From owner-obm-l@sucuri.mat.puc-rio.br Tue Apr 1 00:21:45 2003 Return-Path: Received: (from majordom@localhost) by sucuri.mat.puc-rio.br (8.9.3/8.9.3) id AAA00428 for obm-l-MTTP; Tue, 1 Apr 2003 00:20:08 -0300 Received: from paiol.terra.com.br (paiol.terra.com.br [200.176.3.18]) by sucuri.mat.puc-rio.br (8.9.3/8.9.3) with ESMTP id AAA00422 for ; Tue, 1 Apr 2003 00:20:05 -0300 Received: from una.terra.com.br (una.terra.com.br [200.176.3.32]) by paiol.terra.com.br (Postfix) with ESMTP id 0390C880A0 for ; Tue, 1 Apr 2003 00:19:35 -0300 (BRT) Received: from [200.177.188.125] (dl-nas7-sao-C8B1BC7D.p001.terra.com.br [200.177.188.125]) by una.terra.com.br (Postfix) with ESMTP id D06C12F0016 for ; Tue, 1 Apr 2003 00:19:33 -0300 (BRT) User-Agent: Microsoft-Outlook-Express-Macintosh-Edition/5.02.2022 Date: Tue, 01 Apr 2003 00:17:42 -0300 Subject: Re: [obm-l] AJUDA From: Claudio Buffara To: Message-ID: In-Reply-To: <3E88E7FC.1090505@centroin.com.br> Mime-version: 1.0 Content-type: multipart/alternative; boundary="MS_Mac_OE_3132001063_309203_MIME_Part" Sender: owner-obm-l@sucuri.mat.puc-rio.br Precedence: bulk Reply-To: obm-l@mat.puc-rio.br > This message is in MIME format. Since your mail reader does not understand this format, some or all of this message may not be legible. --MS_Mac_OE_3132001063_309203_MIME_Part Content-type: text/plain; charset="ISO-8859-1" Content-transfer-encoding: quoted-printable Uma curiosidade: o numero desejado eh justamente aquele formado pelos 6 algarismos do periodo de 1/7 quando expresso em decimal. 1/7 =3D 0,142857 142857 1428.... Logo, N =3D 142857 e SD(N) =3D 1+4+2+8+5+7 =3D 27. on 31.03.03 22:14, A. C. Morgado at morgado@centroin.com.br wrote: 1)=20 a=3D1 (se a>1, 6N nao poderia ter a mesma quantidade de algarismos de N). 3N =3D bcdef1 (o 1=3Da so pode aparecer na ultima posi=E7ao no 3N, pois o 5N nao pode terminar em 1 e os outros sao pares). Logo, N termina em 7, f=3D7. 2N termina em 4, 4N termina em 8 e 6N termina em 2, 5N termina em 5. Os algarismos sao 1(inicial), 7(final), 4, 8, 2 e 5 (nao sei em que posi=E7oes) S =3D 27 Se o problema tem solu=E7ao, a solu=E7ao eh 27. Daniel Pini wrote: =20 OL=E1, alguem poderia me ajudar? 1-O n=FAmero de seis algarismos N=3Dabcdef =E9 tal que quando multipicamos por 2= , 3, 4, 5, 6 obtemos n=FAmeros com os mesmos algarismos permutados ciclicamente= . A soma dos alg. de N =E9: R:27 --MS_Mac_OE_3132001063_309203_MIME_Part Content-type: text/html; charset="ISO-8859-1" Content-transfer-encoding: quoted-printable Re: [obm-l] AJUDA Uma curiosidade: o numero desejado eh justamente aquele formado pelos 6 alg= arismos do periodo de 1/7 quando expresso em decimal.

1/7 =3D 0,142857 142857 1428....

Logo, N =3D 142857 e SD(N) =3D 1+4+2+8+5+7 =3D 27.

on 31.03.03 22:14, A. C. Morgado at morgado@centroin.com.br wrote:

1)
a=3D1 (se a>1, 6N nao poderia ter a mesma quantidade de algarismos de N).<= BR> 3N =3D bcdef1 (o 1=3Da so pode aparecer na ultima posi=E7ao no 3N, pois o 5N nao = pode terminar em 1 e os outros sao pares). Logo, N termina em 7, f=3D7.
2N termina em 4, 4N termina em 8 e 6N termina em 2, 5N termina em 5.
Os algarismos sao 1(inicial), 7(final), 4, 8, 2 e 5 (nao sei em que posi=E7oe= s)
S =3D 27
Se o problema tem solu=E7ao, a solu=E7ao eh 27.

Daniel Pini wrote:
   
OL=E1, alguem poderia me ajudar?
1-O n=FAmero de seis algarismos N=3Dabcdef =E9 tal que quando  multipicamos = por 2, 3, 4, 5, 6 obtemos n=FAmeros com os mesmos algarismos permutados ciclic= amente. A soma dos alg. de N =E9: R:27     



--MS_Mac_OE_3132001063_309203_MIME_Part-- ========================================================================= Instruções para entrar na lista, sair da lista e usar a lista em http://www.mat.puc-rio.br/~nicolau/olimp/obm-l.html O administrador desta lista é ========================================================================= From owner-obm-l@sucuri.mat.puc-rio.br Tue Apr 1 00:46:54 2003 Return-Path: Received: (from majordom@localhost) by sucuri.mat.puc-rio.br (8.9.3/8.9.3) id AAA01287 for obm-l-MTTP; Tue, 1 Apr 2003 00:45:34 -0300 Received: from artemis.opendf.com.br (artemis.opengate.com.br [200.181.71.15]) by sucuri.mat.puc-rio.br (8.9.3/8.9.3) with ESMTP id AAA01277 for ; Tue, 1 Apr 2003 00:45:29 -0300 Received: from localhost (localhost [127.0.0.1]) by artemis.opendf.com.br (Postfix) with ESMTP id 1FEAF1C07D for ; Tue, 1 Apr 2003 00:44:53 -0300 (EST) Received: from computer (200-181-088-024.bsace7001.dsl.brasiltelecom.net.br [200.181.88.24]) by artemis.opendf.com.br (Postfix) with ESMTP id 11CD51C03E for ; Tue, 1 Apr 2003 00:44:49 -0300 (EST) From: "Artur Costa Steiner" To: Subject: [obm-l] Mais Problemas em Aberto - Topologia Date: Tue, 1 Apr 2003 00:44:59 -0300 Organization: Steiner Consultoria LTDA Message-ID: <001901c2f801$11e7e0e0$9865fea9@computer> MIME-Version: 1.0 Content-Type: text/plain; charset="iso-8859-1" X-Priority: 3 (Normal) X-MSMail-Priority: Normal X-Mailer: Microsoft Outlook, Build 10.0.2627 X-MIMEOLE: Produced By Microsoft MimeOLE V6.00.2800.1106 In-Reply-To: Importance: Normal X-Virus-Scanned: by AMaViS new-20020517 Content-Transfer-Encoding: 8bit X-MIME-Autoconverted: from quoted-printable to 8bit by sucuri.mat.puc-rio.br id AAA01284 Sender: owner-obm-l@sucuri.mat.puc-rio.br Precedence: bulk Reply-To: obm-l@mat.puc-rio.br > Caros colegas da lista:   > Aqui vai mais uma compilação de problemas que foram propostos mas cujas > soluções nunca foram publicadas na lista. [Artur Costa Steiner] Sobre, Topologia, para os que curtem, aqui vão algumas soluções:   5) Alguns de topologia geral:   Definamos x como ponto de condensação de um subconjunto E de R^n se qualquer vizinhança V de x contiver um número incontável de elementos de E (isto é, se V inter E não for numerável). Seja P o conjunto dos pontos de condensação de E. Mostre que 5.1) E é numerável se, e somente se, P for vazio ( o que acarreta automaticamente que E não é numerável sse P não for vazio) Sabemos que R^n possui uma base numerável, como, por exemplo, a coleção das bolas abertas de raios racionais e centros em elementos de coordenadas racionais. Seja B = {B_n} esta coleção. Para maior clareza, provaremos primeiro o item 5.2 5.2) O conjunto dos elementos de E que não são pontos de condensação do mesmo (E inter complementar de P) é numerável. Definamos W como a união de todos os conjuntos básicos B_n cujas intercessões com E sejam numeráveis. Seja cP o complementar de P. Vamos mostrar que W = cP. Se x pertence a W, então x possui uma vizinhança básica B_n cuja intercessão com E é numerável. Da definição de ponto de condensação, segue-se que x não é um de tais pontos e que portanto, x pertence a cP. Se, por outro lado, x pertence a cP, então x possui uma vizinhança, logo uma vizinhança básica, cuja intercessão com E é numerável. Da definição de W, segue-se que x pertence a W. Concluimos assim que W está contido em cP e vice-versa. Logo W = cP. Desta conclusão, segue-se agora que E inter cP = E inter W = (E inter B_1) U (E inter B_2) U....(E inter B_n)..... Como cada (E inter B_n) é numerável, vemos que E inter cP é dado por uma união numerável de conjuntos numeráveis. Logo E inter cP é numerável, o que prova 5.2. Voltando-se a 5.1, observamos que E = (E inter P) U (E inter cP). Se P for vazio (isto é, se E não possuir pontos de condensação) então E = E inter cP, equação que, em virtude do que acabamos de ver, mostra-nos que E é numerável. Se, por outro lado, P não for vazio, então E possui um ponto de condensação x e qualquer vizinhança V de x é tal que V inter E não é numerável. Dado que V inter E é um subconjunto de E, segue-se que E não é numerável. Isto prova 5.1. Tomando-se as contrapositivas de tais conclusões, constatamos imediatamente que E não é numerável sse P não for vazio. 5.3) P é perfeito (é fechado e todos seus elementos são pontos de acumulação do mesmo). Na realidade, todo elemento de P é ponto de condensação do mesmo. Vimos que cP = W é dado por uma união de conjuntos abertos. Logo, cP é aberto e P é fechado. Alternativamente, podemos chegar a esta mesma conclusão observando que, se x pertence a cP, então x possui uma vizinhança V cuja intercessão com E é numerável. Como V é vizinhança de todos os seus elementos, segue-se que igual condição vale para todo elemento de V, o que nos mostra que V está contida em cP. Todo elemento de cP é portanto ponto interior do mesmo, do que deduzimos que cP é aberto e P é fechado. Sejam agora p pertencente a P e V uma vizinhaça qualquer de p. Temos a seguinte equação: V inter E = (V inter E inter P) U (V inter E inter cP). Pela definição de ponto de condensação, V inter E não é numerável e, conforme já vimos, E inter cP é numerável. Logo, V inter E inter cP é numerável, pois é subconjunto de E inter cP. Para que a equação citada possa vigorar, temos então, necessariamente, que V inter E inter P não pode ser numerável. Como V é arbitrária, concluimos que p é ponto de condensação de E inter P e, consequentemente, do próprio P. E como todo ponto de condensação de um conjunto é, automaticamente, ponto de acumulação do mesmo, concluimos que todo elemento de P é ponto de acumulação do mesmo. Logo P é perfeito. OBS. Nesta demonstração admitimos que P não é vazio. Se P for vazio, então P é trivialmente perfeito. 5.4)Todo elemento de P é ponto de condensação de E inter P Conseqüência imediata da demonstração de 5.3. Como corolário, segue-se que, se P não for vazio, então E inter P não é numerável. Como outro corolário, temos que todo elemento de E inter P é ponto de condensação do mesmo. 5.5) O fecho de E inter P é o próprio P Se x pertence a fecho de E inter P, então então toda vizinhança V de x intercepta E inter P e, portanto, intercepta P. Logo, V contém um ponto de condensação de E, o que acarreta que V inter E não seja numerável. Segue-se que x é ponto de condensação de E e, face a isto, x pertence a P. Se, por outro lado, x pertence a P, então, conforme vimos, x é ponto de condensação de E inter P. É então imediato que x pertence ao fecho de E inter P. Isto prova 5.5 5.6) Todo conjunto fechado é dado pela união disjunta de um conjunto perfeito com um conjunto numerável (podendo ser que um destes conjuntos seja vazio). Este é o Teorema de Cantor-Bendixon. Suponhamos que E seja fechado. Temos que E = (E inter P) U (E inter cP). Conforme vimos, P é fechado, o que acarreta que E inter P também seja fechado. Nos corolários de 5.4 vimos que todo elemento de E inter P é ponto de condensação, e portanto de acumulação, do mesmo. Logo, E inter P é perfeito. Além disto, vimos também que E inter cP é numerável. E como E inter P e E inter cP são disjuntos, o teorema fica demonstrado Um abraço para todos Artur ========================================================================= Instruções para entrar na lista, sair da lista e usar a lista em http://www.mat.puc-rio.br/~nicolau/olimp/obm-l.html O administrador desta lista é ========================================================================= From owner-obm-l@sucuri.mat.puc-rio.br Tue Apr 1 02:25:41 2003 Return-Path: Received: (from majordom@localhost) by sucuri.mat.puc-rio.br (8.9.3/8.9.3) id CAA03284 for obm-l-MTTP; Tue, 1 Apr 2003 02:24:10 -0300 Received: from itaqui.terra.com.br (itaqui.terra.com.br [200.176.3.19]) by sucuri.mat.puc-rio.br (8.9.3/8.9.3) with ESMTP id CAA03279 for ; Tue, 1 Apr 2003 02:24:06 -0300 Received: from botucatu.terra.com.br (botucatu.terra.com.br [200.176.3.78]) by itaqui.terra.com.br (Postfix) with ESMTP id 7BBAE3BC5D6 for ; Tue, 1 Apr 2003 02:23:36 -0300 (BRT) Received: from [200.177.176.168] (dl-nas1-sao-C8B1B0A8.p001.terra.com.br [200.177.176.168]) by botucatu.terra.com.br (Postfix) with ESMTP id 707C429C072 for ; Tue, 1 Apr 2003 02:23:35 -0300 (BRT) User-Agent: Microsoft-Outlook-Express-Macintosh-Edition/5.02.2022 Date: Tue, 01 Apr 2003 02:21:45 -0300 Subject: Re: [obm-l] K- =?ISO-8859-1?B?6XNpbW8gbvptZXJvIGRhIHNlcXXqbmNpYSEgKEEgcmVlbmNhcm5h5+M=?=o) From: Claudio Buffara To: Message-ID: In-Reply-To: <20030401021625.93984.qmail@web13001.mail.yahoo.com> Mime-version: 1.0 Content-type: text/plain; charset="ISO-8859-1" Content-Transfer-Encoding: 8bit X-MIME-Autoconverted: from quoted-printable to 8bit by sucuri.mat.puc-rio.br id CAA03280 Sender: owner-obm-l@sucuri.mat.puc-rio.br Precedence: bulk Reply-To: obm-l@mat.puc-rio.br Essa eh a chamada sequencia de Farey de ordem N (F(N)). a/b pertence a F(N) <==> 0 <= a <= b <= N e mdc(a,b) = 1. Alem disso, se o k-esimo termo eh a/b e o (k+1)-esimo eh c/d entao: a/b < c/d e bc - ad = 1, ou seja: c/d = a/b + 1/(bd). Apesar de nao fornecer uma formula, o algoritmo abaixo (descrito em "An Introduction to the Theory of Numbers" - Hardy-Wright - sec. 3.4) permite determinar o c/d conhecendo-se a/b: Como mdc(a,b) =1, existem inteiros x e y tais que bx - ay = 1. Se (p,q) eh uma solucao particular dessa equacao diofantina (ou seja, bp - aq = 1) entao, a solucao geral serah: x = p + a*k y = q + b*k (k em Z) Podemos escolher k de modo que N - b < q + b*k = y <= N. Dessa forma, teremos uma solucao (x,y) tal que: mdc(x,y) = 1 e N - b < y <= N ==> x/y pertence a F(N) e x/y = a/b + 1/(by) > a/b. Vamos provar, por absurdo, que x/y = c/d. Suponhamos que x/y <> c/d. Entao, x/y > c/d ==> x/y - c/d = (dx - cy)/(dy) >= 1/(dy) Por outro lado, c/d - a/b = (bc - ad)/(bd) = 1/(bd) Assim: 1/(by) = (bx - ay)/(by) = x/y - a/b >= 1/(dy) + 1/(bd) = = (b + y)/(bdy) > N/(bdy) >= 1/(by) ==> contradicao ==> x/y = c/d Um abraco, Claudio. on 31.03.03 23:16, Helder Suzuki at heldersuzuki@yahoo.com.br wrote: > Se temos todas frações reduzidas entre 0/1 e 1/1 > (inclusive) com denominadores <= N e ordenadas, qual a > K-ésima fração em função de N e K? > > por exemplo > se N = 3 > temos: > (0, 1/3, 1/2, 2/3, 1) > A1 = 0, A2 = 1/3, ..., A5 = 1 > > Abraços, > Helder Toshiro Suzuki > > _______________________________________________________________________ > Yahoo! GeoCities > Tudo para criar o seu site: ferramentas fáceis de usar, espaço de sobra e > acessórios. > http://br.geocities.yahoo.com/ > ========================================================================= > Instruções para entrar na lista, sair da lista e usar a lista em > http://www.mat.puc-rio.br/~nicolau/olimp/obm-l.html > O administrador desta lista é > ========================================================================= > ========================================================================= Instruções para entrar na lista, sair da lista e usar a lista em http://www.mat.puc-rio.br/~nicolau/olimp/obm-l.html O administrador desta lista é ========================================================================= From owner-obm-l@sucuri.mat.puc-rio.br Tue Apr 1 10:02:02 2003 Return-Path: Received: (from majordom@localhost) by sucuri.mat.puc-rio.br (8.9.3/8.9.3) id KAA09109 for obm-l-MTTP; Tue, 1 Apr 2003 10:00:23 -0300 Received: from paiol.terra.com.br (paiol.terra.com.br [200.176.3.18]) by sucuri.mat.puc-rio.br (8.9.3/8.9.3) with ESMTP id KAA09092 for ; Tue, 1 Apr 2003 10:00:15 -0300 Received: from botucatu.terra.com.br (botucatu.terra.com.br [200.176.3.78]) by paiol.terra.com.br (Postfix) with ESMTP id 12B7D886E2 for ; Tue, 1 Apr 2003 09:59:39 -0300 (BRT) Received: from [200.177.187.75] (dl-nas4-sao-C8B1BB4B.p001.terra.com.br [200.177.187.75]) by botucatu.terra.com.br (Postfix) with ESMTP id 4796E29C08F for ; Tue, 1 Apr 2003 09:59:38 -0300 (BRT) User-Agent: Microsoft-Outlook-Express-Macintosh-Edition/5.02.2022 Date: Tue, 01 Apr 2003 09:57:45 -0300 Subject: Re: [obm-l] Mais Problemas em Aberto From: Claudio Buffara To: Message-ID: In-Reply-To: <008e01c2f7e0$d6082c30$de609ec8@gauss> Mime-version: 1.0 Content-type: multipart/alternative; boundary="MS_Mac_OE_3132035909_438481_MIME_Part" Sender: owner-obm-l@sucuri.mat.puc-rio.br Precedence: bulk Reply-To: obm-l@mat.puc-rio.br > This message is in MIME format. Since your mail reader does not understand this format, some or all of this message may not be legible. --MS_Mac_OE_3132035909_438481_MIME_Part Content-type: text/plain; charset="ISO-8859-1" Content-transfer-encoding: quoted-printable Tah certo. Foi aquele problema do arquivo muito grande, neh? Desculpe a falha. O credito e de voces. Um abraco, Claudio. on 31.03.03 20:54, Domingos Jr. at dopikas@uol.com.br wrote: p=F4, o 7.2 eu e o Wendel j=E1 provamos: http://www.linux.ime.usp.br/~domingos/problema.ps http://www.linux.ime.usp.br/~domingos/problema.pdf =20 --MS_Mac_OE_3132035909_438481_MIME_Part Content-type: text/html; charset="ISO-8859-1" Content-transfer-encoding: quoted-printable Re: [obm-l] Mais Problemas em Aberto Tah certo. Foi aquele problema do arquivo muito grande, neh?

Desculpe a falha. O credito e de voces.

Um abraco,
Claudio.

on 31.03.03 20:54, Domingos Jr. at dopikas@uol.com.br wrote:

p=F4, o 7.2 eu e o Wendel j=E1 pr= ovamos:
http://www.linux.ime.usp.br/~domingos/problema.ps
http://www.linux.ime.usp.br/~domingos/problema.pdf



--MS_Mac_OE_3132035909_438481_MIME_Part-- ========================================================================= Instruções para entrar na lista, sair da lista e usar a lista em http://www.mat.puc-rio.br/~nicolau/olimp/obm-l.html O administrador desta lista é ========================================================================= From owner-obm-l@sucuri.mat.puc-rio.br Tue Apr 1 10:02:02 2003 Return-Path: Received: (from majordom@localhost) by sucuri.mat.puc-rio.br (8.9.3/8.9.3) id KAA09096 for obm-l-MTTP; Tue, 1 Apr 2003 10:00:19 -0300 Received: from ivoti.terra.com.br (ivoti.terra.com.br [200.176.3.20]) by sucuri.mat.puc-rio.br (8.9.3/8.9.3) with ESMTP id KAA09088 for ; Tue, 1 Apr 2003 10:00:09 -0300 Received: from botucatu.terra.com.br (botucatu.terra.com.br [200.176.3.78]) by ivoti.terra.com.br (Postfix) with ESMTP id 2C448408B94 for ; Tue, 1 Apr 2003 09:59:37 -0300 (BRT) Received: from [200.177.187.75] (dl-nas4-sao-C8B1BB4B.p001.terra.com.br [200.177.187.75]) by botucatu.terra.com.br (Postfix) with ESMTP id 597D529C12B for ; Tue, 1 Apr 2003 09:59:35 -0300 (BRT) User-Agent: Microsoft-Outlook-Express-Macintosh-Edition/5.02.2022 Date: Tue, 01 Apr 2003 09:57:45 -0300 Subject: Re: [obm-l] Mais Problemas em Aberto From: Claudio Buffara To: Message-ID: In-Reply-To: <01e201c2f7b1$472e8ba0$3300c57d@bovespa.com> Mime-version: 1.0 Content-type: multipart/alternative; boundary="MS_Mac_OE_3132035865_435841_MIME_Part" Sender: owner-obm-l@sucuri.mat.puc-rio.br Precedence: bulk Reply-To: obm-l@mat.puc-rio.br > This message is in MIME format. Since your mail reader does not understand this format, some or all of this message may not be legible. --MS_Mac_OE_3132035865_435841_MIME_Part Content-type: text/plain; charset="ISO-8859-1" Content-transfer-encoding: quoted-printable 4) Seja f:N---->R uma fun=E7=E3o tal que f(1)=3D3 e f(m+n)+f(m-n)-m+n-1=3D(f(2m)+f(2n))/2 para todos os inteiros n=E3o negativos m = e n com m>=3Dn.=20 Determine a express=E3o de f(m). m =3D n =3D=3D>=20 f(2n) + f(0) - 1 =3D f(2n) =3D=3D> f(0) =3D 1 n =3D 0 =3D=3D> f(m) + f(m) - m - 1 =3D [f(2m) + f(0)]/2 =3D=3D> 4f(m) - 2m - 2 =3D f(2m) + 1 =3D=3D> f(2m) =3D 4f(m) - 2m - 3 Fazendo m+n =3D p e m-n =3D q =3D=3D> p >=3D q e m =3D (p+q)/2 e n =3D (p-q)/2 =3D=3D> f(p) + f(q) - (p+q)/2 + (p-q)/2 -1 =3D [f(p+q)+f(p-q)]/2 =3D=3D> f(p) + f(q) - q - 1 =3D [f(p+q)+f(p-q)]/2 p =3D q+1 =3D=3D> f(q+1) + f(q) - q - 1 =3D [f(2q+1)+f(1)]/2 =3D=3D> f(2q+1) =3D 2[f(q+1) + f(q) - q] - 5 =3D=3D> Resumindo, temos: f(0) =3D 1 f(1) =3D 3=20 e para todo n >=3D 0: f(2n) =3D 4f(n) - 2n - 3 f(2n+1) =3D 2[f(n+1) + f(n) - n] - 5 Calculando os valores seguintes de f, chegamos a: f(2) =3D 7 f(3) =3D 13 f(4) =3D 21 f(5) =3D 31 f(6) =3D 43 f(7) =3D 57 Reparamos que vale, para todo k, com 1 <=3D k <=3D 7: f(k) =3D f(k-1) + 2k. Juntamente com f(0) =3D 1, esta equacao implica que: f(k) =3D k^2 + k + 1 para 0 <=3D k <=3D 7. Temos agora a nossa hipotese de inducao: Suponhamos que, para 0 <=3D k < n, f(k) =3D k^2 + k + 1. n eh par =3D=3D> n =3D 2m =3D=3D> f(2m) =3D 4f(m) - 2m - 3 =3D 4(m^2 + m + 1) - 2m - 3 =3D 4m^2 + 2m + 1 =3D (2m)^2 + (2m) + 1 =3D n^2 + n =3D 1 =3D=3D> Se n eh par, entao f(n) =3D n^2 + n =3D 1. n eh impar =3D=3D> n =3D 2m+1 =3D=3D> f(2m+1) =3D 2[f(m+1) + f(m) - m] - 5 =3D 2[(m^2+2m+1) + (m+1) + 1 + m^2 + m + 1 - m] - 5 =3D 4m^2 + 6m + 3 =3D (4m^2 + 4m + 1) + (2m + 1) + 1 =3D (2m+1)^2 + (2m+1) + 1 =3D n^2 + n + 1 =3D=3D> Se n eh impar, entao f(n) =3D n^2 + n =3D 1. Logo, para todo n >=3D 0, f(n) =3D n^2 + n + 1. Um abraco, Claudio. --MS_Mac_OE_3132035865_435841_MIME_Part Content-type: text/html; charset="ISO-8859-1" Content-transfer-encoding: quoted-printable Re: [obm-l] Mais Problemas em Aberto

4) Seja f:N---->R uma fun=E7=E3o tal que f(1)=3D3 e
f(m+n)+f(m-n)-m+n-1=3D(f(2m)+f(2n))/2 para todos os inteiros n=E3o negativos m = e n com m>=3Dn.
Determine a express=E3o de f(m).

m =3D n =3D=3D>
f(2n) + f(0) - 1 =3D f(2n) =3D=3D>
f(0) =3D 1

n =3D 0 =3D=3D>
f(m) + f(m) - m - 1 =3D [f(2m) + f(0)]/2 =3D=3D>
4f(m) - 2m - 2 =3D f(2m) + 1 =3D=3D>
f(2m) =3D 4f(m) - 2m - 3

Fazendo m+n =3D p   e   m-n =3D q =3D=3D>
p >=3D q   e   m =3D (p+q)/2   e   n= =3D (p-q)/2 =3D=3D>
f(p) + f(q) - (p+q)/2 + (p-q)/2 -1 =3D [f(p+q)+f(p-q)]/2 =3D=3D>
f(p) + f(q) - q - 1 =3D [f(p+q)+f(p-q)]/2

p =3D q+1 =3D=3D>
f(q+1) + f(q) - q - 1 =3D [f(2q+1)+f(1)]/2 =3D=3D>
f(2q+1) =3D 2[f(q+1) + f(q) - q] - 5 =3D=3D>

Resumindo, temos:
f(0) =3D 1
f(1) =3D 3
e para todo n >=3D 0:
f(2n) =3D 4f(n) - 2n - 3
f(2n+1) =3D 2[f(n+1) + f(n) - n] - 5

Calculando os valores seguintes de f, chegamos a:
f(2) =3D 7
f(3) =3D 13
f(4) =3D 21
f(5) =3D 31
f(6) =3D 43
f(7) =3D 57

Reparamos que vale, para todo k, com 1 <=3D k <=3D 7:
f(k) =3D f(k-1) + 2k.

Juntamente com f(0) =3D 1, esta equacao implica que:
f(k) =3D k^2 + k + 1  para 0 <=3D k <=3D 7.

Temos agora a nossa hipotese de inducao:
Suponhamos que, para 0 <=3D k < n, f(k) =3D k^2 + k + 1.

n eh par =3D=3D>
n =3D 2m =3D=3D>
f(2m) =3D 4f(m) - 2m - 3 =3D 4(m^2 + m + 1) - 2m - 3 =3D
4m^2 + 2m + 1 =3D (2m)^2 + (2m) + 1 =3D n^2 + n =3D 1 =3D=3D>
Se n eh par, entao f(n) =3D n^2 + n =3D 1.

n eh impar =3D=3D>
n =3D 2m+1 =3D=3D>
f(2m+1) =3D 2[f(m+1) + f(m) - m] - 5 =3D
2[(m^2+2m+1) + (m+1) + 1 + m^2 + m + 1 - m] - 5 =3D
4m^2 + 6m + 3 =3D
(4m^2 + 4m + 1) + (2m + 1) + 1 =3D
(2m+1)^2 + (2m+1) + 1 =3D n^2 + n + 1 =3D=3D>
Se n eh impar, entao f(n) =3D n^2 + n =3D 1.

Logo,  para todo n >=3D 0, f(n) =3D n^2 + n + 1.

Um abraco,
Claudio.




--MS_Mac_OE_3132035865_435841_MIME_Part-- ========================================================================= Instruções para entrar na lista, sair da lista e usar a lista em http://www.mat.puc-rio.br/~nicolau/olimp/obm-l.html O administrador desta lista é ========================================================================= From owner-obm-l@sucuri.mat.puc-rio.br Tue Apr 1 10:02:03 2003 Return-Path: Received: (from majordom@localhost) by sucuri.mat.puc-rio.br (8.9.3/8.9.3) id KAA09103 for obm-l-MTTP; Tue, 1 Apr 2003 10:00:21 -0300 Received: from itaqui.terra.com.br (itaqui.terra.com.br [200.176.3.19]) by sucuri.mat.puc-rio.br (8.9.3/8.9.3) with ESMTP id KAA09089 for ; Tue, 1 Apr 2003 10:00:09 -0300 Received: from botucatu.terra.com.br (botucatu.terra.com.br [200.176.3.78]) by itaqui.terra.com.br (Postfix) with ESMTP id ED45B3BCC67 for ; Tue, 1 Apr 2003 09:59:37 -0300 (BRT) Received: from [200.177.187.75] (dl-nas4-sao-C8B1BB4B.p001.terra.com.br [200.177.187.75]) by botucatu.terra.com.br (Postfix) with ESMTP id 0B6A329C0FA for ; Tue, 1 Apr 2003 09:59:37 -0300 (BRT) User-Agent: Microsoft-Outlook-Express-Macintosh-Edition/5.02.2022 Date: Tue, 01 Apr 2003 09:57:45 -0300 Subject: Re: [obm-l] Mais Probls em Aberto II From: Claudio Buffara To: Message-ID: In-Reply-To: <03f001c2f7b4$aa8349e0$3300c57d@bovespa.com> Mime-version: 1.0 Content-type: multipart/alternative; boundary="MS_Mac_OE_3132035908_438406_MIME_Part" Sender: owner-obm-l@sucuri.mat.puc-rio.br Precedence: bulk Reply-To: obm-l@mat.puc-rio.br > This message is in MIME format. Since your mail reader does not understand this format, some or all of this message may not be legible. --MS_Mac_OE_3132035908_438406_MIME_Part Content-type: text/plain; charset="US-ASCII" Content-transfer-encoding: 7bit 12)Sejam m e n inteiros positivos tal que n== n >= 1 Inducao em m, supondo n fixo: m = n: (m+n)!/(m-n)! = (2n)! = produto de todos os naturais de 1 a 2n >= produto de todos os pares de 1 a 2n = 2^n*n! Suponhamos que m >= n e que (m+n)!/(m-n)! >= 2^n * n!. (m+1+n)!/(m+1-n)! = (m+n)!/(m-n)! * (m+n+1)/(m-n+1) >= (m+n)!/(m-n)! >= 2^n*n!. ------ Parte 2: (m+n)!/(m-n)! <= (m^2+m)^n para m >= n >= 1 (m+n)!/(m-n)! = (m+n)*(m+n-1)*....*(m-n+1) = PRODUTO(1<=k<=n) (m+n+1-k)*(m-n+k) = PRODUTO(1<=k<=n) [m^2-n^2+m-n + (2n+1)*k - k^2] Por outro lado: (m^2+m)^n = PRODUTO(1<=k<= n) (m^2+m) Entretanto: (m^2 - m) - [m^2-n^2+m-n + (2n+1)*k - k^2] = k^2 - (2n+1)*k + n*(n+1) = (k - n)*(k - n - 1) >= 0 se k <= n ou k >= n + 1 ==> (k - n)*(k - n - 1) >= 0 para todo k inteiro ==> PRODUTO(1<=k<=n) (m+n+1-k)*(m-n+k) <= (m^2+m)^n ==> (m+n)/(m-n)! <= (m^2+m)^n para m >=n >= 0. Um abraco, Claudio. --MS_Mac_OE_3132035908_438406_MIME_Part Content-type: text/html; charset="US-ASCII" Content-transfer-encoding: quoted-printable Re: [obm-l] Mais Probls em Aberto II

12)Sejam m e n inteiros positivos tal que n=3D<m. Prove
que (2^n)*n!=3D< (m+n)!/(m-n)! =3D<(m^2+m)^n.

Parte 1:
(2^n)*n! <=3D (m+n)!/(m-n)! para m >=3D n >=3D 1
Inducao em m, supondo n fixo:

m =3D n:
(m+n)!/(m-n)! =3D (2n)! =3D produto de todos os naturais de 1 a 2n >=3D
produto de todos os pares de 1 a 2n =3D 2^n*n!

Suponhamos que m >=3D n e que (m+n)!/(m-n)! >=3D 2^n * n!.

(m+1+n)!/(m+1-n)! =3D (m+n)!/(m-n)! * (m+n+1)/(m-n+1) >=3D
(m+n)!/(m-n)! >=3D 2^n*n!.

------

Parte 2:
(m+n)!/(m-n)! <=3D (m^2+m)^n para m >=3D n >=3D 1

(m+n)!/(m-n)! =3D (m+n)*(m+n-1)*....*(m-n+1) =3D
PRODUTO(1<=3Dk<=3Dn) (m+n+1-k)*(m-n+k) =3D
PRODUTO(1<=3Dk<=3Dn) [m^2-n^2+m-n + (2n+1)*k - k^2]

Por outro lado:
(m^2+m)^n =3D PRODUTO(1<=3Dk<=3D n) (m^2+m)

Entretanto:
(m^2 - m) - [m^2-n^2+m-n + (2n+1)*k - k^2] =3D
k^2 - (2n+1)*k + n*(n+1) =3D
(k - n)*(k - n - 1) >=3D 0 se k <=3D n ou k >=3D n + 1 =3D=3D>

(k - n)*(k - n - 1) >=3D 0 para todo k inteiro =3D=3D>

PRODUTO(1<=3Dk<=3Dn) (m+n+1-k)*(m-n+k) <=3D (m^2+m)^n =3D=3D>

(m+n)/(m-n)! <=3D (m^2+m)^n para m >=3Dn >=3D 0.


Um abraco,
Claudio.
--MS_Mac_OE_3132035908_438406_MIME_Part-- ========================================================================= Instruções para entrar na lista, sair da lista e usar a lista em http://www.mat.puc-rio.br/~nicolau/olimp/obm-l.html O administrador desta lista é ========================================================================= From owner-obm-l@sucuri.mat.puc-rio.br Tue Apr 1 13:59:26 2003 Return-Path: Received: (from majordom@localhost) by sucuri.mat.puc-rio.br (8.9.3/8.9.3) id NAA14142 for obm-l-MTTP; Tue, 1 Apr 2003 13:56:44 -0300 Received: from ns3bind.localdomain ([200.230.34.5]) by sucuri.mat.puc-rio.br (8.9.3/8.9.3) with ESMTP id NAA14138 for ; Tue, 1 Apr 2003 13:56:40 -0300 Received: from servico2 ([200.230.34.229]) by ns3bind.localdomain (8.11.6/X.XX.X) with SMTP id h31Gql825003 for ; Tue, 1 Apr 2003 13:52:47 -0300 Message-ID: <013801c2f86f$aba3eb20$3300c57d@bovespa.com> From: "=?iso-8859-1?Q?Cl=E1udio_\=28Pr=E1tica\=29?=" To: References: <002401c2f7e0$e718eaa0$9a75fea9@windows98> Subject: Re: [obm-l] AJUDA Date: Tue, 1 Apr 2003 13:56:38 -0300 MIME-Version: 1.0 Content-Type: multipart/alternative; boundary="----=_NextPart_000_0135_01C2F856.83F2D280" X-Priority: 3 X-MSMail-Priority: Normal X-Mailer: Microsoft Outlook Express 5.50.4920.2300 X-MimeOLE: Produced By Microsoft MimeOLE V5.50.4920.2300 Sender: owner-obm-l@sucuri.mat.puc-rio.br Precedence: bulk Reply-To: obm-l@mat.puc-rio.br This is a multi-part message in MIME format. ------=_NextPart_000_0135_01C2F856.83F2D280 Content-Type: text/plain; charset="iso-8859-1" Content-Transfer-Encoding: quoted-printable 4-Fatore: a^4+b^4-c^4-2a^b^2+4abc^2 Supondo que a express=E3o seja: a^4+b^4-c^4-2a^2b^2+4abc^2, fa=E7amos: F(c) =3D -c^4 + 4abc^2 + (a^4+b^4-2a^2b^2) =3D polin=F4mio biquadrado em = c. Delta =3D 16a^2b^2 + 4(a^4+b^4-2a^2b^2) =3D =3D 4(a^4+b^4+2a^2b^2) =3D 4(a^2+b^2)^2 =3D=3D> raiz(Delta) =3D 2(a^2+b^2) Logo, as ra=EDzes ser=E3o: c^2 =3D [-4ab +ou- 2(a^2+b^2)]/(-2), ou seja: c^2 =3D 2ab + a^2 + b^2 ou c^2 =3D 2ab - a^2 - b^2 =3D=3D> c^2 =3D (a+b)^2 ou c^2 =3D -(a-b)^2 Logo, F(c) se fatora como: F(c) =3D ((a+b)^2 - c^2)((a-b)^2 + c^2) =3D=3D> F(c) =3D (a + b + c)(a + b - c)(c^2 + (a-b)^2) Um abra=E7o, Claudio. ------=_NextPart_000_0135_01C2F856.83F2D280 Content-Type: text/html; charset="iso-8859-1" Content-Transfer-Encoding: quoted-printable
 
4-Fatore: a^4+b^4-c^4-2a^b^2+4abc^2
 
Supondo que a express=E3o seja:
a^4+b^4-c^4-2a^2b^2+4abc^2, = fa=E7amos:
F(c) =3D -c^4 + 4abc^2 + (a^4+b^4-2a^2b^2) =3D = polin=F4mio=20 biquadrado em c.
 
Delta =3D 16a^2b^2 + 4(a^4+b^4-2a^2b^2) = =3D
=3D 4(a^4+b^4+2a^2b^2) =3D 4(a^2+b^2)^2 = =3D=3D>
 
raiz(Delta) =3D 2(a^2+b^2)
 
Logo, as ra=EDzes ser=E3o:
c^2 =3D [-4ab +ou- 2(a^2+b^2)]/(-2), ou = seja:
c^2 =3D 2ab + a^2 + b^2  ou  c^2 =3D = 2ab - a^2 - b^2=20 =3D=3D>
c^2 =3D (a+b)^2 ou c^2 =3D -(a-b)^2
 
Logo, F(c) se fatora como:
F(c) =3D ((a+b)^2 - c^2)((a-b)^2 + c^2) = =3D=3D>
 
F(c) =3D (a + b + c)(a + b - c)(c^2 +=20 (a-b)^2)
 
Um abra=E7o,
Claudio.
------=_NextPart_000_0135_01C2F856.83F2D280-- ========================================================================= Instruções para entrar na lista, sair da lista e usar a lista em http://www.mat.puc-rio.br/~nicolau/olimp/obm-l.html O administrador desta lista é ========================================================================= From owner-obm-l@sucuri.mat.puc-rio.br Tue Apr 1 14:08:24 2003 Return-Path: Received: (from majordom@localhost) by sucuri.mat.puc-rio.br (8.9.3/8.9.3) id OAA14367 for obm-l-MTTP; Tue, 1 Apr 2003 14:06:54 -0300 Received: from hotmail.com (bay1-f13.bay1.hotmail.com [65.54.245.13]) by sucuri.mat.puc-rio.br (8.9.3/8.9.3) with ESMTP id OAA14363 for ; Tue, 1 Apr 2003 14:06:51 -0300 Received: from mail pickup service by hotmail.com with Microsoft SMTPSVC; Tue, 1 Apr 2003 09:06:19 -0800 Received: from 146.164.44.48 by by1fd.bay1.hotmail.msn.com with HTTP; Tue, 01 Apr 2003 17:06:19 GMT X-Originating-IP: [146.164.44.48] X-Originating-Email: [marcelo_souza7@hotmail.com] From: "Marcelo Souza" To: obm-l@mat.puc-rio.br Subject: [obm-l] equação Date: Tue, 01 Apr 2003 17:06:19 +0000 Mime-Version: 1.0 Content-Type: text/html Message-ID: X-OriginalArrivalTime: 01 Apr 2003 17:06:19.0642 (UTC) FILETIME=[03AC6DA0:01C2F871] Sender: owner-obm-l@sucuri.mat.puc-rio.br Precedence: bulk Reply-To: obm-l@mat.puc-rio.br
Alguém poderia achar as raizes da equação (usando um computador que a mao esta dificil)
 
[4/(sqrt(x^2-900) + 6/(sqrt(x^(2)-400)] = 15
obrigado pela ajuda
[]'s, M.


Add photos to your messages with MSN 8. Get 2 months FREE*. ========================================================================= Instruções para entrar na lista, sair da lista e usar a lista em http://www.mat.puc-rio.br/~nicolau/olimp/obm-l.html O administrador desta lista é ========================================================================= From owner-obm-l@sucuri.mat.puc-rio.br Tue Apr 1 14:12:52 2003 Return-Path: Received: (from majordom@localhost) by sucuri.mat.puc-rio.br (8.9.3/8.9.3) id OAA14514 for obm-l-MTTP; Tue, 1 Apr 2003 14:11:01 -0300 Received: from smtp-29.ig.com.br (smtp-29.ig.com.br [200.226.132.157]) by sucuri.mat.puc-rio.br (8.9.3/8.9.3) with SMTP id OAA14510 for ; Tue, 1 Apr 2003 14:10:56 -0300 Received: (qmail 19220 invoked from network); 1 Apr 2003 17:10:15 -0000 Received: from 169068.telemar.net.br (HELO xxxx) (200.165.169.68) by smtp-29.ig.com.br with SMTP; 1 Apr 2003 17:10:15 -0000 Message-ID: <003601c2f871$f289b860$44a9a5c8@epq.ime.eb.br> From: "Marcio" To: References: Subject: Re: [obm-l] Rearranjo generalizado Date: Tue, 1 Apr 2003 14:12:59 -0300 MIME-Version: 1.0 Content-Type: text/plain; charset="iso-8859-1" Content-Transfer-Encoding: 7bit X-Priority: 3 X-MSMail-Priority: Normal X-Mailer: Microsoft Outlook Express 5.50.4133.2400 X-MIMEOLE: Produced By Microsoft MimeOLE V5.50.4133.2400 Sender: owner-obm-l@sucuri.mat.puc-rio.br Precedence: bulk Reply-To: obm-l@mat.puc-rio.br Por que a linha "Naturalmente o valor de S eh o mesmo nos 3 casos" eh verdadeira? Por exemplo, se vc tiver A =(a1,a2)= (1,2), B = (3,2), C=(2, 3/2), entao vc tem: a1b1 < a2b2 e c1 < c2. Por outro lado, embora voce tenha a1 < a2, vc nao tem b1c1 < b2c2... Portanto, nesse caso, a soma S nao eh exatamente a mesma nos seus 3 casos.. Certo? Marcio > Inicialmente, aplicamos a hipotese de inducao as M-1 sequencias (A_i*B_i), > (C_i), ..., (Z_i) e concluimos que S eh maxima quando todas estas as > sequencias tem a mesma ordenacao, digamos: > 0 < A_1*B_1 <= ... <= A_n*B_n, > 0 < C_1 <= ... <= C_n, > ... > 0 < Z_1 <= ... <= Z_n. > > Agora, aplicamos a h.i. as M-1 sequencias (A_i), (B_i*C_i), ..., (Z_i) e > concluimos que S eh maxima quando: > 0 < A_1 <= ... <= A_n, > 0 < B_1*C_1 <= ... <= B_n*C_n, > ... > 0 < Z_1 <= ... <= Z_n. > > Finalmente, aplicamos a h.i. as M-1 sequencias (A_i*C_i), (B_i), ..., (Z_i) > e concluimos que S eh maxima quando: > 0 < A-1*C_1 <= ... <= A_n*C_n, > 0 < B_1 <= ... <= B_n, > ... > 0 < Z_1 <= ... <= Z_n. > > Naturalmente, o valor maximo de S serah o mesmo em cada um dos tres casos > acima. > > Estas tres aplicacoes da h.i. implicam que S eh maxima quando: > 0 < A_1 <= ... <= A_n, > 0 < B_1 <= ... <= B_n, > 0 < C_1 <= ... <= C_n, > ... > 0 < Z_1 <= ... <= Z_n, > ou seja, quando as M sequencias tiverem a a mesma ordenacao. > ========================================================================= Instruções para entrar na lista, sair da lista e usar a lista em http://www.mat.puc-rio.br/~nicolau/olimp/obm-l.html O administrador desta lista é ========================================================================= From owner-obm-l@sucuri.mat.puc-rio.br Tue Apr 1 17:28:09 2003 Return-Path: Received: (from majordom@localhost) by sucuri.mat.puc-rio.br (8.9.3/8.9.3) id RAA19036 for obm-l-MTTP; Tue, 1 Apr 2003 17:25:39 -0300 Received: from ns3bind.localdomain ([200.230.34.5]) by sucuri.mat.puc-rio.br (8.9.3/8.9.3) with ESMTP id RAA19032 for ; Tue, 1 Apr 2003 17:25:35 -0300 Received: from servico2 ([200.230.34.229]) by ns3bind.localdomain (8.11.6/X.XX.X) with SMTP id h31KLf806634 for ; Tue, 1 Apr 2003 17:21:41 -0300 Message-ID: <017b01c2f88c$dba92340$3300c57d@bovespa.com> From: "=?iso-8859-1?Q?Cl=E1udio_\=28Pr=E1tica\=29?=" To: References: <003601c2f871$f289b860$44a9a5c8@epq.ime.eb.br> Subject: Re: [obm-l] Rearranjo generalizado Date: Tue, 1 Apr 2003 17:25:31 -0300 MIME-Version: 1.0 Content-Type: text/plain; charset="iso-8859-1" Content-Transfer-Encoding: 8bit X-Priority: 3 X-MSMail-Priority: Normal X-Mailer: Microsoft Outlook Express 5.50.4920.2300 X-MimeOLE: Produced By Microsoft MimeOLE V5.50.4920.2300 Sender: owner-obm-l@sucuri.mat.puc-rio.br Precedence: bulk Reply-To: obm-l@mat.puc-rio.br ----- Original Message ----- From: "Marcio" To: Sent: Tuesday, April 01, 2003 2:12 PM Subject: Re: [obm-l] Rearranjo generalizado > Por que a linha "Naturalmente o valor de S eh o mesmo nos 3 casos" eh > verdadeira? > Por exemplo, se vc tiver A =(a1,a2)= (1,2), B = (3,2), C=(2, 3/2), entao > vc tem: > a1b1 < a2b2 e c1 < c2. > Por outro lado, embora voce tenha a1 < a2, vc nao tem b1c1 < b2c2... > Portanto, nesse caso, a soma S nao eh exatamente a mesma nos seus 3 casos.. > Certo? > Marcio > > Oi Márcio: Acho que a sua sequencia C deveria ser (3/2,2). Nesse caso, a1b1 = 3 < 4 = a2b2 e c1 = 3/2 < 2 = c2. Por outro lado, a1 = 1 < 2 = a2, mas b1c1 = 9/2 > 4 = b2c2. Na verdade a soma nos dois casos é a mesma: S = 1*3*3/2 + 2*2*2 = 12,5. Só que esta soma não é a maior possível (que é 15). Assim, foi possível ter b1c1 > b2c2. O que eu disse é que as somas MÁXIMAS em cada caso são iguais. De qualquer forma, concordo que esta afirmativa precisa ser justificada com algo mais do que um "naturalmente". Vou pensar num argumento aceitável e te aviso quando achar. Um abraço, Claudio. > > > Inicialmente, aplicamos a hipotese de inducao as M-1 sequencias (A_i*B_i), > > (C_i), ..., (Z_i) e concluimos que S eh maxima quando todas estas as > > sequencias tem a mesma ordenacao, digamos: > > 0 < A_1*B_1 <= ... <= A_n*B_n, > > 0 < C_1 <= ... <= C_n, > > ... > > 0 < Z_1 <= ... <= Z_n. > > > > Agora, aplicamos a h.i. as M-1 sequencias (A_i), (B_i*C_i), ..., (Z_i) e > > concluimos que S eh maxima quando: > > 0 < A_1 <= ... <= A_n, > > 0 < B_1*C_1 <= ... <= B_n*C_n, > > ... > > 0 < Z_1 <= ... <= Z_n. > > > > Finalmente, aplicamos a h.i. as M-1 sequencias (A_i*C_i), (B_i), ..., > (Z_i) > > e concluimos que S eh maxima quando: > > 0 < A-1*C_1 <= ... <= A_n*C_n, > > 0 < B_1 <= ... <= B_n, > > ... > > 0 < Z_1 <= ... <= Z_n. > > > > Naturalmente, o valor maximo de S serah o mesmo em cada um dos tres casos > > acima. > > > > Estas tres aplicacoes da h.i. implicam que S eh maxima quando: > > 0 < A_1 <= ... <= A_n, > > 0 < B_1 <= ... <= B_n, > > 0 < C_1 <= ... <= C_n, > > ... > > 0 < Z_1 <= ... <= Z_n, > > ou seja, quando as M sequencias tiverem a a mesma ordenacao. > > ========================================================================= Instruções para entrar na lista, sair da lista e usar a lista em http://www.mat.puc-rio.br/~nicolau/olimp/obm-l.html O administrador desta lista é ========================================================================= From owner-obm-l@sucuri.mat.puc-rio.br Tue Apr 1 18:13:48 2003 Return-Path: Received: (from majordom@localhost) by sucuri.mat.puc-rio.br (8.9.3/8.9.3) id SAA20254 for obm-l-MTTP; Tue, 1 Apr 2003 18:11:54 -0300 Received: from paiol.terra.com.br (paiol.terra.com.br [200.176.3.18]) by sucuri.mat.puc-rio.br (8.9.3/8.9.3) with ESMTP id SAA20246 for ; Tue, 1 Apr 2003 18:11:49 -0300 Received: from canela.terra.com.br (canela.terra.com.br [200.176.3.79]) by paiol.terra.com.br (Postfix) with ESMTP id 40A9C88A1C for ; Tue, 1 Apr 2003 18:11:19 -0300 (BRT) Received: from usuario (200-180-188-144.paemt7005.dsl.brasiltelecom.net.br [200.180.188.144]) (authenticated user marioappereira) by canela.terra.com.br (Postfix) with ESMTP id CD39D11405B for ; Tue, 1 Apr 2003 18:11:18 -0300 (BRT) Message-ID: <003501c2f893$3d6efcc0$0301a8c0@usuario> From: =?iso-8859-1?Q?M=E1rio_Pereira?= To: Subject: [obm-l] volume Date: Tue, 1 Apr 2003 18:11:18 -0300 MIME-Version: 1.0 Content-Type: multipart/related; type="multipart/alternative"; boundary="----=_NextPart_000_0031_01C2F87A.17D0BE90" X-Priority: 3 X-MSMail-Priority: Normal X-Mailer: Microsoft Outlook Express 6.00.2600.0000 X-MimeOLE: Produced By Microsoft MimeOLE V6.00.2600.0000 Sender: owner-obm-l@sucuri.mat.puc-rio.br Precedence: bulk Reply-To: obm-l@mat.puc-rio.br This is a multi-part message in MIME format. ------=_NextPart_000_0031_01C2F87A.17D0BE90 Content-Type: multipart/alternative; boundary="----=_NextPart_001_0032_01C2F87A.17D57980" ------=_NextPart_001_0032_01C2F87A.17D57980 Content-Type: text/plain; charset="iso-8859-1" Content-Transfer-Encoding: quoted-printable Por favor ajudem a resolver:=20 Um tonel em forma de um cilindro regular encontra-se deitado no solo, = com um certo volume de =F3leo dentro.=20 O diametro base =E9 1,90 metros e o comprimento do tonel (altura) =E9 = 5,5 metros. Estando deitado, a altura do l=EDquido dentro do tonel = equivale a 1,13 metros. Qual o volume de =F3leo dentro do tonel? Obrigado,=20 M=E1rio =20 =20 ------=_NextPart_001_0032_01C2F87A.17D57980 Content-Type: text/html; charset="iso-8859-1" Content-Transfer-Encoding: quoted-printable
Por favor ajudem a resolver: =
 
Um tonel em forma de um cilindro = regular=20 encontra-se deitado no solo, com um certo volume de =F3leo dentro. =
O diametro base =E9 1,90 metros e o = comprimento do=20 tonel (altura) =E9 5,5 metros. Estando deitado, a altura do l=EDquido = dentro do=20 tonel equivale a 1,13 metros.
Qual o volume de =F3leo dentro do = tonel?
 
Obrigado,
M=E1rio
 
 
 
 
 
------=_NextPart_001_0032_01C2F87A.17D57980-- ------=_NextPart_000_0031_01C2F87A.17D0BE90 Content-Type: image/gif; name="clip_image001.gif" Content-Transfer-Encoding: base64 Content-ID: <003001c2f893$3d193ba0$0301a8c0@usuario> R0lGODlhygBjAHcAMSH+GlNvZnR3YXJlOiBNaWNyb3NvZnQgT2ZmaWNlACH5BAEAAAAALAEAAQDJ AGEAgQAAAAAAAP///wECAwL/hB2py+0Po5y02ovzBOn4fwSCKJTmiabqyrbuC8fyTNdqUgbgTtr+ DwwKh0TcaAfq9YjMpvMJPS11SIQ0is1qt6tp1XrlisdkoPe7HJXX7HYqTa3C3fS6OH1E4vH2vj+4 J/f2R1joE6g3aLjI2ILIc9MoOamGEpcUSalJyJeHqbgZWtd5GdIliupGCpmZ6jq2+gn6SpsV+0Fa q4t165G7C9zUa3oabAzYxdp6zDzTmSM72zzt8uxJvEytnW0ZbbkNzi3lHRZuXpmMy2J9fmxd+t5u /q6+Lh9O7+t4D56PLc7PmD8wxQJOG8hhn8GDjurZW8gMITt2EFNJrFbRXUN9/woz7prosKDHWiA5 PhxJ8sWliSh1UVyJsSWtlyZFykRF8x/Am5RyErTJc5PPhB2DhhrK0qgopDGVClVZc6dTQ0yLTpVU 9eRVrFB1Stu6KCtQsIXESiVrx+xXtH/UfmMbtuvPs3DbuC1Xt61colrz9rlrgqJfMoBzDNZbLera w2wKo2NMp6TXt5AjJ1ZMubKqjZPxam7Mee7iz3dC86VL2pbpeKntrrbaGtbrvrFLrws5ujav2WN1 q76NObBvwqufaTiOPLny5cyPO2JwOvNwLQije56+G3jwx9izp9suuDsy7Z25i4cyTLTw87/fKJPO /kl664bje+9Grr599N+Dh2bfX8N89P0HoDP9YUZggTEIyEeCCsLwCDkOPqiSe4nAR+F441x4XYZm WMihfh4KEaE3E47ITSkhnYjiMiqu2GIRG6KxXowaXnMhizZKQcKLkOi4Yw49fuFLc0YeiWSSSkrA gY8GFAAAOw== ------=_NextPart_000_0031_01C2F87A.17D0BE90-- ========================================================================= Instruções para entrar na lista, sair da lista e usar a lista em http://www.mat.puc-rio.br/~nicolau/olimp/obm-l.html O administrador desta lista é ========================================================================= From owner-obm-l@sucuri.mat.puc-rio.br Tue Apr 1 19:22:40 2003 Return-Path: Received: (from majordom@localhost) by sucuri.mat.puc-rio.br (8.9.3/8.9.3) id TAA30344 for obm-l-MTTP; Tue, 1 Apr 2003 19:21:00 -0300 Received: from traven10.uol.com.br (traven10.uol.com.br [200.221.29.45]) by sucuri.mat.puc-rio.br (8.9.3/8.9.3) with ESMTP id TAA30340 for ; Tue, 1 Apr 2003 19:20:57 -0300 Received: from gauss ([200.158.96.173]) by traven10.uol.com.br (8.9.1/8.9.1) with SMTP id TAA04095 for ; Tue, 1 Apr 2003 19:20:25 -0300 (BRT) Message-ID: <000d01c2f89d$5bf768d0$2accfea9@gauss> From: "Domingos Jr." To: References: <01e201c2f7b1$472e8ba0$3300c57d@bovespa.com> Subject: Re: [obm-l] Mais Problemas em Aberto Date: Tue, 1 Apr 2003 19:23:44 -0300 MIME-Version: 1.0 Content-Type: text/plain; charset="Windows-1252" Content-Transfer-Encoding: 8bit X-Priority: 3 X-MSMail-Priority: Normal X-Mailer: Microsoft Outlook Express 6.00.2800.1106 X-MimeOLE: Produced By Microsoft MimeOLE V6.00.2800.1106 Sender: owner-obm-l@sucuri.mat.puc-rio.br Precedence: bulk Reply-To: obm-l@mat.puc-rio.br Consegui estimar um limitante inferior para o número de grupos de crianças: Considere uma matriz com elementos A[i, j] = (i, j) pertence a (Zp)² O problema proposto é equivalente a calcular o número de combinações de elementos de A cuja soma dê (0, 0). Agora desenhando a matriz A e separando a última linha e a última coluna, formamos uma matriz A', com (p-1) x (p-1) elementos em (Zp)². Os elementos da última linha são: { (1, 0), (2, 0), ..., (p-1, 0) , (0,0)} e os da última coluna são: { (0, 1), (0, 2), ..., (0, p-1) , (0,0)} * o elemento (0, 0) é compartilhado! Repare que toda combinação de elementos da matriz A' tem soma em (Zp)² e todo elemento e (Zp)² tem um oposto aditivo em (Zp)², além disso, é possível obter todos os elementos de (Zp)² através dos elementos da última linha e da última coluna (basta tomar a soma de dois deles, por exemplo), por tanto, para cada combinação de A' existe pelo menos uma maneira de selecionar elemento(s) na última linha e coluna de A tal que a soma total dê (0, 0). Por tanto, um limitante inferior para o número de grupo crianças do problema é: 2^[(p-1)²] == total de combinações em A'. Esse limitante tem bastante folga pois na verdade existe várias maneiras de obter o mesmo elemento de (Zp)² através da última linha e da última coluna. por exemplo o elemento (3, 2) = (3,0) + (0,2) = (2,0) + (1,0) + (0,2) = ... Idéias? [ ]'s ----- 7.5)(Guilherme Issao)Existem p²,onde p e primo,crianças dispostas num bairro como um tabuleiro p por p.Ha tambem duas distribuidoras de doces,a Cledmilson Marmotta e a Estrogonofre's.A Cledmilson Marmotta manda um vendedor para cada uma das p linhas horizontais,sendo que o vendedor da i-esima linha tem i Kg de doce de jilo e distribui igualmente entre as p crianças.Da mesma forma Estrogonofre's manda um vendedor para cada uma das p linhas verticais,sendo que o vendedor da j-esima linha tem j Kg de doce de jaca e distribui igualmente entre as p crianças.De quantas maneiras podemos escolher um grupo de crianças desse bairro para roubar-lhes os doces de modo que a quantidade de cada tipo de doce roubada seja inteira?[6] ========================================================================= Instruções para entrar na lista, sair da lista e usar a lista em http://www.mat.puc-rio.br/~nicolau/olimp/obm-l.html O administrador desta lista é ========================================================================= From owner-obm-l@sucuri.mat.puc-rio.br Tue Apr 1 19:34:13 2003 Return-Path: Received: (from majordom@localhost) by sucuri.mat.puc-rio.br (8.9.3/8.9.3) id TAA30588 for obm-l-MTTP; Tue, 1 Apr 2003 19:32:52 -0300 Received: from fnn.net ([200.175.38.9]) by sucuri.mat.puc-rio.br (8.9.3/8.9.3) with SMTP id TAA30584 for ; Tue, 1 Apr 2003 19:32:46 -0300 Received: (qmail 14369 invoked from network); 1 Apr 2003 22:18:57 -0000 Received: from unknown (HELO windows98) (200.175.39.54) by fnn.net with SMTP; 1 Apr 2003 22:18:57 -0000 Message-ID: <002d01c2f8a7$d79b47e0$9a75fea9@windows98> From: "Daniel Pini" To: Subject: [obm-l] nova ajuda Date: Tue, 1 Apr 2003 20:38:12 -0300 MIME-Version: 1.0 Content-Type: multipart/alternative; boundary="----=_NextPart_000_0029_01C2F88E.9D49D020" X-Priority: 3 X-MSMail-Priority: Normal X-Mailer: Microsoft Outlook Express 5.00.2615.200 X-MimeOLE: Produced By Microsoft MimeOLE V5.00.2615.200 Sender: owner-obm-l@sucuri.mat.puc-rio.br Precedence: bulk Reply-To: obm-l@mat.puc-rio.br This is a multi-part message in MIME format. ------=_NextPart_000_0029_01C2F88E.9D49D020 Content-Type: text/plain; charset="iso-8859-1" Content-Transfer-Encoding: quoted-printable As cidades A e B distam 5 quilometros uma da outra. Deseja-se construir = uma escola onde estudar=E3o 1000 crian=E7as da cidade A e 500 crian=E7as = da cidade B. A que distancia, em quilometros, da cidade A deve ser = construida a escola de modo que a distancia total percorrida por todas = as 1500 crian=E7as seja a menor poss=EDvel?=20 R: 0 Considere a sequencia x(1), x(2), x(3), ... definida por x(1)=3D 3^1/3, = x(2)=3D (x(1))^3^1/3 e, em geral, x(n)=3D (x(n-1))^3^1/3, pra n maior = que 1. O menor valor de n para o qual x(n) =E9 inteiro vale: R:4 A sequencia crescente 2, 3, 5, 6, 7, 10, 11,... consiste de todos os = inteiros positivos que n=E3o s=E3o quadrados nem cubos de um inteiro = positivo. O 500=BA termo dessa sequencia =E9 : R:528 Se (5^2 + 9^2) (12^2 + 17^2) for escrito sob a forma a^2 + b^2 ent=E3o = a+b =E9 igual a:R: 60 ou 153 ------=_NextPart_000_0029_01C2F88E.9D49D020 Content-Type: text/html; charset="iso-8859-1" Content-Transfer-Encoding: quoted-printable
As cidades  A e B distam 5 = quilometros uma da=20 outra. Deseja-se construir uma escola onde estudar=E3o 1000 crian=E7as = da cidade A e=20 500 crian=E7as da cidade B. A que distancia, em quilometros, da cidade A = deve ser=20 construida a escola de modo que a distancia total percorrida por todas = as 1500=20 crian=E7as seja a menor poss=EDvel?
R: 0
Considere a sequencia x(1), x(2), x(3), = ...  definida=20 por x(1)=3D 3^1/3, x(2)=3D (x(1))^3^1/3 e, em geral, x(n)=3D = (x(n-1))^3^1/3, pra n=20 maior que 1. O menor valor de n para o qual x(n) =E9 inteiro vale:=20 R:4
 
A sequencia crescente 2, 3, 5, 6, 7, 10, 11,... = consiste=20 de todos os inteiros positivos que n=E3o s=E3o quadrados nem cubos de um = inteiro=20 positivo. O 500=BA termo dessa sequencia =E9 :
R:528
 
Se (5^2 + 9^2) (12^2 + 17^2) for escrito sob a = forma a^2 +=20 b^2 ent=E3o a+b  =E9 igual a:R: 60 ou 153
 
------=_NextPart_000_0029_01C2F88E.9D49D020-- ========================================================================= Instruções para entrar na lista, sair da lista e usar a lista em http://www.mat.puc-rio.br/~nicolau/olimp/obm-l.html O administrador desta lista é ========================================================================= From owner-obm-l@sucuri.mat.puc-rio.br Tue Apr 1 19:46:20 2003 Return-Path: Received: (from majordom@localhost) by sucuri.mat.puc-rio.br (8.9.3/8.9.3) id TAA30942 for obm-l-MTTP; Tue, 1 Apr 2003 19:44:58 -0300 Received: from fnn.net ([200.175.38.9]) by sucuri.mat.puc-rio.br (8.9.3/8.9.3) with SMTP id TAA30937 for ; Tue, 1 Apr 2003 19:44:52 -0300 Received: (qmail 11262 invoked from network); 1 Apr 2003 22:30:57 -0000 Received: from unknown (HELO windows98) (200.175.39.54) by fnn.net with SMTP; 1 Apr 2003 22:30:57 -0000 Message-ID: <004801c2f8a9$854be420$9a75fea9@windows98> From: "Daniel Pini" To: Subject: [obm-l] =?iso-8859-1?Q?quest=E3o_enviada_com_erro?= Date: Tue, 1 Apr 2003 20:50:47 -0300 MIME-Version: 1.0 Content-Type: multipart/alternative; boundary="----=_NextPart_000_0045_01C2F890.5EF21E20" X-Priority: 3 X-MSMail-Priority: Normal X-Mailer: Microsoft Outlook Express 5.00.2615.200 X-MimeOLE: Produced By Microsoft MimeOLE V5.00.2615.200 Sender: owner-obm-l@sucuri.mat.puc-rio.br Precedence: bulk Reply-To: obm-l@mat.puc-rio.br This is a multi-part message in MIME format. ------=_NextPart_000_0045_01C2F890.5EF21E20 Content-Type: text/plain; charset="iso-8859-1" Content-Transfer-Encoding: quoted-printable Pe=E7o desculpa pelo erro idiota que fiz na quest=E3o de fatora=E7=E3o. A express=E3o correta =E9: a^4 + b^4 - c^4 -2a=B2b=B2+4abc=B2=20 ------=_NextPart_000_0045_01C2F890.5EF21E20 Content-Type: text/html; charset="iso-8859-1" Content-Transfer-Encoding: quoted-printable
Pe=E7o desculpa pelo erro idiota que = fiz na quest=E3o=20 de fatora=E7=E3o.
A express=E3o correta =E9:
a^4 + b^4 - c^4 -2a=B2b=B2+4abc=B2 =
------=_NextPart_000_0045_01C2F890.5EF21E20-- ========================================================================= Instruções para entrar na lista, sair da lista e usar a lista em http://www.mat.puc-rio.br/~nicolau/olimp/obm-l.html O administrador desta lista é ========================================================================= From owner-obm-l@sucuri.mat.puc-rio.br Tue Apr 1 22:14:21 2003 Return-Path: Received: (from majordom@localhost) by sucuri.mat.puc-rio.br (8.9.3/8.9.3) id WAA01053 for obm-l-MTTP; Tue, 1 Apr 2003 22:12:39 -0300 Received: from itaqui.terra.com.br (itaqui.terra.com.br [200.176.3.19]) by sucuri.mat.puc-rio.br (8.9.3/8.9.3) with ESMTP id WAA01049 for ; Tue, 1 Apr 2003 22:12:36 -0300 Received: from marova.terra.com.br (marova.terra.com.br [200.176.3.39]) by itaqui.terra.com.br (Postfix) with ESMTP id 938323BE550 for ; Tue, 1 Apr 2003 22:12:05 -0300 (BRT) Received: from riemann.localdomain (RJ175052.user.veloxzone.com.br [200.149.175.52]) (authenticated user fabio.dias.moreira) by marova.terra.com.br (Postfix) with ESMTP id 1F4013DC08C for ; Tue, 1 Apr 2003 22:12:05 -0300 (BRT) Content-Type: text/plain; charset="iso-8859-1" From: =?iso-8859-1?q?F=E1bio=20Dias=20Moreira?= To: obm-l@mat.puc-rio.br Subject: Re: [obm-l] nova ajuda Date: Tue, 1 Apr 2003 22:12:18 -0300 User-Agent: KMail/1.4.3 References: <002d01c2f8a7$d79b47e0$9a75fea9@windows98> In-Reply-To: <002d01c2f8a7$d79b47e0$9a75fea9@windows98> MIME-Version: 1.0 Content-Transfer-Encoding: 8bit Message-Id: <200304012212.25980.fabio.dias.moreira@terra.com.br> Sender: owner-obm-l@sucuri.mat.puc-rio.br Precedence: bulk Reply-To: obm-l@mat.puc-rio.br -----BEGIN PGP SIGNED MESSAGE----- Hash: SHA1 On Tuesday 01 April 2003 20:38, Daniel Pini wrote: > As cidades A e B distam 5 quilometros uma da outra. Deseja-se construir > uma escola onde estudarão 1000 crianças da cidade A e 500 crianças da > cidade B. A que distancia, em quilometros, da cidade A deve ser construida > a escola de modo que a distancia total percorrida por todas as 1500 > crianças seja a menor possível? R: 0 > [...] Seja d a distância da escola a A. Então a distância a B é de 5 - d. Queremos minimizar 1000*d + (5-d)*500 = 1000*d + 2500 - 500*d = 500*d + 2500 com 0 <= d <= 5. > [...] > Considere a sequencia x(1), x(2), x(3), ... definida por x(1)= 3^1/3, > x(2)= (x(1))^3^1/3 e, em geral, x(n)= (x(n-1))^3^1/3, pra n maior que 1. O > menor valor de n para o qual x(n) é inteiro vale: R:4 > [...] É fácil ver que x(n) = (3^1/3)^((3^1/3)^n) (indução!). Existe um teorema diz que se a e b são reais que são raízes de um polinômio de coeficientes inteiros (por exemplo, x^3 - 3 = 0) e b não for racional, então a^b não é raiz de nenhum polinômio de coeficientes inteiros (e em particular, a^b não é inteiro). (Existe uma demonstração mais elementar de que x(2) e x(3) não são inteiros que não envolva estimar 3^1/3 por cima e por baixo por racionais?) > [...] > A sequencia crescente 2, 3, 5, 6, 7, 10, 11,... consiste de todos os > inteiros positivos que não são quadrados nem cubos de um inteiro positivo. > O 500º termo dessa sequencia é : R:528 > [...] Calcule primeiro a posição do número 500 e conte até o 500-ésimo termo na mão (cuidado com as sextas potências!). > [...] > Se (5^2 + 9^2) (12^2 + 17^2) for escrito sob a forma a^2 + b^2 então a+b é > igual a:R: 60 ou 153 > [...] Seja z' o conjugado de um complexo z. Use os seguintes fatos: a) Se z = a+bi, zz' = a^2 + b^2 b) zz'ww' = zwz'w' = zw'z'w []s, - -- Fábio "ctg \pi" Dias Moreira -----BEGIN PGP SIGNATURE----- Version: GnuPG v1.0.6 (GNU/Linux) Comment: For info see http://www.gnupg.org iD8DBQE+ijj5alOQFrvzGQoRAp0OAJ0S7v2FPcYIgqqIzO//DA4+WIxSvQCgnmts sbmwPytfJcBaA8r9LBDtYQ8= =s5y0 -----END PGP SIGNATURE----- ========================================================================= Instruções para entrar na lista, sair da lista e usar a lista em http://www.mat.puc-rio.br/~nicolau/olimp/obm-l.html O administrador desta lista é ========================================================================= From owner-obm-l@sucuri.mat.puc-rio.br Wed Apr 2 00:06:49 2003 Return-Path: Received: (from majordom@localhost) by sucuri.mat.puc-rio.br (8.9.3/8.9.3) id AAA02675 for obm-l-MTTP; Wed, 2 Apr 2003 00:05:02 -0300 Received: from paiol.terra.com.br (paiol.terra.com.br [200.176.3.18]) by sucuri.mat.puc-rio.br (8.9.3/8.9.3) with ESMTP id AAA02670 for ; Wed, 2 Apr 2003 00:04:59 -0300 Received: from itaim.terra.com.br (itaim.terra.com.br [200.176.3.76]) by paiol.terra.com.br (Postfix) with ESMTP id 76B8187FF7 for ; Wed, 2 Apr 2003 00:04:29 -0300 (BRT) Received: from [200.177.179.115] (dl-nas3-sao-C8B1B373.p001.terra.com.br [200.177.179.115]) by itaim.terra.com.br (Postfix) with ESMTP id 673A12E006B for ; Wed, 2 Apr 2003 00:04:28 -0300 (BRT) User-Agent: Microsoft-Outlook-Express-Macintosh-Edition/5.02.2022 Date: Wed, 02 Apr 2003 00:02:38 -0300 Subject: [obm-l] Rearranjo generalizado - revisado From: Claudio Buffara To: Lista OBM Message-ID: In-Reply-To: Mime-version: 1.0 Content-type: text/plain; charset="ISO-8859-1" Content-Transfer-Encoding: 8bit X-MIME-Autoconverted: from quoted-printable to 8bit by sucuri.mat.puc-rio.br id AAA02671 Sender: owner-obm-l@sucuri.mat.puc-rio.br Precedence: bulk Reply-To: obm-l@mat.puc-rio.br Oi, Marcio: Dei uma boa mexida na demonstracao e eliminei aquela passagem nao justificada. Por favor de uma lida e me diga se ficou algum furo. O problema: Sejam varias seqs de termos positivos (a), (b), (c), ...e considere as somas do tipo S = a_1*b_1*c_1*... +a_2*b_2*c_2* ... + ... a_n*b_n*c_n*... onde (a_i) eh uma permutacao da 1a sequencia, (b_i) uma permutacao da 2a, e assim por diante. Mostre que S é máxima quando as sequencias tem a mesma ordenacao. O caso com 2 sequencias eh o que se conhece como "desigualdade do rearranjo" ----------- Inducao sobre o numero M de sequencias (M >= 2): O caso base (M = 2) eh a desigualdade do rearranjo. Supondo que o resultado seja verdadeiro para quaisquer M-1 sequencias (M >= 3) de termos positivos, consideremos as M sequencias (A_i), (B_i), (C_i), ..., (Z_i) (achei melhor usar esta notacao do que dois indices) de termos positivos e as somas correspondentes do tipo: S = A_1*B_1*...*Z_1 + ... + A_n*B_n*...*Z_n Inicialmente, aplicamos a hipotese de inducao as M-1 sequencias (A_i*B_i), (C_i), ..., (Z_i) e concluimos que S eh maxima quando todas estas as sequencias tem a mesma ordenacao, digamos: 0 < A_1*B_1 <= ... <= A_n*B_n, 0 < C_1 <= ... <= C_n, ... 0 < Z_1 <= ... <= Z_n. O objetivo agora eh provar que: 0 < A_1 <= ... <= A_n e 0 < B_1 <= ... <= B_n Suponhamos, portanto, que este nao seja o caso. Sem perda de generalidade podemos supor que existem i, j tais que: 1 <= i < j <= n e A_i > A_j. Nesse caso, como A_i*B_i <= A_j*B_j, teremos que B_i < B_j, o que, juntamente com as outras desigualdades decorrentes da h.i., implica que: B_i*...*Z_i < B_j*...*Z_j. Agora, vamos calcular o valor da soma S1, a qual eh obtida de S pela permutacao de A_i e A_j (todos os demais termos ficam iguais). Assim: S1 = S - A_i*B_i*...*Z_i - A_j*B_j*...*Z_j + A_j*B_i*...*Z_i + A_i*B_j*...*Z_j ==> S1 = S + (A_i - A_j)*(B_j*...*Z_j - B_i*...*Z_i) Como A_i > A_j e B_j*...*Z_j > B_i*...*Z_i, temos que S1 > S. Ou seja, colocando em ordem crescente dois termos originalmente "fora de ordem" da sequncia (A_i), conseguimos aumentar o valor da soma. O mesmo raciocinio vale para eventuais termos "fora de ordem" da sequencia (B_i). Portanto, a soma eh maxima quando (A_i) e (B_i) estao em ordem crescente. Em virtude da h.i., podemos concluir que S eh maxima quando: 0 < A_1 <= ... <= A_n, 0 < B_1 <= ... <= B_n, 0 < C_1 <= ... <= C_n, ... 0 < Z_1 <= ... <= Z_n, ou seja, quando as M sequencias tiverem a a mesma ordenacao. FIM Um abraco, Claudio. ========================================================================= Instruções para entrar na lista, sair da lista e usar a lista em http://www.mat.puc-rio.br/~nicolau/olimp/obm-l.html O administrador desta lista é ========================================================================= From owner-obm-l@sucuri.mat.puc-rio.br Wed Apr 2 00:06:49 2003 Return-Path: Received: (from majordom@localhost) by sucuri.mat.puc-rio.br (8.9.3/8.9.3) id AAA02683 for obm-l-MTTP; Wed, 2 Apr 2003 00:05:12 -0300 Received: from paiol.terra.com.br (paiol.terra.com.br [200.176.3.18]) by sucuri.mat.puc-rio.br (8.9.3/8.9.3) with ESMTP id AAA02679 for ; Wed, 2 Apr 2003 00:05:09 -0300 Received: from marova.terra.com.br (marova.terra.com.br [200.176.3.39]) by paiol.terra.com.br (Postfix) with ESMTP id 415F3881C4 for ; Wed, 2 Apr 2003 00:04:39 -0300 (BRT) Received: from [200.177.179.115] (dl-nas3-sao-C8B1B373.p001.terra.com.br [200.177.179.115]) by marova.terra.com.br (Postfix) with ESMTP id EAF233DC090 for ; Wed, 2 Apr 2003 00:04:37 -0300 (BRT) User-Agent: Microsoft-Outlook-Express-Macintosh-Edition/5.02.2022 Date: Wed, 02 Apr 2003 00:03:28 -0300 Subject: Re: [obm-l] volume From: Claudio Buffara To: Message-ID: In-Reply-To: <003501c2f893$3d6efcc0$0301a8c0@usuario> Mime-version: 1.0 Content-type: multipart/alternative; boundary="MS_Mac_OE_3132086608_545458_MIME_Part" Sender: owner-obm-l@sucuri.mat.puc-rio.br Precedence: bulk Reply-To: obm-l@mat.puc-rio.br > This message is in MIME format. Since your mail reader does not understand this format, some or all of this message may not be legible. --MS_Mac_OE_3132086608_545458_MIME_Part Content-type: text/plain; charset="ISO-8859-1" Content-transfer-encoding: quoted-printable on 01.04.03 18:11, M=E1rio Pereira at marioappereira@terra.com.br wrote: Por favor ajudem a resolver: =20 Um tonel em forma de um cilindro regular encontra-se deitado no solo, com u= m certo volume de =F3leo dentro. O diametro base =E9 1,90 metros e o comprimento do tonel (altura) =E9 5,5 metros. Estando deitado, a altura do l=EDquido dentro do tonel equivale a 1,1= 3 metros. Qual o volume de =F3leo dentro do tonel? =20 Obrigado,=20 M=E1rio =20 Oi, Mario: O volume de liquido (desprezando a espessura das paredes, ou considerando a= s medidas como sendo do volume interno) eh igual ao volume de um cilindro ret= o cuja altura eh 5,5 m e cuja base eh um segmento circular de raio 0,95 m e altura 1,13 m. A area da base do cilindro liquido tambem eh igual a diferenca entre a area de um circulo de raio 0,95 m e a area de um segmento deste circulo de altur= a igual a 1,90 - 1,13 =3D 0,77 m. O angulo central A compreendido pelo segmento circular de altura 0,77 m eh tal que cos(A/2) =3D (0,95-0,77)/0,95 =3D 0,18/0,95 =3D 18/95 A area deste segmento eh igual a: (1/2)*0,95^2*(A - sen(A)) cos(A/2) =3D 18/95 =3D=3D> cos(A) =3D 2cos^2(A/2) - 1 =3D 2*18^2/95^2 - 1 =3D -0,928199 =3D=3D> sen(A) =3D raiz(1 - cos^2(A)) =3D 0,372083 =3D=3D> A =3D arccos(-0,928199) =3D 2,760341 radianos =3D=3D> =20 Area do segmento =3D (1/2)*0,95^2*(2,760341 - 0,372083) =3D 1,077701 m^2 =3D=3D> Area da base do Liquido =3D Pi*0,95^2 - 1,077701 =3D 1,757586 m^2 =3D=3D> Volume do Liquido =3D 1,757586 * 5,5 =3D 9,666723 m^3. Um abraco, Claudio.=20 --MS_Mac_OE_3132086608_545458_MIME_Part Content-type: text/html; charset="ISO-8859-1" Content-transfer-encoding: quoted-printable Re: [obm-l] volume on 01.04.03 18:11, M=E1rio Pereira at marioappereira@terra.com.br wrote:

Por favor ajudem a resolver: =

Um tonel em forma de um cilindro regular = encontra-se deitado no solo, com um certo volume de =F3leo dentro.
O diametro base =E9 1,90 metros e o comprimento do tonel (altura) =E9 5,5 metro= s. Estando deitado, a altura do l=EDquido dentro do tonel equivale a 1,13 metr= os.
Qual o volume de =F3leo dentro do tonel?

Obrigado,
M=E1rio

Oi, Mario:

O volume de liquido (desprezando a espessura das paredes, ou considerando a= s medidas como sendo do volume interno) eh igual ao volume de um cilindro re= to cuja altura eh 5,5 m e cuja base eh um segmento circular de raio 0,95 m e= altura 1,13 m.

A area da base do cilindro liquido tambem eh igual a diferenca entre a area= de um circulo de raio 0,95 m e a area de um segmento deste circulo de altur= a igual a 1,90 - 1,13 =3D 0,77 m.

O angulo central A compreendido pelo segmento circular de altura 0,77 m eh = tal que cos(A/2) =3D (0,95-0,77)/0,95 =3D 0,18/0,95 =3D 18/95

A area deste segmento eh igual a:
(1/2)*0,95^2*(A - sen(A))

cos(A/2) =3D 18/95 =3D=3D>
cos(A) =3D 2cos^2(A/2) - 1 =3D 2*18^2/95^2 - 1 =3D -0,928199 =3D=3D>
sen(A) =3D raiz(1 - cos^2(A)) =3D 0,372083 =3D=3D>
A =3D arccos(-0,928199) =3D 2,760341 radianos =3D=3D>
 
Area do segmento =3D (1/2)*0,95^2*(2,760341 - 0,372083) =3D 1,077701 m^2 =3D=3D>=

Area da base do Liquido =3D Pi*0,95^2 - 1,077701 =3D 1,757586 m^2 =3D=3D>

Volume do Liquido =3D 1,757586 * 5,5 =3D 9,666723 m^3.


Um abraco,
Claudio.


--MS_Mac_OE_3132086608_545458_MIME_Part-- ========================================================================= Instruções para entrar na lista, sair da lista e usar a lista em http://www.mat.puc-rio.br/~nicolau/olimp/obm-l.html O administrador desta lista é ========================================================================= From owner-obm-l@sucuri.mat.puc-rio.br Wed Apr 2 03:06:23 2003 Return-Path: Received: (from majordom@localhost) by sucuri.mat.puc-rio.br (8.9.3/8.9.3) id DAA05399 for obm-l-MTTP; Wed, 2 Apr 2003 03:02:30 -0300 Received: from smtp014.mail.yahoo.com (smtp014.mail.yahoo.com [216.136.173.58]) by sucuri.mat.puc-rio.br (8.9.3/8.9.3) with SMTP id DAA05395 for ; Wed, 2 Apr 2003 03:02:27 -0300 Received: from unknown (HELO victorli) (victorluiz16@200.153.15.20 with login) by smtp.mail.vip.sc5.yahoo.com with SMTP; 2 Apr 2003 06:01:55 -0000 Message-ID: <007c01c2f8de$37f7a580$140f99c8@victorli> From: "Victor Luiz" To: Subject: [obm-l] =?iso-8859-1?Q?Palavra_=22Matem=E1tica=22_e_s=EDmbolo_de_multiplica=E7=E3o?= Date: Wed, 2 Apr 2003 03:07:13 -0300 MIME-Version: 1.0 Content-Type: text/plain; charset="iso-8859-1" Content-Transfer-Encoding: 8bit X-Priority: 3 X-MSMail-Priority: Normal X-Mailer: Microsoft Outlook Express 6.00.2800.1106 X-MimeOLE: Produced By Microsoft MimeOLE V6.00.2800.1106 Sender: owner-obm-l@sucuri.mat.puc-rio.br Precedence: bulk Reply-To: obm-l@mat.puc-rio.br Eu gostaria de saber a origem da palavra "Matemática". Outra coisa que eu sempre quis saber é se existe algum motivo especial em depois de algumas séries substituir o símbolo de multiplicação que até então era usado um "x" por um ponto (Ou bolinha, dependendo do ponto de vista). Seria para os alunos não confundirem o sinal de multiplicação com a incógnita x? Grato, Victor Luiz Salgado de Lima. ========================================================================= Instruções para entrar na lista, sair da lista e usar a lista em http://www.mat.puc-rio.br/~nicolau/olimp/obm-l.html O administrador desta lista é ========================================================================= From owner-obm-l@sucuri.mat.puc-rio.br Wed Apr 2 08:11:35 2003 Return-Path: Received: (from majordom@localhost) by sucuri.mat.puc-rio.br (8.9.3/8.9.3) id IAA08784 for obm-l-MTTP; Wed, 2 Apr 2003 08:09:37 -0300 Received: from hotmail.com (f110.law9.hotmail.com [64.4.9.110]) by sucuri.mat.puc-rio.br (8.9.3/8.9.3) with ESMTP id IAA08780 for ; Wed, 2 Apr 2003 08:09:33 -0300 Received: from mail pickup service by hotmail.com with Microsoft SMTPSVC; Wed, 2 Apr 2003 03:09:02 -0800 Received: from 200.222.178.226 by lw9fd.law9.hotmail.msn.com with HTTP; Wed, 02 Apr 2003 11:09:01 GMT X-Originating-IP: [200.222.178.226] X-Originating-Email: [ricardoprins@hotmail.com] From: "Ricardo Prins" To: obm-l@mat.puc-rio.br Subject: Re: [obm-l] Grafos e Casamentos Date: Wed, 02 Apr 2003 11:09:01 +0000 Mime-Version: 1.0 Content-Type: text/html Message-ID: X-OriginalArrivalTime: 02 Apr 2003 11:09:02.0221 (UTC) FILETIME=[446353D0:01C2F908] Sender: owner-obm-l@sucuri.mat.puc-rio.br Precedence: bulk Reply-To: obm-l@mat.puc-rio.br

me intrometendo...

Você pode me enviar a demonstração?

Ricardo

>From: "Cláudio \(Prática\)"
>Reply-To: obm-l@mat.puc-rio.br
>To:
>Subject: Re: [obm-l] Grafos e Casamentos
>Date: Mon, 31 Mar 2003 15:57:27 -0300
>
>Oi, JP:
>
>O enunciado do Teorema dos Casamentos é o seguinte:
>Sejam A(1), A(2), ..., A(n) conjuntos tais que a união da quaisquer k deles
>(1 <= k <= n) contém no mínimo k elementos distintos. Então é possível
>selecionar n elementos distintos, sendo um de cada conjunto.
>
>A demonstração padrão é por indução completa em n, e trata dois casos
>separadamente:
>i) Para cada k (1 <= k < n), a união de cada k conjuntos contém pelo menos
>k+1 elementos;
>ii) Existem k (1 <= k < n) e k conjuntos tais que a sua união tem
>exatamemente k elementos.
>
>Se você quiser, depois eu posso mandar a demonstração.
>
>Um abraço,
>Claudio.
>
>
>
>
>
>----- Original Message -----
>From:
>To:
>Sent: Monday, March 31, 2003 2:23 PM
>Subject: [obm-l] Grafos e Casamentos
>
>
> > Turma,quem conhece o enunciado e a demonstraçao do Teorema dos
>Casamentos?Estava
> > tentando pensar nele ao ver esse problema:
> >
> > Numa festa ha 18 garotos e 18 garotas.Destas 36 pessoas,4 delas tem 2
>amigos
> > cada,16 tem 3 amigos cada e o resto tem 4 amigos cada.Qual o minimo de
>casais
> > amigos diferentes que pode haver na festa?
> >
> > Nao sei se tem algo a ver mas de qualquer modo tai.
> >
> >
> > TEA WITH ME THAT I BOOK YOUR FACE
> >
> >
> > ------------------------------------------
> > Use o melhor sistema de busca da Internet
> > Radar UOL - http://www.radaruol.com.br
> >
> >
> >
> > =========================================================================
> > Instruções para entrar na lista, sair da lista e usar a lista em
> > http://www.mat.puc-rio.br/~nicolau/olimp/obm-l.html
> > O administrador desta lista é
> > =========================================================================
>
>=========================================================================
>Instruções para entrar na lista, sair da lista e usar a lista em
>http://www.mat.puc-rio.br/~nicolau/olimp/obm-l.html
>O administrador desta lista é
>=========================================================================


Help STOP SPAM with the new MSN 8 and get 2 months FREE* ========================================================================= Instruções para entrar na lista, sair da lista e usar a lista em http://www.mat.puc-rio.br/~nicolau/olimp/obm-l.html O administrador desta lista é ========================================================================= From owner-obm-l@sucuri.mat.puc-rio.br Wed Apr 2 08:18:37 2003 Return-Path: Received: (from majordom@localhost) by sucuri.mat.puc-rio.br (8.9.3/8.9.3) id IAA08902 for obm-l-MTTP; Wed, 2 Apr 2003 08:17:16 -0300 Received: from hotmail.com (f150.law9.hotmail.com [64.4.9.150]) by sucuri.mat.puc-rio.br (8.9.3/8.9.3) with ESMTP id IAA08891 for ; Wed, 2 Apr 2003 08:17:10 -0300 Received: from mail pickup service by hotmail.com with Microsoft SMTPSVC; Wed, 2 Apr 2003 03:16:39 -0800 Received: from 200.222.178.226 by lw9fd.law9.hotmail.msn.com with HTTP; Wed, 02 Apr 2003 11:16:38 GMT X-Originating-IP: [200.222.178.226] X-Originating-Email: [ricardoprins@hotmail.com] From: "Ricardo Prins" To: obm-l@mat.puc-rio.br Subject: Re: [obm-l] AJUDA Date: Wed, 02 Apr 2003 11:16:38 +0000 Mime-Version: 1.0 Content-Type: text/html Message-ID: X-OriginalArrivalTime: 02 Apr 2003 11:16:39.0274 (UTC) FILETIME=[54D014A0:01C2F909] Sender: owner-obm-l@sucuri.mat.puc-rio.br Precedence: bulk Reply-To: obm-l@mat.puc-rio.br



Eu preciso me acostumar com essa notação de expoente...haha - achei estranho pois havia lido algo do tipo 2a^(2b^2)...

>From: "Cláudio \(Prática\)"
>Reply-To: obm-l@mat.puc-rio.br
>To:
>Subject: Re: [obm-l] AJUDA
>Date: Tue, 1 Apr 2003 13:56:38 -0300
>
>
>4-Fatore: a^4+b^4-c^4-2a^b^2+4abc^2
>
>Supondo que a expressão seja:
>a^4+b^4-c^4-2a^2b^2+4abc^2, façamos:
>F(c) = -c^4 + 4abc^2 + (a^4+b^4-2a^2b^2) = polinômio biquadrado em c.
>
>Delta = 16a^2b^2 + 4(a^4+b^4-2a^2b^2) =
>= 4(a^4+b^4+2a^2b^2) = 4(a^2+b^2)^2 ==>
>
>raiz(Delta) = 2(a^2+b^2)
>
>Logo, as raízes serão:
>c^2 = [-4ab +ou- 2(a^2+b^2)]/(-2), ou seja:
>c^2 = 2ab + a^2 + b^2 ou c^2 = 2ab - a^2 - b^2 ==>
>c^2 = (a+b)^2 ou c^2 = -(a-b)^2
>
>Logo, F(c) se fatora como:
>F(c) = ((a+b)^2 - c^2)((a-b)^2 + c^2) ==>
>
>F(c) = (a + b + c)(a + b - c)(c^2 + (a-b)^2)
>
>Um abraço,
>Claudio.


The new MSN 8: smart spam protection and 2 months FREE* ========================================================================= Instruções para entrar na lista, sair da lista e usar a lista em http://www.mat.puc-rio.br/~nicolau/olimp/obm-l.html O administrador desta lista é ========================================================================= From owner-obm-l@sucuri.mat.puc-rio.br Wed Apr 2 08:46:34 2003 Return-Path: Received: (from majordom@localhost) by sucuri.mat.puc-rio.br (8.9.3/8.9.3) id IAA09788 for obm-l-MTTP; Wed, 2 Apr 2003 08:45:00 -0300 Received: from hotmail.com (f68.law9.hotmail.com [64.4.9.68]) by sucuri.mat.puc-rio.br (8.9.3/8.9.3) with ESMTP id IAA09784 for ; Wed, 2 Apr 2003 08:44:56 -0300 Received: from mail pickup service by hotmail.com with Microsoft SMTPSVC; Wed, 2 Apr 2003 03:44:25 -0800 Received: from 200.222.178.226 by lw9fd.law9.hotmail.msn.com with HTTP; Wed, 02 Apr 2003 11:44:25 GMT X-Originating-IP: [200.222.178.226] X-Originating-Email: [ricardoprins@hotmail.com] From: "Ricardo Prins" To: obm-l@mat.puc-rio.br Subject: Re: [obm-l] Palavra "Matemática" e símbolo de multiplicação Date: Wed, 02 Apr 2003 11:44:25 +0000 Mime-Version: 1.0 Content-Type: text/html Message-ID: X-OriginalArrivalTime: 02 Apr 2003 11:44:25.0660 (UTC) FILETIME=[360E67C0:01C2F90D] Sender: owner-obm-l@sucuri.mat.puc-rio.br Precedence: bulk Reply-To: obm-l@mat.puc-rio.br


De acordo com o dicionário etimológico da língua portuguesa(feito por portugueses, daí a resposta sem sentido - não se ofendam possíveis relativos à portugueses, isso é apenas uma brincadeira), vem de "mathematiké" - palavra grega que significa "ciência das matemáticas"

Sobre o pontinho...não será apenas por preguiça? hahaha

>From: "Victor Luiz"
>Reply-To: obm-l@mat.puc-rio.br
>To:
>Subject: [obm-l] Palavra "Matemática" e símbolo de multiplicação
>Date: Wed, 2 Apr 2003 03:07:13 -0300
>
>Eu gostaria de saber a origem da palavra "Matemática".
>
>Outra coisa que eu sempre quis saber é se existe algum motivo especial em
>depois de algumas séries substituir o símbolo de multiplicação que até então
>era usado um "x" por um ponto (Ou bolinha, dependendo do ponto de vista).
>Seria para os alunos não confundirem o sinal de multiplicação com a
>incógnita x?
>
>Grato,
>Victor Luiz Salgado de Lima.
>
>=========================================================================
>Instruções para entrar na lista, sair da lista e usar a lista em
>http://www.mat.puc-rio.br/~nicolau/olimp/obm-l.html
>O administrador desta lista é
>=========================================================================


MSN 8 helps ELIMINATE E-MAIL VIRUSES. Get 2 months FREE*. ========================================================================= Instruções para entrar na lista, sair da lista e usar a lista em http://www.mat.puc-rio.br/~nicolau/olimp/obm-l.html O administrador desta lista é ========================================================================= From owner-obm-l@sucuri.mat.puc-rio.br Wed Apr 2 08:47:49 2003 Return-Path: Received: (from majordom@localhost) by sucuri.mat.puc-rio.br (8.9.3/8.9.3) id IAA09843 for obm-l-MTTP; Wed, 2 Apr 2003 08:46:30 -0300 Received: from hotmail.com (f108.law9.hotmail.com [64.4.9.108]) by sucuri.mat.puc-rio.br (8.9.3/8.9.3) with ESMTP id IAA09838 for ; Wed, 2 Apr 2003 08:46:26 -0300 Received: from mail pickup service by hotmail.com with Microsoft SMTPSVC; Wed, 2 Apr 2003 03:45:55 -0800 Received: from 200.222.178.226 by lw9fd.law9.hotmail.msn.com with HTTP; Wed, 02 Apr 2003 11:45:54 GMT X-Originating-IP: [200.222.178.226] X-Originating-Email: [ricardoprins@hotmail.com] From: "Ricardo Prins" To: obm-l@mat.puc-rio.br Subject: Re: [obm-l] Palavra "Matemática" e símbolo de multiplicação Date: Wed, 02 Apr 2003 11:45:54 +0000 Mime-Version: 1.0 Content-Type: text/html Message-ID: X-OriginalArrivalTime: 02 Apr 2003 11:45:55.0019 (UTC) FILETIME=[6B5181B0:01C2F90D] Sender: owner-obm-l@sucuri.mat.puc-rio.br Precedence: bulk Reply-To: obm-l@mat.puc-rio.br
>From: "Victor Luiz"
>Reply-To: obm-l@mat.puc-rio.br
>To:
>Subject: [obm-l] Palavra "Matemática" e símbolo de multiplicação
>Date: Wed, 2 Apr 2003 03:07:13 -0300
>
>Eu gostaria de saber a origem da palavra "Matemática".
>
>Outra coisa que eu sempre quis saber é se existe algum motivo especial em
>depois de algumas séries substituir o símbolo de multiplicação que até então
>era usado um "x" por um ponto (Ou bolinha, dependendo do ponto de vista).
>Seria para os alunos não confundirem o sinal de multiplicação com a
>incógnita x?
>
>Grato,
>Victor Luiz Salgado de Lima.
>
>=========================================================================
>Instruções para entrar na lista, sair da lista e usar a lista em
>http://www.mat.puc-rio.br/~nicolau/olimp/obm-l.html
>O administrador desta lista é
>=========================================================================


Add photos to your e-mail with MSN 8. Get 2 months FREE*. ========================================================================= Instruções para entrar na lista, sair da lista e usar a lista em http://www.mat.puc-rio.br/~nicolau/olimp/obm-l.html O administrador desta lista é ========================================================================= From owner-obm-l@sucuri.mat.puc-rio.br Wed Apr 2 08:48:33 2003 Return-Path: Received: (from majordom@localhost) by sucuri.mat.puc-rio.br (8.9.3/8.9.3) id IAA09883 for obm-l-MTTP; Wed, 2 Apr 2003 08:47:15 -0300 Received: from web40509.mail.yahoo.com (web40509.mail.yahoo.com [66.218.78.126]) by sucuri.mat.puc-rio.br (8.9.3/8.9.3) with SMTP id IAA09871 for ; Wed, 2 Apr 2003 08:47:10 -0300 Message-ID: <20030402114638.68629.qmail@web40509.mail.yahoo.com> Received: from [200.180.172.87] by web40509.mail.yahoo.com via HTTP; Wed, 02 Apr 2003 08:46:38 ART Date: Wed, 2 Apr 2003 08:46:38 -0300 (ART) From: "=?iso-8859-1?q?J.C.=20PAREDE?=" Subject: Re: [obm-l] ajuda2_(correção) To: obm-l@mat.puc-rio.br In-Reply-To: <004801c2f3fa$5cfea100$c300000a@wsjujuba> MIME-Version: 1.0 Content-Type: multipart/alternative; boundary="0-156514777-1049283998=:67823" Content-Transfer-Encoding: 8bit Sender: owner-obm-l@sucuri.mat.puc-rio.br Precedence: bulk Reply-To: obm-l@mat.puc-rio.br --0-156514777-1049283998=:67823 Content-Type: text/plain; charset=iso-8859-1 Content-Transfer-Encoding: 8bit f (x) = -2x + 3 f(1) = -2*1 + 3 f(1) = 1 f(x) = x^2 - 4x + 4 f(1) = 1 - 4 + 4 = 1 ============= f(x) = -2x + 3 -1/5 = -2x + 3 2x = 3 + 1/5 x = (16/5) * (1/2) = 8/5 f(x) = x^2 - 4x + 4 x^2 - 4x + 4 = -1/5 x^2 - 4x + (21/5) = 0 x = (4 + - sqrt(16 - (4 * 1 * 21/5)) / 2 Helter Skelter wrote:2. Dada as funções f (x) = -2x + 3 e f (X) = x^2 - 4x + 4 , determine: a) f (1) b) f (x) = - 1/5 ========================================================================= Instruções para entrar na lista, sair da lista e usar a lista em http://www.mat.puc-rio.br/~nicolau/olimp/obm-l.html O administrador desta lista é ========================================================================= JOÃO CARLOS PAREDE --------------------------------- Yahoo! Mail O melhor e-mail gratuito da internet: 6MB de espaço, antivírus, acesso POP3, filtro contra spam. --0-156514777-1049283998=:67823 Content-Type: text/html; charset=iso-8859-1 Content-Transfer-Encoding: 8bit

f (x) = -2x + 3

f(1) = -2*1 + 3

f(1) = 1

f(x) = x^2 - 4x + 4

f(1) = 1 - 4 + 4 = 1

=============

f(x) = -2x + 3

-1/5 = -2x + 3

2x = 3 + 1/5

x = (16/5) * (1/2) = 8/5

f(x) = x^2 - 4x + 4

x^2 - 4x + 4 = -1/5

x^2 - 4x + (21/5) = 0

x = (4 + - sqrt(16 - (4 * 1 * 21/5)) / 2 

 Helter Skelter <helterskelter@terra.com.br> wrote:

2. Dada as funções f (x) = -2x + 3 e f (X) = x^2 - 4x + 4 , determine:

a) f (1)

b) f (x) = - 1/5


=========================================================================
Instruções para entrar na lista, sair da lista e usar a lista em
http://www.mat.puc-rio.br/~nicolau/olimp/obm-l.html
O administrador desta lista é
=========================================================================


 JOÃO CARLOS PAREDE



Yahoo! Mail
O melhor e-mail gratuito da internet: 6MB de espaço, antivírus, acesso POP3, filtro contra spam. --0-156514777-1049283998=:67823-- ========================================================================= Instruções para entrar na lista, sair da lista e usar a lista em http://www.mat.puc-rio.br/~nicolau/olimp/obm-l.html O administrador desta lista é ========================================================================= From owner-obm-l@sucuri.mat.puc-rio.br Wed Apr 2 08:51:07 2003 Return-Path: Received: (from majordom@localhost) by sucuri.mat.puc-rio.br (8.9.3/8.9.3) id IAA10109 for obm-l-MTTP; Wed, 2 Apr 2003 08:49:43 -0300 Received: from web40505.mail.yahoo.com (web40505.mail.yahoo.com [66.218.78.122]) by sucuri.mat.puc-rio.br (8.9.3/8.9.3) with SMTP id IAA10104 for ; Wed, 2 Apr 2003 08:49:38 -0300 Message-ID: <20030402114907.92290.qmail@web40505.mail.yahoo.com> Received: from [200.180.172.87] by web40505.mail.yahoo.com via HTTP; Wed, 02 Apr 2003 08:49:07 ART Date: Wed, 2 Apr 2003 08:49:07 -0300 (ART) From: "=?iso-8859-1?q?J.C.=20PAREDE?=" Subject: Re: [obm-l] ajuda6 To: obm-l@mat.puc-rio.br In-Reply-To: <005801c2f3fa$eed930e0$c300000a@wsjujuba> MIME-Version: 1.0 Content-Type: multipart/alternative; boundary="0-435745439-1049284147=:91012" Content-Transfer-Encoding: 8bit Sender: owner-obm-l@sucuri.mat.puc-rio.br Precedence: bulk Reply-To: obm-l@mat.puc-rio.br --0-435745439-1049284147=:91012 Content-Type: text/plain; charset=iso-8859-1 Content-Transfer-Encoding: 8bit Como são funções do tipo f(x) = ax + b, se a é positivo a função é crescente e se a é negativo a função é decrescente. Helter Skelter wrote:6. Classifique as funções em crescentes ou decrescentes: a) f (x) = x - 4 b) f(x) = - 3x + 2 c) f (x) = 2x ========================================================================= Instruções para entrar na lista, sair da lista e usar a lista em http://www.mat.puc-rio.br/~nicolau/olimp/obm-l.html O administrador desta lista é ========================================================================= JOÃO CARLOS PAREDE --------------------------------- Yahoo! Mail O melhor e-mail gratuito da internet: 6MB de espaço, antivírus, acesso POP3, filtro contra spam. --0-435745439-1049284147=:91012 Content-Type: text/html; charset=iso-8859-1 Content-Transfer-Encoding: 8bit

Como são funções do tipo f(x) = ax + b, se a é positivo a função é crescente e se a é negativo a função é decrescente.

 Helter Skelter <helterskelter@terra.com.br> wrote:

6. Classifique as funções em crescentes ou decrescentes:

a) f (x) = x - 4

b) f(x) = - 3x + 2

c) f (x) = 2x


=========================================================================
Instruções para entrar na lista, sair da lista e usar a lista em
http://www.mat.puc-rio.br/~nicolau/olimp/obm-l.html
O administrador desta lista é
=========================================================================


 JOÃO CARLOS PAREDE



Yahoo! Mail
O melhor e-mail gratuito da internet: 6MB de espaço, antivírus, acesso POP3, filtro contra spam. --0-435745439-1049284147=:91012-- ========================================================================= Instruções para entrar na lista, sair da lista e usar a lista em http://www.mat.puc-rio.br/~nicolau/olimp/obm-l.html O administrador desta lista é ========================================================================= From owner-obm-l@sucuri.mat.puc-rio.br Wed Apr 2 11:33:35 2003 Return-Path: Received: (from majordom@localhost) by sucuri.mat.puc-rio.br (8.9.3/8.9.3) id LAA14692 for obm-l-MTTP; Wed, 2 Apr 2003 11:31:39 -0300 Received: from mafalda.rantac.com.br ([200.245.60.143]) by sucuri.mat.puc-rio.br (8.9.3/8.9.3) with SMTP id LAA14687 for ; Wed, 2 Apr 2003 11:31:34 -0300 Received: (qmail 31678 invoked from network); 2 Apr 2003 14:30:30 -0000 Received: from unknown (HELO oswaldostanziola) (200.243.181.185) by 0 with SMTP; 2 Apr 2003 14:30:30 -0000 Message-ID: <002f01c2f924$b1c71b70$b9b5f3c8@oswaldostanziola> From: "Oswaldo Stanziola" To: Subject: [obm-l] limites Date: Wed, 2 Apr 2003 11:32:31 -0300 MIME-Version: 1.0 Content-Type: multipart/alternative; boundary="----=_NextPart_000_002C_01C2F90B.8C1F4050" X-Priority: 3 X-MSMail-Priority: Normal X-Mailer: Microsoft Outlook Express 6.00.2800.1106 X-MimeOLE: Produced By Microsoft MimeOLE V6.00.2800.1106 Sender: owner-obm-l@sucuri.mat.puc-rio.br Precedence: bulk Reply-To: obm-l@mat.puc-rio.br This is a multi-part message in MIME format. ------=_NextPart_000_002C_01C2F90B.8C1F4050 Content-Type: text/plain; charset="iso-8859-1" Content-Transfer-Encoding: quoted-printable Olah pessoal, Agradeceria muito pela ajuda na resoluc=E3o do exercicio: Sendo f(x) =3D ( tg x - x)/( x - sen x) entao f(x) eh: x->0=20 Resp.: 2 Obrigado. Oswaldo stanii@rantac.com.br ------=_NextPart_000_002C_01C2F90B.8C1F4050 Content-Type: text/html; charset="iso-8859-1" Content-Transfer-Encoding: quoted-printable
Olah pessoal,
 
Agradeceria muito pela ajuda na = resoluc=E3o do=20 exercicio:
Sendo f(x) =3D ( tg x - x)/( x - sen x) = entao f(x)=20 eh:
          &nbs= p;            = ;            =             &= nbsp;        =20 x->0
Resp.: 2
 
Obrigado.
Oswaldo
stanii@rantac.com.br
 
------=_NextPart_000_002C_01C2F90B.8C1F4050-- ========================================================================= Instruções para entrar na lista, sair da lista e usar a lista em http://www.mat.puc-rio.br/~nicolau/olimp/obm-l.html O administrador desta lista é ========================================================================= From owner-obm-l@sucuri.mat.puc-rio.br Wed Apr 2 12:51:19 2003 Return-Path: Received: (from majordom@localhost) by sucuri.mat.puc-rio.br (8.9.3/8.9.3) id MAA16263 for obm-l-MTTP; Wed, 2 Apr 2003 12:49:07 -0300 Received: from ns3bind.localdomain ([200.230.34.5]) by sucuri.mat.puc-rio.br (8.9.3/8.9.3) with ESMTP id MAA16258 for ; Wed, 2 Apr 2003 12:49:02 -0300 Received: from servico2 ([200.230.34.229]) by ns3bind.localdomain (8.11.6/X.XX.X) with SMTP id h32Fj5815420 for ; Wed, 2 Apr 2003 12:45:06 -0300 Message-ID: <006b01c2f92f$6424aee0$3300c57d@bovespa.com> From: "=?iso-8859-1?Q?Cl=E1udio_\=28Pr=E1tica\=29?=" To: References: <002f01c2f924$b1c71b70$b9b5f3c8@oswaldostanziola> Subject: Re: [obm-l] limites Date: Wed, 2 Apr 2003 12:48:59 -0300 MIME-Version: 1.0 Content-Type: multipart/alternative; boundary="----=_NextPart_000_0068_01C2F916.3AD92020" X-Priority: 3 X-MSMail-Priority: Normal X-Mailer: Microsoft Outlook Express 5.50.4920.2300 X-MimeOLE: Produced By Microsoft MimeOLE V5.50.4920.2300 Sender: owner-obm-l@sucuri.mat.puc-rio.br Precedence: bulk Reply-To: obm-l@mat.puc-rio.br This is a multi-part message in MIME format. ------=_NextPart_000_0068_01C2F916.3AD92020 Content-Type: text/plain; charset="iso-8859-1" Content-Transfer-Encoding: quoted-printable f(x) =3D (sen(x)/cos(x) - x)/(x - sen(x)) =3D (sen(x) - = x*cos(x))/[cos(x)*(x - sen(x))] =3D =3D [(sen(x)/x) - cos(x)] / [cos(x)*(1 - sen(x)/x)] Usando os primeiros dois termos das s=E9ries de Taylor de seno e = cosseno, teremos: sen(x)/x =3D 1 - x^2/6 + O(x^4) e cos(x) =3D 1 - x^2/2 + O(x^4) Assim: f(x) =3D [x^2/2 - x^2/6 + O(x^4)] / [(1 - x^2/2 + O(x^4))*(x^2/6 + = O(x^4))] =3D [1/2 - 1/6 + O(x^4)/x^2] / [1/6 + O(x^4)/x^2]=20 Logo, quando x -> 0, f(x) -> (1/2 - 1/6)/(1/6) =3D 2. Um abra=E7o, Claudio.=20 ----- Original Message -----=20 From: Oswaldo Stanziola=20 To: obm-l@mat.puc-rio.br=20 Sent: Wednesday, April 02, 2003 11:32 AM Subject: [obm-l] limites Olah pessoal, Agradeceria muito pela ajuda na resoluc=E3o do exercicio: Sendo f(x) =3D ( tg x - x)/( x - sen x) entao f(x) eh: x->0=20 Resp.: 2 Obrigado. Oswaldo stanii@rantac.com.br ------=_NextPart_000_0068_01C2F916.3AD92020 Content-Type: text/html; charset="iso-8859-1" Content-Transfer-Encoding: quoted-printable
f(x) =3D (sen(x)/cos(x) - x)/(x - = sen(x)) =3D (sen(x) -=20 x*cos(x))/[cos(x)*(x - sen(x))] =3D
=3D [(sen(x)/x) - cos(x)] / [cos(x)*(1 = -=20 sen(x)/x)]
 
Usando os primeiros dois termos das = s=E9ries de=20 Taylor de seno e cosseno, teremos:
 
sen(x)/x =3D 1 - x^2/6 + = O(x^4)  =20 e   cos(x) =3D 1 - x^2/2 + O(x^4)
 
Assim:
f(x) =3D [x^2/2 - x^2/6 + O(x^4)] / [(1 = - x^2/2 +=20 O(x^4))*(x^2/6 + O(x^4))]
 
=3D [1/2 - 1/6 + O(x^4)/x^2] / = [1/6 +=20 O(x^4)/x^2]
 
Logo, quando x -> 0, f(x) -> (1/2 = -=20 1/6)/(1/6) =3D 2.
 
Um abra=E7o,
Claudio. 
 
----- Original Message -----
From:=20 Oswaldo=20 Stanziola
Sent: Wednesday, April 02, 2003 = 11:32=20 AM
Subject: [obm-l] limites

Olah pessoal,
 
Agradeceria muito pela ajuda na = resoluc=E3o do=20 exercicio:
Sendo f(x) =3D ( tg x - x)/( x - sen = x) entao f(x)=20 eh:
          &nbs= p;            = ;            =             &= nbsp;        =20 x->0
Resp.: 2
 
Obrigado.
Oswaldo
stanii@rantac.com.br
 
------=_NextPart_000_0068_01C2F916.3AD92020-- ========================================================================= Instruções para entrar na lista, sair da lista e usar a lista em http://www.mat.puc-rio.br/~nicolau/olimp/obm-l.html O administrador desta lista é ========================================================================= From owner-obm-l@sucuri.mat.puc-rio.br Wed Apr 2 12:54:28 2003 Return-Path: Received: (from majordom@localhost) by sucuri.mat.puc-rio.br (8.9.3/8.9.3) id MAA16339 for obm-l-MTTP; Wed, 2 Apr 2003 12:53:04 -0300 Received: from hotmail.com (f84.sea1.hotmail.com [207.68.163.84]) by sucuri.mat.puc-rio.br (8.9.3/8.9.3) with ESMTP id MAA16335 for ; Wed, 2 Apr 2003 12:53:00 -0300 Received: from mail pickup service by hotmail.com with Microsoft SMTPSVC; Wed, 2 Apr 2003 07:52:28 -0800 Received: from 198.81.9.3 by sea1fd.sea1.hotmail.msn.com with HTTP; Wed, 02 Apr 2003 15:52:28 GMT X-Originating-IP: [198.81.9.3] X-Originating-Email: [fredericor@hotmail.com] From: "Frederico Reis Marques de Brito" To: obm-l@mat.puc-rio.br Subject: [obm-l] =?iso-8859-1?B?UmU6IFtvYm0tbF0gUGFsYXZyYSAiTWF0ZW3hdGljYSIgZSBz7W1ib2xv?= =?iso-8859-1?B?IGRlIG11bHRpcGxpY2Hn428=?= Date: Wed, 02 Apr 2003 12:52:28 -0300 Mime-Version: 1.0 Content-Type: multipart/mixed; boundary="----=_NextPart_000_70b_4825_3efc" Message-ID: X-OriginalArrivalTime: 02 Apr 2003 15:52:28.0814 (UTC) FILETIME=[DD1B7AE0:01C2F92F] Sender: owner-obm-l@sucuri.mat.puc-rio.br Precedence: bulk Reply-To: obm-l@mat.puc-rio.br This is a multi-part message in MIME format. ------=_NextPart_000_70b_4825_3efc Content-Type: text/plain; charset=iso-8859-1; format=flowed Ao que parece, a palavra matemática foi introduzida por Pitágoras e significava " Aquilo que se pode aprender", sentido que esorbita a ciência matemática. Quanto ao uso do símbolo para a multiplicação, simplificações notacionais são uma constante obcessão dos matemáticos. Inclusive, quase sempre usamos apenas a adjacência xy para indicar x vezes y. Não sei se foi intencionalmente para não confundir-se a variável x com o símbolo do produto, mas sem dúvida a nova notação evita essa ambiguidade, embora os símbolos não fossem idênticos. Fred. >From: "Ricardo Prins" >Reply-To: obm-l@mat.puc-rio.br >To: obm-l@mat.puc-rio.br >Subject: Re: [obm-l] Palavra "Matemática" e símbolo de multiplicação >Date: Wed, 02 Apr 2003 11:45:54 +0000 > _________________________________________________________________ MSN Hotmail, o maior webmail do Brasil. http://www.hotmail.com ------=_NextPart_000_70b_4825_3efc Content-Type: message/rfc822 X-Message-Info: N4u0pqWW+O0FK/zWy/zAaJtag4N4KGPV Received: from mc3-f26.law16.hotmail.com ([65.54.236.161]) by mc3-s7.law16.hotmail.com with Microsoft SMTPSVC(5.0.2195.5600); Wed, 2 Apr 2003 04:15:57 -0800 Received: from sucuri.mat.puc-rio.br ([139.82.27.7]) by mc3-f26.law16.hotmail.com with Microsoft SMTPSVC(5.0.2195.5600); Wed, 2 Apr 2003 04:13:40 -0800 Received: (from majordom@localhost) by sucuri.mat.puc-rio.br (8.9.3/8.9.3) id IAA09843 for obm-l-MTTP; Wed, 2 Apr 2003 08:46:30 -0300 Received: from hotmail.com (f108.law9.hotmail.com [64.4.9.108]) by sucuri.mat.puc-rio.br (8.9.3/8.9.3) with ESMTP id IAA09838 for ; Wed, 2 Apr 2003 08:46:26 -0300 Received: from mail pickup service by hotmail.com with Microsoft SMTPSVC; Wed, 2 Apr 2003 03:45:55 -0800 Received: from 200.222.178.226 by lw9fd.law9.hotmail.msn.com with HTTP; Wed, 02 Apr 2003 11:45:54 GMT X-Originating-IP: [200.222.178.226] X-Originating-Email: [ricardoprins@hotmail.com] From: "Ricardo Prins" To: obm-l@mat.puc-rio.br Subject: Re: [obm-l] Palavra "Matemática" e símbolo de multiplicação Date: Wed, 02 Apr 2003 11:45:54 +0000 Mime-Version: 1.0 Content-Type: text/html Message-ID: X-OriginalArrivalTime: 02 Apr 2003 11:45:55.0019 (UTC) FILETIME=[6B5181B0:01C2F90D] Sender: owner-obm-l@sucuri.mat.puc-rio.br Precedence: bulk Reply-To: obm-l@mat.puc-rio.br Return-Path: owner-obm-l@sucuri.mat.puc-rio.br
>From: "Victor Luiz"
>Reply-To: obm-l@mat.puc-rio.br
>To:
>Subject: [obm-l] Palavra "Matemática" e símbolo de multiplicação
>Date: Wed, 2 Apr 2003 03:07:13 -0300
>
>Eu gostaria de saber a origem da palavra "Matemática".
>
>Outra coisa que eu sempre quis saber é se existe algum motivo especial em
>depois de algumas séries substituir o símbolo de multiplicação que até então
>era usado um "x" por um ponto (Ou bolinha, dependendo do ponto de vista).
>Seria para os alunos não confundirem o sinal de multiplicação com a
>incógnita x?
>
>Grato,
>Victor Luiz Salgado de Lima.
>
>=========================================================================
>Instruções para entrar na lista, sair da lista e usar a lista em
>http://www.mat.puc-rio.br/~nicolau/olimp/obm-l.html
>O administrador desta lista é
>=========================================================================


Add photos to your e-mail with MSN 8. Get 2 months FREE*. ========================================================================= Instruções para entrar na lista, sair da lista e usar a lista em http://www.mat.puc-rio.br/~nicolau/olimp/obm-l.html O administrador desta lista é ========================================================================= ------=_NextPart_000_70b_4825_3efc-- ========================================================================= Instruções para entrar na lista, sair da lista e usar a lista em http://www.mat.puc-rio.br/~nicolau/olimp/obm-l.html O administrador desta lista é ========================================================================= From owner-obm-l@sucuri.mat.puc-rio.br Wed Apr 2 13:08:32 2003 Return-Path: Received: (from majordom@localhost) by sucuri.mat.puc-rio.br (8.9.3/8.9.3) id NAA16888 for obm-l-MTTP; Wed, 2 Apr 2003 13:07:05 -0300 Received: from pagu.dsce.fee.unicamp.br (pagu.dsce.fee.unicamp.br [143.106.14.130]) by sucuri.mat.puc-rio.br (8.9.3/8.9.3) with ESMTP id NAA16883 for ; Wed, 2 Apr 2003 13:06:56 -0300 Received: from monaco (monaco.dsce.fee.unicamp.br [143.106.14.182]) by pagu.dsce.fee.unicamp.br (8.9.3/8.9.3) with SMTP id NAA16481 for ; Wed, 2 Apr 2003 13:01:08 -0300 (EST) Message-ID: <000801c2f932$dc75bda0$b60e6a8f@monaco> From: =?iso-8859-1?Q?M=E1rcio_Ven=EDcio_Pilar_Alc=E2ntara?= To: References: <3E8869D000000863@www.zipmail.com.br> <045401c2f7c5$c3b481c0$3300c57d@bovespa.com> <001201c2f7da$78f41af0$0578a492@ARISTOTELES> <000e01c2f7e3$0d42e580$850e6a8f@malfatti> Subject: [obm-l] =?iso-8859-1?Q?Integral_=28Ningu=E9m_se_habilita=3F=29?= Date: Wed, 2 Apr 2003 13:13:55 -0300 Organization: UNICAMP - FEE - DSCE MIME-Version: 1.0 Content-Type: text/plain; charset="iso-8859-1" Content-Transfer-Encoding: 8bit X-Priority: 3 X-MSMail-Priority: Normal X-Mailer: Microsoft Outlook Express 6.00.2800.1106 X-MimeOLE: Produced By Microsoft MimeOLE V6.00.2800.1106 Sender: owner-obm-l@sucuri.mat.puc-rio.br Precedence: bulk Reply-To: obm-l@mat.puc-rio.br Alguém sabe me dizer como eu calculo a integral indefinida de x^x (x elevado a x)? Consegui calcular a derivada de y = x^x como sendo y' = (1 + lnx) . x^x Aguardo solução de alguém, Márcio Venício P. Alcântara http://www.marcio.ezdir.net marcio@dsce.fee.unicamp.br Departamento de Sistemas e Controle de Energia (DSCE) Faculdade de Engenharia Elétrica e Computação (FEEC) UNICAMP - Campinas - SP - Brasil ========================================================================= Instruções para entrar na lista, sair da lista e usar a lista em http://www.mat.puc-rio.br/~nicolau/olimp/obm-l.html O administrador desta lista é ========================================================================= From owner-obm-l@sucuri.mat.puc-rio.br Wed Apr 2 14:47:12 2003 Return-Path: Received: (from majordom@localhost) by sucuri.mat.puc-rio.br (8.9.3/8.9.3) id OAA19349 for obm-l-MTTP; Wed, 2 Apr 2003 14:44:37 -0300 Received: from ns3bind.localdomain ([200.230.34.5]) by sucuri.mat.puc-rio.br (8.9.3/8.9.3) with ESMTP id OAA19336 for ; Wed, 2 Apr 2003 14:44:32 -0300 Received: from servico2 ([200.230.34.224]) by ns3bind.localdomain (8.11.6/X.XX.X) with SMTP id h32HeZ823437 for ; Wed, 2 Apr 2003 14:40:35 -0300 Message-ID: <007501c2f93f$876fc460$3300c57d@bovespa.com> From: "=?iso-8859-1?Q?Cl=E1udio_\=28Pr=E1tica\=29?=" To: References: <3E8869D000000863@www.zipmail.com.br> <045401c2f7c5$c3b481c0$3300c57d@bovespa.com> <001201c2f7da$78f41af0$0578a492@ARISTOTELES> <000e01c2f7e3$0d42e580$850e6a8f@malfatti> <000801c2f932$dc75bda0$b60e6a8f@monaco> Subject: [obm-l] =?iso-8859-1?Q?Re:_=5Bobm-l=5D_Integral_=28Ningu=E9m_se_habilita=3F=29?= Date: Wed, 2 Apr 2003 14:44:29 -0300 MIME-Version: 1.0 Content-Type: text/plain; charset="iso-8859-1" Content-Transfer-Encoding: 8bit X-Priority: 3 X-MSMail-Priority: Normal X-Mailer: Microsoft Outlook Express 5.50.4920.2300 X-MimeOLE: Produced By Microsoft MimeOLE V5.50.4920.2300 Sender: owner-obm-l@sucuri.mat.puc-rio.br Precedence: bulk Reply-To: obm-l@mat.puc-rio.br Oi, Márcio: Não tenho certeza mas acho que a integral indefinida de x^x = e^(x*Ln(x)) não se expressa como uma combinação de funções elementares. Pelo menos não consta da tabela de integrais do Manual de Fórmulas e Tabelas Matemáticas da Coleção Schaum, que é a mais completa que eu conheço. Por outro lado, lá tem a fórmula: INTEGRAL(1 a +infinito) dx/x^x = SOMA(n = 1 a +infinito) 1/n^n. Um abraço, Claudio. ----- Original Message ----- From: "Márcio Venício Pilar Alcântara" To: Sent: Wednesday, April 02, 2003 1:13 PM Subject: [obm-l] Integral (Ninguém se habilita?) > Alguém sabe me dizer como eu calculo a integral indefinida de x^x (x elevado > a x)? > Consegui calcular a derivada de y = x^x como sendo y' = (1 + lnx) . x^x > > Aguardo solução de alguém, > > Márcio Venício P. Alcântara > http://www.marcio.ezdir.net > marcio@dsce.fee.unicamp.br > Departamento de Sistemas e Controle de Energia (DSCE) > Faculdade de Engenharia Elétrica e Computação (FEEC) > UNICAMP - Campinas - SP - Brasil > > ========================================================================= > Instruções para entrar na lista, sair da lista e usar a lista em > http://www.mat.puc-rio.br/~nicolau/olimp/obm-l.html > O administrador desta lista é > ========================================================================= ========================================================================= Instruções para entrar na lista, sair da lista e usar a lista em http://www.mat.puc-rio.br/~nicolau/olimp/obm-l.html O administrador desta lista é ========================================================================= From owner-obm-l@sucuri.mat.puc-rio.br Wed Apr 2 14:57:29 2003 Return-Path: Received: (from majordom@localhost) by sucuri.mat.puc-rio.br (8.9.3/8.9.3) id OAA19597 for obm-l-MTTP; Wed, 2 Apr 2003 14:55:52 -0300 Received: from hotmail.com (oe12.law10.hotmail.com [64.4.14.116]) by sucuri.mat.puc-rio.br (8.9.3/8.9.3) with ESMTP id OAA19593 for ; Wed, 2 Apr 2003 14:55:48 -0300 Received: from mail pickup service by hotmail.com with Microsoft SMTPSVC; Wed, 2 Apr 2003 09:55:16 -0800 Received: from 64.60.139.18 by oe12.law10.hotmail.com with DAV; Wed, 02 Apr 2003 17:55:16 +0000 X-Originating-IP: [64.60.139.18] X-Originating-Email: [lrecova@hotmail.com] From: =?iso-8859-1?Q?Leandro_Lacorte_Rec=F4va?= To: Subject: [obm-l] =?iso-8859-1?Q?RE:_=5Bobm-l=5D_Integral_=28Ningu=E9m_se_habilita=3F=29?= Date: Wed, 2 Apr 2003 09:55:17 -0800 Message-ID: <002a01c2f941$052eaa50$28029b9b@LeandroRecova> MIME-Version: 1.0 Content-Type: text/plain; charset="iso-8859-1" X-Priority: 3 (Normal) X-MSMail-Priority: Normal X-Mailer: Microsoft Outlook, Build 10.0.3416 X-MimeOLE: Produced By Microsoft MimeOLE V6.00.2800.1106 Importance: Normal In-Reply-To: <000801c2f932$dc75bda0$b60e6a8f@monaco> X-OriginalArrivalTime: 02 Apr 2003 17:55:16.0921 (UTC) FILETIME=[04D76290:01C2F941] Content-Transfer-Encoding: 8bit X-MIME-Autoconverted: from quoted-printable to 8bit by sucuri.mat.puc-rio.br id OAA19594 Sender: owner-obm-l@sucuri.mat.puc-rio.br Precedence: bulk Reply-To: obm-l@mat.puc-rio.br Voce ja tentou algo usando o teorema dos residuos ou Integral de Cauchy ? -----Original Message----- From: owner-obm-l@sucuri.mat.puc-rio.br [mailto:owner-obm-l@sucuri.mat.puc-rio.br] On Behalf Of Márcio Venício Pilar Alcântara Sent: Wednesday, April 02, 2003 8:14 AM To: obm-l@mat.puc-rio.br Subject: [obm-l] Integral (Ninguém se habilita?) Alguém sabe me dizer como eu calculo a integral indefinida de x^x (x elevado a x)? Consegui calcular a derivada de y = x^x como sendo y' = (1 + lnx) . x^x Aguardo solução de alguém, Márcio Venício P. Alcântara http://www.marcio.ezdir.net marcio@dsce.fee.unicamp.br Departamento de Sistemas e Controle de Energia (DSCE) Faculdade de Engenharia Elétrica e Computação (FEEC) UNICAMP - Campinas - SP - Brasil ======================================================================== = Instruções para entrar na lista, sair da lista e usar a lista em http://www.mat.puc-rio.br/~nicolau/olimp/obm-l.html O administrador desta lista é ======================================================================== = ========================================================================= Instruções para entrar na lista, sair da lista e usar a lista em http://www.mat.puc-rio.br/~nicolau/olimp/obm-l.html O administrador desta lista é ========================================================================= From owner-obm-l@sucuri.mat.puc-rio.br Wed Apr 2 15:27:58 2003 Return-Path: Received: (from majordom@localhost) by sucuri.mat.puc-rio.br (8.9.3/8.9.3) id PAA20483 for obm-l-MTTP; Wed, 2 Apr 2003 15:26:12 -0300 Received: from smtp-29.ig.com.br (smtp-29.ig.com.br [200.226.132.157]) by sucuri.mat.puc-rio.br (8.9.3/8.9.3) with SMTP id PAA20479 for ; Wed, 2 Apr 2003 15:26:08 -0300 Received: (qmail 6832 invoked from network); 2 Apr 2003 18:25:35 -0000 Received: from 200-140-080-097.bsace7026.dsl.brasiltelecom.net.br (HELO henrique) (200.140.80.97) by smtp-29.ig.com.br with SMTP; 2 Apr 2003 18:25:35 -0000 Message-ID: <000501c2f945$3d152210$019da8c0@henrique> From: "=?iso-8859-1?Q?Henrique_Patr=EDcio_Sant'Anna_Branco?=" To: References: <3E8869D000000863@www.zipmail.com.br> <045401c2f7c5$c3b481c0$3300c57d@bovespa.com> <001201c2f7da$78f41af0$0578a492@ARISTOTELES> <000e01c2f7e3$0d42e580$850e6a8f@malfatti> <000801c2f932$dc75bda0$b60e6a8f@monaco> Subject: [obm-l] =?iso-8859-1?Q?Re:_=5Bobm-l=5D_Integral_=28Ningu=E9m_se_habilita=3F=29?= Date: Wed, 2 Apr 2003 15:25:28 -0300 MIME-Version: 1.0 Content-Type: text/plain; charset="iso-8859-1" Content-Transfer-Encoding: 8bit X-Priority: 3 X-MSMail-Priority: Normal X-Mailer: Microsoft Outlook Express 6.00.2800.1106 X-MimeOLE: Produced By Microsoft MimeOLE V6.00.2800.1106 Sender: owner-obm-l@sucuri.mat.puc-rio.br Precedence: bulk Reply-To: obm-l@mat.puc-rio.br > Alguém sabe me dizer como eu calculo a integral indefinida de x^x (x elevado > a x)? Essa função não é integrável segundo Riemman. Sobre a demonstração, eu estava pensando em uma usando o critério de Lebesge, mas não sei se está certo. Gostaria que algum membro da lista pudesse me apontar se eu errei e onde errei. Essa função é descontínua em zero. Sendo A o conjunto das descontinuidades desse função, temos que A = {0}. Portanto, tomando o intervalo real I = {-infinito,infinito), vemos que esse intervalo cobre A, pois A está contido nesse intervalo. Por outro lado, pela definição de conjunto de medida nula, temos que o somatório das amplitudes do intervalo I tende ao infinito e não podemos achar um epsilon maior que isso. Portanto, o conjunto não tem medida nula e, assim, não é integrável por Riemman. Deu pra entender? O problema dessa demonstração está no fato de que se A = {0}, então é enumerável e, portanto, não tem medida nula. Talvez eu tenha errado no conjunto das descontinuidades da função (como no ponto zero, temos 0^0, teríamos uma descontinuidade infinita?). Agradeço qualquer ajuda. Henrique. P.S. - Sou aluno de Estatística e ainda estou no Cálculo 1... Portanto, não sejam muito duros se eu falei muita besteira... :-) ========================================================================= Instruções para entrar na lista, sair da lista e usar a lista em http://www.mat.puc-rio.br/~nicolau/olimp/obm-l.html O administrador desta lista é ========================================================================= From owner-obm-l@sucuri.mat.puc-rio.br Wed Apr 2 16:12:20 2003 Return-Path: Received: (from majordom@localhost) by sucuri.mat.puc-rio.br (8.9.3/8.9.3) id QAA21901 for obm-l-MTTP; Wed, 2 Apr 2003 16:10:18 -0300 Received: from ivoti.terra.com.br (ivoti.terra.com.br [200.176.3.20]) by sucuri.mat.puc-rio.br (8.9.3/8.9.3) with ESMTP id QAA21877 for ; Wed, 2 Apr 2003 16:10:12 -0300 Received: from gunga.terra.com.br (gunga.terra.com.br [200.176.3.45]) by ivoti.terra.com.br (Postfix) with ESMTP id 09F31408B19 for ; Wed, 2 Apr 2003 16:09:40 -0300 (BRT) Received: from usuario (200-180-188-144.paemt7005.dsl.brasiltelecom.net.br [200.180.188.144]) (authenticated user marioappereira) by gunga.terra.com.br (Postfix) with ESMTP id BAC5B1280E2 for ; Wed, 2 Apr 2003 16:09:39 -0300 (BRT) Message-ID: <009901c2f94b$6a34e4f0$0301a8c0@usuario> From: =?iso-8859-1?Q?M=E1rio_Pereira?= To: Subject: [obm-l] Date: Wed, 2 Apr 2003 16:09:41 -0300 MIME-Version: 1.0 Content-Type: multipart/alternative; boundary="----=_NextPart_000_0096_01C2F932.44AE7480" X-Priority: 3 X-MSMail-Priority: Normal X-Mailer: Microsoft Outlook Express 6.00.2600.0000 X-MimeOLE: Produced By Microsoft MimeOLE V6.00.2600.0000 Sender: owner-obm-l@sucuri.mat.puc-rio.br Precedence: bulk Reply-To: obm-l@mat.puc-rio.br This is a multi-part message in MIME format. ------=_NextPart_000_0096_01C2F932.44AE7480 Content-Type: text/plain; charset="iso-8859-1" Content-Transfer-Encoding: quoted-printable =C9 possivel calcular a matriz inversa nos seguintes casos? Qual seria o = desenvolvimento? D =3D 1 0 0 1 3 1 1 2 0 E =3D -1 -1 2 2 1 -2 1 1 -1 Obrigado. Mario ------=_NextPart_000_0096_01C2F932.44AE7480 Content-Type: text/html; charset="iso-8859-1" Content-Transfer-Encoding: quoted-printable
=C9 possivel calcular a matriz inversa = nos seguintes=20 casos? Qual seria o desenvolvimento?
 
D =3D 1   0   = 0
      1  =20 3   1
      = 1  =20 2   0
 
 
E =3D -1   -1   = 2
      =20 2    1   -2
      =20 1    1  -1
 
 
Obrigado.
 
Mario
 
------=_NextPart_000_0096_01C2F932.44AE7480-- ========================================================================= Instruções para entrar na lista, sair da lista e usar a lista em http://www.mat.puc-rio.br/~nicolau/olimp/obm-l.html O administrador desta lista é ========================================================================= From owner-obm-l@sucuri.mat.puc-rio.br Wed Apr 2 16:15:47 2003 Return-Path: Received: (from majordom@localhost) by sucuri.mat.puc-rio.br (8.9.3/8.9.3) id QAA22174 for obm-l-MTTP; Wed, 2 Apr 2003 16:14:20 -0300 Received: from mafalda.rantac.com.br ([200.245.60.143]) by sucuri.mat.puc-rio.br (8.9.3/8.9.3) with SMTP id QAA22169 for ; Wed, 2 Apr 2003 16:14:13 -0300 Received: (qmail 29890 invoked from network); 2 Apr 2003 19:13:11 -0000 Received: from unknown (HELO oswaldostanziola) (200.243.181.185) by 0 with SMTP; 2 Apr 2003 19:13:11 -0000 Message-ID: <001101c2f94c$2f67d5c0$b9b5f3c8@oswaldostanziola> From: "Oswaldo Stanziola" To: References: <002f01c2f924$b1c71b70$b9b5f3c8@oswaldostanziola> <006b01c2f92f$6424aee0$3300c57d@bovespa.com> Subject: Re: [obm-l] limites Date: Wed, 2 Apr 2003 16:15:11 -0300 MIME-Version: 1.0 Content-Type: multipart/alternative; boundary="----=_NextPart_000_000E_01C2F933.09713E60" X-Priority: 3 X-MSMail-Priority: Normal X-Mailer: Microsoft Outlook Express 6.00.2800.1106 X-MimeOLE: Produced By Microsoft MimeOLE V6.00.2800.1106 Sender: owner-obm-l@sucuri.mat.puc-rio.br Precedence: bulk Reply-To: obm-l@mat.puc-rio.br This is a multi-part message in MIME format. ------=_NextPart_000_000E_01C2F933.09713E60 Content-Type: text/plain; charset="iso-8859-1" Content-Transfer-Encoding: quoted-printable Oi Claudio. Agradecido pela aten=E7=E3o. ----- Original Message -----=20 From: Cl=E1udio (Pr=E1tica)=20 To: obm-l@mat.puc-rio.br=20 Sent: Wednesday, April 02, 2003 12:48 PM Subject: Re: [obm-l] limites f(x) =3D (sen(x)/cos(x) - x)/(x - sen(x)) =3D (sen(x) - = x*cos(x))/[cos(x)*(x - sen(x))] =3D =3D [(sen(x)/x) - cos(x)] / [cos(x)*(1 - sen(x)/x)] Usando os primeiros dois termos das s=E9ries de Taylor de seno e = cosseno, teremos: sen(x)/x =3D 1 - x^2/6 + O(x^4) e cos(x) =3D 1 - x^2/2 + O(x^4) Assim: f(x) =3D [x^2/2 - x^2/6 + O(x^4)] / [(1 - x^2/2 + O(x^4))*(x^2/6 + = O(x^4))] =3D [1/2 - 1/6 + O(x^4)/x^2] / [1/6 + O(x^4)/x^2]=20 Logo, quando x -> 0, f(x) -> (1/2 - 1/6)/(1/6) =3D 2. Um abra=E7o, Claudio.=20 ----- Original Message -----=20 From: Oswaldo Stanziola=20 To: obm-l@mat.puc-rio.br=20 Sent: Wednesday, April 02, 2003 11:32 AM Subject: [obm-l] limites Olah pessoal, Agradeceria muito pela ajuda na resoluc=E3o do exercicio: Sendo f(x) =3D ( tg x - x)/( x - sen x) entao f(x) eh: x->0=20 Resp.: 2 Obrigado. Oswaldo stanii@rantac.com.br ------=_NextPart_000_000E_01C2F933.09713E60 Content-Type: text/html; charset="iso-8859-1" Content-Transfer-Encoding: quoted-printable
Oi Claudio.
 
Agradecido pela = aten=E7=E3o.
----- Original Message -----
From:=20 Cl=E1udio = (Pr=E1tica)
Sent: Wednesday, April 02, 2003 = 12:48=20 PM
Subject: Re: [obm-l] = limites

f(x) =3D (sen(x)/cos(x) - x)/(x - = sen(x)) =3D (sen(x)=20 - x*cos(x))/[cos(x)*(x - sen(x))] =3D
=3D [(sen(x)/x) - cos(x)] / = [cos(x)*(1 -=20 sen(x)/x)]
 
Usando os primeiros dois termos das = s=E9ries de=20 Taylor de seno e cosseno, teremos:
 
sen(x)/x =3D 1 - x^2/6 + = O(x^4)  =20 e   cos(x) =3D 1 - x^2/2 + O(x^4)
 
Assim:
f(x) =3D [x^2/2 - x^2/6 + O(x^4)] / = [(1 - x^2/2 +=20 O(x^4))*(x^2/6 + O(x^4))]
 
=3D [1/2 - 1/6 + O(x^4)/x^2] / = [1/6 +=20 O(x^4)/x^2]
 
Logo, quando x -> 0, f(x) -> = (1/2 -=20 1/6)/(1/6) =3D 2.
 
Um abra=E7o,
Claudio. 
 
----- Original Message -----
From:=20 Oswaldo=20 Stanziola
Sent: Wednesday, April 02, = 2003 11:32=20 AM
Subject: [obm-l] = limites

Olah pessoal,
 
Agradeceria muito pela ajuda na = resoluc=E3o do=20 exercicio:
Sendo f(x) =3D ( tg x - x)/( x - = sen x) entao=20 f(x) eh:
          &nbs= p;            = ;            =             &= nbsp;        =20 x->0
Resp.: 2
 
Obrigado.
Oswaldo
stanii@rantac.com.br
 
------=_NextPart_000_000E_01C2F933.09713E60-- ========================================================================= Instruções para entrar na lista, sair da lista e usar a lista em http://www.mat.puc-rio.br/~nicolau/olimp/obm-l.html O administrador desta lista é ========================================================================= From owner-obm-l@sucuri.mat.puc-rio.br Wed Apr 2 16:49:16 2003 Return-Path: Received: (from majordom@localhost) by sucuri.mat.puc-rio.br (8.9.3/8.9.3) id QAA23301 for obm-l-MTTP; Wed, 2 Apr 2003 16:47:35 -0300 Received: from web40510.mail.yahoo.com (web40510.mail.yahoo.com [66.218.78.127]) by sucuri.mat.puc-rio.br (8.9.3/8.9.3) with SMTP id QAA23294 for ; Wed, 2 Apr 2003 16:47:30 -0300 Message-ID: <20030402194658.52225.qmail@web40510.mail.yahoo.com> Received: from [200.180.172.87] by web40510.mail.yahoo.com via HTTP; Wed, 02 Apr 2003 16:46:58 ART Date: Wed, 2 Apr 2003 16:46:58 -0300 (ART) From: "=?iso-8859-1?q?J.C.=20PAREDE?=" Subject: Re: [obm-l] onde_está_o_erro???????? To: obm-l@mat.puc-rio.br In-Reply-To: MIME-Version: 1.0 Content-Type: multipart/alternative; boundary="0-1965516642-1049312818=:51960" Content-Transfer-Encoding: 8bit Sender: owner-obm-l@sucuri.mat.puc-rio.br Precedence: bulk Reply-To: obm-l@mat.puc-rio.br --0-1965516642-1049312818=:51960 Content-Type: text/plain; charset=iso-8859-1 Content-Transfer-Encoding: 8bit Na segunda para a terceira linha tu fazes uma divisão por zero!!! Fabricio Taschetto wrote:Olá pessoal, alguém pode me ajudar com o que segue abaixo ??? Acredito que seja pela pré-condição da equação, mas não tenho certeza. Se alguém puder me responder, ficaria muito agradecido.Abs Fabricio X2 - X2 = X2 - X2 X(X-X) = (X+X)(X-X) X = X+X X = 2X 1 = 2 ?!?!?! Onde está o erro??? JOÃO CARLOS PAREDE --------------------------------- Yahoo! Mail O melhor e-mail gratuito da internet: 6MB de espaço, antivírus, acesso POP3, filtro contra spam. --0-1965516642-1049312818=:51960 Content-Type: text/html; charset=iso-8859-1 Content-Transfer-Encoding: 8bit

Na segunda para a terceira linha tu fazes uma divisão por zero!!!

 Fabricio Taschetto <ftaschetto@santander.com.br> wrote:

Olá pessoal, alguém pode me ajudar com o que segue abaixo ??? Acredito que seja pela pré-condição da equação, mas não tenho certeza. Se alguém puder me responder, ficaria muito agradecido.
Abs
 
Fabricio

 

X2 - X2  = X2 - X2

X(X-X) = (X+X)(X-X)

X = X+X

X = 2X

1  =  2   ?!?!?!

Onde está o erro???


 JOÃO CARLOS PAREDE



Yahoo! Mail
O melhor e-mail gratuito da internet: 6MB de espaço, antivírus, acesso POP3, filtro contra spam. --0-1965516642-1049312818=:51960-- ========================================================================= Instruções para entrar na lista, sair da lista e usar a lista em http://www.mat.puc-rio.br/~nicolau/olimp/obm-l.html O administrador desta lista é ========================================================================= From owner-obm-l@sucuri.mat.puc-rio.br Wed Apr 2 16:51:12 2003 Return-Path: Received: (from majordom@localhost) by sucuri.mat.puc-rio.br (8.9.3/8.9.3) id QAA23354 for obm-l-MTTP; Wed, 2 Apr 2003 16:49:53 -0300 Received: from smtp-26.ig.com.br (smtp-26.ig.com.br [200.226.132.160]) by sucuri.mat.puc-rio.br (8.9.3/8.9.3) with SMTP id QAA23350 for ; Wed, 2 Apr 2003 16:49:48 -0300 Received: (qmail 18166 invoked from network); 2 Apr 2003 19:49:26 -0000 Received: from 200-140-080-097.bsace7026.dsl.brasiltelecom.net.br (HELO henrique) (200.140.80.97) by smtp-26.ig.com.br with SMTP; 2 Apr 2003 19:49:26 -0000 Message-ID: <001e01c2f950$f2f900a0$019da8c0@henrique> From: "=?iso-8859-1?Q?Henrique_Patr=EDcio_Sant'Anna_Branco?=" To: "OBM" Subject: [obm-l] O problema do andarilho Date: Wed, 2 Apr 2003 16:49:18 -0300 MIME-Version: 1.0 Content-Type: text/plain; charset="iso-8859-1" Content-Transfer-Encoding: 8bit X-Priority: 3 X-MSMail-Priority: Normal X-Mailer: Microsoft Outlook Express 6.00.2800.1106 X-MimeOLE: Produced By Microsoft MimeOLE V6.00.2800.1106 Sender: owner-obm-l@sucuri.mat.puc-rio.br Precedence: bulk Reply-To: obm-l@mat.puc-rio.br Alguem poderia me ajudar com esse? Uma trilha vai da base de uma montanha até o topo. Um andarilho começa a subir a trilha às 6 horas da manhã e chega ao topo às 6 horas da tarde do mesmo dia. Durante o percurso ele pode parar, voltar atrás, correr, fazer o que quiser desde que chegue ao topo às 6 horas da tarde do mesmo dia. Na manhã seguinte ele começa a descer a trilha às 6 horas da manhã do modo como ele quiser e chega à base exatamente às 6 horas da tarde do mesmo dia. Prove que existe pelo menos um lugar na trilha pelo qual ele passa na mesma hora de cada dia. Grato, Henrique. ========================================================================= Instruções para entrar na lista, sair da lista e usar a lista em http://www.mat.puc-rio.br/~nicolau/olimp/obm-l.html O administrador desta lista é ========================================================================= From owner-obm-l@sucuri.mat.puc-rio.br Wed Apr 2 17:09:33 2003 Return-Path: Received: (from majordom@localhost) by sucuri.mat.puc-rio.br (8.9.3/8.9.3) id RAA24351 for obm-l-MTTP; Wed, 2 Apr 2003 17:08:00 -0300 Received: from imo-m07.mx.aol.com (imo-m07.mx.aol.com [64.12.136.162]) by sucuri.mat.puc-rio.br (8.9.3/8.9.3) with ESMTP id RAA24337 for ; Wed, 2 Apr 2003 17:07:54 -0300 From: DEOLIVEIRASOU@aol.com Received: from DEOLIVEIRASOU@aol.com by imo-m07.mx.aol.com (mail_out_v34.21.) id z.141.e4dbe57 (4539) for ; Wed, 2 Apr 2003 15:07:18 -0500 (EST) Message-ID: <141.e4dbe57.2bbc9cf6@aol.com> Date: Wed, 2 Apr 2003 15:07:18 EST Subject: [obm-l] =?ISO-8859-1?Q?olimp=EDadas=20ao=20redor=20do=20mundo....?= To: obm-l@mat.puc-rio.br MIME-Version: 1.0 Content-Type: multipart/alternative; boundary="part1_141.e4dbe57.2bbc9cf6_boundary" X-Mailer: 7.0 for Windows sub 10501 Sender: owner-obm-l@sucuri.mat.puc-rio.br Precedence: bulk Reply-To: obm-l@mat.puc-rio.br --part1_141.e4dbe57.2bbc9cf6_boundary Content-Type: text/plain; charset="ISO-8859-1" Content-Transfer-Encoding: quoted-printable E a=ED rapaziada....quero perguntar uma coisa sobre o problema abaixo... 1) Determine n natural, tais que n^2+2 divida 2+2001n. Indo direto a=20 defini=E7=E3o, existe k inteiro tal que 2+2001n=3Dn^2*k+2K. A equa=E7=E3o do= segundo=20 grau subjacente tr=E1z delta=3D2001^2-8k(k-1). S=F3 existe n natural se delt= a for=20 um quadrado perfeito....como determinar os valores de k para que isso=20 aconte=E7a?? No bra=E7o??? Se algu=E9m souber, agrade=E7o a ajuda...se alguem conhece outra forma=20= de=20 resolver adoraria conhecer tamb=E9m. Vou aproveitar e mandar outro. 2) No gr=E1fico da par=E1bola y=3Dx^2 no pano cartesiano marcamos os pontos= A, B e=20 C(com A entre B e C). No segmento BC marca-se o ponto N de modo que AN seja=20 paralelo ao eixo das ordenadas. Se S1 e S2 s=E3o as =E1reas dos tri=E2ngulos= ABN e=20 ACN, respectivamente, determine a medida do segmento AN.=20 --part1_141.e4dbe57.2bbc9cf6_boundary Content-Type: text/html; charset="ISO-8859-1" Content-Transfer-Encoding: quoted-printable E a=ED rapaziada....quero perguntar uma coisa sobre o=20= problema abaixo...
1) Determine n natural, tais que n^2+2 divida 2+2001n. Indo direto a defini= =E7=E3o, existe k inteiro tal que 2+2001n=3Dn^2*k+2K. A equa=E7=E3o do segun= do grau subjacente tr=E1z delta=3D2001^2-8k(k-1). S=F3 existe n natural se d= elta for um quadrado perfeito....como determinar os valores de k para que is= so aconte=E7a?? No bra=E7o???
     Se algu=E9m souber, agrade=E7o a ajuda...se alguem=20= conhece outra forma de resolver adoraria conhecer tamb=E9m. Vou aproveitar e= mandar outro.
2) No gr=E1fico da par=E1bola y=3Dx^2 no pano cartesiano marcamos os pontos= A, B e C(com A entre B e C). No segmento BC marca-se o ponto N de modo que=20= AN seja paralelo ao eixo das ordenadas. Se S1 e S2 s=E3o as =E1reas dos tri= =E2ngulos ABN e ACN, respectivamente, determine a medida do segmento AN. --part1_141.e4dbe57.2bbc9cf6_boundary-- ========================================================================= Instruções para entrar na lista, sair da lista e usar a lista em http://www.mat.puc-rio.br/~nicolau/olimp/obm-l.html O administrador desta lista é ========================================================================= From owner-obm-l@sucuri.mat.puc-rio.br Wed Apr 2 18:44:19 2003 Return-Path: Received: (from majordom@localhost) by sucuri.mat.puc-rio.br (8.9.3/8.9.3) id SAA26935 for obm-l-MTTP; Wed, 2 Apr 2003 18:42:34 -0300 Received: from krypton.hosting4u.net (krypton.hosting4u.net [209.15.2.78]) by sucuri.mat.puc-rio.br (8.9.3/8.9.3) with ESMTP id SAA26930 for ; Wed, 2 Apr 2003 18:42:30 -0300 Received: from gargamel (200-158-200-134.dsl.telesp.net.br [200.158.200.134]) by krypton.hosting4u.net (Postfix) with ESMTP id D0803A135C for ; Wed, 2 Apr 2003 15:41:34 -0600 (CST) Message-ID: <200304021843340610.00DA75EE@smtp.watersportsbrazil.com> X-Mailer: Calypso Version 3.30.00.00 (3) Date: Wed, 02 Apr 2003 18:43:34 -0300 From: "Ariel de Silvio" To: obm-l@mat.puc-rio.br Subject: [obm-l] Problema simples Mime-Version: 1.0 Content-Type: text/plain; charset="us-ascii" Content-Transfer-Encoding: 8bit X-MIME-Autoconverted: from quoted-printable to 8bit by sucuri.mat.puc-rio.br id SAA26932 Sender: owner-obm-l@sucuri.mat.puc-rio.br Precedence: bulk Reply-To: obm-l@mat.puc-rio.br Olá, Encontrei o seguinte problema no livro Noções de Matemática V.2 do Aref Antar Neto: "Sendo ab<>0 e a+b<>0, verifique que se 4*(a+b)^(-1)=a^(-1)+b^(-1), então a=b." Desenvolvi da seguinte maneira: 4 * (1/a + 1/b) = 1/a + 1/b 4/a + 4/b = 1/a +1/b 3/a = -3/b Portanto >> a=-b mas a+b<>0 ==> a<>-b ?? Esse tipo de questão pode ser respondido com um "Afirmação incorreta"?? Sei que é um problema bobo, se estou errando em alguma coisa, deve ser algo mto besta, isso nao eh nada que eu nao tenha aprendido até o 3o colegial (que curso) E como ele afirma isso e nao pergunta se eh verdade, imagino que deveria ser possivel verificar né.... []s Ariel ========================================================================= Instruções para entrar na lista, sair da lista e usar a lista em http://www.mat.puc-rio.br/~nicolau/olimp/obm-l.html O administrador desta lista é ========================================================================= From owner-obm-l@sucuri.mat.puc-rio.br Wed Apr 2 19:20:27 2003 Return-Path: Received: (from majordom@localhost) by sucuri.mat.puc-rio.br (8.9.3/8.9.3) id TAA27738 for obm-l-MTTP; Wed, 2 Apr 2003 19:19:02 -0300 Received: from itaqui.terra.com.br (itaqui.terra.com.br [200.176.3.19]) by sucuri.mat.puc-rio.br (8.9.3/8.9.3) with ESMTP id TAA27731 for ; Wed, 2 Apr 2003 19:18:56 -0300 Received: from una.terra.com.br (una.terra.com.br [200.176.3.32]) by itaqui.terra.com.br (Postfix) with ESMTP id 542DA3BC499 for ; Wed, 2 Apr 2003 19:18:26 -0300 (BRT) Received: from [200.177.192.106] (dl-nas2-sao-C8B1C06A.p001.terra.com.br [200.177.192.106]) by una.terra.com.br (Postfix) with ESMTP id 4480F2F006B for ; Wed, 2 Apr 2003 19:18:25 -0300 (BRT) User-Agent: Microsoft-Outlook-Express-Macintosh-Edition/5.02.2022 Date: Wed, 02 Apr 2003 19:16:33 -0300 Subject: Re: [obm-l] Re: [obm-l] Integral (Ningu=?ISO-8859-1?B?6Q==?=m se habilita?) From: Claudio Buffara To: Message-ID: In-Reply-To: <000501c2f945$3d152210$019da8c0@henrique> Mime-version: 1.0 Content-type: text/plain; charset="ISO-8859-1" Content-Transfer-Encoding: 8bit X-MIME-Autoconverted: from quoted-printable to 8bit by sucuri.mat.puc-rio.br id TAA27732 Sender: owner-obm-l@sucuri.mat.puc-rio.br Precedence: bulk Reply-To: obm-l@mat.puc-rio.br Oi, Henrique: Na verdade, o que voce quer eh apenas achar uma funcao F, definida no conjunto dos reais positivos (ja que a definicao de x^x eh, na melhor das hipoteses, problematica para x <= 0), tal que F'(x) = x^x. Repare que o enunciado fala de integral INDEFINIDA. De qualquer forma, para x > 0 a funcao f(x) = x^x eh Riemann-integravel. Um abraco, Claudio. on 02.04.03 15:25, Henrique Patrício Sant'Anna Branco at hpsbranco@superig.com.br wrote: >> Alguém sabe me dizer como eu calculo a integral indefinida de x^x (x > elevado >> a x)? > > Essa função não é integrável segundo Riemman. > Sobre a demonstração, eu estava pensando em uma usando o critério de > Lebesge, mas não sei se está certo. Gostaria que algum membro da lista > pudesse me apontar se eu errei e onde errei. > > Essa função é descontínua em zero. Sendo A o conjunto das descontinuidades > desse função, temos que A = {0}. Portanto, tomando o intervalo real I = > {-infinito,infinito), vemos que esse intervalo cobre A, pois A está contido > nesse intervalo. Por outro lado, pela definição de conjunto de medida nula, > temos que o somatório das amplitudes do intervalo I tende ao infinito e não > podemos achar um epsilon maior que isso. Portanto, o conjunto não tem medida > nula e, assim, não é integrável por Riemman. > > Deu pra entender? > O problema dessa demonstração está no fato de que se A = {0}, então é > enumerável e, portanto, não tem medida nula. > Talvez eu tenha errado no conjunto das descontinuidades da função (como no > ponto zero, temos 0^0, teríamos uma descontinuidade infinita?). > > Agradeço qualquer ajuda. > Henrique. > > P.S. - Sou aluno de Estatística e ainda estou no Cálculo 1... Portanto, não > sejam muito duros se eu falei muita besteira... :-) > > ========================================================================= > Instruções para entrar na lista, sair da lista e usar a lista em > http://www.mat.puc-rio.br/~nicolau/olimp/obm-l.html > O administrador desta lista é > ========================================================================= > ========================================================================= Instruções para entrar na lista, sair da lista e usar a lista em http://www.mat.puc-rio.br/~nicolau/olimp/obm-l.html O administrador desta lista é ========================================================================= From owner-obm-l@sucuri.mat.puc-rio.br Wed Apr 2 19:20:27 2003 Return-Path: Received: (from majordom@localhost) by sucuri.mat.puc-rio.br (8.9.3/8.9.3) id TAA27751 for obm-l-MTTP; Wed, 2 Apr 2003 19:19:07 -0300 Received: from ivoti.terra.com.br (ivoti.terra.com.br [200.176.3.20]) by sucuri.mat.puc-rio.br (8.9.3/8.9.3) with ESMTP id TAA27734 for ; Wed, 2 Apr 2003 19:18:59 -0300 Received: from una.terra.com.br (una.terra.com.br [200.176.3.32]) by ivoti.terra.com.br (Postfix) with ESMTP id 97879408042 for ; Wed, 2 Apr 2003 19:18:28 -0300 (BRT) Received: from [200.177.192.106] (dl-nas2-sao-C8B1C06A.p001.terra.com.br [200.177.192.106]) by una.terra.com.br (Postfix) with ESMTP id CDB862F001C for ; Wed, 2 Apr 2003 19:18:27 -0300 (BRT) User-Agent: Microsoft-Outlook-Express-Macintosh-Edition/5.02.2022 Date: Wed, 02 Apr 2003 19:16:34 -0300 Subject: Re: [obm-l] O problema do andarilho From: Claudio Buffara To: Message-ID: In-Reply-To: <001e01c2f950$f2f900a0$019da8c0@henrique> Mime-version: 1.0 Content-type: text/plain; charset="ISO-8859-1" Content-Transfer-Encoding: 8bit X-MIME-Autoconverted: from quoted-printable to 8bit by sucuri.mat.puc-rio.br id TAA27739 Sender: owner-obm-l@sucuri.mat.puc-rio.br Precedence: bulk Reply-To: obm-l@mat.puc-rio.br on 02.04.03 16:49, Henrique Patrício Sant'Anna Branco at hpsbranco@superig.com.br wrote: > Alguem poderia me ajudar com esse? > > > Uma trilha vai da base de uma montanha até o topo. Um andarilho começa a > subir a trilha às 6 horas da manhã e chega ao topo às 6 horas da tarde do > mesmo dia. Durante o percurso ele pode parar, voltar atrás, correr, fazer o > que quiser desde que chegue ao topo às 6 horas da tarde do mesmo dia. > Na manhã seguinte ele começa a descer a trilha às 6 horas da manhã do modo > como ele quiser e chega à base exatamente às 6 horas da tarde do mesmo dia. > Prove que existe pelo menos um lugar na trilha pelo qual ele passa na mesma > hora de cada dia. > > Grato, > > Henrique. > Oi, Henrique: Essa eh uma aplicacao do Teorema do Valor Intermediario. Associe um numero real x a cada ponto do trajeto, de forma que x = distancia do ponto ate a base da montanha. Voce pode normalizar os valores de x, fazendo: base da montanha: x = 0; topo da montanha: x = 1. Defina duas funcoes, F e G, de [6,18] em [0,1], por: F(t) = ponto do trajeto que o andarilho ocupava no instante t durante a subida; G(t) = ponto do trajeto que o andarilho ocupava no instante t durante a descida; F e G sao continuas, pois a velocidade do andarilho eh finita. Agora, aplique o TVI a funcao H: [6,18] -> [0,1] dada por: H(t) = G(t) - H(t) Como H(6) = 1 e H(18) = -1, deve haver algum t_0 em [6,18] tal que H(t_0) = 0 ==> F(t_0) = G(t_0) ==> Em t = t_0 o andarilho estava no mesmo ponto do trajeto tanto na subida quanto na descida. Um abraco, Claudio. ========================================================================= Instruções para entrar na lista, sair da lista e usar a lista em http://www.mat.puc-rio.br/~nicolau/olimp/obm-l.html O administrador desta lista é ========================================================================= From owner-obm-l@sucuri.mat.puc-rio.br Wed Apr 2 19:20:28 2003 Return-Path: Received: (from majordom@localhost) by sucuri.mat.puc-rio.br (8.9.3/8.9.3) id TAA27745 for obm-l-MTTP; Wed, 2 Apr 2003 19:19:06 -0300 Received: from paiol.terra.com.br (paiol.terra.com.br [200.176.3.18]) by sucuri.mat.puc-rio.br (8.9.3/8.9.3) with ESMTP id TAA27733 for ; Wed, 2 Apr 2003 19:18:58 -0300 Received: from una.terra.com.br (una.terra.com.br [200.176.3.32]) by paiol.terra.com.br (Postfix) with ESMTP id 8988C886C9 for ; Wed, 2 Apr 2003 19:18:27 -0300 (BRT) Received: from [200.177.192.106] (dl-nas2-sao-C8B1C06A.p001.terra.com.br [200.177.192.106]) by una.terra.com.br (Postfix) with ESMTP id A2C692F0042 for ; Wed, 2 Apr 2003 19:18:26 -0300 (BRT) User-Agent: Microsoft-Outlook-Express-Macintosh-Edition/5.02.2022 Date: Wed, 02 Apr 2003 19:16:34 -0300 Subject: Re: [obm-l] From: Claudio Buffara To: Message-ID: In-Reply-To: <009901c2f94b$6a34e4f0$0301a8c0@usuario> Mime-version: 1.0 Content-type: multipart/alternative; boundary="MS_Mac_OE_3132155837_116554_MIME_Part" Sender: owner-obm-l@sucuri.mat.puc-rio.br Precedence: bulk Reply-To: obm-l@mat.puc-rio.br > This message is in MIME format. Since your mail reader does not understand this format, some or all of this message may not be legible. --MS_Mac_OE_3132155837_116554_MIME_Part Content-type: text/plain; charset="ISO-8859-1" Content-transfer-encoding: quoted-printable Use o metodo de reducao a forma escalonada: Escreva: 1 0 0 1 0 0 1 3 1 0 1 0 1 2 0 0 0 1 e use operacoes elementares com linhas para reduzir a submatriz 3x3 da esquerda (igual a D) a matriz identidade. Ao fazer isso, voce estara reduzindo a submatriz 3x3 da direita (inicialmente igual a identidade) a inversa de D. Se, durante a reducao, a submatriz da esquerda ficar com alguma linha nula, entao D serah singular. Faca o mesmo para E. Um abraco, Claudio. =20 on 02.04.03 16:09, M=E1rio Pereira at marioappereira@terra.com.br wrote: =C9 possivel calcular a matriz inversa nos seguintes casos? Qual seria o desenvolvimento? =20 D =3D 1 0 0 1 3 1 1 2 0 =20 E =3D -1 -1 2 2 1 -2 1 1 -1 =20 Obrigado. =20 Mario =20 --MS_Mac_OE_3132155837_116554_MIME_Part Content-type: text/html; charset="ISO-8859-1" Content-transfer-encoding: quoted-printable Re: [obm-l] Use o metodo de reducao a forma escalonada:

Escreva:

1  0  0  1  0  0
1  3  1  0  1  0
1  2  0  0  0  1

e use operacoes elementares com linhas para reduzir a submatriz 3x3 da esqu= erda (igual a D) a matriz identidade.

Ao fazer isso, voce estara reduzindo a submatriz 3x3 da direita (inicialmen= te igual a identidade) a inversa de D.

Se, durante a reducao, a submatriz da esquerda ficar com alguma linha nula,= entao D serah singular.

Faca o mesmo para E.

Um abraco,
Claudio.
  
on 02.04.03 16:09, M=E1rio Pereira at marioappereira@terra.com.br wrote:

=C9 possivel calcular a matriz = inversa nos seguintes casos? Qual seria o desenvolvimento?

D =3D 1   0   0
     1   3   1
     1   2   0


E =3D -1   -1   2
      2    1   -2       1    1  -1


Obrigado.

Mario



--MS_Mac_OE_3132155837_116554_MIME_Part-- ========================================================================= Instruções para entrar na lista, sair da lista e usar a lista em http://www.mat.puc-rio.br/~nicolau/olimp/obm-l.html O administrador desta lista é ========================================================================= From owner-obm-l@sucuri.mat.puc-rio.br Wed Apr 2 19:25:35 2003 Return-Path: Received: (from majordom@localhost) by sucuri.mat.puc-rio.br (8.9.3/8.9.3) id TAA27937 for obm-l-MTTP; Wed, 2 Apr 2003 19:24:17 -0300 Received: from traven10.uol.com.br (traven10.uol.com.br [200.221.29.45]) by sucuri.mat.puc-rio.br (8.9.3/8.9.3) with ESMTP id TAA27933 for ; Wed, 2 Apr 2003 19:24:14 -0300 Received: from gauss ([200.158.96.234]) by traven10.uol.com.br (8.9.1/8.9.1) with SMTP id TAA23067 for ; Wed, 2 Apr 2003 19:23:43 -0300 (BRT) Message-ID: <001601c2f966$ff8b39d0$ea609ec8@gauss> From: "Domingos Jr." To: References: <01e201c2f7b1$472e8ba0$3300c57d@bovespa.com> <000d01c2f89d$5bf768d0$2accfea9@gauss> Subject: Re: [obm-l] Mais Problemas em Aberto Date: Wed, 2 Apr 2003 19:27:07 -0300 MIME-Version: 1.0 Content-Type: text/plain; charset="Windows-1252" Content-Transfer-Encoding: 8bit X-Priority: 3 X-MSMail-Priority: Normal X-Mailer: Microsoft Outlook Express 6.00.2800.1106 X-MimeOLE: Produced By Microsoft MimeOLE V6.00.2800.1106 Sender: owner-obm-l@sucuri.mat.puc-rio.br Precedence: bulk Reply-To: obm-l@mat.puc-rio.br Dá pra melhorar bastante esse limitante: A idéia baseia-se no seguinte fato: todo inteiro entre 1...2^(n+1)-1 pode ser expresso como soma de elementos de uma combinação de {1, 2, 2², ..., 2^n}. Seja k um inteiro tal que 2^(k-1) < p < 2^k Da matriz A já definida, separe os elementos: S1 = {(1, 0); (2, 0); ...; (2^(k-1), 0)} S2 = {(0, 1); ...; (0, 2^(k-1))} |S1| = |S2| = k todo elemento de Zp X Zp pode ser expresso como uma soma de elementos de uma combinação dos elementos de S1 e S2, sendo assim, para toda a combinação de elementos do resto da matriz, há sempre pelo menos uma combinação de elementos de S1 e S2 que se cancela com a combinação do resto. O limitante inferior passa então a ser: 2^[p² - 2k] Onde k < lg(p). Além do limitante inferior dá pra determinar um limitante superior: 2^(k-1) < p < 2^k logo 2^k < 2p => 2^k - 1 <= 2p - 2 o conjunto das possíveis somas é no máximo {0, 1, ..., p-1, p, p+1, ... 2p-2}. repare que nesse conjunto não há nenhum elemento (mod p) que se repete mais do que 2 vezes, sendo assim, no máximo temos 2 maneiras de escrever um mesmo elemento e, por consequência, no máximo 4 maneiras de escrever um par de Zp X Zp como soma dos elementos de S1 e S2. Sob essas condições um limitante superior é 4*2^[p² - 2k] = 2^[p² - 2k + 2] Temos então: 2^[p² - 2k] <= RESPOSTA <= 2^[p² - 2k + 2] [ ]'s > Consegui estimar um limitante inferior para o número de grupos de > crianças: > > Considere uma matriz com elementos A[i, j] = (i, j) pertence a (Zp)² > O problema proposto é equivalente a calcular o número de combinações de > elementos de A cuja soma dê (0, 0). > > Agora desenhando a matriz A e separando a última linha e a última coluna, > formamos uma matriz A', com (p-1) x (p-1) elementos em (Zp)². > > Os elementos da última linha são: > { (1, 0), (2, 0), ..., (p-1, 0) , (0,0)} > e os da última coluna são: > { (0, 1), (0, 2), ..., (0, p-1) , (0,0)} > * o elemento (0, 0) é compartilhado! > > Repare que toda combinação de elementos da matriz A' tem soma em (Zp)² e > todo elemento e (Zp)² tem um oposto aditivo em (Zp)², além disso, é possível > obter todos os elementos de (Zp)² através dos elementos da última linha e da > última coluna (basta tomar a soma de dois deles, por exemplo), por tanto, > para cada combinação de A' existe pelo menos uma maneira de selecionar > elemento(s) na última linha e coluna de A tal que a soma total dê (0, 0). > > Por tanto, um limitante inferior para o número de grupo crianças do problema > é: > > 2^[(p-1)²] == total de combinações em A'. > > Esse limitante tem bastante folga pois na verdade existe várias maneiras de > obter o mesmo elemento de (Zp)² através da última linha e da última coluna. > > por exemplo o elemento (3, 2) = (3,0) + (0,2) = (2,0) + (1,0) + (0,2) = ... > > Idéias? > > [ ]'s > > ----- > > 7.5)(Guilherme Issao)Existem p²,onde p e primo,crianças dispostas num bairro > como um tabuleiro p por p.Ha tambem duas distribuidoras de doces,a > Cledmilson > Marmotta e a Estrogonofre's.A Cledmilson Marmotta manda um vendedor para > cada uma das p linhas horizontais,sendo que o vendedor da i-esima linha > tem i Kg de doce de jilo e distribui igualmente entre as p crianças.Da mesma > forma Estrogonofre's manda um vendedor para cada uma das p linhas > verticais,sendo > que o vendedor da j-esima linha tem j Kg de doce de jaca e distribui > igualmente > entre as p crianças.De quantas maneiras podemos escolher um grupo de > crianças > desse bairro para roubar-lhes os doces de modo que a quantidade de cada > tipo de doce roubada seja inteira?[6] > > ========================================================================= > Instruções para entrar na lista, sair da lista e usar a lista em > http://www.mat.puc-rio.br/~nicolau/olimp/obm-l.html > O administrador desta lista é > ========================================================================= ========================================================================= Instruções para entrar na lista, sair da lista e usar a lista em http://www.mat.puc-rio.br/~nicolau/olimp/obm-l.html O administrador desta lista é ========================================================================= From owner-obm-l@sucuri.mat.puc-rio.br Wed Apr 2 19:28:45 2003 Return-Path: Received: (from majordom@localhost) by sucuri.mat.puc-rio.br (8.9.3/8.9.3) id TAA28048 for obm-l-MTTP; Wed, 2 Apr 2003 19:27:27 -0300 Received: from gorgo.centroin.com.br (gorgo.centroin.com.br [200.225.63.128]) by sucuri.mat.puc-rio.br (8.9.3/8.9.3) with ESMTP id TAA28044 for ; Wed, 2 Apr 2003 19:27:24 -0300 Received: from centroin.com.br (RJ208155.user.veloxzone.com.br [200.165.208.155] (may be forged)) (authenticated bits=0) by gorgo.centroin.com.br (8.12.9/8.12.1) with ESMTP id h32MRFWX024084 for ; Wed, 2 Apr 2003 19:27:15 -0300 (EST) Message-ID: <3E8B63DA.20802@centroin.com.br> Date: Wed, 02 Apr 2003 19:27:38 -0300 From: "A. C. Morgado" User-Agent: Mozilla/5.0 (Windows; U; Windows NT 5.0; en-US; rv:1.0.2) Gecko/20030208 Netscape/7.02 X-Accept-Language: en-us, en MIME-Version: 1.0 To: obm-l@mat.puc-rio.br Subject: Re: [obm-l] Problema simples References: <200304021843340610.00DA75EE@smtp.watersportsbrazil.com> Content-Type: text/plain; charset=ISO-8859-1; format=flowed Content-Transfer-Encoding: 8bit Sender: owner-obm-l@sucuri.mat.puc-rio.br Precedence: bulk Reply-To: obm-l@mat.puc-rio.br Ariel, presta atençao! Olha o que voce fez com (a+b)^(-1) que eh 1/ (a+b). Morgado Ariel de Silvio wrote: >Olá, > >Encontrei o seguinte problema no livro Noções de Matemática V.2 do Aref Antar Neto: > >"Sendo ab<>0 e a+b<>0, verifique que se 4*(a+b)^(-1)=a^(-1)+b^(-1), então a=b." > >Desenvolvi da seguinte maneira: >4 * (1/a + 1/b) = 1/a + 1/b >4/a + 4/b = 1/a +1/b >3/a = -3/b >Portanto >> a=-b > >mas a+b<>0 ==> a<>-b >?? > >Esse tipo de questão pode ser respondido com um "Afirmação incorreta"?? >Sei que é um problema bobo, se estou errando em alguma coisa, deve ser algo mto besta, isso nao eh nada que eu nao tenha aprendido até o 3o colegial (que curso) >E como ele afirma isso e nao pergunta se eh verdade, imagino que deveria ser possivel verificar né.... > >[]s >Ariel > > > >========================================================================= >Instruções para entrar na lista, sair da lista e usar a lista em >http://www.mat.puc-rio.br/~nicolau/olimp/obm-l.html >O administrador desta lista é >========================================================================= > > > > ========================================================================= Instruções para entrar na lista, sair da lista e usar a lista em http://www.mat.puc-rio.br/~nicolau/olimp/obm-l.html O administrador desta lista é ========================================================================= From owner-obm-l@sucuri.mat.puc-rio.br Wed Apr 2 19:33:58 2003 Return-Path: Received: (from majordom@localhost) by sucuri.mat.puc-rio.br (8.9.3/8.9.3) id TAA28420 for obm-l-MTTP; Wed, 2 Apr 2003 19:32:33 -0300 Received: from traven10.uol.com.br (traven10.uol.com.br [200.221.29.45]) by sucuri.mat.puc-rio.br (8.9.3/8.9.3) with ESMTP id TAA28416 for ; Wed, 2 Apr 2003 19:32:30 -0300 Received: from gauss ([200.158.96.234]) by traven10.uol.com.br (8.9.1/8.9.1) with SMTP id TAA00445 for ; Wed, 2 Apr 2003 19:31:59 -0300 (BRT) Message-ID: <004101c2f968$27a51340$ea609ec8@gauss> From: "Domingos Jr." To: References: <001e01c2f950$f2f900a0$019da8c0@henrique> Subject: Re: [obm-l] O problema do andarilho Date: Wed, 2 Apr 2003 19:35:24 -0300 MIME-Version: 1.0 Content-Type: text/plain; charset="iso-8859-1" Content-Transfer-Encoding: 8bit X-Priority: 3 X-MSMail-Priority: Normal X-Mailer: Microsoft Outlook Express 6.00.2800.1106 X-MimeOLE: Produced By Microsoft MimeOLE V6.00.2800.1106 Sender: owner-obm-l@sucuri.mat.puc-rio.br Precedence: bulk Reply-To: obm-l@mat.puc-rio.br > Alguem poderia me ajudar com esse? > > > Uma trilha vai da base de uma montanha até o topo. Um andarilho começa a > subir a trilha às 6 horas da manhã e chega ao topo às 6 horas da tarde do > mesmo dia. Durante o percurso ele pode parar, voltar atrás, correr, fazer o > que quiser desde que chegue ao topo às 6 horas da tarde do mesmo dia. > Na manhã seguinte ele começa a descer a trilha às 6 horas da manhã do modo > como ele quiser e chega à base exatamente às 6 horas da tarde do mesmo dia. > Prove que existe pelo menos um lugar na trilha pelo qual ele passa na mesma > hora de cada dia. > > Grato, > > Henrique. A trilha pode ser considerada um segmento reto ligando dois pontos, normalize o segmento para ficar em [0, 1]. Seja X(t) a posição em [0, 1] do andarilho na ida no tempo t entre 06:00 e 18:00 Seja Y(t) a posição em [0, 1] do andarilho na Volta no tempo t entre 06:00 e 18:00. X(06:00) = 0 X(18:00) = 1 Y(06:00) = 1 Y(18:00) = 0 Plote o gráfico de X e Y, como ambas são contínuas, deve haver pelo menos um ponto de intersecção entre os gráficos, esse ponto de intersecção pode ser interpretado como o horário do dia em que ele passa pelo mesmo ponto da trilha. [ ]'s ========================================================================= Instruções para entrar na lista, sair da lista e usar a lista em http://www.mat.puc-rio.br/~nicolau/olimp/obm-l.html O administrador desta lista é ========================================================================= From owner-obm-l@sucuri.mat.puc-rio.br Wed Apr 2 19:38:01 2003 Return-Path: Received: (from majordom@localhost) by sucuri.mat.puc-rio.br (8.9.3/8.9.3) id TAA28809 for obm-l-MTTP; Wed, 2 Apr 2003 19:36:32 -0300 Received: from smtp-26.ig.com.br (smtp-26.ig.com.br [200.226.132.160]) by sucuri.mat.puc-rio.br (8.9.3/8.9.3) with SMTP id TAA28803 for ; Wed, 2 Apr 2003 19:36:29 -0300 Received: (qmail 541 invoked from network); 2 Apr 2003 22:36:10 -0000 Received: from 200-140-080-097.bsace7026.dsl.brasiltelecom.net.br (HELO henrique) (200.140.80.97) by smtp-26.ig.com.br with SMTP; 2 Apr 2003 22:36:10 -0000 Message-ID: <000d01c2f968$3cb69970$019da8c0@henrique> From: "=?iso-8859-1?Q?Henrique_Patr=EDcio_Sant'Anna_Branco?=" To: References: <200304021843340610.00DA75EE@smtp.watersportsbrazil.com> Subject: Re: [obm-l] Problema simples Date: Wed, 2 Apr 2003 19:36:00 -0300 MIME-Version: 1.0 Content-Type: text/plain; charset="iso-8859-1" Content-Transfer-Encoding: 8bit X-Priority: 3 X-MSMail-Priority: Normal X-Mailer: Microsoft Outlook Express 6.00.2800.1106 X-MimeOLE: Produced By Microsoft MimeOLE V6.00.2800.1106 Sender: owner-obm-l@sucuri.mat.puc-rio.br Precedence: bulk Reply-To: obm-l@mat.puc-rio.br > "Sendo ab<>0 e a+b<>0, verifique que se 4*(a+b)^(-1)=a^(-1)+b^(-1), então a=b." Suponha o contrário do que o enunciado propõe: se a<>b, então 4*(a+b)^(-1)=a^(-1)+b^(-1) e desenvolva como você fez. Com isso, chegará à contradição a = -b. Portanto, a = b. Abraço, Henrique. ========================================================================= Instruções para entrar na lista, sair da lista e usar a lista em http://www.mat.puc-rio.br/~nicolau/olimp/obm-l.html O administrador desta lista é ========================================================================= From owner-obm-l@sucuri.mat.puc-rio.br Wed Apr 2 19:44:12 2003 Return-Path: Received: (from majordom@localhost) by sucuri.mat.puc-rio.br (8.9.3/8.9.3) id TAA29354 for obm-l-MTTP; Wed, 2 Apr 2003 19:42:36 -0300 Received: from fronthost.com (mail.fronthost.com [63.250.6.253]) by sucuri.mat.puc-rio.br (8.9.3/8.9.3) with ESMTP id TAA29350 for ; Wed, 2 Apr 2003 19:42:31 -0300 Received: from [68.49.44.134] by fronthost.com [63.250.6.253] with SmartMax MailMax for obm-l@mat.puc-rio.br at Wed, 02 Apr 2003 17:42:28 -0500 Message-ID: <007201c2f968$daa84110$6501a8c0@TEST4> From: "Alexandre A da Rocha" To: References: <200304021843340610.00DA75EE@smtp.watersportsbrazil.com> Subject: Re: [obm-l] Problema simples Date: Wed, 2 Apr 2003 17:40:25 -0500 MIME-Version: 1.0 Content-Type: text/plain; charset="iso-8859-1" Content-Transfer-Encoding: 8bit X-Priority: 3 X-MSMail-Priority: Normal X-Mailer: Microsoft Outlook Express 5.50.4807.1700 X-MimeOLE: Produced By Microsoft MimeOLE V5.50.4807.1700 Sender: owner-obm-l@sucuri.mat.puc-rio.br Precedence: bulk Reply-To: obm-l@mat.puc-rio.br Ariel, me parece que e so um problema de atencao: 4*(a+b)^-1 <> 4*(1/a +1/b) 4*(a+b)^-1 = 4*[1/(a+b)] ou seja, fica: 4*(1/(a+b))= 1/a + 1/b ==> 4/(a+b) = (a+b)/ab ==> 4ab = (a+b)^2 ==> 4ab = a^2 + 2ab + b^2 ==> a^2 - 2ab + b^2 = 0 ==> (a-b)^2 = 0 ==> a-b = 0 ==> a = b -Auggy ----- Original Message ----- From: "Ariel de Silvio" To: Sent: Wednesday, April 02, 2003 4:43 PM Subject: [obm-l] Problema simples Olá, Encontrei o seguinte problema no livro Noções de Matemática V.2 do Aref Antar Neto: "Sendo ab<>0 e a+b<>0, verifique que se 4*(a+b)^(-1)=a^(-1)+b^(-1), então a=b." Desenvolvi da seguinte maneira: 4 * (1/a + 1/b) = 1/a + 1/b 4/a + 4/b = 1/a +1/b 3/a = -3/b Portanto >> a=-b mas a+b<>0 ==> a<>-b ?? Esse tipo de questão pode ser respondido com um "Afirmação incorreta"?? Sei que é um problema bobo, se estou errando em alguma coisa, deve ser algo mto besta, isso nao eh nada que eu nao tenha aprendido até o 3o colegial (que curso) E como ele afirma isso e nao pergunta se eh verdade, imagino que deveria ser possivel verificar né.... []s Ariel ========================================================================= Instruções para entrar na lista, sair da lista e usar a lista em http://www.mat.puc-rio.br/~nicolau/olimp/obm-l.html O administrador desta lista é ========================================================================= ========================================================================= Instruções para entrar na lista, sair da lista e usar a lista em http://www.mat.puc-rio.br/~nicolau/olimp/obm-l.html O administrador desta lista é ========================================================================= From owner-obm-l@sucuri.mat.puc-rio.br Wed Apr 2 20:58:38 2003 Return-Path: Received: (from majordom@localhost) by sucuri.mat.puc-rio.br (8.9.3/8.9.3) id UAA32410 for obm-l-MTTP; Wed, 2 Apr 2003 20:57:06 -0300 Received: from krypton.hosting4u.net (krypton.hosting4u.net [209.15.2.78]) by sucuri.mat.puc-rio.br (8.9.3/8.9.3) with ESMTP id UAA32406 for ; Wed, 2 Apr 2003 20:57:02 -0300 Received: from gargamel (200-158-200-129.dsl.telesp.net.br [200.158.200.129]) by krypton.hosting4u.net (Postfix) with ESMTP id 6A5479F245 for ; Wed, 2 Apr 2003 17:56:30 -0600 (CST) Message-ID: <200304022058300030.0156016C@smtp.watersportsbrazil.com> In-Reply-To: <007201c2f968$daa84110$6501a8c0@TEST4> References: <200304021843340610.00DA75EE@smtp.watersportsbrazil.com> <007201c2f968$daa84110$6501a8c0@TEST4> X-Mailer: Calypso Version 3.30.00.00 (3) Date: Wed, 02 Apr 2003 20:58:30 -0300 From: "Ariel de Silvio" To: obm-l@mat.puc-rio.br Subject: Re: [obm-l] Problema simples Mime-Version: 1.0 Content-Type: text/plain; charset="ISO-8859-1" Content-Transfer-Encoding: 8bit X-MIME-Autoconverted: from quoted-printable to 8bit by sucuri.mat.puc-rio.br id UAA32407 Sender: owner-obm-l@sucuri.mat.puc-rio.br Precedence: bulk Reply-To: obm-l@mat.puc-rio.br Nossa, é verdade... Ultimamente eu to tendo varios erros de pura falta de atenção, sempre me dei bem em exatas (fis, quim e mat), e mesmo entendendo tudo, to errando coisas bestas!!! Fora q isso é revisão, to estudando pro vestibular (ITA)... só preciso parar com essas faltas de atenção!!! Obrigado Ariel *********** MENSAGEM ORIGINAL *********** As 17:40 de 2/4/2003 Alexandre A da Rocha escreveu: >Ariel, >me parece que e so um problema de atencao: >4*(a+b)^-1 <> 4*(1/a +1/b) >4*(a+b)^-1 = 4*[1/(a+b)] > >ou seja, fica: >4*(1/(a+b))= 1/a + 1/b ==> >4/(a+b) = (a+b)/ab ==> >4ab = (a+b)^2 ==> >4ab = a^2 + 2ab + b^2 ==> >a^2 - 2ab + b^2 = 0 ==> >(a-b)^2 = 0 ==> >a-b = 0 ==> a = b > >-Auggy > > >----- Original Message ----- >From: "Ariel de Silvio" >To: >Sent: Wednesday, April 02, 2003 4:43 PM >Subject: [obm-l] Problema simples > > > >Olá, > >Encontrei o seguinte problema no livro Noções de Matemática V.2 do Aref >Antar Neto: > >"Sendo ab<>0 e a+b<>0, verifique que se 4*(a+b)^(-1)=a^(-1)+b^(-1), então >a=b." > >Desenvolvi da seguinte maneira: >4 * (1/a + 1/b) = 1/a + 1/b >4/a + 4/b = 1/a +1/b >3/a = -3/b >Portanto >> a=-b > >mas a+b<>0 ==> a<>-b >?? > >Esse tipo de questão pode ser respondido com um "Afirmação incorreta"?? >Sei que é um problema bobo, se estou errando em alguma coisa, deve ser algo >mto besta, isso nao eh nada que eu nao tenha aprendido até o 3o colegial >(que curso) >E como ele afirma isso e nao pergunta se eh verdade, imagino que deveria >ser >possivel verificar né.... > >[]s >Ariel > > > >========================================================================= >Instruções para entrar na lista, sair da lista e usar a lista em >http://www.mat.puc-rio.br/~nicolau/olimp/obm-l.html >O administrador desta lista é >========================================================================= > > > >========================================================================= >Instruções para entrar na lista, sair da lista e usar a lista em >http://www.mat.puc-rio.br/~nicolau/olimp/obm-l.html >O administrador desta lista é >========================================================================= ========================================================================= Instruções para entrar na lista, sair da lista e usar a lista em http://www.mat.puc-rio.br/~nicolau/olimp/obm-l.html O administrador desta lista é ========================================================================= From owner-obm-l@sucuri.mat.puc-rio.br Wed Apr 2 22:38:27 2003 Return-Path: Received: (from majordom@localhost) by sucuri.mat.puc-rio.br (8.9.3/8.9.3) id WAA03316 for obm-l-MTTP; Wed, 2 Apr 2003 22:36:41 -0300 Received: from hotmail.com (f14.law9.hotmail.com [64.4.9.14]) by sucuri.mat.puc-rio.br (8.9.3/8.9.3) with ESMTP id WAA03312 for ; Wed, 2 Apr 2003 22:36:38 -0300 Received: from mail pickup service by hotmail.com with Microsoft SMTPSVC; Wed, 2 Apr 2003 17:36:07 -0800 Received: from 200.222.175.215 by lw9fd.law9.hotmail.msn.com with HTTP; Thu, 03 Apr 2003 01:36:07 GMT X-Originating-IP: [200.222.175.215] X-Originating-Email: [ricardoprins@hotmail.com] From: "Ricardo Prins" To: obm-l@mat.puc-rio.br Subject: Re: [obm-l] Re: [obm-l] Palavra "Matemática" e símbolo de multiplicação Date: Thu, 03 Apr 2003 01:36:07 +0000 Mime-Version: 1.0 Content-Type: text/html Message-ID: X-OriginalArrivalTime: 03 Apr 2003 01:36:07.0498 (UTC) FILETIME=[65DE9AA0:01C2F981] Sender: owner-obm-l@sucuri.mat.puc-rio.br Precedence: bulk Reply-To: obm-l@mat.puc-rio.br

Bem, o sinal 'x' veio depois do ponto...ele é bem 'atual', na verdade... e...não assassine a nossa amada língua portuguesa...por favor...hahaha

>From: "Frederico Reis Marques de Brito"
>Reply-To: obm-l@mat.puc-rio.br
>To: obm-l@mat.puc-rio.br
>Subject: [obm-l] Re: [obm-l] Palavra "Matemática" e símbolo de multiplicação
>Date: Wed, 02 Apr 2003 12:52:28 -0300
>
>
>
>Ao que parece, a palavra matemática foi introduzida por Pitágoras e
>significava " Aquilo que se pode aprender", sentido que esorbita a
>ciência matemática. Quanto ao uso do símbolo para a multiplicação,
>simplificações notacionais são uma constante obcessão dos
>matemáticos. Inclusive, quase sempre usamos apenas a adjacência xy
> para indicar x vezes y. Não sei se foi intencionalmente para
>não confundir-se a variável x com o símbolo do produto, mas sem
>dúvida a nova notação evita essa ambiguidade, embora os símbolos não
>fossem idênticos.
>Fred.
>
>
>
>
>>From: "Ricardo Prins"
>>Reply-To: obm-l@mat.puc-rio.br
>>To: obm-l@mat.puc-rio.br
>>Subject: Re: [obm-l] Palavra "Matemática" e símbolo de
>>multiplicação
>>Date: Wed, 02 Apr 2003 11:45:54 +0000
>>
>
>
>_________________________________________________________________
>MSN Hotmail, o maior webmail do Brasil. http://www.hotmail.com
><< message3.txt >>


MSN 8 with e-mail virus protection service: 2 months FREE* ========================================================================= Instruções para entrar na lista, sair da lista e usar a lista em http://www.mat.puc-rio.br/~nicolau/olimp/obm-l.html O administrador desta lista é ========================================================================= From owner-obm-l@sucuri.mat.puc-rio.br Wed Apr 2 22:39:08 2003 Return-Path: Received: (from majordom@localhost) by sucuri.mat.puc-rio.br (8.9.3/8.9.3) id WAA03333 for obm-l-MTTP; Wed, 2 Apr 2003 22:37:50 -0300 Received: from ivoti.terra.com.br (ivoti.terra.com.br [200.176.3.20]) by sucuri.mat.puc-rio.br (8.9.3/8.9.3) with ESMTP id WAA03325 for ; Wed, 2 Apr 2003 22:37:46 -0300 Received: from una.terra.com.br (una.terra.com.br [200.176.3.32]) by ivoti.terra.com.br (Postfix) with ESMTP id 0D633408F0D for ; Wed, 2 Apr 2003 22:37:16 -0300 (BRT) Received: from [200.177.186.15] (dl-nas4-sao-C8B1BA0F.p001.terra.com.br [200.177.186.15]) by una.terra.com.br (Postfix) with ESMTP id 6C5C22F0020 for ; Wed, 2 Apr 2003 22:37:14 -0300 (BRT) User-Agent: Microsoft-Outlook-Express-Macintosh-Edition/5.02.2022 Date: Wed, 02 Apr 2003 22:36:04 -0300 Subject: Re: [obm-l] olimp=?ISO-8859-1?B?7Q==?=adas ao redor do mundo.... From: Claudio Buffara To: Message-ID: In-Reply-To: <141.e4dbe57.2bbc9cf6@aol.com> Mime-version: 1.0 Content-type: multipart/alternative; boundary="MS_Mac_OE_3132167764_833925_MIME_Part" Sender: owner-obm-l@sucuri.mat.puc-rio.br Precedence: bulk Reply-To: obm-l@mat.puc-rio.br > This message is in MIME format. Since your mail reader does not understand this format, some or all of this message may not be legible. --MS_Mac_OE_3132167764_833925_MIME_Part Content-type: text/plain; charset="ISO-8859-1" Content-transfer-encoding: quoted-printable on 02.04.03 17:07, DEOLIVEIRASOU@aol.com at DEOLIVEIRASOU@aol.com wrote: E a=ED rapaziada....quero perguntar uma coisa sobre o problema abaixo... 1) Determine n natural, tais que n^2+2 divida 2+2001n. Indo direto a defini=E7=E3o, existe k inteiro tal que 2+2001n=3Dn^2*k+2K. A equa=E7=E3o do segundo grau subjacente tr=E1z delta=3D2001^2-8k(k-1). S=F3 existe n natural se delta for um quadrado perfeito....como determinar os valores de k para que isso aconte=E7a?? No bra=E7o??? Se algu=E9m souber, agrade=E7o a ajuda...se alguem conhece outra forma de resolver adoraria conhecer tamb=E9m. Vou aproveitar e mandar outro. 2) No gr=E1fico da par=E1bola y=3Dx^2 no pano cartesiano marcamos os pontos A, B = e C(com A entre B e C). No segmento BC marca-se o ponto N de modo que AN seja paralelo ao eixo das ordenadas. Se S1 e S2 s=E3o as =E1reas dos tri=E2ngulos ABN = e ACN, respectivamente, determine a medida do segmento AN. Oi, Crom: A 1a. ja apareceu aqui na lista. Vou dar uma procurada e te mando a solucao se conseguir acha-la. Quanto a sua forma de resolver, eu diria que eh perfeitamente aceitavel, apesar de a solucao poder ser mais complicada do que por outros metodos. Repare, no entanto, que delta =3D quadrado perfeito eh apenas uma condicao necessaria (mas nao suficiente) para que a equacao do 2o. grau tenha solucoes inteiras, pois pode ser que (2001 +ou- raiz(delta)) nao seja divisivel por 2k. Agora o 2o.: A =3D (a,a^2), B =3D (b,b^2), C =3D (c,c^2) com b < a < c. Eq. da reta BC:=20 y - b^2 =3D [(c^2-b^2)/(c-b)](x - b) =3D=3D> y =3D b^2 + (b+c)(x - b) =3D=3D> y =3D (b+c)x - bc AN paralelo ao eixo y =3D=3D> A e N tem a mesma abscissa =3D=3D> N =3D ( a , (b+c)a - bc ) =3D=3D> m(AN) =3D | (b+c)a - bc - a^2 | =3D | ba - bc + ca - a^2 | =3D =3D | b*(a - c) - a*(a - c) | =3D | (b - a)*(a - c) | =3D =3D (a - b)*(c - a) (lembre-se que b < a < c) Repare que, aparentemente, temos um problema dimensional: m(AN) =3D comprimento enquanto que: (a - b)*(c - a) =3D comprimento^2 No entanto, lembre-se que a equacao da parabola eh y =3D x^2. Assim, se x e y devem ter a mesma dimensao (comprimento), deve haver uma constante de proporcionalidade k, tal que: k*y =3D x^2 onde, no caso, k vale 1 unidade de comprimento. Agora, repare que a altura de ABN relativa a base AN e igual a (a - b) =3D=3D> S1 =3D (1/2)*m(AN)*(a - b) =3D=3D> m(AN) =3D 2*S1/(a - b) Analogamente voce acha que: m(AN) =3D 2*S2/(c - a) Multiplicando estas duas ultimas expressoes para m(AN): m(AN)^2 =3D 4*S1*S2/m(AN) =3D=3D> m(AN)^3 =3D 4*S1*S2 =3D=3D> m(AN) =3D (4*S1*S2)^(1/3) Um abraco, Claudio. --MS_Mac_OE_3132167764_833925_MIME_Part Content-type: text/html; charset="ISO-8859-1" Content-transfer-encoding: quoted-printable Re: [obm-l] olimp=EDadas ao redor do mundo....
on 02.04.03 17:07, DEOLIVEIRASOU@aol.com at DEOLIVEIRASOU@aol.com wrote:
E a=ED rapaziada....quero pergu= ntar uma coisa sobre o problema abaixo...
1) Determine n natural, tais que n^2+2 divida 2+2001n. Indo direto a defini= =E7=E3o, existe k inteiro tal que 2+2001n=3Dn^2*k+2K. A equa=E7=E3o do segundo grau su= bjacente tr=E1z delta=3D2001^2-8k(k-1). S=F3 existe n natural se delta for um quad= rado perfeito....como determinar os valores de k para que isso aconte=E7a?? No= bra=E7o???


   Se algu=E9m souber, agrade=E7o a ajuda...se alguem conhece o= utra forma de resolver adoraria conhecer tamb=E9m. Vou aproveitar e mandar out= ro.

2) No gr=E1fico da par=E1bola y=3Dx^2 no pano cartesiano marcamos os pontos A, B = e C(com A entre B e C). No segmento BC marca-se o ponto N de modo que AN sej= a paralelo ao eixo das ordenadas. Se S1 e S2 s=E3o as =E1reas dos tri=E2ngulos ABN= e ACN, respectivamente, determine a medida do segmento AN.

Oi, Crom:

A 1a. ja apareceu aqui na lista. Vou dar uma procurada e te mando a solucao= se conseguir acha-la.

Quanto a sua forma de resolver, eu diria que eh perfeitamente aceitavel, ap= esar de a solucao poder ser mais complicada do que por outros metodos.

Repare, no entanto, que delta =3D quadrado perfeito eh apenas uma condicao ne= cessaria (mas nao suficiente) para que a equacao do 2o. grau tenha solucoes = inteiras, pois pode ser que (2001 +ou- raiz(delta)) nao seja divisivel por 2= k.


Agora o 2o.:

A =3D (a,a^2), B =3D (b,b^2), C =3D (c,c^2)  com b < a < c.

Eq. da reta BC:
y - b^2 =3D [(c^2-b^2)/(c-b)](x - b) =3D=3D>
y =3D b^2 + (b+c)(x - b) =3D=3D>
y =3D (b+c)x - bc

AN paralelo ao eixo y =3D=3D>
A e N tem a mesma abscissa =3D=3D>
N =3D ( a ,  (b+c)a - bc ) =3D=3D>
m(AN) =3D | (b+c)a - bc - a^2 | =3D | ba - bc + ca - a^2 | =3D
=3D | b*(a - c) - a*(a - c) | =3D | (b - a)*(a - c) | =3D
=3D (a - b)*(c - a)   (lembre-se que b < a < c)

Repare que, aparentemente, temos um problema dimensional:
m(AN) =3D comprimento
enquanto que:
(a - b)*(c - a) =3D comprimento^2

No entanto, lembre-se que a equacao da parabola eh y =3D x^2. Assim, se x e y= devem ter a mesma dimensao (comprimento), deve haver uma constante de propo= rcionalidade k, tal que:
k*y =3D x^2  onde, no caso, k vale 1 unidade de comprimento.

Agora, repare que a altura de ABN relativa a base AN e igual a (a - b) =3D=3D&g= t;
S1 =3D (1/2)*m(AN)*(a - b) =3D=3D>
m(AN) =3D 2*S1/(a - b)

Analogamente voce acha que:
m(AN) =3D 2*S2/(c - a)

Multiplicando estas duas ultimas expressoes para m(AN):
m(AN)^2 =3D 4*S1*S2/m(AN) =3D=3D>
m(AN)^3 =3D 4*S1*S2 =3D=3D>
m(AN) =3D (4*S1*S2)^(1/3)

Um abraco,
Claudio.
--MS_Mac_OE_3132167764_833925_MIME_Part-- ========================================================================= Instruções para entrar na lista, sair da lista e usar a lista em http://www.mat.puc-rio.br/~nicolau/olimp/obm-l.html O administrador desta lista é ========================================================================= From owner-obm-l@sucuri.mat.puc-rio.br Wed Apr 2 23:10:09 2003 Return-Path: Received: (from majordom@localhost) by sucuri.mat.puc-rio.br (8.9.3/8.9.3) id XAA04272 for obm-l-MTTP; Wed, 2 Apr 2003 23:08:34 -0300 Received: from hotmail.com (f22.law9.hotmail.com [64.4.9.22]) by sucuri.mat.puc-rio.br (8.9.3/8.9.3) with ESMTP id XAA04268 for ; Wed, 2 Apr 2003 23:08:29 -0300 Received: from mail pickup service by hotmail.com with Microsoft SMTPSVC; Wed, 2 Apr 2003 18:07:58 -0800 Received: from 200.222.175.215 by lw9fd.law9.hotmail.msn.com with HTTP; Thu, 03 Apr 2003 02:07:58 GMT X-Originating-IP: [200.222.175.215] X-Originating-Email: [ricardoprins@hotmail.com] From: "Ricardo Prins" To: obm-l@mat.puc-rio.br Subject: [obm-l] Números complexos Date: Thu, 03 Apr 2003 02:07:58 +0000 Mime-Version: 1.0 Content-Type: text/html Message-ID: X-OriginalArrivalTime: 03 Apr 2003 02:07:58.0731 (UTC) FILETIME=[D90D9DB0:01C2F985] Sender: owner-obm-l@sucuri.mat.puc-rio.br Precedence: bulk Reply-To: obm-l@mat.puc-rio.br
Primeira dúvida: existe representação gráfica da norma de um complexo?
 
outra dúvida:
 
Seja z pertencente aos complexos. Determine z e o módulo do complexo 1 - z, sabendo-se que z é o complexo de módulo máximo tal que | z + sqrt(2)cis (pi)/3 | = 1.
 
e finalmente,
 
prove que se x + x^ (- 1) = 2 cos n, então x^13 + x^(-13) = 2 cos 13n.
 
 


MSN 8 helps ELIMINATE E-MAIL VIRUSES. Get 2 months FREE*. ========================================================================= Instruções para entrar na lista, sair da lista e usar a lista em http://www.mat.puc-rio.br/~nicolau/olimp/obm-l.html O administrador desta lista é ========================================================================= From owner-obm-l@sucuri.mat.puc-rio.br Wed Apr 2 23:46:16 2003 Return-Path: Received: (from majordom@localhost) by sucuri.mat.puc-rio.br (8.9.3/8.9.3) id XAA05338 for obm-l-MTTP; Wed, 2 Apr 2003 23:44:55 -0300 Received: from paiol.terra.com.br (paiol.terra.com.br [200.176.3.18]) by sucuri.mat.puc-rio.br (8.9.3/8.9.3) with ESMTP id XAA05334 for ; Wed, 2 Apr 2003 23:44:52 -0300 Received: from itaim.terra.com.br (itaim.terra.com.br [200.176.3.76]) by paiol.terra.com.br (Postfix) with ESMTP id 6DF5788101 for ; Wed, 2 Apr 2003 23:44:22 -0300 (BRT) Received: from [200.177.179.254] (dl-nas3-sao-C8B1B3FE.p001.terra.com.br [200.177.179.254]) by itaim.terra.com.br (Postfix) with ESMTP id BCBE52E0063 for ; Wed, 2 Apr 2003 23:44:21 -0300 (BRT) User-Agent: Microsoft-Outlook-Express-Macintosh-Edition/5.02.2022 Date: Wed, 02 Apr 2003 23:43:11 -0300 Subject: FW: [obm-l] O problema do andarilho From: Claudio Buffara To: Lista OBM Message-ID: In-Reply-To: Mime-version: 1.0 Content-type: text/plain; charset="ISO-8859-1" Content-Transfer-Encoding: 8bit X-MIME-Autoconverted: from quoted-printable to 8bit by sucuri.mat.puc-rio.br id XAA05335 Sender: owner-obm-l@sucuri.mat.puc-rio.br Precedence: bulk Reply-To: obm-l@mat.puc-rio.br So uma pequena correcao: O contradominio da funcao H definida abaixo eh [-1,1] (ou qualquer subconjunto de R que contenha [-1,1]) Claudio. ---------- From: Claudio Buffara Date: Wed, 02 Apr 2003 19:16:34 -0300 To: Subject: Re: [obm-l] O problema do andarilho on 02.04.03 16:49, Henrique Patrício Sant'Anna Branco at hpsbranco@superig.com.br wrote: > Alguem poderia me ajudar com esse? > > > Uma trilha vai da base de uma montanha até o topo. Um andarilho começa a > subir a trilha às 6 horas da manhã e chega ao topo às 6 horas da tarde do > mesmo dia. Durante o percurso ele pode parar, voltar atrás, correr, fazer o > que quiser desde que chegue ao topo às 6 horas da tarde do mesmo dia. > Na manhã seguinte ele começa a descer a trilha às 6 horas da manhã do modo > como ele quiser e chega à base exatamente às 6 horas da tarde do mesmo dia. > Prove que existe pelo menos um lugar na trilha pelo qual ele passa na mesma > hora de cada dia. > > Grato, > > Henrique. > Oi, Henrique: Essa eh uma aplicacao do Teorema do Valor Intermediario. Associe um numero real x a cada ponto do trajeto, de forma que x = distancia do ponto ate a base da montanha. Voce pode normalizar os valores de x, fazendo: base da montanha: x = 0; topo da montanha: x = 1. Defina duas funcoes, F e G, de [6,18] em [0,1], por: F(t) = ponto do trajeto que o andarilho ocupava no instante t durante a subida; G(t) = ponto do trajeto que o andarilho ocupava no instante t durante a descida; F e G sao continuas, pois a velocidade do andarilho eh finita. Agora, aplique o TVI a funcao H: [6,18] -> [0,1] dada por: H(t) = G(t) - H(t) Como H(6) = 1 e H(18) = -1, deve haver algum t_0 em [6,18] tal que H(t_0) = 0 ==> F(t_0) = G(t_0) ==> Em t = t_0 o andarilho estava no mesmo ponto do trajeto tanto na subida quanto na descida. Um abraco, Claudio. ========================================================================= Instruções para entrar na lista, sair da lista e usar a lista em http://www.mat.puc-rio.br/~nicolau/olimp/obm-l.html O administrador desta lista é ========================================================================= From owner-obm-l@sucuri.mat.puc-rio.br Wed Apr 2 23:46:16 2003 Return-Path: Received: (from majordom@localhost) by sucuri.mat.puc-rio.br (8.9.3/8.9.3) id XAA05330 for obm-l-MTTP; Wed, 2 Apr 2003 23:44:45 -0300 Received: from ivoti.terra.com.br (ivoti.terra.com.br [200.176.3.20]) by sucuri.mat.puc-rio.br (8.9.3/8.9.3) with ESMTP id XAA05326 for ; Wed, 2 Apr 2003 23:44:40 -0300 Received: from sapucaia.terra.com.br (sapucaia.terra.com.br [200.176.3.41]) by ivoti.terra.com.br (Postfix) with ESMTP id 9D827408357 for ; Wed, 2 Apr 2003 23:44:10 -0300 (BRT) Received: from [200.177.179.254] (dl-nas3-sao-C8B1B3FE.p001.terra.com.br [200.177.179.254]) by sapucaia.terra.com.br (Postfix) with ESMTP id 0EC9B304032 for ; Wed, 2 Apr 2003 23:44:09 -0300 (BRT) User-Agent: Microsoft-Outlook-Express-Macintosh-Edition/5.02.2022 Date: Wed, 02 Apr 2003 23:42:18 -0300 Subject: [obm-l] Teorema dos Casamentos From: Claudio Buffara To: Message-ID: In-Reply-To: Mime-version: 1.0 Content-type: multipart/alternative; boundary="MS_Mac_OE_3132171738_1072986_MIME_Part" Sender: owner-obm-l@sucuri.mat.puc-rio.br Precedence: bulk Reply-To: obm-l@mat.puc-rio.br > This message is in MIME format. Since your mail reader does not understand this format, some or all of this message may not be legible. --MS_Mac_OE_3132171738_1072986_MIME_Part Content-type: text/plain; charset="ISO-8859-1" Content-transfer-encoding: quoted-printable Caro Ricardo: Segue abaixo a demonstracao. Teorema dos Casamentos: Sejam A(1), A(2), ..., A(n) conjuntos tais que a uni=E3o da quaisquer i deles (1 <=3D i <=3D n) cont=E9m no m=EDnimo i elementos distintos. Ent=E3o =E9 poss=EDvel selecionar n elementos distintos, sendo um de cada conjunto.=20 Dem: Inducao completa em n (numero de conjuntos): n =3D 1: obvio Hipotese de Inducao: o teorema eh verdadeiro para 1 <=3D n <=3D m-1. Consideremos m conjuntos A(1), ..., A(m) tais que a uniao de quaisquer k deles (1 <=3D i <=3D m) contenha pelo menos i elementos distintos. Temos dois casos a considerar: CASO 1: Para cada k (1 <=3D k <=3D m-1), a uni=E3o de quaisquer k conjuntos cont=E9= m pelo menos k+1 elementos. Nesse caso, escolha um elemento qualquer de A(m) - digamos "a". Como a uniao dos m-1 conjuntos A(1), ..., A(m-1) contem pelo menos m elementos, a uniao de A(1) - {a}, ..., A(m-1) - {a} ira conter, no minimo, m-1 elementos. Assim, pela hipotese de inducao, podemos escolher um elemento distinto de cada um destes conjuntos. Estes m-1 elementos, juntamente com "a", serao m elementos distintos, cada um escolhido de um dos A(i) (1 <=3D i <=3D m) ---------- CASO 2: Existem: (i) um inteiro k (1 <=3D k <=3D m-1) e (ii) k conjuntos tais que a sua uni=E3o contem exatamemente k elementos. Como 1 <=3D k <=3D m-1, podemos aplicar a hipotese de inducao e escolher um elemento distinto de cada um dos k conjuntos cuja uniao contem k elementos. Suponhamos que dentre os m-k conjuntos restantes, existam j ( 1 <=3D j <=3D m-k= ) cuja uniao contenha menos do que j elementos que sejam distintos dos k elementos escolhidos acima. Entao, a uniao dos k conjuntos iniciais com estes j conjuntos ira conter menos do que k + j elementos, o que contradiz a hipotese do teorema sobre estes conjuntos. Logo, dentre os m-k conjuntos restantes, a uniao de quaisquer j ( 1 <=3D j <=3D m-k) ira conter pelo menos j elementos e todos eles serao distintos dos k elementos escolhidos inicialmente. Assim, podemos tambem aplicar a hipotese de inducao a estes m-k conjuntos e escolher um elemento distinto de cada um deles. Alem do mais, podemos fazer isso de forma que estes elementos sejam distintos dos k elementos escolhido= s inicialmente. Em suma, tambem neste caso eh possivel escolher m elementos distintos, send= o um de cada um dos A(i) (1 <=3D i <=3D n) *** FIM *** Um abraco, Claudio. on 02.04.03 08:09, Ricardo Prins at ricardoprins@hotmail.com wrote: me intrometendo...=20 Voc=EA pode me enviar a demonstra=E7=E3o? Ricardo >From: "Cl=E1udio \(Pr=E1tica\)" >Reply-To: obm-l@mat.puc-rio.br >To:=20 >Subject: Re: [obm-l] Grafos e Casamentos >Date: Mon, 31 Mar 2003 15:57:27 -0300 >=20 >Oi, JP:=20 >=20 >O enunciado do Teorema dos Casamentos =E9 o seguinte: >Sejam A(1), A(2), ..., A(n) conjuntos tais que a uni=E3o da quaisquer k dele= s >(1 <=3D k <=3D n) cont=E9m no m=EDnimo k elementos distintos. Ent=E3o =E9 poss=EDvel >selecionar n elementos distintos, sendo um de cada conjunto. >=20 >A demonstra=E7=E3o padr=E3o =E9 por indu=E7=E3o completa em n, e trata dois casos >separadamente:=20 >i) Para cada k (1 <=3D k < n), a uni=E3o de cada k conjuntos cont=E9m pelo menos >k+1 elementos;=20 >ii) Existem k (1 <=3D k < n) e k conjuntos tais que a sua uni=E3o tem >exatamemente k elementos. >=20 >Se voc=EA quiser, depois eu posso mandar a demonstra=E7=E3o. >=20 >Um abra=E7o,=20 >Claudio.=20 --MS_Mac_OE_3132171738_1072986_MIME_Part Content-type: text/html; charset="ISO-8859-1" Content-transfer-encoding: quoted-printable Teorema dos Casamentos
Caro Ricardo:

Segue abaixo a demonstracao.

Teorema dos Casamentos:
Sejam A(1), A(2), ..., A(n) conjuntos tais que a uni=E3o da quaisquer i deles=
(1 <=3D i <=3D n) cont=E9m no m=EDnimo i elementos distintos.
Ent=E3o =E9 poss=EDvel  selecionar n elementos distintos, sendo um de cada c= onjunto.

Dem:
Inducao completa em n (numero de conjuntos):

n =3D 1: obvio

Hipotese de Inducao: o teorema eh verdadeiro para 1 <=3D n <=3D m-1.

Consideremos m conjuntos A(1), ..., A(m) tais que a uniao de quaisquer k de= les (1 <=3D i <=3D m) contenha pelo menos i elementos distintos.


Temos dois casos a considerar:

CASO 1: Para cada k (1 <=3D k <=3D m-1), a uni=E3o de quaisquer k conjuntos= cont=E9m pelo menos k+1 elementos.

Nesse caso, escolha um elemento qualquer de A(m) - digamos "a".
Como a uniao dos m-1 conjuntos A(1), ..., A(m-1) contem pelo menos m elemen= tos, a uniao de A(1) - {a}, ..., A(m-1) - {a} ira conter, no minimo, m-1 ele= mentos.

Assim, pela hipotese de inducao, podemos escolher um elemento distinto de c= ada um destes conjuntos.

Estes m-1 elementos, juntamente com "a", serao m elementos distin= tos, cada um escolhido de um dos A(i) (1 <=3D i <=3D m)

----------

CASO 2: Existem: (i) um inteiro k (1 <=3D k <=3D m-1) e (ii) k conjuntos = tais que a sua uni=E3o contem exatamemente k elementos.

Como 1 <=3D k <=3D m-1, podemos aplicar a hipotese de inducao e escolher = um elemento distinto de cada um dos k conjuntos cuja uniao contem k elemento= s.

Suponhamos que dentre os m-k conjuntos restantes, existam j ( 1 <=3D j <= ;=3D m-k) cuja uniao contenha menos do que j elementos que sejam distintos dos= k elementos escolhidos acima.

Entao, a uniao dos k conjuntos iniciais com estes j conjuntos ira conter me= nos do que k + j elementos, o que contradiz a hipotese do teorema sobre este= s conjuntos.

Logo, dentre os m-k conjuntos restantes, a uniao de quaisquer j ( 1 <=3D j= <=3D m-k) ira conter pelo menos j elementos e todos eles serao distintos d= os k elementos escolhidos inicialmente.

Assim, podemos tambem aplicar a hipotese de inducao a estes m-k conjuntos e= escolher um elemento distinto de cada um deles. Alem do mais, podemos fazer= isso de forma que estes elementos sejam distintos dos k elementos escolhido= s inicialmente.

Em suma, tambem neste caso eh possivel escolher m elementos distintos, send= o um de cada um dos A(i) (1 <=3D i <=3D n)

*** FIM ***

Um abraco,
Claudio.

on 02.04.03 08:09, Ricardo Prins at ricardoprins@hotmail.com wrote:

me intrometendo...

Voc=EA pode me enviar a demonstra=E7=E3o?

Ricardo
>From: "Cl=E1udio \(Pr=E1tica\)"
>Reply-To: obm-l@mat.puc-rio.br
>To:
>Subject: Re: [obm-l] Grafos e Casamentos
>Date: Mon, 31 Mar 2003 15:57:27 -0300
>
>Oi, JP:
>
>O enunciado do Teorema dos Casamentos =E9 o seguinte:
>Sejam A(1), A(2), ..., A(n) conjuntos tais que a uni=E3o da quaisquer k d= eles
>(1 <=3D k <=3D n) cont=E9m no m=EDnimo k elementos distintos. Ent=E3o =E9 pos= s=EDvel
>selecionar n elementos distintos, sendo um de cada conjunto.
>
>A demonstra=E7=E3o padr=E3o =E9 por indu=E7=E3o completa em n, e trata dois casos <= BR> >separadamente:
>i) Para cada k (1 <=3D k < n), a uni=E3o de cada k conjuntos cont=E9m p= elo menos
>k+1 elementos;
>ii) Existem k (1 <=3D k < n) e k conjuntos tais que a sua uni=E3o tem=
>exatamemente k elementos.
>
>Se voc=EA quiser, depois eu posso mandar a demonstra=E7=E3o.
>
>Um abra=E7o,
>Claudio.
--MS_Mac_OE_3132171738_1072986_MIME_Part-- ========================================================================= Instruções para entrar na lista, sair da lista e usar a lista em http://www.mat.puc-rio.br/~nicolau/olimp/obm-l.html O administrador desta lista é ========================================================================= From owner-obm-l@sucuri.mat.puc-rio.br Wed Apr 2 23:56:46 2003 Return-Path: Received: (from majordom@localhost) by sucuri.mat.puc-rio.br (8.9.3/8.9.3) id XAA05704 for obm-l-MTTP; Wed, 2 Apr 2003 23:55:21 -0300 Received: from gorgo.centroin.com.br (gorgo.centroin.com.br [200.225.63.128]) by sucuri.mat.puc-rio.br (8.9.3/8.9.3) with ESMTP id XAA05700 for ; Wed, 2 Apr 2003 23:55:18 -0300 Received: from centroin.com.br (RJ208155.user.veloxzone.com.br [200.165.208.155] (may be forged)) (authenticated bits=0) by gorgo.centroin.com.br (8.12.9/8.12.1) with ESMTP id h332t9WX014293 for ; Wed, 2 Apr 2003 23:55:09 -0300 (EST) Message-ID: <3E8BA2A4.5040904@centroin.com.br> Date: Wed, 02 Apr 2003 23:55:32 -0300 From: "A. C. Morgado" User-Agent: Mozilla/5.0 (Windows; U; Windows NT 5.0; en-US; rv:1.0.2) Gecko/20030208 Netscape/7.02 X-Accept-Language: en-us, en MIME-Version: 1.0 To: obm-l@mat.puc-rio.br Subject: Re: [obm-l] =?ISO-8859-1?Q?N=FAmeros_complexos?= References: Content-Type: multipart/alternative; boundary="------------010806000701030307040407" Sender: owner-obm-l@sucuri.mat.puc-rio.br Precedence: bulk Reply-To: obm-l@mat.puc-rio.br --------------010806000701030307040407 Content-Type: text/plain; charset=ISO-8859-1; format=flowed Content-Transfer-Encoding: 8bit 3) x^2 - x.2cosn +1 = 0 x = cosn (+-) i sen n x^13 = cos 13n (+-) i sen13n x^(-13) = cos 13n (-+) i sen 13n x^13 + x^(-13) = 2cos13n Ricardo Prins wrote: > Primeira dúvida: existe representação gráfica da norma de um complexo? > > outra dúvida: > > Seja z pertencente aos complexos. Determine z e o módulo do complexo 1 > - z, sabendo-se que z é o complexo de módulo máximo tal que | z + > sqrt(2)cis (pi)/3 | = 1. > > e finalmente, > > prove que se x + x^ (- 1) = 2 cos n, então x^13 + x^(-13) = 2 cos 13n. > > > > ------------------------------------------------------------------------ > MSN 8 helps ELIMINATE E-MAIL VIRUSES. > Get 2 months > FREE*.========================================================================= > Instruções para entrar na lista, sair da lista e usar a lista em > http://www.mat.puc-rio.br/~nicolau/olimp/obm-l.html O administrador > desta lista é > ========================================================================= --------------010806000701030307040407 Content-Type: text/html; charset=us-ascii Content-Transfer-Encoding: 7bit 3) x^2 - x.2cosn +1 = 0
x = cosn (+-) i sen n
x^13 = cos 13n (+-) i sen13n
x^(-13) = cos 13n (-+) i sen 13n
x^13 + x^(-13) = 2cos13n
Ricardo Prins wrote:
Primeira dúvida: existe representação gráfica da norma de um complexo?
 
outra dúvida:
 
Seja z pertencente aos complexos. Determine z e o módulo do complexo 1 - z, sabendo-se que z é o complexo de módulo máximo tal que | z + sqrt(2)cis (pi)/3 | = 1.
 
e finalmente,
 
prove que se x + x^ (- 1) = 2 cos n, então x^13 + x^(-13) = 2 cos 13n.
 
 


MSN 8 helps ELIMINATE E-MAIL VIRUSES. Get 2 months FREE*.========================================================================= Instruções para entrar na lista, sair da lista e usar a lista em http://www.mat.puc-rio.br/~nicolau/olimp/obm-l.html O administrador desta lista é =========================================================================

--------------010806000701030307040407-- ========================================================================= Instruções para entrar na lista, sair da lista e usar a lista em http://www.mat.puc-rio.br/~nicolau/olimp/obm-l.html O administrador desta lista é ========================================================================= From owner-obm-l@sucuri.mat.puc-rio.br Thu Apr 3 00:56:33 2003 Return-Path: Received: (from majordom@localhost) by sucuri.mat.puc-rio.br (8.9.3/8.9.3) id AAA07535 for obm-l-MTTP; Thu, 3 Apr 2003 00:54:49 -0300 Received: from ivoti.terra.com.br (ivoti.terra.com.br [200.176.3.20]) by sucuri.mat.puc-rio.br (8.9.3/8.9.3) with ESMTP id AAA07530 for ; Thu, 3 Apr 2003 00:54:46 -0300 Received: from marova.terra.com.br (marova.terra.com.br [200.176.3.39]) by ivoti.terra.com.br (Postfix) with ESMTP id 64B1640835A for ; Thu, 3 Apr 2003 00:54:16 -0300 (BRT) Received: from [200.177.186.92] (dl-nas4-sao-C8B1BA5C.p001.terra.com.br [200.177.186.92]) by marova.terra.com.br (Postfix) with ESMTP id D07873DC077 for ; Thu, 3 Apr 2003 00:54:14 -0300 (BRT) User-Agent: Microsoft-Outlook-Express-Macintosh-Edition/5.02.2022 Date: Thu, 03 Apr 2003 00:53:04 -0300 Subject: Re: [obm-l] N=?ISO-8859-1?B?+g==?=meros complexos From: Claudio Buffara To: Message-ID: In-Reply-To: Mime-version: 1.0 Content-type: multipart/alternative; boundary="MS_Mac_OE_3132175984_1328359_MIME_Part" Sender: owner-obm-l@sucuri.mat.puc-rio.br Precedence: bulk Reply-To: obm-l@mat.puc-rio.br > This message is in MIME format. Since your mail reader does not understand this format, some or all of this message may not be legible. --MS_Mac_OE_3132175984_1328359_MIME_Part Content-type: text/plain; charset="ISO-8859-1" Content-transfer-encoding: quoted-printable on 02.04.03 23:07, Ricardo Prins at ricardoprins@hotmail.com wrote: Primeira d=FAvida: existe representa=E7=E3o gr=E1fica da norma de um complexo? Sim, em 3 dimensoes. A norma de x + iy eh igual a (x^2+y^2) (outras pessoas dizem que a norma eh raiz(x^2+y^2), mas eu chamo isso de modulo. Assim, pra mim: modulo =3D raiz(norma). Cuidado que a nomenclatura nao eh padrao). De qualquer jeito, voce define a funcao N: R^2 --> R tal que: N(x,y) =3D x^2 + y^2 Fazendo o complexo x + iy corresponder ao par ordenado (x,y) voce tem a sua representacao grafica: e o paraboloide de revolucao: z =3D x^2 + y^2. Se N(x,y) =3D raiz(x^2+y^2), entao a representacao grafica sera a folha superior (localizada no semi-espaco z >=3D 0) do cone z^2 - x^2 - y^2 =3D 0 ********* outra d=FAvida:=20 Seja z pertencente aos complexos. Determine z e o m=F3dulo do complexo 1 - z, sabendo-se que z =E9 o complexo de m=F3dulo m=E1ximo tal que | z + sqrt(2)cis (pi)/3 | =3D 1. Tem certeza que eh cis(pi)/3? Isso eh igual a -1/3. Assim, voce vai ter | z - raiz(2)/3 | =3D 1 =3D=3D> z pertence a circunferencia de centro em (raiz(2)/3,0) e raio 1 =3D=3D> |z| eh maximo se z =3D 1 + raiz(2)/3 =3D=3D> | 1 - z | =3D raiz(2)/3. Por outro lado, se for cis(pi/3), voce terah: | z + raiz(2)*cis(pi/3) | =3D 1 =3D=3D> z pertence a circunferencia de centro em (-raiz(2)/2,-raiz(6)/2) e raio 1 =3D=3D> | z | eh maximo se z tambem pertencer a circunferencia de centro na origem = e que tangencia externamente a circunferencia acima. z =3D x + iy =3D=3D> x/(-raiz(2)/2) =3D (1+raiz(2))/raiz(2) e y/(-raiz(6)/2) =3D (1+raiz(2))/raiz(2) =3D=3D> x =3D (1+raiz(2))(-1/2) e y =3D (1+raiz(2))*(-raiz(3)/2) =3D=3D> z =3D (1+raiz(2))*cis(4*pi/3) =3D=3D> | 1 - z | =3D | (3+raiz(2))/2 + i*(1+raiz(2))*(raiz(3)/2) | =3D raiz(20 + 12*raiz(2))/2 =3D raiz(5 + 3*raiz(2)) ********* e finalmente,=20 prove que se x + x^ (- 1) =3D 2 cos n, ent=E3o x^13 + x^(-13) =3D 2 cos 13n. =20 x + 1/x =3D 2*cos(n) =3D=3D> x^2 - 2*cos(n)*x + 1 =3D 0 =3D=3D> x =3D cis(n) e x^(-1) =3D cis(-n) ou =20 x =3D cis(-n) e x^(-1) =3D cis(n) =3D=3D> x^13 =3D cis(13n) e x^(-13) =3D cis(-13n) ou x^13 =3D cis(-13n) e x^(-13) =3D cis(13n) =3D=3D> de qualquer forma, x^13 + x^(-13) =3D 2*cos(13n) --MS_Mac_OE_3132175984_1328359_MIME_Part Content-type: text/html; charset="ISO-8859-1" Content-transfer-encoding: quoted-printable Re: [obm-l] N=FAmeros complexos on 02.04.03 23:07, Ricardo Prins at ricardoprins@hotmail.com wrote:

Primeira d=FAvida: existe representa=E7=E3o gr=E1fica da norma de um= complexo?

Sim, em 3 dimensoes.

A norma de x + iy eh igual a (x^2+y^2) (outras pessoas dizem que a norma eh= raiz(x^2+y^2), mas eu chamo isso de modulo. Assim, pra mim: modulo =3D raiz(n= orma). Cuidado que a nomenclatura nao eh padrao).

De qualquer jeito, voce define a funcao N: R^2 --> R tal que:
N(x,y) =3D x^2 + y^2

Fazendo o complexo x + iy corresponder ao par ordenado (x,y) voce tem a sua= representacao grafica: e o paraboloide de revolucao: z =3D x^2 + y^2.

Se N(x,y) =3D raiz(x^2+y^2), entao a representacao grafica sera a folha super= ior (localizada no semi-espaco z >=3D 0) do cone z^2 - x^2 - y^2 =3D 0

*********

outra d=FAvida:

Seja z pertencente aos complexos. Determine z e o m=F3dulo do complexo 1 -= z, sabendo-se que z =E9 o complexo de m=F3dulo m=E1ximo tal que | z + sqrt(2)cis = (pi)/3 | =3D 1.

Tem certeza que eh cis(pi)/3? Isso eh igual a -1/3.

Assim, voce vai ter | z - raiz(2)/3 | =3D 1 =3D=3D>
z pertence a circunferencia de centro em (raiz(2)/3,0) e raio 1 =3D=3D> |z| eh maximo  se z =3D 1 + raiz(2)/3 =3D=3D>
| 1 - z | =3D raiz(2)/3.

Por outro lado, se for cis(pi/3), voce terah:
| z + raiz(2)*cis(pi/3) | =3D 1 =3D=3D>
z pertence a circunferencia de centro em (-raiz(2)/2,-raiz(6)/2) e raio 1 =3D= =3D>
| z | eh maximo se z tambem pertencer a circunferencia de centro na origem = e que tangencia externamente a circunferencia acima.

z =3D x + iy =3D=3D>
x/(-raiz(2)/2) =3D (1+raiz(2))/raiz(2)
e
y/(-raiz(6)/2) =3D (1+raiz(2))/raiz(2) =3D=3D>
x =3D (1+raiz(2))(-1/2)   e   y =3D (1+raiz(2))*(-raiz(3)/2= ) =3D=3D>
z =3D (1+raiz(2))*cis(4*pi/3) =3D=3D>
| 1 - z | =3D | (3+raiz(2))/2 + i*(1+raiz(2))*(raiz(3)/2) | =3D
raiz(20 + 12*raiz(2))/2 =3D
raiz(5 + 3*raiz(2))

*********

e finalmente,

prove que se x + x^ (- 1) =3D 2 cos n, ent=E3o x^13 + x^(-13) =3D 2 cos 13n.
x + 1/x =3D 2*cos(n) =3D=3D>  

x^2 - 2*cos(n)*x + 1 =3D 0 =3D=3D>

x =3D cis(n)  e  x^(-1) =3D cis(-n)
ou  
x =3D cis(-n)  e  x^(-1) =3D cis(n) =3D=3D>

x^13 =3D cis(13n)  e  x^(-13) =3D cis(-13n)
ou
x^13 =3D cis(-13n)  e  x^(-13) =3D cis(13n) =3D=3D>

de qualquer forma, x^13 + x^(-13) =3D 2*cos(13n)

--MS_Mac_OE_3132175984_1328359_MIME_Part-- ========================================================================= Instruções para entrar na lista, sair da lista e usar a lista em http://www.mat.puc-rio.br/~nicolau/olimp/obm-l.html O administrador desta lista é ========================================================================= From owner-obm-l@sucuri.mat.puc-rio.br Thu Apr 3 02:53:33 2003 Return-Path: Received: (from majordom@localhost) by sucuri.mat.puc-rio.br (8.9.3/8.9.3) id CAA09819 for obm-l-MTTP; Thu, 3 Apr 2003 02:51:40 -0300 Received: from ivoti.terra.com.br (ivoti.terra.com.br [200.176.3.20]) by sucuri.mat.puc-rio.br (8.9.3/8.9.3) with ESMTP id CAA09815 for ; Thu, 3 Apr 2003 02:51:37 -0300 Received: from marova.terra.com.br (marova.terra.com.br [200.176.3.39]) by ivoti.terra.com.br (Postfix) with ESMTP id 8CA90408548 for ; Thu, 3 Apr 2003 02:51:07 -0300 (BRT) Received: from usuario (200-180-188-144.paemt7005.dsl.brasiltelecom.net.br [200.180.188.144]) (authenticated user marioappereira) by marova.terra.com.br (Postfix) with ESMTP id 562D43DC04A for ; Thu, 3 Apr 2003 02:51:07 -0300 (BRT) Message-ID: <006601c2f9a5$0709ad70$0301a8c0@usuario> From: =?iso-8859-1?Q?M=E1rio_Pereira?= To: Subject: [obm-l] Date: Thu, 3 Apr 2003 02:51:09 -0300 MIME-Version: 1.0 Content-Type: multipart/alternative; boundary="----=_NextPart_000_0063_01C2F98B.E161D250" X-Priority: 3 X-MSMail-Priority: Normal X-Mailer: Microsoft Outlook Express 6.00.2600.0000 X-MimeOLE: Produced By Microsoft MimeOLE V6.00.2600.0000 Sender: owner-obm-l@sucuri.mat.puc-rio.br Precedence: bulk Reply-To: obm-l@mat.puc-rio.br This is a multi-part message in MIME format. ------=_NextPart_000_0063_01C2F98B.E161D250 Content-Type: text/plain; charset="iso-8859-1" Content-Transfer-Encoding: quoted-printable Por favor:=20 O n=FAmero N de 2 algarismos positivos e inteiros =E9 tal que se forem = invertidos o segundo n=FAmero excede o primeiro em 27 unidades. Calcule = esse n=FAmero sabendo que a soma dos algarismos de N =3D 11.=20 Observa=E7=E3o: h=E1 uma dica dizendo para resolver por sistema.=20 Obrigado pela ajuda.=20 M=E1rio ------=_NextPart_000_0063_01C2F98B.E161D250 Content-Type: text/html; charset="iso-8859-1" Content-Transfer-Encoding: quoted-printable
Por favor:
 
O n=FAmero N de 2 algarismos positivos = e inteiros =E9=20 tal que se forem invertidos o segundo n=FAmero excede o primeiro em 27 = unidades.=20 Calcule esse n=FAmero sabendo que a soma dos algarismos de N =3D 11. =
Observa=E7=E3o: h=E1 uma dica dizendo = para resolver por=20 sistema.
 
Obrigado pela ajuda.
 
 
M=E1rio
 
------=_NextPart_000_0063_01C2F98B.E161D250-- ========================================================================= Instruções para entrar na lista, sair da lista e usar a lista em http://www.mat.puc-rio.br/~nicolau/olimp/obm-l.html O administrador desta lista é ========================================================================= From owner-obm-l@sucuri.mat.puc-rio.br Thu Apr 3 06:01:24 2003 Return-Path: Received: (from majordom@localhost) by sucuri.mat.puc-rio.br (8.9.3/8.9.3) id FAA12088 for obm-l-MTTP; Thu, 3 Apr 2003 05:59:38 -0300 Received: from sina.bol.com.br (sina.bol.com.br [200.221.24.27]) by sucuri.mat.puc-rio.br (8.9.3/8.9.3) with ESMTP id FAA12084 for ; Thu, 3 Apr 2003 05:59:35 -0300 Received: from vinicius (200.221.24.191) by sina.bol.com.br (5.1.071) id 3E7665FB004EC907 for obm-l@mat.puc-rio.br; Thu, 3 Apr 2003 05:59:05 -0300 Message-ID: <000d01c2f9bf$298be920$05a564c8@vinicius> From: "Vinicius Junqueira Filho" To: References: <006601c2f9a5$0709ad70$0301a8c0@usuario> Subject: Re: [obm-l] Date: Thu, 3 Apr 2003 05:57:04 -0300 MIME-Version: 1.0 Content-Type: text/plain; charset="iso-8859-1" Content-Transfer-Encoding: 8bit X-Priority: 3 X-MSMail-Priority: Normal X-Mailer: Microsoft Outlook Express 5.50.4133.2400 X-MimeOLE: Produced By Microsoft MimeOLE V5.50.4133.2400 X-Sender-IP: 200.100.165.5 Sender: owner-obm-l@sucuri.mat.puc-rio.br Precedence: bulk Reply-To: obm-l@mat.puc-rio.br Mário, chamando os algarismos do número N de "a" e "b", tal que N = ab, e pelas condições dadas: a + b = 11 (I) e ba - ab = 27 (II). Por (I), a solução será um destes pares ordenados: (9;2), (8;3), (7;4), (6;5), (5;6), (4;7), (3;8) ou (2;9). Essa é a primeira restrição a ser observada. Por (II), conclui-se que: b > a. (III) Portanto, diminui-se os pares ordenados possíveis que solucionam o nosso problema. Ficam: (5;6), (4;7), (3;8) e (2;9), porque ordenamos os pares na forma (a;b). Bem, tomando a condição (III) e a (II) simultaneamente, conclui-se que: 10 + a - b = 7 ou b - 1 - a = 2 --- (Leva-se em conta que o número "ba" tem um algarismo menor representando a casa das unidades) e por (I), a + b = 11. Resolvendo o sistema formado pelas duas equações (escolha "10 + a - b = 7" ou "b - 1 - a = 2") , teremos a = 4 e por (II), b = 7, e como resposta para o seu problema teremos N = 47. Um abraço. Continue a mandar problemas. Vinicius Junqueira Filho. ========================================================================= Instruções para entrar na lista, sair da lista e usar a lista em http://www.mat.puc-rio.br/~nicolau/olimp/obm-l.html O administrador desta lista é ========================================================================= From owner-obm-l@sucuri.mat.puc-rio.br Thu Apr 3 08:02:32 2003 Return-Path: Received: (from majordom@localhost) by sucuri.mat.puc-rio.br (8.9.3/8.9.3) id IAA13685 for obm-l-MTTP; Thu, 3 Apr 2003 08:00:49 -0300 Received: from paiol.terra.com.br (paiol.terra.com.br [200.176.3.18]) by sucuri.mat.puc-rio.br (8.9.3/8.9.3) with ESMTP id IAA13680 for ; Thu, 3 Apr 2003 08:00:46 -0300 Received: from marova.terra.com.br (marova.terra.com.br [200.176.3.39]) by paiol.terra.com.br (Postfix) with ESMTP id B23AB88CB2 for ; Thu, 3 Apr 2003 08:00:15 -0300 (BRT) Received: from [200.177.182.84] (dl-nas6-sao-C8B1B654.p001.terra.com.br [200.177.182.84]) by marova.terra.com.br (Postfix) with ESMTP id BEBB73DC07A for ; Thu, 3 Apr 2003 08:00:14 -0300 (BRT) User-Agent: Microsoft-Outlook-Express-Macintosh-Edition/5.02.2022 Date: Thu, 03 Apr 2003 07:58:23 -0300 Subject: Re: [obm-l] From: Claudio Buffara To: Message-ID: In-Reply-To: <006601c2f9a5$0709ad70$0301a8c0@usuario> Mime-version: 1.0 Content-type: multipart/alternative; boundary="MS_Mac_OE_3132201503_28831_MIME_Part" Sender: owner-obm-l@sucuri.mat.puc-rio.br Precedence: bulk Reply-To: obm-l@mat.puc-rio.br > This message is in MIME format. Since your mail reader does not understand this format, some or all of this message may not be legible. --MS_Mac_OE_3132201503_28831_MIME_Part Content-type: text/plain; charset="ISO-8859-1" Content-transfer-encoding: quoted-printable on 03.04.03 02:51, M=E1rio Pereira at marioappereira@terra.com.br wrote: Por favor:=20 =20 O n=FAmero N de 2 algarismos positivos e inteiros =E9 tal que se forem invertidos o segundo n=FAmero excede o primeiro em 27 unidades. Calcule esse n=FAmero sabendo que a soma dos algarismos de N =3D 11. Observa=E7=E3o: h=E1 uma dica dizendo para resolver por sistema. =20 Obrigado pela ajuda. =20 M=E1rio =20 N =3D (ab) =3D 10a + b (10b + a) - (10a + b) =3D 27 =3D=3D> 9(b - a) =3D 27 =3D=3D> b - a =3D 3 Tambem: b + a =3D 11 Logo, voce obtem o "sistema" de equacoes: b + a =3D 11 b - a =3D 3 cuja solucao eh: a =3D 4, b =3D 7 =3D=3D> N =3D 47 Um abraco, Claudio. --MS_Mac_OE_3132201503_28831_MIME_Part Content-type: text/html; charset="ISO-8859-1" Content-transfer-encoding: quoted-printable Re: [obm-l] on 03.04.03 02:51, M=E1rio Pereira at marioappereira@terra.com.br wrote:

Por favor:

O n=FAmero N de 2 algarismos positivos e in= teiros =E9 tal que se forem invertidos o segundo n=FAmero excede o primeiro em 2= 7 unidades. Calcule esse n=FAmero sabendo que a soma dos algarismos de N =3D 11.=
Observa=E7=E3o: h=E1 uma dica dizendo para resolver por sistema.

Obrigado pela ajuda.


M=E1rio


N =3D (ab) =3D 10a + b

(10b + a) - (10a + b) =3D 27 =3D=3D> 9(b - a) =3D 27 =3D=3D> b - a =3D 3

Tambem: b + a =3D 11

Logo, voce obtem o "sistema"  de equacoes:
b + a =3D 11
b - a =3D 3
cuja solucao eh: a =3D 4, b =3D 7 =3D=3D> N =3D 47

Um abraco,
Claudio. --MS_Mac_OE_3132201503_28831_MIME_Part-- ========================================================================= Instruções para entrar na lista, sair da lista e usar a lista em http://www.mat.puc-rio.br/~nicolau/olimp/obm-l.html O administrador desta lista é ========================================================================= From owner-obm-l@sucuri.mat.puc-rio.br Thu Apr 3 16:43:07 2003 Return-Path: Received: (from majordom@localhost) by sucuri.mat.puc-rio.br (8.9.3/8.9.3) id QAA22452 for obm-l-MTTP; Thu, 3 Apr 2003 16:38:30 -0300 Received: from web14305.mail.yahoo.com (web14305.mail.yahoo.com [216.136.173.81]) by sucuri.mat.puc-rio.br (8.9.3/8.9.3) with SMTP id QAA22448 for ; Thu, 3 Apr 2003 16:38:25 -0300 Message-ID: <20030403193752.16540.qmail@web14305.mail.yahoo.com> Received: from [200.17.25.3] by web14305.mail.yahoo.com via HTTP; Thu, 03 Apr 2003 16:37:52 ART Date: Thu, 3 Apr 2003 16:37:52 -0300 (ART) From: =?iso-8859-1?q?Rafael?= Subject: Re: [obm-l] Números_complexos To: obm-l@mat.puc-rio.br In-Reply-To: <3E8BA2A4.5040904@centroin.com.br> MIME-Version: 1.0 Content-Type: text/plain; charset=iso-8859-1 Content-Transfer-Encoding: 8bit Sender: owner-obm-l@sucuri.mat.puc-rio.br Precedence: bulk Reply-To: obm-l@mat.puc-rio.br Outro dia resolvi um assim: "Se x + x^(-1) = a, ao escrever x^(13) + x^(-13) como um polinômio em a, a soma dos coeficientes é igual a:" Vendo este exercício lembrei dele, só que eu só consegui resolver meio braçal mesmo, elevando as potências. Alguém consegue um caminho melhor?? A resposta é 1. Abraços, Rafael. > > prove que se x + x^ (- 1) = 2 cos n, então x^13 + > x^(-13) = 2 cos 13n. --- "A. C. Morgado" escreveu: > 3) x^2 - x.2cosn +1 = 0 > x = cosn (+-) i sen n > x^13 = cos 13n (+-) i sen13n > x^(-13) = cos 13n (-+) i sen 13n > x^13 + x^(-13) = 2cos13n _______________________________________________________________________ Yahoo! Mail O melhor e-mail gratuito da internet: 6MB de espaço, antivírus, acesso POP3, filtro contra spam. http://br.mail.yahoo.com/ ========================================================================= Instruções para entrar na lista, sair da lista e usar a lista em http://www.mat.puc-rio.br/~nicolau/olimp/obm-l.html O administrador desta lista é ========================================================================= From owner-obm-l@sucuri.mat.puc-rio.br Thu Apr 3 21:10:40 2003 Return-Path: Received: (from majordom@localhost) by sucuri.mat.puc-rio.br (8.9.3/8.9.3) id VAA26817 for obm-l-MTTP; Thu, 3 Apr 2003 21:09:01 -0300 Received: from paiol.terra.com.br (paiol.terra.com.br [200.176.3.18]) by sucuri.mat.puc-rio.br (8.9.3/8.9.3) with ESMTP id VAA26813 for ; Thu, 3 Apr 2003 21:08:58 -0300 Received: from jurua.terra.com.br (jurua.terra.com.br [200.176.3.42]) by paiol.terra.com.br (Postfix) with ESMTP id 0BCF787E19 for ; Thu, 3 Apr 2003 21:08:28 -0300 (BRT) Received: from [200.177.176.11] (dl-nas1-sao-C8B1B00B.p001.terra.com.br [200.177.176.11]) by jurua.terra.com.br (Postfix) with ESMTP id 04C64B001A for ; Thu, 3 Apr 2003 21:08:27 -0300 (BRT) User-Agent: Microsoft-Outlook-Express-Macintosh-Edition/5.02.2022 Date: Thu, 03 Apr 2003 21:06:36 -0300 Subject: Re: [obm-l] N=?ISO-8859-1?B?+g==?=meros_complexos From: Claudio Buffara To: Message-ID: In-Reply-To: <20030403193752.16540.qmail@web14305.mail.yahoo.com> Mime-version: 1.0 Content-type: text/plain; charset="ISO-8859-1" Content-Transfer-Encoding: 8bit X-MIME-Autoconverted: from quoted-printable to 8bit by sucuri.mat.puc-rio.br id VAA26814 Sender: owner-obm-l@sucuri.mat.puc-rio.br Precedence: bulk Reply-To: obm-l@mat.puc-rio.br Oi, Rafael: Da uma olhada nisso aqui: x + 1/x = a ==> x^13 + 1/x^13 = P(a) Queremos o valor de P(1). Assim, fazendo a = 1 teremos: x + 1/x = 1 ==> x^2 - x + 1 = 0 ==> x = 1/2 + i*raiz(3)/2 = cis(pi/3) ou x = 1/2 - i*raiz(3)/2 = cis(-pi/3) ==> Ambas as raizes sao raizes sextas da unidade ==> Para ambas, x^12 = 1 ==> x^13 = x = cis(pi/3) ou x^13 = x = cis(-pi/3) ==> 1/x^13 = cis(-pi/3) ou 1/x^13 = cis(pi/3) ==> De qualquer jeito: x^13 + 1/x^13 = cis(pi/3) + cis(-pi/3) = 1 = P(1) Um abraco, Claudio. on 03.04.03 16:37, Rafael at matduvidas@yahoo.com.br wrote: > Outro dia resolvi um assim: > "Se x + x^(-1) = a, ao escrever x^(13) + x^(-13) como > um polinômio em a, a soma dos coeficientes é igual a:" > > > Vendo este exercício lembrei dele, só que eu só > consegui resolver meio braçal mesmo, elevando as > potências. Alguém consegue um caminho melhor?? > > A resposta é 1. > > Abraços, > > Rafael. > >>> prove que se x + x^ (- 1) = 2 cos n, então x^13 + >> x^(-13) = 2 cos 13n. > > --- "A. C. Morgado" > escreveu: > 3) x^2 - x.2cosn +1 = 0 >> x = cosn (+-) i sen n >> x^13 = cos 13n (+-) i sen13n >> x^(-13) = cos 13n (-+) i sen 13n >> x^13 + x^(-13) = 2cos13n > > _______________________________________________________________________ > Yahoo! Mail > O melhor e-mail gratuito da internet: 6MB de espaço, antivírus, acesso POP3, > filtro contra spam. > http://br.mail.yahoo.com/ > ========================================================================= > Instruções para entrar na lista, sair da lista e usar a lista em > http://www.mat.puc-rio.br/~nicolau/olimp/obm-l.html > O administrador desta lista é > ========================================================================= > ========================================================================= Instruções para entrar na lista, sair da lista e usar a lista em http://www.mat.puc-rio.br/~nicolau/olimp/obm-l.html O administrador desta lista é ========================================================================= From owner-obm-l@sucuri.mat.puc-rio.br Thu Apr 3 21:22:24 2003 Return-Path: Received: (from majordom@localhost) by sucuri.mat.puc-rio.br (8.9.3/8.9.3) id VAA27063 for obm-l-MTTP; Thu, 3 Apr 2003 21:21:05 -0300 Received: from ivoti.terra.com.br (ivoti.terra.com.br [200.176.3.20]) by sucuri.mat.puc-rio.br (8.9.3/8.9.3) with ESMTP id VAA27058 for ; Thu, 3 Apr 2003 21:21:02 -0300 Received: from araci.terra.com.br (araci.terra.com.br [200.176.3.44]) by ivoti.terra.com.br (Postfix) with ESMTP id BC74E40949D for ; Thu, 3 Apr 2003 21:20:31 -0300 (BRT) Received: from [200.177.182.216] (dl-nas6-sao-C8B1B6D8.p001.terra.com.br [200.177.182.216]) by araci.terra.com.br (Postfix) with ESMTP id BB9A5C40A3 for ; Thu, 3 Apr 2003 21:20:30 -0300 (BRT) User-Agent: Microsoft-Outlook-Express-Macintosh-Edition/5.02.2022 Date: Thu, 03 Apr 2003 21:18:40 -0300 Subject: FW: [obm-l] N=?ISO-8859-1?B?+g==?=meros_complexos From: Claudio Buffara To: Lista OBM Message-ID: In-Reply-To: Mime-version: 1.0 Content-type: text/plain; charset="ISO-8859-1" Content-Transfer-Encoding: 8bit X-MIME-Autoconverted: from quoted-printable to 8bit by sucuri.mat.puc-rio.br id VAA27059 Sender: owner-obm-l@sucuri.mat.puc-rio.br Precedence: bulk Reply-To: obm-l@mat.puc-rio.br De fato, o metodo abaixo permite que se calcule a soma dos coeficientes de P_n(a) ( igual a P_n(1) ) para todo n natural, onde: P_n(a) = x^n + 1/x^n ==> P_n(1) = cis(n*pi/3) + cis(-n*pi/3) Assim: n = 0 (mod 6) ==> P_n(1) = 2 n = 1 (mod 6) ==> P_n(1) = 1 n = 2 (mod 6) ==> P_n(1) = -1 n = 3 (mod 6) ==> P_n(1) = -2 n = 4 (mod 6) ==> P_n(1) = -1 n = 5 (mod 6) ==> P_n(1) = 1 Um abraco, Claudio. ---------- From: Claudio Buffara Date: Thu, 03 Apr 2003 21:06:36 -0300 To: Subject: Re: [obm-l] Números_complexos Oi, Rafael: Da uma olhada nisso aqui: x + 1/x = a ==> x^13 + 1/x^13 = P(a) Queremos o valor de P(1). Assim, fazendo a = 1 teremos: x + 1/x = 1 ==> x^2 - x + 1 = 0 ==> x = 1/2 + i*raiz(3)/2 = cis(pi/3) ou x = 1/2 - i*raiz(3)/2 = cis(-pi/3) ==> Ambas as raizes sao raizes sextas da unidade ==> Para ambas, x^12 = 1 ==> x^13 = x = cis(pi/3) ou x^13 = x = cis(-pi/3) ==> 1/x^13 = cis(-pi/3) ou 1/x^13 = cis(pi/3) ==> De qualquer jeito: x^13 + 1/x^13 = cis(pi/3) + cis(-pi/3) = 1 = P(1) Um abraco, Claudio. on 03.04.03 16:37, Rafael at matduvidas@yahoo.com.br wrote: > Outro dia resolvi um assim: > "Se x + x^(-1) = a, ao escrever x^(13) + x^(-13) como > um polinômio em a, a soma dos coeficientes é igual a:" > > > Vendo este exercício lembrei dele, só que eu só > consegui resolver meio braçal mesmo, elevando as > potências. Alguém consegue um caminho melhor?? > > A resposta é 1. > > Abraços, > > Rafael. > >>> prove que se x + x^ (- 1) = 2 cos n, então x^13 + >> x^(-13) = 2 cos 13n. > > --- "A. C. Morgado" > escreveu: > 3) x^2 - x.2cosn +1 = 0 >> x = cosn (+-) i sen n >> x^13 = cos 13n (+-) i sen13n >> x^(-13) = cos 13n (-+) i sen 13n >> x^13 + x^(-13) = 2cos13n > > _______________________________________________________________________ > Yahoo! Mail > O melhor e-mail gratuito da internet: 6MB de espaço, antivírus, acesso POP3, > filtro contra spam. > http://br.mail.yahoo.com/ > ========================================================================= > Instruções para entrar na lista, sair da lista e usar a lista em > http://www.mat.puc-rio.br/~nicolau/olimp/obm-l.html > O administrador desta lista é > ========================================================================= > ========================================================================= Instruções para entrar na lista, sair da lista e usar a lista em http://www.mat.puc-rio.br/~nicolau/olimp/obm-l.html O administrador desta lista é ========================================================================= From owner-obm-l@sucuri.mat.puc-rio.br Thu Apr 3 21:49:07 2003 Return-Path: Received: (from majordom@localhost) by sucuri.mat.puc-rio.br (8.9.3/8.9.3) id VAA27843 for obm-l-MTTP; Thu, 3 Apr 2003 21:47:43 -0300 Received: from hotmail.com (f20.law9.hotmail.com [64.4.9.20]) by sucuri.mat.puc-rio.br (8.9.3/8.9.3) with ESMTP id VAA27838 for ; Thu, 3 Apr 2003 21:47:39 -0300 Received: from mail pickup service by hotmail.com with Microsoft SMTPSVC; Thu, 3 Apr 2003 16:47:08 -0800 Received: from 200.222.178.119 by lw9fd.law9.hotmail.msn.com with HTTP; Fri, 04 Apr 2003 00:47:08 GMT X-Originating-IP: [200.222.178.119] X-Originating-Email: [ricardoprins@hotmail.com] From: "Ricardo Prins" To: obm-l@mat.puc-rio.br Subject: Re: [obm-l] Teorema dos Casamentos Date: Fri, 04 Apr 2003 00:47:08 +0000 Mime-Version: 1.0 Content-Type: text/html Message-ID: X-OriginalArrivalTime: 04 Apr 2003 00:47:08.0535 (UTC) FILETIME=[B8863070:01C2FA43] Sender: owner-obm-l@sucuri.mat.puc-rio.br Precedence: bulk Reply-To: obm-l@mat.puc-rio.br

Claudio, muito obrigado. Você pode me recomendar um livro que fale mais profundamente sobre números complexos?

>From: Claudio Buffara
>Reply-To: obm-l@mat.puc-rio.br
>To:
>Subject: [obm-l] Teorema dos Casamentos
>Date: Wed, 02 Apr 2003 23:42:18 -0300
>
>Caro Ricardo:
>
>Segue abaixo a demonstracao.
>
>Teorema dos Casamentos:
>Sejam A(1), A(2), ..., A(n) conjuntos tais que a união da quaisquer i deles
>(1 <= i <= n) contém no mínimo i elementos distintos.
>Então é possível selecionar n elementos distintos, sendo um de cada
>conjunto.
>
>Dem:
>Inducao completa em n (numero de conjuntos):
>
>n = 1: obvio
>
>Hipotese de Inducao: o teorema eh verdadeiro para 1 <= n <= m-1.
>
>Consideremos m conjuntos A(1), ..., A(m) tais que a uniao de quaisquer k
>deles (1 <= i <= m) contenha pelo menos i elementos distintos.
>
>
>Temos dois casos a considerar:
>
>CASO 1: Para cada k (1 <= k <= m-1), a união de quaisquer k conjuntos contém
>pelo menos k+1 elementos.
>
>Nesse caso, escolha um elemento qualquer de A(m) - digamos "a".
>
>Como a uniao dos m-1 conjuntos A(1), ..., A(m-1) contem pelo menos m
>elementos, a uniao de A(1) - {a}, ..., A(m-1) - {a} ira conter, no minimo,
>m-1 elementos.
>
>Assim, pela hipotese de inducao, podemos escolher um elemento distinto de
>cada um destes conjuntos.
>
>Estes m-1 elementos, juntamente com "a", serao m elementos distintos, cada
>um escolhido de um dos A(i) (1 <= i <= m)
>
>----------
>
>CASO 2: Existem: (i) um inteiro k (1 <= k <= m-1) e (ii) k conjuntos tais
>que a sua união contem exatamemente k elementos.
>
>Como 1 <= k <= m-1, podemos aplicar a hipotese de inducao e escolher um
>elemento distinto de cada um dos k conjuntos cuja uniao contem k elementos.
>
>Suponhamos que dentre os m-k conjuntos restantes, existam j ( 1 <= j <= m-k)
>cuja uniao contenha menos do que j elementos que sejam distintos dos k
>elementos escolhidos acima.
>
>Entao, a uniao dos k conjuntos iniciais com estes j conjuntos ira conter
>menos do que k + j elementos, o que contradiz a hipotese do teorema sobre
>estes conjuntos.
>
>Logo, dentre os m-k conjuntos restantes, a uniao de quaisquer j ( 1 <= j <=
>m-k) ira conter pelo menos j elementos e todos eles serao distintos dos k
>elementos escolhidos inicialmente.
>
>Assim, podemos tambem aplicar a hipotese de inducao a estes m-k conjuntos e
>escolher um elemento distinto de cada um deles. Alem do mais, podemos fazer
>isso de forma que estes elementos sejam distintos dos k elementos escolhidos
>inicialmente.
>
>Em suma, tambem neste caso eh possivel escolher m elementos distintos, sendo
>um de cada um dos A(i) (1 <= i <= n)
>
>*** FIM ***
>
>Um abraco,
>Claudio.
>
>on 02.04.03 08:09, Ricardo Prins at ricardoprins@hotmail.com wrote:
>
>me intrometendo...
>
>Você pode me enviar a demonstração?
>
>Ricardo
> >From: "Cláudio \(Prática\)"
> >Reply-To: obm-l@mat.puc-rio.br
> >To:
> >Subject: Re: [obm-l] Grafos e Casamentos
> >Date: Mon, 31 Mar 2003 15:57:27 -0300
> >
> >Oi, JP:
> >
> >O enunciado do Teorema dos Casamentos é o seguinte:
> >Sejam A(1), A(2), ..., A(n) conjuntos tais que a união da quaisquer k deles
> >(1 <= k <= n) contém no mínimo k elementos distintos. Então é possível
> >selecionar n elementos distintos, sendo um de cada conjunto.
> >
> >A demonstração padrão é por indução completa em n, e trata dois casos
> >separadamente:
> >i) Para cada k (1 <= k < n), a união de cada k conjuntos contém pelo menos
> >k+1 elementos;
> >ii) Existem k (1 <= k < n) e k conjuntos tais que a sua união tem
> >exatamemente k elementos.
> >
> >Se você quiser, depois eu posso mandar a demonstração.
> >
> >Um abraço,
> >Claudio.
>


Protect your PC - Click here for McAfee.com VirusScan Online ========================================================================= Instruções para entrar na lista, sair da lista e usar a lista em http://www.mat.puc-rio.br/~nicolau/olimp/obm-l.html O administrador desta lista é ========================================================================= From owner-obm-l@sucuri.mat.puc-rio.br Thu Apr 3 22:15:28 2003 Return-Path: Received: (from majordom@localhost) by sucuri.mat.puc-rio.br (8.9.3/8.9.3) id WAA28611 for obm-l-MTTP; Thu, 3 Apr 2003 22:13:52 -0300 Received: from hotmail.com (f73.law9.hotmail.com [64.4.9.73]) by sucuri.mat.puc-rio.br (8.9.3/8.9.3) with ESMTP id WAA28599 for ; Thu, 3 Apr 2003 22:13:47 -0300 Received: from mail pickup service by hotmail.com with Microsoft SMTPSVC; Thu, 3 Apr 2003 17:13:16 -0800 Received: from 200.222.178.119 by lw9fd.law9.hotmail.msn.com with HTTP; Fri, 04 Apr 2003 01:13:15 GMT X-Originating-IP: [200.222.178.119] X-Originating-Email: [ricardoprins@hotmail.com] From: "Ricardo Prins" To: obm-l@mat.puc-rio.br Subject: [obm-l] Sobre o problema do cubo Date: Fri, 04 Apr 2003 01:13:15 +0000 Mime-Version: 1.0 Content-Type: text/html Message-ID: X-OriginalArrivalTime: 04 Apr 2003 01:13:16.0215 (UTC) FILETIME=[5EEF2C70:01C2FA47] Sender: owner-obm-l@sucuri.mat.puc-rio.br Precedence: bulk Reply-To: obm-l@mat.puc-rio.br
Bem, eu não me lembro exatamente quais eram as opções, mas cheguei a uma conclusão (hilária e intrigante na minha opinião) de que a resposta deve ser dez. Experimentando diversas opções e posições para os uns e menosuns nos vértices, notei um padrão nas somas...todas elas são da forma 2(2n-1) (n inteiro - para n=-1, 0, 1,2 e 4 - funçãozinha descontínua...que pena...) mas mesmo assim, das opções que me recordo, apenas o dez se encaixa nesse padrão. Agora eu vou em busca de uma justificativa mais lógica pra esse padrão...não sou muito fã de experimentação brusca - nada contra os químicos!


Protect your PC - Click here for McAfee.com VirusScan Online ========================================================================= Instruções para entrar na lista, sair da lista e usar a lista em http://www.mat.puc-rio.br/~nicolau/olimp/obm-l.html O administrador desta lista é ========================================================================= From owner-obm-l@sucuri.mat.puc-rio.br Thu Apr 3 22:20:06 2003 Return-Path: Received: (from majordom@localhost) by sucuri.mat.puc-rio.br (8.9.3/8.9.3) id WAA28753 for obm-l-MTTP; Thu, 3 Apr 2003 22:18:48 -0300 Received: from shannon.bol.com.br (shannon.bol.com.br [200.221.24.13]) by sucuri.mat.puc-rio.br (8.9.3/8.9.3) with ESMTP id WAA28749 for ; Thu, 3 Apr 2003 22:18:45 -0300 Received: from bol.com.br (200.221.24.138) by shannon.bol.com.br (5.1.071) id 3E76735F00526BC8 for obm-l@mat.puc-rio.br; Thu, 3 Apr 2003 22:18:15 -0300 Date: Thu, 3 Apr 2003 22:18:15 -0300 Message-Id: Subject: [obm-l] ALGARISMOS SIGNIFICATIVOS MIME-Version: 1.0 Content-Type: text/plain;charset="iso-8859-1" From: "renatinha15a" To: "obm" X-XaM3-API-Version: 2.4 R3 ( B4 ) X-SenderIP: 200.241.108.194 Content-Transfer-Encoding: 8bit X-MIME-Autoconverted: from quoted-printable to 8bit by sucuri.mat.puc-rio.br id WAA28750 Sender: owner-obm-l@sucuri.mat.puc-rio.br Precedence: bulk Reply-To: obm-l@mat.puc-rio.br oi pessoal, estou postando uma dúvida que li num fórum de física, mas, creio, se refere à conceitos matemáticos. Fico grata se alguém me ajudar. A minha dúvida consiste em como definir ou saber o porquê da variação de casas decimais que acontecem no "desenrolar" das equações, pois vejo em muitas resoluções de provas e livros, conflitos quanto ao método. Vou exemplificar minha dúvida com uma solução do manual do professor do livro Física Clássica: (ITA) De um telhado caem gotas de chuva separadas por intervalos de tempo iguais entre si. No momento em que a 5ª gota se desprende, a primeira toca o solo. Qual a distância que separa as duas últimas gotas consecutivas (4ª e 5ª), neste instante, se a altura do telhado é de 20 m: (Fazer g = 10 m/s^2 e não considerar a resistência do ar.) SOLUÇÃO: Cálculo do tempo de queda: 1) s = gt^2/2 ==> 20 = [(10)tq^2]/2 ==> tq = 2,0s Intervalo de tempo entre 2 gotas consecutivas: 2) Δt = 2,0/4 = 0,50s Espaço entre a 5ª e a 4ª gota: 3) Δs = gt^2/2 = (10J)(0,50)^2 Resposta: Δs = 1,25 m (DÚVIDA) O enunciado dá dois valores, s = 20 m e g = 10 m/s^2, ambos com dois algarismos significativos. Em 1) verifica-se que a resposta (tq = 2,0) mantêm os dois alg. significativos. Agora és o problema! Em 2) a resposta contém 3 números significativos. Li num livro (QUÍMICA GERAL - James E. Brady) que existe duas regras, uma para multiplicação/divisão e outra para adição e subtração, para expressar o resultado -o resultado da adição ou mult.,etc e não o resultado final-. Na multi./div. ele diz: "Em geral, para multiplicação e divisão, o produto ou o quociente não deve possuir mais algarismos significatios do que o fator menos preciso utilizado no cálculo." Concluo, então, que o valor menos preciso, em 2), tem dois alg. sign. e, portanto, a resposta deveria ser 0,5 e não 0,50. Com esse fato, 3) fica comprometido. Finalmente, como eu devo entender isso? O que posso fazer numa situação de prova discursiva? []´s Renatinha __________________________________________________________________________ E-mail Premium BOL Antivírus, anti-spam e até 100 MB de espaço. Assine já! http://email.bol.com.br/ ========================================================================= Instruções para entrar na lista, sair da lista e usar a lista em http://www.mat.puc-rio.br/~nicolau/olimp/obm-l.html O administrador desta lista é ========================================================================= From owner-obm-l@sucuri.mat.puc-rio.br Thu Apr 3 23:00:01 2003 Return-Path: Received: (from majordom@localhost) by sucuri.mat.puc-rio.br (8.9.3/8.9.3) id WAA30043 for obm-l-MTTP; Thu, 3 Apr 2003 22:58:40 -0300 Received: from saks.bol.com.br (saks.bol.com.br [200.221.24.16]) by sucuri.mat.puc-rio.br (8.9.3/8.9.3) with ESMTP id WAA30039 for ; Thu, 3 Apr 2003 22:58:37 -0300 Received: from jr (200.221.24.48) by saks.bol.com.br (5.1.071) id 3E766E3A005663AD for obm-l@mat.puc-rio.br; Thu, 3 Apr 2003 22:58:07 -0300 From: "Hely Jr." To: Subject: [obm-l] =?iso-8859-1?Q?Demonstra=E7=F5es?= Date: Thu, 3 Apr 2003 22:58:03 -0300 Message-ID: <005d01c2fa4d$a245e2b0$4f9093c8@jr> MIME-Version: 1.0 Content-Type: multipart/alternative; boundary="----=_NextPart_000_005E_01C2FA34.7CFA3150" X-Priority: 3 (Normal) X-MSMail-Priority: Normal X-Mailer: Microsoft Outlook, Build 10.0.2627 X-MimeOLE: Produced By Microsoft MimeOLE V6.00.2600.0000 Importance: Normal X-Sender-IP: 200.147.144.79 Sender: owner-obm-l@sucuri.mat.puc-rio.br Precedence: bulk Reply-To: obm-l@mat.puc-rio.br This is a multi-part message in MIME format. ------=_NextPart_000_005E_01C2FA34.7CFA3150 Content-Type: text/plain; charset="iso-8859-1" Content-Transfer-Encoding: quoted-printable Alguem poderia me ajudar nestas demonstra=E7=F5es =20 1) sabendo que sqrt(3) e sqrt(5) s=E3o irracionais, verifique que = sqrt(3) + sqrt(5) =E9 irracional. =20 2) sejam p> 0 e q>0 primos distintos. verifique que sqrt(p) + sqrt(q) = =E9 irracional =20 3) se p e q s=E0o inteiros positivos distintos e pelo menos um dos = numeros sqrt(p) ou sqrt(q) =E9 irracional, ent=E3o sqrt(p) + sqrt(q) =E9 tb irracional. =20 desde ja agrade=E7o ------=_NextPart_000_005E_01C2FA34.7CFA3150 Content-Type: text/html; charset="iso-8859-1" Content-Transfer-Encoding: quoted-printable Mensagem
Alguem = poderia me=20 ajudar nestas demonstra=E7=F5es
 
1) = sabendo que=20 sqrt(3) e sqrt(5) s=E3o irracionais, verifique que sqrt(3) + sqrt(5) =E9 = irracional.
 
2) = sejam p> 0 e=20 q>0 primos distintos. verifique que sqrt(p) + sqrt(q) =E9=20 irracional
 
3) se = p e q s=E0o=20 inteiros positivos distintos e pelo menos um dos numeros sqrt(p) ou = sqrt(q) =E9=20 irracional, ent=E3o sqrt(p) + sqrt(q) =E9 tb = irracional.
 
desde = ja=20 agrade=E7o
------=_NextPart_000_005E_01C2FA34.7CFA3150-- ========================================================================= Instruções para entrar na lista, sair da lista e usar a lista em http://www.mat.puc-rio.br/~nicolau/olimp/obm-l.html O administrador desta lista é ========================================================================= From owner-obm-l@sucuri.mat.puc-rio.br Thu Apr 3 23:55:22 2003 Return-Path: Received: (from majordom@localhost) by sucuri.mat.puc-rio.br (8.9.3/8.9.3) id XAA31018 for obm-l-MTTP; Thu, 3 Apr 2003 23:53:55 -0300 Received: from trex.centroin.com.br (trex.centroin.com.br [200.225.63.134]) by sucuri.mat.puc-rio.br (8.9.3/8.9.3) with ESMTP id XAA31014 for ; Thu, 3 Apr 2003 23:53:52 -0300 Received: from fundoc0hgjgn81 (179234231.rjo.virtua.com.br [200.179.234.231]) (authenticated bits=0) by trex.centroin.com.br (8.12.9/8.12.1) with ESMTP id h342rhHR003644 for ; Thu, 3 Apr 2003 23:53:44 -0300 (EST) Message-ID: <001d01c2fa55$593e8970$e7eab3c8@fundoc0hgjgn81> From: "Diego Navarro" To: References: <005d01c2fa4d$a245e2b0$4f9093c8@jr> Subject: [obm-l] =?iso-8859-1?Q?Re:_=5Bobm-l=5D_Demonstra=E7=F5es?= Date: Thu, 3 Apr 2003 23:53:17 -0300 MIME-Version: 1.0 Content-Type: text/plain; charset="iso-8859-1" Content-Transfer-Encoding: 8bit X-Priority: 3 X-MSMail-Priority: Normal X-Mailer: Microsoft Outlook Express 6.00.2600.0000 X-MimeOLE: Produced By Microsoft MimeOLE V6.00.2600.0000 Sender: owner-obm-l@sucuri.mat.puc-rio.br Precedence: bulk Reply-To: obm-l@mat.puc-rio.br MensagemSuponha que não existem complexos. Na verdade, isso é mais por conveniência, já que não sei nada sobre complexos, mas parece razoável que a soma de dois números reais seja real. Um número racional é aquele que pode ser expresso pelo quociente de dois inteiros p e q. Suponha, por absurdo, que sqrt(3)+sqrt(5)=p/q q(sqrt(3)+sqrt(5))=p q*sqrt(3)+q*sqrt(5)=p Ora, para que a soma q*sqrt(3)+q*sqrt(5) seja inteira, é preciso que cada parcela seja racional (big dúvida: será mesmo? Dois números irracionais podem ter soma inteira? Alguém vai ter que conferir isto aqui). A única forma de que isso aconteça é (i) q=a/sqrt(3) e q=b/sqrt(5); a e b racionais a/sqrt(3) = b/sqrt(5) a*sqrt(5)=b*sqrt(3) a/b = sqrt(3)/sqrt(5) <--- irracional. Contradizendo (i). É uma demonstração meio trapaceada. Se for válida, é fácil expandir para quaisquer primos, já que a sqrt() de um primo é sempre irracional. Será que dá para demonstrar que a soma de dois irracionais não pode ser racional - excetuando o 0? Um, suponha, por absurdo, que dois números irracionais podem ter soma racional diferente de zero. (i) x + y = p/q y=(p-qx)/q x-y = x - (p-qx)/q = (qx-p-qx)/q=-p/q Mas por (i), x+y = p/q; logo, -x-y = -p/q x-y=-x-y ==> x = -x, o que só vale para 0. E nesse caso, y = p/q, ou seja, racional. Logo, dois números irracionais diferentes de zero não podem ter soma racional diferente de zero. Tá, esta segunda parte também parece um pouco trapaceada. Acho que preciso de ajuda. Mas se isto for verdade, a demonstração 3) é trivial, e as três estão respondidas. ----- Original Message ----- From: Hely Jr. To: obm-l@mat.puc-rio.br Sent: Thursday, April 03, 2003 10:58 PM Subject: [obm-l] Demonstrações Alguem poderia me ajudar nestas demonstrações 1) sabendo que sqrt(3) e sqrt(5) são irracionais, verifique que sqrt(3) + sqrt(5) é irracional. 2) sejam p> 0 e q>0 primos distintos. verifique que sqrt(p) + sqrt(q) é irracional 3) se p e q sào inteiros positivos distintos e pelo menos um dos numeros sqrt(p) ou sqrt(q) é irracional, então sqrt(p) + sqrt(q) é tb irracional. desde ja agradeço ========================================================================= Instruções para entrar na lista, sair da lista e usar a lista em http://www.mat.puc-rio.br/~nicolau/olimp/obm-l.html O administrador desta lista é ========================================================================= From owner-obm-l@sucuri.mat.puc-rio.br Fri Apr 4 00:55:49 2003 Return-Path: Received: (from majordom@localhost) by sucuri.mat.puc-rio.br (8.9.3/8.9.3) id AAA32164 for obm-l-MTTP; Fri, 4 Apr 2003 00:53:05 -0300 Received: from itaqui.terra.com.br (itaqui.terra.com.br [200.176.3.19]) by sucuri.mat.puc-rio.br (8.9.3/8.9.3) with ESMTP id AAA32160 for ; Fri, 4 Apr 2003 00:53:02 -0300 Received: from sapucaia.terra.com.br (sapucaia.terra.com.br [200.176.3.41]) by itaqui.terra.com.br (Postfix) with ESMTP id 5254D3BD26F for ; Fri, 4 Apr 2003 00:52:32 -0300 (BRT) Received: from [200.177.187.181] (dl-nas4-sao-C8B1BBB5.p001.terra.com.br [200.177.187.181]) by sapucaia.terra.com.br (Postfix) with ESMTP id 2F72F30404D for ; Fri, 4 Apr 2003 00:52:31 -0300 (BRT) User-Agent: Microsoft-Outlook-Express-Macintosh-Edition/5.02.2022 Date: Fri, 04 Apr 2003 00:51:20 -0300 Subject: [obm-l] Livro sobre Nos Complexos From: Claudio Buffara To: Message-ID: In-Reply-To: Mime-version: 1.0 Content-type: multipart/alternative; boundary="MS_Mac_OE_3132262280_980956_MIME_Part" Sender: owner-obm-l@sucuri.mat.puc-rio.br Precedence: bulk Reply-To: obm-l@mat.puc-rio.br > This message is in MIME format. Since your mail reader does not understand this format, some or all of this message may not be legible. --MS_Mac_OE_3132262280_980956_MIME_Part Content-type: text/plain; charset="ISO-8859-1" Content-transfer-encoding: quoted-printable on 03.04.03 21:47, Ricardo Prins at ricardoprins@hotmail.com wrote: Claudio, voc=EA pode me recomendar um livro que fale mais profundamente sobre n=FAmeros complexos? Ricardo: Acho que o volume sobre Numeros Complexos da colecao Fundamentos da Matematica Elementar (vol. 6, se nao me engano) contem tudo o que voce precisa saber sobre a teoria elementar dos nos. complexos (veja bem, elementar nao eh sinonimo de facil) A partir dai, a melhor maneira de se aprofundar eh atraves da resolucao de exercicios - livros russos e compilacoes de problemas de olimpiadas sao as melhores fontes. Tem tambem um artigo bem interessante na Eureka, sobre aplicacoes dos nos. complexos em geometria (um exemplo: prove que o produto dos comprimentos de todas as diagonais que emanam de um mesmo vertice de um n-agono regular inscrito num circulo de raio 1 eh igual a n) Na internet voce tambem encontra alguns sites com problemas interessantes, tais como este aqui: http://math.stanford.edu/~vakil/stanfordputnam/02/putnam3.pdf Alem disso, existe o enorme campo da analise complexa (ou calculo com variaveis complexas) onde a referencia basica eh o livro Complex Analysis d= o Lars Ahlfors. Espero que isso ja seja util. Um abraco, Claudio.=20 --MS_Mac_OE_3132262280_980956_MIME_Part Content-type: text/html; charset="ISO-8859-1" Content-transfer-encoding: quoted-printable Livro sobre Nos Complexos on 03.04.03 21:47, Ricardo Prins at ricardoprins@hotmail.com wrote:

Claudio, voc=EA pode me recomendar um livro que fale mais profund= amente sobre n=FAmeros complexos?

Ricardo:

Acho que o volume sobre Numeros Complexos da colecao Fundamentos da Matemat= ica Elementar (vol. 6, se nao me engano) contem tudo o que voce precisa sabe= r sobre a teoria elementar dos nos. complexos (veja bem, elementar nao eh si= nonimo de facil)

A partir dai, a melhor maneira de se aprofundar eh atraves da resolucao de = exercicios - livros russos e compilacoes de problemas de olimpiadas sao as m= elhores fontes.

Tem tambem um artigo bem interessante na Eureka, sobre aplicacoes dos nos. = complexos em geometria (um exemplo: prove que o produto dos comprimentos de = todas as diagonais que emanam de um mesmo vertice de um n-agono regular insc= rito num circulo de raio 1 eh igual a n)

Na internet voce tambem encontra alguns sites com problemas interessantes, = tais como este aqui:
http://math.stanford.edu/~vakil/stanfordputnam/02/putnam3.pdf

Alem disso, existe o enorme campo da analise complexa (ou calculo com varia= veis complexas) onde a referencia basica eh o livro Complex Analysis do Lars= Ahlfors.

Espero que isso ja seja util.

Um abraco,
Claudio.
--MS_Mac_OE_3132262280_980956_MIME_Part-- ========================================================================= Instruções para entrar na lista, sair da lista e usar a lista em http://www.mat.puc-rio.br/~nicolau/olimp/obm-l.html O administrador desta lista é ========================================================================= From owner-obm-l@sucuri.mat.puc-rio.br Fri Apr 4 01:34:36 2003 Return-Path: Received: (from majordom@localhost) by sucuri.mat.puc-rio.br (8.9.3/8.9.3) id BAA00601 for obm-l-MTTP; Fri, 4 Apr 2003 01:32:48 -0300 Received: from paiol.terra.com.br (paiol.terra.com.br [200.176.3.18]) by sucuri.mat.puc-rio.br (8.9.3/8.9.3) with ESMTP id BAA00596 for ; Fri, 4 Apr 2003 01:32:44 -0300 Received: from itaim.terra.com.br (itaim.terra.com.br [200.176.3.76]) by paiol.terra.com.br (Postfix) with ESMTP id 9D8D787C45 for ; Fri, 4 Apr 2003 01:32:14 -0300 (BRT) Received: from [200.177.188.177] (dl-nas7-sao-C8B1BCB1.p001.terra.com.br [200.177.188.177]) by itaim.terra.com.br (Postfix) with ESMTP id 2E1872E006A for ; Fri, 4 Apr 2003 01:32:13 -0300 (BRT) User-Agent: Microsoft-Outlook-Express-Macintosh-Edition/5.02.2022 Date: Fri, 04 Apr 2003 01:30:22 -0300 Subject: Re: [obm-l] Re: [obm-l] Demonstra=?ISO-8859-1?B?5/U=?=es From: Claudio Buffara To: Message-ID: In-Reply-To: <001d01c2fa55$593e8970$e7eab3c8@fundoc0hgjgn81> Mime-version: 1.0 Content-type: text/plain; charset="ISO-8859-1" Content-Transfer-Encoding: 8bit X-MIME-Autoconverted: from quoted-printable to 8bit by sucuri.mat.puc-rio.br id BAA00597 Sender: owner-obm-l@sucuri.mat.puc-rio.br Precedence: bulk Reply-To: obm-l@mat.puc-rio.br Caro Diego: Tomei a liberdade de inserir alguns comentarios em sua mensagem. Espero que voce nao se importe. on 03.04.03 23:53, Diego Navarro at diego@navarro.mus.br wrote: > MensagemSuponha que não existem complexos. Na verdade, isso é mais por > conveniência, já que não sei nada sobre complexos, mas parece razoável que a > soma de dois números reais seja real. > > Um número racional é aquele que pode ser expresso pelo quociente de dois > inteiros p e q. Suponha, por absurdo, que > > sqrt(3)+sqrt(5)=p/q > q(sqrt(3)+sqrt(5))=p > q*sqrt(3)+q*sqrt(5)=p > > Ora, para que a soma q*sqrt(3)+q*sqrt(5) seja inteira, é preciso que cada > parcela seja racional > (big dúvida: será mesmo? Dois números irracionais > podem ter soma inteira? Sim. Por exemplo: 2 + raiz(2) e 2 - raiz(2) sao ambos irracionais mas tem soma = 4. O problema 1 estah mal formulado, pois da a impressao de que se a e b sao irracionais entao a+b eh irracional, o que, pelo exemplo acima, nao eh sempre verdade, apesar de ser verdade com a = raiz(3) e b = raiz(5). > Alguém vai ter que conferir isto aqui). A única > forma de que isso aconteça é > > (i) q=a/sqrt(3) e q=b/sqrt(5); a e b racionais > > > a/sqrt(3) = b/sqrt(5) > a*sqrt(5)=b*sqrt(3) > a/b = sqrt(3)/sqrt(5) <--- irracional. Contradizendo (i). > Este argumento eh invalido. raiz(3)/raiz(5) eh de fato irracional mas nao decorre simplesmente do fato de raiz(3) e raiz(5) serem ambos irracionais. Por exemplo, raiz(18) e raiz(2) sao ambos irracionais, mas raiz(18)/raiz(2) = 3, que eh racional. > É uma demonstração meio trapaceada. Se for válida, é fácil expandir para > quaisquer primos, já que a sqrt() de um primo é sempre irracional. Verdade, mas isso nao foi demonstrado. >Será que dá para demonstrar que a soma de dois irracionais não pode ser >racional - excetuando o 0? Nao, pois isso nao eh verdade. Vide exemplo acima. > Um, suponha, por absurdo, que dois números irracionais podem ter soma > racional diferente de zero. > > (i) x + y = p/q > y=(p-qx)/q > > x-y = x - (p-qx)/q = (qx-p-qx)/q=-p/q > > Mas por (i), x+y = p/q; logo, -x-y = -p/q > > x-y=-x-y ==> x = -x, o que só vale para 0. E nesse caso, y = p/q, ou seja, > racional. Logo, dois números irracionais diferentes de zero não podem ter > soma racional diferente de zero. > > Tá, esta segunda parte também parece um pouco trapaceada. Acho que preciso > de ajuda. Mas se isto for verdade, a demonstração 3) é trivial, e as três > estão respondidas. > A melhor maneira de se resolver os tres problemas de uma vez so eh provando o seguinte resultado mais geral: Seja N um inteiro nao negativo. Entao: raiz(N) eh racional se e somente se N eh um quadrado perfeito. Dem: Se N = 0 ou N = 1, o resultado eh obvio. Assim, suponhamos que N >= 2. Se N eh um q.p., entao existe um inteiro nao negativo M tal que N = M^2. Assim, raiz(N) = raiz(M^2) = M, que eh inteiro e, portanto, racional. Se raiz(N) eh racional, entao existem inteiros positivos P e Q, primos entre si, tais que raiz(N) = P/Q. Isso implica que N = P^2/Q^2, ou seja, P^2 = N * Q^2. Naturalmente temos que N divide P^2. Por outro lado, como mdc(P,Q) = 1, cada primo que divide P^2 terah necessariamente que dividir N. Isso implica que P^2 divide N. Assim, concluimos que P^2 e N sao dois inteiros positivos que se dividem mutuamente. Logo, sao iguais ==> N = P^2 eh um quadrado perfeito. ------ Repare que provamos um pouco mais do que queriamos, a saber, que se N eh um inteiro nao negativo e raiz(N) eh racional, entao raiz(N) eh inteiro. ****** Agora, fica mais facil resolver os problemas. Por exemplo, o segundo sai assim: p e q primos ==> p*q nao eh quadrado perfeito (por que?) ==> raiz(p*q) eh irracional (consequencia do resultado demonstrado acima) Suponha que a = raiz(p) + raiz(q) seja racional. Entao a^2 = p + q + 2*raiz(p*q) eh racional ==> (a^2 - p - q)/2 = raiz(p*q) eh racional ==> contradicao ==> a = raiz(p) + raiz(q) eh irracional. Um abraco, Claudio. > > > ----- Original Message ----- > From: Hely Jr. > To: obm-l@mat.puc-rio.br > Sent: Thursday, April 03, 2003 10:58 PM > Subject: [obm-l] Demonstrações > > > Alguem poderia me ajudar nestas demonstrações > > 1) sabendo que sqrt(3) e sqrt(5) são irracionais, verifique que sqrt(3) + > sqrt(5) é irracional. > > 2) sejam p> 0 e q>0 primos distintos. verifique que sqrt(p) + sqrt(q) é > irracional > > 3) se p e q sào inteiros positivos distintos e pelo menos um dos numeros > sqrt(p) ou sqrt(q) é irracional, então sqrt(p) + sqrt(q) é tb irracional. > > desde ja agradeço > > ========================================================================= > Instruções para entrar na lista, sair da lista e usar a lista em > http://www.mat.puc-rio.br/~nicolau/olimp/obm-l.html > O administrador desta lista é > ========================================================================= > ========================================================================= Instruções para entrar na lista, sair da lista e usar a lista em http://www.mat.puc-rio.br/~nicolau/olimp/obm-l.html O administrador desta lista é ========================================================================= From owner-obm-l@sucuri.mat.puc-rio.br Fri Apr 4 01:40:33 2003 Return-Path: Received: (from majordom@localhost) by sucuri.mat.puc-rio.br (8.9.3/8.9.3) id BAA00714 for obm-l-MTTP; Fri, 4 Apr 2003 01:39:15 -0300 Received: from ivoti.terra.com.br (ivoti.terra.com.br [200.176.3.20]) by sucuri.mat.puc-rio.br (8.9.3/8.9.3) with ESMTP id BAA00710 for ; Fri, 4 Apr 2003 01:39:12 -0300 Received: from una.terra.com.br (una.terra.com.br [200.176.3.32]) by ivoti.terra.com.br (Postfix) with ESMTP id 37A69408505 for ; Fri, 4 Apr 2003 01:38:42 -0300 (BRT) Received: from [200.177.188.177] (dl-nas7-sao-C8B1BCB1.p001.terra.com.br [200.177.188.177]) by una.terra.com.br (Postfix) with ESMTP id EFDDC2F0036 for ; Fri, 4 Apr 2003 01:38:40 -0300 (BRT) User-Agent: Microsoft-Outlook-Express-Macintosh-Edition/5.02.2022 Date: Fri, 04 Apr 2003 01:37:30 -0300 Subject: Re: [obm-l] Livro sobre Nos Complexos From: Claudio Buffara To: Message-ID: In-Reply-To: Mime-version: 1.0 Content-type: multipart/alternative; boundary="MS_Mac_OE_3132265050_1147536_MIME_Part" Sender: owner-obm-l@sucuri.mat.puc-rio.br Precedence: bulk Reply-To: obm-l@mat.puc-rio.br > This message is in MIME format. Since your mail reader does not understand this format, some or all of this message may not be legible. --MS_Mac_OE_3132265050_1147536_MIME_Part Content-type: text/plain; charset="ISO-8859-1" Content-transfer-encoding: quoted-printable Uma pequena correcao no enunciado do problema abaixo: Prove que o produto dos comprimentos dos dois lados e de todas as diagonais que emanam de um mesmo vertice de um n-agono regular inscrito num circulo d= e raio 1 eh igual a n. Ou seja, trata-se do produto dos comprimentos de todos os segmentos que une= m um dado vertice a cada um dos n-1 outros vertices do n-gono regular. on 04.04.03 00:51, Claudio Buffara at claudio.buffara@terra.com.br wrote: on 03.04.03 21:47, Ricardo Prins at ricardoprins@hotmail.com wrote: Claudio, voc=EA pode me recomendar um livro que fale mais profundamente sobre n=FAmeros complexos? Ricardo: Acho que o volume sobre Numeros Complexos da colecao Fundamentos da Matematica Elementar (vol. 6, se nao me engano) contem tudo o que voce precisa saber sobre a teoria elementar dos nos. complexos (veja bem, elementar nao eh sinonimo de facil) A partir dai, a melhor maneira de se aprofundar eh atraves da resolucao de exercicios - livros russos e compilacoes de problemas de olimpiadas sao as melhores fontes. Tem tambem um artigo bem interessante na Eureka, sobre aplicacoes dos nos. complexos em geometria (um exemplo: prove que o produto dos comprimentos de todas as diagonais que emanam de um mesmo vertice de um n-agono regular inscrito num circulo de raio 1 eh igual a n) Na internet voce tambem encontra alguns sites com problemas interessantes, tais como este aqui: http://math.stanford.edu/~vakil/stanfordputnam/02/putnam3.pdf Alem disso, existe o enorme campo da analise complexa (ou calculo com variaveis complexas) onde a referencia basica eh o livro Complex Analysis d= o Lars Ahlfors. Espero que isso ja seja util. Um abraco, Claudio.=20 --MS_Mac_OE_3132265050_1147536_MIME_Part Content-type: text/html; charset="ISO-8859-1" Content-transfer-encoding: quoted-printable Re: [obm-l] Livro sobre Nos Complexos
Uma pequena correcao no enunciado do problema abaix= o:

Prove que o produto dos comprimentos dos dois lados e de todas as diagonais= que emanam de um mesmo vertice de um n-agono regular inscrito num circulo d= e raio 1 eh igual a n.

Ou seja, trata-se do produto dos comprimentos de todos os segmentos que une= m um dado vertice a cada um dos n-1 outros vertices do n-gono regular.


on 04.04.03 00:51, Claudio Buffara at claudio.buffara@terra.com.br wrote:





on 03.04.03 21:47, Ricardo Prins at ricardoprins@hotmail.com wr= ote:

Claudio, voc=EA pode me recomendar um livro que fale mais profund= amente sobre n=FAmeros complexos?

Ricardo:

Acho que o volume sobre Numeros Complexos da colecao Fundamentos da Matemat= ica Elementar (vol. 6, se nao me engano) contem tudo o que voce precisa sabe= r sobre a teoria elementar dos nos. complexos (veja bem, elementar nao eh si= nonimo de facil)

A partir dai, a melhor maneira de se aprofundar eh atraves da resolucao de = exercicios - livros russos e compilacoes de problemas de olimpiadas sao as m= elhores fontes.

Tem tambem um artigo bem interessante na Eureka, sobre aplicacoes dos nos. = complexos em geometria (um exemplo: prove que o produto dos comprimentos de = todas as diagonais que emanam de um mesmo vertice de um n-agono regular insc= rito num circulo de raio 1 eh igual a n)

Na internet voce tambem encontra alguns sites com problemas interessantes, = tais como este aqui:
http://math.stanford.edu/~vakil/stanfordputnam/02/putnam3.pdf

Alem disso, existe o enorme campo da analise complexa (ou calculo com varia= veis complexas) onde a referencia basica eh o livro Complex Analysis do Lars= Ahlfors.

Espero que isso ja seja util.

Um abraco,
Claudio.


--MS_Mac_OE_3132265050_1147536_MIME_Part-- ========================================================================= Instruções para entrar na lista, sair da lista e usar a lista em http://www.mat.puc-rio.br/~nicolau/olimp/obm-l.html O administrador desta lista é ========================================================================= From owner-obm-l@sucuri.mat.puc-rio.br Fri Apr 4 13:13:14 2003 Return-Path: Received: (from majordom@localhost) by sucuri.mat.puc-rio.br (8.9.3/8.9.3) id NAA10998 for obm-l-MTTP; Fri, 4 Apr 2003 13:10:55 -0300 Received: from hotmail.com (f89.law10.hotmail.com [64.4.15.89]) by sucuri.mat.puc-rio.br (8.9.3/8.9.3) with ESMTP id NAA10994 for ; Fri, 4 Apr 2003 13:10:52 -0300 Received: from mail pickup service by hotmail.com with Microsoft SMTPSVC; Fri, 4 Apr 2003 08:10:20 -0800 Received: from 200.147.130.62 by lw10fd.law10.hotmail.msn.com with HTTP; Fri, 04 Apr 2003 16:10:20 GMT X-Originating-IP: [200.147.130.62] X-Originating-Email: [rhel2002@hotmail.com] From: "Rubens Vilhena" To: obm-l@mat.puc-rio.br Subject: [obm-l] Cartas de Ajuda Date: Fri, 04 Apr 2003 16:10:20 +0000 Mime-Version: 1.0 Content-Type: text/plain; charset=iso-8859-1; format=flowed Message-ID: X-OriginalArrivalTime: 04 Apr 2003 16:10:20.0919 (UTC) FILETIME=[B0F53870:01C2FAC4] Sender: owner-obm-l@sucuri.mat.puc-rio.br Precedence: bulk Reply-To: obm-l@mat.puc-rio.br Alô pessoal, estou estudando problemas matemáticos em mágicas e truques com cartas de baralho. Estou tendo problemas em conseguir bibliografia em português em primeiro lugar e depois em espanhol. Em Inglês deve ter muita coisa, mas como o meu não é dos melhores e este é um trabalho preliminar vou deixar como última opção. Agradeço a ajuda []' _________________________________________________________________ MSN Hotmail, o maior webmail do Brasil. http://www.hotmail.com ========================================================================= Instruções para entrar na lista, sair da lista e usar a lista em http://www.mat.puc-rio.br/~nicolau/olimp/obm-l.html O administrador desta lista é ========================================================================= From owner-obm-l@sucuri.mat.puc-rio.br Fri Apr 4 13:37:27 2003 Return-Path: Received: (from majordom@localhost) by sucuri.mat.puc-rio.br (8.9.3/8.9.3) id NAA11528 for obm-l-MTTP; Fri, 4 Apr 2003 13:35:58 -0300 Received: from trex.centroin.com.br (trex.centroin.com.br [200.225.63.134]) by sucuri.mat.puc-rio.br (8.9.3/8.9.3) with ESMTP id NAA11524 for ; Fri, 4 Apr 2003 13:35:55 -0300 Received: from fundoc0hgjgn81 (179234231.rjo.virtua.com.br [200.179.234.231]) (authenticated bits=0) by trex.centroin.com.br (8.12.9/8.12.1) with ESMTP id h34GZZjE001429 for ; Fri, 4 Apr 2003 13:35:44 -0300 (EST) Message-ID: <003601c2fac8$2ef4bef0$e7eab3c8@fundoc0hgjgn81> From: "Diego Navarro" To: References: Subject: [obm-l] =?iso-8859-1?Q?Re:_=5Bobm-l=5D_Re:_=5Bobm-l=5D_Demonstra=E7=F5es?= Date: Fri, 4 Apr 2003 13:35:05 -0300 MIME-Version: 1.0 Content-Type: text/plain; charset="iso-8859-1" Content-Transfer-Encoding: 8bit X-Priority: 3 X-MSMail-Priority: Normal X-Mailer: Microsoft Outlook Express 6.00.2600.0000 X-MimeOLE: Produced By Microsoft MimeOLE V6.00.2600.0000 Sender: owner-obm-l@sucuri.mat.puc-rio.br Precedence: bulk Reply-To: obm-l@mat.puc-rio.br > Tomei a liberdade de inserir alguns comentarios em sua mensagem. Espero que > voce nao se importe. Ei, é claro que eu não me incomodo. Eu estou tentando aprender alguma coisa. Eu achava mesmo que a minha narrativa tinha algum erro: surgiu um resultado geral demais rápido demais. Na verdade, fiquei com vontade de enviar para a lista justamente por desconfiar da minha solução. Em todo caso, a minha escorregada parece ser não ter reduzido os irracionais a alguns números especiais. E se os separássemos? Seja A o conjunto das raízes quadradas de números primos. Parece-me que podemos dizer que: 1) A é subconjunto de I. 2) para quaisquer x,y em A, x+y é irracional. 3) para quaisquer x,y em A, x/y é irracional. O problema é que não tenho tempo para tentar provar essas três proposiçõezinhas. Fazendo as pequenas alterações, parece que a prova da questão 1) vale. Ou não vale? ----- Original Message ----- From: "Claudio Buffara" To: Sent: Friday, April 04, 2003 1:30 AM Subject: Re: [obm-l] Re: [obm-l] Demonstrações > Caro Diego: > > > on 03.04.03 23:53, Diego Navarro at diego@navarro.mus.br wrote: > > > MensagemSuponha que não existem complexos. Na verdade, isso é mais por > > conveniência, já que não sei nada sobre complexos, mas parece razoável que a > > soma de dois números reais seja real. > > > > Um número racional é aquele que pode ser expresso pelo quociente de dois > > inteiros p e q. Suponha, por absurdo, que > > > > sqrt(3)+sqrt(5)=p/q > > q(sqrt(3)+sqrt(5))=p > > q*sqrt(3)+q*sqrt(5)=p > > > > Ora, para que a soma q*sqrt(3)+q*sqrt(5) seja inteira, é preciso que cada > > parcela seja racional > > > (big dúvida: será mesmo? Dois números irracionais > > podem ter soma inteira? > > Sim. Por exemplo: 2 + raiz(2) e 2 - raiz(2) sao ambos irracionais mas tem > soma = 4. > > O problema 1 estah mal formulado, pois da a impressao de que se a e b sao > irracionais entao a+b eh irracional, o que, pelo exemplo acima, nao eh > sempre verdade, apesar de ser verdade com a = raiz(3) e b = raiz(5). > > > > Alguém vai ter que conferir isto aqui). A única > > forma de que isso aconteça é > > > > (i) q=a/sqrt(3) e q=b/sqrt(5); a e b racionais > > > > > > a/sqrt(3) = b/sqrt(5) > > a*sqrt(5)=b*sqrt(3) > > a/b = sqrt(3)/sqrt(5) <--- irracional. Contradizendo (i). > > > Este argumento eh invalido. > raiz(3)/raiz(5) eh de fato irracional mas nao decorre simplesmente do fato > de raiz(3) e raiz(5) serem ambos irracionais. > > Por exemplo, raiz(18) e raiz(2) sao ambos irracionais, mas raiz(18)/raiz(2) > = 3, que eh racional. > > > É uma demonstração meio trapaceada. Se for válida, é fácil expandir para > > quaisquer primos, já que a sqrt() de um primo é sempre irracional. > Verdade, mas isso nao foi demonstrado. > > >Será que dá para demonstrar que a soma de dois irracionais não pode ser > >racional - excetuando o 0? > Nao, pois isso nao eh verdade. Vide exemplo acima. > > > Um, suponha, por absurdo, que dois números irracionais podem ter soma > > racional diferente de zero. > > > > (i) x + y = p/q > > y=(p-qx)/q > > > > x-y = x - (p-qx)/q = (qx-p-qx)/q=-p/q > > > > Mas por (i), x+y = p/q; logo, -x-y = -p/q > > > > x-y=-x-y ==> x = -x, o que só vale para 0. E nesse caso, y = p/q, ou seja, > > racional. Logo, dois números irracionais diferentes de zero não podem ter > > soma racional diferente de zero. > > > > Tá, esta segunda parte também parece um pouco trapaceada. Acho que preciso > > de ajuda. Mas se isto for verdade, a demonstração 3) é trivial, e as três > > estão respondidas. > > > > A melhor maneira de se resolver os tres problemas de uma vez so eh provando > o seguinte resultado mais geral: > > Seja N um inteiro nao negativo. Entao: > raiz(N) eh racional se e somente se N eh um quadrado perfeito. > Dem: > Se N = 0 ou N = 1, o resultado eh obvio. Assim, suponhamos que N >= 2. > > Se N eh um q.p., entao existe um inteiro nao negativo M tal que N = M^2. > Assim, raiz(N) = raiz(M^2) = M, que eh inteiro e, portanto, racional. > > Se raiz(N) eh racional, entao existem inteiros positivos P e Q, primos entre > si, tais que raiz(N) = P/Q. > Isso implica que N = P^2/Q^2, ou seja, P^2 = N * Q^2. > Naturalmente temos que N divide P^2. > Por outro lado, como mdc(P,Q) = 1, cada primo que divide P^2 terah > necessariamente que dividir N. Isso implica que P^2 divide N. > Assim, concluimos que P^2 e N sao dois inteiros positivos que se dividem > mutuamente. Logo, sao iguais ==> N = P^2 eh um quadrado perfeito. > ------ > > Repare que provamos um pouco mais do que queriamos, a saber, que se N eh um > inteiro nao negativo e raiz(N) eh racional, entao raiz(N) eh inteiro. > > ****** > > Agora, fica mais facil resolver os problemas. > > Por exemplo, o segundo sai assim: > > p e q primos ==> > p*q nao eh quadrado perfeito (por que?) ==> > raiz(p*q) eh irracional (consequencia do resultado demonstrado acima) > > Suponha que a = raiz(p) + raiz(q) seja racional. > Entao a^2 = p + q + 2*raiz(p*q) eh racional ==> > (a^2 - p - q)/2 = raiz(p*q) eh racional ==> > contradicao ==> > a = raiz(p) + raiz(q) eh irracional. > > Um abraco, > Claudio. > > > > > > ----- Original Message ----- > > From: Hely Jr. > > To: obm-l@mat.puc-rio.br > > Sent: Thursday, April 03, 2003 10:58 PM > > Subject: [obm-l] Demonstrações > > > > > > Alguem poderia me ajudar nestas demonstrações > > > > 1) sabendo que sqrt(3) e sqrt(5) são irracionais, verifique que sqrt(3) + > > sqrt(5) é irracional. > > > > 2) sejam p> 0 e q>0 primos distintos. verifique que sqrt(p) + sqrt(q) é > > irracional > > > > 3) se p e q sào inteiros positivos distintos e pelo menos um dos numeros > > sqrt(p) ou sqrt(q) é irracional, então sqrt(p) + sqrt(q) é tb irracional. > > > > desde ja agradeço > > > > ========================================================================= > > Instruções para entrar na lista, sair da lista e usar a lista em > > http://www.mat.puc-rio.br/~nicolau/olimp/obm-l.html > > O administrador desta lista é > > ========================================================================= > > > > ========================================================================= > Instruções para entrar na lista, sair da lista e usar a lista em > http://www.mat.puc-rio.br/~nicolau/olimp/obm-l.html > O administrador desta lista é > ========================================================================= > ========================================================================= Instruções para entrar na lista, sair da lista e usar a lista em http://www.mat.puc-rio.br/~nicolau/olimp/obm-l.html O administrador desta lista é ========================================================================= From owner-obm-l@sucuri.mat.puc-rio.br Fri Apr 4 14:53:04 2003 Return-Path: Received: (from majordom@localhost) by sucuri.mat.puc-rio.br (8.9.3/8.9.3) id OAA13256 for obm-l-MTTP; Fri, 4 Apr 2003 14:48:46 -0300 Received: from ns3bind.localdomain ([200.230.34.5]) by sucuri.mat.puc-rio.br (8.9.3/8.9.3) with ESMTP id OAA13252 for ; Fri, 4 Apr 2003 14:48:42 -0300 Received: from servico2 ([200.230.34.224]) by ns3bind.localdomain (8.11.6/X.XX.X) with SMTP id h34Hid016072 for ; Fri, 4 Apr 2003 14:44:40 -0300 Message-ID: <009501c2fad2$72b16620$3300c57d@bovespa.com> From: "=?iso-8859-1?Q?Cl=E1udio_\=28Pr=E1tica\=29?=" To: References: <003601c2fac8$2ef4bef0$e7eab3c8@fundoc0hgjgn81> Subject: [obm-l] =?iso-8859-1?Q?Re:_=5Bobm-l=5D_Re:_=5Bobm-l=5D_Re:_=5Bobm-l=5D_Demonstr?= =?iso-8859-1?Q?a=E7=F5es?= Date: Fri, 4 Apr 2003 14:48:39 -0300 MIME-Version: 1.0 Content-Type: text/plain; charset="iso-8859-1" Content-Transfer-Encoding: 8bit X-Priority: 3 X-MSMail-Priority: Normal X-Mailer: Microsoft Outlook Express 5.50.4920.2300 X-MimeOLE: Produced By Microsoft MimeOLE V5.50.4920.2300 Sender: owner-obm-l@sucuri.mat.puc-rio.br Precedence: bulk Reply-To: obm-l@mat.puc-rio.br Caro Diego: Sobre a sua afirmativa: > Seja A o conjunto das raízes quadradas de números primos. Parece-me que > podemos dizer que: > > 1) A é subconjunto de I. > 2) para quaisquer x,y em A, x+y é irracional. > 3) para quaisquer x,y em A, x/y é irracional. (1) e (2) são verdadeiras. (3) será verdadeira <==> x <> y. Repare que este resultado é um corolário do resultado mais geral que eu provei no meu e-mail anterior: "Seja N um inteiro nao negativo. Entao: raiz(N) eh racional se e somente se N eh um quadrado perfeito." uma vez que se p e q são primos distintos então nenhum dos números p, q, p*q será quadrado perfeito. Isso pode ser generalizado para raízes n-ésimas: "Seja n um inteiro >= 2 e M um inteiro não negativo. Então: M^(1/n) é racional se e somente se M é igual à n-ésima potência de algum inteiro." Tente provar isso quando tiver um tempo. É um bom exercício. Um abraço, Claudio. ========================================================================= Instruções para entrar na lista, sair da lista e usar a lista em http://www.mat.puc-rio.br/~nicolau/olimp/obm-l.html O administrador desta lista é ========================================================================= From owner-obm-l@sucuri.mat.puc-rio.br Fri Apr 4 15:37:17 2003 Return-Path: Received: (from majordom@localhost) by sucuri.mat.puc-rio.br (8.9.3/8.9.3) id PAA14567 for obm-l-MTTP; Fri, 4 Apr 2003 15:35:17 -0300 Received: from hotmail.com (f84.law8.hotmail.com [216.33.241.84]) by sucuri.mat.puc-rio.br (8.9.3/8.9.3) with ESMTP id PAA14563 for ; Fri, 4 Apr 2003 15:35:13 -0300 Received: from mail pickup service by hotmail.com with Microsoft SMTPSVC; Fri, 4 Apr 2003 10:34:42 -0800 Received: from 200.222.105.178 by lw8fd.law8.hotmail.msn.com with HTTP; Fri, 04 Apr 2003 18:34:42 GMT X-Originating-IP: [200.222.105.178] X-Originating-Email: [osneto@hotmail.com] From: "Antonio Neto" To: obm-l@mat.puc-rio.br Subject: Re: [obm-l] AJUDA Date: Fri, 04 Apr 2003 18:34:42 +0000 Mime-Version: 1.0 Content-Type: text/plain; format=flowed Message-ID: X-OriginalArrivalTime: 04 Apr 2003 18:34:42.0644 (UTC) FILETIME=[DBBF9140:01C2FAD8] Sender: owner-obm-l@sucuri.mat.puc-rio.br Precedence: bulk Reply-To: obm-l@mat.puc-rio.br Nao sei se estou atrasado e alguem ja respondeu, mas soh li hoje. O numero eh 142857, o periodo de 1/7. Isto ocorre sempre que 1/p, pprimo, tem um periodo com p-1 algarismos. Tais numeros sao chamados ciclicos. Posso mandar referencis bibliograficas, mas nao agora, estou longe da minha bisbilhoteca. Abracos, olavo. >From: "A. C. Morgado" >Reply-To: obm-l@mat.puc-rio.br >To: obm-l@mat.puc-rio.br >Subject: Re: [obm-l] AJUDA >Date: Mon, 31 Mar 2003 22:14:36 -0300 > >1) >a=1 (se a>1, 6N nao poderia ter a mesma quantidade de algarismos de N). >3N = bcdef1 (o 1=a so pode aparecer na ultima posiçao no 3N, pois o 5N nao >pode terminar em 1 e os outros sao pares). Logo, N termina em 7, f=7. >2N termina em 4, 4N termina em 8 e 6N termina em 2, 5N termina em 5. >Os algarismos sao 1(inicial), 7(final), 4, 8, 2 e 5 (nao sei em que >posiçoes) >S = 27 >Se o problema tem soluçao, a soluçao eh 27. > >Daniel Pini wrote: > >>OLá, alguem poderia me ajudar? >>1-O número de seis algarismos N=abcdef é tal que quando multipicamos por >>2, 3, 4, 5, 6 obtemos números com os mesmos algarismos permutados >>ciclicamente. A soma dos alg. de N é: R:27 > > _________________________________________________________________ The new MSN 8: advanced junk mail protection and 2 months FREE* http://join.msn.com/?page=features/junkmail ========================================================================= Instruções para entrar na lista, sair da lista e usar a lista em http://www.mat.puc-rio.br/~nicolau/olimp/obm-l.html O administrador desta lista é ========================================================================= From owner-obm-l@sucuri.mat.puc-rio.br Fri Apr 4 23:23:31 2003 Return-Path: Received: (from majordom@localhost) by sucuri.mat.puc-rio.br (8.9.3/8.9.3) id XAA21717 for obm-l-MTTP; Fri, 4 Apr 2003 23:21:43 -0300 Received: from hotmail.com (f113.law9.hotmail.com [64.4.9.113]) by sucuri.mat.puc-rio.br (8.9.3/8.9.3) with ESMTP id XAA21713 for ; Fri, 4 Apr 2003 23:21:40 -0300 Received: from mail pickup service by hotmail.com with Microsoft SMTPSVC; Fri, 4 Apr 2003 18:21:08 -0800 Received: from 200.222.172.43 by lw9fd.law9.hotmail.msn.com with HTTP; Sat, 05 Apr 2003 02:21:08 GMT X-Originating-IP: [200.222.172.43] X-Originating-Email: [ricardoprins@hotmail.com] From: "Ricardo Prins" To: obm-l@mat.puc-rio.br Subject: Re: [obm-l] AJUDA Date: Sat, 05 Apr 2003 02:21:08 +0000 Mime-Version: 1.0 Content-Type: text/html Message-ID: X-OriginalArrivalTime: 05 Apr 2003 02:21:08.0882 (UTC) FILETIME=[04D89720:01C2FB1A] Sender: owner-obm-l@sucuri.mat.puc-rio.br Precedence: bulk Reply-To: obm-l@mat.puc-rio.br


Help STOP SPAM with the new MSN 8 and get 2 months FREE* ========================================================================= Instruções para entrar na lista, sair da lista e usar a lista em http://www.mat.puc-rio.br/~nicolau/olimp/obm-l.html O administrador desta lista é ========================================================================= From owner-obm-l@sucuri.mat.puc-rio.br Fri Apr 4 23:28:45 2003 Return-Path: Received: (from majordom@localhost) by sucuri.mat.puc-rio.br (8.9.3/8.9.3) id XAA21828 for obm-l-MTTP; Fri, 4 Apr 2003 23:27:28 -0300 Received: from hotmail.com (f14.law9.hotmail.com [64.4.9.14]) by sucuri.mat.puc-rio.br (8.9.3/8.9.3) with ESMTP id XAA21824 for ; Fri, 4 Apr 2003 23:27:24 -0300 Received: from mail pickup service by hotmail.com with Microsoft SMTPSVC; Fri, 4 Apr 2003 18:26:53 -0800 Received: from 200.222.172.43 by lw9fd.law9.hotmail.msn.com with HTTP; Sat, 05 Apr 2003 02:26:53 GMT X-Originating-IP: [200.222.172.43] X-Originating-Email: [ricardoprins@hotmail.com] From: "Ricardo Prins" To: obm-l@mat.puc-rio.br Subject: Re: [obm-l] Cartas de Ajuda Date: Sat, 05 Apr 2003 02:26:53 +0000 Mime-Version: 1.0 Content-Type: text/html Message-ID: X-OriginalArrivalTime: 05 Apr 2003 02:26:53.0640 (UTC) FILETIME=[D2567C80:01C2FB1A] Sender: owner-obm-l@sucuri.mat.puc-rio.br Precedence: bulk Reply-To: obm-l@mat.puc-rio.br

hmmm acho que na internet você encontra material em português sobre o truque de cartas de Peirce...é o único que eu conheço. quanto à livros, talvez uma busca em sites de venda como submarino.com.br possa ser melhor... acho que não há um sítio de catálogos de livros no brasil...hahaha

>From: "Rubens Vilhena"
>Reply-To: obm-l@mat.puc-rio.br
>To: obm-l@mat.puc-rio.br
>Subject: [obm-l] Cartas de Ajuda
>Date: Fri, 04 Apr 2003 16:10:20 +0000
>
>
>
>Alô pessoal, estou estudando problemas matemáticos em mágicas e
>truques com cartas de baralho. Estou tendo problemas em conseguir
>bibliografia em português em primeiro lugar e depois em espanhol. Em
>Inglês deve ter muita coisa, mas como o meu não é dos melhores e
>este é um trabalho preliminar vou deixar como última opção.
>Agradeço a ajuda
>
>[]'
>
>_________________________________________________________________
>MSN Hotmail, o maior webmail do Brasil. http://www.hotmail.com
>
>=========================================================================
>Instruções para entrar na lista, sair da lista e usar a lista em
>http://www.mat.puc-rio.br/~nicolau/olimp/obm-l.html
>O administrador desta lista é
>=========================================================================


Add photos to your messages with MSN 8. Get 2 months FREE*. ========================================================================= Instruções para entrar na lista, sair da lista e usar a lista em http://www.mat.puc-rio.br/~nicolau/olimp/obm-l.html O administrador desta lista é ========================================================================= From owner-obm-l@sucuri.mat.puc-rio.br Sat Apr 5 11:55:54 2003 Return-Path: Received: (from majordom@localhost) by sucuri.mat.puc-rio.br (8.9.3/8.9.3) id LAA29443 for obm-l-MTTP; Sat, 5 Apr 2003 11:52:52 -0300 Received: from silva5.uol.com.br (silva5.uol.com.br [200.221.29.52]) by sucuri.mat.puc-rio.br (8.9.3/8.9.3) with ESMTP id LAA29439 for ; Sat, 5 Apr 2003 11:52:49 -0300 Received: from u2z7z2 ([200.158.144.229]) by silva5.uol.com.br (8.9.1/8.9.1) with ESMTP id LAA10497 for ; Sat, 5 Apr 2003 11:52:19 -0300 (BRT) Message-ID: <000401c2fb82$dc81a780$2101a8c0@u2z7z2> From: "Wagner" To: References: <005d01c2fa4d$a245e2b0$4f9093c8@jr> Subject: [obm-l] =?iso-8859-1?Q?Re:_=5Bobm-l=5D_Demonstra=E7=F5es?= Date: Fri, 4 Apr 2003 22:24:46 -0300 Organization: Wagner MIME-Version: 1.0 Content-Type: multipart/alternative; boundary="----=_NextPart_000_0058_01C2FAF8.FFC48DA0" X-Priority: 3 X-MSMail-Priority: Normal X-Mailer: Microsoft Outlook Express 5.50.4133.2400 X-MimeOLE: Produced By Microsoft MimeOLE V5.50.4133.2400 Sender: owner-obm-l@sucuri.mat.puc-rio.br Precedence: bulk Reply-To: obm-l@mat.puc-rio.br This is a multi-part message in MIME format. ------=_NextPart_000_0058_01C2FAF8.FFC48DA0 Content-Type: text/plain; charset="iso-8859-1" Content-Transfer-Encoding: quoted-printable MensagemOi para todos! TEOREMA: Se a =E9 um n=BA natural que n=E3o =E9 um quadrado perfeito, = sqrt(a) =E9 irracional PROVA: Suponha por absurdo que sqrt(a) =E9 racional. Logo sqrt(a) pode = ser escrito na forma p/q , mdc(p,q)=3D1 Logo existe solu=E7=E3o racional para p e q tais que mdc(p,q)=3D1 para = a=3Dp^2/q^2 =3D> a.q^2 =3D p^2 . a,p,q s=E3o inteiros. Logo p =E9 divis=EDvel por a. Logo p =3D a.r para algum valor inteiro de = r . Logo a^2.r^2 =3D a.q^2 =3D> q^2 =3D a.r^2 . a,q,r s=E3o inteiros .Segue que q =E9 divis=EDvel por a. Como a n=E3o =E9 quadrado = perfeito, a>1 . Logo mdc(p,q)>1 . Absurdo ! PROPRIEDADE: Se a =E9 irracional, sqrt(a) tamb=E9m =E9 irracional Usando esses teoremas acima fica f=E1cil provar os 2 primeiros 1)Eleve ( sqrt(3) + sqrt(5)) ao quadrado, voc=EA ter=E1 8 + 2sqrt(15) = que =E9 irracional pois sqrt(15) =E9 irracional, uma vez que 15 n=E3o =E9 quadrado perfeito, logo sqrt(8 + 2sqrt(15)) =3D sqrt(3) + = sqrt(5) =E9 irracional. 2)(sqrt(p) + sqrt(q))^2 =3D p+q + 2sqrt(p.q) . Como p e q s=E3o primos = distintos p.q n=E3o =E9 quadrado perfeito, logo sqrt(p) + sqrt(q) =E9 = irracional=20 Andr=E9 T. ----- Original Message -----=20 From: Hely Jr.=20 To: obm-l@mat.puc-rio.br=20 Sent: Thursday, April 03, 2003 10:58 PM Subject: [obm-l] Demonstra=E7=F5es Alguem poderia me ajudar nestas demonstra=E7=F5es 1) sabendo que sqrt(3) e sqrt(5) s=E3o irracionais, verifique que = sqrt(3) + sqrt(5) =E9 irracional. 2) sejam p> 0 e q>0 primos distintos. verifique que sqrt(p) + sqrt(q) = =E9 irracional 3) se p e q s=E0o inteiros positivos distintos e pelo menos um dos = numeros sqrt(p) ou sqrt(q) =E9 irracional, ent=E3o sqrt(p) + sqrt(q) =E9 = tb irracional. desde ja agrade=E7o ------=_NextPart_000_0058_01C2FAF8.FFC48DA0 Content-Type: text/html; charset="iso-8859-1" Content-Transfer-Encoding: quoted-printable Mensagem
Oi para todos!
 
TEOREMA: Se a =E9 um n=BA = natural que n=E3o =E9=20 um quadrado perfeito, sqrt(a) =E9 irracional
PROVA: Suponha por absurdo que sqrt(a) = =E9 racional.=20 Logo sqrt(a) pode ser escrito na forma p/q , mdc(p,q)=3D1
Logo existe solu=E7=E3o = racional para p e q tais=20 que mdc(p,q)=3D1 para a=3Dp^2/q^2 =3D> a.q^2 =3D p^2 . a,p,q = s=E3o=20 inteiros.
Logo p =E9 divis=EDvel por a. Logo = p =3D a.r para=20 algum valor inteiro de r . Logo a^2.r^2 =3D a.q^2 =3D> q^2 =3D a.r^2 = . a,q,r=20 s=E3o
inteiros .Segue que q =E9 divis=EDvel = por a. Como a n=E3o=20 =E9 quadrado perfeito, a>1 . Logo mdc(p,q)>1 . Absurdo = !
PROPRIEDADE: Se a =E9 irracional, = sqrt(a) tamb=E9m =E9=20 irracional
 
Usando esses teoremas acima fica = f=E1cil provar=20 os 2 primeiros
 
1)Eleve ( sqrt(3) + sqrt(5)) ao = quadrado, voc=EA ter=E1=20 8 + 2sqrt(15) que =E9 irracional pois sqrt(15) =E9 irracional, uma vez=20 que
15 n=E3o =E9 quadrado perfeito, logo = sqrt(8 +=20 2sqrt(15)) =3D sqrt(3) + sqrt(5) =E9 irracional.
2)(sqrt(p) + sqrt(q))^2 =3D p+q + = 2sqrt(p.q) . Como p=20 e q s=E3o primos distintos p.q n=E3o =E9 quadrado perfeito, logo sqrt(p) = +=20 sqrt(q) =E9 irracional 
 
Andr=E9 T.
 
 
----- Original Message -----
From:=20 Hely Jr.=20
Sent: Thursday, April 03, 2003 = 10:58=20 PM
Subject: [obm-l] = Demonstra=E7=F5es

Alguem poderia me=20 ajudar nestas demonstra=E7=F5es
 
1) = sabendo que=20 sqrt(3) e sqrt(5) s=E3o irracionais, verifique que sqrt(3) + sqrt(5) = =E9=20 irracional.
 
2) = sejam p> 0 e=20 q>0 primos distintos. verifique que sqrt(p) + sqrt(q) =E9=20 irracional
 
3) = se p e q s=E0o=20 inteiros positivos distintos e pelo menos um dos numeros sqrt(p) ou = sqrt(q) =E9=20 irracional, ent=E3o sqrt(p) + sqrt(q) =E9 tb = irracional.
 
desde ja=20 agrade=E7o
------=_NextPart_000_0058_01C2FAF8.FFC48DA0-- ========================================================================= Instruções para entrar na lista, sair da lista e usar a lista em http://www.mat.puc-rio.br/~nicolau/olimp/obm-l.html O administrador desta lista é ========================================================================= From owner-obm-l@sucuri.mat.puc-rio.br Sat Apr 5 17:34:29 2003 Return-Path: Received: (from majordom@localhost) by sucuri.mat.puc-rio.br (8.9.3/8.9.3) id RAA00909 for obm-l-MTTP; Sat, 5 Apr 2003 17:32:08 -0300 Received: from artemis.opendf.com.br (artemis.opengate.com.br [200.181.71.15]) by sucuri.mat.puc-rio.br (8.9.3/8.9.3) with ESMTP id RAA00905 for ; Sat, 5 Apr 2003 17:32:05 -0300 Received: from localhost (localhost [127.0.0.1]) by artemis.opendf.com.br (Postfix) with ESMTP id 108402BEB4 for ; Sat, 5 Apr 2003 17:31:34 -0300 (BRT) Received: from artemis.opendf.com.br ([127.0.0.1]) by localhost (artemis.opengate.com.br [127.0.0.1:10024]) (amavisd-new) with ESMTP id 00534-03 for ; Sat, 5 Apr 2003 17:31:31 -0300 (BRT) Received: from computer (200-181-089-125.bsace7001.dsl.brasiltelecom.net.br [200.181.89.125]) by artemis.opendf.com.br (Postfix) with ESMTP id 069AE2BEB0 for ; Sat, 5 Apr 2003 17:01:27 -0300 (BRT) From: "Artur Costa Steiner" To: Subject: [obm-l] =?iso-8859-1?Q?RE:_=5Bobm-l=5D_Re:_=5Bobm-l=5D_Demonstra=E7=F5es?= Date: Sat, 5 Apr 2003 17:01:23 -0300 Organization: Steiner Consultoria LTDA Message-ID: <002a01c2fbae$24d0f330$9865fea9@computer> MIME-Version: 1.0 Content-Type: text/plain; charset="iso-8859-1" X-Priority: 3 (Normal) X-MSMail-Priority: Normal X-Mailer: Microsoft Outlook, Build 10.0.2627 In-Reply-To: <000401c2fb82$dc81a780$2101a8c0@u2z7z2> X-MimeOLE: Produced By Microsoft MimeOLE V6.00.2800.1106 Importance: Normal X-Virus-Scanned: by amavisd-new Content-Transfer-Encoding: 8bit X-MIME-Autoconverted: from quoted-printable to 8bit by sucuri.mat.puc-rio.br id RAA00906 Sender: owner-obm-l@sucuri.mat.puc-rio.br Precedence: bulk Reply-To: obm-l@mat.puc-rio.br Subject: [obm-l] Demonstrações Alguem poderia me ajudar nestas demonstrações   1) sabendo que sqrt(3) e sqrt(5) são irracionais, verifique que sqrt(3) + sqrt(5) é irracional.   2) sejam p> 0 e q>0 primos distintos. verifique que sqrt(p) + sqrt(q) é irracional   3) se p e q sào inteiros positivos distintos e pelo menos um dos numeros sqrt(p) ou sqrt(q) é irracional, então sqrt(p) + sqrt(q) é tb irracional.   desde ja agradeço Oi a todos! Os itens 1 e 2 já foram demonstrados por vários colegas. O 3 , na realidade, vale se p e q forem racionais, ainda que não necessariamente inteiros. E não é preciso que sejam distintos. Seja S = sqrt(p) + sqrt(q) e suponhamos, sem perda de generalidade, que sqrt(q) seja irracional. Segue-se que sqrt(p) = S - sqrt(q) => p = S^2 - 2S sqrt(q) + q. Por hipótese, q é racional. Se S for racional, então s^2 é racional e 2S sqrt(q) é irracional, pois, por hipótese, sqrt(q) é irracional. Logo, = S^2 - 2S sqrt(q) + q é irracional, o que contraria a hipótese de que p é racional. Disso concluimos que S tem necessariamente que ser irracional. Dito de outra forma: se p e q forem racionais, então sqrt(p) + sqrt(q) é racional se, e somente se, sqrt(p) e sqrt(q) também o forem. Artur ========================================================================= Instruções para entrar na lista, sair da lista e usar a lista em http://www.mat.puc-rio.br/~nicolau/olimp/obm-l.html O administrador desta lista é ========================================================================= From owner-obm-l@sucuri.mat.puc-rio.br Sat Apr 5 18:22:09 2003 Return-Path: Received: (from majordom@localhost) by sucuri.mat.puc-rio.br (8.9.3/8.9.3) id SAA01769 for obm-l-MTTP; Sat, 5 Apr 2003 18:20:49 -0300 Received: from lampiao.digi.com.br (lampiao.digi.com.br [200.241.100.60]) by sucuri.mat.puc-rio.br (8.9.3/8.9.3) with ESMTP id SAA01764 for ; Sat, 5 Apr 2003 18:20:45 -0300 Received: from p8c7y1fe9gyrelp.digi.com.br (host187.d.digizap.com.br [200.249.6.187]) by lampiao.digi.com.br (8.11.6/8.11.6) with ESMTP id h35LHel07678 for ; Sat, 5 Apr 2003 18:17:57 -0300 Message-Id: <5.2.0.9.0.20030405180556.00b17a90@mail.digi.com.br> X-Sender: benedito@mail.digi.com.br X-Mailer: QUALCOMM Windows Eudora Version 5.2.0.9 Date: Sat, 05 Apr 2003 18:17:28 -0300 To: obm-l@mat.puc-rio.br From: benedito Subject: Re: [obm-l] Cartas de Ajuda In-Reply-To: Mime-Version: 1.0 Content-Type: text/plain; charset="iso-8859-1"; format=flowed X-MailScanner: Found to be clean Content-Transfer-Encoding: 8bit X-MIME-Autoconverted: from quoted-printable to 8bit by sucuri.mat.puc-rio.br id SAA01765 Sender: owner-obm-l@sucuri.mat.puc-rio.br Precedence: bulk Reply-To: obm-l@mat.puc-rio.br Prezado Rubens, No livro: "Ah, Descobri !" de Martin Gardner, Editora Gradiva (Portugal) (pag. 116 - 118), tem um truque interessantíssimo, para o "mágico" usar cartas. Na verdade, trata-se de uma boa motivação para o ensino do Teorema Chinês dos Restos. Tenho feito este truque para meus estudantes e sinto que eles ficam altamente motivado. Um colega de Departamento, melhorou o truque consideravelmente, a ponto de você dizer um número entre 1 e 27 e a carta escolhida estar na posição do número que você escolheu (e não tornou público). Veja o truque e você vai entender do que estou falando. Recomendo que você veja os livros do Martin Gardner, que tem muita coisa escrita de forma a tornar o interesse sempre em alta. Boa sorte! Benedito Freire At 16:10 4/4/2003 +0000, you wrote: >Alô pessoal, estou estudando problemas matemáticos em mágicas e truques >com cartas de baralho. Estou tendo problemas em conseguir bibliografia em >português em primeiro lugar e depois em espanhol. Em Inglês deve ter muita >coisa, mas como o meu não é dos melhores e este é um trabalho preliminar >vou deixar como última opção. >Agradeço a ajuda > >[]' > >_________________________________________________________________ >MSN Hotmail, o maior webmail do Brasil. http://www.hotmail.com > >========================================================================= >Instruções para entrar na lista, sair da lista e usar a lista em >http://www.mat.puc-rio.br/~nicolau/olimp/obm-l.html >O administrador desta lista é >========================================================================= ========================================================================= Instruções para entrar na lista, sair da lista e usar a lista em http://www.mat.puc-rio.br/~nicolau/olimp/obm-l.html O administrador desta lista é ========================================================================= From owner-obm-l@sucuri.mat.puc-rio.br Sat Apr 5 18:36:11 2003 Return-Path: Received: (from majordom@localhost) by sucuri.mat.puc-rio.br (8.9.3/8.9.3) id SAA02164 for obm-l-MTTP; Sat, 5 Apr 2003 18:34:24 -0300 Received: from lampiao.digi.com.br (lampiao.digi.com.br [200.241.100.60]) by sucuri.mat.puc-rio.br (8.9.3/8.9.3) with ESMTP id SAA02159 for ; Sat, 5 Apr 2003 18:34:21 -0300 Received: from p8c7y1fe9gyrelp.digi.com.br (host187.d.digizap.com.br [200.249.6.187]) by lampiao.digi.com.br (8.11.6/8.11.6) with ESMTP id h35LU3l09889 for ; Sat, 5 Apr 2003 18:30:13 -0300 Message-Id: <5.2.0.9.0.20030405181929.047b0eb0@mail.digi.com.br> X-Sender: benedito@mail.digi.com.br X-Mailer: QUALCOMM Windows Eudora Version 5.2.0.9 Date: Sat, 05 Apr 2003 18:30:07 -0300 To: obm-l@mat.puc-rio.br From: benedito Subject: [obm-l] Problema da Tesoura(O Retorno???) e sqrt(pi) In-Reply-To: <20030331165244.C5866@sucuri.mat.puc-rio.br> References: <3E8869D000000863@www.zipmail.com.br> <3E8869D000000863@www.zipmail.com.br> Mime-Version: 1.0 Content-Type: text/plain; charset="iso-8859-1"; format=flowed X-MailScanner: Found to be clean Content-Transfer-Encoding: 8bit X-MIME-Autoconverted: from quoted-printable to 8bit by sucuri.mat.puc-rio.br id SAA02160 Sender: owner-obm-l@sucuri.mat.puc-rio.br Precedence: bulk Reply-To: obm-l@mat.puc-rio.br A questão seguinte foi a de número 2 da prova para SENIOR, Autumn 1996 (O Level) do Tournament of Towns: "Can a paper circle be cut into pieces and then rearranged into a square of the same area, if only a finite number of cuts is allowed and they must be along segments of straight lines or circular arcs?" Aproveitando, aconselho a leitura no livro "Tournament of Towns 1993-1997". An Australian Mathematics Trust Publication. Este livro tem uma coleção maravilhosa de problemas. Aliás, qualquer livro do Australian Mathematics Trust Publication pertence a categoria de excelente. Para maiores informações, entre na nossa página www.obm.org.br e clique links e, em seguida, Austrália. Benedito Freire At 16:52 31/3/2003 -0300, you wrote: >On Mon, Mar 31, 2003 at 03:07:34PM -0300, >peterdirichlet1985@zipmail.com.br wrote: > > Turma,alguem sabe demonstrar esse teorema estranho que me apareceu na > Semana > > Olimpica? > > "Mostre que e possivel recortar um circulo em varios mas finitos pedaços > > e rearranjar os pedaços sem falhas de modo a formar um quadrado.Cada corte > > deve ser ou um arco de circulo ou um segmento de reta." > > Que tal se esse fosse pra Eureka!? > >Isto me cheira ao problema da quadratura do círculo, versão século XX. >O teorema (que não é fácil) é que é possível cortar um quadrado >em um número finito de peças e juntá-las para formar um disco redondo >de mesma área. Mas as peças são muito complicadas, não é possível >resolver o problema se os cortes forem limitados a curvas bem comportadas. > >Isso parece o paradoxo de Banach-Tarski: é possível decompor uma bola >em um número finito de pedaços e juntá-los para formar duas bolas, >cada uma igual à bola original. O teorema mais geral é que se A e B >são dois subconjuntos de R^3 limitados e de interior nào vazio então >é possível recortar A em um número finito de pedaços e juntá-los >para montar B. Note em particular que não existe preservação de volume; >em R^2 existe, não é possível recortar uma bola pequena para montar >uma bola grande. > >[]s, N. >========================================================================= >Instruções para entrar na lista, sair da lista e usar a lista em >http://www.mat.puc-rio.br/~nicolau/olimp/obm-l.html >O administrador desta lista é >========================================================================= ========================================================================= Instruções para entrar na lista, sair da lista e usar a lista em http://www.mat.puc-rio.br/~nicolau/olimp/obm-l.html O administrador desta lista é ========================================================================= From owner-obm-l@sucuri.mat.puc-rio.br Sat Apr 5 18:43:37 2003 Return-Path: Received: (from majordom@localhost) by sucuri.mat.puc-rio.br (8.9.3/8.9.3) id SAA02370 for obm-l-MTTP; Sat, 5 Apr 2003 18:42:19 -0300 Received: from lampiao.digi.com.br (lampiao.digi.com.br [200.241.100.60]) by sucuri.mat.puc-rio.br (8.9.3/8.9.3) with ESMTP id SAA02366 for ; Sat, 5 Apr 2003 18:42:16 -0300 Received: from p8c7y1fe9gyrelp.digi.com.br (host187.d.digizap.com.br [200.249.6.187]) by lampiao.digi.com.br (8.11.6/8.11.6) with ESMTP id h35Le4l11627 for ; Sat, 5 Apr 2003 18:40:10 -0300 Message-Id: <5.2.0.9.0.20030405183058.00b98680@mail.digi.com.br> X-Sender: benedito@mail.digi.com.br X-Mailer: QUALCOMM Windows Eudora Version 5.2.0.9 Date: Sat, 05 Apr 2003 18:40:16 -0300 To: obm-l@mat.puc-rio.br From: benedito Subject: Re: [obm-l] O problema do andarilho In-Reply-To: <001e01c2f950$f2f900a0$019da8c0@henrique> Mime-Version: 1.0 Content-Type: text/plain; charset="iso-8859-1"; format=flowed X-MailScanner: Found to be clean Content-Transfer-Encoding: 8bit X-MIME-Autoconverted: from quoted-printable to 8bit by sucuri.mat.puc-rio.br id SAA02367 Sender: owner-obm-l@sucuri.mat.puc-rio.br Precedence: bulk Reply-To: obm-l@mat.puc-rio.br Prezado Henrique, Um problema semelhante aparece no (excelente) livro "The Art and Craft of Problem Solving" de Paul Zeitz. Wiley. 1999. É o problema 1.3.1, pag 9, a solução (engenhosa) aparece na página 19. Suponha que no instante em que o andarilho começa a descer, um outro andarilho, nas mesmas condições começa a subir. O ponto de encontro dos dois é o tempo e o lugar que você queria! Benedito Freire At 16:49 2/4/2003 -0300, you wrote: >Alguem poderia me ajudar com esse? > > >Uma trilha vai da base de uma montanha até o topo. Um andarilho começa a >subir a trilha às 6 horas da manhã e chega ao topo às 6 horas da tarde do >mesmo dia. Durante o percurso ele pode parar, voltar atrás, correr, fazer o >que quiser desde que chegue ao topo às 6 horas da tarde do mesmo dia. >Na manhã seguinte ele começa a descer a trilha às 6 horas da manhã do modo >como ele quiser e chega à base exatamente às 6 horas da tarde do mesmo dia. >Prove que existe pelo menos um lugar na trilha pelo qual ele passa na mesma >hora de cada dia. > >Grato, > >Henrique. > >========================================================================= >Instruções para entrar na lista, sair da lista e usar a lista em >http://www.mat.puc-rio.br/~nicolau/olimp/obm-l.html >O administrador desta lista é >========================================================================= ========================================================================= Instruções para entrar na lista, sair da lista e usar a lista em http://www.mat.puc-rio.br/~nicolau/olimp/obm-l.html O administrador desta lista é ========================================================================= From owner-obm-l@sucuri.mat.puc-rio.br Sat Apr 5 19:18:40 2003 Return-Path: Received: (from majordom@localhost) by sucuri.mat.puc-rio.br (8.9.3/8.9.3) id TAA03974 for obm-l-MTTP; Sat, 5 Apr 2003 19:17:16 -0300 Received: from web20508.mail.yahoo.com (web20508.mail.yahoo.com [216.136.226.143]) by sucuri.mat.puc-rio.br (8.9.3/8.9.3) with SMTP id TAA03969 for ; Sat, 5 Apr 2003 19:17:13 -0300 Message-ID: <20030405221641.46397.qmail@web20508.mail.yahoo.com> Received: from [200.100.76.69] by web20508.mail.yahoo.com via HTTP; Sat, 05 Apr 2003 19:16:41 ART Date: Sat, 5 Apr 2003 19:16:41 -0300 (ART) From: =?iso-8859-1?q?Jhonata=20Emerick?= Subject: [obm-l] Grafos To: obm-l@mat.puc-rio.br MIME-Version: 1.0 Content-Type: multipart/alternative; boundary="0-1604562023-1049581001=:45541" Content-Transfer-Encoding: 8bit Sender: owner-obm-l@sucuri.mat.puc-rio.br Precedence: bulk Reply-To: obm-l@mat.puc-rio.br --0-1604562023-1049581001=:45541 Content-Type: text/plain; charset=iso-8859-1 Content-Transfer-Encoding: 8bit Olá pessoal da lista, quem puder me ajudar com esse problema.: Numa matriz A,nxn, em que os elementos aij pode corresponder exclusivamente a 1 ou a 0, sendo que 1 indica ligação e 0 indica falta dessa(ligação) entre os vertices i e j, como eu faço para descobrir o menor caminho entre a e b, sendo estes pares ordenados de inteiros positivos, a<=n e b<=n. --------------------------------- Yahoo! Mail O melhor e-mail gratuito da internet: 6MB de espaço, antivírus, acesso POP3, filtro contra spam. --0-1604562023-1049581001=:45541 Content-Type: text/html; charset=iso-8859-1 Content-Transfer-Encoding: 8bit

Olá pessoal da lista, quem puder me ajudar com esse problema.:

Numa matriz A,nxn, em que os elementos aij pode corresponder exclusivamente a 1 ou a 0, sendo que 1 indica ligação e 0 indica falta dessa(ligação) entre os vertices i e j, como eu faço para descobrir o menor caminho entre a e b, sendo estes pares ordenados de inteiros positivos, a<=n e b<=n.

 



Yahoo! Mail
O melhor e-mail gratuito da internet: 6MB de espaço, antivírus, acesso POP3, filtro contra spam. --0-1604562023-1049581001=:45541-- ========================================================================= Instruções para entrar na lista, sair da lista e usar a lista em http://www.mat.puc-rio.br/~nicolau/olimp/obm-l.html O administrador desta lista é ========================================================================= From owner-obm-l@sucuri.mat.puc-rio.br Sat Apr 5 19:33:49 2003 Return-Path: Received: (from majordom@localhost) by sucuri.mat.puc-rio.br (8.9.3/8.9.3) id TAA04324 for obm-l-MTTP; Sat, 5 Apr 2003 19:32:17 -0300 Received: from imo-m06.mx.aol.com (imo-m06.mx.aol.com [64.12.136.161]) by sucuri.mat.puc-rio.br (8.9.3/8.9.3) with ESMTP id TAA04320 for ; Sat, 5 Apr 2003 19:32:14 -0300 From: Faelccmm@aol.com Received: from Faelccmm@aol.com by imo-m06.mx.aol.com (mail_out_v34.21.) id z.c8.363b0ff4 (18707) for ; Sat, 5 Apr 2003 17:31:32 -0500 (EST) Message-ID: Date: Sat, 5 Apr 2003 17:31:32 EST Subject: [obm-l] geometria plana To: obm-l@mat.puc-rio.br MIME-Version: 1.0 Content-Type: multipart/alternative; boundary="part1_c8.363b0ff4.2bc0b344_boundary" X-Mailer: 6.0 sub 10516 Sender: owner-obm-l@sucuri.mat.puc-rio.br Precedence: bulk Reply-To: obm-l@mat.puc-rio.br --part1_c8.363b0ff4.2bc0b344_boundary Content-Type: text/plain; charset="ISO-8859-1" Content-Transfer-Encoding: quoted-printable Ol=E1 pessoal, As diagonais de um losango medem 18 cm e 24 cm. Qual =E9 a =E1rea do c=EDrcu= lo=20 inscrito neste losango?=20 Obs: Eu n=E3o consigo achar o raio da circunfer=EAncia inscrita, fazendo uma= =20 figura d=E1 para perceber que o raio =E9 um pouco menor que 9, j=E1 que o la= do=20 menor mede 18. Mas procuro um rela=E7=E3o entre estas duas regi=F5es planas,= pois=20 n=E3o temos aqui uma ciircunfer=EAncia inscrita em uma pol=EDgono regular. --part1_c8.363b0ff4.2bc0b344_boundary Content-Type: text/html; charset="ISO-8859-1" Content-Transfer-Encoding: quoted-printable Ol=E1 pessoal,

As diagonais de um losango medem 18 cm e 24 cm. Qual =E9 a =E1rea do c= =EDrculo inscrito neste losango?=20

Obs: Eu n=E3o consigo achar o raio da circunfer=EAncia inscrita, fazendo= uma figura d=E1 para perceber que o raio =E9 um pouco menor que 9, j=E1 que= o lado menor mede 18. Mas procuro um rela=E7=E3o entre estas duas regi=F5es= planas, pois n=E3o temos aqui uma ciircunfer=EAncia inscrita em uma pol=EDg= ono regular.
--part1_c8.363b0ff4.2bc0b344_boundary-- ========================================================================= Instruções para entrar na lista, sair da lista e usar a lista em http://www.mat.puc-rio.br/~nicolau/olimp/obm-l.html O administrador desta lista é ========================================================================= From owner-obm-l@sucuri.mat.puc-rio.br Sat Apr 5 19:46:17 2003 Return-Path: Received: (from majordom@localhost) by sucuri.mat.puc-rio.br (8.9.3/8.9.3) id TAA04814 for obm-l-MTTP; Sat, 5 Apr 2003 19:44:45 -0300 Received: from web20502.mail.yahoo.com (web20502.mail.yahoo.com [216.136.226.137]) by sucuri.mat.puc-rio.br (8.9.3/8.9.3) with SMTP id TAA04809 for ; Sat, 5 Apr 2003 19:44:41 -0300 Message-ID: <20030405224410.48336.qmail@web20502.mail.yahoo.com> Received: from [200.100.76.69] by web20502.mail.yahoo.com via HTTP; Sat, 05 Apr 2003 19:44:10 ART Date: Sat, 5 Apr 2003 19:44:10 -0300 (ART) From: =?iso-8859-1?q?Jhonata=20Emerick?= Subject: [obm-l] Grafos To: obm-l@mat.puc-rio.br MIME-Version: 1.0 Content-Type: multipart/alternative; boundary="0-845988220-1049582650=:48022" Content-Transfer-Encoding: 8bit Sender: owner-obm-l@sucuri.mat.puc-rio.br Precedence: bulk Reply-To: obm-l@mat.puc-rio.br --0-845988220-1049582650=:48022 Content-Type: text/plain; charset=iso-8859-1 Content-Transfer-Encoding: 8bit Olá pessoal da lista, quem puder me ajudar com esse problema.: Numa matriz A,nxn, em que os elementos aij pode corresponder exclusivamente a 1 ou a 0, sendo que 1 indica ligação e 0 indica falta dessa(ligação) entre os vertices i e j, como eu faço para descobrir o menor caminho entre a e b, sendo estes pares ordenados de inteiros positivos, a<=n e b<=n. --------------------------------- Yahoo! Mail O melhor e-mail gratuito da internet: 6MB de espaço, antivírus, acesso POP3, filtro contra spam. --0-845988220-1049582650=:48022 Content-Type: text/html; charset=iso-8859-1 Content-Transfer-Encoding: 8bit

Olá pessoal da lista, quem puder me ajudar com esse problema.:

Numa matriz A,nxn, em que os elementos aij pode corresponder exclusivamente a 1 ou a 0, sendo que 1 indica ligação e 0 indica falta dessa(ligação) entre os vertices i e j, como eu faço para descobrir o menor caminho entre a e b, sendo estes pares ordenados de inteiros positivos, a<=n e b<=n.



Yahoo! Mail
O melhor e-mail gratuito da internet: 6MB de espaço, antivírus, acesso POP3, filtro contra spam. --0-845988220-1049582650=:48022-- ========================================================================= Instruções para entrar na lista, sair da lista e usar a lista em http://www.mat.puc-rio.br/~nicolau/olimp/obm-l.html O administrador desta lista é ========================================================================= From owner-obm-l@sucuri.mat.puc-rio.br Sat Apr 5 20:45:06 2003 Return-Path: Received: (from majordom@localhost) by sucuri.mat.puc-rio.br (8.9.3/8.9.3) id UAA06462 for obm-l-MTTP; Sat, 5 Apr 2003 20:43:41 -0300 Received: from traven.uol.com.br (traven.uol.com.br [200.221.29.39]) by sucuri.mat.puc-rio.br (8.9.3/8.9.3) with ESMTP id UAA06458 for ; Sat, 5 Apr 2003 20:43:38 -0300 Received: from u2z7z2 ([200.158.144.59]) by traven.uol.com.br (8.9.1/8.9.1) with ESMTP id UAA22895 for ; Sat, 5 Apr 2003 20:43:07 -0300 (BRT) Message-ID: <003501c2fbcd$01f38840$3b909ec8@u2z7z2> From: "Wagner" To: References: Subject: Re: [obm-l] geometria plana Date: Sat, 5 Apr 2003 20:40:57 -0300 Organization: Wagner MIME-Version: 1.0 Content-Type: multipart/alternative; boundary="----=_NextPart_000_0032_01C2FBB3.A8F45DA0" X-Priority: 3 X-MSMail-Priority: Normal X-Mailer: Microsoft Outlook Express 5.50.4133.2400 X-MimeOLE: Produced By Microsoft MimeOLE V5.50.4133.2400 Sender: owner-obm-l@sucuri.mat.puc-rio.br Precedence: bulk Reply-To: obm-l@mat.puc-rio.br This is a multi-part message in MIME format. ------=_NextPart_000_0032_01C2FBB3.A8F45DA0 Content-Type: text/plain; charset="iso-8859-1" Content-Transfer-Encoding: quoted-printable Oi para todos! Seja ABCD esse losango. Seja E o encontro das diagonais AC e BD. Logo E = =E9 o centro da circunfer=EAncia. Tome o tri=E2ngulo ret=E2ngulo ABE de catetos 9 cm e 12 cm . Logo AB^2 = =3D 81 + 144 =3D 225 =3D> AB =3D 15 cm. O raio r da circunfer=EAncia =E9 igual a altura de ABE em rela=E7=E3o a = base AB. Seja x a =E1rea do tri=E2ngulo ABE. Ent=E3o x =3D 12.9/2 =3D 15.r/2 =3D> r =3D 7,2 cm =3D> a=3D51,84pi. Genericamente se as diagonais medissem d e D, usando uma resolu=E7=E3o = an=E1loga teriamos a=3D[(d^2.D^2)/4(d^2+D^2)].pi Andr=E9 T. ----- Original Message -----=20 From: Faelccmm@aol.com=20 To: obm-l@mat.puc-rio.br=20 Sent: Saturday, April 05, 2003 7:31 PM Subject: [obm-l] geometria plana Ol=E1 pessoal,=20 As diagonais de um losango medem 18 cm e 24 cm. Qual =E9 a =E1rea do = c=EDrculo inscrito neste losango?=20 Obs: Eu n=E3o consigo achar o raio da circunfer=EAncia inscrita, = fazendo uma figura d=E1 para perceber que o raio =E9 um pouco menor que = 9, j=E1 que o lado menor mede 18. Mas procuro um rela=E7=E3o entre estas = duas regi=F5es planas, pois n=E3o temos aqui uma ciircunfer=EAncia = inscrita em uma pol=EDgono regular.=20 ------=_NextPart_000_0032_01C2FBB3.A8F45DA0 Content-Type: text/html; charset="iso-8859-1" Content-Transfer-Encoding: quoted-printable
Oi para todos!
 
Seja ABCD esse losango. Seja E o = encontro das=20 diagonais AC e BD. Logo E =E9 o centro da circunfer=EAncia.
Tome o tri=E2ngulo ret=E2ngulo ABE de = catetos 9 cm e 12=20 cm . Logo AB^2 =3D 81 + 144 =3D 225 =3D> AB =3D 15 cm.
O raio r da circunfer=EAncia =E9 = igual a altura de=20 ABE em rela=E7=E3o a base AB. Seja x a =E1rea do tri=E2ngulo = ABE.
Ent=E3o x =3D 12.9/2 =3D 15.r/2 =3D> = r =3D 7,2 cm =3D>=20 a=3D51,84pi.
Genericamente se as diagonais medissem = d e D,=20 usando uma resolu=E7=E3o an=E1loga teriamos = a=3D[(d^2.D^2)/4(d^2+D^2)].pi
 
Andr=E9 T.
 
 
 
----- Original Message -----
From:=20 Faelccmm@aol.com=20
Sent: Saturday, April 05, 2003 = 7:31=20 PM
Subject: [obm-l] geometria = plana

Ol=E1 = pessoal,

As=20 diagonais de um losango medem 18 cm e 24 cm. Qual =E9 a =E1rea do = c=EDrculo inscrito=20 neste losango?

Obs: Eu n=E3o consigo achar o raio da = circunfer=EAncia=20 inscrita, fazendo uma figura d=E1 para perceber que o raio =E9 um = pouco menor que=20 9, j=E1 que o lado menor mede 18. Mas procuro um rela=E7=E3o entre = estas duas=20 regi=F5es planas, pois n=E3o temos aqui uma ciircunfer=EAncia inscrita = em uma=20 pol=EDgono regular.
------=_NextPart_000_0032_01C2FBB3.A8F45DA0-- ========================================================================= Instruções para entrar na lista, sair da lista e usar a lista em http://www.mat.puc-rio.br/~nicolau/olimp/obm-l.html O administrador desta lista é ========================================================================= From owner-obm-l@sucuri.mat.puc-rio.br Sat Apr 5 21:19:26 2003 Return-Path: Received: (from majordom@localhost) by sucuri.mat.puc-rio.br (8.9.3/8.9.3) id VAA07204 for obm-l-MTTP; Sat, 5 Apr 2003 21:17:53 -0300 Received: from artemis.opendf.com.br (artemis.opengate.com.br [200.181.71.14] (may be forged)) by sucuri.mat.puc-rio.br (8.9.3/8.9.3) with ESMTP id VAA07200 for ; Sat, 5 Apr 2003 21:17:49 -0300 Received: from localhost (localhost [127.0.0.1]) by artemis.opendf.com.br (Postfix) with ESMTP id 8519D2BEB9 for ; Sat, 5 Apr 2003 21:17:20 -0300 (BRT) Received: from artemis.opendf.com.br ([127.0.0.1]) by localhost (artemis.opengate.com.br [127.0.0.1:10024]) (amavisd-new) with ESMTP id 00563-09 for ; Sat, 5 Apr 2003 21:17:19 -0300 (BRT) Received: from computer (200-181-089-111.bsace7001.dsl.brasiltelecom.net.br [200.181.89.111]) by artemis.opendf.com.br (Postfix) with ESMTP id 4D4FF2BEA5 for ; Sat, 5 Apr 2003 21:17:19 -0300 (BRT) From: "Artur Costa Steiner" To: Subject: [obm-l] Problemas bonitos de Geometria Date: Sat, 5 Apr 2003 21:17:18 -0300 Organization: Steiner Consultoria LTDA Message-ID: <002001c2fbd1$e28fd990$9865fea9@computer> MIME-Version: 1.0 Content-Type: text/plain; charset="iso-8859-1" X-Priority: 3 (Normal) X-MSMail-Priority: Normal X-Mailer: Microsoft Outlook, Build 10.0.2627 Importance: Normal X-MimeOLE: Produced By Microsoft MimeOLE V6.00.2800.1106 X-Virus-Scanned: by amavisd-new Content-Transfer-Encoding: 8bit X-MIME-Autoconverted: from quoted-printable to 8bit by sucuri.mat.puc-rio.br id VAA07201 Sender: owner-obm-l@sucuri.mat.puc-rio.br Precedence: bulk Reply-To: obm-l@mat.puc-rio.br Olá a todos Eu acho estes dois problemas de Geometria Plana muito interesantes, embora eu seja mais ligado em Análise: 1) Considere um triângulo ABC, de lados a, b e c (na convenção usual) e o círculo C nele inscrito, Sejam P e Q os pontos em que AB e AC tangenciam C. Por algum ponto do arco PQ, distinto de P e de Q, tracemos a reta tangente a C, a qual intercepta AB e AC nos pontos M e N. Determine o perímetro do triângulo AMN. Resposta: P = b+c-a 2) No plano cartesiano, consideremos um círculo C e dois círculos C1 e C2, ambos interiores a C, tais que C1 e C2 sejam tangentes a C e tangenciem-se entre si. Sobre a tangente comum a C1 e C2, consideremos o segmento de comprimento t, compreendido entre os pontos em que a tangente intercepta C. Seja S a área da região do plano interior a C e exterior a cada um dos círculos C1 e C2. Determine S em função de t. Resposta: S = PI t^2/8 Estes problemas, embora bonitos, são simples. PROVA: eu consegui resolvê-los quando fiz vestibular para Engenharia. Artur ========================================================================= Instruções para entrar na lista, sair da lista e usar a lista em http://www.mat.puc-rio.br/~nicolau/olimp/obm-l.html O administrador desta lista é ========================================================================= From owner-obm-l@sucuri.mat.puc-rio.br Sat Apr 5 22:27:42 2003 Return-Path: Received: (from majordom@localhost) by sucuri.mat.puc-rio.br (8.9.3/8.9.3) id WAA08462 for obm-l-MTTP; Sat, 5 Apr 2003 22:25:49 -0300 Received: from mail.gmx.net (mail.gmx.net [213.165.64.20]) by sucuri.mat.puc-rio.br (8.9.3/8.9.3) with SMTP id WAA08458 for ; Sat, 5 Apr 2003 22:25:45 -0300 Received: (qmail 6728 invoked by uid 65534); 6 Apr 2003 01:25:12 -0000 Received: from unknown (EHLO localhost) (200.149.213.54) by mail.gmx.net (mp014-rz3) with SMTP; 06 Apr 2003 03:25:12 +0200 Date: Sat, 5 Apr 2003 22:24:48 -0300 From: Igor GomeZZ X-Mailer: The Bat! (v1.61) Organization: -- X-Priority: 3 (Normal) Message-ID: <19718063373.20030405222448@gmx.net> To: OBM Subject: [obm-l] Limite: Escola Naval/2002 MIME-Version: 1.0 Content-Type: text/plain; charset=ISO-8859-1 Content-Transfer-Encoding: 8bit Sender: owner-obm-l@sucuri.mat.puc-rio.br Precedence: bulk Reply-To: obm-l@mat.puc-rio.br Fala galera... O problema eh o seguinte: (pode-se usar L'Hôpital, era uma questão de múltiplas escolhas) Lim[x->0] [(cotgx)^(1/lnx)], cuja resposta eh e^(-1) ** Meu início de resolução: Seja (cotgx)^(1/lnx) = f(x) DEM1 cotgx = y ln(cotgx) = lny y = e^ln(cotgx) f(x) = (e^(ln(cotgx)))^(1/lnx) = e^(ln(cotgx)/lnx) = e^g(x) g(x) = ln(cotgx)/lnx = ln(cosx/senx)/lnx = ln(cosx)/lnx - ln(senx)/lnx , aqui pode-se mudar de base, mas não vejo utilidade... f(x) = e^(ln(cosx)/lnx - ln(senx)/lnx) = e^(ln(cosx)/lnx) / e^(ln(senx)/lnx) = E...? Qualquer dica tah valendo :-) Ateh! Fui! ####### Igor GomeZZ ######## UIN: 29249895 Vitória, Espírito Santo, Brasil Criação: 5/4/2003 (22:24) #################################### Pare para pensar: A diferença entre a genialidade e a estupidez é que a genialidade tem limites. (Autor Desconhecido) #################################### ========================================================================= Instruções para entrar na lista, sair da lista e usar a lista em http://www.mat.puc-rio.br/~nicolau/olimp/obm-l.html O administrador desta lista é ========================================================================= From owner-obm-l@sucuri.mat.puc-rio.br Sat Apr 5 22:40:57 2003 Return-Path: Received: (from majordom@localhost) by sucuri.mat.puc-rio.br (8.9.3/8.9.3) id WAA08721 for obm-l-MTTP; Sat, 5 Apr 2003 22:39:38 -0300 Received: from trex.centroin.com.br (trex.centroin.com.br [200.225.63.134]) by sucuri.mat.puc-rio.br (8.9.3/8.9.3) with ESMTP id WAA08717 for ; Sat, 5 Apr 2003 22:39:35 -0300 Received: from trex.centroin.com.br (localhost [127.0.0.1]) by trex.centroin.com.br (8.12.9/8.12.1) with ESMTP id h361dRBn018107 for ; Sat, 5 Apr 2003 22:39:27 -0300 (EST) Received: by trex.centroin.com.br (8.12.9/8.12.5/Submit) id h361dRoJ018106; Sat, 5 Apr 2003 22:39:27 -0300 (EST) Message-Id: <200304060139.h361dRoJ018106@trex.centroin.com.br> Received: from 200.225.58.88 by trex.centroin.com.br (CIPWM versao 1.4C1) with HTTPS for ; Sat, 5 Apr 2003 22:39:27 -0300 (EST) Date: Sat, 5 Apr 2003 22:39:27 -0300 (EST) From: Augusto Cesar de Oliveira Morgado To: obm-l@mat.puc-rio.br Subject: Re: [obm-l] Limite: Escola Naval/2002 MIME-Version: 1.0 X-Mailer: CentroIn Internet Provider WebMail v. 1.4C1 (http://www.centroin.com.br/) Content-Type: text/plain; charset="iso-8859-1" Content-Transfer-Encoding: 8bit X-MIME-Autoconverted: from quoted-printable to 8bit by sucuri.mat.puc-rio.br id WAA08718 Sender: owner-obm-l@sucuri.mat.puc-rio.br Precedence: bulk Reply-To: obm-l@mat.puc-rio.br (cotgx)^(1/lnx) = e ^[ln cotx /lnx] ln cot x / ln x encontra-se na forma infinito/infinito Por L Hopital, lim [ln cotx/ln x] = lim [- (cosec x)^2] * x /cotx = lim [- x/ senx * cosx] = -1 porque o co-seno tende a 1 e x/sen x tende a 1. Logo, e ^[ln cotx /lnx] tende a e^(-1) Morgado Em Sat, 5 Apr 2003 22:24:48 -0300, Igor GomeZZ disse: O problema eh o seguinte: (pode-se usar L'Hôpital, era uma questão de múltiplas escolhas) Lim[x->0] [(cotgx)^(1/lnx)], cuja resposta eh e^(-1) ####### Igor GomeZZ ######## > UIN: 29249895 > Vitória, Espírito Santo, Brasil > Criação: 5/4/2003 (22:24) > #################################### > Pare para pensar: > > A diferença entre a genialidade e > a estupidez é que a genialidade > tem limites. (Autor Desconhecido) > > #################################### ========================================================================= Instruções para entrar na lista, sair da lista e usar a lista em http://www.mat.puc-rio.br/~nicolau/olimp/obm-l.html O administrador desta lista é ========================================================================= From owner-obm-l@sucuri.mat.puc-rio.br Sun Apr 6 01:10:55 2003 Return-Path: Received: (from majordom@localhost) by sucuri.mat.puc-rio.br (8.9.3/8.9.3) id BAA11080 for obm-l-MTTP; Sun, 6 Apr 2003 01:08:39 -0300 Received: from mail.gmx.net (mail.gmx.net [213.165.64.20]) by sucuri.mat.puc-rio.br (8.9.3/8.9.3) with SMTP id BAA11076 for ; Sun, 6 Apr 2003 01:08:35 -0300 Received: (qmail 7778 invoked by uid 65534); 6 Apr 2003 04:08:03 -0000 Received: from unknown (EHLO localhost) (200.149.213.54) by mail.gmx.net (mp013-rz3) with SMTP; 06 Apr 2003 06:08:03 +0200 Date: Sun, 6 Apr 2003 01:07:46 -0300 From: Igor GomeZZ X-Mailer: The Bat! (v1.61) Organization: -- X-Priority: 3 (Normal) Message-ID: <17627841073.20030406010746@gmx.net> To: Augusto Cesar de Oliveira Morgado Subject: Re[2]: [obm-l] Limite: Escola Naval/2002 In-Reply-To: <200304060139.h361dRoJ018106@trex.centroin.com.br> References: <200304060139.h361dRoJ018106@trex.centroin.com.br> MIME-Version: 1.0 Content-Type: text/plain; charset=ISO-8859-1 Content-Transfer-Encoding: 8bit Sender: owner-obm-l@sucuri.mat.puc-rio.br Precedence: bulk Reply-To: obm-l@mat.puc-rio.br Em 5/4/2003, 22:39, Augusto (morgado@centroin.com.br) disse: > (cotgx)^(1/lnx) = e ^[ln cotx /lnx] > ln cot x / ln x encontra-se na forma infinito/infinito > Por L Hopital, lim [ln cotx/ln x] = lim [- (cosec x)^2] * x /cotx = > lim [- x/ senx * cosx] = -1 porque o co-seno tende a 1 e x/sen x tende a 1. > Logo, e ^[ln cotx /lnx] tende a e^(-1) > Morgado Morgado, mais uma vez muito obrigado! Fui! ####### Igor GomeZZ ######## UIN: 29249895 Vitória, Espírito Santo, Brasil Criação: 6/4/2003 (01:06) #################################### Pare para pensar: Amigo: alguém que sabe de tudo a teu respeito e gosta de ti assim mesmo. (Elbert Hubbard) #################################### ========================================================================= Instruções para entrar na lista, sair da lista e usar a lista em http://www.mat.puc-rio.br/~nicolau/olimp/obm-l.html O administrador desta lista é ========================================================================= From owner-obm-l@sucuri.mat.puc-rio.br Sun Apr 6 07:47:19 2003 Return-Path: Received: (from majordom@localhost) by sucuri.mat.puc-rio.br (8.9.3/8.9.3) id HAA17496 for obm-l-MTTP; Sun, 6 Apr 2003 07:45:42 -0300 Received: from imo-d08.mx.aol.com (imo-d08.mx.aol.com [205.188.157.40]) by sucuri.mat.puc-rio.br (8.9.3/8.9.3) with ESMTP id HAA17492 for ; Sun, 6 Apr 2003 07:45:38 -0300 From: Faelccmm@aol.com Received: from Faelccmm@aol.com by imo-d08.mx.aol.com (mail_out_v34.21.) id z.1e6.60d9446 (4362) for ; Sun, 6 Apr 2003 06:45:03 -0400 (EDT) Message-ID: <1e6.60d9446.2bc15f2f@aol.com> Date: Sun, 6 Apr 2003 06:45:03 EDT Subject: [obm-l] geometria plana II To: obm-l@mat.puc-rio.br MIME-Version: 1.0 Content-Type: multipart/alternative; boundary="part1_1e6.60d9446.2bc15f2f_boundary" X-Mailer: 6.0 sub 10516 Sender: owner-obm-l@sucuri.mat.puc-rio.br Precedence: bulk Reply-To: obm-l@mat.puc-rio.br --part1_1e6.60d9446.2bc15f2f_boundary Content-Type: text/plain; charset="ISO-8859-1" Content-Transfer-Encoding: quoted-printable Ol=E1 pessoal, Vejam a quest=E3o: Dada uma circunfer=EAncia cujo raio mede 6 cm, calcular a =E1rea de um segme= nto=20 circular cujo arco A=3D120 graus.=20 Resposta: =C1rea setor =3D m(A).pi.r^2/360 =3D 120.pi.6^2/360 =3D12 pi cm2 =C1rea tri=E2ngulo =3D 6 R[3] 3/2 =3D 9 R[3] cm2 =C1rea segmento =3D =C1rea setor - =C1rea tri=E2ngulo =3D (12 pi - 9R[3]) cm= 2 =20 Obs: A =FAnica coisa que n=E3o entendi na resolu=E7=E3o foi o por que da=20= =C1rea=20 tri=E2ngulo =3D 6 R[3] 3/2 --part1_1e6.60d9446.2bc15f2f_boundary Content-Type: text/html; charset="ISO-8859-1" Content-Transfer-Encoding: quoted-printable Ol=E1 pessoal,

Vejam a quest=E3o:

Dada uma circunfer=EAncia cujo raio mede 6 cm, calcular a =E1rea de um <= B>segmento circular cujo arco A=3D120 graus.=20

Resposta: =C1rea setor =3D m(A).pi.r^2/360 =3D 120.pi.6^2/360 =3D12 pi= cm2

=C1rea tri=E2ngulo =3D 6 R[3] 3/2 =3D 9 R[3] cm2

=C1rea segmento =3D =C1rea setor - =C1rea tri=E2ngulo =3D (12 pi - 9R[3]= ) cm2 =20

Obs: A =FAnica coisa que n=E3o entendi na resolu=E7=E3o foi o por que da=
=C1rea tri=E2ngulo =3D 6 R[3] 3/2 --part1_1e6.60d9446.2bc15f2f_boundary-- ========================================================================= Instruções para entrar na lista, sair da lista e usar a lista em http://www.mat.puc-rio.br/~nicolau/olimp/obm-l.html O administrador desta lista é ========================================================================= From owner-obm-l@sucuri.mat.puc-rio.br Sun Apr 6 11:46:07 2003 Return-Path: Received: (from majordom@localhost) by sucuri.mat.puc-rio.br (8.9.3/8.9.3) id LAA19741 for obm-l-MTTP; Sun, 6 Apr 2003 11:44:19 -0300 Received: from ivoti.terra.com.br (ivoti.terra.com.br [200.176.3.20]) by sucuri.mat.puc-rio.br (8.9.3/8.9.3) with ESMTP id LAA19737 for ; Sun, 6 Apr 2003 11:44:17 -0300 Received: from jurua.terra.com.br (jurua.terra.com.br [200.176.3.42]) by ivoti.terra.com.br (Postfix) with ESMTP id E465F408743 for ; Sun, 6 Apr 2003 11:43:46 -0300 (BRT) Received: from [200.177.182.229] (dl-nas6-sao-C8B1B6E5.p001.terra.com.br [200.177.182.229]) by jurua.terra.com.br (Postfix) with ESMTP id 972FCB002E for ; Sun, 6 Apr 2003 11:43:45 -0300 (BRT) User-Agent: Microsoft-Outlook-Express-Macintosh-Edition/5.02.2022 Date: Sun, 06 Apr 2003 11:41:56 -0300 Subject: Re: [obm-l] geometria plana II From: Claudio Buffara To: Message-ID: In-Reply-To: <1e6.60d9446.2bc15f2f@aol.com> Mime-version: 1.0 Content-type: multipart/alternative; boundary="MS_Mac_OE_3132474116_67240_MIME_Part" Sender: owner-obm-l@sucuri.mat.puc-rio.br Precedence: bulk Reply-To: obm-l@mat.puc-rio.br > This message is in MIME format. Since your mail reader does not understand this format, some or all of this message may not be legible. --MS_Mac_OE_3132474116_67240_MIME_Part Content-type: text/plain; charset="ISO-8859-1" Content-transfer-encoding: quoted-printable on 06.04.03 07:45, Faelccmm@aol.com at Faelccmm@aol.com wrote: Ol=E1 pessoal,=20 Vejam a quest=E3o:=20 Dada uma circunfer=EAncia cujo raio mede 6 cm, calcular a =E1rea de um segmento circular cujo arco A=3D120 graus. Resposta: =C1rea setor =3D m(A).pi.r^2/360 =3D 120.pi.6^2/360 =3D12 pi cm2 =C1rea tri=E2ngulo =3D 6 R[3] 3/2 =3D 9 R[3] cm2 =C1rea segmento =3D =C1rea setor - =C1rea tri=E2ngulo =3D (12 pi - 9R[3]) cm2 Obs: A =FAnica coisa que n=E3o entendi na resolu=E7=E3o foi o por que da =C1rea tri=E2ngulo =3D 6 R[3] 3/2 Antes de mais nada, Theta =3D 120 graus =3D 2Pi/3 radianos. Assim, teremos: Area do Setor =3D (1/2)*R^2*Theta =3D (1/2)*6^2*(2Pi/3) =3D 12*Pi cm^2 Area do Triangulo =3D (1/2)*R*R*sen(Theta) =3D (1/2)*6*6*sen(2Pi/3) =3D =3D (1/2)*36*raiz(3)/2 =3D 9*raiz(3) cm^2 Logo: Area do Segmento =3D Area do Setor - Area do Triangulo =3D =3D 12*Pi - 9*raiz(3) cm^2 Eu calculei a area do triangulo pela formula que envolve o comprimento de dois lados e o seno do angulo compreendido =3D (1/2)*Lado1*Lado2*sen(Angulo). Para expressar a area do triangulo da forma que fez, o elaborador do seu gabarito deve te-la calculado da maneira mais complicada possivel. Algo assim: O triangulo eh isosceles com dois lados medindo 6 cm e o angulo compreendid= o igua a 120 graus =3D=3D> os dois angulos iguais medem 30 graus cada. Pela lei dos cossenos, voce deduz que o terceiro lado mede 6*raiz(3) cm. A altura "h" relativa a este lado maior eh tal que h/6 =3D sen(30) =3D 1/2 =3D=3D> = h =3D 3 cm. Agora, use Area =3D Base * Altura / 2 e voce obterah: Area =3D 6*raiz(3)*3/2. Ou seja, mesmo quando dah a resposta certa, o seu gabarito escolhe a soluca= o "errada" (no sentido de ser a mais complicada) Um abraco, Claudio. --MS_Mac_OE_3132474116_67240_MIME_Part Content-type: text/html; charset="ISO-8859-1" Content-transfer-encoding: quoted-printable Re: [obm-l] geometria plana II on 06.04.03 07:45, Faelccmm@aol.com at Faelccmm@aol.com wrote:

Ol=E1 pessoal,

Vejam a quest=E3o:

Dada uma circunfer=EAncia cujo raio mede 6 cm, calcular a =E1rea de um segme= nto circular cujo arco A=3D120 graus.

Resposta:   =C1rea setor =3D m(A).pi.r^2/360 =3D 120.pi.6^2/360 =3D12 pi = cm2

=C1rea tri=E2ngulo =3D 6 R[3] 3/2= =3D 9 R[3] cm2

=C1rea segmento =3D =C1rea setor - =C1rea tri=E2ngulo =3D (12 pi - 9R[3]) cm2  &nb= sp;

Obs: A =FAnica coisa que n=E3o entendi na resolu=E7=E3o foi o por que da =C1rea tri=E2ngulo =3D 6 R[3] 3/2


Antes de mais nada, Theta =3D 120 graus =3D 2Pi/3 radianos.

Assim, teremos:
Area do Setor =3D (1/2)*R^2*Theta =3D (1/2)*6^2*(2Pi/3) =3D 12*Pi cm^2

Area do Triangulo =3D (1/2)*R*R*sen(Theta) =3D (1/2)*6*6*sen(2Pi/3) =3D
=3D (1/2)*36*raiz(3)/2 =3D 9*raiz(3) cm^2

Logo:
Area do Segmento =3D Area do Setor - Area do Triangulo =3D
=3D 12*Pi - 9*raiz(3) cm^2


Eu calculei a area do triangulo pela formula que envolve o comprimento de d= ois lados e o seno do angulo compreendido =3D (1/2)*Lado1*Lado2*sen(Angulo).
Para expressar a area do triangulo da forma que fez, o elaborador do seu ga= barito deve te-la calculado da maneira mais complicada possivel. Algo assim:=

O triangulo eh isosceles com dois lados medindo 6 cm e o angulo compreendid= o igua a 120 graus =3D=3D> os dois angulos iguais medem 30 graus cada.

Pela lei dos cossenos, voce deduz que o terceiro lado mede 6*raiz(3) cm.
A altura "h" relativa a este lado maior eh tal que h/6 =3D sen(30) = =3D 1/2 =3D=3D> h =3D 3 cm.

Agora, use Area =3D Base * Altura / 2 e voce obterah:
Area =3D 6*raiz(3)*3/2.

Ou seja, mesmo quando dah a resposta certa, o seu gabarito escolhe a soluca= o "errada" (no sentido de ser a mais complicada)


Um abraco,
Claudio. --MS_Mac_OE_3132474116_67240_MIME_Part-- ========================================================================= Instruções para entrar na lista, sair da lista e usar a lista em http://www.mat.puc-rio.br/~nicolau/olimp/obm-l.html O administrador desta lista é ========================================================================= From owner-obm-l@sucuri.mat.puc-rio.br Sun Apr 6 12:06:03 2003 Return-Path: Received: (from majordom@localhost) by sucuri.mat.puc-rio.br (8.9.3/8.9.3) id MAA20248 for obm-l-MTTP; Sun, 6 Apr 2003 12:04:43 -0300 Received: from itaqui.terra.com.br (itaqui.terra.com.br [200.176.3.19]) by sucuri.mat.puc-rio.br (8.9.3/8.9.3) with ESMTP id MAA20244 for ; Sun, 6 Apr 2003 12:04:40 -0300 Received: from sapucaia.terra.com.br (sapucaia.terra.com.br [200.176.3.41]) by itaqui.terra.com.br (Postfix) with ESMTP id 9CC4F3BC0BD for ; Sun, 6 Apr 2003 12:04:10 -0300 (BRT) Received: from [200.177.186.134] (dl-nas4-sao-C8B1BA86.p001.terra.com.br [200.177.186.134]) by sapucaia.terra.com.br (Postfix) with ESMTP id 728FD304039 for ; Sun, 6 Apr 2003 12:04:09 -0300 (BRT) User-Agent: Microsoft-Outlook-Express-Macintosh-Edition/5.02.2022 Date: Sun, 06 Apr 2003 12:02:19 -0300 Subject: Re: [obm-l] Grafos From: Claudio Buffara To: Message-ID: In-Reply-To: <20030405221641.46397.qmail@web20508.mail.yahoo.com> Mime-version: 1.0 Content-type: multipart/alternative; boundary="MS_Mac_OE_3132475340_140850_MIME_Part" Sender: owner-obm-l@sucuri.mat.puc-rio.br Precedence: bulk Reply-To: obm-l@mat.puc-rio.br > This message is in MIME format. Since your mail reader does not understand this format, some or all of this message may not be legible. --MS_Mac_OE_3132475340_140850_MIME_Part Content-type: text/plain; charset="ISO-8859-1" Content-transfer-encoding: quoted-printable on 05.04.03 19:16, Jhonata Emerick at jhonobm@yahoo.com.br wrote: Ol=E1 pessoal da lista, quem puder me ajudar com esse problema.: Numa matriz A,nxn, em que os elementos aij pode corresponder exclusivamente a 1 ou a 0, sendo que 1 indica liga=E7=E3o e 0 indica falta dessa(liga=E7=E3o) entr= e os vertices i e j, como eu fa=E7o para descobrir o menor caminho entre a e b, sendo estes pares ordenados de inteiros positivos, a<=3Dn e b<=3Dn. Seja L =3D comprimento do caminho de a ate b: Se a =3D b entao L =3D 0. Se A(a,b) =3D 1 =3D=3D> L =3D 1 (a e b sao ligados por uma aresta) Caso contrario, L =3D menor inteiro positivo n tal que o elemento (a,b) de A^= n eh <> 0. Ou seja, voce vai calculando potencias sucessivas de A =3D matriz de incidencia do seu grafo ateh que o elemento cujos indices correspondem ao seus dois vertices seja diferente de zero. O comprimento do seu caminho minimo serah justamente o expoente no qual isso acontece. Um abraco, Claudio. =20 Yahoo! Mail O melhor e-mail gratuito da internet: 6MB de espa=E7o, antiv=EDrus, acesso POP3= , filtro contra spam. --MS_Mac_OE_3132475340_140850_MIME_Part Content-type: text/html; charset="ISO-8859-1" Content-transfer-encoding: quoted-printable Re: [obm-l] Grafos on 05.04.03 19:16, Jhonata Emerick at jhonobm@yahoo.com.br wrote:


Ol=E1 pessoal da lista, quem puder me ajudar com esse problema.:

Numa matriz A,nxn, em que os elementos aij pode corresponder exclusivamente= a 1 ou a 0, sendo que 1 indica liga=E7=E3o e 0 indica falta dessa(liga=E7=E3o) entr= e os vertices i e j, como eu fa=E7o para descobrir o menor caminho entre a e b= , sendo estes pares ordenados de inteiros positivos, a<=3Dn e b<=3Dn.

Seja L =3D comprimento do caminho de a ate b:

Se a =3D b entao L =3D 0.

Se A(a,b) =3D 1 =3D=3D> L =3D 1 (a e b sao ligados por uma aresta)

Caso contrario, L =3D menor inteiro positivo n tal que o elemento (a,b) de A^= n eh <> 0.

Ou seja, voce vai calculando potencias sucessivas de A =3D matriz de incidenc= ia do seu grafo ateh que o elemento cujos indices correspondem ao seus dois = vertices seja diferente de zero. O comprimento do seu caminho minimo serah j= ustamente o expoente no qual isso acontece.

Um abraco,
Claudio.

 




Yahoo! Mail  <http://br.mail.yahoo.com/>
O melhor e-mail gratuito da internet: 6MB de espa=E7o, antiv=EDrus, acesso = POP3, filtro contra spam.

--MS_Mac_OE_3132475340_140850_MIME_Part-- ========================================================================= Instruções para entrar na lista, sair da lista e usar a lista em http://www.mat.puc-rio.br/~nicolau/olimp/obm-l.html O administrador desta lista é ========================================================================= From owner-obm-l@sucuri.mat.puc-rio.br Sun Apr 6 17:35:20 2003 Return-Path: Received: (from majordom@localhost) by sucuri.mat.puc-rio.br (8.9.3/8.9.3) id RAA23799 for obm-l-MTTP; Sun, 6 Apr 2003 17:32:54 -0300 Received: from shannon.bol.com.br (shannon.bol.com.br [200.221.24.13]) by sucuri.mat.puc-rio.br (8.9.3/8.9.3) with ESMTP id RAA23795 for ; Sun, 6 Apr 2003 17:32:51 -0300 From: renatinha15a@bol.com.br Received: from bol.com.br (200.221.24.133) by shannon.bol.com.br (5.1.071) id 3E76735F00620B95 for obm-l@mat.puc-rio.br; Sun, 6 Apr 2003 17:32:21 -0300 Date: Sun, 6 Apr 2003 17:32:21 -0300 Message-Id: Subject: [obm-l] =?iso-8859-1?q?=FAltimo_n=FAmero?= MIME-Version: 1.0 Content-Type: text/plain;charset="iso-8859-1" To: "obm" X-XaM3-API-Version: 2.4 R3 ( B4 ) X-SenderIP: 200.241.108.194 Content-Transfer-Encoding: 8bit X-MIME-Autoconverted: from quoted-printable to 8bit by sucuri.mat.puc-rio.br id RAA23796 Sender: owner-obm-l@sucuri.mat.puc-rio.br Precedence: bulk Reply-To: obm-l@mat.puc-rio.br Oi gente, fico grata se alguém me ajudar. Determine o último algarismo (algarismo das unidades) do número 14^(14^14). []´s Renatinha __________________________________________________________________________ E-mail Premium BOL Antivírus, anti-spam e até 100 MB de espaço. Assine já! http://email.bol.com.br/ ========================================================================= Instruções para entrar na lista, sair da lista e usar a lista em http://www.mat.puc-rio.br/~nicolau/olimp/obm-l.html O administrador desta lista é ========================================================================= From owner-obm-l@sucuri.mat.puc-rio.br Sun Apr 6 18:28:11 2003 Return-Path: Received: (from majordom@localhost) by sucuri.mat.puc-rio.br (8.9.3/8.9.3) id SAA24936 for obm-l-MTTP; Sun, 6 Apr 2003 18:26:41 -0300 Received: from imo-r07.mx.aol.com (imo-r07.mx.aol.com [152.163.225.103]) by sucuri.mat.puc-rio.br (8.9.3/8.9.3) with ESMTP id SAA24931 for ; Sun, 6 Apr 2003 18:26:38 -0300 From: Faelccmm@aol.com Received: from Faelccmm@aol.com by imo-r07.mx.aol.com (mail_out_v34.21.) id z.12b.27286c8e (4116) for ; Sun, 6 Apr 2003 17:26:01 -0400 (EDT) Message-ID: <12b.27286c8e.2bc1f569@aol.com> Date: Sun, 6 Apr 2003 17:26:01 EDT Subject: [obm-l] geometria plana III To: obm-l@mat.puc-rio.br MIME-Version: 1.0 Content-Type: multipart/alternative; boundary="part1_12b.27286c8e.2bc1f569_boundary" X-Mailer: 6.0 sub 10516 Sender: owner-obm-l@sucuri.mat.puc-rio.br Precedence: bulk Reply-To: obm-l@mat.puc-rio.br --part1_12b.27286c8e.2bc1f569_boundary Content-Type: text/plain; charset="ISO-8859-1" Content-Transfer-Encoding: quoted-printable Ol=E1 pessoal, Como resolver a quest=E3o 23 do site: http://pessoal.sercomtel.com.br/matematica/gplana/209/exe209b.htm Obs: Estou me complicando na intersec=E7=E3o das circunfer=EAncias =20 --part1_12b.27286c8e.2bc1f569_boundary Content-Type: text/html; charset="ISO-8859-1" Content-Transfer-Encoding: quoted-printable Ol=E1 pessoal,

Como resolver a quest=E3o 23 do site:
http://pessoal.sercomtel.com.br/matematica/gplana/209/exe209b.htm
Obs: Estou me complicando na intersec=E7=E3o das circunfer=EAncias  = ;
--part1_12b.27286c8e.2bc1f569_boundary-- ========================================================================= Instruções para entrar na lista, sair da lista e usar a lista em http://www.mat.puc-rio.br/~nicolau/olimp/obm-l.html O administrador desta lista é ========================================================================= From owner-obm-l@sucuri.mat.puc-rio.br Sun Apr 6 18:48:10 2003 Return-Path: Received: (from majordom@localhost) by sucuri.mat.puc-rio.br (8.9.3/8.9.3) id SAA25357 for obm-l-MTTP; Sun, 6 Apr 2003 18:46:46 -0300 Received: from itaqui.terra.com.br (itaqui.terra.com.br [200.176.3.19]) by sucuri.mat.puc-rio.br (8.9.3/8.9.3) with ESMTP id SAA25353 for ; Sun, 6 Apr 2003 18:46:44 -0300 Received: from marova.terra.com.br (marova.terra.com.br [200.176.3.39]) by itaqui.terra.com.br (Postfix) with ESMTP id 32ED43BC591 for ; Sun, 6 Apr 2003 18:46:14 -0300 (BRT) Received: from [200.177.179.29] (dl-nas3-sao-C8B1B31D.p001.terra.com.br [200.177.179.29]) by marova.terra.com.br (Postfix) with ESMTP id 3307B3DC076 for ; Sun, 6 Apr 2003 18:46:13 -0300 (BRT) User-Agent: Microsoft-Outlook-Express-Macintosh-Edition/5.02.2022 Date: Sun, 06 Apr 2003 18:44:22 -0300 Subject: Re: [obm-l] =?ISO-8859-1?B?+mx0aW1vIG76?=mero From: Claudio Buffara To: Message-ID: In-Reply-To: Mime-version: 1.0 Content-type: text/plain; charset="ISO-8859-1" Content-Transfer-Encoding: 8bit X-MIME-Autoconverted: from quoted-printable to 8bit by sucuri.mat.puc-rio.br id SAA25354 Sender: owner-obm-l@sucuri.mat.puc-rio.br Precedence: bulk Reply-To: obm-l@mat.puc-rio.br on 06.04.03 17:32, renatinha15a@bol.com.br at renatinha15a@bol.com.br wrote: > Oi gente, fico grata se alguém me ajudar. > > Determine o último algarismo (algarismo das unidades) do > número 14^(14^14). > > []´s > Renatinha > > > __________________________________________________________________________ > E-mail Premium BOL > Antivírus, anti-spam e até 100 MB de espaço. Assine já! > http://email.bol.com.br/ > > > ========================================================================= > Instruções para entrar na lista, sair da lista e usar a lista em > http://www.mat.puc-rio.br/~nicolau/olimp/obm-l.html > O administrador desta lista é > ========================================================================= > Use o seguinte fato (facilmente demonstrado via propriedades das congruencias): Se os inteiros M e N tem o mesmo algarismo das unidades, entao, para todo inteiro positivo k, M^k e N^k tem o mesmo algarismo das unidades. Isso se representa assim: Se M = N (mod 10) entao M^k = N^k (mod 10) 14 = 4 (mod 10) 14^2 = 4^2 = 16 = 6 (mod 10) 14^3 = 4^3 = 64 = 4 (mod 10) 14^4 = 4^4 = 256 = 6 (mod 10) Ou seja,em geral: k eh par ==> 14^k = 4^k = 6 (mod 10) k eh impar ==> 14^k = 4^k = 4 (mod 10) Como 14^14 eh par, temos que 14^(14^14) = 6 (mod 10) Logo, o algarismo das unidades de 14^(14^14) eh 6. ========================================================================= Instruções para entrar na lista, sair da lista e usar a lista em http://www.mat.puc-rio.br/~nicolau/olimp/obm-l.html O administrador desta lista é ========================================================================= From owner-obm-l@sucuri.mat.puc-rio.br Sun Apr 6 19:05:01 2003 Return-Path: Received: (from majordom@localhost) by sucuri.mat.puc-rio.br (8.9.3/8.9.3) id TAA26003 for obm-l-MTTP; Sun, 6 Apr 2003 19:03:41 -0300 Received: from orion.netbank.com.br (orion.netbank.com.br [200.203.199.90]) by sucuri.mat.puc-rio.br (8.9.3/8.9.3) with ESMTP id TAA25998 for ; Sun, 6 Apr 2003 19:03:38 -0300 Received: from [200.193.162.249] (helo=smtp.leitner.homeip.net) by orion.netbank.com.br with asmtp (Exim 3.33 #1) id 192IIN-0004Vq-00 for obm-l@mat.puc-rio.br; Sun, 06 Apr 2003 19:07:23 -0300 Received: from darkstar.leitner.homeip.net (darkstar [192.168.1.3]) by smtp.leitner.homeip.net (Postfix) with ESMTP id 45CC4901F for ; Sun, 6 Apr 2003 19:06:16 -0300 (BRT) Received: by darkstar.leitner.homeip.net (Postfix, from userid 502) id 996D4271B11; Sun, 6 Apr 2003 19:06:31 -0300 (BRT) Date: Sun, 6 Apr 2003 19:06:31 -0300 From: Eduardo Henrique Leitner To: obm-l@mat.puc-rio.br Subject: Re: [obm-l] =?iso-8859-1?Q?=FAltimo_n=FAme?= =?iso-8859-1?Q?ro?= Message-ID: <20030406220631.GA15000@darkstar.leitner.homeip.net> References: Mime-Version: 1.0 Content-Type: text/plain; charset=iso-8859-1 Content-Disposition: inline In-Reply-To: User-Agent: Mutt/1.5.4i Content-Transfer-Encoding: 8bit X-MIME-Autoconverted: from quoted-printable to 8bit by sucuri.mat.puc-rio.br id TAA26000 Sender: owner-obm-l@sucuri.mat.puc-rio.br Precedence: bulk Reply-To: obm-l@mat.puc-rio.br simples: 14^1 : 4 14^2 : 6 14^3 : 4 14^4 : 6 como o q importa eh o ultimo algarismo, temos sempre: 4 * 4 = 6; 6 * 4 = 4 ... podemos concluir q qndo 14 estah elevado a um numero impar, o ultimo algarismo eh 4, e quando 14 estah elevado a um algarismo par, o ultimo algarismo é 6 como (14^14) eh par, o ultimo algarismo de 14^(14^14) eh 6 []'s On Sun, Apr 06, 2003 at 05:32:21PM -0300, renatinha15a@bol.com.br wrote: > Oi gente, fico grata se alguém me ajudar. > > Determine o último algarismo (algarismo das unidades) do > número 14^(14^14). > > []´s > Renatinha > > > __________________________________________________________________________ > E-mail Premium BOL > Antivírus, anti-spam e até 100 MB de espaço. Assine já! > http://email.bol.com.br/ > > > ========================================================================= > Instruções para entrar na lista, sair da lista e usar a lista em > http://www.mat.puc-rio.br/~nicolau/olimp/obm-l.html > O administrador desta lista é > ========================================================================= ========================================================================= Instruções para entrar na lista, sair da lista e usar a lista em http://www.mat.puc-rio.br/~nicolau/olimp/obm-l.html O administrador desta lista é ========================================================================= From owner-obm-l@sucuri.mat.puc-rio.br Sun Apr 6 19:53:51 2003 Return-Path: Received: (from majordom@localhost) by sucuri.mat.puc-rio.br (8.9.3/8.9.3) id TAA27492 for obm-l-MTTP; Sun, 6 Apr 2003 19:51:10 -0300 Received: from fnn.net ([200.175.38.9]) by sucuri.mat.puc-rio.br (8.9.3/8.9.3) with SMTP id TAA27488 for ; Sun, 6 Apr 2003 19:51:07 -0300 Received: (qmail 26667 invoked from network); 6 Apr 2003 22:37:01 -0000 Received: from unknown (HELO windows98) (200.175.39.122) by fnn.net with SMTP; 6 Apr 2003 22:37:01 -0000 Message-ID: <002201c2fc98$4ec420a0$9a75fea9@windows98> From: "Daniel Pini" To: Subject: [obm-l] =?iso-8859-1?B?ZmF0b3Jh5+Nv?= Date: Sun, 6 Apr 2003 20:57:39 -0300 MIME-Version: 1.0 Content-Type: multipart/alternative; boundary="----=_NextPart_000_001F_01C2FC7F.289CB540" X-Priority: 3 X-MSMail-Priority: Normal X-Mailer: Microsoft Outlook Express 5.00.2615.200 X-MimeOLE: Produced By Microsoft MimeOLE V5.00.2615.200 Sender: owner-obm-l@sucuri.mat.puc-rio.br Precedence: bulk Reply-To: obm-l@mat.puc-rio.br This is a multi-part message in MIME format. ------=_NextPart_000_001F_01C2FC7F.289CB540 Content-Type: text/plain; charset="iso-8859-1" Content-Transfer-Encoding: quoted-printable N=E3o consegui fatorar as seguintes express=F5es. Por favor me ajudem a = resolve-las. Fatore: a) a^4 + b^4 - c^4 - 2a=B2b=B2 + 4abc=B2 b) a=B3+b=B3+c=B3-3abc c)1+y(1+x)=B2(1+xy) d)3xyz+x(y=B2+z=B2)+y(z=B2+x=B2)+z(x=B2+y=B2) 2-Se n =E9 um n=FAmero par ent=E3o 2^n - 1 =E9 sempre divisivel por: R:3 3-Os dois n=FAmeros entre 60 e 70 que dividem 2^48 -1 s=E3o? R:63 e 65 4- Simplifique:=20 {bx(a=B2x=B2+2a=B2y=B2+b=B2y=B2)+ay(a=B2x=B2+2b=B2x=B2+b=B2y=B2)}/S S=3D bx + ay R:(ax+by)=B2 Obrigado. Daniel.=20 ------=_NextPart_000_001F_01C2FC7F.289CB540 Content-Type: text/html; charset="iso-8859-1" Content-Transfer-Encoding: quoted-printable
N=E3o consegui fatorar as seguintes = express=F5es. Por favor me=20 ajudem a resolve-las.
Fatore:
a) a^4 + b^4 - c^4 - 2a=B2b=B2 + = 4abc=B2
b) a=B3+b=B3+c=B3-3abc
c)1+y(1+x)=B2(1+xy)
d)3xyz+x(y=B2+z=B2)+y(z=B2+x=B2)+z(x=B2+y=B2)
2-Se n =E9 um n=FAmero par ent=E3o 2^n - 1 =E9 = sempre divisivel=20 por: R:3
3-Os dois n=FAmeros entre 60 e 70 = que dividem 2^48=20 -1 s=E3o? R:63 e 65
4- Simplifique: 
 {bx(a=B2x=B2+2a=B2y=B2+b=B2y=B2)+ay(a=B2x=B2+2b=B2x=B2= +b=B2y=B2)}/S
S=3D bx + ay
R:(ax+by)=B2
 
Obrigado. = Daniel. 
------=_NextPart_000_001F_01C2FC7F.289CB540-- ========================================================================= Instruções para entrar na lista, sair da lista e usar a lista em http://www.mat.puc-rio.br/~nicolau/olimp/obm-l.html O administrador desta lista é ========================================================================= From owner-obm-l@sucuri.mat.puc-rio.br Sun Apr 6 20:51:41 2003 Return-Path: Received: (from majordom@localhost) by sucuri.mat.puc-rio.br (8.9.3/8.9.3) id UAA28607 for obm-l-MTTP; Sun, 6 Apr 2003 20:50:17 -0300 Received: from orion.netbank.com.br (orion.netbank.com.br [200.203.199.90]) by sucuri.mat.puc-rio.br (8.9.3/8.9.3) with ESMTP id UAA28603 for ; Sun, 6 Apr 2003 20:50:14 -0300 Received: from [200.193.162.249] (helo=smtp.leitner.homeip.net) by orion.netbank.com.br with asmtp (Exim 3.33 #1) id 192JxY-0004l8-00 for obm-l@mat.puc-rio.br; Sun, 06 Apr 2003 20:54:00 -0300 Received: from darkstar.leitner.homeip.net (darkstar [192.168.1.3]) by smtp.leitner.homeip.net (Postfix) with ESMTP id 002AB901F for ; Sun, 6 Apr 2003 20:52:52 -0300 (BRT) Received: by darkstar.leitner.homeip.net (Postfix, from userid 502) id 15C6D271B11; Sun, 6 Apr 2003 20:53:08 -0300 (BRT) Date: Sun, 6 Apr 2003 20:53:07 -0300 From: Eduardo Henrique Leitner To: obm-l@mat.puc-rio.br Subject: Re: [obm-l] =?iso-8859-1?B?ZmF0b3Jh5+Nv?= Message-ID: <20030406235307.GA15255@darkstar.leitner.homeip.net> References: <002201c2fc98$4ec420a0$9a75fea9@windows98> Mime-Version: 1.0 Content-Type: text/plain; charset=iso-8859-1 Content-Disposition: inline In-Reply-To: <002201c2fc98$4ec420a0$9a75fea9@windows98> User-Agent: Mutt/1.5.4i Content-Transfer-Encoding: 8bit X-MIME-Autoconverted: from quoted-printable to 8bit by sucuri.mat.puc-rio.br id UAA28604 Sender: owner-obm-l@sucuri.mat.puc-rio.br Precedence: bulk Reply-To: obm-l@mat.puc-rio.br > 3-Os dois números entre 60 e 70 que dividem 2^48 -1 são? R:63 e 65 2^48 - 1 = (2^24 + 1)(2^24 - 1) = (2^24 + 1)(2^12 + 1)(2^12 - 1)= (2^24 + 1)(2^12 + 1)(2^6 + 1)(2^6 - 1)= (2^24 + 1)(2^12 + 1)(64 + 1)(64 - 1)= (2^24 + 1)(2^12 + 1)*65*63 portanto 2^48 - 1 eh divisivel por 63 e 65 []'s On Sun, Apr 06, 2003 at 08:57:39PM -0300, Daniel Pini wrote: > Não consegui fatorar as seguintes expressões. Por favor me ajudem a resolve-las. > Fatore: > a) a^4 + b^4 - c^4 - 2a²b² + 4abc² > b) a³+b³+c³-3abc > c)1+y(1+x)²(1+xy) > d)3xyz+x(y²+z²)+y(z²+x²)+z(x²+y²) > 2-Se n é um número par então 2^n - 1 é sempre divisivel por: R:3 > 3-Os dois números entre 60 e 70 que dividem 2^48 -1 são? R:63 e 65 > 4- Simplifique: > {bx(a²x²+2a²y²+b²y²)+ay(a²x²+2b²x²+b²y²)}/S > S= bx + ay > R:(ax+by)² > > Obrigado. Daniel. ========================================================================= Instruções para entrar na lista, sair da lista e usar a lista em http://www.mat.puc-rio.br/~nicolau/olimp/obm-l.html O administrador desta lista é ========================================================================= From owner-obm-l@sucuri.mat.puc-rio.br Sun Apr 6 22:26:29 2003 Return-Path: Received: (from majordom@localhost) by sucuri.mat.puc-rio.br (8.9.3/8.9.3) id WAA29795 for obm-l-MTTP; Sun, 6 Apr 2003 22:22:47 -0300 Received: from orion.netbank.com.br (orion.netbank.com.br [200.203.199.90]) by sucuri.mat.puc-rio.br (8.9.3/8.9.3) with ESMTP id WAA29791 for ; Sun, 6 Apr 2003 22:22:44 -0300 Received: from [200.193.162.249] (helo=smtp.leitner.homeip.net) by orion.netbank.com.br with asmtp (Exim 3.33 #1) id 192LP4-00050v-00 for obm-l@mat.puc-rio.br; Sun, 06 Apr 2003 22:26:30 -0300 Received: from darkstar.leitner.homeip.net (darkstar [192.168.1.3]) by smtp.leitner.homeip.net (Postfix) with ESMTP id B2762901F for ; Sun, 6 Apr 2003 22:25:22 -0300 (BRT) Received: by darkstar.leitner.homeip.net (Postfix, from userid 502) id 906EF271B11; Sun, 6 Apr 2003 22:25:37 -0300 (BRT) Date: Sun, 6 Apr 2003 22:25:37 -0300 From: Eduardo Henrique Leitner To: obm-l@mat.puc-rio.br Subject: Re: [obm-l] =?iso-8859-1?B?ZmF0b3Jh5+Nv?= Message-ID: <20030407012537.GA15449@darkstar.leitner.homeip.net> References: <002201c2fc98$4ec420a0$9a75fea9@windows98> <20030406235307.GA15255@darkstar.leitner.homeip.net> Mime-Version: 1.0 Content-Type: text/plain; charset=iso-8859-1 Content-Disposition: inline In-Reply-To: <20030406235307.GA15255@darkstar.leitner.homeip.net> User-Agent: Mutt/1.5.4i Content-Transfer-Encoding: 8bit X-MIME-Autoconverted: from quoted-printable to 8bit by sucuri.mat.puc-rio.br id WAA29792 Sender: owner-obm-l@sucuri.mat.puc-rio.br Precedence: bulk Reply-To: obm-l@mat.puc-rio.br > > 2-Se n é um número par então 2^n - 1 é sempre divisivel por: R:3 essa eu acho q dah pra fazer por indução, tipo: 2^2 - 1 = 3, entao vamos ver se sempre será divisivel por 3 admitindo q para 2K a expressão abaixo seja verdadeira [2^(2K) - 1] / 3 = Z, tal q Z eh um numero natural. entao temos q: 3Z + 1 = 2^2K testemos se para K+1 a expressão tambem serah divisivel por 3 2^[2(K+1)] - 1 = 2^[2K + 2] - 1 = 2^(2K)*2^2 - 1 = sabemos que 2^2K = 3Z + 1, entao (3Z + 1)4 - 1 = 12Z + 4 - 1 = 12Z + 3 = 3(4Z + 1) e portanto, eh divihsivel por 3 entao provamos que 2^n - 1 é sempre divisivel por 3 se n for um numero par acho q eh isso, qualquer erro ou conceito errado q usei por favor me corrijam []'s On Sun, Apr 06, 2003 at 08:53:07PM -0300, Eduardo Henrique Leitner wrote: > > 3-Os dois números entre 60 e 70 que dividem 2^48 -1 são? R:63 e 65 > > 2^48 - 1 = > (2^24 + 1)(2^24 - 1) = > (2^24 + 1)(2^12 + 1)(2^12 - 1)= > (2^24 + 1)(2^12 + 1)(2^6 + 1)(2^6 - 1)= > (2^24 + 1)(2^12 + 1)(64 + 1)(64 - 1)= > (2^24 + 1)(2^12 + 1)*65*63 > > portanto 2^48 - 1 eh divisivel por 63 e 65 > > []'s > > On Sun, Apr 06, 2003 at 08:57:39PM -0300, Daniel Pini wrote: > > Não consegui fatorar as seguintes expressões. Por favor me ajudem a resolve-las. > > Fatore: > > a) a^4 + b^4 - c^4 - 2a²b² + 4abc² > > b) a³+b³+c³-3abc > > c)1+y(1+x)²(1+xy) > > d)3xyz+x(y²+z²)+y(z²+x²)+z(x²+y²) > > 2-Se n é um número par então 2^n - 1 é sempre divisivel por: R:3 > > 3-Os dois números entre 60 e 70 que dividem 2^48 -1 são? R:63 e 65 > > 4- Simplifique: > > {bx(a²x²+2a²y²+b²y²)+ay(a²x²+2b²x²+b²y²)}/S > > S= bx + ay > > R:(ax+by)² > > > > Obrigado. Daniel. > ========================================================================= > Instruções para entrar na lista, sair da lista e usar a lista em > http://www.mat.puc-rio.br/~nicolau/olimp/obm-l.html > O administrador desta lista é > ========================================================================= ========================================================================= Instruções para entrar na lista, sair da lista e usar a lista em http://www.mat.puc-rio.br/~nicolau/olimp/obm-l.html O administrador desta lista é ========================================================================= From owner-obm-l@sucuri.mat.puc-rio.br Sun Apr 6 23:12:26 2003 Return-Path: Received: (from majordom@localhost) by sucuri.mat.puc-rio.br (8.9.3/8.9.3) id XAA30787 for obm-l-MTTP; Sun, 6 Apr 2003 23:11:06 -0300 Received: from shannon.bol.com.br (shannon.bol.com.br [200.221.24.13]) by sucuri.mat.puc-rio.br (8.9.3/8.9.3) with ESMTP id XAA30783 for ; Sun, 6 Apr 2003 23:11:03 -0300 Received: from bol.com.br (200.221.24.128) by shannon.bol.com.br (5.1.071) id 3E76735F00639C4B for obm-l@mat.puc-rio.br; Sun, 6 Apr 2003 23:10:33 -0300 Date: Sun, 6 Apr 2003 23:10:33 -0300 Message-Id: Subject: [obm-l] =?iso-8859-1?q?Re=3A_=5Bobm=2Dl=5D_=FAltimo_n=FAmero_=28grata=29?= MIME-Version: 1.0 Content-Type: text/plain;charset="iso-8859-1" From: "renatinha15a" To: obm-l@mat.puc-rio.br X-XaM3-API-Version: 2.4 R3 ( B4 ) X-SenderIP: 200.241.108.194 Content-Transfer-Encoding: 8bit X-MIME-Autoconverted: from quoted-printable to 8bit by sucuri.mat.puc-rio.br id XAA30784 Sender: owner-obm-l@sucuri.mat.puc-rio.br Precedence: bulk Reply-To: obm-l@mat.puc-rio.br Obrigado pela explicação, Claudio e Eduardo. []´s, Renatinha __________________________________________________________________________ E-mail Premium BOL Antivírus, anti-spam e até 100 MB de espaço. Assine já! http://email.bol.com.br/ ========================================================================= Instruções para entrar na lista, sair da lista e usar a lista em http://www.mat.puc-rio.br/~nicolau/olimp/obm-l.html O administrador desta lista é ========================================================================= From owner-obm-l@sucuri.mat.puc-rio.br Sun Apr 6 23:23:29 2003 Return-Path: Received: (from majordom@localhost) by sucuri.mat.puc-rio.br (8.9.3/8.9.3) id XAA31104 for obm-l-MTTP; Sun, 6 Apr 2003 23:22:10 -0300 Received: from ivoti.terra.com.br (ivoti.terra.com.br [200.176.3.20]) by sucuri.mat.puc-rio.br (8.9.3/8.9.3) with ESMTP id XAA31100 for ; Sun, 6 Apr 2003 23:22:07 -0300 Received: from una.terra.com.br (una.terra.com.br [200.176.3.32]) by ivoti.terra.com.br (Postfix) with ESMTP id 66C09408EC8 for ; Sun, 6 Apr 2003 23:21:37 -0300 (BRT) Received: from [200.177.190.55] (dl-nas5-sao-C8B1BE37.p001.terra.com.br [200.177.190.55]) by una.terra.com.br (Postfix) with ESMTP id 3623F2F005D for ; Sun, 6 Apr 2003 23:21:36 -0300 (BRT) User-Agent: Microsoft-Outlook-Express-Macintosh-Edition/5.02.2022 Date: Sun, 06 Apr 2003 23:19:45 -0300 Subject: Re: [obm-l] fatora=?ISO-8859-1?B?5+M=?=o From: Claudio Buffara To: Message-ID: In-Reply-To: <002201c2fc98$4ec420a0$9a75fea9@windows98> Mime-version: 1.0 Content-type: multipart/alternative; boundary="MS_Mac_OE_3132515986_102842_MIME_Part" Sender: owner-obm-l@sucuri.mat.puc-rio.br Precedence: bulk Reply-To: obm-l@mat.puc-rio.br > This message is in MIME format. Since your mail reader does not understand this format, some or all of this message may not be legible. --MS_Mac_OE_3132515986_102842_MIME_Part Content-type: text/plain; charset="ISO-8859-1" Content-transfer-encoding: quoted-printable Oi, Daniel: Aqui vao uma dicas para os problemas abaixo. Um abraco, Claudio. on 06.04.03 20:57, Daniel Pini at daniel@fnn.net wrote: N=E3o consegui fatorar as seguintes express=F5es. Por favor me ajudem a resolve-las. Fatore: a) a^4 + b^4 - c^4 - 2a=B2b=B2 + 4abc=B2 Dica: Escreva a expessao como um polinomio biquadrado em c e calcule o discriminante (delta) b) a=B3+b=B3+c=B3-3abc Dica: Suponha que c =3D -(a+b) e veja o que acontece c)1+y(1+x)=B2(1+xy) Dica: Escreva a expressao como um polinomio quadratico em y e calcule o discriminante =20 d)3xyz+x(y=B2+z=B2)+y(z=B2+x=B2)+z(x=B2+y=B2) Dica: Suponha que z =3D -(x+y) e veja o que acontece 2-Se n =E9 um n=FAmero par ent=E3o 2^n - 1 =E9 sempre divisivel por: R:3 Dica: Se n eh par entao 2^n =3D 4^m para algum inteiro m. Para que valores de m (4^m - 1) eh divisivel por (4 - 1)? 3-Os dois n=FAmeros entre 60 e 70 que dividem 2^48 -1 s=E3o? R:63 e 65 Dica: Fatore 2^48 - 1 o maximo que voce puder 4- Simplifique:=20 {bx(a=B2x=B2+2a=B2y=B2+b=B2y=B2)+ay(a=B2x=B2+2b=B2x=B2+b=B2y=B2)}/S S=3D bx + ay R:(ax+by)=B2 Dica: Tente isolar o fator (ax+by) no numerador Obrigado. Daniel.=20 --MS_Mac_OE_3132515986_102842_MIME_Part Content-type: text/html; charset="ISO-8859-1" Content-transfer-encoding: quoted-printable Re: [obm-l] fatora=E7=E3o Oi, Daniel:

Aqui vao uma dicas para os problemas abaixo.

Um abraco,
Claudio.

on 06.04.03 20:57, Daniel Pini at daniel@fnn.net wrote:

N=E3o consegui fatorar as seguintes express=F5es= . Por favor me ajudem a resolve-las.
Fatore:
a) a^4 + b^4 - c^4 - 2a=B2b=B2 + 4abc=B2
Dica: Escreva a expessao como um polinomio biquadrado em c e calcule o disc= riminante (delta)

b) a=B3+b=B3+c=B3-3abc
Dica: Suponha que c =3D -(a+b) e veja o que acontece

c)1+y(1+x)=B2(1+xy)
Dica: Escreva a expressao como um polinomio quadratico em y e calcule o dis= criminante
 
d)3xyz+x(y=B2+z=B2)+y(z=B2+x=B2)+z(x=B2+y=B2)
Dica: Suponha que z =3D -(x+y) e veja o que acontece

2-Se n =E9 um n=FAmero par ent=E3o 2^n - 1 =E9 sempre divisivel = por: R:3
Dica: Se n eh par entao 2^n =3D 4^m para algum inteiro m. Para que val= ores de m (4^m - 1) eh divisivel por (4 - 1)?

3-Os dois n=FAmeros entre 60 e 70 que dividem 2^48 -1 s=E3o?= R:63 e 65
Dica: Fatore 2^48 - 1 o maximo que voce puder

4- Simplifique:
{bx(a=B2x=B2+2a=B2y=B2+b=B2y=B2)+ay(a=B2x=B2+2b=B2x=B2+b=B2y=B2)}/S
S=3D bx + ay
R:(ax+by)=B2
Dica: Tente isolar o fator (ax+by) no numerador

Obrigado. Daniel.


--MS_Mac_OE_3132515986_102842_MIME_Part-- ========================================================================= Instruções para entrar na lista, sair da lista e usar a lista em http://www.mat.puc-rio.br/~nicolau/olimp/obm-l.html O administrador desta lista é ========================================================================= From owner-obm-l@sucuri.mat.puc-rio.br Sun Apr 6 23:37:32 2003 Return-Path: Received: (from majordom@localhost) by sucuri.mat.puc-rio.br (8.9.3/8.9.3) id XAA31628 for obm-l-MTTP; Sun, 6 Apr 2003 23:35:53 -0300 Received: from paiol.terra.com.br (paiol.terra.com.br [200.176.3.18]) by sucuri.mat.puc-rio.br (8.9.3/8.9.3) with ESMTP id XAA31624 for ; Sun, 6 Apr 2003 23:35:50 -0300 Received: from botucatu.terra.com.br (botucatu.terra.com.br [200.176.3.78]) by paiol.terra.com.br (Postfix) with ESMTP id DEAAB87DF7 for ; Sun, 6 Apr 2003 23:35:19 -0300 (BRT) Received: from [200.177.190.232] (dl-nas5-sao-C8B1BEE8.p001.terra.com.br [200.177.190.232]) by botucatu.terra.com.br (Postfix) with ESMTP id D364D29C088 for ; Sun, 6 Apr 2003 23:35:18 -0300 (BRT) User-Agent: Microsoft-Outlook-Express-Macintosh-Edition/5.02.2022 Date: Sun, 06 Apr 2003 23:33:28 -0300 Subject: Re: [obm-l] geometria plana III From: Claudio Buffara To: Message-ID: In-Reply-To: <12b.27286c8e.2bc1f569@aol.com> Mime-version: 1.0 Content-type: multipart/alternative; boundary="MS_Mac_OE_3132516808_152325_MIME_Part" Sender: owner-obm-l@sucuri.mat.puc-rio.br Precedence: bulk Reply-To: obm-l@mat.puc-rio.br > This message is in MIME format. Since your mail reader does not understand this format, some or all of this message may not be legible. --MS_Mac_OE_3132516808_152325_MIME_Part Content-type: text/plain; charset="ISO-8859-1" Content-transfer-encoding: quoted-printable on 06.04.03 18:26, Faelccmm@aol.com at Faelccmm@aol.com wrote: Ol=E1 pessoal,=20 Como resolver a quest=E3o 23 do site: http://pessoal.sercomtel.com.br/matematica/gplana/209/exe209b.htm Obs: Estou me complicando na intersec=E7=E3o das circunfer=EAncias Oi, Fael: Semicircunfer=EAncias s=E3o tra=E7ados sobre dois lados de um quadrado de lados medindo 6 cm. Calcular a =E1rea da regi=E3o pintada na figura ao lado. Essa eh uma aplicacao do principio da inclusao-exclusao em geometria. Area Pedida =3D Area do Quadrado - 2*Area do Semicirculo + Area da Lunula Area do Quadrado =3D 6*6 =3D 36 cm^2 Area do Semicirculo =3D pi*3^2/2) =3D 9*pi/2 cm^2 Area da Lunula =3D 2*Area do Segmento Circular Area do Segmento Circular =3D Area do Setor Circular - Area do Triangulo =3D =3D (1/2)*3^2*(pi/2) - (1/2)*3^2*sen(pi/2) =3D =3D (9*pi/4 - 9/2) cm^2 =3D=3D> Area da Lunula =3D 2*(9*pi/4 - 9/2) =3D (9*pi/2 - 9) cm^2 Logo, Area Pedida =3D 36 - 2*(9*pi/2) + 9*pi/2 - 9 =3D (27 - 9*pi/2) cm^2 Um abraco, Claudio. --MS_Mac_OE_3132516808_152325_MIME_Part Content-type: text/html; charset="ISO-8859-1" Content-transfer-encoding: quoted-printable Re: [obm-l] geometria plana III on 06.04.03 18:26, Faelccmm@aol.com at Faelccmm@aol.com wrote:

Ol=E1 pessoal,

Como resolver a quest=E3o 23 do site:
http://pessoal.sercomtel.com.br/matematica/gplana/209/exe209b.htm
Obs: Estou me complicando na intersec=E7=E3o das circunfer=EAncias  


Oi, Fael:

Semicircunfer=EAncias s=E3o tra=E7ados sobre dois lados de um quadrado de lados m= edindo 6 cm. Calcular a =E1rea da regi=E3o pintada na figura ao lado.

Essa eh uma aplicacao do principio da inclusao-exclusao em geometria.

Area Pedida =3D Area do Quadrado - 2*Area do Semicirculo + Area da Lunula

Area do Quadrado =3D 6*6 =3D 36 cm^2

Area do Semicirculo =3D pi*3^2/2) =3D 9*pi/2 cm^2

Area da Lunula =3D 2*Area do Segmento Circular

Area do Segmento Circular =3D Area do Setor Circular - Area do Triangulo =3D =3D (1/2)*3^2*(pi/2) - (1/2)*3^2*sen(pi/2) =3D
=3D (9*pi/4 - 9/2) cm^2 =3D=3D>

Area da Lunula =3D 2*(9*pi/4 - 9/2) =3D (9*pi/2 - 9) cm^2

Logo, Area Pedida =3D 36 - 2*(9*pi/2) + 9*pi/2 - 9 =3D (27 - 9*pi/2) cm^2

Um abraco,
Claudio.

--MS_Mac_OE_3132516808_152325_MIME_Part-- ========================================================================= Instruções para entrar na lista, sair da lista e usar a lista em http://www.mat.puc-rio.br/~nicolau/olimp/obm-l.html O administrador desta lista é ========================================================================= From owner-obm-l@sucuri.mat.puc-rio.br Mon Apr 7 00:05:14 2003 Return-Path: Received: (from majordom@localhost) by sucuri.mat.puc-rio.br (8.9.3/8.9.3) id AAA32586 for obm-l-MTTP; Mon, 7 Apr 2003 00:03:54 -0300 Received: from artemis.opendf.com.br (artemis.opengate.com.br [200.181.71.14]) by sucuri.mat.puc-rio.br (8.9.3/8.9.3) with ESMTP id AAA32582 for ; Mon, 7 Apr 2003 00:03:51 -0300 Received: from localhost (localhost [127.0.0.1]) by artemis.opendf.com.br (Postfix) with ESMTP id E65E32BEBE for ; Mon, 7 Apr 2003 00:03:20 -0300 (BRT) Received: from artemis.opendf.com.br ([127.0.0.1]) by localhost (artemis.opengate.com.br [127.0.0.1:10024]) (amavisd-new) with ESMTP id 13518-09 for ; Mon, 7 Apr 2003 00:03:20 -0300 (BRT) Received: from computer (200-181-088-086.bsace7001.dsl.brasiltelecom.net.br [200.181.88.86]) by artemis.opendf.com.br (Postfix) with ESMTP id 4A29B2BEA5 for ; Mon, 7 Apr 2003 00:03:19 -0300 (BRT) From: "Artur Costa Steiner" To: Subject: [obm-l] =?iso-8859-1?Q?RE:_=5Bobm-l=5D_fatora=E7=E3o?= Date: Mon, 7 Apr 2003 00:03:16 -0300 Organization: Steiner Consultoria LTDA Message-ID: <000201c2fcb2$3e9537e0$9865fea9@computer> MIME-Version: 1.0 Content-Type: text/plain; charset="iso-8859-1" X-Priority: 3 (Normal) X-MSMail-Priority: Normal X-Mailer: Microsoft Outlook, Build 10.0.2627 In-Reply-To: <20030407012537.GA15449@darkstar.leitner.homeip.net> Importance: Normal X-MimeOLE: Produced By Microsoft MimeOLE V6.00.2800.1106 X-Virus-Scanned: by amavisd-new Content-Transfer-Encoding: 8bit X-MIME-Autoconverted: from quoted-printable to 8bit by sucuri.mat.puc-rio.br id AAA32583 Sender: owner-obm-l@sucuri.mat.puc-rio.br Precedence: bulk Reply-To: obm-l@mat.puc-rio.br >> > 2-Se n é um número par então 2^n - 1 é sempre divisivel por: R:3 Outra forma de provar isto é considerr o polionômio dado por P(x) = x^n -1. O resto da divisão de P pelo binômio x+1 é p(-1) = (-1)^n -1. Se n for par, então o resto é zero, do que concluímos que, se x for um inteiro positivo, então x é divisível por x+1. Logo, em tais casos 2^n -1 é divisível por 3. Por um raciocínio similar, observamos que, se n for par, então x^n -1 é divisível por x-1, x natural. É por isso que, se representarmos os números em uma base b>1,, então m é divisível por b-1 sse a soma de seus algarismos os for, e m é divisível por b+1 sse a soma de seus algarismos de ordem ímpar menos a soma de seus algarismos de ordem par for divisível por b+1. Na base 10, isto também vale para 3 (a soma ser divisível por 3) porque 10 = 3^2 + 1 [Artur Costa Steiner] >essa eu acho q dah pra fazer por indução, tipo: > >2^2 - 1 = 3, entao vamos ver se sempre será divisivel por 3 > >admitindo q para 2K a expressão abaixo seja verdadeira >[2^(2K) - 1] / 3 = Z, tal q Z eh um numero natural. > >entao temos q: > >3Z + 1 = 2^2K > >testemos se para K+1 a expressão tambem serah divisivel por 3 > >2^[2(K+1)] - 1 = >2^[2K + 2] - 1 = >2^(2K)*2^2 - 1 = sabemos que 2^2K = 3Z + 1, entao >(3Z + 1)4 - 1 = >12Z + 4 - 1 = >12Z + 3 = >3(4Z + 1) > >e portanto, eh divihsivel por 3 > >entao provamos que 2^n - 1 é sempre divisivel por 3 se n for um numero par > > >acho q eh isso, qualquer erro ou conceito errado q usei por favor me >corrijam > >[]'s > >On Sun, Apr 06, 2003 at 08:53:07PM -0300, Eduardo Henrique Leitner wrote: >> > 3-Os dois números entre 60 e 70 que dividem 2^48 -1 são? R:63 e 65 >> >> 2^48 - 1 = >> (2^24 + 1)(2^24 - 1) = >> (2^24 + 1)(2^12 + 1)(2^12 - 1)= >> (2^24 + 1)(2^12 + 1)(2^6 + 1)(2^6 - 1)= >> (2^24 + 1)(2^12 + 1)(64 + 1)(64 - 1)= >> (2^24 + 1)(2^12 + 1)*65*63 >> >> portanto 2^48 - 1 eh divisivel por 63 e 65 >> >> []'s >> >> On Sun, Apr 06, 2003 at 08:57:39PM -0300, Daniel Pini wrote: >> > Não consegui fatorar as seguintes expressões. Por favor me ajudem a >resolve-las. >> > Fatore: >> > a) a^4 + b^4 - c^4 - 2a²b² + 4abc² >> > b) a³+b³+c³-3abc >> > c)1+y(1+x)²(1+xy) >> > d)3xyz+x(y²+z²)+y(z²+x²)+z(x²+y²) >> > 2-Se n é um número par então 2^n - 1 é sempre divisivel por: R:3 >> > 3-Os dois números entre 60 e 70 que dividem 2^48 -1 são? R:63 e 65 >> > 4- Simplifique: >> > {bx(a²x²+2a²y²+b²y²)+ay(a²x²+2b²x²+b²y²)}/S >> > S= bx + ay >> > R:(ax+by)² >> > >> > Obrigado. Daniel. >> ======================================================================== = >> Instruções para entrar na lista, sair da lista e usar a lista em >> http://www.mat.puc-rio.br/~nicolau/olimp/obm-l.html >> O administrador desta lista é >> ======================================================================== = >======================================================================= == >Instruções para entrar na lista, sair da lista e usar a lista em >http://www.mat.puc-rio.br/~nicolau/olimp/obm-l.html >O administrador desta lista é >======================================================================= == ========================================================================= Instruções para entrar na lista, sair da lista e usar a lista em http://www.mat.puc-rio.br/~nicolau/olimp/obm-l.html O administrador desta lista é ========================================================================= From owner-obm-l@sucuri.mat.puc-rio.br Mon Apr 7 00:10:35 2003 Return-Path: Received: (from majordom@localhost) by sucuri.mat.puc-rio.br (8.9.3/8.9.3) id AAA32726 for obm-l-MTTP; Mon, 7 Apr 2003 00:09:17 -0300 Received: from web80409.mail.yahoo.com (web80409.mail.yahoo.com [66.218.79.64]) by sucuri.mat.puc-rio.br (8.9.3/8.9.3) with SMTP id AAA32721 for ; Mon, 7 Apr 2003 00:09:14 -0300 Message-ID: <20030407030842.21079.qmail@web80409.mail.yahoo.com> Received: from [200.175.1.161] by web80409.mail.yahoo.com via HTTP; Mon, 07 Apr 2003 00:08:42 ART Date: Mon, 7 Apr 2003 00:08:42 -0300 (ART) From: "=?iso-8859-1?q?guilherme=20S.?=" Subject: Re: [obm-l] geometria plana III To: obm-l@mat.puc-rio.br In-Reply-To: <12b.27286c8e.2bc1f569@aol.com> MIME-Version: 1.0 Content-Type: text/plain; charset=iso-8859-1 Content-Transfer-Encoding: 8bit Sender: owner-obm-l@sucuri.mat.puc-rio.br Precedence: bulk Reply-To: obm-l@mat.puc-rio.br a ÁREA A é dada pela area metade do triangulo(s) menos duas vezes a área do setor circular(c) de 90°: s=6*6/2=18 cm^2 c=1/4*pi*3^2-3*3/2=[-18+9pi]/4 A=18-[-18+9pi]/2=27-9*pi/2 _______________________________________________________________________ Yahoo! Mail O melhor e-mail gratuito da internet: 6MB de espaço, antivírus, acesso POP3, filtro contra spam. http://br.mail.yahoo.com/ ========================================================================= Instruções para entrar na lista, sair da lista e usar a lista em http://www.mat.puc-rio.br/~nicolau/olimp/obm-l.html O administrador desta lista é ========================================================================= From owner-obm-l@sucuri.mat.puc-rio.br Mon Apr 7 00:35:11 2003 Return-Path: Received: (from majordom@localhost) by sucuri.mat.puc-rio.br (8.9.3/8.9.3) id AAA01480 for obm-l-MTTP; Mon, 7 Apr 2003 00:33:52 -0300 Received: from artemis.opendf.com.br (artemis.opengate.com.br [200.181.71.14]) by sucuri.mat.puc-rio.br (8.9.3/8.9.3) with ESMTP id AAA01476 for ; Mon, 7 Apr 2003 00:33:49 -0300 Received: from localhost (localhost [127.0.0.1]) by artemis.opendf.com.br (Postfix) with ESMTP id D33A52BEC4 for ; Mon, 7 Apr 2003 00:33:18 -0300 (BRT) Received: from artemis.opendf.com.br ([127.0.0.1]) by localhost (artemis.opengate.com.br [127.0.0.1:10024]) (amavisd-new) with ESMTP id 14031-07 for ; Mon, 7 Apr 2003 00:33:18 -0300 (BRT) Received: from computer (200-181-088-086.bsace7001.dsl.brasiltelecom.net.br [200.181.88.86]) by artemis.opendf.com.br (Postfix) with ESMTP id 5415F2BEC3 for ; Mon, 7 Apr 2003 00:33:17 -0300 (BRT) From: "Artur Costa Steiner" To: Subject: [obm-l] =?iso-8859-1?Q?RE:_=5Bobm-l=5D_fatora=E7=E3o?= Date: Mon, 7 Apr 2003 00:33:17 -0300 Organization: Steiner Consultoria LTDA Message-ID: <000a01c2fcb6$6e8ee0a0$9865fea9@computer> MIME-Version: 1.0 Content-Type: text/plain; charset="iso-8859-1" X-Priority: 3 (Normal) X-MSMail-Priority: Normal X-Mailer: Microsoft Outlook, Build 10.0.2627 In-Reply-To: Importance: Normal X-MimeOLE: Produced By Microsoft MimeOLE V6.00.2800.1106 X-Virus-Scanned: by amavisd-new Content-Transfer-Encoding: 8bit X-MIME-Autoconverted: from quoted-printable to 8bit by sucuri.mat.puc-rio.br id AAA01477 Sender: owner-obm-l@sucuri.mat.puc-rio.br Precedence: bulk Reply-To: obm-l@mat.puc-rio.br Uma correção: na realidade x^n -1, (x>2 , inteiro) , é sempre divisível por x-1, seja n par ou ímpar. Porque, neste caso, P(1) = 0 para qualquer natural n. Artur >> >>Outra forma de provar isto é considerr o polionômio dado por P(x) = x^n - >1. >>O resto da divisão de P pelo binômio x+1 é p(-1) = (-1)^n -1. Se n for >par, >>então o resto é zero, do que concluímos que, se x for um inteiro positivo, >>então x é divisível por x+1. Logo, em tais casos 2^n -1 é divisível por 3. >>Por um raciocínio similar, observamos que, se n for par, então x^n -1 é >>divisível por x-1, x natural. É por isso que, se representarmos os números >>em uma base b>1,, então m é divisível por b-1 sse a soma de seus >algarismos >>os for, e m é divisível por b+1 sse a soma de seus algarismos de ordem >>ímpar menos a soma de seus algarismos de ordem par for divisível por b+1. >>Na base 10, isto também vale para 3 (a soma ser divisível por 3) porque 10 >>= 3^2 + 1=========================================================== ========================================================================= Instruções para entrar na lista, sair da lista e usar a lista em http://www.mat.puc-rio.br/~nicolau/olimp/obm-l.html O administrador desta lista é ========================================================================= From owner-obm-l@sucuri.mat.puc-rio.br Mon Apr 7 01:03:34 2003 Return-Path: Received: (from majordom@localhost) by sucuri.mat.puc-rio.br (8.9.3/8.9.3) id BAA02505 for obm-l-MTTP; Mon, 7 Apr 2003 01:01:48 -0300 Received: from artemis.opendf.com.br (artemis.opengate.com.br [200.181.71.14]) by sucuri.mat.puc-rio.br (8.9.3/8.9.3) with ESMTP id BAA02501 for ; Mon, 7 Apr 2003 01:01:45 -0300 Received: from localhost (localhost [127.0.0.1]) by artemis.opendf.com.br (Postfix) with ESMTP id 6B0A32BEC4 for ; Mon, 7 Apr 2003 01:01:15 -0300 (BRT) Received: from artemis.opendf.com.br ([127.0.0.1]) by localhost (artemis.opengate.com.br [127.0.0.1:10024]) (amavisd-new) with ESMTP id 14031-09 for ; Mon, 7 Apr 2003 01:01:14 -0300 (BRT) Received: from computer (200-181-088-086.bsace7001.dsl.brasiltelecom.net.br [200.181.88.86]) by artemis.opendf.com.br (Postfix) with ESMTP id 455952BEC3 for ; Mon, 7 Apr 2003 01:01:14 -0300 (BRT) From: "Artur Costa Steiner" To: Subject: [obm-l] =?utf-8?Q?Problemas_de_An=C3=A1lise?= Date: Mon, 7 Apr 2003 01:01:11 -0300 Organization: Steiner Consultoria LTDA Message-ID: <000b01c2fcba$55b61c70$9865fea9@computer> MIME-Version: 1.0 Content-Type: text/plain; charset="utf-8" X-Priority: 3 (Normal) X-MSMail-Priority: Normal X-Mailer: Microsoft Outlook, Build 10.0.2627 Importance: Normal X-MimeOLE: Produced By Microsoft MimeOLE V6.00.2800.1106 X-Virus-Scanned: by amavisd-new Content-Transfer-Encoding: 8bit X-MIME-Autoconverted: from quoted-printable to 8bit by sucuri.mat.puc-rio.br id BAA02502 Sender: owner-obm-l@sucuri.mat.puc-rio.br Precedence: bulk Reply-To: obm-l@mat.puc-rio.br Para os que estão estudadando Análise, recomendo os seguintes exercícios. São instrutivos. 1) Seja f::R→ R uma função não idênticamente nula tal que f(x+y) = f(x) f(y) para todos x, y em R. Mostre que: a) f(x)>0 para todo x real b) f(-x) = 1/f(x) para todo x real c) sendo a = f(1), então f(x) = a^x para todo racional x. d) se for contínua em algum w em R, então f é contínua em todo R. e) se f for diferenciável em algum w em R, então f é diferenciável em todo R. Neste caso, para todo x real temos que f(x) = a^x f) sem assumir diferenciabilidade em R, é possível garantir que f seja uma função exponencial? 2) Seja f definida e contínua em um subconjunto D de R^p e com valores em R^q. Mostre que, se f apresentar limite em todos os pontos de acumulação de D, então f possui uma única extensão contínua para o fecho de D. Isto é, sendo D'o fecho de D, existe uma única função g:D' --> R^q, contínua em D', tal que g(x) = f(x) para todo x em D'. Artur ========================================================================= Instruções para entrar na lista, sair da lista e usar a lista em http://www.mat.puc-rio.br/~nicolau/olimp/obm-l.html O administrador desta lista é ========================================================================= From owner-obm-l@sucuri.mat.puc-rio.br Mon Apr 7 10:44:49 2003 Return-Path: Received: (from majordom@localhost) by sucuri.mat.puc-rio.br (8.9.3/8.9.3) id KAA09338 for obm-l-MTTP; Mon, 7 Apr 2003 10:41:32 -0300 Received: from web14309.mail.yahoo.com (web14309.mail.yahoo.com [216.136.224.59]) by sucuri.mat.puc-rio.br (8.9.3/8.9.3) with SMTP id KAA09329 for ; Mon, 7 Apr 2003 10:41:29 -0300 Message-ID: <20030407134057.64056.qmail@web14309.mail.yahoo.com> Received: from [200.144.49.45] by web14309.mail.yahoo.com via HTTP; Mon, 07 Apr 2003 10:40:56 ART Date: Mon, 7 Apr 2003 10:40:56 -0300 (ART) From: =?iso-8859-1?q?Rafael?= Subject: [obm-l] pontos notaveis To: OBM MIME-Version: 1.0 Content-Type: text/plain; charset=iso-8859-1 Content-Transfer-Encoding: 8bit Sender: owner-obm-l@sucuri.mat.puc-rio.br Precedence: bulk Reply-To: obm-l@mat.puc-rio.br Oi pessoal! Tenho essa questão aqui que não consigo resovler. Como parece-me trabalhosa, principalmente para digitar aqui, agradeço pelo menos umas dicas dos caminhos que devo seguir: Num triangulo ABC, a distância do baricentro ao ortocentro é igual a 4cm. Então a distãncia do baricentro ao circuncentro vale: a)0,5 b)1 c)2 d)4 e)8 Valeu! Rafael. _______________________________________________________________________ Yahoo! Mail O melhor e-mail gratuito da internet: 6MB de espaço, antivírus, acesso POP3, filtro contra spam. http://br.mail.yahoo.com/ ========================================================================= Instruções para entrar na lista, sair da lista e usar a lista em http://www.mat.puc-rio.br/~nicolau/olimp/obm-l.html O administrador desta lista é ========================================================================= From owner-obm-l@sucuri.mat.puc-rio.br Mon Apr 7 11:32:48 2003 Return-Path: Received: (from majordom@localhost) by sucuri.mat.puc-rio.br (8.9.3/8.9.3) id LAA10947 for obm-l-MTTP; Mon, 7 Apr 2003 11:31:03 -0300 Received: from mail.ccet.ufrn.br (venus.ccet.ufrn.br [200.19.174.45]) by sucuri.mat.puc-rio.br (8.9.3/8.9.3) with ESMTP id LAA10942 for ; Mon, 7 Apr 2003 11:30:59 -0300 Received: from scheelita (schelita.ccet.ufrn.br [10.9.0.137]) by mail.ccet.ufrn.br (Postfix) with SMTP id 4164614499 for ; Mon, 7 Apr 2003 11:30:29 -0300 (GMT+3) Message-ID: <004801c2fd11$78e155a0$8900090a@cceta.ufrn.br> From: "bene" To: References: Subject: [obm-l] =?iso-8859-1?Q?Re:_=5Bobm-l=5D_=FAltimo_n=FAmero?= Date: Mon, 7 Apr 2003 11:25:00 -0300 MIME-Version: 1.0 Content-Type: text/plain; charset="iso-8859-1" Content-Transfer-Encoding: 8bit X-Priority: 3 X-MSMail-Priority: Normal X-Mailer: Microsoft Outlook Express 5.00.2615.200 X-MimeOLE: Produced By Microsoft MimeOLE V5.00.2615.200 Sender: owner-obm-l@sucuri.mat.puc-rio.br Precedence: bulk Reply-To: obm-l@mat.puc-rio.br O algarismo das unidades de qualquer potência de 14 ou é 4 ou é 6. E é 4 para as potências ímpares. 14^14 é um número par. Logo, o alagrismo das unidades pedido é 6. Benedito ----- Original Message ----- From: Claudio Buffara To: Sent: Sunday, April 06, 2003 6:44 PM Subject: Re: [obm-l] último número on 06.04.03 17:32, renatinha15a@bol.com.br at renatinha15a@bol.com.br wrote: > Oi gente, fico grata se alguém me ajudar. > > Determine o último algarismo (algarismo das unidades) do > número 14^(14^14). > > []´s > Renatinha > > > __________________________________________________________________________ > E-mail Premium BOL > Antivírus, anti-spam e até 100 MB de espaço. Assine já! > http://email.bol.com.br/ > > > ========================================================================= > Instruções para entrar na lista, sair da lista e usar a lista em > http://www.mat.puc-rio.br/~nicolau/olimp/obm-l.html > O administrador desta lista é > ========================================================================= > Use o seguinte fato (facilmente demonstrado via propriedades das congruencias): Se os inteiros M e N tem o mesmo algarismo das unidades, entao, para todo inteiro positivo k, M^k e N^k tem o mesmo algarismo das unidades. Isso se representa assim: Se M = N (mod 10) entao M^k = N^k (mod 10) 14 = 4 (mod 10) 14^2 = 4^2 = 16 = 6 (mod 10) 14^3 = 4^3 = 64 = 4 (mod 10) 14^4 = 4^4 = 256 = 6 (mod 10) Ou seja,em geral: k eh par ==> 14^k = 4^k = 6 (mod 10) k eh impar ==> 14^k = 4^k = 4 (mod 10) Como 14^14 eh par, temos que 14^(14^14) = 6 (mod 10) Logo, o algarismo das unidades de 14^(14^14) eh 6. ========================================================================= Instruções para entrar na lista, sair da lista e usar a lista em http://www.mat.puc-rio.br/~nicolau/olimp/obm-l.html O administrador desta lista é ========================================================================= ========================================================================= Instruções para entrar na lista, sair da lista e usar a lista em http://www.mat.puc-rio.br/~nicolau/olimp/obm-l.html O administrador desta lista é ========================================================================= From owner-obm-l@sucuri.mat.puc-rio.br Mon Apr 7 11:36:27 2003 Return-Path: Received: (from majordom@localhost) by sucuri.mat.puc-rio.br (8.9.3/8.9.3) id LAA11041 for obm-l-MTTP; Mon, 7 Apr 2003 11:34:52 -0300 Received: from birosca.ime.usp.br (birosca.ime.usp.br [143.107.45.59]) by sucuri.mat.puc-rio.br (8.9.3/8.9.3) with SMTP id LAA11035 for ; Mon, 7 Apr 2003 11:34:48 -0300 Received: (qmail 11772 invoked from network); 7 Apr 2003 14:34:08 -0000 Received: from rebutosa.ime.usp.br (143.107.45.16) by birosca.ime.usp.br with SMTP; 7 Apr 2003 14:34:08 -0000 Received: (qmail 26172 invoked by uid 1604); 7 Apr 2003 14:34:16 -0000 Date: Mon, 7 Apr 2003 11:34:16 -0300 (EST) From: Salvador Addas Zanata X-Sender: sazanata@rebutosa To: OBM Subject: Re: [obm-l] pontos notaveis In-Reply-To: <20030407134057.64056.qmail@web14309.mail.yahoo.com> Message-ID: MIME-Version: 1.0 Content-Type: TEXT/PLAIN; charset=ISO-8859-1 Content-Transfer-Encoding: 8bit X-MIME-Autoconverted: from QUOTED-PRINTABLE to 8bit by sucuri.mat.puc-rio.br id LAA11038 Sender: owner-obm-l@sucuri.mat.puc-rio.br Precedence: bulk Reply-To: obm-l@mat.puc-rio.br Nao lembro exatamente se esses tres pontos sao os da reta de Euler, mas se forem eles, sao colineares e o do meio, que eu novamente nao lembro qual eh, divide a reta em 2x1. De uma olhada num livrinho chamado "100 great problems of elementary mathematics", autor eh Heinrich Dorrie. La tem uma demonstracao MUITO simples desse resultado, alem de mais 99 coisas fantasticas. Abraco, Salvador On Mon, 7 Apr 2003, Rafael wrote: > Oi pessoal! > > Tenho essa questão aqui que não consigo resovler. Como > parece-me trabalhosa, principalmente para digitar > aqui, agradeço pelo menos umas dicas dos caminhos que > devo seguir: > > Num triangulo ABC, a distância do baricentro ao > ortocentro é igual a 4cm. Então a distãncia do > baricentro ao circuncentro vale: > a)0,5 b)1 c)2 d)4 e)8 > > Valeu! > > Rafael. > > _______________________________________________________________________ > Yahoo! Mail > O melhor e-mail gratuito da internet: 6MB de espaço, antivírus, acesso POP3, filtro contra spam. > http://br.mail.yahoo.com/ > ========================================================================= > Instruções para entrar na lista, sair da lista e usar a lista em > http://www.mat.puc-rio.br/~nicolau/olimp/obm-l.html > O administrador desta lista é > ========================================================================= > ========================================================================= Instruções para entrar na lista, sair da lista e usar a lista em http://www.mat.puc-rio.br/~nicolau/olimp/obm-l.html O administrador desta lista é ========================================================================= From owner-obm-l@sucuri.mat.puc-rio.br Mon Apr 7 11:57:27 2003 Return-Path: Received: (from majordom@localhost) by sucuri.mat.puc-rio.br (8.9.3/8.9.3) id LAA12057 for obm-l-MTTP; Mon, 7 Apr 2003 11:55:25 -0300 Received: from birosca.ime.usp.br (birosca.ime.usp.br [143.107.45.59]) by sucuri.mat.puc-rio.br (8.9.3/8.9.3) with SMTP id LAA12053 for ; Mon, 7 Apr 2003 11:55:22 -0300 Received: (qmail 15033 invoked from network); 7 Apr 2003 14:54:42 -0000 Received: from rebutosa.ime.usp.br (143.107.45.16) by birosca.ime.usp.br with SMTP; 7 Apr 2003 14:54:42 -0000 Received: (qmail 26589 invoked by uid 1604); 7 Apr 2003 14:54:50 -0000 Date: Mon, 7 Apr 2003 11:54:50 -0300 (EST) From: Salvador Addas Zanata X-Sender: sazanata@rebutosa To: obm-l@mat.puc-rio.br Subject: Re: [obm-l] geometria plana In-Reply-To: Message-ID: MIME-Version: 1.0 Content-Type: TEXT/PLAIN; charset=ISO-8859-1 Content-Transfer-Encoding: 8bit X-MIME-Autoconverted: from QUOTED-PRINTABLE to 8bit by sucuri.mat.puc-rio.br id LAA12054 Sender: owner-obm-l@sucuri.mat.puc-rio.br Precedence: bulk Reply-To: obm-l@mat.puc-rio.br O centro do circulo eh o centro do losango. As diagonais do losango se cruzam formando um angulo de 90 graus. Assim o lado do losango mede 15. O circulo e tangente aos lados, assim se voce unir o centro do losango ao ponto de tangencia do circulo, obtem um angulo de 90 graus. Logo o raio do circulo eh a altura ralativa a hipotenusa de um triangulo de lados 9, 12, 15. Abraco, Salvador On Sat, 5 Apr 2003 Faelccmm@aol.com wrote: > Olá pessoal, > > As diagonais de um losango medem 18 cm e 24 cm. Qual é a área do círculo > inscrito neste losango? > > Obs: Eu não consigo achar o raio da circunferência inscrita, fazendo uma > figura dá para perceber que o raio é um pouco menor que 9, já que o lado > menor mede 18. Mas procuro um relação entre estas duas regiões planas, pois > não temos aqui uma ciircunferência inscrita em uma polígono regular. > ========================================================================= Instruções para entrar na lista, sair da lista e usar a lista em http://www.mat.puc-rio.br/~nicolau/olimp/obm-l.html O administrador desta lista é ========================================================================= From owner-obm-l@sucuri.mat.puc-rio.br Mon Apr 7 12:16:55 2003 Return-Path: Received: (from majordom@localhost) by sucuri.mat.puc-rio.br (8.9.3/8.9.3) id MAA12727 for obm-l-MTTP; Mon, 7 Apr 2003 12:14:58 -0300 Received: from ns3bind.localdomain ([200.230.34.5]) by sucuri.mat.puc-rio.br (8.9.3/8.9.3) with ESMTP id MAA12723 for ; Mon, 7 Apr 2003 12:14:54 -0300 Received: from servico2 ([200.230.34.227]) by ns3bind.localdomain (8.11.6/X.XX.X) with SMTP id h37FAmb09461 for ; Mon, 7 Apr 2003 12:10:48 -0300 Message-ID: <008f01c2fd18$7918b520$3300c57d@bovespa.com> From: "=?iso-8859-1?Q?Cl=E1udio_\=28Pr=E1tica\=29?=" To: References: <20030407134057.64056.qmail@web14309.mail.yahoo.com> Subject: Re: [obm-l] pontos notaveis Date: Mon, 7 Apr 2003 12:15:05 -0300 MIME-Version: 1.0 Content-Type: text/plain; charset="iso-8859-1" Content-Transfer-Encoding: 8bit X-Priority: 3 X-MSMail-Priority: Normal X-Mailer: Microsoft Outlook Express 5.50.4920.2300 X-MimeOLE: Produced By Microsoft MimeOLE V5.50.4920.2300 Sender: owner-obm-l@sucuri.mat.puc-rio.br Precedence: bulk Reply-To: obm-l@mat.puc-rio.br Oi, Rafael: Essa sai pela aplicação do teorema da reta de Euler, que diz o seguinte: O baricentro (B), o circuncentro (C) e o ortocentro (H) de um triângulo qualquer são colineares (a reta suporte chama-se Reta de Euler) com B entre C e H e tais que: m(BH) = 2*m(BC). Do enunciado, temos que m(BH) = 4 cm. Logo, m(BC) = m(BH)/2 = 2 cm ==> alternativa (c). Um abraço, Claudio. ----- Original Message ----- From: "Rafael" To: "OBM" Sent: Monday, April 07, 2003 10:40 AM Subject: [obm-l] pontos notaveis > Oi pessoal! > > Tenho essa questão aqui que não consigo resovler. Como > parece-me trabalhosa, principalmente para digitar > aqui, agradeço pelo menos umas dicas dos caminhos que > devo seguir: > > Num triangulo ABC, a distância do baricentro ao > ortocentro é igual a 4cm. Então a distãncia do > baricentro ao circuncentro vale: > a)0,5 b)1 c)2 d)4 e)8 > > Valeu! > > Rafael. ========================================================================= Instruções para entrar na lista, sair da lista e usar a lista em http://www.mat.puc-rio.br/~nicolau/olimp/obm-l.html O administrador desta lista é ========================================================================= From owner-obm-l@sucuri.mat.puc-rio.br Mon Apr 7 12:25:19 2003 Return-Path: Received: (from majordom@localhost) by sucuri.mat.puc-rio.br (8.9.3/8.9.3) id MAA13194 for obm-l-MTTP; Mon, 7 Apr 2003 12:23:59 -0300 Received: from trex.centroin.com.br (trex.centroin.com.br [200.225.63.134]) by sucuri.mat.puc-rio.br (8.9.3/8.9.3) with ESMTP id MAA13190 for ; Mon, 7 Apr 2003 12:23:56 -0300 Received: from trex.centroin.com.br (localhost [127.0.0.1]) by trex.centroin.com.br (8.12.9/8.12.9) with ESMTP id h37FNmGU016021 for ; Mon, 7 Apr 2003 12:23:48 -0300 (EST) Received: (from morgado@localhost) by trex.centroin.com.br (8.12.9/8.12.5/Submit) id h37FNmNU016020; Mon, 7 Apr 2003 12:23:48 -0300 (EST) Message-Id: <200304071523.h37FNmNU016020@trex.centroin.com.br> Received: from 200.225.58.148 by trex.centroin.com.br (CIPWM versao 1.4C1) with HTTPS for ; Mon, 7 Apr 2003 12:23:48 -0300 (EST) Date: Mon, 7 Apr 2003 12:23:48 -0300 (EST) From: Augusto Cesar de Oliveira Morgado To: obm-l@mat.puc-rio.br Subject: Re: [obm-l] pontos notaveis MIME-Version: 1.0 X-Mailer: CentroIn Internet Provider WebMail v. 1.4C1 (http://www.centroin.com.br/) Content-Type: text/plain; charset="iso-8859-1" Content-Transfer-Encoding: 8bit X-MIME-Autoconverted: from quoted-printable to 8bit by sucuri.mat.puc-rio.br id MAA13191 Sender: owner-obm-l@sucuri.mat.puc-rio.br Precedence: bulk Reply-To: obm-l@mat.puc-rio.br Ha um artigo meu na RPM sobre isso. O titulo eh "coordenadas para os centros dos triangulos". Procure nos arquivos da lista que voce vai encontrar uma discussao sobre isso, se nao me engano no fim do ano passado. Em Mon, 7 Apr 2003 10:40:56 -0300 (ART), Rafael disse: > Oi pessoal! > > Tenho essa questão aqui que não consigo resovler. Como > parece-me trabalhosa, principalmente para digitar > aqui, agradeço pelo menos umas dicas dos caminhos que > devo seguir: > > Num triangulo ABC, a distância do baricentro ao > ortocentro é igual a 4cm. Então a distãncia do > baricentro ao circuncentro vale: > a)0,5 b)1 c)2 d)4 e)8 > > Valeu! > > Rafael. > > _______________________________________________________________________ > Yahoo! Mail > O melhor e-mail gratuito da internet: 6MB de espaço, antivírus, acesso POP3, filtro contra spam. > http://br.mail.yahoo.com/ > ========================================================================= > Instruções para entrar na lista, sair da lista e usar a lista em > http://www.mat.puc-rio.br/~nicolau/olimp/obm-l.html > O administrador desta lista é > ========================================================================= > > ========================================================================= Instruções para entrar na lista, sair da lista e usar a lista em http://www.mat.puc-rio.br/~nicolau/olimp/obm-l.html O administrador desta lista é ========================================================================= From owner-obm-l@sucuri.mat.puc-rio.br Mon Apr 7 13:54:17 2003 Return-Path: Received: (from majordom@localhost) by sucuri.mat.puc-rio.br (8.9.3/8.9.3) id NAA16198 for obm-l-MTTP; Mon, 7 Apr 2003 13:52:11 -0300 Received: from sidney8.bol.com.br (sidney8.bol.com.br [200.221.24.108]) by sucuri.mat.puc-rio.br (8.9.3/8.9.3) with ESMTP id NAA16194 for ; Mon, 7 Apr 2003 13:52:01 -0300 Received: from bol.com.br (200.221.24.130) by sidney8.bol.com.br (5.1.071) id 3E8D10A30009B569 for obm-l@mat.puc-rio.br; Mon, 7 Apr 2003 13:51:31 -0300 Date: Mon, 7 Apr 2003 13:51:30 -0300 Message-Id: Subject: [obm-l] =?iso-8859-1?q?divis=E3o_exata?= MIME-Version: 1.0 Content-Type: text/plain;charset="iso-8859-1" From: "marcelo.paiva.jr4" To: obm-l@mat.puc-rio.br X-XaM3-API-Version: 2.4 R3 ( B4 ) X-SenderIP: 200.241.108.194 Content-Transfer-Encoding: 8bit X-MIME-Autoconverted: from quoted-printable to 8bit by sucuri.mat.puc-rio.br id NAA16195 Sender: owner-obm-l@sucuri.mat.puc-rio.br Precedence: bulk Reply-To: obm-l@mat.puc-rio.br Oi pessoal, gostaria de saber como provar algebricamente esta questão: Numa divisão, cujo resto não é nulo, o menor número que se deve adicionar ao dividendo para que se torne exata é: (d - r), sendo d o divisor e r o resto. Testando-se com valores aleatórios, se constata que essa afirmação é verdadeira. Mas quando fui tentar de forma algébrica, não consegui provar. Eu fiz o seguinte: (D = dividendo, d = divisor, q = quociente e r = resto) ==> D = d*q + r ==> D + (d - r) = d*q + r ==> D = d*q + r - d + r ==> D = d(q - 1) + 2r (parei aqui) Estou errado, pois para que a divisão seja exata "r" tem que ser iqual a zero. Qual é a resposta mais adequada? obrigado pela atenção de todos. __________________________________________________________________________ E-mail Premium BOL Antivírus, anti-spam e até 100 MB de espaço. Assine já! http://email.bol.com.br/ ========================================================================= Instruções para entrar na lista, sair da lista e usar a lista em http://www.mat.puc-rio.br/~nicolau/olimp/obm-l.html O administrador desta lista é ========================================================================= From owner-obm-l@sucuri.mat.puc-rio.br Mon Apr 7 14:48:08 2003 Return-Path: Received: (from majordom@localhost) by sucuri.mat.puc-rio.br (8.9.3/8.9.3) id OAA17521 for obm-l-MTTP; Mon, 7 Apr 2003 14:46:05 -0300 Received: from cmsrelay04.mx.net (cmsrelay04.mx.net [165.212.11.113]) by sucuri.mat.puc-rio.br (8.9.3/8.9.3) with SMTP id OAA17515 for ; Mon, 7 Apr 2003 14:45:59 -0300 Received: from cmsapps02.cms.usa.net (HELO localhost) (165.212.11.138) by cmsoutbound.mx.net with SMTP; 7 Apr 2003 17:45:25 -0000 Received: from smtp.postoffice.net [165.212.8.22] by cmsapps02.cms.usa.net (ASMTP/) via mtad (C8.MAIN.2.05) with ESMTP id 273HDgRty0334M38; Mon, 07 Apr 2003 17:45:24 GMT Received: from 200.181.4.100 [200.181.4.100] by uwdvg022.cms.usa.net (USANET web-mailer CM.0402.5.2B); Mon, 07 Apr 2003 17:45:23 -0000 Date: Mon, 07 Apr 2003 14:45:23 -0300 From: Artur Costa Steiner To: Subject: [obm-l] =?ISO-8859-1?Q?Re=3A=20=5B=5Bobm=2Dl=5D=20divis=E3o=20exa?= =?ISO-8859-1?Q?ta=5D?= X-Mailer: USANET web-mailer (CM.0402.5.2B) Mime-Version: 1.0 Message-ID: <299HDgRtX9312S22.1049737523@uwdvg022.cms.usa.net> Content-Type: text/plain; charset=ISO-8859-1 Content-Transfer-Encoding: 8bit X-MIME-Autoconverted: from quoted-printable to 8bit by sucuri.mat.puc-rio.br id OAA17517 Sender: owner-obm-l@sucuri.mat.puc-rio.br Precedence: bulk Reply-To: obm-l@mat.puc-rio.br Oi Marcelo, Temos que D = d*q + r. Adicionando-se s a D, desejamos ter D+s = d*q1. Logo, s = dq1 - dq -r e q1 = q + (s+r)/d. Como desejamos que q1 seja inteiro e r>0, o menor valor de s que satisfaz é tal que s+r = d. Logo, s = d-r. Você começou certo. Artur "> > Numa divisão, cujo resto não é nulo, o menor número que > se deve adicionar ao dividendo para que se torne exata > é: (d - r), sendo d o divisor e r o resto. > > Testando-se com valores aleatórios, se > constata que essa afirmação é verdadeira. Mas quando fui > tentar de forma algébrica, não consegui provar. Eu fiz o > seguinte: (D = dividendo, d = divisor, q = quociente e r > = resto) > ==> D = d*q + r ==> D + (d - r) = d*q + r ==> D = d*q + > r - d + r ==> D = d(q - 1) + 2r (parei aqui) > Estou errado, pois para que a divisão seja exata "r" tem > que ser iqual a zero. > Qual é a resposta mais adequada? > > obrigado pela atenção de todos. ========================================================================= Instruções para entrar na lista, sair da lista e usar a lista em http://www.mat.puc-rio.br/~nicolau/olimp/obm-l.html O administrador desta lista é ========================================================================= From owner-obm-l@sucuri.mat.puc-rio.br Mon Apr 7 14:48:09 2003 Return-Path: Received: (from majordom@localhost) by sucuri.mat.puc-rio.br (8.9.3/8.9.3) id OAA17527 for obm-l-MTTP; Mon, 7 Apr 2003 14:46:06 -0300 Received: from web14305.mail.yahoo.com (web14305.mail.yahoo.com [216.136.173.81]) by sucuri.mat.puc-rio.br (8.9.3/8.9.3) with SMTP id OAA17516 for ; Mon, 7 Apr 2003 14:46:00 -0300 Message-ID: <20030407174528.92363.qmail@web14305.mail.yahoo.com> Received: from [200.144.49.41] by web14305.mail.yahoo.com via HTTP; Mon, 07 Apr 2003 10:45:28 PDT Date: Mon, 7 Apr 2003 10:45:28 -0700 (PDT) From: Rafael Subject: [obm-l] circunferencias To: OBM MIME-Version: 1.0 Content-Type: text/plain; charset=us-ascii Sender: owner-obm-l@sucuri.mat.puc-rio.br Precedence: bulk Reply-To: obm-l@mat.puc-rio.br Pessoal, tem essa questão que já fiz duas vezes e a resposta não bate com a que eu tenho aqui. Talvez eu não esteja conseguido escrever a resposta que eu encontrei de outra maneira para ficar como a que eu tenho. Vejam: Duas circunferências de raios R e r são tangentes externamente em A. Traça-se a tangente comum externa BC (B e C pontos de contato). Calcular o raio da circunferência inscrita no triângulo mistilíneo ABC. resp: Rr/(R+r+2raiz(Rr)) A resposta que eu chego é: Rr/[raiz(R+r).(raiz(R) + raiz(r) + raiz(R+r))] Alguém consegue transformar uma resposta na outra?? Abraços, Rafael. __________________________________________________ Do you Yahoo!? Yahoo! Tax Center - File online, calculators, forms, and more http://tax.yahoo.com ========================================================================= Instruções para entrar na lista, sair da lista e usar a lista em http://www.mat.puc-rio.br/~nicolau/olimp/obm-l.html O administrador desta lista é ========================================================================= From owner-obm-l@sucuri.mat.puc-rio.br Mon Apr 7 15:10:49 2003 Return-Path: Received: (from majordom@localhost) by sucuri.mat.puc-rio.br (8.9.3/8.9.3) id PAA18403 for obm-l-MTTP; Mon, 7 Apr 2003 15:08:17 -0300 Received: from mail.fronthost.com (mail.fronthost.com [63.250.6.253]) by sucuri.mat.puc-rio.br (8.9.3/8.9.3) with ESMTP id PAA18388 for ; Mon, 7 Apr 2003 15:08:12 -0300 Received: from [68.49.44.134] by fronthost.com [63.250.6.253] with SmartMax MailMax for at Mon, 07 Apr 2003 14:08:02 -0400 Message-ID: <011101c2fd30$53efd7c0$6501a8c0@TEST4> From: "Alexandre A da Rocha" To: References: Subject: [obm-l] =?iso-8859-1?Q?Re:_=5Bobm-l=5D_divis=E3o_exata?= Date: Mon, 7 Apr 2003 14:05:49 -0400 MIME-Version: 1.0 Content-Type: text/plain; charset="iso-8859-1" Content-Transfer-Encoding: 8bit X-Priority: 3 X-MSMail-Priority: Normal X-Mailer: Microsoft Outlook Express 5.50.4807.1700 X-MimeOLE: Produced By Microsoft MimeOLE V5.50.4807.1700 Sender: owner-obm-l@sucuri.mat.puc-rio.br Precedence: bulk Reply-To: obm-l@mat.puc-rio.br Marcelo, Talvez seja so falta de atencao....vamos ver Se D = d*q + r , e supondo que realmente adicionar ( d - r ) elimina o resto ==> D + ( d - r ) = d*q + r ... nao esta certo... o certo seria: D + ( d - r ) = d*(q + 1) + 0 ( ja que o prosto e justamente eliminar o resto ) continuando... D + d - r = d*q + d ==> D = d*q + r +d -d ==> D = d*q + r ==> verdadeiro Acho que e o suficiente pra provar, favor replicar se tiver furos -Auggy ----- Original Message ----- From: "marcelo.paiva.jr4" To: Sent: Monday, April 07, 2003 12:51 PM Subject: [obm-l] divisão exata Oi pessoal, gostaria de saber como provar algebricamente esta questão: Numa divisão, cujo resto não é nulo, o menor número que se deve adicionar ao dividendo para que se torne exata é: (d - r), sendo d o divisor e r o resto. Testando-se com valores aleatórios, se constata que essa afirmação é verdadeira. Mas quando fui tentar de forma algébrica, não consegui provar. Eu fiz o seguinte: (D = dividendo, d = divisor, q = quociente e r = resto) ==> D = d*q + r ==> D + (d - r) = d*q + r ==> D = d*q + r - d + r ==> D = d(q - 1) + 2r (parei aqui) Estou errado, pois para que a divisão seja exata "r" tem que ser iqual a zero. Qual é a resposta mais adequada? obrigado pela atenção de todos. __________________________________________________________________________ E-mail Premium BOL Antivírus, anti-spam e até 100 MB de espaço. Assine já! http://email.bol.com.br/ ========================================================================= Instruções para entrar na lista, sair da lista e usar a lista em http://www.mat.puc-rio.br/~nicolau/olimp/obm-l.html O administrador desta lista é ========================================================================= ========================================================================= Instruções para entrar na lista, sair da lista e usar a lista em http://www.mat.puc-rio.br/~nicolau/olimp/obm-l.html O administrador desta lista é ========================================================================= From owner-obm-l@sucuri.mat.puc-rio.br Mon Apr 7 15:14:16 2003 Return-Path: Received: (from majordom@localhost) by sucuri.mat.puc-rio.br (8.9.3/8.9.3) id PAA18588 for obm-l-MTTP; Mon, 7 Apr 2003 15:12:27 -0300 Received: from shannon.bol.com.br (shannon.bol.com.br [200.221.24.13]) by sucuri.mat.puc-rio.br (8.9.3/8.9.3) with ESMTP id PAA18584 for ; Mon, 7 Apr 2003 15:12:19 -0300 From: renatinha15a@bol.com.br Received: from bol.com.br (200.221.24.130) by shannon.bol.com.br (5.1.071) id 3E76735F0067FC47 for obm-l@mat.puc-rio.br; Mon, 7 Apr 2003 15:11:47 -0300 Date: Mon, 7 Apr 2003 15:11:46 -0300 Message-Id: Subject: [obm-l] =?iso-8859-1?q?base_da_fun=E7=E3o_exponencial?= MIME-Version: 1.0 Content-Type: text/plain;charset="iso-8859-1" To: "obm" X-XaM3-API-Version: 2.4 R3 ( B4 ) X-SenderIP: 200.241.108.194 Content-Transfer-Encoding: 8bit X-MIME-Autoconverted: from quoted-printable to 8bit by sucuri.mat.puc-rio.br id PAA18585 Sender: owner-obm-l@sucuri.mat.puc-rio.br Precedence: bulk Reply-To: obm-l@mat.puc-rio.br oi pessoal da lista, estou com uma dúvida bem básica. Por que a base "a" de uma função exponencial não pode pertencer a R* - {+-1}? []´s Renatinha __________________________________________________________________________ E-mail Premium BOL Antivírus, anti-spam e até 100 MB de espaço. Assine já! http://email.bol.com.br/ ========================================================================= Instruções para entrar na lista, sair da lista e usar a lista em http://www.mat.puc-rio.br/~nicolau/olimp/obm-l.html O administrador desta lista é ========================================================================= From owner-obm-l@sucuri.mat.puc-rio.br Mon Apr 7 16:19:57 2003 Return-Path: Received: (from majordom@localhost) by sucuri.mat.puc-rio.br (8.9.3/8.9.3) id QAA21302 for obm-l-MTTP; Mon, 7 Apr 2003 16:17:22 -0300 Received: from cmsrelay05.mx.net (cmsrelay05.mx.net [165.212.11.2]) by sucuri.mat.puc-rio.br (8.9.3/8.9.3) with SMTP id QAA21298 for ; Mon, 7 Apr 2003 16:17:17 -0300 Received: from cmsapps02.cms.usa.net (HELO localhost) (165.212.11.138) by cmsoutbound.mx.net with SMTP; 7 Apr 2003 19:16:43 -0000 Received: from smtp.postoffice.net [165.212.8.17] by cmsapps02.cms.usa.net (ASMTP/) via mtad (C8.MAIN.2.05) with ESMTP id 787HDgTqp0436M38; Mon, 07 Apr 2003 19:16:41 GMT Received: from 200.181.4.100 [200.181.4.100] by uwdvg017.cms.usa.net (USANET web-mailer CM.0402.5.2B); Mon, 07 Apr 2003 19:16:40 -0000 Date: Mon, 07 Apr 2003 16:16:40 -0300 From: Artur Costa Steiner To: Subject: [obm-l] =?ISO-8859-1?Q?Re=3A=20=5B=5Bobm=2Dl=5D=20base=20da=20fun?= =?ISO-8859-1?Q?=E7=E3o=20exponencial=5D?= X-Mailer: USANET web-mailer (CM.0402.5.2B) Mime-Version: 1.0 Message-ID: <167HDgTqO1552S17.1049743000@uwdvg017.cms.usa.net> Content-Type: text/plain; charset=ISO-8859-1 Content-Transfer-Encoding: 8bit X-MIME-Autoconverted: from quoted-printable to 8bit by sucuri.mat.puc-rio.br id QAA21299 Sender: owner-obm-l@sucuri.mat.puc-rio.br Precedence: bulk Reply-To: obm-l@mat.puc-rio.br Bom , na realidade devemos ter a>0 e a<>1. Tudo começa com a definicão da função exponencial da base e, dada por E(x) = 1 + x +x^2/2!...=..x^n/n!..., isto é, por uma série de potências (esta definição vale inclusive para números complexos). Na reta real, podemos provar que E é estritamente positiva e estritamente crescente. Além disto, é imediato que E(0) =1. Temos então a função L, log neperiano, definida, para y>0, como a inversa de E. Para todo real y, definimos "a elevado a y", por a^y = E(y La), o que só faz sentido para a>0. L não é definida para valores negativos ou para a =0, visto que E é estritamente positiva na reta real. Se a =1, observamos que a^y torna-se constante e igual a 1, pois L(1) = 0, uma vez que E(0) =1. . Embora isso faça sentido, não é uma função muito nteressante e, por isso, normalmente não se definem exponenciais de base a para a =1. Artur renatinha15a@bol.com.br wrote: > oi pessoal da lista, estou com uma dúvida bem básica. > Por que a base "a" de uma função exponencial não pode > pertencer a R* - {+-1}? > > []´s ========================================================================= Instruções para entrar na lista, sair da lista e usar a lista em http://www.mat.puc-rio.br/~nicolau/olimp/obm-l.html O administrador desta lista é ========================================================================= From owner-obm-l@sucuri.mat.puc-rio.br Mon Apr 7 17:29:51 2003 Return-Path: Received: (from majordom@localhost) by sucuri.mat.puc-rio.br (8.9.3/8.9.3) id RAA23094 for obm-l-MTTP; Mon, 7 Apr 2003 17:27:48 -0300 Received: from hotmail.com (oe57.law10.hotmail.com [64.4.14.192]) by sucuri.mat.puc-rio.br (8.9.3/8.9.3) with ESMTP id RAA23085 for ; Mon, 7 Apr 2003 17:27:43 -0300 Received: from mail pickup service by hotmail.com with Microsoft SMTPSVC; Mon, 7 Apr 2003 13:27:11 -0700 Received: from 64.60.139.18 by oe57.law10.hotmail.com with DAV; Mon, 07 Apr 2003 20:27:10 +0000 X-Originating-IP: [64.60.139.18] X-Originating-Email: [lrecova@hotmail.com] From: =?iso-8859-1?Q?Leandro_Lacorte_Rec=F4va?= To: Subject: [obm-l] =?iso-8859-1?Q?RE:_=5Bobm-l=5D_Re:_=5B=5Bobm-l=5D_base_da_fun=E7=E3o_expo?= =?iso-8859-1?Q?nencial=5D?= Date: Mon, 7 Apr 2003 13:27:13 -0700 Message-ID: <003d01c2fd44$128ae090$28029b9b@LeandroRecova> MIME-Version: 1.0 Content-Type: text/plain; charset="iso-8859-1" X-Priority: 3 (Normal) X-MSMail-Priority: Normal X-Mailer: Microsoft Outlook, Build 10.0.3416 X-MimeOLE: Produced By Microsoft MimeOLE V6.00.2800.1106 Importance: Normal In-Reply-To: <167HDgTqO1552S17.1049743000@uwdvg017.cms.usa.net> X-OriginalArrivalTime: 07 Apr 2003 20:27:11.0313 (UTC) FILETIME=[1181FC10:01C2FD44] Content-Transfer-Encoding: 8bit X-MIME-Autoconverted: from quoted-printable to 8bit by sucuri.mat.puc-rio.br id RAA23091 Sender: owner-obm-l@sucuri.mat.puc-rio.br Precedence: bulk Reply-To: obm-l@mat.puc-rio.br Arthur, Tenho certeza que ela ainda nao viu o conceito de serie de potencias. -----Original Message----- From: owner-obm-l@sucuri.mat.puc-rio.br [mailto:owner-obm-l@sucuri.mat.puc-rio.br] On Behalf Of Artur Costa Steiner Sent: Monday, April 07, 2003 12:17 PM To: obm-l@mat.puc-rio.br Subject: [obm-l] Re: [[obm-l] base da função exponencial] Bom , na realidade devemos ter a>0 e a<>1. Tudo começa com a definicão da função exponencial da base e, dada por E(x) = 1 + x +x^2/2!...=..x^n/n!..., isto é, por uma série de potências (esta definição vale inclusive para números complexos). Na reta real, podemos provar que E é estritamente positiva e estritamente crescente. Além disto, é imediato que E(0) =1. Temos então a função L, log neperiano, definida, para y>0, como a inversa de E. Para todo real y, definimos "a elevado a y", por a^y = E(y La), o que só faz sentido para a>0. L não é definida para valores negativos ou para a =0, visto que E é estritamente positiva na reta real. Se a =1, observamos que a^y torna-se constante e igual a 1, pois L(1) = 0, uma vez que E(0) =1. . Embora isso faça sentido, não é uma função muito nteressante e, por isso, normalmente não se definem exponenciais de base a para a =1. Artur renatinha15a@bol.com.br wrote: > oi pessoal da lista, estou com uma dúvida bem básica. > Por que a base "a" de uma função exponencial não pode > pertencer a R* - {+-1}? > > []´s ======================================================================== = Instruções para entrar na lista, sair da lista e usar a lista em http://www.mat.puc-rio.br/~nicolau/olimp/obm-l.html O administrador desta lista é ======================================================================== = ========================================================================= Instruções para entrar na lista, sair da lista e usar a lista em http://www.mat.puc-rio.br/~nicolau/olimp/obm-l.html O administrador desta lista é ========================================================================= From owner-obm-l@sucuri.mat.puc-rio.br Mon Apr 7 17:37:15 2003 Return-Path: Received: (from majordom@localhost) by sucuri.mat.puc-rio.br (8.9.3/8.9.3) id RAA23334 for obm-l-MTTP; Mon, 7 Apr 2003 17:35:40 -0300 Received: from Euler.impa.br (euler.impa.br [147.65.1.3]) by sucuri.mat.puc-rio.br (8.9.3/8.9.3) with ESMTP id RAA23330 for ; Mon, 7 Apr 2003 17:35:37 -0300 Received: from Gauss.impa.br (Gauss [147.65.4.1]) by Euler.impa.br (8.11.6p2/8.11.6) with ESMTP id h37KZ7215517 for ; Mon, 7 Apr 2003 17:35:07 -0300 (EST) From: Carlos Gustavo Tamm de Araujo Moreira Received: by Gauss.impa.br (8.11.6p2) id h37KZ6f13307; Mon, 7 Apr 2003 17:35:06 -0300 (EST) Message-Id: <200304072035.h37KZ6f13307@Gauss.impa.br> Subject: Re: [obm-l] =?iso-8859-1?q?base_da_fun=E7=E3o_exponencial?= To: obm-l@mat.puc-rio.br Date: Mon, 7 Apr 2003 17:35:05 -0300 (EST) In-Reply-To: from "renatinha15a@bol.com.br" at Apr 7, 3 03:11:46 pm X-Mailer: ELM [version 2.4 PL25] MIME-Version: 1.0 Content-Type: text/plain; charset=US-ASCII Content-Transfer-Encoding: 7bit Sender: owner-obm-l@sucuri.mat.puc-rio.br Precedence: bulk Reply-To: obm-l@mat.puc-rio.br Cara Renatinha, O problema principal e' que nao e' obvio como definir potencias de base negativa com expoente irracional. Por exemplo: o que e' (-2)^pi ? Se usarmos complexos obteremos infinitos valores, sem razao clara para escolher um deles... Abracos, Gugu > >oi pessoal da lista, estou com uma dúvida bem básica. >Por que a base "a" de uma função exponencial não pode >pertencer a R* - {+-1}? > >[]´s > Renatinha > > >__________________________________________________________________________ >E-mail Premium BOL >Antivírus, anti-spam e até 100 MB de espaço. Assine já! >http://email.bol.com.br/ > > >========================================================================= >Instruções para entrar na lista, sair da lista e usar a lista em >http://www.mat.puc-rio.br/~nicolau/olimp/obm-l.html >O administrador desta lista é >========================================================================= ========================================================================= Instruções para entrar na lista, sair da lista e usar a lista em http://www.mat.puc-rio.br/~nicolau/olimp/obm-l.html O administrador desta lista é ========================================================================= From owner-obm-l@sucuri.mat.puc-rio.br Mon Apr 7 17:57:01 2003 Return-Path: Received: (from majordom@localhost) by sucuri.mat.puc-rio.br (8.9.3/8.9.3) id RAA24358 for obm-l-MTTP; Mon, 7 Apr 2003 17:55:31 -0300 Received: from Euler.impa.br (euler.impa.br [147.65.1.3]) by sucuri.mat.puc-rio.br (8.9.3/8.9.3) with ESMTP id RAA24352 for ; Mon, 7 Apr 2003 17:55:28 -0300 Received: from Gauss.impa.br (Gauss [147.65.4.1]) by Euler.impa.br (8.11.6p2/8.11.6) with ESMTP id h37Ksv216806 for ; Mon, 7 Apr 2003 17:54:57 -0300 (EST) From: Carlos Gustavo Tamm de Araujo Moreira Received: by Gauss.impa.br (8.11.6p2) id h37Ksvu15145; Mon, 7 Apr 2003 17:54:57 -0300 (EST) Message-Id: <200304072054.h37Ksvu15145@Gauss.impa.br> Subject: Re: [obm-l] Rearranjo generalizado - revisado To: obm-l@mat.puc-rio.br Date: Mon, 7 Apr 2003 17:54:57 -0300 (EST) In-Reply-To: from "Claudio Buffara" at Apr 2, 3 00:02:38 am X-Mailer: ELM [version 2.4 PL25] MIME-Version: 1.0 Content-Type: text/plain; charset=US-ASCII Content-Transfer-Encoding: 7bit Sender: owner-obm-l@sucuri.mat.puc-rio.br Precedence: bulk Reply-To: obm-l@mat.puc-rio.br Oi pessoal, A gente discutiu esse problema hoje no treinamento do IMPA. A solucao do Claudio parece muito com a do Alex. A minha e' um pouco diferente, e segue dos seguintes lemas (as provas sao faceis e deixo como exercicio): Def: (a_1,a_2,...,a_n)<<(b_1,b_2,...,b_n) se para todo k<=n, a_k+a_(k+1)+...+a_n<=b_k+b_(k+1)+...+b_n. Lema 1: dadas duas permutacoes (i1,i2,...,in) e (j1,j2,...,jn) de (1,2,...,n) e duas sequencias nao-decrescentes (a_1,a_2,..,a_n) e (b_1,b_2,...,b_n) temos (a_i1.b_j1,a_i2.b_j2,...,a_in.b_jn)<<(a_1.b_1,a_2.b_2,....,a_n.b_n). Lema 2: se (a_1,a_2,...,a_n)<<(b_1,b_2,...,b_n) e (c_1,c_2,...,c_n) e' uma sequencia nao-decrescente entao (c_1.a_1,c_2.a_2,...,c_n.a_n)<<(c_1.b_1,c_2.b_2,...,c_n.b_n). Esses lemas implicam um resultado um pouco mais forte: a sequencia dos produtos com todas as sequencias na ordem certa e' maxima na ordem <<. Abracos, Gugu P.S.: As sequencias acima sao todas positivas. Vale a pena notar que o resultado com duas sequencias vale mesmo sem supor que os termos sao nao-negativos, mas em geral precisamos disso (como mostra o exemplo das sequencias (-1,-1,1),(-1,-1,1),(-1,-1,1)). > >Oi, Marcio: > >Dei uma boa mexida na demonstracao e eliminei aquela passagem nao >justificada. Por favor de uma lida e me diga se ficou algum furo. > >O problema: > >Sejam varias seqs de termos positivos (a), (b), (c), ...e considere as somas >do tipo S = a_1*b_1*c_1*... +a_2*b_2*c_2* ... + ... a_n*b_n*c_n*... onde >(a_i) eh uma permutacao da 1a sequencia, (b_i) uma permutacao da 2a, e assim >por diante. > Mostre que S é máxima quando as sequencias tem a mesma ordenacao. > O caso com 2 sequencias eh o que se conhece como "desigualdade do >rearranjo" > >----------- > >Inducao sobre o numero M de sequencias (M >= 2): > >O caso base (M = 2) eh a desigualdade do rearranjo. > >Supondo que o resultado seja verdadeiro para quaisquer M-1 sequencias (M >= >3) de termos positivos, consideremos as M sequencias (A_i), (B_i), (C_i), >..., (Z_i) (achei melhor usar esta notacao do que dois indices) de termos >positivos e as somas correspondentes do tipo: >S = A_1*B_1*...*Z_1 + ... + A_n*B_n*...*Z_n > >Inicialmente, aplicamos a hipotese de inducao as M-1 sequencias (A_i*B_i), >(C_i), ..., (Z_i) e concluimos que S eh maxima quando todas estas as >sequencias tem a mesma ordenacao, digamos: >0 < A_1*B_1 <= ... <= A_n*B_n, >0 < C_1 <= ... <= C_n, >... >0 < Z_1 <= ... <= Z_n. > >O objetivo agora eh provar que: >0 < A_1 <= ... <= A_n e 0 < B_1 <= ... <= B_n > >Suponhamos, portanto, que este nao seja o caso. >Sem perda de generalidade podemos supor que existem i, j tais que: >1 <= i < j <= n e A_i > A_j. > >Nesse caso, como A_i*B_i <= A_j*B_j, teremos que B_i < B_j, o que, >juntamente com as outras desigualdades decorrentes da h.i., implica que: >B_i*...*Z_i < B_j*...*Z_j. > >Agora, vamos calcular o valor da soma S1, a qual eh obtida de S pela >permutacao de A_i e A_j (todos os demais termos ficam iguais). Assim: > >S1 = S - A_i*B_i*...*Z_i - A_j*B_j*...*Z_j + A_j*B_i*...*Z_i + >A_i*B_j*...*Z_j ==> > >S1 = S + (A_i - A_j)*(B_j*...*Z_j - B_i*...*Z_i) > >Como A_i > A_j e B_j*...*Z_j > B_i*...*Z_i, temos que S1 > S. > >Ou seja, colocando em ordem crescente dois termos originalmente "fora de >ordem" da sequncia (A_i), conseguimos aumentar o valor da soma. O mesmo >raciocinio vale para eventuais termos "fora de ordem" da sequencia (B_i). > >Portanto, a soma eh maxima quando (A_i) e (B_i) estao em ordem crescente. > >Em virtude da h.i., podemos concluir que S eh maxima quando: >0 < A_1 <= ... <= A_n, >0 < B_1 <= ... <= B_n, >0 < C_1 <= ... <= C_n, >... >0 < Z_1 <= ... <= Z_n, >ou seja, quando as M sequencias tiverem a a mesma ordenacao. > >FIM > > >Um abraco, >Claudio. > > >========================================================================= >Instruções para entrar na lista, sair da lista e usar a lista em >http://www.mat.puc-rio.br/~nicolau/olimp/obm-l.html >O administrador desta lista é >========================================================================= ========================================================================= Instruções para entrar na lista, sair da lista e usar a lista em http://www.mat.puc-rio.br/~nicolau/olimp/obm-l.html O administrador desta lista é ========================================================================= From owner-obm-l@sucuri.mat.puc-rio.br Mon Apr 7 18:01:11 2003 Return-Path: Received: (from majordom@localhost) by sucuri.mat.puc-rio.br (8.9.3/8.9.3) id RAA24472 for obm-l-MTTP; Mon, 7 Apr 2003 17:59:50 -0300 Received: from shannon.bol.com.br (shannon.bol.com.br [200.221.24.13]) by sucuri.mat.puc-rio.br (8.9.3/8.9.3) with ESMTP id RAA24466 for ; Mon, 7 Apr 2003 17:59:45 -0300 Received: from bol.com.br (200.221.24.132) by shannon.bol.com.br (5.1.071) id 3E76735F00691279 for obm-l@mat.puc-rio.br; Mon, 7 Apr 2003 17:59:14 -0300 Date: Mon, 7 Apr 2003 17:59:14 -0300 Message-Id: Subject: [obm-l] =?iso-8859-1?q?fun=E7=E3o_exponencial_=28de_novo=29?= MIME-Version: 1.0 Content-Type: text/plain;charset="iso-8859-1" From: "renatinha15a" To: "obm" X-XaM3-API-Version: 2.4 R3 ( B4 ) X-SenderIP: 200.241.108.194 Content-Transfer-Encoding: 8bit X-MIME-Autoconverted: from quoted-printable to 8bit by sucuri.mat.puc-rio.br id RAA24467 Sender: owner-obm-l@sucuri.mat.puc-rio.br Precedence: bulk Reply-To: obm-l@mat.puc-rio.br Primeiramente, obrigada Artur Costa pela explicação sobre a base da função exponencial -confesso que ainda me falta conhecimento matemático para entende-la, mas estou batalhando para isso.- Estudo pelos livros fundamentos de mat. elementar, e não entendi uma parte da solução proposta pelo livro (f. exponencial/logaritimo, vol. 2). Fico grata por qualquer esclarecimento. Resolva as equações em R+: a) x^(x^2 - 5x + 6) = 1 [SOLUÇÃO DO LIVRO]: Devemos examinar inicialmente se 0 ou 1 são soluções da equação. Substituindo x = 0 na equação proposta, temos: 0^6 = 1 (falso) logo, 0 não é solução. Substituindo x = 1 na equação, temos: 1^2 = 1 (verdadeiro) logo 1 é solução da equação. [DÚVIDA] Por que examinar inicialmente se 0 e 1 são soluções? Digo isso pois, se x = a (a = base), então temos 0 < x =/= 1. (=/= "diferente") []´s Renatinha __________________________________________________________________________ E-mail Premium BOL Antivírus, anti-spam e até 100 MB de espaço. Assine já! http://email.bol.com.br/ ========================================================================= Instruções para entrar na lista, sair da lista e usar a lista em http://www.mat.puc-rio.br/~nicolau/olimp/obm-l.html O administrador desta lista é ========================================================================= From owner-obm-l@sucuri.mat.puc-rio.br Mon Apr 7 18:05:13 2003 Return-Path: Received: (from majordom@localhost) by sucuri.mat.puc-rio.br (8.9.3/8.9.3) id SAA24666 for obm-l-MTTP; Mon, 7 Apr 2003 18:03:41 -0300 Received: from cmsrelay03.mx.net (cmsrelay03.mx.net [165.212.11.112]) by sucuri.mat.puc-rio.br (8.9.3/8.9.3) with SMTP id SAA24659 for ; Mon, 7 Apr 2003 18:03:36 -0300 Received: from uadvg130.cms.usa.net (HELO localhost) (165.212.11.130) by cmsoutbound.mx.net with SMTP; 7 Apr 2003 21:03:02 -0000 Received: from smtp.postoffice.net [165.212.8.7] by uadvg130.cms.usa.net (ASMTP/) via mtad (C8.MAIN.2.05) with ESMTP id 612HDgVDc0054M30; Mon, 07 Apr 2003 21:03:03 GMT Received: from 200.181.4.100 [200.181.4.100] by uwdvg007.cms.usa.net (USANET web-mailer CM.0402.5.2B); Mon, 07 Apr 2003 21:03:02 -0000 Date: Mon, 07 Apr 2003 18:03:02 -0300 From: Artur Costa Steiner To: Subject: [obm-l] =?ISO-8859-1?Q?Re=3A=20=5B=5Bobm=2Dl=5D=20RE=3A=20=5Bobm=2Dl=5D=20?= =?ISO-8859-1?Q?Re=3A=20=5B=5Bobm=2Dl=5D=20base=20da=20fun?= =?ISO-8859-1?Q?=E7=E3o=20exponencial=5D=5D?= X-Mailer: USANET web-mailer (CM.0402.5.2B) Mime-Version: 1.0 Message-ID: <161HDgVDc3184S07.1049749382@uwdvg007.cms.usa.net> Content-Type: text/plain; charset=ISO-8859-1 Content-Transfer-Encoding: 8bit X-MIME-Autoconverted: from quoted-printable to 8bit by sucuri.mat.puc-rio.br id SAA24661 Sender: owner-obm-l@sucuri.mat.puc-rio.br Precedence: bulk Reply-To: obm-l@mat.puc-rio.br Leandro Lacorte Recôva wrote: > Arthur, > > Tenho certeza que ela ainda nao viu o conceito de serie de potencias.> OK. Mas, de qualquer forma, se alguém aceita que a função E é estritamente positiva e que a^x = E(x Lna), acho que dá para aceitar que devemos ter a>0. Artur ========================================================================= Instruções para entrar na lista, sair da lista e usar a lista em http://www.mat.puc-rio.br/~nicolau/olimp/obm-l.html O administrador desta lista é ========================================================================= From owner-obm-l@sucuri.mat.puc-rio.br Mon Apr 7 18:33:56 2003 Return-Path: Received: (from majordom@localhost) by sucuri.mat.puc-rio.br (8.9.3/8.9.3) id SAA26044 for obm-l-MTTP; Mon, 7 Apr 2003 18:32:04 -0300 Received: from cmsrelay01.mx.net (cmsrelay01.mx.net [165.212.11.110]) by sucuri.mat.puc-rio.br (8.9.3/8.9.3) with SMTP id SAA26038 for ; Mon, 7 Apr 2003 18:32:00 -0300 Received: from uadvg128.cms.usa.net (HELO localhost) (165.212.11.128) by cmsoutbound.mx.net with SMTP; 7 Apr 2003 21:24:09 -0000 Received: from smtp.postoffice.net [165.212.8.20] by uadvg128.cms.usa.net (ASMTP/) via mtad (C8.MAIN.2.05) with ESMTP id 562HDgVyH0345M28; Mon, 07 Apr 2003 21:24:07 GMT Received: from 200.181.4.100 [200.181.4.100] by uwdvg020.cms.usa.net (USANET web-mailer CM.0402.5.2B); Mon, 07 Apr 2003 21:24:06 -0000 Date: Mon, 07 Apr 2003 18:24:06 -0300 From: Artur Costa Steiner To: Subject: [obm-l] =?ISO-8859-1?Q?Re=3A=20=5BRe=3A=20=5Bobm=2Dl=5D=20base=20da=20?= =?ISO-8859-1?Q?fun=E7=E3o=20exponencial=5D?= X-Mailer: USANET web-mailer (CM.0402.5.2B) Mime-Version: 1.0 Message-ID: <849HDgVyg2976S20.1049750646@uwdvg020.cms.usa.net> Content-Type: text/plain; charset=ISO-8859-1 Content-Transfer-Encoding: 8bit X-MIME-Autoconverted: from quoted-printable to 8bit by sucuri.mat.puc-rio.br id SAA26039 Sender: owner-obm-l@sucuri.mat.puc-rio.br Precedence: bulk Reply-To: obm-l@mat.puc-rio.br > >oi pessoal da lista, estou com uma dúvida bem básica. > >Por que a base "a" de uma função exponencial não pode > >pertencer a R* - {+-1}? > > > >[]´s > > Renatinha Acredito que uma outra forma de perceber isto é considerar o sentido natural de um número elevado a outro. Este conceito, da forma como está enunciado, só faz sentido quando o expoente é um natural maior do que 1, pois, na definição "natural", elevar o número à potência n significa multiplicar a por a n vezes, isto é, calcular a x a ...x a --> n vezes. Mesmo para n=1 este conceito não é perfeitamente preciso, e se convenciona que a elevado a 1 é a. Daí, parte-se para a convenção de que a^0 =1 e de que a^(-n) = 1/(a^n). E podemos ainda chegar aos racionais, quando definimos que a^r = raiz_n (a^m), sendo r =m/n e m e n inteiros. Mas quando vamos para os irracionais (e mais ainda para os complexos) este processo progressivo tem uma quebra, pois, conforme um colega disse, não há modo trivial de definirmos a^x. Daí se define a função exponencial com base num conceito bem diferente, conhecido por série de potências, e se prova que a função completa assim definida "bate", para agumentos racionais, com aquela outra mais "natural" definida até os racionais. Ma como a função completa não é definida para base <= 0, sempre se consideram bases positivas. Espero que isto seja mais claro. Artur ========================================================================= Instruções para entrar na lista, sair da lista e usar a lista em http://www.mat.puc-rio.br/~nicolau/olimp/obm-l.html O administrador desta lista é ========================================================================= From owner-obm-l@sucuri.mat.puc-rio.br Mon Apr 7 18:55:55 2003 Return-Path: Received: (from majordom@localhost) by sucuri.mat.puc-rio.br (8.9.3/8.9.3) id SAA26901 for obm-l-MTTP; Mon, 7 Apr 2003 18:54:24 -0300 Received: from Euler.impa.br (euler.impa.br [147.65.1.3]) by sucuri.mat.puc-rio.br (8.9.3/8.9.3) with ESMTP id SAA26897 for ; Mon, 7 Apr 2003 18:54:21 -0300 Received: from Gauss.impa.br (Gauss [147.65.4.1]) by Euler.impa.br (8.11.6p2/8.11.6) with ESMTP id h37Lro220080 for ; Mon, 7 Apr 2003 18:53:50 -0300 (EST) From: Carlos Gustavo Tamm de Araujo Moreira Received: by Gauss.impa.br (8.11.6p2) id h37Lro720302; Mon, 7 Apr 2003 18:53:50 -0300 (EST) Message-Id: <200304072153.h37Lro720302@Gauss.impa.br> Subject: Re: [obm-l] Mais Probls em Aberto II To: obm-l@mat.puc-rio.br Date: Mon, 7 Apr 2003 18:53:49 -0300 (EST) In-Reply-To: <03f001c2f7b4$aa8349e0$3300c57d@bovespa.com> from "=?Windows-1252?Q?Cl=E1udio_\=28Pr=E1tica\=29?=" at Mar 31, 3 03:38:03 pm X-Mailer: ELM [version 2.4 PL25] MIME-Version: 1.0 Content-Type: text/plain; charset=US-ASCII Content-Transfer-Encoding: 7bit Sender: owner-obm-l@sucuri.mat.puc-rio.br Precedence: bulk Reply-To: obm-l@mat.puc-rio.br > >9)Prove, para todo n=FAmero real positivo x,y,z, a=20 >seguinte inequa=E7=E3o:=20 >(xy+yz+zx)*[1/(x+y)=B2 + 1/(y+z)=B2 + 1/(z+x)=B2]>=3D1/4. > Isso e' trivial pois, sendo z o maior dos tres numeros, (xy+xz+yz)(1/(x+y)^2+1/(x+z)^2+1/(y+z)^2)>=(xz+yz)(1/(x+y)^2)=z/(x+y)>=1/2. Seria mais interessante tentar provar que (xy+xz+yz)(1/(x+y)^2+1/(x+z)^2+1/(y+z)^2)>=9/4 (o que parece ser o valor minimo dessa expressao - eu acho que sei provar isso usando multiplicadores de Lagrange, mas ainda nao achei nenhuma prova elementar)... >***** > >10)Resolva o sistema de equa=E7=F5es: > i)raiz(3x)*[1+1/(x+y)]=3D2 >ii)raiz(7y)*[1-1/(x+y)]=3D4*raiz(2) > Isso nao parece ter uma resposta muito bonitinha... Fazendo s=x+y eu obtive (com alguma ajuda do mathematica, por preguica...) os seguintes valores possiveis para s (isolamos x e y em funcao de s e somamos: deve dar s): -5 - 2 I Sqrt[6] -5 + 2 I Sqrt[6] Out[13]= {{s -> ----------------}, {s -> ----------------}, 7 7 11 - 4 Sqrt[7] 11 + 4 Sqrt[7] {s -> --------------}, {s -> --------------}} 3 3 Depois eu escrevo mais. Abracos, Gugu ========================================================================= Instruções para entrar na lista, sair da lista e usar a lista em http://www.mat.puc-rio.br/~nicolau/olimp/obm-l.html O administrador desta lista é ========================================================================= From owner-obm-l@sucuri.mat.puc-rio.br Mon Apr 7 18:56:40 2003 Return-Path: Received: (from majordom@localhost) by sucuri.mat.puc-rio.br (8.9.3/8.9.3) id SAA26978 for obm-l-MTTP; Mon, 7 Apr 2003 18:55:22 -0300 Received: from Euler.impa.br (euler.impa.br [147.65.1.3]) by sucuri.mat.puc-rio.br (8.9.3/8.9.3) with ESMTP id SAA26974 for ; Mon, 7 Apr 2003 18:55:19 -0300 Received: from Gauss.impa.br (Gauss [147.65.4.1]) by Euler.impa.br (8.11.6p2/8.11.6) with ESMTP id h37Lsn220137 for ; Mon, 7 Apr 2003 18:54:49 -0300 (EST) From: Carlos Gustavo Tamm de Araujo Moreira Received: by Gauss.impa.br (8.11.6p2) id h37Lsmh20337; Mon, 7 Apr 2003 18:54:48 -0300 (EST) Message-Id: <200304072154.h37Lsmh20337@Gauss.impa.br> Subject: Re: [obm-l] Mais Problemas em Aberto To: obm-l@mat.puc-rio.br Date: Mon, 7 Apr 2003 18:54:48 -0300 (EST) In-Reply-To: <01e201c2f7b1$472e8ba0$3300c57d@bovespa.com> from "=?Windows-1252?Q?Cl=E1udio_\=28Pr=E1tica\=29?=" at Mar 31, 3 03:13:46 pm X-Mailer: ELM [version 2.4 PL25] MIME-Version: 1.0 Content-Type: text/plain; charset=US-ASCII Content-Transfer-Encoding: 7bit Sender: owner-obm-l@sucuri.mat.puc-rio.br Precedence: bulk Reply-To: obm-l@mat.puc-rio.br > >3)Determine todos os inteiros positivos que podem ser representados de = >maneira =FAnica sob a forma ( x^2+y)/(xy+1). > Se n=(x^2+y)/(xy+1) e' inteiro entao (y^2-x)/(xy+1)=ny-x tambem e'. Temos entao dois casos: ou y^2=x (e nesse caso n=(y^4+y)/(y^3+1)=y) ou |xy+1|<=|y^2-x| e |xy+1|<=|x^2+y|. Se admitirmos x ou y negativos temos solucoes com y=-1, quando n=-(x+1), e todo inteiro positivo tem pelo menos duas representacoes. Vamos considerar entao x e y naturais. Nesse caso, se y=0, temos n=x^2 com duas representacoes. Se y e' positivo, devemos ter y^2-x>=xy+1, donde y>x, ou seja, y>=x+1. De x^2+y>=xy+1 temos y(x-1)<=x^2-1, o que da' x=1 (o que da' sempre n=1, que tem pois infinitas representacoes) ou y<=x+1, e portanto y=x+1, o que tambem da' n=1. Assim, se supusermos x e y naturais todos os nao quadrados tem representacao unica. Se supusermos x e y inteiros positivos todos os inteiros positivos, exceto n=1, tem representacao unica, mas se deixarmos x e y serem inteiros quaisquer entao nenhum inteiro positivo tem representacao unica. ========================================================================= Instruções para entrar na lista, sair da lista e usar a lista em http://www.mat.puc-rio.br/~nicolau/olimp/obm-l.html O administrador desta lista é ========================================================================= From owner-obm-l@sucuri.mat.puc-rio.br Mon Apr 7 19:05:44 2003 Return-Path: Received: (from majordom@localhost) by sucuri.mat.puc-rio.br (8.9.3/8.9.3) id TAA27504 for obm-l-MTTP; Mon, 7 Apr 2003 19:04:18 -0300 Received: from mafalda.rantac.com.br ([200.245.60.141]) by sucuri.mat.puc-rio.br (8.9.3/8.9.3) with SMTP id TAA27497 for ; Mon, 7 Apr 2003 19:04:14 -0300 Received: (qmail 24001 invoked from network); 7 Apr 2003 22:03:01 -0000 Received: from unknown (HELO oswaldostanziola) (200.243.181.99) by 0 with SMTP; 7 Apr 2003 22:03:01 -0000 Message-ID: <001101c2fd51$ca6d6360$63b5f3c8@oswaldostanziola> From: "Oswaldo Stanziola" To: References: <20030407134057.64056.qmail@web14309.mail.yahoo.com> Subject: Re: [obm-l] pontos notaveis Date: Mon, 7 Apr 2003 19:05:23 -0300 MIME-Version: 1.0 Content-Type: text/plain; charset="iso-8859-1" Content-Transfer-Encoding: 8bit X-Priority: 3 X-MSMail-Priority: Normal X-Mailer: Microsoft Outlook Express 6.00.2800.1106 X-MimeOLE: Produced By Microsoft MimeOLE V6.00.2800.1106 Sender: owner-obm-l@sucuri.mat.puc-rio.br Precedence: bulk Reply-To: obm-l@mat.puc-rio.br Rafael, No excelente livro dos professores Morgado/Wagner/Jorge - Geometria II Livraria Francisco Alves- pág. 260 está a dica para a resolução do problema. Abraços Oswaldo ----- Original Message ----- From: "Rafael" To: "OBM" Sent: Monday, April 07, 2003 10:40 AM Subject: [obm-l] pontos notaveis > Oi pessoal! > > Tenho essa questão aqui que não consigo resovler. Como > parece-me trabalhosa, principalmente para digitar > aqui, agradeço pelo menos umas dicas dos caminhos que > devo seguir: > > Num triangulo ABC, a distância do baricentro ao > ortocentro é igual a 4cm. Então a distãncia do > baricentro ao circuncentro vale: > a)0,5 b)1 c)2 d)4 e)8 > > Valeu! > > Rafael. > > _______________________________________________________________________ > Yahoo! Mail > O melhor e-mail gratuito da internet: 6MB de espaço, antivírus, acesso POP3, filtro contra spam. > http://br.mail.yahoo.com/ > ========================================================================= > Instruções para entrar na lista, sair da lista e usar a lista em > http://www.mat.puc-rio.br/~nicolau/olimp/obm-l.html > O administrador desta lista é > ========================================================================= > > ========================================================================= Instruções para entrar na lista, sair da lista e usar a lista em http://www.mat.puc-rio.br/~nicolau/olimp/obm-l.html O administrador desta lista é ========================================================================= From owner-obm-l@sucuri.mat.puc-rio.br Mon Apr 7 19:13:07 2003 Return-Path: Received: (from majordom@localhost) by sucuri.mat.puc-rio.br (8.9.3/8.9.3) id TAA28084 for obm-l-MTTP; Mon, 7 Apr 2003 19:11:45 -0300 Received: from hotmail.com (oe49.law10.hotmail.com [64.4.14.21]) by sucuri.mat.puc-rio.br (8.9.3/8.9.3) with ESMTP id TAA28080 for ; Mon, 7 Apr 2003 19:11:41 -0300 Received: from mail pickup service by hotmail.com with Microsoft SMTPSVC; Mon, 7 Apr 2003 15:11:10 -0700 Received: from 64.60.139.18 by OE49.law10.internal.hotmail.com with DAV; Mon, 07 Apr 2003 22:11:09 +0000 X-Originating-IP: [64.60.139.18] X-Originating-Email: [lrecova@hotmail.com] From: =?iso-8859-1?Q?Leandro_Lacorte_Rec=F4va?= To: Subject: [obm-l] =?iso-8859-1?Q?RE:_=5Bobm-l=5D_Re:_=5B=5Bobm-l=5D_RE:_=5Bobm-l=5D_Re:_=5B?= =?iso-8859-1?Q?=5Bobm-l=5D_base_da_fun=E7=E3o_exponencial=5D=5D?= Date: Mon, 7 Apr 2003 15:11:11 -0700 Message-ID: <00aa01c2fd52$993c59d0$28029b9b@LeandroRecova> MIME-Version: 1.0 Content-Type: text/plain; charset="iso-8859-1" X-Priority: 3 (Normal) X-MSMail-Priority: Normal X-Mailer: Microsoft Outlook, Build 10.0.3416 X-MimeOLE: Produced By Microsoft MimeOLE V6.00.2800.1106 Importance: Normal In-Reply-To: <161HDgVDc3184S07.1049749382@uwdvg007.cms.usa.net> X-OriginalArrivalTime: 07 Apr 2003 22:11:10.0208 (UTC) FILETIME=[982DD000:01C2FD52] Content-Transfer-Encoding: 8bit X-MIME-Autoconverted: from quoted-printable to 8bit by sucuri.mat.puc-rio.br id TAA28081 Sender: owner-obm-l@sucuri.mat.puc-rio.br Precedence: bulk Reply-To: obm-l@mat.puc-rio.br Arthur, Eu entendi o que voce colocou. So me preocupei pelo fato da Renata ainda estar no 1o ano Colegial e nao saber bem o conceito de serie de potencias. Sua explicacao foi muito boa e clara. Saudacoes Rubro-Negras, Leandro. -----Original Message----- From: owner-obm-l@sucuri.mat.puc-rio.br [mailto:owner-obm-l@sucuri.mat.puc-rio.br] On Behalf Of Artur Costa Steiner Sent: Monday, April 07, 2003 2:03 PM To: obm-l@mat.puc-rio.br Subject: [obm-l] Re: [[obm-l] RE: [obm-l] Re: [[obm-l] base da função exponencial]] Leandro Lacorte Recôva wrote: > Arthur, > > Tenho certeza que ela ainda nao viu o conceito de serie de potencias.> OK. Mas, de qualquer forma, se alguém aceita que a função E é estritamente positiva e que a^x = E(x Lna), acho que dá para aceitar que devemos ter a>0. Artur ======================================================================== = Instruções para entrar na lista, sair da lista e usar a lista em http://www.mat.puc-rio.br/~nicolau/olimp/obm-l.html O administrador desta lista é ======================================================================== = ========================================================================= Instruções para entrar na lista, sair da lista e usar a lista em http://www.mat.puc-rio.br/~nicolau/olimp/obm-l.html O administrador desta lista é ========================================================================= From owner-obm-l@sucuri.mat.puc-rio.br Mon Apr 7 19:15:28 2003 Return-Path: Received: (from majordom@localhost) by sucuri.mat.puc-rio.br (8.9.3/8.9.3) id TAA28334 for obm-l-MTTP; Mon, 7 Apr 2003 19:14:10 -0300 Received: from Euler.impa.br (euler.impa.br [147.65.1.3]) by sucuri.mat.puc-rio.br (8.9.3/8.9.3) with ESMTP id TAA28330 for ; Mon, 7 Apr 2003 19:14:07 -0300 Received: from Gauss.impa.br (Gauss [147.65.4.1]) by Euler.impa.br (8.11.6p2/8.11.6) with ESMTP id h37MDb221185 for ; Mon, 7 Apr 2003 19:13:37 -0300 (EST) From: Carlos Gustavo Tamm de Araujo Moreira Received: by Gauss.impa.br (8.11.6p2) id h37MDaG22078; Mon, 7 Apr 2003 19:13:36 -0300 (EST) Message-Id: <200304072213.h37MDaG22078@Gauss.impa.br> Subject: Re: [obm-l] Mais Probls em Aberto II To: obm-l@mat.puc-rio.br Date: Mon, 7 Apr 2003 19:13:36 -0300 (EST) In-Reply-To: <00cd01c2f7ef$54520120$2101a8c0@u2z7z2> from "Wagner" at Mar 31, 3 10:37:59 pm X-Mailer: ELM [version 2.4 PL25] MIME-Version: 1.0 Content-Type: text/plain; charset=US-ASCII Content-Transfer-Encoding: 7bit Sender: owner-obm-l@sucuri.mat.puc-rio.br Precedence: bulk Reply-To: obm-l@mat.puc-rio.br Caros companheiros, Eu nao me convenci com o argumento que o Fabio usou para mostrar que a ordem de 10 modulo 3^2002 e' 3^2000 (acho que ele so' provou que essa ordem divide 3^2000). Vou dar outro argumento, por inducao, de que para todo n>=2, a ordem de 10 modulo 3^n (e portanto o tamanho do periodo de 1/3^n) e' 3^(n-2). Para isso, vou provar que para todo n>=0, 10^(3^n)=1+b(n).3^(n+2), onde 3 nao diide b(n). Isso vale para n=0, e, se vale para n, teremos 10^(3^(n+1))=(10^(3^n))^3=(1+b(n).3^(n+2))^3=1+3.b(n).3^(n+2)+3.(b(n).3^(n+2))^2+ +(b(n).3^(n+2))^3=1+b(n).3^(n+3)(mod 3^(n+4))=1+b(n+1).3^(n+3), onde b(n+1)=b(n)(mod 3), e portanto 3 nao divide b(n+1). Assim, 10^(3^2000)=1(mod 3^2002), mas 10^(3^1999)=1+b(1999).3^2001 nao e' 1 mod. 3^2002, e portanto a ordem de 10 modulo 3^2002 e' 3^2000. Abracos, Gugu > >Oi para todos! > >Isso também é a prova das 2 hipóteses que eu sugeri para resolver o problema >(Mas essas hipóteses não eram suficientes para chegar na resposta, já que a >resposta poderia ser 3^2000 ou 3^2001) > >André T. > > > >> > 17) >> > a) Ao escrevermos a fração 1/3^2002 como um número decimal, obtemos uma >> > dízima periódica. Qual o número de algarismos da período? b) Existe >algum >> > inteiro positivo n tal que 1/3^n é uma dízima periódica cujo período tem >um >> > número par de algarismos? >> > [...] >> >> 17) >> a) >> O conjunto formados pelos invertíveis módulo 3^2002 que são congruentes a >1 >> módulo 9 é invariante por uma multiplicação por 10. Esse conjunto tem >> \phi(3^2002)/\phi(9) = 3^2000 elementos. Seja P o produto de todos os seus >> elementos. Então >> >> P === 10^(3^2000)*P (mod 3^2002) >> 10^(3^2000) === 1 (mod 3^2002) >> >> Logo a ordem de 10 (mod 3^2002) divide 3^2000. Como a ordem de 10 (mod >3^2002) >> é o número de elementos do menor conjunto invariante por uma multiplicação >> por 10, mas como 10 !== 1 (mod 27), não existem conjuntos com >> \phi(3^2002)/\phi(27) === 3^1999 elementos. Logo a ordem de 10 (mod >3^2002) é >> mesmo 3^2000, *logo o período de 1/3^2002 é 3^2000*. >> >> b) >> Não. Seja K = {x | x é invertível (mod 3^n) e x === 1 (mod 9)}. É óbvio >que >> 10K = K. Se P é o produto dos elementos de K, então >> >> P === 10^(#(K))*P (mod 3^n) >> 10^(#(K)) === 1 (mod 3^n) >> ord_3^2002(10) | #(K) = \phi(3^n)/\phi(9) = 3^(n-2) >> >> Mas 2 | ord_3^2002(10) <=> 2 | 3^(n-2), *absurdo*. Logo 1/3^n sempre tem >> período ímpar (em particular, seu período sempre é uma potência de três). >> >> []s, >> >> - -- >> Fábio "ctg \pi" Dias Moreira >> -----BEGIN PGP SIGNATURE----- >> Version: GnuPG v1.0.6 (GNU/Linux) >> Comment: For info see http://www.gnupg.org >> >> iD8DBQE+iNnmalOQFrvzGQoRAopiAKCnIycHoC8alkkUs3Rs40pYdFi3oACcDmo+ >> aegviRKBOA7fJIQz24jyDWk= >> =+m/H >> -----END PGP SIGNATURE----- >> >> ========================================================================= >> Instruções para entrar na lista, sair da lista e usar a lista em >> http://www.mat.puc-rio.br/~nicolau/olimp/obm-l.html >> O administrador desta lista é >> ========================================================================= > > >========================================================================= >Instruções para entrar na lista, sair da lista e usar a lista em >http://www.mat.puc-rio.br/~nicolau/olimp/obm-l.html >O administrador desta lista é >========================================================================= ========================================================================= Instruções para entrar na lista, sair da lista e usar a lista em http://www.mat.puc-rio.br/~nicolau/olimp/obm-l.html O administrador desta lista é ========================================================================= From owner-obm-l@sucuri.mat.puc-rio.br Mon Apr 7 19:57:14 2003 Return-Path: Received: (from majordom@localhost) by sucuri.mat.puc-rio.br (8.9.3/8.9.3) id TAA30514 for obm-l-MTTP; Mon, 7 Apr 2003 19:55:30 -0300 Received: from fgvrj23.fgv.br (fgvrj23.fgv.br [200.20.164.23]) by sucuri.mat.puc-rio.br (8.9.3/8.9.3) with ESMTP id TAA30504 for ; Mon, 7 Apr 2003 19:55:21 -0300 Received: by FGVRJ23 with Internet Mail Service (5.5.2655.55) id <23V2DJMR>; Mon, 7 Apr 2003 19:56:31 -0300 Message-ID: <3BE65222F383D611BE1E00D0B7B60A55E02834@FGVRJ23> From: Ralph Teixeira To: "'obm-l@mat.puc-rio.br'" Subject: [obm-l] =?iso-8859-1?Q?RES=3A_=5Bobm-l=5D_Pent=E1gono?= Date: Mon, 7 Apr 2003 19:56:27 -0300 MIME-Version: 1.0 X-Mailer: Internet Mail Service (5.5.2655.55) Content-Type: multipart/mixed; boundary="----_=_NextPart_000_01C2FD58.EBD55010" Sender: owner-obm-l@sucuri.mat.puc-rio.br Precedence: bulk Reply-To: obm-l@mat.puc-rio.br This message is in MIME format. Since your mail reader does not understand this format, some or all of this message may not be legible. ------_=_NextPart_000_01C2FD58.EBD55010 Content-Type: multipart/alternative; boundary="----_=_NextPart_001_01C2FD58.EBD55010" ------_=_NextPart_001_01C2FD58.EBD55010 Content-Type: text/plain; charset="iso-8859-1" Content-Transfer-Encoding: quoted-printable Sejam AO=3D2x e AQ=3Dy. Escreva tamb=E9m AB=3DL. =20 Dividindo o pent=E1gono em 5 tri=E2ngulos como OCD, v=EA-se que sua = =E1rea =E9 5.L.1/2=3D 5L/2. Por outro lado, usando as =E1reas de ABC, ACD e ADE, chegamos a = Ly/2+L.(2x+1)/2+Ly/2 =20 Igualando essas coisas, L(x+y)+L/2=3D5L/2, x+y=3D2; 2x+2y=3D4, que = =E9 a resposta desejada. ABCDE =E9 um pent=E1gono regular. AP, AQ e AR s=E3o perpendiculares = tra=E7adas de A at=E9 CD e os prolongamentos de CB e DE, = respectivamente. Se O =E9 o centro do pent=E1gono e OP=3D1, ent=E3o = AO+AQ+AR =E9 igual a: =20 a) 3 b) 1+ c) 4 d) 2+ e) 5 =20 =20 =20 =20 =20 =20 ------_=_NextPart_001_01C2FD58.EBD55010 Content-Type: text/html; charset="iso-8859-1" Content-Transfer-Encoding: quoted-printable
   =20 Sejam AO=3D2x e AQ=3Dy. Escreva tamb=E9m=20 AB=3DL.
 
    Dividindo o pent=E1gono = em 5=20 tri=E2ngulos como OCD, v=EA-se que sua =E1rea =E9 5.L.1/2=3D = 5L/2.
    Por outro lado, usando as = =E1reas de=20 ABC, ACD e ADE, chegamos a Ly/2+L.(2x+1)/2+Ly/2
 
    Igualando essas coisas,=20 L(x+y)+L/2=3D5L/2, x+y=3D2; 2x+2y=3D4, que =E9 a resposta = desejada.

ABCDE =E9 um = pent=E1gono regular.=20 AP, AQ e AR s=E3o perpendiculares tra=E7adas de A at=E9 CD e os = prolongamentos de CB=20 e DE, respectivamente. Se O =E9 o centro do pent=E1gono e OP=3D1, = ent=E3o AO+AQ+AR =E9=20 igual a:

 

a)     =20 3

b)     =20 1+

c)     =20 4

d)     =20 2+

e)     =20 = 5

            &= nbsp;           &= nbsp;           &= nbsp;          =20

 

 

 

 

------_=_NextPart_001_01C2FD58.EBD55010-- ------_=_NextPart_000_01C2FD58.EBD55010 Content-Type: image/gif; name="clip_image002.gif" Content-Transfer-Encoding: base64 Content-Disposition: attachment; filename="clip_image002.gif" Content-ID: <834365322@07042003-113c> R0lGODlhGAAYAHcAMSH+GlNvZnR3YXJlOiBNaWNyb3NvZnQgT2ZmaWNlACH5BAEAAAAALAMAAwAS ABIAhIGBgQAAAAECAwECAwECAwECAwECAwECAwECAwECAwECAwECAwECAwECAwECAwECAwECAwEC AwECAwECAwECAwECAwECAwECAwECAwECAwECAwECAwECAwECAwECAwECAwIphH+hu+jhYoSy0iov fojhq1ndZoDNl6EpoCQoBKrlLLps3YXeFJI3WQAAOw== ------_=_NextPart_000_01C2FD58.EBD55010 Content-Type: image/gif; name="clip_image003.gif" Content-Transfer-Encoding: base64 Content-Disposition: attachment; filename="clip_image003.gif" Content-ID: <834365322@07042003-1143> R0lGODlhGAAYAHcAMSH+GlNvZnR3YXJlOiBNaWNyb3NvZnQgT2ZmaWNlACH5BAEAAAAALAMAAwAS ABIAhIGBgQAAAAECAwECAwECAwECAwECAwECAwECAwECAwECAwECAwECAwECAwECAwECAwECAwEC AwECAwECAwECAwECAwECAwECAwECAwECAwECAwECAwECAwECAwECAwECAwIphH+hu+jhYoSy0iov fojhq1ndZoDNl6EpoCQoBKrlLLps3YXeFJI3WQAAOw== ------_=_NextPart_000_01C2FD58.EBD55010 Content-Type: image/jpeg; name="clip_image005.jpg" Content-Transfer-Encoding: base64 Content-Disposition: attachment; filename="clip_image005.jpg" Content-ID: <834365322@07042003-114a> /9j/4AAQSkZJRgABAQEAYABgAAD//gAcU29mdHdhcmU6IE1pY3Jvc29mdCBPZmZpY2X/2wBDAAgG BgcGBQgHBwcJCQgKDBQNDAsLDBkSEw8UHRofHh0aHBwgJC4nICIsIxwcKDcpLDAxNDQ0Hyc5PTgy PC4zNDL/2wBDAQkJCQwLDBgNDRgyIRwhMjIyMjIyMjIyMjIyMjIyMjIyMjIyMjIyMjIyMjIyMjIy MjIyMjIyMjIyMjIyMjIyMjL/wAARCACzAQADASIAAhEBAxEB/8QAGwABAQEBAQEBAQAAAAAAAAAA AAUEBgMCBwH/xABDEAABBAIABAMCCwQJAwUAAAABAAIDBAURBhIhMRNBUSJhFBUlMjZicXWBkrMj VpHTBxYkNEJSU3KhM0NzNUSCsdH/xAAUAQEAAAAAAAAAAAAAAAAAAAAA/8QAFBEBAAAAAAAAAAAA AAAAAAAAAP/dAAQAKP/aAAwDAQACEQMRAD8A/f0REBERAREQEREBERAREQEREBERAREQEREBERAR EQEREBERARFko5TH5MTGhfq2/BeY5fAmbJyO/wArtHofcUGtERAUHi/CX89gZKuMy9rF3GHxYpq0 hYXODXANcR15SSN69FeRBjxeQjyuLr3omuY2ZgcY365o3dnMd6OaQWkeRBC2KDW+SeJp6rulbKHx 4PPUzW6kHu20NI8ujvMq8gIiICIiAiIgIiICIiAiIgIiICIiAiIgIiICIiAiIgw5jIjFYua1yGSQ abFEO8jydNaB5kkjosvDfD1Th3GfB68ELJ5XGWzJGwDxZD1JPr31s+QC8ZflfikVz7VPFtbJIO7X WXdWA+9jfa0exkjIV5AREQEREE7N45+RxrmQODLUThLXkP8Ahkadj+Pb8V64rIsyuMguMaWeI32m O7scOhafeCtigw/JHFEkDvZqZX9pD6NsNaS9noOZjQ8dyS2Qk9kF5ERAREQEREBERAREQFw2a48n xnGsONiqwSYeu6KLJ3HSgOryzb8JoHMD5bPQ9HDsuuvZShjQw3bkFfn2GCR4BefRo7uPuHVfls2R 4LyGWvVo+HrOSoZBsl2fORxvlcHAgOEbtc4a08gBadN3oa0UH//Q/f0UnD57DZOGKLG5KCwRGC1n i7kLRocxB9ojt1VZAREQEREBERAREQFjyuRZisbLckY5/JytZG3u97iGsaD5bcQNnoN7PRbFBn+V uKI6x/u2LDZ3++dwIaPsDST6e17kGzBY5+Lw8Ned7ZLLi6azI3s+Z7i+Qj0Bc46HkNDyVJEQEREB ERAU7N452TxcsMMgits/a1Z9b8GZvVjvf17jsQSDsEhUUQYsTkW5XGxW2xmNzi5kkZO/De0lrm78 9OBG/Putq52exBw9xDNYtTR18bkGgullcGRsnb06k9AXN11PctAXtZ4nggqy2o6OQlqwtMk03wd0 YawdSQH8pd06+yDvyQXEX8a4OaHNIII2CPNfMsscMZklkbGwd3POgPxQfaKR/WKnP/6cybJD/PTZ zx79PE2Gb9wOx5pzZ612jpY+M9uZzp5CD6gcrWkfa8de/TqFdTbGfxsE7q4stntN1utW/ayjfUba 3ZA7dTodR6ry+IIrHXJW7WQPk2Z4Yxv2MYGtP2uBI9V6yXKOJYylVr80g3yVKjBzAdyeUaDR17nQ 2R5kIPL4wylzpSxZrt/1r7w0a+qxpLj9juX8eym2X2Z7D60t67esN6SVMc1sMTPPT5D7QI2NjnBI 17PXrS+BX8l1yMvwaue9OtJvnH138oP4N0PIlwVKtWgp12QV4mxRM+axo0B5oINHhSuxz5LrYnGT QdBCHBjh/leSeaUf7un1R1XrN9PqX3XP+rCrygzfT6n91z/qwoPTJ8N1LxdKyOLxS/xDFOzxIJHe roz039Yad79bBnw1/gcza77d/FTk8rXsl8WtKfIN8QODd+nsu3sbPQnql8TQxWIXQzxslieNOY9o cCPeCgl+Jnqw26GjfjHU+G90EhH1QeZpJ8gXNHv8w/rFTi/vsVqjru6zCWsb67eNsA9+9J8WWcee bETNbH3dUsuc6M/7DsmP06baO/L66KmWhsy+BLHJUtd/AsANc4erdEhw94J/5CDZHLHMwPika9p7 Fp2F9qXJw7iXvL2U215P9Sq4wuB9dsIP/wC9l8fAczW618uLQHXkuwN273c0Ybr8p170FdFI+Nch X6XMLN7305WzMP2b5XfxaPxXvVzmLuPMUV6Hxh86CQ8krP8Acx2nN/EBBQRCdDZXP0c3lLccttmK jmx/ivbC+GxqZzGnXMY3ADuD/i326IKuTvx4zHTWpOvINMaBsvcejWgDqSToABZ8DQkx+JjZY62p SZrB3vcjuruvnrtv0CmC4OI8/BWZBYjrYx/jW2zRlv7fQMUZ8joO8ToTrUfqumQEREBERAREQFE4 m4li4ahxz5Ks1l9+9FQhZEWj9pJvlJJI0Oitrn+LOFoeK4cVDPM2OKjkobzmOiEgmDObcZBI0Dzd +v2IMc2WZxBkL/DclSWllqtdl2Fz3Nc1pJIY4OaTrTh19xK+cBxmeJ8a92NxNqWWJ5rzmzqGJsgA 5hs7JGz5NP8AxpbMfwlXxPFU+Yx0jK1exWbBNRjhDYyWkkPbogA9SDsHfuXo75I4qa8+zSyreUn/ AAstM7fjIzY30AMQHUvCDzxeEy1bHRVJMqK0MfMGRVWB5jYSSGiR46gDQGmN0ABrzW6Lh7HNkE1i E3LA/wC9bd4rvw30aPc0ADfQBVF5WbMFOu6exK2OJvdzig9Vlu5KpjmsNmYNdIdRxjq+Q+jWjq4+ 4LF8JyGT9mnE+jWP/uZ49SH/AGxuG2/a4fgtdLGV6LnyMMktiQafPM8ve4emz2bvZ5RpoJOgNoMm srk+vO7F1v8ALytdYd797LWj3acfeFup4+rQjLa0LWF3V7z1e8+rnHq49T1PqtKICIiAoM30+p/d c/6sKvKDN9Pqf3XP+rCgvIiICz26Va9F4dqBkrR1HMOrT6g9wfeOq0IgkeDkcV1rvdfpt/7EhJna PqyOPt69Hdeu+bpo7KWRrXgRE/UrP+pC/pJGfRze4/8Ao+Wwtax3cZXukSODorDfmWITyyN+w+nu PRBsXhapVbrAy1WinaOwkYHa/ip/wjJYzpbjdfrf69eMCSMfXZv2unXbOp7Bio1bde7CJqs8c0Z6 c0bg4b9OiCVY4ajdWlhoZG/QbI0tLYpvEYB5AMkDg0Dr0byj+A1OyVzN8IcOPlr0aOUgpxMZFCyV 1WTQIa1g2JA93bzbs9ANldYoN75V4ir4351Sm0WbLe7XP3+zafs0XaPoD5IOajyuWxtzhvDFjKWT 4hdYvX7EoEjoHMa17owOx0C2MOJPK1g6FWeDeI7GbnzVG0+GeTFXPg4tQ9GztLQ4O5eoB666E715 K5k8Nj8xHE29WEpheJInhxY+N3q17SHNP2FeWE4exfDlaWviqprxSyGR7fEe/bj3PtE90FNERARE Qf/R/f0REBERBjyWVoYeqLWRtR1oC9sYfIdAuPQD7SsNubHcR4q1XhsOPhhsgka0tdE9p5mPGx3D mg9j2Ur+kfB38/w3BVx0Dppo79edzGyNY7kY/btFxA3rt1VrCCVkUsclPJQAEODr87JS7fkC17tA a89d0EbhjjGTjDA17mGjjkfrks2JWPZCyQD2gwH2ndeo7dNbIKu1sNXisNtWXOuXGdWT2NOMe+/I NaZvz1rfTe9BYoAMVxXJABy1smwzMA7CZgAd/Fujv3K8gIiICIiAiIgKDN9Pqf3XP+rCrygzfT6n 91z/AKsKC8iIgIiICIiAp1rEskmNmpM+na85Ivmv9OdvZw/59CFRRBz+R4k/q3QmtcQsEVeFuzeh H7F58gW7LmOJ8jsdhzEnSm1c7Bw/UxTbME9vIcQTvmjbV5CHPLefW3ODQAzQ3vR171QzkEOczFLB TRMmqRgXr0b2hzS1p1Ewj60gLveIXA91l4r4POfuYGaAUhDipnyfBbEPNFI0s5Q3Q7AfZ5IKmH4j q5jI5LHNgsVr2NdG2zXnDdtEjeZhBa5zSCN9jsa66VhcxwtwtJgMpm7zpK8bMlJC9lOpHyQwckfI S0eru56DsF06AiIgIiICIiAiIgIiIJPEVKW3ijJVaXXKr22a4Hm9nXXp1G29fXfkt1C7FkcfXuwO Dop4xI0j0I2tCg4H+wZHJ4V3QRS/C6/vhmc469ByvEjdDsA31QXkREBERAREQclL/SHioatjIGtd diK9j4NNkgxvgsdvlJI5uflDtNJDe/u6rfN9PqX3XP8AqwqBP/Rw+XBXeHG5RrcDctmxLGYNzgF/ O5gk5tDbgOvL0HT3rfYw8D/6V6GTM1gSsxMzRGJP2fSRg+b/APM/wHog65ERAREQEREBfEsscMT5 ZXtjjY0ue950GgdSSfIL7ULiNxtiphYyQ6/JqXXlC3q/+PRuvMEoPrhqKSatYy9hjmWMlL4/I8ad HEByxMIPYhgBI2RzOeR3VtfxrWsaGtADQNADyC/qAiIgIiICIiAiIgIiICIiAoPEf9gkpZ9vT4ve 5tj31pNCT8payToCT4XKPnFXl5zwtsV5IX/MkaWn7CEHoii8LzOGJGPm/vGOd8Ef9YN+Y737bynf rtWkBERAREQFBm+n1P7rn/VhV5QZvp9T+65/1YUH/9L9/REQEREBERAUHC/KOVv5k9Yy41Kv/jYf aP4v30I6cvoVo4kuTVMM9tUgW7UjKsBI3p8jg0O15huy4+5pW+lThx9GvTrgiGCNsbATs6A11PmU HuiIgIiICIiAiIgIiICIiAiIgIiIINv5L4pq2x0gyI+Cy/8AlaCYz79jmHoNK8p2dx78phbNSJzW zkB8JcdN8Rjg5nN9XmaN6662vTE5BmVxVa6xrm+KwFzHDTmO7Oa4eRB2CPIhBtREQEREBQZvp9T+ 65/1YVeUGb6fU/uuf9WFBeREQEREBEWTKX2YvGWLsjeYRM2Gb0Xu7NaPeSQPxQTB8q8XuJ61sOzQ Hrakbsn7WROGj1B8c+bVeUzAUH47DwxTO5rMhdNYdrXNK8lzzry9onoqaAiIgIiICIiAiIgIiICI iAiIgIiICg4/5L4muY09K95pv1h5NfsNnaPQcxY/3mR/otPEuer8McO3c1aillgqMD3si1zEEgdN kDzXKZh7eIOLeHKtu3l8DbjEtuk2J8bmWyGgPBI5gC1hPcdnnv1QfoCKR8TXP3iyn5a/8pPia5+8 WU/LX/lIK6KR8TXP3iyn5a/8pPia5+8WU/LX/lIK65jJxW5uOqIp2mV3jGTkufD4mx4sXTWwqHxN c/eLKflr/wApcTZbEeOL9uLP8TT2cNXEFtkMUHhsa9viebQDsaPzT80IOrsz5jG5TDxzX69iC5bN eRgq8hA8GV+weY+bB5ea6JctTxkHEdDG5avxFlJ4ARaqycsLdEsc3evCH+F7ho+qpfE1z94sp+Wv /KQV0Uj4mufvFlPy1/5SfE1z94sp+Wv/ACkFdQcl8p8QUsY3rBW/tdn02OkbT9p2dHuBsdl7/E1z 94sp+Wv/AClz2Cmw3DWYyrLPE1vJWrdljJZbTS9kDuvJEZGt5Gn2ujSR5dEHcIiICIiAiIgIiICI iD//0/39ERAREQEREBERBF4u4f8A61cK5DCfCfgvwtgZ43h8/JpwO+XY329Vjh4VsTcR4zM5XIxW JcXBJFTir1jC1pkAa9ztveXEgAAdANHvtdMiAiIgIiIC46rwMw8VcS5W/YlfXyzoPDir2podNZFy ODwxwDt/j0XYog8KVKrjaMNKlAyCtAwMjijGmtaPIL3REBERAXEYPB5zhoZHH1qlO7XuZN91tqWb lDGSO5nAx6JJbrQ0dHv07Lt0QEREBERAREQEREBERAREQEREBERAREQEREBERAREQEREBERAREQE REBERAREQEREBERAREQf/9k= ------_=_NextPart_000_01C2FD58.EBD55010-- ========================================================================= Instruções para entrar na lista, sair da lista e usar a lista em http://www.mat.puc-rio.br/~nicolau/olimp/obm-l.html O administrador desta lista é ========================================================================= From owner-obm-l@sucuri.mat.puc-rio.br Mon Apr 7 19:59:22 2003 Return-Path: Received: (from majordom@localhost) by sucuri.mat.puc-rio.br (8.9.3/8.9.3) id TAA30591 for obm-l-MTTP; Mon, 7 Apr 2003 19:58:05 -0300 Received: from shannon.bol.com.br (shannon.bol.com.br [200.221.24.13]) by sucuri.mat.puc-rio.br (8.9.3/8.9.3) with ESMTP id TAA30583 for ; Mon, 7 Apr 2003 19:58:01 -0300 Received: from bol.com.br (200.221.24.140) by shannon.bol.com.br (5.1.071) id 3E76735F0069B3A6 for obm-l@mat.puc-rio.br; Mon, 7 Apr 2003 19:57:31 -0300 Date: Mon, 7 Apr 2003 19:57:31 -0300 Message-Id: Subject: [obm-l] =?iso-8859-1?q?Re=3A=5Bobm=2Dl=5D_Problemas_de_An=C3=26iexl=3Blise?= MIME-Version: 1.0 Content-Type: text/plain;charset="iso-8859-1" From: "rafaelc.l" To: obm-l@mat.puc-rio.br X-XaM3-API-Version: 2.4 R3 ( B4 ) X-SenderIP: 200.187.198.177 Content-Transfer-Encoding: 8bit X-MIME-Autoconverted: from quoted-printable to 8bit by sucuri.mat.puc-rio.br id TAA30584 Sender: owner-obm-l@sucuri.mat.puc-rio.br Precedence: bulk Reply-To: obm-l@mat.puc-rio.br Alguém pode me indicar alguma bibliografia sobre problemas de análise desse tipo(com funções)? Obrigado Rafael __________________________________________________________________________ E-mail Premium BOL Antivírus, anti-spam e até 100 MB de espaço. Assine já! http://email.bol.com.br/ ========================================================================= Instruções para entrar na lista, sair da lista e usar a lista em http://www.mat.puc-rio.br/~nicolau/olimp/obm-l.html O administrador desta lista é ========================================================================= From owner-obm-l@sucuri.mat.puc-rio.br Mon Apr 7 23:24:03 2003 Return-Path: Received: (from majordom@localhost) by sucuri.mat.puc-rio.br (8.9.3/8.9.3) id XAA02311 for obm-l-MTTP; Mon, 7 Apr 2003 23:22:20 -0300 Received: from paiol.terra.com.br (paiol.terra.com.br [200.176.3.18]) by sucuri.mat.puc-rio.br (8.9.3/8.9.3) with ESMTP id XAA02307 for ; Mon, 7 Apr 2003 23:22:17 -0300 Received: from jurua.terra.com.br (jurua.terra.com.br [200.176.3.42]) by paiol.terra.com.br (Postfix) with ESMTP id 8506788217 for ; Mon, 7 Apr 2003 23:21:46 -0300 (BRT) Received: from niski.com (unknown [200.148.198.44]) (authenticated user fniski) by jurua.terra.com.br (Postfix) with ESMTP id 960ACB0058 for ; Mon, 7 Apr 2003 23:21:45 -0300 (BRT) Message-ID: <3E92323A.5050508@niski.com> Date: Mon, 07 Apr 2003 19:21:46 -0700 From: niski User-Agent: Mozilla/5.0 (Windows; U; Windows NT 5.1; en-US; rv:1.0.2) Gecko/20030208 Netscape/7.02 X-Accept-Language: en-us, en MIME-Version: 1.0 To: obm-l@mat.puc-rio.br Subject: [obm-l] sobre matrizes simetricas Content-Type: text/plain; charset=ISO-8859-1; format=flowed Content-Transfer-Encoding: 8bit Sender: owner-obm-l@sucuri.mat.puc-rio.br Precedence: bulk Reply-To: obm-l@mat.puc-rio.br Olá colegas. Estou me embanando para calcular o numero maximo de entradas distintas que uma matriz simetrica e anti-simétrica comportam, gostaria de ler sugestoes e resolucoes. Muito obrigado Niski -- [about him:] It is rare to find learned men who are clean, do not stink and have a sense of humour. -Gottfried Whilhem Leibniz ========================================================================= Instruções para entrar na lista, sair da lista e usar a lista em http://www.mat.puc-rio.br/~nicolau/olimp/obm-l.html O administrador desta lista é ========================================================================= From owner-obm-l@sucuri.mat.puc-rio.br Mon Apr 7 23:38:14 2003 Return-Path: Received: (from majordom@localhost) by sucuri.mat.puc-rio.br (8.9.3/8.9.3) id XAA02726 for obm-l-MTTP; Mon, 7 Apr 2003 23:36:55 -0300 Received: from artemis.opendf.com.br (artemis.opengate.com.br [200.181.71.14]) by sucuri.mat.puc-rio.br (8.9.3/8.9.3) with ESMTP id XAA02721 for ; Mon, 7 Apr 2003 23:36:51 -0300 Received: from localhost (localhost [127.0.0.1]) by artemis.opendf.com.br (Postfix) with ESMTP id B2D222BECB for ; Mon, 7 Apr 2003 23:36:24 -0300 (BRT) Received: from artemis.opendf.com.br ([127.0.0.1]) by localhost (artemis.opengate.com.br [127.0.0.1:10024]) (amavisd-new) with ESMTP id 22297-04 for ; Mon, 7 Apr 2003 23:36:23 -0300 (BRT) Received: from artur (200-181-089-129.bsace7001.dsl.brasiltelecom.net.br [200.181.89.129]) by artemis.opendf.com.br (Postfix) with ESMTP id F1F392BEC0 for ; Mon, 7 Apr 2003 23:36:22 -0300 (BRT) From: "Artur Costa Steiner" To: Subject: [obm-l] =?iso-8859-1?Q?RE:_=5Bobm-l=5D_fun=E7=E3o_exponencial_=28de_novo=29?= Date: Mon, 7 Apr 2003 23:38:44 -0700 Message-ID: MIME-Version: 1.0 Content-Type: multipart/mixed; boundary="----=_NextPart_000_003A_01C2FD5E.D448F720" X-Priority: 3 (Normal) X-MSMail-Priority: Normal X-Mailer: Microsoft Outlook, Build 10.0.2627 X-MimeOLE: Produced By Microsoft MimeOLE V6.00.2800.1106 Importance: Normal In-Reply-To: X-MS-TNEF-Correlator: 00000000881CAD9D1A69CE40BD8EA975009E8893E4E32100 X-Virus-Scanned: by amavisd-new Sender: owner-obm-l@sucuri.mat.puc-rio.br Precedence: bulk Reply-To: obm-l@mat.puc-rio.br This is a multi-part message in MIME format. ------=_NextPart_000_003A_01C2FD5E.D448F720 Content-Type: text/plain; charset="iso-8859-1" Content-Transfer-Encoding: quoted-printable > -----Original Message----- > From: owner-obm-l@sucuri.mat.puc-rio.br [mailto:owner-obm- > l@sucuri.mat.puc-rio.br] On Behalf Of renatinha15a > Sent: Monday, April 07, 2003 12:59 PM > To: obm > Subject: [obm-l] fun=E7=E3o exponencial (de novo) >=20 > Primeiramente, obrigada Artur Costa pela explica=E7=E3o > sobre a base da fun=E7=E3o exponencial -confesso que ainda > me falta conhecimento matem=E1tico para entende-la, mas > estou batalhando para isso.- Nenhum problema nisso, nada a "confessar". No colegial n=E3o se ensina = mesmo a defini=E7=E3o precisa da fun=E7=E3o exponencial. A defini=E7=E3o baseada = em s=E9ries de pot=EAncias vem num est=E1gio um pouco adiante. > Estudo pelos livros fundamentos de mat. elementar, e > n=E3o entendi uma parte da solu=E7=E3o proposta pelo livro > (f. exponencial/logaritimo, vol. 2). Fico grata por > qualquer esclarecimento. >=20 > Resolva as equa=E7=F5es em R+: > a) x^(x^2 - 5x + 6) =3D 1 > [SOLU=C7=C3O DO LIVRO]: > Devemos examinar inicialmente se 0 ou 1 s=E3o solu=E7=F5es da > equa=E7=E3o. > Substituindo x =3D 0 na equa=E7=E3o proposta, temos: > 0^6 =3D 1 (falso) N=E0o entendi bem porque testar x=3D0. Zero elevado a qualquer n=FAmero = >0 =E9 sempre zero, logo nunca vai atender =E3 equa=E7=E3o apresentada. E zero = elevado a um n=FAmero <=3D0 n=E3o =E9 definido. Suponhamos agora que x<>0. Sabemos que 1 elevado a qualquer n=FAmero =E9 = 1. Logo, 1 =E9 trivialmente uma solu=E7=E3o da equa=E7=E3o dada. Vc est=E1 = achando estranho porque foi dito que normalmente n=E3o se considera base 1. Isto porque = uma fun=E7=E3o exponencial de base 1 =E9 uma constante e n=E3o h=E1 = interesse em consider=E1-la como uma exponencial. Mas o n=FAmero 1 elevado a y existe = sempre e =E9 o pr=F3prio 1. Pensando assim, h=E1 sentido em considerar a = solu=E7=E3o x =3D1. De fato 1^2 =3D 1 =E9 verdadeiro. Isto =E9 mesmo um tanto confuso. 1^x = existe e =E9 1, mas por simplicidade =20 , quando se trata da FUN=C7=C3O EXPONENCIAL, n=E3o se costuma considerar = a base 1, pois isto nada de interessante agregaria. =20 > logo, 0 n=E3o =E9 solu=E7=E3o. Vimos que n=E3o > Substituindo x =3D 1 na equa=E7=E3o, temos: > 1^2 =3D 1 (verdadeiro) De fato, pelo que dissemos > logo 1 =E9 solu=E7=E3o da equa=E7=E3o. > [D=DAVIDA] Por que examinar inicialmente se 0 e 1 s=E3o > solu=E7=F5es? Digo isso pois, se x =3D a (a =3D base), ent=E3o > temos 0 < x =3D/=3D 1. (=3D/=3D "diferente") O zero me pareceu meio sem sentido. O 1, OK pelo que vimos Mas n=E3o cabou por aqui. Dado que qualquer n=FAmero =3D/=3D 0 elevado a = zero =E9 1, h=E1 que verificar se o expoente se anula para valores de x n=E3o nulos. = E como o expoente =E9 um bin^mio do segundo grau, resolvemos a aequa=E7=E3o do = segundo grau correlata x^2 - 5 x + 6 =3D0 e ncontramos x=3D2 e x=3D3, ambas = solu=E7=F5es n=E3o nulas. Logo 2 e 3 s=E3o solu=E7=F5es da equa=E7=E3o. OK? Artur ------=_NextPart_000_003A_01C2FD5E.D448F720 Content-Type: application/ms-tnef; name="winmail.dat" Content-Transfer-Encoding: base64 Content-Disposition: attachment; filename="winmail.dat" eJ8+Ii0GAQaQCAAEAAAAAAABAAEAAQeQBgAIAAAA5AQAAAAAAADoAAEIgAcAGAAAAElQTS5NaWNy b3NvZnQgTWFpbC5Ob3RlADEIAQ2ABAACAAAAAgACAAEGAAcAAQAAAAAAAAEGgAMADgAAANMHBAAI AAAAJgAAAAIADgEBA5AGALQMAAAuAAAACwACAAEAAAALACMAAAAAAAMAJgAAAAAACwApAAAAAAAD AC4AAAAAAAIBMQABAAAAGAAAAAAAAACIHK2dGmnOQL2OqXUAnoiTpOMhAAMANgAAAAAAHgBwAAEA AAAlAAAAW29ibS1sXSBmdW7n428gZXhwb25lbmNpYWwgKGRlIG5vdm8pAAAAAAIBcQABAAAAFgAA AAHC/ZgmDfp6NRFQLUD9hszdVntulVYAAAIBHQwBAAAAGwAAAFNNVFA6QVJUVVJfU1RFSU5FUkBV U0EuTkVUAAALAAEOAAAAAEAABg4APJjHof3CAQIBCg4BAAAAGAAAAAAAAACIHK2dGmnOQL2OqXUA noiTwoAAAAMAFA4AAAAACwAfDgEAAAAeACgOAQAAACoAAAAwMDAwMDAwNQFhcnR1cl9zdGVpbmVy QHVzYS5uZXQBbmV0YWRkcmVzcwAAAB4AKQ4BAAAAKgAAADAwMDAwMDA1AWFydHVyX3N0ZWluZXJA dXNhLm5ldAFuZXRhZGRyZXNzAAAAAgEJEAEAAACIBwAAhAcAAGENAABMWkZ1hyQ0eQMACgByY3Bn MTI14jIDQ3RleAVBAQMB908KgAKkA+MCAGNoCsBz8GV0MCAHEwKAD/MAUH8EVghVB7IRxQ5RAwEQ xzL3BgAGwxHFMwRGEMkS2xHT2wjvCfc7GL8OMDURwgxgzmMAUAsJAWQzNhFQC6ZHCuMKhAqAPiAt HnJPPQUQZwuAB0AF0AeQc2EsZ2Uecx32RgNhOiAIb3duBJAtb2JtsC1sQHMbcAhxLgDAKHQucBtw LQUQby4aYgXAWwDAAxB0bzqPIPgd9iGfIqRdIE8DoORCZRDwbGYmMCbAGMCNHxB0C4AQ8DE1YR32 FwZgAjAg0E0CIGRheZosEWBwBRADIDA3KRARAdAwMyAOIDo1OeggUE0d9lQjcCDgIWCxJ9d1YmoF kCiBWyFTiSYgZnULkCdlNy1RaDNvIA7AcAIgCfBjCQcxICgBACBub3Zsbykd9h32UAUQB4Bp7HJh B4ACMGUpECFQHtG2YSjgEWF0CHASwXMBkNggcGULYC3ibA3gJ8A7LWYd9nMxMS7QMaBiYV0RICAx kS0vLjYtBaBu9mYfYS3QcQpQNKALgCjgvx32B4A1QAdAMlE28WgFkG8HcQIwLdAiIWUrgDWQMf8n UAWgMnAKwDKxMOEo0B+wfwtgKRAAwBCwHgUHkCNgdfs0wQGQbBDwKNA6tQQBIsDnIAUHwCdwdW0y cANgAmD/OiAxoAMAN0EpEB8QMZExoOYiNvUKwCIuB7At0Bgx/mUe8B8hNXItwTTxCfAAkPsfEDiB cwRgNKEBAQuAAKB/NZcpQDlhH4A1Hy35QUBBv0OvNNIxgjogNEBFkTkIgfcEIC7BLhB0RZEAcC5h BCD6dkihbj7xPJE6Qh7wLdC/PvIIYDqhMYAHMDDhLh2cekUyQHU9cjKQMjAkoGn+dgNgBCBFYSjg OZNJUyIi3y3gP2EoYQrAKRBlHfZCJf07FWlLkTJhCsAOsEUiNFD7CkBEGm8uEDJFLdBOYx327Chm T/EuCC8YUDFwBRCfJ1AEYCkQLxBG0TIpQUDeRjqSCcA9AUmRch32N4DvB0A3gQXAB5BjC2BEwjmT 20ylL6hSB5AG8HZAUQQgZmVZYUWUZjUHkUihUgQrOh32YSkgeF4GKF6QFOAtIDV4IGgrIDZecD0q EB32W1BTT0xVRZBjRcFjEDNPIERhIExJVshST11d10RlSnFOMf8OwDCwHwEFwEPhLmIww0KC+xFQ PMExSMNCYlM3XTM3+L9cmC2xWtcsETJAVwB1N9H/LdBfMF+gEVBDAWbrU/cpEMc6ETIwXdcwXjZf klWh1wdANFAvNU5FkTAt0VHlfmJIoVixN4IOsDJBBcB49D0wQUBaBJAt0T9gXDB/PXExoFlnRYE4 wAeAb9E+lxzhSPFCgW1EsSB6b8H/KRBWsS3QSrAuUDGgXDBSMN86ATthCtE10WmdYUSxESD3UGEo 4EFARXKzb/k+8XE4/jxvcEIWcgNDtD1wTKssEH8uERDwYqIfkAWwcIIu0Hj+PHHQQUAGEG5BTjE3 gmTA13APcRpyAzFBQExzMSkQ62TAcgN0BRB2Y7dSUlMsz0hyabso4HZSVmNK5jSg/xDhPWI8kTCg J3A6sW6UAhDfUjBMQDnBN4Iu8HIAwGPVv0IoNvEAkASBNLV/cUk8of9udlJSRW8uNi7BiNRx9FJS 74gyAZBkAod3aIQzC4AOsP914UKSPwCINjpCO5E5EUNhe1JSRjtNSkFzUXFHfTp5/y3hBABkE3Jz LtByA0SSXRHuMylBkrFBQFBCwT1TNOD9AJBtKRCORHYCecGPOgrBv1MOaRF/cWJgOLE5wTFe0f9s QnIDSnALIDGAMIEiwIlEv3IDQzQ+8Y1SQXI3EHU+Ee+akV8wk7WUlTE7w1iiNED/B3Ay8nnAm7FR sB2UKRBZYZ89YjTxhSEyUTGRRlVtUQFgxkVYUE9ORU7wQ0lBTEABh6lNoYzl/5hXiNQpEC4QBAA9 4TnBQCP/LsGOpo1jH5AYwFbSdmAdlL8vt3MiKRB4nFMrTKVWVyH/fNRCJGfPaNZkwGl+aw6apv4o m3gvNZolpqFUojeCTEB/jwFioanKf/aBj2bvHclbhkSKgCjgVklEQSYg/lAFsTeCYu9j+H0SZOQz 2P1lej9hMB7gLdA98qazKRB3NPFpEjGgKDGgX6A00ilfUKECMDOLayNkYTxpAi/rX6FBQCjB4iJM QDcgJxH9DrAiLzVhIHazOJEKsQWQ/mU80DBxQnJIsZd0QUBhIPmfcU9Ls5iAoLSHHZSR0t9CJTMg BuA80J/iYTeAIgD/YTB9kjeCff8t0MHiu5F9Z/92s58mjkTG0wZyMxGgAS7Q9y3UY/YAcHUyoTrT XDAYUf9JRF8wQiXPwTIwdnKQc87Zd4yFNMALgF66MLaxQnJn+07hWDN1KRB14VwRYpNAUfeCndRM ORFyGMALYDJRXsX/aQFfUV+Qu5IuUAIhMKFOMedvYBTge/E9MykQMLA00f9lTtE4NOB/hCnAjaEq AGTv92X1tv3GQT8dmjHDHZodlAJ94wAeAEIQAQAAAEEAAAA8SENaUk1RJEl5eUZNajNkazBhMXl4 WnVER29scDVIQ3BVRHZiSWVWMkVHMTJCV2NfVTYzQGJvbC5jb20uYnI+AAAAAAMAkhABAAAAAgEU OgEAAAAQAAAAFlKwcNoyokab1y3F/zD7AQMA3j+vbwAAAwAJWQEAAAADAEBlAAAAAAsAE4AIIAYA AAAAAMAAAAAAAABGAAAAAAOFAAAAAAAAAwAVgAggBgAAAAAAwAAAAAAAAEYAAAAAEIUAAAAAAAAD ABuACCAGAAAAAADAAAAAAAAARgAAAABShQAA45ABAAMASYAIIAYAAAAAAMAAAAAAAABGAAAAAAGF AAAAAAAAQABKgAggBgAAAAAAwAAAAAAAAEYAAAAAYIUAAABAIw5DAAAAHgB3gAggBgAAAAAAwAAA AAAAAEYAAAAAVIUAAAEAAAAFAAAAMTAuMAAAAAALAHiACCAGAAAAAADAAAAAAAAARgAAAAAGhQAA AAAAAAsAfIAIIAYAAAAAAMAAAAAAAABGAAAAAA6FAAAAAAAAAwB/gAggBgAAAAAAwAAAAAAAAEYA AAAAGIUAAAAAAAALAJSACCAGAAAAAADAAAAAAAAARgAAAACChQAAAQAAAAIB+A8BAAAAEAAAAIgc rZ0aac5AvY6pdQCeiJMCAfoPAQAAABAAAACIHK2dGmnOQL2OqXUAnoiTAgH7DwEAAABwAAAAAAAA ADihuxAF5RAaobsIACsqVsIAAG1zcHN0LmRsbAAAAAAATklUQfm/uAEAqgA32W4AAABDOlxXSU5E T1dTXEFwcGxpY2F0aW9uIERhdGFcTWljcm9zb2Z0XE91dGxvb2tcT3V0bG9vazEucHN0AAMA/g8F AAAAAwANNP03AgACARQ0AQAAABAAAABOSVRB+b+4AQCqADfZbgAAAgF/AAEAAAAxAAAAMDAwMDAw MDA4ODFDQUQ5RDFBNjlDRTQwQkQ4RUE5NzUwMDlFODg5M0U0RTMyMTAwAAAAAAMABhC60y+7AwAH EPQHAAADABAQAQAAAAMAERAAAAAAHgAIEAEAAABlAAAALS0tLS1PUklHSU5BTE1FU1NBR0UtLS0t LUZST006T1dORVItT0JNLUxAU1VDVVJJTUFUUFVDLVJJT0JSTUFJTFRPOk9XTkVSLU9CTS1MQFNV Q1VSSU1BVFBVQy1SSU9CUk9OQgAAAAAR2g== ------=_NextPart_000_003A_01C2FD5E.D448F720-- ========================================================================= Instruções para entrar na lista, sair da lista e usar a lista em http://www.mat.puc-rio.br/~nicolau/olimp/obm-l.html O administrador desta lista é ========================================================================= From owner-obm-l@sucuri.mat.puc-rio.br Tue Apr 8 00:16:46 2003 Return-Path: Received: (from majordom@localhost) by sucuri.mat.puc-rio.br (8.9.3/8.9.3) id AAA03836 for obm-l-MTTP; Tue, 8 Apr 2003 00:15:26 -0300 Received: from smtp-27.ig.com.br (smtp-27.ig.com.br [200.226.132.159]) by sucuri.mat.puc-rio.br (8.9.3/8.9.3) with SMTP id AAA03832 for ; Tue, 8 Apr 2003 00:15:23 -0300 Received: (qmail 30575 invoked from network); 8 Apr 2003 03:14:51 -0000 Received: from unknown (HELO henrique) (200.140.6.35) by smtp-27.ig.com.br with SMTP; 8 Apr 2003 03:14:51 -0000 Message-ID: <005801c2fd7d$0728d110$019da8c0@henrique> From: "=?iso-8859-1?Q?Henrique_Patr=EDcio_Sant'Anna_Branco?=" To: References: Subject: [obm-l] =?iso-8859-1?Q?Re:_=5Bobm-l=5D_fun=E7=E3o_exponencial_=28de_novo=29?= Date: Tue, 8 Apr 2003 00:14:54 -0300 MIME-Version: 1.0 Content-Type: text/plain; charset="iso-8859-1" Content-Transfer-Encoding: 8bit X-Priority: 3 X-MSMail-Priority: Normal X-Mailer: Microsoft Outlook Express 6.00.2800.1106 X-MimeOLE: Produced By Microsoft MimeOLE V6.00.2800.1106 Sender: owner-obm-l@sucuri.mat.puc-rio.br Precedence: bulk Reply-To: obm-l@mat.puc-rio.br > Resolva as equações em R+: > a) x^(x^2 - 5x + 6) = 1 > [SOLUÇÃO DO LIVRO]: > Devemos examinar inicialmente se 0 ou 1 são soluções da > equação. > Substituindo x = 0 na equação proposta, temos: > 0^6 = 1 (falso) > logo, 0 não é solução. > Substituindo x = 1 na equação, temos: > 1^2 = 1 (verdadeiro) > logo 1 é solução da equação. > [DÚVIDA] Por que examinar inicialmente se 0 e 1 são > soluções? Digo isso pois, se x = a (a = base), então > temos 0 < x =/= 1. (=/= "diferente") Olá, Renatinha Na verdade, essa função que você colocou é e^((x^2 - 5x + 6)*log(x)) = 1. Não há problema nenhum em considerar x = 1, pois log(1) = 0 está definido para os reais. Portanto, isso aí acaba em e^0 que, de fato, é igual 1. Se tentarmos usar o 0, log(0) não está definido, e essa expressão não faz sentido. Note que a equação ainda tem outras raízes, se assumirmos x^2 - 5x + 6 = 0. Talvez o livro queira examinar primeiramente o zero ou o um por serem fáceis de verificar e úteis para reduzir o grau de certas equações polinomiais. Por exemplo, -3x^3 + 2x^2 + 1 = 0. É fácil ver que 1 é raiz da equação pois a soma dos coeficientes é negativa. Portanto, se 1 é raiz, podemos reduzir essa equação a uma de grau dois e achar facilmente as outras raízes. Abraços, Henrique. ========================================================================= Instruções para entrar na lista, sair da lista e usar a lista em http://www.mat.puc-rio.br/~nicolau/olimp/obm-l.html O administrador desta lista é ========================================================================= From owner-obm-l@sucuri.mat.puc-rio.br Tue Apr 8 07:44:32 2003 Return-Path: Received: (from majordom@localhost) by sucuri.mat.puc-rio.br (8.9.3/8.9.3) id HAA08363 for obm-l-MTTP; Tue, 8 Apr 2003 07:43:01 -0300 Received: from pretoria.ime.unicamp.br (pretoria.ime.unicamp.br [143.106.22.7]) by sucuri.mat.puc-rio.br (8.9.3/8.9.3) with ESMTP id HAA08359 for ; Tue, 8 Apr 2003 07:42:58 -0300 Received: from lei060 (lei060.lei.ime.unicamp.br [143.106.118.60]) by pretoria.ime.unicamp.br (8.9.0/8.9.0) with ESMTP id HAA16414 for ; Tue, 8 Apr 2003 07:42:35 -0300 (EST) From: "Mario Salvatierra Junior" To: Subject: RES: [obm-l] sobre matrizes simetricas Date: Tue, 8 Apr 2003 07:42:18 -0300 Message-ID: <000401c2fdbb$87a42b30$3c766a8f@lei.ime.unicamp.br> MIME-Version: 1.0 Content-Type: text/plain; charset="iso-8859-1" X-Priority: 3 (Normal) X-MSMail-Priority: Normal X-Mailer: Microsoft Outlook, Build 10.0.2627 X-MimeOLE: Produced By Microsoft MimeOLE V6.00.2600.0000 Importance: Normal In-Reply-To: <3E92323A.5050508@niski.com> Content-Transfer-Encoding: 8bit X-MIME-Autoconverted: from quoted-printable to 8bit by sucuri.mat.puc-rio.br id HAA08360 Sender: owner-obm-l@sucuri.mat.puc-rio.br Precedence: bulk Reply-To: obm-l@mat.puc-rio.br Numa matriz simétrica só precisamos dos seus elementos da diagonal e dos elementos acima da diagonal, pois os q estão abaixo da diagonal são os mesmos dos que estão acima. Então a quantidade de elementos necessários para representar uma matriz simétrica NxN são: N+ (N-1)+ (N-2)+..+1=N(N+1)/2 (Progressao aritmética) N=numero de elementos da diagonal N-1= numero de elementos na primeira superdiagonal......etc..... -----Mensagem original----- De: owner-obm-l@sucuri.mat.puc-rio.br [mailto:owner-obm-l@sucuri.mat.puc-rio.br] Em nome de niski Enviada em: segunda-feira, 7 de abril de 2003 23:22 Para: obm-l@mat.puc-rio.br Assunto: [obm-l] sobre matrizes simetricas Olá colegas. Estou me embanando para calcular o numero maximo de entradas distintas que uma matriz simetrica e anti-simétrica comportam, gostaria de ler sugestoes e resolucoes. Muito obrigado Niski -- [about him:] It is rare to find learned men who are clean, do not stink and have a sense of humour. -Gottfried Whilhem Leibniz ======================================================================== = Instruções para entrar na lista, sair da lista e usar a lista em http://www.mat.puc-rio.br/~nicolau/olimp/obm-l.html O administrador desta lista é ======================================================================== = ========================================================================= Instruções para entrar na lista, sair da lista e usar a lista em http://www.mat.puc-rio.br/~nicolau/olimp/obm-l.html O administrador desta lista é ========================================================================= From owner-obm-l@sucuri.mat.puc-rio.br Tue Apr 8 13:46:40 2003 Return-Path: Received: (from majordom@localhost) by sucuri.mat.puc-rio.br (8.9.3/8.9.3) id NAA14355 for obm-l-MTTP; Tue, 8 Apr 2003 13:43:15 -0300 Received: from shannon.bol.com.br (shannon.bol.com.br [200.221.24.13]) by sucuri.mat.puc-rio.br (8.9.3/8.9.3) with ESMTP id NAA14351 for ; Tue, 8 Apr 2003 13:43:07 -0300 Received: from bol.com.br (200.221.24.140) by shannon.bol.com.br (5.1.071) id 3E76735F006DA110 for obm-l@mat.puc-rio.br; Tue, 8 Apr 2003 13:42:36 -0300 Date: Tue, 8 Apr 2003 13:42:36 -0300 Message-Id: Subject: [obm-l] f. exponencial (agradecimentos) MIME-Version: 1.0 Content-Type: text/plain;charset="iso-8859-1" From: "renatinha15a" To: "obm" X-XaM3-API-Version: 2.4 R3 ( B4 ) X-SenderIP: 200.241.108.194 Content-Transfer-Encoding: 8bit X-MIME-Autoconverted: from quoted-printable to 8bit by sucuri.mat.puc-rio.br id NAA14352 Sender: owner-obm-l@sucuri.mat.puc-rio.br Precedence: bulk Reply-To: obm-l@mat.puc-rio.br Obrigada Artur e Henrique, pelo esclarecimento, atenção, e paciência. []´s Renatinha __________________________________________________________________________ E-mail Premium BOL Antivírus, anti-spam e até 100 MB de espaço. Assine já! http://email.bol.com.br/ ========================================================================= Instruções para entrar na lista, sair da lista e usar a lista em http://www.mat.puc-rio.br/~nicolau/olimp/obm-l.html O administrador desta lista é ========================================================================= From owner-obm-l@sucuri.mat.puc-rio.br Tue Apr 8 14:04:26 2003 Return-Path: Received: (from majordom@localhost) by sucuri.mat.puc-rio.br (8.9.3/8.9.3) id OAA14838 for obm-l-MTTP; Tue, 8 Apr 2003 14:02:37 -0300 Received: from itaqui.terra.com.br (itaqui.terra.com.br [200.176.3.19]) by sucuri.mat.puc-rio.br (8.9.3/8.9.3) with ESMTP id OAA14834 for ; Tue, 8 Apr 2003 14:02:31 -0300 Received: from altamira.terra.com.br (altamira.terra.com.br [200.176.3.40]) by itaqui.terra.com.br (Postfix) with ESMTP id 6017C3BCC3E for ; Tue, 8 Apr 2003 14:01:54 -0300 (BRT) Received: from niski.com (unknown [200.148.194.31]) (authenticated user fniski) by altamira.terra.com.br (Postfix) with ESMTP id F23F53DC101 for ; Tue, 8 Apr 2003 14:01:43 -0300 (BRT) Message-ID: <3E93006D.7000706@niski.com> Date: Tue, 08 Apr 2003 10:01:33 -0700 From: niski User-Agent: Mozilla/5.0 (Windows; U; Windows NT 5.1; en-US; rv:1.0.2) Gecko/20030208 Netscape/7.02 X-Accept-Language: en-us, en MIME-Version: 1.0 To: obm-l@mat.puc-rio.br Subject: Re: RES: [obm-l] sobre matrizes simetricas References: <000401c2fdbb$87a42b30$3c766a8f@lei.ime.unicamp.br> Content-Type: text/plain; charset=ISO-8859-1; format=flowed Content-Transfer-Encoding: 8bit Sender: owner-obm-l@sucuri.mat.puc-rio.br Precedence: bulk Reply-To: obm-l@mat.puc-rio.br Mario Salvatierra Junior wrote: > Numa matriz simétrica só precisamos dos seus elementos da diagonal e dos > elementos acima da diagonal, pois os q estão abaixo da diagonal são os > mesmos dos que estão acima. Então a quantidade de elementos necessários > para representar uma matriz simétrica NxN são: > N+ (N-1)+ (N-2)+..+1=N(N+1)/2 (Progressao aritmética) > N=numero de elementos da diagonal > N-1= numero de elementos na primeira superdiagonal......etc..... Estava cometendo uma besteira! Só agora que vi que [a e i k] poderia ser simetrica...estava considerando apenas [e f m n] [i m k o] [k n o p] [a e i m] que poderia ser simetrica! [e f m n] [i m k o] [m n o p] de qq forma, obrigado Mario! -- [about him:] It is rare to find learned men who are clean, do not stink and have a sense of humour. -Gottfried Whilhem Leibniz ========================================================================= Instruções para entrar na lista, sair da lista e usar a lista em http://www.mat.puc-rio.br/~nicolau/olimp/obm-l.html O administrador desta lista é ========================================================================= From owner-obm-l@sucuri.mat.puc-rio.br Tue Apr 8 16:02:23 2003 Return-Path: Received: (from majordom@localhost) by sucuri.mat.puc-rio.br (8.9.3/8.9.3) id PAA18170 for obm-l-MTTP; Tue, 8 Apr 2003 15:59:39 -0300 Received: (from nicolau@localhost) by sucuri.mat.puc-rio.br (8.9.3/8.9.3) id PAA18165 for obm-l@mat.puc-rio.br; Tue, 8 Apr 2003 15:59:38 -0300 Date: Tue, 8 Apr 2003 15:59:38 -0300 From: "Nicolau C. Saldanha" To: obm-l@mat.puc-rio.br Subject: [obm-l] Re: =?iso-8859-1?Q?=5Bobm-l=5D_RE:_=5Bobm-l=5D_fun=E7=E3o_exponencial_=28de_?= =?iso-8859-1?Q?novo=29?= Message-ID: <20030408155938.C17703@sucuri.mat.puc-rio.br> References: Mime-Version: 1.0 Content-Type: text/plain; charset=iso-8859-1 Content-Disposition: inline Content-Transfer-Encoding: 8bit User-Agent: Mutt/1.2.5i In-Reply-To: ; from artur_steiner@usa.net on Mon, Apr 07, 2003 at 11:38:44PM -0700 Sender: owner-obm-l@sucuri.mat.puc-rio.br Precedence: bulk Reply-To: obm-l@mat.puc-rio.br On Mon, Apr 07, 2003 at 11:38:44PM -0700, Artur Costa Steiner wrote: > > Primeiramente, obrigada Artur Costa pela explicação > > sobre a base da função exponencial -confesso que ainda > > me falta conhecimento matemático para entende-la, mas > > estou batalhando para isso.- > Nenhum problema nisso, nada a "confessar". No colegial não se ensina mesmo a > definição precisa da função exponencial. A definição baseada em séries de > potências vem num estágio um pouco adiante. Desculpem eu me meter no meio da conversa, mas para mim a definição mais apropriada de exponencial para o ensino médio é a seguinte: Teorema: Dado a > 1 existe uma única função crescente f: R -> R com f(0) = 1, f(1) = a, f(x+y) = f(x) f(y). Demonstração: Omite-se, é muito técnica. Mas as idéias principais você conhece. Para 0 < a < 1 há um teorema análogo com f decrescente. Dado este teorema, definimos a^b = f(b). Observe que não é necessário falar de limite nem continuidade nem derivada nem séries de potências nem sei lá que outras idéias que só aparecem em cálculo... Vocês provavelmente também conhecem a definição de log(a) para a > 1 como a área da região 1 <= x <= a, 0 <= y <= 1/x. É tão elementar quanto o conceito de área (ahem...). []s, N. ========================================================================= Instruções para entrar na lista, sair da lista e usar a lista em http://www.mat.puc-rio.br/~nicolau/olimp/obm-l.html O administrador desta lista é ========================================================================= From owner-obm-l@sucuri.mat.puc-rio.br Tue Apr 8 16:24:46 2003 Return-Path: Received: (from majordom@localhost) by sucuri.mat.puc-rio.br (8.9.3/8.9.3) id QAA18804 for obm-l-MTTP; Tue, 8 Apr 2003 16:23:16 -0300 Received: from web14311.mail.yahoo.com (web14311.mail.yahoo.com [216.136.224.61]) by sucuri.mat.puc-rio.br (8.9.3/8.9.3) with SMTP id QAA18799 for ; Tue, 8 Apr 2003 16:23:11 -0300 Message-ID: <20030408192239.78086.qmail@web14311.mail.yahoo.com> Received: from [200.17.25.3] by web14311.mail.yahoo.com via HTTP; Tue, 08 Apr 2003 16:22:39 ART Date: Tue, 8 Apr 2003 16:22:39 -0300 (ART) From: =?iso-8859-1?q?Rafael?= Subject: [obm-l] circunferencias To: OBM MIME-Version: 1.0 Content-Type: text/plain; charset=iso-8859-1 Content-Transfer-Encoding: 8bit Sender: owner-obm-l@sucuri.mat.puc-rio.br Precedence: bulk Reply-To: obm-l@mat.puc-rio.br Alguém consegue me ajudar com essa? Eu agradeço se puderem me dar pelo menos o caminho, não precisa deixar a resolução, pode deixar o trabalho braçal comigo. Já tentei muita coisa e até agora nada: Três circunferência de raios r , r' e R são tangentes, duas a duas, externamente. A tangente comun interna às duas primeiras circunferências intercepta a circunferência de raio R nos pontos A e B. Calcular a corda AB. Abraços, Rafael. _______________________________________________________________________ Yahoo! Mail O melhor e-mail gratuito da internet: 6MB de espaço, antivírus, acesso POP3, filtro contra spam. http://br.mail.yahoo.com/ ========================================================================= Instruções para entrar na lista, sair da lista e usar a lista em http://www.mat.puc-rio.br/~nicolau/olimp/obm-l.html O administrador desta lista é ========================================================================= From owner-obm-l@sucuri.mat.puc-rio.br Tue Apr 8 17:20:21 2003 Return-Path: Received: (from majordom@localhost) by sucuri.mat.puc-rio.br (8.9.3/8.9.3) id RAA20618 for obm-l-MTTP; Tue, 8 Apr 2003 17:18:21 -0300 Received: from ns3bind.localdomain ([200.230.34.5]) by sucuri.mat.puc-rio.br (8.9.3/8.9.3) with ESMTP id RAA20614 for ; Tue, 8 Apr 2003 17:18:17 -0300 Received: from servico2 ([200.230.34.224]) by ns3bind.localdomain (8.11.6/X.XX.X) with SMTP id h38KE8r25310 for ; Tue, 8 Apr 2003 17:14:08 -0300 Message-ID: <009301c2fe0c$06f6dea0$3300c57d@bovespa.com> From: "=?iso-8859-1?Q?Cl=E1udio_\=28Pr=E1tica\=29?=" To: References: <20030408155938.C17703@sucuri.mat.puc-rio.br> Subject: [obm-l] =?iso-8859-1?Q?Re:_=5Bobm-l=5D_Re:_=5Bobm-l=5D_RE:_=5Bobm-l=5D_fu?= =?iso-8859-1?Q?n=E7=E3o_exponencial_=28de_novo=29?= Date: Tue, 8 Apr 2003 17:18:24 -0300 MIME-Version: 1.0 Content-Type: text/plain; charset="iso-8859-1" Content-Transfer-Encoding: 8bit X-Priority: 3 X-MSMail-Priority: Normal X-Mailer: Microsoft Outlook Express 5.50.4920.2300 X-MimeOLE: Produced By Microsoft MimeOLE V5.50.4920.2300 Sender: owner-obm-l@sucuri.mat.puc-rio.br Precedence: bulk Reply-To: obm-l@mat.puc-rio.br > > Desculpem eu me meter no meio da conversa, mas para mim a definição > mais apropriada de exponencial para o ensino médio é a seguinte: > > Teorema: > Dado a > 1 existe uma única função crescente f: R -> R com f(0) = 1, > f(1) = a, f(x+y) = f(x) f(y). > > Demonstração: > Omite-se, é muito técnica. Mas as idéias principais você conhece. > > Para 0 < a < 1 há um teorema análogo com f decrescente. > > Dado este teorema, definimos a^b = f(b). > > Observe que não é necessário falar de limite nem continuidade > nem derivada nem séries de potências nem sei lá que outras > idéias que só aparecem em cálculo... > > Vocês provavelmente também conhecem a definição de log(a) > para a > 1 como a área da região 1 <= x <= a, 0 <= y <= 1/x. > É tão elementar quanto o conceito de área (ahem...). > > []s, N. > Oi, Nicolau: Eu gostaria de ver a demonstração do Teorema acima, especialmente a passagem dos racionais para os reais na parte da existência. Com se faz isso sem usar limites? Um abraço, Claudio. ========================================================================= Instruções para entrar na lista, sair da lista e usar a lista em http://www.mat.puc-rio.br/~nicolau/olimp/obm-l.html O administrador desta lista é ========================================================================= From owner-obm-l@sucuri.mat.puc-rio.br Tue Apr 8 18:35:13 2003 Return-Path: Received: (from majordom@localhost) by sucuri.mat.puc-rio.br (8.9.3/8.9.3) id SAA22452 for obm-l-MTTP; Tue, 8 Apr 2003 18:32:07 -0300 Received: from spf1.us.outblaze.com (205-158-62-158.outblaze.com [205.158.62.158]) by sucuri.mat.puc-rio.br (8.9.3/8.9.3) with SMTP id SAA22443 for ; Tue, 8 Apr 2003 18:32:00 -0300 Received: (qmail 19056 invoked from network); 8 Apr 2003 21:30:42 -0000 Received: from unknown (205.158.62.68) by spf1.us.outblaze.com with QMQP; 8 Apr 2003 21:30:42 -0000 Received: (qmail 48763 invoked from network); 8 Apr 2003 21:31:22 -0000 Received: from unknown (HELO ws1-1.us4.outblaze.com) (205.158.62.49) by 205-158-62-153.outblaze.com with SMTP; 8 Apr 2003 21:31:22 -0000 Received: (qmail 75646 invoked by uid 1001); 8 Apr 2003 21:31:21 -0000 Message-ID: <20030408213121.75645.qmail@mail.com> Content-Type: text/plain; charset="iso-8859-1" Content-Disposition: inline Content-Transfer-Encoding: 7bit MIME-Version: 1.0 X-Mailer: MIME-tools 5.41 (Entity 5.404) Received: from [200.153.128.2] by ws1-1.us4.outblaze.com with http for radiohead_fan@popstar.com; Tue, 08 Apr 2003 18:31:21 -0300 From: "Rodrigo I." To: obm-l@mat.puc-rio.br Date: Tue, 08 Apr 2003 18:31:21 -0300 Subject: [obm-l] duvida porcentagem e aritmetica X-Originating-Ip: 200.153.128.2 X-Originating-Server: ws1-1.us4.outblaze.com Sender: owner-obm-l@sucuri.mat.puc-rio.br Precedence: bulk Reply-To: obm-l@mat.puc-rio.br Por favor me ajudem, estou precisando de ajuda na resolução de alguns exercícios: (ESPM-SP) Um artigo de revista especializada informa que um fabricante de sapatos, situado na cidade de Franca, produziu, em 2000, 15% a mais de pares de sapatos que no ano de 1999, quando, ele exportou 25% de sua produção. Já em 2000, exportou 7000 pares de sapatos produzidos a mais que no ano anterior. O número de pares de sapatos produzidos por essa fábrica no ano de 1999 foi: a. 72000 b. 80000 c. 95000 d. 98000 e. 14000 (IME-RJ) Calcule a soma dos números entre 200 e 500 que são múltiplos de 6 e 14, mas não simultaneamente de ambos. -- __________________________________________________________ Sign-up for your own FREE Personalized E-mail at Mail.com http://www.mail.com/?sr=signup ========================================================================= Instruções para entrar na lista, sair da lista e usar a lista em http://www.mat.puc-rio.br/~nicolau/olimp/obm-l.html O administrador desta lista é ========================================================================= From owner-obm-l@sucuri.mat.puc-rio.br Tue Apr 8 19:02:28 2003 Return-Path: Received: (from majordom@localhost) by sucuri.mat.puc-rio.br (8.9.3/8.9.3) id TAA22750 for obm-l-MTTP; Tue, 8 Apr 2003 19:01:07 -0300 Received: from fnn.net ([200.175.38.9]) by sucuri.mat.puc-rio.br (8.9.3/8.9.3) with SMTP id TAA22745 for ; Tue, 8 Apr 2003 19:01:03 -0300 Received: (qmail 179 invoked from network); 8 Apr 2003 21:46:47 -0000 Received: from unknown (HELO windows98) (200.175.39.126) by fnn.net with SMTP; 8 Apr 2003 21:46:47 -0000 Message-ID: <006701c2fe23$a9f4d320$9a75fea9@windows98> From: "Daniel Pini" To: Subject: [obm-l] =?iso-8859-1?B?KyBmYXRvcmHn428=?= Date: Tue, 8 Apr 2003 20:07:43 -0300 MIME-Version: 1.0 Content-Type: multipart/alternative; boundary="----=_NextPart_000_0064_01C2FE0A.83D508E0" X-Priority: 3 X-MSMail-Priority: Normal X-Mailer: Microsoft Outlook Express 5.00.2615.200 X-MimeOLE: Produced By Microsoft MimeOLE V5.00.2615.200 Sender: owner-obm-l@sucuri.mat.puc-rio.br Precedence: bulk Reply-To: obm-l@mat.puc-rio.br This is a multi-part message in MIME format. ------=_NextPart_000_0064_01C2FE0A.83D508E0 Content-Type: text/plain; charset="iso-8859-1" Content-Transfer-Encoding: quoted-printable Seja D=3Da=B2+b=B2+c=B2 onde a e b s=E3o inteiros consecutivos e c=3Dab. = Ent=E3o prove que a raiz quadrada de D =E9 sempre um inteiro =EDmpar. Se xyz=3D1 ent=E3o (1/1+x+xy)+(1/1+y+yz)+(1/1+z+xz) =E9 igual a?R:1 Se 1-y for usado como aproxima=E7=E3o de 1/1+y com | y | menor que 1, a = raz=E3o do erro cometido para o valor exato =E9: R:y=B2 =20 ------=_NextPart_000_0064_01C2FE0A.83D508E0 Content-Type: text/html; charset="iso-8859-1" Content-Transfer-Encoding: quoted-printable
Seja D=3Da=B2+b=B2+c=B2 onde a e b s=E3o = inteiros consecutivos e=20 c=3Dab. Ent=E3o prove que a raiz quadrada de D =E9 sempre um inteiro=20 =EDmpar.
 
Se xyz=3D1 ent=E3o = (1/1+x+xy)+(1/1+y+yz)+(1/1+z+xz) =E9 igual=20 a?R:1
 
Se 1-y for usado como aproxima=E7=E3o de = 1/1+y  com=20 | y | menor que 1, a raz=E3o do erro cometido para o valor = exato =E9:=20 R:y=B2
   
------=_NextPart_000_0064_01C2FE0A.83D508E0-- ========================================================================= Instruções para entrar na lista, sair da lista e usar a lista em http://www.mat.puc-rio.br/~nicolau/olimp/obm-l.html O administrador desta lista é ========================================================================= From owner-obm-l@sucuri.mat.puc-rio.br Tue Apr 8 19:09:55 2003 Return-Path: Received: (from majordom@localhost) by sucuri.mat.puc-rio.br (8.9.3/8.9.3) id TAA22977 for obm-l-MTTP; Tue, 8 Apr 2003 19:08:36 -0300 Received: from fnn.net ([200.175.38.9]) by sucuri.mat.puc-rio.br (8.9.3/8.9.3) with SMTP id TAA22972 for ; Tue, 8 Apr 2003 19:08:33 -0300 Received: (qmail 19769 invoked from network); 8 Apr 2003 21:54:21 -0000 Received: from unknown (HELO windows98) (200.175.39.126) by fnn.net with SMTP; 8 Apr 2003 21:54:21 -0000 Message-ID: <007001c2fe24$b906bee0$9a75fea9@windows98> From: "Daniel Pini" To: Subject: [obm-l] ajuda Date: Tue, 8 Apr 2003 20:15:18 -0300 MIME-Version: 1.0 Content-Type: multipart/alternative; boundary="----=_NextPart_000_006D_01C2FE0B.9311AE20" X-Priority: 3 X-MSMail-Priority: Normal X-Mailer: Microsoft Outlook Express 5.00.2615.200 X-MimeOLE: Produced By Microsoft MimeOLE V5.00.2615.200 Sender: owner-obm-l@sucuri.mat.puc-rio.br Precedence: bulk Reply-To: obm-l@mat.puc-rio.br This is a multi-part message in MIME format. ------=_NextPart_000_006D_01C2FE0B.9311AE20 Content-Type: text/plain; charset="iso-8859-1" Content-Transfer-Encoding: quoted-printable Se 2^8 + 2^11 + 2^n =E9 um quadrado perfeito ent=E3o o valor de n = =E9:R:multiplo de 3 Se a, b, c s=E3o n=FAmeros reais n=E3o nulos tais que = (a+b-c)/c=3D(a-b+c)/b=3D(-a+b+c)/a e=20 x=3D(a+b)(b+c)(c+a)/abc e x =E9 menor que 0. Ent=E3o x =E9 igual a: R: = -1 ------=_NextPart_000_006D_01C2FE0B.9311AE20 Content-Type: text/html; charset="iso-8859-1" Content-Transfer-Encoding: quoted-printable
Se 2^8 + 2^11 + 2^n =E9 um quadrado = perfeito ent=E3o o=20 valor de n =E9:R:multiplo de 3
 
Se a, b, c s=E3o n=FAmeros reais n=E3o nulos = tais que=20 (a+b-c)/c=3D(a-b+c)/b=3D(-a+b+c)/a   e
x=3D(a+b)(b+c)(c+a)/abc e x =E9 menor que 0. = Ent=E3o x =E9 igual=20 a: R: -1
------=_NextPart_000_006D_01C2FE0B.9311AE20-- ========================================================================= Instruções para entrar na lista, sair da lista e usar a lista em http://www.mat.puc-rio.br/~nicolau/olimp/obm-l.html O administrador desta lista é ========================================================================= From owner-obm-l@sucuri.mat.puc-rio.br Tue Apr 8 19:17:18 2003 Return-Path: Received: (from majordom@localhost) by sucuri.mat.puc-rio.br (8.9.3/8.9.3) id TAA23175 for obm-l-MTTP; Tue, 8 Apr 2003 19:16:00 -0300 Received: from fnn.net ([200.175.38.9]) by sucuri.mat.puc-rio.br (8.9.3/8.9.3) with SMTP id TAA23171 for ; Tue, 8 Apr 2003 19:15:56 -0300 Received: (qmail 23440 invoked from network); 8 Apr 2003 22:01:44 -0000 Received: from unknown (HELO windows98) (200.175.39.126) by fnn.net with SMTP; 8 Apr 2003 22:01:44 -0000 Message-ID: <007901c2fe25$c0ed08c0$9a75fea9@windows98> From: "Daniel Pini" To: Subject: [obm-l] ajuda2 Date: Tue, 8 Apr 2003 20:22:41 -0300 MIME-Version: 1.0 Content-Type: multipart/alternative; boundary="----=_NextPart_000_0076_01C2FE0C.9ACEC520" X-Priority: 3 X-MSMail-Priority: Normal X-Mailer: Microsoft Outlook Express 5.00.2615.200 X-MimeOLE: Produced By Microsoft MimeOLE V5.00.2615.200 Sender: owner-obm-l@sucuri.mat.puc-rio.br Precedence: bulk Reply-To: obm-l@mat.puc-rio.br This is a multi-part message in MIME format. ------=_NextPart_000_0076_01C2FE0C.9ACEC520 Content-Type: text/plain; charset="iso-8859-1" Content-Transfer-Encoding: quoted-printable O valor de (95*94*93*92+1)^1/2 =E9 igual a: R:8741 =20 Obs: qual o meio onde n=E3o se efetua esse produto? =20 O valor de [52+6(43)^1/2]^3/2 - [52-6(43)^1/2]^3/2? R;828 Por favor me ajudem a sair da atual situa=E7=E3o. Grato, Daniel. ------=_NextPart_000_0076_01C2FE0C.9ACEC520 Content-Type: text/html; charset="iso-8859-1" Content-Transfer-Encoding: quoted-printable
O valor de (95*94*93*92+1)^1/2 =E9 = igual a:=20 R:8741   
Obs: qual o meio onde n=E3o se efetua esse=20 produto?   
O valor de [52+6(43)^1/2]^3/2 - = [52-6(43)^1/2]^3/2?=20 R;828
Por favor me ajudem a sair da atual = situa=E7=E3o. Grato,=20 Daniel.
------=_NextPart_000_0076_01C2FE0C.9ACEC520-- ========================================================================= Instruções para entrar na lista, sair da lista e usar a lista em http://www.mat.puc-rio.br/~nicolau/olimp/obm-l.html O administrador desta lista é ========================================================================= From owner-obm-l@sucuri.mat.puc-rio.br Tue Apr 8 19:42:17 2003 Return-Path: Received: (from majordom@localhost) by sucuri.mat.puc-rio.br (8.9.3/8.9.3) id TAA24417 for obm-l-MTTP; Tue, 8 Apr 2003 19:40:45 -0300 Received: from Euler.impa.br (euler.impa.br [147.65.1.3]) by sucuri.mat.puc-rio.br (8.9.3/8.9.3) with ESMTP id TAA24412 for ; Tue, 8 Apr 2003 19:40:41 -0300 Received: from Gauss.impa.br (Gauss [147.65.4.1]) by Euler.impa.br (8.11.6p2/8.11.6) with ESMTP id h38MeB207257 for ; Tue, 8 Apr 2003 19:40:11 -0300 (EST) From: Carlos Gustavo Tamm de Araujo Moreira Received: by Gauss.impa.br (8.11.6p2) id h38MeAM12965; Tue, 8 Apr 2003 19:40:10 -0300 (EST) Message-Id: <200304082240.h38MeAM12965@Gauss.impa.br> Subject: Re: [obm-l] ajuda2 To: obm-l@mat.puc-rio.br Date: Tue, 8 Apr 2003 19:40:10 -0300 (EST) In-Reply-To: <007901c2fe25$c0ed08c0$9a75fea9@windows98> from "Daniel Pini" at Apr 8, 3 08:22:41 pm X-Mailer: ELM [version 2.4 PL25] MIME-Version: 1.0 Content-Type: text/plain; charset=US-ASCII Content-Transfer-Encoding: 7bit Sender: owner-obm-l@sucuri.mat.puc-rio.br Precedence: bulk Reply-To: obm-l@mat.puc-rio.br > >O valor de (95*94*93*92+1)^1/2 =E9 igual a: R:8741 =20 >Obs: qual o meio onde n=E3o se efetua esse produto? =20 Note que (u+2)(u+1)u(u-1)+1=(u^2+u-1)^2, e faca u=93. >O valor de [52+6(43)^1/2]^3/2 - [52-6(43)^1/2]^3/2? R;828 Note que 52+6(43)^1/2=(3+(43)^1/2)^2, donde [52+6(43)^1/2]^1/2= =3+(43)^(1/2) e [52-6(43)^1/2]^1/2=(43)^1/2-3 (pois (43)^1/2>3). Assim, a expressao vale (3+(43)^1/2)^3+(3-(43)^1/2)^3= =6.(104-(9-43))=828 (usando x^3+y^3=(x+y)(x^2-xy+y^2)). >Por favor me ajudem a sair da atual situa=E7=E3o. Grato, Daniel. > Abracos, Gugu ========================================================================= Instruções para entrar na lista, sair da lista e usar a lista em http://www.mat.puc-rio.br/~nicolau/olimp/obm-l.html O administrador desta lista é ========================================================================= From owner-obm-l@sucuri.mat.puc-rio.br Tue Apr 8 20:15:35 2003 Return-Path: Received: (from majordom@localhost) by sucuri.mat.puc-rio.br (8.9.3/8.9.3) id UAA25621 for obm-l-MTTP; Tue, 8 Apr 2003 20:13:53 -0300 Received: from traven.uol.com.br (traven.uol.com.br [200.221.29.39]) by sucuri.mat.puc-rio.br (8.9.3/8.9.3) with ESMTP id UAA25617 for ; Tue, 8 Apr 2003 20:13:50 -0300 Received: from gauss ([200.158.97.56]) by traven.uol.com.br (8.9.1/8.9.1) with SMTP id UAA29046 for ; Tue, 8 Apr 2003 20:13:19 -0300 (BRT) Message-ID: <001001c2fe24$fc55e2c0$38619ec8@gauss> From: "Domingos Jr." To: References: <006701c2fe23$a9f4d320$9a75fea9@windows98> Subject: [obm-l] =?iso-8859-1?Q?Re:_=5Bobm-l=5D_+_fatora=E7=E3o?= Date: Tue, 8 Apr 2003 20:17:11 -0300 MIME-Version: 1.0 Content-Type: multipart/alternative; boundary="----=_NextPart_000_000D_01C2FE0B.D68E71B0" X-Priority: 3 X-MSMail-Priority: Normal X-Mailer: Microsoft Outlook Express 6.00.2800.1106 X-MimeOLE: Produced By Microsoft MimeOLE V6.00.2800.1106 Sender: owner-obm-l@sucuri.mat.puc-rio.br Precedence: bulk Reply-To: obm-l@mat.puc-rio.br This is a multi-part message in MIME format. ------=_NextPart_000_000D_01C2FE0B.D68E71B0 Content-Type: text/plain; charset="iso-8859-1" Content-Transfer-Encoding: quoted-printable seja d =3D raizquadrada(D) d =3D D (mod 2) pois x=B2 =3D x (mod 2) para todo x d =3D D =3D a=B2 + (a+1)=B2 + a=B2(a+1)=B2 =3D a + (a+1) + a(a+1) =3D 2a = + 1 + a=B2 + a =3D 1 + a + a =3D 2a + 1 =3D 1 (mod 2) d =E9 =EDmpar note que eu n=E3o provei que D =E9 quadrado perfeito! isso eu deixo a = seu cargo... ----- Original Message -----=20 From: Daniel Pini=20 To: obm-l@mat.puc-rio.br=20 Sent: Tuesday, April 08, 2003 8:07 PM Subject: [obm-l] + fatora=E7=E3o Seja D=3Da=B2+b=B2+c=B2 onde a e b s=E3o inteiros consecutivos e = c=3Dab. Ent=E3o prove que a raiz quadrada de D =E9 sempre um inteiro = =EDmpar. Se xyz=3D1 ent=E3o (1/1+x+xy)+(1/1+y+yz)+(1/1+z+xz) =E9 igual a?R:1 Se 1-y for usado como aproxima=E7=E3o de 1/1+y com | y | menor que 1, = a raz=E3o do erro cometido para o valor exato =E9: R:y=B2 ------=_NextPart_000_000D_01C2FE0B.D68E71B0 Content-Type: text/html; charset="iso-8859-1" Content-Transfer-Encoding: quoted-printable
seja d =3D raizquadrada(D)
d =3D D (mod 2)
 
pois x=B2 =3D x (mod 2) para todo = x
 
d =3D D =3D a=B2 + (a+1)=B2 + = a=B2(a+1)=B2 =3D a + (a+1) + a(a+1)=20 =3D 2a + 1 + a=B2 + a =3D 1 + a + a =3D 2a + 1 =3D 1 (mod = 2)
 
d =E9 =EDmpar
 
note que eu n=E3o provei que D =E9 = quadrado perfeito!=20 isso eu deixo a seu cargo...
 
----- Original Message -----
From:=20 Daniel = Pini
Sent: Tuesday, April 08, 2003 = 8:07=20 PM
Subject: [obm-l] + = fatora=E7=E3o

Seja D=3Da=B2+b=B2+c=B2 onde a e b s=E3o = inteiros consecutivos e=20 c=3Dab. Ent=E3o prove que a raiz quadrada de D =E9 sempre um inteiro=20 =EDmpar.
 
Se xyz=3D1 ent=E3o = (1/1+x+xy)+(1/1+y+yz)+(1/1+z+xz) =E9 igual=20 a?R:1
 
Se 1-y for usado como aproxima=E7=E3o de = 1/1+y =20 com | y | menor que 1, a raz=E3o do erro cometido para o = valor=20 exato =E9: R:y=B2
   =20
------=_NextPart_000_000D_01C2FE0B.D68E71B0-- ========================================================================= Instruções para entrar na lista, sair da lista e usar a lista em http://www.mat.puc-rio.br/~nicolau/olimp/obm-l.html O administrador desta lista é ========================================================================= From owner-obm-l@sucuri.mat.puc-rio.br Tue Apr 8 20:31:01 2003 Return-Path: Received: (from majordom@localhost) by sucuri.mat.puc-rio.br (8.9.3/8.9.3) id UAA26067 for obm-l-MTTP; Tue, 8 Apr 2003 20:29:27 -0300 Received: from Euler.impa.br (euler.impa.br [147.65.1.3]) by sucuri.mat.puc-rio.br (8.9.3/8.9.3) with ESMTP id UAA26062 for ; Tue, 8 Apr 2003 20:29:24 -0300 Received: from Gauss.impa.br (Gauss [147.65.4.1]) by Euler.impa.br (8.11.6p2/8.11.6) with ESMTP id h38NSr209480 for ; Tue, 8 Apr 2003 20:28:53 -0300 (EST) From: Carlos Gustavo Tamm de Araujo Moreira Received: by Gauss.impa.br (8.11.6p2) id h38NSq616889; Tue, 8 Apr 2003 20:28:52 -0300 (EST) Message-Id: <200304082328.h38NSq616889@Gauss.impa.br> Subject: Re: [obm-l] =?iso-8859-1?Q?Re:_=5Bobm-l=5D_+_fatora=E7=E3o?= To: obm-l@mat.puc-rio.br Date: Tue, 8 Apr 2003 20:28:52 -0300 (EST) In-Reply-To: <001001c2fe24$fc55e2c0$38619ec8@gauss> from "Domingos Jr." at Apr 8, 3 08:17:11 pm X-Mailer: ELM [version 2.4 PL25] MIME-Version: 1.0 Content-Type: text/plain; charset=US-ASCII Content-Transfer-Encoding: 7bit Sender: owner-obm-l@sucuri.mat.puc-rio.br Precedence: bulk Reply-To: obm-l@mat.puc-rio.br > >seja d =3D raizquadrada(D) >d =3D D (mod 2) > >pois x=B2 =3D x (mod 2) para todo x > >d =3D D =3D a=B2 + (a+1)=B2 + a=B2(a+1)=B2 =3D a + (a+1) + a(a+1) =3D 2a = >+ 1 + a=B2 + a =3D 1 + a + a =3D 2a + 1 =3D 1 (mod 2) > >d =E9 =EDmpar > >note que eu n=E3o provei que D =E9 quadrado perfeito! isso eu deixo a = >seu cargo... De fato a^2+(a+1)^2+a^2.(a+1)^2=(a^2+a+1)^2. > > ----- Original Message -----=20 > From: Daniel Pini=20 > To: obm-l@mat.puc-rio.br=20 > Sent: Tuesday, April 08, 2003 8:07 PM > Subject: [obm-l] + fatora=E7=E3o > > > Seja D=3Da=B2+b=B2+c=B2 onde a e b s=E3o inteiros consecutivos e = >c=3Dab. Ent=E3o prove que a raiz quadrada de D =E9 sempre um inteiro = >=EDmpar. > > Se xyz=3D1 ent=E3o (1/1+x+xy)+(1/1+y+yz)+(1/1+z+xz) =E9 igual a?R:1 De fato isso 1/(1+x+xy)=1/(1+x+1/z)=z/(1+z+xz) e 1/(1+y+yz)=1/(1+y+1/x)= =x/(1+x+xy)=xz/(1+z+xz), donde 1/(1+x+xy)+1/(1+y+yz)+1/(1+z+xz)= =z/(1+z+xz)+xz/(1+z+xz)+1/(1+z+xz)=(z+xz+1)/(1+z+xz)=1. > > Se 1-y for usado como aproxima=E7=E3o de 1/1+y com | y | menor que 1, = >a raz=E3o do erro cometido para o valor exato =E9: R:y=B2 > De fato 1/(1+y)-(1-y)=(1-(1-y^2))/(1+y)=y^2/(1+y). Abracos, Gugu ========================================================================= Instruções para entrar na lista, sair da lista e usar a lista em http://www.mat.puc-rio.br/~nicolau/olimp/obm-l.html O administrador desta lista é ========================================================================= From owner-obm-l@sucuri.mat.puc-rio.br Tue Apr 8 20:43:22 2003 Return-Path: Received: (from majordom@localhost) by sucuri.mat.puc-rio.br (8.9.3/8.9.3) id UAA26582 for obm-l-MTTP; Tue, 8 Apr 2003 20:41:54 -0300 Received: from traven.uol.com.br (traven.uol.com.br [200.221.29.39]) by sucuri.mat.puc-rio.br (8.9.3/8.9.3) with ESMTP id UAA26570 for ; Tue, 8 Apr 2003 20:41:49 -0300 Received: from u2z7z2 ([200.158.144.162]) by traven.uol.com.br (8.9.1/8.9.1) with ESMTP id UAA20309 for ; Tue, 8 Apr 2003 20:41:18 -0300 (BRT) Message-ID: <006201c2fe28$410e5340$a2909ec8@u2z7z2> From: "Wagner" To: Subject: [obm-l] Fw: sqrt(12a^3 - 3) Date: Tue, 8 Apr 2003 20:40:35 -0300 Organization: Wagner MIME-Version: 1.0 Content-Type: multipart/alternative; boundary="----=_NextPart_000_005F_01C2FE0F.1B54C4E0" X-Priority: 3 X-MSMail-Priority: Normal X-Mailer: Microsoft Outlook Express 5.50.4133.2400 X-MimeOLE: Produced By Microsoft MimeOLE V5.50.4133.2400 Sender: owner-obm-l@sucuri.mat.puc-rio.br Precedence: bulk Reply-To: obm-l@mat.puc-rio.br This is a multi-part message in MIME format. ------=_NextPart_000_005F_01C2FE0F.1B54C4E0 Content-Type: text/plain; charset="iso-8859-1" Content-Transfer-Encoding: quoted-printable ----- Original Message -----=20 From: Cl=E1udio (Pr=E1tica)=20 To: Wagner=20 Sent: Monday, April 07, 2003 4:02 PM Subject: Re: sqrt(12a^3 - 3) Oi, Andr=E9: Gostei muito do problema. Realmente, o UTDF nunca chegou a me passar = pela cabe=E7a - foi, sem d=FAvida, uma =F3tima id=E9ia. Acho que voc=EA deveria mandar esta solu=E7=E3o pra lista.=20 Obrigado e um abra=E7o, Claudio. ----- Original Message -----=20 From: Wagner=20 To: Cl=E1udio (Pr=E1tica)=20 Sent: Friday, April 04, 2003 9:52 PM Subject: Re: sqrt(12a^3 - 3) Oi Cl=E1udio Fui eu que inventei esse problema. A solu=E7=E3o =E9 muito mais dif=EDcil do que parece Foi assim que eu criei esse problema: Se a,b,c s=E3o tr=EAs n=FAmeros inteiros, tais que: (a+b)^3 =3D a^3 + c^3. Ent=E3o segundo o =FAltimo teorema de Fermat=20 (a+b),a ou c =E9 igual a zero. Pois se eles fossem todos n=E3o nulos, isso seria uma contradi=E7=E3o do teorema no caso n=3D3. Se (a+b)^3 =3D a^3 + c^3 . Ent=E3o: ((a+b)/b)^3 =3D (a/b)^3 + (c/b)^3. Para b diferente de zero. Logo: ((a/b)+1)^3 =3D (a/b)^3 + (c/b)^3. Sejam x e y dois n=FAmeros racionais tais que: x=3Da/b e y=3Dc/b. Ent=E3o: (x+1)^3 =3D x^3 + y^3 =3D> x^3 + 3x^2 + 3x + 1 - x^3 - y^3 =3D 0 =3D> 3x^2 + 3x + (1-y^3) =3D 0. Vamos calcular x em fun=E7=E3o de y: delta =3D 9 - 12(1 - y^3) =3D 12y^3 - 3. x =3D( -3 + - sqrt(delta))/6. ( i ) Agora suponha que a+b=3D0. Ent=E3o a =3D -b =3D> 0 =3D -b^3 + c^3 =3D> b=3Dc =3D> y=3D1 Se a =3D 0, b^3 =3D c^3 =3D> b=3Dc =3D> y=3D1=20 Se c =3D 0, (a+b)^3 =3D a^3 =3D> b=3D0 e ent=E3o nem x nem y fazem = sentido. Note que sempre que c =E9 diferente de zero, b =E9 diferente de zero. Se a,b e c forem n=FAmeros inteiros e c for diferente de zero, ent=E3o x e y v=E3o ser n=FAmeros racionais. Mas segundo o teorema de = Fermat isso implica que y =3D 1. Logo x =E9 racional se e somente se y =3D 1. Mas temos de ( i ) que x =E9 racional se e somente se sqrt(delta) =3D = sqrt(12y^3 - 3) for racional. Logo sqrt(12y^3 - 3) s=F3 =E9 racional se y=3D1. Na verdade a maior dificuldade dessa solu=E7=E3o =E9 associar o = problema ao teorema de Fermat (o que =E9 na verdade muito dif=EDcil) Andr=E9 T. ----- Original Message -----=20 From: Cl=E1udio (Pr=E1tica)=20 To: timpa@uol.com.br=20 Cc: claudio.buffara@terra.com.br=20 Sent: Friday, April 04, 2003 5:00 PM Subject: sqrt(12a^3 - 3) Oi, Andre: Voc=EA j=E1 conseguiu provar que se "a" e sqrt(12a^3 - 3) s=E3o = racionais, ent=E3o a =3D 1? De onde voc=EA tirou esse problema? Parece que =E9 f=E1cil mas h=E1 dias eu tenho tentado sem sucesso. Um abra=E7o, Claudio. ------=_NextPart_000_005F_01C2FE0F.1B54C4E0 Content-Type: text/html; charset="iso-8859-1" Content-Transfer-Encoding: quoted-printable
 
----- Original Message -----=20
From: Cl=E1udio = (Pr=E1tica)
To: Wagner
Sent: Monday, April 07, 2003 4:02 PM
Subject: Re: sqrt(12a^3 - 3)

Oi, Andr=E9:
 
Gostei muito do problema. Realmente, o = UTDF nunca=20 chegou a me passar pela cabe=E7a - foi, sem d=FAvida, uma =F3tima=20 id=E9ia.
 
Acho que voc=EA deveria mandar esta = solu=E7=E3o pra=20 lista.
 
Obrigado e um abra=E7o,
Claudio.
----- Original Message -----
From:=20 Wagner
To: Cl=E1udio = (Pr=E1tica)
Sent: Friday, April 04, 2003 = 9:52=20 PM
Subject: Re: sqrt(12a^3 - = 3)

Oi Cl=E1udio
 
Fui eu que inventei esse = problema.
A solu=E7=E3o =E9 muito mais = dif=EDcil do que=20 parece
Foi assim que eu criei esse=20 problema:
Se a,b,c s=E3o tr=EAs n=FAmeros = inteiros, tais=20 que:
(a+b)^3 =3D a^3 + c^3.
Ent=E3o segundo o =FAltimo teorema de = Fermat=20
(a+b),a ou c =E9 igual a zero. Pois = se eles=20 fossem
todos n=E3o nulos, isso seria uma = contradi=E7=E3o=20 do
teorema no caso n=3D3.
Se (a+b)^3 =3D a^3 + c^3 . = Ent=E3o:
((a+b)/b)^3 =3D (a/b)^3 + (c/b)^3. = Para b diferente=20 de zero.
Logo: ((a/b)+1)^3 =3D (a/b)^3 +=20 (c/b)^3.
Sejam x e y dois n=FAmeros racionais = tais=20 que:
x=3Da/b e y=3Dc/b.
Ent=E3o: (x+1)^3 =3D x^3 + y^3 = =3D>
x^3 + 3x^2 + 3x + 1 - x^3 - y^3 =3D 0 = =3D>
3x^2 + 3x + (1-y^3) =3D = 0.
Vamos calcular x em fun=E7=E3o de = y:
delta =3D 9 - 12(1 - y^3) =3D 12y^3 - = 3.
x =3D( -3 + - sqrt(delta))/6. ( i = )
 
Agora suponha que = a+b=3D0.
Ent=E3o a =3D -b =3D> 0 =3D -b^3 + = c^3 =3D> b=3Dc =3D>=20 y=3D1
Se a =3D 0, b^3 =3D c^3 =3D> b=3Dc = =3D>=20 y=3D1 
Se c =3D 0, (a+b)^3 =3D a^3 =3D> = b=3D0 e ent=E3o nem x=20 nem y fazem sentido.
Note que sempre que c =E9 diferente = de zero, b =E9=20 diferente de zero.
Se a,b e c forem n=FAmeros = inteiros e c for=20 diferente de zero,
ent=E3o x e y v=E3o ser n=FAmeros = racionais. Mas=20 segundo o teorema de Fermat
isso implica que y =3D 1. Logo x = =E9 racional se=20 e somente se y =3D 1.
Mas temos de ( i ) que x =E9 racional = se e somente=20 se sqrt(delta) =3D sqrt(12y^3 - 3)
for racional. Logo sqrt(12y^3 - 3) = s=F3 =E9 racional=20 se y=3D1.
 
Na verdade a maior dificuldade dessa = solu=E7=E3o =E9=20 associar o problema ao teorema
de Fermat (o que =E9 na verdade muito = dif=EDcil)
 
 
Andr=E9 T.
 
 
----- Original Message -----
From:=20 Cl=E1udio = (Pr=E1tica)
Cc: claudio.buffara@terra.com.br= =20
Sent: Friday, April 04, 2003 = 5:00=20 PM
Subject: sqrt(12a^3 - = 3)

Oi, Andre:
 
Voc=EA j=E1 conseguiu provar que se = "a" e=20 sqrt(12a^3 - 3) s=E3o racionais, ent=E3o a =3D 1?
De onde voc=EA tirou esse = problema?
 
Parece que =E9 f=E1cil mas h=E1 = dias eu tenho tentado=20 sem sucesso.
 
Um abra=E7o,
Claudio.
 
------=_NextPart_000_005F_01C2FE0F.1B54C4E0-- ========================================================================= Instruções para entrar na lista, sair da lista e usar a lista em http://www.mat.puc-rio.br/~nicolau/olimp/obm-l.html O administrador desta lista é ========================================================================= From owner-obm-l@sucuri.mat.puc-rio.br Tue Apr 8 21:03:33 2003 Return-Path: Received: (from majordom@localhost) by sucuri.mat.puc-rio.br (8.9.3/8.9.3) id VAA27348 for obm-l-MTTP; Tue, 8 Apr 2003 21:02:12 -0300 Received: from smtp-29.ig.com.br (smtp-29.ig.com.br [200.226.132.157]) by sucuri.mat.puc-rio.br (8.9.3/8.9.3) with SMTP id VAA27344 for ; Tue, 8 Apr 2003 21:02:08 -0300 Received: (qmail 17869 invoked from network); 9 Apr 2003 00:01:49 -0000 Received: from unknown (HELO henrique) (200.140.80.157) by smtp-29.ig.com.br with SMTP; 9 Apr 2003 00:01:49 -0000 Message-ID: <013001c2fe2b$32c70db0$019da8c0@henrique> From: "=?iso-8859-1?Q?Henrique_Patr=EDcio_Sant'Anna_Branco?=" To: References: <20030408213121.75645.qmail@mail.com> Subject: Re: [obm-l] duvida porcentagem e aritmetica Date: Tue, 8 Apr 2003 21:01:40 -0300 MIME-Version: 1.0 Content-Type: text/plain; charset="iso-8859-1" Content-Transfer-Encoding: 8bit X-Priority: 3 X-MSMail-Priority: Normal X-Mailer: Microsoft Outlook Express 6.00.2800.1106 X-MimeOLE: Produced By Microsoft MimeOLE V6.00.2800.1106 Sender: owner-obm-l@sucuri.mat.puc-rio.br Precedence: bulk Reply-To: obm-l@mat.puc-rio.br Olá, Rodrigo! > (IME-RJ) Calcule a soma dos números entre 200 e 500 que são múltiplos de 6 e 14, mas não simultaneamente de ambos. Esses números estão em P.A., portanto, podemos aplicar a fórmula da soma das duas P.A.'s com um detalhe: devemos subtrair, no final, duas vezes a soma dos múltiplos de 6 e 14, pois estes foram contados duas vezes (quando somamos os múltiplos de 6 e de 14 simultaneamente). Vamos calcular os múltiplos de 6. Primeiro, devemos descobrir quantos são: 498 = 204 + (n-1)*6 ==> n = 50 Calculando a soma dos múltiplos de 6, temos (204+498)*50/2 = 17550 Fazendo o mesmo para os múltiplos de 14. Aqui pode surgir uma dificuldade, em determinar quais números são divisíveis por 14 mas calculando: 200 mod 14 = 4 201 mod 14 = 5 202 mod 14 = 6 203 mod 14 = 7 (etc) Assim, temos que 210 mod 14 = 0, portanto, é o primeiro valor da P.A. De modo análogo, temos que o último valor é 490. 490 = 210 + (n-1)*6 ==> n = 21 (210+490)*21/2 = 7350 Agora, temos que mmc(6,14) = 42, ou seja, os múltiplos simultâneos de 6 e 14 são múltiplos de 42. Primeiro termo = 210, último termo = 462 ==> 462 = 210 + (n-1)*42 ==> n = 7 Soma: (210 + 462)*7/2 = 2352 Portanto, a resposta do problema é dada por: 17550 + 7350 - 2*2352 = 20196 Abraços, Henrique. ========================================================================= Instruções para entrar na lista, sair da lista e usar a lista em http://www.mat.puc-rio.br/~nicolau/olimp/obm-l.html O administrador desta lista é ========================================================================= From owner-obm-l@sucuri.mat.puc-rio.br Tue Apr 8 21:04:44 2003 Return-Path: Received: (from majordom@localhost) by sucuri.mat.puc-rio.br (8.9.3/8.9.3) id VAA27393 for obm-l-MTTP; Tue, 8 Apr 2003 21:03:28 -0300 Received: from lampiao.digi.com.br (lampiao.digi.com.br [200.241.100.60]) by sucuri.mat.puc-rio.br (8.9.3/8.9.3) with ESMTP id VAA27389 for ; Tue, 8 Apr 2003 21:03:24 -0300 Received: from p8c7y1fe9gyrelp.digi.com.br (host175.e.digizap.com.br [200.249.8.175]) by lampiao.digi.com.br (8.11.6/8.11.6) with ESMTP id h3900n018992 for ; Tue, 8 Apr 2003 21:00:56 -0300 Message-Id: <5.2.0.9.0.20030408205344.00b17f10@mail.digi.com.br> X-Sender: benedito@mail.digi.com.br X-Mailer: QUALCOMM Windows Eudora Version 5.2.0.9 Date: Tue, 08 Apr 2003 21:01:18 -0300 To: obm-l@mat.puc-rio.br From: benedito Subject: Re: [obm-l] Cartas de Ajuda In-Reply-To: <5.2.0.9.0.20030405180556.00b17a90@mail.digi.com.br> References: Mime-Version: 1.0 Content-Type: text/plain; charset="iso-8859-1"; format=flowed X-MailScanner: Found to be clean Content-Transfer-Encoding: 8bit X-MIME-Autoconverted: from quoted-printable to 8bit by sucuri.mat.puc-rio.br id VAA27390 Sender: owner-obm-l@sucuri.mat.puc-rio.br Precedence: bulk Reply-To: obm-l@mat.puc-rio.br Desculpem-me, mas há um claro equívoco. O truque das cartas não tem nada com o Teorema Chinês dos Restos. A brincadeira com o Teorema Chinês dos Restos é aquela em que você pensa num número, por exemplo de 1 a 100, e diz os restos da divisão desse número por 3, 5 e 7. Em seguida, o "mágico" diz o número pensado. O truque das cartas é uma questão de base 3, muito interessante e vale a pena ser feito. Desculpem-me pela má formalização do problema. Benedito At 18:17 5/4/2003 -0300, you wrote: >Prezado Rubens, > >No livro: "Ah, Descobri !" de Martin Gardner, Editora Gradiva (Portugal) >(pag. 116 - 118), tem um truque interessantíssimo, para o "mágico" usar >cartas. Na verdade, trata-se de uma boa motivação para o ensino do Teorema >Chinês dos Restos. Tenho feito este truque para meus estudantes e sinto >que eles ficam altamente motivado. Um colega de Departamento, melhorou o >truque consideravelmente, a ponto de você dizer um número entre >1 e 27 e a carta escolhida estar na posição do número que você escolheu >(e não tornou público). Veja o truque e você vai entender do que estou falando. >Recomendo que você veja os livros do Martin Gardner, que tem muita coisa >escrita de forma a tornar o interesse sempre em alta. >Boa sorte! > >Benedito Freire > >At 16:10 4/4/2003 +0000, you wrote: > > >>Alô pessoal, estou estudando problemas matemáticos em mágicas e truques >>com cartas de baralho. Estou tendo problemas em conseguir bibliografia em >>português em primeiro lugar e depois em espanhol. Em Inglês deve ter >>muita coisa, mas como o meu não é dos melhores e este é um trabalho >>preliminar vou deixar como última opção. >>Agradeço a ajuda >> >>[]' >> >>_________________________________________________________________ >>MSN Hotmail, o maior webmail do Brasil. http://www.hotmail.com >> >>========================================================================= >>Instruções para entrar na lista, sair da lista e usar a lista em >>http://www.mat.puc-rio.br/~nicolau/olimp/obm-l.html >>O administrador desta lista é >>========================================================================= > > >========================================================================= >Instruções para entrar na lista, sair da lista e usar a lista em >http://www.mat.puc-rio.br/~nicolau/olimp/obm-l.html >O administrador desta lista é >========================================================================= ========================================================================= Instruções para entrar na lista, sair da lista e usar a lista em http://www.mat.puc-rio.br/~nicolau/olimp/obm-l.html O administrador desta lista é ========================================================================= From owner-obm-l@sucuri.mat.puc-rio.br Tue Apr 8 21:13:47 2003 Return-Path: Received: (from majordom@localhost) by sucuri.mat.puc-rio.br (8.9.3/8.9.3) id VAA28321 for obm-l-MTTP; Tue, 8 Apr 2003 21:12:24 -0300 Received: from smtp-29.ig.com.br (smtp-29.ig.com.br [200.226.132.157]) by sucuri.mat.puc-rio.br (8.9.3/8.9.3) with SMTP id VAA28297 for ; Tue, 8 Apr 2003 21:12:17 -0300 Received: (qmail 2958 invoked from network); 9 Apr 2003 00:11:57 -0000 Received: from unknown (HELO henrique) (200.140.80.157) by smtp-29.ig.com.br with SMTP; 9 Apr 2003 00:11:57 -0000 Message-ID: <013a01c2fe2c$9d7fcdd0$019da8c0@henrique> From: "=?iso-8859-1?Q?Henrique_Patr=EDcio_Sant'Anna_Branco?=" To: References: <200304082240.h38MeAM12965@Gauss.impa.br> Subject: Re: [obm-l] ajuda2 Date: Tue, 8 Apr 2003 21:11:49 -0300 MIME-Version: 1.0 Content-Type: text/plain; charset="iso-8859-1" Content-Transfer-Encoding: 8bit X-Priority: 3 X-MSMail-Priority: Normal X-Mailer: Microsoft Outlook Express 6.00.2800.1106 X-MimeOLE: Produced By Microsoft MimeOLE V6.00.2800.1106 Sender: owner-obm-l@sucuri.mat.puc-rio.br Precedence: bulk Reply-To: obm-l@mat.puc-rio.br Carlos, (...) > Note que (u+2)(u+1)u(u-1)+1=(u^2+u-1)^2, e faca u=93. (...) Pergunta de quem não entende pouco de fatoração: como você fatorou isso? Sei que (u+2)(u+1)u(u-1)+1 = u^4+2u^3-u^2-2u+1, mas a partir daí, não sei proceder. Abraço, Henrique. ========================================================================= Instruções para entrar na lista, sair da lista e usar a lista em http://www.mat.puc-rio.br/~nicolau/olimp/obm-l.html O administrador desta lista é ========================================================================= From owner-obm-l@sucuri.mat.puc-rio.br Tue Apr 8 22:01:25 2003 Return-Path: Received: (from majordom@localhost) by sucuri.mat.puc-rio.br (8.9.3/8.9.3) id VAA30230 for obm-l-MTTP; Tue, 8 Apr 2003 21:59:56 -0300 Received: from paiol.terra.com.br (paiol.terra.com.br [200.176.3.18]) by sucuri.mat.puc-rio.br (8.9.3/8.9.3) with ESMTP id VAA30226 for ; Tue, 8 Apr 2003 21:59:53 -0300 Received: from botucatu.terra.com.br (botucatu.terra.com.br [200.176.3.78]) by paiol.terra.com.br (Postfix) with ESMTP id B00DD87EC1 for ; Tue, 8 Apr 2003 21:59:22 -0300 (BRT) Received: from [200.177.182.107] (dl-nas6-sao-C8B1B66B.p001.terra.com.br [200.177.182.107]) by botucatu.terra.com.br (Postfix) with ESMTP id 9EAF329C0A2 for ; Tue, 8 Apr 2003 21:59:21 -0300 (BRT) User-Agent: Microsoft-Outlook-Express-Macintosh-Edition/5.02.2022 Date: Tue, 08 Apr 2003 21:57:30 -0300 Subject: Re: [obm-l] + fatora=?ISO-8859-1?B?5+M=?=o From: Claudio Buffara To: Message-ID: In-Reply-To: <006701c2fe23$a9f4d320$9a75fea9@windows98> Mime-version: 1.0 Content-type: multipart/alternative; boundary="MS_Mac_OE_3132683850_830048_MIME_Part" Sender: owner-obm-l@sucuri.mat.puc-rio.br Precedence: bulk Reply-To: obm-l@mat.puc-rio.br > This message is in MIME format. Since your mail reader does not understand this format, some or all of this message may not be legible. --MS_Mac_OE_3132683850_830048_MIME_Part Content-type: text/plain; charset="ISO-8859-1" Content-transfer-encoding: quoted-printable on 08.04.03 20:07, Daniel Pini at daniel@fnn.net wrote: Seja D=3Da=B2+b=B2+c=B2 onde a e b s=E3o inteiros consecutivos e c=3Dab. Ent=E3o prove qu= e a raiz quadrada de D =E9 sempre um inteiro =EDmpar. =20 Se xyz=3D1 ent=E3o (1/1+x+xy)+(1/1+y+yz)+(1/1+z+xz) =E9 igual a?R:1 =20 Se 1-y for usado como aproxima=E7=E3o de 1/1+y com | y | menor que 1, a raz=E3o do erro cometido para o valor exato =E9: R:y=B2 =20 O primeiro sai assim: b =3D a+1 e c =3D ab =3D a(a+1) D =3D a^2 + b^2 + c^2 =3D a^2 + (a+1)^2 + a^2(a+1)^2 =3D=3D> D =3D a^2 + a^2 + 2a + 1 + a^4 + 2a^3 + a^2 =3D=3D> D =3D a^4 + 2a^3 + 3a^2 + 2a + 1 =3D=3D> D =3D (a^2 + a + 1)^2 Mas a^2 + a =3D a(a+1) eh sempre par =3D=3D> a^2 + a + 1 eh sempre impar =3D=3D> sqrt(D) =3D a^2 + a + 1 eh sempre um inteiro impar *********** =20 O terceiro eh: Erro =3D | (1-y) - 1/(1+y) | =3D | (1 - y^2 - 1)/(1+y) | =3D | -y^2/(1+y) = | =3D y^2/(1+y) Valor Exato =3D 1/(1+y) Logo: Erro/Valor Exato =3D y^2. Um abraco, Claudio. --MS_Mac_OE_3132683850_830048_MIME_Part Content-type: text/html; charset="ISO-8859-1" Content-transfer-encoding: quoted-printable Re: [obm-l] + fatora=E7=E3o on 08.04.03 20:07, Daniel Pini at daniel@fnn.net wrote:

Seja D=3Da=B2+b=B2+c=B2 onde a e b s=E3o inteiros cons= ecutivos e c=3Dab. Ent=E3o prove que a raiz quadrada de D =E9 sempre um inteiro =EDm= par.

Se xyz=3D1 ent=E3o (1/1+x+xy)+(1/1+y+yz)+(1/1+z+xz) =E9 igual = a?R:1

Se 1-y for usado como aproxima=E7=E3o de 1/1+y  com | y= | menor que 1, a raz=E3o do erro cometido para o valor exato =E9: R:y=B2
   
O primeiro sai assim:

b =3D a+1   e   c =3D ab =3D a(a+1)

D =3D a^2 + b^2 + c^2 =3D a^2 + (a+1)^2 + a^2(a+1)^2  =3D=3D>
D =3D a^2 + a^2 + 2a + 1 + a^4 + 2a^3 + a^2 =3D=3D>
D =3D a^4 + 2a^3 + 3a^2 + 2a + 1 =3D=3D>
D =3D (a^2 + a + 1)^2

Mas a^2 + a =3D a(a+1) eh sempre par =3D=3D>
a^2 + a + 1 eh sempre impar =3D=3D>
sqrt(D) =3D a^2 + a + 1 eh sempre um inteiro impar

       ***********
       
       O terceiro eh:

       Erro =3D | (1-y) - 1/(1+y) | =3D | (= 1 - y^2 - 1)/(1+y) | =3D | -y^2/(1+y) | =3D y^2/(1+y)

       Valor Exato =3D 1/(1+y)

       Logo: Erro/Valor Exato =3D y^2. &n= bsp;       


       Um abraco,
       Claudio. --MS_Mac_OE_3132683850_830048_MIME_Part-- ========================================================================= Instruções para entrar na lista, sair da lista e usar a lista em http://www.mat.puc-rio.br/~nicolau/olimp/obm-l.html O administrador desta lista é ========================================================================= From owner-obm-l@sucuri.mat.puc-rio.br Tue Apr 8 22:01:26 2003 Return-Path: Received: (from majordom@localhost) by sucuri.mat.puc-rio.br (8.9.3/8.9.3) id WAA30245 for obm-l-MTTP; Tue, 8 Apr 2003 22:00:05 -0300 Received: from itaqui.terra.com.br (itaqui.terra.com.br [200.176.3.19]) by sucuri.mat.puc-rio.br (8.9.3/8.9.3) with ESMTP id WAA30240 for ; Tue, 8 Apr 2003 22:00:01 -0300 Received: from una.terra.com.br (una.terra.com.br [200.176.3.32]) by itaqui.terra.com.br (Postfix) with ESMTP id 88E363BC1CB for ; Tue, 8 Apr 2003 21:59:31 -0300 (BRT) Received: from [200.177.182.107] (dl-nas6-sao-C8B1B66B.p001.terra.com.br [200.177.182.107]) by una.terra.com.br (Postfix) with ESMTP id 4A8562F0069 for ; Tue, 8 Apr 2003 21:59:30 -0300 (BRT) User-Agent: Microsoft-Outlook-Express-Macintosh-Edition/5.02.2022 Date: Tue, 08 Apr 2003 21:58:19 -0300 Subject: Re: [obm-l] ajuda From: Claudio Buffara To: Message-ID: In-Reply-To: <007001c2fe24$b906bee0$9a75fea9@windows98> Mime-version: 1.0 Content-type: multipart/alternative; boundary="MS_Mac_OE_3132683899_833010_MIME_Part" Sender: owner-obm-l@sucuri.mat.puc-rio.br Precedence: bulk Reply-To: obm-l@mat.puc-rio.br > This message is in MIME format. Since your mail reader does not understand this format, some or all of this message may not be legible. --MS_Mac_OE_3132683899_833010_MIME_Part Content-type: text/plain; charset="ISO-8859-1" Content-transfer-encoding: quoted-printable on 08.04.03 20:15, Daniel Pini at daniel@fnn.net wrote: Se 2^8 + 2^11 + 2^n =E9 um quadrado perfeito ent=E3o o valor de n =E9:R:multiplo de 3 =20 Se a, b, c s=E3o n=FAmeros reais n=E3o nulos tais que (a+b-c)/c=3D(a-b+c)/b=3D(-a+b+c)/a e x=3D(a+b)(b+c)(c+a)/abc e x =E9 menor que 0. Ent=E3o x =E9 igual a: R: -1 2^8 + 2^11 =3D 2^8*(1 + 2^3) =3D 16^2*3^2 =3D (16*3)^2 =3D 48^2 =3D=3D> 2^8 + 2^11 + 2^n =3D 48^2 + 2^n 48^2 + 2^n =3D M^2 =3D=3D> 2^n =3D M^2 - 48^2 =3D=3D> 2^n =3D (M - 48)*(M + 48) =3D=3D> M - 48 =3D 2^x e M + 48 =3D 2^(n-x) com x < n - x =3D=3D> subtraindo, obtemos: 96 =3D 2^(n-x) - 2^x =3D=3D> 2^5*3 =3D 2^x*(2^(n-2x) - 1) =3D=3D> 2^5 =3D 2^x e 3 =3D 2^(n-2x) - 1 =3D=3D> x =3D 5 e n-2x =3D 2 =3D=3D> n =3D 12 ****** (a+b-c)/c =3D (a-b+c)/b =3D (-a+b+c)/a =3D=3D> (a+b)/c =3D (a+c)/b =3D (b+c)/a =3D=3D> X =3D (a+b)/c * (a+c)/b * (b+c)/a < 0 Podemos supor s.p.d.g. que a <=3D b <=3D c. Suponhamos que c < 0 ou a > 0 =3D=3D> (a+b)/c, (a+c)/b e (b+c)/a sao positivos =3D=3D> X eh positivo =3D=3D> contradicao =3D=3D> a < 0 e c > 0 Suponhamos que b <> c =3D=3D> (a+b)/c =3D (a+c)/b =3D=3D> ab + b^2 =3D ac + c^2 =3D=3D> a(b - c) =3D c^2 - b^2 =3D=3D> a(b - c) =3D (c + b)(c - b) =3D=3D> -a =3D b+c =3D=3D> (b+c)/a =3D -1 =3D=3D>=20 X =3D -1 Suponhamos agora que b =3Dc =3D=3D> b =3D c > a, pois a < 0 e c > 0 =3D=3D> (a+b)/b =3D 2b/a (=3D (b+c)/a) =3D=3D> a^2 + ab =3D 2b^2 =3D=3D> ab - b^2 =3D b^2 - a^2 =3D=3D> b(a - b) =3D (b + a)(b - a) =3D=3D> -b =3D b + a =3D=3D> a =3D -2b =3D -2c =3D=3D> (a+b)/c =3D (-2b+b)/b =3D -1 =3D=3D> X =3D -1 Assim, de qualquer jeito teremos X =3D -1. Um abraco, Claudio. --MS_Mac_OE_3132683899_833010_MIME_Part Content-type: text/html; charset="ISO-8859-1" Content-transfer-encoding: quoted-printable Re: [obm-l] ajuda on 08.04.03 20:15, Daniel Pini at daniel@fnn.net wrote:

Se 2^8 + 2^11 + 2^n =E9 um quadrado perfeito e= nt=E3o o valor de n =E9:R:multiplo de 3

Se a, b, c s=E3o n=FAmeros reais n=E3o nulos tais que (a+b-c)/= c=3D(a-b+c)/b=3D(-a+b+c)/a   e
x=3D(a+b)(b+c)(c+a)/abc e x =E9 menor que 0. Ent=E3o x =E9 igual a: R: -1


2^8 + 2^11 =3D 2^8*(1 + 2^3) =3D 16^2*3^2 =3D (16*3)^2 =3D 48^2 =3D=3D>
2^8 + 2^11 + 2^n =3D 48^2 + 2^n

48^2 + 2^n =3D M^2 =3D=3D>
2^n =3D M^2 - 48^2 =3D=3D>
2^n =3D (M - 48)*(M + 48) =3D=3D>
M - 48 =3D 2^x  e  M + 48 =3D 2^(n-x)  com  x < n - x =3D=3D= >
subtraindo, obtemos: 96 =3D 2^(n-x) - 2^x =3D=3D>
2^5*3 =3D 2^x*(2^(n-2x) - 1) =3D=3D>
2^5 =3D 2^x  e  3 =3D 2^(n-2x) - 1 =3D=3D>
x =3D 5  e  n-2x =3D 2 =3D=3D> n =3D 12

******

(a+b-c)/c =3D (a-b+c)/b =3D (-a+b+c)/a  =3D=3D>
(a+b)/c =3D (a+c)/b =3D (b+c)/a  =3D=3D>
X =3D (a+b)/c * (a+c)/b * (b+c)/a < 0

Podemos supor s.p.d.g. que a <=3D b <=3D c.

Suponhamos que c < 0 ou a > 0 =3D=3D>
(a+b)/c, (a+c)/b e (b+c)/a sao positivos =3D=3D>
X eh positivo =3D=3D>
contradicao =3D=3D>
a < 0 e c > 0

Suponhamos que b <> c =3D=3D>
(a+b)/c =3D (a+c)/b =3D=3D>
ab + b^2 =3D ac + c^2 =3D=3D>
a(b - c) =3D c^2 - b^2 =3D=3D>
a(b - c) =3D (c + b)(c - b) =3D=3D>
-a =3D b+c =3D=3D>
(b+c)/a =3D -1 =3D=3D>
X =3D -1

Suponhamos agora que b =3Dc =3D=3D>
b =3D c > a, pois a < 0 e c > 0 =3D=3D>
(a+b)/b =3D 2b/a (=3D (b+c)/a) =3D=3D>
a^2 + ab =3D 2b^2 =3D=3D>
ab - b^2 =3D b^2 - a^2 =3D=3D>
b(a - b) =3D (b + a)(b - a) =3D=3D>
-b =3D b + a =3D=3D>
a =3D -2b =3D -2c =3D=3D>
(a+b)/c =3D (-2b+b)/b =3D -1 =3D=3D>
X =3D -1

Assim, de qualquer jeito teremos X =3D -1.

Um abraco,
Claudio.

--MS_Mac_OE_3132683899_833010_MIME_Part-- ========================================================================= Instruções para entrar na lista, sair da lista e usar a lista em http://www.mat.puc-rio.br/~nicolau/olimp/obm-l.html O administrador desta lista é ========================================================================= From owner-obm-l@sucuri.mat.puc-rio.br Tue Apr 8 22:28:47 2003 Return-Path: Received: (from majordom@localhost) by sucuri.mat.puc-rio.br (8.9.3/8.9.3) id WAA31191 for obm-l-MTTP; Tue, 8 Apr 2003 22:27:30 -0300 Received: from itaqui.terra.com.br (itaqui.terra.com.br [200.176.3.19]) by sucuri.mat.puc-rio.br (8.9.3/8.9.3) with ESMTP id WAA31187 for ; Tue, 8 Apr 2003 22:27:26 -0300 Received: from gunga.terra.com.br (gunga.terra.com.br [200.176.3.45]) by itaqui.terra.com.br (Postfix) with ESMTP id 8E6493BC495 for ; Tue, 8 Apr 2003 22:26:56 -0300 (BRT) Received: from [200.177.182.221] (dl-nas6-sao-C8B1B6DD.p001.terra.com.br [200.177.182.221]) by gunga.terra.com.br (Postfix) with ESMTP id 4D3AF12808B for ; Tue, 8 Apr 2003 22:26:55 -0300 (BRT) User-Agent: Microsoft-Outlook-Express-Macintosh-Edition/5.02.2022 Date: Tue, 08 Apr 2003 22:25:04 -0300 Subject: Re: [obm-l] ajuda2 From: Claudio Buffara To: Message-ID: In-Reply-To: <013a01c2fe2c$9d7fcdd0$019da8c0@henrique> Mime-version: 1.0 Content-type: text/plain; charset="ISO-8859-1" Content-Transfer-Encoding: 8bit X-MIME-Autoconverted: from quoted-printable to 8bit by sucuri.mat.puc-rio.br id WAA31188 Sender: owner-obm-l@sucuri.mat.puc-rio.br Precedence: bulk Reply-To: obm-l@mat.puc-rio.br Oi, Henrique: Estou me intrometendo mas acho que posso ajudar. Ha algum tempo, o Marcio Cohen deu a seguinte dica aqui na lista: Em exames, quando aparece algum polinomio de 4o. grau com coeficientes inteiros, ha uma boa chance de que ele seja fatoravel em polinomios lineares ou pelo menos de 2o. grau tambem com coeficientes inteiros. Quando o polinomio eh monico (isto eh, o coeficiente do termo de maior grau eh 1), a coisa fica ainda mais facil. Assim, se p(x) = x^4 + 2x^3 - x^2 - 2x + 1, podemos procurar uma fatoracao da forma: p(x) = (x^2 + ax + b)(x^2 + cx + d) Multiplicando:, vem: p(x) = x^4 + (a + c)x^3 + (ac + b + d)x^2 + (ad + bc)x + bd Igualando os coeficientes, teremos: a + c = 2 ac + b + d = -1 ad + bc = -2 bd = 1 A ultima equacao implica que: b = d = 1 ou b = d = -1. Caso 1: b = d = 1 ==> Usando as duas primeiras equacoes, teremos: a + c = 2 ac = -3 ==> a e c sao raizes de x^2 - 2x - 3 = 0 ==> a = 3 e c = -1. Testando na 3a. equacao: ad + bc = 3 - 1 = 2 <> - 2 ==> b = d = 1 nao eh solucao Caso 2: b = d = -1 ==> a + c = 2 ac = 1 a e c sao raizes de x^2 - 2x + 1 = 0 ==> a = c = 1 Testando: ad + bc = -1 + -1 = -2 ==> OK. Logo, temos que: a = c = 1, b = d = -1 ==> p(x) = (x^2 + x - 1)^2. ********* Repare que a algebra acima, apesar de um pouco trabalhosa, eh bastante elementar. Naturalmente, para este problema especifico (provar que 1 + o produto de quatro inteiros consecutivos eh sempre um quadrado perfeito) existe uma forma melhor: Sejam os 4 inteiros consectivos: u - 3/2, u - 1/2, u + 1/2, u + 3/2 (quem disse que u eh inteiro??? De fato, u = m+1/2, para algum inteiro m) Assim, o produto eh (rearranjando os termos): (u - 3/2)(u + 3/2)(u - 1/2)(u + 1/2) = (u^2 - 9/4)(u^2 - 1/4) = u^4 - (5/2)u^2 + 9/16. Logo, 1 + produto = u^4 - (5/2)u^2 + 25/16 = = u^4 - 2*(5/4)*u^2 + (5/4)^2 = = (u^2 - 5/4)^2 Mas lembre-se de que u = m+1/2 ==> u^2 = m^2 + m + 1/4 ==> (u^2 - 5/4)^2 = (m^2 + m - 1)^2 e acabou... Aqui vai mais uma dica: antes de sair multiplicando e fatorando feito um maluco, pense um pouco e veja se tem alguma mudanca de variaveis que simplifique o problema. Um abraco, Claudio. on 08.04.03 21:11, Henrique Patrício Sant'Anna Branco at hpsbranco@superig.com.br wrote: > Carlos, > > (...) >> Note que (u+2)(u+1)u(u-1)+1=(u^2+u-1)^2, e faca u=93. > (...) > > Pergunta de quem não entende pouco de fatoração: como você fatorou isso? > Sei que (u+2)(u+1)u(u-1)+1 = u^4+2u^3-u^2-2u+1, mas a partir daí, não sei > proceder. > > Abraço, > Henrique. > > ========================================================================= > Instruções para entrar na lista, sair da lista e usar a lista em > http://www.mat.puc-rio.br/~nicolau/olimp/obm-l.html > O administrador desta lista é > ========================================================================= > ========================================================================= Instruções para entrar na lista, sair da lista e usar a lista em http://www.mat.puc-rio.br/~nicolau/olimp/obm-l.html O administrador desta lista é ========================================================================= From owner-obm-l@sucuri.mat.puc-rio.br Tue Apr 8 22:41:46 2003 Return-Path: Received: (from majordom@localhost) by sucuri.mat.puc-rio.br (8.9.3/8.9.3) id WAA31699 for obm-l-MTTP; Tue, 8 Apr 2003 22:40:14 -0300 Received: from salem.bol.com.br (salem.bol.com.br [200.221.24.25]) by sucuri.mat.puc-rio.br (8.9.3/8.9.3) with ESMTP id WAA31694; Tue, 8 Apr 2003 22:40:10 -0300 Received: from bol.com.br (200.221.24.135) by salem.bol.com.br (5.1.071) id 3E7668AC004A482E; Tue, 8 Apr 2003 22:39:40 -0300 Date: Tue, 8 Apr 2003 22:39:40 -0300 Message-Id: Subject: [obm-l] Re; ajuda MIME-Version: 1.0 Content-Type: text/plain;charset="iso-8859-1" From: "thor-oliveira" To: obm-l@mat.puc-rio.br Cc: obm-l@mat.puc-rio.br X-XaM3-API-Version: 2.4 R3 ( B4 ) X-SenderIP: 200.151.161.15 Content-Transfer-Encoding: 8bit X-MIME-Autoconverted: from quoted-printable to 8bit by sucuri.mat.puc-rio.br id WAA31695 Sender: owner-obm-l@sucuri.mat.puc-rio.br Precedence: bulk Reply-To: obm-l@mat.puc-rio.br Me ajudem a resolver esses probleminhas: Os juros cobrados numa operação de descontos composto foram iguais a R$ 370,00. Sabendo que o valor do título é igual a R$ 1800,00 e que o faltavam 75 dias para o vencimento deste, a taxa de desconto mensal aplicada é de ? Uma casa está sendo vendida por R$ 240.000,00 à vista, se desejamos financiar pelo seguinte plano: R$ 45.000,00 após 60 dias e mais 36 prestações mensais iguais sendo a primeira após 30 dias, o valor das prestações se a taxa de juros composto é de 3% a.m. é de ? __________________________________________________________________________ E-mail Premium BOL Antivírus, anti-spam e até 100 MB de espaço. Assine já! http://email.bol.com.br/ ========================================================================= Instruções para entrar na lista, sair da lista e usar a lista em http://www.mat.puc-rio.br/~nicolau/olimp/obm-l.html O administrador desta lista é ========================================================================= From owner-obm-l@sucuri.mat.puc-rio.br Tue Apr 8 22:41:46 2003 Return-Path: Received: (from majordom@localhost) by sucuri.mat.puc-rio.br (8.9.3/8.9.3) id WAA31699 for obm-l-MTTP; Tue, 8 Apr 2003 22:40:14 -0300 Received: from salem.bol.com.br (salem.bol.com.br [200.221.24.25]) by sucuri.mat.puc-rio.br (8.9.3/8.9.3) with ESMTP id WAA31694; Tue, 8 Apr 2003 22:40:10 -0300 Received: from bol.com.br (200.221.24.135) by salem.bol.com.br (5.1.071) id 3E7668AC004A482E; Tue, 8 Apr 2003 22:39:40 -0300 Date: Tue, 8 Apr 2003 22:39:40 -0300 Message-Id: Subject: [obm-l] Re; ajuda MIME-Version: 1.0 Content-Type: text/plain;charset="iso-8859-1" From: "thor-oliveira" To: obm-l@mat.puc-rio.br Cc: obm-l@mat.puc-rio.br X-XaM3-API-Version: 2.4 R3 ( B4 ) X-SenderIP: 200.151.161.15 Content-Transfer-Encoding: 8bit X-MIME-Autoconverted: from quoted-printable to 8bit by sucuri.mat.puc-rio.br id WAA31695 Sender: owner-obm-l@sucuri.mat.puc-rio.br Precedence: bulk Reply-To: obm-l@mat.puc-rio.br Me ajudem a resolver esses probleminhas: Os juros cobrados numa operação de descontos composto foram iguais a R$ 370,00. Sabendo que o valor do título é igual a R$ 1800,00 e que o faltavam 75 dias para o vencimento deste, a taxa de desconto mensal aplicada é de ? Uma casa está sendo vendida por R$ 240.000,00 à vista, se desejamos financiar pelo seguinte plano: R$ 45.000,00 após 60 dias e mais 36 prestações mensais iguais sendo a primeira após 30 dias, o valor das prestações se a taxa de juros composto é de 3% a.m. é de ? __________________________________________________________________________ E-mail Premium BOL Antivírus, anti-spam e até 100 MB de espaço. Assine já! http://email.bol.com.br/ ========================================================================= Instruções para entrar na lista, sair da lista e usar a lista em http://www.mat.puc-rio.br/~nicolau/olimp/obm-l.html O administrador desta lista é ========================================================================= From owner-obm-l@sucuri.mat.puc-rio.br Tue Apr 8 22:50:21 2003 Return-Path: Received: (from majordom@localhost) by sucuri.mat.puc-rio.br (8.9.3/8.9.3) id WAA32030 for obm-l-MTTP; Tue, 8 Apr 2003 22:49:03 -0300 Received: from itaqui.terra.com.br (itaqui.terra.com.br [200.176.3.19]) by sucuri.mat.puc-rio.br (8.9.3/8.9.3) with ESMTP id WAA32025 for ; Tue, 8 Apr 2003 22:49:00 -0300 Received: from marova.terra.com.br (marova.terra.com.br [200.176.3.39]) by itaqui.terra.com.br (Postfix) with ESMTP id B37D53BC4BD for ; Tue, 8 Apr 2003 22:48:29 -0300 (BRT) Received: from [200.177.188.197] (dl-nas7-sao-C8B1BCC5.p001.terra.com.br [200.177.188.197]) by marova.terra.com.br (Postfix) with ESMTP id 36BF43DC090 for ; Tue, 8 Apr 2003 22:48:29 -0300 (BRT) User-Agent: Microsoft-Outlook-Express-Macintosh-Edition/5.02.2022 Date: Tue, 08 Apr 2003 22:47:19 -0300 Subject: [obm-l] Bijecao From: Claudio Buffara To: Lista OBM Message-ID: In-Reply-To: Mime-version: 1.0 Content-type: text/plain; charset="US-ASCII" Content-transfer-encoding: 7bit Sender: owner-obm-l@sucuri.mat.puc-rio.br Precedence: bulk Reply-To: obm-l@mat.puc-rio.br Caros colegas da lista: Dado um conjunto infinito A, seja B o conjunto de todas as sequencias finitas cujos termos pertencem a A. Pergunta: existe uma bijecao entre A e B? Eu sei que a resposta eh sim quando A eh enumeravel e suspeito que seja sim em geral, mas nao estou conseguindo amarrar o argumento. Esta duvida apareceu ao tentar resolver o seguinte problema: Seja R um anel tal que cada funcao de R em R pode ser expressa como um polinomio com coeficientes em R. Prove que R eh um corpo finito. Agradeco qualquer ajuda. Um abraco, Claudio. ========================================================================= Instruções para entrar na lista, sair da lista e usar a lista em http://www.mat.puc-rio.br/~nicolau/olimp/obm-l.html O administrador desta lista é ========================================================================= From owner-obm-l@sucuri.mat.puc-rio.br Tue Apr 8 23:34:35 2003 Return-Path: Received: (from majordom@localhost) by sucuri.mat.puc-rio.br (8.9.3/8.9.3) id XAA01013 for obm-l-MTTP; Tue, 8 Apr 2003 23:32:59 -0300 Received: from imo-d05.mx.aol.com (imo-d05.mx.aol.com [205.188.157.37]) by sucuri.mat.puc-rio.br (8.9.3/8.9.3) with ESMTP id XAA01008 for ; Tue, 8 Apr 2003 23:32:56 -0300 From: Lltmdrtm@aol.com Received: from Lltmdrtm@aol.com by imo-d05.mx.aol.com (mail_out_v34.21.) id z.14f.1dba9f39 (4410) for ; Tue, 8 Apr 2003 22:32:14 -0400 (EDT) Message-ID: <14f.1dba9f39.2bc4e02e@aol.com> Date: Tue, 8 Apr 2003 22:32:14 EDT Subject: [obm-l] ajuda-colmeia To: obm-l@mat.puc-rio.br MIME-Version: 1.0 Content-Type: multipart/alternative; boundary="part1_14f.1dba9f39.2bc4e02e_boundary" X-Mailer: 7.0 for Windows sub 537 Sender: owner-obm-l@sucuri.mat.puc-rio.br Precedence: bulk Reply-To: obm-l@mat.puc-rio.br --part1_14f.1dba9f39.2bc4e02e_boundary Content-Type: text/plain; charset="ISO-8859-1" Content-Transfer-Encoding: quoted-printable Uma colmeia nova tem 8000 abelhas. Destas, a cada dia que passa, morrem 200.= =20 Do 21=BA dia em diante, nascem diariamente 2000 abelhas que vivem, em m=E9di= a, 40=20 dias. Ap=F3s um certo tempo, o n=FAmero de abelhas dessa colmeia se estabili= zar=E1=20 em, aproximadamente quanto? --part1_14f.1dba9f39.2bc4e02e_boundary Content-Type: text/html; charset="ISO-8859-1" Content-Transfer-Encoding: quoted-printable Uma colmeia nova tem 8000 abelhas. Destas, a cada dia=20= que passa, morrem 200. Do 21=BA dia em diante, nascem diariamente 2000 abelh= as que vivem, em m=E9dia, 40 dias. Ap=F3s um certo tempo, o n=FAmero de abel= has dessa colmeia se estabilizar=E1 em, aproximadamente quanto? --part1_14f.1dba9f39.2bc4e02e_boundary-- ========================================================================= Instruções para entrar na lista, sair da lista e usar a lista em http://www.mat.puc-rio.br/~nicolau/olimp/obm-l.html O administrador desta lista é ========================================================================= From owner-obm-l@sucuri.mat.puc-rio.br Wed Apr 9 00:21:24 2003 Return-Path: Received: (from majordom@localhost) by sucuri.mat.puc-rio.br (8.9.3/8.9.3) id AAA01949 for obm-l-MTTP; Wed, 9 Apr 2003 00:19:40 -0300 Received: from ivoti.terra.com.br (ivoti.terra.com.br [200.176.3.20]) by sucuri.mat.puc-rio.br (8.9.3/8.9.3) with ESMTP id AAA01945 for ; Wed, 9 Apr 2003 00:19:37 -0300 Received: from itaim.terra.com.br (itaim.terra.com.br [200.176.3.76]) by ivoti.terra.com.br (Postfix) with ESMTP id A949E40852C for ; Wed, 9 Apr 2003 00:19:06 -0300 (BRT) Received: from oem (150172.cps.virtua.com.br [200.174.150.172]) (authenticated user euraul) by itaim.terra.com.br (Postfix) with ESMTP id 5A1E72E006C for ; Wed, 9 Apr 2003 00:19:06 -0300 (BRT) Message-ID: <001a01c2fe45$f664e0c0$ac96aec8@soc.virtua.com.br> From: "Raul" To: References: <14f.1dba9f39.2bc4e02e@aol.com> Subject: [obm-l] =?iso-8859-1?B?T3Bpbmnjbw==?= Date: Wed, 9 Apr 2003 00:13:14 -0300 MIME-Version: 1.0 Content-Type: multipart/alternative; boundary="----=_NextPart_000_0017_01C2FE2C.D055DF60" X-Priority: 3 X-MSMail-Priority: Normal X-Mailer: Microsoft Outlook Express 5.00.2615.200 X-MimeOLE: Produced By Microsoft MimeOLE V5.00.2615.200 Sender: owner-obm-l@sucuri.mat.puc-rio.br Precedence: bulk Reply-To: obm-l@mat.puc-rio.br This is a multi-part message in MIME format. ------=_NextPart_000_0017_01C2FE2C.D055DF60 Content-Type: text/plain; charset="iso-8859-1" Content-Transfer-Encoding: quoted-printable Bom dia a todos. Um aluno me fez uma pergunta que gostaria de saber a opini=E3o da = lista para que eu n=E3o d=EA a ele uma resposta parcial. A pergunta =E9 = : quais s=E3o as melhores faculdades de Matem=E1tica Pura e de = Matem=E1tica Aplicada por aqui (Rio-S=E3o Paulo)? Gostaria de saber o que vcs pensam e j=E1 me desculpo se a pergunta = for fora do prop=F3sito da lista. Obrigado, Raul ------=_NextPart_000_0017_01C2FE2C.D055DF60 Content-Type: text/html; charset="iso-8859-1" Content-Transfer-Encoding: quoted-printable
    Bom dia a = todos.
    Um aluno me fez uma = pergunta que=20 gostaria de saber a opini=E3o da lista para que eu n=E3o d=EA a ele uma = resposta=20 parcial. A pergunta =E9 : quais s=E3o as melhores faculdades de = Matem=E1tica Pura e de=20 Matem=E1tica Aplicada por aqui (Rio-S=E3o Paulo)?
    Gostaria de saber o = que vcs=20 pensam e j=E1 me desculpo se a pergunta for fora do prop=F3sito da=20 lista.
    = Obrigado,
       =20 Raul
------=_NextPart_000_0017_01C2FE2C.D055DF60-- ========================================================================= Instruções para entrar na lista, sair da lista e usar a lista em http://www.mat.puc-rio.br/~nicolau/olimp/obm-l.html O administrador desta lista é ========================================================================= From owner-obm-l@sucuri.mat.puc-rio.br Wed Apr 9 00:29:12 2003 Return-Path: Received: (from majordom@localhost) by sucuri.mat.puc-rio.br (8.9.3/8.9.3) id AAA02047 for obm-l-MTTP; Wed, 9 Apr 2003 00:27:55 -0300 Received: from puma.unisys.com.br (smtp.unisys.com.br [200.220.64.7]) by sucuri.mat.puc-rio.br (8.9.3/8.9.3) with ESMTP id AAA02037 for ; Wed, 9 Apr 2003 00:27:50 -0300 Received: from jf (riohiper01p24.uninet.com.br [200.220.2.24]) by puma.unisys.com.br (8.12.9/8.12.3) with SMTP id h393RBLE001962 for ; Wed, 9 Apr 2003 00:27:19 -0300 (EST) X-Spam-Filter: check_local@puma.unisys.com.br by digitalanswers.org Message-ID: <040101c2fe48$1e0771e0$1802dcc8@jf> From: "Jose Francisco Guimaraes Costa" To: References: <000801c2f7db$f6f2c900$9a6ef9c8@kuki> Subject: [obm-l] =?iso-8859-1?Q?Quest=E3o_interessante_-_n=E3o=2C_quest=E3o_imbecil?= Date: Wed, 9 Apr 2003 00:02:50 -0300 MIME-Version: 1.0 Content-Type: multipart/alternative; boundary="----=_NextPart_000_03B1_01C2FE2B.5C225D40" X-Priority: 3 X-MSMail-Priority: Normal X-Mailer: Microsoft Outlook Express 6.00.2800.1106 X-MIMEOLE: Produced By Microsoft MimeOLE V6.00.2800.1106 Sender: owner-obm-l@sucuri.mat.puc-rio.br Precedence: bulk Reply-To: obm-l@mat.puc-rio.br This is a multi-part message in MIME format. ------=_NextPart_000_03B1_01C2FE2B.5C225D40 Content-Type: text/plain; charset="iso-8859-1" Content-Transfer-Encoding: quoted-printable Estive ausente deste forum por uma semana, e foi com misto de profunda = tristeza e indigna=E7=E3o que li a mensagem abaixo. Dentro de cada tanque existem seres humanos. Que s=E3o filhos e t=EAm = filhos. Espero que quem teve a est=FApida id=E9ia de propor este problema venha = a se envergonhar de seu feito, sem que para isso tenha que sofrer a = perda de um ente querido em uma situa=E7=E3o de viol=EAncia. JF ----- Original Message -----=20 From: Renato Lira=20 To: obm-l@mat.puc-rio.br=20 Sent: Monday, March 31, 2003 8:18 PM Subject: [obm-l] Quest=E3o interessante Voc=EA =E9 um piloto de um helic=F3ptero Apache e avista uma = fileira de tanques inimigos em forma de combate no vale do rio tigre, = logo a frente [...]. [...] Sabendo-se que seu helic=F3ptero pode destruir o numero de tanques = [...] quantos tanques em forma=E7=E3o restar=E3o? ------=_NextPart_000_03B1_01C2FE2B.5C225D40 Content-Type: text/html; charset="iso-8859-1" Content-Transfer-Encoding: quoted-printable
Estive ausente deste forum por uma = semana, e foi=20 com misto de profunda tristeza e indigna=E7=E3o que li a mensagem=20 abaixo.
 
Dentro de cada tanque existem = seres humanos.=20 Que s=E3o filhos e t=EAm filhos.
 
Espero que quem teve a est=FApida = id=E9ia de propor=20 este problema venha a se envergonhar de seu feito, sem que para = isso tenha=20 que sofrer a perda de um ente querido em uma situa=E7=E3o de = viol=EAncia.
 
JF
 
----- Original Message -----
From:=20 Renato Lira
Sent: Monday, March 31, 2003 = 8:18=20 PM
Subject: [obm-l] Quest=E3o=20 interessante

    Voc=EA =E9 um = piloto de um=20 helic=F3ptero Apache e avista uma fileira de tanques inimigos em = forma de=20 combate no vale do rio tigre, logo a frente [...].
[...]
    Sabendo-se que seu = helic=F3ptero=20 pode destruir o numero de tanques [...] quantos tanques em = forma=E7=E3o=20 restar=E3o?
------=_NextPart_000_03B1_01C2FE2B.5C225D40-- ========================================================================= Instruções para entrar na lista, sair da lista e usar a lista em http://www.mat.puc-rio.br/~nicolau/olimp/obm-l.html O administrador desta lista é ========================================================================= From owner-obm-l@sucuri.mat.puc-rio.br Wed Apr 9 00:29:14 2003 Return-Path: Received: (from majordom@localhost) by sucuri.mat.puc-rio.br (8.9.3/8.9.3) id AAA02041 for obm-l-MTTP; Wed, 9 Apr 2003 00:27:53 -0300 Received: from puma.unisys.com.br (smtp.unisys.com.br [200.220.64.7]) by sucuri.mat.puc-rio.br (8.9.3/8.9.3) with ESMTP id AAA02035 for ; Wed, 9 Apr 2003 00:27:48 -0300 Received: from jf (riohiper01p24.uninet.com.br [200.220.2.24]) by puma.unisys.com.br (8.12.9/8.12.3) with SMTP id h393RBLC001962 for ; Wed, 9 Apr 2003 00:27:16 -0300 (EST) X-Spam-Filter: check_local@puma.unisys.com.br by digitalanswers.org Message-ID: <040001c2fe48$1c0fe7a0$1802dcc8@jf> From: "Jose Francisco Guimaraes Costa" To: References: Subject: Re: [obm-l] ALGARISMOS SIGNIFICATIVOS Date: Wed, 9 Apr 2003 00:02:00 -0300 MIME-Version: 1.0 Content-Type: multipart/alternative; boundary="----=_NextPart_000_03A6_01C2FE2B.3E473CA0" X-Priority: 3 X-MSMail-Priority: Normal X-Mailer: Microsoft Outlook Express 6.00.2800.1106 X-MIMEOLE: Produced By Microsoft MimeOLE V6.00.2800.1106 Sender: owner-obm-l@sucuri.mat.puc-rio.br Precedence: bulk Reply-To: obm-l@mat.puc-rio.br This is a multi-part message in MIME format. ------=_NextPart_000_03A6_01C2FE2B.3E473CA0 Content-Type: text/plain; charset="iso-8859-1" Content-Transfer-Encoding: quoted-printable De uma maneira muito geral, a regra que V menciona - o resultado n=E3o = deve possuir mais algarismos significativos do que o fator menos preciso = - vale para qualquer opera=E7=E3o aritm=E9tica, e n=E3o apenas para = multiplica=E7=E3o e divis=E3o.=20 Mas se formos mais rigorosos, veremos que o resultado pode ter menos = algarismos significativos que o fator menos preciso.=20 Some 0,50 com 0,40.=20 0,90 seria o resultado? Pode n=E3o ser. Escreva 0,50 como 0,50X e 0,40 = como 0,40X, com X significando um algarismo que V n=E3o sabe qual =E9. = S=F3 sabe que =E9 menor ou igual a 4. Digamos que seja 4. Logo, 0,504 = mais 0,404 =E9 igual a 0,908, ou seja, 0,91. Isso significa que V s=F3 = pode garantir um algarismo significativo para o resultado! O primeiro problema mencionado por V teria ca=EDdo num vestibular do = ITA.=20 Para V ter uma id=E9ia de como aquela escola d=E1 import=E2ncia =E0 = quest=E3o de algarismos significativos, numa das cadeiras de f=EDsica do = primeiro per=EDodo do primeiro ano passa-se cerca de dois meses = discutindo o assunto. A decepcionante primeira experi=EAncia executada = pelo aluno do ITA no laborat=F3rio de f=EDsica =E9 medir a =E1rea do = tampo da mesa onde ele est=E1, usando para isso uma r=E9gua de madeira = de 30 cm lascada, empenada e dentada. Em geral ele d=E1 o resultado com = seis ou sete algarismos significativos, j=E1 que ele acha que a mesa = mede - digamos - 1,205 por 0,802 metros (afinal, a r=E9gua est=E1 = graduada em mil=EDmetros), e 1,205 x 0,802 =E9 igual a 0,966410. S=E3o = necess=E1rios dois meses para convenc=EA-lo que o resultado tem um = algarismo significativo apenas, e olhe l=E1! Respondendo sua pergunta, numa prova discursiva siga a regra b=E1sica: o = resultado de uma opera=E7=E3o aritm=E9tica tem tantos algarismos = significativos quanto o fator menos preciso. =20 JF ----- Original Message -----=20 From: "renatinha15a" To: "obm" Sent: Thursday, April 03, 2003 10:18 PM Subject: [obm-l] ALGARISMOS SIGNIFICATIVOS > oi pessoal, estou postando uma d=FAvida que li num f=F3rum=20 > de f=EDsica, mas, creio, se refere =E0 conceitos=20 > matem=E1ticos. Fico grata se algu=E9m me ajudar. >=20 > A minha d=FAvida consiste em como definir ou saber o=20 > porqu=EA da varia=E7=E3o de casas decimais que acontecem=20 > no "desenrolar" das equa=E7=F5es, pois vejo em muitas=20 > resolu=E7=F5es de provas e livros, conflitos quanto ao=20 > m=E9todo. Vou exemplificar minha d=FAvida com uma solu=E7=E3o do=20 > manual do professor do livro F=EDsica Cl=E1ssica: > (ITA) De um telhado caem gotas de chuva separadas por=20 > intervalos de tempo iguais entre si. No momento em que a=20 > 5=AA gota se desprende, a primeira toca o solo. Qual a=20 > dist=E2ncia que separa as duas =FAltimas gotas consecutivas=20 > (4=AA e 5=AA), neste instante, se a altura do telhado =E9 de=20 > 20 m: > (Fazer g =3D 10 m/s^2 e n=E3o considerar a resist=EAncia do=20 > ar.) > SOLU=C7=C3O: > C=E1lculo do tempo de queda: > 1) s =3D gt^2/2 =3D=3D> 20 =3D [(10)tq^2]/2 =3D=3D> tq =3D 2,0s > Intervalo de tempo entre 2 gotas consecutivas: > 2) Δt =3D 2,0/4 =3D 0,50s > Espa=E7o entre a 5=AA e a 4=AA gota: > 3) Δs =3D gt^2/2 =3D (10J)(0,50)^2 > Resposta: Δs =3D 1,25 m >=20 > (D=DAVIDA) O enunciado d=E1 dois valores, s =3D 20 m e g =3D 10=20 > m/s^2, ambos com dois algarismos significativos. Em 1)=20 > verifica-se que a resposta (tq =3D 2,0) mant=EAm os dois=20 > alg. significativos. Agora =E9s o problema! Em 2) a=20 > resposta cont=E9m 3 n=FAmeros significativos. Li num livro=20 > (QU=CDMICA GERAL - James E. Brady) que existe duas regras,=20 > uma para multiplica=E7=E3o/divis=E3o e outra para adi=E7=E3o e=20 > subtra=E7=E3o, para expressar o resultado -o resultado da=20 > adi=E7=E3o ou mult.,etc e n=E3o o resultado final-. Na=20 > multi./div. ele diz: "Em geral, para multiplica=E7=E3o e=20 > divis=E3o, o produto ou o quociente n=E3o deve possuir mais=20 > algarismos significatios do que o fator menos preciso=20 > utilizado no c=E1lculo." Concluo, ent=E3o, que o valor menos=20 > preciso, em 2), tem dois alg. sign. e, portanto, a=20 > resposta deveria ser 0,5 e n=E3o 0,50. Com esse fato, 3)=20 > fica comprometido. Finalmente, como eu devo entender=20 > isso? O que posso fazer numa situa=E7=E3o de prova=20 > discursiva? >=20 >=20 > []=B4s > Renatinha >=20 > =20 > = _________________________________________________________________________= _ > E-mail Premium BOL > Antiv=EDrus, anti-spam e at=E9 100 MB de espa=E7o. Assine j=E1! > http://email.bol.com.br/ >=20 >=20 > = =3D=3D=3D=3D=3D=3D=3D=3D=3D=3D=3D=3D=3D=3D=3D=3D=3D=3D=3D=3D=3D=3D=3D=3D=3D= =3D=3D=3D=3D=3D=3D=3D=3D=3D=3D=3D=3D=3D=3D=3D=3D=3D=3D=3D=3D=3D=3D=3D=3D=3D= =3D=3D=3D=3D=3D=3D=3D=3D=3D=3D=3D=3D=3D=3D=3D=3D=3D=3D=3D=3D=3D=3D=3D > Instru=E7=F5es para entrar na lista, sair da lista e usar a lista em > http://www.mat.puc-rio.br/~nicolau/olimp/obm-l.html > O administrador desta lista =E9 > = =3D=3D=3D=3D=3D=3D=3D=3D=3D=3D=3D=3D=3D=3D=3D=3D=3D=3D=3D=3D=3D=3D=3D=3D=3D= =3D=3D=3D=3D=3D=3D=3D=3D=3D=3D=3D=3D=3D=3D=3D=3D=3D=3D=3D=3D=3D=3D=3D=3D=3D= =3D=3D=3D=3D=3D=3D=3D=3D=3D=3D=3D=3D=3D=3D=3D=3D=3D=3D=3D=3D=3D=3D=3D > ------=_NextPart_000_03A6_01C2FE2B.3E473CA0 Content-Type: text/html; charset="iso-8859-1" Content-Transfer-Encoding: quoted-printable
De uma maneira muito geral, a regra que = V menciona=20 - o resultado n=E3o deve possuir mais algarismos significativos do = que o=20 fator menos preciso - vale para qualquer opera=E7=E3o aritm=E9tica, e = n=E3o apenas para=20 multiplica=E7=E3o e divis=E3o.
 
Mas se formos mais rigorosos, veremos = que o=20 resultado pode ter menos algarismos significativos que o fator menos = preciso.=20
 
Some 0,50 com 0,40.
 
0,90 seria o resultado? Pode n=E3o ser. = Escreva 0,50=20 como 0,50X e 0,40 como 0,40X, com X significando um algarismo que V = n=E3o sabe=20 qual =E9. S=F3 sabe que =E9 menor ou igual a 4. Digamos que seja 4. = Logo, 0,504 mais=20 0,404 =E9 igual a 0,908, ou seja, 0,91. Isso significa que V s=F3 pode = garantir um=20 algarismo significativo para o resultado!
 
O primeiro problema mencionado por V = teria ca=EDdo=20 num vestibular do ITA.
 
Para V ter uma id=E9ia de como aquela = escola d=E1=20 import=E2ncia =E0 quest=E3o de algarismos significativos, numa das = cadeiras de f=EDsica=20 do primeiro per=EDodo do primeiro ano passa-se cerca de dois meses = discutindo o=20 assunto. A decepcionante primeira experi=EAncia executada pelo = aluno do ITA=20 no laborat=F3rio de f=EDsica =E9 medir a =E1rea do tampo da mesa onde = ele est=E1, usando=20 para isso uma r=E9gua de madeira de 30 cm lascada, empenada e dentada. = Em geral=20 ele d=E1 o resultado com seis ou sete algarismos significativos, j=E1 = que=20 ele acha que a mesa mede - digamos - 1,205 por 0,802 metros = (afinal, a=20 r=E9gua est=E1 graduada em mil=EDmetros), e 1,205 x 0,802 =E9 igual a = 0,966410. S=E3o=20 necess=E1rios dois meses para convenc=EA-lo que o resultado tem um = algarismo=20 significativo apenas, e olhe l=E1!
 
Respondendo sua pergunta, numa prova = discursiva=20 siga a regra b=E1sica: o resultado de uma opera=E7=E3o aritm=E9tica tem = tantos=20 algarismos significativos quanto o fator menos = preciso.  
 
JF
 
----- Original Message ----- =
From: "renatinha15a" <renatinha15a@bol.com.br>
To: "obm" <obm-l@mat.puc-rio.br>
Sent: Thursday, April 03, 2003 10:18=20 PM
Subject: [obm-l] ALGARISMOS=20 SIGNIFICATIVOS

> oi pessoal, estou postando uma d=FAvida que li num f=F3rum =
> de=20 f=EDsica, mas, creio, se refere =E0 conceitos
> matem=E1ticos. = Fico grata se=20 algu=E9m me ajudar.
>
> A minha d=FAvida consiste em como = definir ou=20 saber o
> porqu=EA da varia=E7=E3o de casas decimais que = acontecem
> no=20 "desenrolar" das equa=E7=F5es, pois vejo em muitas
> = resolu=E7=F5es de provas e=20 livros, conflitos quanto ao
> m=E9todo. Vou exemplificar minha = d=FAvida com=20 uma solu=E7=E3o do
> manual do professor do livro F=EDsica = Cl=E1ssica:
>=20 (ITA) De um telhado caem gotas de chuva separadas por
> = intervalos de=20 tempo iguais entre si. No momento em que a
> 5=AA gota se = desprende, a=20 primeira toca o solo. Qual a
> dist=E2ncia que separa as duas = =FAltimas gotas=20 consecutivas
> (4=AA e 5=AA), neste instante, se a altura do = telhado =E9 de=20
> 20 m:
> (Fazer g =3D 10 m/s^2 e n=E3o considerar a = resist=EAncia do=20
> ar.)
> SOLU=C7=C3O:
> C=E1lculo do tempo de = queda:
> 1) s =3D=20 gt^2/2 =3D=3D> 20 =3D [(10)tq^2]/2 =3D=3D> tq =3D 2,0s
> = Intervalo de tempo=20 entre 2 gotas consecutivas:
> 2) &#916;t =3D 2,0/4 =3D = 0,50s
>=20 Espa=E7o entre a 5=AA e a 4=AA gota:
> 3) &#916;s =3D gt^2/2 = =3D=20 (10J)(0,50)^2
> Resposta: &#916;s =3D 1,25 m
>
> = (D=DAVIDA) O=20 enunciado d=E1 dois valores, s =3D 20 m e g =3D 10
> m/s^2, ambos = com dois=20 algarismos significativos. Em 1)
> verifica-se que a resposta (tq = =3D 2,0)=20 mant=EAm os dois
> alg. significativos. Agora =E9s o problema! Em = 2) a=20
> resposta cont=E9m 3 n=FAmeros significativos. Li num livro =
>=20 (QU=CDMICA GERAL - James E. Brady) que existe duas regras,
> uma = para=20 multiplica=E7=E3o/divis=E3o e outra para adi=E7=E3o e
> = subtra=E7=E3o, para expressar o=20 resultado -o resultado da
> adi=E7=E3o ou mult.,etc e n=E3o o = resultado final-.=20 Na
> multi./div. ele diz: "Em geral, para multiplica=E7=E3o e =
>=20 divis=E3o, o produto ou o quociente n=E3o deve possuir mais
> = algarismos=20 significatios do que o fator menos preciso
> utilizado no = c=E1lculo."=20 Concluo, ent=E3o, que o valor menos
> preciso, em 2), tem dois = alg. sign.=20 e, portanto, a
> resposta deveria ser 0,5 e n=E3o 0,50. Com esse = fato, 3)=20
> fica comprometido. Finalmente, como eu devo entender
> = isso? O=20 que posso fazer numa situa=E7=E3o de prova
> discursiva?
> =
>=20
> []=B4s
>    Renatinha
>
> =  
>=20 _________________________________________________________________________= _
>=20 E-mail Premium BOL
> Antiv=EDrus, anti-spam e at=E9 100 MB de = espa=E7o. Assine=20 j=E1!
>
http://email.bol.com.br/
>=20
>
>=20 =3D=3D=3D=3D=3D=3D=3D=3D=3D=3D=3D=3D=3D=3D=3D=3D=3D=3D=3D=3D=3D=3D=3D=3D=3D= =3D=3D=3D=3D=3D=3D=3D=3D=3D=3D=3D=3D=3D=3D=3D=3D=3D=3D=3D=3D=3D=3D=3D=3D=3D= =3D=3D=3D=3D=3D=3D=3D=3D=3D=3D=3D=3D=3D=3D=3D=3D=3D=3D=3D=3D=3D=3D=3D
= >=20 Instru=E7=F5es para entrar na lista, sair da lista e usar a lista = em
>=20
http://www.mat.puc-rio.br/~nicolau/olimp/obm-l.html
> O administrador desta lista =E9 <
nicolau@mat.puc-rio.br>
>=20 =3D=3D=3D=3D=3D=3D=3D=3D=3D=3D=3D=3D=3D=3D=3D=3D=3D=3D=3D=3D=3D=3D=3D=3D=3D= =3D=3D=3D=3D=3D=3D=3D=3D=3D=3D=3D=3D=3D=3D=3D=3D=3D=3D=3D=3D=3D=3D=3D=3D=3D= =3D=3D=3D=3D=3D=3D=3D=3D=3D=3D=3D=3D=3D=3D=3D=3D=3D=3D=3D=3D=3D=3D=3D
= >=20
------=_NextPart_000_03A6_01C2FE2B.3E473CA0-- ========================================================================= Instruções para entrar na lista, sair da lista e usar a lista em http://www.mat.puc-rio.br/~nicolau/olimp/obm-l.html O administrador desta lista é ========================================================================= From owner-obm-l@sucuri.mat.puc-rio.br Wed Apr 9 01:53:26 2003 Return-Path: Received: (from majordom@localhost) by sucuri.mat.puc-rio.br (8.9.3/8.9.3) id BAA04125 for obm-l-MTTP; Wed, 9 Apr 2003 01:51:54 -0300 Received: from spf1.us.outblaze.com (205-158-62-158.outblaze.com [205.158.62.158]) by sucuri.mat.puc-rio.br (8.9.3/8.9.3) with SMTP id BAA04121 for ; Wed, 9 Apr 2003 01:51:50 -0300 Received: (qmail 30342 invoked from network); 9 Apr 2003 04:50:28 -0000 Received: from unknown (205.158.62.68) by spf1.us.outblaze.com with QMQP; 9 Apr 2003 04:50:28 -0000 Received: (qmail 39528 invoked from network); 9 Apr 2003 04:51:10 -0000 Received: from unknown (HELO ws1-10.us4.outblaze.com) (205.158.62.111) by 205-158-62-153.outblaze.com with SMTP; 9 Apr 2003 04:51:10 -0000 Received: (qmail 4470 invoked by uid 1001); 9 Apr 2003 04:51:10 -0000 Message-ID: <20030409045110.4469.qmail@mail.com> Content-Type: multipart/mixed; boundary="----------=_1049863870-90095-1" Content-Transfer-Encoding: 7bit MIME-Version: 1.0 X-Mailer: MIME-tools 5.41 (Entity 5.404) Received: from [200.153.128.167] by ws1-10.us4.outblaze.com with http for radiohead_fan@popstar.com; Wed, 09 Apr 2003 01:51:10 -0300 From: "Rodrigo I." To: obm-l@mat.puc-rio.br Date: Wed, 09 Apr 2003 01:51:10 -0300 Subject: =?iso-8859-1?Q?Fw: [obm-l] Quest=E3o_interessante_-_n=E3o,_quest= E3o_imbecil?= X-Originating-Ip: 200.153.128.167 X-Originating-Server: ws1-10.us4.outblaze.com Sender: owner-obm-l@sucuri.mat.puc-rio.br Precedence: bulk Reply-To: obm-l@mat.puc-rio.br This is a multi-part message in MIME format... ------------=_1049863870-90095-1 Content-Type: text/plain; charset="iso-8859-1" Content-Disposition: inline Content-Transfer-Encoding: 7bit Realmente... Desculpa se no momento mando uma mensagem sem ser dúvidas, mas de fato algo tem de ser dito... A guerra de verdade não é vídeo-gueime. A guerra de verdade tem seres humanos de carne osso que podem morrer e que depois não tem outra vida pra contar a história... Alguns deixam suas respectivas famílias, seus trabalhos, até seus filhos... para servir à guerra... Filhos estes que se desesperam com a face obscura de uma guerra, com a incerteza constante de um míssel cair sobre suas cabeças em qualquer instante... São pessoas comuns... A minha intenção não é mudar a cabeça de ninguém, só acho que fazer piada sobre esse assunto, é algo muito sério. Rodrigo Note: forwarded message attached. ----- Original Message ----- From: "Jose Francisco Guimaraes Costa" Date: Wed, 9 Apr 2003 00:02:50 -0300 To: Subject: [obm-l] Questão_interessante_-_não,_questão_imbecil -- __________________________________________________________ Sign-up for your own FREE Personalized E-mail at Mail.com http://www.mail.com/?sr=signup ------------=_1049863870-90095-1 Content-Type: message/rfc822 Content-Disposition: inline Content-Transfer-Encoding: 7bit Return-Path: Received: (qmail 29508 invoked from network); 9 Apr 2003 04:08:21 -0000 Received: from unknown (HELO spf2.us4.outblaze.com) (205.158.62.24) by 205-158-62-61.outblaze.com with SMTP; 9 Apr 2003 04:08:21 -0000 Received: from sucuri.mat.puc-rio.br (sucuri.mat.puc-rio.br [139.82.27.7]) by spf2.us4.outblaze.com (Postfix) with ESMTP id 27716230CE for ; Wed, 9 Apr 2003 04:08:18 +0000 (GMT) Received: (from majordom@localhost) by sucuri.mat.puc-rio.br (8.9.3/8.9.3) id AAA02047 for obm-l-MTTP; Wed, 9 Apr 2003 00:27:55 -0300 Received: from puma.unisys.com.br (smtp.unisys.com.br [200.220.64.7]) by sucuri.mat.puc-rio.br (8.9.3/8.9.3) with ESMTP id AAA02037 for ; Wed, 9 Apr 2003 00:27:50 -0300 Received: from jf (riohiper01p24.uninet.com.br [200.220.2.24]) by puma.unisys.com.br (8.12.9/8.12.3) with SMTP id h393RBLE001962 for ; Wed, 9 Apr 2003 00:27:19 -0300 (EST) X-Spam-Filter: check_local@puma.unisys.com.br by digitalanswers.org Message-ID: <040101c2fe48$1e0771e0$1802dcc8@jf> From: "Jose Francisco Guimaraes Costa" To: References: <000801c2f7db$f6f2c900$9a6ef9c8@kuki> Subject: [obm-l] =?iso-8859-1?Q?Quest=E3o_interessante_-_n=E3o=2C_quest=E3o_imbecil?= Date: Wed, 9 Apr 2003 00:02:50 -0300 MIME-Version: 1.0 Content-Type: multipart/alternative; boundary="----=_NextPart_000_03B1_01C2FE2B.5C225D40" X-Priority: 3 X-MSMail-Priority: Normal X-Mailer: Microsoft Outlook Express 6.00.2800.1106 X-MIMEOLE: Produced By Microsoft MimeOLE V6.00.2800.1106 Sender: owner-obm-l@sucuri.mat.puc-rio.br Precedence: bulk Reply-To: obm-l@mat.puc-rio.br This is a multi-part message in MIME format. ------=_NextPart_000_03B1_01C2FE2B.5C225D40 Content-Type: text/plain; charset="iso-8859-1" Content-Transfer-Encoding: quoted-printable Estive ausente deste forum por uma semana, e foi com misto de profunda = tristeza e indigna=E7=E3o que li a mensagem abaixo. Dentro de cada tanque existem seres humanos. Que s=E3o filhos e t=EAm = filhos. Espero que quem teve a est=FApida id=E9ia de propor este problema venha = a se envergonhar de seu feito, sem que para isso tenha que sofrer a = perda de um ente querido em uma situa=E7=E3o de viol=EAncia. JF ----- Original Message -----=20 From: Renato Lira=20 To: obm-l@mat.puc-rio.br=20 Sent: Monday, March 31, 2003 8:18 PM Subject: [obm-l] Quest=E3o interessante Voc=EA =E9 um piloto de um helic=F3ptero Apache e avista uma = fileira de tanques inimigos em forma de combate no vale do rio tigre, = logo a frente [...]. [...] Sabendo-se que seu helic=F3ptero pode destruir o numero de tanques = [...] quantos tanques em forma=E7=E3o restar=E3o? ------=_NextPart_000_03B1_01C2FE2B.5C225D40 Content-Type: text/html; charset="iso-8859-1" Content-Transfer-Encoding: quoted-printable
Estive ausente deste forum por uma = semana, e foi=20 com misto de profunda tristeza e indigna=E7=E3o que li a mensagem=20 abaixo.
 
Dentro de cada tanque existem = seres humanos.=20 Que s=E3o filhos e t=EAm filhos.
 
Espero que quem teve a est=FApida = id=E9ia de propor=20 este problema venha a se envergonhar de seu feito, sem que para = isso tenha=20 que sofrer a perda de um ente querido em uma situa=E7=E3o de = viol=EAncia.
 
JF
 
----- Original Message -----
From:=20 Renato Lira
Sent: Monday, March 31, 2003 = 8:18=20 PM
Subject: [obm-l] Quest=E3o=20 interessante

    Voc=EA =E9 um = piloto de um=20 helic=F3ptero Apache e avista uma fileira de tanques inimigos em = forma de=20 combate no vale do rio tigre, logo a frente [...].
[...]
    Sabendo-se que seu = helic=F3ptero=20 pode destruir o numero de tanques [...] quantos tanques em = forma=E7=E3o=20 restar=E3o?
------=_NextPart_000_03B1_01C2FE2B.5C225D40-- ========================================================================= Instruções para entrar na lista, sair da lista e usar a lista em http://www.mat.puc-rio.br/~nicolau/olimp/obm-l.html O administrador desta lista é ========================================================================= ------------=_1049863870-90095-1-- ========================================================================= Instruções para entrar na lista, sair da lista e usar a lista em http://www.mat.puc-rio.br/~nicolau/olimp/obm-l.html O administrador desta lista é ========================================================================= From owner-obm-l@sucuri.mat.puc-rio.br Wed Apr 9 02:04:07 2003 Return-Path: Received: (from majordom@localhost) by sucuri.mat.puc-rio.br (8.9.3/8.9.3) id CAA04341 for obm-l-MTTP; Wed, 9 Apr 2003 02:02:47 -0300 Received: from mail.fronthost.com (mail.fronthost.com [63.250.6.253]) by sucuri.mat.puc-rio.br (8.9.3/8.9.3) with ESMTP id CAA04337 for ; Wed, 9 Apr 2003 02:02:42 -0300 Received: from [66.44.43.155] by fronthost.com [63.250.6.253] with SmartMax MailMax for at Wed, 09 Apr 2003 01:02:34 -0400 Message-ID: <005301c2fe54$ed1db410$9b2b2c42@TEST4> From: "Alexandre A da Rocha" To: References: Subject: [obm-l] =?iso-8859-1?Q?Re:_=5Bobm-l=5D_+_fatora=E7=E3o?= Date: Wed, 9 Apr 2003 01:00:21 -0400 MIME-Version: 1.0 Content-Type: multipart/alternative; boundary="----=_NextPart_000_0050_01C2FE33.654B8330" X-Priority: 3 X-MSMail-Priority: Normal X-Mailer: Microsoft Outlook Express 5.50.4807.1700 X-MimeOLE: Produced By Microsoft MimeOLE V5.50.4807.1700 Sender: owner-obm-l@sucuri.mat.puc-rio.br Precedence: bulk Reply-To: obm-l@mat.puc-rio.br This is a multi-part message in MIME format. ------=_NextPart_000_0050_01C2FE33.654B8330 Content-Type: text/plain; charset="iso-8859-1" Content-Transfer-Encoding: quoted-printable Re: [obm-l] + fatora=E7=E3oCorro o risco de estar martelando um ponto = morto, mas vamos la.... Apesar de os problemas ja terem sidos solucionados por varios colegas, = gostaria de adcionar que e possivel resolver o primeiro problema com = conhecimentos basicos de fatoracao ( sem que tenhamos que ser capazes de = fatorar polinomios de 4o grau )...=20 basta um pouco de imaginacao para reescrever a=B2+b=B2+c=B2 em funcao de = c, usando apenas o produto notavel (a+b)^2 e a fatoracao equivalente... a^2 + b^2 + c^2, b =3D a+1 =3D=3D> a2 + (a+1)^2 + c^2 =3D=3D> a^2 + a^2 + 2a + 1 + c^2 =3D=3D> 2a^2 + 2a + 1 + c^2 =3D=3D>=20 2a*a + 2a + 1 + c^2, a =3D b-1 =3D=3D> 2a(b-1) + 2a + 1 + c^2 =3D=3D> 2ab -2a + 2a + 1 + c^2 =3D=3D> c^2 +2ab + 1, c =3D ab =3D=3D> c^2 +2c + 1 =3D=3D> (c + 1)^2 =3D=3D> c =3D impar * par =3D par, logo (c+1) =3D impar da muito mais trabalho, mas eu gostei da 'simplicidade' -Auggy ----- Original Message -----=20 From: Claudio Buffara=20 To: obm-l@mat.puc-rio.br=20 Sent: Tuesday, April 08, 2003 8:57 PM Subject: Re: [obm-l] + fatora=E7=E3o =20 [snip] O primeiro sai assim: b =3D a+1 e c =3D ab =3D a(a+1) D =3D a^2 + b^2 + c^2 =3D a^2 + (a+1)^2 + a^2(a+1)^2 =3D=3D> D =3D a^2 + a^2 + 2a + 1 + a^4 + 2a^3 + a^2 =3D=3D> D =3D a^4 + 2a^3 + 3a^2 + 2a + 1 =3D=3D> D =3D (a^2 + a + 1)^2 Mas a^2 + a =3D a(a+1) eh sempre par =3D=3D> a^2 + a + 1 eh sempre impar =3D=3D> sqrt(D) =3D a^2 + a + 1 eh sempre um inteiro impar [snip] ------=_NextPart_000_0050_01C2FE33.654B8330 Content-Type: text/html; charset="iso-8859-1" Content-Transfer-Encoding: quoted-printable Re: [obm-l] + fatora=E7=E3o
Corro o risco de estar martelando um = ponto morto,=20 mas vamos la....
Apesar de os problemas ja terem sidos = solucionados por varios colegas, gostaria de adcionar que e possivel = resolver o=20 primeiro problema com conhecimentos basicos de fatoracao ( sem que = tenhamos que=20 ser capazes de fatorar polinomios de 4o grau )...
basta um pouco de imaginacao para = reescrever=20 a=B2+b=B2+c=B2 em funcao de c, usando = apenas o produto=20 notavel (a+b)^2 e a fatoracao equivalente...
 
a^2 + b^2 + c^2, b =3D a+1 = =3D=3D>
a2 + (a+1)^2 + c^2 = =3D=3D>
a^2 + a^2 + 2a + 1 + c^2 = =3D=3D>
2a^2 + 2a + 1 + c^2 =3D=3D> =
2a*a + 2a + 1 + c^2, a =3D b-1 = =3D=3D>
2a(b-1) + 2a + 1 + c^2 = =3D=3D>
2ab -2a + 2a + 1 + c^2 = =3D=3D>
c^2 +2ab + 1, c =3D ab = =3D=3D>
c^2 +2c + 1 =3D=3D>
(c + 1)^2 =3D=3D> c =3D impar * = par =3D par, logo=20 (c+1) =3D impar
 
da muito mais trabalho, mas eu gostei = da=20 'simplicidade'
 
-Auggy
----- Original Message -----
From:=20 Claudio Buffara =
Sent: Tuesday, April 08, 2003 = 8:57=20 PM
Subject: Re: [obm-l] + = fatora=E7=E3o   
[snip]
 
O primeiro sai assim:

b =3D a+1 =   e   c =3D ab =3D a(a+1)

D =3D a^2 + b^2 + = c^2 =3D a^2 +=20 (a+1)^2 + a^2(a+1)^2  =3D=3D>
D =3D a^2 + a^2 + 2a + 1 + = a^4 + 2a^3 + a^2=20 =3D=3D>
D =3D a^4 + 2a^3 + 3a^2 + 2a + 1 =3D=3D>
D =3D = (a^2 + a +=20 1)^2

Mas a^2 + a =3D a(a+1) eh sempre par =3D=3D>
a^2 + a = + 1 eh sempre=20 impar =3D=3D>
sqrt(D) =3D a^2 + a + 1 eh sempre um inteiro=20 impar
[snip]
------=_NextPart_000_0050_01C2FE33.654B8330-- ========================================================================= Instruções para entrar na lista, sair da lista e usar a lista em http://www.mat.puc-rio.br/~nicolau/olimp/obm-l.html O administrador desta lista é ========================================================================= From owner-obm-l@sucuri.mat.puc-rio.br Wed Apr 9 07:14:58 2003 Return-Path: Received: (from majordom@localhost) by sucuri.mat.puc-rio.br (8.9.3/8.9.3) id HAA09493 for obm-l-MTTP; Wed, 9 Apr 2003 07:12:14 -0300 Received: from saulo.bol.com.br (saulo.bol.com.br [200.221.24.31]) by sucuri.mat.puc-rio.br (8.9.3/8.9.3) with ESMTP id HAA09488 for ; Wed, 9 Apr 2003 07:12:10 -0300 Received: from bol.com.br (200.221.24.118) by saulo.bol.com.br (5.1.071) id 3E7672F20053160F for obm-l@mat.puc-rio.br; Wed, 9 Apr 2003 07:11:40 -0300 Date: Wed, 9 Apr 2003 07:11:40 -0300 Message-Id: Subject: [obm-l] Combinatoria MIME-Version: 1.0 Content-Type: text/plain;charset="iso-8859-1" From: "amurpe" To: obm-l@mat.puc-rio.br X-XaM3-API-Version: 2.4 R3 ( B4 ) X-SenderIP: 200.255.10.147 Content-Transfer-Encoding: 8bit X-MIME-Autoconverted: from quoted-printable to 8bit by sucuri.mat.puc-rio.br id HAA09489 Sender: owner-obm-l@sucuri.mat.puc-rio.br Precedence: bulk Reply-To: obm-l@mat.puc-rio.br Por favor me ajudem nesses problemas. 1)(CCSPUC-1969)- Numa urna há m bolas numeradas de 1 a m.sacam-se, uma a uma , todas as bolas da urna.Pede-se o número de casos em que os p últimos numeros aparecemnas p últimas posições. Resp: ( m-p)!p!. 2)( EPUC-1959)em uma urna há 10 bolas , numeradas de 1 a 10.Sacam-se, uma a uma , toadas as bolas da urna. a) de quantos modos se pode esvaziar a urna ? b)Quantos são os casos em que os 4 últimos numeros aparecem nas 4 ultimas sacadas? c)Quantos são os casos em que as bolas de numero impar aparecem nas sacadas de ordem par ? Res: a)10! , b)6!.4! c)(5!)^2. 3)(IFUFRJ-1969)-dados n pontos de um plano, dos quais p (p0. onde a base do primeiro log é 1/2, o do segundo log é 2 e a do terceiro log é x-1. Resposta: 4< x <10 , x pertence aos reais. Muito obrigado e um abraço. Amurpe __________________________________________________________________________ E-mail Premium BOL Antivírus, anti-spam e até 100 MB de espaço. Assine já! http://email.bol.com.br/ ========================================================================= Instruções para entrar na lista, sair da lista e usar a lista em http://www.mat.puc-rio.br/~nicolau/olimp/obm-l.html O administrador desta lista é ========================================================================= From owner-obm-l@sucuri.mat.puc-rio.br Wed Apr 9 11:46:48 2003 Return-Path: Received: (from majordom@localhost) by sucuri.mat.puc-rio.br (8.9.3/8.9.3) id LAA13702 for obm-l-MTTP; Wed, 9 Apr 2003 11:42:53 -0300 Received: from birosca.ime.usp.br (birosca.ime.usp.br [143.107.45.59]) by sucuri.mat.puc-rio.br (8.9.3/8.9.3) with SMTP id LAA13698 for ; Wed, 9 Apr 2003 11:42:47 -0300 Received: (qmail 27623 invoked from network); 9 Apr 2003 14:42:03 -0000 Received: from mafalda.ime.usp.br (HELO bidu.ime.usp.br) (143.107.45.13) by birosca.ime.usp.br with SMTP; 9 Apr 2003 14:42:03 -0000 Received: (qmail 9742 invoked by uid 1604); 9 Apr 2003 14:39:50 -0000 Date: Wed, 9 Apr 2003 11:39:50 -0300 (EST) From: Salvador Addas Zanata X-Sender: sazanata@mafalda To: obm-l@mat.puc-rio.br Subject: Re: [obm-l] Fw: sqrt(12a^3 - 3) In-Reply-To: <006201c2fe28$410e5340$a2909ec8@u2z7z2> Message-ID: MIME-Version: 1.0 Content-Type: TEXT/PLAIN; charset=ISO-8859-1 Content-Transfer-Encoding: 8bit X-MIME-Autoconverted: from QUOTED-PRINTABLE to 8bit by sucuri.mat.puc-rio.br id LAA13699 Sender: owner-obm-l@sucuri.mat.puc-rio.br Precedence: bulk Reply-To: obm-l@mat.puc-rio.br So um pequeno pitaco: A versao do ultimo teorema de fermat para n=3 nao e muito dificil de provar, se nao me engano foi provada pelo proprio. Acho que saiu na rpm, tem tambem no livro "100 great problems of elementary mathematics", Dorrie. Usa o principio da descida infinita (nao sei se o nome eh exatamente esse). A ideia eh a partir de uma sol., construir outra onde pelo menos um dos numeros eh estritamente menor que o outro. Abraco, Salvador On Tue, 8 Apr 2003, Wagner wrote: > > ----- Original Message ----- > From: Cláudio (Prática) > To: Wagner > Sent: Monday, April 07, 2003 4:02 PM > Subject: Re: sqrt(12a^3 - 3) > > > Oi, André: > > Gostei muito do problema. Realmente, o UTDF nunca chegou a me passar pela cabeça - foi, sem dúvida, uma ótima idéia. > > Acho que você deveria mandar esta solução pra lista. > > Obrigado e um abraço, > Claudio. > ----- Original Message ----- > From: Wagner > To: Cláudio (Prática) > Sent: Friday, April 04, 2003 9:52 PM > Subject: Re: sqrt(12a^3 - 3) > > > Oi Cláudio > > Fui eu que inventei esse problema. > A solução é muito mais difícil do que parece > Foi assim que eu criei esse problema: > Se a,b,c são três números inteiros, tais que: > (a+b)^3 = a^3 + c^3. > Então segundo o último teorema de Fermat > (a+b),a ou c é igual a zero. Pois se eles fossem > todos não nulos, isso seria uma contradição do > teorema no caso n=3. > Se (a+b)^3 = a^3 + c^3 . Então: > ((a+b)/b)^3 = (a/b)^3 + (c/b)^3. Para b diferente de zero. > Logo: ((a/b)+1)^3 = (a/b)^3 + (c/b)^3. > Sejam x e y dois números racionais tais que: > x=a/b e y=c/b. > Então: (x+1)^3 = x^3 + y^3 => > x^3 + 3x^2 + 3x + 1 - x^3 - y^3 = 0 => > 3x^2 + 3x + (1-y^3) = 0. > Vamos calcular x em função de y: > delta = 9 - 12(1 - y^3) = 12y^3 - 3. > x =( -3 + - sqrt(delta))/6. ( i ) > > Agora suponha que a+b=0. > Então a = -b => 0 = -b^3 + c^3 => b=c => y=1 > Se a = 0, b^3 = c^3 => b=c => y=1 > Se c = 0, (a+b)^3 = a^3 => b=0 e então nem x nem y fazem sentido. > Note que sempre que c é diferente de zero, b é diferente de zero. > Se a,b e c forem números inteiros e c for diferente de zero, > então x e y vão ser números racionais. Mas segundo o teorema de Fermat > isso implica que y = 1. Logo x é racional se e somente se y = 1. > Mas temos de ( i ) que x é racional se e somente se sqrt(delta) = sqrt(12y^3 - 3) > for racional. Logo sqrt(12y^3 - 3) só é racional se y=1. > > Na verdade a maior dificuldade dessa solução é associar o problema ao teorema > de Fermat (o que é na verdade muito difícil) > > > André T. > > > ----- Original Message ----- > From: Cláudio (Prática) > To: timpa@uol.com.br > Cc: claudio.buffara@terra.com.br > Sent: Friday, April 04, 2003 5:00 PM > Subject: sqrt(12a^3 - 3) > > > Oi, Andre: > > Você já conseguiu provar que se "a" e sqrt(12a^3 - 3) são racionais, então a = 1? > De onde você tirou esse problema? > > Parece que é fácil mas há dias eu tenho tentado sem sucesso. > > Um abraço, > Claudio. > > ========================================================================= Instruções para entrar na lista, sair da lista e usar a lista em http://www.mat.puc-rio.br/~nicolau/olimp/obm-l.html O administrador desta lista é ========================================================================= From owner-obm-l@sucuri.mat.puc-rio.br Wed Apr 9 11:59:53 2003 Return-Path: Received: (from majordom@localhost) by sucuri.mat.puc-rio.br (8.9.3/8.9.3) id LAA14003 for obm-l-MTTP; Wed, 9 Apr 2003 11:58:03 -0300 Received: from smtp-26.ig.com.br (smtp-26.ig.com.br [200.226.132.160]) by sucuri.mat.puc-rio.br (8.9.3/8.9.3) with SMTP id LAA13999 for ; Wed, 9 Apr 2003 11:57:58 -0300 Received: (qmail 10660 invoked from network); 9 Apr 2003 14:57:32 -0000 Received: from unknown (HELO xxxx) (200.165.170.238) by smtp-26.ig.com.br with SMTP; 9 Apr 2003 14:57:32 -0000 Message-ID: <002c01c2fea8$eddade60$eeaaa5c8@epq.ime.eb.br> From: "Marcio" To: References: Subject: [obm-l] Fibonacci Date: Wed, 9 Apr 2003 12:01:41 -0300 MIME-Version: 1.0 Content-Type: text/plain; charset="iso-8859-1" Content-Transfer-Encoding: 8bit X-Priority: 3 X-MSMail-Priority: Normal X-Mailer: Microsoft Outlook Express 5.50.4133.2400 X-MimeOLE: Produced By Microsoft MimeOLE V5.50.4133.2400 Sender: owner-obm-l@sucuri.mat.puc-rio.br Precedence: bulk Reply-To: obm-l@mat.puc-rio.br Obrigado ao pessoal que se manifestou na questao do rearranjo! Segue aqui um outro problema legal, que tambem ja circulou (sem resposta) pela lista. Esse eu consegui fazer (na época eu não tinha conseguido), mas minha solução é meio feia. Fica aqui pra voces tentarem também. Se alguém quiser depois eu mando a solução. Seja F_n o n-esimo nr. de fibonacci. Mostre que a serie 1/(F_n) converge, e determine sua soma. Abracos, Marcio PS: Eu iria mandar pra Eureka como proposto, mas achei universitario demais. ========================================================================= Instruções para entrar na lista, sair da lista e usar a lista em http://www.mat.puc-rio.br/~nicolau/olimp/obm-l.html O administrador desta lista é ========================================================================= From owner-obm-l@sucuri.mat.puc-rio.br Wed Apr 9 12:08:49 2003 Return-Path: Received: (from majordom@localhost) by sucuri.mat.puc-rio.br (8.9.3/8.9.3) id MAA14293 for obm-l-MTTP; Wed, 9 Apr 2003 12:07:08 -0300 Received: from hotmail.com (f23.sea2.hotmail.com [207.68.165.23]) by sucuri.mat.puc-rio.br (8.9.3/8.9.3) with ESMTP id MAA14289 for ; Wed, 9 Apr 2003 12:07:04 -0300 Received: from mail pickup service by hotmail.com with Microsoft SMTPSVC; Wed, 9 Apr 2003 08:06:32 -0700 Received: from 200.142.58.18 by sea2fd.sea2.hotmail.msn.com with HTTP; Wed, 09 Apr 2003 15:06:32 GMT X-Originating-IP: [200.142.58.18] X-Originating-Email: [p_ssr@hotmail.com] From: "Paulo Santa Rita" To: obm-l@mat.puc-rio.br Subject: [obm-l] =?iso-8859-1?B?UmU6IFtvYm0tbF0gUXVlc3TjbyBpbnRlcmVzc2FudGVd?= Date: Wed, 09 Apr 2003 15:06:32 +0000 Mime-Version: 1.0 Content-Type: text/plain; charset=iso-8859-1; format=flowed Message-ID: X-OriginalArrivalTime: 09 Apr 2003 15:06:32.0937 (UTC) FILETIME=[9B5E4590:01C2FEA9] Sender: owner-obm-l@sucuri.mat.puc-rio.br Precedence: bulk Reply-To: obm-l@mat.puc-rio.br Ola Renato e demais colegas desta lista ... OBM-L Pelos meus calculos sobrariam todos os tangues ... Nao tenho nem um pouco de admiracao por pastores ou padres, mas considero certo que todos os homens sao nossos irmaos e que a paz, sem duvida, e a unica forma de nos tornarmos mais humanos. E Considero todo uso da matematica para fins belicos como uma atividade mediocre e repugnante e que sao lixo e escoria da humanidade aqueles que a este mister se dedicam. Por esta razao, penso que uma contextualizacao belica pressupoe e sugere atitudes que nao sao saudaveis, sobretudo quando o publico alvo pode ser jovens com o discernimento ainda em formacao. Um Abraco Paulo Santa Rita 4,1205,090403 >Original Message -------- Subject: [obm-l] Questão interessante >Date: Mon, 31 Mar 2003 20:18:43 -0300 >From: "Renato Lira" >Reply-To: obm-l@mat.puc-rio.br >To: > > > >Você é um piloto de um helicóptero Apache e avista uma fileira de >tanques >inimigos em forma de combate no vale do rio tigre, logo a >frente distante >46km. >Sabe-se que: >a) Você se aproxima obedecendo uma P.A.(Progressão Aritmética) de >números >inteiros. > >b) Você pode atacar os tanques inimigos a partir de 7,5 km de >distancia, o >que ocorre entre o oitavo e o nono termo da P.A. > >c) O número de tanques em formação é o sétimo termo de uma P.G. >(Progressão geométrica) cuja razao é o inverso da razão da P.A. > >d) O oposto do sexto termo da P.G. é o sêxtuplo do inverso do sétimo >termo >de uma P.H.(Progressão Harmônica) e também igual ao inverso do >quarto >termo desta mesma P.H., cujo primeiro termo vale 1/145. > >Pergunta-se: >Sabendo-se que seu helicóptero pode destruir o numero de tanques dado >pelo >sétimo termo da P.A., quantos tanques em formação restarão? _________________________________________________________________ MSN Messenger: converse com os seus amigos online. http://messenger.msn.com.br ========================================================================= Instruções para entrar na lista, sair da lista e usar a lista em http://www.mat.puc-rio.br/~nicolau/olimp/obm-l.html O administrador desta lista é ========================================================================= From owner-obm-l@sucuri.mat.puc-rio.br Wed Apr 9 12:12:00 2003 Return-Path: Received: (from majordom@localhost) by sucuri.mat.puc-rio.br (8.9.3/8.9.3) id MAA14418 for obm-l-MTTP; Wed, 9 Apr 2003 12:10:37 -0300 Received: from cmsrelay05.mx.net (cmsrelay05.mx.net [165.212.11.2]) by sucuri.mat.puc-rio.br (8.9.3/8.9.3) with SMTP id MAA14414 for ; Wed, 9 Apr 2003 12:10:32 -0300 Received: from cmsapps01.cms.usa.net (HELO localhost) (165.212.11.136) by cmsoutbound.mx.net with SMTP; 9 Apr 2003 15:09:59 -0000 Received: from smtp.postoffice.net [165.212.8.7] by cmsapps01.cms.usa.net (ASMTP/) via mtad (C8.MAIN.2.05) with ESMTP id 544HDiPJ50264M36; Wed, 09 Apr 2003 15:09:57 GMT Received: from 200.181.4.100 [200.181.4.100] by uwdvg007.cms.usa.net (USANET web-mailer CM.0402.5.2B); Wed, 09 Apr 2003 15:09:55 -0000 Date: Wed, 09 Apr 2003 12:09:55 -0300 From: Artur Costa Steiner To: , Subject: [obm-l] =?ISO-8859-1?Q?Re=3A=20=5Bobm=2Dl=5D=20fun=E7=E3o=20expo?= =?ISO-8859-1?Q?nencial=20=2D=20uma=20defini=E7=E3o?= X-Mailer: USANET web-mailer (CM.0402.5.2B) Mime-Version: 1.0 Message-ID: <741HDiooH3760S06.1049900995@uwdvg007.cms.usa.net> Content-Type: text/plain; charset=ISO-8859-1 Content-Transfer-Encoding: 8bit X-MIME-Autoconverted: from quoted-printable to 8bit by sucuri.mat.puc-rio.br id MAA14415 Sender: owner-obm-l@sucuri.mat.puc-rio.br Precedence: bulk Reply-To: obm-l@mat.puc-rio.br Artur Costa Steiner wrote: > > > -----Mensagem original----- > De: Artur Costa Steiner [mailto:artur_steiner@usa.net] > Enviada em: quarta-feira, 9 de abril de 2003 08:26 > Para: artur.steiner@mme.gov.br > Assunto: FW: [obm-l] Re: [obm-l] Re: [obm-l] RE: [obm-l] função > exponencial (de novo) > > > > > Artur Costa Steiner > > > Oi, Nicolau: > > Eu gostaria de ver a demonstração do Teorema acima, especialmente a > passagem > dos racionais para os reais na parte da existência. > Com se faz isso sem usar limites? > > Um abraço, > Claudio. Cláudio, sem nos basearmos em séries de potências, eu creio que podemos seguir o seguintes passos. Inicialmente, verificamos que, com base na definicção que o Nicolau deu, podemos sem maiores difuldades, concluir que, para qualquer racional r, f(r) = a^r, sendo a^r definido, para racionais, da forma usual (a^r = raiz índice n de a^m), r=m/n. Aliás, eu há alguns dias sugeri esta demonstração para a lista, é bonita e não é difícil. Concluímos também as clássicas propriedades da função exponencial, como f(0) = 1 e f(-r) = 1/f(r), f estrit. positiva. Para estendermos a f para x real, consideremos por ora o caso de base a>1. Verificamos inicialmente Se r2>r1 são ambos racionais positivos, então existem inteiros m1 e m2, m2>m1, e um mesmo int, n>0 tais que r1 =m1/n e r2 = m2/n. Como a>1, segue-se que f é estritamente crescente em Q+. Considerando-se que a^(-r) = 1/a^r, podemos estender esta propriedade para todo o Q. para todo x real, definamos A(x) = {a^r |r<=x, r racional}. Como existe um racional s>x e f é estrit. crescente em Q, segue-se que A(x) é limitado superioremente por f(s), existindo assim supremo A(x). Definamos agora g(x) = a^x = sup A(x), x real. É imediato que se x for racional então g(x) = f(x). Vamos agora mostrar que g atende ás demais propriedades da função exponencial. 1) g é estrit, crescente em R ---- Observamos que se x1 < x2, existem racionais r1 e r2 tais que x10, e A(x) e A(y) possuem supremo, segue-se que B também possui e que sup B = sup A(x) sup A(y). Logo, sup B = g(x) g(y). Vamos agora mostrar que B = A(x+y). Se z pertence a B, então z = uv para u em A(x), v em A(y). Das definiçoes de A(x) e A(y) temos que existem racionais r1<=x e r2<=y tais que u = a^r1 e v = a^r2. Logo, z = a^(r1+r2) e r1+r2 = r<= x+y, do que deduzimos que z está em A(x+y) pois r é racional. Se, por outro lado, z está em A(x+y), então z = a^r para um racional r<= x+y. Podemos encontrar racionais r1<=x e r2<= y tais que r1+r2=r, de modo que z = a^(r1+r2) = a^r1 a^r2 . temos então que a^r1 está em A(x), a^r2 está em A(y) e, portanto, z está em B. Logo, A(x+y) = B e sup A(x+y) = g(x+y) = sup B = g(x)g(y) = a^x a^y, conforme desejado. A função g assim definida satisfaz a g(0)=1, g(1) =a e g(x+y) = g(x) g(y) para todos x e y em R. Sabemos que função exponencial E definida pela série de potências e por E(x) = a^x = e^(x Lna) satisfaz a estas mesma propriedades, é contínua e diferenciável em R, e para racionais, coincide com a nossa g. Se provarmos agora que a nosssa g é contínua em um único elemento de R, teremos automaticamente provado que g é contínua em todo R. Neste caso, teremos E = g, pois E e g sã ambas cont´nusa em R e idênticas em Q, o qual é um subconjunto denso de R ( o fecho de Q é R). Acho que não deve ser muito complicado provar que g é contínua em, digamos, zero (não provei ainda). O caso 0 ========================================================================= From owner-obm-l@sucuri.mat.puc-rio.br Wed Apr 9 13:23:22 2003 Return-Path: Received: (from majordom@localhost) by sucuri.mat.puc-rio.br (8.9.3/8.9.3) id NAA17147 for obm-l-MTTP; Wed, 9 Apr 2003 13:21:06 -0300 Received: from ns3bind.localdomain ([200.230.34.5]) by sucuri.mat.puc-rio.br (8.9.3/8.9.3) with ESMTP id NAA17143 for ; Wed, 9 Apr 2003 13:21:02 -0300 Received: from servico2 ([200.230.34.227]) by ns3bind.localdomain (8.11.6/X.XX.X) with SMTP id h39GGmg04157 for ; Wed, 9 Apr 2003 13:16:48 -0300 Message-ID: <00b801c2feb4$0b73c4e0$3300c57d@bovespa.com> From: "=?iso-8859-1?Q?Cl=E1udio_\=28Pr=E1tica\=29?=" To: References: Subject: Re: [obm-l] Fw: sqrt(12a^3 - 3) Date: Wed, 9 Apr 2003 13:21:14 -0300 MIME-Version: 1.0 Content-Type: text/plain; charset="iso-8859-1" Content-Transfer-Encoding: 8bit X-Priority: 3 X-MSMail-Priority: Normal X-Mailer: Microsoft Outlook Express 5.50.4920.2300 X-MimeOLE: Produced By Microsoft MimeOLE V5.50.4920.2300 Sender: owner-obm-l@sucuri.mat.puc-rio.br Precedence: bulk Reply-To: obm-l@mat.puc-rio.br Oi, Salvador: Acho que Fermat provou o caso n = 4 (de fato, usando descida infinita). O caso n = 3 foi provado por Euler usando aritmética em Z[raiz(-3)], se não me engano, e a demonstração é mais difícil do que o caso n = 4. Um abraço, Claudio. ----- Original Message ----- From: "Salvador Addas Zanata" To: Sent: Wednesday, April 09, 2003 11:39 AM Subject: Re: [obm-l] Fw: sqrt(12a^3 - 3) > > So um pequeno pitaco: A versao do ultimo teorema de fermat para n=3 nao e > muito dificil de provar, se nao me engano foi provada pelo proprio. Acho > que saiu na rpm, tem tambem no livro "100 great problems of elementary > mathematics", Dorrie. Usa o principio da descida infinita (nao sei se o > nome eh exatamente esse). A ideia eh a partir de uma sol., construir outra > onde pelo menos um dos numeros eh estritamente menor que o outro. > > > Abraco, > > Salvador > > > On Tue, 8 Apr 2003, Wagner wrote: > > > > > ----- Original Message ----- > > From: Cláudio (Prática) > > To: Wagner > > Sent: Monday, April 07, 2003 4:02 PM > > Subject: Re: sqrt(12a^3 - 3) > > > > > > Oi, André: > > > > Gostei muito do problema. Realmente, o UTDF nunca chegou a me passar pela cabeça - foi, sem dúvida, uma ótima idéia. > > > > Acho que você deveria mandar esta solução pra lista. > > > > Obrigado e um abraço, > > Claudio. > > ----- Original Message ----- > > From: Wagner > > To: Cláudio (Prática) > > Sent: Friday, April 04, 2003 9:52 PM > > Subject: Re: sqrt(12a^3 - 3) > > > > > > Oi Cláudio > > > > Fui eu que inventei esse problema. > > A solução é muito mais difícil do que parece > > Foi assim que eu criei esse problema: > > Se a,b,c são três números inteiros, tais que: > > (a+b)^3 = a^3 + c^3. > > Então segundo o último teorema de Fermat > > (a+b),a ou c é igual a zero. Pois se eles fossem > > todos não nulos, isso seria uma contradição do > > teorema no caso n=3. > > Se (a+b)^3 = a^3 + c^3 . Então: > > ((a+b)/b)^3 = (a/b)^3 + (c/b)^3. Para b diferente de zero. > > Logo: ((a/b)+1)^3 = (a/b)^3 + (c/b)^3. > > Sejam x e y dois números racionais tais que: > > x=a/b e y=c/b. > > Então: (x+1)^3 = x^3 + y^3 => > > x^3 + 3x^2 + 3x + 1 - x^3 - y^3 = 0 => > > 3x^2 + 3x + (1-y^3) = 0. > > Vamos calcular x em função de y: > > delta = 9 - 12(1 - y^3) = 12y^3 - 3. > > x =( -3 + - sqrt(delta))/6. ( i ) > > > > Agora suponha que a+b=0. > > Então a = -b => 0 = -b^3 + c^3 => b=c => y=1 > > Se a = 0, b^3 = c^3 => b=c => y=1 > > Se c = 0, (a+b)^3 = a^3 => b=0 e então nem x nem y fazem sentido. > > Note que sempre que c é diferente de zero, b é diferente de zero. > > Se a,b e c forem números inteiros e c for diferente de zero, > > então x e y vão ser números racionais. Mas segundo o teorema de Fermat > > isso implica que y = 1. Logo x é racional se e somente se y = 1. > > Mas temos de ( i ) que x é racional se e somente se sqrt(delta) = sqrt(12y^3 - 3) > > for racional. Logo sqrt(12y^3 - 3) só é racional se y=1. > > > > Na verdade a maior dificuldade dessa solução é associar o problema ao teorema > > de Fermat (o que é na verdade muito difícil) > > > > > > André T. > > > > > > ----- Original Message ----- > > From: Cláudio (Prática) > > To: timpa@uol.com.br > > Cc: claudio.buffara@terra.com.br > > Sent: Friday, April 04, 2003 5:00 PM > > Subject: sqrt(12a^3 - 3) > > > > > > Oi, Andre: > > > > Você já conseguiu provar que se "a" e sqrt(12a^3 - 3) são racionais, então a = 1? > > De onde você tirou esse problema? > > > > Parece que é fácil mas há dias eu tenho tentado sem sucesso. > > > > Um abraço, > > Claudio. > > > > > > ========================================================================= > Instruções para entrar na lista, sair da lista e usar a lista em > http://www.mat.puc-rio.br/~nicolau/olimp/obm-l.html > O administrador desta lista é > ========================================================================= ========================================================================= Instruções para entrar na lista, sair da lista e usar a lista em http://www.mat.puc-rio.br/~nicolau/olimp/obm-l.html O administrador desta lista é ========================================================================= From owner-obm-l@sucuri.mat.puc-rio.br Wed Apr 9 13:28:26 2003 Return-Path: Received: (from majordom@localhost) by sucuri.mat.puc-rio.br (8.9.3/8.9.3) id NAA17278 for obm-l-MTTP; Wed, 9 Apr 2003 13:26:58 -0300 Received: from traven9.uol.com.br (traven9.uol.com.br [200.221.29.35]) by sucuri.mat.puc-rio.br (8.9.3/8.9.3) with ESMTP id NAA17273 for ; Wed, 9 Apr 2003 13:26:42 -0300 Received: from gauss ([200.158.96.217]) by traven9.uol.com.br (8.9.1/8.9.1) with SMTP id NAA25290 for ; Wed, 9 Apr 2003 13:25:54 -0300 (BRT) Message-ID: <007101c2feb5$40dee3c0$2accfea9@gauss> From: "Domingos Jr." To: References: Subject: Re: [obm-l] Bijecao Date: Wed, 9 Apr 2003 13:26:45 -0300 MIME-Version: 1.0 Content-Type: text/plain; charset="iso-8859-1" Content-Transfer-Encoding: 8bit X-Priority: 3 X-MSMail-Priority: Normal X-Mailer: Microsoft Outlook Express 6.00.2800.1106 X-MimeOLE: Produced By Microsoft MimeOLE V6.00.2800.1106 Sender: owner-obm-l@sucuri.mat.puc-rio.br Precedence: bulk Reply-To: obm-l@mat.puc-rio.br > Seja R um anel tal que cada funcao de R em R pode ser expressa como um > polinomio com coeficientes em R. > Prove que R eh um corpo finito. Acho que uma possível idéia para esse problema é: Seja R o anel e suponha R infinito, suponha que toda função de R em R pode ser expressa como um polinômio com coef. em R. Seja f : R -> R, f(x) = {a, se x = 0 e 0 se x != 0}, onde a é um elemento não nulo do anel. Essa função deve ser expressa por um polinômio, mas se p é um polinômio dessa forma, p(x) != 0 pois p(0) = a != 0 e, no entando, existem infinitas raízes para p, um absurdo. [ ]'s PS: Depois eu penso no problema da bijeção. ========================================================================= Instruções para entrar na lista, sair da lista e usar a lista em http://www.mat.puc-rio.br/~nicolau/olimp/obm-l.html O administrador desta lista é ========================================================================= From owner-obm-l@sucuri.mat.puc-rio.br Wed Apr 9 13:33:29 2003 Return-Path: Received: (from majordom@localhost) by sucuri.mat.puc-rio.br (8.9.3/8.9.3) id NAA17433 for obm-l-MTTP; Wed, 9 Apr 2003 13:31:56 -0300 Received: from ns3bind.localdomain ([200.230.34.5]) by sucuri.mat.puc-rio.br (8.9.3/8.9.3) with ESMTP id NAA17429 for ; Wed, 9 Apr 2003 13:31:52 -0300 Received: from servico2 ([200.230.34.224]) by ns3bind.localdomain (8.11.6/X.XX.X) with SMTP id h39GRfg04874 for ; Wed, 9 Apr 2003 13:27:41 -0300 Message-ID: <00bf01c2feb5$909c6720$3300c57d@bovespa.com> From: "=?iso-8859-1?Q?Cl=E1udio_\=28Pr=E1tica\=29?=" To: References: Subject: [obm-l] =?iso-8859-1?Q?Re:_=5Bobm-l=5D_Re:_=5Bobm-l=5D_Quest=E3o_interessante=5D?= Date: Wed, 9 Apr 2003 13:32:07 -0300 MIME-Version: 1.0 Content-Type: text/plain; charset="iso-8859-1" Content-Transfer-Encoding: 8bit X-Priority: 3 X-MSMail-Priority: Normal X-Mailer: Microsoft Outlook Express 5.50.4920.2300 X-MimeOLE: Produced By Microsoft MimeOLE V5.50.4920.2300 Sender: owner-obm-l@sucuri.mat.puc-rio.br Precedence: bulk Reply-To: obm-l@mat.puc-rio.br Além de todas as críticas de conteúdo moral e filosófico, com as quais estou de acordo, eu gostaria de dizer que o enunciado está ambíguo. Por exemplo, o que quer dizer "se aproxima obedecendo uma PA"? Por favor, clarifique o enunciado e, principalmente, mande os helicópteros de volta às bases e troque os tanques por termos de uma sequencia que, ao invés de destruídos, sejam escolhidos ou retirados, ou algo do gênero, sem que ninguém precise morrer. Um abraço, Claudio. ----- Original Message ----- From: "Paulo Santa Rita" To: Sent: Wednesday, April 09, 2003 12:06 PM Subject: [obm-l] Re: [obm-l] Questão interessante] > Ola Renato e demais > colegas desta lista ... OBM-L > > Pelos meus calculos sobrariam todos os tangues ... > > Nao tenho nem um pouco de admiracao por pastores ou padres, mas considero > certo que todos os homens sao nossos irmaos e que a paz, sem duvida, e a > unica forma de nos tornarmos mais humanos. > > E Considero todo uso da matematica para fins belicos como uma atividade > mediocre e repugnante e que sao lixo e escoria da humanidade aqueles que a > este mister se dedicam. > > Por esta razao, penso que uma contextualizacao belica pressupoe e sugere > atitudes que nao sao saudaveis, sobretudo quando o publico alvo pode ser > jovens com o discernimento ainda em formacao. > > Um Abraco > Paulo Santa Rita > 4,1205,090403 > > > >Original Message -------- Subject: [obm-l] Questão interessante > >Date: Mon, 31 Mar 2003 20:18:43 -0300 > >From: "Renato Lira" > >Reply-To: obm-l@mat.puc-rio.br > >To: > > > > > > > >Você é um piloto de um helicóptero Apache e avista uma fileira de >tanques > >inimigos em forma de combate no vale do rio tigre, logo a >frente distante > >46km. > >Sabe-se que: > >a) Você se aproxima obedecendo uma P.A.(Progressão Aritmética) de >números > >inteiros. > > > >b) Você pode atacar os tanques inimigos a partir de 7,5 km de >distancia, o > >que ocorre entre o oitavo e o nono termo da P.A. > > > >c) O número de tanques em formação é o sétimo termo de uma P.G. > >(Progressão geométrica) cuja razao é o inverso da razão da P.A. > > > >d) O oposto do sexto termo da P.G. é o sêxtuplo do inverso do sétimo >termo > >de uma P.H.(Progressão Harmônica) e também igual ao inverso do >quarto > >termo desta mesma P.H., cujo primeiro termo vale 1/145. > > > >Pergunta-se: > >Sabendo-se que seu helicóptero pode destruir o numero de tanques dado >pelo > >sétimo termo da P.A., quantos tanques em formação restarão? > > > _________________________________________________________________ > MSN Messenger: converse com os seus amigos online. > http://messenger.msn.com.br > > ========================================================================= > Instruções para entrar na lista, sair da lista e usar a lista em > http://www.mat.puc-rio.br/~nicolau/olimp/obm-l.html > O administrador desta lista é > ========================================================================= ========================================================================= Instruções para entrar na lista, sair da lista e usar a lista em http://www.mat.puc-rio.br/~nicolau/olimp/obm-l.html O administrador desta lista é ========================================================================= From owner-obm-l@sucuri.mat.puc-rio.br Wed Apr 9 13:59:10 2003 Return-Path: Received: (from majordom@localhost) by sucuri.mat.puc-rio.br (8.9.3/8.9.3) id NAA18483 for obm-l-MTTP; Wed, 9 Apr 2003 13:55:33 -0300 Received: (from nicolau@localhost) by sucuri.mat.puc-rio.br (8.9.3/8.9.3) id NAA18477 for obm-l@mat.puc-rio.br; Wed, 9 Apr 2003 13:55:32 -0300 Date: Wed, 9 Apr 2003 13:55:32 -0300 From: "Nicolau C. Saldanha" To: obm-l@mat.puc-rio.br Subject: [obm-l] Re: =?iso-8859-1?Q?=5Bobm-l=5D_Opini=E3o?= Message-ID: <20030409135532.A17897@sucuri.mat.puc-rio.br> References: <14f.1dba9f39.2bc4e02e@aol.com> <001a01c2fe45$f664e0c0$ac96aec8@soc.virtua.com.br> Mime-Version: 1.0 Content-Type: text/plain; charset=iso-8859-1 Content-Disposition: inline Content-Transfer-Encoding: 8bit User-Agent: Mutt/1.2.5i In-Reply-To: <001a01c2fe45$f664e0c0$ac96aec8@soc.virtua.com.br>; from euraul@terra.com.br on Wed, Apr 09, 2003 at 12:13:14AM -0300 Sender: owner-obm-l@sucuri.mat.puc-rio.br Precedence: bulk Reply-To: obm-l@mat.puc-rio.br On Wed, Apr 09, 2003 at 12:13:14AM -0300, Raul wrote: > Bom dia a todos. Um aluno me fez uma pergunta que gostaria de saber a > opinião da lista para que eu não dê a ele uma resposta parcial. A > pergunta é : quais são as melhores faculdades de Matemática Pura e de > Matemática Aplicada por aqui (Rio-São Paulo)? Gostaria de saber o que > vcs pensam e já me desculpo se a pergunta for fora do propósito da lista. > Obrigado, Raul Eu sou professor da PUC-Rio, então a resposta que eu vou dar pode ser justa mas não é isenta. Mas sem falsa modéstia acho que a graduação da PUC-Rio é claramente a melhor do Rio e uma das melhores do Brasil. Ficamos várias vezes em primeiro lugar nacional no provão e várias outras avaliações também nos colocam em ótima posição. Vale a pena chamar a atenção para o fato de estarmos falando de turmas pequenas (talvez 5 alunos por ano) o que obviamente significa que cada aluno recebe muito mais atenção dos professores do que em outros lugares onde há turmas maiores. A melhor graduação da cidade de São Paulo é a da USP e a da Unicamp também é muito boa. O Impa também é uma instituição excelente mas não tem graduação. []s, N. ========================================================================= Instruções para entrar na lista, sair da lista e usar a lista em http://www.mat.puc-rio.br/~nicolau/olimp/obm-l.html O administrador desta lista é ========================================================================= From owner-obm-l@sucuri.mat.puc-rio.br Wed Apr 9 14:06:21 2003 Return-Path: Received: (from majordom@localhost) by sucuri.mat.puc-rio.br (8.9.3/8.9.3) id OAA18841 for obm-l-MTTP; Wed, 9 Apr 2003 14:04:50 -0300 Received: from web12908.mail.yahoo.com (web12908.mail.yahoo.com [216.136.174.75]) by sucuri.mat.puc-rio.br (8.9.3/8.9.3) with SMTP id OAA18835 for ; Wed, 9 Apr 2003 14:04:46 -0300 Message-ID: <20030409170413.51670.qmail@web12908.mail.yahoo.com> Received: from [200.206.103.3] by web12908.mail.yahoo.com via HTTP; Wed, 09 Apr 2003 14:04:13 ART Date: Wed, 9 Apr 2003 14:04:13 -0300 (ART) From: =?iso-8859-1?q?Johann=20Peter=20Gustav=20Lejeune=20Dirichlet?= Subject: [obm-l] Um de matrizes e outro To: obm-l@mat.puc-rio.br MIME-Version: 1.0 Content-Type: multipart/alternative; boundary="0-984156818-1049907853=:51045" Content-Transfer-Encoding: 8bit Sender: owner-obm-l@sucuri.mat.puc-rio.br Precedence: bulk Reply-To: obm-l@mat.puc-rio.br --0-984156818-1049907853=:51045 Content-Type: text/plain; charset=iso-8859-1 Content-Transfer-Encoding: 8bit Oi turma,tudo beleza?Tenho duas perguntas: Como se resolve esse treco horrivel? "Temos uma matriz A n*n simetrica cujos elementos sao 0 ou 1.Sabe-se que ela tem as seguintes propriedades: i)Para todos os i, j e k com a_ij=a_jk=1 tem-se a_ik=1 ii)Se a_ij=0 entao existem i' e j' tais que a_ii'=a_ji'=a_ij'=a_jj'=1 Mostre que n-1 deve ser ukm numero triangular(do tipo 1+2+3+...+k para algum k natural)." Faz um certo tempo que peguei esse problema mas nao resolvi porque nao manjo de matrizes. Ah,quem poderia me indicar algo sobre o seguinte problema(discuti isso com o ET mas nao achei o livro): Imagine uma colmeia do tipo hexagonal daquelas de abelhas.Dela destacamos um pedaço de arranjo triangular(como um podio de formula 1).Mostre que e impossivel cobri-lo sem falhas usando um azulejo reto com 3 hexagonos(ele cobre tres casas consecutivas em linha reta,mais ou menos como tres hexagonos,com somente um deles vizinho aos outros dois). Nao sei direito mas acho que o Saldanha falou disso na lista.So nao sei quando... Te mais!!!Ass.:Johann TRANSIRE SVVM PECTVS MVNDOQVE POTIRI CONGREGATI EX TOTO ORBE MATHEMATICI OB SCRIPTA INSIGNIA TRIBVERE Fields Medal(John Charles Fields) --------------------------------- Yahoo! Mail O melhor e-mail gratuito da internet: 6MB de espaço, antivírus, acesso POP3, filtro contra spam. --0-984156818-1049907853=:51045 Content-Type: text/html; charset=iso-8859-1 Content-Transfer-Encoding: 8bit

Oi turma,tudo beleza?Tenho duas perguntas:

Como se resolve esse treco horrivel?

"Temos uma matriz A n*n simetrica cujos elementos sao 0 ou 1.Sabe-se que ela tem as seguintes propriedades:

i)Para todos os  i, j e k com a_ij=a_jk=1 tem-se a_ik=1

ii)Se a_ij=0 entao existem i' e j' tais que a_ii'=a_ji'=a_ij'=a_jj'=1

Mostre que n-1 deve ser ukm numero triangular(do tipo 1+2+3+...+k para algum k natural)."

Faz um certo tempo que peguei esse problema mas nao resolvi porque nao manjo de matrizes.

Ah,quem poderia me indicar algo sobre o seguinte problema(discuti isso com o ET mas nao achei o livro):

Imagine uma colmeia do tipo hexagonal daquelas de abelhas.Dela destacamos um pedaço de arranjo triangular(como um podio de formula 1).Mostre que e impossivel cobri-lo sem falhas usando um azulejo reto com 3 hexagonos(ele cobre tres casas consecutivas em linha reta,mais ou menos como tres hexagonos,com somente um deles vizinho aos outros dois).

Nao sei direito mas acho que o Saldanha falou disso na lista.So nao sei quando...

Te mais!!!Ass.:Johann



TRANSIRE SVVM PECTVS MVNDOQVE POTIRI

CONGREGATI EX TOTO ORBE MATHEMATICI OB SCRIPTA INSIGNIA TRIBVERE

Fields Medal(John Charles Fields)



Yahoo! Mail
O melhor e-mail gratuito da internet: 6MB de espaço, antivírus, acesso POP3, filtro contra spam. --0-984156818-1049907853=:51045-- ========================================================================= Instruções para entrar na lista, sair da lista e usar a lista em http://www.mat.puc-rio.br/~nicolau/olimp/obm-l.html O administrador desta lista é ========================================================================= From owner-obm-l@sucuri.mat.puc-rio.br Wed Apr 9 14:07:12 2003 Return-Path: Received: (from majordom@localhost) by sucuri.mat.puc-rio.br (8.9.3/8.9.3) id OAA18876 for obm-l-MTTP; Wed, 9 Apr 2003 14:05:46 -0300 Received: (from nicolau@localhost) by sucuri.mat.puc-rio.br (8.9.3/8.9.3) id OAA18871 for obm-l@mat.puc-rio.br; Wed, 9 Apr 2003 14:05:45 -0300 Date: Wed, 9 Apr 2003 14:05:45 -0300 From: "Nicolau C. Saldanha" To: obm-l@mat.puc-rio.br Subject: Re: [obm-l] Bijecao Message-ID: <20030409140545.B17897@sucuri.mat.puc-rio.br> References: <007101c2feb5$40dee3c0$2accfea9@gauss> Mime-Version: 1.0 Content-Type: text/plain; charset=iso-8859-1 Content-Disposition: inline Content-Transfer-Encoding: 8bit User-Agent: Mutt/1.2.5i In-Reply-To: <007101c2feb5$40dee3c0$2accfea9@gauss>; from dopikas@uol.com.br on Wed, Apr 09, 2003 at 01:26:45PM -0300 Sender: owner-obm-l@sucuri.mat.puc-rio.br Precedence: bulk Reply-To: obm-l@mat.puc-rio.br On Wed, Apr 09, 2003 at 01:26:45PM -0300, Domingos Jr. wrote: > > Seja R um anel tal que cada funcao de R em R pode ser expressa como um > > polinomio com coeficientes em R. > > Prove que R eh um corpo finito. > > Acho que uma possível idéia para esse problema é: > > Seja R o anel e suponha R infinito, suponha que toda função de R em R pode > ser expressa como um polinômio com coef. em R. > > Seja f : R -> R, f(x) = {a, se x = 0 e 0 se x != 0}, onde a é um elemento > não nulo do anel. > Essa função deve ser expressa por um polinômio, mas se p é um polinômio > dessa forma, p(x) != 0 pois p(0) = a != 0 e, no entando, existem infinitas > raízes para p, um absurdo. A demonstração infelizemnte não está correta. Seja A o anel de todas as funções de N em Z com soma e produto coordenada a coordenada. O polinômio x^2 - x admite infinitas raízes em A: todas as funções x com x(n) = 0 ou 1 para todo n. O teorema que você quer (um polinômio de grau n tem no máximo n raízes) só vale em um domínio, um anel comutativo com unidade onde ab = 0 -> a = 0 ou b = 0 []s, N. ========================================================================= Instruções para entrar na lista, sair da lista e usar a lista em http://www.mat.puc-rio.br/~nicolau/olimp/obm-l.html O administrador desta lista é ========================================================================= From owner-obm-l@sucuri.mat.puc-rio.br Wed Apr 9 14:25:36 2003 Return-Path: Received: (from majordom@localhost) by sucuri.mat.puc-rio.br (8.9.3/8.9.3) id OAA19946 for obm-l-MTTP; Wed, 9 Apr 2003 14:23:15 -0300 Received: (from nicolau@localhost) by sucuri.mat.puc-rio.br (8.9.3/8.9.3) id OAA19941 for obm-l@mat.puc-rio.br; Wed, 9 Apr 2003 14:23:14 -0300 Date: Wed, 9 Apr 2003 14:23:14 -0300 From: "Nicolau C. Saldanha" To: obm-l@mat.puc-rio.br Subject: Re: [obm-l] Bijecao Message-ID: <20030409142314.C17897@sucuri.mat.puc-rio.br> References: Mime-Version: 1.0 Content-Type: text/plain; charset=iso-8859-1 Content-Disposition: inline Content-Transfer-Encoding: 8bit User-Agent: Mutt/1.2.5i In-Reply-To: ; from claudio.buffara@terra.com.br on Tue, Apr 08, 2003 at 10:47:19PM -0300 Sender: owner-obm-l@sucuri.mat.puc-rio.br Precedence: bulk Reply-To: obm-l@mat.puc-rio.br On Tue, Apr 08, 2003 at 10:47:19PM -0300, Claudio Buffara wrote: > Caros colegas da lista: > > Dado um conjunto infinito A, seja B o conjunto de todas as sequencias > finitas cujos termos pertencem a A. > > Pergunta: existe uma bijecao entre A e B? Sim. Mas a demonstração geral não é muito fácil. Tente provar que sempre existe uma bijeção entre AxA e A para A infinito, já não é fácil. Eu tenho quase certeza que você precisa do axioma da escolha. O livro 'Set Theory' de Jech tem tudo o que você quer. > Eu sei que a resposta eh sim quando A eh enumeravel e suspeito que seja sim > em geral, mas nao estou conseguindo amarrar o argumento. > > Esta duvida apareceu ao tentar resolver o seguinte problema: > > Seja R um anel tal que cada funcao de R em R pode ser expressa como um > polinomio com coeficientes em R. > Prove que R eh um corpo finito. > > Agradeco qualquer ajuda. O que você parece querer fazer é provar que se R é infinito então o cardinal do conjunto de todas as funções de R em R é maior que o cardinal do conjunto dos polinômios com coeficientes em R. Isto é verdade e acho que pode ser demonstrado por uma variação do argumento diagonal de Cantor, deve ser mais fácil do que a outra pergunta que você fez. De qualquer forma o problema é bem legal. []s, N. ========================================================================= Instruções para entrar na lista, sair da lista e usar a lista em http://www.mat.puc-rio.br/~nicolau/olimp/obm-l.html O administrador desta lista é ========================================================================= From owner-obm-l@sucuri.mat.puc-rio.br Wed Apr 9 14:28:22 2003 Return-Path: Received: (from majordom@localhost) by sucuri.mat.puc-rio.br (8.9.3/8.9.3) id OAA20153 for obm-l-MTTP; Wed, 9 Apr 2003 14:26:59 -0300 Received: (from nicolau@localhost) by sucuri.mat.puc-rio.br (8.9.3/8.9.3) id OAA20148 for obm-l@mat.puc-rio.br; Wed, 9 Apr 2003 14:26:58 -0300 Date: Wed, 9 Apr 2003 14:26:58 -0300 From: "Nicolau C. Saldanha" To: obm-l@mat.puc-rio.br Subject: Re: [obm-l] Um de matrizes e outro Message-ID: <20030409142658.D17897@sucuri.mat.puc-rio.br> References: <20030409170413.51670.qmail@web12908.mail.yahoo.com> Mime-Version: 1.0 Content-Type: text/plain; charset=iso-8859-1 Content-Disposition: inline Content-Transfer-Encoding: 8bit User-Agent: Mutt/1.2.5i In-Reply-To: <20030409170413.51670.qmail@web12908.mail.yahoo.com>; from peterdirichlet2002@yahoo.com.br on Wed, Apr 09, 2003 at 02:04:13PM -0300 Sender: owner-obm-l@sucuri.mat.puc-rio.br Precedence: bulk Reply-To: obm-l@mat.puc-rio.br On Wed, Apr 09, 2003 at 02:04:13PM -0300, Johann Peter Gustav Lejeune Dirichlet wrote: > > Oi turma,tudo beleza?Tenho duas perguntas: > > Como se resolve esse treco horrivel? > > "Temos uma matriz A n*n simetrica cujos elementos sao 0 ou 1.Sabe-se que ela tem as seguintes propriedades: > > i)Para todos os i, j e k com a_ij=a_jk=1 tem-se a_ik=1 > > ii)Se a_ij=0 entao existem i' e j' tais que a_ii'=a_ji'=a_ij'=a_jj'=1 > > Mostre que n-1 deve ser ukm numero triangular(do tipo 1+2+3+...+k para algum k natural)." Certamente há um erro no enunciado: estes itens implicam que a matriz A tem todas as entradas iguais a 1 e não tem nada a ver com o valor de n. []s, N. ========================================================================= Instruções para entrar na lista, sair da lista e usar a lista em http://www.mat.puc-rio.br/~nicolau/olimp/obm-l.html O administrador desta lista é ========================================================================= From owner-obm-l@sucuri.mat.puc-rio.br Wed Apr 9 14:28:28 2003 Return-Path: Received: (from majordom@localhost) by sucuri.mat.puc-rio.br (8.9.3/8.9.3) id OAA20166 for obm-l-MTTP; Wed, 9 Apr 2003 14:27:05 -0300 Received: from web12906.mail.yahoo.com (web12906.mail.yahoo.com [216.136.174.73]) by sucuri.mat.puc-rio.br (8.9.3/8.9.3) with SMTP id OAA20157 for ; Wed, 9 Apr 2003 14:26:59 -0300 Message-ID: <20030409172627.25600.qmail@web12906.mail.yahoo.com> Received: from [200.206.103.3] by web12906.mail.yahoo.com via HTTP; Wed, 09 Apr 2003 14:26:27 ART Date: Wed, 9 Apr 2003 14:26:27 -0300 (ART) From: =?iso-8859-1?q?Johann=20Peter=20Gustav=20Lejeune=20Dirichlet?= Subject: Re: [obm-l] Fw: sqrt(12a^3 - 3) To: obm-l@mat.puc-rio.br In-Reply-To: <00b801c2feb4$0b73c4e0$3300c57d@bovespa.com> MIME-Version: 1.0 Content-Type: multipart/alternative; boundary="0-967178673-1049909187=:25249" Content-Transfer-Encoding: 8bit Sender: owner-obm-l@sucuri.mat.puc-rio.br Precedence: bulk Reply-To: obm-l@mat.puc-rio.br --0-967178673-1049909187=:25249 Content-Type: text/plain; charset=iso-8859-1 Content-Transfer-Encoding: 8bit Bem,tenho que dizer uma coisa:a demonstração de Euler,e bastante chata pelo que eu bem to sabendo,e o Gauss foi quem tene a ideia de usar outros dominios para resolver Fermat.Alias essa soluçao do Fermat tem na Eureka! sete. Cláudio_(Prática) wrote:Oi, Salvador: Acho que Fermat provou o caso n = 4 (de fato, usando descida infinita). O caso n = 3 foi provado por Euler usando aritmética em Z[raiz(-3)], se não me engano, e a demonstração é mais difícil do que o caso n = 4. Um abraço, Claudio. ----- Original Message ----- From: "Salvador Addas Zanata" To: Sent: Wednesday, April 09, 2003 11:39 AM Subject: Re: [obm-l] Fw: sqrt(12a^3 - 3) > > So um pequeno pitaco: A versao do ultimo teorema de fermat para n=3 nao e > muito dificil de provar, se nao me engano foi provada pelo proprio. Acho > que saiu na rpm, tem tambem no livro "100 great problems of elementary > mathematics", Dorrie. Usa o principio da descida infinita (nao sei se o > nome eh exatamente esse). A ideia eh a partir de uma sol., construir outra > onde pelo menos um dos numeros eh estritamente menor que o outro. > > > Abraco, > > Salvador > > > On Tue, 8 Apr 2003, Wagner wrote: > > > > > ----- Original Message ----- > > From: Cláudio (Prática) > > To: Wagner > > Sent: Monday, April 07, 2003 4:02 PM > > Subject: Re: sqrt(12a^3 - 3) > > > > > > Oi, André: > > > > Gostei muito do problema. Realmente, o UTF nunca chegou a me passar pela cabeça - foi, sem dúvida, uma ótima idéia. > > > > Acho que você deveria mandar esta solução pra lista. > > > > Obrigado e um abraço, > > Claudio. > > ----- Original Message ----- > > From: Wagner > > To: Cláudio (Prática) > > Sent: Friday, April 04, 2003 9:52 PM > > Subject: Re: sqrt(12a^3 - 3) > > > > > > Oi Cláudio > > > > Fui eu que inventei esse problema. > > A solução é muito mais difícil do que parece > > Foi assim que eu criei esse problema: > > Se a,b,c são três números inteiros, tais que: > > (a+b)^3 = a^3 + c^3. > > Então segundo o último teorema de Fermat > > (a+b),a ou c é igual a zero. Pois se eles fossem > > todos não nulos, isso seria uma contradição do > > teorema no caso n=3. > > Se (a+b)^3 = a^3 + c^3 . Então: > > ((a+b)/b)^3 = (a/b)^3 + (c/b)^3. Para b diferente de zero. > > Logo: ((a/b)+1)^3 = (a/b)^3 + (c/b)^3. > > Sejam x e y dois números racionais tais que: > > x=a/b e y=c/b. > > Então: (x+1)^3 = x^3 + y^3 => > > x^3 + 3x^2 + 3x + 1 - x^3 - y^3 = 0 => > > 3x^2 + 3x + (1-y^3) = 0. > > Vamos calcular x em função de y: > > delta = 9 - 12(1 - y^3) = 12y^3 - 3. > > x =( -3 + - sqrt(delta))/6. ( i ) > > > > Agora suponha que a+b=0. > > Então a = -b => 0 = -b^3 + c^3 => b=c => y=1 > > Se a = 0, b^3 = c^3 => b=c => y=1 > > Se c = 0, (a+b)^3 = a^3 => b=0 e então nem x nem y fazem sentido. > > Note que sempre que c é diferente de zero, b é diferente de zero. > > Se a,b e c forem números inteiros e c for diferente de zero, > > então x e y vão ser números racionais. Mas segundo o teorema de Fermat > > isso implica que y = 1. Logo x é racional se e somente se y = 1. > > Mas temos de ( i ) que x é racional se e somente se sqrt(delta) = sqrt(12y^3 - 3) > > for racional. Logo sqrt(12y^3 - 3) só é racional se y=1. > > > > Na verdade a maior dificuldade dessa solução é associar o problema ao teorema > > de Fermat (o que é na verdade muito difícil) > > > > > > André T. > > > > > > ----- Original Message ----- > > From: Cláudio (Prática) > > To: timpa@uol.com.br > > Cc: claudio.buffara@terra.com.br > > Sent: Friday, April 04, 2003 5:00 PM > > Subject: sqrt(12a^3 - 3) > > > > > > Oi, Andre: > > > > Você já conseguiu provar que se "a" e sqrt(12a^3 - 3) são racionais, então a = 1? > > De onde você tirou esse problema? > > > > Parece que é fácil mas há dias eu tenho tentado sem sucesso. > > > > Um abraço, > > Claudio. > > > > > > ========================================================================= > Instruções para entrar na lista, sair da lista e usar a lista em > http://www.mat.puc-rio.br/~nicolau/olimp/obm-l.html > O administrador desta lista é > ========================================================================= ========================================================================= Instruções para entrar na lista, sair da lista e usar a lista em http://www.mat.puc-rio.br/~nicolau/olimp/obm-l.html O administrador desta lista é ========================================================================= --------------------------------- Yahoo! Mail O melhor e-mail gratuito da internet: 6MB de espaço, antivírus, acesso POP3, filtro contra spam. --0-967178673-1049909187=:25249 Content-Type: text/html; charset=iso-8859-1 Content-Transfer-Encoding: 8bit

Bem,tenho que dizer uma coisa:a demonstração de Euler,e bastante chata pelo que eu bem to sabendo,e o Gauss foi quem tene a ideia de usar outros dominios para resolver Fermat.Alias essa soluçao do Fermat tem na Eureka! sete.

 Cláudio_(Prática) <claudio@praticacorretora.com.br> wrote:

Oi, Salvador:

Acho que Fermat provou o caso n = 4 (de fato, usando descida infinita). O
caso n = 3 foi provado por Euler usando aritmética em Z[raiz(-3)], se não me
engano, e a demonstração é mais difícil do que o caso n = 4.

Um abraço,
Claudio.
----- Original Message -----
From: "Salvador Addas Zanata"
To:
Sent: Wednesday, April 09, 2003 11:39 AM
Subject: Re: [obm-l] Fw: sqrt(12a^3 - 3)


>
> So um pequeno pitaco: A versao do ultimo teorema de fermat para n=3 nao e
> muito dificil de provar, se nao me engano foi provada pelo proprio. Acho
> que saiu na rpm, tem tambem no livro "100 great problems of elementary
> mathematics", Dorrie. Usa o principio da descida infinita (nao sei se o
> nome eh exatamente esse). A ideia eh a partir de uma sol., construir outra
> onde pelo menos um dos numeros eh estritamente menor que o outro.
>
>
> Abraco,
>
> Salvador
>
>
> On Tue, 8 Apr 2003, Wagner wrote:
>
> >
> > ----- Original Message -----
> > From: Cláudio (Prática)
> > To: Wagner
> > Sent: Monday, April 07, 2003 4:02 PM
> > Subject: Re: sqrt(12a^3 - 3)
> >
> >
> > Oi, André:
> >
> > Gostei muito do problema. Realmente, o UTF nunca chegou a me passar
pela cabeça - foi, sem dúvida, uma ótima idéia.
> >
> > Acho que você deveria mandar esta solução pra lista.
> >
> > Obrigado e um abraço,
> > Claudio.
> > ----- Original Message -----
> > From: Wagner
> > To: Cláudio (Prática)
> > Sent: Friday, April 04, 2003 9:52 PM
> > Subject: Re: sqrt(12a^3 - 3)
> >
> >
> > Oi Cláudio
> >
> > Fui eu que inventei esse problema.
> > A solução é muito mais difícil do que parece
> > Foi assim que eu criei esse problema:
> > Se a,b,c são três números inteiros, tais que:
> > (a+b)^3 = a^3 + c^3.
> > Então segundo o último teorema de Fermat
> > (a+b),a ou c é igual a zero. Pois se eles fossem
> > todos não nulos, isso seria uma contradição do
> > teorema no caso n=3.
> > Se (a+b)^3 = a^3 + c^3 . Então:
> > ((a+b)/b)^3 = (a/b)^3 + (c/b)^3. Para b diferente de zero.
> > Logo: ((a/b)+1)^3 = (a/b)^3 + (c/b)^3.
> > Sejam x e y dois números racionais tais que:
> > x=a/b e y=c/b.
> > Então: (x+1)^3 = x^3 + y^3 =>
> > x^3 + 3x^2 + 3x + 1 - x^3 - y^3 = 0 =>
> > 3x^2 + 3x + (1-y^3) = 0.
> > Vamos calcular x em função de y:
> > delta = 9 - 12(1 - y^3) = 12y^3 - 3.
> > x =( -3 + - sqrt(delta))/6. ( i )
> >
> > Agora suponha que a+b=0.
> > Então a = -b => 0 = -b^3 + c^3 => b=c => y=1
> > Se a = 0, b^3 = c^3 => b=c => y=1
> > Se c = 0, (a+b)^3 = a^3 => b=0 e então nem x nem y fazem sentido.
> > Note que sempre que c é diferente de zero, b é diferente de zero.
> > Se a,b e c forem números inteiros e c for diferente de zero,
> > então x e y vão ser números racionais. Mas segundo o teorema de Fermat
> > isso implica que y = 1. Logo x é racional se e somente se y = 1.
> > Mas temos de ( i ) que x é racional se e somente se sqrt(delta) =
sqrt(12y^3 - 3)
> > for racional. Logo sqrt(12y^3 - 3) só é racional se y=1.
> >
> > Na verdade a maior dificuldade dessa solução é associar o problema ao
teorema
> > de Fermat (o que é na verdade muito difícil)
> >
> >
> > André T.
> >
> >
> > ----- Original Message -----
> > From: Cláudio (Prática)
> > To: timpa@uol.com.br
> > Cc: claudio.buffara@terra.com.br
> > Sent: Friday, April 04, 2003 5:00 PM
> > Subject: sqrt(12a^3 - 3)
> >
> >
> > Oi, Andre:
> >
> > Você já conseguiu provar que se "a" e sqrt(12a^3 - 3) são racionais,
então a = 1?
> > De onde você tirou esse problema?
> >
> > Parece que é fácil mas há dias eu tenho tentado sem sucesso.
> >
> > Um abraço,
> > Claudio.
> >
> >
>
> =========================================================================
> Instruções para entrar na lista, sair da lista e usar a lista em
> http://www.mat.puc-rio.br/~nicolau/olimp/obm-l.html
> O administrador desta lista é
> =========================================================================

=========================================================================
Instruções para entrar na lista, sair da lista e usar a lista em
http://www.mat.puc-rio.br/~nicolau/olimp/obm-l.html
O administrador desta lista é
=========================================================================



Yahoo! Mail
O melhor e-mail gratuito da internet: 6MB de espaço, antivírus, acesso POP3, filtro contra spam. --0-967178673-1049909187=:25249-- ========================================================================= Instruções para entrar na lista, sair da lista e usar a lista em http://www.mat.puc-rio.br/~nicolau/olimp/obm-l.html O administrador desta lista é ========================================================================= From owner-obm-l@sucuri.mat.puc-rio.br Wed Apr 9 15:14:30 2003 Return-Path: Received: (from majordom@localhost) by sucuri.mat.puc-rio.br (8.9.3/8.9.3) id PAA22594 for obm-l-MTTP; Wed, 9 Apr 2003 15:12:28 -0300 Received: from hotmail.com (f9.sea2.hotmail.com [207.68.165.9]) by sucuri.mat.puc-rio.br (8.9.3/8.9.3) with ESMTP id PAA22589 for ; Wed, 9 Apr 2003 15:12:22 -0300 Received: from mail pickup service by hotmail.com with Microsoft SMTPSVC; Wed, 9 Apr 2003 11:11:48 -0700 Received: from 200.216.62.82 by sea2fd.sea2.hotmail.msn.com with HTTP; Wed, 09 Apr 2003 18:11:48 GMT X-Originating-IP: [200.216.62.82] X-Originating-Email: [p_ssr@hotmail.com] From: "Paulo Santa Rita" To: obm-l@mat.puc-rio.br Subject: [obm-l] Uma Observacao Date: Wed, 09 Apr 2003 18:11:48 +0000 Mime-Version: 1.0 Content-Type: text/plain; charset=iso-8859-1; format=flowed Message-ID: X-OriginalArrivalTime: 09 Apr 2003 18:11:48.0682 (UTC) FILETIME=[7CDE5AA0:01C2FEC3] Sender: owner-obm-l@sucuri.mat.puc-rio.br Precedence: bulk Reply-To: obm-l@mat.puc-rio.br Ola caros colegas desta lista ... OBM-L, Muitos colegas desta lista me escrevem diretamente, por diversas razoes. Ora pedem a solucao que eu daria para algum determinado problema, ora solicitam que eu verifique se uma determinacao solucao esta correta ou mesmo desejam apenas esclarecimentos sobre algum conceito matematico. E com a alegria de saber que estou sendo util que tenho respondido, dentro da brevidade que esta minha vida tao corrida e exigente me permite. Nas ultimas semanas, todavia, ja acumularam-se dezenas de mensagens que nao respondi, por absoluta falta de tempo. Por esta razao estou enviando esta mensagem publica, pensando que assim respondo a todas as mensagens brasileiras, pois em sua imensa maioria sao de membros desta lista de discussao. Caros colegas, nao e indiferenca ou ma vontade. E realmente falta de tempo. Tao logo as coisas se tranquilizem, vou responder a todos e voltar a participar mais desta lista. Um Abraco Paulo Santa Rita 4,1511,090403 _________________________________________________________________ MSN Messenger: converse com os seus amigos online. http://messenger.msn.com.br ========================================================================= Instruções para entrar na lista, sair da lista e usar a lista em http://www.mat.puc-rio.br/~nicolau/olimp/obm-l.html O administrador desta lista é ========================================================================= From owner-obm-l@sucuri.mat.puc-rio.br Wed Apr 9 15:16:49 2003 Return-Path: Received: (from majordom@localhost) by sucuri.mat.puc-rio.br (8.9.3/8.9.3) id PAA22685 for obm-l-MTTP; Wed, 9 Apr 2003 15:15:21 -0300 Received: from itaqui.terra.com.br (itaqui.terra.com.br [200.176.3.19]) by sucuri.mat.puc-rio.br (8.9.3/8.9.3) with ESMTP id PAA22681 for ; Wed, 9 Apr 2003 15:15:09 -0300 Received: from marova.terra.com.br (marova.terra.com.br [200.176.3.39]) by itaqui.terra.com.br (Postfix) with ESMTP id 09AE23BC30C for ; Wed, 9 Apr 2003 15:14:35 -0300 (BRT) Received: from niski.com (unknown [200.148.199.29]) (authenticated user fniski) by marova.terra.com.br (Postfix) with ESMTP id 02D183DC06E for ; Wed, 9 Apr 2003 15:14:34 -0300 (BRT) Message-ID: <3E942AD2.40002@niski.com> Date: Wed, 09 Apr 2003 11:14:42 -0300 From: niski User-Agent: Mozilla/5.0 (X11; U; Linux i686; en-US; rv:1.3) Gecko/20030312 X-Accept-Language: en-us, en MIME-Version: 1.0 To: obm-l@mat.puc-rio.br Subject: [obm-l] pequena duvida sobre matrizez Content-Type: text/plain; charset=ISO-8859-1; format=flowed Content-Transfer-Encoding: 8bit Sender: owner-obm-l@sucuri.mat.puc-rio.br Precedence: bulk Reply-To: obm-l@mat.puc-rio.br Colegas, por favor me ajudem... Eu sei que o produto de qualquer matrizes invertiveis é invertivel. Mas uma matriz invertivel pode surgir do produto de uma invertivel com outra não invertivel? Minha opinião é que não. pois seja A invertivel det(AB) = det(A). det(B) det(AB) = m.n Sabe-se que m é diferente de 0. Mas B por não ser invertivel tem determinante = 0 Logo det(AB) = 0 e entao a matriz AB nao é invertivel. Essa demonstracao esta correta? Há como provar sem o uso de determinantes? Obrigado Fabio Niski www.linux.ime.usp.br/~niski -- [about him:] It is rare to find learned men who are clean, do not stink and have a sense of humour. Gottfried Whilhem Leibniz ========================================================================= Instruções para entrar na lista, sair da lista e usar a lista em http://www.mat.puc-rio.br/~nicolau/olimp/obm-l.html O administrador desta lista é ========================================================================= From owner-obm-l@sucuri.mat.puc-rio.br Wed Apr 9 16:07:20 2003 Return-Path: Received: (from majordom@localhost) by sucuri.mat.puc-rio.br (8.9.3/8.9.3) id QAA24355 for obm-l-MTTP; Wed, 9 Apr 2003 16:04:29 -0300 Received: from cmsrelay02.mx.net (cmsrelay02.mx.net [165.212.11.111]) by sucuri.mat.puc-rio.br (8.9.3/8.9.3) with SMTP id QAA24349 for ; Wed, 9 Apr 2003 16:04:24 -0300 Received: from uadvg128.cms.usa.net (HELO localhost) (165.212.11.128) by cmsoutbound.mx.net with SMTP; 9 Apr 2003 19:03:47 -0000 Received: from smtp.postoffice.net [165.212.8.23] by uadvg128.cms.usa.net (ASMTP/) via mtad (C8.MAIN.2.05) with ESMTP id 194HDiTDt0269M28; Wed, 09 Apr 2003 19:03:45 GMT Received: from 200.181.4.100 [200.181.4.100] by cmsweb06.cms.usa.net (USANET web-mailer CM.0402.5.2B); Wed, 09 Apr 2003 19:03:44 -0000 Date: Wed, 09 Apr 2003 16:03:44 -0300 From: Artur Costa Steiner To: Subject: Re: [[obm-l] pequena duvida sobre matrizez] X-Mailer: USANET web-mailer (CM.0402.5.2B) Mime-Version: 1.0 Message-ID: <676HDiTDS0672S06.1049915024@cmsweb06.cms.usa.net> Content-Type: text/plain; charset=ISO-8859-1 Content-Transfer-Encoding: 8bit X-MIME-Autoconverted: from quoted-printable to 8bit by sucuri.mat.puc-rio.br id QAA24352 Sender: owner-obm-l@sucuri.mat.puc-rio.br Precedence: bulk Reply-To: obm-l@mat.puc-rio.br niski wrote: > Colegas, por favor me ajudem... > Eu sei que o produto de qualquer matrizes invertiveis é invertivel. > Mas uma matriz invertivel pode surgir do produto de uma invertivel com > outra não invertivel? Minha opinião é que não. > pois seja A invertivel > det(AB) = det(A). det(B) > det(AB) = m.n > > Sabe-se que m é diferente de 0. > Mas B por não ser invertivel tem determinante = 0 > Logo det(AB) = 0 e entao a matriz AB nao é invertivel. > > Essa demonstracao esta correta? Há como provar sem o uso de determinantes? > Obrigado Está correto. E é uma demonstração simples, elegante e inquestionável. Uma alternativa equivalente é mostrar que as linhas, ou colunas, de AB são linearmente dependentes....o que vem a ser a mesma coisa. Artur ========================================================================= Instruções para entrar na lista, sair da lista e usar a lista em http://www.mat.puc-rio.br/~nicolau/olimp/obm-l.html O administrador desta lista é ========================================================================= From owner-obm-l@sucuri.mat.puc-rio.br Wed Apr 9 16:45:01 2003 Return-Path: Received: (from majordom@localhost) by sucuri.mat.puc-rio.br (8.9.3/8.9.3) id QAA25662 for obm-l-MTTP; Wed, 9 Apr 2003 16:43:11 -0300 Received: from traven9.uol.com.br (traven9.uol.com.br [200.221.29.35]) by sucuri.mat.puc-rio.br (8.9.3/8.9.3) with ESMTP id QAA25658 for ; Wed, 9 Apr 2003 16:43:06 -0300 Received: from gauss ([200.158.96.217]) by traven9.uol.com.br (8.9.1/8.9.1) with SMTP id QAA10271 for ; Wed, 9 Apr 2003 16:40:42 -0300 (BRT) Message-ID: <002501c2fed0$7ad81770$2accfea9@gauss> From: "Domingos Jr." To: References: <007101c2feb5$40dee3c0$2accfea9@gauss> <20030409140545.B17897@sucuri.mat.puc-rio.br> Subject: Re: [obm-l] Bijecao Date: Wed, 9 Apr 2003 16:44:18 -0300 MIME-Version: 1.0 Content-Type: text/plain; charset="iso-8859-1" Content-Transfer-Encoding: 8bit X-Priority: 3 X-MSMail-Priority: Normal X-Mailer: Microsoft Outlook Express 6.00.2800.1106 X-MimeOLE: Produced By Microsoft MimeOLE V6.00.2800.1106 Sender: owner-obm-l@sucuri.mat.puc-rio.br Precedence: bulk Reply-To: obm-l@mat.puc-rio.br > On Wed, Apr 09, 2003 at 01:26:45PM -0300, Domingos Jr. wrote: > > > Seja R um anel tal que cada funcao de R em R pode ser expressa como um > > > polinomio com coeficientes em R. > > > Prove que R eh um corpo finito. > > > > Acho que uma possível idéia para esse problema é: > > > > Seja R o anel e suponha R infinito, suponha que toda função de R em R pode > > ser expressa como um polinômio com coef. em R. > > > > Seja f : R -> R, f(x) = {a, se x = 0 e 0 se x != 0}, onde a é um elemento > > não nulo do anel. > > Essa função deve ser expressa por um polinômio, mas se p é um polinômio > > dessa forma, p(x) != 0 pois p(0) = a != 0 e, no entando, existem infinitas > > raízes para p, um absurdo. > > A demonstração infelizemnte não está correta. > Seja A o anel de todas as funções de N em Z > com soma e produto coordenada a coordenada. > O polinômio x^2 - x admite infinitas raízes em A: > todas as funções x com x(n) = 0 ou 1 para todo n. > > O teorema que você quer (um polinômio de grau n > tem no máximo n raízes) só vale em um domínio, > um anel comutativo com unidade onde > ab = 0 -> a = 0 ou b = 0 Eu senti que estava fácil demais! Se tivermos um domínio de integridade é só imaginar a fatoração do polinômio no fecho algébrico do corpo de frações desse domínio para determinar que há n raízes nesse fecho algébrico e por tanto no máximo n raízes no domínio, certo? [ ]'s ========================================================================= Instruções para entrar na lista, sair da lista e usar a lista em http://www.mat.puc-rio.br/~nicolau/olimp/obm-l.html O administrador desta lista é ========================================================================= From owner-obm-l@sucuri.mat.puc-rio.br Wed Apr 9 18:11:11 2003 Return-Path: Received: (from majordom@localhost) by sucuri.mat.puc-rio.br (8.9.3/8.9.3) id SAA27889 for obm-l-MTTP; Wed, 9 Apr 2003 18:09:18 -0300 Received: from ivoti.terra.com.br (ivoti.terra.com.br [200.176.3.20]) by sucuri.mat.puc-rio.br (8.9.3/8.9.3) with ESMTP id SAA27884 for ; Wed, 9 Apr 2003 18:09:14 -0300 Received: from itaim.terra.com.br (itaim.terra.com.br [200.176.3.76]) by ivoti.terra.com.br (Postfix) with ESMTP id D65DE408A44 for ; Wed, 9 Apr 2003 18:08:41 -0300 (BRT) Received: from niski.com (unknown [200.148.199.29]) (authenticated user fniski) by itaim.terra.com.br (Postfix) with ESMTP id CB2A02E0084 for ; Wed, 9 Apr 2003 18:08:40 -0300 (BRT) Message-ID: <3E948BE1.20904@niski.com> Date: Wed, 09 Apr 2003 14:08:49 -0700 From: niski User-Agent: Mozilla/5.0 (Windows; U; Windows NT 5.1; en-US; rv:1.0.2) Gecko/20030208 Netscape/7.02 X-Accept-Language: en-us, en MIME-Version: 1.0 To: obm-l@mat.puc-rio.br Subject: Re: [[obm-l] pequena duvida sobre matrizez] References: <676HDiTDS0672S06.1049915024@cmsweb06.cms.usa.net> Content-Type: text/plain; charset=us-ascii; format=flowed Content-Transfer-Encoding: 7bit Sender: owner-obm-l@sucuri.mat.puc-rio.br Precedence: bulk Reply-To: obm-l@mat.puc-rio.br muito obrigado arthur -- [about him:] It is rare to find learned men who are clean, do not stink and have a sense of humour. -Gottfried Whilhem Leibniz ========================================================================= Instruções para entrar na lista, sair da lista e usar a lista em http://www.mat.puc-rio.br/~nicolau/olimp/obm-l.html O administrador desta lista é ========================================================================= From owner-obm-l@sucuri.mat.puc-rio.br Wed Apr 9 18:15:43 2003 Return-Path: Received: (from majordom@localhost) by sucuri.mat.puc-rio.br (8.9.3/8.9.3) id SAA28026 for obm-l-MTTP; Wed, 9 Apr 2003 18:14:11 -0300 Received: from itaqui.terra.com.br (itaqui.terra.com.br [200.176.3.19]) by sucuri.mat.puc-rio.br (8.9.3/8.9.3) with ESMTP id SAA28022 for ; Wed, 9 Apr 2003 18:14:07 -0300 Received: from itaim.terra.com.br (itaim.terra.com.br [200.176.3.76]) by itaqui.terra.com.br (Postfix) with ESMTP id A338E3BC510 for ; Wed, 9 Apr 2003 18:13:35 -0300 (BRT) Received: from niski.com (unknown [200.148.199.29]) (authenticated user fniski) by itaim.terra.com.br (Postfix) with ESMTP id 5931E2E004F for ; Wed, 9 Apr 2003 18:13:34 -0300 (BRT) Message-ID: <3E948D06.2040509@niski.com> Date: Wed, 09 Apr 2003 14:13:42 -0700 From: niski User-Agent: Mozilla/5.0 (Windows; U; Windows NT 5.1; en-US; rv:1.0.2) Gecko/20030208 Netscape/7.02 X-Accept-Language: en-us, en MIME-Version: 1.0 To: obm-l@mat.puc-rio.br Subject: [obm-l] duvida em estatistica Content-Type: text/plain; charset=ISO-8859-1; format=flowed Content-Transfer-Encoding: 8bit Sender: owner-obm-l@sucuri.mat.puc-rio.br Precedence: bulk Reply-To: obm-l@mat.puc-rio.br pessoal...estou estudando variaveis aleatorias discretas, e não consigo deduzir que Var(X) = E(X^2) - [(E(X)]^2 , dadas as definicoes de variancia: Var(X) = Somatorio[i=1 até n] ((x[i] - E(X))^2)p[i] e Esperança Matematica E(X) = Somatorio[i=1 até n] x[i]p[i] Aceito qualquer ajuda. obrigado a todos niski www.linux.ime.usp.br/~niski -- [about him:] It is rare to find learned men who are clean, do not stink and have a sense of humour. -Gottfried Whilhem Leibniz ========================================================================= Instruções para entrar na lista, sair da lista e usar a lista em http://www.mat.puc-rio.br/~nicolau/olimp/obm-l.html O administrador desta lista é ========================================================================= From owner-obm-l@sucuri.mat.puc-rio.br Wed Apr 9 18:22:57 2003 Return-Path: Received: (from majordom@localhost) by sucuri.mat.puc-rio.br (8.9.3/8.9.3) id SAA28285 for obm-l-MTTP; Wed, 9 Apr 2003 18:21:21 -0300 Received: from trex.centroin.com.br (trex.centroin.com.br [200.225.63.134]) by sucuri.mat.puc-rio.br (8.9.3/8.9.3) with ESMTP id SAA28281 for ; Wed, 9 Apr 2003 18:21:18 -0300 Received: from trex.centroin.com.br (localhost [127.0.0.1]) by trex.centroin.com.br (8.12.9/8.12.9) with ESMTP id h39LL9Yr025102 for ; Wed, 9 Apr 2003 18:21:09 -0300 (EST) Received: by trex.centroin.com.br (8.12.9/8.12.5/Submit) id h39LL9r7025096; Wed, 9 Apr 2003 18:21:09 -0300 (EST) Message-Id: <200304092121.h39LL9r7025096@trex.centroin.com.br> Received: from 200.141.119.127 by trex.centroin.com.br (CIPWM versao 1.4C1) with HTTPS for ; Wed, 9 Apr 2003 18:21:09 -0300 (EST) Date: Wed, 9 Apr 2003 18:21:09 -0300 (EST) From: Augusto Cesar de Oliveira Morgado To: obm-l@mat.puc-rio.br Subject: Re: [obm-l] duvida em estatistica MIME-Version: 1.0 X-Mailer: CentroIn Internet Provider WebMail v. 1.4C1 (http://www.centroin.com.br/) Content-Type: text/plain; charset="iso-8859-1" Content-Transfer-Encoding: 8bit X-MIME-Autoconverted: from quoted-printable to 8bit by sucuri.mat.puc-rio.br id SAA28282 Sender: owner-obm-l@sucuri.mat.puc-rio.br Precedence: bulk Reply-To: obm-l@mat.puc-rio.br Veja a prova em Variáveis Aleatórias 1 dentro de www.fgv.br em graduação RJ ciclo básico, disciplina Teoria da Probabilidade, materiais. Em Wed, 09 Apr 2003 14:13:42 -0700, niski disse: > pessoal...estou estudando variaveis aleatorias discretas, e não consigo > deduzir que > Var(X) = E(X^2) - [(E(X)]^2 , dadas as definicoes de variancia: > > Var(X) = Somatorio[i=1 até n] ((x[i] - E(X))^2)p[i] > e > Esperança Matematica > E(X) = Somatorio[i=1 até n] x[i]p[i] > > Aceito qualquer ajuda. > obrigado a todos > > niski > www.linux.ime.usp.br/~niski > -- > [about him:] > It is rare to find learned men who are clean, do not stink and have a > sense of humour. > -Gottfried Whilhem Leibniz > > ========================================================================= > Instruções para entrar na lista, sair da lista e usar a lista em > http://www.mat.puc-rio.br/~nicolau/olimp/obm-l.html > O administrador desta lista é > ========================================================================= > > ========================================================================= Instruções para entrar na lista, sair da lista e usar a lista em http://www.mat.puc-rio.br/~nicolau/olimp/obm-l.html O administrador desta lista é ========================================================================= From owner-obm-l@sucuri.mat.puc-rio.br Wed Apr 9 18:32:28 2003 Return-Path: Received: (from majordom@localhost) by sucuri.mat.puc-rio.br (8.9.3/8.9.3) id SAA28737 for obm-l-MTTP; Wed, 9 Apr 2003 18:31:09 -0300 Received: from cmsrelay03.mx.net (cmsrelay03.mx.net [165.212.11.112]) by sucuri.mat.puc-rio.br (8.9.3/8.9.3) with SMTP id SAA28723 for ; Wed, 9 Apr 2003 18:31:02 -0300 Received: from uadvg129.cms.usa.net (HELO localhost) (165.212.11.129) by cmsoutbound.mx.net with SMTP; 9 Apr 2003 21:30:26 -0000 Received: from smtp.postoffice.net [165.212.8.7] by uadvg129.cms.usa.net (ASMTP/) via mtad (C8.MAIN.2.05) with ESMTP id 497HDiVEb0089M29; Wed, 09 Apr 2003 21:30:27 GMT Received: from 200.181.4.100 [200.181.4.100] by uwdvg007.cms.usa.net (USANET web-mailer CM.0402.5.2B); Wed, 09 Apr 2003 21:30:26 -0000 Date: Wed, 09 Apr 2003 18:30:26 -0300 From: Artur Costa Steiner To: Subject: [obm-l] =?ISO-8859-1?Q?func=E3o=20exponencial=20=2D=20alg?= =?ISO-8859-1?Q?uns=20pontos=20interessantes?= X-Mailer: USANET web-mailer (CM.0402.5.2B) Mime-Version: 1.0 Message-ID: <915HDiVEA3968S07.1049923826@uwdvg007.cms.usa.net> Content-Type: text/plain; charset=ISO-8859-1 Content-Transfer-Encoding: 8bit X-MIME-Autoconverted: from quoted-printable to 8bit by sucuri.mat.puc-rio.br id SAA28724 Sender: owner-obm-l@sucuri.mat.puc-rio.br Precedence: bulk Reply-To: obm-l@mat.puc-rio.br Neste dias a Renatinha motivou discissões sobre a função exponencial. Eu coloquei uma mensagem sobre uma definição alternativa para tal função, tomando por base um exercício que consta no livro do Bartle e que eu procurei resolver. Vou agora citar uns pontos interesante, com base na definição que o Nicolau apresentou. Seja então f uma função não identicamente nula tal que f(x+y) = f(x) f(y) para toda x e y em R. Vamos tirar algumas conclusões: 1) f é estritamente positiva --- inicialamente, mostraremos que f nunca se anula. Se f(w) =0 para algum w, então, para todo x em R, temos que f(x) = f(w + x - w) = f(w) f(x-w) = 0 f(x-w) =0, o que contraria a hipótese de que f não é identicamente nula. Logo, f nunca se anula em R. Para todo real x, temos então que f(x) = f(x/2+x/2) = f(x/2)f(x/2) = [f(x/2)]^2>0, conforme desejado. 2) f(0)= 1. Fixemos um real x. Temos que f(x) = f(x+0) = f(x) f(0). Como f(x) <>0, segue-se que f(0) =1. 3) f(-x) = f(x). Basta ver que f(-x+x) = f(-x) f(x) = f(0) =1. Com f não se anula, a conclusão é imediata 4) Para todo inteiro m, f(m) = a^m, sendo a = f(1). Se m é natural, basta aplicar indução finita observando que f(1) = a e que f(m+1) = (m+1) f(m). Se m=0, f(0) = 1 = a^0 e, utilizando (3), estendemos facilmente a conclusão para todo inteiro m. 5) Para todo real x e todo inteiro m, f(mx) = f(x)^m. Demonstração análoga a (4) 6) Para todo real r, f(r) = a^r. Temos que r =m/n. m e n inteiros. Logo f(r) = f(m . 1/n)= f(1/n)^m. Mas f(n . 1/n) = f(1/n)^n = f(1) = a , do que deduzimos que f(1/n)= a^(1/n). E temos portanto que f(r) = [a^(1/n)] ^m = a^(m/n) = a^r. Disto deduzimos que f é idêntica, em Q, à função exponencial usualmente definida para tal conjunto. A generalização para irracionais já não é asim tão "linear" , há uma quebra no processo. Mas há ainda un spontos interessantes. 7) Se f for contínua em algum real w , f é contínua em todo R. Como w é ponto de acumulação de R, seguese que lim h --> 0 f(w+h) = f(w). Mas f(w+h) = f(w) f(h). Da existência de lim h --> 0 f(w+h), seguese então que f(w) lim h --> 0 f(h) = f(w) e , como f(w)<>). vem lim h --> 0 f(h)= 1 = f(0), acarretando continuidade em zero. Para todo real x, temos agora que f(x+h) = f(x) f(h). Como lim h --> 0 f(h)= 1, seguese imediatamente que lim h-->0 f(x+h) = f(x).1 = f(x), implicando continuiddae em todo o R. 8) Se f for diferenciável em algum real w, então f é dierenciável em todo R. Por definição f'(w) = lim h-->0 {f(+h) - f(w)]/h = lim h -->0 f(w)[f(h) -1]/h. Da existência (hipõtese) de f'(w), temos que a expressão entre [] apresenta limite em zero e que lim h -->0 )[f(h) -1]/h] = f'(0) (pois f(0) =1) = f'(w)/f(w). Logo, f é diferenciável em zero. Para todo x em R, temos agora que [f(x+h) - f(x)]/h, h<>0, = f(x) [f(h) -1]/h]. Da conclusão anterior, segue-se então que lim h-->0 [f(x+h) - f(x)]/h existe e iguala-se a f(x) f'(0). Logo , f é diferenciável em todo o R. Pontos a ponderar: Se assumirmos que existe tal f e f é contínua em um único ponto de R, então f é necessariamente a função exponencial dad pela conhecida série de potências. Um abraço a todos Artur ========================================================================= Instruções para entrar na lista, sair da lista e usar a lista em http://www.mat.puc-rio.br/~nicolau/olimp/obm-l.html O administrador desta lista é ========================================================================= From owner-obm-l@sucuri.mat.puc-rio.br Wed Apr 9 19:25:02 2003 Return-Path: Received: (from majordom@localhost) by sucuri.mat.puc-rio.br (8.9.3/8.9.3) id TAA30693 for obm-l-MTTP; Wed, 9 Apr 2003 19:23:27 -0300 Received: from pretoria.ime.unicamp.br (pretoria.ime.unicamp.br [143.106.22.7]) by sucuri.mat.puc-rio.br (8.9.3/8.9.3) with ESMTP id TAA30689 for ; Wed, 9 Apr 2003 19:23:22 -0300 Received: from lei060 (lei060.lei.ime.unicamp.br [143.106.118.60]) by pretoria.ime.unicamp.br (8.9.0/8.9.0) with ESMTP id TAA10281 for ; Wed, 9 Apr 2003 19:22:58 -0300 (EST) From: "Mario Salvatierra Junior" To: Subject: RES: [obm-l] duvida em estatistica Date: Wed, 9 Apr 2003 19:22:47 -0300 Message-ID: <000701c2fee6$8fdb91e0$3c766a8f@lei.ime.unicamp.br> MIME-Version: 1.0 Content-Type: text/plain; charset="iso-8859-1" X-Priority: 3 (Normal) X-MSMail-Priority: Normal X-Mailer: Microsoft Outlook, Build 10.0.2627 X-MimeOLE: Produced By Microsoft MimeOLE V6.00.2600.0000 In-Reply-To: <3E948D06.2040509@niski.com> Importance: Normal Content-Transfer-Encoding: 8bit X-MIME-Autoconverted: from quoted-printable to 8bit by sucuri.mat.puc-rio.br id TAA30690 Sender: owner-obm-l@sucuri.mat.puc-rio.br Precedence: bulk Reply-To: obm-l@mat.puc-rio.br Por definição, para qualquer variável aleatória X, continua ou discreta, VarX=E[(X-EX)^2]. Então, VarX=E[(X-EX)^2]=E[X^2-2XE(X)+[E(X)]^2]=E(X^2)-2E(X)E(X)+[E(X)]^2= =E(X^2)-[E(X)]^2 A segunda igualdade vem pelo fato da linearidade da esperança................ E lembre que E(constante)=constante, assim E[[E(X)]^2]= [E(X)]^2 -----Mensagem original----- De: owner-obm-l@sucuri.mat.puc-rio.br [mailto:owner-obm-l@sucuri.mat.puc-rio.br] Em nome de niski Enviada em: quarta-feira, 9 de abril de 2003 18:14 Para: obm-l@mat.puc-rio.br Assunto: [obm-l] duvida em estatistica pessoal...estou estudando variaveis aleatorias discretas, e não consigo deduzir que Var(X) = E(X^2) - [(E(X)]^2 , dadas as definicoes de variancia: Var(X) = Somatorio[i=1 até n] ((x[i] - E(X))^2)p[i] e Esperança Matematica E(X) = Somatorio[i=1 até n] x[i]p[i] Aceito qualquer ajuda. obrigado a todos niski www.linux.ime.usp.br/~niski -- [about him:] It is rare to find learned men who are clean, do not stink and have a sense of humour. -Gottfried Whilhem Leibniz ======================================================================== = Instruções para entrar na lista, sair da lista e usar a lista em http://www.mat.puc-rio.br/~nicolau/olimp/obm-l.html O administrador desta lista é ======================================================================== = ========================================================================= Instruções para entrar na lista, sair da lista e usar a lista em http://www.mat.puc-rio.br/~nicolau/olimp/obm-l.html O administrador desta lista é ========================================================================= From owner-obm-l@sucuri.mat.puc-rio.br Wed Apr 9 19:35:19 2003 Return-Path: Received: (from majordom@localhost) by sucuri.mat.puc-rio.br (8.9.3/8.9.3) id TAA31006 for obm-l-MTTP; Wed, 9 Apr 2003 19:34:00 -0300 Received: from pretoria.ime.unicamp.br (pretoria.ime.unicamp.br [143.106.22.7]) by sucuri.mat.puc-rio.br (8.9.3/8.9.3) with ESMTP id TAA31001 for ; Wed, 9 Apr 2003 19:33:56 -0300 Received: from lei060 (lei060.lei.ime.unicamp.br [143.106.118.60]) by pretoria.ime.unicamp.br (8.9.0/8.9.0) with ESMTP id TAA10437 for ; Wed, 9 Apr 2003 19:33:34 -0300 (EST) From: "Mario Salvatierra Junior" To: Subject: RES: [obm-l] pequena duvida sobre matrizez Date: Wed, 9 Apr 2003 19:33:24 -0300 Message-ID: <000801c2fee8$0b61c5e0$3c766a8f@lei.ime.unicamp.br> MIME-Version: 1.0 Content-Type: text/plain; charset="iso-8859-1" X-Priority: 3 (Normal) X-MSMail-Priority: Normal X-Mailer: Microsoft Outlook, Build 10.0.2627 X-MimeOLE: Produced By Microsoft MimeOLE V6.00.2600.0000 In-Reply-To: <3E942AD2.40002@niski.com> Importance: Normal Content-Transfer-Encoding: 8bit X-MIME-Autoconverted: from quoted-printable to 8bit by sucuri.mat.puc-rio.br id TAA31002 Sender: owner-obm-l@sucuri.mat.puc-rio.br Precedence: bulk Reply-To: obm-l@mat.puc-rio.br A sua demonstracao está correta e é a forma mais simples de se provar este fato. Vamos provar o seguinte: Sejam A e B matrizes NxN e P=AB. 1) Suponha que A é invertível e B é não-invertível. Então P é não-invertível. 2) Suponha que B é invertível e A é não-invertível. Então P é não-invertível. Demonstração: 1) Como B é não invertível , existe um vetor x diferente de 0 em R^N, tal que Bx=0. Assim Px=ABx=A0=0. Logo P é não invertivel. 2) Como A é não invertível, existe y diferente de 0 tal que Ay=0. Como B é invertivel existe x tal que Bx=y e alem disso x é diferente de 0. Assim Px=ABx=Ay=0. Logo P é não invertivel. c.q.d Usamos acima o fato de q uma matriz A NxN é não-invertivel se e somente se existe um vetor x não nulo tal que Ax=0. -----Mensagem original----- De: owner-obm-l@sucuri.mat.puc-rio.br [mailto:owner-obm-l@sucuri.mat.puc-rio.br] Em nome de niski Enviada em: quarta-feira, 9 de abril de 2003 11:15 Para: obm-l@mat.puc-rio.br Assunto: [obm-l] pequena duvida sobre matrizez Colegas, por favor me ajudem... Eu sei que o produto de qualquer matrizes invertiveis é invertivel. Mas uma matriz invertivel pode surgir do produto de uma invertivel com outra não invertivel? Minha opinião é que não. pois seja A invertivel det(AB) = det(A). det(B) det(AB) = m.n Sabe-se que m é diferente de 0. Mas B por não ser invertivel tem determinante = 0 Logo det(AB) = 0 e entao a matriz AB nao é invertivel. Essa demonstracao esta correta? Há como provar sem o uso de determinantes? Obrigado Fabio Niski www.linux.ime.usp.br/~niski -- [about him:] It is rare to find learned men who are clean, do not stink and have a sense of humour. Gottfried Whilhem Leibniz ======================================================================== = Instruções para entrar na lista, sair da lista e usar a lista em http://www.mat.puc-rio.br/~nicolau/olimp/obm-l.html O administrador desta lista é ======================================================================== = ========================================================================= Instruções para entrar na lista, sair da lista e usar a lista em http://www.mat.puc-rio.br/~nicolau/olimp/obm-l.html O administrador desta lista é ========================================================================= From owner-obm-l@sucuri.mat.puc-rio.br Wed Apr 9 19:56:11 2003 Return-Path: Received: (from majordom@localhost) by sucuri.mat.puc-rio.br (8.9.3/8.9.3) id TAA31713 for obm-l-MTTP; Wed, 9 Apr 2003 19:54:47 -0300 Received: from paiol.terra.com.br (paiol.terra.com.br [200.176.3.18]) by sucuri.mat.puc-rio.br (8.9.3/8.9.3) with ESMTP id TAA31709 for ; Wed, 9 Apr 2003 19:54:43 -0300 Received: from gunga.terra.com.br (gunga.terra.com.br [200.176.3.45]) by paiol.terra.com.br (Postfix) with ESMTP id BCFFF88311 for ; Wed, 9 Apr 2003 19:54:12 -0300 (BRT) Received: from niski.com (unknown [200.148.199.29]) (authenticated user fniski) by gunga.terra.com.br (Postfix) with ESMTP id 9CCAB12806D for ; Wed, 9 Apr 2003 19:54:11 -0300 (BRT) Message-ID: <3E94A49C.4000609@niski.com> Date: Wed, 09 Apr 2003 15:54:20 -0700 From: niski User-Agent: Mozilla/5.0 (Windows; U; Windows NT 5.1; en-US; rv:1.0.2) Gecko/20030208 Netscape/7.02 X-Accept-Language: en-us, en MIME-Version: 1.0 To: obm-l@mat.puc-rio.br Subject: Re: [obm-l] duvida em estatistica References: <200304092121.h39LL9r7025096@trex.centroin.com.br> Content-Type: text/plain; charset=ISO-8859-1; format=flowed Content-Transfer-Encoding: 8bit Sender: owner-obm-l@sucuri.mat.puc-rio.br Precedence: bulk Reply-To: obm-l@mat.puc-rio.br Olá prof. Obrigado pelo texto! Muito bom o material... Me deu um estalo, e eu entendi o que eu nao estava entendendo! O meu livro primeiro define variancia como Var(x) = Somatorio[i=1 até n] ((x[i] - E(x))^2)p[i] O que eu achei melhor do que no seu texto, já que para mim é mais paupavel do que definir como E[(X-E(X))^2]. O problema, é que de uma hora pra outra o livro começa a usar a variancia direto como sendo E[(X-E(X))^2]...e eu nao entendia como de uma formula se chegava na outra...dai eu vi que de fato, fazendo a variavel aleatoria valer (X-E(X))^2 e subistituindo na definicao de esperança matematica eu caia primeira formula apresentada pelo meu livro. Então eu te pergunto professor..qual é a definição original de variancia? Me parece a que foi apresentada inicialmente pelo meu livro pois me pareceu muito artificioso escolher uma variavel aleatoria (X-E(X))^2 e "entuchar" isso na definicao de esperanca para cair na formula que voce apresentou como a de variancia. Augusto Cesar de Oliveira Morgado wrote: > Veja a prova em Variáveis Aleatórias 1 dentro de www.fgv.br em graduação RJ ciclo básico, disciplina Teoria da Probabilidade, materiais. > > > > Em Wed, 09 Apr 2003 14:13:42 -0700, niski disse: > > >>pessoal...estou estudando variaveis aleatorias discretas, e não consigo >>deduzir que >>Var(X) = E(X^2) - [(E(X)]^2 , dadas as definicoes de variancia: >> >>Var(X) = Somatorio[i=1 até n] ((x[i] - E(X))^2)p[i] >>e >>Esperança Matematica >>E(X) = Somatorio[i=1 até n] x[i]p[i] >> >>Aceito qualquer ajuda. >>obrigado a todos >> >>niski >>www.linux.ime.usp.br/~niski >>-- >>[about him:] >> It is rare to find learned men who are clean, do not stink and have a >>sense of humour. >>-Gottfried Whilhem Leibniz >> >>========================================================================= >>Instruções para entrar na lista, sair da lista e usar a lista em >>http://www.mat.puc-rio.br/~nicolau/olimp/obm-l.html >>O administrador desta lista é >>========================================================================= >> >> > > > ========================================================================= > Instruções para entrar na lista, sair da lista e usar a lista em > http://www.mat.puc-rio.br/~nicolau/olimp/obm-l.html > O administrador desta lista é > ========================================================================= > > -- [about him:] It is rare to find learned men who are clean, do not stink and have a sense of humour. -Gottfried Whilhem Leibniz ========================================================================= Instruções para entrar na lista, sair da lista e usar a lista em http://www.mat.puc-rio.br/~nicolau/olimp/obm-l.html O administrador desta lista é ========================================================================= From owner-obm-l@sucuri.mat.puc-rio.br Wed Apr 9 20:09:38 2003 Return-Path: Received: (from majordom@localhost) by sucuri.mat.puc-rio.br (8.9.3/8.9.3) id UAA32370 for obm-l-MTTP; Wed, 9 Apr 2003 20:08:18 -0300 Received: from ivoti.terra.com.br (ivoti.terra.com.br [200.176.3.20]) by sucuri.mat.puc-rio.br (8.9.3/8.9.3) with ESMTP id UAA32365 for ; Wed, 9 Apr 2003 20:08:15 -0300 Received: from bertioga.terra.com.br (bertioga.terra.com.br [200.176.3.77]) by ivoti.terra.com.br (Postfix) with ESMTP id 28329408174 for ; Wed, 9 Apr 2003 20:07:44 -0300 (BRT) Received: from niski.com (unknown [200.148.199.29]) (authenticated user fniski) by bertioga.terra.com.br (Postfix) with ESMTP id 2C6453F8034 for ; Wed, 9 Apr 2003 20:07:43 -0300 (BRT) Message-ID: <3E94A7C7.9050202@niski.com> Date: Wed, 09 Apr 2003 16:07:51 -0700 From: niski User-Agent: Mozilla/5.0 (Windows; U; Windows NT 5.1; en-US; rv:1.0.2) Gecko/20030208 Netscape/7.02 X-Accept-Language: en-us, en MIME-Version: 1.0 To: obm-l@mat.puc-rio.br Subject: Re: RES: [obm-l] duvida em estatistica References: <000701c2fee6$8fdb91e0$3c766a8f@lei.ime.unicamp.br> Content-Type: text/plain; charset=ISO-8859-1; format=flowed Content-Transfer-Encoding: 8bit Sender: owner-obm-l@sucuri.mat.puc-rio.br Precedence: bulk Reply-To: obm-l@mat.puc-rio.br Mario Salvatierra Junior wrote: > Por definição, para qualquer variável aleatória X, continua ou discreta, > VarX=E[(X-EX)^2]. > Então, > VarX=E[(X-EX)^2]=E[X^2-2XE(X)+[E(X)]^2]=E(X^2)-2E(X)E(X)+[E(X)]^2= > =E(X^2)-[E(X)]^2 > A segunda igualdade vem pelo fato da linearidade da > esperança................ > E lembre que E(constante)=constante, assim E[[E(X)]^2]= [E(X)]^2 Se eu entendi bem a ultima linha, fazendo todas as passagems fica E[[E(X)]^2] = E[1*[E(X)]^2] = [E(X)]^2. E[1] = [E(X)]^2 certo!? -- [about him:] It is rare to find learned men who are clean, do not stink and have a sense of humour. -Gottfried Whilhem Leibniz ========================================================================= Instruções para entrar na lista, sair da lista e usar a lista em http://www.mat.puc-rio.br/~nicolau/olimp/obm-l.html O administrador desta lista é ========================================================================= From owner-obm-l@sucuri.mat.puc-rio.br Wed Apr 9 20:38:55 2003 Return-Path: Received: (from majordom@localhost) by sucuri.mat.puc-rio.br (8.9.3/8.9.3) id UAA01152 for obm-l-MTTP; Wed, 9 Apr 2003 20:36:52 -0300 Received: from imo-r08.mx.aol.com (imo-r08.mx.aol.com [152.163.225.104]) by sucuri.mat.puc-rio.br (8.9.3/8.9.3) with ESMTP id UAA01131 for ; Wed, 9 Apr 2003 20:36:45 -0300 From: Lltmdrtm@aol.com Received: from Lltmdrtm@aol.com by imo-r08.mx.aol.com (mail_out_v34.21.) id z.6b.e667a00 (4184) for ; Wed, 9 Apr 2003 19:36:08 -0400 (EDT) Message-ID: <6b.e667a00.2bc60868@aol.com> Date: Wed, 9 Apr 2003 19:36:08 EDT Subject: Re: [obm-l] ajuda-colmeia To: obm-l@mat.puc-rio.br MIME-Version: 1.0 Content-Type: text/plain; charset="US-ASCII" Content-Transfer-Encoding: 7bit X-Mailer: 7.0 for Windows sub 10501 Sender: owner-obm-l@sucuri.mat.puc-rio.br Precedence: bulk Reply-To: obm-l@mat.puc-rio.br ========================================================================= Instruções para entrar na lista, sair da lista e usar a lista em http://www.mat.puc-rio.br/~nicolau/olimp/obm-l.html O administrador desta lista é ========================================================================= From owner-obm-l@sucuri.mat.puc-rio.br Wed Apr 9 20:47:31 2003 Return-Path: Received: (from majordom@localhost) by sucuri.mat.puc-rio.br (8.9.3/8.9.3) id UAA01438 for obm-l-MTTP; Wed, 9 Apr 2003 20:46:03 -0300 Received: from artemis.opendf.com.br (artemis.opengate.com.br [200.181.71.14]) by sucuri.mat.puc-rio.br (8.9.3/8.9.3) with ESMTP id UAA01434 for ; Wed, 9 Apr 2003 20:45:59 -0300 Received: from localhost (localhost [127.0.0.1]) by artemis.opendf.com.br (Postfix) with ESMTP id F16252BEC4 for ; Wed, 9 Apr 2003 20:45:56 -0300 (BRT) Received: from artemis.opendf.com.br ([127.0.0.1]) by localhost (artemis.opengate.com.br [127.0.0.1:10024]) (amavisd-new) with ESMTP id 16734-08 for ; Wed, 9 Apr 2003 20:45:56 -0300 (BRT) Received: from artur (200-181-089-242.bsace7001.dsl.brasiltelecom.net.br [200.181.89.242]) by artemis.opendf.com.br (Postfix) with ESMTP id D6E452BEC2 for ; Wed, 9 Apr 2003 20:45:55 -0300 (BRT) From: "Artur Costa Steiner" To: Subject: RE: [obm-l] duvida em estatistica Date: Wed, 9 Apr 2003 20:47:54 -0700 Message-ID: <002701c2ff13$f80da320$0c01a8c0@mshome.net> MIME-Version: 1.0 Content-Type: text/plain; charset="iso-8859-1" X-Priority: 3 (Normal) X-MSMail-Priority: Normal X-Mailer: Microsoft Outlook, Build 10.0.2627 X-MimeOLE: Produced By Microsoft MimeOLE V6.00.2800.1106 In-Reply-To: <3E94A49C.4000609@niski.com> Importance: Normal X-Virus-Scanned: by amavisd-new Content-Transfer-Encoding: 8bit X-MIME-Autoconverted: from quoted-printable to 8bit by sucuri.mat.puc-rio.br id UAA01435 Sender: owner-obm-l@sucuri.mat.puc-rio.br Precedence: bulk Reply-To: obm-l@mat.puc-rio.br Oi Niski, Eu não sou o Morgado, mas posso dar uma pequena ajuda porque trabalho variâncias e desvios padrões de parâmetros associados a sistemas elétricos. A definição usual de variância é E[X - E(x)]^2. A fórmula enevolvendo somatórios ou integrais é consequência desta definição. A raiz quadrada da variância é conhecida por desvio padrão. Qual o significado destas fórmulas? Observe que a vari6ancia mede de fato uma variação enm torno da esperança e sempre gera um número postivo. É uma medida da \dispersão dos valores em torno de sua esperança. A esperança é uma medida de tendência central, assim como outros parâmetros como a mediana e a moda. A vari6ancia e a esperança podem também ser definidos para um conjunto de observações, de uma forma bem semelhante a que se adapta a variáveis aleatórias com distribuição conhecida. Observações muito dispersas tem alta variância. Note que a variância é dimensionalmente expressa numa unidae que é o quadrado da variável analisada, ap paso que o desvio padrão é dadao na mesma unidade. Abraços. ========================================================================= Instruções para entrar na lista, sair da lista e usar a lista em http://www.mat.puc-rio.br/~nicolau/olimp/obm-l.html O administrador desta lista é ========================================================================= From owner-obm-l@sucuri.mat.puc-rio.br Wed Apr 9 21:18:20 2003 Return-Path: Received: (from majordom@localhost) by sucuri.mat.puc-rio.br (8.9.3/8.9.3) id VAA02765 for obm-l-MTTP; Wed, 9 Apr 2003 21:16:44 -0300 Received: from itaqui.terra.com.br (itaqui.terra.com.br [200.176.3.19]) by sucuri.mat.puc-rio.br (8.9.3/8.9.3) with ESMTP id VAA02759 for ; Wed, 9 Apr 2003 21:16:40 -0300 Received: from canela.terra.com.br (canela.terra.com.br [200.176.3.79]) by itaqui.terra.com.br (Postfix) with ESMTP id C15D73BC767 for ; Wed, 9 Apr 2003 21:16:09 -0300 (BRT) Received: from [200.177.176.145] (dl-nas1-sao-C8B1B091.p001.terra.com.br [200.177.176.145]) by canela.terra.com.br (Postfix) with ESMTP id BBFB011400E for ; Wed, 9 Apr 2003 21:16:08 -0300 (BRT) User-Agent: Microsoft-Outlook-Express-Macintosh-Edition/5.02.2022 Date: Wed, 09 Apr 2003 21:14:56 -0300 Subject: Re: [obm-l] Bijecao From: Claudio Buffara To: Message-ID: In-Reply-To: <20030409142314.C17897@sucuri.mat.puc-rio.br> Mime-version: 1.0 Content-type: text/plain; charset="ISO-8859-1" Content-Transfer-Encoding: 8bit X-MIME-Autoconverted: from quoted-printable to 8bit by sucuri.mat.puc-rio.br id VAA02760 Sender: owner-obm-l@sucuri.mat.puc-rio.br Precedence: bulk Reply-To: obm-l@mat.puc-rio.br on 09.04.03 14:23, Nicolau C. Saldanha at nicolau@sucuri.mat.puc-rio.br wrote: > On Tue, Apr 08, 2003 at 10:47:19PM -0300, Claudio Buffara wrote: >> Caros colegas da lista: >> >> Dado um conjunto infinito A, seja B o conjunto de todas as sequencias >> finitas cujos termos pertencem a A. >> >> Pergunta: existe uma bijecao entre A e B? > > Sim. Mas a demonstração geral não é muito fácil. Tente provar > que sempre existe uma bijeção entre AxA e A para A infinito, > já não é fácil. Eu tenho quase certeza que você precisa > do axioma da escolha. > > O livro 'Set Theory' de Jech tem tudo o que você quer. > >> Eu sei que a resposta eh sim quando A eh enumeravel e suspeito que seja sim >> em geral, mas nao estou conseguindo amarrar o argumento. >> >> Esta duvida apareceu ao tentar resolver o seguinte problema: >> >> Seja R um anel tal que cada funcao de R em R pode ser expressa como um >> polinomio com coeficientes em R. >> Prove que R eh um corpo finito. >> >> Agradeco qualquer ajuda. > > O que você parece querer fazer é provar que se R é infinito > então o cardinal do conjunto de todas as funções de R em R > é maior que o cardinal do conjunto dos polinômios com coeficientes > em R. Isto é verdade e acho que pode ser demonstrado por uma variação do > argumento diagonal de Cantor, deve ser mais fácil do que a outra > pergunta que você fez. > Oi, Nicolau. Eh exatamente o que eu tenho em mente. Vou tentar provar isso e procurar o livro que voce indicou. Quanto ao restante do problema sobre o anel, supondo que R eh um anel finito com 1 eu fiz o seguinte: Tome um elemento arbitrario "c" de R (c <> 0). Considere a funcao (polinomial, por hipotese) de R em R tal que: f(0) = 0 e f(c) = 1. Como f nao eh constante, existe um inteiro positivo n e n+1 elementos (nao necessariamente distintos) de R: b_0, b_1, ..., b_n tais que: f(x) = b_0 + b_1*x + ... + b_n*x^n f(0) = 0 ==> b_0 + b_1*0 + ... + b_n*0^n = 0 ==> b_0 = 0 f(c) = b_1*c + ... + b_n*c^n = 1 ==> (b_1 + b_2*c + ... + b_n*c^(n-1))*c = 1 ==> d = b_1 + b_2*c + ... + b_n*c^(n-1) eh um inverso a esquerda de c Conclusao: em R todo elemento <> 0 tem um inverso a esquerda. Agora, tome a em R. Pelo que acabamos de ver, existe b em R tal que b*a = 1. Alem disso, existe c em R tal que c*b = 1. Entao: a*b = (1*a)*b = ((c*b)*a)*b = (c*(b*a))*b = (c*1)*b = c*b = 1. Logo, os inversos a direita e a esquerda de a sao iguais ==> a eh invertivel ==> R eh um anel de divisao finito ==> R eh um corpo finito. Minhas perguntas: 1) Como provar, usando apenas que R eh um anel finito onde todas as funcoes de R em r sao polinomiais, que R tem uma identidade? 2) Como provar que estas mesmas condicoes implicam que R eh comutativo, sem usar o teorema de Wedderburn (anel de divisao finito ==> corpo)? Obrigado pelas dicas e um abraco, Claudio. ========================================================================= Instruções para entrar na lista, sair da lista e usar a lista em http://www.mat.puc-rio.br/~nicolau/olimp/obm-l.html O administrador desta lista é ========================================================================= From owner-obm-l@sucuri.mat.puc-rio.br Wed Apr 9 21:18:20 2003 Return-Path: Received: (from majordom@localhost) by sucuri.mat.puc-rio.br (8.9.3/8.9.3) id VAA02771 for obm-l-MTTP; Wed, 9 Apr 2003 21:16:46 -0300 Received: from paiol.terra.com.br (paiol.terra.com.br [200.176.3.18]) by sucuri.mat.puc-rio.br (8.9.3/8.9.3) with ESMTP id VAA02761 for ; Wed, 9 Apr 2003 21:16:41 -0300 Received: from canela.terra.com.br (canela.terra.com.br [200.176.3.79]) by paiol.terra.com.br (Postfix) with ESMTP id D9F8087EC6 for ; Wed, 9 Apr 2003 21:16:10 -0300 (BRT) Received: from [200.177.176.145] (dl-nas1-sao-C8B1B091.p001.terra.com.br [200.177.176.145]) by canela.terra.com.br (Postfix) with ESMTP id 1BD4D11400E for ; Wed, 9 Apr 2003 21:16:10 -0300 (BRT) User-Agent: Microsoft-Outlook-Express-Macintosh-Edition/5.02.2022 Date: Wed, 09 Apr 2003 21:14:56 -0300 Subject: Re: [obm-l] Fibonacci From: Claudio Buffara To: Message-ID: In-Reply-To: <002c01c2fea8$eddade60$eeaaa5c8@epq.ime.eb.br> Mime-version: 1.0 Content-type: text/plain; charset="ISO-8859-1" Content-Transfer-Encoding: 8bit X-MIME-Autoconverted: from quoted-printable to 8bit by sucuri.mat.puc-rio.br id VAA02764 Sender: owner-obm-l@sucuri.mat.puc-rio.br Precedence: bulk Reply-To: obm-l@mat.puc-rio.br on 09.04.03 12:01, Marcio at marciocohen@superig.com.br wrote: > Obrigado ao pessoal que se manifestou na questao do rearranjo! > Segue aqui um outro problema legal, que tambem ja circulou (sem resposta) > pela lista. > > Esse eu consegui fazer (na época eu não tinha conseguido), mas minha solução > é meio feia. Fica aqui pra voces tentarem também. Se alguém quiser depois eu > mando a solução. > > Seja F_n o n-esimo nr. de fibonacci. Mostre que a serie 1/(F_n) converge, e > determine sua soma. > > Abracos, > Marcio > > PS: Eu iria mandar pra Eureka como proposto, mas achei universitario demais. > Oi, Marcio: Estou supondo que F(1) = F(2) = 1 e F(n) = F(n-1) + F(n-2) para n >= 3. A convergencia eh consequencia do seguinte resultado, que pode ser provado por inducao completa: Para todo n >= 3 F(n) > (5/4)^n Dem: F(3) = 2 > (5/4)^3 = 1,953125 Suponha que para 3 <= k <= n-1 tenhamos F(k) > (5/4)^k Entao: F(n) = F(n-1) + F(n-2) > (5/4)^(n-1) + (5/4)^(n-2) = = (5/4 + 1)*(5/4)^(n-2) = (9/4)*(5/4)^(n-2) > (25/16)*(5/4)^(n-2) = (5/4)^n ----- Como, para n >=3, F(n) > (5/4)^n, temos que: para n >= 3, 0 < 1/F(n) < (4/5)^n. Alem disso, SOMA(n>=3) (4/5)^n converge. Logo SOMA(n>=3) 1/F(n) converge, pelo teste da comparacao. ******* Acho que a soma pode sair atraves da formula de Binet: F(n) = (1/raiz(5))*(A^n - B^n), onde: A = (1+raiz(5))/2 e B = (1-raiz(5))/2 mas ainda nao encontrei o caminho. Gostei do problema. Seria uma pena se todas as solucoes fossem feias, pois a sequencia de Fibonacci eh tao "bonitinha"... Um abraco, Claudio. ========================================================================= Instruções para entrar na lista, sair da lista e usar a lista em http://www.mat.puc-rio.br/~nicolau/olimp/obm-l.html O administrador desta lista é ========================================================================= From owner-obm-l@sucuri.mat.puc-rio.br Wed Apr 9 22:45:28 2003 Return-Path: Received: (from majordom@localhost) by sucuri.mat.puc-rio.br (8.9.3/8.9.3) id WAA05080 for obm-l-MTTP; Wed, 9 Apr 2003 22:43:51 -0300 Received: from puma.unisys.com.br (smtp.unisys.com.br [200.220.64.7]) by sucuri.mat.puc-rio.br (8.9.3/8.9.3) with ESMTP id WAA05075 for ; Wed, 9 Apr 2003 22:43:46 -0300 Received: from jf (riopm18p134.uninet.com.br [200.220.16.134]) by puma.unisys.com.br (8.12.9/8.12.3) with SMTP id h3A1hALC010405 for ; Wed, 9 Apr 2003 22:43:13 -0300 (EST) X-Spam-Filter: check_local@puma.unisys.com.br by digitalanswers.org Message-ID: <00ca01c2ff02$c04ec380$a710dcc8@jf> From: "Jose Francisco Guimaraes Costa" To: "obm-l" Subject: [obm-l] =?iso-8859-1?Q?=DAltimo_Teorema_de_Fermat_para_n=3D3?= Date: Wed, 9 Apr 2003 22:37:20 -0300 MIME-Version: 1.0 Content-Type: multipart/alternative; boundary="----=_NextPart_000_0084_01C2FEE8.94C680A0" X-Priority: 3 X-MSMail-Priority: Normal X-Mailer: Microsoft Outlook Express 6.00.2800.1106 X-MimeOLE: Produced By Microsoft MimeOLE V6.00.2800.1106 Sender: owner-obm-l@sucuri.mat.puc-rio.br Precedence: bulk Reply-To: obm-l@mat.puc-rio.br This is a multi-part message in MIME format. ------=_NextPart_000_0084_01C2FEE8.94C680A0 Content-Type: text/plain; charset="iso-8859-1" Content-Transfer-Encoding: quoted-printable O livro An Introduction to the Theory of Numbers (GH Hardy, EM Wright) = fornece a demonstra=E7=E3o de Fermat para n=3D4, usando o "method of = descent". Segundo o livro, este =E9 o =FAnico caso "f=E1cil" do teorema. = O livro tamb=E9m fornece a demonstra=E7=E3o para n=3D3, sem fazer = qualquer men=E7=E3o a Euler. Para mim, =E9 mais f=E1cil entender a = Cr=EDtica da Raz=E3o Pura (Kant) escrita em aramaico do que a = demonstra=E7=E3o do UTF para n=3D3. JF ----- Original Message -----=20 From: Cl=E1udio (Pr=E1tica)=20 To: obm-l@mat.puc-rio.br=20 Sent: Wednesday, April 09, 2003 1:21 PM Subject: Re: [obm-l] Fw: sqrt(12a^3 - 3) Oi, Salvador: Acho que Fermat provou o caso n =3D 4 (de fato, usando descida = infinita). O caso n =3D 3 foi provado por Euler usando aritm=E9tica em Z[raiz(-3)], = se n=E3o me engano, e a demonstra=E7=E3o =E9 mais dif=EDcil do que o caso n =3D 4. Um abra=E7o, Claudio. ----- Original Message ----- From: "Salvador Addas Zanata" To: Sent: Wednesday, April 09, 2003 11:39 AM Subject: Re: [obm-l] Fw: sqrt(12a^3 - 3) > > So um pequeno pitaco: A versao do ultimo teorema de fermat para n=3D3 = nao e > muito dificil de provar, se nao me engano foi provada pelo proprio. = Acho > que saiu na rpm, tem tambem no livro "100 great problems of elementary > mathematics", Dorrie. Usa o principio da descida infinita (nao sei se = o > nome eh exatamente esse). A ideia eh a partir de uma sol., construir = outra > onde pelo menos um dos numeros eh estritamente menor que o outro. > > > Abraco, > > Salvador > > > On Tue, 8 Apr 2003, Wagner wrote: > > > > > ----- Original Message ----- > > From: Cl=E1udio (Pr=E1tica) > > To: Wagner > > Sent: Monday, April 07, 2003 4:02 PM > > Subject: Re: sqrt(12a^3 - 3) > > > > > > Oi, Andr=E9: > > > > Gostei muito do problema. Realmente, o UTDF nunca chegou a me passar pela cabe=E7a - foi, sem d=FAvida, uma =F3tima id=E9ia. > > > > Acho que voc=EA deveria mandar esta solu=E7=E3o pra lista. > > > > Obrigado e um abra=E7o, > > Claudio. > > ----- Original Message ----- > > From: Wagner > > To: Cl=E1udio (Pr=E1tica) > > Sent: Friday, April 04, 2003 9:52 PM > > Subject: Re: sqrt(12a^3 - 3) > > > > > > Oi Cl=E1udio > > > > Fui eu que inventei esse problema. > > A solu=E7=E3o =E9 muito mais dif=EDcil do que parece > > Foi assim que eu criei esse problema: > > Se a,b,c s=E3o tr=EAs n=FAmeros inteiros, tais que: > > (a+b)^3 =3D a^3 + c^3. > > Ent=E3o segundo o =FAltimo teorema de Fermat > > (a+b),a ou c =E9 igual a zero. Pois se eles fossem > > todos n=E3o nulos, isso seria uma contradi=E7=E3o do > > teorema no caso n=3D3. > > Se (a+b)^3 =3D a^3 + c^3 . Ent=E3o: > > ((a+b)/b)^3 =3D (a/b)^3 + (c/b)^3. Para b diferente de zero. > > Logo: ((a/b)+1)^3 =3D (a/b)^3 + (c/b)^3. > > Sejam x e y dois n=FAmeros racionais tais que: > > x=3Da/b e y=3Dc/b. > > Ent=E3o: (x+1)^3 =3D x^3 + y^3 =3D> > > x^3 + 3x^2 + 3x + 1 - x^3 - y^3 =3D 0 =3D> > > 3x^2 + 3x + (1-y^3) =3D 0. > > Vamos calcular x em fun=E7=E3o de y: > > delta =3D 9 - 12(1 - y^3) =3D 12y^3 - 3. > > x =3D( -3 + - sqrt(delta))/6. ( i ) > > > > Agora suponha que a+b=3D0. > > Ent=E3o a =3D -b =3D> 0 =3D -b^3 + c^3 =3D> b=3Dc =3D> y=3D1 > > Se a =3D 0, b^3 =3D c^3 =3D> b=3Dc =3D> y=3D1 > > Se c =3D 0, (a+b)^3 =3D a^3 =3D> b=3D0 e ent=E3o nem x nem y fazem = sentido. > > Note que sempre que c =E9 diferente de zero, b =E9 diferente de = zero. > > Se a,b e c forem n=FAmeros inteiros e c for diferente de zero, > > ent=E3o x e y v=E3o ser n=FAmeros racionais. Mas segundo o teorema = de Fermat > > isso implica que y =3D 1. Logo x =E9 racional se e somente se y = =3D 1. > > Mas temos de ( i ) que x =E9 racional se e somente se sqrt(delta) = =3D sqrt(12y^3 - 3) > > for racional. Logo sqrt(12y^3 - 3) s=F3 =E9 racional se y=3D1. > > > > Na verdade a maior dificuldade dessa solu=E7=E3o =E9 associar o = problema ao teorema > > de Fermat (o que =E9 na verdade muito dif=EDcil) > > > > > > Andr=E9 T. > > > > > > ----- Original Message ----- > > From: Cl=E1udio (Pr=E1tica) > > To: timpa@uol.com.br > > Cc: claudio.buffara@terra.com.br > > Sent: Friday, April 04, 2003 5:00 PM > > Subject: sqrt(12a^3 - 3) > > > > > > Oi, Andre: > > > > Voc=EA j=E1 conseguiu provar que se "a" e sqrt(12a^3 - 3) s=E3o = racionais, ent=E3o a =3D 1? > > De onde voc=EA tirou esse problema? > > > > Parece que =E9 f=E1cil mas h=E1 dias eu tenho tentado sem = sucesso. > > > > Um abra=E7o, > > Claudio. ------=_NextPart_000_0084_01C2FEE8.94C680A0 Content-Type: text/html; charset="iso-8859-1" Content-Transfer-Encoding: quoted-printable

O livro An Introduction to the Theory = of Numbers=20 (GH Hardy, EM Wright) fornece a demonstra=E7=E3o de Fermat para n=3D4, = usando o=20 "method of descent". Segundo o livro, este =E9 o =FAnico caso "f=E1cil" = do teorema.=20
 
O livro tamb=E9m fornece a = demonstra=E7=E3o para n=3D3, sem=20 fazer qualquer men=E7=E3o a Euler. Para mim, =E9 mais f=E1cil = entender a Cr=EDtica da=20 Raz=E3o Pura (Kant) escrita em aramaico do que a demonstra=E7=E3o do UTF = para=20 n=3D3.
 
JF
 
----- Original Message -----
From: Cl=E1udio (Pr=E1tica) =
To: obm-l@mat.puc-rio.br
Sent: = Wednesday,=20 April 09, 2003 1:21 PM
Subject: Re: [obm-l] Fw: sqrt(12a^3 -=20 3)


Oi, Salvador:

Acho que Fermat provou o caso n =3D 4 = (de fato,=20 usando descida infinita). O
caso n =3D 3 foi provado por Euler usando = aritm=E9tica em Z[raiz(-3)], se n=E3o me
engano, e a demonstra=E7=E3o = =E9 mais dif=EDcil=20 do que o caso n =3D 4.

Um abra=E7o,
Claudio.
----- Original = Message=20 -----
From: "Salvador Addas Zanata" = <sazanata@ime.usp.br>
To:=20 <obm-l@mat.puc-rio.br>
Sent: Wednesday, April 09, 2003 11:39=20 AM
Subject: Re: [obm-l] Fw: sqrt(12a^3 - 3)


>
> = So um=20 pequeno pitaco: A versao do ultimo teorema de fermat para n=3D3 nao = e
>=20 muito dificil de provar, se nao me engano foi provada pelo proprio. = Acho
>=20 que saiu na rpm, tem tambem no livro "100 great problems of = elementary
>=20 mathematics", Dorrie. Usa o principio da descida infinita (nao sei se = o
>=20 nome eh exatamente esse). A ideia eh a partir de uma sol., construir=20 outra
> onde pelo menos um dos numeros eh estritamente menor que o = outro.
>
>
> Abraco,
>
>=20 Salvador
>
>
> On Tue, 8 Apr 2003, Wagner=20 wrote:
>
> >
> > ----- Original Message = -----
>=20 > From: Cl=E1udio (Pr=E1tica)
> > To: Wagner
> > = Sent: Monday,=20 April 07, 2003 4:02 PM
> > Subject: Re: sqrt(12a^3 - 3)
> = >
> >
> > Oi, Andr=E9:
> >
> > = Gostei muito=20 do problema. Realmente, o UTDF nunca chegou a me passar
pela cabe=E7a = - foi,=20 sem d=FAvida, uma =F3tima id=E9ia.
> >
> > Acho que = voc=EA deveria=20 mandar esta solu=E7=E3o pra lista.
> >
> > Obrigado e = um=20 abra=E7o,
> > Claudio.
> >   ----- Original = Message=20 -----
> >   From: Wagner
> >   To: = Cl=E1udio=20 (Pr=E1tica)
> >   Sent: Friday, April 04, 2003 9:52 = PM
>=20 >   Subject: Re: sqrt(12a^3 - 3)
> >
> = >
>=20 >   Oi Cl=E1udio
> >
> >   Fui = eu que=20 inventei esse problema.
> >   A solu=E7=E3o =E9 muito = mais dif=EDcil=20 do que parece
> >   Foi assim que eu criei esse=20 problema:
> >   Se a,b,c s=E3o tr=EAs n=FAmeros = inteiros, tais=20 que:
> >   (a+b)^3 =3D a^3 + c^3.
> = >  =20 Ent=E3o segundo o =FAltimo teorema de Fermat
> >   = (a+b),a ou c =E9=20 igual a zero. Pois se eles fossem
> >   todos n=E3o = nulos, isso=20 seria uma contradi=E7=E3o do
> >   teorema no caso = n=3D3.
>=20 >   Se (a+b)^3 =3D a^3 + c^3 . Ent=E3o:
> = >  =20 ((a+b)/b)^3 =3D (a/b)^3 + (c/b)^3. Para b diferente de zero.
>=20 >   Logo: ((a/b)+1)^3 =3D (a/b)^3 + (c/b)^3.
> = >  =20 Sejam x e y dois n=FAmeros racionais tais que:
> >   = x=3Da/b e=20 y=3Dc/b.
> >   Ent=E3o: (x+1)^3 =3D x^3 + y^3 = =3D>
>=20 >   x^3 + 3x^2 + 3x + 1 - x^3 - y^3 =3D 0 =3D>
>=20 >   3x^2 + 3x + (1-y^3) =3D 0.
> >   = Vamos=20 calcular x em fun=E7=E3o de y:
> >   delta =3D 9 - = 12(1 - y^3) =3D=20 12y^3 - 3.
> >   x =3D( -3 + - sqrt(delta))/6. ( i = )
>=20 >
> >   Agora suponha que a+b=3D0.
> = >  =20 Ent=E3o a =3D -b =3D> 0 =3D -b^3 + c^3 =3D> b=3Dc =3D> = y=3D1
> >  =20 Se a =3D 0, b^3 =3D c^3 =3D> b=3Dc =3D> y=3D1
> = >   Se c =3D 0,=20 (a+b)^3 =3D a^3 =3D> b=3D0 e ent=E3o nem x nem y fazem = sentido.
>=20 >   Note que sempre que c =E9 diferente de zero, b =E9 = diferente de=20 zero.
> >   Se a,b e c forem n=FAmeros inteiros e c = for=20 diferente de zero,
> >   ent=E3o x e y v=E3o ser = n=FAmeros=20 racionais. Mas segundo o teorema de Fermat
> >   isso = implica=20 que y =3D 1. Logo x =E9 racional se e somente se y =3D 1.
> = >   Mas=20 temos de ( i ) que x =E9 racional se e somente se sqrt(delta) = =3D
sqrt(12y^3 -=20 3)
> >   for racional. Logo sqrt(12y^3 - 3) s=F3 =E9 = racional se=20 y=3D1.
> >
> >   Na verdade a maior = dificuldade dessa=20 solu=E7=E3o =E9 associar o problema ao
teorema
> = >   de Fermat=20 (o que =E9 na verdade muito dif=EDcil)
> >
> >
> = >   Andr=E9 T.
> >
> >
>=20 >     ----- Original Message -----
>=20 >     From: Cl=E1udio (Pr=E1tica)
>=20 >     To: timpa@uol.com.br
>=20 >     Cc: claudio.buffara@terra.com.br
>=20 >     Sent: Friday, April 04, 2003 5:00 = PM
>=20 >     Subject: sqrt(12a^3 - 3)
> = >
>=20 >
> >     Oi, Andre:
> = >
>=20 >     Voc=EA j=E1 conseguiu provar que se "a" e = sqrt(12a^3 -=20 3) s=E3o racionais,
ent=E3o a =3D 1?
> = >     De onde=20 voc=EA tirou esse problema?
> >
> = >    =20 Parece que =E9 f=E1cil mas h=E1 dias eu tenho tentado sem = sucesso.
>=20 >
> >     Um abra=E7o,
>=20 >     Claudio.
------=_NextPart_000_0084_01C2FEE8.94C680A0-- ========================================================================= Instruções para entrar na lista, sair da lista e usar a lista em http://www.mat.puc-rio.br/~nicolau/olimp/obm-l.html O administrador desta lista é ========================================================================= From owner-obm-l@sucuri.mat.puc-rio.br Wed Apr 9 23:59:46 2003 Return-Path: Received: (from majordom@localhost) by sucuri.mat.puc-rio.br (8.9.3/8.9.3) id XAA06837 for obm-l-MTTP; Wed, 9 Apr 2003 23:56:45 -0300 Received: from ivoti.terra.com.br (ivoti.terra.com.br [200.176.3.20]) by sucuri.mat.puc-rio.br (8.9.3/8.9.3) with ESMTP id XAA06832 for ; Wed, 9 Apr 2003 23:56:41 -0300 Received: from bertioga.terra.com.br (bertioga.terra.com.br [200.176.3.77]) by ivoti.terra.com.br (Postfix) with ESMTP id EF245408124 for ; Wed, 9 Apr 2003 23:56:11 -0300 (BRT) Received: from [200.177.176.40] (dl-nas1-sao-C8B1B028.p001.terra.com.br [200.177.176.40]) by bertioga.terra.com.br (Postfix) with ESMTP id BD1F73F804E for ; Wed, 9 Apr 2003 23:56:10 -0300 (BRT) User-Agent: Microsoft-Outlook-Express-Macintosh-Edition/5.02.2022 Date: Wed, 09 Apr 2003 23:54:59 -0300 Subject: Re: [obm-l] Fibonacci From: Claudio Buffara To: Message-ID: In-Reply-To: Mime-version: 1.0 Content-type: text/plain; charset="ISO-8859-1" Content-Transfer-Encoding: 8bit X-MIME-Autoconverted: from quoted-printable to 8bit by sucuri.mat.puc-rio.br id XAA06834 Sender: owner-obm-l@sucuri.mat.puc-rio.br Precedence: bulk Reply-To: obm-l@mat.puc-rio.br Marcio: Dei uma pesquisada na internet e parece a prova da irracionalidade da soma dos reciprocos dos numeros de Fibonacci foi um dos problemas propostos por Paul Erdos e que soh foi resolvido na decada de 1980. Assim, o valor exato desta soma nao deve poder ser expresso como uma combinacao de constantes conhecidas (Pi, e, raiz(5), etc.). Sendo assim, eu gostaria muito de ver a sua solucao. Um abraco, Claudio. on 09.04.03 21:14, Claudio Buffara at claudio.buffara@terra.com.br wrote: > on 09.04.03 12:01, Marcio at marciocohen@superig.com.br wrote: > >> Obrigado ao pessoal que se manifestou na questao do rearranjo! >> Segue aqui um outro problema legal, que tambem ja circulou (sem resposta) >> pela lista. >> >> Esse eu consegui fazer (na época eu não tinha conseguido), mas minha solução >> é meio feia. Fica aqui pra voces tentarem também. Se alguém quiser depois eu >> mando a solução. >> >> Seja F_n o n-esimo nr. de fibonacci. Mostre que a serie 1/(F_n) converge, e >> determine sua soma. >> >> Abracos, >> Marcio >> >> PS: Eu iria mandar pra Eureka como proposto, mas achei universitario demais. >> > Oi, Marcio: > > Estou supondo que F(1) = F(2) = 1 e F(n) = F(n-1) + F(n-2) para n >= 3. > > A convergencia eh consequencia do seguinte resultado, que pode ser provado > por inducao completa: > > Para todo n >= 3 F(n) > (5/4)^n > Dem: > F(3) = 2 > (5/4)^3 = 1,953125 > > Suponha que para 3 <= k <= n-1 tenhamos F(k) > (5/4)^k > > Entao: > F(n) = F(n-1) + F(n-2) > (5/4)^(n-1) + (5/4)^(n-2) = > = (5/4 + 1)*(5/4)^(n-2) = (9/4)*(5/4)^(n-2) > (25/16)*(5/4)^(n-2) = (5/4)^n > ----- > > Como, para n >=3, F(n) > (5/4)^n, temos que: > para n >= 3, 0 < 1/F(n) < (4/5)^n. > > Alem disso, SOMA(n>=3) (4/5)^n converge. > > Logo SOMA(n>=3) 1/F(n) converge, pelo teste da comparacao. > > ******* > > Acho que a soma pode sair atraves da formula de Binet: > > F(n) = (1/raiz(5))*(A^n - B^n), onde: > > A = (1+raiz(5))/2 e B = (1-raiz(5))/2 > > mas ainda nao encontrei o caminho. > > Gostei do problema. Seria uma pena se todas as solucoes fossem feias, pois a > sequencia de Fibonacci eh tao "bonitinha"... > > Um abraco, > Claudio. > > > ========================================================================= > Instruções para entrar na lista, sair da lista e usar a lista em > http://www.mat.puc-rio.br/~nicolau/olimp/obm-l.html > O administrador desta lista é > ========================================================================= > ========================================================================= Instruções para entrar na lista, sair da lista e usar a lista em http://www.mat.puc-rio.br/~nicolau/olimp/obm-l.html O administrador desta lista é ========================================================================= From owner-obm-l@sucuri.mat.puc-rio.br Thu Apr 10 07:39:08 2003 Return-Path: Received: (from majordom@localhost) by sucuri.mat.puc-rio.br (8.9.3/8.9.3) id HAA14644 for obm-l-MTTP; Thu, 10 Apr 2003 07:36:03 -0300 Received: from imo-r08.mx.aol.com (imo-r08.mx.aol.com [152.163.225.104]) by sucuri.mat.puc-rio.br (8.9.3/8.9.3) with ESMTP id HAA14640 for ; Thu, 10 Apr 2003 07:35:59 -0300 From: Lltmdrtm@aol.com Received: from Lltmdrtm@aol.com by imo-r08.mx.aol.com (mail_out_v34.21.) id z.1e8.64dd35c (4184) for ; Thu, 10 Apr 2003 06:35:18 -0400 (EDT) Message-ID: <1e8.64dd35c.2bc6a2e6@aol.com> Date: Thu, 10 Apr 2003 06:35:18 EDT Subject: [obm-l] ajuda-colmeia To: obm-l@mat.puc-rio.br MIME-Version: 1.0 Content-Type: multipart/alternative; boundary="part1_1e8.64dd35c.2bc6a2e6_boundary" X-Mailer: 7.0 for Windows sub 10501 Sender: owner-obm-l@sucuri.mat.puc-rio.br Precedence: bulk Reply-To: obm-l@mat.puc-rio.br --part1_1e8.64dd35c.2bc6a2e6_boundary Content-Type: text/plain; charset="ISO-8859-1" Content-Transfer-Encoding: quoted-printable Uma colmeia nova tem 8000 abelhas. Destas, a cada dia que passa, morrem 200.= =20 Do 21=BA dia em diante, nascem diariamente 2000 abelhas que vivem, em m=E9di= a, 40=20 dias. Ap=F3s um certo tempo, o n=FAmero de abelhas dessa colmeia se estabili= zar=E1=20 em, aproximadamente quanto? --part1_1e8.64dd35c.2bc6a2e6_boundary Content-Type: text/html; charset="ISO-8859-1" Content-Transfer-Encoding: quoted-printable Uma colmeia nova tem 8000 abelhas. Destas, a cada dia=20= que passa, morrem 200. Do 21=BA dia em diante, nascem diariamente 2000 abelh= as que vivem, em m=E9dia, 40 dias. Ap=F3s um certo tempo, o n=FAmero de abel= has dessa colmeia se estabilizar=E1 em, aproximadamente quanto?
--part1_1e8.64dd35c.2bc6a2e6_boundary-- ========================================================================= Instruções para entrar na lista, sair da lista e usar a lista em http://www.mat.puc-rio.br/~nicolau/olimp/obm-l.html O administrador desta lista é ========================================================================= From owner-obm-l@sucuri.mat.puc-rio.br Thu Apr 10 09:52:32 2003 Return-Path: Received: (from majordom@localhost) by sucuri.mat.puc-rio.br (8.9.3/8.9.3) id JAA16542 for obm-l-MTTP; Thu, 10 Apr 2003 09:49:38 -0300 Received: from saks.bol.com.br (saks.bol.com.br [200.221.24.16]) by sucuri.mat.puc-rio.br (8.9.3/8.9.3) with ESMTP id JAA16538 for ; Thu, 10 Apr 2003 09:49:31 -0300 Received: from bol.com.br (200.221.24.119) by saks.bol.com.br (5.1.071) id 3E766E3A0074FFC1 for obm-l@mat.puc-rio.br; Thu, 10 Apr 2003 09:48:57 -0300 Date: Thu, 10 Apr 2003 09:48:56 -0300 Message-Id: Subject: [obm-l] =?iso-8859-1?q?Dist=E2ncia_de_ponto=2E?= MIME-Version: 1.0 Content-Type: text/plain;charset="iso-8859-1" From: "helynatal" To: obm-l@mat.puc-rio.br X-XaM3-API-Version: 2.4 R3 ( B4 ) X-SenderIP: 200.222.64.218 Content-Transfer-Encoding: 8bit X-MIME-Autoconverted: from quoted-printable to 8bit by sucuri.mat.puc-rio.br id JAA16539 Sender: owner-obm-l@sucuri.mat.puc-rio.br Precedence: bulk Reply-To: obm-l@mat.puc-rio.br "Qual é o ponto que é equidistante dos pontos (0,0), (3,1) e (1,2)." Posso resolver esta questão dizendo que: d( (x,y),(0,0) ) = d( (x,y),(1,2) ) = d( (x,y), (3,1) ) __________________________________________________________________________ E-mail Premium BOL Antivírus, anti-spam e até 100 MB de espaço. Assine já! http://email.bol.com.br/ ========================================================================= Instruções para entrar na lista, sair da lista e usar a lista em http://www.mat.puc-rio.br/~nicolau/olimp/obm-l.html O administrador desta lista é ========================================================================= From owner-obm-l@sucuri.mat.puc-rio.br Thu Apr 10 11:58:57 2003 Return-Path: Received: (from majordom@localhost) by sucuri.mat.puc-rio.br (8.9.3/8.9.3) id LAA18642 for obm-l-MTTP; Thu, 10 Apr 2003 11:55:25 -0300 Received: from www.zipmail.com.br (smtp.zipmail.com.br [200.221.11.147]) by sucuri.mat.puc-rio.br (8.9.3/8.9.3) with ESMTP id LAA18638 for ; Thu, 10 Apr 2003 11:55:09 -0300 From: yurigomes@zipmail.com.br Received: from [200.253.233.61] by www.zipmail.com.br with HTTP; Thu, 10 Apr 2003 11:51:15 -0300 Message-ID: <3E9570B1000003EE@www.zipmail.com.br> Date: Thu, 10 Apr 2003 11:51:15 -0300 In-Reply-To: Subject: [obm-l] =?iso-8859-1?Q?Re=3A=20=5Bobm=2Dl=5D=20Fibonacci?= To: obm-l@mat.puc-rio.br MIME-Version: 1.0 Content-Type: text/plain; charset="iso-8859-1" Content-Transfer-Encoding: 8bit X-MIME-Autoconverted: from quoted-printable to 8bit by sucuri.mat.puc-rio.br id LAA18639 Sender: owner-obm-l@sucuri.mat.puc-rio.br Precedence: bulk Reply-To: obm-l@mat.puc-rio.br Oi Marcio, Para mostrar que ela eh convergente basta fazer o seguinte: defina F(0)=F(1)=1. Logo, para n>=0, temos F(n+2)>=2F(n), com igualdade apenas qdo n=0. Logo, podemos majorar F(n)^(-1) do seguinte modo. Para n=2k, temos F(2k)>2F(2k-2)>...>2^(k-1).F(2)=2^k e para n=2k+1, temos f(2k+1)>2F(2k-1)>...>2^(k-1).F(3)>2^k, ou seja, temos F(n)>2^[n/2], onde [x]= parte inteira de x. Desse modo, vemos que sum(1/F(n)) converge, pelo teste da comparação. Yuri -- Mensagem original -- >Marcio: > >Dei uma pesquisada na internet e parece a prova da irracionalidade da soma >dos reciprocos dos numeros de Fibonacci foi um dos problemas propostos por >Paul Erdos e que soh foi resolvido na decada de 1980. > >Assim, o valor exato desta soma nao deve poder ser expresso como uma >combinacao de constantes conhecidas (Pi, e, raiz(5), etc.). > >Sendo assim, eu gostaria muito de ver a sua solucao. > >Um abraco, >Claudio. > >on 09.04.03 21:14, Claudio Buffara at claudio.buffara@terra.com.br wrote: > >> on 09.04.03 12:01, Marcio at marciocohen@superig.com.br wrote: >> >>> Obrigado ao pessoal que se manifestou na questao do rearranjo! >>> Segue aqui um outro problema legal, que tambem ja circulou (sem resposta) >>> pela lista. >>> >>> Esse eu consegui fazer (na época eu não tinha conseguido), mas minha solução >>> é meio feia. Fica aqui pra voces tentarem também. Se alguém quiser depois >eu >>> mando a solução. >>> >>> Seja F_n o n-esimo nr. de fibonacci. Mostre que a serie 1/(F_n) converge, >e >>> determine sua soma. >>> >>> Abracos, >>> Marcio >>> >>> PS: Eu iria mandar pra Eureka como proposto, mas achei universitario demais. >>> >> Oi, Marcio: >> >> Estou supondo que F(1) = F(2) = 1 e F(n) = F(n-1) + F(n-2) para n >= >3. >> >> A convergencia eh consequencia do seguinte resultado, que pode ser provado >> por inducao completa: >> >> Para todo n >= 3 F(n) > (5/4)^n >> Dem: >> F(3) = 2 > (5/4)^3 = 1,953125 >> >> Suponha que para 3 <= k <= n-1 tenhamos F(k) > (5/4)^k >> >> Entao: >> F(n) = F(n-1) + F(n-2) > (5/4)^(n-1) + (5/4)^(n-2) = >> = (5/4 + 1)*(5/4)^(n-2) = (9/4)*(5/4)^(n-2) > (25/16)*(5/4)^(n-2) = (5/4)^n >> ----- >> >> Como, para n >=3, F(n) > (5/4)^n, temos que: >> para n >= 3, 0 < 1/F(n) < (4/5)^n. >> >> Alem disso, SOMA(n>=3) (4/5)^n converge. >> >> Logo SOMA(n>=3) 1/F(n) converge, pelo teste da comparacao. >> >> ******* >> >> Acho que a soma pode sair atraves da formula de Binet: >> >> F(n) = (1/raiz(5))*(A^n - B^n), onde: >> >> A = (1+raiz(5))/2 e B = (1-raiz(5))/2 >> >> mas ainda nao encontrei o caminho. >> >> Gostei do problema. Seria uma pena se todas as solucoes fossem feias, pois >a >> sequencia de Fibonacci eh tao "bonitinha"... >> >> Um abraco, >> Claudio. >> >> >> ========================================================================= >> Instruções para entrar na lista, sair da lista e usar a lista em >> http://www.mat.puc-rio.br/~nicolau/olimp/obm-l.html >> O administrador desta lista é >> ========================================================================= >> > >========================================================================= >Instruções para entrar na lista, sair da lista e usar a lista em >http://www.mat.puc-rio.br/~nicolau/olimp/obm-l.html >O administrador desta lista é >========================================================================= > []'s, Yuri ICQ: 64992515 ------------------------------------------ Use o melhor sistema de busca da Internet Radar UOL - http://www.radaruol.com.br ========================================================================= Instruções para entrar na lista, sair da lista e usar a lista em http://www.mat.puc-rio.br/~nicolau/olimp/obm-l.html O administrador desta lista é ========================================================================= From owner-obm-l@sucuri.mat.puc-rio.br Thu Apr 10 14:01:10 2003 Return-Path: Received: (from majordom@localhost) by sucuri.mat.puc-rio.br (8.9.3/8.9.3) id NAA20923 for obm-l-MTTP; Thu, 10 Apr 2003 13:56:40 -0300 Received: from web12905.mail.yahoo.com (web12905.mail.yahoo.com [216.136.174.72]) by sucuri.mat.puc-rio.br (8.9.3/8.9.3) with SMTP id NAA20919 for ; Thu, 10 Apr 2003 13:56:35 -0300 Message-ID: <20030410165602.92114.qmail@web12905.mail.yahoo.com> Received: from [200.206.103.3] by web12905.mail.yahoo.com via HTTP; Thu, 10 Apr 2003 13:56:02 ART Date: Thu, 10 Apr 2003 13:56:02 -0300 (ART) From: =?iso-8859-1?q?Johann=20Peter=20Gustav=20Lejeune=20Dirichlet?= Subject: Re: [obm-l](correçao) Um de matrizes e outro() To: obm-l@mat.puc-rio.br In-Reply-To: <20030409142658.D17897@sucuri.mat.puc-rio.br> MIME-Version: 1.0 Content-Type: multipart/alternative; boundary="0-961696897-1049993762=:90645" Content-Transfer-Encoding: 8bit Sender: owner-obm-l@sucuri.mat.puc-rio.br Precedence: bulk Reply-To: obm-l@mat.puc-rio.br --0-961696897-1049993762=:90645 Content-Type: text/plain; charset=iso-8859-1 Content-Transfer-Encoding: 8bit Fiz besteira!Faltou um dado e errei outro.Versao corrigida abaixo: "Nicolau C. Saldanha" wrote: On Wed, Apr 09, 2003 at 02:04:13PM -0300, Johann Peter Gustav Lejeune Dirichlet wrote: > > Oi turma,tudo beleza?Tenho duas perguntas: > > Como se resolve esse treco horrivel? > > "Temos uma matriz A n*n simetrica cujos elementos sao 0 ou 1.Sabe-se que ela tem as seguintes propriedades: > > i)Para todos os i, j e k com a_ij=a_jk=1 tem-se a_ik=0 > > ii)Se a_ij=0 entao existem i' e j' tais que a_ii'=a_ji'=a_ij'=a_jj'=1.Tais i' e j' sao unicos em relaçao aos i e j. > > Mostre que n-1 deve ser um numero triangular(do tipo 1+2+3+...+k para algum k natural)." --------------------------------- Yahoo! Mail O melhor e-mail gratuito da internet: 6MB de espaço, antivírus, acesso POP3, filtro contra spam. --0-961696897-1049993762=:90645 Content-Type: text/html; charset=iso-8859-1 Content-Transfer-Encoding: 8bit

Fiz besteira!Faltou um dado e errei outro.Versao corrigida abaixo:

 "Nicolau C. Saldanha" <nicolau@sucuri.mat.puc-rio.br> wrote:

On Wed, Apr 09, 2003 at 02:04:13PM -0300, Johann Peter Gustav Lejeune Dirichlet wrote:
>
> Oi turma,tudo beleza?Tenho duas perguntas:
>
> Como se resolve esse treco horrivel?
>
> "Temos uma matriz A n*n simetrica cujos elementos sao 0 ou 1.Sabe-se que ela tem as seguintes propriedades:
>
> i)Para todos os i, j e k com a_ij=a_jk=1 tem-se a_ik=0
>
> ii)Se a_ij=0 entao existem i' e j' tais que a_ii'=a_ji'=a_ij'=a_jj'=1.Tais i' e j' sao unicos em relaçao aos i e j.
>
> Mostre que n-1 deve ser um numero triangular(do tipo 1+2+3+...+k para algum k natural)."

 



Yahoo! Mail
O melhor e-mail gratuito da internet: 6MB de espaço, antivírus, acesso POP3, filtro contra spam. --0-961696897-1049993762=:90645-- ========================================================================= Instruções para entrar na lista, sair da lista e usar a lista em http://www.mat.puc-rio.br/~nicolau/olimp/obm-l.html O administrador desta lista é ========================================================================= From owner-obm-l@sucuri.mat.puc-rio.br Thu Apr 10 14:30:27 2003 Return-Path: Received: (from majordom@localhost) by sucuri.mat.puc-rio.br (8.9.3/8.9.3) id OAA21312 for obm-l-MTTP; Thu, 10 Apr 2003 14:27:06 -0300 Received: from web12901.mail.yahoo.com (web12901.mail.yahoo.com [216.136.174.68]) by sucuri.mat.puc-rio.br (8.9.3/8.9.3) with SMTP id OAA21297 for ; Thu, 10 Apr 2003 14:26:58 -0300 Message-ID: <20030410172625.14665.qmail@web12901.mail.yahoo.com> Received: from [200.206.103.3] by web12901.mail.yahoo.com via HTTP; Thu, 10 Apr 2003 14:26:25 ART Date: Thu, 10 Apr 2003 14:26:25 -0300 (ART) From: =?iso-8859-1?q?Johann=20Peter=20Gustav=20Lejeune=20Dirichlet?= Subject: [obm-l]Último_Teorema_de_Fermat_para_n=4 To: obm-l@mat.puc-rio.br In-Reply-To: <00ca01c2ff02$c04ec380$a710dcc8@jf> MIME-Version: 1.0 Content-Type: multipart/alternative; boundary="0-1306383560-1049995585=:13058" Content-Transfer-Encoding: 8bit Sender: owner-obm-l@sucuri.mat.puc-rio.br Precedence: bulk Reply-To: obm-l@mat.puc-rio.br --0-1306383560-1049995585=:13058 Content-Type: text/plain; charset=iso-8859-1 Content-Transfer-Encoding: 8bit Bem,o metodo da descida infinita ou descenso infinito pode ser descrito assim: i)Suponha que exista uma soluçao em naturais; ii)pegue uma soluçao minima em algum sentido; iii)tente chegar em contradiçao. Na verdade isto e o Principio da Boa Ordem para os Naturais e a relacao >. Vou demonstrar essa joça aqui: Se os naturais a,b,c sao tais que a^4+b^4=c^2 entao algum deles sera zero. Suponha o contrario,que os caras sao inteiros positivos.Entao existe uma soluçao tal que c seja o menor possivel(pelo PBO).Logo MDC(a,b) =1,e existem inteiros positivos u,v com a^2=u^2-v^2,b^2=2uv,c=u^2+v^2 (essa e a soluçao da equaçao diofantina de Pitagoras X^2+Y^2=Z^2).Reaplicando,vemos que existem inteiros positivos p,q com MDC(p,q)=1 e a=p^2-q^2,v=2pq,c=u^2+v^2. Com isso b^2=2uv=4pq(p^2+q^2).Logo como p e q sao primos entre si p,q e p^2+q^2 sao todos quadrados perfeitos.Logo existem A,B e C inteiros positivos tais que p=A^2,q=B^2,p^2+q^2=C^2. Mas com isso A^4+B^4=C^2 e c=u^2+v^2>u=p^2+q^2=C^2>C e ai temos duas contradiçoes: 1)c>C pois coisas grandes sao maiores que coisas pequenas:) ;) 2)c<=C por hipotese de minimalidade. E fim! --------------------------------- Yahoo! Mail O melhor e-mail gratuito da internet: 6MB de espaço, antivírus, acesso POP3, filtro contra spam. --0-1306383560-1049995585=:13058 Content-Type: text/html; charset=iso-8859-1 Content-Transfer-Encoding: 8bit

Bem,o metodo da descida infinita ou descenso infinito pode ser descrito assim:

i)Suponha que exista uma soluçao em naturais;

ii)pegue uma soluçao minima em algum sentido;

iii)tente chegar em contradiçao.

Na verdade isto e o Principio da Boa Ordem para os Naturais e a relacao >.

Vou demonstrar essa joça aqui:

Se os naturais a,b,c sao tais que a^4+b^4=c^2 entao algum deles sera zero.

Suponha o contrario,que os caras sao inteiros positivos.Entao existe uma soluçao tal que c seja o menor possivel(pelo PBO).Logo MDC(a,b) =1,e existem inteiros positivos u,v com

a^2=u^2-v^2,b^2=2uv,c=u^2+v^2

(essa e a soluçao da equaçao diofantina de Pitagoras X^2+Y^2=Z^2).Reaplicando,vemos que existem inteiros positivos p,q com

MDC(p,q)=1 e a=p^2-q^2,v=2pq,c=u^2+v^2.

Com isso b^2=2uv=4pq(p^2+q^2).Logo como p e q sao primos entre si p,q e p^2+q^2 sao todos quadrados perfeitos.Logo existem A,B e C inteiros positivos tais que 

p=A^2,q=B^2,p^2+q^2=C^2.

Mas com isso A^4+B^4=C^2 e c=u^2+v^2>u=p^2+q^2=C^2>C e ai temos duas contradiçoes:

1)c>C pois coisas grandes sao maiores que coisas pequenas:) ;) 

2)c<=C por hipotese de minimalidade. 

E fim!



Yahoo! Mail
O melhor e-mail gratuito da internet: 6MB de espaço, antivírus, acesso POP3, filtro contra spam. --0-1306383560-1049995585=:13058-- ========================================================================= Instruções para entrar na lista, sair da lista e usar a lista em http://www.mat.puc-rio.br/~nicolau/olimp/obm-l.html O administrador desta lista é ========================================================================= From owner-obm-l@sucuri.mat.puc-rio.br Thu Apr 10 14:32:20 2003 Return-Path: Received: (from majordom@localhost) by sucuri.mat.puc-rio.br (8.9.3/8.9.3) id OAA21398 for obm-l-MTTP; Thu, 10 Apr 2003 14:29:38 -0300 Received: from cmsrelay05.mx.net (cmsrelay05.mx.net [165.212.11.2]) by sucuri.mat.puc-rio.br (8.9.3/8.9.3) with SMTP id OAA21394 for ; Thu, 10 Apr 2003 14:29:33 -0300 Received: from uadvg129.cms.usa.net (HELO localhost) (165.212.11.129) by cmsoutbound.mx.net with SMTP; 10 Apr 2003 17:28:59 -0000 Received: from smtp.postoffice.net [165.212.8.2] by uadvg129.cms.usa.net (ASMTP/) via mtad (C8.MAIN.2.05) with ESMTP id 163HDJRC50119M29; Thu, 10 Apr 2003 17:28:56 GMT Received: from 200.181.4.100 [200.181.4.100] by uwdvg002.cms.usa.net (USANET web-mailer CM.0402.5.2B); Thu, 10 Apr 2003 17:28:55 -0000 Date: Thu, 10 Apr 2003 14:28:55 -0300 From: Artur Costa Steiner To: Subject: [obm-l] =?ISO-8859-1?Q?Re=3A=20=5B=5Bobm=2Dl=5D=20Dist=E2ncia=20d?= =?ISO-8859-1?Q?e=20ponto=2E=5D?= X-Mailer: USANET web-mailer (CM.0402.5.2B) Mime-Version: 1.0 Message-ID: <442HDJRC42544S02.1049995735@uwdvg002.cms.usa.net> Content-Type: text/plain; charset=ISO-8859-1 Content-Transfer-Encoding: 8bit X-MIME-Autoconverted: from quoted-printable to 8bit by sucuri.mat.puc-rio.br id OAA21395 Sender: owner-obm-l@sucuri.mat.puc-rio.br Precedence: bulk Reply-To: obm-l@mat.puc-rio.br "helynatal" wrote: > "Qual é o ponto que é equidistante dos pontos (0,0), > (3,1) e (1,2)." > > > Posso resolver esta questão dizendo que: > > d( (x,y),(0,0) ) = d( (x,y),(1,2) ) = d( (x,y), (3,1) ) Sem dúvida. E temos que x^2 + y^2 = (x-1)^2 + (y-2)^2 Desenvolvendo os quadrados, concluimos que 2x + 4y -5 =0. temos também que (x-1)^2 + (y-2)^2 = (x-3)^2 + (y-1)^2 , o que nos leva a 2x + 4y - 5 = 6x + 2y -10. Logo, 3x + y -5 = 0. A solução é x= 1,5, y = 0,5 (se eu não cometi nenhum engano). Observe que existe um e apenas um ponto equidistante, o qual é a intersecção das mediatrizes do triãngulo definido pelos pontos dados. Artur ========================================================================= Instruções para entrar na lista, sair da lista e usar a lista em http://www.mat.puc-rio.br/~nicolau/olimp/obm-l.html O administrador desta lista é ========================================================================= From owner-obm-l@sucuri.mat.puc-rio.br Thu Apr 10 15:12:13 2003 Return-Path: Received: (from majordom@localhost) by sucuri.mat.puc-rio.br (8.9.3/8.9.3) id PAA22233 for obm-l-MTTP; Thu, 10 Apr 2003 15:08:32 -0300 Received: from ns3bind.localdomain ([200.230.34.5]) by sucuri.mat.puc-rio.br (8.9.3/8.9.3) with ESMTP id PAA22227 for ; Thu, 10 Apr 2003 15:08:27 -0300 Received: from servico2 ([200.230.34.229]) by ns3bind.localdomain (8.11.6/X.XX.X) with SMTP id h3AI4FE05178 for ; Thu, 10 Apr 2003 15:04:16 -0300 Message-ID: <043c01c2ff8c$3afda180$3300c57d@bovespa.com> From: "=?iso-8859-1?Q?Cl=E1udio_\=28Pr=E1tica\=29?=" To: References: Subject: [obm-l] =?iso-8859-1?Q?Re:_=5Bobm-l=5D_Dist=E2ncia_de_ponto.?= Date: Thu, 10 Apr 2003 15:08:45 -0300 MIME-Version: 1.0 Content-Type: text/plain; charset="iso-8859-1" Content-Transfer-Encoding: 8bit X-Priority: 3 X-MSMail-Priority: Normal X-Mailer: Microsoft Outlook Express 5.50.4920.2300 X-MimeOLE: Produced By Microsoft MimeOLE V5.50.4920.2300 Sender: owner-obm-l@sucuri.mat.puc-rio.br Precedence: bulk Reply-To: obm-l@mat.puc-rio.br ----- Original Message ----- From: "helynatal" To: Sent: Thursday, April 10, 2003 9:48 AM Subject: [obm-l] Distância de ponto. > "Qual é o ponto que é equidistante dos pontos (0,0), > (3,1) e (1,2)." > > > Posso resolver esta questão dizendo que: > > d( (x,y),(0,0) ) = d( (x,y),(1,2) ) = d( (x,y), (3,1) ) > Certamente você pode e não se preocupe: os termos ao quadrado se cancelam e voce chega num sistema linear de 2 equações e 2 incógnitas fácil de resolver. Um abraço, Claudio. ========================================================================= Instruções para entrar na lista, sair da lista e usar a lista em http://www.mat.puc-rio.br/~nicolau/olimp/obm-l.html O administrador desta lista é ========================================================================= From owner-obm-l@sucuri.mat.puc-rio.br Thu Apr 10 16:42:25 2003 Return-Path: Received: (from majordom@localhost) by sucuri.mat.puc-rio.br (8.9.3/8.9.3) id QAA24771 for obm-l-MTTP; Thu, 10 Apr 2003 16:39:20 -0300 Received: from cmsrelay03.mx.net (cmsrelay03.mx.net [165.212.11.112]) by sucuri.mat.puc-rio.br (8.9.3/8.9.3) with SMTP id QAA24767 for ; Thu, 10 Apr 2003 16:39:16 -0300 Received: from uadvg130.cms.usa.net (HELO localhost) (165.212.11.130) by cmsoutbound.mx.net with SMTP; 10 Apr 2003 19:38:39 -0000 Received: from smtp.postoffice.net [165.212.8.2] by uadvg130.cms.usa.net (ASMTP/) via mtad (C8.MAIN.2.05) with ESMTP id 227HDJTMn0138M30; Thu, 10 Apr 2003 19:38:39 GMT Received: from 200.181.4.100 [200.181.4.100] by uwdvg002.cms.usa.net (USANET web-mailer CM.0402.5.2B); Thu, 10 Apr 2003 19:38:38 -0000 Date: Thu, 10 Apr 2003 16:38:38 -0300 From: Artur Costa Steiner To: Subject: Re: [[obm-l] ajuda-colmeia] X-Mailer: USANET web-mailer (CM.0402.5.2B) Mime-Version: 1.0 Message-ID: <790HDJTMM5824S02.1050003518@uwdvg002.cms.usa.net> Content-Type: text/plain; charset=ISO-8859-1 Content-Transfer-Encoding: 8bit X-MIME-Autoconverted: from quoted-printable to 8bit by sucuri.mat.puc-rio.br id QAA24768 Sender: owner-obm-l@sucuri.mat.puc-rio.br Precedence: bulk Reply-To: obm-l@mat.puc-rio.br > Uma colmeia nova tem 8000 abelhas. Destas, a cada dia que passa, morrem 200. > Do 21º dia em diante, nascem diariamente 2000 abelhas que vivem, em média, 40 > dias. Após um certo tempo, o número de abelhas dessa colmeia se estabilizará > em, aproximadamente quanto? Após 40 dias, todas as abelhas inicialmente existentes terão morrido. A partir daí, você a longo prazo chega num processo em que a cada dia nascem 2000 e morrem 2000 que completam 4o dias de idade. Logo, o total de abelhas se estabilizará no número total das que ainda não completaram 40 dias, ou seja 40 X 2000 = 80000. A menos que eu tenha entendido errado. Um abraço Artur ========================================================================= Instruções para entrar na lista, sair da lista e usar a lista em http://www.mat.puc-rio.br/~nicolau/olimp/obm-l.html O administrador desta lista é ========================================================================= From owner-obm-l@sucuri.mat.puc-rio.br Thu Apr 10 18:46:52 2003 Return-Path: Received: (from majordom@localhost) by sucuri.mat.puc-rio.br (8.9.3/8.9.3) id SAA27822 for obm-l-MTTP; Thu, 10 Apr 2003 18:43:44 -0300 Received: from paiol.terra.com.br (paiol.terra.com.br [200.176.3.18]) by sucuri.mat.puc-rio.br (8.9.3/8.9.3) with ESMTP id SAA27818 for ; Thu, 10 Apr 2003 18:43:40 -0300 Received: from altamira.terra.com.br (altamira.terra.com.br [200.176.3.40]) by paiol.terra.com.br (Postfix) with ESMTP id C7329883F5 for ; Thu, 10 Apr 2003 18:43:10 -0300 (BRT) Received: from [200.177.179.5] (dl-nas3-sao-C8B1B305.p001.terra.com.br [200.177.179.5]) by altamira.terra.com.br (Postfix) with ESMTP id 46B693DC0B8 for ; Thu, 10 Apr 2003 18:43:07 -0300 (BRT) User-Agent: Microsoft-Outlook-Express-Macintosh-Edition/5.02.2022 Date: Thu, 10 Apr 2003 18:41:14 -0300 Subject: [obm-l] FW: Teoria dos grupos From: Claudio Buffara To: Lista OBM Message-ID: In-Reply-To: Mime-version: 1.0 Content-type: text/plain; charset="US-ASCII" Content-transfer-encoding: 7bit Sender: owner-obm-l@sucuri.mat.puc-rio.br Precedence: bulk Reply-To: obm-l@mat.puc-rio.br Caros colegas da lista: Um problema de teoria dos grupos: Seja G um grupo cuja ordem eh diferente de 2. Seja a um elemento de G tal que f(a) = a para todo automorfismo f:G -> G. Prove que a = identidade de G. O resultado eh extremamente razoavel mas eu nao estou conseguindo prova-lo. Agradeco qualquer ajuda. Um abraco, Claudio. ========================================================================= Instruções para entrar na lista, sair da lista e usar a lista em http://www.mat.puc-rio.br/~nicolau/olimp/obm-l.html O administrador desta lista é ========================================================================= From owner-obm-l@sucuri.mat.puc-rio.br Thu Apr 10 20:31:18 2003 Return-Path: Received: (from majordom@localhost) by sucuri.mat.puc-rio.br (8.9.3/8.9.3) id UAA29271 for obm-l-MTTP; Thu, 10 Apr 2003 20:27:55 -0300 Received: from smtp015.mail.yahoo.com (smtp015.mail.yahoo.com [216.136.173.59]) by sucuri.mat.puc-rio.br (8.9.3/8.9.3) with SMTP id UAA29266 for ; Thu, 10 Apr 2003 20:27:51 -0300 Received: from unknown (HELO victorli) (victorluiz16@200.153.15.20 with login) by smtp.mail.vip.sc5.yahoo.com with SMTP; 10 Apr 2003 23:27:19 -0000 Message-ID: <005c01c2ffb9$9582b8c0$140f99c8@victorli> From: "Victor Luiz" To: Subject: [obm-l] OFF-TOPIC: Spam Date: Thu, 10 Apr 2003 20:32:45 -0300 MIME-Version: 1.0 Content-Type: text/plain; charset="iso-8859-1" Content-Transfer-Encoding: 8bit X-Priority: 3 X-MSMail-Priority: Normal X-Mailer: Microsoft Outlook Express 6.00.2800.1106 X-MimeOLE: Produced By Microsoft MimeOLE V6.00.2800.1106 Sender: owner-obm-l@sucuri.mat.puc-rio.br Precedence: bulk Reply-To: obm-l@mat.puc-rio.br Não pude deixar de notar que certo usuário cadastrado da lista têm enviado spams à lista... Bem que o moderador poderia alertar o cara ou em último caso tirar ele da lista já que segundo o majordom "As mensagens submetidas a esta lista devem ser: (a) sobre matemática ou sobre olimpíadas de matemática...". E aliás, já que toquei no assunto, gostaria de saber se existe restrição sobre a pessoa colocar uma URL na assinatura. Só não espero que isso também não vire debate, hehehe... Victor Luiz Salgado de Lima. ========================================================================= Instruções para entrar na lista, sair da lista e usar a lista em http://www.mat.puc-rio.br/~nicolau/olimp/obm-l.html O administrador desta lista é ========================================================================= From owner-obm-l@sucuri.mat.puc-rio.br Thu Apr 10 21:17:05 2003 Return-Path: Received: (from majordom@localhost) by sucuri.mat.puc-rio.br (8.9.3/8.9.3) id VAA30113 for obm-l-MTTP; Thu, 10 Apr 2003 21:14:22 -0300 Received: from silva5.uol.com.br (silva5.uol.com.br [200.221.29.52]) by sucuri.mat.puc-rio.br (8.9.3/8.9.3) with ESMTP id VAA30107 for ; Thu, 10 Apr 2003 21:14:13 -0300 Received: from giulio ([200.168.177.138]) by silva5.uol.com.br (8.9.1/8.9.1) with SMTP id VAA05705 for ; Thu, 10 Apr 2003 21:13:42 -0300 (BRT) Message-ID: <00f901c2ffbe$0eb97720$0200000a@giulio> From: "Fabiano" To: References: <002c01c2fea8$eddade60$eeaaa5c8@epq.ime.eb.br> Subject: Re: [obm-l] Fibonacci Date: Thu, 10 Apr 2003 21:05:27 -0300 MIME-Version: 1.0 Content-Type: text/plain; charset="iso-8859-1" Content-Transfer-Encoding: 7bit X-Priority: 3 X-MSMail-Priority: Normal X-Mailer: Microsoft Outlook Express 6.00.2800.1106 X-MimeOLE: Produced By Microsoft MimeOLE V6.00.2800.1106 Sender: owner-obm-l@sucuri.mat.puc-rio.br Precedence: bulk Reply-To: obm-l@mat.puc-rio.br me desculpem pela mensagem meio "off-topic", mas, oque seria esse "Eureka"? ----- Original Message ----- From: "Marcio" To: Sent: Wednesday, April 09, 2003 12:01 PM Subject: [obm-l] Fibonacci > PS: Eu iria mandar pra Eureka como proposto, mas achei universitario demais. > ========================================================================= Instruções para entrar na lista, sair da lista e usar a lista em http://www.mat.puc-rio.br/~nicolau/olimp/obm-l.html O administrador desta lista é ========================================================================= From owner-obm-l@sucuri.mat.puc-rio.br Thu Apr 10 22:08:23 2003 Return-Path: Received: (from majordom@localhost) by sucuri.mat.puc-rio.br (8.9.3/8.9.3) id WAA31318 for obm-l-MTTP; Thu, 10 Apr 2003 22:05:36 -0300 Received: from gorgo.centroin.com.br (gorgo.centroin.com.br [200.225.63.128]) by sucuri.mat.puc-rio.br (8.9.3/8.9.3) with ESMTP id WAA31314 for ; Thu, 10 Apr 2003 22:05:33 -0300 Received: from centroin.com.br (RJ118216.user.veloxzone.com.br [200.141.118.216] (may be forged)) (authenticated bits=0) by gorgo.centroin.com.br (8.12.9/8.12.9) with ESMTP id h3B14xBm003403 for ; Thu, 10 Apr 2003 22:05:00 -0300 (EST) Message-ID: <3E9614ED.1000905@centroin.com.br> Date: Thu, 10 Apr 2003 22:05:49 -0300 From: "A. C. Morgado" User-Agent: Mozilla/5.0 (Windows; U; Windows NT 5.0; en-US; rv:1.0.2) Gecko/20030208 Netscape/7.02 X-Accept-Language: en-us, en MIME-Version: 1.0 To: obm-l@mat.puc-rio.br Subject: Re: [obm-l] Fibonacci References: <002c01c2fea8$eddade60$eeaaa5c8@epq.ime.eb.br> <00f901c2ffbe$0eb97720$0200000a@giulio> Content-Type: multipart/alternative; boundary="------------030504030006060102010106" Sender: owner-obm-l@sucuri.mat.puc-rio.br Precedence: bulk Reply-To: obm-l@mat.puc-rio.br --------------030504030006060102010106 Content-Type: text/plain; charset=ISO-8859-1; format=flowed Content-Transfer-Encoding: 8bit Eureka é a revista da OBM. Está disponível em www.obm.org.br Fabiano wrote: >me desculpem pela mensagem meio "off-topic", mas, oque seria esse "Eureka"? >----- Original Message ----- >From: "Marcio" >To: >Sent: Wednesday, April 09, 2003 12:01 PM >Subject: [obm-l] Fibonacci > > > > >>PS: Eu iria mandar pra Eureka como proposto, mas achei universitario >> >> >demais. > > > >========================================================================= >Instruções para entrar na lista, sair da lista e usar a lista em >http://www.mat.puc-rio.br/~nicolau/olimp/obm-l.html >O administrador desta lista é >========================================================================= > > > > --------------030504030006060102010106 Content-Type: text/html; charset=us-ascii Content-Transfer-Encoding: 7bit Eureka é a revista da OBM. Está disponível em www.obm.org.br

Fabiano wrote:
me desculpem pela mensagem meio "off-topic", mas, oque seria esse "Eureka"?
----- Original Message -----
From: "Marcio" <marciocohen@superig.com.br>
To: <obm-l@mat.puc-rio.br>
Sent: Wednesday, April 09, 2003 12:01 PM
Subject: [obm-l] Fibonacci


  
PS: Eu iria mandar pra Eureka como proposto, mas achei universitario
    
demais.
  

=========================================================================
Instruções para entrar na lista, sair da lista e usar a lista em
http://www.mat.puc-rio.br/~nicolau/olimp/obm-l.html
O administrador desta lista é <nicolau@mat.puc-rio.br>
=========================================================================


  

--------------030504030006060102010106-- ========================================================================= Instruções para entrar na lista, sair da lista e usar a lista em http://www.mat.puc-rio.br/~nicolau/olimp/obm-l.html O administrador desta lista é ========================================================================= From owner-obm-l@sucuri.mat.puc-rio.br Thu Apr 10 23:59:40 2003 Return-Path: Received: (from majordom@localhost) by sucuri.mat.puc-rio.br (8.9.3/8.9.3) id XAA00752 for obm-l-MTTP; Thu, 10 Apr 2003 23:56:57 -0300 Received: from acsxe0.ac.brahma ([200.212.186.163]) by sucuri.mat.puc-rio.br (8.9.3/8.9.3) with ESMTP id XAA00748 for ; Thu, 10 Apr 2003 23:56:52 -0300 Received: from acsnx2.ac.brahma (acsnx2 [10.100.3.89]) by acsxe0.ac.brahma (AIX4.3/8.9.3/8.9.3) with ESMTP id XAA82696 for ; Thu, 10 Apr 2003 23:56:07 -0300 Received: by acsnx2.brahma with Internet Mail Service (5.5.2653.19) id <2V2ASTS2>; Thu, 10 Apr 2003 23:55:47 -0300 Message-ID: <27BD56F8640DD711A4320006295078E50500B7@pssnx1.brahma> From: "Ricardo de Moraes (PS)" To: "'obm-l@mat.puc-rio.br'" Subject: [obm-l] Date: Thu, 10 Apr 2003 23:55:40 -0300 MIME-Version: 1.0 X-Mailer: Internet Mail Service (5.5.2653.19) Content-Type: multipart/alternative; boundary="----_=_NextPart_001_01C2FFD5.D5DFDC10" Sender: owner-obm-l@sucuri.mat.puc-rio.br Precedence: bulk Reply-To: obm-l@mat.puc-rio.br This message is in MIME format. Since your mail reader does not understand this format, some or all of this message may not be legible. ------_=_NextPart_001_01C2FFD5.D5DFDC10 Content-Type: text/plain; charset="ISO-8859-1" Content-Transfer-Encoding: quoted-printable Boa noite, Estou com dificuldades em resolver alguns problemas, at=E9 que resolvi = alguns deles mas fiquei com duvidas quanto =E0s respostas. Se algu=E9m puder me ajudar...=20 =20 =20 1. De quantos modos podemos distribuir 15 balas entre 5 crian=E7as: a) garantindo que cada crian=E7a receba pelo menos uma bala; b) garantindo que cada crian=E7a receba pelo menos duas balas. =20 2. Um campeonato =E9 disputado por 12 clubes em rodadas de 6 jogos = cada. De quantos modos =E9 poss=EDvel selecionar os jogos da primeira rodada. =20 3. Delegados de 10 pa=EDses diferentes devem se sentar em 10 cadeiras = em fila. De quantos modos isso pode ser feito, se os delegados do Brasil e de Portugal devem sentar juntos e os da Argentina e do Paraguai n=E3o = podem sentar jntos? =20 4. lan=E7am-se 3 dados. Em quantos dos poss=EDveis resultados a soma = dos valores das faces =E9 10? =20 5. Determine o n=FAmero de anagramas da palavra ESTUDANTE que tem E no primeiro lugar ou S no segundo lugar ou T no terceiro lugar? =20 6. Em um torneio no qual cada participante enfrenta todos os demais, = s=E3o disputadas 780 partidas no total. Quantos s=E3o os participantes deste torneio? =20 7. De quantos modos diferentes podem ser escolhidos um presidente e um secret=E1rio de um conselho que tem 12 membros? =20 8. O c=F3digo Morse usa "palavras" contendo de 1 a 4 "letras". As = "letras" s=E3o ponto e tra=E7o. Quantas "palavras" existe no c=F3digo Morse? =20 =20 =20 Ricardo. =20 ------_=_NextPart_001_01C2FFD5.D5DFDC10 Content-Type: text/html; charset="ISO-8859-1" Content-Transfer-Encoding: quoted-printable
Boa=20 noite,
Estou = com=20 dificuldades em resolver alguns problemas, at=E9 que resolvi alguns = deles mas=20 fiquei com duvidas quanto =E0s respostas.
Se = algu=E9m puder me=20 ajudar...
 
 
1. De = quantos modos=20 podemos distribuir 15 balas entre 5 crian=E7as:
a) = garantindo que=20 cada crian=E7a receba pelo menos uma bala;
b) = garantindo que=20 cada crian=E7a receba pelo menos duas balas.
 
2. Um = campeonato =E9=20 disputado por 12 clubes em rodadas de 6 jogos cada. De quantos modos = =E9 poss=EDvel=20 selecionar os jogos da primeira rodada.
 
3. = Delegados de 10=20 pa=EDses diferentes devem se sentar em 10 cadeiras em fila. De quantos = modos isso=20 pode ser feito, se os delegados do Brasil e de Portugal devem sentar = juntos e os=20 da Argentina e do Paraguai n=E3o podem sentar = jntos?
 
4. = lan=E7am-se 3=20 dados. Em quantos dos poss=EDveis resultados a soma dos valores das = faces =E9=20 10?
 
5. = Determine o=20 n=FAmero de anagramas da palavra ESTUDANTE que tem E no primeiro lugar = ou S no=20 segundo lugar ou T no terceiro lugar?
 
6. Em = um torneio no=20 qual cada participante enfrenta todos os demais, s=E3o disputadas 780 = partidas no=20 total. Quantos s=E3o os participantes deste = torneio?
 
7. De = quantos modos=20 diferentes podem ser escolhidos um presidente e um secret=E1rio de um = conselho que=20 tem 12 membros?
 
8. O = c=F3digo Morse=20 usa "palavras" contendo de 1 a 4 "letras". As "letras" s=E3o ponto e = tra=E7o.=20 Quantas "palavras" existe no c=F3digo Morse?
 
 
 
Ricardo.
 
------_=_NextPart_001_01C2FFD5.D5DFDC10-- ========================================================================= Instruções para entrar na lista, sair da lista e usar a lista em http://www.mat.puc-rio.br/~nicolau/olimp/obm-l.html O administrador desta lista é ========================================================================= From owner-obm-l@sucuri.mat.puc-rio.br Thu Apr 10 23:59:40 2003 Return-Path: Received: (from majordom@localhost) by sucuri.mat.puc-rio.br (8.9.3/8.9.3) id XAA00744 for obm-l-MTTP; Thu, 10 Apr 2003 23:56:35 -0300 Received: from web21413.mail.yahoo.com (web21413.mail.yahoo.com [216.136.232.184]) by sucuri.mat.puc-rio.br (8.9.3/8.9.3) with SMTP id XAA00730 for ; Thu, 10 Apr 2003 23:56:29 -0300 Message-ID: <20030411025558.64194.qmail@web21413.mail.yahoo.com> Received: from [200.221.52.211] by web21413.mail.yahoo.com via HTTP; Thu, 10 Apr 2003 19:55:58 PDT Date: Thu, 10 Apr 2003 19:55:58 -0700 (PDT) From: Danilo Pinseta Subject: [obm-l] Polinômios e Grupo Abeliano To: "Nicolau C. Saldanha" Cc: obm-l@sucuri.mat.puc-rio.br In-Reply-To: <20030410170717.B25350@sucuri.mat.puc-rio.br> MIME-Version: 1.0 Content-Type: text/plain; charset=us-ascii Sender: owner-obm-l@sucuri.mat.puc-rio.br Precedence: bulk Reply-To: obm-l@mat.puc-rio.br Gostaria de receber demonstrações de que polinômios de grau <= n) são grupo abeliano para adição. Obrigado DANILO __________________________________________________ Do you Yahoo!? Yahoo! Tax Center - File online, calculators, forms, and more http://tax.yahoo.com ========================================================================= Instruções para entrar na lista, sair da lista e usar a lista em http://www.mat.puc-rio.br/~nicolau/olimp/obm-l.html O administrador desta lista é ========================================================================= From owner-obm-l@sucuri.mat.puc-rio.br Fri Apr 11 00:33:08 2003 Return-Path: Received: (from majordom@localhost) by sucuri.mat.puc-rio.br (8.9.3/8.9.3) id AAA01522 for obm-l-MTTP; Fri, 11 Apr 2003 00:30:28 -0300 Received: from toole.uol.com.br (toole.uol.com.br [200.221.29.26]) by sucuri.mat.puc-rio.br (8.9.3/8.9.3) with ESMTP id AAA01496 for ; Fri, 11 Apr 2003 00:30:07 -0300 Received: from afonso ([200.191.190.39]) by toole.uol.com.br (8.9.1/8.9.1) with SMTP id AAA05462 for ; Fri, 11 Apr 2003 00:29:32 -0300 (BRT) MIME-Version: 1.0 Message-Id: <3E963254.000020.24597@afonso> Date: Fri, 11 Apr 2003 00:11:16 -0300 (Hora padrão leste da Am. Sul) Content-Type: Multipart/related; type="multipart/alternative"; boundary="------------Boundary-00=_SUS5FQR2QL8000000000" X-Mailer: IncrediMail 2001 (1850931) From: "Guilherme Pimentel" References: <20030409135532.A17897@sucuri.mat.puc-rio.br> X-FID: FLAVOR00-NONE-0000-0000-000000000000 X-CNT: ; X-Priority: 3 To: Subject: [obm-l] =?iso-8859-1?B?UmVzOiBbb2JtLWxdIFJlOiBbb2JtLWxdIE9waW5p428=?= Sender: owner-obm-l@sucuri.mat.puc-rio.br Precedence: bulk Reply-To: obm-l@mat.puc-rio.br --------------Boundary-00=_SUS5FQR2QL8000000000 Content-Type: Multipart/Alternative; boundary="------------Boundary-00=_SUS5A0V2QL8000000000" --------------Boundary-00=_SUS5A0V2QL8000000000 Content-Type: Text/Plain; charset="Windows-1252" Content-Transfer-Encoding: quoted-printable Concordo com o Nicolau....=0D Afinal foi la que eu estudei :-)=0D =0D Guilherme Pimentel =0D =0D -------Mensagem original-------=0D =0D De: obm-l@mat.puc-rio.br=0D Data: quarta-feira, 09 de abril de 2003 14:12:25=0D Para: obm-l@mat.puc-rio.br=0D Assunto: [obm-l] Re: [obm-l] Opini=E3o=0D =0D On Wed, Apr 09, 2003 at 12:13:14AM -0300, Raul wrote:=0D > Bom dia a todos. Um aluno me fez uma pergunta que gostaria de saber a=0D > opini=E3o da lista para que eu n=E3o d=EA a ele uma resposta parcial. A= =0D > pergunta =E9 : quais s=E3o as melhores faculdades de Matem=E1tica Pura = e de=0D > Matem=E1tica Aplicada por aqui (Rio-S=E3o Paulo)? Gostaria de saber o q= ue=0D > vcs pensam e j=E1 me desculpo se a pergunta for fora do prop=F3sito da = lista.=0D > Obrigado, Raul=0D =0D Eu sou professor da PUC-Rio, ent=E3o a resposta que eu vou dar pode=0D ser justa mas n=E3o =E9 isenta. Mas sem falsa mod=E9stia acho que a gradu= a=E7=E3o=0D da PUC-Rio =E9 claramente a melhor do Rio e uma das melhores do Brasil.=0D Ficamos v=E1rias vezes em primeiro lugar nacional no prov=E3o e v=E1rias=0D outras avalia=E7=F5es tamb=E9m nos colocam em =F3tima posi=E7=E3o. Vale a= pena chamar=0D a aten=E7=E3o para o fato de estarmos falando de turmas pequenas=0D (talvez 5 alunos por ano) o que obviamente significa que cada aluno=0D recebe muito mais aten=E7=E3o dos professores do que em outros lugares=0D onde h=E1 turmas maiores. =0D =0D A melhor gradua=E7=E3o da cidade de S=E3o Paulo =E9 a da USP e a da Unica= mp=0D tamb=E9m =E9 muito boa. O Impa tamb=E9m =E9 uma institui=E7=E3o excelente= mas=0D n=E3o tem gradua=E7=E3o.=0D =0D []s, N.=0D =0D =0D =3D=3D=3D=3D=3D=3D=3D=3D=3D=3D=3D=3D=3D=3D=3D=3D=3D=3D=3D=3D=3D=3D=3D=3D=3D= =3D=3D=3D=3D=3D=3D=3D=3D=3D=3D=3D=3D=3D=3D=3D=3D=3D=3D=3D=3D=3D=3D=3D=3D=3D= =3D=3D=3D=3D=3D=3D=3D=3D=3D=3D=3D=3D=3D=3D=3D=3D=3D=3D=3D=3D=3D=3D=3D=0D Instru=E7=F5es para entrar na lista, sair da lista e usar a lista em=0D http://www.mat.puc-rio.br/~nicolau/olimp/obm-l.html=0D O administrador desta lista =E9 =0D =3D=3D=3D=3D=3D=3D=3D=3D=3D=3D=3D=3D=3D=3D=3D=3D=3D=3D=3D=3D=3D=3D=3D=3D=3D= =3D=3D=3D=3D=3D=3D=3D=3D=3D=3D=3D=3D=3D=3D=3D=3D=3D=3D=3D=3D=3D=3D=3D=3D=3D= =3D=3D=3D=3D=3D=3D=3D=3D=3D=3D=3D=3D=3D=3D=3D=3D=3D=3D=3D=3D=3D=3D=3D=0D =2E=20 --------------Boundary-00=_SUS5A0V2QL8000000000 Content-Type: Text/HTML; charset="Windows-1252" Content-Transfer-Encoding: quoted-printable
Concordo com o Nicolau....
Afinal foi la que eu estudei :-)
 
Guilherme Pimentel 
 
-------Mensagem original-------
 
Data: quarta-feira= , 09 de abril de 2003 14:12:25
Assunto: [obm-l] R= e: [obm-l] Opini=E3o
 
On Wed, Apr 09, 2003 at 12:13:14AM -0300, Raul wrote:> Bom dia a todos. Um aluno me fez uma pergunta que gostaria de saber= a
> opini=E3o da lista para que eu n=E3o d=EA a ele uma resposta p= arcial. A
> pergunta =E9 : quais s=E3o as melhores faculdades de Ma= tem=E1tica Pura e de
> Matem=E1tica Aplicada por aqui (Rio-S=E3o Pa= ulo)? Gostaria de saber o que
> vcs pensam e j=E1 me desculpo se a = pergunta for fora do prop=F3sito da lista.
> Obrigado, Raul

= Eu sou professor da PUC-Rio, ent=E3o a resposta que eu vou dar pode
se= r justa mas n=E3o =E9 isenta. Mas sem falsa mod=E9stia acho que a gradua=E7= =E3o
da PUC-Rio =E9 claramente a melhor do Rio e uma das melhores do B= rasil.
Ficamos v=E1rias vezes em primeiro lugar nacional no prov=E3o e= v=E1rias
outras avalia=E7=F5es tamb=E9m nos colocam em =F3tima posi=E7= =E3o. Vale a pena chamar
a aten=E7=E3o para o fato de estarmos falando= de turmas pequenas
(talvez 5 alunos por ano) o que obviamente signifi= ca que cada aluno
recebe muito mais aten=E7=E3o dos professores do que= em outros lugares
onde h=E1 turmas maiores.

A melhor gradua=E7= =E3o da cidade de S=E3o Paulo =E9 a da USP e a da Unicamp
tamb=E9m =E9= muito boa. O Impa tamb=E9m =E9 uma institui=E7=E3o excelente mas
n=E3= o tem gradua=E7=E3o.

[]s, N.


=3D=3D=3D=3D=3D=3D=3D=3D=3D= =3D=3D=3D=3D=3D=3D=3D=3D=3D=3D=3D=3D=3D=3D=3D=3D=3D=3D=3D=3D=3D=3D=3D=3D=3D= =3D=3D=3D=3D=3D=3D=3D=3D=3D=3D=3D=3D=3D=3D=3D=3D=3D=3D=3D=3D=3D=3D=3D=3D=3D= =3D=3D=3D=3D=3D=3D=3D=3D=3D=3D=3D=3D=3D=3D
Instru=E7=F5es para entrar = na lista, sair da lista e usar a lista em
http://www.mat.puc-rio.br/~nicolau/ol= imp/obm-l.html
O administrador desta lista =E9 <nicolau@mat.puc-rio.br>
=3D=3D=3D=3D= =3D=3D=3D=3D=3D=3D=3D=3D=3D=3D=3D=3D=3D=3D=3D=3D=3D=3D=3D=3D=3D=3D=3D=3D=3D= =3D=3D=3D=3D=3D=3D=3D=3D=3D=3D=3D=3D=3D=3D=3D=3D=3D=3D=3D=3D=3D=3D=3D=3D=3D= =3D=3D=3D=3D=3D=3D=3D=3D=3D=3D=3D=3D=3D=3D=3D=3D=3D=3D=3D
.
______________________= ______________________________
<= A href=3D"http://www.incredimail.com/redir.asp?ad_id=3D322&lang=3D22"= >3D""  IncrediMail - O mundo = do correio eletr=F4nico finalmente desenvolveu-se - Clique aqui
--------------Boundary-00=_SUS5A0V2QL8000000000-- --------------Boundary-00=_SUS5FQR2QL8000000000 Content-Type: unknown/unknown; name="IMSTP.gif" Content-Transfer-Encoding: base64 Content-ID: <30E042F9-6BA8-11D7-8958-444553540000> R0lGODlhFAAPALMIAP9gAM9gAM8vAM9gL/+QL5AvAGAvAP9gL////wAAAAAAAAAAAAAAAAAAAAAA AAAAACH/C05FVFNDQVBFMi4wAwEAAAAh+QQJFAAIACwAAAAAFAAPAAAEVRDJSaudJuudrxlEKI6B URlCUYyjKpgYAKSgOBSCDEuGDKgrAtC3Q/R+hkPJEDgYCjpKr5A8WK9OaPFZwHoPqm3366VKyeRt E30tVVRscMHDqV/u+AgAIfkEBWQACAAsAAAAABQADwAABBIQyUmrvTjrzbv/YCiOZGmeaAQAIfkE CRQACAAsAgABABAADQAABEoQIUOrpXIOwrsPxiQUheeRAgUA49YNhbCqK1kS9grQhXGAhsDBUJgZ AL2Dcqkk7ogFpvRAokSn0p4PO6UIuUsQggSmFjKXdAgRAQAh+QQFCgAIACwAAAAAFAAPAAAEEhDJ Sau9OOvNu/9gKI5kaZ5oBAAh+QQJFAAIACwCAAEAEAANAAAEShAhQ6ulcg7Cuw/GJBSF55ECBQDj 1g2FsKorWRL2CtCFcYCGwMFQmBkAvYNyqSTuiAWm9ECiRKfSng87pQi5SxCCBKYWMpd0CBEBACH5 BAVkAAgALAAAAAAUAA8AAAQSEMlJq7046827/2AojmRpnmgEADs= --------------Boundary-00=_SUS5FQR2QL8000000000-- ========================================================================= Instruções para entrar na lista, sair da lista e usar a lista em http://www.mat.puc-rio.br/~nicolau/olimp/obm-l.html O administrador desta lista é ========================================================================= From owner-obm-l@sucuri.mat.puc-rio.br Fri Apr 11 01:42:07 2003 Return-Path: Received: (from majordom@localhost) by sucuri.mat.puc-rio.br (8.9.3/8.9.3) id BAA03006 for obm-l-MTTP; Fri, 11 Apr 2003 01:39:18 -0300 Received: from mail.gmx.net (mail.gmx.net [213.165.64.20]) by sucuri.mat.puc-rio.br (8.9.3/8.9.3) with SMTP id BAA02994 for ; Fri, 11 Apr 2003 01:39:11 -0300 Received: (qmail 11720 invoked by uid 65534); 11 Apr 2003 04:38:37 -0000 Received: from unknown (EHLO localhost) (200.217.15.35) by mail.gmx.net (mp022-rz3) with SMTP; 11 Apr 2003 06:38:37 +0200 Date: Fri, 11 Apr 2003 01:38:05 -0300 From: Igor GomeZZ X-Mailer: The Bat! (v1.61) Organization: -- X-Priority: 3 (Normal) Message-ID: <6732305773.20030411013805@gmx.net> To: OBM Subject: [obm-l] =?ISO-8859-1?B?VHJpZ29ub21ldHJpYSBlIFNlcXXqbmNpYXM=?= MIME-Version: 1.0 Content-Type: text/plain; charset=ISO-8859-1 Content-Transfer-Encoding: 8bit Sender: owner-obm-l@sucuri.mat.puc-rio.br Precedence: bulk Reply-To: obm-l@mat.puc-rio.br Fala galera da lista, boa noite... São dois problemas, um OBM e o outro não sei a fonte: **Sequência: 1*2 + 2*3 + 3*4 + 4*5 +...+47*48 + 48*49 + 49*50 Se não me engano ela eh OBM, certo? Consegui resolvê-la como uma Progressão Aritmética de segunda ordem. Para achar o polinômio que gera os termos t(n) eh relativamente demorado, jah para achar o polinômio que define a soma s(n) eh ainda mais demorado. Tem alguma coisa na cara que facilite a questão e não estou vendo? ** Trigonometria: cosa * cos2a * cos4a * cos8a * ... * cos[(2^(n-1))a] , ou seja, Produtório(cos((2^(n-1)a, n=1..(n-1)) Tentei levar para uma resolução com complexos (potências), mas sem sucesso... Alguma dica? Fui! ####### Igor GomeZZ ######## UIN: 29249895 Vitória, Espírito Santo, Brasil Criação: 11/4/2003 (01:27) #################################### Pare para pensar: Preocupe-se mais com seu caráter do que com sua reputação, porque seu caráter é o que você realmente é, enquanto a reputação é apenas o que os outros pensam que você é. (Henfil) #################################### ========================================================================= Instruções para entrar na lista, sair da lista e usar a lista em http://www.mat.puc-rio.br/~nicolau/olimp/obm-l.html O administrador desta lista é ========================================================================= From owner-obm-l@sucuri.mat.puc-rio.br Fri Apr 11 02:24:22 2003 Return-Path: Received: (from majordom@localhost) by sucuri.mat.puc-rio.br (8.9.3/8.9.3) id CAA03600 for obm-l-MTTP; Fri, 11 Apr 2003 02:21:19 -0300 Received: from Euler.impa.br (euler.impa.br [147.65.1.3]) by sucuri.mat.puc-rio.br (8.9.3/8.9.3) with ESMTP id CAA03586 for ; Fri, 11 Apr 2003 02:21:13 -0300 Received: from Gauss.impa.br (Gauss [147.65.4.1]) by Euler.impa.br (8.11.6p2/8.11.6) with ESMTP id h3B5Ke002877 for ; Fri, 11 Apr 2003 02:20:40 -0300 (EST) From: Carlos Gustavo Tamm de Araujo Moreira Received: by Gauss.impa.br (8.11.6p2) id h3B5Ka922795; Fri, 11 Apr 2003 02:20:36 -0300 (EST) Message-Id: <200304110520.h3B5Ka922795@Gauss.impa.br> Subject: Re: [obm-l] Mais Probls em Aberto II To: obm-l@mat.puc-rio.br Date: Fri, 11 Apr 2003 02:20:36 -0300 (EST) In-Reply-To: <03f001c2f7b4$aa8349e0$3300c57d@bovespa.com> from "=?Windows-1252?Q?Cl=E1udio_\=28Pr=E1tica\=29?=" at Mar 31, 3 03:38:03 pm X-Mailer: ELM [version 2.4 PL25] MIME-Version: 1.0 Content-Type: text/plain; charset=US-ASCII Content-Transfer-Encoding: 7bit Sender: owner-obm-l@sucuri.mat.puc-rio.br Precedence: bulk Reply-To: obm-l@mat.puc-rio.br > >11) Seja a,b,c,d 4 n=FAmeros reais n=E3o negativos que=20 >satisfazem a condi=E7=E3o=20 >2*(ab+ac+ad+bc+bd+cd)+abc+abd+acd+bcd=3D16. >Prove que a+b+c+d>=3D2/3*(ab+ac+ad+bc+bd+cd) e determine o=20 >caso de igualdade. > O enunciado implica que o polinomio P(x)=(x-a)(x-b)(x-c)(x-d) tem quatro raizes nao negativas, e portanto sua derivada tambem (pelo teorema do valor medio). Seja entao P'(x)=4(x-u)(x-v)(x-w).Temos 3(a+b+c+d)=4(u+v+w), 2(ab+ac+ad+bc+bd+cd)=4(uv+uw+vw) e abc+abd+acd+bcd=4uvw. Assim, a condicao 2(ab+ac+ad+bc+bd+cd)+abc+abd+acd+bcd=16 equivale a uv+uw+vw+uvw=4. A conclusao equivale a u+v+w>=uv+uw+vw. Multiplicando por u+v+uv e usando w(u+v+uv)=4-uv, temos que a conclusao segue de (u+v)(u+v+uv)+4-uv>=uv(u+v+uv)+(u+v)(4-uv) (note que se u+v+uv=0 entao u=v=0, e nao poderiamos ter uv+uw+vw+uvw=4). Essa desigualdade pode ser escrita como (u+v-2)^2-uv(uv-u-v+1)>=0, ou seja, ((u-1)+(v-1))^2-uv(u-1)(v-1)>=0, e essa ultima desigualdade segue de 0<=uv=4-(uw+vw+uvw)<=4. So' podemos ter igualdade se u=v=1 (quando w tambem e' 1) ou quando uv=0, u+v=2, caso em que w=2, ou quando uv=4 e u=v, caso em que u=v=2 e w=0. Os ultimos casos nao sao possiveis (senao P'(x)=4x(x-2)^2>=0 para x>=0, donde P(x) seria crescente para x>=0, e logo, como abc+abd+acd+bcd=4uvw=0, todas as raizes de P deveriam ser iguais a 0, donde u=v=w=0, absurdo), e no primeiro devemos ter a=b=c=d=1 (pois (a-1)^2+(b-1)^2+(c-1)^2+(d-1)^2=(a+b+c+d)^2-4(ab+ac+ad+bc+bd+cd)+4= =(4(u+v+w)/3)^2-8(uv+uw+vw)+4=4^2-8.3+4=0), que e' assim o unico caso de igualdade. >***** > >13) Seja a,b e c medidas dos lados de um tri=E2ngulo.=20 >Prove que: raiz(a+b-c)+raiz(b+c-a)+raiz(c+a-b)=3D+raiz(b)+raiz(c) > Sejam a+b-c=x, a+c-b=y e b+c-a=z. O problema equivale a mostrar que raiz(2x)+raiz(2y)+raiz(2z)<=raiz(x+y)+raiz(x+z)+raiz(y+z), mas isso segue de raiz(2x)+raiz(2y)<=2.raiz(x+y), raiz(2x)+raiz(2z)<=2.raiz(x+z) e raiz(2y)+raiz(2z)<=2.raiz(y+z), que sao equivalentes. A primeira segue elevando ao quadrado: ela fica equivalente a 2x+2y+4.raiz(xy)<=4(x+y), ou a x-2.raiz(xy)+y=(raiz(x)-raiz(y))^2>=0, que e' obvio. So vale a igualdade se x=y=z, ou seja, se a=b=c. >***** > >14)Demonstrar que para quaisquer valores real e x, y=20 >e z =E9 v=E1lida a desigualdade >4x(x+y)(x+z)(x+y+z)+y=B2z=B2>=3D0 > Escrevendo 4x(x+y)(x+z)(x+y+z)+y^2.z^2 como um polinomio do segundo grau em y, obtemos (z^2+4x(x+z))y^2+(4x(x+z)^2+4x^2.(x+z))y+4x^2.(x+z)^2= =(z+2x)^2.y^2+4x(x+z)(z+2x)^2+4(x(x+z))^2= ((z+2x)y+2x(x+z))^2 >=0. >***** > >15) Se a^(b^c) =3D b^d , c/d pode ser dado em fun=E7=E3o de a e b ? > Nao. Se a^(b^c)=b^d entao a^(b^(c+1))=b^(bd) e a^(b^(c+2))=b^(b^2.d). Se c/d so' dependesse de a e de b teriamos c/d=(c+1)/bd=(c+2)/(b^2.d), donde bc=c+1 e b^2.c=c+2, donde 2=c(b^2-1)=c(b-1)(b+1)=b+1, e logo b=1 e c=c+1, absurdo. >***** > >16) Seja a fun=E7ao f:N*U{0} ->N*U{0} dada pelas = >propriedades:(f(2n+1))=B2-(f(2n))=B2=3D6f(n)+1 e f(2n)>=3Df(n) para todo = >n natural.Ache #{x elemento de N,f(x)<2003}.=20 >(A solu=E7=E3o desse vale um doce - cortesia do Dirichlet!) > Vou mostrar por inducao que se n=soma(j=0 ate' k)(s_j.2^j), com s_j em {0,1} e' a representacao binaria de n entao f(n)=soma(j=0 ate' k)(s_j.3^j). De fato, de f(1)^2-f(0)^2=6f(0)+1 segue (f(0)+3)^2-f(1)^2=8, donde f(0)+3+f(1)=4 e f(0)+3-f(1)=2 (pois f(0)+3 e f(1) sao naturais de mesma paridade), e logo f(1)=1 e f(0)=0. Temos (f(2n+1)+f(2n))(f(2n+1)-f(2n))=6f(n)+1. Assim f(2n+1)>f(2n)>=f(n), donde f(2n+1)+f(2n)>=2f(n)+1 e e' um divisor de 6f(n)+1. Como 6f(n)+1 e' impar e (6f(n)+1)/(2f(n)+1)<3, segue que f(2n+1)+f(2n)=6f(n)+1 e f(2n+1)-f(2n)=1, e logo f(2n+1)=3f(n)+1 e f(2n)=3f(n), o que implica nossa afirmacao para 2n e 2n+1, supondo que ela vale para n. Agora, como f(x) e' uma soma de potencias de 3 distintas, se f(x) < 2003 , como 3^6+3^5+3^4+3^3+3^2+3^1+3^0 < 2003 < 3^7, x deve ser uma soma qualquer de potencias de 2 distintas coem expoentes menores que 7, ou seja, x pode ser qualquer numero menor que 2^7=128. Assim, {x natural,f(x)<2003}={0,1,2,...,127}, donde #{x natural,f(x)<2003}=128. Cade meu doce, Dirichlet ? >***** Abracos, Gugu ========================================================================= Instruções para entrar na lista, sair da lista e usar a lista em http://www.mat.puc-rio.br/~nicolau/olimp/obm-l.html O administrador desta lista é ========================================================================= From owner-obm-l@sucuri.mat.puc-rio.br Fri Apr 11 02:36:10 2003 Return-Path: Received: (from majordom@localhost) by sucuri.mat.puc-rio.br (8.9.3/8.9.3) id CAA03766 for obm-l-MTTP; Fri, 11 Apr 2003 02:33:34 -0300 Received: from Euler.impa.br (euler.impa.br [147.65.1.3]) by sucuri.mat.puc-rio.br (8.9.3/8.9.3) with ESMTP id CAA03762 for ; Fri, 11 Apr 2003 02:33:25 -0300 Received: from Gauss.impa.br (Gauss [147.65.4.1]) by Euler.impa.br (8.11.6p2/8.11.6) with ESMTP id h3B5Wt003523 for ; Fri, 11 Apr 2003 02:32:55 -0300 (EST) From: Carlos Gustavo Tamm de Araujo Moreira Received: by Gauss.impa.br (8.11.6p2) id h3B5WqC23661; Fri, 11 Apr 2003 02:32:52 -0300 (EST) Message-Id: <200304110532.h3B5WqC23661@Gauss.impa.br> Subject: Re: [obm-l] Problemas em Aberto To: obm-l@mat.puc-rio.br Date: Fri, 11 Apr 2003 02:32:52 -0300 (EST) In-Reply-To: <044b01c2de8a$b27413c0$3300c57d@bovespa.com> from "=?Windows-1252?Q?Cl=E1udio_\=28Pr=E1tica\=29?=" at Feb 27, 3 03:04:34 pm X-Mailer: ELM [version 2.4 PL25] MIME-Version: 1.0 Content-Type: text/plain; charset=US-ASCII Content-Transfer-Encoding: 7bit Sender: owner-obm-l@sucuri.mat.puc-rio.br Precedence: bulk Reply-To: obm-l@mat.puc-rio.br > >HelpCaros colegas da lista: > >Muitas vezes um problema =E9 proposto na lista, nenhuma solu=E7=E3o =E9 = >dada nos dias seguintes e logo o problema cai no esquecimento. Assim, = >resolvi fazer uma compila=E7=E3o (temo que incompleta) daqueles = >problemas da lista que ficaram sem solu=E7=E3o. > >1. Seja=20 >A =3D | A1 | > | A2 | >uma matriz m x n com A1 n x n n=E3o singular e A2 uma matriz (m-n) x n = >arbitr=E1ria > >A+ =E9 a pseudo-inversa de A, definida como=20 >A+ =3D (A' * A)^(-1) * A' > >prove que ||A+|| <=3D ||(A1)^(-1)|| =20 > >OBS: A norma aqui =E9 induzida: > ||A|| =3D sup ||Ax|| > ||x|| =3D 1 > Temos que A+(v) e', dentre os vetores u com |Au-v| minimo o que tem a menor norma. Assim, se v=Au, temos |A+(v)|<=|u|. Assim, A+ se anula no complemento ortogonal da imagem de A, logo a norma de A+ e' igual a norma de A+ restrita a imagem de A. Por outro lado,temos que |A1(u)|<=|Au| para todo u, donde ||A+|| <= max |u|/|Au| <= max |u|/|A1(u)| = ||(A1)^(-1)||. >********* > >2. =C9 poss=EDvel que um polin=F4mio de coeficientes inteiros P(X) = >irredut=EDvel se fatore em Z/(n) para todo n natural ? > > Sim. Por exemplo:P(x)=x^4-38.x^2+225= =(x-(raiz(2)+raiz(17))(x-(-raiz(2)+raiz(17))(x-(raiz(2)-raiz(17))(x-(-raiz(2)-raiz(17)). De fato, para qualquer q potencia de primo, 2 ou 17 ou 34 e' um quadrado modulo q (prove primeiro para q primo e depois use inducao no expoente; para q potencia de 2 use que 17 e' quadrado modulo 8). Se 2=a^2 (mod q), entao P(x)=(x^2-2ax-15)(x^2+2ax-15) (mod q). Se 17=b^2 (mod q), entao P(x)=(x^2-2bx+15)(x^2+2bx+15) (mod q), e se 34=c^2 (mod q), entao P(x)=(x^2-(19+2c))(x^2-(19-2c)) (mod q). Agora, dado n, escrevemos n como produto de potencias de primos distintos, e usamos o fato de que modulo qualquer dessas potencias de primo pudemos fatorar P(x) como produto de dois polinomios do segundo grau com coeficientes inteiros junto com o teorema chines dos restos para mostrar que P(x) pode ser escrito modulo n como o produto de dois polinomios do segundo grau com coeficientes inteiros. Um problema relacionado que eu nao sei resolver e' o seguinte: e' possivel que um polinomio irredutivel de coeficientes inteiros tenha raiz modulo p para todo primo p ? >********* Abracos, Gugu ========================================================================= Instruções para entrar na lista, sair da lista e usar a lista em http://www.mat.puc-rio.br/~nicolau/olimp/obm-l.html O administrador desta lista é ========================================================================= From owner-obm-l@sucuri.mat.puc-rio.br Fri Apr 11 06:37:52 2003 Return-Path: Received: (from majordom@localhost) by sucuri.mat.puc-rio.br (8.9.3/8.9.3) id GAA07927 for obm-l-MTTP; Fri, 11 Apr 2003 06:35:02 -0300 Received: from traven9.uol.com.br (traven9.uol.com.br [200.221.29.35]) by sucuri.mat.puc-rio.br (8.9.3/8.9.3) with ESMTP id GAA07923 for ; Fri, 11 Apr 2003 06:34:52 -0300 Received: from giulio ([200.168.176.23]) by traven9.uol.com.br (8.9.1/8.9.1) with SMTP id GAA29317; Fri, 11 Apr 2003 06:33:33 -0300 (BRT) Message-ID: <003601c3000c$554998c0$0200000a@giulio> From: "Fabiano" To: References: <1e8.64dd35c.2bc6a2e6@aol.com> Subject: Re: [obm-l] ajuda-colmeia Date: Fri, 11 Apr 2003 06:25:18 -0300 MIME-Version: 1.0 Content-Type: multipart/alternative; boundary="----=_NextPart_000_0028_01C2FFF3.1F05BA00" X-Priority: 1 X-MSMail-Priority: High X-Mailer: Microsoft Outlook Express 6.00.2800.1106 X-MimeOLE: Produced By Microsoft MimeOLE V6.00.2800.1106 Sender: owner-obm-l@sucuri.mat.puc-rio.br Precedence: bulk Reply-To: obm-l@mat.puc-rio.br This is a multi-part message in MIME format. ------=_NextPart_000_0028_01C2FFF3.1F05BA00 Content-Type: text/plain; charset="iso-8859-1" Content-Transfer-Encoding: quoted-printable ----- Original Message -----=20 From: Lltmdrtm@aol.com=20 To: obm-l@mat.puc-rio.br=20 Sent: Thursday, April 10, 2003 7:35 AM Subject: [obm-l] ajuda-colmeia Uma colmeia nova tem 8000 abelhas. Destas, a cada dia que passa, = morrem 200. Do 21=BA dia em diante, nascem diariamente 2000 abelhas que = vivem, em m=E9dia, 40 dias. Ap=F3s um certo tempo, o n=FAmero de abelhas = dessa colmeia se estabilizar=E1 em, aproximadamente quant Nos primeiros 20 dias, morreriam 4000 abelhas (200*20), ou seja, o = numero da colmeia ficaria na metade do inicial.... Do 21=B0 dia at=E9 o dia 40=B0, o numero de abelhas vivas iria aumentar = numa propor=E7=E3o de 1800 abelhas, totalizando ao final do 40=B0 dia o = valor de 40.000. Do 41=B0 dia at=E9 o dia 61=B0 o n=FAmero de abelhas ia = subir at=E9 chegar em 80.000(2.000*20+40.000), e apartir do dia 61 o = n=FAmero iria se manter est=E1vel.... Bem, eu creio que a resolu=E7=E3o seja essa... Fabiano ------=_NextPart_000_0028_01C2FFF3.1F05BA00 Content-Type: text/html; charset="iso-8859-1" Content-Transfer-Encoding: quoted-printable
 
----- Original Message -----
From:=20 Lltmdrtm@aol.com=20
Sent: Thursday, April 10, 2003 = 7:35=20 AM
Subject: [obm-l] = ajuda-colmeia

Uma colmeia nova tem 8000 abelhas. Destas, a cada = dia que=20 passa, morrem 200. Do 21=BA dia em diante, nascem diariamente 2000 = abelhas que=20 vivem, em m=E9dia, 40 dias. Ap=F3s um certo tempo, o n=FAmero de = abelhas dessa=20 colmeia se estabilizar=E1 em, aproximadamente quant
Nos primeiros 20 dias, morreriam 4000 abelhas (200*20), ou seja, o = numero=20 da colmeia ficaria na metade do inicial....
Do 21=B0 dia at=E9 o dia  40=B0, o numero de abelhas vivas = iria aumentar=20 numa propor=E7=E3o de 1800 abelhas, totalizando ao final do 40=B0 dia o = valor de=20 40.000. Do 41=B0 dia at=E9 o dia 61=B0 o n=FAmero de abelhas ia subir = at=E9 chegar em=20 80.000(2.000*20+40.000), e apartir do dia 61 o n=FAmero iria se manter=20 est=E1vel....
 
Bem, eu creio que a resolu=E7=E3o seja essa...
 
Fabiano
------=_NextPart_000_0028_01C2FFF3.1F05BA00-- ========================================================================= Instruções para entrar na lista, sair da lista e usar a lista em http://www.mat.puc-rio.br/~nicolau/olimp/obm-l.html O administrador desta lista é ========================================================================= From owner-obm-l@sucuri.mat.puc-rio.br Fri Apr 11 08:49:44 2003 Return-Path: Received: (from majordom@localhost) by sucuri.mat.puc-rio.br (8.9.3/8.9.3) id IAA09651 for obm-l-MTTP; Fri, 11 Apr 2003 08:46:55 -0300 Received: from aacpdlotus.net.ms.gov.br (ns1.ms.gov.br [200.181.116.3]) by sucuri.mat.puc-rio.br (8.9.3/8.9.3) with ESMTP id IAA09647 for ; Fri, 11 Apr 2003 08:46:51 -0300 From: JoaoCarlos_Junior@net.ms.gov.br Subject: [obm-l] Elogio e Uso de acentos, cedilha To: obm-l@mat.puc-rio.br X-Mailer: Lotus Notes Release 5.0.9a January 7, 2002 Message-ID: Date: Fri, 11 Apr 2003 07:50:01 -0400 X-MIMETrack: Serialize by Router on aacpdlotus/NETMS(Release 5.0.9a |January 7, 2002) at 04/11/2003 07:50:42 AM MIME-Version: 1.0 Content-type: text/plain; charset=iso-8859-1 Content-Transfer-Encoding: 8bit X-MIME-Autoconverted: from quoted-printable to 8bit by sucuri.mat.puc-rio.br id IAA09648 Sender: owner-obm-l@sucuri.mat.puc-rio.br Precedence: bulk Reply-To: obm-l@mat.puc-rio.br Gostaria de elogiar o professor Carlos Gustavo Tamm de Araujo Moreira, pois que a forma de escrita de suas resolucoes matematicas eh verdadeira poesia. Sempre manifestou-se com lirismo. Outro assunto: sugiro o nao uso de qualquer tipo de acento (de nasalizacao, agudo, grave, diferencial, etc); assim como do cedilha e outros, pois que eles sao representados por um conjunto de no minimo dois outros simbolos nas mensagens, o que dificulta a leitura. Nao sei quais outros sinais saem maus representados. Hah outra forma de resolver isto? Um forte abraço, Joao Carlos. ========================================================================= Instruções para entrar na lista, sair da lista e usar a lista em http://www.mat.puc-rio.br/~nicolau/olimp/obm-l.html O administrador desta lista é ========================================================================= From owner-obm-l@sucuri.mat.puc-rio.br Fri Apr 11 12:33:11 2003 Return-Path: Received: (from majordom@localhost) by sucuri.mat.puc-rio.br (8.9.3/8.9.3) id MAA13975 for obm-l-MTTP; Fri, 11 Apr 2003 12:29:49 -0300 Received: from ns3bind.localdomain ([200.230.34.5]) by sucuri.mat.puc-rio.br (8.9.3/8.9.3) with ESMTP id MAA13971 for ; Fri, 11 Apr 2003 12:29:45 -0300 Received: from servico2 ([200.230.34.229]) by ns3bind.localdomain (8.11.6/X.XX.X) with SMTP id h3BFPVZ17847 for ; Fri, 11 Apr 2003 12:25:32 -0300 Message-ID: <002001c3003f$3a614a20$3300c57d@bovespa.com> From: "=?iso-8859-1?Q?Cl=E1udio_\=28Pr=E1tica\=29?=" To: References: <20030411025558.64194.qmail@web21413.mail.yahoo.com> Subject: [obm-l] =?iso-8859-1?Q?Re:_=5Bobm-l=5D_Polin=F4mios_e_Grupo_Abeliano?= Date: Fri, 11 Apr 2003 12:30:03 -0300 MIME-Version: 1.0 Content-Type: text/plain; charset="iso-8859-1" Content-Transfer-Encoding: 8bit X-Priority: 3 X-MSMail-Priority: Normal X-Mailer: Microsoft Outlook Express 5.50.4920.2300 X-MimeOLE: Produced By Microsoft MimeOLE V5.50.4920.2300 Sender: owner-obm-l@sucuri.mat.puc-rio.br Precedence: bulk Reply-To: obm-l@mat.puc-rio.br Basta você mostrar que se A = conjunto dos polinômios de grau <= n, então: 1) Para todos f, g em A, f + g está em A. 2) Para todos f, g, h em A, f + (g + h) = (f + g) + h 3) 0 = polinômio identicamente nulo está em A 4) Para todo f em A, existe g em A tal que: f + g = g + f = 0 5) Para todos f, g em A, f + g = g + f. Um abraço, Claudio. ----- Original Message ----- From: "Danilo Pinseta" To: "Nicolau C. Saldanha" Cc: Sent: Thursday, April 10, 2003 11:55 PM Subject: [obm-l] Polinômios e Grupo Abeliano > Gostaria de receber demonstrações de que polinômios > de grau <= n) são grupo abeliano para adição. > Obrigado > DANILO > > ========================================================================= Instruções para entrar na lista, sair da lista e usar a lista em http://www.mat.puc-rio.br/~nicolau/olimp/obm-l.html O administrador desta lista é ========================================================================= From owner-obm-l@sucuri.mat.puc-rio.br Fri Apr 11 13:19:34 2003 Return-Path: Received: (from majordom@localhost) by sucuri.mat.puc-rio.br (8.9.3/8.9.3) id NAA14976 for obm-l-MTTP; Fri, 11 Apr 2003 13:17:17 -0300 Received: from web12905.mail.yahoo.com (web12905.mail.yahoo.com [216.136.174.72]) by sucuri.mat.puc-rio.br (8.9.3/8.9.3) with SMTP id NAA14967 for ; Fri, 11 Apr 2003 13:17:12 -0300 Message-ID: <20030411161636.50428.qmail@web12905.mail.yahoo.com> Received: from [200.206.103.3] by web12905.mail.yahoo.com via HTTP; Fri, 11 Apr 2003 13:16:36 ART Date: Fri, 11 Apr 2003 13:16:36 -0300 (ART) From: =?iso-8859-1?q?Johann=20Peter=20Gustav=20Lejeune=20Dirichlet?= Subject: Re: [obm-l] Mais Probls em Aberto II To: obm-l@mat.puc-rio.br In-Reply-To: <200304110520.h3B5Ka922795@Gauss.impa.br> MIME-Version: 1.0 Content-Type: multipart/alternative; boundary="0-51144419-1050077796=:50303" Content-Transfer-Encoding: 8bit Sender: owner-obm-l@sucuri.mat.puc-rio.br Precedence: bulk Reply-To: obm-l@mat.puc-rio.br --0-51144419-1050077796=:50303 Content-Type: text/plain; charset=iso-8859-1 Content-Transfer-Encoding: 8bit Turma esse primeiro eu consegui fazer usando um pouco de Schur.Tente ver por absurdo.Fica bem parecido com a soluçao de tres linhas do Gugu se voce for doido de usar Maclaurin-Newton junto.Depois eu passo... Em desigualdades com lados de um triangulo sempre use a substituiçao de Ravi:a=x+y,b=y+z,c=z+w com x,y,z positivos. Carlos Gustavo Tamm de Araujo Moreira wrote:> >11) Seja a,b,c,d 4 n=FAmeros reais n=E3o negativos que=20 >satisfazem a condi=E7=E3o=20 >2*(ab+ac+ad+bc+bd+cd)+abc+abd+acd+bcd=3D16. >Prove que a+b+c+d>=3D2/3*(ab+ac+ad+bc+bd+cd) e determine o=20 >caso de igualdade. > O enunciado implica que o polinomio P(x)=(x-a)(x-b)(x-c)(x-d) tem quatro raizes nao negativas, e portanto sua derivada tambem (pelo teorema do valor medio). Seja entao P'(x)=4(x-u)(x-v)(x-w).Temos 3(a+b+c+d)=4(u+v+w), 2(ab+ac+ad+bc+bd+cd)=4(uv+uw+vw) e abc+abd+acd+bcd=4uvw. Assim, a condicao 2(ab+ac+ad+bc+bd+cd)+abc+abd+acd+bcd=16 equivale a uv+uw+vw+uvw=4. A conclusao equivale a u+v+w>=uv+uw+vw. Multiplicando por u+v+uv e usando w(u+v+uv)=4-uv, temos que a conclusao segue de (u+v)(u+v+uv)+4-uv>=uv(u+v+uv)+(u+v)(4-uv) (note que se u+v+uv=0 entao u=v=0, e nao poderiamos ter uv+uw+vw+uvw=4). Essa desigualdade pode ser escrita como (u+v-2)^2-uv(uv-u-v+1)>=0, ou seja, ((u-1)+(v-1))^2-uv(u-1)(v-1)>=0, e essa ultima desigualdade segue de 0<=uv=4-(uw+vw+uvw)<=4. So' podemos ter igualdade se u=v=1 (quando w tambem e' 1) ou quando uv=0, u+v=2, caso em que w=2, ou quando uv=4 e u=v, caso em que u=v=2 e w=0. Os ultimos casos nao sao possiveis (senao P'(x)=4x(x-2)^2>=0 para x>=0, donde P(x) seria crescente para x>=0, e logo, como abc+abd+acd+bcd=4uvw=0, todas as raizes de P deveriam ser iguais a 0, donde u=v=w=0, absurdo), e no primeiro devemos ter a=b=c=d=1 (pois (a-1)^2+(b-1)^2+(c-1)^2+(d-1)^2=(a+b+c+d)^2-4(ab+ac+ad+bc+bd+cd)+4= =(4(u+v+w)/3)^2-8(uv+uw+vw)+4=4^2-8.3+4=0), que e' assim o unico caso de igualdade. >***** > >13) Seja a,b e c medidas dos lados de um tri=E2ngulo.=20 >Prove que: raiz(a+b-c)+raiz(b+c-a)+raiz(c+a-b)=3D>+raiz(b)+raiz(c) > Sejam a+b-c=x, a+c-b=y e b+c-a=z. O problema equivale a mostrar que raiz(2x)+raiz(2y)+raiz(2z)<=raiz(x+y)+raiz(x+z)+raiz(y+z), mas isso segue de raiz(2x)+raiz(2y)<=2.raiz(x+y), raiz(2x)+raiz(2z)<=2.raiz(x+z) e raiz(2y)+raiz(2z)<=2.raiz(y+z), que sao equivalentes. A primeira segue elevando ao quadrado: ela fica equivalente a 2x+2y+4.raiz(xy)<=4(x+y), ou a x-2.raiz(xy)+y=(raiz(x)-raiz(y))^2>=0, que e' obvio. So vale a igualdade se x=y=z, ou seja, se a=b=c. >***** > >14)Demonstrar que para quaisquer valores real e x, y=20 >e z =E9 v=E1lida a desigualdade >4x(x+y)(x+z)(x+y+z)+y=B2z=B2>=3D0 > Escrevendo 4x(x+y)(x+z)(x+y+z)+y^2.z^2 como um polinomio do segundo grau em y, obtemos (z^2+4x(x+z))y^2+(4x(x+z)^2+4x^2.(x+z))y+4x^2.(x+z)^2= =(z+2x)^2.y^2+4x(x+z)(z+2x)^2+4(x(x+z))^2= ((z+2x)y+2x(x+z))^2 >=0. >***** > >15) Se a^(b^c) =3D b^d , c/d pode ser dado em fun=E7=E3o de a e b ? > Nao. Se a^(b^c)=b^d entao a^(b^(c+1))=b^(bd) e a^(b^(c+2))=b^(b^2.d). Se c/d so' dependesse de a e de b teriamos c/d=(c+1)/bd=(c+2)/(b^2.d), donde bc=c+1 e b^2.c=c+2, donde 2=c(b^2-1)=c(b-1)(b+1)=b+1, e logo b=1 e c=c+1, absurdo. >***** > >16) Seja a fun=E7ao f:N*U{0} ->N*U{0} dada pelas = >propriedades:(f(2n+1))=B2-(f(2n))=B2=3D6f(n)+1 e f(2n)>=3Df(n) para todo = >n natural.Ache #{x elemento de N,f(x)<2003}.=20 >(A solu=E7=E3o desse vale um doce - cortesia do Dirichlet!) > Vou mostrar por inducao que se n=soma(j=0 ate' k)(s_j.2^j), com s_j em {0,1} e' a representacao binaria de n entao f(n)=soma(j=0 ate' k)(s_j.3^j). De fato, de f(1)^2-f(0)^2=6f(0)+1 segue (f(0)+3)^2-f(1)^2=8, donde f(0)+3+f(1)=4 e f(0)+3-f(1)=2 (pois f(0)+3 e f(1) sao naturais de mesma paridade), e logo f(1)=1 e f(0)=0. Temos (f(2n+1)+f(2n))(f(2n+1)-f(2n))=6f(n)+1. Assim f(2n+1)>f(2n)>=f(n), donde f(2n+1)+f(2n)>=2f(n)+1 e e' um divisor de 6f(n)+1. Como 6f(n)+1 e' impar e (6f(n)+1)/(2f(n)+1)<3, segue que f(2n+1)+f(2n)=6f(n)+1 e f(2n+1)-f(2n)=1, e logo f(2n+1)=3f(n)+1 e f(2n)=3f(n), o que implica nossa afirmacao para 2n e 2n+1, supondo que ela vale para n. Agora, como f(x) e' uma soma de potencias de 3 distintas, se f(x) < 2003 , como 3^6+3^5+3^4+3^3+3^2+3^1+3^0 < 2003 < 3^7, x deve ser uma soma qualquer de potencias de 2 distintas coem expoentes menores que 7, ou seja, x pode ser qualquer numero menor que 2^7=128. Assim, {x natural,f(x)<2003}={0,1,2,...,127}, donde #{x natural,f(x)<2003}=128. Cade meu doce, Dirichlet ? >***** Abracos, Gugu ========================================================================= Instruções para entrar na lista, sair da lista e usar a lista em http://www.mat.puc-rio.br/~nicolau/olimp/obm-l.html O administrador desta lista é ========================================================================= --------------------------------- Yahoo! Mail O melhor e-mail gratuito da internet: 6MB de espaço, antivírus, acesso POP3, filtro contra spam. --0-51144419-1050077796=:50303 Content-Type: text/html; charset=iso-8859-1 Content-Transfer-Encoding: 8bit

Turma esse primeiro eu consegui fazer usando um pouco de Schur.Tente ver por absurdo.Fica bem parecido com a soluçao de tres linhas do Gugu se voce for doido de usar Maclaurin-Newton junto.Depois eu passo...

Em desigualdades com lados de um triangulo sempre use a substituiçao de Ravi:a=x+y,b=y+z,c=z+w com x,y,z positivos.

 Carlos Gustavo Tamm de Araujo Moreira <gugu@impa.br> wrote:

>
>11) Seja a,b,c,d 4 n=FAmeros reais n=E3o negativos que=20
>satisfazem a condi=E7=E3o=20
>2*(ab+ac+ad+bc+bd+cd)+abc+abd+acd+bcd=3D16.
>Prove que a+b+c+d>=3D2/3*(ab+ac+ad+bc+bd+cd) e determine o=20
>caso de igualdade.
>

O enunciado implica que o polinomio P(x)=(x-a)(x-b)(x-c)(x-d) tem quatro
raizes nao negativas, e portanto sua derivada tambem (pelo teorema do valor
medio). Seja entao P'(x)=4(x-u)(x-v)(x-w).Temos 3(a+b+c+d)=4(u+v+w),
2(ab+ac+ad+bc+bd+cd)=4(uv+uw+vw) e abc+abd+acd+bcd=4uvw. Assim, a condicao
2(ab+ac+ad+bc+bd+cd)+abc+abd+acd+bcd=16 equivale a uv+uw+vw+uvw=4. A
conclusao equivale a u+v+w>=uv+uw+vw. Multiplicando por u+v+uv e usando
w(u+v+uv)=4-uv, temos que a conclusao segue de
(u+v)(u+v+uv)+4-uv>=uv(u+v+uv)+(u+v)(4-uv) (note que se u+v+uv=0 entao
u=v=0, e nao poderiamos ter uv+uw+vw+uvw=4). Essa desigualdade pode ser
escrita como (u+v-2)^2-uv(uv-u-v+1)>=0, ou seja,
((u-1)+(v-1))^2-uv(u-1)(v-1)>=0, e essa ultima desigualdade segue de
0<=uv=4-(uw+vw+uvw)<=4. So' podemos ter igualdade se u=v=1 (quando w tambem
e' 1) ou quando uv=0, u+v=2, caso em que w=2, ou quando uv=4 e u=v, caso em
que u=v=2 e w=0. Os ultimos casos nao sao possiveis (senao
P'(x)=4x(x-2)^2>=0 para x>=0, donde P(x) seria crescente para x>=0, e logo,
como abc+abd+acd+bcd=4uvw=0, todas as raizes de P deveriam ser iguais a 0,
donde u=v=w=0, absurdo), e no primeiro devemos ter a=b=c=d=1 (pois
(a-1)^2+(b-1)^2+(c-1)^2+(d-1)^2=(a+b+c+d)^2-4(ab+ac+ad+bc+bd+cd)+4=
=(4(u+v+w)/3)^2-8(uv+uw+vw)+4=4^2-8.3+4=0), que e' assim o unico caso de
igualdade.

>*****
>
>13) Seja a,b e c medidas dos lados de um tri=E2ngulo.=20
>Prove que: raiz(a+b-c)+raiz(b+c-a)+raiz(c+a-b)=3D>+raiz(b)+raiz(c)
>

Sejam a+b-c=x, a+c-b=y e b+c-a=z. O problema equivale a mostrar que
raiz(2x)+raiz(2y)+raiz(2z)<=raiz(x+y)+raiz(x+z)+raiz(y+z), mas isso segue de
raiz(2x)+raiz(2y)<=2.raiz(x+y), raiz(2x)+raiz(2z)<=2.raiz(x+z) e
raiz(2y)+raiz(2z)<=2.raiz(y+z), que sao equivalentes. A primeira segue
elevando ao quadrado: ela fica equivalente a 2x+2y+4.raiz(xy)<=4(x+y), ou a
x-2.raiz(xy)+y=(raiz(x)-raiz(y))^2>=0,
que e' obvio. So vale a igualdade se x=y=z, ou seja, se a=b=c.

>*****
>
>14)Demonstrar que para quaisquer valores real e x, y=20
>e z =E9 v=E1lida a desigualdade
>4x(x+y)(x+z)(x+y+z)+y=B2z=B2>=3D0
>

Escrevendo 4x(x+y)(x+z)(x+y+z)+y^2.z^2 como um polinomio do segundo grau em
y, obtemos (z^2+4x(x+z))y^2+(4x(x+z)^2+4x^2.(x+z))y+4x^2.(x+z)^2=
=(z+2x)^2.y^2+4x(x+z)(z+2x)^2+4(x(x+z))^2= ((z+2x)y+2x(x+z))^2 >=0.

>*****
>
>15) Se a^(b^c) =3D b^d , c/d pode ser dado em fun=E7=E3o de a e b ?
>

Nao. Se a^(b^c)=b^d entao a^(b^(c+1))=b^(bd) e a^(b^(c+2))=b^(b^2.d). Se
c/d so' dependesse de a e de b teriamos c/d=(c+1)/bd=(c+2)/(b^2.d), donde
bc=c+1 e b^2.c=c+2, donde 2=c(b^2-1)=c(b-1)(b+1)=b+1, e logo b=1 e c=c+1,
absurdo.

>*****
>
>16) Seja a fun=E7ao f:N*U{0} ->N*U{0} dada pelas =
>propriedades:(f(2n+1))=B2-(f(2n))=B2=3D6f(n)+1 e f(2n)>=3Df(n) para todo =
>n natural.Ache #{x elemento de N,f(x)<2003}.=20
>(A solu=E7=E3o desse vale um doce - cortesia do Dirichlet!)
>

Vou mostrar por inducao que se n=soma(j=0 ate' k)(s_j.2^j), com s_j em
{0,1} e' a representacao binaria de n entao f(n)=soma(j=0 ate' k)(s_j.3^j).
De fato, de f(1)^2-f(0)^2=6f(0)+1 segue (f(0)+3)^2-f(1)^2=8, donde
f(0)+3+f(1)=4 e f(0)+3-f(1)=2 (pois f(0)+3 e f(1) sao naturais de mesma
paridade), e logo f(1)=1 e f(0)=0. Temos
(f(2n+1)+f(2n))(f(2n+1)-f(2n))=6f(n)+1. Assim f(2n+1)>f(2n)>=f(n), donde
f(2n+1)+f(2n)>=2f(n)+1 e e' um divisor de 6f(n)+1. Como 6f(n)+1 e' impar e
(6f(n)+1)/(2f(n)+1)<3, segue que f(2n+1)+f(2n)=6f(n)+1 e f(2n+1)-f(2n)=1, e
logo f(2n+1)=3f(n)+1 e f(2n)=3f(n), o que implica nossa afirmacao para 2n e
2n+1, supondo que ela vale para n.
Agora, como f(x) e' uma soma de potencias de 3 distintas, se f(x) < 2003
, como 3^6+3^5+3^4+3^3+3^2+3^1+3^0 < 2003 < 3^7, x deve ser uma soma
qualquer de potencias de 2 distintas coem expoentes menores que 7, ou seja,
x pode ser qualquer numero menor que 2^7=128. Assim, {x
natural,f(x)<2003}={0,1,2,...,127}, donde #{x natural,f(x)<2003}=128.
Cade meu doce, Dirichlet ?

>*****
Abracos,
Gugu
=========================================================================
Instruções para entrar na lista, sair da lista e usar a lista em
http://www.mat.puc-rio.br/~nicolau/olimp/obm-l.html
O administrador desta lista é
=========================================================================



Yahoo! Mail
O melhor e-mail gratuito da internet: 6MB de espaço, antivírus, acesso POP3, filtro contra spam. --0-51144419-1050077796=:50303-- ========================================================================= Instruções para entrar na lista, sair da lista e usar a lista em http://www.mat.puc-rio.br/~nicolau/olimp/obm-l.html O administrador desta lista é ========================================================================= From owner-obm-l@sucuri.mat.puc-rio.br Fri Apr 11 13:23:04 2003 Return-Path: Received: (from majordom@localhost) by sucuri.mat.puc-rio.br (8.9.3/8.9.3) id NAA15030 for obm-l-MTTP; Fri, 11 Apr 2003 13:20:30 -0300 Received: from web12905.mail.yahoo.com (web12905.mail.yahoo.com [216.136.174.72]) by sucuri.mat.puc-rio.br (8.9.3/8.9.3) with SMTP id NAA15016 for ; Fri, 11 Apr 2003 13:20:23 -0300 Message-ID: <20030411161952.50992.qmail@web12905.mail.yahoo.com> Received: from [200.206.103.3] by web12905.mail.yahoo.com via HTTP; Fri, 11 Apr 2003 13:19:52 ART Date: Fri, 11 Apr 2003 13:19:52 -0300 (ART) From: =?iso-8859-1?q?Johann=20Peter=20Gustav=20Lejeune=20Dirichlet?= Subject: Re: [obm-l] Elogio e Uso de acentos, cedilha To: obm-l@mat.puc-rio.br In-Reply-To: MIME-Version: 1.0 Content-Type: multipart/alternative; boundary="0-409054994-1050077992=:48692" Content-Transfer-Encoding: 8bit Sender: owner-obm-l@sucuri.mat.puc-rio.br Precedence: bulk Reply-To: obm-l@mat.puc-rio.br --0-409054994-1050077992=:48692 Content-Type: text/plain; charset=iso-8859-1 Content-Transfer-Encoding: 8bit Bem ,quanto ao Gugu eu concordo,to contigo e nao abro.Quanto aos acentos eu ja faço isso. JoaoCarlos_Junior@net.ms.gov.br wrote: Gostaria de elogiar o professor Carlos Gustavo Tamm de Araujo Moreira, pois que a forma de escrita de suas resolucoes matematicas eh verdadeira poesia. Sempre manifestou-se com lirismo. Outro assunto: sugiro o nao uso de qualquer tipo de acento (de nasalizacao, agudo, grave, diferencial, etc); assim como do cedilha e outros, pois que eles sao representados por um conjunto de no minimo dois outros simbolos nas mensagens, o que dificulta a leitura. Nao sei quais outros sinais saem maus representados. Hah outra forma de resolver isto? Um forte abraço, Joao Carlos. ========================================================================= Instruções para entrar na lista, sair da lista e usar a lista em http://www.mat.puc-rio.br/~nicolau/olimp/obm-l.html O administrador desta lista é ========================================================================= --------------------------------- Yahoo! Mail O melhor e-mail gratuito da internet: 6MB de espaço, antivírus, acesso POP3, filtro contra spam. --0-409054994-1050077992=:48692 Content-Type: text/html; charset=iso-8859-1 Content-Transfer-Encoding: 8bit

Bem ,quanto ao Gugu eu concordo,to contigo e nao abro.Quanto aos acentos eu ja faço isso.

 JoaoCarlos_Junior@net.ms.gov.br wrote:


Gostaria de elogiar o professor Carlos Gustavo Tamm de Araujo Moreira, pois
que a forma de escrita de suas resolucoes matematicas eh verdadeira poesia.
Sempre manifestou-se com lirismo.

Outro assunto: sugiro o nao uso de qualquer tipo de acento (de

nasalizacao, agudo, grave, diferencial, etc); assim como do cedilha e

outros, pois que eles sao representados por um conjunto de no minimo dois

outros simbolos nas mensagens, o que dificulta a leitura.

Nao sei quais outros sinais saem maus representados. Hah outra

forma de resolver isto?

Um forte abraço, Joao Carlos.


=========================================================================
Instruções para entrar na lista, sair da lista e usar a lista em
http://www.mat.puc-rio.br/~nicolau/olimp/obm-l.html
O administrador desta lista é
=========================================================================



Yahoo! Mail
O melhor e-mail gratuito da internet: 6MB de espaço, antivírus, acesso POP3, filtro contra spam. --0-409054994-1050077992=:48692-- ========================================================================= Instruções para entrar na lista, sair da lista e usar a lista em http://www.mat.puc-rio.br/~nicolau/olimp/obm-l.html O administrador desta lista é ========================================================================= From owner-obm-l@sucuri.mat.puc-rio.br Fri Apr 11 13:26:47 2003 Return-Path: Received: (from majordom@localhost) by sucuri.mat.puc-rio.br (8.9.3/8.9.3) id NAA15166 for obm-l-MTTP; Fri, 11 Apr 2003 13:24:06 -0300 Received: from mail.fronthost.com (mail.fronthost.com [63.250.6.253]) by sucuri.mat.puc-rio.br (8.9.3/8.9.3) with ESMTP id NAA15153 for ; Fri, 11 Apr 2003 13:24:00 -0300 Received: from [68.49.44.134] by fronthost.com [63.250.6.253] with SmartMax MailMax for at Fri, 11 Apr 2003 12:23:48 -0400 Message-ID: <00a201c30046$6b208570$6401a8c0@TEST4> From: "Alexandre A da Rocha" To: References: Subject: Re: [obm-l] Elogio e Uso de acentos, cedilha Date: Fri, 11 Apr 2003 12:21:33 -0400 MIME-Version: 1.0 Content-Type: multipart/alternative; boundary="----=_NextPart_000_009F_01C30024.E3E11EB0" X-Priority: 3 X-MSMail-Priority: Normal X-Mailer: Microsoft Outlook Express 5.50.4807.1700 X-MimeOLE: Produced By Microsoft MimeOLE V5.50.4807.1700 Sender: owner-obm-l@sucuri.mat.puc-rio.br Precedence: bulk Reply-To: obm-l@mat.puc-rio.br This is a multi-part message in MIME format. ------=_NextPart_000_009F_01C30024.E3E11EB0 Content-Type: text/plain; charset="iso-8859-1" Content-Transfer-Encoding: quoted-printable Do as I say, not as I do eh? :) -Auggy ----- Original Message -----=20 From: To: Sent: Friday, April 11, 2003 7:50 AM Subject: [obm-l] Elogio e Uso de acentos, cedilha Gostaria de elogiar o professor Carlos Gustavo Tamm de Araujo Moreira, = pois que a forma de escrita de suas resolucoes matematicas eh verdadeira = poesia. Sempre manifestou-se com lirismo. Outro assunto: sugiro o nao uso de qualquer tipo de acento = (de nasalizacao, agudo, grave, diferencial, etc); assim como do = cedilha e outros, pois que eles sao representados por um conjunto de no minimo = dois outros simbolos nas mensagens, o que dificulta a leitura. Nao sei quais outros sinais saem maus representados. Hah = outra forma de resolver isto? Um forte abra=E7o, Joao Carlos. =3D=3D=3D=3D=3D=3D=3D=3D=3D=3D=3D=3D=3D=3D=3D=3D=3D=3D=3D=3D=3D=3D=3D=3D=3D= =3D=3D=3D=3D=3D=3D=3D=3D=3D=3D=3D=3D=3D=3D=3D=3D=3D=3D=3D=3D=3D=3D=3D=3D=3D= =3D=3D=3D=3D=3D=3D=3D=3D=3D=3D=3D=3D=3D=3D=3D=3D=3D=3D=3D=3D=3D=3D=3D Instru=E7=F5es para entrar na lista, sair da lista e usar a lista em http://www.mat.puc-rio.br/~nicolau/olimp/obm-l.html O administrador desta lista =E9 =3D=3D=3D=3D=3D=3D=3D=3D=3D=3D=3D=3D=3D=3D=3D=3D=3D=3D=3D=3D=3D=3D=3D=3D=3D= =3D=3D=3D=3D=3D=3D=3D=3D=3D=3D=3D=3D=3D=3D=3D=3D=3D=3D=3D=3D=3D=3D=3D=3D=3D= =3D=3D=3D=3D=3D=3D=3D=3D=3D=3D=3D=3D=3D=3D=3D=3D=3D=3D=3D=3D=3D=3D=3D ------=_NextPart_000_009F_01C30024.E3E11EB0 Content-Type: text/html; charset="iso-8859-1" Content-Transfer-Encoding: quoted-printable
Do as I say, not as I do eh? = :)
 
-Auggy
 
----- Original Message -----
From: <JoaoCarlos_Junior@net.ms.gov.br>
To: <obm-l@mat.puc-rio.br>
Sent: Friday, April 11, 2003 7:50 = AM
Subject: [obm-l] Elogio e Uso de = acentos,=20 cedilha


Gostaria de elogiar o professor Carlos Gustavo = Tamm de=20 Araujo Moreira, pois
que a forma de escrita de suas resolucoes = matematicas eh=20 verdadeira poesia.
Sempre manifestou-se com=20 lirismo.

        Outro =20 assunto:  sugiro  o  nao  uso de qualquer tipo de = acento=20 (de

  nasalizacao,  agudo,  grave, =20 diferencial,  etc); assim como do cedilha = e

 =20 outros, pois que eles sao representados por um conjunto de no minimo=20 dois

  outros simbolos nas mensagens, o que dificulta a=20 leitura.

        Nao  = sei =20 quais  outros  sinais  saem maus representados. Hah=20 outra

  forma de resolver=20 isto?

        Um forte=20 abra=E7o, Joao=20 Carlos.


=3D=3D=3D=3D=3D=3D=3D=3D=3D=3D=3D=3D=3D=3D=3D=3D=3D=3D= =3D=3D=3D=3D=3D=3D=3D=3D=3D=3D=3D=3D=3D=3D=3D=3D=3D=3D=3D=3D=3D=3D=3D=3D=3D= =3D=3D=3D=3D=3D=3D=3D=3D=3D=3D=3D=3D=3D=3D=3D=3D=3D=3D=3D=3D=3D=3D=3D=3D=3D= =3D=3D=3D=3D=3D
Instru=E7=F5es=20 para entrar na lista, sair da lista e usar a lista em
http://www.mat.puc-rio.br/~nicolau/olimp/obm-l.htmlO administrador desta lista =E9 <nicolau@mat.puc-rio.br>
=3D=3D=3D=3D=3D=3D=3D=3D=3D=3D=3D=3D=3D=3D=3D=3D=3D=3D=3D= =3D=3D=3D=3D=3D=3D=3D=3D=3D=3D=3D=3D=3D=3D=3D=3D=3D=3D=3D=3D=3D=3D=3D=3D=3D= =3D=3D=3D=3D=3D=3D=3D=3D=3D=3D=3D=3D=3D=3D=3D=3D=3D=3D=3D=3D=3D=3D=3D=3D=3D= =3D=3D=3D=3D
------=_NextPart_000_009F_01C30024.E3E11EB0-- ========================================================================= Instruções para entrar na lista, sair da lista e usar a lista em http://www.mat.puc-rio.br/~nicolau/olimp/obm-l.html O administrador desta lista é ========================================================================= From owner-obm-l@sucuri.mat.puc-rio.br Fri Apr 11 14:43:46 2003 Return-Path: Received: (from majordom@localhost) by sucuri.mat.puc-rio.br (8.9.3/8.9.3) id OAA17320 for obm-l-MTTP; Fri, 11 Apr 2003 14:39:27 -0300 Received: from smtp.ieg.com.br (sharon.protocoloweb.com.br [200.226.139.12]) by sucuri.mat.puc-rio.br (8.9.3/8.9.3) with ESMTP id OAA17314 for ; Fri, 11 Apr 2003 14:39:22 -0300 From: pergola@ieg.com.br Received: from ieg.com.br (jimi.protocoloweb.com.br [200.226.139.29]) by smtp.ieg.com.br (8.12.8/8.9.3) with SMTP id h3BHW3Wj083811 for ; Fri, 11 Apr 2003 14:32:03 -0300 (BRT) To: obm-l@mat.puc-rio.br Date: Fri, 11 Apr 2003 17:38:32 GMT Subject: [obm-l] GA - vetores X-Mailer: DMailWeb Web to Mail Gateway 2.7v, http://netwinsite.com/top_mail.htm Message-id: <3e96fd98.f88.0@ieg.com.br> X-User-Info: 200.161.156.124 MIME-Version: 1.0 Content-Type: text/plain; charset="iso-8859-1" Content-Transfer-Encoding: 8bit X-MIME-Autoconverted: from quoted-printable to 8bit by sucuri.mat.puc-rio.br id OAA17315 Sender: owner-obm-l@sucuri.mat.puc-rio.br Precedence: bulk Reply-To: obm-l@mat.puc-rio.br Olah pessoal, Estou com duvida nos seguintes exercicios, gostaria de ver a resolucao: Nao sei a notacao para vetores, entao quando for, vou usar colchetes, exemplo: [AB] = -[BA] 1) Dados O, A, B, C, ache G tal que [GA]+[GB]+[GC]=[0] em funcao de O, [OA], [OB], [OC]. 2) Num triangulo ABC, onde M esta na reta AB e N esta na reta AC, a distancia de M a A eh o dobro da distancia de M a B, e a medida de AN eh a terca parte da medida de CN. Exprima X em funcao de A, [AB] e [AC]. 3) Considere o triangulo ABC, e sejam [CA]=[u],[CB]=[v] e [w]=[u] - 2[v]. Calcule k real para que o ponto X=C+k[w] pertenca a reta AB. Obrigado, Gabriel Campos Pérgola http://www.ieg.com.br ========================================================================= Instruções para entrar na lista, sair da lista e usar a lista em http://www.mat.puc-rio.br/~nicolau/olimp/obm-l.html O administrador desta lista é ========================================================================= From owner-obm-l@sucuri.mat.puc-rio.br Fri Apr 11 15:39:20 2003 Return-Path: Received: (from majordom@localhost) by sucuri.mat.puc-rio.br (8.9.3/8.9.3) id PAA19280 for obm-l-MTTP; Fri, 11 Apr 2003 15:36:28 -0300 Received: from hotmail.com (f67.law8.hotmail.com [216.33.241.67]) by sucuri.mat.puc-rio.br (8.9.3/8.9.3) with ESMTP id PAA19275 for ; Fri, 11 Apr 2003 15:36:25 -0300 Received: from mail pickup service by hotmail.com with Microsoft SMTPSVC; Fri, 11 Apr 2003 11:35:53 -0700 Received: from 200.222.105.178 by lw8fd.law8.hotmail.msn.com with HTTP; Fri, 11 Apr 2003 18:35:52 GMT X-Originating-IP: [200.222.105.178] X-Originating-Email: [osneto@hotmail.com] From: "Antonio Neto" To: obm-l@mat.puc-rio.br Subject: [obm-l] Sequências Date: Fri, 11 Apr 2003 18:35:52 +0000 Mime-Version: 1.0 Content-Type: text/plain; format=flowed Message-ID: X-OriginalArrivalTime: 11 Apr 2003 18:35:53.0256 (UTC) FILETIME=[2EBA5A80:01C30059] Sender: owner-obm-l@sucuri.mat.puc-rio.br Precedence: bulk Reply-To: obm-l@mat.puc-rio.br >From: Igor GomeZZ >Reply-To: obm-l@mat.puc-rio.br >To: OBM >Subject: [obm-l] Trigonometria e Sequências >Date: Fri, 11 Apr 2003 01:38:05 -0300 > > > Fala galera da lista, boa noite... São dois problemas, um OBM e o >outro >não sei a fonte: > >**Sequência: > >1*2 + 2*3 + 3*4 + 4*5 +...+47*48 + 48*49 + 49*50 > >Se não me engano ela eh OBM, certo? Consegui resolvê-la como uma >Progressão Aritmética de segunda ordem. Para achar o polinômio >que gera os termos t(n) eh relativamente demorado, jah para achar o >polinômio que define a soma s(n) eh ainda mais demorado. Tem alguma coisa >na cara que facilite a questão e não estou vendo? > Hah uma identidade, que eh C(p, p) + C(p+1, p) + C8p+2, p) +...+C(n, p)= C(n+1, p+1). No seu caso, chamando a soma de S, vem S/2 = C(2, 2)+C(3, 2)+C(4, 2)+...+C(50, 2)= C(51, 3)= 20825. Se nao errei nada, eh isso, abracos, olavo. _________________________________________________________________ STOP MORE SPAM with the new MSN 8 and get 2 months FREE* http://join.msn.com/?page=features/junkmail ========================================================================= Instruções para entrar na lista, sair da lista e usar a lista em http://www.mat.puc-rio.br/~nicolau/olimp/obm-l.html O administrador desta lista é ========================================================================= From owner-obm-l@sucuri.mat.puc-rio.br Fri Apr 11 16:15:55 2003 Return-Path: Received: (from majordom@localhost) by sucuri.mat.puc-rio.br (8.9.3/8.9.3) id QAA20164 for obm-l-MTTP; Fri, 11 Apr 2003 16:12:32 -0300 Received: from ns3bind.localdomain ([200.230.34.5]) by sucuri.mat.puc-rio.br (8.9.3/8.9.3) with ESMTP id QAA20160 for ; Fri, 11 Apr 2003 16:12:27 -0300 Received: from servico2 ([200.230.34.224]) by ns3bind.localdomain (8.11.6/X.XX.X) with SMTP id h3BJ8DD32408 for ; Fri, 11 Apr 2003 16:08:13 -0300 Message-ID: <00ba01c3005e$56faaae0$3300c57d@bovespa.com> From: "=?iso-8859-1?Q?Cl=E1udio_\=28Pr=E1tica\=29?=" To: References: <6732305773.20030411013805@gmx.net> Subject: [obm-l] =?iso-8859-1?Q?Re:_=5Bobm-l=5D_Trigonometria_e_Sequ=EAncias?= Date: Fri, 11 Apr 2003 16:12:46 -0300 MIME-Version: 1.0 Content-Type: text/plain; charset="iso-8859-1" Content-Transfer-Encoding: 8bit X-Priority: 3 X-MSMail-Priority: Normal X-Mailer: Microsoft Outlook Express 5.50.4920.2300 X-MimeOLE: Produced By Microsoft MimeOLE V5.50.4920.2300 Sender: owner-obm-l@sucuri.mat.puc-rio.br Precedence: bulk Reply-To: obm-l@mat.puc-rio.br ----- Original Message ----- From: "Igor GomeZZ" To: "OBM" Sent: Friday, April 11, 2003 1:38 AM Subject: [obm-l] Trigonometria e Sequências > > Fala galera da lista, boa noite... São dois problemas, um OBM e o outro > não sei a fonte: > > **Sequência: > > 1*2 + 2*3 + 3*4 + 4*5 +...+47*48 + 48*49 + 49*50 > > Se não me engano ela eh OBM, certo? Consegui resolvê-la como uma > Progressão Aritmética de segunda ordem. Para achar o polinômio > que gera os termos t(n) eh relativamente demorado, jah para achar o > polinômio que define a soma s(n) eh ainda mais demorado. Tem alguma coisa > na cara que facilite a questão e não estou vendo? > > S = 1*2 + 2*3 + 3*4 + 4*5 +...+47*48 + 48*49 + 49*50 Voce pode dividir a soma por 2 e obter: S/2 = C(2,2) + C(3,2) + ... + C(50,2) onde C(m,p) = numero de subconjuntos de p elementos de um conjunto com m elementos. Ai, usando uma propriedade do Triangulo de Pascal, chegar a conclusao de que: S/2 = C(51,3) ==> S = 2*C(51,3) = 2*51*50*49/6 = 41.650 *************** > ** Trigonometria: > > cosa * cos2a * cos4a * cos8a * ... * cos[(2^(n-1))a] , ou seja, > > Produtório(cos((2^(n-1)a, n=1..(n-1)) > Use a relação: senx * cosx = (1/2) * sen2x Assim: P = cosa * cos2a * cos4a * cos8a * ... * cos[(2^(n-1))a] sena * P = sena * cosa * cos2a * cos4a * cos8a * ... * cos[(2^(n-1))a] ==> sena * P = (1/2) sen2a * cos2a * cos4a * cos8a * ... * cos[(2^(n-1))a] ==> sena * P = (1/4) * sen4a * cos4a * cos8a * ... * cos[(2^(n-1))a] ==> ... sena * P = (1/2^(n-1)) * sen[(2^(n-1))a] * cos[(2^(n-1))a] ==> sena * P = (1/2^n) * sen[ (2^n)a ] ==> P = sen[(2^n)a]/[ (2^n) * sena ] Um abraco, Claudio. ========================================================================= Instruções para entrar na lista, sair da lista e usar a lista em http://www.mat.puc-rio.br/~nicolau/olimp/obm-l.html O administrador desta lista é ========================================================================= From owner-obm-l@sucuri.mat.puc-rio.br Fri Apr 11 16:29:18 2003 Return-Path: Received: (from majordom@localhost) by sucuri.mat.puc-rio.br (8.9.3/8.9.3) id QAA20567 for obm-l-MTTP; Fri, 11 Apr 2003 16:26:22 -0300 Received: from Euler.impa.br (euler.impa.br [147.65.1.3]) by sucuri.mat.puc-rio.br (8.9.3/8.9.3) with ESMTP id QAA20559 for ; Fri, 11 Apr 2003 16:26:17 -0300 Received: from Gauss.impa.br (Gauss [147.65.4.1]) by Euler.impa.br (8.11.6p2/8.11.6) with ESMTP id h3BJPj016349 for ; Fri, 11 Apr 2003 16:25:45 -0300 (EST) From: Carlos Gustavo Tamm de Araujo Moreira Received: by Gauss.impa.br (8.11.6p2) id h3BJPfW03075; Fri, 11 Apr 2003 16:25:41 -0300 (EST) Message-Id: <200304111925.h3BJPfW03075@Gauss.impa.br> Subject: Re: [obm-l] FW: Teoria dos grupos To: obm-l@mat.puc-rio.br Date: Fri, 11 Apr 2003 16:25:40 -0300 (EST) In-Reply-To: from "Claudio Buffara" at Apr 10, 3 06:41:14 pm X-Mailer: ELM [version 2.4 PL25] MIME-Version: 1.0 Content-Type: text/plain; charset=US-ASCII Content-Transfer-Encoding: 7bit Sender: owner-obm-l@sucuri.mat.puc-rio.br Precedence: bulk Reply-To: obm-l@mat.puc-rio.br Caro Claudio, Acho que isso nao esta' certo. Por exemplo, f(2)=2 para todo automorfismo de Z/4Z, pois 2 e' o unico elemento de ordem 2. Abracos, Gugu > > >Caros colegas da lista: > >Um problema de teoria dos grupos: > >Seja G um grupo cuja ordem eh diferente de 2. >Seja a um elemento de G tal que f(a) = a para todo automorfismo f:G -> G. >Prove que a = identidade de G. > >O resultado eh extremamente razoavel mas eu nao estou conseguindo prova-lo. > >Agradeco qualquer ajuda. > >Um abraco, >Claudio. > >========================================================================= >Instruções para entrar na lista, sair da lista e usar a lista em >http://www.mat.puc-rio.br/~nicolau/olimp/obm-l.html >O administrador desta lista é >========================================================================= ========================================================================= Instruções para entrar na lista, sair da lista e usar a lista em http://www.mat.puc-rio.br/~nicolau/olimp/obm-l.html O administrador desta lista é ========================================================================= From owner-obm-l@sucuri.mat.puc-rio.br Fri Apr 11 17:02:20 2003 Return-Path: Received: (from majordom@localhost) by sucuri.mat.puc-rio.br (8.9.3/8.9.3) id QAA21544 for obm-l-MTTP; Fri, 11 Apr 2003 16:59:40 -0300 Received: from web12908.mail.yahoo.com (web12908.mail.yahoo.com [216.136.174.75]) by sucuri.mat.puc-rio.br (8.9.3/8.9.3) with SMTP id QAA21535 for ; Fri, 11 Apr 2003 16:59:34 -0300 Message-ID: <20030411195903.94651.qmail@web12908.mail.yahoo.com> Received: from [200.206.103.3] by web12908.mail.yahoo.com via HTTP; Fri, 11 Apr 2003 16:59:03 ART Date: Fri, 11 Apr 2003 16:59:03 -0300 (ART) From: =?iso-8859-1?q?Johann=20Peter=20Gustav=20Lejeune=20Dirichlet?= Subject: Re: [obm-l] Sequências To: obm-l@mat.puc-rio.br In-Reply-To: MIME-Version: 1.0 Content-Type: multipart/alternative; boundary="0-777945100-1050091143=:92973" Content-Transfer-Encoding: 8bit Sender: owner-obm-l@sucuri.mat.puc-rio.br Precedence: bulk Reply-To: obm-l@mat.puc-rio.br --0-777945100-1050091143=:92973 Content-Type: text/plain; charset=iso-8859-1 Content-Transfer-Encoding: 8bit Antonio Neto wrote: A fonte e Arial simples ;) >From: Igor GomeZZ >Reply-To: obm-l@mat.puc-rio.br >To: OBM >Subject: [obm-l] Trigonometria e Sequências >Date: Fri, 11 Apr 2003 01:38:05 -0300 > > > Fala galera da lista, boa noite... São dois problemas, um OBM e o >outro >não sei a fonte: > >**Sequência: > >1*2 + 2*3 + 3*4 + 4*5 +...+47*48 + 48*49 + 49*50 > >Se não me engano ela eh OBM, certo? Consegui resolvê-la como uma >Progressão Aritmética de segunda ordem. Para achar o polinômio >que gera os termos t(n) eh relativamente demorado, jah para achar o >polinômio que define a soma s(n) eh ainda mais demorado. Tem alguma coisa >na cara que facilite a questão e não estou vendo? > Hah uma identidade, que eh C(p, p) + C(p+1, p) + C8p+2, p) +...+C(n, p)= C(n+1, p+1). No seu caso, chamando a soma de S, vem S/2 = C(2, 2)+C(3, 2)+C(4, 2)+...+C(50, 2)= C(51, 3)= 20825. Se nao errei nada, eh isso, abracos, olavo. _________________________________________________________________ STOP MORE SPAM with the new MSN 8 and get 2 months FREE* http://join.msn.com/?page=features/junkmail ========================================================================= Instruções para entrar na lista, sair da lista e usar a lista em http://www.mat.puc-rio.br/~nicolau/olimp/obm-l.html O administrador desta lista é ========================================================================= --------------------------------- Yahoo! Mail O melhor e-mail gratuito da internet: 6MB de espaço, antivírus, acesso POP3, filtro contra spam. --0-777945100-1050091143=:92973 Content-Type: text/html; charset=iso-8859-1 Content-Transfer-Encoding: 8bit

 

 Antonio Neto <osneto@hotmail.com> wrote:



A fonte e Arial simples ;)




>From: Igor GomeZZ
>Reply-To: obm-l@mat.puc-rio.br
>To: OBM
>Subject: [obm-l] Trigonometria e Sequências
>Date: Fri, 11 Apr 2003 01:38:05 -0300
>
>
> Fala galera da lista, boa noite... São dois problemas, um OBM e o
>outro
>não sei a fonte:
>
>**Sequência:
>
>1*2 + 2*3 + 3*4 + 4*5 +...+47*48 + 48*49 + 49*50
>
>Se não me engano ela eh OBM, certo? Consegui resolvê-la como uma
>Progressão Aritmética de segunda ordem. Para achar o polinômio
>que gera os termos t(n) eh relativamente demorado, jah para achar o
>polinômio que define a soma s(n) eh ainda mais demorado. Tem alguma coisa
>na cara que facilite a questão e não estou vendo?
>
Hah uma identidade, que eh C(p, p) + C(p+1, p) + C8p+2, p) +...+C(n, p)=
C(n+1, p+1). No seu caso, chamando a soma de S, vem S/2 = C(2, 2)+C(3,
2)+C(4, 2)+...+C(50, 2)= C(51, 3)= 20825. Se nao errei nada, eh isso,
abracos, olavo.


_________________________________________________________________
STOP
MORE SPAM with the new MSN 8 and get 2 months FREE*
http://join.msn.com/?page=features/junkmail

=========================================================================
Instruções para entrar na lista, sair da lista e usar a lista em
http://www.mat.puc-rio.br/~nicolau/olimp/obm-l.html
O administrador desta lista é
=========================================================================



Yahoo! Mail
O melhor e-mail gratuito da internet: 6MB de espaço, antivírus, acesso POP3, filtro contra spam. --0-777945100-1050091143=:92973-- ========================================================================= Instruções para entrar na lista, sair da lista e usar a lista em http://www.mat.puc-rio.br/~nicolau/olimp/obm-l.html O administrador desta lista é ========================================================================= From owner-obm-l@sucuri.mat.puc-rio.br Fri Apr 11 17:03:07 2003 Return-Path: Received: (from majordom@localhost) by sucuri.mat.puc-rio.br (8.9.3/8.9.3) id RAA21599 for obm-l-MTTP; Fri, 11 Apr 2003 17:00:34 -0300 Received: from web12901.mail.yahoo.com (web12901.mail.yahoo.com [216.136.174.68]) by sucuri.mat.puc-rio.br (8.9.3/8.9.3) with SMTP id RAA21595 for ; Fri, 11 Apr 2003 17:00:30 -0300 Message-ID: <20030411195959.95561.qmail@web12901.mail.yahoo.com> Received: from [200.206.103.3] by web12901.mail.yahoo.com via HTTP; Fri, 11 Apr 2003 16:59:59 ART Date: Fri, 11 Apr 2003 16:59:59 -0300 (ART) From: =?iso-8859-1?q?Johann=20Peter=20Gustav=20Lejeune=20Dirichlet?= Subject: Re: [obm-l] Re:_[obm-l]_Trigonometria_e_Sequências To: obm-l@mat.puc-rio.br In-Reply-To: <00ba01c3005e$56faaae0$3300c57d@bovespa.com> MIME-Version: 1.0 Content-Type: multipart/alternative; boundary="0-865200952-1050091199=:95177" Content-Transfer-Encoding: 8bit Sender: owner-obm-l@sucuri.mat.puc-rio.br Precedence: bulk Reply-To: obm-l@mat.puc-rio.br --0-865200952-1050091199=:95177 Content-Type: text/plain; charset=iso-8859-1 Content-Transfer-Encoding: 8bit Truque veio..... Cláudio_(Prática) wrote: ----- Original Message ----- From: "Igor GomeZZ" To: "OBM" Sent: Friday, April 11, 2003 1:38 AM Subject: [obm-l] Trigonometria e Sequências > > Fala galera da lista, boa noite... São dois problemas, um OBM e o outro > não sei a fonte: > > **Sequência: > > 1*2 + 2*3 + 3*4 + 4*5 +...+47*48 + 48*49 + 49*50 > > Se não me engano ela eh OBM, certo? Consegui resolvê-la como uma > Progressão Aritmética de segunda ordem. Para achar o polinômio > que gera os termos t(n) eh relativamente demorado, jah para achar o > polinômio que define a soma s(n) eh ainda mais demorado. Tem alguma coisa > na cara que facilite a questão e não estou vendo? > > S = 1*2 + 2*3 + 3*4 + 4*5 +...+47*48 + 48*49 + 49*50 Voce pode dividir a soma por 2 e obter: S/2 = C(2,2) + C(3,2) + ... + C(50,2) onde C(m,p) = numero de subconjuntos de p elementos de um conjunto com m elementos. Ai, usando uma propriedade do Triangulo de Pascal, chegar a conclusao de que: S/2 = C(51,3) ==> S = 2*C(51,3) = 2*51*50*49/6 = 41.650 *************** > ** Trigonometria: > > cosa * cos2a * cos4a * cos8a * ... * cos[(2^(n-1))a] , ou seja, > > Produtório(cos((2^(n-1)a, n=1..(n-1)) > Use a relação: senx * cosx = (1/2) * sen2x Assim: P = cosa * cos2a * cos4a * cos8a * ... * cos[(2^(n-1))a] sena * P = sena * cosa * cos2a * cos4a * cos8a * ... * cos[(2^(n-1))a] ==> sena * P = (1/2) sen2a * cos2a * cos4a * cos8a * ... * cos[(2^(n-1))a] ==> sena * P = (1/4) * sen4a * cos4a * cos8a * ... * cos[(2^(n-1))a] ==> ... sena * P = (1/2^(n-1)) * sen[(2^(n-1))a] * cos[(2^(n-1))a] ==> sena * P = (1/2^n) * sen[ (2^n)a ] ==> P = sen[(2^n)a]/[ (2^n) * sena ] Um abraco, Claudio. ========================================================================= Instruções para entrar na lista, sair da lista e usar a lista em http://www.mat.puc-rio.br/~nicolau/olimp/obm-l.html O administrador desta lista é ========================================================================= --------------------------------- Yahoo! Mail O melhor e-mail gratuito da internet: 6MB de espaço, antivírus, acesso POP3, filtro contra spam. --0-865200952-1050091199=:95177 Content-Type: text/html; charset=iso-8859-1 Content-Transfer-Encoding: 8bit

Truque veio.....

 Cláudio_(Prática) <claudio@praticacorretora.com.br> wrote:


----- Original Message -----
From: "Igor GomeZZ"
To: "OBM"
Sent: Friday, April 11, 2003 1:38 AM
Subject: [obm-l] Trigonometria e Sequências


>
> Fala galera da lista, boa noite... São dois problemas, um OBM e o
outro
> não sei a fonte:
>
> **Sequência:
>
> 1*2 + 2*3 + 3*4 + 4*5 +...+47*48 + 48*49 + 49*50
>
> Se não me engano ela eh OBM, certo? Consegui resolvê-la como uma
> Progressão Aritmética de segunda ordem. Para achar o polinômio
> que gera os termos t(n) eh relativamente demorado, jah para achar o
> polinômio que define a soma s(n) eh ainda mais demorado. Tem alguma coisa
> na cara que facilite a questão e não estou vendo?
>
>
S = 1*2 + 2*3 + 3*4 + 4*5 +...+47*48 + 48*49 + 49*50

Voce pode dividir a soma por 2 e obter:
S/2 = C(2,2) + C(3,2) + ... + C(50,2)
onde C(m,p) = numero de subconjuntos de p elementos de um conjunto com m
elementos.

Ai, usando uma propriedade do Triangulo de Pascal, chegar a conclusao de
que:
S/2 = C(51,3) ==> S = 2*C(51,3) = 2*51*50*49/6 = 41.650

***************

> ** Trigonometria:
>
> cosa * cos2a * cos4a * cos8a * ... * cos[(2^(n-1))a] , ou seja,
>
> Produtório(cos((2^(n-1)a, n=1..(n-1))
>
Use a relação: senx * cosx = (1/2) * sen2x

Assim:
P = cosa * cos2a * cos4a * cos8a * ... * cos[(2^(n-1))a]
sena * P = sena * cosa * cos2a * cos4a * cos8a * ... * cos[(2^(n-1))a] ==>
sena * P = (1/2) sen2a * cos2a * cos4a * cos8a * ... * cos[(2^(n-1))a] ==>
sena * P = (1/4) * sen4a * cos4a * cos8a * ... * cos[(2^(n-1))a] ==>
...
sena * P = (1/2^(n-1)) * sen[(2^(n-1))a] * cos[(2^(n-1))a] ==>
sena * P = (1/2^n) * sen[ (2^n)a ] ==>
P = sen[(2^n)a]/[ (2^n) * sena ]

Um abraco,
Claudio.

=========================================================================
Instruções para entrar na lista, sair da lista e usar a lista em
http://www.mat.puc-rio.br/~nicolau/olimp/obm-l.html
O administrador desta lista é
=========================================================================



Yahoo! Mail
O melhor e-mail gratuito da internet: 6MB de espaço, antivírus, acesso POP3, filtro contra spam. --0-865200952-1050091199=:95177-- ========================================================================= Instruções para entrar na lista, sair da lista e usar a lista em http://www.mat.puc-rio.br/~nicolau/olimp/obm-l.html O administrador desta lista é ========================================================================= From owner-obm-l@sucuri.mat.puc-rio.br Fri Apr 11 17:35:57 2003 Return-Path: Received: (from majordom@localhost) by sucuri.mat.puc-rio.br (8.9.3/8.9.3) id RAA22815 for obm-l-MTTP; Fri, 11 Apr 2003 17:32:59 -0300 Received: from ivoti.terra.com.br (ivoti.terra.com.br [200.176.3.20]) by sucuri.mat.puc-rio.br (8.9.3/8.9.3) with ESMTP id RAA22811 for ; Fri, 11 Apr 2003 17:32:56 -0300 Received: from itaim.terra.com.br (itaim.terra.com.br [200.176.3.76]) by ivoti.terra.com.br (Postfix) with ESMTP id D9B144090E6 for ; Fri, 11 Apr 2003 17:32:23 -0300 (BRT) Received: from nt (RJ231057.user.veloxzone.com.br [200.165.231.57]) (authenticated user ensr) by itaim.terra.com.br (Postfix) with ESMTP id 4AE382E00C2 for ; Fri, 11 Apr 2003 17:32:23 -0300 (BRT) Message-ID: <010a01c30069$5ccfe380$5400a8c0@ensrbr> From: "Luis Lopes" To: References: Subject: [obm-l] =?iso-8859-1?Q?Re:_=5Bobm-l=5D_Sequ=EAncias?= Date: Fri, 11 Apr 2003 17:31:41 -0300 MIME-Version: 1.0 Content-Type: text/plain; charset="iso-8859-1" Content-Transfer-Encoding: 8bit X-Priority: 3 X-MSMail-Priority: Normal X-Mailer: Microsoft Outlook Express 5.50.4807.1700 X-MimeOLE: Produced By Microsoft MimeOLE V5.50.4807.1700 Sender: owner-obm-l@sucuri.mat.puc-rio.br Precedence: bulk Reply-To: obm-l@mat.puc-rio.br Sauda,c~oes, Ha alguns resultados que facilitam estes calculos. O meu preferido eh o seguinte: a seq. eh > >1*2 + 2*3 + 3*4 + 4*5 +...+47*48 + 48*49 + 49*50 ou seja, 2,6,12,20,... 4,6,8 2,2 (PA de 2a. ordem) Vc quer somar S_n^{[k]} = \sum_{i=1}^n a_i, a_i termo geral de PA de ordem k. O resultado geral de S_n^{[k]} para n=49 e k=2 eh S_{49}^{[2]} = 2C(49,3) + 4C(49,2) + 2C(49,1) = 41650. []'s Luís > >From: Igor GomeZZ > >Reply-To: obm-l@mat.puc-rio.br > >To: OBM > >Subject: [obm-l] Trigonometria e Sequências > >Date: Fri, 11 Apr 2003 01:38:05 -0300 > > > > > > Fala galera da lista, boa noite... São dois problemas, um OBM e o > >outro > >não sei a fonte: > > > >**Sequência: > > > >1*2 + 2*3 + 3*4 + 4*5 +...+47*48 + 48*49 + 49*50 > > na cara que facilite a questão e não estou vendo? > > ========================================================================= Instruções para entrar na lista, sair da lista e usar a lista em http://www.mat.puc-rio.br/~nicolau/olimp/obm-l.html O administrador desta lista é ========================================================================= From owner-obm-l@sucuri.mat.puc-rio.br Fri Apr 11 17:54:00 2003 Return-Path: Received: (from majordom@localhost) by sucuri.mat.puc-rio.br (8.9.3/8.9.3) id RAA23353 for obm-l-MTTP; Fri, 11 Apr 2003 17:50:51 -0300 Received: from ivoti.terra.com.br (ivoti.terra.com.br [200.176.3.20]) by sucuri.mat.puc-rio.br (8.9.3/8.9.3) with ESMTP id RAA23349 for ; Fri, 11 Apr 2003 17:50:47 -0300 Received: from itaim.terra.com.br (itaim.terra.com.br [200.176.3.76]) by ivoti.terra.com.br (Postfix) with ESMTP id 72FE6409392 for ; Fri, 11 Apr 2003 17:50:17 -0300 (BRT) Received: from nt (RJ231057.user.veloxzone.com.br [200.165.231.57]) (authenticated user ensr) by itaim.terra.com.br (Postfix) with ESMTP id 946AE2E0099 for ; Fri, 11 Apr 2003 17:50:16 -0300 (BRT) Message-ID: <012a01c3006b$dcae0a80$5400a8c0@ensrbr> From: "Luis Lopes" To: References: <6732305773.20030411013805@gmx.net> <00ba01c3005e$56faaae0$3300c57d@bovespa.com> Subject: [obm-l] =?iso-8859-1?Q?Re:_=5Bobm-l=5D_Re:_=5Bobm-l=5D_Trigonometria_e_Sequ=EAnci?= =?iso-8859-1?Q?as?= Date: Fri, 11 Apr 2003 17:49:35 -0300 MIME-Version: 1.0 Content-Type: text/plain; charset="iso-8859-1" Content-Transfer-Encoding: 8bit X-Priority: 3 X-MSMail-Priority: Normal X-Mailer: Microsoft Outlook Express 5.50.4807.1700 X-MimeOLE: Produced By Microsoft MimeOLE V5.50.4807.1700 Sender: owner-obm-l@sucuri.mat.puc-rio.br Precedence: bulk Reply-To: obm-l@mat.puc-rio.br Sauda,c~oes, Na mensagem do Claudio Buffara sobre aparece o site Na internet voce tambem encontra alguns sites com problemas interessantes, tais como este aqui: http://math.stanford.edu/~vakil/stanfordputnam/02/putnam3.pdf Proponho dois problemas tirados do site: A5. Sendo \cos(\theta) = 1 / \pi , calcule \sum_{n=0}^\infty \cos(n\theta) / 2^n . B2. Para n >= 2, mostre que (produtório) \sin(\pi / n) \sin(2\pi / n) ..... \sin[(n - 1)\pi / n] = n / 2^{n-1} . []'s Luís ========================================================================= Instruções para entrar na lista, sair da lista e usar a lista em http://www.mat.puc-rio.br/~nicolau/olimp/obm-l.html O administrador desta lista é ========================================================================= From owner-obm-l@sucuri.mat.puc-rio.br Fri Apr 11 18:27:10 2003 Return-Path: Received: (from majordom@localhost) by sucuri.mat.puc-rio.br (8.9.3/8.9.3) id SAA24499 for obm-l-MTTP; Fri, 11 Apr 2003 18:24:30 -0300 Received: from ivoti.terra.com.br (ivoti.terra.com.br [200.176.3.20]) by sucuri.mat.puc-rio.br (8.9.3/8.9.3) with ESMTP id SAA24495 for ; Fri, 11 Apr 2003 18:24:26 -0300 Received: from itaim.terra.com.br (itaim.terra.com.br [200.176.3.76]) by ivoti.terra.com.br (Postfix) with ESMTP id 695B740933C for ; Fri, 11 Apr 2003 18:23:56 -0300 (BRT) Received: from nt (RJ231057.user.veloxzone.com.br [200.165.231.57]) (authenticated user ensr) by itaim.terra.com.br (Postfix) with ESMTP id 0D2F22E007A for ; Fri, 11 Apr 2003 18:23:56 -0300 (BRT) Message-ID: <01e601c30070$907f39e0$5400a8c0@ensrbr> From: "Luis Lopes" To: Subject: [obm-l] centro de semelhanca Date: Fri, 11 Apr 2003 18:23:15 -0300 MIME-Version: 1.0 Content-Type: text/plain; charset="iso-8859-1" Content-Transfer-Encoding: 8bit X-Priority: 3 X-MSMail-Priority: Normal X-Mailer: Microsoft Outlook Express 5.50.4807.1700 X-MimeOLE: Produced By Microsoft MimeOLE V5.50.4807.1700 Sender: owner-obm-l@sucuri.mat.puc-rio.br Precedence: bulk Reply-To: obm-l@mat.puc-rio.br Sauda,c~oes, Mensagem recebida de uma outra lista: Some months ago, I wrote: >> I have read the following geometry problem marked >> with "Iran, 1997": >> Let ABC be a triangle and P a varying point on the arc >> BC of the circumcircle of ABC. Prove that the circle >> through P and the incenters of triangles PAB and PAC >> passes through a fixed point independent of P. > Later Jean-Pierre Ehrmann identfied this point as the > intersection of the line AX(56) with the circumcircle, > where X(56) is the external center of similtude of > circumcircle and incircle. > Remarkable: If P lies on the arc CA or on the arc AB, > then the circle through P and the incenters of triangles > PAB and PAC passes through A ! > Darij Grinberg O que eh o centro externo de semelhanca? E o interno? Como determiná-los? []'s Luís ========================================================================= Instruções para entrar na lista, sair da lista e usar a lista em http://www.mat.puc-rio.br/~nicolau/olimp/obm-l.html O administrador desta lista é ========================================================================= From owner-obm-l@sucuri.mat.puc-rio.br Fri Apr 11 18:43:41 2003 Return-Path: Received: (from majordom@localhost) by sucuri.mat.puc-rio.br (8.9.3/8.9.3) id SAA25032 for obm-l-MTTP; Fri, 11 Apr 2003 18:41:00 -0300 Received: from ivoti.terra.com.br (ivoti.terra.com.br [200.176.3.20]) by sucuri.mat.puc-rio.br (8.9.3/8.9.3) with ESMTP id SAA25028 for ; Fri, 11 Apr 2003 18:40:56 -0300 Received: from bertioga.terra.com.br (bertioga.terra.com.br [200.176.3.77]) by ivoti.terra.com.br (Postfix) with ESMTP id 4A66440896D for ; Fri, 11 Apr 2003 18:40:26 -0300 (BRT) Received: from [200.177.190.10] (dl-nas5-sao-C8B1BE0A.p001.terra.com.br [200.177.190.10]) by bertioga.terra.com.br (Postfix) with ESMTP id 67E6E3F8058 for ; Fri, 11 Apr 2003 18:40:25 -0300 (BRT) User-Agent: Microsoft-Outlook-Express-Macintosh-Edition/5.02.2022 Date: Fri, 11 Apr 2003 18:38:34 -0300 Subject: Re: [obm-l] FW: Teoria dos grupos From: Claudio Buffara To: Message-ID: In-Reply-To: <200304111925.h3BJPfW03075@Gauss.impa.br> Mime-version: 1.0 Content-type: text/plain; charset="US-ASCII" Content-transfer-encoding: 7bit Sender: owner-obm-l@sucuri.mat.puc-rio.br Precedence: bulk Reply-To: obm-l@mat.puc-rio.br Oi, Gugu: Voce tem toda a razao. Eu me esqueci de mencionar uma outra hipotese crucial: todos os elementos de G tem ordem <= 2. Este problema apareceu quando eu tentava provar, usando teoria dos grupos, uma generalizacao do teorema de Wilson: Se n eh um inteiro > 2, entao o produto de todos os Phi(n) invertiveis (mod n) eh igual a: -1, se existe uma raiz primitiva mod n ou +1, se nao existem raizes primitivas mod n. Obrigado e um abraco, Claudio. on 11.04.03 16:25, Carlos Gustavo Tamm de Araujo Moreira at gugu@impa.br wrote: > Caro Claudio, > Acho que isso nao esta' certo. Por exemplo, f(2)=2 para todo > automorfismo de Z/4Z, pois 2 e' o unico elemento de ordem 2. > Abracos, > Gugu > >> >> >> Caros colegas da lista: >> >> Um problema de teoria dos grupos: >> >> Seja G um grupo cuja ordem eh diferente de 2. >> Seja a um elemento de G tal que f(a) = a para todo automorfismo f:G -> G. >> Prove que a = identidade de G. >> >> O resultado eh extremamente razoavel mas eu nao estou conseguindo prova-lo. >> >> Agradeco qualquer ajuda. >> >> Um abraco, >> Claudio. >> ========================================================================= Instruções para entrar na lista, sair da lista e usar a lista em http://www.mat.puc-rio.br/~nicolau/olimp/obm-l.html O administrador desta lista é ========================================================================= From owner-obm-l@sucuri.mat.puc-rio.br Fri Apr 11 18:48:40 2003 Return-Path: Received: (from majordom@localhost) by sucuri.mat.puc-rio.br (8.9.3/8.9.3) id SAA25194 for obm-l-MTTP; Fri, 11 Apr 2003 18:45:37 -0300 Received: from ivoti.terra.com.br (ivoti.terra.com.br [200.176.3.20]) by sucuri.mat.puc-rio.br (8.9.3/8.9.3) with ESMTP id SAA25190 for ; Fri, 11 Apr 2003 18:45:33 -0300 Received: from altamira.terra.com.br (altamira.terra.com.br [200.176.3.40]) by ivoti.terra.com.br (Postfix) with ESMTP id E36F2409A21 for ; Fri, 11 Apr 2003 18:45:02 -0300 (BRT) Received: from niski.com (unknown [200.148.206.4]) (authenticated user fniski) by altamira.terra.com.br (Postfix) with ESMTP id 936173DC0CA for ; Fri, 11 Apr 2003 18:45:01 -0300 (BRT) Message-ID: <3E973761.8040705@niski.com> Date: Fri, 11 Apr 2003 14:45:05 -0700 From: niski User-Agent: Mozilla/5.0 (Windows; U; Windows NT 5.1; en-US; rv:1.0.2) Gecko/20030208 Netscape/7.02 X-Accept-Language: en-us, en MIME-Version: 1.0 To: obm-l@mat.puc-rio.br Subject: [obm-l] Demonstracao - matrizes Content-Type: text/plain; charset=ISO-8859-1; format=flowed Content-Transfer-Encoding: 8bit Sender: owner-obm-l@sucuri.mat.puc-rio.br Precedence: bulk Reply-To: obm-l@mat.puc-rio.br Olá pessoal. Essa mensagem não é uma duvida extritamente sobre matrizes e sim como fazer certas demonstracoes. Gostaria que lessem minha demonstracao e se ela realmente convence o leitor e se é a maneira comum de provar coisas do tipo. Obrigado Por exemplo "Mostre que se A tem uma linha de zeros e B é uma matriz qualquer para qual o produto AB esta definido, entao AB tambem tem uma linha de zeros" Começei assim a demonstracao: Sejam A[mXp] = (a[ij]) , B[pXn] = (b[jl]) e AB = C , com C = (c[il]) Da definição do produto de matrizes : c[il] = a[i1]b[1l] + a[i2]b[2l] + a[i3]b[3l] + ... + a[ij]b[jl] para todo i E {1,2,3...m} e todo l E { 1,2,3...n} Seja a p-ésima linha de A a que contem apenas zeros. Sendo assima soma c[pl] = a[p1]b[1l] + a[p2]b[2l] + ... + a[pj]b[jl] para todo l E { 1,2,3,...n} Sempre será zero fazendo com que a matriz C tenha uma linha de zeros. Fabio Niski www.linux.ime.usp.br/~niski -- [about him:] It is rare to find learned men who are clean, do not stink and have a sense of humour. -Gottfried Whilhem Leibniz ========================================================================= Instruções para entrar na lista, sair da lista e usar a lista em http://www.mat.puc-rio.br/~nicolau/olimp/obm-l.html O administrador desta lista é ========================================================================= From owner-obm-l@sucuri.mat.puc-rio.br Fri Apr 11 20:10:30 2003 Return-Path: Received: (from majordom@localhost) by sucuri.mat.puc-rio.br (8.9.3/8.9.3) id UAA26963 for obm-l-MTTP; Fri, 11 Apr 2003 20:07:33 -0300 Received: from web41509.mail.yahoo.com (web41509.mail.yahoo.com [66.218.93.92]) by sucuri.mat.puc-rio.br (8.9.3/8.9.3) with SMTP id UAA26959 for ; Fri, 11 Apr 2003 20:07:29 -0300 Message-ID: <20030411230657.82083.qmail@web41509.mail.yahoo.com> Received: from [200.207.158.20] by web41509.mail.yahoo.com via HTTP; Fri, 11 Apr 2003 16:06:56 PDT Date: Fri, 11 Apr 2003 16:06:56 -0700 (PDT) From: Carlos Yuzo Shine Subject: Re: [obm-l] Re:_[obm-l]_Re:_[obm-l]_Trigonometria_e_Sequências To: obm-l@mat.puc-rio.br In-Reply-To: <012a01c3006b$dcae0a80$5400a8c0@ensrbr> MIME-Version: 1.0 Content-Type: text/plain; charset=us-ascii Sender: owner-obm-l@sucuri.mat.puc-rio.br Precedence: bulk Reply-To: obm-l@mat.puc-rio.br Ambos os problemas podem ser resolvidos usando o fato de que cos x = (e^(ix) + e^(-ix))/2 sen x = (e^(ix) - e^(-ix))/(2i) (x em radianos) Para ver isso, verifique as expansões em polinômio de Taylor de e^x, sen x e cos x e verifique que e^(ix) = cos x + i*sen x e^(-ix) = cos x - i*sen x Veja que com isso o problema A5 vira uma soma de duas progressões geométricas. O problema B2 usa os fatos acima e a fatoração z^n - 1 = (z - 1)(z - w)(z - w^2)...(z - w^(n-1)), em que w = e^(2\pi/n) é uma raiz n-ésima primitiva da unidade. > A5. Sendo \cos(\theta) = 1 / \pi , calcule \sum_{n=0}^\infty \cos(n\theta) / 2^n . > B2. Para n >= 2, mostre que (produtório) > \sin(\pi / n) \sin(2\pi / n) ..... \sin[(n - 1)\pi / > n] = n / 2^{n-1} . > > []'s > Luís __________________________________________________ Do you Yahoo!? Yahoo! Tax Center - File online, calculators, forms, and more http://tax.yahoo.com ========================================================================= Instruções para entrar na lista, sair da lista e usar a lista em http://www.mat.puc-rio.br/~nicolau/olimp/obm-l.html O administrador desta lista é ========================================================================= From owner-obm-l@sucuri.mat.puc-rio.br Fri Apr 11 21:05:13 2003 Return-Path: Received: (from majordom@localhost) by sucuri.mat.puc-rio.br (8.9.3/8.9.3) id VAA27799 for obm-l-MTTP; Fri, 11 Apr 2003 21:02:25 -0300 Received: from itaqui.terra.com.br (itaqui.terra.com.br [200.176.3.19]) by sucuri.mat.puc-rio.br (8.9.3/8.9.3) with ESMTP id VAA27795 for ; Fri, 11 Apr 2003 21:02:22 -0300 Received: from bertioga.terra.com.br (bertioga.terra.com.br [200.176.3.77]) by itaqui.terra.com.br (Postfix) with ESMTP id B2A813BD98F for ; Fri, 11 Apr 2003 21:01:51 -0300 (BRT) Received: from [200.177.179.12] (dl-nas3-sao-C8B1B30C.p001.terra.com.br [200.177.179.12]) by bertioga.terra.com.br (Postfix) with ESMTP id F1D943F8042 for ; Fri, 11 Apr 2003 21:01:50 -0300 (BRT) User-Agent: Microsoft-Outlook-Express-Macintosh-Edition/5.02.2022 Date: Fri, 11 Apr 2003 20:59:56 -0300 Subject: Re: [obm-l] Re: [obm-l] Re: [obm-l] Trigonometria e Sequ =?ISO-8859-1?B?6g==?=ncias From: Claudio Buffara To: Message-ID: In-Reply-To: <012a01c3006b$dcae0a80$5400a8c0@ensrbr> Mime-version: 1.0 Content-type: text/plain; charset="ISO-8859-1" Content-Transfer-Encoding: 8bit X-MIME-Autoconverted: from quoted-printable to 8bit by sucuri.mat.puc-rio.br id VAA27796 Sender: owner-obm-l@sucuri.mat.puc-rio.br Precedence: bulk Reply-To: obm-l@mat.puc-rio.br Oi, Luis: O primeiro eh calcular S = SOMA(n>=0) cos(n*x)/2^n com cos(x) = 1/Pi Considere a soma SOMA(n>=0) (e^(ix)/2)^n = Soma da PG infinita com 1o. termo 1 e razao e^(ix)/2 = = 1/(1 - e^(ix)/2) (a convergencia deve-se ao fato de que |razao| = 1/2 < 1) 1/(1 - e^(ix)/2) = 2/[2 - e^(ix)] = = 2/[2 - cos(x) - i*sen(x)] = 2*[2 - cos(x) + i*sen(x)]/[(2 - cos(x))^2 + sen^2(x)] A parte real da expressao acima eh a soma que nos interessa (S): S = 2*[2 - cos(x)]/[(2 - cos(x))^2 + sen^2(x)] = = 2*[2 - cos(x)]/[5 - 4*cos(x)] cos(x) = 1/Pi ==> S = 2*(2 - 1/Pi)/(5 - 4/Pi) ==> S = 2*(2*Pi - 1)/(5*Pi - 4). Um abraco, Claudio. on 11.04.03 17:49, Luis Lopes at llopes@ensrbr.com.br wrote: > Sauda,c~oes, > > Na mensagem do Claudio Buffara sobre > aparece > o site > > Na internet voce tambem encontra alguns sites com problemas interessantes, > tais como este aqui: > http://math.stanford.edu/~vakil/stanfordputnam/02/putnam3.pdf > > Proponho dois problemas tirados do site: > > A5. Sendo \cos(\theta) = 1 / \pi , calcule > \sum_{n=0}^\infty \cos(n\theta) / 2^n . > > B2. Para n >= 2, mostre que (produtório) > \sin(\pi / n) \sin(2\pi / n) ..... \sin[(n - 1)\pi / n] = n / 2^{n-1} . > > []'s > Luís > > > ========================================================================= > Instruções para entrar na lista, sair da lista e usar a lista em > http://www.mat.puc-rio.br/~nicolau/olimp/obm-l.html > O administrador desta lista é > ========================================================================= > ========================================================================= Instruções para entrar na lista, sair da lista e usar a lista em http://www.mat.puc-rio.br/~nicolau/olimp/obm-l.html O administrador desta lista é ========================================================================= From owner-obm-l@sucuri.mat.puc-rio.br Fri Apr 11 22:06:16 2003 Return-Path: Received: (from majordom@localhost) by sucuri.mat.puc-rio.br (8.9.3/8.9.3) id WAA28756 for obm-l-MTTP; Fri, 11 Apr 2003 22:03:47 -0300 Received: from hotmail.com (f37.law12.hotmail.com [64.4.19.37]) by sucuri.mat.puc-rio.br (8.9.3/8.9.3) with ESMTP id WAA28752 for ; Fri, 11 Apr 2003 22:03:43 -0300 Received: from mail pickup service by hotmail.com with Microsoft SMTPSVC; Fri, 11 Apr 2003 18:03:08 -0700 Received: from 200.151.3.252 by lw12fd.law12.hotmail.msn.com with HTTP; Sat, 12 Apr 2003 01:03:08 GMT X-Originating-IP: [200.151.3.252] X-Originating-Email: [felipensador@hotmail.com] From: "felipe mendona" To: obm-l@mat.puc-rio.br Subject: [obm-l] =?iso-8859-1?B?UkU6Z2VuZXJhbGl6YedhbyBkZSBQQSBkZSAyIG9yZGVt?= Date: Fri, 11 Apr 2003 22:03:08 -0300 Mime-Version: 1.0 Content-Type: text/html; charset=iso-8859-1 Message-ID: X-OriginalArrivalTime: 12 Apr 2003 01:03:08.0484 (UTC) FILETIME=[4800C840:01C3008F] Sender: owner-obm-l@sucuri.mat.puc-rio.br Precedence: bulk Reply-To: obm-l@mat.puc-rio.br
       
           
                                  Boa noite colegas da lista.....
 
 
                   Igor, em essencia voce quer uma maneira rapida e alternativa para calcular o somatorio de qualquer progressao  na forma {(a_1). (b_1),(a_2). (b_2),......,(a_n).( b_n)}na qual os termos a_k seguem uma PA de razao r_a ao passo que os termos b_k  segue uma PA de razao r_b para 1<=k<=n.Vejamos isto:  Vamos convencionar que a_1>=b_1.
                 Temos a_k = a_1 + r_a.(k-1) analogamente temos b_k = b_1 + r_b(k-1) logo temos a_k.b_k = a_1.b_1 + (a_1.r_b + b_1.r_a).(k-1) + r_a.r_b.(k-1)^2.Agora  basta somar todos termos na forma a_k.b_k , lembrando que 1<=k<=n.Teremos entao uma soma S = n.a_1.b_1 + [(n-1)+(n-2)+......+2+1].(a_1.r_b+b_1.r_a)+[(n-1)^2 + (n-2)^2 +...........
+2^2+1^2].(r_a.r_b).   Sabemos que 1+2+......+(n-1)=n(n-1)/2  (vou dispensar a prova desse fato) . Precisamos determinar agora f(n)=1^2 +2^2 + .......+(n-1)^2 .Vamos calcular    f(n) pela identidade polinomal g(n+1)-g(n) = x.n^2 + y.n +z tal que g(k)= k^3.Temos entao que (n+1)^3 - n^3 = 3.n^2 + 3.n +1. Temos entao que o somatorio de (k+1)^3 com k variando de  0 a n - somatorio de k^3 com k variando de 0 a n = (n+1)^3 = somatorio de (3.k^2+3.k+1) com k variando de 0 a n = 3.n.(n+1)/2 + n + 1 + 3.[1^2 +2^2 + .........+ (n-1)^2 +n^2] =(n+1)^3            Fazendo as contas teremos 1^2 +2^2 +.........+n^2 = n^3/3 + n^2/2 + n/6  =  n.(n+1).(2n+1)/6 =f(n+1) , portanto f(n) = n.(n-1).(2n-1)/6 = 1^2 + 2^2 +.........+(n-1)^2.
      Voltando ao problema inicial, teremos S = n.(a_1).(b_1) + (a_1.r_b+b_1.r_a).n.(n-1)/2  +(r_a.r_b). n.(n-1).(2.n-1)/6. O que acabei de fazer é uma generalizaçao para qualquer PA de 2 ordem,portanto vale para a soma que voce  propos:1.2 + 2.3 + ........+ 49.50 = 41650
       Por razoes praticas ,resolva estas questoes por meio do triangulo de pascal ou entao por meio da formula deduzida acima.
                       
                              Forte abraço
 
                                               Felipe Mendonça                    Vitória-ES.
 


MSN Hotmail, o maior webmail do Brasil. Faça o seu agora. ========================================================================= Instruções para entrar na lista, sair da lista e usar a lista em http://www.mat.puc-rio.br/~nicolau/olimp/obm-l.html O administrador desta lista é ========================================================================= From owner-obm-l@sucuri.mat.puc-rio.br Fri Apr 11 22:16:33 2003 Return-Path: Received: (from majordom@localhost) by sucuri.mat.puc-rio.br (8.9.3/8.9.3) id WAA28952 for obm-l-MTTP; Fri, 11 Apr 2003 22:13:47 -0300 Received: from Euler.impa.br (euler.impa.br [147.65.1.3]) by sucuri.mat.puc-rio.br (8.9.3/8.9.3) with ESMTP id WAA28948 for ; Fri, 11 Apr 2003 22:13:44 -0300 Received: from Gauss.impa.br (Gauss [147.65.4.1]) by Euler.impa.br (8.11.6p2/8.11.6) with ESMTP id h3C1DD012756 for ; Fri, 11 Apr 2003 22:13:13 -0300 (EST) From: Carlos Gustavo Tamm de Araujo Moreira Received: by Gauss.impa.br (8.11.6p2) id h3C1D9b28692; Fri, 11 Apr 2003 22:13:09 -0300 (EST) Message-Id: <200304120113.h3C1D9b28692@Gauss.impa.br> Subject: Re: [obm-l] FW: Teoria dos grupos To: obm-l@mat.puc-rio.br Date: Fri, 11 Apr 2003 22:13:09 -0300 (EST) In-Reply-To: from "Claudio Buffara" at Apr 11, 3 06:38:34 pm X-Mailer: ELM [version 2.4 PL25] MIME-Version: 1.0 Content-Type: text/plain; charset=US-ASCII Content-Transfer-Encoding: 7bit Sender: owner-obm-l@sucuri.mat.puc-rio.br Precedence: bulk Reply-To: obm-l@mat.puc-rio.br Oi Claudio, A sua hipotese implica que G e' abeliano, e portanto, pelo menos quando G e' finito, ele deve ser isomorfo a (Z/2Z)^n para algum n natural, e nesse caso o resultado segue facilmente, pois, se n>1, podemos permutar os fatores de (Z/2Z)^n e obter isomorfismos que nao fixam um dado elemento de ordem 2. Para ver que G e' abeliano, devemos mostrar que para quaisquer x e y, xy.x^(-1).y^(-1)=e (o elemento neutro), mas isso equivale a xyxy=e, mas xyxy=(xy)^2=e (pois todo elemento e' o inverso de si mesmo). Um outro jeito de provar essa generalizacao do teorema de Wilson e' o seguinte: no produto de todos os invertiveis modulo n aparecem varios pares a, a^(-1) que se cancelam, e sobram apenas os a com a^2=1 (mod n). se existe raiz primitiva modulo n existem apenas um valor de a diferente de 1 com a^2=1: a=g^(phi(n)/2)=-1, onde g e' uma raiz primitiva modulo n, e portanto o produto dos invertiveis e' um modulo n. No caso em que nao existe raiz primitiva modulo n, n=2^k com k>=3 ou n=bc, com b e c primos entre si e maiores que 2. Note que se a^2=1 entao (-a)^2=1, e esses a vem aos pares. Note que a.(-a)=-a^2=-1, donde nosso produto e' (-1)^(N/2), onde N e' o numero de tais a. Se n=2^k, com k>=3, temos N=|{a|a^2=1(mod 2^k)}|= =|{r.2(k-1)+e,r=0,1, e=-1,1}|=4. Se n=bc, com b e c primos entre si e maiores que 2, pelo teorema chines dos restos, o numero de tais a (mod n) e' o produto do numero de tais a (mod b) pelo numero de tais a (mod c), que e' sempre multiplo de 4, pois como vimos esses a vem sempre aos pares. Em qualquer caso N e' multiplo de 4 e nosso produto e' 1. Abracos, Gugu > >Oi, Gugu: > >Voce tem toda a razao. Eu me esqueci de mencionar uma outra hipotese >crucial: todos os elementos de G tem ordem <= 2. > >Este problema apareceu quando eu tentava provar, usando teoria dos grupos, >uma generalizacao do teorema de Wilson: >Se n eh um inteiro > 2, entao o produto de todos os Phi(n) invertiveis (mod >n) eh igual a: >-1, se existe uma raiz primitiva mod n >ou >+1, se nao existem raizes primitivas mod n. > >Obrigado e um abraco, >Claudio. > > > >on 11.04.03 16:25, Carlos Gustavo Tamm de Araujo Moreira at gugu@impa.br >wrote: > >> Caro Claudio, >> Acho que isso nao esta' certo. Por exemplo, f(2)=2 para todo >> automorfismo de Z/4Z, pois 2 e' o unico elemento de ordem 2. >> Abracos, >> Gugu >> >>> >>> >>> Caros colegas da lista: >>> >>> Um problema de teoria dos grupos: >>> >>> Seja G um grupo cuja ordem eh diferente de 2. >>> Seja a um elemento de G tal que f(a) = a para todo automorfismo f:G -> G. >>> Prove que a = identidade de G. >>> >>> O resultado eh extremamente razoavel mas eu nao estou conseguindo prova-lo. >>> >>> Agradeco qualquer ajuda. >>> >>> Um abraco, >>> Claudio. >>> > >========================================================================= >Instruções para entrar na lista, sair da lista e usar a lista em >http://www.mat.puc-rio.br/~nicolau/olimp/obm-l.html >O administrador desta lista é >========================================================================= ========================================================================= Instruções para entrar na lista, sair da lista e usar a lista em http://www.mat.puc-rio.br/~nicolau/olimp/obm-l.html O administrador desta lista é ========================================================================= From owner-obm-l@sucuri.mat.puc-rio.br Sat Apr 12 00:05:17 2003 Return-Path: Received: (from majordom@localhost) by sucuri.mat.puc-rio.br (8.9.3/8.9.3) id XAA30717 for obm-l-MTTP; Fri, 11 Apr 2003 23:59:35 -0300 Received: from ivoti.terra.com.br (ivoti.terra.com.br [200.176.3.20]) by sucuri.mat.puc-rio.br (8.9.3/8.9.3) with ESMTP id XAA30713 for ; Fri, 11 Apr 2003 23:59:31 -0300 Received: from bertioga.terra.com.br (bertioga.terra.com.br [200.176.3.77]) by ivoti.terra.com.br (Postfix) with ESMTP id 92CAE409ACF for ; Fri, 11 Apr 2003 23:59:01 -0300 (BRT) Received: from [200.177.182.218] (dl-nas6-sao-C8B1B6DA.p001.terra.com.br [200.177.182.218]) by bertioga.terra.com.br (Postfix) with ESMTP id 7AE953F8033 for ; Fri, 11 Apr 2003 23:59:00 -0300 (BRT) User-Agent: Microsoft-Outlook-Express-Macintosh-Edition/5.02.2022 Date: Fri, 11 Apr 2003 23:57:49 -0300 Subject: Re: [obm-l] FW: Teoria dos grupos From: Claudio Buffara To: Message-ID: In-Reply-To: <200304120113.h3C1D9b28692@Gauss.impa.br> Mime-version: 1.0 Content-type: text/plain; charset="US-ASCII" Content-transfer-encoding: 7bit Sender: owner-obm-l@sucuri.mat.puc-rio.br Precedence: bulk Reply-To: obm-l@mat.puc-rio.br Gugu: Mais uma vez muitissimo obrigado. Um grande abraco, Claudio. on 11.04.03 22:13, Carlos Gustavo Tamm de Araujo Moreira at gugu@impa.br wrote: > Oi Claudio, > A sua hipotese implica que G e' abeliano, e portanto, pelo menos quando G > e' finito, ele deve ser isomorfo a (Z/2Z)^n para algum n natural, e nesse > caso o resultado segue facilmente, pois, se n>1, podemos permutar os fatores > de (Z/2Z)^n e obter isomorfismos que nao fixam um dado elemento de ordem 2. > Para ver que G e' abeliano, devemos mostrar que para quaisquer x e y, > xy.x^(-1).y^(-1)=e (o elemento neutro), mas isso equivale a xyxy=e, mas > xyxy=(xy)^2=e (pois todo elemento e' o inverso de si mesmo). > Um outro jeito de provar essa generalizacao do teorema de Wilson e' o > seguinte: no produto de todos os invertiveis modulo n aparecem varios pares > a, a^(-1) que se cancelam, e sobram apenas os a com a^2=1 (mod n). se existe > raiz primitiva modulo n existem apenas um valor de a diferente de 1 com > a^2=1: a=g^(phi(n)/2)=-1, onde g e' uma raiz primitiva modulo n, e portanto > o produto dos invertiveis e' um modulo n. No caso em que nao existe raiz > primitiva modulo n, n=2^k com k>=3 ou n=bc, com b e c primos entre si e > maiores que 2. Note que se a^2=1 entao (-a)^2=1, e esses a vem aos pares. > Note que a.(-a)=-a^2=-1, donde nosso produto e' (-1)^(N/2), onde N e' o > numero de tais a. Se n=2^k, com k>=3, temos N=|{a|a^2=1(mod 2^k)}|= > =|{r.2(k-1)+e,r=0,1, e=-1,1}|=4. Se n=bc, com b e c primos entre si e > maiores que 2, pelo teorema chines dos restos, o numero de tais a (mod n) e' > o produto do numero de tais a (mod b) pelo numero de tais a (mod c), que e' > sempre multiplo de 4, pois como vimos esses a vem sempre aos pares. Em > qualquer caso N e' multiplo de 4 e nosso produto e' 1. > Abracos, > Gugu > > >> >> Oi, Gugu: >> >> Voce tem toda a razao. Eu me esqueci de mencionar uma outra hipotese >> crucial: todos os elementos de G tem ordem <= 2. >> >> Este problema apareceu quando eu tentava provar, usando teoria dos grupos, >> uma generalizacao do teorema de Wilson: >> Se n eh um inteiro > 2, entao o produto de todos os Phi(n) invertiveis (mod >> n) eh igual a: >> -1, se existe uma raiz primitiva mod n >> ou >> +1, se nao existem raizes primitivas mod n. >> >> Obrigado e um abraco, >> Claudio. >> >> >> >> on 11.04.03 16:25, Carlos Gustavo Tamm de Araujo Moreira at gugu@impa.br >> wrote: >> >>> Caro Claudio, >>> Acho que isso nao esta' certo. Por exemplo, f(2)=2 para todo >>> automorfismo de Z/4Z, pois 2 e' o unico elemento de ordem 2. >>> Abracos, >>> Gugu >>> >>>> >>>> >>>> Caros colegas da lista: >>>> >>>> Um problema de teoria dos grupos: >>>> >>>> Seja G um grupo cuja ordem eh diferente de 2. >>>> Seja a um elemento de G tal que f(a) = a para todo automorfismo f:G -> G. >>>> Prove que a = identidade de G. >>>> >>>> O resultado eh extremamente razoavel mas eu nao estou conseguindo prova-lo. >>>> >>>> Agradeco qualquer ajuda. >>>> >>>> Um abraco, >>>> Claudio. >>>> >> >> ========================================================================= >> Instru??es para entrar na lista, sair da lista e usar a lista em >> http://www.mat.puc-rio.br/~nicolau/olimp/obm-l.html >> O administrador desta lista ? >> ========================================================================= > > ========================================================================= > Instru??es para entrar na lista, sair da lista e usar a lista em > http://www.mat.puc-rio.br/~nicolau/olimp/obm-l.html > O administrador desta lista ? > ========================================================================= > ========================================================================= Instruções para entrar na lista, sair da lista e usar a lista em http://www.mat.puc-rio.br/~nicolau/olimp/obm-l.html O administrador desta lista é ========================================================================= From owner-obm-l@sucuri.mat.puc-rio.br Sat Apr 12 00:05:17 2003 Return-Path: Received: (from majordom@localhost) by sucuri.mat.puc-rio.br (8.9.3/8.9.3) id XAA30709 for obm-l-MTTP; Fri, 11 Apr 2003 23:59:26 -0300 Received: from ivoti.terra.com.br (ivoti.terra.com.br [200.176.3.20]) by sucuri.mat.puc-rio.br (8.9.3/8.9.3) with ESMTP id XAA30705 for ; Fri, 11 Apr 2003 23:59:22 -0300 Received: from bertioga.terra.com.br (bertioga.terra.com.br [200.176.3.77]) by ivoti.terra.com.br (Postfix) with ESMTP id 48B40409892 for ; Fri, 11 Apr 2003 23:58:52 -0300 (BRT) Received: from [200.177.182.218] (dl-nas6-sao-C8B1B6DA.p001.terra.com.br [200.177.182.218]) by bertioga.terra.com.br (Postfix) with ESMTP id 2BCAE3F803E for ; Fri, 11 Apr 2003 23:58:51 -0300 (BRT) User-Agent: Microsoft-Outlook-Express-Macintosh-Edition/5.02.2022 Date: Fri, 11 Apr 2003 23:57:00 -0300 Subject: Re: [obm-l] Re:_[obm-l]_Re:_[obm-l]_Trigonometria_e_Sequ =?ISO-8859-1?B?6g==?=ncias From: Claudio Buffara To: Message-ID: In-Reply-To: <20030411230657.82083.qmail@web41509.mail.yahoo.com> Mime-version: 1.0 Content-type: text/plain; charset="US-ASCII" Content-transfer-encoding: 7bit Sender: owner-obm-l@sucuri.mat.puc-rio.br Precedence: bulk Reply-To: obm-l@mat.puc-rio.br So pra complementar o que o Shine escreveu: Estas duas tecnicas: expressar sen(x) e cos(x) em funcao de exp(ix) e exp(-ix) e fatorar x^n - 1 sao suficientes pra se resolver a grande maioria de problemas envolvendo somas e produtos de senos e cossenos. ********* No caso especifico do 2o. problema proposto pelo Luis, as fatoracoes de x^n - 1 que sao relevantes sao as seguintes: x^n - 1 = (x - 1)*p(x) Forma 1: p(x) = x^(n-1) + x^(n-2) + ... + x + 1 (basta usar a formula da soma dos termos de uma PG) Forma 2: Se n eh par: p(x) = (x + 1) * PRODUTO(1<=k<=(n-2)/2) (x^2 - 2*cos(2*k*Pi/n)*x + 1) Se n eh impar: p(x) = PRODUTO(1<=k<=(n-1)/2) (x^2 - 2*cos(2*k*Pi/n)*x + 1) (neste caso, use a fatoracao em termos lineares que o Shine menciona e agrupe os pares de fatores cujas raizes sao complexas conjugadas) ---------- A primeira forma implica que p(1) = n, o que nao parece tao inusitado ateh compararmos esta forma com a fatoracao do Shine: p(x) = (x - w)(x - w^2)...(x - w^(n-1)) ==> (1 - w)(1 - w^2)...(1 - w^(n-1)) = n. Pra pensar: Qual a interpretacao geometrica desta identidade? (dica: pense na disposicao geometrica das raizes n-esimas da unidade no plano complexo e no que o modulo de cada fator da lado esquerdo significa). ----------- A segunda forma resulta em: Se n eh par: p(1) = 2 * PRODUTO(1<=k<=(n-2)/2) (1 - 2*cos(2*k*Pi/n) + 1) ==> p(1) = 2^(n/2) * PRODUTO(1<=k<=(n-2)/2) (1 - cos(2*k*Pi/n)) ==> (usando a identidade 1 - cos(2*x) = 2*sen^2(x) ) p(1) = 2^(n-1) * PRODUTO(1<=k<=(n-2)/2) sen^2(k*Pi/n) ----------- Se n eh impar: p(1) = PRODUTO(1<=k<=(n-1)/2) (1 - 2*cos(2*k*Pi/n) + 1) ==> p(1) = 2^((n-1)/2) * PRODUTO(1<=k<=(n-1)/2) (1 - cos(2*k*Pi/n)) ==> p(1) = 2^(n-1) * PRODUTO(1<=k<=(n-1)/2) sen^2(k*Pi/n) ------------ Agora, levando em conta que sen(Pi/2) = 1 e que, para 0 <= k <= n: sen(k*Pi/n) = sen((n-k)*Pi/n), teremos: Para n par: PRODUTO(1<=k<=(n-2)/2) sen^2(k*pi/n) = = sen((n/2)*Pi/n) * PRODUTO(1<=k<=(n-2)/2) sen(k*Pi/n)*sen((n-k)*Pi/n) = = PRODUTO(1<=k<=n-1) sen(k*Pi/n) Para n impar: PRODUTO(1<=k<=(n-1)/2) sen^2(k*Pi/n) = = PRODUTO(1<=k<=(n-1)/2) sen(k*Pi/n)*sen((n-k)*Pi/n) = = PRODUTO(1<=k<=n-1) sen(k*Pi/n) Assim, de qualquer jeito, teremos: p(1) = 2^(n-1) * PRODUTO(1<=k<=n-1) sen(k*Pi/n) Lembrando que p(1) = n, acabou. Teremos PRODUTO(1<=k<=n-1) sen(k*Pi/n) = n/2^(n-1). Um abraco, Claudio. on 11.04.03 20:06, Carlos Yuzo Shine at cyshine@yahoo.com wrote: > Ambos os problemas podem ser resolvidos usando o fato > de que > cos x = (e^(ix) + e^(-ix))/2 > sen x = (e^(ix) - e^(-ix))/(2i) > (x em radianos) > > Para ver isso, verifique as expans?es em polin?mio de > Taylor de e^x, sen x e cos x e verifique que > e^(ix) = cos x + i*sen x > e^(-ix) = cos x - i*sen x > > Veja que com isso o problema A5 vira uma soma de duas > progress?es geom?tricas. > > O problema B2 usa os fatos acima e a fatora??o > z^n - 1 = (z - 1)(z - w)(z - w^2)...(z - w^(n-1)), > em que w = e^(2\pi/n) ? uma raiz n-?sima primitiva da > unidade. > >> A5. Sendo \cos(\theta) = 1 / \pi , calcule > \sum_{n=0}^\infty \cos(n\theta) / 2^n . > >> B2. Para n >= 2, mostre que (produt?rio) >> \sin(\pi / n) \sin(2\pi / n) ..... \sin[(n - 1)\pi / >> n] = n / 2^{n-1} . >> >> []'s >> Lu?s > ========================================================================= Instruções para entrar na lista, sair da lista e usar a lista em http://www.mat.puc-rio.br/~nicolau/olimp/obm-l.html O administrador desta lista é ========================================================================= From owner-obm-l@sucuri.mat.puc-rio.br Sat Apr 12 00:16:18 2003 Return-Path: Received: (from majordom@localhost) by sucuri.mat.puc-rio.br (8.9.3/8.9.3) id AAA31042 for obm-l-MTTP; Sat, 12 Apr 2003 00:13:41 -0300 Received: from paiol.terra.com.br (paiol.terra.com.br [200.176.3.18]) by sucuri.mat.puc-rio.br (8.9.3/8.9.3) with ESMTP id AAA31034 for ; Sat, 12 Apr 2003 00:13:37 -0300 Received: from itaim.terra.com.br (itaim.terra.com.br [200.176.3.76]) by paiol.terra.com.br (Postfix) with ESMTP id ABDB287D94 for ; Sat, 12 Apr 2003 00:13:08 -0300 (BRT) Received: from [200.177.186.108] (dl-nas4-sao-C8B1BA6C.p001.terra.com.br [200.177.186.108]) by itaim.terra.com.br (Postfix) with ESMTP id D3AB52E0049 for ; Sat, 12 Apr 2003 00:13:07 -0300 (BRT) User-Agent: Microsoft-Outlook-Express-Macintosh-Edition/5.02.2022 Date: Sat, 12 Apr 2003 00:11:17 -0300 Subject: Re: [obm-l] Demonstracao - matrizes From: Claudio Buffara To: Message-ID: In-Reply-To: <3E973761.8040705@niski.com> Mime-version: 1.0 Content-type: text/plain; charset="ISO-8859-1" Content-Transfer-Encoding: 8bit X-MIME-Autoconverted: from quoted-printable to 8bit by sucuri.mat.puc-rio.br id AAA31036 Sender: owner-obm-l@sucuri.mat.puc-rio.br Precedence: bulk Reply-To: obm-l@mat.puc-rio.br Oi, Niski: Sua demonstracao estah perfeita. Da mesma forma voce mostra que se B tem uma coluna de zeros, entao AB tem uma coluna de zeros. Um abraco, Claudio. on 11.04.03 18:45, niski at fabio@niski.com wrote: > Olá pessoal. Essa mensagem não é uma duvida extritamente sobre matrizes > e sim como fazer certas demonstracoes. Gostaria que lessem minha > demonstracao e se ela realmente convence o leitor e se é a maneira comum > de provar coisas do tipo. Obrigado > > Por exemplo > "Mostre que se A tem uma linha de zeros e B é uma matriz qualquer para > qual o produto AB esta definido, entao AB tambem tem uma linha de zeros" > > Começei assim a demonstracao: > Sejam A[mXp] = (a[ij]) , B[pXn] = (b[jl]) e AB = C , com C = (c[il]) > Da definição do produto de matrizes : > c[il] = a[i1]b[1l] + a[i2]b[2l] + a[i3]b[3l] + ... + a[ij]b[jl] > para todo i E {1,2,3...m} e todo l E { 1,2,3...n} > > Seja a p-ésima linha de A a que contem apenas zeros. > Sendo assima soma > c[pl] = a[p1]b[1l] + a[p2]b[2l] + ... + a[pj]b[jl] > para todo l E { 1,2,3,...n} > Sempre será zero fazendo com que a matriz C tenha uma linha de zeros. > > Fabio Niski > www.linux.ime.usp.br/~niski ========================================================================= Instruções para entrar na lista, sair da lista e usar a lista em http://www.mat.puc-rio.br/~nicolau/olimp/obm-l.html O administrador desta lista é ========================================================================= From owner-obm-l@sucuri.mat.puc-rio.br Sat Apr 12 00:40:18 2003 Return-Path: Received: (from majordom@localhost) by sucuri.mat.puc-rio.br (8.9.3/8.9.3) id AAA32292 for obm-l-MTTP; Sat, 12 Apr 2003 00:37:43 -0300 Received: from ivoti.terra.com.br (ivoti.terra.com.br [200.176.3.20]) by sucuri.mat.puc-rio.br (8.9.3/8.9.3) with ESMTP id AAA32287 for ; Sat, 12 Apr 2003 00:37:40 -0300 Received: from botucatu.terra.com.br (botucatu.terra.com.br [200.176.3.78]) by ivoti.terra.com.br (Postfix) with ESMTP id 0DE0F409381 for ; Sat, 12 Apr 2003 00:37:10 -0300 (BRT) Received: from [200.177.186.175] (dl-nas4-sao-C8B1BAAF.p001.terra.com.br [200.177.186.175]) by botucatu.terra.com.br (Postfix) with ESMTP id 326E629C074 for ; Sat, 12 Apr 2003 00:37:09 -0300 (BRT) User-Agent: Microsoft-Outlook-Express-Macintosh-Edition/5.02.2022 Date: Sat, 12 Apr 2003 00:34:56 -0300 Subject: Re: [obm-l] centro de semelhanca From: Claudio Buffara To: Message-ID: In-Reply-To: <01e601c30070$907f39e0$5400a8c0@ensrbr> Mime-version: 1.0 Content-type: text/plain; charset="ISO-8859-1" Content-Transfer-Encoding: 8bit X-MIME-Autoconverted: from quoted-printable to 8bit by sucuri.mat.puc-rio.br id AAA32288 Sender: owner-obm-l@sucuri.mat.puc-rio.br Precedence: bulk Reply-To: obm-l@mat.puc-rio.br on 11.04.03 18:23, Luis Lopes at llopes@ensrbr.com.br wrote: > Sauda,c~oes, > > Mensagem recebida de uma outra lista: > > Some months ago, I wrote: > >>> I have read the following geometry problem marked >>> with "Iran, 1997": > >>> Let ABC be a triangle and P a varying point on the arc >>> BC of the circumcircle of ABC. Prove that the circle >>> through P and the incenters of triangles PAB and PAC >>> passes through a fixed point independent of P. > >> Later Jean-Pierre Ehrmann identfied this point as the >> intersection of the line AX(56) with the circumcircle, >> where X(56) is the external center of similtude of >> circumcircle and incircle. > >> Remarkable: If P lies on the arc CA or on the arc AB, >> then the circle through P and the incenters of triangles >> PAB and PAC passes through A ! > >> Darij Grinberg > > O que eh o centro externo de semelhanca? E o interno? > Como determiná-los? > > []'s > Luís > Oi, Luis: Acho que esta eh uma pergunta pro Eduardo Wagner. De qualquer forma, me ocorreu um outro problema envolvendo incirculos e circuncirculos: Dados dois circulos C1 e C2, tais que C1 estah contido no interior de C2, construa um triangulo cujo incirculo seja C1 e o circuncirculo C2. Este triangulo eh unico? Quais as condicoes para que a construcao seja possivel? Por exemplo, se C1 e C2 forem concentricos e os respectivos raios estiverem na razao 1:2, existira uma infinidade de triangulos equilateros (todos congruentes) inscritos em C2 e circunscritos a C1. Um abraco, Claudio. ========================================================================= Instruções para entrar na lista, sair da lista e usar a lista em http://www.mat.puc-rio.br/~nicolau/olimp/obm-l.html O administrador desta lista é ========================================================================= From owner-obm-l@sucuri.mat.puc-rio.br Sat Apr 12 00:42:38 2003 Return-Path: Received: (from majordom@localhost) by sucuri.mat.puc-rio.br (8.9.3/8.9.3) id AAA32443 for obm-l-MTTP; Sat, 12 Apr 2003 00:40:06 -0300 Received: from smtp-26.ig.com.br (smtp-26.ig.com.br [200.226.132.160]) by sucuri.mat.puc-rio.br (8.9.3/8.9.3) with SMTP id AAA32438 for ; Sat, 12 Apr 2003 00:40:03 -0300 Received: (qmail 15011 invoked from network); 12 Apr 2003 03:39:31 -0000 Received: from unknown (HELO xxxx) (200.165.170.117) by smtp-26.ig.com.br with SMTP; 12 Apr 2003 03:39:31 -0000 Message-ID: <000b01c300a5$b9d73b00$75aaa5c8@epq.ime.eb.br> From: "Marcio" To: References: Subject: Re: [obm-l] Fibonacci Date: Sat, 12 Apr 2003 00:43:47 -0300 MIME-Version: 1.0 Content-Type: text/plain; charset="iso-8859-1" Content-Transfer-Encoding: 7bit X-Priority: 3 X-MSMail-Priority: Normal X-Mailer: Microsoft Outlook Express 5.50.4133.2400 X-MimeOLE: Produced By Microsoft MimeOLE V5.50.4133.2400 Sender: owner-obm-l@sucuri.mat.puc-rio.br Precedence: bulk Reply-To: obm-l@mat.puc-rio.br Oi Claudio! Desculpe a demora na resposta. Assim que li seu email fui conferir minha solucao.. E descobri que ela esta mesmo errada.. Fiquei tentando consertar, mas nao deu (alias, foi mas por teimosia, pq o que vc falou faz total sentido.. se a irracionalidade nao foi decidida ateh 80, eh razoavel que o resultado nao se escreva de maneira mto simples com constantes conhecidas). Como esse eh um exercicio proposto num livro que eu tenho aqui em casa, achei que fosse soh mais um exercicio de rotina, e nao um problema conhecido e dificil.. O que eu tinha feito era olhar para o inverso de F_n*sqrt(5) como 1/(a^n + b^n), onde a,b sao... e ab=-1 donde F_n*sqrt(5) = b^n/(b^2n + 1) para n par, com - para n impar. E ai eu encarava esse ultimo numero como a soma de uma PG e depois fazia uma contagem dupla nas potencias de b (isso se justifica pelo fato da serie ser absolutamente convergente, o que segue por exemplo de 1/|F_n*sqrt(5)| = 1/|a^n-b^n|<2/(a^n) que eh uma serie convergente pois |1/a|<1 (ou entao seguindo o que vc e o yuri responderam em emails passados). O problema eh que eu burramente escrevi o o 1/(b^2n + 1) como 1-b+b^2-b^3+... e nao como uma PG de razao b^2n como deveria ser.. Sendo assim, eu tmb gostaria muito de ver a minha solucao :) Abracos, Marcio PS: O livro em questao eh o Engel - Solving problem strategies.. La ele deixa esse exercicio como proposto, sem mostrar a solucao.. Pensando agora, acho provavel que ele quisesse dizer F_2^n ao inves de F_n.. ----- Original Message ----- From: "Claudio Buffara" To: Sent: Wednesday, April 09, 2003 11:54 PM Subject: Re: [obm-l] Fibonacci > Marcio: > > Dei uma pesquisada na internet e parece a prova da irracionalidade da soma > dos reciprocos dos numeros de Fibonacci foi um dos problemas propostos por > Paul Erdos e que soh foi resolvido na decada de 1980. > > Assim, o valor exato desta soma nao deve poder ser expresso como uma > combinacao de constantes conhecidas (Pi, e, raiz(5), etc.). > > Sendo assim, eu gostaria muito de ver a sua solucao. ========================================================================= Instruções para entrar na lista, sair da lista e usar a lista em http://www.mat.puc-rio.br/~nicolau/olimp/obm-l.html O administrador desta lista é ========================================================================= From owner-obm-l@sucuri.mat.puc-rio.br Sat Apr 12 01:50:27 2003 Return-Path: Received: (from majordom@localhost) by sucuri.mat.puc-rio.br (8.9.3/8.9.3) id BAA01834 for obm-l-MTTP; Sat, 12 Apr 2003 01:47:39 -0300 Received: from mail.gmx.net (mail.gmx.net [213.165.65.60]) by sucuri.mat.puc-rio.br (8.9.3/8.9.3) with SMTP id BAA01830 for ; Sat, 12 Apr 2003 01:47:34 -0300 Received: (qmail 1510 invoked by uid 65534); 12 Apr 2003 04:46:48 -0000 Received: from unknown (EHLO localhost) (200.217.15.94) by mail.gmx.net (mp008-rz3) with SMTP; 12 Apr 2003 06:46:48 +0200 Date: Sat, 12 Apr 2003 01:28:51 -0300 From: Igor GomeZZ X-Mailer: The Bat! (v1.61) Organization: -- X-Priority: 3 (Normal) Message-ID: <394616367.20030412012851@gmx.net> To: =?ISO-8859-1?B?IkNs4XVkaW8gKFBy4XRpY2EpIg==?= Subject: [obm-l] =?ISO-8859-1?B?UmU6IFtvYm0tbF0gUmU6IFtvYm0tbF0gVHJpZ29ub21ldHJpYSBlIFNl?= =?ISO-8859-1?B?cXXqbmNpYXM=?= In-Reply-To: <00ba01c3005e$56faaae0$3300c57d@bovespa.com> References: <6732305773.20030411013805@gmx.net> <00ba01c3005e$56faaae0$3300c57d@bovespa.com> MIME-Version: 1.0 Content-Type: text/plain; charset=ISO-8859-1 Content-Transfer-Encoding: 8bit Sender: owner-obm-l@sucuri.mat.puc-rio.br Precedence: bulk Reply-To: obm-l@mat.puc-rio.br Em 11/4/2003, 16:12, Cláudio (claudio@praticacorretora.com.br) disse: > S = 1*2 + 2*3 + 3*4 + 4*5 +...+47*48 + 48*49 + 49*50 > Ai, usando uma propriedade do Triangulo de Pascal, chegar a conclusao de S/2 = C(51,3) ==>> S = 2*C(51,3) = 2*51*50*49/6 = 41.650 --------- > sena * P = (1/4) * sen4a * cos4a * cos8a * ... * cos[(2^(n-1))a] ==> > sena * P = (1/2^(n-1)) * sen[(2^(n-1))a] * cos[(2^(n-1))a] ==> > sena * P = (1/2^n) * sen[ (2^n)a ] ==> > P = sen[(2^n)a]/[ (2^n) * sena ] Claudio, muito obrigado pelas duas soluções... A primeira foi falta de pensar mais um pouco, mas a sua resolução da segunda questão eu achei fantástica :-) Com uma multiplicação simples vc fez um "estrago"... Muito boa! > Um abraco, > Claudio. Valeu! Fui! ####### Igor GomeZZ ######## UIN: 29249895 Vitória, Espírito Santo, Brasil Criação: 12/4/2003 (01:15) #################################### Pare para pensar: Que Deus me proteja dos meus amigos. Dos inimigos, cuido eu. (Voltaire) #################################### ========================================================================= Instruções para entrar na lista, sair da lista e usar a lista em http://www.mat.puc-rio.br/~nicolau/olimp/obm-l.html O administrador desta lista é ========================================================================= From owner-obm-l@sucuri.mat.puc-rio.br Sat Apr 12 01:50:36 2003 Return-Path: Received: (from majordom@localhost) by sucuri.mat.puc-rio.br (8.9.3/8.9.3) id BAA01856 for obm-l-MTTP; Sat, 12 Apr 2003 01:48:02 -0300 Received: from mail.gmx.net (mail.gmx.net [213.165.65.60]) by sucuri.mat.puc-rio.br (8.9.3/8.9.3) with SMTP id BAA01852 for ; Sat, 12 Apr 2003 01:47:57 -0300 Received: (qmail 2274 invoked by uid 65534); 12 Apr 2003 04:47:17 -0000 Received: from unknown (EHLO localhost) (200.217.15.94) by mail.gmx.net (mp008-rz3) with SMTP; 12 Apr 2003 06:47:17 +0200 Date: Sat, 12 Apr 2003 01:29:11 -0300 From: Igor GomeZZ X-Mailer: The Bat! (v1.61) Organization: -- X-Priority: 3 (Normal) Message-ID: <804636106.20030412012911@gmx.net> To: Antonio Neto Subject: [obm-l] =?ISO-8859-1?B?UmU6IFtvYm0tbF0gU2VxdepuY2lhcw==?= In-Reply-To: References: MIME-Version: 1.0 Content-Type: text/plain; charset=ISO-8859-1 Content-Transfer-Encoding: 8bit Sender: owner-obm-l@sucuri.mat.puc-rio.br Precedence: bulk Reply-To: obm-l@mat.puc-rio.br Em 11/4/2003, 15:35, Antonio (osneto@hotmail.com) disse: > Hah uma identidade, que eh C(p, p) + C(p+1, p) + C8p+2, p) +...+C(n, p)= > C(n+1, p+1). No seu caso, chamando a soma de S, vem S/2 = C(2, 2)+C(3, > 2)+C(4, 2)+...+C(50, 2)= C(51, 3)= 20825. Se nao errei nada, eh isso, > abracos, olavo. Olavo, valeu por relembrar-me de Pascal... Um belo uso! :-) Sua resolução tah perfeita, soh faltou multiplicar pelo dois do primeiro membro, mas tah tranquilo... Muito obrigado! Fui ####### Igor GomeZZ ######## UIN: 29249895 Vitória, Espírito Santo, Brasil Criação: 12/4/2003 (01:25) #################################### Pare para pensar: Quando todos estão contra você quer dizer que você está absolutamente errado, ou absolutamente certo. (Albert Guinon) #################################### ========================================================================= Instruções para entrar na lista, sair da lista e usar a lista em http://www.mat.puc-rio.br/~nicolau/olimp/obm-l.html O administrador desta lista é ========================================================================= From owner-obm-l@sucuri.mat.puc-rio.br Sat Apr 12 01:51:10 2003 Return-Path: Received: (from majordom@localhost) by sucuri.mat.puc-rio.br (8.9.3/8.9.3) id BAA01866 for obm-l-MTTP; Sat, 12 Apr 2003 01:48:39 -0300 Received: from mail.gmx.net (mail.gmx.net [213.165.65.60]) by sucuri.mat.puc-rio.br (8.9.3/8.9.3) with SMTP id BAA01862 for ; Sat, 12 Apr 2003 01:48:34 -0300 Received: (qmail 2915 invoked by uid 65534); 12 Apr 2003 04:47:51 -0000 Received: from unknown (EHLO localhost) (200.217.15.94) by mail.gmx.net (mp008-rz3) with SMTP; 12 Apr 2003 06:47:51 +0200 Date: Sat, 12 Apr 2003 01:29:20 -0300 From: Igor GomeZZ X-Mailer: The Bat! (v1.61) Organization: -- X-Priority: 3 (Normal) Message-ID: <1624644969.20030412012920@gmx.net> To: Johann Peter Gustav Lejeune Dirichlet Subject: [obm-l] =?ISO-8859-1?B?UmVbMl06IFtvYm0tbF0gU2VxdepuY2lhcw==?= In-Reply-To: <20030411195903.94651.qmail@web12908.mail.yahoo.com> References: <20030411195903.94651.qmail@web12908.mail.yahoo.com> MIME-Version: 1.0 Content-Type: text/plain; charset=ISO-8859-1 Content-Transfer-Encoding: 8bit Sender: owner-obm-l@sucuri.mat.puc-rio.br Precedence: bulk Reply-To: obm-l@mat.puc-rio.br Em 11/4/2003, 16:59, Johann (peterdirichlet2002@yahoo.com.br) disse: > A fonte e Arial simples ;) eehehehehe > Truque veio..... Velho eh relativo :-) Fui! ####### Igor GomeZZ ######## UIN: 29249895 Vitória, Espírito Santo, Brasil Criação: 12/4/2003 (01:27) #################################### Pare para pensar: De todos os animais selvagens, o homem jovem é o mais difícil de domar. (Platão) #################################### ========================================================================= Instruções para entrar na lista, sair da lista e usar a lista em http://www.mat.puc-rio.br/~nicolau/olimp/obm-l.html O administrador desta lista é ========================================================================= From owner-obm-l@sucuri.mat.puc-rio.br Sat Apr 12 01:51:39 2003 Return-Path: Received: (from majordom@localhost) by sucuri.mat.puc-rio.br (8.9.3/8.9.3) id BAA01876 for obm-l-MTTP; Sat, 12 Apr 2003 01:49:08 -0300 Received: from mail.gmx.net (mail.gmx.net [213.165.65.60]) by sucuri.mat.puc-rio.br (8.9.3/8.9.3) with SMTP id BAA01872 for ; Sat, 12 Apr 2003 01:49:03 -0300 Received: (qmail 3635 invoked by uid 65534); 12 Apr 2003 04:48:19 -0000 Received: from unknown (EHLO localhost) (200.217.15.94) by mail.gmx.net (mp008-rz3) with SMTP; 12 Apr 2003 06:48:19 +0200 Date: Sat, 12 Apr 2003 01:38:16 -0300 From: Igor GomeZZ X-Mailer: The Bat! (v1.61) Organization: -- X-Priority: 3 (Normal) Message-ID: <595180649.20030412013816@gmx.net> To: Luis Lopes Subject: [obm-l] =?ISO-8859-1?B?UmU6IFtvYm0tbF0gUmU6IFtvYm0tbF0gU2VxdepuY2lhcw==?= In-Reply-To: <010a01c30069$5ccfe380$5400a8c0@ensrbr> References: <010a01c30069$5ccfe380$5400a8c0@ensrbr> MIME-Version: 1.0 Content-Type: text/plain; charset=ISO-8859-1 Content-Transfer-Encoding: 8bit Sender: owner-obm-l@sucuri.mat.puc-rio.br Precedence: bulk Reply-To: obm-l@mat.puc-rio.br Em 11/4/2003, 17:31, Luis (llopes@ensrbr.com.br) disse: > Sauda,c~oes, Fala Luís! > Ha alguns resultados que facilitam estes > calculos. > Vc quer somar S_n^{[k]} = \sum_{i=1}^n a_i, > a_i termo geral de PA de ordem k. > O resultado geral de S_n^{[k]} para n=49 e k=2 eh > S_{49}^{[2]} = > 2C(49,3) + 4C(49,2) + 2C(49,1) = 41650. Entendi bem por cima, acredito que a notação Latex pra quem não estah familiarizado dificulta :-) Pelo que entendi, eh uma forma de fazer a soma de uma PA de ordem k, sabendo-se apenas o termo geral sem precisar calcular o polinômio (grau k+1) que define a soma, correto? Vc poderia mostrar melhor sua resolução? > []'s > Luís Valeuz Luís! Fui! ####### Igor GomeZZ ######## UIN: 29249895 Vitória, Espírito Santo, Brasil Criação: 12/4/2003 (01:30) #################################### Pare para pensar: Algo é só impossível até que alguém duvide e acabe provando o contrário. (Albert Einstein) #################################### ========================================================================= Instruções para entrar na lista, sair da lista e usar a lista em http://www.mat.puc-rio.br/~nicolau/olimp/obm-l.html O administrador desta lista é ========================================================================= From owner-obm-l@sucuri.mat.puc-rio.br Sat Apr 12 02:55:08 2003 Return-Path: Received: (from majordom@localhost) by sucuri.mat.puc-rio.br (8.9.3/8.9.3) id CAA04379 for obm-l-MTTP; Sat, 12 Apr 2003 02:51:27 -0300 Received: from Euler.impa.br (euler.impa.br [147.65.1.3]) by sucuri.mat.puc-rio.br (8.9.3/8.9.3) with ESMTP id CAA04375 for ; Sat, 12 Apr 2003 02:51:23 -0300 Received: from Gauss.impa.br (Gauss [147.65.4.1]) by Euler.impa.br (8.11.6p2/8.11.6) with ESMTP id h3C5or024133 for ; Sat, 12 Apr 2003 02:50:53 -0300 (EST) From: Carlos Gustavo Tamm de Araujo Moreira Received: by Gauss.impa.br (8.11.6p2) id h3C5on517751; Sat, 12 Apr 2003 02:50:49 -0300 (EST) Message-Id: <200304120550.h3C5on517751@Gauss.impa.br> Subject: Re: [obm-l] Problemas em Aberto To: obm-l@mat.puc-rio.br Date: Sat, 12 Apr 2003 02:50:48 -0300 (EST) In-Reply-To: <044b01c2de8a$b27413c0$3300c57d@bovespa.com> from "=?Windows-1252?Q?Cl=E1udio_\=28Pr=E1tica\=29?=" at Feb 27, 3 03:04:34 pm X-Mailer: ELM [version 2.4 PL25] MIME-Version: 1.0 Content-Type: text/plain; charset=US-ASCII Content-Transfer-Encoding: 7bit Sender: owner-obm-l@sucuri.mat.puc-rio.br Precedence: bulk Reply-To: obm-l@mat.puc-rio.br > >6. D=EA um exemplo de uma sequ=EAncia (Xn) de n=FAmeros reais tal que:=20 > >lim ( Xn / n^t ) =3D 0 para todo t > 0=20 >e >lim ( [log(n)]^k / Xn ) =3D 0 para todo k > 0 > Existem muitas, como X_n=2^(raiz(log(n)), ou X_n=(log(n))^log(log(n)). >********* > >7. Um tri=E2ngulo tem lados com medida inteira e =E1rea racional. Prove = >que uma de suas alturas tem medida inteira e que o p=E9 desta altura = >est=E1 a uma dist=E2ncia inteira dos v=E9rtices do tri=E2ngulo. > Parece que isso nao esta' certo. O triangulo de lados 17, 65 e 80 tem area 288 e alturas 576/17, 576/65 e 36/5, que nao sao inteiras... >********* > >9. Seja K um inteiro >=3D 2.=20 > infinito >Seja S =3D SOMAT=D3RIO 1 / K^(n^2) =3D 1/K + 1/K^4 + 1/K^9 + 1/K^16 + = >... > n =3D 1 >Prove que S =E9 irracional. > Se x=p/q e' racional e r/s e' outro racional diferente de x entao |x-r/s|=|(ps-qr)/qs|>=1/qs, ou seja, s|x-r/s|>=1/q. Por outro lado, soma(n=1 ate' m)(1/K^(n^2)) e' um racional com denominador (divisor de) K^(m^2), digamos p/K^(m^2), e |S-p/K^(m^2)|<(1/K^((m+1)^2))(1+1/2+1/4+...)=2/K^((m+1)^2), mas K^(m^2).2/K^((m+1)^2)=2/K^(2m+1) tende a 0 quando m tende a infinito, e portanto S nao pode ser racional. Abracos, Gugu ========================================================================= Instruções para entrar na lista, sair da lista e usar a lista em http://www.mat.puc-rio.br/~nicolau/olimp/obm-l.html O administrador desta lista é ========================================================================= From owner-obm-l@sucuri.mat.puc-rio.br Sat Apr 12 11:44:12 2003 Return-Path: Received: (from majordom@localhost) by sucuri.mat.puc-rio.br (8.9.3/8.9.3) id LAA12127 for obm-l-MTTP; Sat, 12 Apr 2003 11:38:27 -0300 Received: from ivoti.terra.com.br (ivoti.terra.com.br [200.176.3.20]) by sucuri.mat.puc-rio.br (8.9.3/8.9.3) with ESMTP id LAA12123 for ; Sat, 12 Apr 2003 11:38:23 -0300 Received: from marova.terra.com.br (marova.terra.com.br [200.176.3.39]) by ivoti.terra.com.br (Postfix) with ESMTP id F3DF94086D2 for ; Sat, 12 Apr 2003 11:37:52 -0300 (BRT) Received: from [200.177.182.141] (dl-nas6-sao-C8B1B68D.p001.terra.com.br [200.177.182.141]) by marova.terra.com.br (Postfix) with ESMTP id D47103DC078 for ; Sat, 12 Apr 2003 11:37:51 -0300 (BRT) User-Agent: Microsoft-Outlook-Express-Macintosh-Edition/5.02.2022 Date: Sat, 12 Apr 2003 11:36:00 -0300 Subject: Re: [obm-l] Problemas em Aberto From: Claudio Buffara To: Message-ID: In-Reply-To: <200304120550.h3C5on517751@Gauss.impa.br> Mime-version: 1.0 Content-type: text/plain; charset="US-ASCII" Content-transfer-encoding: 7bit Sender: owner-obm-l@sucuri.mat.puc-rio.br Precedence: bulk Reply-To: obm-l@mat.puc-rio.br Oi, Gugu: O no. 7 foi baseado num problema da Eureka, mas eu acabei de ver onde errei. O problema original eh: "Um triangulo tem os lados de medidas inteiras e area racional. Prove que ele eh congruente a um triangulo cujos vertices tem coordenadas inteiras". Eu assumi, erroneamente, que um dos lados do triangulo de reticulado (essa eh a traducao correta de "lattice triangle"?) poderia ser paralelo a um dos eixos coordenados, de forma que o pe' da altura relativa a este lado teria coordenadas inteiras. O seu contra-exemplo mostra que esta hipotese nao eh valida em geral. Um triangulo de reticulado congruente ao seu teria como vertices: (0,0), (15,8), (48,64) ==> nenhum lado eh paralelo aos eixos. De qualquer forma, fica ai o enunciado do problema original. Ao que me consta, a Eureka ainda nao recebeu uma solucao para este. ***** A solucao que eu tinha imaginado pro no. 9 eh: O numero S, quando expresso na base K eh igual a: 0,100100001000000100000000100..., ou seja, uma K-esimal infinita e nao periodica. Logo, S eh irracional. De qualquer forma, ambas as solucoes so facilmente generalizaveis para o caso de: SOMA(n>=1) R^(n^2), onde R eh um racional entre 0 e 1. Um abraco, Claudio. on 12.04.03 02:50, Carlos Gustavo Tamm de Araujo Moreira at gugu@impa.br wrote: >> >> 6. D=EA um exemplo de uma sequ=EAncia (Xn) de n=FAmeros reais tal que:=20 >> >> lim ( Xn / n^t ) =3D 0 para todo t > 0=20 >> e >> lim ( [log(n)]^k / Xn ) =3D 0 para todo k > 0 >> > > Existem muitas, como X_n=2^(raiz(log(n)), ou X_n=(log(n))^log(log(n)). > >> ********* >> >> 7. Um tri=E2ngulo tem lados com medida inteira e =E1rea racional. Prove = >> que uma de suas alturas tem medida inteira e que o p=E9 desta altura = >> est=E1 a uma dist=E2ncia inteira dos v=E9rtices do tri=E2ngulo. >> > > Parece que isso nao esta' certo. O triangulo de lados 17, 65 e 80 tem area > 288 e alturas 576/17, 576/65 e 36/5, que nao sao inteiras... > >> ********* >> >> 9. Seja K um inteiro >=3D 2.=20 >> infinito >> Seja S =3D SOMAT=D3RIO 1 / K^(n^2) =3D 1/K + 1/K^4 + 1/K^9 + 1/K^16 + = >> ... >> n =3D 1 >> Prove que S =E9 irracional. >> > > Se x=p/q e' racional e r/s e' outro racional diferente de x entao > |x-r/s|=|(ps-qr)/qs|>=1/qs, ou seja, s|x-r/s|>=1/q. > Por outro lado, soma(n=1 ate' m)(1/K^(n^2)) e' um racional com denominador > (divisor de) K^(m^2), digamos p/K^(m^2), e > |S-p/K^(m^2)|<(1/K^((m+1)^2))(1+1/2+1/4+...)=2/K^((m+1)^2), mas > K^(m^2).2/K^((m+1)^2)=2/K^(2m+1) tende a 0 quando m tende a infinito, e > portanto S nao pode ser racional. > > Abracos, > Gugu > ========================================================================= > Instru??es para entrar na lista, sair da lista e usar a lista em > http://www.mat.puc-rio.br/~nicolau/olimp/obm-l.html > O administrador desta lista ? > ========================================================================= > ========================================================================= Instruções para entrar na lista, sair da lista e usar a lista em http://www.mat.puc-rio.br/~nicolau/olimp/obm-l.html O administrador desta lista é ========================================================================= From owner-obm-l@sucuri.mat.puc-rio.br Sat Apr 12 15:15:30 2003 Return-Path: Received: (from majordom@localhost) by sucuri.mat.puc-rio.br (8.9.3/8.9.3) id PAA14265 for obm-l-MTTP; Sat, 12 Apr 2003 15:11:47 -0300 Received: from mail.gmx.net (mail.gmx.net [213.165.64.20]) by sucuri.mat.puc-rio.br (8.9.3/8.9.3) with SMTP id PAA14261 for ; Sat, 12 Apr 2003 15:11:43 -0300 Received: (qmail 3686 invoked by uid 65534); 12 Apr 2003 18:10:59 -0000 Received: from unknown (EHLO localhost) (200.217.15.93) by mail.gmx.net (mp023-rz3) with SMTP; 12 Apr 2003 20:10:59 +0200 Date: Sat, 12 Apr 2003 15:10:18 -0300 From: Igor GomeZZ X-Mailer: The Bat! (v1.61) Organization: -- X-Priority: 3 (Normal) Message-ID: <524131200.20030412151018@gmx.net> To: felipe mendona Subject: [obm-l] =?ISO-8859-1?B?UmU6IFtvYm0tbF0gUkU6Z2VuZXJhbGl6YedhbyBkZSBQQSBkZSAyIG9y?= =?ISO-8859-1?B?ZGVt?= In-Reply-To: References: MIME-Version: 1.0 Content-Type: text/plain; charset=ISO-8859-1 Content-Transfer-Encoding: 8bit Sender: owner-obm-l@sucuri.mat.puc-rio.br Precedence: bulk Reply-To: obm-l@mat.puc-rio.br Em 11/4/2003, 22:03, felipe (felipensador@hotmail.com) disse: > Boa noite colegas da lista..... Fala Felipe... >                    Igor, em essencia voce quer uma maneira rapida > e alternativa para calcular o somatorio de qualquer progressao  na forma > {(a_1). (b_1),(a_2). (b_2),......, > (a_n).( b_n)}na qual os termos a_k seguem uma PA de razao r_a  > ao passo que os termos b_k  segue uma PA de razao r_b para 1<=k<=n.Vejamos > isto:  Vamos convencionar que a_1>=b_1. [cortado] >       Voltando ao problema inicial, teremos S = n.(a_1).(b_1) + > (a_1.r_b+b_1.r_a).n.(n-1)/2  +(r_a.r_b). n.(n-1).(2.n-1)/6. O que acabei > de fazer é uma generalizaçao para qualquer PA de 2 > ordem,portanto vale para a soma que voce  propos > :1.2 + 2.3 + ........+ 49.50 = 41650 Bela resolução! Principalmente a idéia de tratar duas progressões como apenas uma e os passos algébricos da resolução... Vou bolar outros problemas semelhantes pra treinar a idéia. > Forte abraço > Felipe Mendonça Vitória-ES. Eh bom ver mais um capixaba na lista... Valeu! Fui! ####### Igor GomeZZ ######## UIN: 29249895 Vitória, Espírito Santo, Brasil Criação: 12/4/2003 (14:21) #################################### Pare para pensar: O rio atinge seus objetivos porque aprendeu a contornar obstáculos. (Lao- Tsé) #################################### ========================================================================= Instruções para entrar na lista, sair da lista e usar a lista em http://www.mat.puc-rio.br/~nicolau/olimp/obm-l.html O administrador desta lista é ========================================================================= From owner-obm-l@sucuri.mat.puc-rio.br Sat Apr 12 15:23:28 2003 Return-Path: Received: (from majordom@localhost) by sucuri.mat.puc-rio.br (8.9.3/8.9.3) id PAA14401 for obm-l-MTTP; Sat, 12 Apr 2003 15:20:53 -0300 Received: from shen.bol.com.br (shen.bol.com.br [200.221.24.14]) by sucuri.mat.puc-rio.br (8.9.3/8.9.3) with ESMTP id PAA14397 for ; Sat, 12 Apr 2003 15:20:45 -0300 Received: from bol.com.br (200.221.24.140) by shen.bol.com.br (5.1.071) id 3E7F61BF004D532F for obm-l@mat.puc-rio.br; Sat, 12 Apr 2003 15:20:14 -0300 Date: Sat, 12 Apr 2003 15:20:14 -0300 Message-Id: Subject: [obm-l] =?iso-8859-1?q?Re=3A=5Bobm=2Dl=5D_Re=3A_=5Bobm=2Dl=5D_c=E1lculo=2Dengen?= =?iso-8859-1?q?haria?= MIME-Version: 1.0 Content-Type: text/plain;charset="iso-8859-1" From: "adr.scr.m" To: obm-l@mat.puc-rio.br X-XaM3-API-Version: 2.4 R3 ( B4 ) X-SenderIP: 200.151.64.201 Content-Transfer-Encoding: 8bit X-MIME-Autoconverted: from quoted-printable to 8bit by sucuri.mat.puc-rio.br id PAA14398 Sender: owner-obm-l@sucuri.mat.puc-rio.br Precedence: bulk Reply-To: obm-l@mat.puc-rio.br entaum qual livro vc me sugere Cláudio ? []´s. Adriano. > Este livro é um dos mais elementares e fáceis de ler qu e eu conheço para > Análise Real. Assim, recomendo o livro como uma ótima i ntrodução ao assunto. > > Quanto à utilidade para engenharia, eu diria o seguinte : > Para engenharia (pelo menos durante o curso) você preci sa de uma boa base em > Cálculo, e Análise Real trata justamente dos fundamento s conceituais do > Cálculo. Assim, é o tipo do conhecimento que vale a pen a ter, se o esforço > para adquiri-lo não for excessivo. > > Por outro lado, se você não adora matemática e pretende ser um engenheiro > "mão na massa", muito mais chegado à prática do que à t eoria, então não se > preocupe em virar um expert em análise - estou convicto de que a maioria dos > engenheiros competentes que existem por aí não sabem o que é um conjunto > compacto ou a diferença entre as integrais de Riemann e de Stieltjes. > > Um abraço, > Claudio. > > ----- Original Message ----- > From: "adr.scr.m" > To: > Sent: Saturday, March 22, 2003 6:12 PM > Subject: [obm-l] cálculo-engenharia > > > > gostaria de saber se o livro do Elon( Análise Real)eh > > bom para quem faz engenharia (1º período)? > > se naum for,por favor,recomendem outros. > > []´s. > > Adriano. > > > > > > _____________________________________________________ _____________________ > > E-mail Premium BOL > > Antivírus, anti- spam e até 100 MB de espaço. Assine já! > > http://email.bol.com.br/ > > > > > > ===================================================== ==================== > > Instruções para entrar na lista, sair da lista e usar a lista em > > http://www.mat.puc-rio.br/~nicolau/olimp/obm-l.html > > O administrador desta lista é > > ===================================================== ==================== > > ======================================================= ================== > Instruções para entrar na lista, sair da lista e usar a lista em > http://www.mat.puc-rio.br/~nicolau/olimp/obm-l.html > O administrador desta lista é > ======================================================= ================== > __________________________________________________________________________ Seleção de Softwares UOL. 10 softwares escolhidos pelo UOL para você e sua família. http://www.uol.com.br/selecao ========================================================================= Instruções para entrar na lista, sair da lista e usar a lista em http://www.mat.puc-rio.br/~nicolau/olimp/obm-l.html O administrador desta lista é ========================================================================= From owner-obm-l@sucuri.mat.puc-rio.br Sat Apr 12 15:46:35 2003 Return-Path: Received: (from majordom@localhost) by sucuri.mat.puc-rio.br (8.9.3/8.9.3) id PAA15152 for obm-l-MTTP; Sat, 12 Apr 2003 15:43:49 -0300 Received: from ivoti.terra.com.br (ivoti.terra.com.br [200.176.3.20]) by sucuri.mat.puc-rio.br (8.9.3/8.9.3) with ESMTP id PAA15148 for ; Sat, 12 Apr 2003 15:43:45 -0300 Received: from sapucaia.terra.com.br (sapucaia.terra.com.br [200.176.3.41]) by ivoti.terra.com.br (Postfix) with ESMTP id D996D408B6B for ; Sat, 12 Apr 2003 15:43:14 -0300 (BRT) Received: from [200.177.182.67] (dl-nas6-sao-C8B1B643.p001.terra.com.br [200.177.182.67]) by sapucaia.terra.com.br (Postfix) with ESMTP id BBEA0304040 for ; Sat, 12 Apr 2003 15:43:13 -0300 (BRT) User-Agent: Microsoft-Outlook-Express-Macintosh-Edition/5.02.2022 Date: Sat, 12 Apr 2003 15:41:21 -0300 Subject: Re: [obm-l] Fibonacci From: Claudio Buffara To: Message-ID: In-Reply-To: <000b01c300a5$b9d73b00$75aaa5c8@epq.ime.eb.br> Mime-version: 1.0 Content-type: text/plain; charset="US-ASCII" Content-transfer-encoding: 7bit Sender: owner-obm-l@sucuri.mat.puc-rio.br Precedence: bulk Reply-To: obm-l@mat.puc-rio.br Oi, Marcio: Mesmo o problema de se achar: S = SOMA(n>=0) 1/F(2^n) esta' longe de ser trivial. Eu sei que S = 4 - A, onde A = (1 + raiz(5))/2, ou seja, S = (7 - raiz(5))/2. Acho que a formula: F(2k) = [F(k+1) + F(k-1)]*F(k) deve entrar em algum lugar na demonstracao e, de algum jeito, a restricao as potencias de 2 deve fazer aparecer alguma PG cuja soma eh S. Um abraco, Claudio. on 12.04.03 00:43, Marcio at marciocohen@superig.com.br wrote: > Oi Claudio! > Desculpe a demora na resposta. Assim que li seu email fui conferir minha > solucao.. E descobri que ela esta mesmo errada.. Fiquei tentando consertar, > mas nao deu (alias, foi mas por teimosia, pq o que vc falou faz total > sentido.. se a irracionalidade nao foi decidida ateh 80, eh razoavel que o > resultado nao se escreva de maneira mto simples com constantes conhecidas). > Como esse eh um exercicio proposto num livro que eu tenho aqui em casa, > achei que fosse soh mais um exercicio de rotina, e nao um problema conhecido > e dificil.. O que eu tinha feito era olhar para o inverso de F_n*sqrt(5) > como 1/(a^n + b^n), onde a,b sao... e ab=-1 donde F_n*sqrt(5) = b^n/(b^2n + > 1) para n par, com - para n impar. > E ai eu encarava esse ultimo numero como a soma de uma PG e depois fazia > uma contagem dupla nas potencias de b (isso se justifica pelo fato da serie > ser absolutamente convergente, o que segue por exemplo de 1/|F_n*sqrt(5)| = > 1/|a^n-b^n|<2/(a^n) que eh uma serie convergente pois |1/a|<1 (ou entao > seguindo o que vc e o yuri responderam em emails passados). O problema eh > que eu burramente escrevi o o 1/(b^2n + 1) como 1-b+b^2-b^3+... e nao como > uma PG de razao b^2n como deveria ser.. > Sendo assim, eu tmb gostaria muito de ver a minha solucao :) > Abracos, > Marcio > PS: O livro em questao eh o Engel - Solving problem strategies.. La ele > deixa esse exercicio como proposto, sem mostrar a solucao.. Pensando agora, > acho provavel que ele quisesse dizer F_2^n ao inves de F_n.. > > ----- Original Message ----- > From: "Claudio Buffara" > To: > Sent: Wednesday, April 09, 2003 11:54 PM > Subject: Re: [obm-l] Fibonacci > > >> Marcio: >> >> Dei uma pesquisada na internet e parece a prova da irracionalidade da soma >> dos reciprocos dos numeros de Fibonacci foi um dos problemas propostos por >> Paul Erdos e que soh foi resolvido na decada de 1980. >> >> Assim, o valor exato desta soma nao deve poder ser expresso como uma >> combinacao de constantes conhecidas (Pi, e, raiz(5), etc.). >> >> Sendo assim, eu gostaria muito de ver a sua solucao. > ========================================================================= Instruções para entrar na lista, sair da lista e usar a lista em http://www.mat.puc-rio.br/~nicolau/olimp/obm-l.html O administrador desta lista é ========================================================================= From owner-obm-l@sucuri.mat.puc-rio.br Sat Apr 12 16:29:22 2003 Return-Path: Received: (from majordom@localhost) by sucuri.mat.puc-rio.br (8.9.3/8.9.3) id QAA16440 for obm-l-MTTP; Sat, 12 Apr 2003 16:26:35 -0300 Received: from ivoti.terra.com.br (ivoti.terra.com.br [200.176.3.20]) by sucuri.mat.puc-rio.br (8.9.3/8.9.3) with ESMTP id QAA16430 for ; Sat, 12 Apr 2003 16:26:31 -0300 Received: from altamira.terra.com.br (altamira.terra.com.br [200.176.3.40]) by ivoti.terra.com.br (Postfix) with ESMTP id B9501408C94 for ; Sat, 12 Apr 2003 16:25:59 -0300 (BRT) Received: from [200.177.186.141] (dl-nas4-sao-C8B1BA8D.p001.terra.com.br [200.177.186.141]) by altamira.terra.com.br (Postfix) with ESMTP id BE8883DC07F for ; Sat, 12 Apr 2003 16:25:58 -0300 (BRT) User-Agent: Microsoft-Outlook-Express-Macintosh-Edition/5.02.2022 Date: Sat, 12 Apr 2003 16:24:08 -0300 Subject: Re: [obm-l] Re:[obm-l] Re: [obm-l] c =?ISO-8859-1?B?4Q==?=lculo-engenharia From: Claudio Buffara To: Message-ID: In-Reply-To: Mime-version: 1.0 Content-type: text/plain; charset="ISO-8859-1" Content-Transfer-Encoding: 8bit X-MIME-Autoconverted: from quoted-printable to 8bit by sucuri.mat.puc-rio.br id QAA16437 Sender: owner-obm-l@sucuri.mat.puc-rio.br Precedence: bulk Reply-To: obm-l@mat.puc-rio.br on 12.04.03 15:20, adr.scr.m at adr.scr.m@bol.com.br wrote: > entaum qual livro vc me sugere Cláudio ? > []´s. > Adriano. > Sugiro esse mesmo, pois eh bem escrito, razoavelmente elementar e relativamente barato, mas so vale o tempo e o esforco de estuda-lo se voce tiver realmente interesse em se aprofundar nos fundamentos do calculo. Volto a repetir: voce pode vir a ser um excelente engenheiro sem conhecer analise matematica a fundo, uma vez que para um engenheiro, matematica eh uma ferramenta e nao o objeto principal de estudo ou trabalho. Um abraco, Claudio. >> Este livro é um dos mais elementares e fáceis de ler qu > e eu conheço para >> Análise Real. Assim, recomendo o livro como uma ótima i > ntrodução ao assunto. >> >> Quanto à utilidade para engenharia, eu diria o seguinte > : >> Para engenharia (pelo menos durante o curso) você preci > sa de uma boa base em >> Cálculo, e Análise Real trata justamente dos fundamento > s conceituais do >> Cálculo. Assim, é o tipo do conhecimento que vale a pen > a ter, se o esforço >> para adquiri-lo não for excessivo. >> >> Por outro lado, se você não adora matemática e pretende > ser um engenheiro >> "mão na massa", muito mais chegado à prática do que à t > eoria, então não se >> preocupe em virar um expert em análise - > estou convicto de que a maioria dos >> engenheiros competentes que existem por aí não sabem o > que é um conjunto >> compacto ou a diferença entre as integrais de Riemann e > de Stieltjes. >> >> Um abraço, >> Claudio. >> >> ----- Original Message ----- >> From: "adr.scr.m" >> To: >> Sent: Saturday, March 22, 2003 6:12 PM >> Subject: [obm-l] cálculo-engenharia >> >> >>> gostaria de saber se o livro do Elon( Análise Real)eh >>> bom para quem faz engenharia (1º período)? >>> se naum for,por favor,recomendem outros. >>> []´s. >>> Adriano. >>> >>> ========================================================================= Instruções para entrar na lista, sair da lista e usar a lista em http://www.mat.puc-rio.br/~nicolau/olimp/obm-l.html O administrador desta lista é ========================================================================= From owner-obm-l@sucuri.mat.puc-rio.br Sat Apr 12 19:28:09 2003 Return-Path: Received: (from majordom@localhost) by sucuri.mat.puc-rio.br (8.9.3/8.9.3) id TAA18832 for obm-l-MTTP; Sat, 12 Apr 2003 19:25:00 -0300 Received: from trex-b.centroin.com.br (trex-b.centroin.com.br [200.225.63.136]) by sucuri.mat.puc-rio.br (8.9.3/8.9.3) with ESMTP id TAA18828 for ; Sat, 12 Apr 2003 19:24:57 -0300 Received: from centroin.com.br (RJ208093.user.veloxzone.com.br [200.165.208.93] (may be forged)) (authenticated bits=0) by trex-b.centroin.com.br (8.12.9/8.12.9) with ESMTP id h3CMOUiL012924 for ; Sat, 12 Apr 2003 19:24:31 -0300 (EST) Message-ID: <3E98924F.80509@centroin.com.br> Date: Sat, 12 Apr 2003 19:25:19 -0300 From: "A. C. Morgado" User-Agent: Mozilla/5.0 (Windows; U; Windows NT 5.0; en-US; rv:1.0.2) Gecko/20030208 Netscape/7.02 X-Accept-Language: en-us, en MIME-Version: 1.0 To: obm-l@mat.puc-rio.br Subject: Re: [obm-l] References: <27BD56F8640DD711A4320006295078E50500B7@pssnx1.brahma> Content-Type: multipart/alternative; boundary="------------050202020307040605050702" Sender: owner-obm-l@sucuri.mat.puc-rio.br Precedence: bulk Reply-To: obm-l@mat.puc-rio.br --------------050202020307040605050702 Content-Type: text/plain; charset=ISO-8859-1; format=flowed Content-Transfer-Encoding: 8bit 7) 12x11 = 132, porque ha 12 modos de escolher o presidente e, depois disso, 11 modos de escolher o secretario. 8) Ha 2 palavrs com uma so letra, 2x2=4 palavras com duas letras, 2x2x2=8 com tres e 2x2x2x2=16 com quatro. A resposta eh 2+4+8+16=30. 6) Se ha n participantes, o numero de partidas eh C(n, 2) = n(n-1)/2. Igualando a 780, obtemos n(n-1) = 1560. Essa equaçao se resolve por formula ou ate mesmo de cabeça (n eh inteiro positivo). A resposta eh n=40 1) Vou supor que as balas sejam iguais.Seja x a quantidade de balas que a primeira criança recebe, etc. a) Devemos descobrir quantas sao as soluçoes inteiras de x+y+z+w+t = 15 com todas as incognitas maiores ou iguais a 1. b) Devemos descobrir quantas sao as soluçoes inteiras de x+y+z+w+t = 15 com todas as incognitas maiores ou iguais a 2. O problema padrao eh achar o numero de soluçoes inteiras de x+y+z+w+t = 15 com todas as incognitas maiores ou iguais a 0. A resposta do problema padrao eh CR(5, 15) = C(5+15-1, 15) = C(19, 15) a) Para fazer esse problema recair no padrao, fazemos x=1+a, y=1+b.... A equaçao se transforma em a+b+c+d+e=10 com todo mundo maior que ou igual a 0. A resposta eh CR(5, 10) = C(14, 10) = 1001 b) Aqui fazemos x=2+a, y+2+b... Recaimos em a+b+c+d+e=5 com todo mundo maior que ou igual a 0. A resposta eh CR(5, 5) = C(9, 5) = 126 Ricardo de Moraes (PS) wrote: > Boa noite, > Estou com dificuldades em resolver alguns problemas, até que resolvi > alguns deles mas fiquei com duvidas quanto às respostas. > Se alguém puder me ajudar... > > > 1. De quantos modos podemos distribuir 15 balas entre 5 crianças: > a) garantindo que cada criança receba pelo menos uma bala; > b) garantindo que cada criança receba pelo menos duas balas. > > 2. Um campeonato é disputado por 12 clubes em rodadas de 6 jogos cada. > De quantos modos é possível selecionar os jogos da primeira rodada. > > 3. Delegados de 10 países diferentes devem se sentar em 10 cadeiras em > fila. De quantos modos isso pode ser feito, se os delegados do Brasil > e de Portugal devem sentar juntos e os da Argentina e do Paraguai não > podem sentar jntos? > > 4. lançam-se 3 dados. Em quantos dos possíveis resultados a soma dos > valores das faces é 10? > > 5. Determine o número de anagramas da palavra ESTUDANTE que tem E no > primeiro lugar ou S no segundo lugar ou T no terceiro lugar? > > 6. Em um torneio no qual cada participante enfrenta todos os demais, > são disputadas 780 partidas no total. Quantos são os participantes > deste torneio? > > 7. De quantos modos diferentes podem ser escolhidos um presidente e um > secretário de um conselho que tem 12 membros? > > 8. O código Morse usa "palavras" contendo de 1 a 4 "letras". As > "letras" são ponto e traço. Quantas "palavras" existe no código Morse? > > > > Ricardo. > --------------050202020307040605050702 Content-Type: text/html; charset=us-ascii Content-Transfer-Encoding: 7bit 7) 12x11 = 132, porque ha 12 modos de escolher o presidente e, depois disso, 11 modos de escolher o secretario.
8) Ha 2 palavrs com uma so letra, 2x2=4 palavras com duas letras, 2x2x2=8 com tres e 2x2x2x2=16 com quatro. A resposta eh 2+4+8+16=30.
6) Se ha n participantes, o numero de partidas eh C(n, 2) = n(n-1)/2. Igualando a 780, obtemos
n(n-1) = 1560. Essa equaçao se resolve por formula ou ate mesmo de cabeça (n eh inteiro positivo).
A resposta eh  n=40
1) Vou supor que as balas sejam iguais.Seja x a quantidade de balas que a primeira criança recebe, etc.
a) Devemos descobrir quantas sao as soluçoes inteiras de x+y+z+w+t = 15 com todas as incognitas maiores ou iguais a 1.
b) Devemos descobrir quantas sao as soluçoes inteiras de x+y+z+w+t = 15 com todas as incognitas maiores ou iguais a 2.
O problema padrao eh achar o numero de soluçoes inteiras de x+y+z+w+t = 15 com todas as incognitas maiores ou iguais a 0. A resposta do problema padrao eh  CR(5, 15) = C(5+15-1, 15) = C(19, 15)
a) Para fazer esse problema recair no padrao, fazemos x=1+a, y=1+b....
A equaçao se transforma em a+b+c+d+e=10 com todo mundo maior que ou igual a 0. A resposta eh CR(5, 10) = C(14, 10) = 1001
b) Aqui fazemos x=2+a, y+2+b... Recaimos em a+b+c+d+e=5 com todo mundo maior que ou igual a 0. A resposta eh CR(5, 5) = C(9, 5) = 126
Ricardo de Moraes (PS) wrote:
Boa noite,
Estou com dificuldades em resolver alguns problemas, até que resolvi alguns deles mas fiquei com duvidas quanto às respostas.
Se alguém puder me ajudar...
 
 
1. De quantos modos podemos distribuir 15 balas entre 5 crianças:
a) garantindo que cada criança receba pelo menos uma bala;
b) garantindo que cada criança receba pelo menos duas balas.
 
2. Um campeonato é disputado por 12 clubes em rodadas de 6 jogos cada. De quantos modos é possível selecionar os jogos da primeira rodada.
 
3. Delegados de 10 países diferentes devem se sentar em 10 cadeiras em fila. De quantos modos isso pode ser feito, se os delegados do Brasil e de Portugal devem sentar juntos e os da Argentina e do Paraguai não podem sentar jntos?
 
4. lançam-se 3 dados. Em quantos dos possíveis resultados a soma dos valores das faces é 10?
 
5. Determine o número de anagramas da palavra ESTUDANTE que tem E no primeiro lugar ou S no segundo lugar ou T no terceiro lugar?
 
6. Em um torneio no qual cada participante enfrenta todos os demais, são disputadas 780 partidas no total. Quantos são os participantes deste torneio?
 
7. De quantos modos diferentes podem ser escolhidos um presidente e um secretário de um conselho que tem 12 membros?
 
8. O código Morse usa "palavras" contendo de 1 a 4 "letras". As "letras" são ponto e traço. Quantas "palavras" existe no código Morse?
 
 
 
Ricardo.
 

--------------050202020307040605050702-- ========================================================================= Instruções para entrar na lista, sair da lista e usar a lista em http://www.mat.puc-rio.br/~nicolau/olimp/obm-l.html O administrador desta lista é ========================================================================= From owner-obm-l@sucuri.mat.puc-rio.br Sat Apr 12 20:09:46 2003 Return-Path: Received: (from majordom@localhost) by sucuri.mat.puc-rio.br (8.9.3/8.9.3) id UAA19589 for obm-l-MTTP; Sat, 12 Apr 2003 20:07:12 -0300 Received: from artemis.opendf.com.br (artemis.opengate.com.br [200.181.71.14]) by sucuri.mat.puc-rio.br (8.9.3/8.9.3) with ESMTP id UAA19585 for ; Sat, 12 Apr 2003 20:07:09 -0300 Received: from localhost (localhost [127.0.0.1]) by artemis.opendf.com.br (Postfix) with ESMTP id A17142BECD for ; Sat, 12 Apr 2003 20:07:43 -0300 (BRT) Received: from artemis.opendf.com.br ([127.0.0.1]) by localhost (artemis.opengate.com.br [127.0.0.1:10024]) (amavisd-new) with ESMTP id 24338-01 for ; Sat, 12 Apr 2003 20:07:42 -0300 (BRT) Received: from computer (200-181-089-062.bsace7001.dsl.brasiltelecom.net.br [200.181.89.62]) by artemis.opendf.com.br (Postfix) with ESMTP id 05D472BEC2 for ; Sat, 12 Apr 2003 20:07:42 -0300 (BRT) From: "Artur Costa Steiner" To: Subject: [obm-l] =?iso-8859-1?Q?RE:_=5Bobm-l=5D_Re:=5Bobm-l=5D_Re:_=5Bobm-l=5D_c=E1lcu?= =?iso-8859-1?Q?lo-engenharia?= Date: Sat, 12 Apr 2003 20:06:35 -0300 Organization: Steiner Consultoria LTDA Message-ID: MIME-Version: 1.0 Content-Type: multipart/mixed; boundary="----=_NextPart_000_0006_01C3012F.056F5960" X-Priority: 3 (Normal) X-MSMail-Priority: Normal X-Mailer: Microsoft Outlook, Build 10.0.2627 In-Reply-To: Importance: Normal X-MS-TNEF-Correlator: 000000005FC3F71D8B4BB941A8AA63D06F261C3664A22A00 X-MimeOLE: Produced By Microsoft MimeOLE V6.00.2800.1106 X-Virus-Scanned: by amavisd-new Sender: owner-obm-l@sucuri.mat.puc-rio.br Precedence: bulk Reply-To: obm-l@mat.puc-rio.br This is a multi-part message in MIME format. ------=_NextPart_000_0006_01C3012F.056F5960 Content-Type: text/plain; charset="iso-8859-1" Content-Transfer-Encoding: quoted-printable Se voce curte um pouquinho de Analise e esta disposto a investir um = pouco mais, um livro que eu recomendo fortemente (em Ingl=EAs) =E9 = Introduction to Real Analysis, de Bartle e Sherbert. Eh realmente excelente, o livro tem = uma linguagem acessivel, excelente did=E1tica, sem qualquer sacificio do = rigor matematico. Robert Bartle tem outros livros e eh de fato um grande = autor.=20 Este livro que estou citando eh uma excelente introducao e se dedica a Analise na reta real. Mas quem estudar por ele ganhara uma solida base = para analise em R^n, nos complexos e mesmo para topicos mais avancados que geralmente so matematicos estudam. O livro chega a apresentar uma = abordagem da integral de Lebesgue, embora de forma bem diferente do que aquela = baseada na teoria de medidas (assunto que eu comecei a estudar e no qual ainda = naum consegui ir para a frente - sou engenheiro e tenho que = trabalhar....risos - do contr=E1rio, nem dah para comer, quanto mais para estudar Analise..). = Eu recomendo este livro mesmo para quem vai ser engenheiro e , de fato, = naum precisa lidar profundamente com epsilons e deltas, medida de Lebesgue, teorema de Heine Borel, etc... Eh de fato verdade que a esmagadora maioria dos engenheiros nao sabe o = que eh um conjunto compacto e nem a diferenca entre integrais de Riemann e = de Stieltjes (muito menos a de Lebesgue). A maioria dis engenheiros nao = gosta muito de matematica. Mas se vc for para uma area ligada a algoritmos e e otimizacao, entao um certo conhecimento de Analise sera util. =20 [Artur Costa Steiner] ------=_NextPart_000_0006_01C3012F.056F5960 Content-Type: application/ms-tnef; name="winmail.dat" Content-Transfer-Encoding: base64 Content-Disposition: attachment; filename="winmail.dat" eJ8+IiQXAQaQCAAEAAAAAAABAAEAAQeQBgAIAAAA5AQAAAAAAADoAAEIgAcAGAAAAElQTS5NaWNy b3NvZnQgTWFpbC5Ob3RlADEIAQ2ABAACAAAAAgACAAEGAAcAAQAAAAAAAAEGgAMADgAAANMHBAAM ABQABgAAAAYACgEBA5AGAAQKAAAuAAAACwACAAEAAAALACMAAAAAAAMAJgAAAAAACwApAAAAAAAD AC4AAAAAAAIBMQABAAAAGAAAAAAAAABfw/cdi0u5QaiqY9BvJhw2ZKEqAAMANgAAAAAAHgBwAAEA AAAyAAAAW29ibS1sXSBSZTpbb2JtLWxdIFJlOiBbb2JtLWxdIGPhbGN1bG8tZW5nZW5oYXJpYQAA AAIBcQABAAAAFgAAAAHDAUfbL+4Qy5Y2cUq+rwt9mxKurXgAAAIBHQwBAAAAGQAAAFNNVFA6QVJU VVJAT1BFTkRGLkNPTS5CUgAAAAALAAEOAAAAAEAABg4A/BwVSAHDAQIBCg4BAAAAGAAAAAAAAABf w/cdi0u5QaiqY9BvJhw2woAAAAMAFA4AAAAACwAfDgEAAAAeACgOAQAAAC8AAAAwMDAwMDAwMgFh cnR1ckBvcGVuZGYuY29tLmJyAXBvcC5vcGVuZGYuY29tLmJyAAAeACkOAQAAAC8AAAAwMDAwMDAw MgFhcnR1ckBvcGVuZGYuY29tLmJyAXBvcC5vcGVuZGYuY29tLmJyAAACAQkQAQAAAMUEAADBBAAA KAcAAExaRnVv8zddAwAKAHJjcGcxMjXiMgNDdGV4BUEBAwH3TwqAAqQD4wIAY2gKwHPwZXQwIAcT AoAP8wBQfwRWCFUHshHFDlEDARDHMvcGAAbDEcUzBEYQyRLbEdPbCO8J9zsYvw4wNRHCDGDOYwBQ CwkBZDM2EVALpqEGUSB2b2MdoGMIcEEOsCB1bSBwCGBxYnULgGhvIAEAEWBu7wdABAAdoB/RcwGQ HzAEAF8eoCAgHyAgQAuAdiARabcFwB50BaAgAMAEACweYjUfoHYDYCAe0B/RdSDnGMAFoAeAbmQf IAIQHjELI9EeQSgkcCBJbmfBGKAnZWFzKQMwJXDqOSURdANgZBtwIWACILogIMFSJYADIB9yeQCQ SyJhH0FCCsB0bB/SUwpoBJBiBJB0LiBF/mgjgQdAJIQOwB3gKIAkof8icB8gIsQkYR5hIEAfoCUw +HVhZyTxANAHkACQITC+bCJwKkcgUQswJXAxIWD8Y2EicBEgHoAe0AdAIyH9BcBzANAGkA3gJrAf MB8gfQUQZwWxAMAkYTDADeBv/ylQCAApEig2K3IIYCZBBCDfIsMEIB/RKYAfQWYwwB8g7x5xCcAA cB9BYTKgBbApUPsKogqARSAgHaAiySCxI3D/L6ABkCPyM5Ersi1IC4AmRP5hN2EuoR8xIGAucCDh H3bfH4AjgSAxKaIpUE0lkCMS8x6AIBF1ZArBHqAFwCpx3zRgAHAQ8SBAK7JzBvAt8P0gQGIlkB2g CrE54R+GHoDoUl5uInBuMtEjsQtQvw7AM0MHgQRgPoQgwHAxIfcEICIyIOB2AHAucCQAO6N/NGAE kCnGPdAwqTNRPCNtbSlQTyK1EOBlPRAg4WH+cBjAESACMArBK7IBoAWwfzxQLFI+ETiRRhBDYR8y TPplKRBzLCAqwSRwR3EgQf8z0QWwK8EpEEfhBpAEkC2U/SMEYSMhC2A+M0KwIEA6oX8OsAWwBzAf MgeALeE7kSh5JZBzdQIwIwcjsh3gaf8g4TwWOoEjAidBC3FMox5x/wWgAIBGEB7gIQA8cT6jJCD9 SxQtPcEjcAnwLFAfAE9w/yLxMjIfAiMiJkABoAdAEPH+LlWRBRA90AQgUzAkAVGR9yZALhIFEG8/ wUfTKYA+k/0jsnIicC7xTnJCEz6TPBb7H3VVkCkpUSN6IBE15kDp/zvDQnBPgBEgPLFTqifzM/L/ P8FRUkaBL6AvgCKxPFMDYL5mTmBFISSTI7Ef4HAAkPcYUAYxOXJsAZAiYU2kSLz3TQIw4R8ySE9w V2AoMGRR+S0SdGNVkTVVM6ghMEeR/x9BS6MgAQDAPRAkAD1hIjH/TSQzQlOnBCAfgB8gL4ApEO8q 4SMjN5JRgmpOY0AiAND/IME6gSxiSsY50SSRGMBIJvtCMR9BUgiQA4EDoDlyBgDLIWBigWoHkSht HuBY4u8J8DLRY0tawUFoqEIxaX7fMHAgInB0TXIwxmE7ZB/B/HZjJCI+hEcjKaEisWhB/UYzbDBx NxAEYDNTKuAhYPhtaXoA0DkQLSFG0TQj/x3gACBWYyjgL6Akgh8rXbGvPXEhYDtQNVQgNUVbBxA/ PDAFwAhQICJvwGTycl0FNVR9f3AAAAAeAEIQAQAAACwAAAA8QkFCREVFMjguNDY0JWNsYXVkaW8u YnVmZmFyYUB0ZXJyYS5jb20uYnI+AAMAkhABAAAAAgEUOgEAAAAQAAAAs00u1b55Q0yfAehttPqt GAMA3j+vbwAAAwAJWQEAAAADAEBlAAAAAAsAE4AIIAYAAAAAAMAAAAAAAABGAAAAAAOFAAAAAAAA AwAVgAggBgAAAAAAwAAAAAAAAEYAAAAAEIUAAAAAAAADABuACCAGAAAAAADAAAAAAAAARgAAAABS hQAA45ABAAMAIoAIIAYAAAAAAMAAAAAAAABGAAAAAAGFAAAAAAAAQAAjgAggBgAAAAAAwAAAAAAA AEYAAAAAYIUAAADQiMMQAAAAHgBBgAggBgAAAAAAwAAAAAAAAEYAAAAAVIUAAAEAAAAFAAAAMTAu MAAAAAALAEKACCAGAAAAAADAAAAAAAAARgAAAAAGhQAAAAAAAAsARoAIIAYAAAAAAMAAAAAAAABG AAAAAA6FAAAAAAAAAwBJgAggBgAAAAAAwAAAAAAAAEYAAAAAGIUAAAAAAAALAF6ACCAGAAAAAADA AAAAAAAARgAAAACChQAAAQAAAAIB+A8BAAAAEAAAAF/D9x2LS7lBqKpj0G8mHDYCAfoPAQAAABAA AABfw/cdi0u5QaiqY9BvJhw2AgH7DwEAAACEAAAAAAAAADihuxAF5RAaobsIACsqVsIAAG1zcHN0 LmRsbAAAAAAATklUQfm/uAEAqgA32W4AAABDOlxEb2N1bWVudHMgYW5kIFNldHRpbmdzXEFydHVy XEFwcGxpY2F0aW9uIERhdGFcTWljcm9zb2Z0XE91dGxvb2tcT3V0bG9vay5wc3QAAwD+DwUAAAAD AA00/TcCAAIBFDQBAAAAEAAAAE5JVEH5v7gBAKoAN9luAAACAX8AAQAAADEAAAAwMDAwMDAwMDVG QzNGNzFEOEI0QkI5NDFBOEFBNjNEMDZGMjYxQzM2NjRBMjJBMDAAAAAAAwAGECyM75cDAAcQ2wQA AAMAEBAAAAAAAwAREAEAAAAeAAgQAQAAAGUAAABTRVZPQ0VDVVJURVVNUE9VUVVJTkhPREVBTkFM SVNFRUVTVEFESVNQT1NUT0FJTlZFU1RJUlVNUE9VQ09NQUlTLFVNTElWUk9RVUVFVVJFQ09NRU5E T0ZPUlRFTUVOVEUoRU1JAAAAACak ------=_NextPart_000_0006_01C3012F.056F5960-- ========================================================================= Instruções para entrar na lista, sair da lista e usar a lista em http://www.mat.puc-rio.br/~nicolau/olimp/obm-l.html O administrador desta lista é ========================================================================= From owner-obm-l@sucuri.mat.puc-rio.br Sat Apr 12 20:15:36 2003 Return-Path: Received: (from majordom@localhost) by sucuri.mat.puc-rio.br (8.9.3/8.9.3) id UAA19741 for obm-l-MTTP; Sat, 12 Apr 2003 20:13:47 -0300 Received: from artemis.opendf.com.br (artemis.opengate.com.br [200.181.71.14]) by sucuri.mat.puc-rio.br (8.9.3/8.9.3) with ESMTP id UAA19737 for ; Sat, 12 Apr 2003 20:13:44 -0300 Received: from localhost (localhost [127.0.0.1]) by artemis.opendf.com.br (Postfix) with ESMTP id 08E972BECE for ; Sat, 12 Apr 2003 20:14:19 -0300 (BRT) Received: from artemis.opendf.com.br ([127.0.0.1]) by localhost (artemis.opengate.com.br [127.0.0.1:10024]) (amavisd-new) with ESMTP id 24191-05 for ; Sat, 12 Apr 2003 20:14:18 -0300 (BRT) Received: from computer (200-181-089-062.bsace7001.dsl.brasiltelecom.net.br [200.181.89.62]) by artemis.opendf.com.br (Postfix) with ESMTP id AAD052BEC2 for ; Sat, 12 Apr 2003 20:14:16 -0300 (BRT) From: "Artur Costa Steiner" To: Subject: [obm-l] =?iso-8859-1?Q?Combiana=E7=F5es_completas_?= Date: Sat, 12 Apr 2003 20:13:09 -0300 Organization: Steiner Consultoria LTDA Message-ID: <000901c30149$16310ea0$9865fea9@computer> MIME-Version: 1.0 Content-Type: multipart/alternative; boundary="----=_NextPart_000_000A_01C3012F.F0F93360" X-Priority: 3 (Normal) X-MSMail-Priority: Normal X-Mailer: Microsoft Outlook, Build 10.0.2627 Importance: Normal X-MimeOLE: Produced By Microsoft MimeOLE V6.00.2800.1106 X-Virus-Scanned: by amavisd-new Sender: owner-obm-l@sucuri.mat.puc-rio.br Precedence: bulk Reply-To: obm-l@mat.puc-rio.br This is a multi-part message in MIME format. ------=_NextPart_000_000A_01C3012F.F0F93360 Content-Type: text/plain; charset="iso-8859-1" Content-Transfer-Encoding: 7bit Boa noite a todos Alguem se lembra da formula das combinacoes completas de n elementos k a k? (para arranjos completos, eh n^p) Estou precisando disto para o meu trabalho e, como ha muitos temp que naum lido com isto, esqueci. Nao tenho um unico livro em casa que trate especificamente de analise combinatoria. Eu tentei deduzir a formula e cheguei a CC(n, k) = n (n+1)....(n+k-1)/k! Acho que esta certo mas nao tenho certeza. Se alguem tiver a deducao, eu gostaria de ver e agradeco. Obrigado e um abraco Artur ------=_NextPart_000_000A_01C3012F.F0F93360 Content-Type: text/html; charset="iso-8859-1" Content-Transfer-Encoding: quoted-printable

Boa noite a = todos

Alguem se lembra da = formula das combinacoes completas de n elementos k a k? (para arranjos = completos, eh n^p) Estou precisando disto para o meu trabalho e, como ha muitos temp que = naum lido com isto, esqueci. Nao tenho um unico livro em casa que trate = especificamente de analise combinatoria. Eu tentei deduzir a formula e cheguei a CC(n, k) = =3D n (n+1)....(n+k-1)/k! Acho que esta certo mas nao tenho certeza. Se alguem = tiver a deducao, eu gostaria de ver e agradeco.

Obrigado e um = abraco

Artur

------=_NextPart_000_000A_01C3012F.F0F93360-- ========================================================================= Instruções para entrar na lista, sair da lista e usar a lista em http://www.mat.puc-rio.br/~nicolau/olimp/obm-l.html O administrador desta lista é ========================================================================= From owner-obm-l@sucuri.mat.puc-rio.br Sat Apr 12 20:37:24 2003 Return-Path: Received: (from majordom@localhost) by sucuri.mat.puc-rio.br (8.9.3/8.9.3) id UAA20441 for obm-l-MTTP; Sat, 12 Apr 2003 20:34:29 -0300 Received: from trex-b.centroin.com.br (trex-b.centroin.com.br [200.225.63.136]) by sucuri.mat.puc-rio.br (8.9.3/8.9.3) with ESMTP id UAA20437 for ; Sat, 12 Apr 2003 20:34:26 -0300 Received: from centroin.com.br (RJ208093.user.veloxzone.com.br [200.165.208.93] (may be forged)) (authenticated bits=0) by trex-b.centroin.com.br (8.12.9/8.12.9) with ESMTP id h3CNXxiL013836 for ; Sat, 12 Apr 2003 20:34:01 -0300 (EST) Message-ID: <3E98A296.5090107@centroin.com.br> Date: Sat, 12 Apr 2003 20:34:46 -0300 From: "A. C. Morgado" User-Agent: Mozilla/5.0 (Windows; U; Windows NT 5.0; en-US; rv:1.0.2) Gecko/20030208 Netscape/7.02 X-Accept-Language: en-us, en MIME-Version: 1.0 To: obm-l@mat.puc-rio.br Subject: Re: [obm-l] =?ISO-8859-1?Q?Combiana=E7=F5es_completas_?= References: <000901c30149$16310ea0$9865fea9@computer> Content-Type: multipart/alternative; boundary="------------060205040702000901040803" Sender: owner-obm-l@sucuri.mat.puc-rio.br Precedence: bulk Reply-To: obm-l@mat.puc-rio.br --------------060205040702000901040803 Content-Type: text/plain; charset=ISO-8859-1; format=flowed Content-Transfer-Encoding: 8bit Ta certo. So que eh mais comum escrever CR e nao CC. CR(n, p) = C(n+p-1,p). Ha uma deduçao em www.fgv.br Entre em graduaçao RJ procure pelas disciplinas do ciclo basico (estao dentro de programaçao academica), em teoria da probabilidade procure por materiais. no arquivo combinatoria 2 voce encontra. [ ]s Morgado Artur Costa Steiner wrote: > Boa noite a todos > > Alguem se lembra da formula das combinacoes completas de n elementos k > a k? (para arranjos completos, eh n^p) Estou precisando disto para o > meu trabalho e, como ha muitos temp que naum lido com isto, esqueci. > Nao tenho um unico livro em casa que trate especificamente de analise > combinatoria. Eu tentei deduzir a formula e cheguei a CC(n, k) = n > (n+1)....(n+k-1)/k! Acho que esta certo mas nao tenho certeza. Se > alguem tiver a deducao, eu gostaria de ver e agradeco. > > Obrigado e um abraco > > Artur > --------------060205040702000901040803 Content-Type: text/html; charset=us-ascii Content-Transfer-Encoding: 7bit Ta certo. So que eh mais comum escrever CR e nao CC.
CR(n, p) = C(n+p-1,p).
Ha uma deduçao em  www.fgv.br  
Entre em graduaçao RJ procure pelas disciplinas do ciclo basico (estao dentro de programaçao academica), em teoria da probabilidade procure por materiais. no arquivo combinatoria 2 voce encontra.
[ ]s
Morgado

Artur Costa Steiner wrote:

Boa noite a todos

Alguem se lembra da formula das combinacoes completas de n elementos k a k? (para arranjos completos, eh n^p) Estou precisando disto para o meu trabalho e, como ha muitos temp que naum lido com isto, esqueci. Nao tenho um unico livro em casa que trate especificamente de analise combinatoria. Eu tentei deduzir a formula e cheguei a CC(n, k) = n (n+1)....(n+k-1)/k! Acho que esta certo mas nao tenho certeza. Se alguem tiver a deducao, eu gostaria de ver e agradeco.

Obrigado e um abraco

Artur


--------------060205040702000901040803-- ========================================================================= Instruções para entrar na lista, sair da lista e usar a lista em http://www.mat.puc-rio.br/~nicolau/olimp/obm-l.html O administrador desta lista é ========================================================================= From owner-obm-l@sucuri.mat.puc-rio.br Sat Apr 12 20:49:56 2003 Return-Path: Received: (from majordom@localhost) by sucuri.mat.puc-rio.br (8.9.3/8.9.3) id UAA21025 for obm-l-MTTP; Sat, 12 Apr 2003 20:47:19 -0300 Received: from trex-b.centroin.com.br (trex-b.centroin.com.br [200.225.63.136]) by sucuri.mat.puc-rio.br (8.9.3/8.9.3) with ESMTP id UAA21020 for ; Sat, 12 Apr 2003 20:47:16 -0300 Received: from centroin.com.br (RJ208093.user.veloxzone.com.br [200.165.208.93] (may be forged)) (authenticated bits=0) by trex-b.centroin.com.br (8.12.9/8.12.9) with ESMTP id h3CNkhiL013993 for ; Sat, 12 Apr 2003 20:46:47 -0300 (EST) Message-ID: <3E98A591.7080903@centroin.com.br> Date: Sat, 12 Apr 2003 20:47:29 -0300 From: "A. C. Morgado" User-Agent: Mozilla/5.0 (Windows; U; Windows NT 5.0; en-US; rv:1.0.2) Gecko/20030208 Netscape/7.02 X-Accept-Language: en-us, en MIME-Version: 1.0 To: obm-l@mat.puc-rio.br Subject: Re: [obm-l] References: <27BD56F8640DD711A4320006295078E50500B7@pssnx1.brahma> Content-Type: multipart/alternative; boundary="------------030402070106010109050108" Sender: owner-obm-l@sucuri.mat.puc-rio.br Precedence: bulk Reply-To: obm-l@mat.puc-rio.br --------------030402070106010109050108 Content-Type: text/plain; charset=ISO-8859-1; format=flowed Content-Transfer-Encoding: 8bit 1) Imagine a tabela da rodada: _ x _ _ x _ _ x _ _ x _ _ x _ _ x _ Em principio voce tem que botar 12 times em 12 lugares e a resposta parece ser 12! Repare que voce contou a mesma seleçao de jogos varias vezes pois a seleçao nao se altera quando voce troca a ordem dos jogos (6! modos) nem quando voce troca a ordem dos times em cada jogo (2^6 modos) A resposta eh 12!/[6! (2^6)] = 10 395 Ricardo de Moraes (PS) wrote: > > 2. Um campeonato é disputado por 12 clubes em rodadas de 6 jogos cada. > De quantos modos é possível selecionar os jogos da primeira rodada. > > 3. Delegados de 10 países diferentes devem se sentar em 10 cadeiras em > fila. De quantos modos isso pode ser feito, se os delegados do Brasil > e de Portugal devem sentar juntos e os da Argentina e do Paraguai não > podem sentar jntos? > > 4. lançam-se 3 dados. Em quantos dos possíveis resultados a soma dos > valores das faces é 10? > > 5. Determine o número de anagramas da palavra ESTUDANTE que tem E no > primeiro lugar ou S no segundo lugar ou T no terceiro lugar? > > Ricardo. > --------------030402070106010109050108 Content-Type: text/html; charset=us-ascii Content-Transfer-Encoding: 7bit 1) Imagine a tabela da rodada:
           _ x _
           _ x _
           _ x _
           _ x _
           _ x _
           _ x _
Em principio voce tem que botar 12 times em 12 lugares e a resposta parece ser 12!
Repare que voce contou a mesma seleçao de jogos varias vezes pois a seleçao nao se altera quando voce troca a ordem dos jogos (6! modos) nem quando voce troca a ordem dos times em cada jogo (2^6 modos)
A resposta eh    12!/[6! (2^6)] = 10 395


Ricardo de Moraes (PS) wrote:
 
2. Um campeonato é disputado por 12 clubes em rodadas de 6 jogos cada. De quantos modos é possível selecionar os jogos da primeira rodada.
 
3. Delegados de 10 países diferentes devem se sentar em 10 cadeiras em fila. De quantos modos isso pode ser feito, se os delegados do Brasil e de Portugal devem sentar juntos e os da Argentina e do Paraguai não podem sentar jntos?
 
4. lançam-se 3 dados. Em quantos dos possíveis resultados a soma dos valores das faces é 10?
 
5. Determine o número de anagramas da palavra ESTUDANTE que tem E no primeiro lugar ou S no segundo lugar ou T no terceiro lugar?
 
Ricardo.
 

--------------030402070106010109050108-- ========================================================================= Instruções para entrar na lista, sair da lista e usar a lista em http://www.mat.puc-rio.br/~nicolau/olimp/obm-l.html O administrador desta lista é ========================================================================= From owner-obm-l@sucuri.mat.puc-rio.br Sat Apr 12 21:06:39 2003 Return-Path: Received: (from majordom@localhost) by sucuri.mat.puc-rio.br (8.9.3/8.9.3) id VAA21522 for obm-l-MTTP; Sat, 12 Apr 2003 21:03:47 -0300 Received: from lampiao.digi.com.br (lampiao.digi.com.br [200.241.100.60]) by sucuri.mat.puc-rio.br (8.9.3/8.9.3) with ESMTP id VAA21518 for ; Sat, 12 Apr 2003 21:03:44 -0300 Received: from p8c7y1fe9gyrelp.digi.com.br (host139.e.digizap.com.br [200.249.8.139]) by lampiao.digi.com.br (8.11.6/8.11.6) with ESMTP id h3D00f011657 for ; Sat, 12 Apr 2003 21:00:41 -0300 Message-Id: <5.2.0.9.0.20030412210037.00b18540@mail.digi.com.br> X-Sender: benedito@mail.digi.com.br X-Mailer: QUALCOMM Windows Eudora Version 5.2.0.9 Date: Sat, 12 Apr 2003 21:01:58 -0300 To: obm-l@mat.puc-rio.br From: benedito Subject: [obm-l] Um problema legal In-Reply-To: <3E98A296.5090107@centroin.com.br> References: <000901c30149$16310ea0$9865fea9@computer> Mime-Version: 1.0 Content-Type: text/plain; charset="iso-8859-1"; format=flowed X-MailScanner: Found to be clean Content-Transfer-Encoding: 8bit X-MIME-Autoconverted: from quoted-printable to 8bit by sucuri.mat.puc-rio.br id VAA21519 Sender: owner-obm-l@sucuri.mat.puc-rio.br Precedence: bulk Reply-To: obm-l@mat.puc-rio.br Um problema interessante: Num salão de jogos, existem três máquinas que imprimem cartões. A primeira máquina funciona da seguinte maneira: se você insere um cartão com quaisquer dois números a e b ela retorna um cartão com os números a + 1 e b + 1. A segunda máquina, aceita somente cartões numerados com dois números pares a e b e retorna um cartão numerado com os números a/2 e b/2. A terceira, aceita somente dois cartões numerados com a e b e c e d, respectivamente, e retorna um cartão numerado com a e c. Se você começa com um cartão numerado com 5 e 19, é possível obter um cartão com os números 1, 1988? Benedito Freire ========================================================================= Instruções para entrar na lista, sair da lista e usar a lista em http://www.mat.puc-rio.br/~nicolau/olimp/obm-l.html O administrador desta lista é ========================================================================= From owner-obm-l@sucuri.mat.puc-rio.br Sat Apr 12 21:15:53 2003 Return-Path: Received: (from majordom@localhost) by sucuri.mat.puc-rio.br (8.9.3/8.9.3) id VAA21982 for obm-l-MTTP; Sat, 12 Apr 2003 21:13:21 -0300 Received: from trex-b.centroin.com.br (trex-b.centroin.com.br [200.225.63.136]) by sucuri.mat.puc-rio.br (8.9.3/8.9.3) with ESMTP id VAA21978 for ; Sat, 12 Apr 2003 21:13:18 -0300 Received: from centroin.com.br (RJ208093.user.veloxzone.com.br [200.165.208.93] (may be forged)) (authenticated bits=0) by trex-b.centroin.com.br (8.12.9/8.12.9) with ESMTP id h3D0CoiL014295 for ; Sat, 12 Apr 2003 21:12:51 -0300 (EST) Message-ID: <3E98ABB5.6040608@centroin.com.br> Date: Sat, 12 Apr 2003 21:13:41 -0300 From: "A. C. Morgado" User-Agent: Mozilla/5.0 (Windows; U; Windows NT 5.0; en-US; rv:1.0.2) Gecko/20030208 Netscape/7.02 X-Accept-Language: en-us, en MIME-Version: 1.0 To: obm-l@mat.puc-rio.br Subject: Re: [obm-l] References: <27BD56F8640DD711A4320006295078E50500B7@pssnx1.brahma> <3E98A591.7080903@centroin.com.br> Content-Type: multipart/alternative; boundary="------------080605080605000907080802" Sender: owner-obm-l@sucuri.mat.puc-rio.br Precedence: bulk Reply-To: obm-l@mat.puc-rio.br --------------080605080605000907080802 Content-Type: text/plain; charset=ISO-8859-1; format=flowed Content-Transfer-Encoding: 8bit > 3) Br e Pt juntos 2*9! ( 2 modos de escolher a ordem deles e 9! > modos de arrumar o bloco dos dois e os outro 8 delegados). Devemos retirar dessas arrumaçoes aquelas em que o argentino eo paraguaio tambem estao juntos, 2*2*8! A resposta eh 2*9! - 4*8! > Ricardo de Moraes (PS) wrote: > >> >> 3. Delegados de 10 países diferentes devem se sentar em 10 cadeiras >> em fila. De quantos modos isso pode ser feito, se os delegados do >> Brasil e de Portugal devem sentar juntos e os da Argentina e do >> Paraguai não podem sentar jntos? >> >> Ricardo. >> > > --------------080605080605000907080802 Content-Type: text/html; charset=us-ascii Content-Transfer-Encoding: 7bit
3) Br e Pt juntos  2*9! ( 2 modos de escolher a ordem deles e 9! modos de arrumar o bloco dos dois e os outro 8 delegados).
Devemos retirar dessas arrumaçoes aquelas em que o argentino eo paraguaio tambem estao juntos, 2*2*8!
A resposta eh 2*9! - 4*8!
Ricardo de Moraes (PS) wrote:
 
3. Delegados de 10 países diferentes devem se sentar em 10 cadeiras em fila. De quantos modos isso pode ser feito, se os delegados do Brasil e de Portugal devem sentar juntos e os da Argentina e do Paraguai não podem sentar jntos?
 
Ricardo.
 


--------------080605080605000907080802-- ========================================================================= Instruções para entrar na lista, sair da lista e usar a lista em http://www.mat.puc-rio.br/~nicolau/olimp/obm-l.html O administrador desta lista é ========================================================================= From owner-obm-l@sucuri.mat.puc-rio.br Sat Apr 12 21:26:07 2003 Return-Path: Received: (from majordom@localhost) by sucuri.mat.puc-rio.br (8.9.3/8.9.3) id VAA22444 for obm-l-MTTP; Sat, 12 Apr 2003 21:23:30 -0300 Received: from trex-b.centroin.com.br (trex-b.centroin.com.br [200.225.63.136]) by sucuri.mat.puc-rio.br (8.9.3/8.9.3) with ESMTP id VAA22440 for ; Sat, 12 Apr 2003 21:23:27 -0300 Received: from centroin.com.br (RJ208093.user.veloxzone.com.br [200.165.208.93] (may be forged)) (authenticated bits=0) by trex-b.centroin.com.br (8.12.9/8.12.9) with ESMTP id h3D0N0iL014416 for ; Sat, 12 Apr 2003 21:23:01 -0300 (EST) Message-ID: <3E98AE15.1010301@centroin.com.br> Date: Sat, 12 Apr 2003 21:23:49 -0300 From: "A. C. Morgado" User-Agent: Mozilla/5.0 (Windows; U; Windows NT 5.0; en-US; rv:1.0.2) Gecko/20030208 Netscape/7.02 X-Accept-Language: en-us, en MIME-Version: 1.0 To: obm-l@mat.puc-rio.br Subject: Re: [obm-l] References: <27BD56F8640DD711A4320006295078E50500B7@pssnx1.brahma> Content-Type: multipart/alternative; boundary="------------080102040606010408020806" Sender: owner-obm-l@sucuri.mat.puc-rio.br Precedence: bulk Reply-To: obm-l@mat.puc-rio.br --------------080102040606010408020806 Content-Type: text/plain; charset=ISO-8859-1; format=flowed Content-Transfer-Encoding: 8bit Voce deve contar todos os que tem E em primeiro, todos que tem S em segundo e todos que tem T em terceiro, 8!+8!+8! = 120 960 Mas aih voce contou duas vezes os anagramas que tem E em 1 e S em 2, e tambem os que tem E em 1 e T em 3, e tambem os que tem S em 2 e T em 3. Corrigindo, a resposta passa a ser 120 960 - 3* 7! = 105 840 Mas os anagramas que tem E em 1, S em 2 e T em 3 foram contados tres vezes e descontados tres vezes. Eles devem ser incluidos na contagem. A resposta eh 105 480 + 6! = 106 200 Ricardo de Moraes (PS) wrote: > > 5. Determine o número de anagramas da palavra ESTUDANTE que tem E no > primeiro lugar ou S no segundo lugar ou T no terceiro lugar. > Ricardo. > --------------080102040606010408020806 Content-Type: text/html; charset=us-ascii Content-Transfer-Encoding: 7bit Voce deve contar todos os que tem E em primeiro, todos que tem S em segundo e todos que tem T em terceiro, 8!+8!+8! = 120 960
Mas aih voce contou duas vezes os anagramas que tem E em 1 e S em 2, e tambem os que tem E em 1 e T em 3, e tambem os que tem S em 2 e T em 3. Corrigindo, a resposta passa a ser
120 960 - 3* 7! = 105 840
Mas os anagramas que tem E em 1, S em 2 e T em 3 foram contados tres vezes e descontados tres vezes. Eles devem ser incluidos na contagem.
A resposta eh 105 480 + 6! = 106 200

Ricardo de Moraes (PS) wrote:

5. Determine o número de anagramas da palavra ESTUDANTE que tem E no primeiro lugar ou S no segundo lugar ou T no terceiro lugar.
Ricardo.
 

--------------080102040606010408020806-- ========================================================================= Instruções para entrar na lista, sair da lista e usar a lista em http://www.mat.puc-rio.br/~nicolau/olimp/obm-l.html O administrador desta lista é ========================================================================= From owner-obm-l@sucuri.mat.puc-rio.br Sat Apr 12 22:00:58 2003 Return-Path: Received: (from majordom@localhost) by sucuri.mat.puc-rio.br (8.9.3/8.9.3) id VAA23407 for obm-l-MTTP; Sat, 12 Apr 2003 21:57:35 -0300 Received: from www.zipmail.com.br (smtp.zipmail.com.br [200.221.11.147]) by sucuri.mat.puc-rio.br (8.9.3/8.9.3) with ESMTP id VAA23398 for ; Sat, 12 Apr 2003 21:57:24 -0300 From: camilojr@zipmail.com.br Received: from [200.214.79.186] by www.zipmail.com.br with HTTP; Sat, 12 Apr 2003 21:56:59 -0300 Message-ID: <3E980F4100001652@www.zipmail.com.br> Date: Sat, 12 Apr 2003 21:56:59 -0300 In-Reply-To: <5.2.0.9.0.20030412210037.00b18540@mail.digi.com.br> Subject: [obm-l] =?iso-8859-1?Q?Re=3A=20=5Bobm=2Dl=5D=20Um=20problema=20legal=20?= To: obm-l@mat.puc-rio.br MIME-Version: 1.0 Content-Type: text/plain; charset="iso-8859-1" Content-Transfer-Encoding: 8bit X-MIME-Autoconverted: from quoted-printable to 8bit by sucuri.mat.puc-rio.br id VAA23400 Sender: owner-obm-l@sucuri.mat.puc-rio.br Precedence: bulk Reply-To: obm-l@mat.puc-rio.br Apesar de não estar escrito, acho que está implícito que após inserir um cartão nós podemos utilizá-lo novamente (caso contrário, como usar a máquina que utiliza 2 cartões?). Mas se é assim mesmo, e não existe um número limite de operações, creio que existam infinitas soluções. Uma mais ou menos simples acho que é essa: 1)Usamos a máquina de somar um por 19 vezes. Guardamos 2 cartões: (8,22) e (24,38). 2)Colocamos esses 2 cartões na máquina de selecionar os primeiros números e teremos (8,24). 3)Usando a máquina de dividir por 3 vezes obtemos (1,3). 4)Pegamos o (24,38) e, se tivermos paciência pra isso, usamos máquina de somar um por 1964 vezes. Obtemos (1988, 2002). 5)Colocamos (1,3) e (1988, 2002) na máquina de selecionar os primeiros números e teremos (1,1988). Era só isso mesmo? Se for, acho que essa é a uma das soluções com o menor número de operações. um abraço, Camilo -- Mensagem original -- >Um problema interessante: > >Num salão de jogos, existem três máquinas que imprimem cartões. A primeira > >máquina funciona da seguinte maneira: se você insere um cartão com >quaisquer dois números a e b ela retorna um cartão com os números a >+ >1 e b + 1. A segunda máquina, aceita somente cartões numerados com dois > >números pares a e b e retorna um cartão numerado com os >números a/2 e b/2. A terceira, aceita somente dois cartões >numerados com a e b e c e d, respectivamente, e retorna um cartão > >numerado com a e c. Se você começa com um cartão numerado com 5 e >19, é possível obter um cartão com os números 1, 1988? > >Benedito Freire > >========================================================================= >Instruções para entrar na lista, sair da lista e usar a lista em >http://www.mat.puc-rio.br/~nicolau/olimp/obm-l.html >O administrador desta lista é >========================================================================= > ------------------------------------------ Use o melhor sistema de busca da Internet Radar UOL - http://www.radaruol.com.br ========================================================================= Instruções para entrar na lista, sair da lista e usar a lista em http://www.mat.puc-rio.br/~nicolau/olimp/obm-l.html O administrador desta lista é ========================================================================= From owner-obm-l@sucuri.mat.puc-rio.br Sat Apr 12 22:47:15 2003 Return-Path: Received: (from majordom@localhost) by sucuri.mat.puc-rio.br (8.9.3/8.9.3) id WAA24347 for obm-l-MTTP; Sat, 12 Apr 2003 22:44:37 -0300 Received: from trex-b.centroin.com.br (trex-b.centroin.com.br [200.225.63.136]) by sucuri.mat.puc-rio.br (8.9.3/8.9.3) with ESMTP id WAA24343 for ; Sat, 12 Apr 2003 22:44:34 -0300 Received: from centroin.com.br (RJ208093.user.veloxzone.com.br [200.165.208.93] (may be forged)) (authenticated bits=0) by trex-b.centroin.com.br (8.12.9/8.12.9) with ESMTP id h3D1i1iL015667 for ; Sat, 12 Apr 2003 22:44:02 -0300 (EST) Message-ID: <3E98C10F.3090100@centroin.com.br> Date: Sat, 12 Apr 2003 22:44:47 -0300 From: "A. C. Morgado" User-Agent: Mozilla/5.0 (Windows; U; Windows NT 5.0; en-US; rv:1.0.2) Gecko/20030208 Netscape/7.02 X-Accept-Language: en-us, en MIME-Version: 1.0 To: obm-l@mat.puc-rio.br Subject: Re: [obm-l] References: <27BD56F8640DD711A4320006295078E50500B7@pssnx1.brahma> Content-Type: multipart/alternative; boundary="------------020307050507070106070003" Sender: owner-obm-l@sucuri.mat.puc-rio.br Precedence: bulk Reply-To: obm-l@mat.puc-rio.br --------------020307050507070106070003 Content-Type: text/plain; charset=ISO-8859-1; format=flowed Content-Transfer-Encoding: 8bit Devemos determinar o numero de soluçoes de x+y+z = 10 com x, y, z inteiros em [1, 6] Pondo x=1+a, y=1+b e z=1+c, devemos determinar o numero de soluçoes inteiras de a+b+c = 7 com a, b, c em [0, 5] O numero de soluçoes inteiras de a+b+c=7 com a, b, c maiores ou iguais a 0 eh CR(3, 7) = C(9, 7) = 36. Devemos subtrair as soluçoes com alguem maior que 5. Com a=6 ha 2 soluçoes, analogo para b=6 , c=6. Com a=7 so ha uma soluçao, analogo para b=7, a=7. A resposta eh 36 - 3*2 - 3*1 = 27 Ricardo de Moraes (PS) wrote: > > 4.Lançam-se 3 dados. Em quantos dos possíveis resultados a soma dos > valores das faces é 10? > > Ricardo. > --------------020307050507070106070003 Content-Type: text/html; charset=us-ascii Content-Transfer-Encoding: 7bit Devemos determinar o numero de soluçoes de
x+y+z = 10 com x, y, z inteiros em [1, 6]
Pondo x=1+a, y=1+b e z=1+c, devemos determinar o numero de soluçoes inteiras de
a+b+c = 7 com a, b, c em [0, 5]
O numero de soluçoes inteiras de a+b+c=7 com a, b, c maiores ou iguais a 0 eh CR(3, 7) =
C(9, 7) = 36. Devemos subtrair as soluçoes com alguem maior que 5.
Com a=6 ha 2 soluçoes, analogo para b=6 , c=6. Com a=7 so ha uma soluçao, analogo para b=7, a=7.
A resposta eh 36 - 3*2 - 3*1 = 27
Ricardo de Moraes (PS) wrote:
 
4.Lançam-se 3 dados. Em quantos dos possíveis resultados a soma dos valores das faces é 10?
 
Ricardo.
 

--------------020307050507070106070003-- ========================================================================= Instruções para entrar na lista, sair da lista e usar a lista em http://www.mat.puc-rio.br/~nicolau/olimp/obm-l.html O administrador desta lista é ========================================================================= From owner-obm-l@sucuri.mat.puc-rio.br Sun Apr 13 02:21:04 2003 Return-Path: Received: (from majordom@localhost) by sucuri.mat.puc-rio.br (8.9.3/8.9.3) id CAA26910 for obm-l-MTTP; Sun, 13 Apr 2003 02:17:58 -0300 Received: from web13708.mail.yahoo.com (web13708.mail.yahoo.com [216.136.175.141]) by sucuri.mat.puc-rio.br (8.9.3/8.9.3) with SMTP id CAA26905 for ; Sun, 13 Apr 2003 02:17:54 -0300 Message-ID: <20030413051723.27045.qmail@web13708.mail.yahoo.com> Received: from [200.213.88.163] by web13708.mail.yahoo.com via HTTP; Sun, 13 Apr 2003 02:17:23 ART Date: Sun, 13 Apr 2003 02:17:23 -0300 (ART) From: =?iso-8859-1?q?pichurin?= Subject: [obm-l] demonstração To: obm-l@mat.puc-rio.br MIME-Version: 1.0 Content-Type: text/plain; charset=iso-8859-1 Content-Transfer-Encoding: 8bit Sender: owner-obm-l@sucuri.mat.puc-rio.br Precedence: bulk Reply-To: obm-l@mat.puc-rio.br Demonstre que a solução de f'(x)=f(x) são as funções f(x)=ke^k. _______________________________________________________________________ Yahoo! Mail O melhor e-mail gratuito da internet: 6MB de espaço, antivírus, acesso POP3, filtro contra spam. http://br.mail.yahoo.com/ ========================================================================= Instruções para entrar na lista, sair da lista e usar a lista em http://www.mat.puc-rio.br/~nicolau/olimp/obm-l.html O administrador desta lista é ========================================================================= From owner-obm-l@sucuri.mat.puc-rio.br Sun Apr 13 02:30:18 2003 Return-Path: Received: (from majordom@localhost) by sucuri.mat.puc-rio.br (8.9.3/8.9.3) id CAA27004 for obm-l-MTTP; Sun, 13 Apr 2003 02:27:32 -0300 Received: from web13703.mail.yahoo.com (web13703.mail.yahoo.com [216.136.175.136]) by sucuri.mat.puc-rio.br (8.9.3/8.9.3) with SMTP id CAA27000 for ; Sun, 13 Apr 2003 02:27:29 -0300 Message-ID: <20030413052658.1653.qmail@web13703.mail.yahoo.com> Received: from [200.213.88.163] by web13703.mail.yahoo.com via HTTP; Sun, 13 Apr 2003 02:26:58 ART Date: Sun, 13 Apr 2003 02:26:58 -0300 (ART) From: =?iso-8859-1?q?pichurin?= Subject: [obm-l] +Demonstrações To: obm-l@mat.puc-rio.br MIME-Version: 1.0 Content-Type: text/plain; charset=iso-8859-1 Content-Transfer-Encoding: 8bit Sender: owner-obm-l@sucuri.mat.puc-rio.br Precedence: bulk Reply-To: obm-l@mat.puc-rio.br Demonstre que o subespaço das soluções de uma equação linear diferencial ordinária homogênea de ordem n com coeficientes constantes tem dimensão n. _______________________________________________________________________ Yahoo! Mail O melhor e-mail gratuito da internet: 6MB de espaço, antivírus, acesso POP3, filtro contra spam. http://br.mail.yahoo.com/ ========================================================================= Instruções para entrar na lista, sair da lista e usar a lista em http://www.mat.puc-rio.br/~nicolau/olimp/obm-l.html O administrador desta lista é ========================================================================= From owner-obm-l@sucuri.mat.puc-rio.br Sun Apr 13 02:36:10 2003 Return-Path: Received: (from majordom@localhost) by sucuri.mat.puc-rio.br (8.9.3/8.9.3) id CAA27261 for obm-l-MTTP; Sun, 13 Apr 2003 02:33:22 -0300 Received: from web13707.mail.yahoo.com (web13707.mail.yahoo.com [216.136.175.140]) by sucuri.mat.puc-rio.br (8.9.3/8.9.3) with SMTP id CAA27257 for ; Sun, 13 Apr 2003 02:33:18 -0300 Message-ID: <20030413053247.30777.qmail@web13707.mail.yahoo.com> Received: from [200.213.88.163] by web13707.mail.yahoo.com via HTTP; Sun, 13 Apr 2003 02:32:47 ART Date: Sun, 13 Apr 2003 02:32:47 -0300 (ART) From: =?iso-8859-1?q?pichurin?= Subject: [obm-l] O que é o que é? To: obm-l@mat.puc-rio.br MIME-Version: 1.0 Content-Type: text/plain; charset=iso-8859-1 Content-Transfer-Encoding: 8bit Sender: owner-obm-l@sucuri.mat.puc-rio.br Precedence: bulk Reply-To: obm-l@mat.puc-rio.br o que é uma equação diferencial ORDINÁRIA?O nominho escroto.... _______________________________________________________________________ Yahoo! Mail O melhor e-mail gratuito da internet: 6MB de espaço, antivírus, acesso POP3, filtro contra spam. http://br.mail.yahoo.com/ ========================================================================= Instruções para entrar na lista, sair da lista e usar a lista em http://www.mat.puc-rio.br/~nicolau/olimp/obm-l.html O administrador desta lista é ========================================================================= From owner-obm-l@sucuri.mat.puc-rio.br Sun Apr 13 02:46:23 2003 Return-Path: Received: (from majordom@localhost) by sucuri.mat.puc-rio.br (8.9.3/8.9.3) id CAA27647 for obm-l-MTTP; Sun, 13 Apr 2003 02:43:28 -0300 Received: from hotmail.com (oe22.law10.hotmail.com [64.4.14.79]) by sucuri.mat.puc-rio.br (8.9.3/8.9.3) with ESMTP id CAA27642 for ; Sun, 13 Apr 2003 02:43:24 -0300 Received: from mail pickup service by hotmail.com with Microsoft SMTPSVC; Sat, 12 Apr 2003 22:42:53 -0700 Received: from 67.25.243.23 by law10-oe22.adinternal.hotmail.com with DAV; Sun, 13 Apr 2003 05:42:53 +0000 X-Originating-IP: [67.25.243.23] X-Originating-Email: [lrecova@hotmail.com] From: =?iso-8859-1?Q?Leandro_Lacorte_Rec=F4va?= To: Subject: [obm-l] =?iso-8859-1?Q?RE:_=5Bobm-l=5D_Re:=5Bobm-l=5D_Re:_=5Bobm-l=5D_c=E1lcu?= =?iso-8859-1?Q?lo-engenharia?= Date: Sat, 12 Apr 2003 22:42:51 -0700 Message-ID: <001101c3017f$85c524f0$17f31943@LeandroRecova> MIME-Version: 1.0 Content-Type: text/plain; charset="iso-8859-1" Content-Transfer-Encoding: 7bit X-Priority: 3 (Normal) X-MSMail-Priority: Normal X-Mailer: Microsoft Outlook, Build 10.0.3416 Importance: Normal X-MimeOLE: Produced By Microsoft MimeOLE V6.00.2800.1106 In-Reply-To: X-OriginalArrivalTime: 13 Apr 2003 05:42:53.0390 (UTC) FILETIME=[86FFEEE0:01C3017F] Sender: owner-obm-l@sucuri.mat.puc-rio.br Precedence: bulk Reply-To: obm-l@mat.puc-rio.br Caro colega, Eu ja usei o livro do Shokowski quando lecionei calculo 1 para uma turma de engenharia eletrica em Brasilia. O livro e bom, mas como o Claudio disse, se voce quiser aprender analise e conhecer os fundamentos do calculo os livros do Elon sao otima referencia. Mas, nao esqueca, mesmo quem faz um curso de matematica pura passa por um curso de calculo antes. Por isso, comece pelos livros do Showkoski, Lethold, e faca bastante exercicios. Tem um livro russo bom de exercicios do Demidovith e um do IEEE. Depois de um curso de calculo, voce nao voltara mais a ver matematica num curso de engenharia (o que e lamentalvemelte uma pena para aqueles que gostam da matematica), e caso tenha interesse em continuar, ai sim, pegue umas materias de analise, algebra que sao base para entender outras disciplinas mais avancadas. Leandro ========================================================================= Instruções para entrar na lista, sair da lista e usar a lista em http://www.mat.puc-rio.br/~nicolau/olimp/obm-l.html O administrador desta lista é ========================================================================= From owner-obm-l@sucuri.mat.puc-rio.br Sun Apr 13 07:51:43 2003 Return-Path: Received: (from majordom@localhost) by sucuri.mat.puc-rio.br (8.9.3/8.9.3) id HAA02155 for obm-l-MTTP; Sun, 13 Apr 2003 07:48:20 -0300 Received: from trex.centroin.com.br (trex.centroin.com.br [200.225.63.134]) by sucuri.mat.puc-rio.br (8.9.3/8.9.3) with ESMTP id HAA02151 for ; Sun, 13 Apr 2003 07:48:16 -0300 Received: from trex.centroin.com.br (localhost [127.0.0.1]) by trex.centroin.com.br (8.12.9/8.12.9) with ESMTP id h3DAlpOp000650 for ; Sun, 13 Apr 2003 07:47:51 -0300 (EST) Received: by trex.centroin.com.br (8.12.9/8.12.5/Submit) id h3DAlpeA000649; Sun, 13 Apr 2003 07:47:51 -0300 (EST) Message-Id: <200304131047.h3DAlpeA000649@trex.centroin.com.br> Received: from 200.165.208.144 by trex.centroin.com.br (CIPWM versao 1.4C1) with HTTPS for ; Sun, 13 Apr 2003 07:47:51 -0300 (EST) Date: Sun, 13 Apr 2003 07:47:51 -0300 (EST) From: Augusto Cesar de Oliveira Morgado To: obm-l@mat.puc-rio.br Subject: =?iso-8859-1?q?Re: [obm-l] O que =E9 o que =E9??= MIME-Version: 1.0 X-Mailer: CentroIn Internet Provider WebMail v. 1.4C1 (http://www.centroin.com.br/) Content-Type: text/plain; charset="iso-8859-1" Content-Transfer-Encoding: 8bit X-MIME-Autoconverted: from quoted-printable to 8bit by sucuri.mat.puc-rio.br id HAA02152 Sender: owner-obm-l@sucuri.mat.puc-rio.br Precedence: bulk Reply-To: obm-l@mat.puc-rio.br Ordinario significa comum! Nos, no Brasil, eh que atribuimos a essa palavra um sentido que ela ordinariamente nao tinha: o de vagabundo, de segunda categoria. Megalomania dos brasucas que querem, comumente, apenas coisas extraordinarias. Mas, no caso das equaçoes diferenciais, o ordinaria eh usado para distingui-las das equaçoes em derivadas parciais. Em Sun, 13 Apr 2003 02:32:47 -0300 (ART), pichurin disse: > o que é uma equação diferencial ORDINÁRIA?O nominho escroto.... > > _______________________________________________________________________ > Yahoo! Mail > O melhor e-mail gratuito da internet: 6MB de espaço, antivírus, acesso POP3, filtro contra spam. > http://br.mail.yahoo.com/ > ========================================================================= > Instruções para entrar na lista, sair da lista e usar a lista em > http://www.mat.puc-rio.br/~nicolau/olimp/obm-l.html > O administrador desta lista é > ========================================================================= > > ========================================================================= Instruções para entrar na lista, sair da lista e usar a lista em http://www.mat.puc-rio.br/~nicolau/olimp/obm-l.html O administrador desta lista é ========================================================================= From owner-obm-l@sucuri.mat.puc-rio.br Sun Apr 13 07:59:34 2003 Return-Path: Received: (from majordom@localhost) by sucuri.mat.puc-rio.br (8.9.3/8.9.3) id HAA02270 for obm-l-MTTP; Sun, 13 Apr 2003 07:56:53 -0300 Received: from trex.centroin.com.br (trex.centroin.com.br [200.225.63.134]) by sucuri.mat.puc-rio.br (8.9.3/8.9.3) with ESMTP id HAA02266 for ; Sun, 13 Apr 2003 07:56:51 -0300 Received: from trex.centroin.com.br (localhost [127.0.0.1]) by trex.centroin.com.br (8.12.9/8.12.9) with ESMTP id h3DAuOOp002567 for ; Sun, 13 Apr 2003 07:56:24 -0300 (EST) Received: by trex.centroin.com.br (8.12.9/8.12.5/Submit) id h3DAuO7f002566; Sun, 13 Apr 2003 07:56:24 -0300 (EST) Message-Id: <200304131056.h3DAuO7f002566@trex.centroin.com.br> Received: from 200.165.208.144 by trex.centroin.com.br (CIPWM versao 1.4C1) with HTTPS for ; Sun, 13 Apr 2003 07:56:24 -0300 (EST) Date: Sun, 13 Apr 2003 07:56:24 -0300 (EST) From: Augusto Cesar de Oliveira Morgado To: obm-l@mat.puc-rio.br Subject: =?iso-8859-1?q?Re: [obm-l] demonstra=E7=E3o?= MIME-Version: 1.0 X-Mailer: CentroIn Internet Provider WebMail v. 1.4C1 (http://www.centroin.com.br/) Content-Type: text/plain; charset="iso-8859-1" Content-Transfer-Encoding: 8bit X-MIME-Autoconverted: from quoted-printable to 8bit by sucuri.mat.puc-rio.br id HAA02267 Sender: owner-obm-l@sucuri.mat.puc-rio.br Precedence: bulk Reply-To: obm-l@mat.puc-rio.br Corrigindo, sao as funçoes f(x)= k e^x Prove que: 1)se f'(x)= f(x), entao f(x)/e^x eh constante. Isso eh facil, para mostrar que eh constante basta achar a derivada e constatar que a derivada vale zero na reta toda 2)se f(x)= k e^x, entao f'(x) = f(x) Em Sun, 13 Apr 2003 02:17:23 -0300 (ART), pichurin disse: > Demonstre que a solução de f'(x)=f(x) são as funções f(x)=ke^k. > > _______________________________________________________________________ > Yahoo! Mail > O melhor e-mail gratuito da internet: 6MB de espaço, antivírus, acesso POP3, filtro contra spam. > http://br.mail.yahoo.com/ > ========================================================================= > Instruções para entrar na lista, sair da lista e usar a lista em > http://www.mat.puc-rio.br/~nicolau/olimp/obm-l.html > O administrador desta lista é > ========================================================================= > > ========================================================================= Instruções para entrar na lista, sair da lista e usar a lista em http://www.mat.puc-rio.br/~nicolau/olimp/obm-l.html O administrador desta lista é ========================================================================= From owner-obm-l@sucuri.mat.puc-rio.br Sun Apr 13 12:49:16 2003 Return-Path: Received: (from majordom@localhost) by sucuri.mat.puc-rio.br (8.9.3/8.9.3) id MAA05083 for obm-l-MTTP; Sun, 13 Apr 2003 12:43:46 -0300 Received: from traven10.uol.com.br (traven10.uol.com.br [200.221.29.45]) by sucuri.mat.puc-rio.br (8.9.3/8.9.3) with ESMTP id MAA05079 for ; Sun, 13 Apr 2003 12:43:43 -0300 Received: from u2z7z2 ([200.158.145.76]) by traven10.uol.com.br (8.9.1/8.9.1) with ESMTP id MAA10685 for ; Sun, 13 Apr 2003 12:43:12 -0300 (BRT) Message-ID: <003401c301d3$4c9b9de0$2101a8c0@u2z7z2> From: "Wagner" To: Subject: [obm-l] =?iso-8859-1?Q?Simplifica=E7=E3o?= Date: Sun, 13 Apr 2003 12:42:32 -0300 Organization: Wagner MIME-Version: 1.0 Content-Type: multipart/alternative; boundary="----=_NextPart_000_0031_01C301BA.26FA7980" X-Priority: 3 X-MSMail-Priority: Normal X-Mailer: Microsoft Outlook Express 5.50.4133.2400 X-MimeOLE: Produced By Microsoft MimeOLE V5.50.4133.2400 Sender: owner-obm-l@sucuri.mat.puc-rio.br Precedence: bulk Reply-To: obm-l@mat.puc-rio.br This is a multi-part message in MIME format. ------=_NextPart_000_0031_01C301BA.26FA7980 Content-Type: text/plain; charset="iso-8859-1" Content-Transfer-Encoding: quoted-printable Oi para todos! Em que casos =E9 poss=EDvel simplificar a express=E3o abaixo ? (a+b)^(1/3) + (a-b)^(1/3) Andr=E9 T. ------=_NextPart_000_0031_01C301BA.26FA7980 Content-Type: text/html; charset="iso-8859-1" Content-Transfer-Encoding: quoted-printable
Oi para todos!
 
Em que casos =E9 poss=EDvel simplificar = a express=E3o=20 abaixo ?
 
(a+b)^(1/3) + (a-b)^(1/3)
 
Andr=E9 T.
------=_NextPart_000_0031_01C301BA.26FA7980-- ========================================================================= Instruções para entrar na lista, sair da lista e usar a lista em http://www.mat.puc-rio.br/~nicolau/olimp/obm-l.html O administrador desta lista é ========================================================================= From owner-obm-l@sucuri.mat.puc-rio.br Sun Apr 13 12:52:09 2003 Return-Path: Received: (from majordom@localhost) by sucuri.mat.puc-rio.br (8.9.3/8.9.3) id MAA05094 for obm-l-MTTP; Sun, 13 Apr 2003 12:46:27 -0300 Received: from artemis.opendf.com.br (artemis.opengate.com.br [200.181.71.14]) by sucuri.mat.puc-rio.br (8.9.3/8.9.3) with ESMTP id MAA05090 for ; Sun, 13 Apr 2003 12:46:23 -0300 Received: from localhost (localhost [127.0.0.1]) by artemis.opendf.com.br (Postfix) with ESMTP id A12312BECD for ; Sun, 13 Apr 2003 12:47:06 -0300 (BRT) Received: from artemis.opendf.com.br ([127.0.0.1]) by localhost (artemis.opengate.com.br [127.0.0.1:10024]) (amavisd-new) with ESMTP id 30886-09 for ; Sun, 13 Apr 2003 12:47:05 -0300 (BRT) Received: from computer (200-181-090-143.bsace7001.dsl.brasiltelecom.net.br [200.181.90.143]) by artemis.opendf.com.br (Postfix) with ESMTP id F233C2BEC2 for ; Sun, 13 Apr 2003 12:47:04 -0300 (BRT) From: "Artur Costa Steiner" To: Subject: [obm-l] =?iso-8859-1?Q?RE:_=5Bobm-l=5D_demonstra=E7=E3o?= Date: Sun, 13 Apr 2003 12:45:50 -0300 Organization: Steiner Consultoria LTDA Message-ID: MIME-Version: 1.0 Content-Type: multipart/mixed; boundary="----=_NextPart_000_0036_01C301BA.9D40F290" X-Priority: 3 (Normal) X-MSMail-Priority: Normal X-Mailer: Microsoft Outlook, Build 10.0.2627 Importance: Normal X-MimeOLE: Produced By Microsoft MimeOLE V6.00.2800.1106 In-Reply-To: <20030413051723.27045.qmail@web13708.mail.yahoo.com> X-MS-TNEF-Correlator: 000000005FC3F71D8B4BB941A8AA63D06F261C3624AD2A00 X-Virus-Scanned: by amavisd-new Sender: owner-obm-l@sucuri.mat.puc-rio.br Precedence: bulk Reply-To: obm-l@mat.puc-rio.br This is a multi-part message in MIME format. ------=_NextPart_000_0036_01C301BA.9D40F290 Content-Type: text/plain; charset="iso-8859-1" Content-Transfer-Encoding: quoted-printable > >Demonstre que a solu=E7=E3o de f'(x)=3Df(x) s=E3o as fun=E7=F5es = f(x)=3Dke^k. [Artur Costa Steiner] Oi Pichurin f(x) =3D ke^x, ok? Para uma demonstra=E7=E3o consistente na reta real, = podemos fazer o seguinte.=20 Atraves de um racioc=EDnio indutivo simples, concluimos que f apresenta derivadas de todas as ordens em R e que f =3D f' =3D f''.... Sendo k =3D = f(0), temos entao que k =3D f(0) =3D f'(0) =3D f''(0)=3D..... Destas = condicoes, segue-se que, para qualquer x real, o Teorema de Taylor aplicase ao intervalo = fechado I de pontos extremos 0 e x. Existe portanto um real a entre 0 e x tal = que, para todo natural n, f(x) =3D f(0) +x f'(0) + ...x^n/n! f_n(a), onde f_n = eh a n-gesima derivada d f. Mas, em virtude do que vimos, temos que f(x) =3D = k + kx + kx^2/2! .... + x^n/n! f(a) =3D P_n(x) + R_n(x), onde P_n (de grau n-1) = eh o polinomio de Taylor e R_n o resto de Lagrange. R_n eh assim uma = sequencia que depende de x, pois a depende de x.=20 Mas, em virtude de sua diferenciabilidade em R, logo no intervalo = fechado I, de pontos extremos 0 e x, segue-se que f eh continua e, portanto, = limitada em I. Existe assim um real M tal que |f(y)| < M para todo y em I. Como a estah em I, isto nos mostra que, para todo n, |R_n(x)| =3D |x^n/n! f(a)| = < M |x^n/n!|. Observemos que M depende de x mas naum de n. Conforme = sabemos, para todo real x a sequencia de funcoes x^n/n! converge para zero, o que acarreta automaticamente que R_n(x)tambem convirja para zero. Logo, para todo real x o resto de Lagrange converge para zero, o que garante que f possa ser expandida em serie infinita de Taylor por f(x) =3D k + kx + kx^2/2!.... kx^n/n!.... Comparando-se esta expans=E3o com a definicao de = e^x, segundo a qual e^x =3D 1 + x + x^2/2!... + x^n/n!...concluimos, com base = nas propriedades dos limites de series, que f(x)=3D ke^x para todo x real. Um abraco Artur ------=_NextPart_000_0036_01C301BA.9D40F290 Content-Type: application/ms-tnef; name="winmail.dat" Content-Transfer-Encoding: base64 Content-Disposition: attachment; filename="winmail.dat" eJ8+IjMPAQaQCAAEAAAAAAABAAEAAQeQBgAIAAAA5AQAAAAAAADoAAEIgAcAGAAAAElQTS5NaWNy b3NvZnQgTWFpbC5Ob3RlADEIAQ2ABAACAAAAAgACAAEGAAcAAQAAAAAAAAEGgAMADgAAANMHBAAN AAwALQAAAAAAJAEBA5AGAEgKAAAuAAAACwACAAEAAAALACMAAAAAAAMAJgAAAAAACwApAAAAAAAD AC4AAAAAAAIBMQABAAAAGAAAAAAAAABfw/cdi0u5QaiqY9BvJhw25KwqAAMANgAAAAAAHgBwAAEA AAAVAAAAW29ibS1sXSBkZW1vbnN0cmHn428AAAAAAgFxAAEAAAAWAAAAAcMB02zVsMgXt9zASiGA AYb4ANsyAQAAAgEdDAEAAAAbAAAAU01UUDpBUlRVUl9TVEVJTkVSQFVTQS5ORVQAAAsAAQ4AAAAA QAAGDgAmI6TTAcMBAgEKDgEAAAAYAAAAAAAAAF/D9x2LS7lBqKpj0G8mHDbCgAAAAwAUDgAAAAAL AB8OAQAAAB4AKA4BAAAAKgAAADAwMDAwMDA0AWFydHVyX3N0ZWluZXJAdXNhLm5ldAFOZXRhZGRy ZXNzAAAAHgApDgEAAAAqAAAAMDAwMDAwMDQBYXJ0dXJfc3RlaW5lckB1c2EubmV0AU5ldGFkZHJl c3MAAAACAQkQAQAAACQFAAAgBQAAYAgAAExaRnXz2O+HAwAKAHJjcGcxMjXiMgNDdGV4BUEBAwH3 TwqAAqQD4wIAY2gKwHPwZXQwIAcTAoAP8wBQfwRWCFUHshHFDlEDARDHMvcGAAbDEcUzBEYQyRLb EdPbCO8J9zsYvw4wNRHCDGDOYwBQCwkBZDM2EVALpowgPgqiCoA+RGUEYCUAgHQYwCBxClAgYYwg cwbwDHAnZTcfUQgzbyABACBmJyhweCk9ZiBBHwAflGGZBCBmdQuQH2NmNQeRgSCCPWtlXmsuHaR6 WwcQdAhwEsEeYB7wUwcOsAuABJBdIE9pIPpQDeBoCHELkB2zHaQggwQ9ICKheCwgb2vuPyTQCsAe 8HUAwB/hHjR+YR9YBaAAgR5gCfAOsCB+bh7wGMAj8RjAB0Am0HDnBHEEYCFRYXoTIR/QESBsZ3UL gA6wLgrjCoBBfSghdgeRH/EngCnQANBpdm8A4B9gZAMAH9ALgGTwdXRpditxB3ALUAeQeybQKPFj CkAHcCrhHrJmfR7gcBjAESACMCeiBRB27GFkIUEf8XQEcCFBIUH/BbABAAYxHiAH8DLgMBUmcPMg IDOzJy40YQZRLmAf0OJrM7IoMCkm0A6wKtJ/MNIf0B6yNRYzszcGN3I9vzRiLBAeECPhBCAo8WQN 4OZvLzIrkmUtESAeoipx3ydCHrAHQB6xBcB4KiUf0DxUZQWwHiAnojwgYXn7GFEwcWwN4CFAHtEu Mg6wfnIxYBhQIBAFkBDwNOFJ/x/iKpACMDXiDtA8YSrhEVDlHpB4LBBFeCkyKoEAIP8AcDHwLTMq QR7hKWEegUCD/zHgQkE6aDHxH9ApsCOBQkFubibQJiU28ys7kDdkKwIgNGF4Xm4vbiHhIBBfbihh NYE5MSABi0dQMuBoHuFuLWcHkN8HcDEIH+AgECwQTSFAJtD5MvF2aSNxH/E04R6ySpDfKtE1ljAj JjVGcWs7kE0i8F4yLzJHIDRjRoBG1rtHcSZhUEdRIKFGgFJPczNHpU9hICgf8QnAYXV9SJExILBI UR/QKpA9cG7/A3AuITy4HpBP8SthMKFBwekf8UxhUUFuSMAsEFOS/0hSBBAHcCdzESAesS+QBzD7 HqMBAHA0wVbyO4AqcgQA3x7hVxosFkofHpBzOxAf4NcGkASQVnNiAxBpMYAf8fcy8ibQGFBnRGE9 /z8CJtB/P19AZTm8MFFIUSjxLpBu31rhOoJBdVyBB3BpAZBJkV0y8UlA11WWQhRNQ0YggnwggHkp fCA8ZMH9Q+h5YxUIUARgQmIj4Uhg/2MiJtApMVzSBCAq0SghQ4/dJtB8T/RlwCZwfE6JZcDLZdNr RXwsEE9iESA+UP9MB2TQVwto8CFBKbAtQR/x6m5nIm4CEHIHgB8AAaD/KsJDykIjO5BWGh/yIYA5 cvdOdijxLNBySMA6pCsxYmH/NlQA0ArAKeNRYDHwAMAukP89kAeAKXIesk/0AZAG0DLxu3QCSqBq HvB0lywQTFyxv3D/O5BT33RhdA91F2cnQb929jBgKpAEEFYSU1F4CrDfOUFi9G0xCJAuQWYLgGLB /24SPNZBYUyPTZU0Y02RRvL/NGNnQTqyNNE6IjjCf0Qg1f8FoC1QJ6KAoj2QH9MmoyuS7zTSOuSH kiZhMU5STlODNd9OVzRhL3gvUi1QYj2iKbD7BCAwkG8wkAiQW/Is8Srh/2KTLOSAIy9BTFYmdEPZ O5TVIuVVhpFiLXFvLDUjcgUdpH2S8B4AQhABAAAANQAAADwyMDAzMDQxMzA1MTcyMy4yNzA0NS5x bWFpbEB3ZWIxMzcwOC5tYWlsLnlhaG9vLmNvbT4AAAAAAwCSEAEAAAACARQ6AQAAABAAAABMXH6k I/FZTIPYndI7Wj5tAwDeP69vAAADAAlZAQAAAAMAQGUAAAAACwATgAggBgAAAAAAwAAAAAAAAEYA AAAAA4UAAAAAAAADABWACCAGAAAAAADAAAAAAAAARgAAAAAQhQAAAAAAAAMAG4AIIAYAAAAAAMAA AAAAAABGAAAAAFKFAADjkAEAAwAigAggBgAAAAAAwAAAAAAAAEYAAAAAAYUAAAAAAABAACOACCAG AAAAAADAAAAAAAAARgAAAABghQAAANCIwxAAAAAeAEGACCAGAAAAAADAAAAAAAAARgAAAABUhQAA AQAAAAUAAAAxMC4wAAAAAAsAQoAIIAYAAAAAAMAAAAAAAABGAAAAAAaFAAAAAAAACwBGgAggBgAA AAAAwAAAAAAAAEYAAAAADoUAAAAAAAADAEmACCAGAAAAAADAAAAAAAAARgAAAAAYhQAAAAAAAAsA XoAIIAYAAAAAAMAAAAAAAABGAAAAAIKFAAABAAAAAgH4DwEAAAAQAAAAX8P3HYtLuUGoqmPQbyYc NgIB+g8BAAAAEAAAAF/D9x2LS7lBqKpj0G8mHDYCAfsPAQAAAIQAAAAAAAAAOKG7EAXlEBqhuwgA KypWwgAAbXNwc3QuZGxsAAAAAABOSVRB+b+4AQCqADfZbgAAAEM6XERvY3VtZW50cyBhbmQgU2V0 dGluZ3NcQXJ0dXJcQXBwbGljYXRpb24gRGF0YVxNaWNyb3NvZnRcT3V0bG9va1xPdXRsb29rLnBz dAADAP4PBQAAAAMADTT9NwIAAgEUNAEAAAAQAAAATklUQfm/uAEAqgA32W4AAAIBfwABAAAAMQAA ADAwMDAwMDAwNUZDM0Y3MUQ4QjRCQjk0MUE4QUE2M0QwNkYyNjFDMzYyNEFEMkEwMAAAAAADAAYQ SlMiqAMABxBPBQAAAwAQEAIAAAADABEQAAAAAB4ACBABAAAAZQAAAERFTU9OU1RSRVFVRUFTT0xV 5+NPREVGKFgpPUYoWClT409BU0ZVTuf1RVNGKFgpPUtFS0FSVFVSQ09TVEFTVEVJTkVST0lQSUNI VVJJTkYoWCk9S0VYLE9LP1BBUkFVTUFERU0AAAAAVt4= ------=_NextPart_000_0036_01C301BA.9D40F290-- ========================================================================= Instruções para entrar na lista, sair da lista e usar a lista em http://www.mat.puc-rio.br/~nicolau/olimp/obm-l.html O administrador desta lista é ========================================================================= From owner-obm-l@sucuri.mat.puc-rio.br Sun Apr 13 13:13:22 2003 Return-Path: Received: (from majordom@localhost) by sucuri.mat.puc-rio.br (8.9.3/8.9.3) id NAA05790 for obm-l-MTTP; Sun, 13 Apr 2003 13:07:57 -0300 Received: from artemis.opendf.com.br (artemis.opengate.com.br [200.181.71.14]) by sucuri.mat.puc-rio.br (8.9.3/8.9.3) with ESMTP id NAA05785 for ; Sun, 13 Apr 2003 13:07:53 -0300 Received: from localhost (localhost [127.0.0.1]) by artemis.opendf.com.br (Postfix) with ESMTP id 4BAC92BECD for ; Sun, 13 Apr 2003 13:08:37 -0300 (BRT) Received: from artemis.opendf.com.br ([127.0.0.1]) by localhost (artemis.opengate.com.br [127.0.0.1:10024]) (amavisd-new) with ESMTP id 31306-06 for ; Sun, 13 Apr 2003 13:08:36 -0300 (BRT) Received: from computer (200-181-090-143.bsace7001.dsl.brasiltelecom.net.br [200.181.90.143]) by artemis.opendf.com.br (Postfix) with ESMTP id 110E72BEC2 for ; Sun, 13 Apr 2003 13:08:36 -0300 (BRT) From: "Artur Costa Steiner" To: Subject: [obm-l] =?iso-8859-1?Q?RE:_=5Bobm-l=5D_Combina=E7=F5es_completas_?= Date: Sun, 13 Apr 2003 13:07:21 -0300 Organization: Steiner Consultoria LTDA Message-ID: <003b01c301d6$c3fd6aa0$9865fea9@computer> MIME-Version: 1.0 Content-Type: text/plain; charset="iso-8859-1" X-Priority: 3 (Normal) X-MSMail-Priority: Normal X-Mailer: Microsoft Outlook, Build 10.0.2627 Importance: Normal X-MimeOLE: Produced By Microsoft MimeOLE V6.00.2800.1106 In-Reply-To: <3E98A296.5090107@centroin.com.br> X-Virus-Scanned: by amavisd-new Content-Transfer-Encoding: 8bit X-MIME-Autoconverted: from quoted-printable to 8bit by sucuri.mat.puc-rio.br id NAA05786 Sender: owner-obm-l@sucuri.mat.puc-rio.br Precedence: bulk Reply-To: obm-l@mat.puc-rio.br Ta certo. So que eh mais comum escrever CR e nao CC. CR(n, p) = C(n+p-1,p). Ha uma deduçao em  www.fgv.br   Entre em graduaçao RJ procure pelas disciplinas do ciclo basico (estao dentro de programaçao academica), em teoria da probabilidade procure por materiais. no arquivo combinatoria 2 voce encontra. [ ]s Morgado [Artur Costa Steiner] Muitro Obrigado ========================================================================= Instruções para entrar na lista, sair da lista e usar a lista em http://www.mat.puc-rio.br/~nicolau/olimp/obm-l.html O administrador desta lista é ========================================================================= From owner-obm-l@sucuri.mat.puc-rio.br Sun Apr 13 13:14:05 2003 Return-Path: Received: (from majordom@localhost) by sucuri.mat.puc-rio.br (8.9.3/8.9.3) id NAA05823 for obm-l-MTTP; Sun, 13 Apr 2003 13:08:47 -0300 Received: from hotmail.com (f15.sea2.hotmail.com [207.68.165.15]) by sucuri.mat.puc-rio.br (8.9.3/8.9.3) with ESMTP id NAA05819 for ; Sun, 13 Apr 2003 13:08:43 -0300 Received: from mail pickup service by hotmail.com with Microsoft SMTPSVC; Sun, 13 Apr 2003 09:08:12 -0700 Received: from 200.214.81.99 by sea2fd.sea2.hotmail.msn.com with HTTP; Sun, 13 Apr 2003 16:08:09 GMT X-Originating-IP: [200.214.81.99] X-Originating-Email: [p_ssr@hotmail.com] From: "Paulo Santa Rita" To: obm-l@mat.puc-rio.br Subject: [obm-l] =?iso-8859-1?B?UmU6IFtvYm0tbF0gU2VxdepuY2lhcw==?= Date: Sun, 13 Apr 2003 16:08:09 +0000 Mime-Version: 1.0 Content-Type: text/plain; charset=iso-8859-1; format=flowed Message-ID: X-OriginalArrivalTime: 13 Apr 2003 16:08:12.0095 (UTC) FILETIME=[E1E3FCF0:01C301D6] Sender: owner-obm-l@sucuri.mat.puc-rio.br Precedence: bulk Reply-To: obm-l@mat.puc-rio.br Ola Igor e demais colegas desta lista ... OBM-L, Sim, existe. Tomando quaisquer tres termos consecutivos ( digamos : Ai-1, Ai e Ai+1 ) pode-se verificar que : Ai+1 - 2*Ai + Ai-1 = 2 => (Ai+1 - Ai) - (Ai - Ai-1) = 2 Isto prova que estamos diante de uma PA2, vale dizer, diante de uma Progressao Aritmetica de 2 ordem. A soma dos N primeiros termos de uma sequencia deste tipo e o "produto escalar" : Sn = < A1,A2-A1,A3-2*A2+A3 >.< BI(N,1),BI(N,2),BI(N,3) > Onde BI(N,P) e o NUMERO BINOMIAL de numerador N e denominador P. Nesta expressao devemos impor que BI(N,P) = 0 se N < P. Por oportuno, e digno de nota que o termo geral tambem admite uma expressao sintetica e economica, nestes termos : An = < A1,A2-A1,A3-2*A2+A3 >.< BI(N-1,0),BI(N-1,1),BI(N-1,2) > Nao e dificil provar estas expressoes, que ficam como exercicio pra voce. Entretanto, o ganho real aqui nao se resume a elas, que a principio sao apenas uma forma inteligente de expressar uma necessidade computacional, mas, sobretudo, a possibilidade que assim se abre de olharmos estas sequencias como irmanadas em nivel mais profundo ... Com efeito, dado que tanto "An" quanto "Sn" podem ser expressas como um produto, tal como apresentamos acima, entao o que diferencia uma sequencia da outra e propriamente o vetor : C = < A1,A2-A1,A3-2*A2+A3 > Este VETOR CARACTERISTICO e portanto, a priori, o responsavel pelas diferencas que existam. Enatural sentirmos que ele pode nos falar mais ... Seja Ai, A2, A3, ... uma PA2 e para cada N natural formemos o produto ( agora vetorial ) : Vn = < A1,A2-A1,A3-2*A2+A3 > X < BI(N,1),BI(N,2),BI(N,3) > Isto gera uma sequencia de vetores ( aqui em R^3 ). Voce consegue descobrir alguma propriedade da sequencia que esteja relacionada a esta sequencia de vetores ? Um Abraco Paulo Santa Rita 1,1306,130403 >From: Igor GomeZZ >Reply-To: obm-l@mat.puc-rio.br >To: OBM >Subject: [obm-l] Trigonometria e Sequências >Date: Fri, 11 Apr 2003 01:38:05 -0300 > > >Fala galera da lista, boa noite... São dois problemas, um OBM e o outro não >sei a fonte: > >**Sequência: > >1*2 + 2*3 + 3*4 + 4*5 +...+47*48 + 48*49 + 49*50 > >Se não me engano ela eh OBM, certo? Consegui resolvê-la como uma >Progressão Aritmética de segunda ordem. Para achar o polinômio >que gera os termos t(n) eh relativamente demorado, jah para achar o >polinômio que define a soma s(n) eh ainda mais demorado. Tem alguma coisa >na cara que facilite a questão e não estou vendo? _________________________________________________________________ MSN Hotmail, o maior webmail do Brasil. http://www.hotmail.com ========================================================================= Instruções para entrar na lista, sair da lista e usar a lista em http://www.mat.puc-rio.br/~nicolau/olimp/obm-l.html O administrador desta lista é ========================================================================= From owner-obm-l@sucuri.mat.puc-rio.br Sun Apr 13 13:28:41 2003 Return-Path: Received: (from majordom@localhost) by sucuri.mat.puc-rio.br (8.9.3/8.9.3) id NAA06544 for obm-l-MTTP; Sun, 13 Apr 2003 13:23:22 -0300 Received: from ivoti.terra.com.br (ivoti.terra.com.br [200.176.3.20]) by sucuri.mat.puc-rio.br (8.9.3/8.9.3) with ESMTP id NAA06540 for ; Sun, 13 Apr 2003 13:23:18 -0300 Received: from araci.terra.com.br (araci.terra.com.br [200.176.3.44]) by ivoti.terra.com.br (Postfix) with ESMTP id 33AFD408E44 for ; Sun, 13 Apr 2003 13:22:48 -0300 (BRT) Received: from riemann.localdomain (RJ171242.user.veloxzone.com.br [200.149.171.242]) (authenticated user fabio.dias.moreira) by araci.terra.com.br (Postfix) with ESMTP id A4B52C4051 for ; Sun, 13 Apr 2003 13:22:47 -0300 (BRT) Content-Type: text/plain; charset="iso-8859-1" From: =?iso-8859-1?q?F=E1bio=20Dias=20Moreira?= To: obm-l@mat.puc-rio.br Subject: Re: [obm-l] =?iso-8859-1?q?Simplifica=E7=E3o?= Date: Sun, 13 Apr 2003 13:23:13 -0300 User-Agent: KMail/1.4.3 References: <003401c301d3$4c9b9de0$2101a8c0@u2z7z2> In-Reply-To: <003401c301d3$4c9b9de0$2101a8c0@u2z7z2> MIME-Version: 1.0 Content-Transfer-Encoding: 8bit Message-Id: <200304131323.30115.fabio.dias.moreira@terra.com.br> Sender: owner-obm-l@sucuri.mat.puc-rio.br Precedence: bulk Reply-To: obm-l@mat.puc-rio.br -----BEGIN PGP SIGNED MESSAGE----- Hash: SHA1 On Sunday 13 April 2003 12:42, Wagner wrote: > Oi para todos! > > Em que casos é possível simplificar a expressão abaixo ? > > (a+b)^(1/3) + (a-b)^(1/3) > [...] Chame a sua expressão de x. Então x^3 = a+b + a-b + [(a+b)(a-b)]^1/3*x x^3 - (a^2-b^2)^3x - 2a = 0 Logo sua expressão é simplificável (?) se e somente se a equação acima possuir sua raiz real simples o suficiente para você (é claro, agora o problema é resolver a equação...) []s, - -- Fábio "ctg \pi" Dias Moreira -----BEGIN PGP SIGNATURE----- Version: GnuPG v1.0.6 (GNU/Linux) Comment: For info see http://www.gnupg.org iD8DBQE+mY8BalOQFrvzGQoRAshuAJ9z6Ha6EayzKRRHfQOkGd9xOeXCswCfbvEm 5iiX/eCbyy3Vn++GGqqww64= =e2Q1 -----END PGP SIGNATURE----- ========================================================================= Instruções para entrar na lista, sair da lista e usar a lista em http://www.mat.puc-rio.br/~nicolau/olimp/obm-l.html O administrador desta lista é ========================================================================= From owner-obm-l@sucuri.mat.puc-rio.br Sun Apr 13 14:45:21 2003 Return-Path: Received: (from majordom@localhost) by sucuri.mat.puc-rio.br (8.9.3/8.9.3) id OAA08392 for obm-l-MTTP; Sun, 13 Apr 2003 14:40:08 -0300 Received: from fnn.net ([200.175.38.9]) by sucuri.mat.puc-rio.br (8.9.3/8.9.3) with SMTP id OAA08381 for ; Sun, 13 Apr 2003 14:40:03 -0300 Received: (qmail 10185 invoked from network); 13 Apr 2003 17:25:31 -0000 Received: from unknown (HELO windows98) (200.175.39.97) by fnn.net with SMTP; 13 Apr 2003 17:25:31 -0000 Message-ID: <003e01c301ed$1adeecc0$9a75fea9@windows98> From: "Daniel Pini" To: Subject: [obm-l] duvida Date: Sun, 13 Apr 2003 15:46:24 -0300 MIME-Version: 1.0 Content-Type: multipart/alternative; boundary="----=_NextPart_000_0038_01C301D3.D6C3F6C0" X-Priority: 3 X-MSMail-Priority: Normal X-Mailer: Microsoft Outlook Express 5.00.2615.200 X-MimeOLE: Produced By Microsoft MimeOLE V5.00.2615.200 Sender: owner-obm-l@sucuri.mat.puc-rio.br Precedence: bulk Reply-To: obm-l@mat.puc-rio.br This is a multi-part message in MIME format. ------=_NextPart_000_0038_01C301D3.D6C3F6C0 Content-Type: text/plain; charset="iso-8859-1" Content-Transfer-Encoding: quoted-printable Por favor, caros col=E9gas, se algu=E9m aqui poder me ajudar com algum = macete pros exerc=EDcios abaixo, ficarei muito grato: a) [45+29(2)^1/2]^1/3=3Da+b(2)^1/2 o valor de a-b =E9? R;2 b)[5+2(13)^1/2]^1/3 + [5-2(13)^1/2]^1/3 c)[68+48(2)^1/2]^1/4 - [25+22(2)^1/2]1/3 Qual seria um jeito pratico de resolver estas express=F5es? ------=_NextPart_000_0038_01C301D3.D6C3F6C0 Content-Type: text/html; charset="iso-8859-1" Content-Transfer-Encoding: quoted-printable
Por favor, caros col=E9gas, se algu=E9m aqui = poder me ajudar=20 com algum macete pros exerc=EDcios abaixo, ficarei muito = grato:
a) [45+29(2)^1/2]^1/3=3Da+b(2)^1/2 o valor de = a-b =E9?=20 R;2
b)[5+2(13)^1/2]^1/3 + = [5-2(13)^1/2]^1/3
c)[68+48(2)^1/2]^1/4 - = [25+22(2)^1/2]1/3
Qual seria um jeito pratico de resolver estas=20 express=F5es?
------=_NextPart_000_0038_01C301D3.D6C3F6C0-- ========================================================================= Instruções para entrar na lista, sair da lista e usar a lista em http://www.mat.puc-rio.br/~nicolau/olimp/obm-l.html O administrador desta lista é ========================================================================= From owner-obm-l@sucuri.mat.puc-rio.br Sun Apr 13 14:45:21 2003 Return-Path: Received: (from majordom@localhost) by sucuri.mat.puc-rio.br (8.9.3/8.9.3) id OAA08384 for obm-l-MTTP; Sun, 13 Apr 2003 14:40:04 -0300 Received: from fnn.net ([200.175.38.9]) by sucuri.mat.puc-rio.br (8.9.3/8.9.3) with SMTP id OAA08374 for ; Sun, 13 Apr 2003 14:40:00 -0300 Received: (qmail 10499 invoked from network); 13 Apr 2003 17:25:32 -0000 Received: from unknown (HELO windows98) (200.175.39.97) by fnn.net with SMTP; 13 Apr 2003 17:25:32 -0000 Message-ID: <003f01c301ed$1b9f2f80$9a75fea9@windows98> From: "Daniel Pini" To: Subject: [obm-l] =?iso-8859-1?B?bm92YSBmYXRvcmHn428=?= Date: Sun, 13 Apr 2003 15:46:28 -0300 MIME-Version: 1.0 Content-Type: multipart/alternative; boundary="----=_NextPart_000_003B_01C301D3.D8F57CC0" X-Priority: 3 X-MSMail-Priority: Normal X-Mailer: Microsoft Outlook Express 5.00.2615.200 X-MimeOLE: Produced By Microsoft MimeOLE V5.00.2615.200 Sender: owner-obm-l@sucuri.mat.puc-rio.br Precedence: bulk Reply-To: obm-l@mat.puc-rio.br This is a multi-part message in MIME format. ------=_NextPart_000_003B_01C301D3.D8F57CC0 Content-Type: text/plain; charset="iso-8859-1" Content-Transfer-Encoding: quoted-printable A soma de todos os fatores da express=E3o : bc(b+c)+ca(c+a)+ab(a+b)+2abc =E9? Um dos fatores da express=E3o 3a^4-4a=B3b+b^4 =E9? R: a-b A soma de todos os fatores da espress=E3o (a+b+c)=B3-a=B3-b=B3-c=B3 =E9:R: 2(a+b+c)+3 ------=_NextPart_000_003B_01C301D3.D8F57CC0 Content-Type: text/html; charset="iso-8859-1" Content-Transfer-Encoding: quoted-printable
A soma de todos os fatores da = express=E3o=20 :
bc(b+c)+ca(c+a)+ab(a+b)+2abc =E9?
Um dos fatores da express=E3o 3a^4-4a=B3b+b^4 = =E9? R:=20 a-b
A soma de todos os fatores da = espress=E3o
(a+b+c)=B3-a=B3-b=B3-c=B3 =E9:R:=20 2(a+b+c)+3
------=_NextPart_000_003B_01C301D3.D8F57CC0-- ========================================================================= Instruções para entrar na lista, sair da lista e usar a lista em http://www.mat.puc-rio.br/~nicolau/olimp/obm-l.html O administrador desta lista é ========================================================================= From owner-obm-l@sucuri.mat.puc-rio.br Sun Apr 13 17:25:10 2003 Return-Path: Received: (from majordom@localhost) by sucuri.mat.puc-rio.br (8.9.3/8.9.3) id RAA10971 for obm-l-MTTP; Sun, 13 Apr 2003 17:20:03 -0300 Received: from smtp-26.ig.com.br (smtp-26.ig.com.br [200.226.132.160]) by sucuri.mat.puc-rio.br (8.9.3/8.9.3) with SMTP id RAA10961 for ; Sun, 13 Apr 2003 17:20:00 -0300 Received: (qmail 18991 invoked from network); 13 Apr 2003 20:19:39 -0000 Received: from unknown (HELO henrique) (200.163.25.75) by smtp-26.ig.com.br with SMTP; 13 Apr 2003 20:19:39 -0000 Message-ID: <002901c301fa$006e3460$019da8c0@henrique> From: "=?iso-8859-1?Q?Henrique_Patr=EDcio_Sant'Anna_Branco?=" To: "OBM" Subject: [obm-l] =?iso-8859-1?Q?Demonstra=E7=E3o_por_indu=E7=E3o?= Date: Sun, 13 Apr 2003 17:19:35 -0300 MIME-Version: 1.0 Content-Type: text/plain; charset="iso-8859-1" Content-Transfer-Encoding: 8bit X-Priority: 3 X-MSMail-Priority: Normal X-Mailer: Microsoft Outlook Express 6.00.2800.1106 X-MimeOLE: Produced By Microsoft MimeOLE V6.00.2800.1106 Sender: owner-obm-l@sucuri.mat.puc-rio.br Precedence: bulk Reply-To: obm-l@mat.puc-rio.br Pessoal, Como eu demonstro, pelo principio da indução finita, o seguinte: Para todo número natural "n", "n" é primo ou produto de primos. Grato, Henrique. ========================================================================= Instruções para entrar na lista, sair da lista e usar a lista em http://www.mat.puc-rio.br/~nicolau/olimp/obm-l.html O administrador desta lista é ========================================================================= From owner-obm-l@sucuri.mat.puc-rio.br Sun Apr 13 18:31:12 2003 Return-Path: Received: (from majordom@localhost) by sucuri.mat.puc-rio.br (8.9.3/8.9.3) id SAA11973 for obm-l-MTTP; Sun, 13 Apr 2003 18:28:12 -0300 Received: from smtp-27.ig.com.br (smtp-27.ig.com.br [200.226.132.159]) by sucuri.mat.puc-rio.br (8.9.3/8.9.3) with SMTP id SAA11968 for ; Sun, 13 Apr 2003 18:28:09 -0300 Received: (qmail 30778 invoked from network); 13 Apr 2003 21:27:48 -0000 Received: from unknown (HELO xxxx) (200.165.255.183) by smtp-27.ig.com.br with SMTP; 13 Apr 2003 21:27:48 -0000 Message-ID: <003001c30204$19c6f5a0$b7ffa5c8@epq.ime.eb.br> From: "Marcio" To: References: Subject: Re: [obm-l] Fibonacci Date: Sun, 13 Apr 2003 18:31:52 -0300 MIME-Version: 1.0 Content-Type: text/plain; charset="iso-8859-1" Content-Transfer-Encoding: 8bit X-Priority: 3 X-MSMail-Priority: Normal X-Mailer: Microsoft Outlook Express 5.50.4133.2400 X-MimeOLE: Produced By Microsoft MimeOLE V5.50.4133.2400 Sender: owner-obm-l@sucuri.mat.puc-rio.br Precedence: bulk Reply-To: obm-l@mat.puc-rio.br Oi Cláudio.. A idéia que eu mencionei antes funciona bastante bem nesse caso. Afinal, se a,b, a>b são as raízes de x^2 = x+1, sabemos que F_n = (a^n - b^n)/sqrt(5). Como ab=-1: Se n eh par, entao 1/sqrt(5)F_n = 1 / [(-1/b)^n - b^n] = b^n / [1 - b^(2n)] = b^n + b^(3n) + b^(5n) + ... (como |b|<1, podemos interpretar a fração como a soma da pg que começa em b^n e tem razao b^(2n) ). Variando n entre as potências de dois pares e somando temos: n=2: b^2 + b^6 + b^10 + b^14 +... (expoente = 2 (mod4)) n=4: b^4 + b^12 + b^20 +... (expoente = 4 (mod8) n=8: b^8 + b^24 + ... (expoente = 8 (mod16)) ... Note que todo expoente par aparece uma e apenas uma vez nessa soma (de fato, se X = (2^r)i, com i ímpar, então X = 2^r (mod 2^(r+1)), e X != 2^k (mod 2^(k+1)) para outros valores de k - se kr tmb, pq isso implicaria que 2^k | X, contradizendo i ser impar ). Logo, [1/sqrt(5)] * [1/F_2 + 1/F_4 + 1/F_8 + 1/F_16 + ...] = b^2 + b^4 + b^6 + ... = b^2 / (1-b^2) Sendo S a soma procurada: S - 1/F_1 = sqrt(5) * b^2 / (1-b^2). Como b = [1-sqrt(5)]/2 e b^2 = 1+b: S = 1 - sqrt(5)*b = 1-sqrt(5)* ( 1-sqrt(5))/2 = (2 - sqrt(5) + 5)/2 = (7-sqrt(5))/2. Note que os passos em que se trocou ordem de somatório são formalmente justificados pelo fato de soh termos tratados de series absolutamente convergentes de termos positivos (pois b^2 > 0). Abraco, Marcio ----- Original Message ----- From: "Claudio Buffara" To: Sent: Saturday, April 12, 2003 3:41 PM Subject: Re: [obm-l] Fibonacci > Oi, Marcio: > > Mesmo o problema de se achar: > S = SOMA(n>=0) 1/F(2^n) > esta' longe de ser trivial. > > Eu sei que S = 4 - A, onde A = (1 + raiz(5))/2, ou seja, > S = (7 - raiz(5))/2. > > Acho que a formula: F(2k) = [F(k+1) + F(k-1)]*F(k) deve entrar em algum > lugar na demonstracao e, de algum jeito, a restricao as potencias de 2 deve > fazer aparecer alguma PG cuja soma eh S. > > Um abraco, > Claudio. > > ========================================================================= Instruções para entrar na lista, sair da lista e usar a lista em http://www.mat.puc-rio.br/~nicolau/olimp/obm-l.html O administrador desta lista é ========================================================================= From owner-obm-l@sucuri.mat.puc-rio.br Sun Apr 13 19:08:40 2003 Return-Path: Received: (from majordom@localhost) by sucuri.mat.puc-rio.br (8.9.3/8.9.3) id TAA12885 for obm-l-MTTP; Sun, 13 Apr 2003 19:06:05 -0300 Received: from ginsberg.uol.com.br (ginsberg.uol.com.br [200.221.29.48]) by sucuri.mat.puc-rio.br (8.9.3/8.9.3) with ESMTP id TAA12881 for ; Sun, 13 Apr 2003 19:06:02 -0300 Received: from gauss ([200.158.96.79]) by ginsberg.uol.com.br (8.9.1/8.9.1) with SMTP id TAA12336 for ; Sun, 13 Apr 2003 19:05:31 -0300 (BRT) Message-ID: <001201c30209$684cf120$4f609ec8@gauss> From: "Domingos Jr." To: References: <002901c301fa$006e3460$019da8c0@henrique> Subject: [obm-l] =?iso-8859-1?Q?Re:_=5Bobm-l=5D_Demonstra=E7=E3o_por_indu=E7=E3o?= Date: Sun, 13 Apr 2003 19:09:51 -0300 MIME-Version: 1.0 Content-Type: text/plain; charset="iso-8859-1" Content-Transfer-Encoding: 8bit X-Priority: 3 X-MSMail-Priority: Normal X-Mailer: Microsoft Outlook Express 6.00.2800.1106 X-MimeOLE: Produced By Microsoft MimeOLE V6.00.2800.1106 Sender: owner-obm-l@sucuri.mat.puc-rio.br Precedence: bulk Reply-To: obm-l@mat.puc-rio.br > Como eu demonstro, pelo principio da indução finita, o seguinte: > > Para todo número natural "n", "n" é primo ou produto de primos. o caso n = 1 é diferente, então vamos provar para n > 1 seja n = 2, 2 é primo então o caso inicial está demonstrado suponha que para todo 2<= n < k os números sejam primos ou produtos de primos. se n + 1 é primo, nada temos a demonstrar, se não é primo, existe 1 < d < n + 1 tq. d é divisor de n + 1, mas então n + 1 = d.q para algum q e d, q <= n, pela hipótese de indução d e q possuem fatoração em primos ou são primos, de qualquer forma n + 1 tem uma fatoração em primos. segue, por indução para todo n >= 2 [ ]'s ========================================================================= Instruções para entrar na lista, sair da lista e usar a lista em http://www.mat.puc-rio.br/~nicolau/olimp/obm-l.html O administrador desta lista é ========================================================================= From owner-obm-l@sucuri.mat.puc-rio.br Sun Apr 13 19:12:57 2003 Return-Path: Received: (from majordom@localhost) by sucuri.mat.puc-rio.br (8.9.3/8.9.3) id TAA13003 for obm-l-MTTP; Sun, 13 Apr 2003 19:10:55 -0300 Received: from hotmail.com (oe26.law10.hotmail.com [64.4.14.83]) by sucuri.mat.puc-rio.br (8.9.3/8.9.3) with ESMTP id TAA12999 for ; Sun, 13 Apr 2003 19:10:52 -0300 Received: from mail pickup service by hotmail.com with Microsoft SMTPSVC; Sun, 13 Apr 2003 15:10:20 -0700 Received: from 67.27.69.114 by Law10-OE26.adinternal.hotmail.com with DAV; Sun, 13 Apr 2003 22:10:20 +0000 X-Originating-IP: [67.27.69.114] X-Originating-Email: [lrecova@hotmail.com] From: =?iso-8859-1?Q?Leandro_Lacorte_Rec=F4va?= To: Subject: [obm-l] =?iso-8859-1?Q?RE:_=5Bobm-l=5D_RE:_=5Bobm-l=5D_demonstra=E7=E3o?= Date: Sun, 13 Apr 2003 15:10:19 -0700 Message-ID: MIME-Version: 1.0 Content-Type: text/plain; charset="iso-8859-1" X-Priority: 3 (Normal) X-MSMail-Priority: Normal X-Mailer: Microsoft Outlook, Build 10.0.3416 X-MimeOLE: Produced By Microsoft MimeOLE V6.00.2800.1106 Importance: Normal In-Reply-To: X-OriginalArrivalTime: 13 Apr 2003 22:10:20.0739 (UTC) FILETIME=[792BF130:01C30209] Content-Transfer-Encoding: 8bit X-MIME-Autoconverted: from quoted-printable to 8bit by sucuri.mat.puc-rio.br id TAA13000 Sender: owner-obm-l@sucuri.mat.puc-rio.br Precedence: bulk Reply-To: obm-l@mat.puc-rio.br >Demonstre que a solução de f'(x)=f(x) são as funções f(x)=ke^k. Resposta: Essa e uma equacao diferencial linear de 1a ordem. Para isso, escreva a equacao da seguinte forma f'(x)/f(x) = 1 ; Integrando a equacao anterior em ambos os lados em relacao a x temos ln(f(x)) = x + C ; ln(f(x)) = x + C => f(x) = e^(x+C) => f(x) = e^C.e^x = k.e^x onde k = e^C, c real. O valor de k vai depender da condicao inicial de contorno no ponto x=0. Leandro. ========================================================================= Instruções para entrar na lista, sair da lista e usar a lista em http://www.mat.puc-rio.br/~nicolau/olimp/obm-l.html O administrador desta lista é ========================================================================= From owner-obm-l@sucuri.mat.puc-rio.br Sun Apr 13 19:56:51 2003 Return-Path: Received: (from majordom@localhost) by sucuri.mat.puc-rio.br (8.9.3/8.9.3) id TAA14270 for obm-l-MTTP; Sun, 13 Apr 2003 19:53:57 -0300 Received: from web13007.mail.yahoo.com (web13007.mail.yahoo.com [216.136.174.17]) by sucuri.mat.puc-rio.br (8.9.3/8.9.3) with SMTP id TAA14266 for ; Sun, 13 Apr 2003 19:53:53 -0300 Message-ID: <20030413225321.62021.qmail@web13007.mail.yahoo.com> Received: from [200.148.197.31] by web13007.mail.yahoo.com via HTTP; Sun, 13 Apr 2003 19:53:21 ART Date: Sun, 13 Apr 2003 19:53:21 -0300 (ART) From: =?iso-8859-1?q?Helder=20Suzuki?= Subject: [obm-l] Combinatória! To: obm-l@mat.puc-rio.br MIME-Version: 1.0 Content-Type: text/plain; charset=iso-8859-1 Content-Transfer-Encoding: 8bit Sender: owner-obm-l@sucuri.mat.puc-rio.br Precedence: bulk Reply-To: obm-l@mat.puc-rio.br De quantas formas podemos fazer uma sequencia de 0's e 1's de n números tal que nunca temos dois 1's adjacentes? exemplo: se n = 3 000, 001, 010 e 100, 101 são válidos, e 011, 110 e 111 não. 5 possibilidades []'s, Helder Toshiro Suzuki obs: algo ai cheira fibonacci, mas não tenho certeza _______________________________________________________________________ Yahoo! Mail O melhor e-mail gratuito da internet: 6MB de espaço, antivírus, acesso POP3, filtro contra spam. http://br.mail.yahoo.com/ ========================================================================= Instruções para entrar na lista, sair da lista e usar a lista em http://www.mat.puc-rio.br/~nicolau/olimp/obm-l.html O administrador desta lista é ========================================================================= From owner-obm-l@sucuri.mat.puc-rio.br Sun Apr 13 19:58:06 2003 Return-Path: Received: (from majordom@localhost) by sucuri.mat.puc-rio.br (8.9.3/8.9.3) id TAA14318 for obm-l-MTTP; Sun, 13 Apr 2003 19:55:35 -0300 Received: from puma.unisys.com.br (ns2.unisys.com.br [200.220.64.7]) by sucuri.mat.puc-rio.br (8.9.3/8.9.3) with ESMTP id TAA14298 for ; Sun, 13 Apr 2003 19:55:28 -0300 Received: from jf (riohiper01p125.uninet.com.br [200.220.2.125]) by puma.unisys.com.br (8.12.9/8.12.3) with SMTP id h3DMsu53019674 for ; Sun, 13 Apr 2003 19:54:59 -0300 (EST) X-Spam-Filter: check_local@puma.unisys.com.br by digitalanswers.org Message-ID: <00d201c3020f$f374f800$7d02dcc8@jf> From: "Jose Francisco Guimaraes Costa" To: References: <001101c3017f$85c524f0$17f31943@LeandroRecova> Subject: [obm-l] =?iso-8859-1?Q?c=E1lculo-engenharia?= Date: Sun, 13 Apr 2003 19:56:39 -0300 MIME-Version: 1.0 Content-Type: text/plain; charset="iso-8859-1" Content-Transfer-Encoding: 8bit X-Priority: 3 X-MSMail-Priority: Normal X-Mailer: Microsoft Outlook Express 6.00.2800.1106 X-MimeOLE: Produced By Microsoft MimeOLE V6.00.2800.1106 Sender: owner-obm-l@sucuri.mat.puc-rio.br Precedence: bulk Reply-To: obm-l@mat.puc-rio.br A menos que V esteja fazendo engenharia no ITA, onde existem cadeiras de matemática até no segundo período do quinto ano. JF ----- Original Message ----- From: "Leandro Lacorte Recôva" To: Sent: Sunday, April 13, 2003 2:42 AM Subject: [obm-l] RE: [obm-l] Re:[obm-l] Re: [obm-l] cálculo-engenharia > Caro colega, > > Eu ja usei o livro do Shokowski quando lecionei calculo 1 para uma turma > de engenharia eletrica em Brasilia. [...] > Depois de um curso de calculo, voce nao voltara mais a ver > matematica num curso de engenharia (o que e lamentalvemelte uma pena > para aqueles que gostam da matematica) [...] > Leandro ========================================================================= Instruções para entrar na lista, sair da lista e usar a lista em http://www.mat.puc-rio.br/~nicolau/olimp/obm-l.html O administrador desta lista é ========================================================================= From owner-obm-l@sucuri.mat.puc-rio.br Sun Apr 13 20:02:57 2003 Return-Path: Received: (from majordom@localhost) by sucuri.mat.puc-rio.br (8.9.3/8.9.3) id UAA14448 for obm-l-MTTP; Sun, 13 Apr 2003 20:00:24 -0300 Received: from puma.unisys.com.br (smtp.unisys.com.br [200.220.64.7]) by sucuri.mat.puc-rio.br (8.9.3/8.9.3) with ESMTP id UAA14434 for ; Sun, 13 Apr 2003 20:00:16 -0300 Received: from jf (riohiper01p125.uninet.com.br [200.220.2.125]) by puma.unisys.com.br (8.12.9/8.12.3) with SMTP id h3DMxl53020313 for ; Sun, 13 Apr 2003 19:59:48 -0300 (EST) X-Spam-Filter: check_local@puma.unisys.com.br by digitalanswers.org Message-ID: <011b01c30210$9f90b8e0$7d02dcc8@jf> From: "Jose Francisco Guimaraes Costa" To: References: <002901c301fa$006e3460$019da8c0@henrique> Subject: [obm-l] =?iso-8859-1?Q?Demonstra=E7=E3o_por_indu=E7=E3o?= Date: Sun, 13 Apr 2003 20:01:30 -0300 MIME-Version: 1.0 Content-Type: text/plain; charset="iso-8859-1" Content-Transfer-Encoding: 8bit X-Priority: 3 X-MSMail-Priority: Normal X-Mailer: Microsoft Outlook Express 6.00.2800.1106 X-MimeOLE: Produced By Microsoft MimeOLE V6.00.2800.1106 Sender: owner-obm-l@sucuri.mat.puc-rio.br Precedence: bulk Reply-To: obm-l@mat.puc-rio.br Isto não é a definição de número composto, e portanto não demonstrável? JF ----- Original Message ----- From: "Henrique Patrício Sant'Anna Branco" To: "OBM" Sent: Sunday, April 13, 2003 5:19 PM Subject: [obm-l] Demonstração por indução > Pessoal, > > Como eu demonstro, pelo principio da indução finita, o seguinte: > > Para todo número natural "n", "n" é primo ou produto de primos. > > Grato, > Henrique. > > ========================================================================= > Instruções para entrar na lista, sair da lista e usar a lista em > http://www.mat.puc-rio.br/~nicolau/olimp/obm-l.html > O administrador desta lista é > ========================================================================= > ========================================================================= Instruções para entrar na lista, sair da lista e usar a lista em http://www.mat.puc-rio.br/~nicolau/olimp/obm-l.html O administrador desta lista é ========================================================================= From owner-obm-l@sucuri.mat.puc-rio.br Sun Apr 13 20:18:52 2003 Return-Path: Received: (from majordom@localhost) by sucuri.mat.puc-rio.br (8.9.3/8.9.3) id UAA15233 for obm-l-MTTP; Sun, 13 Apr 2003 20:16:05 -0300 Received: from trex-b.centroin.com.br (trex-b.centroin.com.br [200.225.63.136]) by sucuri.mat.puc-rio.br (8.9.3/8.9.3) with ESMTP id UAA15229 for ; Sun, 13 Apr 2003 20:16:01 -0300 Received: from centroin.com.br (RJ118023.user.veloxzone.com.br [200.141.118.23] (may be forged)) (authenticated bits=0) by trex-b.centroin.com.br (8.12.9/8.12.9) with ESMTP id h3DNFWiL001825 for ; Sun, 13 Apr 2003 20:15:33 -0300 (EST) Message-ID: <3E99EFC6.10505@centroin.com.br> Date: Sun, 13 Apr 2003 20:16:22 -0300 From: "A. C. Morgado" User-Agent: Mozilla/5.0 (Windows; U; Windows NT 5.0; en-US; rv:1.0.2) Gecko/20030208 Netscape/7.02 X-Accept-Language: en-us, en MIME-Version: 1.0 To: obm-l@mat.puc-rio.br Subject: Re: [obm-l] =?ISO-8859-1?Q?Combinat=F3ria!?= References: <20030413225321.62021.qmail@web13007.mail.yahoo.com> Content-Type: text/plain; charset=ISO-8859-1; format=flowed Content-Transfer-Encoding: 8bit Sender: owner-obm-l@sucuri.mat.puc-rio.br Precedence: bulk Reply-To: obm-l@mat.puc-rio.br Ta com olfato apurado! Seja A(n) o numero de sequencias de n termos com a citada propriedade. A(n) eh a soma do numero de seq. com a prop. que começam por 0 (que eh igual a A(n-1) ) com o numero de seq. com a prop. que começam por 1 (estas tem necessariamente o segundo termo igual a 0 e o restante da seq. pode ser formado de A(n-2) modos). Logo, A(n) = A(n-1) + A(n-2), ou seja A(n) eh uma seq do tipo Fibonacci. A(1) = 2 e A(2) = 3 Eh, dependendo de sua definiçao de Fibonacci uma Fibonacci deslocada. Aproveito para perguntar a todos a sua definiçao de Fibonacci. Uns começam a enumeraçao dos termos em 0, outros em 1. Helder Suzuki wrote: >De quantas formas podemos fazer uma sequencia de 0's e >1's de n números tal que nunca temos dois 1's >adjacentes? > >exemplo: se n = 3 >000, 001, 010 e 100, 101 são válidos, >e 011, 110 e 111 não. >5 possibilidades > >[]'s, >Helder Toshiro Suzuki > >obs: algo ai cheira fibonacci, mas não tenho certeza > >_______________________________________________________________________ >Yahoo! Mail >O melhor e-mail gratuito da internet: 6MB de espaço, antivírus, acesso POP3, filtro contra spam. >http://br.mail.yahoo.com/ >========================================================================= >Instruções para entrar na lista, sair da lista e usar a lista em >http://www.mat.puc-rio.br/~nicolau/olimp/obm-l.html >O administrador desta lista é >========================================================================= > > > > ========================================================================= Instruções para entrar na lista, sair da lista e usar a lista em http://www.mat.puc-rio.br/~nicolau/olimp/obm-l.html O administrador desta lista é ========================================================================= From owner-obm-l@sucuri.mat.puc-rio.br Sun Apr 13 20:23:25 2003 Return-Path: Received: (from majordom@localhost) by sucuri.mat.puc-rio.br (8.9.3/8.9.3) id UAA15461 for obm-l-MTTP; Sun, 13 Apr 2003 20:20:54 -0300 Received: from web13004.mail.yahoo.com (web13004.mail.yahoo.com [216.136.174.14]) by sucuri.mat.puc-rio.br (8.9.3/8.9.3) with SMTP id UAA15456 for ; Sun, 13 Apr 2003 20:20:49 -0300 Message-ID: <20030413232018.41631.qmail@web13004.mail.yahoo.com> Received: from [200.148.197.31] by web13004.mail.yahoo.com via HTTP; Sun, 13 Apr 2003 20:20:18 ART Date: Sun, 13 Apr 2003 20:20:18 -0300 (ART) From: =?iso-8859-1?q?Helder=20Suzuki?= Subject: Re: [obm-l] nova fatoração To: obm-l@mat.puc-rio.br In-Reply-To: <003f01c301ed$1b9f2f80$9a75fea9@windows98> MIME-Version: 1.0 Content-Type: text/plain; charset=iso-8859-1 Content-Transfer-Encoding: 8bit Sender: owner-obm-l@sucuri.mat.puc-rio.br Precedence: bulk Reply-To: obm-l@mat.puc-rio.br --- Daniel Pini escreveu: > A soma de todos os fatores da expressão : > bc(b+c)+ca(c+a)+ab(a+b)+2abc é? > Um dos fatores da expressão 3a^4-4a³b+b^4 é? R: a-b > A soma de todos os fatores da espressão > (a+b+c)³-a³-b³-c³ é:R: 2(a+b+c)+3 > bc(b+c)+ca(c+a)+ab(a+b)+2abc = = bbc + bcc + acc + aac + aab + abb + 2abc = = b(cc + aa + 2ac) + bb(c + a) + ac(a + c)= = b(a + c)(a + b) + bb(a + c) + ac(a + c) = = (a + c)(bb + b(a+c) + ac) = = (a + c)(a + b)(b + c) soma dos fatores = a+c+a+b+c+b = 2(a+b+c)= 2a + 2b + 2c 3a^4 - 4a^3b + b^4 = = 3a^3(a - b) - a^3b + b^4 = = 3a^3(a - b) - b(a^3 - b^3) = = 3a^3(a - b) - b(a - b)(a^2 + ab + b^2)= = (a - b)(3a^3 - a^2b - ab^2 - b^3) um dos fatores: (a-b) (tentem fatorar o outro termo :P) (a+b+c)^3 - a^3 - b^3 - c^3 = = a^3+ 3a(b+c)(a+b+c) + (b+c)^3 - a^3 - b^3 - c^3 = = 3a(b+c)(a+b+c) + 3bc(b+c) = = 3(b+c)(a^2+ab+ac) + 3(b+c)bc= = 3(b+c)(a^2+ab+ac+bc)= = 3(b+c)(a(a+b)+c(a+b)= = 3(b+c)(a+b)(a+c) soma dos fatores: 3+a+b+b+c+a+c = 2(a+b+c) + 3 []'s, Helder Toshiro Suzuki _______________________________________________________________________ Yahoo! Mail O melhor e-mail gratuito da internet: 6MB de espaço, antivírus, acesso POP3, filtro contra spam. http://br.mail.yahoo.com/ ========================================================================= Instruções para entrar na lista, sair da lista e usar a lista em http://www.mat.puc-rio.br/~nicolau/olimp/obm-l.html O administrador desta lista é ========================================================================= From owner-obm-l@sucuri.mat.puc-rio.br Sun Apr 13 21:07:52 2003 Return-Path: Received: (from majordom@localhost) by sucuri.mat.puc-rio.br (8.9.3/8.9.3) id VAA17502 for obm-l-MTTP; Sun, 13 Apr 2003 21:04:51 -0300 Received: from web13004.mail.yahoo.com (web13004.mail.yahoo.com [216.136.174.14]) by sucuri.mat.puc-rio.br (8.9.3/8.9.3) with SMTP id VAA17498 for ; Sun, 13 Apr 2003 21:04:48 -0300 Message-ID: <20030414000417.47542.qmail@web13004.mail.yahoo.com> Received: from [200.148.197.31] by web13004.mail.yahoo.com via HTTP; Sun, 13 Apr 2003 21:04:17 ART Date: Sun, 13 Apr 2003 21:04:17 -0300 (ART) From: =?iso-8859-1?q?Helder=20Suzuki?= Subject: Re: [obm-l] Combinatória! To: obm-l@mat.puc-rio.br In-Reply-To: <3E99EFC6.10505@centroin.com.br> MIME-Version: 1.0 Content-Type: text/plain; charset=iso-8859-1 Content-Transfer-Encoding: 8bit Sender: owner-obm-l@sucuri.mat.puc-rio.br Precedence: bulk Reply-To: obm-l@mat.puc-rio.br Obrigado Morgado! pra min, Fibonacci começa com F(1) = 1 e F(2) = 1 assim: 1, 1, 2, 3, 5, 8, ... []'s, Helder Toshiro Suzuki --- "A. C. Morgado" escreveu: > Ta com olfato apurado! > Seja A(n) o numero de sequencias de n termos com a > citada propriedade. > A(n) eh a soma do numero de seq. com a prop. que > começam por 0 (que eh > igual a A(n-1) ) com o numero de seq. com a prop. > que começam por 1 > (estas tem necessariamente o segundo termo igual a 0 > e o restante da > seq. pode ser formado de A(n-2) modos). > Logo, A(n) = A(n-1) + A(n-2), ou seja A(n) eh uma > seq do tipo Fibonacci. > A(1) = 2 e A(2) = 3 > Eh, dependendo de sua definiçao de Fibonacci uma > Fibonacci deslocada. > Aproveito para perguntar a todos a sua definiçao de > Fibonacci. Uns > começam a enumeraçao dos termos em 0, outros em 1. > Helder Suzuki wrote: > > >De quantas formas podemos fazer uma sequencia de > 0's e > >1's de n números tal que nunca temos dois 1's > >adjacentes? > > > >exemplo: se n = 3 > >000, 001, 010 e 100, 101 são válidos, > >e 011, 110 e 111 não. > >5 possibilidades > > > >[]'s, > >Helder Toshiro Suzuki > > > >obs: algo ai cheira fibonacci, mas não tenho > certeza > > _______________________________________________________________________ Yahoo! Mail O melhor e-mail gratuito da internet: 6MB de espaço, antivírus, acesso POP3, filtro contra spam. http://br.mail.yahoo.com/ ========================================================================= Instruções para entrar na lista, sair da lista e usar a lista em http://www.mat.puc-rio.br/~nicolau/olimp/obm-l.html O administrador desta lista é ========================================================================= From owner-obm-l@sucuri.mat.puc-rio.br Sun Apr 13 21:09:24 2003 Return-Path: Received: (from majordom@localhost) by sucuri.mat.puc-rio.br (8.9.3/8.9.3) id VAA17588 for obm-l-MTTP; Sun, 13 Apr 2003 21:06:44 -0300 Received: from traven10.uol.com.br (traven10.uol.com.br [200.221.29.45]) by sucuri.mat.puc-rio.br (8.9.3/8.9.3) with ESMTP id VAA17584 for ; Sun, 13 Apr 2003 21:06:41 -0300 Received: from Giulio.Teia ([200.168.177.9]) by traven10.uol.com.br (8.9.1/8.9.1) with SMTP id VAA19598 for ; Sun, 13 Apr 2003 21:06:11 -0300 (BRT) Date: Sun, 13 Apr 2003 20:57:55 -0300 From: Fabiano To: obm-l@mat.puc-rio.br Subject: Re: [obm-l] =?ISO-8859-1?Q?Combinat=F3ria!?= Message-Id: <20030413205755.4004affb.fabiano.ufo@uol.com.br> In-Reply-To: <3E99EFC6.10505@centroin.com.br> References: <20030413225321.62021.qmail@web13007.mail.yahoo.com> <3E99EFC6.10505@centroin.com.br> X-Mailer: Sylpheed version 0.8.10 (GTK+ 1.2.10; i686-pc-linux-gnu) Mime-Version: 1.0 Content-Type: text/plain; charset=ISO-8859-1 Content-Transfer-Encoding: 8bit X-MIME-Autoconverted: from quoted-printable to 8bit by sucuri.mat.puc-rio.br id VAA17585 Sender: owner-obm-l@sucuri.mat.puc-rio.br Precedence: bulk Reply-To: obm-l@mat.puc-rio.br On Sun, 13 Apr 2003 20:16:22 -0300 "A. C. Morgado" wrote: > Eh, dependendo de sua definiçao de Fibonacci uma Fibonacci deslocada. > Aproveito para perguntar a todos a sua definiçao de Fibonacci. Uns > começam a enumeraçao dos termos em 0, outros em 1. como assim uma Fibonnacci Deslocada? []s Fabiano ========================================================================= Instruções para entrar na lista, sair da lista e usar a lista em http://www.mat.puc-rio.br/~nicolau/olimp/obm-l.html O administrador desta lista é ========================================================================= From owner-obm-l@sucuri.mat.puc-rio.br Sun Apr 13 21:30:33 2003 Return-Path: Received: (from majordom@localhost) by sucuri.mat.puc-rio.br (8.9.3/8.9.3) id VAA18400 for obm-l-MTTP; Sun, 13 Apr 2003 21:27:44 -0300 Received: from artemis.opendf.com.br (artemis.opengate.com.br [200.181.71.14]) by sucuri.mat.puc-rio.br (8.9.3/8.9.3) with ESMTP id VAA18395 for ; Sun, 13 Apr 2003 21:27:40 -0300 Received: from localhost (localhost [127.0.0.1]) by artemis.opendf.com.br (Postfix) with ESMTP id F33372BEC4 for ; Sun, 13 Apr 2003 21:28:27 -0300 (BRT) Received: from artemis.opendf.com.br ([127.0.0.1]) by localhost (artemis.opengate.com.br [127.0.0.1:10024]) (amavisd-new) with ESMTP id 03460-10 for ; Sun, 13 Apr 2003 21:28:27 -0300 (BRT) Received: from computer (200-181-090-143.bsace7001.dsl.brasiltelecom.net.br [200.181.90.143]) by artemis.opendf.com.br (Postfix) with ESMTP id 6904E2BEC2 for ; Sun, 13 Apr 2003 21:28:26 -0300 (BRT) From: "Artur Costa Steiner" To: Subject: [obm-l] =?iso-8859-1?Q?RE:_=5Bobm-l=5D_RE:_=5Bobm-l=5D_RE:_=5Bobm-l=5D_demonstr?= =?iso-8859-1?Q?a=E7=E3o?= Date: Sun, 13 Apr 2003 21:27:03 -0300 Organization: Steiner Consultoria LTDA Message-ID: <006701c3021c$94a95110$9865fea9@computer> MIME-Version: 1.0 Content-Type: text/plain; charset="iso-8859-1" X-Priority: 3 (Normal) X-MSMail-Priority: Normal X-Mailer: Microsoft Outlook, Build 10.0.2627 Importance: Normal X-MimeOLE: Produced By Microsoft MimeOLE V6.00.2800.1106 In-Reply-To: X-Virus-Scanned: by amavisd-new Content-Transfer-Encoding: 8bit X-MIME-Autoconverted: from quoted-printable to 8bit by sucuri.mat.puc-rio.br id VAA18396 Sender: owner-obm-l@sucuri.mat.puc-rio.br Precedence: bulk Reply-To: obm-l@mat.puc-rio.br Tenicamente falando, eu acho que isto nao eh uma prova. Ao resolver a equacao diferencial, vc integrou f'(x)/f(x) e encontrou ln(f(x)) + c. Eh inquestionavel, mas esta conclusao ja parte do principio de que a derivada de ke^x eh ela propria. A teoria de equacoes diferenciais ja considera como demonstrado que a derivada da funcao exponencial do tipo ke^x e ele propria. Um abraco Artur >>Demonstre que a solução de f'(x)=f(x) são as funções f(x)=ke^k. > >Resposta: Essa e uma equacao diferencial linear de 1a ordem. Para isso, >escreva a equacao da seguinte forma > >f'(x)/f(x) = 1 ; > >Integrando a equacao anterior em ambos os lados em relacao a x temos > >ln(f(x)) = x + C ; >ln(f(x)) = x + C => f(x) = e^(x+C) => f(x) = e^C.e^x = k.e^x onde k = >e^C, c real. > >O valor de k vai depender da condicao inicial de contorno no ponto x=0. > >Leandro. > >======================================================================= == >Instruções para entrar na lista, sair da lista e usar a lista em >http://www.mat.puc-rio.br/~nicolau/olimp/obm-l.html >O administrador desta lista é >======================================================================= == ========================================================================= Instruções para entrar na lista, sair da lista e usar a lista em http://www.mat.puc-rio.br/~nicolau/olimp/obm-l.html O administrador desta lista é ========================================================================= From owner-obm-l@sucuri.mat.puc-rio.br Sun Apr 13 22:48:19 2003 Return-Path: Received: (from majordom@localhost) by sucuri.mat.puc-rio.br (8.9.3/8.9.3) id WAA20473 for obm-l-MTTP; Sun, 13 Apr 2003 22:43:50 -0300 Received: from trex-b.centroin.com.br (trex-b.centroin.com.br [200.225.63.136]) by sucuri.mat.puc-rio.br (8.9.3/8.9.3) with ESMTP id WAA20469 for ; Sun, 13 Apr 2003 22:43:47 -0300 Received: from centroin.com.br (RJ118023.user.veloxzone.com.br [200.141.118.23] (may be forged)) (authenticated bits=0) by trex-b.centroin.com.br (8.12.9/8.12.9) with ESMTP id h3E1hIiL006813 for ; Sun, 13 Apr 2003 22:43:20 -0300 (EST) Message-ID: <3E9A1266.60309@centroin.com.br> Date: Sun, 13 Apr 2003 22:44:06 -0300 From: "A. C. Morgado" User-Agent: Mozilla/5.0 (Windows; U; Windows NT 5.0; en-US; rv:1.0.2) Gecko/20030208 Netscape/7.02 X-Accept-Language: en-us, en MIME-Version: 1.0 To: obm-l@mat.puc-rio.br Subject: Re: [obm-l] =?ISO-8859-1?Q?Combinat=F3ria!?= References: <20030413225321.62021.qmail@web13007.mail.yahoo.com> <3E99EFC6.10505@centroin.com.br> <20030413205755.4004affb.fabiano.ufo@uol.com.br> Content-Type: multipart/alternative; boundary="------------070002070102030805060807" Sender: owner-obm-l@sucuri.mat.puc-rio.br Precedence: bulk Reply-To: obm-l@mat.puc-rio.br --------------070002070102030805060807 Content-Type: text/plain; charset=ISO-8859-1; format=flowed Content-Transfer-Encoding: 8bit Adotando a definiçao do Suzuki, F(1) = 1, F(2) =1 na seq. de Fibonacci (alguns definem F(0) = 1. F(1) = 1, F(2) = 2, ...; o pior eh que eu ateh hoje ainda nao me convenci onde estah a maioria!), a resposta do problema eh F(n+2), ou seja, F(n) deslocada de 2 termos. Fabiano wrote: >On Sun, 13 Apr 2003 20:16:22 -0300 >"A. C. Morgado" wrote: > > >>Eh, dependendo de sua definiçao de Fibonacci uma Fibonacci deslocada. >>Aproveito para perguntar a todos a sua definiçao de Fibonacci. Uns >>começam a enumeraçao dos termos em 0, outros em 1. >> >> > >como assim uma Fibonnacci Deslocada? > >[]s >Fabiano >========================================================================= >Instruções para entrar na lista, sair da lista e usar a lista em >http://www.mat.puc-rio.br/~nicolau/olimp/obm-l.html >O administrador desta lista é >========================================================================= > > > > --------------070002070102030805060807 Content-Type: text/html; charset=us-ascii Content-Transfer-Encoding: 7bit Adotando a definiçao do Suzuki, F(1) = 1, F(2) =1 na seq. de Fibonacci (alguns definem F(0) = 1. F(1) = 1, F(2) = 2, ...; o pior eh que eu ateh hoje ainda nao me convenci onde estah a maioria!), a resposta do problema eh  F(n+2), ou seja, F(n) deslocada de 2 termos.

Fabiano wrote:
On Sun, 13 Apr 2003 20:16:22 -0300
"A. C. Morgado" <morgado@centroin.com.br> wrote:
  
Eh, dependendo de sua definiçao de Fibonacci uma Fibonacci deslocada.
Aproveito para perguntar a todos a sua definiçao de Fibonacci. Uns 
começam a enumeraçao dos termos em 0, outros em 1.
    

como assim uma Fibonnacci Deslocada?

[]s
Fabiano
=========================================================================
Instruções para entrar na lista, sair da lista e usar a lista em
http://www.mat.puc-rio.br/~nicolau/olimp/obm-l.html
O administrador desta lista é <nicolau@mat.puc-rio.br>
=========================================================================


  

--------------070002070102030805060807-- ========================================================================= Instruções para entrar na lista, sair da lista e usar a lista em http://www.mat.puc-rio.br/~nicolau/olimp/obm-l.html O administrador desta lista é ========================================================================= From owner-obm-l@sucuri.mat.puc-rio.br Mon Apr 14 02:46:31 2003 Return-Path: Received: (from majordom@localhost) by sucuri.mat.puc-rio.br (8.9.3/8.9.3) id CAA22794 for obm-l-MTTP; Mon, 14 Apr 2003 02:42:08 -0300 Received: from web14308.mail.yahoo.com (web14308.mail.yahoo.com [216.136.173.156]) by sucuri.mat.puc-rio.br (8.9.3/8.9.3) with SMTP id CAA22789 for ; Mon, 14 Apr 2003 02:42:04 -0300 Message-ID: <20030414054133.12805.qmail@web14308.mail.yahoo.com> Received: from [200.190.89.28] by web14308.mail.yahoo.com via HTTP; Mon, 14 Apr 2003 02:41:33 ART Date: Mon, 14 Apr 2003 02:41:33 -0300 (ART) From: =?iso-8859-1?q?Rafael?= Subject: Re: [obm-l] duvida To: obm-l@mat.puc-rio.br In-Reply-To: <003e01c301ed$1adeecc0$9a75fea9@windows98> MIME-Version: 1.0 Content-Type: text/plain; charset=iso-8859-1 Content-Transfer-Encoding: 8bit Sender: owner-obm-l@sucuri.mat.puc-rio.br Precedence: bulk Reply-To: obm-l@mat.puc-rio.br Olá Daniel! Tenho uma questão aqui resolvida que talvez te ajude. É bem semelhante: > O valor de n que satisfaz a igualdade > [17(5)^1/2 +38]^1/n + [17(5)^1/2 - 38]^1/n = 20^1/2 Primeiro, vendo que os dois radicandos eram parecidos fui ver o que acontecia ao multiplicá-los: = [17.raiz(5) + 38].[17.raiz(5) - 38] = 1 Então se fizermos: a = 17.raiz(5) + 38 b = 17.raiz(5) - 38 Podemos dizer que: ab = 1 a = 1/b E nosso problema vira: [17.raiz(5) + 38]^(1/n) + [17.raiz(5) - 38]^(1/n) = raiz(20) a^(1/n) + b^(1/n) = raiz(20) (1/b)^(1/n) + b^(1/n) = raiz(20) 1/[b^(1/n)] + b^(1/n) = raiz(20) Tirando o mínimo e arrumando tudo teremos: (b²)^(1/n) - raiz(20).b^(1/n) + 1 = 0 E chamando b^(1/n) de x, temos: x² - raiz(20).x + 1 = 0 Resolvendo a equação: x = raiz(5) +- 2 Como x era b^(1/n): b^(1/n) = raiz(5) +- 2 E colocando o valor de b: [17.raiz(5) - 38]^(1/n) = raiz(5) +- 2 Aqui você até pode usar logaritmo para achar o valor aproximado de n. Mas você pode ir elevando o segundo membro ao quadrado, ao cubo, à quarta, etc, para ver se o radicando do primeiro membro é uma potência do que tem no segundo membro e aí para igualar era só tirar a raiz com o índice da potência que encontrou. Foi o que eu fiz: [raiz(5) +- 2]² = 9 +- 4.raiz(5) [raiz(5) +- 2]³ = 17.raiz(5) +- 38 E quando cheguei no cubo já deu certo! Então temos: [17.raiz(5) - 38]^(1/n) = raiz(5) +- 2 [[raiz(5) +- 2]³]^(1/n) = raiz(5) +- 2 n = 3 Abraços, Rafael. --- Daniel Pini escreveu: > Por favor, caros colégas, se alguém aqui poder me > ajudar com algum macete pros exercícios abaixo, > ficarei muito grato: > a) [45+29(2)^1/2]^1/3=a+b(2)^1/2 o valor de a-b é? > R;2 > b)[5+2(13)^1/2]^1/3 + [5-2(13)^1/2]^1/3 > c)[68+48(2)^1/2]^1/4 - [25+22(2)^1/2]1/3 > Qual seria um jeito pratico de resolver estas > expressões? > _______________________________________________________________________ Yahoo! Mail O melhor e-mail gratuito da internet: 6MB de espaço, antivírus, acesso POP3, filtro contra spam. http://br.mail.yahoo.com/ ========================================================================= Instruções para entrar na lista, sair da lista e usar a lista em http://www.mat.puc-rio.br/~nicolau/olimp/obm-l.html O administrador desta lista é ========================================================================= From owner-obm-l@sucuri.mat.puc-rio.br Mon Apr 14 03:04:38 2003 Return-Path: Received: (from majordom@localhost) by sucuri.mat.puc-rio.br (8.9.3/8.9.3) id CAA23001 for obm-l-MTTP; Mon, 14 Apr 2003 02:59:25 -0300 Received: from Euler.impa.br (euler.impa.br [147.65.1.3]) by sucuri.mat.puc-rio.br (8.9.3/8.9.3) with ESMTP id CAA22997 for ; Mon, 14 Apr 2003 02:59:22 -0300 Received: from Gauss.impa.br (Gauss [147.65.4.1]) by Euler.impa.br (8.11.6p2/8.11.6) with ESMTP id h3E5wq009865 for ; Mon, 14 Apr 2003 02:58:52 -0300 (EST) From: Carlos Gustavo Tamm de Araujo Moreira Received: by Gauss.impa.br (8.11.6p2) id h3E5wkp00839; Mon, 14 Apr 2003 02:58:46 -0300 (EST) Message-Id: <200304140558.h3E5wkp00839@Gauss.impa.br> Subject: Re: [obm-l] =?ISO-8859-1?Q?Combinat=F3ria!?= To: obm-l@mat.puc-rio.br Date: Mon, 14 Apr 2003 02:58:45 -0300 (EST) In-Reply-To: <3E9A1266.60309@centroin.com.br> from "A. C. Morgado" at Apr 13, 3 10:44:06 pm X-Mailer: ELM [version 2.4 PL25] MIME-Version: 1.0 Content-Type: text/plain; charset=US-ASCII Content-Transfer-Encoding: 7bit Sender: owner-obm-l@sucuri.mat.puc-rio.br Precedence: bulk Reply-To: obm-l@mat.puc-rio.br Eu acho que e' mais comum a definicao do Suzuki. Eu tenho uma clara preferencia por ela pois ela faz com que valha a bela propriedade n|m => F(n)|F(n). Abracos, Gugu > > Adotando a definiçao do Suzuki, F(1) = 1, F(2) =1 na seq. de Fibonacci >(alguns definem F(0) = 1. F(1) = 1, F(2) = 2, ...; o pior eh que eu ateh >hoje ainda nao me convenci onde estah a maioria!), a resposta do >problema eh F(n+2), ou seja, F(n) deslocada de 2 termos. > >Fabiano wrote: > >>On Sun, 13 Apr 2003 20:16:22 -0300 >>"A. C. Morgado" wrote: >> >> >>>Eh, dependendo de sua definiçao de Fibonacci uma Fibonacci deslocada. >>>Aproveito para perguntar a todos a sua definiçao de Fibonacci. Uns >>>começam a enumeraçao dos termos em 0, outros em 1. >>> >>> >> >>como assim uma Fibonnacci Deslocada? >> >>[]s >>Fabiano ========================================================================= Instruções para entrar na lista, sair da lista e usar a lista em http://www.mat.puc-rio.br/~nicolau/olimp/obm-l.html O administrador desta lista é ========================================================================= From owner-obm-l@sucuri.mat.puc-rio.br Mon Apr 14 03:11:52 2003 Return-Path: Received: (from majordom@localhost) by sucuri.mat.puc-rio.br (8.9.3/8.9.3) id DAA23155 for obm-l-MTTP; Mon, 14 Apr 2003 03:07:06 -0300 Received: from Euler.impa.br (euler.impa.br [147.65.1.3]) by sucuri.mat.puc-rio.br (8.9.3/8.9.3) with ESMTP id DAA23151 for ; Mon, 14 Apr 2003 03:07:03 -0300 Received: from Gauss.impa.br (Gauss [147.65.4.1]) by Euler.impa.br (8.11.6p2/8.11.6) with ESMTP id h3E66X010122 for ; Mon, 14 Apr 2003 03:06:33 -0300 (EST) From: Carlos Gustavo Tamm de Araujo Moreira Received: by Gauss.impa.br (8.11.6p2) id h3E66SA01501; Mon, 14 Apr 2003 03:06:28 -0300 (EST) Message-Id: <200304140606.h3E66SA01501@Gauss.impa.br> Subject: Re: [obm-l] Problemas em Aberto To: obm-l@mat.puc-rio.br Date: Mon, 14 Apr 2003 03:06:27 -0300 (EST) In-Reply-To: from "Claudio Buffara" at Apr 12, 3 11:36:00 am X-Mailer: ELM [version 2.4 PL25] MIME-Version: 1.0 Content-Type: text/plain; charset=US-ASCII Content-Transfer-Encoding: 7bit Sender: owner-obm-l@sucuri.mat.puc-rio.br Precedence: bulk Reply-To: obm-l@mat.puc-rio.br Caro Claudio, Eu tinha proposto esse problema da Eureka para uma IMO antiga, mas nao entrou... Tente primeiro colocar o triangulo com coordenadas racionais no plano com um dos lados no eixo x e depois tente aplicar uma rotacao conveniente (multiplicar por um complexo de modulo 1 com coordenadas racionais conveniente). Um pouco de aritmetica em Z[i] ajuda... Sobre o problema 9, eu nao vejo como as solucoes se generalizam para R racional... Os numeradores podem ficar grandes... Abracos, Gugu > >Oi, Gugu: > >O no. 7 foi baseado num problema da Eureka, mas eu acabei de ver onde errei. >O problema original eh: > >"Um triangulo tem os lados de medidas inteiras e area racional. Prove que >ele eh congruente a um triangulo cujos vertices tem coordenadas inteiras". > >Eu assumi, erroneamente, que um dos lados do triangulo de reticulado (essa >eh a traducao correta de "lattice triangle"?) poderia ser paralelo a um dos >eixos coordenados, de forma que o pe' da altura relativa a este lado teria >coordenadas inteiras. O seu contra-exemplo mostra que esta hipotese nao eh >valida em geral. > >Um triangulo de reticulado congruente ao seu teria como vertices: >(0,0), (15,8), (48,64) ==> nenhum lado eh paralelo aos eixos. > >De qualquer forma, fica ai o enunciado do problema original. Ao que me >consta, a Eureka ainda nao recebeu uma solucao para este. > >***** > >A solucao que eu tinha imaginado pro no. 9 eh: > >O numero S, quando expresso na base K eh igual a: >0,100100001000000100000000100..., ou seja, uma K-esimal infinita e nao >periodica. Logo, S eh irracional. > >De qualquer forma, ambas as solucoes so facilmente generalizaveis para o >caso de: >SOMA(n>=1) R^(n^2), onde R eh um racional entre 0 e 1. > > >Um abraco, >Claudio. > > >on 12.04.03 02:50, Carlos Gustavo Tamm de Araujo Moreira at gugu@impa.br >wrote: > >>> >>> 6. D=EA um exemplo de uma sequ=EAncia (Xn) de n=FAmeros reais tal que:=20 >>> >>> lim ( Xn / n^t ) =3D 0 para todo t > 0=20 >>> e >>> lim ( [log(n)]^k / Xn ) =3D 0 para todo k > 0 >>> >> >> Existem muitas, como X_n=2^(raiz(log(n)), ou X_n=(log(n))^log(log(n)). >> >>> ********* >>> >>> 7. Um tri=E2ngulo tem lados com medida inteira e =E1rea racional. Prove = >>> que uma de suas alturas tem medida inteira e que o p=E9 desta altura = >>> est=E1 a uma dist=E2ncia inteira dos v=E9rtices do tri=E2ngulo. >>> >> >> Parece que isso nao esta' certo. O triangulo de lados 17, 65 e 80 tem area >> 288 e alturas 576/17, 576/65 e 36/5, que nao sao inteiras... >> >>> ********* >>> >>> 9. Seja K um inteiro >=3D 2.=20 >>> infinito >>> Seja S =3D SOMAT=D3RIO 1 / K^(n^2) =3D 1/K + 1/K^4 + 1/K^9 + 1/K^16 + = >>> ... >>> n =3D 1 >>> Prove que S =E9 irracional. >>> >> >> Se x=p/q e' racional e r/s e' outro racional diferente de x entao >> |x-r/s|=|(ps-qr)/qs|>=1/qs, ou seja, s|x-r/s|>=1/q. >> Por outro lado, soma(n=1 ate' m)(1/K^(n^2)) e' um racional com denominador >> (divisor de) K^(m^2), digamos p/K^(m^2), e >> |S-p/K^(m^2)|<(1/K^((m+1)^2))(1+1/2+1/4+...)=2/K^((m+1)^2), mas >> K^(m^2).2/K^((m+1)^2)=2/K^(2m+1) tende a 0 quando m tende a infinito, e >> portanto S nao pode ser racional. >> >> Abracos, >> Gugu >> ========================================================================= >> Instru??es para entrar na lista, sair da lista e usar a lista em >> http://www.mat.puc-rio.br/~nicolau/olimp/obm-l.html >> O administrador desta lista ? >> ========================================================================= >> > >========================================================================= >Instruções para entrar na lista, sair da lista e usar a lista em >http://www.mat.puc-rio.br/~nicolau/olimp/obm-l.html >O administrador desta lista é >========================================================================= ========================================================================= Instruções para entrar na lista, sair da lista e usar a lista em http://www.mat.puc-rio.br/~nicolau/olimp/obm-l.html O administrador desta lista é ========================================================================= From owner-obm-l@sucuri.mat.puc-rio.br Mon Apr 14 03:19:27 2003 Return-Path: Received: (from majordom@localhost) by sucuri.mat.puc-rio.br (8.9.3/8.9.3) id DAA23418 for obm-l-MTTP; Mon, 14 Apr 2003 03:14:23 -0300 Received: from Euler.impa.br (euler.impa.br [147.65.1.3]) by sucuri.mat.puc-rio.br (8.9.3/8.9.3) with ESMTP id DAA23411 for ; Mon, 14 Apr 2003 03:14:20 -0300 Received: from Gauss.impa.br (Gauss [147.65.4.1]) by Euler.impa.br (8.11.6p2/8.11.6) with ESMTP id h3E6Do010443 for ; Mon, 14 Apr 2003 03:13:50 -0300 (EST) From: Carlos Gustavo Tamm de Araujo Moreira Received: by Gauss.impa.br (8.11.6p2) id h3E6Di101879; Mon, 14 Apr 2003 03:13:44 -0300 (EST) Message-Id: <200304140613.h3E6Di101879@Gauss.impa.br> Subject: Re: [obm-l] Problemas em Aberto To: obm-l@mat.puc-rio.br Date: Mon, 14 Apr 2003 03:13:44 -0300 (EST) In-Reply-To: <044b01c2de8a$b27413c0$3300c57d@bovespa.com> from "=?Windows-1252?Q?Cl=E1udio_\=28Pr=E1tica\=29?=" at Feb 27, 3 03:04:34 pm X-Mailer: ELM [version 2.4 PL25] MIME-Version: 1.0 Content-Type: text/plain; charset=US-ASCII Content-Transfer-Encoding: 7bit Sender: owner-obm-l@sucuri.mat.puc-rio.br Precedence: bulk Reply-To: obm-l@mat.puc-rio.br > >4.=20 >A) As medidas dos =E2ngulos agudos de um tri=E2ngulo pitag=F3rico = >(tri=E2ngulo ret=E2ngulo cujos lados t=EAm medida inteira) n=E3o s=E3o = >inteiras (quando expressos em graus). > >B) Se os lados de um tri=E2ngulo t=EAm medida inteira e um de seus = >=E2ngulos tem medida inteira, ent=E3o esse =E2ngulo mede 60, 90 ou 120 = >graus. > >C) Se um tri=E2ngulo tem os tr=EAs lados e os tr=EAs =E2ngulos com = >medida inteira ent=E3o ele =E9 equil=E1tero. > Vamos primeiro achar todos os angulos racionais (em graus) cujo cosseno e' racional. Para isso, note que 2.cos(nx)=P_n(2.cos(x)), onde P_n e' um polinomio de grau n com coeficientes inteiros cujo coeficiente lider e' 1 (talvez a prova mais rapida disso seja por inducao, a partir de 2.cos(x)=e^(ix)+e^(-ix) e 2.cos(nx)=e^(ix)^n+e^(-ix)^n). Assim, como P_q(2.cos(2.p.Pi/q))=2.cos(q.2.p.Pi/q)=2.cos(2.p.Pi)=2, 2.cos(2.p.Pi/q) e' raiz de um polinomio com coeficientes inteiros e coeficiente lider 1, e logo, se cos(2.p.Pi/q) e' racional entao 2.cos(2.p.Pi/q) e' inteiro, donde cos(2.p.Pi/q) pertence a {-1,-1/2,0,1/2,1}. Nesses casos, sen(2.p.Pi/q) tambem e' racional se e somente se cos(2.p.Pi/q) e' 0,1 ou -1. Isso implica imediatamente o item A). Para o item B, note que se a, b e c sao inteiros entao cos(C)=(a^2+b^2-c^2)/2ab (lei dos cossenos) e' racional, donde cos(C) e' -1/2,0 ou 1/2 e logo C e' 120, 90 ou 60 graus. O item C segue imediatamente do item B, pois se os angulos nao fossem todos 60 graus (o minimo possivel nesse caso), sua soma seria maior que 180 graus. >********* > >5. Nos festejos juninos, 20 casais de dan=E7arinos s=E3o colocados em = >c=EDrculo de tal maneira que um homem e uma mulher formando um par = >est=E3o situados diametralmente opostos. Durante a dan=E7a, dois = >dan=E7arinos adjacentes trocam de lugar enquanto todos os outros = >permanecem na mesma posi=E7=E3o. Essa mudan=E7a =E9 repetida com pares = >adjacentes at=E9 que, na posi=E7=E3o final, os dois dan=E7arinos de cada = >par estejam novamente diametralmente opostos, mas na posi=E7=E3o = >contr=E1ria da inicial. Ent=E3o o n=FAmero m=EDnimo de mudan=E7as, de = >dois dan=E7arinos adjacentes, para acontecer isso =E9: > >(a) 20! (b) 400 (c) 10! (d) 19! (e) 20 > A resposta e' 400 (item (b)). Para ver que vao pode ser menos que isso, definimos uma distancia entre duas permutacoes f e g de 1,2,...,40 a soma das distancias no circulo entre f(j) e g(j) para j=1,2,...,40. Uma troca de posicoes entre dois vizinhos leva uma permutacao a outra permutacao a uma distancia 2 dela. A distancia entre a posicao original e a posicao em que os pares diametralmente opostos trocam de posicao e' 20.40=800. Assim, precisamos de no minimo 800/2=400 trocas de posicoes de vizinhos para chegar nessa situacao. Por outro lado e' possivel fazer isso com 400 trocas de vizinhos: suponha que a ordem original seja 1,2,...,40. Fazemos assim: 1 troca de posicao com 40,39,...,21 e vai para a posicao oposta (enquanto 40,39,...,21 andam uma posicao na direcao de 2). Depois 2 troca de posicao com 40,39,...,21 e assim por diante, ate' 20 trocar de posicao com 40,39,...,21, quando todos ficam nas posicoes diametralmente opostas as iniciais. >************* Abracos, Gugu ========================================================================= Instruções para entrar na lista, sair da lista e usar a lista em http://www.mat.puc-rio.br/~nicolau/olimp/obm-l.html O administrador desta lista é ========================================================================= From owner-obm-l@sucuri.mat.puc-rio.br Mon Apr 14 03:25:02 2003 Return-Path: Received: (from majordom@localhost) by sucuri.mat.puc-rio.br (8.9.3/8.9.3) id DAA23637 for obm-l-MTTP; Mon, 14 Apr 2003 03:19:56 -0300 Received: from Euler.impa.br (euler.impa.br [147.65.1.3]) by sucuri.mat.puc-rio.br (8.9.3/8.9.3) with ESMTP id DAA23633 for ; Mon, 14 Apr 2003 03:19:53 -0300 Received: from Gauss.impa.br (Gauss [147.65.4.1]) by Euler.impa.br (8.11.6p2/8.11.6) with ESMTP id h3E6JN010611 for ; Mon, 14 Apr 2003 03:19:23 -0300 (EST) From: Carlos Gustavo Tamm de Araujo Moreira Received: by Gauss.impa.br (8.11.6p2) id h3E6JH002200; Mon, 14 Apr 2003 03:19:17 -0300 (EST) Message-Id: <200304140619.h3E6JH002200@Gauss.impa.br> Subject: Re: [obm-l] Problemas em Aberto II To: obm-l@mat.puc-rio.br Date: Mon, 14 Apr 2003 03:19:17 -0300 (EST) In-Reply-To: <045901c2de8a$b6ac6640$3300c57d@bovespa.com> from "=?Windows-1252?Q?Cl=E1udio_\=28Pr=E1tica\=29?=" at Feb 27, 3 03:04:44 pm X-Mailer: ELM [version 2.4 PL25] MIME-Version: 1.0 Content-Type: text/plain; charset=US-ASCII Content-Transfer-Encoding: 7bit Sender: owner-obm-l@sucuri.mat.puc-rio.br Precedence: bulk Reply-To: obm-l@mat.puc-rio.br > >12. Dada a sequencia a[n+1]=3D 2a[1]*a[n] - a[n-1] definida para todo = >n>=3D1 tal que a[0]=3D100 e a[100]=3D 0.=20 >a) Mostre que | a[1] |<=3D1. > >b) Determine a[2003]. > Esse problema nao esta' certo. Ele parece muito com um problema que apareceu na Eureka 9, na secao olimpiadas ao redor do mundo (e' o problema 34, na pagina 45), que tambem dizia que a[n+1]=2.a[1].a[n]-a[n-1] para todo n>=1 e que a[100]=0, mas dizia que a[0]=1 (e nao a[0]=100), e pedia para calcular a[1996] (na verdade deveria pedir para calcular todos os valores possiveis de a[1996]) em vez de a[2003]. Nesses problemas temos dois casos: a[1] pertence a {-1,1} ou x^2-2.a[1].x+1 tem duas raizes distintas, a[1]+raiz(a[1]^2-1) e a[1]-raiz(a[1]^2-1): nesse segundo caso, a[n] pode ser escrito como A.(a[1]+raiz(a[1]^2-1))^n+B.(a[1]-raiz(a[1]^2-1))^n para certas constantes A e B (ver o artigo sobre recorrencia na Eureka 9). Note que as condicoes do problema (nas duas versoes) implicam A.(a[1]+raiz(a[1]^2-1))+B.(a[1]-raiz(a[1]^2-1))=a[1]. Se a[0]=1, temos A+B=1, e portanto A=B=1/2. Por outro, se a[0]=100 entao A+B=100, B=100-A e A.(a[1]+raiz(a[1]^2-1))+(100-A).(a[1]-raiz(a[1]^2-1))=a[1], donde A=(100.raiz(a[1]^2-1)-99.a[1])/2.raiz(a[1]^2-1) e B=(100.raiz(a[1]^2-1)+99.a[1])/2.raiz(a[1]^2-1). Como a[100]=0, temos A.(a[1]+raiz(a[1]^2-1))^100+B.(a[1]-raiz(a[1]^2-1))^100=0, donde (a[1]+raiz(a[1]^2-1))^200=(a[1]+raiz(a[1]^2-1))^100/(a[1]-raiz(a[1]^2-1))^100=-B/A. No caso em que a[1]=100, isso fica (*) (a[1]+raiz(a[1]^2-1))^200=(99.a[1]+100.raiz(a[1]^2-1))/(99.a[1]-100.raiz(a[1]^2-1)). Quando a[1] tende a 100/raiz(199)(que e' solucao de 99.a[1]-100.raiz(a[1]^2-1)=0), o lado direito de (*) tende a infinito, mas, se a[1]=5/4, o lado esquerdo e' 2^200, que e' maior que 795/195=139/39, que e' o lado direito. Assim, existe a[1] maior que 1 que faz o lado direito ser igual ao lado esquerdo, donde a primeira conclusao do problema e' falsa. Vamos entao resolver o problema trocando a[0]=100 por a[0]=1. Nesse caso, a condicao a[100]=0 equivale a (a[1]+raiz(a[1]^2-1))^200=-1=(a[1]-raiz(a[1]^2-1))^200, donde (a[1]+raiz(a[1]^2-1))=e^((2k+1).Pi.i/200) e (a[1]-raiz(a[1]^2-1))=e^(-(2k+1).Pi.i/200) (para algum k inteiro) tem modulo 1, donde |a(1)|<=1. Dai segue que a[n]=1/2((a[1]+raiz(a[1]^2-1))^n+(a[1]-raiz(a[1]^2-1))^n)= =1/2(e^((2k+1).n.Pi.i/200)+e^(-(2k+1).n.Pi.i/200))=2.cos((2k+1).n.Pi/200) para todo n. Em particular, temos a[2003]=2.cos((2k+1).2003.Pi/200)=2.cos(3.(2k+1).Pi/200). Por outro lado, ainda podemos escolher o valor de k, e temos pois os seguintes 100 valores possiveis para a[2003]: cos(Pi/200),cos(3.Pi/200),...,cos(199.Pi/200). >********** > >13. X, Y e Z s=E3o reais positivos e satisfazem o sistema abaixo, > >X^2 + XY + (Y^2)/3 =3D 25 >(Y^2)/3 + Z^2 =3D 9 >Z^2 + ZX + X^2 =3D 16 > >Encontre o valor de ( XY + 2YZ + 3ZX ). > >SUGEST=C3O : Voc=EA nao precisa, necessariamente, resolver o sistema ... > Somando as duas ultimas equacoes e comparando com a primeira obtemos 2Z^2+ZX=XY, donde Y=Z/X.(2Z+X) (note que X deve ser nao nulo, senao Z tambem seria nulo e nao poderia valer a terceira equacao; Z tambem e' diferente de 0, senao XY=0, e, como X e' nao nulo, Y=0, e a segunda equacao daria 0=9, absurdo). Dividindo a segunda e a terceira equacoes, obtemos 9/16=(Y^2/3+Z^2)/(Z^2+ZX+X^2)=((Y/Z)^2/3+1)/(1+(X/Z)+(X/Z)^2)= =((2(Z/X)+1)^2/3+1)/(1+(X/Z)+(X/Z)^2)=((1+2/a)^2/3+1)/(1+a+a^2)=4/3a^2, onde a=X/Z, donde a^2=64/27 e a=8/raiz(27) ou a=-8/raiz(27). Por outro lado, nesse caso, XY+2YZ+3ZX=z^2(a.Y/Z+2.Y/Z+3a)=Z^2(a(1+2/a)+2(1+2/a)+3a)=4Z^2.(a^2+a+1)/a, mas Z^2.(a^2+a+1)=X^2+XZ+Z^2=16, donde XY+2YZ+3ZX=64/a pertence a {8.raiz(27),-8.raiz(27)} = {24.raiz(3),-24.raiz(3)}. >********** Abracos, Gugu ========================================================================= Instruções para entrar na lista, sair da lista e usar a lista em http://www.mat.puc-rio.br/~nicolau/olimp/obm-l.html O administrador desta lista é ========================================================================= From owner-obm-l@sucuri.mat.puc-rio.br Mon Apr 14 10:26:26 2003 Return-Path: Received: (from majordom@localhost) by sucuri.mat.puc-rio.br (8.9.3/8.9.3) id KAA29859 for obm-l-MTTP; Mon, 14 Apr 2003 10:21:57 -0300 Received: (from nicolau@localhost) by sucuri.mat.puc-rio.br (8.9.3/8.9.3) id KAA29853 for obm-l@mat.puc-rio.br; Mon, 14 Apr 2003 10:21:56 -0300 Date: Mon, 14 Apr 2003 10:21:56 -0300 From: "Nicolau C. Saldanha" To: obm-l@mat.puc-rio.br Subject: [obm-l] Re: =?iso-8859-1?Q?=5Bobm-l=5D_Combinat=F3ria!?= Message-ID: <20030414102156.A29433@sucuri.mat.puc-rio.br> References: <20030413225321.62021.qmail@web13007.mail.yahoo.com> <3E99EFC6.10505@centroin.com.br> Mime-Version: 1.0 Content-Type: text/plain; charset=iso-8859-1 Content-Disposition: inline Content-Transfer-Encoding: 8bit User-Agent: Mutt/1.2.5i In-Reply-To: <3E99EFC6.10505@centroin.com.br>; from morgado@centroin.com.br on Sun, Apr 13, 2003 at 08:16:22PM -0300 Sender: owner-obm-l@sucuri.mat.puc-rio.br Precedence: bulk Reply-To: obm-l@mat.puc-rio.br On Sun, Apr 13, 2003 at 08:16:22PM -0300, A. C. Morgado wrote: ... > Aproveito para perguntar a todos a sua definiçao de Fibonacci. Uns > começam a enumeraçao dos termos em 0, outros em 1. Oi Morgado, nós já falamos sobre isso pessoalmente. Minha definição favorita é F(0) = 0, F(1) = 1. Algumas obras que acompanham esta convenção: The On-Line Encyclopaedia of Integer Sequences: http://www.research.att.com/cgi-bin/access.cgi/as/njas/sequences/eisA.cgi?Anum=A000045 Concrete Mathematics, Graham, Knuth, Patashnik http://www.mcs.surrey.ac.uk/Personal/R.Knott/Fibonacci/fibCalcX.html http://www-gap.dcs.st-and.ac.uk/~history/Mathematicians/Fibonacci.html Algumas propriedades que só valem com esta convenção: n|m -> F(n)|F(m) F(mdc(n,m)) = mdc(F(n),F(m)) []s, N. ========================================================================= Instruções para entrar na lista, sair da lista e usar a lista em http://www.mat.puc-rio.br/~nicolau/olimp/obm-l.html O administrador desta lista é ========================================================================= From owner-obm-l@sucuri.mat.puc-rio.br Mon Apr 14 10:36:35 2003 Return-Path: Received: (from majordom@localhost) by sucuri.mat.puc-rio.br (8.9.3/8.9.3) id KAA30007 for obm-l-MTTP; Mon, 14 Apr 2003 10:32:54 -0300 Received: from web14305.mail.yahoo.com (web14305.mail.yahoo.com [216.136.173.81]) by sucuri.mat.puc-rio.br (8.9.3/8.9.3) with SMTP id KAA30003 for ; Mon, 14 Apr 2003 10:32:50 -0300 Message-ID: <20030414133218.22445.qmail@web14305.mail.yahoo.com> Received: from [200.17.25.3] by web14305.mail.yahoo.com via HTTP; Mon, 14 Apr 2003 06:32:18 PDT Date: Mon, 14 Apr 2003 06:32:18 -0700 (PDT) From: Rafael Subject: [obm-l] 3 semicircunferências To: OBM MIME-Version: 1.0 Content-Type: text/plain; charset=us-ascii Sender: owner-obm-l@sucuri.mat.puc-rio.br Precedence: bulk Reply-To: obm-l@mat.puc-rio.br Olá Pessoas! Alguém consegue resolver esse? A , B e C são três pontos alinhados tais que os segmentos consecutivos AB = 2a e BC = 2b. Sobre os segmentos AB, BC e AC tomados como diâmetros e de um mesmo lado de AC descrevem três semicircunferências. Calcular o raio de circunferência que é tangente a estas três semicircunferências. resposta: [ab(a + b)]/(a² + ab + b²) Abraços, Rafael. __________________________________________________ Do you Yahoo!? Yahoo! Tax Center - File online, calculators, forms, and more http://tax.yahoo.com ========================================================================= Instruções para entrar na lista, sair da lista e usar a lista em http://www.mat.puc-rio.br/~nicolau/olimp/obm-l.html O administrador desta lista é ========================================================================= From owner-obm-l@sucuri.mat.puc-rio.br Mon Apr 14 13:14:04 2003 Return-Path: Received: (from majordom@localhost) by sucuri.mat.puc-rio.br (8.9.3/8.9.3) id NAA01572 for obm-l-MTTP; Mon, 14 Apr 2003 13:10:08 -0300 Received: from hotmail.com (f83.sea1.hotmail.com [207.68.163.83]) by sucuri.mat.puc-rio.br (8.9.3/8.9.3) with ESMTP id NAA01568 for ; Mon, 14 Apr 2003 13:10:03 -0300 Received: from mail pickup service by hotmail.com with Microsoft SMTPSVC; Mon, 14 Apr 2003 09:09:28 -0700 Received: from 198.81.8.1 by sea1fd.sea1.hotmail.msn.com with HTTP; Mon, 14 Apr 2003 16:09:28 GMT X-Originating-IP: [198.81.8.1] X-Originating-Email: [fredericor@hotmail.com] From: "Frederico Reis Marques de Brito" To: obm-l@mat.puc-rio.br Subject: [obm-l] =?iso-8859-1?B?UmU6IFtvYm0tbF0gRGVtb25zdHJh5+NvIHBvciBpbmR15+Nv?= Date: Mon, 14 Apr 2003 13:09:28 -0300 Mime-Version: 1.0 Content-Type: text/plain; charset=iso-8859-1; format=flowed Message-ID: X-OriginalArrivalTime: 14 Apr 2003 16:09:28.0556 (UTC) FILETIME=[39E0C2C0:01C302A0] Sender: owner-obm-l@sucuri.mat.puc-rio.br Precedence: bulk Reply-To: obm-l@mat.puc-rio.br Ainda que fosse, restaria demonstrar que todo número inteiro > 1 ou é primo ou composto. Na realidade, o problema proposto é exatamente a parte que trata da existência no Teorema Fundamental da Aritmética, pelo qual todo inteiro > 1 ou é primo ou se fatora ( de forma única ) como produto de primos. Cabendo lembrar que um número é primo se é inteiro positivo e tem exatamente dois divisores positivos. Para a demonstração, consequência trivial do PIM, veja a resposta do Morgado. Abraço, FREDERICO. >From: "Jose Francisco Guimaraes Costa" >Reply-To: obm-l@mat.puc-rio.br >To: >Subject: [obm-l] Demonstração por indução >Date: Sun, 13 Apr 2003 20:01:30 -0300 > >Isto não é a definição de número composto, e portanto não demonstrável? > >JF > >----- Original Message ----- >From: "Henrique Patrício Sant'Anna Branco" >To: "OBM" >Sent: Sunday, April 13, 2003 5:19 PM >Subject: [obm-l] Demonstração por indução > > > > Pessoal, > > > > Como eu demonstro, pelo principio da indução finita, o seguinte: > > > > Para todo número natural "n", "n" é primo ou produto de primos. > > > > Grato, > > Henrique. > > > > >========================================================================= > > Instruções para entrar na lista, sair da lista e usar a lista em > > http://www.mat.puc-rio.br/~nicolau/olimp/obm-l.html > > O administrador desta lista é > > >========================================================================= > > > >========================================================================= >Instruções para entrar na lista, sair da lista e usar a lista em >http://www.mat.puc-rio.br/~nicolau/olimp/obm-l.html >O administrador desta lista é >========================================================================= _________________________________________________________________ MSN Hotmail, o maior webmail do Brasil. http://www.hotmail.com ========================================================================= Instruções para entrar na lista, sair da lista e usar a lista em http://www.mat.puc-rio.br/~nicolau/olimp/obm-l.html O administrador desta lista é ========================================================================= From owner-obm-l@sucuri.mat.puc-rio.br Mon Apr 14 15:10:07 2003 Return-Path: Received: (from majordom@localhost) by sucuri.mat.puc-rio.br (8.9.3/8.9.3) id PAA04209 for obm-l-MTTP; Mon, 14 Apr 2003 15:05:33 -0300 Received: (from nicolau@localhost) by sucuri.mat.puc-rio.br (8.9.3/8.9.3) id PAA04204 for obm-l@mat.puc-rio.br; Mon, 14 Apr 2003 15:05:33 -0300 Date: Mon, 14 Apr 2003 15:05:32 -0300 From: "Nicolau C. Saldanha" To: obm-l@mat.puc-rio.br Subject: Re: [obm-l] FW: Teoria dos grupos Message-ID: <20030414150532.A2668@sucuri.mat.puc-rio.br> References: <200304111925.h3BJPfW03075@Gauss.impa.br> Mime-Version: 1.0 Content-Type: text/plain; charset=iso-8859-1 Content-Disposition: inline Content-Transfer-Encoding: 8bit User-Agent: Mutt/1.2.5i In-Reply-To: <200304111925.h3BJPfW03075@Gauss.impa.br>; from gugu@impa.br on Fri, Apr 11, 2003 at 04:25:40PM -0300 Sender: owner-obm-l@sucuri.mat.puc-rio.br Precedence: bulk Reply-To: obm-l@mat.puc-rio.br On Fri, Apr 11, 2003 at 04:25:40PM -0300, Carlos Gustavo Tamm de Araujo Moreira wrote: > Acho que isso nao esta' certo. Por exemplo, f(2)=2 para todo > automorfismo de Z/4Z, pois 2 e' o unico elemento de ordem 2. > > > >Seja G um grupo cuja ordem eh diferente de 2. > >Seja a um elemento de G tal que f(a) = a para todo automorfismo f:G -> G. > >Prove que a = identidade de G. Como o Gugu apontou o problema original estava errado mas acho que vale a pena falar um pouco mais. Se o grupo G é não abeliano então as conjugações deixam fixos exatamente os elementos do centro do grupo. Não é claro para mim o que acontece com os elementos do centro. O exemplo G = Q8 x Z/2 (onde Q8 é o grupo de 8 elementos dos quatérnios +-1, +-i, +-j, +-k) tem centro Z/2 x Z/2: o elemento (-1,0) tem ordem 2 e é o único elemento do centro que é quadrado de alguém logo fica fixo por todo automorfismo de G (apesar de existirem automorfismos do centro levando ele noutro elemento). Não sei se existe, por exemplo, um grupo cujo centro seja Z/3 para o qual não existe automorfismo agindo de forma não trivial no centro. Eu tentei o exemplo G = SL(3,Z/7) (matrizes 3x3 de determinante 1 com coeficientes no corpo Z/7) mas o automorfismo A |-> A^(-T) leva 2I em 4I. []s, N. ========================================================================= Instruções para entrar na lista, sair da lista e usar a lista em http://www.mat.puc-rio.br/~nicolau/olimp/obm-l.html O administrador desta lista é ========================================================================= From owner-obm-l@sucuri.mat.puc-rio.br Mon Apr 14 15:11:55 2003 Return-Path: Received: (from majordom@localhost) by sucuri.mat.puc-rio.br (8.9.3/8.9.3) id PAA04288 for obm-l-MTTP; Mon, 14 Apr 2003 15:09:09 -0300 Received: (from nicolau@localhost) by sucuri.mat.puc-rio.br (8.9.3/8.9.3) id PAA04282 for obm-l@mat.puc-rio.br; Mon, 14 Apr 2003 15:09:08 -0300 Date: Mon, 14 Apr 2003 15:09:08 -0300 From: "Nicolau C. Saldanha" To: obm-l@mat.puc-rio.br Subject: [obm-l] Re: =?iso-8859-1?Q?=5Bobm-l=5D_Combiana=E7=F5es_completas?= Message-ID: <20030414150908.B2668@sucuri.mat.puc-rio.br> References: <000901c30149$16310ea0$9865fea9@computer> <3E98A296.5090107@centroin.com.br> Mime-Version: 1.0 Content-Type: text/plain; charset=iso-8859-1 Content-Disposition: inline Content-Transfer-Encoding: 8bit User-Agent: Mutt/1.2.5i In-Reply-To: <3E98A296.5090107@centroin.com.br>; from morgado@centroin.com.br on Sat, Apr 12, 2003 at 08:34:46PM -0300 Sender: owner-obm-l@sucuri.mat.puc-rio.br Precedence: bulk Reply-To: obm-l@mat.puc-rio.br On Sat, Apr 12, 2003 at 08:34:46PM -0300, A. C. Morgado wrote: > Ta certo. So que eh mais comum escrever CR e nao CC. > CR(n, p) = C(n+p-1,p). É um pouco bobo polemizar sobre notação mas eu não recomendaria a criação de um símbolo novo (como CR ou CC) para uma variação tão simples dos números binomiais. Aliás eu também nunca uso C(n,p), eu uso binomial(n,p). No papel ou no quadro negro isso se escreve como um par de () com um número acima do outro. []s, N. ========================================================================= Instruções para entrar na lista, sair da lista e usar a lista em http://www.mat.puc-rio.br/~nicolau/olimp/obm-l.html O administrador desta lista é ========================================================================= From owner-obm-l@sucuri.mat.puc-rio.br Mon Apr 14 15:19:40 2003 Return-Path: Received: (from majordom@localhost) by sucuri.mat.puc-rio.br (8.9.3/8.9.3) id PAA04436 for obm-l-MTTP; Mon, 14 Apr 2003 15:16:48 -0300 Received: from ns3bind.localdomain ([200.230.34.5]) by sucuri.mat.puc-rio.br (8.9.3/8.9.3) with ESMTP id PAA04427 for ; Mon, 14 Apr 2003 15:16:41 -0300 Received: from servico2 ([200.230.34.227]) by ns3bind.localdomain (8.11.6/X.XX.X) with SMTP id h3EICId19598 for ; Mon, 14 Apr 2003 15:12:19 -0300 Message-ID: <010901c302b2$0f61ae80$3300c57d@bovespa.com> From: "=?iso-8859-1?Q?Cl=E1udio_\=28Pr=E1tica\=29?=" To: References: <20030414133218.22445.qmail@web14305.mail.yahoo.com> Subject: [obm-l] =?iso-8859-1?Q?Re:_=5Bobm-l=5D_3_semicircunfer=EAncias?= Date: Mon, 14 Apr 2003 15:16:59 -0300 MIME-Version: 1.0 Content-Type: text/plain; charset="iso-8859-1" Content-Transfer-Encoding: 8bit X-Priority: 3 X-MSMail-Priority: Normal X-Mailer: Microsoft Outlook Express 5.50.4920.2300 X-MimeOLE: Produced By Microsoft MimeOLE V5.50.4920.2300 Sender: owner-obm-l@sucuri.mat.puc-rio.br Precedence: bulk Reply-To: obm-l@mat.puc-rio.br > > A , B e C são três pontos alinhados tais que os > segmentos consecutivos AB = 2a e BC = 2b. Sobre os > segmentos AB, BC e AC tomados como diâmetros e de um > mesmo lado de AC descrevem três semicircunferências. > Calcular o raio de circunferência que é tangente a > estas três semicircunferências. > resposta: [ab(a + b)]/(a² + ab + b²) > Oi, Rafael: Seja R o raio pedido. Sejam os pontos: O: centro da circunferência tangente às três outras; M: ponto médio de AB N: ponto médio de BC P: ponto médio de AC O triângulo OMN tem lados: OM = a + R ON = b + R MN = a + b e uma ceviana: OP = a + b - R tal que: MP = b; PN = a Se o ângulo MPO mede x, então o NPO mede 180 - x. Lei dos cossenos no triângulo MPO: OM^2 = MP^2 + OP^2 - 2*MP*OP*cos(MPO) ==> (a+R)^2 = b^2 + (a+b-R)^2 - 2*b*(a+b-R)*cos(x) ==> cos(x) = [b^2 + (a+b-R)^2 - (a+R)^2]/[2*b*(a+b-R)] Lei dos cossenos no triângulo NPO: ON^2 = PN^2 + OP^2 - 2*PN*OP*cos(NPO) ==> (b+R)^2 = a^2 + (a+b-R)^2 + 2*a*(a+b-R)*cos(x) ==> cos(x) = [(b+R)^2 - a^2 - (a+b-R)^2]/[2*a*(a+b-R)] Igualando as duas expressões para cos(x) e já fazendo algumas simplificações, teremos: a*[b^2 + (a+b-R)^2 - (a+R)^2] = b*[(b+R)^2 - a^2 - (a+b-R)^2] ==> a*b^2 + a^2*b = a^2*R + b^2*R + a*b*R ==> R = a*b*(a+b)/(a^2 + a*b + b^2) Um abraço, Claudio. ========================================================================= Instruções para entrar na lista, sair da lista e usar a lista em http://www.mat.puc-rio.br/~nicolau/olimp/obm-l.html O administrador desta lista é ========================================================================= From owner-obm-l@sucuri.mat.puc-rio.br Mon Apr 14 16:50:59 2003 Return-Path: Received: (from majordom@localhost) by sucuri.mat.puc-rio.br (8.9.3/8.9.3) id QAA08023 for obm-l-MTTP; Mon, 14 Apr 2003 16:47:38 -0300 Received: (from nicolau@localhost) by sucuri.mat.puc-rio.br (8.9.3/8.9.3) id QAA08017 for obm-l@mat.puc-rio.br; Mon, 14 Apr 2003 16:47:38 -0300 Date: Mon, 14 Apr 2003 16:47:37 -0300 From: "Nicolau C. Saldanha" To: obm-l@mat.puc-rio.br Subject: Re: [obm-l] FW: Teoria dos grupos Message-ID: <20030414164737.C7525@sucuri.mat.puc-rio.br> References: <200304111925.h3BJPfW03075@Gauss.impa.br> <20030414150532.A2668@sucuri.mat.puc-rio.br> Mime-Version: 1.0 Content-Type: text/plain; charset=iso-8859-1 Content-Disposition: inline Content-Transfer-Encoding: 8bit User-Agent: Mutt/1.2.5i In-Reply-To: <20030414150532.A2668@sucuri.mat.puc-rio.br>; from nicolau@sucuri.mat.puc-rio.br on Mon, Apr 14, 2003 at 03:05:32PM -0300 Sender: owner-obm-l@sucuri.mat.puc-rio.br Precedence: bulk Reply-To: obm-l@mat.puc-rio.br On Mon, Apr 14, 2003 at 03:05:32PM -0300, Nicolau C. Saldanha wrote: > On Fri, Apr 11, 2003 at 04:25:40PM -0300, Carlos Gustavo Tamm de Araujo Moreira wrote: > > Acho que isso nao esta' certo. Por exemplo, f(2)=2 para todo > > automorfismo de Z/4Z, pois 2 e' o unico elemento de ordem 2. > > > > > >Seja G um grupo cuja ordem eh diferente de 2. > > >Seja a um elemento de G tal que f(a) = a para todo automorfismo f:G -> G. > > >Prove que a = identidade de G. > > Como o Gugu apontou o problema original estava errado mas acho que vale > a pena falar um pouco mais. > > Se o grupo G é não abeliano então as conjugações deixam fixos exatamente > os elementos do centro do grupo. Não é claro para mim o que acontece > com os elementos do centro. O exemplo G = Q8 x Z/2 (onde Q8 é o grupo > de 8 elementos dos quatérnios +-1, +-i, +-j, +-k) tem centro Z/2 x Z/2: > o elemento (-1,0) tem ordem 2 e é o único elemento do centro que é > quadrado de alguém logo fica fixo por todo automorfismo de G > (apesar de existirem automorfismos do centro levando ele noutro elemento). > Não sei se existe, por exemplo, um grupo cujo centro seja Z/3 para o qual > não existe automorfismo agindo de forma não trivial no centro. > Eu tentei o exemplo G = SL(3,Z/7) (matrizes 3x3 de determinante 1 > com coeficientes no corpo Z/7) mas o automorfismo A |-> A^(-T) > leva 2I em 4I. Outro exemplo instrutivo é SL(5,Z/11) (matrizes 5x5 de determinante 1 com coeficientes no corpo Z/11). O centro é isomorfo a Z/5, são as matrizes diagonais com entrada diagonal 1, 3, 4, 5 ou 9. As conjugações por elementos de GL(5,Z/11) mantem estes 5 caras parados e o automorfismo externo A |-> A^(-T) (a inversa da transposta) troca 3 <-> 4, 5 <-> 9. Consultando o Atlas of Finite Groups aprendi que não existem outros automorfismos além dos gerados por estes exemplos. Assim não existe, por exemplo, um automorfismo que leve 3 em 5, apesar de eu não ver uma propriedade "simples" destes elementos que deixe claro pq um tal automorfismo não possa existir. Note que existe um automorfismo do centro nele mesmo levando 3 em 5. []s, N. ========================================================================= Instruções para entrar na lista, sair da lista e usar a lista em http://www.mat.puc-rio.br/~nicolau/olimp/obm-l.html O administrador desta lista é ========================================================================= From owner-obm-l@sucuri.mat.puc-rio.br Mon Apr 14 17:15:15 2003 Return-Path: Received: (from majordom@localhost) by sucuri.mat.puc-rio.br (8.9.3/8.9.3) id RAA08761 for obm-l-MTTP; Mon, 14 Apr 2003 17:12:33 -0300 Received: from ns3bind.localdomain ([200.230.34.5]) by sucuri.mat.puc-rio.br (8.9.3/8.9.3) with ESMTP id RAA08757 for ; Mon, 14 Apr 2003 17:12:29 -0300 Received: from servico2 ([200.230.34.224]) by ns3bind.localdomain (8.11.6/X.XX.X) with SMTP id h3EK88d29861 for ; Mon, 14 Apr 2003 17:08:08 -0300 Message-ID: <01b901c302c2$3db52ae0$3300c57d@bovespa.com> From: "=?iso-8859-1?Q?Cl=E1udio_\=28Pr=E1tica\=29?=" To: References: <200304140606.h3E66SA01501@Gauss.impa.br> Subject: Re: [obm-l] Problemas em Aberto Date: Mon, 14 Apr 2003 17:12:52 -0300 MIME-Version: 1.0 Content-Type: text/plain; charset="iso-8859-1" Content-Transfer-Encoding: 8bit X-Priority: 3 X-MSMail-Priority: Normal X-Mailer: Microsoft Outlook Express 5.50.4920.2300 X-MimeOLE: Produced By Microsoft MimeOLE V5.50.4920.2300 Sender: owner-obm-l@sucuri.mat.puc-rio.br Precedence: bulk Reply-To: obm-l@mat.puc-rio.br Oi, Gugu: Vou seguir sua sugestão e ver se consigo fazer o da Eureka. Também vou dar uma pensada na generalização do no. 9. Talvez dê pra provar que, apesar do numerador crescer, a representação Q-esimal (supondo R = P/Q, com 0 < P < Q e (P,Q) = 1) jamais se torna periódica. No caso da sua demonstração, talvez dê pra estreitar a desigualdade: |S-p/K^(m^2)|<(1/K^((m+1)^2))(1+1/2+1/4+...) de forma que no fim também se chegue a algo que tende a zero qaundo m -> infinito. Um abraço, Claudio. ----- Original Message ----- From: "Carlos Gustavo Tamm de Araujo Moreira" To: Sent: Monday, April 14, 2003 3:06 AM Subject: Re: [obm-l] Problemas em Aberto > Caro Claudio, > Eu tinha proposto esse problema da Eureka para uma IMO antiga, mas nao > entrou... Tente primeiro colocar o triangulo com coordenadas racionais no > plano com um dos lados no eixo x e depois tente aplicar uma rotacao > conveniente (multiplicar por um complexo de modulo 1 com coordenadas > racionais conveniente). Um pouco de aritmetica em Z[i] ajuda... > Sobre o problema 9, eu nao vejo como as solucoes se generalizam para R > racional... Os numeradores podem ficar grandes... > Abracos, > Gugu > > > > >Oi, Gugu: > > > >O no. 7 foi baseado num problema da Eureka, mas eu acabei de ver onde errei. > >O problema original eh: > > > >"Um triangulo tem os lados de medidas inteiras e area racional. Prove que > >ele eh congruente a um triangulo cujos vertices tem coordenadas inteiras". > > > >Eu assumi, erroneamente, que um dos lados do triangulo de reticulado (essa > >eh a traducao correta de "lattice triangle"?) poderia ser paralelo a um dos > >eixos coordenados, de forma que o pe' da altura relativa a este lado teria > >coordenadas inteiras. O seu contra-exemplo mostra que esta hipotese nao eh > >valida em geral. > > > >Um triangulo de reticulado congruente ao seu teria como vertices: > >(0,0), (15,8), (48,64) ==> nenhum lado eh paralelo aos eixos. > > > >De qualquer forma, fica ai o enunciado do problema original. Ao que me > >consta, a Eureka ainda nao recebeu uma solucao para este. > > > >***** > > > >A solucao que eu tinha imaginado pro no. 9 eh: > > > >O numero S, quando expresso na base K eh igual a: > >0,100100001000000100000000100..., ou seja, uma K-esimal infinita e nao > >periodica. Logo, S eh irracional. > > > >De qualquer forma, ambas as solucoes so facilmente generalizaveis para o > >caso de: > >SOMA(n>=1) R^(n^2), onde R eh um racional entre 0 e 1. > > > > > >Um abraco, > >Claudio. > > > > > >on 12.04.03 02:50, Carlos Gustavo Tamm de Araujo Moreira at gugu@impa.br > >wrote: > > > >>> > >>> 6. D=EA um exemplo de uma sequ=EAncia (Xn) de n=FAmeros reais tal que:=20 > >>> > >>> lim ( Xn / n^t ) =3D 0 para todo t > 0=20 > >>> e > >>> lim ( [log(n)]^k / Xn ) =3D 0 para todo k > 0 > >>> > >> > >> Existem muitas, como X_n=2^(raiz(log(n)), ou X_n=(log(n))^log(log(n)). > >> > >>> ********* > >>> > >>> 7. Um tri=E2ngulo tem lados com medida inteira e =E1rea racional. Prove = > >>> que uma de suas alturas tem medida inteira e que o p=E9 desta altura = > >>> est=E1 a uma dist=E2ncia inteira dos v=E9rtices do tri=E2ngulo. > >>> > >> > >> Parece que isso nao esta' certo. O triangulo de lados 17, 65 e 80 tem area > >> 288 e alturas 576/17, 576/65 e 36/5, que nao sao inteiras... > >> > >>> ********* > >>> > >>> 9. Seja K um inteiro >=3D 2.=20 > >>> infinito > >>> Seja S =3D SOMAT=D3RIO 1 / K^(n^2) =3D 1/K + 1/K^4 + 1/K^9 + 1/K^16 + = > >>> ... > >>> n =3D 1 > >>> Prove que S =E9 irracional. > >>> > >> > >> Se x=p/q e' racional e r/s e' outro racional diferente de x entao > >> |x-r/s|=|(ps-qr)/qs|>=1/qs, ou seja, s|x-r/s|>=1/q. > >> Por outro lado, soma(n=1 ate' m)(1/K^(n^2)) e' um racional com denominador > >> (divisor de) K^(m^2), digamos p/K^(m^2), e > >> |S-p/K^(m^2)|<(1/K^((m+1)^2))(1+1/2+1/4+...)=2/K^((m+1)^2), mas > >> K^(m^2).2/K^((m+1)^2)=2/K^(2m+1) tende a 0 quando m tende a infinito, e > >> portanto S nao pode ser racional. > >> > >> Abracos, > >> Gugu > >> ========================================================================= > >> Instru??es para entrar na lista, sair da lista e usar a lista em > >> http://www.mat.puc-rio.br/~nicolau/olimp/obm-l.html > >> O administrador desta lista ? > >> ========================================================================= > >> > > > >========================================================================= > >Instruções para entrar na lista, sair da lista e usar a lista em > >http://www.mat.puc-rio.br/~nicolau/olimp/obm-l.html > >O administrador desta lista é > >========================================================================= > > ========================================================================= > Instruções para entrar na lista, sair da lista e usar a lista em > http://www.mat.puc-rio.br/~nicolau/olimp/obm-l.html > O administrador desta lista é > ========================================================================= ========================================================================= Instruções para entrar na lista, sair da lista e usar a lista em http://www.mat.puc-rio.br/~nicolau/olimp/obm-l.html O administrador desta lista é ========================================================================= From owner-obm-l@sucuri.mat.puc-rio.br Mon Apr 14 17:32:43 2003 Return-Path: Received: (from majordom@localhost) by sucuri.mat.puc-rio.br (8.9.3/8.9.3) id RAA09389 for obm-l-MTTP; Mon, 14 Apr 2003 17:29:09 -0300 Received: from puma.unisys.com.br (smtp.unisys.com.br [200.220.64.7]) by sucuri.mat.puc-rio.br (8.9.3/8.9.3) with ESMTP id RAA09385 for ; Mon, 14 Apr 2003 17:29:05 -0300 Received: from n8x4f9 (riohiper01p96.uninet.com.br [200.220.2.96]) by puma.unisys.com.br (8.12.9/8.12.3) with SMTP id h3EKSWRW007529 for ; Mon, 14 Apr 2003 17:28:35 -0300 (EST) X-Spam-Filter: check_local@puma.unisys.com.br by digitalanswers.org Message-ID: <00eb01c302c4$eda27b40$2302dcc8@n8x4f9> From: "Jose Francisco Guimaraes Costa" To: References: Subject: [obm-l] =?iso-8859-1?B?RGVtb25zdHJh5+NvIHBvciBpbmR15+Nv?= Date: Mon, 14 Apr 2003 17:31:11 -0300 MIME-Version: 1.0 Content-Type: multipart/alternative; boundary="----=_NextPart_000_00C8_01C302AB.A47B19E0" X-Priority: 3 X-MSMail-Priority: Normal X-Mailer: Microsoft Outlook Express 5.00.2615.200 X-MimeOLE: Produced By Microsoft MimeOLE V5.00.2615.200 Sender: owner-obm-l@sucuri.mat.puc-rio.br Precedence: bulk Reply-To: obm-l@mat.puc-rio.br This is a multi-part message in MIME format. ------=_NextPart_000_00C8_01C302AB.A47B19E0 Content-Type: text/plain; charset="iso-8859-1" Content-Transfer-Encoding: quoted-printable Na minha simploriamente c=E2ndida forma de ver o assunto, se um n=FAmero = inteiro > 1 n=E3o =E9 primo ele =E9 composto, e se n=E3o =E9 composto = ele =E9 primo. Logo n=E3o h=E1 o que demonstrar aqui. Ou (parafraseando o Caetano Veloso) n=E3o? JF ----- Original Message -----=20 From: Frederico Reis Marques de Brito=20 Sent: Monday, April 14, 2003 1:09 PM Subject: [obm-l] Re: [obm-l] Demonstra=E7=E3o por indu=E7=E3o Ainda que fosse, restaria demonstrar que todo n=FAmero inteiro > 1 ou = =E9 primo=20 ou composto.=20 [...] FREDERICO. >From: "Jose Francisco Guimaraes Costa" >Subject: [obm-l] Demonstra=E7=E3o por indu=E7=E3o >Date: Sun, 13 Apr 2003 20:01:30 -0300 > >Isto n=E3o =E9 a defini=E7=E3o de n=FAmero composto, e portanto n=E3o = demonstr=E1vel? > >JF > >----- Original Message ----- >From: "Henrique Patr=EDcio Sant'Anna Branco" = >Sent: Sunday, April 13, 2003 5:19 PM >Subject: [obm-l] Demonstra=E7=E3o por indu=E7=E3o > > > Como eu demonstro, pelo principio da indu=E7=E3o finita, o = seguinte: > > > > Para todo n=FAmero natural "n", "n" =E9 primo ou produto de = primos. > > > > Henrique. ------=_NextPart_000_00C8_01C302AB.A47B19E0 Content-Type: text/html; charset="iso-8859-1" Content-Transfer-Encoding: quoted-printable
Na minha simploriamente c=E2ndida forma = de ver o=20 assunto, se um n=FAmero inteiro > 1 n=E3o =E9 primo ele =E9 composto, = e se n=E3o =E9=20 composto ele =E9 primo. Logo n=E3o h=E1 o que demonstrar = aqui.
 
Ou (parafraseando o Caetano Veloso)=20 n=E3o?
 
JF
----- Original Message -----
From:=20 Frederico=20 Reis Marques de Brito
Sent: Monday, April 14, 2003 = 1:09=20 PM
Subject: [obm-l] Re: [obm-l] = Demonstra=E7=E3o=20 por indu=E7=E3o

Ainda que fosse, restaria demonstrar que todo n=FAmero inteiro = > 1 =20 ou =E9 primo
ou composto.
 
[...]
 
FREDERICO.

>From: "Jose Francisco Guimaraes Costa" = <jfgcosta@unisys.com.br>
= >Subject:=20 [obm-l] Demonstra=E7=E3o por indu=E7=E3o
>Date: Sun, 13 Apr 2003 = 20:01:30=20 -0300
>
>Isto n=E3o =E9 a defini=E7=E3o de n=FAmero = composto, e portanto n=E3o=20 demonstr=E1vel?
>
>JF
>
>----- Original = Message=20 -----
>From: "Henrique Patr=EDcio Sant'Anna Branco" <hpsbranco@superig.com.br>=
>Sent:=20 Sunday, April 13, 2003 5:19 PM
>Subject: [obm-l] = Demonstra=E7=E3o por=20 indu=E7=E3o
>
> > Como eu demonstro, pelo principio da = indu=E7=E3o=20 finita, o seguinte:
> >
> > Para todo n=FAmero = natural "n",=20 "n" =E9 primo ou produto de primos.
> >
> >=20 Henrique.
------=_NextPart_000_00C8_01C302AB.A47B19E0-- ========================================================================= Instruções para entrar na lista, sair da lista e usar a lista em http://www.mat.puc-rio.br/~nicolau/olimp/obm-l.html O administrador desta lista é ========================================================================= From owner-obm-l@sucuri.mat.puc-rio.br Mon Apr 14 18:31:06 2003 Return-Path: Received: (from majordom@localhost) by sucuri.mat.puc-rio.br (8.9.3/8.9.3) id SAA11051 for obm-l-MTTP; Mon, 14 Apr 2003 18:28:17 -0300 Received: from traven9.uol.com.br (traven9.uol.com.br [200.221.29.35]) by sucuri.mat.puc-rio.br (8.9.3/8.9.3) with ESMTP id SAA11025 for ; Mon, 14 Apr 2003 18:28:06 -0300 Received: from gauss ([200.158.97.49]) by traven9.uol.com.br (8.9.1/8.9.1) with SMTP id SAA08101 for ; Mon, 14 Apr 2003 18:25:33 -0300 (BRT) Message-ID: <002701c302cd$0ede0b50$31619ec8@gauss> From: "Domingos Jr." To: Subject: [obm-l] automorfismo Date: Mon, 14 Apr 2003 18:29:22 -0300 MIME-Version: 1.0 Content-Type: text/plain; charset="iso-8859-1" Content-Transfer-Encoding: 8bit X-Priority: 3 X-MSMail-Priority: Normal X-Mailer: Microsoft Outlook Express 6.00.2800.1106 X-MimeOLE: Produced By Microsoft MimeOLE V6.00.2800.1106 Sender: owner-obm-l@sucuri.mat.puc-rio.br Precedence: bulk Reply-To: obm-l@mat.puc-rio.br Seja f um automorfismo em um corpo E, se f^4 = 1, e f(a) + f³(a) = a + f²(a) para todo a pertencendo a E, mostre que f²= 1. Vejam o que eu fiz.... Se f != 1 existe um a tq. f(a) = b != a b + f²(b) = a + f(b) se f²(b) = b, f(f(b)) = b, logo f(b) = a pois f é um automorfismo logo b + b = a + a dá pra ver que isso tá errado... prop. distributiva a(b + b) = ab + ab b(a + a) = ba + ba prop. comutativa ab + ab = ba + ba logo a(b + b) = b(a + a), como a + a = b + b != 0 a = b (lei do cancelamento) absurdo... temos então que se f != 1, f² != 1 É isso mesmo? Temos que provar que na verdade f é a identidade??? [ ]'s ========================================================================= Instruções para entrar na lista, sair da lista e usar a lista em http://www.mat.puc-rio.br/~nicolau/olimp/obm-l.html O administrador desta lista é ========================================================================= From owner-obm-l@sucuri.mat.puc-rio.br Mon Apr 14 21:19:57 2003 Return-Path: Received: (from majordom@localhost) by sucuri.mat.puc-rio.br (8.9.3/8.9.3) id VAA14108 for obm-l-MTTP; Mon, 14 Apr 2003 21:16:26 -0300 Received: from ivoti.terra.com.br (ivoti.terra.com.br [200.176.3.20]) by sucuri.mat.puc-rio.br (8.9.3/8.9.3) with ESMTP id VAA14104 for ; Mon, 14 Apr 2003 21:16:23 -0300 Received: from gunga.terra.com.br (gunga.terra.com.br [200.176.3.45]) by ivoti.terra.com.br (Postfix) with ESMTP id 08373422D96 for ; Mon, 14 Apr 2003 16:19:41 -0300 (BRT) Received: from niski.com (unknown [200.148.196.81]) (authenticated user fniski) by gunga.terra.com.br (Postfix) with ESMTP id 7F198145A20 for ; Mon, 14 Apr 2003 16:19:38 -0300 (BRT) Message-ID: <3E9B09C8.6050802@niski.com> Date: Mon, 14 Apr 2003 12:19:36 -0700 From: niski User-Agent: Mozilla/5.0 (Windows; U; Windows NT 5.1; en-US; rv:1.0.2) Gecko/20030208 Netscape/7.02 X-Accept-Language: en-us, en MIME-Version: 1.0 To: obm-l@mat.puc-rio.br Subject: [obm-l] Sobre os livros didaticos Content-Type: text/plain; charset=ISO-8859-1; format=flowed Content-Transfer-Encoding: 8bit Sender: owner-obm-l@sucuri.mat.puc-rio.br Precedence: bulk Reply-To: obm-l@mat.puc-rio.br Olá colegas! Agora que acabo de entrar no mundo universitario e tomei conhecimento do jeitão do material didático que as vezes é adotado..gostaria de ouvir opinioes dos colegas mais experientes. Bom, os livros que adquiri (Thomas Calculus e Anton&Rorres Linear Algebra with applications) parecem razoaveis quanto a teoria e o didatismo das explicacoes, porem desconfio que o livro é feito apenas 50% para o aluno, pois a parte de respostas de exercicios realmente deixa a desejar. A maioria dos livros do tipo "americano" que vi tem a palhacada de dar apenas as respostas dos exercicios pares ou impares, isso de fato é um estimulo ao professor incopetente que tendo em maos o manual exclusivo para o professor possa elaborar listas de exercicios e provas sem se dar ao trabalho de procurar os mesmos em outros livros e trabalhos. É lamentavel, mas quem procurar pode encontrar por ai os "Student solutions manual to accompany XXXXX" ou seja pagando mais um pouquinho voce compra outro livrinho e tem acesso as respostas. Estão fazendo do ensino uma feira?! Um livro que se diz didatico tem que dar total apoio ao aluno e não ao professor. Defendo que todo livro didatico deve ter no minimo todas as respostas impressas e no caso ideal todas as solucoes. Alguem poderia me dar um motivo para que não seja assim? Em livros do pré primário eu concordo que as respostas devem ser suprimidas pois todos nós qdo criancas, fomos tentados a apenas copiar as resolucoes sem pensar no que estamos fazendo para poder ir brincar de boneco mais cedo. Mas quando chegamos a universidade é natural supor que o aluno sabe que pouco adiantará copiar as respostas pois isso nao o ajudará na hora de resolver as provas (supondo que o professor procure exercicios e problemas de outras referencias, obviamente). Se o problema for monetario, por motivos de impressao e etc, que publiquem as respostas na internet ou coisa do tipo. Lamentavel tb que livros de excelente nível como o do Guidorrizi nao contenha todas as respostas impressas no fim do livro e muitas respostas contem apenas uma dica e etc. É razoavel supor que alguem já com conhecimento matematico solido se satisfaça apenas com as respostas impares pois é provavel que alguem com tal conhecimento nem precise conferir as respostas por isso essa pessoa pode classificar o livro como excelente, perfeito e maravilhoso, mas o novato (como eu) precisa de um suporte maior para ter total confianca no que esta escrevendo, o livro deve mostrar o caminho correto para o aluno e nao o deixar na duvida pensando se o que ele fez esta correto ou nao. Segue um exemplo banal que me deixou um pouco frustrado. "Se A é uma matriz quadrada e n é um inteiro positivo, é verdade que (A^n)^T = (A^T)^n ?" Eu resolvi assim. Seja A^n = A(A^(n-1)) (A^n)^T = (A(A^(n-1)))^T (A^n)^T = (A^(n-1))^T (A)^T (A^n)^T = ((A^T)^(n-1)) ((A)^T)^1 (A^n)^T = ((A^T)^(n)) Portanto para mim a resposta é sim, e a justificativa esta ai em cima. O livro responde apenas "sim" Então, neste caso creio que nao fiz nada de errado, mas nao tenho certeza!!! Afinal mexo com matrizes a apenas 2 ou 3 anos, certamente alguem que já mexe com isso há 10, 20 anos nao precise de nada mais do que um simples "sim" como resposta. É bem possivel que em um outro problema eu chegue na resolucao correta porem cometendo um erro e nunca perceba isto. Isso deixa o estudante novato desconfiado..afinal ele é _novato_! Não concordam que seria melhor, se todos os livros de alguma maneira disponibilizassem as resolucoes de todos os problemas?! Niski www.linux.ime.usp.br/~niski -- [about him:] It is rare to find learned men who are clean, do not stink and have a sense of humour. -Gottfried Whilhem Leibniz ========================================================================= Instruções para entrar na lista, sair da lista e usar a lista em http://www.mat.puc-rio.br/~nicolau/olimp/obm-l.html O administrador desta lista é ========================================================================= From owner-obm-l@sucuri.mat.puc-rio.br Mon Apr 14 23:30:25 2003 Return-Path: Received: (from majordom@localhost) by sucuri.mat.puc-rio.br (8.9.3/8.9.3) id XAA16260 for obm-l-MTTP; Mon, 14 Apr 2003 23:27:28 -0300 Received: from hotmail.com (f59.sea1.hotmail.com [207.68.163.59]) by sucuri.mat.puc-rio.br (8.9.3/8.9.3) with ESMTP id XAA16257 for ; Mon, 14 Apr 2003 23:27:25 -0300 Received: from mail pickup service by hotmail.com with Microsoft SMTPSVC; Mon, 14 Apr 2003 19:26:52 -0700 Received: from 198.81.8.3 by sea1fd.sea1.hotmail.msn.com with HTTP; Tue, 15 Apr 2003 02:26:52 GMT X-Originating-IP: [198.81.8.3] X-Originating-Email: [fredericor@hotmail.com] From: "Frederico Reis Marques de Brito" To: obm-l@mat.puc-rio.br Subject: [obm-l] =?iso-8859-1?B?UmU6IFtvYm0tbF0gRGVtb25zdHJh5+NvIHBvciBpbmR15+Nv?= Date: Mon, 14 Apr 2003 23:26:52 -0300 Mime-Version: 1.0 Content-Type: text/plain; charset=iso-8859-1; format=flowed Message-ID: X-OriginalArrivalTime: 15 Apr 2003 02:26:52.0530 (UTC) FILETIME=[79CE9D20:01C302F6] Sender: owner-obm-l@sucuri.mat.puc-rio.br Precedence: bulk Reply-To: obm-l@mat.puc-rio.br É necesssário acompanhar a sequência dos email's para entender o que escrevi. Já lecionei a disciplina teoria dos números pelo menos umas 4 vezes e sei perfeitamente bem que todo inteiro maior que 1 ou é primo ou composto, como já foi dito, por definição aliás. Entretanto, a questão inicial era se todo inteiro >1 ou é primo ou se decompõe como produto de primos. Carece de demonstração o fato: Todo número composto pode ser completamente fatorado em primos. Agora, se você define número composto como aquele que se fatora em primos, como alguém argumentou, então deve demonstrar que todo inteiro > 1 ou é primo ou composto. Olha, e francamente, acho que tem muita gente confundindo conhecimento com arrogância e abusando de ironias que mais atrapalham que qualquer outra coisa. Frederico. >From: "Jose Francisco Guimaraes Costa" >Reply-To: obm-l@mat.puc-rio.br >To: >Subject: [obm-l] Demonstração por indução >Date: Mon, 14 Apr 2003 17:31:11 -0300 > >Na minha simploriamente cândida forma de ver o assunto, se um número >inteiro > 1 não é primo ele é composto, e se não é composto ele é primo. >Logo não há o que demonstrar aqui. > >Ou (parafraseando o Caetano Veloso) não? > >JF > ----- Original Message ----- > From: Frederico Reis Marques de Brito > Sent: Monday, April 14, 2003 1:09 PM > Subject: [obm-l] Re: [obm-l] Demonstração por indução > > > Ainda que fosse, restaria demonstrar que todo número inteiro > 1 ou é >primo > ou composto. > > [...] > > FREDERICO. > > >From: "Jose Francisco Guimaraes Costa" > >Subject: [obm-l] Demonstração por indução > >Date: Sun, 13 Apr 2003 20:01:30 -0300 > > > >Isto não é a definição de número composto, e portanto não demonstrável? > > > >JF > > > >----- Original Message ----- > >From: "Henrique Patrício Sant'Anna Branco" > >Sent: Sunday, April 13, 2003 5:19 PM > >Subject: [obm-l] Demonstração por indução > > > > > Como eu demonstro, pelo principio da indução finita, o seguinte: > > > > > > Para todo número natural "n", "n" é primo ou produto de primos. > > > > > > Henrique. _________________________________________________________________ MSN Hotmail, o maior webmail do Brasil. http://www.hotmail.com ========================================================================= Instruções para entrar na lista, sair da lista e usar a lista em http://www.mat.puc-rio.br/~nicolau/olimp/obm-l.html O administrador desta lista é ========================================================================= From owner-obm-l@sucuri.mat.puc-rio.br Mon Apr 14 23:38:20 2003 Return-Path: Received: (from majordom@localhost) by sucuri.mat.puc-rio.br (8.9.3/8.9.3) id XAA16378 for obm-l-MTTP; Mon, 14 Apr 2003 23:35:38 -0300 Received: from fnn.net ([200.175.38.9]) by sucuri.mat.puc-rio.br (8.9.3/8.9.3) with SMTP id XAA16373 for ; Mon, 14 Apr 2003 23:35:34 -0300 Received: (qmail 22187 invoked from network); 15 Apr 2003 02:21:02 -0000 Received: from unknown (HELO windows98) (200.175.39.47) by fnn.net with SMTP; 15 Apr 2003 02:21:02 -0000 Message-ID: <003b01c30301$1c984e20$9a75fea9@windows98> From: "Daniel Pini" To: Subject: [obm-l] + duvidas Date: Tue, 15 Apr 2003 00:42:59 -0300 MIME-Version: 1.0 Content-Type: multipart/alternative; boundary="----=_NextPart_000_0038_01C302E7.F6833240" X-Priority: 3 X-MSMail-Priority: Normal X-Mailer: Microsoft Outlook Express 5.00.2615.200 X-MimeOLE: Produced By Microsoft MimeOLE V5.00.2615.200 Sender: owner-obm-l@sucuri.mat.puc-rio.br Precedence: bulk Reply-To: obm-l@mat.puc-rio.br This is a multi-part message in MIME format. ------=_NextPart_000_0038_01C302E7.F6833240 Content-Type: text/plain; charset="iso-8859-1" Content-Transfer-Encoding: quoted-printable Para variar um pouco do assunto dos meus ultimos e-mails ( = fatora=E7=E3o), vou postar duvidas de problemas de artimetica. O n=FAmero de maneiras de se escolher tr=EAs n=EDmeros distintos no = conjunto {1,2,3,...,100} de modo que a soma desses tr=EAs n=FAmeros = seja igual a 100 e o menor deles seja 25 =E9? R:12 O n=FAmero de maneiras de se escolher tr=EAs n=EDmeros distintos no = conjunto {1,2,3,...,100} de modo que a soma desses tr=EAs n=FAmeros = seja igual a 100 =E9? R:784 Se x=B2+y=B2=3D9797 onde x e y s=E3o inteiros positivos tais que x =E9 = maior que y, existem exatamente dois pares ordenados de inteiros (x,y) = que satisfazem tal equa=E7=E3o. A soma das coordenadas deste dois pares = =E9? R:260 Se (5=B2+9=B2)(12=B2+17=B2) for escrito sob a forma a=B2+b=B2 ent=E3o = a+b =E9 igual a? R:236 ou 286 (Obs: eu recebi uma dica p/ usar n=FAmeros complexos, mas = queria saber se existe algum outro modo de se resolver este problema?) ------=_NextPart_000_0038_01C302E7.F6833240 Content-Type: text/html; charset="iso-8859-1" Content-Transfer-Encoding: quoted-printable
Para variar um pouco do assunto dos = meus ultimos=20 e-mails ( fatora=E7=E3o), vou postar duvidas de problemas de=20 artimetica.
O n=FAmero de maneiras de se escolher tr=EAs = n=EDmeros distintos=20 no conjunto {1,2,3,...,100} de modo que a soma desses tr=EAs  = n=FAmeros seja=20 igual a 100 e o menor deles seja 25 =E9? R:12
 
O n=FAmero de maneiras de se = escolher tr=EAs=20 n=EDmeros distintos no conjunto {1,2,3,...,100} de modo que a soma = desses=20 tr=EAs  n=FAmeros seja igual a 100 =E9?
R:784
 
Se x=B2+y=B2=3D9797 onde x e y s=E3o inteiros = positivos tais que x=20 =E9 maior que y, existem exatamente dois pares ordenados de inteiros = (x,y) que=20 satisfazem tal equa=E7=E3o. A soma das coordenadas deste dois pares =E9? = R:260
 
Se (5=B2+9=B2)(12=B2+17=B2) for escrito sob a = forma a=B2+b=B2 ent=E3o=20 a+b =E9 igual a?
R:236 ou 286 (Obs: eu recebi uma dica p/ usar = n=FAmeros=20 complexos, mas queria saber se existe algum outro modo de se resolver = este=20 problema?)
------=_NextPart_000_0038_01C302E7.F6833240-- ========================================================================= Instruções para entrar na lista, sair da lista e usar a lista em http://www.mat.puc-rio.br/~nicolau/olimp/obm-l.html O administrador desta lista é ========================================================================= From owner-obm-l@sucuri.mat.puc-rio.br Mon Apr 14 23:38:52 2003 Return-Path: Received: (from majordom@localhost) by sucuri.mat.puc-rio.br (8.9.3/8.9.3) id XAA16408 for obm-l-MTTP; Mon, 14 Apr 2003 23:36:17 -0300 Received: from hotmail.com (f44.sea1.hotmail.com [207.68.163.44]) by sucuri.mat.puc-rio.br (8.9.3/8.9.3) with ESMTP id XAA16403 for ; Mon, 14 Apr 2003 23:36:13 -0300 Received: from mail pickup service by hotmail.com with Microsoft SMTPSVC; Mon, 14 Apr 2003 19:35:41 -0700 Received: from 198.81.8.3 by sea1fd.sea1.hotmail.msn.com with HTTP; Tue, 15 Apr 2003 02:35:41 GMT X-Originating-IP: [198.81.8.3] X-Originating-Email: [fredericor@hotmail.com] From: "Frederico Reis Marques de Brito" To: obm-l@mat.puc-rio.br Subject: Re: [obm-l] Sobre os livros didaticos Date: Mon, 14 Apr 2003 23:35:41 -0300 Mime-Version: 1.0 Content-Type: text/plain; charset=iso-8859-1; format=flowed Message-ID: X-OriginalArrivalTime: 15 Apr 2003 02:35:41.0404 (UTC) FILETIME=[B50A61C0:01C302F7] Sender: owner-obm-l@sucuri.mat.puc-rio.br Precedence: bulk Reply-To: obm-l@mat.puc-rio.br Caro Niski, infelizmenmte boa parte dos alunos dos cursos básicos, tais como Cálculo e Geometrioa Analítica não são, mas agem como crianças, o que creio não seja seu caso. Ademais, não creio ser a função do livro didático resolver os exercícios que propõe. Para isso, existem, professor, monitores e colegas. Também ´há que se estimular a auto-confiança dos alunos. Quanto ao exemplo que vc citou, ou você usou indução, embora não tenha deixado isso explícito, ou você usou a tese, e nesse caso, a demonstração está errada, afinal $ ( A^{n-1})T= (AT)^{n-1}$ é equivalente ao que vc deseja demonstrar inicialmente. Frederico. >From: niski >Reply-To: obm-l@mat.puc-rio.br >To: obm-l@mat.puc-rio.br >Subject: [obm-l] Sobre os livros didaticos >Date: Mon, 14 Apr 2003 12:19:36 -0700 > >Olá colegas! >Agora que acabo de entrar no mundo universitario e tomei conhecimento do >jeitão do material didático que as vezes é adotado..gostaria de ouvir >opinioes dos colegas mais experientes. >Bom, os livros que adquiri (Thomas Calculus e Anton&Rorres Linear Algebra >with applications) parecem razoaveis quanto a teoria e o didatismo das >explicacoes, porem desconfio que o livro é feito apenas 50% para o aluno, >pois a parte de respostas de exercicios realmente deixa a desejar. A >maioria dos livros do tipo "americano" que vi tem a palhacada de dar apenas >as respostas dos exercicios pares ou impares, isso de fato é um estimulo ao >professor incopetente que tendo em maos o manual exclusivo para o professor >possa elaborar listas de exercicios e provas sem se dar ao trabalho de >procurar os mesmos em outros livros e trabalhos. É lamentavel, mas quem >procurar pode encontrar por ai os "Student solutions manual to accompany >XXXXX" ou seja pagando mais um pouquinho voce compra outro livrinho e tem >acesso as respostas. Estão fazendo do ensino uma feira?! Um livro que se >diz didatico tem que dar total apoio ao aluno e não ao professor. Defendo >que todo livro didatico deve ter no minimo todas as respostas impressas e >no caso ideal todas as solucoes. Alguem poderia me dar um motivo para que >não seja assim? >Em livros do pré primário eu concordo que as respostas devem ser suprimidas >pois todos nós qdo criancas, fomos tentados a apenas copiar as resolucoes >sem pensar no que estamos fazendo para poder ir brincar de boneco mais >cedo. Mas quando chegamos a universidade é natural supor que o aluno sabe >que pouco adiantará copiar as respostas pois isso nao o ajudará na hora de >resolver as provas (supondo que o professor procure exercicios e problemas >de outras referencias, obviamente). Se o problema for monetario, por >motivos de impressao e etc, que publiquem as respostas na internet ou coisa >do tipo. >Lamentavel tb que livros de excelente nível como o do Guidorrizi nao >contenha todas as respostas impressas no fim do livro e muitas respostas >contem apenas uma dica e etc. É razoavel supor que alguem já com >conhecimento matematico solido se satisfaça apenas com as respostas impares >pois é provavel que alguem com tal conhecimento nem precise conferir as >respostas por isso essa pessoa pode classificar o livro como excelente, >perfeito e maravilhoso, mas o novato (como eu) precisa de um suporte maior >para ter total confianca no que esta escrevendo, o livro deve mostrar o >caminho correto para o aluno e nao o deixar na duvida pensando se o que ele >fez esta correto ou nao. >Segue um exemplo banal que me deixou um pouco frustrado. > >"Se A é uma matriz quadrada e n é um inteiro positivo, é verdade que >(A^n)^T = (A^T)^n ?" > >Eu resolvi assim. >Seja A^n = A(A^(n-1)) >(A^n)^T = (A(A^(n-1)))^T >(A^n)^T = (A^(n-1))^T (A)^T >(A^n)^T = ((A^T)^(n-1)) ((A)^T)^1 >(A^n)^T = ((A^T)^(n)) >Portanto para mim a resposta é sim, e a justificativa esta ai em cima. >O livro responde apenas >"sim" > >Então, neste caso creio que nao fiz nada de errado, mas nao tenho >certeza!!! Afinal mexo com matrizes a apenas 2 ou 3 anos, certamente alguem >que já mexe com isso há 10, 20 anos nao precise de nada mais do que um >simples "sim" como resposta. É bem possivel que em um outro problema eu >chegue na resolucao correta porem cometendo um erro e nunca perceba isto. >Isso deixa o estudante novato desconfiado..afinal ele é _novato_! Não >concordam que seria melhor, se todos os livros de alguma maneira >disponibilizassem as resolucoes de todos os problemas?! > > >Niski >www.linux.ime.usp.br/~niski >-- >[about him:] > It is rare to find learned men who are clean, do not stink and have a >sense of humour. >-Gottfried Whilhem Leibniz > >========================================================================= >Instruções para entrar na lista, sair da lista e usar a lista em >http://www.mat.puc-rio.br/~nicolau/olimp/obm-l.html >O administrador desta lista é >========================================================================= _________________________________________________________________ MSN Messenger: converse com os seus amigos online. http://messenger.msn.com.br ========================================================================= Instruções para entrar na lista, sair da lista e usar a lista em http://www.mat.puc-rio.br/~nicolau/olimp/obm-l.html O administrador desta lista é ========================================================================= From owner-obm-l@sucuri.mat.puc-rio.br Tue Apr 15 03:06:32 2003 Return-Path: Received: (from majordom@localhost) by sucuri.mat.puc-rio.br (8.9.3/8.9.3) id DAA19540 for obm-l-MTTP; Tue, 15 Apr 2003 03:03:32 -0300 Received: from web13707.mail.yahoo.com (web13707.mail.yahoo.com [216.136.175.140]) by sucuri.mat.puc-rio.br (8.9.3/8.9.3) with SMTP id DAA19536 for ; Tue, 15 Apr 2003 03:03:28 -0300 Message-ID: <20030415060257.31650.qmail@web13707.mail.yahoo.com> Received: from [200.213.88.166] by web13707.mail.yahoo.com via HTTP; Tue, 15 Apr 2003 03:02:57 ART Date: Tue, 15 Apr 2003 03:02:57 -0300 (ART) From: =?iso-8859-1?q?pichurin?= Subject: [obm-l] REGRA DA CADEIA To: obm-l@mat.puc-rio.br MIME-Version: 1.0 Content-Type: text/plain; charset=iso-8859-1 Content-Transfer-Encoding: 8bit Sender: owner-obm-l@sucuri.mat.puc-rio.br Precedence: bulk Reply-To: obm-l@mat.puc-rio.br qual é a demonstração da regra da cadeia? Essa regra pode serr aplicada para expoentes(x^f(x))?Como? _______________________________________________________________________ Yahoo! Mail O melhor e-mail gratuito da internet: 6MB de espaço, antivírus, acesso POP3, filtro contra spam. http://br.mail.yahoo.com/ ========================================================================= Instruções para entrar na lista, sair da lista e usar a lista em http://www.mat.puc-rio.br/~nicolau/olimp/obm-l.html O administrador desta lista é ========================================================================= From owner-obm-l@sucuri.mat.puc-rio.br Tue Apr 15 09:23:05 2003 Return-Path: Received: (from majordom@localhost) by sucuri.mat.puc-rio.br (8.9.3/8.9.3) id JAA23420 for obm-l-MTTP; Tue, 15 Apr 2003 09:19:31 -0300 Received: from aacpdlotus.net.ms.gov.br (ns1.ms.gov.br [200.181.116.3]) by sucuri.mat.puc-rio.br (8.9.3/8.9.3) with ESMTP id JAA23415 for ; Tue, 15 Apr 2003 09:19:27 -0300 From: JoaoCarlos_Junior@net.ms.gov.br Subject: [obm-l] Re: + =?iso-8859-1?Q?D=FAvidas?= To: obm-l@mat.puc-rio.br X-Mailer: Lotus Notes Release 5.0.9a January 7, 2002 Message-ID: Date: Tue, 15 Apr 2003 08:23:01 -0400 X-MIMETrack: Serialize by Router on aacpdlotus/NETMS(Release 5.0.9a |January 7, 2002) at 04/15/2003 08:23:32 AM MIME-Version: 1.0 Content-type: text/plain; charset=iso-8859-1 Content-Transfer-Encoding: 8bit X-MIME-Autoconverted: from quoted-printable to 8bit by sucuri.mat.puc-rio.br id JAA23416 Sender: owner-obm-l@sucuri.mat.puc-rio.br Precedence: bulk Reply-To: obm-l@mat.puc-rio.br Querido Daniel, Desde jah, peço que me corrijam, caso haja erro em minha solução. 1) O numero de maneiras de se escolher tres numeros distintos no conjunto {1,2,3,...,100} de modo que a soma desses três números seja igual a 100 e o menor deles seja 25 eh? Tentativa de resolucao: Sejam 25=x < y < z os numeros distintos no conjunto dado. x+y+z=100 entao y+z=75 entao 26<=y<75?26=49. Na PA de razão 1, a1=26, an=49, os termos do meio sao 37 e 38. Como de 26 a 37, hah 12 termos, a resposta eh entao 12. ========================================================================= Instruções para entrar na lista, sair da lista e usar a lista em http://www.mat.puc-rio.br/~nicolau/olimp/obm-l.html O administrador desta lista é ========================================================================= From owner-obm-l@sucuri.mat.puc-rio.br Tue Apr 15 09:34:02 2003 Return-Path: Received: (from majordom@localhost) by sucuri.mat.puc-rio.br (8.9.3/8.9.3) id JAA23685 for obm-l-MTTP; Tue, 15 Apr 2003 09:31:12 -0300 Received: from aacpdlotus.net.ms.gov.br (ns1.ms.gov.br [200.181.116.3]) by sucuri.mat.puc-rio.br (8.9.3/8.9.3) with ESMTP id JAA23681 for ; Tue, 15 Apr 2003 09:31:08 -0300 From: JoaoCarlos_Junior@net.ms.gov.br Importance: High X-Priority: 1 (High) Subject: [obm-l] Re: + =?iso-8859-1?Q?D=FAvidas?= To: obm-l@mat.puc-rio.br X-Mailer: Lotus Notes Release 5.0.9a January 7, 2002 Message-ID: Date: Tue, 15 Apr 2003 08:34:42 -0400 X-MIMETrack: Serialize by Router on aacpdlotus/NETMS(Release 5.0.9a |January 7, 2002) at 04/15/2003 08:35:13 AM MIME-Version: 1.0 Content-type: text/plain; charset=iso-8859-1 Content-Transfer-Encoding: 8bit X-MIME-Autoconverted: from quoted-printable to 8bit by sucuri.mat.puc-rio.br id JAA23682 Sender: owner-obm-l@sucuri.mat.puc-rio.br Precedence: bulk Reply-To: obm-l@mat.puc-rio.br Querido Daniel, Desde jah, peço que me corrijam, caso haja erro em minha solução. 1) O numero de maneiras de se escolher tres numeros distintos no conjunto {1,2,3,...,100} de modo que a soma desses três números seja igual a 100 e o menor deles seja 25 eh? Tentativa de resolucao: Sejam 25=x < y < z os numeros distintos no conjunto dado. x+y+z=100 entao y+z=75 entao 26<=y ========================================================================= From owner-obm-l@sucuri.mat.puc-rio.br Tue Apr 15 11:04:41 2003 Return-Path: Received: (from majordom@localhost) by sucuri.mat.puc-rio.br (8.9.3/8.9.3) id LAA26039 for obm-l-MTTP; Tue, 15 Apr 2003 11:01:29 -0300 Received: from traven9.uol.com.br (traven9.uol.com.br [200.221.29.35]) by sucuri.mat.puc-rio.br (8.9.3/8.9.3) with ESMTP id LAA26035 for ; Tue, 15 Apr 2003 11:01:24 -0300 Received: from gauss ([200.158.96.222]) by traven9.uol.com.br (8.9.1/8.9.1) with SMTP id LAA16347 for ; Tue, 15 Apr 2003 11:00:51 -0300 (BRT) Message-ID: <006301c30358$0c86c4b0$7d07fea9@gauss> From: "Domingos Jr." To: References: <20030415060257.31650.qmail@web13707.mail.yahoo.com> Subject: Re: [obm-l] REGRA DA CADEIA Date: Tue, 15 Apr 2003 11:05:19 -0300 MIME-Version: 1.0 Content-Type: text/plain; charset="iso-8859-1" Content-Transfer-Encoding: 8bit X-Priority: 3 X-MSMail-Priority: Normal X-Mailer: Microsoft Outlook Express 6.00.2800.1106 X-MimeOLE: Produced By Microsoft MimeOLE V6.00.2800.1106 Sender: owner-obm-l@sucuri.mat.puc-rio.br Precedence: bulk Reply-To: obm-l@mat.puc-rio.br > qual é a demonstração da regra da cadeia? > Essa regra pode serr aplicada para expoentes(x^f(x))?Como? Depende, a regra da cadeia de funções com 1 variável sai direto da definição de derivada através de limites, já foram colocadas algumas demonstrações diversas aqui nessa lista mesmo. Funções de R^n -> R^m já são mais complicadas e envolvem a matriz Jacobiana... (eu nem conheço as demonstrações, se alguém puder indicar um material interessante...) as regras (sem demonstração) você encontra aqui: http://mathworld.wolfram.com/ChainRule.html PARTE 2 entenda x^f(x) como (e^lnx)^f(x) = e^(f(x).lnx) sendo assim, se g(x) = x^f(x) g(x) = e^(f(x).lnx) dg/dx = e^(f(x).lnx).[f(x).lnx]' ... [ ]'s ========================================================================= Instruções para entrar na lista, sair da lista e usar a lista em http://www.mat.puc-rio.br/~nicolau/olimp/obm-l.html O administrador desta lista é ========================================================================= From owner-obm-l@sucuri.mat.puc-rio.br Tue Apr 15 12:06:30 2003 Return-Path: Received: (from majordom@localhost) by sucuri.mat.puc-rio.br (8.9.3/8.9.3) id MAA27208 for obm-l-MTTP; Tue, 15 Apr 2003 12:02:56 -0300 Received: from traven9.uol.com.br (traven9.uol.com.br [200.221.29.35]) by sucuri.mat.puc-rio.br (8.9.3/8.9.3) with ESMTP id MAA27204 for ; Tue, 15 Apr 2003 12:02:45 -0300 Received: from u ([200.207.152.104]) by traven9.uol.com.br (8.9.1/8.9.1) with SMTP id LAA06561 for ; Tue, 15 Apr 2003 11:16:12 -0300 (BRT) Message-ID: <001101c3035a$d317d400$6898cfc8@u> From: "Afemano" To: References: <20030415060257.31650.qmail@web13707.mail.yahoo.com> Subject: Re: [obm-l] REGRA DA CADEIA Date: Tue, 15 Apr 2003 11:25:11 -0300 MIME-Version: 1.0 Content-Type: text/plain; charset="iso-8859-1" Content-Transfer-Encoding: 8bit X-Priority: 3 X-MSMail-Priority: Normal X-Mailer: Microsoft Outlook Express 6.00.2600.0000 X-MimeOLE: Produced By Microsoft MimeOLE V6.00.2600.0000 Sender: owner-obm-l@sucuri.mat.puc-rio.br Precedence: bulk Reply-To: obm-l@mat.puc-rio.br A demonstração deixo pros mais experientes da lista heheheheh mas não é dificil.. Para exponecial a regra da cadeia se aplica sim... seja f(x)=x^g(x) entao f'(x) = x^g(x) * lnx * g'(x) Abraços Gabriel ----- Original Message ----- From: "pichurin" To: Sent: Tuesday, April 15, 2003 3:02 AM Subject: [obm-l] REGRA DA CADEIA > qual é a demonstração da regra da cadeia? > Essa regra pode serr aplicada para expoentes(x^f(x))?Como? > > _______________________________________________________________________ > Yahoo! Mail > O melhor e-mail gratuito da internet: 6MB de espaço, antivírus, acesso POP3, filtro contra spam. > http://br.mail.yahoo.com/ > ========================================================================= > Instruções para entrar na lista, sair da lista e usar a lista em > http://www.mat.puc-rio.br/~nicolau/olimp/obm-l.html > O administrador desta lista é > ========================================================================= ========================================================================= Instruções para entrar na lista, sair da lista e usar a lista em http://www.mat.puc-rio.br/~nicolau/olimp/obm-l.html O administrador desta lista é ========================================================================= From owner-obm-l@sucuri.mat.puc-rio.br Tue Apr 15 12:13:09 2003 Return-Path: Received: (from majordom@localhost) by sucuri.mat.puc-rio.br (8.9.3/8.9.3) id MAA27298 for obm-l-MTTP; Tue, 15 Apr 2003 12:10:14 -0300 Received: from cmsrelay05.mx.net (cmsrelay05.mx.net [165.212.11.2]) by sucuri.mat.puc-rio.br (8.9.3/8.9.3) with SMTP id MAA27293 for ; Tue, 15 Apr 2003 12:10:09 -0300 Received: from cmsapps02.cms.usa.net (HELO localhost) (165.212.11.138) by cmsoutbound.mx.net with SMTP; 15 Apr 2003 15:09:32 -0000 Received: from smtp.postoffice.net [165.212.8.7] by cmsapps02.cms.usa.net (ASMTP/) via mtad (C8.MAIN.2.05) with ESMTP id 169HDoPJE0010M38; Tue, 15 Apr 2003 15:09:30 GMT Received: from 200.181.4.100 [200.181.4.100] by uwdvg007.cms.usa.net (USANET web-mailer CM.0402.5.2B); Tue, 15 Apr 2003 15:09:30 -0000 Date: Tue, 15 Apr 2003 12:09:30 -0300 From: Artur Costa Steiner To: Subject: Re: [[obm-l] REGRA DA CADEIA] X-Mailer: USANET web-mailer (CM.0402.5.2B) Mime-Version: 1.0 Message-ID: <590HDooKj1664S20.1050419370@uwdvg007.cms.usa.net> Content-Type: text/plain; charset=ISO-8859-1 Content-Transfer-Encoding: 8bit X-MIME-Autoconverted: from quoted-printable to 8bit by sucuri.mat.puc-rio.br id MAA27294 Sender: owner-obm-l@sucuri.mat.puc-rio.br Precedence: bulk Reply-To: obm-l@mat.puc-rio.br pichurin wrote: > qual é a demonstração da regra da cadeia? > Essa regra pode ser aplicada para expoentes(x^f(x))?Como? Há alguns dias alguém da lista informou um site onde há uma demonstracao bem interessante, baseada em funções auxiliares.. Infelizmente, nao tenho o site aqui no momento. Mas, no caso, de funções de R em R eu acho que a seguinte demonstracao permite sentir melhor o que está acontecendo: Suponhamos que g seja definida em um intervalo aberto I contendo a, que g seja diferenciavel em a e que f seja diferenciavel em g(a) e definida em um intervalo aberto contendo f(I). Então, h, a funcao composta de f e g , eh diferenciavel em a e h'(a) = f'(g(a)) g(a). A outra demonstracao que eu citei e que estah naquele site tem a vantagem de eliminar o problema de denominadores nulos, mas eu acho que esta que dei permite um melhor "insight". A regra vale tambem para funcoes que estejam em expoentes, desde que as condicoes que citei permanecam validas Demonstracao: suponhamos inicialmente que exista uma vizinhanca V de a, contida em I, na qual g(x)<>g(a) para x<>a. Para todo x<>a em V, temos entao que [h(x)-h(a)]/(x-a) = f[g(x)]-f[g(a)]/(x-a) =f[g(x)]-f[g(a)]/[(g(x)-g(a)].[(g(x)-g(a)]/(x-a) (1). Sendo v a funcao definida no domínio de f para u<>g(a) por v(u) = [f(u)-f(g(a)]/(u-g(a)), segue-se da diferenciabilidae de f em g(a) que lim (u-->g(a)) v(u) = f'(g(a). Da diferenciabilidade de g em a segue-se que g eh continua em a. E como g(x)<>g(a) para x<> a em V, podemos aplicar aquele teorma relativo a limites de composicao de funcoes e concluir que, como v(g(x))= f[g(x)]-f[g(a)]/(g(x-g(a)), entao lim (x-->a)v(g(x))= lim (x-->a) f[g(x)]-f[g(a)]/(g(x)-g(a)) = lim (u-->g(a)) v(u) = f'(g(a)). Por outro lado, da diferenciabilidade de g em a eh imediato que lim (x-->a) [g(x) - g(a)]/(x-a) = g'(a). Considerando-se (1) e a existência dos limites em a das funcoes cujo produto compoe o primeiro membro de (1), concluimos que lim (x-->a) [h(x)-h(a)]/(x-a) = h'(a) existe e iguala-se a f'(g(a)) g'(a). Isto prova a regra para o caso em questao. Mas existe a possibilidade de que g seja um tanto "patologica" e g(x) - g(a) se anule para x<> a em qualquer vizinhanca de a. Neste caso, o procedimento acima nao se aplica, pois teriamos denominador nulo. Observamos porem que, nesta nova sitauacao, a diferenciabilidade de g em acarreta automaticamente que g'(a) = 0 (de outra forma, g-a seria estritamente positiva ou negativa em uma vizinhanca de a, contrariando nossa hipotese). Da diferenciabilidade de f em g(a), segue-se das propriedades das derivadas que existem M>0 e uma vizinhanca U de g(a) contida em f(I) na qual |f(y) - f(g(a)| <= M (y-g(a)). Logo, |f(g(x) - f(g(a)| <= M (g(x) - g(a)) para todo x em I. Para x<> a, temos entao que|f(g(x) - f(g(a)|/(x-a) <= M [(g(x) - g(a)]/(x-a). Como g'(a) = 0, o segundo membro tende a zero quando x-->a e, consequentemente, o mesmo ocorre para o primeiro membro. Logo h'(a) = 0 = f'(g(a). g'(a), o que mostra que a R. da cadeia eh valida tambem para este ultimo caso. No caso que vc citou, temos que h(x)= x^f(x) = e^[f(x) ln(x)] Supomdo-se x>0 e f diferenciavel em x, a aplicacao da r. da cadeia leva a que h'(x) = e^[f(x) ln(x)] d/dx [f(x) ln(x)] , visto que (e^u)' = e^u u'. Logo, h'(x) = x^f(x) [f(x)/x + f'(x) ln(x] Um abraco Artur ========================================================================= Instruções para entrar na lista, sair da lista e usar a lista em http://www.mat.puc-rio.br/~nicolau/olimp/obm-l.html O administrador desta lista é ========================================================================= From owner-obm-l@sucuri.mat.puc-rio.br Tue Apr 15 12:33:29 2003 Return-Path: Received: (from majordom@localhost) by sucuri.mat.puc-rio.br (8.9.3/8.9.3) id MAA27991 for obm-l-MTTP; Tue, 15 Apr 2003 12:30:37 -0300 Received: from cmsrelay02.mx.net (cmsrelay02.mx.net [165.212.11.111]) by sucuri.mat.puc-rio.br (8.9.3/8.9.3) with SMTP id MAA27987 for ; Tue, 15 Apr 2003 12:30:32 -0300 Received: from uadvg130.cms.usa.net (HELO localhost) (165.212.11.130) by cmsoutbound.mx.net with SMTP; 15 Apr 2003 15:29:58 -0000 Received: from smtp.postoffice.net [165.212.8.22] by uadvg130.cms.usa.net (ASMTP/) via mtad (C8.MAIN.2.05) with ESMTP id 763HDoPd40434M30; Tue, 15 Apr 2003 15:29:56 GMT Received: from 200.181.4.100 [200.181.4.100] by uwdvg022.cms.usa.net (USANET web-mailer CM.0402.5.2B); Tue, 15 Apr 2003 15:29:54 -0000 Date: Tue, 15 Apr 2003 12:29:54 -0300 From: Artur Costa Steiner To: Subject: Re: [Re: [obm-l] REGRA DA CADEIA] X-Mailer: USANET web-mailer (CM.0402.5.2B) Mime-Version: 1.0 Message-ID: <689HDoPd30272S22.1050420594@uwdvg022.cms.usa.net> Content-Type: text/plain; charset=ISO-8859-1 Content-Transfer-Encoding: 8bit X-MIME-Autoconverted: from quoted-printable to 8bit by sucuri.mat.puc-rio.br id MAA27988 Sender: owner-obm-l@sucuri.mat.puc-rio.br Precedence: bulk Reply-To: obm-l@mat.puc-rio.br "Domingos Jr." wrote: > > > qual é a demonstração da regra da cadeia? > > Essa regra pode serr aplicada para expoentes(x^f(x))?Como? > > Depende, a regra da cadeia de funções com 1 variável sai direto da definição > de derivada através de limites, já foram colocadas algumas demonstrações > diversas aqui nessa lista mesmo. > Funções de R^n -> R^m já são mais complicadas e envolvem a matriz > Jacobiana... (eu nem conheço as demonstrações, se alguém puder indicar um > material interessante...) Nos livros do Bartle (The Elements of Real Analysis) e do Apostol (Real Analysis) há as demonstracoes da regra da cadeia para funcoes de R^m em R^n. Lembro que para funcoes de dominio em R^n, n>1, o conceito de derivada eh diferente daquele do caso n=1. Se n>1, ao valor da derivada de f em um ponto x de seu dominio nao eh um numero ou um vetor mas si uma funcao linear que aproxima f em uma vizinhanca de x. Dizemos que f de dominio em R^n e valores em R^m eh diferenciavel em x se existir uma funcao linear L tal que, dado qualquer eps>0, existir um d>0, tal que, se x estiver no dominio de f e 0<||x-a||1, uma condicao suficiente (porem nao necessaria) para que f seja diferenciavel em x e que uma de suas derivadas parciais exista em x e todas as outras derivadas parciais existam em uma vizinhanca de x e sejam continuas em x. Logo, a continuidade das derivadas parciais de f e m x eh uma condicao mais forte do que a diferenciabilidade em x. Artur ========================================================================= Instruções para entrar na lista, sair da lista e usar a lista em http://www.mat.puc-rio.br/~nicolau/olimp/obm-l.html O administrador desta lista é ========================================================================= From owner-obm-l@sucuri.mat.puc-rio.br Tue Apr 15 12:43:24 2003 Return-Path: Received: (from majordom@localhost) by sucuri.mat.puc-rio.br (8.9.3/8.9.3) id MAA28292 for obm-l-MTTP; Tue, 15 Apr 2003 12:40:41 -0300 Received: (from nicolau@localhost) by sucuri.mat.puc-rio.br (8.9.3/8.9.3) id MAA28286 for obm-l@mat.puc-rio.br; Tue, 15 Apr 2003 12:40:40 -0300 Date: Tue, 15 Apr 2003 12:40:40 -0300 From: "Nicolau C. Saldanha" To: obm-l@mat.puc-rio.br Subject: Re: [obm-l] REGRA DA CADEIA Message-ID: <20030415124040.C27455@sucuri.mat.puc-rio.br> References: <20030415060257.31650.qmail@web13707.mail.yahoo.com> <006301c30358$0c86c4b0$7d07fea9@gauss> Mime-Version: 1.0 Content-Type: text/plain; charset=iso-8859-1 Content-Disposition: inline Content-Transfer-Encoding: 8bit User-Agent: Mutt/1.2.5i In-Reply-To: <006301c30358$0c86c4b0$7d07fea9@gauss>; from dopikas@uol.com.br on Tue, Apr 15, 2003 at 11:05:19AM -0300 Sender: owner-obm-l@sucuri.mat.puc-rio.br Precedence: bulk Reply-To: obm-l@mat.puc-rio.br On Tue, Apr 15, 2003 at 11:05:19AM -0300, Domingos Jr. wrote: > > > qual é a demonstração da regra da cadeia? > > Essa regra pode serr aplicada para expoentes(x^f(x))?Como? > > Depende, a regra da cadeia de funções com 1 variável sai direto da definição > de derivada através de limites, já foram colocadas algumas demonstrações > diversas aqui nessa lista mesmo. > Funções de R^n -> R^m já são mais complicadas e envolvem a matriz > Jacobiana... (eu nem conheço as demonstrações, se alguém puder indicar um > material interessante...) Interessantemente a demonstração do caso geral é talvez mais simples do que a do caso particular (em cálculo 1 costuma-se dividir em casos conforme uma das duas derivadas é igual a 0 ou não). Sejam V e W espaços de Banach (se isso for ir longe demais pense em V e W como R^n e R^m). Para uma função f: V -> W temos f'(x0) = A (onde A é uma transformação linear contínua de V em W; no caso de R^n e R^m toda transformação linear é contínua) se podemos escrever f(x + x0) = f(x0) + Ax + r(x) onde |r(x)| << |x|, ou mais precisamente, se para todo epsilon > 0 existir delta > 0 tal que |x| < delta implica |r(x)| < epsilon |x|. Agora é só juntar as peças: se f(x0) = y0 e f(x + x0) = f(x0) + Ax + r(x) g(y + y0) = g(y0) + By + s(y) então g(f(x+x0)) = g(f(x0) + Ax + r(x)) = g(f(x0)) + BAx + B(r(x)) + s(Ax + r(x)) e basta verificar que |B(r(x)) + s(Ax + r(x))| << |x|, o que não é difícil. []s, N. ========================================================================= Instruções para entrar na lista, sair da lista e usar a lista em http://www.mat.puc-rio.br/~nicolau/olimp/obm-l.html O administrador desta lista é ========================================================================= From owner-obm-l@sucuri.mat.puc-rio.br Tue Apr 15 12:43:46 2003 Return-Path: Received: (from majordom@localhost) by sucuri.mat.puc-rio.br (8.9.3/8.9.3) id MAA28312 for obm-l-MTTP; Tue, 15 Apr 2003 12:41:11 -0300 Received: from hotmail.com (oe36.law10.hotmail.com [64.4.14.93]) by sucuri.mat.puc-rio.br (8.9.3/8.9.3) with ESMTP id MAA28308 for ; Tue, 15 Apr 2003 12:41:06 -0300 Received: from mail pickup service by hotmail.com with Microsoft SMTPSVC; Tue, 15 Apr 2003 08:40:33 -0700 Received: from 64.60.139.18 by oe36.law10.hotmail.com with DAV; Tue, 15 Apr 2003 15:40:33 +0000 X-Originating-IP: [64.60.139.18] X-Originating-Email: [lrecova@hotmail.com] From: =?iso-8859-1?Q?Leandro_Lacorte_Rec=F4va?= To: Subject: RE: [obm-l] REGRA DA CADEIA Date: Tue, 15 Apr 2003 08:40:33 -0700 Message-ID: <003201c30365$5a017110$28029b9b@LeandroRecova> MIME-Version: 1.0 Content-Type: text/plain; charset="iso-8859-1" Content-Transfer-Encoding: 7bit X-Priority: 3 (Normal) X-MSMail-Priority: Normal X-Mailer: Microsoft Outlook, Build 10.0.3416 X-MimeOLE: Produced By Microsoft MimeOLE V6.00.2800.1106 Importance: Normal In-Reply-To: <006301c30358$0c86c4b0$7d07fea9@gauss> X-OriginalArrivalTime: 15 Apr 2003 15:40:33.0352 (UTC) FILETIME=[5A076480:01C30365] Sender: owner-obm-l@sucuri.mat.puc-rio.br Precedence: bulk Reply-To: obm-l@mat.puc-rio.br De uma olhada no livro do Elon, Analise 1 para o caso de uma variavel e no caso de R^n no livro 2. Sao bem interessantes !!! ========================================================================= Instruções para entrar na lista, sair da lista e usar a lista em http://www.mat.puc-rio.br/~nicolau/olimp/obm-l.html O administrador desta lista é ========================================================================= From owner-obm-l@sucuri.mat.puc-rio.br Tue Apr 15 12:47:40 2003 Return-Path: Received: (from majordom@localhost) by sucuri.mat.puc-rio.br (8.9.3/8.9.3) id MAA28453 for obm-l-MTTP; Tue, 15 Apr 2003 12:45:04 -0300 Received: (from nicolau@localhost) by sucuri.mat.puc-rio.br (8.9.3/8.9.3) id MAA28444 for obm-l@mat.puc-rio.br; Tue, 15 Apr 2003 12:45:03 -0300 Date: Tue, 15 Apr 2003 12:45:03 -0300 From: "Nicolau C. Saldanha" To: obm-l@mat.puc-rio.br Subject: Re: [Re: [obm-l] REGRA DA CADEIA] Message-ID: <20030415124503.D27455@sucuri.mat.puc-rio.br> References: <689HDoPd30272S22.1050420594@uwdvg022.cms.usa.net> Mime-Version: 1.0 Content-Type: text/plain; charset=iso-8859-1 Content-Disposition: inline Content-Transfer-Encoding: 8bit User-Agent: Mutt/1.2.5i In-Reply-To: <689HDoPd30272S22.1050420594@uwdvg022.cms.usa.net>; from artur_steiner@usa.net on Tue, Apr 15, 2003 at 12:29:54PM -0300 Sender: owner-obm-l@sucuri.mat.puc-rio.br Precedence: bulk Reply-To: obm-l@mat.puc-rio.br On Tue, Apr 15, 2003 at 12:29:54PM -0300, Artur Costa Steiner wrote: > > > "Domingos Jr." wrote: > > > > > qual é a demonstração da regra da cadeia? > > > Essa regra pode serr aplicada para expoentes(x^f(x))?Como? > > > > Depende, a regra da cadeia de funções com 1 variável sai direto da > definição > > de derivada através de limites, já foram colocadas algumas > demonstrações > > diversas aqui nessa lista mesmo. > > Funções de R^n -> R^m já são mais complicadas e envolvem a matriz > > Jacobiana... (eu nem conheço as demonstrações, se alguém puder indicar > um > > material interessante...) > > Nos livros do Bartle (The Elements of Real Analysis) e do Apostol (Real > Analysis) há as demonstracoes da regra da cadeia para funcoes de R^m em R^n. > Lembro que para funcoes de dominio em R^n, n>1, o conceito de derivada eh > diferente daquele do caso n=1. Se n>1, ao valor da derivada de f em um ponto x > de seu dominio nao eh um numero ou um vetor mas si uma funcao linear que > aproxima f em uma vizinhanca de x. O caso n=1 é um caso particular. Apenas seria um pedantismo dizer para o aluno de cálculo 1 que f'(x) é uma transformação linear de R em R. Ora, toda transformação linear de R em R é da forma x |-> ax e pode ser naturalmente identificada com o número a. É como identificar uma matriz 1x1 com um número. Para quem se perdeu nos espaços de Banach da mensagem anterior eu devo indicar um livro de análise mais avançado, como o Real and Complex Analysis, do Rudin. []s, N. ========================================================================= Instruções para entrar na lista, sair da lista e usar a lista em http://www.mat.puc-rio.br/~nicolau/olimp/obm-l.html O administrador desta lista é ========================================================================= From owner-obm-l@sucuri.mat.puc-rio.br Tue Apr 15 12:49:58 2003 Return-Path: Received: (from majordom@localhost) by sucuri.mat.puc-rio.br (8.9.3/8.9.3) id MAA28709 for obm-l-MTTP; Tue, 15 Apr 2003 12:47:22 -0300 Received: (from nicolau@localhost) by sucuri.mat.puc-rio.br (8.9.3/8.9.3) id MAA28703 for obm-l@mat.puc-rio.br; Tue, 15 Apr 2003 12:47:21 -0300 Date: Tue, 15 Apr 2003 12:47:21 -0300 From: "Nicolau C. Saldanha" To: obm-l@mat.puc-rio.br Subject: Re: [obm-l] REGRA DA CADEIA Message-ID: <20030415124721.E27455@sucuri.mat.puc-rio.br> References: <006301c30358$0c86c4b0$7d07fea9@gauss> <003201c30365$5a017110$28029b9b@LeandroRecova> Mime-Version: 1.0 Content-Type: text/plain; charset=iso-8859-1 Content-Disposition: inline Content-Transfer-Encoding: 8bit User-Agent: Mutt/1.2.5i In-Reply-To: <003201c30365$5a017110$28029b9b@LeandroRecova>; from lrecova@hotmail.com on Tue, Apr 15, 2003 at 08:40:33AM -0700 Sender: owner-obm-l@sucuri.mat.puc-rio.br Precedence: bulk Reply-To: obm-l@mat.puc-rio.br On Tue, Apr 15, 2003 at 08:40:33AM -0700, Leandro Lacorte Recôva wrote: > De uma olhada no livro do Elon, Analise 1 para o caso de uma variavel e > no caso de R^n no livro 2. Sao bem interessantes !!! Para ficar com o Elon eu recomendaria para o caso de R^n um livrinho mais antigo, acho que se chama análise no R^n. []s, N. ========================================================================= Instruções para entrar na lista, sair da lista e usar a lista em http://www.mat.puc-rio.br/~nicolau/olimp/obm-l.html O administrador desta lista é ========================================================================= From owner-obm-l@sucuri.mat.puc-rio.br Tue Apr 15 12:50:11 2003 Return-Path: Received: (from majordom@localhost) by sucuri.mat.puc-rio.br (8.9.3/8.9.3) id MAA28691 for obm-l-MTTP; Tue, 15 Apr 2003 12:47:19 -0300 Received: from cmsrelay02.mx.net (cmsrelay02.mx.net [165.212.11.111]) by sucuri.mat.puc-rio.br (8.9.3/8.9.3) with SMTP id MAA28665 for ; Tue, 15 Apr 2003 12:47:11 -0300 Received: from cmsapps02.cms.usa.net (HELO localhost) (165.212.11.138) by cmsoutbound.mx.net with SMTP; 15 Apr 2003 15:46:38 -0000 Received: from smtp.postoffice.net [165.212.8.22] by cmsapps02.cms.usa.net (ASMTP/) via mtad (C8.MAIN.2.05) with ESMTP id 534HDoPUl0418M38; Tue, 15 Apr 2003 15:46:37 GMT Received: from 200.181.4.100 [200.181.4.100] by uwdvg022.cms.usa.net (USANET web-mailer CM.0402.5.2B); Tue, 15 Apr 2003 15:46:37 -0000 Date: Tue, 15 Apr 2003 12:46:37 -0300 From: Artur Costa Steiner To: Subject: Re: [Re: [[obm-l] REGRA DA CADEIA]] X-Mailer: USANET web-mailer (CM.0402.5.2B) Mime-Version: 1.0 Message-ID: <628HDoPUl0544S22.1050421597@uwdvg022.cms.usa.net> Content-Type: text/plain; charset=ISO-8859-1 Content-Transfer-Encoding: 8bit X-MIME-Autoconverted: from quoted-printable to 8bit by sucuri.mat.puc-rio.br id MAA28681 Sender: owner-obm-l@sucuri.mat.puc-rio.br Precedence: bulk Reply-To: obm-l@mat.puc-rio.br Na primeira mensagem que enviei, o enunciado da Regra da cadeia, saiu errado. Temos que h'(a) = f'(g(a) g'(a) (e não g(a), como acabei esctrevando..) Eho que dah digitar rapido e sem oculos Artur ========================================================================= Instruções para entrar na lista, sair da lista e usar a lista em http://www.mat.puc-rio.br/~nicolau/olimp/obm-l.html O administrador desta lista é ========================================================================= From owner-obm-l@sucuri.mat.puc-rio.br Tue Apr 15 13:31:13 2003 Return-Path: Received: (from majordom@localhost) by sucuri.mat.puc-rio.br (8.9.3/8.9.3) id NAA31625 for obm-l-MTTP; Tue, 15 Apr 2003 13:28:01 -0300 Received: from traven9.uol.com.br (traven9.uol.com.br [200.221.29.35]) by sucuri.mat.puc-rio.br (8.9.3/8.9.3) with ESMTP id NAA31620 for ; Tue, 15 Apr 2003 13:27:51 -0300 Received: from gauss ([200.158.96.222]) by traven9.uol.com.br (8.9.1/8.9.1) with SMTP id NAA14694 for ; Tue, 15 Apr 2003 13:27:19 -0300 (BRT) Message-ID: <00a701c3036c$81d29a00$7d07fea9@gauss> From: "Domingos Jr." To: References: <689HDoPd30272S22.1050420594@uwdvg022.cms.usa.net> Subject: Re: [Re: [obm-l] REGRA DA CADEIA] Date: Tue, 15 Apr 2003 13:31:45 -0300 MIME-Version: 1.0 Content-Type: text/plain; charset="iso-8859-1" Content-Transfer-Encoding: 8bit X-Priority: 3 X-MSMail-Priority: Normal X-Mailer: Microsoft Outlook Express 6.00.2800.1106 X-MimeOLE: Produced By Microsoft MimeOLE V6.00.2800.1106 Sender: owner-obm-l@sucuri.mat.puc-rio.br Precedence: bulk Reply-To: obm-l@mat.puc-rio.br Nos livros do Bartle (The Elements of Real Analysis) e do Apostol (Real > Analysis) há as demonstracoes da regra da cadeia para funcoes de R^m em R^n. > Lembro que para funcoes de dominio em R^n, n>1, o conceito de derivada eh > diferente daquele do caso n=1. Se n>1, ao valor da derivada de f em um ponto x > de seu dominio nao eh um numero ou um vetor mas si uma funcao linear que > aproxima f em uma vizinhanca de x. Dizemos que f de dominio em R^n e valores > em R^m eh diferenciavel em x se existir uma funcao linear L tal que, dado > qualquer eps>0, existir um d>0, tal que, se x estiver no dominio de f e > 0<||x-a|| que a funcao linear L eh a derivada de f em x. (|| signfica a norma de vetores > em R^n e R^m). fazendo-se uma analogia com o caso na reta real, poderiamos > dizer que a derivada de f em x eh a funcao linear que a cada real u associa o > numero f'(x) u, sendo f'(x) conforme a definicao classica. > No caso de R^n, n>1, uma condicao suficiente (porem nao necessaria) para que f > seja diferenciavel em x e que uma de suas derivadas parciais exista em x e > todas as outras derivadas parciais existam em uma vizinhanca de x e sejam > continuas em x. Logo, a continuidade das derivadas parciais de f e m x eh uma > condicao mais forte do que a diferenciabilidade em x. > > Artur Pelo que me lembro, é isso mesmo! Obrigado pela bibliografia... vai pra lista de livros pelos quais tenho interesse. [ ]'s ========================================================================= Instruções para entrar na lista, sair da lista e usar a lista em http://www.mat.puc-rio.br/~nicolau/olimp/obm-l.html O administrador desta lista é ========================================================================= From owner-obm-l@sucuri.mat.puc-rio.br Tue Apr 15 15:00:50 2003 Return-Path: Received: (from majordom@localhost) by sucuri.mat.puc-rio.br (8.9.3/8.9.3) id OAA02625 for obm-l-MTTP; Tue, 15 Apr 2003 14:58:23 -0300 Received: from hotmail.com (oe42.law10.hotmail.com [64.4.14.100]) by sucuri.mat.puc-rio.br (8.9.3/8.9.3) with ESMTP id OAA02621 for ; Tue, 15 Apr 2003 14:58:17 -0300 Received: from mail pickup service by hotmail.com with Microsoft SMTPSVC; Tue, 15 Apr 2003 10:57:45 -0700 Received: from 64.60.139.18 by oe42.law10.hotmail.com with DAV; Tue, 15 Apr 2003 17:57:45 +0000 X-Originating-IP: [64.60.139.18] X-Originating-Email: [lrecova@hotmail.com] From: =?iso-8859-1?Q?Leandro_Lacorte_Rec=F4va?= To: Subject: [obm-l] FW: Integral da funcao de Planck Date: Tue, 15 Apr 2003 10:57:45 -0700 Message-ID: <000901c30378$84d471e0$28029b9b@LeandroRecova> MIME-Version: 1.0 Content-Type: multipart/related; boundary="----=_NextPart_000_000A_01C3033D.D87599E0" X-Priority: 3 (Normal) X-MSMail-Priority: Normal X-Mailer: Microsoft Outlook, Build 10.0.3416 X-MIMEOLE: Produced By Microsoft MimeOLE V6.00.2800.1106 Importance: Normal X-OriginalArrivalTime: 15 Apr 2003 17:57:45.0296 (UTC) FILETIME=[84A65D00:01C30378] Sender: owner-obm-l@sucuri.mat.puc-rio.br Precedence: bulk Reply-To: obm-l@mat.puc-rio.br This is a multi-part message in MIME format. ------=_NextPart_000_000A_01C3033D.D87599E0 Content-Type: multipart/alternative; boundary="----=_NextPart_001_000B_01C3033D.D87599E0" ------=_NextPart_001_000B_01C3033D.D87599E0 Content-Type: text/plain; charset="iso-8859-1" Content-Transfer-Encoding: quoted-printable A integral abaixo parece ser um bom exercicio para os membros de plantao.=20 =20 Regards, =20 Leandro =20 -----Original Message----- From: Frederico Elsner [mailto:frederico_elsner@msn.com]=20 Sent: Tuesday, April 15, 2003 10:49 AM To: Leandro Lacorte Rec=F4va Subject: Integral da funcao de Planck =20 Leandro, =20 A integral abaixo =E9 bastante interessante, e =E9 aquela de que eu te = falei =20 =20 O que acha? =20 Frederico =20 =20 ------=_NextPart_001_000B_01C3033D.D87599E0 Content-Type: text/html; charset="iso-8859-1" Content-Transfer-Encoding: quoted-printable

A integral = abaixo parece ser um bom exercicio para os membros de plantao.

 

Regards,

 

Leandro

 

-----Original Message-----
From: Frederico Elsner [mailto:frederico_elsner@msn.com]
Sent:
Tuesday, April 15, 2003 10:49 = AM
To: Leandro Lacorte = Rec=F4va
Subject: Integral da = funcao de Planck

 

Leandro,

 

A integral abaixo =E9 bastante interessante, e =E9 = aquela de que eu te falei

 

 

O que acha?

 

Frederico

 

 

------=_NextPart_001_000B_01C3033D.D87599E0-- ------=_NextPart_000_000A_01C3033D.D87599E0 Content-Type: image/jpeg; name="image001.jpg" Content-Transfer-Encoding: base64 Content-ID: /9j/4AAQSkZJRgABAQEAYABgAAD//gAcU29mdHdhcmU6IE1pY3Jvc29mdCBPZmZpY2X/2wBDAAoH BwgHBgoICAgLCgoLDhgQDg0NDh0VFhEYIx8lJCIfIiEmKzcvJik0KSEiMEExNDk7Pj4+JS5ESUM8 SDc9Pjv/wAALCABDAJ0BAREA/8QAGQABAAMBAQAAAAAAAAAAAAAAAAQFBgMH/8QAMxAAAgEEAQIF AgUDBAMAAAAAAQIDAAQFERITIQYUIjFBFSMyVmGT0wcWUTNGdsIkRHH/3QAEACj/2gAIAQEAAD8A 9mpSlKUpSlKUpSlKUpSsxl81m7TKTQW9t07deIWX6dNdaTiD1vtkcvX9vpD1j/U3xGq0FlLPPYwT XVt5W4kiVpYOYfpMRsryHY6PbY99V3pSlKUpSlKUpSlK/9D2aqjxLm5cFijcWtg+RvJG4W9mjFWn YKXYA6PcIjtrXfjodyKm4y6lvsVaXk9q9pLcQJJJbvvlEzKCUOwO4J17D2qv/uaHzGvIXnkvM+V8 /wDb6XV6nS48efU/1PRvhrff8Pqq6qE+TRM5FiWgmDzWz3CTenpkIyqy+/LkOan21o+/uKm1n81P k8fkrW6S8m8rLcxQmMRReXjVnRPuE7lLsXYKU0oPDkAAzNoKUpVfd5q1s8vZYySO5M96xWNlgbpD SO/eTXHeoz6dlvY613qwpSlKUpWM/qP/ALV/5JZ/962dYXztr9W4+Zh8x9X1/b/UHLXV4eY4fi5f +x7cNerjy+9UKCK2mztkL3PZGO1yVzPLayS5aaLzSRrHEEVVdV+5LI0ylOxUKAAG0vTxBc3E8+Vv /PvY217PHibKS1d1mmeMvybt6gY5GuDxBBlMaINA/dtrg3o8RZLw5ZzXnSv4o7qS6Mrs1ksnVWTp sTpe8ScF36WkZgCqlRey4OxnyIvpvMySBlcRvdymEMuuJ6RbhsEAg8exAPv3qwpSlZ/xHe2ttm/D EdxcwxPJkn4LJIFLf+NMvYH39ToP/rKPkVoKUpWfyPie1tspZJDlMd5RZZkyLSSD7ARQN9TkFRhI 8SlSCT1BrWjV4ZolnSAyoJXVnWMsOTKCASB8gFl2f1H+a6V//9H2asZ/Uf8A2r/ySz/71r5o2lgk jSZ4WdSokQAshI9xyBGx+oI/Sqn6FkfzZl/2rT+Cn0LI/mzL/tWn8FRr/H5OygV4/EHiC8kduKQ2 8FkWY6J92hCqNA92IHwO5ALHWV5k7JbqHxVmkBZ43R4bPlG6MUdTqEjYZWGwSDrsSNGpP0LI/mzL /tWn8FSo8bdpjpLVs5fPK7clu2SDqxjt2AEYTXY+6k9z39tRfoWR/NmX/atP4KjY6zuMrZLeWfi/ NNEzOnrtbaNgysVYFWtwQQykaI+KXtncY4RG58X5oGZmSNUtbaRnYI0hAC25JPFGOvnWh3IFQr28 gsMNa5i48bZoWV2oeGRLO3cspjMm+K2xYDgrMSQNAHeqm46zuMrZLeWfi/NNEzOnrtbaNgysVYFW twQQykaI+Kk/Qsj+bMv+1afwU+hZH82Zf9q0/grjksfl7TDXC2uSyOVmklhBjYwRSCLqL1QjKsYD GMt3J2CBog96hYXE3k/iLJXOU8P2dpYvYxWNrEem/G3G26ehsdy7B17KOCAGQeoTfCeKyVtjba6z z9XKeWjh9UnUMChEDLy+WZ1LsfkkDbBFNaClYLxI2X8R+LrDBw4O5htsRlLXIfUpNiCWNEJcA6/F twoA5bO98QDW9pULK5NMXarKYJrmWSVYobeDj1JWY+yhiB2G2PfsqsT2BqE8ucvMbcQ3OCxzStKY mglv2aGaEoCW5dEn3JUqyD2J3rW5WCsJcZiIrSZkLKzsEjJKQqzsyxLvXpQEIOw7KOw9hYVzmmit oJJ55UiiiUvJI7BVRQNkkn2AHzWctPFd/f3U8NthNuli9ylvJdKtwjgIY4p0AKwtJyJXbE6UkgaI FTa+Gs9irKS0s5r65ewbHJaPLe9KG4WJh1W0rHgvBijIUOxGrfcY9puV8O5S6tmtC73TfS3sXu3c bme6kQTy8SfSIxHzCDsQ4VePGi2Ob+kY/H3ePubieXIQTXt2LxZQhieN2kCuw4q7oeKJ2VDy0G+3 WpvLuOxtXuZlmZE1sQwvK/c67KgLH3+BUKz8QWV9dJbQwZFXfejNjLmJOw33Z0Cj2+TUPx3aR3fg fM9Rpl6NjPKvSmePZEbaDcSOS/5U7B+RWgpSlf/S9mqizPiKXFz3Yhsknix1oL29d5yjLES+umAp 5tqJ+xKD8PfudXtKhZPGJko4fvzW01vL1YJ4ePOJuLKSAwZTtXYdwfxf50R2s7bylqkHXmnK7LSz PydyTsk/A7k9gAB7AAAAd6VwvbODIWNxZXUfUt7mJopU2RyVhojY7jsfioWDwUOBtTbwXE0qeyq6 xxpGNk6WOJUQd2YkhdnfcnQ1aUpSlQsxjEzOHusZLPNBFdxGKR4ePPiezAcgR3Gx7fPbR71JhjaK CON5nmZFCmRwAzkD3PEAbP6AD9K6UpSqLM+HZcpPdmG9SCLI2gsr1HgLs0QL66ZDDg2pX7kOPw9u x3e0pSlKUpSlKUpSlK//0/ZqUpSlKUpSlKUpSlKV/9k= ------=_NextPart_000_000A_01C3033D.D87599E0-- ========================================================================= Instruções para entrar na lista, sair da lista e usar a lista em http://www.mat.puc-rio.br/~nicolau/olimp/obm-l.html O administrador desta lista é ========================================================================= From owner-obm-l@sucuri.mat.puc-rio.br Tue Apr 15 17:04:03 2003 Return-Path: Received: (from majordom@localhost) by sucuri.mat.puc-rio.br (8.9.3/8.9.3) id RAA05129 for obm-l-MTTP; Tue, 15 Apr 2003 17:01:52 -0300 Received: from itaqui.terra.com.br (itaqui.terra.com.br [200.176.3.19]) by sucuri.mat.puc-rio.br (8.9.3/8.9.3) with ESMTP id RAA05123 for ; Tue, 15 Apr 2003 17:01:46 -0300 Received: from bertioga.terra.com.br (bertioga.terra.com.br [200.176.3.77]) by itaqui.terra.com.br (Postfix) with ESMTP id E36FE73B0 for ; Tue, 15 Apr 2003 08:29:50 -0300 (BRT) Received: from [200.177.188.2] (dl-nas7-sao-C8B1BC02.p001.terra.com.br [200.177.188.2]) by bertioga.terra.com.br (Postfix) with ESMTP id AC5E73F8074 for ; Tue, 15 Apr 2003 08:29:49 -0300 (BRT) User-Agent: Microsoft-Outlook-Express-Macintosh-Edition/5.02.2022 Date: Sat, 02 Jan 1904 20:45:10 -0200 Subject: Re: [obm-l] Fibonacci From: Claudio Buffara To: Message-ID: <27541.487%claudio.buffara@terra.com.br> In-Reply-To: <003001c30204$19c6f5a0$b7ffa5c8@epq.ime.eb.br> Mime-version: 1.0 Content-type: text/plain; charset="ISO-8859-1" Content-Transfer-Encoding: 8bit X-MIME-Autoconverted: from quoted-printable to 8bit by sucuri.mat.puc-rio.br id RAA05124 Sender: owner-obm-l@sucuri.mat.puc-rio.br Precedence: bulk Reply-To: obm-l@mat.puc-rio.br Oi, Marcio: Gostei bastante da demonstracao, especialmente da sua particao do conjunto dos dos nos. pares - simples, mas eu nunca tinha pensado nisso. Eu me pergunto se nao ha outras aplicacoes... Valeu ,mesmo! Um abraco, Claudio. on 13.04.03 19:31, Marcio at marciocohen@superig.com.br wrote: > Oi Cláudio.. A idéia que eu mencionei antes funciona bastante bem nesse > caso. > Afinal, se a,b, a>b são as raízes de x^2 = x+1, sabemos que F_n = (a^n - > b^n)/sqrt(5). Como ab=-1: > Se n eh par, entao 1/sqrt(5)F_n = 1 / [(-1/b)^n - b^n] = b^n / [1 - > b^(2n)] = b^n + b^(3n) + b^(5n) + ... > (como |b|<1, podemos interpretar a fração como a soma da pg que começa em > b^n e tem razao b^(2n) ). > Variando n entre as potências de dois pares e somando temos: > n=2: b^2 + b^6 + b^10 + b^14 +... (expoente = 2 (mod4)) > n=4: b^4 + b^12 + b^20 +... (expoente = 4 (mod8) > n=8: b^8 + b^24 + ... (expoente = 8 (mod16)) > ... > Note que todo expoente par aparece uma e apenas uma vez nessa soma (de fato, > se X = (2^r)i, com i ímpar, então X = 2^r (mod 2^(r+1)), e > X != 2^r (mod 2^(k+1)) para outros valores de k - se k k>r tmb, pq isso implicaria que 2^k | X, contradizendo i ser impar ). > > Logo, [1/sqrt(5)] * [1/F_2 + 1/F_4 + 1/F_8 + 1/F_16 + ...] = b^2 + b^4 + b^6 > + ... = b^2 / (1-b^2) > Sendo S a soma procurada: S - 1/F_1 = sqrt(5) * b^2 / (1-b^2). > Como b = [1-sqrt(5)]/2 e b^2 = 1+b: > S = 1 - sqrt(5)*b = 1-sqrt(5)* ( 1-sqrt(5))/2 = (2 - sqrt(5) + 5)/2 = > (7-sqrt(5))/2. > > Note que os passos em que se trocou ordem de somatório são formalmente > justificados pelo fato de soh termos tratados de series absolutamente > convergentes de termos positivos (pois b^2 > 0). > > Abraco, > Marcio > > ----- Original Message ----- > From: "Claudio Buffara" > To: > Sent: Saturday, April 12, 2003 3:41 PM > Subject: Re: [obm-l] Fibonacci > > >> Oi, Marcio: >> >> Mesmo o problema de se achar: >> S = SOMA(n>=0) 1/F(2^n) >> esta' longe de ser trivial. >> >> Eu sei que S = 4 - A, onde A = (1 + raiz(5))/2, ou seja, >> S = (7 - raiz(5))/2. >> >> Acho que a formula: F(2k) = [F(k+1) + F(k-1)]*F(k) deve entrar em algum >> lugar na demonstracao e, de algum jeito, a restricao as potencias de 2 > deve >> fazer aparecer alguma PG cuja soma eh S. >> >> Um abraco, >> Claudio. >> >> ========================================================================= Instruções para entrar na lista, sair da lista e usar a lista em http://www.mat.puc-rio.br/~nicolau/olimp/obm-l.html O administrador desta lista é ========================================================================= From owner-obm-l@sucuri.mat.puc-rio.br Tue Apr 15 17:04:03 2003 Return-Path: Received: (from majordom@localhost) by sucuri.mat.puc-rio.br (8.9.3/8.9.3) id RAA05119 for obm-l-MTTP; Tue, 15 Apr 2003 17:01:37 -0300 Received: from Euler.impa.br (euler.impa.br [147.65.1.3]) by sucuri.mat.puc-rio.br (8.9.3/8.9.3) with ESMTP id RAA05093 for ; Tue, 15 Apr 2003 17:01:33 -0300 Received: from Gauss.impa.br (Gauss [147.65.4.1]) by Euler.impa.br (8.11.6p2/8.11.6) with ESMTP id h3FK12018496 for ; Tue, 15 Apr 2003 17:01:02 -0300 (EST) From: Carlos Gustavo Tamm de Araujo Moreira Received: by Gauss.impa.br (8.11.6p2) id h3FK0tU01273; Tue, 15 Apr 2003 17:00:55 -0300 (EST) Message-Id: <200304152000.h3FK0tU01273@Gauss.impa.br> Subject: Re: [obm-l] Problemas em Aberto To: obm-l@mat.puc-rio.br Date: Tue, 15 Apr 2003 17:00:54 -0300 (EST) In-Reply-To: <01b901c302c2$3db52ae0$3300c57d@bovespa.com> from "=?iso-8859-1?Q?Cl=E1udio_\=28Pr=E1tica\=29?=" at Apr 14, 3 05:12:52 pm X-Mailer: ELM [version 2.4 PL25] MIME-Version: 1.0 Content-Type: text/plain; charset=US-ASCII Content-Transfer-Encoding: 7bit Sender: owner-obm-l@sucuri.mat.puc-rio.br Precedence: bulk Reply-To: obm-l@mat.puc-rio.br Oi Claudio, Na desigualdade |S-p/K^(m^2)|<(1/K^((m+1)^2))(1+1/2+1/4+...) e' importante o fato de que nao so' o lado direito tende a zero, mas tende a zero mais rapido que 1/K^(m^2) (o inverso do denominador da fracao que aparece do lado esquerdo). E' isso que eu acho que se torna dificil no caso racional geral (embora eu nao tenha duvidas de que no fim o resultado deve ser sempre irracional, e mesmo transcendente). Abracos, Gugu > >Oi, Gugu: > >Vou seguir sua sugestão e ver se consigo fazer o da Eureka. > >Também vou dar uma pensada na generalização do no. 9. Talvez dê pra provar >que, apesar do numerador crescer, a representação Q-esimal (supondo R = P/Q, >com 0 < P < Q e (P,Q) = 1) jamais se torna periódica. > >No caso da sua demonstração, talvez dê pra estreitar a desigualdade: >|S-p/K^(m^2)|<(1/K^((m+1)^2))(1+1/2+1/4+...) >de forma que no fim também se chegue a algo que tende a zero qaundo m -> >infinito. > >Um abraço, >Claudio. > >----- Original Message ----- >From: "Carlos Gustavo Tamm de Araujo Moreira" >To: >Sent: Monday, April 14, 2003 3:06 AM >Subject: Re: [obm-l] Problemas em Aberto > > >> Caro Claudio, >> Eu tinha proposto esse problema da Eureka para uma IMO antiga, mas nao >> entrou... Tente primeiro colocar o triangulo com coordenadas racionais no >> plano com um dos lados no eixo x e depois tente aplicar uma rotacao >> conveniente (multiplicar por um complexo de modulo 1 com coordenadas >> racionais conveniente). Um pouco de aritmetica em Z[i] ajuda... >> Sobre o problema 9, eu nao vejo como as solucoes se generalizam para R >> racional... Os numeradores podem ficar grandes... >> Abracos, >> Gugu >> >> > >> >Oi, Gugu: >> > >> >O no. 7 foi baseado num problema da Eureka, mas eu acabei de ver onde >errei. >> >O problema original eh: >> > >> >"Um triangulo tem os lados de medidas inteiras e area racional. Prove que >> >ele eh congruente a um triangulo cujos vertices tem coordenadas >inteiras". >> > >> >Eu assumi, erroneamente, que um dos lados do triangulo de reticulado >(essa >> >eh a traducao correta de "lattice triangle"?) poderia ser paralelo a um >dos >> >eixos coordenados, de forma que o pe' da altura relativa a este lado >teria >> >coordenadas inteiras. O seu contra-exemplo mostra que esta hipotese nao >eh >> >valida em geral. >> > >> >Um triangulo de reticulado congruente ao seu teria como vertices: >> >(0,0), (15,8), (48,64) ==> nenhum lado eh paralelo aos eixos. >> > >> >De qualquer forma, fica ai o enunciado do problema original. Ao que me >> >consta, a Eureka ainda nao recebeu uma solucao para este. >> > >> >***** >> > >> >A solucao que eu tinha imaginado pro no. 9 eh: >> > >> >O numero S, quando expresso na base K eh igual a: >> >0,100100001000000100000000100..., ou seja, uma K-esimal infinita e nao >> >periodica. Logo, S eh irracional. >> > >> >De qualquer forma, ambas as solucoes so facilmente generalizaveis para o >> >caso de: >> >SOMA(n>=1) R^(n^2), onde R eh um racional entre 0 e 1. >> > >> > >> >Um abraco, >> >Claudio. >> > >> > >> >on 12.04.03 02:50, Carlos Gustavo Tamm de Araujo Moreira at gugu@impa.br >> >wrote: >> > >> >>> >> >>> 6. D=EA um exemplo de uma sequ=EAncia (Xn) de n=FAmeros reais tal >que:=20 >> >>> >> >>> lim ( Xn / n^t ) =3D 0 para todo t > 0=20 >> >>> e >> >>> lim ( [log(n)]^k / Xn ) =3D 0 para todo k > 0 >> >>> >> >> >> >> Existem muitas, como X_n=2^(raiz(log(n)), ou X_n=(log(n))^log(log(n)). >> >> >> >>> ********* >> >>> >> >>> 7. Um tri=E2ngulo tem lados com medida inteira e =E1rea racional. >Prove = >> >>> que uma de suas alturas tem medida inteira e que o p=E9 desta altura = >> >>> est=E1 a uma dist=E2ncia inteira dos v=E9rtices do tri=E2ngulo. >> >>> >> >> >> >> Parece que isso nao esta' certo. O triangulo de lados 17, 65 e 80 tem >area >> >> 288 e alturas 576/17, 576/65 e 36/5, que nao sao inteiras... >> >> >> >>> ********* >> >>> >> >>> 9. Seja K um inteiro >=3D 2.=20 >> >>> infinito >> >>> Seja S =3D SOMAT=D3RIO 1 / K^(n^2) =3D 1/K + 1/K^4 + 1/K^9 + 1/K^16 >+ = >> >>> ... >> >>> n =3D 1 >> >>> Prove que S =E9 irracional. >> >>> >> >> >> >> Se x=p/q e' racional e r/s e' outro racional diferente de x entao >> >> |x-r/s|=|(ps-qr)/qs|>=1/qs, ou seja, s|x-r/s|>=1/q. >> >> Por outro lado, soma(n=1 ate' m)(1/K^(n^2)) e' um racional com >denominador >> >> (divisor de) K^(m^2), digamos p/K^(m^2), e >> >> |S-p/K^(m^2)|<(1/K^((m+1)^2))(1+1/2+1/4+...)=2/K^((m+1)^2), mas >> >> K^(m^2).2/K^((m+1)^2)=2/K^(2m+1) tende a 0 quando m tende a infinito, e >> >> portanto S nao pode ser racional. >> >> >> >> Abracos, >> >> Gugu >> >> >========================================================================= >> >> Instru??es para entrar na lista, sair da lista e usar a lista em >> >> http://www.mat.puc-rio.br/~nicolau/olimp/obm-l.html >> >> O administrador desta lista ? >> >> >========================================================================= >> >> >> > >> >========================================================================= >> >Instruções para entrar na lista, sair da lista e usar a lista em >> >http://www.mat.puc-rio.br/~nicolau/olimp/obm-l.html >> >O administrador desta lista é >> >========================================================================= >> >> ========================================================================= >> Instruções para entrar na lista, sair da lista e usar a lista em >> http://www.mat.puc-rio.br/~nicolau/olimp/obm-l.html >> O administrador desta lista é >> ========================================================================= > >========================================================================= >Instruções para entrar na lista, sair da lista e usar a lista em >http://www.mat.puc-rio.br/~nicolau/olimp/obm-l.html >O administrador desta lista é >========================================================================= ========================================================================= Instruções para entrar na lista, sair da lista e usar a lista em http://www.mat.puc-rio.br/~nicolau/olimp/obm-l.html O administrador desta lista é ========================================================================= From owner-obm-l@sucuri.mat.puc-rio.br Tue Apr 15 22:14:47 2003 Return-Path: Received: (from majordom@localhost) by sucuri.mat.puc-rio.br (8.9.3/8.9.3) id WAA09247 for obm-l-MTTP; Tue, 15 Apr 2003 22:12:23 -0300 Received: from pina.terra.com.br (pina.terra.com.br [200.176.3.31]) by sucuri.mat.puc-rio.br (8.9.3/8.9.3) with ESMTP id WAA09243 for ; Tue, 15 Apr 2003 22:12:20 -0300 Received: from altamira.terra.com.br (altamira.terra.com.br [200.176.3.40]) by pina.terra.com.br (Postfix) with ESMTP id 67C6B35A3B3 for ; Tue, 15 Apr 2003 20:32:02 -0300 (BRT) Received: from niski.com (unknown [200.148.203.85]) (authenticated user fniski) by altamira.terra.com.br (Postfix) with ESMTP id 53CAD3DC0D6 for ; Tue, 15 Apr 2003 20:31:49 -0300 (BRT) Message-ID: <3E9C9674.1020109@niski.com> Date: Tue, 15 Apr 2003 16:32:04 -0700 From: niski User-Agent: Mozilla/5.0 (Windows; U; Windows NT 5.1; en-US; rv:1.0.2) Gecko/20030208 Netscape/7.02 X-Accept-Language: en-us, en MIME-Version: 1.0 To: obm-l@mat.puc-rio.br Subject: [obm-l] um problema Content-Type: text/plain; charset=ISO-8859-1; format=flowed Content-Transfer-Encoding: 8bit Sender: owner-obm-l@sucuri.mat.puc-rio.br Precedence: bulk Reply-To: obm-l@mat.puc-rio.br pessoal, atribuindo a cada letra um valor de 0 ou 1, qual é o valor maximo que a expressao abaixo pode ter? eimqu-djmqu-ehnqu+cjnqu+dhoqu-cioqu-eilru+ djlru+egnru-bjnru-dgoru+bioru+ehlsu-cjlsu- egmsu+bjmsu+cgosu-bhosu-dhltu+ciltu+dgmtu- bimtu-cgntu+bhntu-eimpv+djmpv+ehnpv-cjnpv- dhopv+ciopv+eikrv-djkrv-efnrv+ajnrv+dforv- aiorv-ehksv+cjksv+efmsv-ajmsv-cfosv+ahosv+ dhktv-ciktv-dfmtv+aimtv+cfntv-ahntv+eilpx- djlpx-egnpx+bjnpx+dgopx-biopx-eikqx+djkqx+ efnqx-ajnqx-dfoqx+aioqx+egksx-bjksx-eflsx+ ajlsx+bfosx-agosx-dgktx+biktx+dfltx-ailtx- bfntx+agntx-ehlpy+cjlpy+egmpy-bjmpy-cgopy+ bhopy+ehkqy-cjkqy-efmqy+ajmqy+cfoqy-ahoqy- egkry+bjkry+eflry-ajlry-bfory+agory+cgkty- bhkty-cflty+ahlty+bfmty-agmty+dhlpz-cilpz- dgmpz+bimpz+cgnpz-bhnpz-dhkqz+cikqz+dfmqz- aimqz-cfnqz+ahnqz+dgkrz-bikrz-dflrz+ailrz+ bfnrz-agnrz-cgksz+bhksz+cflsz-ahlsz-bfmsz+ agmsz obrigado Niski www.linux.ime.usp.br/~niski -- [about him:] It is rare to find learned men who are clean, do not stink and have a sense of humour. -Gottfried Whilhem Leibniz ========================================================================= Instruções para entrar na lista, sair da lista e usar a lista em http://www.mat.puc-rio.br/~nicolau/olimp/obm-l.html O administrador desta lista é ========================================================================= From owner-obm-l@sucuri.mat.puc-rio.br Tue Apr 15 22:28:38 2003 Return-Path: Received: (from majordom@localhost) by sucuri.mat.puc-rio.br (8.9.3/8.9.3) id WAA09534 for obm-l-MTTP; Tue, 15 Apr 2003 22:27:18 -0300 Received: from smtp-29.ig.com.br (smtp-29.ig.com.br [200.226.132.157]) by sucuri.mat.puc-rio.br (8.9.3/8.9.3) with SMTP id WAA09526 for ; Tue, 15 Apr 2003 22:27:13 -0300 Received: (qmail 25745 invoked from network); 16 Apr 2003 01:26:55 -0000 Received: from unknown (HELO xxxx) (200.165.168.8) by smtp-29.ig.com.br with SMTP; 16 Apr 2003 01:26:55 -0000 Message-ID: <000901c303b7$ceac4240$08a8a5c8@epq.ime.eb.br> From: "Marcio" To: References: <200304152000.h3FK0tU01273@Gauss.impa.br> Subject: [obm-l] Desigualdade legal Date: Tue, 15 Apr 2003 22:30:46 -0300 MIME-Version: 1.0 Content-Type: text/plain; charset="iso-8859-1" Content-Transfer-Encoding: 7bit X-Priority: 3 X-MSMail-Priority: Normal X-Mailer: Microsoft Outlook Express 5.50.4133.2400 X-MimeOLE: Produced By Microsoft MimeOLE V5.50.4133.2400 Sender: owner-obm-l@sucuri.mat.puc-rio.br Precedence: bulk Reply-To: obm-l@mat.puc-rio.br Segue pro pessoal tentar uma desigualdade legal que apareceu em outra lista: Se a,b,c sao reais positivos e abc <=1, mostre que: a/c + b/a + c/b >= a + b + c Abracos, Marcio ========================================================================= Instruções para entrar na lista, sair da lista e usar a lista em http://www.mat.puc-rio.br/~nicolau/olimp/obm-l.html O administrador desta lista é ========================================================================= From owner-obm-l@sucuri.mat.puc-rio.br Tue Apr 15 22:53:17 2003 Return-Path: Received: (from majordom@localhost) by sucuri.mat.puc-rio.br (8.9.3/8.9.3) id WAA10237 for obm-l-MTTP; Tue, 15 Apr 2003 22:51:55 -0300 Received: from krypton.hosting4u.net (krypton.hosting4u.net [209.15.2.78]) by sucuri.mat.puc-rio.br (8.9.3/8.9.3) with ESMTP id WAA10232 for ; Tue, 15 Apr 2003 22:51:51 -0300 Received: from gargamel (200-158-201-195.dsl.telesp.net.br [200.158.201.195]) by krypton.hosting4u.net (Postfix) with ESMTP id 82AECA08BF for ; Tue, 15 Apr 2003 20:51:18 -0500 (CDT) Message-ID: <200304152254330590.01550783@smtp.watersportsbrazil.com> X-Mailer: Calypso Version 3.30.00.00 (3) Date: Tue, 15 Apr 2003 22:54:33 -0300 From: "Ariel de Silvio" To: obm-l@mat.puc-rio.br Subject: [obm-l] pontos colineares, cade o erro? Mime-Version: 1.0 Content-Type: text/plain; charset="us-ascii" Content-Transfer-Encoding: 8bit X-MIME-Autoconverted: from quoted-printable to 8bit by sucuri.mat.puc-rio.br id WAA10233 Sender: owner-obm-l@sucuri.mat.puc-rio.br Precedence: bulk Reply-To: obm-l@mat.puc-rio.br Ola, mais uma vez estou cometendo algum erro de atencao (provavelmente) "Mostre que A(a,-3a), B(a+3,-3a-1) e C(a+5,-3a-2) sao colineares para todo valor real de a" Mas nao foi o que encontrei, cheguei a considerar a=2, e meu resultado nao foram pontos colineares... Se alguem puder me ajudar a encontrar meu erro, agradeco Tentei atraves da relacao: (xb-xa)/(xc-xb)=(yb-ya)/(yc-yb) cheguei a 3/2=1 (?!?) E tambem tentei atraves de determinates, mas cheguei a nada também E quando considerei a=2 na determinante, cheguei num resultado 27, que deveria ser 0... Obrigado, Ariel ========================================================================= Instruções para entrar na lista, sair da lista e usar a lista em http://www.mat.puc-rio.br/~nicolau/olimp/obm-l.html O administrador desta lista é ========================================================================= From owner-obm-l@sucuri.mat.puc-rio.br Tue Apr 15 23:45:10 2003 Return-Path: Received: (from majordom@localhost) by sucuri.mat.puc-rio.br (8.9.3/8.9.3) id XAA11423 for obm-l-MTTP; Tue, 15 Apr 2003 23:43:26 -0300 Received: from web14302.mail.yahoo.com (web14302.mail.yahoo.com [216.136.173.78]) by sucuri.mat.puc-rio.br (8.9.3/8.9.3) with SMTP id XAA11419 for ; Tue, 15 Apr 2003 23:43:22 -0300 Message-ID: <20030416024251.80728.qmail@web14302.mail.yahoo.com> Received: from [200.167.100.162] by web14302.mail.yahoo.com via HTTP; Tue, 15 Apr 2003 23:42:51 ART Date: Tue, 15 Apr 2003 23:42:51 -0300 (ART) From: =?iso-8859-1?q?Rafael?= Subject: [obm-l] outras 3 circunferencias To: OBM MIME-Version: 1.0 Content-Type: text/plain; charset=iso-8859-1 Content-Transfer-Encoding: 8bit Sender: owner-obm-l@sucuri.mat.puc-rio.br Precedence: bulk Reply-To: obm-l@mat.puc-rio.br Aí pessoal, alguém consegue resolver mais essa?? Já desisti de quebrar a cabeça. Duas circunferências de raios R e r cortam-se sob um ângulo de 120º. Traça-se a tangente comum externa AB(A e B pontos de contato). Determine o raio da circunferência que é tangente às duas primeiras e tangente à reta AB. resposta: 3/4 Rr/[R + r + 2(Rr)^1/2] Abraços, Rafael. _______________________________________________________________________ Yahoo! Mail O melhor e-mail gratuito da internet: 6MB de espaço, antivírus, acesso POP3, filtro contra spam. http://br.mail.yahoo.com/ ========================================================================= Instruções para entrar na lista, sair da lista e usar a lista em http://www.mat.puc-rio.br/~nicolau/olimp/obm-l.html O administrador desta lista é ========================================================================= From owner-obm-l@sucuri.mat.puc-rio.br Wed Apr 16 00:14:47 2003 Return-Path: Received: (from majordom@localhost) by sucuri.mat.puc-rio.br (8.9.3/8.9.3) id AAA12092 for obm-l-MTTP; Wed, 16 Apr 2003 00:13:28 -0300 Received: from trex-b.centroin.com.br (trex-b.centroin.com.br [200.225.63.136]) by sucuri.mat.puc-rio.br (8.9.3/8.9.3) with ESMTP id AAA12088 for ; Wed, 16 Apr 2003 00:13:25 -0300 Received: from centroin.com.br (RJ092013.user.veloxzone.com.br [200.141.92.13] (may be forged)) (authenticated bits=0) by trex-b.centroin.com.br (8.12.9/8.12.9) with ESMTP id h3G3CpVi012328 for ; Wed, 16 Apr 2003 00:12:51 -0300 (EST) Message-ID: <3E9CCA69.3090706@centroin.com.br> Date: Wed, 16 Apr 2003 00:13:45 -0300 From: "A. C. Morgado" User-Agent: Mozilla/5.0 (Windows; U; Windows NT 5.0; en-US; rv:1.0.2) Gecko/20030208 Netscape/7.02 X-Accept-Language: en-us, en MIME-Version: 1.0 To: obm-l@mat.puc-rio.br Subject: Re: [obm-l] pontos colineares, cade o erro? References: <200304152254330590.01550783@smtp.watersportsbrazil.com> Content-Type: text/plain; charset=ISO-8859-1; format=flowed Content-Transfer-Encoding: 8bit Sender: owner-obm-l@sucuri.mat.puc-rio.br Precedence: bulk Reply-To: obm-l@mat.puc-rio.br Esquenta nao que deve ser de digitaçao no enunciado. Se o 5 fosse 6, os pontos seriam colineares. Ariel de Silvio wrote: >Ola, > >mais uma vez estou cometendo algum erro de atencao (provavelmente) > >"Mostre que A(a,-3a), B(a+3,-3a-1) e C(a+5,-3a-2) sao colineares para todo valor real de a" > >Mas nao foi o que encontrei, cheguei a considerar a=2, e meu resultado nao foram pontos colineares... > >Se alguem puder me ajudar a encontrar meu erro, agradeco > >Tentei atraves da relacao: >(xb-xa)/(xc-xb)=(yb-ya)/(yc-yb) >cheguei a 3/2=1 (?!?) > >E tambem tentei atraves de determinates, mas cheguei a nada também > >E quando considerei a=2 na determinante, cheguei num resultado 27, que deveria ser 0... > >Obrigado, >Ariel > >========================================================================= >Instruções para entrar na lista, sair da lista e usar a lista em >http://www.mat.puc-rio.br/~nicolau/olimp/obm-l.html >O administrador desta lista é >========================================================================= > > > > ========================================================================= Instruções para entrar na lista, sair da lista e usar a lista em http://www.mat.puc-rio.br/~nicolau/olimp/obm-l.html O administrador desta lista é ========================================================================= From owner-obm-l@sucuri.mat.puc-rio.br Wed Apr 16 00:50:08 2003 Return-Path: Received: (from majordom@localhost) by sucuri.mat.puc-rio.br (8.9.3/8.9.3) id AAA12816 for obm-l-MTTP; Wed, 16 Apr 2003 00:48:40 -0300 Received: from sidney2.bol.com.br (sidney2.bol.com.br [200.221.24.141]) by sucuri.mat.puc-rio.br (8.9.3/8.9.3) with ESMTP id AAA12809 for ; Wed, 16 Apr 2003 00:48:23 -0300 Received: from moyra (200.221.24.99) by sidney2.bol.com.br (5.1.071) id 3E766D9200651027 for obm-l@mat.puc-rio.br; Wed, 16 Apr 2003 00:47:27 -0300 Message-ID: <005c01c303ca$ee779f80$8929d8c8@moyra> From: "ponciomineiro" To: References: <200304152254330590.01550783@smtp.watersportsbrazil.com> Subject: Re: [obm-l] pontos colineares, cade o erro? Date: Wed, 16 Apr 2003 00:47:39 -0300 MIME-Version: 1.0 Content-Type: text/plain; charset="iso-8859-1" Content-Transfer-Encoding: 8bit X-Priority: 3 X-MSMail-Priority: Normal X-Mailer: Microsoft Outlook Express 5.00.2919.6600 X-MimeOLE: Produced By Microsoft MimeOLE V5.00.2919.6600 X-Sender-IP: 200.149.104.145 Sender: owner-obm-l@sucuri.mat.puc-rio.br Precedence: bulk Reply-To: obm-l@mat.puc-rio.br ----- Original Message ----- From: "Ariel de Silvio" To: Sent: Tuesday, April 15, 2003 10:54 PM Subject: [obm-l] pontos colineares, cade o erro? > E-mail Premium BOL > Antivírus, anti-spam e até 100 MB de espaço. Assine já! > http://email.bol.com.br/ > Ola, > > mais uma vez estou cometendo algum erro de atencao (provavelmente) > > "Mostre que A(a,-3a), B(a+3,-3a-1) e C(a+5,-3a-2) sao colineares para todo valor real de a" > > Mas nao foi o que encontrei, cheguei a considerar a=2, e meu resultado nao foram pontos colineares... > > Se alguem puder me ajudar a encontrar meu erro, agradeco > > Tentei atraves da relacao: > (xb-xa)/(xc-xb)=(yb-ya)/(yc-yb) > cheguei a 3/2=1 (?!?) > > E tambem tentei atraves de determinates, mas cheguei a nada também > > E quando considerei a=2 na determinante, cheguei num resultado 27, que deveria ser 0... > > Obrigado, > Ariel > > ========================================================================= > Instruções para entrar na lista, sair da lista e usar a lista em > http://www.mat.puc-rio.br/~nicolau/olimp/obm-l.html > O administrador desta lista é > ========================================================================= > > Caro Ariel, os pontos não seriam A(a,-3a), B(a+5/2,-3a-1) e C(a+5,-3a-2) ? Isso faria sentido, visto que acréscimos iguais a x conduziriam acréscimos iguais à y e, portanto, A, B e C seriam colineares...Um grande abraço, Poncio Mineiro ========================================================================= Instruções para entrar na lista, sair da lista e usar a lista em http://www.mat.puc-rio.br/~nicolau/olimp/obm-l.html O administrador desta lista é ========================================================================= From owner-obm-l@sucuri.mat.puc-rio.br Wed Apr 16 01:38:21 2003 Return-Path: Received: (from majordom@localhost) by sucuri.mat.puc-rio.br (8.9.3/8.9.3) id BAA13903 for obm-l-MTTP; Wed, 16 Apr 2003 01:36:56 -0300 Received: from web13705.mail.yahoo.com (web13705.mail.yahoo.com [216.136.175.138]) by sucuri.mat.puc-rio.br (8.9.3/8.9.3) with SMTP id BAA13899 for ; Wed, 16 Apr 2003 01:36:53 -0300 Message-ID: <20030416043622.42259.qmail@web13705.mail.yahoo.com> Received: from [200.213.88.166] by web13705.mail.yahoo.com via HTTP; Wed, 16 Apr 2003 01:36:22 ART Date: Wed, 16 Apr 2003 01:36:22 -0300 (ART) From: =?iso-8859-1?q?pichurin?= Subject: Re: [[obm-l] REGRA DA CADEIA] To: obm-l@mat.puc-rio.br In-Reply-To: <590HDooKj1664S20.1050419370@uwdvg007.cms.usa.net> MIME-Version: 1.0 Content-Type: text/plain; charset=iso-8859-1 Content-Transfer-Encoding: 8bit Sender: owner-obm-l@sucuri.mat.puc-rio.br Precedence: bulk Reply-To: obm-l@mat.puc-rio.br Demonstracao: suponhamos inicialmente que exista uma > vizinhanca V de a, > contida em I, na qual g(x)<>g(a) para x<>a. Para > todo x<>a em V, temos entao > que [h(x)-h(a)]/(x-a) = f[g(x)]-f[g(a)]/(x-a) > =f[g(x)]-f[g(a)]/[(g(x)-g(a)].[(g(x)-g(a)]/(x-a) > (1). Não entendo essa passagem.....como que {f[g(x)]-f[g(a)]}/(x-a)={f[g(x)]-f[g(a)]}/[(g(x)-(g(a)].[g(x)-g(a)]/(x-a)? Sendo v a funcao > definida no domínio de f para u<>g(a) por v(u) = > [f(u)-f(g(a)]/(u-g(a)), > segue-se da diferenciabilidae de f em g(a) que lim > (u-->g(a)) v(u) = f'(g(a). > Da diferenciabilidade de g em a segue-se que g eh > continua em a. E como > g(x)<>g(a) para x<> a em V, podemos aplicar aquele > teorma relativo a limites > de composicao de funcoes e concluir que, como > v(g(x))= > f[g(x)]-f[g(a)]/(g(x-g(a)), entao lim > (x-->a)v(g(x))= lim (x-->a) > f[g(x)]-f[g(a)]/(g(x)-g(a)) = lim (u-->g(a)) v(u) = > f'(g(a)). Por outro lado, > da diferenciabilidade de g em a eh imediato que lim > (x-->a) [g(x) - > g(a)]/(x-a) = g'(a). Considerando-se (1) e a > existência dos limites em a das > funcoes cujo produto compoe o primeiro membro de > (1), concluimos que lim > (x-->a) [h(x)-h(a)]/(x-a) = h'(a) existe e > iguala-se a f'(g(a)) g'(a). Isto > prova a regra para o caso em questao. > > Mas existe a possibilidade de que g seja um tanto > "patologica" e g(x) - g(a) > se anule para x<> a em qualquer vizinhanca de a. > Neste caso, o procedimento > acima nao se aplica, pois teriamos denominador nulo. > Observamos porem que, > nesta nova sitauacao, a diferenciabilidade de g em > acarreta automaticamente > que g'(a) = 0 (de outra forma, g-a seria > estritamente positiva ou negativa em > uma vizinhanca de a, contrariando nossa hipotese). > Da diferenciabilidade de f > em g(a), segue-se das propriedades das derivadas que > existem M>0 e uma > vizinhanca U de g(a) contida em f(I) na qual |f(y) - > f(g(a)| <= M (y-g(a)). > Logo, |f(g(x) - f(g(a)| <= M (g(x) - g(a)) para todo > x em I. Para x<> a, temos > entao que|f(g(x) - f(g(a)|/(x-a) <= M [(g(x) - > g(a)]/(x-a). Como g'(a) = 0, o > segundo membro tende a zero quando x-->a e, > consequentemente, o mesmo ocorre > para o primeiro membro. Logo h'(a) = 0 = f'(g(a). > g'(a), o que mostra que a R. > da cadeia eh valida tambem para este ultimo caso. > > No caso que vc citou, temos que h(x)= x^f(x) = > e^[f(x) ln(x)] Supomdo-se x>0 e > f diferenciavel em x, a aplicacao da r. da cadeia > leva a que h'(x) = e^[f(x) > ln(x)] d/dx [f(x) ln(x)] , visto que (e^u)' = e^u > u'. Logo, h'(x) = x^f(x) > [f(x)/x + f'(x) ln(x] > > Um abraco > Artur > > ========================================================================= > Instruções para entrar na lista, sair da lista e > usar a lista em > http://www.mat.puc-rio.br/~nicolau/olimp/obm-l.html > O administrador desta lista é > > ========================================================================= _______________________________________________________________________ Yahoo! Mail O melhor e-mail gratuito da internet: 6MB de espaço, antivírus, acesso POP3, filtro contra spam. http://br.mail.yahoo.com/ ========================================================================= Instruções para entrar na lista, sair da lista e usar a lista em http://www.mat.puc-rio.br/~nicolau/olimp/obm-l.html O administrador desta lista é ========================================================================= From owner-obm-l@sucuri.mat.puc-rio.br Wed Apr 16 01:44:10 2003 Return-Path: Received: (from majordom@localhost) by sucuri.mat.puc-rio.br (8.9.3/8.9.3) id BAA14044 for obm-l-MTTP; Wed, 16 Apr 2003 01:42:50 -0300 Received: from web13709.mail.yahoo.com (web13709.mail.yahoo.com [216.136.175.251]) by sucuri.mat.puc-rio.br (8.9.3/8.9.3) with SMTP id BAA14039 for ; Wed, 16 Apr 2003 01:42:45 -0300 Message-ID: <20030416044214.96197.qmail@web13709.mail.yahoo.com> Received: from [200.213.88.166] by web13709.mail.yahoo.com via HTTP; Wed, 16 Apr 2003 01:42:14 ART Date: Wed, 16 Apr 2003 01:42:14 -0300 (ART) From: =?iso-8859-1?q?pichurin?= Subject: [obm-l] +Demonstrações To: obm-l@mat.puc-rio.br MIME-Version: 1.0 Content-Type: text/plain; charset=iso-8859-1 Content-Transfer-Encoding: 8bit Sender: owner-obm-l@sucuri.mat.puc-rio.br Precedence: bulk Reply-To: obm-l@mat.puc-rio.br Demonstre que a base do conjunto das soluções de y^(n) + A(n-1)y^(n-1) + ...+ A2y'' + A1y'+ A0y=0, sendo A(n-1),A2,A1 constantes reais tem dimensão n. _______________________________________________________________________ Yahoo! Mail O melhor e-mail gratuito da internet: 6MB de espaço, antivírus, acesso POP3, filtro contra spam. http://br.mail.yahoo.com/ ========================================================================= Instruções para entrar na lista, sair da lista e usar a lista em http://www.mat.puc-rio.br/~nicolau/olimp/obm-l.html O administrador desta lista é ========================================================================= From owner-obm-l@sucuri.mat.puc-rio.br Wed Apr 16 02:46:01 2003 Return-Path: Received: (from majordom@localhost) by sucuri.mat.puc-rio.br (8.9.3/8.9.3) id CAA15657 for obm-l-MTTP; Wed, 16 Apr 2003 02:43:15 -0300 Received: from Euler.impa.br (euler.impa.br [147.65.1.3]) by sucuri.mat.puc-rio.br (8.9.3/8.9.3) with ESMTP id CAA15653 for ; Wed, 16 Apr 2003 02:43:12 -0300 Received: from Gauss.impa.br (Gauss [147.65.4.1]) by Euler.impa.br (8.11.6p2/8.11.6) with ESMTP id h3G5gg025179 for ; Wed, 16 Apr 2003 02:42:42 -0300 (EST) From: Carlos Gustavo Tamm de Araujo Moreira Received: by Gauss.impa.br (8.11.6p2) id h3G5gZR11558; Wed, 16 Apr 2003 02:42:35 -0300 (EST) Message-Id: <200304160542.h3G5gZR11558@Gauss.impa.br> Subject: Re: [obm-l] Problemas em Aberto II To: obm-l@mat.puc-rio.br Date: Wed, 16 Apr 2003 02:42:34 -0300 (EST) In-Reply-To: <045901c2de8a$b6ac6640$3300c57d@bovespa.com> from "=?Windows-1252?Q?Cl=E1udio_\=28Pr=E1tica\=29?=" at Feb 27, 3 03:04:44 pm X-Mailer: ELM [version 2.4 PL25] MIME-Version: 1.0 Content-Type: text/plain; charset=US-ASCII Content-Transfer-Encoding: 7bit Sender: owner-obm-l@sucuri.mat.puc-rio.br Precedence: bulk Reply-To: obm-l@mat.puc-rio.br > >16. Seja f uma fun=E7=E3o cont=EDnua em [a,b] e diferenci=E1vel em = >(a,b). > >A) =C9 poss=EDvel que, apesar de existir, f' seja descont=EDnua em todo = >ponto de (a,b). > >B) Em caso afirmativo, ser=E1 que a condi=E7=E3o f(a) < f(b) =E9 = >suficiente para garantir que exista um sub-intervalo [c,d] (a <=3D c < d = ><=3D b) onde f =E9 crescente? > > A) Nao. Vou usar (mais de uma vez) o teorema de Baire: toda intersecao enumeravel de abertos densos (num intervalo) e' densa (nesse intervalo). Suponha que f:(a,b)->R seja derivavel e f' seja descontinua em todo ponto de (a,b). Dado x em (a,b) definimos o salto de f' em x como w(f',x)=lim(c->0)(sup{f'(y), y em (x-c,x+c)}-inf{f'(y), y em (x-c,x+c)}). Nao e' dificil ver que se x_n tende a x e w(f',x_n) >= d para todo n entao w(f',x) >= d. Assim, para todo n inteiro positivo, {x em (a,b) | w(f,x) < 1/n} e' aberto. se esses conjuntos forem densos em (a,b) para todo n entao o conjunto dos pontos de continuidade de f', que e' dado por {x em (a,b) | w(f',x)=0}={x em (a,b) | w(f',x) < 1/n para todo n} seria uma intersecao enumeravel de abertos densos em (a,b) e portanto seria denso em (a,b), e logo nao-vazio, absurdo. Assim, existem n natural e um intervalo (c,d) contido em (a,b) tais que w(f',x) >= 1/n para todo x em (c,d). Isso implica que, para todo inteiro positivo k e todo x em (c,d) existe y em (c,d) com |y-x| < 1/k e |f'(y)-f'(x)| > 1/3n, e logo existem z em {x,y}, h e t em R com 0<|t|<|h|=|y-x|<1/k tais que |(f(z+h)-f(z))/h-(f(z+t)-f(z))/t| > 1/4n. Assim, para todo k (usando a continuidade de f), Z_k:={z em (c,d) | existem h e t em R com 0<|t|<|h|<1/k tais que |(f(z+h)-f(z))/h-(f(z+t)-f(t))/t| > 1/4n} e' aberto e denso em (c,d), donde a intersecao dos Z_k para todo inteiro positivo k e' densa e portanto nao-vazia em (c,d), mas em todo elemento dessa intersecao f nao pode ser derivavel, absurdo. B)Existem funcoes diferenciaveis que nao sao monotonas em nenhum intervalo, mas isso nao e' muito facil de provar. Ha' artigos sobre isso que datam do fim do seculo XIX. Vou dar uma referencia: "Everywhere differentiable, nowhere monotone, functions", de Y. Katznelson e Karl Stromberg, American Mathematical Monthly 81 (1974), 349-354. Tambem existem funcoes ("escada") nao-constantes diferenciaveis em todo ponto que sao localmente constantes (i.e., sua derivada e' nula) num aberto denso. A minha construcao daquelas funcoes usa essas. >********** > >17. a, b, c, d s=E3o n=FAmeros reais n=E3o-negativos tais que: > > ab+ac+ad+bc+bd+cd+abc+abd+acd+bcd=3D2. > >Mostre que: > >3(a+b+c+d)>=3D4(ab+ac+ad+bc+bd+cd). > Esse problema e' equivalente ao problema 11 de "Mais problemas em aberto II", que eu ja' resolvi: faca a=A/2, b=B/2, c=C/2 e d=D/2 Teremos 2(AB+AC+AD+BC+BC+CD)+ABC+ABD+ACD+BCD=16 e a conclusao 3(a+b+c+d)>=4(ab+ac+ad+bc+bd+cd) equivale a A+B+C+D>=2/3*(AB+AC+AD+BC+BD+CD). >********* > >18. Numa loteria sao sorteados 7 numeros escolhidos aleatoriamente de = >{1,2,3,...,48,49}. Cada cartao de apostas deve ser preenchido com = >exatamente 7 numeros. Uma pessoa pode pode apostar quantos cartoes = >desejar sem pagar nada, desde que quaisquer dois cartoes de sua aposta = >tenham, NO MAXIMO, uma dezena em comum. O primeiro premio e dado a = >pessoa que acertar o maior numero de triplos. >A) Exiba uma aposta gratuita que tenha a maxima probabibilidade de = >ganhar o primeiro premio. >B) Qual o valor da probabilidade acima ? > Nao entendi bem essa condicao sobre os triplos, mas a aposta gratuita com a maior quantidade de cartoes faz com que todos os pares de dezenas aparecam em exatamente um dos cartoes apostados (as condicoes do problema obrigam cada par de dezenas a aparecer em no maximo um cartao da aposta gratuita, o que limita o numero de cartoes em C(49,2)/C(7,2)=56, pois cada cartao tem 7 dezenas e C(7,2) pares de dezenas). Um jeito de fazer isso e' pensar no conjunto das 49 dezenas como o plano afim sobre Z/7Z dado por P=Z/7Z x Z/7Z, e atribuir um cartao a cada reta {(x,y) em P | Ax+By=C} para (A,B,C) em (Z/7Z)^3 diferente de (0,0,0). Podemos supor (para contar cada reta uma vez so') que A=1 ou (A,B)=(0,1), o que nos da' 8.7=56 retas (para cada escolha de (A,B) podemos escolher qualquer C em Z/7Z). Como por cada par de pontos passa exatamente uma reta essa construcao funciona. >*********** > >19. Suponha que os n=FAmeros da forma 2^x * 3^y (x, y: inteiros n=E3o = >negativos) s=E3o colocados em ordem crescente. Prove que existem termos = >consecutivos - digamos 2^a * 3^b e 2^c * 3^d - tais que um dos = >n=FAmeros | a - c | ou | b - d | =E9 t=E3o grande quanto se queira. > >************* > >20. Duas de An=E1lise Real: > >A) Prove que se f:{a, b) -> R =E9 cont=EDnua em c em (a,b) e lim x-> c >f'(x) =3D L, ent=E3o f'(c) =3D L. A partir da=ED, conclua que derivadas = >jamais >apresentam descontinuidades do tipo salto. Conclua tamb=E9m que se f' = >=E9 >monot=F4nica em um intervalo I, ent=E3o f'=E9 cont=EDnua em I. > >B) Suponhamos que f seja diferenci=E1vel em R e seja k<>0. Mostre que: >B.1) se k>0, ent=E3o lim x -> infinito f'(x) + k f(x) =3D L, L em R, = >implica >que lim x-> infinito f('x) =3D 0 e lim x-> infinito f(x) =3D L/k >B.2) se k<0, ent=E3o lim x-> infinito f'(x) + k f(x) =3D L, L em R, s=F3 = >=E9 >poss=EDvel se lim x-> e^(kx) f(x) =3D 0, caso em que temos tamb=E9m lim = >x-> >infinito f('x) =3D 0 e lim x-> infinito f(x) =3D L/k >sugest=E3o : defina h(x) =3D e^(kx) f(x) g(x) =3D e^(kx) . Logo, f(x) = >=3D >h(x)/g(x). Use L'Hopital. > > >************** > >------=_NextPart_000_0450_01C2DE71.8F8ABC60 >Content-Type: text/html; > charset="Windows-1252" >Content-Transfer-Encoding: quoted-printable > > >Help >charset=3Dwindows-1252">href=3Dfile://C:\WINDOWS\> > > > > >
Continuando a compila=E7=E3o de = >problemas n=E3o=20 >resolvidos da lista:
>
 
>
11. Dado um corredor com 1 metro de = >largura, que=20 >faz uma "curva" de 90 graus e
continua com a mesma largura, = >qual a=20 >figura plana de maior =E1rea poss=EDvel que pode fazer
essa curva? = >Observe que o=20 >formato dessa area pode ser qualquer e, obviamente, ela =E9 suposta = > >rigida.
>
(Acho que este problema ainda est=E1 em = >aberto - e=20 >n=E3o s=F3 aqui na lista. De qualquer forma....)
>
 
>
**********
>
 
>
12. Dada a sequencia a[n+1]=3D = >2a[1]*a[n] -=20 >a[n-1] definida para todo n>=3D1 tal que a[0]=3D100 e a[100]=3D = >0.=20 >

a) Mostre que | a[1] = >|<=3D1.

>

b) Determine a[2003].

>

**********

>

13.  X, Y e Z s=E3o reais positivos=20 >e satisfazem o sistema abaixo,

X^2 + XY + (Y^2)/3 =3D = >25
(Y^2)/3 +=20 >Z^2 =3D 9
Z^2 + ZX + X^2 =3D 16

Encontre o valor de ( XY + 2YZ = >+ 3ZX=20 >).

SUGEST=C3O : Voc=EA nao precisa, necessariamente, resolver o = >sistema=20 >...

>

**********

>

14. De quantas formas podemos colocar N = >rainhas em=20 >um
tabuleiro NxN tal que nenhuma rainha possa = >enxergar
outra?

obs:=20 >uma rainha enxerga outra se ambas estiverem na
mesma coluna, linha ou = > >diagonal.
(Este problema tamb=E9m est=E1 em aberto. Talvez valha a = >pena tentar=20 >com Torres e Bispos ao inv=E9s de Rainhas)

>

***********

>

15. 
>
> _ _ _ _ _ _ _ = >1 2 ... n=20 >_
> _|_| |_|_| |_|_|_|_|_|_|_
> B   \_\=20 >/_/      A
>     = > =20 >\_|_/
>        |_|
>&= >nbsp;     =20 > |_|
>        |_|=20 >C
>        |o|
>
> = >Imagine=20 >que o 'desenho' acima =E9 uma linha f=E9rrea,
> aonde o segmento B = >=E9 extens=E3o=20 >do segmento A e o
> segmento C se conecta com ambos = >segmentos.
> Os=20 >numeros no segmento A representam n vag=F5es
> _soltos_ e=20 >enumerados.
> Os vagoes podem se mover de A -> B, A -> C e C = >->=20 >B,
> mas nunca de C -> A nem B -> A nem B -> C..
> = >
>=20 >De quantas formas eh possivel reagrupar os vag=F5es no
> segmento = >B?
>=20 >
> (h=E1 espa=E7o suficiente para n vag=F5es tanto em A,
> = >quanto em B e=20 >em C)

>

************

>

16. Seja f uma fun=E7=E3o cont=EDnua em = >[a,b] e=20 >diferenci=E1vel em (a,b).

>

A) =C9 poss=EDvel que, apesar de = >existir, f' seja=20 >descont=EDnua em todo ponto de (a,b).

>

B) Em caso afirmativo, ser=E1 que a = >condi=E7=E3o f(a) <=20 >f(b) =E9 suficiente para garantir que exista um sub-intervalo [c,d] (a = ><=3D c=20 >< d <=3D b) onde f =E9 crescente?

>

**********

>

17. a, b, c, d s=E3o n=FAmeros reais = >n=E3o-negativos=20 >tais que:

>

size=3D2> ab+ac+ad+bc+bd+cd+abc+abd+acd+bcd=3D2.

>

Mostre que:

>

size=3D2>3(a+b+c+d)>=3D4(ab+ac+ad+bc+bd+cd).

>

*********

>

18. Numa loteria sao sorteados 7 numeros = >escolhidos=20 >aleatoriamente de {1,2,3,...,48,49}. Cada cartao de apostas deve ser = >preenchido=20 >com exatamente 7 numeros. Uma pessoa pode pode apostar quantos = >cartoes=20 >desejar sem pagar nada, desde que quaisquer dois cartoes de sua aposta = >tenham,=20 >NO MAXIMO, uma dezena em comum. O primeiro premio e dado a pessoa que = >acertar o=20 >maior numero de triplos.
A) Exiba uma aposta gratuita que tenha a = >maxima=20 >probabibilidade de ganhar o primeiro premio.
B) Qual o valor da = >probabilidade=20 >acima ?

>

***********

>

19. Suponha que os n=FAmeros da forma 2^x = >* 3^y (x, y:=20 >inteiros n=E3o negativos) s=E3o colocados em ordem crescente. Prove que = >existem=20 >termos consecutivos - digamos 2^a * 3^b  e  2^c * 3^d - tais = >que um=20 >dos n=FAmeros | a - c |  ou  | b - d | =E9 t=E3o grande quanto = >se=20 >queira.

>

*************

>

20. Duas de An=E1lise Real:

>

A) Prove que se f:{a, b) -> R  = >=E9 cont=EDnua em=20 >c em (a,b) e lim x-> c
f'(x) =3D L, ent=E3o f'(c) =3D L. A partir = >da=ED, conclua=20 >que derivadas jamais
apresentam descontinuidades do tipo salto. = >Conclua=20 >tamb=E9m que se f' =E9
monot=F4nica em um intervalo I, ent=E3o f'=E9 = >cont=EDnua em=20 >I.

B) Suponhamos que f seja diferenci=E1vel em R e seja = >k<>0. Mostre=20 >que:
B.1) se k>0, ent=E3o lim x -> infinito f'(x) + k f(x) =3D = >L, L em=20 >R,  implica
que lim x-> infinito f('x) =3D 0 e lim x-> = >infinito f(x)=20 >=3D L/k
B.2) se k<0, ent=E3o lim x-> infinito f'(x) + k f(x) = >=3D L, L em R,=20 >s=F3 =E9
poss=EDvel se lim x-> e^(kx) f(x) =3D 0, caso em que = >temos tamb=E9m lim=20 >x->
infinito f('x) =3D 0 e lim x-> infinito  f(x) =3D = >L/k
sugest=E3o=20 >: defina h(x) =3D e^(kx) f(x) g(x) =3D e^(kx) . Logo, f(x) = >=3D
h(x)/g(x). Use=20 >L'Hopital.

>

size=3D2>**************

> >------=_NextPart_000_0450_01C2DE71.8F8ABC60-- > >========================================================================= >Instruções para entrar na lista, sair da lista e usar a lista em >http://www.mat.puc-rio.br/~nicolau/olimp/obm-l.html >O administrador desta lista é >========================================================================= ========================================================================= Instruções para entrar na lista, sair da lista e usar a lista em http://www.mat.puc-rio.br/~nicolau/olimp/obm-l.html O administrador desta lista é ========================================================================= From owner-obm-l@sucuri.mat.puc-rio.br Wed Apr 16 07:13:30 2003 Return-Path: Received: (from majordom@localhost) by sucuri.mat.puc-rio.br (8.9.3/8.9.3) id HAA19466 for obm-l-MTTP; Wed, 16 Apr 2003 07:10:27 -0300 Received: from spinoza.ime.usp.br (spinoza.ime.usp.br [143.107.45.30]) by sucuri.mat.puc-rio.br (8.9.3/8.9.3) with SMTP id HAA19462 for ; Wed, 16 Apr 2003 07:10:24 -0300 Received: (qmail 12278 invoked from network); 16 Apr 2003 10:09:54 -0000 Received: from coliseu.linux.ime.usp.br (192.168.240.9) by spinoza.linux.ime.usp.br with SMTP; 16 Apr 2003 10:09:54 -0000 Received: (qmail 11456 invoked from network); 16 Apr 2003 10:09:54 -0000 Received: from capibaribe.linux.ime.usp.br (192.168.240.206) by coliseu.linux.ime.usp.br with QMQP; 16 Apr 2003 10:09:54 -0000 Date: Wed, 16 Apr 2003 07:09:53 -0300 From: Wendel Scardua To: obm-l@mat.puc-rio.br Subject: Re: [[obm-l] REGRA DA CADEIA] Message-ID: <20030416100912.GA21143@linux.ime.usp.br> Mail-Followup-To: obm-l@mat.puc-rio.br References: <590HDooKj1664S20.1050419370@uwdvg007.cms.usa.net> <20030416043622.42259.qmail@web13705.mail.yahoo.com> Mime-Version: 1.0 Content-Type: text/plain; charset=iso-8859-1 Content-Disposition: inline Content-Transfer-Encoding: 8bit In-Reply-To: <20030416043622.42259.qmail@web13705.mail.yahoo.com> User-Agent: Mutt/1.3.28i Sender: owner-obm-l@sucuri.mat.puc-rio.br Precedence: bulk Reply-To: obm-l@mat.puc-rio.br > Não entendo essa passagem.....como que > {f[g(x)]-f[g(a)]}/(x-a)={f[g(x)]-f[g(a)]}/[(g(x)-(g(a)].[g(x)-g(a)]/(x-a)? > É só notar que está divindo e multiplicando um mesmo valor (que podia ser qualquer coisa) : x / y = (x / z) * (z / y) Wendel Scardua -------------------------------------------- ========================================================================= Instruções para entrar na lista, sair da lista e usar a lista em http://www.mat.puc-rio.br/~nicolau/olimp/obm-l.html O administrador desta lista é ========================================================================= From owner-obm-l@sucuri.mat.puc-rio.br Wed Apr 16 09:56:25 2003 Return-Path: Received: (from majordom@localhost) by sucuri.mat.puc-rio.br (8.9.3/8.9.3) id JAA21922 for obm-l-MTTP; Wed, 16 Apr 2003 09:54:30 -0300 Received: from smtp1.vesper.com.br ([200.218.62.20]) by sucuri.mat.puc-rio.br (8.9.3/8.9.3) with ESMTP id JAA21917 for ; Wed, 16 Apr 2003 09:54:26 -0300 Received: from ipcsrvnws02.vespersa.com.br ([10.11.255.107]) by smtp1.vesper.com.br with Microsoft SMTPSVC(5.0.2195.5329); Wed, 16 Apr 2003 09:54:02 -0300 Received: by ipcsrvnws02.vespersa.com.br with Internet Mail Service (5.5.2653.19) id ; Wed, 16 Apr 2003 09:53:40 -0300 Message-ID: From: =?iso-8859-1?Q?Jo=E3o_Gilberto_Ponciano_Pereira?= To: "'obm-l@mat.puc-rio.br'" Subject: RE: [obm-l] um problema Date: Wed, 16 Apr 2003 09:53:31 -0300 MIME-Version: 1.0 X-Mailer: Internet Mail Service (5.5.2653.19) Content-Type: text/plain; charset="iso-8859-1" X-OriginalArrivalTime: 16 Apr 2003 12:54:02.0218 (UTC) FILETIME=[414314A0:01C30417] Content-Transfer-Encoding: 8bit X-MIME-Autoconverted: from quoted-printable to 8bit by sucuri.mat.puc-rio.br id JAA21919 Sender: owner-obm-l@sucuri.mat.puc-rio.br Precedence: bulk Reply-To: obm-l@mat.puc-rio.br Putz, que problema chato! Vamos separar o conjunto nos números de Soma (60 no total) e nos números de subtração(60 também). Se vocês repararem bem, verão que no quinto dígito (10000) só temos as letras de A a E. Se repararem melhor ainda, verão que os números de Soma, 12 começam com A, 12 com B,... 12 com E. O mesmo acontece com os números de Subtração. Logo, para qualquer combinação, a soma deste dígito será 0. O mesmo acontece com os outros dígitos. Logo, a soma sempre será zero. -----Original Message----- From: niski [mailto:fabio@niski.com] Sent: Tuesday, April 15, 2003 8:32 PM To: obm-l@mat.puc-rio.br Subject: [obm-l] um problema pessoal, atribuindo a cada letra um valor de 0 ou 1, qual é o valor maximo que a expressao abaixo pode ter? eimqu-djmqu-ehnqu+cjnqu+dhoqu-cioqu-eilru+ djlru+egnru-bjnru-dgoru+bioru+ehlsu-cjlsu- egmsu+bjmsu+cgosu-bhosu-dhltu+ciltu+dgmtu- bimtu-cgntu+bhntu-eimpv+djmpv+ehnpv-cjnpv- dhopv+ciopv+eikrv-djkrv-efnrv+ajnrv+dforv- aiorv-ehksv+cjksv+efmsv-ajmsv-cfosv+ahosv+ dhktv-ciktv-dfmtv+aimtv+cfntv-ahntv+eilpx- djlpx-egnpx+bjnpx+dgopx-biopx-eikqx+djkqx+ efnqx-ajnqx-dfoqx+aioqx+egksx-bjksx-eflsx+ ajlsx+bfosx-agosx-dgktx+biktx+dfltx-ailtx- bfntx+agntx-ehlpy+cjlpy+egmpy-bjmpy-cgopy+ bhopy+ehkqy-cjkqy-efmqy+ajmqy+cfoqy-ahoqy- egkry+bjkry+eflry-ajlry-bfory+agory+cgkty- bhkty-cflty+ahlty+bfmty-agmty+dhlpz-cilpz- dgmpz+bimpz+cgnpz-bhnpz-dhkqz+cikqz+dfmqz- aimqz-cfnqz+ahnqz+dgkrz-bikrz-dflrz+ailrz+ bfnrz-agnrz-cgksz+bhksz+cflsz-ahlsz-bfmsz+ agmsz obrigado Niski www.linux.ime.usp.br/~niski -- [about him:] It is rare to find learned men who are clean, do not stink and have a sense of humour. -Gottfried Whilhem Leibniz ========================================================================= Instruções para entrar na lista, sair da lista e usar a lista em http://www.mat.puc-rio.br/~nicolau/olimp/obm-l.html O administrador desta lista é ========================================================================= ========================================================================= Instruções para entrar na lista, sair da lista e usar a lista em http://www.mat.puc-rio.br/~nicolau/olimp/obm-l.html O administrador desta lista é ========================================================================= From owner-obm-l@sucuri.mat.puc-rio.br Wed Apr 16 10:42:29 2003 Return-Path: Received: (from majordom@localhost) by sucuri.mat.puc-rio.br (8.9.3/8.9.3) id KAA23259 for obm-l-MTTP; Wed, 16 Apr 2003 10:40:03 -0300 Received: from cmsrelay04.mx.net (cmsrelay04.mx.net [165.212.11.113]) by sucuri.mat.puc-rio.br (8.9.3/8.9.3) with SMTP id KAA23250 for ; Wed, 16 Apr 2003 10:39:58 -0300 Received: from uadvg131.cms.usa.net (HELO localhost) (165.212.11.131) by cmsoutbound.mx.net with SMTP; 16 Apr 2003 13:38:50 -0000 Received: from smtp.postoffice.net [165.212.8.7] by uadvg131.cms.usa.net (ASMTP/) via mtad (C8.MAIN.2.05) with ESMTP id 896HDPNMv0473M31; Wed, 16 Apr 2003 13:38:47 GMT Received: from 200.181.4.100 [200.181.4.100] by uwdvg007.cms.usa.net (USANET web-mailer CM.0402.5.2B); Wed, 16 Apr 2003 13:38:46 -0000 Date: Wed, 16 Apr 2003 10:38:46 -0300 From: Artur Costa Steiner To: Subject: Re: [Re: [[obm-l] REGRA DA CADEIA]] X-Mailer: USANET web-mailer (CM.0402.5.2B) Mime-Version: 1.0 Message-ID: <930HDPNMU8928S07.1050500326@uwdvg007.cms.usa.net> Content-Type: text/plain; charset=ISO-8859-1 Content-Transfer-Encoding: 8bit X-MIME-Autoconverted: from quoted-printable to 8bit by sucuri.mat.puc-rio.br id KAA23251 Sender: owner-obm-l@sucuri.mat.puc-rio.br Precedence: bulk Reply-To: obm-l@mat.puc-rio.br Wendel Scardua wrote: > > Não entendo essa passagem.....como que > > {f[g(x)]-f[g(a)]}/(x-a)={f[g(x)]-f[g(a)]}/[(g(x)-(g(a)].[g(x)-g(a)]/(x-a)? > > > > É só notar que está divindo e multiplicando um mesmo valor (que podia > ser qualquer coisa) : > > x / y = (x / z) * (z / y) Exatamente. E observando que, pelas hipoteses feitas, o denominador nao se anula. Artur ========================================================================= Instruções para entrar na lista, sair da lista e usar a lista em http://www.mat.puc-rio.br/~nicolau/olimp/obm-l.html O administrador desta lista é ========================================================================= From owner-obm-l@sucuri.mat.puc-rio.br Wed Apr 16 11:44:32 2003 Return-Path: Received: (from majordom@localhost) by sucuri.mat.puc-rio.br (8.9.3/8.9.3) id LAA25191 for obm-l-MTTP; Wed, 16 Apr 2003 11:40:44 -0300 Received: from ginsberg.uol.com.br (ginsberg.uol.com.br [200.221.29.48]) by sucuri.mat.puc-rio.br (8.9.3/8.9.3) with ESMTP id LAA25174 for ; Wed, 16 Apr 2003 11:40:25 -0300 Received: from gauss ([200.153.212.67]) by ginsberg.uol.com.br (8.9.1/8.9.1) with SMTP id LAA29636 for ; Wed, 16 Apr 2003 11:39:50 -0300 (BRT) Message-ID: <000901c30426$ac5adc50$7d07fea9@gauss> From: "Domingos Jr." To: References: <002701c302cd$0ede0b50$31619ec8@gauss> Subject: Re: [obm-l] automorfismo Date: Wed, 16 Apr 2003 11:44:23 -0300 MIME-Version: 1.0 Content-Type: text/plain; charset="iso-8859-1" Content-Transfer-Encoding: 8bit X-Priority: 3 X-MSMail-Priority: Normal X-Mailer: Microsoft Outlook Express 6.00.2800.1106 X-MimeOLE: Produced By Microsoft MimeOLE V6.00.2800.1106 Sender: owner-obm-l@sucuri.mat.puc-rio.br Precedence: bulk Reply-To: obm-l@mat.puc-rio.br é isso mesmo, ou f = 1 ou a característica de E é 2 (caso em que a + a = b + b = 0 pode ocorrer mesmo com a != b). dá pra resolver esse problema usando o teorema de Dedekind: tma: se F1 e F2 são corpos, e f1, f2, ..., fn um conjunto de homomorfismos (distintos), f1, ..., fn são linearmente independentes em F2, ou seja, se a1, ..., an são tais que a1f1 + a2f2 + ... an.fn = 0, a1 = a2 = ... = an = 0 para o problema, basta ver que se f² != 1, { 1, f, f², f³ } são homomorfismos (automorfismos, de fato) distintos, logo são linearmente independentes, mas f(a) + f³(a) - a - f²(a) = 0 é uma combinação linear desses homomorfismos sendo que nem todos os coeficientes são nulos, uma contradição. f² = 1. [ ]'s > Seja f um automorfismo em um corpo E, se f^4 = 1, e > f(a) + f³(a) = a + f²(a) para todo a pertencendo a E, > mostre que f²= 1. > > > Vejam o que eu fiz.... > > Se f != 1 existe um a tq. f(a) = b != a > b + f²(b) = a + f(b) > se f²(b) = b, f(f(b)) = b, logo f(b) = a pois f é um automorfismo > logo > b + b = a + a > dá pra ver que isso tá errado... > prop. distributiva > a(b + b) = ab + ab > b(a + a) = ba + ba > prop. comutativa > ab + ab = ba + ba > logo > a(b + b) = b(a + a), como a + a = b + b != 0 > a = b (lei do cancelamento) > absurdo... > > temos então que se f != 1, f² != 1 > > É isso mesmo? Temos que provar que na verdade f é a identidade??? > > [ ]'s > > ========================================================================= > Instruções para entrar na lista, sair da lista e usar a lista em > http://www.mat.puc-rio.br/~nicolau/olimp/obm-l.html > O administrador desta lista é > ========================================================================= ========================================================================= Instruções para entrar na lista, sair da lista e usar a lista em http://www.mat.puc-rio.br/~nicolau/olimp/obm-l.html O administrador desta lista é ========================================================================= From owner-obm-l@sucuri.mat.puc-rio.br Wed Apr 16 14:14:34 2003 Return-Path: Received: (from majordom@localhost) by sucuri.mat.puc-rio.br (8.9.3/8.9.3) id OAA28206 for obm-l-MTTP; Wed, 16 Apr 2003 14:09:47 -0300 Received: from paiol.terra.com.br (paiol.terra.com.br [200.176.3.18]) by sucuri.mat.puc-rio.br (8.9.3/8.9.3) with ESMTP id OAA28201 for ; Wed, 16 Apr 2003 14:09:43 -0300 Received: from una.terra.com.br (una.terra.com.br [200.176.3.32]) by paiol.terra.com.br (Postfix) with ESMTP id 40C5287E89 for ; Wed, 16 Apr 2003 14:09:12 -0300 (BRT) Received: from oem (150172.cps.virtua.com.br [200.174.150.172]) (authenticated user euraul) by una.terra.com.br (Postfix) with ESMTP id 1263E2F00EF for ; Wed, 16 Apr 2003 14:09:10 -0300 (BRT) Message-ID: <000e01c3043a$132f7f40$ac96aec8@soc.virtua.com.br> From: "Raul" To: Subject: [obm-l] Sistema Date: Wed, 16 Apr 2003 14:03:12 -0300 MIME-Version: 1.0 Content-Type: multipart/alternative; boundary="----=_NextPart_000_000B_01C30420.EB22D920" X-Priority: 3 X-MSMail-Priority: Normal X-Mailer: Microsoft Outlook Express 5.00.2615.200 X-MimeOLE: Produced By Microsoft MimeOLE V5.00.2615.200 Sender: owner-obm-l@sucuri.mat.puc-rio.br Precedence: bulk Reply-To: obm-l@mat.puc-rio.br This is a multi-part message in MIME format. ------=_NextPart_000_000B_01C30420.EB22D920 Content-Type: text/plain; charset="iso-8859-1" Content-Transfer-Encoding: quoted-printable Ol=E1, queria saber como demonstrar que a =FAnica solu=E7=E3o = positiva do seguinte sistema =E9 x=3Dy=3Dz=3D1. x+y^2+z^3=3D3 y+z^2+x^3=3D3 z+x^^2+y^3=3D3 Obrigado a todos que puderem colaborar. Raul ------=_NextPart_000_000B_01C30420.EB22D920 Content-Type: text/html; charset="iso-8859-1" Content-Transfer-Encoding: quoted-printable
    Ol=E1, queria saber = como=20 demonstrar que a =FAnica solu=E7=E3o positiva do seguinte sistema =E9=20 x=3Dy=3Dz=3D1.
    = x+y^2+z^3=3D3
    = y+z^2+x^3=3D3
    = z+x^^2+y^3=3D3
    Obrigado a todos que = puderem=20 colaborar.
       =20 Raul
------=_NextPart_000_000B_01C30420.EB22D920-- ========================================================================= Instruções para entrar na lista, sair da lista e usar a lista em http://www.mat.puc-rio.br/~nicolau/olimp/obm-l.html O administrador desta lista é ========================================================================= From owner-obm-l@sucuri.mat.puc-rio.br Wed Apr 16 14:41:03 2003 Return-Path: Received: (from majordom@localhost) by sucuri.mat.puc-rio.br (8.9.3/8.9.3) id OAA28976 for obm-l-MTTP; Wed, 16 Apr 2003 14:38:19 -0300 Received: from zeus.hotlink.com.br (zeus.hotlink.com.br [200.249.243.250]) by sucuri.mat.puc-rio.br (8.9.3/8.9.3) with SMTP id OAA28958 for ; Wed, 16 Apr 2003 14:38:12 -0300 Received: (qmail 4153 invoked by uid 504); 16 Apr 2003 17:35:52 -0000 Received: from veloz-221-183.hotlink.com.br (HELO pureza) (200.164.221.183) by 0 with SMTP; 16 Apr 2003 17:35:51 -0000 Message-ID: <007501c3043e$ea749c20$0e00a8c0@hotlink.com.br> From: "gabriel" To: References: <590HDooKj1664S20.1050419370@uwdvg007.cms.usa.net> <20030416043622.42259.qmail@web13705.mail.yahoo.com> <20030416100912.GA21143@linux.ime.usp.br> Subject: [obm-l] riemann Date: Wed, 16 Apr 2003 14:37:53 -0300 MIME-Version: 1.0 Content-Type: text/plain; charset="iso-8859-1" Content-Transfer-Encoding: 8bit X-Priority: 3 X-MSMail-Priority: Normal X-Mailer: Microsoft Outlook Express 6.00.2800.1106 X-MimeOLE: Produced By Microsoft MimeOLE V6.00.2800.1106 Sender: owner-obm-l@sucuri.mat.puc-rio.br Precedence: bulk Reply-To: obm-l@mat.puc-rio.br ALguem sabe dizer se a hipotese de Riemann foi realmente provada?E a conjectura de Poincare? Agradeço desde ja, Gabriel Guedes ========================================================================= Instruções para entrar na lista, sair da lista e usar a lista em http://www.mat.puc-rio.br/~nicolau/olimp/obm-l.html O administrador desta lista é ========================================================================= From owner-obm-l@sucuri.mat.puc-rio.br Wed Apr 16 16:38:53 2003 Return-Path: Received: (from majordom@localhost) by sucuri.mat.puc-rio.br (8.9.3/8.9.3) id QAA32181 for obm-l-MTTP; Wed, 16 Apr 2003 16:36:31 -0300 Received: from paiol.terra.com.br (paiol.terra.com.br [200.176.3.18]) by sucuri.mat.puc-rio.br (8.9.3/8.9.3) with ESMTP id QAA32177 for ; Wed, 16 Apr 2003 16:36:26 -0300 Received: from botucatu.terra.com.br (botucatu.terra.com.br [200.176.3.78]) by paiol.terra.com.br (Postfix) with ESMTP id 9A23487F8E for ; Wed, 16 Apr 2003 16:35:51 -0300 (BRT) Received: from nt (RJ231083.user.veloxzone.com.br [200.165.231.83]) (authenticated user ensr) by botucatu.terra.com.br (Postfix) with ESMTP id 2E42329C2E4 for ; Wed, 16 Apr 2003 16:35:51 -0300 (BRT) Message-ID: <014b01c3044f$46e8df60$5400a8c0@ensrbr> From: "Luis Lopes" To: References: Subject: Re: [obm-l] Fibonacci Date: Wed, 16 Apr 2003 16:35:02 -0300 MIME-Version: 1.0 Content-Type: text/plain; charset="iso-8859-1" Content-Transfer-Encoding: 8bit X-Priority: 3 X-MSMail-Priority: Normal X-Mailer: Microsoft Outlook Express 5.50.4807.1700 X-MimeOLE: Produced By Microsoft MimeOLE V5.50.4807.1700 Sender: owner-obm-l@sucuri.mat.puc-rio.br Precedence: bulk Reply-To: obm-l@mat.puc-rio.br Sauda,c~oes, Oi Claudio, Depois de 3 dias sem o Terra soh vai dar pra comentar uma msg hoje. > Mesmo o problema de se achar: > S = SOMA(n>=0) 1/F(2^n) > esta' longe de ser trivial. Concordo. Mas tenho uma solução elegante. > Acho que a formula: F(2k) = [F(k+1) + F(k-1)]*F(k) > deve entrar em algum lugar na demonstracao e, > de algum jeito, a restricao as potencias de 2 deve > fazer aparecer alguma PG cuja soma eh S. Mais ou menos. Continue a ler. > Eu sei que S = 4 - A, onde A = (1 + raiz(5))/2, ou seja, > S = (7 - raiz(5))/2. É verdade. Mas podemos encontrar também S_n. Seja S_n = \sum_{i=0}^n 1 / F_{2^i}. Para calcular S_n e depois S comece mostrando por indução que F_{k-1}F_m - F_kF_{m-1} = (-1)^k F_{m-k} para m >= k. (*) (e F_0 = 0 lembrando uma outra mensagem) Feito isso, coloque k=n e m=2n em (*). Obtemos F_{n-1}F_{2n} - F_nF_{2n-1} = (-1)^n F_n. Assim, 1/F_{2n} = F_{n-1}/F_n - F_{2n-1}/F_{2n} para n par. (**) Logo, para n>= 2 e usando (**), S_n = 1/F_1 + 1/F_2 + (F_1/F_2 - F_3/F_4) + (F_3/F_4 - F_7/F_8) + .... + (F_{2^{n-1}-1}/F_{2^{n-1}} - F_{2^n-1}/F_{2^n}). Que se "telescopia" a S_n = 3 - F_{2^n-1}/F_{2^n}. E como lim F_{n-1}/F_n = (sqrt5 - 1)/2 , S = 3 - (sqrt5 - 1)/2 = (7 - sqrt5)/2. []'s Luís -----Mensagem Original----- De: "Claudio Buffara" Para: Enviada em: sábado, 12 de abril de 2003 15:41 Assunto: Re: [obm-l] Fibonacci > Oi, Marcio: > > Mesmo o problema de se achar: > S = SOMA(n>=0) 1/F(2^n) > esta' longe de ser trivial. > > Eu sei que S = 4 - A, onde A = (1 + raiz(5))/2, ou seja, > S = (7 - raiz(5))/2. > > Acho que a formula: F(2k) = [F(k+1) + F(k-1)]*F(k) deve entrar em algum > lugar na demonstracao e, de algum jeito, a restricao as potencias de 2 deve > fazer aparecer alguma PG cuja soma eh S. > > Um abraco, > Claudio. > ========================================================================= Instruções para entrar na lista, sair da lista e usar a lista em http://www.mat.puc-rio.br/~nicolau/olimp/obm-l.html O administrador desta lista é ========================================================================= From owner-obm-l@sucuri.mat.puc-rio.br Wed Apr 16 16:54:22 2003 Return-Path: Received: (from majordom@localhost) by sucuri.mat.puc-rio.br (8.9.3/8.9.3) id QAA32474 for obm-l-MTTP; Wed, 16 Apr 2003 16:52:43 -0300 Received: from birosca.ime.usp.br (birosca.ime.usp.br [143.107.45.59]) by sucuri.mat.puc-rio.br (8.9.3/8.9.3) with SMTP id QAA32470 for ; Wed, 16 Apr 2003 16:52:38 -0300 Received: (qmail 26157 invoked from network); 16 Apr 2003 19:51:53 -0000 Received: from fradim.ime.usp.br (143.107.45.37) by birosca.ime.usp.br with SMTP; 16 Apr 2003 19:51:53 -0000 Received: (qmail 29882 invoked by uid 217); 16 Apr 2003 19:52:05 -0000 Date: Wed, 16 Apr 2003 16:52:05 -0300 (EST) From: Manuel Valentim Pera X-Sender: mane@fradim To: obm-l@mat.puc-rio.br Subject: Re: [obm-l] Sistema In-Reply-To: <000e01c3043a$132f7f40$ac96aec8@soc.virtua.com.br> Message-ID: MIME-Version: 1.0 Content-Type: TEXT/PLAIN; charset=ISO-8859-1 Content-Transfer-Encoding: 8bit X-MIME-Autoconverted: from QUOTED-PRINTABLE to 8bit by sucuri.mat.puc-rio.br id QAA32471 Sender: owner-obm-l@sucuri.mat.puc-rio.br Precedence: bulk Reply-To: obm-l@mat.puc-rio.br Boa tarde. Quanto `a pergunta: > Olá, queria saber como demonstrar que a única solução positiva do seguinte sistema é x=y=z=1. > x+y^2+z^3=3 > y+z^2+x^3=3 > z+x^^2+y^3=3 E' mais ou menos sabido que se a<1 entso a^n e' decrescente e se a>1 entao a^n e' estritamente crescente. Suponha que (x,y,z) e' uma terna de numeros estritamente positivos, resolve o sistema acima e uma das coordenadas e' estritamente maior do que 1, digamos, z>1 (as outras possibilidades sao tratadas analogamente). Ha' tres situacoes possiveis A) x<1 e y>=1. B) x>=1 e y<1 C) X<=1 e y<=1. Se A) veja que y<=y^2, z^2 ========================================================================= From owner-obm-l@sucuri.mat.puc-rio.br Wed Apr 16 17:24:53 2003 Return-Path: Received: (from majordom@localhost) by sucuri.mat.puc-rio.br (8.9.3/8.9.3) id RAA00982 for obm-l-MTTP; Wed, 16 Apr 2003 17:23:27 -0300 Received: from web12902.mail.yahoo.com (web12902.mail.yahoo.com [216.136.174.69]) by sucuri.mat.puc-rio.br (8.9.3/8.9.3) with SMTP id RAA00978 for ; Wed, 16 Apr 2003 17:23:23 -0300 Message-ID: <20030416202252.40002.qmail@web12902.mail.yahoo.com> Received: from [200.144.43.151] by web12902.mail.yahoo.com via HTTP; Wed, 16 Apr 2003 17:22:52 ART Date: Wed, 16 Apr 2003 17:22:52 -0300 (ART) From: =?iso-8859-1?q?Johann=20Peter=20Gustav=20Lejeune=20Dirichlet?= Subject: [obm-l] Sobre as olimpiadas ao redor do mundo(e um certo DEOLIVEIRASOU...) To: obm-l@mat.puc-rio.br MIME-Version: 1.0 Content-Type: multipart/alternative; boundary="0-990580002-1050524572=:39272" Content-Transfer-Encoding: 8bit Sender: owner-obm-l@sucuri.mat.puc-rio.br Precedence: bulk Reply-To: obm-l@mat.puc-rio.br --0-990580002-1050524572=:39272 Content-Type: text/plain; charset=iso-8859-1 Content-Transfer-Encoding: 8bit Esse aqui e do Olimpiadas ao Redor do mundo e ja foi sem o menor sucesso disculido na lista.Bem,o problema finlandes era pra achar os n tais que n^2+2 dividia 2001n+2.A ideia e verificar que se a divide b entao a divide b^2.Aplica isso e usa linearidade ou d divide.So dou a deica e o resto que se d...virem!!! --------------------------------- Yahoo! Mail O melhor e-mail gratuito da internet: 6MB de espaço, antivírus, acesso POP3, filtro contra spam. --0-990580002-1050524572=:39272 Content-Type: text/html; charset=iso-8859-1 Content-Transfer-Encoding: 8bit
Esse aqui e do Olimpiadas ao Redor do mundo e ja foi sem o menor sucesso disculido na lista.Bem,o problema finlandes era pra achar os n tais que n^2+2 dividia 2001n+2.A ideia e verificar que se a divide b entao a divide b^2.Aplica isso e usa linearidade ou d divide.So dou a deica e o resto que se d...virem!!!



Yahoo! Mail
O melhor e-mail gratuito da internet: 6MB de espaço, antivírus, acesso POP3, filtro contra spam. --0-990580002-1050524572=:39272-- ========================================================================= Instruções para entrar na lista, sair da lista e usar a lista em http://www.mat.puc-rio.br/~nicolau/olimp/obm-l.html O administrador desta lista é ========================================================================= From owner-obm-l@sucuri.mat.puc-rio.br Wed Apr 16 17:30:53 2003 Return-Path: Received: (from majordom@localhost) by sucuri.mat.puc-rio.br (8.9.3/8.9.3) id RAA01203 for obm-l-MTTP; Wed, 16 Apr 2003 17:29:35 -0300 Received: from web12902.mail.yahoo.com (web12902.mail.yahoo.com [216.136.174.69]) by sucuri.mat.puc-rio.br (8.9.3/8.9.3) with SMTP id RAA01190 for ; Wed, 16 Apr 2003 17:29:29 -0300 Message-ID: <20030416202859.41018.qmail@web12902.mail.yahoo.com> Received: from [200.144.43.151] by web12902.mail.yahoo.com via HTTP; Wed, 16 Apr 2003 17:28:59 ART Date: Wed, 16 Apr 2003 17:28:59 -0300 (ART) From: =?iso-8859-1?q?Johann=20Peter=20Gustav=20Lejeune=20Dirichlet?= Subject: Re: [obm-l] Re:_[obm-l]_Re:_[obm-l]_Trigonometria_e_Sequências To: obm-l@mat.puc-rio.br In-Reply-To: <20030411230657.82083.qmail@web41509.mail.yahoo.com> MIME-Version: 1.0 Content-Type: multipart/alternative; boundary="0-803146043-1050524939=:40198" Content-Transfer-Encoding: 8bit Sender: owner-obm-l@sucuri.mat.puc-rio.br Precedence: bulk Reply-To: obm-l@mat.puc-rio.br --0-803146043-1050524939=:40198 Content-Type: text/plain; charset=iso-8859-1 Content-Transfer-Encoding: 8bit Problemas de sequencias de senos e cossenos da pra resolver ou com complexos ou com prostafereses espertas e induçao. Carlos Yuzo Shine wrote:Ambos os problemas podem ser resolvidos usando o fato de que cos x = (e^(ix) + e^(-ix))/2 sen x = (e^(ix) - e^(-ix))/(2i) (x em radianos) Para ver isso, verifique as expansões em polinômio de Taylor de e^x, sen x e cos x e verifique que e^(ix) = cos x + i*sen x e^(-ix) = cos x - i*sen x Veja que com isso o problema A5 vira uma soma de duas progressões geométricas. O problema B2 usa os fatos acima e a fatoração z^n - 1 = (z - 1)(z - w)(z - w^2)...(z - w^(n-1)), em que w = e^(2\pi/n) é uma raiz n-ésima primitiva da unidade. > A5. Sendo \cos(\theta) = 1 / \pi , calcule \sum_{n=0}^\infty \cos(n\theta) / 2^n . > B2. Para n >= 2, mostre que (produtório) > \sin(\pi / n) \sin(2\pi / n) ..... \sin[(n - 1)\pi / > n] = n / 2^{n-1} . > > []'s > Luís __________________________________________________ Do you Yahoo!? Yahoo! Tax Center - File online, calculators, forms, and more http://tax.yahoo.com ========================================================================= Instruções para entrar na lista, sair da lista e usar a lista em http://www.mat.puc-rio.br/~nicolau/olimp/obm-l.html O administrador desta lista é ========================================================================= --------------------------------- Yahoo! Mail O melhor e-mail gratuito da internet: 6MB de espaço, antivírus, acesso POP3, filtro contra spam. --0-803146043-1050524939=:40198 Content-Type: text/html; charset=iso-8859-1 Content-Transfer-Encoding: 8bit

Problemas de sequencias de senos e cossenos da pra resolver ou com complexos ou com prostafereses espertas e induçao.

 Carlos Yuzo Shine <cyshine@yahoo.com> wrote:

Ambos os problemas podem ser resolvidos usando o fato
de que
cos x = (e^(ix) + e^(-ix))/2
sen x = (e^(ix) - e^(-ix))/(2i)
(x em radianos)

Para ver isso, verifique as expansões em polinômio de
Taylor de e^x, sen x e cos x e verifique que
e^(ix) = cos x + i*sen x
e^(-ix) = cos x - i*sen x

Veja que com isso o problema A5 vira uma soma de duas
progressões geométricas.

O problema B2 usa os fatos acima e a fatoração
z^n - 1 = (z - 1)(z - w)(z - w^2)...(z - w^(n-1)),
em que w = e^(2\pi/n) é uma raiz n-ésima primitiva da
unidade.

> A5. Sendo \cos(\theta) = 1 / \pi , calcule
\sum_{n=0}^\infty \cos(n\theta) / 2^n .

> B2. Para n >= 2, mostre que (produtório)
> \sin(\pi / n) \sin(2\pi / n) ..... \sin[(n - 1)\pi /
> n] = n / 2^{n-1} .
>
> []'s
> Luís


__________________________________________________
Do you Yahoo!?
Yahoo! Tax Center - File online, calculators, forms, and more
http://tax.yahoo.com
=========================================================================
Instruções para entrar na lista, sair da lista e usar a lista em
http://www.mat.puc-rio.br/~nicolau/olimp/obm-l.html
O administrador desta lista é
=========================================================================



Yahoo! Mail
O melhor e-mail gratuito da internet: 6MB de espaço, antivírus, acesso POP3, filtro contra spam. --0-803146043-1050524939=:40198-- ========================================================================= Instruções para entrar na lista, sair da lista e usar a lista em http://www.mat.puc-rio.br/~nicolau/olimp/obm-l.html O administrador desta lista é ========================================================================= From owner-obm-l@sucuri.mat.puc-rio.br Wed Apr 16 17:48:51 2003 Return-Path: Received: (from majordom@localhost) by sucuri.mat.puc-rio.br (8.9.3/8.9.3) id RAA02119 for obm-l-MTTP; Wed, 16 Apr 2003 17:47:08 -0300 Received: from ginsberg.uol.com.br (ginsberg.uol.com.br [200.221.29.48]) by sucuri.mat.puc-rio.br (8.9.3/8.9.3) with ESMTP id RAA02114 for ; Wed, 16 Apr 2003 17:47:05 -0300 Received: from gauss ([200.158.97.182]) by ginsberg.uol.com.br (8.9.1/8.9.1) with SMTP id RAA08183 for ; Wed, 16 Apr 2003 17:46:33 -0300 (BRT) Message-ID: <003001c30459$e7950790$b6619ec8@gauss> From: "Domingos Jr." To: References: <200304152000.h3FK0tU01273@Gauss.impa.br> <000901c303b7$ceac4240$08a8a5c8@epq.ime.eb.br> Subject: Re: [obm-l] Desigualdade legal Date: Wed, 16 Apr 2003 17:51:07 -0300 MIME-Version: 1.0 Content-Type: text/plain; charset="iso-8859-1" Content-Transfer-Encoding: 8bit X-Priority: 3 X-MSMail-Priority: Normal X-Mailer: Microsoft Outlook Express 6.00.2800.1106 X-MimeOLE: Produced By Microsoft MimeOLE V6.00.2800.1106 Sender: owner-obm-l@sucuri.mat.puc-rio.br Precedence: bulk Reply-To: obm-l@mat.puc-rio.br > Segue pro pessoal tentar uma desigualdade legal que apareceu em outra lista: > Se a,b,c sao reais positivos e abc <=1, mostre que: > a/c + b/a + c/b >= a + b + c talvez saia dessa maneira: sejam x e y reais positivos tais que b = ax c = ay temos então a/ay + ax/a + ay/ax >= a(1 + x + y) 1/y + x + y/x >= a(1 + x + y) no entanto abc = a³xy <= 1 => a <= (xy)^(-1/3) sendo assim, se provarmos que 1/y + x + y/x >= (xy)^(-1/3)(1 + x + y) teremos provado a desigualdade original. acho que se fizermos f(x, y) = 1/y + x + y/x - (xy)^(-1/3)(1 + x + y) talvez se usarmos os métodos de cálculo para determinar qual o valor mínimo de f e verificarmos que esse valor é >= 0 a resposta saia. [ ]'s ========================================================================= Instruções para entrar na lista, sair da lista e usar a lista em http://www.mat.puc-rio.br/~nicolau/olimp/obm-l.html O administrador desta lista é ========================================================================= From owner-obm-l@sucuri.mat.puc-rio.br Wed Apr 16 18:13:41 2003 Return-Path: Received: (from majordom@localhost) by sucuri.mat.puc-rio.br (8.9.3/8.9.3) id SAA03226 for obm-l-MTTP; Wed, 16 Apr 2003 18:12:09 -0300 Received: (from nicolau@localhost) by sucuri.mat.puc-rio.br (8.9.3/8.9.3) id SAA03221 for obm-l@mat.puc-rio.br; Wed, 16 Apr 2003 18:12:09 -0300 Date: Wed, 16 Apr 2003 18:12:08 -0300 From: "Nicolau C. Saldanha" To: obm-l@mat.puc-rio.br Subject: Re: [obm-l] riemann Message-ID: <20030416181208.B2662@sucuri.mat.puc-rio.br> References: <590HDooKj1664S20.1050419370@uwdvg007.cms.usa.net> <20030416043622.42259.qmail@web13705.mail.yahoo.com> <20030416100912.GA21143@linux.ime.usp.br> <007501c3043e$ea749c20$0e00a8c0@hotlink.com.br> Mime-Version: 1.0 Content-Type: text/plain; charset=iso-8859-1 Content-Disposition: inline Content-Transfer-Encoding: 8bit User-Agent: Mutt/1.2.5i In-Reply-To: <007501c3043e$ea749c20$0e00a8c0@hotlink.com.br>; from gabriel@hotlink.com.br on Wed, Apr 16, 2003 at 02:37:53PM -0300 Sender: owner-obm-l@sucuri.mat.puc-rio.br Precedence: bulk Reply-To: obm-l@mat.puc-rio.br On Wed, Apr 16, 2003 at 02:37:53PM -0300, gabriel wrote: > ALguem sabe dizer se a hipotese de Riemann foi realmente provada?E a > conjectura de Poincare? > Agradeço desde ja, > Gabriel Guedes Eu tenho conhecimento de uns preprint escritos por físicos que alegam ter demonstrado a hipótese de Riemann mas acho que esses preprints não devem ser levados muito a sério. O caso da conjectura de Poincaré é diferente. Existem alguns matemáticos sérios que afirmam ter demonstrado a conjectura. Uma notícia recente deste tipo é http://www.nytimes.com/2003/04/15/science/15MATH.html mas existem outros. Mesmo assim é muito cedo para dizer que a conjectura foi provada. O trabalho não foi nem escrito. Considera-se que um teorema desta importância foi provado só depois do trabalho ter sido publicado e lido por um número significativo de especialistas da área. []s, N. ========================================================================= Instruções para entrar na lista, sair da lista e usar a lista em http://www.mat.puc-rio.br/~nicolau/olimp/obm-l.html O administrador desta lista é ========================================================================= From owner-obm-l@sucuri.mat.puc-rio.br Wed Apr 16 18:15:45 2003 Return-Path: Received: (from majordom@localhost) by sucuri.mat.puc-rio.br (8.9.3/8.9.3) id SAA03362 for obm-l-MTTP; Wed, 16 Apr 2003 18:14:27 -0300 Received: from web12901.mail.yahoo.com (web12901.mail.yahoo.com [216.136.174.68]) by sucuri.mat.puc-rio.br (8.9.3/8.9.3) with SMTP id SAA03353 for ; Wed, 16 Apr 2003 18:14:20 -0300 Message-ID: <20030416211349.78622.qmail@web12901.mail.yahoo.com> Received: from [200.144.43.151] by web12901.mail.yahoo.com via HTTP; Wed, 16 Apr 2003 18:13:49 ART Date: Wed, 16 Apr 2003 18:13:49 -0300 (ART) From: =?iso-8859-1?q?Johann=20Peter=20Gustav=20Lejeune=20Dirichlet?= Subject: [obm-l] Sobre o Torneio das Cidades e a Eureka! To: obm-l@mat.puc-rio.br, gugu@impa.br MIME-Version: 1.0 Content-Type: multipart/alternative; boundary="0-1824483615-1050527629=:78481" Content-Transfer-Encoding: 8bit Sender: owner-obm-l@sucuri.mat.puc-rio.br Precedence: bulk Reply-To: obm-l@mat.puc-rio.br --0-1824483615-1050527629=:78481 Content-Type: text/plain; charset=iso-8859-1 Content-Transfer-Encoding: 8bit Ola Gugu!!!!Ola turma da lista!!!!Estava pensando na possibilidade de publicar soluçoes dos leitores da Eureka para as provas do torneio das Cidades,que foi publicada na Eureka 14.Ja consegui fazer tres.O que voces acham?Te mais!!!!!!!Ass.:Johann --------------------------------- Yahoo! Mail O melhor e-mail gratuito da internet: 6MB de espaço, antivírus, acesso POP3, filtro contra spam. --0-1824483615-1050527629=:78481 Content-Type: text/html; charset=iso-8859-1 Content-Transfer-Encoding: 8bit
Ola Gugu!!!!Ola turma da lista!!!!Estava pensando na possibilidade de publicar soluçoes dos leitores da Eureka para as provas do torneio das Cidades,que foi publicada na Eureka 14.Ja consegui fazer tres.O que voces acham?
Te mais!!!!!!!Ass.:Johann



Yahoo! Mail
O melhor e-mail gratuito da internet: 6MB de espaço, antivírus, acesso POP3, filtro contra spam. --0-1824483615-1050527629=:78481-- ========================================================================= Instruções para entrar na lista, sair da lista e usar a lista em http://www.mat.puc-rio.br/~nicolau/olimp/obm-l.html O administrador desta lista é ========================================================================= From owner-obm-l@sucuri.mat.puc-rio.br Wed Apr 16 18:27:44 2003 Return-Path: Received: (from majordom@localhost) by sucuri.mat.puc-rio.br (8.9.3/8.9.3) id SAA04158 for obm-l-MTTP; Wed, 16 Apr 2003 18:26:22 -0300 Received: from Euler.impa.br (euler.impa.br [147.65.1.3]) by sucuri.mat.puc-rio.br (8.9.3/8.9.3) with ESMTP id SAA04154 for ; Wed, 16 Apr 2003 18:26:18 -0300 Received: from Gauss.impa.br (Gauss [147.65.4.1]) by Euler.impa.br (8.11.6p2/8.11.6) with ESMTP id h3GLPl012787 for ; Wed, 16 Apr 2003 18:25:47 -0300 (EST) From: Carlos Gustavo Tamm de Araujo Moreira Received: by Gauss.impa.br (8.11.6p2) id h3GLPdm13594; Wed, 16 Apr 2003 18:25:39 -0300 (EST) Message-Id: <200304162125.h3GLPdm13594@Gauss.impa.br> Subject: Re: [obm-l] Problemas em Aberto II To: obm-l@mat.puc-rio.br Date: Wed, 16 Apr 2003 18:25:39 -0300 (EST) In-Reply-To: <045901c2de8a$b6ac6640$3300c57d@bovespa.com> from "=?Windows-1252?Q?Cl=E1udio_\=28Pr=E1tica\=29?=" at Feb 27, 3 03:04:44 pm X-Mailer: ELM [version 2.4 PL25] MIME-Version: 1.0 Content-Type: text/plain; charset=US-ASCII Content-Transfer-Encoding: 7bit Sender: owner-obm-l@sucuri.mat.puc-rio.br Precedence: bulk Reply-To: obm-l@mat.puc-rio.br > >19. Suponha que os n=FAmeros da forma 2^x * 3^y (x, y: inteiros n=E3o = >negativos) s=E3o colocados em ordem crescente. Prove que existem termos = >consecutivos - digamos 2^a * 3^b e 2^c * 3^d - tais que um dos = >n=FAmeros | a - c | ou | b - d | =E9 t=E3o grande quanto se queira. > Ha' pelo menos (n+1)^2 numeros dessa forma menores ou iguais a 6^n (por exemplo os numeros da forma 2^a.3^b com 0<=a,b<=n). Assim, se r > 1 e' a menor razao entre dois numeros consecutivos dessa forma que sao menores que 6^n, temos r^((n+1)^2-1) <= 6^n, donde r <= 6^(1/(n+2)), que tende a 6^0=1 quando n cresce. Se |a-c| e |b-d| fossem limitados para os pares de numeros consecutivos da forma 2^a.3^b e 2^c.3^d, a razao 2^(c-a).3^(d-b) entre dois termos consecutivos pertenceria a um conjunto finito de numeros maiores que 1, e nao poderia aproximar-se arbitrariamente de 1, absurdo. >************* > >20. Duas de An=E1lise Real: > >A) Prove que se f:{a, b) -> R =E9 cont=EDnua em c em (a,b) e lim x-> c >f'(x) =3D L, ent=E3o f'(c) =3D L. A partir da=ED, conclua que derivadas = >jamais >apresentam descontinuidades do tipo salto. Conclua tamb=E9m que se f' = >=E9 >monot=F4nica em um intervalo I, ent=E3o f'=E9 cont=EDnua em I. > >B) Suponhamos que f seja diferenci=E1vel em R e seja k<>0. Mostre que: >B.1) se k>0, ent=E3o lim x -> infinito f'(x) + k f(x) =3D L, L em R, = >implica >que lim x-> infinito f('x) =3D 0 e lim x-> infinito f(x) =3D L/k >B.2) se k<0, ent=E3o lim x-> infinito f'(x) + k f(x) =3D L, L em R, s=F3 = >=E9 >poss=EDvel se lim x-> e^(kx) f(x) =3D 0, caso em que temos tamb=E9m lim = >x-> >infinito f('x) =3D 0 e lim x-> infinito f(x) =3D L/k >sugest=E3o : defina h(x) =3D e^(kx) f(x) g(x) =3D e^(kx) . Logo, f(x) = >=3D >h(x)/g(x). Use L'Hopital. > > A) A primeira afirmacao segue imediatamente do teorema do valor medio: Se f e' continua em [x,x+h] e derivavel em (x,x+h) entao (f(x+h)-f(x))/h=f'(c), para algum c em (x,x+h). As outras afirmacoes seguem do teorema do valor intermediario para derivadas: se f e' derivavel e s esta' entre f'(a) e f'(b) entao (mesmo que f' nao seja continua) existe c entre a e b com f'(c)=s. Para provar isso, seja g(x)=f(x)-sx. Temos g'(x)=f'(x)-s, donde g'(a)=f'(a)-s e g'(b)=f'(b)-s tem sinais opostos. Queremos mostrar que existe c em (a,b) com g'(c)=0. Suponha sem perda de generalidade que g'(a)>0 e g'(b)<0. Como g e' derivavel, e logo continua, em [a,b], existe c em [a,b] onde g(c) e' maximo. Como g'(a)>0, g(x)>g(a) para todo x um pouco maior que a, e como g'(b)<0, g(x)>g(b) para todo x um pouco menor que b, donde c deve pertencer a (a,b), e portanto g'(c)=0 (e f'(c)=s). B) Se k>0, e^(kx) tende a infinito quando x tende a infinito, e podemos usar a regra de l'Hopital para calcular lim(x->infinito)(f(x))=lim(x->infinito)((e^(kx).f(x))/e^(kx))= =lim(x->infinito)((e^(kx).f'(x)+k.e^(kx).f(x))/(k.e^(kx)))= =lim(x->infinito)((f'(x)+k.f(x))/k)=L/k. (E logo lim(x->infinito)(f'(x))=lim(x->infinito)(f'(x)+k.f(x))-k.lim(x->infinito)(f(x))=L-k.(L/k)=0) Por outro lado, se k<0 a situacao e' mais delicada, e a conclusao a que eu cheguei nao e' bem a do enunciado (de fato, se f(x)=e^(-kx)+L/k, vale lim(x->infinito)(f'(x)+k.f(x))=L, mas nao e' verdade que lim(x->infinito)(e^(kx).f(x))=0). O que e' verdade nesse caso e' que f(x)=c.e^(-kx)+h(x) (para algum c real), onde lim(x->infinito)(h(x))=L/k e lim(x->infinito)(h'(x))=0. Para mostrar isso, seja g(x)=e^(kx).f(x). Temos g'(x)=e^(kx).(k.f(x)+f'(x))=O(e^(kx)) pois k.f(x)+f'(x) converge. Assim, dados m < n naturais (grandes), |g(n)-g(m)|<= <=soma(j=1 ate' n-m)(|g(m+j)-g(m+j-1)|=O(soma(j=1 ate' n-m)(e^(k(m+j)))=O(e^(km)), donde (g(n)) e' uma sequencia de Cauchy, e portanto converge a um certo c (e alem disso nossas estimativas mostram que |g(m)-c|=O(e^(km))). Como |g(x)-g([x])|=O(e^(kx)), temos lim(x->infinito)(g(x))=c e |g(x)-c|=O(e^(kx)). Assim, h(x):=f(x)-c.e^(-kx)=e^(-kx).(g(x)-c)= =O(e^(-kx).e^(kx))=O(1), donde lim(x->infinito)(e^(kx).h(x))=lim(x->infinito)(e^(kx))=0, e podemos usar a regra de l'Hopital para calcular lim(x->infinito)(h(x))= =lim(x->infinito)((e^(kx).h(x))/e^(kx))= lim(x->infinito)((k.e^(kx).h(x)+e^(kx).h'(x))/(k.e^(kx)))= =lim(x->infinito)((k.h(x)+h'(x))/k)=L/k, pois h'(x)=f'(x)+k.c.e^(-kx), donde k.h(x)+h'(x)= =k.f(x)+f'(x), que tende a L quando x tende a infinito. Do mesmo jeito que antes, lim(x->infinito)(h'(x))=lim(x->infinito)(h'(x)+k.h(x))-k.lim(x->infinito)(h(x))=L-k.(L/k)=0. Abracos, Gugu ========================================================================= Instruções para entrar na lista, sair da lista e usar a lista em http://www.mat.puc-rio.br/~nicolau/olimp/obm-l.html O administrador desta lista é ========================================================================= From owner-obm-l@sucuri.mat.puc-rio.br Wed Apr 16 18:58:32 2003 Return-Path: Received: (from majordom@localhost) by sucuri.mat.puc-rio.br (8.9.3/8.9.3) id SAA05399 for obm-l-MTTP; Wed, 16 Apr 2003 18:56:36 -0300 Received: from zeus.hotlink.com.br (zeus.hotlink.com.br [200.249.243.250]) by sucuri.mat.puc-rio.br (8.9.3/8.9.3) with SMTP id SAA05395 for ; Wed, 16 Apr 2003 18:56:32 -0300 Received: (qmail 12266 invoked by uid 504); 16 Apr 2003 21:54:18 -0000 Received: from veloz-221-183.hotlink.com.br (HELO pureza) (200.164.221.183) by 0 with SMTP; 16 Apr 2003 21:54:17 -0000 Message-ID: <001a01c30463$05622f60$0e00a8c0@hotlink.com.br> From: "gabriel" To: References: <590HDooKj1664S20.1050419370@uwdvg007.cms.usa.net> <20030416043622.42259.qmail@web13705.mail.yahoo.com> <20030416100912.GA21143@linux.ime.usp.br> <007501c3043e$ea749c20$0e00a8c0@hotlink.com.br> <20030416181208.B2662@sucuri.mat.puc-rio.br> Subject: Re: [obm-l] riemann Date: Wed, 16 Apr 2003 18:56:22 -0300 MIME-Version: 1.0 Content-Type: multipart/alternative; boundary="----=_NextPart_000_0017_01C30449.DFB04220" X-Priority: 3 X-MSMail-Priority: Normal X-Mailer: Microsoft Outlook Express 6.00.2800.1106 X-MimeOLE: Produced By Microsoft MimeOLE V6.00.2800.1106 Sender: owner-obm-l@sucuri.mat.puc-rio.br Precedence: bulk Reply-To: obm-l@mat.puc-rio.br This is a multi-part message in MIME format. ------=_NextPart_000_0017_01C30449.DFB04220 Content-Type: text/plain; charset="iso-8859-1" Content-Transfer-Encoding: quoted-printable Caro Nicolau, So nao entendi uma coisa; porque estes preprints nao devem ser levados a = serios?? Fisicos nao resolvem problemas de matematica?? : ) Agrade=E7o desde ja, Gabriel Guedes =20 ----- Original Message -----=20 From: "Nicolau C. Saldanha" To: Sent: Wednesday, April 16, 2003 6:12 PM Subject: Re: [obm-l] riemann > On Wed, Apr 16, 2003 at 02:37:53PM -0300, gabriel wrote: > > ALguem sabe dizer se a hipotese de Riemann foi realmente provada?E a > > conjectura de Poincare? > > Agrade=E7o desde ja, > > Gabriel Guedes >=20 > Eu tenho conhecimento de uns preprint escritos por f=EDsicos que = alegam > ter demonstrado a hip=F3tese de Riemann mas acho que esses preprints > n=E3o devem ser levados muito a s=E9rio. >=20 > O caso da conjectura de Poincar=E9 =E9 diferente. Existem alguns = matem=E1ticos > s=E9rios que afirmam ter demonstrado a conjectura. Uma not=EDcia = recente > deste tipo =E9 > http://www.nytimes.com/2003/04/15/science/15MATH.html > mas existem outros. >=20 > Mesmo assim =E9 muito cedo para dizer que a conjectura foi provada. > O trabalho n=E3o foi nem escrito. Considera-se que um teorema desta > import=E2ncia foi provado s=F3 depois do trabalho ter sido publicado > e lido por um n=FAmero significativo de especialistas da =E1rea. >=20 > []s, N. > = =3D=3D=3D=3D=3D=3D=3D=3D=3D=3D=3D=3D=3D=3D=3D=3D=3D=3D=3D=3D=3D=3D=3D=3D=3D= =3D=3D=3D=3D=3D=3D=3D=3D=3D=3D=3D=3D=3D=3D=3D=3D=3D=3D=3D=3D=3D=3D=3D=3D=3D= =3D=3D=3D=3D=3D=3D=3D=3D=3D=3D=3D=3D=3D=3D=3D=3D=3D=3D=3D=3D=3D=3D=3D > Instru=E7=F5es para entrar na lista, sair da lista e usar a lista em > http://www.mat.puc-rio.br/~nicolau/olimp/obm-l.html > O administrador desta lista =E9 > = =3D=3D=3D=3D=3D=3D=3D=3D=3D=3D=3D=3D=3D=3D=3D=3D=3D=3D=3D=3D=3D=3D=3D=3D=3D= =3D=3D=3D=3D=3D=3D=3D=3D=3D=3D=3D=3D=3D=3D=3D=3D=3D=3D=3D=3D=3D=3D=3D=3D=3D= =3D=3D=3D=3D=3D=3D=3D=3D=3D=3D=3D=3D=3D=3D=3D=3D=3D=3D=3D=3D=3D=3D=3D > ------=_NextPart_000_0017_01C30449.DFB04220 Content-Type: text/html; charset="iso-8859-1" Content-Transfer-Encoding: quoted-printable
Caro Nicolau,
So nao entendi uma coisa; porque = estes=20 preprints nao devem ser levados a serios?? Fisicos nao resolvem = problemas=20 de matematica?? : )
Agrade=E7o desde ja,
Gabriel = Guedes
 
----- Original Message -----
From: "Nicolau C. Saldanha" = <nicolau@sucuri.mat.puc-rio.br>
To: <obm-l@mat.puc-rio.br>
Sent: Wednesday, April 16, 2003 6:12=20 PM
Subject: Re: [obm-l] = riemann

> On Wed, Apr 16, 2003 at 02:37:53PM -0300, gabriel = wrote:
>=20 > ALguem sabe dizer se a hipotese de Riemann foi realmente provada?E=20 a
> > conjectura de Poincare?
> > Agrade=E7o desde = ja,
>=20 > Gabriel Guedes
>
> Eu tenho conhecimento de uns = preprint=20 escritos por f=EDsicos que alegam
> ter demonstrado a hip=F3tese = de Riemann=20 mas acho que esses preprints
> n=E3o devem ser levados muito a=20 s=E9rio.
>
> O caso da conjectura de Poincar=E9 =E9 = diferente. Existem=20 alguns matem=E1ticos
> s=E9rios que afirmam ter demonstrado a = conjectura. Uma=20 not=EDcia recente
> deste tipo =E9
>
http://www.nytimes.com/2003/04/15/science/15MATH.html=
> mas existem outros.
>
> Mesmo = assim =E9 muito=20 cedo para dizer que a conjectura foi provada.
> O trabalho n=E3o = foi nem=20 escrito. Considera-se que um teorema desta
> import=E2ncia foi = provado s=F3=20 depois do trabalho ter sido publicado
> e lido por um n=FAmero = significativo=20 de especialistas da =E1rea.
>
> []s, N.
>=20 =3D=3D=3D=3D=3D=3D=3D=3D=3D=3D=3D=3D=3D=3D=3D=3D=3D=3D=3D=3D=3D=3D=3D=3D=3D= =3D=3D=3D=3D=3D=3D=3D=3D=3D=3D=3D=3D=3D=3D=3D=3D=3D=3D=3D=3D=3D=3D=3D=3D=3D= =3D=3D=3D=3D=3D=3D=3D=3D=3D=3D=3D=3D=3D=3D=3D=3D=3D=3D=3D=3D=3D=3D=3D
= >=20 Instru=E7=F5es para entrar na lista, sair da lista e usar a lista = em
>=20
http://www.mat.puc-rio.br/~nicolau/olimp/obm-l.html
> O administrador desta lista =E9 <
nicolau@mat.puc-rio.br>
>=20 =3D=3D=3D=3D=3D=3D=3D=3D=3D=3D=3D=3D=3D=3D=3D=3D=3D=3D=3D=3D=3D=3D=3D=3D=3D= =3D=3D=3D=3D=3D=3D=3D=3D=3D=3D=3D=3D=3D=3D=3D=3D=3D=3D=3D=3D=3D=3D=3D=3D=3D= =3D=3D=3D=3D=3D=3D=3D=3D=3D=3D=3D=3D=3D=3D=3D=3D=3D=3D=3D=3D=3D=3D=3D
= >=20
------=_NextPart_000_0017_01C30449.DFB04220-- ========================================================================= Instruções para entrar na lista, sair da lista e usar a lista em http://www.mat.puc-rio.br/~nicolau/olimp/obm-l.html O administrador desta lista é ========================================================================= From owner-obm-l@sucuri.mat.puc-rio.br Wed Apr 16 19:42:39 2003 Return-Path: Received: (from majordom@localhost) by sucuri.mat.puc-rio.br (8.9.3/8.9.3) id TAA06583 for obm-l-MTTP; Wed, 16 Apr 2003 19:41:01 -0300 Received: from imo-m03.mx.aol.com (imo-m03.mx.aol.com [64.12.136.6]) by sucuri.mat.puc-rio.br (8.9.3/8.9.3) with ESMTP id TAA06578 for ; Wed, 16 Apr 2003 19:40:57 -0300 From: DEOLIVEIRASOU@aol.com Received: from DEOLIVEIRASOU@aol.com by imo-m03.mx.aol.com (mail_out_v34.21.) id z.25.37797ad5 (25508) for ; Wed, 16 Apr 2003 18:40:22 -0400 (EDT) Message-ID: <25.37797ad5.2bcf35d5@aol.com> Date: Wed, 16 Apr 2003 18:40:21 EDT Subject: Re: [obm-l] Sobre as olimpiadas ao redor do mundo(e um certo DEOLIVEIRASOU...) To: obm-l@mat.puc-rio.br MIME-Version: 1.0 Content-Type: multipart/alternative; boundary="part1_25.37797ad5.2bcf35d5_boundary" X-Mailer: 7.0 for Windows sub 10501 Sender: owner-obm-l@sucuri.mat.puc-rio.br Precedence: bulk Reply-To: obm-l@mat.puc-rio.br --part1_25.37797ad5.2bcf35d5_boundary Content-Type: text/plain; charset="ISO-8859-1" Content-Transfer-Encoding: quoted-printable Valeu grande dirichlet....o Claudio pratica( grande Claudiao) me mandou a=20 solucao...de qualquer forma obrigado pela dica. Aproveitando a deixa , esse recado vai para o Claudio sobre o problema da=20 eureka( Olimpiadas ao redor do mundo. O enunciado segue na integra...meu=20 teclado esta com problemas , entao vou tentar nao ser ambiguo com o=20 enunciado... Problema------- Determine todos os restos possiveis da divisao do quadrado d= e=20 um numero primo com 120 por 120( Est=F6nia 2000). Vi uma ideia num problema=20 russo de 1940(47?). Fiz um esboco, mas ainda nao parei pra trabalhar nele...= . Antes que me esqueca, valeu a resolucao do problema dos primos da forma=20 101010.......101, Claudio. Crom --part1_25.37797ad5.2bcf35d5_boundary Content-Type: text/html; charset="ISO-8859-1" Content-Transfer-Encoding: quoted-printable Valeu grande dirichlet....o Claudio pratica( grande Cl= audiao) me mandou a solucao...de qualquer forma obrigado pela dica.
Aproveitando a deixa , esse recado vai para o Claudio sobre o problema da eu= reka( Olimpiadas ao redor do mundo. O enunciado segue na integra...meu tecla= do esta com problemas , entao vou tentar nao ser ambiguo com o enunciado...<= BR> Problema------- Determine todos os restos possiveis da divisao do quadrado d= e um numero primo com 120 por 120( Est=F6nia 2000). Vi uma ideia num problem= a russo de 1940(47?). Fiz um esboco, mas ainda nao parei pra trabalhar nele.= ...
  Antes que me esqueca, valeu a resolucao do problema dos primos da for= ma 101010.......101, Claudio.
        Crom
--part1_25.37797ad5.2bcf35d5_boundary-- ========================================================================= Instruções para entrar na lista, sair da lista e usar a lista em http://www.mat.puc-rio.br/~nicolau/olimp/obm-l.html O administrador desta lista é ========================================================================= From owner-obm-l@sucuri.mat.puc-rio.br Wed Apr 16 21:45:32 2003 Return-Path: Received: (from majordom@localhost) by sucuri.mat.puc-rio.br (8.9.3/8.9.3) id VAA09352 for obm-l-MTTP; Wed, 16 Apr 2003 21:43:52 -0300 Received: (from nicolau@localhost) by sucuri.mat.puc-rio.br (8.9.3/8.9.3) id VAA09347 for obm-l@mat.puc-rio.br; Wed, 16 Apr 2003 21:43:51 -0300 Date: Wed, 16 Apr 2003 21:43:51 -0300 From: "Nicolau C. Saldanha" To: obm-l@mat.puc-rio.br Subject: Re: [obm-l] riemann Message-ID: <20030416214351.A9315@sucuri.mat.puc-rio.br> References: <590HDooKj1664S20.1050419370@uwdvg007.cms.usa.net> <20030416043622.42259.qmail@web13705.mail.yahoo.com> <20030416100912.GA21143@linux.ime.usp.br> <007501c3043e$ea749c20$0e00a8c0@hotlink.com.br> <20030416181208.B2662@sucuri.mat.puc-rio.br> <001a01c30463$05622f60$0e00a8c0@hotlink.com.br> Mime-Version: 1.0 Content-Type: text/plain; charset=iso-8859-1 Content-Disposition: inline Content-Transfer-Encoding: 8bit User-Agent: Mutt/1.2.5i In-Reply-To: <001a01c30463$05622f60$0e00a8c0@hotlink.com.br>; from gabriel@hotlink.com.br on Wed, Apr 16, 2003 at 06:56:22PM -0300 Sender: owner-obm-l@sucuri.mat.puc-rio.br Precedence: bulk Reply-To: obm-l@mat.puc-rio.br On Wed, Apr 16, 2003 at 06:56:22PM -0300, gabriel wrote: > Caro Nicolau, > So nao entendi uma coisa; porque estes preprints nao devem ser levados > a serios?? Fisicos nao resolvem problemas de matematica?? : ) Em muitos casos sim. Mas *estes* preprints em particular são escritos de maneira muito imprecisa demais. A pedido do Gugu segue abaixo o artigo do NYTimes. []s, N. ========================================================================== The New York Times April 15, 2003 Celebrated Math Problem Solved, Russian Reports By SARA ROBINSON A Russian mathematician is reporting that he has proved the Poincaré Conjecture, one of the most famous unsolved problems in mathematics. The mathematician, Dr. Grigori Perelman of the Steklov Institute of Mathematics of the Russian Academy of Sciences in St. Petersburg, is describing his work in a series of papers, not yet completed. It will be months before the proof can be thoroughly checked. But if true, it will verify a statement about three-dimensional objects that has haunted mathematicians for nearly a century, and its consequences will reverberate through geometry and physics. If his proof is accepted for publication in a refereed research journal and survives two years of scrutiny, Dr. Perelman could be eligible for a $1 million prize sponsored by the Clay Mathematics Institute in Cambridge, Mass., for solving what the institute identifies as one of the seven most important unsolved mathematics problems of the millennium. Rumors about Dr. Perelman's work have been circulating since November, when he posted the first of his papers reporting the result on an Internet preprint server. Last week at the Massachusetts Institute of Technology, he gave his first formal lectures on his work to a packed auditorium. Dr. Perelman will give another lecture series at the State University of New York at Stony Brook starting on Monday. Dr. Perelman declined to be interviewed, saying publicity would be premature. For two months, Dr. Tomasz S. Mrowka, a mathematician at M.I.T., has been attending a seminar on Dr. Perelman's work, which relies on ideas pioneered by another mathematician, Richard Hamilton. So far, Dr. Mrowka said, every time someone brings up an issue or objection, Dr. Perelman has a clear and succinct response. "It's not certain, but we're taking it very seriously," Dr. Mrowka said. "He's obviously thought about this stuff very hard for a long time, and it will be very hard to find any mistakes." Formulated by the French mathematician Henri Poincaré in 1904, the Poincaré Conjecture is a central question in topology, the study of the geometrical properties of objects that do not change when the object is stretched, twisted or shrunk. The hollow shell of the surface of the earth is what topologists would call a two-dimensional sphere. It has the property that every lasso of string encircling it can be pulled tight to one spot. On the surface of a doughnut, by contrast, a lasso passing through the hole in the center cannot be shrunk to a point without cutting through the surface. Since the 19th century, mathematicians have known that the sphere is the only bounded two-dimensional space with this property, but what about higher dimensions? The Poincaré Conjecture makes a corresponding statement about the three-dimensional sphere, a concept that is a stretch for the nonmathematician to visualize. It says, essentially, that the three-dimensional sphere is the only bounded three-dimensional space with no holes. "The hard part is how to tell globally what a space looks like when you can only see a little piece of it at a time," said Dr. Benson Farb, a professor of mathematics at the University of Chicago. "It was pretty reasonable to think the earth was flat." That conjecture is notorious for the many "solutions" that later proved false. Indeed, Poincaré himself demonstrated that his earliest version of his conjecture was wrong. Since then, dozens of mathematicians have asserted that they had proofs until experts found fatal flaws. Although many experts say they are excited and hopeful about Dr. Perelman's effort, they also urge caution, noting that not all of the proof has been written down and that even the most reliable researchers make mistakes. That was the case in 1993 with Dr. Andrew J. Wiles, the Princeton professor whose celebrated proof for Fermat's Last Theorem turned out to have a serious gap that was repaired after months of effort by Dr. Wiles and a former student, Dr. Richard Taylor. Dr. Perelman's results go well beyond a solution to the problem at hand, as did those of Dr. Wiles. Dr. Perelman's results say he has proved a much broader conjecture about the geometry of three-dimensional spaces made in the 1970's. The Poincaré Conjecture is but a small part of that. Dr. Perelman's personal story has parallels to that of Dr. Wiles, who, without confiding in his colleagues, worked alone in his attic on Fermat's Last Theorem. Though his early work has earned him a reputation as a brilliant mathematician, Dr. Perelman spent the last eight years sequestered in Russia, not publishing. In his paper posted in November, Dr. Perelman, now in his late 30's, thanks the Courant Institute at New York University, SUNY Stony Brook and the University of California at Berkeley, because his savings from visiting positions at those institutions helped support him in Russia. His papers say that he has proved what is known as the Geometrization Conjecture, a complete characterization of the geometry of three-dimensional spaces. Since the 19th century, mathematicians have known that a type of two-dimensional space called a manifold can be given a rigid geometric structure that looks the same everywhere. Mathematicians could list all the possible shapes for two-dimensional manifolds and explain how a creature living on the surface of one can tell what kind of space he is on. In the 1950's, however, a Russian mathematician proved that the problem was impossible to resolve in four dimensions and that even for three dimensions, the question looked hopelessly complex. In the early 1970's, Dr. William P. Thurston, a professor at the University of California at Davis, conjectured that three-dimensional manifolds are composed of many homogeneous pieces that can be put together only in prescribed ways and proved that in many cases his conjecture was correct. Dr. Thurston won a Fields Medal, the highest honor in mathematics, for his work. Dr. Perelman's work, if correct, would provide the final piece of a complete description of the structure of three-dimensional manifolds and, almost as an afterthought, would resolve Poincaré's famous question. Dr. Perelman's approach uses a technique known as the Ricci flow, devised by Dr. Hamilton, who is now at Columbia University. The Ricci flow is an averaging process used to smooth out the bumps of a manifold and make it look more uniform. Dr. Hamilton uses the Ricci flow to prove the Geometrization Conjecture in some cases and outlined a general program of how it could be used to prove the Geometrization Conjecture in all cases. He ran into problems, however, coping with certain types of large lumps that tended to grow uncontrollably under the averaging process. "What Perelman has done is to figure out some new and interesting ways to tame these singularities," Dr. Mrowka said. "His work relies heavily on Hamilton's work but makes amazing new contributions to that program." If Dr. Perelman succeeds in resolving Poincaré, he will probably share the Clay Mathematics Institute Award with Dr. Hamilton, mathematicians said. Even if Dr. Perelman's work does not prove the Geometrization Conjecture, mathematicians said, it is clear that his work will make a substantial contribution to mathematics. "This is one of those happy circumstances where it's going to be fun no matter what," Dr. Mrowka said. "Either he's done it or he's made some really significant progress, and we're going to learn from it." ========================================================================= Instruções para entrar na lista, sair da lista e usar a lista em http://www.mat.puc-rio.br/~nicolau/olimp/obm-l.html O administrador desta lista é ========================================================================= From owner-obm-l@sucuri.mat.puc-rio.br Wed Apr 16 21:59:20 2003 Return-Path: Received: (from majordom@localhost) by sucuri.mat.puc-rio.br (8.9.3/8.9.3) id VAA11219 for obm-l-MTTP; Wed, 16 Apr 2003 21:57:45 -0300 Received: (from nicolau@localhost) by sucuri.mat.puc-rio.br (8.9.3/8.9.3) id VAA11213 for obm-l@sucuri.mat.puc-rio.br; Wed, 16 Apr 2003 21:57:44 -0300 Date: Wed, 16 Apr 2003 21:57:44 -0300 From: "Nicolau C. Saldanha" To: obm-l@sucuri.mat.puc-rio.br Subject: [obm-l] Polinomio irredutivel Message-ID: <20030416215744.A11097@sucuri.mat.puc-rio.br> Mime-Version: 1.0 Content-Type: text/plain; charset=iso-8859-1 Content-Disposition: inline Content-Transfer-Encoding: 8bit User-Agent: Mutt/1.2.5i Sender: owner-obm-l@sucuri.mat.puc-rio.br Precedence: bulk Reply-To: obm-l@mat.puc-rio.br O Gugu perguntou se existe um polinomio irredutivel sobre Z com raiz em todo Z/(p). A resposta parece ser não, a solução é do João Pedro. Está aqui: http://www.mat.puc-rio.br/~nicolau/joaopedro []s, N. ========================================================================= Instruções para entrar na lista, sair da lista e usar a lista em http://www.mat.puc-rio.br/~nicolau/olimp/obm-l.html O administrador desta lista é ========================================================================= From owner-obm-l@sucuri.mat.puc-rio.br Wed Apr 16 23:31:22 2003 Return-Path: Received: (from majordom@localhost) by sucuri.mat.puc-rio.br (8.9.3/8.9.3) id XAA13035 for obm-l-MTTP; Wed, 16 Apr 2003 23:28:06 -0300 Received: from acsxe0.ac.brahma (smtp01.ambev.com.br [200.212.186.163]) by sucuri.mat.puc-rio.br (8.9.3/8.9.3) with ESMTP id XAA13030 for ; Wed, 16 Apr 2003 23:28:00 -0300 Received: from acsnx2.ac.brahma (acsnx2 [10.100.3.89]) by acsxe0.ac.brahma (AIX4.3/8.9.3/8.9.3) with ESMTP id XAA52432 for ; Wed, 16 Apr 2003 23:27:14 -0300 Received: by acsnx2.brahma with Internet Mail Service (5.5.2653.19) id <20X57FXJ>; Wed, 16 Apr 2003 23:26:48 -0300 Message-ID: <27BD56F8640DD711A4320006295078E50500BE@pssnx1.brahma> From: "Ricardo de Moraes (PS)" To: "'obm-l@mat.puc-rio.br'" Subject: [obm-l] =?ISO-8859-1?Q?Combinat=F3ria?= Date: Wed, 16 Apr 2003 23:26:36 -0300 MIME-Version: 1.0 X-Mailer: Internet Mail Service (5.5.2653.19) Content-Type: multipart/alternative; boundary="----_=_NextPart_001_01C30488.C534EFB0" Sender: owner-obm-l@sucuri.mat.puc-rio.br Precedence: bulk Reply-To: obm-l@mat.puc-rio.br This message is in MIME format. Since your mail reader does not understand this format, some or all of this message may not be legible. ------_=_NextPart_001_01C30488.C534EFB0 Content-Type: text/plain; charset="ISO-8859-1" Content-Transfer-Encoding: quoted-printable Seguema alguns problemas de combinatoria. =20 =20 1. De quantas formas podemos agrupar 20 pessoas em 4 grupos de 5 = pessoas? =20 2. Quantas s=E3o as solu=E7oes poss=EDveis para: a) x + y + z + w + t < =3D 13 b) x + y + z + w =3D 22 , com x > 2 e y >=3D 1 =20 3. Quantos s=E3o os anagramas de INCONSISTENTE que: =20 A a) Come=E7=E3o com I; b) Tem CO juntos nesta ordem; c) Terminam por E. B Quantos anagramas pertemcem a exatamente 2 destes conjuntos? (a, = b, c) =20 4. Quantas s=E3o as trajet=F3rias poss=EDveis de (0,0) =E0 (18,6): a) No total: b) Tocando a reta y=3D -2: c) N=E3o tocando a reta y=3D -3. ------_=_NextPart_001_01C30488.C534EFB0 Content-Type: text/html; charset="ISO-8859-1" Content-Transfer-Encoding: quoted-printable
Seguema alguns=20 problemas de combinatoria.
 
 
1. De = quantas formas=20 podemos agrupar 20 pessoas em 4 grupos de 5 = pessoas?
 
2. = Quantas s=E3o as=20 solu=E7oes poss=EDveis para:
    a) x + y + z + w + t < =3D 13
    b) x + y + z + w =3D 22 , com x > 2 e y >=3D = 1
 
3. = Quantos s=E3o os=20 anagramas de INCONSISTENTE que:
 
    A=20 a) Come=E7=E3o com I;
       b) Tem CO juntos nesta = ordem;
       c) Terminam por E.
    B=20 Quantos anagramas pertemcem a exatamente 2 destes conjuntos? (a,=20 b, c)
 
4. = Quantas s=E3o as=20 trajet=F3rias poss=EDveis de (0,0) =E0 (18,6):
    a) No total:
    b) Tocando a reta y=3D -2:
   =20 c) N=E3o tocando a reta y=3D -3.
------_=_NextPart_001_01C30488.C534EFB0-- ========================================================================= Instruções para entrar na lista, sair da lista e usar a lista em http://www.mat.puc-rio.br/~nicolau/olimp/obm-l.html O administrador desta lista é ========================================================================= From owner-obm-l@sucuri.mat.puc-rio.br Wed Apr 16 23:34:20 2003 Return-Path: Received: (from majordom@localhost) by sucuri.mat.puc-rio.br (8.9.3/8.9.3) id XAA13080 for obm-l-MTTP; Wed, 16 Apr 2003 23:31:48 -0300 Received: from paiol.terra.com.br (paiol.terra.com.br [200.176.3.18]) by sucuri.mat.puc-rio.br (8.9.3/8.9.3) with ESMTP id XAA13073 for ; Wed, 16 Apr 2003 23:31:44 -0300 Received: from altamira.terra.com.br (altamira.terra.com.br [200.176.3.40]) by paiol.terra.com.br (Postfix) with ESMTP id 094748803F for ; Wed, 16 Apr 2003 23:31:09 -0300 (BRT) Received: from [200.177.188.57] (dl-nas7-sao-C8B1BC39.p001.terra.com.br [200.177.188.57]) by altamira.terra.com.br (Postfix) with ESMTP id AF7BF3DC0AE for ; Wed, 16 Apr 2003 23:31:06 -0300 (BRT) User-Agent: Microsoft-Outlook-Express-Macintosh-Edition/5.02.2022 Date: Sat, 02 Jan 1904 11:23:42 -0200 Subject: Re: [obm-l] Sobre as olimpiadas ao redor do mundo(e um certo DEOLIVEIRASOU...) From: Claudio Buffara To: Message-ID: <1F1BE.4C9%claudio.buffara@terra.com.br> In-Reply-To: <25.37797ad5.2bcf35d5@aol.com> Mime-version: 1.0 Content-type: multipart/alternative; boundary="MS_Mac_OE_127422_772307_MIME_Part" Sender: owner-obm-l@sucuri.mat.puc-rio.br Precedence: bulk Reply-To: obm-l@mat.puc-rio.br > This message is in MIME format. Since your mail reader does not understand this format, some or all of this message may not be legible. --MS_Mac_OE_127422_772307_MIME_Part Content-type: text/plain; charset="ISO-8859-1" Content-transfer-encoding: quoted-printable on 16.04.03 20:40, DEOLIVEIRASOU@aol.com at DEOLIVEIRASOU@aol.com wrote: Valeu grande dirichlet....o Claudio pratica( grande Claudiao) me mandou a solucao...de qualquer forma obrigado pela dica. Aproveitando a deixa , esse recado vai para o Claudio sobre o problema da eureka( Olimpiadas ao redor do mundo. O enunciado segue na integra...meu teclado esta com problemas , entao vou tentar nao ser ambiguo com o enunciado... Problema------- Determine todos os restos possiveis da divisao do quadrado de um numero primo com 120 por 120( Est=F6nia 2000). Vi uma ideia num problem= a russo de 1940(47?). Fiz um esboco, mas ainda nao parei pra trabalhar nele.... Antes que me esqueca, valeu a resolucao do problema dos primos da forma 101010.......101, Claudio. Crom=20 Oi, Crom: Agora eu entendi! O problema eh: Seja n um inteiro tal que mdc(n,120) =3D 1. Quais os restos possiveis da divisao de n^2 por 120? O algorimo da divisao diz que existem inteiros q e r tais que: n^2 =3D 120q + r com 0 <=3D r <=3D 119. Alem disso, temos que: mdc(n,120) =3D 1 <=3D=3D> mdc(n^2,120) =3D 1 Assim: mdc(n^2,120) =3D mdc(120q + r,120) =3D mdc(r,120) =3D 1. Logo, r limita-se aos Phi(120) =3D Phi(8*3*5) =3D (8-4)*(3-1)*(5-1) =3D 32 inteiros tais que 1 <=3D r <=3D 120 e mdc(r,120) =3D 1. A questao agora eh: variando-se n, todos os 32 valores de r sao de fato obtidos? A resposta eh nao. Eis o porque: A equacao n^2 =3D 120q + r eh equivalente a congruencia: n^2 =3D=3D r (mod 120), ou seja, r eh um resto quadratico (mod 120) Assim, a resposta do problema eh: r pertence a intersecao dos seguintes 3 conjuntos: A =3D {0,1,2,3,...,119} B =3D {x inteiros tais que mdc(x,120) =3D 1} C =3D {x inteiros tais que x eh um resto quadratico (mod 120)} A inter B inter C eh obtido tomando-se os 16 elementos de A inter B que sao menores do que 60, elevando-os ao quadrado e reduzindo-os (mod 120). Isso porque: mdc(x,120) =3D 1 <=3D=3D> mdc(120 - x,120) e x^2 =3D=3D (120-x)^2 (mod 120) Assim: A inter B inter {1,2,...,60} =3D {01,07,11,13, 17,19,23,29, 31,37,41,43, 47,49,53,59} Os quadrados correspondentes (reduzidos mod 120 serao): {01,49,01,49, 49,01,49,01, 01,49,01,49, 49,01,49,01} Ou seja, os restos possiveis sao 1 e 49. ***** Repare que a solucao acima, apesar de correta, segue a linha do "eu sou burro mas nao sou cego", ou seja, usa conceitos e ferramentas que sao mais ou menos obvios dado o enunciado (algoritmo da divisao - pois o enunciado fala em resto da divisao; propriedades basicas do mdc e da funcao Phi de Euler - pois n eh dito ser primo com 120; finalmente, a definicao de resto quadratico - pois o problema pede os restos de n ao quadrado - essa eh a parte do "eu nao sou cego") mas no final calcula no braco os quadrados de 1= 6 numeros e os reduz mod 120 (essa eh a parte do "eu sou burro") e chega num resultado aparentemente surpreendente: estes 16 quadrados sao congruentes apenas a 1 ou 49 (mod 120). Uma maneira mais sofisticada de se resolver o problema seria observar que: mdc(n,120) =3D 1 =3D=3D>=20 mdc(n,8*3*5) =3D 1 =3D=3D> mdc(n,8) =3D mdc(n,3) =3D mdc(n,5) =3D 1 No entanto: mdc(n,8) =3D 1 =3D=3D> n eh impar =3D=3D> n =3D=3D 1, 3, 5 ou 7 (mod 8) =3D=3D> n^2 =3D=3D 1 (mod 8) mdc(n,3) =3D 1 e o pequeno teorema de Fermat =3D=3D> n^2 =3D=3D 1 (mod 3) mdc(n,5) =3D 1 e o pequeno teorema de Fermat =3D=3D> n^4 =3D=3D 1 (mod 5) =3D=3D> n^2 =3D=3D 1 (mod 5) ou n^2 =3D=3D -1 (mod 5) Pelo teorema chines dos restos, existe uma unica solucao em n^2 (mod 120) para cada um dos dois sistemas de congruencias resultante: n^2 =3D=3D 1 (mod 8) n^2 =3D=3D 1 (mod 3) n^2 =3D=3D 1 (mod 5) ou n^2 =3D=3D 1 (mod 8) n^2 =3D=3D 1 (mod 3) n^2 =3D=3D -1 (mod 5) A solucao do primeiro eh obviamente n^2 =3D=3D 1 (mod 120) A solucao do segundo pode ser obtida da seguinte forma (solucao excessivamente detalhada para ilustrar o metodo): n^2 =3D=3D -1 (mod 5) =3D=3D> n^2 =3D -1 + 5a (a inteiro) =3D=3D> -1 + 5a =3D=3D 1 (mod 3) =3D=3D> 5a =3D=3D 2 (mod 3) =3D=3D> 2a =3D=3D 2 (mod 3) =3D=3D> a =3D=3D 1 (mod 3) =3D=3D> a =3D 1 + 3b (b inteiro) =3D=3D> n^2 =3D -1 + 5(1 + 3b) =3D=3D> n^2 =3D 4 + 15b =3D=3D> 4 + 15b =3D=3D 1 (mod 8) =3D=3D> 15b =3D=3D -3 (mod 8) =3D=3D> -b =3D=3D -3 (mod 8) =3D=3D> b =3D=3D 3 (mod 8) =3D=3D> b =3D 3 + 8c (c inteiro) =3D=3D> n^2 =3D 4 + 15(3 + 8c) =3D=3D> n^2 =3D 49 + 120c =3D=3D> n^2 =3D=3D 49 (mod 120) Ou seja, os restos possiveis sao 1 e 49. ***** A primeira solucao que me ocorreu foi a bracal. No entanto, depois de verificar que aqueles 16 quadrados eram congruentes a apenas dois numeros, eu passei a achar que deveria haver algo especial com quadrados modulo 120. Foi so entao que eu imaginei a segunda solucao. Acho que a medida em que a pessoa vai ficando mais experiente, as solucoes mais sofisticadas (e, espera-se, menos bracais) vao aparecedo com mais naturalidade. Conclusao: tambem em matematica, parece que vale 1% de inspiracao e 99% de transpiracao - quanto mais voce estuda e pratica, mais inteligente voce fica... Um abraco, Claudio. --MS_Mac_OE_127422_772307_MIME_Part Content-type: text/html; charset="ISO-8859-1" Content-transfer-encoding: quoted-printable Re: [obm-l] Sobre as olimpiadas ao redor do mundo(e um certo DEOLIVE= IRASOU...) on 16.04.03 20:40, DEOLIVEIRASOU@aol.com at DEOLIVEIRASOU@aol.com wrote:
Valeu grande dirichlet....o C= laudio pratica( grande Claudiao) me mandou a solucao...de qualquer forma obr= igado pela dica.
Aproveitando a deixa , esse recado vai para o Claudio sobre o problema da e= ureka( Olimpiadas ao redor do mundo. O enunciado segue na integra...meu tecl= ado esta com problemas , entao vou tentar nao ser ambiguo com o enunciado...=
Problema------- Determine todos os restos possiveis da divisao do quadrado = de um numero primo com 120 por 120( Est=F6nia 2000). Vi uma ideia num problema= russo de 1940(47?). Fiz um esboco, mas ainda nao parei pra trabalhar nele..= ..
 Antes que me esqueca, valeu a resolucao do problema dos primos da fo= rma 101010.......101, Claudio.
       Crom


Oi, Crom:

Agora eu entendi! O problema eh:
Seja n um inteiro tal que mdc(n,120) =3D 1.
Quais os restos possiveis da divisao de n^2 por 120?

O algorimo da divisao diz que existem inteiros q e r tais que:
n^2 =3D 120q + r   com   0 <=3D r <=3D 119.

Alem disso, temos que:
mdc(n,120) =3D 1 <=3D=3D> mdc(n^2,120) =3D 1

Assim:
mdc(n^2,120) =3D mdc(120q + r,120) =3D mdc(r,120) =3D 1.

Logo, r limita-se aos Phi(120) =3D Phi(8*3*5) =3D (8-4)*(3-1)*(5-1) =3D 32 inteir= os tais que 1 <=3D r <=3D 120 e mdc(r,120) =3D 1.

A questao agora eh: variando-se n, todos os 32 valores de r sao de fato obt= idos?

A resposta eh nao. Eis o porque:

A equacao n^2 =3D 120q + r eh equivalente a congruencia:
n^2 =3D=3D r (mod 120), ou seja, r eh um resto quadratico (mod 120)

Assim, a resposta do problema eh: r pertence a intersecao dos seguintes 3 c= onjuntos:
A =3D {0,1,2,3,...,119}
B =3D {x inteiros tais que mdc(x,120) =3D 1}
C =3D {x inteiros tais que x eh um resto quadratico (mod 120)}

A inter B inter C eh obtido tomando-se os 16 elementos de A inter B que sao= menores do que 60, elevando-os ao quadrado e reduzindo-os (mod 120).

Isso porque:
mdc(x,120) =3D 1 <=3D=3D> mdc(120 - x,120)
e
x^2 =3D=3D (120-x)^2 (mod 120)

Assim:
A inter B inter {1,2,...,60} =3D
{01,07,11,13,   17,19,23,29,   31,37,41,43,   = ;47,49,53,59}

Os quadrados correspondentes (reduzidos mod 120 serao):
{01,49,01,49,   49,01,49,01,   01,49,01,49,   = ;49,01,49,01}

Ou seja, os restos possiveis sao 1 e 49.

*****

Repare que a solucao acima, apesar de correta, segue a linha do "eu so= u burro mas nao sou cego", ou seja, usa conceitos e ferramentas que sao= mais ou menos obvios dado o enunciado  (algoritmo da divisao - pois o = enunciado fala em resto da divisao; propriedades basicas do mdc e da funcao = Phi de Euler - pois n eh dito ser primo com 120; finalmente, a definicao de = resto quadratico - pois o problema pede os restos de n ao quadrado - essa eh= a parte do "eu nao sou cego") mas no final calcula no braco os qu= adrados de 16 numeros e os reduz mod 120 (essa eh a parte do "eu sou bu= rro") e chega num resultado aparentemente surpreendente: estes 16  = ;quadrados sao congruentes apenas a 1 ou 49 (mod 120).

Uma maneira mais sofisticada de se resolver o problema seria observar que:<= BR> mdc(n,120) =3D 1 =3D=3D>
mdc(n,8*3*5) =3D 1 =3D=3D>
mdc(n,8) =3D mdc(n,3) =3D mdc(n,5) =3D 1

No entanto:
mdc(n,8) =3D 1 =3D=3D>
n eh impar =3D=3D>
n =3D=3D 1, 3, 5 ou 7 (mod 8) =3D=3D>
n^2 =3D=3D 1 (mod 8)

mdc(n,3) =3D 1 e o pequeno teorema de Fermat =3D=3D>
n^2 =3D=3D 1 (mod 3)

mdc(n,5) =3D 1 e o pequeno teorema de Fermat =3D=3D>
n^4 =3D=3D 1 (mod 5) =3D=3D>
n^2 =3D=3D 1 (mod 5)   ou   n^2 =3D=3D -1 (mod 5)

Pelo teorema chines dos restos, existe uma unica solucao em n^2 (mod 120) p= ara cada um dos dois sistemas de congruencias resultante:
n^2 =3D=3D 1 (mod 8)
n^2 =3D=3D 1 (mod 3)
n^2 =3D=3D 1 (mod 5)

ou

n^2 =3D=3D 1 (mod 8)
n^2 =3D=3D 1 (mod 3)
n^2 =3D=3D -1 (mod 5)

A solucao do primeiro eh obviamente n^2 =3D=3D 1 (mod 120)

A solucao do segundo pode ser obtida da seguinte forma (solucao excessivame= nte detalhada para ilustrar o metodo):
n^2 =3D=3D -1 (mod 5) =3D=3D>
n^2 =3D -1 + 5a (a inteiro) =3D=3D>

-1 + 5a =3D=3D 1 (mod 3) =3D=3D>
5a =3D=3D 2 (mod 3) =3D=3D>
2a =3D=3D 2 (mod 3) =3D=3D>
a =3D=3D 1 (mod 3) =3D=3D>
a =3D 1 + 3b (b inteiro) =3D=3D>

n^2 =3D -1 + 5(1 + 3b) =3D=3D>
n^2 =3D 4 + 15b =3D=3D>

4 + 15b =3D=3D 1 (mod 8) =3D=3D>
15b =3D=3D -3 (mod 8) =3D=3D>
-b =3D=3D -3 (mod 8) =3D=3D>
b =3D=3D 3 (mod 8) =3D=3D>
b =3D 3 + 8c (c inteiro) =3D=3D>

n^2 =3D 4 + 15(3 + 8c) =3D=3D>
n^2 =3D 49 + 120c =3D=3D>
n^2 =3D=3D 49 (mod 120)

Ou seja, os restos possiveis sao 1 e 49.

*****

A primeira solucao que me ocorreu foi a bracal. No entanto, depois de verif= icar que aqueles 16 quadrados eram congruentes a apenas dois numeros, eu pas= sei a achar que deveria haver algo especial com quadrados modulo 120. Foi so= entao que eu imaginei a segunda solucao. Acho que a medida em que a pessoa = vai ficando mais experiente, as solucoes mais sofisticadas (e, espera-se, me= nos bracais) vao aparecedo com mais naturalidade.

Conclusao: tambem em matematica, parece que vale 1% de inspiracao e 99% de = transpiracao - quanto mais voce estuda e pratica, mais inteligente voce fica= ...

Um abraco,
Claudio. --MS_Mac_OE_127422_772307_MIME_Part-- ========================================================================= Instruções para entrar na lista, sair da lista e usar a lista em http://www.mat.puc-rio.br/~nicolau/olimp/obm-l.html O administrador desta lista é ========================================================================= From owner-obm-l@sucuri.mat.puc-rio.br Wed Apr 16 23:42:36 2003 Return-Path: Received: (from majordom@localhost) by sucuri.mat.puc-rio.br (8.9.3/8.9.3) id XAA13311 for obm-l-MTTP; Wed, 16 Apr 2003 23:39:57 -0300 Received: from saks.bol.com.br (saks.bol.com.br [200.221.24.16]) by sucuri.mat.puc-rio.br (8.9.3/8.9.3) with ESMTP id XAA13305 for ; Wed, 16 Apr 2003 23:39:49 -0300 Received: from y7q1a7 (200.221.24.191) by saks.bol.com.br (5.1.071) id 3E766E3A00952C01 for obm-l@mat.puc-rio.br; Wed, 16 Apr 2003 23:39:18 -0300 Message-ID: <004e01c3048a$51c886c0$9cb3fea9@y7q1a7> From: "Igor Correia Oliveira" To: References: <590HDooKj1664S20.1050419370@uwdvg007.cms.usa.net> <20030416043622.42259.qmail@web13705.mail.yahoo.com> <20030416100912.GA21143@linux.ime.usp.br> <007501c3043e$ea749c20$0e00a8c0@hotlink.com.br> <20030416181208.B2662@sucuri.mat.puc-rio.br> <001a01c30463$05622f60$0e00a8c0@hotlink.com.br> <20030416214351.A9315@sucuri.mat.puc-rio.br> Subject: [obm-l] =?iso-8859-1?Q?O_Retorno=2C_parte_-=5Be^=28Pi*i=29=5D._Os_matem=E1tic?= =?iso-8859-1?Q?os_nunca_morrem.?= Date: Wed, 16 Apr 2003 23:37:39 -0300 MIME-Version: 1.0 Content-Type: text/plain; charset="iso-8859-1" Content-Transfer-Encoding: 8bit X-Priority: 3 X-MSMail-Priority: Normal X-Mailer: Microsoft Outlook Express 5.00.2919.6600 X-MimeOLE: Produced By Microsoft MimeOLE V5.00.2919.6600 X-Sender-IP: 200.253.251.100 Sender: owner-obm-l@sucuri.mat.puc-rio.br Precedence: bulk Reply-To: obm-l@mat.puc-rio.br Saudações "terráquios", Retorno da minha "hibernação" da lista com as seguintes dúvidas: 1°) Como eu posso provar o Teorema de Erdös-Suranyi: "Todo inteiro positivo K poder ser escrito na forma K=±1² ±2² ±3² ... ±m² para uma escolha conveniente dos sinais + e -, e de m." (Amém!¡) 2°) Como eu provo que a distância do centro de gravidade do semi-círculo para o centro do círculo é (4*Pi)/(3*r). 3°) Encontre todos os inteiros positivos de 4 dígitos e cumprem a seguinte condição: o cubo da soma dos seus dígitos é igual ao quadrado do referido número. 4°) Exitem quantos livros Contest com questões internacionais variadas? Existe a de 2002? É possível fazer downloads deles pela internet como PDF ou arquivos de texto. Se, somente se, puder, qual é o site? 5°) *Apelo: Jovem humilde procura fundos de tela, figuras interressantes de matemática ou teoremas para fazer camisas com esse subsídio, poster ou material de divulgação. Quem possuir, envia-me por anexo. Ele será muito grato por uma colaboração caridosa e filantrópica. "Encontra-se oportunidade para fazer o mal cem vezes por dia e para fazer o bem uma vez por ano". - Voltaire ¡Grácias! Igor Correia Oliveira. ========================================================================= Instruções para entrar na lista, sair da lista e usar a lista em http://www.mat.puc-rio.br/~nicolau/olimp/obm-l.html O administrador desta lista é ========================================================================= From owner-obm-l@sucuri.mat.puc-rio.br Wed Apr 16 23:44:56 2003 Return-Path: Received: (from majordom@localhost) by sucuri.mat.puc-rio.br (8.9.3/8.9.3) id XAA13426 for obm-l-MTTP; Wed, 16 Apr 2003 23:42:20 -0300 Received: from krypton.hosting4u.net ([209.15.2.78]) by sucuri.mat.puc-rio.br (8.9.3/8.9.3) with ESMTP id XAA13415 for ; Wed, 16 Apr 2003 23:42:15 -0300 Received: from gargamel (200-158-201-146.dsl.telesp.net.br [200.158.201.146]) by krypton.hosting4u.net (Postfix) with ESMTP id C21439F574 for ; Wed, 16 Apr 2003 21:41:07 -0500 (CDT) Message-ID: <200304162344250770.06A92F93@smtp.watersportsbrazil.com> In-Reply-To: <3E9CCA69.3090706@centroin.com.br> References: <200304152254330590.01550783@smtp.watersportsbrazil.com> <3E9CCA69.3090706@centroin.com.br> X-Mailer: Calypso Version 3.30.00.00 (3) Date: Wed, 16 Apr 2003 23:44:25 -0300 From: "Ariel de Silvio" To: obm-l@mat.puc-rio.br Subject: Re: [obm-l] pontos colineares, cade o erro? Mime-Version: 1.0 Content-Type: text/plain; charset="ISO-8859-1" Content-Transfer-Encoding: 8bit X-MIME-Autoconverted: from quoted-printable to 8bit by sucuri.mat.puc-rio.br id XAA13417 Sender: owner-obm-l@sucuri.mat.puc-rio.br Precedence: bulk Reply-To: obm-l@mat.puc-rio.br Valeu pessoal, pelo menos dessa vez o erro não foi meu, é um exercicio do Cap 1 do V.7 do Fundamentos da Matematica Elementar do Gelson Iezzi... Ele errou... Valeu pela ajuda Ariel *********** MENSAGEM ORIGINAL *********** As 00:13 de 16/4/2003 A. C. Morgado escreveu: >Esquenta nao que deve ser de digitaçao no enunciado. Se o 5 fosse 6, os >pontos seriam colineares. > >Ariel de Silvio wrote: > >>Ola, >> >>mais uma vez estou cometendo algum erro de atencao (provavelmente) >> >>"Mostre que A(a,-3a), B(a+3,-3a-1) e C(a+5,-3a-2) sao colineares para >todo valor real de a" >> >>Mas nao foi o que encontrei, cheguei a considerar a=2, e meu resultado >nao foram pontos colineares... >> >>Se alguem puder me ajudar a encontrar meu erro, agradeco >> >>Tentei atraves da relacao: >>(xb-xa)/(xc-xb)=(yb-ya)/(yc-yb) >>cheguei a 3/2=1 (?!?) >> >>E tambem tentei atraves de determinates, mas cheguei a nada também >> >>E quando considerei a=2 na determinante, cheguei num resultado 27, que >deveria ser 0... >> >>Obrigado, >>Ariel >> >>========================================================================= >>Instruções para entrar na lista, sair da lista e usar a lista em >>http://www.mat.puc-rio.br/~nicolau/olimp/obm-l.html >>O administrador desta lista é >>========================================================================= >> >> >> >> > > >========================================================================= >Instruções para entrar na lista, sair da lista e usar a lista em >http://www.mat.puc-rio.br/~nicolau/olimp/obm-l.html >O administrador desta lista é >========================================================================= ========================================================================= Instruções para entrar na lista, sair da lista e usar a lista em http://www.mat.puc-rio.br/~nicolau/olimp/obm-l.html O administrador desta lista é ========================================================================= From owner-obm-l@sucuri.mat.puc-rio.br Thu Apr 17 00:53:57 2003 Return-Path: Received: (from majordom@localhost) by sucuri.mat.puc-rio.br (8.9.3/8.9.3) id AAA15698 for obm-l-MTTP; Thu, 17 Apr 2003 00:51:05 -0300 Received: from smtp-27.ig.com.br (smtp-27.ig.com.br [200.226.132.159]) by sucuri.mat.puc-rio.br (8.9.3/8.9.3) with SMTP id AAA15694 for ; Thu, 17 Apr 2003 00:51:02 -0300 Received: (qmail 6263 invoked from network); 17 Apr 2003 03:50:26 -0000 Received: from unknown (HELO xxxx) (200.165.171.96) by smtp-27.ig.com.br with SMTP; 17 Apr 2003 03:50:26 -0000 Message-ID: <004701c30495$0b6f7d40$60aba5c8@epq.ime.eb.br> From: "Marcio" To: References: <200304152000.h3FK0tU01273@Gauss.impa.br> <000901c303b7$ceac4240$08a8a5c8@epq.ime.eb.br> <003001c30459$e7950790$b6619ec8@gauss> Subject: Re: [obm-l] Desigualdade legal Date: Thu, 17 Apr 2003 00:54:11 -0300 MIME-Version: 1.0 Content-Type: text/plain; charset="iso-8859-1" Content-Transfer-Encoding: 8bit X-Priority: 3 X-MSMail-Priority: Normal X-Mailer: Microsoft Outlook Express 5.50.4133.2400 X-MimeOLE: Produced By Microsoft MimeOLE V5.50.4133.2400 Sender: owner-obm-l@sucuri.mat.puc-rio.br Precedence: bulk Reply-To: obm-l@mat.puc-rio.br É... Minha solução foi a seguinte: Voce comeca mostrando o caso em que abc = 1. Esse caso é típico. Uma solucao é fazer a=u/v, b=v/w, c=w/u e depois usar a desigualdade do rearranjo. Fica u^2/wv + v^2/uw + w^2/uv >= u/v + v/w + w/u sse u^3 + v^3 + w^3 >= u^2w + v^2u + w^2v (rearranjo com [u^2 v^2 w^2] e [u v w]. Agora, suponha abc < 1. Se todos os três forem menores que 1, a desigualdade segue direto de a/c >= a, ... Suponha então que c > 1 (os outros dois casos são tratados da mesma forma). Considere f(x) = x/c + b/x + c/b - x - b - c. Como 1/c - 1 < 0, essa função é decrescente, e portanto para 0 f(1/bc). Por outro lado, fazendo a = 1/bc sabemos que f(a) > 0 da primeira parte, e consequentemente f(x) > 0 sempre. t+ ----- Original Message ----- From: "Domingos Jr." To: Sent: Wednesday, April 16, 2003 5:51 PM Subject: Re: [obm-l] Desigualdade legal > > Segue pro pessoal tentar uma desigualdade legal que apareceu em outra > lista: > > Se a,b,c sao reais positivos e abc <=1, mostre que: > > a/c + b/a + c/b >= a + b + c > > > talvez saia dessa maneira: > sejam x e y reais positivos tais que > b = ax > c = ay > temos então > > a/ay + ax/a + ay/ax >= a(1 + x + y)9 > 1/y + x + y/x >= a(1 + x + y) > > no entanto abc = a³xy <= 1 => a <= (xy)^(-1/3) > sendo assim, se provarmos que > 1/y + x + y/x >= (xy)^(-1/3)(1 + x + y) > teremos provado a desigualdade original. > acho que se fizermos > f(x, y) = 1/y + x + y/x - (xy)^(-1/3)(1 + x + y) > talvez se usarmos os métodos de cálculo para determinar qual o valor mínimo > de f e verificarmos que esse valor é >= 0 a resposta saia. ========================================================================= Instruções para entrar na lista, sair da lista e usar a lista em http://www.mat.puc-rio.br/~nicolau/olimp/obm-l.html O administrador desta lista é ========================================================================= From owner-obm-l@sucuri.mat.puc-rio.br Thu Apr 17 01:19:41 2003 Return-Path: Received: (from majordom@localhost) by sucuri.mat.puc-rio.br (8.9.3/8.9.3) id BAA16497 for obm-l-MTTP; Thu, 17 Apr 2003 01:17:02 -0300 Received: from Euler.impa.br (euler.impa.br [147.65.1.3]) by sucuri.mat.puc-rio.br (8.9.3/8.9.3) with ESMTP id BAA16491 for ; Thu, 17 Apr 2003 01:17:00 -0300 Received: from Gauss.impa.br (Gauss [147.65.4.1]) by Euler.impa.br (8.11.6p2/8.11.6) with ESMTP id h3H4GT012130 for ; Thu, 17 Apr 2003 01:16:29 -0300 (EST) From: Carlos Gustavo Tamm de Araujo Moreira Received: by Gauss.impa.br (8.11.6p2) id h3H4GL303573; Thu, 17 Apr 2003 01:16:21 -0300 (EST) Message-Id: <200304170416.h3H4GL303573@Gauss.impa.br> Subject: Re: [obm-l] Desigualdade legal To: obm-l@mat.puc-rio.br Date: Thu, 17 Apr 2003 01:16:21 -0300 (EST) In-Reply-To: <004701c30495$0b6f7d40$60aba5c8@epq.ime.eb.br> from "Marcio" at Apr 17, 3 00:54:11 am X-Mailer: ELM [version 2.4 PL25] MIME-Version: 1.0 Content-Type: text/plain; charset=US-ASCII Content-Transfer-Encoding: 7bit Sender: owner-obm-l@sucuri.mat.puc-rio.br Precedence: bulk Reply-To: obm-l@mat.puc-rio.br Oi Marcio, Bacana, mas a maior parte da sua solucao trata do caso abc<1, e nao precisa: se abc<1, multiplicamos a,b e c por 1/raiz_cubica(abc). Ai o lado esquerdo nao muda, o direito aumenta e passamos a ter ABC=1. Abracos, Gugu > > É... > Minha solução foi a seguinte: > Voce comeca mostrando o caso em que abc = 1. Esse caso é típico. Uma >solucao é fazer a=u/v, b=v/w, c=w/u e depois usar a desigualdade do >rearranjo. >Fica u^2/wv + v^2/uw + w^2/uv >= u/v + v/w + w/u sse u^3 + v^3 + w^3 >= u^2w >+ v^2u + w^2v (rearranjo com [u^2 v^2 w^2] e [u v w]. > Agora, suponha abc < 1. Se todos os três forem menores que 1, a >desigualdade segue direto de a/c >= a, ... > Suponha então que c > 1 (os outros dois casos são tratados da mesma >forma). Considere f(x) = x/c + b/x + c/b - x - b - c. > Como 1/c - 1 < 0, essa função é decrescente, e portanto para 0temos temos f(x) > f(1/bc). > Por outro lado, fazendo a = 1/bc sabemos que f(a) > 0 da primeira parte, >e consequentemente f(x) > 0 sempre. > t+ > >----- Original Message ----- >From: "Domingos Jr." >To: >Sent: Wednesday, April 16, 2003 5:51 PM >Subject: Re: [obm-l] Desigualdade legal > > >> > Segue pro pessoal tentar uma desigualdade legal que apareceu em outra >> lista: >> > Se a,b,c sao reais positivos e abc <=1, mostre que: >> > a/c + b/a + c/b >= a + b + c >> >> >> talvez saia dessa maneira: >> sejam x e y reais positivos tais que >> b = ax >> c = ay >> temos então >> >> a/ay + ax/a + ay/ax >= a(1 + x + y)9 >> 1/y + x + y/x >= a(1 + x + y) >> >> no entanto abc = a³xy <= 1 => a <= (xy)^(-1/3) >> sendo assim, se provarmos que >> 1/y + x + y/x >= (xy)^(-1/3)(1 + x + y) >> teremos provado a desigualdade original. >> acho que se fizermos >> f(x, y) = 1/y + x + y/x - (xy)^(-1/3)(1 + x + y) >> talvez se usarmos os métodos de cálculo para determinar qual o valor >mínimo >> de f e verificarmos que esse valor é >= 0 a resposta saia. > > >========================================================================= >Instruções para entrar na lista, sair da lista e usar a lista em >http://www.mat.puc-rio.br/~nicolau/olimp/obm-l.html >O administrador desta lista é >========================================================================= ========================================================================= Instruções para entrar na lista, sair da lista e usar a lista em http://www.mat.puc-rio.br/~nicolau/olimp/obm-l.html O administrador desta lista é ========================================================================= From owner-obm-l@sucuri.mat.puc-rio.br Thu Apr 17 01:24:18 2003 Return-Path: Received: (from majordom@localhost) by sucuri.mat.puc-rio.br (8.9.3/8.9.3) id BAA16682 for obm-l-MTTP; Thu, 17 Apr 2003 01:21:46 -0300 Received: from Euler.impa.br (euler.impa.br [147.65.1.3]) by sucuri.mat.puc-rio.br (8.9.3/8.9.3) with ESMTP id BAA16676 for ; Thu, 17 Apr 2003 01:21:43 -0300 Received: from Gauss.impa.br (Gauss [147.65.4.1]) by Euler.impa.br (8.11.6p2/8.11.6) with ESMTP id h3H4LC012328 for ; Thu, 17 Apr 2003 01:21:12 -0300 (EST) From: Carlos Gustavo Tamm de Araujo Moreira Received: by Gauss.impa.br (8.11.6p2) id h3H4L4V04228; Thu, 17 Apr 2003 01:21:04 -0300 (EST) Message-Id: <200304170421.h3H4L4V04228@Gauss.impa.br> Subject: Re: [obm-l] Problemas em Aberto III To: obm-l@mat.puc-rio.br Date: Thu, 17 Apr 2003 01:21:04 -0300 (EST) In-Reply-To: <045f01c2de8a$b87abda0$3300c57d@bovespa.com> from "=?Windows-1252?Q?Cl=E1udio_\=28Pr=E1tica\=29?=" at Feb 27, 3 03:04:48 pm X-Mailer: ELM [version 2.4 PL25] MIME-Version: 1.0 Content-Type: text/plain; charset=US-ASCII Content-Transfer-Encoding: 7bit Sender: owner-obm-l@sucuri.mat.puc-rio.br Precedence: bulk Reply-To: obm-l@mat.puc-rio.br > >21) (CHINA) 10 pessoas chegaram a uma livraria. Sabe-se que : > >A) Todos as pessoas compraram livros de 3 disciplinas >B) Para quaisquer duas pessoas existe ao menos uma disciplina sobre a = >qual=20 >ambas compraram livros. > >Enumerando-se as disciplinas sobre as quais ha livros na livraria, seja = >M(i)=20 >o numero de pessoas que compraram livros da disciplina "i". Qual e o = >menor=20 >valor positivo possivel para o MAXIMO de {M(1), M(2), ... } ? > Cada pessoa comprou livros de 3 disciplinas. Se M(j)<=4 para todo j, dada uma pessoa, cada disciplina foi comprada por exatamente 3 outras pessoas (pois ha' 10 pessoas no total), e em particular a intersecao de duas pessoas e' exatamente uma disciplina, e M(j)=4 para todo j. Mas isso nao pode acontecer, pois nesse caso a soma dos M(j) seria multiplo de 4, e tambem deve ser igual a soma dos numeros de livros comprados pelas 10 pessoas, que e' 30. Assim, o menor valor possivel para o maximo dos M(j) e' 5, e isso e' possivel, como mostram os seguintes 10 conjuntos de disciplinas (obtidos escolhendo metade dos C(6,3)=20 subconjuntos de 3 elementos de um conjunto de 6 disciplinas de modo que dado A com tres elementos, ou A ou seu complementar aparece na lista): {1,2,3},{3,5,6},{1,2,5},{3,4,5},{2,3,4},{1,4,5},{1,4,6},{1,3,6},{2,4,6} e {2,5,6}. >************** > >26) Ache todos os pares (x,y) de inteiros positivos tais que >z=3D( 9*x^2 + 50*x*y + 9*y^2)^1/2 seja tamb=E9m um n=FAmero=20 >inteiro. > Vamos achar as solucoes racionais de 9x^2+50xy+9y^2=z^2 com y nao nulo (se y=0 entao z=3x ou z=-3x): dividindo por y^2 e fazendo u=x/y e v=z/y obtemos 9u^2+50u+9=v^2, ou (9u+25)^2-(3v)^2=644, ou (9u+25-3v)(9u+25+3v)=544. Sendo 9u+25-3v=2t, temos 9u+25+3v=272/t, e logo 9u+25=t+136/t e 3v=136/t-t. Fazendo t=p/q, obtemos finalmente z/y=v=(136q^2-p^2)/3pq e x/y=u=(p^2-25pq+136q^2)/9pq=(p-8q)(p-17q)/9pq. Assim, para a solucao inteira geral, teremos x=(k/d).(p-8q)(p-17q), y=(k/d).9pq e z=(k/d).3(136q^2-p^2), onde podemos tomar p e q inteiros com q>0 e mdc(p,q)=1, k e' um inteiro qualquer e d=mdc((p-8q)(p-17q),9pq)= =mdc(p+q,3)^2.mdc(p,136). Como queremos x e y inteiros positivos, devemos tomar k positivo, p positivo, p>17q ou p<8q. Por exemplo, a solucao x=4,y=5,z=37 e' obtida tomando p=5,q=1,d=9 e k=1, enquanto a solucao x=5,y=4,z=37 e' obtida tomando p=17, q=4, d=153 e k=1. Abracos, Gugu ========================================================================= Instruções para entrar na lista, sair da lista e usar a lista em http://www.mat.puc-rio.br/~nicolau/olimp/obm-l.html O administrador desta lista é ========================================================================= From owner-obm-l@sucuri.mat.puc-rio.br Thu Apr 17 02:00:43 2003 Return-Path: Received: (from majordom@localhost) by sucuri.mat.puc-rio.br (8.9.3/8.9.3) id BAA17967 for obm-l-MTTP; Thu, 17 Apr 2003 01:58:59 -0300 Received: from itaqui.terra.com.br (itaqui.terra.com.br [200.176.3.19]) by sucuri.mat.puc-rio.br (8.9.3/8.9.3) with ESMTP id BAA17953 for ; Thu, 17 Apr 2003 01:58:52 -0300 Received: from itaim.terra.com.br (itaim.terra.com.br [200.176.3.76]) by itaqui.terra.com.br (Postfix) with ESMTP id 25FAE3BC3CC for ; Thu, 17 Apr 2003 01:58:22 -0300 (BRT) Received: from [200.177.179.16] (dl-nas3-sao-C8B1B310.p001.terra.com.br [200.177.179.16]) by itaim.terra.com.br (Postfix) with ESMTP id 6DD582E006B for ; Thu, 17 Apr 2003 01:58:20 -0300 (BRT) User-Agent: Microsoft-Outlook-Express-Macintosh-Edition/5.02.2022 Date: Sat, 02 Jan 1904 13:50:52 -0200 Subject: Re: [obm-l] Fibonacci From: Claudio Buffara To: Message-ID: <20B43.4CD%claudio.buffara@terra.com.br> In-Reply-To: <014b01c3044f$46e8df60$5400a8c0@ensrbr> Mime-version: 1.0 Content-type: text/plain; charset="ISO-8859-1" Content-Transfer-Encoding: 8bit X-MIME-Autoconverted: from quoted-printable to 8bit by sucuri.mat.puc-rio.br id BAA17955 Sender: owner-obm-l@sucuri.mat.puc-rio.br Precedence: bulk Reply-To: obm-l@mat.puc-rio.br Oi, Luis: Realmente a solucao eh muito elegante, apesar do mega-coelho que voce tira da cartola - a identidade razoavelmente obscura: F(k-1)*F(m) - F(k)*F(m-1) = (-1)^k*F(m-k) com k = n e m = 2n. Assim, a solucao do Marcio, apesar de ser mais trabalhosa, me parece ser mais natural (no sentido de ser mais provavel de ser encontrada sem um conhecimento previo de identidades de Fibonacci). Mas valeu! Agora temos 2 solucoes bem diferentes pro mesmo problema. Quem sabe combinando as duas da pra obter alguma generalizacao... Um abraco, Claudio. > > para m >= k. (*) (e F_0 = 0 lembrando > uma outra mensagem) on 16.04.03 17:35, Luis Lopes at llopes@ensrbr.com.br wrote: > Sauda,c~oes, > > Oi Claudio, > > Depois de 3 dias sem o Terra soh vai dar > pra comentar uma msg hoje. > >> Mesmo o problema de se achar: >> S = SOMA(n>=0) 1/F(2^n) >> esta' longe de ser trivial. > Concordo. Mas tenho uma solução elegante. > >> Acho que a formula: F(2k) = [F(k+1) + F(k-1)]*F(k) >> deve entrar em algum lugar na demonstracao e, >> de algum jeito, a restricao as potencias de 2 deve >> fazer aparecer alguma PG cuja soma eh S. > Mais ou menos. Continue a ler. > >> Eu sei que S = 4 - A, onde A = (1 + raiz(5))/2, ou seja, >> S = (7 - raiz(5))/2. > É verdade. Mas podemos encontrar também S_n. > > Seja S_n = \sum_{i=0}^n 1 / F_{2^i}. > > Para calcular S_n e depois S comece mostrando > por indução que > > F_{k-1}F_m - F_kF_{m-1} = (-1)^k F_{m-k} > > para m >= k. (*) (e F_0 = 0 lembrando > uma outra mensagem) > > Feito isso, coloque k=n e m=2n em (*). > > Obtemos F_{n-1}F_{2n} - F_nF_{2n-1} = (-1)^n F_n. > Assim, > > 1/F_{2n} = F_{n-1}/F_n - F_{2n-1}/F_{2n} > para n par. (**) > > Logo, para n>= 2 e usando (**), > > S_n = 1/F_1 + 1/F_2 + (F_1/F_2 - F_3/F_4) + > (F_3/F_4 - F_7/F_8) + .... + > (F_{2^{n-1}-1}/F_{2^{n-1}} - F_{2^n-1}/F_{2^n}). > > Que se "telescopia" a S_n = 3 - F_{2^n-1}/F_{2^n}. > > E como lim F_{n-1}/F_n = (sqrt5 - 1)/2 , > > S = 3 - (sqrt5 - 1)/2 = (7 - sqrt5)/2. > > []'s > Luís > > > -----Mensagem Original----- > De: "Claudio Buffara" > Para: > Enviada em: sábado, 12 de abril de 2003 15:41 > Assunto: Re: [obm-l] Fibonacci > > >> Oi, Marcio: >> >> Mesmo o problema de se achar: >> S = SOMA(n>=0) 1/F(2^n) >> esta' longe de ser trivial. >> >> Eu sei que S = 4 - A, onde A = (1 + raiz(5))/2, ou seja, >> S = (7 - raiz(5))/2. >> >> Acho que a formula: F(2k) = [F(k+1) + F(k-1)]*F(k) deve entrar em algum >> lugar na demonstracao e, de algum jeito, a restricao as potencias de 2 > deve >> fazer aparecer alguma PG cuja soma eh S. >> >> Um abraco, >> Claudio. >> > ========================================================================= Instruções para entrar na lista, sair da lista e usar a lista em http://www.mat.puc-rio.br/~nicolau/olimp/obm-l.html O administrador desta lista é ========================================================================= From owner-obm-l@sucuri.mat.puc-rio.br Thu Apr 17 02:38:03 2003 Return-Path: Received: (from majordom@localhost) by sucuri.mat.puc-rio.br (8.9.3/8.9.3) id CAA19350 for obm-l-MTTP; Thu, 17 Apr 2003 02:36:42 -0300 Received: from paiol.terra.com.br (paiol.terra.com.br [200.176.3.18]) by sucuri.mat.puc-rio.br (8.9.3/8.9.3) with ESMTP id CAA19341 for ; Thu, 17 Apr 2003 02:36:37 -0300 Received: from itaim.terra.com.br (itaim.terra.com.br [200.176.3.76]) by paiol.terra.com.br (Postfix) with ESMTP id 4F04987D18 for ; Thu, 17 Apr 2003 02:36:07 -0300 (BRT) Received: from [200.177.176.199] (dl-nas1-sao-C8B1B0C7.p001.terra.com.br [200.177.176.199]) by itaim.terra.com.br (Postfix) with ESMTP id 5C8A22E0059 for ; Thu, 17 Apr 2003 02:36:06 -0300 (BRT) User-Agent: Microsoft-Outlook-Express-Macintosh-Edition/5.02.2022 Date: Sat, 02 Jan 1904 14:28:41 -0200 Subject: Re: [obm-l] O Retorno, parte -[e^(Pi*i)]. Os matem =?ISO-8859-1?B?4Q==?=ticos nunca morrem. From: Claudio Buffara To: Message-ID: <21D19.4D5%claudio.buffara@terra.com.br> In-Reply-To: <004e01c3048a$51c886c0$9cb3fea9@y7q1a7> Mime-version: 1.0 Content-type: text/plain; charset="ISO-8859-1" Content-Transfer-Encoding: 8bit X-MIME-Autoconverted: from quoted-printable to 8bit by sucuri.mat.puc-rio.br id CAA19343 Sender: owner-obm-l@sucuri.mat.puc-rio.br Precedence: bulk Reply-To: obm-l@mat.puc-rio.br on 17.04.03 00:37, Igor Correia Oliveira at basketboy_igor@bol.com.br wrote: > Saudações "terráquios", > Retorno da minha "hibernação" da lista com as seguintes dúvidas: > 1°) Como eu posso provar o Teorema de Erdös-Suranyi: "Todo inteiro positivo > K poder ser escrito na forma K=±1² ±2² ±3² ... ±m² para uma escolha > conveniente dos sinais + e -, e de m." (Amém!¡) Na verdade, todo inteiro positivo diferente de 2 e os sinais dos quadrados utilizados sao alternados (+ - + - ....) Tente identificar a lei de formacao das expressoes abaixo e depois prova-la por inducao: 1 = 1 3 = 4 - 1 4 = 4 5 = 9 - 4 6 = 9 - 4 + 1 7 = 16 - 9 8 = 9 - 1 9 = 9 10 = 16 - 9 + 4 - 1 11 = 16 - 9 + 4 12 = 16 - 4 13 = 16 - 4 + 1 14 = 25 - 9 15 = 16 - 1 16 = 16 17 = 25 - 9 + 1 18 = 25 - 16 + 9 19 = 25 - 9 + 4 - 1 20 = 25 - 9 + 4 21 = 25 - 4 22 = 25 - 4 + 1 23 = 36 - 16 + 4 - 1 24 = 25 - 1 25 = 25 26 = 36 - 16 + 9 - 4 + 1 27 = 36 - 16 + 9 - 4 28 = 36 - 9 + 1 29 = 36 - 16 + 9 30 = 36 - 9 + 4 - 1 31 = 36 - 9 + 4 32 = 36 - 4 33 = 36 - 4 + 1 34 = 49 - 25 + 16 - 9 + 4 - 1 35 = 36 - 1 36 = 36 > > 2°) Como eu provo que a distância do centro de gravidade do semi-círculo > para o centro do círculo é (4*Pi)/(3*r). > Use a formula de Pappus para o volume de um solido de revolucao em funcao da area e da distancia do centroide ao eixo e rotacao. No caso, teremos: V(esfera) = 2 * Pi * A(semi-circulo) * d ==> (4/3)*Pi*r^3 = 2 * Pi * (Pi*r^2/2) * d ==> d = 4*r/(3*Pi). > 3°) Encontre todos os inteiros positivos de 4 dígitos e cumprem a seguinte > condição: o cubo da soma dos seus dígitos é igual ao quadrado do referido > número. > Ou seja: (A + B + C + D)^3 = (1000A + 100B + 10C + D)^2 Com 1 <= A <= 9 e 0 <= B,C,D <= 9 (A+B+C+D)^3 <= (9+9+9+9)^3 = 46656 Logo, 1000A + 100B + 10C + D <= raiz(46656) = 216 ==> A = 0 ==> contradicao, pois o inteiro deve ter 4 digitos Logo, nao ha inteiros positivos de 4 digitos que cumpram as condicoes do enunciado. Um abraco, Claudio. ========================================================================= Instruções para entrar na lista, sair da lista e usar a lista em http://www.mat.puc-rio.br/~nicolau/olimp/obm-l.html O administrador desta lista é ========================================================================= From owner-obm-l@sucuri.mat.puc-rio.br Thu Apr 17 11:06:33 2003 Return-Path: Received: (from majordom@localhost) by sucuri.mat.puc-rio.br (8.9.3/8.9.3) id LAA25483 for obm-l-MTTP; Thu, 17 Apr 2003 11:03:12 -0300 Received: (from nicolau@localhost) by sucuri.mat.puc-rio.br (8.9.3/8.9.3) id LAA25478 for obm-l@mat.puc-rio.br; Thu, 17 Apr 2003 11:03:11 -0300 Date: Thu, 17 Apr 2003 11:03:11 -0300 From: "Nicolau C. Saldanha" To: obm-l@mat.puc-rio.br Subject: Re: [obm-l] Fibonacci Message-ID: <20030417110311.A25394@sucuri.mat.puc-rio.br> References: <014b01c3044f$46e8df60$5400a8c0@ensrbr> <20B43.4CD%claudio.buffara@terra.com.br> Mime-Version: 1.0 Content-Type: text/plain; charset=iso-8859-1 Content-Disposition: inline Content-Transfer-Encoding: 8bit User-Agent: Mutt/1.2.5i In-Reply-To: <20B43.4CD%claudio.buffara@terra.com.br>; from claudio.buffara@terra.com.br on Thu, Apr 17, 2003 at 03:00:43AM -0200 Sender: owner-obm-l@sucuri.mat.puc-rio.br Precedence: bulk Reply-To: obm-l@mat.puc-rio.br On Thu, Apr 17, 2003 at 03:00:43AM -0200, Claudio Buffara wrote: > Oi, Luis: > > Realmente a solucao eh muito elegante, apesar do mega-coelho que voce tira > da cartola - a identidade razoavelmente obscura: > F(k-1)*F(m) - F(k)*F(m-1) = (-1)^k*F(m-k) Esta identidade para mim não é obscura. Convenciono F(0) = 0, F(1) = 1; seja M a matriz [[0,1],[1,1]]. Uma das propriedades mais fundamentais da seq de Fibo é (0 1)^n (F(n-1) F(n) ) M^n = ( ) = ( ) (1 1) ( F(n) F(n+1)) em particular invertendo M^n (que tem determinante (-1)^n) temos F(-n) = (-1)^(n+1) F(n). Escreva M^(-k) * M^m = M^(m-k) e a identidade segue. []s, N. ========================================================================= Instruções para entrar na lista, sair da lista e usar a lista em http://www.mat.puc-rio.br/~nicolau/olimp/obm-l.html O administrador desta lista é ========================================================================= From owner-obm-l@sucuri.mat.puc-rio.br Thu Apr 17 15:47:51 2003 Return-Path: Received: (from majordom@localhost) by sucuri.mat.puc-rio.br (8.9.3/8.9.3) id PAA29717 for obm-l-MTTP; Thu, 17 Apr 2003 15:45:51 -0300 Received: from cmsrelay03.mx.net (cmsrelay03.mx.net [165.212.11.112]) by sucuri.mat.puc-rio.br (8.9.3/8.9.3) with SMTP id PAA29713 for ; Thu, 17 Apr 2003 15:45:48 -0300 Received: from uadvg128.cms.usa.net (HELO localhost) (165.212.11.128) by cmsoutbound.mx.net with SMTP; 17 Apr 2003 18:45:14 -0000 Received: from smtp.postoffice.net [165.212.8.3] by uadvg128.cms.usa.net (ASMTP/) via mtad (C8.MAIN.2.05) with ESMTP id 861HDqstN0435M28; Thu, 17 Apr 2003 18:45:13 GMT Received: from 200.181.4.100 [200.181.4.100] by uwdvg003.cms.usa.net (USANET web-mailer CM.0402.5.2B); Thu, 17 Apr 2003 18:45:12 -0000 Date: Thu, 17 Apr 2003 15:45:12 -0300 From: Artur Costa Steiner To: Subject: [obm-l] =?ISO-8859-1?Q?Integral=20impr=F3pria?= X-Mailer: USANET web-mailer (CM.0402.5.2B) Mime-Version: 1.0 Message-ID: <944HDqstm8784S03.1050605112@uwdvg003.cms.usa.net> Content-Type: text/plain; charset=ISO-8859-1 Content-Transfer-Encoding: 8bit X-MIME-Autoconverted: from quoted-printable to 8bit by sucuri.mat.puc-rio.br id PAA29714 Sender: owner-obm-l@sucuri.mat.puc-rio.br Precedence: bulk Reply-To: obm-l@mat.puc-rio.br Olah a todos! Há alguns dias alguem levantou na lista a questao de que se a existência da integral imprópria (Riemann) de f sobre o intervalo [a, inf), a em R, acarreta que lim (x --> inf) f(x) =0. Nao sei se a questao ficou esclarecida mas, de qualquer forma, a resposta e nao. Uma vez achei que o limite tinha mesmo que ser zero, tentei demonstrar e, eh claro, nao consegui, pois isto eh falso. Me deram o seguinte contra-exemplo, relativamente simples (ha muitos): para cada natural n, definamos f por f(x) =1 se x estah em (n, n +1/n^2) e f(x) =0 caso contrario. Eh facil ver que a integral de f sobre [0, inf) eh a serie 1 + 1/2^2 + 1/3^2....., a qual sabemos ser convergente. Logo, a integral converge para o limite da serie. Entretanto, f não apresenta limite no infinito. (Para vermos isto, observemos que as sequencias (n) e (n + 1/(2n^2)) sao ambas propriamente divergentes. Entretanto, a imagem da primeira sob f converge para 0 e a da segunda converge para 1). Se, por outro lado, lim (x --> inf) f(x) existir, entao ele eh necessariamente nulo. De outra forma, a integral impropria iria para + ou - infinito, contrariando a hipotese de sua convergencia. Vemos assi que analogia com o caso de series infinitas nao eh total. Ha ateh exemplos nos quais a f nao e tao "patologica", sendo ateh mesmo continua. Exemplos como a f que dei sao muito interssante sob o ponto de vista matematico, mas cabe mencionar que, em termos praticos, nao servem para nada, pois nenhum fenomeno fisico, economico, social ou seja la o que for e representado por funcoes como esta. Um abraco Artur ========================================================================= Instruções para entrar na lista, sair da lista e usar a lista em http://www.mat.puc-rio.br/~nicolau/olimp/obm-l.html O administrador desta lista é ========================================================================= From owner-obm-l@sucuri.mat.puc-rio.br Thu Apr 17 15:51:01 2003 Return-Path: Received: (from majordom@localhost) by sucuri.mat.puc-rio.br (8.9.3/8.9.3) id PAA29811 for obm-l-MTTP; Thu, 17 Apr 2003 15:49:45 -0300 Received: from itaqui.terra.com.br (itaqui.terra.com.br [200.176.3.19]) by sucuri.mat.puc-rio.br (8.9.3/8.9.3) with ESMTP id PAA29805 for ; Thu, 17 Apr 2003 15:49:40 -0300 Received: from gunga.terra.com.br (gunga.terra.com.br [200.176.3.45]) by itaqui.terra.com.br (Postfix) with ESMTP id 7DED33BC5B9 for ; Thu, 17 Apr 2003 15:49:10 -0300 (BRT) Received: from nt (RJ231083.user.veloxzone.com.br [200.165.231.83]) (authenticated user ensr) by gunga.terra.com.br (Postfix) with ESMTP id 445AF128266 for ; Thu, 17 Apr 2003 15:49:01 -0300 (BRT) Message-ID: <00e801c30511$e90f9000$5400a8c0@ensrbr> From: "Luis Lopes" To: References: <20030411230657.82083.qmail@web41509.mail.yahoo.com> Subject: [obm-l] =?iso-8859-1?Q?Re:_=5Bobm-l=5D_Re:=5F=5Bobm-l=5D=5FRe:=5F=5Bobm-l?= =?iso-8859-1?Q?=5D=5FTrigonometria=5Fe=5FSequ=EAncias?= Date: Thu, 17 Apr 2003 15:47:57 -0300 MIME-Version: 1.0 Content-Type: text/plain; charset="iso-8859-1" Content-Transfer-Encoding: 8bit X-Priority: 3 X-MSMail-Priority: Normal X-Mailer: Microsoft Outlook Express 5.50.4807.1700 X-MimeOLE: Produced By Microsoft MimeOLE V5.50.4807.1700 Sender: owner-obm-l@sucuri.mat.puc-rio.br Precedence: bulk Reply-To: obm-l@mat.puc-rio.br Sauda,c~oes, A soma S_n(q,\alpha) = \sum_{k=0}^n q^k \cos k\alpha pode ser calculada com manipulações algébricas e trigonométricas também. Mas concordo que usando números complexos eh mais objetivo e rico por gerar problemas (e soluções) como B2 e cuja solução eu não conhecia. De algum modo mostramos que S_n(q,\alpha)= A / B, onde A = q^{n+2}\cos n\alpha - q^{n+1}\cos(n+1)\alpha - q\cos\alpha + 1 B = 1 - 2q\cos\alpha + q^2 Se |q| < 1, S = lim S_n = (1 - q\cos\alpha) / B []'s Luís -----Mensagem Original----- De: "Carlos Yuzo Shine" Para: Enviada em: sexta-feira, 11 de abril de 2003 20:06 Assunto: Re: [obm-l] Re:_[obm-l]_Re:_[obm-l]_Trigonometria_e_Sequências > Ambos os problemas podem ser resolvidos usando o fato > de que > cos x = (e^(ix) + e^(-ix))/2 > sen x = (e^(ix) - e^(-ix))/(2i) > (x em radianos) > > Para ver isso, verifique as expansões em polinômio de > Taylor de e^x, sen x e cos x e verifique que > e^(ix) = cos x + i*sen x > e^(-ix) = cos x - i*sen x > > Veja que com isso o problema A5 vira uma soma de duas > progressões geométricas. > > O problema B2 usa os fatos acima e a fatoração > z^n - 1 = (z - 1)(z - w)(z - w^2)...(z - w^(n-1)), > em que w = e^(2\pi/n) é uma raiz n-ésima primitiva da > unidade. > > > A5. Sendo \cos(\theta) = 1 / \pi , calcule > \sum_{n=0}^\infty \cos(n\theta) / 2^n . > > > B2. Para n >= 2, mostre que (produtório) > > \sin(\pi / n) \sin(2\pi / n) ..... \sin[(n - 1)\pi / > > n] = n / 2^{n-1} . > > > > []'s > > Luís ========================================================================= Instruções para entrar na lista, sair da lista e usar a lista em http://www.mat.puc-rio.br/~nicolau/olimp/obm-l.html O administrador desta lista é ========================================================================= From owner-obm-l@sucuri.mat.puc-rio.br Thu Apr 17 15:58:00 2003 Return-Path: Received: (from majordom@localhost) by sucuri.mat.puc-rio.br (8.9.3/8.9.3) id PAA30276 for obm-l-MTTP; Thu, 17 Apr 2003 15:56:37 -0300 Received: from ns3bind.localdomain ([200.230.34.5]) by sucuri.mat.puc-rio.br (8.9.3/8.9.3) with ESMTP id PAA30266 for ; Thu, 17 Apr 2003 15:56:31 -0300 Received: from servico2 ([200.230.34.227]) by ns3bind.localdomain (8.11.6/X.XX.X) with SMTP id h3HIq5r19559 for ; Thu, 17 Apr 2003 15:52:05 -0300 Message-ID: <00e301c30513$233fb100$3300c57d@bovespa.com> From: "=?iso-8859-1?Q?Cl=E1udio_\=28Pr=E1tica\=29?=" To: References: <014b01c3044f$46e8df60$5400a8c0@ensrbr> <20B43.4CD%claudio.buffara@terra.com.br> <20030417110311.A25394@sucuri.mat.puc-rio.br> Subject: Re: [obm-l] Fibonacci Date: Thu, 17 Apr 2003 15:57:03 -0300 MIME-Version: 1.0 Content-Type: text/plain; charset="iso-8859-1" Content-Transfer-Encoding: 8bit X-Priority: 3 X-MSMail-Priority: Normal X-Mailer: Microsoft Outlook Express 5.50.4920.2300 X-MimeOLE: Produced By Microsoft MimeOLE V5.50.4920.2300 Sender: owner-obm-l@sucuri.mat.puc-rio.br Precedence: bulk Reply-To: obm-l@mat.puc-rio.br Oi, Nicolau e Luis: Bem, pra mim agora também não é mais!!! Boa Páscoa para todos. Um abraço, Claudio. ----- Original Message ----- From: "Nicolau C. Saldanha" To: Sent: Thursday, April 17, 2003 11:03 AM Subject: Re: [obm-l] Fibonacci > On Thu, Apr 17, 2003 at 03:00:43AM -0200, Claudio Buffara wrote: > > Oi, Luis: > > > > Realmente a solucao eh muito elegante, apesar do mega-coelho que voce tira > > da cartola - a identidade razoavelmente obscura: > > F(k-1)*F(m) - F(k)*F(m-1) = (-1)^k*F(m-k) > > Esta identidade para mim não é obscura. > > Convenciono F(0) = 0, F(1) = 1; seja M a matriz [[0,1],[1,1]]. > Uma das propriedades mais fundamentais da seq de Fibo é > > (0 1)^n (F(n-1) F(n) ) > M^n = ( ) = ( ) > (1 1) ( F(n) F(n+1)) > > em particular invertendo M^n (que tem determinante (-1)^n) > temos F(-n) = (-1)^(n+1) F(n). Escreva M^(-k) * M^m = M^(m-k) > e a identidade segue. > > []s, N. > ========================================================================= > Instruções para entrar na lista, sair da lista e usar a lista em > http://www.mat.puc-rio.br/~nicolau/olimp/obm-l.html > O administrador desta lista é > ========================================================================= ========================================================================= Instruções para entrar na lista, sair da lista e usar a lista em http://www.mat.puc-rio.br/~nicolau/olimp/obm-l.html O administrador desta lista é ========================================================================= From owner-obm-l@sucuri.mat.puc-rio.br Thu Apr 17 16:12:52 2003 Return-Path: Received: (from majordom@localhost) by sucuri.mat.puc-rio.br (8.9.3/8.9.3) id QAA31014 for obm-l-MTTP; Thu, 17 Apr 2003 16:11:14 -0300 Received: from itaqui.terra.com.br (itaqui.terra.com.br [200.176.3.19]) by sucuri.mat.puc-rio.br (8.9.3/8.9.3) with ESMTP id QAA31009 for ; Thu, 17 Apr 2003 16:11:11 -0300 Received: from gunga.terra.com.br (gunga.terra.com.br [200.176.3.45]) by itaqui.terra.com.br (Postfix) with ESMTP id 6287B3BC470 for ; Thu, 17 Apr 2003 16:10:40 -0300 (BRT) Received: from nt (RJ231083.user.veloxzone.com.br [200.165.231.83]) (authenticated user ensr) by gunga.terra.com.br (Postfix) with ESMTP id 051BE1282A6 for ; Thu, 17 Apr 2003 16:10:36 -0300 (BRT) Message-ID: <00ed01c30514$eb7c9100$5400a8c0@ensrbr> From: "Luis Lopes" To: References: <6732305773.20030411013805@gmx.net> <00ba01c3005e$56faaae0$3300c57d@bovespa.com> <012a01c3006b$dcae0a80$5400a8c0@ensrbr> Subject: [obm-l] =?iso-8859-1?Q?Dois_problemas_do_mesmo_site_=5Bera:Trigonometria_e_Sequ?= =?iso-8859-1?Q?=EAncias=5D?= Date: Thu, 17 Apr 2003 16:09:40 -0300 MIME-Version: 1.0 Content-Type: text/plain; charset="iso-8859-1" Content-Transfer-Encoding: 8bit X-Priority: 3 X-MSMail-Priority: Normal X-Mailer: Microsoft Outlook Express 5.50.4807.1700 X-MimeOLE: Produced By Microsoft MimeOLE V5.50.4807.1700 Sender: owner-obm-l@sucuri.mat.puc-rio.br Precedence: bulk Reply-To: obm-l@mat.puc-rio.br Sauda,c~oes, Na mensagem do Claudio Buffara sobre aparece o site Na internet voce tambem encontra alguns sites com problemas interessantes, tais como este aqui: http://math.stanford.edu/~vakil/stanfordputnam/02/putnam3.pdf Vasculhando o site acima encontrei acho que em http://math.stanford.edu/~vakil/stanfordputnam/02/putnam1.pdf alguns problemas. Proponho novamente dois problemas, mas desta vez não tenho a solução de nenhum dos dois: Com uma notação resumida, 1) mostre que existe n tal que 2^n = 2002....... 2) 10000! = ..........n0000000000...000. Calcule n. Obs.: o número de zeros em n0000000000...000 eh um problema conhecido. []'s Luís ========================================================================= Instruções para entrar na lista, sair da lista e usar a lista em http://www.mat.puc-rio.br/~nicolau/olimp/obm-l.html O administrador desta lista é ========================================================================= From owner-obm-l@sucuri.mat.puc-rio.br Thu Apr 17 16:44:21 2003 Return-Path: Received: (from majordom@localhost) by sucuri.mat.puc-rio.br (8.9.3/8.9.3) id QAA31919 for obm-l-MTTP; Thu, 17 Apr 2003 16:29:23 -0300 Received: from puma.unisys.com.br (smtp.unisys.com.br [200.220.64.7]) by sucuri.mat.puc-rio.br (8.9.3/8.9.3) with ESMTP id QAA31915 for ; Thu, 17 Apr 2003 16:29:18 -0300 Received: from n8x4f9 (riopm18p120.uninet.com.br [200.220.16.120]) by puma.unisys.com.br (8.12.9/8.12.3) with SMTP id h3HJSn0e011729 for ; Thu, 17 Apr 2003 16:28:50 -0300 (EST) X-Spam-Filter: check_local@puma.unisys.com.br by digitalanswers.org Message-ID: <001b01c30518$172a2da0$7810dcc8@n8x4f9> From: "Jose Francisco Guimaraes Costa" To: References: <590HDooKj1664S20.1050419370@uwdvg007.cms.usa.net> <20030416043622.42259.qmail@web13705.mail.yahoo.com> <20030416100912.GA21143@linux.ime.usp.br> <007501c3043e$ea749c20$0e00a8c0@hotlink.com.br> Subject: [obm-l] =?iso-8859-1?Q?conjecturas_de_riemann_e_poincar=E9?= Date: Thu, 17 Apr 2003 16:32:29 -0300 MIME-Version: 1.0 Content-Type: multipart/alternative; boundary="----=_NextPart_000_0018_01C304FE.F006FD20" X-Priority: 3 X-MSMail-Priority: Normal X-Mailer: Microsoft Outlook Express 5.00.2615.200 X-MimeOLE: Produced By Microsoft MimeOLE V5.00.2615.200 Sender: owner-obm-l@sucuri.mat.puc-rio.br Precedence: bulk Reply-To: obm-l@mat.puc-rio.br This is a multi-part message in MIME format. ------=_NextPart_000_0018_01C304FE.F006FD20 Content-Type: text/plain; charset="iso-8859-1" Content-Transfer-Encoding: quoted-printable V=E1 at=E9 http://mathworld.wolfram.com/ e pesquise (search) "poincare" = que V ter=E1 o status da Conjectura de Poincar=E9, inclusive pelo menos = uma demonstra=E7=E3o em = http://www.maths.soton.ac.uk/pure/viewabstract.phtml?entry=3D655 . Se V pesquisar "riemann" vai ter informa=E7=F5es sobre a conjectura que = leva o nome dele, mas tanto quanto pude ver, n=E3o existe nenhum link = =E0s "demonstra=E7=E3o dos f=EDsicos" mencionada pelo N. JF ----- Original Message -----=20 From: gabriel=20 To: obm-l@mat.puc-rio.br=20 Sent: Wednesday, April 16, 2003 2:37 PM Subject: [obm-l] riemann ALguem sabe dizer se a hipotese de Riemann foi realmente provada?E a conjectura de Poincare? Gabriel Guedes ------=_NextPart_000_0018_01C304FE.F006FD20 Content-Type: text/html; charset="iso-8859-1" Content-Transfer-Encoding: quoted-printable
V=E1 at=E9 http://mathworld.wolfram.com/<= FONT=20 face=3D"Times New Roman" size=3D3> e pesquise = (search) "poincare"=20 que V ter=E1 o status da Conjectura de Poincar=E9, inclusive pelo menos = uma=20 demonstra=E7=E3o em http://www.maths.soton.ac.uk/pure/viewabstract.phtml?entry=3D655=20 .
 
Se V pesquisar "riemann" vai ter = informa=E7=F5es sobre=20 a conjectura que leva o nome dele, mas tanto quanto pude ver, n=E3o = existe nenhum=20 link =E0s "demonstra=E7=E3o dos f=EDsicos" mencionada pelo = N.
 
JF
 
----- Original Message -----
From:=20 gabriel
Sent: Wednesday, April 16, 2003 = 2:37=20 PM
Subject: [obm-l] riemann

ALguem sabe dizer se a hipotese de Riemann foi = realmente=20 provada?E a
conjectura de Poincare?
Gabriel=20 Guedes
------=_NextPart_000_0018_01C304FE.F006FD20-- ========================================================================= Instruções para entrar na lista, sair da lista e usar a lista em http://www.mat.puc-rio.br/~nicolau/olimp/obm-l.html O administrador desta lista é ========================================================================= From owner-obm-l@sucuri.mat.puc-rio.br Thu Apr 17 18:24:59 2003 Return-Path: Received: (from majordom@localhost) by sucuri.mat.puc-rio.br (8.9.3/8.9.3) id SAA00895 for obm-l-MTTP; Thu, 17 Apr 2003 18:22:23 -0300 Received: from ivoti.terra.com.br (ivoti.terra.com.br [200.176.3.20]) by sucuri.mat.puc-rio.br (8.9.3/8.9.3) with ESMTP id SAA00890 for ; Thu, 17 Apr 2003 18:22:19 -0300 Received: from araci.terra.com.br (araci.terra.com.br [200.176.3.44]) by ivoti.terra.com.br (Postfix) with ESMTP id D16E040816A for ; Thu, 17 Apr 2003 18:21:42 -0300 (BRT) Received: from nt (RJ231083.user.veloxzone.com.br [200.165.231.83]) (authenticated user ensr) by araci.terra.com.br (Postfix) with ESMTP id 47ED721EF93 for ; Thu, 17 Apr 2003 18:21:42 -0300 (BRT) Message-ID: <014901c30527$3ba77700$5400a8c0@ensrbr> From: "Luis Lopes" To: References: <014b01c3044f$46e8df60$5400a8c0@ensrbr> <20B43.4CD%claudio.buffara@terra.com.br> <20030417110311.A25394@sucuri.mat.puc-rio.br> Subject: [obm-l] serie interessante Date: Thu, 17 Apr 2003 18:20:55 -0300 MIME-Version: 1.0 Content-Type: text/plain; charset="iso-8859-1" Content-Transfer-Encoding: 8bit X-Priority: 3 X-MSMail-Priority: Normal X-Mailer: Microsoft Outlook Express 5.50.4807.1700 X-MimeOLE: Produced By Microsoft MimeOLE V5.50.4807.1700 Sender: owner-obm-l@sucuri.mat.puc-rio.br Precedence: bulk Reply-To: obm-l@mat.puc-rio.br Sauda,c~oes, O professor Rousseau acabou de me mandar o seguinte email: Dear Luis: Thanks. I am just writing up a solution set for one of my classes that includes a most remarkable sum. Unfortunately, I don't know even an Eulerian approach to this problem; only complex analysis works as far as I know. Anyway, here is the series \sum_{k=1}^{\infty} 1/x_k^2 = 1/10, where 0 < x_1 < x_2 < x_3 < \cdots are the positive roots of the equation x = \tan x. What's so remarkable about this series is that we don't have a precise value for any one of the terms (although they can be approximated to many decimal places by numerical computation), but we do have a precise value (1/10) for the sum. Something to ponder. Cecil Comentário: não sei se os x_k e os 1/x_k^2 são transcendentes mas se forem, teríamos uma soma infinita de termos transcendentes dando um número racional. []'s Luís ========================================================================= Instruções para entrar na lista, sair da lista e usar a lista em http://www.mat.puc-rio.br/~nicolau/olimp/obm-l.html O administrador desta lista é ========================================================================= From owner-obm-l@sucuri.mat.puc-rio.br Thu Apr 17 19:39:52 2003 Return-Path: Received: (from majordom@localhost) by sucuri.mat.puc-rio.br (8.9.3/8.9.3) id TAA02778 for obm-l-MTTP; Thu, 17 Apr 2003 19:37:11 -0300 Received: from ivoti.terra.com.br (ivoti.terra.com.br [200.176.3.20]) by sucuri.mat.puc-rio.br (8.9.3/8.9.3) with ESMTP id TAA02774 for ; Thu, 17 Apr 2003 19:37:08 -0300 Received: from araci.terra.com.br (araci.terra.com.br [200.176.3.44]) by ivoti.terra.com.br (Postfix) with ESMTP id B3C72408704 for ; Thu, 17 Apr 2003 19:36:37 -0300 (BRT) Received: from nt (RJ231083.user.veloxzone.com.br [200.165.231.83]) (authenticated user ensr) by araci.terra.com.br (Postfix) with ESMTP id 44D4421EF59 for ; Thu, 17 Apr 2003 19:36:37 -0300 (BRT) Message-ID: <016901c30531$b33d9600$5400a8c0@ensrbr> From: "Luis Lopes" To: References: <010a01c30069$5ccfe380$5400a8c0@ensrbr> <595180649.20030412013816@gmx.net> Subject: [obm-l] =?iso-8859-1?Q?Re:_=5Bobm-l=5D_Re:_=5Bobm-l=5D_Re:_=5Bobm-l=5D_Sequ=EAnci?= =?iso-8859-1?Q?as?= Date: Thu, 17 Apr 2003 19:35:50 -0300 MIME-Version: 1.0 Content-Type: text/plain; charset="iso-8859-1" Content-Transfer-Encoding: 8bit X-Priority: 3 X-MSMail-Priority: Normal X-Mailer: Microsoft Outlook Express 5.50.4807.1700 X-MimeOLE: Produced By Microsoft MimeOLE V5.50.4807.1700 Sender: owner-obm-l@sucuri.mat.puc-rio.br Precedence: bulk Reply-To: obm-l@mat.puc-rio.br Sauda,c~oes, Vou falar, Igor. Não me lembro de onde tirei a prova deste resultado. Talvez de um livro de cálculo numérico. Vou dar mais um exemplo de seu uso para a PA de ordem k=3. Seja a PA de ordem 3 1,3,19,61,141,271,... a_i Vamos gerar outras PAs fazendo a_{i+1} - a_i: 2,16,42,80,130 Delta a_i 14,26,38,50 Delta^2 a_i 12,12,12 Delta^3 a_i a_i = a_1 + Delta a_1 binom(i-1,1) + Delta^2 a_1 binom(i-1,2) + Delta^3 a_1 binom(i-1,3) a_i = 1 + 2(i-1) + 14(i-1)(i-2)/2 + 12(i-1)(i-2)(i-3)/6 a_i = 2i^3 - 5i^2 + 3i + 1 S_n = a_1 binom(n,1) + Delta a_1 binom(n,2) + Delta^2 a_1 binom(n,3) + Delta^3 binom(n,4) S_n = n[3n^3 - 4n^2 - 3n + 10] / 6 Para um outro método, usando polinômios fatoriais e antidiferenças, ver o meu livro de Progressões. []'s Luís -----Mensagem Original----- De: "Igor GomeZZ" Para: "Luis Lopes" Enviada em: sábado, 12 de abril de 2003 01:38 Assunto: [obm-l] Re: [obm-l] Re: [obm-l] Sequências > > Em 11/4/2003, 17:31, Luis (llopes@ensrbr.com.br) disse: > > > Sauda,c~oes, > > Fala Luís! > > > Ha alguns resultados que facilitam estes > > calculos. > > Vc quer somar S_n^{[k]} = \sum_{i=1}^n a_i, > > a_i termo geral de PA de ordem k. > > O resultado geral de S_n^{[k]} para n=49 e k=2 eh > > S_{49}^{[2]} = > > 2C(49,3) + 4C(49,2) + 2C(49,1) = 41650. > > Entendi bem por cima, acredito que a notação Latex pra quem não estah > familiarizado dificulta :-) > > Pelo que entendi, eh uma forma de fazer a soma de uma PA de ordem k, > sabendo-se apenas o termo geral sem precisar calcular o polinômio (grau > k+1) que define a soma, correto? > > Vc poderia mostrar melhor sua resolução? > > > []'s > > Luís > > Valeuz Luís! > > Fui! > > > ####### Igor GomeZZ ######## > UIN: 29249895 > Vitória, Espírito Santo, Brasil > Criação: 12/4/2003 (01:30) > #################################### > Pare para pensar: > > Algo é só impossível até que > alguém duvide e acabe provando o > contrário. (Albert Einstein) > > #################################### > ========================================================================= Instruções para entrar na lista, sair da lista e usar a lista em http://www.mat.puc-rio.br/~nicolau/olimp/obm-l.html O administrador desta lista é ========================================================================= From owner-obm-l@sucuri.mat.puc-rio.br Thu Apr 17 22:10:58 2003 Return-Path: Received: (from majordom@localhost) by sucuri.mat.puc-rio.br (8.9.3/8.9.3) id WAA05694 for obm-l-MTTP; Thu, 17 Apr 2003 22:08:51 -0300 Received: from puma.unisys.com.br (smtp.unisys.com.br [200.220.64.7]) by sucuri.mat.puc-rio.br (8.9.3/8.9.3) with ESMTP id WAA05690 for ; Thu, 17 Apr 2003 22:08:48 -0300 Received: from jf (riohiper01p169.uninet.com.br [200.220.2.169]) by puma.unisys.com.br (8.12.9/8.12.3) with SMTP id h3I18F0g028955 for ; Thu, 17 Apr 2003 22:08:20 -0300 (EST) X-Spam-Filter: check_local@puma.unisys.com.br by digitalanswers.org Message-ID: <009d01c30547$495f38e0$a902dcc8@jf> From: "Jose Francisco Guimaraes Costa" To: References: <010a01c30069$5ccfe380$5400a8c0@ensrbr> <595180649.20030412013816@gmx.net> <016901c30531$b33d9600$5400a8c0@ensrbr> Subject: [obm-l] PA de ordem k Date: Thu, 17 Apr 2003 22:09:24 -0300 MIME-Version: 1.0 Content-Type: multipart/alternative; boundary="----=_NextPart_000_0097_01C3052E.017FE7E0" X-Priority: 3 X-MSMail-Priority: Normal X-Mailer: Microsoft Outlook Express 6.00.2800.1106 X-MimeOLE: Produced By Microsoft MimeOLE V6.00.2800.1106 Sender: owner-obm-l@sucuri.mat.puc-rio.br Precedence: bulk Reply-To: obm-l@mat.puc-rio.br This is a multi-part message in MIME format. ------=_NextPart_000_0097_01C3052E.017FE7E0 Content-Type: text/plain; charset="iso-8859-1" Content-Transfer-Encoding: quoted-printable O que =E9 uma PA de ordem k? JF ----- Original Message -----=20 From: "Luis Lopes" To: Sent: Thursday, April 17, 2003 7:35 PM Subject: [obm-l] Re: [obm-l] Re: [obm-l] Re: [obm-l] Sequ=EAncias > Seja a PA de ordem 3 >=20 > 1,3,19,61,141,271,... a_i >=20 [,,,] > Lu=EDs ------=_NextPart_000_0097_01C3052E.017FE7E0 Content-Type: text/html; charset="iso-8859-1" Content-Transfer-Encoding: quoted-printable
O que =E9 uma PA de ordem = k?
 
JF
 
----- Original Message ----- =
From: "Luis Lopes" <llopes@ensrbr.com.br>
To: <obm-l@mat.puc-rio.br>
Sent: Thursday, April 17, 2003 7:35 = PM
Subject: [obm-l] Re: [obm-l] Re: = [obm-l] Re:=20 [obm-l] Sequ=EAncias

> Seja a PA de ordem 3
> =
>=20 1,3,19,61,141,271,...     a_i
> =
[,,,]
> = Lu=EDs
------=_NextPart_000_0097_01C3052E.017FE7E0-- ========================================================================= Instruções para entrar na lista, sair da lista e usar a lista em http://www.mat.puc-rio.br/~nicolau/olimp/obm-l.html O administrador desta lista é ========================================================================= From owner-obm-l@sucuri.mat.puc-rio.br Thu Apr 17 22:29:08 2003 Return-Path: Received: (from majordom@localhost) by sucuri.mat.puc-rio.br (8.9.3/8.9.3) id WAA06131 for obm-l-MTTP; Thu, 17 Apr 2003 22:27:31 -0300 Received: from trex-b.centroin.com.br (trex-b.centroin.com.br [200.225.63.136]) by sucuri.mat.puc-rio.br (8.9.3/8.9.3) with ESMTP id WAA06127 for ; Thu, 17 Apr 2003 22:27:25 -0300 Received: from centroin.com.br (RJ091082.user.veloxzone.com.br [200.141.91.82] (may be forged)) (authenticated bits=0) by trex-b.centroin.com.br (8.12.9/8.12.9) with ESMTP id h3I1QrVi009728 for ; Thu, 17 Apr 2003 22:26:53 -0300 (EST) Message-ID: <3E9F5494.7060605@centroin.com.br> Date: Thu, 17 Apr 2003 22:27:48 -0300 From: "A. C. Morgado" User-Agent: Mozilla/5.0 (Windows; U; Windows NT 5.0; en-US; rv:1.0.2) Gecko/20030208 Netscape/7.02 X-Accept-Language: en-us, en MIME-Version: 1.0 To: obm-l@mat.puc-rio.br Subject: Re: [obm-l] PA de ordem k References: <010a01c30069$5ccfe380$5400a8c0@ensrbr> <595180649.20030412013816@gmx.net> <016901c30531$b33d9600$5400a8c0@ensrbr> <009d01c30547$495f38e0$a902dcc8@jf> Content-Type: multipart/alternative; boundary="------------070704080700060209070308" Sender: owner-obm-l@sucuri.mat.puc-rio.br Precedence: bulk Reply-To: obm-l@mat.puc-rio.br --------------070704080700060209070308 Content-Type: text/plain; charset=ISO-8859-1; format=flowed Content-Transfer-Encoding: 8bit Voce pode definir como sendo uma sequencia cujo termo de ordem n eh dado por um polinomio em n de grau k. PA nao constantes seriam PAs de ordem 1 e as constantes seriam de ordem 0. Definindo assim eh um teorema que as diferenças entre termos consecutivos de uma PA de ordem k formam uma PA de ordem k-1. Ou pode fazer ao contrario definindo recursivamente por: PA de ordem 0 eh PA constante, PA de ordem 1 eh PA nao constante e PA de ordem k eh sequencia em que as diferenças entre termos consecutivos formam PA de ordem k-1. Aih, eh teorema que numa PA de ordem k o termo de ordem n eh dado por um polinomio em n de grau k. Jose Francisco Guimaraes Costa wrote: > O que é uma PA de ordem k? > > JF > > ----- Original Message ----- > From: "Luis Lopes" > > To: > > Sent: Thursday, April 17, 2003 7:35 PM > Subject: [obm-l] Re: [obm-l] Re: [obm-l] Re: [obm-l] Sequências > > > Seja a PA de ordem 3 > > > > 1,3,19,61,141,271,... a_i > > > [,,,] > > Luís --------------070704080700060209070308 Content-Type: text/html; charset=us-ascii Content-Transfer-Encoding: 7bit Voce pode definir como sendo uma sequencia cujo termo de ordem n eh dado por um polinomio em n de grau k. PA nao constantes seriam PAs de ordem 1 e as constantes seriam de ordem 0. Definindo assim eh um teorema que as diferenças entre termos consecutivos de uma PA de ordem k formam uma PA de ordem k-1.
Ou pode fazer ao contrario definindo recursivamente por:  PA de ordem 0 eh PA constante, PA de ordem 1 eh PA nao constante e PA de ordem k eh sequencia em que  as diferenças entre termos consecutivos formam PA de ordem k-1. Aih, eh teorema que numa PA de ordem k o termo de ordem n eh dado por um polinomio em n de grau k.

Jose Francisco Guimaraes Costa wrote:
O que é uma PA de ordem k?
 
JF
 
----- Original Message -----
From: "Luis Lopes" <llopes@ensrbr.com.br>
Sent: Thursday, April 17, 2003 7:35 PM
Subject: [obm-l] Re: [obm-l] Re: [obm-l] Re: [obm-l] Sequências

> Seja a PA de ordem 3
>
> 1,3,19,61,141,271,...     a_i
>
[,,,]
> Luís

--------------070704080700060209070308-- ========================================================================= Instruções para entrar na lista, sair da lista e usar a lista em http://www.mat.puc-rio.br/~nicolau/olimp/obm-l.html O administrador desta lista é ========================================================================= From owner-obm-l@sucuri.mat.puc-rio.br Thu Apr 17 22:34:07 2003 Return-Path: Received: (from majordom@localhost) by sucuri.mat.puc-rio.br (8.9.3/8.9.3) id WAA06330 for obm-l-MTTP; Thu, 17 Apr 2003 22:32:45 -0300 Received: from imo-m05.mx.aol.com (imo-m05.mx.aol.com [64.12.136.8]) by sucuri.mat.puc-rio.br (8.9.3/8.9.3) with ESMTP id WAA06326 for ; Thu, 17 Apr 2003 22:32:41 -0300 From: DEOLIVEIRASOU@aol.com Received: from DEOLIVEIRASOU@aol.com by imo-m05.mx.aol.com (mail_out_v34.21.) id z.10f.20f85442 (4184) for ; Thu, 17 Apr 2003 21:32:02 -0400 (EDT) Message-ID: <10f.20f85442.2bd0af92@aol.com> Date: Thu, 17 Apr 2003 21:32:02 EDT Subject: Re: [obm-l] Sobre as olimpiadas ao redor do mundo(e um certo DEOLIVEIRASOU...) To: obm-l@mat.puc-rio.br MIME-Version: 1.0 Content-Type: multipart/alternative; boundary="part1_10f.20f85442.2bd0af92_boundary" X-Mailer: 7.0 for Windows sub 10501 Sender: owner-obm-l@sucuri.mat.puc-rio.br Precedence: bulk Reply-To: obm-l@mat.puc-rio.br --part1_10f.20f85442.2bd0af92_boundary Content-Type: text/plain; charset="US-ASCII" Content-Transfer-Encoding: 7bit Ola Claudio, Voce tem tanta razao sobre a transpiracao em detrimento da inspiracao, que eu, ja consegui fazer sozinho uns seis ou sete problemas das olimpiadas ao redor do mundo...somando-se a essa conquista, os problemas que vc tem resolvido, acho que meu desenvolvimento em rudimentos de teoria dos numeros ja esta aparecendo....Valeu muito. Para que essa mensagem nao fique off-topic, vou mandar aos participantes da lista o se guinte problema.... Mostre que a equacao diofantina x^3+y^3+z^3=0 so tem solucoes triviais, ou seja, xyz=0. obs...esse exercicio vem depois da exposicao do artigo sobre inteiros de Gauss e inteiros de Einsenstein da eureka 14, que confesso ainda nao ter lido....pergunta----ele so pode ser resolvido utilizando-se as teorias subjacentes ao artigo. Um Abraco, Crom --part1_10f.20f85442.2bd0af92_boundary Content-Type: text/html; charset="US-ASCII" Content-Transfer-Encoding: quoted-printable Ola Claudio,
Voce tem tanta razao sobre a transpiracao em detrimento da inspiracao, que e= u, ja consegui fazer sozinho uns seis ou sete problemas das olimpiadas ao re= dor do mundo...somando-se a essa conquista, os problemas que vc tem  re= solvido, acho que meu desenvolvimento em rudimentos de teoria dos numeros ja= esta aparecendo....Valeu muito.
Para que essa mensagem nao fique off-topic, vou mandar aos participantes da=20= lista o se guinte problema....
Mostre que a equacao diofantina x^3+y^3+z^3=3D0 so tem solucoes triviais, ou= seja, xyz=3D0.
obs...esse exercicio vem depois da exposicao do artigo sobre inteiros de Gau= ss e inteiros de Einsenstein da eureka 14, que confesso ainda nao ter lido..= ..pergunta----ele so pode ser resolvido utilizando-se as teorias subjacentes= ao artigo.
         Um Abraco,
             Cro= m
--part1_10f.20f85442.2bd0af92_boundary-- ========================================================================= Instruções para entrar na lista, sair da lista e usar a lista em http://www.mat.puc-rio.br/~nicolau/olimp/obm-l.html O administrador desta lista é ========================================================================= From owner-obm-l@sucuri.mat.puc-rio.br Fri Apr 18 00:32:41 2003 Return-Path: Received: (from majordom@localhost) by sucuri.mat.puc-rio.br (8.9.3/8.9.3) id AAA08439 for obm-l-MTTP; Fri, 18 Apr 2003 00:28:23 -0300 Received: from mail.gmx.net (mail.gmx.net [213.165.65.60]) by sucuri.mat.puc-rio.br (8.9.3/8.9.3) with SMTP id AAA08434 for ; Fri, 18 Apr 2003 00:28:19 -0300 Received: (qmail 3596 invoked by uid 65534); 18 Apr 2003 03:27:46 -0000 Received: from unknown (EHLO localhost) (200.217.15.121) by mail.gmx.net (mp001-rz3) with SMTP; 18 Apr 2003 05:27:46 +0200 Date: Fri, 18 Apr 2003 00:27:35 -0300 From: Igor GomeZZ X-Mailer: The Bat! (v1.61) Organization: -- X-Priority: 3 (Normal) Message-ID: <1731697330.20030418002735@gmx.net> To: Luis Lopes Subject: [obm-l] =?ISO-8859-1?B?UmU6IFtvYm0tbF0gU2VxdepuY2lhcw==?= In-Reply-To: <016901c30531$b33d9600$5400a8c0@ensrbr> References: <010a01c30069$5ccfe380$5400a8c0@ensrbr> <595180649.20030412013816@gmx.net> <016901c30531$b33d9600$5400a8c0@ensrbr> MIME-Version: 1.0 Content-Type: text/plain; charset=ISO-8859-1 Content-Transfer-Encoding: 8bit Sender: owner-obm-l@sucuri.mat.puc-rio.br Precedence: bulk Reply-To: obm-l@mat.puc-rio.br Em 17/4/2003, 19:35, Luis (llopes@ensrbr.com.br) disse: > Vou falar, Igor. Isso eh bom :-) > Para um outro método, usando polinômios > fatoriais e antidiferenças, ver o meu livro de > Progressões. Perfeito agora! Entendido... Outro autor na lista, fantástico :-) Vou procurá-lo > []'s > Luís Flws! Obrigado. Fui! ####### Igor GomeZZ ######## UIN: 29249895 Vitória, Espírito Santo, Brasil Criação: 18/4/2003 (00:25) #################################### Pare para pensar: A verdade é filha do tempo, não da autoridade. (Francis Bacon) #################################### ========================================================================= Instruções para entrar na lista, sair da lista e usar a lista em http://www.mat.puc-rio.br/~nicolau/olimp/obm-l.html O administrador desta lista é ========================================================================= From owner-obm-l@sucuri.mat.puc-rio.br Fri Apr 18 01:09:01 2003 Return-Path: Received: (from majordom@localhost) by sucuri.mat.puc-rio.br (8.9.3/8.9.3) id BAA09249 for obm-l-MTTP; Fri, 18 Apr 2003 01:07:23 -0300 Received: from smtp-29.ig.com.br (smtp-29.ig.com.br [200.226.132.157]) by sucuri.mat.puc-rio.br (8.9.3/8.9.3) with SMTP id BAA09245 for ; Fri, 18 Apr 2003 01:07:19 -0300 Received: (qmail 30732 invoked from network); 18 Apr 2003 04:06:48 -0000 Received: from unknown (HELO henrique) (200.140.80.114) by smtp-29.ig.com.br with SMTP; 18 Apr 2003 04:06:48 -0000 Message-ID: <000501c3055f$f00f1c60$019da8c0@henrique> From: "=?iso-8859-1?Q?Henrique_Patr=EDcio_Sant'Anna_Branco?=" To: "OBM" Subject: [obm-l] Vetores Date: Fri, 18 Apr 2003 01:06:49 -0300 MIME-Version: 1.0 Content-Type: text/plain; charset="iso-8859-1" Content-Transfer-Encoding: 8bit X-Priority: 3 X-MSMail-Priority: Normal X-Mailer: Microsoft Outlook Express 6.00.2800.1106 X-MimeOLE: Produced By Microsoft MimeOLE V6.00.2800.1106 Sender: owner-obm-l@sucuri.mat.puc-rio.br Precedence: bulk Reply-To: obm-l@mat.puc-rio.br Pessoal, Tem uma questão de Algebra Linear que tá me causando dúvidas. É pra dizer se é V ou F. Segue: O vetor (-3, 1, 2) pertence ao plano do R^3 que é gerado pelos vetores (1, 0, -1) e (0, 0, 1). Resolvi da seguinte forma: (-3, 1, 2) = a (1, 0, -1) + b (0, 0, 1) = (a, 0, -a) + (0, 0, b) = (a, 0, -a + b) (-3, 1, 2) = (a, 0, -a+b) Impossível, pois não podemos fazer 0 = 1. Onde estou errando? Ou a afirmação é realmente falsa? Grato, Henrique. ========================================================================= Instruções para entrar na lista, sair da lista e usar a lista em http://www.mat.puc-rio.br/~nicolau/olimp/obm-l.html O administrador desta lista é ========================================================================= From owner-obm-l@sucuri.mat.puc-rio.br Fri Apr 18 01:30:32 2003 Return-Path: Received: (from majordom@localhost) by sucuri.mat.puc-rio.br (8.9.3/8.9.3) id BAA09934 for obm-l-MTTP; Fri, 18 Apr 2003 01:29:03 -0300 Received: from web13701.mail.yahoo.com (web13701.mail.yahoo.com [216.136.175.134]) by sucuri.mat.puc-rio.br (8.9.3/8.9.3) with SMTP id BAA09930 for ; Fri, 18 Apr 2003 01:28:56 -0300 Message-ID: <20030418042752.39269.qmail@web13701.mail.yahoo.com> Received: from [200.213.88.152] by web13701.mail.yahoo.com via HTTP; Fri, 18 Apr 2003 01:27:52 ART Date: Fri, 18 Apr 2003 01:27:52 -0300 (ART) From: =?iso-8859-1?q?pichurin?= Subject: Re: [obm-l] Vetores To: obm-l@mat.puc-rio.br In-Reply-To: <000501c3055f$f00f1c60$019da8c0@henrique> MIME-Version: 1.0 Content-Type: text/plain; charset=iso-8859-1 Content-Transfer-Encoding: 8bit Sender: owner-obm-l@sucuri.mat.puc-rio.br Precedence: bulk Reply-To: obm-l@mat.puc-rio.br Vc está certo. Nao eh possível obter o vetor(-3,1,2) com os otros dois vetores --- Henrique_Patrício_Sant'Anna_Branco escreveu: > Pessoal, > > Tem uma questão de Algebra Linear que tá me causando > dúvidas. É pra dizer se > é V ou F. > Segue: > > O vetor (-3, 1, 2) pertence ao plano do R^3 que é > gerado pelos vetores (1, > 0, -1) e (0, 0, 1). > > Resolvi da seguinte forma: > (-3, 1, 2) = a (1, 0, -1) + b (0, 0, 1) > = (a, 0, -a) + (0, 0, b) > = (a, 0, -a + b) > (-3, 1, 2) = (a, 0, -a+b) > > Impossível, pois não podemos fazer 0 = 1. > Onde estou errando? Ou a afirmação é realmente > falsa? > > Grato, > Henrique. > > ========================================================================= > Instruções para entrar na lista, sair da lista e > usar a lista em > http://www.mat.puc-rio.br/~nicolau/olimp/obm-l.html > O administrador desta lista é > > ========================================================================= _______________________________________________________________________ Yahoo! Mail O melhor e-mail gratuito da internet: 6MB de espaço, antivírus, acesso POP3, filtro contra spam. http://br.mail.yahoo.com/ ========================================================================= Instruções para entrar na lista, sair da lista e usar a lista em http://www.mat.puc-rio.br/~nicolau/olimp/obm-l.html O administrador desta lista é ========================================================================= From owner-obm-l@sucuri.mat.puc-rio.br Fri Apr 18 08:47:59 2003 Return-Path: Received: (from majordom@localhost) by sucuri.mat.puc-rio.br (8.9.3/8.9.3) id IAA14254 for obm-l-MTTP; Fri, 18 Apr 2003 08:46:06 -0300 Received: from mediterraneo.rjnet.com.br (mediterraneo.rjnet.com.br [200.152.115.30]) by sucuri.mat.puc-rio.br (8.9.3/8.9.3) with ESMTP id IAA14249 for ; Fri, 18 Apr 2003 08:46:02 -0300 Received: from locutus.rjnet.com.br (root@locutus.rjnet.com.br [200.152.115.10]) by mediterraneo.rjnet.com.br (8.11.4/8.11.4) with ESMTP id h3IBikv18359 for ; Fri, 18 Apr 2003 08:44:46 -0300 Received: from computador (rosamaior.rjnet.com.br [200.152.116.14]) by locutus.rjnet.com.br (8.11.2/8.11.2) with SMTP id h3IBOmb14797 for ; Fri, 18 Apr 2003 08:24:48 -0300 Message-ID: <006701c3059f$fb5a1670$158c000a@computador> From: "Felipe Villela Dias" To: References: <590HDooKj1664S20.1050419370@uwdvg007.cms.usa.net> <20030416043622.42259.qmail@web13705.mail.yahoo.com> <20030416100912.GA21143@linux.ime.usp.br> <007501c3043e$ea749c20$0e00a8c0@hotlink.com.br> <20030416181208.B2662@sucuri.mat.puc-rio.br> <001a01c30463$05622f60$0e00a8c0@hotlink.com.br> <20030416214351.A9315@sucuri.mat.puc-rio.br> Subject: Re: [obm-l] riemann Date: Fri, 18 Apr 2003 08:45:16 -0300 MIME-Version: 1.0 Content-Type: text/plain; charset="iso-8859-1" Content-Transfer-Encoding: 8bit X-Priority: 3 X-MSMail-Priority: Normal X-Mailer: Microsoft Outlook Express 6.00.2720.3000 X-MimeOLE: Produced By Microsoft MimeOLE V6.00.2600.0000 Sender: owner-obm-l@sucuri.mat.puc-rio.br Precedence: bulk Reply-To: obm-l@mat.puc-rio.br Olá a todos, No texto do NY Times, dizem que a conjecutra de Poincaré é um dos 7 problemas mais importantes do milenio. Quais seriam os outros seis??? abraços ----- Original Message ----- From: "Nicolau C. Saldanha" To: Sent: Wednesday, April 16, 2003 9:43 PM Subject: Re: [obm-l] riemann > On Wed, Apr 16, 2003 at 06:56:22PM -0300, gabriel wrote: > > Caro Nicolau, > > So nao entendi uma coisa; porque estes preprints nao devem ser levados > > a serios?? Fisicos nao resolvem problemas de matematica?? : ) > > Em muitos casos sim. Mas *estes* preprints em particular são escritos > de maneira muito imprecisa demais. > > A pedido do Gugu segue abaixo o artigo do NYTimes. > > []s, N. > > ========================================================================== > > The New York Times > > April 15, 2003 > > Celebrated Math Problem Solved, Russian Reports > > By SARA ROBINSON > > > A Russian mathematician is reporting that he has proved the Poincaré > Conjecture, one of the most famous unsolved problems in mathematics. > The mathematician, Dr. Grigori Perelman of the Steklov Institute of > Mathematics of the Russian Academy of Sciences in St. Petersburg, is > describing his work in a series of papers, not yet completed. > It will be months before the proof can be thoroughly checked. But if true, > it will verify a statement about three-dimensional objects that has haunted > mathematicians for nearly a century, and its consequences will reverberate > through geometry and physics. > If his proof is accepted for publication in a refereed research journal and > survives two years of scrutiny, Dr. Perelman could be eligible for a $1 > million prize sponsored by the Clay Mathematics Institute in Cambridge, > Mass., for solving what the institute identifies as one of the seven most > important unsolved mathematics problems of the millennium. > Rumors about Dr. Perelman's work have been circulating since November, whe n > he posted the first of his papers reporting the result on an Internet > preprint server. > Last week at the Massachusetts Institute of Technology, he gave his first > formal lectures on his work to a packed auditorium. Dr. Perelman will give > another lecture series at the State University of New York at Stony Brook > starting on Monday. > Dr. Perelman declined to be interviewed, saying publicity would be > premature. > For two months, Dr. Tomasz S. Mrowka, a mathematician at M.I.T., has been > attending a seminar on Dr. Perelman's work, which relies on ideas pioneered > by another mathematician, Richard Hamilton. So far, Dr. Mrowka said, every > time someone brings up an issue or objection, Dr. Perelman has a clear and > succinct response. > "It's not certain, but we're taking it very seriously," Dr. Mrowka said. > "He's obviously thought about this stuff very hard for a long time, and it > will be very hard to find any mistakes." > Formulated by the French mathematician Henri Poincaré in 1904, the Poincaré > Conjecture is a central question in topology, the study of the geometrical > properties of objects that do not change when the object is stretched, > twisted or shrunk. > The hollow shell of the surface of the earth is what topologists would call > a two-dimensional sphere. It has the property that every lasso of string > encircling it can be pulled tight to one spot. > On the surface of a doughnut, by contrast, a lasso passing through the hole > in the center cannot be shrunk to a point without cutting through the > surface. > Since the 19th century, mathematicians have known that the sphere is the > only bounded two-dimensional space with this property, but what about higher > dimensions? > The Poincaré Conjecture makes a corresponding statement about the > three-dimensional sphere, a concept that is a stretch for the > nonmathematician to visualize. It says, essentially, that the > three-dimensional sphere is the only bounded three-dimensional space with no > holes. > "The hard part is how to tell globally what a space looks like when you can > only see a little piece of it at a time," said Dr. Benson Farb, a professor > of mathematics at the University of Chicago. "It was pretty reasonable to > think the earth was flat." > That conjecture is notorious for the many "solutions" that later proved > false. Indeed, Poincaré himself demonstrated that his earliest version of > his conjecture was wrong. Since then, dozens of mathematicians have asserted > that they had proofs until experts found fatal flaws. > Although many experts say they are excited and hopeful about Dr. Perelman's > effort, they also urge caution, noting that not all of the proof has been > written down and that even the most reliable researchers make mistakes. > That was the case in 1993 with Dr. Andrew J. Wiles, the Princeton professor > whose celebrated proof for Fermat's Last Theorem turned out to have a > serious gap that was repaired after months of effort by Dr. Wiles and a > former student, Dr. Richard Taylor. > Dr. Perelman's results go well beyond a solution to the problem at hand, as > did those of Dr. Wiles. Dr. Perelman's results say he has proved a much > broader conjecture about the geometry of three-dimensional spaces made in > the 1970's. The Poincaré Conjecture is but a small part of that. > Dr. Perelman's personal story has parallels to that of Dr. Wiles, who, > without confiding in his colleagues, worked alone in his attic on Fermat's > Last Theorem. Though his early work has earned him a reputation as a > brilliant mathematician, Dr. Perelman spent the last eight years sequestered > in Russia, not publishing. > In his paper posted in November, Dr. Perelman, now in his late 30's, thanks > the Courant Institute at New York University, SUNY Stony Brook and the > University of California at Berkeley, because his savings from visiting > positions at those institutions helped support him in Russia. > His papers say that he has proved what is known as the Geometrization > Conjecture, a complete characterization of the geometry of three-dimensional > spaces. > Since the 19th century, mathematicians have known that a type of > two-dimensional space called a manifold can be given a rigid geometric > structure that looks the same everywhere. Mathematicians could list all the > possible shapes for two-dimensional manifolds and explain how a creature > living on the surface of one can tell what kind of space he is on. > In the 1950's, however, a Russian mathematician proved that the problem was > impossible to resolve in four dimensions and that even for three dimensions, > the question looked hopelessly complex. > In the early 1970's, Dr. William P. Thurston, a professor at the University > of California at Davis, conjectured that three-dimensional manifolds are > composed of many homogeneous pieces that can be put together only in > prescribed ways and proved that in many cases his conjecture was correct. > Dr. Thurston won a Fields Medal, the highest honor in mathematics, for his > work. > Dr. Perelman's work, if correct, would provide the final piece of a complete > description of the structure of three-dimensional manifolds and, almost as > an afterthought, would resolve Poincaré's famous question. Dr. Perelman's > approach uses a technique known as the Ricci flow, devised by Dr. Hamilton, > who is now at Columbia University. > The Ricci flow is an averaging process used to smooth out the bumps of a > manifold and make it look more uniform. Dr. Hamilton uses the Ricci flow to > prove the Geometrization Conjecture in some cases and outlined a general > program of how it could be used to prove the Geometrization Conjecture in > all cases. He ran into problems, however, coping with certain types of large > lumps that tended to grow uncontrollably under the averaging process. > "What Perelman has done is to figure out some new and interesting ways to > tame these singularities," Dr. Mrowka said. "His work relies heavily on > Hamilton's work but makes amazing new contributions to that program." > If Dr. Perelman succeeds in resolving Poincaré, he will probably share the > Clay Mathematics Institute Award with Dr. Hamilton, mathematicians said. > Even if Dr. Perelman's work does not prove the Geometrization Conjecture, > mathematicians said, it is clear that his work will make a substantial > contribution to mathematics. > "This is one of those happy circumstances where it's going to be fun no > matter what," Dr. Mrowka said. "Either he's done it or he's made some really > significant progress, and we're going to learn from it." > > > > > ========================================================================= > Instruções para entrar na lista, sair da lista e usar a lista em > http://www.mat.puc-rio.br/~nicolau/olimp/obm-l.html > O administrador desta lista é > ========================================================================= > ========================================================================= Instruções para entrar na lista, sair da lista e usar a lista em http://www.mat.puc-rio.br/~nicolau/olimp/obm-l.html O administrador desta lista é ========================================================================= From owner-obm-l@sucuri.mat.puc-rio.br Fri Apr 18 11:18:33 2003 Return-Path: Received: (from majordom@localhost) by sucuri.mat.puc-rio.br (8.9.3/8.9.3) id LAA15430 for obm-l-MTTP; Fri, 18 Apr 2003 11:17:07 -0300 Received: from puma.unisys.com.br (smtp.unisys.com.br [200.220.64.7]) by sucuri.mat.puc-rio.br (8.9.3/8.9.3) with ESMTP id LAA15426 for ; Fri, 18 Apr 2003 11:17:04 -0300 Received: from jf (riohiper01p245.uninet.com.br [200.220.2.245]) by puma.unisys.com.br (8.12.9/8.12.3) with SMTP id h3IEGX0e029028 for ; Fri, 18 Apr 2003 11:16:33 -0300 (EST) X-Spam-Filter: check_local@puma.unisys.com.br by digitalanswers.org Message-ID: <006801c305b5$696741a0$f502dcc8@jf> From: "Jose Francisco Guimaraes Costa" To: References: <590HDooKj1664S20.1050419370@uwdvg007.cms.usa.net> <20030416043622.42259.qmail@web13705.mail.yahoo.com> <20030416100912.GA21143@linux.ime.usp.br> <007501c3043e$ea749c20$0e00a8c0@hotlink.com.br> <20030416181208.B2662@sucuri.mat.puc-rio.br> <001a01c30463$05622f60$0e00a8c0@hotlink.com.br> <20030416214351.A9315@sucuri.mat.puc-rio.br> <006701c3059f$fb5a1670$158c000a@computador> Subject: Re: [obm-l] riemann Date: Fri, 18 Apr 2003 11:18:39 -0300 MIME-Version: 1.0 Content-Type: text/plain; charset="iso-8859-1" Content-Transfer-Encoding: 8bit X-Priority: 3 X-MSMail-Priority: Normal X-Mailer: Microsoft Outlook Express 6.00.2800.1106 X-MimeOLE: Produced By Microsoft MimeOLE V6.00.2800.1106 Sender: owner-obm-l@sucuri.mat.puc-rio.br Precedence: bulk Reply-To: obm-l@mat.puc-rio.br Veja-os em http://www.claymath.org/Millennium_Prize_Problems/ JF ----- Original Message ----- From: "Felipe Villela Dias" To: Sent: Friday, April 18, 2003 8:45 AM Subject: Re: [obm-l] riemann > Olá a todos, > No texto do NY Times, dizem que a conjecutra de Poincaré é um dos 7 > problemas mais importantes do milenio. Quais seriam os outros seis??? > abraços ========================================================================= Instruções para entrar na lista, sair da lista e usar a lista em http://www.mat.puc-rio.br/~nicolau/olimp/obm-l.html O administrador desta lista é ========================================================================= From owner-obm-l@sucuri.mat.puc-rio.br Fri Apr 18 13:07:57 2003 Return-Path: Received: (from majordom@localhost) by sucuri.mat.puc-rio.br (8.9.3/8.9.3) id NAA16913 for obm-l-MTTP; Fri, 18 Apr 2003 13:06:28 -0300 Received: from sidney4.bol.com.br (sidney4.bol.com.br [200.221.24.120]) by sucuri.mat.puc-rio.br (8.9.3/8.9.3) with ESMTP id NAA16909 for ; Fri, 18 Apr 2003 13:06:25 -0300 Received: from bol.com.br (200.221.24.136) by sidney4.bol.com.br (5.1.071) id 3E990A66000EE467 for obm-l@mat.puc-rio.br; Fri, 18 Apr 2003 13:05:54 -0300 Date: Fri, 18 Apr 2003 13:05:54 -0300 Message-Id: Subject: [obm-l] L.I MIME-Version: 1.0 Content-Type: text/plain;charset="iso-8859-1" From: "marcio.lis" To: obm-l@mat.puc-rio.br X-XaM3-API-Version: 2.4 R3 ( B4 ) X-SenderIP: 200.151.172.172 Content-Transfer-Encoding: 8bit X-MIME-Autoconverted: from quoted-printable to 8bit by sucuri.mat.puc-rio.br id NAA16910 Sender: owner-obm-l@sucuri.mat.puc-rio.br Precedence: bulk Reply-To: obm-l@mat.puc-rio.br em uma questão do livro do elon de algébra linear ele pede p/ mostrar que:(1,e^x,e2^x,e^3x,e^4x) é L.I e coloca como sugestão sair derivando e dividindo por e^x e consegui chegar a solucao por ai porém eu gostaria de saber se eu posso fazer da seguinte forma:substituir e^x por y ai fica um polinômio em y de um lado e do outro lado 0.eu posso concluir por igualdade de polinômio que os coeficientes são todos nulos?e que portanto o conjunto é L.I __________________________________________________________________________ Seleção de Softwares UOL. 10 softwares escolhidos pelo UOL para você e sua família. http://www.uol.com.br/selecao ========================================================================= Instruções para entrar na lista, sair da lista e usar a lista em http://www.mat.puc-rio.br/~nicolau/olimp/obm-l.html O administrador desta lista é ========================================================================= From owner-obm-l@sucuri.mat.puc-rio.br Fri Apr 18 14:18:06 2003 Return-Path: Received: (from majordom@localhost) by sucuri.mat.puc-rio.br (8.9.3/8.9.3) id OAA17984 for obm-l-MTTP; Fri, 18 Apr 2003 14:16:35 -0300 Received: from seki.bol.com.br (seki.bol.com.br [200.221.24.26]) by sucuri.mat.puc-rio.br (8.9.3/8.9.3) with ESMTP id OAA17980 for ; Fri, 18 Apr 2003 14:16:27 -0300 Received: from bol.com.br (200.221.24.131) by seki.bol.com.br (5.1.071) id 3E9EA3BD00058C3C for obm-l@mat.puc-rio.br; Fri, 18 Apr 2003 14:15:57 -0300 Date: Fri, 18 Apr 2003 14:15:57 -0300 Message-Id: Subject: Re:[obm-l] pontos colineares, cade o erro? MIME-Version: 1.0 Content-Type: text/plain;charset="iso-8859-1" From: "osvaldomellospq" To: obm-l@mat.puc-rio.br X-XaM3-API-Version: 2.4 R3 ( B4 ) X-SenderIP: 200.153.155.93 Content-Transfer-Encoding: 8bit X-MIME-Autoconverted: from quoted-printable to 8bit by sucuri.mat.puc-rio.br id OAA17981 Sender: owner-obm-l@sucuri.mat.puc-rio.br Precedence: bulk Reply-To: obm-l@mat.puc-rio.br > > ---------- Início da mensagem original ----------- > De: owner-obm-l@sucuri.mat.puc-rio.br > Para: obm-l@mat.puc-rio.br > Cc: > Data: Tue, 15 Apr 2003 22:54:33 -0300 > Assunto: [obm-l] pontos colineares, cade o erro? > Ola, > > mais uma vez estou cometendo algum erro de atencao (pro vavelmente) > > "Mostre que A(a,-3a), B(a+3,-3a-1) e C(a+5,-3a- 2) sao colineares para todo valor real de a" > > Mas nao foi o que encontrei, cheguei a considerar a=2, e meu resultado nao foram pontos colineares... > > Se alguem puder me ajudar a encontrar meu erro, agradec o > > Tentei atraves da relacao: > (xb-xa)/(xc-xb)=(yb-ya)/(yc-yb) > cheguei a 3/2=1 (?!?) > > E tambem tentei atraves de determinates, mas cheguei a nada também > > E quando considerei a=2 na determinante, cheguei num re sultado 27, que deveria ser 0... > > Obrigado, > Ariel > > ======================================================= ================== > Instruções para entrar na lista, sair da lista e usar a lista em > http://www.mat.puc-rio.br/~nicolau/olimp/obm-l.html > O administrador desta lista é > ======================================================= ================== Ariel, esta aí uma dica mto simples vc já deve ter ouvido falar de HPs né? Pronto é simples joga as coordenadas dos três pontos lá e impões que formem uma reta, daí vc vai ter os valores possíveis de a. Se para todo real a união dos três pontos for uma reta então a=R > __________________________________________________________________________ Seleção de Softwares UOL. 10 softwares escolhidos pelo UOL para você e sua família. http://www.uol.com.br/selecao ========================================================================= Instruções para entrar na lista, sair da lista e usar a lista em http://www.mat.puc-rio.br/~nicolau/olimp/obm-l.html O administrador desta lista é ========================================================================= From owner-obm-l@sucuri.mat.puc-rio.br Fri Apr 18 15:16:23 2003 Return-Path: Received: (from majordom@localhost) by sucuri.mat.puc-rio.br (8.9.3/8.9.3) id PAA19060 for obm-l-MTTP; Fri, 18 Apr 2003 15:14:57 -0300 Received: from sang.bol.com.br (sang.bol.com.br [200.221.24.24]) by sucuri.mat.puc-rio.br (8.9.3/8.9.3) with ESMTP id PAA19056 for ; Fri, 18 Apr 2003 15:14:55 -0300 Received: from bol.com.br (200.221.24.136) by sang.bol.com.br (5.1.071) id 3E76704C008B425E for obm-l@mat.puc-rio.br; Fri, 18 Apr 2003 15:14:24 -0300 Date: Fri, 18 Apr 2003 15:14:24 -0300 Message-Id: Subject: [obm-l] =?iso-8859-1?q?Constru=E7l=E3o=21=21=21?= MIME-Version: 1.0 Content-Type: text/plain;charset="iso-8859-1" From: "cfgauss77" To: "Lista OBM" X-XaM3-API-Version: 2.4 R3 ( B4 ) X-SenderIP: 200.193.251.183 Content-Transfer-Encoding: 8bit X-MIME-Autoconverted: from quoted-printable to 8bit by sucuri.mat.puc-rio.br id PAA19057 Sender: owner-obm-l@sucuri.mat.puc-rio.br Precedence: bulk Reply-To: obm-l@mat.puc-rio.br Sabemos que é possível localizar na reta real o iarracional sqrt(2) usando regua e compasso. Minha dúvida é se o mesmo pode ser feito para localizar o irracional Pi (3,14159...). Desde já, agradeço a quem puder me ajudar!!! __________________________________________________________________________ Seleção de Softwares UOL. 10 softwares escolhidos pelo UOL para você e sua família. http://www.uol.com.br/selecao ========================================================================= Instruções para entrar na lista, sair da lista e usar a lista em http://www.mat.puc-rio.br/~nicolau/olimp/obm-l.html O administrador desta lista é ========================================================================= From owner-obm-l@sucuri.mat.puc-rio.br Fri Apr 18 15:16:29 2003 Return-Path: Received: (from majordom@localhost) by sucuri.mat.puc-rio.br (8.9.3/8.9.3) id PAA19076 for obm-l-MTTP; Fri, 18 Apr 2003 15:15:12 -0300 Received: from sang.bol.com.br (sang.bol.com.br [200.221.24.24]) by sucuri.mat.puc-rio.br (8.9.3/8.9.3) with ESMTP id PAA19072 for ; Fri, 18 Apr 2003 15:15:09 -0300 Received: from bol.com.br (200.221.24.136) by sang.bol.com.br (5.1.071) id 3E76704C008B4278 for obm-l@mat.puc-rio.br; Fri, 18 Apr 2003 15:14:38 -0300 Date: Fri, 18 Apr 2003 15:14:38 -0300 Message-Id: Subject: [obm-l] =?iso-8859-1?q?Constru=E7=E3o=21=21=21?= MIME-Version: 1.0 Content-Type: text/plain;charset="iso-8859-1" From: "cfgauss77" To: "Lista OBM" X-XaM3-API-Version: 2.4 R3 ( B4 ) X-SenderIP: 200.193.251.183 Content-Transfer-Encoding: 8bit X-MIME-Autoconverted: from quoted-printable to 8bit by sucuri.mat.puc-rio.br id PAA19073 Sender: owner-obm-l@sucuri.mat.puc-rio.br Precedence: bulk Reply-To: obm-l@mat.puc-rio.br Sabemos que é possível localizar na reta real o iarracional sqrt(2) usando regua e compasso. Minha dúvida é se o mesmo pode ser feito para localizar o irracional Pi (3,14159...). Desde já, agradeço a quem puder me ajudar!!! __________________________________________________________________________ Seleção de Softwares UOL. 10 softwares escolhidos pelo UOL para você e sua família. http://www.uol.com.br/selecao ========================================================================= Instruções para entrar na lista, sair da lista e usar a lista em http://www.mat.puc-rio.br/~nicolau/olimp/obm-l.html O administrador desta lista é ========================================================================= From owner-obm-l@sucuri.mat.puc-rio.br Fri Apr 18 17:39:02 2003 Return-Path: Received: (from majordom@localhost) by sucuri.mat.puc-rio.br (8.9.3/8.9.3) id RAA21441 for obm-l-MTTP; Fri, 18 Apr 2003 17:37:17 -0300 Received: from saks.bol.com.br (saks.bol.com.br [200.221.24.16]) by sucuri.mat.puc-rio.br (8.9.3/8.9.3) with ESMTP id RAA21427 for ; Fri, 18 Apr 2003 17:37:12 -0300 Received: from xx (200.221.24.191) by saks.bol.com.br (5.1.071) id 3E766E3A009CC8F1 for obm-l@mat.puc-rio.br; Fri, 18 Apr 2003 17:36:42 -0300 Message-ID: <001501c305ea$3b3d38e0$f4befea9@xx> From: "Blue Ice" To: Subject: [obm-l] =?iso-8859-1?Q?Quando_ser=E1_a_olimp=EDada_Internacional=3F=3F?= Date: Fri, 18 Apr 2003 17:36:45 -0300 MIME-Version: 1.0 Content-Type: multipart/alternative; boundary="----=_NextPart_000_0012_01C305D1.14FA0820" X-Priority: 3 X-MSMail-Priority: Normal X-Mailer: Microsoft Outlook Express 6.00.2600.0000 X-MimeOLE: Produced By Microsoft MimeOLE V6.00.2600.0000 X-Sender-IP: 200.148.35.125 Sender: owner-obm-l@sucuri.mat.puc-rio.br Precedence: bulk Reply-To: obm-l@mat.puc-rio.br This is a multi-part message in MIME format. ------=_NextPart_000_0012_01C305D1.14FA0820 Content-Type: text/plain; charset="iso-8859-1" Content-Transfer-Encoding: quoted-printable ------=_NextPart_000_0012_01C305D1.14FA0820 Content-Type: text/html; charset="iso-8859-1" Content-Transfer-Encoding: quoted-printable
 
------=_NextPart_000_0012_01C305D1.14FA0820-- ========================================================================= Instruções para entrar na lista, sair da lista e usar a lista em http://www.mat.puc-rio.br/~nicolau/olimp/obm-l.html O administrador desta lista é ========================================================================= From owner-obm-l@sucuri.mat.puc-rio.br Fri Apr 18 18:00:46 2003 Return-Path: Received: (from majordom@localhost) by sucuri.mat.puc-rio.br (8.9.3/8.9.3) id RAA21935 for obm-l-MTTP; Fri, 18 Apr 2003 17:59:23 -0300 Received: from krypton.hosting4u.net (krypton.hosting4u.net [209.15.2.78]) by sucuri.mat.puc-rio.br (8.9.3/8.9.3) with ESMTP id RAA21931 for ; Fri, 18 Apr 2003 17:59:20 -0300 Received: from gargamel (200-158-200-129.dsl.telesp.net.br [200.158.200.129]) by krypton.hosting4u.net (Postfix) with ESMTP id B762FA0C79 for ; Fri, 18 Apr 2003 15:58:45 -0500 (CDT) Message-ID: <200304181802160540.00122780@smtp.watersportsbrazil.com> X-Mailer: Calypso Version 3.30.00.00 (3) Date: Fri, 18 Apr 2003 18:02:16 -0300 From: "Ariel de Silvio" To: obm-l@mat.puc-rio.br Subject: [obm-l] ITA - Off-Topic Mime-Version: 1.0 Content-Type: text/plain; charset="us-ascii" Content-Transfer-Encoding: 8bit X-MIME-Autoconverted: from quoted-printable to 8bit by sucuri.mat.puc-rio.br id RAA21932 Sender: owner-obm-l@sucuri.mat.puc-rio.br Precedence: bulk Reply-To: obm-l@mat.puc-rio.br Olá, sei que é off-topic, por isso pretendo ser breve... Sei que já foi discutido algumas vezes aqui nessa lista esse assunto, e sei que alguns participantes da lista cursam ou cursaram o ITA... gostaria de conversar com algumas pessoas a esse respeito, se puderem me enviar emails em pvt... ariel@watersportsbrazil.com Quero fazer ITA, estou cursando o 3o colegial do Colegio Rio Branco (SP), e gostaria de informações de pessoas que ja passaram por isso ou passam ainda... como é? se é tão maluco quanto alguns dizem... Como se preparar... e coisas do tipo... Obrigado aos que puderem me ajudar... Ariel de Silvio ========================================================================= Instruções para entrar na lista, sair da lista e usar a lista em http://www.mat.puc-rio.br/~nicolau/olimp/obm-l.html O administrador desta lista é ========================================================================= From owner-obm-l@sucuri.mat.puc-rio.br Fri Apr 18 19:04:47 2003 Return-Path: Received: (from majordom@localhost) by sucuri.mat.puc-rio.br (8.9.3/8.9.3) id TAA23139 for obm-l-MTTP; Fri, 18 Apr 2003 19:02:29 -0300 Received: from sidney3.bol.com.br (sidney3.bol.com.br [200.221.24.143]) by sucuri.mat.puc-rio.br (8.9.3/8.9.3) with ESMTP id TAA23133 for ; Fri, 18 Apr 2003 19:02:21 -0300 Received: from xx (200.221.24.192) by sidney3.bol.com.br (5.1.071) id 3E6CB30F00883851 for obm-l@mat.puc-rio.br; Fri, 18 Apr 2003 19:01:49 -0300 Message-ID: <000801c305f6$203761e0$f4befea9@xx> From: "Blue Ice" To: References: <200304181802160540.00122780@smtp.watersportsbrazil.com> Subject: Re: [obm-l] ITA - Off-Topic Date: Fri, 18 Apr 2003 19:01:54 -0300 MIME-Version: 1.0 Content-Type: text/plain; charset="iso-8859-1" Content-Transfer-Encoding: 8bit X-Priority: 3 X-MSMail-Priority: Normal X-Mailer: Microsoft Outlook Express 6.00.2600.0000 X-MimeOLE: Produced By Microsoft MimeOLE V6.00.2600.0000 X-Sender-IP: 200.148.35.162 Sender: owner-obm-l@sucuri.mat.puc-rio.br Precedence: bulk Reply-To: obm-l@mat.puc-rio.br ----- Original Message ----- From: "Ariel de Silvio" To: Sent: Friday, April 18, 2003 6:02 PM Subject: [obm-l] ITA - Off-Topic > E-mail Premium BOL > Antivírus, anti-spam e até 100 MB de espaço. Assine já! > http://email.bol.com.br/ > Olá, > > sei que é off-topic, por isso pretendo ser breve... > > Sei que já foi discutido algumas vezes aqui nessa lista esse assunto, e sei que alguns participantes da lista cursam ou cursaram o ITA... gostaria de conversar com algumas pessoas a esse respeito, se puderem me enviar emails em pvt... ariel@watersportsbrazil.com > > Quero fazer ITA, estou cursando o 3o colegial do Colegio Rio Branco (SP), e gostaria de informações de pessoas que ja passaram por isso ou passam ainda... como é? se é tão maluco quanto alguns dizem... Como se preparar... e coisas do tipo... > > Obrigado aos que puderem me ajudar... > Ariel de Silvio > > ========================================================================= > Instruções para entrar na lista, sair da lista e usar a lista em > http://www.mat.puc-rio.br/~nicolau/olimp/obm-l.html > O administrador desta lista é Olá colega, tb gostaria de cursar o ITA, tenho ICQ se quiser conversar.Acredito que dividimos a mesma paixão pela matemática. []´s Ice ICQ:177782914 Email:Vinicius84@hotmail.com Win XP Home Full/Speedy 256kbps ========================================================================= Instruções para entrar na lista, sair da lista e usar a lista em http://www.mat.puc-rio.br/~nicolau/olimp/obm-l.html O administrador desta lista é ========================================================================= From owner-obm-l@sucuri.mat.puc-rio.br Fri Apr 18 19:13:11 2003 Return-Path: Received: (from majordom@localhost) by sucuri.mat.puc-rio.br (8.9.3/8.9.3) id TAA23308 for obm-l-MTTP; Fri, 18 Apr 2003 19:11:54 -0300 Received: from artemis.opendf.com.br (artemis.opengate.com.br [200.181.71.14]) by sucuri.mat.puc-rio.br (8.9.3/8.9.3) with ESMTP id TAA23304 for ; Fri, 18 Apr 2003 19:11:50 -0300 Received: from localhost (localhost [127.0.0.1]) by artemis.opendf.com.br (Postfix) with ESMTP id B32A42BECF for ; Fri, 18 Apr 2003 19:12:19 -0300 (BRT) Received: from artemis.opendf.com.br ([127.0.0.1]) by localhost (artemis.opengate.com.br [127.0.0.1:10024]) (amavisd-new) with ESMTP id 05516-08 for ; Fri, 18 Apr 2003 19:12:18 -0300 (BRT) Received: from computer (200-181-088-055.bsace7001.dsl.brasiltelecom.net.br [200.181.88.55]) by artemis.opendf.com.br (Postfix) with ESMTP id 3B0622BECA for ; Fri, 18 Apr 2003 19:12:18 -0300 (BRT) From: "Artur Costa Steiner" To: Subject: RE: [obm-l] L.I Date: Fri, 18 Apr 2003 19:11:19 -0300 Organization: Steiner Consultoria LTDA Message-ID: <007c01c305f7$71464f00$9865fea9@computer> MIME-Version: 1.0 Content-Type: text/plain; charset="iso-8859-1" X-Priority: 3 (Normal) X-MSMail-Priority: Normal X-Mailer: Microsoft Outlook, Build 10.0.2627 X-MimeOLE: Produced By Microsoft MimeOLE V6.00.2800.1106 In-Reply-To: Importance: Normal X-Virus-Scanned: by amavisd-new Content-Transfer-Encoding: 8bit X-MIME-Autoconverted: from quoted-printable to 8bit by sucuri.mat.puc-rio.br id TAA23305 Sender: owner-obm-l@sucuri.mat.puc-rio.br Precedence: bulk Reply-To: obm-l@mat.puc-rio.br De fato, se e^x = y, então e^2x = y^2 e assim sucessivamente. Se existirem números a1...a4 tais a1 y + ...a4y^4 = 0 para todo x, isto significa que a1 y + ...a4y^4 = 0 para todo y>0. A única possibilidade e entao que o polinomio em y seja identicamente nulo, com a1 = a2= a3= a4 = 0. Logo o conjunto eh LI. Podemos facilmente extender tal conclusoa para qualquer natural n. Um abraco Artur >-----Original Message----- >From: owner-obm-l@sucuri.mat.puc-rio.br [mailto:owner-obm-l@sucuri.mat.puc- >rio.br] On Behalf Of marcio.lis >Sent: Friday, April 18, 2003 1:06 PM >To: obm-l@mat.puc-rio.br >Subject: [obm-l] L.I > > em uma questão do livro do elon de algébra linear ele >pede p/ mostrar que:(1,e^x,e2^x,e^3x,e^4x) é L.I e >coloca como sugestão sair derivando e dividindo por e^x >e consegui chegar a solucao por ai porém eu gostaria de >saber se eu posso fazer da seguinte forma:substituir e^x >por y ai fica um polinômio em y de um lado e do outro >lado 0.eu posso concluir por igualdade de polinômio que >os coeficientes são todos nulos?e que portanto o >conjunto é L.I > > >_______________________________________________________________________ ___ >Seleção de Softwares UOL. >10 softwares escolhidos pelo UOL para você e sua família. >http://www.uol.com.br/selecao > > >======================================================================= == >Instruções para entrar na lista, sair da lista e usar a lista em >http://www.mat.puc-rio.br/~nicolau/olimp/obm-l.html >O administrador desta lista é >======================================================================= == ========================================================================= Instruções para entrar na lista, sair da lista e usar a lista em http://www.mat.puc-rio.br/~nicolau/olimp/obm-l.html O administrador desta lista é ========================================================================= From owner-obm-l@sucuri.mat.puc-rio.br Fri Apr 18 19:29:27 2003 Return-Path: Received: (from majordom@localhost) by sucuri.mat.puc-rio.br (8.9.3/8.9.3) id TAA23924 for obm-l-MTTP; Fri, 18 Apr 2003 19:28:08 -0300 Received: from artemis.opendf.com.br (artemis.opengate.com.br [200.181.71.14]) by sucuri.mat.puc-rio.br (8.9.3/8.9.3) with ESMTP id TAA23920 for ; Fri, 18 Apr 2003 19:28:05 -0300 Received: from localhost (localhost [127.0.0.1]) by artemis.opendf.com.br (Postfix) with ESMTP id 997212BECD for ; Fri, 18 Apr 2003 19:28:34 -0300 (BRT) Received: from artemis.opendf.com.br ([127.0.0.1]) by localhost (artemis.opengate.com.br [127.0.0.1:10024]) (amavisd-new) with ESMTP id 06124-08 for ; Fri, 18 Apr 2003 19:28:33 -0300 (BRT) Received: from computer (200-181-088-055.bsace7001.dsl.brasiltelecom.net.br [200.181.88.55]) by artemis.opendf.com.br (Postfix) with ESMTP id 4A8682BEC2 for ; Fri, 18 Apr 2003 19:28:33 -0300 (BRT) From: "Artur Costa Steiner" To: Subject: RE: [obm-l] pontos colineares, cade o erro? Date: Fri, 18 Apr 2003 19:27:32 -0300 Organization: Steiner Consultoria LTDA Message-ID: <007d01c305f9$b6505760$9865fea9@computer> MIME-Version: 1.0 Content-Type: text/plain; charset="us-ascii" Content-Transfer-Encoding: 7bit X-Priority: 3 (Normal) X-MSMail-Priority: Normal X-Mailer: Microsoft Outlook, Build 10.0.2627 X-MimeOLE: Produced By Microsoft MimeOLE V6.00.2800.1106 In-Reply-To: Importance: Normal X-Virus-Scanned: by amavisd-new Sender: owner-obm-l@sucuri.mat.puc-rio.br Precedence: bulk Reply-To: obm-l@mat.puc-rio.br >> Ola, >> >> mais uma vez estou cometendo algum erro de atencao (pro >vavelmente) >> >> "Mostre que A(a,-3a), B(a+3,-3a-1) e C(a+5,-3a- >2) sao colineares para todo valor real de a" [Artur Costa Steiner] Temos que [(-3a-1)-(-3a)]/[(a+3)-a] = -1/3 para todo a. Por outro lado, [(-3a-2)-((-3a-1)/[[(a+5)-(a+3)] = -1/2 =/= -1/3. O que concluimos e que os pontos NAO sao colineares para NENHUM real a... Tem certeza de que o enunciado estah correto? Artur ========================================================================= Instruções para entrar na lista, sair da lista e usar a lista em http://www.mat.puc-rio.br/~nicolau/olimp/obm-l.html O administrador desta lista é ========================================================================= From owner-obm-l@sucuri.mat.puc-rio.br Fri Apr 18 19:59:10 2003 Return-Path: Received: (from majordom@localhost) by sucuri.mat.puc-rio.br (8.9.3/8.9.3) id TAA24987 for obm-l-MTTP; Fri, 18 Apr 2003 19:57:46 -0300 Received: from hotmail.com (oe14.law10.hotmail.com [64.4.14.118]) by sucuri.mat.puc-rio.br (8.9.3/8.9.3) with ESMTP id TAA24983 for ; Fri, 18 Apr 2003 19:57:43 -0300 Received: from mail pickup service by hotmail.com with Microsoft SMTPSVC; Fri, 18 Apr 2003 15:57:11 -0700 Received: from 64.60.139.18 by oe14.law10.hotmail.com with DAV; Fri, 18 Apr 2003 22:57:11 +0000 X-Originating-IP: [64.60.139.18] X-Originating-Email: [lrecova@hotmail.com] From: =?iso-8859-1?Q?Leandro_Lacorte_Rec=F4va?= To: Subject: RE: [obm-l] ITA - Off-Topic Date: Fri, 18 Apr 2003 15:57:11 -0700 Message-ID: <001901c305fd$d85acd00$28029b9b@LeandroRecova> MIME-Version: 1.0 Content-Type: text/plain; charset="iso-8859-1" X-Priority: 3 (Normal) X-MSMail-Priority: Normal X-Mailer: Microsoft Outlook, Build 10.0.3416 X-MimeOLE: Produced By Microsoft MimeOLE V6.00.2800.1106 In-Reply-To: <000801c305f6$203761e0$f4befea9@xx> Importance: Normal X-OriginalArrivalTime: 18 Apr 2003 22:57:11.0854 (UTC) FILETIME=[D8CACCE0:01C305FD] Content-Transfer-Encoding: 8bit X-MIME-Autoconverted: from quoted-printable to 8bit by sucuri.mat.puc-rio.br id TAA24984 Sender: owner-obm-l@sucuri.mat.puc-rio.br Precedence: bulk Reply-To: obm-l@mat.puc-rio.br Um conselho: Se voces tem paixao pela matematica, facam um curso de matematica na PUC-RJ, UFRJ, UNICAMP, USP,UnB, etc...pois na engenharia so irao usa-la como ferramenta e no futuro usarao terno e gravata. Caso gostem, otimo, mas se querem conviver com a matematica, facam um curso de bacharelado em matematica. Leandro. -----Original Message----- From: owner-obm-l@sucuri.mat.puc-rio.br [mailto:owner-obm-l@sucuri.mat.puc-rio.br] On Behalf Of Blue Ice Sent: Friday, April 18, 2003 3:02 PM To: obm-l@mat.puc-rio.br Subject: Re: [obm-l] ITA - Off-Topic ----- Original Message ----- From: "Ariel de Silvio" To: Sent: Friday, April 18, 2003 6:02 PM Subject: [obm-l] ITA - Off-Topic > E-mail Premium BOL > Antivírus, anti-spam e até 100 MB de espaço. Assine já! > http://email.bol.com.br/ > Olá, > > sei que é off-topic, por isso pretendo ser breve... > > Sei que já foi discutido algumas vezes aqui nessa lista esse assunto, e sei que alguns participantes da lista cursam ou cursaram o ITA... gostaria de conversar com algumas pessoas a esse respeito, se puderem me enviar emails em pvt... ariel@watersportsbrazil.com > > Quero fazer ITA, estou cursando o 3o colegial do Colegio Rio Branco (SP), e gostaria de informações de pessoas que ja passaram por isso ou passam ainda... como é? se é tão maluco quanto alguns dizem... Como se preparar... e coisas do tipo... > > Obrigado aos que puderem me ajudar... > Ariel de Silvio > > ======================================================================== = > Instruções para entrar na lista, sair da lista e usar a lista em > http://www.mat.puc-rio.br/~nicolau/olimp/obm-l.html > O administrador desta lista é Olá colega, tb gostaria de cursar o ITA, tenho ICQ se quiser conversar.Acredito que dividimos a mesma paixão pela matemática. []´s Ice ICQ:177782914 Email:Vinicius84@hotmail.com Win XP Home Full/Speedy 256kbps ======================================================================== = Instruções para entrar na lista, sair da lista e usar a lista em http://www.mat.puc-rio.br/~nicolau/olimp/obm-l.html O administrador desta lista é ======================================================================== = ========================================================================= Instruções para entrar na lista, sair da lista e usar a lista em http://www.mat.puc-rio.br/~nicolau/olimp/obm-l.html O administrador desta lista é ========================================================================= From owner-obm-l@sucuri.mat.puc-rio.br Fri Apr 18 20:01:00 2003 Return-Path: Received: (from majordom@localhost) by sucuri.mat.puc-rio.br (8.9.3/8.9.3) id TAA25038 for obm-l-MTTP; Fri, 18 Apr 2003 19:59:40 -0300 Received: from Euler.impa.br (euler.impa.br [147.65.1.3]) by sucuri.mat.puc-rio.br (8.9.3/8.9.3) with ESMTP id TAA25034 for ; Fri, 18 Apr 2003 19:59:36 -0300 Received: from Gauss.impa.br (Gauss [147.65.4.1]) by Euler.impa.br (8.11.6p2/8.11.6) with ESMTP id h3IMx5027247 for ; Fri, 18 Apr 2003 19:59:05 -0300 (EST) From: Carlos Gustavo Tamm de Araujo Moreira Received: by Gauss.impa.br (8.11.6p2) id h3IMwuu06634; Fri, 18 Apr 2003 19:58:56 -0300 (EST) Message-Id: <200304182258.h3IMwuu06634@Gauss.impa.br> Subject: [obm-l] Premio Abel (fwd) To: obm-l@mat.puc-rio.br Date: Fri, 18 Apr 2003 19:58:56 -0300 (EST) X-Mailer: ELM [version 2.4 PL25] MIME-Version: 1.0 Content-Type: text/plain; charset=US-ASCII Content-Transfer-Encoding: 7bit Sender: owner-obm-l@sucuri.mat.puc-rio.br Precedence: bulk Reply-To: obm-l@mat.puc-rio.br Caros colegas, Acho que ainda nao foi mandada para a lista a noticia sobre o premio Abel. Ai vai. Abracos, Gugu > >Como prometido, segue em attachment, um texto que preparei em portugues,=20 >baseado quase que integralmente no que foi feito para divulgacao publica=20 >pela Comissao do Premio Abel, sobre Jean-Pierre Serre, o primeiro laureado= >=20 >com este reconhecimento maior as contribuicoes cientificas em Matematica e= >=20 >suas Aplicacoes. > >Abracos, > >Jacob > > > >Caros Colegas, > >Foi hoje anunciado o primeiro Premio Abel que tera lugar a cada ano no=20 >valor de cerca de US$ 850,000.00, sendo entregue pelo Rei da Noruega a um,= >=20 >dois ou tres matematicos. > >O primeiro premiado este ano foi Jean-Pierre Serre do College de France e a= >=20 >cerimonia tera lugar em 3 junho proximo. Veja a informacao no link abaixo >http://www.abelprisen.no/index_english.html > >Observo que Serre visitou o IMPA em 1997 para participar do Col=F3quio=20 >Brasileiro de Matematica, do qual foi plenarista. > >Tive a honra de fazer parte do Comite do Premio Abel este ano: os nomes dos= >=20 >cinco membros estao tambem no link acima. > >A Matematica tem agora dois grandes premios bem distintos em sua natureza: > >- A Medalha Fields para matematicos ate 40 anos ( US$ 10,000.00) e entregue > a cada 4 anos para dois, tres ou quatro matematicos > >- O Premio Abel para matematicos sem limite de idade (da ordem de > US$ 850,000.00) > >Pretendo escrever alguns paragrafos amanha cedo sobre Serre e o Premio Abel= >=20 >baseado em resumo do Comite. > >Salute!=20 ========================================================================= Instruções para entrar na lista, sair da lista e usar a lista em http://www.mat.puc-rio.br/~nicolau/olimp/obm-l.html O administrador desta lista é ========================================================================= From owner-obm-l@sucuri.mat.puc-rio.br Fri Apr 18 22:17:41 2003 Return-Path: Received: (from majordom@localhost) by sucuri.mat.puc-rio.br (8.9.3/8.9.3) id WAA27399 for obm-l-MTTP; Fri, 18 Apr 2003 22:13:29 -0300 Received: from puma.unisys.com.br (smtp.unisys.com.br [200.220.64.7]) by sucuri.mat.puc-rio.br (8.9.3/8.9.3) with ESMTP id WAA27380 for ; Fri, 18 Apr 2003 22:13:24 -0300 Received: from jf (riohiper01p142.uninet.com.br [200.220.2.142]) by puma.unisys.com.br (8.12.9/8.12.3) with SMTP id h3J1Co0g017925 for ; Fri, 18 Apr 2003 22:12:52 -0300 (EST) X-Spam-Filter: check_local@puma.unisys.com.br by digitalanswers.org Message-ID: <00e701c30611$1a8d89c0$1102dcc8@jf> From: "Jose Francisco Guimaraes Costa" To: "obm-l" Subject: [obm-l] =?iso-8859-1?Q?problemas_do_mil=EAnio?= Date: Fri, 18 Apr 2003 22:11:30 -0300 MIME-Version: 1.0 Content-Type: multipart/alternative; boundary="----=_NextPart_000_00D5_01C305F7.771483C0" X-Priority: 3 X-MSMail-Priority: Normal X-Mailer: Microsoft Outlook Express 6.00.2800.1106 X-MimeOLE: Produced By Microsoft MimeOLE V6.00.2800.1106 Sender: owner-obm-l@sucuri.mat.puc-rio.br Precedence: bulk Reply-To: obm-l@mat.puc-rio.br This is a multi-part message in MIME format. ------=_NextPart_000_00D5_01C305F7.771483C0 Content-Type: text/plain; charset="iso-8859-1" Content-Transfer-Encoding: quoted-printable Por conta da recente discuss=E3o neste forum das conjecturas de = Poincar=E9 e Reimann, eu vim a tomar conhecimento do s=E9timo problema = do mil=EAnio, a Teoria de Yang-Mills, de que nunca havia ouvido falar. Sobre ela diz o Clay Mathematics Institute, que =E9 "dedicado ao aumento = e dissemina=E7=E3o do conhecimento matem=E1tico", que instituiu os = Problemas do Mil=EAnio: "The laws of quantum physics stand to the world of elementary particles = in the way that Newton's laws of classical mechanics stand to the = macroscopic world. Almost half a century ago, Yang and Mills introduced = a remarkable new framework to describe elementary particles using = structures that also occur in geometry. Quantum Yang-Mills theory is now = the foundation of most of elementary particle theory, and its = predictions have been tested at many experimental laboratories, but its = mathematical foundation is still unclear. The successful use of = Yang-Mills theory to describe the strong interactions of elementary = particles depends on a subtle quantum mechanical property called the = "mass gap:" the quantum particles have positive masses, even though the = classical waves travel at the speed of light. This property has been = discovered by physicists from experiment and confirmed by computer = simulations, but it still has not been understood from a theoretical = point of view. Progress in establishing the existence of the Yang-Mills = theory and a mass gap and will require the introduction of fundamental = new ideas both in physics and in mathematics". A comunidade da f=EDsica Newtoniana (cl=E1ssica) diz - dizia, pelo menos = - de forma cr=EDtica que enquanto na f=EDsica cl=E1ssica observa-se um = fen=F4meno (uma ma=E7=E3 caindo, por exemplo) e a partir da=ED = procura-se a equa=E7=E3o matem=E1tica que o descreve, na f=EDsica = qu=E2ntica ocorre o inverso, isto =E9, bola-se uma equa=E7=E3o e a = partir da=ED procura-se um fen=F4meno que seja descrito por ela. Ficaram = famosos os di=E1logos entre Einstein (f=EDsica cl=E1ssica) e Bohr = (f=EDsica qu=E2ntica) sobre o n=E3o determinismo desta =FAltima, que = Einstein resumia em uma frase: "God does not play dice".=20 Isso posto, lan=E7o =E0 discuss=E3o aqui se a Teoria de Yang-Mills =E9 = um problema matem=E1tico ou f=EDsico. JF ------=_NextPart_000_00D5_01C305F7.771483C0 Content-Type: text/html; charset="iso-8859-1" Content-Transfer-Encoding: quoted-printable
Por conta da recente discuss=E3o neste = forum das=20 conjecturas de Poincar=E9 e Reimann, eu vim a tomar conhecimento do = s=E9timo=20 problema do mil=EAnio, a Teoria de Yang-Mills, de que nunca havia ouvido = falar.
 
Sobre ela diz o Clay Mathematics = Institute, que =E9=20 "dedicado ao aumento e dissemina=E7=E3o do conhecimento matem=E1tico", = que instituiu=20 os Problemas do Mil=EAnio:
 
"The laws of quantum physics stand to = the world of=20 elementary particles in the way that Newton's laws of classical = mechanics stand=20 to the macroscopic world. Almost half a century ago, Yang and Mills = introduced a=20 remarkable new framework to describe elementary particles using = structures that=20 also occur in geometry. Quantum Yang-Mills theory is now the foundation = of most=20 of elementary particle theory, and its predictions have been tested at = many=20 experimental laboratories, but its mathematical foundation is still = unclear. The=20 successful use of Yang-Mills theory to describe the strong interactions = of=20 elementary particles depends on a subtle quantum mechanical property = called the=20 "mass gap:" the quantum particles have positive masses, even though the=20 classical waves travel at the speed of light. This property has been = discovered=20 by physicists from experiment and confirmed by computer simulations, but = it=20 still has not been understood from a theoretical point of view. Progress = in=20 establishing the existence of the Yang-Mills theory and a mass gap and = will=20 require the introduction of fundamental new ideas both in physics and in = mathematics".
 
A comunidade da f=EDsica Newtoniana = (cl=E1ssica) diz -=20 dizia, pelo menos - de forma cr=EDtica que enquanto na = f=EDsica=20 cl=E1ssica observa-se um fen=F4meno (uma ma=E7=E3 caindo, por exemplo) e = a partir da=ED=20 procura-se a equa=E7=E3o matem=E1tica que o descreve, na f=EDsica = qu=E2ntica ocorre o=20 inverso, isto =E9, bola-se uma equa=E7=E3o e a partir da=ED = procura-se um fen=F4meno=20 que seja descrito por ela. Ficaram famosos os di=E1logos entre Einstein = (f=EDsica=20 cl=E1ssica) e Bohr (f=EDsica qu=E2ntica) sobre o n=E3o determinismo = desta =FAltima,=20 que Einstein resumia em uma frase: "God does not play dice". =
 
Isso posto, lan=E7o =E0 = discuss=E3o=20 aqui se a Teoria de Yang-Mills =E9 um problema matem=E1tico ou=20 f=EDsico.
 
JF
 
------=_NextPart_000_00D5_01C305F7.771483C0-- ========================================================================= Instruções para entrar na lista, sair da lista e usar a lista em http://www.mat.puc-rio.br/~nicolau/olimp/obm-l.html O administrador desta lista é ========================================================================= From owner-obm-l@sucuri.mat.puc-rio.br Fri Apr 18 22:17:42 2003 Return-Path: Received: (from majordom@localhost) by sucuri.mat.puc-rio.br (8.9.3/8.9.3) id WAA27384 for obm-l-MTTP; Fri, 18 Apr 2003 22:13:25 -0300 Received: from puma.unisys.com.br (smtp.unisys.com.br [200.220.64.7]) by sucuri.mat.puc-rio.br (8.9.3/8.9.3) with ESMTP id WAA27378 for ; Fri, 18 Apr 2003 22:13:22 -0300 Received: from jf (riohiper01p142.uninet.com.br [200.220.2.142]) by puma.unisys.com.br (8.12.9/8.12.3) with SMTP id h3J1Co0e017925 for ; Fri, 18 Apr 2003 22:12:50 -0300 (EST) X-Spam-Filter: check_local@puma.unisys.com.br by digitalanswers.org Message-ID: <00e601c30611$1991c4a0$1102dcc8@jf> From: "Jose Francisco Guimaraes Costa" To: References: <001901c305fd$d85acd00$28029b9b@LeandroRecova> Subject: [obm-l] =?iso-8859-1?Q?palet=F3_e_gravata?= Date: Fri, 18 Apr 2003 21:23:46 -0300 MIME-Version: 1.0 Content-Type: multipart/alternative; boundary="----=_NextPart_000_00AD_01C305F0.CB7D9200" X-Priority: 3 X-MSMail-Priority: Normal X-Mailer: Microsoft Outlook Express 6.00.2800.1106 X-MimeOLE: Produced By Microsoft MimeOLE V6.00.2800.1106 Sender: owner-obm-l@sucuri.mat.puc-rio.br Precedence: bulk Reply-To: obm-l@mat.puc-rio.br This is a multi-part message in MIME format. ------=_NextPart_000_00AD_01C305F0.CB7D9200 Content-Type: text/plain; charset="iso-8859-1" Content-Transfer-Encoding: quoted-printable Responda r=E1pido, antes que o N brigue com a gente: V j=E1 viu algum = engenheiro usando palet=F3 e gravata no alto de um pr=E9dio em = constru=E7=E3o? Hoje em dia, a =FAnica profiss=E3o que requer o uso de = palet=F3 e gravata =E9 a de advogado. JF ----- Original Message -----=20 From: "Leandro Lacorte Rec=F4va" To: Sent: Friday, April 18, 2003 7:57 PM Subject: RE: [obm-l] ITA - Off-Topic > Um conselho: >=20 > Se voces tem paixao pela matematica, facam um curso de matematica na > PUC-RJ, UFRJ, UNICAMP, USP,UnB, etc...pois na engenharia so irao = usa-la > como ferramenta e no futuro usarao terno e gravata. Caso gostem, = otimo, > mas se querem conviver com a matematica, facam um curso de bacharelado > em matematica. >=20 > Leandro.=20 >=20 > ----- Original Message ----- > From: "Ariel de Silvio" > To: > Sent: Friday, April 18, 2003 6:02 PM > Subject: [obm-l] ITA - Off-Topic >=20 > ----- Original Message ----- > From: "Blue Ice" > To: > Sent: Friday, April 18, 2003 7:01 PM > Subject: Re: [obm-l] ITA - Off-Topic ------=_NextPart_000_00AD_01C305F0.CB7D9200 Content-Type: text/html; charset="iso-8859-1" Content-Transfer-Encoding: quoted-printable
Responda r=E1pido, antes que o N brigue = com a gente:=20 V j=E1 viu algum engenheiro usando palet=F3 e gravata no alto de um = pr=E9dio em=20 constru=E7=E3o? Hoje em dia, a =FAnica profiss=E3o que requer o uso de = palet=F3 e gravata=20 =E9 a de advogado.
 
JF
 
----- Original Message ----- =
From: "Leandro Lacorte Rec=F4va" = <lrecova@hotmail.com>
To: <obm-l@mat.puc-rio.br>
Sent: Friday, April 18, 2003 7:57 = PM
Subject: RE: [obm-l] ITA - = Off-Topic

> Um conselho:
>
> Se voces tem paixao pela = matematica,=20 facam um curso de matematica na
> PUC-RJ, UFRJ, UNICAMP, = USP,UnB, =20 etc...pois na engenharia so irao usa-la
> como ferramenta e no = futuro=20 usarao terno e gravata. Caso gostem, otimo,
> mas se querem = conviver com a=20 matematica, facam um curso de bacharelado
> em matematica.
> =
> Leandro.
>
> = -----=20 Original Message -----
> From: "Ariel de Silvio" <
ariel@watersportsbrazil.com>
> To: <
obm-l@mat.puc-rio.br
>
> Sent: Friday, April 18, 2003 6:02 PM
> = Subject:=20 [obm-l] ITA - Off-Topic

> ----- Original Message -----
> From: "Blue Ice" <vinimaila35@bol.com.br>
> To: <obm-l@mat.puc-rio.br>
> Sent: Friday, April 18, 2003 7:01 PM
> Subject: Re: [obm-l] ITA - = Off-Topic
------=_NextPart_000_00AD_01C305F0.CB7D9200-- ========================================================================= Instruções para entrar na lista, sair da lista e usar a lista em http://www.mat.puc-rio.br/~nicolau/olimp/obm-l.html O administrador desta lista é ========================================================================= From owner-obm-l@sucuri.mat.puc-rio.br Fri Apr 18 23:37:54 2003 Return-Path: Received: (from majordom@localhost) by sucuri.mat.puc-rio.br (8.9.3/8.9.3) id XAA29090 for obm-l-MTTP; Fri, 18 Apr 2003 23:33:50 -0300 Received: from krypton.hosting4u.net (krypton.hosting4u.net [209.15.2.78]) by sucuri.mat.puc-rio.br (8.9.3/8.9.3) with ESMTP id XAA29086 for ; Fri, 18 Apr 2003 23:33:46 -0300 Received: from gargamel (200-158-201-221.dsl.telesp.net.br [200.158.201.221]) by krypton.hosting4u.net (Postfix) with ESMTP id 2AF0D9F707 for ; Fri, 18 Apr 2003 21:33:14 -0500 (CDT) Message-ID: <200304182336440060.00133E65@smtp.watersportsbrazil.com> In-Reply-To: <200304181802160540.00122780@smtp.watersportsbrazil.com> References: <200304181802160540.00122780@smtp.watersportsbrazil.com> X-Mailer: Calypso Version 3.30.00.00 (3) Date: Fri, 18 Apr 2003 23:36:44 -0300 From: "Ariel de Silvio" To: obm-l@mat.puc-rio.br Subject: Re: [obm-l] ITA - Off-Topic Mime-Version: 1.0 Content-Type: text/plain; charset="us-ascii" Content-Transfer-Encoding: 8bit X-MIME-Autoconverted: from quoted-printable to 8bit by sucuri.mat.puc-rio.br id XAA29087 Sender: owner-obm-l@sucuri.mat.puc-rio.br Precedence: bulk Reply-To: obm-l@mat.puc-rio.br Muito obrigado aos que me responderam no email, meu ICQ é 29383018. Quem mais puder me ajudar... Gosto muito de matemática, mas prefiro ela como instrumento mesmo... hehehe E espero nao ter que usar terno no futuro, hehehe... meu pai é engenheiro (de minas) e a primeira festa de 15 anos que fui na vida tive que desempoerar o unico terno dele... Acho que a unica vez q vi ele usando terno foi pra defender o mestrado dele... hehehe Bom, fim de papo off-topic Thanks, Ariel *********** MENSAGEM ORIGINAL *********** As 18:02 de 18/4/2003 Ariel de Silvio escreveu: >Olá, > >sei que é off-topic, por isso pretendo ser breve... > >Sei que já foi discutido algumas vezes aqui nessa lista esse assunto, e >sei que alguns participantes da lista cursam ou cursaram o ITA... gostaria >de conversar com algumas pessoas a esse respeito, se puderem me enviar >emails em pvt... ariel@watersportsbrazil.com > >Quero fazer ITA, estou cursando o 3o colegial do Colegio Rio Branco (SP), >e gostaria de informações de pessoas que ja passaram por isso ou passam >ainda... como é? se é tão maluco quanto alguns dizem... Como se >preparar... e coisas do tipo... > >Obrigado aos que puderem me ajudar... >Ariel de Silvio > >========================================================================= >Instruções para entrar na lista, sair da lista e usar a lista em >http://www.mat.puc-rio.br/~nicolau/olimp/obm-l.html >O administrador desta lista é >========================================================================= ========================================================================= Instruções para entrar na lista, sair da lista e usar a lista em http://www.mat.puc-rio.br/~nicolau/olimp/obm-l.html O administrador desta lista é ========================================================================= From owner-obm-l@sucuri.mat.puc-rio.br Sat Apr 19 00:53:12 2003 Return-Path: Received: (from majordom@localhost) by sucuri.mat.puc-rio.br (8.9.3/8.9.3) id AAA30063 for obm-l-MTTP; Sat, 19 Apr 2003 00:51:11 -0300 Received: from www.zipmail.com.br (smtp.zipmail.com.br [200.221.11.147]) by sucuri.mat.puc-rio.br (8.9.3/8.9.3) with ESMTP id AAA30059 for ; Sat, 19 Apr 2003 00:51:08 -0300 From: luizhenriquerick@zipmail.com.br Received: from [200.216.33.62] by www.zipmail.com.br with HTTP; Sat, 19 Apr 2003 00:48:51 -0300 Message-ID: <3E9EC18900002846@www.zipmail.com.br> Date: Sat, 19 Apr 2003 00:48:51 -0300 In-Reply-To: <005d01c2fa4d$a245e2b0$4f9093c8@jr> Subject: [obm-l] =?iso-8859-1?Q?Re=3A=20=5Bobm=2Dl=5D=20Demonstra=E7=F5es?= To: obm-l@mat.puc-rio.br MIME-Version: 1.0 Content-Type: text/plain; charset="iso-8859-1" Content-Transfer-Encoding: 8bit X-MIME-Autoconverted: from quoted-printable to 8bit by sucuri.mat.puc-rio.br id AAA30060 Sender: owner-obm-l@sucuri.mat.puc-rio.br Precedence: bulk Reply-To: obm-l@mat.puc-rio.br -- Mensagem original -- >Alguem poderia me ajudar nestas demonstrações > >1) sabendo que sqrt(3) e sqrt(5) são irracionais, verifique que sqrt(3) >+ sqrt(5) é irracional. > >2) sejam p> 0 e q>0 primos distintos. verifique que sqrt(p) + sqrt(q) é >irracional > >3) se p e q sào inteiros positivos distintos e pelo menos um dos numeros >sqrt(p) ou sqrt(q) é irracional, então sqrt(p) + sqrt(q) é tb >irracional. > >desde ja agradeço > ========================== Olá. Não sei se já responderam sua mensagem , mas , vou tentar responder . Hipótese => sqrt3 e sqrt5 são irracionais Tese => sqrt3 + sqrt5 é irracional Supondo que (sqrt3 + sqrt5) seja racional , ou seja , possa ser escrito na forma a/b , onde a,b pertence aos inteiros , b seja diferente de zero e mdc(a,b) = 1, temos: sqrt3 + sqrt5 = a / b b ( sqrt3 + sqrt5 ) = a b(sqrt3) + b(sqrt5 ) = a Pelo lema de que um ( Inteiro) x ( Irracional ) = Irracional ,sendo o inteiro diferente de zero , vem : b(sqrt3) = Irracional + b(sqrt5) = Irracional --------------------------- a = Inteiro ( => <= ) Contradição. Não sei se está muito superficial . Espero que tenho ajudado em alguma coisa. Use a mesma idéia para fazer 2 e 3 . ================================== Agora prove para mim : a) sqrt3 é Irracional b) sqrt5 é Irracional c) sqrt7 Irracional d) sqrt3 + sqrt5 + sqrt7 é Irracional ================================= Abraços Luiz B. Rocha www.olympicmaths.hpg.com.br ------------------------------------------ Use o melhor sistema de busca da Internet Radar UOL - http://www.radaruol.com.br ========================================================================= Instruções para entrar na lista, sair da lista e usar a lista em http://www.mat.puc-rio.br/~nicolau/olimp/obm-l.html O administrador desta lista é ========================================================================= From owner-obm-l@sucuri.mat.puc-rio.br Sat Apr 19 03:33:49 2003 Return-Path: Received: (from majordom@localhost) by sucuri.mat.puc-rio.br (8.9.3/8.9.3) id DAA31593 for obm-l-MTTP; Sat, 19 Apr 2003 03:31:56 -0300 Received: from smtp018.mail.yahoo.com (smtp018.mail.yahoo.com [216.136.174.115]) by sucuri.mat.puc-rio.br (8.9.3/8.9.3) with SMTP id DAA31589 for ; Sat, 19 Apr 2003 03:31:52 -0300 Received: from 200-180-165-193.paemt7002.dsl.brasiltelecom.net.br (HELO servidor) (marcus?math@200.180.165.193 with login) by smtp.mail.vip.sc5.yahoo.com with SMTP; 19 Apr 2003 06:31:19 -0000 Message-ID: <000b01c3063d$49c69a70$6fe9fea9@servidor> From: "Marcus Alexandre Nunes" To: References: <3E9EC18900002846@www.zipmail.com.br> Subject: [obm-l] =?iso-8859-1?Q?Re:_=5Bobm-l=5D_Re:_=5Bobm-l=5D_Demonstra=E7=F5es?= Date: Sat, 19 Apr 2003 03:31:15 -0300 MIME-Version: 1.0 Content-Type: multipart/alternative; boundary="----=_NextPart_000_0008_01C30624.21EB6490" X-Priority: 3 X-MSMail-Priority: Normal X-Mailer: Microsoft Outlook Express 6.00.2800.1106 x-mimeole: Produced By Microsoft MimeOLE V6.00.2800.1106 Sender: owner-obm-l@sucuri.mat.puc-rio.br Precedence: bulk Reply-To: obm-l@mat.puc-rio.br This is a multi-part message in MIME format. ------=_NextPart_000_0008_01C30624.21EB6490 Content-Type: text/plain; charset="iso-8859-1" Content-Transfer-Encoding: quoted-printable b(sqrt3) =3D Irracional=20 + b(sqrt5) =3D Irracional=20 --------------------------- =20 a =3D Inteiro ( =3D> <=3D ) Contradi=E7=E3o. Para mim isto n=E3o prova nada, pois (sqrt2 + 2) - (sqrt2 - 2) =3D 4, = onde temos uma soma de irracionais que resulta em inteiro. Corrijam-me se eu errei na minha observa=E7=E3o. ---------------------------------------------- Marcus Alexandre Nunes marcus_math@yahoo.com.br UIN 114153703 ------=_NextPart_000_0008_01C30624.21EB6490 Content-Type: text/html; charset="iso-8859-1" Content-Transfer-Encoding: quoted-printable
b(sqrt3) =3D Irracional
   +
b(sqrt5) =3D = Irracional=20
---------------------------  =
       a=20 =3D Inteiro       ( =3D> <=3D = )   =20 Contradi=E7=E3o.
Para mim isto n=E3o prova nada, pois = (sqrt2 + 2) -=20 (sqrt2 - 2) =3D 4, onde temos uma soma de irracionais que resulta em=20 inteiro.
 
Corrijam-me se eu errei na minha=20 observa=E7=E3o.
 
----------------------------------------------
Marcus = Alexandre=20 Nunes
marcus_math@yahoo.com.br
= UIN=20 114153703
------=_NextPart_000_0008_01C30624.21EB6490-- ========================================================================= Instruções para entrar na lista, sair da lista e usar a lista em http://www.mat.puc-rio.br/~nicolau/olimp/obm-l.html O administrador desta lista é ========================================================================= From owner-obm-l@sucuri.mat.puc-rio.br Sat Apr 19 07:06:59 2003 Return-Path: Received: (from majordom@localhost) by sucuri.mat.puc-rio.br (8.9.3/8.9.3) id HAA00918 for obm-l-MTTP; Sat, 19 Apr 2003 07:05:29 -0300 Received: from smtp-26.ig.com.br (smtp-26.ig.com.br [200.226.132.160]) by sucuri.mat.puc-rio.br (8.9.3/8.9.3) with SMTP id HAA00914 for ; Sat, 19 Apr 2003 07:05:24 -0300 Received: (qmail 29394 invoked from network); 19 Apr 2003 10:04:57 -0000 Received: from unknown (HELO xxxx) (200.165.255.197) by smtp-26.ig.com.br with SMTP; 19 Apr 2003 10:04:57 -0000 Message-ID: <001501c3065b$b2aeda20$c5ffa5c8@epq.ime.eb.br> From: "Marcio" To: References: <3E9EC18900002846@www.zipmail.com.br> <000b01c3063d$49c69a70$6fe9fea9@servidor> Subject: [obm-l] =?iso-8859-1?Q?Re:_=5Bobm-l=5D_Re:_=5Bobm-l=5D_Re:_=5Bobm-l=5D_Demonstr?= =?iso-8859-1?Q?a=E7=F5es?= Date: Sat, 19 Apr 2003 07:09:00 -0300 MIME-Version: 1.0 Content-Type: multipart/alternative; boundary="----=_NextPart_000_0012_01C30642.8D1556E0" X-Priority: 3 X-MSMail-Priority: Normal X-Mailer: Microsoft Outlook Express 5.50.4133.2400 X-MimeOLE: Produced By Microsoft MimeOLE V5.50.4133.2400 Sender: owner-obm-l@sucuri.mat.puc-rio.br Precedence: bulk Reply-To: obm-l@mat.puc-rio.br This is a multi-part message in MIME format. ------=_NextPart_000_0012_01C30642.8D1556E0 Content-Type: text/plain; charset="iso-8859-1" Content-Transfer-Encoding: quoted-printable Voce esta certissimo.. De fato, nao acho que o fato de sqrt(3) e = sqrt(5) serem irracionais ajude muito na prova.. Uma maneira de provar seria a seguinte: Suponha que X =3D sqrt(3) + = sqrt(5) seja irracional. Entao, X^2 - 8 =3D sqrt(15) tambem seria = irracional. Mas se p/q =3Dsqrt(15), entao 15p^2 =3D q^2 e chegamos a uma = contradicao, pois ao colocar ambos os lados na forma 'produto de fatores = primos' (que =E9 =FAnica), vemos que o expoente de 3 =E9 =EDmpar no lado = esquerdo, e par no lado direito... contradicao.. Outra opcao eh escrever direto x =3D p/q =3D sqrt(3) + sqrt(5), ir = mexendo ateh chegar numa equacao com coeficientes inteiros apenas e = ficar fazendo testes de divisibilidade (ajuda mostrar antes que se p/q = eh raiz de uma equacao com coeficientes inteiros, entao p divide o termo = independente, e q divide o termo lider) ----- Original Message -----=20 From: Marcus Alexandre Nunes=20 To: obm-l@mat.puc-rio.br=20 Sent: Saturday, April 19, 2003 3:31 AM Subject: [obm-l] Re: [obm-l] Re: [obm-l] Demonstra=E7=F5es b(sqrt3) =3D Irracional=20 + b(sqrt5) =3D Irracional=20 --------------------------- =20 a =3D Inteiro ( =3D> <=3D ) Contradi=E7=E3o. Para mim isto n=E3o prova nada, pois (sqrt2 + 2) - (sqrt2 - 2) =3D 4, = onde temos uma soma de irracionais que resulta em inteiro. Corrijam-me se eu errei na minha observa=E7=E3o. ---------------------------------------------- Marcus Alexandre Nunes marcus_math@yahoo.com.br UIN 114153703 ------=_NextPart_000_0012_01C30642.8D1556E0 Content-Type: text/html; charset="iso-8859-1" Content-Transfer-Encoding: quoted-printable
    Voce esta = certissimo..  De=20 fato, nao acho que o fato de sqrt(3) e sqrt(5) serem irracionais ajude = muito na=20 prova..
    Uma maneira de = provar seria a=20 seguinte: Suponha que X =3D sqrt(3) + sqrt(5) seja irracional. Entao, = X^2 - 8 =3D=20 sqrt(15) tambem seria irracional.
Mas se p/q =3Dsqrt(15), entao 15p^2 =3D = q^2 e chegamos=20 a uma contradicao, pois ao colocar ambos os lados na forma 'produto de = fatores=20 primos' (que =E9 =FAnica), vemos que o expoente de 3 =E9 =EDmpar no lado = esquerdo, e par=20 no lado direito... contradicao..
    Outra opcao eh = escrever direto x=20 =3D p/q =3D sqrt(3) + sqrt(5), ir mexendo ateh chegar numa equacao com = coeficientes=20 inteiros apenas e ficar fazendo testes de divisibilidade (ajuda mostrar = antes=20 que se p/q eh raiz de uma equacao com coeficientes inteiros, entao p = divide o=20 termo independente, e q divide o termo lider)
----- Original Message -----
From:=20 Marcus Alexandre Nunes =
Sent: Saturday, April 19, 2003 = 3:31=20 AM
Subject: [obm-l] Re: [obm-l] = Re: [obm-l]=20 Demonstra=E7=F5es

b(sqrt3) =3D Irracional
   +
b(sqrt5) =3D = Irracional=20
---------------------------  =
      =20 a =3D Inteiro       ( =3D> <=3D=20 )    Contradi=E7=E3o.
Para mim isto n=E3o prova nada, = pois (sqrt2 + 2)=20 - (sqrt2 - 2) =3D 4, onde temos uma soma de irracionais que resulta em = inteiro.
 
Corrijam-me se eu errei na minha=20 observa=E7=E3o.
 
----------------------------------------------
Marcus = Alexandre=20 Nunes
marcus_math@yahoo.com.br
= UIN=20 114153703
------=_NextPart_000_0012_01C30642.8D1556E0-- ========================================================================= Instruções para entrar na lista, sair da lista e usar a lista em http://www.mat.puc-rio.br/~nicolau/olimp/obm-l.html O administrador desta lista é ========================================================================= From owner-obm-l@sucuri.mat.puc-rio.br Sat Apr 19 11:54:30 2003 Return-Path: Received: (from majordom@localhost) by sucuri.mat.puc-rio.br (8.9.3/8.9.3) id LAA03395 for obm-l-MTTP; Sat, 19 Apr 2003 11:52:36 -0300 Received: from imo-m04.mx.aol.com (imo-m04.mx.aol.com [64.12.136.7]) by sucuri.mat.puc-rio.br (8.9.3/8.9.3) with ESMTP id LAA03390 for ; Sat, 19 Apr 2003 11:52:32 -0300 From: Jkwasinsky@aol.com Received: from Jkwasinsky@aol.com by imo-m04.mx.aol.com (mail_out_v34.22.) id z.149.f4c2d9b (4116) for ; Sat, 19 Apr 2003 10:51:55 -0400 (EDT) Message-ID: <149.f4c2d9b.2bd2bc8b@aol.com> Date: Sat, 19 Apr 2003 10:51:55 EDT Subject: =?ISO-8859-1?Q?Re:=20[obm-l]=20Constru=E7l=E3o!!!?= To: obm-l@mat.puc-rio.br MIME-Version: 1.0 Content-Type: multipart/alternative; boundary="part1_149.f4c2d9b.2bd2bc8b_boundary" X-Mailer: 7.0 for Windows sub 537 Sender: owner-obm-l@sucuri.mat.puc-rio.br Precedence: bulk Reply-To: obm-l@mat.puc-rio.br --part1_149.f4c2d9b.2bd2bc8b_boundary Content-Type: text/plain; charset="ISO-8859-1" Content-Transfer-Encoding: quoted-printable ola aqui eh o kwasinsky, esta eh minha primeira mensagem. Estou gostando de=20 participar deste grupo de discussao . aqui vai a resposta sobre a=20 possibilidade ou nao da construcao de Pi com regua e compasso. Eh possivel mostrar que todo numero real que se pode consrtruir com regua e=20 compasso eh algebrico!!!!!!!!! =20 O numero Pi nao eh algebrico ( nao eh solucao de uma equacao polinomial de=20 coeficientes inteiros ).Voce pode encontrar esses conceitos e suas=20 demonstrcoes nos livros do impa (se nao me engano o titulo eh : =A8numeos=20 irracionais e transcendentes"). =20 --part1_149.f4c2d9b.2bd2bc8b_boundary Content-Type: text/html; charset="ISO-8859-1" Content-Transfer-Encoding: quoted-printable ola aqui eh o kwasinsky, esta eh minha primeira mensag= em. Estou gostando de participar deste grupo de discussao . aqui vai a respo= sta sobre a possibilidade ou nao da construcao de Pi com regua e compasso.
Eh possivel mostrar que todo numero real que se pode consrtruir com regua e=20= compasso  eh    algebrico!!!!!!!!!  

O numero Pi nao eh algebrico ( nao eh solucao de uma equacao polinomial de c= oeficientes inteiros ).Voce pode encontrar esses conceitos e suas demonstrco= es nos livros  do impa (se nao me engano o titulo eh : =A8numeos irraci= onais e transcendentes").     
--part1_149.f4c2d9b.2bd2bc8b_boundary-- ========================================================================= Instruções para entrar na lista, sair da lista e usar a lista em http://www.mat.puc-rio.br/~nicolau/olimp/obm-l.html O administrador desta lista é ========================================================================= From owner-obm-l@sucuri.mat.puc-rio.br Sat Apr 19 12:33:03 2003 Return-Path: Received: (from majordom@localhost) by sucuri.mat.puc-rio.br (8.9.3/8.9.3) id MAA04219 for obm-l-MTTP; Sat, 19 Apr 2003 12:31:44 -0300 Received: from artemis.opendf.com.br (artemis.opengate.com.br [200.181.71.14]) by sucuri.mat.puc-rio.br (8.9.3/8.9.3) with ESMTP id MAA04215 for ; Sat, 19 Apr 2003 12:31:41 -0300 Received: from localhost (localhost [127.0.0.1]) by artemis.opendf.com.br (Postfix) with ESMTP id 942ED2BEC9 for ; Sat, 19 Apr 2003 12:32:18 -0300 (BRT) Received: from artemis.opendf.com.br ([127.0.0.1]) by localhost (artemis.opengate.com.br [127.0.0.1:10024]) (amavisd-new) with ESMTP id 20734-01 for ; Sat, 19 Apr 2003 12:32:17 -0300 (BRT) Received: from computer (200-181-089-244.bsace7001.dsl.brasiltelecom.net.br [200.181.89.244]) by artemis.opendf.com.br (Postfix) with ESMTP id 323472BEC4 for ; Sat, 19 Apr 2003 12:32:17 -0300 (BRT) From: "Artur Costa Steiner" To: Subject: [obm-l] =?iso-8859-1?Q?RE:_=5Bobm-l=5D_Re:_=5Bobm-l=5D_Demonstra=E7=F5es?= Date: Sat, 19 Apr 2003 12:31:13 -0300 Organization: Steiner Consultoria LTDA Message-ID: <005901c30688$b6510db0$9865fea9@computer> MIME-Version: 1.0 Content-Type: text/plain; charset="iso-8859-1" X-Priority: 3 (Normal) X-MSMail-Priority: Normal X-Mailer: Microsoft Outlook, Build 10.0.2627 X-MimeOLE: Produced By Microsoft MimeOLE V6.00.2800.1106 In-Reply-To: <3E9EC18900002846@www.zipmail.com.br> Importance: Normal X-Virus-Scanned: by amavisd-new Content-Transfer-Encoding: 8bit X-MIME-Autoconverted: from quoted-printable to 8bit by sucuri.mat.puc-rio.br id MAA04216 Sender: owner-obm-l@sucuri.mat.puc-rio.br Precedence: bulk Reply-To: obm-l@mat.puc-rio.br A demonstracao destas afirmacoes jah foi apresentada na lista ha uns 15. Consulte os arquivos. Artur >-----Original Message----- >From: owner-obm-l@sucuri.mat.puc-rio.br [mailto:owner-obm-l@sucuri.mat.puc- >rio.br] On Behalf Of luizhenriquerick@zipmail.com.br >Sent: Saturday, April 19, 2003 12:49 AM >To: obm-l@mat.puc-rio.br >Subject: [obm-l] Re: [obm-l] Demonstrações > > > >-- Mensagem original -- > >>Alguem poderia me ajudar nestas demonstrações >> >>1) sabendo que sqrt(3) e sqrt(5) são irracionais, verifique que sqrt(3) >>+ sqrt(5) é irracional. >> >>2) sejam p> 0 e q>0 primos distintos. verifique que sqrt(p) + sqrt(q) é >>irracional >> >>3) se p e q sào inteiros positivos distintos e pelo menos um dos numeros >>sqrt(p) ou sqrt(q) é irracional, então sqrt(p) + sqrt(q) é tb >>irracional. >> >>desde ja agradeço >> > > >========================== > > >Olá. > >Não sei se já responderam sua mensagem , mas , vou tentar responder . > >Hipótese => sqrt3 e sqrt5 são irracionais >Tese => sqrt3 + sqrt5 é irracional > >Supondo que (sqrt3 + sqrt5) seja racional , ou seja , possa ser escrito >na forma a/b , onde a,b pertence aos inteiros , b seja diferente de zero >e mdc(a,b) = 1, temos: > >sqrt3 + sqrt5 = a / b >b ( sqrt3 + sqrt5 ) = a >b(sqrt3) + b(sqrt5 ) = a > >Pelo lema de que um ( Inteiro) x ( Irracional ) = Irracional ,sendo o >inteiro >diferente de zero , vem : >b(sqrt3) = Irracional > + >b(sqrt5) = Irracional >--------------------------- > a = Inteiro ( => <= ) Contradição. > >Não sei se está muito superficial . Espero que tenho ajudado em alguma >coisa. > > >Use a mesma idéia para fazer 2 e 3 . > >================================== > > >Agora prove para mim : > >a) sqrt3 é Irracional >b) sqrt5 é Irracional >c) sqrt7 Irracional >d) sqrt3 + sqrt5 + sqrt7 é Irracional > > >================================= > >Abraços > >Luiz B. Rocha > > > >www.olympicmaths.hpg.com.br > > >------------------------------------------ >Use o melhor sistema de busca da Internet >Radar UOL - http://www.radaruol.com.br > > > >======================================================================= == >Instruções para entrar na lista, sair da lista e usar a lista em >http://www.mat.puc-rio.br/~nicolau/olimp/obm-l.html >O administrador desta lista é >======================================================================= == ========================================================================= Instruções para entrar na lista, sair da lista e usar a lista em http://www.mat.puc-rio.br/~nicolau/olimp/obm-l.html O administrador desta lista é ========================================================================= From owner-obm-l@sucuri.mat.puc-rio.br Sat Apr 19 12:44:50 2003 Return-Path: Received: (from majordom@localhost) by sucuri.mat.puc-rio.br (8.9.3/8.9.3) id MAA04488 for obm-l-MTTP; Sat, 19 Apr 2003 12:43:12 -0300 Received: from artemis.opendf.com.br (artemis.opengate.com.br [200.181.71.14]) by sucuri.mat.puc-rio.br (8.9.3/8.9.3) with ESMTP id MAA04484 for ; Sat, 19 Apr 2003 12:43:08 -0300 Received: from localhost (localhost [127.0.0.1]) by artemis.opendf.com.br (Postfix) with ESMTP id 385322BEC9 for ; Sat, 19 Apr 2003 12:43:47 -0300 (BRT) Received: from artemis.opendf.com.br ([127.0.0.1]) by localhost (artemis.opengate.com.br [127.0.0.1:10024]) (amavisd-new) with ESMTP id 20734-05 for ; Sat, 19 Apr 2003 12:43:46 -0300 (BRT) Received: from computer (200-181-089-244.bsace7001.dsl.brasiltelecom.net.br [200.181.89.244]) by artemis.opendf.com.br (Postfix) with ESMTP id CA0D32BEC4 for ; Sat, 19 Apr 2003 12:43:45 -0300 (BRT) From: "Artur Costa Steiner" To: Subject: RE: [obm-l] ITA - Off-Topic Date: Sat, 19 Apr 2003 12:42:42 -0300 Organization: Steiner Consultoria LTDA Message-ID: <005a01c3068a$50abb670$9865fea9@computer> MIME-Version: 1.0 Content-Type: text/plain; charset="iso-8859-1" X-Priority: 3 (Normal) X-MSMail-Priority: Normal X-Mailer: Microsoft Outlook, Build 10.0.2627 X-MimeOLE: Produced By Microsoft MimeOLE V6.00.2800.1106 In-Reply-To: <001901c305fd$d85acd00$28029b9b@LeandroRecova> Importance: Normal X-Virus-Scanned: by amavisd-new Content-Transfer-Encoding: 8bit X-MIME-Autoconverted: from quoted-printable to 8bit by sucuri.mat.puc-rio.br id MAA04485 Sender: owner-obm-l@sucuri.mat.puc-rio.br Precedence: bulk Reply-To: obm-l@mat.puc-rio.br Ou entao tente fazer comop eu. Quando a situacao permite, compre um bom livro na area da matematica que vc gosta e estude. Quando nao emtender mesmo alguma passagem, peca ajuda a esta lista ou a outras. O dificil eh arranjar tempo para estudar matematica sendo engenheiro, quer vc ande de terno e gravata ou nao. Estou ha meses tentando estudar a teoria de medidas e nao consigo ir pra a frente. Artur >-----Original Message----- >From: owner-obm-l@sucuri.mat.puc-rio.br [mailto:owner-obm-l@sucuri.mat.puc- >rio.br] On Behalf Of Leandro Lacorte Recôva >Sent: Friday, April 18, 2003 7:57 PM >To: obm-l@mat.puc-rio.br >Subject: RE: [obm-l] ITA - Off-Topic > >Um conselho: > >Se voces tem paixao pela matematica, facam um curso de matematica na >PUC-RJ, UFRJ, UNICAMP, USP,UnB, etc...pois na engenharia so irao usa-la >como ferramenta e no futuro usarao terno e gravata. Caso gostem, otimo, >mas se querem conviver com a matematica, facam um curso de bacharelado >em matematica. > >Leandro. > >-----Original Message----- >From: owner-obm-l@sucuri.mat.puc-rio.br >[mailto:owner-obm-l@sucuri.mat.puc-rio.br] On Behalf Of Blue Ice >Sent: Friday, April 18, 2003 3:02 PM >To: obm-l@mat.puc-rio.br >Subject: Re: [obm-l] ITA - Off-Topic > > >----- Original Message ----- >From: "Ariel de Silvio" >To: >Sent: Friday, April 18, 2003 6:02 PM >Subject: [obm-l] ITA - Off-Topic > > >> E-mail Premium BOL >> Antivírus, anti-spam e até 100 MB de espaço. Assine já! >> http://email.bol.com.br/ >> Olá, >> >> sei que é off-topic, por isso pretendo ser breve... >> >> Sei que já foi discutido algumas vezes aqui nessa lista esse assunto, >e >sei que alguns participantes da lista cursam ou cursaram o ITA... >gostaria >de conversar com algumas pessoas a esse respeito, se puderem me enviar >emails em pvt... ariel@watersportsbrazil.com >> >> Quero fazer ITA, estou cursando o 3o colegial do Colegio Rio Branco >(SP), >e gostaria de informações de pessoas que ja passaram por isso ou passam >ainda... como é? se é tão maluco quanto alguns dizem... Como se >preparar... >e coisas do tipo... >> >> Obrigado aos que puderem me ajudar... >> Ariel de Silvio >> >> >======================================================================= = >= >> Instruções para entrar na lista, sair da lista e usar a lista em >> http://www.mat.puc-rio.br/~nicolau/olimp/obm-l.html >> O administrador desta lista é > > >Olá colega, tb gostaria de cursar o ITA, tenho ICQ se quiser >conversar.Acredito que dividimos a mesma paixão pela matemática. > > >[]´s > >Ice > >ICQ:177782914 Email:Vinicius84@hotmail.com >Win XP Home Full/Speedy 256kbps > > >======================================================================= = >= >Instruções para entrar na lista, sair da lista e usar a lista em >http://www.mat.puc-rio.br/~nicolau/olimp/obm-l.html >O administrador desta lista é >======================================================================= = >= >======================================================================= == >Instruções para entrar na lista, sair da lista e usar a lista em >http://www.mat.puc-rio.br/~nicolau/olimp/obm-l.html >O administrador desta lista é >======================================================================= == ========================================================================= Instruções para entrar na lista, sair da lista e usar a lista em http://www.mat.puc-rio.br/~nicolau/olimp/obm-l.html O administrador desta lista é ========================================================================= From owner-obm-l@sucuri.mat.puc-rio.br Sat Apr 19 13:20:33 2003 Return-Path: Received: (from majordom@localhost) by sucuri.mat.puc-rio.br (8.9.3/8.9.3) id NAA05695 for obm-l-MTTP; Sat, 19 Apr 2003 13:19:00 -0300 Received: from www.zipmail.com.br (smtp.zipmail.com.br [200.221.11.147]) by sucuri.mat.puc-rio.br (8.9.3/8.9.3) with ESMTP id NAA05690 for ; Sat, 19 Apr 2003 13:18:51 -0300 From: luizhenriquerick@zipmail.com.br Received: from [200.216.32.17] by www.zipmail.com.br with HTTP; Sat, 19 Apr 2003 13:14:44 -0300 Message-ID: <3E9D4BDE00003C3F@www.zipmail.com.br> Date: Sat, 19 Apr 2003 13:14:44 -0300 In-Reply-To: <001501c3065b$b2aeda20$c5ffa5c8@epq.ime.eb.br> Subject: [obm-l] =?iso-8859-1?Q?Re=3A=20=5Bobm=2Dl=5D=20Demonstra=E7=F5es?= To: obm-l@mat.puc-rio.br MIME-Version: 1.0 Content-Type: text/plain; charset="iso-8859-1" Content-Transfer-Encoding: 8bit X-MIME-Autoconverted: from quoted-printable to 8bit by sucuri.mat.puc-rio.br id NAA05692 Sender: owner-obm-l@sucuri.mat.puc-rio.br Precedence: bulk Reply-To: obm-l@mat.puc-rio.br -- Mensagem original -- > Voce esta certissimo.. De fato, nao acho que o fato de sqrt(3) e sqrt(5) >serem irracionais ajude muito na prova.. > Uma maneira de provar seria a seguinte: Suponha que X = sqrt(3) + sqrt(5) >seja irracional. Entao, X^2 - 8 = sqrt(15) tambem seria irracional. >Mas se p/q =sqrt(15), entao 15p^2 = q^2 e chegamos a uma contradicao, pois >ao colocar ambos os lados na forma 'produto de fatores primos' (que é única), >vemos que o expoente de 3 é ímpar no lado esquerdo, e par no lado direito... >contradicao.. > Outra opcao eh escrever direto x = p/q = sqrt(3) + sqrt(5), ir mexendo >ateh chegar numa equacao com coeficientes inteiros apenas e ficar fazendo >testes de divisibilidade (ajuda mostrar antes que se p/q eh raiz de uma equacao >com coeficientes inteiros, entao p divide o termo independente, e q divide >o termo lider) > ----- Original Message ----- > From: Marcus Alexandre Nunes > To: obm-l@mat.puc-rio.br > Sent: Saturday, April 19, 2003 3:31 AM > Subject: [obm-l] Re: [obm-l] Re: [obm-l] Demonstrações > > > b(sqrt3) = Irracional > + > b(sqrt5) = Irracional > --------------------------- > a = Inteiro ( => <= ) Contradição. > > Para mim isto não prova nada, pois (sqrt2 + 2) - (sqrt2 - 2) = 4, onde >temos uma soma de irracionais que resulta em inteiro. > > Corrijam-me se eu errei na minha observação. > > ---------------------------------------------- > Marcus Alexandre Nunes > marcus_math@yahoo.com.br > UIN 114153703 > ========================= O que você fez está certo , mais não tem nada haver com a questão . Abraços. Luiz Barbosa www.olympicmaths.hpg.com.br ------------------------------------------ Use o melhor sistema de busca da Internet Radar UOL - http://www.radaruol.com.br ========================================================================= Instruções para entrar na lista, sair da lista e usar a lista em http://www.mat.puc-rio.br/~nicolau/olimp/obm-l.html O administrador desta lista é ========================================================================= From owner-obm-l@sucuri.mat.puc-rio.br Sat Apr 19 13:33:18 2003 Return-Path: Received: (from majordom@localhost) by sucuri.mat.puc-rio.br (8.9.3/8.9.3) id NAA06000 for obm-l-MTTP; Sat, 19 Apr 2003 13:31:59 -0300 Received: from www.zipmail.com.br (smtp.zipmail.com.br [200.221.11.147]) by sucuri.mat.puc-rio.br (8.9.3/8.9.3) with ESMTP id NAA05987 for ; Sat, 19 Apr 2003 13:31:55 -0300 From: luizhenriquerick@zipmail.com.br Received: from [200.216.32.17] by www.zipmail.com.br with HTTP; Sat, 19 Apr 2003 13:06:45 -0300 Message-ID: <3E9D4BDE00003C2B@www.zipmail.com.br> Date: Sat, 19 Apr 2003 13:06:45 -0300 In-Reply-To: <005901c30688$b6510db0$9865fea9@computer> Subject: [obm-l] =?iso-8859-1?Q?Re=3A=20=5Bobm=2Dl=5D=20RE=3A=20=5Bobm=2Dl=5D=20Re=3A=20=5Bobm=2Dl=5D=20Demonstra=E7=F5es?= To: obm-l@mat.puc-rio.br MIME-Version: 1.0 Content-Type: text/plain; charset="iso-8859-1" Content-Transfer-Encoding: 8bit X-MIME-Autoconverted: from quoted-printable to 8bit by sucuri.mat.puc-rio.br id NAA05994 Sender: owner-obm-l@sucuri.mat.puc-rio.br Precedence: bulk Reply-To: obm-l@mat.puc-rio.br Uso zipmail . Costumo deletar os e-mail mais antigos para esvazia a caixa de e-mail e quando não faço isso , perco muitas mensagens. Abraço. Luiz B. www.olympicmaths.hpg.com.br ------------------------------------------ Use o melhor sistema de busca da Internet Radar UOL - http://www.radaruol.com.br ========================================================================= Instruções para entrar na lista, sair da lista e usar a lista em http://www.mat.puc-rio.br/~nicolau/olimp/obm-l.html O administrador desta lista é ========================================================================= From owner-obm-l@sucuri.mat.puc-rio.br Sat Apr 19 15:10:03 2003 Return-Path: Received: (from majordom@localhost) by sucuri.mat.puc-rio.br (8.9.3/8.9.3) id PAA07583 for obm-l-MTTP; Sat, 19 Apr 2003 15:08:15 -0300 Received: from imo-r02.mx.aol.com (imo-r02.mx.aol.com [152.163.225.98]) by sucuri.mat.puc-rio.br (8.9.3/8.9.3) with ESMTP id PAA07579 for ; Sat, 19 Apr 2003 15:08:12 -0300 From: Faelccmm@aol.com Received: from Faelccmm@aol.com by imo-r02.mx.aol.com (mail_out_v34.22.) id z.f5.2c1497f4 (3310) for ; Sat, 19 Apr 2003 14:07:37 -0400 (EDT) Message-ID: Date: Sat, 19 Apr 2003 14:07:37 EDT Subject: [obm-l] MDC To: obm-l@mat.puc-rio.br MIME-Version: 1.0 Content-Type: multipart/alternative; boundary="part1_f5.2c1497f4.2bd2ea69_boundary" X-Mailer: 6.0 sub 10516 Sender: owner-obm-l@sucuri.mat.puc-rio.br Precedence: bulk Reply-To: obm-l@mat.puc-rio.br --part1_f5.2c1497f4.2bd2ea69_boundary Content-Type: text/plain; charset="ISO-8859-1" Content-Transfer-Encoding: quoted-printable Ol=E1 pessoal, Vejam a quest=E3o: Um grupo de escoteiros deve montar guarda nos 4 postos Norte, Sul, Leste e=20 Oeste de seu acampamento. No posto Norte a guarda deve ser dia e noite; no=20 posto Sul a guarda vai das 6 da manh=E3 =E0s 18 da tarde; no posto Leste, da= s 22=20 da noite =E0s 6 da manh=E3 e no posto Oeste, das 14 da tarde =E0s 6 da manh= =E3. Qual=20 deve ser o turno m=E1ximo de guarda, de modo que todos permane=E7am igual te= mpo=20 de sentinela (sem mudar de lugar) ? Quantos escoteiros s=E3o necess=E1rios p= ara=20 24 horas de guarda, de modo que para escoteiro n=E3o d=EA duas guardas no me= smo=20 dia ? Obs: A primeira pergunta eu resolvi tirando o m.d.c das amplitudes hor=E1r= ias=20 de cada posto de guarda e cheguei =E0 resposta do livro R: 4 horas. Mas n= =E3o=20 estou conseguindo resolver a outra pergunta.=20 resp:=20 a) 4 horas=20 b) 15 escoteiros --part1_f5.2c1497f4.2bd2ea69_boundary Content-Type: text/html; charset="ISO-8859-1" Content-Transfer-Encoding: quoted-printable Ol=E1 pessoal,

Vejam a quest=E3o:

Um grupo de escoteiros deve montar guarda nos 4 postos Norte, Sul, Leste= e Oeste de seu acampamento. No posto Norte a guarda deve ser dia e noite; n= o posto Sul a guarda vai das 6 da manh=E3 =E0s 18 da tarde; no posto Leste,=20= das 22 da noite =E0s 6 da manh=E3 e no posto Oeste, das 14 da tarde =E0s 6 d= a manh=E3. Qual deve ser o turno m=E1ximo de guarda, de modo que todos perma= ne=E7am igual tempo de sentinela (sem mudar de lugar) ? Quantos escoteiros s= =E3o necess=E1rios para 24 horas de guarda, de modo que para escoteiro n=E3o= d=EA duas guardas no mesmo dia ?

Obs: A primeira pergunta eu resolvi  tirando o  m.d.c das ampl= itudes hor=E1rias de cada posto de guarda e cheguei =E0 resposta do livro R:= 4 horas. Mas n=E3o estou conseguindo resolver a outra pergunta.=20

resp:=20

a) 4 horas=20
b) 15 escoteiros
--part1_f5.2c1497f4.2bd2ea69_boundary-- ========================================================================= Instruções para entrar na lista, sair da lista e usar a lista em http://www.mat.puc-rio.br/~nicolau/olimp/obm-l.html O administrador desta lista é ========================================================================= From owner-obm-l@sucuri.mat.puc-rio.br Sat Apr 19 16:04:19 2003 Return-Path: Received: (from majordom@localhost) by sucuri.mat.puc-rio.br (8.9.3/8.9.3) id QAA08622 for obm-l-MTTP; Sat, 19 Apr 2003 16:02:36 -0300 Received: from paiol.terra.com.br (paiol.terra.com.br [200.176.3.18]) by sucuri.mat.puc-rio.br (8.9.3/8.9.3) with ESMTP id QAA08618 for ; Sat, 19 Apr 2003 16:02:33 -0300 Received: from marova.terra.com.br (marova.terra.com.br [200.176.3.39]) by paiol.terra.com.br (Postfix) with ESMTP id 61D0487F9C for ; Sat, 19 Apr 2003 16:02:03 -0300 (BRT) Received: from riemann.localdomain (RJ173171.user.veloxzone.com.br [200.149.173.171]) (authenticated user fabio.dias.moreira) by marova.terra.com.br (Postfix) with ESMTP id 6F62B3DC06C for ; Sat, 19 Apr 2003 16:02:02 -0300 (BRT) Content-Type: text/plain; charset="iso-8859-1" From: =?iso-8859-1?q?F=E1bio=20Dias=20Moreira?= To: obm-l@mat.puc-rio.br Subject: Re: [obm-l] Re: [obm-l] RE: [obm-l] Re: [obm-l] =?iso-8859-1?q?Demonstra=E7=F5es?= Date: Sat, 19 Apr 2003 16:01:41 -0300 User-Agent: KMail/1.4.3 References: <3E9D4BDE00003C2B@www.zipmail.com.br> In-Reply-To: <3E9D4BDE00003C2B@www.zipmail.com.br> MIME-Version: 1.0 Content-Transfer-Encoding: 8bit Message-Id: <200304191601.13564.fabio.dias.moreira@terra.com.br> Sender: owner-obm-l@sucuri.mat.puc-rio.br Precedence: bulk Reply-To: obm-l@mat.puc-rio.br -----BEGIN PGP SIGNED MESSAGE----- Hash: SHA1 On Saturday 19 April 2003 13:06, luizhenriquerick@zipmail.com.br wrote: > Uso zipmail . > Costumo deletar os e-mail mais antigos para esvazia a caixa de e-mail e > quando não faço isso , perco muitas mensagens. > [...] Sim, mas o site da obm-l (que está no rodapé deste email) arquiva todas as mensagens enviadas à lista. []s, - -- Fábio "ctg \pi" Dias Moreira -----BEGIN PGP SIGNATURE----- Version: GnuPG v1.0.6 (GNU/Linux) Comment: For info see http://www.gnupg.org iD8DBQE+oZ0falOQFrvzGQoRApDpAKDJg9iHiafxptLlpkYLGT1t7RWvIACggEBB AZy5jUZSUKyUbNgZn+W56B4= =WVlb -----END PGP SIGNATURE----- ========================================================================= Instruções para entrar na lista, sair da lista e usar a lista em http://www.mat.puc-rio.br/~nicolau/olimp/obm-l.html O administrador desta lista é ========================================================================= From owner-obm-l@sucuri.mat.puc-rio.br Sat Apr 19 16:07:27 2003 Return-Path: Received: (from majordom@localhost) by sucuri.mat.puc-rio.br (8.9.3/8.9.3) id QAA08702 for obm-l-MTTP; Sat, 19 Apr 2003 16:05:57 -0300 Received: from salvatore2.bol.com.br (salvatore2.bol.com.br [200.221.24.87]) by sucuri.mat.puc-rio.br (8.9.3/8.9.3) with ESMTP id QAA08698 for ; Sat, 19 Apr 2003 16:05:54 -0300 Received: from xx (200.221.24.48) by salvatore2.bol.com.br (5.1.071) id 3E766FC90078E507 for obm-l@mat.puc-rio.br; Sat, 19 Apr 2003 16:05:24 -0300 Message-ID: <001701c306a6$a04f1160$f4befea9@xx> From: "Blue Ice" To: References: Subject: Re: [obm-l] MDC Date: Sat, 19 Apr 2003 16:05:20 -0300 MIME-Version: 1.0 Content-Type: multipart/alternative; boundary="----=_NextPart_000_0014_01C3068D.79F06960" X-Priority: 3 X-MSMail-Priority: Normal X-Mailer: Microsoft Outlook Express 6.00.2600.0000 X-MimeOLE: Produced By Microsoft MimeOLE V6.00.2600.0000 X-Sender-IP: 200.148.36.99 Sender: owner-obm-l@sucuri.mat.puc-rio.br Precedence: bulk Reply-To: obm-l@mat.puc-rio.br This is a multi-part message in MIME format. ------=_NextPart_000_0014_01C3068D.79F06960 Content-Type: text/plain; charset="iso-8859-1" Content-Transfer-Encoding: quoted-printable ----- Original Message -----=20 From: Faelccmm@aol.com=20 To: obm-l@mat.puc-rio.br=20 Sent: Saturday, April 19, 2003 3:07 PM Subject: [obm-l] MDC -------------------------------------------------------------------------= ----- E-mail Premium BOL=20 Antiv=EDrus, anti-spam e at=E9 100 MB de espa=E7o. Assine j=E1! http://email.bol.com.br=20 -------------------------------------------------------------------------= ----- Ol=E1 pessoal,=20 Vejam a quest=E3o:=20 Um grupo de escoteiros deve montar guarda nos 4 postos Norte, Sul, = Leste e Oeste de seu acampamento. No posto Norte a guarda deve ser dia e = noite; no posto Sul a guarda vai das 6 da manh=E3 =E0s 18 da tarde; no = posto Leste, das 22 da noite =E0s 6 da manh=E3 e no posto Oeste, das 14 = da tarde =E0s 6 da manh=E3. Qual deve ser o turno m=E1ximo de guarda, de = modo que todos permane=E7am igual tempo de sentinela (sem mudar de = lugar) ? Quantos escoteiros s=E3o necess=E1rios para 24 horas de guarda, = de modo que para escoteiro n=E3o d=EA duas guardas no mesmo dia ?=20 Obs: A primeira pergunta eu resolvi tirando o m.d.c das amplitudes = hor=E1rias de cada posto de guarda e cheguei =E0 resposta do livro R: 4 = horas. Mas n=E3o estou conseguindo resolver a outra pergunta.=20 resp:=20 a) 4 horas=20 b) 15 escoteiros(*) =20 Solu=E7=E3o:somei todos(norte,sul,leste,oeste) e dividi por 4: N24+S12+L8+016/4: 60/4:15=20 ------=_NextPart_000_0014_01C3068D.79F06960 Content-Type: text/html; charset="iso-8859-1" Content-Transfer-Encoding: quoted-printable
 
----- Original Message -----
From:=20 Faelccmm@aol.com=20
Sent: Saturday, April 19, 2003 = 3:07=20 PM
Subject: [obm-l] MDC


E-mail Premium BOL
Antiv=EDrus, anti-spam e at=E9 100 MB de = espa=E7o.=20 Assine j=E1!
http://email.bol.com.br

Ol=E1 pessoal, =

Vejam a=20 quest=E3o:

Um grupo de escoteiros deve montar guarda nos 4 = postos Norte,=20 Sul, Leste e Oeste de seu acampamento. No posto Norte a guarda deve = ser dia e=20 noite; no posto Sul a guarda vai das 6 da manh=E3 =E0s 18 da tarde; no = posto=20 Leste, das 22 da noite =E0s 6 da manh=E3 e no posto Oeste, das 14 da = tarde =E0s 6 da=20 manh=E3. Qual deve ser o turno m=E1ximo de guarda, de modo que todos = permane=E7am=20 igual tempo de sentinela (sem mudar de lugar) ? Quantos escoteiros = s=E3o=20 necess=E1rios para 24 horas de guarda, de modo que para escoteiro = n=E3o d=EA duas=20 guardas no mesmo dia ?

Obs: A primeira pergunta eu resolvi=20  tirando o  m.d.c das amplitudes hor=E1rias de cada posto de = guarda e=20 cheguei =E0 resposta do livro R: 4 horas. Mas n=E3o estou conseguindo = resolver a=20 outra pergunta.

resp:

a) 4 horas
b) 15 = escoteiros(*)=20
 
 
 
 
Solu=E7=E3o:somei = todos(norte,sul,leste,oeste) e=20 dividi  por 4:
 
N24+S12+L8+016/4:
60/4:15
 
 
 
 
 
------=_NextPart_000_0014_01C3068D.79F06960-- ========================================================================= Instruções para entrar na lista, sair da lista e usar a lista em http://www.mat.puc-rio.br/~nicolau/olimp/obm-l.html O administrador desta lista é ========================================================================= From owner-obm-l@sucuri.mat.puc-rio.br Sat Apr 19 16:17:32 2003 Return-Path: Received: (from majordom@localhost) by sucuri.mat.puc-rio.br (8.9.3/8.9.3) id QAA09062 for obm-l-MTTP; Sat, 19 Apr 2003 16:15:28 -0300 Received: from krypton.hosting4u.net (krypton.hosting4u.net [209.15.2.78]) by sucuri.mat.puc-rio.br (8.9.3/8.9.3) with ESMTP id QAA09057 for ; Sat, 19 Apr 2003 16:15:24 -0300 Received: from gargamel (200-158-200-50.dsl.telesp.net.br [200.158.200.50]) by krypton.hosting4u.net (Postfix) with ESMTP id C70CCA0DFC for ; Sat, 19 Apr 2003 14:13:31 -0500 (CDT) Message-ID: <200304191616520750.0155912D@smtp.watersportsbrazil.com> In-Reply-To: References: X-Mailer: Calypso Version 3.30.00.00 (3) Date: Sat, 19 Apr 2003 16:16:52 -0300 From: "Ariel de Silvio" To: obm-l@mat.puc-rio.br Subject: Re: [obm-l] MDC Mime-Version: 1.0 Content-Type: multipart/alternative; boundary="=====_105077981219939=_" Sender: owner-obm-l@sucuri.mat.puc-rio.br Precedence: bulk Reply-To: obm-l@mat.puc-rio.br --=====_105077981219939=_ Content-Type: text/plain; charset="us-ascii" Content-Transfer-Encoding: quoted-printable Eita, acho que vai ser minha primeira colabora=E7=E3o na lista, hehehe A segunda =E9 at=E9 mais simples que a primeira, pense em 1 dia de guarda= nas torres: N: 24 h S: 12 h E: 8 h O: 16 h Tempo total de guarda: 60 horas Se cada escoteiro faz guardas de 4 horas: 60 / 4 =3D 15 escoteiros Eh isso n=E3o? []s Ariel *********** MENSAGEM ORIGINAL *********** As 14:07 de 19/4/2003 Faelccmm@aol.com escreveu: Ol=E1 pessoal, Vejam a quest=E3o: Um grupo de escoteiros deve montar guarda nos 4 postos Norte, Sul, Leste e= Oeste de seu acampamento. No posto Norte a guarda deve ser dia e noite; no= posto Sul a guarda vai das 6 da manh=E3 =E0s 18 da tarde; no posto Leste,= das 22 da noite =E0s 6 da manh=E3 e no posto Oeste, das 14 da tarde =E0s 6= da manh=E3. Qual deve ser o turno m=E1ximo de guarda, de modo que todos= permane=E7am igual tempo de sentinela (sem mudar de lugar) ? Quantos= escoteiros s=E3o necess=E1rios para 24 horas de guarda, de modo que para= escoteiro n=E3o d=EA duas guardas no mesmo dia ? Obs: A primeira pergunta eu resolvi tirando o m.d.c das amplitudes= hor=E1rias de cada posto de guarda e cheguei =E0 resposta do livro R: 4= horas. Mas n=E3o estou conseguindo resolver a outra pergunta. resp: a) 4 horas b) 15 escoteiros --=====_105077981219939=_ Content-Type: text/html; charset="us-ascii"
Eita, acho que vai ser minha primeira colaboração na lista, hehehe
 
A segunda é até mais simples que a primeira, pense em 1 dia de guarda nas torres:
    N: 24 h
    S: 12 h
    E: 8 h
    O: 16 h
Tempo total de guarda: 60 horas
 
Se cada escoteiro faz guardas de 4 horas:
 
60 / 4 = 15 escoteiros
 
Eh isso não?
[]s
Ariel

*********** MENSAGEM ORIGINAL ***********

As 14:07 de 19/4/2003 Faelccmm@aol.com escreveu:
Olá pessoal,

Vejam a questão:

Um grupo de escoteiros deve montar guarda nos 4 postos Norte, Sul, Leste e Oeste de seu acampamento. No posto Norte a guarda deve ser dia e noite; no posto Sul a guarda vai das 6 da manhã às 18 da tarde; no posto Leste, das 22 da noite às 6 da manhã e no posto Oeste, das 14 da tarde às 6 da manhã. Qual deve ser o turno máximo de guarda, de modo que todos permaneçam igual tempo de sentinela (sem mudar de lugar) ? Quantos escoteiros são necessários para 24 horas de guarda, de modo que para escoteiro não dê duas guardas no mesmo dia ?

Obs: A primeira pergunta eu resolvi  tirando o  m.d.c das amplitudes horárias de cada posto de guarda e cheguei à resposta do livro R: 4 horas. Mas não estou conseguindo resolver a outra pergunta.

resp:

a) 4 horas
b) 15 escoteiros
--=====_105077981219939=_-- ========================================================================= Instruções para entrar na lista, sair da lista e usar a lista em http://www.mat.puc-rio.br/~nicolau/olimp/obm-l.html O administrador desta lista é ========================================================================= From owner-obm-l@sucuri.mat.puc-rio.br Sat Apr 19 16:35:50 2003 Return-Path: Received: (from majordom@localhost) by sucuri.mat.puc-rio.br (8.9.3/8.9.3) id QAA09958 for obm-l-MTTP; Sat, 19 Apr 2003 16:34:16 -0300 Received: from trex-b.centroin.com.br (trex-b.centroin.com.br [200.225.63.136]) by sucuri.mat.puc-rio.br (8.9.3/8.9.3) with ESMTP id QAA09954 for ; Sat, 19 Apr 2003 16:34:13 -0300 Received: from centroin.com.br (RJ090116.user.veloxzone.com.br [200.141.90.116] (may be forged)) (authenticated bits=0) by trex-b.centroin.com.br (8.12.9/8.12.9) with ESMTP id h3JJXhVi002896 for ; Sat, 19 Apr 2003 16:33:44 -0300 (EST) Message-ID: <3EA1A4D0.4050205@centroin.com.br> Date: Sat, 19 Apr 2003 16:34:40 -0300 From: "A. C. Morgado" User-Agent: Mozilla/5.0 (Windows; U; Windows NT 5.0; en-US; rv:1.0.2) Gecko/20030208 Netscape/7.02 X-Accept-Language: en-us, en MIME-Version: 1.0 To: obm-l@mat.puc-rio.br Subject: Re: [obm-l] MDC References: <001701c306a6$a04f1160$f4befea9@xx> Content-Type: multipart/alternative; boundary="------------020400000107080600070208" Sender: owner-obm-l@sucuri.mat.puc-rio.br Precedence: bulk Reply-To: obm-l@mat.puc-rio.br --------------020400000107080600070208 Content-Type: text/plain; charset=ISO-8859-1; format=flowed Content-Transfer-Encoding: 8bit Soh uma provocaçaozinha para aguçar o espirito critico dos companheiros. Eu acho (acho eh bondade minha; eu tenho certeza!) que a resposta do item b eh 30. Morgado Blue Ice wrote: > > > ----- Original Message ----- > From: Faelccmm@aol.com > To: obm-l@mat.puc-rio.br > Sent: Saturday, April 19, 2003 3:07 PM > Subject: [obm-l] MDC > > ------------------------------------------------------------------------ > E-mail Premium BOL > Antivírus, anti-spam e até 100 MB de espaço. Assine já! > http://email.bol.com.br > ------------------------------------------------------------------------ > Olá pessoal, > > Vejam a questão: > > Um grupo de escoteiros deve montar guarda nos 4 postos Norte, Sul, > Leste e Oeste de seu acampamento. No posto Norte a guarda deve ser > dia e noite; no posto Sul a guarda vai das 6 da manhã às 18 da > tarde; no posto Leste, das 22 da noite às 6 da manhã e no posto > Oeste, das 14 da tarde às 6 da manhã. Qual deve ser o turno máximo > de guarda, de modo que todos permaneçam igual tempo de sentinela > (sem mudar de lugar) ? Quantos escoteiros são necessários para 24 > horas de guarda, de modo que para escoteiro não dê duas guardas no > mesmo dia ? > > Obs: A primeira pergunta eu resolvi tirando o m.d.c das > amplitudes horárias de cada posto de guarda e cheguei à resposta > do livro R: 4 horas. Mas não estou conseguindo resolver a outra > pergunta. > > resp: > > a) 4 horas > b) 15 escoteiros(*) > > > > Solução:somei todos(norte,sul,leste,oeste) e dividi por 4: > > N24+S12+L8+016/4: > 60/4:15 > > > > > > --------------020400000107080600070208 Content-Type: text/html; charset=us-ascii Content-Transfer-Encoding: 7bit Soh uma provocaçaozinha para aguçar o espirito critico dos companheiros. Eu acho (acho eh bondade minha; eu tenho certeza!) que a resposta do item b eh 30.
Morgado

Blue Ice wrote:
 
----- Original Message -----
Sent: Saturday, April 19, 2003 3:07 PM
Subject: [obm-l] MDC


E-mail Premium BOL
Antivírus, anti-spam e até 100 MB de espaço. Assine já!
http://email.bol.com.br

Olá pessoal,

Vejam a questão:

Um grupo de escoteiros deve montar guarda nos 4 postos Norte, Sul, Leste e Oeste de seu acampamento. No posto Norte a guarda deve ser dia e noite; no posto Sul a guarda vai das 6 da manhã às 18 da tarde; no posto Leste, das 22 da noite às 6 da manhã e no posto Oeste, das 14 da tarde às 6 da manhã. Qual deve ser o turno máximo de guarda, de modo que todos permaneçam igual tempo de sentinela (sem mudar de lugar) ? Quantos escoteiros são necessários para 24 horas de guarda, de modo que para escoteiro não dê duas guardas no mesmo dia ?

Obs: A primeira pergunta eu resolvi  tirando o  m.d.c das amplitudes horárias de cada posto de guarda e cheguei à resposta do livro R: 4 horas. Mas não estou conseguindo resolver a outra pergunta.

resp:

a) 4 horas
b) 15 escoteiros(*)
 
 
 
 
Solução:somei todos(norte,sul,leste,oeste) e dividi  por 4:
 
N24+S12+L8+016/4:
60/4:15
 
 
 
 
 

--------------020400000107080600070208-- ========================================================================= Instruções para entrar na lista, sair da lista e usar a lista em http://www.mat.puc-rio.br/~nicolau/olimp/obm-l.html O administrador desta lista é ========================================================================= From owner-obm-l@sucuri.mat.puc-rio.br Sat Apr 19 17:48:13 2003 Return-Path: Received: (from majordom@localhost) by sucuri.mat.puc-rio.br (8.9.3/8.9.3) id RAA11551 for obm-l-MTTP; Sat, 19 Apr 2003 17:44:20 -0300 Received: from krypton.hosting4u.net (krypton.hosting4u.net [209.15.2.78]) by sucuri.mat.puc-rio.br (8.9.3/8.9.3) with ESMTP id RAA11547 for ; Sat, 19 Apr 2003 17:44:15 -0300 Received: from gargamel (200-158-200-201.dsl.telesp.net.br [200.158.200.201]) by krypton.hosting4u.net (Postfix) with ESMTP id 1849DA0E4D for ; Sat, 19 Apr 2003 15:42:05 -0500 (CDT) Message-ID: <200304191745230710.01A69E56@smtp.watersportsbrazil.com> In-Reply-To: <3EA1A4D0.4050205@centroin.com.br> References: <001701c306a6$a04f1160$f4befea9@xx> <3EA1A4D0.4050205@centroin.com.br> X-Mailer: Calypso Version 3.30.00.00 (3) Date: Sat, 19 Apr 2003 17:45:23 -0300 From: "Ariel de Silvio" To: obm-l@mat.puc-rio.br Subject: Re: [obm-l] MDC Mime-Version: 1.0 Content-Type: multipart/alternative; boundary="=====_105078512329766=_" Sender: owner-obm-l@sucuri.mat.puc-rio.br Precedence: bulk Reply-To: obm-l@mat.puc-rio.br --=====_105078512329766=_ Content-Type: text/plain; charset="us-ascii" Content-Transfer-Encoding: quoted-printable N=E3o entendi pq 30... Putz, tirou toda minha felicidade... :( primeira vez q eu falo e ainda vem= me falar q eu errei :(( se puder explicar agradeco... eheheh falou Ariel *********** MENSAGEM ORIGINAL *********** As 16:34 de 19/4/2003 A. C. Morgado escreveu: Soh uma provoca=E7aozinha para agu=E7ar o espirito critico dos= companheiros. Eu acho (acho eh bondade minha; eu tenho certeza!) que a= resposta do item b eh 30. Morgado Blue Ice wrote: ----- Original Message ----- From: Faelccmm@aol.com To: obm-l@mat.puc-rio.br Sent: Saturday, April 19, 2003 3:07 PM Subject: [obm-l] MDC E-mail Premium BOL Antiv=EDrus, anti-spam e at=E9 100 MB de espa=E7o. Assine j=E1! http://email.bol.com.br Ol=E1 pessoal, Vejam a quest=E3o: Um grupo de escoteiros deve montar guarda nos 4 postos Norte, Sul, Leste e= Oeste de seu acampamento. No posto Norte a guarda deve ser dia e noite; no= posto Sul a guarda vai das 6 da manh=E3 =E0s 18 da tarde; no posto Leste,= das 22 da noite =E0s 6 da manh=E3 e no posto Oeste, das 14 da tarde =E0s 6= da manh=E3. Qual deve ser o turno m=E1ximo de guarda, de modo que todos= permane=E7am igual tempo de sentinela (sem mudar de lugar) ? Quantos= escoteiros s=E3o necess=E1rios para 24 horas de guarda, de modo que para= escoteiro n=E3o d=EA duas guardas no mesmo dia ? Obs: A primeira pergunta eu resolvi tirando o m.d.c das amplitudes= hor=E1rias de cada posto de guarda e cheguei =E0 resposta do livro R: 4= horas. Mas n=E3o estou conseguindo resolver a outra pergunta. resp: a) 4 horas b) 15 escoteiros(*) Solu=E7=E3o:somei todos(norte,sul,leste,oeste) e dividi por 4: N24+S12+L8+016/4: 60/4:15 --=====_105078512329766=_ Content-Type: text/html; charset="us-ascii"
Não entendi pq 30...
Putz, tirou toda minha felicidade... :( primeira vez q eu falo e ainda vem me falar q eu errei :((
 
se puder explicar agradeco...
 
eheheh
falou
Ariel

*********** MENSAGEM ORIGINAL ***********

As 16:34 de 19/4/2003 A. C. Morgado escreveu:
Soh uma provocaçaozinha para aguçar o espirito critico dos companheiros. Eu acho (acho eh bondade minha; eu tenho certeza!) que a resposta do item b eh 30.
Morgado

Blue Ice wrote:
 
----- Original Message -----
Sent: Saturday, April 19, 2003 3:07 PM
Subject: [obm-l] MDC


E-mail Premium BOL
Antivírus, anti-spam e até 100 MB de espaço. Assine já!
http://email.bol.com.br

Olá pessoal,

Vejam a questão:

Um grupo de escoteiros deve montar guarda nos 4 postos Norte, Sul, Leste e Oeste de seu acampamento. No posto Norte a guarda deve ser dia e noite; no posto Sul a guarda vai das 6 da manhã às 18 da tarde; no posto Leste, das 22 da noite às 6 da manhã e no posto Oeste, das 14 da tarde às 6 da manhã. Qual deve ser o turno máximo de guarda, de modo que todos permaneçam igual tempo de sentinela (sem mudar de lugar) ? Quantos escoteiros são necessários para 24 horas de guarda, de modo que para escoteiro não dê duas guardas no mesmo dia ?

Obs: A primeira pergunta eu resolvi  tirando o  m.d.c das amplitudes horárias de cada posto de guarda e cheguei à resposta do livro R: 4 horas. Mas não estou conseguindo resolver a outra pergunta.

resp:

a) 4 horas
b) 15 escoteiros(*)
 
 
 
 
Solução:somei todos(norte,sul,leste,oeste) e dividi  por 4:
 
N24+S12+L8+016/4:
60/4:15
 
 
 
 
 

--=====_105078512329766=_-- ========================================================================= Instruções para entrar na lista, sair da lista e usar a lista em http://www.mat.puc-rio.br/~nicolau/olimp/obm-l.html O administrador desta lista é ========================================================================= From owner-obm-l@sucuri.mat.puc-rio.br Sat Apr 19 17:59:05 2003 Return-Path: Received: (from majordom@localhost) by sucuri.mat.puc-rio.br (8.9.3/8.9.3) id RAA11822 for obm-l-MTTP; Sat, 19 Apr 2003 17:56:11 -0300 Received: from krypton.hosting4u.net (krypton.hosting4u.net [209.15.2.78]) by sucuri.mat.puc-rio.br (8.9.3/8.9.3) with ESMTP id RAA11818 for ; Sat, 19 Apr 2003 17:56:07 -0300 Received: from gargamel (200-158-200-201.dsl.telesp.net.br [200.158.200.201]) by krypton.hosting4u.net (Postfix) with ESMTP id DD9DFA08E8 for ; Sat, 19 Apr 2003 15:55:33 -0500 (CDT) Message-ID: <200304191758510720.01B2F31B@smtp.watersportsbrazil.com> In-Reply-To: <3EA1A4D0.4050205@centroin.com.br> References: <001701c306a6$a04f1160$f4befea9@xx> <3EA1A4D0.4050205@centroin.com.br> X-Mailer: Calypso Version 3.30.00.00 (3) Date: Sat, 19 Apr 2003 17:58:51 -0300 From: "Ariel de Silvio" To: obm-l@mat.puc-rio.br Subject: Re: [obm-l] MDC Mime-Version: 1.0 Content-Type: multipart/alternative; boundary="=====_105078593111224=_" Sender: owner-obm-l@sucuri.mat.puc-rio.br Precedence: bulk Reply-To: obm-l@mat.puc-rio.br --=====_105078593111224=_ Content-Type: text/plain; charset="us-ascii" Content-Transfer-Encoding: quoted-printable Logo apos enviar o outro email, analizei com outros olhos, e o que vc diz= tem sentido... mesmo sem saber sua justificativa.... Se vc considerar o inicio das 24 horas, meia noite... como ele diz q eh um= dia, consideremos o dia oficial, eh esse o caminho? Sendo das 0h as 24h... O turno da Torre Leste inicia-se =E0s 22h, portanto at=E9 as 24 hrs se= passaram 2h apenas... isso significa q todos os turnos devem durar apenas= 2h para serem iguais e ninguem virar o dia de guarda... Ent=E3o soma-se as 60h de guarda, q deve dividida por turnos de 2h,= resultando em 30 escoteiros... O que salva essa resolu=E7=E3o =E9 o finalzinho do enunciado "n=E3o d=EA= duas guardas NO MESMO DIA"... Agora ficou certo?? []s Ariel *********** MENSAGEM ORIGINAL *********** As 16:34 de 19/4/2003 A. C. Morgado escreveu: Soh uma provoca=E7aozinha para agu=E7ar o espirito critico dos= companheiros. Eu acho (acho eh bondade minha; eu tenho certeza!) que a= resposta do item b eh 30. Morgado Blue Ice wrote: ----- Original Message ----- From: Faelccmm@aol.com To: obm-l@mat.puc-rio.br Sent: Saturday, April 19, 2003 3:07 PM Subject: [obm-l] MDC E-mail Premium BOL Antiv=EDrus, anti-spam e at=E9 100 MB de espa=E7o. Assine j=E1! http://email.bol.com.br Ol=E1 pessoal, Vejam a quest=E3o: Um grupo de escoteiros deve montar guarda nos 4 postos Norte, Sul, Leste e= Oeste de seu acampamento. No posto Norte a guarda deve ser dia e noite; no= posto Sul a guarda vai das 6 da manh=E3 =E0s 18 da tarde; no posto Leste,= das 22 da noite =E0s 6 da manh=E3 e no posto Oeste, das 14 da tarde =E0s 6= da manh=E3. Qual deve ser o turno m=E1ximo de guarda, de modo que todos= permane=E7am igual tempo de sentinela (sem mudar de lugar) ? Quantos= escoteiros s=E3o necess=E1rios para 24 horas de guarda, de modo que para= escoteiro n=E3o d=EA duas guardas no mesmo dia ? Obs: A primeira pergunta eu resolvi tirando o m.d.c das amplitudes= hor=E1rias de cada posto de guarda e cheguei =E0 resposta do livro R: 4= horas. Mas n=E3o estou conseguindo resolver a outra pergunta. resp: a) 4 horas b) 15 escoteiros(*) Solu=E7=E3o:somei todos(norte,sul,leste,oeste) e dividi por 4: N24+S12+L8+016/4: 60/4:15 --=====_105078593111224=_ Content-Type: text/html; charset="us-ascii"
Logo apos enviar o outro email, analizei com outros olhos, e o que vc diz tem sentido... mesmo sem saber sua justificativa....
 
Se vc considerar o inicio das 24 horas, meia noite... como ele diz q eh um dia, consideremos o dia oficial, eh esse o caminho?
Sendo das 0h as 24h...
O turno da Torre Leste inicia-se às 22h, portanto até as 24 hrs se passaram 2h apenas... isso significa q todos os turnos devem durar apenas 2h para serem iguais e ninguem virar o dia de guarda...
 
Então soma-se as 60h de guarda, q deve dividida por turnos de 2h, resultando em 30 escoteiros...
 
O que salva essa resolução é o finalzinho do enunciado "não dê duas guardas NO MESMO DIA"...
 
Agora ficou certo??
 
[]s
Ariel
 
*********** MENSAGEM ORIGINAL ***********

As 16:34 de 19/4/2003 A. C. Morgado escreveu:
Soh uma provocaçaozinha para aguçar o espirito critico dos companheiros. Eu acho (acho eh bondade minha; eu tenho certeza!) que a resposta do item b eh 30.
Morgado

Blue Ice wrote:
 
----- Original Message -----
Sent: Saturday, April 19, 2003 3:07 PM
Subject: [obm-l] MDC


E-mail Premium BOL
Antivírus, anti-spam e até 100 MB de espaço. Assine já!
http://email.bol.com.br

Olá pessoal,

Vejam a questão:

Um grupo de escoteiros deve montar guarda nos 4 postos Norte, Sul, Leste e Oeste de seu acampamento. No posto Norte a guarda deve ser dia e noite; no posto Sul a guarda vai das 6 da manhã às 18 da tarde; no posto Leste, das 22 da noite às 6 da manhã e no posto Oeste, das 14 da tarde às 6 da manhã. Qual deve ser o turno máximo de guarda, de modo que todos permaneçam igual tempo de sentinela (sem mudar de lugar) ? Quantos escoteiros são necessários para 24 horas de guarda, de modo que para escoteiro não dê duas guardas no mesmo dia ?

Obs: A primeira pergunta eu resolvi  tirando o  m.d.c das amplitudes horárias de cada posto de guarda e cheguei à resposta do livro R: 4 horas. Mas não estou conseguindo resolver a outra pergunta.

resp:

a) 4 horas
b) 15 escoteiros(*)
 
 
 
 
Solução:somei todos(norte,sul,leste,oeste) e dividi  por 4:
 
N24+S12+L8+016/4:
60/4:15
 
 
 
 
 

--=====_105078593111224=_-- ========================================================================= Instruções para entrar na lista, sair da lista e usar a lista em http://www.mat.puc-rio.br/~nicolau/olimp/obm-l.html O administrador desta lista é ========================================================================= From owner-obm-l@sucuri.mat.puc-rio.br Sat Apr 19 19:06:06 2003 Return-Path: Received: (from majordom@localhost) by sucuri.mat.puc-rio.br (8.9.3/8.9.3) id TAA13279 for obm-l-MTTP; Sat, 19 Apr 2003 19:02:54 -0300 Received: from trex-b.centroin.com.br (trex-b.centroin.com.br [200.225.63.136]) by sucuri.mat.puc-rio.br (8.9.3/8.9.3) with ESMTP id TAA13275 for ; Sat, 19 Apr 2003 19:02:51 -0300 Received: from centroin.com.br (RJ090116.user.veloxzone.com.br [200.141.90.116] (may be forged)) (authenticated bits=0) by trex-b.centroin.com.br (8.12.9/8.12.9) with ESMTP id h3JM2JVi004520 for ; Sat, 19 Apr 2003 19:02:20 -0300 (EST) Message-ID: <3EA1C7A5.4050409@centroin.com.br> Date: Sat, 19 Apr 2003 19:03:17 -0300 From: "A. C. Morgado" User-Agent: Mozilla/5.0 (Windows; U; Windows NT 5.0; en-US; rv:1.0.2) Gecko/20030208 Netscape/7.02 X-Accept-Language: en-us, en MIME-Version: 1.0 To: obm-l@mat.puc-rio.br Subject: Re: [obm-l] MDC References: <001701c306a6$a04f1160$f4befea9@xx> <3EA1A4D0.4050205@centroin.com.br> <200304191758510720.01B2F31B@smtp.watersportsbrazil.com> Content-Type: multipart/alternative; boundary="------------060309000800080706000701" Sender: owner-obm-l@sucuri.mat.puc-rio.br Precedence: bulk Reply-To: obm-l@mat.puc-rio.br --------------060309000800080706000701 Content-Type: text/plain; charset=ISO-8859-1; format=flowed Content-Transfer-Encoding: 8bit Eh isso aih, garoto! Na realidade eu mandei essa provocaçao porque hoje em dia a regra em vestibulares e concursos eh a presença de questoes com enunciados dubios e o que mais me espanta eh a passividade de alunos e principalmente de professores diante de tais enunciados. Nao tenho a menor duvida de que o gabarito oficial de tal questao seja 15. Tambem nao tenho duvida que isso foi questao de prova e o gabarito 15 foi aceito sem grandes contestaçoes. Eh preciso protestar contra essas coisas. []s Morgado Ariel de Silvio wrote: > Logo apos enviar o outro email, analizei com outros olhos, e o que vc > diz tem sentido... mesmo sem saber sua justificativa.... > > Se vc considerar o inicio das 24 horas, meia noite... como ele diz q > eh um dia, consideremos o dia oficial, eh esse o caminho? > Sendo das 0h as 24h... > O turno da Torre Leste inicia-se às 22h, portanto até as 24 hrs se > passaram 2h apenas... isso significa q todos os turnos devem durar > apenas 2h para serem iguais e ninguem virar o dia de guarda... > > Então soma-se as 60h de guarda, q deve dividida por turnos de 2h, > resultando em 30 escoteiros... > > O que salva essa resolução é o finalzinho do enunciado "não dê duas > guardas NO MESMO DIA"... > > Agora ficou certo?? > > []s > Ariel > > *********** MENSAGEM ORIGINAL *********** > > As 16:34 de 19/4/2003 A. C. Morgado escreveu: > > Soh uma provocaçaozinha para aguçar o espirito critico dos > companheiros. Eu acho (acho eh bondade minha; eu tenho certeza!) > que a resposta do item b eh 30. > Morgado > > Blue Ice wrote: > >> >> >> ----- Original Message ----- >> From: Faelccmm@aol.com >> To: obm-l@mat.puc-rio.br >> Sent: Saturday, April 19, 2003 3:07 PM >> Subject: [obm-l] MDC >> >> ------------------------------------------------------------------------ >> E-mail Premium BOL >> Antivírus, anti-spam e até 100 MB de espaço. Assine já! >> http://email.bol.com.br >> ------------------------------------------------------------------------ >> Olá pessoal, >> >> Vejam a questão: >> >> Um grupo de escoteiros deve montar guarda nos 4 postos Norte, >> Sul, Leste e Oeste de seu acampamento. No posto Norte a >> guarda deve ser dia e noite; no posto Sul a guarda vai das 6 >> da manhã às 18 da tarde; no posto Leste, das 22 da noite às 6 >> da manhã e no posto Oeste, das 14 da tarde às 6 da manhã. >> Qual deve ser o turno máximo de guarda, de modo que todos >> permaneçam igual tempo de sentinela (sem mudar de lugar) ? >> Quantos escoteiros são necessários para 24 horas de guarda, >> de modo que para escoteiro não dê duas guardas no mesmo dia ? >> >> Obs: A primeira pergunta eu resolvi tirando o m.d.c das >> amplitudes horárias de cada posto de guarda e cheguei à >> resposta do livro R: 4 horas. Mas não estou conseguindo >> resolver a outra pergunta. >> >> resp: >> >> a) 4 horas >> b) 15 escoteiros(*) >> >> >> >> Solução:somei todos(norte,sul,leste,oeste) e dividi por 4: >> >> N24+S12+L8+016/4: >> 60/4:15 >> >> >> >> >> >> > --------------060309000800080706000701 Content-Type: text/html; charset=us-ascii Content-Transfer-Encoding: 7bit Eh isso aih, garoto!
Na realidade eu mandei essa provocaçao porque hoje em dia a regra em vestibulares e concursos eh a presença de questoes com enunciados dubios e o que mais me espanta eh a passividade de alunos e principalmente de professores diante de tais enunciados. Nao tenho a menor duvida de que o gabarito oficial de tal questao seja 15. Tambem nao tenho duvida que isso foi questao de prova e o gabarito 15 foi aceito sem grandes contestaçoes. Eh preciso protestar contra essas coisas.
[]s
Morgado

Ariel de Silvio wrote:
Logo apos enviar o outro email, analizei com outros olhos, e o que vc diz tem sentido... mesmo sem saber sua justificativa....
 
Se vc considerar o inicio das 24 horas, meia noite... como ele diz q eh um dia, consideremos o dia oficial, eh esse o caminho?
Sendo das 0h as 24h...
O turno da Torre Leste inicia-se às 22h, portanto até as 24 hrs se passaram 2h apenas... isso significa q todos os turnos devem durar apenas 2h para serem iguais e ninguem virar o dia de guarda...
 
Então soma-se as 60h de guarda, q deve dividida por turnos de 2h, resultando em 30 escoteiros...
 
O que salva essa resolução é o finalzinho do enunciado "não dê duas guardas NO MESMO DIA"...
 
Agora ficou certo??
 
[]s
Ariel
 
*********** MENSAGEM ORIGINAL ***********

As 16:34 de 19/4/2003 A. C. Morgado escreveu:
Soh uma provocaçaozinha para aguçar o espirito critico dos companheiros. Eu acho (acho eh bondade minha; eu tenho certeza!) que a resposta do item b eh 30.
Morgado

Blue Ice wrote:
 
----- Original Message -----
Sent: Saturday, April 19, 2003 3:07 PM
Subject: [obm-l] MDC


E-mail Premium BOL
Antivírus, anti-spam e até 100 MB de espaço. Assine já!
http://email.bol.com.br

Olá pessoal,

Vejam a questão:

Um grupo de escoteiros deve montar guarda nos 4 postos Norte, Sul, Leste e Oeste de seu acampamento. No posto Norte a guarda deve ser dia e noite; no posto Sul a guarda vai das 6 da manhã às 18 da tarde; no posto Leste, das 22 da noite às 6 da manhã e no posto Oeste, das 14 da tarde às 6 da manhã. Qual deve ser o turno máximo de guarda, de modo que todos permaneçam igual tempo de sentinela (sem mudar de lugar) ? Quantos escoteiros são necessários para 24 horas de guarda, de modo que para escoteiro não dê duas guardas no mesmo dia ?

Obs: A primeira pergunta eu resolvi  tirando o  m.d.c das amplitudes horárias de cada posto de guarda e cheguei à resposta do livro R: 4 horas. Mas não estou conseguindo resolver a outra pergunta.

resp:

a) 4 horas
b) 15 escoteiros(*)
 
 
 
 
Solução:somei todos(norte,sul,leste,oeste) e dividi  por 4:
 
N24+S12+L8+016/4:
60/4:15
 
 
 
 
 


--------------060309000800080706000701-- ========================================================================= Instruções para entrar na lista, sair da lista e usar a lista em http://www.mat.puc-rio.br/~nicolau/olimp/obm-l.html O administrador desta lista é ========================================================================= From owner-obm-l@sucuri.mat.puc-rio.br Sat Apr 19 19:06:30 2003 Return-Path: Received: (from majordom@localhost) by sucuri.mat.puc-rio.br (8.9.3/8.9.3) id TAA13289 for obm-l-MTTP; Sat, 19 Apr 2003 19:03:58 -0300 Received: from sina.bol.com.br (sina.bol.com.br [200.221.24.27]) by sucuri.mat.puc-rio.br (8.9.3/8.9.3) with ESMTP id TAA13285 for ; Sat, 19 Apr 2003 19:03:55 -0300 Received: from xx (200.221.24.192) by sina.bol.com.br (5.1.071) id 3E7665FB00959267 for obm-l@mat.puc-rio.br; Sat, 19 Apr 2003 19:03:24 -0300 Message-ID: <000901c306bf$82427720$f4befea9@xx> From: "Blue Ice" To: References: <001701c306a6$a04f1160$f4befea9@xx> <3EA1A4D0.4050205@centroin.com.br> <200304191758510720.01B2F31B@smtp.watersportsbrazil.com> Subject: Re: [obm-l] MDC Date: Sat, 19 Apr 2003 19:03:26 -0300 MIME-Version: 1.0 Content-Type: multipart/alternative; boundary="----=_NextPart_000_0006_01C306A6.5BB45AB0" X-Priority: 3 X-MSMail-Priority: Normal X-Mailer: Microsoft Outlook Express 6.00.2600.0000 X-MimeOLE: Produced By Microsoft MimeOLE V6.00.2600.0000 X-Sender-IP: 200.148.35.14 Sender: owner-obm-l@sucuri.mat.puc-rio.br Precedence: bulk Reply-To: obm-l@mat.puc-rio.br This is a multi-part message in MIME format. ------=_NextPart_000_0006_01C306A6.5BB45AB0 Content-Type: text/plain; charset="iso-8859-1" Content-Transfer-Encoding: quoted-printable ----- Original Message -----=20 From: Ariel de Silvio=20 To: obm-l@mat.puc-rio.br=20 Sent: Saturday, April 19, 2003 5:58 PM Subject: Re: [obm-l] MDC Logo apos enviar o outro email, analizei com outros olhos, e o que vc = diz tem sentido... mesmo sem saber sua justificativa.... Se vc considerar o inicio das 24 horas, meia noite... como ele diz q = eh um dia, consideremos o dia oficial, eh esse o caminho? Sendo das 0h as 24h... O turno da Torre Leste inicia-se =E0s 22h, portanto at=E9 as 24 hrs se = passaram 2h apenas... isso significa q todos os turnos devem durar = apenas 2h para serem iguais e ninguem virar o dia de guarda... Ent=E3o soma-se as 60h de guarda, q deve dividida por turnos de 2h, = resultando em 30 escoteiros... O que salva essa resolu=E7=E3o =E9 o finalzinho do enunciado "n=E3o = d=EA duas guardas NO MESMO DIA"... Agora ficou certo?? []s Ariel Meu racioc=EDnio foi que na primeira torre somei 24,foi isso.somei = todas e deu o resultado de 60h vc acertou, mas agora vc teria que fazer = a divis=E3o por 4. Pq??? Ora, s=E3o 4 torres, ent=E3o p se achar o = resultado, divida por 4, obtendo 15 escoteiros!n=E3o 30. []=B4s Ice ------=_NextPart_000_0006_01C306A6.5BB45AB0 Content-Type: text/html; charset="iso-8859-1" Content-Transfer-Encoding: quoted-printable
 
----- Original Message -----
From:=20 Ariel de Silvio
Sent: Saturday, April 19, 2003 = 5:58=20 PM
Subject: Re: [obm-l] MDC

Logo apos enviar o outro email, analizei com outros olhos, e o = que vc diz=20 tem sentido... mesmo sem saber sua justificativa....
 
Se vc considerar o inicio das 24 horas, meia noite... como ele = diz q eh=20 um dia, consideremos o dia oficial, eh esse o caminho?
Sendo das 0h as 24h...
O turno da Torre Leste inicia-se =E0s 22h, portanto at=E9 as 24 = hrs se=20 passaram 2h apenas... isso significa q todos os turnos devem durar = apenas 2h=20 para serem iguais e ninguem virar o dia de guarda...
 
Ent=E3o soma-se=20 as 60h de guarda, q deve dividida por turnos de 2h, resultando em 30=20 escoteiros...
 
O que salva essa resolu=E7=E3o =E9 o finalzinho do enunciado = "n=E3o d=EA duas guardas NO MESMO DIA"...
 
Agora ficou certo??
 
[]s
Ariel

Meu racioc=EDnio foi que na primeira torre somei 24,foi isso.somei = todas e=20 deu o resultado de 60h vc acertou, mas agora vc teria que fazer a = divis=E3o por=20 4. Pq??? Ora, s=E3o 4 torres, ent=E3o p se achar o resultado, divida = por 4,=20 obtendo 15 escoteiros!n=E3o 30.

 

 

[]=B4s

 

Ice

 

 


------=_NextPart_000_0006_01C306A6.5BB45AB0-- ========================================================================= Instruções para entrar na lista, sair da lista e usar a lista em http://www.mat.puc-rio.br/~nicolau/olimp/obm-l.html O administrador desta lista é ========================================================================= From owner-obm-l@sucuri.mat.puc-rio.br Sat Apr 19 19:10:53 2003 Return-Path: Received: (from majordom@localhost) by sucuri.mat.puc-rio.br (8.9.3/8.9.3) id TAA13419 for obm-l-MTTP; Sat, 19 Apr 2003 19:08:21 -0300 Received: from sina.bol.com.br (sina.bol.com.br [200.221.24.27]) by sucuri.mat.puc-rio.br (8.9.3/8.9.3) with ESMTP id TAA13414 for ; Sat, 19 Apr 2003 19:08:18 -0300 Received: from xx (200.221.24.192) by sina.bol.com.br (5.1.071) id 3E7665FB009594F5 for obm-l@mat.puc-rio.br; Sat, 19 Apr 2003 19:07:48 -0300 Message-ID: <002301c306c0$22c46690$f4befea9@xx> From: "Blue Ice" To: References: <001701c306a6$a04f1160$f4befea9@xx> <3EA1A4D0.4050205@centroin.com.br> Subject: Re: [obm-l] MDC Date: Sat, 19 Apr 2003 19:07:56 -0300 MIME-Version: 1.0 Content-Type: multipart/alternative; boundary="----=_NextPart_000_0020_01C306A6.FC981930" X-Priority: 3 X-MSMail-Priority: Normal X-Mailer: Microsoft Outlook Express 6.00.2600.0000 X-MimeOLE: Produced By Microsoft MimeOLE V6.00.2600.0000 X-Sender-IP: 200.148.35.14 Sender: owner-obm-l@sucuri.mat.puc-rio.br Precedence: bulk Reply-To: obm-l@mat.puc-rio.br This is a multi-part message in MIME format. ------=_NextPart_000_0020_01C306A6.FC981930 Content-Type: text/plain; charset="iso-8859-1" Content-Transfer-Encoding: quoted-printable ----- Original Message -----=20 From: A. C. Morgado=20 To: obm-l@mat.puc-rio.br=20 Sent: Saturday, April 19, 2003 4:34 PM Subject: Re: [obm-l] MDC Soh uma provoca=E7aozinha para agu=E7ar o espirito critico dos = companheiros. Eu acho (acho eh bondade minha; eu tenho certeza!) que a = resposta do item b eh 30. Morgado Blue Ice wrote: Seu racioc=EDnio est=E1 correto ,colega,por=E9m o meu tamb=E9m pode = ser tomado como certo ,pq vc levou em conta 2h de turno de cada = sentinela. 60/2:30 eu levei em conta um turno de 4 horas. 60/4:15 []=B4s Ice ICQ:177782914 Email:Vinicius84@hotmail.com Win XP Home Full/Speedy 256kbps=20 ------=_NextPart_000_0020_01C306A6.FC981930 Content-Type: text/html; charset="iso-8859-1" Content-Transfer-Encoding: quoted-printable
 
----- Original Message -----
From:=20 A. C.=20 Morgado
Sent: Saturday, April 19, 2003 = 4:34=20 PM
Subject: Re: [obm-l] MDC

Soh uma provoca=E7aozinha para agu=E7ar o espirito critico dos = companheiros.=20 Eu acho (acho eh bondade minha; eu tenho certeza!) que a resposta do = item b eh=20 30.
Morgado

Blue Ice wrote:
 
Seu racioc=EDnio est=E1 correto = ,colega,por=E9m o meu=20 tamb=E9m pode ser tomado como certo ,pq vc levou em conta 2h de turno = de cada=20 sentinela.
 
 
60/2:30
 
 
eu levei em conta um turno de 4=20 horas.
 
 
60/4:15
 
[]=B4s
 
Ice
 
 
ICQ:177782914 = Email:Vinicius84@hotmail.com
Win=20 XP Home Full/Speedy 256kbps
------=_NextPart_000_0020_01C306A6.FC981930-- ========================================================================= Instruções para entrar na lista, sair da lista e usar a lista em http://www.mat.puc-rio.br/~nicolau/olimp/obm-l.html O administrador desta lista é ========================================================================= From owner-obm-l@sucuri.mat.puc-rio.br Sat Apr 19 19:24:26 2003 Return-Path: Received: (from majordom@localhost) by sucuri.mat.puc-rio.br (8.9.3/8.9.3) id TAA14163 for obm-l-MTTP; Sat, 19 Apr 2003 19:21:51 -0300 Received: from salem.bol.com.br (salem.bol.com.br [200.221.24.25]) by sucuri.mat.puc-rio.br (8.9.3/8.9.3) with ESMTP id TAA14158 for ; Sat, 19 Apr 2003 19:21:47 -0300 Received: from bol.com.br (200.221.24.119) by salem.bol.com.br (5.1.071) id 3E98B814002122F4 for obm-l@mat.puc-rio.br; Sat, 19 Apr 2003 19:21:13 -0300 Date: Sat, 19 Apr 2003 19:21:13 -0300 Message-Id: Subject: [obm-l] Problema proposto 74 eureka 15... MIME-Version: 1.0 Content-Type: text/plain;charset="iso-8859-1" From: "rmr-olimp" To: obm-l@mat.puc-rio.br X-XaM3-API-Version: 2.4 R3 ( B4 ) X-SenderIP: 200.226.207.107 Content-Transfer-Encoding: 8bit X-MIME-Autoconverted: from quoted-printable to 8bit by sucuri.mat.puc-rio.br id TAA14159 Sender: owner-obm-l@sucuri.mat.puc-rio.br Precedence: bulk Reply-To: obm-l@mat.puc-rio.br Problema proposto 74 eureka 15... Ache todas as funçoes f: R -> R tais que: f(x+y)+f(x-y)=2.f(x).cos(y) fazendo: x+y=a x-y=b Substituindo: f(a)+f(b)=2.f((a+b)/2).cos((a-b)/2) Da trigonometria: sen(a)+sen(b)=2.sen((a+b)/2).cos((a-b)/2) cos(a)+sen(b)=2.cos((a+b)/2).cos((a-b)/2) Logo f(x)=sen(x) ou f(x)=cos(x) O que eu não sei provar é se essas são as únicas soluções e caso existam encontrá-las.. Se alguém puder quebrar um galho e me ajudar seria ótimo!! Rodrigo __________________________________________________________________________ Seleção de Softwares UOL. 10 softwares escolhidos pelo UOL para você e sua família. http://www.uol.com.br/selecao ========================================================================= Instruções para entrar na lista, sair da lista e usar a lista em http://www.mat.puc-rio.br/~nicolau/olimp/obm-l.html O administrador desta lista é ========================================================================= From owner-obm-l@sucuri.mat.puc-rio.br Sat Apr 19 19:31:13 2003 Return-Path: Received: (from majordom@localhost) by sucuri.mat.puc-rio.br (8.9.3/8.9.3) id TAA14485 for obm-l-MTTP; Sat, 19 Apr 2003 19:28:41 -0300 Received: from smtp-26.ig.com.br (smtp-26.ig.com.br [200.226.132.160]) by sucuri.mat.puc-rio.br (8.9.3/8.9.3) with SMTP id TAA14481 for ; Sat, 19 Apr 2003 19:28:38 -0300 Received: (qmail 22404 invoked from network); 19 Apr 2003 22:28:18 -0000 Received: from unknown (HELO xxxx) (200.165.170.19) by smtp-26.ig.com.br with SMTP; 19 Apr 2003 22:28:18 -0000 Message-ID: <004101c306c3$8901ece0$13aaa5c8@epq.ime.eb.br> From: "Marcio" To: References: <3E9D4BDE00003C3F@www.zipmail.com.br> Subject: [obm-l] =?iso-8859-1?Q?Re:_=5Bobm-l=5D_Demonstra=E7=F5es?= Date: Sat, 19 Apr 2003 19:32:17 -0300 MIME-Version: 1.0 Content-Type: text/plain; charset="iso-8859-1" Content-Transfer-Encoding: 8bit X-Priority: 3 X-MSMail-Priority: Normal X-Mailer: Microsoft Outlook Express 5.50.4133.2400 x-mimeole: Produced By Microsoft MimeOLE V5.50.4133.2400 Sender: owner-obm-l@sucuri.mat.puc-rio.br Precedence: bulk Reply-To: obm-l@mat.puc-rio.br O objetivo da questao era mostrar que sqrt(3) + sqrt(5) era irracional, e isso foi feito. O que eu fiz foi (a menos de uma pequena confusao.. onde escrevo X seja irracional na verdade quis dizer X racional, e depois X^2 - 8 racional.. ver '*') uma maneira correta de mostrar isso. Se a questao fosse: Sabendo que 2 eh positivo, mostre que sqrt(7) eh irracional. Voce deixaria de fazer por nao conseguir ligar diretamente o dado com o pedido?? Voce simplesmente agradece a informacao de que 2 eh positivo e a usa da melhor maneira possivel (no caso, descartando-a).. Na verdade, meu objetivo com a mensagem era apenas confirmar o comentario do Marcus sobre o erro na sua conclusao "a = Inteiro (contradicao)".. Afinal, a soma de dois irracionais nao eh necessariamente irracional (como mostra o contra-exemplo que mandaram).. Marcio > > > Voce esta certissimo.. De fato, nao acho que o fato de sqrt(3) e sqrt(5) > >serem irracionais ajude muito na prova.. > > Uma maneira de provar seria a seguinte: Suponha que X = sqrt(3) + sqrt(5) > >seja *irracional*. Entao, X^2 - 8 = sqrt(15) tambem seria *irracional*. > >Mas se p/q =sqrt(15), entao 15p^2 = q^2 e chegamos a uma contradicao, pois > >ao colocar ambos os lados na forma 'produto de fatores primos' (que é única), > >vemos que o expoente de 3 é ímpar no lado esquerdo, e par no lado direito... > >contradicao.. > > b(sqrt3) = Irracional > > + > > b(sqrt5) = Irracional > > --------------------------- > > a = Inteiro ( => <= ) Contradição. > > > > Para mim isto não prova nada, pois (sqrt2 + 2) - (sqrt2 - 2) = 4, onde > >temos uma soma de irracionais que resulta em inteiro. > >> > O que você fez está certo , mais não tem nada haver com a questão . > > Abraços. > > Luiz Barbosa > ========================================================================= Instruções para entrar na lista, sair da lista e usar a lista em http://www.mat.puc-rio.br/~nicolau/olimp/obm-l.html O administrador desta lista é ========================================================================= From owner-obm-l@sucuri.mat.puc-rio.br Sat Apr 19 19:47:59 2003 Return-Path: Received: (from majordom@localhost) by sucuri.mat.puc-rio.br (8.9.3/8.9.3) id TAA15717 for obm-l-MTTP; Sat, 19 Apr 2003 19:45:24 -0300 Received: from paiol.terra.com.br (paiol.terra.com.br [200.176.3.18]) by sucuri.mat.puc-rio.br (8.9.3/8.9.3) with ESMTP id TAA15711 for ; Sat, 19 Apr 2003 19:45:18 -0300 Received: from botucatu.terra.com.br (botucatu.terra.com.br [200.176.3.78]) by paiol.terra.com.br (Postfix) with ESMTP id 9303287D32 for ; Sat, 19 Apr 2003 19:44:48 -0300 (BRT) Received: from terra.com.br (200-158-174-130.dsl.telesp.net.br [200.158.174.130]) (authenticated user matematika) by botucatu.terra.com.br (Postfix) with ESMTP id 558DC29C096 for ; Sat, 19 Apr 2003 19:44:48 -0300 (BRT) Message-ID: <3EA1D12A.8070101@terra.com.br> Date: Sat, 19 Apr 2003 19:43:54 -0300 From: Eduardo Botelho User-Agent: Mozilla/5.0 (Windows; U; Windows NT 5.0; en-US; rv:1.0.2) Gecko/20030208 Netscape/7.02 X-Accept-Language: en-us, en MIME-Version: 1.0 To: obm-l@mat.puc-rio.br Subject: [obm-l] =?ISO-8859-1?Q?D=FAvida_sobre_grupos_diedrais?= References: <001701c306a6$a04f1160$f4befea9@xx> <3EA1A4D0.4050205@centroin.com.br> <200304191758510720.01B2F31B@smtp.watersportsbrazil.com> Content-Type: text/plain; charset=ISO-8859-1; format=flowed Content-Transfer-Encoding: 8bit Sender: owner-obm-l@sucuri.mat.puc-rio.br Precedence: bulk Reply-To: obm-l@mat.puc-rio.br Olá pessoal da lista. Estive lendo o Higino Domingues para meu curso de Álgebra e não consigo acreditar no que diz o livro. Vejamos: ele me diz que um grupo diedral, como o proprio nome já diz, é um grupo. Mas vale a associativa? Se ´a´ é tomado para especificar rotação e ' b' para refletir em torno de um eixo, 'e' é tal que b^2 = e, e D(2n) = {e, a,a^2,...,a^(n-1),b,...,ba^(n-1)} é o conjunto em questão, vale (a^2 a)b = a^2(ab), por exemplo ? Veja só a contradição(pelo menos pra mim) : Ele mostra que ab = ba^(n-1). Jóia. Aí dá o exemplo D6: (para ficar claro, vou vou associar cada vértice do triângulo a um ângulo imaginário a partir de seu centro): e: 1(0),2(120),3(240) a: 1(120),2(240), 3(0) a^2: 1(240),2(0),3(120) até aqui, jóia. prosseguindo... b: 1(0),2(240),3(120) b^2: 1(240),2(120),3(0) b^3: 1(120),2(0),3(240) ..ou seja, as rotações são horárias. Nesse caso, não valeria ab = ba^(n-1) Qualquer ajuda agradeço Abraço Eduardo ========================================================================= Instruções para entrar na lista, sair da lista e usar a lista em http://www.mat.puc-rio.br/~nicolau/olimp/obm-l.html O administrador desta lista é ========================================================================= From owner-obm-l@sucuri.mat.puc-rio.br Sat Apr 19 19:47:59 2003 Return-Path: Received: (from majordom@localhost) by sucuri.mat.puc-rio.br (8.9.3/8.9.3) id TAA15723 for obm-l-MTTP; Sat, 19 Apr 2003 19:45:25 -0300 Received: from imo-d08.mx.aol.com (imo-d08.mx.aol.com [205.188.157.40]) by sucuri.mat.puc-rio.br (8.9.3/8.9.3) with ESMTP id TAA15713 for ; Sat, 19 Apr 2003 19:45:20 -0300 From: Faelccmm@aol.com Received: from Faelccmm@aol.com by imo-d08.mx.aol.com (mail_out_v34.22.) id z.bc.36dc4191 (4116) for ; Sat, 19 Apr 2003 18:44:43 -0400 (EDT) Message-ID: Date: Sat, 19 Apr 2003 18:44:43 EDT Subject: [obm-l] mdc (ponto final) To: obm-l@mat.puc-rio.br MIME-Version: 1.0 Content-Type: multipart/alternative; boundary="part1_bc.36dc4191.2bd32b5b_boundary" X-Mailer: 6.0 sub 10516 Sender: owner-obm-l@sucuri.mat.puc-rio.br Precedence: bulk Reply-To: obm-l@mat.puc-rio.br --part1_bc.36dc4191.2bd32b5b_boundary Content-Type: text/plain; charset="ISO-8859-1" Content-Transfer-Encoding: quoted-printable Ol=E1 pessoal, N=E3o passou pela minha cabe=E7a que uma quest=E3o de ensino FUNDAMENTAL iri= a=20 trazer tanta confus=E3o. Por isso que gosto da lista, se tivesse falado que=20= era=20 de ensino fundamental as resposta j=E1 iriam come=E7ar com um : ... essa=20= =E9 f=E1cil=20 ! ...Ou sen=E3o: ... Para que enviar quest=E3o desse N=CDVEL (:-) pra lista= ... ?=20 Com exce=E7=E3o de poucos membros como o sr. e o Cl=E1udio que compreendem q= ue=20 Matem=E1tica =E9 um verdadeiro puzzle e n=E3o uma Ci=EAncia r=EDgida e infle= x=EDvel. --part1_bc.36dc4191.2bd32b5b_boundary Content-Type: text/html; charset="ISO-8859-1" Content-Transfer-Encoding: quoted-printable Ol=E1 pessoal,

N=E3o passou pela minha cabe=E7a que uma quest=E3o de ensino FUNDAMENTAL= iria trazer tanta confus=E3o. Por isso que gosto da lista, se tivesse falad= o que era de ensino fundamental as resposta j=E1 iriam come=E7ar com um : &n= bsp;... essa =E9 f=E1cil ! ...Ou sen=E3o: ... Para que enviar quest=E3o dess= e N=CDVEL  (:-) pra lista ... ? Com exce=E7=E3o de poucos membros como=20= o sr. e o Cl=E1udio que compreendem que Matem=E1tica =E9 um verdadeiro puzzl= e e n=E3o uma Ci=EAncia r=EDgida e inflex=EDvel.
--part1_bc.36dc4191.2bd32b5b_boundary-- ========================================================================= Instruções para entrar na lista, sair da lista e usar a lista em http://www.mat.puc-rio.br/~nicolau/olimp/obm-l.html O administrador desta lista é ========================================================================= From owner-obm-l@sucuri.mat.puc-rio.br Sat Apr 19 19:51:02 2003 Return-Path: Received: (from majordom@localhost) by sucuri.mat.puc-rio.br (8.9.3/8.9.3) id TAA15878 for obm-l-MTTP; Sat, 19 Apr 2003 19:48:26 -0300 Received: from sidney1.bol.com.br (sidney1.bol.com.br [200.221.24.206]) by sucuri.mat.puc-rio.br (8.9.3/8.9.3) with ESMTP id TAA15874 for ; Sat, 19 Apr 2003 19:48:18 -0300 Received: from xx (200.221.24.191) by sidney1.bol.com.br (5.1.071) id 3E766A3D00785670 for obm-l@mat.puc-rio.br; Sat, 19 Apr 2003 19:47:48 -0300 Message-ID: <000d01c306c5$b8eb53e0$f4befea9@xx> From: "Blue Ice" To: References: <001701c306a6$a04f1160$f4befea9@xx> <3EA1A4D0.4050205@centroin.com.br> <200304191758510720.01B2F31B@smtp.watersportsbrazil.com> <3EA1C7A5.4050409@centroin.com.br> Subject: Re: [obm-l] MDC Date: Sat, 19 Apr 2003 19:47:51 -0300 MIME-Version: 1.0 Content-Type: multipart/alternative; boundary="----=_NextPart_000_000A_01C306AC.903EC440" X-Priority: 3 X-MSMail-Priority: Normal X-Mailer: Microsoft Outlook Express 6.00.2600.0000 X-MimeOLE: Produced By Microsoft MimeOLE V6.00.2600.0000 X-Sender-IP: 200.148.36.227 Sender: owner-obm-l@sucuri.mat.puc-rio.br Precedence: bulk Reply-To: obm-l@mat.puc-rio.br This is a multi-part message in MIME format. ------=_NextPart_000_000A_01C306AC.903EC440 Content-Type: text/plain; charset="iso-8859-1" Content-Transfer-Encoding: quoted-printable ----- Original Message -----=20 From: A. C. Morgado=20 To: obm-l@mat.puc-rio.br=20 Sent: Saturday, April 19, 2003 7:03 PM Subject: Re: [obm-l] MDC Eh isso aih, garoto! Na realidade eu mandei essa provoca=E7ao porque hoje em dia a regra em = vestibulares e concursos eh a presen=E7a de questoes com enunciados = dubios e o que mais me espanta eh a passividade de alunos e = principalmente de professores diante de tais enunciados. Nao tenho a = menor duvida de que o gabarito oficial de tal questao seja 15. Tambem = nao tenho duvida que isso foi questao de prova e o gabarito 15 foi = aceito sem grandes contesta=E7oes. Eh preciso protestar contra essas = coisas. []s Morgado Ariel de Silvio wrote Concordo tb colega, mas pq acham o resultado 15 incorreto?? Concordo q = t=EAm quest=F5es as quais podem ter diversas interpreta=E7=F5es, e = resultados, mas com qual vcs ficariam:com o 15 ou 30, e porqu=EA. []=B4s Ice ICQ:177782914 Email:Vinicius84@hotmail.com Win XP Home Full/Speedy 256kbps=20 ------=_NextPart_000_000A_01C306AC.903EC440 Content-Type: text/html; charset="iso-8859-1" Content-Transfer-Encoding: quoted-printable
 
----- Original Message -----
From:=20 A. C.=20 Morgado
Sent: Saturday, April 19, 2003 = 7:03=20 PM
Subject: Re: [obm-l] MDC

Eh isso aih, garoto!
Na realidade eu mandei essa provoca=E7ao = porque=20 hoje em dia a regra em vestibulares e concursos eh a presen=E7a de = questoes com=20 enunciados dubios e o que mais me espanta eh a passividade de alunos e = principalmente de professores diante de tais enunciados. Nao tenho a = menor=20 duvida de que o gabarito oficial de tal questao seja 15. Tambem nao = tenho=20 duvida que isso foi questao de prova e o gabarito 15 foi aceito sem = grandes=20 contesta=E7oes. Eh preciso protestar contra essas=20 coisas.
[]s
Morgado

Ariel de Silvio wrote
 
 
Concordo tb colega, mas pq acham o = resultado 15=20 incorreto?? Concordo q t=EAm quest=F5es as quais podem ter diversas=20 interpreta=E7=F5es, e resultados, mas com qual vcs ficariam:com o 15 = ou 30, e=20 porqu=EA.
 
 
[]=B4s
 
Ice
 
ICQ:177782914 = Email:Vinicius84@hotmail.com
Win=20 XP Home Full/Speedy 256kbps
------=_NextPart_000_000A_01C306AC.903EC440-- ========================================================================= Instruções para entrar na lista, sair da lista e usar a lista em http://www.mat.puc-rio.br/~nicolau/olimp/obm-l.html O administrador desta lista é ========================================================================= From owner-obm-l@sucuri.mat.puc-rio.br Sat Apr 19 20:25:52 2003 Return-Path: Received: (from majordom@localhost) by sucuri.mat.puc-rio.br (8.9.3/8.9.3) id UAA17968 for obm-l-MTTP; Sat, 19 Apr 2003 20:23:01 -0300 Received: from krypton.hosting4u.net (krypton.hosting4u.net [209.15.2.78]) by sucuri.mat.puc-rio.br (8.9.3/8.9.3) with ESMTP id UAA17962 for ; Sat, 19 Apr 2003 20:22:57 -0300 Received: from gargamel (200-158-200-2.dsl.telesp.net.br [200.158.200.2]) by krypton.hosting4u.net (Postfix) with ESMTP id AE0DC9F71F for ; Sat, 19 Apr 2003 18:22:23 -0500 (CDT) Message-ID: <200304192025370600.0239563D@smtp.watersportsbrazil.com> In-Reply-To: <3EA1C7A5.4050409@centroin.com.br> References: <001701c306a6$a04f1160$f4befea9@xx> <3EA1A4D0.4050205@centroin.com.br> <200304191758510720.01B2F31B@smtp.watersportsbrazil.com> <3EA1C7A5.4050409@centroin.com.br> X-Mailer: Calypso Version 3.30.00.00 (3) Date: Sat, 19 Apr 2003 20:25:37 -0300 From: "Ariel de Silvio" To: obm-l@mat.puc-rio.br Subject: Re: [obm-l] MDC Mime-Version: 1.0 Content-Type: multipart/alternative; boundary="=====_105079473729805=_" Sender: owner-obm-l@sucuri.mat.puc-rio.br Precedence: bulk Reply-To: obm-l@mat.puc-rio.br --=====_105079473729805=_ Content-Type: text/plain; charset="us-ascii" Content-Transfer-Encoding: quoted-printable Enquanto eu fazia a segunda resolu=E7=E3o pensei exatamente o que vc falou= agora, qual seria a resposta do gabarito numa prova... Ser=E1 que= considerariam tal detalhe, que realmente o enunciado leva a duas= interpreta=E7=F5es... quer dizer, uma se vc analizar ele a pe da letra....= com certeza a resposta numa prova seria 15.... eh so pensar q 1 dia s=E3o 24h, mas 24h n=E3o s=E3o NECESSARIAMENTE somente= um dia, pode comecar em um e terminar no outro.... se analizarmos melhor= um dia seria 23h59m59s.... mas ai ja eh demais ne.... muito interessante sua "provocacao" hehehe falou Ariel *********** MENSAGEM ORIGINAL *********** As 19:03 de 19/4/2003 A. C. Morgado escreveu: Eh isso aih, garoto! Na realidade eu mandei essa provoca=E7ao porque hoje em dia a regra em= vestibulares e concursos eh a presen=E7a de questoes com enunciados dubios= e o que mais me espanta eh a passividade de alunos e principalmente de= professores diante de tais enunciados. Nao tenho a menor duvida de que o= gabarito oficial de tal questao seja 15. Tambem nao tenho duvida que isso= foi questao de prova e o gabarito 15 foi aceito sem grandes= contesta=E7oes. Eh preciso protestar contra essas coisas. []s Morgado Ariel de Silvio wrote: Logo apos enviar o outro email, analizei com outros olhos, e o que vc diz= tem sentido... mesmo sem saber sua justificativa.... Se vc considerar o inicio das 24 horas, meia noite... como ele diz q eh um= dia, consideremos o dia oficial, eh esse o caminho? Sendo das 0h as 24h... O turno da Torre Leste inicia-se =E0s 22h, portanto at=E9 as 24 hrs se= passaram 2h apenas... isso significa q todos os turnos devem durar apenas= 2h para serem iguais e ninguem virar o dia de guarda... Ent=E3o soma-se as 60h de guarda, q deve dividida por turnos de 2h,= resultando em 30 escoteiros... O que salva essa resolu=E7=E3o =E9 o finalzinho do enunciado "n=E3o d=EA= duas guardas NO MESMO DIA"... Agora ficou certo?? []s Ariel *********** MENSAGEM ORIGINAL *********** As 16:34 de 19/4/2003 A. C. Morgado escreveu: Soh uma provoca=E7aozinha para agu=E7ar o espirito critico dos= companheiros. Eu acho (acho eh bondade minha; eu tenho certeza!) que a= resposta do item b eh 30. Morgado Blue Ice wrote: ----- Original Message ----- From: Faelccmm@aol.com To: obm-l@mat.puc-rio.br Sent: Saturday, April 19, 2003 3:07 PM Subject: [obm-l] MDC E-mail Premium BOL Antiv=EDrus, anti-spam e at=E9 100 MB de espa=E7o. Assine j=E1! http://email.bol.com.br Ol=E1 pessoal, Vejam a quest=E3o: Um grupo de escoteiros deve montar guarda nos 4 postos Norte, Sul, Leste e= Oeste de seu acampamento. No posto Norte a guarda deve ser dia e noite; no= posto Sul a guarda vai das 6 da manh=E3 =E0s 18 da tarde; no posto Leste,= das 22 da noite =E0s 6 da manh=E3 e no posto Oeste, das 14 da tarde =E0s 6= da manh=E3. Qual deve ser o turno m=E1ximo de guarda, de modo que todos= permane=E7am igual tempo de sentinela (sem mudar de lugar) ? Quantos= escoteiros s=E3o necess=E1rios para 24 horas de guarda, de modo que para= escoteiro n=E3o d=EA duas guardas no mesmo dia ? Obs: A primeira pergunta eu resolvi tirando o m.d.c das amplitudes= hor=E1rias de cada posto de guarda e cheguei =E0 resposta do livro R: 4= horas. Mas n=E3o estou conseguindo resolver a outra pergunta. resp: a) 4 horas b) 15 escoteiros(*) Solu=E7=E3o:somei todos(norte,sul,leste,oeste) e dividi por 4: N24+S12+L8+016/4: 60/4:15 --=====_105079473729805=_ Content-Type: text/html; charset="us-ascii"
Enquanto eu fazia a segunda resolução pensei exatamente o que vc falou agora, qual seria a resposta do gabarito numa prova... Será que considerariam tal detalhe, que realmente o enunciado leva a duas interpretações... quer dizer, uma se vc analizar ele a pe da letra....
com certeza a resposta numa prova seria 15....
 
eh so pensar q 1 dia são 24h, mas 24h não são NECESSARIAMENTE somente um dia, pode comecar em um e terminar no outro.... se analizarmos melhor um dia seria 23h59m59s.... mas ai ja eh demais ne....
 
muito interessante sua "provocacao" hehehe
 
falou
Ariel
 

*********** MENSAGEM ORIGINAL ***********

As 19:03 de 19/4/2003 A. C. Morgado escreveu:
Eh isso aih, garoto!
Na realidade eu mandei essa provocaçao porque hoje em dia a regra em vestibulares e concursos eh a presença de questoes com enunciados dubios e o que mais me espanta eh a passividade de alunos e principalmente de professores diante de tais enunciados. Nao tenho a menor duvida de que o gabarito oficial de tal questao seja 15. Tambem nao tenho duvida que isso foi questao de prova e o gabarito 15 foi aceito sem grandes contestaçoes. Eh preciso protestar contra essas coisas.
[]s
Morgado

Ariel de Silvio wrote:
Logo apos enviar o outro email, analizei com outros olhos, e o que vc diz tem sentido... mesmo sem saber sua justificativa....
 
Se vc considerar o inicio das 24 horas, meia noite... como ele diz q eh um dia, consideremos o dia oficial, eh esse o caminho?
Sendo das 0h as 24h...
O turno da Torre Leste inicia-se às 22h, portanto até as 24 hrs se passaram 2h apenas... isso significa q todos os turnos devem durar apenas 2h para serem iguais e ninguem virar o dia de guarda...
 
Então soma-se as 60h de guarda, q deve dividida por turnos de 2h, resultando em 30 escoteiros...
 
O que salva essa resolução é o finalzinho do enunciado "não dê duas guardas NO MESMO DIA"...
 
Agora ficou certo??
 
[]s
Ariel
 
*********** MENSAGEM ORIGINAL ***********

As 16:34 de 19/4/2003 A. C. Morgado escreveu:
Soh uma provocaçaozinha para aguçar o espirito critico dos companheiros. Eu acho (acho eh bondade minha; eu tenho certeza!) que a resposta do item b eh 30.
Morgado

Blue Ice wrote:
 
----- Original Message -----
Sent: Saturday, April 19, 2003 3:07 PM
Subject: [obm-l] MDC


E-mail Premium BOL
Antivírus, anti-spam e até 100 MB de espaço. Assine já!
http://email.bol.com.br

Olá pessoal,

Vejam a questão:

Um grupo de escoteiros deve montar guarda nos 4 postos Norte, Sul, Leste e Oeste de seu acampamento. No posto Norte a guarda deve ser dia e noite; no posto Sul a guarda vai das 6 da manhã às 18 da tarde; no posto Leste, das 22 da noite às 6 da manhã e no posto Oeste, das 14 da tarde às 6 da manhã. Qual deve ser o turno máximo de guarda, de modo que todos permaneçam igual tempo de sentinela (sem mudar de lugar) ? Quantos escoteiros são necessários para 24 horas de guarda, de modo que para escoteiro não dê duas guardas no mesmo dia ?

Obs: A primeira pergunta eu resolvi  tirando o  m.d.c das amplitudes horárias de cada posto de guarda e cheguei à resposta do livro R: 4 horas. Mas não estou conseguindo resolver a outra pergunta.

resp:

a) 4 horas
b) 15 escoteiros(*)
 
 
 
 
Solução:somei todos(norte,sul,leste,oeste) e dividi  por 4:
 
N24+S12+L8+016/4:
60/4:15
 
 
 
 
 


--=====_105079473729805=_-- ========================================================================= Instruções para entrar na lista, sair da lista e usar a lista em http://www.mat.puc-rio.br/~nicolau/olimp/obm-l.html O administrador desta lista é ========================================================================= From owner-obm-l@sucuri.mat.puc-rio.br Sat Apr 19 20:30:31 2003 Return-Path: Received: (from majordom@localhost) by sucuri.mat.puc-rio.br (8.9.3/8.9.3) id UAA18123 for obm-l-MTTP; Sat, 19 Apr 2003 20:27:58 -0300 Received: from artemis.opendf.com.br (artemis.opengate.com.br [200.181.71.14]) by sucuri.mat.puc-rio.br (8.9.3/8.9.3) with ESMTP id UAA18118 for ; Sat, 19 Apr 2003 20:27:55 -0300 Received: from localhost (localhost [127.0.0.1]) by artemis.opendf.com.br (Postfix) with ESMTP id AFA742BECC for ; Sat, 19 Apr 2003 20:28:36 -0300 (BRT) Received: from artemis.opendf.com.br ([127.0.0.1]) by localhost (artemis.opengate.com.br [127.0.0.1:10024]) (amavisd-new) with ESMTP id 24938-03 for ; Sat, 19 Apr 2003 20:28:35 -0300 (BRT) Received: from computer (200-181-090-079.bsace7001.dsl.brasiltelecom.net.br [200.181.90.79]) by artemis.opendf.com.br (Postfix) with ESMTP id 3BF7B2BEC2 for ; Sat, 19 Apr 2003 20:28:35 -0300 (BRT) From: "Artur Costa Steiner" To: Subject: RE: [obm-l] Problema proposto 74 eureka 15... Date: Sat, 19 Apr 2003 20:27:25 -0300 Organization: Steiner Consultoria LTDA Message-ID: <003601c306cb$3da88d50$9865fea9@computer> MIME-Version: 1.0 Content-Type: text/plain; charset="iso-8859-1" X-Priority: 3 (Normal) X-MSMail-Priority: Normal X-Mailer: Microsoft Outlook, Build 10.0.2627 In-Reply-To: X-MimeOLE: Produced By Microsoft MimeOLE V6.00.2800.1106 Importance: Normal X-Virus-Scanned: by amavisd-new Content-Transfer-Encoding: 8bit X-MIME-Autoconverted: from quoted-printable to 8bit by sucuri.mat.puc-rio.br id UAA18119 Sender: owner-obm-l@sucuri.mat.puc-rio.br Precedence: bulk Reply-To: obm-l@mat.puc-rio.br Este problema jah circulou na lista. O Claudio chegou a uma soluçao interessante assumindo diferenciabilidae de f. As sua solucoes, que naum sao as unicas, sao um caso particular da funcao que o Claudio achou. Uma outra hipotese trivial e a funcao identicamente nula em R (acho que tambem estah englobada na solucao do Claudio) Eu acho que sem assumir alguma condicao como diferenciabilidade fica muito dificil garantir que se encontraram todas as funcoes. Artur >-----Original Message----- >From: owner-obm-l@sucuri.mat.puc-rio.br [mailto:owner-obm-l@sucuri.mat.puc- >rio.br] On Behalf Of rmr-olimp >Sent: Saturday, April 19, 2003 7:21 PM >To: obm-l@mat.puc-rio.br >Subject: [obm-l] Problema proposto 74 eureka 15... > >Problema proposto 74 eureka 15... > >Ache todas as funçoes f: R -> R tais que: >f(x+y)+f(x-y)=2.f(x).cos(y) > >fazendo: >x+y=a >x-y=b > >Substituindo: >f(a)+f(b)=2.f((a+b)/2).cos((a-b)/2) > >Da trigonometria: >sen(a)+sen(b)=2.sen((a+b)/2).cos((a-b)/2) >cos(a)+sen(b)=2.cos((a+b)/2).cos((a-b)/2) > >Logo f(x)=sen(x) ou f(x)=cos(x) > >O que eu não sei provar é se essas são as únicas soluções >e caso existam encontrá-las.. > >Se alguém puder quebrar um galho e me ajudar seria ótimo!! > >Rodrigo > > > >_______________________________________________________________________ ___ >Seleção de Softwares UOL. >10 softwares escolhidos pelo UOL para você e sua família. >http://www.uol.com.br/selecao > > >======================================================================= == >Instruções para entrar na lista, sair da lista e usar a lista em >http://www.mat.puc-rio.br/~nicolau/olimp/obm-l.html >O administrador desta lista é >======================================================================= == ========================================================================= Instruções para entrar na lista, sair da lista e usar a lista em http://www.mat.puc-rio.br/~nicolau/olimp/obm-l.html O administrador desta lista é ========================================================================= From owner-obm-l@sucuri.mat.puc-rio.br Sat Apr 19 20:45:02 2003 Return-Path: Received: (from majordom@localhost) by sucuri.mat.puc-rio.br (8.9.3/8.9.3) id UAA18687 for obm-l-MTTP; Sat, 19 Apr 2003 20:42:24 -0300 Received: from trex-b.centroin.com.br (trex-b.centroin.com.br [200.225.63.136]) by sucuri.mat.puc-rio.br (8.9.3/8.9.3) with ESMTP id UAA18683 for ; Sat, 19 Apr 2003 20:42:21 -0300 Received: from centroin.com.br (RJ090116.user.veloxzone.com.br [200.141.90.116] (may be forged)) (authenticated bits=0) by trex-b.centroin.com.br (8.12.9/8.12.9) with ESMTP id h3JNfgVi005554 for ; Sat, 19 Apr 2003 20:41:46 -0300 (EST) Message-ID: <3EA1DEEF.8080803@centroin.com.br> Date: Sat, 19 Apr 2003 20:42:39 -0300 From: "A. C. Morgado" User-Agent: Mozilla/5.0 (Windows; U; Windows NT 5.0; en-US; rv:1.0.2) Gecko/20030208 Netscape/7.02 X-Accept-Language: en-us, en MIME-Version: 1.0 To: obm-l@mat.puc-rio.br Subject: Re: [obm-l] MDC References: <001701c306a6$a04f1160$f4befea9@xx> <3EA1A4D0.4050205@centroin.com.br> <200304191758510720.01B2F31B@smtp.watersportsbrazil.com> <3EA1C7A5.4050409@centroin.com.br> <200304192025370600.0239563D@smtp.watersportsbrazil.com> Content-Type: multipart/alternative; boundary="------------010808040406060400050000" Sender: owner-obm-l@sucuri.mat.puc-rio.br Precedence: bulk Reply-To: obm-l@mat.puc-rio.br --------------010808040406060400050000 Content-Type: text/plain; charset=ISO-8859-1; format=flowed Content-Transfer-Encoding: 8bit Eh quase isso ai. Na verdade o problema nao tem duas interpretaçoes. A resposta eh 30. Mas que eh pegadinha, eh. Ariel de Silvio wrote: > Enquanto eu fazia a segunda resolução pensei exatamente o que vc falou > agora, qual seria a resposta do gabarito numa prova... Será que > considerariam tal detalhe, que realmente o enunciado leva a duas > interpretações... quer dizer, uma se vc analizar ele a pe da letra.... > com certeza a resposta numa prova seria 15.... > > eh so pensar q 1 dia são 24h, mas 24h não são NECESSARIAMENTE somente > um dia, pode comecar em um e terminar no outro.... se analizarmos > melhor um dia seria 23h59m59s.... mas ai ja eh demais ne.... > > muito interessante sua "provocacao" hehehe > > falou > Ariel > > > *********** MENSAGEM ORIGINAL *********** > > As 19:03 de 19/4/2003 A. C. Morgado escreveu: > > Eh isso aih, garoto! > Na realidade eu mandei essa provocaçao porque hoje em dia a regra > em vestibulares e concursos eh a presença de questoes com > enunciados dubios e o que mais me espanta eh a passividade de > alunos e principalmente de professores diante de tais enunciados. > Nao tenho a menor duvida de que o gabarito oficial de tal questao > seja 15. Tambem nao tenho duvida que isso foi questao de prova e o > gabarito 15 foi aceito sem grandes contestaçoes. Eh preciso > protestar contra essas coisas. > []s > Morgado > > Ariel de Silvio wrote: > >> Logo apos enviar o outro email, analizei com outros olhos, e o >> que vc diz tem sentido... mesmo sem saber sua justificativa.... >> >> Se vc considerar o inicio das 24 horas, meia noite... como ele >> diz q eh um dia, consideremos o dia oficial, eh esse o caminho? >> Sendo das 0h as 24h... >> O turno da Torre Leste inicia-se às 22h, portanto até as 24 hrs >> se passaram 2h apenas... isso significa q todos os turnos devem >> durar apenas 2h para serem iguais e ninguem virar o dia de guarda... >> >> Então soma-se as 60h de guarda, q deve dividida por turnos de 2h, >> resultando em 30 escoteiros... >> >> O que salva essa resolução é o finalzinho do enunciado "não dê >> duas guardas NO MESMO DIA"... >> >> Agora ficou certo?? >> >> []s >> Ariel >> >> *********** MENSAGEM ORIGINAL *********** >> >> As 16:34 de 19/4/2003 A. C. Morgado escreveu: >> >> Soh uma provocaçaozinha para aguçar o espirito critico dos >> companheiros. Eu acho (acho eh bondade minha; eu tenho >> certeza!) que a resposta do item b eh 30. >> Morgado >> >> Blue Ice wrote: >> >>> >>> >>> ----- Original Message ----- >>> From: Faelccmm@aol.com >>> To: obm-l@mat.puc-rio.br >>> Sent: Saturday, April 19, 2003 3:07 PM >>> Subject: [obm-l] MDC >>> >>> ------------------------------------------------------------------------ >>> E-mail Premium BOL >>> Antivírus, anti-spam e até 100 MB de espaço. Assine já! >>> http://email.bol.com.br >>> ------------------------------------------------------------------------ >>> Olá pessoal, >>> >>> Vejam a questão: >>> >>> Um grupo de escoteiros deve montar guarda nos 4 postos >>> Norte, Sul, Leste e Oeste de seu acampamento. No posto >>> Norte a guarda deve ser dia e noite; no posto Sul a >>> guarda vai das 6 da manhã às 18 da tarde; no posto >>> Leste, das 22 da noite às 6 da manhã e no posto Oeste, >>> das 14 da tarde às 6 da manhã. Qual deve ser o turno >>> máximo de guarda, de modo que todos permaneçam igual >>> tempo de sentinela (sem mudar de lugar) ? Quantos >>> escoteiros são necessários para 24 horas de guarda, de >>> modo que para escoteiro não dê duas guardas no mesmo dia ? >>> >>> Obs: A primeira pergunta eu resolvi tirando o m.d.c >>> das amplitudes horárias de cada posto de guarda e >>> cheguei à resposta do livro R: 4 horas. Mas não estou >>> conseguindo resolver a outra pergunta. >>> >>> resp: >>> >>> a) 4 horas >>> b) 15 escoteiros(*) >>> >>> >>> >>> Solução:somei todos(norte,sul,leste,oeste) e dividi por 4: >>> >>> N24+S12+L8+016/4: >>> 60/4:15 >>> >>> >>> >>> >>> >>> >> > --------------010808040406060400050000 Content-Type: text/html; charset=us-ascii Content-Transfer-Encoding: 7bit Eh quase isso ai. Na verdade o problema nao tem duas interpretaçoes. A resposta eh 30. Mas que eh pegadinha, eh.

Ariel de Silvio wrote:
Enquanto eu fazia a segunda resolução pensei exatamente o que vc falou agora, qual seria a resposta do gabarito numa prova... Será que considerariam tal detalhe, que realmente o enunciado leva a duas interpretações... quer dizer, uma se vc analizar ele a pe da letra....
com certeza a resposta numa prova seria 15....
 
eh so pensar q 1 dia são 24h, mas 24h não são NECESSARIAMENTE somente um dia, pode comecar em um e terminar no outro.... se analizarmos melhor um dia seria 23h59m59s.... mas ai ja eh demais ne....
 
muito interessante sua "provocacao" hehehe
 
falou
Ariel
 

*********** MENSAGEM ORIGINAL ***********

As 19:03 de 19/4/2003 A. C. Morgado escreveu:
Eh isso aih, garoto!
Na realidade eu mandei essa provocaçao porque hoje em dia a regra em vestibulares e concursos eh a presença de questoes com enunciados dubios e o que mais me espanta eh a passividade de alunos e principalmente de professores diante de tais enunciados. Nao tenho a menor duvida de que o gabarito oficial de tal questao seja 15. Tambem nao tenho duvida que isso foi questao de prova e o gabarito 15 foi aceito sem grandes contestaçoes. Eh preciso protestar contra essas coisas.
[]s
Morgado

Ariel de Silvio wrote:
Logo apos enviar o outro email, analizei com outros olhos, e o que vc diz tem sentido... mesmo sem saber sua justificativa....
 
Se vc considerar o inicio das 24 horas, meia noite... como ele diz q eh um dia, consideremos o dia oficial, eh esse o caminho?
Sendo das 0h as 24h...
O turno da Torre Leste inicia-se às 22h, portanto até as 24 hrs se passaram 2h apenas... isso significa q todos os turnos devem durar apenas 2h para serem iguais e ninguem virar o dia de guarda...
 
Então soma-se as 60h de guarda, q deve dividida por turnos de 2h, resultando em 30 escoteiros...
 
O que salva essa resolução é o finalzinho do enunciado "não dê duas guardas NO MESMO DIA"...
 
Agora ficou certo??
 
[]s
Ariel
 
*********** MENSAGEM ORIGINAL ***********

As 16:34 de 19/4/2003 A. C. Morgado escreveu:
Soh uma provocaçaozinha para aguçar o espirito critico dos companheiros. Eu acho (acho eh bondade minha; eu tenho certeza!) que a resposta do item b eh 30.
Morgado

Blue Ice wrote:
 
----- Original Message -----
Sent: Saturday, April 19, 2003 3:07 PM
Subject: [obm-l] MDC


E-mail Premium BOL
Antivírus, anti-spam e até 100 MB de espaço. Assine já!
http://email.bol.com.br

Olá pessoal,

Vejam a questão:

Um grupo de escoteiros deve montar guarda nos 4 postos Norte, Sul, Leste e Oeste de seu acampamento. No posto Norte a guarda deve ser dia e noite; no posto Sul a guarda vai das 6 da manhã às 18 da tarde; no posto Leste, das 22 da noite às 6 da manhã e no posto Oeste, das 14 da tarde às 6 da manhã. Qual deve ser o turno máximo de guarda, de modo que todos permaneçam igual tempo de sentinela (sem mudar de lugar) ? Quantos escoteiros são necessários para 24 horas de guarda, de modo que para escoteiro não dê duas guardas no mesmo dia ?

Obs: A primeira pergunta eu resolvi  tirando o  m.d.c das amplitudes horárias de cada posto de guarda e cheguei à resposta do livro R: 4 horas. Mas não estou conseguindo resolver a outra pergunta.

resp:

a) 4 horas
b) 15 escoteiros(*)
 
 
 
 
Solução:somei todos(norte,sul,leste,oeste) e dividi  por 4:
 
N24+S12+L8+016/4:
60/4:15
 
 
 
 
 



--------------010808040406060400050000-- ========================================================================= Instruções para entrar na lista, sair da lista e usar a lista em http://www.mat.puc-rio.br/~nicolau/olimp/obm-l.html O administrador desta lista é ========================================================================= From owner-obm-l@sucuri.mat.puc-rio.br Sat Apr 19 20:47:14 2003 Return-Path: Received: (from majordom@localhost) by sucuri.mat.puc-rio.br (8.9.3/8.9.3) id UAA18790 for obm-l-MTTP; Sat, 19 Apr 2003 20:44:41 -0300 Received: from trex-b.centroin.com.br (trex-b.centroin.com.br [200.225.63.136]) by sucuri.mat.puc-rio.br (8.9.3/8.9.3) with ESMTP id UAA18786 for ; Sat, 19 Apr 2003 20:44:38 -0300 Received: from centroin.com.br (RJ090116.user.veloxzone.com.br [200.141.90.116] (may be forged)) (authenticated bits=0) by trex-b.centroin.com.br (8.12.9/8.12.9) with ESMTP id h3JNi9Vi005582 for ; Sat, 19 Apr 2003 20:44:09 -0300 (EST) Message-ID: <3EA1DF81.6030506@centroin.com.br> Date: Sat, 19 Apr 2003 20:45:05 -0300 From: "A. C. Morgado" User-Agent: Mozilla/5.0 (Windows; U; Windows NT 5.0; en-US; rv:1.0.2) Gecko/20030208 Netscape/7.02 X-Accept-Language: en-us, en MIME-Version: 1.0 To: obm-l@mat.puc-rio.br Subject: Re: [obm-l] MDC References: <001701c306a6$a04f1160$f4befea9@xx> <3EA1A4D0.4050205@centroin.com.br> <200304191758510720.01B2F31B@smtp.watersportsbrazil.com> <3EA1C7A5.4050409@centroin.com.br> <000d01c306c5$b8eb53e0$f4befea9@xx> Content-Type: multipart/alternative; boundary="------------070107050709070403000503" Sender: owner-obm-l@sucuri.mat.puc-rio.br Precedence: bulk Reply-To: obm-l@mat.puc-rio.br --------------070107050709070403000503 Content-Type: text/plain; charset=ISO-8859-1; format=flowed Content-Transfer-Encoding: 8bit Um grupo de escoteiros deve montar guarda nos 4 postos Norte, Sul, Leste e Oeste de seu acampamento. No posto Norte a guarda deve ser dia e noite; no posto Sul a guarda vai das 6 da manhã às 18 da tarde; no posto Leste, das 22 da noite às 6 da manhã e no posto Oeste, das 14 da tarde às 6 da manhã. Qual deve ser o turno máximo de guarda, de modo que todos permaneçam igual tempo de sentinela (sem mudar de lugar) ? Quantos escoteiros são necessários para 24 horas de guarda, de modo que para escoteiro não dê duas guardas no mesmo dia ? Vamos fazer uma ultima tentativa de convencer os recalcitrantes de que eh impossivel fazer turnos de 4 horas e usar apenas 15 escoteiros: Escala de guarda dos turnos iniciados no dia 19: Torre Norte: escoteiro 1 (0h/4h); 2 (4h/8h); 3(8h/12h); 4(12h/16h); 5 (16h/20h); 6(20h/24h) Torre Sul: 7 (6h/10h); 8 (10h/14h); 9 (14h/18h). Torre Leste: 10 ( 22h/ 2h do dia 20); 11 (2h/6h) Torre Oeste: 12 (14h/18h); 13 (18h/22h); 14 (22h/2h do dia 20); 15 (2h/6h) Voce pode ate dar duas interpretaçoes: primeira: sao apenas 24 horas de guarda, ou seja, eh so para fazer escala do dia 19; nesse caso os escoteiros 10 e 14 teriam sua guarda dividida em duas partes e mudariam de lugar, da torre para o descanso e do descanso para a torre. segunda: o acampamento dura mais de 24h; nesse caso o 14 eo 10 nao poderiam trabalhar no dia 20 pois um escoteiro nao pode dar duas guardas no mesmo dia. Em qualquer caso serao necessarios mais dois escoteiros e como todos devem trabalhar o mesmo numero de horas serao necessarios mais 15 escoteiros. --------------070107050709070403000503 Content-Type: text/html; charset=us-ascii Content-Transfer-Encoding: 7bit
Um grupo de escoteiros deve montar guarda nos 4 postos Norte, Sul, Leste e Oeste de seu acampamento. No posto Norte a guarda deve ser dia e noite; no posto Sul a guarda vai das 6 da manhã às 18 da tarde; no posto Leste, das 22 da noite às 6 da manhã e no posto Oeste, das 14 da tarde às 6 da manhã. Qual deve ser o turno máximo de guarda, de modo que todos permaneçam igual tempo de sentinela (sem mudar de lugar) ? Quantos escoteiros são necessários para 24 horas de guarda, de modo que para escoteiro não dê duas guardas no mesmo dia ?
Vamos fazer uma ultima tentativa de convencer os recalcitrantes de que eh impossivel fazer turnos de 4 horas e usar apenas 15 escoteiros:
Escala de guarda dos turnos iniciados no dia 19:
Torre Norte: escoteiro 1 (0h/4h); 2 (4h/8h); 3(8h/12h); 4(12h/16h); 5 (16h/20h); 6(20h/24h)
Torre Sul: 7 (6h/10h); 8 (10h/14h); 9 (14h/18h).
Torre Leste: 10 ( 22h/ 2h do dia 20); 11 (2h/6h)
Torre Oeste: 12 (14h/18h); 13 (18h/22h); 14 (22h/2h do dia 20); 15 (2h/6h)
Voce pode ate dar duas interpretaçoes:
primeira: sao apenas 24 horas de guarda, ou seja, eh so para fazer escala do dia 19; nesse caso os escoteiros 10 e 14 teriam sua guarda dividida em duas partes e mudariam de lugar, da torre para o descanso e do descanso para a torre.
segunda: o acampamento dura mais de 24h; nesse caso o 14 eo 10 nao poderiam trabalhar no dia 20 pois um escoteiro nao pode dar duas guardas no mesmo dia.
Em qualquer caso serao necessarios mais dois escoteiros e como todos devem trabalhar o mesmo numero de horas serao necessarios mais 15 escoteiros.



--------------070107050709070403000503-- ========================================================================= Instruções para entrar na lista, sair da lista e usar a lista em http://www.mat.puc-rio.br/~nicolau/olimp/obm-l.html O administrador desta lista é ========================================================================= From owner-obm-l@sucuri.mat.puc-rio.br Sat Apr 19 22:30:26 2003 Return-Path: Received: (from majordom@localhost) by sucuri.mat.puc-rio.br (8.9.3/8.9.3) id WAA21168 for obm-l-MTTP; Sat, 19 Apr 2003 22:28:36 -0300 Received: from ivoti.terra.com.br (ivoti.terra.com.br [200.176.3.20]) by sucuri.mat.puc-rio.br (8.9.3/8.9.3) with ESMTP id WAA21164 for ; Sat, 19 Apr 2003 22:28:33 -0300 Received: from altamira.terra.com.br (altamira.terra.com.br [200.176.3.40]) by ivoti.terra.com.br (Postfix) with ESMTP id AB4EB4081F8 for ; Sat, 19 Apr 2003 22:28:02 -0300 (BRT) Received: from terra.com.br (200-158-174-89.dsl.telesp.net.br [200.158.174.89]) (authenticated user matematika) by altamira.terra.com.br (Postfix) with ESMTP id 668403DC05B for ; Sat, 19 Apr 2003 22:28:02 -0300 (BRT) Message-ID: <3EA1F772.8090600@terra.com.br> Date: Sat, 19 Apr 2003 22:27:14 -0300 From: Eduardo Botelho User-Agent: Mozilla/5.0 (Windows; U; Windows NT 5.0; en-US; rv:1.0.2) Gecko/20030208 Netscape/7.02 X-Accept-Language: en-us, en MIME-Version: 1.0 To: obm-l@mat.puc-rio.br Subject: [obm-l] =?ISO-8859-1?Q?D=FAvida_sobre_grupos_diedrais_II?= References: <001701c306a6$a04f1160$f4befea9@xx> <3EA1A4D0.4050205@centroin.com.br> <200304191758510720.01B2F31B@smtp.watersportsbrazil.com> <3EA1D12A.8070101@terra.com.br> Content-Type: text/plain; charset=ISO-8859-1; format=flowed Content-Transfer-Encoding: 8bit Sender: owner-obm-l@sucuri.mat.puc-rio.br Precedence: bulk Reply-To: obm-l@mat.puc-rio.br aqui o certo é: > b: 1(0),2(240),3(120) > ba^2: 1(240),2(120),3(0) > ba^3: 1(120),2(0),3(240) ..ou seja, as rotações são horárias. Nesse > caso, não valeria ab = ba^(n-1) Isso foi o que quiz dizer... ========================================================================= Instruções para entrar na lista, sair da lista e usar a lista em http://www.mat.puc-rio.br/~nicolau/olimp/obm-l.html O administrador desta lista é ========================================================================= From owner-obm-l@sucuri.mat.puc-rio.br Sun Apr 20 00:42:43 2003 Return-Path: Received: (from majordom@localhost) by sucuri.mat.puc-rio.br (8.9.3/8.9.3) id AAA22735 for obm-l-MTTP; Sun, 20 Apr 2003 00:40:39 -0300 Received: from sang.bol.com.br (sang.bol.com.br [200.221.24.24]) by sucuri.mat.puc-rio.br (8.9.3/8.9.3) with ESMTP id AAA22730 for ; Sun, 20 Apr 2003 00:40:36 -0300 Received: from bol.com.br (200.221.24.134) by sang.bol.com.br (5.1.071) id 3E76704C008F6408 for obm-l@mat.puc-rio.br; Sun, 20 Apr 2003 00:39:55 -0300 Date: Sun, 20 Apr 2003 00:39:55 -0300 Message-Id: Subject: [obm-l] =?iso-8859-1?q?Tang=EAncia?= MIME-Version: 1.0 Content-Type: multipart/mixed; boundary="_=__=_XaM3_Boundary.1050809995.2A.999077.42.26033.52.42.101010.1270519487" From: "cfgauss77" To: "Lista OBM" X-XaM3-API-Version: 2.4 R3 ( B4 ) X-SenderIP: 200.193.251.130 Sender: owner-obm-l@sucuri.mat.puc-rio.br Precedence: bulk Reply-To: obm-l@mat.puc-rio.br --_=__=_XaM3_Boundary.1050809995.2A.999077.42.26033.52.42.101010.1270519487 Content-Type: text/plain;charset="iso-8859-1" Content-Transfer-Encoding: quoted-printable Gostei desta quest=E3o: Dado un tri=E1ngulo ABC y un punto D en el lado BC, se trazan las circunferencias C1 y C2 inscritas respectivamente en los tri=E1ngulos ABD y ADC. Mostrar que las circunferencias C1 y C2 son mutuamente tangentes si y s=F3lo si D es el punto de tangencia de la circunferencia C0, inscrita del tri=E1ngulo ABC, en el lado BC. __________________________________________________________________________ Sele=E7=E3o de Softwares UOL. 10 softwares escolhidos pelo UOL para voc=EA e sua fam=EDlia. http://www.uol.com.br/selecao --_=__=_XaM3_Boundary.1050809995.2A.999077.42.26033.52.42.101010.1270519487 Content-Type: application/octet-stream; name="prb15001.gif" Content-Transfer-Encoding: base64 R0lGODlhQQHVAPcAAAAAAIAAAACAAICAAAAAgIAAgACAgMDAwMDcwKbK8P///wAAhAAA//8A AP////////////////////////////////////////////////////////////////////// //////////////////////////////////////////////////////////////////////// //////////////////////////////////////////////////////////////////////// //////////////////////////////////////////////////////////////////////// //////////////////////////////////////////////////////////////////////// //////////////////////////////////////////////////////////////////////// //////////////////////////////////////////////////////////////////////// //////////////////////////////////////////////////////////////////////// //////////////////////////////////////////////////////////////////////// //////////////////////////////////////////////////////////////////////// //////////////////////////////////////////////////////////////////////// //////////////////////////////////////////////////////////////////////// ///////////////////////////////////////////////////////////////////78KCg pICAgP8AAAD/AP//AAAA//8A/wD//////yH5BAEAAAoALAAAAABBAdUAQAj+ABUIHEiwoMGD CBMqXMiwocOHECNKnEixosWLGDNqxAig48aPIEOKHEmypMmTKFOqXMmypcuXCAkokDmQJsyb OHPq3InRpkCfCQkA5Um0qNGjG4fWvKgUqdOnUG82LTi1Z9WoWLNqbXjVYFeRX7eKHZsz7MGm DNIyIKl27U+zZOPKZQrRrVeGHgcCcGl3rt+/XBH2hQiXYMe9NxkIBcw4rs3BdI22nUy5MtXG mHfKhAyycMS0fBeCzkxaJOeSngue1rla8MzSsB+2Nml2tuSKkBfHJk3A9km0fn0z9J16t1QF q9VeVM7Qp/Coz0Vb1G0c52PkEPMKRGzRLs3oT8H+Sx9ZvPrD3q4tHsaoWPxtkO41lq/uO75p ufYVDtZufb5c9J/lNBpW+Y2XHVH+HeVcdyv19R1RBRpYHXVYTRWhQBeqFtNBGTbUYXrmSZSg fBJ9ONCACUUXFooemsihiyEGNaJCDRDQQEmTvcjcRJYJdKNMN2JIGY+VFZljjGPVGGRD2vGH 5JNQcsSdAlNGqRB/Tlqp5ZZcdunlQ4eF+eWYZG6kZJloprmQTU1RqOabUSrl2Yxw1snTVXTO lKedfIJ1Xkp79iloYCL2N+ihPXXnG5YmpRUoolZexWJQFHFX5UmTQlomUBc+2pJbnmoa1YN+ spTpRpA5KmpmAP72EYz+FD2n2KpZwbpmibXGuhStONnqkKT4LScjryS1ap1Bvqp0oXih1kkq dnhdiutdGDbWYYdu8rqZhhwJu+1fJiZLULOkzSaubMHdp1O2VkZ3rodjwQoZo9tlqRK5mlXL 40ugvqsrW7zhSxuy7GFKLbQwJesvUgJX9NWH+dlWlYviLswYu64WPNKpv8bqFsevWuxleU1K S+zJKJeEpckpy2VvyzDHLPPMNNecskcv26wzS07mvPPPQAetpW4NC200TUNhbPTSDo+7ZtFM E5s0YVFXrVCbTVtddVUjKq11zF0F6vXXq4ZFLtRkxwYX1Gin/decMLXttlbFwdVjikOeJff+ 3EaV91hrYbK83Fpj880qkbsu5DOOIht+r0OzfRW4XpSn1LjjIU0cUXGW8rs35llzS9HnqOoJ ulOckgiYg6fnq+9HpCMHcl1Cgtp6S9+Sh9Lsr+J9O2rnBsp7oy3+rtHCI16uX12x2yxyccpD vu9rxkv4OO25/pt49cay9DBZBXJWuNbd4758sMJe3Tya5bsEXLrpd7w1wsO1pePwlxEUfe8Z xbc+ks860ZWmJDiCUep14CodU/4HG+c8Z3GyGUwA5wKxzDHwbfq6EARTtJT9KfB4GYNU+TRY sPbFCz7m41NyeIK/8IRkf+ODkrlYeMFpoVAvBMRdDUMDL9bERV7+BdngSnZoMOz1yoNGvKFE ChjC0uQuiXwxYVFsBR4mviSGFZogFC33E/pBCEe7wWJRUscgLiYOiV5UIgCz8r74vfAsomsQ 8cZERGAdL0OtaaNpFNan5tXmjQEilO/cuDE0/gdt5ZHXYPxXl9a0UFE1E2OhUPVIHd2PahFE 0Y7uaEgu0akBoNzNkpZUvaeUrJSGO6V5AmdFVG5EiF1amStRQq80sXKWuMylLnfJy146ZT2+ tBrOWhlMmamymEI7JjJ/1jNiLvOZ0PwSKEkZzZ/VSJLV1Bb1sJlNEVJve91sGZvgGM6UjfOA 5Syb05qTTm/mT5DttNPUCEPEeN5pQyL+qqc9j0PO0elzn0O8WqIAOjSBqo6gMQpbZxBqHoWW iqG8kZ/uINoYs/3mnxSF560AhdGM9lOiHPUoGzd3xY5SNDWkMylCUcoeIwlHpfH0W/0qKZhN dlGkxxqdamg6ONvhNIUOm10tX0i4n16vREWN1qdmZdRifSY3Sk1YJ6u5otV0pZbAxNRUkfm9 jU6kc8qCaSk196evVs5UTdUpiMq6xLN+SqyY0yNJKfjNtKJTgNNZ3U3t+k4E5lWvpuPrXtMY ugTWBK5HwyvsdmI/jw2Im93UYkb2ZtP+6c9RiI2kXxdruYrV9KSETUoRlWW9dj5xooDkYWkj Gzw1Jgxd4ZT+4kJByELmURV5GpviZyDbOovNZ6v6wxVvDde41ADXgIEcbtqKa0OoxMdBmR2T 8v64FftI0JXTlZ5YIvbR08FwOD8sY3eJm1MOhle8+OSbbFFC1vOi965fWy971wo+y3IlurxB olKOG0hCPi1qp81kYwu72frmlp6JtU1lRdNCMhr2wHPdmWQ5NjnIRS64gIXwRPC7lQAKNYeN TC9/m+tftYItuMKB5fnyN+Itvneg5jxRdIY6vZu2GLa0zdzJSFVFMbm1vwF2r33Jw+F1ISxD zjRgkA2cY9QU+YpeJCGDlrxd19LmyXJErnr6J18CWZm9WJ4jh1h4Y/B++ajsWyH+Y8u82hID VbptVq2QP1ivrKJ5S/Wp7ZybTCUQvzlKwkEjm7XMPxwiKE6yYs2gFWtlGpcUSeBB4qJDO2TD +JhKPx5imK8mHg/ylF8Aa2tZNu0V9+xvLWWmoqgPfThKr/hTr2mxqiNSYVYDRrI9hHWsnTJr J/rlOrjRNa4TA8YJOQbD2tvdOjvpq0nDuEKEJvHG+urpYsepo/uttLrWWWAzhtqT+pRrsLfN bUYr29p0rKEdNfzicpt72uhOk3JFG+cajxu+yD5zofkUO+re0d4GpS8kxYyovdmNqDhmp5kR R3BasY3h8DlNJQ9evNHCTGC/VaTsCFzxy+7O2f9xav+AwvUxniZywSED+cXEtsdI42/ACDmT flAu40/LzuYzmyZGrimUnvv850APutB9XpmhG/3oPXepYpDO9KY7/elQj7rUp071qjc9lA65 JZJ0LtjsWCrJXYeS1sMeo1oP0NBkX6WfEzL26ihTpCrekix/6ugxzZ3ul7bl19POd0gFBAAA Ow== --_=__=_XaM3_Boundary.1050809995.2A.999077.42.26033.52.42.101010.1270519487-- ========================================================================= Instruções para entrar na lista, sair da lista e usar a lista em http://www.mat.puc-rio.br/~nicolau/olimp/obm-l.html O administrador desta lista é ========================================================================= From owner-obm-l@sucuri.mat.puc-rio.br Sun Apr 20 10:44:48 2003 Return-Path: Received: (from majordom@localhost) by sucuri.mat.puc-rio.br (8.9.3/8.9.3) id KAA30188 for obm-l-MTTP; Sun, 20 Apr 2003 10:41:12 -0300 Received: from shannon.bol.com.br (shannon.bol.com.br [200.221.24.13]) by sucuri.mat.puc-rio.br (8.9.3/8.9.3) with ESMTP id KAA30184 for ; Sun, 20 Apr 2003 10:41:09 -0300 Received: from bol.com.br (200.221.24.135) by shannon.bol.com.br (5.1.071) id 3E9EDE39000BDDDB for obm-l@mat.puc-rio.br; Sun, 20 Apr 2003 10:40:35 -0300 Date: Sun, 20 Apr 2003 10:40:35 -0300 Message-Id: Subject: [obm-l] =?iso-8859-1?q?quest=E3o_sobre_conjuntos?= MIME-Version: 1.0 Content-Type: text/plain;charset="iso-8859-1" From: "renatinha15a" To: "obm" X-XaM3-API-Version: 2.4 R3 ( B4 ) X-SenderIP: 200.241.108.194 Content-Transfer-Encoding: 8bit X-MIME-Autoconverted: from quoted-printable to 8bit by sucuri.mat.puc-rio.br id KAA30185 Sender: owner-obm-l@sucuri.mat.puc-rio.br Precedence: bulk Reply-To: obm-l@mat.puc-rio.br Oi pessoal da lista, não consegui fazer esta questão, se alguém puder me ajudar, ficarei bastante grata. (U.F. VIÇOSA-89) Um conjunto A tem 8 elementos distintos. O número de subconjuntos de A, com 5 elementos distintos cada um, é: P.S: Gostaria, se possível, que a solução não tivesse probabilidade ou combinatória, pois ainda faço 1º ano do ensino médio. []´s Renatinha __________________________________________________________________________ Seleção de Softwares UOL. 10 softwares escolhidos pelo UOL para você e sua família. http://www.uol.com.br/selecao ========================================================================= Instruções para entrar na lista, sair da lista e usar a lista em http://www.mat.puc-rio.br/~nicolau/olimp/obm-l.html O administrador desta lista é ========================================================================= From owner-obm-l@sucuri.mat.puc-rio.br Sun Apr 20 11:58:43 2003 Return-Path: Received: (from majordom@localhost) by sucuri.mat.puc-rio.br (8.9.3/8.9.3) id LAA31181 for obm-l-MTTP; Sun, 20 Apr 2003 11:56:30 -0300 Received: from salem.bol.com.br (salem.bol.com.br [200.221.24.25]) by sucuri.mat.puc-rio.br (8.9.3/8.9.3) with ESMTP id LAA31177 for ; Sun, 20 Apr 2003 11:56:22 -0300 Received: from bol.com.br (200.221.24.140) by salem.bol.com.br (5.1.071) id 3E98B8140023564B for obm-l@mat.puc-rio.br; Sun, 20 Apr 2003 11:55:52 -0300 Date: Sun, 20 Apr 2003 11:55:52 -0300 Message-Id: Subject: [obm-l] =?iso-8859-1?q?Re=3A=5Bobm=2Dl=5D_quest=E3o_sobre_conjuntos?= MIME-Version: 1.0 Content-Type: text/plain;charset="iso-8859-1" From: "rmr-olimp" To: obm-l@mat.puc-rio.br X-XaM3-API-Version: 2.4 R3 ( B4 ) X-SenderIP: 200.225.149.154 Content-Transfer-Encoding: 8bit X-MIME-Autoconverted: from quoted-printable to 8bit by sucuri.mat.puc-rio.br id LAA31178 Sender: owner-obm-l@sucuri.mat.puc-rio.br Precedence: bulk Reply-To: obm-l@mat.puc-rio.br Olá Ranatinha, é um problema de análise combinatória clássico, a resposta é a combinação de 8 elementos tomados 5 a 5, ou seja: C 8,5 = ((8!)/(5!.(8-5)!)= 8.7=56 Espero ter ajudado, Rodrigo > Oi pessoal da lista, não consegui fazer esta questão, se > alguém puder me ajudar, ficarei bastante grata. > > (U.F. VIÇOSA-89) Um conjunto A tem 8 elementos > distintos. O número de subconjuntos de A, com 5 > elementos distintos cada um, é: > > P.S: Gostaria, se possível, que a solução não tivesse > probabilidade ou combinatória, pois ainda faço 1º ano do > ensino médio. > > []´s > Renatinha > > > ________________________________________________________ __________________ > Seleção de Softwares UOL. > 10 softwares escolhidos pelo UOL para você e sua família . > http://www.uol.com.br/selecao > > > ======================================================== ================= > Instruções para entrar na lista, sair da lista e usar a lista em > http://www.mat.puc-rio.br/~nicolau/olimp/obm-l.html > O administrador desta lista é > ======================================================== ================= > __________________________________________________________________________ Seleção de Softwares UOL. 10 softwares escolhidos pelo UOL para você e sua família. http://www.uol.com.br/selecao ========================================================================= Instruções para entrar na lista, sair da lista e usar a lista em http://www.mat.puc-rio.br/~nicolau/olimp/obm-l.html O administrador desta lista é ========================================================================= From owner-obm-l@sucuri.mat.puc-rio.br Sun Apr 20 12:03:38 2003 Return-Path: Received: (from majordom@localhost) by sucuri.mat.puc-rio.br (8.9.3/8.9.3) id MAA31260 for obm-l-MTTP; Sun, 20 Apr 2003 12:02:17 -0300 Received: from salvatore3.bol.com.br (salvatore3.bol.com.br [200.221.24.51]) by sucuri.mat.puc-rio.br (8.9.3/8.9.3) with ESMTP id MAA31256 for ; Sun, 20 Apr 2003 12:02:14 -0300 Received: from xx (200.221.24.48) by salvatore3.bol.com.br (5.1.071) id 3E76704800778594 for obm-l@mat.puc-rio.br; Sun, 20 Apr 2003 12:01:43 -0300 Message-ID: <000b01c3074d$c370c650$f4befea9@xx> From: "Blue Ice" To: References: Subject: [obm-l] =?iso-8859-1?Q?Re:_=5Bobm-l=5D_quest=E3o_sobre_conjuntos?= Date: Sun, 20 Apr 2003 12:01:45 -0300 MIME-Version: 1.0 Content-Type: text/plain; charset="iso-8859-1" Content-Transfer-Encoding: 8bit X-Priority: 3 X-MSMail-Priority: Normal X-Mailer: Microsoft Outlook Express 6.00.2600.0000 X-MimeOLE: Produced By Microsoft MimeOLE V6.00.2600.0000 X-Sender-IP: 200.148.35.226 Sender: owner-obm-l@sucuri.mat.puc-rio.br Precedence: bulk Reply-To: obm-l@mat.puc-rio.br ----- Original Message ----- From: "renatinha15a" To: "obm" Sent: Sunday, April 20, 2003 10:40 AM Subject: [obm-l] questão sobre conjuntos > E-mail Premium BOL > Antivírus, anti-spam e até 100 MB de espaço. Assine já! > http://email.bol.com.br/ > Oi pessoal da lista, não consegui fazer esta questão, se > alguém puder me ajudar, ficarei bastante grata. > > (U.F. VIÇOSA-89) Um conjunto A tem 8 elementos > distintos. O número de subconjuntos de A, com 5 > elementos distintos cada um, é: > > P.S: Gostaria, se possível, que a solução não tivesse > probabilidade ou combinatória, pois ainda faço 1º ano do > ensino médio. > > []´s > Renatinha > R=8.5=40 e possibilidades com o 40: 40/8:5 e 40/5>8 5+8:13 []´s Ice ICQ:177782914 Email:Vinicius84@hotmail.com Win XP Home Full/Speedy 256kbps ========================================================================= Instruções para entrar na lista, sair da lista e usar a lista em http://www.mat.puc-rio.br/~nicolau/olimp/obm-l.html O administrador desta lista é ========================================================================= From owner-obm-l@sucuri.mat.puc-rio.br Sun Apr 20 12:28:10 2003 Return-Path: Received: (from majordom@localhost) by sucuri.mat.puc-rio.br (8.9.3/8.9.3) id MAA32002 for obm-l-MTTP; Sun, 20 Apr 2003 12:26:51 -0300 Received: from ivoti.terra.com.br (ivoti.terra.com.br [200.176.3.20]) by sucuri.mat.puc-rio.br (8.9.3/8.9.3) with ESMTP id MAA31998 for ; Sun, 20 Apr 2003 12:26:47 -0300 Received: from botucatu.terra.com.br (botucatu.terra.com.br [200.176.3.78]) by ivoti.terra.com.br (Postfix) with ESMTP id 0C1F8408092 for ; Sun, 20 Apr 2003 12:26:14 -0300 (BRT) Received: from terra.com.br (200-158-174-176.dsl.telesp.net.br [200.158.174.176]) (authenticated user matematika) by botucatu.terra.com.br (Postfix) with ESMTP id A6DE829C070 for ; Sun, 20 Apr 2003 12:26:13 -0300 (BRT) Message-ID: <3EA2BBE6.90703@terra.com.br> Date: Sun, 20 Apr 2003 12:25:26 -0300 From: Eduardo Botelho User-Agent: Mozilla/5.0 (Windows; U; Windows NT 5.0; en-US; rv:1.0.2) Gecko/20030208 Netscape/7.02 X-Accept-Language: en-us, en MIME-Version: 1.0 To: obm-l@mat.puc-rio.br Subject: [obm-l] =?ISO-8859-1?Q?D=FAvida_sobre_grupos_diedrais_III?= References: <001701c306a6$a04f1160$f4befea9@xx> <3EA1A4D0.4050205@centroin.com.br> <200304191758510720.01B2F31B@smtp.watersportsbrazil.com> <3EA1D12A.8070101@terra.com.br> <3EA1F772.8090600@terra.com.br> Content-Type: text/plain; charset=ISO-8859-1; format=flowed Content-Transfer-Encoding: 8bit Sender: owner-obm-l@sucuri.mat.puc-rio.br Precedence: bulk Reply-To: obm-l@mat.puc-rio.br até na hora de corrigir, erro. O que quis dizer mesmo foi isso: > >> b: 1(0),2(240),3(120) >> ba: 1(240),2(120),3(0) >> ba^2: 1(120),2(0),3(240) ..ou seja, as rotações são horárias. Nesse >> caso, não valeria ab = ba^(n-1) > > Agora, sim.... ========================================================================= Instruções para entrar na lista, sair da lista e usar a lista em http://www.mat.puc-rio.br/~nicolau/olimp/obm-l.html O administrador desta lista é ========================================================================= From owner-obm-l@sucuri.mat.puc-rio.br Sun Apr 20 12:52:39 2003 Return-Path: Received: (from majordom@localhost) by sucuri.mat.puc-rio.br (8.9.3/8.9.3) id MAA32710 for obm-l-MTTP; Sun, 20 Apr 2003 12:50:57 -0300 Received: from www.zipmail.com.br (smtp.zipmail.com.br [200.221.11.147]) by sucuri.mat.puc-rio.br (8.9.3/8.9.3) with ESMTP id MAA32706 for ; Sun, 20 Apr 2003 12:50:53 -0300 From: luizhenriquerick@zipmail.com.br Received: from [200.216.33.73] by www.zipmail.com.br with HTTP; Sun, 20 Apr 2003 12:48:54 -0300 Message-ID: <3E9EC18900003EBA@www.zipmail.com.br> Date: Sun, 20 Apr 2003 12:48:54 -0300 Subject: [obm-l] =?iso-8859-1?Q?Geo=20Plana?= To: obm-l@mat.puc-rio.br MIME-Version: 1.0 Content-Type: multipart/mixed; boundary="=========3E9EC18900003EBA/www.zipmail.com.br" Sender: owner-obm-l@sucuri.mat.puc-rio.br Precedence: bulk Reply-To: obm-l@mat.puc-rio.br --=========3E9EC18900003EBA/www.zipmail.com.br Content-Type: text/plain; charset="iso-8859-1" Content-Transfer-Encoding: quoted-printable Seja ABCD um quadril=E1tero convexo tal que suas diagonais AC e BD s=E3o = perpendiculares. Seja P a interse=E7=E3o de AC e BD e seja M o ponto m=E9dio de AB. Mostre= que o quadril=E1tero ABCD =E9 inscrit=EDvel se, e somente se, as retas PM e C= D s=E3o perpendiculares. Temos de provar que #ABCD =E9 c=EDclico <=3D> PM perpendicular a DC. 1=B0 parte : #ABCD =E9 c=EDclico =3D> PM perpendicular a DC Hip=F3tese : #ABCD =E9 c=EDclico Tese : PM perpendicular a DC ** Lema : - Seja um tri=E2ngulo ABC de di=E2metro BC e centro =3D M . - Se o ponto A ( gen=E9rico ) esta entre o arco BC , ent=E3o o =E2ngulo A= BC =3D 90 - Como BC =E9 di=E2metro , tra=E7amos AM , que =E9 a mediana de BC e vale= BC/2 . - Observamos que em qualquer tri=E2ngulo ret=E2ngulo , a medida do lado o= posto ao =E2ngulo de 90=B0 sempre ser=E1 metade desse mesmo lado . ( Olhando na figura anexa ) - Seja o # ABCD inscrito em uma circunfer=EAncia de centro O . - Fazendo os =E2ngulos : BAC =3D a , ABD =3D b , BDC =3D g e ACD =3D f . - Como M =E9 m=E9dio de AB e o =E2ngulo APB =3D 90=B0 , temos pelo lema q= ue MP =3D AM =3D MB . - Assim os =E2ngulos APM =3D a e BPM =3D b . - Como =E2ngulos opostos pelo v=E9rtice s=E3o iguais , ent=E3o : DPM1 =3D BPM =3D b CPM1 =3D APM =3D a - Sendo os =E2ngulos MM1C =3D k e MM1D =3D k2. - Como a e g (falam) para um mesmo arco , ent=E3o s=E3o iguais .O mesmo o= corre com os =E2ngulos b e f . - No tri=E2ngulo ABP : a + b + 90 =3D 180 =3D> a + b =3D 90 ( i ) - Nos tri=E2ngulos PM1D e PM1C , temos : b + g + k2 =3D 180 e a + f + k =3D 180 ou ( b + a ) + k2 =3D 180 ( ii ) e ( a + b) + k =3D 180 ( iii ) De ( i ) em ( ii ) e ( iii ) , vem : 90 + k2 =3D 180 e 90 + k =3D 180 k2 =3D 90 e k =3D 90 Agora , como fa=E7o para provar a 2=B0 parte do problema ? PM perpendicular a DC =3D> #ABCD =E9 c=EDclico . Qualquer ajuda ser=E1 bem vinda . Abra=E7os . Luiz H. Barbosa . www.olympicmaths.hpg.com.br ------------------------------------------ Use o melhor sistema de busca da Internet Radar UOL - http://www.radaruol.com.br --=========3E9EC18900003EBA/www.zipmail.com.br Content-Type: image/gif Content-Transfer-Encoding: base64 Content-Disposition: attachment; filename="Con02.gif" R0lGODdhJQE0AfcAAAAAAAAAQAAAgAAA/wAgAAAgQAAggAAg/wBAAABAQABAgABA/wBgAABgQABg gABg/wCAAACAQACAgACA/wCgAACgQACggACg/wDAAADAQADAgADA/wD/AAD/QAD/gAD//yAAACAA QCAAgCAA/yAgACAgQCAggCAg/yBAACBAQCBAgCBA/yBgACBgQCBggCBg/yCAACCAQCCAgCCA/yCg ACCgQCCggCCg/yDAACDAQCDAgCDA/yD/ACD/QCD/gCD//0AAAEAAQEAAgEAA/0AgAEAgQEAggEAg /0BAAEBAQEBAgEBA/0BgAEBgQEBggEBg/0CAAECAQECAgECA/0CgAECgQECggECg/0DAAEDAQEDA gEDA/0D/AED/QED/gED//2AAAGAAQGAAgGAA/2AgAGAgQGAggGAg/2BAAGBAQGBAgGBA/2BgAGBg QGBggGBg/2CAAGCAQGCAgGCA/2CgAGCgQGCggGCg/2DAAGDAQGDAgGDA/2D/AGD/QGD/gGD//4AA AIAAQIAAgIAA/4AgAIAgQIAggIAg/4BAAIBAQIBAgIBA/4BgAIBgQIBggIBg/4CAAICAQICAgICA /4CgAICgQICggICg/4DAAIDAQIDAgIDA/4D/AID/QID/gID//6AAAKAAQKAAgKAA/6AgAKAgQKAg gKAg/6BAAKBAQKBAgKBA/6BgAKBgQKBggKBg/6CAAKCAQKCAgKCA/6CgAKCgQKCggKCg/6DAAKDA QKDAgKDA/6D/AKD/QKD/gKD//8AAAMAAQMAAgMAA/8AgAMAgQMAggMAg/8BAAMBAQMBAgMBA/8Bg AMBgQMBggMBg/8CAAMCAQMCAgMCA/8CgAMCgQMCggMCg/8DAAMDAQMDAgMDA/8D/AMD/QMD/gMD/ //8AAP8AQP8AgP8A//8gAP8gQP8ggP8g//9AAP9AQP9AgP9A//9gAP9gQP9ggP9g//+AAP+AQP+A gP+A//+gAP+gQP+ggP+g///AAP/AQP/AgP/A////AP//QP//gP///yH5BAAAAAAALAAAAAAlATQB AAjjAP8JHEiwoMGDCBMqXMiwocOHECNKnEixosWLGDNq3Mixo8ePIEOKHEmypMmTKFOqXMmypcuX MGPKnEmzps2bOHPq3Mmzp8+fQIMKHUq0qNGjSJPyBKC0qdOhTJ9KnZozKtWrWF9azcq1a8mtXsOK 3Qh2rNmzEcuiXcsWodq2cNm+jUt37Ny6eLnezct36t6+gJX+DUy46ODCiIEeTsx45+LGkG0+jkw5 5uTKmFlezswZ5ebOoEV+Dk264+jSqDGeTs164urWsB2+jk074ezauAnezp17N+/avn/HDi68NfHi rI//Iy+tfHlosACiS59Ovbrz1NVHZ7/e2Hpz69zrTj/Y3OD48GelLyxvOzp6veobsl8f/31T92ll 4rdveH/+mfXx55N/rt1EoICSHVggTgEiaFlw812koIMpTShhTxZSKNp3P2WoIUceqhZUiB9WRKKI Qp1Y4kMRCtTiRy9ep2JGMXo044rmqVQjjDhKdKNGO8IYJGxD/lOkjT3S59KRNk6WHZNrQQklkhD5 9yNoUxop1WNWZunVlSB5CeJh+4GJmZhaUjUYeLyhmaaaDJXpJpySZbVXl7jN+eZVd+E5nJ578vmW n0A2iBagLoZV1pOqnUcXookqGqajeUEaqaSNUgqYOaWXYpqWpohx2qmn7YHa3YjpmWdqZaLqdih1 xqWoKKy0teqqoPUxZWaoRNkaJ6056hprr0fBemekvv8ulSCBjiYL7EB9QistaXo2GKClbLpl27Tc dlatkVbputWcz8q37ajJGlgVuNJa+eCqsmnb7aiUISousuNqBi9FixakVrr6OeaisOzOCyR5+9J4 68L+nikwu1Exi2S2rqmIX7/ksaosuOFCR1bCFfuYsbynblzwyfRWSSlxNw6qEMCaLXUxtNeqDC/L XJKsc2Ewn1vqrpmS6PJ6ifXs88Agn3TxsTs3zZfRbpVL09IvVx0nz05JzWDEczF99abFPisq1U4z TLRYZKa479hcl2221V+uiaHWbtck7tBnxxv3X9UmXbdi5lb5JcojK/2y318jJfd/dg4Mt0nj0n2h 4nrjC66X438LaWiYxbK4IK6rzuddhUblLDJWkRdumqk19mz66X6pmmOhmzfsmWGM5551fGJbRHGd UMEuvFlOIh6wrLonj/avtQ+IvPKeH4qw8Q8DPvz1g3MssWCofs6vXZIHWrr13n+P+nkhDto88Bj6 juLum6+NMOYm14899IqFr2XER8/vvP33w59jWoas6U0PaLernvned7lLeSxq9AOg3Rg4uQbuyT0Z Ilv7dPKa3UCtcoFak/4AxEGFHawr+bLdltZlwhaiToVvS8q3TkhDPs3OXx+MoWXIAiILGuyHuGOf Cyn/uELVwbBzE+ShaV5oxCMGkYQ9XKINm4hDyk1NilGkExWd2L3jZVGJsetf5hS4JB5RqYhjFB/5 ymhGLD7Fa0oKHkyUc5wc3jCNKZMZAk0UkjpqUYyPG+Ae3ce5Nr4xcSDcYMz6WMhDIlI2rTpSeTjk yEcmkoUrYQ8ls3bJTgoxgaJhZCUt6ckrRm2QOnSjKmUYvZBhsoqEq+BINnmfVi7wkxFEZYtoyT1b ls+UukndEEXZyF768pdepFkwh0nMYlrxmK4EpjKnSUjIzZKTArQcFGG5xxhpEpvaJCIbVdVN0oWy lgFMpznPCUpnIjGbn9ohO9tpyGeGs4aLbKbh5mnPsXsyk56r/ApJ7KjGwAV0n/VEqD7fqU5+DYmO LREdOBvKRx3dsaKZHOhE4VlNgIpToe4cH0YTqtEvZvSa6LylkDw60nwuVKQqDalMY2pRlBqTpme0 6UdZelBidZSfM+VoSXXaT2SuFKhGrSlRGYrToDJwfXhcKkmf2NKh5jSWeZujQFP6U6uadEZiihBB C+rPgV4mdLA0aDKdKseuWpNFbbvoCNEk0VFW9a3xlGtco4rXl/8ytalePVzthGkhN4kVjbKsEN/2 CkMNclGrgS0qYLfanggyzLFAhKxU/zpZyp4yfsvE6gw3y9mkglR4d9uiPJHKyn8q9q58XadfJdvZ vpZVtWudKm1NK1to4na1sy0tb3vLPLSWMLKtxadWFkswauZRmsEVLkWJ69x/ZRa6uk3uTsf5NvWR FbtXtWti14q3i76SrTB1rVZ0WF4yZle7211Sc837XVz2lKvxXS8gbfXNxoGRQRnD2P9YK163Qvde qQQwgQsM26mFi36L+p1+o4vf/O7wwfyTnWUnjF74WvhB+9swhMNKWsR+eI4PxurJUKneE3t4vMDD IGZXTGIKM7i1thxeGQ6ph9wWh1G5SaQZBrvFYpPe948+ZmPHjljk/4YXhUbeZrvKlNYgPxnKUdZP ivkHHai69MhMBDKAuJziDnUYyTC2216bvGADZ8/Jyyqgiu2b5DDXOZ/CeqB77zxFMSdzyI+Vcpbt MmgOb3i+dE4z8QodUSoyFrwu9q+fYyZgy5LrytKDs34fONhENzjTfLZq5Phm5VBjedIAHQ+ZvASh vngQuFHNkm/GqulP2/a39KWum//h8mruAjKrJ0U1r4Xd5utW9su7flSte0xf+QU70qBWNFFBy81o KVXalTI1MxXUJzjeerh4uU3rINhskLH510IVj7YbzG0NzyyQxSYlr6DNbh1X27dglve8sb3sgrlr x1429roTjDV+t9ixs56kwQnTwRJj7t+MTjawHSZxfiP8vbaGN8UrXtuLYxzcsQ3MaiBqLmv1sdUg LxmO063xiN924s/hOL5ZvnKY6xtLGac5ul1+85BD5jOz7veYFi5wnNe85UcnOq4DjRqgD9znOY/1 b5yudLV+2+pIJ1LKoZ71SU2X1oqcec8/HfCfMX3pYB+w2HeObTPJONdoD4+ToonraRFDsusCT/uI FrMZhR/ORHci9bHtw3edZ3OEAO951yiUtrXHF+HbCe3N2yugxltdwuqccRz/vni4Ex7rO9O82c8u +sHrrN1nJ7y3m1Z6d+uZ3F2imIcq3alza73r/1pz5DVs7Nb7r2wpfK6OeExotquvZZo6 VtmF3PnClfnkT8K8yJufa6A/ur6kHfKuoi99agH/+Lnavb9pr2sh/4z7eldbsw/dfYBvr9HuR3+S zFvp12O/lE1CP5Xnn1d06XX5Y1dc8gdX/LdAwUdwJ6R/3Fd3BfgrcuZ5QWN7/deAgJdhqRd/+ld0 X9FtgP9GgaWiLQoIgBc4S90meR6IgYziVc2iHiJoeuSmgXIXgrxHXOG3ZBIiQpUXgiMngU1Ug871 KXyXfmOmgMNHT/eCYHP2dxP4HOJnIDzof9png5BkfSfYJNMGMTR2dzc4glUoWH50KxqUQS34gvfX hXAVdGCoewakOUK4N2OYeLLEgnczh0jThtghhTfIY333hmbYKD/4MfojQsTXh0P3h5OigylIiFpm gvCniAMiTIbmiBwEib4miVWBh5VoiU5oiM+miU44iGfmiU8jihpDipFhh6bIc6mobKvIGKjYivQG i4smiwVHiyJniwyHi7eoi67Gi73oi6MIjOEmjMNIjKwOaIzDhozJqIxywYxt8Yoyzrh00Yhm0/hm 1XiN2JiN2riN3NiN3viN4BiO4jiO5FiO5niO6JiO6riO7NiO7ogWAQEAOw== --=========3E9EC18900003EBA/www.zipmail.com.br-- ========================================================================= Instruções para entrar na lista, sair da lista e usar a lista em http://www.mat.puc-rio.br/~nicolau/olimp/obm-l.html O administrador desta lista é ========================================================================= From owner-obm-l@sucuri.mat.puc-rio.br Sun Apr 20 13:00:00 2003 Return-Path: Received: (from majordom@localhost) by sucuri.mat.puc-rio.br (8.9.3/8.9.3) id MAA00405 for obm-l-MTTP; Sun, 20 Apr 2003 12:58:25 -0300 Received: from salvatore3.bol.com.br (salvatore3.bol.com.br [200.221.24.51]) by sucuri.mat.puc-rio.br (8.9.3/8.9.3) with ESMTP id MAA00401 for ; Sun, 20 Apr 2003 12:58:21 -0300 Received: from xx (200.221.24.48) by salvatore3.bol.com.br (5.1.071) id 3E76704800779EB0 for obm-l@mat.puc-rio.br; Sun, 20 Apr 2003 12:57:51 -0300 Message-ID: <000f01c30755$a082d810$f4befea9@xx> From: "Blue Ice" To: References: Subject: [obm-l] =?iso-8859-1?Q?Re:_=5Bobm-l=5D_Re:=5Bobm-l=5D_quest=E3o_sobre_conjuntos?= Date: Sun, 20 Apr 2003 12:58:03 -0300 MIME-Version: 1.0 Content-Type: text/plain; charset="iso-8859-1" Content-Transfer-Encoding: 8bit X-Priority: 3 X-MSMail-Priority: Normal X-Mailer: Microsoft Outlook Express 6.00.2600.0000 X-MimeOLE: Produced By Microsoft MimeOLE V6.00.2600.0000 X-Sender-IP: 200.148.35.232 Sender: owner-obm-l@sucuri.mat.puc-rio.br Precedence: bulk Reply-To: obm-l@mat.puc-rio.br ----- Original Message ----- From: "rmr-olimp" To: Sent: Sunday, April 20, 2003 11:55 AM Subject: [obm-l] Re:[obm-l] questão sobre conjuntos > E-mail Premium BOL > Antivírus, anti-spam e até 100 MB de espaço. Assine já! > http://email.bol.com.br/ > Olá Ranatinha, > > é um problema de análise combinatória clássico, a > resposta é a combinação de 8 elementos tomados 5 a 5, ou > seja: C 8,5 = ((8!)/(5!.(8-5)!)= 8.7=56 Interessante sua solução!boiei nele.... eu ainda não aprendi essa matéria, mas ás vezes vale a pena ´´chutar´´ ;)) []´s Ice ICQ:177782914 Email:Vinicius84@hotmail.com Win XP Home Full/Speedy 256kbps ========================================================================= Instruções para entrar na lista, sair da lista e usar a lista em http://www.mat.puc-rio.br/~nicolau/olimp/obm-l.html O administrador desta lista é ========================================================================= From owner-obm-l@sucuri.mat.puc-rio.br Sun Apr 20 15:12:20 2003 Return-Path: Received: (from majordom@localhost) by sucuri.mat.puc-rio.br (8.9.3/8.9.3) id PAA02862 for obm-l-MTTP; Sun, 20 Apr 2003 15:08:14 -0300 Received: from web41501.mail.yahoo.com (web41501.mail.yahoo.com [66.218.93.84]) by sucuri.mat.puc-rio.br (8.9.3/8.9.3) with SMTP id PAA02858 for ; Sun, 20 Apr 2003 15:08:10 -0300 Message-ID: <20030420180738.40811.qmail@web41501.mail.yahoo.com> Received: from [200.190.14.115] by web41501.mail.yahoo.com via HTTP; Sun, 20 Apr 2003 11:07:38 PDT Date: Sun, 20 Apr 2003 11:07:38 -0700 (PDT) From: Carlos Yuzo Shine Subject: RE: [obm-l] Problema proposto 74 eureka 15... To: obm-l@mat.puc-rio.br In-Reply-To: <003601c306cb$3da88d50$9865fea9@computer> MIME-Version: 1.0 Content-Type: text/plain; charset=us-ascii Sender: owner-obm-l@sucuri.mat.puc-rio.br Precedence: bulk Reply-To: obm-l@mat.puc-rio.br Oi pessoal da lista, Acho que consegui fazer esse problema... > >Ache todas as funçoes f: R -> R tais que: f(x+y)+f(x-y)=2.f(x).cos(y) Faça x = 0: f(y) + f(-y) = 2f(0)cos(y) Troque x e y de lugar: f(x+y)+f(y-x)=2f(y)cos(x) Some a original com a última: 2f(x+y) + f(x-y) + f(y-x) = 2f(x)cos(y) + 2f(y)cos(x) Mas f(x-y) + f(y-x) = 2f(0)cos(y), logo f(x+y) + f(0)cos(y) = f(x)cos(y) + f(y)cos(x) Faça y=0: f(x) + f(0) = f(x) + f(0)cos(x) => f(0) = 0 Agora temos f(x+y) = f(x)cos(y) + f(y)cos(x) Faça y = pi/2: f(x+pi/2) = f(pi/2)cos(x) Logo f(x) = f(pi/2)sen(x), ou seja, f(x) = A.sen(x), sendo A um real qualquer. Pode-se verificar que todas essas servem. []'s Shine __________________________________________________ Do you Yahoo!? The New Yahoo! Search - Faster. Easier. Bingo http://search.yahoo.com ========================================================================= Instruções para entrar na lista, sair da lista e usar a lista em http://www.mat.puc-rio.br/~nicolau/olimp/obm-l.html O administrador desta lista é ========================================================================= From owner-obm-l@sucuri.mat.puc-rio.br Sun Apr 20 15:17:56 2003 Return-Path: Received: (from majordom@localhost) by sucuri.mat.puc-rio.br (8.9.3/8.9.3) id PAA03017 for obm-l-MTTP; Sun, 20 Apr 2003 15:14:04 -0300 Received: from krypton.hosting4u.net (krypton.hosting4u.net [209.15.2.78]) by sucuri.mat.puc-rio.br (8.9.3/8.9.3) with ESMTP id PAA03013 for ; Sun, 20 Apr 2003 15:14:01 -0300 Received: from gargamel (200-158-200-72.dsl.telesp.net.br [200.158.200.72]) by krypton.hosting4u.net (Postfix) with ESMTP id 3A091A0808 for ; Sun, 20 Apr 2003 13:13:28 -0500 (CDT) Message-ID: <200304201517060500.000A8A62@smtp.watersportsbrazil.com> In-Reply-To: <000f01c30755$a082d810$f4befea9@xx> References: <000f01c30755$a082d810$f4befea9@xx> X-Mailer: Calypso Version 3.30.00.00 (3) Date: Sun, 20 Apr 2003 15:17:06 -0300 From: "Ariel de Silvio" To: obm-l@mat.puc-rio.br Subject: Re: [obm-l] Re: [obm-l] Re:[obm-l] =?ISO-8859-1?Q?quest=E3o_sobre_conjuntos?= Mime-Version: 1.0 Content-Type: text/plain; charset="ISO-8859-1" Content-Transfer-Encoding: 8bit X-MIME-Autoconverted: from quoted-printable to 8bit by sucuri.mat.puc-rio.br id PAA03014 Sender: owner-obm-l@sucuri.mat.puc-rio.br Precedence: bulk Reply-To: obm-l@mat.puc-rio.br É Ice, Análise Combinatória é matéria de 2o colegial... Eu achei mto interessante qdo aprendi, mesmo demorando pra pegar pq perdi algumas aulas... O mais dificil neles é a interpretação do problema, não é como uma equação que ele fala x=2+2 => x=4 Vc precisa interpretar o problema, e mta gente roda por erro de portugues... heheheh Depois q vc achou os valores normalmente são a mesma coisa... Não se torture por nao saber isso, na sua idade eu nem imaginava o q era isso... hehehehe falou Ariel *********** MENSAGEM ORIGINAL *********** As 12:58 de 20/4/2003 Blue Ice escreveu: >----- Original Message ----- >From: "rmr-olimp" >To: >Sent: Sunday, April 20, 2003 11:55 AM >Subject: [obm-l] Re:[obm-l] questão sobre conjuntos > > >> E-mail Premium BOL >> Antivírus, anti-spam e até 100 MB de espaço. Assine já! >> http://email.bol.com.br/ >> Olá Ranatinha, >> >> é um problema de análise combinatória clássico, a >> resposta é a combinação de 8 elementos tomados 5 a 5, ou >> seja: C 8,5 = ((8!)/(5!.(8-5)!)= 8.7=56 > > >Interessante sua solução!boiei nele.... eu ainda não aprendi essa matéria, >mas ás vezes vale a pena ´´chutar´´ ;)) > > >[]´s > >Ice > > >ICQ:177782914 Email:Vinicius84@hotmail.com >Win XP Home Full/Speedy 256kbps > > > > >========================================================================= >Instruções para entrar na lista, sair da lista e usar a lista em >http://www.mat.puc-rio.br/~nicolau/olimp/obm-l.html >O administrador desta lista é >========================================================================= ========================================================================= Instruções para entrar na lista, sair da lista e usar a lista em http://www.mat.puc-rio.br/~nicolau/olimp/obm-l.html O administrador desta lista é ========================================================================= From owner-obm-l@sucuri.mat.puc-rio.br Sun Apr 20 15:22:15 2003 Return-Path: Received: (from majordom@localhost) by sucuri.mat.puc-rio.br (8.9.3/8.9.3) id PAA03213 for obm-l-MTTP; Sun, 20 Apr 2003 15:20:52 -0300 Received: from Euler.impa.br (euler.impa.br [147.65.1.3]) by sucuri.mat.puc-rio.br (8.9.3/8.9.3) with ESMTP id PAA03196 for ; Sun, 20 Apr 2003 15:20:45 -0300 Received: from Gauss.impa.br (Gauss [147.65.4.1]) by Euler.impa.br (8.11.6p2/8.11.6) with ESMTP id h3KIJp027827 for ; Sun, 20 Apr 2003 15:19:51 -0300 (EST) From: Carlos Gustavo Tamm de Araujo Moreira Received: by Gauss.impa.br (8.11.6p2) id h3KIJfQ05163; Sun, 20 Apr 2003 15:19:41 -0300 (EST) Message-Id: <200304201819.h3KIJfQ05163@Gauss.impa.br> Subject: Re: [obm-l] Problema proposto 74 eureka 15... To: obm-l@mat.puc-rio.br Date: Sun, 20 Apr 2003 15:19:41 -0300 (EST) In-Reply-To: <20030420180738.40811.qmail@web41501.mail.yahoo.com> from "Carlos Yuzo Shine" at Apr 20, 3 11:07:38 am X-Mailer: ELM [version 2.4 PL25] MIME-Version: 1.0 Content-Type: text/plain; charset=US-ASCII Content-Transfer-Encoding: 7bit Sender: owner-obm-l@sucuri.mat.puc-rio.br Precedence: bulk Reply-To: obm-l@mat.puc-rio.br Oi Shine, Note que f(x)=cos(x) e' solucao, donde f(0) nao e' sempre 0. De qualquer jeito e' facil corrigir a sua solucao: trocando f(x) por f(x)-f(0).cos(x) podemos supor que f(0)=0. Abracos, Gugu > >Oi pessoal da lista, > >Acho que consegui fazer esse problema... > >> >Ache todas as funçoes f: R -> R tais que: >f(x+y)+f(x-y)=2.f(x).cos(y) > >Faça x = 0: >f(y) + f(-y) = 2f(0)cos(y) > >Troque x e y de lugar: >f(x+y)+f(y-x)=2f(y)cos(x) > >Some a original com a última: >2f(x+y) + f(x-y) + f(y-x) = 2f(x)cos(y) + 2f(y)cos(x) > >Mas f(x-y) + f(y-x) = 2f(0)cos(y), logo Aqui f(x-y)+f(y-x)=2f(0)cos(x-y). >f(x+y) + f(0)cos(y) = f(x)cos(y) + f(y)cos(x) > >Faça y=0: >f(x) + f(0) = f(x) + f(0)cos(x) => f(0) = 0 > >Agora temos >f(x+y) = f(x)cos(y) + f(y)cos(x) > >Faça y = pi/2: >f(x+pi/2) = f(pi/2)cos(x) > >Logo >f(x) = f(pi/2)sen(x), ou seja, f(x) = A.sen(x), sendo >A um real qualquer. Pode-se verificar que todas essas >servem. > >[]'s >Shine > >__________________________________________________ >Do you Yahoo!? >The New Yahoo! Search - Faster. Easier. Bingo >http://search.yahoo.com >========================================================================= >Instruções para entrar na lista, sair da lista e usar a lista em >http://www.mat.puc-rio.br/~nicolau/olimp/obm-l.html >O administrador desta lista é >========================================================================= ========================================================================= Instruções para entrar na lista, sair da lista e usar a lista em http://www.mat.puc-rio.br/~nicolau/olimp/obm-l.html O administrador desta lista é ========================================================================= From owner-obm-l@sucuri.mat.puc-rio.br Sun Apr 20 16:31:46 2003 Return-Path: Received: (from majordom@localhost) by sucuri.mat.puc-rio.br (8.9.3/8.9.3) id QAA05037 for obm-l-MTTP; Sun, 20 Apr 2003 16:27:59 -0300 Received: from shannon.bol.com.br (shannon.bol.com.br [200.221.24.13]) by sucuri.mat.puc-rio.br (8.9.3/8.9.3) with ESMTP id QAA05033 for ; Sun, 20 Apr 2003 16:27:56 -0300 Received: from bol.com.br (200.221.24.130) by shannon.bol.com.br (5.1.071) id 3E9EDE39000CC904 for obm-l@mat.puc-rio.br; Sun, 20 Apr 2003 16:27:25 -0300 Date: Sun, 20 Apr 2003 16:27:25 -0300 Message-Id: Subject: [obm-l] =?iso-8859-1?q?Re=3A=5Bobm=2Dl=5D_quest=E3o_sobre_conjuntos?= MIME-Version: 1.0 Content-Type: text/plain;charset="iso-8859-1" From: "renatinha15a" To: obm-l@mat.puc-rio.br X-XaM3-API-Version: 2.4 R3 ( B4 ) X-SenderIP: 200.241.108.194 Content-Transfer-Encoding: 8bit X-MIME-Autoconverted: from quoted-printable to 8bit by sucuri.mat.puc-rio.br id QAA05034 Sender: owner-obm-l@sucuri.mat.puc-rio.br Precedence: bulk Reply-To: obm-l@mat.puc-rio.br Primeiramente, obrigada Rodrigo pela resposta da questão e Ice pela tentativa. Mas ainda gostaria de saber se existe outra forma de resolver essa questão. Digo isso, pois ela é uma questão do livro Fundamentos de mat. elementar, e estou estranhando o fato dela está na matéria de conjuntos. Talves seja somente erro didático dos autores. De qualquer forma, ficarei grata por qualquer esclarecimento. []´s Renatinha __________________________________________________________________________ Seleção de Softwares UOL. 10 softwares escolhidos pelo UOL para você e sua família. http://www.uol.com.br/selecao ========================================================================= Instruções para entrar na lista, sair da lista e usar a lista em http://www.mat.puc-rio.br/~nicolau/olimp/obm-l.html O administrador desta lista é ========================================================================= From owner-obm-l@sucuri.mat.puc-rio.br Sun Apr 20 17:03:45 2003 Return-Path: Received: (from majordom@localhost) by sucuri.mat.puc-rio.br (8.9.3/8.9.3) id RAA05793 for obm-l-MTTP; Sun, 20 Apr 2003 17:01:14 -0300 Received: from imo-d09.mx.aol.com (imo-d09.mx.aol.com [205.188.157.41]) by sucuri.mat.puc-rio.br (8.9.3/8.9.3) with ESMTP id RAA05781 for ; Sun, 20 Apr 2003 17:01:07 -0300 From: Jkwasinsky@aol.com Received: from Jkwasinsky@aol.com by imo-d09.mx.aol.com (mail_out_v34.22.) id z.159.1e7d9564 (3310) for ; Sun, 20 Apr 2003 16:00:28 -0400 (EDT) Message-ID: <159.1e7d9564.2bd4565c@aol.com> Date: Sun, 20 Apr 2003 16:00:28 EDT Subject: =?ISO-8859-1?Q?Re:=20[obm-l]=20quest=E3o=20sobre=20conjuntos?= To: obm-l@mat.puc-rio.br MIME-Version: 1.0 Content-Type: multipart/alternative; boundary="part1_159.1e7d9564.2bd4565c_boundary" X-Mailer: 7.0 for Windows sub 537 Sender: owner-obm-l@sucuri.mat.puc-rio.br Precedence: bulk Reply-To: obm-l@mat.puc-rio.br --part1_159.1e7d9564.2bd4565c_boundary Content-Type: text/plain; charset="ISO-8859-1" Content-Transfer-Encoding: quoted-printable Ola renatinha, acho um pouco dificil nao utilizar um raciocinio que nao leve= =20 em consideracao ,pelo menos, o principio multiplicativo. a=ED vai!!!!!!!!! Voce lembra de como se demonstra a formula que da o numero de diagonais de= =20 um poligono convexo????? n(n-3)/2. Lembre de que o significado da divisao por dois ( 2 ) se devia ao= =20 fato de que ao estabelecer um processo de congem, voce associava =E0s diagon= ais=20 os segmentos de reta AB, AC , ...... conforme o numero de lados ( que=20 coincide com o de vertices ), e ao adotar essa tecnica, voce deve lembrar qu= e=20 o segmento AB, representava o mesmo segmento BA. Da=ED em diante voce conhec= e o=20 procedimento.Voce tambem poderia pensar no problema dos apertos de maos, no=20 problema de se organizar um campeonato de futebol (onde os times se enfrenta= m=20 apenas uma vez ), Imagine agora que voce deseja convidar um grupo de cinco=20 amigos de um total de oito. Quantos grupos poderia formar? Pense em primairo lugar, para fixar ideias, em todas as maneiras poss=EDveis= de=20 grupar 5 letras de um total de 8, digamos A,B,C,D,E,F,G,H. =C9 facil conclu= ir=20 que sao no total de 8.7.6.5.4 , que coresponde a 8 escolhas para a primeira= ,=20 7 para a segunda ... .Tomemos por exemlo as pessoas correspondentes as letra= s=20 ABCDE este grupo e o mesmo que BACDE e portanto, esse grupamento se repetiri= a=20 5.4.3.2.1 vezes. Como a cada grupamento distinto deste ,por exemplo , ADEFG=20 temos tambem um total de 5.4.3.2.1 repeticoes . Portanto , a resposta ao seu= =20 problema =E9 : 8.7.6.5.4/ 5.4.3.2.1 que da 56 subconjuntos . --part1_159.1e7d9564.2bd4565c_boundary Content-Type: text/html; charset="ISO-8859-1" Content-Transfer-Encoding: quoted-printable Ola renatinha, acho um pouco dificil nao utilizar um r= aciocinio que nao leve em consideracao ,pelo menos, o principio multiplicati= vo. a=ED vai!!!!!!!!!

  Voce lembra de como se demonstra a formula que da o numero de diagona= is de um poligono convexo?????

n(n-3)/2. Lembre de que o significado da divisao por dois ( 2 ) se devia ao= fato de que ao estabelecer um processo de congem, voce associava =E0s diago= nais os segmentos de reta AB, AC , ...... conforme o numero de lados ( que c= oincide com o de vertices ), e ao adotar essa tecnica, voce deve lembrar que= o segmento AB, representava o mesmo segmento BA. Da=ED em diante voce conhe= ce o procedimento.Voce tambem poderia pensar no problema dos apertos de maos= , no problema de se organizar um campeonato de futebol (onde os times se enf= rentam apenas uma vez ), Imagine agora que voce deseja convidar um grupo de=20= cinco amigos de um total de oito. Quantos grupos poderia formar?
Pense em primairo lugar, para fixar ideias, em todas as maneiras poss=EDveis= de grupar 5 letras de um total de 8, digamos A,B,C,D,E,F,G,H.  =C9 fac= il concluir que sao no total de  8.7.6.5.4 , que coresponde a 8 escolha= s para a primeira, 7 para a segunda ... .Tomemos por exemlo as pessoas corre= spondentes as letras ABCDE este grupo e o mesmo que BACDE e portanto, esse g= rupamento se repetiria 5.4.3.2.1 vezes. Como a cada grupamento distinto dest= e ,por exemplo , ADEFG temos tambem um total de 5.4.3.2.1 repeticoes . Porta= nto , a resposta ao seu problema =E9 :

            8.7.6.5.4= / 5.4.3.2.1 que da 56 subconjuntos .
--part1_159.1e7d9564.2bd4565c_boundary-- ========================================================================= Instruções para entrar na lista, sair da lista e usar a lista em http://www.mat.puc-rio.br/~nicolau/olimp/obm-l.html O administrador desta lista é ========================================================================= From owner-obm-l@sucuri.mat.puc-rio.br Sun Apr 20 17:03:47 2003 Return-Path: Received: (from majordom@localhost) by sucuri.mat.puc-rio.br (8.9.3/8.9.3) id RAA05777 for obm-l-MTTP; Sun, 20 Apr 2003 17:01:07 -0300 Received: from imo-r02.mx.aol.com (imo-r02.mx.aol.com [152.163.225.98]) by sucuri.mat.puc-rio.br (8.9.3/8.9.3) with ESMTP id RAA05772 for ; Sun, 20 Apr 2003 17:01:03 -0300 From: Jkwasinsky@aol.com Received: from Jkwasinsky@aol.com by imo-r02.mx.aol.com (mail_out_v34.22.) id z.ac.3e390c2d (3310) for ; Sun, 20 Apr 2003 16:00:29 -0400 (EDT) Message-ID: Date: Sun, 20 Apr 2003 16:00:29 EDT Subject: =?ISO-8859-1?Q?Re:=20[obm-l]=20quest=E3o=20sobre=20conjuntos?= To: obm-l@mat.puc-rio.br MIME-Version: 1.0 Content-Type: text/plain; charset="US-ASCII" Content-Transfer-Encoding: 7bit X-Mailer: 7.0 for Windows sub 537 Sender: owner-obm-l@sucuri.mat.puc-rio.br Precedence: bulk Reply-To: obm-l@mat.puc-rio.br ========================================================================= Instruções para entrar na lista, sair da lista e usar a lista em http://www.mat.puc-rio.br/~nicolau/olimp/obm-l.html O administrador desta lista é ========================================================================= From owner-obm-l@sucuri.mat.puc-rio.br Sun Apr 20 17:03:49 2003 Return-Path: Received: (from majordom@localhost) by sucuri.mat.puc-rio.br (8.9.3/8.9.3) id RAA05787 for obm-l-MTTP; Sun, 20 Apr 2003 17:01:13 -0300 Received: from imo-r03.mx.aol.com (imo-r03.mx.aol.com [152.163.225.99]) by sucuri.mat.puc-rio.br (8.9.3/8.9.3) with ESMTP id RAA05776 for ; Sun, 20 Apr 2003 17:01:06 -0300 From: Jkwasinsky@aol.com Received: from Jkwasinsky@aol.com by imo-r03.mx.aol.com (mail_out_v34.22.) id z.48.1b8aeba9 (3310) for ; Sun, 20 Apr 2003 16:00:30 -0400 (EDT) Message-ID: <48.1b8aeba9.2bd4565e@aol.com> Date: Sun, 20 Apr 2003 16:00:30 EDT Subject: =?ISO-8859-1?Q?Re:=20[obm-l]=20quest=E3o=20sobre=20conjuntos?= To: obm-l@mat.puc-rio.br MIME-Version: 1.0 Content-Type: text/plain; charset="US-ASCII" Content-Transfer-Encoding: 7bit X-Mailer: 7.0 for Windows sub 537 Sender: owner-obm-l@sucuri.mat.puc-rio.br Precedence: bulk Reply-To: obm-l@mat.puc-rio.br ========================================================================= Instruções para entrar na lista, sair da lista e usar a lista em http://www.mat.puc-rio.br/~nicolau/olimp/obm-l.html O administrador desta lista é ========================================================================= From owner-obm-l@sucuri.mat.puc-rio.br Sun Apr 20 17:50:34 2003 Return-Path: Received: (from majordom@localhost) by sucuri.mat.puc-rio.br (8.9.3/8.9.3) id RAA07362 for obm-l-MTTP; Sun, 20 Apr 2003 17:47:26 -0300 Received: from artemis.opendf.com.br (artemis.opengate.com.br [200.181.71.14]) by sucuri.mat.puc-rio.br (8.9.3/8.9.3) with ESMTP id RAA07357 for ; Sun, 20 Apr 2003 17:47:21 -0300 Received: from localhost (localhost [127.0.0.1]) by artemis.opendf.com.br (Postfix) with ESMTP id A33FE2BEC9 for ; Sun, 20 Apr 2003 17:48:14 -0300 (BRT) Received: from artemis.opendf.com.br ([127.0.0.1]) by localhost (artemis.opengate.com.br [127.0.0.1:10024]) (amavisd-new) with ESMTP id 03650-04 for ; Sun, 20 Apr 2003 17:48:13 -0300 (BRT) Received: from computer (200-181-090-222.bsace7001.dsl.brasiltelecom.net.br [200.181.90.222]) by artemis.opendf.com.br (Postfix) with ESMTP id 4E6AB2BEC2 for ; Sun, 20 Apr 2003 17:48:13 -0300 (BRT) From: "Artur Costa Steiner" To: Subject: RE: [obm-l] Problema proposto 74 eureka 15... Date: Sun, 20 Apr 2003 17:46:54 -0300 Organization: Steiner Consultoria LTDA Message-ID: <005601c3077d$fb2c5980$9865fea9@computer> MIME-Version: 1.0 Content-Type: text/plain; charset="iso-8859-1" X-Priority: 3 (Normal) X-MSMail-Priority: Normal X-Mailer: Microsoft Outlook, Build 10.0.2627 Importance: Normal X-MimeOLE: Produced By Microsoft MimeOLE V6.00.2800.1106 In-Reply-To: <20030420180738.40811.qmail@web41501.mail.yahoo.com> X-Virus-Scanned: by amavisd-new Content-Transfer-Encoding: 8bit X-MIME-Autoconverted: from quoted-printable to 8bit by sucuri.mat.puc-rio.br id RAA07358 Sender: owner-obm-l@sucuri.mat.puc-rio.br Precedence: bulk Reply-To: obm-l@mat.puc-rio.br >-----Original Message----- >From: owner-obm-l@sucuri.mat.puc-rio.br [mailto:owner-obm-l@sucuri.mat.puc- >rio.br] On Behalf Of Carlos Yuzo Shine >Sent: Sunday, April 20, 2003 3:08 PM >To: obm-l@mat.puc-rio.br >Subject: RE: [obm-l] Problema proposto 74 eureka 15... > >Oi pessoal da lista, > >Acho que consegui fazer esse problema... > >> >Ache todas as funçoes f: R -> R tais que: >f(x+y)+f(x-y)=2.f(x).cos(y) > >Faça x = 0: >f(y) + f(-y) = 2f(0)cos(y) > >Troque x e y de lugar: >f(x+y)+f(y-x)=2f(y)cos(x) > >Some a original com a última: >2f(x+y) + f(x-y) + f(y-x) = 2f(x)cos(y) + 2f(y)cos(x) > >Mas f(x-y) + f(y-x) = 2f(0)cos(y), logo [Artur Costa Steiner] Acho que f(x-y) + f(y-x) = 2f(0)cos(x-y). Nao eh isto? >f(x+y) + f(0)cos(y) = f(x)cos(y) + f(y)cos(x) > >Faça y=0: >f(x) + f(0) = f(x) + f(0)cos(x) => f(0) = 0 > >Agora temos >f(x+y) = f(x)cos(y) + f(y)cos(x) > >Faça y = pi/2: >f(x+pi/2) = f(pi/2)cos(x) > >Logo >f(x) = f(pi/2)sen(x), ou seja, f(x) = A.sen(x), sendo >A um real qualquer. Pode-se verificar que todas essas >servem. [Artur Costa Steiner] De fato. Mas nao eh a unica solucao. Observe que f(x) = A cos x tambem satisfaz a condicao desejada. Artur > >[]'s >Shine > >__________________________________________________ >Do you Yahoo!? >The New Yahoo! Search - Faster. Easier. Bingo >http://search.yahoo.com >======================================================================= == >Instruções para entrar na lista, sair da lista e usar a lista em >http://www.mat.puc-rio.br/~nicolau/olimp/obm-l.html >O administrador desta lista é >======================================================================= == ========================================================================= Instruções para entrar na lista, sair da lista e usar a lista em http://www.mat.puc-rio.br/~nicolau/olimp/obm-l.html O administrador desta lista é ========================================================================= From owner-obm-l@sucuri.mat.puc-rio.br Sun Apr 20 18:17:35 2003 Return-Path: Received: (from majordom@localhost) by sucuri.mat.puc-rio.br (8.9.3/8.9.3) id SAA08071 for obm-l-MTTP; Sun, 20 Apr 2003 18:13:10 -0300 Received: from mail1c.webmessenger.it (mail2.webmessenger.it [193.70.193.55]) by sucuri.mat.puc-rio.br (8.9.3/8.9.3) with ESMTP id SAA08066 for ; Sun, 20 Apr 2003 18:13:06 -0300 Received: from iloveusa (200.217.173.243) by mail1c.webmessenger.it (6.7.016) (authenticated as roxette_lover@email.it) id 3E8868D400A122D2 for obm-l@mat.puc-rio.br; Sun, 20 Apr 2003 23:12:34 +0200 Message-ID: <007901c30781$bfa14a20$f3add9c8@iloveusa> From: "=?iso-8859-1?Q?J.Paulo_roxer_=B4til_the_end?=" To: Subject: [obm-l] =?iso-8859-1?Q?D=FAvida_Sobre_a_Utilidade_da_Matem=E1tica_Ensinada_nas_Es?= =?iso-8859-1?Q?colas?= Date: Sun, 20 Apr 2003 17:44:54 -0300 MIME-Version: 1.0 Content-Type: multipart/alternative; boundary="----=_NextPart_000_0012_01C30764.8DC61980" X-Priority: 3 X-MSMail-Priority: Normal X-Mailer: Microsoft Outlook Express 6.00.2800.1106 X-MimeOLE: Produced By Microsoft MimeOLE V6.00.2800.1106 Sender: owner-obm-l@sucuri.mat.puc-rio.br Precedence: bulk Reply-To: obm-l@mat.puc-rio.br This is a multi-part message in MIME format. ------=_NextPart_000_0012_01C30764.8DC61980 Content-Type: text/plain; charset="iso-8859-1" Content-Transfer-Encoding: quoted-printable Ol=E1 Algu=E9m poderia dizer pra que serve conjuntos,an=E1lise = combinat=F3ria,bissetriz,equa=E7=E3o do 1=BA e 2=BA grau,produtos = not=E1veis,f=F3rmulas(Como a de B=E1skara), entre outros assuntos que = parecem n=E3o fazer parte do mundo real? Existe algum emprego em q essas coisas s=E3o usadas? [ ]=B4s Jo=E3o Paulo -- Email.it, the professional e-mail, gratis per te: http://www.email.it/f Sponsor: Libri e CD musicali nuovi con sconti dal 60 all'80%! Clicca qui: http://adv.email.it/cgi-bin/foclick.cgi?mid=3D814&d=3D20-4 ------=_NextPart_000_0012_01C30764.8DC61980 Content-Type: text/html; charset="iso-8859-1" Content-Transfer-Encoding: quoted-printable
Ol=E1
 
Algu=E9m poderia dizer pra que serve conjuntos,an=E1lise=20 combinat=F3ria,bissetriz,equa=E7=E3o do 1=BA e 2=BA grau,produtos=20 not=E1veis,f=F3rmulas(Como a de B=E1skara), entre outros assuntos que = parecem n=E3o=20 fazer parte do mundo real?
Existe algum emprego em q essas coisas s=E3o usadas?
 
[ ]=B4s
 
Jo=E3o Paulo

----
Email.it, the professional e-mail, gratis per te: clicca qui

Sponsor:
Libri e CD musicali nuovi con sconti dal 60 all'80%!
Clicca qui

------=_NextPart_000_0012_01C30764.8DC61980-- ========================================================================= Instruções para entrar na lista, sair da lista e usar a lista em http://www.mat.puc-rio.br/~nicolau/olimp/obm-l.html O administrador desta lista é ========================================================================= From owner-obm-l@sucuri.mat.puc-rio.br Sun Apr 20 18:30:36 2003 Return-Path: Received: (from majordom@localhost) by sucuri.mat.puc-rio.br (8.9.3/8.9.3) id SAA08459 for obm-l-MTTP; Sun, 20 Apr 2003 18:26:28 -0300 Received: from smtp-27.ig.com.br (smtp-27.ig.com.br [200.226.132.159]) by sucuri.mat.puc-rio.br (8.9.3/8.9.3) with SMTP id SAA08446 for ; Sun, 20 Apr 2003 18:26:22 -0300 Received: (qmail 8061 invoked from network); 20 Apr 2003 21:25:37 -0000 Received: from unknown (HELO xxxx) (200.165.168.157) by smtp-27.ig.com.br with SMTP; 20 Apr 2003 21:25:37 -0000 Message-ID: <000f01c30783$f0903a40$9da8a5c8@epq.ime.eb.br> From: "Marcio" To: References: <005601c3077d$fb2c5980$9865fea9@computer> Subject: Re: [obm-l] Problema proposto 74 eureka 15... Date: Sun, 20 Apr 2003 18:29:34 -0300 MIME-Version: 1.0 Content-Type: text/plain; charset="iso-8859-1" Content-Transfer-Encoding: 7bit X-Priority: 3 X-MSMail-Priority: Normal X-Mailer: Microsoft Outlook Express 5.50.4133.2400 X-MimeOLE: Produced By Microsoft MimeOLE V5.50.4133.2400 Sender: owner-obm-l@sucuri.mat.puc-rio.br Precedence: bulk Reply-To: obm-l@mat.puc-rio.br A mensagem que o Gugu mandou corrige esse detalhe e mostra todas as solucoes. Marcio > >Logo > >f(x) = f(pi/2)sen(x), ou seja, f(x) = A.sen(x), sendo > >A um real qualquer. Pode-se verificar que todas essas > >servem. > [Artur Costa Steiner] > De fato. Mas nao eh a unica solucao. Observe que f(x) = A cos x tambem > satisfaz a condicao desejada. > Artur > ========================================================================= Instruções para entrar na lista, sair da lista e usar a lista em http://www.mat.puc-rio.br/~nicolau/olimp/obm-l.html O administrador desta lista é ========================================================================= From owner-obm-l@sucuri.mat.puc-rio.br Sun Apr 20 18:34:47 2003 Return-Path: Received: (from majordom@localhost) by sucuri.mat.puc-rio.br (8.9.3/8.9.3) id SAA08656 for obm-l-MTTP; Sun, 20 Apr 2003 18:30:59 -0300 Received: from sina.bol.com.br (sina.bol.com.br [200.221.24.27]) by sucuri.mat.puc-rio.br (8.9.3/8.9.3) with ESMTP id SAA08652 for ; Sun, 20 Apr 2003 18:30:51 -0300 Received: from xx (200.221.24.99) by sina.bol.com.br (5.1.071) id 3E7665FB00987408 for obm-l@mat.puc-rio.br; Sun, 20 Apr 2003 18:30:21 -0300 Message-ID: <001c01c30784$10282610$f4befea9@xx> From: "Blue Ice" To: References: <000f01c30755$a082d810$f4befea9@xx> <200304201517060500.000A8A62@smtp.watersportsbrazil.com> Subject: [obm-l] =?iso-8859-1?Q?Re:_=5Bobm-l=5D_Re:_=5Bobm-l=5D_Re:=5Bobm-l=5D__ques?= =?iso-8859-1?Q?t=E3o_sobre_conjuntos?= Date: Sun, 20 Apr 2003 18:30:26 -0300 MIME-Version: 1.0 Content-Type: text/plain; charset="iso-8859-1" Content-Transfer-Encoding: 8bit X-Priority: 3 X-MSMail-Priority: Normal X-Mailer: Microsoft Outlook Express 6.00.2600.0000 X-MimeOLE: Produced By Microsoft MimeOLE V6.00.2600.0000 X-Sender-IP: 200.148.35.151 Sender: owner-obm-l@sucuri.mat.puc-rio.br Precedence: bulk Reply-To: obm-l@mat.puc-rio.br ----- Original Message ----- From: "Ariel de Silvio" To: Sent: Sunday, April 20, 2003 3:17 PM Subject: Re: [obm-l] Re: [obm-l] Re:[obm-l] questão sobre conjuntos > E-mail Premium BOL > Antivírus, anti-spam e até 100 MB de espaço. Assine já! > http://email.bol.com.br/ > É Ice, Análise Combinatória é matéria de 2o colegial... > > Eu achei mto interessante qdo aprendi, mesmo demorando pra pegar pq perdi algumas aulas... O mais dificil neles é a interpretação do problema, não é como uma equação que ele fala x=2+2 => x=4 > Vc precisa interpretar o problema, e mta gente roda por erro de portugues... heheheh > > Depois q vc achou os valores normalmente são a mesma coisa... > > Não se torture por nao saber isso, na sua idade eu nem imaginava o q era isso... hehehehe > > falou > Ariel Normal,vivendo e aprendendo ;) []´s Ice ICQ:177782914 Email:Vinicius84@hotmail.com Win XP Home Full/Speedy 256kbps ========================================================================= Instruções para entrar na lista, sair da lista e usar a lista em http://www.mat.puc-rio.br/~nicolau/olimp/obm-l.html O administrador desta lista é ========================================================================= From owner-obm-l@sucuri.mat.puc-rio.br Sun Apr 20 18:37:09 2003 Return-Path: Received: (from majordom@localhost) by sucuri.mat.puc-rio.br (8.9.3/8.9.3) id SAA08756 for obm-l-MTTP; Sun, 20 Apr 2003 18:33:21 -0300 Received: from krypton.hosting4u.net (krypton.hosting4u.net [209.15.2.78]) by sucuri.mat.puc-rio.br (8.9.3/8.9.3) with ESMTP id SAA08752 for ; Sun, 20 Apr 2003 18:33:17 -0300 Received: from gargamel (200-158-200-72.dsl.telesp.net.br [200.158.200.72]) by krypton.hosting4u.net (Postfix) with ESMTP id 934DD9F0B3 for ; Sun, 20 Apr 2003 16:32:44 -0500 (CDT) Message-ID: <200304201836210910.00C0FE44@smtp.watersportsbrazil.com> In-Reply-To: References: X-Mailer: Calypso Version 3.30.00.00 (3) Date: Sun, 20 Apr 2003 18:36:21 -0300 From: "Ariel de Silvio" To: obm-l@mat.puc-rio.br Subject: =?ISO-8859-1?Q?Re:_[obm-l]_Re:[obm-l]_quest=E3o_sobre_?= conjuntos Mime-Version: 1.0 Content-Type: text/plain; charset="ISO-8859-1" Content-Transfer-Encoding: 8bit X-MIME-Autoconverted: from quoted-printable to 8bit by sucuri.mat.puc-rio.br id SAA08753 Sender: owner-obm-l@sucuri.mat.puc-rio.br Precedence: bulk Reply-To: obm-l@mat.puc-rio.br Fundamentos de Matematica Elementar eu to usando pra estudar pro ITA... heheheh Deve ser o mesmo, não é do Gelson Iezzi? *********** MENSAGEM ORIGINAL *********** As 16:27 de 20/4/2003 renatinha15a escreveu: >Primeiramente, obrigada Rodrigo pela resposta da questão >e Ice pela tentativa. Mas ainda gostaria de saber se >existe outra forma de resolver essa questão. Digo isso, >pois ela é uma questão do livro Fundamentos de mat. >elementar, e estou estranhando o fato dela está na >matéria de conjuntos. Talves seja somente erro didático >dos autores. De qualquer forma, ficarei grata por >qualquer esclarecimento. > >[]´s > Renatinha > > >__________________________________________________________________________ >Seleção de Softwares UOL. >10 softwares escolhidos pelo UOL para você e sua família. >http://www.uol.com.br/selecao > > >========================================================================= >Instruções para entrar na lista, sair da lista e usar a lista em >http://www.mat.puc-rio.br/~nicolau/olimp/obm-l.html >O administrador desta lista é >========================================================================= ========================================================================= Instruções para entrar na lista, sair da lista e usar a lista em http://www.mat.puc-rio.br/~nicolau/olimp/obm-l.html O administrador desta lista é ========================================================================= From owner-obm-l@sucuri.mat.puc-rio.br Sun Apr 20 20:13:55 2003 Return-Path: Received: (from majordom@localhost) by sucuri.mat.puc-rio.br (8.9.3/8.9.3) id UAA11691 for obm-l-MTTP; Sun, 20 Apr 2003 20:12:10 -0300 Received: from sidney4.bol.com.br (sidney4.bol.com.br [200.221.24.120]) by sucuri.mat.puc-rio.br (8.9.3/8.9.3) with ESMTP id UAA11686 for ; Sun, 20 Apr 2003 20:12:07 -0300 Received: from bol.com.br (200.221.24.128) by sidney4.bol.com.br (5.1.071) id 3E990A6600138E9D for obm-l@mat.puc-rio.br; Sun, 20 Apr 2003 20:11:36 -0300 Date: Sun, 20 Apr 2003 20:11:36 -0300 Message-Id: Subject: RE: [obm-l] L.I MIME-Version: 1.0 Content-Type: text/plain;charset="iso-8859-1" From: "marcio.lis" To: obm-l@mat.puc-rio.br X-XaM3-API-Version: 2.4 R3 ( B4 ) X-SenderIP: 200.151.200.211 Content-Transfer-Encoding: 8bit X-MIME-Autoconverted: from quoted-printable to 8bit by sucuri.mat.puc-rio.br id UAA11687 Sender: owner-obm-l@sucuri.mat.puc-rio.br Precedence: bulk Reply-To: obm-l@mat.puc-rio.br obrigado,pela resposta pois estava intrigado c/ essa questão,agora só fiquei c/ uma dúvida pq ele dava essa sugestão?p/ dificultar? __________________________________________________________________________ Seleção de Softwares UOL. 10 softwares escolhidos pelo UOL para você e sua família. http://www.uol.com.br/selecao ========================================================================= Instruções para entrar na lista, sair da lista e usar a lista em http://www.mat.puc-rio.br/~nicolau/olimp/obm-l.html O administrador desta lista é ========================================================================= From owner-obm-l@sucuri.mat.puc-rio.br Sun Apr 20 20:15:12 2003 Return-Path: Received: (from majordom@localhost) by sucuri.mat.puc-rio.br (8.9.3/8.9.3) id UAA11705 for obm-l-MTTP; Sun, 20 Apr 2003 20:13:55 -0300 Received: from artemis.opendf.com.br (artemis.opengate.com.br [200.181.71.14]) by sucuri.mat.puc-rio.br (8.9.3/8.9.3) with ESMTP id UAA11699 for ; Sun, 20 Apr 2003 20:13:51 -0300 Received: from localhost (localhost [127.0.0.1]) by artemis.opendf.com.br (Postfix) with ESMTP id 4D2B42BECB for ; Sun, 20 Apr 2003 20:14:45 -0300 (BRT) Received: from artemis.opendf.com.br ([127.0.0.1]) by localhost (artemis.opengate.com.br [127.0.0.1:10024]) (amavisd-new) with ESMTP id 04184-07 for ; Sun, 20 Apr 2003 20:14:44 -0300 (BRT) Received: from computer (200-181-090-222.bsace7001.dsl.brasiltelecom.net.br [200.181.90.222]) by artemis.opendf.com.br (Postfix) with ESMTP id 21ED42BEC2 for ; Sun, 20 Apr 2003 20:14:43 -0300 (BRT) From: "Artur Costa Steiner" To: Subject: [obm-l] =?iso-8859-1?Q?RE:_=5Bobm-l=5D_D=FAvida_Sobre_a_Utilidade_da_Matem=E1tica?= =?iso-8859-1?Q?_Ensinada_nas_Escolas?= Date: Sun, 20 Apr 2003 20:13:24 -0300 Organization: Steiner Consultoria LTDA Message-ID: MIME-Version: 1.0 Content-Type: multipart/mixed; boundary="----=_NextPart_000_000C_01C30779.4C06D7E0" X-Priority: 3 (Normal) X-MSMail-Priority: Normal X-Mailer: Microsoft Outlook, Build 10.0.2627 Importance: Normal X-MimeOLE: Produced By Microsoft MimeOLE V6.00.2800.1106 In-Reply-To: <007901c30781$bfa14a20$f3add9c8@iloveusa> X-MS-TNEF-Correlator: 000000005FC3F71D8B4BB941A8AA63D06F261C3684FB2A00 X-Virus-Scanned: by amavisd-new Sender: owner-obm-l@sucuri.mat.puc-rio.br Precedence: bulk Reply-To: obm-l@mat.puc-rio.br This is a multi-part message in MIME format. ------=_NextPart_000_000C_01C30779.4C06D7E0 Content-Type: text/plain; charset="iso-8859-1" Content-Transfer-Encoding: quoted-printable Ol=E1 =A0 Algu=E9m poderia dizer pra que serve conjuntos,an=E1lise combinat=F3ria,bissetriz,equa=E7=E3o do 1=BA=A0e 2=BA grau,produtos not=E1veis,f=F3rmulas(Como a de B=E1skara), entre outros assuntos que = parecem n=E3o fazer parte do mundo real? Existe algum emprego em q essas coisas s=E3o usadas? [Artur Costa Steiner]=20 =A0Huuuummmm... vou tentar mostrar que estas coisas nao sao totalmente inuteis e nem interessam apenas a um bando de desocupados que nao tem = mais o que fazer... Conjuntos - Sao a base de quase toda a matematica. Apresentam vital importancia em ramos da matematica como Analise, Algebra, Calculo. Estes ultimos, embora muita gente nao se de conta, tem aplicacoes na vida = pratica no projeto de coisa hoje triviais como televispes , automoveis, DVDs, computadores, etc. Vc sabe quanto formulas e calculos e mesmo equacoes diferenciais tiveram que ser resolvidas para que vc pudesse acionar um = CD player? Conjuntos e assuntos correlatos como Teoria dos Grafos, sao = tambe utilizados na achamada arquitetura de computadores. Analise combinatoria - em muitos algoritmos que envolvem problemas = discretos (isto eh, baseados em numeros inteiros), em an=E1lises de alternativas = de investimentos, quano h=E1 um numero finito de possibilidaes, no calculo = de probabilidade, tambem relacionado a analise de alternativas.=20 Equacoes de modo geral - Acho que todo mundo jah tentou resolver algo similar a seguinte questao: Tenho R$ 200 e aquele som que quero comprar custa R$300. Logo, preciso arranjar mais $100. Vc se deu conta de que = para isso resoveu a equacao do primeiro grau 200 + x =3D300? E equacoes mais complexas aparecem nos mais diverso campos do conhecimento humano. Sem = ir muito longe, quando um gerente de banco calcula quanto vc vai pagar por = mes sobre um emprestimo que o banco vai dar a vc em, por exemplo 12 meses, a calculadora financeira dele resoveu uma equacao. A formula de Baskara, = alias muito bem bolada e na Idae Media, acho, resove equacoes do segundo grau. Bisstriz - geometria, em geral , eh usada em arquitetura, projeto de maquinas, veiculos, etc.=20 Formulas- em quase tudo se usa algum tipo de formula para calcular = alguma coisa. Diametros de eixos de automoveis, potencia de usinas = hidreletricas, valores financeiros a constar em clausulas contratuais entre grandes empresas, imposto de renda a pagar ou a receber de volta. Toda vez que alguem desenvolve num,a planilha Excel um modelo para a ser aplicado = seja no que for estah usando uma porcao de formulas. Abra uma planilha Excel e = cique naquele icone das formulas. Ha mais de 200 formulas rel;ativas aos mais variados assuntos (finacas , engenharia, estatistica...) E se estao la e porque os clientes da Microsoft acham necessario. Eu trabalho no planejamento de sistemas eletricos para o Brasil. = Utilizamos varios programas que analisam as melhores formas de se implantar usinas = e linhas de transmissao de forma a atender aos requesitos do pais e = minimizar o custo do atendimento. Usamos uma serie de tecnicas matematicas, como programacao dinamica estocastica, decomposicao de Benders, programacoa lineare nao linear que se baseiam em conjuntos , equacoes, formulas, = etc.=20 Eu sou insuspeito para dizer isso - sou engenheiro, nao matematico. Artur=A0 ------=_NextPart_000_000C_01C30779.4C06D7E0 Content-Type: application/ms-tnef; name="winmail.dat" Content-Transfer-Encoding: base64 Content-Disposition: attachment; filename="winmail.dat" eJ8+IhgXAQaQCAAEAAAAAAABAAEAAQeQBgAIAAAA5AQAAAAAAADoAAEIgAcAGAAAAElQTS5NaWNy b3NvZnQgTWFpbC5Ob3RlADEIAQ2ABAACAAAAAgACAAEGAAcAAQAAAAAAAAEGgAMADgAAANMHBAAU ABQADQAAAAAAEwEBA5AGAGAOAAAuAAAACwACAAEAAAALACMAAAAAAAMAJgAAAAAACwApAAAAAAAD AC4AAAAAAAIBMQABAAAAGAAAAAAAAABfw/cdi0u5QaiqY9BvJhw2RPsqAAMANgAAAAAAHgBwAAEA AABEAAAAW29ibS1sXSBE+nZpZGEgU29icmUgYSBVdGlsaWRhZGUgZGEgTWF0ZW3hdGljYSBFbnNp bmFkYSBuYXMgRXNjb2xhcwACAXEAAQAAABYAAAABwweSTF76gRkWylROroRrznDmLjJYAAACAR0M AQAAABkAAABTTVRQOkFSVFVSQE9QRU5ERi5DT00uQlIAAAAACwABDgAAAABAAAYOAObBYpIHwwEC AQoOAQAAABgAAAAAAAAAX8P3HYtLuUGoqmPQbyYcNsKAAAADABQOAAAAAAsAHw4BAAAAHgAoDgEA AAAvAAAAMDAwMDAwMDIBYXJ0dXJAb3BlbmRmLmNvbS5icgFwb3Aub3BlbmRmLmNvbS5icgAAHgAp DgEAAAAvAAAAMDAwMDAwMDIBYXJ0dXJAb3BlbmRmLmNvbS5icgFwb3Aub3BlbmRmLmNvbS5icgAA AgEJEAEAAAASCQAADgkAAEcOAABMWkZ1XpeptgMACgByY3BnMTI14jIDQ3RleAVBAQMB908KgAKk A+MCAGNoCsBz8GV0MCAHEwKAD/MAUH8EVghVB7IRxQ5RAwEQxzL3BgAGwxHFMwRGEMkS2xHT2wjv Cfc7GL8OMDURwgxgzmMAUAsJAWQzNhFQC6bLCuMKgE8YoCdlG5MKgYwnYQFAHaNBbGcMcGEeIDlt IHAEcgcwIEhkaXoTIXByIFBxZQpQIBEgcnYhMAWgbsRqdQIwb3MsAHAeEp5sBAAhkgbQC4BhdB4Q exXAByEsIzAEEBEwBRB6VCxlIRBhHhE3HhEzzm8gYCVgAFAnYiTRHtAXITAOUCXRIAnAYXUsiyDQ BHB1IhEgbm8jcSceMCGABAAsZiODbXXZC2BzKAhQBGAgIFEhMBJCHhJzawrAYSks/iAJ8CRQITAI YCRQJ3Eo8N5zIfMhAwqxBZBlH9AiYvklUWZhIJMKwA6wJXIowC5uJYEYwAdAPx2URXh/BAAtsQdA H3Af0CyAINBl9mclYCyBcSqABBAo8CGhrwQAMPEQsCUzdTDgZCjw9S61WwcQdAhwEsEvUCBQblMO sAuABJBdHYUeskgadTThbTUhHhA4NSD0IHYIYCAOsAIwCsEEYP8vUCDgBcAhEgeQAZAxByNQ3yVg MOAlYCIQAZBsB4ACMP8hMAuAJ1AoISqAJ5AsgQuAzw6wGMAw0R/QYXAJ8DDxPyBQL8El4C4yKZEB AHNv/GN1CrAlgCvUOAIOsB/Q3wDAOYElYCESLSMuPpAeVecdowhQIdUgLQYBKVIl4P8i0SmRJLEi 0SIQMjApYQDA5z1RI2AN4GEuEWAwEREg8zYhH9B2aTihOSAwABhh/QBwYyBBLIEg4DZxIGBB2nsi 8iVgQSNQIrIqcB9RZfZiIOAqcEMHQDxQGFBCkP5FL1EEICjQQlA2cSpxBtH/IOEowENxJrA44zgD QNMhsf8BkCpwPVI6wCKwQnAFoAeRfyNQQ1FBoSDRRaMnoCDBb85qETA7wzEjIGhNYTXwnwUQQ2A9 okXzDrBsZUNg/nM60AQgKnAm4CIQBGAoE7AgRFZESLEjAXAnUCc8gTpRKnF0Y0KQVmP9ODFiQQMi AS0QBbAowzmh/0JwR4M5kgeBKUEkokvDIHD+ZjpBRDI5gUJQIYBEsSEGey5hPDBsTEIEIAqxIPR2 31KwUaA8ESLRANBpAiAKwVkvwUNEH+ALYHkEkD//EsE/pi9xK3YFoRjAC2BcFLkpQVRlBbAgQidx RyDg/wIQSLE4QzqQUvEx8EJQIrB+ejyDTBEA0BDwAMBBonL/IRBDcBEwCHApc1F6PrtGRf8i911z QCA9YklhKzIfYF1x9nRE0jbzbjXAV/AsgScRvwJgQiFVwgTxTYEEICgvQf0wUWgqcECiPIMsgi/A BJD7J3E6EmkrISpiOqEiZ0Tx/y9yOjEjUU7AZyI5EiGAL1DvB3EiA1MTJWBoHhI7QmlUvy0QC4Bk 4SmCH/AEEGkjMP8isDIwUhJNEVRFb1NmwQGg/2/zAQBLAV7SRJFckVmzJYHfQbFjNWt9QpA+ykVV Z1TRvwRwJWBG8CDgAyBAIEEQ4Mc95EGBLfZqYWg18zXR/1e0EyFlMiFAB3ADEArBIFCfESAfcDoS IQIzYW86XUHSbnehUiQmUDARUC9x/yERT6AhQANwIQMhEW7BUXJvNrI8UDNifHAzfLBCkEz+bzBA KnAwEURAPDBgkSDg5yHQNkI9oiQxf2JSojvy/zXgSrNA5CwTIFAEAS5SPDD/IYA14ERhVWI4ESWB INAHcQdqESazfJMrIHggPXt/UVrgRVVIPZNRck+geP87Aiw3J3E9kyBwVtGAUUJw/0PRRPF+Mnww gCFtE05AL8C/bbFCkAZgOfE2UWTSIBhQ/m5G8G10JYEvwXcBOOMpkX87gQWgVDUg8lNDWPFsMGn9 LCFnCsFD4VTifWFHECEw/y/GbOI95CVgjqSQIjIwerL/WPEsgH/hBbEOwC/xcQEOIP9U4lISRdGP FFHhIFBu8UQh/2oBKXJ9QYPWi6GEZkKRU4b/KYMo8CoiUDEisDDxjIRygucG4WBiPPIgSXAhBdAJ gP8j0WACf9GD1FVJSjIfcC4y+ybCPrtCBAEkUkARRvADcP8kQkcxLIF3BCpxeOAyA2pz/2C4f+JN ZwDAYME68SpwKBH3VHNSNT7KRlOlZHNBJVkw/0ozMgEvhUJQH/ApgpkGWGPfjwV545gSMSNCkEQH MKDC80TiNxFpeKwEUFof8DYB/0RCKZEyAKRyTkBMUFyBJEL/QnCkogdAUfKWiCsyIaIzYfuUcR/Q YwtgMgBT00qyTKH/JMA5giqjJsEuMAeRkdSkkv9DwmgSKZFWIUGjkGQ10SBQfyxSUvAFwCmRZkEB kEKQVP9BkiGAoFAhEi+SLIE8EWYlv2lCIkBacgMBEPBH4HgscP8DIC/BBGJxAVhjetJ54Utz/54j eMBNAj4DlGIzYaIDjYTfTHEFsISzqTd1AUFHEb6k+7osIZFpPNR9FA3gAiAtwfsw8b+YSEHSiXIm QXyxU5f9XIE7bAU4EId1bDAHITyS/StnKJaSr4Ghwo0BfDDGwr8qcTNhQlBs4UJxPpAphtH/ItF7 o4zQm5JD4ZKDMQEisNs44kTzTQ3gKyFvAYBgBP85wSxwMNEFEEfAdTs14SDg/yXgumAlYE0SuiG8 8Ys0KZF/AJAvUWcSryUncVhjJWBC6yDgAJBsQpBVX0RE0say/9ICA2AmwWcSuFNGUzqSh3H/T5BO ULATU6FsRCLRQ8G6If82Mq51ITAisMjBa1M2oQCA/3pwMNGpB0GyQgGz0XnhJ3H/GMB7gmTjhPKz Ay4AbwF6cP9fgLYxfsMlY9qDbQTTATqB/ydxmBIhUQiQ2NMFkAMAyCL/QfhRRE0j1HOEsyNBenBF wf83MTxAKPBCUipwBYKKQkJh3TvDQtqjrYLhtm8gUNhx/y6AKsE4AuYUIQVAk6uhsaP/P5cqcVVl KnBTlqVfzvM8MP814AuAK4BP4W8iWGMgdIOD/0Ag66LIhGoCcGE4EUH3zjYXMuMeuB2UffFgAAAe AEIQAQAAACoAAAA8MDA3OTAxYzMwNzgxJGJmYTE0YTIwJGYzYWRkOWM4QGlsb3ZldXNhPgAAAAMA khABAAAAAgEUOgEAAAAQAAAAVSW0qgPy0EWJ8k4lYd9ZAgMA3j+vbwAAAwAJWQEAAAADAEBlAAAA AAsAE4AIIAYAAAAAAMAAAAAAAABGAAAAAAOFAAAAAAAAAwAVgAggBgAAAAAAwAAAAAAAAEYAAAAA EIUAAAAAAAADABuACCAGAAAAAADAAAAAAAAARgAAAABShQAA45ABAAMAIoAIIAYAAAAAAMAAAAAA AABGAAAAAAGFAAAAAAAAQAAjgAggBgAAAAAAwAAAAAAAAEYAAAAAYIUAAADQiMMQAAAAHgBBgAgg BgAAAAAAwAAAAAAAAEYAAAAAVIUAAAEAAAAFAAAAMTAuMAAAAAALAEKACCAGAAAAAADAAAAAAAAA RgAAAAAGhQAAAAAAAAsARoAIIAYAAAAAAMAAAAAAAABGAAAAAA6FAAAAAAAAAwBJgAggBgAAAAAA wAAAAAAAAEYAAAAAGIUAAAAAAAALAF6ACCAGAAAAAADAAAAAAAAARgAAAACChQAAAQAAAAIB+A8B AAAAEAAAAF/D9x2LS7lBqKpj0G8mHDYCAfoPAQAAABAAAABfw/cdi0u5QaiqY9BvJhw2AgH7DwEA AACEAAAAAAAAADihuxAF5RAaobsIACsqVsIAAG1zcHN0LmRsbAAAAAAATklUQfm/uAEAqgA32W4A AABDOlxEb2N1bWVudHMgYW5kIFNldHRpbmdzXEFydHVyXEFwcGxpY2F0aW9uIERhdGFcTWljcm9z b2Z0XE91dGxvb2tcT3V0bG9vay5wc3QAAwD+DwUAAAADAA00/TcCAAIBFDQBAAAAEAAAAE5JVEH5 v7gBAKoAN9luAAACAX8AAQAAADEAAAAwMDAwMDAwMDVGQzNGNzFEOEI0QkI5NDFBOEFBNjNEMDZG MjYxQzM2ODRGQjJBMDAAAAAAAwAGEEPHKsoDAAcQZQoAAAMAEBAAAAAAAwAREAAAAAAeAAgQAQAA AGUAAABPTOGgQUxHVelNUE9ERVJJQURJWkVSUFJBUVVFU0VSVkVDT05KVU5UT1MsQU7hTElTRUNP TUJJTkFU81JJQSxCSVNTRVRSSVosRVFVQefjT0RPMbqgRTK6R1JBVSxQUk9EVVRPAAAAADd4 ------=_NextPart_000_000C_01C30779.4C06D7E0-- ========================================================================= Instruções para entrar na lista, sair da lista e usar a lista em http://www.mat.puc-rio.br/~nicolau/olimp/obm-l.html O administrador desta lista é ========================================================================= From owner-obm-l@sucuri.mat.puc-rio.br Sun Apr 20 21:00:52 2003 Return-Path: Received: (from majordom@localhost) by sucuri.mat.puc-rio.br (8.9.3/8.9.3) id UAA13079 for obm-l-MTTP; Sun, 20 Apr 2003 20:59:09 -0300 Received: from artemis.opendf.com.br (artemis.opengate.com.br [200.181.71.14]) by sucuri.mat.puc-rio.br (8.9.3/8.9.3) with ESMTP id UAA13075 for ; Sun, 20 Apr 2003 20:59:05 -0300 Received: from localhost (localhost [127.0.0.1]) by artemis.opendf.com.br (Postfix) with ESMTP id E43182BEC8 for ; Sun, 20 Apr 2003 20:59:53 -0300 (BRT) Received: from artemis.opendf.com.br ([127.0.0.1]) by localhost (artemis.opengate.com.br [127.0.0.1:10024]) (amavisd-new) with ESMTP id 05650-09 for ; Sun, 20 Apr 2003 20:59:53 -0300 (BRT) Received: from computer (200-181-090-222.bsace7001.dsl.brasiltelecom.net.br [200.181.90.222]) by artemis.opendf.com.br (Postfix) with ESMTP id 99F972BEC2 for ; Sun, 20 Apr 2003 20:59:52 -0300 (BRT) From: "Artur Costa Steiner" To: Subject: RE: [obm-l] L.I Date: Sun, 20 Apr 2003 20:58:32 -0300 Organization: Steiner Consultoria LTDA Message-ID: <001501c30798$c00d0cd0$9865fea9@computer> MIME-Version: 1.0 Content-Type: text/plain; charset="iso-8859-1" X-Priority: 3 (Normal) X-MSMail-Priority: Normal X-Mailer: Microsoft Outlook, Build 10.0.2627 Importance: Normal X-MimeOLE: Produced By Microsoft MimeOLE V6.00.2800.1106 In-Reply-To: X-Virus-Scanned: by amavisd-new Content-Transfer-Encoding: 8bit X-MIME-Autoconverted: from quoted-printable to 8bit by sucuri.mat.puc-rio.br id UAA13076 Sender: owner-obm-l@sucuri.mat.puc-rio.br Precedence: bulk Reply-To: obm-l@mat.puc-rio.br > > obrigado,pela resposta pois estava intrigado c/ essa >questão,agora só fiquei c/ uma dúvida pq ele dava essa >sugestão?p/ dificultar? [Artur Costa Steiner] Nao, nao creio. Ele provavelmente acha a outra solucao mais bonita ou mais pratica. Artur ========================================================================= Instruções para entrar na lista, sair da lista e usar a lista em http://www.mat.puc-rio.br/~nicolau/olimp/obm-l.html O administrador desta lista é ========================================================================= From owner-obm-l@sucuri.mat.puc-rio.br Sun Apr 20 21:27:11 2003 Return-Path: Received: (from majordom@localhost) by sucuri.mat.puc-rio.br (8.9.3/8.9.3) id VAA13775 for obm-l-MTTP; Sun, 20 Apr 2003 21:25:11 -0300 Received: from pop2 (pop2.directnet.com.br [200.152.0.15]) by sucuri.mat.puc-rio.br (8.9.3/8.9.3) with ESMTP id VAA13771 for ; Sun, 20 Apr 2003 21:24:58 -0300 Received: from Home ([200.152.14.165]) by pop2.directnet.com.br (iPlanet Messaging Server 5.2 HotFix 1.08 (built Dec 6 2002)) with SMTP id <0HDO0037C3RNLP@pop2.directnet.com.br> for obm-l@mat.puc-rio.br; Sun, 20 Apr 2003 21:23:48 -0300 (EST) Date: Sun, 20 Apr 2003 21:30:15 -0300 From: Daniel Subject: [obm-l] =?iso-8859-1?Q?Re:_=5Bobm-l=5D_D=FAvida_Sobre_a_Utilidade_da_Matem=E1tica?= =?iso-8859-1?Q?_Ensinada_nas_Escolas?= To: obm-l@mat.puc-rio.br Message-id: <005001c3079d$2e9ce540$0100a8c0@directnet.com.br> MIME-version: 1.0 X-MIMEOLE: Produced By Microsoft MimeOLE V6.00.2800.1106 X-Mailer: Microsoft Outlook Express 6.00.2800.1106 Content-type: multipart/alternative; boundary="Boundary_(ID_kDE3LxGxF0i/N10O6E5r1Q)" X-Priority: 3 X-MSMail-priority: Normal References: <007901c30781$bfa14a20$f3add9c8@iloveusa> Sender: owner-obm-l@sucuri.mat.puc-rio.br Precedence: bulk Reply-To: obm-l@mat.puc-rio.br This is a multi-part message in MIME format. --Boundary_(ID_kDE3LxGxF0i/N10O6E5r1Q) Content-type: text/plain; charset=iso-8859-1 Content-transfer-encoding: quoted-printable Jo=E3o Paulo, A Matem=E1tica =E9 uma Ci=EAncia e como tal possui uma = seq=FC=EAncia em que os fatos foram descobertos e uma seq=FC=EAncia = did=E1tica para ensin=E1-los. Tudo bem, nem todas as pessoas da face da terra ir=E3o trabalhar = com Matem=E1tica Superiror, mas a base, justamente o que =E9 ensinado = nas escolas isso todas ir=E3o usar, e as que n=E3o usarem ter=E3o = s=E9rias dificuldades em arrumarem empregos ou progredir em sua = carreira, vamos a alguns exemplos leves: Conjuntos: O respons=E1vel por organizar dados do censo, com certeza ele = trabalha constantemente com conjuntos de pessoas, separando em = subconjuntos para tornar a an=E1lise dos dados mais f=E1cil. Analise Combinat=F3ria: A arte de contar sem contar; Imagine o = respons=E1vel pelo estoque de uma loja de roupas (camisas, cal=E7as, = meias..) que deve saber extamente quantos conjuntos de roupas diferente = ele possui, para organizar as vendas, se ele n=E3o souber t=E9cnicas de = contagem, as coisas v=E3o ficar bem trabalhosas e muiiiito demoradas, = n=E3o se esque=E7a tempo =E9 dinheiro!! Bissetriz: Tra=E7ados geom=E9tricos s=E3o usados por Arquitetos e = Engenheiros o tempo todo para construir pr=E9dios, casas, etc... Equa=E7=E3o do 1=BA: Um carro se move com velocidade constante e voc=EA = deja saber em quantas horas chegar=E1 a seu destino, conhecida a = dist=E2ncia, sem coment=E1rios... Equa=E7=E3o do 2=BA: Uma sala retangular deve ser constru=EDda de modo = que ela possua um per=EDmetro fixo, pois n=E3o se disp=F5e de cimento = indefinido para fazer o muro, al=E9m do que a =E1rea n=E3o pode passar = de determinado valor, pois h=E1 restri=E7=E3o de espa=E7o f=EDsico, = voc=EA acha que se usar=E1 uma equa=E7=E3o de que grau para construir = esta sala?=20 F=F3rmula de B=E1skara: Como voc=EA acha que a equa=E7=E3o acima =E9 = rapidamente resolvida?!! Produtos Not=E1veis: O dono de uma grande loja que est=E1 negociando o = pre=E7o de mercadorias, e precisa comprar 99 pe=E7as por 101 reais, como = ele est=E1 no meio de uma negocia=E7=E3o n=E3o vai ficar fazendo contas = na calculadora, rapidamente e faz (100 + 1) (100 - 1) =3D 100^2 - 1 =3D = 9.999. Agilidade nos c=E1lculos =E9 muito importante, em qualquer area. Isso tudo =E9 apenas uma pontinha do Ice Berg, a Matem=E1tica est=E1 = presente em todos os componentes eletr=F4nicos (video, tv, som, = computadores...), Mec=E2nicos (carros, motos, m=E1quinas...), na = constru=E7=E3o civil, no mercado econ=F4mico e em muitas outras areas. Eu por exemplo estou estudando Eng. Aeron=E1utica, se voc=EA visse a = quantidade de equa=E7=F5es matem=E1ticas que devem ser resolvidas para = botar um avi=E3o no ar, de modo que ele possua baixo custo de = produ=E7=E3o, opera=E7=E3o e manuten=E7=E3o, sem que caia causando = acidentes perigosos e fatais, ficaria imprecionado. Tenha certeza, a Matem=E1tica ensinada nas escolas al=E9m de ajudar = muito as pessoas que n=E3o ir=E3o trabalhar com isso, d=E3o a base = necess=E1ria para Engenheiros, Cientistas e Pesquisadores, encontrarem = solu=E7=F5es para problemas bem reais!!!! Daniel --Boundary_(ID_kDE3LxGxF0i/N10O6E5r1Q) Content-type: text/html; charset=iso-8859-1 Content-transfer-encoding: quoted-printable

Jo=E3o Paulo,
 
        A=20 Matem=E1tica =E9 uma Ci=EAncia e como tal possui uma seq=FC=EAncia em = que os fatos foram=20 descobertos e uma seq=FC=EAncia did=E1tica para = ensin=E1-los.
    =     Tudo bem,=20 nem todas as pessoas da face da terra ir=E3o trabalhar com Matem=E1tica = Superiror,=20 mas a base, justamente o que =E9 ensinado nas escolas isso todas ir=E3o = usar, e as=20 que n=E3o usarem ter=E3o s=E9rias dificuldades em arrumarem empregos ou = progredir em=20 sua carreira, vamos a alguns exemplos leves:
 
Conjuntos: O respons=E1vel por = organizar dados do=20 censo, com certeza ele trabalha constantemente com conjuntos de pessoas, = separando em subconjuntos para tornar a an=E1lise dos dados mais=20 f=E1cil.
Analise Combinat=F3ria: A arte de = contar sem=20 contar; Imagine o respons=E1vel pelo estoque de uma loja de roupas = (camisas,=20 cal=E7as, meias..) que deve saber extamente quantos conjuntos de roupas = diferente=20 ele possui, para organizar as vendas, se ele n=E3o souber t=E9cnicas de = contagem, as=20 coisas v=E3o ficar bem trabalhosas e muiiiito demoradas, n=E3o se = esque=E7a tempo =E9=20 dinheiro!!
Bissetriz: Tra=E7ados geom=E9tricos = s=E3o usados por=20 Arquitetos e Engenheiros o tempo todo para construir pr=E9dios, casas,=20 etc...
Equa=E7=E3o do 1=BA: Um carro se = move com velocidade=20 constante e voc=EA deja saber em quantas horas chegar=E1 a seu destino, = conhecida a=20 dist=E2ncia, sem coment=E1rios...
Equa=E7=E3o do 2=BA: Uma sala=20 retangular  deve ser constru=EDda de modo que ela possua um = per=EDmetro=20 fixo, pois n=E3o se disp=F5e de cimento indefinido para fazer o = muro, al=E9m do=20 que a =E1rea n=E3o pode passar de determinado valor, pois h=E1 = restri=E7=E3o de=20 espa=E7o f=EDsico, voc=EA acha que se usar=E1 uma equa=E7=E3o de = que grau=20 para construir esta sala? 
F=F3rmula de B=E1skara: Como voc=EA = acha que a equa=E7=E3o=20 acima =E9 rapidamente resolvida?!!
Produtos Not=E1veis: O dono de uma = grande loja que=20 est=E1 negociando o pre=E7o de mercadorias, e precisa comprar 99 pe=E7as = por 101=20 reais, como ele est=E1 no meio de uma negocia=E7=E3o n=E3o vai ficar = fazendo contas na=20 calculadora, rapidamente e faz (100 + 1) (100 - 1) =3D 100^2 - 1 =3D = 9.999.=20 Agilidade nos c=E1lculos =E9 muito importante, em qualquer = area.
 
Isso tudo =E9 apenas uma pontinha do = Ice Berg, a=20 Matem=E1tica est=E1 presente em todos os componentes eletr=F4nicos = (video, tv, som,=20 computadores...), Mec=E2nicos (carros, motos, m=E1quinas...), na = constru=E7=E3o civil,=20 no mercado econ=F4mico e em muitas outras areas.
Eu por exemplo estou estudando Eng. = Aeron=E1utica,=20 se voc=EA visse a quantidade de equa=E7=F5es matem=E1ticas que devem ser = resolvidas para=20 botar um avi=E3o no ar, de modo que ele possua baixo custo de = produ=E7=E3o,=20 opera=E7=E3o e manuten=E7=E3o, sem que caia causando acidentes perigosos = e fatais,=20 ficaria imprecionado.
Tenha certeza, a Matem=E1tica = ensinada nas escolas=20 al=E9m de ajudar muito as pessoas que n=E3o ir=E3o trabalhar com isso, = d=E3o a base=20 necess=E1ria para Engenheiros, Cientistas e Pesquisadores, encontrarem = solu=E7=F5es=20 para problemas bem reais!!!!
 
Daniel
 
 
 
--Boundary_(ID_kDE3LxGxF0i/N10O6E5r1Q)-- ========================================================================= Instruções para entrar na lista, sair da lista e usar a lista em http://www.mat.puc-rio.br/~nicolau/olimp/obm-l.html O administrador desta lista é ========================================================================= From owner-obm-l@sucuri.mat.puc-rio.br Sun Apr 20 21:39:02 2003 Return-Path: Received: (from majordom@localhost) by sucuri.mat.puc-rio.br (8.9.3/8.9.3) id VAA14157 for obm-l-MTTP; Sun, 20 Apr 2003 21:37:01 -0300 Received: from mercury.antares.com.br (mercury.antares.com.br [200.255.168.252]) by sucuri.mat.puc-rio.br (8.9.3/8.9.3) with ESMTP id VAA14153 for ; Sun, 20 Apr 2003 21:36:53 -0300 Received: from exellon.antares.com.br (exellon.antares.com.br [200.255.168.11]) by mercury.antares.com.br (8.11.6/linuxconf) with SMTP id h3L0aFL21899 for ; Sun, 20 Apr 2003 21:36:15 -0300 Received: from [200.255.168.104] (helo=fabio) by exellon.antares.com.br with smtp (Exim 3.35 #1 (Debian)) id 197PIa-0003tM-00 for ; Sun, 20 Apr 2003 21:36:45 -0300 Message-ID: <002f01c3079e$503fede0$68a8ffc8@fabio> From: =?iso-8859-1?Q?F=E1bio_Nunes_Ribeiro_Maia?= To: =?iso-8859-1?Q?Lista_de_discuss=E3o_de_Matem=E1tica?= References: Subject: [obm-l] =?iso-8859-1?Q?Re:_=5Bobm-l=5D_quest=E3o_sobre_conjuntos?= Date: Sun, 20 Apr 2003 21:33:19 -0300 Organization: Home Office MIME-Version: 1.0 Content-Type: text/plain; charset="iso-8859-1" Content-Transfer-Encoding: 8bit X-Priority: 3 X-MSMail-Priority: Normal X-Mailer: Microsoft Outlook Express 6.00.2600.0000 X-MimeOLE: Produced By Microsoft MimeOLE V6.00.2600.0000 X-Envelope-To: obm-l@mat.puc-rio.br Sender: owner-obm-l@sucuri.mat.puc-rio.br Precedence: bulk Reply-To: obm-l@mat.puc-rio.br Primeiramente, olá pessoal da lista, e em especial pra Renatinha. É a primeira vez que escrevo pra esta lista, apesar de já estar acompanhando-a a pelo menos 6 meses. Eu observei que, nas várias soluções que enviaram ao problema, nenhum de vocês deve ter reparado na observação que nossa amiga escreveu logo após: P.S: "Gostaria, se possível, que a solução não tivesse probabilidade ou combinatória, pois ainda faço 1º ano do ensino médio." Como também estou no primeiro ano do EM e também tive que resolver essa questão, fiz isso da seguinte maneira: Se um conjunto qualquer possui n elementos, o número de subconjuntos é dado pela fórmula 2^n. Assim, um conjunto vazio possui 1 subconjunto, também vazio; um unitário possui 2 subconjuntos (um vazio e um unitário); um de 2 possui 4 subconjuntos (um vazio, 2 unitários e um de 2 elementos); um outro de 3 possui 8 subconjuntos (um vazio, 3 unitários, 3 de 2 elementos e um de 3 elementos) e assim sucessivamente. Assim, pude construir a seguinte pirâmide (se eu não me engano ela é usada pra resolver um outro tipo de problema, e seu nome tem a ver com Newton, um amigo me disse isso): 1 1 1 1 2 1 1 3 3 1 1 4 6 4 1 1 5 10 10 5 1 1 6 15 20 15 6 1 1 7 21 35 35 21 7 1 1 8 28 56 70 56 28 8 1 (...) O processo de formação dessa pirâmide é o seguinte: começando com a segunda linha, a cada linha depois dela deve-se adicionar o número um nas extremidades, e embaixo de cada espaço da linha anterior escreve-se a soma dos números que se encontram ao lado dele. Nela, cada linha se refere a um conjunto com um número de elementos que é de 0 (conjunto vazio) na primeira linha, e assim prossegue até o infinito, e os números de cada linha indicam quantos subconjuntos existem com cada número de elementos do conjunto original. Assim: Conjunto com 3 elementos (quarta linha): 1 conjunto vazio 3 " unitário 3 " com 2 elementos 1 " com 3 elementos Conjunto com 8 elementos (nona linha): 1 conjunto vazio 8 " unitário 28 " com 2 elementos 56 " com 3 elementos 70 " com 4 elementos 56 " com 5 elementos 28 " com 6 elementos 8 " com 7 elementos 1 " com 8 elementos Assim, fica fácil de perceber qual a resposta da questão: O conjunto A, de 8 elementos, possui 56 subconjuntos com 5 elementos distintos cada um. P.S.: Desculpe se a resposta foi demorada, mas foi a maneira mais fácil que consegui elaborar para resolver essa questão com a matéria que nos foi dada a mim e à Renatinha até agora. Espero que eu tenha podido ser útil. ----- Original Message ----- From: "renatinha15a" To: "obm" Sent: Sunday, April 20, 2003 10:40 AM Subject: [obm-l] questão sobre conjuntos > Oi pessoal da lista, não consegui fazer esta questão, se > alguém puder me ajudar, ficarei bastante grata. > > (U.F. VIÇOSA-89) Um conjunto A tem 8 elementos > distintos. O número de subconjuntos de A, com 5 > elementos distintos cada um, é: > > P.S: Gostaria, se possível, que a solução não tivesse > probabilidade ou combinatória, pois ainda faço 1º ano do > ensino médio. > > []´s > Renatinha > > > __________________________________________________________________________ > Seleção de Softwares UOL. > 10 softwares escolhidos pelo UOL para você e sua família. > http://www.uol.com.br/selecao > > > ========================================================================= > Instruções para entrar na lista, sair da lista e usar a lista em > http://www.mat.puc-rio.br/~nicolau/olimp/obm-l.html > O administrador desta lista é > ========================================================================= ========================================================================= Instruções para entrar na lista, sair da lista e usar a lista em http://www.mat.puc-rio.br/~nicolau/olimp/obm-l.html O administrador desta lista é ========================================================================= From owner-obm-l@sucuri.mat.puc-rio.br Sun Apr 20 21:51:03 2003 Return-Path: Received: (from majordom@localhost) by sucuri.mat.puc-rio.br (8.9.3/8.9.3) id VAA14531 for obm-l-MTTP; Sun, 20 Apr 2003 21:49:19 -0300 Received: from mercury.antares.com.br (mercury.antares.com.br [200.255.168.252]) by sucuri.mat.puc-rio.br (8.9.3/8.9.3) with ESMTP id VAA14527 for ; Sun, 20 Apr 2003 21:49:11 -0300 Received: from exellon.antares.com.br (exellon.antares.com.br [200.255.168.11]) by mercury.antares.com.br (8.11.6/linuxconf) with SMTP id h3L0mXL22654 for ; Sun, 20 Apr 2003 21:48:33 -0300 Received: from [200.255.168.104] (helo=fabio) by exellon.antares.com.br with smtp (Exim 3.35 #1 (Debian)) id 197PUT-0004Ct-00 for ; Sun, 20 Apr 2003 21:49:02 -0300 Message-ID: <003b01c307a0$078f4ee0$68a8ffc8@fabio> From: =?iso-8859-1?Q?F=E1bio_Nunes_Ribeiro_Maia?= To: References: <001501c305ea$3b3d38e0$f4befea9@xx> Subject: [obm-l] =?iso-8859-1?Q?Re:_=5Bobm-l=5D_Quando_ser=E1_a_olimp=EDada_Internacio?= =?iso-8859-1?Q?nal=3F=3F?= Date: Sun, 20 Apr 2003 21:50:28 -0300 Organization: Home Office MIME-Version: 1.0 Content-Type: multipart/alternative; boundary="----=_NextPart_000_0038_01C30786.DB6A6160" X-Priority: 3 X-MSMail-Priority: Normal X-Mailer: Microsoft Outlook Express 6.00.2600.0000 X-MimeOLE: Produced By Microsoft MimeOLE V6.00.2600.0000 X-Envelope-To: obm-l@mat.puc-rio.br Sender: owner-obm-l@sucuri.mat.puc-rio.br Precedence: bulk Reply-To: obm-l@mat.puc-rio.br This is a multi-part message in MIME format. ------=_NextPart_000_0038_01C30786.DB6A6160 Content-Type: text/plain; charset="iso-8859-1" Content-Transfer-Encoding: quoted-printable Este ano a 44a. IMO ser=E1 realizada nos dias 07 a 19 de julho no = Jap=E3o. Mais informa=E7=F5es nos site da OBM = http://www.obm.org.br/imo.htm, da IMO http://imo.math.ca/, ou no site = oficial do evento: http://www.imo2003.com/=20 Um abra=E7o, =20 F=E1bio Maia ----- Original Message -----=20 From: Blue Ice=20 To: obm-l@mat.puc-rio.br=20 Sent: Friday, April 18, 2003 5:36 PM Subject: [obm-l] Quando ser=E1 a olimp=EDada Internacional?? ------=_NextPart_000_0038_01C30786.DB6A6160 Content-Type: text/html; charset="iso-8859-1" Content-Transfer-Encoding: quoted-printable
Este ano a 44a. IMO ser=E1 realizada nos dias 07 a 19 de = julho no Jap=E3o. Mais informa=E7=F5es nos site da OBM http://www.obm.org.br/imo.htm,=  da=20 IMO http://imo.math.ca/, ou no site = oficial do=20 evento: http://www.imo2003.com/ 
 
Um abra=E7o,    =
F=E1bio Maia
----- Original Message -----
From:=20 Blue=20 Ice
Sent: Friday, April 18, 2003 = 5:36=20 PM
Subject: [obm-l] Quando ser=E1 = a olimp=EDada=20 Internacional??

 
------=_NextPart_000_0038_01C30786.DB6A6160-- ========================================================================= Instruções para entrar na lista, sair da lista e usar a lista em http://www.mat.puc-rio.br/~nicolau/olimp/obm-l.html O administrador desta lista é ========================================================================= From owner-obm-l@sucuri.mat.puc-rio.br Sun Apr 20 22:52:18 2003 Return-Path: Received: (from majordom@localhost) by sucuri.mat.puc-rio.br (8.9.3/8.9.3) id WAA16089 for obm-l-MTTP; Sun, 20 Apr 2003 22:50:26 -0300 Received: from mail1c.webmessenger.it (mail2.webmessenger.it [193.70.193.55]) by sucuri.mat.puc-rio.br (8.9.3/8.9.3) with ESMTP id WAA16078 for ; Sun, 20 Apr 2003 22:50:21 -0300 Received: from iloveusa (200.217.173.243) by mail1c.webmessenger.it (6.7.016) (authenticated as roxette_lover@email.it) id 3E8868D400A24BA3 for obm-l@mat.puc-rio.br; Mon, 21 Apr 2003 03:49:31 +0200 Message-ID: <016b01c307a8$6635ae00$f3add9c8@iloveusa> From: "=?iso-8859-1?Q?J.Paulo_roxer_=B4til_the_end?=" To: References: Subject: [obm-l] =?iso-8859-1?Q?Re:_RE:D=FAvida_Sobre_a_Utilidade_da_Matem=E1tica_Ensinada?= =?iso-8859-1?Q?_nas_Escolas?= Date: Sun, 20 Apr 2003 22:47:02 -0300 MIME-Version: 1.0 Content-Type: text/plain; charset="iso-8859-1" Content-Transfer-Encoding: 8bit X-Priority: 3 X-MSMail-Priority: Normal X-Mailer: Microsoft Outlook Express 6.00.2800.1106 X-MimeOLE: Produced By Microsoft MimeOLE V6.00.2800.1106 Sender: owner-obm-l@sucuri.mat.puc-rio.br Precedence: bulk Reply-To: obm-l@mat.puc-rio.br Sei q é uma pergunta meio esquisita,mas nunca entendi pra q isso servia.Pelo menos nunca vi ninguém fazendo esses cálculos nem usando coisas como x-10= etc etc. Pelo menos deveriam não colocar coisas q a pessoa não vai utilizar.Estou no pré vestibular e tenho muita dificuldade pra entender questões matemáticas.Algumas são propositalmente feitas pra confundir o candidato a um curso universitário. Aproveitando,vc ou outra pessoa pode dizer se a seguinte questão está bem formulada? (PUC-MG)Em uma empresa,60% dos funcionários lêem a revista A,80 % lêem a revista B e todo funcionário é leitor de pelo menos uma dessas revistas.O percentual de funcionários que lêem as duas revistas é: Segundo a resolução feita pelo meu prof.: 60-x+x+80-x=100 140-x=100 -x=100-140 -x= -40.(-1) x= 40 Não seria melhor ir direto ao ponto e fazer 60+ 80=100 140-100 140-100=40? E minha principal dúvida: 100% não é um todo?Se é,como pode uma empresa dividir os funcionários em 60% q lêem a revista A e 80% q lêem a B? (80+60=140%) Pelo q entendi,deveria ser um número como por exemplo,40% q lêem a revista A e 60% q lêem a revista B(40+60=100.O máximo,100%) Bem,sei q estou errado nessas coisas,por isso,fica aqui a solicitação de ajuda aos magos da matemática :) Obrigado desde já _________________________ João Paulo Cisne http://www.roxette.se http://www.roxette.nl http://runto.roxette.org http://www.dailyroxette.com _________________________ ----- Original Message ----- From: "Artur Costa Steiner" To: Sent: Sunday, April 20, 2003 8:13 PM Subject: [obm-l] RE: [obm-l] Dúvida Sobre a Utilidade da Matemática Ensinada nas Escolas Olá Alguém poderia dizer pra que serve conjuntos,análise combinatória,bissetriz,equação do 1º e 2º grau,produtos notáveis,fórmulas(Como a de Báskara), entre outros assuntos que parecem não fazer parte do mundo real? Existe algum emprego em q essas coisas são usadas? [Artur Costa Steiner] Huuuummmm... vou tentar mostrar que estas coisas nao sao totalmente inuteis e nem interessam apenas a um bando de desocupados que nao tem mais o que fazer... Conjuntos - Sao a base de quase toda a matematica. Apresentam vital importancia em ramos da matematica como Analise, Algebra, Calculo. Estes ultimos, embora muita gente nao se de conta, tem aplicacoes na vida pratica no projeto de coisa hoje triviais como televispes , automoveis, DVDs, computadores, etc. Vc sabe quanto formulas e calculos e mesmo equacoes diferenciais tiveram que ser resolvidas para que vc pudesse acionar um CD player? Conjuntos e assuntos correlatos como Teoria dos Grafos, sao tambe utilizados na achamada arquitetura de computadores. Analise combinatoria - em muitos algoritmos que envolvem problemas discretos (isto eh, baseados em numeros inteiros), em análises de alternativas de investimentos, quano há um numero finito de possibilidaes, no calculo de probabilidade, tambem relacionado a analise de alternativas. Equacoes de modo geral - Acho que todo mundo jah tentou resolver algo similar a seguinte questao: Tenho R$ 200 e aquele som que quero comprar custa R$300. Logo, preciso arranjar mais $100. Vc se deu conta de que para isso resoveu a equacao do primeiro grau 200 + x =300? E equacoes mais complexas aparecem nos mais diverso campos do conhecimento humano. Sem ir muito longe, quando um gerente de banco calcula quanto vc vai pagar por mes sobre um emprestimo que o banco vai dar a vc em, por exemplo 12 meses, a calculadora financeira dele resoveu uma equacao. A formula de Baskara, alias muito bem bolada e na Idae Media, acho, resove equacoes do segundo grau. Bisstriz - geometria, em geral , eh usada em arquitetura, projeto de maquinas, veiculos, etc. Formulas- em quase tudo se usa algum tipo de formula para calcular alguma coisa. Diametros de eixos de automoveis, potencia de usinas hidreletricas, valores financeiros a constar em clausulas contratuais entre grandes empresas, imposto de renda a pagar ou a receber de volta. Toda vez que alguem desenvolve num,a planilha Excel um modelo para a ser aplicado seja no que for estah usando uma porcao de formulas. Abra uma planilha Excel e cique naquele icone das formulas. Ha mais de 200 formulas rel;ativas aos mais variados assuntos (finacas , engenharia, estatistica...) E se estao la e porque os clientes da Microsoft acham necessario. Eu trabalho no planejamento de sistemas eletricos para o Brasil. Utilizamos varios programas que analisam as melhores formas de se implantar usinas e linhas de transmissao de forma a atender aos requesitos do pais e minimizar o custo do atendimento. Usamos uma serie de tecnicas matematicas, como programacao dinamica estocastica, decomposicao de Benders, programacoa lineare nao linear que se baseiam em conjuntos , equacoes, formulas, etc. Eu sou insuspeito para dizer isso - sou engenheiro, nao matematico. Artur -- Email.it, the professional e-mail, gratis per te: http://www.email.it/f Sponsor: Vuoi essere alla moda a prezzi contenuti? Miotti.it - sunglasses and more! Clicca qui: http://adv.email.it/cgi-bin/foclick.cgi?mid=1455&d=21-4 ========================================================================= Instruções para entrar na lista, sair da lista e usar a lista em http://www.mat.puc-rio.br/~nicolau/olimp/obm-l.html O administrador desta lista é ========================================================================= From owner-obm-l@sucuri.mat.puc-rio.br Sun Apr 20 23:21:23 2003 Return-Path: Received: (from majordom@localhost) by sucuri.mat.puc-rio.br (8.9.3/8.9.3) id XAA16807 for obm-l-MTTP; Sun, 20 Apr 2003 23:19:57 -0300 Received: from traven9.uol.com.br (traven9.uol.com.br [200.221.29.35]) by sucuri.mat.puc-rio.br (8.9.3/8.9.3) with ESMTP id XAA16802 for ; Sun, 20 Apr 2003 23:19:54 -0300 Received: from gauss ([200.158.96.55]) by traven9.uol.com.br (8.9.1/8.9.1) with SMTP id XAA28173 for ; Sun, 20 Apr 2003 23:19:23 -0300 (BRT) Message-ID: <010401c307ad$1cd16f10$37609ec8@gauss> From: "Domingos Jr." To: Subject: [obm-l] =?iso-8859-1?Q?Crian=E7as_e_doces_roubados?= Date: Sun, 20 Apr 2003 23:24:18 -0300 MIME-Version: 1.0 Content-Type: text/plain; charset="iso-8859-1" Content-Transfer-Encoding: 8bit X-Priority: 3 X-MSMail-Priority: Normal X-Mailer: Microsoft Outlook Express 6.00.2800.1106 X-MimeOLE: Produced By Microsoft MimeOLE V6.00.2800.1106 Sender: owner-obm-l@sucuri.mat.puc-rio.br Precedence: bulk Reply-To: obm-l@mat.puc-rio.br Aqui vai a minha solução para o problema: http://www.linux.ime.usp.br/~domingos/doces.ps http://www.linux.ime.usp.br/~domingos/doces.pdf Acho que o Postscript está mais legível... [ ]'s ========================================================================= Instruções para entrar na lista, sair da lista e usar a lista em http://www.mat.puc-rio.br/~nicolau/olimp/obm-l.html O administrador desta lista é ========================================================================= From owner-obm-l@sucuri.mat.puc-rio.br Sun Apr 20 23:49:02 2003 Return-Path: Received: (from majordom@localhost) by sucuri.mat.puc-rio.br (8.9.3/8.9.3) id XAA17471 for obm-l-MTTP; Sun, 20 Apr 2003 23:46:48 -0300 Received: from artemis.opendf.com.br (artemis.opengate.com.br [200.181.71.14]) by sucuri.mat.puc-rio.br (8.9.3/8.9.3) with ESMTP id XAA17467 for ; Sun, 20 Apr 2003 23:46:45 -0300 Received: from localhost (localhost [127.0.0.1]) by artemis.opendf.com.br (Postfix) with ESMTP id 1B6C12BEC9 for ; Sun, 20 Apr 2003 23:47:41 -0300 (BRT) Received: from artemis.opendf.com.br ([127.0.0.1]) by localhost (artemis.opengate.com.br [127.0.0.1:10024]) (amavisd-new) with ESMTP id 06997-04 for ; Sun, 20 Apr 2003 23:47:40 -0300 (BRT) Received: from computer (200-181-090-222.bsace7001.dsl.brasiltelecom.net.br [200.181.90.222]) by artemis.opendf.com.br (Postfix) with ESMTP id 82ABB2BEC2 for ; Sun, 20 Apr 2003 23:47:39 -0300 (BRT) From: "Artur Costa Steiner" To: Subject: [obm-l] =?iso-8859-1?Q?RE:_=5Bobm-l=5D_Re:_RE:D=FAvida_Sobre_a_Utilidade_da_Mat?= =?iso-8859-1?Q?em=E1tica_Ensinada_nas_Escolas?= Date: Sun, 20 Apr 2003 23:46:14 -0300 Organization: Steiner Consultoria LTDA Message-ID: <002201c307b0$2f76a8d0$9865fea9@computer> MIME-Version: 1.0 Content-Type: text/plain; charset="iso-8859-1" X-Priority: 3 (Normal) X-MSMail-Priority: Normal X-Mailer: Microsoft Outlook, Build 10.0.2627 Importance: Normal X-MimeOLE: Produced By Microsoft MimeOLE V6.00.2800.1106 In-Reply-To: <016b01c307a8$6635ae00$f3add9c8@iloveusa> X-Virus-Scanned: by amavisd-new Content-Transfer-Encoding: 8bit X-MIME-Autoconverted: from quoted-printable to 8bit by sucuri.mat.puc-rio.br id XAA17468 Sender: owner-obm-l@sucuri.mat.puc-rio.br Precedence: bulk Reply-To: obm-l@mat.puc-rio.br > >Sei q é uma pergunta meio esquisita,mas nunca entendi pra q isso >servia.Pelo >menos nunca vi ninguém fazendo esses cálculos nem usando coisas como x-10= >etc etc. [Artur Costa Steiner] Nao eh esquesita, nem todos curtem matematica e nem todos tem que se aprofundar nela. Eh como a nossa lingua. Todos temos que ser capazes de nos expressar clara e corretamente, mas apenas alguns tem que conhecer profundamente asuntos como literatura e analise sintatica. > >Pelo menos deveriam não colocar coisas q a pessoa não vai utilizar.Estou no >pré vestibular e tenho muita dificuldade pra entender questões >matemáticas.Algumas são propositalmente feitas pra confundir o candidato a >um curso universitário. [Artur Costa Steiner] Isto eh de fato lamentavel. > >Aproveitando,vc ou outra pessoa pode dizer se a seguinte questão está bem >formulada? >(PUC-MG)Em uma empresa,60% dos funcionários lêem a revista A,80 % lêem a >revista B e todo funcionário é leitor de pelo menos uma dessas revistas.O >percentual de funcionários que lêem as duas revistas é: [Artur Costa Steiner] Eu creio que eh mais facil de entender se definirmos Ca como o conjunto dos funcionarios que leem a revista A, Cb como o conjunto dos funcionarios que lem a revista B e C como o conjunto dos funcionários que leem ao menos uma das duas revistas. Por hipotese, todo funcionario da empresa le pelo menos uma das duas revistas, de modo que C se confunde com o conjunto dos funcionarios da empresa. Em termos percentuais, podemos entao dizer que nC, o numero de elementos de C (no caso em percentual), eh 100. O que o problema pede eh o numero de elementos da intercecao de Ca e Cb (conjunto dos que leem as duas revistas). E temos que a uniao de Ca e Cb e o proprio C, pois todo funcionario le A ou B (observe que o "ou" nao eh excludente). Da teoria dos conjuntos, temos que n(Ca inter Cb) = nCa + nCb - nC = 60 + 80m - 100 =40. Que eh o que vc fez. Ao dizermos que o numero de elementos de uma uniao de dois conjuntos e soma dos numeros de elementos de cada um dos conjunto menos o numero de elementos da intercecao deles, estamos evitando que os elementos da intercecao sejam contados duas vezes. O seu professor fez o mesmo, so que talvez tenha explicado de modo um pouco difrerente. Um abraco. Artur ========================================================================= Instruções para entrar na lista, sair da lista e usar a lista em http://www.mat.puc-rio.br/~nicolau/olimp/obm-l.html O administrador desta lista é ========================================================================= From owner-obm-l@sucuri.mat.puc-rio.br Mon Apr 21 00:01:25 2003 Return-Path: Received: (from majordom@localhost) by sucuri.mat.puc-rio.br (8.9.3/8.9.3) id XAA17730 for obm-l-MTTP; Sun, 20 Apr 2003 23:59:45 -0300 Received: from artemis.opendf.com.br (artemis.opengate.com.br [200.181.71.14]) by sucuri.mat.puc-rio.br (8.9.3/8.9.3) with ESMTP id XAA17725 for ; Sun, 20 Apr 2003 23:59:41 -0300 Received: from localhost (localhost [127.0.0.1]) by artemis.opendf.com.br (Postfix) with ESMTP id 212942BEC4 for ; Sun, 20 Apr 2003 23:59:11 -0300 (BRT) Received: from artemis.opendf.com.br ([127.0.0.1]) by localhost (artemis.opengate.com.br [127.0.0.1:10024]) (amavisd-new) with ESMTP id 07113-03 for ; Sun, 20 Apr 2003 23:59:10 -0300 (BRT) Received: from computer (200-181-090-222.bsace7001.dsl.brasiltelecom.net.br [200.181.90.222]) by artemis.opendf.com.br (Postfix) with ESMTP id D5F332BEC2 for ; Sun, 20 Apr 2003 23:59:09 -0300 (BRT) From: "Artur Costa Steiner" To: Subject: [obm-l] =?iso-8859-1?Q?RE:_=5Bobm-l=5D_Re:_RE:D=FAvida_Sobre_a_Utilidade_da_Mat?= =?iso-8859-1?Q?em=E1tica_Ensinada_nas_Escolas?= Date: Sun, 20 Apr 2003 23:59:12 -0300 Organization: Steiner Consultoria LTDA Message-ID: <002301c307b1$fea7c700$9865fea9@computer> MIME-Version: 1.0 Content-Type: text/plain; charset="iso-8859-1" X-Priority: 3 (Normal) X-MSMail-Priority: Normal X-Mailer: Microsoft Outlook, Build 10.0.2627 Importance: Normal X-MimeOLE: Produced By Microsoft MimeOLE V6.00.2800.1106 In-Reply-To: <016b01c307a8$6635ae00$f3add9c8@iloveusa> X-Virus-Scanned: by amavisd-new Content-Transfer-Encoding: 8bit X-MIME-Autoconverted: from quoted-printable to 8bit by sucuri.mat.puc-rio.br id XAA17726 Sender: owner-obm-l@sucuri.mat.puc-rio.br Precedence: bulk Reply-To: obm-l@mat.puc-rio.br Ah, esqueci de responder a esta duvida [Artur Costa Steiner] >E minha principal dúvida: 100% não é um todo?Se é,como pode uma empresa >dividir os funcionários em 60% q lêem a revista A e 80% q lêem a B? >(80+60=140%) Pelo q entendi,deveria ser um número como por exemplo,40% q >lêem a revista A e 60% q lêem a revista B(40+60=100.O máximo,100%) >Bem,sei q estou errado nessas coisas,por isso,fica aqui a solicitação de >ajuda aos magos da matemática :) [Artur Costa Steiner] Observe que os conjuntos nao sao disjuntos, isto eh, ha funcionarios que leem as duas revistas. Eh por isso que a soma dos respectivos percentuais NAO tem que ser 100%. Por exemplo considere um conjunto de 5 pessoas no qual todas tenham nivel superior. E perfeitamente possivel que 4 sejam advogadas e 4 sejam engenheiras, embora 4+4 =8 >5. Neste caso, 3 pessoas sao simultanemanete advogadas e emgenheiras. Quando vc soma 4+4, esta contando duas vezes as pessoas que sao simultaneamente engenheiras e advogadas. Certo? Artur ========================================================================= Instruções para entrar na lista, sair da lista e usar a lista em http://www.mat.puc-rio.br/~nicolau/olimp/obm-l.html O administrador desta lista é ========================================================================= From owner-obm-l@sucuri.mat.puc-rio.br Mon Apr 21 00:51:51 2003 Return-Path: Received: (from majordom@localhost) by sucuri.mat.puc-rio.br (8.9.3/8.9.3) id AAA18637 for obm-l-MTTP; Mon, 21 Apr 2003 00:50:14 -0300 Received: from smtp-29.ig.com.br (smtp-29.ig.com.br [200.226.132.157]) by sucuri.mat.puc-rio.br (8.9.3/8.9.3) with SMTP id AAA18631 for ; Mon, 21 Apr 2003 00:50:07 -0300 Received: (qmail 26338 invoked from network); 21 Apr 2003 03:49:32 -0000 Received: from unknown (HELO henrique) (200.140.6.162) by smtp-29.ig.com.br with SMTP; 21 Apr 2003 03:49:32 -0000 Message-ID: <002d01c307b9$06ee2380$019da8c0@henrique> From: "=?iso-8859-1?Q?Henrique_Patr=EDcio_Sant'Anna_Branco?=" To: References: <016b01c307a8$6635ae00$f3add9c8@iloveusa> Subject: [obm-l] =?iso-8859-1?Q?Re:_=5Bobm-l=5D_Re:_RE:D=FAvida_Sobre_a_Utilidade_da_Mat?= =?iso-8859-1?Q?em=E1tica_Ensinada_nas_Escolas?= Date: Mon, 21 Apr 2003 00:49:35 -0300 MIME-Version: 1.0 Content-Type: text/plain; charset="iso-8859-1" Content-Transfer-Encoding: 8bit X-Priority: 3 X-MSMail-Priority: Normal X-Mailer: Microsoft Outlook Express 6.00.2800.1106 X-MIMEOLE: Produced By Microsoft MimeOLE V6.00.2800.1106 Sender: owner-obm-l@sucuri.mat.puc-rio.br Precedence: bulk Reply-To: obm-l@mat.puc-rio.br > E minha principal dúvida: 100% não é um todo?Se é,como pode uma empresa > dividir os funcionários em 60% q lêem a revista A e 80% q lêem a B? > (80+60=140%) Pelo q entendi,deveria ser um número como por exemplo,40% q > lêem a revista A e 60% q lêem a revista B(40+60=100.O máximo,100%) J. Paulo, preste atenção ao enunciado: todo funcionário é leitor de *pelo menos* uma dessas revistas. Isso quer dizer que um mesmo funcionário pode ler mais de uma revista. Isso é exatamente uma noção da teoria dos conjuntos da qual você falou. O conjunto A e B são os conjuntos dos funcionários que lêem a revista A e a revista B, respectivamente. Na intersecção desses dois conjuntos, está a parte "que sobra" segundo você: os que lêem as duas revistas. Quanto à questão da utilidade da matemática, esta é uma matemática bem simples, que serve para utilização no dia a dia e base para quem vai fazer um curso de exatas. Acredite, logo no Cálculo I, você já usa 90% do que você viu no colégio. E o pior: isso tudo tem que ser como um "axioma" pra você: claro e evidente :-) assim como as quatro operações básicas são de uso imediato no Segundo Grau. Você usa sem nem mesmo pensar nelas! Abraços, Henrique. ========================================================================= Instruções para entrar na lista, sair da lista e usar a lista em http://www.mat.puc-rio.br/~nicolau/olimp/obm-l.html O administrador desta lista é ========================================================================= From owner-obm-l@sucuri.mat.puc-rio.br Mon Apr 21 00:53:22 2003 Return-Path: Received: (from majordom@localhost) by sucuri.mat.puc-rio.br (8.9.3/8.9.3) id AAA18660 for obm-l-MTTP; Mon, 21 Apr 2003 00:50:45 -0300 Received: from www.zipmail.com.br (smtp.zipmail.com.br [200.221.11.147]) by sucuri.mat.puc-rio.br (8.9.3/8.9.3) with ESMTP id AAA18656 for ; Mon, 21 Apr 2003 00:50:42 -0300 From: luizhenriquerick@zipmail.com.br Received: from [200.216.34.124] by www.zipmail.com.br with HTTP; Mon, 21 Apr 2003 00:49:31 -0300 Message-ID: <3E9C60630000C01B@www.zipmail.com.br> Date: Mon, 21 Apr 2003 00:49:31 -0300 In-Reply-To: <016b01c307a8$6635ae00$f3add9c8@iloveusa> Subject: [obm-l] =?iso-8859-1?Q?RE=3AD=FAvida=20Sobre=20a=20Utilidade=20da=20Matem=E1tica=20Ensinada=20nas=20Escolas?= To: obm-l@mat.puc-rio.br MIME-Version: 1.0 Content-Type: multipart/mixed; boundary="=========3E9C60630000C01B/www.zipmail.com.br" Sender: owner-obm-l@sucuri.mat.puc-rio.br Precedence: bulk Reply-To: obm-l@mat.puc-rio.br --=========3E9C60630000C01B/www.zipmail.com.br Content-Type: text/plain; charset="iso-8859-1" Content-Transfer-Encoding: quoted-printable PUC-MG)Em uma empresa,60% dos funcion=E1rios l=EAem a revista A,80 % l=EA= em a revista B e todo funcion=E1rio =E9 leitor de pelo menos uma dessas revist= as.O percentual de funcion=E1rios que l=EAem as duas revistas =E9: Ol=E1 amigo , espero ajudar voc=EA. ( Diagrama de veen anexo ) Sendo y o n=FAmero de quem l=EA somente A , z de quem l=EA somente B , x = de quem l=EA A e B e p o total de funcion=E1rios , temos: * Se 60% l=EAem A , quer dizer que 0,6p l=EAem A . * Se 80% l=EAem B , quer dizer que 0,8p l=EAem B . Assim , fica: x + y l=EAem A , pois y l=EA somente A e x l=EAem A e B : x + y =3D 0,6 ( i ) x + z l=EAem B , o racioc=EDnio =E9 o mesmo do item anterior : x + z =3D 0,8p ( ii ) Como o total de funcion=E1rios que l=EAem =E9 igual a x + y + z e ele diz= que todos l=EAem A ou B , voc=EA pode afirmar que x + y + z =3D p (iii) . Assim , utilizando as equa=E7=F5es i , ii e iii para montar um sistema , = conseguimos chegar a conclus=E3o de que x =3D 0,4p , ou seja , 40%p , ou ainda 40% do= total. Na verdade esse problema =E9 bem b=E1sico . O que acontece com voc=EA e com qualquer outro ser humano =E9 que quando = nos deparamos com um assunto que o nosso c=E9rebro n=E3o consegue entender, n= a maioria matem=E1ticos , o extinto de muitos , talvez da maioria , exceto os amant= es da matem=E1tica , =E9 abandonar esse assunto e sem perceber come=E7ar a i= nventar justificativas paro o fato de tal abandono . Reflita sobre isso e por mais estranho que pare=E7a , nunca fique com pen= a de si mesmo , isso ajuda bastante ... Abra=E7os... Luiz H. Barbosa www.olympicmaths.hpg.com.br ------------------------------------------ Use o melhor sistema de busca da Internet Radar UOL - http://www.radaruol.com.br --=========3E9C60630000C01B/www.zipmail.com.br Content-Type: image/gif Content-Transfer-Encoding: base64 Content-Disposition: attachment; filename="veen.gif" R0lGODdhFAGyAPcAAAAAAAAAQAAAgAAA/wAgAAAgQAAggAAg/wBAAABAQABAgABA/wBgAABgQABg gABg/wCAAACAQACAgACA/wCgAACgQACggACg/wDAAADAQADAgADA/wD/AAD/QAD/gAD//yAAACAA QCAAgCAA/yAgACAgQCAggCAg/yBAACBAQCBAgCBA/yBgACBgQCBggCBg/yCAACCAQCCAgCCA/yCg ACCgQCCggCCg/yDAACDAQCDAgCDA/yD/ACD/QCD/gCD//0AAAEAAQEAAgEAA/0AgAEAgQEAggEAg /0BAAEBAQEBAgEBA/0BgAEBgQEBggEBg/0CAAECAQECAgECA/0CgAECgQECggECg/0DAAEDAQEDA gEDA/0D/AED/QED/gED//2AAAGAAQGAAgGAA/2AgAGAgQGAggGAg/2BAAGBAQGBAgGBA/2BgAGBg QGBggGBg/2CAAGCAQGCAgGCA/2CgAGCgQGCggGCg/2DAAGDAQGDAgGDA/2D/AGD/QGD/gGD//4AA AIAAQIAAgIAA/4AgAIAgQIAggIAg/4BAAIBAQIBAgIBA/4BgAIBgQIBggIBg/4CAAICAQICAgICA /4CgAICgQICggICg/4DAAIDAQIDAgIDA/4D/AID/QID/gID//6AAAKAAQKAAgKAA/6AgAKAgQKAg gKAg/6BAAKBAQKBAgKBA/6BgAKBgQKBggKBg/6CAAKCAQKCAgKCA/6CgAKCgQKCggKCg/6DAAKDA QKDAgKDA/6D/AKD/QKD/gKD//8AAAMAAQMAAgMAA/8AgAMAgQMAggMAg/8BAAMBAQMBAgMBA/8Bg AMBgQMBggMBg/8CAAMCAQMCAgMCA/8CgAMCgQMCggMCg/8DAAMDAQMDAgMDA/8D/AMD/QMD/gMD/ //8AAP8AQP8AgP8A//8gAP8gQP8ggP8g//9AAP9AQP9AgP9A//9gAP9gQP9ggP9g//+AAP+AQP+A gP+A//+gAP+gQP+ggP+g///AAP/AQP/AgP/A////AP//QP//gP///yH5BAAAAAAALAAAAAAUAbIA AAj/AP8JHEiwoMGDCBMqXMiwocOHECNKnEixosWLGDNq3Mixo8ePIDUCGEmypMmTKFOqXMmypcuX MGPKnEmzJsmQHgHg3Mmzp8+POn9eDCq0qNGjR4kihah0qdOnUCs2jZpwKtWrWLFazTpwK9evYH16 5To2rNmzG8v+S8l0JEa1aOPKZcowqNu1CLfetQh3rt+/BvuuLatXcFXAiBMrFLx3sN2bNwU2Ptm1 ZGDFmDPjdbjX7V3POj/3BV3QsObTVBkTJT2YoGjXJmG3do26tlzVlWeLft2QdVfbwM3ilhz6cWve dYuXDs6cLOfIjisXR9kb+m+Jpptr7521MW3s2beL/z8YHqf3623Lj1+/ObV19A8ts2e+kvvtiNPn a4f+nvxc0/npR99k5y13X3zSCRicdf0ZGJdhqxWoIGb8Sfjdg+nFFlJMYXFoFFt1/YcgbOrlJd9z DQqlIYoWdlShWiUWxVhpKeLXYnoq1mjjjUMRSJiIyXW2IngxVsVjj+YVaaJSOraHlpITNZnWkTj2 JOWOJC4EpZUfbmljRlTmtOWLIR4olpcUhRkYmlHGWF+ZGIqlFXhasckZkDupeeaIzn2I54bCJSec nYv9KeaTXhGak4qGcqQoUP41CqiZLgLG5KMg6ZmmpG8lZheFPGHq6KKKadrnpHGmlZmpp0JKaaeg OhNZKqpPqhqrZKcRKqqtsN6K66qZ/3KKn2ZW7Zqnq6lKRexhwFb6KnbLFtosr7XyNS2z1/Za7abZ 5hWtSMLCOeud3Sr7LJ/j2ufrUOFKu664noJ7rrrxDvuttfNqeS+69bKbr7v92luuwNvyi5h6xpqn 7VnZJUwrwQFzmyy9EUM7sMEdQnywv+9qnDFdHXtsKb4T63sxyCGTWzLAFUe5r8hfQfiyyyfDG6jK 6S68scQFm5yyxT/73DPLO4M5M84MY/zXmEdTfLPTI8sbNNEfQ+2Xlw4/rHTMSBctdc4oD+3txbE1 mTWyMLdKtaOWXbk1cZ8KzRdlctK8cqRJruaskaFZbbHbX4edtN9sx03tkl1zC7jRdpKLjXfesjK+ GJVYs2ou0I5fZuXiia8JV+VZl3f25XJvyDnhcH8+5dmitwtfnnb+GF3paXb2U+v/Xhhq7HxPJ/tb yIXK8+A2K0zq4bXTzWXjxNMeLNqSg/12d52vHnngOkeNefPOi3S64CRLP/2c1Xt/LPbab/908Usj n37a1JefO/jv0w/WjDXbfzXp3K/dfvReU/9f1VB3N/gVkIBXkVn+ENg//amte65j4PoEeD8DZk5+ B5QgVADUNAxeUINP4WAH2RdBCMbPgfPDlvgsmBoK7m94AWTeBD3YQBKW0ITkY+EMQci17FVQhvXD 4QfHl0AYxtB/KaRhVBA2Qt21zIg/BGIQH3dE/u2QiDV00Aqh2EMrTvF1VfRiFLn4wrFtUYxdRGMZ tRjG8GURhc8a3aigd0UXGkqOczzeELH4NDy6D31j9OH+/AjIQj6QY2dMohpP+Mf7EBKRz8sgGfv4 SDoJT5E6LCIb37i3PU6SkU4c4CUlOTdETW5QMrrhIkOoxFT6CZNSqSSN2lInpMiSh6tkVJtsNujH W9rwUCF0kxwt18hDws6WYyJmlTaoysKdSVePUqYeObk7aO7qRD2SJh1FuZTvkUebbVrckNJITRlh M0jg9N5L1ljObsJkQk7xpRDhSU9c2rKe+HRjHfPJTxW2s58AlScSAUrQX5GyoPwU4Kg/EcrQ6wWy oQhVqBkhylCJUpGiBLWo5jAa0WZylD0a3eRH8xnSUI4UnyUF40nrmVKDrpSkHn1pc1rqUJkKiKY0 tanWuKnTm8a0p7XBKVDpKdShTqioRvUpLJOaq58yNZfGO+hTY9k3Vi51qnyqJFKxmsfZxNOpXMVW Vb961bBKy3DdBKtZL6M3q0p1rdVxaVrLCtdvphOS+6wrH2+nVr0+lKd+XWALA2ubrRK2iZo8bFPp qlhQ5rWxjHUlZKeW2Mk+kZyW/d9bM+vYDtnks6ANrWhHS9rSeoizqE2talfLCNrWuva1Og0IADs= --=========3E9C60630000C01B/www.zipmail.com.br-- ========================================================================= Instruções para entrar na lista, sair da lista e usar a lista em http://www.mat.puc-rio.br/~nicolau/olimp/obm-l.html O administrador desta lista é ========================================================================= From owner-obm-l@sucuri.mat.puc-rio.br Mon Apr 21 01:29:39 2003 Return-Path: Received: (from majordom@localhost) by sucuri.mat.puc-rio.br (8.9.3/8.9.3) id BAA20166 for obm-l-MTTP; Mon, 21 Apr 2003 01:27:53 -0300 Received: from acsxe0.ac.brahma (smtp01.ambev.com.br [200.212.186.163]) by sucuri.mat.puc-rio.br (8.9.3/8.9.3) with ESMTP id BAA20154 for ; Mon, 21 Apr 2003 01:27:25 -0300 Received: from acsnx2.ac.brahma (acsnx2 [10.100.3.89]) by acsxe0.ac.brahma (AIX4.3/8.9.3/8.9.3) with ESMTP id BAA75128 for ; Mon, 21 Apr 2003 01:26:18 -0300 Received: by acsnx2.brahma with Internet Mail Service (5.5.2653.19) id <20X626WR>; Mon, 21 Apr 2003 01:25:47 -0300 Message-ID: <27BD56F8640DD711A4320006295078E50500C1@pssnx1.brahma> From: "Ricardo de Moraes (PS)" To: "'obm-l@mat.puc-rio.br'" Subject: [obm-l] Equacao diferencial Date: Mon, 21 Apr 2003 01:25:46 -0300 MIME-Version: 1.0 X-Mailer: Internet Mail Service (5.5.2653.19) Content-Type: multipart/alternative; boundary="----_=_NextPart_001_01C307BE.14B60640" Sender: owner-obm-l@sucuri.mat.puc-rio.br Precedence: bulk Reply-To: obm-l@mat.puc-rio.br This message is in MIME format. Since your mail reader does not understand this format, some or all of this message may not be legible. ------_=_NextPart_001_01C307BE.14B60640 Content-Type: text/plain; charset="ISO-8859-1" Content-Transfer-Encoding: quoted-printable Boa noite, Nao consegui chegar a mesma resposta que o livro traz para esta = quest=E3o... =20 Uma gota esferica evapora a uma taxa proporcional a sua area de = superficie. Escreva uma equacao diferencial para o volume de uma gota de chuva em = funcao do tempo. =20 Resposta: V' =3D - kV^2/3 =20 Obrigado. ------_=_NextPart_001_01C307BE.14B60640 Content-Type: text/html; charset="ISO-8859-1" Content-Transfer-Encoding: quoted-printable
Boa=20 noite,
Nao = consegui chegar=20 a mesma resposta que o livro traz para esta = quest=E3o...
 
Uma = gota esferica=20 evapora a uma taxa proporcional a sua area de superficie. Escreva uma = equacao=20 diferencial para o volume de uma gota de chuva em funcao do=20 tempo.
 
Resposta: V' =3D -=20 kV^2/3
 
Obrigado.
------_=_NextPart_001_01C307BE.14B60640-- ========================================================================= Instruções para entrar na lista, sair da lista e usar a lista em http://www.mat.puc-rio.br/~nicolau/olimp/obm-l.html O administrador desta lista é ========================================================================= From owner-obm-l@sucuri.mat.puc-rio.br Mon Apr 21 02:17:54 2003 Return-Path: Received: (from majordom@localhost) by sucuri.mat.puc-rio.br (8.9.3/8.9.3) id CAA21121 for obm-l-MTTP; Mon, 21 Apr 2003 02:16:35 -0300 Received: from www.zipmail.com.br (smtp.zipmail.com.br [200.221.11.147]) by sucuri.mat.puc-rio.br (8.9.3/8.9.3) with ESMTP id CAA21113 for ; Mon, 21 Apr 2003 02:16:30 -0300 From: camilojr@zipmail.com.br Received: from [200.214.79.186] by www.zipmail.com.br with HTTP; Mon, 21 Apr 2003 02:15:17 -0300 Message-ID: <3E9C60630000C16A@www.zipmail.com.br> Date: Mon, 21 Apr 2003 02:15:17 -0300 In-Reply-To: <27BD56F8640DD711A4320006295078E50500C1@pssnx1.brahma> Subject: [obm-l] =?iso-8859-1?Q?Re=3A=20=5Bobm=2Dl=5D=20Equacao=20diferencial?= To: obm-l@mat.puc-rio.br MIME-Version: 1.0 Content-Type: text/plain; charset="iso-8859-1" Content-Transfer-Encoding: 8bit X-MIME-Autoconverted: from quoted-printable to 8bit by sucuri.mat.puc-rio.br id CAA21114 Sender: owner-obm-l@sucuri.mat.puc-rio.br Precedence: bulk Reply-To: obm-l@mat.puc-rio.br Oi Ricardo, Sabendo que a área superficial é proporcional ao raio(r) ao quadrado e o volume ao raio ao cubo é meio que imediato. Se não, vejamos: O volume diminui proporcionalmente á área superficial, assim V' = -c1*r^2 (1) em que c1 é cte. e o sinal lembra o fato do volume estar diminuindo. Mas: V = c2*r^3 => r = c3*V^(1/3) (2) em que c2 e c3 são ctes.. Usando (1) e (2) você chega à resposta. um abraço, Camilo -- Mensagem original -- >Boa noite, >Nao consegui chegar a mesma resposta que o livro traz para esta questão... > >Uma gota esferica evapora a uma taxa proporcional a sua area de superficie. >Escreva uma equacao diferencial para o volume de uma gota de chuva em funcao >do tempo. > >Resposta: V' = - kV^2/3 > >Obrigado. > ------------------------------------------ Use o melhor sistema de busca da Internet Radar UOL - http://www.radaruol.com.br ========================================================================= Instruções para entrar na lista, sair da lista e usar a lista em http://www.mat.puc-rio.br/~nicolau/olimp/obm-l.html O administrador desta lista é ========================================================================= From owner-obm-l@sucuri.mat.puc-rio.br Mon Apr 21 08:54:40 2003 Return-Path: Received: (from majordom@localhost) by sucuri.mat.puc-rio.br (8.9.3/8.9.3) id IAA24804 for obm-l-MTTP; Mon, 21 Apr 2003 08:53:05 -0300 Received: from hotmail.com (f168.sea1.hotmail.com [207.68.163.168]) by sucuri.mat.puc-rio.br (8.9.3/8.9.3) with ESMTP id IAA24800 for ; Mon, 21 Apr 2003 08:53:01 -0300 Received: from mail pickup service by hotmail.com with Microsoft SMTPSVC; Mon, 21 Apr 2003 04:52:29 -0700 Received: from 198.81.9.3 by sea1fd.sea1.hotmail.msn.com with HTTP; Mon, 21 Apr 2003 11:52:28 GMT X-Originating-IP: [198.81.9.3] X-Originating-Email: [fredericor@hotmail.com] From: "Frederico Reis Marques de Brito" To: obm-l@mat.puc-rio.br Subject: [obm-l] =?iso-8859-1?B?UmU6IFtvYm0tbF0gUmU6IFtvYm0tbF0gcXVlc3TjbyBzb2JyZSBjb25q?= =?iso-8859-1?B?dW50b3M=?= Date: Mon, 21 Apr 2003 08:52:28 -0300 Mime-Version: 1.0 Content-Type: text/plain; charset=iso-8859-1; format=flowed Message-ID: X-OriginalArrivalTime: 21 Apr 2003 11:52:29.0199 (UTC) FILETIME=[7C1DD9F0:01C307FC] Sender: owner-obm-l@sucuri.mat.puc-rio.br Precedence: bulk Reply-To: obm-l@mat.puc-rio.br Qualquer princípio de contagem constitui análise combinatória. Logo a sua solução, embora maios elementar, tb usa anal. comb. Frederico. >From: Fábio Nunes Ribeiro Maia >Reply-To: obm-l@mat.puc-rio.br >To: Lista de discussão de Matemática >Subject: [obm-l] Re: [obm-l] questão sobre conjuntos >Date: Sun, 20 Apr 2003 21:33:19 -0300 > > Primeiramente, olá pessoal da lista, e em especial pra Renatinha. É a >primeira vez que escrevo pra esta lista, apesar de já estar acompanhando-a >a >pelo menos 6 meses. Eu observei que, nas várias soluções que enviaram ao >problema, nenhum de vocês deve ter reparado na observação que nossa amiga >escreveu logo após: > >P.S: "Gostaria, se possível, que a solução não tivesse probabilidade ou >combinatória, pois ainda faço 1º ano do ensino médio." > > Como também estou no primeiro ano do EM e também tive que resolver >essa >questão, fiz isso da seguinte maneira: > >Se um conjunto qualquer possui n elementos, o número de subconjuntos é dado >pela fórmula 2^n. Assim, um conjunto vazio possui 1 subconjunto, também >vazio; um unitário possui 2 subconjuntos (um vazio e um unitário); um de 2 >possui 4 subconjuntos (um vazio, 2 unitários e um de 2 elementos); um outro >de 3 possui 8 subconjuntos (um vazio, 3 unitários, 3 de 2 elementos e um de >3 elementos) e assim sucessivamente. Assim, pude construir a seguinte >pirâmide (se eu não me engano ela é usada pra resolver um outro tipo de >problema, e seu nome tem a ver com Newton, um amigo me disse isso): > > 1 > 1 1 > 1 2 1 > 1 3 3 1 > 1 4 6 4 1 > 1 5 10 10 5 1 > 1 6 15 20 15 6 1 > 1 7 21 35 35 21 7 1 > 1 8 28 56 70 56 28 8 1 > (...) > >O processo de formação dessa pirâmide é o seguinte: começando com a segunda >linha, a cada linha depois dela deve-se adicionar o número um nas >extremidades, e embaixo de cada espaço da linha anterior escreve-se a soma >dos números que se encontram ao lado dele. >Nela, cada linha se refere a um conjunto com um número de elementos que é >de >0 (conjunto vazio) na primeira linha, e assim prossegue até o infinito, e >os >números de cada linha indicam quantos subconjuntos existem com cada número >de elementos do conjunto original. Assim: > > Conjunto com 3 elementos (quarta linha): >1 conjunto vazio >3 " unitário >3 " com 2 elementos >1 " com 3 elementos > > Conjunto com 8 elementos (nona linha): >1 conjunto vazio >8 " unitário >28 " com 2 elementos >56 " com 3 elementos >70 " com 4 elementos >56 " com 5 elementos >28 " com 6 elementos >8 " com 7 elementos >1 " com 8 elementos > >Assim, fica fácil de perceber qual a resposta da questão: > O conjunto A, de 8 elementos, possui 56 subconjuntos com 5 elementos >distintos cada um. > >P.S.: Desculpe se a resposta foi demorada, mas foi a maneira mais fácil que >consegui elaborar para resolver essa questão com a matéria que nos foi dada >a mim e à Renatinha até agora. Espero que eu tenha podido ser útil. > > > >----- Original Message ----- >From: "renatinha15a" >To: "obm" >Sent: Sunday, April 20, 2003 10:40 AM >Subject: [obm-l] questão sobre conjuntos > > > > Oi pessoal da lista, não consegui fazer esta questão, se > > alguém puder me ajudar, ficarei bastante grata. > > > > (U.F. VIÇOSA-89) Um conjunto A tem 8 elementos > > distintos. O número de subconjuntos de A, com 5 > > elementos distintos cada um, é: > > > > P.S: Gostaria, se possível, que a solução não tivesse > > probabilidade ou combinatória, pois ainda faço 1º ano do > > ensino médio. > > > > []´s > > Renatinha > > > > > > >__________________________________________________________________________ > > Seleção de Softwares UOL. > > 10 softwares escolhidos pelo UOL para você e sua família. > > http://www.uol.com.br/selecao > > > > > > >========================================================================= > > Instruções para entrar na lista, sair da lista e usar a lista em > > http://www.mat.puc-rio.br/~nicolau/olimp/obm-l.html > > O administrador desta lista é > > >========================================================================= > > > >========================================================================= >Instruções para entrar na lista, sair da lista e usar a lista em >http://www.mat.puc-rio.br/~nicolau/olimp/obm-l.html >O administrador desta lista é >========================================================================= _________________________________________________________________ MSN Messenger: converse com os seus amigos online. http://messenger.msn.com.br ========================================================================= Instruções para entrar na lista, sair da lista e usar a lista em http://www.mat.puc-rio.br/~nicolau/olimp/obm-l.html O administrador desta lista é ========================================================================= From owner-obm-l@sucuri.mat.puc-rio.br Mon Apr 21 09:13:23 2003 Return-Path: Received: (from majordom@localhost) by sucuri.mat.puc-rio.br (8.9.3/8.9.3) id JAA25047 for obm-l-MTTP; Mon, 21 Apr 2003 09:11:39 -0300 Received: from hotmail.com (f68.sea1.hotmail.com [207.68.163.68]) by sucuri.mat.puc-rio.br (8.9.3/8.9.3) with ESMTP id JAA25042 for ; Mon, 21 Apr 2003 09:11:35 -0300 Received: from mail pickup service by hotmail.com with Microsoft SMTPSVC; Mon, 21 Apr 2003 05:11:03 -0700 Received: from 198.81.9.1 by sea1fd.sea1.hotmail.msn.com with HTTP; Mon, 21 Apr 2003 12:11:03 GMT X-Originating-IP: [198.81.9.1] X-Originating-Email: [fredericor@hotmail.com] From: "Frederico Reis Marques de Brito" To: obm-l@mat.puc-rio.br Subject: [obm-l] =?iso-8859-1?B?UmU6IFtvYm0tbF0gRPp2aWRhIFNvYnJlIGEgVXRpbGlkYWRlIGRhIE1h?= =?iso-8859-1?B?dGVt4XRpY2EgRW5zaW5hZGEgbmFzIEVzY29sYXM=?= Date: Mon, 21 Apr 2003 09:11:03 -0300 Mime-Version: 1.0 Content-Type: text/plain; charset=iso-8859-1; format=flowed Message-ID: X-OriginalArrivalTime: 21 Apr 2003 12:11:03.0956 (UTC) FILETIME=[14904540:01C307FF] Sender: owner-obm-l@sucuri.mat.puc-rio.br Precedence: bulk Reply-To: obm-l@mat.puc-rio.br O imediatismo é um dos maiores entraves ao conhecimento. Não podemos de forma alguma banalizar a Matemática, ou qualquer outra ciência, a ponto de exigirmos que seu estudo seja necessário para irmos até a esquina compar um jornal, por exemplo. Aconselho ler: G. H > Hardy, "EM DEFEASA DE UM MATEMÁTICO". E o estudo de todas as ciências em níveis elementares se explica pelo fato de que qq pessoa deve ter um mínimo de cultura e formação gerais, para que, dentre outras coisas, possa escolher com alguma razão uma área para especializar-se. Ademais, no caso específico da Matemática, embora muitas vezes tratada erroneamente pelos professores dos ensinos fundamental e médio, seu estudo desenvolve capacidades específicas do pensar, como abstração e formalismo, que são por demais importantes a quase todas as pessoas. " Seja lá o que você venha a fazer, se você souber Matemática fará bem melhor!" ( Arrabal ) "Na Matemática não posso achar deficiência, a não ser que as pessoas não compreendem suficientemente o uso excelente da Matemática pura.''(F. Bacon) "O abandono da Matemática traz dano a todo o conhecimento."(R. Bacon) ``Toda educação científica que não se inicia com a Matemática é naturalmente imperfeita em sua base.'' (A. Comte) Frederico. >From: "J.Paulo roxer ´til the end" >Reply-To: obm-l@mat.puc-rio.br >To: >Subject: [obm-l] Dúvida Sobre a Utilidade da Matemática Ensinada nas >Escolas >Date: Sun, 20 Apr 2003 17:44:54 -0300 > >Olá > >Alguém poderia dizer pra que serve conjuntos,análise >combinatória,bissetriz,equação do 1º e 2º grau,produtos >notáveis,fórmulas(Como a de Báskara), entre outros assuntos que parecem não >fazer parte do mundo real? >Existe algum emprego em q essas coisas são usadas? > >[ ]´s > >João Paulo > > >-- >Email.it, the professional e-mail, gratis per te: http://www.email.it/f > >Sponsor: >Libri e CD musicali nuovi con sconti dal 60 all'80%! >Clicca qui: http://adv.email.it/cgi-bin/foclick.cgi?mid=814&d=20-4 _________________________________________________________________ MSN Hotmail, o maior webmail do Brasil. http://www.hotmail.com ========================================================================= Instruções para entrar na lista, sair da lista e usar a lista em http://www.mat.puc-rio.br/~nicolau/olimp/obm-l.html O administrador desta lista é ========================================================================= From owner-obm-l@sucuri.mat.puc-rio.br Mon Apr 21 12:20:28 2003 Return-Path: Received: (from majordom@localhost) by sucuri.mat.puc-rio.br (8.9.3/8.9.3) id MAA27321 for obm-l-MTTP; Mon, 21 Apr 2003 12:17:06 -0300 Received: from web13007.mail.yahoo.com (web13007.mail.yahoo.com [216.136.174.17]) by sucuri.mat.puc-rio.br (8.9.3/8.9.3) with SMTP id MAA27316 for ; Mon, 21 Apr 2003 12:17:01 -0300 Message-ID: <20030421151630.66488.qmail@web13007.mail.yahoo.com> Received: from [200.148.194.13] by web13007.mail.yahoo.com via HTTP; Mon, 21 Apr 2003 12:16:30 ART Date: Mon, 21 Apr 2003 12:16:30 -0300 (ART) From: =?iso-8859-1?q?Helder=20Suzuki?= Subject: [obm-l] 4 coisinhas To: obm-l@mat.puc-rio.br MIME-Version: 1.0 Content-Type: text/plain; charset=iso-8859-1 Content-Transfer-Encoding: 8bit Sender: owner-obm-l@sucuri.mat.puc-rio.br Precedence: bulk Reply-To: obm-l@mat.puc-rio.br 1) De quantas formas podemos dividir um retangulo 2xN em partes de 2x1 e/ou 2x2? 2) Dois jogadores começam com dois inteiros, a e b, a>b>0. Em cada jogada um jogador subtrai do maior numero um multiplo positivo do menor numero, tal que o resultado nao seja negativo. Se um jogador conseguir deixar um 0, ele ganha o jogo. Por exemplo, se A e B estao jogando e A comeca com a=25 e b=7: 25 7 <- inicial 11 7 <- A retira 2*7 de 25 = 11 4 7 <- B retira 1*7 de 11 = 4 4 3 <- A retira 1*4 de 7 = 3 1 3 <- B retira 1*3 de 4 = 1 1 0 <- A retira 3*1 de 3 e ganha o jogo Assumindo que A e B joguem perfeitamente. Se A sempre começa o jogo, que condições a e b devem respeitar para que A ganhe? e para que B ganhe? 3) A area de um triangulo em funcao do comprimento das medianas? 4) Calcular S = 1 + 8 + 27 + ... + x^3 Abraços, Helder Toshiro Suzuki _______________________________________________________________________ Yahoo! Mail O melhor e-mail gratuito da internet: 6MB de espaço, antivírus, acesso POP3, filtro contra spam. http://br.mail.yahoo.com/ ========================================================================= Instruções para entrar na lista, sair da lista e usar a lista em http://www.mat.puc-rio.br/~nicolau/olimp/obm-l.html O administrador desta lista é ========================================================================= From owner-obm-l@sucuri.mat.puc-rio.br Mon Apr 21 12:42:01 2003 Return-Path: Received: (from majordom@localhost) by sucuri.mat.puc-rio.br (8.9.3/8.9.3) id MAA27733 for obm-l-MTTP; Mon, 21 Apr 2003 12:39:27 -0300 Received: from sina.bol.com.br (sina.bol.com.br [200.221.24.27]) by sucuri.mat.puc-rio.br (8.9.3/8.9.3) with ESMTP id MAA27729 for ; Mon, 21 Apr 2003 12:39:23 -0300 Received: from flavio (200.221.24.192) by sina.bol.com.br (5.1.071) id 3E7665FB009AD9A1 for obm-l@mat.puc-rio.br; Mon, 21 Apr 2003 12:38:53 -0300 To: obm-l@mat.puc-rio.br Subject: [obm-l] =?utf-8?Q?D=C3=BAvidas_sobre_geometria...?= References: Message-ID: Content-Type: text/plain; charset=utf-8; format=flowed From: Rodrigo Badia Piccinini MIME-Version: 1.0 Date: Mon, 21 Apr 2003 12:41:36 -0300 In-Reply-To: User-Agent: Opera7.10/Win32 M2 build 2840 X-Sender-IP: 200.152.112.80 Sender: owner-obm-l@sucuri.mat.puc-rio.br Precedence: bulk Reply-To: obm-l@mat.puc-rio.br Aêe, >> Gostaria de esclarecer algumas dúvidas sobre geometria: -Por quê quando calculamos a área lateral de um cone podemos supor a planificação do lado como um triângulo, valendo a equação (r*g)/2 ? Isso vale para outros casos? -Se o arco de uma circunferência vale raiz(2)*R, então posso "esticá-lo" e dizer que sua área é igual a de um triângulo retângulo de catetos R e hipotenusa raiz(2)*R? -Eu estudo eng Mecânica na UFF e não encontrei bons livros de geometria plana lá... Quais livros são recomendados e onde posso conseguí-los? >>> Obrigado! ========================================================================= Instruções para entrar na lista, sair da lista e usar a lista em http://www.mat.puc-rio.br/~nicolau/olimp/obm-l.html O administrador desta lista é ========================================================================= From owner-obm-l@sucuri.mat.puc-rio.br Mon Apr 21 13:02:09 2003 Return-Path: Received: (from majordom@localhost) by sucuri.mat.puc-rio.br (8.9.3/8.9.3) id MAA28315 for obm-l-MTTP; Mon, 21 Apr 2003 12:59:19 -0300 Received: from sina.bol.com.br (sina.bol.com.br [200.221.24.27]) by sucuri.mat.puc-rio.br (8.9.3/8.9.3) with ESMTP id MAA28304 for ; Mon, 21 Apr 2003 12:59:14 -0300 Received: from xx (200.221.24.192) by sina.bol.com.br (5.1.071) id 3E7665FB009AE909 for obm-l@mat.puc-rio.br; Mon, 21 Apr 2003 12:58:43 -0300 Message-ID: <001f01c3081e$e5f278a0$f4befea9@xx> From: "Blue Ice" To: References: <001501c305ea$3b3d38e0$f4befea9@xx> <003b01c307a0$078f4ee0$68a8ffc8@fabio> Subject: [obm-l] =?iso-8859-1?Q?Re:_=5Bobm-l=5D_Re:_=5Bobm-l=5D_Quando_ser=E1_a_olim?= =?iso-8859-1?Q?p=EDada_Internacional=3F=3F?= Date: Mon, 21 Apr 2003 12:58:44 -0300 MIME-Version: 1.0 Content-Type: multipart/alternative; boundary="----=_NextPart_000_001C_01C30805.BD7B77F0" X-Priority: 3 X-MSMail-Priority: Normal X-Mailer: Microsoft Outlook Express 6.00.2600.0000 X-MIMEOLE: Produced By Microsoft MimeOLE V6.00.2600.0000 X-Sender-IP: 200.148.36.24 Sender: owner-obm-l@sucuri.mat.puc-rio.br Precedence: bulk Reply-To: obm-l@mat.puc-rio.br This is a multi-part message in MIME format. ------=_NextPart_000_001C_01C30805.BD7B77F0 Content-Type: text/plain; charset="iso-8859-1" Content-Transfer-Encoding: quoted-printable Obrigado colega! []=B4s Ice ICQ:177782914 Email:Vinicius84@hotmail.com Win XP Home Full/Speedy 256kbps=20 ----- Original Message -----=20 From: F=E1bio Nunes Ribeiro Maia=20 To: obm-l@mat.puc-rio.br=20 Sent: Sunday, April 20, 2003 9:50 PM Subject: [obm-l] Re: [obm-l] Quando ser=E1 a olimp=EDada = Internacional?? -------------------------------------------------------------------------= ----- E-mail Premium BOL=20 Antiv=EDrus, anti-spam e at=E9 100 MB de espa=E7o. Assine j=E1! http://email.bol.com.br=20 -------------------------------------------------------------------------= ----- Este ano a 44a. IMO ser=E1 realizada nos dias 07 a 19 de julho no = Jap=E3o. Mais informa=E7=F5es nos site da OBM = http://www.obm.org.br/imo.htm, da IMO http://imo.math.ca/, ou no site = oficial do evento: http://www.imo2003.com/=20 Um abra=E7o, =20 F=E1bio Maia ----- Original Message -----=20 From: Blue Ice=20 To: obm-l@mat.puc-rio.br=20 Sent: Friday, April 18, 2003 5:36 PM Subject: [obm-l] Quando ser=E1 a olimp=EDada Internacional?? ------=_NextPart_000_001C_01C30805.BD7B77F0 Content-Type: text/html; charset="iso-8859-1" Content-Transfer-Encoding: quoted-printable
Obrigado colega!
 
 
[]=B4s
 
Ice
 
ICQ:177782914 = Email:Vinicius84@hotmail.com
Win=20 XP Home Full/Speedy 256kbps
----- Original Message -----
From:=20 F=E1bio=20 Nunes Ribeiro Maia
Sent: Sunday, April 20, 2003 = 9:50=20 PM
Subject: [obm-l] Re: [obm-l] = Quando ser=E1=20 a olimp=EDada Internacional??


E-mail Premium BOL
Antiv=EDrus, anti-spam e at=E9 100 MB de = espa=E7o. Assine=20 j=E1!
http://email.bol.com.br=20
Este ano a 44a. IMO ser=E1 realizada nos dias 07 a 19 = de=20 julho no Jap=E3o. Mais informa=E7=F5es nos site da OBM http://www.obm.org.br/imo.htm,=  da=20 IMO http://imo.math.ca/, ou no = site oficial=20 do evento: http://www.imo2003.com/ 
 
Um abra=E7o,    =
F=E1bio Maia
----- Original Message -----
From:=20 Blue=20 Ice
Sent: Friday, April 18, 2003 = 5:36=20 PM
Subject: [obm-l] Quando = ser=E1 a=20 olimp=EDada Internacional??

 
------=_NextPart_000_001C_01C30805.BD7B77F0-- ========================================================================= Instruções para entrar na lista, sair da lista e usar a lista em http://www.mat.puc-rio.br/~nicolau/olimp/obm-l.html O administrador desta lista é ========================================================================= From owner-obm-l@sucuri.mat.puc-rio.br Mon Apr 21 13:02:54 2003 Return-Path: Received: (from majordom@localhost) by sucuri.mat.puc-rio.br (8.9.3/8.9.3) id NAA28329 for obm-l-MTTP; Mon, 21 Apr 2003 13:00:22 -0300 Received: from imo-m08.mx.aol.com (imo-m08.mx.aol.com [64.12.136.163]) by sucuri.mat.puc-rio.br (8.9.3/8.9.3) with ESMTP id NAA28325 for ; Mon, 21 Apr 2003 13:00:18 -0300 From: Lltmdrtm@aol.com Received: from Lltmdrtm@aol.com by imo-m08.mx.aol.com (mail_out_v34.22.) id z.174.19c6feb5 (4362) for ; Mon, 21 Apr 2003 11:59:40 -0400 (EDT) Message-ID: <174.19c6feb5.2bd56f6c@aol.com> Date: Mon, 21 Apr 2003 11:59:40 EDT Subject: =?ISO-8859-1?Q?Re:=20[obm-l]=20Re:=20[obm-l]=20D=FAvida=20Sobre=20?= =?ISO-8859-1?Q?a=20Utilidade=20da=20Ma=20tem=E1tica=20Ensinada=20nas=20.?= =?ISO-8859-1?Q?..?= To: obm-l@mat.puc-rio.br MIME-Version: 1.0 Content-Type: multipart/alternative; boundary="part1_174.19c6feb5.2bd56f6c_boundary" X-Mailer: 7.0 for Windows sub 10501 Sender: owner-obm-l@sucuri.mat.puc-rio.br Precedence: bulk Reply-To: obm-l@mat.puc-rio.br --part1_174.19c6feb5.2bd56f6c_boundary Content-Type: text/plain; charset="ISO-8859-1" Content-Transfer-Encoding: quoted-printable Tudo na vida, exige bom senso e flexibilidade. N=E3o podemos nos amarrar a=20 determinados pontos de vista, sem coloc=E1-los a prova. Dizem os mais velhos= =20 que dif=EDcil =E9 come=E7ar, mas, mais dif=EDcil ainda =E9 voc=EA querer ver= =20 imediatamente a utilidade de tudo que se cria, de tudo que se faz. Quando=20 pensamos, automaticamente crescemos. Em sala de aula quando discutimos o=20 assunto n=FAmeros complexos, =E9 muito comum a pergunta: Qual a utilidade? M= uitas=20 vezes nossas limita=E7=F5es nos impede de dar uma resposta naquele momento,=20= mas=20 n=E3o significa que n=E3o exista a tal utilidade. De repente a resposta=20 ultrapassa os limites do universo da pessoa que pergunta. Cuidado com os=20 atropelos. Se meu filho de 5 anos afirma que 3-5=3D2 posso tentar convenc= =EA-lo=20 de que n=E3o =E9 verdade, mas tenho que ter o despreendimento, caso perceba=20= que=20 ele n=E3o tem maturidade naquele momento para compreender meus argumentos e=20= ai=20 tenho que aceitar no sentido de respeitar. Muitas coisas na vida que eu=20 duvidava da exist=EAncia e utilidade, hoje , tenho algumas respostas e outra= s=20 est=E3o por vir. Talvez uma exist=EAncia n=E3o seja suficiente para se ter t= odas as=20 respostas para as nossas indaga=E7=F5es. N=E3o v=EA o Teorema de Fermat, qua= nto tempo=20 demorou para ser demonstrado. Existem muitas coisas em nossas vidas que=20 existem, no entanto n=E3o compreendemos bem naquele momento, e por n=E3o=20 compreender bem, duvidamos de sua exist=EAncia e utilidade. Muitas vezes=20 brinco com determinados colegas, dizendo que a exist=EAncia e utilidade de=20 determinadas coisas =E9 uma quest=E3o de f=E9, porque temos nossas limita= =E7=F5es. De=20 repente pode ocorrer de voc=EA n=E3o usufruir da utilidade, mas os que vir= =E3o ,=20 terem o privil=E9gio. A pergunta: Pra que serve ? =E9 importante existir, ma= s=20 temos que ter a capacidade de reconhecer que a resposta num primeiro momento= ,=20 n=E3o =E9 interessante, n=E3o =E9 completa, pois id=E9ias se modificam, se a= primoram, =20 com o tempo. N=E3o podemos castrar id=E9ias com tais perguntas , ou inibir a= =20 continuidade das coisas com perguntas que exigem respostas imediatas. Quando= =20 jogamos uma partida de futebol, ser=E1 que pensamos mesmo em todos os=20 benef=EDcios que aquele momento nos proporciona. Dizem que a paci=EAncia=20= =E9 uma=20 =E1rvore de raiz amarga, mas de bons frutos: o tempo tem comprovado. Que a=20 for=E7a esteja com voc=EAs. --part1_174.19c6feb5.2bd56f6c_boundary Content-Type: text/html; charset="ISO-8859-1" Content-Transfer-Encoding: quoted-printable Tudo na vida, exige bom senso e flexibilidade. N=E3o podemos nos=20= amarrar a determinados pontos de vista, sem coloc=E1-los a prova. Dizem os m= ais velhos que dif=EDcil =E9 come=E7ar, mas, mais dif=EDcil ainda =E9 voc= =EA querer ver imediatamente a utilidade de tudo que se cria, de tudo que se= faz. Quando pensamos, automaticamente crescemos. Em sala de aula quando dis= cutimos o assunto n=FAmeros complexos, =E9 muito comum a pergunta: Qual a ut= ilidade? Muitas vezes nossas limita=E7=F5es nos impede de dar uma resposta n= aquele momento, mas n=E3o significa que n=E3o exista a tal utilidade. De rep= ente a resposta ultrapassa os limites do universo da pessoa que pergunta. Cu= idado com os atropelos. Se meu filho de 5 anos afirma que 3-5=3D2 posso tent= ar convenc=EA-lo de que n=E3o =E9 verdade, mas tenho que ter o despreendimen= to, caso perceba que ele n=E3o tem maturidade naquele momento para compreend= er meus argumentos e ai tenho que aceitar no sentido de respeitar. Muitas co= isas na vida que eu duvidava da exist=EAncia e utilidade, hoje , tenho algum= as respostas e outras est=E3o por vir. Talvez uma exist=EAncia n=E3o seja su= ficiente para se ter todas as respostas para as nossas indaga=E7=F5es. N=E3o= v=EA o Teorema de Fermat, quanto tempo demorou para ser demonstrado. Existe= m muitas coisas em nossas vidas que existem, no entanto n=E3o compreendemos=20= bem naquele momento, e por n=E3o compreender bem,  duvidamos de sua exi= st=EAncia e utilidade. Muitas vezes brinco com determinados colegas, dizendo= que a exist=EAncia e utilidade de determinadas coisas =E9 uma quest=E3o de=20= f=E9,  porque temos nossas limita=E7=F5es. De repente pode ocorrer de v= oc=EA n=E3o usufruir da utilidade, mas os que vir=E3o , terem o privil=E9gio= . A pergunta: Pra que serve ? =E9 importante existir, mas temos que ter a ca= pacidade de reconhecer que a resposta num primeiro momento, n=E3o =E9 intere= ssante, n=E3o =E9 completa, pois id=E9ias se modificam, se aprimoram, =20= com o tempo. N=E3o podemos castrar id=E9ias com tais perguntas , ou inibir a= continuidade das coisas com perguntas que exigem respostas imediatas. Quand= o jogamos uma partida de futebol, ser=E1 que pensamos mesmo em todos os bene= f=EDcios que aquele momento nos proporciona. Dizem que a paci=EAncia =E9 uma= =E1rvore de raiz amarga, mas de bons frutos: o tempo tem comprovado. Que a=20= for=E7a esteja com voc=EAs. --part1_174.19c6feb5.2bd56f6c_boundary-- ========================================================================= Instruções para entrar na lista, sair da lista e usar a lista em http://www.mat.puc-rio.br/~nicolau/olimp/obm-l.html O administrador desta lista é ========================================================================= From owner-obm-l@sucuri.mat.puc-rio.br Mon Apr 21 13:06:28 2003 Return-Path: Received: (from majordom@localhost) by sucuri.mat.puc-rio.br (8.9.3/8.9.3) id NAA28473 for obm-l-MTTP; Mon, 21 Apr 2003 13:03:52 -0300 Received: from imo-m02.mx.aol.com (imo-m02.mx.aol.com [64.12.136.5]) by sucuri.mat.puc-rio.br (8.9.3/8.9.3) with ESMTP id NAA28469 for ; Mon, 21 Apr 2003 13:03:48 -0300 From: Lltmdrtm@aol.com Received: from Lltmdrtm@aol.com by imo-m02.mx.aol.com (mail_out_v34.22.) id z.1dd.7fa5de0 (4362) for ; Mon, 21 Apr 2003 12:03:07 -0400 (EDT) Message-ID: <1dd.7fa5de0.2bd5703b@aol.com> Date: Mon, 21 Apr 2003 12:03:07 EDT Subject: =?ISO-8859-1?Q?Re:=20[obm-l]=20Re:=20[obm-l]=20D=FAvida=20Sobre=20?= =?ISO-8859-1?Q?a=20Utilidade=20da=20Ma=20tem=E1tica=20Ensinada=20nas=20.?= =?ISO-8859-1?Q?..?= To: obm-l@mat.puc-rio.br MIME-Version: 1.0 Content-Type: multipart/alternative; boundary="part1_1dd.7fa5de0.2bd5703b_boundary" X-Mailer: 7.0 for Windows sub 10501 Sender: owner-obm-l@sucuri.mat.puc-rio.br Precedence: bulk Reply-To: obm-l@mat.puc-rio.br --part1_1dd.7fa5de0.2bd5703b_boundary Content-Type: text/plain; charset="ISO-8859-1" Content-Transfer-Encoding: quoted-printable Tudo na vida, exige bom senso e flexibilidade. N=E3o podemos nos amarrar a=20 determinados pontos de vista, sem coloc=E1-los a prova. Dizem os mais velhos= =20 que dif=EDcil =E9 come=E7ar, mas, mais dif=EDcil ainda =E9 voc=EA querer ver= =20 imediatamente a utilidade de tudo que se cria, de tudo que se faz. Quando=20 pensamos, automaticamente crescemos. Em sala de aula quando discutimos o=20 assunto n=FAmeros complexos, =E9 muito comum a pergunta: Qual a utilidade? M= uitas=20 vezes nossas limita=E7=F5es nos impede de dar uma resposta naquele momento,=20= mas=20 n=E3o significa que n=E3o exista a tal utilidade. De repente a resposta=20 ultrapassa os limites do universo da pessoa que pergunta. Cuidado com os=20 atropelos. Se meu filho de 5 anos afirma que 3-5=3D2 posso tentar convenc= =EA-lo=20 de que n=E3o =E9 verdade, mas tenho que ter o despreendimento, caso perceba=20= que=20 ele n=E3o tem maturidade naquele momento para compreender meus argumentos e=20= ai=20 tenho que aceitar no sentido de respeitar. Muitas coisas na vida que eu=20 duvidava da exist=EAncia e utilidade, hoje , tenho algumas respostas e outra= s=20 est=E3o por vir. Talvez uma exist=EAncia n=E3o seja suficiente para se ter t= odas as=20 respostas para as nossas indaga=E7=F5es. N=E3o v=EA o Teorema de Fermat, qua= nto tempo=20 demorou para ser demonstrado. Existem muitas coisas em nossas vidas que=20 existem, no entanto n=E3o compreendemos bem naquele momento, e por n=E3o=20 compreender bem, duvidamos de sua exist=EAncia e utilidade. Muitas vezes=20 brinco com determinados colegas, dizendo que a exist=EAncia e utilidade de=20 determinadas coisas =E9 uma quest=E3o de f=E9, porque temos nossas limita= =E7=F5es. De=20 repente pode ocorrer de voc=EA n=E3o usufruir da utilidade, mas os que vir= =E3o ,=20 terem o privil=E9gio. A pergunta: Pra que serve ? =E9 importante existir, ma= s=20 temos que ter a capacidade de reconhecer que a resposta num primeiro momento= ,=20 n=E3o =E9 interessante, n=E3o =E9 completa, pois id=E9ias se modificam, se a= primoram, =20 com o tempo. N=E3o podemos castrar id=E9ias com tais perguntas , ou inibir a= =20 continuidade das coisas com perguntas que exigem respostas imediatas. Quando= =20 jogamos uma partida de futebol, ser=E1 que pensamos mesmo em todos os=20 benef=EDcios que aquele momento nos proporciona. Dizem que a paci=EAncia=20= =E9 uma=20 =E1rvore de raiz amarga, mas de bons frutos: o tempo tem comprovado. Que a=20 for=E7a esteja com voc=EAs. --part1_1dd.7fa5de0.2bd5703b_boundary Content-Type: text/html; charset="ISO-8859-1" Content-Transfer-Encoding: quoted-printable Tudo na vida, exige bom senso e flexibilidade. N=E3o podemos nos=20= amarrar a determinados pontos de vista, sem coloc=E1-los a prova. Dizem os m= ais velhos que dif=EDcil =E9 come=E7ar, mas, mais dif=EDcil ainda =E9 voc= =EA querer ver imediatamente a utilidade de tudo que se cria, de tudo que se= faz. Quando pensamos, automaticamente crescemos. Em sala de aula quando dis= cutimos o assunto n=FAmeros complexos, =E9 muito comum a pergunta: Qual a ut= ilidade? Muitas vezes nossas limita=E7=F5es nos impede de dar uma resposta n= aquele momento, mas n=E3o significa que n=E3o exista a tal utilidade. De rep= ente a resposta ultrapassa os limites do universo da pessoa que pergunta. Cu= idado com os atropelos. Se meu filho de 5 anos afirma que 3-5=3D2 posso tent= ar convenc=EA-lo de que n=E3o =E9 verdade, mas tenho que ter o despreendimen= to, caso perceba que ele n=E3o tem maturidade naquele momento para compreend= er meus argumentos e ai tenho que aceitar no sentido de respeitar. Muitas co= isas na vida que eu duvidava da exist=EAncia e utilidade, hoje , tenho algum= as respostas e outras est=E3o por vir. Talvez uma exist=EAncia n=E3o seja su= ficiente para se ter todas as respostas para as nossas indaga=E7=F5es. N=E3o= v=EA o Teorema de Fermat, quanto tempo demorou para ser demonstrado. Existe= m muitas coisas em nossas vidas que existem, no entanto n=E3o compreendemos=20= bem naquele momento, e por n=E3o compreender bem,  duvidamos de sua exi= st=EAncia e utilidade. Muitas vezes brinco com determinados colegas, dizendo= que a exist=EAncia e utilidade de determinadas coisas =E9 uma quest=E3o de=20= f=E9,  porque temos nossas limita=E7=F5es. De repente pode ocorrer de v= oc=EA n=E3o usufruir da utilidade, mas os que vir=E3o , terem o privil=E9gio= . A pergunta: Pra que serve ? =E9 importante existir, mas temos que ter a ca= pacidade de reconhecer que a resposta num primeiro momento, n=E3o =E9 intere= ssante, n=E3o =E9 completa, pois id=E9ias se modificam, se aprimoram, =20= com o tempo. N=E3o podemos castrar id=E9ias com tais perguntas , ou inibir a= continuidade das coisas com perguntas que exigem respostas imediatas. Quand= o jogamos uma partida de futebol, ser=E1 que pensamos mesmo em todos os bene= f=EDcios que aquele momento nos proporciona. Dizem que a paci=EAncia =E9 uma= =E1rvore de raiz amarga, mas de bons frutos: o tempo tem comprovado. Que a=20= for=E7a esteja com voc=EAs.
--part1_1dd.7fa5de0.2bd5703b_boundary-- ========================================================================= Instruções para entrar na lista, sair da lista e usar a lista em http://www.mat.puc-rio.br/~nicolau/olimp/obm-l.html O administrador desta lista é ========================================================================= From owner-obm-l@sucuri.mat.puc-rio.br Mon Apr 21 13:45:29 2003 Return-Path: Received: (from majordom@localhost) by sucuri.mat.puc-rio.br (8.9.3/8.9.3) id NAA29896 for obm-l-MTTP; Mon, 21 Apr 2003 13:40:36 -0300 Received: from artemis.opendf.com.br (artemis.opengate.com.br [200.181.71.14]) by sucuri.mat.puc-rio.br (8.9.3/8.9.3) with ESMTP id NAA29892 for ; Mon, 21 Apr 2003 13:40:32 -0300 Received: from localhost (localhost [127.0.0.1]) by artemis.opendf.com.br (Postfix) with ESMTP id 3C0792BECA for ; Mon, 21 Apr 2003 13:40:08 -0300 (BRT) Received: from artemis.opendf.com.br ([127.0.0.1]) by localhost (artemis.opengate.com.br [127.0.0.1:10024]) (amavisd-new) with ESMTP id 13563-05 for ; Mon, 21 Apr 2003 13:40:07 -0300 (BRT) Received: from computer (200-181-089-149.bsace7001.dsl.brasiltelecom.net.br [200.181.89.149]) by artemis.opendf.com.br (Postfix) with ESMTP id F2DC42BEC2 for ; Mon, 21 Apr 2003 13:40:06 -0300 (BRT) From: "Artur Costa Steiner" To: Subject: RE: [obm-l] Equacao diferencial Date: Mon, 21 Apr 2003 13:40:06 -0300 Organization: Steiner Consultoria LTDA Message-ID: <004801c30824$ab08a060$9865fea9@computer> MIME-Version: 1.0 Content-Type: text/plain; charset="iso-8859-1" X-Priority: 3 (Normal) X-MSMail-Priority: Normal X-Mailer: Microsoft Outlook, Build 10.0.2627 Importance: Normal X-MimeOLE: Produced By Microsoft MimeOLE V6.00.2800.1106 In-Reply-To: <27BD56F8640DD711A4320006295078E50500C1@pssnx1.brahma> X-Virus-Scanned: by amavisd-new Content-Transfer-Encoding: 8bit X-MIME-Autoconverted: from quoted-printable to 8bit by sucuri.mat.puc-rio.br id NAA29893 Sender: owner-obm-l@sucuri.mat.puc-rio.br Precedence: bulk Reply-To: obm-l@mat.puc-rio.br Oi, Sendo V o volume da esfera e S a area de sua superficie, temos pelo enunciado que V' = dV/dt = -KS. K eh uma constante positiva e o sinal de - eh para considerar o fato de o volume diminui ao longo do tempo com a evaporacao. Sendo r o raio da esfera, temos que V= 4/3 pi r^3 e S = 4 pi r^2. Explicite S em funcao de V e substitua na equacao diferencia acima. Deve bater com o livro. Artur -----Original Message----- From: owner-obm-l@sucuri.mat.puc-rio.br [mailto:owner-obm-l@sucuri.mat.puc-rio.br] On Behalf Of Ricardo de Moraes (PS) Sent: Monday, April 21, 2003 1:26 AM To: 'obm-l@mat.puc-rio.br' Subject: [obm-l] Equacao diferencial Boa noite, Nao consegui chegar a mesma resposta que o livro traz para esta questão...   Uma gota esferica evapora a uma taxa proporcional a sua area de superficie. Escreva uma equacao diferencial para o volume de uma gota de chuva em funcao do tempo.   Resposta: V' = - kV^2/3   Obrigado. ========================================================================= Instruções para entrar na lista, sair da lista e usar a lista em http://www.mat.puc-rio.br/~nicolau/olimp/obm-l.html O administrador desta lista é ========================================================================= From owner-obm-l@sucuri.mat.puc-rio.br Mon Apr 21 13:45:41 2003 Return-Path: Received: (from majordom@localhost) by sucuri.mat.puc-rio.br (8.9.3/8.9.3) id NAA29911 for obm-l-MTTP; Mon, 21 Apr 2003 13:41:17 -0300 Received: from hotmail.com (f13.sea1.hotmail.com [207.68.163.13]) by sucuri.mat.puc-rio.br (8.9.3/8.9.3) with ESMTP id NAA29907 for ; Mon, 21 Apr 2003 13:41:13 -0300 Received: from mail pickup service by hotmail.com with Microsoft SMTPSVC; Mon, 21 Apr 2003 09:40:41 -0700 Received: from 198.81.8.1 by sea1fd.sea1.hotmail.msn.com with HTTP; Mon, 21 Apr 2003 16:40:41 GMT X-Originating-IP: [198.81.8.1] X-Originating-Email: [fredericor@hotmail.com] From: "Frederico Reis Marques de Brito" To: obm-l@mat.puc-rio.br Subject: [obm-l] =?iso-8859-1?B?UmU6IFtvYm0tbF0gRMO6dmlkYXMgc29icmUgZ2VvbWV0cmlhLi4u?= Date: Mon, 21 Apr 2003 13:40:41 -0300 Mime-Version: 1.0 Content-Type: text/plain; charset=iso-8859-1; format=flowed Message-ID: X-OriginalArrivalTime: 21 Apr 2003 16:40:41.0872 (UTC) FILETIME=[BF5A5900:01C30824] Sender: owner-obm-l@sucuri.mat.puc-rio.br Precedence: bulk Reply-To: obm-l@mat.puc-rio.br A fórmula da a´rea lateral do cone é obtida, de forma elementar, cortando-se o cone ao longo de uma reta geratriz, isto é, uma reta no cone que passe pelo vértice do cone. Cortando-o conforme essa reta, e "abrindo" o cone, obtemos um setor circular, cujo raio é a altura do cone. Com auxílio de uma regra de três, podemos determinar essa a´rea. Isso ocorre com superfícies "conformes" ao plano, como cilindros ou cones, mas não com a esfera, por exemplo. Desculpe-me mas não entendi a 2a pergunta. E quanto a terceira, um excelente livro de Geometria Plana é o do João Lucas Barbosa, GEOMETRIA PLANA, da SBM. Consta que tb há um bom livro de autoria do Morgado, mas nunca consegui encontrá-lo. Boa Sorte! >From: Rodrigo Badia Piccinini >Reply-To: obm-l@mat.puc-rio.br >To: obm-l@mat.puc-rio.br >Subject: [obm-l] Dúvidas sobre geometria... >Date: Mon, 21 Apr 2003 12:41:36 -0300 > > >Aêe, >>>Gostaria de esclarecer algumas dúvidas sobre geometria: > > -Por quê quando calculamos a área lateral de um cone podemos supor a >planificação do lado como um triângulo, valendo a equação (r*g)/2 ? >Isso vale para outros casos? > > -Se o arco de uma circunferência vale raiz(2)*R, então posso >"esticá-lo" e dizer que sua área é igual a de um triângulo retângulo >de catetos R e hipotenusa raiz(2)*R? > > -Eu estudo eng Mecânica na UFF e não encontrei bons livros de geometria >plana lá... Quais livros são recomendados e onde posso conseguí-los? > >>>>Obrigado! >========================================================================= >Instrues para entrar na lista, sair da lista e usar a lista em >http://www.mat.puc-rio.br/~nicolau/olimp/obm-l.html >O administrador desta lista >========================================================================= _________________________________________________________________ MSN Hotmail, o maior webmail do Brasil. http://www.hotmail.com ========================================================================= Instruções para entrar na lista, sair da lista e usar a lista em http://www.mat.puc-rio.br/~nicolau/olimp/obm-l.html O administrador desta lista é ========================================================================= From owner-obm-l@sucuri.mat.puc-rio.br Mon Apr 21 14:27:58 2003 Return-Path: Received: (from majordom@localhost) by sucuri.mat.puc-rio.br (8.9.3/8.9.3) id OAA31798 for obm-l-MTTP; Mon, 21 Apr 2003 14:25:14 -0300 Received: from mail.fronthost.com (mail.fronthost.com [63.250.6.253]) by sucuri.mat.puc-rio.br (8.9.3/8.9.3) with ESMTP id OAA31791 for ; Mon, 21 Apr 2003 14:25:08 -0300 Received: from [68.49.44.134] by fronthost.com [63.250.6.253] with SmartMax MailMax for at Mon, 21 Apr 2003 13:24:55 -0400 Message-ID: <016001c3082a$984fa170$6401a8c0@TEST4> From: "Alexandre A da Rocha" To: References: <001701c306a6$a04f1160$f4befea9@xx> <3EA1A4D0.4050205@centroin.com.br> <200304191758510720.01B2F31B@smtp.watersportsbrazil.com> <3EA1C7A5.4050409@centroin.com.br> <200304192025370600.0239563D@smtp.watersportsbrazil.com> <3EA1DEEF.8080803@centroin.com.br> Subject: Re: [obm-l] MDC Date: Mon, 21 Apr 2003 13:22:32 -0400 MIME-Version: 1.0 Content-Type: multipart/alternative; boundary="----=_NextPart_000_015D_01C30809.110712F0" X-Priority: 3 X-MSMail-Priority: Normal X-Mailer: Microsoft Outlook Express 5.50.4807.1700 X-MimeOLE: Produced By Microsoft MimeOLE V5.50.4807.1700 Sender: owner-obm-l@sucuri.mat.puc-rio.br Precedence: bulk Reply-To: obm-l@mat.puc-rio.br This is a multi-part message in MIME format. ------=_NextPart_000_015D_01C30809.110712F0 Content-Type: text/plain; charset="iso-8859-1" Content-Transfer-Encoding: quoted-printable Discordo... o problema tem tantas interpretacoes quanto definicaoes da = palavra dia. O fato de que voce escolheu a definicao restritiva de dia = civil, nao anula as demais. Embora o enunciado da questao nao limite, a = alusao a dia como periodo de 24 horas e clara. just my $0.02, Auggy dia=20 do Lat. *dia por dies s. m.,=20 claridade que o Sol envia =E0 Terra; espa=E7o de tempo que decorre entre o nascer e o p=F4r do Sol (dia = natural); Astr.,=20 intervalo de tempo entre duas passagens consecutivas do Sol no mesmo = meridiano (dia solar verdadeiro); espa=E7o de tempo que decorre desde a meia-noite de um dia at=E9 =E0 = meia-noite do dia seguinte (dia civil); per=EDodo de uma rota=E7=E3o terrestre; espa=E7o de 24 horas. ... =20 ----- Original Message -----=20 From: A. C. Morgado=20 To: obm-l@mat.puc-rio.br=20 Sent: Saturday, April 19, 2003 7:42 PM Subject: Re: [obm-l] MDC Eh quase isso ai. Na verdade o problema nao tem duas interpreta=E7oes. = A resposta eh 30. Mas que eh pegadinha, eh. Ariel de Silvio wrote: Enquanto eu fazia a segunda resolu=E7=E3o pensei exatamente o que vc = falou agora, qual seria a resposta do gabarito numa prova... Ser=E1 que = considerariam tal detalhe, que realmente o enunciado leva a duas = interpreta=E7=F5es... quer dizer, uma se vc analizar ele a pe da = letra....=20 com certeza a resposta numa prova seria 15....=20 eh so pensar q 1 dia s=E3o 24h, mas 24h n=E3o s=E3o NECESSARIAMENTE = somente um dia, pode comecar em um e terminar no outro.... se = analizarmos melhor um dia seria 23h59m59s.... mas ai ja eh demais ne.... muito interessante sua "provocacao" hehehe falou Ariel *********** MENSAGEM ORIGINAL *********** As 19:03 de 19/4/2003 A. C. Morgado escreveu: Eh isso aih, garoto! Na realidade eu mandei essa provoca=E7ao porque hoje em dia a = regra em vestibulares e concursos eh a presen=E7a de questoes com = enunciados dubios e o que mais me espanta eh a passividade de alunos e = principalmente de professores diante de tais enunciados. Nao tenho a = menor duvida de que o gabarito oficial de tal questao seja 15. Tambem = nao tenho duvida que isso foi questao de prova e o gabarito 15 foi = aceito sem grandes contesta=E7oes. Eh preciso protestar contra essas = coisas. []s Morgado Ariel de Silvio wrote: Logo apos enviar o outro email, analizei com outros olhos, e o = que vc diz tem sentido... mesmo sem saber sua justificativa.... Se vc considerar o inicio das 24 horas, meia noite... como ele = diz q eh um dia, consideremos o dia oficial, eh esse o caminho? Sendo das 0h as 24h... O turno da Torre Leste inicia-se =E0s 22h, portanto at=E9 as 24 = hrs se passaram 2h apenas... isso significa q todos os turnos devem = durar apenas 2h para serem iguais e ninguem virar o dia de guarda... Ent=E3o soma-se as 60h de guarda, q deve dividida por turnos de = 2h, resultando em 30 escoteiros... O que salva essa resolu=E7=E3o =E9 o finalzinho do enunciado = "n=E3o d=EA duas guardas NO MESMO DIA"... Agora ficou certo?? []s Ariel *********** MENSAGEM ORIGINAL *********** As 16:34 de 19/4/2003 A. C. Morgado escreveu: Soh uma provoca=E7aozinha para agu=E7ar o espirito critico dos = companheiros. Eu acho (acho eh bondade minha; eu tenho certeza!) que a = resposta do item b eh 30. Morgado Blue Ice wrote: ----- Original Message -----=20 From: Faelccmm@aol.com=20 To: obm-l@mat.puc-rio.br=20 Sent: Saturday, April 19, 2003 3:07 PM Subject: [obm-l] MDC ------------------------------------------------------------------ E-mail Premium BOL=20 Antiv=EDrus, anti-spam e at=E9 100 MB de espa=E7o. Assine = j=E1! http://email.bol.com.br=20 ------------------------------------------------------------------ Ol=E1 pessoal,=20 Vejam a quest=E3o:=20 Um grupo de escoteiros deve montar guarda nos 4 postos = Norte, Sul, Leste e Oeste de seu acampamento. No posto Norte a guarda = deve ser dia e noite; no posto Sul a guarda vai das 6 da manh=E3 =E0s 18 = da tarde; no posto Leste, das 22 da noite =E0s 6 da manh=E3 e no posto = Oeste, das 14 da tarde =E0s 6 da manh=E3. Qual deve ser o turno m=E1ximo = de guarda, de modo que todos permane=E7am igual tempo de sentinela (sem = mudar de lugar) ? Quantos escoteiros s=E3o necess=E1rios para 24 horas = de guarda, de modo que para escoteiro n=E3o d=EA duas guardas no mesmo = dia ?=20 Obs: A primeira pergunta eu resolvi tirando o m.d.c das = amplitudes hor=E1rias de cada posto de guarda e cheguei =E0 resposta do = livro R: 4 horas. Mas n=E3o estou conseguindo resolver a outra pergunta. = resp:=20 a) 4 horas=20 b) 15 escoteiros(*) =20 Solu=E7=E3o:somei todos(norte,sul,leste,oeste) e dividi = por 4: N24+S12+L8+016/4: 60/4:15=20 ------=_NextPart_000_015D_01C30809.110712F0 Content-Type: text/html; charset="iso-8859-1" Content-Transfer-Encoding: quoted-printable
Discordo... o problema tem tantas = interpretacoes=20 quanto definicaoes da palavra dia.  O fato de que voce escolheu a = definicao=20 restritiva de dia civil, nao anula as demais. Embora o enunciado da = questao nao=20 limite, a alusao a dia como periodo de 24 horas e clara.
 
just my $0.02,
Auggy
 
dia
 
do Lat. *dia por = dies

s.=20 m.,=20
claridade que o Sol envia =E0 Terra;
espa=E7o de tempo que decorre entre o nascer e o p=F4r do Sol (dia = natural);
Astr.,=20
intervalo de tempo entre duas passagens consecutivas do Sol no = mesmo=20 meridiano (dia solar verdadeiro);
espa=E7o de tempo que decorre desde a meia-noite de um dia at=E9 = =E0 meia-noite=20 do dia seguinte (dia civil);
per=EDodo de uma rota=E7=E3o terrestre;
espa=E7o de 24 horas.
...
 
----- Original Message -----
From:=20 A. C.=20 Morgado
Sent: Saturday, April 19, 2003 = 7:42=20 PM
Subject: Re: [obm-l] MDC

Eh quase isso ai. Na verdade o problema nao tem duas=20 interpreta=E7oes. A resposta eh 30. Mas que eh pegadinha, = eh.

Ariel de=20 Silvio wrote:
Enquanto eu fazia a segunda resolu=E7=E3o pensei exatamente o = que vc falou=20 agora, qual seria a resposta do gabarito numa prova... Ser=E1 que=20 considerariam tal detalhe, que realmente o enunciado leva a duas=20 interpreta=E7=F5es... quer dizer, uma se vc analizar ele a pe da = letra....=20
com certeza a resposta numa prova seria 15....
 
eh so pensar q 1 dia s=E3o 24h, mas 24h n=E3o s=E3o = NECESSARIAMENTE somente=20 um dia, pode comecar em um e terminar no outro.... se analizarmos = melhor um=20 dia seria 23h59m59s.... mas ai ja eh demais ne....
 
muito interessante sua "provocacao" hehehe
 
falou
Ariel
 

*********** MENSAGEM ORIGINAL=20 ***********

As 19:03 de 19/4/2003 A. C. Morgado=20 escreveu:
Eh=20 isso aih, garoto!
Na realidade eu mandei essa provoca=E7ao = porque hoje em=20 dia a regra em vestibulares e concursos eh a presen=E7a de = questoes com=20 enunciados dubios e o que mais me espanta eh a passividade de = alunos e=20 principalmente de professores diante de tais enunciados. Nao tenho = a menor=20 duvida de que o gabarito oficial de tal questao seja 15. Tambem = nao tenho=20 duvida que isso foi questao de prova e o gabarito 15 foi aceito = sem=20 grandes contesta=E7oes. Eh preciso protestar contra essas=20 coisas.
[]s
Morgado

Ariel de Silvio wrote:
Logo apos enviar o outro email, analizei com outros olhos, = e o que=20 vc diz tem sentido... mesmo sem saber sua = justificativa....
 
Se vc considerar o inicio das 24 horas, meia noite... como = ele diz=20 q eh um dia, consideremos o dia oficial, eh esse o = caminho?
Sendo das 0h as 24h...
O turno da Torre Leste inicia-se =E0s 22h, portanto at=E9 = as 24 hrs se=20 passaram 2h apenas... isso significa q todos os turnos devem = durar=20 apenas 2h para serem iguais e ninguem virar o dia de = guarda...
 
Ent=E3o=20 soma-se as 60h de guarda, q deve dividida por turnos de 2h, = resultando=20 em 30 escoteiros...
 
O que salva essa resolu=E7=E3o =E9 o finalzinho do = enunciado "n=E3o d=EA duas guardas NO MESMO = DIA"...
 
Agora ficou = certo??
 
[]s
Ariel
 
*********** MENSAGEM ORIGINAL=20 ***********

As 16:34 de 19/4/2003 A. C. Morgado=20 escreveu:
Soh=20 uma provoca=E7aozinha para agu=E7ar o espirito critico dos = companheiros.=20 Eu acho (acho eh bondade minha; eu tenho certeza!) que a = resposta do=20 item b eh 30.
Morgado

Blue Ice wrote:
 
-----=20 Original Message ----- From:=20 Faelccmm@aol.com = To:=20 obm-l@mat.puc-rio.br Sent:=20 Saturday, April 19, 2003 3:07 PM Subject:=20 [obm-l] MDC


E-mail Premium BOL
Antiv=EDrus, anti-spam e at=E9 = 100 MB de=20 espa=E7o. Assine j=E1!
http://email.bol.com.br

Ol=E1 = pessoal,=20

Vejam a quest=E3o:

Um grupo de escoteiros = deve=20 montar guarda nos 4 postos Norte, Sul, Leste e Oeste de = seu=20 acampamento. No posto Norte a guarda deve ser dia e noite; = no=20 posto Sul a guarda vai das 6 da manh=E3 =E0s 18 da tarde; = no posto=20 Leste, das 22 da noite =E0s 6 da manh=E3 e no posto Oeste, = das 14 da=20 tarde =E0s 6 da manh=E3. Qual deve ser o turno m=E1ximo de = guarda, de=20 modo que todos permane=E7am igual tempo de sentinela (sem = mudar de=20 lugar) ? Quantos escoteiros s=E3o necess=E1rios para 24 = horas de=20 guarda, de modo que para escoteiro n=E3o d=EA duas guardas = no mesmo=20 dia ?

Obs: A primeira pergunta eu resolvi =  tirando o=20  m.d.c das amplitudes hor=E1rias de cada posto de = guarda e=20 cheguei =E0 resposta do livro R: 4 horas. Mas n=E3o estou = conseguindo=20 resolver a outra pergunta.

resp:

a) 4 = horas
b)=20 15 escoteiros(*)
 
 
 
 
Solu=E7=E3o:somei=20 todos(norte,sul,leste,oeste) e dividi  por = 4:
 
N24+S12+L8+016/4:
60/4:15
 
 
 
 
=
 

=

------=_NextPart_000_015D_01C30809.110712F0-- ========================================================================= Instruções para entrar na lista, sair da lista e usar a lista em http://www.mat.puc-rio.br/~nicolau/olimp/obm-l.html O administrador desta lista é ========================================================================= From owner-obm-l@sucuri.mat.puc-rio.br Mon Apr 21 15:04:19 2003 Return-Path: Received: (from majordom@localhost) by sucuri.mat.puc-rio.br (8.9.3/8.9.3) id PAA32623 for obm-l-MTTP; Mon, 21 Apr 2003 15:02:00 -0300 Received: from traven10.uol.com.br (traven10.uol.com.br [200.221.29.45]) by sucuri.mat.puc-rio.br (8.9.3/8.9.3) with ESMTP id PAA32618 for ; Mon, 21 Apr 2003 15:01:56 -0300 Received: from atila ([200.100.193.103]) by traven10.uol.com.br (8.9.1/8.9.1) with SMTP id PAA10197 for ; Mon, 21 Apr 2003 15:01:25 -0300 (BRT) Message-ID: <001b01c30830$4d5563c0$67c164c8@atila> From: "H. Batista" To: References: <590HDooKj1664S20.1050419370@uwdvg007.cms.usa.net> <20030416043622.42259.qmail@web13705.mail.yahoo.com> <20030416100912.GA21143@linux.ime.usp.br> <007501c3043e$ea749c20$0e00a8c0@hotlink.com.br> <20030416181208.B2662@sucuri.mat.puc-rio.br> <001a01c30463$05622f60$0e00a8c0@hotlink.com.br> <20030416214351.A9315@sucuri.mat.puc-rio.br> <006701c3059f$fb5a1670$158c000a@computador> Subject: Re: [obm-l] riemann Date: Mon, 21 Apr 2003 15:03:22 -0300 MIME-Version: 1.0 Content-Type: text/plain; charset="iso-8859-1" Content-Transfer-Encoding: 8bit X-Priority: 3 X-MSMail-Priority: Normal X-Mailer: Microsoft Outlook Express 6.00.2600.0000 X-MimeOLE: Produced By Microsoft MimeOLE V6.00.2600.0000 Sender: owner-obm-l@sucuri.mat.puc-rio.br Precedence: bulk Reply-To: obm-l@mat.puc-rio.br Olá Felipe, Você encontra os sete problemas no endereço abaixo.... Divirta-se... http://www.zaz.com.br/istoe/1609/comportamento/1609bons.htm Helena ========================================================================= Instruções para entrar na lista, sair da lista e usar a lista em http://www.mat.puc-rio.br/~nicolau/olimp/obm-l.html O administrador desta lista é ========================================================================= From owner-obm-l@sucuri.mat.puc-rio.br Mon Apr 21 15:16:46 2003 Return-Path: Received: (from majordom@localhost) by sucuri.mat.puc-rio.br (8.9.3/8.9.3) id PAA00452 for obm-l-MTTP; Mon, 21 Apr 2003 15:15:04 -0300 Received: from web40105.mail.yahoo.com (web40105.mail.yahoo.com [66.218.78.39]) by sucuri.mat.puc-rio.br (8.9.3/8.9.3) with SMTP id PAA00448 for ; Mon, 21 Apr 2003 15:15:00 -0300 Message-ID: <20030421181428.60521.qmail@web40105.mail.yahoo.com> Received: from [200.216.81.173] by web40105.mail.yahoo.com via HTTP; Mon, 21 Apr 2003 11:14:28 PDT Date: Mon, 21 Apr 2003 11:14:28 -0700 (PDT) From: fabio fortes Subject: Re: [obm-l] Re: [obm-l] Dúvida Sobre a Utilidade da Ma temática Ensinada nas ... To: obm-l@mat.puc-rio.br In-Reply-To: <1dd.7fa5de0.2bd5703b@aol.com> MIME-Version: 1.0 Content-Type: text/plain; charset=us-ascii Sender: owner-obm-l@sucuri.mat.puc-rio.br Precedence: bulk Reply-To: obm-l@mat.puc-rio.br mesmo que você não goste de matemática, você irá precisar dela para processos de seleção de emprego (toda empresa pede que vc faça um teste de raciocínio lógico e quantitativo), para praticamente todos os cursos de graduação (o único que ela não é usada me parece ser direito), terá mais facilidade em investir seu dinheiro, perceberá truques de lojistas, não será roubado na conta do barzinho e inclusive será beneficiado em áreas que a princípio nada tem a ver com a matéria, como escrever melhor (vc aprende a articular melhor seus pensamentos) e com as mulheres ( vc usa probabilidade e teoria dos jogos) abraços __________________________________________________ Do you Yahoo!? The New Yahoo! Search - Faster. Easier. Bingo http://search.yahoo.com ========================================================================= Instruções para entrar na lista, sair da lista e usar a lista em http://www.mat.puc-rio.br/~nicolau/olimp/obm-l.html O administrador desta lista é ========================================================================= From owner-obm-l@sucuri.mat.puc-rio.br Mon Apr 21 16:20:36 2003 Return-Path: Received: (from majordom@localhost) by sucuri.mat.puc-rio.br (8.9.3/8.9.3) id QAA02126 for obm-l-MTTP; Mon, 21 Apr 2003 16:18:54 -0300 Received: from trex-b.centroin.com.br (trex-b.centroin.com.br [200.225.63.136]) by sucuri.mat.puc-rio.br (8.9.3/8.9.3) with ESMTP id QAA02122 for ; Mon, 21 Apr 2003 16:18:51 -0300 Received: from centroin.com.br (RJ091120.user.veloxzone.com.br [200.141.91.120] (may be forged)) (authenticated bits=0) by trex-b.centroin.com.br (8.12.9/8.12.9) with ESMTP id h3LJIKVi026096 for ; Mon, 21 Apr 2003 16:18:21 -0300 (EST) Message-ID: <3EA44437.5060608@centroin.com.br> Date: Mon, 21 Apr 2003 16:19:19 -0300 From: "A. C. Morgado" User-Agent: Mozilla/5.0 (Windows; U; Windows NT 5.0; en-US; rv:1.0.2) Gecko/20030208 Netscape/7.02 X-Accept-Language: en-us, en MIME-Version: 1.0 To: obm-l@mat.puc-rio.br Subject: Re: [obm-l] Re: [obm-l] =?ISO-8859-1?Q?D=FAvida_Sobre_a_Ut?= =?ISO-8859-1?Q?ilidade_da_Ma_tem=E1tica_Ensinada_nas_=2E=2E=2E?= References: <20030421181428.60521.qmail@web40105.mail.yahoo.com> Content-Type: text/plain; charset=ISO-8859-1; format=flowed Content-Transfer-Encoding: 8bit Sender: owner-obm-l@sucuri.mat.puc-rio.br Precedence: bulk Reply-To: obm-l@mat.puc-rio.br Acrescentando: no curso de direito da FGV-RJ, que pretende formar especialistas em direito empresarial, ha a disciplina Matematica Financeira. fabio fortes wrote: >mesmo que você não goste de matemática, você irá >precisar dela para processos de seleção de emprego >(toda empresa pede que vc faça um teste de raciocínio >lógico e quantitativo), para praticamente todos os >cursos de graduação (o único que ela não é usada me >parece ser direito), terá mais facilidade em investir >seu dinheiro, perceberá truques de lojistas, não será >roubado na conta do barzinho e inclusive será >beneficiado em áreas que a princípio nada tem a ver >com a matéria, como escrever melhor (vc aprende a >articular melhor seus pensamentos) e com as mulheres ( >vc usa probabilidade e teoria dos jogos) >abraços > > >__________________________________________________ >Do you Yahoo!? >The New Yahoo! Search - Faster. Easier. Bingo >http://search.yahoo.com >========================================================================= >Instruções para entrar na lista, sair da lista e usar a lista em >http://www.mat.puc-rio.br/~nicolau/olimp/obm-l.html >O administrador desta lista é >========================================================================= > > > > ========================================================================= Instruções para entrar na lista, sair da lista e usar a lista em http://www.mat.puc-rio.br/~nicolau/olimp/obm-l.html O administrador desta lista é ========================================================================= From owner-obm-l@sucuri.mat.puc-rio.br Mon Apr 21 16:56:26 2003 Return-Path: Received: (from majordom@localhost) by sucuri.mat.puc-rio.br (8.9.3/8.9.3) id QAA02753 for obm-l-MTTP; Mon, 21 Apr 2003 16:54:04 -0300 Received: from trex-b.centroin.com.br (trex-b.centroin.com.br [200.225.63.136]) by sucuri.mat.puc-rio.br (8.9.3/8.9.3) with ESMTP id QAA02749 for ; Mon, 21 Apr 2003 16:54:01 -0300 Received: from centroin.com.br (RJ091120.user.veloxzone.com.br [200.141.91.120] (may be forged)) (authenticated bits=0) by trex-b.centroin.com.br (8.12.9/8.12.9) with ESMTP id h3LJrUVi026608 for ; Mon, 21 Apr 2003 16:53:30 -0300 (EST) Message-ID: <3EA44C75.9050405@centroin.com.br> Date: Mon, 21 Apr 2003 16:54:29 -0300 From: "A. C. Morgado" User-Agent: Mozilla/5.0 (Windows; U; Windows NT 5.0; en-US; rv:1.0.2) Gecko/20030208 Netscape/7.02 X-Accept-Language: en-us, en MIME-Version: 1.0 To: obm-l@mat.puc-rio.br Subject: Re: [obm-l] MDC References: <001701c306a6$a04f1160$f4befea9@xx> <3EA1A4D0.4050205@centroin.com.br> <200304191758510720.01B2F31B@smtp.watersportsbrazil.com> <3EA1C7A5.4050409@centroin.com.br> <200304192025370600.0239563D@smtp.watersportsbrazil.com> <3EA1DEEF.8080803@centroin.com.br> <016001c3082a$984fa170$6401a8c0@TEST4> Content-Type: multipart/alternative; boundary="------------040709090805040704040003" Sender: owner-obm-l@sucuri.mat.puc-rio.br Precedence: bulk Reply-To: obm-l@mat.puc-rio.br --------------040709090805040704040003 Content-Type: text/plain; charset=ISO-8859-1; format=flowed Content-Transfer-Encoding: 8bit Alexandre, voce esta apelando e sabe que esta apelando. Ao contrario do que voce diz, a alusao a dia como periodo de 24 horas nao eh clara, muito pelo contrario: o enunciado fala em 24 horas de guarda e duas guardas no mesmo dia; poderia ter falado duas guardas nesse periodo. Alem do mais, a discussao nao eh propriamente sobre o significado de dia e sim sobre o significado de "no mesmo dia". Parece uma pessoa que eu conheço (e alguns da lista tambem) que fuma sem parar e diz que so fuma um cigarro por dia, que jamais fuma dois cigarros no mesmo dia e que depois de haver acendido um cigarro as 9h, ao ser surpreendida acendendo um cigarro as 9h5min diz que o primeiro eh do dia (periodo de 24 horas) que se iniciou as 9h e que o outro eh do dia que se iniciou as 9h5min e que esse dias sao diferentes, ou seja, nao sao identicos, e que nao tem culpa de eles nao serem disjuntos. Ou como o garoto que recebe uma caixa de chocolates e proibiçao da mae de comer mais de um no mesmo dia e come dois em seguida. Pela sua forçada interpretaçao, o garoto nao descumpriu a recomendaçao materna. Mas admitamos, para argumentar, que voce tenha razao. Chegamos ao ponto crucial: questoes nao devem ser enunciadas desse jeito dubio e eh preciso reclamar de coisas assim. Essa era a finalidade da minha provocaçao. Alexandre A da Rocha wrote: > Discordo... o problema tem tantas interpretacoes quanto definicaoes da > palavra dia. O fato de que voce escolheu a definicao restritiva de > dia civil, nao anula as demais. Embora o enunciado da questao nao > limite, a alusao a dia como periodo de 24 horas e clara. > > just my $0.02, > Auggy > > dia > > do Lat. *dia por dies > > s. m., > claridade que o Sol envia à Terra; > > espaço de tempo que decorre entre o nascer e o pôr do Sol (dia > natural); > Astr., > intervalo de tempo entre duas passagens consecutivas do Sol no > mesmo meridiano (dia solar verdadeiro); > > espaço de tempo que decorre desde a meia-noite de um dia até à > meia-noite do dia seguinte (dia civil); > > período de uma rotação terrestre; > > espaço de 24 horas. > ... > >> Olá pessoal, >> >> Vejam a questão: >> >> Um grupo de escoteiros deve montar guarda nos 4 postos Norte, >> Sul, Leste e Oeste de seu acampamento. No posto Norte a guarda >> deve ser dia e noite; no posto Sul a guarda vai das 6 da manhã às >> 18 da tarde; no posto Leste, das 22 da noite às 6 da manhã e no >> posto Oeste, das 14 da tarde às 6 da manhã. Qual deve ser o turno >> máximo de guarda, de modo que todos permaneçam igual tempo de >> sentinela (sem mudar de lugar) ? Quantos escoteiros são >> necessários para 24 horas de guarda, de modo que para escoteiro >> não dê duas guardas no mesmo dia ? >> ------------------------------------------------------------------------ >> --------------040709090805040704040003 Content-Type: text/html; charset=us-ascii Content-Transfer-Encoding: 7bit Alexandre,
voce esta apelando e sabe que esta apelando. Ao contrario do que voce diz, a alusao a dia como periodo de 24 horas nao eh clara, muito pelo contrario: o enunciado fala em 24 horas de guarda e duas guardas no mesmo dia; poderia ter falado duas guardas nesse periodo. Alem do mais, a discussao nao eh propriamente sobre o significado de dia e sim sobre o significado de "no mesmo dia". Parece uma pessoa que eu conheço (e alguns da lista tambem) que fuma sem parar e diz que so fuma um cigarro por dia, que jamais fuma dois cigarros no mesmo dia e que depois de haver acendido um cigarro as 9h, ao ser surpreendida acendendo um cigarro as 9h5min diz que o primeiro eh do dia (periodo de 24 horas) que se iniciou as 9h e que o outro eh do dia que se iniciou as 9h5min e que esse dias sao diferentes, ou seja, nao sao identicos, e que nao tem culpa de eles nao serem disjuntos.
Ou como o garoto que recebe uma caixa de chocolates e proibiçao da mae de comer mais de um no mesmo dia e come dois em seguida. Pela sua forçada interpretaçao, o garoto nao descumpriu a recomendaçao materna.
Mas admitamos, para argumentar, que voce tenha razao. Chegamos ao ponto crucial: questoes nao devem ser enunciadas desse jeito dubio e eh preciso reclamar de coisas assim. Essa era a finalidade da minha provocaçao.

Alexandre A da Rocha wrote:
Discordo... o problema tem tantas interpretacoes quanto definicaoes da palavra dia.  O fato de que voce escolheu a definicao restritiva de dia civil, nao anula as demais. Embora o enunciado da questao nao limite, a alusao a dia como periodo de 24 horas e clara.
 
just my $0.02,
Auggy
 
dia
 
do Lat. *dia por dies

s. m.,
claridade que o Sol envia à Terra;

espaço de tempo que decorre entre o nascer e o pôr do Sol (dia natural);
Astr.,
intervalo de tempo entre duas passagens consecutivas do Sol no mesmo meridiano (dia solar verdadeiro);

espaço de tempo que decorre desde a meia-noite de um dia até à meia-noite do dia seguinte (dia civil);

período de uma rotação terrestre;

espaço de 24 horas.
... 
Olá pessoal,

Vejam a questão:

Um grupo de escoteiros deve montar guarda nos 4 postos Norte, Sul, Leste e Oeste de seu acampamento. No posto Norte a guarda deve ser dia e noite; no posto Sul a guarda vai das 6 da manhã às 18 da tarde; no posto Leste, das 22 da noite às 6 da manhã e no posto Oeste, das 14 da tarde às 6 da manhã. Qual deve ser o turno máximo de guarda, de modo que todos permaneçam igual tempo de sentinela (sem mudar de lugar) ? Quantos escoteiros são necessários para 24 horas de guarda, de modo que para escoteiro não dê duas guardas no mesmo dia ?

--------------040709090805040704040003-- ========================================================================= Instruções para entrar na lista, sair da lista e usar a lista em http://www.mat.puc-rio.br/~nicolau/olimp/obm-l.html O administrador desta lista é ========================================================================= From owner-obm-l@sucuri.mat.puc-rio.br Mon Apr 21 18:00:40 2003 Return-Path: Received: (from majordom@localhost) by sucuri.mat.puc-rio.br (8.9.3/8.9.3) id RAA04384 for obm-l-MTTP; Mon, 21 Apr 2003 17:58:51 -0300 Received: from mail.fronthost.com (mail.fronthost.com [63.250.6.253]) by sucuri.mat.puc-rio.br (8.9.3/8.9.3) with ESMTP id RAA04380 for ; Mon, 21 Apr 2003 17:58:46 -0300 Received: from [68.49.44.134] by fronthost.com [63.250.6.253] with SmartMax MailMax for at Mon, 21 Apr 2003 16:58:34 -0400 Message-ID: <01cc01c30848$70f15ec0$6401a8c0@TEST4> From: "Alexandre A da Rocha" To: References: <001701c306a6$a04f1160$f4befea9@xx> <3EA1A4D0.4050205@centroin.com.br> <200304191758510720.01B2F31B@smtp.watersportsbrazil.com> <3EA1C7A5.4050409@centroin.com.br> <200304192025370600.0239563D@smtp.watersportsbrazil.com> <3EA1DEEF.8080803@centroin.com.br> <016001c3082a$984fa170$6401a8c0@TEST4> <3EA44C75.9050405@centroin.com.br> Subject: Re: [obm-l] MDC Date: Mon, 21 Apr 2003 16:56:11 -0400 MIME-Version: 1.0 Content-Type: multipart/alternative; boundary="----=_NextPart_000_01C9_01C30826.E9AA56E0" X-Priority: 3 X-MSMail-Priority: Normal X-Mailer: Microsoft Outlook Express 5.50.4807.1700 X-MimeOLE: Produced By Microsoft MimeOLE V5.50.4807.1700 Sender: owner-obm-l@sucuri.mat.puc-rio.br Precedence: bulk Reply-To: obm-l@mat.puc-rio.br This is a multi-part message in MIME format. ------=_NextPart_000_01C9_01C30826.E9AA56E0 Content-Type: text/plain; charset="iso-8859-1" Content-Transfer-Encoding: quoted-printable :), Auggy ----- Original Message -----=20 From: A. C. Morgado=20 To: obm-l@mat.puc-rio.br=20 Sent: Monday, April 21, 2003 3:54 PM Subject: Re: [obm-l] MDC Alexandre, voce esta apelando e sabe que esta apelando. Ao contrario do que voce = diz, a alusao a dia como periodo de 24 horas nao eh clara, muito pelo = contrario: o enunciado fala em 24 horas de guarda e duas guardas no = mesmo dia; poderia ter falado duas guardas nesse periodo. Alem do mais, = a discussao nao eh propriamente sobre o significado de dia e sim sobre o = significado de "no mesmo dia". Parece uma pessoa que eu conhe=E7o (e = alguns da lista tambem) que fuma sem parar e diz que so fuma um cigarro = por dia, que jamais fuma dois cigarros no mesmo dia e que depois de = haver acendido um cigarro as 9h, ao ser surpreendida acendendo um = cigarro as 9h5min diz que o primeiro eh do dia (periodo de 24 horas) que = se iniciou as 9h e que o outro eh do dia que se iniciou as 9h5min e que = esse dias sao diferentes, ou seja, nao sao identicos, e que nao tem = culpa de eles nao serem disjuntos. Ou como o garoto que recebe uma caixa de chocolates e proibi=E7ao da = mae de comer mais de um no mesmo dia e come dois em seguida. Pela sua = for=E7ada interpreta=E7ao, o garoto nao descumpriu a recomenda=E7ao = materna. Mas admitamos, para argumentar, que voce tenha razao. Chegamos ao = ponto crucial: questoes nao devem ser enunciadas desse jeito dubio e eh = preciso reclamar de coisas assim. Essa era a finalidade da minha = provoca=E7ao.=20 Alexandre A da Rocha wrote: Discordo... o problema tem tantas interpretacoes quanto definicaoes = da palavra dia. O fato de que voce escolheu a definicao restritiva de = dia civil, nao anula as demais. Embora o enunciado da questao nao = limite, a alusao a dia como periodo de 24 horas e clara. just my $0.02, Auggy dia=20 =20 do Lat. *dia por dies s. m.,=20 claridade que o Sol envia =E0 Terra; espa=E7o de tempo que decorre entre o nascer e o p=F4r do Sol (dia = natural); Astr.,=20 intervalo de tempo entre duas passagens consecutivas do Sol no = mesmo meridiano (dia solar verdadeiro); espa=E7o de tempo que decorre desde a meia-noite de um dia at=E9 = =E0 meia-noite do dia seguinte (dia civil); per=EDodo de uma rota=E7=E3o terrestre; espa=E7o de 24 horas. ... =20 Ol=E1 pessoal,=20 Vejam a quest=E3o:=20 Um grupo de escoteiros deve montar guarda nos 4 postos Norte, = Sul, Leste e Oeste de seu acampamento. No posto Norte a guarda deve ser = dia e noite; no posto Sul a guarda vai das 6 da manh=E3 =E0s 18 da = tarde; no posto Leste, das 22 da noite =E0s 6 da manh=E3 e no posto = Oeste, das 14 da tarde =E0s 6 da manh=E3. Qual deve ser o turno m=E1ximo = de guarda, de modo que todos permane=E7am igual tempo de sentinela (sem = mudar de lugar) ? Quantos escoteiros s=E3o necess=E1rios para 24 horas = de guarda, de modo que para escoteiro n=E3o d=EA duas guardas no mesmo = dia ?=20 ------------------------------------------------------------------------ ------=_NextPart_000_01C9_01C30826.E9AA56E0 Content-Type: text/html; charset="iso-8859-1" Content-Transfer-Encoding: quoted-printable
:),
Auggy
----- Original Message -----
From:=20 A. C.=20 Morgado
Sent: Monday, April 21, 2003 = 3:54=20 PM
Subject: Re: [obm-l] MDC

Alexandre,
voce esta apelando e sabe que esta = apelando. Ao=20 contrario do que voce diz, a alusao a dia como periodo de 24 horas nao = eh=20 clara, muito pelo contrario: o enunciado fala em 24 horas de guarda e = duas=20 guardas no mesmo dia; poderia ter falado duas guardas nesse periodo. = Alem do=20 mais, a discussao nao eh propriamente sobre o significado de dia e sim = sobre o=20 significado de "no mesmo dia". Parece uma pessoa que eu conhe=E7o (e = alguns da=20 lista tambem) que fuma sem parar e diz que so fuma um cigarro por dia, = que=20 jamais fuma dois cigarros no mesmo dia e que depois de haver acendido = um=20 cigarro as 9h, ao ser surpreendida acendendo um cigarro as 9h5min diz = que o=20 primeiro eh do dia (periodo de 24 horas) que se iniciou as 9h e que o = outro eh=20 do dia que se iniciou as 9h5min e que esse dias sao diferentes, ou = seja, nao=20 sao identicos, e que nao tem culpa de eles nao serem disjuntos.
Ou = como o=20 garoto que recebe uma caixa de chocolates e proibi=E7ao da mae de = comer mais de=20 um no mesmo dia e come dois em seguida. Pela sua for=E7ada = interpreta=E7ao, o=20 garoto nao descumpriu a recomenda=E7ao materna.
Mas admitamos, para = argumentar, que voce tenha razao. Chegamos ao ponto crucial: questoes = nao=20 devem ser enunciadas desse jeito dubio e eh preciso reclamar de coisas = assim.=20 Essa era a finalidade da minha provoca=E7ao.

Alexandre A da = Rocha=20 wrote:
Discordo... o problema tem tantas = interpretacoes quanto definicaoes da palavra dia.  O fato de = que voce=20 escolheu a definicao restritiva de dia civil, nao anula as demais. = Embora o=20 enunciado da questao nao limite, a alusao a dia como periodo de 24 = horas e=20 clara.
 
just my $0.02,
Auggy
 
dia
 
do Lat. *dia=20 por dies

s. m.,
claridade que o Sol envia =E0=20 Terra;

espa=E7o de tempo que decorre = entre o nascer e=20 o p=F4r do Sol (dia natural);
Astr.,
intervalo de tempo entre duas = passagens=20 consecutivas do Sol no mesmo meridiano (dia solar = verdadeiro);

espa=E7o de tempo que decorre = desde a=20 meia-noite de um dia at=E9 =E0 meia-noite do dia seguinte (dia=20 civil);

per=EDodo de uma rota=E7=E3o = terrestre;

espa=E7o de 24 horas.
... 
Ol=E1 pessoal, =

Vejam=20 a quest=E3o:

Um grupo de escoteiros deve montar guarda = nos 4=20 postos Norte, Sul, Leste e Oeste de seu acampamento. No posto = Norte a=20 guarda deve ser dia e noite; no posto Sul a guarda vai das 6 da = manh=E3 =E0s=20 18 da tarde; no posto Leste, das 22 da noite =E0s 6 da manh=E3 e = no posto=20 Oeste, das 14 da tarde =E0s 6 da manh=E3. Qual deve ser o turno = m=E1ximo de=20 guarda, de modo que todos permane=E7am igual tempo de sentinela = (sem mudar=20 de lugar) ? Quantos escoteiros s=E3o necess=E1rios para 24 horas = de guarda,=20 de modo que para escoteiro n=E3o d=EA duas guardas no mesmo dia=20 ?
=20
------=_NextPart_000_01C9_01C30826.E9AA56E0-- ========================================================================= Instruções para entrar na lista, sair da lista e usar a lista em http://www.mat.puc-rio.br/~nicolau/olimp/obm-l.html O administrador desta lista é ========================================================================= From owner-obm-l@sucuri.mat.puc-rio.br Mon Apr 21 18:29:16 2003 Return-Path: Received: (from majordom@localhost) by sucuri.mat.puc-rio.br (8.9.3/8.9.3) id SAA05281 for obm-l-MTTP; Mon, 21 Apr 2003 18:27:57 -0300 Received: from trex-b.centroin.com.br (trex-b.centroin.com.br [200.225.63.136]) by sucuri.mat.puc-rio.br (8.9.3/8.9.3) with ESMTP id SAA05276 for ; Mon, 21 Apr 2003 18:27:54 -0300 Received: from centroin.com.br (RJ091120.user.veloxzone.com.br [200.141.91.120] (may be forged)) (authenticated bits=0) by trex-b.centroin.com.br (8.12.9/8.12.9) with ESMTP id h3LLROVi027948 for ; Mon, 21 Apr 2003 18:27:24 -0300 (EST) Message-ID: <3EA46277.3050004@centroin.com.br> Date: Mon, 21 Apr 2003 18:28:23 -0300 From: "A. C. Morgado" User-Agent: Mozilla/5.0 (Windows; U; Windows NT 5.0; en-US; rv:1.0.2) Gecko/20030208 Netscape/7.02 X-Accept-Language: en-us, en MIME-Version: 1.0 To: obm-l@mat.puc-rio.br Subject: Re: [obm-l] 4 coisinhas References: <20030421151630.66488.qmail@web13007.mail.yahoo.com> Content-Type: multipart/alternative; boundary="------------040303030707050108080300" Sender: owner-obm-l@sucuri.mat.puc-rio.br Precedence: bulk Reply-To: obm-l@mat.puc-rio.br --------------040303030707050108080300 Content-Type: text/plain; charset=ISO-8859-1; format=flowed Content-Transfer-Encoding: 8bit Um processo eficiente e menos conhecido do que deveria para calcular essas somas polinomiais eh escrever o polinomio em potencias fatoriais e nao em potencias ordinarias: Potencia ordinaria: x^3 = x*x*x Potencia fatorial : (x)^3 = x*(x-1)*(x-2) na realidade a notaçao usual eh (x) com indice 3, que se le x baixado a 3. Uma conta boba transforma x^3 = x(x-1)(x-2) + 3x(x-1) + x [x^3 = ax(x-1)(x-2) + bx(x-1) + cx + d, obriga os polinomios a serem identicos e pronto] Portanto, S = somatorio de x(x-1)(x-2) + 3x(x-1) + x = (x)^3 + 3 (x)^2 + (x)^1, x variando de 1 a n (aqui as potencias sao fatoriais) Eh (muito) facil provar que se F(x) eh uma antidiferença de f(x) (ou seja, se f(x) eh uma diferença de F(x) ) (ou seja, se F(x+1) - F(x) = f(x) ), somatorio de 1 a n de f(x) eh igual a F(n+1) - F(1). Realmente, f(1) + f(2) +...+f(n) = F(2) - F(1) + F(3) - F(2) + ...+ F(n+1) - F(n) = F(n+1) - F(1). Isso eh conhecido como teorema fundamental da somaçao e eh analogo ao teorema fundamental do calculo integral. Agora, para potencias fatoriais eh facil provar que diferença de (x)^n = n*(x)^(n-1) e antidiferença de (x)^n = [(x)^(n+1)]/(n+1) + constante (note a analogia com derivada e integral) Portanto, S = somatorio de (x)^3 + 3 (x)^2 + (x)^1, (x variando de 1 a n) = F(n+1) - F(1) sendo F(x) = (x)^4 /4 + (x)^3 + (x)^2 /2 = x(x-1)(x-2)(x-3)/4 + x(x-1)(x-2) + x(x-1)/2. Calculando F(n+1) - F(1) obtem-se [n^2] * [(n+1)^2] /4 Helder Suzuki wrote: >4) >Calcular S = 1 + 8 + 27 + ... + n^3 > >Abraços, >Helder Toshiro Suzuki > > > --------------040303030707050108080300 Content-Type: text/html; charset=us-ascii Content-Transfer-Encoding: 7bit Um processo eficiente e menos conhecido do que deveria para calcular essas somas polinomiais eh escrever o polinomio em potencias fatoriais e nao em potencias ordinarias:
Potencia ordinaria: x^3 = x*x*x
Potencia fatorial : (x)^3 = x*(x-1)*(x-2)  na realidade a notaçao usual eh (x) com indice 3, que se le x baixado a 3.
Uma conta boba transforma x^3 = x(x-1)(x-2) + 3x(x-1) + x  [x^3 = ax(x-1)(x-2) + bx(x-1) + cx + d, obriga os polinomios a serem identicos e pronto]
Portanto, S = somatorio de x(x-1)(x-2) + 3x(x-1) + x = (x)^3 + 3 (x)^2 + (x)^1, x variando de 1 a n (aqui as potencias sao fatoriais)
Eh (muito) facil provar que se F(x) eh uma antidiferença de f(x) (ou seja, se f(x) eh uma diferença de F(x) ) (ou seja, se
F(x+1) - F(x) = f(x) ), somatorio de 1 a n de f(x) eh igual a F(n+1) - F(1). Realmente, f(1) + f(2) +...+f(n) = F(2) - F(1) + F(3) - F(2) + ...+ F(n+1) - F(n) = F(n+1) - F(1). Isso eh conhecido como teorema fundamental da somaçao e eh analogo ao teorema fundamental do calculo integral.
Agora, para potencias fatoriais eh facil provar que diferença de (x)^n = n*(x)^(n-1) e antidiferença de (x)^n = [(x)^(n+1)]/(n+1) + constante (note a analogia com derivada e integral)
Portanto, S = somatorio de (x)^3 + 3 (x)^2 + (x)^1, (x variando de 1 a n)  = F(n+1) - F(1) sendo
F(x) = (x)^4 /4 + (x)^3 + (x)^2 /2 = x(x-1)(x-2)(x-3)/4 + x(x-1)(x-2) + x(x-1)/2.
Calculando F(n+1) - F(1) obtem-se  [n^2] * [(n+1)^2] /4


Helder Suzuki wrote:
4)
Calcular S = 1 + 8 + 27 + ... + n^3

Abraços,
Helder Toshiro Suzuki

  

--------------040303030707050108080300-- ========================================================================= Instruções para entrar na lista, sair da lista e usar a lista em http://www.mat.puc-rio.br/~nicolau/olimp/obm-l.html O administrador desta lista é ========================================================================= From owner-obm-l@sucuri.mat.puc-rio.br Mon Apr 21 18:45:24 2003 Return-Path: Received: (from majordom@localhost) by sucuri.mat.puc-rio.br (8.9.3/8.9.3) id SAA05704 for obm-l-MTTP; Mon, 21 Apr 2003 18:44:03 -0300 Received: from trex-b.centroin.com.br (trex-b.centroin.com.br [200.225.63.136]) by sucuri.mat.puc-rio.br (8.9.3/8.9.3) with ESMTP id SAA05698 for ; Mon, 21 Apr 2003 18:44:00 -0300 Received: from centroin.com.br (RJ091120.user.veloxzone.com.br [200.141.91.120] (may be forged)) (authenticated bits=0) by trex-b.centroin.com.br (8.12.9/8.12.9) with ESMTP id h3LLhUVi028167 for ; Mon, 21 Apr 2003 18:43:30 -0300 (EST) Message-ID: <3EA4663C.4060809@centroin.com.br> Date: Mon, 21 Apr 2003 18:44:28 -0300 From: "A. C. Morgado" User-Agent: Mozilla/5.0 (Windows; U; Windows NT 5.0; en-US; rv:1.0.2) Gecko/20030208 Netscape/7.02 X-Accept-Language: en-us, en MIME-Version: 1.0 To: obm-l@mat.puc-rio.br Subject: Re: [obm-l] 4 coisinhas References: <20030421151630.66488.qmail@web13007.mail.yahoo.com> Content-Type: text/plain; charset=ISO-8859-1; format=flowed Content-Transfer-Encoding: 8bit Sender: owner-obm-l@sucuri.mat.puc-rio.br Precedence: bulk Reply-To: obm-l@mat.puc-rio.br 2) Acho (?) que ha um artigo do Pitombeira sobre o problema 2 em uma RPM antiga, com o titulo "o jogo de Euclides". Peço ajuda os colegas organizados para ver se eh isso mesmo e qual eh o numero da RPM. 3) N>1 Seja f(n) a resposta. Pegue a casa 11 da ponta do retangulo. Ela pode ser coberta com um retangulo vertical, ou com um horizontal (isso força o preenchimento das casa 21 e 22 com um retangulo horizontal) ou por um quadrado. f(n) = f(n-1) + f(n-2) + f(n-2) f(n) - f(n-1) - 2f(n-2) = 0 com f(2) = 3 e f(3) = 5 Salvo erro de conta da [ (-1)^n + 2 (2^n) ] /3 Helder Suzuki wrote: >1) >De quantas formas podemos dividir um retangulo 2xN em >partes de 2x1 e/ou 2x2? > >2) >Dois jogadores começam com dois inteiros, a e b, >a>b>0. >Em cada jogada um jogador subtrai do maior numero um >multiplo positivo do menor numero, tal que o resultado >nao seja negativo. >Se um jogador conseguir deixar um 0, ele ganha o jogo. > >Por exemplo, se A e B estao jogando e A comeca com >a=25 e b=7: > >25 7 <- inicial >11 7 <- A retira 2*7 de 25 = 11 > 4 7 <- B retira 1*7 de 11 = 4 > 4 3 <- A retira 1*4 de 7 = 3 > 1 3 <- B retira 1*3 de 4 = 1 > 1 0 <- A retira 3*1 de 3 e ganha o jogo > >Assumindo que A e B joguem perfeitamente. Se A sempre >começa o jogo, que condições a e b devem respeitar >para que A ganhe? e para que B ganhe? > >Abraços, >Helder Toshiro Suzuki > >_______________________________________________________________________ > > > ========================================================================= Instruções para entrar na lista, sair da lista e usar a lista em http://www.mat.puc-rio.br/~nicolau/olimp/obm-l.html O administrador desta lista é ========================================================================= From owner-obm-l@sucuri.mat.puc-rio.br Mon Apr 21 19:16:49 2003 Return-Path: Received: (from majordom@localhost) by sucuri.mat.puc-rio.br (8.9.3/8.9.3) id TAA06796 for obm-l-MTTP; Mon, 21 Apr 2003 19:14:12 -0300 Received: (from nicolau@localhost) by sucuri.mat.puc-rio.br (8.9.3/8.9.3) id TAA06791 for obm-l@mat.puc-rio.br; Mon, 21 Apr 2003 19:14:11 -0300 Date: Mon, 21 Apr 2003 19:14:11 -0300 From: "Nicolau C. Saldanha" To: obm-l@mat.puc-rio.br Subject: Re: [obm-l] L.I Message-ID: <20030421191411.C6341@sucuri.mat.puc-rio.br> References: Mime-Version: 1.0 Content-Type: text/plain; charset=iso-8859-1 Content-Disposition: inline Content-Transfer-Encoding: 8bit User-Agent: Mutt/1.2.5i In-Reply-To: ; from marcio.lis@bol.com.br on Fri, Apr 18, 2003 at 01:05:54PM -0300 Sender: owner-obm-l@sucuri.mat.puc-rio.br Precedence: bulk Reply-To: obm-l@mat.puc-rio.br On Fri, Apr 18, 2003 at 01:05:54PM -0300, marcio.lis wrote: > em uma questão do livro do elon de algébra linear ele > pede p/ mostrar que:(1,e^x,e2^x,e^3x,e^4x) é L.I e > coloca como sugestão sair derivando e dividindo por e^x > e consegui chegar a solucao por ai porém eu gostaria de > saber se eu posso fazer da seguinte forma:substituir e^x > por y ai fica um polinômio em y de um lado e do outro > lado 0.eu posso concluir por igualdade de polinômio que > os coeficientes são todos nulos?e que portanto o > conjunto é L.I Uma generalização natural desta questão é provar que as funções f_a(x) = exp(ax) são todas LI. Meu método favorito de resolver é olhar o comportamento em +infinito. Mais exatamente, suponha que (c1 f_a1 + c2 f_a2 + ... + cn f_an)(x) = 0 para todo x, a1 < a2 < ... < an, todos os ck, k = 1..n, diferentes de 0. Então lim_{x -> infinito}(c1 f_a1 + c2 f_a2 + ... + cn f_an)(x) / f_an(x) = 0 Mas lim_{x -> infinito}(c1 f_a1 + c2 f_a2 + ... + cn f_an)(x) / f_an(x) = c1 lim ( f_a1(x)/f_an(x) ) + ... + cn lim ( f_an(x)/f_an(x) ) = 0 + 0 + ... + 0 + cn Donde cn = 0, absurdo. []s, N. ========================================================================= Instruções para entrar na lista, sair da lista e usar a lista em http://www.mat.puc-rio.br/~nicolau/olimp/obm-l.html O administrador desta lista é ========================================================================= From owner-obm-l@sucuri.mat.puc-rio.br Mon Apr 21 19:30:41 2003 Return-Path: Received: (from majordom@localhost) by sucuri.mat.puc-rio.br (8.9.3/8.9.3) id TAA07201 for obm-l-MTTP; Mon, 21 Apr 2003 19:28:57 -0300 Received: from hotmail.com (f85.law10.hotmail.com [64.4.15.85]) by sucuri.mat.puc-rio.br (8.9.3/8.9.3) with ESMTP id TAA07197 for ; Mon, 21 Apr 2003 19:28:53 -0300 Received: from mail pickup service by hotmail.com with Microsoft SMTPSVC; Mon, 21 Apr 2003 15:28:22 -0700 Received: from 200.147.46.139 by lw10fd.law10.hotmail.msn.com with HTTP; Mon, 21 Apr 2003 22:28:22 GMT X-Originating-IP: [200.147.46.139] X-Originating-Email: [rhel2002@hotmail.com] From: "Rubens Vilhena" To: obm-l@mat.puc-rio.br Subject: Re: [obm-l] 4 coisinhas Date: Mon, 21 Apr 2003 22:28:22 +0000 Mime-Version: 1.0 Content-Type: text/plain; charset=iso-8859-1; format=flowed Message-ID: X-OriginalArrivalTime: 21 Apr 2003 22:28:22.0296 (UTC) FILETIME=[51227180:01C30855] Sender: owner-obm-l@sucuri.mat.puc-rio.br Precedence: bulk Reply-To: obm-l@mat.puc-rio.br 2) Acho (?) que ha um artigo do Pitombeira sobre o problema 2 em uma RPM antiga, com o titulo "o jogo de Euclides". Peço ajuda os colegas organizados para ver se eh isso mesmo e qual eh o numero da RPM. Respondendo apenas sobre a RPM onde está o artigo, é a de número 14. []' _________________________________________________________________ MSN Hotmail, o maior webmail do Brasil. http://www.hotmail.com ========================================================================= Instruções para entrar na lista, sair da lista e usar a lista em http://www.mat.puc-rio.br/~nicolau/olimp/obm-l.html O administrador desta lista é ========================================================================= From owner-obm-l@sucuri.mat.puc-rio.br Mon Apr 21 19:31:55 2003 Return-Path: Received: (from majordom@localhost) by sucuri.mat.puc-rio.br (8.9.3/8.9.3) id TAA07254 for obm-l-MTTP; Mon, 21 Apr 2003 19:30:37 -0300 Received: (from nicolau@localhost) by sucuri.mat.puc-rio.br (8.9.3/8.9.3) id TAA07247 for obm-l@mat.puc-rio.br; Mon, 21 Apr 2003 19:30:36 -0300 Date: Mon, 21 Apr 2003 19:30:36 -0300 From: "Nicolau C. Saldanha" To: obm-l@mat.puc-rio.br Subject: [obm-l] Ensino de matematica no Brasil Message-ID: <20030421193036.D6341@sucuri.mat.puc-rio.br> Mime-Version: 1.0 Content-Type: text/plain; charset=iso-8859-1 Content-Disposition: inline Content-Transfer-Encoding: 8bit User-Agent: Mutt/1.2.5i Sender: owner-obm-l@sucuri.mat.puc-rio.br Precedence: bulk Reply-To: obm-l@mat.puc-rio.br Não é exatamente o assunto da lista, mas a Suely Druck (presidente da SBM) publicou um artigo na Folha sobre o ensino de matemática no Brasil. As olimpíadas de matemática são mencionadas mas não são o tema do artigo. Nem todos os leitores gostaram gostaram. Vejam os links abaixo (obrigado ao Gugu pela referência) vale a pena ler e refletir, talvez participar do debate. []s, N. http://www1.folha.uol.com.br/folha/sinapse/ult1063u343.shtml http://www.jornaldaciencia.org.br/Detalhe.jsp?id=8868 http://www.jornaldaciencia.org.br/Detalhe.jsp?id=9238 http://www.jornaldaciencia.org.br/Detalhe.jsp?id=9105 ========================================================================= Instruções para entrar na lista, sair da lista e usar a lista em http://www.mat.puc-rio.br/~nicolau/olimp/obm-l.html O administrador desta lista é ========================================================================= From owner-obm-l@sucuri.mat.puc-rio.br Mon Apr 21 20:28:21 2003 Return-Path: Received: (from majordom@localhost) by sucuri.mat.puc-rio.br (8.9.3/8.9.3) id UAA09141 for obm-l-MTTP; Mon, 21 Apr 2003 20:26:33 -0300 Received: from lampiao.digi.com.br (lampiao.digi.com.br [200.241.100.60]) by sucuri.mat.puc-rio.br (8.9.3/8.9.3) with ESMTP id UAA09137 for ; Mon, 21 Apr 2003 20:26:30 -0300 Received: from p8c7y1fe9gyrelp.digi.com.br (host221.d.digizap.com.br [200.249.6.221]) by lampiao.digi.com.br (8.11.6/8.11.6) with ESMTP id h3LNKin10919 for ; Mon, 21 Apr 2003 20:20:49 -0300 Message-Id: <5.2.0.9.0.20030421201915.00b1b518@mail.digi.com.br> X-Sender: benedito@mail.digi.com.br X-Mailer: QUALCOMM Windows Eudora Version 5.2.0.9 Date: Mon, 21 Apr 2003 20:22:14 -0300 To: obm-l@mat.puc-rio.br From: benedito Subject: Re: [obm-l] Ensino de matematica no Brasil In-Reply-To: <20030421193036.D6341@sucuri.mat.puc-rio.br> Mime-Version: 1.0 Content-Type: text/plain; charset="iso-8859-1"; format=flowed X-MailScanner: Found to be clean Content-Transfer-Encoding: 8bit X-MIME-Autoconverted: from quoted-printable to 8bit by sucuri.mat.puc-rio.br id UAA09138 Sender: owner-obm-l@sucuri.mat.puc-rio.br Precedence: bulk Reply-To: obm-l@mat.puc-rio.br Muita gente pode não ter gostado....Mas as declarações da Presidente tem muita verdade, isso tem. Incomoda alguns, especialmente aqueles que focam o ensino nos métodos, esquecendo o principal: o conteúdo... Benedito Freire At 19:30 21/4/2003 -0300, you wrote: >Não é exatamente o assunto da lista, mas a Suely Druck (presidente da SBM) >publicou um artigo na Folha sobre o ensino de matemática no Brasil. >As olimpíadas de matemática são mencionadas mas não são o tema do artigo. >Nem todos os leitores gostaram gostaram. Vejam os links abaixo >(obrigado ao Gugu pela referência) vale a pena ler e refletir, >talvez participar do debate. []s, N. > >http://www1.folha.uol.com.br/folha/sinapse/ult1063u343.shtml >http://www.jornaldaciencia.org.br/Detalhe.jsp?id=8868 >http://www.jornaldaciencia.org.br/Detalhe.jsp?id=9238 >http://www.jornaldaciencia.org.br/Detalhe.jsp?id=9105 > > > >========================================================================= >Instruções para entrar na lista, sair da lista e usar a lista em >http://www.mat.puc-rio.br/~nicolau/olimp/obm-l.html >O administrador desta lista é >========================================================================= ========================================================================= Instruções para entrar na lista, sair da lista e usar a lista em http://www.mat.puc-rio.br/~nicolau/olimp/obm-l.html O administrador desta lista é ========================================================================= From owner-obm-l@sucuri.mat.puc-rio.br Mon Apr 21 21:39:31 2003 Return-Path: Received: (from majordom@localhost) by sucuri.mat.puc-rio.br (8.9.3/8.9.3) id VAA10176 for obm-l-MTTP; Mon, 21 Apr 2003 21:37:20 -0300 Received: from Euler.impa.br (euler.impa.br [147.65.1.3]) by sucuri.mat.puc-rio.br (8.9.3/8.9.3) with ESMTP id VAA10172 for ; Mon, 21 Apr 2003 21:37:17 -0300 Received: from Gauss.impa.br (Gauss [147.65.4.1]) by Euler.impa.br (8.11.6p2/8.11.6) with ESMTP id h3M0ak015171 for ; Mon, 21 Apr 2003 21:36:47 -0300 (EST) From: Carlos Gustavo Tamm de Araujo Moreira Received: by Gauss.impa.br (8.11.6p2) id h3M0aZK07271; Mon, 21 Apr 2003 21:36:35 -0300 (EST) Message-Id: <200304220036.h3M0aZK07271@Gauss.impa.br> Subject: Re: [obm-l] FW: Integral da funcao de Planck To: obm-l@mat.puc-rio.br Date: Mon, 21 Apr 2003 21:36:34 -0300 (EST) In-Reply-To: <000901c30378$84d471e0$28029b9b@LeandroRecova> from "=?iso-8859-1?Q?Leandro_Lacorte_Rec=F4va?=" at Apr 15, 3 10:57:45 am X-Mailer: ELM [version 2.4 PL25] MIME-Version: 1.0 Content-Type: text/plain; charset=US-ASCII Content-Transfer-Encoding: 7bit Sender: owner-obm-l@sucuri.mat.puc-rio.br Precedence: bulk Reply-To: obm-l@mat.puc-rio.br Bem, a integral em questao e' algo como Integral(0 a infinito)(a.v^3/(e^(bv)-1) dv), onde a=8.pi.h/c^3 e b=h/kT. Fazendo bv=x, dv=dx/b, nossa integral fica (supondo, e' claro, que b>0, senao a integral nao converge) (a/b^4).Integral(0 a infinito)(x^3/(e^x-1))dx, mas, escrevendo 1/(e^x-1)=soma(n=1 a infinito)(e^(-nx)), temos que nossa integral e' (a/b^4)soma(n=1 a infinito)(6/n^4) (pois, fazendo y=nx, vemos que integral(0 a infinito)(x^3.e^(-nx) dx)= =(1/n^4).integral(0 a infinito)(x^3.e^(-x)dx)=6/n^4, pois, para todo k, integral(0 a infinito)(x^k.e^(-x)dx)=k! (inducao, funcao gama). Como soma(n=1 a infinito)(1/n^4)=pi^4/90, nossa integral e' (a.pi^4)/(15.b^4))=(8.pi^5.(kT)^4)/(15.(ch)^3). Abracos, Gugu > >A integral abaixo parece ser um bom exercicio para os membros de >plantao.=20 > >=20 > >Regards, > >=20 > >Leandro > >=20 > >-----Original Message----- >From: Frederico Elsner [mailto:frederico_elsner@msn.com]=20 >Sent: Tuesday, April 15, 2003 10:49 AM >To: Leandro Lacorte Rec=F4va >Subject: Integral da funcao de Planck > >=20 > >Leandro, > >=20 > >A integral abaixo =E9 bastante interessante, e =E9 aquela de que eu te = >falei > >=20 > > > >=20 > >O que acha? > >=20 > >Frederico > ========================================================================= Instruções para entrar na lista, sair da lista e usar a lista em http://www.mat.puc-rio.br/~nicolau/olimp/obm-l.html O administrador desta lista é ========================================================================= From owner-obm-l@sucuri.mat.puc-rio.br Mon Apr 21 21:41:06 2003 Return-Path: Received: (from majordom@localhost) by sucuri.mat.puc-rio.br (8.9.3/8.9.3) id VAA10232 for obm-l-MTTP; Mon, 21 Apr 2003 21:39:48 -0300 Received: from hotmail.com (f31.law12.hotmail.com [64.4.19.31]) by sucuri.mat.puc-rio.br (8.9.3/8.9.3) with ESMTP id VAA10227 for ; Mon, 21 Apr 2003 21:39:44 -0300 Received: from mail pickup service by hotmail.com with Microsoft SMTPSVC; Mon, 21 Apr 2003 17:39:13 -0700 Received: from 200.151.104.83 by lw12fd.law12.hotmail.msn.com with HTTP; Tue, 22 Apr 2003 00:39:12 GMT X-Originating-IP: [200.151.104.83] X-Originating-Email: [felipensador@hotmail.com] From: "felipe mendona" To: obm-l@mat.puc-rio.br Subject: [obm-l] re:2 coisinhas Date: Mon, 21 Apr 2003 21:39:12 -0300 Mime-Version: 1.0 Content-Type: text/html; charset=iso-8859-1 Message-ID: X-OriginalArrivalTime: 22 Apr 2003 00:39:13.0163 (UTC) FILETIME=[989DADB0:01C30867] Sender: owner-obm-l@sucuri.mat.puc-rio.br Precedence: bulk Reply-To: obm-l@mat.puc-rio.br
          
         
                    Fala Helder .....  na paz?//!!
 
       
                      
                              Rapaz , fiz o 1 e o ultimo, ve se concorda comigo:
  
                         
                 1-)       Consideremos [A,B] como o conjunto de peças 2x1 e 2x2   que definem um quadriculado   2xN , ou seja , [A,B] indica que precisamos de A peças 2x1 e B peças 2x2 para se contruir um quadriculado 2xN.
                                               Com isso em mente , vamos analizar o caso que N é par e o caso que N é impar separadamente:
 
                                 *N par:
                                                                        
   *----Usando apenas peças 2x1>>>>De uma só maneira constroi-se um quadriculado 2xN.
   *----Usando apenas peças 2x2>>>>De uma só maneira constroi-se um quadriculado 2xN.
   *----Usando simultaneamente peças 2x1  e  2x2>>>>Podemos dizer  que [2,(n-2)/2] define um quadriculado 2xN ,ou seja ,podemos construir o quadriculado 2xN usando 2 peças 2x1   e  (n-2)/2 peças  2x2 .Podemos dizer tambem que [4,(n-4)/2] define um quadriculado 2xN. Estendendo esse raciocinio , chegaremos a um conjunto S :                                         S={[2,(n-2)/2],[4,(n-4)/2],..............,[(n-4),2],[(n-2),1]}onde cada elemento define um quadriculado 2xN. Temos que o numero de elementos de S determina o numero de maneiras que podemos construir o quadriculado 2xN usando simultaneamente as peças 2x1 e 2x2. Dai vem que , podemos construir de (n-2)/2 maneiras o quadriculado 2xN, quando N é par , usando simultaneamente peças 2x1 e 2x2.
         Podemos construir de (n-2)/2 + 2 maneiras o quadriculado 2xN quando N é par.
 
 
                               *N impar
 
    
*----Usando apenas peças 2x1>>>>De uma só maneira constroi-se um quadriculado 2xN.
*----Usando apenas peças 2x2>>>>Impossivel construir um quadriculado 2xN.
*----Usando simultaneamente peças 2x1 e 2x2>>>>Podemos dizer que [1,(n-1)/2] define o quadriculado 2xN , assim como [3,(n-3)/2] ..........................  assim como [(n-2),1] ou seja ,
o numero de elementos do conjunto T={1,(n-1)/2],[3,(n-3)/2],................,[(n-4),2],[(n-2),1]} determina o numero de maneiras de se contruir o quadriculado 2xN usando simultaneamente peças 2x1 e 2x2,esse numero é:(n-1)/2
          Podemos construir de (n-1)/2 +1 maneiras o quadriculado 2xN quando N é impar.
 
 
----------------------------------------------------------------------------------------------
 
              2-) Voce pode calcular 1^3 + 2^3 + ..................+ n^3 =f(n) de diversas formas , uma delas é voce procurar calcular por induçao.Primeiramente voce tem que tomar algo como hipotese.Como fazer isso?Eu aconselho voce nesses casos procurar uma lei de formaçao para cubos pequenos,ou seja , pegando por exemplo 2^3 ,voce percebera que 2^3 = 3 + 5 ,
assim como se voce pegar 3^3 , vc vera que 3^3 = 7 + 9 + 11 , assim como 1^3 =1.O que vc observa nesses casos ? Se observa que os cubos sao resultado aparente da soma dos termos de uma PA de razao 2 ( hipotese) , se observa (para n={1,2,3})que n^3 é resultado da soma dos termos de uma PA de razao 2 com n termos, onde a_1 = 1 + 2.[1+2+3+.......+(n-1)] = n^2 - n + 1 =a_1 (hipotese ).
        Temos pelo menos 2 hipoteses embasadas no que observamos:
*(hipotese 1)---- n^3 é resultado da soma de uma PA de n termos.
           *(hipotese 2)---- n^3 é resultado da soma de uma PA de n termos onde:                a_1  = n^2 - n +1 e  a_n = n^2 + n -1.
 
                    Vamos agora calcular a soma dos termos de uma PA de n termos de razao 2 , com a_1 = n^2-n+1 e a_n = n^2 +n -1.(Estamos supondo que a soma dos termos dessa PA seja n^3.)           
                                   s_n = n . (n^2 -n + 1 + n^2 +n - 1)/2 = n . (2.n^2)/2 = n . n^2 = n^3 
                       s_n = n^3   Nossa hipotese inicial é verdadeira!!!Ou seja, podemos escrever 1^3 = 1 ,  2^3 = 3 +5 ,  3^3 = 7 + 9 +11 ,    4^3 = 13 + 15 +17 +19 .............................  
 .............,n^3 = (n^2 - n + 1 ) + (n^2 - n +1 +2.1 ) +.......................+[n^2 -n +1+2(n-1)] .
 
       Calcular 1^3 + 2^3 + ....................+ n^3 = f(n) se torna equivalente a calcular 1 + 3 + 5 + ...............................+ (n^2 + n - 1)>>>........>>>----------f(n)=n.(n+1).n.(n+1)/4.
  
[(n^2 +n)/2]^2 = 1^3 + 2^3 + ........................+n^3.
  1      
3              5     
7                    9                 11    
13                            15              17                       19           
..............................................................................................................
..........................................................................................................................................
 
Forte abraço.                                           
                                                               
                                                        Felipe Mendonça    Vitória - ES.


MSN Hotmail, o maior webmail do Brasil. Faça o seu agora. smart spam protection and 2 months FREE* ========================================================================= Instruções para entrar na lista, sair da lista e usar a lista em http://www.mat.puc-rio.br/~nicolau/olimp/obm-l.html O administrador desta lista é ========================================================================= From owner-obm-l@sucuri.mat.puc-rio.br Mon Apr 21 22:09:27 2003 Return-Path: Received: (from majordom@localhost) by sucuri.mat.puc-rio.br (8.9.3/8.9.3) id WAA10985 for obm-l-MTTP; Mon, 21 Apr 2003 22:08:04 -0300 Received: from Euler.impa.br (euler.impa.br [147.65.1.3]) by sucuri.mat.puc-rio.br (8.9.3/8.9.3) with ESMTP id WAA10981 for ; Mon, 21 Apr 2003 22:08:01 -0300 Received: from Gauss.impa.br (Gauss [147.65.4.1]) by Euler.impa.br (8.11.6p2/8.11.6) with ESMTP id h3M17U017151 for ; Mon, 21 Apr 2003 22:07:30 -0300 (EST) From: Carlos Gustavo Tamm de Araujo Moreira Received: by Gauss.impa.br (8.11.6p2) id h3M17J210776; Mon, 21 Apr 2003 22:07:19 -0300 (EST) Message-Id: <200304220107.h3M17J210776@Gauss.impa.br> Subject: Re: [obm-l] serie interessante To: obm-l@mat.puc-rio.br Date: Mon, 21 Apr 2003 22:07:17 -0300 (EST) In-Reply-To: <014901c30527$3ba77700$5400a8c0@ensrbr> from "Luis Lopes" at Apr 17, 3 06:20:55 pm X-Mailer: ELM [version 2.4 PL25] MIME-Version: 1.0 Content-Type: text/plain; charset=US-ASCII Content-Transfer-Encoding: 7bit Sender: owner-obm-l@sucuri.mat.puc-rio.br Precedence: bulk Reply-To: obm-l@mat.puc-rio.br Saudacoes, Eu gostei muito desse problema. De fato e' possivel provar isso usando tecnicas de analise complexa (residuos; pode-se comecar observando que o residuo de 1/(tan(x)-x) em x_k solucao nao-nula de tan(x)=x e' 1/(x_k)^2, pois a derivada de tan(x)-x em x_k e' sec^2(x_k)-1=tan^2(x_k)=(x_k)^2, mas da' um pouco de trabalho provar tudo.). Os x_k sao todos transcendentes, pois tan(x)=x significa (e^(ix)-e^(-ix))/(e^(ix)+e^(-ix))=ix, donde (1-ix)e^(ix)-(1+ix)e^(-ix)=0. Se x fosse um algebrico nao nulo,isso contradiria uma das formas do teorema de Hermite-Lindemann-Weierstrass (que tambem implica que e e pi sao transcendentes): se a_1,a_2,...,a_n sao algebricos distintos entao e^(a_1),e^(a_2),...,e^(a_n) sao linearmente independentes sobre os algebricos. Ha' uma prova desse teorema na dissertacao de mestrado do Joao Pedro (que foi meu aluno no IMPA e agora esta' em Cambridge): http://www.preprint.impa.br/Shadows/SERIE_B/2002/3.html Abracos, Gugu P.S.:8=pi+(4-pi)+pi/2+(4-pi)/2+pi/4+(4-pi)/4+... tambem e' uma soma infinita de termos transcendentes que da' racional... (:-) > >Sauda,c~oes, > >O professor Rousseau acabou de me mandar >o seguinte email: > >Dear Luis: > > Thanks. I am just writing up a solution set for one of >my classes that includes a most remarkable sum. >Unfortunately, I don't know even an Eulerian approach to >this problem; only complex analysis works as far as >I know. Anyway, here is the series > >\sum_{k=1}^{\infty} 1/x_k^2 = 1/10, > >where 0 < x_1 < x_2 < x_3 < \cdots are the positive >roots of the equation x = \tan x. What's so remarkable >about this series is that we don't have a precise value for >any one of the terms (although they can be approximated >to many decimal places by numerical computation), but >we do have a precise value (1/10) for the sum. >Something to ponder. > >Cecil > >Comentário: não sei se os x_k e os 1/x_k^2 são >transcendentes mas se forem, teríamos uma soma >infinita de termos transcendentes dando um número >racional. > >[]'s >Luís > > >========================================================================= >Instruções para entrar na lista, sair da lista e usar a lista em >http://www.mat.puc-rio.br/~nicolau/olimp/obm-l.html >O administrador desta lista é >========================================================================= ========================================================================= Instruções para entrar na lista, sair da lista e usar a lista em http://www.mat.puc-rio.br/~nicolau/olimp/obm-l.html O administrador desta lista é ========================================================================= From owner-obm-l@sucuri.mat.puc-rio.br Mon Apr 21 23:18:32 2003 Return-Path: Received: (from majordom@localhost) by sucuri.mat.puc-rio.br (8.9.3/8.9.3) id XAA12486 for obm-l-MTTP; Mon, 21 Apr 2003 23:16:53 -0300 Received: from smtp-29.ig.com.br (smtp-29.ig.com.br [200.226.132.157]) by sucuri.mat.puc-rio.br (8.9.3/8.9.3) with SMTP id XAA12482 for ; Mon, 21 Apr 2003 23:16:50 -0300 Received: (qmail 20999 invoked from network); 22 Apr 2003 02:16:32 -0000 Received: from unknown (HELO xxxx) (200.165.255.193) by smtp-29.ig.com.br with SMTP; 22 Apr 2003 02:16:32 -0000 Message-ID: <001b01c30875$be5f0540$c1ffa5c8@epq.ime.eb.br> From: "Marcio" To: References: <200304220036.h3M0aZK07271@Gauss.impa.br> Subject: Re: [obm-l] FW: Integral da funcao de Planck Date: Mon, 21 Apr 2003 23:20:28 -0300 MIME-Version: 1.0 Content-Type: text/plain; charset="iso-8859-1" Content-Transfer-Encoding: 8bit X-Priority: 3 X-MSMail-Priority: Normal X-Mailer: Microsoft Outlook Express 5.50.4133.2400 X-MimeOLE: Produced By Microsoft MimeOLE V5.50.4133.2400 Sender: owner-obm-l@sucuri.mat.puc-rio.br Precedence: bulk Reply-To: obm-l@mat.puc-rio.br Quando exatamente eu posso trocar a ordem da integral com o somatorio? Por exemplo, considere a serie cujo n-o termo eh s_n = 1/(4n+1) + 1/(4n+3) - 1/(2n+2), cujos termos são todos positivos. Ela converge, por comparação com a série a/n^2. Para calcular Somatorio (0 a infinito) s_n, eu pensei em calcular: Somatorio(0 a infinito)_Integral (0 a 1) [x^4n + x^(4n+2) - x^(2n+1)] Trocando a ordem, ficamos com algumas PG's e: Integral (0 a 1) [1/(1-x^4) + x^2 / (1-x^4) - x/(1-x^2)] = Integral (0 a 1) [1/(1+x)] = ln2 Mas eu vi que essa soma vale, na verdade, 1.5 ln2 (inclusive me provaram isso, e parece estar certo).. Por outro lado, em outros problemas esse método funciona bem.. Por exemplo, para calcular Somatorio ( 1/[(3n+1)*(3n+2)*(3n+3)] ) a resposta parece dar correta.. Abracos, Marcio ----- Original Message ----- From: "Carlos Gustavo Tamm de Araujo Moreira" To: Sent: Monday, April 21, 2003 9:36 PM Subject: Re: [obm-l] FW: Integral da funcao de Planck > Bem, a integral em questao e' algo como > Integral(0 a infinito)(a.v^3/(e^(bv)-1) dv), onde a=8.pi.h/c^3 e b=h/kT. > Fazendo bv=x, dv=dx/b, nossa integral fica (supondo, e' claro, que b>0, > senao a integral nao converge) (a/b^4).Integral(0 a infinito)(x^3/(e^x-1))dx, > mas, escrevendo 1/(e^x-1)=soma(n=1 a infinito)(e^(-nx)), temos que nossa > integral e' (a/b^4)soma(n=1 a infinito)(6/n^4) (pois, fazendo y=nx, vemos > que integral(0 a infinito)(x^3.e^(-nx) dx)= > =(1/n^4).integral(0 a infinito)(x^3.e^(-x)dx)=6/n^4, pois, para todo k, > integral(0 a infinito)(x^k.e^(-x)dx)=k! (inducao, funcao gama). > Como soma(n=1 a infinito)(1/n^4)=pi^4/90, nossa integral e' > (a.pi^4)/(15.b^4))=(8.pi^5.(kT)^4)/(15.(ch)^3). > Abracos, > Gugu > > > > >A integral abaixo parece ser um bom exercicio para os membros de > >plantao.=20 > > > >=20 > > > >Regards, > > > >=20 > > > >Leandro > > > >=20 > > > >-----Original Message----- > >From: Frederico Elsner [mailto:frederico_elsner@msn.com]=20 > >Sent: Tuesday, April 15, 2003 10:49 AM > >To: Leandro Lacorte Rec=F4va > >Subject: Integral da funcao de Planck > > > >=20 > > > >Leandro, > > > >=20 > > > >A integral abaixo =E9 bastante interessante, e =E9 aquela de que eu te = > >falei > > > >=20 > > > > > > > >=20 > > > >O que acha? > > > >=20 > > > >Frederico > > > ========================================================================= > Instruções para entrar na lista, sair da lista e usar a lista em > http://www.mat.puc-rio.br/~nicolau/olimp/obm-l.html > O administrador desta lista é > ========================================================================= > ========================================================================= Instruções para entrar na lista, sair da lista e usar a lista em http://www.mat.puc-rio.br/~nicolau/olimp/obm-l.html O administrador desta lista é ========================================================================= From owner-obm-l@sucuri.mat.puc-rio.br Mon Apr 21 23:57:05 2003 Return-Path: Received: (from majordom@localhost) by sucuri.mat.puc-rio.br (8.9.3/8.9.3) id XAA13369 for obm-l-MTTP; Mon, 21 Apr 2003 23:55:42 -0300 Received: from imo-m06.mx.aol.com (imo-m06.mx.aol.com [64.12.136.161]) by sucuri.mat.puc-rio.br (8.9.3/8.9.3) with ESMTP id XAA13345 for ; Mon, 21 Apr 2003 23:55:35 -0300 From: SiarJoes@aol.com Received: from SiarJoes@aol.com by imo-m06.mx.aol.com (mail_out_v34.22.) id z.ea.381a1607 (657) for ; Mon, 21 Apr 2003 22:54:57 -0400 (EDT) Message-ID: Date: Mon, 21 Apr 2003 22:54:56 EDT Subject: [obm-l] =?ISO-8859-1?Q?An=E1lise=20Combinat=F3ria?= To: obm-l@mat.puc-rio.br MIME-Version: 1.0 Content-Type: multipart/alternative; boundary="part1_ea.381a1607.2bd60900_boundary" X-Mailer: 7.0 for Windows sub 10501 Sender: owner-obm-l@sucuri.mat.puc-rio.br Precedence: bulk Reply-To: obm-l@mat.puc-rio.br --part1_ea.381a1607.2bd60900_boundary Content-Type: text/plain; charset="ISO-8859-1" Content-Transfer-Encoding: quoted-printable Alguem por favor poderia me ajudar nesses problemas de combinat=F3ria? 1) AS atuais placas de licenciamento de autom=F3veis constam de sete s=EDmbo= los,=20 tendo tr=EAs letra, dentre as 26 do alfabeto, seguida de 4 algarismos. a) quantas placas distintas podemos ter sem o algarismo zero na 1=AA posi= =E7=E3o de=20 algarismo? b) No Conjunto de todas as placas distintas poss=EDveis, qual a porcentagem=20 daquelas que t=EAm as duas primeiras letras iguais? 2? A escrita em brasile =E9 um sistema de s=EDmbolos em que cada um dos=20 caracteres =E9 formado por uma matriz de seis pondos, dos quais pelo menos u= m=20 se destaca .Assim: A B * . * . . . * . . . . . Qual o n=FAmero m=E1ximo de caracteres distindos que podem ser representados= =20 nesse sitema? obrigado Junior --part1_ea.381a1607.2bd60900_boundary Content-Type: text/html; charset="ISO-8859-1" Content-Transfer-Encoding: quoted-printable Alguem por favor poderia me ajudar nesses problemas de= combinat=F3ria?

1) AS atuais placas de licenciamento de autom=F3veis constam de sete s=EDmbo= los, tendo tr=EAs letra, dentre as 26 do alfabeto, seguida de 4 algarismos.<= BR> a) quantas placas distintas podemos ter sem o algarismo zero na 1=AA posi= =E7=E3o de algarismo?
b) No Conjunto de todas as placas distintas poss=EDveis, qual a porcentagem=20= daquelas que t=EAm as duas primeiras letras iguais?

2? A escrita em brasile =E9 um sistema de s=EDmbolos em que cada um dos cara= cteres =E9 formado por uma matriz de seis pondos, dos quais pelo menos um se= destaca .Assim:

        A     = ;       B
   
*<= FONT COLOR=3D"#000000" style=3D"BACKGROUND-COLOR: #ffffff" SIZE=3D2 FAMILY= =3D"SANSSERIF" FACE=3D"Arial" LANG=3D"8">     . &nb= sp;       *    .
    .     .     =     *    .
    .     .     =     .    .

Qual o n=FAmero m=E1ximo de caracteres distindos que podem ser representados= nesse sitema?

obrigado
Junior
--part1_ea.381a1607.2bd60900_boundary-- ========================================================================= Instruções para entrar na lista, sair da lista e usar a lista em http://www.mat.puc-rio.br/~nicolau/olimp/obm-l.html O administrador desta lista é ========================================================================= From owner-obm-l@sucuri.mat.puc-rio.br Tue Apr 22 00:08:46 2003 Return-Path: Received: (from majordom@localhost) by sucuri.mat.puc-rio.br (8.9.3/8.9.3) id AAA13700 for obm-l-MTTP; Tue, 22 Apr 2003 00:07:23 -0300 Received: from imo-r07.mx.aol.com (imo-r07.mx.aol.com [152.163.225.103]) by sucuri.mat.puc-rio.br (8.9.3/8.9.3) with ESMTP id AAA13693 for ; Tue, 22 Apr 2003 00:07:20 -0300 From: DEOLIVEIRASOU@aol.com Received: from DEOLIVEIRASOU@aol.com by imo-r07.mx.aol.com (mail_out_v34.22.) id z.1ef.72af400 (4410) for ; Mon, 21 Apr 2003 23:06:40 -0400 (EDT) Message-ID: <1ef.72af400.2bd60bbf@aol.com> Date: Mon, 21 Apr 2003 23:06:39 EDT Subject: =?ISO-8859-1?Q?Re:=20[obm-l]=20Re:=20[obm-l]=20D=FAvida=20Sobre=20?= =?ISO-8859-1?Q?=20a=20Utilidade=20da=20Ma=20tem=E1tica=20Ensinada=20nas.?= =?ISO-8859-1?Q?..?= To: obm-l@mat.puc-rio.br MIME-Version: 1.0 Content-Type: multipart/alternative; boundary="part1_1ef.72af400.2bd60bbf_boundary" X-Mailer: 7.0 for Windows sub 10501 Sender: owner-obm-l@sucuri.mat.puc-rio.br Precedence: bulk Reply-To: obm-l@mat.puc-rio.br --part1_1ef.72af400.2bd60bbf_boundary Content-Type: text/plain; charset="ISO-8859-1" Content-Transfer-Encoding: quoted-printable Rapaz, que belo texto. Fiquei impressionado! Algumas pessoas acham que=20 professores de matem=E1tica n=E3o tem sensibilidade e discernimento para esc= rever=20 ou interpretar aquilo que n=E3o esteja circunscrito =E0 matem=E1tica....cheg= am a=20 dizer mesmo que quem trabalha na =E1rea de exatas =E9 um tanto quanto " alie= nado=20 ". Muitas pessoas que contribuiram para a melhoria do ensino, pode ter=20 certeza, s=E3o matem=E1ticos, com forma=E7=E3o inclusive em matem=E1tica pur= a. Ubiratan=20 D=E1mbr=F3zio por exemplo. Seu texto =E9 digno de impress=E3o, pois, seria e= lucidador=20 para as pessoas que equivocadamente acreditam que a matem=E1tica =E9 uma ci= =EAncia=20 cega que n=E3o serve para nada.Vou imprimi-lo, pois, qual professor nunca ou= viu=20 de um aluno a pergunta: " Por que se multiplica matriz assim , psor??(=20 Composi=E7=E3o de transforma=E7=F5es lineares n=E3o =E9 um assunto que voc= =EA possa=20 explicar para um aluno que n=E3o seja um talento olimpico). Um abra=E7o, Ruy --part1_1ef.72af400.2bd60bbf_boundary Content-Type: text/html; charset="ISO-8859-1" Content-Transfer-Encoding: quoted-printable Rapaz, que belo texto. Fiquei impressionado! Algumas p= essoas acham que professores de matem=E1tica n=E3o tem sensibilidade e disce= rnimento para escrever ou interpretar aquilo que n=E3o esteja circunscrito=20= =E0 matem=E1tica....chegam a dizer mesmo que quem trabalha na =E1rea de exat= as =E9 um tanto quanto " alienado ". Muitas pessoas que contribuiram para a=20= melhoria do ensino, pode ter certeza, s=E3o matem=E1ticos, com forma=E7=E3o=20= inclusive em matem=E1tica pura. Ubiratan D=E1mbr=F3zio por exemplo. Seu text= o =E9 digno de impress=E3o, pois, seria elucidador para as pessoas que equiv= ocadamente acreditam que a matem=E1tica =E9 uma ci=EAncia cega que n=E3o ser= ve para nada.Vou imprimi-lo, pois, qual professor nunca ouviu de um aluno a=20= pergunta: " Por que se multiplica matriz assim , psor??( Composi=E7=E3o de t= ransforma=E7=F5es lineares n=E3o =E9 um assunto que voc=EA possa explicar pa= ra um aluno que n=E3o seja um talento olimpico).
     Um abra=E7o,
            &nbs= p;       Ruy
--part1_1ef.72af400.2bd60bbf_boundary-- ========================================================================= Instruções para entrar na lista, sair da lista e usar a lista em http://www.mat.puc-rio.br/~nicolau/olimp/obm-l.html O administrador desta lista é ========================================================================= From owner-obm-l@sucuri.mat.puc-rio.br Tue Apr 22 00:47:18 2003 Return-Path: Received: (from majordom@localhost) by sucuri.mat.puc-rio.br (8.9.3/8.9.3) id AAA14930 for obm-l-MTTP; Tue, 22 Apr 2003 00:45:51 -0300 Received: from Euler.impa.br (euler.impa.br [147.65.1.3]) by sucuri.mat.puc-rio.br (8.9.3/8.9.3) with ESMTP id AAA14925 for ; Tue, 22 Apr 2003 00:45:47 -0300 Received: from Gauss.impa.br (Gauss [147.65.4.1]) by Euler.impa.br (8.11.6p2/8.11.6) with ESMTP id h3M3jH022763 for ; Tue, 22 Apr 2003 00:45:17 -0300 (EST) From: Carlos Gustavo Tamm de Araujo Moreira Received: by Gauss.impa.br (8.11.6p2) id h3M3j5W28949; Tue, 22 Apr 2003 00:45:05 -0300 (EST) Message-Id: <200304220345.h3M3j5W28949@Gauss.impa.br> Subject: Re: [obm-l] FW: Integral da funcao de Planck To: obm-l@mat.puc-rio.br Date: Tue, 22 Apr 2003 00:45:04 -0300 (EST) In-Reply-To: <001b01c30875$be5f0540$c1ffa5c8@epq.ime.eb.br> from "Marcio" at Apr 21, 3 11:20:28 pm X-Mailer: ELM [version 2.4 PL25] MIME-Version: 1.0 Content-Type: text/plain; charset=US-ASCII Content-Transfer-Encoding: 7bit Sender: owner-obm-l@sucuri.mat.puc-rio.br Precedence: bulk Reply-To: obm-l@mat.puc-rio.br Oi Marcio, Para poder trocar a ordem em integral(soma(f_n))=soma(integral(f_n)) e' necessario e suficiente que integral(soma(n >= k)(f_n)) tenda a 0 quando k tende a infinito (e sempre podemos quando as f_n sao todas positivas, que e' o caso do problema que eu resolvi). No seu caso isso nao acontece, pois essa integral e' integral(0 a 1)(x^4k/(1-x^4)+x^(4k+2)/(1-x^4)-x^(2k+1)/(1-x^2))= integral(0 a 1)((x^(4k)-x^(2k+1))/(1-x^2)), que NAO tende a 0 (mas sim a -ln(2)/2) quando k tende a infinito. Abracos, Gugu > > Quando exatamente eu posso trocar a ordem da integral com o somatorio? >Por exemplo, considere a serie cujo n-o termo eh s_n = 1/(4n+1) + 1/(4n+3) - >1/(2n+2), cujos termos são todos positivos. >Ela converge, por comparação com a série a/n^2. > >Para calcular Somatorio (0 a infinito) s_n, eu pensei em calcular: > Somatorio(0 a infinito)_Integral (0 a 1) [x^4n + x^(4n+2) - x^(2n+1)] > Trocando a ordem, ficamos com algumas PG's e: > Integral (0 a 1) [1/(1-x^4) + x^2 / (1-x^4) - x/(1-x^2)] = > Integral (0 a 1) [1/(1+x)] = ln2 > >Mas eu vi que essa soma vale, na verdade, 1.5 ln2 (inclusive me provaram >isso, e parece estar certo).. > >Por outro lado, em outros problemas esse método funciona bem.. Por exemplo, >para calcular Somatorio ( 1/[(3n+1)*(3n+2)*(3n+3)] ) a resposta parece dar >correta.. > > Abracos, > Marcio > > > >----- Original Message ----- >From: "Carlos Gustavo Tamm de Araujo Moreira" >To: >Sent: Monday, April 21, 2003 9:36 PM >Subject: Re: [obm-l] FW: Integral da funcao de Planck > > >> Bem, a integral em questao e' algo como >> Integral(0 a infinito)(a.v^3/(e^(bv)-1) dv), onde a=8.pi.h/c^3 e b=h/kT. >> Fazendo bv=x, dv=dx/b, nossa integral fica (supondo, e' claro, que b>0, >> senao a integral nao converge) (a/b^4).Integral(0 a >infinito)(x^3/(e^x-1))dx, >> mas, escrevendo 1/(e^x-1)=soma(n=1 a infinito)(e^(-nx)), temos que nossa >> integral e' (a/b^4)soma(n=1 a infinito)(6/n^4) (pois, fazendo y=nx, vemos >> que integral(0 a infinito)(x^3.e^(-nx) dx)= >> =(1/n^4).integral(0 a infinito)(x^3.e^(-x)dx)=6/n^4, pois, para todo k, >> integral(0 a infinito)(x^k.e^(-x)dx)=k! (inducao, funcao gama). >> Como soma(n=1 a infinito)(1/n^4)=pi^4/90, nossa integral e' >> (a.pi^4)/(15.b^4))=(8.pi^5.(kT)^4)/(15.(ch)^3). >> Abracos, >> Gugu >> >> > >> >A integral abaixo parece ser um bom exercicio para os membros de >> >plantao.=20 >> > >> >=20 >> > >> >Regards, >> > >> >=20 >> > >> >Leandro >> > >> >=20 >> > >> >-----Original Message----- >> >From: Frederico Elsner [mailto:frederico_elsner@msn.com]=20 >> >Sent: Tuesday, April 15, 2003 10:49 AM >> >To: Leandro Lacorte Rec=F4va >> >Subject: Integral da funcao de Planck >> > >> >=20 >> > >> >Leandro, >> > >> >=20 >> > >> >A integral abaixo =E9 bastante interessante, e =E9 aquela de que eu te = >> >falei >> > >> >=20 >> > >> > >> > >> >=20 >> > >> >O que acha? >> > >> >=20 >> > >> >Frederico >> > >> ========================================================================= >> Instruções para entrar na lista, sair da lista e usar a lista em >> http://www.mat.puc-rio.br/~nicolau/olimp/obm-l.html >> O administrador desta lista é >> ========================================================================= >> > >========================================================================= >Instruções para entrar na lista, sair da lista e usar a lista em >http://www.mat.puc-rio.br/~nicolau/olimp/obm-l.html >O administrador desta lista é >========================================================================= ========================================================================= Instruções para entrar na lista, sair da lista e usar a lista em http://www.mat.puc-rio.br/~nicolau/olimp/obm-l.html O administrador desta lista é ========================================================================= From owner-obm-l@sucuri.mat.puc-rio.br Tue Apr 22 05:22:18 2003 Return-Path: Received: (from majordom@localhost) by sucuri.mat.puc-rio.br (8.9.3/8.9.3) id FAA17990 for obm-l-MTTP; Tue, 22 Apr 2003 05:20:30 -0300 Received: from trex.centroin.com.br (trex.centroin.com.br [200.225.63.134]) by sucuri.mat.puc-rio.br (8.9.3/8.9.3) with ESMTP id FAA17986 for ; Tue, 22 Apr 2003 05:20:26 -0300 Received: from trex.centroin.com.br (localhost [127.0.0.1]) by trex.centroin.com.br (8.12.9/8.12.9) with ESMTP id h3M8JurL004184 for ; Tue, 22 Apr 2003 05:19:56 -0300 (EST) Received: by trex.centroin.com.br (8.12.9/8.12.5/Submit) id h3M8JuoC004183; Tue, 22 Apr 2003 05:19:56 -0300 (EST) Message-Id: <200304220819.h3M8JuoC004183@trex.centroin.com.br> Received: from 200.165.208.59 by trex.centroin.com.br (CIPWM versao 1.4C1) with HTTPS for ; Tue, 22 Apr 2003 05:19:56 -0300 (EST) Date: Tue, 22 Apr 2003 05:19:56 -0300 (EST) From: Augusto Cesar de Oliveira Morgado To: obm-l@mat.puc-rio.br Subject: Re: [obm-l] re:2 coisinhas MIME-Version: 1.0 X-Mailer: CentroIn Internet Provider WebMail v. 1.4C1 (http://www.centroin.com.br/) Content-Type: text/plain; charset="iso-8859-1" Content-Transfer-Encoding: 8bit X-MIME-Autoconverted: from quoted-printable to 8bit by sucuri.mat.puc-rio.br id FAA17987 Sender: owner-obm-l@sucuri.mat.puc-rio.br Precedence: bulk Reply-To: obm-l@mat.puc-rio.br Interessante. Mandei uma soluçao para o problema 1, diferente desta, mas me passou pela cabeça a interpretaçao do Felipe. Na minha soluçao, retangulo 2x1 era retangulo de 2 linhas e 1 coluna OU VICE-VERSA. Qual eh a sua interpretaçao, Helder? Em Mon, 21 Apr 2003 21:39:12 -0300, felipe mendona disse: ========================================================================= Instruções para entrar na lista, sair da lista e usar a lista em http://www.mat.puc-rio.br/~nicolau/olimp/obm-l.html O administrador desta lista é ========================================================================= From owner-obm-l@sucuri.mat.puc-rio.br Tue Apr 22 05:32:15 2003 Return-Path: Received: (from majordom@localhost) by sucuri.mat.puc-rio.br (8.9.3/8.9.3) id FAA18116 for obm-l-MTTP; Tue, 22 Apr 2003 05:30:54 -0300 Received: from trex.centroin.com.br (trex.centroin.com.br [200.225.63.134]) by sucuri.mat.puc-rio.br (8.9.3/8.9.3) with ESMTP id FAA18112 for ; Tue, 22 Apr 2003 05:30:51 -0300 Received: from trex.centroin.com.br (localhost [127.0.0.1]) by trex.centroin.com.br (8.12.9/8.12.9) with ESMTP id h3M8ULrL005727 for ; Tue, 22 Apr 2003 05:30:21 -0300 (EST) Received: by trex.centroin.com.br (8.12.9/8.12.5/Submit) id h3M8ULfa005726; Tue, 22 Apr 2003 05:30:21 -0300 (EST) Message-Id: <200304220830.h3M8ULfa005726@trex.centroin.com.br> Received: from 200.165.208.59 by trex.centroin.com.br (CIPWM versao 1.4C1) with HTTPS for ; Tue, 22 Apr 2003 05:30:21 -0300 (EST) Date: Tue, 22 Apr 2003 05:30:21 -0300 (EST) From: Augusto Cesar de Oliveira Morgado To: obm-l@mat.puc-rio.br Subject: =?iso-8859-1?q?Re: [obm-l] An=E1lise Combinat=F3ria?= MIME-Version: 1.0 X-Mailer: CentroIn Internet Provider WebMail v. 1.4C1 (http://www.centroin.com.br/) Content-Type: text/plain; charset="iso-8859-1" Content-Transfer-Encoding: 8bit X-MIME-Autoconverted: from quoted-printable to 8bit by sucuri.mat.puc-rio.br id FAA18113 Sender: owner-obm-l@sucuri.mat.puc-rio.br Precedence: bulk Reply-To: obm-l@mat.puc-rio.br 1a)26x26x26x9x10x10x10 1b)1/26 2) Essa caiu em vestibular em Sao Paulo e deu um rolo... Voce tem que escolher o subconjunto dos simbolos a destacar, o que voce pode fazer de 2^6 = 64 modos. Nao vale o vazio (pelo menos um deve se destacar) e a resposta (oficial) eh 63. Muitos professores reclamaram do gabarito oficial alegando que se todos se destacam, ninguem se destaca. Para eles a resposta era 62. Em Mon, 21 Apr 2003 22:54:56 EDT, SiarJoes@aol.com disse: > Alguem por favor poderia me ajudar nesses problemas de combinatória? > > 1) AS atuais placas de licenciamento de automóveis constam de sete símbolos, > tendo três letra, dentre as 26 do alfabeto, seguida de 4 algarismos. > a) quantas placas distintas podemos ter sem o algarismo zero na 1ª posição de > algarismo? > b) No Conjunto de todas as placas distintas possíveis, qual a porcentagem > daquelas que têm as duas primeiras letras iguais? > > 2? A escrita em brasile é um sistema de símbolos em que cada um dos > caracteres é formado por uma matriz de seis pondos, dos quais pelo menos um > se destaca .Assim: > > A B > * . * . > . . * . > . . . . > > Qual o número máximo de caracteres distindos que podem ser representados > nesse sitema? > > obrigado > Junior ========================================================================= Instruções para entrar na lista, sair da lista e usar a lista em http://www.mat.puc-rio.br/~nicolau/olimp/obm-l.html O administrador desta lista é ========================================================================= From owner-obm-l@sucuri.mat.puc-rio.br Tue Apr 22 09:42:56 2003 Return-Path: Received: (from majordom@localhost) by sucuri.mat.puc-rio.br (8.9.3/8.9.3) id JAA21645 for obm-l-MTTP; Tue, 22 Apr 2003 09:40:43 -0300 Received: from imo-r05.mx.aol.com (imo-r05.mx.aol.com [152.163.225.101]) by sucuri.mat.puc-rio.br (8.9.3/8.9.3) with ESMTP id JAA21641 for ; Tue, 22 Apr 2003 09:40:40 -0300 From: SiarJoes@aol.com Received: from SiarJoes@aol.com by imo-r05.mx.aol.com (mail_out_v34.22.) id z.6.f4e108a (4116) for ; Tue, 22 Apr 2003 08:40:03 -0400 (EDT) Message-ID: <6.f4e108a.2bd69223@aol.com> Date: Tue, 22 Apr 2003 08:40:03 EDT Subject: Re: =?iso-8859-1?q?Re: [obm-l] An=E1lise Combinat=F3ria?= To: obm-l@mat.puc-rio.br MIME-Version: 1.0 Content-Type: multipart/alternative; boundary="part1_6.f4e108a.2bd69223_boundary" X-Mailer: 7.0 for Windows sub 10501 Sender: owner-obm-l@sucuri.mat.puc-rio.br Precedence: bulk Reply-To: obm-l@mat.puc-rio.br --part1_6.f4e108a.2bd69223_boundary Content-Type: text/plain; charset="ISO-8859-1" Content-Transfer-Encoding: quoted-printable Muito Obrigado Morgado!!!!! Sidmar Junior ps: s=F3 uma d=FAvida, nessa n=FAmero dois eu consegui fazendo combina=E7= =E3o 1 em 6,=20 depois de 2 em 6, depois de 3 em 6, e assim at=E9 combina=E7=E3o de 6 em 6,=20= depois=20 eu somei, e deu a mesma coisa, =E9 poss=EDvel fazer isso? e como voc=EA fez=20= para=20 colocar 2^6 que eu n=E3o intendi direito? muito agradecido Junior --part1_6.f4e108a.2bd69223_boundary Content-Type: text/html; charset="ISO-8859-1" Content-Transfer-Encoding: quoted-printable Muito Obrigado Morgado!!!!!
Sidmar Junior
ps: s=F3 uma d=FAvida, nessa n=FAmero dois eu consegui fazendo combina=E7= =E3o 1 em 6, depois de 2 em 6, depois de 3 em 6, e assim at=E9 combina=E7= =E3o de 6 em 6, depois eu somei, e deu a mesma coisa, =E9 poss=EDvel fazer i= sso? e como voc=EA fez para colocar 2^6 que eu n=E3o intendi direito?

muito agradecido
Junior
--part1_6.f4e108a.2bd69223_boundary-- ========================================================================= Instruções para entrar na lista, sair da lista e usar a lista em http://www.mat.puc-rio.br/~nicolau/olimp/obm-l.html O administrador desta lista é ========================================================================= From owner-obm-l@sucuri.mat.puc-rio.br Tue Apr 22 09:49:59 2003 Return-Path: Received: (from majordom@localhost) by sucuri.mat.puc-rio.br (8.9.3/8.9.3) id JAA21806 for obm-l-MTTP; Tue, 22 Apr 2003 09:48:36 -0300 Received: from hotmail.com (bay3-f18.bay3.hotmail.com [65.54.169.18]) by sucuri.mat.puc-rio.br (8.9.3/8.9.3) with ESMTP id JAA21801 for ; Tue, 22 Apr 2003 09:48:32 -0300 Received: from mail pickup service by hotmail.com with Microsoft SMTPSVC; Tue, 22 Apr 2003 05:48:01 -0700 Received: from 200.147.91.227 by by3fd.bay3.hotmail.msn.com with HTTP; Tue, 22 Apr 2003 12:48:00 GMT X-Originating-IP: [200.147.91.227] X-Originating-Email: [rhilbert1990@msn.com] From: "Oblomov Insistenko" To: obm-l@mat.puc-rio.br Subject: [obm-l] =?iso-8859-1?B?UGFyZWNlIGbhY2lsLi4ubWFzLCBu428gY29uc2lnby4=?= Date: Tue, 22 Apr 2003 09:48:00 -0300 Mime-Version: 1.0 Content-Type: text/plain; charset=iso-8859-1; format=flowed Message-ID: X-OriginalArrivalTime: 22 Apr 2003 12:48:01.0014 (UTC) FILETIME=[68723960:01C308CD] Sender: owner-obm-l@sucuri.mat.puc-rio.br Precedence: bulk Reply-To: obm-l@mat.puc-rio.br Agradeço a quem puder me ajudar a resolver esse problema. Demonstrar que, se m e n são inteiros ímpares, então 8|(m^4 + n^4 - 2). Obrigado! _________________________________________________________________ MSN Hotmail, o maior webmail do Brasil. http://www.hotmail.com ========================================================================= Instruções para entrar na lista, sair da lista e usar a lista em http://www.mat.puc-rio.br/~nicolau/olimp/obm-l.html O administrador desta lista é ========================================================================= From owner-obm-l@sucuri.mat.puc-rio.br Tue Apr 22 10:16:42 2003 Return-Path: Received: (from majordom@localhost) by sucuri.mat.puc-rio.br (8.9.3/8.9.3) id KAA22629 for obm-l-MTTP; Tue, 22 Apr 2003 10:14:21 -0300 Received: from ns3bind.localdomain ([200.230.34.5]) by sucuri.mat.puc-rio.br (8.9.3/8.9.3) with ESMTP id KAA22625 for ; Tue, 22 Apr 2003 10:14:18 -0300 Received: from servico2 ([200.230.34.228]) by ns3bind.localdomain (8.11.6/X.XX.X) with SMTP id h3MD9fR04699 for ; Tue, 22 Apr 2003 10:09:42 -0300 Message-ID: <001c01c308d1$2cac9200$3300c57d@bovespa.com> From: "=?iso-8859-1?Q?Cl=E1udio_\=28Pr=E1tica\=29?=" To: References: <10f.20f85442.2bd0af92@aol.com> Subject: Re: [obm-l] Sobre as olimpiadas ao redor do mundo(e um certo DEOLIVEIRASOU...) Date: Tue, 22 Apr 2003 10:14:56 -0300 MIME-Version: 1.0 Content-Type: multipart/alternative; boundary="----=_NextPart_000_0019_01C308B8.05CE0BA0" X-Priority: 3 X-MSMail-Priority: Normal X-Mailer: Microsoft Outlook Express 5.50.4920.2300 X-MimeOLE: Produced By Microsoft MimeOLE V5.50.4920.2300 Sender: owner-obm-l@sucuri.mat.puc-rio.br Precedence: bulk Reply-To: obm-l@mat.puc-rio.br This is a multi-part message in MIME format. ------=_NextPart_000_0019_01C308B8.05CE0BA0 Content-Type: text/plain; charset="iso-8859-1" Content-Transfer-Encoding: quoted-printable Oi, Crom: Essa equa=E7=E3o nada mais =E9 do que o caso n =3D 3 do =DAltimo Teorema = de Fermat. Eu estou convencido de que o resultado pode ser provado sem se usar = inteiros de Eisenstein, mas apenas por meio de considera=E7=F5es de = divisibilidade (inclusive congru=EAncias) e indu=E7=E3o matem=E1tica = (possivelmente na forma de descida infinita). H=E1 algum tempo, o Jos=E9 Francisco Guimar=E3es Costa disse que era = mais f=E1cil entender a Cr=EDtica da Raz=E3o Pura de Kant em aramaico do = que a demonstra=E7=E3o do caso n =3D 3 do UTF. Eu estou procurando uma demonstra=E7=E3o elementar desse teorema cujo = objetivo =E9 convencer o JF do contr=E1rio (a menos que ele seja um = expert em Kant e aramaico!). Assim que eu a encontrar, mando pra lista. Um abra=E7o, Claudio. ----- Original Message -----=20 From: DEOLIVEIRASOU@aol.com=20 To: obm-l@mat.puc-rio.br=20 Sent: Thursday, April 17, 2003 10:32 PM Subject: Re: [obm-l] Sobre as olimpiadas ao redor do mundo(e um certo = DEOLIVEIRASOU...) Ola Claudio, Voce tem tanta razao sobre a transpiracao em detrimento da inspiracao, = que eu, ja consegui fazer sozinho uns seis ou sete problemas das = olimpiadas ao redor do mundo...somando-se a essa conquista, os problemas = que vc tem resolvido, acho que meu desenvolvimento em rudimentos de = teoria dos numeros ja esta aparecendo....Valeu muito. Para que essa mensagem nao fique off-topic, vou mandar aos = participantes da lista o se guinte problema.... Mostre que a equacao diofantina x^3+y^3+z^3=3D0 so tem solucoes = triviais, ou seja, xyz=3D0. obs...esse exercicio vem depois da exposicao do artigo sobre inteiros = de Gauss e inteiros de Einsenstein da eureka 14, que confesso ainda nao = ter lido....pergunta----ele so pode ser resolvido utilizando-se as = teorias subjacentes ao artigo. Um Abraco, Crom ------=_NextPart_000_0019_01C308B8.05CE0BA0 Content-Type: text/html; charset="iso-8859-1" Content-Transfer-Encoding: quoted-printable
Oi, Crom:
 
Essa equa=E7=E3o nada mais =E9 do que o = caso n =3D 3 do=20 =DAltimo Teorema de Fermat.
 
Eu estou convencido de que o resultado = pode ser=20 provado sem se usar inteiros de Eisenstein, mas apenas por meio de = considera=E7=F5es=20 de divisibilidade (inclusive congru=EAncias) e indu=E7=E3o matem=E1tica = (possivelmente=20 na forma de descida infinita).
 
H=E1 algum tempo, o Jos=E9 Francisco = Guimar=E3es Costa=20 disse que era mais f=E1cil entender a Cr=EDtica da Raz=E3o Pura de Kant = em aramaico do=20 que a demonstra=E7=E3o do caso n =3D 3 do UTF.
 
Eu estou procurando uma = demonstra=E7=E3o elementar=20 desse teorema cujo objetivo =E9 convencer o JF do contr=E1rio (a menos = que ele seja=20 um expert em Kant e aramaico!). Assim que eu a encontrar, mando pra=20 lista.
 
Um abra=E7o,
Claudio.
----- Original Message -----
From:=20 DEOLIVEIRASOU@aol.com
Sent: Thursday, April 17, 2003 = 10:32=20 PM
Subject: Re: [obm-l] Sobre as = olimpiadas=20 ao redor do mundo(e um certo DEOLIVEIRASOU...)

Ola Claudio,
Voce tem tanta razao sobre a = transpiracao=20 em detrimento da inspiracao, que eu, ja consegui fazer sozinho uns = seis ou=20 sete problemas das olimpiadas ao redor do mundo...somando-se a essa = conquista,=20 os problemas que vc tem  resolvido, acho que meu desenvolvimento = em=20 rudimentos de teoria dos numeros ja esta aparecendo....Valeu = muito.
Para=20 que essa mensagem nao fique off-topic, vou mandar aos participantes da = lista o=20 se guinte problema....
Mostre que a equacao diofantina = x^3+y^3+z^3=3D0 so tem=20 solucoes triviais, ou seja, xyz=3D0.
obs...esse exercicio vem = depois da=20 exposicao do artigo sobre inteiros de Gauss e inteiros de Einsenstein = da=20 eureka 14, que confesso ainda nao ter lido....pergunta----ele so pode = ser=20 resolvido utilizando-se as teorias subjacentes ao=20 artigo.
         Um=20 = Abraco,
          &n= bsp; =20 Crom
------=_NextPart_000_0019_01C308B8.05CE0BA0-- ========================================================================= Instruções para entrar na lista, sair da lista e usar a lista em http://www.mat.puc-rio.br/~nicolau/olimp/obm-l.html O administrador desta lista é ========================================================================= From owner-obm-l@sucuri.mat.puc-rio.br Tue Apr 22 10:38:31 2003 Return-Path: Received: (from majordom@localhost) by sucuri.mat.puc-rio.br (8.9.3/8.9.3) id KAA23460 for obm-l-MTTP; Tue, 22 Apr 2003 10:37:07 -0300 Received: from cmsrelay05.mx.net (cmsrelay05.mx.net [165.212.11.2]) by sucuri.mat.puc-rio.br (8.9.3/8.9.3) with SMTP id KAA23456 for ; Tue, 22 Apr 2003 10:37:04 -0300 Received: from uadvg137.cms.usa.net (HELO localhost) (165.212.11.137) by cmsoutbound.mx.net with SMTP; 22 Apr 2003 13:36:31 -0000 Received: from smtp.postoffice.net [165.212.8.26] by uadvg137.cms.usa.net (ASMTP/) via mtad (C8.MAIN.2.05) with ESMTP id 038HDVNKd0234M37; Tue, 22 Apr 2003 13:36:29 GMT Received: from 200.181.4.100 [200.181.4.100] by cmsweb10.cms.usa.net (USANET web-mailer CM.0402.5.2B); Tue, 22 Apr 2003 13:36:27 -0000 Date: Tue, 22 Apr 2003 10:36:27 -0300 From: Artur Costa Steiner To: Subject: [obm-l] =?ISO-8859-1?Q?Re=3A=20=5B=5Bobm=2Dl=5D=20Parece=20f=E1ci?= =?ISO-8859-1?Q?l=2E=2E=2Emas=2C=20n=E3o=20consigo=2E=5D?= X-Mailer: USANET web-mailer (CM.0402.5.2B) Mime-Version: 1.0 Message-ID: <902HDVNKb6016S10.1051018587@cmsweb10.cms.usa.net> Content-Type: text/plain; charset=ISO-8859-1 Content-Transfer-Encoding: 8bit X-MIME-Autoconverted: from quoted-printable to 8bit by sucuri.mat.puc-rio.br id KAA23457 Sender: owner-obm-l@sucuri.mat.puc-rio.br Precedence: bulk Reply-To: obm-l@mat.puc-rio.br Temos que (m^4 + n^4 - 2) = m^4 -1 + n^4 -1 = (m^2+1) (m^2 -1) +(n^2+1) (n^2 -1). Como m eh impar, m= 2k-1 para um inteiro k. Logo, m^2= 4k^2 - 4k +1. Substituindo, segue-se que m^4 -1 = (m^2+1) (m^2 -1) = (4k^2 - 4k +2) (4k^2 -4k) = 8(2k^2 - 2k +1)(k^2 -k). Como k eh inteiro, o produto dos dois parenteses eh inteiro, do que deduzimos que m^4 -1 eh multiplo de 8. Dado que este mesmo raciocinio vale obviamente para n^4 -1, concluimos que a soma m^4 -1 + n^4 -1 = (m^4 + n^4 - 2) eh multipla de 8. Abracos Artur > Agradeço a quem puder me ajudar a resolver esse problema. > > Demonstrar que, se m e n são inteiros ímpares, então > 8|(m^4 + n^4 - 2). > > Obrigado! > > > _________________________________________________________________ > MSN Hotmail, o maior webmail do Brasil. http://www.hotmail.com > > ========================================================================= > Instruções para entrar na lista, sair da lista e usar a lista em > http://www.mat.puc-rio.br/~nicolau/olimp/obm-l.html > O administrador desta lista é > ========================================================================= ========================================================================= Instruções para entrar na lista, sair da lista e usar a lista em http://www.mat.puc-rio.br/~nicolau/olimp/obm-l.html O administrador desta lista é ========================================================================= From owner-obm-l@sucuri.mat.puc-rio.br Tue Apr 22 10:46:56 2003 Return-Path: Received: (from majordom@localhost) by sucuri.mat.puc-rio.br (8.9.3/8.9.3) id KAA23682 for obm-l-MTTP; Tue, 22 Apr 2003 10:44:54 -0300 Received: from soling.fortalnet.com.br ([200.164.93.238]) by sucuri.mat.puc-rio.br (8.9.3/8.9.3) with ESMTP id KAA23678 for ; Tue, 22 Apr 2003 10:44:50 -0300 Received: from CemagCerver (ip-ip61.fortalnet.com.br [200.253.221.190]) by soling.fortalnet.com.br (8.11.6/8.11.6) with SMTP id h3MDYqa00769 for ; Tue, 22 Apr 2003 10:34:52 -0300 Message-ID: <001f01c308d4$e6c401c0$0203a8c0@CemagCerver> From: "Davidson Estanislau" To: Subject: [obm-l] =?iso-8859-1?Q?Fw:_=5Bobm-l=5D_Parece_f=E1cil...mas=2C_n=E3o_consigo.?= Date: Tue, 22 Apr 2003 10:41:34 -0300 MIME-Version: 1.0 Content-Type: multipart/alternative; boundary="----=_NextPart_000_001C_01C308BB.BE5A4780" X-Priority: 3 X-MSMail-Priority: Normal X-Mailer: Microsoft Outlook Express 6.00.2800.1106 X-MimeOLE: Produced By Microsoft MimeOLE V6.00.2800.1106 Sender: owner-obm-l@sucuri.mat.puc-rio.br Precedence: bulk Reply-To: obm-l@mat.puc-rio.br This is a multi-part message in MIME format. ------=_NextPart_000_001C_01C308BB.BE5A4780 Content-Type: text/plain; charset="iso-8859-1" Content-Transfer-Encoding: quoted-printable Como m e n s=E3o inteiros =EDmpares, podemos escrever: m =3D 2a + 1; n =3D 2b + 1. Substituindo, temos: (2a+1)^4 + (2b+1)^4 - 2 =3D 8(2a^4 + 4a^3 + 3a^2 + a + 2b^4 + 4b^3 + = 3b^2 + b), que =E9 divis=EDvel por 8. Felicidades. Davidson Estanislau -----Mensagem Original-----=20 De: "Oblomov Insistenko" Para: Enviada em: Ter=E7a-feira, 22 de Abril de 2003 09:48 Assunto: [obm-l] Parece f=E1cil...mas, n=E3o consigo. >=20 > Agrade=E7o a quem puder me ajudar a resolver esse problema. >=20 > Demonstrar que, se m e n s=E3o inteiros =EDmpares, ent=E3o > 8|(m^4 + n^4 - 2). >=20 > Obrigado! ------=_NextPart_000_001C_01C308BB.BE5A4780 Content-Type: text/html; charset="iso-8859-1" Content-Transfer-Encoding: quoted-printable
 
 Como m e n = s=E3o=20 inteiros =EDmpares, podemos escrever:
 
 m =3D 2a + 1;
 n =3D 2b + 1.
 
  Substituindo, temos:
  (2a+1)^4 + (2b+1)^4 - 2 =3D 8(2a^4 + 4a^3 + = 3a^2 + a +=20 2b^4 + 4b^3 + 3b^2 + b), que =E9 divis=EDvel por 8.
 
  Felicidades.
 
  Davidson Estanislau
 
 
 
 
 
-----Mensagem Original-----
De: "Oblomov Insistenko" <rhilbert1990@msn.com>
Enviada em: Ter=E7a-feira, 22 de Abril de 2003=20 09:48
Assunto: [obm-l] Parece f=E1cil...mas, n=E3o=20 consigo.

>
> = Agrade=E7o a quem=20 puder me ajudar a resolver esse problema.
>
> Demonstrar = que, se=20 m  e  n  s=E3o inteiros =EDmpares, ent=E3o
> 8|(m^4 = + n^4 -=20 2).
>
> Obrigado!
------=_NextPart_000_001C_01C308BB.BE5A4780-- ========================================================================= Instruções para entrar na lista, sair da lista e usar a lista em http://www.mat.puc-rio.br/~nicolau/olimp/obm-l.html O administrador desta lista é ========================================================================= From owner-obm-l@sucuri.mat.puc-rio.br Tue Apr 22 10:46:56 2003 Return-Path: Received: (from majordom@localhost) by sucuri.mat.puc-rio.br (8.9.3/8.9.3) id KAA23671 for obm-l-MTTP; Tue, 22 Apr 2003 10:44:43 -0300 Received: from traven10.uol.com.br (traven10.uol.com.br [200.221.29.45]) by sucuri.mat.puc-rio.br (8.9.3/8.9.3) with ESMTP id KAA23667 for ; Tue, 22 Apr 2003 10:44:39 -0300 Received: from ui.uol.com.br ([200.147.147.166]) by traven10.uol.com.br (8.9.1/8.9.1) with ESMTP id KAA24064; Tue, 22 Apr 2003 10:44:07 -0300 (BRT) Message-Id: <5.1.0.14.2.20030422102944.02efdec8@pop3.uol.com.br> X-Sender: cavictor@pop3.uol.com.br X-Mailer: QUALCOMM Windows Eudora Version 5.1 Date: Tue, 22 Apr 2003 10:44:00 -0300 To: obm-l@mat.puc-rio.br, obm-l@mat.puc-rio.br From: Carlos Victor Subject: =?iso-8859-1?Q?Re:_[obm-l]_Parece_f=E1cil...mas,_n=E3o_?= consigo. In-Reply-To: Mime-Version: 1.0 Content-Type: text/plain; charset="iso-8859-1"; format=flowed Content-Transfer-Encoding: 8bit X-MIME-Autoconverted: from quoted-printable to 8bit by sucuri.mat.puc-rio.br id KAA23668 Sender: owner-obm-l@sucuri.mat.puc-rio.br Precedence: bulk Reply-To: obm-l@mat.puc-rio.br Olá Oblomov , S =m^4 + n^4 - 2 = (m^2 +n^2)^2 - 2(m^2.n^2 +1) , faça m=2k+1 e n = 2p+1 e, verifique que S = ( 4t +2)^2 - 2.( 4d+2) = 8.y , para t, d e y devidamente escolhidos ;ok ? []´s Carlos Victor At 09:48 22/4/2003 -0300, Oblomov Insistenko wrote: >Agradeço a quem puder me ajudar a resolver esse problema. > >Demonstrar que, se m e n são inteiros ímpares, então >8|(m^4 + n^4 - 2). > >Obrigado! > > >_________________________________________________________________ >MSN Hotmail, o maior webmail do Brasil. http://www.hotmail.com > >========================================================================= >Instruções para entrar na lista, sair da lista e usar a lista em >http://www.mat.puc-rio.br/~nicolau/olimp/obm-l.html >O administrador desta lista é >========================================================================= ========================================================================= Instruções para entrar na lista, sair da lista e usar a lista em http://www.mat.puc-rio.br/~nicolau/olimp/obm-l.html O administrador desta lista é ========================================================================= From owner-obm-l@sucuri.mat.puc-rio.br Tue Apr 22 10:50:48 2003 Return-Path: Received: (from majordom@localhost) by sucuri.mat.puc-rio.br (8.9.3/8.9.3) id KAA23911 for obm-l-MTTP; Tue, 22 Apr 2003 10:49:29 -0300 Received: from paiol.terra.com.br (paiol.terra.com.br [200.176.3.18]) by sucuri.mat.puc-rio.br (8.9.3/8.9.3) with ESMTP id KAA23898 for ; Tue, 22 Apr 2003 10:49:24 -0300 Received: from gunga.terra.com.br (gunga.terra.com.br [200.176.3.45]) by paiol.terra.com.br (Postfix) with ESMTP id 2E90A881EF for ; Tue, 22 Apr 2003 10:48:53 -0300 (BRT) Received: from terra.com.br (200-148-121-137.dsl.telesp.net.br [200.148.121.137]) (authenticated user matematika) by gunga.terra.com.br (Postfix) with ESMTP id BFF901280A2 for ; Tue, 22 Apr 2003 10:48:52 -0300 (BRT) Message-ID: <3EA54819.6040807@terra.com.br> Date: Tue, 22 Apr 2003 10:48:09 -0300 From: Eduardo Botelho User-Agent: Mozilla/5.0 (Windows; U; Windows NT 5.0; en-US; rv:1.0.2) Gecko/20030208 Netscape/7.02 X-Accept-Language: en-us, en MIME-Version: 1.0 To: obm-l@mat.puc-rio.br Subject: [obm-l] Re: Parece =?ISO-8859-1?Q?f=E1cil=2E=2E=2Emas=2C_n=E3o_consi?= =?ISO-8859-1?Q?go=2E?= References: Content-Type: text/plain; charset=ISO-8859-1; format=flowed Content-Transfer-Encoding: 8bit Sender: owner-obm-l@sucuri.mat.puc-rio.br Precedence: bulk Reply-To: obm-l@mat.puc-rio.br Todo número ímpar ao quadrado é congruente a 1, mod 8. Assim, dois deles somados são congruentes a 2, mod 8. Abraço Eduardo Oblomov Insistenko wrote: > > Agradeço a quem puder me ajudar a resolver esse problema. > > Demonstrar que, se m e n são inteiros ímpares, então > 8|(m^4 + n^4 - 2). > > Obrigado! > > > _________________________________________________________________ > MSN Hotmail, o maior webmail do Brasil. http://www.hotmail.com > > ========================================================================= > Instruções para entrar na lista, sair da lista e usar a lista em > http://www.mat.puc-rio.br/~nicolau/olimp/obm-l.html > O administrador desta lista é > ========================================================================= > > ========================================================================= Instruções para entrar na lista, sair da lista e usar a lista em http://www.mat.puc-rio.br/~nicolau/olimp/obm-l.html O administrador desta lista é ========================================================================= From owner-obm-l@sucuri.mat.puc-rio.br Tue Apr 22 11:02:52 2003 Return-Path: Received: (from majordom@localhost) by sucuri.mat.puc-rio.br (8.9.3/8.9.3) id LAA24578 for obm-l-MTTP; Tue, 22 Apr 2003 11:01:21 -0300 Received: from hotmail.com (f112.law10.hotmail.com [64.4.15.112]) by sucuri.mat.puc-rio.br (8.9.3/8.9.3) with ESMTP id LAA24574 for ; Tue, 22 Apr 2003 11:01:17 -0300 Received: from mail pickup service by hotmail.com with Microsoft SMTPSVC; Tue, 22 Apr 2003 07:00:46 -0700 Received: from 200.147.154.143 by lw10fd.law10.hotmail.msn.com with HTTP; Tue, 22 Apr 2003 14:00:46 GMT X-Originating-IP: [200.147.154.143] X-Originating-Email: [rhel2002@hotmail.com] From: "Rubens Vilhena" To: obm-l@mat.puc-rio.br Subject: [obm-l] =?iso-8859-1?B?UmU6IFtvYm0tbF0gUGFyZWNlIGbhY2lsLi4ubWFzLCBu428gY29uc2ln?= =?iso-8859-1?B?by4=?= Date: Tue, 22 Apr 2003 14:00:46 +0000 Mime-Version: 1.0 Content-Type: text/plain; charset=iso-8859-1; format=flowed Message-ID: X-OriginalArrivalTime: 22 Apr 2003 14:00:46.0582 (UTC) FILETIME=[92870960:01C308D7] Sender: owner-obm-l@sucuri.mat.puc-rio.br Precedence: bulk Reply-To: obm-l@mat.puc-rio.br >Agradeço a quem puder me ajudar a resolver esse problema. > >Demonstrar que, se m e n são inteiros ímpares, então >8|(m^4 + n^4 - 2). > >Obrigado! Amigo, se não estou enganado, você pode fazer o seguinte: m^4 + n^4 - 2 = m^4 - 2m2n2 + n^4 + 2m2n2 - 2. m^4 + n^4 - 2 = (m2 - n2)2 + 2(m2n2 - 1). m^4 + n^4 - 2 = (m-n)2(m+n)2 + 2(mn-1)(mn+1). Como m e n são inteiros ímpares, então m - n , m + n , mn - 1 e mn + 1 são inteiros pares. Fazendo a substituição m - n = 2k1 , m + n = 2k2 , mn - 1 = 2k3 e mn + 1 = 2k4 , temos : m^4 + n^4 - 2 = (2k1)2(2k2)2 + 2(2k3)(2k4). m^4 + n^4 - 2 = 16k12k22 + 8k3k4 ou m^4 + n^4 - 2 = 8(2k12k22 + k3k4) . De onde concluímos que 8 | (m^4 + n^4 - 2) Bem, se eu estiver errado, alguém da lista me ajude. []' _________________________________________________________________ MSN Hotmail, o maior webmail do Brasil. http://www.hotmail.com ========================================================================= Instruções para entrar na lista, sair da lista e usar a lista em http://www.mat.puc-rio.br/~nicolau/olimp/obm-l.html O administrador desta lista é ========================================================================= From owner-obm-l@sucuri.mat.puc-rio.br Tue Apr 22 11:49:15 2003 Return-Path: Received: (from majordom@localhost) by sucuri.mat.puc-rio.br (8.9.3/8.9.3) id LAA26776 for obm-l-MTTP; Tue, 22 Apr 2003 11:47:00 -0300 Received: from trex.centroin.com.br (trex.centroin.com.br [200.225.63.134]) by sucuri.mat.puc-rio.br (8.9.3/8.9.3) with ESMTP id LAA26772 for ; Tue, 22 Apr 2003 11:46:57 -0300 Received: from trex.centroin.com.br (localhost [127.0.0.1]) by trex.centroin.com.br (8.12.9/8.12.9) with ESMTP id h3MEkRrL026426 for ; Tue, 22 Apr 2003 11:46:27 -0300 (EST) Received: by trex.centroin.com.br (8.12.9/8.12.5/Submit) id h3MEkR2E026420; Tue, 22 Apr 2003 11:46:27 -0300 (EST) Message-Id: <200304221446.h3MEkR2E026420@trex.centroin.com.br> Received: from 200.141.91.246 by trex.centroin.com.br (CIPWM versao 1.4C1) with HTTPS for ; Tue, 22 Apr 2003 11:46:27 -0300 (EST) Date: Tue, 22 Apr 2003 11:46:27 -0300 (EST) From: Augusto Cesar de Oliveira Morgado To: obm-l@mat.puc-rio.br Subject: =?iso-8859-1?q?Re: Re: [obm-l] An=E1lise Combinat=F3ria?= MIME-Version: 1.0 X-Mailer: CentroIn Internet Provider WebMail v. 1.4C1 (http://www.centroin.com.br/) Content-Type: text/plain; charset="iso-8859-1" Content-Transfer-Encoding: 8bit X-MIME-Autoconverted: from quoted-printable to 8bit by sucuri.mat.puc-rio.br id LAA26773 Sender: owner-obm-l@sucuri.mat.puc-rio.br Precedence: bulk Reply-To: obm-l@mat.puc-rio.br O que voce fez ta OK. Para formar o subconjunto, voce pode pensar assim: cada elemento pode figurar ou nao no subconjunto. Voce 2 modos de decidir se o primeiro elemento figura, 2 modos de decidir se o segundo elemento figura, etc. O numero de subconjuntos eh 2^6. Em Tue, 22 Apr 2003 08:40:03 EDT, SiarJoes@aol.com disse: > Muito Obrigado Morgado!!!!! > Sidmar Junior > ps: só uma dúvida, nessa número dois eu consegui fazendo combinação 1 em 6, > depois de 2 em 6, depois de 3 em 6, e assim até combinação de 6 em 6, depois > eu somei, e deu a mesma coisa, é possível fazer isso? e como você fez para > colocar 2^6 que eu não intendi direito? > > muito agradecido > Junior ========================================================================= Instruções para entrar na lista, sair da lista e usar a lista em http://www.mat.puc-rio.br/~nicolau/olimp/obm-l.html O administrador desta lista é ========================================================================= From owner-obm-l@sucuri.mat.puc-rio.br Tue Apr 22 13:31:20 2003 Return-Path: Received: (from majordom@localhost) by sucuri.mat.puc-rio.br (8.9.3/8.9.3) id NAA28906 for obm-l-MTTP; Tue, 22 Apr 2003 13:28:00 -0300 Received: from web12901.mail.yahoo.com (web12901.mail.yahoo.com [216.136.174.68]) by sucuri.mat.puc-rio.br (8.9.3/8.9.3) with SMTP id NAA28902 for ; Tue, 22 Apr 2003 13:27:56 -0300 Message-ID: <20030422162724.24689.qmail@web12901.mail.yahoo.com> Received: from [200.206.103.3] by web12901.mail.yahoo.com via HTTP; Tue, 22 Apr 2003 13:27:24 ART Date: Tue, 22 Apr 2003 13:27:24 -0300 (ART) From: =?iso-8859-1?q?Johann=20Peter=20Gustav=20Lejeune=20Dirichlet?= Subject: RE: [obm-l] Problema proposto 74 eureka 15... To: obm-l@mat.puc-rio.br In-Reply-To: <003601c306cb$3da88d50$9865fea9@computer> MIME-Version: 1.0 Content-Type: multipart/alternative; boundary="0-2026124008-1051028844=:23457" Content-Transfer-Encoding: 8bit Sender: owner-obm-l@sucuri.mat.puc-rio.br Precedence: bulk Reply-To: obm-l@mat.puc-rio.br --0-2026124008-1051028844=:23457 Content-Type: text/plain; charset=iso-8859-1 Content-Transfer-Encoding: 8bit Nao consegui muita coisa alem disso mas acho que esse cosseno nao e o mais importante poisc a diferencialidade se estende a varias funçoes... Artur Costa Steiner wrote:Este problema jah circulou na lista. O Claudio chegou a uma soluçao interessante assumindo diferenciabilidae de f. As sua solucoes, que naum sao as unicas, sao um caso particular da funcao que o Claudio achou. Uma outra hipotese trivial e a funcao identicamente nula em R (acho que tambem estah englobada na solucao do Claudio) Eu acho que sem assumir alguma condicao como diferenciabilidade fica muito dificil garantir que se encontraram todas as funcoes. Artur >-----Original Message----- >From: owner-obm-l@sucuri.mat.puc-rio.br [mailto:owner-obm-l@sucuri.mat.puc- >rio.br] On Behalf Of rmr-olimp >Sent: Saturday, April 19, 2003 7:21 PM >To: obm-l@mat.puc-rio.br >Subject: [obm-l] Problema proposto 74 eureka 15... > >Problema proposto 74 eureka 15... > >Ache todas as funçoes f: R -> R tais que: >f(x+y)+f(x-y)=2.f(x).cos(y) > >fazendo: >x+y=a >x-y=b > >Substituindo: >f(a)+f(b)=2.f((a+b)/2).cos((a-b)/2) > >Da trigonometria: >sen(a)+sen(b)=2.sen((a+b)/2).cos((a-b)/2) >cos(a)+sen(b)=2.cos((a+b)/2).cos((a-b)/2) > >Logo f(x)=sen(x) ou f(x)=cos(x) > >O que eu não sei provar é se essas são as únicas soluções >e caso existam encontrá-las.. > >Se alguém puder quebrar um galho e me ajudar seria ótimo!! > >Rodrigo > > > >_______________________________________________________________________ ___ >Seleção de Softwares UOL. >10 softwares escolhidos pelo UOL para você e sua família. >http://www.uol.com.br/selecao > > >======================================================================= == >Instruções para entrar na lista, sair da lista e usar a lista em >http://www.mat.puc-rio.br/~nicolau/olimp/obm-l.html >O administrador desta lista é >======================================================================= == ========================================================================= Instruções para entrar na lista, sair da lista e usar a lista em http://www.mat.puc-rio.br/~nicolau/olimp/obm-l.html O administrador desta lista é ========================================================================= --------------------------------- Yahoo! Mail O melhor e-mail gratuito da internet: 6MB de espaço, antivírus, acesso POP3, filtro contra spam. --0-2026124008-1051028844=:23457 Content-Type: text/html; charset=iso-8859-1 Content-Transfer-Encoding: 8bit

Nao consegui muita coisa alem disso mas acho que esse cosseno nao e o mais importante poisc a diferencialidade se estende a varias funçoes...

 Artur Costa Steiner <artur_steiner@usa.net> wrote:

Este problema jah circulou na lista. O Claudio chegou a uma soluçao
interessante assumindo diferenciabilidae de f. As sua solucoes, que naum
sao as unicas, sao um caso particular da funcao que o Claudio achou.
Uma outra hipotese trivial e a funcao identicamente nula em R (acho que
tambem estah englobada na solucao do Claudio)
Eu acho que sem assumir alguma condicao como diferenciabilidade fica
muito dificil garantir que se encontraram todas as funcoes.
Artur

>-----Original Message-----
>From: owner-obm-l@sucuri.mat.puc-rio.br
[mailto:owner-obm-l@sucuri.mat.puc-
>rio.br] On Behalf Of rmr-olimp
>Sent: Saturday, April 19, 2003 7:21 PM
>To: obm-l@mat.puc-rio.br
>Subject: [obm-l] Problema proposto 74 eureka 15...
>
>Problema proposto 74 eureka 15...
>
>Ache todas as funçoes f: R -> R tais que:
>f(x+y)+f(x-y)=2.f(x).cos(y)
>
>fazendo:
>x+y=a
>x-y=b
>
>Substituindo:
>f(a)+f(b)=2.f((a+b)/2).cos((a-b)/2)
>
>Da trigonometria:
>sen(a)+sen(b)=2.sen((a+b)/2).cos((a-b)/2)
>cos(a)+sen(b)=2.cos((a+b)/2).cos((a-b)/2)
>
>Logo f(x)=sen(x) ou f(x)=cos(x)
>
>O que eu não sei provar é se essas são as únicas soluções
>e caso existam encontrá-las..
>
>Se alguém puder quebrar um galho e me ajudar seria ótimo!!
>
>Rodrigo
>
>
>
>_______________________________________________________________________
___
>Seleção de Softwares UOL.
>10 softwares escolhidos pelo UOL para você e sua família.
>http://www.uol.com.br/selecao
>
>
>=======================================================================
==
>Instruções para entrar na lista, sair da lista e usar a lista em
>http://www.mat.puc-rio.br/~nicolau/olimp/obm-l.html
>O administrador desta lista é
>=======================================================================
==

=========================================================================
Instruções para entrar na lista, sair da lista e usar a lista em
http://www.mat.puc-rio.br/~nicolau/olimp/obm-l.html
O administrador desta lista é
=========================================================================



Yahoo! Mail
O melhor e-mail gratuito da internet: 6MB de espaço, antivírus, acesso POP3, filtro contra spam. --0-2026124008-1051028844=:23457-- ========================================================================= Instruções para entrar na lista, sair da lista e usar a lista em http://www.mat.puc-rio.br/~nicolau/olimp/obm-l.html O administrador desta lista é ========================================================================= From owner-obm-l@sucuri.mat.puc-rio.br Tue Apr 22 13:36:15 2003 Return-Path: Received: (from majordom@localhost) by sucuri.mat.puc-rio.br (8.9.3/8.9.3) id NAA29020 for obm-l-MTTP; Tue, 22 Apr 2003 13:33:43 -0300 Received: from web13001.mail.yahoo.com (web13001.mail.yahoo.com [216.136.174.11]) by sucuri.mat.puc-rio.br (8.9.3/8.9.3) with SMTP id NAA29014 for ; Tue, 22 Apr 2003 13:33:39 -0300 Message-ID: <20030422163307.13329.qmail@web13001.mail.yahoo.com> Received: from [200.148.196.25] by web13001.mail.yahoo.com via HTTP; Tue, 22 Apr 2003 13:33:07 ART Date: Tue, 22 Apr 2003 13:33:07 -0300 (ART) From: =?iso-8859-1?q?Helder=20Suzuki?= Subject: Re: [obm-l] re:2 coisinhas To: obm-l@mat.puc-rio.br In-Reply-To: <200304220819.h3M8JuoC004183@trex.centroin.com.br> MIME-Version: 1.0 Content-Type: text/plain; charset=iso-8859-1 Content-Transfer-Encoding: 8bit Sender: owner-obm-l@sucuri.mat.puc-rio.br Precedence: bulk Reply-To: obm-l@mat.puc-rio.br Pra min é exatamente isto. Você pode colocar os retângulos 2x1 em pé ou deitado. []'s, Helder Toshiro Suzuki --- Augusto Cesar de Oliveira Morgado escreveu: > Interessante. Mandei uma soluçao para o problema 1, > diferente desta, mas me passou pela cabeça a > interpretaçao do Felipe. Na minha soluçao, retangulo > 2x1 era retangulo de 2 linhas e 1 coluna OU > VICE-VERSA. > Qual eh a sua interpretaçao, Helder? > > > Em Mon, 21 Apr 2003 21:39:12 -0300, felipe mendona > disse: _______________________________________________________________________ Yahoo! GeoCities Tudo para criar o seu site: ferramentas fáceis de usar, espaço de sobra e acessórios. http://br.geocities.yahoo.com/ ========================================================================= Instruções para entrar na lista, sair da lista e usar a lista em http://www.mat.puc-rio.br/~nicolau/olimp/obm-l.html O administrador desta lista é ========================================================================= From owner-obm-l@sucuri.mat.puc-rio.br Tue Apr 22 13:58:33 2003 Return-Path: Received: (from majordom@localhost) by sucuri.mat.puc-rio.br (8.9.3/8.9.3) id NAA29720 for obm-l-MTTP; Tue, 22 Apr 2003 13:55:13 -0300 Received: from Euler.impa.br (euler.impa.br [147.65.1.3]) by sucuri.mat.puc-rio.br (8.9.3/8.9.3) with ESMTP id NAA29715 for ; Tue, 22 Apr 2003 13:55:10 -0300 Received: from Gauss.impa.br (Gauss [147.65.4.1]) by Euler.impa.br (8.11.6p2/8.11.6) with ESMTP id h3MGsd014793 for ; Tue, 22 Apr 2003 13:54:39 -0300 (EST) From: Carlos Gustavo Tamm de Araujo Moreira Received: by Gauss.impa.br (8.11.6p2) id h3MGsRr05101; Tue, 22 Apr 2003 13:54:27 -0300 (EST) Message-Id: <200304221654.h3MGsRr05101@Gauss.impa.br> Subject: Re: [obm-l] Problema proposto 74 eureka 15... To: obm-l@mat.puc-rio.br Date: Tue, 22 Apr 2003 13:54:27 -0300 (EST) In-Reply-To: <20030422162724.24689.qmail@web12901.mail.yahoo.com> from "=?iso-8859-1?q?Johann=20Peter=20Gustav=20Lejeune=20Dirichlet?=" at Apr 22, 3 01:27:24 pm X-Mailer: ELM [version 2.4 PL25] MIME-Version: 1.0 Content-Type: text/plain; charset=US-ASCII Content-Transfer-Encoding: 7bit Sender: owner-obm-l@sucuri.mat.puc-rio.br Precedence: bulk Reply-To: obm-l@mat.puc-rio.br Em e-mails recentes (de 20/4) o Shine e eu provamos sem hipoteses adicionais que f(x)=f(0)cos(x)+f(pi/2)sen(x) para todo x. Assim, f pode ser qualquer combinacao linear de sen(x) e cos(x). Abracos, Gugu > >Nao consegui muita coisa alem disso mas acho que esse cosseno nao e o mais importante poisc a diferencialidade se estende a varias funçoes... > Artur Costa Steiner wrote:Este problema jah circulou na lista. O Claudio chegou a uma soluçao >interessante assumindo diferenciabilidae de f. As sua solucoes, que naum >sao as unicas, sao um caso particular da funcao que o Claudio achou. >Uma outra hipotese trivial e a funcao identicamente nula em R (acho que >tambem estah englobada na solucao do Claudio) >Eu acho que sem assumir alguma condicao como diferenciabilidade fica >muito dificil garantir que se encontraram todas as funcoes. >Artur > >>-----Original Message----- >>From: owner-obm-l@sucuri.mat.puc-rio.br >[mailto:owner-obm-l@sucuri.mat.puc- >>rio.br] On Behalf Of rmr-olimp >>Sent: Saturday, April 19, 2003 7:21 PM >>To: obm-l@mat.puc-rio.br >>Subject: [obm-l] Problema proposto 74 eureka 15... >> >>Problema proposto 74 eureka 15... >> >>Ache todas as funçoes f: R -> R tais que: >>f(x+y)+f(x-y)=2.f(x).cos(y) >> >>fazendo: >>x+y=a >>x-y=b >> >>Substituindo: >>f(a)+f(b)=2.f((a+b)/2).cos((a-b)/2) >> >>Da trigonometria: >>sen(a)+sen(b)=2.sen((a+b)/2).cos((a-b)/2) >>cos(a)+sen(b)=2.cos((a+b)/2).cos((a-b)/2) >> >>Logo f(x)=sen(x) ou f(x)=cos(x) >> >>O que eu não sei provar é se essas são as únicas soluções >>e caso existam encontrá-las.. >> >>Se alguém puder quebrar um galho e me ajudar seria ótimo!! >> >>Rodrigo >> >> >> >>_______________________________________________________________________ >___ >>Seleção de Softwares UOL. >>10 softwares escolhidos pelo UOL para você e sua família. >>http://www.uol.com.br/selecao >> >> >>======================================================================= >== >>Instruções para entrar na lista, sair da lista e usar a lista em >>http://www.mat.puc-rio.br/~nicolau/olimp/obm-l.html >>O administrador desta lista é >>======================================================================= >== > >========================================================================= >Instruções para entrar na lista, sair da lista e usar a lista em >http://www.mat.puc-rio.br/~nicolau/olimp/obm-l.html >O administrador desta lista é >========================================================================= > > >--------------------------------- >Yahoo! Mail >O melhor e-mail gratuito da internet: 6MB de espaço, antivírus, acesso POP3, filtro contra spam. >--0-2026124008-1051028844=:23457 >Content-Type: text/html; charset=iso-8859-1 >Content-Transfer-Encoding: 8bit > >
>

Nao consegui muita coisa alem disso mas acho que esse cosseno nao e o mais importante poisc a diferencialidade se estende a varias funçoes... >

 Artur Costa Steiner <artur_steiner@usa.net> wrote: >

Este problema jah circulou na lista. O Claudio chegou a uma soluçao
interessante assumindo diferenciabilidae de f. As sua solucoes, que naum
sao as unicas, sao um caso particular da funcao que o Claudio achou.
Uma outra hipotese trivial e a funcao identicamente nula em R (acho que
tambem estah englobada na solucao do Claudio)
Eu acho que sem assumir alguma condicao como diferenciabilidade fica
muito dificil garantir que se encontraram todas as funcoes.
Artur

>-----Original Message-----
>From: owner-obm-l@sucuri.mat.puc-rio.br
[mailto:owner-obm-l@sucuri.mat.puc-
>rio.br] On Behalf Of rmr-olimp
>Sent: Saturday, April 19, 2003 7:21 PM
>To: obm-l@mat.puc-rio.br
>Subject: [obm-l] Problema proposto 74 eureka 15...
>
>Problema proposto 74 eureka 15...
>
>Ache todas as funçoes f: R -> R tais que:
>f! (x+! >y)+f(x-y)=2.f(x).cos(y)
>
>fazendo:
>x+y=a
>x-y=b
>
>Substituindo:
>f(a)+f(b)=2.f((a+b)/2).cos((a-b)/2)
>
>Da trigonometria:
>sen(a)+sen(b)=2.sen((a+b)/2).cos((a-b)/2)
>cos(a)+sen(b)=2.cos((a+b)/2).cos((a-b)/2)
>
>Logo f(x)=sen(x) ou f(x)=cos(x)
>
>O que eu não sei provar é se essas são as únicas soluções
>e caso existam encontrá-las..
>
>Se alguém puder quebrar um galho e me ajudar seria ótimo!!
>
>Rodrigo
>
>
>
>_______________________________________________________________________
___
>Seleção de Softwares UOL.
>10 softwares escolhidos pelo UOL para você e sua família.
>http://www.uol.com.br/selecao
>
>
>=======================================================================
==
>Instruções para entrar na lista, sair da lista e usar a lista em
>http://www.mat.puc-rio.br/~nicol! au/! >olimp/obm-l.html
>O administrador desta lista é
>=======================================================================
==

=========================================================================
Instruções para entrar na lista, sair da lista e usar a lista em
http://www.mat.puc-rio.br/~nicolau/olimp/obm-l.html
O administrador desta lista é
=========================================================================



Yahoo! Mail
>O melhor e-mail gratuito da internet: 6MB de espaço, antivírus, acesso POP3, filtro contra spam. >--0-2026124008-1051028844=:23457-- >========================================================================= >Instrugues para entrar na lista, sair da lista e usar a lista em >http://www.mat.puc-rio.br/~nicolau/olimp/obm-l.html >O administrador desta lista i >========================================================================= ========================================================================= Instruções para entrar na lista, sair da lista e usar a lista em http://www.mat.puc-rio.br/~nicolau/olimp/obm-l.html O administrador desta lista é ========================================================================= From owner-obm-l@sucuri.mat.puc-rio.br Tue Apr 22 14:07:16 2003 Return-Path: Received: (from majordom@localhost) by sucuri.mat.puc-rio.br (8.9.3/8.9.3) id OAA30000 for obm-l-MTTP; Tue, 22 Apr 2003 14:04:43 -0300 Received: from web12903.mail.yahoo.com (web12903.mail.yahoo.com [216.136.174.70]) by sucuri.mat.puc-rio.br (8.9.3/8.9.3) with SMTP id OAA29996 for ; Tue, 22 Apr 2003 14:04:38 -0300 Message-ID: <20030422170406.79208.qmail@web12903.mail.yahoo.com> Received: from [200.206.103.3] by web12903.mail.yahoo.com via HTTP; Tue, 22 Apr 2003 14:04:06 ART Date: Tue, 22 Apr 2003 14:04:06 -0300 (ART) From: =?iso-8859-1?q?Johann=20Peter=20Gustav=20Lejeune=20Dirichlet?= Subject: [obm-l] Problema proposto Eureka 15 To: obm-l@mat.puc-rio.br MIME-Version: 1.0 Content-Type: multipart/alternative; boundary="0-599731614-1051031046=:78096" Content-Transfer-Encoding: 8bit Sender: owner-obm-l@sucuri.mat.puc-rio.br Precedence: bulk Reply-To: obm-l@mat.puc-rio.br --0-599731614-1051031046=:78096 Content-Type: text/plain; charset=iso-8859-1 Content-Transfer-Encoding: 8bit Acho que depois de muita viajem e depressao por ter passado dias entrevado na cama de casa com gripe,consegui me restabelecer e ter uma ideia pra esse da Eureka!Ache todas as funçoes de R em R tais que f(x+y)+f(x-y)=2f(x)*cos y para todos os reais x e y Bem,revertendo:f(y+x)+f(y-x)=2f(y)*cos x.Soma as duas: 2f(x+y)+f(x-y)+f(y-x)=2(f(x)*cos y+f(y)*cos x).Mas f(x-y)+f(y-x)=2f(0)*cos(x-y)(faz x<-0 e y<-(x-y) e confere na primeira)Logo f(x+y)+f(0)*cos(x-y)=f(x)*cos y+f(y)*cos x.Pondo f(0)=z (z de zero,oras!!!)f(2a)+z*cos 2b=f(a+b)*cos(a-b)+f(a-b)*cos(a+b) por substituiçao direta.Assim da pra zuar mais:f(2a)+z=2f(a)*cos af(pi)+z=0 Com isso da pra prosseguir.Parece prostaferese,nao? --------------------------------- Yahoo! Mail O melhor e-mail gratuito da internet: 6MB de espaço, antivírus, acesso POP3, filtro contra spam. --0-599731614-1051031046=:78096 Content-Type: text/html; charset=iso-8859-1 Content-Transfer-Encoding: 8bit
Acho que depois de muita viajem e depressao por ter passado dias entrevado na cama de casa com gripe,consegui me restabelecer e ter uma ideia pra esse da Eureka!
Ache todas as funçoes de R em R tais que f(x+y)+f(x-y)=2f(x)*cos y para todos os reais x e y
 
Bem,revertendo:
f(y+x)+f(y-x)=2f(y)*cos x.
Soma as duas:
2f(x+y)+f(x-y)+f(y-x)=2(f(x)*cos y+f(y)*cos x).
Mas
f(x-y)+f(y-x)=2f(0)*cos(x-y)
(faz x<-0 e y<-(x-y) e confere na primeira)
Logo
f(x+y)+f(0)*cos(x-y)=f(x)*cos y+f(y)*cos x.
Pondo f(0)=z (z de zero,oras!!!)
f(2a)+z*cos 2b=f(a+b)*cos(a-b)+f(a-b)*cos(a+b) por substituiçao direta.
Assim da pra zuar mais:f(2a)+z=2f(a)*cos a
f(pi)+z=0
 Com isso da pra prosseguir.Parece prostaferese,nao?



Yahoo! Mail
O melhor e-mail gratuito da internet: 6MB de espaço, antivírus, acesso POP3, filtro contra spam. --0-599731614-1051031046=:78096-- ========================================================================= Instruções para entrar na lista, sair da lista e usar a lista em http://www.mat.puc-rio.br/~nicolau/olimp/obm-l.html O administrador desta lista é ========================================================================= From owner-obm-l@sucuri.mat.puc-rio.br Tue Apr 22 14:15:49 2003 Return-Path: Received: (from majordom@localhost) by sucuri.mat.puc-rio.br (8.9.3/8.9.3) id OAA30334 for obm-l-MTTP; Tue, 22 Apr 2003 14:13:11 -0300 Received: from web12905.mail.yahoo.com (web12905.mail.yahoo.com [216.136.174.72]) by sucuri.mat.puc-rio.br (8.9.3/8.9.3) with SMTP id OAA30329 for ; Tue, 22 Apr 2003 14:13:06 -0300 Message-ID: <20030422171235.20928.qmail@web12905.mail.yahoo.com> Received: from [200.206.103.3] by web12905.mail.yahoo.com via HTTP; Tue, 22 Apr 2003 14:12:35 ART Date: Tue, 22 Apr 2003 14:12:35 -0300 (ART) From: =?iso-8859-1?q?Johann=20Peter=20Gustav=20Lejeune=20Dirichlet?= Subject: Re: [obm-l] Tangência To: obm-l@mat.puc-rio.br In-Reply-To: MIME-Version: 1.0 Content-Type: multipart/alternative; boundary="0-1905800097-1051031555=:20887" Content-Transfer-Encoding: 8bit Sender: owner-obm-l@sucuri.mat.puc-rio.br Precedence: bulk Reply-To: obm-l@mat.puc-rio.br --0-1905800097-1051031555=:20887 Content-Type: text/plain; charset=iso-8859-1 Content-Transfer-Encoding: 8bit Nao ha muito o que fazer alem de contas.Pode ser chato mas e assim mesmo;):> Tente resolver esse,bem parecido: Dado un triángulo ABC y un punto D en el lado BC, se trazan las circunferencias C1 y C2 tangentes a los lados AB y BD,AD y DC respectivamente en los triángulos ABD y ADC,y tangentes a lo circuncirculo do triangulo ABC.Mostrar que las circunferencias C1 y C2 son mutuamente tangentes si y sólo si D es el punto de tangencia de la circunferencia inscrita del triángulo ABC en el lado BC. cfgauss77 wrote:Gostei desta questão: Dado un triángulo ABC y un punto D en el lado BC, se trazan las circunferencias C1 y C2 inscritas respectivamente en los triángulos ABD y ADC. Mostrar que las circunferencias C1 y C2 son mutuamente tangentes si y sólo si D es el punto de tangencia de la circunferencia C0, inscrita del triángulo ABC, en el lado BC. __________________________________________________________________________ Seleção de Softwares UOL. 10 softwares escolhidos pelo UOL para você e sua família. http://www.uol.com.br/selecao > ATTACHMENT part 2 application/octet-stream name=prb15001.gif --------------------------------- Yahoo! Mail O melhor e-mail gratuito da internet: 6MB de espaço, antivírus, acesso POP3, filtro contra spam. --0-1905800097-1051031555=:20887 Content-Type: text/html; charset=iso-8859-1 Content-Transfer-Encoding: 8bit

Nao ha muito o que fazer alem de contas.Pode ser chato mas e assim mesmo;):>

Tente resolver esse,bem parecido:

Dado un triángulo ABC y un punto D en el lado BC, se
trazan las circunferencias C1 y C2 tangentes a los lados
AB y BD,AD y DC respectivamente en los triángulos ABD y ADC,y tangentes a lo circuncirculo do triangulo ABC.Mostrar que
las circunferencias C1 y C2 son mutuamente tangentes si
y sólo si D es el punto de tangencia de la
circunferencia inscrita del triángulo ABC en el
lado BC.
 

cfgauss77 <cfgauss77@bol.com.br> wrote:

Gostei desta questão:
Dado un triángulo ABC y un punto D en el lado BC, se
trazan las circunferencias C1 y C2 inscritas
respectivamente en los triángulos ABD y ADC. Mostrar que
las circunferencias C1 y C2 son mutuamente tangentes si
y sólo si D es el punto de tangencia de la
circunferencia C0, inscrita del triángulo ABC, en el
lado BC.




__________________________________________________________________________
Seleção de Softwares UOL.
10 softwares escolhidos pelo UOL para você e sua família.
http://www.uol.com.br/selecao
> ATTACHMENT part 2 application/octet-stream name=prb15001.gif



Yahoo! Mail
O melhor e-mail gratuito da internet: 6MB de espaço, antivírus, acesso POP3, filtro contra spam. --0-1905800097-1051031555=:20887-- ========================================================================= Instruções para entrar na lista, sair da lista e usar a lista em http://www.mat.puc-rio.br/~nicolau/olimp/obm-l.html O administrador desta lista é ========================================================================= From owner-obm-l@sucuri.mat.puc-rio.br Tue Apr 22 14:29:24 2003 Return-Path: Received: (from majordom@localhost) by sucuri.mat.puc-rio.br (8.9.3/8.9.3) id OAA31257 for obm-l-MTTP; Tue, 22 Apr 2003 14:26:33 -0300 Received: from web12903.mail.yahoo.com (web12903.mail.yahoo.com [216.136.174.70]) by sucuri.mat.puc-rio.br (8.9.3/8.9.3) with SMTP id OAA31253 for ; Tue, 22 Apr 2003 14:26:29 -0300 Message-ID: <20030422172557.83275.qmail@web12903.mail.yahoo.com> Received: from [200.206.103.3] by web12903.mail.yahoo.com via HTTP; Tue, 22 Apr 2003 14:25:57 ART Date: Tue, 22 Apr 2003 14:25:57 -0300 (ART) From: =?iso-8859-1?q?Johann=20Peter=20Gustav=20Lejeune=20Dirichlet?= Subject: Re: [obm-l] Geo Plana To: obm-l@mat.puc-rio.br In-Reply-To: <3E9EC18900003EBA@www.zipmail.com.br> MIME-Version: 1.0 Content-Type: multipart/alternative; boundary="0-1878427731-1051032357=:81989" Content-Transfer-Encoding: 8bit Sender: owner-obm-l@sucuri.mat.puc-rio.br Precedence: bulk Reply-To: obm-l@mat.puc-rio.br --0-1878427731-1051032357=:81989 Content-Type: text/plain; charset=iso-8859-1 Content-Transfer-Encoding: 8bit Simplesmente demais!!!!Esse caiu na Cone Sul e foi o insano do Emanuel que propos.Segundo Tio Eddie,ou Edmilson,como quiser,era pra esse ter ido pra OBM!!!!! Enfim,as duas partes saem direto uma da outra.Basta que voce chame o angulo que ce quer que seja reto de y e ver o que acontece quando ele e 90 e comparar com o que acontece quando ABCD e ciclico luizhenriquerick@zipmail.com.br wrote:Seja ABCD um quadrilátero convexo tal que suas diagonais AC e BD são perpendiculares. Seja P a interseção de AC e BD e seja M o ponto médio de AB. Mostre que o quadrilátero ABCD é inscritível se, e somente se, as retas PM e CD são perpendiculares. Temos de provar que #ABCD é cíclico <=> PM perpendicular a DC. 1° parte : #ABCD é cíclico => PM perpendicular a DC Hipótese : #ABCD é cíclico Tese : PM perpendicular a DC ** Lema : - Seja um triângulo ABC de diâmetro BC e centro = M . - Se o ponto A ( genérico ) esta entre o arco BC , então o ângulo ABC = 90 - Como BC é diâmetro , traçamos AM , que é a mediana de BC e vale BC/2 . - Observamos que em qualquer triângulo retângulo , a medida do lado oposto ao ângulo de 90° sempre será metade desse mesmo lado . ( Olhando na figura anexa ) - Seja o # ABCD inscrito em uma circunferência de centro O . - Fazendo os ângulos : BAC = a , ABD = b , BDC = g e ACD = f . - Como M é médio de AB e o ângulo APB = 90° , temos pelo lema que MP = AM = MB . - Assim os ângulos APM = a e BPM = b . - Como ângulos opostos pelo vértice são iguais , então : DPM1 = BPM = b CPM1 = APM = a - Sendo os ângulos MM1C = k e MM1D = k2. - Como a e g (falam) para um mesmo arco , então são iguais .O mesmo ocorre com os ângulos b e f . - No triângulo ABP : a + b + 90 = 180 => a + b = 90 ( i ) - Nos triângulos PM1D e PM1C , temos : b + g + k2 = 180 e a + f + k = 180 ou ( b + a ) + k2 = 180 ( ii ) e ( a + b) + k = 180 ( iii ) De ( i ) em ( ii ) e ( iii ) , vem : 90 + k2 = 180 e 90 + k = 180 k2 = 90 e k = 90 Agora , como faço para provar a 2° parte do problema ? PM perpendicular a DC => #ABCD é cíclico . Qualquer ajuda será bem vinda . Abraços . Luiz H. Barbosa . www.olympicmaths.hpg.com.br ------------------------------------------ Use o melhor sistema de busca da Internet Radar UOL - http://www.radaruol.com.br > ATTACHMENT part 2 image/gif --------------------------------- Yahoo! Mail O melhor e-mail gratuito da internet: 6MB de espaço, antivírus, acesso POP3, filtro contra spam. --0-1878427731-1051032357=:81989 Content-Type: text/html; charset=iso-8859-1 Content-Transfer-Encoding: 8bit

Simplesmente demais!!!!Esse caiu na Cone Sul e foi o insano do Emanuel que propos.Segundo Tio Eddie,ou Edmilson,como quiser,era pra esse ter ido pra OBM!!!!!

Enfim,as duas partes saem direto uma da outra.Basta que voce chame o angulo que ce quer que seja reto de y e ver o que acontece quando ele e 90 e comparar com o que acontece quando ABCD e ciclico

 luizhenriquerick@zipmail.com.br wrote:

Seja ABCD um quadrilátero convexo tal que suas diagonais AC e BD são perpendiculares.
Seja P a interseção de AC e BD e seja M o ponto médio de AB. Mostre que
o quadrilátero ABCD é inscritível se, e somente se, as retas PM e CD são
perpendiculares.


Temos de provar que #ABCD é cíclico <=> PM perpendicular a DC.

1° parte :
#ABCD é cíclico => PM perpendicular a DC

Hipótese : #ABCD é cíclico
Tese : PM perpendicular a DC

** Lema :
- Seja um triângulo ABC de diâmetro BC e centro = M .
- Se o ponto A ( genérico ) esta entre o arco BC , então o ângulo ABC =
90
- Como BC é diâmetro , traçamos AM , que é a mediana de BC e vale BC/2 .
- Observamos que em qualquer triângulo retângulo , a medida do lado oposto
ao ângulo de 90° sempre será metade desse mesmo lado .


( Olhando na figura anexa )

- Seja o # ABCD inscrito em uma circunferência de centro O .
- Fazendo os ângulos :
BAC = a , ABD = b , BDC = g e ACD = f .
- Como M é médio de AB e o ângulo APB = 90° , temos pelo lema que MP = AM
= MB .
- Assim os ângulos APM = a e BPM = b .
- Como ângulos opostos pelo vértice são iguais , então :
DPM1 = BPM = b
CPM1 = APM = a
- Sendo os ângulos MM1C = k e MM1D = k2.
- Como a e g (falam) para um mesmo arco , então são iguais .O mesmo ocorre
com os ângulos b e f .
- No triângulo ABP :

a + b + 90 = 180 => a + b = 90 ( i )

- Nos triângulos PM1D e PM1C , temos :
b + g + k2 = 180 e a + f + k = 180

ou

( b + a ) + k2 = 180 ( ii ) e ( a + b) + k = 180 ( iii )

De ( i ) em ( ii ) e ( iii ) , vem :

90 + k2 = 180 e 90 + k = 180

k2 = 90 e k = 90



Agora , como faço para provar a 2° parte do problema ?

PM perpendicular a DC => #ABCD é cíclico .


Qualquer ajuda será bem vinda .

Abraços .

Luiz H. Barbosa .





www.olympicmaths.hpg.com.br


------------------------------------------
Use o melhor sistema de busca da Internet
Radar UOL - http://www.radaruol.com.br




> ATTACHMENT part 2 image/gif



Yahoo! Mail
O melhor e-mail gratuito da internet: 6MB de espaço, antivírus, acesso POP3, filtro contra spam. --0-1878427731-1051032357=:81989-- ========================================================================= Instruções para entrar na lista, sair da lista e usar a lista em http://www.mat.puc-rio.br/~nicolau/olimp/obm-l.html O administrador desta lista é ========================================================================= From owner-obm-l@sucuri.mat.puc-rio.br Tue Apr 22 15:27:17 2003 Return-Path: Received: (from majordom@localhost) by sucuri.mat.puc-rio.br (8.9.3/8.9.3) id PAA00808 for obm-l-MTTP; Tue, 22 Apr 2003 15:23:28 -0300 Received: from cmsrelay02.mx.net (cmsrelay02.mx.net [165.212.11.111]) by sucuri.mat.puc-rio.br (8.9.3/8.9.3) with SMTP id PAA00794 for ; Tue, 22 Apr 2003 15:23:22 -0300 Received: from uadvg128.cms.usa.net (HELO localhost) (165.212.11.128) by cmsoutbound.mx.net with SMTP; 22 Apr 2003 18:22:47 -0000 Received: from smtp.postoffice.net [165.212.8.26] by uadvg128.cms.usa.net (ASMTP/) via mtad (C8.MAIN.2.05) with ESMTP id 452HDVswt0495M28; Tue, 22 Apr 2003 18:22:46 GMT Received: from 200.181.4.100 [200.181.4.100] by cmsweb10.cms.usa.net (USANET web-mailer CM.0402.5.2B); Tue, 22 Apr 2003 18:22:45 -0000 Date: Tue, 22 Apr 2003 15:22:45 -0300 From: Artur Costa Steiner To: Subject: Re: [[obm-l] Problema proposto Eureka 15] X-Mailer: USANET web-mailer (CM.0402.5.2B) Mime-Version: 1.0 Message-ID: <839HDVswt6304S10.1051035765@cmsweb10.cms.usa.net> Content-Type: text/plain; charset=ISO-8859-1 Content-Transfer-Encoding: 8bit X-MIME-Autoconverted: from quoted-printable to 8bit by sucuri.mat.puc-rio.br id PAA00805 Sender: owner-obm-l@sucuri.mat.puc-rio.br Precedence: bulk Reply-To: obm-l@mat.puc-rio.br Melhoras para vc! Eu realmente nao sei o que eh prostaferese! O que vc fez, acho que esta OK Um abraco Artur > --------------------------------------------- > Acho que depois de muita viajem e depressao por ter passado dias entrevado na cama de casa com gripe,consegui me restabelecer e ter uma ideia pra esse da Eureka!Ache todas as funçoes de R em R tais que f(x+y)+f(x-y)=2f(x)*cos y para todos os reais x e y Bem,revertendo:f(y+x)+f(y-x)=2f(y)*cos x.Soma as duas: > 2f(x+y)+f(x-y)+f(y-x)=2(f(x)*cos y+f(y)*cos x).Mas f(x-y)+f(y-x)=2f(0)*cos(x-y)(faz x<-0 e y<-(x-y) e confere na primeira)Logo f(x+y)+f(0)*cos(x-y)=f(x)*cos y+f(y)*cos x.Pondo f(0)=z (z de zero,oras!!!)f(2a)+z*cos 2b=f(a+b)*cos(a-b)+f(a-b)*cos(a+b) por substituiçao direta.Assim da pra zuar mais:f(2a)+z=2f(a)*cos af(pi)+z=0 Com isso da pra prosseguir.Parece prostaferese,nao? > > > --------------------------------- > Yahoo! Mail > O melhor e-mail gratuito da internet: 6MB de espaço, antivírus, acesso POP3, filtro contra spam. ========================================================================= Instruções para entrar na lista, sair da lista e usar a lista em http://www.mat.puc-rio.br/~nicolau/olimp/obm-l.html ========================================================================= From owner-obm-l@sucuri.mat.puc-rio.br Tue Apr 22 15:35:35 2003 Return-Path: Received: (from majordom@localhost) by sucuri.mat.puc-rio.br (8.9.3/8.9.3) id PAA01023 for obm-l-MTTP; Tue, 22 Apr 2003 15:33:02 -0300 Received: from trex.centroin.com.br (trex.centroin.com.br [200.225.63.134]) by sucuri.mat.puc-rio.br (8.9.3/8.9.3) with ESMTP id PAA01019 for ; Tue, 22 Apr 2003 15:32:58 -0300 Received: from trex.centroin.com.br (localhost [127.0.0.1]) by trex.centroin.com.br (8.12.9/8.12.9) with ESMTP id h3MIWTrL010896 for ; Tue, 22 Apr 2003 15:32:29 -0300 (EST) Received: by trex.centroin.com.br (8.12.9/8.12.5/Submit) id h3MIWSl7010895; Tue, 22 Apr 2003 15:32:28 -0300 (EST) Message-Id: <200304221832.h3MIWSl7010895@trex.centroin.com.br> Received: from 200.165.208.164 by trex.centroin.com.br (CIPWM versao 1.4C1) with HTTPS for ; Tue, 22 Apr 2003 15:32:28 -0300 (EST) Date: Tue, 22 Apr 2003 15:32:28 -0300 (EST) From: Augusto Cesar de Oliveira Morgado To: obm-l@mat.puc-rio.br Subject: [obm-l] =?iso-8859-1?q?En: quest=F5es_mal_formuladas?= MIME-Version: 1.0 X-Mailer: CentroIn Internet Provider WebMail v. 1.4C1 (http://www.centroin.com.br/) Content-Type: text/plain; charset="iso-8859-1" Content-Transfer-Encoding: 8bit X-MIME-Autoconverted: from quoted-printable to 8bit by sucuri.mat.puc-rio.br id PAA01020 Sender: owner-obm-l@sucuri.mat.puc-rio.br Precedence: bulk Reply-To: obm-l@mat.puc-rio.br Vou tomar a liberdade de repassar uma mensagem particular que mostra que o problema dos enunciados mal formulados eh antigo. E olha que nao eh nenhum MDC. Observo que, embora nao seja cego, a escrita em Braile eh formada por pontos em relevo. Em Tue, 22 Apr 2003 14:36:30 -0300, Jose Francisco Guimaraes Costa disse: Morgado, Ainda bem que já lá se foi meu tempo de vestibular. No caso abaixo, só masoquismo explica o uso do termo "destacar" por quem propos o problema. O próprio Aurélio admite as duas interpretações: "ser diferente de", quando a resposta seria 62, e "estar em relevo", quando a resposta seria 63. Na realidade é "evidente" que o objetivo é o segundo, já que se não existe nenhum ponto em alto relevo, o cego não vai conseguir saber que existe um caracter ali. Mas o pobre do vestibulando fica na maior m***** tentando descobrir qual deve ser a interpretação de "destacar", e não consegue fazer mais nada. No _meu_ tempo de vestibular eu tive um problema desses. Quando fui fazer os vestibulares da PUC e da Escola Nacional de Engenharia eu já sabia que havia sido aprovado na escola que eu realmente queria cursar. Fiz ambos por pura pressão doméstica. Com a cuca o mais leve possivel, ia fazer as provas de calção por baixo da roupa, para poder ir para a praia direto depois da prova. Na primeira prova da PUC, numa questão envolvendo elipse, os dados eram incompatíveis. Eu vi logo que eram, chamei o fiscal e mostrei o problema. Recebi como resposta algo do tipo "meu filho, os dados estão certos; não fique nervoso, pule a questão, e se der tempo, no fim volte a ela". Uns quinze minutos depois veio o aviso: "na questão número x, onde está y leia-se z". Em seguida veio o fiscal me perguntar como é que eu havia tido cabeça para ver que o problema era impossivel. Tenho certeza que muita gente se enterrou no vestibular naquela questão. É apavorante o número de questões mal formuladas, com sentidos múltiplos, que vejo circular por este forum. Aplica-se aqui um dos dois principais lemas dos engenheiros: KISS (Keep It Simple, Stupid). O outro é "if it ain't broke, don't fix it". JF ----- Original Message ----- From: "Augusto Cesar de Oliveira Morgado" To: Sent: Tuesday, April 22, 2003 5:30 AM Subject: Re: [obm-l] Análise Combinatória 2) Essa caiu em vestibular em Sao Paulo e deu um rolo... Voce tem que escolher o subconjunto dos simbolos a destacar, o que voce pode fazer de 2^6 = 64 modos. Nao vale o vazio (pelo menos um deve se destacar) e a resposta (oficial) eh 63. Muitos professores reclamaram do gabarito oficial alegando que se todos se destacam, ninguem se destaca. Para eles a resposta era 62. Em Mon, 21 Apr 2003 22:54:56 EDT, SiarJoes@aol.com disse: A escrita em Braile é um sistema de símbolos em que cada um dos caracteres é formado por uma matriz de seis pondos, dos quais pelo menos um se destaca .Assim: A B * . * . . . * . . . . . Qual o número máximo de caracteres distintos que podem ser representados nesse sitema? obrigado Junior ========================================================================= Instruções para entrar na lista, sair da lista e usar a lista em http://www.mat.puc-rio.br/~nicolau/olimp/obm-l.html ========================================================================= From owner-obm-l@sucuri.mat.puc-rio.br Tue Apr 22 15:36:12 2003 Return-Path: Received: (from majordom@localhost) by sucuri.mat.puc-rio.br (8.9.3/8.9.3) id PAA01052 for obm-l-MTTP; Tue, 22 Apr 2003 15:33:41 -0300 Received: from ns3bind.localdomain ([200.230.34.5]) by sucuri.mat.puc-rio.br (8.9.3/8.9.3) with ESMTP id PAA01048 for ; Tue, 22 Apr 2003 15:33:37 -0300 Received: from servico2 ([200.230.34.229]) by ns3bind.localdomain (8.11.6/X.XX.X) with SMTP id h3MIT1R27812 for ; Tue, 22 Apr 2003 15:29:01 -0300 Message-ID: <018a01c308fd$c99a8140$3300c57d@bovespa.com> From: "=?iso-8859-1?Q?Cl=E1udio_\=28Pr=E1tica\=29?=" To: References: <200304221654.h3MGsRr05101@Gauss.impa.br> Subject: Re: [obm-l] Problema proposto 74 eureka 15... Date: Tue, 22 Apr 2003 15:34:18 -0300 MIME-Version: 1.0 Content-Type: text/plain; charset="iso-8859-1" Content-Transfer-Encoding: 8bit X-Priority: 3 X-MSMail-Priority: Normal X-Mailer: Microsoft Outlook Express 5.50.4920.2300 X-MimeOLE: Produced By Microsoft MimeOLE V5.50.4920.2300 Sender: owner-obm-l@sucuri.mat.puc-rio.br Precedence: bulk Reply-To: obm-l@mat.puc-rio.br Caros Gugu, Shine e Artur: Estou plenamente convencido. O mais interessante é que eu achei a mesma solução sob a hipótese restritiva de que f é diferenciável, e que o efeito relevante de se diferenciar f é justamente o de se deslocar a variável independente de Pi/2, artifício que o Shine e o Gugu usaram na solução deles: D(sen(x)) = cos(x) = sen(x + Pi/2) D(cos(x)) = - sen(x) = cos(x + Pi/2) Ou seja, ao invés de diferenciar f, bastava avaliá-la em x + Pi/2. Naturalmente, fica óbvio depois que alguém mostra o caminho. Obrigado e um abraço, Claudio. ----- Original Message ----- From: "Carlos Gustavo Tamm de Araujo Moreira" To: Sent: Tuesday, April 22, 2003 1:54 PM Subject: Re: [obm-l] Problema proposto 74 eureka 15... > Em e-mails recentes (de 20/4) o Shine e eu provamos sem hipoteses > adicionais que f(x)=f(0)cos(x)+f(pi/2)sen(x) para todo x. Assim, f pode ser > qualquer combinacao linear de sen(x) e cos(x). > Abracos, > Gugu > > > > >Nao consegui muita coisa alem disso mas acho que esse cosseno nao e o mais importante poisc a diferencialidade se estende a varias funçoes... > > Artur Costa Steiner wrote:Este problema jah circulou na lista. O Claudio chegou a uma soluçao > >interessante assumindo diferenciabilidae de f. As sua solucoes, que naum > >sao as unicas, sao um caso particular da funcao que o Claudio achou. > >Uma outra hipotese trivial e a funcao identicamente nula em R (acho que > >tambem estah englobada na solucao do Claudio) > >Eu acho que sem assumir alguma condicao como diferenciabilidade fica > >muito dificil garantir que se encontraram todas as funcoes. > >Artur > > > >>-----Original Message----- > >>From: owner-obm-l@sucuri.mat.puc-rio.br > >[mailto:owner-obm-l@sucuri.mat.puc- > >>rio.br] On Behalf Of rmr-olimp > >>Sent: Saturday, April 19, 2003 7:21 PM > >>To: obm-l@mat.puc-rio.br > >>Subject: [obm-l] Problema proposto 74 eureka 15... > >> > >>Problema proposto 74 eureka 15... > >> > >>Ache todas as funçoes f: R -> R tais que: > >>f(x+y)+f(x-y)=2.f(x).cos(y) > >> > >>fazendo: > >>x+y=a > >>x-y=b > >> > >>Substituindo: > >>f(a)+f(b)=2.f((a+b)/2).cos((a-b)/2) > >> > >>Da trigonometria: > >>sen(a)+sen(b)=2.sen((a+b)/2).cos((a-b)/2) > >>cos(a)+sen(b)=2.cos((a+b)/2).cos((a-b)/2) > >> > >>Logo f(x)=sen(x) ou f(x)=cos(x) > >> > >>O que eu não sei provar é se essas são as únicas soluções > >>e caso existam encontrá-las.. > >> > >>Se alguém puder quebrar um galho e me ajudar seria ótimo!! > >> > >>Rodrigo > >> > >> > >> > >>_______________________________________________________________________ > >___ > >>Seleção de Softwares UOL. > >>10 softwares escolhidos pelo UOL para você e sua família. > >>http://www.uol.com.br/selecao > >> > >> > >>======================================================================= > >== > >>Instruções para entrar na lista, sair da lista e usar a lista em > >>http://www.mat.puc-rio.br/~nicolau/olimp/obm-l.html > >>O administrador desta lista é > >>======================================================================= > >== > > > >========================================================================= > >Instruções para entrar na lista, sair da lista e usar a lista em > >http://www.mat.puc-rio.br/~nicolau/olimp/obm-l.html > >O administrador desta lista é > >========================================================================= > > > > > >--------------------------------- > >Yahoo! Mail > >O melhor e-mail gratuito da internet: 6MB de espaço, antivírus, acesso POP3, filtro contra spam. > >--0-2026124008-1051028844=:23457 > >Content-Type: text/html; charset=iso-8859-1 > >Content-Transfer-Encoding: 8bit > > > >
> >

Nao consegui muita coisa alem disso mas acho que esse cosseno nao e o mais importante poisc a diferencialidade se estende a varias funçoes... > >

 Artur Costa Steiner <artur_steiner@usa.net> wrote: > >

Este problema jah circulou na lista. O Claudio chegou a uma soluçao
interessante assumindo diferenciabilidae de f. As sua solucoes, que naum
sao as unicas, sao um caso particular da funcao que o Claudio achou.
Uma outra hipotese trivial e a funcao identicamente nula em R (acho que
tambem estah englobada na solucao do Claudio)
Eu acho que sem assumir alguma condicao como diferenciabilidade fica
muito dificil garantir que se encontraram todas as funcoes.
Artur

>-----Original Message-----
>From: owner-obm-l@sucuri.mat.puc-rio.br
[mailto:owner-obm-l@sucuri.mat.puc-
>rio.br] On Behalf Of rmr-olimp
>Sent: Saturday, April 19, 2003 7:21 PM
>To: obm-l@mat.puc-rio.br
>Subject: [obm-l] Problema proposto 74 eureka 15...
>
>Problema proposto 74 eureka 15...
>
>Ache todas as funçoes f: R -> R tais que:
>f! > (x+! > >y)+f(x-y)=2.f(x).cos(y)
>
>fazendo:
>x+y=a
>x-y=b>
>Substituindo:
>f(a)+f(b)=2.f((a+b)/2).cos((a-b)/2)
&g t;
>Da trigonometria:
>sen(a)+sen(b)=2.sen((a+b)/2).cos((a-b)/2)
>cos(a )+sen(b)=2.cos((a+b)/2).cos((a-b)/2)
>
>Logo f(x)=sen(x) ou f(x)=cos(x)
>
>O que eu não sei provar é se essas são as únicas soluções
>e caso existam encontrá-las..
>
>Se alguém puder quebrar um galho e me ajudar seria ótimo!!
>
>Rodrigo
>
>
>
>______________ _________________________________________________________
___
>Sele ção de Softwares UOL.
>10 softwares escolhidos pelo UOL para você e sua família.
>http://www.uol.com.br/selecao
>
>
>======= ================================================================
==
&g t;Instruções para entrar na lista, sair da lista e usar a lista em
>http://www.mat.puc-rio.br/~nicol! > au/! > >olimp/obm-l.html
>O administrador desta lista é
>============================================ ===========================
==

=================================== ======================================
Instruções para entrar na lista, sair da lista e usar a lista em
http://www.mat.puc-rio.br/~nicolau/olimp/obm-l.html
O administrador desta lista é
================================================ =========================



Yahoo! Mail
> >O melhor e-mail gratuito da internet: 6MB de espaço, antivírus, acesso POP3, filtro contra spam. > >--0-2026124008-1051028844=:23457-- > >========================================================================= > >Instrugues para entrar na lista, sair da lista e usar a lista em > >http://www.mat.puc-rio.br/~nicolau/olimp/obm-l.html > >O administrador desta lista i > >========================================================================= > > ========================================================================= > Instruções para entrar na lista, sair da lista e usar a lista em > http://www.mat.puc-rio.br/~nicolau/olimp/obm-l.html > O administrador desta lista é > ========================================================================= ========================================================================= Instruções para entrar na lista, sair da lista e usar a lista em http://www.mat.puc-rio.br/~nicolau/olimp/obm-l.html ========================================================================= From owner-obm-l@sucuri.mat.puc-rio.br Tue Apr 22 15:56:54 2003 Return-Path: Received: (from majordom@localhost) by sucuri.mat.puc-rio.br (8.9.3/8.9.3) id PAA01981 for obm-l-MTTP; Tue, 22 Apr 2003 15:53:30 -0300 Received: from hotmail.com (f74.law9.hotmail.com [64.4.9.74]) by sucuri.mat.puc-rio.br (8.9.3/8.9.3) with ESMTP id PAA01976 for ; Tue, 22 Apr 2003 15:53:26 -0300 Received: from mail pickup service by hotmail.com with Microsoft SMTPSVC; Tue, 22 Apr 2003 11:52:52 -0700 Received: from 200.151.189.81 by lw9fd.law9.hotmail.msn.com with HTTP; Tue, 22 Apr 2003 18:52:52 GMT X-Originating-IP: [200.151.189.81] X-Originating-Email: [santanahenrique@hotmail.com] From: "Henrique Lima Santana" To: obm-l@mat.puc-rio.br Subject: Re: [obm-l] Parece fácil...mas, não consigo. Date: Tue, 22 Apr 2003 18:52:52 +0000 Mime-Version: 1.0 Content-Type: text/plain; format=flowed Message-ID: X-OriginalArrivalTime: 22 Apr 2003 18:52:52.0375 (UTC) FILETIME=[60B6D270:01C30900] Sender: owner-obm-l@sucuri.mat.puc-rio.br Precedence: bulk Reply-To: obm-l@mat.puc-rio.br usando congruencias: sabemos q um nº impar eh da forma 8k+-1 ou 8k+-3, portanto sua quarta potencia soh poderah ser da forma 8k+1 ou seja , sendo i um nº impar => i^4==1(mod8) assim, m^4+n^4==2(mod8) => => m^4 + n^4 -2==0(mod8) => 8|m^4+n^4-2 c.q.d. Henrique >From: "Oblomov Insistenko" >Reply-To: obm-l@mat.puc-rio.br >To: obm-l@mat.puc-rio.br >Subject: [obm-l] Parece fácil...mas, não consigo. >Date: Tue, 22 Apr 2003 09:48:00 -0300 > > >Agradeço a quem puder me ajudar a resolver esse problema. > >Demonstrar que, se m e n são inteiros ímpares, então >8|(m^4 + n^4 - 2). > >Obrigado! > > >_________________________________________________________________ >MSN Hotmail, o maior webmail do Brasil. http://www.hotmail.com > >========================================================================= >Instruções para entrar na lista, sair da lista e usar a lista em >http://www.mat.puc-rio.br/~nicolau/olimp/obm-l.html >O administrador desta lista é >====================================================================== _________________________________________________________________ Add photos to your messages with MSN 8. Get 2 months FREE*. http://join.msn.com/?page=features/featuredemail ========================================================================= Instruções para entrar na lista, sair da lista e usar a lista em http://www.mat.puc-rio.br/~nicolau/olimp/obm-l.html ========================================================================= From owner-obm-l@sucuri.mat.puc-rio.br Tue Apr 22 16:09:40 2003 Return-Path: Received: (from majordom@localhost) by sucuri.mat.puc-rio.br (8.9.3/8.9.3) id QAA02582 for obm-l-MTTP; Tue, 22 Apr 2003 16:08:00 -0300 Received: from ns3bind.localdomain ([200.230.34.5]) by sucuri.mat.puc-rio.br (8.9.3/8.9.3) with ESMTP id QAA02574 for ; Tue, 22 Apr 2003 16:07:55 -0300 Received: from servico2 ([200.230.34.227]) by ns3bind.localdomain (8.11.6/X.XX.X) with SMTP id h3MJ3JR30449 for ; Tue, 22 Apr 2003 16:03:19 -0300 Message-ID: <01b701c30902$946f1bc0$3300c57d@bovespa.com> From: "=?iso-8859-1?Q?Cl=E1udio_\=28Pr=E1tica\=29?=" To: References: <6732305773.20030411013805@gmx.net> <00ba01c3005e$56faaae0$3300c57d@bovespa.com> <012a01c3006b$dcae0a80$5400a8c0@ensrbr> <00ed01c30514$eb7c9100$5400a8c0@ensrbr> Subject: [obm-l] =?iso-8859-1?Q?Re:_=5Bobm-l=5D_Dois_problemas_do_mesmo_site_=5Bera:Trigon?= =?iso-8859-1?Q?ometria_e_Sequ=EAncias=5D?= Date: Tue, 22 Apr 2003 16:08:31 -0300 MIME-Version: 1.0 Content-Type: text/plain; charset="iso-8859-1" Content-Transfer-Encoding: 8bit X-Priority: 3 X-MSMail-Priority: Normal X-Mailer: Microsoft Outlook Express 5.50.4920.2300 X-MimeOLE: Produced By Microsoft MimeOLE V5.50.4920.2300 Sender: owner-obm-l@sucuri.mat.puc-rio.br Precedence: bulk Reply-To: obm-l@mat.puc-rio.br ----- Original Message ----- From: "Luis Lopes" To: Sent: Thursday, April 17, 2003 4:09 PM Subject: [obm-l] Dois problemas do mesmo site [era:Trigonometria e Sequências] > Sauda,c~oes, > > Na mensagem do Claudio Buffara sobre > aparece > o site > > Na internet voce tambem encontra alguns sites com > problemas interessantes, tais como este aqui: > > http://math.stanford.edu/~vakil/stanfordputnam/02/putnam3.pdf > > Vasculhando o site acima encontrei acho que em > > http://math.stanford.edu/~vakil/stanfordputnam/02/putnam1.pdf > > alguns problemas. Proponho novamente dois problemas, > mas desta vez não tenho a solução de nenhum dos dois: > > Com uma notação resumida, > > 1) mostre que existe n tal que 2^n = 2002....... > > 2) 10000! = ..........n0000000000...000. Calcule n. > > Obs.: o número de zeros em n0000000000...000 > eh um problema conhecido. > > []'s > Luís > Oi, Luis: O no. 1 é consequencia do fato de que log(2) na base 10 é irracional e, portanto, existem inteiros m e n tais que: log(2002) + n < m*log(2) < log(2003) + n. Eu apresentei esse problema há alguns meses ao site Ask Dr. Math e um dos matemáticos de lá me deu a dica crucial. A discussão está aqui: http://mathforum.org/library/drmath/view/61545.html O no. 2 é menos conhecido do que o problema sobre o número de zeros no final de 10.000! mas, mesmo assim, é um belo problema de teoria dos números. Uma solução está aqui: http://www.sci.csuhayward.edu/mathcs/pom/pom95.html Um abraço, Claudio. ========================================================================= Instruções para entrar na lista, sair da lista e usar a lista em http://www.mat.puc-rio.br/~nicolau/olimp/obm-l.html ========================================================================= From owner-obm-l@sucuri.mat.puc-rio.br Tue Apr 22 16:21:59 2003 Return-Path: Received: (from majordom@localhost) by sucuri.mat.puc-rio.br (8.9.3/8.9.3) id QAA03037 for obm-l-MTTP; Tue, 22 Apr 2003 16:18:24 -0300 Received: from cmsrelay02.mx.net (cmsrelay02.mx.net [165.212.11.111]) by sucuri.mat.puc-rio.br (8.9.3/8.9.3) with SMTP id QAA03032 for ; Tue, 22 Apr 2003 16:18:20 -0300 Received: from uadvg131.cms.usa.net (HELO localhost) (165.212.11.131) by cmsoutbound.mx.net with SMTP; 22 Apr 2003 19:17:45 -0000 Received: from smtp.postoffice.net [165.212.8.11] by uadvg131.cms.usa.net (ASMTP/) via mtad (C8.MAIN.2.05) with ESMTP id 438HDVTRr0379M31; Tue, 22 Apr 2003 19:17:43 GMT Received: from 200.181.4.100 [200.181.4.100] by uwdvg001.cms.usa.net (USANET web-mailer CM.0402.5.2B); Tue, 22 Apr 2003 19:17:42 -0000 Date: Tue, 22 Apr 2003 16:17:42 -0300 From: Artur Costa Steiner To: Subject: [obm-l] =?ISO-8859-1?Q?Re=3A=20=5B=5Bobm=2Dl=5D=20En=3A=20quest=F5e?= =?ISO-8859-1?Q?s=20mal=20formuladas=5D?= X-Mailer: USANET web-mailer (CM.0402.5.2B) Mime-Version: 1.0 Message-ID: <578HDVTRQ9248S01.1051039062@uwdvg001.cms.usa.net> Content-Type: text/plain; charset=ISO-8859-1 Content-Transfer-Encoding: 8bit X-MIME-Autoconverted: from quoted-printable to 8bit by sucuri.mat.puc-rio.br id QAA03033 Sender: owner-obm-l@sucuri.mat.puc-rio.br Precedence: bulk Reply-To: obm-l@mat.puc-rio.br > > Aplica-se aqui um dos dois principais lemas dos engenheiros: KISS (Keep It Simple, Stupid). O outro é "if it ain't broke, don't fix it". > Talvez seja por isso que alguns dizem que engenheiro eh aquele que consegue fazer de forma barata o que qualquer um faria de forma cara.... :) Artur ========================================================================= Instruções para entrar na lista, sair da lista e usar a lista em http://www.mat.puc-rio.br/~nicolau/olimp/obm-l.html ========================================================================= From owner-obm-l@sucuri.mat.puc-rio.br Tue Apr 22 17:17:08 2003 Return-Path: Received: (from majordom@localhost) by sucuri.mat.puc-rio.br (8.9.3/8.9.3) id RAA05304 for obm-l-MTTP; Tue, 22 Apr 2003 17:15:12 -0300 Received: from ns3bind.localdomain ([200.230.34.5]) by sucuri.mat.puc-rio.br (8.9.3/8.9.3) with ESMTP id RAA05300 for ; Tue, 22 Apr 2003 17:15:09 -0300 Received: from servico2 ([200.230.34.227]) by ns3bind.localdomain (8.11.6/X.XX.X) with SMTP id h3MKATR03382 for ; Tue, 22 Apr 2003 17:10:29 -0300 Message-ID: <029b01c3090b$f897e420$3300c57d@bovespa.com> From: "=?iso-8859-1?Q?Cl=E1udio_\=28Pr=E1tica\=29?=" To: References: <001701c306a6$a04f1160$f4befea9@xx> <3EA1A4D0.4050205@centroin.com.br> <200304191758510720.01B2F31B@smtp.watersportsbrazil.com> <3EA1D12A.8070101@terra.com.br> Subject: [obm-l] =?iso-8859-1?Q?Re:_=5Bobm-l=5D_D=FAvida_sobre_grupos_diedrais?= Date: Tue, 22 Apr 2003 17:15:41 -0300 MIME-Version: 1.0 Content-Type: text/plain; charset="iso-8859-1" Content-Transfer-Encoding: 8bit X-Priority: 3 X-MSMail-Priority: Normal X-Mailer: Microsoft Outlook Express 5.50.4920.2300 X-MimeOLE: Produced By Microsoft MimeOLE V5.50.4920.2300 Sender: owner-obm-l@sucuri.mat.puc-rio.br Precedence: bulk Reply-To: obm-l@mat.puc-rio.br Caro Eduardo: Demonstrar associatividade normalmente é um saco. No caso de D(2n), a melhor forma é obter uma representação (ou seja, uma instância "concreta") do grupo como uma série de operações no plano complexo. Por exemplo, dado um número complexo z qualquer, você pode definir duas funções, "a" e "b" de C em C (C = conjunto dos números complexos) tais que: a(z) = exp(2*Pi/n)*z e b(z) = conjugado(z) Ou seja, "a" efetua uma rotação de z de um ângulo igual a 2*Pi/n em torno da origem no sentido anti-horário, e "b" efetua a reflexão de z no eixo real. Assim, teríamos: a^k(z) = exp(2*k*Pi/n)*z = a(a^(k-1)(z)) para todo k inteiro. Logo: 1) ab(z) = a(b(z)) = a(conjugado(z)) = exp(2*Pi/n)*conjugado(z) 2) ba^(n-1)(z) = b(a^(n-1)(z)) = b(exp(2*(n-1)*Pi/n)*z) = b(exp(-2*Pi/n)*z) = = conjugado(exp(-2*Pi/n)*z) = conjugado(exp(-2*Pi/n))*conjugado(z) = = exp(2*Pi/n)*conjugado(z) Ou seja, ab(z) = ba^(n-1)(z) para todo z complexo. Logo, ab = ba^(n-1) As outras propriedades que definem D(2n) são provadas de forma análoga (por exemplo, o elemento neutro seria a rotação de um ângulo igual a zero, etc...) Qual a vantagem disso tudo? A associatividade em D(2n) passa a ser uma consequencia da associatividade da composição de funções. Espero que tenha ficado claro. Um abraço, Claudio. ----- Original Message ----- From: "Eduardo Botelho" To: Sent: Saturday, April 19, 2003 7:43 PM Subject: [obm-l] Dúvida sobre grupos diedrais > Olá pessoal da lista. > Estive lendo o Higino Domingues para meu curso de Álgebra e não > consigo acreditar no que diz o livro. Vejamos: ele me diz que um grupo > diedral, como o proprio nome já diz, é um grupo. > Mas vale a associativa? > Se ´a´ é tomado para especificar rotação e ' b' para refletir em torno > de um eixo, 'e' é tal que b^2 = e, e D(2n) = {e, > a,a^2,...,a^(n-1),b,...,ba^(n-1)} é o conjunto em questão, vale (a^2 a)b > = a^2(ab), por exemplo ? > Veja só a contradição(pelo menos pra mim) : > Ele mostra que ab = ba^(n-1). Jóia. > Aí dá o exemplo D6: (para ficar claro, vou vou associar cada vértice > do triângulo a um ângulo imaginário a partir de seu centro): > e: 1(0),2(120),3(240) > a: 1(120),2(240), 3(0) > a^2: 1(240),2(0),3(120) até aqui, jóia. > prosseguindo... > b: 1(0),2(240),3(120) > b^2: 1(240),2(120),3(0) > b^3: 1(120),2(0),3(240) ..ou seja, as rotações são horárias. Nesse > caso, não valeria ab = ba^(n-1) > > Qualquer ajuda agradeço > > Abraço > Eduardo > > > > > > ========================================================================= > Instruções para entrar na lista, sair da lista e usar a lista em > http://www.mat.puc-rio.br/~nicolau/olimp/obm-l.html > O administrador desta lista é > ========================================================================= ========================================================================= Instruções para entrar na lista, sair da lista e usar a lista em http://www.mat.puc-rio.br/~nicolau/olimp/obm-l.html ========================================================================= From owner-obm-l@sucuri.mat.puc-rio.br Tue Apr 22 17:17:09 2003 Return-Path: Received: (from majordom@localhost) by sucuri.mat.puc-rio.br (8.9.3/8.9.3) id RAA05295 for obm-l-MTTP; Tue, 22 Apr 2003 17:14:49 -0300 Received: from itaqui.terra.com.br (itaqui.terra.com.br [200.176.3.19]) by sucuri.mat.puc-rio.br (8.9.3/8.9.3) with ESMTP id RAA05291 for ; Tue, 22 Apr 2003 17:14:46 -0300 Received: from una.terra.com.br (una.terra.com.br [200.176.3.32]) by itaqui.terra.com.br (Postfix) with ESMTP id 7F88A3BC660 for ; Tue, 22 Apr 2003 17:14:14 -0300 (BRT) Received: from nt (RJ231083.user.veloxzone.com.br [200.165.231.83]) (authenticated user ensr) by una.terra.com.br (Postfix) with ESMTP id 0DEAE2F00D3 for ; Tue, 22 Apr 2003 17:14:14 -0300 (BRT) Message-ID: <027d01c3090b$a009e880$5400a8c0@ensrbr> From: "Luis Lopes" To: References: <20030421151630.66488.qmail@web13007.mail.yahoo.com> Subject: Re: [obm-l] 4 coisinhas Date: Tue, 22 Apr 2003 17:13:22 -0300 MIME-Version: 1.0 Content-Type: text/plain; charset="iso-8859-1" Content-Transfer-Encoding: 8bit X-Priority: 3 X-MSMail-Priority: Normal X-Mailer: Microsoft Outlook Express 5.50.4807.1700 X-MIMEOLE: Produced By Microsoft MimeOLE V5.50.4807.1700 Sender: owner-obm-l@sucuri.mat.puc-rio.br Precedence: bulk Reply-To: obm-l@mat.puc-rio.br Sauda,c~oes, > 3) > A area de um triangulo em funcao do comprimento das > medianas? Vi isso num livro em inglês mas acho que o livro do Edgard Alencar "Problemas de Geometria" também tem. S = 4\sqrt{m(m - m_a)(m - m_b)(m - m_c)} / 3 onde m = (m_a + m_b + m_c) / 2. > 4) > Calcular S = 1 + 8 + 27 + ... + x^3 Este o Igor deverá saber fazer. Fala, Igor! Copiando e colando a msg do Morgado. Uma conta boba transforma x^3 = x(x-1)(x-2) + 3x(x-1) + x [x^3 = ax(x-1)(x-2) + bx(x-1) + cx + d, obriga os polinomios a serem identicos e pronto] Podemos calcular os coeficientes da Potencia fatorial da seguinte maneira: p_n(x) = a_0 + a_1x + .... + a_n x^n p_n(x) = r_0 + r_1(x)^(1) + ... + r_n(x)^(n) onde os r_i, i=0,1,2,....n são os restos das seguintes divisões: p_n(x) = r_0 + xq_0(x) q_0(x) = r_1 + (x-1)q_1(x) q_1(x) = r_2 + (x-2)q_2(x) ..... q_{n-1}(x) = r_n Se p_n(x) = x^3, resulta: x^3 = 0 + x x^2 x^2 = 1 + (x-1)(x+1) x+1 = 3 + (x-2)(1) x^3 = 0 + 1(x)^(1) + 3(x)^(2) + 1(x)^(3) = 0 + x + 3x(x-1) + x(x-1)(x-2) = x(x-1)(x-2) + 3x(x-1) + x []'s Luís -----Mensagem Original----- De: "Helder Suzuki" Para: Enviada em: segunda-feira, 21 de abril de 2003 12:16 Assunto: [obm-l] 4 coisinhas > 3) > A area de um triangulo em funcao do comprimento das > medianas? > > 4) > Calcular S = 1 + 8 + 27 + ... + x^3 > > Abraços, > Helder Toshiro Suzuki > ========================================================================= Instruções para entrar na lista, sair da lista e usar a lista em http://www.mat.puc-rio.br/~nicolau/olimp/obm-l.html ========================================================================= From owner-obm-l@sucuri.mat.puc-rio.br Tue Apr 22 17:18:42 2003 Return-Path: Received: (from majordom@localhost) by sucuri.mat.puc-rio.br (8.9.3/8.9.3) id RAA05418 for obm-l-MTTP; Tue, 22 Apr 2003 17:17:20 -0300 Received: from smtp012.mail.yahoo.com (smtp012.mail.yahoo.com [216.136.173.32]) by sucuri.mat.puc-rio.br (8.9.3/8.9.3) with SMTP id RAA05413 for ; Tue, 22 Apr 2003 17:17:16 -0300 Received: from 200-180-182-081.paemt7002.dsl.brasiltelecom.net.br (HELO servidor) (marcus?math@200.180.182.81 with login) by smtp.mail.vip.sc5.yahoo.com with SMTP; 22 Apr 2003 20:16:39 -0000 Message-ID: <002101c3090c$19fd3cf0$6fe9fea9@servidor> From: "Marcus Alexandre Nunes" To: References: Subject: [obm-l] =?iso-8859-1?Q?Re:_=5Bobm-l=5D_Re:_=5Bobm-l=5D_Parece_f=E1cil...m?= =?iso-8859-1?Q?as=2C_n=E3o_consigo.?= Date: Tue, 22 Apr 2003 17:16:40 -0300 MIME-Version: 1.0 Content-Type: multipart/alternative; boundary="----=_NextPart_000_001C_01C308F2.F03467E0" X-Priority: 3 X-MSMail-Priority: Normal X-Mailer: Microsoft Outlook Express 6.00.2800.1106 X-MimeOLE: Produced By Microsoft MimeOLE V6.00.2800.1106 Sender: owner-obm-l@sucuri.mat.puc-rio.br Precedence: bulk Reply-To: obm-l@mat.puc-rio.br This is a multi-part message in MIME format. ------=_NextPart_000_001C_01C308F2.F03467E0 Content-Type: text/plain; charset="iso-8859-1" Content-Transfer-Encoding: quoted-printable Eu achei as respostas dadas para este problema um tanto complicadas, = ent=E3o estou enviando a minha: m, n =EDmpares =3D> 8|(m^4 + n^4 -2) Como m, n s=E3o =EDmpares, podemos escrever m =3D 2k + 1, k pertence aos inteiros; n =3D 2l + 1, l pertence aos inteiros. m^4 =3D (2k+1)^4 =3D 16k^4 + 32k^3 + 24k^2 +8k + 1 n^4 =3D (2l+1)^4 =3D 16l^4 + 32l^3 + 24l^2 +8l + 1 Ent=E3o temos 8|[ (16k^4 + 32k^3 + 24k^2 +8k + 1) + (16l^4 + 32l^3 + 24l^2 +8l + 1) - = 2] 8|[ 16k^4 + 32k^3 + 24k^2 + 8k + 16l^4 + 32l^3 + 24l^2 + 8l ] Colocamos 8 em evid=EAncia dentro dos colchetes e ficamos com 8|8[ 2k^4 + 4k^3 + 3k^2 + k + 2l^4 + 4l^3 + 3l^2 + l ] O que prova que 8|(m^4 + n^4 -2), se m, n s=E3o =EDmpares. ---------------------------------------------- Marcus Alexandre Nunes marcus_math@yahoo.com.br UIN 114153703 ------=_NextPart_000_001C_01C308F2.F03467E0 Content-Type: text/html; charset="iso-8859-1" Content-Transfer-Encoding: quoted-printable
Eu achei as respostas dadas para este = problema um=20 tanto complicadas, ent=E3o estou enviando a minha:
 
m, n =EDmpares =3D> 8|(m^4 + n^4 = -2)
 
Como m, n s=E3o =EDmpares, podemos=20 escrever
 
m =3D 2k + 1, k pertence aos = inteiros;
n =3D 2l + 1, l pertence aos = inteiros.
 
m^4 =3D (2k+1)^4 =3D 16k^4 + 32k^3 + = 24k^2 +8k +=20 1
n^4 =3D (2l+1)^4 =3D 16l^4 + 32l^3 + = 24l^2 +8l +=20 1
 
Ent=E3o temos
 
8|[ (16k^4 + 32k^3 + 24k^2 +8k + 1) + = (16l^4 +=20 32l^3 + 24l^2 +8l + 1) - 2]
 
8|[ 16k^4 + 32k^3 + 24k^2 + 8k + = 16l^4 + 32l^3 +=20 24l^2 + 8l ]
 
Colocamos 8 em evid=EAncia dentro dos colchetes e ficamos com
 
8|8[ 2k^4 + 4k^3 + 3k^2 + k + 2l^4 + 4l^3 + 3l^2 + l ]
 
O que prova que 8|(m^4 + n^4 -2), se m, n s=E3o =EDmpares.
 
----------------------------------------------
Marcus Alexandre=20 Nunes
marcus_math@yahoo.com.br
= UIN=20 114153703
------=_NextPart_000_001C_01C308F2.F03467E0-- ========================================================================= Instruções para entrar na lista, sair da lista e usar a lista em http://www.mat.puc-rio.br/~nicolau/olimp/obm-l.html ========================================================================= From owner-obm-l@sucuri.mat.puc-rio.br Tue Apr 22 17:45:45 2003 Return-Path: Received: (from majordom@localhost) by sucuri.mat.puc-rio.br (8.9.3/8.9.3) id RAA06766 for obm-l-MTTP; Tue, 22 Apr 2003 17:43:34 -0300 Received: from itaqui.terra.com.br (itaqui.terra.com.br [200.176.3.19]) by sucuri.mat.puc-rio.br (8.9.3/8.9.3) with ESMTP id RAA06762 for ; Tue, 22 Apr 2003 17:43:30 -0300 Received: from itaim.terra.com.br (itaim.terra.com.br [200.176.3.76]) by itaqui.terra.com.br (Postfix) with ESMTP id 504E93BC816 for ; Tue, 22 Apr 2003 17:42:59 -0300 (BRT) Received: from nt (RJ231083.user.veloxzone.com.br [200.165.231.83]) (authenticated user ensr) by itaim.terra.com.br (Postfix) with ESMTP id F051A2E0090 for ; Tue, 22 Apr 2003 17:42:54 -0300 (BRT) Message-ID: <029001c3090f$a329f060$5400a8c0@ensrbr> From: "Luis Lopes" To: References: <6732305773.20030411013805@gmx.net> <00ba01c3005e$56faaae0$3300c57d@bovespa.com> <012a01c3006b$dcae0a80$5400a8c0@ensrbr> <00ed01c30514$eb7c9100$5400a8c0@ensrbr> <01b701c30902$946f1bc0$3300c57d@bovespa.com> Subject: [obm-l] =?iso-8859-1?Q?Re:_=5Bobm-l=5D_Re:_=5Bobm-l=5D_Dois_problemas_do_mesmo_si?= =?iso-8859-1?Q?te_=5Bera:Trigonometria_e_Sequ=EAncias=5D?= Date: Tue, 22 Apr 2003 17:41:56 -0300 MIME-Version: 1.0 Content-Type: text/plain; charset="iso-8859-1" Content-Transfer-Encoding: 8bit X-Priority: 3 X-MSMail-Priority: Normal X-Mailer: Microsoft Outlook Express 5.50.4807.1700 X-MIMEOLE: Produced By Microsoft MimeOLE V5.50.4807.1700 Sender: owner-obm-l@sucuri.mat.puc-rio.br Precedence: bulk Reply-To: obm-l@mat.puc-rio.br Sauda,c~oes, Oi Claudio, Valeu! Não sabia que estes problemas eram tão elaborados, principalmente o segundo. []'s Luís > > Proponho novamente dois problemas, mas desta > > vez não tenho a solução de nenhum dos dois: > > > > Com uma notação resumida, > > > > 1) mostre que existe n tal que 2^n = 2002....... > > > > 2) 10000! = ..........n0000000000...000. Calcule n. > > > > Obs.: o número de zeros em n0000000000...000 > > eh um problema conhecido. > > > > []'s > > Luís > > > Oi, Luis: > > O no. 1 é consequencia do fato de que log(2) na base 10 é irracional e, > portanto, existem inteiros m e n tais que: > log(2002) + n < m*log(2) < log(2003) + n. > Eu apresentei esse problema há alguns meses ao site Ask Dr. Math e um dos > matemáticos de lá me deu a dica crucial. A discussão está aqui: > http://mathforum.org/library/drmath/view/61545.html > > O no. 2 é menos conhecido do que o problema sobre o número de zeros no final > de 10.000! mas, mesmo assim, é um belo problema de teoria dos números. Uma > solução está aqui: > http://www.sci.csuhayward.edu/mathcs/pom/pom95.html > > Um abraço, > Claudio. > ========================================================================= Instruções para entrar na lista, sair da lista e usar a lista em http://www.mat.puc-rio.br/~nicolau/olimp/obm-l.html ========================================================================= From owner-obm-l@sucuri.mat.puc-rio.br Tue Apr 22 17:59:51 2003 Return-Path: Received: (from majordom@localhost) by sucuri.mat.puc-rio.br (8.9.3/8.9.3) id RAA07365 for obm-l-MTTP; Tue, 22 Apr 2003 17:58:15 -0300 Received: from cmsrelay01.mx.net (cmsrelay01.mx.net [165.212.11.110]) by sucuri.mat.puc-rio.br (8.9.3/8.9.3) with SMTP id RAA07359 for ; Tue, 22 Apr 2003 17:58:11 -0300 Received: from uadvg131.cms.usa.net (HELO localhost) (165.212.11.131) by cmsoutbound.mx.net with SMTP; 22 Apr 2003 20:57:33 -0000 Received: from smtp.postoffice.net [165.212.8.17] by uadvg131.cms.usa.net (ASMTP/) via mtad (C8.MAIN.2.05) with ESMTP id 441HDVu6E0090M31; Tue, 22 Apr 2003 20:57:31 GMT Received: from 200.181.4.100 [200.181.4.100] by uwdvg017.cms.usa.net (USANET web-mailer CM.0402.5.2B); Tue, 22 Apr 2003 20:57:30 -0000 Date: Tue, 22 Apr 2003 17:57:30 -0300 From: Artur Costa Steiner To: Subject: Re: [Re: [obm-l] 4 coisinhas] X-Mailer: USANET web-mailer (CM.0402.5.2B) Mime-Version: 1.0 Message-ID: <672HDVu6E1552S17.1051045050@uwdvg017.cms.usa.net> Content-Type: text/plain; charset=ISO-8859-1 Content-Transfer-Encoding: 8bit X-MIME-Autoconverted: from quoted-printable to 8bit by sucuri.mat.puc-rio.br id RAA07360 Sender: owner-obm-l@sucuri.mat.puc-rio.br Precedence: bulk Reply-To: obm-l@mat.puc-rio.br > > > 4) > > Calcular S = 1 + 8 + 27 + ... + x^3 > Este o Igor deverá saber fazer. Fala, Igor! > Um fato interessante e razoavelmente bem conhecido (talvez nao tanto) eh que a soma dos cubos dos n primeiros numeros naturais eh o quadrado das soma destes numeros. Isto pode ser provado utilizando aquele processo classico (um tanto bracal e cansativo), por recorrencia, ou por inducao finita - neste ultimo caso, supondo, eh claro, que vc ja desconfia do resultado. Logo, 1^3 + 2^3 ....+ n^3 = [n(n+1)/2]^2. A obtencao da soma dos n primeiros naturais eh trivial, por P. Aritmetica PS. Por forca do habito, usei n e nao x para representar um natural... Artur ========================================================================= Instruções para entrar na lista, sair da lista e usar a lista em http://www.mat.puc-rio.br/~nicolau/olimp/obm-l.html ========================================================================= From owner-obm-l@sucuri.mat.puc-rio.br Tue Apr 22 18:05:37 2003 Return-Path: Received: (from majordom@localhost) by sucuri.mat.puc-rio.br (8.9.3/8.9.3) id SAA07896 for obm-l-MTTP; Tue, 22 Apr 2003 18:04:20 -0300 Received: from cmsrelay02.mx.net (cmsrelay02.mx.net [165.212.11.111]) by sucuri.mat.puc-rio.br (8.9.3/8.9.3) with SMTP id SAA07892 for ; Tue, 22 Apr 2003 18:04:15 -0300 Received: from uadvg130.cms.usa.net (HELO localhost) (165.212.11.130) by cmsoutbound.mx.net with SMTP; 22 Apr 2003 21:03:44 -0000 Received: from smtp.postoffice.net [165.212.8.23] by uadvg130.cms.usa.net (ASMTP/) via mtad (C8.MAIN.2.05) with ESMTP id 679HDVVDp0295M30; Tue, 22 Apr 2003 21:03:41 GMT Received: from 200.181.4.100 [200.181.4.100] by cmsweb06.cms.usa.net (USANET web-mailer CM.0402.5.2B); Tue, 22 Apr 2003 21:03:40 -0000 Date: Tue, 22 Apr 2003 18:03:40 -0300 From: Artur Costa Steiner To: Subject: [obm-l] =?ISO-8859-1?Q?Re=3A=20=5B=5Bobm=2Dl=5D=20Re=3A=20=5Bobm=2Dl=5D=20?= =?ISO-8859-1?Q?Re=3A=20=5Bobm=2Dl=5D=20Parece=20f=E1cil?= =?ISO-8859-1?Q?=2E=2E=2Emas=2C=20n=E3o=20consigo=2E=5D?= X-Mailer: USANET web-mailer (CM.0402.5.2B) Mime-Version: 1.0 Message-ID: <659HDVVDO6400S06.1051045420@cmsweb06.cms.usa.net> Content-Type: text/plain; charset=ISO-8859-1 Content-Transfer-Encoding: 8bit X-MIME-Autoconverted: from quoted-printable to 8bit by sucuri.mat.puc-rio.br id SAA07893 Sender: owner-obm-l@sucuri.mat.puc-rio.br Precedence: bulk Reply-To: obm-l@mat.puc-rio.br "Marcus Alexandre Nunes" wrote: > --------------------------------------------- > --------------------------------------------- > Eu achei as respostas dadas para este problema um tanto complicadas, então estou enviando a minha: Acho que todas elas, inclusive a sua, estao no mesmo "nivel de complicacao", exceto as basedas em congruencias. Mas estas jah assumem que se sabe que, se m eh impar, entao m^2 = 1 (mod 8), cuja demonstracao acaba caindo no fato de que m = 2k+1, etc... Artur ========================================================================= Instruções para entrar na lista, sair da lista e usar a lista em http://www.mat.puc-rio.br/~nicolau/olimp/obm-l.html ========================================================================= From owner-obm-l@sucuri.mat.puc-rio.br Tue Apr 22 18:28:06 2003 Return-Path: Received: (from majordom@localhost) by sucuri.mat.puc-rio.br (8.9.3/8.9.3) id SAA09069 for obm-l-MTTP; Tue, 22 Apr 2003 18:25:02 -0300 Received: from itaqui.terra.com.br (itaqui.terra.com.br [200.176.3.19]) by sucuri.mat.puc-rio.br (8.9.3/8.9.3) with ESMTP id SAA09058 for ; Tue, 22 Apr 2003 18:24:57 -0300 Received: from araci.terra.com.br (araci.terra.com.br [200.176.3.44]) by itaqui.terra.com.br (Postfix) with ESMTP id DF9C43BC2CC for ; Tue, 22 Apr 2003 18:24:26 -0300 (BRT) Received: from usuario (200-203-035-087.paemt7005.dsl.brasiltelecom.net.br [200.203.35.87]) (authenticated user marioappereira) by araci.terra.com.br (Postfix) with ESMTP id 9FF2621EF67 for ; Tue, 22 Apr 2003 18:24:26 -0300 (BRT) Message-ID: <004101c30915$959b6c20$0301a8c0@usuario> From: =?iso-8859-1?Q?M=E1rio_Pereira?= To: Subject: [obm-l] euclidiana Date: Tue, 22 Apr 2003 18:24:39 -0300 MIME-Version: 1.0 Content-Type: multipart/alternative; boundary="----=_NextPart_000_003E_01C308FC.6FC3CE70" X-Priority: 3 X-MSMail-Priority: Normal X-Mailer: Microsoft Outlook Express 6.00.2600.0000 X-MimeOLE: Produced By Microsoft MimeOLE V6.00.2600.0000 Sender: owner-obm-l@sucuri.mat.puc-rio.br Precedence: bulk Reply-To: obm-l@mat.puc-rio.br This is a multi-part message in MIME format. ------=_NextPart_000_003E_01C308FC.6FC3CE70 Content-Type: text/plain; charset="iso-8859-1" Content-Transfer-Encoding: quoted-printable Como podemos verificar que um sistema de unidades de medidas =E9, de = fato, um sistema? Obrigado,=20 M=E1rio.=20 ------=_NextPart_000_003E_01C308FC.6FC3CE70 Content-Type: text/html; charset="iso-8859-1" Content-Transfer-Encoding: quoted-printable
 
Como podemos verificar que um sistema = de unidades=20 de medidas =E9, de fato, um sistema?
 
Obrigado,
 
M=E1rio.
------=_NextPart_000_003E_01C308FC.6FC3CE70-- ========================================================================= Instruções para entrar na lista, sair da lista e usar a lista em http://www.mat.puc-rio.br/~nicolau/olimp/obm-l.html ========================================================================= From owner-obm-l@sucuri.mat.puc-rio.br Tue Apr 22 18:41:54 2003 Return-Path: Received: (from majordom@localhost) by sucuri.mat.puc-rio.br (8.9.3/8.9.3) id SAA09882 for obm-l-MTTP; Tue, 22 Apr 2003 18:38:57 -0300 Received: from paiol.terra.com.br (paiol.terra.com.br [200.176.3.18]) by sucuri.mat.puc-rio.br (8.9.3/8.9.3) with ESMTP id SAA09878 for ; Tue, 22 Apr 2003 18:38:52 -0300 Received: from marova.terra.com.br (marova.terra.com.br [200.176.3.39]) by paiol.terra.com.br (Postfix) with ESMTP id D20F487F06 for ; Tue, 22 Apr 2003 18:38:20 -0300 (BRT) Received: from nt (RJ231083.user.veloxzone.com.br [200.165.231.83]) (authenticated user ensr) by marova.terra.com.br (Postfix) with ESMTP id 878113DC075 for ; Tue, 22 Apr 2003 18:38:20 -0300 (BRT) Message-ID: <030301c30917$5fb98f40$5400a8c0@ensrbr> From: "Luis Lopes" To: References: <672HDVu6E1552S17.1051045050@uwdvg017.cms.usa.net> Subject: Re: [Re: [obm-l] 4 coisinhas] Date: Tue, 22 Apr 2003 18:37:28 -0300 MIME-Version: 1.0 Content-Type: text/plain; charset="iso-8859-1" Content-Transfer-Encoding: 8bit X-Priority: 3 X-MSMail-Priority: Normal X-Mailer: Microsoft Outlook Express 5.50.4807.1700 X-MIMEOLE: Produced By Microsoft MimeOLE V5.50.4807.1700 Sender: owner-obm-l@sucuri.mat.puc-rio.br Precedence: bulk Reply-To: obm-l@mat.puc-rio.br Sauda,c~oes, O que sabemos é que [S_n^(1)]^2 = S_n^(3). E o que vc diria de 3[S_n^(2)]^2 = S_n^(3) +2S_n^(5) 2[S_n^(3)]^2 = S_n^(5) +S_n^(7) onde S_n^(k) = 1^k + 2^k + .... + n^k ? Estas identidades foram tiradas de um livro antigo, autor Rio Nogueira. Título? Hum, Lições de Combinatória. []'s Luís -----Mensagem Original----- De: "Artur Costa Steiner" Para: Enviada em: terça-feira, 22 de abril de 2003 17:57 Assunto: Re: [Re: [obm-l] 4 coisinhas] > > > > > 4) > > > Calcular S = 1 + 8 + 27 + ... + x^3 > > Este o Igor deverá saber fazer. Fala, Igor! > > > Um fato interessante e razoavelmente bem conhecido (talvez nao tanto) eh que a > soma dos cubos dos n primeiros numeros naturais eh o quadrado das soma destes > numeros. Isto pode ser provado utilizando aquele processo classico (um tanto > bracal e cansativo), por recorrencia, ou por inducao finita - neste ultimo > caso, supondo, eh claro, que vc ja desconfia do resultado. Logo, 1^3 + 2^3 > ....+ n^3 = [n(n+1)/2]^2. A obtencao da soma dos n primeiros naturais eh > trivial, por P. Aritmetica > > PS. Por forca do habito, usei n e nao x para representar um natural... > Artur > ========================================================================= Instruções para entrar na lista, sair da lista e usar a lista em http://www.mat.puc-rio.br/~nicolau/olimp/obm-l.html ========================================================================= From owner-obm-l@sucuri.mat.puc-rio.br Tue Apr 22 19:02:29 2003 Return-Path: Received: (from majordom@localhost) by sucuri.mat.puc-rio.br (8.9.3/8.9.3) id SAA10736 for obm-l-MTTP; Tue, 22 Apr 2003 18:59:38 -0300 Received: from web12906.mail.yahoo.com (web12906.mail.yahoo.com [216.136.174.73]) by sucuri.mat.puc-rio.br (8.9.3/8.9.3) with SMTP id SAA10732 for ; Tue, 22 Apr 2003 18:59:34 -0300 Message-ID: <20030422215902.78545.qmail@web12906.mail.yahoo.com> Received: from [200.144.43.151] by web12906.mail.yahoo.com via HTTP; Tue, 22 Apr 2003 18:59:02 ART Date: Tue, 22 Apr 2003 18:59:02 -0300 (ART) From: =?iso-8859-1?q?Johann=20Peter=20Gustav=20Lejeune=20Dirichlet?= Subject: Re: [[obm-l] Problema proposto Eureka 15] To: obm-l@mat.puc-rio.br In-Reply-To: <839HDVswt6304S10.1051035765@cmsweb10.cms.usa.net> MIME-Version: 1.0 Content-Type: multipart/alternative; boundary="0-1555627697-1051048742=:77930" Content-Transfer-Encoding: 8bit Sender: owner-obm-l@sucuri.mat.puc-rio.br Precedence: bulk Reply-To: obm-l@mat.puc-rio.br --0-1555627697-1051048742=:77930 Content-Type: text/plain; charset=iso-8859-1 Content-Transfer-Encoding: 8bit Bem,prostaferizar e aquele treco de soma e produto. Artur Costa Steiner wrote:Melhoras para vc! Eu realmente nao sei o que eh prostaferese! O que vc fez, acho que esta OK Um abraco Artur > --------------------------------------------- > Acho que depois de muita viajem e depressao por ter passado dias entrevado na cama de casa com gripe,consegui me restabelecer e ter uma ideia pra esse da Eureka!Ache todas as funçoes de R em R tais que f(x+y)+f(x-y)=2f(x)*cos y para todos os reais x e y Bem,revertendo:f(y+x)+f(y-x)=2f(y)*cos x.Soma as duas: > 2f(x+y)+f(x-y)+f(y-x)=2(f(x)*cos y+f(y)*cos x).Mas f(x-y)+f(y-x)=2f(0)*cos(x-y)(faz x<-0 e y<-(x-y) e confere na primeira)Logo f(x+y)+f(0)*cos(x-y)=f(x)*cos y+f(y)*cos x.Pondo f(0)=z (z de zero,oras!!!)f(2a)+z*cos 2b=f(a+b)*cos(a-b)+f(a-b)*cos(a+b) por substituiçao direta.Assim da pra zuar mais:f(2a)+z=2f(a)*cos af(pi)+z=0 Com isso da pra prosseguir.Parece prostaferese,nao? > > > --------------------------------- > Yahoo! Mail > O melhor e-mail gratuito da internet: 6MB de espaço, antivírus, acesso POP3, filtro contra spam. ========================================================================= Instruções para entrar na lista, sair da lista e usar a lista em http://www.mat.puc-rio.br/~nicolau/olimp/obm-l.html ========================================================================= --------------------------------- Yahoo! Mail O melhor e-mail gratuito da internet: 6MB de espaço, antivírus, acesso POP3, filtro contra spam. --0-1555627697-1051048742=:77930 Content-Type: text/html; charset=iso-8859-1 Content-Transfer-Encoding: 8bit

Bem,prostaferizar e aquele treco de soma e produto.

 Artur Costa Steiner <artur_steiner@usa.net> wrote:

Melhoras para vc!
Eu realmente nao sei o que eh prostaferese!
O que vc fez, acho que esta OK
Um abraco
Artur

> ---------------------------------------------
> Acho que depois de muita viajem e depressao por ter passado dias entrevado
na cama de casa com gripe,consegui me restabelecer e ter uma ideia pra esse da
Eureka!Ache todas as funçoes de R em R tais que f(x+y)+f(x-y)=2f(x)*cos y
para todos os reais x e y Bem,revertendo:f(y+x)+f(y-x)=2f(y)*cos x.Soma as
duas:
> 2f(x+y)+f(x-y)+f(y-x)=2(f(x)*cos y+f(y)*cos x).Mas
f(x-y)+f(y-x)=2f(0)*cos(x-y)(faz x<-0 e y<-(x-y) e confere na primeira)Logo
f(x+y)+f(0)*cos(x-y)=f(x)*cos y+f(y)*cos x.Pondo f(0)=z (z de
zero,oras!!!)f(2a)+z*cos 2b=f(a+b)*cos(a-b)+f(a-b)*cos(a+b) por substituiçao
direta.Assim da pra zuar mais:f(2a)+z=2f(a)*cos af(pi)+z=0 Com isso da pra
prosseguir.Parece prostaferese,nao?
>
>
> ---------------------------------
> Yahoo! Mail
> O melhor e-mail gratuito da internet: 6MB de espaço, antivírus, acesso
POP3, filtro contra spam.


=========================================================================
Instruções para entrar na lista, sair da lista e usar a lista em
http://www.mat.puc-rio.br/~nicolau/olimp/obm-l.html
=========================================================================



Yahoo! Mail
O melhor e-mail gratuito da internet: 6MB de espaço, antivírus, acesso POP3, filtro contra spam. --0-1555627697-1051048742=:77930-- ========================================================================= Instruções para entrar na lista, sair da lista e usar a lista em http://www.mat.puc-rio.br/~nicolau/olimp/obm-l.html ========================================================================= From owner-obm-l@sucuri.mat.puc-rio.br Tue Apr 22 19:17:52 2003 Return-Path: Received: (from majordom@localhost) by sucuri.mat.puc-rio.br (8.9.3/8.9.3) id TAA11460 for obm-l-MTTP; Tue, 22 Apr 2003 19:15:15 -0300 Received: from web13007.mail.yahoo.com (web13007.mail.yahoo.com [216.136.174.17]) by sucuri.mat.puc-rio.br (8.9.3/8.9.3) with SMTP id TAA11456 for ; Tue, 22 Apr 2003 19:15:11 -0300 Message-ID: <20030422221439.11707.qmail@web13007.mail.yahoo.com> Received: from [200.148.192.169] by web13007.mail.yahoo.com via HTTP; Tue, 22 Apr 2003 19:14:39 ART Date: Tue, 22 Apr 2003 19:14:39 -0300 (ART) From: =?iso-8859-1?q?Helder=20Suzuki?= Subject: [obm-l] Mais uma coisinha To: obm-l@mat.puc-rio.br MIME-Version: 1.0 Content-Type: text/plain; charset=iso-8859-1 Content-Transfer-Encoding: 8bit Sender: owner-obm-l@sucuri.mat.puc-rio.br Precedence: bulk Reply-To: obm-l@mat.puc-rio.br Olá! Bom, primeiro eu agradeço a todos que responderam ao "4 coisinhas", embora o problema do jogo não foi lá bem muito resolvido e eu não sei aonde achar a tal revista hehehe :P 1) João e Maria brincam de multiplicar. Antes de começar o jogo, eles escolhem um natural n>1. João começa com o número 1 e multiplica-o por um natural do intervalo [2, 9], Maria pega o resultado faz o mesmo, e assim sucessivamente. Ganha o jogo o primeiro que conseguir fazer o resultado >= n. Se João e Maria jogam perfeitamente, e João sempre começa o jogo, para quais valores de n João ganha? e Maria? []'s, Helder Toshiro Suzuki _______________________________________________________________________ Yahoo! Mail O melhor e-mail gratuito da internet: 6MB de espaço, antivírus, acesso POP3, filtro contra spam. http://br.mail.yahoo.com/ ========================================================================= Instruções para entrar na lista, sair da lista e usar a lista em http://www.mat.puc-rio.br/~nicolau/olimp/obm-l.html ========================================================================= From owner-obm-l@sucuri.mat.puc-rio.br Tue Apr 22 19:30:48 2003 Return-Path: Received: (from majordom@localhost) by sucuri.mat.puc-rio.br (8.9.3/8.9.3) id TAA12018 for obm-l-MTTP; Tue, 22 Apr 2003 19:27:33 -0300 Received: from web12906.mail.yahoo.com (web12906.mail.yahoo.com [216.136.174.73]) by sucuri.mat.puc-rio.br (8.9.3/8.9.3) with SMTP id TAA12013 for ; Tue, 22 Apr 2003 19:27:28 -0300 Message-ID: <20030422222657.84669.qmail@web12906.mail.yahoo.com> Received: from [200.144.43.151] by web12906.mail.yahoo.com via HTTP; Tue, 22 Apr 2003 19:26:57 ART Date: Tue, 22 Apr 2003 19:26:57 -0300 (ART) From: =?iso-8859-1?q?Johann=20Peter=20Gustav=20Lejeune=20Dirichlet?= Subject: [obm-l] Abertos da Eureka!-68,69,71,73 To: obm-l@mat.puc-rio.br MIME-Version: 1.0 Content-Type: multipart/alternative; boundary="0-1992702694-1051050417=:83718" Content-Transfer-Encoding: 8bit Sender: owner-obm-l@sucuri.mat.puc-rio.br Precedence: bulk Reply-To: obm-l@mat.puc-rio.br --0-1992702694-1051050417=:83718 Content-Type: text/plain; charset=iso-8859-1 Content-Transfer-Encoding: 8bit Ai turma!!!!A Eureka 14 foi uma das mais menos respondidas da face da Terra!!!!!!Vou por na lista ate que alguem possa resolver: 68)Seja ABC um triângulo de lados inteiros e área racional.Prove que existem pontos X,Y,Z de cooordenadas inteiras de R² tais que XYZ e ABC sao congruentes. 69)Se os naturais a e b sao tais que a^n-1 divide b^n-1 para todo n natural,mostre que b e potencia perfeita de a. Comments:o Gugu ja resolveu esse na Semana mas perdi a soluçao.Mas assim sendo eu me lembrava de que era algo como tecnicas de n suficientemente grande... 71)Considere tres circunferencias tangentes duas a duas.Mostre que so ha duas circunferencias tangentes simultaneamente as tres,e construa-as. Comments:Trivial!!! 73(Torneio das Cidades)Mostre que para cada inteiro positivo n existe uma PA crescente de n inteiros positivos cujas somas dos digitos formam uma PA crescente,mas o mesmo nao e valido se a sequencia e infinita. --------------------------------- Yahoo! Mail O melhor e-mail gratuito da internet: 6MB de espaço, antivírus, acesso POP3, filtro contra spam. --0-1992702694-1051050417=:83718 Content-Type: text/html; charset=iso-8859-1 Content-Transfer-Encoding: 8bit
Ai turma!!!!A Eureka 14 foi uma das mais menos respondidas da face da Terra!!!!!!Vou por na lista ate que alguem possa resolver:
 
68)Seja ABC um triângulo de lados inteiros e área racional.Prove que existem pontos X,Y,Z de cooordenadas inteiras de R² tais que XYZ e ABC sao congruentes.
 
69)Se os naturais a e b sao tais que a^n-1 divide b^n-1 para todo n natural,mostre que b e potencia perfeita de a.
 
Comments:o Gugu ja resolveu esse na Semana mas perdi a soluçao.Mas assim sendo eu me lembrava de que era algo como tecnicas de n suficientemente grande...
 
71)Considere tres circunferencias tangentes duas a duas.Mostre que so ha duas circunferencias tangentes simultaneamente as tres,e construa-as.
 
Comments:Trivial!!!
 
73(Torneio das Cidades)Mostre que para cada inteiro positivo n existe uma PA crescente de n inteiros positivos cujas somas dos digitos formam uma PA crescente,mas o mesmo nao e valido se a sequencia e infinita.



Yahoo! Mail
O melhor e-mail gratuito da internet: 6MB de espaço, antivírus, acesso POP3, filtro contra spam. --0-1992702694-1051050417=:83718-- ========================================================================= Instruções para entrar na lista, sair da lista e usar a lista em http://www.mat.puc-rio.br/~nicolau/olimp/obm-l.html ========================================================================= From owner-obm-l@sucuri.mat.puc-rio.br Tue Apr 22 19:36:55 2003 Return-Path: Received: (from majordom@localhost) by sucuri.mat.puc-rio.br (8.9.3/8.9.3) id TAA12309 for obm-l-MTTP; Tue, 22 Apr 2003 19:34:21 -0300 Received: from web12908.mail.yahoo.com (web12908.mail.yahoo.com [216.136.174.75]) by sucuri.mat.puc-rio.br (8.9.3/8.9.3) with SMTP id TAA12303 for ; Tue, 22 Apr 2003 19:34:16 -0300 Message-ID: <20030422223345.30175.qmail@web12908.mail.yahoo.com> Received: from [200.144.43.151] by web12908.mail.yahoo.com via HTTP; Tue, 22 Apr 2003 19:33:45 ART Date: Tue, 22 Apr 2003 19:33:45 -0300 (ART) From: =?iso-8859-1?q?Johann=20Peter=20Gustav=20Lejeune=20Dirichlet?= Subject: [obm-l] Polinomialmente assim... To: obm-l@mat.puc-rio.br MIME-Version: 1.0 Content-Type: multipart/alternative; boundary="0-299959220-1051050825=:29140" Content-Transfer-Encoding: 8bit Sender: owner-obm-l@sucuri.mat.puc-rio.br Precedence: bulk Reply-To: obm-l@mat.puc-rio.br --0-299959220-1051050825=:29140 Content-Type: text/plain; charset=iso-8859-1 Content-Transfer-Encoding: 8bit Oi turma,estou as voltas com esse aqui,vejam:Qual o produto e a soma das raizes POSITIVAS de (x^2+2x-12)^2=9x^3-108x ? --------------------------------- Yahoo! Mail O melhor e-mail gratuito da internet: 6MB de espaço, antivírus, acesso POP3, filtro contra spam. --0-299959220-1051050825=:29140 Content-Type: text/html; charset=iso-8859-1 Content-Transfer-Encoding: 8bit
Oi turma,estou as voltas com esse aqui,vejam:
Qual o produto e a soma das raizes POSITIVAS de (x^2+2x-12)^2=9x^3-108x ?



Yahoo! Mail
O melhor e-mail gratuito da internet: 6MB de espaço, antivírus, acesso POP3, filtro contra spam. --0-299959220-1051050825=:29140-- ========================================================================= Instruções para entrar na lista, sair da lista e usar a lista em http://www.mat.puc-rio.br/~nicolau/olimp/obm-l.html ========================================================================= From owner-obm-l@sucuri.mat.puc-rio.br Tue Apr 22 19:46:13 2003 Return-Path: Received: (from majordom@localhost) by sucuri.mat.puc-rio.br (8.9.3/8.9.3) id TAA12734 for obm-l-MTTP; Tue, 22 Apr 2003 19:43:32 -0300 Received: from web12902.mail.yahoo.com (web12902.mail.yahoo.com [216.136.174.69]) by sucuri.mat.puc-rio.br (8.9.3/8.9.3) with SMTP id TAA12729 for ; Tue, 22 Apr 2003 19:43:27 -0300 Message-ID: <20030422224256.27938.qmail@web12902.mail.yahoo.com> Received: from [200.144.43.151] by web12902.mail.yahoo.com via HTTP; Tue, 22 Apr 2003 19:42:56 ART Date: Tue, 22 Apr 2003 19:42:56 -0300 (ART) From: =?iso-8859-1?q?Johann=20Peter=20Gustav=20Lejeune=20Dirichlet?= Subject: Re: [Re: [obm-l] 4 coisinhas] To: obm-l@mat.puc-rio.br In-Reply-To: <030301c30917$5fb98f40$5400a8c0@ensrbr> MIME-Version: 1.0 Content-Type: multipart/alternative; boundary="0-1934890832-1051051376=:25849" Content-Transfer-Encoding: 8bit Sender: owner-obm-l@sucuri.mat.puc-rio.br Precedence: bulk Reply-To: obm-l@mat.puc-rio.br --0-1934890832-1051051376=:25849 Content-Type: text/plain; charset=iso-8859-1 Content-Transfer-Encoding: 8bit Use Tabela de Diferenças pra ver que da polinomio. Luis Lopes wrote:Sauda,c~oes, O que sabemos é que [S_n^(1)]^2 = S_n^(3). E o que vc diria de 3[S_n^(2)]^2 = S_n^(3) +2S_n^(5) 2[S_n^(3)]^2 = S_n^(5) +S_n^(7) onde S_n^(k) = 1^k + 2^k + .... + n^k ? Estas identidades foram tiradas de um livro antigo, autor Rio Nogueira. Título? Hum, Lições de Combinatória. []'s Luís -----Mensagem Original----- De: "Artur Costa Steiner" Para: Enviada em: terça-feira, 22 de abril de 2003 17:57 Assunto: Re: [Re: [obm-l] 4 coisinhas] > > > > > 4) > > > Calcular S = 1 + 8 + 27 + ... + x^3 > > Este o Igor deverá saber fazer. Fala, Igor! > > > Um fato interessante e razoavelmente bem conhecido (talvez nao tanto) eh que a > soma dos cubos dos n primeiros numeros naturais eh o quadrado das soma destes > numeros. Isto pode ser provado utilizando aquele processo classico (um tanto > bracal e cansativo), por recorrencia, ou por inducao finita - neste ultimo > caso, supondo, eh claro, que vc ja desconfia do resultado. Logo, 1^3 + 2^3 > ....+ n^3 = [n(n+1)/2]^2. A obtencao da soma dos n primeiros naturais eh > trivial, por P. Aritmetica > > PS. Por forca do habito, usei n e nao x para representar um natural... > Artur > ========================================================================= Instruções para entrar na lista, sair da lista e usar a lista em http://www.mat.puc-rio.br/~nicolau/olimp/obm-l.html ========================================================================= --------------------------------- Yahoo! Mail O melhor e-mail gratuito da internet: 6MB de espaço, antivírus, acesso POP3, filtro contra spam. --0-1934890832-1051051376=:25849 Content-Type: text/html; charset=iso-8859-1 Content-Transfer-Encoding: 8bit

Use Tabela de Diferenças pra ver que da polinomio.

 Luis Lopes <llopes@ensrbr.com.br> wrote:

Sauda,c~oes,

O que sabemos é que

[S_n^(1)]^2 = S_n^(3).

E o que vc diria de

3[S_n^(2)]^2 = S_n^(3) +2S_n^(5)

2[S_n^(3)]^2 = S_n^(5) +S_n^(7)

onde S_n^(k) = 1^k + 2^k + .... + n^k ?

Estas identidades foram tiradas de um livro
antigo, autor Rio Nogueira. Título? Hum,
Lições de Combinatória.

[]'s
Luís

-----Mensagem Original-----
De: "Artur Costa Steiner"
Para:
Enviada em: terça-feira, 22 de abril de 2003 17:57
Assunto: Re: [Re: [obm-l] 4 coisinhas]


> >
> > > 4)
> > > Calcular S = 1 + 8 + 27 + ... + x^3
> > Este o Igor deverá saber fazer. Fala, Igor!
> >
> Um fato interessante e razoavelmente bem conhecido (talvez nao tanto) eh
que a
> soma dos cubos dos n primeiros numeros naturais eh o quadrado das soma
destes
> numeros. Isto pode ser provado utilizando aquele processo classico (um
tanto
> bracal e cansativo), por recorrencia, ou por inducao finita - neste ultimo
> caso, supondo, eh claro, que vc ja desconfia do resultado. Logo, 1^3 + 2^3
> ....+ n^3 = [n(n+1)/2]^2. A obtencao da soma dos n primeiros naturais eh
> trivial, por P. Aritmetica
>
> PS. Por forca do habito, usei n e nao x para representar um natural...
> Artur
>


=========================================================================
Instruções para entrar na lista, sair da lista e usar a lista em
http://www.mat.puc-rio.br/~nicolau/olimp/obm-l.html
=========================================================================



Yahoo! Mail
O melhor e-mail gratuito da internet: 6MB de espaço, antivírus, acesso POP3, filtro contra spam. --0-1934890832-1051051376=:25849-- ========================================================================= Instruções para entrar na lista, sair da lista e usar a lista em http://www.mat.puc-rio.br/~nicolau/olimp/obm-l.html ========================================================================= From owner-obm-l@sucuri.mat.puc-rio.br Tue Apr 22 19:58:05 2003 Return-Path: Received: (from majordom@localhost) by sucuri.mat.puc-rio.br (8.9.3/8.9.3) id TAA13639 for obm-l-MTTP; Tue, 22 Apr 2003 19:55:13 -0300 Received: from imo-r03.mx.aol.com (imo-r03.mx.aol.com [152.163.225.99]) by sucuri.mat.puc-rio.br (8.9.3/8.9.3) with ESMTP id TAA13635 for ; Tue, 22 Apr 2003 19:55:09 -0300 From: SiarJoes@aol.com Received: from SiarJoes@aol.com by imo-r03.mx.aol.com (mail_out_v34.22.) id z.128.2855cf7d (30970) for ; Tue, 22 Apr 2003 18:54:26 -0400 (EDT) Message-ID: <128.2855cf7d.2bd72222@aol.com> Date: Tue, 22 Apr 2003 18:54:26 EDT Subject: Re: =?iso-8859-1?q?Re: Re: [obm-l] An=E1lise Combinat=F3ria?= To: obm-l@mat.puc-rio.br MIME-Version: 1.0 Content-Type: multipart/alternative; boundary="part1_128.2855cf7d.2bd72222_boundary" X-Mailer: 7.0 for Windows sub 10501 Sender: owner-obm-l@sucuri.mat.puc-rio.br Precedence: bulk Reply-To: obm-l@mat.puc-rio.br --part1_128.2855cf7d.2bd72222_boundary Content-Type: text/plain; charset="US-ASCII" Content-Transfer-Encoding: 7bit entendi, muito obrigado rapaz!!!! Sidmar Junior --part1_128.2855cf7d.2bd72222_boundary Content-Type: text/html; charset="US-ASCII" Content-Transfer-Encoding: quoted-printable entendi, muito obrigado rapaz!!!!<= BR> Sidmar Junior --part1_128.2855cf7d.2bd72222_boundary-- ========================================================================= Instruções para entrar na lista, sair da lista e usar a lista em http://www.mat.puc-rio.br/~nicolau/olimp/obm-l.html ========================================================================= From owner-obm-l@sucuri.mat.puc-rio.br Tue Apr 22 20:03:22 2003 Return-Path: Received: (from majordom@localhost) by sucuri.mat.puc-rio.br (8.9.3/8.9.3) id UAA13999 for obm-l-MTTP; Tue, 22 Apr 2003 20:00:47 -0300 Received: from mail.gmx.net (mail.gmx.net [213.165.65.60]) by sucuri.mat.puc-rio.br (8.9.3/8.9.3) with SMTP id UAA13985 for ; Tue, 22 Apr 2003 20:00:41 -0300 Received: (qmail 7428 invoked by uid 65534); 22 Apr 2003 23:00:09 -0000 Received: from unknown (EHLO localhost) (200.217.15.247) by mail.gmx.net (mp003-rz3) with SMTP; 23 Apr 2003 01:00:09 +0200 Date: Tue, 22 Apr 2003 19:54:30 -0300 From: Igor GomeZZ X-Mailer: The Bat! (v1.61) Organization: -- X-Priority: 3 (Normal) Message-ID: <9821226982.20030422195430@gmx.net> To: Luis Lopes Subject: Re[2]: [obm-l] 4 coisinhas In-Reply-To: <027d01c3090b$a009e880$5400a8c0@ensrbr> References: <20030421151630.66488.qmail@web13007.mail.yahoo.com> <027d01c3090b$a009e880$5400a8c0@ensrbr> MIME-Version: 1.0 Content-Type: text/plain; charset=ISO-8859-1 Content-Transfer-Encoding: 8bit Sender: owner-obm-l@sucuri.mat.puc-rio.br Precedence: bulk Reply-To: obm-l@mat.puc-rio.br Em 22/4/2003, 17:13, Luis (llopes@ensrbr.com.br) disse: >> 4) >> Calcular S = 1 + 8 + 27 + ... + x^3 > Este o Igor deverá saber fazer. Fala, Igor! Fala Luis! :-) /\[0](1, 8, 27, 64, 125, 216...) << PA original /\[1](7, 19, 37, 61, 91...) /\[2](12, 18, 24, 30...) /\[3](6, 6, 6...) << PA de ordem 3 O polinômio que descreve a soma eh de grau 3 + 1 = 4, logo: ** Solução no braço (Perde-se muito tempo numa prova) S(n) = ax^4 + bx^3 + cx^2 + dx + e, faz-se S(1) = 1 S(2) = 9 S(3) = 36 S(4) = 100 S(5) = 225, 5 equações e 5 incógnitas, resolve-se o sistema. [resolvendo pelo Maple] {e = 0, a = 1/4, b = 1/2, c = 1/4, d = 0} .:. S(x) = x^4/4 + x^3/2 + x^2/4 ** Solução à la Luis (Qual o nome desse método?) S(n) = (/\[0]a[1])*binomial(n,1) + (/\[1]a[1])*binomial(n,2) + + (/\[2]a[1])*binomial(n,3) + (/\[3]a[1])*binom(n,4) S(n) = 1*binomial(n,1) + 7*binomial(n,2) + 12*binomial(n,3) + 6*binomial(n,4) .:. S(n) = n^4/4 + n^3/2 + n^2/4 Fui! ####### Igor GomeZZ ######## UIN: 29249895 Vitória, Espírito Santo, Brasil Criação: 22/4/2003 (19:08) #################################### Pare para pensar: Nunca desencoraje ninguém que continuamente faz progresso, não importa quão devagar. (Platão) #################################### ========================================================================= Instruções para entrar na lista, sair da lista e usar a lista em http://www.mat.puc-rio.br/~nicolau/olimp/obm-l.html ========================================================================= From owner-obm-l@sucuri.mat.puc-rio.br Tue Apr 22 20:05:47 2003 Return-Path: Received: (from majordom@localhost) by sucuri.mat.puc-rio.br (8.9.3/8.9.3) id UAA14195 for obm-l-MTTP; Tue, 22 Apr 2003 20:03:02 -0300 Received: from web12907.mail.yahoo.com (web12907.mail.yahoo.com [216.136.174.74]) by sucuri.mat.puc-rio.br (8.9.3/8.9.3) with SMTP id UAA14182 for ; Tue, 22 Apr 2003 20:02:57 -0300 Message-ID: <20030422230225.98447.qmail@web12907.mail.yahoo.com> Received: from [200.144.43.151] by web12907.mail.yahoo.com via HTTP; Tue, 22 Apr 2003 20:02:25 ART Date: Tue, 22 Apr 2003 20:02:25 -0300 (ART) From: =?iso-8859-1?q?Johann=20Peter=20Gustav=20Lejeune=20Dirichlet?= Subject: [obm-l] Combinatoria e |Desordens To: obm-l@mat.puc-rio.br MIME-Version: 1.0 Content-Type: multipart/alternative; boundary="0-949939697-1051052545=:97549" Content-Transfer-Encoding: 8bit Sender: owner-obm-l@sucuri.mat.puc-rio.br Precedence: bulk Reply-To: obm-l@mat.puc-rio.br --0-949939697-1051052545=:97549 Content-Type: text/plain; charset=iso-8859-1 Content-Transfer-Encoding: 8bit Oi genten!!!!E ai,se alguem manja de permutaçao caotica(sem pontos fixos) de (1,2,3,...,n),se chamarmos K_n o numero de jeitos de permutar caoticamente,sera que da pra demonstrar COMBINATRORIALMENTE que K_n=n*K_(n-1)+(-1)^n? --------------------------------- Yahoo! Mail O melhor e-mail gratuito da internet: 6MB de espaço, antivírus, acesso POP3, filtro contra spam. --0-949939697-1051052545=:97549 Content-Type: text/html; charset=iso-8859-1 Content-Transfer-Encoding: 8bit
Oi genten!!!!E ai,se alguem manja de permutaçao caotica(sem pontos fixos) de (1,2,3,...,n),se chamarmos K_n o numero de jeitos de permutar caoticamente,sera que da pra demonstrar COMBINATRORIALMENTE que
K_n=n*K_(n-1)+(-1)^n?



Yahoo! Mail
O melhor e-mail gratuito da internet: 6MB de espaço, antivírus, acesso POP3, filtro contra spam. --0-949939697-1051052545=:97549-- ========================================================================= Instruções para entrar na lista, sair da lista e usar a lista em http://www.mat.puc-rio.br/~nicolau/olimp/obm-l.html ========================================================================= From owner-obm-l@sucuri.mat.puc-rio.br Tue Apr 22 20:16:53 2003 Return-Path: Received: (from majordom@localhost) by sucuri.mat.puc-rio.br (8.9.3/8.9.3) id UAA15106 for obm-l-MTTP; Tue, 22 Apr 2003 20:14:14 -0300 Received: from web14303.mail.yahoo.com (web14303.mail.yahoo.com [216.136.173.79]) by sucuri.mat.puc-rio.br (8.9.3/8.9.3) with SMTP id UAA15088 for ; Tue, 22 Apr 2003 20:14:09 -0300 Message-ID: <20030422231335.45324.qmail@web14303.mail.yahoo.com> Received: from [200.17.25.3] by web14303.mail.yahoo.com via HTTP; Tue, 22 Apr 2003 20:13:35 ART Date: Tue, 22 Apr 2003 20:13:35 -0300 (ART) From: =?iso-8859-1?q?Rafael?= Subject: [obm-l] congruencia To: OBM MIME-Version: 1.0 Content-Type: text/plain; charset=iso-8859-1 Content-Transfer-Encoding: 8bit Sender: owner-obm-l@sucuri.mat.puc-rio.br Precedence: bulk Reply-To: obm-l@mat.puc-rio.br Olá pessoal! Acho que essa sai por congruências mas ainda não consegui: O número de valores inteiros de n para os quais a fração (15n + 2)/(14n + 3) pode ser simplificada é: resposta: 118 Se alguém puder me dar uma luz... Abraços, Rafael. _______________________________________________________________________ Yahoo! Mail O melhor e-mail gratuito da internet: 6MB de espaço, antivírus, acesso POP3, filtro contra spam. http://br.mail.yahoo.com/ ========================================================================= Instruções para entrar na lista, sair da lista e usar a lista em http://www.mat.puc-rio.br/~nicolau/olimp/obm-l.html ========================================================================= From owner-obm-l@sucuri.mat.puc-rio.br Tue Apr 22 20:30:09 2003 Return-Path: Received: (from majordom@localhost) by sucuri.mat.puc-rio.br (8.9.3/8.9.3) id UAA15885 for obm-l-MTTP; Tue, 22 Apr 2003 20:26:31 -0300 Received: from web12905.mail.yahoo.com (web12905.mail.yahoo.com [216.136.174.72]) by sucuri.mat.puc-rio.br (8.9.3/8.9.3) with SMTP id UAA15880 for ; Tue, 22 Apr 2003 20:26:27 -0300 Message-ID: <20030422232555.96652.qmail@web12905.mail.yahoo.com> Received: from [200.144.43.151] by web12905.mail.yahoo.com via HTTP; Tue, 22 Apr 2003 20:25:55 ART Date: Tue, 22 Apr 2003 20:25:55 -0300 (ART) From: =?iso-8859-1?q?Johann=20Peter=20Gustav=20Lejeune=20Dirichlet?= Subject: [obm-l] TEOIREMA DE CRISTEA::Geometria,pontos estranhos e problemas legais!!!! To: OBM-L@mat.puc-rio.br MIME-Version: 1.0 Content-Type: multipart/alternative; boundary="0-1534575639-1051053955=:96631" Content-Transfer-Encoding: 8bit Sender: owner-obm-l@sucuri.mat.puc-rio.br Precedence: bulk Reply-To: obm-l@mat.puc-rio.br --0-1534575639-1051053955=:96631 Content-Type: text/plain; charset=iso-8859-1 Content-Transfer-Encoding: 8bit Ola gente!!!!Estou tentando resolver problemas de Geometria do Mathematical excalibur mas preciso de um teorema.Quem pode demonstrar pra mim: Considere o triangulo ABC e um ponto T.Duas cevianas CT_c e BT_b se cortam erm T e os pontos R_c,R_b e T sao alinhados,com R_c em AB e R_b em AC.Mostre que AT_c*BR_c/T_cB/R_cA + ATb*CR_b//T_bC/R_bA = 1 Nao sei se e isso mesmo mas vale o risco... --------------------------------- Yahoo! Mail O melhor e-mail gratuito da internet: 6MB de espaço, antivírus, acesso POP3, filtro contra spam. --0-1534575639-1051053955=:96631 Content-Type: text/html; charset=iso-8859-1 Content-Transfer-Encoding: 8bit
Ola gente!!!!Estou tentando resolver problemas de Geometria do Mathematical excalibur mas preciso de um teorema.Quem pode demonstrar pra mim:
 
Considere o triangulo ABC e um ponto T.Duas cevianas CT_c e BT_b se cortam erm T e os pontos R_c,R_b e T sao alinhados,com R_c em AB e R_b em AC.Mostre que
 
AT_c*BR_c/T_cB/R_cA + ATb*CR_b//T_bC/R_bA = 1
 
Nao sei se e isso mesmo mas vale o risco...



Yahoo! Mail
O melhor e-mail gratuito da internet: 6MB de espaço, antivírus, acesso POP3, filtro contra spam. --0-1534575639-1051053955=:96631-- ========================================================================= Instruções para entrar na lista, sair da lista e usar a lista em http://www.mat.puc-rio.br/~nicolau/olimp/obm-l.html ========================================================================= From owner-obm-l@sucuri.mat.puc-rio.br Tue Apr 22 21:26:10 2003 Return-Path: Received: (from majordom@localhost) by sucuri.mat.puc-rio.br (8.9.3/8.9.3) id VAA18424 for obm-l-MTTP; Tue, 22 Apr 2003 21:23:21 -0300 Received: from trex.centroin.com.br (trex.centroin.com.br [200.225.63.134]) by sucuri.mat.puc-rio.br (8.9.3/8.9.3) with ESMTP id VAA18420 for ; Tue, 22 Apr 2003 21:23:17 -0300 Received: from trex.centroin.com.br (localhost [127.0.0.1]) by trex.centroin.com.br (8.12.9/8.12.9) with ESMTP id h3N0MmrL015800 for ; Tue, 22 Apr 2003 21:22:48 -0300 (EST) Received: (from morgado@localhost) by trex.centroin.com.br (8.12.9/8.12.5/Submit) id h3N0MmX9015799; Tue, 22 Apr 2003 21:22:48 -0300 (EST) Message-Id: <200304230022.h3N0MmX9015799@trex.centroin.com.br> Received: from 200.141.119.111 by trex.centroin.com.br (CIPWM versao 1.4C1) with HTTPS for ; Tue, 22 Apr 2003 21:22:48 -0300 (EST) Date: Tue, 22 Apr 2003 21:22:48 -0300 (EST) From: Augusto Cesar de Oliveira Morgado To: obm-l@mat.puc-rio.br Subject: Re: [obm-l] Mais uma coisinha MIME-Version: 1.0 X-Mailer: CentroIn Internet Provider WebMail v. 1.4C1 (http://www.centroin.com.br/) Content-Type: text/plain; charset="iso-8859-1" Content-Transfer-Encoding: 8bit X-MIME-Autoconverted: from quoted-printable to 8bit by sucuri.mat.puc-rio.br id VAA18421 Sender: owner-obm-l@sucuri.mat.puc-rio.br Precedence: bulk Reply-To: obm-l@mat.puc-rio.br Helder, mensagens do tipo onde achar livro, revista etc. devem sempre vir acompanhadas da cidade do autor da mensagem. Morgado Em Tue, 22 Apr 2003 19:14:39 -0300 (ART), Helder Suzuki disse: > Olá! > > Bom, primeiro eu agradeço a todos que responderam ao > "4 coisinhas", embora o problema do jogo não foi lá > bem muito resolvido e eu não sei aonde achar a tal > revista hehehe :P > > 1) > João e Maria brincam de multiplicar. > Antes de começar o jogo, eles escolhem um natural n>1. > > João começa com o número 1 e multiplica-o por um > natural do intervalo [2, 9], Maria pega o resultado > faz o mesmo, e assim sucessivamente. > > Ganha o jogo o primeiro que conseguir fazer o > resultado >= n. > > Se João e Maria jogam perfeitamente, e João sempre > começa o jogo, para quais valores de n João ganha? e > Maria? > > []'s, > > Helder Toshiro Suzuki > > _______________________________________________________________________ > Yahoo! Mail > O melhor e-mail gratuito da internet: 6MB de espaço, antivírus, acesso POP3, filtro contra spam. > http://br.mail.yahoo.com/ > ========================================================================= > Instruções para entrar na lista, sair da lista e usar a lista em > http://www.mat.puc-rio.br/~nicolau/olimp/obm-l.html > ========================================================================= > > ========================================================================= Instruções para entrar na lista, sair da lista e usar a lista em http://www.mat.puc-rio.br/~nicolau/olimp/obm-l.html ========================================================================= From owner-obm-l@sucuri.mat.puc-rio.br Tue Apr 22 21:27:22 2003 Return-Path: Received: (from majordom@localhost) by sucuri.mat.puc-rio.br (8.9.3/8.9.3) id VAA18451 for obm-l-MTTP; Tue, 22 Apr 2003 21:24:49 -0300 Received: from trex.centroin.com.br (trex.centroin.com.br [200.225.63.134]) by sucuri.mat.puc-rio.br (8.9.3/8.9.3) with ESMTP id VAA18442 for ; Tue, 22 Apr 2003 21:24:44 -0300 Received: from trex.centroin.com.br (localhost [127.0.0.1]) by trex.centroin.com.br (8.12.9/8.12.9) with ESMTP id h3N0OErL016336 for ; Tue, 22 Apr 2003 21:24:14 -0300 (EST) Received: by trex.centroin.com.br (8.12.9/8.12.5/Submit) id h3N0OE0W016335; Tue, 22 Apr 2003 21:24:14 -0300 (EST) Message-Id: <200304230024.h3N0OE0W016335@trex.centroin.com.br> Received: from 200.141.119.111 by trex.centroin.com.br (CIPWM versao 1.4C1) with HTTPS for ; Tue, 22 Apr 2003 21:24:14 -0300 (EST) Date: Tue, 22 Apr 2003 21:24:14 -0300 (EST) From: Augusto Cesar de Oliveira Morgado To: obm-l@mat.puc-rio.br Subject: =?iso-8859-1?q?Re: Re: Re: [obm-l] An=E1lise Combinat=F3ria?= MIME-Version: 1.0 X-Mailer: CentroIn Internet Provider WebMail v. 1.4C1 (http://www.centroin.com.br/) Content-Type: text/plain Sender: owner-obm-l@sucuri.mat.puc-rio.br Precedence: bulk Reply-To: obm-l@mat.puc-rio.br Do alto dos meus 59 anos, obrigado pelo rapaz. Em Tue, 22 Apr 2003 18:54:26 EDT, SiarJoes@aol.com disse: > entendi, muito obrigado rapaz!!!! > Sidmar Junior ========================================================================= Instruções para entrar na lista, sair da lista e usar a lista em http://www.mat.puc-rio.br/~nicolau/olimp/obm-l.html ========================================================================= From owner-obm-l@sucuri.mat.puc-rio.br Tue Apr 22 21:37:32 2003 Return-Path: Received: (from majordom@localhost) by sucuri.mat.puc-rio.br (8.9.3/8.9.3) id VAA18702 for obm-l-MTTP; Tue, 22 Apr 2003 21:34:49 -0300 Received: from trex.centroin.com.br (trex.centroin.com.br [200.225.63.134]) by sucuri.mat.puc-rio.br (8.9.3/8.9.3) with ESMTP id VAA18689 for ; Tue, 22 Apr 2003 21:34:44 -0300 Received: from trex.centroin.com.br (localhost [127.0.0.1]) by trex.centroin.com.br (8.12.9/8.12.9) with ESMTP id h3N0YErL021208 for ; Tue, 22 Apr 2003 21:34:14 -0300 (EST) Received: by trex.centroin.com.br (8.12.9/8.12.5/Submit) id h3N0YEO4021205; Tue, 22 Apr 2003 21:34:14 -0300 (EST) Message-Id: <200304230034.h3N0YEO4021205@trex.centroin.com.br> Received: from 200.141.119.111 by trex.centroin.com.br (CIPWM versao 1.4C1) with HTTPS for ; Tue, 22 Apr 2003 21:34:14 -0300 (EST) Date: Tue, 22 Apr 2003 21:34:14 -0300 (EST) From: Augusto Cesar de Oliveira Morgado To: obm-l@mat.puc-rio.br Subject: =?iso-8859-1?q?Re: [obm-l] Re:_[obm-l]_Re:_[obm-l]_Parece_f=E1cil...m as,_n=E3o_consigo= Sender: owner-obm-l@sucuri.mat.puc-rio.br Precedence: bulk Reply-To: obm-l@mat.puc-rio.br .?= MIME-Version: 1.0 X-Mailer: CentroIn Internet Provider WebMail v. 1.4C1 (http://www.centroin.com.br/) Content-Type: text/plain; charset="iso-8859-1" Content-Transfer-Encoding: quoted-printable Oh Marcus, teu ach=F4metro ta com defeito. Alguem, infelizmente deletei a mensagem, deu a solu=E7ao do Livro para es= se problema!Aqui vai ela de novo, talvez um pouco mais explicadinha. (2k+1)^2 =3D 4k(k+1)+1 k(k+1)eh sempre par , logo o quadrado de um impar eh sempre igual a um mu= ltiplo de 8 mais 1. Logo, se m e n sao impares, m^4 + n^4 - 2 eh uma soma de dois multiplos d= e 8. Em Tue, 22 Apr 2003 17:16:40 -0300, Marcus Alexandre Nunes disse: > Eu achei as respostas dadas para este problema um tanto complicadas, en= t=E3o estou enviando a minha: >=20 > m, n =EDmpares =3D> 8|(m^4 + n^4 -2) >=20 > Como m, n s=E3o =EDmpares, podemos escrever >=20 > m =3D 2k + 1, k pertence aos inteiros; > n =3D 2l + 1, l pertence aos inteiros. >=20 > m^4 =3D (2k+1)^4 =3D 16k^4 + 32k^3 + 24k^2 +8k + 1 > n^4 =3D (2l+1)^4 =3D 16l^4 + 32l^3 + 24l^2 +8l + 1 >=20 > Ent=E3o temos >=20 > 8|[ (16k^4 + 32k^3 + 24k^2 +8k + 1) + (16l^4 + 32l^3 + 24l^2 +8l + 1) -= 2] >=20 > 8|[ 16k^4 + 32k^3 + 24k^2 + 8k + 16l^4 + 32l^3 + 24l^2 + 8l ] >=20 > Colocamos 8 em evid=EAncia dentro dos colchetes e ficamos com >=20 > 8|8[ 2k^4 + 4k^3 + 3k^2 + k + 2l^4 + 4l^3 + 3l^2 + l ] >=20 > O que prova que 8|(m^4 + n^4 -2), se m, n s=E3o =EDmpares. >=20 > ---------------------------------------------- > Marcus Alexandre Nunes > marcus_math@yahoo.com.br > UIN 114153703 ========================================================================= Instruções para entrar na lista, sair da lista e usar a lista em http://www.mat.puc-rio.br/~nicolau/olimp/obm-l.html ========================================================================= From owner-obm-l@sucuri.mat.puc-rio.br Tue Apr 22 22:01:48 2003 Return-Path: Received: (from majordom@localhost) by sucuri.mat.puc-rio.br (8.9.3/8.9.3) id VAA19778 for obm-l-MTTP; Tue, 22 Apr 2003 21:59:08 -0300 Received: from trex.centroin.com.br (trex.centroin.com.br [200.225.63.134]) by sucuri.mat.puc-rio.br (8.9.3/8.9.3) with ESMTP id VAA19756 for ; Tue, 22 Apr 2003 21:59:00 -0300 Received: from trex.centroin.com.br (localhost [127.0.0.1]) by trex.centroin.com.br (8.12.9/8.12.9) with ESMTP id h3N0wSrL000620 for ; Tue, 22 Apr 2003 21:58:28 -0300 (EST) Received: by trex.centroin.com.br (8.12.9/8.12.5/Submit) id h3N0wSKR000619; Tue, 22 Apr 2003 21:58:28 -0300 (EST) Message-Id: <200304230058.h3N0wSKR000619@trex.centroin.com.br> Received: from 200.141.119.111 by trex.centroin.com.br (CIPWM versao 1.4C1) with HTTPS for ; Tue, 22 Apr 2003 21:58:28 -0300 (EST) Date: Tue, 22 Apr 2003 21:58:28 -0300 (EST) From: Augusto Cesar de Oliveira Morgado To: obm-l@mat.puc-rio.br Subject: Re: [obm-l] congruencia MIME-Version: 1.0 X-Mailer: CentroIn Internet Provider WebMail v. 1.4C1 (http://www.centroin.com.br/) Content-Type: text/plain; charset="iso-8859-1" Content-Transfer-Encoding: 8bit X-MIME-Autoconverted: from quoted-printable to 8bit by sucuri.mat.puc-rio.br id VAA19766 Sender: owner-obm-l@sucuri.mat.puc-rio.br Precedence: bulk Reply-To: obm-l@mat.puc-rio.br (15n+2)/(14n+3)= 1 + [(n-1)/(14n+3)]eh simplificavel sse (14n+3)/(n-1) o for. (14n+3)/(n-1)=14 + [17/(n-1)]eh simplificavel sse 17/(n-1) o for: Resposta: n - 1 múltiplo de 17 Ha infinitos valores para n Em Tue, 22 Apr 2003 20:13:35 -0300 (ART), Rafael disse: > Olá pessoal! > > Acho que essa sai por congruências mas ainda não > consegui: > O número de valores inteiros de n para os quais a > fração (15n + 2)/(14n + 3) pode ser simplificada é: > > resposta: 118 > > Se alguém puder me dar uma luz... > > Abraços, > > Rafael. > > _______________________________________________________________________ > Yahoo! Mail > O melhor e-mail gratuito da internet: 6MB de espaço, antivírus, acesso POP3, filtro contra spam. > http://br.mail.yahoo.com/ > ========================================================================= > Instruções para entrar na lista, sair da lista e usar a lista em > http://www.mat.puc-rio.br/~nicolau/olimp/obm-l.html > ========================================================================= > > ========================================================================= Instruções para entrar na lista, sair da lista e usar a lista em http://www.mat.puc-rio.br/~nicolau/olimp/obm-l.html ========================================================================= From owner-obm-l@sucuri.mat.puc-rio.br Tue Apr 22 22:32:35 2003 Return-Path: Received: (from majordom@localhost) by sucuri.mat.puc-rio.br (8.9.3/8.9.3) id WAA20913 for obm-l-MTTP; Tue, 22 Apr 2003 22:29:59 -0300 Received: from itaqui.terra.com.br (itaqui.terra.com.br [200.176.3.19]) by sucuri.mat.puc-rio.br (8.9.3/8.9.3) with ESMTP id WAA20907 for ; Tue, 22 Apr 2003 22:29:54 -0300 Received: from altamira.terra.com.br (altamira.terra.com.br [200.176.3.40]) by itaqui.terra.com.br (Postfix) with ESMTP id EBB7B3BC0D6 for ; Tue, 22 Apr 2003 22:29:23 -0300 (BRT) Received: from [200.177.182.32] (dl-nas6-sao-C8B1B620.p001.terra.com.br [200.177.182.32]) by altamira.terra.com.br (Postfix) with ESMTP id 2BEB23DC093 for ; Tue, 22 Apr 2003 22:29:23 -0300 (BRT) User-Agent: Microsoft-Outlook-Express-Macintosh-Edition/5.02.2022 Date: Fri, 08 Jan 1904 10:22:08 -0200 Subject: Re: [obm-l] congruencia From: Claudio Buffara To: Message-ID: <9C9B9.589%claudio.buffara@terra.com.br> In-Reply-To: <20030422231335.45324.qmail@web14303.mail.yahoo.com> Mime-version: 1.0 Content-type: text/plain; charset="ISO-8859-1" Content-Transfer-Encoding: 8bit X-MIME-Autoconverted: from quoted-printable to 8bit by sucuri.mat.puc-rio.br id WAA20908 Sender: owner-obm-l@sucuri.mat.puc-rio.br Precedence: bulk Reply-To: obm-l@mat.puc-rio.br on 22.04.03 21:13, Rafael at matduvidas@yahoo.com.br wrote: > Olá pessoal! > > Acho que essa sai por congruências mas ainda não > consegui: > O número de valores inteiros de n para os quais a > fração (15n + 2)/(14n + 3) pode ser simplificada é: > > resposta: 118 > > Se alguém puder me dar uma luz... > > Abraços, > > Rafael. > Oi, Rafael: Repare que: -14*(15n+2) + 15*(14n+3) = 17 (independente de n). Logo, pelo teorema de Bezout, mdc(15n+2,14n+3) so pode ser 1 ou 17. Se a fracao pode ser simplificada, entao o mdc deve ser igual a 17. Isso implica que: 15n + 2 == 0 (mod 17) e 14n + 3 == 0 (mod 17) Ou seja: n == 1 (mod 17) Em outras palavras: Se n = 17k + 1, para algum inteiro k, entao, a fracao serah: (15n+2)/(14n+3) = [15(17k+1)+2]/[14(17k+1)+3] = = [17*(15k+1)]/[17*(14k+1)], a qual pode ser simplificada (para todo k) e resultara na fracao irredutivel (15k+1)/(14k+1). Em suma, ha infinitos valores inteiros de n para os quais a fracao pode ser simplificada (a saber, aqueles da forma 17k + 1, com k inteiro). Um abraco, Claudio. ========================================================================= Instruções para entrar na lista, sair da lista e usar a lista em http://www.mat.puc-rio.br/~nicolau/olimp/obm-l.html ========================================================================= From owner-obm-l@sucuri.mat.puc-rio.br Tue Apr 22 22:32:40 2003 Return-Path: Received: (from majordom@localhost) by sucuri.mat.puc-rio.br (8.9.3/8.9.3) id WAA20927 for obm-l-MTTP; Tue, 22 Apr 2003 22:30:01 -0300 Received: from itaqui.terra.com.br (itaqui.terra.com.br [200.176.3.19]) by sucuri.mat.puc-rio.br (8.9.3/8.9.3) with ESMTP id WAA20909 for ; Tue, 22 Apr 2003 22:29:55 -0300 Received: from altamira.terra.com.br (altamira.terra.com.br [200.176.3.40]) by itaqui.terra.com.br (Postfix) with ESMTP id 38F063BCE15 for ; Tue, 22 Apr 2003 22:29:25 -0300 (BRT) Received: from [200.177.182.32] (dl-nas6-sao-C8B1B620.p001.terra.com.br [200.177.182.32]) by altamira.terra.com.br (Postfix) with ESMTP id 537393DC093 for ; Tue, 22 Apr 2003 22:29:24 -0300 (BRT) User-Agent: Microsoft-Outlook-Express-Macintosh-Edition/5.02.2022 Date: Fri, 08 Jan 1904 10:22:09 -0200 Subject: Re: [obm-l] Polinomialmente assim... From: Claudio Buffara To: Message-ID: <9CC42.58A%claudio.buffara@terra.com.br> In-Reply-To: <20030422223345.30175.qmail@web12908.mail.yahoo.com> Mime-version: 1.0 Content-type: multipart/alternative; boundary="MS_Mac_OE_642170_678313_MIME_Part" Sender: owner-obm-l@sucuri.mat.puc-rio.br Precedence: bulk Reply-To: obm-l@mat.puc-rio.br > This message is in MIME format. Since your mail reader does not understand this format, some or all of this message may not be legible. --MS_Mac_OE_642170_678313_MIME_Part Content-type: text/plain; charset="US-ASCII" Content-transfer-encoding: 7bit on 22.04.03 20:33, Johann Peter Gustav Lejeune Dirichlet at peterdirichlet2002@yahoo.com.br wrote: Oi turma,estou as voltas com esse aqui,vejam: Qual o produto e a soma das raizes POSITIVAS de (x^2+2x-12)^2=9x^3-108x ? Oi, JP: A equacao eh: (x^2+2x-12)^2 = 9x(x^2-12) Faca y = x^2 - 12. Entao: (y+2x)^2 = 9xy ==> y^2 + 4x^2 + 4xy = 9xy ==> y^2 - 5xy + 4x^2 = 0 ==> y^2 - 2*(5x/2)*y + 25x^2/4 - 9x^2/4 = 0 ==> (y - 5x/2)^2 - (3x/2)^2 = 0 ==> (y - 4x)(y - x) = 0 ==> y = 4x ou y = x ==> x^2 - 4x - 12 = 0 ou x^2 - x - 12 = 0 ==> x = -2 ou x = 6 ou x = 4 ou x = - 3 ==> as raizes positivas sao 4 e 6 ==> Produto = 24 e Soma = 10 Um abraco, Claudio. --MS_Mac_OE_642170_678313_MIME_Part Content-type: text/html; charset="US-ASCII" Content-transfer-encoding: quoted-printable Re: [obm-l] Polinomialmente assim... on 22.04.03 20:33, Johann Peter Gustav Lejeune Dirichlet at peterdirichlet2= 002@yahoo.com.br wrote:

Oi turma,estou as voltas com esse aqui,vejam:
Qual o produto e a soma das raizes POSITIVAS de (x^2+2x-12)^2=3D9x^3-108x ?
Oi, JP:

A equacao eh:
(x^2+2x-12)^2 =3D 9x(x^2-12)

Faca y =3D x^2 - 12. Entao:

(y+2x)^2 =3D 9xy =3D=3D>
y^2 + 4x^2 + 4xy =3D 9xy =3D=3D>
y^2 - 5xy + 4x^2 =3D 0 =3D=3D>
y^2 - 2*(5x/2)*y + 25x^2/4 - 9x^2/4 =3D 0 =3D=3D>
(y - 5x/2)^2 - (3x/2)^2 =3D 0 =3D=3D>
(y - 4x)(y - x) =3D 0 =3D=3D>
y =3D 4x   ou   y =3D x =3D=3D>
x^2 - 4x - 12 =3D 0   ou   x^2 - x - 12 =3D 0 =3D=3D>
x =3D -2  ou  x =3D 6  ou  x =3D 4  ou  x =3D - 3 =3D=3D&= gt;
as raizes positivas sao 4 e 6 =3D=3D>

Produto =3D 24  e  Soma =3D 10

Um abraco,
Claudio.

--MS_Mac_OE_642170_678313_MIME_Part-- ========================================================================= Instruções para entrar na lista, sair da lista e usar a lista em http://www.mat.puc-rio.br/~nicolau/olimp/obm-l.html ========================================================================= From owner-obm-l@sucuri.mat.puc-rio.br Tue Apr 22 22:36:45 2003 Return-Path: Received: (from majordom@localhost) by sucuri.mat.puc-rio.br (8.9.3/8.9.3) id WAA21071 for obm-l-MTTP; Tue, 22 Apr 2003 22:34:13 -0300 Received: from smtp-27.ig.com.br (smtp-27.ig.com.br [200.226.132.159]) by sucuri.mat.puc-rio.br (8.9.3/8.9.3) with SMTP id WAA21067 for ; Tue, 22 Apr 2003 22:34:09 -0300 Received: (qmail 13815 invoked from network); 23 Apr 2003 01:33:51 -0000 Received: from unknown (HELO xxxx) (200.165.168.185) by smtp-27.ig.com.br with SMTP; 23 Apr 2003 01:33:51 -0000 Message-ID: <006f01c30938$ebfc39a0$b9a8a5c8@epq.ime.eb.br> From: "Marcio" To: References: <20030422221439.11707.qmail@web13007.mail.yahoo.com> Subject: Re: [obm-l] Mais uma coisinha Date: Tue, 22 Apr 2003 22:37:37 -0300 MIME-Version: 1.0 Content-Type: text/plain; charset="iso-8859-1" Content-Transfer-Encoding: 8bit X-Priority: 3 X-MSMail-Priority: Normal X-Mailer: Microsoft Outlook Express 5.50.4133.2400 X-MimeOLE: Produced By Microsoft MimeOLE V5.50.4133.2400 Sender: owner-obm-l@sucuri.mat.puc-rio.br Precedence: bulk Reply-To: obm-l@mat.puc-rio.br Oi Helder. Legal esse problema.. Minha solucao ficou um pouco longa pq eu fui escrevendo direto no computador.. Espero que esteja certa.. Vamos montar, a partir do final, uma estratégia vencedora para o jogador A. (1) A vence na rodada k sse receber um número x1 que seja maior ou igual a n/9. (n/9 <= x1 < n) Como A pode forcar B a responder um x1 nessas condicoes? (2)Para que B fique sem saida, A manda um número y1 tal que 2y1 > n/9, isto é y1 > n/18.. Alem disso, A deve se preocupar para que B nao venca, logo ele deve mandar y1 < n/9. Ou seja, se o jogador A puder deixar y1 em [n/18, n/9), então B fatalmente retorna um número x1 em [n/9,n) e A vence. (1) Para poder garantir que B recebe esse y1, A deve receber um número x2 em [n/18.9, n/18) (e ai A vai escolher um multiplicador de 2 a 9 para garantir o y1. Se x2 for n/18.9, o multiplicador pode ser 9. Se for n/18, A escolhe 2). (2) Para que B fique sem saida, A manda um y2 em [n/18.18, n/18.9) E assim por diante.. Agora vamos reverter o processo.. Para comecar, voce deve descobrir em que tipo de intervalo o 1 vai cair.. Por exemplo, se existir k tq n / (9.18^k) <= 1 < n/ (18^k), entao Joao eh A, e vence o jogo. Seja k = [(log n)/(log 18)] (função teto), ou seja, 18^k < n <= 18^(k+1). Entao, 1 pertence ao intervalo I = [ n / (18^(k+1)) , n/(18^k) [. Mas I = [n/ (18^(k+1)), n/ (9.18^k) [ U [n/(9.18^k), n/(18^k) [ = X U Y (nessa ordem). Agora eh facil reverter o processo acima e concluir que se 1 pertence a X, então Joao comeca na situacao que acima fora descrita como sendo a de B, ou seja, Joao eh perdedor. Por outro lado, se 1 pertence a Y, entao Joao eh vencedor.. Voce pode formalizar isso se quiser, fazendo o caminho logico correto.. Por exemplo, se 1 esta em X, entao: Joao pega um numero em [n/ (18^(k+1)), n/ (9.18^k) [ .. Como Joao multiplica no minimo por 2 e no maximo por 9: Maria recebe um numero em [n / (9.18^k), n/18^k [. Maria pode entao escolher um multiplicador dentre 2,3,...,9 para forcar que: Joao receba um numero em [n / (18^k), n/ (9. 18^(k-1) ) [. e assim por diante.. Agora, o expoente do 18 vai diminuindo. Quando esse expoente chegar em 0: Maria recebe um numero em [n/9, n[, e ganha o jogo multiplicando esse nr por 9. O outro caso (em que 1 esta em Y) eh analogo. ----- Original Message ----- From: "Helder Suzuki" To: Sent: Tuesday, April 22, 2003 7:14 PM Subject: [obm-l] Mais uma coisinha > Olá! > > Bom, primeiro eu agradeço a todos que responderam ao > "4 coisinhas", embora o problema do jogo não foi lá > bem muito resolvido e eu não sei aonde achar a tal > revista hehehe :P > > 1) > João e Maria brincam de multiplicar. > Antes de começar o jogo, eles escolhem um natural n>1. > > João começa com o número 1 e multiplica-o por um > natural do intervalo [2, 9], Maria pega o resultado > faz o mesmo, e assim sucessivamente. > > Ganha o jogo o primeiro que conseguir fazer o > resultado >= n. > > Se João e Maria jogam perfeitamente, e João sempre > começa o jogo, para quais valores de n João ganha? e > Maria? ========================================================================= Instruções para entrar na lista, sair da lista e usar a lista em http://www.mat.puc-rio.br/~nicolau/olimp/obm-l.html ========================================================================= From owner-obm-l@sucuri.mat.puc-rio.br Tue Apr 22 22:52:09 2003 Return-Path: Received: (from majordom@localhost) by sucuri.mat.puc-rio.br (8.9.3/8.9.3) id WAA21927 for obm-l-MTTP; Tue, 22 Apr 2003 22:49:22 -0300 Received: from artemis.opendf.com.br (artemis.opengate.com.br [200.181.71.14]) by sucuri.mat.puc-rio.br (8.9.3/8.9.3) with ESMTP id WAA21922 for ; Tue, 22 Apr 2003 22:49:18 -0300 Received: from localhost (localhost [127.0.0.1]) by artemis.opendf.com.br (Postfix) with ESMTP id D3D2A2BED3 for ; Tue, 22 Apr 2003 22:49:10 -0300 (BRT) Received: from artemis.opendf.com.br ([127.0.0.1]) by localhost (artemis.opengate.com.br [127.0.0.1:10024]) (amavisd-new) with ESMTP id 10819-05 for ; Tue, 22 Apr 2003 22:49:10 -0300 (BRT) Received: from computer (200-181-090-074.bsace7001.dsl.brasiltelecom.net.br [200.181.90.74]) by artemis.opendf.com.br (Postfix) with ESMTP id A58322BECD for ; Tue, 22 Apr 2003 22:49:09 -0300 (BRT) From: "Artur Costa Steiner" To: Subject: [obm-l] =?iso-8859-1?Q?RE:_Re:_Re:_=5Bobm-l=5D_An=E1lise_Combinat=F3ria?= Date: Tue, 22 Apr 2003 22:48:44 -0300 Organization: Steiner Consultoria LTDA Message-ID: <003701c3093a$7a753280$9865fea9@computer> MIME-Version: 1.0 Content-Type: text/plain; charset="iso-8859-1" X-Priority: 3 (Normal) X-MSMail-Priority: Normal X-Mailer: Microsoft Outlook, Build 10.0.2627 X-MimeOLE: Produced By Microsoft MimeOLE V6.00.2800.1106 In-Reply-To: <200304230024.h3N0OE0W016335@trex.centroin.com.br> Importance: Normal X-Virus-Scanned: by amavisd-new Content-Transfer-Encoding: 8bit X-MIME-Autoconverted: from quoted-printable to 8bit by sucuri.mat.puc-rio.br id WAA21923 Sender: owner-obm-l@sucuri.mat.puc-rio.br Precedence: bulk Reply-To: obm-l@mat.puc-rio.br >Do alto dos meus 59 anos, obrigado pelo rapaz. > > [Artur Costa Steiner] Os bons em matematica (e em qualquer ciência) nunca envelhecem. Artur ========================================================================= Instruções para entrar na lista, sair da lista e usar a lista em http://www.mat.puc-rio.br/~nicolau/olimp/obm-l.html ========================================================================= From owner-obm-l@sucuri.mat.puc-rio.br Tue Apr 22 23:09:44 2003 Return-Path: Received: (from majordom@localhost) by sucuri.mat.puc-rio.br (8.9.3/8.9.3) id XAA22630 for obm-l-MTTP; Tue, 22 Apr 2003 23:07:08 -0300 Received: from imo-r06.mx.aol.com (imo-r06.mx.aol.com [152.163.225.102]) by sucuri.mat.puc-rio.br (8.9.3/8.9.3) with ESMTP id XAA22605 for ; Tue, 22 Apr 2003 23:06:59 -0300 From: Korshinoi@aol.com Received: from Korshinoi@aol.com by imo-r06.mx.aol.com (mail_out_v34.22.) id z.148.fdb1870 (4116) for ; Tue, 22 Apr 2003 22:06:16 -0400 (EDT) Message-ID: <148.fdb1870.2bd74f18@aol.com> Date: Tue, 22 Apr 2003 22:06:16 EDT Subject: [obm-l] =?ISO-8859-1?Q?Olimp=EDadas=20ao=20redor=20do=20mundo...?= To: obm-l@mat.puc-rio.br MIME-Version: 1.0 Content-Type: multipart/alternative; boundary="part1_148.fdb1870.2bd74f18_boundary" X-Mailer: 7.0 for Windows sub 10501 Sender: owner-obm-l@sucuri.mat.puc-rio.br Precedence: bulk Reply-To: obm-l@mat.puc-rio.br --part1_148.fdb1870.2bd74f18_boundary Content-Type: text/plain; charset="ISO-8859-1" Content-Transfer-Encoding: quoted-printable O=E1 rapaziada...trabalhei nesse problema, achando que fazia a coisa certa,=20 seguindo estritamente os conceitos...tudo parecia l=F3gico e derrepente=20 absurdos apareceram. Fica dificil descrever aqui e por esse motivo vou passa= r=20 o problema a lista e esperar poss=EDveis solu=E7=F5es. Desde j=E1 agrade=E7o= quem por=20 ventura mandar uma id=E9ia ou solu=E7=E3o. Problema: Um polin=F4mio quadr=E1tico de coeficientes inteiros e coeficiente= do=20 segundo grau igual a 1 assume valores primos em tr=EAs valores inteiros e=20 consecutivos. Mostre que ele assume um valor primo em pelo menos mais um=20 valor inteiro. Valeu, Korshin=F3i --part1_148.fdb1870.2bd74f18_boundary Content-Type: text/html; charset="ISO-8859-1" Content-Transfer-Encoding: quoted-printable O=E1 rapaziada...trabalhei nesse problema, achando que= fazia a coisa certa, seguindo estritamente os conceitos...tudo parecia l= =F3gico e derrepente absurdos apareceram. Fica dificil descrever aqui e por=20= esse motivo vou passar o problema a lista e esperar poss=EDveis solu=E7=F5es= . Desde j=E1 agrade=E7o quem por ventura mandar uma id=E9ia ou solu=E7=E3o.<= BR> Problema: Um polin=F4mio quadr=E1tico de coeficientes inteiros e coeficiente= do segundo grau igual a 1 assume valores primos em tr=EAs valores inteiros=20= e consecutivos. Mostre que ele assume um valor primo em pelo menos mais um v= alor inteiro.
      Valeu,
            &nbs= p; Korshin=F3i
--part1_148.fdb1870.2bd74f18_boundary-- ========================================================================= Instruções para entrar na lista, sair da lista e usar a lista em http://www.mat.puc-rio.br/~nicolau/olimp/obm-l.html ========================================================================= From owner-obm-l@sucuri.mat.puc-rio.br Tue Apr 22 23:17:58 2003 Return-Path: Received: (from majordom@localhost) by sucuri.mat.puc-rio.br (8.9.3/8.9.3) id XAA23226 for obm-l-MTTP; Tue, 22 Apr 2003 23:16:37 -0300 Received: from web13001.mail.yahoo.com (web13001.mail.yahoo.com [216.136.174.11]) by sucuri.mat.puc-rio.br (8.9.3/8.9.3) with SMTP id XAA23222 for ; Tue, 22 Apr 2003 23:16:33 -0300 Message-ID: <20030423021601.33834.qmail@web13001.mail.yahoo.com> Received: from [200.148.192.169] by web13001.mail.yahoo.com via HTTP; Tue, 22 Apr 2003 23:16:01 ART Date: Tue, 22 Apr 2003 23:16:01 -0300 (ART) From: =?iso-8859-1?q?Helder=20Suzuki?= Subject: Re: [obm-l] Mais uma coisinha To: obm-l@mat.puc-rio.br In-Reply-To: <200304230022.h3N0MmX9015799@trex.centroin.com.br> MIME-Version: 1.0 Content-Type: text/plain; charset=iso-8859-1 Content-Transfer-Encoding: 8bit Sender: owner-obm-l@sucuri.mat.puc-rio.br Precedence: bulk Reply-To: obm-l@mat.puc-rio.br Ah! É verdade! :) Sou de São Paulo. []'s, Helder Toshiro Suzuki --- Augusto Cesar de Oliveira Morgado escreveu: > Helder, > mensagens do tipo onde achar livro, revista etc. > devem sempre vir acompanhadas da cidade do autor da > mensagem. > Morgado > > > Em Tue, 22 Apr 2003 19:14:39 -0300 (ART), Helder > Suzuki disse: > > > Olá! > > > > Bom, primeiro eu agradeço a todos que responderam > ao > > "4 coisinhas", embora o problema do jogo não foi > lá > > bem muito resolvido e eu não sei aonde achar a tal > > revista hehehe :P > > > > 1) > > João e Maria brincam de multiplicar. > > Antes de começar o jogo, eles escolhem um natural > n>1. > > > > João começa com o número 1 e multiplica-o por um > > natural do intervalo [2, 9], Maria pega o > resultado > > faz o mesmo, e assim sucessivamente. > > > > Ganha o jogo o primeiro que conseguir fazer o > > resultado >= n. > > > > Se João e Maria jogam perfeitamente, e João sempre > > começa o jogo, para quais valores de n João ganha? > e > > Maria? > > > > []'s, > > > > Helder Toshiro Suzuki > > > > > _______________________________________________________________________ > > Yahoo! Mail > > O melhor e-mail gratuito da internet: 6MB de > espaço, antivírus, acesso POP3, filtro contra spam. > > http://br.mail.yahoo.com/ > > > ========================================================================= > > Instruções para entrar na lista, sair da lista e > usar a lista em > > > http://www.mat.puc-rio.br/~nicolau/olimp/obm-l.html > > > ========================================================================= > > > > > > ========================================================================= > Instruções para entrar na lista, sair da lista e > usar a lista em > http://www.mat.puc-rio.br/~nicolau/olimp/obm-l.html > ========================================================================= _______________________________________________________________________ Yahoo! GeoCities Tudo para criar o seu site: ferramentas fáceis de usar, espaço de sobra e acessórios. http://br.geocities.yahoo.com/ ========================================================================= Instruções para entrar na lista, sair da lista e usar a lista em http://www.mat.puc-rio.br/~nicolau/olimp/obm-l.html ========================================================================= From owner-obm-l@sucuri.mat.puc-rio.br Tue Apr 22 23:35:06 2003 Return-Path: Received: (from majordom@localhost) by sucuri.mat.puc-rio.br (8.9.3/8.9.3) id XAA24075 for obm-l-MTTP; Tue, 22 Apr 2003 23:32:27 -0300 Received: from trex-b.centroin.com.br (trex-b.centroin.com.br [200.225.63.136]) by sucuri.mat.puc-rio.br (8.9.3/8.9.3) with ESMTP id XAA24064 for ; Tue, 22 Apr 2003 23:32:22 -0300 Received: from centroin.com.br (RJ119111.user.veloxzone.com.br [200.141.119.111] (may be forged)) (authenticated bits=0) by trex-b.centroin.com.br (8.12.9/8.12.9) with ESMTP id h3N2VjVi029761 for ; Tue, 22 Apr 2003 23:31:46 -0300 (EST) Message-ID: <3EA5FB4C.2040403@centroin.com.br> Date: Tue, 22 Apr 2003 23:32:44 -0300 From: "A. C. Morgado" User-Agent: Mozilla/5.0 (Windows; U; Windows NT 5.0; en-US; rv:1.0.2) Gecko/20030208 Netscape/7.02 X-Accept-Language: en-us, en MIME-Version: 1.0 To: obm-l@mat.puc-rio.br Subject: Re: [obm-l] Mais uma coisinha References: <20030423021601.33834.qmail@web13001.mail.yahoo.com> Content-Type: multipart/alternative; boundary="------------010001090309050106070201" Sender: owner-obm-l@sucuri.mat.puc-rio.br Precedence: bulk Reply-To: obm-l@mat.puc-rio.br --------------010001090309050106070201 Content-Type: text/plain; charset=ISO-8859-1; format=flowed Content-Transfer-Encoding: 8bit A sede da RPM eh no IME/USP. Helder Suzuki wrote: >Ah! >É verdade! :) >Sou de São Paulo. > >[]'s, >Helder Toshiro Suzuki > > --- Augusto Cesar de Oliveira Morgado > escreveu: > Helder, > > >>mensagens do tipo onde achar livro, revista etc. >>devem sempre vir acompanhadas da cidade do autor da >>mensagem. >>Morgado >> >> >>Em Tue, 22 Apr 2003 19:14:39 -0300 (ART), Helder >>Suzuki disse: >> >> >> >>>Olá! >>> >>>Bom, primeiro eu agradeço a todos que responderam >>> >>> >>ao >> >> >>>"4 coisinhas", embora o problema do jogo não foi >>> >>> >>lá >> >> >>>bem muito resolvido e eu não sei aonde achar a tal >>>revista hehehe :P >>> >>>1) >>>João e Maria brincam de multiplicar. >>>Antes de começar o jogo, eles escolhem um natural >>> >>> >>n>1. >> >> >>>João começa com o número 1 e multiplica-o por um >>>natural do intervalo [2, 9], Maria pega o >>> >>> >>resultado >> >> >>>faz o mesmo, e assim sucessivamente. >>> >>>Ganha o jogo o primeiro que conseguir fazer o >>>resultado >= n. >>> >>>Se João e Maria jogam perfeitamente, e João sempre >>>começa o jogo, para quais valores de n João ganha? >>> >>> >>e >> >> >>>Maria? >>> >>>[]'s, >>> >>>Helder Toshiro Suzuki >>> >>> >>> >>> >_______________________________________________________________________ > > >>>Yahoo! Mail >>>O melhor e-mail gratuito da internet: 6MB de >>> >>> >>espaço, antivírus, acesso POP3, filtro contra spam. >> >> >>>http://br.mail.yahoo.com/ >>> >>> >>> >========================================================================= > > >>>Instruções para entrar na lista, sair da lista e >>> >>> >>usar a lista em >> >> >>http://www.mat.puc-rio.br/~nicolau/olimp/obm-l.html >> >> >========================================================================= > > >>> >>> >> >> >========================================================================= > > >>Instruções para entrar na lista, sair da lista e >>usar a lista em >>http://www.mat.puc-rio.br/~nicolau/olimp/obm-l.html >> >> >> >========================================================================= > >_______________________________________________________________________ >Yahoo! GeoCities >Tudo para criar o seu site: ferramentas fáceis de usar, espaço de sobra e acessórios. >http://br.geocities.yahoo.com/ >========================================================================= >Instruções para entrar na lista, sair da lista e usar a lista em >http://www.mat.puc-rio.br/~nicolau/olimp/obm-l.html >========================================================================= > > > > --------------010001090309050106070201 Content-Type: text/html; charset=us-ascii Content-Transfer-Encoding: 7bit A sede da RPM eh no IME/USP.

Helder Suzuki wrote:
Ah!
É verdade! :)
Sou de São Paulo.

[]'s,
Helder Toshiro Suzuki

 --- Augusto Cesar de Oliveira Morgado
<morgado@centroin.com.br> escreveu: > Helder,
  
mensagens do tipo onde achar livro, revista etc.
devem sempre vir acompanhadas da cidade do autor da
mensagem.
Morgado


Em Tue, 22 Apr 2003 19:14:39 -0300 (ART), Helder
Suzuki <heldersuzuki@yahoo.com.br> disse:

    
Olá!

Bom, primeiro eu agradeço a todos que responderam
      
ao
    
"4 coisinhas", embora o problema do jogo não foi
      
bem muito resolvido e eu não sei aonde achar a tal
revista hehehe :P

1)
João e Maria brincam de multiplicar.
Antes de começar o jogo, eles escolhem um natural
      
n>1.
    
João começa com o número 1 e multiplica-o por um
natural do intervalo [2, 9], Maria pega o
      
resultado
    
faz o mesmo, e assim sucessivamente.

Ganha o jogo o primeiro que conseguir fazer o
resultado >= n.

Se João e Maria jogam perfeitamente, e João sempre
começa o jogo, para quais valores de n João ganha?
      
e
    
Maria?

[]'s,

Helder Toshiro Suzuki


      
_______________________________________________________________________
  
Yahoo! Mail
O melhor e-mail gratuito da internet: 6MB de
      
espaço, antivírus, acesso POP3, filtro contra spam. 
    
http://br.mail.yahoo.com/

      
=========================================================================
  
Instruções para entrar na lista, sair da lista e
      
usar a lista em
    
http://www.mat.puc-rio.br/~nicolau/olimp/obm-l.html
    
=========================================================================
  
      
    
=========================================================================
  
Instruções para entrar na lista, sair da lista e
usar a lista em
http://www.mat.puc-rio.br/~nicolau/olimp/obm-l.html

    
========================================================================= 

_______________________________________________________________________
Yahoo! GeoCities
Tudo para criar o seu site: ferramentas fáceis de usar, espaço de sobra e acessórios.
http://br.geocities.yahoo.com/
=========================================================================
Instruções para entrar na lista, sair da lista e usar a lista em
http://www.mat.puc-rio.br/~nicolau/olimp/obm-l.html
=========================================================================


  

--------------010001090309050106070201-- ========================================================================= Instruções para entrar na lista, sair da lista e usar a lista em http://www.mat.puc-rio.br/~nicolau/olimp/obm-l.html ========================================================================= From owner-obm-l@sucuri.mat.puc-rio.br Tue Apr 22 23:44:02 2003 Return-Path: Received: (from majordom@localhost) by sucuri.mat.puc-rio.br (8.9.3/8.9.3) id XAA24461 for obm-l-MTTP; Tue, 22 Apr 2003 23:41:29 -0300 Received: from itaqui.terra.com.br (itaqui.terra.com.br [200.176.3.19]) by sucuri.mat.puc-rio.br (8.9.3/8.9.3) with ESMTP id XAA24457 for ; Tue, 22 Apr 2003 23:41:25 -0300 Received: from barra.terra.com.br (barra.terra.com.br [200.176.3.52]) by itaqui.terra.com.br (Postfix) with ESMTP id 60DE53BCE19 for ; Tue, 22 Apr 2003 23:40:55 -0300 (BRT) Received: from [200.177.176.10] (dl-nas1-sao-C8B1B00A.p001.terra.com.br [200.177.176.10]) by barra.terra.com.br (Postfix) with ESMTP id 5A3AE234051 for ; Tue, 22 Apr 2003 23:40:54 -0300 (BRT) User-Agent: Microsoft-Outlook-Express-Macintosh-Edition/5.02.2022 Date: Tue, 22 Apr 2003 23:42:22 -0300 Subject: Re: [obm-l] TEOIREMA DE CRISTEA::Geometria,pontos estranhos e problemas legais!!!! From: Claudio Buffara To: Message-ID: In-Reply-To: <20030422232555.96652.qmail@web12905.mail.yahoo.com> Mime-version: 1.0 Content-type: multipart/alternative; boundary="MS_Mac_OE_3133899743_933890_MIME_Part" Sender: owner-obm-l@sucuri.mat.puc-rio.br Precedence: bulk Reply-To: obm-l@mat.puc-rio.br > This message is in MIME format. Since your mail reader does not understand this format, some or all of this message may not be legible. --MS_Mac_OE_3133899743_933890_MIME_Part Content-type: text/plain; charset="US-ASCII" Content-transfer-encoding: 7bit on 22.04.03 21:25, Johann Peter Gustav Lejeune Dirichlet at peterdirichlet2002@yahoo.com.br wrote: Ola gente!!!!Estou tentando resolver problemas de Geometria do Mathematical excalibur mas preciso de um teorema.Quem pode demonstrar pra mim: Considere o triangulo ABC e um ponto T.Duas cevianas CT_c e BT_b se cortam erm T e os pontos R_c,R_b e T sao alinhados,com R_c em AB e R_b em AC.Mostre que AT_c*BR_c/T_cB/R_cA + ATb*CR_b//T_bC/R_bA = 1 Nao sei se e isso mesmo mas vale o risco... Oi, JP: Supondo que a expressao acima seja: (AT_c * BR_c) / (BT_c * AR_c) + ( AT_b * CR_b ) / (CT_b * AR_b ) = 1 eu diria que uma boa aposta seria o teorema de Menelau (aplicado mais de uma vez). Um abraco, Claudio. --MS_Mac_OE_3133899743_933890_MIME_Part Content-type: text/html; charset="US-ASCII" Content-transfer-encoding: quoted-printable Re: [obm-l] TEOIREMA DE CRISTEA::Geometria,pontos estranhos e proble= mas legais!!!! on 22.04.03 21:25, Johann Peter Gustav Lejeune Dirichlet at peterdirichlet2= 002@yahoo.com.br wrote:

Ola gente!!!!Estou tentando resolver problemas de Geometria do = Mathematical excalibur mas preciso de um teorema.Quem pode demonstrar pra mi= m:

Considere o triangulo ABC e um ponto T.Duas cevianas CT_c e BT_b se cortam = erm T e os pontos R_c,R_b e T sao alinhados,com R_c em AB e R_b em AC.Mostre= que

AT_c*BR_c/T_cB/R_cA + ATb*CR_b//T_bC/R_bA =3D 1

Nao sei se e isso mesmo mas vale o risco...


Oi, JP:

Supondo que a expressao acima seja:

(AT_c * BR_c) / (BT_c * AR_c)  +  ( AT_b * CR_b ) / (CT_b * AR_b = ) =3D 1

eu diria que uma boa aposta seria o teorema de Menelau (aplicado mais de um= a vez).


Um abraco,
Claudio.

--MS_Mac_OE_3133899743_933890_MIME_Part-- ========================================================================= Instruções para entrar na lista, sair da lista e usar a lista em http://www.mat.puc-rio.br/~nicolau/olimp/obm-l.html ========================================================================= From owner-obm-l@sucuri.mat.puc-rio.br Wed Apr 23 10:32:59 2003 Return-Path: Received: (from majordom@localhost) by sucuri.mat.puc-rio.br (8.9.3/8.9.3) id KAA02769 for obm-l-MTTP; Wed, 23 Apr 2003 10:28:33 -0300 Received: from ivoti.terra.com.br (ivoti.terra.com.br [200.176.3.20]) by sucuri.mat.puc-rio.br (8.9.3/8.9.3) with ESMTP id KAA02762 for ; Wed, 23 Apr 2003 10:28:28 -0300 Received: from canela.terra.com.br (canela.terra.com.br [200.176.3.79]) by ivoti.terra.com.br (Postfix) with ESMTP id 0A921408CE5 for ; Wed, 23 Apr 2003 10:27:58 -0300 (BRT) Received: from [200.177.177.69] (dl-nas1-sao-C8B1B145.p001.terra.com.br [200.177.177.69]) by canela.terra.com.br (Postfix) with ESMTP id 33E25224080 for ; Wed, 23 Apr 2003 10:27:57 -0300 (BRT) User-Agent: Microsoft-Outlook-Express-Macintosh-Edition/5.02.2022 Date: Wed, 23 Apr 2003 10:29:19 -0300 Subject: [obm-l] FW: Problema dificil From: Claudio Buffara To: Lista OBM Message-ID: In-Reply-To: <9A635.57D%buffara1@terra.com.br> Mime-version: 1.0 Content-type: text/plain; charset="US-ASCII" Content-transfer-encoding: 7bit Sender: owner-obm-l@sucuri.mat.puc-rio.br Precedence: bulk Reply-To: obm-l@mat.puc-rio.br Caros colegas da lista: Aqui vai um problema que eu levei algumas semanas pra resolver e, mesmo assim, somente depois que me deram uma dica: Uma sequencia de 100 numeros reais tem a seguinte propriedade: Para cada subsequencia de 8 termos, existe uma subsequencia de 9 termos que tem a mesma media aritmetica que a de 8 termos. Prove que a sequencia eh constante. Daqui a alguns dias eu dou a dica. Tenho muito interesse em ver alguma solucao diferente da minha (e de preferencia mais simples). Um abraco, Claudio. ========================================================================= Instruções para entrar na lista, sair da lista e usar a lista em http://www.mat.puc-rio.br/~nicolau/olimp/obm-l.html ========================================================================= From owner-obm-l@sucuri.mat.puc-rio.br Wed Apr 23 10:32:59 2003 Return-Path: Received: (from majordom@localhost) by sucuri.mat.puc-rio.br (8.9.3/8.9.3) id KAA02775 for obm-l-MTTP; Wed, 23 Apr 2003 10:28:35 -0300 Received: from paiol.terra.com.br (paiol.terra.com.br [200.176.3.18]) by sucuri.mat.puc-rio.br (8.9.3/8.9.3) with ESMTP id KAA02764 for ; Wed, 23 Apr 2003 10:28:29 -0300 Received: from canela.terra.com.br (canela.terra.com.br [200.176.3.79]) by paiol.terra.com.br (Postfix) with ESMTP id 1C4A28817A for ; Wed, 23 Apr 2003 10:27:59 -0300 (BRT) Received: from [200.177.177.69] (dl-nas1-sao-C8B1B145.p001.terra.com.br [200.177.177.69]) by canela.terra.com.br (Postfix) with ESMTP id 2E8CC224080 for ; Wed, 23 Apr 2003 10:27:58 -0300 (BRT) User-Agent: Microsoft-Outlook-Express-Macintosh-Edition/5.02.2022 Date: Wed, 23 Apr 2003 10:29:19 -0300 Subject: FW: [obm-l] Polinomialmente assim... From: Claudio Buffara To: Lista OBM Message-ID: In-Reply-To: <9CC42.58A%claudio.buffara@terra.com.br> Mime-version: 1.0 Content-type: multipart/alternative; boundary="MS_Mac_OE_3133938608_309935_MIME_Part" Sender: owner-obm-l@sucuri.mat.puc-rio.br Precedence: bulk Reply-To: obm-l@mat.puc-rio.br > This message is in MIME format. Since your mail reader does not understand this format, some or all of this message may not be legible. --MS_Mac_OE_3133938608_309935_MIME_Part Content-type: text/plain; charset="US-ASCII" Content-transfer-encoding: 7bit on 22.04.03 20:33, Johann Peter Gustav Lejeune Dirichlet at peterdirichlet2002@yahoo.com.br wrote: Oi turma,estou as voltas com esse aqui,vejam: Qual o produto e a soma das raizes POSITIVAS de (x^2+2x-12)^2=9x^3-108x ? Oi, JP: A equacao eh: (x^2+2x-12)^2 = 9x(x^2-12) Faca y = x^2 - 12. Entao: (y+2x)^2 = 9xy ==> y^2 + 4x^2 + 4xy = 9xy ==> y^2 - 5xy + 4x^2 = 0 ==> y^2 - 2*(5x/2)*y + 25x^2/4 - 9x^2/4 = 0 ==> (y - 5x/2)^2 - (3x/2)^2 = 0 ==> (y - 4x)(y - x) = 0 ==> y = 4x ou y = x ==> x^2 - 4x - 12 = 0 ou x^2 - x - 12 = 0 ==> x = -2 ou x = 6 ou x = 4 ou x = - 3 ==> as raizes positivas sao 4 e 6 ==> Produto = 24 e Soma = 10 Um abraco, Claudio. --MS_Mac_OE_3133938608_309935_MIME_Part Content-type: text/html; charset="US-ASCII" Content-transfer-encoding: quoted-printable FW: [obm-l] Polinomialmente assim...
on 22.04.03 20:33, Johann Peter Gustav Lejeune Dirichlet at peterdirichlet2= 002@yahoo.com.br wrote:

Oi turma,estou as voltas com esse aqui,vejam:
Qual o produto e a soma das raizes POSITIVAS de (x^2+2x-12)^2=3D9x^3-108x ?

Oi, JP:

A equacao eh:
(x^2+2x-12)^2 =3D 9x(x^2-12)

Faca y =3D x^2 - 12. Entao:

(y+2x)^2 =3D 9xy =3D=3D>
y^2 + 4x^2 + 4xy =3D 9xy =3D=3D>
y^2 - 5xy + 4x^2 =3D 0 =3D=3D>
y^2 - 2*(5x/2)*y + 25x^2/4 - 9x^2/4 =3D 0 =3D=3D>
(y - 5x/2)^2 - (3x/2)^2 =3D 0 =3D=3D>
(y - 4x)(y - x) =3D 0 =3D=3D>
y =3D 4x   ou   y =3D x =3D=3D>
x^2 - 4x - 12 =3D 0   ou   x^2 - x - 12 =3D 0 =3D=3D>
x =3D -2  ou  x =3D 6  ou  x =3D 4  ou  x =3D - 3 =3D=3D&= gt;
as raizes positivas sao 4 e 6 =3D=3D>

Produto =3D 24  e  Soma =3D 10

Um abraco,
Claudio.


--MS_Mac_OE_3133938608_309935_MIME_Part-- ========================================================================= Instruções para entrar na lista, sair da lista e usar a lista em http://www.mat.puc-rio.br/~nicolau/olimp/obm-l.html ========================================================================= From owner-obm-l@sucuri.mat.puc-rio.br Wed Apr 23 10:55:24 2003 Return-Path: Received: (from majordom@localhost) by sucuri.mat.puc-rio.br (8.9.3/8.9.3) id KAA03455 for obm-l-MTTP; Wed, 23 Apr 2003 10:52:17 -0300 Received: from Euler.impa.br (euler.impa.br [147.65.1.3]) by sucuri.mat.puc-rio.br (8.9.3/8.9.3) with ESMTP id KAA03450 for ; Wed, 23 Apr 2003 10:52:13 -0300 Received: from sbm.impa.br (obm-01.impa.br [147.65.2.170]) by Euler.impa.br (8.11.6p2/8.11.6) with ESMTP id h3NDpg011419 for ; Wed, 23 Apr 2003 10:51:42 -0300 (EST) Message-Id: <5.0.0.25.0.20030423224524.00a09370@pop.impa.br> X-Sender: obm@pop.impa.br X-Mailer: QUALCOMM Windows Eudora Version 5.0 Date: Wed, 23 Apr 2003 22:52:27 -0700 To: obm-l@mat.puc-rio.br From: Olimpiada Brasileira de Matematica Subject: [obm-l] Materia sobre Olimpiadas. In-Reply-To: <5.2.0.9.0.20030421201915.00b1b518@mail.digi.com.br> References: <20030421193036.D6341@sucuri.mat.puc-rio.br> Mime-Version: 1.0 Content-Type: text/plain; charset="us-ascii"; format=flowed Sender: owner-obm-l@sucuri.mat.puc-rio.br Precedence: bulk Reply-To: obm-l@mat.puc-rio.br Caros(as) amigos(as) da lista, Foi publicada uma materia sobre a OBM na Folha Dirigida, os interessados podem dar uma olhada no seguinte endereco: http://www.folhadirigida.com.br/educacao/FdgDetPrm2003_04_22.htm Abracos, Nelly. ========================================================================= Instruções para entrar na lista, sair da lista e usar a lista em http://www.mat.puc-rio.br/~nicolau/olimp/obm-l.html ========================================================================= From owner-obm-l@sucuri.mat.puc-rio.br Wed Apr 23 13:13:08 2003 Return-Path: Received: (from majordom@localhost) by sucuri.mat.puc-rio.br (8.9.3/8.9.3) id NAA06216 for obm-l-MTTP; Wed, 23 Apr 2003 13:10:07 -0300 Received: from ns3bind.localdomain ([200.230.34.5]) by sucuri.mat.puc-rio.br (8.9.3/8.9.3) with ESMTP id NAA06212 for ; Wed, 23 Apr 2003 13:10:04 -0300 Received: from servico2 ([200.230.34.229]) by ns3bind.localdomain (8.11.6/X.XX.X) with SMTP id h3NG5Qu23042 for ; Wed, 23 Apr 2003 13:05:26 -0300 Message-ID: <002101c309b2$e73bf200$3300c57d@bovespa.com> From: "=?iso-8859-1?Q?Cl=E1udio_\=28Pr=E1tica\=29?=" To: References: <20030422230225.98447.qmail@web12907.mail.yahoo.com> Subject: Re: [obm-l] Combinatoria e |Desordens Date: Wed, 23 Apr 2003 13:10:46 -0300 MIME-Version: 1.0 Content-Type: multipart/alternative; boundary="----=_NextPart_000_001E_01C30999.C0D16320" X-Priority: 3 X-MSMail-Priority: Normal X-Mailer: Microsoft Outlook Express 5.50.4920.2300 X-MimeOLE: Produced By Microsoft MimeOLE V5.50.4920.2300 Sender: owner-obm-l@sucuri.mat.puc-rio.br Precedence: bulk Reply-To: obm-l@mat.puc-rio.br This is a multi-part message in MIME format. ------=_NextPart_000_001E_01C30999.C0D16320 Content-Type: text/plain; charset="iso-8859-1" Content-Transfer-Encoding: quoted-printable Oi, JP: Segue abaixo um pequeno artigo que, inicialmente, d=E1 uma = demonstra=E7=E3o combinat=F3ria da seguinte equa=E7=E3o de = recorr=EAncia: K(1) =3D 0 K(2) =3D 1 K(n) =3D (n-1)*[K(n-1) + K(n-2)], para n >=3D 3 e em seguida, prova a sua por indu=E7=E3o, com base na anterior. http://www.unc.edu/~rowlett/combin/notes/Derangements.pdf Espero que isso resolva. Um abra=E7o, Claudio. ----- Original Message -----=20 From: Johann Peter Gustav Lejeune Dirichlet=20 To: obm-l@mat.puc-rio.br=20 Sent: Tuesday, April 22, 2003 8:02 PM Subject: [obm-l] Combinatoria e |Desordens Oi genten!!!!E ai,se alguem manja de permuta=E7ao caotica(sem pontos = fixos) de (1,2,3,...,n),se chamarmos K_n o numero de jeitos de permutar = caoticamente,sera que da pra demonstrar COMBINATRORIALMENTE que=20 K_n=3Dn*K_(n-1)+(-1)^n? -------------------------------------------------------------------------= ----- Yahoo! Mail=20 O melhor e-mail gratuito da internet: 6MB de espa=E7o, antiv=EDrus, = acesso POP3, filtro contra spam. ------=_NextPart_000_001E_01C30999.C0D16320 Content-Type: text/html; charset="iso-8859-1" Content-Transfer-Encoding: quoted-printable
Oi, JP:
 
Segue abaixo um pequeno artigo que, = inicialmente,=20 d=E1 uma demonstra=E7=E3o combinat=F3ria da seguinte equa=E7=E3o de=20 recorr=EAncia:
 
K(1) =3D 0
K(2) =3D 1
K(n) =3D (n-1)*[K(n-1) + K(n-2)], para = n >=3D=20 3
 
e em seguida, prova a sua por = indu=E7=E3o, com base na=20 anterior.
 
http:/= /www.unc.edu/~rowlett/combin/notes/Derangements.pdf
 
Espero que isso resolva.
 
Um abra=E7o,
Claudio.
 
----- Original Message -----
From:=20 Johann Peter Gustav = Lejeune=20 Dirichlet
Sent: Tuesday, April 22, 2003 = 8:02=20 PM
Subject: [obm-l] Combinatoria e = |Desordens

Oi genten!!!!E ai,se alguem manja de permuta=E7ao caotica(sem = pontos fixos)=20 de (1,2,3,...,n),se chamarmos K_n o numero de jeitos de permutar=20 caoticamente,sera que da pra demonstrar COMBINATRORIALMENTE que =
K_n=3Dn*K_(n-1)+(-1)^n?



Yahoo! Mail
O = melhor e-mail=20 gratuito da internet: 6MB de espa=E7o, antiv=EDrus, acesso POP3, = filtro contra=20 spam. ------=_NextPart_000_001E_01C30999.C0D16320-- ========================================================================= Instruções para entrar na lista, sair da lista e usar a lista em http://www.mat.puc-rio.br/~nicolau/olimp/obm-l.html ========================================================================= From owner-obm-l@sucuri.mat.puc-rio.br Wed Apr 23 13:55:10 2003 Return-Path: Received: (from majordom@localhost) by sucuri.mat.puc-rio.br (8.9.3/8.9.3) id NAA07220 for obm-l-MTTP; Wed, 23 Apr 2003 13:50:22 -0300 Received: (from nicolau@localhost) by sucuri.mat.puc-rio.br (8.9.3/8.9.3) id NAA07215 for obm-l@mat.puc-rio.br; Wed, 23 Apr 2003 13:50:22 -0300 Date: Wed, 23 Apr 2003 13:50:22 -0300 From: "Nicolau C. Saldanha" To: obm-l@mat.puc-rio.br Subject: [obm-l] Re: =?iso-8859-1?Q?=5Bobm-l=5D_Olimp=EDadas_ao_redor_do_mundo=2E=2E=2E?= Message-ID: <20030423135022.A5984@sucuri.mat.puc-rio.br> References: <148.fdb1870.2bd74f18@aol.com> Mime-Version: 1.0 Content-Type: text/plain; charset=iso-8859-1 Content-Disposition: inline Content-Transfer-Encoding: 8bit User-Agent: Mutt/1.2.5i In-Reply-To: <148.fdb1870.2bd74f18@aol.com>; from Korshinoi@aol.com on Tue, Apr 22, 2003 at 10:06:16PM -0400 Sender: owner-obm-l@sucuri.mat.puc-rio.br Precedence: bulk Reply-To: obm-l@mat.puc-rio.br On Tue, Apr 22, 2003 at 10:06:16PM -0400, Korshinoi@aol.com wrote: > Problema: Um polinômio quadrático de coeficientes inteiros e coeficiente do > segundo grau igual a 1 assume valores primos em três valores inteiros e > consecutivos. Mostre que ele assume um valor primo em pelo menos mais um > valor inteiro. O problema é um pouco enganador, parece que está perguntando algo difícil mas... O polinômio é da forma P(x) = x^2 + bx + c. Vamos dividir em casos: b ímpar: A identidade P(-b-x) = P(x) garante que cada valor de P(n) aparece exatamente duas vezes. Se temos três valores inteiros de n para os quais P(n) é primo tem que existir um quarto. b par: Transladando em x podemos supor que os três valores primos são P(-1), P(0), P(1). Observe que P(x+1) - P(x) = 2x + 1 + b, (P(x+2) - P(x+1)) - (P(x+1) - P(x)) = 2. P(n) alterna entre valores pares e ímpares. A única forma de termos 3 primos consecutivos é portanto, usando as observações acima, P(-1) = 3, P(0) = 2, P(1) = 3 Neste caso P(x) = x^2 + 2 e P(3) = 11. []s, N. ========================================================================= Instruções para entrar na lista, sair da lista e usar a lista em http://www.mat.puc-rio.br/~nicolau/olimp/obm-l.html ========================================================================= From owner-obm-l@sucuri.mat.puc-rio.br Wed Apr 23 14:20:32 2003 Return-Path: Received: (from majordom@localhost) by sucuri.mat.puc-rio.br (8.9.3/8.9.3) id OAA07928 for obm-l-MTTP; Wed, 23 Apr 2003 14:17:25 -0300 Received: from mail.fronthost.com (mail.fronthost.com [63.250.6.253]) by sucuri.mat.puc-rio.br (8.9.3/8.9.3) with ESMTP id OAA07923 for ; Wed, 23 Apr 2003 14:17:20 -0300 Received: from [68.49.44.134] by fronthost.com [63.250.6.253] with SmartMax MailMax for at Wed, 23 Apr 2003 13:17:06 -0400 Message-ID: <007d01c309bc$18124ce0$6401a8c0@TEST4> From: "Alexandre A da Rocha" To: References: <20030421193036.D6341@sucuri.mat.puc-rio.br> <5.0.0.25.0.20030423224524.00a09370@pop.impa.br> Subject: Re: [obm-l] Materia sobre Olimpiadas. Date: Wed, 23 Apr 2003 13:16:32 -0400 MIME-Version: 1.0 Content-Type: multipart/alternative; boundary="----=_NextPart_000_007A_01C3099A.8F20F060" X-Priority: 3 X-MSMail-Priority: Normal X-Mailer: Microsoft Outlook Express 5.50.4807.1700 X-MIMEOLE: Produced By Microsoft MimeOLE V5.50.4807.1700 Sender: owner-obm-l@sucuri.mat.puc-rio.br Precedence: bulk Reply-To: obm-l@mat.puc-rio.br This is a multi-part message in MIME format. ------=_NextPart_000_007A_01C3099A.8F20F060 Content-Type: text/plain; charset="iso-8859-1" Content-Transfer-Encoding: quoted-printable Ao clicar no link 'O sonho de ouro'... [snip] "Na Olimp=EDada Internacional de Matem=E1tica, considerada = competi=E7=E3o mais importante da =E1rea, os brasileiros tamb=E9m t=EAm = presen=E7a marcante. O pa=EDs participa desde 1979 e ganhou sua primeira = medalha (de ouro!) j=E1 em 1981, com Nicolau Cor=E7=E3o Saldanha. Desde = ent=E3o, foram 52 medalhas, sendo 6 de ouro, 8 de prata e 38 de bronze, = al=E9m de 15 men=E7=F5es honrosas (uma esp=E9cie de medalha de 4=BA = lugar)." [snip] Foi inda outro dia que mandei um email pro Nicolau perguntando uma = besteira qualquer e que tomei conhecimento dessa lista... nao sabia que = deveria o ter tratato por Sua Excelencia... very impressive. :), Auggy ----- Original Message -----=20 From: "Olimpiada Brasileira de Matematica" To: Sent: Thursday, April 24, 2003 1:52 AM Subject: [obm-l] Materia sobre Olimpiadas. Caros(as) amigos(as) da lista, Foi publicada uma materia sobre a OBM na Folha Dirigida, os interessados podem dar uma olhada no seguinte endereco: http://www.folhadirigida.com.br/educacao/FdgDetPrm2003_04_22.htm Abracos, Nelly. =3D=3D=3D=3D=3D=3D=3D=3D=3D=3D=3D=3D=3D=3D=3D=3D=3D=3D=3D=3D=3D=3D=3D=3D=3D= =3D=3D=3D=3D=3D=3D=3D=3D=3D=3D=3D=3D=3D=3D=3D=3D=3D=3D=3D=3D=3D=3D=3D=3D=3D= =3D=3D=3D=3D=3D=3D=3D=3D=3D=3D=3D=3D=3D=3D=3D=3D=3D=3D=3D=3D=3D=3D=3D Instru=E7=F5es para entrar na lista, sair da lista e usar a lista em http://www.mat.puc-rio.br/~nicolau/olimp/obm-l.html =3D=3D=3D=3D=3D=3D=3D=3D=3D=3D=3D=3D=3D=3D=3D=3D=3D=3D=3D=3D=3D=3D=3D=3D=3D= =3D=3D=3D=3D=3D=3D=3D=3D=3D=3D=3D=3D=3D=3D=3D=3D=3D=3D=3D=3D=3D=3D=3D=3D=3D= =3D=3D=3D=3D=3D=3D=3D=3D=3D=3D=3D=3D=3D=3D=3D=3D=3D=3D=3D=3D=3D=3D=3D ------=_NextPart_000_007A_01C3099A.8F20F060 Content-Type: text/html; charset="iso-8859-1" Content-Transfer-Encoding: quoted-printable
Ao clicar no link 'O sonho de=20 ouro'...
 
[snip]
"Na Olimp=EDada Internacional de = Matem=E1tica,=20 considerada competi=E7=E3o mais importante da =E1rea, os brasileiros = tamb=E9m t=EAm=20 presen=E7a marcante. O pa=EDs participa desde 1979 e ganhou sua primeira = medalha (de=20 ouro!) j=E1 em 1981, com Nicolau Cor=E7=E3o Saldanha. = Desde ent=E3o,=20 foram 52 medalhas, sendo 6 de ouro, 8 de prata e 38 de bronze, al=E9m de = 15=20 men=E7=F5es honrosas (uma esp=E9cie de medalha de 4=BA = lugar)."
[snip]
 
Foi inda outro dia que mandei um = email pro=20 Nicolau perguntando uma besteira qualquer e que tomei conhecimento dessa = lista... nao sabia que deveria o ter tratato por Sua Excelencia... very=20 impressive.
:),
Auggy
 
 
----- Original Message -----
From: "Olimpiada Brasileira de = Matematica"=20 <obm@impa.br>
To: <obm-l@mat.puc-rio.br>
Sent: Thursday, April 24, 2003 1:52=20 AM
Subject: [obm-l] Materia sobre=20 Olimpiadas.

Caros(as) amigos(as) da lista,

Foi publicada uma materia = sobre a=20 OBM na Folha Dirigida,
os interessados podem dar uma olhada no = seguinte=20 endereco:

http://www.folhadirigida.com.br/educacao/FdgDetPrm2003_04_22.htm=


Abracos,=20 Nelly.

=3D=3D=3D=3D=3D=3D=3D=3D=3D=3D=3D=3D=3D=3D=3D=3D=3D=3D=3D=3D= =3D=3D=3D=3D=3D=3D=3D=3D=3D=3D=3D=3D=3D=3D=3D=3D=3D=3D=3D=3D=3D=3D=3D=3D=3D= =3D=3D=3D=3D=3D=3D=3D=3D=3D=3D=3D=3D=3D=3D=3D=3D=3D=3D=3D=3D=3D=3D=3D=3D=3D= =3D=3D=3D
Instru=E7=F5es=20 para entrar na lista, sair da lista e usar a lista em
http://www.mat.puc-rio.br/~nicolau/olimp/obm-l.html=3D=3D=3D=3D=3D=3D=3D=3D=3D=3D=3D=3D=3D=3D=3D=3D=3D=3D=3D=3D=3D=3D= =3D=3D=3D=3D=3D=3D=3D=3D=3D=3D=3D=3D=3D=3D=3D=3D=3D=3D=3D=3D=3D=3D=3D=3D=3D= =3D=3D=3D=3D=3D=3D=3D=3D=3D=3D=3D=3D=3D=3D=3D=3D=3D=3D=3D=3D=3D=3D=3D=3D=3D= =3D
------=_NextPart_000_007A_01C3099A.8F20F060-- ========================================================================= Instruções para entrar na lista, sair da lista e usar a lista em http://www.mat.puc-rio.br/~nicolau/olimp/obm-l.html ========================================================================= From owner-obm-l@sucuri.mat.puc-rio.br Wed Apr 23 17:30:21 2003 Return-Path: Received: (from majordom@localhost) by sucuri.mat.puc-rio.br (8.9.3/8.9.3) id RAA11069 for obm-l-MTTP; Wed, 23 Apr 2003 17:28:30 -0300 Received: from web14305.mail.yahoo.com (web14305.mail.yahoo.com [216.136.173.81]) by sucuri.mat.puc-rio.br (8.9.3/8.9.3) with SMTP id RAA11065 for ; Wed, 23 Apr 2003 17:28:26 -0300 Message-ID: <20030423202754.21166.qmail@web14305.mail.yahoo.com> Received: from [200.17.25.3] by web14305.mail.yahoo.com via HTTP; Wed, 23 Apr 2003 17:27:54 ART Date: Wed, 23 Apr 2003 17:27:54 -0300 (ART) From: =?iso-8859-1?q?Rafael?= Subject: [obm-l] circunferências To: OBM MIME-Version: 1.0 Content-Type: text/plain; charset=iso-8859-1 Content-Transfer-Encoding: 8bit Sender: owner-obm-l@sucuri.mat.puc-rio.br Precedence: bulk Reply-To: obm-l@mat.puc-rio.br Olá pessoas! Esse aqui tá difícil! Uma circunferência de raio 1m tem o seu centro distante 0,1m de uma segunda circunferência, e divide esta em duas partes iguais. Calcular a área do círculo correspondente à segunda circunferência. Resposta 1,3485 m² ou 1,7914 m² Se alguém puder me ajudar... Abraços, Rafael. _______________________________________________________________________ Yahoo! Mail O melhor e-mail gratuito da internet: 6MB de espaço, antivírus, acesso POP3, filtro contra spam. http://br.mail.yahoo.com/ ========================================================================= Instruções para entrar na lista, sair da lista e usar a lista em http://www.mat.puc-rio.br/~nicolau/olimp/obm-l.html ========================================================================= From owner-obm-l@sucuri.mat.puc-rio.br Wed Apr 23 17:32:34 2003 Return-Path: Received: (from majordom@localhost) by sucuri.mat.puc-rio.br (8.9.3/8.9.3) id RAA11149 for obm-l-MTTP; Wed, 23 Apr 2003 17:31:10 -0300 Received: from ivoti.terra.com.br (ivoti.terra.com.br [200.176.3.20]) by sucuri.mat.puc-rio.br (8.9.3/8.9.3) with ESMTP id RAA11145 for ; Wed, 23 Apr 2003 17:31:06 -0300 Received: from gunga.terra.com.br (gunga.terra.com.br [200.176.3.45]) by ivoti.terra.com.br (Postfix) with ESMTP id A245C408E6F for ; Wed, 23 Apr 2003 17:30:34 -0300 (BRT) Received: from niski.com (unknown [200.148.194.69]) (authenticated user fniski) by gunga.terra.com.br (Postfix) with ESMTP id 461931280DC for ; Wed, 23 Apr 2003 17:30:33 -0300 (BRT) Message-ID: <3EA6E9DE.40006@niski.com> Date: Wed, 23 Apr 2003 12:30:38 -0700 From: niski User-Agent: Mozilla/5.0 (Windows; U; Windows NT 5.1; en-US; rv:1.0.2) Gecko/20030208 Netscape/7.02 X-Accept-Language: en-us, en MIME-Version: 1.0 To: obm-l@mat.puc-rio.br Subject: [obm-l] simples de variaveis aleatorias Content-Type: text/plain; charset=ISO-8859-1; format=flowed Content-Transfer-Encoding: 8bit Sender: owner-obm-l@sucuri.mat.puc-rio.br Precedence: bulk Reply-To: obm-l@mat.puc-rio.br Olá pessoal da lista. Gostaria que me explicassem o que fiz de errado neste problema. "Duas moedas estão sobre a mesa, uma delas tem duas caras e a outra tem probabilidade igual de cara e coroa. Sorteamos, ao acaso, uma dessas moedas e a lançamos duas vezes. Seja X a variável aleatória que conta o número de caras nos dois lançamentos. Qual a esperança de X? Qual a variância de X? R: 1,5 e 9/16" Bem, fiz o calculos e conclui que p(X=0) = 1/8 p(X=1) = 2/8 p(X=2) = 5/8 Então, a esperança é dada por : E(X) = X[0].p(X=0) + X[1].p(X=1) + X[2].p(X=2) E(X) = 0.(1/8) + 1.(2/8) + 2.(5/8) E(X) = 1,5 A variancia de X é dada por: V(X) = E(X^2) - [E(X)]^2 Para o calculo de E(X^2) eu procedi assim : Se X assume os valores 0 , 1 e 2 X^2 assumirá os valores 0, 1 e 4 porem e as probabilidades associadas a estes valores, mudam!? De inicio, calculei como se não mudassem: E(X^2) = 0.(1/8) + 1.(2/8) + 4.(5/8) E(X^2) = 11/4 Então V(X) = 11/4 - 9/4 V(X) = 2/4 = 0,5 O que não bate com a resposta do gabarito. Depois, fiquei pensando que se a variavel X assume o valor 4 e a variavel X é o numero de caras, então certamente para sair 4 caras a probabilidade deve mudar, mas o numero de lançamento de moedas é constante ("...lançamos duas vezes") e igual a 2 então como conseguir 4 caras?! Fiquei travado nesse aparente "paradoxo" que construi sem querer. Aguardo do pessoal da lista alguma ajuda na resolução do exercicio Niski www.linux.ime.usp.br/~niski ========================================================================= Instruções para entrar na lista, sair da lista e usar a lista em http://www.mat.puc-rio.br/~nicolau/olimp/obm-l.html ========================================================================= From owner-obm-l@sucuri.mat.puc-rio.br Wed Apr 23 17:56:09 2003 Return-Path: Received: (from majordom@localhost) by sucuri.mat.puc-rio.br (8.9.3/8.9.3) id RAA11696 for obm-l-MTTP; Wed, 23 Apr 2003 17:54:47 -0300 Received: from trex-b.centroin.com.br (trex-b.centroin.com.br [200.225.63.136]) by sucuri.mat.puc-rio.br (8.9.3/8.9.3) with ESMTP id RAA11693 for ; Wed, 23 Apr 2003 17:54:44 -0300 Received: from centroin.com.br (RJ118229.user.veloxzone.com.br [200.141.118.229] (may be forged)) (authenticated bits=0) by trex-b.centroin.com.br (8.12.9/8.12.9) with ESMTP id h3NKsCVi016219 for ; Wed, 23 Apr 2003 17:54:13 -0300 (EST) Message-ID: <3EA6FDB1.8000704@centroin.com.br> Date: Wed, 23 Apr 2003 17:55:13 -0300 From: "A. C. Morgado" User-Agent: Mozilla/5.0 (Windows; U; Windows NT 5.0; en-US; rv:1.0.2) Gecko/20030208 Netscape/7.02 X-Accept-Language: en-us, en MIME-Version: 1.0 To: obm-l@mat.puc-rio.br Subject: Re: [obm-l] simples de variaveis aleatorias References: <3EA6E9DE.40006@niski.com> Content-Type: text/plain; charset=ISO-8859-1; format=flowed Content-Transfer-Encoding: 8bit Sender: owner-obm-l@sucuri.mat.puc-rio.br Precedence: bulk Reply-To: obm-l@mat.puc-rio.br Voce estah certo e o seu gabarito, errado. niski wrote: > Olá pessoal da lista. Gostaria que me explicassem o que fiz de errado > neste problema. > > "Duas moedas estão sobre a mesa, uma delas tem duas caras e a outra > tem probabilidade igual de cara e coroa. Sorteamos, ao acaso, uma > dessas moedas e a lançamos duas vezes. Seja X a variável aleatória que > conta o número de caras nos dois lançamentos. Qual a esperança de X? > Qual a variância de X? R: 1,5 e 9/16" > > Bem, fiz o calculos e conclui que > p(X=0) = 1/8 > p(X=1) = 2/8 > p(X=2) = 5/8 > > Então, a esperança é dada por : > E(X) = X[0].p(X=0) + X[1].p(X=1) + X[2].p(X=2) > E(X) = 0.(1/8) + 1.(2/8) + 2.(5/8) > E(X) = 1,5 > > A variancia de X é dada por: > V(X) = E(X^2) - [E(X)]^2 > Para o calculo de E(X^2) eu procedi assim : > Se X assume os valores 0 , 1 e 2 X^2 assumirá os valores 0, 1 e 4 > porem e as probabilidades associadas a estes valores, mudam!? > De inicio, calculei como se não mudassem: > E(X^2) = 0.(1/8) + 1.(2/8) + 4.(5/8) > E(X^2) = 11/4 > > Então > V(X) = 11/4 - 9/4 > V(X) = 2/4 = 0,5 > > O que não bate com a resposta do gabarito. > > Depois, fiquei pensando que se a variavel X assume o valor 4 e a > variavel X é o numero de caras, então certamente para sair 4 caras a > probabilidade deve mudar, mas o numero de lançamento de moedas é > constante ("...lançamos duas vezes") e igual a 2 então como conseguir > 4 caras?! > > Fiquei travado nesse aparente "paradoxo" que construi sem querer. > > Aguardo do pessoal da lista alguma ajuda na resolução do exercicio > > Niski > www.linux.ime.usp.br/~niski > > ========================================================================= > Instruções para entrar na lista, sair da lista e usar a lista em > http://www.mat.puc-rio.br/~nicolau/olimp/obm-l.html > ========================================================================= > > ========================================================================= Instruções para entrar na lista, sair da lista e usar a lista em http://www.mat.puc-rio.br/~nicolau/olimp/obm-l.html ========================================================================= From owner-obm-l@sucuri.mat.puc-rio.br Wed Apr 23 18:42:22 2003 Return-Path: Received: (from majordom@localhost) by sucuri.mat.puc-rio.br (8.9.3/8.9.3) id SAA13029 for obm-l-MTTP; Wed, 23 Apr 2003 18:40:38 -0300 Received: from ivoti.terra.com.br (ivoti.terra.com.br [200.176.3.20]) by sucuri.mat.puc-rio.br (8.9.3/8.9.3) with ESMTP id SAA13025 for ; Wed, 23 Apr 2003 18:40:35 -0300 Received: from jurere.terra.com.br (jurere.terra.com.br [200.176.3.49]) by ivoti.terra.com.br (Postfix) with ESMTP id 2DAB4408923 for ; Wed, 23 Apr 2003 18:40:01 -0300 (BRT) Received: from riemann.localdomain (RJ170210.user.veloxzone.com.br [200.149.170.210]) (authenticated user fabio.dias.moreira) by jurere.terra.com.br (Postfix) with ESMTP id B3E081380CC for ; Wed, 23 Apr 2003 18:40:00 -0300 (BRT) Content-Type: text/plain; charset="iso-8859-1" From: =?iso-8859-1?q?F=E1bio=20Dias=20Moreira?= To: obm-l@mat.puc-rio.br Subject: Re: [obm-l] =?iso-8859-1?q?circunfer=EAncias?= Date: Wed, 23 Apr 2003 18:39:27 -0300 User-Agent: KMail/1.4.3 References: <20030423202754.21166.qmail@web14305.mail.yahoo.com> In-Reply-To: <20030423202754.21166.qmail@web14305.mail.yahoo.com> MIME-Version: 1.0 Content-Transfer-Encoding: 8bit Message-Id: <200304231840.04725.fabio.dias.moreira@terra.com.br> Sender: owner-obm-l@sucuri.mat.puc-rio.br Precedence: bulk Reply-To: obm-l@mat.puc-rio.br -----BEGIN PGP SIGNED MESSAGE----- Hash: SHA1 On Wednesday 23 April 2003 17:27, Rafael wrote: > [...] > Uma circunferência de raio 1m tem o seu centro > distante 0,1m de uma segunda circunferência, e divide > esta em duas partes iguais. Calcular a área do círculo > correspondente à segunda circunferência. > [...] Existem duas possibilidades para o centro O da primeira circunferência: ele está dentro da segunda circunferência ou fora da segunda circunhferência. Seja O' o centro da segunda circunferência e P um dos pontos de contato. Caso I -- O dentro de O'P: Então OO'P é retângulo de lados R-0,1, R e 10 (hipotenusa). Logo R^2 - 0,2R + 0,01 + R^2 = 1 <==> R = 0,76 m <==> S = 1,79 m^2 Caso II -- O fora de O'P: Então OO'P é retângulo de lados R+0,1, R e 10 (hipotenusa). Analogamente, S = 1,35 m^2. []s, - -- Fábio "ctg \pi" Dias Moreira -----BEGIN PGP SIGNATURE----- Version: GnuPG v1.0.6 (GNU/Linux) Comment: For info see http://www.gnupg.org iD8DBQE+pwg0alOQFrvzGQoRAqgZAJ94YaK/T4lZxlh6wcZ1XgM68eZpRQCgipIx u324fM9W2iQoT+PP4KLTQBU= =+QE2 -----END PGP SIGNATURE----- ========================================================================= Instruções para entrar na lista, sair da lista e usar a lista em http://www.mat.puc-rio.br/~nicolau/olimp/obm-l.html ========================================================================= From owner-obm-l@sucuri.mat.puc-rio.br Wed Apr 23 19:40:11 2003 Return-Path: Received: (from majordom@localhost) by sucuri.mat.puc-rio.br (8.9.3/8.9.3) id TAA14579 for obm-l-MTTP; Wed, 23 Apr 2003 19:38:45 -0300 Received: from web14304.mail.yahoo.com (web14304.mail.yahoo.com [216.136.173.80]) by sucuri.mat.puc-rio.br (8.9.3/8.9.3) with SMTP id TAA14574 for ; Wed, 23 Apr 2003 19:38:41 -0300 Message-ID: <20030423223809.4152.qmail@web14304.mail.yahoo.com> Received: from [200.144.49.43] by web14304.mail.yahoo.com via HTTP; Wed, 23 Apr 2003 19:38:09 ART Date: Wed, 23 Apr 2003 19:38:09 -0300 (ART) From: =?iso-8859-1?q?Rafael?= Subject: [obm-l] triangulo e bissetrizes To: obm-l@mat.puc-rio.br In-Reply-To: <200304231840.04725.fabio.dias.moreira@terra.com.br> MIME-Version: 1.0 Content-Type: text/plain; charset=iso-8859-1 Content-Transfer-Encoding: 8bit Sender: owner-obm-l@sucuri.mat.puc-rio.br Precedence: bulk Reply-To: obm-l@mat.puc-rio.br Valeu Fabio pelo problema das circunferências. Agora tem esse aqui também: ABC é um triângulo cujas medidas dos lados são a,b,c e M ,N,P são os pés das três bissetrizes internas desse triângulo. Achar a razão entre as áreas dos triângulos MNP e ABC resposta: 2abc/(a+b)(a+c)(b+c) Alguém me ajuda? Abraços, Rafael. _______________________________________________________________________ Yahoo! Mail O melhor e-mail gratuito da internet: 6MB de espaço, antivírus, acesso POP3, filtro contra spam. http://br.mail.yahoo.com/ ========================================================================= Instruções para entrar na lista, sair da lista e usar a lista em http://www.mat.puc-rio.br/~nicolau/olimp/obm-l.html ========================================================================= From owner-obm-l@sucuri.mat.puc-rio.br Wed Apr 23 19:52:01 2003 Return-Path: Received: (from majordom@localhost) by sucuri.mat.puc-rio.br (8.9.3/8.9.3) id TAA14842 for obm-l-MTTP; Wed, 23 Apr 2003 19:50:36 -0300 Received: from traven9.uol.com.br (traven9.uol.com.br [200.221.29.35]) by sucuri.mat.puc-rio.br (8.9.3/8.9.3) with ESMTP id TAA14838 for ; Wed, 23 Apr 2003 19:50:27 -0300 Received: from gauss ([200.158.97.212]) by traven9.uol.com.br (8.9.1/8.9.1) with SMTP id TAA04772 for ; Wed, 23 Apr 2003 19:49:56 -0300 (BRT) Message-ID: <000b01c309eb$5fcbeb70$d4619ec8@gauss> From: "Domingos Jr." To: References: Subject: Re: [obm-l] FW: Problema dificil Date: Wed, 23 Apr 2003 19:55:01 -0300 MIME-Version: 1.0 Content-Type: text/plain; charset="iso-8859-1" Content-Transfer-Encoding: 8bit X-Priority: 3 X-MSMail-Priority: Normal X-Mailer: Microsoft Outlook Express 6.00.2800.1106 X-MimeOLE: Produced By Microsoft MimeOLE V6.00.2800.1106 Sender: owner-obm-l@sucuri.mat.puc-rio.br Precedence: bulk Reply-To: obm-l@mat.puc-rio.br A propósito, vc tinha me pedido pra dar uma olhada naquele problema da Vingança Olímpica (roubando doces das crianças!). Eu deixei uma solução no meu site: www.linux.ime.usp.br/~domingos/doces.pdf tem tb um ps para a mesma solução... ainda não tive idéias para o problema que vc propôs... > Caros colegas da lista: > > Aqui vai um problema que eu levei algumas semanas pra resolver e, mesmo > assim, somente depois que me deram uma dica: > > Uma sequencia de 100 numeros reais tem a seguinte propriedade: > Para cada subsequencia de 8 termos, existe uma subsequencia de 9 termos que > tem a mesma media aritmetica que a de 8 termos. > Prove que a sequencia eh constante. > > Daqui a alguns dias eu dou a dica. > > Tenho muito interesse em ver alguma solucao diferente da minha (e de > preferencia mais simples). > > Um abraco, > Claudio. > > > ========================================================================= > Instruções para entrar na lista, sair da lista e usar a lista em > http://www.mat.puc-rio.br/~nicolau/olimp/obm-l.html > ========================================================================= ========================================================================= Instruções para entrar na lista, sair da lista e usar a lista em http://www.mat.puc-rio.br/~nicolau/olimp/obm-l.html ========================================================================= From owner-obm-l@sucuri.mat.puc-rio.br Wed Apr 23 21:08:33 2003 Return-Path: Received: (from majordom@localhost) by sucuri.mat.puc-rio.br (8.9.3/8.9.3) id VAA16918 for obm-l-MTTP; Wed, 23 Apr 2003 21:06:44 -0300 Received: from Euler.impa.br (euler.impa.br [147.65.1.3]) by sucuri.mat.puc-rio.br (8.9.3/8.9.3) with ESMTP id VAA16914 for ; Wed, 23 Apr 2003 21:06:41 -0300 Received: from [147.65.11.1] (dial01.impa.br [147.65.11.1]) by Euler.impa.br (8.11.6p2/8.11.6) with ESMTP id h3O066001829 for ; Wed, 23 Apr 2003 21:06:06 -0300 (EST) Message-Id: <200304240006.h3O066001829@Euler.impa.br> X-Mailer: Microsoft Outlook Express Macintosh Edition - 4.5 (0410) Date: Wed, 23 Apr 2003 21:04:53 -0300 Subject: Re: [obm-l] Ensino de matematica no Brasil From: "Eduardo Wagner" To: obm-l@mat.puc-rio.br Mime-version: 1.0 X-Priority: 3 Content-type: text/plain; charset="ISO-8859-1" Content-Transfer-Encoding: 8bit X-MIME-Autoconverted: from quoted-printable to 8bit by sucuri.mat.puc-rio.br id VAA16915 Sender: owner-obm-l@sucuri.mat.puc-rio.br Precedence: bulk Reply-To: obm-l@mat.puc-rio.br A lista nao eh realmente o forum adequado para discussao sobre ensino de matematica, mas nao posso deixar de registrar o meu total apoio em relacao ao que disse a prof. Suely Druck. Lamentamos o artigo do prof. Kulesza que mostrou nao compreender o que leu e que nem soube rabater os solidos argumentos da prof. Suely. O artigo do prof. Kulesza eh inconsistente e mediocre. A resposta do prof. Michel Spira nao poderia ser melhor. Disse tudo o que era necessario. A preocupacao com a melhoria do conteudo dos professores do ensino fundamental em medio eh tema atual em inumeros paises. Para a informacao de todos, a OEI (Organizacao dos Estados Iberoamericanos para Ciencia e Cultura) esta promovendo nos dias 28 a 30 de abril um grande simposio em Antigua, reunindo professores de matematica, educadores e autoridades em educacao de todos os paises de lingua espanhola e portuguesa para tratar exatamente deste tema: a melhoria do conteudo matematico dos professores para um melhor ensino. O Brasil estara fortemente presente, transmitindo a experiencia que temos no assunto atraves dos programas patrocinados pelo IMPA e pela Sociedade Brasileira de Matematica. Eduardo Wagner. PS. Benedito: foi bom voce ter escrito pois me motivou a escrever tambem. Sera que alguem pode ensinar portugues sem conhecer a lingua portuguesa? ---------- >From: benedito >To: obm-l@mat.puc-rio.br >Subject: Re: [obm-l] Ensino de matematica no Brasil >Date: Mon, Apr 21, 2003, 8:22 PM > > Muita gente pode não ter gostado....Mas as declarações da Presidente tem > muita verdade, isso tem. Incomoda alguns, especialmente aqueles que focam o > ensino nos métodos, esquecendo o principal: o conteúdo... > Benedito Freire > > At 19:30 21/4/2003 -0300, you wrote: >>Não é exatamente o assunto da lista, mas a Suely Druck (presidente da SBM) >>publicou um artigo na Folha sobre o ensino de matemática no Brasil. >>As olimpíadas de matemática são mencionadas mas não são o tema do artigo. >>Nem todos os leitores gostaram gostaram. Vejam os links abaixo >>(obrigado ao Gugu pela referência) vale a pena ler e refletir, >>talvez participar do debate. []s, N. >> >>http://www1.folha.uol.com.br/folha/sinapse/ult1063u343.shtml >>http://www.jornaldaciencia.org.br/Detalhe.jsp?id=8868 >>http://www.jornaldaciencia.org.br/Detalhe.jsp?id=9238 >>http://www.jornaldaciencia.org.br/Detalhe.jsp?id=9105 >> >> >> >>========================================================================= >>Instruções para entrar na lista, sair da lista e usar a lista em >>http://www.mat.puc-rio.br/~nicolau/olimp/obm-l.html >>O administrador desta lista é >>========================================================================= > > > ========================================================================= > Instruções para entrar na lista, sair da lista e usar a lista em > http://www.mat.puc-rio.br/~nicolau/olimp/obm-l.html > O administrador desta lista é > ========================================================================= ========================================================================= Instruções para entrar na lista, sair da lista e usar a lista em http://www.mat.puc-rio.br/~nicolau/olimp/obm-l.html ========================================================================= From owner-obm-l@sucuri.mat.puc-rio.br Wed Apr 23 22:28:43 2003 Return-Path: Received: (from majordom@localhost) by sucuri.mat.puc-rio.br (8.9.3/8.9.3) id WAA18207 for obm-l-MTTP; Wed, 23 Apr 2003 22:26:59 -0300 Received: from itaqui.terra.com.br (itaqui.terra.com.br [200.176.3.19]) by sucuri.mat.puc-rio.br (8.9.3/8.9.3) with ESMTP id WAA18202 for ; Wed, 23 Apr 2003 22:26:55 -0300 Received: from marova.terra.com.br (marova.terra.com.br [200.176.3.39]) by itaqui.terra.com.br (Postfix) with ESMTP id 3EAAF3BC863 for ; Wed, 23 Apr 2003 22:26:24 -0300 (BRT) Received: from niski.com (unknown [200.148.194.69]) (authenticated user fniski) by marova.terra.com.br (Postfix) with ESMTP id 004373DC084 for ; Wed, 23 Apr 2003 22:26:23 -0300 (BRT) Message-ID: <3EA72F34.3040006@niski.com> Date: Wed, 23 Apr 2003 17:26:28 -0700 From: niski User-Agent: Mozilla/5.0 (Windows; U; Windows NT 5.1; en-US; rv:1.0.2) Gecko/20030208 Netscape/7.02 X-Accept-Language: en-us, en MIME-Version: 1.0 To: obm-l@mat.puc-rio.br Subject: Re: [obm-l] simples de variaveis aleatorias References: <3EA6E9DE.40006@niski.com> <3EA6FDB1.8000704@centroin.com.br> Content-Type: text/plain; charset=ISO-8859-1; format=flowed Content-Transfer-Encoding: 8bit Sender: owner-obm-l@sucuri.mat.puc-rio.br Precedence: bulk Reply-To: obm-l@mat.puc-rio.br Certo professor. Mas quanto ao problema de elevar o valor da variavel...de 2 ela passou para 4..mas X=4 significa "tirar 4 caras" ,mas isso seria impossivel dado que são apenas 2 lançamentos...o que estou pensando errado!? obrigado A. C. Morgado wrote: > Voce estah certo e o seu gabarito, errado. > > niski wrote: > >> Olá pessoal da lista. Gostaria que me explicassem o que fiz de errado >> neste problema. >> >> "Duas moedas estão sobre a mesa, uma delas tem duas caras e a outra >> tem probabilidade igual de cara e coroa. Sorteamos, ao acaso, uma >> dessas moedas e a lançamos duas vezes. Seja X a variável aleatória que >> conta o número de caras nos dois lançamentos. Qual a esperança de X? >> Qual a variância de X? R: 1,5 e 9/16" -- [about him:] It is rare to find learned men who are clean, do not stink and have a sense of humour. -Gottfried Whilhem Leibniz ========================================================================= Instruções para entrar na lista, sair da lista e usar a lista em http://www.mat.puc-rio.br/~nicolau/olimp/obm-l.html ========================================================================= From owner-obm-l@sucuri.mat.puc-rio.br Wed Apr 23 22:30:47 2003 Return-Path: Received: (from majordom@localhost) by sucuri.mat.puc-rio.br (8.9.3/8.9.3) id WAA18259 for obm-l-MTTP; Wed, 23 Apr 2003 22:29:30 -0300 Received: from itaqui.terra.com.br (itaqui.terra.com.br [200.176.3.19]) by sucuri.mat.puc-rio.br (8.9.3/8.9.3) with ESMTP id WAA18254 for ; Wed, 23 Apr 2003 22:29:27 -0300 Received: from araci.terra.com.br (araci.terra.com.br [200.176.3.44]) by itaqui.terra.com.br (Postfix) with ESMTP id D65163BC0C2 for ; Wed, 23 Apr 2003 22:28:56 -0300 (BRT) Received: from [200.177.190.13] (dl-nas5-sao-C8B1BE0D.p001.terra.com.br [200.177.190.13]) by araci.terra.com.br (Postfix) with ESMTP id 01C2421EF7A for ; Wed, 23 Apr 2003 22:28:56 -0300 (BRT) User-Agent: Microsoft-Outlook-Express-Macintosh-Edition/5.02.2022 Date: Wed, 23 Apr 2003 22:30:20 -0300 Subject: Re: [obm-l] triangulo e bissetrizes From: Claudio Buffara To: Message-ID: In-Reply-To: <20030423223809.4152.qmail@web14304.mail.yahoo.com> Mime-version: 1.0 Content-type: text/plain; charset="ISO-8859-1" Content-Transfer-Encoding: 8bit X-MIME-Autoconverted: from quoted-printable to 8bit by sucuri.mat.puc-rio.br id WAA18255 Sender: owner-obm-l@sucuri.mat.puc-rio.br Precedence: bulk Reply-To: obm-l@mat.puc-rio.br on 23.04.03 19:38, Rafael at matduvidas@yahoo.com.br wrote: > Valeu Fabio pelo problema das circunferências. Agora > tem esse aqui também: > > ABC é um triângulo cujas medidas dos lados são a,b,c > e M ,N,P são os pés das três bissetrizes internas > desse triângulo. Achar a razão entre as áreas dos > triângulos MNP e ABC > resposta: 2abc/(a+b)(a+c)(b+c) > > Alguém me ajuda? > > Abraços, > > Rafael. > Oi, Rafael: Aqui acho que vale a pena usar o fato de que a bissetriz interna de um angulo divide o lado oposto a esse angulo em partes proporcionais aos outros dois lados. Assim, sejam as bissetrizes AM, BN e CP, com M em BC, N em AC e P em AB. Teremos: BC = a ==> BM = a*c/(b+c) e MC = a*b/(b+c) AC = b ==> AN = b*c/(a+c) e NC = b*a/(a+c) AB = c ==> AP = c*b/(a+b) e PB = c*a/(a+b) Alem disso, decompondo o triangulo ABC, teremos: [MNP] + [ANP] + [BMP] + [CMN] = [ABC] ==> [MNP]/[ABC] = 1 - [ANP]/[ABC] - [BMP]/[ABC] - [CMN]/[ABC] Mas: [ABC] = (1/2)*AB*AC*sen(A) = (1/2)*b*c*sen(A) e [ANP] = (1/2)*AN*AP*sen(A) = (1/2)*b^2*c^2*sen(A)/((a+c)(a+b)) Ou seja: [ANP]/[ABC] = bc/((a+c)(a+b)) = bc(b+c)/((a+b)(a+c)(b+c)) Analogamente, encontramos: [BMP]/[ABC] = ac(a+c)/((a+b)(a+c)(b+c)) e [CMN]/[ABC] = ab(a+b)/(((a+b)(a+c)(b+c)) Assim, temos que: [MNP]/[ABC] = 1 - (bc(b+c) + ac(a+c) + ab(a+b))/((a+b)(a+c)(b+c)) E, apos alguma algebra, chegamos finalmente a: [MNP]/[ABC] = 2abc/((a+b)(a+c)(b+c)) Um abraco, Claudio. ========================================================================= Instruções para entrar na lista, sair da lista e usar a lista em http://www.mat.puc-rio.br/~nicolau/olimp/obm-l.html ========================================================================= From owner-obm-l@sucuri.mat.puc-rio.br Wed Apr 23 22:32:09 2003 Return-Path: Received: (from majordom@localhost) by sucuri.mat.puc-rio.br (8.9.3/8.9.3) id WAA18333 for obm-l-MTTP; Wed, 23 Apr 2003 22:30:53 -0300 Received: from smtp-26.ig.com.br (smtp-26.ig.com.br [200.226.132.160]) by sucuri.mat.puc-rio.br (8.9.3/8.9.3) with SMTP id WAA18326 for ; Wed, 23 Apr 2003 22:30:48 -0300 Received: (qmail 5682 invoked from network); 24 Apr 2003 01:30:30 -0000 Received: from unknown (HELO xxxx) (200.165.206.112) by smtp-26.ig.com.br with SMTP; 24 Apr 2003 01:30:30 -0000 Message-ID: <00d401c30a01$e3eb4e80$70cea5c8@epq.ime.eb.br> From: "Marcio" To: References: Subject: Re: [obm-l] FW: Problema dificil Date: Wed, 23 Apr 2003 22:36:12 -0300 MIME-Version: 1.0 Content-Type: text/plain; charset="iso-8859-1" Content-Transfer-Encoding: 7bit X-Priority: 3 X-MSMail-Priority: Normal X-Mailer: Microsoft Outlook Express 5.50.4133.2400 X-MimeOLE: Produced By Microsoft MimeOLE V5.50.4133.2400 Sender: owner-obm-l@sucuri.mat.puc-rio.br Precedence: bulk Reply-To: obm-l@mat.puc-rio.br Oi Claudio. Acho que o que eu fiz esta certo.. De uma conferida. Chame os termos da sua sequencia de b1,b2,...,b100, com b1 <= b2 <= b3 <= b4 <=... <= b100 (note que essa reordenacao mantem a propriedade mencionada no enunciado). Para facilitar a notacao, voce pode ainda diminuir b1 de todo mundo e criar uma nova sequencia c1, c2, ..., c100 que ainda satisfaz a propriedade (tanto a media de 8 como a de 9 termos apenas foram diminuidas de b1, e portanto as relacoes de igualdade se mantem). Note que c1 = 0= c9. Por outro lado, m <= (c9+c9+...+c9)/8 = c9 , pois c1 <=c9, c2 <= c9, etc.. Logo, deve ocorrer a igualdade acima, e portanto 0=c1=c2=...=c9 Agora voce usa um raciocinio semelhante.. Suponha por absurdo que c10 > 0 Considere a media m = c10 / 8 (pegando com outros 7 zeros dentre c1,c2,...,c9). A menor media de 9 termos nao nula eh M = c10/9, que nao serve pois c10 eh nao nulo. A segunda menor eh M' = (c10+c11)/9 > 2(c10) / 9 > (c10)/8. Contradicao. Logo, c10 = 0. De forma analoga, voce conclui que c11=c12=...=c100=0. Logo, a sequencia original era constante. Marcio ----- Original Message ----- From: "Claudio Buffara" To: "Lista OBM" Sent: Wednesday, April 23, 2003 10:29 AM Subject: [obm-l] FW: Problema dificil > > Caros colegas da lista: > > Aqui vai um problema que eu levei algumas semanas pra resolver e, mesmo > assim, somente depois que me deram uma dica: > > Uma sequencia de 100 numeros reais tem a seguinte propriedade: > Para cada subsequencia de 8 termos, existe uma subsequencia de 9 termos que > tem a mesma media aritmetica que a de 8 termos. > Prove que a sequencia eh constante. > > Daqui a alguns dias eu dou a dica. > > Tenho muito interesse em ver alguma solucao diferente da minha (e de > preferencia mais simples). > > Um abraco, > Claudio. ========================================================================= Instruções para entrar na lista, sair da lista e usar a lista em http://www.mat.puc-rio.br/~nicolau/olimp/obm-l.html ========================================================================= From owner-obm-l@sucuri.mat.puc-rio.br Wed Apr 23 22:36:05 2003 Return-Path: Received: (from majordom@localhost) by sucuri.mat.puc-rio.br (8.9.3/8.9.3) id WAA18513 for obm-l-MTTP; Wed, 23 Apr 2003 22:34:47 -0300 Received: from trex-b.centroin.com.br (trex-b.centroin.com.br [200.225.63.136]) by sucuri.mat.puc-rio.br (8.9.3/8.9.3) with ESMTP id WAA18505 for ; Wed, 23 Apr 2003 22:34:43 -0300 Received: from centroin.com.br (RJ118229.user.veloxzone.com.br [200.141.118.229] (may be forged)) (authenticated bits=0) by trex-b.centroin.com.br (8.12.9/8.12.9) with ESMTP id h3O1YCVi022265 for ; Wed, 23 Apr 2003 22:34:13 -0300 (EST) Message-ID: <3EA73F51.10207@centroin.com.br> Date: Wed, 23 Apr 2003 22:35:13 -0300 From: "A. C. Morgado" User-Agent: Mozilla/5.0 (Windows; U; Windows NT 5.0; en-US; rv:1.0.2) Gecko/20030208 Netscape/7.02 X-Accept-Language: en-us, en MIME-Version: 1.0 To: obm-l@mat.puc-rio.br Subject: Re: [obm-l] simples de variaveis aleatorias References: <3EA6E9DE.40006@niski.com> <3EA6FDB1.8000704@centroin.com.br> <3EA72F34.3040006@niski.com> Content-Type: text/plain; charset=ISO-8859-1; format=flowed Content-Transfer-Encoding: 8bit Sender: owner-obm-l@sucuri.mat.puc-rio.br Precedence: bulk Reply-To: obm-l@mat.puc-rio.br Oh Niski, nao eh X=4, eh X^2 = 4. niski wrote: > Certo professor. > Mas quanto ao problema de elevar o valor da variavel...de 2 ela passou > para 4..mas X=4 significa "tirar 4 caras" ,mas isso seria impossivel > dado que são apenas 2 lançamentos...o que estou pensando errado!? > > obrigado > > A. C. Morgado wrote: > >> Voce estah certo e o seu gabarito, errado. >> >> niski wrote: >> >>> Olá pessoal da lista. Gostaria que me explicassem o que fiz de >>> errado neste problema. >>> >>> "Duas moedas estão sobre a mesa, uma delas tem duas caras e a outra >>> tem probabilidade igual de cara e coroa. Sorteamos, ao acaso, uma >>> dessas moedas e a lançamos duas vezes. Seja X a variável aleatória >>> que conta o número de caras nos dois lançamentos. Qual a esperança >>> de X? Qual a variância de X? R: 1,5 e 9/16" >> > ========================================================================= Instruções para entrar na lista, sair da lista e usar a lista em http://www.mat.puc-rio.br/~nicolau/olimp/obm-l.html ========================================================================= From owner-obm-l@sucuri.mat.puc-rio.br Wed Apr 23 22:54:22 2003 Return-Path: Received: (from majordom@localhost) by sucuri.mat.puc-rio.br (8.9.3/8.9.3) id WAA19534 for obm-l-MTTP; Wed, 23 Apr 2003 22:53:00 -0300 Received: from paiol.terra.com.br (paiol.terra.com.br [200.176.3.18]) by sucuri.mat.puc-rio.br (8.9.3/8.9.3) with ESMTP id WAA19529 for ; Wed, 23 Apr 2003 22:52:56 -0300 Received: from barra.terra.com.br (barra.terra.com.br [200.176.3.52]) by paiol.terra.com.br (Postfix) with ESMTP id 189AD8814D for ; Wed, 23 Apr 2003 22:52:26 -0300 (BRT) Received: from [200.177.176.64] (dl-nas1-sao-C8B1B040.p001.terra.com.br [200.177.176.64]) by barra.terra.com.br (Postfix) with ESMTP id 1A958234059 for ; Wed, 23 Apr 2003 22:52:25 -0300 (BRT) User-Agent: Microsoft-Outlook-Express-Macintosh-Edition/5.02.2022 Date: Wed, 23 Apr 2003 22:53:54 -0300 Subject: Re: [obm-l] FW: Problema dificil From: Claudio Buffara To: Message-ID: In-Reply-To: <000b01c309eb$5fcbeb70$d4619ec8@gauss> Mime-version: 1.0 Content-type: text/plain; charset="ISO-8859-1" Content-Transfer-Encoding: 8bit X-MIME-Autoconverted: from quoted-printable to 8bit by sucuri.mat.puc-rio.br id WAA19531 Sender: owner-obm-l@sucuri.mat.puc-rio.br Precedence: bulk Reply-To: obm-l@mat.puc-rio.br Oi, Domingos: Agradeco a sua atencao ao problema. De fato, eu ja imprimi uma copia da sua solucao pra analisar com calma. Nao eh falta de interesse - muito pelo contrario. So que eu nao tenho conseguido me organizar pra dar uma estudada mais a fundo nos problemas mais dificeis. Estou com uma pilha de e-mails seus e de varios outros colegas da lista aos quais ainda nao consegui dar a atencao devida. Isso pra nao falar duns 30 artigos, problemas, "lecture notes" e trechos de livros que tambem separei pra olhar com calma quando tiver um tempo. O chato eh que tem umas coisas que eu nao consigo nem comecar a ler na tela do computador. Tenho que imprimir e reescrever todo o argumento se quiser ter uma chance de entender... No momento, so tem dado pra acompanhar a lista e mandar as solucoes dos problemas que eu consigo resolver no computador mesmo. Um abraco, Claudio. on 23.04.03 19:55, Domingos Jr. at dopikas@uol.com.br wrote: > A propósito, vc tinha me pedido pra dar uma olhada naquele problema da > Vingança Olímpica (roubando doces das crianças!). > Eu deixei uma solução no meu site: www.linux.ime.usp.br/~domingos/doces.pdf > tem tb um ps para a mesma solução... > > > ainda não tive idéias para o problema que vc propôs... > >> Caros colegas da lista: >> >> Aqui vai um problema que eu levei algumas semanas pra resolver e, mesmo >> assim, somente depois que me deram uma dica: >> >> Uma sequencia de 100 numeros reais tem a seguinte propriedade: >> Para cada subsequencia de 8 termos, existe uma subsequencia de 9 termos > que >> tem a mesma media aritmetica que a de 8 termos. >> Prove que a sequencia eh constante. >> >> Daqui a alguns dias eu dou a dica. >> >> Tenho muito interesse em ver alguma solucao diferente da minha (e de >> preferencia mais simples). >> >> Um abraco, >> Claudio. >> >> >> ========================================================================= >> Instruções para entrar na lista, sair da lista e usar a lista em >> http://www.mat.puc-rio.br/~nicolau/olimp/obm-l.html >> ========================================================================= > > ========================================================================= > Instruções para entrar na lista, sair da lista e usar a lista em > http://www.mat.puc-rio.br/~nicolau/olimp/obm-l.html > ========================================================================= > ========================================================================= Instruções para entrar na lista, sair da lista e usar a lista em http://www.mat.puc-rio.br/~nicolau/olimp/obm-l.html ========================================================================= From owner-obm-l@sucuri.mat.puc-rio.br Wed Apr 23 23:19:43 2003 Return-Path: Received: (from majordom@localhost) by sucuri.mat.puc-rio.br (8.9.3/8.9.3) id XAA21169 for obm-l-MTTP; Wed, 23 Apr 2003 23:18:22 -0300 Received: from ivoti.terra.com.br (ivoti.terra.com.br [200.176.3.20]) by sucuri.mat.puc-rio.br (8.9.3/8.9.3) with ESMTP id XAA21164 for ; Wed, 23 Apr 2003 23:18:19 -0300 Received: from bertioga.terra.com.br (bertioga.terra.com.br [200.176.3.77]) by ivoti.terra.com.br (Postfix) with ESMTP id 9CFE340878A for ; Wed, 23 Apr 2003 23:17:47 -0300 (BRT) Received: from niski.com (unknown [200.148.194.69]) (authenticated user fniski) by bertioga.terra.com.br (Postfix) with ESMTP id 751FD3F803C for ; Wed, 23 Apr 2003 23:17:46 -0300 (BRT) Message-ID: <3EA73B3F.8030909@niski.com> Date: Wed, 23 Apr 2003 18:17:51 -0700 From: niski User-Agent: Mozilla/5.0 (Windows; U; Windows NT 5.1; en-US; rv:1.0.2) Gecko/20030208 Netscape/7.02 X-Accept-Language: en-us, en MIME-Version: 1.0 To: obm-l@mat.puc-rio.br Subject: Re: [obm-l] simples de variaveis aleatorias References: <3EA6E9DE.40006@niski.com> <3EA6FDB1.8000704@centroin.com.br> <3EA72F34.3040006@niski.com> <3EA73F51.10207@centroin.com.br> Content-Type: text/plain; charset=ISO-8859-1; format=flowed Content-Transfer-Encoding: 8bit Sender: owner-obm-l@sucuri.mat.puc-rio.br Precedence: bulk Reply-To: obm-l@mat.puc-rio.br Professor Morgado. obrigado pela elucidação, do X^2. É verdade. Gostaria que se possivel, olhasse se novamente o gabarito esta errado. Um exame consta de 25 questões e, para ser aprovado um estudante deverá responder corretamente pelo menos 17 dessas questões. Um determinado estudante sabe 60% do material sobre o qual serão formuladas as questões. Qual a probabilidade desse estudante ser aprovado? Eu resolvi calculando o somatorio de i variando de 17 até 25 para ((0,6)^i).((0,4)^(25-i)).C[25,i] Onde C[25,i] é combinacao de 25 elementos tomados i a i. Este somatorio é igual a 0.273531 , porem no gabarito a resposta é 1-0.273531 = 0.726469 O que intuitivamente, me parece mais correto. O sr. (ou qualquer outro participante da lista) poderia me falar o que esta de errado!? obrigado -- [about him:] It is rare to find learned men who are clean, do not stink and have a sense of humour. -Gottfried Whilhem Leibniz A. C. Morgado wrote: > Oh Niski, nao eh X=4, eh X^2 = 4. ========================================================================= Instruções para entrar na lista, sair da lista e usar a lista em http://www.mat.puc-rio.br/~nicolau/olimp/obm-l.html ========================================================================= From owner-obm-l@sucuri.mat.puc-rio.br Thu Apr 24 09:38:01 2003 Return-Path: Received: (from majordom@localhost) by sucuri.mat.puc-rio.br (8.9.3/8.9.3) id JAA28749 for obm-l-MTTP; Thu, 24 Apr 2003 09:36:04 -0300 Received: from trex.centroin.com.br (trex.centroin.com.br [200.225.63.134]) by sucuri.mat.puc-rio.br (8.9.3/8.9.3) with ESMTP id JAA28745 for ; Thu, 24 Apr 2003 09:36:00 -0300 Received: from trex.centroin.com.br (localhost [127.0.0.1]) by trex.centroin.com.br (8.12.9/8.12.9) with ESMTP id h3OCZUrL013433 for ; Thu, 24 Apr 2003 09:35:30 -0300 (EST) Received: by trex.centroin.com.br (8.12.9/8.12.5/Submit) id h3OCZUCC013431; Thu, 24 Apr 2003 09:35:30 -0300 (EST) Message-Id: <200304241235.h3OCZUCC013431@trex.centroin.com.br> Received: from 200.141.91.20 by trex.centroin.com.br (CIPWM versao 1.4C1) with HTTPS for ; Thu, 24 Apr 2003 09:35:29 -0300 (EST) Date: Thu, 24 Apr 2003 09:35:29 -0300 (EST) From: Augusto Cesar de Oliveira Morgado To: obm-l@mat.puc-rio.br Subject: Re: [obm-l] simples de variaveis aleatorias MIME-Version: 1.0 X-Mailer: CentroIn Internet Provider WebMail v. 1.4C1 (http://www.centroin.com.br/) Content-Type: text/plain; charset="iso-8859-1" Content-Transfer-Encoding: 8bit X-MIME-Autoconverted: from quoted-printable to 8bit by sucuri.mat.puc-rio.br id JAA28746 Sender: owner-obm-l@sucuri.mat.puc-rio.br Precedence: bulk Reply-To: obm-l@mat.puc-rio.br Nao conferi as contas, mas o que voce fez estah certo. Em Wed, 23 Apr 2003 18:17:51 -0700, niski disse: > Professor Morgado. obrigado pela elucidação, do X^2. É verdade. > Gostaria que se possivel, olhasse se novamente o gabarito esta errado. > > Um exame consta de 25 questões e, para ser aprovado um estudante deverá > responder corretamente pelo menos 17 dessas questões. Um determinado > estudante sabe 60% do material sobre o qual serão formuladas as > questões. Qual a probabilidade desse estudante ser aprovado? > > Eu resolvi calculando o somatorio de i variando de 17 até 25 para > ((0,6)^i).((0,4)^(25-i)).C[25,i] > Onde C[25,i] é combinacao de 25 elementos tomados i a i. > > Este somatorio é igual a 0.273531 , porem no gabarito a resposta é > 1-0.273531 = 0.726469 > O que intuitivamente, me parece mais correto. > > O sr. (ou qualquer outro participante da lista) poderia me falar o que > esta de errado!? > obrigado > > -- > [about him:] > It is rare to find learned men who are clean, do not stink and have a > sense of humour. > -Gottfried Whilhem Leibniz > > A. C. Morgado wrote: > > Oh Niski, nao eh X=4, eh X^2 = 4. > > ========================================================================= > Instruções para entrar na lista, sair da lista e usar a lista em > http://www.mat.puc-rio.br/~nicolau/olimp/obm-l.html > ========================================================================= > > ========================================================================= Instruções para entrar na lista, sair da lista e usar a lista em http://www.mat.puc-rio.br/~nicolau/olimp/obm-l.html ========================================================================= From owner-obm-l@sucuri.mat.puc-rio.br Thu Apr 24 09:59:33 2003 Return-Path: Received: (from majordom@localhost) by sucuri.mat.puc-rio.br (8.9.3/8.9.3) id JAA29354 for obm-l-MTTP; Thu, 24 Apr 2003 09:57:50 -0300 Received: from itaqui.terra.com.br (itaqui.terra.com.br [200.176.3.19]) by sucuri.mat.puc-rio.br (8.9.3/8.9.3) with ESMTP id JAA29350 for ; Thu, 24 Apr 2003 09:57:45 -0300 Received: from gunga.terra.com.br (gunga.terra.com.br [200.176.3.45]) by itaqui.terra.com.br (Postfix) with ESMTP id A1A333BD100 for ; Thu, 24 Apr 2003 09:57:11 -0300 (BRT) Received: from o0f7o8 (200-158-198-31.dsl.telesp.net.br [200.158.198.31]) (authenticated user ssalomao) by gunga.terra.com.br (Postfix) with ESMTP id 2BA89128065 for ; Thu, 24 Apr 2003 09:57:11 -0300 (BRT) Message-ID: <002201c30a61$11be5e60$9cddfea9@o0f7o8> From: =?iso-8859-1?Q?Pedro_Antonio_Santoro_Salom=E3o?= To: References: <00d401c30a01$e3eb4e80$70cea5c8@epq.ime.eb.br> Subject: Re: [obm-l] FW: Problema dificil Date: Thu, 24 Apr 2003 09:57:31 -0300 MIME-Version: 1.0 Content-Type: text/plain; charset="iso-8859-1" Content-Transfer-Encoding: 8bit X-Priority: 3 X-MSMail-Priority: Normal X-Mailer: Microsoft Outlook Express 6.00.2600.0000 X-MimeOLE: Produced By Microsoft MimeOLE V6.00.2600.0000 Sender: owner-obm-l@sucuri.mat.puc-rio.br Precedence: bulk Reply-To: obm-l@mat.puc-rio.br ----- Original Message ----- From: "Marcio" To: Sent: Wednesday, April 23, 2003 10:36 PM Subject: Re: [obm-l] FW: Problema dificil > Oi Claudio. Acho que o que eu fiz esta certo.. De uma conferida. > Chame os termos da sua sequencia de b1,b2,...,b100, com b1 <= b2 <= b3 > <= b4 <=... <= b100 > (note que essa reordenacao mantem a propriedade mencionada no enunciado). > Para facilitar a notacao, voce pode ainda diminuir b1 de todo mundo e > criar uma nova sequencia c1, c2, ..., c100 que ainda satisfaz a propriedade > (tanto a media de 8 como a de 9 termos apenas foram diminuidas de b1, e > portanto as relacoes de igualdade se mantem). Note que c1 = > 0 > Agora considere a media m = (c1+c2+...+c8)/8. > Veja que, devido a ordenacao que fizemos, a menor media de 9 numeros que > pode existir eh M = (c1+c2+..+c8+c9)/9 = (8m+c9)/9 > Em particular, essa menor media nao pode ser maior que m (pq se ela for, > todas as outras tmb serao!). > Logo, (8m+c9)/9 <= m donde m >= c9. > Por outro lado, m <= (c9+c9+...+c9)/8 = c9 , pois c1 <=c9, c2 <= c9, etc.. > Logo, deve ocorrer a igualdade acima, e portanto 0=c1=c2=...=c9 > > Agora voce usa um raciocinio semelhante.. Suponha por absurdo que c10 > 0 > Considere a media m = c10 / 8 (pegando com outros 7 zeros dentre > c1,c2,...,c9). > A menor media de 9 termos nao nula eh M = c10/9, que nao serve pois c10 eh > nao nulo. > A segunda menor eh M' = (c10+c11)/9 > 2(c10) / 9 > (c10)/8. Contradicao. > Logo, c10 = 0. Aqui tem um probleminha. A segunda menor media não nula com 9 números poderia ser c11/9, pegando outros 8 zeros de c1,...,c9. A partir daí as coisas começam a complicar. Pelo menos por esse caminho. Abraço. Pedro. > > De forma analoga, voce conclui que c11=c12=...=c100=0. Logo, a sequencia > original era constante. > > Marcio > > ----- Original Message ----- > From: "Claudio Buffara" > To: "Lista OBM" > Sent: Wednesday, April 23, 2003 10:29 AM > Subject: [obm-l] FW: Problema dificil > > > > > > Caros colegas da lista: > > > > Aqui vai um problema que eu levei algumas semanas pra resolver e, mesmo > > assim, somente depois que me deram uma dica: > > > > Uma sequencia de 100 numeros reais tem a seguinte propriedade: > > Para cada subsequencia de 8 termos, existe uma subsequencia de 9 termos > que > > tem a mesma media aritmetica que a de 8 termos. > > Prove que a sequencia eh constante. > > > > Daqui a alguns dias eu dou a dica. > > > > Tenho muito interesse em ver alguma solucao diferente da minha (e de > > preferencia mais simples). > > > > Um abraco, > > Claudio. > > > ========================================================================= > Instruções para entrar na lista, sair da lista e usar a lista em > http://www.mat.puc-rio.br/~nicolau/olimp/obm-l.html > ========================================================================= > ========================================================================= Instruções para entrar na lista, sair da lista e usar a lista em http://www.mat.puc-rio.br/~nicolau/olimp/obm-l.html ========================================================================= From owner-obm-l@sucuri.mat.puc-rio.br Thu Apr 24 10:11:47 2003 Return-Path: Received: (from majordom@localhost) by sucuri.mat.puc-rio.br (8.9.3/8.9.3) id KAA29894 for obm-l-MTTP; Thu, 24 Apr 2003 10:10:11 -0300 Received: from hotmail.com (f71.law8.hotmail.com [216.33.241.71]) by sucuri.mat.puc-rio.br (8.9.3/8.9.3) with ESMTP id KAA29874 for ; Thu, 24 Apr 2003 10:10:04 -0300 Received: from mail pickup service by hotmail.com with Microsoft SMTPSVC; Thu, 24 Apr 2003 06:09:32 -0700 Received: from 200.222.105.178 by lw8fd.law8.hotmail.msn.com with HTTP; Thu, 24 Apr 2003 13:09:31 GMT X-Originating-IP: [200.222.105.178] X-Originating-Email: [osneto@hotmail.com] From: "Antonio Neto" To: obm-l@mat.puc-rio.br Subject: Re: Re[2]: [obm-l] 4 coisinhas Date: Thu, 24 Apr 2003 13:09:31 +0000 Mime-Version: 1.0 Content-Type: text/plain; format=flowed Message-ID: X-OriginalArrivalTime: 24 Apr 2003 13:09:32.0002 (UTC) FILETIME=[BEC2DC20:01C30A62] Sender: owner-obm-l@sucuri.mat.puc-rio.br Precedence: bulk Reply-To: obm-l@mat.puc-rio.br >O polinômio que descreve a soma eh de grau 3 + 1 = 4, logo: > >** Solução no braço (Perde-se muito tempo numa prova) > >S(n) = ax^4 + bx^3 + cx^2 + dx + e, faz-se >S(1) = 1 >S(2) = 9 >S(3) = 36 >S(4) = 100 >S(5) = 225, 5 equações e 5 incógnitas, resolve-se o sistema. > >[resolvendo pelo Maple] > >{e = 0, a = 1/4, b = 1/2, c = 1/4, d = 0} > >.:. S(x) = x^4/4 + x^3/2 + x^2/4 Olah, povo da PA de ordem superior. Posso sugerir que em vez de calcular de S(1) a S(5) calculemos de S(0) a S(4)? Se o aluno deve mesmo resolver o sistema na prova, sem Maple, que resolva um sistema mais simples. No caso em particular, acho que d S(-2) a S(2) seria mais em conta, mas nao fiz o calculo. Abracos, olavo. _________________________________________________________________ MSN 8 with e-mail virus protection service: 2 months FREE* http://join.msn.com/?page=features/virus ========================================================================= Instruções para entrar na lista, sair da lista e usar a lista em http://www.mat.puc-rio.br/~nicolau/olimp/obm-l.html ========================================================================= From owner-obm-l@sucuri.mat.puc-rio.br Thu Apr 24 11:05:27 2003 Return-Path: Received: (from majordom@localhost) by sucuri.mat.puc-rio.br (8.9.3/8.9.3) id LAA31674 for obm-l-MTTP; Thu, 24 Apr 2003 11:03:11 -0300 Received: from paiol.terra.com.br (paiol.terra.com.br [200.176.3.18]) by sucuri.mat.puc-rio.br (8.9.3/8.9.3) with ESMTP id LAA31667 for ; Thu, 24 Apr 2003 11:03:06 -0300 Received: from barra.terra.com.br (barra.terra.com.br [200.176.3.52]) by paiol.terra.com.br (Postfix) with ESMTP id 744F188912 for ; Thu, 24 Apr 2003 11:02:35 -0300 (BRT) Received: from nt (RJ231083.user.veloxzone.com.br [200.165.231.83]) (authenticated user ensr) by barra.terra.com.br (Postfix) with ESMTP id 02BA723408C for ; Thu, 24 Apr 2003 11:02:35 -0300 (BRT) Message-ID: <007801c30a6a$05d59380$5400a8c0@ensrbr> From: "Luis Lopes" To: References: <20030421151630.66488.qmail@web13007.mail.yahoo.com> <027d01c3090b$a009e880$5400a8c0@ensrbr> <9821226982.20030422195430@gmx.net> Subject: Re: Re[2]: [obm-l] 4 coisinhas Date: Thu, 24 Apr 2003 11:01:37 -0300 MIME-Version: 1.0 Content-Type: text/plain; charset="iso-8859-1" Content-Transfer-Encoding: 8bit X-Priority: 3 X-MSMail-Priority: Normal X-Mailer: Microsoft Outlook Express 5.50.4807.1700 X-MimeOLE: Produced By Microsoft MimeOLE V5.50.4807.1700 Sender: owner-obm-l@sucuri.mat.puc-rio.br Precedence: bulk Reply-To: obm-l@mat.puc-rio.br Sauda,c~oes, Oi Igor, Falou e disse! Legal o método, não? Não tem nome, faz parte da teoria do cálculo de diferenças. ===Msg do Olavo Olah, povo da PA de ordem superior. Posso sugerir que em vez de calcular de S(1) a S(5) calculemos de S(0) a S(4)? Se o aluno deve mesmo resolver o sistema na prova, sem Maple, que resolva um sistema mais simples. No caso em particular, acho que d S(-2) a S(2) seria mais em conta, mas nao fiz o calculo. Abracos, olavo. === Na verdade S(1) a S(4) pois a_0 (ou e) = 0 para qq PA de ordem k. Ou S(-1), S(1) a S(3). >S(n) = ax^4 + bx^3 + cx^2 + dx + e, faz-se > [resolvendo pelo Maple] > > {e = 0, a = 1/4, b = 1/2, c = 1/4, d = 0} > > .:. S(x) = x^4/4 + x^3/2 + x^2/4 > []'s Luís -----Mensagem Original----- De: "Igor GomeZZ" Para: "Luis Lopes" Enviada em: terça-feira, 22 de abril de 2003 19:54 Assunto: Re[2]: [obm-l] 4 coisinhas > > Em 22/4/2003, 17:13, Luis (llopes@ensrbr.com.br) disse: > > >> 4) > >> Calcular S = 1 + 8 + 27 + ... + x^3 > > Este o Igor deverá saber fazer. Fala, Igor! > . > > [resolvendo pelo Maple] > > {e = 0, a = 1/4, b = 1/2, c = 1/4, d = 0} > > .:. S(x) = x^4/4 + x^3/2 + x^2/4 > > > > ** Solução à la Luis (Qual o nome desse método?) > > S(n) = (/\[0]a[1])*binomial(n,1) + (/\[1]a[1])*binomial(n,2) + > + (/\[2]a[1])*binomial(n,3) + (/\[3]a[1])*binom(n,4) > > S(n) = 1*binomial(n,1) + 7*binomial(n,2) + 12*binomial(n,3) + 6*binomial(n,4) > > .:. S(n) = n^4/4 + n^3/2 + n^2/4 > > ========================================================================= Instruções para entrar na lista, sair da lista e usar a lista em http://www.mat.puc-rio.br/~nicolau/olimp/obm-l.html ========================================================================= From owner-obm-l@sucuri.mat.puc-rio.br Thu Apr 24 12:08:01 2003 Return-Path: Received: (from majordom@localhost) by sucuri.mat.puc-rio.br (8.9.3/8.9.3) id MAA00884 for obm-l-MTTP; Thu, 24 Apr 2003 12:05:03 -0300 Received: from paiol.terra.com.br (paiol.terra.com.br [200.176.3.18]) by sucuri.mat.puc-rio.br (8.9.3/8.9.3) with ESMTP id MAA00880 for ; Thu, 24 Apr 2003 12:04:59 -0300 Received: from botucatu.terra.com.br (botucatu.terra.com.br [200.176.3.78]) by paiol.terra.com.br (Postfix) with ESMTP id A80B888965 for ; Thu, 24 Apr 2003 12:04:23 -0300 (BRT) Received: from niski.com (unknown [200.148.202.95]) (authenticated user fniski) by botucatu.terra.com.br (Postfix) with ESMTP id 7C18C29C0DE for ; Thu, 24 Apr 2003 12:04:22 -0300 (BRT) Message-ID: <3EA7EEEC.5060607@niski.com> Date: Thu, 24 Apr 2003 07:04:28 -0700 From: niski User-Agent: Mozilla/5.0 (Windows; U; Windows NT 5.1; en-US; rv:1.0.2) Gecko/20030208 Netscape/7.02 X-Accept-Language: en-us, en MIME-Version: 1.0 To: obm-l@mat.puc-rio.br Subject: Re: [obm-l] simples de variaveis aleatorias References: <200304241235.h3OCZUCC013431@trex.centroin.com.br> Content-Type: text/plain; charset=us-ascii; format=flowed Content-Transfer-Encoding: 7bit Sender: owner-obm-l@sucuri.mat.puc-rio.br Precedence: bulk Reply-To: obm-l@mat.puc-rio.br Augusto Cesar de Oliveira Morgado wrote: > Nao conferi as contas, mas o que voce fez estah certo. Obrigado. -- [about him:] It is rare to find learned men who are clean, do not stink and have a sense of humour. -Gottfried Whilhem Leibniz ========================================================================= Instruções para entrar na lista, sair da lista e usar a lista em http://www.mat.puc-rio.br/~nicolau/olimp/obm-l.html ========================================================================= From owner-obm-l@sucuri.mat.puc-rio.br Thu Apr 24 13:03:47 2003 Return-Path: Received: (from majordom@localhost) by sucuri.mat.puc-rio.br (8.9.3/8.9.3) id NAA02117 for obm-l-MTTP; Thu, 24 Apr 2003 13:01:58 -0300 Received: from www.zipmail.com.br (smtp.zipmail.com.br [200.221.11.147]) by sucuri.mat.puc-rio.br (8.9.3/8.9.3) with ESMTP id NAA02113 for ; Thu, 24 Apr 2003 13:01:55 -0300 From: peterdirichlet1985@zipmail.com.br Received: from [200.206.103.3] by www.zipmail.com.br with HTTP; Thu, 24 Apr 2003 12:59:48 -0300 Message-ID: <3E9EC1890000EB42@www.zipmail.com.br> Date: Thu, 24 Apr 2003 12:59:48 -0300 In-Reply-To: Subject: [obm-l] =?iso-8859-1?Q?Re=3A=20=5Bobm=2Dl=5D=20TEOIREMA=20DE=20CRISTEA=3A=3AGeometria=2Cpontos=20estranhos=20e=09problemas=20legais=21=21=21=21?= To: obm-l@mat.puc-rio.br MIME-Version: 1.0 Content-Type: text/plain; charset="iso-8859-1" Content-Transfer-Encoding: 8bit X-MIME-Autoconverted: from quoted-printable to 8bit by sucuri.mat.puc-rio.br id NAA02114 Sender: owner-obm-l@sucuri.mat.puc-rio.br Precedence: bulk Reply-To: obm-l@mat.puc-rio.br MAS COMO???!?!??!!:???!Ja estou ha semanas tentando sem sucesso ou expectativa.Nenhum triangulo da muitas esperanças... -- Mensagem original -- >on 22.04.03 21:25, Johann Peter Gustav Lejeune Dirichlet at >peterdirichlet2002@yahoo.com.br wrote: > >Ola gente!!!!Estou tentando resolver problemas de Geometria do Mathematical >excalibur mas preciso de um teorema.Quem pode demonstrar pra mim: > >Considere o triangulo ABC e um ponto T.Duas cevianas CT_c e BT_b se cortam >erm T e os pontos R_c,R_b e T sao alinhados,com R_c em AB e R_b em AC.Mostre >que > >AT_c*BR_c/T_cB/R_cA + ATb*CR_b//T_bC/R_bA = 1 > >Nao sei se e isso mesmo mas vale o risco... > > >Oi, JP: > >Supondo que a expressao acima seja: > >(AT_c * BR_c) / (BT_c * AR_c) + ( AT_b * CR_b ) / (CT_b * AR_b ) = 1 > >eu diria que uma boa aposta seria o teorema de Menelau (aplicado mais de >uma >vez). > > >Um abraco, >Claudio. > > > TEA WITH ME THAT I BOOK YOUR FACE ------------------------------------------ Use o melhor sistema de busca da Internet Radar UOL - http://www.radaruol.com.br ========================================================================= Instruções para entrar na lista, sair da lista e usar a lista em http://www.mat.puc-rio.br/~nicolau/olimp/obm-l.html ========================================================================= From owner-obm-l@sucuri.mat.puc-rio.br Thu Apr 24 13:05:30 2003 Return-Path: Received: (from majordom@localhost) by sucuri.mat.puc-rio.br (8.9.3/8.9.3) id NAA02189 for obm-l-MTTP; Thu, 24 Apr 2003 13:04:03 -0300 Received: from www.zipmail.com.br (smtp.zipmail.com.br [200.221.11.147]) by sucuri.mat.puc-rio.br (8.9.3/8.9.3) with ESMTP id NAA02178 for ; Thu, 24 Apr 2003 13:03:58 -0300 From: peterdirichlet1985@zipmail.com.br Received: from [200.206.103.3] by www.zipmail.com.br with HTTP; Thu, 24 Apr 2003 13:01:51 -0300 Message-ID: <3E9EC1890000EB71@www.zipmail.com.br> Date: Thu, 24 Apr 2003 13:01:51 -0300 In-Reply-To: Subject: [obm-l] =?iso-8859-1?Q?Re=3A=20=5Bobm=2Dl=5D=20triangulo=20e=20bissetrizes?= To: obm-l@mat.puc-rio.br MIME-Version: 1.0 Content-Type: text/plain; charset="iso-8859-1" Content-Transfer-Encoding: 8bit X-MIME-Autoconverted: from quoted-printable to 8bit by sucuri.mat.puc-rio.br id NAA02183 Sender: owner-obm-l@sucuri.mat.puc-rio.br Precedence: bulk Reply-To: obm-l@mat.puc-rio.br Tente com trigonometria sempre que for preciso!!!!!!!!!! -- Mensagem original -- >on 23.04.03 19:38, Rafael at matduvidas@yahoo.com.br wrote: > >> Valeu Fabio pelo problema das circunferências. Agora >> tem esse aqui também: >> >> ABC é um triângulo cujas medidas dos lados são a,b,c >> e M ,N,P são os pés das três bissetrizes internas >> desse triângulo. Achar a razão entre as áreas dos >> triângulos MNP e ABC >> resposta: 2abc/(a+b)(a+c)(b+c) >> >> Alguém me ajuda? >> >> Abraços, >> >> Rafael. >> >Oi, Rafael: > >Aqui acho que vale a pena usar o fato de que a bissetriz interna de um >angulo divide o lado oposto a esse angulo em partes proporcionais aos outros >dois lados. > >Assim, sejam as bissetrizes AM, BN e CP, com M em BC, N em AC e P em AB. > >Teremos: >BC = a ==> BM = a*c/(b+c) e MC = a*b/(b+c) >AC = b ==> AN = b*c/(a+c) e NC = b*a/(a+c) >AB = c ==> AP = c*b/(a+b) e PB = c*a/(a+b) > >Alem disso, decompondo o triangulo ABC, teremos: >[MNP] + [ANP] + [BMP] + [CMN] = [ABC] ==> >[MNP]/[ABC] = 1 - [ANP]/[ABC] - [BMP]/[ABC] - [CMN]/[ABC] > >Mas: >[ABC] = (1/2)*AB*AC*sen(A) = (1/2)*b*c*sen(A) >e >[ANP] = (1/2)*AN*AP*sen(A) = (1/2)*b^2*c^2*sen(A)/((a+c)(a+b)) > >Ou seja: >[ANP]/[ABC] = bc/((a+c)(a+b)) = bc(b+c)/((a+b)(a+c)(b+c)) > >Analogamente, encontramos: >[BMP]/[ABC] = ac(a+c)/((a+b)(a+c)(b+c)) >e >[CMN]/[ABC] = ab(a+b)/(((a+b)(a+c)(b+c)) > >Assim, temos que: >[MNP]/[ABC] = 1 - (bc(b+c) + ac(a+c) + ab(a+b))/((a+b)(a+c)(b+c)) > >E, apos alguma algebra, chegamos finalmente a: >[MNP]/[ABC] = 2abc/((a+b)(a+c)(b+c)) > >Um abraco, >Claudio. > >========================================================================= >Instruções para entrar na lista, sair da lista e usar a lista em >http://www.mat.puc-rio.br/~nicolau/olimp/obm-l.html >========================================================================= > TEA WITH ME THAT I BOOK YOUR FACE ------------------------------------------ Use o melhor sistema de busca da Internet Radar UOL - http://www.radaruol.com.br ========================================================================= Instruções para entrar na lista, sair da lista e usar a lista em http://www.mat.puc-rio.br/~nicolau/olimp/obm-l.html ========================================================================= From owner-obm-l@sucuri.mat.puc-rio.br Thu Apr 24 13:05:41 2003 Return-Path: Received: (from majordom@localhost) by sucuri.mat.puc-rio.br (8.9.3/8.9.3) id NAA02200 for obm-l-MTTP; Thu, 24 Apr 2003 13:04:23 -0300 Received: from www.zipmail.com.br (smtp.zipmail.com.br [200.221.11.147]) by sucuri.mat.puc-rio.br (8.9.3/8.9.3) with ESMTP id NAA02195 for ; Thu, 24 Apr 2003 13:04:19 -0300 From: peterdirichlet1985@zipmail.com.br Received: from [200.206.103.3] by www.zipmail.com.br with HTTP; Thu, 24 Apr 2003 13:02:47 -0300 Message-ID: <3E9EC1890000EB87@www.zipmail.com.br> Date: Thu, 24 Apr 2003 13:02:47 -0300 In-Reply-To: <004101c30915$959b6c20$0301a8c0@usuario> Subject: [obm-l] =?iso-8859-1?Q?Re=3A=20=5Bobm=2Dl=5D=20euclidiana?= To: obm-l@mat.puc-rio.br MIME-Version: 1.0 Content-Type: text/plain; charset="iso-8859-1" Content-Transfer-Encoding: 8bit X-MIME-Autoconverted: from quoted-printable to 8bit by sucuri.mat.puc-rio.br id NAA02196 Sender: owner-obm-l@sucuri.mat.puc-rio.br Precedence: bulk Reply-To: obm-l@mat.puc-rio.br Pode traduzir? -- Mensagem original -- > >Como podemos verificar que um sistema de unidades de medidas é, de fato, >um sistema? > >Obrigado, > >Mário. > TEA WITH ME THAT I BOOK YOUR FACE ------------------------------------------ Use o melhor sistema de busca da Internet Radar UOL - http://www.radaruol.com.br ========================================================================= Instruções para entrar na lista, sair da lista e usar a lista em http://www.mat.puc-rio.br/~nicolau/olimp/obm-l.html ========================================================================= From owner-obm-l@sucuri.mat.puc-rio.br Thu Apr 24 13:10:55 2003 Return-Path: Received: (from majordom@localhost) by sucuri.mat.puc-rio.br (8.9.3/8.9.3) id NAA02390 for obm-l-MTTP; Thu, 24 Apr 2003 13:09:11 -0300 Received: from www.zipmail.com.br (smtp.zipmail.com.br [200.221.11.147]) by sucuri.mat.puc-rio.br (8.9.3/8.9.3) with ESMTP id NAA02384 for ; Thu, 24 Apr 2003 13:09:01 -0300 From: peterdirichlet1985@zipmail.com.br Received: from [200.206.103.3] by www.zipmail.com.br with HTTP; Thu, 24 Apr 2003 13:06:49 -0300 Message-ID: <3E9EC1890000EBD2@www.zipmail.com.br> Date: Thu, 24 Apr 2003 13:06:49 -0300 In-Reply-To: Subject: [obm-l] =?iso-8859-1?Q?Re=3A=20Re=5B2=5D=3A=20=5Bobm=2Dl=5D=204=20coisinhas?= To: obm-l@mat.puc-rio.br MIME-Version: 1.0 Content-Type: text/plain; charset="iso-8859-1" Content-Transfer-Encoding: 8bit X-MIME-Autoconverted: from quoted-printable to 8bit by sucuri.mat.puc-rio.br id NAA02387 Sender: owner-obm-l@sucuri.mat.puc-rio.br Precedence: bulk Reply-To: obm-l@mat.puc-rio.br Polamordedeos!!!!!!Ces querem evitar o inevitavel!!!!!Isso tudo e so conta,nada mais.Seja como for nao ha o que fazer. Talvez tabela de diferenças seja mais util...Mas nao tenha medo de contas.A nao ser,como disse o Caminha,que seja mais facil simplificar que complicar:) -- Mensagem original -- >>O polinômio que descreve a soma eh de grau 3 + 1 = 4, logo: >> >>** Solução no braço (Perde-se muito tempo numa prova) >> >>S(n) = ax^4 + bx^3 + cx^2 + dx + e, faz-se >>S(1) = 1 >>S(2) = 9 >>S(3) = 36 >>S(4) = 100 >>S(5) = 225, 5 equações e 5 incógnitas, resolve-se o sistema. >> >>[resolvendo pelo Maple] >> >>{e = 0, a = 1/4, b = 1/2, c = 1/4, d = 0} >> >>.:. S(x) = x^4/4 + x^3/2 + x^2/4 > > >Olah, povo da PA de ordem superior. Posso sugerir que em vez de calcular >de >S(1) a S(5) calculemos de S(0) a S(4)? Se o aluno deve mesmo resolver o >sistema na prova, sem Maple, que resolva um sistema mais simples. No caso >em >particular, acho que d S(-2) a S(2) seria mais em conta, mas nao fiz o >calculo. Abracos, olavo. > >_________________________________________________________________ >MSN 8 with e-mail virus protection service: 2 months FREE* >http://join.msn.com/?page=features/virus > >========================================================================= >Instruções para entrar na lista, sair da lista e usar a lista em >http://www.mat.puc-rio.br/~nicolau/olimp/obm-l.html >========================================================================= > TEA WITH ME THAT I BOOK YOUR FACE ------------------------------------------ Use o melhor sistema de busca da Internet Radar UOL - http://www.radaruol.com.br ========================================================================= Instruções para entrar na lista, sair da lista e usar a lista em http://www.mat.puc-rio.br/~nicolau/olimp/obm-l.html ========================================================================= From owner-obm-l@sucuri.mat.puc-rio.br Thu Apr 24 13:13:11 2003 Return-Path: Received: (from majordom@localhost) by sucuri.mat.puc-rio.br (8.9.3/8.9.3) id NAA02467 for obm-l-MTTP; Thu, 24 Apr 2003 13:11:42 -0300 Received: from www.zipmail.com.br (smtp.zipmail.com.br [200.221.11.147]) by sucuri.mat.puc-rio.br (8.9.3/8.9.3) with ESMTP id NAA02463 for ; Thu, 24 Apr 2003 13:11:37 -0300 From: peterdirichlet1985@zipmail.com.br Received: from [200.206.103.3] by www.zipmail.com.br with HTTP; Thu, 24 Apr 2003 13:09:34 -0300 Message-ID: <3E9EC1890000EC0C@www.zipmail.com.br> Date: Thu, 24 Apr 2003 13:09:34 -0300 In-Reply-To: <007801c30a6a$05d59380$5400a8c0@ensrbr> Subject: [obm-l] =?iso-8859-1?Q?Re=3A=20Re=5B2=5D=3A=20=5Bobm=2Dl=5D=204=20coisinhas?= To: obm-l@mat.puc-rio.br MIME-Version: 1.0 Content-Type: text/plain; charset="iso-8859-1" Content-Transfer-Encoding: 8bit X-MIME-Autoconverted: from quoted-printable to 8bit by sucuri.mat.puc-rio.br id NAA02464 Sender: owner-obm-l@sucuri.mat.puc-rio.br Precedence: bulk Reply-To: obm-l@mat.puc-rio.br Falando serio isso nada mais e que Stifel disfarçado.Pega a Eureka 8 e divirta-se com o tio Quinhoes(ae JP,volta pra lista!!!!!!Sua presença e NECESSARIA!!!!!) -- Mensagem original -- >Sauda,c~oes, > >Oi Igor, > >Falou e disse! Legal o método, não? >Não tem nome, faz parte da teoria do cálculo >de diferenças. > >===Msg do Olavo >Olah, povo da PA de ordem superior. Posso sugerir que em vez de calcular >de >S(1) a S(5) calculemos de S(0) a S(4)? Se o aluno deve mesmo resolver o >sistema na prova, sem Maple, que resolva um sistema mais simples. No caso >em >particular, acho que d S(-2) a S(2) seria mais em conta, mas nao fiz o >calculo. Abracos, olavo. >=== >Na verdade S(1) a S(4) pois a_0 (ou e) = 0 para >qq PA de ordem k. Ou S(-1), S(1) a S(3). > >>S(n) = ax^4 + bx^3 + cx^2 + dx + e, faz-se > >> [resolvendo pelo Maple] >> >> {e = 0, a = 1/4, b = 1/2, c = 1/4, d = 0} >> >> .:. S(x) = x^4/4 + x^3/2 + x^2/4 >> >[]'s >Luís > >-----Mensagem Original----- >De: "Igor GomeZZ" >Para: "Luis Lopes" >Enviada em: terça-feira, 22 de abril de 2003 19:54 >Assunto: Re[2]: [obm-l] 4 coisinhas > > >> >> Em 22/4/2003, 17:13, Luis (llopes@ensrbr.com.br) disse: >> >> >> 4) >> >> Calcular S = 1 + 8 + 27 + ... + x^3 >> > Este o Igor deverá saber fazer. Fala, Igor! >> >. >> >> [resolvendo pelo Maple] >> >> {e = 0, a = 1/4, b = 1/2, c = 1/4, d = 0} >> >> .:. S(x) = x^4/4 + x^3/2 + x^2/4 >> >> >> >> ** Solução à la Luis (Qual o nome desse método?) >> >> S(n) = (/\[0]a[1])*binomial(n,1) + (/\[1]a[1])*binomial(n,2) + >> + (/\[2]a[1])*binomial(n,3) + (/\[3]a[1])*binom(n,4) >> >> S(n) = 1*binomial(n,1) + 7*binomial(n,2) + 12*binomial(n,3) + >6*binomial(n,4) >> >> .:. S(n) = n^4/4 + n^3/2 + n^2/4 >> >> > > >========================================================================= >Instruções para entrar na lista, sair da lista e usar a lista em >http://www.mat.puc-rio.br/~nicolau/olimp/obm-l.html >========================================================================= > TEA WITH ME THAT I BOOK YOUR FACE ------------------------------------------ Use o melhor sistema de busca da Internet Radar UOL - http://www.radaruol.com.br ========================================================================= Instruções para entrar na lista, sair da lista e usar a lista em http://www.mat.puc-rio.br/~nicolau/olimp/obm-l.html ========================================================================= From owner-obm-l@sucuri.mat.puc-rio.br Thu Apr 24 13:32:28 2003 Return-Path: Received: (from majordom@localhost) by sucuri.mat.puc-rio.br (8.9.3/8.9.3) id NAA03735 for obm-l-MTTP; Thu, 24 Apr 2003 13:30:55 -0300 Received: from www.zipmail.com.br (smtp.zipmail.com.br [200.221.11.147]) by sucuri.mat.puc-rio.br (8.9.3/8.9.3) with ESMTP id NAA03712 for ; Thu, 24 Apr 2003 13:30:46 -0300 From: peterdirichlet1985@zipmail.com.br Received: from [200.206.103.3] by www.zipmail.com.br with HTTP; Thu, 24 Apr 2003 13:28:37 -0300 Message-ID: <3E9EC1890000ED75@www.zipmail.com.br> Date: Thu, 24 Apr 2003 13:28:37 -0300 Subject: [obm-l] =?iso-8859-1?Q?Problema=20antigo=20sempre=20da=20historia=2E=2E=2E?= To: OBM-L@mat.puc-rio.br, edsonabe@terra.com.br MIME-Version: 1.0 Content-Type: text/plain; charset="iso-8859-1" Content-Transfer-Encoding: 8bit X-MIME-Autoconverted: from quoted-printable to 8bit by sucuri.mat.puc-rio.br id NAA03731 Sender: owner-obm-l@sucuri.mat.puc-rio.br Precedence: bulk Reply-To: obm-l@mat.puc-rio.br Oi gente!!!!!!!Tenho um problema de geometria que ja discuti na lista : "Considere o quadrilatero ABCD,tal que angDBC=60,angACB=50,angABD=20,angACD=30.Calcule todos os angulos do quadrilatero." Ja vi uma soluçao cearense bastante magica(segundo o autor!!!!!)nesta lista.Eu tenho de 4 a 5 soluçoes para isso.Mas sera que tem mais?Conto com voces para isso. TEA WITH ME THAT I BOOK YOUR FACE ------------------------------------------ Use o melhor sistema de busca da Internet Radar UOL - http://www.radaruol.com.br ========================================================================= Instruções para entrar na lista, sair da lista e usar a lista em http://www.mat.puc-rio.br/~nicolau/olimp/obm-l.html ========================================================================= From owner-obm-l@sucuri.mat.puc-rio.br Thu Apr 24 13:34:10 2003 Return-Path: Received: (from majordom@localhost) by sucuri.mat.puc-rio.br (8.9.3/8.9.3) id NAA03810 for obm-l-MTTP; Thu, 24 Apr 2003 13:32:45 -0300 Received: from www.zipmail.com.br (smtp.zipmail.com.br [200.221.11.147]) by sucuri.mat.puc-rio.br (8.9.3/8.9.3) with ESMTP id NAA03793 for ; Thu, 24 Apr 2003 13:32:22 -0300 From: peterdirichlet1985@zipmail.com.br Received: from [200.206.103.3] by www.zipmail.com.br with HTTP; Thu, 24 Apr 2003 13:28:37 -0300 Message-ID: <3E9EC1890000ED74@www.zipmail.com.br> Date: Thu, 24 Apr 2003 13:28:37 -0300 Subject: [obm-l] =?iso-8859-1?Q?Problema=20antigo=20sempre=20da=20historia=2E=2E=2E?= To: OBM-L@mat.puc-rio.br, edsonabe@terra.com.br MIME-Version: 1.0 Content-Type: text/plain; charset="iso-8859-1" Content-Transfer-Encoding: 8bit X-MIME-Autoconverted: from quoted-printable to 8bit by sucuri.mat.puc-rio.br id NAA03807 Sender: owner-obm-l@sucuri.mat.puc-rio.br Precedence: bulk Reply-To: obm-l@mat.puc-rio.br Oi gente!!!!!!!Tenho um problema de geometria que ja discuti na lista : "Considere o quadrilatero ABCD,tal que angDBC=60,angACB=50,angABD=20,angACD=30.Calcule todos os angulos do quadrilatero." Ja vi uma soluçao cearense bastante magica(segundo o autor!!!!!)nesta lista.Eu tenho de 4 a 5 soluçoes para isso.Mas sera que tem mais?Conto com voces para isso. TEA WITH ME THAT I BOOK YOUR FACE ------------------------------------------ Use o melhor sistema de busca da Internet Radar UOL - http://www.radaruol.com.br ========================================================================= Instruções para entrar na lista, sair da lista e usar a lista em http://www.mat.puc-rio.br/~nicolau/olimp/obm-l.html ========================================================================= From owner-obm-l@sucuri.mat.puc-rio.br Thu Apr 24 13:37:03 2003 Return-Path: Received: (from majordom@localhost) by sucuri.mat.puc-rio.br (8.9.3/8.9.3) id NAA03954 for obm-l-MTTP; Thu, 24 Apr 2003 13:35:42 -0300 Received: from traven9.uol.com.br (traven9.uol.com.br [200.221.29.35]) by sucuri.mat.puc-rio.br (8.9.3/8.9.3) with ESMTP id NAA03933 for ; Thu, 24 Apr 2003 13:35:19 -0300 Received: from gauss ([200.158.97.252]) by traven9.uol.com.br (8.9.1/8.9.1) with SMTP id NAA05737 for ; Thu, 24 Apr 2003 13:34:37 -0300 (BRT) Message-ID: <001601c30a80$211a9df0$7d07fea9@gauss> From: "Domingos Jr." To: References: Subject: Re: [obm-l] FW: Problema dificil Date: Thu, 24 Apr 2003 13:39:50 -0300 MIME-Version: 1.0 Content-Type: text/plain; charset="iso-8859-1" Content-Transfer-Encoding: 8bit X-Priority: 3 X-MSMail-Priority: Normal X-Mailer: Microsoft Outlook Express 6.00.2800.1106 X-MimeOLE: Produced By Microsoft MimeOLE V6.00.2800.1106 Sender: owner-obm-l@sucuri.mat.puc-rio.br Precedence: bulk Reply-To: obm-l@mat.puc-rio.br Olá, Cláudio não esquenta não, eu só mandei a mensagem pq pode acontecer de você não ter conseguido pegar o arquivo ou simplesmente não viu a mensagem postada anteriormente... a propósito, ontem mesmo eu tinha chegado num resultado parcial dado pelo Márcio: os 9 primeiros termos da seq. são todos iguais (creio que, analogamente, os 9 últimos tb são iguais), porém, daí a concluir que todos os elementos são iguais parece ser um pouco precipitado, acho que precisa de alguma sacada pra chegar lá, mas ainda não vi a luz! uma pergunta: o que vc faz (curso universitário/mestrado, outros...)? [ ]'s > Oi, Domingos: > > Agradeco a sua atencao ao problema. De fato, eu ja imprimi uma copia da sua > solucao pra analisar com calma. > > Nao eh falta de interesse - muito pelo contrario. So que eu nao tenho > conseguido me organizar pra dar uma estudada mais a fundo nos problemas mais > dificeis. > > Estou com uma pilha de e-mails seus e de varios outros colegas da lista aos > quais ainda nao consegui dar a atencao devida. Isso pra nao falar duns 30 > artigos, problemas, "lecture notes" e trechos de livros que tambem separei > pra olhar com calma quando tiver um tempo. > > O chato eh que tem umas coisas que eu nao consigo nem comecar a ler na tela > do computador. Tenho que imprimir e reescrever todo o argumento se quiser > ter uma chance de entender... > > No momento, so tem dado pra acompanhar a lista e mandar as solucoes dos > problemas que eu consigo resolver no computador mesmo. > > > Um abraco, > Claudio. ========================================================================= Instruções para entrar na lista, sair da lista e usar a lista em http://www.mat.puc-rio.br/~nicolau/olimp/obm-l.html ========================================================================= From owner-obm-l@sucuri.mat.puc-rio.br Thu Apr 24 14:05:43 2003 Return-Path: Received: (from majordom@localhost) by sucuri.mat.puc-rio.br (8.9.3/8.9.3) id OAA06125 for obm-l-MTTP; Thu, 24 Apr 2003 14:02:45 -0300 Received: from ns-3.idc.dglnet.com.br (ns-3.idc.dglnet.com.br [200.218.161.4]) by sucuri.mat.puc-rio.br (8.9.3/8.9.3) with ESMTP id OAA06119 for ; Thu, 24 Apr 2003 14:02:38 -0300 Received: from juciz (200-100-107-37.dsl.telesp.net.br [200.100.107.37]) by ns-3.idc.dglnet.com.br (Postfix) with ESMTP id 406B8150038 for ; Thu, 24 Apr 2003 14:02:10 -0300 (BRT) Message-ID: <000901c30a83$2b552620$256b64c8@dglnet.com.br> From: "Piola" To: References: <200304240006.h3O066001829@Euler.impa.br> Subject: Re: [obm-l] Ensino de matematica no Brasil Date: Thu, 24 Apr 2003 14:01:33 -0300 MIME-Version: 1.0 Content-Type: text/plain; charset="iso-8859-1" Content-Transfer-Encoding: 8bit X-Priority: 3 X-MSMail-Priority: Normal X-Mailer: Microsoft Outlook Express 5.00.2314.1300 X-MimeOLE: Produced By Microsoft MimeOLE V5.00.2314.1300 Sender: owner-obm-l@sucuri.mat.puc-rio.br Precedence: bulk Reply-To: obm-l@mat.puc-rio.br Caro Professor Eduardo Seria excelente se todos nós pudéssemos receber notícias sobre esse grande encontro em Antigua. Penso que o assunto é sim do interesse dessa Lista! É claro que não se pode ensinar aquilo que mal se sabe. Prof. Piola. ----- Original Message ----- From: Eduardo Wagner To: Sent: Wednesday, April 23, 2003 9:04 PM Subject: Re: [obm-l] Ensino de matematica no Brasil > A lista nao eh realmente o forum adequado para discussao > sobre ensino de matematica, mas nao posso deixar de > registrar o meu total apoio em relacao ao que disse > a prof. Suely Druck. > Lamentamos o artigo do prof. Kulesza que mostrou nao > compreender o que leu e que nem soube rabater os solidos > argumentos da prof. Suely. O artigo do prof. Kulesza eh > inconsistente e mediocre. A resposta do prof. Michel Spira > nao poderia ser melhor. Disse tudo o que era necessario. > A preocupacao com a melhoria do conteudo dos professores > do ensino fundamental em medio eh tema atual em inumeros > paises. > Para a informacao de todos, a OEI (Organizacao > dos Estados Iberoamericanos para Ciencia e Cultura) > esta promovendo nos dias 28 a 30 de abril um grande > simposio em Antigua, reunindo professores de matematica, > educadores e autoridades em educacao de todos os paises > de lingua espanhola e portuguesa para tratar exatamente > deste tema: a melhoria do conteudo matematico dos > professores para um melhor ensino. O Brasil estara > fortemente presente, transmitindo a experiencia que > temos no assunto atraves dos programas patrocinados > pelo IMPA e pela Sociedade Brasileira de Matematica. > > Eduardo Wagner. > > PS. Benedito: foi bom voce ter escrito pois me motivou a > escrever tambem. Sera que alguem pode ensinar portugues > sem conhecer a lingua portuguesa? > > ---------- > >From: benedito > >To: obm-l@mat.puc-rio.br > >Subject: Re: [obm-l] Ensino de matematica no Brasil > >Date: Mon, Apr 21, 2003, 8:22 PM > > > > > Muita gente pode não ter gostado....Mas as declarações da Presidente tem > > muita verdade, isso tem. Incomoda alguns, especialmente aqueles que focam o > > ensino nos métodos, esquecendo o principal: o conteúdo... > > Benedito Freire > > > > At 19:30 21/4/2003 -0300, you wrote: > >>Não é exatamente o assunto da lista, mas a Suely Druck (presidente da SBM) > >>publicou um artigo na Folha sobre o ensino de matemática no Brasil. > >>As olimpíadas de matemática são mencionadas mas não são o tema do artigo. > >>Nem todos os leitores gostaram gostaram. Vejam os links abaixo > >>(obrigado ao Gugu pela referência) vale a pena ler e refletir, > >>talvez participar do debate. []s, N. > >> > >>http://www1.folha.uol.com.br/folha/sinapse/ult1063u343.shtml > >>http://www.jornaldaciencia.org.br/Detalhe.jsp?id=8868 > >>http://www.jornaldaciencia.org.br/Detalhe.jsp?id=9238 > >>http://www.jornaldaciencia.org.br/Detalhe.jsp?id=9105 > >> > >> > >> > >>========================================================================= > >>Instruções para entrar na lista, sair da lista e usar a lista em > >>http://www.mat.puc-rio.br/~nicolau/olimp/obm-l.html > >>O administrador desta lista é > >>========================================================================= > > > > > > ========================================================================= > > Instruções para entrar na lista, sair da lista e usar a lista em > > http://www.mat.puc-rio.br/~nicolau/olimp/obm-l.html > > O administrador desta lista é > > ========================================================================= > ========================================================================= > Instruções para entrar na lista, sair da lista e usar a lista em > http://www.mat.puc-rio.br/~nicolau/olimp/obm-l.html > ========================================================================= > ========================================================================= Instruções para entrar na lista, sair da lista e usar a lista em http://www.mat.puc-rio.br/~nicolau/olimp/obm-l.html ========================================================================= From owner-obm-l@sucuri.mat.puc-rio.br Thu Apr 24 14:44:56 2003 Return-Path: Received: (from majordom@localhost) by sucuri.mat.puc-rio.br (8.9.3/8.9.3) id OAA07681 for obm-l-MTTP; Thu, 24 Apr 2003 14:43:00 -0300 Received: from mail.vetor.com.br (wool.vetor.com.br [200.160.244.7]) by sucuri.mat.puc-rio.br (8.9.3/8.9.3) with ESMTP id OAA07677 for ; Thu, 24 Apr 2003 14:42:56 -0300 Received: from UebiMiau (wool [200.160.244.7]) by mail.vetor.com.br (8.12.5) with SMTP id h3OHgMnY002841 for ; Thu, 24 Apr 2003 14:42:22 -0300 Message-Id: <200304241742.h3OHgMnY002841@mail.vetor.com.br> Received: from client 200.160.236.245 for UebiMiau (webmail client); Thu, 24 Apr 2003 14:42:22 -0200 Date: Thu, 24 Apr 2003 14:42:22 -0200 From: Villard To: Subject: Re: [obm-l] Problema antigo sempre da historia... X-Priority: 3 X-Mailer: UebiMiau 2.5 Content-Transfer-Encoding: 8bit X-MSMail-Priority: Medium Importance: Medium Content-Type: text/html; charset="iso-8859-1"; MIME-Version: 1.0 X-Virus-Scanned: by amavis at mail.vetor.com.br Sender: owner-obm-l@sucuri.mat.puc-rio.br Precedence: bulk Reply-To: obm-l@mat.puc-rio.br

Pelo visto voce quer trigonometria... então vai (sem figura, é claro)

Suponha sem perdas AB=1 (logo BC=1, pois o triangulo ABC é isósceles). Seja ang(ABD)=u.

Lei dos senos em ACD : AD*sen(u)=sen20

Lei dos senos em ABD : AD*sen(u+40)=AC*sen30.

Como AC=2*cos50, então dividindo uma equação pela outra temos que sen(u+40)*sen20 = sen(u)*cos50 = sen(u)*sen40 = sen(u)*2*sen20*cos20, logo sen(u+40)=2sen(u)cos20=sen(u+20)+sen(u-20), ou seja sen(u+40) - sen(u-20) = sen(u+20)

Transformando em produto... 2*sen30*cos(u)=sen(u+20), então sen(90-u)=sen(u+20). Como (90-u)+(u+20)=110, então 90-u e u+20 são côngruos ou seja (u+20)-(90-u) =2u-70= 360*k (Aqui já é de se esperar que u=35 seja a única solução).

Agora, é fácil ver que 0<u<110 (pra soma dos ângulos do triângulo não passar de 180), logo -70/360 < k < 70/360 e como k é inteiro temos k=0, então u = 35.

Abraços,

Villard

--------- Mensagem Original --------
De: peterdirichlet1985@zipmail.com.br
Para: "OBM-L@mat.puc-rio.br" <OBM-L@mat.puc-rio.br>, "edsonabe@terra.com.br" <edsonabe@terra.com.br>
Assunto: [obm-l] Problema antigo sempre da historia...
Data: 24/04/03 16:55

Oi gente!!!!!!!Tenho um problema de geometria que ja discuti na lista :
"Considere o quadrilatero ABCD,tal que angDBC=60,angACB=50,angABD=20,angACD=30.Calcule
todos os angulos do quadrilatero."
Ja vi uma soluçao cearense bastante magica(segundo o autor!!!!!)nesta lista.Eu
tenho de 4 a 5 soluçoes para isso.Mas sera que tem mais?Conto com voces
para isso.

TEA WITH ME THAT I BOOK YOUR FACE


------------------------------------------
Use o melhor sistema de busca da Internet
Radar UOL - http://www.radaruol.com.br



=========================================================================
Instruções para entrar na lista, sair da lista e usar a lista em
http://www.mat.puc-rio.br/~nicolau/olimp/obm-l.html
=========================================================================

========================================================================= Instruções para entrar na lista, sair da lista e usar a lista em http://www.mat.puc-rio.br/~nicolau/olimp/obm-l.html ========================================================================= From owner-obm-l@sucuri.mat.puc-rio.br Thu Apr 24 15:10:30 2003 Return-Path: Received: (from majordom@localhost) by sucuri.mat.puc-rio.br (8.9.3/8.9.3) id PAA08819 for obm-l-MTTP; Thu, 24 Apr 2003 15:08:39 -0300 Received: from itaqui.terra.com.br (itaqui.terra.com.br [200.176.3.19]) by sucuri.mat.puc-rio.br (8.9.3/8.9.3) with ESMTP id PAA08813 for ; Thu, 24 Apr 2003 15:08:35 -0300 Received: from altamira.terra.com.br (altamira.terra.com.br [200.176.3.40]) by itaqui.terra.com.br (Postfix) with ESMTP id 613893BD55F for ; Thu, 24 Apr 2003 15:07:54 -0300 (BRT) Received: from usuario (200-203-035-087.paemt7005.dsl.brasiltelecom.net.br [200.203.35.87]) (authenticated user marioappereira) by altamira.terra.com.br (Postfix) with ESMTP id DDC183DC0ED for ; Thu, 24 Apr 2003 15:07:53 -0300 (BRT) Message-ID: <00a601c30a8c$758deac0$0301a8c0@usuario> From: =?iso-8859-1?Q?M=E1rio_Pereira?= To: References: <3E9EC1890000EB87@www.zipmail.com.br> Subject: Re: [obm-l] Re: [obm-l] euclidiana Date: Thu, 24 Apr 2003 15:08:07 -0300 MIME-Version: 1.0 Content-Type: text/plain; charset="iso-8859-1" Content-Transfer-Encoding: 8bit X-Priority: 3 X-MSMail-Priority: Normal X-Mailer: Microsoft Outlook Express 6.00.2600.0000 X-MimeOLE: Produced By Microsoft MimeOLE V6.00.2600.0000 Sender: owner-obm-l@sucuri.mat.puc-rio.br Precedence: bulk Reply-To: obm-l@mat.puc-rio.br A pergunta da prova foi essa. Mário ----- Original Message ----- From: To: Sent: Thursday, April 24, 2003 1:02 PM Subject: [obm-l] Re: [obm-l] euclidiana > > Pode traduzir? > -- Mensagem original -- > > > > >Como podemos verificar que um sistema de unidades de medidas é, de fato, > >um sistema? > > > >Obrigado, > > > >Mário. > > > > TEA WITH ME THAT I BOOK YOUR FACE > > > ------------------------------------------ > Use o melhor sistema de busca da Internet > Radar UOL - http://www.radaruol.com.br > > > > ========================================================================= > Instruções para entrar na lista, sair da lista e usar a lista em > http://www.mat.puc-rio.br/~nicolau/olimp/obm-l.html > ========================================================================= > > ========================================================================= Instruções para entrar na lista, sair da lista e usar a lista em http://www.mat.puc-rio.br/~nicolau/olimp/obm-l.html ========================================================================= From owner-obm-l@sucuri.mat.puc-rio.br Thu Apr 24 15:29:01 2003 Return-Path: Received: (from majordom@localhost) by sucuri.mat.puc-rio.br (8.9.3/8.9.3) id PAA09492 for obm-l-MTTP; Thu, 24 Apr 2003 15:27:11 -0300 Received: from ns3bind.localdomain ([200.230.34.5]) by sucuri.mat.puc-rio.br (8.9.3/8.9.3) with ESMTP id PAA09488 for ; Thu, 24 Apr 2003 15:27:07 -0300 Received: from servico2 ([200.230.34.229]) by ns3bind.localdomain (8.11.6/X.XX.X) with SMTP id h3OIMRa04224 for ; Thu, 24 Apr 2003 15:22:27 -0300 Message-ID: <007401c30a8f$3837fbe0$3300c57d@bovespa.com> From: "=?iso-8859-1?Q?Cl=E1udio_\=28Pr=E1tica\=29?=" To: References: <3E9EC1890000EB42@www.zipmail.com.br> Subject: Re: [obm-l] Re: [obm-l] TEOIREMA DE CRISTEA::Geometria,pontos estranhos e problemas legais!!!! Date: Thu, 24 Apr 2003 15:27:52 -0300 MIME-Version: 1.0 Content-Type: text/plain; charset="iso-8859-1" Content-Transfer-Encoding: 8bit X-Priority: 3 X-MSMail-Priority: Normal X-Mailer: Microsoft Outlook Express 5.50.4920.2300 X-MimeOLE: Produced By Microsoft MimeOLE V5.50.4920.2300 Sender: owner-obm-l@sucuri.mat.puc-rio.br Precedence: bulk Reply-To: obm-l@mat.puc-rio.br Caro Dirichlet: Consegui demonstrar este resultado. Estou disposto a mandar minha solução pra lista sob uma condição: que, de hoje em diante, você só mande mensagens relevantes, com nexo, e caso tenha comentários pra fazer, que eles sejam polidos, pertinentes, e que ajudem as pessoas a entender melhor um problema ou conceito. Eu sei que você é capaz, afinal a demonstração do Último Teorema de Fermat para n = 4 que você mandou pra lista foi uma das melhores que eu já vi. Se você concordar, eu mando a solução imediatamente e "free of charge". Topa? Um abraço, Claudio. ----- Original Message ----- From: To: Sent: Thursday, April 24, 2003 12:59 PM Subject: [obm-l] Re: [obm-l] TEOIREMA DE CRISTEA::Geometria,pontos estranhos e problemas legais!!!! > MAS COMO???!?!??!!:???!Ja estou ha semanas tentando sem sucesso ou expectativa.Nenhum > triangulo da muitas esperanças... > > > -- Mensagem original -- > > >on 22.04.03 21:25, Johann Peter Gustav Lejeune Dirichlet at > >peterdirichlet2002@yahoo.com.br wrote: > > > >Ola gente!!!!Estou tentando resolver problemas de Geometria do Mathematical > >excalibur mas preciso de um teorema.Quem pode demonstrar pra mim: > > > >Considere o triangulo ABC e um ponto T.Duas cevianas CT_c e BT_b se cortam > >erm T e os pontos R_c,R_b e T sao alinhados,com R_c em AB e R_b em AC.Mostre > >que > > > >AT_c*BR_c/T_cB/R_cA + ATb*CR_b//T_bC/R_bA = 1 > > > >Nao sei se e isso mesmo mas vale o risco... > > > > > >Oi, JP: > > > >Supondo que a expressao acima seja: > > > >(AT_c * BR_c) / (BT_c * AR_c) + ( AT_b * CR_b ) / (CT_b * AR_b ) = 1 > > > >eu diria que uma boa aposta seria o teorema de Menelau (aplicado mais de > >uma > >vez). > > > > > >Um abraco, > >Claudio. > > > > > > > > TEA WITH ME THAT I BOOK YOUR FACE > > > ------------------------------------------ > Use o melhor sistema de busca da Internet > Radar UOL - http://www.radaruol.com.br > > > > ========================================================================= > Instruções para entrar na lista, sair da lista e usar a lista em > http://www.mat.puc-rio.br/~nicolau/olimp/obm-l.html > ========================================================================= ========================================================================= Instruções para entrar na lista, sair da lista e usar a lista em http://www.mat.puc-rio.br/~nicolau/olimp/obm-l.html ========================================================================= From owner-obm-l@sucuri.mat.puc-rio.br Thu Apr 24 15:34:42 2003 Return-Path: Received: (from majordom@localhost) by sucuri.mat.puc-rio.br (8.9.3/8.9.3) id PAA09780 for obm-l-MTTP; Thu, 24 Apr 2003 15:33:10 -0300 Received: from web12904.mail.yahoo.com (web12904.mail.yahoo.com [216.136.174.71]) by sucuri.mat.puc-rio.br (8.9.3/8.9.3) with SMTP id PAA09774 for ; Thu, 24 Apr 2003 15:33:05 -0300 Message-ID: <20030424183233.71365.qmail@web12904.mail.yahoo.com> Received: from [200.206.103.3] by web12904.mail.yahoo.com via HTTP; Thu, 24 Apr 2003 15:32:33 ART Date: Thu, 24 Apr 2003 15:32:33 -0300 (ART) From: =?iso-8859-1?q?Johann=20Peter=20Gustav=20Lejeune=20Dirichlet?= Subject: Re: [obm-l] Ensino de matematica no Brasil To: obm-l@mat.puc-rio.br In-Reply-To: <000901c30a83$2b552620$256b64c8@dglnet.com.br> MIME-Version: 1.0 Content-Type: multipart/alternative; boundary="0-343641750-1051209153=:70924" Content-Transfer-Encoding: 8bit Sender: owner-obm-l@sucuri.mat.puc-rio.br Precedence: bulk Reply-To: obm-l@mat.puc-rio.br --0-343641750-1051209153=:70924 Content-Type: text/plain; charset=iso-8859-1 Content-Transfer-Encoding: 8bit Piola wrote: Caro Professor Eduardo Seria excelente se todos nós pudéssemos receber notícias sobre esse grande encontro em Antigua. Penso que o assunto é sim do interesse dessa Lista! É claro que não se pode ensinar aquilo que mal se sabe. Prof. Piola. ----- Original Message ----- From: Eduardo Wagner To: Sent: Wednesday, April 23, 2003 9:04 PM Subject: Re: [obm-l] Ensino de matematica no Brasil > A lista nao eh realmente o forum adequado para discussao > sobre ensino de matematica, mas nao posso deixar de > registrar o meu total apoio em relacao ao que disse > a prof. Suely Druck. > Lamentamos o artigo do prof. Kulesza que mostrou nao > compreender o que leu e que nem soube rabater os solidos > argumentos da prof. Suely. O artigo do prof. Kulesza eh > inconsistente e mediocre. A resposta do prof. Michel Spira > nao poderia ser melhor. Disse tudo o que era necessario. > A preocupacao com a melhoria do conteudo dos professores > do ensino fundamental em medio eh tema atual em inumeros > paises. > Para a informacao de todos, a OEI (Organizacao > dos Estados Iberoamericanos para Ciencia e Cultura) > esta promovendo nos dias 28 a 30 de abril um grande > simposio em Antigua, reunindo professores de matematica, > educadores e autoridades em educacao de todos os paises > de lingua espanhola e portuguesa para tratar exatamente > deste tema: a melhoria do conteudo matematico dos > professores para um melhor ensino. O Brasil estara > fortemente presente, transmitindo a experiencia que > temos no assunto atraves dos programas patrocinados > pelo IMPA e pela Sociedade Brasileira de Matematica. > > Eduardo Wagner. > > PS. Benedito: foi bom voce ter escrito pois me motivou a > escrever tambem. Sera que alguem pode ensinar portugues > sem conhecer a lingua portuguesa? > > ---------- > >From: benedito > >To: obm-l@mat.puc-rio.br > >Subject: Re: [obm-l] Ensino de matematica no Brasil > >Date: Mon, Apr 21, 2003, 8:22 PM > > > > > Muita gente pode não ter gostado....Mas as declarações da Presidente tem > > muita verdade, isso tem. Incomoda alguns, especialmente aqueles que focam o > > ensino nos métodos, esquecendo o principal: o conteúdo... > > Benedito Freire > > Quanto a isso eu realmente concordo.Quale a deles,ensinar sem saber?????? Alias algo um pouco fora de questao:desfde quando se odeia (matematica,neste caso) a primeira vista? --------------------------------- Yahoo! Mail O melhor e-mail gratuito da internet: 6MB de espaço, antivírus, acesso POP3, filtro contra spam. --0-343641750-1051209153=:70924 Content-Type: text/html; charset=iso-8859-1 Content-Transfer-Encoding: 8bit

 

 Piola <barzeus@dglnet.com.br> wrote:

Caro Professor Eduardo
Seria excelente se todos nós pudéssemos receber notícias sobre esse grande
encontro em Antigua. Penso que o assunto é sim do interesse dessa Lista!
É claro que não se pode ensinar aquilo que mal se sabe.
Prof. Piola.
----- Original Message -----
From: Eduardo Wagner
To:
Sent: Wednesday, April 23, 2003 9:04 PM
Subject: Re: [obm-l] Ensino de matematica no Brasil


> A lista nao eh realmente o forum adequado para discussao
> sobre ensino de matematica, mas nao posso deixar de
> registrar o meu total apoio em relacao ao que disse
> a prof. Suely Druck.
> Lamentamos o artigo do prof. Kulesza que mostrou nao
> compreender o que leu e que nem soube rabater os solidos
> argumentos da prof. Suely. O artigo do prof. Kulesza eh
> inconsistente e mediocre. A resposta do prof. Michel Spira
> nao poderia ser melhor. Disse tudo o que era necessario.
> A preocupacao com a melhoria do conteudo dos professores
> do ensino fundamental em medio eh tema atual em inumeros
> paises.
> Para a informacao de todos, a OEI (Organizacao
> dos Estados Iberoamericanos para Ciencia e Cultura)
> esta promovendo nos dias 28 a 30 de abril um grande
> simposio em Antigua, reunindo professores de matematica,
> educadores e autoridades em educacao de todos os paises
> de lingua espanhola e portuguesa para tratar exatamente
> deste tema: a melhoria do conteudo matematico dos
> professores para um melhor ensino. O Brasil estara
> fortemente presente, transmitindo a experiencia que
> temos no assunto atraves dos programas patrocinados
> pelo IMPA e pela Sociedade Brasileira de Matematica.
>
> Eduardo Wagner.
>
> PS. Benedito: foi bom voce ter escrito pois me motivou a
> escrever tambem. Sera que alguem pode ensinar portugues
> sem conhecer a lingua portuguesa?
>
> ----------
> >From: benedito
> >To: obm-l@mat.puc-rio.br
> >Subject: Re: [obm-l] Ensino de matematica no Brasil
> >Date: Mon, Apr 21, 2003, 8:22 PM
> >
>
> > Muita gente pode não ter gostado....Mas as declarações da Presidente
tem
> > muita verdade, isso tem. Incomoda alguns, especialmente aqueles que
focam o
> > ensino nos métodos, esquecendo o principal: o conteúdo...
> > Benedito Freire
> >
Quanto a isso eu realmente concordo.Quale a deles,ensinar sem saber??????

Alias algo um pouco fora de questao:desfde quando se odeia (matematica,neste caso) a primeira vista?



Yahoo! Mail
O melhor e-mail gratuito da internet: 6MB de espaço, antivírus, acesso POP3, filtro contra spam. --0-343641750-1051209153=:70924-- ========================================================================= Instruções para entrar na lista, sair da lista e usar a lista em http://www.mat.puc-rio.br/~nicolau/olimp/obm-l.html ========================================================================= From owner-obm-l@sucuri.mat.puc-rio.br Thu Apr 24 15:37:28 2003 Return-Path: Received: (from majordom@localhost) by sucuri.mat.puc-rio.br (8.9.3/8.9.3) id PAA09898 for obm-l-MTTP; Thu, 24 Apr 2003 15:36:03 -0300 Received: from web12904.mail.yahoo.com (web12904.mail.yahoo.com [216.136.174.71]) by sucuri.mat.puc-rio.br (8.9.3/8.9.3) with SMTP id PAA09875 for ; Thu, 24 Apr 2003 15:35:53 -0300 Message-ID: <20030424183521.71873.qmail@web12904.mail.yahoo.com> Received: from [200.206.103.3] by web12904.mail.yahoo.com via HTTP; Thu, 24 Apr 2003 15:35:21 ART Date: Thu, 24 Apr 2003 15:35:21 -0300 (ART) From: =?iso-8859-1?q?Johann=20Peter=20Gustav=20Lejeune=20Dirichlet?= Subject: Re: [obm-l] Combinatoria e |Desordens To: obm-l@mat.puc-rio.br In-Reply-To: <002101c309b2$e73bf200$3300c57d@bovespa.com> MIME-Version: 1.0 Content-Type: multipart/alternative; boundary="0-1309486872-1051209321=:70960" Content-Transfer-Encoding: 8bit Sender: owner-obm-l@sucuri.mat.puc-rio.br Precedence: bulk Reply-To: obm-l@mat.puc-rio.br --0-1309486872-1051209321=:70960 Content-Type: text/plain; charset=iso-8859-1 Content-Transfer-Encoding: 8bit Meu,eu ja fiz isso!!!!! Cláudio_(Prática) wrote:Oi, JP: Segue abaixo um pequeno artigo que, inicialmente, dá uma demonstração combinatória da seguinte equação de recorrência: K(1) = 0K(2) = 1K(n) = (n-1)*[K(n-1) + K(n-2)], para n >= 3 e em seguida, prova a sua por indução, com base na anterior. http://www.unc.edu/~rowlett/combin/notes/Derangements.pdf Espero que isso resolva. Um abraço,Claudio. ----- Original Message ----- From: Johann Peter Gustav Lejeune Dirichlet To: obm-l@mat.puc-rio.br Sent: Tuesday, April 22, 2003 8:02 PMSubject: [obm-l] Combinatoria e |Desordens Oi genten!!!!E ai,se alguem manja de permutaçao caotica(sem pontos fixos) de (1,2,3,...,n),se chamarmos K_n o numero de jeitos de permutar caoticamente,sera que da pra demonstrar COMBINATRORIALMENTE que K_n=n*K_(n-1)+(-1)^n? --------------------------------- Yahoo! Mail O melhor e-mail gratuito da internet: 6MB de espaço, antivírus, acesso POP3, filtro contra spam. --------------------------------- Yahoo! Mail O melhor e-mail gratuito da internet: 6MB de espaço, antivírus, acesso POP3, filtro contra spam. --0-1309486872-1051209321=:70960 Content-Type: text/html; charset=iso-8859-1 Content-Transfer-Encoding: 8bit

Meu,eu ja fiz isso!!!!!

 Cláudio_(Prática) <claudio@praticacorretora.com.br> wrote:

Oi, JP:
 
Segue abaixo um pequeno artigo que, inicialmente, dá uma demonstração combinatória da seguinte equação de recorrência:
 
K(1) = 0
K(2) = 1
K(n) = (n-1)*[K(n-1) + K(n-2)], para n >= 3
 
e em seguida, prova a sua por indução, com base na anterior.
 
 
Espero que isso resolva.
 
Um abraço,
Claudio.
 
----- Original Message -----
Sent: Tuesday, April 22, 2003 8:02 PM
Subject: [obm-l] Combinatoria e |Desordens

Oi genten!!!!E ai,se alguem manja de permutaçao caotica(sem pontos fixos) de (1,2,3,...,n),se chamarmos K_n o numero de jeitos de permutar caoticamente,sera que da pra demonstrar COMBINATRORIALMENTE que
K_n=n*K_(n-1)+(-1)^n?



Yahoo! Mail
O melhor e-mail gratuito da internet: 6MB de espaço, antivírus, acesso POP3, filtro contra spam.



Yahoo! Mail
O melhor e-mail gratuito da internet: 6MB de espaço, antivírus, acesso POP3, filtro contra spam. --0-1309486872-1051209321=:70960-- ========================================================================= Instruções para entrar na lista, sair da lista e usar a lista em http://www.mat.puc-rio.br/~nicolau/olimp/obm-l.html ========================================================================= From owner-obm-l@sucuri.mat.puc-rio.br Thu Apr 24 15:40:41 2003 Return-Path: Received: (from majordom@localhost) by sucuri.mat.puc-rio.br (8.9.3/8.9.3) id PAA10182 for obm-l-MTTP; Thu, 24 Apr 2003 15:39:15 -0300 Received: from ns3bind.localdomain ([200.230.34.5]) by sucuri.mat.puc-rio.br (8.9.3/8.9.3) with ESMTP id PAA10178 for ; Thu, 24 Apr 2003 15:39:10 -0300 Received: from servico2 ([200.230.34.229]) by ns3bind.localdomain (8.11.6/X.XX.X) with SMTP id h3OIYUX05335 for ; Thu, 24 Apr 2003 15:34:30 -0300 Message-ID: <007f01c30a90$e779e680$3300c57d@bovespa.com> From: "=?iso-8859-1?Q?Cl=E1udio_\=28Pr=E1tica\=29?=" To: References: <001601c30a80$211a9df0$7d07fea9@gauss> Subject: Re: [obm-l] FW: Problema dificil Date: Thu, 24 Apr 2003 15:39:55 -0300 MIME-Version: 1.0 Content-Type: text/plain; charset="iso-8859-1" Content-Transfer-Encoding: 8bit X-Priority: 3 X-MSMail-Priority: Normal X-Mailer: Microsoft Outlook Express 5.50.4920.2300 X-MimeOLE: Produced By Microsoft MimeOLE V5.50.4920.2300 Sender: owner-obm-l@sucuri.mat.puc-rio.br Precedence: bulk Reply-To: obm-l@mat.puc-rio.br Oi, Domingos: Eu sou só um amador. Sou formado em engenharia (graduação apenas - PUC-RJ/87), mas não exerço. Pra mim matemática é um apenas um hobby, mas ao qual eu tento me dedicar o máximo que eu posso. O problema é que eu também tenho que trabalhar, o que toma um pouco do meu tempo!!!!.... Sobre o meu problema, a primeira parte (provar que os primeiros 9 termos são iguais) acabou sendo útil no resto da minha solução. Espero que você ou alguém da lista consiga dar uma solução diferente. Daí talvez dê pra gente descobrir alguma generalização interessante. Um abraço, Claudio. ----- Original Message ----- From: "Domingos Jr." To: Sent: Thursday, April 24, 2003 1:39 PM Subject: Re: [obm-l] FW: Problema dificil > Olá, Cláudio > > não esquenta não, eu só mandei a mensagem pq pode acontecer de você não ter > conseguido pegar o arquivo ou simplesmente não viu a mensagem postada > anteriormente... > > a propósito, ontem mesmo eu tinha chegado num resultado parcial dado pelo > Márcio: os 9 primeiros termos da seq. são todos iguais (creio que, > analogamente, os 9 últimos tb são iguais), porém, daí a concluir que todos > os elementos são iguais parece ser um pouco precipitado, acho que precisa de > alguma sacada pra chegar lá, mas ainda não vi a luz! > > uma pergunta: o que vc faz (curso universitário/mestrado, outros...)? > > [ ]'s > > > Oi, Domingos: > > > > Agradeco a sua atencao ao problema. De fato, eu ja imprimi uma copia da > sua > > solucao pra analisar com calma. > > > > Nao eh falta de interesse - muito pelo contrario. So que eu nao tenho > > conseguido me organizar pra dar uma estudada mais a fundo nos problemas > mais > > dificeis. > > > > Estou com uma pilha de e-mails seus e de varios outros colegas da lista > aos > > quais ainda nao consegui dar a atencao devida. Isso pra nao falar duns 30 > > artigos, problemas, "lecture notes" e trechos de livros que tambem separei > > pra olhar com calma quando tiver um tempo. > > > > O chato eh que tem umas coisas que eu nao consigo nem comecar a ler na > tela > > do computador. Tenho que imprimir e reescrever todo o argumento se quiser > > ter uma chance de entender... > > > > No momento, so tem dado pra acompanhar a lista e mandar as solucoes dos > > problemas que eu consigo resolver no computador mesmo. > > > > > > Um abraco, > > Claudio. > > > ========================================================================= > Instruções para entrar na lista, sair da lista e usar a lista em > http://www.mat.puc-rio.br/~nicolau/olimp/obm-l.html > ========================================================================= ========================================================================= Instruções para entrar na lista, sair da lista e usar a lista em http://www.mat.puc-rio.br/~nicolau/olimp/obm-l.html ========================================================================= From owner-obm-l@sucuri.mat.puc-rio.br Thu Apr 24 15:50:25 2003 Return-Path: Received: (from majordom@localhost) by sucuri.mat.puc-rio.br (8.9.3/8.9.3) id PAA10699 for obm-l-MTTP; Thu, 24 Apr 2003 15:48:08 -0300 Received: from web12905.mail.yahoo.com (web12905.mail.yahoo.com [216.136.174.72]) by sucuri.mat.puc-rio.br (8.9.3/8.9.3) with SMTP id PAA10695 for ; Thu, 24 Apr 2003 15:48:03 -0300 Message-ID: <20030424184730.61006.qmail@web12905.mail.yahoo.com> Received: from [200.206.103.3] by web12905.mail.yahoo.com via HTTP; Thu, 24 Apr 2003 15:47:30 ART Date: Thu, 24 Apr 2003 15:47:30 -0300 (ART) From: =?iso-8859-1?q?Johann=20Peter=20Gustav=20Lejeune=20Dirichlet?= Subject: Re: [obm-l] Problema antigo sempre da historia... To: obm-l@mat.puc-rio.br In-Reply-To: <200304241742.h3OHgMnY002841@mail.vetor.com.br> MIME-Version: 1.0 Content-Type: multipart/alternative; boundary="0-1898713677-1051210050=:60084" Content-Transfer-Encoding: 8bit Sender: owner-obm-l@sucuri.mat.puc-rio.br Precedence: bulk Reply-To: obm-l@mat.puc-rio.br --0-1898713677-1051210050=:60084 Content-Type: text/plain; charset=iso-8859-1 Content-Transfer-Encoding: 8bit Valeu!!!!Eu ja tinha uma com trigonometria so que bem mais dificil.Com essa ja da 6!!!!!Depois passo todas. Villard wrote: Pelo visto voce quer trigonometria... então vai (sem figura, é claro) Suponha sem perdas AB=1 (logo BC=1, pois o triangulo ABC é isósceles). Seja ang(ABD)=u. Lei dos senos em ACD : AD*sen(u)=sen20 Lei dos senos em ABD : AD*sen(u+40)=AC*sen30. Como AC=2*cos50, então dividindo uma equação pela outra temos que sen(u+40)*sen20 = sen(u)*cos50 = sen(u)*sen40 = sen(u)*2*sen20*cos20, logo sen(u+40)=2sen(u)cos20=sen(u+20)+sen(u-20), ou seja sen(u+40) - sen(u-20) = sen(u+20) Transformando em produto... 2*sen30*cos(u)=sen(u+20), então sen(90-u)=sen(u+20). Como (90-u)+(u+20)=110, então 90-u e u+20 são côngruos ou seja (u+20)-(90-u) =2u-70= 360*k (Aqui já é de se esperar que u=35 seja a única solução). Agora, é fácil ver que 0, "edsonabe@terra.com.br" Assunto: [obm-l] Problema antigo sempre da historia... Data: 24/04/03 16:55 Oi gente!!!!!!!Tenho um problema de geometria que ja discuti na lista : "Considere o quadrilatero ABCD,tal que angDBC=60,angACB=50,angABD=20,angACD=30.Calcule todos os angulos do quadrilatero." Ja vi uma soluçao cearense bastante magica(segundo o autor!!!!!)nesta lista.Eu tenho de 4 a 5 soluçoes para isso.Mas sera que tem mais?Conto com voces para isso. TEA WITH ME THAT I BOOK YOUR FACE ------------------------------------------ Use o melhor sistema de busca da Internet Radar UOL - http://www.radaruol.com.br ========================================================================= Instruções para entrar na lista, sair da lista e usar a lista em http://www.mat.puc-rio.br/~nicolau/olimp/obm-l.html ========================================================================= ========================================================================= Instruções para entrar na lista, sair da lista e usar a lista em http://www.mat.puc-rio.br/~nicolau/olimp/obm-l.html ========================================================================= --------------------------------- Yahoo! Mail O melhor e-mail gratuito da internet: 6MB de espaço, antivírus, acesso POP3, filtro contra spam. --0-1898713677-1051210050=:60084 Content-Type: text/html; charset=iso-8859-1 Content-Transfer-Encoding: 8bit

Valeu!!!!Eu ja tinha uma com trigonometria so que bem mais dificil.Com essa ja da 6!!!!!Depois passo todas.

 Villard <villard@vetor.com.br> wrote:

Pelo visto voce quer trigonometria... então vai (sem figura, é claro)

Suponha sem perdas AB=1 (logo BC=1, pois o triangulo ABC é isósceles). Seja ang(ABD)=u.

Lei dos senos em ACD : AD*sen(u)=sen20

Lei dos senos em ABD : AD*sen(u+40)=AC*sen30.

Como AC=2*cos50, então dividindo uma equação pela outra temos que sen(u+40)*sen20 = sen(u)*cos50 = sen(u)*sen40 = sen(u)*2*sen20*cos20, logo sen(u+40)=2sen(u)cos20=sen(u+20)+sen(u-20), ou seja sen(u+40) - sen(u-20) = sen(u+20)

Transformando em produto... 2*sen30*cos(u)=sen(u+20), então sen(90-u)=sen(u+20). Como (90-u)+(u+20)=110, então 90-u e u+20 são côngruos ou seja (u+20)-(90-u) =2u-70= 360*k (Aqui já é de se esperar que u=35 seja a única solução).

Agora, é fácil ver que 0<u<110 (pra soma dos ângulos do triângulo não passar de 180), logo -70/360 < k < 70/360 e como k é inteiro temos k=0, então u = 35.

Abraços,

Villard

--------- Mensagem Original --------
De: peterdirichlet1985@zipmail.com.br
Para: "OBM-L@mat.puc-rio.br" <OBM-L@mat.puc-rio.br>, "edsonabe@terra.com.br" <edsonabe@terra.com.br>
Assunto: [obm-l] Problema antigo sempre da historia...
Data: 24/04/03 16:55

Oi gente!!!!!!!Tenho um problema de geometria que ja discuti na lista :
"Considere o quadrilatero ABCD,tal que angDBC=60,angACB=50,angABD=20,angACD=30.Calcule
todos os angulos do quadrilatero."
Ja vi uma soluçao cearense bastante magica(segundo o autor!!!!!)nesta lista.Eu
tenho de 4 a 5 soluçoes para isso.Mas sera que tem mais?Conto com voces
para isso.

TEA WITH ME THAT I BOOK YOUR FACE


------------------------------------------
Use o melhor sistema de busca da Internet
Radar UOL - http://www.radaruol.com.br



=========================================================================
Instruções para entrar na lista, sair da lista e usar a lista em
http://www.mat.puc-rio.br/~nicolau/olimp/obm-l.html
=========================================================================

========================================================================= Instruções para entrar na lista, sair da lista e usar a lista em http://www.mat.puc-rio.br/~nicolau/olimp/obm-l.html =========================================================================



Yahoo! Mail
O melhor e-mail gratuito da internet: 6MB de espaço, antivírus, acesso POP3, filtro contra spam. --0-1898713677-1051210050=:60084-- ========================================================================= Instruções para entrar na lista, sair da lista e usar a lista em http://www.mat.puc-rio.br/~nicolau/olimp/obm-l.html ========================================================================= From owner-obm-l@sucuri.mat.puc-rio.br Thu Apr 24 15:53:52 2003 Return-Path: Received: (from majordom@localhost) by sucuri.mat.puc-rio.br (8.9.3/8.9.3) id PAA11069 for obm-l-MTTP; Thu, 24 Apr 2003 15:52:19 -0300 Received: from ivoti.terra.com.br (ivoti.terra.com.br [200.176.3.20]) by sucuri.mat.puc-rio.br (8.9.3/8.9.3) with ESMTP id PAA11064 for ; Thu, 24 Apr 2003 15:52:14 -0300 Received: from barra.terra.com.br (barra.terra.com.br [200.176.3.52]) by ivoti.terra.com.br (Postfix) with ESMTP id DD6A94091A4 for ; Thu, 24 Apr 2003 15:51:41 -0300 (BRT) Received: from niski.com (unknown [200.148.202.95]) (authenticated user fniski) by barra.terra.com.br (Postfix) with ESMTP id A6781234090 for ; Thu, 24 Apr 2003 15:51:34 -0300 (BRT) Message-ID: <3EA8242C.1050002@niski.com> Date: Thu, 24 Apr 2003 10:51:40 -0700 From: niski User-Agent: Mozilla/5.0 (Windows; U; Windows NT 5.1; en-US; rv:1.0.2) Gecko/20030208 Netscape/7.02 X-Accept-Language: en-us, en MIME-Version: 1.0 To: obm-l@mat.puc-rio.br Subject: [obm-l] e mais probabilidades! Content-Type: text/plain; charset=ISO-8859-1; format=flowed Content-Transfer-Encoding: 8bit Sender: owner-obm-l@sucuri.mat.puc-rio.br Precedence: bulk Reply-To: obm-l@mat.puc-rio.br Ola pessoal...segue outro problema que nao sei onde estou errando, gostaria que me ajudassem. Muito obrigado! As cinco primeiras repeticoes de um experimento custam $10,00 cada. Todas as repeticoes subsequentes custam $5,00 cada. Suponha que o experimento seja repetido até que o primeiro sucesso ocorra. Se a probabilidade de sucesso de uma repeticao é igual a 0,9, e se as repeticoes sao independentes, qual é o custo esperado da operação? Minha resolucao : Trate-se de calcular a esperança matematica. Dividirei em dois casos i) Esperanca para até 5 repeticoes ii) Esperanca para 6 ou mais repeticoes Somarei i com ii e esta deverá ser a resposta do problema. Seja X a variavel aleatoria que representa o custo por operacao. i) Temos a seguinte distribuição: X = 10 -> p(X=10) = 0,9 X = 20 -> p(X=20) = (0,9).(0,1) ... X = 50 -> p(X=50) = (0,9).(0,1)^4 E'(X) , para até 5 eventos = 11,1105 ii) Cabe notar que a função que dá o custo pelo numero n de repeticoes (n > 5) é dada por C(n) = 50 + (((n-1)-5)+1).5 ou C(n) = 50 + (n-5).5 Então a esperança matematica E''(x) é dada pela somatoria Sum[n=6 até infinito] (50 + (n-5)*5).(0,9).(0,1)^(n-1) , calculei isso no mathematica , teria como calcular na mao( soma de pg infinita por exemplo?) E''(X) = 0,000555556 Então E(X) = E'(X) + E''(X) E(X) = 11,111055556 Porem, a minha resposta indica : 9,99 Gostaria de saber qual dos dois esta errado, e se é possivel o livro ter abordado outra solucao para chegar neste outro resultado no caso de eu e o livro estarmos certos. Obrigado. -- [about him:] It is rare to find learned men who are clean, do not stink and have a sense of humour. -Gottfried Whilhem Leibniz ========================================================================= Instruções para entrar na lista, sair da lista e usar a lista em http://www.mat.puc-rio.br/~nicolau/olimp/obm-l.html ========================================================================= From owner-obm-l@sucuri.mat.puc-rio.br Thu Apr 24 16:08:55 2003 Return-Path: Received: (from majordom@localhost) by sucuri.mat.puc-rio.br (8.9.3/8.9.3) id QAA11828 for obm-l-MTTP; Thu, 24 Apr 2003 16:06:41 -0300 Received: from ns3bind.localdomain ([200.230.34.5]) by sucuri.mat.puc-rio.br (8.9.3/8.9.3) with ESMTP id QAA11787 for ; Thu, 24 Apr 2003 16:06:28 -0300 Received: from servico2 ([200.230.34.227]) by ns3bind.localdomain (8.11.6/X.XX.X) with SMTP id h3OJ1kq07866 for ; Thu, 24 Apr 2003 16:01:46 -0300 Message-ID: <00af01c30a94$b6e98260$3300c57d@bovespa.com> From: "=?iso-8859-1?Q?Cl=E1udio_\=28Pr=E1tica\=29?=" To: References: <3E9EC1890000EB87@www.zipmail.com.br> <00a601c30a8c$758deac0$0301a8c0@usuario> Subject: Re: [obm-l] Re: [obm-l] euclidiana Date: Thu, 24 Apr 2003 16:07:10 -0300 MIME-Version: 1.0 Content-Type: text/plain; charset="iso-8859-1" Content-Transfer-Encoding: 8bit X-Priority: 3 X-MSMail-Priority: Normal X-Mailer: Microsoft Outlook Express 5.50.4920.2300 X-MimeOLE: Produced By Microsoft MimeOLE V5.50.4920.2300 Sender: owner-obm-l@sucuri.mat.puc-rio.br Precedence: bulk Reply-To: obm-l@mat.puc-rio.br Oi, Mário: O que parece estar faltando é a definição formal de "Sistema de Unidades de Medidas" e de "sistema". Além disso, quais são as formas válidas de verificação? Em matemática, é uma demonstração formal usando lógica. Em física e outras ciências naturais, é um experimento e/ou observação. Isso foi questão de uma prova de matemática ou física? Um abraço, Claudio. ----- Original Message ----- From: "Mário Pereira" To: Sent: Thursday, April 24, 2003 3:08 PM Subject: Re: [obm-l] Re: [obm-l] euclidiana > A pergunta da prova foi essa. > > > Mário > > ----- Original Message ----- > From: > To: > Sent: Thursday, April 24, 2003 1:02 PM > Subject: [obm-l] Re: [obm-l] euclidiana > > > > > > Pode traduzir? > > -- Mensagem original -- > > > > > > > >Como podemos verificar que um sistema de unidades de medidas é, de fato, > > >um sistema? > > > > > >Obrigado, > > > > > >Mário. > > > > > ========================================================================= Instruções para entrar na lista, sair da lista e usar a lista em http://www.mat.puc-rio.br/~nicolau/olimp/obm-l.html ========================================================================= From owner-obm-l@sucuri.mat.puc-rio.br Thu Apr 24 16:16:13 2003 Return-Path: Received: (from majordom@localhost) by sucuri.mat.puc-rio.br (8.9.3/8.9.3) id QAA12275 for obm-l-MTTP; Thu, 24 Apr 2003 16:14:37 -0300 Received: from puma.unisys.com.br (ns2.unisys.com.br [200.220.64.7]) by sucuri.mat.puc-rio.br (8.9.3/8.9.3) with ESMTP id QAA12256 for ; Thu, 24 Apr 2003 16:14:25 -0300 Received: from n8x4f9 (riohiper01p99.uninet.com.br [200.220.2.99]) by puma.unisys.com.br (8.12.9/8.12.3) with SMTP id h3OJDlEj027184 for ; Thu, 24 Apr 2003 16:13:53 -0300 (EST) X-Spam-Filter: check_local@puma.unisys.com.br by digitalanswers.org Message-ID: <006001c30a96$2dd34ae0$6302dcc8@n8x4f9> From: "Jose Francisco Guimaraes Costa" To: "obm-l" Subject: [obm-l] =?iso-8859-1?Q?Teorema_de_Erd=F6s-Suranyi?= Date: Thu, 24 Apr 2003 16:14:37 -0300 MIME-Version: 1.0 Content-Type: multipart/alternative; boundary="----=_NextPart_000_0056_01C30A7C.9AE30EA0" X-Priority: 3 X-MSMail-Priority: Normal X-Mailer: Microsoft Outlook Express 5.00.2615.200 X-MimeOLE: Produced By Microsoft MimeOLE V5.00.2615.200 Sender: owner-obm-l@sucuri.mat.puc-rio.br Precedence: bulk Reply-To: obm-l@mat.puc-rio.br This is a multi-part message in MIME format. ------=_NextPart_000_0056_01C30A7C.9AE30EA0 Content-Type: text/plain; charset="iso-8859-1" Content-Transfer-Encoding: quoted-printable Temo estar falando tremenda bobagem, mas l=E1 vai ... Se K=3D=B11=B2 =B12=B2 =B13=B2 ... =B1i=B2 ... =B1m=B2, n=E3o existe a = possiblidade de um dos termos i Como eu posso provar o Teorema de Erd=F6s-Suranyi: "Todo inteiro = positivo > K poder ser escrito na forma K=3D=B11=B2 =B12=B2 =B13=B2 ... =B1m=B2 = para uma escolha > conveniente dos sinais + e -, e de m." (Am=E9m!=A1) Na verdade, todo inteiro positivo diferente de 2 e os sinais dos = quadrados utilizados sao alternados (+ - + - ....) 1 =3D 1 3 =3D 4 - 1 4 =3D 4 5 =3D 9 - 4 6 =3D 9 - 4 + 1 7 =3D 16 - 9 [...] Claudio. ------=_NextPart_000_0056_01C30A7C.9AE30EA0 Content-Type: text/html; charset="iso-8859-1" Content-Transfer-Encoding: quoted-printable
Temo estar falando tremenda bobagem, = mas l=E1 vai=20 ...
 
Se K=3D=B11=B2 =B12=B2 =B13=B2 ... = =B1i=B2 ... =B1m=B2, n=E3o existe a=20 possiblidade de um dos termos i<m n=E3o existir, como = em 5=3D9-4, onde=20 1=B2 n=E3o existe.
 
JF
 
PS: Pela primeira vez o site da Wolfram (http://mathworld.wolfram.com) = n=E3o me deu=20 uma resposta. Tentei encontrar esse Teorema de Erd=F6s-Suranyi l=E1 e = obtive "no=20 matches found" por conta do "Suranyi". Ele seria tamb=E9m conhecido por = um outro=20 nome?
 
----- Original Message -----=20
From: Claudio Buffara
Sent: Saturday, January 02, 1904 1:28 PM
Subject: Re: [obm-l] O Retorno, parte -[e^(Pi*i)]. Os = matem=E1ticos=20 nunca morrem.

 
on 17.04.03 00:37, Igor Correia Oliveira at basketboy_igor@bol.com.br=20 wrote:

> Como eu posso provar o Teorema de Erd=F6s-Suranyi: = "Todo=20 inteiro positivo
> K poder ser escrito na forma K=3D=B11=B2 = =B12=B2 =B13=B2 ... =B1m=B2 para=20 uma escolha
> conveniente dos sinais + e -, e de m." = (Am=E9m!=A1)

Na=20 verdade, todo inteiro positivo diferente de 2 e os sinais dos=20 quadrados
utilizados sao alternados (+ - + - ....)
 
1 =3D 1

3 =3D 4 - 1
4 =3D 4

5 =3D 9 - 4
6 =3D 9 = - 4 + 1
7 =3D 16=20 - 9
[...]
 
Claudio.
------=_NextPart_000_0056_01C30A7C.9AE30EA0-- ========================================================================= Instruções para entrar na lista, sair da lista e usar a lista em http://www.mat.puc-rio.br/~nicolau/olimp/obm-l.html ========================================================================= From owner-obm-l@sucuri.mat.puc-rio.br Thu Apr 24 17:18:48 2003 Return-Path: Received: (from majordom@localhost) by sucuri.mat.puc-rio.br (8.9.3/8.9.3) id RAA15908 for obm-l-MTTP; Thu, 24 Apr 2003 17:16:13 -0300 Received: from hotmail.com (bay2-f170.bay2.hotmail.com [65.54.247.170]) by sucuri.mat.puc-rio.br (8.9.3/8.9.3) with ESMTP id RAA15900 for ; Thu, 24 Apr 2003 17:16:08 -0300 Received: from mail pickup service by hotmail.com with Microsoft SMTPSVC; Thu, 24 Apr 2003 13:15:36 -0700 Received: from 200.18.136.130 by by2fd.bay2.hotmail.msn.com with HTTP; Thu, 24 Apr 2003 20:15:36 GMT X-Originating-IP: [200.18.136.130] X-Originating-Email: [rep_tcheka@hotmail.com] From: "Republica Tcheka" To: obm-l@mat.puc-rio.br Subject: [obm-l] UFMG 2002 Date: Thu, 24 Apr 2003 17:15:36 -0300 Mime-Version: 1.0 Content-Type: text/plain; charset=iso-8859-1; format=flowed Message-ID: X-OriginalArrivalTime: 24 Apr 2003 20:15:36.0234 (UTC) FILETIME=[443B18A0:01C30A9E] Sender: owner-obm-l@sucuri.mat.puc-rio.br Precedence: bulk Reply-To: obm-l@mat.puc-rio.br estou com uma duvida d qual a resposta d uma questao aberta d matematica: uma escola tem 10 professores q lecionam pela manha, 8 pela tarde e 5 pela noite.d qnts forma podem ser formadas comissoes d 4 professores d forma q tenha pelo menos 1 prof. q lecione pela manha e pelo menos 1 q lecione pela tarde. OBS:essa e a letra C da questao. Obrigado _________________________________________________________________ MSN Messenger: converse com os seus amigos online. http://messenger.msn.com.br ========================================================================= Instruções para entrar na lista, sair da lista e usar a lista em http://www.mat.puc-rio.br/~nicolau/olimp/obm-l.html ========================================================================= From owner-obm-l@sucuri.mat.puc-rio.br Thu Apr 24 17:27:53 2003 Return-Path: Received: (from majordom@localhost) by sucuri.mat.puc-rio.br (8.9.3/8.9.3) id RAA16224 for obm-l-MTTP; Thu, 24 Apr 2003 17:26:26 -0300 Received: from paiol.terra.com.br (paiol.terra.com.br [200.176.3.18]) by sucuri.mat.puc-rio.br (8.9.3/8.9.3) with ESMTP id RAA16220 for ; Thu, 24 Apr 2003 17:26:22 -0300 Received: from itaim.terra.com.br (itaim.terra.com.br [200.176.3.76]) by paiol.terra.com.br (Postfix) with ESMTP id 1BE0188DC6 for ; Thu, 24 Apr 2003 17:25:52 -0300 (BRT) Received: from usuario (200-203-035-087.paemt7005.dsl.brasiltelecom.net.br [200.203.35.87]) (authenticated user marioappereira) by itaim.terra.com.br (Postfix) with ESMTP id CB2232E00C6 for ; Thu, 24 Apr 2003 17:25:51 -0300 (BRT) Message-ID: <00ac01c30a9f$bba7a5b0$0301a8c0@usuario> From: =?iso-8859-1?Q?M=E1rio_Pereira?= To: References: <3E9EC1890000EB87@www.zipmail.com.br> <00a601c30a8c$758deac0$0301a8c0@usuario> <00af01c30a94$b6e98260$3300c57d@bovespa.com> Subject: Re: [obm-l] Re: [obm-l] euclidiana Date: Thu, 24 Apr 2003 17:26:05 -0300 MIME-Version: 1.0 Content-Type: text/plain; charset="iso-8859-1" Content-Transfer-Encoding: 8bit X-Priority: 3 X-MSMail-Priority: Normal X-Mailer: Microsoft Outlook Express 6.00.2600.0000 X-MimeOLE: Produced By Microsoft MimeOLE V6.00.2600.0000 Sender: owner-obm-l@sucuri.mat.puc-rio.br Precedence: bulk Reply-To: obm-l@mat.puc-rio.br Matemática. Geometria Euclidiana. Mário. ----- Original Message ----- From: "Cláudio (Prática)" To: Sent: Thursday, April 24, 2003 4:07 PM Subject: Re: [obm-l] Re: [obm-l] euclidiana > Oi, Mário: > > O que parece estar faltando é a definição formal de "Sistema de Unidades de > Medidas" e de "sistema". Além disso, quais são as formas válidas de > verificação? Em matemática, é uma demonstração formal usando lógica. Em > física e outras ciências naturais, é um experimento e/ou observação. > > Isso foi questão de uma prova de matemática ou física? > > Um abraço, > Claudio. > > ----- Original Message ----- > From: "Mário Pereira" > To: > Sent: Thursday, April 24, 2003 3:08 PM > Subject: Re: [obm-l] Re: [obm-l] euclidiana > > > > A pergunta da prova foi essa. > > > > > > Mário > > > > ----- Original Message ----- > > From: > > To: > > Sent: Thursday, April 24, 2003 1:02 PM > > Subject: [obm-l] Re: [obm-l] euclidiana > > > > > > > > > > Pode traduzir? > > > -- Mensagem original -- > > > > > > > > > > >Como podemos verificar que um sistema de unidades de medidas é, de > fato, > > > >um sistema? > > > > > > > >Obrigado, > > > > > > > >Mário. > > > > > > > > > ========================================================================= > Instruções para entrar na lista, sair da lista e usar a lista em > http://www.mat.puc-rio.br/~nicolau/olimp/obm-l.html > ========================================================================= > > ========================================================================= Instruções para entrar na lista, sair da lista e usar a lista em http://www.mat.puc-rio.br/~nicolau/olimp/obm-l.html ========================================================================= From owner-obm-l@sucuri.mat.puc-rio.br Thu Apr 24 17:29:32 2003 Return-Path: Received: (from majordom@localhost) by sucuri.mat.puc-rio.br (8.9.3/8.9.3) id RAA16311 for obm-l-MTTP; Thu, 24 Apr 2003 17:28:05 -0300 Received: from ns3bind.localdomain ([200.230.34.5]) by sucuri.mat.puc-rio.br (8.9.3/8.9.3) with ESMTP id RAA16305 for ; Thu, 24 Apr 2003 17:28:01 -0300 Received: from servico2 ([200.230.34.227]) by ns3bind.localdomain (8.11.6/X.XX.X) with SMTP id h3OKNHx16834 for ; Thu, 24 Apr 2003 17:23:17 -0300 Message-ID: <01d501c30aa0$1a579b60$3300c57d@bovespa.com> From: "=?iso-8859-1?Q?Cl=E1udio_\=28Pr=E1tica\=29?=" To: References: <006001c30a96$2dd34ae0$6302dcc8@n8x4f9> Subject: [obm-l] =?iso-8859-1?Q?Re:_=5Bobm-l=5D_Teorema_de_Erd=F6s-Suranyi?= Date: Thu, 24 Apr 2003 17:28:43 -0300 MIME-Version: 1.0 Content-Type: multipart/alternative; boundary="----=_NextPart_000_01D2_01C30A86.F4303000" X-Priority: 3 X-MSMail-Priority: Normal X-Mailer: Microsoft Outlook Express 5.50.4920.2300 X-MimeOLE: Produced By Microsoft MimeOLE V5.50.4920.2300 Sender: owner-obm-l@sucuri.mat.puc-rio.br Precedence: bulk Reply-To: obm-l@mat.puc-rio.br This is a multi-part message in MIME format. ------=_NextPart_000_01D2_01C30A86.F4303000 Content-Type: text/plain; charset="iso-8859-1" Content-Transfer-Encoding: quoted-printable Oi, JF: Bobagem nenhuma. O enunciado correto deveria ser: Prove que todo inteiro positivo diferente de 2 pode ser expresso como = uma soma alternada de quadrados perfeitos dispostos em ordem = estritamente crescente (ou estritamente decrescente - d=E1 no mesmo).=20 N=E3o =E9 necess=E1rio que todos os quadrados estejam presentes. De = fato, como voc=EA mesmo notou, com esta condi=E7=E3o o resultado n=E3o = =E9 mais v=E1lido. Ali=E1s, acho que =E9 um resultado meio mixuruca pra estar associado ao = nome de algu=E9m, especialmente do Erdos. Um abra=E7o, Claudio. ----- Original Message -----=20 From: Jose Francisco Guimaraes Costa=20 To: obm-l=20 Sent: Thursday, April 24, 2003 4:14 PM Subject: [obm-l] Teorema de Erd=F6s-Suranyi Temo estar falando tremenda bobagem, mas l=E1 vai ... Se K=3D=B11=B2 =B12=B2 =B13=B2 ... =B1i=B2 ... =B1m=B2, n=E3o existe a = possiblidade de um dos termos i Como eu posso provar o Teorema de Erd=F6s-Suranyi: "Todo inteiro = positivo > K poder ser escrito na forma K=3D=B11=B2 =B12=B2 =B13=B2 ... =B1m=B2 = para uma escolha > conveniente dos sinais + e -, e de m." (Am=E9m!=A1) Na verdade, todo inteiro positivo diferente de 2 e os sinais dos = quadrados utilizados sao alternados (+ - + - ....) 1 =3D 1 3 =3D 4 - 1 4 =3D 4 5 =3D 9 - 4 6 =3D 9 - 4 + 1 7 =3D 16 - 9 [...] Claudio. ------=_NextPart_000_01D2_01C30A86.F4303000 Content-Type: text/html; charset="iso-8859-1" Content-Transfer-Encoding: quoted-printable
Oi, JF:
 
Bobagem nenhuma. O enunciado correto = deveria=20 ser:
 
Prove que todo inteiro positivo = diferente de 2 pode=20 ser expresso como uma soma alternada de quadrados perfeitos dispostos em = ordem=20 estritamente crescente (ou estritamente decrescente - d=E1 no mesmo).=20
 
N=E3o =E9 necess=E1rio que todos os = quadrados estejam=20 presentes. De fato, como voc=EA mesmo notou, com esta condi=E7=E3o o = resultado n=E3o =E9=20 mais v=E1lido.
 
Ali=E1s, acho que =E9 um resultado meio = mixuruca pra=20 estar associado ao nome de algu=E9m, especialmente do = Erdos.
 
Um abra=E7o,
Claudio.
----- Original Message -----
From:=20 Jose=20 Francisco Guimaraes Costa
To: obm-l
Sent: Thursday, April 24, 2003 = 4:14=20 PM
Subject: [obm-l] Teorema de=20 Erd=F6s-Suranyi

Temo estar falando tremenda bobagem, = mas l=E1 vai=20 ...
 
Se K=3D=B11=B2 =B12=B2 =B13=B2 ... = =B1i=B2 ... =B1m=B2, n=E3o existe a=20 possiblidade de um dos termos i<m n=E3o existir, como = em 5=3D9-4,=20 onde 1=B2 n=E3o existe.
 
JF
 
PS: Pela primeira vez o site da Wolfram (http://mathworld.wolfram.com) = n=E3o me=20 deu uma resposta. Tentei encontrar esse Teorema de Erd=F6s-Suranyi = l=E1 e obtive=20 "no matches found" por conta do "Suranyi". Ele seria tamb=E9m = conhecido por um=20 outro nome?
 
----- Original Message -----=20
From: Claudio Buffara =
Sent: Saturday, January 02, 1904 1:28 PM
Subject: Re: [obm-l] O Retorno, parte -[e^(Pi*i)]. Os = matem=E1ticos=20 nunca morrem.

 
on 17.04.03 00:37, Igor Correia Oliveira at basketboy_igor@bol.com.br=20 wrote:

> Como eu posso provar o Teorema de Erd=F6s-Suranyi: = "Todo=20 inteiro positivo
> K poder ser escrito na forma K=3D=B11=B2 = =B12=B2 =B13=B2 ... =B1m=B2=20 para uma escolha
> conveniente dos sinais + e -, e de m."=20 (Am=E9m!=A1)

Na verdade, todo inteiro positivo diferente de 2 e = os sinais=20 dos quadrados
utilizados sao alternados (+ - + - ....)
 
1 =3D 1

3 =3D 4 - 1
4 =3D 4

5 =3D 9 - 4
6 =3D = 9 - 4 + 1
7 =3D=20 16 - 9
[...]
 
Claudio.
------=_NextPart_000_01D2_01C30A86.F4303000-- ========================================================================= Instruções para entrar na lista, sair da lista e usar a lista em http://www.mat.puc-rio.br/~nicolau/olimp/obm-l.html ========================================================================= From owner-obm-l@sucuri.mat.puc-rio.br Thu Apr 24 17:31:20 2003 Return-Path: Received: (from majordom@localhost) by sucuri.mat.puc-rio.br (8.9.3/8.9.3) id RAA16406 for obm-l-MTTP; Thu, 24 Apr 2003 17:29:08 -0300 Received: from ivoti.terra.com.br (ivoti.terra.com.br [200.176.3.20]) by sucuri.mat.puc-rio.br (8.9.3/8.9.3) with ESMTP id RAA16401 for ; Thu, 24 Apr 2003 17:29:00 -0300 Received: from marova.terra.com.br (marova.terra.com.br [200.176.3.39]) by ivoti.terra.com.br (Postfix) with ESMTP id 5FD344093C9 for ; Thu, 24 Apr 2003 17:28:29 -0300 (BRT) Received: from usuario (200-203-035-087.paemt7005.dsl.brasiltelecom.net.br [200.203.35.87]) (authenticated user marioappereira) by marova.terra.com.br (Postfix) with ESMTP id 011CE3DC134 for ; Thu, 24 Apr 2003 17:28:29 -0300 (BRT) Message-ID: <00bb01c30aa0$195630f0$0301a8c0@usuario> From: =?iso-8859-1?Q?M=E1rio_Pereira?= To: Subject: [obm-l] Date: Thu, 24 Apr 2003 17:28:42 -0300 MIME-Version: 1.0 Content-Type: multipart/alternative; boundary="----=_NextPart_000_00B8_01C30A86.F3DBA760" X-Priority: 3 X-MSMail-Priority: Normal X-Mailer: Microsoft Outlook Express 6.00.2600.0000 X-MimeOLE: Produced By Microsoft MimeOLE V6.00.2600.0000 Sender: owner-obm-l@sucuri.mat.puc-rio.br Precedence: bulk Reply-To: obm-l@mat.puc-rio.br This is a multi-part message in MIME format. ------=_NextPart_000_00B8_01C30A86.F3DBA760 Content-Type: text/plain; charset="iso-8859-1" Content-Transfer-Encoding: quoted-printable como calcular: (log x)^2 (logaritmo neperiano de x, tudo elevado ao quadrado) A resposta =E9 2logx/x (parece que o desenvolvimento =E9 para ser feito pela defini=E7=E3o de = limite). Obrigado.=20 M=E1rio.=20 ------=_NextPart_000_00B8_01C30A86.F3DBA760 Content-Type: text/html; charset="iso-8859-1" Content-Transfer-Encoding: quoted-printable
como calcular:
 
(log x)^2
 
(logaritmo neperiano de x, tudo elevado = ao=20 quadrado)
 
A resposta =E9 2logx/x
 
(parece que o desenvolvimento =E9 para = ser feito pela=20 defini=E7=E3o de limite).
 
Obrigado.
 
M=E1rio.
------=_NextPart_000_00B8_01C30A86.F3DBA760-- ========================================================================= Instruções para entrar na lista, sair da lista e usar a lista em http://www.mat.puc-rio.br/~nicolau/olimp/obm-l.html ========================================================================= From owner-obm-l@sucuri.mat.puc-rio.br Thu Apr 24 17:38:03 2003 Return-Path: Received: (from majordom@localhost) by sucuri.mat.puc-rio.br (8.9.3/8.9.3) id RAA16870 for obm-l-MTTP; Thu, 24 Apr 2003 17:36:24 -0300 Received: from web40508.mail.yahoo.com (web40508.mail.yahoo.com [66.218.78.125]) by sucuri.mat.puc-rio.br (8.9.3/8.9.3) with SMTP id RAA16847 for ; Thu, 24 Apr 2003 17:36:16 -0300 Message-ID: <20030424203543.53705.qmail@web40508.mail.yahoo.com> Received: from [143.54.41.17] by web40508.mail.yahoo.com via HTTP; Thu, 24 Apr 2003 17:35:43 ART Date: Thu, 24 Apr 2003 17:35:43 -0300 (ART) From: "=?iso-8859-1?q?J.C.=20PAREDE?=" Subject: Re: [obm-l] To: obm-l@mat.puc-rio.br In-Reply-To: <27BD56F8640DD711A4320006295078E50500B7@pssnx1.brahma> MIME-Version: 1.0 Content-Type: multipart/alternative; boundary="0-1551112636-1051216543=:53118" Content-Transfer-Encoding: 8bit Sender: owner-obm-l@sucuri.mat.puc-rio.br Precedence: bulk Reply-To: obm-l@mat.puc-rio.br --0-1551112636-1051216543=:53118 Content-Type: text/plain; charset=iso-8859-1 Content-Transfer-Encoding: 8bit Boa noite,Estou com dificuldades em resolver alguns problemas, até que resolvi alguns deles mas fiquei com duvidas quanto às respostas.Se alguém puder me ajudar... 2. Um campeonato é disputado por 12 clubes em rodadas de 6 jogos cada. De quantos modos é possível selecionar os jogos da primeira rodada. 12!/2 ? 8. O código Morse usa "palavras" contendo de 1 a 4 "letras". As "letras" são ponto e traço. Quantas "palavras" existe no código Morse? 1 letra = 22 letras = 43 letras = 84 letras = 32TOTAL = 46 JOÃO CARLOS PAREDE --------------------------------- Yahoo! Mail O melhor e-mail gratuito da internet: 6MB de espaço, antivírus, acesso POP3, filtro contra spam. --0-1551112636-1051216543=:53118 Content-Type: text/html; charset=iso-8859-1 Content-Transfer-Encoding: 8bit
Boa noite,
Estou com dificuldades em resolver alguns problemas, até que resolvi alguns deles mas fiquei com duvidas quanto às respostas.
Se alguém puder me ajudar...
 
 
2. Um campeonato é disputado por 12 clubes em rodadas de 6 jogos cada. De quantos modos é possível selecionar os jogos da primeira rodada.
 
12!/2 ?
 
 
8. O código Morse usa "palavras" contendo de 1 a 4 "letras". As "letras" são ponto e traço. Quantas "palavras" existe no código Morse?
 
1 letra = 2
2 letras = 4
3 letras = 8
4 letras = 32
TOTAL = 46


 JOÃO CARLOS PAREDE



Yahoo! Mail
O melhor e-mail gratuito da internet: 6MB de espaço, antivírus, acesso POP3, filtro contra spam. --0-1551112636-1051216543=:53118-- ========================================================================= Instruções para entrar na lista, sair da lista e usar a lista em http://www.mat.puc-rio.br/~nicolau/olimp/obm-l.html ========================================================================= From owner-obm-l@sucuri.mat.puc-rio.br Thu Apr 24 17:42:26 2003 Return-Path: Received: (from majordom@localhost) by sucuri.mat.puc-rio.br (8.9.3/8.9.3) id RAA17264 for obm-l-MTTP; Thu, 24 Apr 2003 17:41:01 -0300 Received: from aacpdlotus.net.ms.gov.br (ns1.ms.gov.br [200.181.116.3]) by sucuri.mat.puc-rio.br (8.9.3/8.9.3) with ESMTP id RAA17255 for ; Thu, 24 Apr 2003 17:40:55 -0300 From: JoaoCarlos_Junior@net.ms.gov.br Importance: High X-Priority: 1 (High) Subject: Re: [obm-l] Re: [obm-l] TEOIREMA DE CRISTEA::Geometria,pontos estranhos e problemas legais!!!! To: obm-l@mat.puc-rio.br X-Mailer: Lotus Notes Release 5.0.9a January 7, 2002 Message-ID: Date: Thu, 24 Apr 2003 16:44:58 -0400 X-MIMETrack: Serialize by Router on aacpdlotus/NETMS(Release 5.0.9a |January 7, 2002) at 04/24/2003 04:45:31 PM MIME-Version: 1.0 Content-type: text/plain; charset=iso-8859-1 Content-Transfer-Encoding: 8bit X-MIME-Autoconverted: from quoted-printable to 8bit by sucuri.mat.puc-rio.br id RAA17257 Sender: owner-obm-l@sucuri.mat.puc-rio.br Precedence: bulk Reply-To: obm-l@mat.puc-rio.br Querido Cláudio, Gostaria de dar uma olhada na resolução do Dirichlet para Fermat com n=4, se você assim a elogia. Desta forma, pergunto-te como faço para olhá-la, em que site? Um forte abraço, João Carlos. "Cláudio \(Prática\)" ora.com.br> cc: Enviado Por: Assunto: Re: [obm-l] Re: [obm-l] TEOIREMA DE owner-obm-l@sucuri.mat CRISTEA::Geometria,pontos estranhos e problemas .puc-rio.br legais!!!! 24/04/2003 15:27 Favor responder a obm-l Caro Dirichlet: Consegui demonstrar este resultado. Estou disposto a mandar minha solução pra lista sob uma condição: que, de hoje em diante, você só mande mensagens relevantes, com nexo, e caso tenha comentários pra fazer, que eles sejam polidos, pertinentes, e que ajudem as pessoas a entender melhor um problema ou conceito. Eu sei que você é capaz, afinal a demonstração do Último Teorema de Fermat para n = 4 que você mandou pra lista foi uma das melhores que eu já vi. Se você concordar, eu mando a solução imediatamente e "free of charge". Topa? Um abraço, Claudio. ----- Original Message ----- From: To: Sent: Thursday, April 24, 2003 12:59 PM Subject: [obm-l] Re: [obm-l] TEOIREMA DE CRISTEA::Geometria,pontos estranhos e problemas legais!!!! > MAS COMO???!?!??!!:???!Ja estou ha semanas tentando sem sucesso ou expectativa.Nenhum > triangulo da muitas esperanças... > > > -- Mensagem original -- > > >on 22.04.03 21:25, Johann Peter Gustav Lejeune Dirichlet at > >peterdirichlet2002@yahoo.com.br wrote: > > > >Ola gente!!!!Estou tentando resolver problemas de Geometria do Mathematical > >excalibur mas preciso de um teorema.Quem pode demonstrar pra mim: > > > >Considere o triangulo ABC e um ponto T.Duas cevianas CT_c e BT_b se cortam > >erm T e os pontos R_c,R_b e T sao alinhados,com R_c em AB e R_b em AC.Mostre > >que > > > >AT_c*BR_c/T_cB/R_cA + ATb*CR_b//T_bC/R_bA = 1 > > > >Nao sei se e isso mesmo mas vale o risco... > > > > > >Oi, JP: > > > >Supondo que a expressao acima seja: > > > >(AT_c * BR_c) / (BT_c * AR_c) + ( AT_b * CR_b ) / (CT_b * AR_b ) = 1 > > > >eu diria que uma boa aposta seria o teorema de Menelau (aplicado mais de > >uma > >vez). > > > > > >Um abraco, > >Claudio. > > > > > > > > TEA WITH ME THAT I BOOK YOUR FACE > > > ------------------------------------------ > Use o melhor sistema de busca da Internet > Radar UOL - http://www.radaruol.com.br > > > > ========================================================================= > Instruções para entrar na lista, sair da lista e usar a lista em > http://www.mat.puc-rio.br/~nicolau/olimp/obm-l.html > ========================================================================= ========================================================================= Instruções para entrar na lista, sair da lista e usar a lista em http://www.mat.puc-rio.br/~nicolau/olimp/obm-l.html ========================================================================= ========================================================================= Instruções para entrar na lista, sair da lista e usar a lista em http://www.mat.puc-rio.br/~nicolau/olimp/obm-l.html ========================================================================= From owner-obm-l@sucuri.mat.puc-rio.br Thu Apr 24 17:50:49 2003 Return-Path: Received: (from majordom@localhost) by sucuri.mat.puc-rio.br (8.9.3/8.9.3) id RAA17935 for obm-l-MTTP; Thu, 24 Apr 2003 17:49:27 -0300 Received: from cmsrelay01.mx.net (cmsrelay01.mx.net [165.212.11.110]) by sucuri.mat.puc-rio.br (8.9.3/8.9.3) with SMTP id RAA17921 for ; Thu, 24 Apr 2003 17:49:20 -0300 Received: from uadvg137.cms.usa.net (HELO localhost) (165.212.11.137) by cmsoutbound.mx.net with SMTP; 24 Apr 2003 20:48:45 -0000 Received: from smtp.postoffice.net [165.212.8.17] by uadvg137.cms.usa.net (ASMTP/) via mtad (C8.MAIN.2.05) with ESMTP id 689HDXuWr0031M37; Thu, 24 Apr 2003 20:48:43 GMT Received: from 200.181.4.100 [200.181.4.100] by uwdvg017.cms.usa.net (USANET web-mailer CM.0402.5.2B); Thu, 24 Apr 2003 20:48:42 -0000 Date: Thu, 24 Apr 2003 17:48:42 -0300 From: Artur Costa Steiner To: Subject: Re: [[obm-l] e mais probabilidades!] X-Mailer: USANET web-mailer (CM.0402.5.2B) Mime-Version: 1.0 Message-ID: <812HDXuWQ8672S17.1051217322@uwdvg017.cms.usa.net> Content-Type: text/plain; charset=ISO-8859-1 Content-Transfer-Encoding: 8bit X-MIME-Autoconverted: from quoted-printable to 8bit by sucuri.mat.puc-rio.br id RAA17931 Sender: owner-obm-l@sucuri.mat.puc-rio.br Precedence: bulk Reply-To: obm-l@mat.puc-rio.br niski wrote: > Ola pessoal...segue outro problema que nao sei onde estou errando, > gostaria que me ajudassem. Muito obrigado! > > As cinco primeiras repeticoes de um experimento custam $10,00 cada. > Todas as repeticoes subsequentes custam $5,00 cada. Suponha que o > experimento seja repetido até que o primeiro sucesso ocorra. Se a > probabilidade de sucesso de uma repeticao é igual a 0,9, e se as > repeticoes sao independentes, qual é o custo esperado da operação? > > Minha resolucao : > Trate-se de calcular a esperança matematica. > Dividirei em dois casos > i) Esperanca para até 5 repeticoes > ii) Esperanca para 6 ou mais repeticoes Nao eh bem isso A esperanca eh uma so, composta por duas somas, uma finita de 1 ateh 5, a outra infinita (o limite de uma serie) de 6 a infinito. > > Somarei i com ii e esta deverá ser a resposta do problema. > > Seja X a variavel aleatoria que representa o custo por operacao. > i) > > Temos a seguinte distribuição: > > X = 10 -> p(X=10) = 0,9 > X = 20 -> p(X=20) = (0,9).(0,1) > .. > X = 50 -> p(X=50) = (0,9).(0,1)^4 nao, nao...De modo geral, P(x=n) = 0,9. (0,1)^(n-1) - uma distrib. geometrica > > E'(X) , para até 5 eventos = 11,1105 nao existe "esperanca ateh 5 eventos", vc tem eh a esperanca de toda a distribuicao. Mas no caso interessa a Esp. de Custo , nao a de X. Para calcular esperanca de C, vc tem que considerar a composicao de das funcoes, nao eh possivle trabalhar isoladamente em X. > > ii) > > Cabe notar que a função que dá o custo pelo numero n de repeticoes (n > > 5) é dada por > > C(n) = 50 + (((n-1)-5)+1).5 ou > C(n) = 50 + (n-5).5 isto para n>=6. Para n<=5, C(n) = 10n > > Então a esperança matematica E''(x) é dada pela somatoria > Sum[n=6 até infinito] (50 + (n-5)*5).(0,9).(0,1)^(n-1) , nao, nao. A esperanca de C, E(c) eh dada por E(c) = Soma (n=1, 5) 9n. (0,1)^(n-1) + Soma (k=6, inf) [50 + 5(n-5)]. 9. (0,1)^(n-1) O segundo somatorio eh o limite de uma serie infinita de potências que converge. Podemos calcular seu limite analiticamente, mas exige um pouco de trabalho. Um abraco Artur ========================================================================= Instruções para entrar na lista, sair da lista e usar a lista em http://www.mat.puc-rio.br/~nicolau/olimp/obm-l.html ========================================================================= From owner-obm-l@sucuri.mat.puc-rio.br Thu Apr 24 18:02:37 2003 Return-Path: Received: (from majordom@localhost) by sucuri.mat.puc-rio.br (8.9.3/8.9.3) id SAA18762 for obm-l-MTTP; Thu, 24 Apr 2003 18:01:13 -0300 Received: from ivoti.terra.com.br (ivoti.terra.com.br [200.176.3.20]) by sucuri.mat.puc-rio.br (8.9.3/8.9.3) with ESMTP id SAA18758 for ; Thu, 24 Apr 2003 18:01:08 -0300 Received: from itaim.terra.com.br (itaim.terra.com.br [200.176.3.76]) by ivoti.terra.com.br (Postfix) with ESMTP id 7F2BB408639 for ; Thu, 24 Apr 2003 18:00:36 -0300 (BRT) Received: from niski.com (unknown [200.148.202.95]) (authenticated user fniski) by itaim.terra.com.br (Postfix) with ESMTP id 3C1542E007A for ; Thu, 24 Apr 2003 18:00:35 -0300 (BRT) Message-ID: <3EA84268.6020305@niski.com> Date: Thu, 24 Apr 2003 13:00:40 -0700 From: niski User-Agent: Mozilla/5.0 (Windows; U; Windows NT 5.1; en-US; rv:1.0.2) Gecko/20030208 Netscape/7.02 X-Accept-Language: en-us, en MIME-Version: 1.0 To: obm-l@mat.puc-rio.br Subject: Re: [obm-l] UFMG 2002 References: Content-Type: text/plain; charset=ISO-8859-1; format=flowed Content-Transfer-Encoding: 8bit Sender: owner-obm-l@sucuri.mat.puc-rio.br Precedence: bulk Reply-To: obm-l@mat.puc-rio.br Devemos então escolher inicialmente 1 professor que lecione pela manha C(10,1) e 1 que lecione de tarde C(8,1). Sobraram 2 dos 21 professores p/ serem selecionados, e fazemos isso com C(21,2). Então o total de maneiras possiveis é C(10,1).C(8,1).C(21,2) = 16800. Republica Tcheka wrote: > estou com uma duvida d qual a resposta d uma questao aberta d matematica: > > uma escola tem 10 professores q lecionam pela manha, 8 pela tarde e 5 > pela noite.d qnts forma podem ser formadas comissoes d 4 professores d > forma q tenha pelo menos 1 prof. q lecione pela manha e pelo menos 1 q > lecione pela tarde. > > OBS:essa e a letra C da questao. > > > Obrigado > > > > > _________________________________________________________________ > MSN Messenger: converse com os seus amigos online. > http://messenger.msn.com.br > > ========================================================================= > Instruções para entrar na lista, sair da lista e usar a lista em > http://www.mat.puc-rio.br/~nicolau/olimp/obm-l.html > ========================================================================= > > -- [about him:] It is rare to find learned men who are clean, do not stink and have a sense of humour. -Gottfried Whilhem Leibniz ========================================================================= Instruções para entrar na lista, sair da lista e usar a lista em http://www.mat.puc-rio.br/~nicolau/olimp/obm-l.html ========================================================================= From owner-obm-l@sucuri.mat.puc-rio.br Thu Apr 24 18:22:00 2003 Return-Path: Received: (from majordom@localhost) by sucuri.mat.puc-rio.br (8.9.3/8.9.3) id SAA20389 for obm-l-MTTP; Thu, 24 Apr 2003 18:20:16 -0300 Received: from cmsrelay04.mx.net (cmsrelay04.mx.net [165.212.11.113]) by sucuri.mat.puc-rio.br (8.9.3/8.9.3) with SMTP id SAA20376 for ; Thu, 24 Apr 2003 18:20:07 -0300 Received: from uadvg129.cms.usa.net (HELO localhost) (165.212.11.129) by cmsoutbound.mx.net with SMTP; 24 Apr 2003 21:19:32 -0000 Received: from smtp.postoffice.net [165.212.8.26] by uadvg129.cms.usa.net (ASMTP/) via mtad (C8.MAIN.2.05) with ESMTP id 167HDXVTE0222M29; Thu, 24 Apr 2003 21:19:30 GMT Received: from 200.181.4.100 [200.181.4.100] by cmsweb10.cms.usa.net (USANET web-mailer CM.0402.5.2B); Thu, 24 Apr 2003 21:19:29 -0000 Date: Thu, 24 Apr 2003 18:19:29 -0300 From: Artur Costa Steiner To: Subject: Re: [[obm-l] ] X-Mailer: USANET web-mailer (CM.0402.5.2B) Mime-Version: 1.0 Message-ID: <110HDXVTd6704S10.1051219169@cmsweb10.cms.usa.net> Content-Type: text/plain; charset=ISO-8859-1 Content-Transfer-Encoding: 8bit X-MIME-Autoconverted: from quoted-printable to 8bit by sucuri.mat.puc-rio.br id SAA20379 Sender: owner-obm-l@sucuri.mat.puc-rio.br Precedence: bulk Reply-To: obm-l@mat.puc-rio.br Mário Pereira wrote: > --------------------------------------------- > Attachment:  > MIME Type: multipart/alternative > --------------------------------------------- > como calcular: > > (log x)^2 > > (logaritmo neperiano de x, tudo elevado ao quadrado) > > A resposta é 2logx/x > > (parece que o desenvolvimento é para ser feito pela definição de limite). > > Obrigado. > > Mário. Nao me lembro de uma expressao interessante para (log x)^2, mas 2logx/x DECIDIDAMENTE nao eh. Basta observar que, quando x vai para infinito,log x)^2 tambem vai, ao passo que 2logx/x tende a zero. Plote as dua funcoes em um grafico e constate que uma, nem de longe, aproxima a outra. Um abraco Artur ========================================================================= Instruções para entrar na lista, sair da lista e usar a lista em http://www.mat.puc-rio.br/~nicolau/olimp/obm-l.html ========================================================================= From owner-obm-l@sucuri.mat.puc-rio.br Thu Apr 24 18:23:19 2003 Return-Path: Received: (from majordom@localhost) by sucuri.mat.puc-rio.br (8.9.3/8.9.3) id SAA20516 for obm-l-MTTP; Thu, 24 Apr 2003 18:22:00 -0300 Received: from trex.centroin.com.br (trex.centroin.com.br [200.225.63.134]) by sucuri.mat.puc-rio.br (8.9.3/8.9.3) with ESMTP id SAA20509 for ; Thu, 24 Apr 2003 18:21:56 -0300 Received: from trex.centroin.com.br (localhost [127.0.0.1]) by trex.centroin.com.br (8.12.9/8.12.9) with ESMTP id h3OLLPh5003271 for ; Thu, 24 Apr 2003 18:21:25 -0300 (EST) Received: by trex.centroin.com.br (8.12.9/8.12.5/Submit) id h3OLLP6S003267; Thu, 24 Apr 2003 18:21:25 -0300 (EST) Message-Id: <200304242121.h3OLLP6S003267@trex.centroin.com.br> Received: from 200.165.208.247 by trex.centroin.com.br (CIPWM versao 1.4C1) with HTTPS for ; Thu, 24 Apr 2003 18:21:25 -0300 (EST) Date: Thu, 24 Apr 2003 18:21:25 -0300 (EST) From: Augusto Cesar de Oliveira Morgado To: obm-l@mat.puc-rio.br Subject: Re: [obm-l] UFMG 2002 MIME-Version: 1.0 X-Mailer: CentroIn Internet Provider WebMail v. 1.4C1 (http://www.centroin.com.br/) Content-Type: text/plain; charset="iso-8859-1" Content-Transfer-Encoding: 8bit X-MIME-Autoconverted: from quoted-printable to 8bit by sucuri.mat.puc-rio.br id SAA20511 Sender: owner-obm-l@sucuri.mat.puc-rio.br Precedence: bulk Reply-To: obm-l@mat.puc-rio.br Infelizmente esta soluçao estah errada. Morgado Em Thu, 24 Apr 2003 13:00:40 -0700, niski disse: > Devemos então escolher inicialmente 1 professor que lecione pela manha > C(10,1) e 1 que lecione de tarde C(8,1). Sobraram 2 dos 21 professores > p/ serem selecionados, e fazemos isso com C(21,2). > Então o total de maneiras possiveis é > C(10,1).C(8,1).C(21,2) = 16800. > > > > Republica Tcheka wrote: > > estou com uma duvida d qual a resposta d uma questao aberta d matematica: > > > > uma escola tem 10 professores q lecionam pela manha, 8 pela tarde e 5 > > pela noite.d qnts forma podem ser formadas comissoes d 4 professores d > > forma q tenha pelo menos 1 prof. q lecione pela manha e pelo menos 1 q > > lecione pela tarde. > > > > OBS:essa e a letra C da questao. > > > > > > Obrigado > > > > > > > > > > _________________________________________________________________ > > MSN Messenger: converse com os seus amigos online. > > http://messenger.msn.com.br > > > > ========================================================================= > > Instruções para entrar na lista, sair da lista e usar a lista em > > http://www.mat.puc-rio.br/~nicolau/olimp/obm-l.html > > ========================================================================= > > > > > > > -- > [about him:] > It is rare to find learned men who are clean, do not stink and have a > sense of humour. > -Gottfried Whilhem Leibniz > > ========================================================================= > Instruções para entrar na lista, sair da lista e usar a lista em > http://www.mat.puc-rio.br/~nicolau/olimp/obm-l.html > ========================================================================= > > ========================================================================= Instruções para entrar na lista, sair da lista e usar a lista em http://www.mat.puc-rio.br/~nicolau/olimp/obm-l.html ========================================================================= From owner-obm-l@sucuri.mat.puc-rio.br Thu Apr 24 18:24:14 2003 Return-Path: Received: (from majordom@localhost) by sucuri.mat.puc-rio.br (8.9.3/8.9.3) id SAA20574 for obm-l-MTTP; Thu, 24 Apr 2003 18:22:49 -0300 Received: from smtp-26.ig.com.br (smtp-26.ig.com.br [200.226.132.160]) by sucuri.mat.puc-rio.br (8.9.3/8.9.3) with SMTP id SAA20568 for ; Thu, 24 Apr 2003 18:22:45 -0300 Received: (qmail 14563 invoked from network); 24 Apr 2003 21:22:25 -0000 Received: from unknown (HELO xxxx) (200.165.206.174) by smtp-26.ig.com.br with SMTP; 24 Apr 2003 21:22:25 -0000 Message-ID: <001901c30aa8$66d93400$aecea5c8@epq.ime.eb.br> From: "Marcio" To: References: <001601c30a80$211a9df0$7d07fea9@gauss> Subject: Re: [obm-l] FW: Problema dificil Date: Thu, 24 Apr 2003 18:28:08 -0300 MIME-Version: 1.0 Content-Type: text/plain; charset="iso-8859-1" Content-Transfer-Encoding: 8bit X-Priority: 3 X-MSMail-Priority: Normal X-Mailer: Microsoft Outlook Express 5.50.4133.2400 X-MimeOLE: Produced By Microsoft MimeOLE V5.50.4133.2400 Sender: owner-obm-l@sucuri.mat.puc-rio.br Precedence: bulk Reply-To: obm-l@mat.puc-rio.br Eh verdade, o Claudio ja havia me alertado sobre isso.. Estou pensando numa maneira de completar a solucao.. De fato, o mesmo raciocinio pode ser usado para mostrar que os 9 ultimos sao iguais (iguais a 1 se voce quiser).. Ainda estou tentando completar... []'s Marcio ----- Original Message ----- From: "Domingos Jr." To: Sent: Thursday, April 24, 2003 1:39 PM Subject: Re: [obm-l] FW: Problema dificil > Olá, Cláudio > > não esquenta não, eu só mandei a mensagem pq pode acontecer de você não ter > conseguido pegar o arquivo ou simplesmente não viu a mensagem postada > anteriormente... > > a propósito, ontem mesmo eu tinha chegado num resultado parcial dado pelo > Márcio: os 9 primeiros termos da seq. são todos iguais (creio que, > analogamente, os 9 últimos tb são iguais), porém, daí a concluir que todos > os elementos são iguais parece ser um pouco precipitado, acho que precisa de > alguma sacada pra chegar lá, mas ainda não vi a luz! > > uma pergunta: o que vc faz (curso universitário/mestrado, outros...)? > > [ ]'s ========================================================================= Instruções para entrar na lista, sair da lista e usar a lista em http://www.mat.puc-rio.br/~nicolau/olimp/obm-l.html ========================================================================= From owner-obm-l@sucuri.mat.puc-rio.br Thu Apr 24 18:25:48 2003 Return-Path: Received: (from majordom@localhost) by sucuri.mat.puc-rio.br (8.9.3/8.9.3) id SAA20716 for obm-l-MTTP; Thu, 24 Apr 2003 18:24:30 -0300 Received: from trex.centroin.com.br (trex.centroin.com.br [200.225.63.134]) by sucuri.mat.puc-rio.br (8.9.3/8.9.3) with ESMTP id SAA20711 for ; Thu, 24 Apr 2003 18:24:26 -0300 Received: from trex.centroin.com.br (localhost [127.0.0.1]) by trex.centroin.com.br (8.12.9/8.12.9) with ESMTP id h3OLNth5004659 for ; Thu, 24 Apr 2003 18:23:56 -0300 (EST) Received: by trex.centroin.com.br (8.12.9/8.12.5/Submit) id h3OLNtBv004652; Thu, 24 Apr 2003 18:23:55 -0300 (EST) Message-Id: <200304242123.h3OLNtBv004652@trex.centroin.com.br> Received: from 200.165.208.247 by trex.centroin.com.br (CIPWM versao 1.4C1) with HTTPS for ; Thu, 24 Apr 2003 18:23:55 -0300 (EST) Date: Thu, 24 Apr 2003 18:23:55 -0300 (EST) From: Augusto Cesar de Oliveira Morgado To: obm-l@mat.puc-rio.br Subject: Re: [obm-l] MIME-Version: 1.0 X-Mailer: CentroIn Internet Provider WebMail v. 1.4C1 (http://www.centroin.com.br/) Content-Type: text/plain; charset="iso-8859-1" Content-Transfer-Encoding: 8bit X-MIME-Autoconverted: from quoted-printable to 8bit by sucuri.mat.puc-rio.br id SAA20712 Sender: owner-obm-l@sucuri.mat.puc-rio.br Precedence: bulk Reply-To: obm-l@mat.puc-rio.br O raciocinio do problema 8 estah correto, mas ha uma distraçao gritante nas palavras de 4 letras, que sao 16 e nao 32,o que faz com que a resposta seja 30 (e nao 46) Morgado Em Thu, 24 Apr 2003 17:35:43 -0300 (ART), "J.C. PAREDE" disse: > Boa noite,Estou com dificuldades em resolver alguns problemas, até que resolvi alguns deles mas fiquei com duvidas quanto às respostas.Se alguém puder me ajudar... 2. Um campeonato é disputado por 12 clubes em rodadas de 6 jogos cada. De quantos modos é possível selecionar os jogos da primeira rodada. 12!/2 ? 8. O código Morse usa "palavras" contendo de 1 a 4 "letras". As "letras" são ponto e traço. Quantas "palavras" existe no código Morse? 1 letra = 22 letras = 43 letras = 84 letras = 32TOTAL = 46 > > > JOÃO CARLOS PAREDE > > > > --------------------------------- > Yahoo! Mail > O melhor e-mail gratuito da internet: 6MB de espaço, antivírus, acesso POP3, filtro contra spam. ========================================================================= Instruções para entrar na lista, sair da lista e usar a lista em http://www.mat.puc-rio.br/~nicolau/olimp/obm-l.html ========================================================================= From owner-obm-l@sucuri.mat.puc-rio.br Thu Apr 24 19:02:42 2003 Return-Path: Received: (from majordom@localhost) by sucuri.mat.puc-rio.br (8.9.3/8.9.3) id TAA23741 for obm-l-MTTP; Thu, 24 Apr 2003 19:01:11 -0300 Received: from ivoti.terra.com.br (ivoti.terra.com.br [200.176.3.20]) by sucuri.mat.puc-rio.br (8.9.3/8.9.3) with ESMTP id TAA23737 for ; Thu, 24 Apr 2003 19:01:07 -0300 Received: from bertioga.terra.com.br (bertioga.terra.com.br [200.176.3.77]) by ivoti.terra.com.br (Postfix) with ESMTP id 4D1D840886D for ; Thu, 24 Apr 2003 19:00:37 -0300 (BRT) Received: from [200.177.179.115] (dl-nas3-sao-C8B1B373.p001.terra.com.br [200.177.179.115]) by bertioga.terra.com.br (Postfix) with ESMTP id 4DDD03F8067 for ; Thu, 24 Apr 2003 19:00:36 -0300 (BRT) User-Agent: Microsoft-Outlook-Express-Macintosh-Edition/5.02.2022 Date: Thu, 24 Apr 2003 19:02:05 -0300 Subject: Re: [obm-l] Re: [obm-l] TEOIREMA DE CRISTEA::Geometria,pontos estranhos e problemas legais!!!! From: Claudio Buffara To: Message-ID: In-Reply-To: Mime-version: 1.0 X-Priority: 1 Content-type: text/plain; charset="ISO-8859-1" Content-Transfer-Encoding: 8bit X-MIME-Autoconverted: from quoted-printable to 8bit by sucuri.mat.puc-rio.br id TAA23738 Sender: owner-obm-l@sucuri.mat.puc-rio.br Precedence: bulk Reply-To: obm-l@mat.puc-rio.br Oi, Joao Carlos: Aqui estah a demonstracao do Dirichlet de que se existem inteiros a, b, c tais que a^4 + b^4 = c^2, entao a*b*c = 0. Com a palavra, o sr. Dirichlet: " Bem,o metodo da descida infinita ou descenso infinito pode ser descrito assim: i)Suponha que exista uma soluçao em naturais; ii)pegue uma soluçao minima em algum sentido; iii)tente chegar em contradiçao. Na verdade isto e o Principio da Boa Ordem para os Naturais e a relacao >. Vou demonstrar essa joça aqui: Se os naturais a,b,c sao tais que a^4+b^4=c^2 entao algum deles sera zero. Suponha o contrario,que os caras sao inteiros positivos.Entao existe uma soluçao tal que c seja o menor possivel(pelo PBO).Logo MDC(a,b) =1,e existem inteiros positivos u,v com a^2=u^2-v^2,b^2=2uv,c=u^2+v^2 (essa e a soluçao da equaçao diofantina de Pitagoras X^2+Y^2=Z^2).Reaplicando,vemos que existem inteiros positivos p,q com MDC(p,q)=1 e a=p^2-q^2,v=2pq,c=u^2+v^2. Com isso b^2=2uv=4pq(p^2+q^2).Logo como p e q sao primos entre si p,q e p^2+q^2 sao todos quadrados perfeitos.Logo existem A,B e C inteiros positivos tais que p=A^2,q=B^2,p^2+q^2=C^2. Mas com isso A^4+B^4=C^2 e c=u^2+v^2>u=p^2+q^2=C^2>C e ai temos duas contradiçoes: 1)c>C pois coisas grandes sao maiores que coisas pequenas:) ;) 2)c<=C por hipotese de minimalidade. E fim! " ***** Um abraco, Claudio. on 24.04.03 17:44, JoaoCarlos_Junior@net.ms.gov.br at JoaoCarlos_Junior@net.ms.gov.br wrote: > > Querido Cláudio, > > Gostaria de dar uma olhada na resolução do Dirichlet para Fermat com > n=4, se você assim a elogia. Desta forma, pergunto-te como faço para > olhá-la, em que site? > > Um forte abraço, João Carlos. > > ========================================================================= Instruções para entrar na lista, sair da lista e usar a lista em http://www.mat.puc-rio.br/~nicolau/olimp/obm-l.html ========================================================================= From owner-obm-l@sucuri.mat.puc-rio.br Thu Apr 24 19:05:20 2003 Return-Path: Received: (from majordom@localhost) by sucuri.mat.puc-rio.br (8.9.3/8.9.3) id TAA23943 for obm-l-MTTP; Thu, 24 Apr 2003 19:04:02 -0300 Received: from itaqui.terra.com.br (itaqui.terra.com.br [200.176.3.19]) by sucuri.mat.puc-rio.br (8.9.3/8.9.3) with ESMTP id TAA23912 for ; Thu, 24 Apr 2003 19:03:47 -0300 Received: from canela.terra.com.br (canela.terra.com.br [200.176.3.79]) by itaqui.terra.com.br (Postfix) with ESMTP id A045A3BD63A for ; Thu, 24 Apr 2003 19:03:16 -0300 (BRT) Received: from [200.177.179.115] (dl-nas3-sao-C8B1B373.p001.terra.com.br [200.177.179.115]) by canela.terra.com.br (Postfix) with ESMTP id C8F612240B2 for ; Thu, 24 Apr 2003 19:03:15 -0300 (BRT) User-Agent: Microsoft-Outlook-Express-Macintosh-Edition/5.02.2022 Date: Thu, 24 Apr 2003 19:05:25 -0300 Subject: Re: [obm-l] (log(x))^2 From: Claudio Buffara To: Message-ID: In-Reply-To: <00bb01c30aa0$195630f0$0301a8c0@usuario> Mime-version: 1.0 Content-type: multipart/alternative; boundary="MS_Mac_OE_3134055925_78173_MIME_Part" Sender: owner-obm-l@sucuri.mat.puc-rio.br Precedence: bulk Reply-To: obm-l@mat.puc-rio.br > This message is in MIME format. Since your mail reader does not understand this format, some or all of this message may not be legible. --MS_Mac_OE_3134055925_78173_MIME_Part Content-type: text/plain; charset="ISO-8859-1" Content-transfer-encoding: quoted-printable on 24.04.03 17:28, M=E1rio Pereira at marioappereira@terra.com.br wrote: como calcular: =20 (log x)^2 =20 (logaritmo neperiano de x, tudo elevado ao quadrado) =20 A resposta =E9 2logx/x =20 (parece que o desenvolvimento =E9 para ser feito pela defini=E7=E3o de limite). =20 Obrigado.=20 =20 M=E1rio.=20 Oi, Mario: O que voce quer eh a derivada de (log(x))^2, certo? Eh so aplicar a regra da cadeia e lembrar que D(log(x)) =3D 1/x. Assim: D( (log(x))^2 ) =3D 2*log(x)*D(log(x)) =3D 2*log(x)*(1/x). Um abraco, Claudio. --MS_Mac_OE_3134055925_78173_MIME_Part Content-type: text/html; charset="ISO-8859-1" Content-transfer-encoding: quoted-printable Re: [obm-l] (log(x))^2 on 24.04.03 17:28, M=E1rio Pereira at marioappereira@terra.com.br wrote:

como calcular:

(log x)^2

(logaritmo neperiano de x, tudo elevado a= o quadrado)

A resposta =E9 2logx/x

(parece que o desenvolvimento =E9 para ser = feito pela defini=E7=E3o de limite).

Obrigado.

M=E1rio.


Oi, Mario:

O que voce quer eh a derivada de (log(x))^2, certo?

Eh so aplicar a regra da cadeia e lembrar que D(log(x)) =3D 1/x. Assim:

D( (log(x))^2 ) =3D 2*log(x)*D(log(x)) =3D 2*log(x)*(1/x).

Um abraco,
Claudio.

--MS_Mac_OE_3134055925_78173_MIME_Part-- ========================================================================= Instruções para entrar na lista, sair da lista e usar a lista em http://www.mat.puc-rio.br/~nicolau/olimp/obm-l.html ========================================================================= From owner-obm-l@sucuri.mat.puc-rio.br Thu Apr 24 19:05:21 2003 Return-Path: Received: (from majordom@localhost) by sucuri.mat.puc-rio.br (8.9.3/8.9.3) id TAA23936 for obm-l-MTTP; Thu, 24 Apr 2003 19:04:01 -0300 Received: from trex.centroin.com.br (trex.centroin.com.br [200.225.63.134]) by sucuri.mat.puc-rio.br (8.9.3/8.9.3) with ESMTP id TAA23909 for ; Thu, 24 Apr 2003 19:03:45 -0300 Received: from trex.centroin.com.br (localhost [127.0.0.1]) by trex.centroin.com.br (8.12.9/8.12.9) with ESMTP id h3OM3Eh5028898 for ; Thu, 24 Apr 2003 19:03:14 -0300 (EST) Received: (from morgado@localhost) by trex.centroin.com.br (8.12.9/8.12.5/Submit) id h3OM3ERP028897; Thu, 24 Apr 2003 19:03:14 -0300 (EST) Message-Id: <200304242203.h3OM3ERP028897@trex.centroin.com.br> Received: from 200.165.208.247 by trex.centroin.com.br (CIPWM versao 1.4C1) with HTTPS for ; Thu, 24 Apr 2003 19:03:14 -0300 (EST) Date: Thu, 24 Apr 2003 19:03:14 -0300 (EST) From: Augusto Cesar de Oliveira Morgado To: obm-l@mat.puc-rio.br Subject: Re: [obm-l] MIME-Version: 1.0 X-Mailer: CentroIn Internet Provider WebMail v. 1.4C1 (http://www.centroin.com.br/) Content-Type: text/plain; charset="iso-8859-1" Content-Transfer-Encoding: 8bit X-MIME-Autoconverted: from quoted-printable to 8bit by sucuri.mat.puc-rio.br id TAA23911 Sender: owner-obm-l@sucuri.mat.puc-rio.br Precedence: bulk Reply-To: obm-l@mat.puc-rio.br Nao. Imagine a tabela da primeira rodada: -x- -x- -x- -x- -x- -x- Voce tem que por os 12 times nesses 12 lugares, o que pode ser feito de 12! modos. Mas, assim, voce conta a mesma tabela varias vezes porque voce pode trocar a ordem dos jogos (6! modos) e a ordem dos times em cada jogo(2^6 = 64 modos) sem alterar a tabela. A respsta eh 12!/[6!*64] Em Thu, 24 Apr 2003 17:35:43 -0300 (ART), "J.C. PAREDE" joaocarlosparede@yahoo.com.br> disse: Boa noite,Estou com dificuldades em resolver alguns problemas, até que resolvi alguns deles mas fiquei com duvidas quanto às respostas.Se alguém puder me ajudar... 2. Um campeonato é disputado por 12 clubes em rodadas de 6 jogos cada. De quantos modos é possível selecionar os jogos da primeira rodada. 12!/2 ? JOÃO CARLOS PAREDE ========================================================================= Instruções para entrar na lista, sair da lista e usar a lista em http://www.mat.puc-rio.br/~nicolau/olimp/obm-l.html ========================================================================= From owner-obm-l@sucuri.mat.puc-rio.br Thu Apr 24 19:07:14 2003 Return-Path: Received: (from majordom@localhost) by sucuri.mat.puc-rio.br (8.9.3/8.9.3) id TAA24086 for obm-l-MTTP; Thu, 24 Apr 2003 19:05:55 -0300 Received: from spinoza.ime.usp.br (spinoza.ime.usp.br [143.107.45.30]) by sucuri.mat.puc-rio.br (8.9.3/8.9.3) with SMTP id TAA24072 for ; Thu, 24 Apr 2003 19:05:47 -0300 Received: (qmail 20978 invoked from network); 24 Apr 2003 22:05:17 -0000 Received: from coliseu.linux.ime.usp.br (192.168.240.9) by spinoza.linux.ime.usp.br with SMTP; 24 Apr 2003 22:05:17 -0000 Received: (qmail 1122 invoked from network); 24 Apr 2003 22:05:17 -0000 Received: from tamanduatei.linux.ime.usp.br (192.168.240.209) by coliseu.linux.ime.usp.br with QMQP; 24 Apr 2003 22:05:17 -0000 Date: Thu, 24 Apr 2003 19:05:16 -0300 From: Wendel Scardua To: obm-l@mat.puc-rio.br Subject: Re: [[obm-l] ] Message-ID: <20030424220516.GD23868@linux.ime.usp.br> Mail-Followup-To: obm-l@mat.puc-rio.br References: <110HDXVTd6704S10.1051219169@cmsweb10.cms.usa.net> Mime-Version: 1.0 Content-Type: text/plain; charset=iso-8859-1 Content-Disposition: inline Content-Transfer-Encoding: 8bit In-Reply-To: <110HDXVTd6704S10.1051219169@cmsweb10.cms.usa.net> User-Agent: Mutt/1.3.28i Sender: owner-obm-l@sucuri.mat.puc-rio.br Precedence: bulk Reply-To: obm-l@mat.puc-rio.br > > como calcular: > > > > (log x)^2 > > > > A resposta é 2logx/x > > > Nao me lembro de uma expressao interessante para (log x)^2, mas 2logx/x > DECIDIDAMENTE nao eh. Talvez ele tenha confundido com: ( (log x)^2 )' Isso sim é 2(logx)/x []'s Wendel Scardua -------------------------------------------- ========================================================================= Instruções para entrar na lista, sair da lista e usar a lista em http://www.mat.puc-rio.br/~nicolau/olimp/obm-l.html ========================================================================= From owner-obm-l@sucuri.mat.puc-rio.br Thu Apr 24 19:37:05 2003 Return-Path: Received: (from majordom@localhost) by sucuri.mat.puc-rio.br (8.9.3/8.9.3) id TAA25968 for obm-l-MTTP; Thu, 24 Apr 2003 19:35:21 -0300 Received: from itaqui.terra.com.br (itaqui.terra.com.br [200.176.3.19]) by sucuri.mat.puc-rio.br (8.9.3/8.9.3) with ESMTP id TAA25962 for ; Thu, 24 Apr 2003 19:35:16 -0300 Received: from araci.terra.com.br (araci.terra.com.br [200.176.3.44]) by itaqui.terra.com.br (Postfix) with ESMTP id BDAA43BD981 for ; Thu, 24 Apr 2003 19:34:44 -0300 (BRT) Received: from niski.com (unknown [200.148.202.95]) (authenticated user fniski) by araci.terra.com.br (Postfix) with ESMTP id 79C9821EF74 for ; Thu, 24 Apr 2003 19:34:43 -0300 (BRT) Message-ID: <3EA86680.9040704@niski.com> Date: Thu, 24 Apr 2003 15:34:40 -0700 From: niski User-Agent: Mozilla/5.0 (Windows; U; Windows NT 5.1; en-US; rv:1.0.2) Gecko/20030208 Netscape/7.02 X-Accept-Language: en-us, en MIME-Version: 1.0 To: obm-l@mat.puc-rio.br Subject: Re: [obm-l] UFMG 2002 References: <200304242121.h3OLLP6S003267@trex.centroin.com.br> Content-Type: text/plain; charset=ISO-8859-1; format=flowed Content-Transfer-Encoding: 8bit Sender: owner-obm-l@sucuri.mat.puc-rio.br Precedence: bulk Reply-To: obm-l@mat.puc-rio.br Augusto Cesar de Oliveira Morgado wrote: > Infelizmente esta soluçao estah errada. > Morgado De fato prof. morgado. Acho q me precipitei... Vou tentar novamente. Escolhendo 3 profs. da manha e 1 da tarde temos C(10,3). C(8,1) Escolhendo 2 profs da manha e 2 da tarde temos C(10,2).C(8,2) Escolhendo 1 prof da manha e 3 da tarde temos C(10,1).C(8,3) Escolhendo 1 da manha , 1 da tarde e 2 da noite temos C(10,1).C(8,1).C(4,2) Escolhendo 2 da manha , 1 da tarde e 1 da noite temos C(10,2).C(8,1).C(4,1) Escolhendo 1 da manha , 2 da tarde e 1 da noite temos C(10,1).C(8,2).C(4,1) Somando tudo 5820 maneiras > > Em Thu, 24 Apr 2003 13:00:40 -0700, niski disse: > > >>Devemos então escolher inicialmente 1 professor que lecione pela manha >>C(10,1) e 1 que lecione de tarde C(8,1). Sobraram 2 dos 21 professores >>p/ serem selecionados, e fazemos isso com C(21,2). >>Então o total de maneiras possiveis é >>C(10,1).C(8,1).C(21,2) = 16800. >> >> >> >>Republica Tcheka wrote: >> >>>estou com uma duvida d qual a resposta d uma questao aberta d matematica: >>> >>>uma escola tem 10 professores q lecionam pela manha, 8 pela tarde e 5 >>>pela noite.d qnts forma podem ser formadas comissoes d 4 professores d >>>forma q tenha pelo menos 1 prof. q lecione pela manha e pelo menos 1 q >>>lecione pela tarde. >>> >>>OBS:essa e a letra C da questao. >>> >>> >>>Obrigado >>> >>> >>> >>> >>>_________________________________________________________________ >>>MSN Messenger: converse com os seus amigos online. >>>http://messenger.msn.com.br >>> >>>========================================================================= >>>Instruções para entrar na lista, sair da lista e usar a lista em >>>http://www.mat.puc-rio.br/~nicolau/olimp/obm-l.html >>>========================================================================= >>> >>> >> >> >>-- >>[about him:] >> It is rare to find learned men who are clean, do not stink and have a >>sense of humour. >>-Gottfried Whilhem Leibniz >> >>========================================================================= >>Instruções para entrar na lista, sair da lista e usar a lista em >>http://www.mat.puc-rio.br/~nicolau/olimp/obm-l.html >>========================================================================= >> >> > > > ========================================================================= > Instruções para entrar na lista, sair da lista e usar a lista em > http://www.mat.puc-rio.br/~nicolau/olimp/obm-l.html > ========================================================================= > > -- [about him:] It is rare to find learned men who are clean, do not stink and have a sense of humour. -Gottfried Whilhem Leibniz ========================================================================= Instruções para entrar na lista, sair da lista e usar a lista em http://www.mat.puc-rio.br/~nicolau/olimp/obm-l.html ========================================================================= From owner-obm-l@sucuri.mat.puc-rio.br Thu Apr 24 19:39:52 2003 Return-Path: Received: (from majordom@localhost) by sucuri.mat.puc-rio.br (8.9.3/8.9.3) id TAA26118 for obm-l-MTTP; Thu, 24 Apr 2003 19:38:31 -0300 Received: from paiol.terra.com.br (paiol.terra.com.br [200.176.3.18]) by sucuri.mat.puc-rio.br (8.9.3/8.9.3) with ESMTP id TAA26111 for ; Thu, 24 Apr 2003 19:38:24 -0300 Received: from araci.terra.com.br (araci.terra.com.br [200.176.3.44]) by paiol.terra.com.br (Postfix) with ESMTP id C2C5D87DC3 for ; Thu, 24 Apr 2003 19:37:53 -0300 (BRT) Received: from [200.177.182.121] (dl-nas6-sao-C8B1B679.p001.terra.com.br [200.177.182.121]) by araci.terra.com.br (Postfix) with ESMTP id DA8D821EF75 for ; Thu, 24 Apr 2003 19:37:52 -0300 (BRT) User-Agent: Microsoft-Outlook-Express-Macintosh-Edition/5.02.2022 Date: Thu, 24 Apr 2003 19:39:22 -0300 Subject: Re: [obm-l] Re: [obm-l] euclidiana From: Claudio Buffara To: Message-ID: In-Reply-To: <00ac01c30a9f$bba7a5b0$0301a8c0@usuario> Mime-version: 1.0 Content-type: text/plain; charset="ISO-8859-1" Content-Transfer-Encoding: 8bit X-MIME-Autoconverted: from quoted-printable to 8bit by sucuri.mat.puc-rio.br id TAA26112 Sender: owner-obm-l@sucuri.mat.puc-rio.br Precedence: bulk Reply-To: obm-l@mat.puc-rio.br Qual a definicao que o seu livro-texto e/ou professor(a) da para "sistema de unidades de medidas" ? on 24.04.03 17:26, Mário Pereira at marioappereira@terra.com.br wrote: > Matemática. Geometria Euclidiana. > > Mário. > > > ----- Original Message ----- > From: "Cláudio (Prática)" > To: > Sent: Thursday, April 24, 2003 4:07 PM > Subject: Re: [obm-l] Re: [obm-l] euclidiana > > >> Oi, Mário: >> >> O que parece estar faltando é a definição formal de "Sistema de Unidades > de >> Medidas" e de "sistema". Além disso, quais são as formas válidas de >> verificação? Em matemática, é uma demonstração formal usando lógica. Em >> física e outras ciências naturais, é um experimento e/ou observação. >> >> Isso foi questão de uma prova de matemática ou física? >> >> Um abraço, >> Claudio. >> >> ----- Original Message ----- >> From: "Mário Pereira" >> To: >> Sent: Thursday, April 24, 2003 3:08 PM >> Subject: Re: [obm-l] Re: [obm-l] euclidiana >> >> >>> A pergunta da prova foi essa. >>> >>> >>> Mário >>> >>> ----- Original Message ----- >>> From: >>> To: >>> Sent: Thursday, April 24, 2003 1:02 PM >>> Subject: [obm-l] Re: [obm-l] euclidiana >>> >>> >>>> >>>> Pode traduzir? >>>> -- Mensagem original -- >>>> >>>>> >>>>> Como podemos verificar que um sistema de unidades de medidas é, de >> fato, >>>>> um sistema? >>>>> >>>>> Obrigado, >>>>> >>>>> Mário. >>>>> >>>> ========================================================================= Instruções para entrar na lista, sair da lista e usar a lista em http://www.mat.puc-rio.br/~nicolau/olimp/obm-l.html ========================================================================= From owner-obm-l@sucuri.mat.puc-rio.br Thu Apr 24 19:39:53 2003 Return-Path: Received: (from majordom@localhost) by sucuri.mat.puc-rio.br (8.9.3/8.9.3) id TAA26132 for obm-l-MTTP; Thu, 24 Apr 2003 19:38:36 -0300 Received: from ivoti.terra.com.br (ivoti.terra.com.br [200.176.3.20]) by sucuri.mat.puc-rio.br (8.9.3/8.9.3) with ESMTP id TAA26114 for ; Thu, 24 Apr 2003 19:38:28 -0300 Received: from araci.terra.com.br (araci.terra.com.br [200.176.3.44]) by ivoti.terra.com.br (Postfix) with ESMTP id 504EE40898E for ; Thu, 24 Apr 2003 19:37:57 -0300 (BRT) Received: from [200.177.182.121] (dl-nas6-sao-C8B1B679.p001.terra.com.br [200.177.182.121]) by araci.terra.com.br (Postfix) with ESMTP id 5992421EF91 for ; Thu, 24 Apr 2003 19:37:56 -0300 (BRT) User-Agent: Microsoft-Outlook-Express-Macintosh-Edition/5.02.2022 Date: Thu, 24 Apr 2003 19:39:22 -0300 Subject: Re: [obm-l] From: Claudio Buffara To: Message-ID: In-Reply-To: <20030424203543.53705.qmail@web40508.mail.yahoo.com> Mime-version: 1.0 Content-type: multipart/alternative; boundary="MS_Mac_OE_3134058006_203354_MIME_Part" Sender: owner-obm-l@sucuri.mat.puc-rio.br Precedence: bulk Reply-To: obm-l@mat.puc-rio.br > This message is in MIME format. Since your mail reader does not understand this format, some or all of this message may not be legible. --MS_Mac_OE_3134058006_203354_MIME_Part Content-type: text/plain; charset="ISO-8859-1" Content-transfer-encoding: quoted-printable on 24.04.03 17:35, J.C. PAREDE at joaocarlosparede@yahoo.com.br wrote: Boa noite, Estou com dificuldades em resolver alguns problemas, at=E9 que resolvi alguns deles mas fiquei com duvidas quanto =E0s respostas. Se algu=E9m puder me ajudar... =20 2. Um campeonato =E9 disputado por 12 clubes em rodadas de 6 jogos cada. De quantos modos =E9 poss=EDvel selecionar os jogos da primeira rodada. =20 12!/2 ? Na verdade eh: C(12,2)*C(10,2)*C(8,2)*C(6,2)*C(4,2)*C(2,2)/6! =3D 12!/(2^6 * 6!) (escolhe os 6 pares de times e depois divide pelo numero de permutacoes destes 6 pares, para evitar dupla contagem) Um abraco, Claudio. --MS_Mac_OE_3134058006_203354_MIME_Part Content-type: text/html; charset="ISO-8859-1" Content-transfer-encoding: quoted-printable Re: [obm-l] on 24.04.03 17:35, J.C. PAREDE at joaocarlosparede@yahoo.com.br wrote:

Boa noite,
Estou com dificuldades em resolver alguns problemas, at=E9 que resolvi alguns= deles mas fiquei com duvidas quanto =E0s respostas.
Se algu=E9m puder me ajudar...


2. Um campeonato =E9 disputado por 12 clube= s em rodadas de 6 jogos cada. De quantos modos =E9 poss=EDvel selecionar os jogo= s da primeira rodada.

12!/2 ?

Na verdade eh:
C(12,2)*C(10,2)*C(8,2)*C(6,2)*C(4,2)*C(2,2)/6! =3D 12!/(2^6 * 6!)

(escolhe os 6 pares de times e depois divide pelo numero de permutacoes des= tes 6 pares, para evitar dupla contagem)


Um abraco,
Claudio.
--MS_Mac_OE_3134058006_203354_MIME_Part-- ========================================================================= Instruções para entrar na lista, sair da lista e usar a lista em http://www.mat.puc-rio.br/~nicolau/olimp/obm-l.html ========================================================================= From owner-obm-l@sucuri.mat.puc-rio.br Thu Apr 24 19:39:54 2003 Return-Path: Received: (from majordom@localhost) by sucuri.mat.puc-rio.br (8.9.3/8.9.3) id TAA26126 for obm-l-MTTP; Thu, 24 Apr 2003 19:38:34 -0300 Received: from ivoti.terra.com.br (ivoti.terra.com.br [200.176.3.20]) by sucuri.mat.puc-rio.br (8.9.3/8.9.3) with ESMTP id TAA26113 for ; Thu, 24 Apr 2003 19:38:26 -0300 Received: from araci.terra.com.br (araci.terra.com.br [200.176.3.44]) by ivoti.terra.com.br (Postfix) with ESMTP id DBC164084BF for ; Thu, 24 Apr 2003 19:37:55 -0300 (BRT) Received: from [200.177.182.121] (dl-nas6-sao-C8B1B679.p001.terra.com.br [200.177.182.121]) by araci.terra.com.br (Postfix) with ESMTP id 19EE721EF91 for ; Thu, 24 Apr 2003 19:37:54 -0300 (BRT) User-Agent: Microsoft-Outlook-Express-Macintosh-Edition/5.02.2022 Date: Thu, 24 Apr 2003 19:39:22 -0300 Subject: Re: [obm-l] UFMG 2002 From: Claudio Buffara To: Message-ID: In-Reply-To: Mime-version: 1.0 Content-type: text/plain; charset="US-ASCII" Content-transfer-encoding: 7bit Sender: owner-obm-l@sucuri.mat.puc-rio.br Precedence: bulk Reply-To: obm-l@mat.puc-rio.br on 24.04.03 17:15, Republica Tcheka at rep_tcheka@hotmail.com wrote: > estou com uma duvida d qual a resposta d uma questao aberta d matematica: > > uma escola tem 10 professores q lecionam pela manha, 8 pela tarde e 5 pela > noite.d qnts forma podem ser formadas comissoes d 4 professores d forma q > tenha pelo menos 1 prof. q lecione pela manha e pelo menos 1 q lecione pela > tarde. > > OBS:essa e a letra C da questao. > Numero de comissoes de 4 professores sem restricao: C(10+8+5,4) = C(23,4) Numero de comissoes de 4 professores sem nenhum que lecione de manha: C(13,4) (so sobram 8 da tarde + 5 da noite pra formar a comissao) Numero de comissoes de 4 professores sem nenhum que lecione de tarde: C(15,4) (so sobram 10 da manha + 5 da noite) Numero de comissoes de 4 professores sem nenhum que lecione de manha nem de tarde: C(5,4) (so sobram os 5 da noite) Logo, o numero de comissoes distintas que podem ser formadas nas condicoes do enunciado eh (pelo principio da inclusao-exclusao): C(23,4) - C(13,4) - C(15,4) + C(5,4) = 8.855 - 715 - 1.365 + 5 = 6.780. Um abraco, Claudio. ========================================================================= Instruções para entrar na lista, sair da lista e usar a lista em http://www.mat.puc-rio.br/~nicolau/olimp/obm-l.html ========================================================================= From owner-obm-l@sucuri.mat.puc-rio.br Thu Apr 24 19:40:09 2003 Return-Path: Received: (from majordom@localhost) by sucuri.mat.puc-rio.br (8.9.3/8.9.3) id TAA26154 for obm-l-MTTP; Thu, 24 Apr 2003 19:38:50 -0300 Received: from trex-b.centroin.com.br (trex-b.centroin.com.br [200.225.63.136]) by sucuri.mat.puc-rio.br (8.9.3/8.9.3) with ESMTP id TAA26137 for ; Thu, 24 Apr 2003 19:38:43 -0300 Received: from centroin.com.br (RJ208247.user.veloxzone.com.br [200.165.208.247] (may be forged)) (authenticated bits=0) by trex-b.centroin.com.br (8.12.9/8.12.9) with ESMTP id h3OMcDVi020037 for ; Thu, 24 Apr 2003 19:38:13 -0300 (EST) Message-ID: <3EA86792.8070205@centroin.com.br> Date: Thu, 24 Apr 2003 19:39:14 -0300 From: "A. C. Morgado" User-Agent: Mozilla/5.0 (Windows; U; Windows NT 5.0; en-US; rv:1.0.2) Gecko/20030208 Netscape/7.02 X-Accept-Language: en-us, en MIME-Version: 1.0 To: obm-l@mat.puc-rio.br Subject: Re: [obm-l] UFMG 2002 References: <200304242121.h3OLLP6S003267@trex.centroin.com.br> Content-Type: multipart/alternative; boundary="------------000904060308080405000308" Sender: owner-obm-l@sucuri.mat.puc-rio.br Precedence: bulk Reply-To: obm-l@mat.puc-rio.br --------------000904060308080405000308 Content-Type: text/plain; charset=ISO-8859-1; format=flowed Content-Transfer-Encoding: 8bit 1 da manha, 1 tarde, 2 da noite 10* 8* C(5,2) = 800 1 da manha, 2 da tarde, 1 da noite 10*C(8,2)*5 = 1 400 1da manha, 3 da tarde 10*C(8,3) = 560 2 da manha, 1 da tarde, 1 da noite C(10,2)*8*5 = 1800 2 da manha, 2 da tarde C(10,2)*C(8,2) = 1260 3 da manha , 1 da tarde C(10,3)*8 = 960 A resposta eh a soma 6780. >>Republica Tcheka wrote: >> >> >>>estou com uma duvida d qual a resposta d uma questao aberta d matematica: >>> >>>uma escola tem 10 professores q lecionam pela manha, 8 pela tarde e 5 >>>pela noite.d qnts forma podem ser formadas comissoes d 4 professores d >>>forma q tenha pelo menos 1 prof. q lecione pela manha e pelo menos 1 q >>>lecione pela tarde. >>> >>>OBS:essa e a letra C da questao. >>> >>> >>>Obrigado >>> >>> >>> >>> >>>_________________________________________________________________ >>>MSN Messenger: converse com os seus amigos online. >>>http://messenger.msn.com.br >>> >>>========================================================================= >>>Instruções para entrar na lista, sair da lista e usar a lista em >>>http://www.mat.puc-rio.br/~nicolau/olimp/obm-l.html >>>========================================================================= >>> >>> >>> >>> >>-- >>[about him:] >> It is rare to find learned men who are clean, do not stink and have a >>sense of humour. >>-Gottfried Whilhem Leibniz >> >>========================================================================= >>Instruções para entrar na lista, sair da lista e usar a lista em >>http://www.mat.puc-rio.br/~nicolau/olimp/obm-l.html >>========================================================================= >> >> >> >> > >========================================================================= >Instruções para entrar na lista, sair da lista e usar a lista em >http://www.mat.puc-rio.br/~nicolau/olimp/obm-l.html >========================================================================= > > > > --------------000904060308080405000308 Content-Type: text/html; charset=us-ascii Content-Transfer-Encoding: 7bit 1 da manha, 1 tarde, 2 da noite   10* 8* C(5,2) = 800
1 da manha, 2 da tarde, 1 da noite  10*C(8,2)*5 = 1 400
1da manha, 3 da tarde     10*C(8,3) = 560
2 da manha, 1 da tarde, 1 da noite   C(10,2)*8*5 = 1800
2 da manha, 2 da tarde  C(10,2)*C(8,2) = 1260
3 da manha , 1 da tarde  C(10,3)*8 = 960
A resposta eh a soma 6780.

Republica Tcheka wrote:
    
estou com uma duvida d qual a resposta d uma questao aberta d matematica:

uma escola tem 10 professores q lecionam pela manha, 8 pela tarde e 5 
pela noite.d qnts forma podem ser formadas comissoes d 4 professores d 
forma q tenha pelo menos 1 prof. q lecione pela manha e pelo menos 1 q 
lecione pela tarde.

OBS:essa e a letra C da questao.


Obrigado




_________________________________________________________________
MSN Messenger: converse com os seus amigos online.  
http://messenger.msn.com.br

=========================================================================
Instruções para entrar na lista, sair da lista e usar a lista em
http://www.mat.puc-rio.br/~nicolau/olimp/obm-l.html
=========================================================================


      
-- 
[about him:]
  It is rare to find learned men who are clean, do not stink and have a 
sense of humour.
-Gottfried Whilhem Leibniz

=========================================================================
Instruções para entrar na lista, sair da lista e usar a lista em
http://www.mat.puc-rio.br/~nicolau/olimp/obm-l.html
=========================================================================


    

=========================================================================
Instruções para entrar na lista, sair da lista e usar a lista em
http://www.mat.puc-rio.br/~nicolau/olimp/obm-l.html
=========================================================================


  

--------------000904060308080405000308-- ========================================================================= Instruções para entrar na lista, sair da lista e usar a lista em http://www.mat.puc-rio.br/~nicolau/olimp/obm-l.html ========================================================================= From owner-obm-l@sucuri.mat.puc-rio.br Thu Apr 24 19:55:06 2003 Return-Path: Received: (from majordom@localhost) by sucuri.mat.puc-rio.br (8.9.3/8.9.3) id TAA27885 for obm-l-MTTP; Thu, 24 Apr 2003 19:53:31 -0300 Received: from trex-b.centroin.com.br (trex-b.centroin.com.br [200.225.63.136]) by sucuri.mat.puc-rio.br (8.9.3/8.9.3) with ESMTP id TAA27876 for ; Thu, 24 Apr 2003 19:53:27 -0300 Received: from centroin.com.br (RJ208247.user.veloxzone.com.br [200.165.208.247] (may be forged)) (authenticated bits=0) by trex-b.centroin.com.br (8.12.9/8.12.9) with ESMTP id h3OMqtVi020404 for ; Thu, 24 Apr 2003 19:52:56 -0300 (EST) Message-ID: <3EA86B06.50201@centroin.com.br> Date: Thu, 24 Apr 2003 19:53:58 -0300 From: "A. C. Morgado" User-Agent: Mozilla/5.0 (Windows; U; Windows NT 5.0; en-US; rv:1.0.2) Gecko/20030208 Netscape/7.02 X-Accept-Language: en-us, en MIME-Version: 1.0 To: obm-l@mat.puc-rio.br Subject: Re: [[obm-l] e mais probabilidades!] References: <812HDXuWQ8672S17.1051217322@uwdvg017.cms.usa.net> Content-Type: text/plain; charset=ISO-8859-1; format=flowed Content-Transfer-Encoding: 8bit Sender: owner-obm-l@sucuri.mat.puc-rio.br Precedence: bulk Reply-To: obm-l@mat.puc-rio.br >>As cinco primeiras repeticoes de um experimento custam $10,00 cada. >>Todas as repeticoes subsequentes custam $5,00 cada. Suponha que o >>experimento seja repetido até que o primeiro sucesso ocorra. Se a >>probabilidade de sucesso de uma repeticao é igual a 0,9, e se as >>repeticoes sao independentes, qual é o custo esperado da operação? >> Pense no custo C como formado de duas parcelas, o custo fixo F de 5 por experiencia mais o custo adicional A que eh de 5 nas 5 primeiras experiencias e eh de zero nas demais. C = F + A E(C) = E(F) + E(A) Seja X o numero de experiencias realizadas ateh o primeiro sucesso (inclusive). X tem distribuiçao geometrica com parametro p=0,9 e, portanto com media 1/0,9 = 10/9. F= 5X, logo E(F) = 5E(X) = 50/9. E(A) a gente calcula no braço. E(A) = 5*0,9 + 10*0,1*0,9 + 15*(0,1^2)*0,9 + 20*(0,1^3)*0,9 + 25*(0,1^4)*0,9 + 0 = 5,55525 E(C) = 10/9 + 5,555 25 ========================================================================= Instruções para entrar na lista, sair da lista e usar a lista em http://www.mat.puc-rio.br/~nicolau/olimp/obm-l.html ========================================================================= From owner-obm-l@sucuri.mat.puc-rio.br Thu Apr 24 21:41:23 2003 Return-Path: Received: (from majordom@localhost) by sucuri.mat.puc-rio.br (8.9.3/8.9.3) id VAA31735 for obm-l-MTTP; Thu, 24 Apr 2003 21:39:42 -0300 Received: from traven.uol.com.br (traven.uol.com.br [200.221.29.39]) by sucuri.mat.puc-rio.br (8.9.3/8.9.3) with ESMTP id VAA31731 for ; Thu, 24 Apr 2003 21:39:40 -0300 Received: from gauss ([200.158.96.74]) by traven.uol.com.br (8.9.1/8.9.1) with SMTP id VAA28210 for ; Thu, 24 Apr 2003 21:39:09 -0300 (BRT) Message-ID: <001301c30ac3$cfb98bd0$7d07fea9@gauss> From: "Domingos Jr." To: References: Subject: Re: [obm-l] Re: [obm-l] TEOIREMA DE CRISTEA::Geometria,pontosestranhos e problemas legais!!!! Date: Thu, 24 Apr 2003 21:44:19 -0300 MIME-Version: 1.0 Content-Type: text/plain; charset="iso-8859-1" Content-Transfer-Encoding: 8bit X-Priority: 3 X-MSMail-Priority: Normal X-Mailer: Microsoft Outlook Express 6.00.2800.1158 X-MimeOLE: Produced By Microsoft MimeOLE V6.00.2800.1165 Sender: owner-obm-l@sucuri.mat.puc-rio.br Precedence: bulk Reply-To: obm-l@mat.puc-rio.br > Aqui estah a demonstracao do Dirichlet de que se existem inteiros a, b, c > tais que a^4 + b^4 = c^2, entao a*b*c = 0. > > Com a palavra, o sr. Dirichlet: essa é uma demonstração que eu já tinha visto em um livro sobre o teorema de Fermat (acho que era "lectures on fermat's last theorem"), na verdade essa demonstração usando "infinite descent" (descida infinita) foi feita pelo próprio Fermat! tb é uma das minhas favoritas... [ ]'s ========================================================================= Instruções para entrar na lista, sair da lista e usar a lista em http://www.mat.puc-rio.br/~nicolau/olimp/obm-l.html ========================================================================= From owner-obm-l@sucuri.mat.puc-rio.br Thu Apr 24 22:18:08 2003 Return-Path: Received: (from majordom@localhost) by sucuri.mat.puc-rio.br (8.9.3/8.9.3) id WAA32696 for obm-l-MTTP; Thu, 24 Apr 2003 22:16:38 -0300 Received: from Euler.impa.br (euler.impa.br [147.65.1.3]) by sucuri.mat.puc-rio.br (8.9.3/8.9.3) with ESMTP id WAA32692 for ; Thu, 24 Apr 2003 22:16:35 -0300 Received: from [147.65.11.5] (dial05.impa.br [147.65.11.5]) by Euler.impa.br (8.11.6p2/8.11.6) with ESMTP id h3P1G2023653 for ; Thu, 24 Apr 2003 22:16:02 -0300 (EST) Message-Id: <200304250116.h3P1G2023653@Euler.impa.br> X-Mailer: Microsoft Outlook Express Macintosh Edition - 4.5 (0410) Date: Thu, 24 Apr 2003 22:14:50 -0300 Subject: Re: [obm-l] Ensino de matematica no Brasil From: "Eduardo Wagner" To: obm-l@mat.puc-rio.br Mime-version: 1.0 X-Priority: 3 Content-type: text/plain; charset="ISO-8859-1" Content-Transfer-Encoding: 8bit X-MIME-Autoconverted: from quoted-printable to 8bit by sucuri.mat.puc-rio.br id WAA32693 Sender: owner-obm-l@sucuri.mat.puc-rio.br Precedence: bulk Reply-To: obm-l@mat.puc-rio.br Caro prof. Piola: Todos os anos, nos dias que antecedem a Olimpiada Iberoamericana, a OEI promove um Simposio sobre ensino de Matematica reunindo um grande numero de professores locais (do pais que sedia a Olimpiada) e um numero expressivo de professores de outros paises, uns convidados e outros que serao lideres na Olimpiada e puderam chegar antes para participar. Os temas tratados sao muito variados: experiencias em sala de aula, tecnicas de resolucao de problemas, ensino de geometria (tema sempre presente), a utilizacao do computador no ensino, etc. Eu tenho estado nestes Simposios em diversos paises e posso relatar que sempre, sempre, o maior desejo do professor eh aumentar seu conhecimento da materia que ensina. Todos sentem que sabendo mais, vao ensinar melhor, terao mais alternativas para ensinar e experimentar o efeito no aprendizado dos alunos. Em suma, quero enfatizar que os professores que participam dos Simposios buscam, em sua imensa maioria, melhorar seu conteudo matematico. A reuniao de Antigua (a cidade mais linda da America Central) sera realizada agora no fim do mes, por solicitacao de professores da centroamerica que desejam promover uma melhoria no ensino de matematica em seus paises e perceberam que isto so sera possivel melhorando o conteudo matematico dos professores. Em principio, esta reuniao seria dedicada aos professores dos paises da America Central, mas a OEI pediu auxilio a varios outros paises que ja possuem projetos nesse sentido e o Brasil tem uma bela experiencia a relatar. ---------- >From: "Piola" >To: >Subject: Re: [obm-l] Ensino de matematica no Brasil >Date: Thu, Apr 24, 2003, 2:01 PM > > Caro Professor Eduardo > Seria excelente se todos nós pudéssemos receber notícias sobre esse grande > encontro em Antigua. Penso que o assunto é sim do interesse dessa Lista! > É claro que não se pode ensinar aquilo que mal se sabe. > Prof. Piola. > ----- Original Message ----- > From: Eduardo Wagner > To: > Sent: Wednesday, April 23, 2003 9:04 PM > Subject: Re: [obm-l] Ensino de matematica no Brasil > > >> A lista nao eh realmente o forum adequado para discussao >> sobre ensino de matematica, mas nao posso deixar de >> registrar o meu total apoio em relacao ao que disse >> a prof. Suely Druck. >> Lamentamos o artigo do prof. Kulesza que mostrou nao >> compreender o que leu e que nem soube rabater os solidos >> argumentos da prof. Suely. O artigo do prof. Kulesza eh >> inconsistente e mediocre. A resposta do prof. Michel Spira >> nao poderia ser melhor. Disse tudo o que era necessario. >> A preocupacao com a melhoria do conteudo dos professores >> do ensino fundamental em medio eh tema atual em inumeros >> paises. >> Para a informacao de todos, a OEI (Organizacao >> dos Estados Iberoamericanos para Ciencia e Cultura) >> esta promovendo nos dias 28 a 30 de abril um grande >> simposio em Antigua, reunindo professores de matematica, >> educadores e autoridades em educacao de todos os paises >> de lingua espanhola e portuguesa para tratar exatamente >> deste tema: a melhoria do conteudo matematico dos >> professores para um melhor ensino. O Brasil estara >> fortemente presente, transmitindo a experiencia que >> temos no assunto atraves dos programas patrocinados >> pelo IMPA e pela Sociedade Brasileira de Matematica. >> >> Eduardo Wagner. >> >> PS. Benedito: foi bom voce ter escrito pois me motivou a >> escrever tambem. Sera que alguem pode ensinar portugues >> sem conhecer a lingua portuguesa? >> >> ---------- >> >From: benedito >> >To: obm-l@mat.puc-rio.br >> >Subject: Re: [obm-l] Ensino de matematica no Brasil >> >Date: Mon, Apr 21, 2003, 8:22 PM >> > >> >> > Muita gente pode não ter gostado....Mas as declarações da Presidente > tem >> > muita verdade, isso tem. Incomoda alguns, especialmente aqueles que > focam o >> > ensino nos métodos, esquecendo o principal: o conteúdo... >> > Benedito Freire >> > >> > At 19:30 21/4/2003 -0300, you wrote: >> >>Não é exatamente o assunto da lista, mas a Suely Druck (presidente da > SBM) >> >>publicou um artigo na Folha sobre o ensino de matemática no Brasil. >> >>As olimpíadas de matemática são mencionadas mas não são o tema do > artigo. >> >>Nem todos os leitores gostaram gostaram. Vejam os links abaixo >> >>(obrigado ao Gugu pela referência) vale a pena ler e refletir, >> >>talvez participar do debate. []s, N. >> >> >> >>http://www1.folha.uol.com.br/folha/sinapse/ult1063u343.shtml >> >>http://www.jornaldaciencia.org.br/Detalhe.jsp?id=8868 >> >>http://www.jornaldaciencia.org.br/Detalhe.jsp?id=9238 >> >>http://www.jornaldaciencia.org.br/Detalhe.jsp?id=9105 >> >> >> >> >> >> >> >>>========================================================================= >> >>Instruções para entrar na lista, sair da lista e usar a lista em >> >>http://www.mat.puc-rio.br/~nicolau/olimp/obm-l.html >> >>O administrador desta lista é >> >>>========================================================================= >> > >> > >> > > ========================================================================= >> > Instruções para entrar na lista, sair da lista e usar a lista em >> > http://www.mat.puc-rio.br/~nicolau/olimp/obm-l.html >> > O administrador desta lista é >> > > ========================================================================= >> ========================================================================= >> Instruções para entrar na lista, sair da lista e usar a lista em >> http://www.mat.puc-rio.br/~nicolau/olimp/obm-l.html >> ========================================================================= >> > > ========================================================================= > Instruções para entrar na lista, sair da lista e usar a lista em > http://www.mat.puc-rio.br/~nicolau/olimp/obm-l.html > ========================================================================= ========================================================================= Instruções para entrar na lista, sair da lista e usar a lista em http://www.mat.puc-rio.br/~nicolau/olimp/obm-l.html ========================================================================= From owner-obm-l@sucuri.mat.puc-rio.br Thu Apr 24 22:56:08 2003 Return-Path: Received: (from majordom@localhost) by sucuri.mat.puc-rio.br (8.9.3/8.9.3) id WAA01355 for obm-l-MTTP; Thu, 24 Apr 2003 22:54:23 -0300 Received: from www.zipmail.com.br (smtp.zipmail.com.br [200.221.11.147]) by sucuri.mat.puc-rio.br (8.9.3/8.9.3) with ESMTP id WAA01351 for ; Thu, 24 Apr 2003 22:54:18 -0300 From: yurigomes@zipmail.com.br Received: from [200.253.233.61] by www.zipmail.com.br with HTTP; Thu, 24 Apr 2003 22:46:43 -0300 Message-ID: <3EA5430200008723@www.zipmail.com.br> Date: Thu, 24 Apr 2003 22:46:43 -0300 In-Reply-To: <200304182258.h3IMwuu06634@Gauss.impa.br> To: obm-l@mat.puc-rio.br MIME-Version: 1.0 Content-Type: text/plain; charset="iso-8859-1" Content-Transfer-Encoding: 8bit X-MIME-Autoconverted: from quoted-printable to 8bit by sucuri.mat.puc-rio.br id WAA01352 Sender: owner-obm-l@sucuri.mat.puc-rio.br Precedence: bulk Reply-To: obm-l@mat.puc-rio.br Oi, Alguém poderia resolver a questão abaixo: Seja f: R^n -> R uma função convexa. Sabemos que o gradiente gradf(a) existe para todo ponto a pertencente a R^n e também que existe L>0 tq, para todos x1, x2 pertencentes a R^n, tem-se |gradf(x1)-gradf(x2)| <= L.|x1-x2| Prove que (|gradf(x1)-gradf(x2)|)^2 <= L. , para todos x1, x2 pertencentes a R^n PS: é o produto escalar de a e b. Abraços, Yuri []'s, Yuri ICQ: 64992515 ------------------------------------------ Use o melhor sistema de busca da Internet Radar UOL - http://www.radaruol.com.br ========================================================================= Instruções para entrar na lista, sair da lista e usar a lista em http://www.mat.puc-rio.br/~nicolau/olimp/obm-l.html ========================================================================= From owner-obm-l@sucuri.mat.puc-rio.br Thu Apr 24 23:31:47 2003 Return-Path: Received: (from majordom@localhost) by sucuri.mat.puc-rio.br (8.9.3/8.9.3) id XAA02266 for obm-l-MTTP; Thu, 24 Apr 2003 23:30:22 -0300 Received: from puma.unisys.com.br (smtp.unisys.com.br [200.220.64.7]) by sucuri.mat.puc-rio.br (8.9.3/8.9.3) with ESMTP id XAA02262 for ; Thu, 24 Apr 2003 23:30:19 -0300 Received: from jf (riohiper01p94.uninet.com.br [200.220.2.94]) by puma.unisys.com.br (8.12.9/8.12.3) with SMTP id h3P2TkEh008920 for ; Thu, 24 Apr 2003 23:29:47 -0300 (EST) X-Spam-Filter: check_local@puma.unisys.com.br by digitalanswers.org Message-ID: <00dc01c30adb$4815a700$5e02dcc8@jf> From: "Jose Francisco Guimaraes Costa" To: "obm-l" Subject: [obm-l] data a ser reverenciada Date: Fri, 25 Apr 2003 00:22:48 -0300 MIME-Version: 1.0 Content-Type: multipart/alternative; boundary="----=_NextPart_000_00C3_01C30AC0.CCF65480" X-Priority: 3 X-MSMail-Priority: Normal X-Mailer: Microsoft Outlook Express 6.00.2800.1106 X-MimeOLE: Produced By Microsoft MimeOLE V6.00.2800.1106 Sender: owner-obm-l@sucuri.mat.puc-rio.br Precedence: bulk Reply-To: obm-l@mat.puc-rio.br This is a multi-part message in MIME format. ------=_NextPart_000_00C3_01C30AC0.CCF65480 Content-Type: text/plain; charset="iso-8859-1" Content-Transfer-Encoding: quoted-printable Esta mensagem n=E3o =E9 sobre matem=E1tica. =C9 sobre uma descoberta que deve ser reverenciada por todos, em = especial aqueles envolvidos com Ci=EAncia, classe =E0 qual pertencem os = participantes deste forum. Hoje - 25 de abril de 2003 - comemora-se o cinquenten=E1rio da = publica=E7=E3o, na revista Nature, do artigo "Uma Estrutura para o = =C1cido Nucleico Deoxyribose", de James Watson e Francis Crick, uma obra = que rivaliza em import=E2ncia para a ci=EAncia com a Teoria da = Relatividade de Einstein, A Origem das Esp=E9cies de Darwin, Principia = de Newton e Sobre a Revolu=E7=E3o das Esferas Celestes de Copernicus. Enquanto estes s=E3o livros ou conjuntos de papers, o artigo sobre a = estrutura do DNA ocupa apenas uma p=E1gina, e =E9 iniciado = simploriamente dizendo "We wish to suggest a structure for the DNA. This = structure has novel features which are of considerable biological = interest". Watson e Crick, juntamente com Maurice Watkins, receberam o Premio Nobel = de Medicina de 1962 pelo trabalho sobre DNA. JF ------=_NextPart_000_00C3_01C30AC0.CCF65480 Content-Type: text/html; charset="iso-8859-1" Content-Transfer-Encoding: quoted-printable
Esta mensagem n=E3o =E9 sobre = matem=E1tica.
 
=C9 sobre uma descoberta que deve = ser=20 reverenciada por todos, em especial aqueles envolvidos com Ci=EAncia, = classe =E0=20 qual pertencem os participantes deste forum.
 
Hoje - 25 de abril de 2003 - = comemora-se o=20 cinquenten=E1rio da publica=E7=E3o, na revista Nature, do artigo "Uma = Estrutura para o=20 =C1cido Nucleico Deoxyribose", de James Watson e Francis Crick, uma obra = que=20 rivaliza em import=E2ncia para a ci=EAncia com a Teoria da Relatividade = de=20 Einstein,  A Origem das Esp=E9cies de Darwin, Principia de = Newton e=20 Sobre a Revolu=E7=E3o das Esferas Celestes de = Copernicus.
 
Enquanto estes s=E3o livros ou = conjuntos de=20 papers, o artigo sobre a estrutura do DNA ocupa apenas uma = p=E1gina, e =E9=20 iniciado simploriamente dizendo "We wish to suggest a structure for = the DNA. This structure has novel features which are of = considerable=20 biological interest".
 
Watson e Crick, juntamente com Maurice = Watkins,=20 receberam o Premio Nobel de Medicina de 1962 pelo trabalho=20 sobre DNA.
 
JF
 
------=_NextPart_000_00C3_01C30AC0.CCF65480-- ========================================================================= Instruções para entrar na lista, sair da lista e usar a lista em http://www.mat.puc-rio.br/~nicolau/olimp/obm-l.html ========================================================================= From owner-obm-l@sucuri.mat.puc-rio.br Fri Apr 25 00:05:43 2003 Return-Path: Received: (from majordom@localhost) by sucuri.mat.puc-rio.br (8.9.3/8.9.3) id AAA03306 for obm-l-MTTP; Fri, 25 Apr 2003 00:04:21 -0300 Received: from web14808.mail.yahoo.com (web14808.mail.yahoo.com [216.136.224.224]) by sucuri.mat.puc-rio.br (8.9.3/8.9.3) with SMTP id AAA03302 for ; Fri, 25 Apr 2003 00:04:16 -0300 Message-ID: <20030425030345.70815.qmail@web14808.mail.yahoo.com> Received: from [200.190.17.27] by web14808.mail.yahoo.com via HTTP; Fri, 25 Apr 2003 00:03:45 ART Date: Fri, 25 Apr 2003 00:03:45 -0300 (ART) From: =?iso-8859-1?q?Helder=20Oliveira=20de=20Castro?= Subject: [obm-l] !HELP! To: obm-l@mat.puc-rio.br MIME-Version: 1.0 Content-Type: multipart/alternative; boundary="0-1073930362-1051239825=:69578" Content-Transfer-Encoding: 8bit Sender: owner-obm-l@sucuri.mat.puc-rio.br Precedence: bulk Reply-To: obm-l@mat.puc-rio.br --0-1073930362-1051239825=:69578 Content-Type: text/plain; charset=iso-8859-1 Content-Transfer-Encoding: 8bit Amigos da lista, sou péssimo em problemas de tabuleiro, fico perdido entre as milhares de possibilidades para a resolução destes problemas e não consigo "driblar mirando o gol", ou seja, pensar em maneiras que atendam ao enunciado, que vão direto ao ponto, sem rodeios, talvez porque nunca estudei algo parecido. Quanto mais penso mais acho furos no que pensei e tão logo desisto. Alguém pode me ajudar? Help!Grato, Helder. --------------------------------- Yahoo! Mail O melhor e-mail gratuito da internet: 6MB de espaço, antivírus, acesso POP3, filtro contra spam. --0-1073930362-1051239825=:69578 Content-Type: text/html; charset=iso-8859-1 Content-Transfer-Encoding: 8bit
Amigos da lista,
                       sou péssimo em problemas de tabuleiro, fico perdido entre as milhares de possibilidades para a resolução destes problemas e não consigo "driblar mirando o gol", ou seja, pensar em maneiras que atendam ao enunciado, que vão direto ao ponto, sem rodeios, talvez porque nunca estudei algo parecido. Quanto mais penso mais acho furos no que pensei e tão logo desisto. Alguém pode me ajudar? Help!
Grato,
        Helder.
 



Yahoo! Mail
O melhor e-mail gratuito da internet: 6MB de espaço, antivírus, acesso POP3, filtro contra spam. --0-1073930362-1051239825=:69578-- ========================================================================= Instruções para entrar na lista, sair da lista e usar a lista em http://www.mat.puc-rio.br/~nicolau/olimp/obm-l.html ========================================================================= From owner-obm-l@sucuri.mat.puc-rio.br Fri Apr 25 00:08:14 2003 Return-Path: Received: (from majordom@localhost) by sucuri.mat.puc-rio.br (8.9.3/8.9.3) id AAA03364 for obm-l-MTTP; Fri, 25 Apr 2003 00:06:54 -0300 Received: from mail.gmx.net (mail.gmx.net [213.165.65.60]) by sucuri.mat.puc-rio.br (8.9.3/8.9.3) with SMTP id AAA03357 for ; Fri, 25 Apr 2003 00:06:49 -0300 Received: (qmail 10316 invoked by uid 65534); 25 Apr 2003 03:06:16 -0000 Received: from unknown (EHLO localhost) (200.217.15.193) by mail.gmx.net (mp010-rz3) with SMTP; 25 Apr 2003 05:06:16 +0200 Date: Thu, 24 Apr 2003 15:04:38 -0300 From: Igor GomeZZ X-Mailer: The Bat! (v1.61) Organization: -- X-Priority: 3 (Normal) Message-ID: <1163623370.20030424150438@gmx.net> To: Luis Lopes Subject: Re[4]: [obm-l] 4 coisinhas In-Reply-To: <007801c30a6a$05d59380$5400a8c0@ensrbr> References: <20030421151630.66488.qmail@web13007.mail.yahoo.com> <027d01c3090b$a009e880$5400a8c0@ensrbr> <9821226982.20030422195430@gmx.net> <007801c30a6a$05d59380$5400a8c0@ensrbr> MIME-Version: 1.0 Content-Type: text/plain; charset=ISO-8859-1 Content-Transfer-Encoding: 8bit Sender: owner-obm-l@sucuri.mat.puc-rio.br Precedence: bulk Reply-To: obm-l@mat.puc-rio.br Em 24/4/2003, 11:01, Luis (llopes@ensrbr.com.br) disse: > Oi Igor, Aê Luis! > Falou e disse! Legal o método, não? > Não tem nome, faz parte da teoria do cálculo > de diferenças. Eu curti... Logo vou ler sobre esse método na biblioteca (meu curso começa em Maio por causa das greves nas federais), e arrumar outras coleções boas de livros. Ateh mais! Fui ####### Igor GomeZZ ######## UIN: 29249895 Vitória, Espírito Santo, Brasil Criação: 24/4/2003 (14:39) #################################### Pare para pensar: Uma comissão consiste de uma reunião de pessoas importantes que, sozinhas, não podem fazer nada, mas que, juntas, decidem que nada pode ser feito. (Fred Allen) #################################### ========================================================================= Instruções para entrar na lista, sair da lista e usar a lista em http://www.mat.puc-rio.br/~nicolau/olimp/obm-l.html ========================================================================= From owner-obm-l@sucuri.mat.puc-rio.br Fri Apr 25 00:08:20 2003 Return-Path: Received: (from majordom@localhost) by sucuri.mat.puc-rio.br (8.9.3/8.9.3) id AAA03381 for obm-l-MTTP; Fri, 25 Apr 2003 00:07:01 -0300 Received: from mail.gmx.net (mail.gmx.net [213.165.65.60]) by sucuri.mat.puc-rio.br (8.9.3/8.9.3) with SMTP id AAA03359 for ; Fri, 25 Apr 2003 00:06:51 -0300 Received: (qmail 10380 invoked by uid 65534); 25 Apr 2003 03:06:19 -0000 Received: from unknown (EHLO localhost) (200.217.15.193) by mail.gmx.net (mp010-rz3) with SMTP; 25 Apr 2003 05:06:19 +0200 Date: Thu, 24 Apr 2003 15:04:54 -0300 From: Igor GomeZZ X-Mailer: The Bat! (v1.61) Organization: -- X-Priority: 3 (Normal) Message-ID: <813639333.20030424150454@gmx.net> To: Antonio Neto Subject: Re[4]: [obm-l] 4 coisinhas In-Reply-To: References: MIME-Version: 1.0 Content-Type: text/plain; charset=ISO-8859-1 Content-Transfer-Encoding: 8bit Sender: owner-obm-l@sucuri.mat.puc-rio.br Precedence: bulk Reply-To: obm-l@mat.puc-rio.br Em 24/4/2003, 10:09, Antonio Neto (osneto@hotmail.com) disse: > Olah, povo da PA de ordem superior. Posso sugerir que em vez de calcular de > S(1) a S(5) calculemos de S(0) a S(4)? Se o aluno deve mesmo resolver o > sistema na prova, sem Maple, que resolva um sistema mais simples. No caso em > particular, acho que d S(-2) a S(2) seria mais em conta, mas nao fiz o > calculo. Abracos, olavo. Perfeito, S(0) jah tira uma incógnita do caminho... Foi falta de atenção mesmo :-) Mas no segundo caso, usando S(-2) à S(2), qual o significado de S(-2) da sequência (1, 8, 27, 64...)? Seria um extensão dela para antes do termo 1: (-1, 0, 1, 8, 27, 64...)? S(2): Soma dos dois primeiros; S(1): Soma do primeiro (ou o primeiro termo); S(-2): ? Fui! ####### Igor GomeZZ ######## UIN: 29249895 Vitória, Espírito Santo, Brasil Criação: 24/4/2003 (14:32) #################################### Pare para pensar: Quando a lei eh a fome, o direito eh o saque! (MST) #################################### ========================================================================= Instruções para entrar na lista, sair da lista e usar a lista em http://www.mat.puc-rio.br/~nicolau/olimp/obm-l.html ========================================================================= From owner-obm-l@sucuri.mat.puc-rio.br Fri Apr 25 00:08:26 2003 Return-Path: Received: (from majordom@localhost) by sucuri.mat.puc-rio.br (8.9.3/8.9.3) id AAA03396 for obm-l-MTTP; Fri, 25 Apr 2003 00:07:04 -0300 Received: from mail.gmx.net (mail.gmx.net [213.165.65.60]) by sucuri.mat.puc-rio.br (8.9.3/8.9.3) with SMTP id AAA03363 for ; Fri, 25 Apr 2003 00:06:53 -0300 Received: (qmail 10457 invoked by uid 65534); 25 Apr 2003 03:06:21 -0000 Received: from unknown (EHLO localhost) (200.217.15.193) by mail.gmx.net (mp010-rz3) with SMTP; 25 Apr 2003 05:06:21 +0200 Date: Thu, 24 Apr 2003 15:16:40 -0300 From: Igor GomeZZ X-Mailer: The Bat! (v1.61) Organization: -- X-Priority: 3 (Normal) Message-ID: <794345278.20030424151640@gmx.net> To: "peterdirichlet1985@zipmail.com.br" Subject: Re: Re[2]: [obm-l] 4 coisinhas In-Reply-To: <3E9EC1890000EBD2@www.zipmail.com.br> References: <3E9EC1890000EBD2@www.zipmail.com.br> MIME-Version: 1.0 Content-Type: text/plain; charset=ISO-8859-1 Content-Transfer-Encoding: 8bit Sender: owner-obm-l@sucuri.mat.puc-rio.br Precedence: bulk Reply-To: obm-l@mat.puc-rio.br Em 24/4/2003, 13:06, peterdirichlet1985 (peterdirichlet1985@zipmail.com.br) disse: > Polamordedeos!!!!!!Ces querem evitar o inevitavel!!!!!Isso tudo e so conta,nada > mais.Seja como for nao ha o que fazer. Eh verdade... Conta pura. Nenhum livro de ensino médio (tirando os da SBM) trata de progressões de ordens maiores que 1, ou seja, não eh inevitável. E o método de binomiais reduz muito a quantidade de cálculos... E o "inevitável" eh relativo tb. Vc jah deve ter visto trocentas vezes esse método binomial, mas pra mim foi uma bela surpresa > Talvez tabela de diferenças seja mais util...Mas nao tenha medo de contas.A > nao ser,como disse o Caminha,que seja mais facil simplificar que complicar:) Com certeza :-) Fui! ####### Igor GomeZZ ######## UIN: 29249895 Vitória, Espírito Santo, Brasil Criação: 24/4/2003 (15:05) #################################### Pare para pensar: Uma sociedade sem religião é como um navio sem bússola, difícil de ser controlado (Napoleão Bonaparte) #################################### ========================================================================= Instruções para entrar na lista, sair da lista e usar a lista em http://www.mat.puc-rio.br/~nicolau/olimp/obm-l.html ========================================================================= From owner-obm-l@sucuri.mat.puc-rio.br Fri Apr 25 00:23:42 2003 Return-Path: Received: (from majordom@localhost) by sucuri.mat.puc-rio.br (8.9.3/8.9.3) id AAA04464 for obm-l-MTTP; Fri, 25 Apr 2003 00:21:59 -0300 Received: from mail.vetor.com.br (wool.vetor.com.br [200.160.244.7]) by sucuri.mat.puc-rio.br (8.9.3/8.9.3) with ESMTP id AAA04449 for ; Fri, 25 Apr 2003 00:21:52 -0300 Received: from rodrigo (dl-ct-ip83.wb.com.br [200.216.73.83] (may be forged)) by mail.vetor.com.br (8.12.5) with SMTP id h3P3LCgT010924 for ; Fri, 25 Apr 2003 00:21:12 -0300 Message-ID: <001901c30ad9$2cc07fe0$5349d8c8@rodrigo> From: "Rodrigo Villard Milet" To: Subject: Re: [obm-l] Problema antigo sempre da historia... Date: Fri, 25 Apr 2003 00:17:15 -0300 MIME-Version: 1.0 Content-Type: multipart/alternative; boundary="----=_NextPart_000_0016_01C30AC0.065F18C0" X-Priority: 3 X-MSMail-Priority: Normal X-Mailer: Microsoft Outlook Express 4.72.3110.5 X-MimeOLE: Produced By Microsoft MimeOLE V4.72.3110.3 X-Virus-Scanned: by amavis at mail.vetor.com.br Sender: owner-obm-l@sucuri.mat.puc-rio.br Precedence: bulk Reply-To: obm-l@mat.puc-rio.br This is a multi-part message in MIME format. ------=_NextPart_000_0016_01C30AC0.065F18C0 Content-Type: text/plain; charset="iso-8859-1" Content-Transfer-Encoding: quoted-printable Bem, acho que voc=EA precisa rever essas suas infinitas solu=E7=F5es... = eu errei uma conta no meio, na verdade a resposta =E9 30 graus... segue = abaixo o texto corrigido. Abra=E7os,=20 Villard -----Mensagem original----- De: Johann Peter Gustav Lejeune Dirichlet = Para: obm-l@mat.puc-rio.br Data: Quinta-feira, 24 de Abril de 2003 16:23 Assunto: Re: [obm-l] Problema antigo sempre da historia... Valeu!!!!Eu ja tinha uma com trigonometria so que bem mais dificil.Com = essa ja da 6!!!!!Depois passo todas.=20 Villard wrote:=20 Pelo visto voce quer trigonometria... ent=E3o vai (sem figura, =E9 = claro)=20 Suponha sem perdas AB=3D1 (logo BC=3D1, pois o triangulo ABC =E9 = is=F3sceles). Seja ang(ABD)=3Du.=20 Lei dos senos em ACD : AD*sen(u)=3Dsen20 Lei dos senos em ABD : AD*sen(u+40)=3DAC*sen30. Como AC=3D2*cos50, ent=E3o dividindo uma equa=E7=E3o pela outra = temos que sen(u+40)*sen20 =3D sen(u)*cos50 =3D sen(u)*sen40 =3D = sen(u)*2*sen20*cos20, logo = sen(u+40)=3D2sen(u)cos20=3Dsen(u+20)+sen(u-20), ou seja sen(u+40) - = sen(u-20) =3D sen(u+20) Transformando em produto... 2*sen30*cos(u+10)=3Dsen(u+20), ent=E3o = sen(80-u)=3Dsen(u+20). Como (80-u)+(u+20)=3D100, ent=E3o 80-u e u+20 = s=E3o c=F4ngruos ou seja (u+20)-(80-u) =3D2u-60=3D 360*k (Aqui j=E1 =E9 = de se esperar que u=3D30 seja a =FAnica solu=E7=E3o). Agora, =E9 f=E1cil ver que 0, = "edsonabe@terra.com.br" Assunto: [obm-l] Problema antigo sempre da historia... Data: 24/04/03 16:55 Oi gente!!!!!!!Tenho um problema de geometria que ja discuti na = lista : "Considere o quadrilatero ABCD,tal que = angDBC=3D60,angACB=3D50,angABD=3D20,angACD=3D30.Calcule todos os angulos do quadrilatero." Ja vi uma solu=E7ao cearense bastante magica(segundo o = autor!!!!!)nesta lista.Eu tenho de 4 a 5 solu=E7oes para isso.Mas sera que tem mais?Conto = com voces para isso. TEA WITH ME THAT I BOOK YOUR FACE ------------------------------------------ Use o melhor sistema de busca da Internet Radar UOL - http://www.radaruol.com.br =3D=3D=3D=3D=3D=3D=3D! = =3D=3D=3D=3D=3D=3D=3D=3D=3D=3D=3D=3D=3D=3D=3D=3D=3D=3D=3D=3D=3D=3D=3D=3D=3D= =3D=3D=3D=3D=3D=3D=3D=3D=3D=3D=3D=3D=3D=3D=3D=3D=3D=3D=3D=3D=3D=3D=3D=3D=3D= =3D=3D=3D=3D=3D=3D=3D=3D=3D=3D=3D=3D=3D=3D=3D=3D Instru=E7=F5es para entrar na lista, sair da lista e usar a lista = em http://www.mat.puc-rio.br/~nicolau/olimp/obm-l.html = =3D=3D=3D=3D=3D=3D=3D=3D=3D=3D=3D=3D=3D=3D=3D=3D=3D=3D=3D=3D=3D=3D=3D=3D=3D= =3D=3D=3D=3D=3D=3D=3D=3D=3D=3D=3D=3D=3D=3D=3D=3D=3D=3D=3D=3D=3D=3D=3D=3D=3D= =3D=3D=3D=3D=3D=3D=3D=3D=3D=3D=3D=3D=3D=3D=3D=3D=3D=3D=3D=3D=3D=3D=3D = =3D=3D=3D=3D=3D=3D=3D=3D=3D=3D=3D=3D=3D=3D=3D=3D=3D=3D=3D=3D=3D=3D=3D=3D=3D= =3D=3D=3D=3D=3D=3D=3D=3D=3D=3D=3D=3D=3D=3D=3D=3D=3D=3D=3D=3D=3D=3D=3D=3D=3D= =3D=3D=3D=3D=3D=3D=3D=3D=3D=3D=3D=3D=3D=3D=3D=3D=3D=3D=3D=3D=3D=3D=3D = Instru=E7=F5es para entrar na lista, sair da lista e usar a lista em = http://www.mat.puc-rio.br/~nicolau/olimp/obm-l.html = =3D=3D=3D=3D=3D=3D=3D=3D=3D=3D=3D=3D=3D=3D=3D=3D=3D=3D=3D=3D=3D=3D=3D=3D=3D= =3D=3D=3D=3D=3D=3D=3D=3D=3D=3D=3D=3D=3D=3D=3D=3D=3D=3D=3D=3D=3D=3D=3D=3D=3D= =3D=3D=3D=3D=3D=3D=3D=3D=3D=3D=3D=3D=3D=3D=3D=3D=3D=3D=3D=3D=3D=3D=3D -------------------------------------------------------------------------= ----- Yahoo! Mail=20 O melhor e-mail gratuito da internet: 6MB de espa=E7o, antiv=EDrus, = acesso POP3, filtro contra spam. ------=_NextPart_000_0016_01C30AC0.065F18C0 Content-Type: text/html; charset="iso-8859-1" Content-Transfer-Encoding: quoted-printable
Bem, acho que voc=EA precisa rever essas suas infinitas = solu=E7=F5es... eu=20 errei uma conta no meio, na verdade a resposta =E9 30 graus... segue = abaixo o=20 texto corrigido.
Abra=E7os,
 Villard
-----Mensagem = original-----
De:=20 Johann Peter Gustav Lejeune Dirichlet <peterdirichlet2002@yahoo.= com.br>
Para:=20 obm-l@mat.puc-rio.br = <obm-l@mat.puc-rio.br>
D= ata:=20 Quinta-feira, 24 de Abril de 2003 16:23
Assunto: Re: = [obm-l]=20 Problema antigo sempre da historia...

Valeu!!!!Eu ja tinha uma com trigonometria so que bem mais = dificil.Com essa=20 ja da 6!!!!!Depois passo todas.=20

 Villard <villard@vetor.com.br> wrote:=20

Pelo visto voce quer trigonometria... ent=E3o vai (sem figura, = =E9 claro)=20

Suponha sem perdas AB=3D1 (logo BC=3D1, pois o triangulo ABC =E9 = is=F3sceles).=20 Seja ang(ABD)=3Du.

Lei dos senos em ACD : AD*sen(u)=3Dsen20

Lei dos senos em ABD : AD*sen(u+40)=3DAC*sen30.

Como AC=3D2*cos50, ent=E3o dividindo uma equa=E7=E3o pela outra = temos que=20 sen(u+40)*sen20 =3D sen(u)*cos50 =3D sen(u)*sen40 =3D = sen(u)*2*sen20*cos20, logo=20 sen(u+40)=3D2sen(u)cos20=3Dsen(u+20)+sen(u-20), ou seja sen(u+40) - = sen(u-20) =3D=20 sen(u+20)

Transformando em produto... 2*sen30*cos(u+10)=3Dsen(u+20), = ent=E3o=20 sen(80-u)=3Dsen(u+20). Como (80-u)+(u+20)=3D100, ent=E3o 80-u e u+20 = s=E3o c=F4ngruos=20 ou seja (u+20)-(80-u) =3D2u-60=3D 360*k (Aqui j=E1 =E9 de se esperar = que u=3D30 seja a=20 =FAnica solu=E7=E3o).

Agora, =E9 f=E1cil ver que 0<u<110 (pra soma dos = =E2ngulos do=20 tri=E2ngulo n=E3o passar de 180), logo -30/180 < k < 80/180 e = como k =E9=20 inteiro temos k=3D0, ent=E3o u =3D 30.

Abra=E7os,

Villard

--------- Mensagem Original = --------
De:=20 peterdirichlet1985@zipmail.com.br
Para: "OBM-L@mat.puc-rio.br"=20 <OBM-L@mat.puc-rio.br>, "edsonabe@terra.com.br"=20 <edsonabe@terra.com.br>
Assunto: [obm-l] Problema antigo = sempre=20 da historia...
Data: 24/04/03 16:55

Oi gente!!!!!!!Tenho um problema de geometria que ja = discuti na=20 lista :
"Considere o quadrilatero ABCD,tal que=20 angDBC=3D60,angACB=3D50,angABD=3D20,angACD=3D30.Calcule
todos = os angulos do=20 quadrilatero."
Ja vi uma solu=E7ao cearense bastante = magica(segundo o=20 autor!!!!!)nesta lista.Eu
tenho de 4 a 5 solu=E7oes para = isso.Mas sera=20 que tem mais?Conto com voces
para isso.

TEA WITH ME THAT = I BOOK=20 YOUR = FACE


------------------------------------------
Use o=20 melhor sistema de busca da Internet
Radar UOL - http://www.radaruol.com.br



=3D=3D=3D=3D= =3D=3D=3D!=20 = =3D=3D=3D=3D=3D=3D=3D=3D=3D=3D=3D=3D=3D=3D=3D=3D=3D=3D=3D=3D=3D=3D=3D=3D=3D= =3D=3D=3D=3D=3D=3D=3D=3D=3D=3D=3D=3D=3D=3D=3D=3D=3D=3D=3D=3D=3D=3D=3D=3D=3D= =3D=3D=3D=3D=3D=3D=3D=3D=3D=3D=3D=3D=3D=3D=3D=3D
Instru=E7=F5es=20 para entrar na lista, sair da lista e usar a lista em
http://www.mat.puc-rio.br/~nicolau/olimp/obm-l.html=3D=3D=3D=3D=3D=3D=3D=3D=3D=3D=3D=3D=3D=3D=3D=3D=3D=3D=3D=3D=3D=3D=3D=3D= =3D=3D=3D=3D=3D=3D=3D=3D=3D=3D=3D=3D=3D=3D=3D=3D=3D=3D=3D=3D=3D=3D=3D=3D=3D= =3D=3D=3D=3D=3D=3D=3D=3D=3D=3D=3D=3D=3D=3D=3D=3D=3D=3D=3D=3D=3D=3D=3D=3D<= BR>
=3D=3D=3D=3D=3D=3D=3D=3D=3D=3D=3D=3D=3D=3D= =3D=3D=3D=3D=3D=3D=3D=3D=3D=3D=3D=3D=3D=3D=3D=3D=3D=3D=3D=3D=3D=3D=3D=3D=3D= =3D=3D=3D=3D=3D=3D=3D=3D=3D=3D=3D=3D=3D=3D=3D=3D=3D=3D=3D=3D=3D=3D=3D=3D=3D= =3D=3D=3D=3D=3D=3D=3D=3D=3D=20 Instru=E7=F5es para entrar na lista, sair da lista e usar a lista em = http://www.mat.puc-rio.br/~nicolau/olimp/obm-l.html=20 = =3D=3D=3D=3D=3D=3D=3D=3D=3D=3D=3D=3D=3D=3D=3D=3D=3D=3D=3D=3D=3D=3D=3D=3D=3D= =3D=3D=3D=3D=3D=3D=3D=3D=3D=3D=3D=3D=3D=3D=3D=3D=3D=3D=3D=3D=3D=3D=3D=3D=3D= =3D=3D=3D=3D=3D=3D=3D=3D=3D=3D=3D=3D=3D=3D=3D=3D=3D=3D=3D=3D=3D=3D=3D



Yahoo! Mail
O = melhor e-mail=20 gratuito da internet: 6MB de espa=E7o, antiv=EDrus, acesso POP3, = filtro contra=20 spam. ------=_NextPart_000_0016_01C30AC0.065F18C0-- ========================================================================= Instruções para entrar na lista, sair da lista e usar a lista em http://www.mat.puc-rio.br/~nicolau/olimp/obm-l.html ========================================================================= From owner-obm-l@sucuri.mat.puc-rio.br Fri Apr 25 07:03:37 2003 Return-Path: Received: (from majordom@localhost) by sucuri.mat.puc-rio.br (8.9.3/8.9.3) id HAA10918 for obm-l-MTTP; Fri, 25 Apr 2003 07:01:59 -0300 Received: from hotmail.com (bay3-f27.bay3.hotmail.com [65.54.169.27]) by sucuri.mat.puc-rio.br (8.9.3/8.9.3) with ESMTP id HAA10914 for ; Fri, 25 Apr 2003 07:01:55 -0300 Received: from mail pickup service by hotmail.com with Microsoft SMTPSVC; Fri, 25 Apr 2003 03:01:24 -0700 Received: from 200.147.132.149 by by3fd.bay3.hotmail.msn.com with HTTP; Fri, 25 Apr 2003 10:01:23 GMT X-Originating-IP: [200.147.132.149] X-Originating-Email: [rhilbert1990@msn.com] From: "Oblomov Insistenko" To: obm-l@mat.puc-rio.br Subject: [obm-l] =?iso-8859-1?B?RXNzYSDpIHZlbGhhLi4ubWFzLCBu428gc2VpIQ==?= Date: Fri, 25 Apr 2003 07:01:23 -0300 Mime-Version: 1.0 Content-Type: text/plain; charset=iso-8859-1; format=flowed Message-ID: X-OriginalArrivalTime: 25 Apr 2003 10:01:24.0195 (UTC) FILETIME=[A11DF330:01C30B11] Sender: owner-obm-l@sucuri.mat.puc-rio.br Precedence: bulk Reply-To: obm-l@mat.puc-rio.br Caros colegas, deve existir a resposta em algum livro mas até agora não encontrei, então se algúem puder me ajudar gostaria de saber a demonstração para os seguintes fatos: 1) O mmc de dois ou mais números naturais é o produto dos fatores primos comuns e não comuns, tomados com o maior expoente. 2)O mdc de dois ou mais números naturais é o produto dos fatores primos comuns tomados com o menor expoente. Agradeço qualquer ajuda. (-_-)? _________________________________________________________________ MSN Messenger: converse com os seus amigos online. http://messenger.msn.com.br ========================================================================= Instruções para entrar na lista, sair da lista e usar a lista em http://www.mat.puc-rio.br/~nicolau/olimp/obm-l.html ========================================================================= From owner-obm-l@sucuri.mat.puc-rio.br Fri Apr 25 08:05:55 2003 Return-Path: Received: (from majordom@localhost) by sucuri.mat.puc-rio.br (8.9.3/8.9.3) id IAA11993 for obm-l-MTTP; Fri, 25 Apr 2003 08:04:21 -0300 Received: from artemis.opendf.com.br (artemis.opengate.com.br [200.181.71.14]) by sucuri.mat.puc-rio.br (8.9.3/8.9.3) with ESMTP id IAA11982 for ; Fri, 25 Apr 2003 08:04:13 -0300 Received: from localhost (localhost [127.0.0.1]) by artemis.opendf.com.br (Postfix) with ESMTP id 56B2B2BEC4 for ; Fri, 25 Apr 2003 08:04:33 -0300 (BRT) Received: from artemis.opendf.com.br ([127.0.0.1]) by localhost (artemis.opengate.com.br [127.0.0.1:10024]) (amavisd-new) with ESMTP id 10968-06 for ; Fri, 25 Apr 2003 08:04:32 -0300 (BRT) Received: from artur (200-181-089-218.bsace7001.dsl.brasiltelecom.net.br [200.181.89.218]) by artemis.opendf.com.br (Postfix) with ESMTP id 72E4F2BEC2 for ; Fri, 25 Apr 2003 08:04:30 -0300 (BRT) From: "Artur Costa Steiner" To: Subject: [obm-l] =?iso-8859-1?Q?RE:_=5Bobm-l=5D_Essa_=E9_velha...mas=2C_n=E3o_sei!?= Date: Fri, 25 Apr 2003 08:06:11 -0700 Message-ID: MIME-Version: 1.0 Content-Type: multipart/mixed; boundary="----=_NextPart_000_0010_01C30B01.8A4A3DE0" X-Priority: 3 (Normal) X-MSMail-Priority: Normal X-Mailer: Microsoft Outlook, Build 10.0.2627 Importance: Normal In-Reply-To: X-MS-TNEF-Correlator: 00000000881CAD9D1A69CE40BD8EA975009E889324032200 X-MimeOLE: Produced By Microsoft MimeOLE V6.00.2800.1106 X-Virus-Scanned: by amavisd-new Sender: owner-obm-l@sucuri.mat.puc-rio.br Precedence: bulk Reply-To: obm-l@mat.puc-rio.br This is a multi-part message in MIME format. ------=_NextPart_000_0010_01C30B01.8A4A3DE0 Content-Type: text/plain; charset="iso-8859-1" Content-Transfer-Encoding: quoted-printable > -----Original Message----- > From: owner-obm-l@sucuri.mat.puc-rio.br [mailto:owner-obm- > l@sucuri.mat.puc-rio.br] On Behalf Of Oblomov Insistenko > Sent: Friday, April 25, 2003 2:01 AM > To: obm-l@mat.puc-rio.br > Subject: [obm-l] Essa =E9 velha...mas, n=E3o sei! >=20 >=20 >=20 > Caros colegas, deve existir a resposta em algum livro mas at=E9 agora = n=E3o > encontrei, ent=E3o se alg=FAem puder me ajudar gostaria de saber a > demonstra=E7=E3o > para os seguintes fatos: >=20 > 1) O mmc de dois ou mais n=FAmeros naturais =E9 o produto dos fatores = primos > comuns e n=E3o comuns, tomados com o maior expoente. >=20 > 2)O mdc de dois ou mais n=FAmeros naturais =E9 o produto dos fatores = primos > comuns tomados com o menor expoente. >=20 > Agrade=E7o qualquer ajuda. Bom dia, Se a e b sao numeros naturais, entao, pelo T. Fundamental da Aritmetica, = a =3D Produto (p_i^m_i) i=3D1,...J e b =3D Produto (q_i^n_i) , i=3D1,...K, = sendo os p_i e os q_i numeros primos e os m_i e os n_i numeros naturais. Os p_i sao distintos 2 a dois, assim como os q_i. Consideremos a interseccao do conjunto dos multiplos de a com os de b e seja C este conjunto. Se c pertence a C, entao c tem necesariamente (se C nao for vazio) que = possuir, em sua fatoracao, todos os numeros primos que comparecem nas fatoracoes = de a e de b. Alem disto, se r eh um dos primos que entram na fatoracao de c, entao o expoente de r tem necessariamente que ser >=3D max{m_i,n_i}. = Pelo T. Fundamental da Aritmetica, se esta condicoes nao fossem satisfeitas, = entao c nao poderia pertencer a C. Temos portanto que todo c em C eh da forma N *Produto (r_i^l_i) 1 , i=3D1,...M onde os r_i sao primos que comparecem = nas fatoracoes de a e b, l_i eh um expoente >=3D max{m_i,n_i} e N eh um = natural em cuja fatoracao nao hah qualquer dos primos que compoem a ou compoem b. = Eh imediato que C eh limitado inferiormente e que, desta forma possui um elemento minimo (C e subconjunto dos naturais, o qual eh bem ordenado). = Da compposicao dos c em C, observamos que este minimo eh obtido quando = N=3D1 e os expoentes de cada r_i sao estabelecidos no menor valor admissivel, ou = seja, em r_i=3D max{m_i,n_i. Com isto, fica demonstrada o teormea para o mmc. Para o mdc o raciocinio eh muito semelhante. Nao o coloco aqui gaora por falta de tempo, estopu indo para o trabalho. Umabraco Artur ------=_NextPart_000_0010_01C30B01.8A4A3DE0 Content-Type: application/ms-tnef; name="winmail.dat" Content-Transfer-Encoding: base64 Content-Disposition: attachment; filename="winmail.dat" eJ8+Ig0PAQaQCAAEAAAAAAABAAEAAQeQBgAIAAAA5AQAAAAAAADoAAEIgAcAGAAAAElQTS5NaWNy b3NvZnQgTWFpbC5Ob3RlADEIAQ2ABAACAAAAAgACAAEGAAcAAQAAAAAAAAEGgAMADgAAANMHBAAZ AAkABgAAAAUACwEBA5AGAGwLAAAuAAAACwACAAEAAAALACMAAAAAAAMAJgAAAAAACwApAAAAAAAD AC4AAAAAAAIBMQABAAAAGAAAAAAAAACIHK2dGmnOQL2OqXUAnoiTpAIiAAMANgAAAAAAHgBwAAEA AAAlAAAAW29ibS1sXSBFc3NhIOkgdmVsaGEuLi5tYXMsIG7jbyBzZWkhAAAAAAIBcQABAAAAFgAA AAHDCzufkf0kpGiomkmuhhgAvtZL0u8AAAIBHQwBAAAAGwAAAFNNVFA6QVJUVVJfU1RFSU5FUkBV U0EuTkVUAAALAAEOAAAAAEAABg4A5ByQRAvDAQIBCg4BAAAAGAAAAAAAAACIHK2dGmnOQL2OqXUA noiTwoAAAAMAFA4AAAAACwAfDgEAAAAeACgOAQAAACoAAAAwMDAwMDAwNQFhcnR1cl9zdGVpbmVy QHVzYS5uZXQBbmV0YWRkcmVzcwAAAB4AKQ4BAAAAKgAAADAwMDAwMDA1AWFydHVyX3N0ZWluZXJA dXNhLm5ldAFuZXRhZGRyZXNzAAAAAgEJEAEAAABYBgAAVAYAALoKAABMWkZ1BYBBiAMACgByY3Bn MTI14jIDQ3RleAVBAQMB908KgAKkA+MCAGNoCsBz8GV0MCAHEwKAD/MAUH8EVghVB7IRxQ5RAwEQ xzL3BgAGwxHFMwRGEMkS2xHT2wjvCfc7GL8OMDURwgxgzmMAUAsJAWQzNhFQC6ZHCuMKhAqAPiAt HnJPPQUQZwuAB0AF0AeQc2EsZ2Uecx32RgNhOiAIb3duBJAtb2JtsC1sQHMbcAhxLgDAKHQucBtw LQUQby4aYgXAWwDAAxB0bzqPIPgd9iGfIqRdIE8DoORCZRDwbGYmMCbBAmChA3BvdiBJAIFzDrD4 bmtvHfYGYAIwINAgkKBpZGF5LBFgcAUQTwMgDjApcAHQMDMp4DrUMDERYE0d9lQjcCDgxyFjIiwo N3ViagWQKOE2WyFTJiBFH3EDMCdloDkgdmVsEPAuL3AdAMBzKXALkC7gM28guREgaSEd9jCvHgVD CsDmbwQgGDFlZy+yAQAvIHYgDsAnwWkFwC6wGMBzLnAy0AGQM9BtNFBsZ6J1NSBsaXYDYCAvof00 UHQu0x+QBbAusC/0HfbvCfAFoAIwGMBpKXAowTAG8zUyLtBmYTURImAEgTXw/TlBajoQCsE2wDTR ByEzgfcwUAGgEyFhHfYBAARgAID3OEA8IC7gNzcrCrEusDLRfxEgNWALgA6wBCA5sCNgczI6MS4x KSYwNfBtY9k7gmRvBAAg4HU18UGx3y/xObAHgDLCHxB0CHBCIu8u0zBAKaAEcHUjYEGBP1Q/NIFE AQdwMtAd9gWgbXX/BjEzwC/1RjQpcCNgAMBEkn9GMUPhIyEFsQ7ANLAowWXSLjEuMilBAWRBS0IP /0MfRC9FP0ZER70J8EjPMXk2QQnAR+BlPUIwQHF1vwdAU+A78jqiUhUdlEJIQXpkBzAsHZQGYDRR M8Bi+zuxMEBuNXBMeziDVxApcOZwLzAwQFQuIIBGYClA31ExAZADIClAEWJ0B4A0IEZjVfA0UT0g UE3VKGBwX2lebVvgQOAg0Gk9MSwvcUpWs1spunFb4W5cIilwXHVLKXD/ESBZcDBAMtFb0UaRMtFd 8f9XN08EYARcEWAEXjFXP1kw/k9ftFcCVdA0EQIwMtEU4P87cUGhWuEEEAdwRiJfg13x/1kwCFAA gQSBPME2IVxgPyHdERFjWsBOIjLxbjqgZNH/TjNGUCNQBSAYUAQgO5EusE9IM2ojVuA7oWVqLrBD /zPQJ9Fox1kwVnFBQFjQACD7N/FWgkNYRWzxDrA1ICEQ322AH4AHIVmiM8AoOTFrwI8fEDBAAhAF wHZheiKw/0DgVCE58DLQJNA0MDiBNSD/JNAusE6DANBYkiNgTkJMkv9gvHECRjEKsW7RbqE2EXJF 31HAaiVBcVbBWTBBMyBVss8n0FihOTFRcWggNXFOQr90CSjBNuB1QnI4O4Jjbeb/MEBRljuCBcBu h28JcQIRII0FwD5bIADAeFx7XBHSLF4xXH1ZMFBY71n//zkxa+FqcoAQDeB2EnAUBBD3cdIiMAQA ZjBwAZBYNmzxv3ASNLAEgTthbSY0QkN/0P5UZ3M0sAAgAHBOEXECcvL/bPE1EWvBeEBaEXBRAMBL oKhOICpbR3Jb4WxcIvsqsF6HTSDggBBgE4nxVvP3dA91H3YoYilwijF4JXun/35PRpGJMHg0TMQD IDURJPD/a5F6KHASEPB4QFPneJ2NAt9RwDUhQdKV1nbxRXhAB3HvVdGHNYhDNaBtg+FfcQuA/4PA IqGI4G9jfaMzcjTSiMT/cUSPo3cxKME14QuATyFbsPtrwXHxYmjbV8hTxHgiO+CvSFELIAnwR+Ep f9BEanO+cDSxWrFOJIf0nrFifgH/cJB5B2vynHV4MSFQNCBfcftT4V9iTlyAYARRlmojWsDfWhGL 1oHSO+AzIGOkIUvx71EWcJAYUTRQZJjAZdEvIf+esUHwa3Jxs4nxflxm0jUg/XeEZlqxPKhaETBA DrCZku+FYT5jQRJUy1Ct5UrRMED/coEisKeAAwCjo0ZQWmAwQv8HgC9BUeIHsHtTGDIFoDRQ/1Pg X/AzQDbShtE/YSNQO3P/boE0sDiBd5EiYGfRX3Gt1bU9EWIHQGgiwB2UVQDAvyLgdfEdlAcQTOEd lH24kB4AQhABAAAAKgAAADxCQVkzLUYyN2FzNEc2cU1xRXJiMDAwMDRmNDFAaG90bWFpbC5jb20+ AAAAAwCSEAEAAAACARQ6AQAAABAAAADoyPOzUxE/RY0NvsIX7y8iAwDeP69vAAADAAlZAQAAAAMA QGUAAAAACwATgAggBgAAAAAAwAAAAAAAAEYAAAAAA4UAAAAAAAADABWACCAGAAAAAADAAAAAAAAA RgAAAAAQhQAAAAAAAAMAG4AIIAYAAAAAAMAAAAAAAABGAAAAAFKFAADjkAEAAwBJgAggBgAAAAAA wAAAAAAAAEYAAAAAAYUAAAAAAABAAEqACCAGAAAAAADAAAAAAAAARgAAAABghQAAAEAjDkMAAAAe AHeACCAGAAAAAADAAAAAAAAARgAAAABUhQAAAQAAAAUAAAAxMC4wAAAAAAsAeIAIIAYAAAAAAMAA AAAAAABGAAAAAAaFAAAAAAAACwB8gAggBgAAAAAAwAAAAAAAAEYAAAAADoUAAAAAAAADAH+ACCAG AAAAAADAAAAAAAAARgAAAAAYhQAAAAAAAAsAlIAIIAYAAAAAAMAAAAAAAABGAAAAAIKFAAABAAAA AgH4DwEAAAAQAAAAiBytnRppzkC9jql1AJ6IkwIB+g8BAAAAEAAAAIgcrZ0aac5AvY6pdQCeiJMC AfsPAQAAAHAAAAAAAAAAOKG7EAXlEBqhuwgAKypWwgAAbXNwc3QuZGxsAAAAAABOSVRB+b+4AQCq ADfZbgAAAEM6XFdJTkRPV1NcQXBwbGljYXRpb24gRGF0YVxNaWNyb3NvZnRcT3V0bG9va1xPdXRs b29rMS5wc3QAAwD+DwUAAAADAA00/TcCAAIBFDQBAAAAEAAAAE5JVEH5v7gBAKoAN9luAAACAX8A AQAAADEAAAAwMDAwMDAwMDg4MUNBRDlEMUE2OUNFNDBCRDhFQTk3NTAwOUU4ODkzMjQwMzIyMDAA AAAAAwAGEJ7q75YDAAcQ4AYAAAMAEBABAAAAAwAREAAAAAAeAAgQAQAAAGUAAAAtLS0tLU9SSUdJ TkFMTUVTU0FHRS0tLS0tRlJPTTpPV05FUi1PQk0tTEBTVUNVUklNQVRQVUMtUklPQlJNQUlMVE86 T1dORVItT0JNLUxAU1VDVVJJTUFUUFVDLVJJT0JST05CAAAAAGpe ------=_NextPart_000_0010_01C30B01.8A4A3DE0-- ========================================================================= Instruções para entrar na lista, sair da lista e usar a lista em http://www.mat.puc-rio.br/~nicolau/olimp/obm-l.html ========================================================================= From owner-obm-l@sucuri.mat.puc-rio.br Fri Apr 25 08:06:48 2003 Return-Path: Received: (from majordom@localhost) by sucuri.mat.puc-rio.br (8.9.3/8.9.3) id IAA12014 for obm-l-MTTP; Fri, 25 Apr 2003 08:05:31 -0300 Received: from artemis.opendf.com.br (artemis.opengate.com.br [200.181.71.14]) by sucuri.mat.puc-rio.br (8.9.3/8.9.3) with ESMTP id IAA12010 for ; Fri, 25 Apr 2003 08:05:25 -0300 Received: from localhost (localhost [127.0.0.1]) by artemis.opendf.com.br (Postfix) with ESMTP id 525E82BEC4 for ; Fri, 25 Apr 2003 08:05:47 -0300 (BRT) Received: from artemis.opendf.com.br ([127.0.0.1]) by localhost (artemis.opengate.com.br [127.0.0.1:10024]) (amavisd-new) with ESMTP id 10968-08 for ; Fri, 25 Apr 2003 08:05:46 -0300 (BRT) Received: from artur (200-181-089-218.bsace7001.dsl.brasiltelecom.net.br [200.181.89.218]) by artemis.opendf.com.br (Postfix) with ESMTP id 8BA3F2BEC2 for ; Fri, 25 Apr 2003 08:05:44 -0300 (BRT) From: "Artur Costa Steiner" To: Subject: [obm-l] =?iso-8859-1?Q?RE:_=5Bobm-l=5D_Essa_=E9_velha...mas=2C_n=E3o_sei!?= Date: Fri, 25 Apr 2003 08:07:25 -0700 Message-ID: MIME-Version: 1.0 Content-Type: multipart/mixed; boundary="----=_NextPart_000_0013_01C30B01.B662B1A0" X-Priority: 3 (Normal) X-MSMail-Priority: Normal X-Mailer: Microsoft Outlook, Build 10.0.2627 Importance: Normal In-Reply-To: X-MS-TNEF-Correlator: 00000000881CAD9D1A69CE40BD8EA975009E889384032200 X-MimeOLE: Produced By Microsoft MimeOLE V6.00.2800.1106 X-Virus-Scanned: by amavisd-new Sender: owner-obm-l@sucuri.mat.puc-rio.br Precedence: bulk Reply-To: obm-l@mat.puc-rio.br This is a multi-part message in MIME format. ------=_NextPart_000_0013_01C30B01.B662B1A0 Content-Type: text/plain; charset="iso-8859-1" Content-Transfer-Encoding: quoted-printable > -----Original Message----- > From: owner-obm-l@sucuri.mat.puc-rio.br [mailto:owner-obm- > l@sucuri.mat.puc-rio.br] On Behalf Of Oblomov Insistenko > Sent: Friday, April 25, 2003 2:01 AM > To: obm-l@mat.puc-rio.br > Subject: [obm-l] Essa =E9 velha...mas, n=E3o sei! >=20 >=20 >=20 > Caros colegas, deve existir a resposta em algum livro mas at=E9 agora = n=E3o > encontrei, ent=E3o se alg=FAem puder me ajudar gostaria de saber a > demonstra=E7=E3o > para os seguintes fatos: >=20 > 1) O mmc de dois ou mais n=FAmeros naturais =E9 o produto dos fatores = primos > comuns e n=E3o comuns, tomados com o maior expoente. >=20 > 2)O mdc de dois ou mais n=FAmeros naturais =E9 o produto dos fatores = primos > comuns tomados com o menor expoente. >=20 > Agrade=E7o qualquer ajuda. Bom dia, Se a e b sao numeros naturais, entao, pelo T. Fundamental da Aritmetica, = a =3D Produto (p_i^m_i) i=3D1,...J e b =3D Produto (q_i^n_i) , i=3D1,...K, = sendo os p_i e os q_i numeros primos e os m_i e os n_i numeros naturais. Os p_i sao distintos 2 a dois, assim como os q_i. Consideremos a interseccao do conjunto dos multiplos de a com os de b e seja C este conjunto. Se c pertence a C, entao c tem necesariamente (se C nao for vazio) que = possuir, em sua fatoracao, todos os numeros primos que comparecem nas fatoracoes = de a e de b. Alem disto, se r eh um dos primos que entram na fatoracao de c, entao o expoente de r tem necessariamente que ser >=3D max{m_i,n_i}. = Pelo T. Fundamental da Aritmetica, se esta condicoes nao fossem satisfeitas, = entao c nao poderia pertencer a C. Temos portanto que todo c em C eh da forma N *Produto (r_i^l_i) 1 , i=3D1,...M onde os r_i sao primos que comparecem = nas fatoracoes de a e b, l_i eh um expoente >=3D max{m_i,n_i} e N eh um = natural em cuja fatoracao nao hah qualquer dos primos que compoem a ou compoem b. = Eh imediato que C eh limitado inferiormente e que, desta forma possui um elemento minimo (C e subconjunto dos naturais, o qual eh bem ordenado). = Da compposicao dos c em C, observamos que este minimo eh obtido quando = N=3D1 e os expoentes de cada r_i sao estabelecidos no menor valor admissivel, ou = seja, em r_i=3D max{m_i,n_i. Com isto, fica demonstrada o teormea para o mmc. Para o mdc o raciocinio eh muito semelhante. Nao o coloco aqui gaora por falta de tempo, estopu indo para o trabalho. Umabraco Artur ------=_NextPart_000_0013_01C30B01.B662B1A0 Content-Type: application/ms-tnef; name="winmail.dat" Content-Transfer-Encoding: base64 Content-Disposition: attachment; filename="winmail.dat" eJ8+IhsPAQaQCAAEAAAAAAABAAEAAQeQBgAIAAAA5AQAAAAAAADoAAEIgAcAGAAAAElQTS5NaWNy b3NvZnQgTWFpbC5Ob3RlADEIAQ2ABAACAAAAAgACAAEGAAcAAQAAAAAAAAEGgAMADgAAANMHBAAZ AAkABwAAAAUADAEBA5AGAGwLAAAuAAAACwACAAEAAAALACMAAAAAAAMAJgAAAAAACwApAAAAAAAD AC4AAAAAAAIBMQABAAAAGAAAAAAAAACIHK2dGmnOQL2OqXUAnoiTRAEiAAMANgAAAAAAHgBwAAEA AAAlAAAAW29ibS1sXSBFc3NhIOkgdmVsaGEuLi5tYXMsIG7jbyBzZWkhAAAAAAIBcQABAAAAFgAA AAHDCzufkf0kpGiomkmuhhgAvtZL0u8AAAIBHQwBAAAAGwAAAFNNVFA6QVJUVVJfU1RFSU5FUkBV U0EuTkVUAAALAAEOAAAAAEAABg4AKuCzRAvDAQIBCg4BAAAAGAAAAAAAAACIHK2dGmnOQL2OqXUA noiTwoAAAAMAFA4AAAAACwAfDgEAAAAeACgOAQAAACoAAAAwMDAwMDAwNQFhcnR1cl9zdGVpbmVy QHVzYS5uZXQBbmV0YWRkcmVzcwAAAB4AKQ4BAAAAKgAAADAwMDAwMDA1AWFydHVyX3N0ZWluZXJA dXNhLm5ldAFuZXRhZGRyZXNzAAAAAgEJEAEAAABYBgAAVAYAALoKAABMWkZ1BYBBiAMACgByY3Bn MTI14jIDQ3RleAVBAQMB908KgAKkA+MCAGNoCsBz8GV0MCAHEwKAD/MAUH8EVghVB7IRxQ5RAwEQ xzL3BgAGwxHFMwRGEMkS2xHT2wjvCfc7GL8OMDURwgxgzmMAUAsJAWQzNhFQC6ZHCuMKhAqAPiAt HnJPPQUQZwuAB0AF0AeQc2EsZ2Uecx32RgNhOiAIb3duBJAtb2JtsC1sQHMbcAhxLgDAKHQucBtw LQUQby4aYgXAWwDAAxB0bzqPIPgd9iGfIqRdIE8DoORCZRDwbGYmMCbBAmChA3BvdiBJAIFzDrD4 bmtvHfYGYAIwINAgkKBpZGF5LBFgcAUQTwMgDjApcAHQMDMp4DrUMDERYE0d9lQjcCDgxyFjIiwo N3ViagWQKOE2WyFTJiBFH3EDMCdloDkgdmVsEPAuL3AdAMBzKXALkC7gM28guREgaSEd9jCvHgVD CsDmbwQgGDFlZy+yAQAvIHYgDsAnwWkFwC6wGMBzLnAy0AGQM9BtNFBsZ6J1NSBsaXYDYCAvof00 UHQu0x+QBbAusC/0HfbvCfAFoAIwGMBpKXAowTAG8zUyLtBmYTURImAEgTXw/TlBajoQCsE2wDTR ByEzgfcwUAGgEyFhHfYBAARgAID3OEA8IC7gNzcrCrEusDLRfxEgNWALgA6wBCA5sCNgczI6MS4x KSYwNfBtY9k7gmRvBAAg4HU18UGx3y/xObAHgDLCHxB0CHBCIu8u0zBAKaAEcHUjYEGBP1Q/NIFE AQdwMtAd9gWgbXX/BjEzwC/1RjQpcCNgAMBEkn9GMUPhIyEFsQ7ANLAowWXSLjEuMilBAWRBS0IP /0MfRC9FP0ZER70J8EjPMXk2QQnAR+BlPUIwQHF1vwdAU+A78jqiUhUdlEJIQXpkBzAsHZQGYDRR M8Bi+zuxMEBuNXBMeziDVxApcOZwLzAwQFQuIIBGYClA31ExAZADIClAEWJ0B4A0IEZjVfA0UT0g UE3VKGBwX2lebVvgQOAg0Gk9MSwvcUpWs1spunFb4W5cIilwXHVLKXD/ESBZcDBAMtFb0UaRMtFd 8f9XN08EYARcEWAEXjFXP1kw/k9ftFcCVdA0EQIwMtEU4P87cUGhWuEEEAdwRiJfg13x/1kwCFAA gQSBPME2IVxgPyHdERFjWsBOIjLxbjqgZNH/TjNGUCNQBSAYUAQgO5EusE9IM2ojVuA7oWVqLrBD /zPQJ9Fox1kwVnFBQFjQACD7N/FWgkNYRWzxDrA1ICEQ322AH4AHIVmiM8AoOTFrwI8fEDBAAhAF wHZheiKw/0DgVCE58DLQJNA0MDiBNSD/JNAusE6DANBYkiNgTkJMkv9gvHECRjEKsW7RbqE2EXJF 31HAaiVBcVbBWTBBMyBVss8n0FihOTFRcWggNXFOQr90CSjBNuB1QnI4O4Jjbeb/MEBRljuCBcBu h28JcQIRII0FwD5bIADAeFx7XBHSLF4xXH1ZMFBY71n//zkxa+FqcoAQDeB2EnAUBBD3cdIiMAQA ZjBwAZBYNmzxv3ASNLAEgTthbSY0QkN/0P5UZ3M0sAAgAHBOEXECcvL/bPE1EWvBeEBaEXBRAMBL oKhOICpbR3Jb4WxcIvsqsF6HTSDggBBgE4nxVvP3dA91H3YoYilwijF4JXun/35PRpGJMHg0TMQD IDURJPD/a5F6KHASEPB4QFPneJ2NAt9RwDUhQdKV1nbxRXhAB3HvVdGHNYhDNaBtg+FfcQuA/4PA IqGI4G9jfaMzcjTSiMT/cUSPo3cxKME14QuATyFbsPtrwXHxYmjbV8hTxHgiO+CvSFELIAnwR+Ep f9BEanO+cDSxWrFOJIf0nrFifgH/cJB5B2vynHV4MSFQNCBfcftT4V9iTlyAYARRlmojWsDfWhGL 1oHSO+AzIGOkIUvx71EWcJAYUTRQZJjAZdEvIf+esUHwa3Jxs4nxflxm0jUg/XeEZlqxPKhaETBA DrCZku+FYT5jQRJUy1Ct5UrRMED/coEisKeAAwCjo0ZQWmAwQv8HgC9BUeIHsHtTGDIFoDRQ/1Pg X/AzQDbShtE/YSNQO3P/boE0sDiBd5EiYGfRX3Gt1bU9EWIHQGgiwB2UVQDAvyLgdfEdlAcQTOEd lH24kB4AQhABAAAAKgAAADxCQVkzLUYyN2FzNEc2cU1xRXJiMDAwMDRmNDFAaG90bWFpbC5jb20+ AAAAAwCSEAEAAAACARQ6AQAAABAAAADoyPOzUxE/RY0NvsIX7y8iAwDeP69vAAADAAlZAQAAAAMA QGUAAAAACwATgAggBgAAAAAAwAAAAAAAAEYAAAAAA4UAAAAAAAADABWACCAGAAAAAADAAAAAAAAA RgAAAAAQhQAAAAAAAAMAG4AIIAYAAAAAAMAAAAAAAABGAAAAAFKFAADjkAEAAwBJgAggBgAAAAAA wAAAAAAAAEYAAAAAAYUAAAAAAABAAEqACCAGAAAAAADAAAAAAAAARgAAAABghQAAAEAjDkMAAAAe AHeACCAGAAAAAADAAAAAAAAARgAAAABUhQAAAQAAAAUAAAAxMC4wAAAAAAsAeIAIIAYAAAAAAMAA AAAAAABGAAAAAAaFAAAAAAAACwB8gAggBgAAAAAAwAAAAAAAAEYAAAAADoUAAAAAAAADAH+ACCAG AAAAAADAAAAAAAAARgAAAAAYhQAAAAAAAAsAlIAIIAYAAAAAAMAAAAAAAABGAAAAAIKFAAABAAAA AgH4DwEAAAAQAAAAiBytnRppzkC9jql1AJ6IkwIB+g8BAAAAEAAAAIgcrZ0aac5AvY6pdQCeiJMC AfsPAQAAAHAAAAAAAAAAOKG7EAXlEBqhuwgAKypWwgAAbXNwc3QuZGxsAAAAAABOSVRB+b+4AQCq ADfZbgAAAEM6XFdJTkRPV1NcQXBwbGljYXRpb24gRGF0YVxNaWNyb3NvZnRcT3V0bG9va1xPdXRs b29rMS5wc3QAAwD+DwUAAAADAA00/TcCAAIBFDQBAAAAEAAAAE5JVEH5v7gBAKoAN9luAAACAX8A AQAAADEAAAAwMDAwMDAwMDg4MUNBRDlEMUE2OUNFNDBCRDhFQTk3NTAwOUU4ODkzODQwMzIyMDAA AAAAAwAGEJ7q75YDAAcQ4AYAAAMAEBABAAAAAwAREAAAAAAeAAgQAQAAAGUAAAAtLS0tLU9SSUdJ TkFMTUVTU0FHRS0tLS0tRlJPTTpPV05FUi1PQk0tTEBTVUNVUklNQVRQVUMtUklPQlJNQUlMVE86 T1dORVItT0JNLUxAU1VDVVJJTUFUUFVDLVJJT0JST05CAAAAADxe ------=_NextPart_000_0013_01C30B01.B662B1A0-- ========================================================================= Instruções para entrar na lista, sair da lista e usar a lista em http://www.mat.puc-rio.br/~nicolau/olimp/obm-l.html ========================================================================= From owner-obm-l@sucuri.mat.puc-rio.br Fri Apr 25 08:23:30 2003 Return-Path: Received: (from majordom@localhost) by sucuri.mat.puc-rio.br (8.9.3/8.9.3) id IAA12848 for obm-l-MTTP; Fri, 25 Apr 2003 08:22:07 -0300 Received: from artemis.opendf.com.br (artemis.opengate.com.br [200.181.71.14]) by sucuri.mat.puc-rio.br (8.9.3/8.9.3) with ESMTP id IAA12840 for ; Fri, 25 Apr 2003 08:22:00 -0300 Received: from localhost (localhost [127.0.0.1]) by artemis.opendf.com.br (Postfix) with ESMTP id 18A802BECB for ; Fri, 25 Apr 2003 08:22:23 -0300 (BRT) Received: from artemis.opendf.com.br ([127.0.0.1]) by localhost (artemis.opengate.com.br [127.0.0.1:10024]) (amavisd-new) with ESMTP id 11359-04 for ; Fri, 25 Apr 2003 08:22:22 -0300 (BRT) Received: from artur (200-181-089-218.bsace7001.dsl.brasiltelecom.net.br [200.181.89.218]) by artemis.opendf.com.br (Postfix) with ESMTP id 3C1D12BEC8 for ; Fri, 25 Apr 2003 08:22:21 -0300 (BRT) From: "Artur Costa Steiner" To: Subject: [obm-l] =?iso-8859-1?Q?RE:_=5Bobm-l=5D_Essa_=E9_velha...mas=2C_n=E3o_sei!?= Date: Fri, 25 Apr 2003 08:24:02 -0700 Message-ID: MIME-Version: 1.0 Content-Type: text/plain; charset="iso-8859-1" Content-Transfer-Encoding: 7bit X-Priority: 3 (Normal) X-MSMail-Priority: Normal X-Mailer: Microsoft Outlook, Build 10.0.2627 Importance: Normal In-Reply-To: X-MimeOLE: Produced By Microsoft MimeOLE V6.00.2800.1106 X-Virus-Scanned: by amavisd-new Sender: owner-obm-l@sucuri.mat.puc-rio.br Precedence: bulk Reply-To: obm-l@mat.puc-rio.br Ah, um detalhe. A demonstracao que sdei na outra mensagem, para dois numeros, eh imediatamente estendida, por inducao, para qualquer n numeros naturias. Artur ========================================================================= Instruções para entrar na lista, sair da lista e usar a lista em http://www.mat.puc-rio.br/~nicolau/olimp/obm-l.html ========================================================================= From owner-obm-l@sucuri.mat.puc-rio.br Fri Apr 25 09:53:26 2003 Return-Path: Received: (from majordom@localhost) by sucuri.mat.puc-rio.br (8.9.3/8.9.3) id JAA16214 for obm-l-MTTP; Fri, 25 Apr 2003 09:51:06 -0300 Received: from spinoza.ime.usp.br (spinoza.ime.usp.br [143.107.45.30]) by sucuri.mat.puc-rio.br (8.9.3/8.9.3) with SMTP id JAA16210 for ; Fri, 25 Apr 2003 09:51:00 -0300 Received: (qmail 11780 invoked from network); 25 Apr 2003 12:50:29 -0000 Received: from coliseu.linux.ime.usp.br (192.168.240.9) by spinoza.linux.ime.usp.br with SMTP; 25 Apr 2003 12:50:29 -0000 Received: (qmail 18141 invoked from network); 25 Apr 2003 12:50:30 -0000 Received: from araguaia.linux.ime.usp.br (192.168.240.202) by coliseu.linux.ime.usp.br with QMQP; 25 Apr 2003 12:50:30 -0000 Date: Fri, 25 Apr 2003 09:50:30 -0300 From: guifujiwara@yahoo.com.br To: obm-l@mat.puc-rio.br Subject: [obm-l] Re: your mail Message-ID: <20030425125029.GA22937@linux.ime.usp.br> References: <200304182258.h3IMwuu06634@Gauss.impa.br> <3EA5430200008723@www.zipmail.com.br> Mime-Version: 1.0 Content-Type: text/plain; charset=iso-8859-1 Content-Disposition: inline Content-Transfer-Encoding: 8bit In-Reply-To: <3EA5430200008723@www.zipmail.com.br> User-Agent: Mutt/1.3.28i Sender: owner-obm-l@sucuri.mat.puc-rio.br Precedence: bulk Reply-To: obm-l@mat.puc-rio.br Eu acho que isso ´´´e Cau: (|gradf(x1)-gradf(x2)|÷)^2 <=|gradf(x1)-gradf(x2)|*|x1-x2|*L <= L., sendo a primeira desigualdade ´ a aplicacao da hip´otesee a segunda e Cauchy: |a|*|b|<=. Issao. yurigomes@zipmail.com.br writes: > Oi, > Alguém poderia resolver a questão abaixo: > Seja f: R^n -> R uma função convexa. Sabemos que o gradiente gradf(a) existe > para todo ponto a pertencente a R^n e também que existe L>0 tq, para todos > x1, x2 pertencentes a R^n, tem-se > |gradf(x1)-gradf(x2)| <= L.|x1-x2| > Prove que > (|gradf(x1)-gradf(x2)|)^2 <= L. , para todos > x1, x2 pertencentes a R^n > PS: é o produto escalar de a e b. > > Abraços, > Yuri > > []'s, Yuri > ICQ: 64992515 > > > ------------------------------------------ > Use o melhor sistema de busca da Internet > Radar UOL - http://www.radaruol.com.br > > > > ========================================================================= > Instruções para entrar na lista, sair da lista e usar a lista em > http://www.mat.puc-rio.br/~nicolau/olimp/obm-l.html > ========================================================================= Guilherme Issao Camarinha Fujiwara ========================================================================= Instruções para entrar na lista, sair da lista e usar a lista em http://www.mat.puc-rio.br/~nicolau/olimp/obm-l.html ========================================================================= From owner-obm-l@sucuri.mat.puc-rio.br Fri Apr 25 10:31:54 2003 Return-Path: Received: (from majordom@localhost) by sucuri.mat.puc-rio.br (8.9.3/8.9.3) id KAA17182 for obm-l-MTTP; Fri, 25 Apr 2003 10:30:02 -0300 Received: from hotmail.com (f16.law8.hotmail.com [216.33.241.16]) by sucuri.mat.puc-rio.br (8.9.3/8.9.3) with ESMTP id KAA17168 for ; Fri, 25 Apr 2003 10:29:58 -0300 Received: from mail pickup service by hotmail.com with Microsoft SMTPSVC; Fri, 25 Apr 2003 06:29:25 -0700 Received: from 200.222.105.178 by lw8fd.law8.hotmail.msn.com with HTTP; Fri, 25 Apr 2003 13:29:25 GMT X-Originating-IP: [200.222.105.178] X-Originating-Email: [osneto@hotmail.com] From: "Antonio Neto" To: obm-l@mat.puc-rio.br Subject: [obm-l] Falha nossa Date: Fri, 25 Apr 2003 13:29:25 +0000 Mime-Version: 1.0 Content-Type: text/plain; format=flowed Message-ID: X-OriginalArrivalTime: 25 Apr 2003 13:29:25.0462 (UTC) FILETIME=[B0882760:01C30B2E] Sender: owner-obm-l@sucuri.mat.puc-rio.br Precedence: bulk Reply-To: obm-l@mat.puc-rio.br Falei besteira, Igor. Nao reparei que era a soma, e troquei pela determinacao do termo geral. O que eu disse aplica-se ao termo geral, mas nao aa soma. Para tirar a dúvida, fui aos arquivos da lista para pegar os valores de a, b, c, d e e. Escrevi o polinomio e calculei S(-1)= 1 e S(-2)= 0, o que nao faz sentido, pelo menos para mim. Desculpem a falha, abracos, olavo. >Em 24/4/2003, 10:09, Antonio Neto (osneto@hotmail.com) disse: > > > Olah, povo da PA de ordem superior. Posso sugerir que em vez de calcular >de > > S(1) a S(5) calculemos de S(0) a S(4)? Se o aluno deve mesmo resolver o > > sistema na prova, sem Maple, que resolva um sistema mais simples. No >caso em > > particular, acho que d S(-2) a S(2) seria mais em conta, mas nao fiz o > > calculo. Abracos, olavo. > >Perfeito, S(0) jah tira uma incógnita do caminho... Foi falta de atenção >mesmo :-) > >Mas no segundo caso, usando S(-2) à S(2), qual o significado de S(-2) da >sequência (1, 8, 27, 64...)? Seria um extensão dela para antes do termo 1: >(-1, 0, 1, 8, 27, 64...)? > >S(2): Soma dos dois primeiros; >S(1): Soma do primeiro (ou o primeiro termo); >S(-2): ? _________________________________________________________________ The new MSN 8: advanced junk mail protection and 2 months FREE* http://join.msn.com/?page=features/junkmail ========================================================================= Instruções para entrar na lista, sair da lista e usar a lista em http://www.mat.puc-rio.br/~nicolau/olimp/obm-l.html ========================================================================= From owner-obm-l@sucuri.mat.puc-rio.br Fri Apr 25 11:35:22 2003 Return-Path: Received: (from majordom@localhost) by sucuri.mat.puc-rio.br (8.9.3/8.9.3) id LAA18525 for obm-l-MTTP; Fri, 25 Apr 2003 11:33:00 -0300 Received: from itaqui.terra.com.br (itaqui.terra.com.br [200.176.3.19]) by sucuri.mat.puc-rio.br (8.9.3/8.9.3) with ESMTP id LAA18517 for ; Fri, 25 Apr 2003 11:32:54 -0300 Received: from itaim.terra.com.br (itaim.terra.com.br [200.176.3.76]) by itaqui.terra.com.br (Postfix) with ESMTP id 9CF8F3BC4FE for ; Fri, 25 Apr 2003 11:32:23 -0300 (BRT) Received: from nt (RJ231083.user.veloxzone.com.br [200.165.231.83]) (authenticated user ensr) by itaim.terra.com.br (Postfix) with ESMTP id 3F0F42E0078 for ; Fri, 25 Apr 2003 11:32:23 -0300 (BRT) Message-ID: <016f01c30b37$59171e60$5400a8c0@ensrbr> From: "Luis Lopes" To: References: <001601c30a80$211a9df0$7d07fea9@gauss> <001901c30aa8$66d93400$aecea5c8@epq.ime.eb.br> Subject: Re: [obm-l] FW: Problema dificil Date: Fri, 25 Apr 2003 11:31:23 -0300 MIME-Version: 1.0 Content-Type: text/plain; charset="iso-8859-1" Content-Transfer-Encoding: 8bit X-Priority: 3 X-MSMail-Priority: Normal X-Mailer: Microsoft Outlook Express 5.50.4807.1700 X-MimeOLE: Produced By Microsoft MimeOLE V5.50.4807.1700 Sender: owner-obm-l@sucuri.mat.puc-rio.br Precedence: bulk Reply-To: obm-l@mat.puc-rio.br -----Mensagem Original----- De: "Marcio" Para: Enviada em: quinta-feira, 24 de abril de 2003 18:28 Assunto: Re: [obm-l] FW: Problema dificil > Eh verdade, o Claudio ja havia me alertado sobre isso.. Estou pensando > numa maneira de completar a solucao.. De fato, o mesmo raciocinio pode ser > usado para mostrar que os 9 ultimos sao iguais (iguais a 1 se voce quiser).. > Ainda estou tentando completar... > []'s > Marcio > > > ----- Original Message ----- > From: "Domingos Jr." > To: > Sent: Thursday, April 24, 2003 1:39 PM > Subject: Re: [obm-l] FW: Problema dificil > > > > Olá, Cláudio > > > > não esquenta não, eu só mandei a mensagem pq pode acontecer de você não > ter > > conseguido pegar o arquivo ou simplesmente não viu a mensagem postada > > anteriormente... > > > > a propósito, ontem mesmo eu tinha chegado num resultado parcial dado pelo > > Márcio: os 9 primeiros termos da seq. são todos iguais (creio que, > > analogamente, os 9 últimos tb são iguais), porém, daí a concluir que todos > > os elementos são iguais parece ser um pouco precipitado, acho que precisa > de > > alguma sacada pra chegar lá, mas ainda não vi a luz! > > > > uma pergunta: o que vc faz (curso universitário/mestrado, outros...)? > > > > [ ]'s > > ========================================================================= > Instruções para entrar na lista, sair da lista e usar a lista em > http://www.mat.puc-rio.br/~nicolau/olimp/obm-l.html > ========================================================================= > > ========================================================================= Instruções para entrar na lista, sair da lista e usar a lista em http://www.mat.puc-rio.br/~nicolau/olimp/obm-l.html ========================================================================= From owner-obm-l@sucuri.mat.puc-rio.br Fri Apr 25 11:42:47 2003 Return-Path: Received: (from majordom@localhost) by sucuri.mat.puc-rio.br (8.9.3/8.9.3) id LAA18686 for obm-l-MTTP; Fri, 25 Apr 2003 11:41:29 -0300 Received: from ivoti.terra.com.br (ivoti.terra.com.br [200.176.3.20]) by sucuri.mat.puc-rio.br (8.9.3/8.9.3) with ESMTP id LAA18682 for ; Fri, 25 Apr 2003 11:41:25 -0300 Received: from canela.terra.com.br (canela.terra.com.br [200.176.3.79]) by ivoti.terra.com.br (Postfix) with ESMTP id A2CB94088FC for ; Fri, 25 Apr 2003 11:40:53 -0300 (BRT) Received: from nt (RJ231083.user.veloxzone.com.br [200.165.231.83]) (authenticated user ensr) by canela.terra.com.br (Postfix) with ESMTP id DD39A2240EB for ; Fri, 25 Apr 2003 11:40:52 -0300 (BRT) Message-ID: <018001c30b38$8920ad00$5400a8c0@ensrbr> From: "Luis Lopes" To: References: <001601c30a80$211a9df0$7d07fea9@gauss> <001901c30aa8$66d93400$aecea5c8@epq.ime.eb.br> Subject: [obm-l] serie do Marcio Date: Fri, 25 Apr 2003 11:39:53 -0300 MIME-Version: 1.0 Content-Type: text/plain; charset="iso-8859-1" Content-Transfer-Encoding: 8bit X-Priority: 3 X-MSMail-Priority: Normal X-Mailer: Microsoft Outlook Express 5.50.4807.1700 X-MimeOLE: Produced By Microsoft MimeOLE V5.50.4807.1700 Sender: owner-obm-l@sucuri.mat.puc-rio.br Precedence: bulk Reply-To: obm-l@mat.puc-rio.br Sauda,c~oes, Acabo de mandar uma msg boba por bater no botão errado. Não sei se estou falando com o mesmo Marcio. >Quando exatamente eu posso trocar a ordem da >integral com o somatorio? >Por exemplo, considere a serie cujo n-o termo eh >s_n = 1/(4n+1) + 1/(4n+3) - 1/(2n+2), cujos termos >são todos positivos. >Ela converge, por comparação com a série a/n^2. > >Para calcular Somatorio (0 a infinito) s_n, eu pensei >em calcular: > Somatorio(0 a infinito)_Integral (0 a 1) >[x^4n + x^(4n+2) - x^(2n+1)] > Trocando a ordem, ficamos com algumas PG's e: > Integral (0 a 1) [1/(1-x^4) + x^2 / (1-x^4) - x/(1-x^2)] = > Integral (0 a 1) [1/(1+x)] = ln2 > >Mas eu vi que essa soma vale, na verdade, 1.5 ln2 >(inclusive me provaram isso, e parece estar certo).. > >Por outro lado, em outros problemas esse método >funciona bem.. Por exemplo, >para calcular Somatorio ( 1/[(3n+1)*(3n+2)*(3n+3)] ) >a resposta parece dar correta.. > > Abracos, > Marcio Tentei e não consegui mostrar que dá 1.5 ln2. Como faz? []'s Luís ========================================================================= Instruções para entrar na lista, sair da lista e usar a lista em http://www.mat.puc-rio.br/~nicolau/olimp/obm-l.html ========================================================================= From owner-obm-l@sucuri.mat.puc-rio.br Fri Apr 25 12:14:32 2003 Return-Path: Received: (from majordom@localhost) by sucuri.mat.puc-rio.br (8.9.3/8.9.3) id MAA19667 for obm-l-MTTP; Fri, 25 Apr 2003 12:12:56 -0300 Received: from itaqui.terra.com.br (itaqui.terra.com.br [200.176.3.19]) by sucuri.mat.puc-rio.br (8.9.3/8.9.3) with ESMTP id MAA19663 for ; Fri, 25 Apr 2003 12:12:50 -0300 Received: from marova.terra.com.br (marova.terra.com.br [200.176.3.39]) by itaqui.terra.com.br (Postfix) with ESMTP id 629943BC2E3 for ; Fri, 25 Apr 2003 12:12:19 -0300 (BRT) Received: from nt (RJ231083.user.veloxzone.com.br [200.165.231.83]) (authenticated user ensr) by marova.terra.com.br (Postfix) with ESMTP id E16393DC06D for ; Fri, 25 Apr 2003 12:12:18 -0300 (BRT) Message-ID: <019301c30b3c$ed3ac1a0$5400a8c0@ensrbr> From: "Luis Lopes" To: References: Subject: Re: [obm-l] Falha nossa Nossa! Date: Fri, 25 Apr 2003 12:11:19 -0300 MIME-Version: 1.0 Content-Type: text/plain; charset="iso-8859-1" Content-Transfer-Encoding: 8bit X-Priority: 3 X-MSMail-Priority: Normal X-Mailer: Microsoft Outlook Express 5.50.4807.1700 X-MimeOLE: Produced By Microsoft MimeOLE V5.50.4807.1700 Sender: owner-obm-l@sucuri.mat.puc-rio.br Precedence: bulk Reply-To: obm-l@mat.puc-rio.br Sauda,c~oes, Nossa mãe, é verdade! Falei besteira também. O que eu e o Olavo quisemos dizer foi o seguinte: Já vimos que 1,3,19,61,141,271,.... é uma PA de ordem 3. Mas ....,1,3,19,61,141,271,.... também é uma PA de ordem 3 ! Transcrevo agora um texto do meu livro de Progressões. "O termo geral a_i é válido também para valores de i tais que i<=0, já que a seqüência {a_i} não "começa" em a_1=1. Consultando a tabela da página 11 (das diferenças), obtemos rapidamente a seqüência \Delta^2 a_i "expandida": {-22,-10, 2,14,26,38,...}. Com estes valores e subindo uma linha na mesma tabela, encontramos outros termos para a seqüência \Delta a_i: {32,10,0,2,16,...}. Com a seqüência \Delta a_i geramos os termos a_0 (a_0=1) e a_{-1} (a_{-1}= - 9), obtendo a seqüência a_i expandida: {-9,1,1,3,19,...}. Com esta seqüência, a determinação dos coeficientes \alpha_i é muito mais fácil. De imediato obtemos \alpha_0=1 pois \alpha_0=a_0. Fazendo i=-1,1,2 e resolvendo o sistema....." Note que a_i = \alpha_3 i^3 + ... + \alpha_0. []'s Luís -----Mensagem Original----- De: "Antonio Neto" Para: Enviada em: sexta-feira, 25 de abril de 2003 10:29 Assunto: [obm-l] Falha nossa > Falei besteira, Igor. Nao reparei que era a soma, e troquei pela > determinacao do termo geral. O que eu disse aplica-se ao termo geral, mas > nao aa soma. Para tirar a dúvida, fui aos arquivos da lista para pegar os > valores de a, b, c, d e e. Escrevi o polinomio e calculei S(-1)= 1 e S(-2)= > 0, o que nao faz sentido, pelo menos para mim. Desculpem a falha, abracos, > olavo. > >Em 24/4/2003, 10:09, Antonio Neto (osneto@hotmail.com) disse: > > > > > Olah, povo da PA de ordem superior. Posso sugerir que em vez de calcular > >de > > > S(1) a S(5) calculemos de S(0) a S(4)? Se o aluno deve mesmo resolver o > > > sistema na prova, sem Maple, que resolva um sistema mais simples. No > >caso em > > > particular, acho que d S(-2) a S(2) seria mais em conta, mas nao fiz o > > > calculo. Abracos, olavo. > > > >Perfeito, S(0) jah tira uma incógnita do caminho... Foi falta de atenção > >mesmo :-) > > > >Mas no segundo caso, usando S(-2) à S(2), qual o significado de S(-2) da > >sequência (1, 8, 27, 64...)? Seria um extensão dela para antes do termo 1: > >(-1, 0, 1, 8, 27, 64...)? > > > >S(2): Soma dos dois primeiros; > >S(1): Soma do primeiro (ou o primeiro termo); > >S(-2): ? > > > _________________________________________________________________ > The new MSN 8: advanced junk mail protection and 2 months FREE* > http://join.msn.com/?page=features/junkmail > > ========================================================================= > Instruções para entrar na lista, sair da lista e usar a lista em > http://www.mat.puc-rio.br/~nicolau/olimp/obm-l.html > ========================================================================= > > ========================================================================= Instruções para entrar na lista, sair da lista e usar a lista em http://www.mat.puc-rio.br/~nicolau/olimp/obm-l.html ========================================================================= From owner-obm-l@sucuri.mat.puc-rio.br Fri Apr 25 13:27:53 2003 Return-Path: Received: (from majordom@localhost) by sucuri.mat.puc-rio.br (8.9.3/8.9.3) id NAA21268 for obm-l-MTTP; Fri, 25 Apr 2003 13:24:21 -0300 Received: from web12907.mail.yahoo.com (web12907.mail.yahoo.com [216.136.174.74]) by sucuri.mat.puc-rio.br (8.9.3/8.9.3) with SMTP id NAA21263 for ; Fri, 25 Apr 2003 13:24:15 -0300 Message-ID: <20030425162342.82096.qmail@web12907.mail.yahoo.com> Received: from [200.206.103.3] by web12907.mail.yahoo.com via HTTP; Fri, 25 Apr 2003 13:23:42 ART Date: Fri, 25 Apr 2003 13:23:42 -0300 (ART) From: =?iso-8859-1?q?Johann=20Peter=20Gustav=20Lejeune=20Dirichlet?= Subject: [obm-l] TEOREMA DE CRISTEA:por favor,Claudio!!!!!! To: obm-l@mat.puc-rio.br MIME-Version: 1.0 Content-Type: multipart/alternative; boundary="0-2020767180-1051287822=:80569" Content-Transfer-Encoding: 8bit Sender: owner-obm-l@sucuri.mat.puc-rio.br Precedence: bulk Reply-To: obm-l@mat.puc-rio.br --0-2020767180-1051287822=:80569 Content-Type: text/plain; charset=iso-8859-1 Content-Transfer-Encoding: 8bit Bem,ja que e assim,sou digamos obrigado a aceitar as exigencias.Mas como e essa historia de comentario pertinente?Eu comento coisas sem nexo a cada minuto,mas nao tao gravemente.Quanto ao polidamente,isto e dificil pois escrever e sempre dificil de entender.Mas tudo bem,e a vida em grupo,digo,lista.Mas pode mandar a demo que meu desespero ja esta no fio do barril.... Tente resolver esse:uma reta passando pelo incentro do triangulo ABC bissecta a area se e so se bissecta o perimetro.E do Excalibur. --------------------------------- Yahoo! Mail O melhor e-mail gratuito da internet: 6MB de espaço, antivírus, acesso POP3, filtro contra spam. --0-2020767180-1051287822=:80569 Content-Type: text/html; charset=iso-8859-1 Content-Transfer-Encoding: 8bit
Bem,ja que e assim,sou digamos obrigado a aceitar as exigencias.Mas como e essa historia de comentario pertinente?Eu comento coisas sem nexo a cada minuto,mas nao tao gravemente.Quanto ao polidamente,isto e dificil pois escrever e sempre dificil de entender.Mas tudo bem,e a vida em grupo,digo,lista.Mas pode mandar a demo que meu desespero ja esta no fio do barril....
 
Tente resolver esse:uma reta passando pelo incentro do triangulo ABC bissecta a area se e so se bissecta o perimetro.E do Excalibur.



Yahoo! Mail
O melhor e-mail gratuito da internet: 6MB de espaço, antivírus, acesso POP3, filtro contra spam. --0-2020767180-1051287822=:80569-- ========================================================================= Instruções para entrar na lista, sair da lista e usar a lista em http://www.mat.puc-rio.br/~nicolau/olimp/obm-l.html ========================================================================= From owner-obm-l@sucuri.mat.puc-rio.br Fri Apr 25 13:45:49 2003 Return-Path: Received: (from majordom@localhost) by sucuri.mat.puc-rio.br (8.9.3/8.9.3) id NAA21776 for obm-l-MTTP; Fri, 25 Apr 2003 13:42:54 -0300 Received: from web12907.mail.yahoo.com (web12907.mail.yahoo.com [216.136.174.74]) by sucuri.mat.puc-rio.br (8.9.3/8.9.3) with SMTP id NAA21772 for ; Fri, 25 Apr 2003 13:42:48 -0300 Message-ID: <20030425164217.85761.qmail@web12907.mail.yahoo.com> Received: from [200.206.103.3] by web12907.mail.yahoo.com via HTTP; Fri, 25 Apr 2003 13:42:17 ART Date: Fri, 25 Apr 2003 13:42:17 -0300 (ART) From: =?iso-8859-1?q?Johann=20Peter=20Gustav=20Lejeune=20Dirichlet?= Subject: Re: [obm-l] Re: [obm-l] TEOIREMA DE CRISTEA::Geometria,pontos estranhos e problemas legais!!!! To: obm-l@mat.puc-rio.br In-Reply-To: <007401c30a8f$3837fbe0$3300c57d@bovespa.com> MIME-Version: 1.0 Content-Type: multipart/alternative; boundary="0-1722510293-1051288937=:85172" Content-Transfer-Encoding: 8bit Sender: owner-obm-l@sucuri.mat.puc-rio.br Precedence: bulk Reply-To: obm-l@mat.puc-rio.br --0-1722510293-1051288937=:85172 Content-Type: text/plain; charset=iso-8859-1 Content-Transfer-Encoding: 8bit Tudo bem.Mas o que voce quis dizer sobre mensagens relevantes?Tenho que rever os meus conceitos!! O que e afinal um comentario polido e relevante?Tudo bem,geralmente uso palavras destruidoras de animo mas nunca fui bom de convivencia em grupo.Isso descontando outras coisas....Mas enfim eu concordo.Vamos la,cumpre a sua parte.!! Te mais!"!!!!!!!Ass.:Johann Cláudio_(Prática) wrote:Caro Dirichlet: Consegui demonstrar este resultado. Estou disposto a mandar minha solução pra lista sob uma condição: que, de hoje em diante, você só mande mensagens relevantes, com nexo, e caso tenha comentários pra fazer, que eles sejam polidos, pertinentes, e que ajudem as pessoas a entender melhor um problema ou conceito. Eu sei que você é capaz, afinal a demonstração do Último Teorema de Fermat para n = 4 que você mandou pra lista foi uma das melhores que eu já vi. Se você concordar, eu mando a solução imediatamente e "free of charge". Topa? Um abraço, Claudio. ----- Original Message ----- From: To: Sent: Thursday, April 24, 2003 12:59 PM Subject: [obm-l] Re: [obm-l] TEOIREMA DE CRISTEA::Geometria,pontos estranhos e problemas legais!!!! > MAS COMO???!?!??!!:???!Ja estou ha semanas tentando sem sucesso ou expectativa.Nenhum > triangulo da muitas esperanças... > > > -- Mensagem original -- > > >on 22.04.03 21:25, Johann Peter Gustav Lejeune Dirichlet at > >peterdirichlet2002@yahoo.com.br wrote: > > > >Ola gente!!!!Estou tentando resolver problemas de Geometria do Mathematical > >excalibur mas preciso de um teorema.Quem pode demonstrar pra mim: > > > >Considere o triangulo ABC e um ponto T.Duas cevianas CT_c e BT_b se cortam > >erm T e os pontos R_c,R_b e T sao alinhados,com R_c em AB e R_b em AC.Mostre > >que > > > >AT_c*BR_c/T_cB/R_cA + ATb*CR_b//T_bC/R_bA = 1 > > > >Nao sei se e isso mesmo mas vale o risco... > > > > > >Oi, JP: > > > >Supondo que a expressao acima seja: > > > >(AT_c * BR_c) / (BT_c * AR_c) + ( AT_b * CR_b ) / (CT_b * AR_b ) = 1 > > > >eu diria que uma boa aposta seria o teorema de Menelau (aplicado mais de > >uma > >vez). > > > > > >Um abraco, > >Claudio. > > > > > > > > TEA WITH ME THAT I BOOK YOUR FACE > > > ------------------------------------------ > Use o melhor sistema de busca da Internet > Radar UOL - http://www.radaruol.com.br > > > > ========================================================================= > Instruções para entrar na lista, sair da lista e usar a lista em > http://www.mat.puc-rio.br/~nicolau/olimp/obm-l.html > ========================================================================= ========================================================================= Instruções para entrar na lista, sair da lista e usar a lista em http://www.mat.puc-rio.br/~nicolau/olimp/obm-l.html ========================================================================= TRANSIRE SVVM PECTVS MVNDOQVE POTIRI CONGREGATI EX TOTO ORBE MATHEMATICI OB SCRIPTA INSIGNIA TRIBVERE Fields Medal(John Charles Fields) --------------------------------- Yahoo! Mail O melhor e-mail gratuito da internet: 6MB de espaço, antivírus, acesso POP3, filtro contra spam. --0-1722510293-1051288937=:85172 Content-Type: text/html; charset=iso-8859-1 Content-Transfer-Encoding: 8bit

Tudo bem.Mas o que voce quis dizer sobre mensagens relevantes?Tenho que rever os meus conceitos!!

O que e afinal um comentario polido e relevante?Tudo bem,geralmente uso palavras destruidoras de animo mas nunca fui bom de convivencia em grupo.Isso descontando outras coisas....Mas enfim eu concordo.Vamos la,cumpre a sua parte.!!

Te mais!"!!!!!!!Ass.:Johann

 Cláudio_(Prática) <claudio@praticacorretora.com.br> wrote:

Caro Dirichlet:

Consegui demonstrar este resultado. Estou disposto a mandar minha solução
pra lista sob uma condição:
que, de hoje em diante, você só mande mensagens relevantes, com nexo, e caso
tenha comentários pra fazer, que eles sejam polidos, pertinentes, e que
ajudem as pessoas a entender melhor um problema ou conceito.

Eu sei que você é capaz, afinal a demonstração do Último Teorema de Fermat
para n = 4 que você mandou pra lista foi uma das melhores que eu já vi.

Se você concordar, eu mando a solução imediatamente e "free of charge".
Topa?

Um abraço,
Claudio.



----- Original Message -----
From:
To:
Sent: Thursday, April 24, 2003 12:59 PM
Subject: [obm-l] Re: [obm-l] TEOIREMA DE CRISTEA::Geometria,pontos estranhos
e problemas legais!!!!


> MAS COMO???!?!??!!:???!Ja estou ha semanas tentando sem sucesso ou
expectativa.Nenhum
> triangulo da muitas esperanças...
>
>
> -- Mensagem original --
>
> >on 22.04.03 21:25, Johann Peter Gustav Lejeune Dirichlet at
> >peterdirichlet2002@yahoo.com.br wrote:
> >
> >Ola gente!!!!Estou tentando resolver problemas de Geometria do
Mathematical
> >excalibur mas preciso de um teorema.Quem pode demonstrar pra mim:
> >
> >Considere o triangulo ABC e um ponto T.Duas cevianas CT_c e BT_b se
cortam
> >erm T e os pontos R_c,R_b e T sao alinhados,com R_c em AB e R_b em
AC.Mostre
> >que
> >
> >AT_c*BR_c/T_cB/R_cA + ATb*CR_b//T_bC/R_bA = 1
> >
> >Nao sei se e isso mesmo mas vale o risco...
> >
> >
> >Oi, JP:
> >
> >Supondo que a expressao acima seja:
> >
> >(AT_c * BR_c) / (BT_c * AR_c) + ( AT_b * CR_b ) / (CT_b * AR_b ) = 1
> >
> >eu diria que uma boa aposta seria o teorema de Menelau (aplicado mais de
> >uma
> >vez).
> >
> >
> >Um abraco,
> >Claudio.
> >
> >
> >
>
> TEA WITH ME THAT I BOOK YOUR FACE
>
>
> ------------------------------------------
> Use o melhor sistema de busca da Internet
> Radar UOL - http://www.radaruol.com.br
>
>
>
> =========================================================================
> Instruções para entrar na lista, sair da lista e usar a lista em
> http://www.mat.puc-rio.br/~nicolau/olimp/obm-l.html
> =========================================================================

=========================================================================
Instruções para entrar na lista, sair da lista e usar a lista em
http://www.mat.puc-rio.br/~nicolau/olimp/obm-l.html
=========================================================================


TRANSIRE SVVM PECTVS MVNDOQVE POTIRI

CONGREGATI EX TOTO ORBE MATHEMATICI OB SCRIPTA INSIGNIA TRIBVERE

Fields Medal(John Charles Fields)



Yahoo! Mail
O melhor e-mail gratuito da internet: 6MB de espaço, antivírus, acesso POP3, filtro contra spam. --0-1722510293-1051288937=:85172-- ========================================================================= Instruções para entrar na lista, sair da lista e usar a lista em http://www.mat.puc-rio.br/~nicolau/olimp/obm-l.html ========================================================================= From owner-obm-l@sucuri.mat.puc-rio.br Fri Apr 25 14:05:13 2003 Return-Path: Received: (from majordom@localhost) by sucuri.mat.puc-rio.br (8.9.3/8.9.3) id OAA22369 for obm-l-MTTP; Fri, 25 Apr 2003 14:02:22 -0300 Received: from web12903.mail.yahoo.com (web12903.mail.yahoo.com [216.136.174.70]) by sucuri.mat.puc-rio.br (8.9.3/8.9.3) with SMTP id OAA22365 for ; Fri, 25 Apr 2003 14:02:16 -0300 Message-ID: <20030425170142.60483.qmail@web12903.mail.yahoo.com> Received: from [200.206.103.3] by web12903.mail.yahoo.com via HTTP; Fri, 25 Apr 2003 14:01:42 ART Date: Fri, 25 Apr 2003 14:01:42 -0300 (ART) From: =?iso-8859-1?q?Johann=20Peter=20Gustav=20Lejeune=20Dirichlet?= Subject: Re:re: [obm-l] Problema antigo sempre da historia... To: obm-l@mat.puc-rio.br In-Reply-To: <001901c30ad9$2cc07fe0$5349d8c8@rodrigo> MIME-Version: 1.0 Content-Type: multipart/alternative; boundary="0-1522988829-1051290102=:59011" Content-Transfer-Encoding: 8bit Sender: owner-obm-l@sucuri.mat.puc-rio.br Precedence: bulk Reply-To: obm-l@mat.puc-rio.br --0-1522988829-1051290102=:59011 Content-Type: text/plain; charset=iso-8859-1 Content-Transfer-Encoding: 8bit Eis UMA soluçao cearense.Aqui m(XYZ)=medida do angulo XYZ em graus;e se um triangulo ABC e isosceles subentende-se que AB=BC. Seja X um ponto de BC tal que m(XAB)=20.Assim AB=AX(marcando angulos). XAB isosceles e DAB isosceles acarreta DA=AX e como m(DAX)=60,DAX e equilatero.E CXA e isosceles pois m(ACX)=40(angulo externo no CXA).Logo AX=XC e como DX=XA,temos CX=XC e o CXD e isosceles.Logo m(DCX)=70.Com isso da pra calcular tudo. E melhor que esteja munido de uma otima figura. FUI!!!Ass.:Johann Rodrigo Villard Milet wrote:Bem, acho que você precisa rever essas suas infinitas soluções... eu errei uma conta no meio, na verdade a resposta é 30 graus... segue abaixo o texto corrigido.Abraços, Villard-----Mensagem original----- De: Johann Peter Gustav Lejeune Dirichlet Para: obm-l@mat.puc-rio.br Data: Quinta-feira, 24 de Abril de 2003 16:23 Assunto: Re: [obm-l] Problema antigo sempre da historia... Valeu!!!!Eu ja tinha uma com trigonometria so que bem mais dificil.Com essa ja da 6!!!!!Depois passo todas. Villard wrote: Pelo visto voce quer trigonometria... então vai (sem figura, é claro) Suponha sem perdas AB=1 (logo BC=1, pois o triangulo ABC é isósceles). Seja ang(ABD)=u. Lei dos senos em ACD : AD*sen(u)=sen20 Lei dos senos em ABD : AD*sen(u+40)=AC*sen30. Como AC=2*cos50, então dividindo uma equação pela outra temos que sen(u+40)*sen20 = sen(u)*cos50 = sen(u)*sen40 = sen(u)*2*sen20*cos20, logo sen(u+40)=2sen(u)cos20=sen(u+20)+sen(u-20), ou seja sen(u+40) - sen(u-20) = sen(u+20) Transformando em produto... 2*sen30*cos(u+10)=sen(u+20), então sen(80-u)=sen(u+20). Como (80-u)+(u+20)=100, então 80-u e u+20 são côngruos ou seja (u+20)-(80-u) =2u-60= 360*k (Aqui já é de se esperar que u=30 seja a única solução). Agora, é fácil ver que 0, "edsonabe@terra.com.br" Assunto: [obm-l] Problema antigo sempre da historia... Data: 24/04/03 16:55 Oi gente!!!!!!!Tenho um problema de geometria que ja discuti na lista : "Considere o quadrilatero ABCD,tal que angDBC=60,angACB=50,angABD=20,angACD=30.Calcule todos os angulos do quadrilatero." Ja vi uma soluçao cearense bastante magica(segundo o autor!!!!!)nesta lista.Eu tenho de 4 a 5 soluçoes para isso.Mas sera que tem mais?Conto com voces para isso. TEA WITH ME THAT I BOOK YOUR FACE ------------------------------------------ Use o melhor sistema de busca da Internet Radar UOL - http://www.radaruol.com.br =======! ================================================================== Instruções para entrar na lista, sair da lista e usar a lista em http://www.mat.puc-rio.br/~nicolau/olimp/obm-l.html ========================================================================= ========================================================================= Instruções para entrar na lista, sair da lista e usar a lista em http://www.mat.puc-rio.br/~nicolau/olimp/obm-l.html ========================================================================= --------------------------------- Yahoo! Mail O melhor e-mail gratuito da internet: 6MB de espaço, antivírus, acesso POP3, filtro contra spam. TRANSIRE SVVM PECTVS MVNDOQVE POTIRI CONGREGATI EX TOTO ORBE MATHEMATICI OB SCRIPTA INSIGNIA TRIBVERE Fields Medal(John Charles Fields) --------------------------------- Yahoo! Mail O melhor e-mail gratuito da internet: 6MB de espaço, antivírus, acesso POP3, filtro contra spam. --0-1522988829-1051290102=:59011 Content-Type: text/html; charset=iso-8859-1 Content-Transfer-Encoding: 8bit

Eis UMA soluçao cearense.Aqui m(XYZ)=medida do angulo XYZ em graus;e se um triangulo ABC e isosceles subentende-se que AB=BC.

Seja X um ponto de BC tal que m(XAB)=20.Assim AB=AX(marcando angulos).

XAB isosceles e DAB isosceles acarreta DA=AX e como m(DAX)=60,DAX e equilatero.E CXA e isosceles pois m(ACX)=40(angulo externo no CXA).Logo AX=XC e como DX=XA,temos CX=XC e o CXD e isosceles.Logo m(DCX)=70.Com isso da pra calcular tudo.

E melhor que esteja munido de uma otima figura.

FUI!!!Ass.:Johann

 Rodrigo Villard Milet <villard@vetor.com.br> wrote:

Bem, acho que você precisa rever essas suas infinitas soluções... eu errei uma conta no meio, na verdade a resposta é 30 graus... segue abaixo o texto corrigido.
Abraços,
 Villard
-----Mensagem original-----
De: Johann Peter Gustav Lejeune Dirichlet <peterdirichlet2002@yahoo.com.br>
Para: obm-l@mat.puc-rio.br <obm-l@mat.puc-rio.br>
Data: Quinta-feira, 24 de Abril de 2003 16:23
Assunto: Re: [obm-l] Problema antigo sempre da historia...

Valeu!!!!Eu ja tinha uma com trigonometria so que bem mais dificil.Com essa ja da 6!!!!!Depois passo todas.

 Villard <villard@vetor.com.br> wrote:

Pelo visto voce quer trigonometria... então vai (sem figura, é claro)

Suponha sem perdas AB=1 (logo BC=1, pois o triangulo ABC é isósceles). Seja ang(ABD)=u.

Lei dos senos em ACD : AD*sen(u)=sen20

Lei dos senos em ABD : AD*sen(u+40)=AC*sen30.

Como AC=2*cos50, então dividindo uma equação pela outra temos que sen(u+40)*sen20 = sen(u)*cos50 = sen(u)*sen40 = sen(u)*2*sen20*cos20, logo sen(u+40)=2sen(u)cos20=sen(u+20)+sen(u-20), ou seja sen(u+40) - sen(u-20) = sen(u+20)

Transformando em produto... 2*sen30*cos(u+10)=sen(u+20), então sen(80-u)=sen(u+20). Como (80-u)+(u+20)=100, então 80-u e u+20 são côngruos ou seja (u+20)-(80-u) =2u-60= 360*k (Aqui já é de se esperar que u=30 seja a única solução).

Agora, é fácil ver que 0<u<110 (pra soma dos ângulos do triângulo não passar de 180), logo -30/180 < k < 80/180 e como k é inteiro temos k=0, então u = 30.

Abraços,

Villard

--------- Mensagem Original --------
De: peterdirichlet1985@zipmail.com.br
Para: "OBM-L@mat.puc-rio.br" <OBM-L@mat.puc-rio.br>, "edsonabe@terra.com.br" <edsonabe@terra.com.br>
Assunto: [obm-l] Problema antigo sempre da historia...
Data: 24/04/03 16:55

Oi gente!!!!!!!Tenho um problema de geometria que ja discuti na lista :
"Considere o quadrilatero ABCD,tal que angDBC=60,angACB=50,angABD=20,angACD=30.Calcule
todos os angulos do quadrilatero."
Ja vi uma soluçao cearense bastante magica(segundo o autor!!!!!)nesta lista.Eu
tenho de 4 a 5 soluçoes para isso.Mas sera que tem mais?Conto com voces
para isso.

TEA WITH ME THAT I BOOK YOUR FACE


------------------------------------------
Use o melhor sistema de busca da Internet
Radar UOL - http://www.radaruol.com.br



=======! ==================================================================
Instruções para entrar na lista, sair da lista e usar a lista em
http://www.mat.puc-rio.br/~nicolau/olimp/obm-l.html
=========================================================================

========================================================================= Instruções para entrar na lista, sair da lista e usar a lista em http://www.mat.puc-rio.br/~nicolau/olimp/obm-l.html =========================================================================



Yahoo! Mail
O melhor e-mail gratuito da internet: 6MB de espaço, antivírus, acesso POP3, filtro contra spam.


TRANSIRE SVVM PECTVS MVNDOQVE POTIRI

CONGREGATI EX TOTO ORBE MATHEMATICI OB SCRIPTA INSIGNIA TRIBVERE

Fields Medal(John Charles Fields)



Yahoo! Mail
O melhor e-mail gratuito da internet: 6MB de espaço, antivírus, acesso POP3, filtro contra spam. --0-1522988829-1051290102=:59011-- ========================================================================= Instruções para entrar na lista, sair da lista e usar a lista em http://www.mat.puc-rio.br/~nicolau/olimp/obm-l.html ========================================================================= From owner-obm-l@sucuri.mat.puc-rio.br Fri Apr 25 14:45:55 2003 Return-Path: Received: (from majordom@localhost) by sucuri.mat.puc-rio.br (8.9.3/8.9.3) id OAA23671 for obm-l-MTTP; Fri, 25 Apr 2003 14:43:07 -0300 Received: from suter.bol.com.br (suter.bol.com.br [200.221.24.19]) by sucuri.mat.puc-rio.br (8.9.3/8.9.3) with ESMTP id OAA23667 for ; Fri, 25 Apr 2003 14:43:02 -0300 Received: from bol.com.br (200.221.24.136) by suter.bol.com.br (5.1.071) id 3E7672CB00C5CCC4 for obm-l@mat.puc-rio.br; Fri, 25 Apr 2003 14:42:31 -0300 Date: Fri, 25 Apr 2003 14:42:31 -0300 Message-Id: Subject: [obm-l] Problema MIME-Version: 1.0 Content-Type: text/plain;charset="iso-8859-1" From: "hadock" To: obm-l@mat.puc-rio.br X-XaM3-API-Version: 2.4 R3 ( B4 ) X-SenderIP: 200.153.242.163 Content-Transfer-Encoding: 8bit X-MIME-Autoconverted: from quoted-printable to 8bit by sucuri.mat.puc-rio.br id OAA23668 Sender: owner-obm-l@sucuri.mat.puc-rio.br Precedence: bulk Reply-To: obm-l@mat.puc-rio.br Olá pessoal da lista! Esse é meu primeiro e-mail. Para quais n E N, a função f(n)=12n^3 - 5n^2 - 251n + 389 / 6n^2 - 37n +45 assume valores inteiros Eu estou meio perdido em como resolvê-lo. Se alguém puder ajudar __________________________________________________________________________ Seleção de Softwares UOL. 10 softwares escolhidos pelo UOL para você e sua família. http://www.uol.com.br/selecao ========================================================================= Instruções para entrar na lista, sair da lista e usar a lista em http://www.mat.puc-rio.br/~nicolau/olimp/obm-l.html ========================================================================= From owner-obm-l@sucuri.mat.puc-rio.br Fri Apr 25 15:18:23 2003 Return-Path: Received: (from majordom@localhost) by sucuri.mat.puc-rio.br (8.9.3/8.9.3) id PAA24838 for obm-l-MTTP; Fri, 25 Apr 2003 15:16:35 -0300 Received: from cmsrelay03.mx.net (cmsrelay03.mx.net [165.212.11.112]) by sucuri.mat.puc-rio.br (8.9.3/8.9.3) with SMTP id PAA24835 for ; Fri, 25 Apr 2003 15:16:30 -0300 Received: from uadvg137.cms.usa.net (HELO localhost) (165.212.11.137) by cmsoutbound.mx.net with SMTP; 25 Apr 2003 18:15:58 -0000 Received: from cmsweb10.cms.usa.net [165.212.8.26] by uadvg137.cms.usa.net (ASMTP/) via mtad (C8.MAIN.2.05) with ESMTP id 963HDysP50487M37; Fri, 25 Apr 2003 18:15:56 GMT Received: from 200.181.4.100 [200.181.4.100] by cmsweb10.cms.usa.net (USANET web-mailer CM.0402.5.2B); Fri, 25 Apr 2003 18:15:55 -0000 Date: Fri, 25 Apr 2003 15:15:55 -0300 From: Artur Costa Steiner To: Subject: Re: [[obm-l] Re: your mail] X-Mailer: USANET web-mailer (CM.0402.5.2B) Mime-Version: 1.0 Message-ID: <150HDysP40640S10.1051294555@cmsweb10.cms.usa.net> Content-Type: text/plain; charset=ISO-8859-1 Content-Transfer-Encoding: 8bit X-MIME-Autoconverted: from quoted-printable to 8bit by sucuri.mat.puc-rio.br id PAA24836 Sender: owner-obm-l@sucuri.mat.puc-rio.br Precedence: bulk Reply-To: obm-l@mat.puc-rio.br guifujiwara@yahoo.com.br wrote: > Eu acho que isso ´´´e Cau: > (|gradf(x1)-gradf(x2)|÷)^2 <=|gradf(x1)-gradf(x2)|*|x1-x2|*L <= > L., sendo a primeira desigualdade ´ a aplicacao da > hip´otesee a segunda e Cauchy: |a|*|b|<=. O sentido desta desigualdade e ao contrario... O certo eh <= |a| |b| Aplicando-se a desigualdade de Cauchy-Schwarz o que concluimos eh que <= |gradf(x1)-gradf(x2)| |x1 - x2| <= L |x1 - x2|^2. Interessante, mas nao eh a desigualdade pedida. Mais tarde vou tentar analisa-la. Acho interessante observar que algumas hipoteses estao sendo assumidas. O simples fato de f ser convexa em R^n nao garante a existencia do gradiente em todo R^n. Artur > > > yurigomes@zipmail.com.br writes: > > Oi, > > Alguém poderia resolver a questão abaixo: > > Seja f: R^n -> R uma função convexa. Sabemos que o gradiente gradf(a) existe > > para todo ponto a pertencente a R^n e também que existe L>0 tq, para todos > > x1, x2 pertencentes a R^n, tem-se > > |gradf(x1)-gradf(x2)| <= L.|x1-x2| > > Prove que > > (|gradf(x1)-gradf(x2)|)^2 <= L. , para todos > > x1, x2 pertencentes a R^n > > PS: é o produto escalar de a e b. > > > > Abraços, > > Yuri > > > > []'s, Yuri > > ICQ: 64992515 > > > > > > ------------------------------------------ > > Use o melhor sistema de busca da Internet > > Radar UOL - http://www.radaruol.com.br > > > > > > > > ========================================================================= > > Instruções para entrar na lista, sair da lista e usar a lista em > > http://www.mat.puc-rio.br/~nicolau/olimp/obm-l.html > > ========================================================================= > Guilherme Issao Camarinha Fujiwara > ========================================================================= > Instruções para entrar na lista, sair da lista e usar a lista em > http://www.mat.puc-rio.br/~nicolau/olimp/obm-l.html > ========================================================================= ========================================================================= Instruções para entrar na lista, sair da lista e usar a lista em http://www.mat.puc-rio.br/~nicolau/olimp/obm-l.html ========================================================================= From owner-obm-l@sucuri.mat.puc-rio.br Fri Apr 25 15:22:28 2003 Return-Path: Received: (from majordom@localhost) by sucuri.mat.puc-rio.br (8.9.3/8.9.3) id PAA24944 for obm-l-MTTP; Fri, 25 Apr 2003 15:21:02 -0300 Received: from saks.bol.com.br (saks.bol.com.br [200.221.24.16]) by sucuri.mat.puc-rio.br (8.9.3/8.9.3) with ESMTP id PAA24936 for ; Fri, 25 Apr 2003 15:20:44 -0300 Received: from power (200.221.24.192) by saks.bol.com.br (5.1.071) id 3EA835850005CF4C for obm-l@mat.puc-rio.br; Fri, 25 Apr 2003 15:20:07 -0300 Message-ID: <001501c30b57$235f8a30$2101a8c0@power> From: "Guilherme F. Moleiro" To: "OBM" Subject: [obm-l] Problema Date: Fri, 25 Apr 2003 14:40:25 -0300 MIME-Version: 1.0 Content-Type: multipart/alternative; boundary="----=_NextPart_000_00C7_01C30B38.9B72C510" X-Priority: 3 X-MSMail-Priority: Normal X-Mailer: Microsoft Outlook Express 6.00.2800.1158 X-MimeOLE: Produced By Microsoft MimeOLE V6.00.2800.1165 X-Sender-IP: 200.153.242.163 Sender: owner-obm-l@sucuri.mat.puc-rio.br Precedence: bulk Reply-To: obm-l@mat.puc-rio.br This is a multi-part message in MIME format. ------=_NextPart_000_00C7_01C30B38.9B72C510 Content-Type: text/plain; charset="iso-8859-1" Content-Transfer-Encoding: quoted-printable Ol=E1 pessoal da lista! Esse =E9 meu primeiro e-mail para a lista. Para quais n E N, a fun=E7=E3o f(n)=3D12n^3 - 5n^2 - 251n + 389 / 6n^2 - = 37n +45 assume valores inteiros Eu estou meio perdido em como resolv=EA-lo. Se algu=E9m puder ajudar... ------=_NextPart_000_00C7_01C30B38.9B72C510 Content-Type: text/html; charset="iso-8859-1" Content-Transfer-Encoding: quoted-printable
Ol=E1 pessoal da lista! Esse =E9 meu = primeiro e-mail=20 para a lista.
 
Para quais n E N, a fun=E7=E3o = f(n)=3D12n^3 - 5n^2 - 251n + 389 / 6n^2 - 37n +45 assume = valores=20 inteiros
 
Eu estou meio perdido em como = resolv=EA-lo. Se algu=E9m=20 puder ajudar...
------=_NextPart_000_00C7_01C30B38.9B72C510-- ========================================================================= Instruções para entrar na lista, sair da lista e usar a lista em http://www.mat.puc-rio.br/~nicolau/olimp/obm-l.html ========================================================================= From owner-obm-l@sucuri.mat.puc-rio.br Fri Apr 25 15:26:42 2003 Return-Path: Received: (from majordom@localhost) by sucuri.mat.puc-rio.br (8.9.3/8.9.3) id PAA25033 for obm-l-MTTP; Fri, 25 Apr 2003 15:25:23 -0300 Received: from ns3bind.localdomain ([200.230.34.5]) by sucuri.mat.puc-rio.br (8.9.3/8.9.3) with ESMTP id PAA25030 for ; Fri, 25 Apr 2003 15:25:19 -0300 Received: from servico2 ([200.230.34.227]) by ns3bind.localdomain (8.11.6/X.XX.X) with SMTP id h3PIKaD07719 for ; Fri, 25 Apr 2003 15:20:36 -0300 Message-ID: <010401c30b58$22b942a0$3300c57d@bovespa.com> From: "=?Windows-1252?Q?Cl=E1udio_\=28Pr=E1tica\=29?=" To: Subject: [obm-l] Problema do Dirichlet Date: Fri, 25 Apr 2003 15:26:04 -0300 MIME-Version: 1.0 Content-Type: multipart/alternative; boundary="----=_NextPart_000_0101_01C30B3E.FC935DE0" X-Priority: 3 X-MSMail-Priority: Normal X-Mailer: Microsoft Outlook Express 5.50.4920.2300 X-MimeOLE: Produced By Microsoft MimeOLE V5.50.4920.2300 Sender: owner-obm-l@sucuri.mat.puc-rio.br Precedence: bulk Reply-To: obm-l@mat.puc-rio.br This is a multi-part message in MIME format. ------=_NextPart_000_0101_01C30B3E.FC935DE0 Content-Type: text/plain; charset="Windows-1252" Content-Transfer-Encoding: quoted-printable HelpOk, seu Dirichlet: Como voc=EA est=E1 se compromentando publicamente a ter nexo, = relev=E2ncia e educa=E7=E3o (pelo menos nas suas mensagens pra lista), = eu vou cumprir minha parte do trato. Aqui vai: O problema: Considere o triangulo ABC e um ponto T. Duas cevianas CT_c e BT_b se = cortam em T e os pontos R_c,R_b e T sao alinhados,com R_c em AB e R_b em AC. Mostre que: (AT_c * BR_c) / (BT_c * AR_c) + ( AT_b * CR_b ) / (CT_b * AR_b ) =3D 1 ***** Solu=E7=E3o usando vetores: Tomando A como origem, sejam U e V vetores unit=E1rios nas dire=E7=F5es = de AB e AC, respectivamente. Como ABC =E9 n=E3o-degenerado, U e V s=E3o L.I. Assim, existem n=FAmeros reais h, k, a, b, c, d tais que: AB =3D hU, AC =3D kV, AT_c =3D aU, AR_c =3D bU, AT_b =3D cV, AR_b =3D dV Como | U | =3D | V | =3D 1, podemos re-escrever escrever a express=E3o = do enunciado em fun=E7=E3o apenas de h, k, a, b, c, d. Assim: (AT_c * BR_c) / (BT_c * AR_c) + ( AT_b * CR_b ) / (CT_b * AR_b ) =3D =3D (a*(h - b))/((h - a)*b) + (c*(k - d))/((k - c)*d) =3D =3D (ha - ab)/(hb - ab) + (kc - cd)/(kd - cd) =3D=20 =3D NUM / DEN onde: NUM =3D hkad - hacd - kabd + abcd + hkbc - hbcd - kabc + abcd e DEN =3D hkbd - hbcd - kabd + abcd Temos que provar que NUM / DEN =3D 1, ou seja, que NUM =3D DEN. Em outras palavras, basta provar que: NUM - DEN =3D abcd - hacd - kabc + hkad + hkbc - hkbd =3D 0 ***** Vamos agora =E0 geometria: R_c, T, R_b s=E3o colineares =3D=3D> existe um no. real x tal que: AT =3D x*AR_c + (1-x)*AR_b =3D=3D> AT =3D xbU + (1-x)dV (1) B, T, T_b s=E3o colineares =3D=3D> existe um no. real y tal que: AT =3D y*AB + (1-y)*AT_b =3D=3D> AT =3D yhU + (1-y)cV (2) C, T, T_c s=E3o colineares =3D=3D> existe um no. real z tal que: AT =3D z*AC + (1-z)*AT_c =3D=3D> AT =3D zkV + (1-z)aU (3) (1) e (2) =3D=3D> xbU + (1-x)dV =3D yhU + (1-y)cV =3D=3D> (xb - yh)U + ((1-x)d - (1-y)c)V =3D 0 (1) e (3) =3D=3D> xbU + (1-x)dV =3D zkV + (1-z)aU =3D=3D> (xb - (1-z)a)U + ((1-x)d - zk)V =3D 0 Como U e V s=E3o LI, podemos concluir que: xb - yh =3D 0 (1-x)d - (1-y)c =3D 0 xb - (1-z)a =3D 0 (1-x)d - zk =3D 0 Ou seja, rearranjando: bx - hy =3D 0 -dx + cy =3D c - d bx + az =3D a dx + kz =3D d Resolvendo para x, achamos que: x =3D (hc - hd)/(bc - hd) e x =3D (ka - ad)/(kb - ad) Igualando estas express=F5es, multiplicando e simplificando, ca=EDmos = em: abcd - hacd - kabc + hkad + hkbc - hkbd =3D 0. No entanto, esta express=E3o =E9 precisamente igual a NUM - DEN (Lembre-se: NUM - DEN =3D abcd - hacd - kabc + hkad + hkbc - hkbd). Logo, NUM - DEN =3D 0 =3D=3D>=20 NUM =3D DEN =3D=3D> =20 NUM / DEN =3D 1 =3D=3D> =20 acabou!!! ***** Repare que, apesar da =E1lgebra meio bra=E7al no final, a id=E9ia = central da solu=E7=E3o =E9 bastante simples, o que ilustra (espero) o = grande poder do m=E9todo vetorial em geometria. Pense na sua dificuldade = em resolver este problema via Menelaus ou algum outro teorema mais = elegante... Um abra=E7o, Claudio. ------=_NextPart_000_0101_01C30B3E.FC935DE0 Content-Type: text/html; charset="Windows-1252" Content-Transfer-Encoding: quoted-printable Help
Ok, seu Dirichlet:
 
Como voc=EA est=E1 se compromentando publicamente a ter nexo, = relev=E2ncia e=20 educa=E7=E3o (pelo menos nas suas mensagens pra lista), eu vou cumprir = minha parte=20 do trato. Aqui vai:
 
O problema:
 
Considere o triangulo ABC e um ponto T. Duas cevianas CT_c e BT_b = se=20 cortam
em T e os pontos R_c,R_b e T sao alinhados,com R_c em AB e R_b = em=20 AC.
Mostre que:

(AT_c * BR_c) / (BT_c * AR_c)  +  ( = AT_b *=20 CR_b ) / (CT_b * AR_b ) =3D 1

*****

Solu=E7=E3o usando=20 vetores:

Tomando A como origem, sejam U e V vetores unit=E1rios = nas=20 dire=E7=F5es de AB e AC, respectivamente.
Como ABC =E9 = n=E3o-degenerado, U e V s=E3o=20 L.I.

Assim, existem n=FAmeros reais h, k, a, b, c, d tais = que:
AB =3D hU,=20 AC =3D kV, AT_c =3D aU, AR_c =3D bU, AT_b =3D cV, AR_b =3D = dV

Como | U | =3D | V | =3D=20 1, podemos re-escrever escrever a express=E3o do enunciado em = fun=E7=E3o
apenas de=20 h, k, a, b, c, d.
Assim:
(AT_c * BR_c) / (BT_c * AR_c)  = +  (=20 AT_b * CR_b ) / (CT_b * AR_b ) =3D
=3D (a*(h - b))/((h - a)*b) + (c*(k - d))/((k - c)*d) =3D
=3D (ha - ab)/(hb - ab) + (kc - cd)/(kd - cd) =3D
 
=3D NUM / DEN
onde:
NUM =3D hkad - hacd - kabd + abcd + hkbc = - hbcd -=20 kabc + abcd
e
DEN =3D hkbd - hbcd - kabd + abcd

Temos que = provar que=20 NUM / DEN =3D 1, ou seja, que NUM =3D DEN.
Em outras palavras, basta provar que:
NUM - DEN =3D abcd - hacd = - kabc +=20 hkad + hkbc - hkbd =3D 0

*****

Vamos agora =E0 = geometria:

R_c,=20 T, R_b s=E3o colineares =3D=3D>
existe um no. real x tal que: AT = =3D x*AR_c +=20 (1-x)*AR_b =3D=3D>
AT =3D xbU + (1-x)dV  (1)

B, T, T_b = s=E3o=20 colineares =3D=3D>
existe um no. real y tal que: AT =3D y*AB + = (1-y)*AT_b=20 =3D=3D>
AT =3D yhU + (1-y)cV  (2)

C, T, T_c s=E3o = colineares=20 =3D=3D>
existe um no. real z tal que: AT =3D z*AC + (1-z)*AT_c = =3D=3D>
AT =3D=20 zkV + (1-z)aU  (3)

(1) e (2) =3D=3D> xbU + (1-x)dV =3D = yhU + (1-y)cV=20 =3D=3D>
(xb - yh)U + ((1-x)d - (1-y)c)V =3D 0

(1) e (3) = =3D=3D> xbU +=20 (1-x)dV =3D zkV + (1-z)aU =3D=3D>
(xb - (1-z)a)U + ((1-x)d - zk)V = =3D=20 0

Como U e V s=E3o LI, podemos concluir que:
xb - yh =3D = 0
(1-x)d -=20 (1-y)c =3D 0
xb - (1-z)a =3D 0
(1-x)d - zk =3D 0

Ou seja,=20 rearranjando:
bx - hy =3D 0
-dx + cy =3D c - d
bx + az =3D = a
dx + kz =3D=20 d

Resolvendo para x, achamos que:
x =3D (hc - hd)/(bc - = hd)
e
x =3D=20 (ka - ad)/(kb - ad)

Igualando estas express=F5es, multiplicando e = simplificando, ca=EDmos em:
abcd - hacd - kabc + hkad + hkbc - hkbd = =3D=20 0.

No entanto, esta express=E3o =E9 precisamente igual a NUM -=20 DEN
(Lembre-se: NUM - DEN =3D abcd - hacd - kabc + hkad + hkbc -=20 hkbd).

Logo, NUM - DEN =3D 0  =3D=3D>
NUM =3D DEN  =3D=3D> 
NUM / DEN =3D 1  =3D=3D> 
acabou!!!
*****
Repare que, apesar da =E1lgebra meio bra=E7al no final, a = id=E9ia central da=20 solu=E7=E3o =E9 bastante simples, o que ilustra (espero) o grande poder = do m=E9todo=20 vetorial em geometria. Pense na sua dificuldade em resolver este = problema via=20 Menelaus ou algum outro teorema mais elegante...
 
 
Um abra=E7o,
Claudio.
------=_NextPart_000_0101_01C30B3E.FC935DE0-- ========================================================================= Instruções para entrar na lista, sair da lista e usar a lista em http://www.mat.puc-rio.br/~nicolau/olimp/obm-l.html ========================================================================= From owner-obm-l@sucuri.mat.puc-rio.br Fri Apr 25 16:00:06 2003 Return-Path: Received: (from majordom@localhost) by sucuri.mat.puc-rio.br (8.9.3/8.9.3) id PAA26052 for obm-l-MTTP; Fri, 25 Apr 2003 15:56:12 -0300 Received: from ns3bind.localdomain ([200.230.34.5]) by sucuri.mat.puc-rio.br (8.9.3/8.9.3) with ESMTP id PAA26039 for ; Fri, 25 Apr 2003 15:56:03 -0300 Received: from servico2 ([200.230.34.227]) by ns3bind.localdomain (8.11.6/X.XX.X) with SMTP id h3PIpKD09797 for ; Fri, 25 Apr 2003 15:51:20 -0300 Message-ID: <000201c30b5c$6e33ef60$3300c57d@bovespa.com> From: "=?iso-8859-1?Q?Cl=E1udio_\=28Pr=E1tica\=29?=" To: References: <20030425162342.82096.qmail@web12907.mail.yahoo.com> Subject: [obm-l] =?iso-8859-1?Q?Bisse=E7=E3o_de_=C1rea_e_Per=EDmetro?= Date: Fri, 25 Apr 2003 15:51:35 -0300 MIME-Version: 1.0 Content-Type: multipart/alternative; boundary="----=_NextPart_000_0111_01C30B42.8D14BF00" X-Priority: 3 X-MSMail-Priority: Normal X-Mailer: Microsoft Outlook Express 5.50.4920.2300 X-MimeOLE: Produced By Microsoft MimeOLE V5.50.4920.2300 Sender: owner-obm-l@sucuri.mat.puc-rio.br Precedence: bulk Reply-To: obm-l@mat.puc-rio.br This is a multi-part message in MIME format. ------=_NextPart_000_0111_01C30B42.8D14BF00 Content-Type: text/plain; charset="iso-8859-1" Content-Transfer-Encoding: quoted-printable Uma reta passando pelo incentro do triangulo ABC bissecta a area se e so = se bissecta o perimetro. Sejam os pontos X em AB e Y em AC. Suponha que a reta XY contenha o incentro I de ABC. Suponha que o c=EDrculo inscrito tangencie BC, AC e AB nos pontos M, N e = P, respectivamente. Assim, IM =3D IN =3D IP =3D R =3D raio do c=EDrculo inscrito, e cada um = destes segmentos =E9 perpendicular ao lado correspondente. Temos que: [ABC] =3D (1/2)*R*(AB+AC+BC) e [AXY] =3D [AXI] + [AYI] =3D (1/2)*AX*PI + (1/2)*AY*NI =3D = (1/2)*R*(AX+AY) XY bissecta a =E1rea <=3D=3D> [ABC] =3D 2*[AXY] <=3D=3D> (1/2)*R*(AB+AC+BC) =3D 2*(1/2)*R*(AX+AY) <=3D=3D> AB+AC+BC =3D 2*(AX+AY) <=3D=3D> XY bissecta o pr=EDmetro Um abra=E7o, Claudio. P.S.: existe o verbo bissectar? Acho que o correto =E9 bissecionar ou = bisseccionar ou bi-secionar....mas como esta =E9 uma lista de = matem=E1tica.... ----- Original Message -----=20 From: Johann Peter Gustav Lejeune Dirichlet=20 To: obm-l@mat.puc-rio.br=20 Sent: Friday, April 25, 2003 1:23 PM Subject: [obm-l] TEOREMA DE CRISTEA:por favor,Claudio!!!!!! Bem,ja que e assim,sou digamos obrigado a aceitar as exigencias.Mas = como e essa historia de comentario pertinente?Eu comento coisas sem nexo = a cada minuto,mas nao tao gravemente.Quanto ao polidamente,isto e = dificil pois escrever e sempre dificil de entender.Mas tudo bem,e a vida = em grupo,digo,lista.Mas pode mandar a demo que meu desespero ja esta no = fio do barril.... Tente resolver esse:uma reta passando pelo incentro do triangulo ABC = bissecta a area se e so se bissecta o perimetro.E do Excalibur. -------------------------------------------------------------------------= ----- Yahoo! Mail=20 O melhor e-mail gratuito da internet: 6MB de espa=E7o, antiv=EDrus, = acesso POP3, filtro contra spam. ------=_NextPart_000_0111_01C30B42.8D14BF00 Content-Type: text/html; charset="iso-8859-1" Content-Transfer-Encoding: quoted-printable
Uma reta passando pelo incentro do triangulo ABC bissecta a area se = e so se=20 bissecta o perimetro.
 
Sejam os pontos X em AB e Y em = AC.
Suponha que a reta XY contenha o = incentro I de=20 ABC.
Suponha que o c=EDrculo inscrito = tangencie BC, AC e=20 AB nos pontos M, N e P, respectivamente.
Assim, IM =3D IN =3D IP =3D R =3D = raio do c=EDrculo=20 inscrito, e cada um destes segmentos =E9 perpendicular ao lado=20 correspondente.
 
Temos que:
[ABC] =3D = (1/2)*R*(AB+AC+BC)
e
[AXY] =3D [AXI] + [AYI] =3D (1/2)*AX*PI = + (1/2)*AY*NI =3D=20 (1/2)*R*(AX+AY)
 
XY bissecta a =E1rea  = <=3D=3D>
[ABC] =3D 2*[AXY]  = <=3D=3D>
(1/2)*R*(AB+AC+BC) =3D = 2*(1/2)*R*(AX+AY) =20 <=3D=3D>
AB+AC+BC =3D 2*(AX+AY)  = <=3D=3D>
XY bissecta o pr=EDmetro
 
Um abra=E7o,
Claudio.
 
P.S.: existe o verbo bissectar? Acho = que o correto=20 =E9 bissecionar ou bisseccionar ou bi-secionar....mas como esta =E9 uma = lista de=20 matem=E1tica....
 
----- Original Message -----
From:=20 Johann Peter Gustav = Lejeune=20 Dirichlet
Sent: Friday, April 25, 2003 = 1:23=20 PM
Subject: [obm-l] TEOREMA DE = CRISTEA:por=20 favor,Claudio!!!!!!

Bem,ja que e assim,sou digamos obrigado a aceitar as = exigencias.Mas=20 como e essa historia de comentario pertinente?Eu comento coisas sem = nexo a=20 cada minuto,mas nao tao gravemente.Quanto ao polidamente,isto e = dificil pois=20 escrever e sempre dificil de entender.Mas tudo bem,e a vida em=20 grupo,digo,lista.Mas pode mandar a demo que meu desespero ja esta no = fio do=20 barril....
 
Tente resolver esse:uma reta passando pelo incentro do triangulo = ABC=20 bissecta a area se e so se bissecta o perimetro.E do Excalibur.
 
 



Yahoo! Mail
O = melhor e-mail=20 gratuito da internet: 6MB de espa=E7o, antiv=EDrus, acesso POP3, = filtro contra=20 spam. ------=_NextPart_000_0111_01C30B42.8D14BF00-- ========================================================================= Instruções para entrar na lista, sair da lista e usar a lista em http://www.mat.puc-rio.br/~nicolau/olimp/obm-l.html ========================================================================= From owner-obm-l@sucuri.mat.puc-rio.br Fri Apr 25 16:35:30 2003 Return-Path: Received: (from majordom@localhost) by sucuri.mat.puc-rio.br (8.9.3/8.9.3) id QAA27224 for obm-l-MTTP; Fri, 25 Apr 2003 16:32:06 -0300 Received: from sina.bol.com.br (sina.bol.com.br [200.221.24.27]) by sucuri.mat.puc-rio.br (8.9.3/8.9.3) with ESMTP id QAA27221 for ; Fri, 25 Apr 2003 16:31:54 -0300 Received: from bol.com.br (200.221.24.132) by sina.bol.com.br (5.1.071) id 3E7665FB00AF8F30 for obm-l@mat.puc-rio.br; Fri, 25 Apr 2003 16:31:22 -0300 Date: Fri, 25 Apr 2003 16:31:22 -0300 Message-Id: Subject: [obm-l] =?iso-8859-1?q?Problema_de_aproxima=E7=E3o?= MIME-Version: 1.0 Content-Type: text/plain;charset="iso-8859-1" From: "butkov-cam" To: obm-l@mat.puc-rio.br X-XaM3-API-Version: 2.4 R3 ( B4 ) X-SenderIP: 200.152.12.196 Content-Transfer-Encoding: 8bit X-MIME-Autoconverted: from quoted-printable to 8bit by sucuri.mat.puc-rio.br id QAA27222 Sender: owner-obm-l@sucuri.mat.puc-rio.br Precedence: bulk Reply-To: obm-l@mat.puc-rio.br Estou com dúvida no seguinte problema: Seja X o espaço das funções contínuas entre zero e um usando-se a norma 2. Seja f um elemento deste espaço. Queremos determinar um polinômio p(t) de grau menor ou igual a dois que minimiza a integral de zero a um do módulo de f(t) - p(t) de forma que a integral de zero a um de p(t) seja zero. Minha dúvida é se o problema fica inteiramente resolvido se projetarmos a função f(t) no subespaço gerado pelos polinômios que satisfazem a restrição imposta (integral de zero a um de p(t) é zero ). Assim teríamos: ********o subspaço dos p(t) que satisfazem é gerado por [1-3t^2, 2t-3t^2]. Chamemos e_1=1-3t^2 e e_2=2t-3t^2. p_solucao=alfa*e_1+beta*e_2 onde alfa e beta seriam soluçoes do sistema abaixo: 4/5*alfa + 3/10*beta =integral de zero a um de f(t)*e_1 3/10*alfa+2/15*beta = integral de zero a um de f(t)*e_2 Meu raciocínio está correto???? Grata por qualquer ajuda, Camila. __________________________________________________________________________ Seleção de Softwares UOL. 10 softwares escolhidos pelo UOL para você e sua família. http://www.uol.com.br/selecao ========================================================================= Instruções para entrar na lista, sair da lista e usar a lista em http://www.mat.puc-rio.br/~nicolau/olimp/obm-l.html ========================================================================= From owner-obm-l@sucuri.mat.puc-rio.br Fri Apr 25 16:46:14 2003 Return-Path: Received: (from majordom@localhost) by sucuri.mat.puc-rio.br (8.9.3/8.9.3) id QAA27462 for obm-l-MTTP; Fri, 25 Apr 2003 16:43:26 -0300 Received: from sina.bol.com.br (sina.bol.com.br [200.221.24.27]) by sucuri.mat.puc-rio.br (8.9.3/8.9.3) with ESMTP id QAA27456 for ; Fri, 25 Apr 2003 16:43:16 -0300 Received: from bol.com.br (200.221.24.132) by sina.bol.com.br (5.1.071) id 3E7665FB00AF9D3D for obm-l@mat.puc-rio.br; Fri, 25 Apr 2003 16:42:43 -0300 Date: Fri, 25 Apr 2003 16:42:42 -0300 Message-Id: Subject: [obm-l] Desigualdade de Gram MIME-Version: 1.0 Content-Type: text/plain;charset="iso-8859-1" From: "butkov-cam" To: obm-l@mat.puc-rio.br X-XaM3-API-Version: 2.4 R3 ( B4 ) X-SenderIP: 200.152.12.196 Content-Transfer-Encoding: 8bit X-MIME-Autoconverted: from quoted-printable to 8bit by sucuri.mat.puc-rio.br id QAA27457 Sender: owner-obm-l@sucuri.mat.puc-rio.br Precedence: bulk Reply-To: obm-l@mat.puc-rio.br Colegas estou com dificuldades em demonstrar a desigualdade de Gram no caso geral (n qualquer natural) Os casos n = 1 e n = 2 (Cauchy-Schwarz) são triviais. O problema é o seguinte: Dados x_1,x_2, ..., x_n de um espaço de hilbert. O determinante da matriz de Gram é maior ou igual a zero. matriz de gram = G(x_1, ..., X_n) = [] onde 0 Received: (from majordom@localhost) by sucuri.mat.puc-rio.br (8.9.3/8.9.3) id RAA28180 for obm-l-MTTP; Fri, 25 Apr 2003 17:07:18 -0300 Received: from ns3bind.localdomain ([200.230.34.5]) by sucuri.mat.puc-rio.br (8.9.3/8.9.3) with ESMTP id RAA28175 for ; Fri, 25 Apr 2003 17:07:13 -0300 Received: from servico2 ([200.230.34.227]) by ns3bind.localdomain (8.11.6/X.XX.X) with SMTP id h3PK2UD13887 for ; Fri, 25 Apr 2003 17:02:30 -0300 Message-ID: <005e01c30b66$5fc19e00$3300c57d@bovespa.com> From: "=?Windows-1252?Q?Cl=E1udio_\=28Pr=E1tica\=29?=" To: Subject: [obm-l] PARI-GP Date: Fri, 25 Apr 2003 17:07:55 -0300 MIME-Version: 1.0 Content-Type: multipart/alternative; boundary="----=_NextPart_000_005B_01C30B4D.36B932C0" X-Priority: 3 X-MSMail-Priority: Normal X-Mailer: Microsoft Outlook Express 5.50.4920.2300 X-MimeOLE: Produced By Microsoft MimeOLE V5.50.4920.2300 Sender: owner-obm-l@sucuri.mat.puc-rio.br Precedence: bulk Reply-To: obm-l@mat.puc-rio.br This is a multi-part message in MIME format. ------=_NextPart_000_005B_01C30B4D.36B932C0 Content-Type: text/plain; charset="Windows-1252" Content-Transfer-Encoding: quoted-printable HelpCaros colegas da lista: Pra quem gosta dos aspectos computacionais da teoria dos n=FAmeros, tem = um software GRATUITO que pode ser baixado da internet. Chama-se PARI-GP = e =E9 bem bom pra fazer pesquisa caseira (que =E9 o meu caso - = certamente existem programas melhores e mais sofisticados, mas em termos = de custo-benef=EDcio esse =E9 imbat=EDvel).=20 Mais informa=E7=F5es no site: http://www.math.u-psud.fr/~belabas/gp.html A p=E1gina de "downloads" =E9: http://www.gn-50uma.de/ftp/pari/00index.html L=E1 voc=EA tamb=E9m encontrar=E1 o manual, o guia de instala=E7=E3o e = um "tutorial". Bom proveito. Um abra=E7o, Claudio. ------=_NextPart_000_005B_01C30B4D.36B932C0 Content-Type: text/html; charset="Windows-1252" Content-Transfer-Encoding: quoted-printable Help
Caros colegas da lista:
 
Pra quem gosta dos aspectos computacionais da teoria dos n=FAmeros, = tem um=20 software GRATUITO que pode ser baixado da internet. Chama-se PARI-GP e = =E9 bem bom=20 pra fazer pesquisa caseira (que =E9 o meu caso - certamente existem = programas=20 melhores e mais sofisticados, mas em termos de custo-benef=EDcio esse = =E9=20 imbat=EDvel).
 
Mais informa=E7=F5es no site:
http://www.math.u-psu= d.fr/~belabas/gp.html
 
A p=E1gina de "downloads" =E9:
http://www.gn-50uma= .de/ftp/pari/00index.html
 
L=E1 voc=EA tamb=E9m encontrar=E1 o manual, o guia de = instala=E7=E3o e um=20 "tutorial".
 
Bom proveito.
 
Um abra=E7o,
Claudio.
 
 
------=_NextPart_000_005B_01C30B4D.36B932C0-- ========================================================================= Instruções para entrar na lista, sair da lista e usar a lista em http://www.mat.puc-rio.br/~nicolau/olimp/obm-l.html ========================================================================= From owner-obm-l@sucuri.mat.puc-rio.br Fri Apr 25 17:51:02 2003 Return-Path: Received: (from majordom@localhost) by sucuri.mat.puc-rio.br (8.9.3/8.9.3) id RAA29688 for obm-l-MTTP; Fri, 25 Apr 2003 17:47:32 -0300 Received: from cmsrelay01.mx.net (cmsrelay01.mx.net [165.212.11.110]) by sucuri.mat.puc-rio.br (8.9.3/8.9.3) with SMTP id RAA29684 for ; Fri, 25 Apr 2003 17:47:27 -0300 Received: from cmsapps02.cms.usa.net (165.212.11.138) by cmsoutbound.mx.net with SMTP; 25 Apr 2003 20:46:56 -0000 Received: from uwdvg003.cms.usa.net [165.212.8.3] by cmsapps02.cms.usa.net (ASMTP/) via mtad (C8.MAIN.3.05) with ESMTP id 167HDyuU40370M38; Fri, 25 Apr 2003 20:46:55 GMT Received: from 200.181.4.100 [200.181.4.100] by uwdvg003.cms.usa.net (USANET web-mailer CM.0402.5.2B); Fri, 25 Apr 2003 20:46:54 -0000 Date: Fri, 25 Apr 2003 17:46:54 -0300 From: Artur Costa Steiner To: Subject: [obm-l] =?ISO-8859-1?Q?Re=3A=20=5B=5Bobm=2Dl=5D=20Problema=20de=20?= =?ISO-8859-1?Q?aproxima=E7=E3o=5D?= X-Mailer: USANET web-mailer (CM.0402.5.2B) Mime-Version: 1.0 Message-ID: <606HDyuU37584S03.1051303614@uwdvg003.cms.usa.net> Content-Type: text/plain; charset=ISO-8859-1 Content-Transfer-Encoding: 8bit X-MIME-Autoconverted: from quoted-printable to 8bit by sucuri.mat.puc-rio.br id RAA29685 Sender: owner-obm-l@sucuri.mat.puc-rio.br Precedence: bulk Reply-To: obm-l@mat.puc-rio.br Eu nao entendi bem sua solucao (tambem nao pude ainda analisar em detalhes). O que me ocorreu foi fazer o seguinte: Sejam a, b e c os coeficientes do pol. procurado (a do 2 grau, b do primeiro, c termo independente). Entao verificamos que a integral do polinomio de 0 a 1 eh a/3 + b/2 + c. Logo, a/3 + b/2 + c = 0 eh uma restricao do problema. A integral de f(t) p(t) de 0 a 1 eh dada por a * Int ( 0 a 1) f(t) t^2 dt + b* Int (0 a 1) t f(t)dt + c Int (0 a 1) f(t) dtt. Assumindo que vc consiga, ao menos numericamente, determinar tais integrais e chamando cada uma dela de A, B e C, chegamos ao seguinte problema de otimizacao: Minimizar |a *A + b* B + c* C| sujeito a a/3 + b/2 + c = 0. Como a restricao eh linear e a funcao objetivo eh o valor absoluto de uma funcao linear em a, b e c, fica facil resolver. Artur "butkov-cam" wrote: > Estou com dúvida no seguinte problema: > > Seja X o espaço das funções contínuas entre zero e um > usando-se a norma 2. Seja f um elemento deste espaço. > > Queremos determinar um polinômio p(t) de grau menor ou > igual a dois que minimiza a integral de zero a um do > módulo de f(t) - p(t) de forma que a integral de zero a > um de p(t) seja zero. > > Minha dúvida é se o problema fica inteiramente resolvido > se projetarmos a função f(t) no subespaço gerado pelos > polinômios que satisfazem a restrição imposta (integral > de zero a um de p(t) é zero ). > > Assim teríamos: > > ********o subspaço dos p(t) que satisfazem é gerado por > [1-3t^2, 2t-3t^2]. Chamemos e_1=1-3t^2 e e_2=2t-3t^2. > > p_solucao=alfa*e_1+beta*e_2 onde alfa e beta seriam > soluçoes do sistema abaixo: > > 4/5*alfa + 3/10*beta =integral de zero a um de f(t)*e_1 > 3/10*alfa+2/15*beta = integral de zero a um de f(t)*e_2 > > Meu raciocínio está correto???? > > > Grata por qualquer ajuda, > Camila. > > > > __________________________________________________________________________ > Seleção de Softwares UOL. > 10 softwares escolhidos pelo UOL para você e sua família. > http://www.uol.com.br/selecao > > > ========================================================================= > Instruções para entrar na lista, sair da lista e usar a lista em > http://www.mat.puc-rio.br/~nicolau/olimp/obm-l.html > ========================================================================= > ========================================================================= Instruções para entrar na lista, sair da lista e usar a lista em http://www.mat.puc-rio.br/~nicolau/olimp/obm-l.html ========================================================================= From owner-obm-l@sucuri.mat.puc-rio.br Fri Apr 25 18:41:44 2003 Return-Path: Received: (from majordom@localhost) by sucuri.mat.puc-rio.br (8.9.3/8.9.3) id SAA30992 for obm-l-MTTP; Fri, 25 Apr 2003 18:38:30 -0300 Received: from seki.bol.com.br (seki.bol.com.br [200.221.24.26]) by sucuri.mat.puc-rio.br (8.9.3/8.9.3) with ESMTP id SAA30988 for ; Fri, 25 Apr 2003 18:38:27 -0300 Received: from bol.com.br (200.221.24.138) by seki.bol.com.br (5.1.071) id 3E9EA3BD00230DBA for obm-l@mat.puc-rio.br; Fri, 25 Apr 2003 18:37:50 -0300 Date: Fri, 25 Apr 2003 18:37:50 -0300 Message-Id: Subject: [obm-l] Funcao MIME-Version: 1.0 Content-Type: text/plain;charset="iso-8859-1" From: "osvaldomellospq" To: obm-l@mat.puc-rio.br X-XaM3-API-Version: 2.4 R3 ( B4 ) X-SenderIP: 200.145.242.28 Content-Transfer-Encoding: 8bit X-MIME-Autoconverted: from quoted-printable to 8bit by sucuri.mat.puc-rio.br id SAA30989 Sender: owner-obm-l@sucuri.mat.puc-rio.br Precedence: bulk Reply-To: obm-l@mat.puc-rio.br Gostaria que alguem me ajudasse a resolver esse problema que estou entusiasmado porem nao consegui demonstrar minhas hipoteses: "Encontre todos os x pertencentes a R tal que f.f(2x)=fof(2x+1),sabendo que f nao e multiplicativa" __________________________________________________________________________ Seleção de Softwares UOL. 10 softwares escolhidos pelo UOL para você e sua família. http://www.uol.com.br/selecao ========================================================================= Instruções para entrar na lista, sair da lista e usar a lista em http://www.mat.puc-rio.br/~nicolau/olimp/obm-l.html ========================================================================= From owner-obm-l@sucuri.mat.puc-rio.br Fri Apr 25 19:28:05 2003 Return-Path: Received: (from majordom@localhost) by sucuri.mat.puc-rio.br (8.9.3/8.9.3) id TAA32191 for obm-l-MTTP; Fri, 25 Apr 2003 19:25:30 -0300 Received: from mail.gmx.net (mail.gmx.net [213.165.65.60]) by sucuri.mat.puc-rio.br (8.9.3/8.9.3) with SMTP id TAA32181 for ; Fri, 25 Apr 2003 19:25:25 -0300 Received: (qmail 23049 invoked by uid 65534); 25 Apr 2003 22:24:52 -0000 Received: from unknown (EHLO localhost) (200.149.213.105) by mail.gmx.net (mp005-rz3) with SMTP; 26 Apr 2003 00:24:52 +0200 Date: Fri, 25 Apr 2003 14:32:00 -0300 From: Igor GomeZZ X-Mailer: The Bat! (v1.61) Organization: -- X-Priority: 3 (Normal) Message-ID: <6018417533.20030425143200@gmx.net> To: Luis Lopes Subject: Re[2]: [obm-l] Falha nossa Nossa! In-Reply-To: <019301c30b3c$ed3ac1a0$5400a8c0@ensrbr> References: <019301c30b3c$ed3ac1a0$5400a8c0@ensrbr> MIME-Version: 1.0 Content-Type: text/plain; charset=ISO-8859-1 Content-Transfer-Encoding: 8bit Sender: owner-obm-l@sucuri.mat.puc-rio.br Precedence: bulk Reply-To: obm-l@mat.puc-rio.br Em 25/4/2003, 12:11, Luis (llopes@ensrbr.com.br) disse: > Sauda,c~oes, > Nossa mãe, é verdade! Falei besteira > também. Tranquilo Luis... Os melhores admitem a falha (note o singular, pq não são comuns :-)) > Transcrevo agora um texto do meu livro de > Progressões. Cuidado com o copyright... Vc vai ser preso por copiar seu próprio livro ehehehehe > "O termo geral a_i é válido também para valores > de i tais que i<=0, já que a seqüência {a_i} não > "começa" em a_1=1. Consultando a tabela da > página 11 (das diferenças), obtemos rapidamente > a seqüência \Delta^2 a_i "expandida": {-22,-10, > 2,14,26,38,...}. Com estes valores e subindo uma > linha na mesma tabela, encontramos outros termos > para a seqüência \Delta a_i: {32,10,0,2,16,...}. Com > a seqüência \Delta a_i geramos os termos a_0 > (a_0=1) e a_{-1} (a_{-1}= - 9), obtendo a seqüência > a_i expandida: {-9,1,1,3,19,...}. Com esta seqüência, > a determinação dos coeficientes \alpha_i é muito > mais fácil. De imediato obtemos \alpha_0=1 pois > \alpha_0=a_0. Fazendo i=-1,1,2 e resolvendo o > sistema....." > Note que a_i = \alpha_3 i^3 + ... + \alpha_0. Ainda não tinha experimentado com índices menores que 1, tah anotada a dica... Facilita fácil > []'s > Luís Flws! Fui! ####### Igor GomeZZ ######## UIN: 29249895 Vitória, Espírito Santo, Brasil Criação: 25/4/2003 (14:27) #################################### Pare para pensar: Antes de temer o marxismo porque ele se declara ateu, devemos nos perguntar sempre que tipo de sociedade justa temos construído no mundo que se confessa cristão. (Frei Betto) #################################### ========================================================================= Instruções para entrar na lista, sair da lista e usar a lista em http://www.mat.puc-rio.br/~nicolau/olimp/obm-l.html ========================================================================= From owner-obm-l@sucuri.mat.puc-rio.br Fri Apr 25 19:28:06 2003 Return-Path: Received: (from majordom@localhost) by sucuri.mat.puc-rio.br (8.9.3/8.9.3) id TAA32184 for obm-l-MTTP; Fri, 25 Apr 2003 19:25:26 -0300 Received: from mail.gmx.net (mail.gmx.net [213.165.65.60]) by sucuri.mat.puc-rio.br (8.9.3/8.9.3) with SMTP id TAA32178 for ; Fri, 25 Apr 2003 19:25:22 -0300 Received: (qmail 22948 invoked by uid 65534); 25 Apr 2003 22:24:50 -0000 Received: from unknown (EHLO localhost) (200.149.213.105) by mail.gmx.net (mp005-rz3) with SMTP; 26 Apr 2003 00:24:50 +0200 Date: Fri, 25 Apr 2003 14:25:12 -0300 From: Igor GomeZZ X-Mailer: The Bat! (v1.61) Organization: -- X-Priority: 3 (Normal) Message-ID: <14218008855.20030425142512@gmx.net> To: Antonio Neto Subject: Re: [obm-l] Falha nossa In-Reply-To: References: MIME-Version: 1.0 Content-Type: text/plain; charset=ISO-8859-1 Content-Transfer-Encoding: 8bit Sender: owner-obm-l@sucuri.mat.puc-rio.br Precedence: bulk Reply-To: obm-l@mat.puc-rio.br Em 25/4/2003, 10:29, Antonio Neto (osneto@hotmail.com) disse: > Falei besteira, Igor. Nao reparei que era a soma, e troquei pela > determinacao do termo geral. O que eu disse aplica-se ao termo geral, mas > nao aa soma. Para tirar a dúvida, fui aos arquivos da lista para pegar os > valores de a, b, c, d e e. Escrevi o polinomio e calculei S(-1)= 1 e S(-2)= > 0, o que nao faz sentido, pelo menos para mim. Desculpem a falha, abracos, > olavo. Tranquilo então :-) Eu tb cheguei a calcular S(-1) e S(-2) mas não vi nenhuma relação com a sequência de cubos... Valeu pela ajuda Olavo! Fui! ####### Igor GomeZZ ######## UIN: 29249895 Vitória, Espírito Santo, Brasil Criação: 25/4/2003 (14:21) #################################### Pare para pensar: Um punho aberto ainda é um punho? (Lao-Tsé) #################################### ========================================================================= Instruções para entrar na lista, sair da lista e usar a lista em http://www.mat.puc-rio.br/~nicolau/olimp/obm-l.html ========================================================================= From owner-obm-l@sucuri.mat.puc-rio.br Fri Apr 25 21:25:19 2003 Return-Path: Received: (from majordom@localhost) by sucuri.mat.puc-rio.br (8.9.3/8.9.3) id VAA02029 for obm-l-MTTP; Fri, 25 Apr 2003 21:22:34 -0300 Received: from Euler.impa.br (euler.impa.br [147.65.1.3]) by sucuri.mat.puc-rio.br (8.9.3/8.9.3) with ESMTP id VAA02025 for ; Fri, 25 Apr 2003 21:22:31 -0300 Received: from [147.65.11.2] (dial02.impa.br [147.65.11.2]) by Euler.impa.br (8.11.6p2/8.11.6) with ESMTP id h3Q0Lw001283 for ; Fri, 25 Apr 2003 21:21:58 -0300 (EST) Message-Id: <200304260021.h3Q0Lw001283@Euler.impa.br> X-Mailer: Microsoft Outlook Express Macintosh Edition - 4.5 (0410) Date: Fri, 25 Apr 2003 21:20:46 -0300 Subject: Re: [obm-l] Bisse=?ISO-8859-1?B?5+NvIGRlIMFyZWEgZSBQZXLt?= metro From: "Eduardo Wagner" To: obm-l@mat.puc-rio.br Mime-version: 1.0 X-Priority: 3 Content-type: multipart/alternative; boundary="MS_Mac_OE_3134150446_236295_MIME_Part" Sender: owner-obm-l@sucuri.mat.puc-rio.br Precedence: bulk Reply-To: obm-l@mat.puc-rio.br > THIS MESSAGE IS IN MIME FORMAT. Since your mail reader does not understand this format, some or all of this message may not be legible. --MS_Mac_OE_3134150446_236295_MIME_Part Content-type: text/plain; charset="ISO-8859-1" Content-transfer-encoding: quoted-printable Bissectar esta perfeitamente correto e devemos usar. Ainda nao esta nos dicionarios mas vai aparecer logo. Existe bisseccao e o verbo correspondente eh exatamente bissectar. Outra coisa incrivel eh que fazer a intersecao eh, naturalmente, intersectar, outro verbo que ainda nao esta nos dicionarios mas que devemos usar. Repare que frequentemente usam erradamente o verbo interceptar com o sentido de fazer a intersecao. Interceptar eh outra coisa. Significa interromper um movimento (o carro da policia interceptou o dos bandidos..., o missil interceptou o foguete...). Assim, por exemplo, duas retas se intersectam no ponto P (e nao se interceptam). ---------- From: "Cl=E1udio \(Pr=E1tica\)" To: Subject: [obm-l] Bisse=E7=E3o de =C1rea e Per=EDmetro Date: Fri, Apr 25, 2003, 3:51 PM Uma reta passando pelo incentro do triangulo ABC bissecta a area se e so se bissecta o perimetro. Sejam os pontos X em AB e Y em AC. Suponha que a reta XY contenha o incentro I de ABC. Suponha que o c=EDrculo inscrito tangencie BC, AC e AB nos pontos M, N e P, respectivamente. Assim, IM =3D IN =3D IP =3D R =3D raio do c=EDrculo inscrito, e cada um destes segmentos =E9 perpendicular ao lado correspondente. Temos que: [ABC] =3D (1/2)*R*(AB+AC+BC) e [AXY] =3D [AXI] + [AYI] =3D (1/2)*AX*PI + (1/2)*AY*NI =3D (1/2)*R*(AX+AY) XY bissecta a =E1rea <=3D=3D> [ABC] =3D 2*[AXY] <=3D=3D> (1/2)*R*(AB+AC+BC) =3D 2*(1/2)*R*(AX+AY) <=3D=3D> AB+AC+BC =3D 2*(AX+AY) <=3D=3D> XY bissecta o pr=EDmetro Um abra=E7o, Claudio. P.S.: existe o verbo bissectar? Acho que o correto =E9 bissecionar ou bisseccionar ou bi-secionar....mas como esta =E9 uma lista de matem=E1tica.... ----- Original Message ----- From: Johann Peter Gustav Lejeune Dirichlet To: obm-l@mat.puc-rio.br Sent: Friday, April 25, 2003 1:23 PM Subject: [obm-l] TEOREMA DE CRISTEA:por favor,Claudio!!!!!! Bem,ja que e assim,sou digamos obrigado a aceitar as exigencias.Mas como e essa historia de comentario pertinente?Eu comento coisas sem nexo a cada minuto,mas nao tao gravemente.Quanto ao polidamente,isto e dificil pois escrever e sempre dificil de entender.Mas tudo bem,e a vida em grupo,digo,lista.Mas pode mandar a demo que meu desespero ja esta no fio do barril.... Tente resolver esse:uma reta passando pelo incentro do triangulo ABC bissecta a area se e so se bissecta o perimetro.E do Excalibur. ---------------------------------------------------------------------------= - Yahoo! Mail O melhor e-mail gratuito da internet: 6MB de espa=E7o, antiv=EDrus, acesso POP3= , filtro contra spam. --MS_Mac_OE_3134150446_236295_MIME_Part Content-type: text/html; charset="ISO-8859-1" Content-transfer-encoding: quoted-printable Re: [obm-l] Bisse=E7=E3o de =C1rea e Per=EDmetro Bissectar esta perfeitamente correto e devemos usar.
Ainda nao esta nos dicionarios mas vai aparecer logo.
Existe bisseccao e o verbo correspondente eh exatamente
bissectar.
Outra coisa incrivel eh que fazer a intersecao eh, naturalmente,
intersectar, outro verbo que ainda nao esta nos dicionarios
mas que devemos usar. Repare que frequentemente usam
erradamente o verbo interceptar com o sentido de fazer
a intersecao. Interceptar eh outra coisa. Significa interromper
um movimento (o carro da policia interceptou o dos bandidos...,
o missil interceptou o foguete...). Assim, por exemplo, duas
retas se intersectam no ponto P (e nao se interceptam).

----------
From: "Cl=E1udio \(Pr=E1tica\)" <claudio@praticacorretora.com.br&g= t;
To: <obm-l@mat.puc-rio.br>
Subject: [obm-l] Bisse=E7=E3o de =C1rea e Per=EDmetro
Date: Fri, Apr 25, 2003, 3:51 PM


Uma reta passando pelo incentro do triangulo ABC bissecta a are= a se e so se bissecta o perimetro.
 
Sejam os pontos X em AB e Y em AC.
Suponha que a reta XY contenha o incentro I de ABC.
Suponha que o c=EDrculo inscrito tangencie BC, AC e AB nos pontos M, N e P, r= espectivamente.
Assim, IM =3D IN =3D IP =3D R =3D raio do c=EDrculo inscrito, e cada um destes segmen= tos =E9 perpendicular ao lado correspondente.
 
Temos que:
[ABC] =3D (1/2)*R*(AB+AC+BC)
e
[AXY] =3D [AXI] + [AYI] =3D (1/2)*AX*PI + (1/2)*AY*NI =3D (1/2)*R*(AX+AY)
 
XY bissecta a =E1rea  <=3D=3D>
[ABC] =3D 2*[AXY]  <=3D=3D>
(1/2)*R*(AB+AC+BC) =3D 2*(1/2)*R*(AX+AY)  <=3D=3D>
AB+AC+BC =3D 2*(AX+AY)  <=3D=3D>
XY bissecta o pr=EDmetro
 
Um abra=E7o,
Claudio.
 
P.S.: existe o verbo bissectar? Acho que = o correto =E9 bissecionar ou bisseccionar ou bi-secionar....mas como esta =E9 um= a lista de matem=E1tica....
 
----- Original Message -----
From: Johann Peter Gustav Lejeune Dirichlet <mailto:peterdirichlet2002@yahoo.com.br>  
To: obm-l@mat.puc-rio.br <mailto:obm-l@mat.puc-rio.br>  
Sent: Friday, April 25, 2003 1:23 PM
Subject: [obm-l] TEOREMA DE CRISTEA:por favor,Claudio!!!!!!

Bem,ja que e assim,sou digamos obrigado a aceitar as exigencias.Mas como e = essa historia de comentario pertinente?Eu comento coisas sem nexo a cada min= uto,mas nao tao gravemente.Quanto ao polidamente,isto e dificil pois escreve= r e sempre dificil de entender.Mas tudo bem,e a vida em grupo,digo,lista.Mas= pode mandar a demo que meu desespero ja esta no fio do barril....
 
Tente resolver esse:uma reta passando pelo incentro do triangulo ABC bissec= ta a area se e so se bissecta o perimetro.E do Excalibur.
 
 



Yahoo! Mail  <http://br.mail.yahoo.= com/>
O melhor e-mail gratuito da internet: 6MB de espa=E7o, antiv=EDrus, acesso = POP3, filtro contra spam.

--MS_Mac_OE_3134150446_236295_MIME_Part-- ========================================================================= Instruções para entrar na lista, sair da lista e usar a lista em http://www.mat.puc-rio.br/~nicolau/olimp/obm-l.html ========================================================================= From owner-obm-l@sucuri.mat.puc-rio.br Sat Apr 26 10:43:49 2003 Return-Path: Received: (from majordom@localhost) by sucuri.mat.puc-rio.br (8.9.3/8.9.3) id KAA08344 for obm-l-MTTP; Sat, 26 Apr 2003 10:40:40 -0300 Received: (from nicolau@localhost) by sucuri.mat.puc-rio.br (8.9.3/8.9.3) id KAA08339 for obm-l@mat.puc-rio.br; Sat, 26 Apr 2003 10:40:39 -0300 Date: Sat, 26 Apr 2003 10:40:39 -0300 From: "Nicolau C. Saldanha" To: obm-l@mat.puc-rio.br Subject: Re: [obm-l] Falha nossa Message-ID: <20030426104039.A8053@sucuri.mat.puc-rio.br> References: Mime-Version: 1.0 Content-Type: text/plain; charset=iso-8859-1 Content-Disposition: inline Content-Transfer-Encoding: 8bit User-Agent: Mutt/1.2.5i In-Reply-To: ; from osneto@hotmail.com on Fri, Apr 25, 2003 at 01:29:25PM +0000 Sender: owner-obm-l@sucuri.mat.puc-rio.br Precedence: bulk Reply-To: obm-l@mat.puc-rio.br On Fri, Apr 25, 2003 at 01:29:25PM +0000, Antonio Neto wrote: > Falei besteira, Igor. Nao reparei que era a soma, e troquei pela > determinacao do termo geral. O que eu disse aplica-se ao termo geral, mas > nao aa soma. Para tirar a dúvida, fui aos arquivos da lista para pegar os > valores de a, b, c, d e e. Escrevi o polinomio e calculei S(-1)= 1 e S(-2)= > 0, o que nao faz sentido, pelo menos para mim. Desculpem a falha, abracos, > olavo. Para mim faz todo o sentido falar em S(n) para qualquer inteiro. Temos S(1) = 1^3 = 1 S(2) = 1^3 + 2^3 = 9 S(3) = 1^3 + 2^3 + 3^3 = 36 A propriedade importante é S(n+1) = S(n) + (n+1)^3 o que, junto com S(0) = 0, define S. Mas se você desejar uma interpretação mais explícita, tome S(n) = 1^3 + 2^3 + ... + n^3, n >= 1 S(n) = - ( (-1)^3 + (-2)^3 + ... + (n+1)^3 ), n < -1 Isto é um pouco como definir uma integral de 1 a 0 como menos a integral de 0 a 1, só é preciso ter cuidado com as pontas (se incluí-las ou não). Observe que temos S(n) = S(-1-n). Os valores acima você obviamente trocou, o certo seria S(-1) = 0, S(-2) = 1, S(-3) = 9, S(-4) = 36. Se um aluno precisar resolver um sistema por força bruta (sem calculadora) seria ótima idéia não apenas usar estes valores (que são menores) mas usar a simetria S(n) = S(-1-n) para ver que, supondo S um polinômio de grau <= 4, podemos escrever S(n) = a + b (2n+1)^2 + c (2n+1)^4 Você pode pensar nisso como uma expansão em Taylor ao redor do centro de simetria n=1/2. Os termos de grau ímpar são iguais a 0. S(0) = a + b + c = 0 S(1) = a + 9b + 81c = 1 S(2) = a + 25b + 625c = 9 o que fazendo eliminação gaussina dá a + b + c = 0 b + 10c = 1/8 b + 26c = 9/24 = 3/8 16c = 2/8 = 1/4 -----> c = 1/64 b + 10/64 = 1/8 = 8/64 ----> b = -2/64 a + b + c = 0 --------------> a = 1/64 S(n) = (1 - 2(2n+1)^2 + (2n+1)^4)/64 = (((2n+1)^2 - 1)/8)^2 = ((4n^2 - 4n)/8)^2 = (n(n-1)/2)^2 A fórmula está certa. Se você já sabe que a resposta deve ser um polinômio de grau 4, a demonstração acaba aqui. Se você não sabe (apenas conjecturou) você precisa usar a fórmula e fazer uma pequena demonstração por indução. É claro que tudo isso é para um aluno muito tímido para conjecturar. Afinal, vendo os números 0, 1, 9, 36 lá no início o aluno *deveria* ter conjecturado que S(n) = (f(n))^2 onde f é alguma função inteira simples, talvez um polinômio. E os primeiros valores de f (0,1,3,6,10,...) deveriam acabar de entregar a resposta... []s, N. ========================================================================= Instruções para entrar na lista, sair da lista e usar a lista em http://www.mat.puc-rio.br/~nicolau/olimp/obm-l.html ========================================================================= From owner-obm-l@sucuri.mat.puc-rio.br Sat Apr 26 10:43:54 2003 Return-Path: Received: (from majordom@localhost) by sucuri.mat.puc-rio.br (8.9.3/8.9.3) id KAA08355 for obm-l-MTTP; Sat, 26 Apr 2003 10:41:23 -0300 Received: from artemis.opendf.com.br (artemis.opengate.com.br [200.181.71.14]) by sucuri.mat.puc-rio.br (8.9.3/8.9.3) with ESMTP id KAA08351 for ; Sat, 26 Apr 2003 10:41:19 -0300 Received: from localhost (localhost [127.0.0.1]) by artemis.opendf.com.br (Postfix) with ESMTP id 09B6A2BED0 for ; Sat, 26 Apr 2003 10:41:56 -0300 (BRT) Received: from artemis.opendf.com.br ([127.0.0.1]) by localhost (artemis.opengate.com.br [127.0.0.1:10024]) (amavisd-new) with ESMTP id 23749-07 for ; Sat, 26 Apr 2003 10:41:55 -0300 (BRT) Received: from computer (200-181-089-199.bsace7001.dsl.brasiltelecom.net.br [200.181.89.199]) by artemis.opendf.com.br (Postfix) with ESMTP id 77E3C2BECE for ; Sat, 26 Apr 2003 10:41:54 -0300 (BRT) From: "Artur Costa Steiner" To: Subject: RE: [obm-l] Desigualdade de Gram Date: Sat, 26 Apr 2003 10:40:45 -0300 Organization: Steiner Consultoria LTDA Message-ID: <001001c30bf9$731edab0$9865fea9@computer> MIME-Version: 1.0 Content-Type: text/plain; charset="iso-8859-1" X-Priority: 3 (Normal) X-MSMail-Priority: Normal X-Mailer: Microsoft Outlook, Build 10.0.2627 Importance: Normal X-MimeOLE: Produced By Microsoft MimeOLE V6.00.2800.1165 In-Reply-To: X-Virus-Scanned: by amavisd-new Content-Transfer-Encoding: 8bit X-MIME-Autoconverted: from quoted-printable to 8bit by sucuri.mat.puc-rio.br id KAA08352 Sender: owner-obm-l@sucuri.mat.puc-rio.br Precedence: bulk Reply-To: obm-l@mat.puc-rio.br Definamos H como a matriz cujas colunas sao os vetores x_1,....x_n do espaco de Hilbert. Eh uma matriz n X m, onde m eh a dimensao do espaco, considerand0=-se os vetores expressos como combinacoes lineares de uma base do espaco. Se G eh a matriz de Gram associada a x_1,...x_n, entao temos que G = H'H, onde H' eh a transposta de H. Sendo y um vetor qualquer do espaco, segue-se que y'G y = y'H'H y = (Hy)' Hy = Hy. Hy, onde o ponto denota o produto interno. Logo y'G y>=0, sendo 0 sse y=0. isto nos mostra que a matriz de Gram eh positiva semidefinida, e se seus vetores forem LI entao eh positiva definida. Neste ultimo caso, seu determinante eh > e todos seus autovalores tambem o sao. Um abraco Artur >Subject: [obm-l] Desigualdade de Gram > >Colegas estou com dificuldades em demonstrar a >desigualdade de Gram no caso geral (n qualquer natural) > >Os casos n = 1 e n = 2 (Cauchy-Schwarz) são triviais. > >O problema é o seguinte: > >Dados x_1,x_2, ..., x_n de um espaço de hilbert. > >O determinante da matriz de Gram é maior ou igual a zero. > >matriz de gram = G(x_1, ..., X_n) = [] onde >0 > >P.S. Não é difícil concluir que o determinante de GRAM >será igual a zero se e somente se x_1,...,x_n forem LD. >Esta parte eu inclusive já fiz. Falta mostra que ele é >sempre maior que zero se x_1,...,x_n forem LI.... > >Grata por qualquer ajuda, >Camila. > > >_______________________________________________________________________ ___ >Seleção de Softwares UOL. >10 softwares escolhidos pelo UOL para você e sua família. >http://www.uol.com.br/selecao > > >======================================================================= == >Instruções para entrar na lista, sair da lista e usar a lista em >http://www.mat.puc-rio.br/~nicolau/olimp/obm-l.html >======================================================================= == ========================================================================= Instruções para entrar na lista, sair da lista e usar a lista em http://www.mat.puc-rio.br/~nicolau/olimp/obm-l.html ========================================================================= From owner-obm-l@sucuri.mat.puc-rio.br Sat Apr 26 10:46:03 2003 Return-Path: Received: (from majordom@localhost) by sucuri.mat.puc-rio.br (8.9.3/8.9.3) id KAA08367 for obm-l-MTTP; Sat, 26 Apr 2003 10:43:32 -0300 Received: from shen.bol.com.br (shen.bol.com.br [200.221.24.14]) by sucuri.mat.puc-rio.br (8.9.3/8.9.3) with ESMTP id KAA08362 for ; Sat, 26 Apr 2003 10:43:24 -0300 Received: from bol.com.br (200.221.24.131) by shen.bol.com.br (5.1.071) id 3E7F61BF007E8837 for obm-l@mat.puc-rio.br; Sat, 26 Apr 2003 10:42:53 -0300 Date: Sat, 26 Apr 2003 10:42:53 -0300 Message-Id: Subject: Re:[obm-l] Desigualdade de Gram MIME-Version: 1.0 Content-Type: text/plain;charset="iso-8859-1" From: "arakelov" To: obm-l@mat.puc-rio.br X-XaM3-API-Version: 2.4 R3 ( B4 ) X-SenderIP: 200.151.222.155 Content-Transfer-Encoding: 8bit X-MIME-Autoconverted: from quoted-printable to 8bit by sucuri.mat.puc-rio.br id KAA08363 Sender: owner-obm-l@sucuri.mat.puc-rio.br Precedence: bulk Reply-To: obm-l@mat.puc-rio.br > Colegas estou com dificuldades em demonstrar a > desigualdade de Gram no caso geral (n qualquer natural) > > Os casos n = 1 e n = 2 (Cauchy-Schwarz) são triviais. > > O problema é o seguinte: > > Dados x_1,x_2, ..., x_n de um espaço de hilbert. > > O determinante da matriz de Gram é maior ou igual a zer o. > > matriz de gram = G(x_1, ..., X_n) = [] onde > 0 > > P.S. Não é difícil concluir que o determinante de GRAM > será igual a zero se e somente se x_1,...,x_n forem LD. > Esta parte eu inclusive já fiz. Falta mostra que ele é > sempre maior que zero se x_1,...,x_n forem LI.... > > Grata por qualquer ajuda, > Camila. > > > _______________________________________________________ ___________________ > Seleção de Softwares UOL. > 10 softwares escolhidos pelo UOL para você e sua famíli a. > http://www.uol.com.br/selecao > > > ======================================================= ================== > Instruções para entrar na lista, sair da lista e usar a lista em > http://www.mat.puc-rio.br/~nicolau/olimp/obm-l.html > ======================================================= ================== > __________________________________________________________________________ Seleção de Softwares UOL. 10 softwares escolhidos pelo UOL para você e sua família. http://www.uol.com.br/selecao ========================================================================= Instruções para entrar na lista, sair da lista e usar a lista em http://www.mat.puc-rio.br/~nicolau/olimp/obm-l.html ========================================================================= From owner-obm-l@sucuri.mat.puc-rio.br Sat Apr 26 10:56:38 2003 Return-Path: Received: (from majordom@localhost) by sucuri.mat.puc-rio.br (8.9.3/8.9.3) id KAA08648 for obm-l-MTTP; Sat, 26 Apr 2003 10:52:23 -0300 Received: from salem.bol.com.br (salem.bol.com.br [200.221.24.25]) by sucuri.mat.puc-rio.br (8.9.3/8.9.3) with ESMTP id KAA08644 for ; Sat, 26 Apr 2003 10:52:20 -0300 Received: from bol.com.br (200.221.24.132) by salem.bol.com.br (5.1.071) id 3EA6DA33000F0F3F for obm-l@mat.puc-rio.br; Sat, 26 Apr 2003 10:51:43 -0300 Date: Sat, 26 Apr 2003 10:51:43 -0300 Message-Id: Subject: [obm-l] duvida de geometria MIME-Version: 1.0 Content-Type: text/plain;charset="iso-8859-1" From: "thor-oliveira" To: obm-l@mat.puc-rio.br X-XaM3-API-Version: 2.4 R3 ( B4 ) X-SenderIP: 200.151.222.155 Content-Transfer-Encoding: 8bit X-MIME-Autoconverted: from quoted-printable to 8bit by sucuri.mat.puc-rio.br id KAA08645 Sender: owner-obm-l@sucuri.mat.puc-rio.br Precedence: bulk Reply-To: obm-l@mat.puc-rio.br Oi estou com a seguinte duvida em geometria,qual a soma dos ângulos internos de um poligono concavo?se existe qual e ela e se possivel for me enviem uma demonstração.agradeço pela sua atenção. __________________________________________________________________________ Seleção de Softwares UOL. 10 softwares escolhidos pelo UOL para você e sua família. http://www.uol.com.br/selecao ========================================================================= Instruções para entrar na lista, sair da lista e usar a lista em http://www.mat.puc-rio.br/~nicolau/olimp/obm-l.html ========================================================================= From owner-obm-l@sucuri.mat.puc-rio.br Sat Apr 26 11:13:55 2003 Return-Path: Received: (from majordom@localhost) by sucuri.mat.puc-rio.br (8.9.3/8.9.3) id LAA09523 for obm-l-MTTP; Sat, 26 Apr 2003 11:11:20 -0300 Received: from artemis.opendf.com.br (artemis.opengate.com.br [200.181.71.14]) by sucuri.mat.puc-rio.br (8.9.3/8.9.3) with ESMTP id LAA09519 for ; Sat, 26 Apr 2003 11:11:17 -0300 Received: from localhost (localhost [127.0.0.1]) by artemis.opendf.com.br (Postfix) with ESMTP id 9B7B92BEC8 for ; Sat, 26 Apr 2003 11:11:53 -0300 (BRT) Received: from artemis.opendf.com.br ([127.0.0.1]) by localhost (artemis.opengate.com.br [127.0.0.1:10024]) (amavisd-new) with ESMTP id 23992-07 for ; Sat, 26 Apr 2003 11:11:52 -0300 (BRT) Received: from computer (200-181-089-199.bsace7001.dsl.brasiltelecom.net.br [200.181.89.199]) by artemis.opendf.com.br (Postfix) with ESMTP id 7781B2BEC5 for ; Sat, 26 Apr 2003 11:11:52 -0300 (BRT) From: "Artur Costa Steiner" To: Subject: RE: [obm-l] Funcao Date: Sat, 26 Apr 2003 11:10:47 -0300 Organization: Steiner Consultoria LTDA Message-ID: <001b01c30bfd$a282ac10$9865fea9@computer> MIME-Version: 1.0 Content-Type: text/plain; charset="us-ascii" Content-Transfer-Encoding: 7bit X-Priority: 3 (Normal) X-MSMail-Priority: Normal X-Mailer: Microsoft Outlook, Build 10.0.2627 Importance: Normal X-MimeOLE: Produced By Microsoft MimeOLE V6.00.2800.1165 In-Reply-To: X-Virus-Scanned: by amavisd-new Sender: owner-obm-l@sucuri.mat.puc-rio.br Precedence: bulk Reply-To: obm-l@mat.puc-rio.br >-----Original Message----- >From: owner-obm-l@sucuri.mat.puc-rio.br [mailto:owner-obm-l@sucuri.mat.puc- >rio.br] On Behalf Of osvaldomellospq >Sent: Friday, April 25, 2003 6:38 PM >To: obm-l@mat.puc-rio.br >Subject: [obm-l] Funcao > >Gostaria que alguem me ajudasse a resolver esse problema que estou >entusiasmado porem nao consegui demonstrar minhas hipoteses: > >"Encontre todos os x pertencentes a R tal que f.f(2x)=fof(2x+1),sabendo >que f nao e multiplicativa" [Artur Costa Steiner] O que vc quer dizer exatamente por f.f(2x)? Nao estah claro Artur ========================================================================= Instruções para entrar na lista, sair da lista e usar a lista em http://www.mat.puc-rio.br/~nicolau/olimp/obm-l.html ========================================================================= From owner-obm-l@sucuri.mat.puc-rio.br Sat Apr 26 12:55:26 2003 Return-Path: Received: (from majordom@localhost) by sucuri.mat.puc-rio.br (8.9.3/8.9.3) id MAA11520 for obm-l-MTTP; Sat, 26 Apr 2003 12:52:11 -0300 Received: from paiol.terra.com.br (paiol.terra.com.br [200.176.3.18]) by sucuri.mat.puc-rio.br (8.9.3/8.9.3) with ESMTP id MAA11516 for ; Sat, 26 Apr 2003 12:52:07 -0300 Received: from araci.terra.com.br (araci.terra.com.br [200.176.3.44]) by paiol.terra.com.br (Postfix) with ESMTP id 953C28803D for ; Sat, 26 Apr 2003 12:51:34 -0300 (BRT) Received: from [200.177.176.165] (dl-nas1-sao-C8B1B0A5.p001.terra.com.br [200.177.176.165]) by araci.terra.com.br (Postfix) with ESMTP id D80ED21EF6E for ; Sat, 26 Apr 2003 12:51:33 -0300 (BRT) User-Agent: Microsoft-Outlook-Express-Macintosh-Edition/5.02.2022 Date: Sat, 26 Apr 2003 12:52:39 -0300 Subject: Re: [obm-l] duvida de geometria From: Claudio Buffara To: Message-ID: In-Reply-To: Mime-version: 1.0 Content-type: text/plain; charset="ISO-8859-1" Content-Transfer-Encoding: 8bit X-MIME-Autoconverted: from quoted-printable to 8bit by sucuri.mat.puc-rio.br id MAA11517 Sender: owner-obm-l@sucuri.mat.puc-rio.br Precedence: bulk Reply-To: obm-l@mat.puc-rio.br on 26.04.03 10:51, thor-oliveira at thor-oliveira@bol.com.br wrote: > Oi estou com a seguinte duvida em geometria,qual a soma > dos ângulos internos de um poligono concavo?se existe > qual e ela e se possivel for me enviem uma > demonstração.agradeço pela sua atenção. > > Supondo que dois lados quaisquer do poligono so se encontrem num vertice, a soma tambem serah igual a 180*(n - 2) graus, onde n eh o numero de lados. Uma demonstracao disso encontra-se nas pags. 108-110 do livro Meu professor de Matematica, escrito pelo Elon lages Lima e publicado pela SBM. A ideia basica eh mostrar que um poligono concavo de n lados (da mesma forma que um convexo) tambem pode ser decomposto como uma reuniao de n - 2 triangulos justapostos, cujos vertices sao vertices do poligono dado. Um abraco, Claudio. ========================================================================= Instruções para entrar na lista, sair da lista e usar a lista em http://www.mat.puc-rio.br/~nicolau/olimp/obm-l.html ========================================================================= From owner-obm-l@sucuri.mat.puc-rio.br Sat Apr 26 13:24:42 2003 Return-Path: Received: (from majordom@localhost) by sucuri.mat.puc-rio.br (8.9.3/8.9.3) id NAA12113 for obm-l-MTTP; Sat, 26 Apr 2003 13:22:04 -0300 Received: from hotmail.com (f47.law15.hotmail.com [64.4.23.47]) by sucuri.mat.puc-rio.br (8.9.3/8.9.3) with ESMTP id NAA12108 for ; Sat, 26 Apr 2003 13:22:00 -0300 Received: from mail pickup service by hotmail.com with Microsoft SMTPSVC; Sat, 26 Apr 2003 09:21:28 -0700 Received: from 150.162.198.2 by lw15fd.law15.hotmail.msn.com with HTTP; Sat, 26 Apr 2003 16:21:28 GMT X-Originating-IP: [150.162.198.2] X-Originating-Email: [fabiomatias@hotmail.com] From: "fabio matias" To: obm-l@mat.puc-rio.br Subject: [obm-l] derivada Date: Sat, 26 Apr 2003 13:21:28 -0300 Mime-Version: 1.0 Content-Type: text/plain; charset=iso-8859-1; format=flowed Message-ID: X-OriginalArrivalTime: 26 Apr 2003 16:21:28.0616 (UTC) FILETIME=[E4063280:01C30C0F] Sender: owner-obm-l@sucuri.mat.puc-rio.br Precedence: bulk Reply-To: obm-l@mat.puc-rio.br Oi Pessoal, voces poderiam me ajuda com uma pesquisa que estou fazendo sobre a derivada de Caratheodory. Poderiam me indicando algum livro, site ou artigo que contenha esse assunto. Se não for pedir demais, poderiam me indicar algumas aplicações para essa derivada. Agradeço desde ja a conpreencao de todos. Fábio _________________________________________________________________ MSN Messenger: converse com os seus amigos online. http://messenger.msn.com.br ========================================================================= Instruções para entrar na lista, sair da lista e usar a lista em http://www.mat.puc-rio.br/~nicolau/olimp/obm-l.html ========================================================================= From owner-obm-l@sucuri.mat.puc-rio.br Sat Apr 26 13:36:19 2003 Return-Path: Received: (from majordom@localhost) by sucuri.mat.puc-rio.br (8.9.3/8.9.3) id NAA12470 for obm-l-MTTP; Sat, 26 Apr 2003 13:33:41 -0300 Received: from paiol.terra.com.br (paiol.terra.com.br [200.176.3.18]) by sucuri.mat.puc-rio.br (8.9.3/8.9.3) with ESMTP id NAA12464 for ; Sat, 26 Apr 2003 13:33:37 -0300 Received: from botucatu.terra.com.br (botucatu.terra.com.br [200.176.3.78]) by paiol.terra.com.br (Postfix) with ESMTP id AA79F87DEA for ; Sat, 26 Apr 2003 13:33:05 -0300 (BRT) Received: from niski.com (unknown [200.148.198.60]) (authenticated user fniski) by botucatu.terra.com.br (Postfix) with ESMTP id E098129C096 for ; Sat, 26 Apr 2003 13:33:03 -0300 (BRT) Message-ID: <3EAAB4BF.4080002@niski.com> Date: Sat, 26 Apr 2003 09:33:03 -0700 From: niski User-Agent: Mozilla/5.0 (Windows; U; Windows NT 5.1; en-US; rv:1.0.2) Gecko/20030208 Netscape/7.02 X-Accept-Language: en-us, en MIME-Version: 1.0 To: obm-l@mat.puc-rio.br Subject: [obm-l] Provar continuidade Content-Type: text/plain; charset=ISO-8859-1; format=flowed Content-Transfer-Encoding: 8bit Sender: owner-obm-l@sucuri.mat.puc-rio.br Precedence: bulk Reply-To: obm-l@mat.puc-rio.br Ola pessoal... este problema esta no meu livro mas nao tem resposta...gostaria que o pessoal conferisse se procedi corretamente. Muito obrigado. provar que f(x) = (x)^(1/n) é continua. Demonstraçao : Dado Eps > 0 existe um intervalo aberto I , p pertencente a I , tal que x pertence a I => f(p) - Eps < f(x) < f(p) + Eps Daí vem que (p)^(1/n) - Eps < (x)^(1/n) < (p)^(1/n) + Eps ((p)^(1/n) - Eps)^n < x < ((p)^(1/n) + Eps)^n tomando-se I = ]((p)^(1/n) - Eps)^n, ((p)^(1/n) + Eps)^n[ , p pertencente a I x pertence a I => f(p) - Eps < f(x) < f(p) + Eps logo f(x) = (x)^1/n é continua em todo p real. Mais uma vez, obrigado -- [about him:] It is rare to find learned men who are clean, do not stink and have a sense of humour. -Gottfried Whilhem Leibniz ========================================================================= Instruções para entrar na lista, sair da lista e usar a lista em http://www.mat.puc-rio.br/~nicolau/olimp/obm-l.html ========================================================================= From owner-obm-l@sucuri.mat.puc-rio.br Sat Apr 26 13:55:16 2003 Return-Path: Received: (from majordom@localhost) by sucuri.mat.puc-rio.br (8.9.3/8.9.3) id NAA13147 for obm-l-MTTP; Sat, 26 Apr 2003 13:52:07 -0300 Received: from artemis.opendf.com.br (artemis.opengate.com.br [200.181.71.14]) by sucuri.mat.puc-rio.br (8.9.3/8.9.3) with ESMTP id NAA13142 for ; Sat, 26 Apr 2003 13:52:03 -0300 Received: from localhost (localhost [127.0.0.1]) by artemis.opendf.com.br (Postfix) with ESMTP id F2B0E2BECE for ; Sat, 26 Apr 2003 13:52:40 -0300 (BRT) Received: from artemis.opendf.com.br ([127.0.0.1]) by localhost (artemis.opengate.com.br [127.0.0.1:10024]) (amavisd-new) with ESMTP id 26758-10 for ; Sat, 26 Apr 2003 13:52:40 -0300 (BRT) Received: from computer (200-181-089-199.bsace7001.dsl.brasiltelecom.net.br [200.181.89.199]) by artemis.opendf.com.br (Postfix) with ESMTP id 905D92BEC8 for ; Sat, 26 Apr 2003 13:52:38 -0300 (BRT) From: "Artur Costa Steiner" To: Subject: [obm-l] extensao continua de uma funcao Date: Sat, 26 Apr 2003 13:51:31 -0300 Organization: Steiner Consultoria LTDA Message-ID: MIME-Version: 1.0 Content-Type: multipart/mixed; boundary="----=_NextPart_000_0028_01C30BFA.F1DDF290" X-Priority: 3 (Normal) X-MSMail-Priority: Normal X-Mailer: Microsoft Outlook, Build 10.0.2627 Importance: Normal X-MimeOLE: Produced By Microsoft MimeOLE V6.00.2800.1165 X-MS-TNEF-Correlator: 000000005FC3F71D8B4BB941A8AA63D06F261C36A4442B00 X-Virus-Scanned: by amavisd-new Sender: owner-obm-l@sucuri.mat.puc-rio.br Precedence: bulk Reply-To: obm-l@mat.puc-rio.br This is a multi-part message in MIME format. ------=_NextPart_000_0028_01C30BFA.F1DDF290 Content-Type: text/plain; charset="iso-8859-1" Content-Transfer-Encoding: quoted-printable Ol=E1 a todos Ha cerca de um mes eu sugeri um problema de Analise para a lista (que, alias, constava na lista do Claudio de problemas em aberto). Um = companheiro, nao sei se desta lista, pois acabei perdendo o email dele, me solicitou = que apresentasse a respctiva demonstracao. Sejam D um subconjunto de R^n e f:D--> R^m uma funcao que apresenta = limite em todos os pontos de acumulacao de D. Entao, f possui uma unica = extensao continua para o fecho de D, D*.=20 Prova: Inicialmente, observamos que, se existir efetivamente uma f* = como a desejada, entao teremos necessariamente que f*(x) =3D f(x), se x = pertence a D, e f*(x)=3D lim y-->x f(y) se x nao pertence a D. Logo, se esta f* = existir, entao ela eh unica e tem a definicao dada. Nao ha outra candidata. Vamos agora mostrar que f* eh continua em D*. Seja x pertencente a D*. = Temos que f eh continua em x (se x estiver em D) ou (nao excludente) apresenta limite em x (se x nao estiver em D). Considerando-se a definicao de f*, segue-se que, dado qualquer eps>0, existe d>0 tal que |f*(y) - f*(x)| < eps para todo y em D tal que |y-x|0, existe algum u' em D tal que |u'- u| < d - |u-x| e |f*(u')- f*(u)| < = eps'. Temos entao, pela Desigualdade do Triangulo, que |u'-x| <=3D |u'-u| + = |u-x| < d- |u-x| + |u-x| =3Dd, do que se segue, face a (1), que |f*(u') - f*(x)| = < eps. Aplicando-se novamente a Desigualdade do Triangulo, concluimos que |f*(u) - f*(x)| <=3D |f*(u) - f*(u')| + |f*(u') - f*(x)| < eps' + eps. = Como eps' eh arbitrario, concluimos que |f*(u) - f*(x)| <=3D eps para todo u = em D* tal que |u-x Received: (from majordom@localhost) by sucuri.mat.puc-rio.br (8.9.3/8.9.3) id OAA13507 for obm-l-MTTP; Sat, 26 Apr 2003 14:03:56 -0300 Received: (from nicolau@localhost) by sucuri.mat.puc-rio.br (8.9.3/8.9.3) id OAA13502 for obm-l@mat.puc-rio.br; Sat, 26 Apr 2003 14:03:55 -0300 Date: Sat, 26 Apr 2003 14:03:55 -0300 From: "Nicolau C. Saldanha" To: obm-l@mat.puc-rio.br Subject: Re: [obm-l] Falha nossa Message-ID: <20030426140355.B13311@sucuri.mat.puc-rio.br> References: <20030426104039.A8053@sucuri.mat.puc-rio.br> Mime-Version: 1.0 Content-Type: text/plain; charset=us-ascii Content-Disposition: inline User-Agent: Mutt/1.2.5i In-Reply-To: <20030426104039.A8053@sucuri.mat.puc-rio.br>; from nicolau@sucuri.mat.puc-rio.br on Sat, Apr 26, 2003 at 10:40:39AM -0300 Sender: owner-obm-l@sucuri.mat.puc-rio.br Precedence: bulk Reply-To: obm-l@mat.puc-rio.br On Sat, Apr 26, 2003 at 10:40:39AM -0300, Nicolau C. Saldanha wrote: > S(n) = (1 - 2(2n+1)^2 + (2n+1)^4)/64 = (((2n+1)^2 - 1)/8)^2 > = ((4n^2 - 4n)/8)^2 = (n(n-1)/2)^2 Oops... deveria ser: S(n) = (1 - 2(2n+1)^2 + (2n+1)^4)/64 = (((2n+1)^2 - 1)/8)^2 = ((4n^2 + 4n)/8)^2 = (n(n+1)/2)^2 Obrigado ao Morgado por apontar este erro. []s, N. ========================================================================= Instruções para entrar na lista, sair da lista e usar a lista em http://www.mat.puc-rio.br/~nicolau/olimp/obm-l.html ========================================================================= From owner-obm-l@sucuri.mat.puc-rio.br Sat Apr 26 14:15:55 2003 Return-Path: Received: (from majordom@localhost) by sucuri.mat.puc-rio.br (8.9.3/8.9.3) id OAA13902 for obm-l-MTTP; Sat, 26 Apr 2003 14:13:23 -0300 Received: from artemis.opendf.com.br (artemis.opengate.com.br [200.181.71.14]) by sucuri.mat.puc-rio.br (8.9.3/8.9.3) with ESMTP id OAA13897 for ; Sat, 26 Apr 2003 14:13:20 -0300 Received: from localhost (localhost [127.0.0.1]) by artemis.opendf.com.br (Postfix) with ESMTP id F2ADF2BECE for ; Sat, 26 Apr 2003 14:13:57 -0300 (BRT) Received: from artemis.opendf.com.br ([127.0.0.1]) by localhost (artemis.opengate.com.br [127.0.0.1:10024]) (amavisd-new) with ESMTP id 27310-01 for ; Sat, 26 Apr 2003 14:13:57 -0300 (BRT) Received: from computer (200-181-089-199.bsace7001.dsl.brasiltelecom.net.br [200.181.89.199]) by artemis.opendf.com.br (Postfix) with ESMTP id 6DD472BEC8 for ; Sat, 26 Apr 2003 14:13:56 -0300 (BRT) From: "Artur Costa Steiner" To: Subject: RE: [obm-l] derivada Date: Sat, 26 Apr 2003 14:12:49 -0300 Organization: Steiner Consultoria LTDA Message-ID: <002b01c30c17$10aead10$9865fea9@computer> MIME-Version: 1.0 Content-Type: text/plain; charset="iso-8859-1" X-Priority: 3 (Normal) X-MSMail-Priority: Normal X-Mailer: Microsoft Outlook, Build 10.0.2627 Importance: Normal X-MimeOLE: Produced By Microsoft MimeOLE V6.00.2800.1165 In-Reply-To: X-Virus-Scanned: by amavisd-new Content-Transfer-Encoding: 8bit X-MIME-Autoconverted: from quoted-printable to 8bit by sucuri.mat.puc-rio.br id OAA13898 Sender: owner-obm-l@sucuri.mat.puc-rio.br Precedence: bulk Reply-To: obm-l@mat.puc-rio.br > >Oi Pessoal, voces poderiam me ajuda com uma pesquisa que estou fazendo >sobre a derivada de Caratheodory. >Poderiam me indicando algum livro, site ou artigo que contenha esse >assunto. >Se não for pedir demais, poderiam me indicar algumas aplicações para essa >derivada. > >Agradeço desde ja a conpreencao de todos. > >Fábio [Artur Costa Steiner] Oi Fabio Sobre Caratheodory, eu sei que existe um afuncao de Ccaratheodory, que aparece na teoria de medidas e estah ligada a sigmas - algebras, conjuntos de Borel e assuntos correlatos. Ha ainda um um Teorema de Caratheodory, ligado tambem a teoria de medidas. Especificamente sobre derivada de Caratheodory, eu nunca ouvi falar. Correlato a derivadas, eu conheco um teorema dele que diz "Sendo f uma funcao (real, de variavel real) diferenciavel em um ponto a de um intervalo I, existe entao uma funcao g, continua em a, tal que f(x) - f(a) = (x-a) g(x) para todo x pertencente a I e tal que g(a) = f'(a). Este teorema da uma forma bem interessante para provar a Regra da Cadeia, no caso unidimensional, e tambem para a famoasa formula da derivada da inversa de uma funcao estritamente monotonica e difrenciavel em um intervalo. Com relacao a este ultimo assunto, eu conheco o livro Introduction to real Analysis, de Robert Bartle e Donald Sherbert. Artur ========================================================================= Instruções para entrar na lista, sair da lista e usar a lista em http://www.mat.puc-rio.br/~nicolau/olimp/obm-l.html ========================================================================= From owner-obm-l@sucuri.mat.puc-rio.br Sat Apr 26 15:16:43 2003 Return-Path: Received: (from majordom@localhost) by sucuri.mat.puc-rio.br (8.9.3/8.9.3) id PAA15726 for obm-l-MTTP; Sat, 26 Apr 2003 15:13:47 -0300 Received: from artemis.opendf.com.br (artemis.opengate.com.br [200.181.71.14]) by sucuri.mat.puc-rio.br (8.9.3/8.9.3) with ESMTP id PAA15721 for ; Sat, 26 Apr 2003 15:13:44 -0300 Received: from localhost (localhost [127.0.0.1]) by artemis.opendf.com.br (Postfix) with ESMTP id 3D1FD2BEDA for ; Sat, 26 Apr 2003 15:14:22 -0300 (BRT) Received: from artemis.opendf.com.br ([127.0.0.1]) by localhost (artemis.opengate.com.br [127.0.0.1:10024]) (amavisd-new) with ESMTP id 27899-04 for ; Sat, 26 Apr 2003 15:14:21 -0300 (BRT) Received: from computer (200-181-089-199.bsace7001.dsl.brasiltelecom.net.br [200.181.89.199]) by artemis.opendf.com.br (Postfix) with ESMTP id 5C8AA2BED7 for ; Sat, 26 Apr 2003 15:14:20 -0300 (BRT) From: "Artur Costa Steiner" To: Subject: RE: [obm-l] Provar continuidade Date: Sat, 26 Apr 2003 15:13:12 -0300 Organization: Steiner Consultoria LTDA Message-ID: MIME-Version: 1.0 Content-Type: multipart/mixed; boundary="----=_NextPart_000_003F_01C30C06.5AE8F720" X-Priority: 3 (Normal) X-MSMail-Priority: Normal X-Mailer: Microsoft Outlook, Build 10.0.2627 Importance: Normal X-MimeOLE: Produced By Microsoft MimeOLE V6.00.2800.1165 In-Reply-To: <3EAAB4BF.4080002@niski.com> X-MS-TNEF-Correlator: 000000005FC3F71D8B4BB941A8AA63D06F261C3604492B00 X-Virus-Scanned: by amavisd-new Sender: owner-obm-l@sucuri.mat.puc-rio.br Precedence: bulk Reply-To: obm-l@mat.puc-rio.br This is a multi-part message in MIME format. ------=_NextPart_000_003F_01C30C06.5AE8F720 Content-Type: text/plain; charset="iso-8859-1" Content-Transfer-Encoding: quoted-printable >Sent: Saturday, April 26, 2003 1:33 PM >To: obm-l@mat.puc-rio.br >Subject: [obm-l] Provar continuidade > >Ola pessoal... este problema esta no meu livro mas nao tem >resposta...gostaria que o pessoal conferisse se procedi corretamente. >Muito obrigado. > >provar que f(x) =3D (x)^(1/n) =E9 continua. > >Demonstra=E7ao : >Dado Eps > 0 existe um intervalo aberto I , p pertencente a I , tal que > >x pertence a I =3D> f(p) - Eps < f(x) < f(p) + Eps [Artur Costa Steiner]=20 Ei!! Isto eh a definicao de continuidade em p! Eh justamente o que vc = quer provar! Eh como provar que x eh maior que zero partindo do procipio que = x eh maior que zero.=20 Suponhamos p>0 e que |x-p| < d, d

0, temos que |x^1/n - = p^1/n| < (p+d)^(1/n) - p^1/n =3D p^(1/n)(1+d/p)^(1/n). Pela desigualdade de = Bernouille, vem |x^1/n - p^1/n| < (p^1/n)(1+d/pn) - p^1/n =3D d/n p^(1/n -1). Logo, = dado Eps>0, para tornarmos |x^1/n - p^1/n| 0 podemos portanto achar um d que satIsfaca a condicao de continuidae em p. Logo, f em continua em todo p >=3D0. = Observe que esta funcao, normalmente, so eh definida para x>=3D0.=20 Uma outra forma mais facil de provarmos isto eh verificando que g(x) =3D = x eh obviamente continua e que , pelos teoremas sobre funcoes continuas, para todo n inteiro g^n tambem eh. Como g^n eh estritamente crescente , sua inversa f =3D g^(1/n) eh continua. Vc desenvolveu um raciocinio certo, so se perdeu um pouco na logica. Artur =20 > >(p)^(1/n) - Eps < (x)^(1/n) < (p)^(1/n) + Eps >((p)^(1/n) - Eps)^n < x < ((p)^(1/n) + Eps)^n >tomando-se I =3D ]((p)^(1/n) - Eps)^n, ((p)^(1/n) + Eps)^n[ , p >pertencente a I > >x pertence a I =3D> f(p) - Eps < f(x) < f(p) + Eps >logo f(x) =3D (x)^1/n =E9 continua em todo p real. > >Mais uma vez, obrigado >-- >[about him:] > It is rare to find learned men who are clean, do not stink and have a >sense of humour. >-Gottfried Whilhem Leibniz > >=3D=3D=3D=3D=3D=3D=3D=3D=3D=3D=3D=3D=3D=3D=3D=3D=3D=3D=3D=3D=3D=3D=3D=3D= =3D=3D=3D=3D=3D=3D=3D=3D=3D=3D=3D=3D=3D=3D=3D=3D=3D=3D=3D=3D=3D=3D=3D=3D=3D= =3D=3D=3D=3D=3D=3D=3D=3D=3D=3D=3D=3D=3D=3D=3D=3D=3D=3D=3D=3D=3D=3D=3D=3D >Instru=E7=F5es para entrar na lista, sair da lista e usar a lista em >http://www.mat.puc-rio.br/~nicolau/olimp/obm-l.html >=3D=3D=3D=3D=3D=3D=3D=3D=3D=3D=3D=3D=3D=3D=3D=3D=3D=3D=3D=3D=3D=3D=3D=3D= =3D=3D=3D=3D=3D=3D=3D=3D=3D=3D=3D=3D=3D=3D=3D=3D=3D=3D=3D=3D=3D=3D=3D=3D=3D= =3D=3D=3D=3D=3D=3D=3D=3D=3D=3D=3D=3D=3D=3D=3D=3D=3D=3D=3D=3D=3D=3D=3D=3D ------=_NextPart_000_003F_01C30C06.5AE8F720 Content-Type: application/ms-tnef; name="winmail.dat" Content-Transfer-Encoding: base64 Content-Disposition: attachment; filename="winmail.dat" eJ8+Ig0SAQaQCAAEAAAAAAABAAEAAQeQBgAIAAAA5AQAAAAAAADoAAEIgAcAGAAAAElQTS5NaWNy b3NvZnQgTWFpbC5Ob3RlADEIAQ2ABAACAAAAAgACAAEGAAcAAQAAAAAAAAEGgAMADgAAANMHBAAa AA8ADQAAAAYAGgEBA5AGAKgLAAAuAAAACwACAAEAAAALACMAAAAAAAMAJgAAAAAACwApAAAAAAAD AC4AAAAAAAIBMQABAAAAGAAAAAAAAABfw/cdi0u5QaiqY9BvJhw2xEgrAAMANgAAAAAAHgBwAAEA AAAcAAAAW29ibS1sXSBQcm92YXIgY29udGludWlkYWRlAAIBcQABAAAAFgAAAAHDDB9Aq1Vfj26T +ks2lVGT7IbkJ6kAAAIBHQwBAAAAGwAAAFNNVFA6QVJUVVJfU1RFSU5FUkBVU0EuTkVUAAALAAEO AAAAAEAABg4AXmZ4HwzDAQIBCg4BAAAAGAAAAAAAAABfw/cdi0u5QaiqY9BvJhw2woAAAAMAFA4A AAAACwAfDgEAAAAeACgOAQAAACoAAAAwMDAwMDAwNAFhcnR1cl9zdGVpbmVyQHVzYS5uZXQBTmV0 YWRkcmVzcwAAAB4AKQ4BAAAAKgAAADAwMDAwMDA0AWFydHVyX3N0ZWluZXJAdXNhLm5ldAFOZXRh ZGRyZXNzAAAAAgEJEAEAAACWBgAAkgYAAGILAABMWkZ1jqjYQwMACgByY3BnMTI14jIDQ3RleAVB AQMB908KgAKkA+MCAGNoCsBz8GV0MCAHEwKAD/MAUH8EVghVB7IRxQ5RAwEQxzL3BgAGwxHFMwRG EMkS2xHT2wjvCfc7GL8OMDURwgxgzmMAUAsJAWQzNhFQC6asID4GYAIwOgYBdAhw0GRheSwRYHAF EAMgDDI2HoAB0DAzIDFkOjMfcFBNCqIKgD4EVG8d4G9ibS1sokAAwHQucBtwLQUQEG8uYnIgBVN1 YtZqBZAd0Vsgs10f0ANgnnYKwQWgAjALgHVpHlCXAQAgBSAFTwtgIHAHkNRzbwdALiYwIAeQDrB3 JbADYAJgZQDAJmIloG4EbyAHgHUgbGl2ywNgJ8BhBCBuYSewDrDGbSAFGMBzcG8nYSYx5mcp0gch IHEKUCCgJbZ9I9JmBnEEECagLDEmwWP1CYBpI9FyGMABkAeAAjDUZS4gBU0kQHQnsCCw/QUQZyRw IbAkrCbBI6Iq0oBmKHgpID0gMOFgXigxL24xAAMwJ9xlOSPWKgAkrEQnEAIgqSaAcmEx8TcowToz RpEvASBFcAQgPiARUMcOwAQAJpF1bSALgA6wdnIjoBhQIAGgBJAugUn6IB6AcCWxACAJ8CzALaG/ NzA3owGQAyAq0SSseDgHrTjDPTXAMNBwMQAtNXNuPDDEPFI7wSs1ciAEW+8HEB4hEsEnYlMOsAuA BJDDI1AgBEVpISE3oCaA+SewZWg4wQEBC4AN4CjBrwEAI9smYDaAcD/QRUBQPGp1J2EtgysBKtJ2 Y/8qwgXAMCRCcwWgBGBERSrD3zqAQEEAwCGgMHR6BJArEf8KwCQRNVE1USySBSAhoEW/30bFJlAg BCJgKcBuEPAEYBkEIHA+NeEqw3x4LZRwfDxBZB6AZDw38MxMbypAN9EtZDxBOoCRPFBwK2QmUFBl JZFzQJkloGZ1OGAowSnAdP8x8QBwSDAloQrAJ4E2cEcB/wQgNCBIMAIgC3AEICahTiH/SxEDYB6x CYAkcQQgLxAokf9Qxk/wBAA5ETPBBCAq0juw500wMWY78HBeMZFNUlXj/1W4O7BN0FU+EsFFISey BtD/KEFPAQeQRAJZgkBQPvAu8P0kEHZHI0BxS0BUOkvQVmr/TBJXX1YBMSBV0DGDMYBN0F4vO8Ax dE33AJBnMqBs+0HjQTFCBJEIYAMQJwAegP52QjFcT1ThVeJfJV37X2D3A6BexDvgMV/xTMQkYTVj f1tiUGMugASgCsBK8mJ/Zb01kWIogCdxB5AYMWgEkHVK8jBMMTwxIAOgNYEv/ihlaV/yZ2Rmtk+R AQBK8/8YYQBwLoEA0BDxNmJNQCrS9nMeED/wZgDQJaAloCPh7yzgQP9CFGYWZkIiMkZCIiNswzfw Pj0wJlBPYv8RIDbgS3QnU080HoAnoGfw/wdALYMegCxQQCNAlE8BUGPueHPzIAQgBFUnIQhgNBH/ MMB14kZCBCBvwQMRQTEwJP9K8jYhQCNiMAaBcHFHkirS7mcw5EjTILB2BzBDFHLI/yrDN9FOIEsB DrAFsCcRBCDvJfAh0DCxT0FvB5EyRlQhL2dVNVEDoDaiaShBZ17/A6AtYTdQNoBAQCZQWKODUv9A QSZxBRBC9gUAafE4gx6A/nNzIQuAfBFvcHJhMSCDUO8xhEBBMk0gBFZD4FLhCfD9WtBsYjAn8DZx UTNIMAMA/yewLMA3cYZxJ7AsYlnxilTfKcAbcIKRJaAYUGdwcS3V/z30eEskrDuxVUc8FDFIXUL/ MWY9OZAwkE0xYFYTXTKSLd+UgSAFLoADgS8QLSwxN7B9MRFdk4+DYB6AlS+DYFt/N8IvtjgsOa86 vzvPPN8+/41hJ7Aw2VXyMftzSVPhJiC9JKxNUZI2cCWgYjB6HoDTLrYgBS0tIAVbAaB5UWQgaAdw Ol0gBTHQSf8FQFGhNCCAsS6BQLFNQCcAjwrAPvBNQC2BIHdoNyH9gLFjqSGY8YKCT7AsUCQR/ms3 MKjxEPB0kTQwIBSJ4XUsMW9ycGg2cAhhLdYtzkdPsAAwUpIgV6cQajFVNoBMPtBiAwB6JKw9v7Cv sb+yz7PftCIgBUkz8ucMcDRRMfBmNQeRUGMdsfeoUY0jNiFhhnELcAXATwG/t6NLYULQCsG4dykW aAJAoHA6Ly93uuAuIRyML35A0QbwYXUvBvCtB3BwvJAgwi66cG0YoF+wT76/v8/A37SPfcMQAAAe AEIQAQAAAB0AAAA8M0VBQUI0QkYuNDA4MDAwMkBuaXNraS5jb20+AAAAAAMAkhABAAAAAgEUOgEA AAAQAAAAk/rWr4KsXEWM1GGmyCxBlAMA3j+vbwAAAwAJWQEAAAADAEBlAAAAAAsAE4AIIAYAAAAA AMAAAAAAAABGAAAAAAOFAAAAAAAAAwAVgAggBgAAAAAAwAAAAAAAAEYAAAAAEIUAAAAAAAADABuA CCAGAAAAAADAAAAAAAAARgAAAABShQAA45ABAAMAIoAIIAYAAAAAAMAAAAAAAABGAAAAAAGFAAAA AAAAQAAjgAggBgAAAAAAwAAAAAAAAEYAAAAAYIUAAADQiMMQAAAAHgBBgAggBgAAAAAAwAAAAAAA AEYAAAAAVIUAAAEAAAAFAAAAMTAuMAAAAAALAEKACCAGAAAAAADAAAAAAAAARgAAAAAGhQAAAAAA AAsARoAIIAYAAAAAAMAAAAAAAABGAAAAAA6FAAAAAAAAAwBJgAggBgAAAAAAwAAAAAAAAEYAAAAA GIUAAAAAAAALAF6ACCAGAAAAAADAAAAAAAAARgAAAACChQAAAQAAAAIB+A8BAAAAEAAAAF/D9x2L S7lBqKpj0G8mHDYCAfoPAQAAABAAAABfw/cdi0u5QaiqY9BvJhw2AgH7DwEAAACEAAAAAAAAADih uxAF5RAaobsIACsqVsIAAG1zcHN0LmRsbAAAAAAATklUQfm/uAEAqgA32W4AAABDOlxEb2N1bWVu dHMgYW5kIFNldHRpbmdzXEFydHVyXEFwcGxpY2F0aW9uIERhdGFcTWljcm9zb2Z0XE91dGxvb2tc T3V0bG9vay5wc3QAAwD+DwUAAAADAA00/TcCAAIBFDQBAAAAEAAAAE5JVEH5v7gBAKoAN9luAAAC AX8AAQAAADEAAAAwMDAwMDAwMDVGQzNGNzFEOEI0QkI5NDFBOEFBNjNEMDZGMjYxQzM2MDQ0OTJC MDAAAAAAAwAGEMh3YzoDAAcQRAcAAAMAEBABAAAAAwAREAAAAAAeAAgQAQAAAGUAAABTRU5UOlNB VFVSREFZLEFQUklMMjYsMjAwMzE6MzNQTVRPOk9CTS1MQE1BVFBVQy1SSU9CUlNVQkpFQ1Q6T0JN LUxQUk9WQVJDT05USU5VSURBREVPTEFQRVNTT0FMRVNURVBSAAAAAKdW ------=_NextPart_000_003F_01C30C06.5AE8F720-- ========================================================================= Instruções para entrar na lista, sair da lista e usar a lista em http://www.mat.puc-rio.br/~nicolau/olimp/obm-l.html ========================================================================= From owner-obm-l@sucuri.mat.puc-rio.br Sat Apr 26 16:03:34 2003 Return-Path: Received: (from majordom@localhost) by sucuri.mat.puc-rio.br (8.9.3/8.9.3) id QAA16541 for obm-l-MTTP; Sat, 26 Apr 2003 16:00:38 -0300 Received: from itaqui.terra.com.br (itaqui.terra.com.br [200.176.3.19]) by sucuri.mat.puc-rio.br (8.9.3/8.9.3) with ESMTP id QAA16537 for ; Sat, 26 Apr 2003 16:00:35 -0300 Received: from gunga.terra.com.br (gunga.terra.com.br [200.176.3.45]) by itaqui.terra.com.br (Postfix) with ESMTP id 362A43BC565 for ; Sat, 26 Apr 2003 16:00:04 -0300 (BRT) Received: from niski.com (unknown [200.148.198.60]) (authenticated user fniski) by gunga.terra.com.br (Postfix) with ESMTP id 03C72128055 for ; Sat, 26 Apr 2003 16:00:03 -0300 (BRT) Message-ID: <3EAAD73A.2040708@niski.com> Date: Sat, 26 Apr 2003 12:00:10 -0700 From: niski User-Agent: Mozilla/5.0 (Windows; U; Windows NT 5.1; en-US; rv:1.0.2) Gecko/20030208 Netscape/7.02 X-Accept-Language: en-us, en MIME-Version: 1.0 To: obm-l@mat.puc-rio.br Subject: Re: [obm-l] Provar continuidade References: Content-Type: text/plain; charset=ISO-8859-1; format=flowed Content-Transfer-Encoding: 8bit Sender: owner-obm-l@sucuri.mat.puc-rio.br Precedence: bulk Reply-To: obm-l@mat.puc-rio.br >> >>provar que f(x) = (x)^(1/n) é continua. >> >>Demonstraçao : >>Dado Eps > 0 existe um intervalo aberto I , p pertencente a I , tal que >> >>x pertence a I => f(p) - Eps < f(x) < f(p) + Eps > > [Artur Costa Steiner] > Ei!! Isto eh a definicao de continuidade em p! Eh justamente o que vc quer > provar! Eh como provar que x eh maior que zero partindo do procipio que x eh > maior que zero. > Vc desenvolveu um raciocinio certo, so se perdeu um pouco na logica. > Artur Artur, entendo o que voce quer dizer..mas é o seguinte.. É de comum acordo que se achar um intervalo aberto I, com p neste intervalo tal que x pertence a I => f(p) - Eps < f(x) < f(p) + Eps Certo? Otimo...agora vamos supor que eu vá para um "rascunho" e tente achar esse intervalo usando a hipotese! Note que "ninguem estará vendo" Então no rascunho : (p)^(1/n) - Eps < (x)^(1/n) < (p)^(1/n) + Eps ((p)^(1/n) - Eps)^n < x < ((p)^(1/n) + Eps)^n Pronto! achei o intervalo, agora continuando efetivamente a prova tomando-se I = ]((p)^(1/n) - Eps)^n, ((p)^(1/n) + Eps)^n[ , p pertencente a I x pertence a I => f(p) - Eps < f(x) < f(p) + Eps logo f(x) = (x)^1/n é continua em todo p real. Então para na mensagem não ficar muito "criptico" de onde saiu o intervalo, eu mostrei o raciocinio que utilizei para encontra-lo. E voce pode confirmar graficamente que esse intervalo esta correto. Feito estas observacoes, pergunto novamente se minha demonstracao continua errada. obrigado -- [about him:] It is rare to find learned men who are clean, do not stink and have a sense of humour. -Gottfried Whilhem Leibniz ========================================================================= Instruções para entrar na lista, sair da lista e usar a lista em http://www.mat.puc-rio.br/~nicolau/olimp/obm-l.html ========================================================================= From owner-obm-l@sucuri.mat.puc-rio.br Sat Apr 26 16:21:42 2003 Return-Path: Received: (from majordom@localhost) by sucuri.mat.puc-rio.br (8.9.3/8.9.3) id QAA16754 for obm-l-MTTP; Sat, 26 Apr 2003 16:18:41 -0300 Received: from ivoti.terra.com.br (ivoti.terra.com.br [200.176.3.20]) by sucuri.mat.puc-rio.br (8.9.3/8.9.3) with ESMTP id QAA16750 for ; Sat, 26 Apr 2003 16:18:37 -0300 Received: from itaim.terra.com.br (itaim.terra.com.br [200.176.3.76]) by ivoti.terra.com.br (Postfix) with ESMTP id 12C3F40872A for ; Sat, 26 Apr 2003 16:18:06 -0300 (BRT) Received: from niski.com (unknown [200.148.198.60]) (authenticated user fniski) by itaim.terra.com.br (Postfix) with ESMTP id E3E652E0072 for ; Sat, 26 Apr 2003 16:18:04 -0300 (BRT) Message-ID: <3EAADB73.1070401@niski.com> Date: Sat, 26 Apr 2003 12:18:11 -0700 From: niski User-Agent: Mozilla/5.0 (Windows; U; Windows NT 5.1; en-US; rv:1.0.2) Gecko/20030208 Netscape/7.02 X-Accept-Language: en-us, en MIME-Version: 1.0 To: obm-l@mat.puc-rio.br Subject: [obm-l] provar desiguldade.. Content-Type: text/plain; charset=ISO-8859-1; format=flowed Content-Transfer-Encoding: 8bit Sender: owner-obm-l@sucuri.mat.puc-rio.br Precedence: bulk Reply-To: obm-l@mat.puc-rio.br Por favor pessoal, me ajudem nesta questão, obrigado: Seja f(x) = x + 1/x Prove que |f(x) - f(1)| <= 3|x-1| , para x > 1/2 Demonstracao |x+ (1/x) - 2 | <= 3|x-1| |(x^2 -2x +1)/x| <= 3|x-1| (|x-1||x-1|)/x <= 3|x-1| |x-1|/x <= 3 |x-1| <= 3x Se 1/2 < x < 1 -x+1 < 3x 1 < 4x De fato pois o menor valor que 4x pode assumir é maior do que 4(1/2) = 2. Se x >= 1 |x-1| <= 3x x-1 < 3x -1 < 4x De fato pois o menor valor que 4x pode assumir é 4. Esta certa esta demonstração?! Obrigado. -- [about him:] It is rare to find learned men who are clean, do not stink and have a sense of humour. -Gottfried Whilhem Leibniz ========================================================================= Instruções para entrar na lista, sair da lista e usar a lista em http://www.mat.puc-rio.br/~nicolau/olimp/obm-l.html ========================================================================= From owner-obm-l@sucuri.mat.puc-rio.br Sat Apr 26 17:01:19 2003 Return-Path: Received: (from majordom@localhost) by sucuri.mat.puc-rio.br (8.9.3/8.9.3) id QAA18261 for obm-l-MTTP; Sat, 26 Apr 2003 16:58:38 -0300 Received: from hotmail.com (f7.law15.hotmail.com [64.4.23.7]) by sucuri.mat.puc-rio.br (8.9.3/8.9.3) with ESMTP id QAA18257 for ; Sat, 26 Apr 2003 16:58:34 -0300 Received: from mail pickup service by hotmail.com with Microsoft SMTPSVC; Sat, 26 Apr 2003 12:58:02 -0700 Received: from 200.174.69.242 by lw15fd.law15.hotmail.msn.com with HTTP; Sat, 26 Apr 2003 19:58:02 GMT X-Originating-IP: [200.174.69.242] X-Originating-Email: [piuwee@hotmail.com] From: "Felipe Marinho" To: obm-l@mat.puc-rio.br Subject: [obm-l] Espaco Vetorial Date: Sat, 26 Apr 2003 15:58:02 -0400 Mime-Version: 1.0 Content-Type: text/plain; charset=iso-8859-1; format=flowed Message-ID: X-OriginalArrivalTime: 26 Apr 2003 19:58:02.0667 (UTC) FILETIME=[251507B0:01C30C2E] Sender: owner-obm-l@sucuri.mat.puc-rio.br Precedence: bulk Reply-To: obm-l@mat.puc-rio.br Bom pessoal, Estou voltando a lista depois de um tempo afastado. Gostaria antes de tudo de mandar minhas saudações a vocês todos. :) Agora o lance é o seguinte: Um Espaço Vetorial pode ser formado por 2 vetores distintos ? Bem pessoal, jah venho pensando nessa questao ha tempos e nao consigo chegar a uma conclusao para o mesmo. Será que alguem poderia me dar uma ajuda ? Se for possivel ou nao.. me mostrando a prova ?! Agradeço desde já qualquer tipo de ajuda, E muito Obrigado a todos. Felipe Marinho _________________________________________________________________ MSN Hotmail, o maior webmail do Brasil. http://www.hotmail.com ========================================================================= Instruções para entrar na lista, sair da lista e usar a lista em http://www.mat.puc-rio.br/~nicolau/olimp/obm-l.html ========================================================================= From owner-obm-l@sucuri.mat.puc-rio.br Sat Apr 26 17:39:54 2003 Return-Path: Received: (from majordom@localhost) by sucuri.mat.puc-rio.br (8.9.3/8.9.3) id RAA19032 for obm-l-MTTP; Sat, 26 Apr 2003 17:37:06 -0300 Received: from sidney4.bol.com.br (sidney4.bol.com.br [200.221.24.120]) by sucuri.mat.puc-rio.br (8.9.3/8.9.3) with ESMTP id RAA19012 for ; Sat, 26 Apr 2003 17:36:54 -0300 Received: from bol.com.br (200.221.24.128) by sidney4.bol.com.br (5.1.071) id 3E990A6600232C8C for obm-l@mat.puc-rio.br; Sat, 26 Apr 2003 17:36:24 -0300 Date: Sat, 26 Apr 2003 17:36:24 -0300 Message-Id: Subject: [obm-l] =?iso-8859-1?q?s=E9rie?= MIME-Version: 1.0 Content-Type: text/plain;charset="iso-8859-1" From: "marcio.lis" To: obm-l@mat.puc-rio.br X-XaM3-API-Version: 2.4 R3 ( B4 ) X-SenderIP: 200.151.156.249 Content-Transfer-Encoding: 8bit X-MIME-Autoconverted: from quoted-printable to 8bit by sucuri.mat.puc-rio.br id RAA19024 Sender: owner-obm-l@sucuri.mat.puc-rio.br Precedence: bulk Reply-To: obm-l@mat.puc-rio.br p/ quanto converge a série 1/n^2????salvo engano o gugu me falou uma vez sobre esse valor mas eu não lembro __________________________________________________________________________ Seleção de Softwares UOL. 10 softwares escolhidos pelo UOL para você e sua família. http://www.uol.com.br/selecao ========================================================================= Instruções para entrar na lista, sair da lista e usar a lista em http://www.mat.puc-rio.br/~nicolau/olimp/obm-l.html ========================================================================= From owner-obm-l@sucuri.mat.puc-rio.br Sat Apr 26 17:46:04 2003 Return-Path: Received: (from majordom@localhost) by sucuri.mat.puc-rio.br (8.9.3/8.9.3) id RAA19182 for obm-l-MTTP; Sat, 26 Apr 2003 17:43:28 -0300 Received: from hotmail.com (f68.law15.hotmail.com [64.4.23.68]) by sucuri.mat.puc-rio.br (8.9.3/8.9.3) with ESMTP id RAA19178 for ; Sat, 26 Apr 2003 17:43:24 -0300 Received: from mail pickup service by hotmail.com with Microsoft SMTPSVC; Sat, 26 Apr 2003 13:42:53 -0700 Received: from 200.174.69.242 by lw15fd.law15.hotmail.msn.com with HTTP; Sat, 26 Apr 2003 20:42:53 GMT X-Originating-IP: [200.174.69.242] X-Originating-Email: [piuwee@hotmail.com] From: "Felipe Marinho" To: obm-l@mat.puc-rio.br Subject: [obm-l] Espaco Vetorial: Corrigindo Date: Sat, 26 Apr 2003 16:42:53 -0400 Mime-Version: 1.0 Content-Type: text/plain; charset=iso-8859-1; format=flowed Message-ID: X-OriginalArrivalTime: 26 Apr 2003 20:42:53.0770 (UTC) FILETIME=[691AC2A0:01C30C34] Sender: owner-obm-l@sucuri.mat.puc-rio.br Precedence: bulk Reply-To: obm-l@mat.puc-rio.br Bom pessoal, Estou voltando a lista depois de um tempo afastado. Gostaria antes de tudo de mandar minhas saudações a vocês todos. :) Agora o lance é o seguinte: Um Espaço Vetorial pode ser formado por EXATAMENTE 2 vetores distintos ? Bem pessoal, jah venho pensando nessa questao ha tempos e nao consigo chegar a uma conclusao para o mesmo. Será que alguem poderia me dar uma ajuda ? Se for possivel ou nao.. me mostrando a prova ?! Agradeço desde já qualquer tipo de ajuda, E muito Obrigado a todos. Felipe Marinho > >========================================================================= >Instruções para entrar na lista, sair da lista e usar a lista em >http://www.mat.puc-rio.br/~nicolau/olimp/obm-l.html >========================================================================= _________________________________________________________________ MSN Messenger: converse com os seus amigos online. http://messenger.msn.com.br ========================================================================= Instruções para entrar na lista, sair da lista e usar a lista em http://www.mat.puc-rio.br/~nicolau/olimp/obm-l.html ========================================================================= From owner-obm-l@sucuri.mat.puc-rio.br Sat Apr 26 17:47:12 2003 Return-Path: Received: (from majordom@localhost) by sucuri.mat.puc-rio.br (8.9.3/8.9.3) id RAA19195 for obm-l-MTTP; Sat, 26 Apr 2003 17:44:40 -0300 Received: from sidney4.bol.com.br (sidney4.bol.com.br [200.221.24.120]) by sucuri.mat.puc-rio.br (8.9.3/8.9.3) with ESMTP id RAA19191 for ; Sat, 26 Apr 2003 17:44:37 -0300 Received: from bol.com.br (200.221.24.128) by sidney4.bol.com.br (5.1.071) id 3E990A6600232F5D for obm-l@mat.puc-rio.br; Sat, 26 Apr 2003 17:44:06 -0300 Date: Sat, 26 Apr 2003 17:44:06 -0300 Message-Id: Subject: [obm-l] sequencia MIME-Version: 1.0 Content-Type: text/plain;charset="iso-8859-1" From: "marcio.lis" To: obm-l@mat.puc-rio.br X-XaM3-API-Version: 2.4 R3 ( B4 ) X-SenderIP: 200.151.156.249 Content-Transfer-Encoding: 8bit X-MIME-Autoconverted: from quoted-printable to 8bit by sucuri.mat.puc-rio.br id RAA19192 Sender: owner-obm-l@sucuri.mat.puc-rio.br Precedence: bulk Reply-To: obm-l@mat.puc-rio.br determine todas as sequencias de numeros reais a1,a2,....,a1995 tal que:2(an-(n-1))^1/2>=a(n+1)- (n-1),p/ n=1,2,.........,1994.e 2(a1995-1994)^1/2.=a1+1 essa questão e da asian pacific de 1995 __________________________________________________________________________ Seleção de Softwares UOL. 10 softwares escolhidos pelo UOL para você e sua família. http://www.uol.com.br/selecao ========================================================================= Instruções para entrar na lista, sair da lista e usar a lista em http://www.mat.puc-rio.br/~nicolau/olimp/obm-l.html ========================================================================= From owner-obm-l@sucuri.mat.puc-rio.br Sat Apr 26 18:38:57 2003 Return-Path: Received: (from majordom@localhost) by sucuri.mat.puc-rio.br (8.9.3/8.9.3) id SAA21036 for obm-l-MTTP; Sat, 26 Apr 2003 18:36:22 -0300 Received: from paiol.terra.com.br (paiol.terra.com.br [200.176.3.18]) by sucuri.mat.puc-rio.br (8.9.3/8.9.3) with ESMTP id SAA21032 for ; Sat, 26 Apr 2003 18:36:18 -0300 Received: from botucatu.terra.com.br (botucatu.terra.com.br [200.176.3.78]) by paiol.terra.com.br (Postfix) with ESMTP id 837C98872C for ; Sat, 26 Apr 2003 18:35:48 -0300 (BRT) Received: from [200.177.180.148] (dl-nas3-sao-C8B1B494.p001.terra.com.br [200.177.180.148]) by botucatu.terra.com.br (Postfix) with ESMTP id BDD7F29C078 for ; Sat, 26 Apr 2003 18:35:47 -0300 (BRT) User-Agent: Microsoft-Outlook-Express-Macintosh-Edition/5.02.2022 Date: Sat, 26 Apr 2003 18:37:15 -0300 Subject: Re: [obm-l] Espaco Vetorial From: Claudio Buffara To: Message-ID: In-Reply-To: Mime-version: 1.0 Content-type: text/plain; charset="ISO-8859-1" Content-Transfer-Encoding: 8bit X-MIME-Autoconverted: from quoted-printable to 8bit by sucuri.mat.puc-rio.br id SAA21033 Sender: owner-obm-l@sucuri.mat.puc-rio.br Precedence: bulk Reply-To: obm-l@mat.puc-rio.br on 26.04.03 16:58, Felipe Marinho at piuwee@hotmail.com wrote: > Bom pessoal, > > Estou voltando a lista depois de um tempo afastado. > Gostaria antes de tudo de mandar minhas saudações a vocês todos. :) > > Agora o lance é o seguinte: > > Um Espaço Vetorial pode ser formado por 2 vetores distintos ? > > > Bem pessoal, jah venho pensando nessa questao ha tempos e nao consigo chegar > a uma conclusao para o mesmo. Será que alguem poderia me dar uma ajuda ? Se > for possivel ou nao.. me mostrando a prova ?! > > > Agradeço desde já qualquer tipo de ajuda, > E muito Obrigado a todos. > > Felipe Marinho > > Sim, apesar de ser um caso meio patologico. Tome V = {0, u} sobre o corpo Z/(2), onde u eh um vetor tal que: u + u = 0 Naturalmente, 0 eh o vetor nulo: 0 + 0 = 0 e 0 + u = u + 0 = 0. Alem disso, tem-se 1*v = v*1 = v e 0*v = v*0 = 0 para todo v em V, onde o 1 e os dois primeiros 0's sao de Z/(2) e o terceiro 0 de V. Eh facil, mas meio chato, verificar que V e Z/(2) possuem todas as propriedades que definem um espaco vetorial sobre um corpo. Um abraco, Claudio. ========================================================================= Instruções para entrar na lista, sair da lista e usar a lista em http://www.mat.puc-rio.br/~nicolau/olimp/obm-l.html ========================================================================= From owner-obm-l@sucuri.mat.puc-rio.br Sat Apr 26 18:39:02 2003 Return-Path: Received: (from majordom@localhost) by sucuri.mat.puc-rio.br (8.9.3/8.9.3) id SAA21058 for obm-l-MTTP; Sat, 26 Apr 2003 18:36:31 -0300 Received: from paiol.terra.com.br (paiol.terra.com.br [200.176.3.18]) by sucuri.mat.puc-rio.br (8.9.3/8.9.3) with ESMTP id SAA21041 for ; Sat, 26 Apr 2003 18:36:26 -0300 Received: from gunga.terra.com.br (gunga.terra.com.br [200.176.3.45]) by paiol.terra.com.br (Postfix) with ESMTP id 15BB78874C for ; Sat, 26 Apr 2003 18:35:56 -0300 (BRT) Received: from [200.177.180.148] (dl-nas3-sao-C8B1B494.p001.terra.com.br [200.177.180.148]) by gunga.terra.com.br (Postfix) with ESMTP id 413C212805C for ; Sat, 26 Apr 2003 18:35:55 -0300 (BRT) User-Agent: Microsoft-Outlook-Express-Macintosh-Edition/5.02.2022 Date: Sat, 26 Apr 2003 18:38:03 -0300 Subject: Re: [obm-l] provar desiguldade.. From: Claudio Buffara To: Message-ID: In-Reply-To: <3EAADB73.1070401@niski.com> Mime-version: 1.0 Content-type: text/plain; charset="ISO-8859-1" Content-Transfer-Encoding: 8bit X-MIME-Autoconverted: from quoted-printable to 8bit by sucuri.mat.puc-rio.br id SAA21051 Sender: owner-obm-l@sucuri.mat.puc-rio.br Precedence: bulk Reply-To: obm-l@mat.puc-rio.br on 26.04.03 16:18, niski at fabio@niski.com wrote: > Por favor pessoal, me ajudem nesta questão, obrigado: > > Seja f(x) = x + 1/x > > Prove que > > |f(x) - f(1)| <= 3|x-1| , para x > 1/2 > > Demonstracao > |x+ (1/x) - 2 | <= 3|x-1| > |(x^2 -2x +1)/x| <= 3|x-1| > (|x-1||x-1|)/x <= 3|x-1| > |x-1|/x <= 3 > |x-1| <= 3x > > Se 1/2 < x < 1 > > -x+1 < 3x > 1 < 4x > > De fato pois o menor valor que 4x pode assumir é maior do que 4(1/2) = 2. > > Se x >= 1 > > |x-1| <= 3x > x-1 < 3x > -1 < 4x > > De fato pois o menor valor que 4x pode assumir é 4. > > Esta certa esta demonstração?! > Obrigado. > > > Oi, Niski: O seu raciocinio esta certinho, mas falta uma certa formalizacao. Para isso, o mais facil eh definir F:(1/2,+infinito) --> R por: F(x) = |x + 1/x - 2| - 3|x - 1| Assim, a ideia eh mostrar que F(x) <= 0 para todo x > 1/2. Como voce mesmo viu acima, |x + 1/x - 2| = |x^2 - 2x + 1|/|x| Mas o numerador eh um quadrado perfeito, logo >= 0 e o denominador tambem eh positivo, dado o dominio de F. Assim, teremos: |x + 1/x - 2| = (x - 1)^2/x Agora, dividimos em 3 casos: A) 1/2 < x < 1 ==> 3|x - 1| = 3(1 - x) ==> F(x) = (x - 1)^2/x + 3(x - 1) ==> F(x) = (x - 1)*(4 - 1/x) Como 1/2 < x < 1, teremos x - 1 < 0 e 4 - 1/x > 0 ==> F(x) < 0 B) x = 1 ==> F(x) = 0 C) x > 1 ==> 3|x - 1| = 3(x - 1) ==> F(x) = (x - 1)^2/x - 3(x - 1) ==> F(x) = - (x - 1)(2 + 1/x) Como x > 1, teremos x - 1 > 0 e 2 = 1/x > 0 ==> F(x) < 0 Assim, F(x) <= 0, com igualdade se e somente se x = 1, ou seja: |x + 1/x - 2| <= 3|x - 1| para todo x > 1/2, com igualdade <==> x = 1. Um abraco, Claudio. ========================================================================= Instruções para entrar na lista, sair da lista e usar a lista em http://www.mat.puc-rio.br/~nicolau/olimp/obm-l.html ========================================================================= From owner-obm-l@sucuri.mat.puc-rio.br Sat Apr 26 18:40:54 2003 Return-Path: Received: (from majordom@localhost) by sucuri.mat.puc-rio.br (8.9.3/8.9.3) id SAA21087 for obm-l-MTTP; Sat, 26 Apr 2003 18:38:04 -0300 Received: from smtp-29.ig.com.br (smtp-29.ig.com.br [200.226.132.157]) by sucuri.mat.puc-rio.br (8.9.3/8.9.3) with SMTP id SAA21083 for ; Sat, 26 Apr 2003 18:38:01 -0300 Received: (qmail 14609 invoked from network); 26 Apr 2003 21:37:40 -0000 Received: from unknown (HELO henrique) (200.140.6.162) by smtp-29.ig.com.br with SMTP; 26 Apr 2003 21:37:40 -0000 Message-ID: <000f01c30c3c$0bd5fc10$019da8c0@henrique> From: "=?iso-8859-1?Q?Henrique_Patr=EDcio_Sant'Anna_Branco?=" To: References: Subject: Re: [obm-l] Espaco Vetorial: Corrigindo Date: Sat, 26 Apr 2003 18:37:32 -0300 MIME-Version: 1.0 Content-Type: text/plain; charset="iso-8859-1" Content-Transfer-Encoding: 8bit X-Priority: 3 X-MSMail-Priority: Normal X-Mailer: Microsoft Outlook Express 6.00.2800.1158 X-MimeOLE: Produced By Microsoft MimeOLE V6.00.2800.1165 Sender: owner-obm-l@sucuri.mat.puc-rio.br Precedence: bulk Reply-To: obm-l@mat.puc-rio.br Felipe, > Um Espaço Vetorial pode ser formado por EXATAMENTE 2 vetores distintos ? Acho que não. Não tenho certeza disso pois comecei a estudar algebra linear agora. Mas se todo subespaço vetorial é formado por uma base (x_1, x_2, x_3, ... , x_n), (y_1, y_2, y_3, ... y_n) etc (dependendo da dimensão do mesmo), podemos formar uma infinidade de vetores usando a combinações lineares da base, i.e., variando os escalares que multiplicamos por cada vetor da base. Alguém pode me corrigir, talvez essa minha explanação fure em algum caso, mas não consigo enxergar. Abraços, Henrique. ========================================================================= Instruções para entrar na lista, sair da lista e usar a lista em http://www.mat.puc-rio.br/~nicolau/olimp/obm-l.html ========================================================================= From owner-obm-l@sucuri.mat.puc-rio.br Sat Apr 26 18:44:19 2003 Return-Path: Received: (from majordom@localhost) by sucuri.mat.puc-rio.br (8.9.3/8.9.3) id SAA21202 for obm-l-MTTP; Sat, 26 Apr 2003 18:41:43 -0300 Received: from ivoti.terra.com.br (ivoti.terra.com.br [200.176.3.20]) by sucuri.mat.puc-rio.br (8.9.3/8.9.3) with ESMTP id SAA21197 for ; Sat, 26 Apr 2003 18:41:39 -0300 Received: from gunga.terra.com.br (gunga.terra.com.br [200.176.3.45]) by ivoti.terra.com.br (Postfix) with ESMTP id D6851408AF9 for ; Sat, 26 Apr 2003 18:41:08 -0300 (BRT) Received: from [200.177.188.30] (dl-nas7-sao-C8B1BC1E.p001.terra.com.br [200.177.188.30]) by gunga.terra.com.br (Postfix) with ESMTP id 6FCB2128068 for ; Sat, 26 Apr 2003 18:41:08 -0300 (BRT) User-Agent: Microsoft-Outlook-Express-Macintosh-Edition/5.02.2022 Date: Sat, 26 Apr 2003 18:42:37 -0300 Subject: Re: [obm-l] s=?ISO-8859-1?B?6Q==?=rie From: Claudio Buffara To: Message-ID: In-Reply-To: Mime-version: 1.0 Content-type: text/plain; charset="ISO-8859-1" Content-Transfer-Encoding: 8bit X-MIME-Autoconverted: from quoted-printable to 8bit by sucuri.mat.puc-rio.br id SAA21198 Sender: owner-obm-l@sucuri.mat.puc-rio.br Precedence: bulk Reply-To: obm-l@mat.puc-rio.br on 26.04.03 17:36, marcio.lis at marcio.lis@bol.com.br wrote: > p/ quanto converge a série 1/n^2????salvo > engano o gugu me falou uma vez sobre esse valor mas eu > não lembro > > Pi^2/6. ========================================================================= Instruções para entrar na lista, sair da lista e usar a lista em http://www.mat.puc-rio.br/~nicolau/olimp/obm-l.html ========================================================================= From owner-obm-l@sucuri.mat.puc-rio.br Sat Apr 26 18:44:52 2003 Return-Path: Received: (from majordom@localhost) by sucuri.mat.puc-rio.br (8.9.3/8.9.3) id SAA21250 for obm-l-MTTP; Sat, 26 Apr 2003 18:42:20 -0300 Received: from smtp-29.ig.com.br (smtp-29.ig.com.br [200.226.132.157]) by sucuri.mat.puc-rio.br (8.9.3/8.9.3) with SMTP id SAA21246 for ; Sat, 26 Apr 2003 18:42:16 -0300 Received: (qmail 21458 invoked from network); 26 Apr 2003 21:41:52 -0000 Received: from unknown (HELO henrique) (200.140.6.162) by smtp-29.ig.com.br with SMTP; 26 Apr 2003 21:41:52 -0000 Message-ID: <001601c30c3c$a28879d0$019da8c0@henrique> From: "=?iso-8859-1?Q?Henrique_Patr=EDcio_Sant'Anna_Branco?=" To: References: Subject: Re: [obm-l] Espaco Vetorial: Corrigindo Date: Sat, 26 Apr 2003 18:41:45 -0300 MIME-Version: 1.0 Content-Type: text/plain; charset="iso-8859-1" Content-Transfer-Encoding: 8bit X-Priority: 3 X-MSMail-Priority: Normal X-Mailer: Microsoft Outlook Express 6.00.2800.1158 X-MimeOLE: Produced By Microsoft MimeOLE V6.00.2800.1165 Sender: owner-obm-l@sucuri.mat.puc-rio.br Precedence: bulk Reply-To: obm-l@mat.puc-rio.br Só pra complementar minha resposta anterior, eu entendi que "espaço formado por dois vetores" é diferente de "espaço gerado por dois vetores". Se for o segundo caso, é claramente possível, pois podemos escolher um espaço de dimensão 2, que terá exatamente dois vetores na base. Abraços, Henrique. ========================================================================= Instruções para entrar na lista, sair da lista e usar a lista em http://www.mat.puc-rio.br/~nicolau/olimp/obm-l.html ========================================================================= From owner-obm-l@sucuri.mat.puc-rio.br Sat Apr 26 18:54:11 2003 Return-Path: Received: (from majordom@localhost) by sucuri.mat.puc-rio.br (8.9.3/8.9.3) id SAA22240 for obm-l-MTTP; Sat, 26 Apr 2003 18:51:33 -0300 Received: from itaqui.terra.com.br (itaqui.terra.com.br [200.176.3.19]) by sucuri.mat.puc-rio.br (8.9.3/8.9.3) with ESMTP id SAA22205 for ; Sat, 26 Apr 2003 18:51:18 -0300 Received: from jurere.terra.com.br (jurere.terra.com.br [200.176.3.49]) by itaqui.terra.com.br (Postfix) with ESMTP id E474D3BC757 for ; Sat, 26 Apr 2003 18:50:46 -0300 (BRT) Received: from niski.com (unknown [200.148.198.60]) (authenticated user fniski) by jurere.terra.com.br (Postfix) with ESMTP id B6D911380BC for ; Sat, 26 Apr 2003 18:50:45 -0300 (BRT) Message-ID: <3EAAFF3D.2030706@niski.com> Date: Sat, 26 Apr 2003 14:50:53 -0700 From: niski User-Agent: Mozilla/5.0 (Windows; U; Windows NT 5.1; en-US; rv:1.0.2) Gecko/20030208 Netscape/7.02 X-Accept-Language: en-us, en MIME-Version: 1.0 To: obm-l@mat.puc-rio.br Subject: [obm-l] Limites Content-Type: text/plain; charset=ISO-8859-1; format=flowed Content-Transfer-Encoding: 8bit Sender: owner-obm-l@sucuri.mat.puc-rio.br Precedence: bulk Reply-To: obm-l@mat.puc-rio.br Pessoal, por favor resolvam estes limites. nao me vem nenhuma ideia..obrigado lim[x->pi/2] cos(x)/cbrt((1-sinx)^2) Me ocorreu fazer a seguinte transformação : pi/2 - x = z Daí lim[z->0] sin(z)/cbrt((1-sin((pi/2)-z))^2) dai eu travei... outra coisa...gostaria de saber se sempre é valida subistituicao de variaveis...se nao for..quando nao é? obrigado niski ========================================================================= Instruções para entrar na lista, sair da lista e usar a lista em http://www.mat.puc-rio.br/~nicolau/olimp/obm-l.html ========================================================================= From owner-obm-l@sucuri.mat.puc-rio.br Sat Apr 26 19:17:19 2003 Return-Path: Received: (from majordom@localhost) by sucuri.mat.puc-rio.br (8.9.3/8.9.3) id TAA24437 for obm-l-MTTP; Sat, 26 Apr 2003 19:14:40 -0300 Received: from itaqui.terra.com.br (itaqui.terra.com.br [200.176.3.19]) by sucuri.mat.puc-rio.br (8.9.3/8.9.3) with ESMTP id TAA24427 for ; Sat, 26 Apr 2003 19:14:34 -0300 Received: from marova.terra.com.br (marova.terra.com.br [200.176.3.39]) by itaqui.terra.com.br (Postfix) with ESMTP id 70D933BC6D0 for ; Sat, 26 Apr 2003 19:14:03 -0300 (BRT) Received: from niski.com (unknown [200.148.198.60]) (authenticated user fniski) by marova.terra.com.br (Postfix) with ESMTP id 3FA953DC055 for ; Sat, 26 Apr 2003 19:14:02 -0300 (BRT) Message-ID: <3EAB04B1.60806@niski.com> Date: Sat, 26 Apr 2003 15:14:09 -0700 From: niski User-Agent: Mozilla/5.0 (Windows; U; Windows NT 5.1; en-US; rv:1.0.2) Gecko/20030208 Netscape/7.02 X-Accept-Language: en-us, en MIME-Version: 1.0 To: obm-l@mat.puc-rio.br Subject: Re: [obm-l] provar desiguldade.. References: Content-Type: text/plain; charset=ISO-8859-1; format=flowed Content-Transfer-Encoding: 8bit Sender: owner-obm-l@sucuri.mat.puc-rio.br Precedence: bulk Reply-To: obm-l@mat.puc-rio.br Claudio, creio que sua demonstracao ficou mais clara do que a minha , pq eu utilizei "De fato pois o menor valor que 4x pode assumir é maior do que 4(1/2) = 2." Certo? Se sim, pq isso é falta de formalização? Será que esta passagem foi muito intuitiva e por isso voce a classificou como não formal!? obrigado Claudio Buffara wrote: > on 26.04.03 16:18, niski at fabio@niski.com wrote: > > >>Por favor pessoal, me ajudem nesta questão, obrigado: >> >>Seja f(x) = x + 1/x >> >>Prove que >> >>|f(x) - f(1)| <= 3|x-1| , para x > 1/2 >> >>Demonstracao >>|x+ (1/x) - 2 | <= 3|x-1| >>|(x^2 -2x +1)/x| <= 3|x-1| >>(|x-1||x-1|)/x <= 3|x-1| >>|x-1|/x <= 3 >>|x-1| <= 3x >> >>Se 1/2 < x < 1 >> >>-x+1 < 3x >>1 < 4x >> >>De fato pois o menor valor que 4x pode assumir é maior do que 4(1/2) = 2. >> >>Se x >= 1 >> >>|x-1| <= 3x >>x-1 < 3x >>-1 < 4x >> >>De fato pois o menor valor que 4x pode assumir é 4. >> >>Esta certa esta demonstração?! >>Obrigado. >> -- [about him:] It is rare to find learned men who are clean, do not stink and have a sense of humour. -Gottfried Whilhem Leibniz ========================================================================= Instruções para entrar na lista, sair da lista e usar a lista em http://www.mat.puc-rio.br/~nicolau/olimp/obm-l.html ========================================================================= From owner-obm-l@sucuri.mat.puc-rio.br Sat Apr 26 19:27:54 2003 Return-Path: Received: (from majordom@localhost) by sucuri.mat.puc-rio.br (8.9.3/8.9.3) id TAA24904 for obm-l-MTTP; Sat, 26 Apr 2003 19:25:20 -0300 Received: from paiol.terra.com.br (paiol.terra.com.br [200.176.3.18]) by sucuri.mat.puc-rio.br (8.9.3/8.9.3) with ESMTP id TAA24900 for ; Sat, 26 Apr 2003 19:25:17 -0300 Received: from canela.terra.com.br (canela.terra.com.br [200.176.3.79]) by paiol.terra.com.br (Postfix) with ESMTP id ACB3B8877E for ; Sat, 26 Apr 2003 19:24:46 -0300 (BRT) Received: from [200.177.179.185] (dl-nas3-sao-C8B1B3B9.p001.terra.com.br [200.177.179.185]) by canela.terra.com.br (Postfix) with ESMTP id AC0A2224088 for ; Sat, 26 Apr 2003 19:24:45 -0300 (BRT) User-Agent: Microsoft-Outlook-Express-Macintosh-Edition/5.02.2022 Date: Sat, 26 Apr 2003 19:26:14 -0300 Subject: Re: [obm-l] serie do Marcio From: Claudio Buffara To: Message-ID: In-Reply-To: <018001c30b38$8920ad00$5400a8c0@ensrbr> Mime-version: 1.0 Content-type: text/plain; charset="ISO-8859-1" Content-Transfer-Encoding: 8bit X-MIME-Autoconverted: from quoted-printable to 8bit by sucuri.mat.puc-rio.br id TAA24901 Sender: owner-obm-l@sucuri.mat.puc-rio.br Precedence: bulk Reply-To: obm-l@mat.puc-rio.br on 25.04.03 11:39, Luis Lopes at llopes@ensrbr.com.br wrote: > Sauda,c~oes, > > Acabo de mandar uma msg boba por > bater no botão errado. > > Não sei se estou falando com o mesmo Marcio. > >> Quando exatamente eu posso trocar a ordem da >> integral com o somatorio? >> Por exemplo, considere a serie cujo n-o termo eh >> s_n = 1/(4n+1) + 1/(4n+3) - 1/(2n+2), cujos termos >> são todos positivos. >> Ela converge, por comparação com a série a/n^2. >> >> Para calcular Somatorio (0 a infinito) s_n, eu pensei >> em calcular: >> Somatorio(0 a infinito)_Integral (0 a 1) >> [x^4n + x^(4n+2) - x^(2n+1)] >> Trocando a ordem, ficamos com algumas PG's e: >> Integral (0 a 1) [1/(1-x^4) + x^2 / (1-x^4) - x/(1-x^2)] = >> Integral (0 a 1) [1/(1+x)] = ln2 >> >> Mas eu vi que essa soma vale, na verdade, 1.5 ln2 >> (inclusive me provaram isso, e parece estar certo).. >> >> Por outro lado, em outros problemas esse método >> funciona bem.. Por exemplo, >> para calcular Somatorio ( 1/[(3n+1)*(3n+2)*(3n+3)] ) >> a resposta parece dar correta.. >> >> Abracos, >> Marcio > > Tentei e não consegui mostrar que dá 1.5 ln2. > > Como faz? > > []'s > Luís > > Oi, Marcio e Luis: Realmente eh um problema intrigante. Empiricamente (com uma planilha) eu me convenci que a soma eh 1,5*Ln(2). Alem disso, vejam esse algebrismo: Ln(2) = 1-1/2+1/3-1/4+1/5-1/6+1/7-1/8+1/9-1/10+1/11-1/12+1/13-1/14+... 0,5*Ln(2) = 1/2 -1/4 +1/6 -1/8 +1/10 -1/12 +1/14 - ... Somando: 1,5*Ln(2) = 1 +1/3-1/2+1/5 +1/7-1/4+1/9 +1/11-1/6+1/3 + ... Rearranjando os termos: 1,5*Ln(2) = (1+1/3-1/2) + (1/5+1/7-1/4) + (1/9+1/11-1/6) + ... Ou seja, 1,5*Ln(2) = SOMA(n >= 0) [1/(4n+1) + 1/(4n+3) - 1/(2n+2)] Assim, o algebrismo funciona apesar de eu nao ter certeza do rigor, dado que estamos lidando com uma serie condicionalmente convergente. Pergunta: o que eu fiz pode ser justificado com rigor ou eu achei o resultado correto por pura sorte? Um abraco, Claudio. ========================================================================= Instruções para entrar na lista, sair da lista e usar a lista em http://www.mat.puc-rio.br/~nicolau/olimp/obm-l.html ========================================================================= From owner-obm-l@sucuri.mat.puc-rio.br Sat Apr 26 20:04:53 2003 Return-Path: Received: (from majordom@localhost) by sucuri.mat.puc-rio.br (8.9.3/8.9.3) id UAA26161 for obm-l-MTTP; Sat, 26 Apr 2003 20:02:00 -0300 Received: from artemis.opendf.com.br (artemis.opengate.com.br [200.181.71.14]) by sucuri.mat.puc-rio.br (8.9.3/8.9.3) with ESMTP id UAA26157 for ; Sat, 26 Apr 2003 20:01:56 -0300 Received: from localhost (localhost [127.0.0.1]) by artemis.opendf.com.br (Postfix) with ESMTP id C18602BEE2 for ; Sat, 26 Apr 2003 20:02:37 -0300 (BRT) Received: from artemis.opendf.com.br ([127.0.0.1]) by localhost (artemis.opengate.com.br [127.0.0.1:10024]) (amavisd-new) with ESMTP id 01999-03 for ; Sat, 26 Apr 2003 20:02:37 -0300 (BRT) Received: from computer (200-181-088-208.bsace7001.dsl.brasiltelecom.net.br [200.181.88.208]) by artemis.opendf.com.br (Postfix) with ESMTP id 8DC462BEE0 for ; Sat, 26 Apr 2003 20:02:36 -0300 (BRT) From: "Artur Costa Steiner" To: Subject: RE: [obm-l] Espaco Vetorial: Corrigindo Date: Sat, 26 Apr 2003 20:01:27 -0300 Organization: Steiner Consultoria LTDA Message-ID: <000c01c30c47$c4c71aa0$9865fea9@computer> MIME-Version: 1.0 Content-Type: text/plain; charset="iso-8859-1" X-Priority: 3 (Normal) X-MSMail-Priority: Normal X-Mailer: Microsoft Outlook, Build 10.0.2627 In-Reply-To: <000f01c30c3c$0bd5fc10$019da8c0@henrique> X-MimeOLE: Produced By Microsoft MimeOLE V6.00.2800.1165 Importance: Normal X-Virus-Scanned: by amavisd-new Content-Transfer-Encoding: 8bit X-MIME-Autoconverted: from quoted-printable to 8bit by sucuri.mat.puc-rio.br id UAA26158 Sender: owner-obm-l@sucuri.mat.puc-rio.br Precedence: bulk Reply-To: obm-l@mat.puc-rio.br [Artur Costa Steiner] De fato, nao pode mesmo nao. E o unico espaco vetorial finto e aquele composto unicamente pelo vetor nulo. Outra forma de ver que um espaco vetorial nao pode ter apenas ter dois vetores distintos: por definicao, um deles tem que ser o vetor nulo. E como o outro vetor somado com o nulo e ele proprio, vemos que este outro vetor nao tem elemento simetrico, contrariamente a definicao de espaco vetorial. Artur >Felipe, > >> Um Espaço Vetorial pode ser formado por EXATAMENTE 2 vetores distintos ? > >Acho que não. Não tenho certeza disso pois comecei a estudar algebra linear >agora. >Mas se todo subespaço vetorial é formado por uma base (x_1, x_2, x_3, ... , >x_n), (y_1, y_2, y_3, ... y_n) etc (dependendo da dimensão do mesmo), >podemos formar uma infinidade de vetores usando a combinações lineares da >base, i.e., variando os escalares que multiplicamos por cada vetor da base. >Alguém pode me corrigir, talvez essa minha explanação fure em algum caso, >mas não consigo enxergar. > >Abraços, >Henrique. > >======================================================================= == >Instruções para entrar na lista, sair da lista e usar a lista em >http://www.mat.puc-rio.br/~nicolau/olimp/obm-l.html >======================================================================= == ========================================================================= Instruções para entrar na lista, sair da lista e usar a lista em http://www.mat.puc-rio.br/~nicolau/olimp/obm-l.html ========================================================================= From owner-obm-l@sucuri.mat.puc-rio.br Sat Apr 26 20:33:23 2003 Return-Path: Received: (from majordom@localhost) by sucuri.mat.puc-rio.br (8.9.3/8.9.3) id UAA26955 for obm-l-MTTP; Sat, 26 Apr 2003 20:30:49 -0300 Received: from trex.centroin.com.br (trex.centroin.com.br [200.225.63.134]) by sucuri.mat.puc-rio.br (8.9.3/8.9.3) with ESMTP id UAA26950 for ; Sat, 26 Apr 2003 20:30:46 -0300 Received: from trex.centroin.com.br (localhost [127.0.0.1]) by trex.centroin.com.br (8.12.9/8.12.9) with ESMTP id h3QNUGkZ006099 for ; Sat, 26 Apr 2003 20:30:16 -0300 (EST) Received: by trex.centroin.com.br (8.12.9/8.12.5/Submit) id h3QNUG66006098; Sat, 26 Apr 2003 20:30:16 -0300 (EST) Message-Id: <200304262330.h3QNUG66006098@trex.centroin.com.br> Received: from 200.165.208.231 by trex.centroin.com.br (CIPWM versao 1.4C1) with HTTPS for ; Sat, 26 Apr 2003 20:30:16 -0300 (EST) Date: Sat, 26 Apr 2003 20:30:16 -0300 (EST) From: Augusto Cesar de Oliveira Morgado To: obm-l@mat.puc-rio.br Subject: Re: [obm-l] Limites MIME-Version: 1.0 X-Mailer: CentroIn Internet Provider WebMail v. 1.4C1 (http://www.centroin.com.br/) Content-Type: text/plain; charset="iso-8859-1" Content-Transfer-Encoding: 8bit X-MIME-Autoconverted: from quoted-printable to 8bit by sucuri.mat.puc-rio.br id UAA26952 Sender: owner-obm-l@sucuri.mat.puc-rio.br Precedence: bulk Reply-To: obm-l@mat.puc-rio.br Que diabo eh cbrt? Em Sat, 26 Apr 2003 14:50:53 -0700, niski disse: > Pessoal, por favor resolvam estes limites. nao me vem nenhuma > ideia..obrigado > > lim[x->pi/2] cos(x)/cbrt((1-sinx)^2) > > Me ocorreu fazer a seguinte transformação : > > pi/2 - x = z > > Daí > > lim[z->0] sin(z)/cbrt((1-sin((pi/2)-z))^2) > > dai eu travei... > > outra coisa...gostaria de saber se sempre é valida subistituicao de > variaveis...se nao for..quando nao é? > > obrigado > > niski > > ========================================================================= > Instruções para entrar na lista, sair da lista e usar a lista em > http://www.mat.puc-rio.br/~nicolau/olimp/obm-l.html > ========================================================================= > > ========================================================================= Instruções para entrar na lista, sair da lista e usar a lista em http://www.mat.puc-rio.br/~nicolau/olimp/obm-l.html ========================================================================= From owner-obm-l@sucuri.mat.puc-rio.br Sat Apr 26 20:33:33 2003 Return-Path: Received: (from majordom@localhost) by sucuri.mat.puc-rio.br (8.9.3/8.9.3) id UAA26964 for obm-l-MTTP; Sat, 26 Apr 2003 20:31:02 -0300 Received: from trex.centroin.com.br (trex.centroin.com.br [200.225.63.134]) by sucuri.mat.puc-rio.br (8.9.3/8.9.3) with ESMTP id UAA26960 for ; Sat, 26 Apr 2003 20:30:59 -0300 Received: from trex.centroin.com.br (localhost [127.0.0.1]) by trex.centroin.com.br (8.12.9/8.12.9) with ESMTP id h3QNUTkZ006174 for ; Sat, 26 Apr 2003 20:30:29 -0300 (EST) Received: by trex.centroin.com.br (8.12.9/8.12.5/Submit) id h3QNUTBm006172; Sat, 26 Apr 2003 20:30:29 -0300 (EST) Message-Id: <200304262330.h3QNUTBm006172@trex.centroin.com.br> Received: from 200.165.208.231 by trex.centroin.com.br (CIPWM versao 1.4C1) with HTTPS for ; Sat, 26 Apr 2003 20:30:29 -0300 (EST) Date: Sat, 26 Apr 2003 20:30:29 -0300 (EST) From: Augusto Cesar de Oliveira Morgado To: obm-l@mat.puc-rio.br Subject: Re: [obm-l] Limites MIME-Version: 1.0 X-Mailer: CentroIn Internet Provider WebMail v. 1.4C1 (http://www.centroin.com.br/) Content-Type: text/plain; charset="iso-8859-1" Content-Transfer-Encoding: 8bit X-MIME-Autoconverted: from quoted-printable to 8bit by sucuri.mat.puc-rio.br id UAA26961 Sender: owner-obm-l@sucuri.mat.puc-rio.br Precedence: bulk Reply-To: obm-l@mat.puc-rio.br Que diabo eh cbrt? Em Sat, 26 Apr 2003 14:50:53 -0700, niski disse: > Pessoal, por favor resolvam estes limites. nao me vem nenhuma > ideia..obrigado > > lim[x->pi/2] cos(x)/cbrt((1-sinx)^2) > > Me ocorreu fazer a seguinte transformação : > > pi/2 - x = z > > Daí > > lim[z->0] sin(z)/cbrt((1-sin((pi/2)-z))^2) > > dai eu travei... > > outra coisa...gostaria de saber se sempre é valida subistituicao de > variaveis...se nao for..quando nao é? > > obrigado > > niski > > ========================================================================= > Instruções para entrar na lista, sair da lista e usar a lista em > http://www.mat.puc-rio.br/~nicolau/olimp/obm-l.html > ========================================================================= > > ========================================================================= Instruções para entrar na lista, sair da lista e usar a lista em http://www.mat.puc-rio.br/~nicolau/olimp/obm-l.html ========================================================================= From owner-obm-l@sucuri.mat.puc-rio.br Sat Apr 26 20:33:56 2003 Return-Path: Received: (from majordom@localhost) by sucuri.mat.puc-rio.br (8.9.3/8.9.3) id UAA26974 for obm-l-MTTP; Sat, 26 Apr 2003 20:31:24 -0300 Received: from trex.centroin.com.br (trex.centroin.com.br [200.225.63.134]) by sucuri.mat.puc-rio.br (8.9.3/8.9.3) with ESMTP id UAA26970 for ; Sat, 26 Apr 2003 20:31:21 -0300 Received: from trex.centroin.com.br (localhost [127.0.0.1]) by trex.centroin.com.br (8.12.9/8.12.9) with ESMTP id h3QNUpkZ006279 for ; Sat, 26 Apr 2003 20:30:51 -0300 (EST) Received: by trex.centroin.com.br (8.12.9/8.12.5/Submit) id h3QNUp2i006278; Sat, 26 Apr 2003 20:30:51 -0300 (EST) Message-Id: <200304262330.h3QNUp2i006278@trex.centroin.com.br> Received: from 200.165.208.231 by trex.centroin.com.br (CIPWM versao 1.4C1) with HTTPS for ; Sat, 26 Apr 2003 20:30:51 -0300 (EST) Date: Sat, 26 Apr 2003 20:30:51 -0300 (EST) From: Augusto Cesar de Oliveira Morgado To: obm-l@mat.puc-rio.br Subject: Re: [obm-l] Limites MIME-Version: 1.0 X-Mailer: CentroIn Internet Provider WebMail v. 1.4C1 (http://www.centroin.com.br/) Content-Type: text/plain; charset="iso-8859-1" Content-Transfer-Encoding: 8bit X-MIME-Autoconverted: from quoted-printable to 8bit by sucuri.mat.puc-rio.br id UAA26971 Sender: owner-obm-l@sucuri.mat.puc-rio.br Precedence: bulk Reply-To: obm-l@mat.puc-rio.br Que diabo eh cbrt? Em Sat, 26 Apr 2003 14:50:53 -0700, niski disse: > Pessoal, por favor resolvam estes limites. nao me vem nenhuma > ideia..obrigado > > lim[x->pi/2] cos(x)/cbrt((1-sinx)^2) > > Me ocorreu fazer a seguinte transformação : > > pi/2 - x = z > > Daí > > lim[z->0] sin(z)/cbrt((1-sin((pi/2)-z))^2) > > dai eu travei... > > outra coisa...gostaria de saber se sempre é valida subistituicao de > variaveis...se nao for..quando nao é? > > obrigado > > niski > > ========================================================================= > Instruções para entrar na lista, sair da lista e usar a lista em > http://www.mat.puc-rio.br/~nicolau/olimp/obm-l.html > ========================================================================= > > ========================================================================= Instruções para entrar na lista, sair da lista e usar a lista em http://www.mat.puc-rio.br/~nicolau/olimp/obm-l.html ========================================================================= From owner-obm-l@sucuri.mat.puc-rio.br Sat Apr 26 20:40:26 2003 Return-Path: Received: (from majordom@localhost) by sucuri.mat.puc-rio.br (8.9.3/8.9.3) id UAA27157 for obm-l-MTTP; Sat, 26 Apr 2003 20:37:53 -0300 Received: from hotmail.com (oe63.law10.hotmail.com [64.4.14.198]) by sucuri.mat.puc-rio.br (8.9.3/8.9.3) with ESMTP id UAA27142 for ; Sat, 26 Apr 2003 20:37:46 -0300 Received: from mail pickup service by hotmail.com with Microsoft SMTPSVC; Sat, 26 Apr 2003 16:37:14 -0700 Received: from 67.24.125.40 by oe63.law10.hotmail.com with DAV; Sat, 26 Apr 2003 23:37:14 +0000 X-Originating-IP: [67.24.125.40] X-Originating-Email: [lrecova@hotmail.com] From: =?ISO-8859-1?Q?Leandro_Lacorte_Rec=F4va?= To: Subject: RE: [obm-l] Limites Date: Sat, 26 Apr 2003 16:37:13 -0700 Message-ID: <000001c30c4c$c388dc00$287d1843@LeandroRecova> MIME-Version: 1.0 Content-Type: text/plain; charset="ISO-8859-1" X-Priority: 3 (Normal) X-MSMail-Priority: Normal X-Mailer: Microsoft Outlook, Build 10.0.3416 In-Reply-To: <3EAAFF3D.2030706@niski.com> Importance: Normal X-MimeOLE: Produced By Microsoft MimeOLE V6.00.2800.1165 X-OriginalArrivalTime: 26 Apr 2003 23:37:14.0958 (UTC) FILETIME=[C4755EE0:01C30C4C] Content-Transfer-Encoding: 8bit X-MIME-Autoconverted: from quoted-printable to 8bit by sucuri.mat.puc-rio.br id UAA27144 Sender: owner-obm-l@sucuri.mat.puc-rio.br Precedence: bulk Reply-To: obm-l@mat.puc-rio.br Niski, Cbrt significa Raiz Cubica ? Se for isso, a solucao abaixo: Coloque o numerador no denominador dentro da raiz, o que esta no denominador e a expressao de cos^2(x) que simplificara com cos^3(x), lim[x->pi/2] cos(x)/cbrt((1-sinx)^2) = = lim[x->pi/2] 1/cbrt((1-sinx)^2/cos^3(x))) = lim[x->pi/2] 1/cbrt((cos^2(x)/cos^3(x))) = lim[x->pi/2] cbrt(cos(x)) = 0. Leandro. -----Original Message----- From: owner-obm-l@sucuri.mat.puc-rio.br [mailto:owner-obm-l@sucuri.mat.puc-rio.br] On Behalf Of niski Sent: Saturday, April 26, 2003 2:51 PM To: obm-l@mat.puc-rio.br Subject: [obm-l] Limites Pessoal, por favor resolvam estes limites. nao me vem nenhuma ideia..obrigado lim[x->pi/2] cos(x)/cbrt((1-sinx)^2) Me ocorreu fazer a seguinte transformação : pi/2 - x = z Daí lim[z->0] sin(z)/cbrt((1-sin((pi/2)-z))^2) dai eu travei... outra coisa...gostaria de saber se sempre é valida subistituicao de variaveis...se nao for..quando nao é? obrigado niski ======================================================================== = Instruções para entrar na lista, sair da lista e usar a lista em http://www.mat.puc-rio.br/~nicolau/olimp/obm-l.html ======================================================================== = ========================================================================= Instruções para entrar na lista, sair da lista e usar a lista em http://www.mat.puc-rio.br/~nicolau/olimp/obm-l.html ========================================================================= From owner-obm-l@sucuri.mat.puc-rio.br Sat Apr 26 20:55:02 2003 Return-Path: Received: (from majordom@localhost) by sucuri.mat.puc-rio.br (8.9.3/8.9.3) id UAA28132 for obm-l-MTTP; Sat, 26 Apr 2003 20:52:29 -0300 Received: from artemis.opendf.com.br (artemis.opengate.com.br [200.181.71.14]) by sucuri.mat.puc-rio.br (8.9.3/8.9.3) with ESMTP id UAA28128 for ; Sat, 26 Apr 2003 20:52:25 -0300 Received: from localhost (localhost [127.0.0.1]) by artemis.opendf.com.br (Postfix) with ESMTP id CAAB12BEF7 for ; Sat, 26 Apr 2003 20:53:06 -0300 (BRT) Received: from artemis.opendf.com.br ([127.0.0.1]) by localhost (artemis.opengate.com.br [127.0.0.1:10024]) (amavisd-new) with ESMTP id 02731-08 for ; Sat, 26 Apr 2003 20:53:06 -0300 (BRT) Received: from computer (200-181-088-208.bsace7001.dsl.brasiltelecom.net.br [200.181.88.208]) by artemis.opendf.com.br (Postfix) with ESMTP id 74FBC2BEDD for ; Sat, 26 Apr 2003 20:53:05 -0300 (BRT) From: "Artur Costa Steiner" To: Subject: RE: [obm-l] Provar continuidade Date: Sat, 26 Apr 2003 20:51:52 -0300 Organization: Steiner Consultoria LTDA Message-ID: <000d01c30c4e$d1df2370$9865fea9@computer> MIME-Version: 1.0 Content-Type: text/plain; charset="iso-8859-1" X-Priority: 3 (Normal) X-MSMail-Priority: Normal X-Mailer: Microsoft Outlook, Build 10.0.2627 In-Reply-To: <3EAAD73A.2040708@niski.com> X-MimeOLE: Produced By Microsoft MimeOLE V6.00.2800.1165 Importance: Normal X-Virus-Scanned: by amavisd-new Content-Transfer-Encoding: 8bit X-MIME-Autoconverted: from quoted-printable to 8bit by sucuri.mat.puc-rio.br id UAA28129 Sender: owner-obm-l@sucuri.mat.puc-rio.br Precedence: bulk Reply-To: obm-l@mat.puc-rio.br >>>provar que f(x) = (x)^(1/n) é continua. >>> >>>Demonstraçao : >>>Dado Eps > 0 existe um intervalo aberto I , p pertencente a I , tal que >>> >>>x pertence a I => f(p) - Eps < f(x) < f(p) + Eps >> >> [Artur Costa Steiner] >> Ei!! Isto eh a definicao de continuidade em p! Eh justamente o que vc >quer >> provar! Eh como provar que x eh maior que zero partindo do procipio que x >eh >> maior que zero. > > > >> Vc desenvolveu um raciocinio certo, so se perdeu um pouco na logica. >> Artur > >Artur, entendo o que voce quer dizer..mas é o seguinte.. >É de comum acordo que se achar um intervalo aberto I, com p neste >intervalo tal que x pertence a I => f(p) - Eps < f(x) < f(p) + Eps >Certo? [Artur Costa Steiner] Sem duvida. > >Otimo...agora vamos supor que eu vá para um "rascunho" e tente achar >esse intervalo usando a hipotese! Note que "ninguem estará vendo" >Então no rascunho : > >(p)^(1/n) - Eps < (x)^(1/n) < (p)^(1/n) + Eps >((p)^(1/n) - Eps)^n < x < ((p)^(1/n) + Eps)^n > >Pronto! achei o intervalo, agora continuando efetivamente a prova > >tomando-se I = ]((p)^(1/n) - Eps)^n, ((p)^(1/n) + Eps)^n[ , p >pertencente a I > >x pertence a I => f(p) - Eps < f(x) < f(p) + Eps >logo f(x) = (x)^1/n é continua em todo p real. > >Então para na mensagem não ficar muito "criptico" de onde saiu o >intervalo, eu mostrei o raciocinio que utilizei para encontra-lo. >E voce pode confirmar graficamente que esse intervalo esta correto. > >Feito estas observacoes, pergunto novamente se minha demonstracao >continua errada. [Artur Costa Steiner] Nao, ela estah OK. Na realidae, vc baseou-se no fato de que, para n natural, x^n eh continua e estritamente crescente para x>= 0 (nao em todo o R). Logo, a sua inversa x^(1/n) tambem e continua e crescente. Artur ========================================================================= Instruções para entrar na lista, sair da lista e usar a lista em http://www.mat.puc-rio.br/~nicolau/olimp/obm-l.html ========================================================================= From owner-obm-l@sucuri.mat.puc-rio.br Sat Apr 26 20:57:30 2003 Return-Path: Received: (from majordom@localhost) by sucuri.mat.puc-rio.br (8.9.3/8.9.3) id UAA28225 for obm-l-MTTP; Sat, 26 Apr 2003 20:54:56 -0300 Received: from artemis.opendf.com.br (artemis.opengate.com.br [200.181.71.14]) by sucuri.mat.puc-rio.br (8.9.3/8.9.3) with ESMTP id UAA28221 for ; Sat, 26 Apr 2003 20:54:52 -0300 Received: from localhost (localhost [127.0.0.1]) by artemis.opendf.com.br (Postfix) with ESMTP id 9FAC32BEF7 for ; Sat, 26 Apr 2003 20:55:34 -0300 (BRT) Received: from artemis.opendf.com.br ([127.0.0.1]) by localhost (artemis.opengate.com.br [127.0.0.1:10024]) (amavisd-new) with ESMTP id 02731-09 for ; Sat, 26 Apr 2003 20:55:33 -0300 (BRT) Received: from computer (200-181-088-208.bsace7001.dsl.brasiltelecom.net.br [200.181.88.208]) by artemis.opendf.com.br (Postfix) with ESMTP id 31FB02BEDD for ; Sat, 26 Apr 2003 20:55:33 -0300 (BRT) From: "Artur Costa Steiner" To: Subject: RE: [obm-l] Espaco Vetorial: Corrigindo Date: Sat, 26 Apr 2003 20:54:23 -0300 Organization: Steiner Consultoria LTDA Message-ID: MIME-Version: 1.0 Content-Type: multipart/mixed; boundary="----=_NextPart_000_0014_01C30C36.04849170" X-Priority: 3 (Normal) X-MSMail-Priority: Normal X-Mailer: Microsoft Outlook, Build 10.0.2627 In-Reply-To: <000f01c30c3c$0bd5fc10$019da8c0@henrique> X-MimeOLE: Produced By Microsoft MimeOLE V6.00.2800.1165 Importance: Normal X-MS-TNEF-Correlator: 000000005FC3F71D8B4BB941A8AA63D06F261C3664512B00 X-Virus-Scanned: by amavisd-new Sender: owner-obm-l@sucuri.mat.puc-rio.br Precedence: bulk Reply-To: obm-l@mat.puc-rio.br This is a multi-part message in MIME format. ------=_NextPart_000_0014_01C30C36.04849170 Content-Type: text/plain; charset="us-ascii" Content-Transfer-Encoding: quoted-printable Oi, Quando eu mandei minha primeira mensagem, eu estava pensando em espacos vetoriais definidos sobre o conjunto dos numeros reais. Sobre outros = corpos e possivel ter espacos vetoriais finitos que nao contenham apenas o = vetor nulo. Por exemplo, com as operacoes usuais, {-1, 0, 1} eh um espaco = vetorial de dimensao 1 definido sobre o corpo {-1, 0, 1}=20 Artur ------=_NextPart_000_0014_01C30C36.04849170 Content-Type: application/ms-tnef; name="winmail.dat" Content-Transfer-Encoding: base64 Content-Disposition: attachment; filename="winmail.dat" eJ8+IhcXAQaQCAAEAAAAAAABAAEAAQeQBgAIAAAA5AQAAAAAAADoAAEIgAcAGAAAAElQTS5NaWNy b3NvZnQgTWFpbC5Ob3RlADEIAQ2ABAACAAAAAgACAAEGAAcAAQAAAAAAAAEGgAMADgAAANMHBAAa ABQANgAAAAYASAEBA5AGANQGAAAuAAAACwACAAEAAAALACMAAAAAAAMAJgAAAAAACwApAAAAAAAD AC4AAAAAAAIBMQABAAAAGAAAAAAAAABfw/cdi0u5QaiqY9BvJhw2xE8rAAMANgAAAAAAHgBwAAEA AAAkAAAAW29ibS1sXSBFc3BhY28gVmV0b3JpYWw6IENvcnJpZ2luZG8AAgFxAAEAAAAWAAAAAcMM TIEdQQ3/+DaITSiLqsQoLBcBPwAAAgEdDAEAAAAZAAAAU01UUDpBUlRVUkBPUEVOREYuQ09NLkJS AAAAAAsAAQ4AAAAAQAAGDgCcghtPDMMBAgEKDgEAAAAYAAAAAAAAAF/D9x2LS7lBqKpj0G8mHDbC gAAAAwAUDgAAAAALAB8OAQAAAB4AKA4BAAAALwAAADAwMDAwMDAyAWFydHVyQG9wZW5kZi5jb20u YnIBcG9wLm9wZW5kZi5jb20uYnIAAB4AKQ4BAAAALwAAADAwMDAwMDAyAWFydHVyQG9wZW5kZi5j b20uYnIBcG9wLm9wZW5kZi5jb20uYnIAAAIBCRABAAAApQEAAKEBAAB6AgAATFpGdaDbsPIDAAoA cmNwZzEyNeIyA0N0ZXgFQQEDAff/CoACpAPkBxMCgA/zAFAEVj8IVQeyESUOUQMBAgBjaOEKwHNl dDIGAAbDESX2MwRGE7cwEiwRMwjvCfe2OxgfDjA1ESIMYGMAUDMLCQFkMzYWUAumIE9MaSwKogqA UXUAcGRgbyBldSADgQEAaWceEAuAE+AgcAUQB4BpDnIe0AeAAIBhZ2Vt7iwd4geQAZB2HtEfkh2z 3m0gQQqwBaAEIHYUIAWw2wcwBAAgAQELgGkdwAQgaHNvYhggIB3QBaBunGp1AjAd0CMCbnUHgG8D YAQgGCAiYS4GACNUdfZ0JNIFoXAh0SOAJoEAkN8iAAMgDrAFwCGPICLCIiDtBCBxClAkgGEjow6w HrH1IWBhIMFhBCAd0CIDJIF7F7AlYFAFsQ7AH+ALUG//IAAFoCpRKsEgwB9QBaAHkRx1cx2QBAAg AFx7LVoxIAAwIAAAUH0d4Gj/LZAhZiH2AyABACKQB3Efof0d0DEilyM5JnEuHB0kBxAWdAhwHSR9 NJAAAAAeAEIQAQAAACoAAAA8MDAwZjAxYzMwYzNjJDBiZDVmYzEwJDAxOWRhOGMwQGhlbnJpcXVl PgAAAAMAkhAAAAAAAgEUOgEAAAAQAAAAJ5o5XaOq4UiU6RD4HkfD4AMA3j+fTgAAAwAJWQEAAAAD AEBlAAAAAAsAE4AIIAYAAAAAAMAAAAAAAABGAAAAAAOFAAAAAAAAAwAVgAggBgAAAAAAwAAAAAAA AEYAAAAAEIUAAAAAAAADABuACCAGAAAAAADAAAAAAAAARgAAAABShQAA45ABAAMAIoAIIAYAAAAA AMAAAAAAAABGAAAAAAGFAAAAAAAAQAAjgAggBgAAAAAAwAAAAAAAAEYAAAAAYIUAAADQiMMQAAAA HgBBgAggBgAAAAAAwAAAAAAAAEYAAAAAVIUAAAEAAAAFAAAAMTAuMAAAAAALAEKACCAGAAAAAADA AAAAAAAARgAAAAAGhQAAAAAAAAsARoAIIAYAAAAAAMAAAAAAAABGAAAAAA6FAAAAAAAAAwBJgAgg BgAAAAAAwAAAAAAAAEYAAAAAGIUAAAAAAAALAF6ACCAGAAAAAADAAAAAAAAARgAAAACChQAAAQAA AAIB+A8BAAAAEAAAAF/D9x2LS7lBqKpj0G8mHDYCAfoPAQAAABAAAABfw/cdi0u5QaiqY9BvJhw2 AgH7DwEAAACEAAAAAAAAADihuxAF5RAaobsIACsqVsIAAG1zcHN0LmRsbAAAAAAATklUQfm/uAEA qgA32W4AAABDOlxEb2N1bWVudHMgYW5kIFNldHRpbmdzXEFydHVyXEFwcGxpY2F0aW9uIERhdGFc TWljcm9zb2Z0XE91dGxvb2tcT3V0bG9vay5wc3QAAwD+DwUAAAADAA00/TcCAAIBFDQBAAAAEAAA AE5JVEH5v7gBAKoAN9luAAACAX8AAQAAADEAAAAwMDAwMDAwMDVGQzNGNzFEOEI0QkI5NDFBOEFB NjNEMDZGMjYxQzM2NjQ1MTJCMDAAAAAAAwAGEIPbbvsDAAcQIwEAAAMAEBAAAAAAAwAREAAAAAAe AAgQAQAAAGUAAABPSSxRVUFORE9FVU1BTkRFSU1JTkhBUFJJTUVJUkFNRU5TQUdFTSxFVUVTVEFW QVBFTlNBTkRPRU1FU1BBQ09TVkVUT1JJQUlTREVGSU5JRE9TU09CUkVPQ09OSlVOVE9ET1NOAAAA AKl8 ------=_NextPart_000_0014_01C30C36.04849170-- ========================================================================= Instruções para entrar na lista, sair da lista e usar a lista em http://www.mat.puc-rio.br/~nicolau/olimp/obm-l.html ========================================================================= From owner-obm-l@sucuri.mat.puc-rio.br Sat Apr 26 21:21:01 2003 Return-Path: Received: (from majordom@localhost) by sucuri.mat.puc-rio.br (8.9.3/8.9.3) id VAA29866 for obm-l-MTTP; Sat, 26 Apr 2003 21:18:10 -0300 Received: from itaqui.terra.com.br (itaqui.terra.com.br [200.176.3.19]) by sucuri.mat.puc-rio.br (8.9.3/8.9.3) with ESMTP id VAA29860 for ; Sat, 26 Apr 2003 21:18:06 -0300 Received: from altamira.terra.com.br (altamira.terra.com.br [200.176.3.40]) by itaqui.terra.com.br (Postfix) with ESMTP id E9BB43BC978 for ; Sat, 26 Apr 2003 21:17:33 -0300 (BRT) Received: from [200.177.182.69] (dl-nas6-sao-C8B1B645.p001.terra.com.br [200.177.182.69]) by altamira.terra.com.br (Postfix) with ESMTP id ECE053DC086 for ; Sat, 26 Apr 2003 21:17:32 -0300 (BRT) User-Agent: Microsoft-Outlook-Express-Macintosh-Edition/5.02.2022 Date: Sat, 26 Apr 2003 21:19:01 -0300 Subject: Re: [obm-l] provar desiguldade.. From: Claudio Buffara To: Message-ID: In-Reply-To: <3EAB04B1.60806@niski.com> Mime-version: 1.0 Content-type: text/plain; charset="ISO-8859-1" Content-Transfer-Encoding: 8bit X-MIME-Autoconverted: from quoted-printable to 8bit by sucuri.mat.puc-rio.br id VAA29862 Sender: owner-obm-l@sucuri.mat.puc-rio.br Precedence: bulk Reply-To: obm-l@mat.puc-rio.br on 26.04.03 19:14, niski at fabio@niski.com wrote: > Claudio, creio que sua demonstracao ficou mais clara do que a minha , pq > eu utilizei "De fato pois o menor valor que 4x pode assumir é maior do > que 4(1/2) = 2." Certo? > Se sim, pq isso é falta de formalização? Será que esta passagem foi > muito intuitiva e por isso voce a classificou como não formal!? > obrigado > Oi, Niski: Repare que voce comecou operando com a desigualdade que queria provar e chegou em: |x - 1| <= 3x Entretanto, no meio voce dividiu ambos os membros por |x - 1|, o que nao eh valido se x = 1 (um valor pertencente ao dominio da variavel x). Por sorte, isso nao afetou o sentido da desigualdade. Alem disso, todos os outros passos eram reversiveis, de forma que, dado que x > 1/2: |x - 1| <= 3x se e somente se |x + 1/x - 2| <= 3|x - 1|, mas esse poderia nao ter sido o caso. Assim, para tornar sua solucao 100% rigorosa, faltou explicitar que, para x > 1/2, |x + 1/x - 2| <= 3|x - 1| eh equivalente a |x - 1| <= 3x. A partir dai tudo OK - voce apenas separou os casos 1/2 < x < 1 e x >= 1 e concluiu que, em ambos, a desigualdade eh verdadeira. Assim, a sua analise foi correta, porem incompleta e, digamos, "perigosa". De qualquer forma, o que voce fez corresponde a uma tecnica bastante eficaz de exploracao de um problema - resolve-lo de tras pra frente (tomando sempre o cuidado de se verificar que cada passo eh reversivel, ou seja, da forma "se e somente se"). Alem disso, so por uma questao de rigor, como o dominio da variavel x eh (1/2,+infinito), ou seja, um intervalo aberto, 4x nao atinge, de fato, um valor minimo (apesar de termos inf(4x) = 2). Espero que tenha ficado claro. Um abraco, Claudio. > Claudio Buffara wrote: >> on 26.04.03 16:18, niski at fabio@niski.com wrote: >> >> >>> Por favor pessoal, me ajudem nesta questão, obrigado: >>> >>> Seja f(x) = x + 1/x >>> >>> Prove que >>> >>> |f(x) - f(1)| <= 3|x-1| , para x > 1/2 >>> >>> Demonstracao >>> |x+ (1/x) - 2 | <= 3|x-1| >>> |(x^2 -2x +1)/x| <= 3|x-1| >>> (|x-1||x-1|)/x <= 3|x-1| >>> |x-1|/x <= 3 >>> |x-1| <= 3x >>> >>> Se 1/2 < x < 1 >>> >>> -x+1 < 3x >>> 1 < 4x >>> >>> De fato pois o menor valor que 4x pode assumir é maior do que 4(1/2) = 2. >>> >>> Se x >= 1 >>> >>> |x-1| <= 3x >>> x-1 < 3x >>> -1 < 4x >>> >>> De fato pois o menor valor que 4x pode assumir é 4. >>> >>> Esta certa esta demonstração?! >>> Obrigado. >>> ========================================================================= Instruções para entrar na lista, sair da lista e usar a lista em http://www.mat.puc-rio.br/~nicolau/olimp/obm-l.html ========================================================================= From owner-obm-l@sucuri.mat.puc-rio.br Sat Apr 26 21:27:11 2003 Return-Path: Received: (from majordom@localhost) by sucuri.mat.puc-rio.br (8.9.3/8.9.3) id VAA30219 for obm-l-MTTP; Sat, 26 Apr 2003 21:24:31 -0300 Received: from traven.uol.com.br (traven.uol.com.br [200.221.29.39]) by sucuri.mat.puc-rio.br (8.9.3/8.9.3) with ESMTP id VAA30214 for ; Sat, 26 Apr 2003 21:24:28 -0300 Received: from gauss ([200.158.96.143]) by traven.uol.com.br (8.9.1/8.9.1) with SMTP id VAA22708 for ; Sat, 26 Apr 2003 21:23:56 -0300 (BRT) Message-ID: <008b01c30c54$0b6a5510$8f609ec8@gauss> From: "Domingos Jr." To: Subject: [obm-l] integral de 1/x Date: Sat, 26 Apr 2003 21:29:17 -0300 MIME-Version: 1.0 Content-Type: text/plain; charset="iso-8859-1" Content-Transfer-Encoding: 8bit X-Priority: 3 X-MSMail-Priority: Normal X-Mailer: Microsoft Outlook Express 6.00.2800.1158 X-MimeOLE: Produced By Microsoft MimeOLE V6.00.2800.1165 Sender: owner-obm-l@sucuri.mat.puc-rio.br Precedence: bulk Reply-To: obm-l@mat.puc-rio.br Um amigo me perguntou como demonstro que Integral(1/x) = ln(x), não soube dar a Resposta mas imaginei que talvez fosse possível usar série de Taylor/MacLaurin para chegar no resultado. Alguém conhece uma maneira de provar o resultado? ========================================================================= Instruções para entrar na lista, sair da lista e usar a lista em http://www.mat.puc-rio.br/~nicolau/olimp/obm-l.html ========================================================================= From owner-obm-l@sucuri.mat.puc-rio.br Sat Apr 26 21:31:31 2003 Return-Path: Received: (from majordom@localhost) by sucuri.mat.puc-rio.br (8.9.3/8.9.3) id VAA30361 for obm-l-MTTP; Sat, 26 Apr 2003 21:28:58 -0300 Received: from traven.uol.com.br (traven.uol.com.br [200.221.29.39]) by sucuri.mat.puc-rio.br (8.9.3/8.9.3) with ESMTP id VAA30354 for ; Sat, 26 Apr 2003 21:28:54 -0300 Received: from gauss ([200.158.96.143]) by traven.uol.com.br (8.9.1/8.9.1) with SMTP id VAA25551 for ; Sat, 26 Apr 2003 21:28:20 -0300 (BRT) Message-ID: <00a501c30c54$a8d95f30$8f609ec8@gauss> From: "Domingos Jr." To: References: Subject: [obm-l] =?iso-8859-1?Q?Re:_=5Bobm-l=5D_s=E9rie?= Date: Sat, 26 Apr 2003 21:33:43 -0300 MIME-Version: 1.0 Content-Type: text/plain; charset="iso-8859-1" Content-Transfer-Encoding: 8bit X-Priority: 3 X-MSMail-Priority: Normal X-Mailer: Microsoft Outlook Express 6.00.2800.1158 X-MimeOLE: Produced By Microsoft MimeOLE V6.00.2800.1165 Sender: owner-obm-l@sucuri.mat.puc-rio.br Precedence: bulk Reply-To: obm-l@mat.puc-rio.br > > p/ quanto converge a série 1/n^2????salvo > > engano o gugu me falou uma vez sobre esse valor mas eu > > não lembro > > > > > Pi^2/6. Dá pra chegar nesse resultado usando série de fourier, mas não lembro como... ========================================================================= Instruções para entrar na lista, sair da lista e usar a lista em http://www.mat.puc-rio.br/~nicolau/olimp/obm-l.html ========================================================================= From owner-obm-l@sucuri.mat.puc-rio.br Sat Apr 26 21:45:37 2003 Return-Path: Received: (from majordom@localhost) by sucuri.mat.puc-rio.br (8.9.3/8.9.3) id VAA31141 for obm-l-MTTP; Sat, 26 Apr 2003 21:42:32 -0300 Received: from paiol.terra.com.br (paiol.terra.com.br [200.176.3.18]) by sucuri.mat.puc-rio.br (8.9.3/8.9.3) with ESMTP id VAA31137 for ; Sat, 26 Apr 2003 21:42:28 -0300 Received: from barra.terra.com.br (barra.terra.com.br [200.176.3.52]) by paiol.terra.com.br (Postfix) with ESMTP id AF3738831B for ; Sat, 26 Apr 2003 21:41:58 -0300 (BRT) Received: from [200.177.192.141] (dl-nas2-sao-C8B1C08D.p001.terra.com.br [200.177.192.141]) by barra.terra.com.br (Postfix) with ESMTP id D180A234056 for ; Sat, 26 Apr 2003 21:41:57 -0300 (BRT) User-Agent: Microsoft-Outlook-Express-Macintosh-Edition/5.02.2022 Date: Sat, 26 Apr 2003 21:43:27 -0300 Subject: Re: [obm-l] Limites From: Claudio Buffara To: Message-ID: In-Reply-To: <200304262330.h3QNUG66006098@trex.centroin.com.br> Mime-version: 1.0 Content-type: text/plain; charset="ISO-8859-1" Content-Transfer-Encoding: 8bit X-MIME-Autoconverted: from quoted-printable to 8bit by sucuri.mat.puc-rio.br id VAA31138 Sender: owner-obm-l@sucuri.mat.puc-rio.br Precedence: bulk Reply-To: obm-l@mat.puc-rio.br Oi, Morgado: Eu diria que se sqrt eh raiz quadrada, entao cbrt eh raiz cubica. Aqui vai o meu apelo: caros colegas da lista, sqrt ainda dah pra encarar (apesar de eu preferir raiz(.)), mas cbrt eh um pouco demais. Vamos dar uma chance aos expoentes fracionarios! Quanto ao problema em si, acho que a ideia da mudanca de variaveis foi boa. x = pi/2 - z ==> x --> Pi/2 ==> z --> 0 cos(x) = cos(pi/2 - z) = sen(z) sen(x) = sen(pi/2 - z) = cos(z) (sen com "e", por favor - estamos no Brasil!) Assim, o limite fica: lim(z --> 0) sen(z)/(1 - cos(z))^(2/3) Usando aproximacao por series de Taylor, teremos: sen(z) = z - z^3/6 + O(z^5) cos(z) = 1 - z^2/2 + O(z^4) ==> sen(z)/(1 - cos(z))^(2/3) = (z - z^3/6 + O(z^5))/(z^2/2 + O(z^4))^(2/3) = (z + O(z^3))/O(z^(4/3)). Quando z --> 0+, o limite eh +infinito. Quando z --> 0-, o limite eh -infinito. Logo, esse limite nao existe. Um abraco, Claudio. on 26.04.03 20:30, Augusto Cesar de Oliveira Morgado at morgado@centroin.com.br wrote: > Que diabo eh cbrt? > > > Em Sat, 26 Apr 2003 14:50:53 -0700, niski disse: > >> Pessoal, por favor resolvam estes limites. nao me vem nenhuma >> ideia..obrigado >> >> lim[x->pi/2] cos(x)/cbrt((1-sinx)^2) >> >> Me ocorreu fazer a seguinte transformação : >> >> pi/2 - x = z >> >> Daí >> >> lim[z->0] sin(z)/cbrt((1-sin((pi/2)-z))^2) >> >> dai eu travei... >> >> outra coisa...gostaria de saber se sempre é valida subistituicao de >> variaveis...se nao for..quando nao é? >> >> obrigado >> >> niski ========================================================================= Instruções para entrar na lista, sair da lista e usar a lista em http://www.mat.puc-rio.br/~nicolau/olimp/obm-l.html ========================================================================= From owner-obm-l@sucuri.mat.puc-rio.br Sat Apr 26 22:24:48 2003 Return-Path: Received: (from majordom@localhost) by sucuri.mat.puc-rio.br (8.9.3/8.9.3) id WAA32522 for obm-l-MTTP; Sat, 26 Apr 2003 22:22:01 -0300 Received: from trex.centroin.com.br (trex.centroin.com.br [200.225.63.134]) by sucuri.mat.puc-rio.br (8.9.3/8.9.3) with ESMTP id WAA32509 for ; Sat, 26 Apr 2003 22:21:53 -0300 Received: from trex.centroin.com.br (localhost [127.0.0.1]) by trex.centroin.com.br (8.12.9/8.12.9) with ESMTP id h3R1LNkZ011611 for ; Sat, 26 Apr 2003 22:21:23 -0300 (EST) Received: by trex.centroin.com.br (8.12.9/8.12.5/Submit) id h3R1LNNF011610; Sat, 26 Apr 2003 22:21:23 -0300 (EST) Message-Id: <200304270121.h3R1LNNF011610@trex.centroin.com.br> Received: from 200.141.90.214 by trex.centroin.com.br (CIPWM versao 1.4C1) with HTTPS for ; Sat, 26 Apr 2003 22:21:23 -0300 (EST) Date: Sat, 26 Apr 2003 22:21:23 -0300 (EST) From: Augusto Cesar de Oliveira Morgado To: obm-l@mat.puc-rio.br Subject: =?iso-8859-1?q?Re: [obm-l] Re:_[obm-l]_s=E9rie?= MIME-Version: 1.0 X-Mailer: CentroIn Internet Provider WebMail v. 1.4C1 (http://www.centroin.com.br/) Content-Type: text/plain; charset="iso-8859-1" Content-Transfer-Encoding: 8bit X-MIME-Autoconverted: from quoted-printable to 8bit by sucuri.mat.puc-rio.br id WAA32513 Sender: owner-obm-l@sucuri.mat.puc-rio.br Precedence: bulk Reply-To: obm-l@mat.puc-rio.br Desenvolva f(x) = x e use a identidade da soma dos quadrados dos coeficientes. (ou desenvolva f(x) = x^2 e deh a x um valor conveniente) Em Sat, 26 Apr 2003 21:33:43 -0300, "Domingos Jr." disse: > > > p/ quanto converge a série 1/n^2????salvo > > > engano o gugu me falou uma vez sobre esse valor mas eu > > > não lembro > > > > > > > > Pi^2/6. > > Dá pra chegar nesse resultado usando série de fourier, mas não lembro > como... > > ========================================================================= > Instruções para entrar na lista, sair da lista e usar a lista em > http://www.mat.puc-rio.br/~nicolau/olimp/obm-l.html > ========================================================================= > > ========================================================================= Instruções para entrar na lista, sair da lista e usar a lista em http://www.mat.puc-rio.br/~nicolau/olimp/obm-l.html ========================================================================= From owner-obm-l@sucuri.mat.puc-rio.br Sat Apr 26 22:53:33 2003 Return-Path: Received: (from majordom@localhost) by sucuri.mat.puc-rio.br (8.9.3/8.9.3) id WAA00692 for obm-l-MTTP; Sat, 26 Apr 2003 22:50:26 -0300 Received: from smtp011.mail.yahoo.com (smtp011.mail.yahoo.com [216.136.173.31]) by sucuri.mat.puc-rio.br (8.9.3/8.9.3) with SMTP id WAA00688 for ; Sat, 26 Apr 2003 22:50:21 -0300 Received: from unknown (HELO ricardofilho) (lhradiohead@200.164.123.170 with login) by smtp.mail.vip.sc5.yahoo.com with SMTP; 27 Apr 2003 01:49:49 -0000 Message-ID: <002701c30bc9$85be9e60$54cdfea9@ricardofilho> From: "Ricardo Filho" To: "obm" Subject: [obm-l] =?iso-8859-1?Q?polin=F4mio?= Date: Sat, 26 Apr 2003 04:57:37 -0300 MIME-Version: 1.0 Content-Type: text/plain; charset="iso-8859-1" Content-Transfer-Encoding: 7bit X-Priority: 3 X-MSMail-Priority: Normal X-Mailer: Microsoft Outlook Express 5.50.4807.1700 X-MimeOLE: Produced By Microsoft MimeOLE V5.50.4807.1700 Sender: owner-obm-l@sucuri.mat.puc-rio.br Precedence: bulk Reply-To: obm-l@mat.puc-rio.br Ola amigos da lista,alguem pode me ajudar nessa questao? Se x1 e x2 sao as raizes do polinomio x^2 - 6x + 1 = 0, entao podemos afirmar que x1^2001+x2^2001: a)nao e um numero inteiro b)e zero c)e um inteiro multiplo de 5 d)e um inteiro nao multiplo de 5 e)e 2001 RESPOSTA: D Obrigado ========================================================================= Instruções para entrar na lista, sair da lista e usar a lista em http://www.mat.puc-rio.br/~nicolau/olimp/obm-l.html ========================================================================= From owner-obm-l@sucuri.mat.puc-rio.br Sat Apr 26 23:22:41 2003 Return-Path: Received: (from majordom@localhost) by sucuri.mat.puc-rio.br (8.9.3/8.9.3) id XAA01542 for obm-l-MTTP; Sat, 26 Apr 2003 23:20:03 -0300 Received: from ivoti.terra.com.br (ivoti.terra.com.br [200.176.3.20]) by sucuri.mat.puc-rio.br (8.9.3/8.9.3) with ESMTP id XAA01532 for ; Sat, 26 Apr 2003 23:19:57 -0300 Received: from botucatu.terra.com.br (botucatu.terra.com.br [200.176.3.78]) by ivoti.terra.com.br (Postfix) with ESMTP id 8FAB640849B for ; Sat, 26 Apr 2003 23:19:26 -0300 (BRT) Received: from riemann.localdomain (RJ173061.user.veloxzone.com.br [200.149.173.61]) (authenticated user fabio.dias.moreira) by botucatu.terra.com.br (Postfix) with ESMTP id 1798E29C05B for ; Sat, 26 Apr 2003 23:19:26 -0300 (BRT) Content-Type: text/plain; charset="iso-8859-1" From: =?iso-8859-1?q?F=E1bio=20Dias=20Moreira?= To: obm-l@mat.puc-rio.br Subject: Re: [obm-l] =?iso-8859-1?q?polin=F4mio?= Date: Sat, 26 Apr 2003 23:19:09 -0300 User-Agent: KMail/1.4.3 References: <002701c30bc9$85be9e60$54cdfea9@ricardofilho> In-Reply-To: <002701c30bc9$85be9e60$54cdfea9@ricardofilho> MIME-Version: 1.0 Content-Transfer-Encoding: 8bit Message-Id: <200304262319.22486.fabio.dias.moreira@terra.com.br> Sender: owner-obm-l@sucuri.mat.puc-rio.br Precedence: bulk Reply-To: obm-l@mat.puc-rio.br -----BEGIN PGP SIGNED MESSAGE----- Hash: SHA1 On Saturday 26 April 2003 04:57, Ricardo Filho wrote: > Ola amigos da lista,alguem pode me ajudar nessa questao? > > Se x1 e x2 sao as raizes do polinomio x^2 - 6x + 1 = 0, entao podemos > afirmar que x1^2001+x2^2001: > > a)nao e um numero inteiro > b)e zero > c)e um inteiro multiplo de 5 > d)e um inteiro nao multiplo de 5 > e)e 2001 > > RESPOSTA: D > [...] Seja z_n = x_1^n + x_2^n. Então z_0 = 2, z_1 = 6 e z_{i+2} = 6*z_{i+1} - z_i. Então não é muito difícil ver que z_n é sempre inteiro e crescente -- logo (a) e (b) são impossíveis. A recorrência cresce bastante rápido, então (e) também está errada. Para decidir entre (c) e (d), escreva a equação de recorrência módulo 5, ache valores de z_n até surgir um período e conclua quanto vale z_2001 (mod 5). []s, - -- Fábio "ctg \pi" Dias Moreira -----BEGIN PGP SIGNATURE----- Version: GnuPG v1.0.6 (GNU/Linux) Comment: For info see http://www.gnupg.org iD8DBQE+qz4qalOQFrvzGQoRAjPZAJ9I4iXNHJaRHfmMAvVJ53z1YdFdlwCgsd3i DgvBXbDhu0MMCH4K8SCGsBk= =u4Z2 -----END PGP SIGNATURE----- ========================================================================= Instruções para entrar na lista, sair da lista e usar a lista em http://www.mat.puc-rio.br/~nicolau/olimp/obm-l.html ========================================================================= From owner-obm-l@sucuri.mat.puc-rio.br Sun Apr 27 01:08:03 2003 Return-Path: Received: (from majordom@localhost) by sucuri.mat.puc-rio.br (8.9.3/8.9.3) id BAA03055 for obm-l-MTTP; Sun, 27 Apr 2003 01:05:21 -0300 Received: from web21304.mail.yahoo.com (web21304.mail.yahoo.com [216.136.129.190]) by sucuri.mat.puc-rio.br (8.9.3/8.9.3) with SMTP id BAA03051 for ; Sun, 27 Apr 2003 01:05:16 -0300 Message-ID: <20030427040445.10703.qmail@web21304.mail.yahoo.com> Received: from [200.147.124.106] by web21304.mail.yahoo.com via HTTP; Sun, 27 Apr 2003 01:04:45 ART Date: Sun, 27 Apr 2003 01:04:45 -0300 (ART) From: =?iso-8859-1?q?Marcos=20Reynaldo?= Subject: Re: [obm-l] integral de 1/x To: obm-l@mat.puc-rio.br In-Reply-To: <008b01c30c54$0b6a5510$8f609ec8@gauss> MIME-Version: 1.0 Content-Type: text/plain; charset=iso-8859-1 Content-Transfer-Encoding: 8bit Sender: owner-obm-l@sucuri.mat.puc-rio.br Precedence: bulk Reply-To: obm-l@mat.puc-rio.br O resultado da integral não seria ln |x| ? ln|x|=ln(-x) se x<0 e ln|x|=ln(x) se x>0. Para o primeiro caso a derivada, de ln(-x), é -1/(-x)=1/x. Para o segundo caso, temos também 1/x. Assim em ambos os casos a derivada é 1/x. Então ln|x| é uma primitiva de 1/x, e portanto a sua integral é ln|x|+C. Agora no caso de integrais definidas, uma maneira intuitiva de ver isso, é que a área entre a curva 1/x o eixo x e retas 1 e x é ln(x) (x>0). Assim a integral de 1/x é ln(x). Não sei se minha explicação convence mas alguém da lista deve dar uma demonstração mais formal. []'s Marcos _______________________________________________________________________ Yahoo! Mail O melhor e-mail gratuito da internet: 6MB de espaço, antivírus, acesso POP3, filtro contra spam. http://br.mail.yahoo.com/ ========================================================================= Instruções para entrar na lista, sair da lista e usar a lista em http://www.mat.puc-rio.br/~nicolau/olimp/obm-l.html ========================================================================= From owner-obm-l@sucuri.mat.puc-rio.br Sun Apr 27 01:19:49 2003 Return-Path: Received: (from majordom@localhost) by sucuri.mat.puc-rio.br (8.9.3/8.9.3) id BAA03289 for obm-l-MTTP; Sun, 27 Apr 2003 01:17:10 -0300 Received: from Euler.impa.br (euler.impa.br [147.65.1.3]) by sucuri.mat.puc-rio.br (8.9.3/8.9.3) with ESMTP id BAA03285 for ; Sun, 27 Apr 2003 01:17:06 -0300 Received: from Gauss.impa.br (Gauss [147.65.4.1]) by Euler.impa.br (8.11.6p2/8.11.6) with ESMTP id h3R4Ga019288 for ; Sun, 27 Apr 2003 01:16:36 -0300 (EST) From: Carlos Gustavo Tamm de Araujo Moreira Received: by Gauss.impa.br (8.11.6p2) id h3R4GKp10274; Sun, 27 Apr 2003 01:16:20 -0300 (EST) Message-Id: <200304270416.h3R4GKp10274@Gauss.impa.br> Subject: Re: [obm-l] serie do Marcio To: obm-l@mat.puc-rio.br Date: Sun, 27 Apr 2003 01:16:20 -0300 (EST) In-Reply-To: from "Claudio Buffara" at Apr 26, 3 07:26:14 pm X-Mailer: ELM [version 2.4 PL25] MIME-Version: 1.0 Content-Type: text/plain; charset=US-ASCII Content-Transfer-Encoding: 7bit Sender: owner-obm-l@sucuri.mat.puc-rio.br Precedence: bulk Reply-To: obm-l@mat.puc-rio.br Oi Claudio, Sua prova esta' certa, e segue da seguinte proposicao facil de provar: se soma(n=1 a infinito)(a_n) converge a A e soma(n=1 a infinito)(b_n) converge a B entao a_1+a_2+...+a_k(1)+b_1+a_(k(1)+1)+a_(k(1)+2)+...+a_k(2)+b_2+... converge a A+B, para qualquer sequencia crescente de inteiros positivos k(n) (note que os termos de a(n) e de b(n) estao entrando na ordem original e se somamos muitos termos da terceira serie estamos somando muitos termos da primeira serie e muitos termos da segunda serie). Abracos, Gugu >Oi, Marcio e Luis: > >Realmente eh um problema intrigante. Empiricamente (com uma planilha) eu me >convenci que a soma eh 1,5*Ln(2). > >Alem disso, vejam esse algebrismo: > > Ln(2) = 1-1/2+1/3-1/4+1/5-1/6+1/7-1/8+1/9-1/10+1/11-1/12+1/13-1/14+... > >0,5*Ln(2) = 1/2 -1/4 +1/6 -1/8 +1/10 -1/12 +1/14 - ... > >Somando: >1,5*Ln(2) = 1 +1/3-1/2+1/5 +1/7-1/4+1/9 +1/11-1/6+1/3 + ... > >Rearranjando os termos: >1,5*Ln(2) = (1+1/3-1/2) + (1/5+1/7-1/4) + (1/9+1/11-1/6) + ... > >Ou seja, >1,5*Ln(2) = SOMA(n >= 0) [1/(4n+1) + 1/(4n+3) - 1/(2n+2)] > >Assim, o algebrismo funciona apesar de eu nao ter certeza do rigor, dado que >estamos lidando com uma serie condicionalmente convergente. > >Pergunta: o que eu fiz pode ser justificado com rigor ou eu achei o >resultado correto por pura sorte? > >Um abraco, >Claudio. > > >on 25.04.03 11:39, Luis Lopes at llopes@ensrbr.com.br wrote: > >> Sauda,c~oes, >> >> Acabo de mandar uma msg boba por >> bater no botco errado. >> >> Nco sei se estou falando com o mesmo Marcio. >> >>> Quando exatamente eu posso trocar a ordem da >>> integral com o somatorio? >>> Por exemplo, considere a serie cujo n-o termo eh >>> s_n = 1/(4n+1) + 1/(4n+3) - 1/(2n+2), cujos termos >>> sco todos positivos. >>> Ela converge, por comparagco com a sirie a/n^2. >>> >>> Para calcular Somatorio (0 a infinito) s_n, eu pensei >>> em calcular: >>> Somatorio(0 a infinito)_Integral (0 a 1) >>> [x^4n + x^(4n+2) - x^(2n+1)] >>> Trocando a ordem, ficamos com algumas PG's e: >>> Integral (0 a 1) [1/(1-x^4) + x^2 / (1-x^4) - x/(1-x^2)] = >>> Integral (0 a 1) [1/(1+x)] = ln2 >>> >>> Mas eu vi que essa soma vale, na verdade, 1.5 ln2 >>> (inclusive me provaram isso, e parece estar certo).. >>> >>> Por outro lado, em outros problemas esse mitodo >>> funciona bem.. Por exemplo, >>> para calcular Somatorio ( 1/[(3n+1)*(3n+2)*(3n+3)] ) >>> a resposta parece dar correta.. >>> >>> Abracos, >>> Marcio >> >> Tentei e nco consegui mostrar que da 1.5 ln2. >> >> Como faz? >> >> []'s >> Lums >> >> ========================================================================= Instruções para entrar na lista, sair da lista e usar a lista em http://www.mat.puc-rio.br/~nicolau/olimp/obm-l.html ========================================================================= From owner-obm-l@sucuri.mat.puc-rio.br Sun Apr 27 01:29:28 2003 Return-Path: Received: (from majordom@localhost) by sucuri.mat.puc-rio.br (8.9.3/8.9.3) id BAA03495 for obm-l-MTTP; Sun, 27 Apr 2003 01:26:54 -0300 Received: from itaqui.terra.com.br (itaqui.terra.com.br [200.176.3.19]) by sucuri.mat.puc-rio.br (8.9.3/8.9.3) with ESMTP id BAA03491 for ; Sun, 27 Apr 2003 01:26:51 -0300 Received: from barra.terra.com.br (barra.terra.com.br [200.176.3.52]) by itaqui.terra.com.br (Postfix) with ESMTP id 975ED3BC59F for ; Sun, 27 Apr 2003 01:26:20 -0300 (BRT) Received: from [200.177.179.96] (dl-nas3-sao-C8B1B360.p001.terra.com.br [200.177.179.96]) by barra.terra.com.br (Postfix) with ESMTP id ED58223404A for ; Sun, 27 Apr 2003 01:26:19 -0300 (BRT) User-Agent: Microsoft-Outlook-Express-Macintosh-Edition/5.02.2022 Date: Sun, 27 Apr 2003 01:27:50 -0300 Subject: [obm-l] Pontos pintados From: Claudio Buffara To: Lista OBM Message-ID: Mime-version: 1.0 Content-type: text/plain; charset="US-ASCII" Content-transfer-encoding: 7bit Sender: owner-obm-l@sucuri.mat.puc-rio.br Precedence: bulk Reply-To: obm-l@mat.puc-rio.br Caros colegas da lista: Outro probleminha que estah me dando trabalho: Cada ponto do plano eh pintado de uma cor, dentre tres cores possiveis. Prove que existe um segmento unitario cujas extremidades tem a mesma cor. ****** Uma equacao diofantina bonitinha: Prove que x^2 + (x+1)^2 = y^3 nao tem solucao em inteiros positivos. ****** E aqui vai a dica pro problema da sequencia de 100 numeros reais e das subsequencias de 8 e 9 termos com mesma media: suponha inicialmente que os termos da sequencia sao racionais. Em seguida, use o fato de que R eh um espaco vetorial sobre Q. Seria otimo se alguem descobrisse uma solucao que nao usasse a dica. Um abraco, Claudio. ========================================================================= Instruções para entrar na lista, sair da lista e usar a lista em http://www.mat.puc-rio.br/~nicolau/olimp/obm-l.html ========================================================================= From owner-obm-l@sucuri.mat.puc-rio.br Sun Apr 27 02:00:52 2003 Return-Path: Received: (from majordom@localhost) by sucuri.mat.puc-rio.br (8.9.3/8.9.3) id BAA04645 for obm-l-MTTP; Sun, 27 Apr 2003 01:58:16 -0300 Received: from mail.gmx.net (mail.gmx.net [213.165.64.20]) by sucuri.mat.puc-rio.br (8.9.3/8.9.3) with SMTP id BAA04641 for ; Sun, 27 Apr 2003 01:58:12 -0300 Received: (qmail 1916 invoked by uid 65534); 27 Apr 2003 04:57:34 -0000 Received: from unknown (EHLO localhost) (200.217.15.178) by mail.gmx.net (mp012-rz3) with SMTP; 27 Apr 2003 06:57:34 +0200 Date: Sun, 27 Apr 2003 01:56:58 -0300 From: Igor GomeZZ X-Mailer: The Bat! (v1.61) Organization: -- X-Priority: 3 (Normal) Message-ID: <3142753235.20030427015658@gmx.net> To: =?ISO-8859-1?B?TGVhbmRybyBMYWNvcnRlIFJlY/R2YQ==?= Subject: Re[2]: [obm-l] Limites In-Reply-To: <000001c30c4c$c388dc00$287d1843@LeandroRecova> References: <000001c30c4c$c388dc00$287d1843@LeandroRecova> MIME-Version: 1.0 Content-Type: text/plain; charset=ISO-8859-1 Content-Transfer-Encoding: 8bit Sender: owner-obm-l@sucuri.mat.puc-rio.br Precedence: bulk Reply-To: obm-l@mat.puc-rio.br Em 26/4/2003, 20:37, Leandro (lrecova@hotmail.com) disse: >>= lim[x->>pi/2] 1/cbrt((1-sinx)^2/cos^3(x))) >>= lim[x->>pi/2] 1/cbrt((cos^2(x)/cos^3(x))) Aqui tem uma falha: (1-sen(x))^2 != 1 - sen(x)^2 e portanto, diferente de cos(x)^2 obs: A simbologia pra diferente foi != lim[x->pi/2] cos(x)/cbrt((1-sinx)^2); * cbrt[(1+senx)^2] lim[x->pi/2] cos(x)cbrt[(1+senx)^2] / cbrt[(1-senx)^2(1+senx)^2] lim[x->pi/2] cos(x)cbrt[(1+senx)^2] / cbrt[(1-sen(x)^2)^2] lim[x->pi/2] cos(x)cbrt[(1+senx)^2] / cbrt(cos(x)^4) lim[x->pi/2] cbrt[(1+senx)^2] / (cbrt(cos(x)^4)/cosx) lim[x->pi/2] cbrt[(1+senx)^2] / (cbrt(cos(x)^4/cos(x)^3)) lim[x->pi/2] cbrt[(1+senx)^2] / cbrt(cosx) lim[x->pi/2] cbrt[(1+senx)^2 / cosx] lim[x->pi/2] cbrt[(1+2senx+sen(x)^2 / cosx] lim[x->pi/2] cbrt[(1+1+2senx+(sen(x)^2 - 1) / cosx] lim[x->pi/2] cbrt[(2 + 2senx - cos(x)^2 / cosx] lim[x->pi/2] cbrt[2secx + 2tgx - cos(x)] Mas esse limite por um lado eh +inf e por outro -inf, logo ele não existe. Fui! ####### Igor GomeZZ ######## UIN: 29249895 Vitória, Espírito Santo, Brasil Criação: 26/4/2003 (22:51) #################################### Pare para pensar: O rio atinge seus objetivos porque aprendeu a contornar obstáculos. (Lao- Tsé) #################################### ========================================================================= Instruções para entrar na lista, sair da lista e usar a lista em http://www.mat.puc-rio.br/~nicolau/olimp/obm-l.html ========================================================================= From owner-obm-l@sucuri.mat.puc-rio.br Sun Apr 27 02:10:25 2003 Return-Path: Received: (from majordom@localhost) by sucuri.mat.puc-rio.br (8.9.3/8.9.3) id CAA04897 for obm-l-MTTP; Sun, 27 Apr 2003 02:07:46 -0300 Received: from Euler.impa.br (euler.impa.br [147.65.1.3]) by sucuri.mat.puc-rio.br (8.9.3/8.9.3) with ESMTP id CAA04893 for ; Sun, 27 Apr 2003 02:07:42 -0300 Received: from Gauss.impa.br (Gauss [147.65.4.1]) by Euler.impa.br (8.11.6p2/8.11.6) with ESMTP id h3R57B020788 for ; Sun, 27 Apr 2003 02:07:11 -0300 (EST) From: Carlos Gustavo Tamm de Araujo Moreira Received: by Gauss.impa.br (8.11.6p2) id h3R56uI15953; Sun, 27 Apr 2003 02:06:56 -0300 (EST) Message-Id: <200304270506.h3R56uI15953@Gauss.impa.br> Subject: Re: [obm-l] Pontos pintados To: obm-l@mat.puc-rio.br Date: Sun, 27 Apr 2003 02:06:56 -0300 (EST) In-Reply-To: from "Claudio Buffara" at Apr 27, 3 01:27:50 am X-Mailer: ELM [version 2.4 PL25] MIME-Version: 1.0 Content-Type: text/plain; charset=US-ASCII Content-Transfer-Encoding: 7bit Sender: owner-obm-l@sucuri.mat.puc-rio.br Precedence: bulk Reply-To: obm-l@mat.puc-rio.br Caro Claudio, Suponha que em todo segmento unitario as extremidades tem cores distintas.Se X e Y estao a distancia raiz(3), e' possivel achar A e B no plano tais que os triangulos XAB e ABY sejam equilateros de lado 1. Assim, A e B tem cores distintas e diferentes da cor de X, donde a cor de Y deve ser igual a cor de X. Considere agora um triangulo XYZ com lados XY=XZ=raiz(3) e YZ=1. Temos que as cores de Y e de Z devem ser iguais a cor de X mas a cor de Y deve ser distinta da cor de Z, absurdo. Eu sei fazer o seu problema da equacao diofantina usando um pouco de aritmetica em Z[i]. Voce tem uma solucao que nao usa isso ? Abracos, Gugu > >Caros colegas da lista: > >Outro probleminha que estah me dando trabalho: > >Cada ponto do plano eh pintado de uma cor, dentre tres cores possiveis. >Prove que existe um segmento unitario cujas extremidades tem a mesma cor. > >****** > >Uma equacao diofantina bonitinha: > >Prove que x^2 + (x+1)^2 = y^3 nao tem solucao em inteiros positivos. > >****** > >E aqui vai a dica pro problema da sequencia de 100 numeros reais e das >subsequencias de 8 e 9 termos com mesma media: suponha inicialmente que os >termos da sequencia sao racionais. Em seguida, use o fato de que R eh um >espaco vetorial sobre Q. > >Seria otimo se alguem descobrisse uma solucao que nao usasse a dica. > >Um abraco, >Claudio. > >========================================================================= >Instruções para entrar na lista, sair da lista e usar a lista em >http://www.mat.puc-rio.br/~nicolau/olimp/obm-l.html >========================================================================= ========================================================================= Instruções para entrar na lista, sair da lista e usar a lista em http://www.mat.puc-rio.br/~nicolau/olimp/obm-l.html ========================================================================= From owner-obm-l@sucuri.mat.puc-rio.br Sun Apr 27 02:11:24 2003 Return-Path: Received: (from majordom@localhost) by sucuri.mat.puc-rio.br (8.9.3/8.9.3) id CAA04952 for obm-l-MTTP; Sun, 27 Apr 2003 02:08:50 -0300 Received: from ivoti.terra.com.br (ivoti.terra.com.br [200.176.3.20]) by sucuri.mat.puc-rio.br (8.9.3/8.9.3) with ESMTP id CAA04948 for ; Sun, 27 Apr 2003 02:08:46 -0300 Received: from canela.terra.com.br (canela.terra.com.br [200.176.3.79]) by ivoti.terra.com.br (Postfix) with ESMTP id 52F17408212 for ; Sun, 27 Apr 2003 02:08:15 -0300 (BRT) Received: from niski.com (unknown [200.148.201.35]) (authenticated user fniski) by canela.terra.com.br (Postfix) with ESMTP id 58E1D22408B for ; Sun, 27 Apr 2003 02:08:14 -0300 (BRT) Message-ID: <3EAB65C5.7030704@niski.com> Date: Sat, 26 Apr 2003 22:08:21 -0700 From: niski User-Agent: Mozilla/5.0 (Windows; U; Windows NT 5.1; en-US; rv:1.0.2) Gecko/20030208 Netscape/7.02 X-Accept-Language: en-us, en MIME-Version: 1.0 To: obm-l@mat.puc-rio.br Subject: Re: [obm-l] Limites References: Content-Type: text/plain; charset=ISO-8859-1; format=flowed Content-Transfer-Encoding: 8bit Sender: owner-obm-l@sucuri.mat.puc-rio.br Precedence: bulk Reply-To: obm-l@mat.puc-rio.br Pretendo nesta mensagem responder ao Leandro Recova, ao prof. Morgado, e ao Claudio Buffara. prof. Morgado, desculpe.Eu estava acostumado com o acrônimo cbrt (raiz cubica) e não fui cuidadoso o suficiente para fazer tal referencia. Claudio e Leandro : Muito estranho... Para o Leandro o limite deu 0, para o Buffara não existe, no Mathematica deu -infinito e a resposta do livro é infinito !!!! e agora? Quanto a utilizar sin no lugar de sen , não acho que seja um grande problema, alias sin remete ao termo original em latim "sinus". Na minha opinião nao acho uma boa a lingua portuguesa mudar alguns termos originais...matriz por exemplo..pq este Z? pq nao deixar do originial cunhado por Sylvester!?!? Já vi coisas tb do tipo "Onduletas" para designar wavelets...eu acho que seria a mesma coisa que se referir ao Newton como "Isaque Newton". -- [about him:] It is rare to find learned men who are clean, do not stink and have a sense of humour. -Gottfried Whilhem Leibniz ========================================================================= Instruções para entrar na lista, sair da lista e usar a lista em http://www.mat.puc-rio.br/~nicolau/olimp/obm-l.html ========================================================================= From owner-obm-l@sucuri.mat.puc-rio.br Sun Apr 27 02:15:27 2003 Return-Path: Received: (from majordom@localhost) by sucuri.mat.puc-rio.br (8.9.3/8.9.3) id CAA05046 for obm-l-MTTP; Sun, 27 Apr 2003 02:12:55 -0300 Received: from itaqui.terra.com.br (itaqui.terra.com.br [200.176.3.19]) by sucuri.mat.puc-rio.br (8.9.3/8.9.3) with ESMTP id CAA05042 for ; Sun, 27 Apr 2003 02:12:52 -0300 Received: from bertioga.terra.com.br (bertioga.terra.com.br [200.176.3.77]) by itaqui.terra.com.br (Postfix) with ESMTP id EDE463BC986 for ; Sun, 27 Apr 2003 02:12:20 -0300 (BRT) Received: from niski.com (unknown [200.148.201.35]) (authenticated user fniski) by bertioga.terra.com.br (Postfix) with ESMTP id 0E8CB3F8039 for ; Sun, 27 Apr 2003 02:12:20 -0300 (BRT) Message-ID: <3EAB66B8.4040904@niski.com> Date: Sat, 26 Apr 2003 22:12:24 -0700 From: niski User-Agent: Mozilla/5.0 (Windows; U; Windows NT 5.1; en-US; rv:1.0.2) Gecko/20030208 Netscape/7.02 X-Accept-Language: en-us, en MIME-Version: 1.0 To: obm-l@mat.puc-rio.br Subject: Re: [obm-l] provar desiguldade.. References: Content-Type: text/plain; charset=us-ascii; format=flowed Content-Transfer-Encoding: 7bit Sender: owner-obm-l@sucuri.mat.puc-rio.br Precedence: bulk Reply-To: obm-l@mat.puc-rio.br > Alem disso, so por uma questao de rigor, como o dominio da variavel x eh > (1/2,+infinito), ou seja, um intervalo aberto, 4x nao atinge, de fato, um > valor minimo (apesar de termos inf(4x) = 2). > > Espero que tenha ficado claro. Sim Claudio. Obrigado pelas suas explicacoes! Niski -- [about him:] It is rare to find learned men who are clean, do not stink and have a sense of humour. -Gottfried Whilhem Leibniz ========================================================================= Instruções para entrar na lista, sair da lista e usar a lista em http://www.mat.puc-rio.br/~nicolau/olimp/obm-l.html ========================================================================= From owner-obm-l@sucuri.mat.puc-rio.br Sun Apr 27 03:58:27 2003 Return-Path: Received: (from majordom@localhost) by sucuri.mat.puc-rio.br (8.9.3/8.9.3) id DAA07654 for obm-l-MTTP; Sun, 27 Apr 2003 03:55:09 -0300 Received: from acsxe0.ac.brahma (smtp01.ambev.com.br [200.212.186.163]) by sucuri.mat.puc-rio.br (8.9.3/8.9.3) with ESMTP id DAA07650 for ; Sun, 27 Apr 2003 03:55:03 -0300 Received: from acsnx2.ac.brahma (acsnx2 [10.100.3.89]) by acsxe0.ac.brahma (AIX4.3/8.9.3/8.9.3) with ESMTP id DAA104930 for ; Sun, 27 Apr 2003 03:54:16 -0300 Received: by acsnx2.brahma with Internet Mail Service (5.5.2653.19) id ; Sun, 27 Apr 2003 03:53:39 -0300 Message-ID: <27BD56F8640DD711A4320006295078E50500C7@pssnx1.brahma> From: "Ricardo de Moraes (PS)" To: "'obm-l@mat.puc-rio.br'" Subject: [obm-l] Equacoes diferenciais Date: Sun, 27 Apr 2003 03:53:39 -0300 MIME-Version: 1.0 X-Mailer: Internet Mail Service (5.5.2653.19) Content-Type: multipart/alternative; boundary="----_=_NextPart_001_01C30C89.BBA23AD0" Sender: owner-obm-l@sucuri.mat.puc-rio.br Precedence: bulk Reply-To: obm-l@mat.puc-rio.br This message is in MIME format. Since your mail reader does not understand this format, some or all of this message may not be legible. ------_=_NextPart_001_01C30C89.BBA23AD0 Content-Type: text/plain; charset="ISO-8859-1" Content-Transfer-Encoding: quoted-printable Boa noite. =20 Resolvendo a equa=E7=E3o q segue, cheguei a um resultado um pouco = diferente do q o livro traz. E =F1 consegui encontrar o meu erro... =20 O problema diz,resolva a equa=E7=E3o: =20 t^2y' + 2ty - y^3 =3D 0, t>0 =20 Somando y^3 dos dois lados, dividindo por t^2 e por y^3 chego em=20 y'y^(-3) + (2/t)y^(-2) =3D t^(-2) (a) =20 Fazendo v =3D y^(-2) e v' =3D -2y^(-3)y' (daqui vem q v'/-2 =3D = y^(-3)y') e substituindo em (a) (v'/-2) + 2t^(-1)v =3D t^(-2) multiplicando por -2 v' - 4t^(-1)v =3D -2t^(-2) q =E9 linear de 1=AA ordem(?) =20 Resolvendo esta equa=E7=E3o em v, depois retornando para y ( v =3D = y^(-2) ) =20 A resposta q o livro traz =E9: y =3D + - [5t/(2 + 5ct^5)]^(1/2) =20 Eu achei: y =3D + - [5t/(2 - 10ct^5)]^(1/2) =20 Obs.: =E9 uma equa=E7=E3o de Bernoulli do livro Equa=E7=F5es = Diferenciais e Problemas de Valores de Contorno, de Boyce e Diprima. 7=AA ed. pag.40 n=BA28. =20 Espero q n=E3o tenha ficado muito confuso... =20 Obrigado, =20 Ricardo. ------_=_NextPart_001_01C30C89.BBA23AD0 Content-Type: text/html; charset="ISO-8859-1" Content-Transfer-Encoding: quoted-printable

Boa=20 noite.
 
Resolvendo a equa=E7=E3o=20 q segue, cheguei a um resultado um pouco diferente do q o livro traz. E = =F1=20 consegui encontrar o meu erro...
 
O = problema=20 diz,resolva a equa=E7=E3o:
 
t^2y' = + 2ty - y^3 =3D=20 0,      t>0
 
Somando y^3 dos dois=20 lados, dividindo por t^2 e por y^3 chego em
y'y^(-3) +=20 (2/t)y^(-2) =3D t^(-2) (a)
 
Fazendo v =3D y^(-2) e=20 v' =3D -2y^(-3)y' (daqui vem q v'/-2 =3D y^(-3)y') e substituindo em=20 (a)
(v'/-2) + 2t^(-1)v =3D=20 t^(-2) multiplicando por -2
v' - = 4t^(-1)v =3D=20 -2t^(-2) q =E9 linear de 1=AA ordem(?)
 
Resolvendo esta=20 equa=E7=E3o em v, depois retornando para y ( v =3D y^(-2) = )
 
A = resposta q o livro=20 traz =E9: y =3D + - [5t/(2 + 5ct^5)]^(1/2)
 
Eu = achei: y =3D + -=20 [5t/(2 - 10ct^5)]^(1/2)
 
Obs.: = =E9 uma equa=E7=E3o=20 de Bernoulli do livro Equa=E7=F5es Diferenciais e Problemas de = Valores de=20 Contorno, de Boyce e Diprima. 7=AA ed. pag.40 = n=BA28.
 
Espero q n=E3o tenha=20 ficado muito confuso...
 
Obrigado,
 
Ricardo.
------_=_NextPart_001_01C30C89.BBA23AD0-- ========================================================================= Instruções para entrar na lista, sair da lista e usar a lista em http://www.mat.puc-rio.br/~nicolau/olimp/obm-l.html ========================================================================= From owner-obm-l@sucuri.mat.puc-rio.br Sun Apr 27 05:36:25 2003 Return-Path: Received: (from majordom@localhost) by sucuri.mat.puc-rio.br (8.9.3/8.9.3) id FAA11939 for obm-l-MTTP; Sun, 27 Apr 2003 05:33:36 -0300 Received: from web21302.mail.yahoo.com (web21302.mail.yahoo.com [216.136.173.210]) by sucuri.mat.puc-rio.br (8.9.3/8.9.3) with SMTP id FAA11935 for ; Sun, 27 Apr 2003 05:33:30 -0300 Message-ID: <20030427083259.27051.qmail@web21302.mail.yahoo.com> Received: from [200.147.124.106] by web21302.mail.yahoo.com via HTTP; Sun, 27 Apr 2003 05:32:59 ART Date: Sun, 27 Apr 2003 05:32:59 -0300 (ART) From: =?iso-8859-1?q?Marcos=20Reynaldo?= Subject: Re: [obm-l] Pontos pintados To: obm-l@mat.puc-rio.br In-Reply-To: MIME-Version: 1.0 Content-Type: text/plain; charset=iso-8859-1 Content-Transfer-Encoding: 8bit Sender: owner-obm-l@sucuri.mat.puc-rio.br Precedence: bulk Reply-To: obm-l@mat.puc-rio.br > Uma equacao diofantina bonitinha: > > Prove que x^2 + (x+1)^2 = y^3 nao tem solucao em > inteiros positivos. Com relação ao problema acima ai vai uma tentativa: Admita que a equação tem solução em inteiros positivos. Neste caso, você conclui que y só pode ser um número impar (pois se x é par, x^2 é par, x+1 é impar e (x+1)^2 é impar e a soma de um numero par com um impar é impar; analogamente, se x for impar, conclui-se que y é impar). Bom então y é da forma 2n+1, onde n é inteiro (o caso em que n é 1 é facilmente verificado substituindo y por 1 donde resulta que x ou é 0 ou -1). Substituindo na expressão , vem x^2 + x^2+2x+1=2n+1 --> 2x^2 + 2x - 2n = 0 --> x^2 + x - n = 0 --> x = raiz(1+4n)/2. Ora, mas o resultado acima não é inteiro. Se fosse, x^2=(1+4n)/2 tambem seria, o que é um absurdo. Logo, o fato de considerarmos que a equação possui solução em inteiros positivos gerou uma contradição. Assim, a equação não possui solução em inteiros positivos. Dá uma olhada, se tiver algum erro me diz. []'s Marcos _______________________________________________________________________ Yahoo! Mail O melhor e-mail gratuito da internet: 6MB de espaço, antivírus, acesso POP3, filtro contra spam. http://br.mail.yahoo.com/ ========================================================================= Instruções para entrar na lista, sair da lista e usar a lista em http://www.mat.puc-rio.br/~nicolau/olimp/obm-l.html ========================================================================= From owner-obm-l@sucuri.mat.puc-rio.br Sun Apr 27 06:38:51 2003 Return-Path: Received: (from majordom@localhost) by sucuri.mat.puc-rio.br (8.9.3/8.9.3) id GAA12754 for obm-l-MTTP; Sun, 27 Apr 2003 06:36:05 -0300 Received: from trex.centroin.com.br (trex.centroin.com.br [200.225.63.134]) by sucuri.mat.puc-rio.br (8.9.3/8.9.3) with ESMTP id GAA12750 for ; Sun, 27 Apr 2003 06:36:02 -0300 Received: from trex.centroin.com.br (localhost [127.0.0.1]) by trex.centroin.com.br (8.12.9/8.12.9) with ESMTP id h3R9ZTkZ012033 for ; Sun, 27 Apr 2003 06:35:29 -0300 (EST) Received: by trex.centroin.com.br (8.12.9/8.12.5/Submit) id h3R9ZTxs012032; Sun, 27 Apr 2003 06:35:29 -0300 (EST) Message-Id: <200304270935.h3R9ZTxs012032@trex.centroin.com.br> Received: from 200.141.119.132 by trex.centroin.com.br (CIPWM versao 1.4C1) with HTTPS for ; Sun, 27 Apr 2003 06:35:29 -0300 (EST) Date: Sun, 27 Apr 2003 06:35:29 -0300 (EST) From: Augusto Cesar de Oliveira Morgado To: obm-l@mat.puc-rio.br Subject: Re: [obm-l] Limites MIME-Version: 1.0 X-Mailer: CentroIn Internet Provider WebMail v. 1.4C1 (http://www.centroin.com.br/) Content-Type: text/plain; charset="iso-8859-1" Content-Transfer-Encoding: 8bit X-MIME-Autoconverted: from quoted-printable to 8bit by sucuri.mat.puc-rio.br id GAA12751 Sender: owner-obm-l@sucuri.mat.puc-rio.br Precedence: bulk Reply-To: obm-l@mat.puc-rio.br 1) Quanto ao limite, o Claudio estah certo: o Leandro se distraiu (nao deve ter maximizado a tela) e leu (1-sin^2) onde estava (1-sin)^2. Faz parte... Eu ja fiz coisas bem piores. Alguns autores (o Thomas, por exemplo) fazem uma coisa horrorosa que eh dizer que o limite eh infinito quando o limite eh mais infinito ou menos infinito. Isso pode explicar, embora, a meu ver nao justifique, a resposta do seu livro. 2) Onduletas eh dose! Mas ondaletas eh comum e aceitavel. 3) Eu nao sou tao nacionalista assim. Defendo SIMBOLOS universais. Sou a favor de sin. Mas cbrt nao eh universal e ^(1/3) eh. Mas os nomes, ah! os nomes. Nos ja estamos importando nomes via EUA. Por exemplo, o Bin Laden: o nome do cara se escreve no original com aqueles caracters arabes; pronuncia-se Ussama; nossos irmaos do norte escrevem Osama e pronunciam Ussama. Os papagaios aqui escrevem Osama e pronunciam Ozama. Chebishev se escreve no original em cirílico. Os americanos escrevem Chebyshev. Por que diabos eu vou botar esse y no nome dele? Ou escrevo em cirílico ou em portugues. Eles ainda estao no Iraque e vao para a Siria. Nao nos afobemos. Morgado Em Sat, 26 Apr 2003 22:08:21 -0700, niski disse: > Pretendo nesta mensagem responder ao Leandro Recova, ao prof. Morgado, e > ao Claudio Buffara. > > prof. Morgado, desculpe.Eu estava acostumado com o acrônimo cbrt (raiz > cubica) e não fui cuidadoso o suficiente para fazer tal referencia. > > Claudio e Leandro : > Muito estranho... > Para o Leandro o limite deu 0, para o Buffara não existe, no Mathematica > deu -infinito e a resposta do livro é infinito !!!! e agora? > > Quanto a utilizar sin no lugar de sen , não acho que seja um grande > problema, alias sin remete ao termo original em latim "sinus". > Na minha opinião nao acho uma boa a lingua portuguesa mudar alguns > termos originais...matriz por exemplo..pq este Z? pq nao deixar do > originial cunhado por Sylvester!?!? Já vi coisas tb do tipo "Onduletas" > para designar wavelets...eu acho que seria a mesma coisa que se referir > ao Newton como "Isaque Newton". > > -- > [about him:] > It is rare to find learned men who are clean, do not stink and have a > sense of humour. > -Gottfried Whilhem Leibniz > > ========================================================================= > Instruções para entrar na lista, sair da lista e usar a lista em > http://www.mat.puc-rio.br/~nicolau/olimp/obm-l.html > ========================================================================= > > ========================================================================= Instruções para entrar na lista, sair da lista e usar a lista em http://www.mat.puc-rio.br/~nicolau/olimp/obm-l.html ========================================================================= From owner-obm-l@sucuri.mat.puc-rio.br Sun Apr 27 06:38:54 2003 Return-Path: Received: (from majordom@localhost) by sucuri.mat.puc-rio.br (8.9.3/8.9.3) id GAA12767 for obm-l-MTTP; Sun, 27 Apr 2003 06:36:19 -0300 Received: from trex.centroin.com.br (trex.centroin.com.br [200.225.63.134]) by sucuri.mat.puc-rio.br (8.9.3/8.9.3) with ESMTP id GAA12761 for ; Sun, 27 Apr 2003 06:36:15 -0300 Received: from trex.centroin.com.br (localhost [127.0.0.1]) by trex.centroin.com.br (8.12.9/8.12.9) with ESMTP id h3R9ZjkZ012074 for ; Sun, 27 Apr 2003 06:35:45 -0300 (EST) Received: by trex.centroin.com.br (8.12.9/8.12.5/Submit) id h3R9Zj5T012073; Sun, 27 Apr 2003 06:35:45 -0300 (EST) Message-Id: <200304270935.h3R9Zj5T012073@trex.centroin.com.br> Received: from 200.141.119.132 by trex.centroin.com.br (CIPWM versao 1.4C1) with HTTPS for ; Sun, 27 Apr 2003 06:35:45 -0300 (EST) Date: Sun, 27 Apr 2003 06:35:45 -0300 (EST) From: Augusto Cesar de Oliveira Morgado To: obm-l@mat.puc-rio.br Subject: Re: [obm-l] Limites MIME-Version: 1.0 X-Mailer: CentroIn Internet Provider WebMail v. 1.4C1 (http://www.centroin.com.br/) Content-Type: text/plain; charset="iso-8859-1" Content-Transfer-Encoding: 8bit X-MIME-Autoconverted: from quoted-printable to 8bit by sucuri.mat.puc-rio.br id GAA12764 Sender: owner-obm-l@sucuri.mat.puc-rio.br Precedence: bulk Reply-To: obm-l@mat.puc-rio.br 1) Quanto ao limite, o Claudio estah certo: o Leandro se distraiu (nao deve ter maximizado a tela) e leu (1-sin^2) onde estava (1-sin)^2. Faz parte... Eu ja fiz coisas bem piores. Alguns autores (o Thomas, por exemplo) fazem uma coisa horrorosa que eh dizer que o limite eh infinito quando o limite eh mais infinito ou menos infinito. Isso pode explicar, embora, a meu ver nao justifique, a resposta do seu livro. 2) Onduletas eh dose! Mas ondaletas eh comum e aceitavel. 3) Eu nao sou tao nacionalista assim. Defendo SIMBOLOS universais. Sou a favor de sin. Mas cbrt nao eh universal e ^(1/3) eh. Mas os nomes, ah! os nomes. Nos ja estamos importando nomes via EUA. Por exemplo, o Bin Laden: o nome do cara se escreve no original com aqueles caracters arabes; pronuncia-se Ussama; nossos irmaos do norte escrevem Osama e pronunciam Ussama. Os papagaios aqui escrevem Osama e pronunciam Ozama. Chebishev se escreve no original em cirílico. Os americanos escrevem Chebyshev. Por que diabos eu vou botar esse y no nome dele? Ou escrevo em cirílico ou em portugues. Eles ainda estao no Iraque e vao para a Siria. Nao nos afobemos. Morgado Em Sat, 26 Apr 2003 22:08:21 -0700, niski disse: > Pretendo nesta mensagem responder ao Leandro Recova, ao prof. Morgado, e > ao Claudio Buffara. > > prof. Morgado, desculpe.Eu estava acostumado com o acrônimo cbrt (raiz > cubica) e não fui cuidadoso o suficiente para fazer tal referencia. > > Claudio e Leandro : > Muito estranho... > Para o Leandro o limite deu 0, para o Buffara não existe, no Mathematica > deu -infinito e a resposta do livro é infinito !!!! e agora? > > Quanto a utilizar sin no lugar de sen , não acho que seja um grande > problema, alias sin remete ao termo original em latim "sinus". > Na minha opinião nao acho uma boa a lingua portuguesa mudar alguns > termos originais...matriz por exemplo..pq este Z? pq nao deixar do > originial cunhado por Sylvester!?!? Já vi coisas tb do tipo "Onduletas" > para designar wavelets...eu acho que seria a mesma coisa que se referir > ao Newton como "Isaque Newton". > > -- > [about him:] > It is rare to find learned men who are clean, do not stink and have a > sense of humour. > -Gottfried Whilhem Leibniz > > ========================================================================= > Instruções para entrar na lista, sair da lista e usar a lista em > http://www.mat.puc-rio.br/~nicolau/olimp/obm-l.html > ========================================================================= > > ========================================================================= Instruções para entrar na lista, sair da lista e usar a lista em http://www.mat.puc-rio.br/~nicolau/olimp/obm-l.html ========================================================================= From owner-obm-l@sucuri.mat.puc-rio.br Sun Apr 27 06:39:03 2003 Return-Path: Received: (from majordom@localhost) by sucuri.mat.puc-rio.br (8.9.3/8.9.3) id GAA12775 for obm-l-MTTP; Sun, 27 Apr 2003 06:36:32 -0300 Received: from trex.centroin.com.br (trex.centroin.com.br [200.225.63.134]) by sucuri.mat.puc-rio.br (8.9.3/8.9.3) with ESMTP id GAA12771 for ; Sun, 27 Apr 2003 06:36:28 -0300 Received: from trex.centroin.com.br (localhost [127.0.0.1]) by trex.centroin.com.br (8.12.9/8.12.9) with ESMTP id h3R9ZwkZ012102 for ; Sun, 27 Apr 2003 06:35:58 -0300 (EST) Received: by trex.centroin.com.br (8.12.9/8.12.5/Submit) id h3R9ZwnW012101; Sun, 27 Apr 2003 06:35:58 -0300 (EST) Message-Id: <200304270935.h3R9ZwnW012101@trex.centroin.com.br> Received: from 200.141.119.132 by trex.centroin.com.br (CIPWM versao 1.4C1) with HTTPS for ; Sun, 27 Apr 2003 06:35:58 -0300 (EST) Date: Sun, 27 Apr 2003 06:35:58 -0300 (EST) From: Augusto Cesar de Oliveira Morgado To: obm-l@mat.puc-rio.br Subject: Re: [obm-l] Limites MIME-Version: 1.0 X-Mailer: CentroIn Internet Provider WebMail v. 1.4C1 (http://www.centroin.com.br/) Content-Type: text/plain; charset="iso-8859-1" Content-Transfer-Encoding: 8bit X-MIME-Autoconverted: from quoted-printable to 8bit by sucuri.mat.puc-rio.br id GAA12772 Sender: owner-obm-l@sucuri.mat.puc-rio.br Precedence: bulk Reply-To: obm-l@mat.puc-rio.br 1) Quanto ao limite, o Claudio estah certo: o Leandro se distraiu (nao deve ter maximizado a tela) e leu (1-sin^2) onde estava (1-sin)^2. Faz parte... Eu ja fiz coisas bem piores. Alguns autores (o Thomas, por exemplo) fazem uma coisa horrorosa que eh dizer que o limite eh infinito quando o limite eh mais infinito ou menos infinito. Isso pode explicar, embora, a meu ver nao justifique, a resposta do seu livro. 2) Onduletas eh dose! Mas ondaletas eh comum e aceitavel. 3) Eu nao sou tao nacionalista assim. Defendo SIMBOLOS universais. Sou a favor de sin. Mas cbrt nao eh universal e ^(1/3) eh. Mas os nomes, ah! os nomes. Nos ja estamos importando nomes via EUA. Por exemplo, o Bin Laden: o nome do cara se escreve no original com aqueles caracters arabes; pronuncia-se Ussama; nossos irmaos do norte escrevem Osama e pronunciam Ussama. Os papagaios aqui escrevem Osama e pronunciam Ozama. Chebishev se escreve no original em cirílico. Os americanos escrevem Chebyshev. Por que diabos eu vou botar esse y no nome dele? Ou escrevo em cirílico ou em portugues. Eles ainda estao no Iraque e vao para a Siria. Nao nos afobemos. Morgado Em Sat, 26 Apr 2003 22:08:21 -0700, niski disse: > Pretendo nesta mensagem responder ao Leandro Recova, ao prof. Morgado, e > ao Claudio Buffara. > > prof. Morgado, desculpe.Eu estava acostumado com o acrônimo cbrt (raiz > cubica) e não fui cuidadoso o suficiente para fazer tal referencia. > > Claudio e Leandro : > Muito estranho... > Para o Leandro o limite deu 0, para o Buffara não existe, no Mathematica > deu -infinito e a resposta do livro é infinito !!!! e agora? > > Quanto a utilizar sin no lugar de sen , não acho que seja um grande > problema, alias sin remete ao termo original em latim "sinus". > Na minha opinião nao acho uma boa a lingua portuguesa mudar alguns > termos originais...matriz por exemplo..pq este Z? pq nao deixar do > originial cunhado por Sylvester!?!? Já vi coisas tb do tipo "Onduletas" > para designar wavelets...eu acho que seria a mesma coisa que se referir > ao Newton como "Isaque Newton". > > -- > [about him:] > It is rare to find learned men who are clean, do not stink and have a > sense of humour. > -Gottfried Whilhem Leibniz > > ========================================================================= > Instruções para entrar na lista, sair da lista e usar a lista em > http://www.mat.puc-rio.br/~nicolau/olimp/obm-l.html > ========================================================================= > > ========================================================================= Instruções para entrar na lista, sair da lista e usar a lista em http://www.mat.puc-rio.br/~nicolau/olimp/obm-l.html ========================================================================= From owner-obm-l@sucuri.mat.puc-rio.br Sun Apr 27 06:44:39 2003 Return-Path: Received: (from majordom@localhost) by sucuri.mat.puc-rio.br (8.9.3/8.9.3) id GAA12896 for obm-l-MTTP; Sun, 27 Apr 2003 06:42:08 -0300 Received: from trex.centroin.com.br (trex.centroin.com.br [200.225.63.134]) by sucuri.mat.puc-rio.br (8.9.3/8.9.3) with ESMTP id GAA12891 for ; Sun, 27 Apr 2003 06:42:04 -0300 Received: from trex.centroin.com.br (localhost [127.0.0.1]) by trex.centroin.com.br (8.12.9/8.12.9) with ESMTP id h3R9fYkZ012681 for ; Sun, 27 Apr 2003 06:41:34 -0300 (EST) Received: by trex.centroin.com.br (8.12.9/8.12.5/Submit) id h3R9fY6H012680; Sun, 27 Apr 2003 06:41:34 -0300 (EST) Message-Id: <200304270941.h3R9fY6H012680@trex.centroin.com.br> Received: from 200.141.119.132 by trex.centroin.com.br (CIPWM versao 1.4C1) with HTTPS for ; Sun, 27 Apr 2003 06:41:34 -0300 (EST) Date: Sun, 27 Apr 2003 06:41:34 -0300 (EST) From: Augusto Cesar de Oliveira Morgado To: obm-l@mat.puc-rio.br Subject: Re: [obm-l] Equacoes diferenciais MIME-Version: 1.0 X-Mailer: CentroIn Internet Provider WebMail v. 1.4C1 (http://www.centroin.com.br/) Content-Type: text/plain; charset="iso-8859-1" Content-Transfer-Encoding: 8bit X-MIME-Autoconverted: from quoted-printable to 8bit by sucuri.mat.puc-rio.br id GAA12892 Sender: owner-obm-l@sucuri.mat.puc-rio.br Precedence: bulk Reply-To: obm-l@mat.puc-rio.br Embora nao haja conferido suas contas, observo que a sua resposta descreve a mesma familia de funçoes que o seu livro descreve. Apenas as indexaçoes sao diferentes. A funçao que voce obtem fazendo c=k, seu livro obtem fazendo c=2k. Em Sun, 27 Apr 2003 03:53:39 -0300, Ricardo de Moraes (PS) disse: > Boa noite. > > Resolvendo a equação q segue, cheguei a um resultado um pouco diferente do q > o livro traz. E ñ consegui encontrar o meu erro... > > O problema diz,resolva a equação: > > t^2y' + 2ty - y^3 = 0, t>0 > > Somando y^3 dos dois lados, dividindo por t^2 e por y^3 chego em > y'y^(-3) + (2/t)y^(-2) = t^(-2) (a) > > Fazendo v = y^(-2) e v' = -2y^(-3)y' (daqui vem q v'/-2 = y^(-3)y') e > substituindo em (a) > (v'/-2) + 2t^(-1)v = t^(-2) multiplicando por -2 > v' - 4t^(-1)v = -2t^(-2) q é linear de 1ª ordem(?) > > Resolvendo esta equação em v, depois retornando para y ( v = y^(-2) ) > > A resposta q o livro traz é: y = + - [5t/(2 + 5ct^5)]^(1/2) > > Eu achei: y = + - [5t/(2 - 10ct^5)]^(1/2) > > Obs.: é uma equação de Bernoulli do livro Equações Diferenciais e Problemas > de Valores de Contorno, de Boyce e Diprima. 7ª ed. pag.40 nº28. > > Espero q não tenha ficado muito confuso... > > Obrigado, > > Ricardo. ========================================================================= Instruções para entrar na lista, sair da lista e usar a lista em http://www.mat.puc-rio.br/~nicolau/olimp/obm-l.html ========================================================================= From owner-obm-l@sucuri.mat.puc-rio.br Sun Apr 27 07:58:47 2003 Return-Path: Received: (from majordom@localhost) by sucuri.mat.puc-rio.br (8.9.3/8.9.3) id HAA15474 for obm-l-MTTP; Sun, 27 Apr 2003 07:55:55 -0300 Received: from trex-b.centroin.com.br (trex-b.centroin.com.br [200.225.63.136]) by sucuri.mat.puc-rio.br (8.9.3/8.9.3) with ESMTP id HAA15469 for ; Sun, 27 Apr 2003 07:55:52 -0300 Received: from centroin.com.br (RJ119132.user.veloxzone.com.br [200.141.119.132] (may be forged)) (authenticated bits=0) by trex-b.centroin.com.br (8.12.9/8.12.9) with ESMTP id h3RAtCVi013300 for ; Sun, 27 Apr 2003 07:55:12 -0300 (EST) Message-ID: <3EABB750.6020006@centroin.com.br> Date: Sun, 27 Apr 2003 07:56:16 -0300 From: "A. C. Morgado" User-Agent: Mozilla/5.0 (Windows; U; Windows NT 5.0; en-US; rv:1.0.2) Gecko/20030208 Netscape/7.02 X-Accept-Language: en-us, en MIME-Version: 1.0 To: obm-l@mat.puc-rio.br Subject: Re: [obm-l] Pontos pintados References: <20030427083259.27051.qmail@web21302.mail.yahoo.com> Content-Type: multipart/alternative; boundary="------------030609020608080508080206" Sender: owner-obm-l@sucuri.mat.puc-rio.br Precedence: bulk Reply-To: obm-l@mat.puc-rio.br --------------030609020608080508080206 Content-Type: text/plain; charset=ISO-8859-1; format=flowed Content-Transfer-Encoding: 8bit Epa, houve uma distraçao no final. Estah x^2 + x - n = 0 --> x = raiz(1+4n)/2. e deveria estar x^2 + x - n = 0 --> x = [raiz(1+4n)-1]/2. Marcos Reynaldo wrote: >>Uma equacao diofantina bonitinha: >> >>Prove que x^2 + (x+1)^2 = y^3 nao tem solucao em >>inteiros positivos. >> >> > >Com relação ao problema acima ai vai uma tentativa: > >Admita que a equação tem solução em inteiros >positivos. Neste caso, você conclui que y só pode ser >um número impar (pois se x é par, x^2 é par, x+1 é >impar e (x+1)^2 é impar e a soma de um numero par com >um impar é impar; analogamente, se x for impar, >conclui-se que y é impar). >Bom então y é da forma 2n+1, onde n é inteiro (o caso >em que n é 1 é facilmente verificado substituindo y >por 1 donde resulta que x ou é 0 ou -1). >Substituindo na expressão , vem >x^2 + x^2+2x+1=2n+1 --> 2x^2 + 2x - 2n = 0 --> > >x^2 + x - n = 0 --> x = raiz(1+4n)/2. >Ora, mas o resultado acima não é inteiro. Se fosse, >x^2=(1+4n)/2 tambem seria, o que é um absurdo. >Logo, o fato de considerarmos que a equação possui >solução em inteiros positivos gerou uma contradição. >Assim, a equação não possui solução em inteiros >positivos. > >Dá uma olhada, se tiver algum erro me diz. > >[]'s Marcos > >_______________________________________________________________________ >Yahoo! Mail >O melhor e-mail gratuito da internet: 6MB de espaço, antivírus, acesso POP3, filtro contra spam. >http://br.mail.yahoo.com/ >========================================================================= >Instruções para entrar na lista, sair da lista e usar a lista em >http://www.mat.puc-rio.br/~nicolau/olimp/obm-l.html >========================================================================= > > > > --------------030609020608080508080206 Content-Type: text/html; charset=us-ascii Content-Transfer-Encoding: 7bit Epa, houve uma distraçao no final.
Estah
x^2 + x - n = 0  -->  x = raiz(1+4n)/2.
e deveria estar
x^2 + x - n = 0  -->  x = [raiz(1+4n)-1]/2.


Marcos Reynaldo wrote:
Uma equacao diofantina bonitinha:

Prove que x^2 + (x+1)^2 = y^3 nao tem solucao em
inteiros positivos.
    

Com relação ao problema acima ai vai uma tentativa:

Admita que a equação tem solução em inteiros
positivos. Neste caso, você conclui que y só pode ser
um número impar (pois se x é par, x^2 é par, x+1 é
impar e (x+1)^2 é impar e a soma de um numero par com
um impar é impar; analogamente, se x for impar,
conclui-se que y é impar).
Bom então y é da forma 2n+1, onde n é inteiro (o caso
em que n é 1 é facilmente verificado substituindo y
por 1 donde resulta que x ou é 0 ou -1).
Substituindo na expressão , vem
x^2 + x^2+2x+1=2n+1 -->  2x^2 + 2x - 2n = 0 --> 

x^2 + x - n = 0  -->  x = raiz(1+4n)/2.
Ora, mas o resultado acima não é inteiro. Se fosse,
x^2=(1+4n)/2 tambem seria, o que é um absurdo.
Logo, o fato de considerarmos que a equação possui
solução em inteiros positivos gerou uma contradição.
Assim, a equação não possui solução em inteiros
positivos.

Dá uma olhada, se tiver algum erro me diz.

[]'s Marcos

_______________________________________________________________________
Yahoo! Mail
O melhor e-mail gratuito da internet: 6MB de espaço, antivírus, acesso POP3, filtro contra spam. 
http://br.mail.yahoo.com/
=========================================================================
Instruções para entrar na lista, sair da lista e usar a lista em
http://www.mat.puc-rio.br/~nicolau/olimp/obm-l.html
=========================================================================


  

--------------030609020608080508080206-- ========================================================================= Instruções para entrar na lista, sair da lista e usar a lista em http://www.mat.puc-rio.br/~nicolau/olimp/obm-l.html ========================================================================= From owner-obm-l@sucuri.mat.puc-rio.br Sun Apr 27 08:45:06 2003 Return-Path: Received: (from majordom@localhost) by sucuri.mat.puc-rio.br (8.9.3/8.9.3) id IAA16170 for obm-l-MTTP; Sun, 27 Apr 2003 08:42:28 -0300 Received: from toole.uol.com.br (toole.uol.com.br [200.221.29.26]) by sucuri.mat.puc-rio.br (8.9.3/8.9.3) with ESMTP id IAA16166 for ; Sun, 27 Apr 2003 08:42:24 -0300 Received: from gauss ([200.158.96.144]) by toole.uol.com.br (8.9.1/8.9.1) with SMTP id IAA05132 for ; Sun, 27 Apr 2003 08:41:53 -0300 (BRT) Message-ID: <001701c30cb2$c33eca50$90609ec8@gauss> From: "Domingos Jr." To: References: <20030427040445.10703.qmail@web21304.mail.yahoo.com> Subject: Re: [obm-l] integral de 1/x Date: Sun, 27 Apr 2003 08:47:20 -0300 MIME-Version: 1.0 Content-Type: text/plain; charset="iso-8859-1" Content-Transfer-Encoding: 8bit X-Priority: 3 X-MSMail-Priority: Normal X-Mailer: Microsoft Outlook Express 6.00.2800.1158 X-MimeOLE: Produced By Microsoft MimeOLE V6.00.2800.1165 Sender: owner-obm-l@sucuri.mat.puc-rio.br Precedence: bulk Reply-To: obm-l@mat.puc-rio.br sim, ln|x| falar que a derivada de ln|x| é 1/x tb serve, mas aí você precisaria demonstrar esse resultado. acho q calcular a área do gráfico é trabalhoso e não vai dar uma precisão que lhe permita afirmar que a integral é realmente ln|x|. [ ]'s > O resultado da integral não seria ln |x| ? > > ln|x|=ln(-x) se x<0 e ln|x|=ln(x) se x>0. > Para o primeiro caso a derivada, de ln(-x), é > -1/(-x)=1/x. Para o segundo caso, temos também 1/x. > Assim em ambos os casos a derivada é 1/x. Então ln|x| > é uma primitiva de 1/x, e portanto a sua integral é > ln|x|+C. > > Agora no caso de integrais definidas, uma maneira > intuitiva de ver isso, é que a área entre a curva 1/x > o eixo x e retas 1 e x é ln(x) (x>0). Assim a integral > de 1/x é ln(x). > > Não sei se minha explicação convence mas alguém da > lista deve dar uma demonstração mais formal. > > []'s Marcos ========================================================================= Instruções para entrar na lista, sair da lista e usar a lista em http://www.mat.puc-rio.br/~nicolau/olimp/obm-l.html ========================================================================= From owner-obm-l@sucuri.mat.puc-rio.br Sun Apr 27 09:05:30 2003 Return-Path: Received: (from majordom@localhost) by sucuri.mat.puc-rio.br (8.9.3/8.9.3) id JAA16529 for obm-l-MTTP; Sun, 27 Apr 2003 09:01:39 -0300 Received: from artemis.opendf.com.br (artemis.opengate.com.br [200.181.71.14]) by sucuri.mat.puc-rio.br (8.9.3/8.9.3) with ESMTP id JAA16525 for ; Sun, 27 Apr 2003 09:01:34 -0300 Received: from localhost (localhost [127.0.0.1]) by artemis.opendf.com.br (Postfix) with ESMTP id 2D0972BEE4 for ; Sun, 27 Apr 2003 09:02:22 -0300 (BRT) Received: from artemis.opendf.com.br ([127.0.0.1]) by localhost (artemis.opengate.com.br [127.0.0.1:10024]) (amavisd-new) with ESMTP id 05241-07 for ; Sun, 27 Apr 2003 09:02:21 -0300 (BRT) Received: from computer (200-181-088-208.bsace7001.dsl.brasiltelecom.net.br [200.181.88.208]) by artemis.opendf.com.br (Postfix) with ESMTP id 322542BED3 for ; Sun, 27 Apr 2003 09:02:20 -0300 (BRT) From: "Artur Costa Steiner" To: Subject: RE: [obm-l] integral de 1/x Date: Sun, 27 Apr 2003 09:01:04 -0300 Organization: Steiner Consultoria LTDA Message-ID: MIME-Version: 1.0 Content-Type: multipart/mixed; boundary="----=_NextPart_000_0059_01C30C9B.88841E90" X-Priority: 3 (Normal) X-MSMail-Priority: Normal X-Mailer: Microsoft Outlook, Build 10.0.2627 In-Reply-To: <20030427040445.10703.qmail@web21304.mail.yahoo.com> X-MimeOLE: Produced By Microsoft MimeOLE V6.00.2800.1165 Importance: Normal X-MS-TNEF-Correlator: 000000005FC3F71D8B4BB941A8AA63D06F261C36045E2B00 X-Virus-Scanned: by amavisd-new Sender: owner-obm-l@sucuri.mat.puc-rio.br Precedence: bulk Reply-To: obm-l@mat.puc-rio.br This is a multi-part message in MIME format. ------=_NextPart_000_0059_01C30C9B.88841E90 Content-Type: text/plain; charset="iso-8859-1" Content-Transfer-Encoding: quoted-printable Na realidade, isto depende de como definamos a funcao logaritmica = natural. Usualmente, define-se a mesma como a inversa da funcao exponencial de = base e, ou seja, ln eh a funcao tal que ln(e^x) =3D x para x em R (estou me restringindo =E0 reta real). Como a funcao exponencial eh estritamente crescente em R, a funcao ln entao existe e eh estritamente crescente = para x>0 (que eh o conjunton imagem da exponencial). E como a exponencial eh diferenciavel em R e nunca se anula, segue-se que ln eh diferenciavel = para x>0 e que podemos aplicar aquele teorema da derivada da funcao inversa. Logo, para x>0 temos que (lnx)' =3D 1/(e)'(lnx) =3D 1/e^lnx =3D 1/x. = Agora, vemos que 1/x eh continua para x=3D/=3D 0, logo integravel em qualquer = intervalo fechado que nao contenha o zero. Como ln(1) =3D e^0 =3D 1, temos pelo T. Fundamental do Calculo Integral que lnx =3D Integral (1 a x) 1/t dt. Em = termos de integral indefinida, temos que entao que ln eh uma das integrais indefinidas de 1/x. (pois para todo C, ln + C , C constante, tambem o = eh). Se considerarmos valores negativos para 1/x, podemos de fato dizer que = ln|x| e uma primitiva de 1/x, conforme o colega mostrou.=20 A questao relativa a area entre a curva 1/x e o eixo do x, x>0, eh consequencia de que lnx =3D Integral (1 a x) 1/t dt.=20 Alguns autores definem ln diretamente pela integral, isto eh, lnx =3D Integral (1 a x) 1/t dt por definicao. Neste caso, a conclusao eh = imediata. Mas usualmente define-se o logaritmo neperiano como a inversa da funcao exponencial de base e.=20 Um abraco Artur >O resultado da integral n=E3o seria ln |x| ? > >ln|x|=3Dln(-x) se x<0 e ln|x|=3Dln(x) se x>0. >Para o primeiro caso a derivada, de ln(-x), =E9 >-1/(-x)=3D1/x. Para o segundo caso, temos tamb=E9m 1/x. >Assim em ambos os casos a derivada =E9 1/x. Ent=E3o ln|x| >=E9 uma primitiva de 1/x, e portanto a sua integral =E9 >ln|x|+C. > >Agora no caso de integrais definidas, uma maneira >intuitiva de ver isso, =E9 que a =E1rea entre a curva 1/x >o eixo x e retas 1 e x =E9 ln(x) (x>0). Assim a integral >de 1/x =E9 ln(x). > >N=E3o sei se minha explica=E7=E3o convence mas algu=E9m da >lista deve dar uma demonstra=E7=E3o mais formal. > >[]'s Marcos > >_______________________________________________________________________ >Yahoo! Mail >O melhor e-mail gratuito da internet: 6MB de espa=E7o, antiv=EDrus, = acesso >POP3, filtro contra spam. >http://br.mail.yahoo.com/ >=3D=3D=3D=3D=3D=3D=3D=3D=3D=3D=3D=3D=3D=3D=3D=3D=3D=3D=3D=3D=3D=3D=3D=3D= =3D=3D=3D=3D=3D=3D=3D=3D=3D=3D=3D=3D=3D=3D=3D=3D=3D=3D=3D=3D=3D=3D=3D=3D=3D= =3D=3D=3D=3D=3D=3D=3D=3D=3D=3D=3D=3D=3D=3D=3D=3D=3D=3D=3D=3D=3D=3D=3D=3D >Instru=E7=F5es para entrar na lista, sair da lista e usar a lista em >http://www.mat.puc-rio.br/~nicolau/olimp/obm-l.html >=3D=3D=3D=3D=3D=3D=3D=3D=3D=3D=3D=3D=3D=3D=3D=3D=3D=3D=3D=3D=3D=3D=3D=3D= =3D=3D=3D=3D=3D=3D=3D=3D=3D=3D=3D=3D=3D=3D=3D=3D=3D=3D=3D=3D=3D=3D=3D=3D=3D= =3D=3D=3D=3D=3D=3D=3D=3D=3D=3D=3D=3D=3D=3D=3D=3D=3D=3D=3D=3D=3D=3D=3D=3D ------=_NextPart_000_0059_01C30C9B.88841E90 Content-Type: application/ms-tnef; name="winmail.dat" Content-Transfer-Encoding: base64 Content-Disposition: attachment; filename="winmail.dat" eJ8+IgQMAQaQCAAEAAAAAAABAAEAAQeQBgAIAAAA5AQAAAAAAADoAAEIgAcAGAAAAElQTS5NaWNy b3NvZnQgTWFpbC5Ob3RlADEIAQ2ABAACAAAAAgACAAEGAAcAAQAAAAAAAAEGgAMADgAAANMHBAAb AAkAAQAAAAAAAwEBA5AGAOgLAAAuAAAACwACAAEAAAALACMAAAAAAAMAJgAAAAAACwApAAAAAAAD AC4AAAAAAAIBMQABAAAAGAAAAAAAAABfw/cdi0u5QaiqY9BvJhw2JF0rAAMANgAAAAAAHgBwAAEA AAAYAAAAW29ibS1sXSBpbnRlZ3JhbCBkZSAxL3gAAgFxAAEAAAAWAAAAAcMMtIMS/uK9aHfqS92b AjxhbmC1UwAAAgEdDAEAAAAbAAAAU01UUDpBUlRVUl9TVEVJTkVSQFVTQS5ORVQAAAsAAQ4AAAAA QAAGDgBmDqu0DMMBAgEKDgEAAAAYAAAAAAAAAF/D9x2LS7lBqKpj0G8mHDbCgAAAAwAUDgAAAAAL AB8OAQAAAB4AKA4BAAAAKgAAADAwMDAwMDA0AWFydHVyX3N0ZWluZXJAdXNhLm5ldAFOZXRhZGRy ZXNzAAAAHgApDgEAAAAqAAAAMDAwMDAwMDQBYXJ0dXJfc3RlaW5lckB1c2EubmV0AU5ldGFkZHJl c3MAAAACAQkQAQAAAMIGAAC+BgAALgwAAExaRnUd0CTsAwAKAHJjcGcxMjXiMgNDdGV4BUEBAwH3 TwqAAqQD4wIAY2gKwHPwZXQwIAcTAoAP8wBQfwRWCFUHshHFDlEDARDHMvcGAAbDEcUzBEYQyRLb EdPbCO8J9zsYvw4wNRHCDGDOYwBQCwkBZDM2EVALphkHsGEgGMAHQGlkYYkBACwgBAB0byABAO5w CfABAB6xIAWgBGAesTpmC4BhBGAEIB2gZnVYbmNhHqAYUGcKwGncdG0N4B2gIAB0CHAHQOAuIFVz dQdAB4ACMPMeQR/DZS0RICBRB4EAwNcfZB2gC4B2BJBzHaAeENsgdg7AcAIgCfBjBzEfMlxiYSNx HkEIYCARIGrKYR5QbAOgZWggWAGQDQMgcQpQJ6EoZV54wCkgPSB4IAqxHaDpKcBlbQfwIClAHoEn ME8HgB2xHoAFEG5nC4BkeR6gXCcKYB2xAZAdsyn/IiAIUCRDJU8DICfhK1MBkPcikx9gK0FjL3Mq Uh5QIGj/J8ECMCWDHnEm0S6/L8gp5Ho+EVAoKOIn4R6gBaBunmogkB6QA6AHcGFnKlG3JREluSzx RSQWLg1kBpC/BJAmAySwLqEqYh8gbiCSvydBIFA6MAtgHlARIGcKUP8jYijkOK80CDukJdABACAj 1wtQIXEFwGEo4WwfIA6wzwWwKlAk8wSBaXYeICT66ySVIiBMIQBvHlA0Fw6wWyAiKOIoJ7ApcCcp kTH+LylAQ+BDo0QDKVBDsUQD+ngiIEFCMCHwHlAksEMmd0WxJ9I1QXQLgCJgKdU9+i8poDAnkUIh JIEOsAnAvzl2KOAHQCjhBcBJMnJAkP8YUCBwBZAQ8CvhKOIgADUjrQ6wbhDwNRF6BJBvLQX9KSEx KYIpUBFQRBEeUEME6x7gSxFUIiBGIJAeECKSayZCHqBDB0BjOuAeoEn/SUQoxkVyUQdNwCBRKXFE MPkFQGR0N0EqYErBICIfEf9JNQMgK8Ef0h4BTpY0szGT/TvIdT/zBCBJNQQAVRlUQ/1Fsygl0Fhx KeMekFBSJ5P6KxLAIB5QW2A1QR6AAHDbIsIy8WIqUSWRaCzwCqL/CoAGYFuzHgAEkArAICJK8v8r QSGgO0AhwECATuIp8kWxt0JRPkUfEWYhwB6haUzxsSjVfHh8MkFXgnAFEP8hYF9xQDJgFDVBAhBU EB8g/zUiP3AhECOwIDArcAhgIiD9XURBKNJb8R6gGMALYGMj/2cRHcExchjAIFFQsErhRwT5XNJp eB6hHqBgQTRhHlD7J+Fbs2Uo4SYSHzJRn1KvzWWWbDtQBjFhdR6QXvL/IwQqYCexOOAsYi9kTxEk cv9RJB5VJ+AeUGt/Uts+IQXAv1VEILEiIAfAMhIgsHNCQf8kAiCgCkAk4FziNdEJgAcw+wGQIiBN V9F1wCJmIvkg2O8eoCNAHuAHIW41IiRPLa/7JmhlhlUqYAGgIfAFoGWlzwAgCHBdRF1EPk8rMjrg vwGQK+F6wVSXC5AsIDMeoO8RIAchJ6JiMj9+1X7VYhOaPSkhLSlxI3F4PD2yz4L3g5U0cF01PlAp 8h6g/2LSaQADYHTjIFFAViLiKRLXg4EeUCwROX7VLUQxg4H+PUWzhfU7MivSdPRDBFxy74iyKmBF sn7VQQQQB3AqQv9cgSAxIDF08iBCQFeIskWk/kUCMIDUYhOCFo9yYp9gI/8+EgAgXBF6AiJRVIiI uWIT/CtDhXaMxkYSIaCG1VRov1hxWMceUFeCA4GGoWF+1f1JMXVjF3phHmF1Eo9jKOL9j0MxZ29F on7VaOVooixi/wQgUtCD0Y9UhLSE0DRwLPG/jTRwaH7VWWSfuZXNToDWfmk6giFgTJJ7YT7SLBE3 /4DUNUF6YCCgpRFX0QdAO1D/jCQeEIKGHnFAMnpgerEFwP9Xgz5hW+GZoaYXAMBYcWQiYyIBghxb XScEIHbgcvcFoBCwgitfrj+vT7BfsW8DsZB+1VlhaG9vIe920RQxfuYHgGyzMAXAI1B/qwEDIElx mlJ/5wSgETA68CA2TUIfMgeQCrCmAst1IkehdiwRZHJ1wDChxzAAm1GFhk9QMx5QH+Bff6CGwgIh KgG3EW2FdmihAkBwOi8vfXAutOJULnmzIi4fcS9+1T3/vW++f7+PwJ/A4n7VUQArYfOnoqYxZjUH kSnjnNIKwf8gACDgqLI7EQtwc8HERWJi13qQPxHFRm26/HfHoLvxdVOQcBtwLQUQTSB9cC9ufnQh BvBuQC8G8AdwcLHJUGJtLSIQu2BtodZfwO/L38zvzf/BqX3P0AAAHgBCEAEAAAA1AAAAPDIwMDMw NDI3MDQwNDQ1LjEwNzAzLnFtYWlsQHdlYjIxMzA0Lm1haWwueWFob28uY29tPgAAAAADAJIQAQAA AAIBFDoBAAAAEAAAAMXAD1EoxaJJqzGDxMYadj4DAN4/r28AAAMACVkBAAAAAwBAZQAAAAALABOA CCAGAAAAAADAAAAAAAAARgAAAAADhQAAAAAAAAMAFYAIIAYAAAAAAMAAAAAAAABGAAAAABCFAAAA AAAAAwAbgAggBgAAAAAAwAAAAAAAAEYAAAAAUoUAAOOQAQADACKACCAGAAAAAADAAAAAAAAARgAA AAABhQAAAAAAAEAAI4AIIAYAAAAAAMAAAAAAAABGAAAAAGCFAAAA0IjDEAAAAB4AQYAIIAYAAAAA AMAAAAAAAABGAAAAAFSFAAABAAAABQAAADEwLjAAAAAACwBCgAggBgAAAAAAwAAAAAAAAEYAAAAA BoUAAAAAAAALAEaACCAGAAAAAADAAAAAAAAARgAAAAAOhQAAAAAAAAMASYAIIAYAAAAAAMAAAAAA AABGAAAAABiFAAAAAAAACwBegAggBgAAAAAAwAAAAAAAAEYAAAAAgoUAAAEAAAACAfgPAQAAABAA AABfw/cdi0u5QaiqY9BvJhw2AgH6DwEAAAAQAAAAX8P3HYtLuUGoqmPQbyYcNgIB+w8BAAAAhAAA AAAAAAA4obsQBeUQGqG7CAArKlbCAABtc3BzdC5kbGwAAAAAAE5JVEH5v7gBAKoAN9luAAAAQzpc RG9jdW1lbnRzIGFuZCBTZXR0aW5nc1xBcnR1clxBcHBsaWNhdGlvbiBEYXRhXE1pY3Jvc29mdFxP dXRsb29rXE91dGxvb2sucHN0AAMA/g8FAAAAAwANNP03AgACARQ0AQAAABAAAABOSVRB+b+4AQCq ADfZbgAAAgF/AAEAAAAxAAAAMDAwMDAwMDA1RkMzRjcxRDhCNEJCOTQxQThBQTYzRDA2RjI2MUMz NjA0NUUyQjAwAAAAAAMABhDyLU1XAwAHEOEHAAADABAQAAAAAAMAERAAAAAAHgAIEAEAAABlAAAA TkFSRUFMSURBREUsSVNUT0RFUEVOREVERUNPTU9ERUZJTkFNT1NBRlVOQ0FPTE9HQVJJVE1JQ0FO QVRVUkFMVVNVQUxNRU5URSxERUZJTkUtU0VBTUVTTUFDT01PQUlOVkVSUwAAAADVew== ------=_NextPart_000_0059_01C30C9B.88841E90-- ========================================================================= Instruções para entrar na lista, sair da lista e usar a lista em http://www.mat.puc-rio.br/~nicolau/olimp/obm-l.html ========================================================================= From owner-obm-l@sucuri.mat.puc-rio.br Sun Apr 27 09:09:23 2003 Return-Path: Received: (from majordom@localhost) by sucuri.mat.puc-rio.br (8.9.3/8.9.3) id JAA16654 for obm-l-MTTP; Sun, 27 Apr 2003 09:05:35 -0300 Received: from hotmail.com (f79.sea1.hotmail.com [207.68.163.79]) by sucuri.mat.puc-rio.br (8.9.3/8.9.3) with ESMTP id JAA16642 for ; Sun, 27 Apr 2003 09:05:29 -0300 Received: from mail pickup service by hotmail.com with Microsoft SMTPSVC; Sun, 27 Apr 2003 05:04:58 -0700 Received: from 198.81.9.1 by sea1fd.sea1.hotmail.msn.com with HTTP; Sun, 27 Apr 2003 12:04:58 GMT X-Originating-IP: [198.81.9.1] X-Originating-Email: [fredericor@hotmail.com] From: "Frederico Reis Marques de Brito" To: obm-l@mat.puc-rio.br Subject: [obm-l] REPUNITS ( Meio off-topic?) Date: Sun, 27 Apr 2003 09:04:58 -0300 Mime-Version: 1.0 Content-Type: text/plain; charset=iso-8859-1; format=flowed Message-ID: X-OriginalArrivalTime: 27 Apr 2003 12:04:58.0341 (UTC) FILETIME=[391E5950:01C30CB5] Sender: owner-obm-l@sucuri.mat.puc-rio.br Precedence: bulk Reply-To: obm-l@mat.puc-rio.br Bem, estrangeirismos a parte, gostaria de saber se alguém conhece o termo em português equivalente a "repunits" e ainda quando foi introduzido e por quem. Já procurei em diversos livros de Hist da Mat e de Teor dos Num. mas não encontrei. Tb já busquei na Internet e nada. Antecipo meus agradecimentos. Frederico. _________________________________________________________________ MSN Messenger: converse com os seus amigos online. http://messenger.msn.com.br ========================================================================= Instruções para entrar na lista, sair da lista e usar a lista em http://www.mat.puc-rio.br/~nicolau/olimp/obm-l.html ========================================================================= From owner-obm-l@sucuri.mat.puc-rio.br Sun Apr 27 09:33:48 2003 Return-Path: Received: (from majordom@localhost) by sucuri.mat.puc-rio.br (8.9.3/8.9.3) id JAA17544 for obm-l-MTTP; Sun, 27 Apr 2003 09:30:01 -0300 Received: from trex.centroin.com.br (trex.centroin.com.br [200.225.63.134]) by sucuri.mat.puc-rio.br (8.9.3/8.9.3) with ESMTP id JAA17525 for ; Sun, 27 Apr 2003 09:29:56 -0300 Received: from trex.centroin.com.br (localhost [127.0.0.1]) by trex.centroin.com.br (8.12.9/8.12.9) with ESMTP id h3RCTQkZ008375 for ; Sun, 27 Apr 2003 09:29:26 -0300 (EST) Received: by trex.centroin.com.br (8.12.9/8.12.5/Submit) id h3RCTQYC008374; Sun, 27 Apr 2003 09:29:26 -0300 (EST) Message-Id: <200304271229.h3RCTQYC008374@trex.centroin.com.br> Received: from 200.141.119.61 by trex.centroin.com.br (CIPWM versao 1.4C1) with HTTPS for ; Sun, 27 Apr 2003 09:29:26 -0300 (EST) Date: Sun, 27 Apr 2003 09:29:26 -0300 (EST) From: Augusto Cesar de Oliveira Morgado To: obm-l@mat.puc-rio.br Subject: Re: [obm-l] REPUNITS ( Meio off-topic?) MIME-Version: 1.0 X-Mailer: CentroIn Internet Provider WebMail v. 1.4C1 (http://www.centroin.com.br/) Content-Type: text/plain; charset="iso-8859-1" Content-Transfer-Encoding: 8bit X-MIME-Autoconverted: from quoted-printable to 8bit by sucuri.mat.puc-rio.br id JAA17532 Sender: owner-obm-l@sucuri.mat.puc-rio.br Precedence: bulk Reply-To: obm-l@mat.puc-rio.br Que tal unidade replicada? Em Sun, 27 Apr 2003 09:04:58 -0300, Frederico Reis Marques de Brito disse: > > Bem, estrangeirismos a parte, gostaria de saber se alguém conhece o termo > em português equivalente a "repunits" e ainda quando foi introduzido e por > quem. Já procurei em diversos livros de Hist da Mat e de Teor dos Num. mas > não encontrei. Tb já busquei na Internet e nada. > > Antecipo meus agradecimentos. > Frederico. > > > > > > > > _________________________________________________________________ > MSN Messenger: converse com os seus amigos online. > http://messenger.msn.com.br > > ========================================================================= > Instruções para entrar na lista, sair da lista e usar a lista em > http://www.mat.puc-rio.br/~nicolau/olimp/obm-l.html > ========================================================================= > > ========================================================================= Instruções para entrar na lista, sair da lista e usar a lista em http://www.mat.puc-rio.br/~nicolau/olimp/obm-l.html ========================================================================= From owner-obm-l@sucuri.mat.puc-rio.br Sun Apr 27 09:33:52 2003 Return-Path: Received: (from majordom@localhost) by sucuri.mat.puc-rio.br (8.9.3/8.9.3) id JAA17528 for obm-l-MTTP; Sun, 27 Apr 2003 09:29:57 -0300 Received: from trex.centroin.com.br (trex.centroin.com.br [200.225.63.134]) by sucuri.mat.puc-rio.br (8.9.3/8.9.3) with ESMTP id JAA17516 for ; Sun, 27 Apr 2003 09:29:51 -0300 Received: from trex.centroin.com.br (localhost [127.0.0.1]) by trex.centroin.com.br (8.12.9/8.12.9) with ESMTP id h3RCTLkZ008349 for ; Sun, 27 Apr 2003 09:29:21 -0300 (EST) Received: by trex.centroin.com.br (8.12.9/8.12.5/Submit) id h3RCTLsQ008348; Sun, 27 Apr 2003 09:29:21 -0300 (EST) Message-Id: <200304271229.h3RCTLsQ008348@trex.centroin.com.br> Received: from 200.141.119.61 by trex.centroin.com.br (CIPWM versao 1.4C1) with HTTPS for ; Sun, 27 Apr 2003 09:29:21 -0300 (EST) Date: Sun, 27 Apr 2003 09:29:21 -0300 (EST) From: Augusto Cesar de Oliveira Morgado To: obm-l@mat.puc-rio.br Subject: Re: [obm-l] REPUNITS ( Meio off-topic?) MIME-Version: 1.0 X-Mailer: CentroIn Internet Provider WebMail v. 1.4C1 (http://www.centroin.com.br/) Content-Type: text/plain; charset="iso-8859-1" Content-Transfer-Encoding: 8bit X-MIME-Autoconverted: from quoted-printable to 8bit by sucuri.mat.puc-rio.br id JAA17518 Sender: owner-obm-l@sucuri.mat.puc-rio.br Precedence: bulk Reply-To: obm-l@mat.puc-rio.br Que tal unidade replicada? Em Sun, 27 Apr 2003 09:04:58 -0300, Frederico Reis Marques de Brito disse: > > Bem, estrangeirismos a parte, gostaria de saber se alguém conhece o termo > em português equivalente a "repunits" e ainda quando foi introduzido e por > quem. Já procurei em diversos livros de Hist da Mat e de Teor dos Num. mas > não encontrei. Tb já busquei na Internet e nada. > > Antecipo meus agradecimentos. > Frederico. > > > > > > > > _________________________________________________________________ > MSN Messenger: converse com os seus amigos online. > http://messenger.msn.com.br > > ========================================================================= > Instruções para entrar na lista, sair da lista e usar a lista em > http://www.mat.puc-rio.br/~nicolau/olimp/obm-l.html > ========================================================================= > > ========================================================================= Instruções para entrar na lista, sair da lista e usar a lista em http://www.mat.puc-rio.br/~nicolau/olimp/obm-l.html ========================================================================= From owner-obm-l@sucuri.mat.puc-rio.br Sun Apr 27 09:33:55 2003 Return-Path: Received: (from majordom@localhost) by sucuri.mat.puc-rio.br (8.9.3/8.9.3) id JAA17552 for obm-l-MTTP; Sun, 27 Apr 2003 09:30:07 -0300 Received: from trex.centroin.com.br (trex.centroin.com.br [200.225.63.134]) by sucuri.mat.puc-rio.br (8.9.3/8.9.3) with ESMTP id JAA17548 for ; Sun, 27 Apr 2003 09:30:04 -0300 Received: from trex.centroin.com.br (localhost [127.0.0.1]) by trex.centroin.com.br (8.12.9/8.12.9) with ESMTP id h3RCTYkZ008398 for ; Sun, 27 Apr 2003 09:29:34 -0300 (EST) Received: by trex.centroin.com.br (8.12.9/8.12.5/Submit) id h3RCTYvm008397; Sun, 27 Apr 2003 09:29:34 -0300 (EST) Message-Id: <200304271229.h3RCTYvm008397@trex.centroin.com.br> Received: from 200.141.119.61 by trex.centroin.com.br (CIPWM versao 1.4C1) with HTTPS for ; Sun, 27 Apr 2003 09:29:34 -0300 (EST) Date: Sun, 27 Apr 2003 09:29:34 -0300 (EST) From: Augusto Cesar de Oliveira Morgado To: obm-l@mat.puc-rio.br Subject: Re: [obm-l] REPUNITS ( Meio off-topic?) MIME-Version: 1.0 X-Mailer: CentroIn Internet Provider WebMail v. 1.4C1 (http://www.centroin.com.br/) Content-Type: text/plain; charset="iso-8859-1" Content-Transfer-Encoding: 8bit X-MIME-Autoconverted: from quoted-printable to 8bit by sucuri.mat.puc-rio.br id JAA17549 Sender: owner-obm-l@sucuri.mat.puc-rio.br Precedence: bulk Reply-To: obm-l@mat.puc-rio.br Que tal unidade replicada? Em Sun, 27 Apr 2003 09:04:58 -0300, Frederico Reis Marques de Brito disse: > > Bem, estrangeirismos a parte, gostaria de saber se alguém conhece o termo > em português equivalente a "repunits" e ainda quando foi introduzido e por > quem. Já procurei em diversos livros de Hist da Mat e de Teor dos Num. mas > não encontrei. Tb já busquei na Internet e nada. > > Antecipo meus agradecimentos. > Frederico. > > > > > > > > _________________________________________________________________ > MSN Messenger: converse com os seus amigos online. > http://messenger.msn.com.br > > ========================================================================= > Instruções para entrar na lista, sair da lista e usar a lista em > http://www.mat.puc-rio.br/~nicolau/olimp/obm-l.html > ========================================================================= > > ========================================================================= Instruções para entrar na lista, sair da lista e usar a lista em http://www.mat.puc-rio.br/~nicolau/olimp/obm-l.html ========================================================================= From owner-obm-l@sucuri.mat.puc-rio.br Sun Apr 27 09:34:01 2003 Return-Path: Received: (from majordom@localhost) by sucuri.mat.puc-rio.br (8.9.3/8.9.3) id JAA17560 for obm-l-MTTP; Sun, 27 Apr 2003 09:30:15 -0300 Received: from trex.centroin.com.br (trex.centroin.com.br [200.225.63.134]) by sucuri.mat.puc-rio.br (8.9.3/8.9.3) with ESMTP id JAA17556 for ; Sun, 27 Apr 2003 09:30:11 -0300 Received: from trex.centroin.com.br (localhost [127.0.0.1]) by trex.centroin.com.br (8.12.9/8.12.9) with ESMTP id h3RCTfkZ008413 for ; Sun, 27 Apr 2003 09:29:41 -0300 (EST) Received: by trex.centroin.com.br (8.12.9/8.12.5/Submit) id h3RCTfbS008412; Sun, 27 Apr 2003 09:29:41 -0300 (EST) Message-Id: <200304271229.h3RCTfbS008412@trex.centroin.com.br> Received: from 200.141.119.61 by trex.centroin.com.br (CIPWM versao 1.4C1) with HTTPS for ; Sun, 27 Apr 2003 09:29:41 -0300 (EST) Date: Sun, 27 Apr 2003 09:29:41 -0300 (EST) From: Augusto Cesar de Oliveira Morgado To: obm-l@mat.puc-rio.br Subject: Re: [obm-l] REPUNITS ( Meio off-topic?) MIME-Version: 1.0 X-Mailer: CentroIn Internet Provider WebMail v. 1.4C1 (http://www.centroin.com.br/) Content-Type: text/plain; charset="iso-8859-1" Content-Transfer-Encoding: 8bit X-MIME-Autoconverted: from quoted-printable to 8bit by sucuri.mat.puc-rio.br id JAA17557 Sender: owner-obm-l@sucuri.mat.puc-rio.br Precedence: bulk Reply-To: obm-l@mat.puc-rio.br Que tal unidade replicada? Em Sun, 27 Apr 2003 09:04:58 -0300, Frederico Reis Marques de Brito disse: > > Bem, estrangeirismos a parte, gostaria de saber se alguém conhece o termo > em português equivalente a "repunits" e ainda quando foi introduzido e por > quem. Já procurei em diversos livros de Hist da Mat e de Teor dos Num. mas > não encontrei. Tb já busquei na Internet e nada. > > Antecipo meus agradecimentos. > Frederico. > > > > > > > > _________________________________________________________________ > MSN Messenger: converse com os seus amigos online. > http://messenger.msn.com.br > > ========================================================================= > Instruções para entrar na lista, sair da lista e usar a lista em > http://www.mat.puc-rio.br/~nicolau/olimp/obm-l.html > ========================================================================= > > ========================================================================= Instruções para entrar na lista, sair da lista e usar a lista em http://www.mat.puc-rio.br/~nicolau/olimp/obm-l.html ========================================================================= From owner-obm-l@sucuri.mat.puc-rio.br Sun Apr 27 09:34:10 2003 Return-Path: Received: (from majordom@localhost) by sucuri.mat.puc-rio.br (8.9.3/8.9.3) id JAA17568 for obm-l-MTTP; Sun, 27 Apr 2003 09:30:21 -0300 Received: from trex.centroin.com.br (trex.centroin.com.br [200.225.63.134]) by sucuri.mat.puc-rio.br (8.9.3/8.9.3) with ESMTP id JAA17564 for ; Sun, 27 Apr 2003 09:30:17 -0300 Received: from trex.centroin.com.br (localhost [127.0.0.1]) by trex.centroin.com.br (8.12.9/8.12.9) with ESMTP id h3RCTlkZ008432 for ; Sun, 27 Apr 2003 09:29:47 -0300 (EST) Received: by trex.centroin.com.br (8.12.9/8.12.5/Submit) id h3RCTlJO008431; Sun, 27 Apr 2003 09:29:47 -0300 (EST) Message-Id: <200304271229.h3RCTlJO008431@trex.centroin.com.br> Received: from 200.141.119.61 by trex.centroin.com.br (CIPWM versao 1.4C1) with HTTPS for ; Sun, 27 Apr 2003 09:29:47 -0300 (EST) Date: Sun, 27 Apr 2003 09:29:47 -0300 (EST) From: Augusto Cesar de Oliveira Morgado To: obm-l@mat.puc-rio.br Subject: Re: [obm-l] REPUNITS ( Meio off-topic?) MIME-Version: 1.0 X-Mailer: CentroIn Internet Provider WebMail v. 1.4C1 (http://www.centroin.com.br/) Content-Type: text/plain; charset="iso-8859-1" Content-Transfer-Encoding: 8bit X-MIME-Autoconverted: from quoted-printable to 8bit by sucuri.mat.puc-rio.br id JAA17565 Sender: owner-obm-l@sucuri.mat.puc-rio.br Precedence: bulk Reply-To: obm-l@mat.puc-rio.br Que tal unidade replicada? Em Sun, 27 Apr 2003 09:04:58 -0300, Frederico Reis Marques de Brito disse: > > Bem, estrangeirismos a parte, gostaria de saber se alguém conhece o termo > em português equivalente a "repunits" e ainda quando foi introduzido e por > quem. Já procurei em diversos livros de Hist da Mat e de Teor dos Num. mas > não encontrei. Tb já busquei na Internet e nada. > > Antecipo meus agradecimentos. > Frederico. > > > > > > > > _________________________________________________________________ > MSN Messenger: converse com os seus amigos online. > http://messenger.msn.com.br > > ========================================================================= > Instruções para entrar na lista, sair da lista e usar a lista em > http://www.mat.puc-rio.br/~nicolau/olimp/obm-l.html > ========================================================================= > > ========================================================================= Instruções para entrar na lista, sair da lista e usar a lista em http://www.mat.puc-rio.br/~nicolau/olimp/obm-l.html ========================================================================= From owner-obm-l@sucuri.mat.puc-rio.br Sun Apr 27 09:50:17 2003 Return-Path: Received: (from majordom@localhost) by sucuri.mat.puc-rio.br (8.9.3/8.9.3) id JAA18655 for obm-l-MTTP; Sun, 27 Apr 2003 09:45:53 -0300 Received: from web13006.mail.yahoo.com (web13006.mail.yahoo.com [216.136.174.16]) by sucuri.mat.puc-rio.br (8.9.3/8.9.3) with SMTP id JAA18642 for ; Sun, 27 Apr 2003 09:45:48 -0300 Message-ID: <20030427124516.7613.qmail@web13006.mail.yahoo.com> Received: from [200.148.194.17] by web13006.mail.yahoo.com via HTTP; Sun, 27 Apr 2003 09:45:16 ART Date: Sun, 27 Apr 2003 09:45:16 -0300 (ART) From: =?iso-8859-1?q?Helder=20Suzuki?= Subject: [obm-l] Cavalos de xadrez To: obm-l@mat.puc-rio.br MIME-Version: 1.0 Content-Type: text/plain; charset=iso-8859-1 Content-Transfer-Encoding: 8bit Sender: owner-obm-l@sucuri.mat.puc-rio.br Precedence: bulk Reply-To: obm-l@mat.puc-rio.br Olá! (1) Quantos cavalos de xadrez podemos colocar em um tabuleiro NxM tal que nenhum seja ameaçado por outro? []'s, Helder Toshiro Suzuki _______________________________________________________________________ Yahoo! Mail O melhor e-mail gratuito da internet: 6MB de espaço, antivírus, acesso POP3, filtro contra spam. http://br.mail.yahoo.com/ ========================================================================= Instruções para entrar na lista, sair da lista e usar a lista em http://www.mat.puc-rio.br/~nicolau/olimp/obm-l.html ========================================================================= From owner-obm-l@sucuri.mat.puc-rio.br Sun Apr 27 09:55:19 2003 Return-Path: Received: (from majordom@localhost) by sucuri.mat.puc-rio.br (8.9.3/8.9.3) id JAA19094 for obm-l-MTTP; Sun, 27 Apr 2003 09:51:33 -0300 Received: from itaqui.terra.com.br (itaqui.terra.com.br [200.176.3.19]) by sucuri.mat.puc-rio.br (8.9.3/8.9.3) with ESMTP id JAA19090 for ; Sun, 27 Apr 2003 09:51:28 -0300 Received: from araci.terra.com.br (araci.terra.com.br [200.176.3.44]) by itaqui.terra.com.br (Postfix) with ESMTP id 4E6CD3BC235 for ; Sun, 27 Apr 2003 09:50:58 -0300 (BRT) Received: from [200.177.176.23] (dl-nas1-sao-C8B1B017.p001.terra.com.br [200.177.176.23]) by araci.terra.com.br (Postfix) with ESMTP id 69E5221EF49 for ; Sun, 27 Apr 2003 09:50:57 -0300 (BRT) User-Agent: Microsoft-Outlook-Express-Macintosh-Edition/5.02.2022 Date: Sun, 27 Apr 2003 09:52:25 -0300 Subject: Re: [obm-l] Pontos pintados From: Claudio Buffara To: Message-ID: In-Reply-To: <20030427083259.27051.qmail@web21302.mail.yahoo.com> Mime-version: 1.0 Content-type: text/plain; charset="ISO-8859-1" Content-Transfer-Encoding: 8bit X-MIME-Autoconverted: from quoted-printable to 8bit by sucuri.mat.puc-rio.br id JAA19091 Sender: owner-obm-l@sucuri.mat.puc-rio.br Precedence: bulk Reply-To: obm-l@mat.puc-rio.br Oi, Marcos: Veja os comentarios abaixo: on 27.04.03 05:32, Marcos Reynaldo at marc_reybr@yahoo.com.br wrote: >> Uma equacao diofantina bonitinha: >> >> Prove que x^2 + (x+1)^2 = y^3 nao tem solucao em >> inteiros positivos. > > Com relação ao problema acima ai vai uma tentativa: > > Admita que a equação tem solução em inteiros > positivos. Neste caso, você conclui que y só pode ser > um número impar (pois se x é par, x^2 é par, x+1 é > impar e (x+1)^2 é impar e a soma de um numero par com > um impar é impar; analogamente, se x for impar, > conclui-se que y é impar). > Bom então y é da forma 2n+1, onde n é inteiro (o caso > em que n é 1 é facilmente verificado substituindo y > por 1 donde resulta que x ou é 0 ou -1). > Substituindo na expressão , vem > x^2 + x^2+2x+1=2n+1 --> 2x^2 + 2x - 2n = 0 --> > Ateh aqui - tudo certo. > x^2 + x - n = 0 --> x = raiz(1+4n)/2. > Ora, mas o resultado acima não é inteiro. Se fosse, > x^2=(1+4n)/2 tambem seria, o que é um absurdo. Na verdade, resolvendo aquela equacao do 2o. grau, voce acha que: x = (-1 + raiz(1+4n))/2, que eh inteiro e positivo para uma infinidade de valores de n: n = 2 ==> x = 1 n = 6 ==> x = 2 n = 12 ==> x = 3 em geral: n = k^2 + k ==> x = k > Logo, o fato de considerarmos que a equação possui > solução em inteiros positivos gerou uma contradição. > Assim, a equação não possui solução em inteiros > positivos. > Assim, nao vale a sua conclusao. > Dá uma olhada, se tiver algum erro me diz. > > []'s Marcos > Um abraco, Claudio. ========================================================================= Instruções para entrar na lista, sair da lista e usar a lista em http://www.mat.puc-rio.br/~nicolau/olimp/obm-l.html ========================================================================= From owner-obm-l@sucuri.mat.puc-rio.br Sun Apr 27 10:14:46 2003 Return-Path: Received: (from majordom@localhost) by sucuri.mat.puc-rio.br (8.9.3/8.9.3) id KAA20683 for obm-l-MTTP; Sun, 27 Apr 2003 10:10:53 -0300 Received: from paiol.terra.com.br (paiol.terra.com.br [200.176.3.18]) by sucuri.mat.puc-rio.br (8.9.3/8.9.3) with ESMTP id KAA20679 for ; Sun, 27 Apr 2003 10:10:48 -0300 Received: from altamira.terra.com.br (altamira.terra.com.br [200.176.3.40]) by paiol.terra.com.br (Postfix) with ESMTP id 8899687D5C for ; Sun, 27 Apr 2003 10:10:18 -0300 (BRT) Received: from [200.177.176.23] (dl-nas1-sao-C8B1B017.p001.terra.com.br [200.177.176.23]) by altamira.terra.com.br (Postfix) with ESMTP id 7B8393DC074 for ; Sun, 27 Apr 2003 10:10:17 -0300 (BRT) User-Agent: Microsoft-Outlook-Express-Macintosh-Edition/5.02.2022 Date: Sun, 27 Apr 2003 10:12:25 -0300 Subject: Re: [obm-l] Bisse=?ISO-8859-1?B?5+NvIGRlIMFyZWEgZSBQZXLt?= metro From: Claudio Buffara To: Message-ID: In-Reply-To: <200304260021.h3Q0Lw001283@Euler.impa.br> Mime-version: 1.0 Content-type: multipart/alternative; boundary="MS_Mac_OE_3134283145_134758_MIME_Part" Sender: owner-obm-l@sucuri.mat.puc-rio.br Precedence: bulk Reply-To: obm-l@mat.puc-rio.br > This message is in MIME format. Since your mail reader does not understand this format, some or all of this message may not be legible. --MS_Mac_OE_3134283145_134758_MIME_Part Content-type: text/plain; charset="US-ASCII" Content-transfer-encoding: 7bit Oi, Eduardo: Fiquei muito feliz com sua resposta. A partir de hoje, "bissectar" e "intersectar" estarao devidamente incorporados ao meu vocabulario. Obrigado e um abraco, Claudio. on 25.04.03 21:20, Eduardo Wagner at wagner@impa.br wrote: Bissectar esta perfeitamente correto e devemos usar. Ainda nao esta nos dicionarios mas vai aparecer logo. Existe bisseccao e o verbo correspondente eh exatamente bissectar. Outra coisa incrivel eh que fazer a intersecao eh, naturalmente, intersectar, outro verbo que ainda nao esta nos dicionarios mas que devemos usar. Repare que frequentemente usam erradamente o verbo interceptar com o sentido de fazer a intersecao. Interceptar eh outra coisa. Significa interromper um movimento (o carro da policia interceptou o dos bandidos..., o missil interceptou o foguete...). Assim, por exemplo, duas retas se intersectam no ponto P (e nao se interceptam). --MS_Mac_OE_3134283145_134758_MIME_Part Content-type: text/html; charset="US-ASCII" Content-transfer-encoding: quoted-printable Re: [obm-l] Bisse??o de ?rea e Per? metro Oi, Eduardo:

Fiquei muito feliz com sua resposta. A partir de hoje, "bissectar"= ; e "intersectar" estarao devidamente incorporados ao meu vocabula= rio.

Obrigado e um abraco,
Claudio.


on 25.04.03 21:20, Eduardo Wagner at wagner@impa.br wrote:

Bissectar esta perfeitamente correto e devemos usar.
Ainda nao esta nos dicionarios mas vai aparecer logo.
Existe bisseccao e o verbo correspondente eh exatamente
bissectar.
Outra coisa incrivel eh que fazer a intersecao eh, naturalmente,
intersectar, outro verbo que ainda nao esta nos dicionarios
mas que devemos usar. Repare que frequentemente usam
erradamente o verbo interceptar com o sentido de fazer
a intersecao. Interceptar eh outra coisa. Significa interromper
um movimento (o carro da policia interceptou o dos bandidos...,
o missil interceptou o foguete...). Assim, por exemplo, duas
retas se intersectam no ponto P (e nao se interceptam).

--MS_Mac_OE_3134283145_134758_MIME_Part-- ========================================================================= Instruções para entrar na lista, sair da lista e usar a lista em http://www.mat.puc-rio.br/~nicolau/olimp/obm-l.html ========================================================================= From owner-obm-l@sucuri.mat.puc-rio.br Sun Apr 27 10:14:49 2003 Return-Path: Received: (from majordom@localhost) by sucuri.mat.puc-rio.br (8.9.3/8.9.3) id KAA20675 for obm-l-MTTP; Sun, 27 Apr 2003 10:10:47 -0300 Received: from ivoti.terra.com.br (ivoti.terra.com.br [200.176.3.20]) by sucuri.mat.puc-rio.br (8.9.3/8.9.3) with ESMTP id KAA20671 for ; Sun, 27 Apr 2003 10:10:43 -0300 Received: from jurere.terra.com.br (jurere.terra.com.br [200.176.3.49]) by ivoti.terra.com.br (Postfix) with ESMTP id 750B040827B for ; Sun, 27 Apr 2003 10:10:13 -0300 (BRT) Received: from [200.177.176.23] (dl-nas1-sao-C8B1B017.p001.terra.com.br [200.177.176.23]) by jurere.terra.com.br (Postfix) with ESMTP id 9FDC9138058 for ; Sun, 27 Apr 2003 10:10:12 -0300 (BRT) User-Agent: Microsoft-Outlook-Express-Macintosh-Edition/5.02.2022 Date: Sun, 27 Apr 2003 10:12:20 -0300 Subject: Re: [obm-l] Limites From: Claudio Buffara To: Message-ID: In-Reply-To: <3EAB65C5.7030704@niski.com> Mime-version: 1.0 Content-type: text/plain; charset="ISO-8859-1" Content-Transfer-Encoding: 8bit X-MIME-Autoconverted: from quoted-printable to 8bit by sucuri.mat.puc-rio.br id KAA20672 Sender: owner-obm-l@sucuri.mat.puc-rio.br Precedence: bulk Reply-To: obm-l@mat.puc-rio.br on 27.04.03 02:08, niski at fabio@niski.com wrote: > Pretendo nesta mensagem responder ao Leandro Recova, ao prof. Morgado, e > ao Claudio Buffara. > > prof. Morgado, desculpe.Eu estava acostumado com o acrônimo cbrt (raiz > cubica) e não fui cuidadoso o suficiente para fazer tal referencia. > > Claudio e Leandro : > Muito estranho... > Para o Leandro o limite deu 0, para o Buffara não existe, no Mathematica > deu -infinito e a resposta do livro é infinito !!!! e agora? > > Quanto a utilizar sin no lugar de sen , não acho que seja um grande > problema, alias sin remete ao termo original em latim "sinus". > Na minha opinião nao acho uma boa a lingua portuguesa mudar alguns > termos originais...matriz por exemplo..pq este Z? pq nao deixar do > originial cunhado por Sylvester!?!? Já vi coisas tb do tipo "Onduletas" > para designar wavelets...eu acho que seria a mesma coisa que se referir > ao Newton como "Isaque Newton". Oi, Niski: Nao ligue muito pros meus comentarios sobre notacao - o importante eh se fazer entender e isso voce conseguiu (mesmo com cbrt e sin). No mais, toda vez que eu fizer um comentario mais acido, pode ter certeza que eh soh brincadeira - eu sou o tipo de pessoa que "perde o amigo mas nao perde a piada" - portanto, nao me leve muito a serio nestas horas. Eu ja me ferrei varias vezes porque nem too mundo tem o mesmo senso de humor, mas nao consigo evitar. Assim, ja fica o aviso... Sobre o limite, eu cheguei a conclusao da que os limites laterais eram + e - infinito mas fiquei com um pouco de medo de ter errado as contas. Sabe o que eu fiz? Calculei no computador o valor da expressao original em x = 1,5707 e 1,5709 e achei valores enormes com sinais opostos - essa evidencia empirica me convenceu. Nao eh aceitavel como prova, mas muita matematica de 1a. qualidade ja foi descoberta atraves de observacoes empiricas (por exemplo, Gauss conjecturou o teorema dos nos. primos com base em analise numerica de uma tabela de primos - haja falto do que fazer, hem???) Um abraco, Claudio. ========================================================================= Instruções para entrar na lista, sair da lista e usar a lista em http://www.mat.puc-rio.br/~nicolau/olimp/obm-l.html ========================================================================= From owner-obm-l@sucuri.mat.puc-rio.br Sun Apr 27 10:31:54 2003 Return-Path: Received: (from majordom@localhost) by sucuri.mat.puc-rio.br (8.9.3/8.9.3) id KAA21765 for obm-l-MTTP; Sun, 27 Apr 2003 10:27:53 -0300 Received: from ivoti.terra.com.br (ivoti.terra.com.br [200.176.3.20]) by sucuri.mat.puc-rio.br (8.9.3/8.9.3) with ESMTP id KAA21748 for ; Sun, 27 Apr 2003 10:27:47 -0300 Received: from gunga.terra.com.br (gunga.terra.com.br [200.176.3.45]) by ivoti.terra.com.br (Postfix) with ESMTP id 094344081C3 for ; Sun, 27 Apr 2003 10:27:16 -0300 (BRT) Received: from [200.177.186.127] (dl-nas4-sao-C8B1BA7F.p001.terra.com.br [200.177.186.127]) by gunga.terra.com.br (Postfix) with ESMTP id 0D651128057 for ; Sun, 27 Apr 2003 10:27:15 -0300 (BRT) User-Agent: Microsoft-Outlook-Express-Macintosh-Edition/5.02.2022 Date: Sun, 27 Apr 2003 10:28:29 -0300 Subject: Re: [obm-l] Pontos pintados From: Claudio Buffara To: Message-ID: In-Reply-To: <200304270506.h3R56uI15953@Gauss.impa.br> Mime-version: 1.0 Content-type: text/plain; charset="US-ASCII" Content-transfer-encoding: 7bit Sender: owner-obm-l@sucuri.mat.puc-rio.br Precedence: bulk Reply-To: obm-l@mat.puc-rio.br on 27.04.03 02:06, Carlos Gustavo Tamm de Araujo Moreira at gugu@impa.br wrote: > Caro Claudio, > Suponha que em todo segmento unitario as extremidades tem cores > distintas.Se X e Y estao a distancia raiz(3), e' possivel achar A e B no > plano tais que os triangulos XAB e ABY sejam equilateros de lado 1. Assim, > A e B tem cores distintas e diferentes da cor de X, donde a cor de Y deve > ser igual a cor de X. Considere agora um triangulo XYZ com lados > XY=XZ=raiz(3) e YZ=1. Temos que as cores de Y e de Z devem ser iguais a cor > de X mas a cor de Y deve ser distinta da cor de Z, absurdo. > Eu sei fazer o seu problema da equacao diofantina usando um pouco de > aritmetica em Z[i]. Voce tem uma solucao que nao usa isso ? > Abracos, > Gugu > > Oi, Gugu: Entao, A e B pertencem a mediatriz do segmento XY e distam, cada um, de 1/2 do ponto medio de XY. Z pertence a circunferencia de centro X e raio raiz(3). Eu nao entendi por que as cores de Z e X devem ser iguais. *** Na equacao diofantina eu estava procurando uma solucao por consideracoes de congruencia, mas nao consegui achar. Vou tentar usar aritmetica Z[i] e ver no que dah. Alias, tai um bom topico pra outro artigo da Eureka: o uso de Z[i], Z[raiz(-2)], etc. na solucao de equacoes diofantinas. Este tipo de equacao nao foi abordado no artigo do Antonio Caminha na Eureka 7. Um abraco, Claudio. >> >> Caros colegas da lista: >> >> Outro probleminha que estah me dando trabalho: >> >> Cada ponto do plano eh pintado de uma cor, dentre tres cores possiveis. >> Prove que existe um segmento unitario cujas extremidades tem a mesma cor. >> >> ****** >> >> Uma equacao diofantina bonitinha: >> >> Prove que x^2 + (x+1)^2 = y^3 nao tem solucao em inteiros positivos. >> >> ****** >> >> E aqui vai a dica pro problema da sequencia de 100 numeros reais e das >> subsequencias de 8 e 9 termos com mesma media: suponha inicialmente que os >> termos da sequencia sao racionais. Em seguida, use o fato de que R eh um >> espaco vetorial sobre Q. >> >> Seria otimo se alguem descobrisse uma solucao que nao usasse a dica. >> >> Um abraco, >> Claudio. >> >> ========================================================================= >> Instru??es para entrar na lista, sair da lista e usar a lista em >> http://www.mat.puc-rio.br/~nicolau/olimp/obm-l.html >> ========================================================================= > > ========================================================================= > Instru??es para entrar na lista, sair da lista e usar a lista em > http://www.mat.puc-rio.br/~nicolau/olimp/obm-l.html > ========================================================================= > ========================================================================= Instruções para entrar na lista, sair da lista e usar a lista em http://www.mat.puc-rio.br/~nicolau/olimp/obm-l.html ========================================================================= From owner-obm-l@sucuri.mat.puc-rio.br Sun Apr 27 10:33:39 2003 Return-Path: Received: (from majordom@localhost) by sucuri.mat.puc-rio.br (8.9.3/8.9.3) id KAA21891 for obm-l-MTTP; Sun, 27 Apr 2003 10:29:53 -0300 Received: from web80513.mail.yahoo.com (web80513.mail.yahoo.com [66.218.79.83]) by sucuri.mat.puc-rio.br (8.9.3/8.9.3) with SMTP id KAA21877 for ; Sun, 27 Apr 2003 10:29:46 -0300 Message-ID: <20030427132914.40459.qmail@web80513.mail.yahoo.com> Received: from [200.146.8.48] by web80513.mail.yahoo.com via HTTP; Sun, 27 Apr 2003 10:29:14 ART Date: Sun, 27 Apr 2003 10:29:14 -0300 (ART) From: "=?iso-8859-1?q?guilherme=20S.?=" Subject: [obm-l] ajuda com desigualdade To: obm-l@mat.puc-rio.br MIME-Version: 1.0 Content-Type: text/plain; charset=iso-8859-1 Content-Transfer-Encoding: 8bit Sender: owner-obm-l@sucuri.mat.puc-rio.br Precedence: bulk Reply-To: obm-l@mat.puc-rio.br prove que: 1/(a^3+b^3+abc)+1/(a^3+c^3+abc)+1/(b^3+c^3+abc)<=1/abc sendo a,b e c reais positivos _______________________________________________________________________ Yahoo! Mail O melhor e-mail gratuito da internet: 6MB de espaço, antivírus, acesso POP3, filtro contra spam. http://br.mail.yahoo.com/ ========================================================================= Instruções para entrar na lista, sair da lista e usar a lista em http://www.mat.puc-rio.br/~nicolau/olimp/obm-l.html ========================================================================= From owner-obm-l@sucuri.mat.puc-rio.br Sun Apr 27 10:59:23 2003 Return-Path: Received: (from majordom@localhost) by sucuri.mat.puc-rio.br (8.9.3/8.9.3) id KAA23240 for obm-l-MTTP; Sun, 27 Apr 2003 10:55:23 -0300 Received: from artemis.opendf.com.br (artemis.opengate.com.br [200.181.71.14]) by sucuri.mat.puc-rio.br (8.9.3/8.9.3) with ESMTP id KAA23235 for ; Sun, 27 Apr 2003 10:55:18 -0300 Received: from localhost (localhost [127.0.0.1]) by artemis.opendf.com.br (Postfix) with ESMTP id BFEF52BF0C for ; Sun, 27 Apr 2003 10:56:06 -0300 (BRT) Received: from artemis.opendf.com.br ([127.0.0.1]) by localhost (artemis.opengate.com.br [127.0.0.1:10024]) (amavisd-new) with ESMTP id 07803-10 for ; Sun, 27 Apr 2003 10:56:06 -0300 (BRT) Received: from computer (200-181-088-208.bsace7001.dsl.brasiltelecom.net.br [200.181.88.208]) by artemis.opendf.com.br (Postfix) with ESMTP id 911AC2BF0A for ; Sun, 27 Apr 2003 10:56:05 -0300 (BRT) From: "Artur Costa Steiner" To: Subject: RE: [obm-l] integral de 1/x - uma retificacao Date: Sun, 27 Apr 2003 10:54:48 -0300 Organization: Steiner Consultoria LTDA Message-ID: MIME-Version: 1.0 Content-Type: text/plain; charset="us-ascii" Content-Transfer-Encoding: 7bit X-Priority: 3 (Normal) X-MSMail-Priority: Normal X-Mailer: Microsoft Outlook, Build 10.0.2627 In-Reply-To: X-MimeOLE: Produced By Microsoft MimeOLE V6.00.2800.1165 Importance: Normal X-Virus-Scanned: by amavisd-new Sender: owner-obm-l@sucuri.mat.puc-rio.br Precedence: bulk Reply-To: obm-l@mat.puc-rio.br Oi Na minha outra mensagem sobre este assunto, acabei escrevendo uma frase de forma imprecisa, na realidade, errada. Se f possui inversa, eh diferenciavel em x e continua em algum intervalo contendo x, entao o que garante a diferenciabilidade de sua inversa g em a eh o fato de f'(a) ser diferente de zero. Na outra mensagem, pensando longe demais e considerando que a derivada de e^x e ela propria, cacabei escrevendo que ln eh diferenciavel para x >0 porque e^x >0 para todo x real. Tecnicamente, o certo eh que ln eh diferenciavel porque (e^x)' = e^x >0. Acaba dando na mesma , mas eh importante observar que se trata do fato de que a derivada de e^x nao se anula. Artur ========================================================================= Instruções para entrar na lista, sair da lista e usar a lista em http://www.mat.puc-rio.br/~nicolau/olimp/obm-l.html ========================================================================= From owner-obm-l@sucuri.mat.puc-rio.br Sun Apr 27 11:02:36 2003 Return-Path: Received: (from majordom@localhost) by sucuri.mat.puc-rio.br (8.9.3/8.9.3) id KAA23429 for obm-l-MTTP; Sun, 27 Apr 2003 10:58:49 -0300 Received: from smtp.ieg.com.br (sharon.protocoloweb.com.br [200.226.139.12]) by sucuri.mat.puc-rio.br (8.9.3/8.9.3) with ESMTP id KAA23424 for ; Sun, 27 Apr 2003 10:58:43 -0300 Received: from computador (SHASTA199232.ig.com.br [200.151.199.232] (may be forged)) by smtp.ieg.com.br (8.12.8/8.9.3) with SMTP id h3RDogWj013471 for ; Sun, 27 Apr 2003 10:50:44 -0300 (BRT) Message-ID: <003201c30cc4$56050a00$e8c797c8@computador> From: "=?iso-8859-1?B?QW5kcukgTHXteg==?=" To: "Lista OBM" Subject: [obm-l] =?iso-8859-1?Q?Indu=E7=E3o_Finita?= Date: Sun, 27 Apr 2003 10:53:04 -0300 MIME-Version: 1.0 Content-Type: multipart/alternative; boundary="----=_NextPart_000_002F_01C30CAB.2E19F5A0" X-Priority: 3 X-MSMail-Priority: Normal X-Mailer: Microsoft Outlook Express 6.00.2600.0000 X-MimeOLE: Produced By Microsoft MimeOLE V6.00.2600.0000 Sender: owner-obm-l@sucuri.mat.puc-rio.br Precedence: bulk Reply-To: obm-l@mat.puc-rio.br This is a multi-part message in MIME format. ------=_NextPart_000_002F_01C30CAB.2E19F5A0 Content-Type: text/plain; charset="iso-8859-1" Content-Transfer-Encoding: quoted-printable Oi amigos, gostaria que voc=EAs me ajudasse nessas tr=EAs quest=F5es de = indu=E7=E3o: 1) Para n >=3D 3 mostre que (n+1)^n < n^(n+1) 2) Para n >=3D 2 mostre que 1 * 3 * 5 * ..... * (2n - 1) < n^n 3) Mostre que =E9 poss=EDvel pagar, sem receber troco, qualquer quantia = inteira de reais, maior do que 7, com notas de 3 reais e 5 reais. 4) Para n >=3D 3, mostre que 2^n + 1 =E9 um n=FAmero composto se n n=E3o = =E9 uma pot=EAncia de 2. Desde j=E1 muito obrigado. ------=_NextPart_000_002F_01C30CAB.2E19F5A0 Content-Type: text/html; charset="iso-8859-1" Content-Transfer-Encoding: quoted-printable
Oi amigos, gostaria que voc=EAs me = ajudasse nessas=20 tr=EAs quest=F5es de indu=E7=E3o:
 
1) Para n >=3D 3 mostre que (n+1)^n = <=20 n^(n+1)
 
2) Para n >=3D 2 mostre que 1 * = 3 * 5 * .....=20 * (2n - 1) < n^n
 
3) Mostre que =E9 poss=EDvel = pagar, sem receber=20 troco, qualquer quantia inteira de reais, maior do que 7, com notas = de 3=20 reais e 5 reais.
 
4) Para n >=3D 3, mostre que 2^n + 1 = =E9 um n=FAmero=20 composto se n n=E3o =E9 uma pot=EAncia de 2.
 
Desde j=E1 muito = obrigado.
------=_NextPart_000_002F_01C30CAB.2E19F5A0-- ========================================================================= Instruções para entrar na lista, sair da lista e usar a lista em http://www.mat.puc-rio.br/~nicolau/olimp/obm-l.html ========================================================================= From owner-obm-l@sucuri.mat.puc-rio.br Sun Apr 27 11:35:23 2003 Return-Path: Received: (from majordom@localhost) by sucuri.mat.puc-rio.br (8.9.3/8.9.3) id LAA24181 for obm-l-MTTP; Sun, 27 Apr 2003 11:19:54 -0300 Received: from itaqui.terra.com.br (itaqui.terra.com.br [200.176.3.19]) by sucuri.mat.puc-rio.br (8.9.3/8.9.3) with ESMTP id LAA24177 for ; Sun, 27 Apr 2003 11:19:50 -0300 Received: from canela.terra.com.br (canela.terra.com.br [200.176.3.79]) by itaqui.terra.com.br (Postfix) with ESMTP id 7554B3BC222 for ; Sun, 27 Apr 2003 11:19:09 -0300 (BRT) Received: from niski.com (unknown [200.148.200.81]) (authenticated user fniski) by canela.terra.com.br (Postfix) with ESMTP id 5CF7D224086 for ; Sun, 27 Apr 2003 11:19:08 -0300 (BRT) Message-ID: <3EABE6E4.1080401@niski.com> Date: Sun, 27 Apr 2003 07:19:16 -0700 From: niski User-Agent: Mozilla/5.0 (Windows; U; Windows NT 5.1; en-US; rv:1.0.2) Gecko/20030208 Netscape/7.02 X-Accept-Language: en-us, en MIME-Version: 1.0 To: obm-l@mat.puc-rio.br Subject: Re: [obm-l] Limites References: Content-Type: text/plain; charset=ISO-8859-1; format=flowed Content-Transfer-Encoding: 8bit Sender: owner-obm-l@sucuri.mat.puc-rio.br Precedence: bulk Reply-To: obm-l@mat.puc-rio.br > Oi, Niski: > > Nao ligue muito pros meus comentarios sobre notacao - o importante eh se > fazer entender e isso voce conseguiu (mesmo com cbrt e sin). No mais, toda > vez que eu fizer um comentario mais acido, pode ter certeza que eh soh > brincadeira - eu sou o tipo de pessoa que "perde o amigo mas nao perde a > piada" - portanto, nao me leve muito a serio nestas horas. Eu ja me ferrei > varias vezes porque nem too mundo tem o mesmo senso de humor, mas nao > consigo evitar. Assim, ja fica o aviso... Claudio, nao tem problema. É dificil eu ficar nervoso por uma discussao via internet, e tb em outros foruns que participo eu até mesmo gosto de atiçar uma polemica ! > Sobre o limite, eu cheguei a conclusao da que os limites laterais eram + e - > infinito mas fiquei com um pouco de medo de ter errado as contas. > Sabe o que eu fiz? Calculei no computador o valor da expressao original em x > = 1,5707 e 1,5709 e achei valores enormes com sinais opostos - essa > evidencia empirica me convenceu. Nao eh aceitavel como prova, mas muita > matematica de 1a. qualidade ja foi descoberta atraves de observacoes > empiricas (por exemplo, Gauss conjecturou o teorema dos nos. primos com base > em analise numerica de uma tabela de primos - haja falto do que fazer, > hem???) pode crer! falou niski -- [about him:] It is rare to find learned men who are clean, do not stink and have a sense of humour. -Gottfried Whilhem Leibniz ========================================================================= Instruções para entrar na lista, sair da lista e usar a lista em http://www.mat.puc-rio.br/~nicolau/olimp/obm-l.html ========================================================================= From owner-obm-l@sucuri.mat.puc-rio.br Sun Apr 27 11:42:16 2003 Return-Path: Received: (from majordom@localhost) by sucuri.mat.puc-rio.br (8.9.3/8.9.3) id LAA24471 for obm-l-MTTP; Sun, 27 Apr 2003 11:30:53 -0300 Received: from ivoti.terra.com.br (ivoti.terra.com.br [200.176.3.20]) by sucuri.mat.puc-rio.br (8.9.3/8.9.3) with ESMTP id LAA24457 for ; Sun, 27 Apr 2003 11:30:46 -0300 Received: from altamira.terra.com.br (altamira.terra.com.br [200.176.3.40]) by ivoti.terra.com.br (Postfix) with ESMTP id 2E2804082DE for ; Sun, 27 Apr 2003 11:30:16 -0300 (BRT) Received: from [200.177.192.28] (dl-nas2-sao-C8B1C01C.p001.terra.com.br [200.177.192.28]) by altamira.terra.com.br (Postfix) with ESMTP id 20BFB3DC06F for ; Sun, 27 Apr 2003 11:30:15 -0300 (BRT) User-Agent: Microsoft-Outlook-Express-Macintosh-Edition/5.02.2022 Date: Sun, 27 Apr 2003 11:31:43 -0300 Subject: Re: [obm-l] Pontos pintados From: Claudio Buffara To: Message-ID: In-Reply-To: <200304270506.h3R56uI15953@Gauss.impa.br> Mime-version: 1.0 Content-type: text/plain; charset="US-ASCII" Content-transfer-encoding: 7bit Sender: owner-obm-l@sucuri.mat.puc-rio.br Precedence: bulk Reply-To: obm-l@mat.puc-rio.br Oi, Gugu: Acho que resolvi usando aritmetica em Z[i]: Suponhamos que equacao diofantina: y^3 = (x+1)^2 + x^2 tenha solucoes inteiras positivas. Fatorando o lado direito (em Z[i]), teremos: y^3 = (x + 1 + ix)(x + 1 - ix) x eh inteiro positivo ==> mdc(x+1+ix,x+1-ix) = mdc(x+1+ix,2ix). Mas mdc(x+1+ix,2ix) divide mdc(2(x+1),2ix) = 2mdc(x+1,ix) = 2. Como x+1 e x tem paridades distintas, concluimos que: mdc(x+1+ix,x+1-ix) = 1. Logo, como Z[i] eh um dominio de fatoracao unica, existem inteiros a, b tais que: x + 1 + ix = (a + bi)^3 e x + 1 - ix = (a - bi)^3 ==> x + 1 = a^3 - 3ab^2 x = 3a^2b - b^3 Subtraindo: 1 = a^3 - 3a^2b - 3ab^2 + b^3 ==> 1 = a^3 + b^3 - 3ab(a+b) ==> 1 = (a+b)^3 - 6ab(a+b) ==> 1 = (a+b)(a^2 - 4ab + b^2) ==> a + b = 1 e a^2 - 4ab + b^2 = 1 ou a + b = -1 e a^2 - 4ab + b^2 = -1 A primeira alternativa resulta em: a^2 - 4a(1 - a) + (1 - a)^2 = 1 ==> 6a^2 - 6a = 0 ==> (a = 0 e b = 1) ou (a = 1 e b = 0) ==> x = -1 ou x = 0 ==> nao servem (x tem que ser >= 1) A segunda alternativa resulta em: a^2 - 4a(-1 - a) + (-1 - a)^2 = 1 ==> 6a^2 + 6a = 0 ==> (a = 0 e b = -1) ou (a = -1 e b = 0) ==> (x = 1 e x+1 = 0) ou (x = -2 e x = 0) ==> contradicao Logo, concluimos que a equacao original nao tem solucoes inteiras positivas. Um abraco, Claudio. on 27.04.03 02:06, Carlos Gustavo Tamm de Araujo Moreira at gugu@impa.br wrote: > Caro Claudio, > Suponha que em todo segmento unitario as extremidades tem cores > distintas.Se X e Y estao a distancia raiz(3), e' possivel achar A e B no > plano tais que os triangulos XAB e ABY sejam equilateros de lado 1. Assim, > A e B tem cores distintas e diferentes da cor de X, donde a cor de Y deve > ser igual a cor de X. Considere agora um triangulo XYZ com lados > XY=XZ=raiz(3) e YZ=1. Temos que as cores de Y e de Z devem ser iguais a cor > de X mas a cor de Y deve ser distinta da cor de Z, absurdo. > Eu sei fazer o seu problema da equacao diofantina usando um pouco de > aritmetica em Z[i]. Voce tem uma solucao que nao usa isso ? > Abracos, > Gugu > > >> >> Caros colegas da lista: >> >> Outro probleminha que estah me dando trabalho: >> >> Cada ponto do plano eh pintado de uma cor, dentre tres cores possiveis. >> Prove que existe um segmento unitario cujas extremidades tem a mesma cor. >> >> ****** >> >> Uma equacao diofantina bonitinha: >> >> Prove que x^2 + (x+1)^2 = y^3 nao tem solucao em inteiros positivos. >> >> ****** >> >> E aqui vai a dica pro problema da sequencia de 100 numeros reais e das >> subsequencias de 8 e 9 termos com mesma media: suponha inicialmente que os >> termos da sequencia sao racionais. Em seguida, use o fato de que R eh um >> espaco vetorial sobre Q. >> >> Seria otimo se alguem descobrisse uma solucao que nao usasse a dica. >> >> Um abraco, >> Claudio. ========================================================================= Instruções para entrar na lista, sair da lista e usar a lista em http://www.mat.puc-rio.br/~nicolau/olimp/obm-l.html ========================================================================= From owner-obm-l@sucuri.mat.puc-rio.br Sun Apr 27 12:14:11 2003 Return-Path: Received: (from majordom@localhost) by sucuri.mat.puc-rio.br (8.9.3/8.9.3) id MAA25967 for obm-l-MTTP; Sun, 27 Apr 2003 12:10:17 -0300 Received: from ivoti.terra.com.br (ivoti.terra.com.br [200.176.3.20]) by sucuri.mat.puc-rio.br (8.9.3/8.9.3) with ESMTP id MAA25963 for ; Sun, 27 Apr 2003 12:10:13 -0300 Received: from marova.terra.com.br (marova.terra.com.br [200.176.3.39]) by ivoti.terra.com.br (Postfix) with ESMTP id 33DFD4083FE for ; Sun, 27 Apr 2003 12:09:42 -0300 (BRT) Received: from niski.com (unknown [200.148.200.81]) (authenticated user fniski) by marova.terra.com.br (Postfix) with ESMTP id 144D83DC065 for ; Sun, 27 Apr 2003 12:09:41 -0300 (BRT) Message-ID: <3EABF2BA.5090705@niski.com> Date: Sun, 27 Apr 2003 08:09:46 -0700 From: niski User-Agent: Mozilla/5.0 (Windows; U; Windows NT 5.1; en-US; rv:1.0.2) Gecko/20030208 Netscape/7.02 X-Accept-Language: en-us, en MIME-Version: 1.0 To: obm-l@mat.puc-rio.br Subject: [obm-l] Mais calculo Content-Type: text/plain; charset=ISO-8859-1; format=flowed Content-Transfer-Encoding: 8bit Sender: owner-obm-l@sucuri.mat.puc-rio.br Precedence: bulk Reply-To: obm-l@mat.puc-rio.br Ola pessoal, travei neste daqui..alguem poderia me ajudar?! Obrigado Seja f definida em R e suponha que existe M > 0 tal que, para todo x, |f(x) - f(p)| <= M|x-p|^2 a) Mostre que f é continua b) Calcule, caso exista, lim[x->p] (f(x)-f(p))/(x-p) Obrigado. niski -- [about him:] It is rare to find learned men who are clean, do not stink and have a sense of humour. -Gottfried Whilhem Leibniz ========================================================================= Instruções para entrar na lista, sair da lista e usar a lista em http://www.mat.puc-rio.br/~nicolau/olimp/obm-l.html ========================================================================= From owner-obm-l@sucuri.mat.puc-rio.br Sun Apr 27 12:18:09 2003 Return-Path: Received: (from majordom@localhost) by sucuri.mat.puc-rio.br (8.9.3/8.9.3) id MAA26063 for obm-l-MTTP; Sun, 27 Apr 2003 12:14:18 -0300 Received: from traven9.uol.com.br (traven9.uol.com.br [200.221.29.35]) by sucuri.mat.puc-rio.br (8.9.3/8.9.3) with ESMTP id MAA26058 for ; Sun, 27 Apr 2003 12:14:15 -0300 Received: from gauss ([200.158.96.92]) by traven9.uol.com.br (8.9.1/8.9.1) with SMTP id MAA16888 for ; Sun, 27 Apr 2003 12:13:44 -0300 (BRT) Message-ID: <002501c30cd0$5bb56010$5c609ec8@gauss> From: "Domingos Jr." To: References: Subject: Re: [obm-l] integral de 1/x - uma retificacao Date: Sun, 27 Apr 2003 12:19:11 -0300 MIME-Version: 1.0 Content-Type: text/plain; charset="iso-8859-1" Content-Transfer-Encoding: 8bit X-Priority: 3 X-MSMail-Priority: Normal X-Mailer: Microsoft Outlook Express 6.00.2800.1158 X-MimeOLE: Produced By Microsoft MimeOLE V6.00.2800.1165 Sender: owner-obm-l@sucuri.mat.puc-rio.br Precedence: bulk Reply-To: obm-l@mat.puc-rio.br > Oi > Na minha outra mensagem sobre este assunto, acabei escrevendo uma frase > de forma imprecisa, na realidade, errada. Se f possui inversa, eh > diferenciavel em x e continua em algum intervalo contendo x, entao o que > garante a diferenciabilidade de sua inversa g em a eh o fato de f'(a) > ser diferente de zero. Na outra mensagem, pensando longe demais e > considerando que a derivada de e^x e ela propria, cacabei escrevendo que > ln eh diferenciavel para x >0 porque e^x >0 para todo x real. > Tecnicamente, o certo eh que ln eh diferenciavel porque (e^x)' = e^x >0. > Acaba dando na mesma , mas eh importante observar que se trata do fato > de que a derivada de e^x nao se anula. > Artur > tem razão! nem me passou pela cabeça utilizar a definição de lnx como a f. inversa da exponencial e depois derivar, fica realmente bem simples: ln(e^x) = x [ln(e^x)]' = x' = 1 ln'(e^x)[e^x]' = e^x.ln(e^x) = 1 ln'(e^x) = 1/(e^x) agora vemos que para todo y real positivo, existe um único valor x tal que y = e^x (e^x é bijetora em IR+). logo ln'(y) = 1/y [ ]'s ========================================================================= Instruções para entrar na lista, sair da lista e usar a lista em http://www.mat.puc-rio.br/~nicolau/olimp/obm-l.html ========================================================================= From owner-obm-l@sucuri.mat.puc-rio.br Sun Apr 27 12:19:26 2003 Return-Path: Received: (from majordom@localhost) by sucuri.mat.puc-rio.br (8.9.3/8.9.3) id MAA26151 for obm-l-MTTP; Sun, 27 Apr 2003 12:15:39 -0300 Received: from sidney1.bol.com.br (sidney1.bol.com.br [200.221.24.206]) by sucuri.mat.puc-rio.br (8.9.3/8.9.3) with ESMTP id MAA26147 for ; Sun, 27 Apr 2003 12:15:35 -0300 Received: from bol.com.br (200.221.24.129) by sidney1.bol.com.br (5.1.071) id 3E766A3D008D0EDD for obm-l@mat.puc-rio.br; Sun, 27 Apr 2003 12:15:05 -0300 Date: Sun, 27 Apr 2003 12:15:05 -0300 Message-Id: Subject: [obm-l] ???? MIME-Version: 1.0 Content-Type: text/plain;charset="iso-8859-1" From: "THIAGO" To: obm-l@mat.puc-rio.br X-XaM3-API-Version: 2.4 R3 ( B4 ) X-SenderIP: 200.151.32.108 Content-Transfer-Encoding: 8bit X-MIME-Autoconverted: from quoted-printable to 8bit by sucuri.mat.puc-rio.br id MAA26148 Sender: owner-obm-l@sucuri.mat.puc-rio.br Precedence: bulk Reply-To: obm-l@mat.puc-rio.br Bom dia! Meu nome é Thiago e acabo de entrar na lista....Mas tenho uma dúvida cruel: acabei o ensino médio; faço cursinho pré-vestibular e gostaria de saber se eu posso participar da Olímpíada Brasileira de Matemática, mesmo não estando no 2º grau. Um dos meus professores de matematica do curso ministrou a olimpiada ano passado no meu colégio..... O que posso fazer? Desde já agradecido Thiago Barros da Silva - RJ __________________________________________________________________________ Seleção de Softwares UOL. 10 softwares escolhidos pelo UOL para você e sua família. http://www.uol.com.br/selecao ========================================================================= Instruções para entrar na lista, sair da lista e usar a lista em http://www.mat.puc-rio.br/~nicolau/olimp/obm-l.html ========================================================================= From owner-obm-l@sucuri.mat.puc-rio.br Sun Apr 27 13:31:35 2003 Return-Path: Received: (from majordom@localhost) by sucuri.mat.puc-rio.br (8.9.3/8.9.3) id NAA28360 for obm-l-MTTP; Sun, 27 Apr 2003 13:27:38 -0300 Received: from artemis.opendf.com.br (artemis.opengate.com.br [200.181.71.14]) by sucuri.mat.puc-rio.br (8.9.3/8.9.3) with ESMTP id NAA28356 for ; Sun, 27 Apr 2003 13:27:34 -0300 Received: from localhost (localhost [127.0.0.1]) by artemis.opendf.com.br (Postfix) with ESMTP id 7D81A2BED7 for ; Sun, 27 Apr 2003 13:28:24 -0300 (BRT) Received: from artemis.opendf.com.br ([127.0.0.1]) by localhost (artemis.opengate.com.br [127.0.0.1:10024]) (amavisd-new) with ESMTP id 10247-02 for ; Sun, 27 Apr 2003 13:28:23 -0300 (BRT) Received: from computer (200-181-088-208.bsace7001.dsl.brasiltelecom.net.br [200.181.88.208]) by artemis.opendf.com.br (Postfix) with ESMTP id 0D8962BED3 for ; Sun, 27 Apr 2003 13:28:23 -0300 (BRT) From: "Artur Costa Steiner" To: Subject: [obm-l] =?iso-8859-1?Q?RE:_=5Bobm-l=5D_Indu=E7=E3o_Finita?= Date: Sun, 27 Apr 2003 13:26:59 -0300 Organization: Steiner Consultoria LTDA Message-ID: <007501c30cd9$d6f20e50$9865fea9@computer> MIME-Version: 1.0 Content-Type: text/plain; charset="iso-8859-1" X-Priority: 3 (Normal) X-MSMail-Priority: Normal X-Mailer: Microsoft Outlook, Build 10.0.2627 In-Reply-To: <003201c30cc4$56050a00$e8c797c8@computador> X-MimeOLE: Produced By Microsoft MimeOLE V6.00.2800.1165 Importance: Normal X-Virus-Scanned: by amavisd-new Content-Transfer-Encoding: 8bit X-MIME-Autoconverted: from quoted-printable to 8bit by sucuri.mat.puc-rio.br id NAA28357 Sender: owner-obm-l@sucuri.mat.puc-rio.br Precedence: bulk Reply-To: obm-l@mat.puc-rio.br Oi amigos, gostaria que vocês me ajudasse nessas três questões de indução:   1) Para n >= 3 mostre que (n+1)^n < n^(n+1) Para qualquer natural n, temos que [(n+1)^n]/[n^(n+1)] =[(n+1)^n]/[n. n^n] = 1/n . [(n+1)/n]^n = 1/n [1+ 1/n]^n. Dentro dos colchetes temos a classica sequencia que converge para o numero e, a qual sabemos ser estritamente crescente. Logo, para todo n temos que [(n+1)^n]/[n^(n+1)] < 1/n e = e/n. Se n>=3, entao [(n+1)^n]/[n^(n+1)] < 1, pois e= 2,7182818...... Logo, para n>=3 temos (n+1)^n < n^(n+1), conforme desejado.   2) Para n >= 2 mostre que 1 * 3 * 5 * ..... * (2n - 1) < n^n Em virtude da desigualdade das medias aritmetica e geometrica, temos, para n>=2, que [1 * 3 * 5 * ..... * (2n - 1)]^(1/n) < [1 + 3 +...(2n -1)]/n. (observe a desigualdade estrita, pois para n>=2 os numeros sao diferentes). No numerador do segundo membro, temos a soma dos n primeiros impares, a soma de uma PA. Sabemos que esta soma eh n^2. Logo, [1 * 3 * 5 * ..... * (2n - 1)]^(1/n) Received: (from majordom@localhost) by sucuri.mat.puc-rio.br (8.9.3/8.9.3) id NAA28568 for obm-l-MTTP; Sun, 27 Apr 2003 13:40:26 -0300 Received: from paiol.terra.com.br (paiol.terra.com.br [200.176.3.18]) by sucuri.mat.puc-rio.br (8.9.3/8.9.3) with ESMTP id NAA28559 for ; Sun, 27 Apr 2003 13:40:16 -0300 Received: from canela.terra.com.br (canela.terra.com.br [200.176.3.79]) by paiol.terra.com.br (Postfix) with ESMTP id 89B6987D1D for ; Sun, 27 Apr 2003 13:39:46 -0300 (BRT) Received: from riemann.localdomain (RJ173173.user.veloxzone.com.br [200.149.173.173]) (authenticated user fabio.dias.moreira) by canela.terra.com.br (Postfix) with ESMTP id 2883F22406E for ; Sun, 27 Apr 2003 13:39:46 -0300 (BRT) Content-Type: text/plain; charset="iso-8859-1" From: =?iso-8859-1?q?F=E1bio=20Dias=20Moreira?= To: obm-l@mat.puc-rio.br Subject: Re: [obm-l] Cavalos de xadrez Date: Sun, 27 Apr 2003 13:35:39 -0300 User-Agent: KMail/1.4.3 References: <20030427124516.7613.qmail@web13006.mail.yahoo.com> In-Reply-To: <20030427124516.7613.qmail@web13006.mail.yahoo.com> MIME-Version: 1.0 Content-Transfer-Encoding: 8bit Message-Id: <200304271335.51398.fabio.dias.moreira@terra.com.br> Sender: owner-obm-l@sucuri.mat.puc-rio.br Precedence: bulk Reply-To: obm-l@mat.puc-rio.br -----BEGIN PGP SIGNED MESSAGE----- Hash: SHA1 On Sunday 27 April 2003 09:45, Helder Suzuki wrote: > Olá! > > (1) Quantos cavalos de xadrez podemos colocar em um > tabuleiro NxM tal que nenhum seja ameaçado por outro? > [...] Pinte o tablueiro de preto e branco e coloque um cavalo em uma casa branca. Qual a cor das casas que ele ameaça? Isso acha uma cota inferior para o resultado. Por outro lado, em uma configuração maximal, quadrados vazios são atacados pelo maior número possível de cavalos (isso é óbvio?). Logo, como a configuração gerada pela idéia acima atende a isso, também é cota superior. []s, - -- Fábio "ctg \pi" Dias Moreira -----BEGIN PGP SIGNATURE----- Version: GnuPG v1.0.6 (GNU/Linux) Comment: For info see http://www.gnupg.org iD8DBQE+rAbmalOQFrvzGQoRAlByAJ9tsp55XHjT6SCDdhL2cLfNFXKGfQCfb1nL J5D9jtwD99PJ7LSvp3nowdo= =8wwr -----END PGP SIGNATURE----- ========================================================================= Instruções para entrar na lista, sair da lista e usar a lista em http://www.mat.puc-rio.br/~nicolau/olimp/obm-l.html ========================================================================= From owner-obm-l@sucuri.mat.puc-rio.br Sun Apr 27 13:46:06 2003 Return-Path: Received: (from majordom@localhost) by sucuri.mat.puc-rio.br (8.9.3/8.9.3) id NAA28657 for obm-l-MTTP; Sun, 27 Apr 2003 13:42:19 -0300 Received: from artemis.opendf.com.br (artemis.opengate.com.br [200.181.71.14]) by sucuri.mat.puc-rio.br (8.9.3/8.9.3) with ESMTP id NAA28653 for ; Sun, 27 Apr 2003 13:42:15 -0300 Received: from localhost (localhost [127.0.0.1]) by artemis.opendf.com.br (Postfix) with ESMTP id 67CD02BED7 for ; Sun, 27 Apr 2003 13:43:05 -0300 (BRT) Received: from artemis.opendf.com.br ([127.0.0.1]) by localhost (artemis.opengate.com.br [127.0.0.1:10024]) (amavisd-new) with ESMTP id 10247-05 for ; Sun, 27 Apr 2003 13:43:04 -0300 (BRT) Received: from computer (200-181-088-208.bsace7001.dsl.brasiltelecom.net.br [200.181.88.208]) by artemis.opendf.com.br (Postfix) with ESMTP id 616AA2BED3 for ; Sun, 27 Apr 2003 13:43:04 -0300 (BRT) From: "Artur Costa Steiner" To: Subject: RE: [obm-l] integral de 1/x - uma retificacao Date: Sun, 27 Apr 2003 13:41:45 -0300 Organization: Steiner Consultoria LTDA Message-ID: <007601c30cdb$e3fb8a70$9865fea9@computer> MIME-Version: 1.0 Content-Type: text/plain; charset="iso-8859-1" X-Priority: 3 (Normal) X-MSMail-Priority: Normal X-Mailer: Microsoft Outlook, Build 10.0.2627 In-Reply-To: <002501c30cd0$5bb56010$5c609ec8@gauss> X-MimeOLE: Produced By Microsoft MimeOLE V6.00.2800.1165 Importance: Normal X-Virus-Scanned: by amavisd-new Content-Transfer-Encoding: 8bit X-MIME-Autoconverted: from quoted-printable to 8bit by sucuri.mat.puc-rio.br id NAA28654 Sender: owner-obm-l@sucuri.mat.puc-rio.br Precedence: bulk Reply-To: obm-l@mat.puc-rio.br >tem razão! >nem me passou pela cabeça utilizar a definição de lnx como a f. inversa da >exponencial e depois derivar, fica realmente bem simples: [Artur Costa Steiner] Esta, na realidade eh a definicao de ln > >ln(e^x) = x >[ln(e^x)]' = x' = 1 >ln'(e^x)[e^x]' = e^x.ln(e^x) = 1 >ln'(e^x) = 1/(e^x) > >agora vemos que para todo y real positivo, existe um único valor x tal que >y >= e^x (e^x é bijetora em IR+). >logo ln'(y) = 1/y [Artur Costa Steiner] E isso ai, amigo! Artur ========================================================================= Instruções para entrar na lista, sair da lista e usar a lista em http://www.mat.puc-rio.br/~nicolau/olimp/obm-l.html ========================================================================= From owner-obm-l@sucuri.mat.puc-rio.br Sun Apr 27 13:55:40 2003 Return-Path: Received: (from majordom@localhost) by sucuri.mat.puc-rio.br (8.9.3/8.9.3) id NAA28925 for obm-l-MTTP; Sun, 27 Apr 2003 13:51:48 -0300 Received: from silva5.uol.com.br (silva5.uol.com.br [200.221.29.52]) by sucuri.mat.puc-rio.br (8.9.3/8.9.3) with ESMTP id NAA28918 for ; Sun, 27 Apr 2003 13:51:39 -0300 Received: from gauss ([200.158.96.92]) by silva5.uol.com.br (8.9.1/8.9.1) with SMTP id NAA16939 for ; Sun, 27 Apr 2003 13:51:07 -0300 (BRT) Message-ID: <000f01c30cdd$f665ac70$5c609ec8@gauss> From: "Domingos Jr." To: References: <003201c30cc4$56050a00$e8c797c8@computador> Subject: [obm-l] =?iso-8859-1?Q?Re:_=5Bobm-l=5D_Indu=E7=E3o_Finita?= Date: Sun, 27 Apr 2003 13:56:34 -0300 MIME-Version: 1.0 Content-Type: multipart/alternative; boundary="----=_NextPart_000_000C_01C30CC4.D057E390" X-Priority: 3 X-MSMail-Priority: Normal X-Mailer: Microsoft Outlook Express 6.00.2800.1158 X-MimeOLE: Produced By Microsoft MimeOLE V6.00.2800.1165 Sender: owner-obm-l@sucuri.mat.puc-rio.br Precedence: bulk Reply-To: obm-l@mat.puc-rio.br This is a multi-part message in MIME format. ------=_NextPart_000_000C_01C30CC4.D057E390 Content-Type: text/plain; charset="iso-8859-1" Content-Transfer-Encoding: quoted-printable 1) prove o caso n =3D 3, suponha que (n+1)^n < n^(n+1) para 3 <=3D n < k (n+2)^(n+1) < (n+1)^(n+2) <=3D>=20 [(n+2)-1]^(n+2) > (n+2)^(n+1) <=3D> soma{ i =3D 0..n-2 } [C(n+2,i) * (n+2)^i * (-1)^(n+2-i)] + (n+2)^(n+2) - = C(n+2, n+1)*(n+2)^(n+1) + C(n+2, n)*(n+2)^n - C(n+2, n-1)*(n+2)^(n-1). temos que C(n+2,i) * (n+2)^i < C(n+2,i+1) * (n+2)^(i+1) pois C(n+2,i+1) * (n+2)^(i+1) =3D {[(n+2)(n+2-i)]/(i+1)} * C(n+2,i) * = (n+2)^i e {[(n+2)(n+2-i)]/(i+1)} > 1 temos ent=E3o que soma{ i =3D 0..n-2 } [C(n+2,i) * (n+2)^i * = (-1)^(n+2-i)] > 0, basta verificar que para todo termo negativo o termo = seguinte =E9 positivo e maior em m=F3dulo do que ele. basta agora provar que (n+2)^(n+2) - C(n+2, n+1)*(n+2)^(n+1) + C(n+2, n)*(n+2)^n - C(n+2, = n-1)*(n+2)^(n-1) > (n+2)^(n+1) C(n+2, n+1)*(n+2)^(n+1) =3D (n+2)*(n+2)^(n+1) =3D (n+2)^(n+2) C(n+2, n)*(n+2)^n - C(n+2, n-1)*(n+2)^(n-1) =3D (n+2)(n+1)/2 * (n+2)^n = - (n+2)(n+1)n/6 * (n+2)^(n-1) =3D (n+1)/2 * [(n+2)^(n+1) - [n(n+1)(n+2)^(n-1)]/3] > (n+1)/2 [(n+2)^(n+1) - = [(n+2)^(n+1)]/3] =3D [(n+1)(n+2)^(n+1)]/3 como n+1 > 4, temos a desigualdade desejada... pra ser sincero, essa n=E3o foi uma prova por indu=E7=E3o :-) talvez tenha um jeito mais simples... [ ]'s ----- Original Message -----=20 From: Andr=E9 Lu=EDz=20 To: Lista OBM=20 Sent: Sunday, April 27, 2003 10:53 AM Subject: [obm-l] Indu=E7=E3o Finita Oi amigos, gostaria que voc=EAs me ajudasse nessas tr=EAs quest=F5es = de indu=E7=E3o: 1) Para n >=3D 3 mostre que (n+1)^n < n^(n+1) 2) Para n >=3D 2 mostre que 1 * 3 * 5 * ..... * (2n - 1) < n^n 3) Mostre que =E9 poss=EDvel pagar, sem receber troco, qualquer = quantia inteira de reais, maior do que 7, com notas de 3 reais e 5 = reais. 4) Para n >=3D 3, mostre que 2^n + 1 =E9 um n=FAmero composto se n = n=E3o =E9 uma pot=EAncia de 2. Desde j=E1 muito obrigado. ------=_NextPart_000_000C_01C30CC4.D057E390 Content-Type: text/html; charset="iso-8859-1" Content-Transfer-Encoding: quoted-printable
1)
prove o caso n =3D 3,
suponha que (n+1)^n < n^(n+1) para 3 = <=3D n <=20 k
 
(n+2)^(n+1) = <=20 (n+1)^(n+2) <=3D> =
[(n+2)-1]^(n+2) > (n+2)^(n+1)=20 <=3D>
soma{ i =3D 0..n-2 } = [C(n+2,i) * (n+2)^i *=20 (-1)^(n+2-i)] + (n+2)^(n+2) - C(n+2, n+1)*(n+2)^(n+1) + C(n+2, = n)*(n+2)^n -=20 C(n+2, n-1)*(n+2)^(n-1).
 
temos que C(n+2,i) * (n+2)^i < = C(n+2,i+1) *=20 (n+2)^(i+1)
pois C(n+2,i+1) * (n+2)^(i+1) =3D=20 {[(n+2)(n+2-i)]/(i+1)} * C(n+2,i) * (n+2)^i  e
{[(n+2)(n+2-i)]/(i+1)} > = 1
 
temos ent=E3o que soma{ i = =3D 0..n-2 }=20 [C(n+2,i) * (n+2)^i * (-1)^(n+2-i)] > 0, basta verificar que para = todo termo=20 negativo o termo seguinte =E9 positivo e maior em m=F3dulo do que = ele.
 
basta agora provar que
(n+2)^(n+2) - C(n+2, n+1)*(n+2)^(n+1) + = C(n+2,=20 n)*(n+2)^n  - C(n+2, n-1)*(n+2)^(n-1) > (n+2)^(n+1)
C(n+2, n+1)*(n+2)^(n+1) =3D = (n+2)*(n+2)^(n+1)  =3D=20 (n+2)^(n+2)
 
C(n+2, n)*(n+2)^n  - C(n+2, = n-1)*(n+2)^(n-1) =3D=20 (n+2)(n+1)/2 * (n+2)^n - (n+2)(n+1)n/6 * (n+2)^(n-1) =3D
(n+1)/2 * [(n+2)^(n+1) - = [n(n+1)(n+2)^(n-1)]/3]=20 > (n+1)/2 [(n+2)^(n+1) - [(n+2)^(n+1)]/3] =3D = [(n+1)(n+2)^(n+1)]/3
 
como n+1 > 4, temos a desigualdade=20 desejada...
pra ser sincero, essa n=E3o foi uma = prova por indu=E7=E3o=20 :-)
talvez tenha um jeito mais = simples...
 
[ ]'s
----- Original Message -----
From:=20 Andr=E9=20 Lu=EDz
Sent: Sunday, April 27, 2003 = 10:53=20 AM
Subject: [obm-l] Indu=E7=E3o = Finita

Oi amigos, gostaria que voc=EAs me = ajudasse nessas=20 tr=EAs quest=F5es de indu=E7=E3o:
 
1) Para n >=3D 3 mostre que = (n+1)^n <=20 n^(n+1)
 
2) Para n >=3D 2 mostre que 1 * = 3 * 5 *=20 ..... * (2n - 1) < n^n
 
3) Mostre que =E9 poss=EDvel = pagar, sem receber=20 troco, qualquer quantia inteira de reais, maior do que 7, com = notas de 3=20 reais e 5 reais.
 
4) Para n >=3D 3, mostre que 2^n + = 1 =E9 um n=FAmero=20 composto se n n=E3o =E9 uma pot=EAncia de 2.
 
Desde j=E1 muito=20 obrigado.
------=_NextPart_000_000C_01C30CC4.D057E390-- ========================================================================= Instruções para entrar na lista, sair da lista e usar a lista em http://www.mat.puc-rio.br/~nicolau/olimp/obm-l.html ========================================================================= From owner-obm-l@sucuri.mat.puc-rio.br Sun Apr 27 13:56:03 2003 Return-Path: Received: (from majordom@localhost) by sucuri.mat.puc-rio.br (8.9.3/8.9.3) id NAA28938 for obm-l-MTTP; Sun, 27 Apr 2003 13:52:09 -0300 Received: from traven.uol.com.br (traven.uol.com.br [200.221.29.39]) by sucuri.mat.puc-rio.br (8.9.3/8.9.3) with ESMTP id NAA28933 for ; Sun, 27 Apr 2003 13:52:05 -0300 Received: from n2r0c0.uol.com.br ([200.147.12.160]) by traven.uol.com.br (8.9.1/8.9.1) with ESMTP id NAA09134; Sun, 27 Apr 2003 13:51:34 -0300 (BRT) Message-Id: <5.1.0.14.2.20030427134058.00b272d0@pop3.uol.com.br> X-Sender: cavictor@pop3.uol.com.br X-Mailer: QUALCOMM Windows Eudora Version 5.1 Date: Sun, 27 Apr 2003 13:51:08 -0300 To: obm-l@mat.puc-rio.br, obm-l@mat.puc-rio.br From: Carlos Victor Subject: Re: [obm-l] Mais calculo In-Reply-To: <3EABF2BA.5090705@niski.com> Mime-Version: 1.0 Content-Type: text/plain; charset="iso-8859-1"; format=flowed Content-Transfer-Encoding: 8bit X-MIME-Autoconverted: from quoted-printable to 8bit by sucuri.mat.puc-rio.br id NAA28935 Sender: owner-obm-l@sucuri.mat.puc-rio.br Precedence: bulk Reply-To: obm-l@mat.puc-rio.br Oi Niski, a) -M(x-p)^2 <= (f(x) - f(p))<= M(x-p)^2 , como as funções -M(x-p)^2 e M(x-p)^2 possuem limites iguais a zero , temos pelo teorema do confronto que lim( f(x)- f(p)) =0 para x-> p ; ou seja lim(f(x)) = f(p) . b) Divida a expressão do ítem (a) por (x-p) e use o mesmo teorema , concluindo que o limite é zero . ok ? []´s Carlos Victor At 08:09 27/4/2003 -0700, niski wrote: >Ola pessoal, travei neste daqui..alguem poderia me ajudar?! Obrigado > >Seja f definida em R e suponha que existe M > 0 tal que, para todo x, >|f(x) - f(p)| <= M|x-p|^2 > >a) Mostre que f é continua >b) Calcule, caso exista, lim[x->p] (f(x)-f(p))/(x-p) > >Obrigado. > >niski >-- >[about him:] > It is rare to find learned men who are clean, do not stink and have a > sense of humour. >-Gottfried Whilhem Leibniz > >========================================================================= >Instruções para entrar na lista, sair da lista e usar a lista em >http://www.mat.puc-rio.br/~nicolau/olimp/obm-l.html >========================================================================= ========================================================================= Instruções para entrar na lista, sair da lista e usar a lista em http://www.mat.puc-rio.br/~nicolau/olimp/obm-l.html ========================================================================= From owner-obm-l@sucuri.mat.puc-rio.br Sun Apr 27 13:56:11 2003 Return-Path: Received: (from majordom@localhost) by sucuri.mat.puc-rio.br (8.9.3/8.9.3) id NAA28971 for obm-l-MTTP; Sun, 27 Apr 2003 13:52:24 -0300 Received: from itaqui.terra.com.br (itaqui.terra.com.br [200.176.3.19]) by sucuri.mat.puc-rio.br (8.9.3/8.9.3) with ESMTP id NAA28958 for ; Sun, 27 Apr 2003 13:52:19 -0300 Received: from itaim.terra.com.br (itaim.terra.com.br [200.176.3.76]) by itaqui.terra.com.br (Postfix) with ESMTP id B9B2F3BC1CA for ; Sun, 27 Apr 2003 13:51:48 -0300 (BRT) Received: from riemann.localdomain (RJ173173.user.veloxzone.com.br [200.149.173.173]) (authenticated user fabio.dias.moreira) by itaim.terra.com.br (Postfix) with ESMTP id 4A9392E004A for ; Sun, 27 Apr 2003 13:51:48 -0300 (BRT) Content-Type: text/plain; charset="iso-8859-1" From: =?iso-8859-1?q?F=E1bio=20Dias=20Moreira?= To: obm-l@mat.puc-rio.br Subject: Re: [obm-l] =?iso-8859-1?q?Indu=E7=E3o?= Finita Date: Sun, 27 Apr 2003 13:51:37 -0300 User-Agent: KMail/1.4.3 References: <003201c30cc4$56050a00$e8c797c8@computador> In-Reply-To: <003201c30cc4$56050a00$e8c797c8@computador> MIME-Version: 1.0 Content-Transfer-Encoding: 8bit Message-Id: <200304271351.46131.fabio.dias.moreira@terra.com.br> Sender: owner-obm-l@sucuri.mat.puc-rio.br Precedence: bulk Reply-To: obm-l@mat.puc-rio.br -----BEGIN PGP SIGNED MESSAGE----- Hash: SHA1 On Sunday 27 April 2003 10:53, André Luíz wrote: > [...] > 3) Mostre que é possível pagar, sem receber troco, qualquer quantia inteira > de reais, maior do que 7, com notas de 3 reais e 5 reais. > [...] Lema: Se é possível pagar a quantia n (n >= 8) com b notas de 5 (e algumas notas de 3) então é possível pagar até a quantia n+b com notas de 3 e 5. Prova: Se n = a*3 + b*5, então: n+1 = (a+2)*3 + (b-1)*5 n+2 = (a+4)*3 + (b-2)*5 ... n+b = (a+2*b)*3 + 0*5 Mas: 8 se paga como 3+5 (também se paga até o 9) 10 se paga como 5+5 (também se paga até o 12) 13 se paga como 5+5+3 (também se paga até o 15) 16 se paga como 5+5+3+3 (também se paga até o 18) 19 se paga como 5+5+3+3+3 5*k (k >= 4) se paga como 5+5+5+...+5, onde há k cincos (também se paga até o 5*k + k >= 5*k + 4) Com o último caso é possível cobrir todos os inteiros maiores ou iguais a 20. []s, - -- Fábio "ctg \pi" Dias Moreira -----BEGIN PGP SIGNATURE----- Version: GnuPG v1.0.6 (GNU/Linux) Comment: For info see http://www.gnupg.org iD8DBQE+rAqhalOQFrvzGQoRAofrAKDhkNOVQLZE+fJ7MPAf+4pMF9gjawCffNRW yyrxJVoma7JPiKuaPuEAgqw= =ZPcY -----END PGP SIGNATURE----- ========================================================================= Instruções para entrar na lista, sair da lista e usar a lista em http://www.mat.puc-rio.br/~nicolau/olimp/obm-l.html ========================================================================= From owner-obm-l@sucuri.mat.puc-rio.br Sun Apr 27 13:58:45 2003 Return-Path: Received: (from majordom@localhost) by sucuri.mat.puc-rio.br (8.9.3/8.9.3) id NAA29210 for obm-l-MTTP; Sun, 27 Apr 2003 13:54:52 -0300 Received: from artemis.opendf.com.br (artemis.opengate.com.br [200.181.71.14]) by sucuri.mat.puc-rio.br (8.9.3/8.9.3) with ESMTP id NAA29197 for ; Sun, 27 Apr 2003 13:54:46 -0300 Received: from localhost (localhost [127.0.0.1]) by artemis.opendf.com.br (Postfix) with ESMTP id A64AF2BED7 for ; Sun, 27 Apr 2003 13:55:36 -0300 (BRT) Received: from artemis.opendf.com.br ([127.0.0.1]) by localhost (artemis.opengate.com.br [127.0.0.1:10024]) (amavisd-new) with ESMTP id 10247-06 for ; Sun, 27 Apr 2003 13:55:35 -0300 (BRT) Received: from computer (200-181-088-208.bsace7001.dsl.brasiltelecom.net.br [200.181.88.208]) by artemis.opendf.com.br (Postfix) with ESMTP id 7133B2BED3 for ; Sun, 27 Apr 2003 13:55:35 -0300 (BRT) From: "Artur Costa Steiner" To: Subject: RE: [obm-l] Mais calculo Date: Sun, 27 Apr 2003 13:54:13 -0300 Organization: Steiner Consultoria LTDA Message-ID: <007701c30cdd$a3ac4e30$9865fea9@computer> MIME-Version: 1.0 Content-Type: text/plain; charset="iso-8859-1" X-Priority: 3 (Normal) X-MSMail-Priority: Normal X-Mailer: Microsoft Outlook, Build 10.0.2627 In-Reply-To: <3EABF2BA.5090705@niski.com> X-MimeOLE: Produced By Microsoft MimeOLE V6.00.2800.1165 Importance: Normal X-Virus-Scanned: by amavisd-new Content-Transfer-Encoding: 8bit X-MIME-Autoconverted: from quoted-printable to 8bit by sucuri.mat.puc-rio.br id NAA29202 Sender: owner-obm-l@sucuri.mat.puc-rio.br Precedence: bulk Reply-To: obm-l@mat.puc-rio.br Para todo x =/= p, temos que |f(x) - f(p)|/|(x-p)| <= M |x-p|. Eh imediato que, quando x--> p, o segundo membro tende para zero. Pelo teorema do confronto, segue-se entao que o limite quando x tende a p do primeiro membro e tambem zero. Por definicao, lim x -->p [f(x) - f(p)]/(x-p), quando existe - e no caso, existe - e f'(p). Logo f'(p) = 0, o que responde b. E se b e valido em p, a e automaticamente verificado. Se eu nao me engano, diz -se que uma funcao deste tipo satsfaz a condicao de Lipschitz de segunda ordem em p. Se em vez de 2 tivermos algum n, entao e condicao de ordem n. Artur > >Ola pessoal, travei neste daqui..alguem poderia me ajudar?! Obrigado > >Seja f definida em R e suponha que existe M > 0 tal que, para todo x, >|f(x) - f(p)| <= M|x-p|^2 > >a) Mostre que f é continua >b) Calcule, caso exista, lim[x->p] (f(x)-f(p))/(x-p) > >Obrigado. > >niski ========================================================================= Instruções para entrar na lista, sair da lista e usar a lista em http://www.mat.puc-rio.br/~nicolau/olimp/obm-l.html ========================================================================= From owner-obm-l@sucuri.mat.puc-rio.br Sun Apr 27 15:41:03 2003 Return-Path: Received: (from majordom@localhost) by sucuri.mat.puc-rio.br (8.9.3/8.9.3) id PAA00390 for obm-l-MTTP; Sun, 27 Apr 2003 15:36:07 -0300 Received: from pina.terra.com.br (pina.terra.com.br [200.176.3.31]) by sucuri.mat.puc-rio.br (8.9.3/8.9.3) with ESMTP id PAA00386 for ; Sun, 27 Apr 2003 15:36:04 -0300 Received: from paiol.terra.com.br (paiol.terra.com.br [200.176.3.18]) by pina.terra.com.br (Postfix) with ESMTP id C5C242D75BE for ; Sun, 27 Apr 2003 11:17:22 -0300 (BRT) Received: from canela.terra.com.br (canela.terra.com.br [200.176.3.79]) by paiol.terra.com.br (Postfix) with ESMTP id 0796B87D99 for ; Sun, 27 Apr 2003 11:16:52 -0300 (BRT) Received: from niski.com (unknown [200.148.200.81]) (authenticated user fniski) by canela.terra.com.br (Postfix) with ESMTP id C3F9B22409F for ; Sun, 27 Apr 2003 11:16:50 -0300 (BRT) Message-ID: <3EABE658.9080500@niski.com> Date: Sun, 27 Apr 2003 07:16:56 -0700 From: niski User-Agent: Mozilla/5.0 (Windows; U; Windows NT 5.1; en-US; rv:1.0.2) Gecko/20030208 Netscape/7.02 X-Accept-Language: en-us, en MIME-Version: 1.0 To: obm-l@mat.puc-rio.br Subject: Re: [obm-l] Limites References: <200304270935.h3R9ZTxs012032@trex.centroin.com.br> Content-Type: text/plain; charset=ISO-8859-1; format=flowed Content-Transfer-Encoding: 8bit Sender: owner-obm-l@sucuri.mat.puc-rio.br Precedence: bulk Reply-To: obm-l@mat.puc-rio.br > Alguns autores (o Thomas, por exemplo) fazem uma coisa horrorosa que eh dizer que o limite eh infinito quando o limite eh mais infinito ou menos infinito. Isso pode explicar, embora, a meu ver nao justifique, a resposta do seu livro. No caso estou usando o livro do Maron (MIR) lembro de algum lugar ele ter feito algo semelhante...procurei denovo no livro e nao consigo achar mais para transcrever aqui :( ! > 2) Onduletas eh dose! Mas ondaletas eh comum e aceitavel. Desculpe, me equivoquei, onduletas é em espanhol. De qualquer forma eu prefiro muito mais Wavelets. Acho que deve ser traduzidos os substantivos por exemplo multiplicadores ao inves de multipliers, wavelets significa pequena ondas...ondaletas eu nao encontrei em nenhum dicionario. ========================================================================= Instruções para entrar na lista, sair da lista e usar a lista em http://www.mat.puc-rio.br/~nicolau/olimp/obm-l.html ========================================================================= From owner-obm-l@sucuri.mat.puc-rio.br Sun Apr 27 15:49:28 2003 Return-Path: Received: (from majordom@localhost) by sucuri.mat.puc-rio.br (8.9.3/8.9.3) id PAA00590 for obm-l-MTTP; Sun, 27 Apr 2003 15:45:41 -0300 Received: from traven10.uol.com.br (traven10.uol.com.br [200.221.29.45]) by sucuri.mat.puc-rio.br (8.9.3/8.9.3) with ESMTP id PAA00586 for ; Sun, 27 Apr 2003 15:45:38 -0300 Received: from n2r0c0.uol.com.br ([200.147.136.11]) by traven10.uol.com.br (8.9.1/8.9.1) with ESMTP id PAA27081 for ; Sun, 27 Apr 2003 15:45:05 -0300 (BRT) Message-Id: <5.1.0.14.2.20030427152542.00b27408@pop3.uol.com.br> X-Sender: cavictor@pop3.uol.com.br X-Mailer: QUALCOMM Windows Eudora Version 5.1 Date: Sun, 27 Apr 2003 15:44:46 -0300 To: obm-l@mat.puc-rio.br From: Carlos Victor Subject: [obm-l] Voltando ao Problema proposto na Eureka ! Mime-Version: 1.0 Content-Type: text/plain; charset="iso-8859-1"; format=flowed Content-Transfer-Encoding: 8bit X-MIME-Autoconverted: from quoted-printable to 8bit by sucuri.mat.puc-rio.br id PAA00587 Sender: owner-obm-l@sucuri.mat.puc-rio.br Precedence: bulk Reply-To: obm-l@mat.puc-rio.br Olá pessoal , O que vocês acham da idéia abaixo ? (nâo tenho acompanhado a discussão desta questão por falta de tempo ). f(x+y)+f(x-y)=2f(x)*cos y . É possível mostrar que f(x+pi) = -f(x) e como f(pi+x) +f(pi-x) =2f(pi).cosx ; colocando no lugar de x o valor pi/2 e no lugar de y o valor pi/2 -x , teremos f(pi-x) + f(x) = 2f(pi/2) senx e , consequentemente f(x) = f(pi/2)senx - f(pi)cosx , como sendo as funções que satisfazem ao enunciado . Concordam ? []´s Carlos Victor ========================================================================= Instruções para entrar na lista, sair da lista e usar a lista em http://www.mat.puc-rio.br/~nicolau/olimp/obm-l.html ========================================================================= From owner-obm-l@sucuri.mat.puc-rio.br Sun Apr 27 16:03:00 2003 Return-Path: Received: (from majordom@localhost) by sucuri.mat.puc-rio.br (8.9.3/8.9.3) id PAA00976 for obm-l-MTTP; Sun, 27 Apr 2003 15:59:08 -0300 Received: from smtp012.mail.yahoo.com (smtp012.mail.yahoo.com [216.136.173.32]) by sucuri.mat.puc-rio.br (8.9.3/8.9.3) with SMTP id PAA00972 for ; Sun, 27 Apr 2003 15:59:03 -0300 Received: from dial-up-017-a-tc01.samnet.com.br (HELO itchy) (davidrvp@200.241.109.23 with login) by smtp.mail.vip.sc5.yahoo.com with SMTP; 27 Apr 2003 18:58:29 -0000 Message-ID: <00d301c30cef$05231c50$176df1c8@itchy> From: "David Ricardo" To: References: <003201c30cc4$56050a00$e8c797c8@computador> Subject: [obm-l] =?iso-8859-1?Q?Re:_=5Bobm-l=5D_Indu=E7=E3o_Finita?= Date: Sun, 27 Apr 2003 15:58:35 -0300 MIME-Version: 1.0 Content-Type: text/plain; charset="iso-8859-1" Content-Transfer-Encoding: 8bit X-Priority: 3 X-MSMail-Priority: Normal X-Mailer: Microsoft Outlook Express 6.00.2600.0000 X-MimeOLE: Produced By Microsoft MimeOLE V6.00.2600.0000 Sender: owner-obm-l@sucuri.mat.puc-rio.br Precedence: bulk Reply-To: obm-l@mat.puc-rio.br 3) Mostre que é possível pagar, sem receber troco, qualquer quantia inteira de reais, maior do que 7, com notas de 3 reais e 5 reais. Defina o predicado P(k) como sendo 'é possível pagar uma quantia k com notas de 3 e 5 reais'. Mostre que P(8) é verdadeiro (3 + 5 = 8). Assuma que P(k) é verdadeiro e tente provar que se P(k) é verdadeiro, P(k+1) também é para os inteiros maiores que 7. Para provar isso, veja os seguintes casos: 1) Se existe uma nota de 5 reais fazendo parte da quantia k, troque os 5 reais por duas notas de três reais para obter a quantia k+1. 2) Se não tiver nenhuma nota de 5 reais, deve haver pelo menos 3 notas de três reais na quantia k (para satisfazer a condição k > 7). Troque essas três notas de três reais por duas notas de 5 reais para obter a quantia k+1. Então é possível a partir de uma quantidade k se chegar a uma quantidade k+1 com notas de 3 e 5 reais para k > 7, portanto o predicado é verdadeiro pra todo k inteiro maior que 7. Não sei se é a melhor solução para o problema, mas eu acho que tá certo. []s David ========================================================================= Instruções para entrar na lista, sair da lista e usar a lista em http://www.mat.puc-rio.br/~nicolau/olimp/obm-l.html ========================================================================= From owner-obm-l@sucuri.mat.puc-rio.br Sun Apr 27 16:41:41 2003 Return-Path: Received: (from majordom@localhost) by sucuri.mat.puc-rio.br (8.9.3/8.9.3) id QAA02238 for obm-l-MTTP; Sun, 27 Apr 2003 16:37:39 -0300 Received: from seki.bol.com.br (seki.bol.com.br [200.221.24.26]) by sucuri.mat.puc-rio.br (8.9.3/8.9.3) with ESMTP id QAA02233; Sun, 27 Apr 2003 16:37:26 -0300 Received: from bol.com.br (200.221.24.134) by seki.bol.com.br (5.1.071) id 3E9EA3BD002A1418; Sun, 27 Apr 2003 16:36:56 -0300 Date: Sun, 27 Apr 2003 16:36:56 -0300 Message-Id: Subject: RE: [obm-l] Funcao MIME-Version: 1.0 Content-Type: text/plain;charset="iso-8859-1" From: "osvaldomellospq" To: obm-l@mat.puc-rio.br Cc: obm-l@mat.puc-rio.br X-XaM3-API-Version: 2.4 R3 ( B4 ) X-SenderIP: 200.206.96.2 Content-Transfer-Encoding: 8bit X-MIME-Autoconverted: from quoted-printable to 8bit by sucuri.mat.puc-rio.br id QAA02235 Sender: owner-obm-l@sucuri.mat.puc-rio.br Precedence: bulk Reply-To: obm-l@mat.puc-rio.br > > ---------- Início da mensagem original ----------- > De: owner-obm-l@sucuri.mat.puc-rio.br > Para: > Cc: > Data: Sat, 26 Apr 2003 11:10:47 -0300 > Assunto: RE: [obm-l] Funcao > > >-----Original Message----- > >From: owner-obm-l@sucuri.mat.puc-rio.br > [mailto:owner-obm-l@sucuri.mat.puc- > >rio.br] On Behalf Of osvaldomellospq > >Sent: Friday, April 25, 2003 6:38 PM > >To: obm-l@mat.puc-rio.br > >Subject: [obm-l] Funcao > > > >Gostaria que alguem me ajudasse a resolver esse proble ma que estou > >entusiasmado porem nao consegui demonstrar minhas hipo teses: > > > >"Encontre todos os x pertencentes a R tal que f.f(2x) =fof(2x+1),sabendo > >que f nao e multiplicativa" > [Artur Costa Steiner] > O que vc quer dizer exatamente por f.f (2x)? Nao estah claro > Artur > > ======================================================= ================== > Instruções para entrar na lista, sair da lista e usar a lista em > http://www.mat.puc-rio.br/~nicolau/olimp/obm-l.html > ======================================================= ================== > Desculpem-me pelo erro grosseiro; não é f.f(2x) e sim f(2x).f(2x) __________________________________________________________________________ Seleção de Softwares UOL. 10 softwares escolhidos pelo UOL para você e sua família. http://www.uol.com.br/selecao ========================================================================= Instruções para entrar na lista, sair da lista e usar a lista em http://www.mat.puc-rio.br/~nicolau/olimp/obm-l.html ========================================================================= From owner-obm-l@sucuri.mat.puc-rio.br Sun Apr 27 16:41:41 2003 Return-Path: Received: (from majordom@localhost) by sucuri.mat.puc-rio.br (8.9.3/8.9.3) id QAA02238 for obm-l-MTTP; Sun, 27 Apr 2003 16:37:39 -0300 Received: from seki.bol.com.br (seki.bol.com.br [200.221.24.26]) by sucuri.mat.puc-rio.br (8.9.3/8.9.3) with ESMTP id QAA02233; Sun, 27 Apr 2003 16:37:26 -0300 Received: from bol.com.br (200.221.24.134) by seki.bol.com.br (5.1.071) id 3E9EA3BD002A1418; Sun, 27 Apr 2003 16:36:56 -0300 Date: Sun, 27 Apr 2003 16:36:56 -0300 Message-Id: Subject: RE: [obm-l] Funcao MIME-Version: 1.0 Content-Type: text/plain;charset="iso-8859-1" From: "osvaldomellospq" To: obm-l@mat.puc-rio.br Cc: obm-l@mat.puc-rio.br X-XaM3-API-Version: 2.4 R3 ( B4 ) X-SenderIP: 200.206.96.2 Content-Transfer-Encoding: 8bit X-MIME-Autoconverted: from quoted-printable to 8bit by sucuri.mat.puc-rio.br id QAA02235 Sender: owner-obm-l@sucuri.mat.puc-rio.br Precedence: bulk Reply-To: obm-l@mat.puc-rio.br > > ---------- Início da mensagem original ----------- > De: owner-obm-l@sucuri.mat.puc-rio.br > Para: > Cc: > Data: Sat, 26 Apr 2003 11:10:47 -0300 > Assunto: RE: [obm-l] Funcao > > >-----Original Message----- > >From: owner-obm-l@sucuri.mat.puc-rio.br > [mailto:owner-obm-l@sucuri.mat.puc- > >rio.br] On Behalf Of osvaldomellospq > >Sent: Friday, April 25, 2003 6:38 PM > >To: obm-l@mat.puc-rio.br > >Subject: [obm-l] Funcao > > > >Gostaria que alguem me ajudasse a resolver esse proble ma que estou > >entusiasmado porem nao consegui demonstrar minhas hipo teses: > > > >"Encontre todos os x pertencentes a R tal que f.f(2x) =fof(2x+1),sabendo > >que f nao e multiplicativa" > [Artur Costa Steiner] > O que vc quer dizer exatamente por f.f (2x)? Nao estah claro > Artur > > ======================================================= ================== > Instruções para entrar na lista, sair da lista e usar a lista em > http://www.mat.puc-rio.br/~nicolau/olimp/obm-l.html > ======================================================= ================== > Desculpem-me pelo erro grosseiro; não é f.f(2x) e sim f(2x).f(2x) __________________________________________________________________________ Seleção de Softwares UOL. 10 softwares escolhidos pelo UOL para você e sua família. http://www.uol.com.br/selecao ========================================================================= Instruções para entrar na lista, sair da lista e usar a lista em http://www.mat.puc-rio.br/~nicolau/olimp/obm-l.html ========================================================================= From owner-obm-l@sucuri.mat.puc-rio.br Sun Apr 27 17:11:02 2003 Return-Path: Received: (from majordom@localhost) by sucuri.mat.puc-rio.br (8.9.3/8.9.3) id RAA03032 for obm-l-MTTP; Sun, 27 Apr 2003 17:08:27 -0300 Received: from sporus.bol.com.br (sporus.bol.com.br [200.221.24.23]) by sucuri.mat.puc-rio.br (8.9.3/8.9.3) with ESMTP id RAA03027 for ; Sun, 27 Apr 2003 17:08:23 -0300 Received: from bol.com.br (200.221.24.137) by sporus.bol.com.br (5.1.071) id 3EA97E7200070818 for obm-l@mat.puc-rio.br; Sun, 27 Apr 2003 17:07:52 -0300 Date: Sun, 27 Apr 2003 17:07:52 -0300 Message-Id: Subject: [obm-l] =?iso-8859-1?q?C=E1lculo_em_porvas_militares?= MIME-Version: 1.0 Content-Type: text/plain;charset="iso-8859-1" From: "basketboy_igor" To: obm-l@mat.puc-rio.br X-XaM3-API-Version: 2.4 R3 ( B4 ) X-SenderIP: 200.164.109.200 Content-Transfer-Encoding: 8bit X-MIME-Autoconverted: from quoted-printable to 8bit by sucuri.mat.puc-rio.br id RAA03028 Sender: owner-obm-l@sucuri.mat.puc-rio.br Precedence: bulk Reply-To: obm-l@mat.puc-rio.br Saldações à todos, Estou com algumas dúvidas em quetões de calculo I em provas militeres: i)(EN/95) O lim(x->0) [raiz(x+b)+raiz(x+a)-raiz(b)-raiz (a)]/x é igual a: resp: 1/[2*raiz(b)] + 1/[2*raiz(a)] ii)(EN/95) Se f(x) = e^(2x) + (x+1)*cos(x), então f'(0) é igual a: resp: 3. iii)(EN/97) O valor da integral de raiz[1 + 9*x*(dx)] é: a)(2/27)*(1+9*x)^(3/2) + c iv)(EN/98)A equação do movimento de um progétil que se desloca ao longo do eixo x é x(t)={e^[-(t – Pi/4)]}*sen (t) + cotg²(t), t >=0 . A aceleração do projétil no instante t=0 é: Resp: d) 16 –2*raiz(2) v)(MM/98) Sendo A = Lim(x->0) {2*raiz[x*sen(6x)]}/ {[cossec(6x)]*[1 - cos²(6x)]} e B = Lim (x->log 2 na base 3)[2^(2x+1], [(A²)*B]/2 vale: Resp: b) 6 Igor Correia, #Mathematics __________________________________________________________________________ Seleção de Softwares UOL. 10 softwares escolhidos pelo UOL para você e sua família. http://www.uol.com.br/selecao ========================================================================= Instruções para entrar na lista, sair da lista e usar a lista em http://www.mat.puc-rio.br/~nicolau/olimp/obm-l.html ========================================================================= From owner-obm-l@sucuri.mat.puc-rio.br Sun Apr 27 17:45:06 2003 Return-Path: Received: (from majordom@localhost) by sucuri.mat.puc-rio.br (8.9.3/8.9.3) id RAA03871 for obm-l-MTTP; Sun, 27 Apr 2003 17:41:03 -0300 Received: from seki.bol.com.br (seki.bol.com.br [200.221.24.26]) by sucuri.mat.puc-rio.br (8.9.3/8.9.3) with ESMTP id RAA03867; Sun, 27 Apr 2003 17:41:00 -0300 Received: from bol.com.br (200.221.24.129) by seki.bol.com.br (5.1.071) id 3E9EA3BD002A3AD0; Sun, 27 Apr 2003 17:40:30 -0300 Date: Sun, 27 Apr 2003 17:40:30 -0300 Message-Id: Subject: [obm-l] =?iso-8859-1?q?Re=3A=5Bobm=2Dl=5D_Indu=E7=E3o_Finita?= MIME-Version: 1.0 Content-Type: text/plain;charset="iso-8859-1" From: "osvaldomellospq" To: obm-l@mat.puc-rio.br Cc: obm-l@mat.puc-rio.br X-XaM3-API-Version: 2.4 R3 ( B4 ) X-SenderIP: 200.206.96.2 Content-Transfer-Encoding: 8bit X-MIME-Autoconverted: from quoted-printable to 8bit by sucuri.mat.puc-rio.br id RAA03868 Sender: owner-obm-l@sucuri.mat.puc-rio.br Precedence: bulk Reply-To: obm-l@mat.puc-rio.br > > ---------- Início da mensagem original ----------- > De: owner-obm-l@sucuri.mat.puc-rio.br > Para: "Lista OBM" > Cc: > Data: Sun, 27 Apr 2003 10:53:04 -0300 > Assunto: [obm-l] Indução Finita > Oi amigos, gostaria que vocês me ajudasse nessas três q uestões de indução: > > 1) Para n >= 3 mostre que (n+1)^n < n^(n+1) > > 2) Para n >= 2 mostre que 1 * 3 * 5 * ..... * (2n - 1) < n^n > > 3) Mostre que é possível pagar, sem receber troco, qual quer quantia inteira de reais, maior do que 7, com notas de 3 reais e 5 reais. > > 4) Para n >= 3, mostre que 2^n + 1 é um número composto se n não é uma potência de 2. > > Desde já muito obrigado. 1)Analisando os casos em que n=3 e n=4, temos que a desigualdade se verifica. Suponha (n+1)^n < n^(n+1) Como n^(n+1)= n.n^(n+1) => nossa hipótese se simplifica a [(n+1)/n]^n [(n+1)/n]^(n+1) < (n+1) Portanto temos (n+1)^n < n^(n+1) qualquer que seja n>=3 pertencente a N. (c.q.d.) Osvaldo Mello Sponquiado __________________________________________________________________________ Seleção de Softwares UOL. 10 softwares escolhidos pelo UOL para você e sua família. http://www.uol.com.br/selecao ========================================================================= Instruções para entrar na lista, sair da lista e usar a lista em http://www.mat.puc-rio.br/~nicolau/olimp/obm-l.html ========================================================================= From owner-obm-l@sucuri.mat.puc-rio.br Sun Apr 27 17:45:06 2003 Return-Path: Received: (from majordom@localhost) by sucuri.mat.puc-rio.br (8.9.3/8.9.3) id RAA03871 for obm-l-MTTP; Sun, 27 Apr 2003 17:41:03 -0300 Received: from seki.bol.com.br (seki.bol.com.br [200.221.24.26]) by sucuri.mat.puc-rio.br (8.9.3/8.9.3) with ESMTP id RAA03867; Sun, 27 Apr 2003 17:41:00 -0300 Received: from bol.com.br (200.221.24.129) by seki.bol.com.br (5.1.071) id 3E9EA3BD002A3AD0; Sun, 27 Apr 2003 17:40:30 -0300 Date: Sun, 27 Apr 2003 17:40:30 -0300 Message-Id: Subject: [obm-l] =?iso-8859-1?q?Re=3A=5Bobm=2Dl=5D_Indu=E7=E3o_Finita?= MIME-Version: 1.0 Content-Type: text/plain;charset="iso-8859-1" From: "osvaldomellospq" To: obm-l@mat.puc-rio.br Cc: obm-l@mat.puc-rio.br X-XaM3-API-Version: 2.4 R3 ( B4 ) X-SenderIP: 200.206.96.2 Content-Transfer-Encoding: 8bit X-MIME-Autoconverted: from quoted-printable to 8bit by sucuri.mat.puc-rio.br id RAA03868 Sender: owner-obm-l@sucuri.mat.puc-rio.br Precedence: bulk Reply-To: obm-l@mat.puc-rio.br > > ---------- Início da mensagem original ----------- > De: owner-obm-l@sucuri.mat.puc-rio.br > Para: "Lista OBM" > Cc: > Data: Sun, 27 Apr 2003 10:53:04 -0300 > Assunto: [obm-l] Indução Finita > Oi amigos, gostaria que vocês me ajudasse nessas três q uestões de indução: > > 1) Para n >= 3 mostre que (n+1)^n < n^(n+1) > > 2) Para n >= 2 mostre que 1 * 3 * 5 * ..... * (2n - 1) < n^n > > 3) Mostre que é possível pagar, sem receber troco, qual quer quantia inteira de reais, maior do que 7, com notas de 3 reais e 5 reais. > > 4) Para n >= 3, mostre que 2^n + 1 é um número composto se n não é uma potência de 2. > > Desde já muito obrigado. 1)Analisando os casos em que n=3 e n=4, temos que a desigualdade se verifica. Suponha (n+1)^n < n^(n+1) Como n^(n+1)= n.n^(n+1) => nossa hipótese se simplifica a [(n+1)/n]^n [(n+1)/n]^(n+1) < (n+1) Portanto temos (n+1)^n < n^(n+1) qualquer que seja n>=3 pertencente a N. (c.q.d.) Osvaldo Mello Sponquiado __________________________________________________________________________ Seleção de Softwares UOL. 10 softwares escolhidos pelo UOL para você e sua família. http://www.uol.com.br/selecao ========================================================================= Instruções para entrar na lista, sair da lista e usar a lista em http://www.mat.puc-rio.br/~nicolau/olimp/obm-l.html ========================================================================= From owner-obm-l@sucuri.mat.puc-rio.br Sun Apr 27 18:09:19 2003 Return-Path: Received: (from majordom@localhost) by sucuri.mat.puc-rio.br (8.9.3/8.9.3) id SAA04612 for obm-l-MTTP; Sun, 27 Apr 2003 18:06:44 -0300 Received: from web13002.mail.yahoo.com (web13002.mail.yahoo.com [216.136.174.12]) by sucuri.mat.puc-rio.br (8.9.3/8.9.3) with SMTP id SAA04608 for ; Sun, 27 Apr 2003 18:06:39 -0300 Message-ID: <20030427210607.92514.qmail@web13002.mail.yahoo.com> Received: from [200.148.192.21] by web13002.mail.yahoo.com via HTTP; Sun, 27 Apr 2003 18:06:07 ART Date: Sun, 27 Apr 2003 18:06:07 -0300 (ART) From: =?iso-8859-1?q?Helder=20Suzuki?= Subject: Re: [obm-l] Cavalos de xadrez To: obm-l@mat.puc-rio.br In-Reply-To: <200304271335.51398.fabio.dias.moreira@terra.com.br> MIME-Version: 1.0 Content-Type: text/plain; charset=iso-8859-1 Content-Transfer-Encoding: 8bit Sender: owner-obm-l@sucuri.mat.puc-rio.br Precedence: bulk Reply-To: obm-l@mat.puc-rio.br --- Fábio Dias Moreira escreveu: > -----BEGIN PGP SIGNED MESSAGE----- > Hash: SHA1 > > On Sunday 27 April 2003 09:45, Helder Suzuki wrote: > > Olá! > > > > (1) Quantos cavalos de xadrez podemos colocar em > um > > tabuleiro NxM tal que nenhum seja ameaçado por > outro? > > [...] > > Pinte o tablueiro de preto e branco e coloque um > cavalo em uma casa branca. > Qual a cor das casas que ele ameaça? > > Isso acha uma cota inferior para o resultado. Por > outro lado, em uma > configuração maximal, quadrados vazios são atacados > pelo maior número > possível de cavalos (isso é óbvio?). Logo, como a > configuração gerada pela > idéia acima atende a isso, também é cota superior. > > []s, Ai teriamos sempre N*M/2 cavalos, certo? E em um tabuleiro 2x3 como ficaria? Se eu fizer pintando de preto e branco e colocando cavalos nas casas brancas, por exemplo, eu colocaria apenas 3 cavalos, mas na verdade cabem 4. Já em um tabuleiro 1xM, cabem M cavalos! talvez exista um grupo de casos especiais, vou pensar aqui []'s, Helder Toshiro Suzuki _______________________________________________________________________ Yahoo! Mail O melhor e-mail gratuito da internet: 6MB de espaço, antivírus, acesso POP3, filtro contra spam. http://br.mail.yahoo.com/ ========================================================================= Instruções para entrar na lista, sair da lista e usar a lista em http://www.mat.puc-rio.br/~nicolau/olimp/obm-l.html ========================================================================= From owner-obm-l@sucuri.mat.puc-rio.br Sun Apr 27 18:15:44 2003 Return-Path: Received: (from majordom@localhost) by sucuri.mat.puc-rio.br (8.9.3/8.9.3) id SAA04798 for obm-l-MTTP; Sun, 27 Apr 2003 18:13:10 -0300 Received: from Euler.impa.br (euler.impa.br [147.65.1.3]) by sucuri.mat.puc-rio.br (8.9.3/8.9.3) with ESMTP id SAA04793 for ; Sun, 27 Apr 2003 18:13:07 -0300 Received: from Gauss.impa.br (Gauss [147.65.4.1]) by Euler.impa.br (8.11.6p2/8.11.6) with ESMTP id h3RLCb001825 for ; Sun, 27 Apr 2003 18:12:37 -0300 (EST) From: Carlos Gustavo Tamm de Araujo Moreira Received: by Gauss.impa.br (8.11.6p2) id h3RLCL507340; Sun, 27 Apr 2003 18:12:21 -0300 (EST) Message-Id: <200304272112.h3RLCL507340@Gauss.impa.br> Subject: Re: [obm-l] Pontos pintados To: obm-l@mat.puc-rio.br Date: Sun, 27 Apr 2003 18:12:20 -0300 (EST) In-Reply-To: from "Claudio Buffara" at Apr 27, 3 10:28:29 am X-Mailer: ELM [version 2.4 PL25] MIME-Version: 1.0 Content-Type: text/plain; charset=US-ASCII Content-Transfer-Encoding: 7bit Sender: owner-obm-l@sucuri.mat.puc-rio.br Precedence: bulk Reply-To: obm-l@mat.puc-rio.br Oi, Claudio, > >on 27.04.03 02:06, Carlos Gustavo Tamm de Araujo Moreira at gugu@impa.br >wrote: > >> Caro Claudio, >> Suponha que em todo segmento unitario as extremidades tem cores >> distintas.Se X e Y estao a distancia raiz(3), e' possivel achar A e B no >> plano tais que os triangulos XAB e ABY sejam equilateros de lado 1. Assim, >> A e B tem cores distintas e diferentes da cor de X, donde a cor de Y deve >> ser igual a cor de X. Considere agora um triangulo XYZ com lados >> XY=XZ=raiz(3) e YZ=1. Temos que as cores de Y e de Z devem ser iguais a cor >> de X mas a cor de Y deve ser distinta da cor de Z, absurdo. >> Eu sei fazer o seu problema da equacao diofantina usando um pouco de >> aritmetica em Z[i]. Voce tem uma solucao que nao usa isso ? >> Abracos, >> Gugu >> >> >Oi, Gugu: > >Entao, A e B pertencem a mediatriz do segmento XY e distam, cada um, de 1/2 >do ponto medio de XY. > >Z pertence a circunferencia de centro X e raio raiz(3). > >Eu nao entendi por que as cores de Z e X devem ser iguais. Temos ZX=raiz(3) e eu tinha mostrado que quaisquer dois pontos a distancia raiz(3) devem ter a mesma cor. > >*** > >Na equacao diofantina eu estava procurando uma solucao por consideracoes de >congruencia, mas nao consegui achar. Vou tentar usar aritmetica Z[i] e ver >no que dah. Alias, tai um bom topico pra outro artigo da Eureka: o uso de >Z[i], Z[raiz(-2)], etc. na solucao de equacoes diofantinas. Este tipo de >equacao nao foi abordado no artigo do Antonio Caminha na Eureka 7. > > >Um abraco, >Claudio. >>> >>> Caros colegas da lista: >>> >>> Outro probleminha que estah me dando trabalho: >>> >>> Cada ponto do plano eh pintado de uma cor, dentre tres cores possiveis. >>> Prove que existe um segmento unitario cujas extremidades tem a mesma cor. >>> >>> ****** >>> >>> Uma equacao diofantina bonitinha: >>> >>> Prove que x^2 + (x+1)^2 = y^3 nao tem solucao em inteiros positivos. >>> >>> ****** >>> >>> E aqui vai a dica pro problema da sequencia de 100 numeros reais e das >>> subsequencias de 8 e 9 termos com mesma media: suponha inicialmente que os >>> termos da sequencia sao racionais. Em seguida, use o fato de que R eh um >>> espaco vetorial sobre Q. >>> >>> Seria otimo se alguem descobrisse uma solucao que nao usasse a dica. >>> >>> Um abraco, >>> Claudio. >>> Abracos, Gugu ========================================================================= Instruções para entrar na lista, sair da lista e usar a lista em http://www.mat.puc-rio.br/~nicolau/olimp/obm-l.html ========================================================================= From owner-obm-l@sucuri.mat.puc-rio.br Sun Apr 27 19:11:12 2003 Return-Path: Received: (from majordom@localhost) by sucuri.mat.puc-rio.br (8.9.3/8.9.3) id TAA06580 for obm-l-MTTP; Sun, 27 Apr 2003 19:08:16 -0300 Received: from mx.pop.com.br (relay1.pop.com.br [200.175.8.37]) by sucuri.mat.puc-rio.br (8.9.3/8.9.3) with ESMTP id TAA06576 for ; Sun, 27 Apr 2003 19:08:12 -0300 Received: from smtp.pop.com.br (smtp1.pop.com.br [200.175.8.30]) by mx.pop.com.br (Postfix) with SMTP id 7141475B81 for ; Sun, 27 Apr 2003 19:07:42 -0300 (BRT) Received: (qmail 14775 invoked by uid 0); 27 Apr 2003 22:07:39 -0000 Received: from 200-175-14-202.dial.pop.com.br (HELO MJEPYON) (200.175.14.202) by smtp1.pop.com.br with SMTP; 27 Apr 2003 22:07:39 -0000 Message-ID: <004a01c30d09$63e4ed80$ca0eafc8@MJEPYON> From: "Marcelo" To: Subject: [obm-l] calculo c Date: Sun, 27 Apr 2003 19:07:17 -0300 MIME-Version: 1.0 Content-Type: multipart/alternative; boundary="----=_NextPart_000_0047_01C30CF0.38503210" X-Priority: 3 X-MSMail-Priority: Normal X-Mailer: Microsoft Outlook Express 6.00.2720.3000 X-MimeOLE: Produced By Microsoft MimeOLE V6.00.2600.0000 Sender: owner-obm-l@sucuri.mat.puc-rio.br Precedence: bulk Reply-To: obm-l@mat.puc-rio.br This is a multi-part message in MIME format. ------=_NextPart_000_0047_01C30CF0.38503210 Content-Type: text/plain; charset="iso-8859-1" Content-Transfer-Encoding: quoted-printable ol=E1 pessoal... n=E3o estou conseguindo provar essas quest=F5es: a) x=B3-y=B3 =3D (x-y)(x=B2 + xy + y=B2) na verdade essa eu provei, mas n=E3o sei se est=E1 certo... x=B3-y=B3 =3D x=B3 + x=B2y - xy=B2 - x=B2y - xy=B2 - y=B3 x=B3-y=B3 =3D x=B3-y=B3 b) x^n - y^n =3D (x-y)(x^n-1 + x^n-2*y + ... + xy^n-2 + y^n-1) c) x=B3+y=B3 =3D (x+y)(x=B2 - xy + y=B2) --- Outgoing mail is certified Virus Free. Checked by AVG anti-virus system (http://www.grisoft.com). Version: 6.0.474 / Virus Database: 272 - Release Date: 18/4/2003 ------=_NextPart_000_0047_01C30CF0.38503210 Content-Type: text/html; charset="iso-8859-1" Content-Transfer-Encoding: quoted-printable
ol=E1 pessoal...
 
n=E3o estou conseguindo provar = essas=20 quest=F5es:
 
a) x=B3-y=B3 =3D (x-y)(x=B2 + = xy + y=B2)
 
na verdade essa eu provei, mas = n=E3o sei se=20 est=E1 certo...
 
x=B3-y=B3 =3D x=B3 + x=B2y - = xy=B2 - x=B2y - xy=B2 -=20 y=B3
x=B3-y=B3 =3D = x=B3-y=B3
 
b) x^n - y^n =3D (x-y)(x^n-1 + = x^n-2*y + ...=20 + xy^n-2 + y^n-1)
 
c) x=B3+y=B3 =3D (x+y)(x=B2 - = xy + y=B2)
 

---
Outgoing mail is = certified Virus=20 Free.
Checked by AVG anti-virus system (http://www.grisoft.com).
Version: = 6.0.474 /=20 Virus Database: 272 - Release Date: 18/4/2003
------=_NextPart_000_0047_01C30CF0.38503210-- ========================================================================= Instruções para entrar na lista, sair da lista e usar a lista em http://www.mat.puc-rio.br/~nicolau/olimp/obm-l.html ========================================================================= From owner-obm-l@sucuri.mat.puc-rio.br Sun Apr 27 19:18:19 2003 Return-Path: Received: (from majordom@localhost) by sucuri.mat.puc-rio.br (8.9.3/8.9.3) id TAA06736 for obm-l-MTTP; Sun, 27 Apr 2003 19:15:45 -0300 Received: from ivoti.terra.com.br (ivoti.terra.com.br [200.176.3.20]) by sucuri.mat.puc-rio.br (8.9.3/8.9.3) with ESMTP id TAA06732 for ; Sun, 27 Apr 2003 19:15:42 -0300 Received: from altamira.terra.com.br (altamira.terra.com.br [200.176.3.40]) by ivoti.terra.com.br (Postfix) with ESMTP id 0B084408035 for ; Sun, 27 Apr 2003 19:15:12 -0300 (BRT) Received: from [200.177.176.104] (dl-nas1-sao-C8B1B068.p001.terra.com.br [200.177.176.104]) by altamira.terra.com.br (Postfix) with ESMTP id E0C363DC0A8 for ; Sun, 27 Apr 2003 19:15:10 -0300 (BRT) User-Agent: Microsoft-Outlook-Express-Macintosh-Edition/5.02.2022 Date: Sun, 27 Apr 2003 19:16:37 -0300 Subject: Re: [obm-l] Indu=?ISO-8859-1?B?5+M=?=o Finita From: Claudio Buffara To: Message-ID: In-Reply-To: <003201c30cc4$56050a00$e8c797c8@computador> Mime-version: 1.0 Content-type: multipart/alternative; boundary="MS_Mac_OE_3134315797_56193_MIME_Part" Sender: owner-obm-l@sucuri.mat.puc-rio.br Precedence: bulk Reply-To: obm-l@mat.puc-rio.br > This message is in MIME format. Since your mail reader does not understand this format, some or all of this message may not be legible. --MS_Mac_OE_3134315797_56193_MIME_Part Content-type: text/plain; charset="ISO-8859-1" Content-transfer-encoding: quoted-printable on 27.04.03 10:53, Andr=E9 Lu=EDz at conectado2002@ieg.com.br wrote: Oi amigos, gostaria que voc=EAs me ajudasse nessas tr=EAs quest=F5es de indu=E7=E3o: =20 1) Para n >=3D 3 mostre que (n+1)^n < n^(n+1) =20 2) Para n >=3D 2 mostre que 1 * 3 * 5 * ..... * (2n - 1) < n^n =20 3) Mostre que =E9 poss=EDvel pagar, sem receber troco, qualquer quantia inteira de reais, maior do que 7, com notas de 3 reais e 5 reais. =20 4) Para n >=3D 3, mostre que 2^n + 1 =E9 um n=FAmero composto se n n=E3o =E9 uma pot=EAncia de 2. =20 Desde j=E1 muito obrigado. O no. 4 pode ser feito sem inducao. Suponha que n nao seja uma potencia de 2. Seja p o maior divisor impar de n. Como n >=3D 3, temos que p >=3D 3, e podemos escrever n =3D (2^k)*p, onde k eh um inteiro nao negativo. Seja a =3D 2^k 2^n + 1 =3D 2^((2^k)*p) + 1 =3D a^p + 1 =3D =3D (a + 1)*(a^(p-1) + a^(p-2) + ... + a + 1). Como a >=3D 1 e p >=3D 3, ambos os fatores sao >=3D 2. Logo, 2^n + 1 eh composto. Um abraco, Claudio. --MS_Mac_OE_3134315797_56193_MIME_Part Content-type: text/html; charset="ISO-8859-1" Content-transfer-encoding: quoted-printable Re: [obm-l] Indu=E7=E3o Finita on 27.04.03 10:53, Andr=E9 Lu=EDz at conectado2002@ieg.com.br wrote:

Oi amigos, gostaria que voc=EAs= me ajudasse nessas tr=EAs quest=F5es de indu=E7=E3o:

1) Para n >=3D 3 mostre que (n+1)^n <= n^(n+1)

2) Para n >=3D 2 mostre que 1 * 3 * 5 * = ..... * (2n - 1) < n^n

3) Mostre que =E9 poss=EDvel pagar, sem receb= er troco, qualquer quantia inteira de reais, maior do que 7, com notas de 3 = reais e 5 reais.

4) Para n >=3D 3, mostre que 2^n + 1 =E9 u= m n=FAmero composto se n n=E3o =E9 uma pot=EAncia de 2.

Desde j=E1 muito obrigado.


O no. 4 pode ser feito sem inducao.

Suponha que n nao seja uma potencia de 2.
Seja p o maior divisor impar de n.
Como n >=3D 3, temos que p >=3D 3, e podemos escrever n =3D (2^k)*p, onde k= eh um inteiro nao negativo.

Seja a =3D 2^k
2^n + 1 =3D 2^((2^k)*p) + 1 =3D a^p + 1 =3D
=3D (a + 1)*(a^(p-1) + a^(p-2) + ... + a + 1).

Como a >=3D 1 e p >=3D 3, ambos os fatores sao >=3D 2. Logo, 2^n + 1 eh = composto.

Um abraco,
Claudio. --MS_Mac_OE_3134315797_56193_MIME_Part-- ========================================================================= Instruções para entrar na lista, sair da lista e usar a lista em http://www.mat.puc-rio.br/~nicolau/olimp/obm-l.html ========================================================================= From owner-obm-l@sucuri.mat.puc-rio.br Sun Apr 27 19:34:11 2003 Return-Path: Received: (from majordom@localhost) by sucuri.mat.puc-rio.br (8.9.3/8.9.3) id TAA07331 for obm-l-MTTP; Sun, 27 Apr 2003 19:31:24 -0300 Received: from www.zipmail.com.br (smtp.zipmail.com.br [200.221.11.147]) by sucuri.mat.puc-rio.br (8.9.3/8.9.3) with ESMTP id TAA07324 for ; Sun, 27 Apr 2003 19:31:12 -0300 From: bmat@zipmail.com.br Received: from [200.220.16.39] by www.zipmail.com.br with HTTP; Sun, 27 Apr 2003 19:27:50 -0300 Message-ID: <3EA9933F000028D2@www.zipmail.com.br> Date: Sun, 27 Apr 2003 19:27:50 -0300 Subject: [obm-l] =?iso-8859-1?Q?Re=3A=20=5Bobm=2Dl=5D=20=20C=E1lculo=20em=20provas=20militares?= To: obm-l@mat.puc-rio.br MIME-Version: 1.0 Content-Type: text/plain; charset="iso-8859-1" Content-Transfer-Encoding: 8bit X-MIME-Autoconverted: from quoted-printable to 8bit by sucuri.mat.puc-rio.br id TAA07328 Sender: owner-obm-l@sucuri.mat.puc-rio.br Precedence: bulk Reply-To: obm-l@mat.puc-rio.br Bom, acho que é isso... -- Mensagem original -- >Saldações à todos, >Estou com algumas dúvidas em quetões de calculo I em >provas militeres: > >i)(EN/95) O lim(x->0) [raiz(x+b)+raiz(x+a)-raiz(b)-raiz >(a)]/x é igual a: >resp: 1/[2*raiz(b)] + 1/[2*raiz(a)] separe primeiro os limites: [ raiz(x+b) - raiz(b) ] / x e [ raiz(x+a) - raiz(a) ] / x e multiplicando em cima e embaixo do primeiro por [ raiz(x+b) + raiz(b) ] / x ( o segundo é igualzinho ) temos [ raiz(x+b) - raiz(b) ] [ raiz(x+b) + raiz(b) ] /( x * [ raiz(x+b) + raiz(b) ] ) = ( x + b - b ) / ( x * [ raiz(x+b) + raiz(b) ] ) = 1/[ raiz(x+b) + raiz(b) ] e fazendo x -> 0 obtemos 1/2raiz(b) somando agora os dois limites, temos 1/2raiz(b) + 1/2raiz(a) > >ii)(EN/95) Se f(x) = e^(2x) + (x+1)*cos(x), então f'(0) >é igual a: >resp: 3. Bom, espero que vc conheça as regras de derivação: ( NOTAÇÃO : derivada de f = f' ) I) derivada da soma = soma das derivadas (f+g)'= f' + g' II)derivada do produto = "coisa feia" (fg)'= f'g + fg' III)derivada da função composta = produto das derivadas (fog)' = (f'og)g' com isso em mãos, vamos lá: f'(x) = (2x)'*e^(2x) + (x+1)'*cos(x) + (x+1)*(cos(x))' = 2*e^(2x) + 1*cos(x) + (1+x)*(-sen(x)) f'(0) = 2*1 + 1 + 2*0 = 3 >iii)(EN/97) O valor da integral de raiz[1 + 9*x*(dx)] é: >a)(2/27)*(1+9*x)^(3/2) + c Bom, eu acho que deve ser integral de raiz[1+9x]dx, pois integral de (dx)^(1/2) ainda não faz sentido pra mim. NEste caso: faça v = 1+9x => dv = 9dx => dx = dv/9 e a integral fica integral de v^(1/2)dv/9 = 1/9 * 2/3 * v^(3/2) + c. Lembrando que v = 1+9x 2/27*(1+9x)^(3/2) + c >iv)(EN/98)A equação do movimento de um progétil que se >desloca ao longo do eixo x é x(t)={e^[-(t ? Pi/4)]}*sen >(t) + cotg²(t), t >=0 . A aceleração do projétil no >instante t=0 é: >Resp: d) 16 ?2*raiz(2) Olha, eu não consegui ler o teu e-mail nesta parte, pois aparece uma '?' em {e^[-(t ? Pi/4)]} e eu não sei sair dessa... mas derive duas vezes, com muita coragem e calcule no ponto 0. >v)(MM/98) Sendo A = Lim(x->0) {2*raiz[x*sen(6x)]}/ >{[cossec(6x)]*[1 - cos²(6x)]} e B = Lim (x->log 2 na >base 3)[2^(2x+1], [(A²)*B]/2 vale: >Resp: b) 6 primeiro, simplifique o enunciado, usando relações trigonométricas básicas, como 1-cos^2 = sen^2 e cossec = 1/sen. Aí vc fica com: lim(x->0)2*raiz(x/sen(6x)). Para aplicar o limite fundamental do seno, temos que corrigir o numerador e assim temos lim(x->0)2*raiz(6x/sen(6x) *1/6) = 2raiz(1/6) = raiz(2/3) = A => A^2 = 2/3 no segundo, eu acho que seja lim x->log 3 na base 2... e daí B = 2^(2log_2(3) + 1) por continuidade, o que dá 2^log_2(18) = 18 A^2 * B/2 = 2/3 * 18/2 = 6 >Igor Correia, >#Mathematics Até mais, Bernardo ------------------------------------------ Use o melhor sistema de busca da Internet Radar UOL - http://www.radaruol.com.br ========================================================================= Instruções para entrar na lista, sair da lista e usar a lista em http://www.mat.puc-rio.br/~nicolau/olimp/obm-l.html ========================================================================= From owner-obm-l@sucuri.mat.puc-rio.br Sun Apr 27 20:05:52 2003 Return-Path: Received: (from majordom@localhost) by sucuri.mat.puc-rio.br (8.9.3/8.9.3) id UAA08462 for obm-l-MTTP; Sun, 27 Apr 2003 20:03:10 -0300 Received: from web14311.mail.yahoo.com (web14311.mail.yahoo.com [216.136.224.61]) by sucuri.mat.puc-rio.br (8.9.3/8.9.3) with SMTP id UAA08457 for ; Sun, 27 Apr 2003 20:03:05 -0300 Message-ID: <20030427230234.66911.qmail@web14311.mail.yahoo.com> Received: from [200.226.209.29] by web14311.mail.yahoo.com via HTTP; Sun, 27 Apr 2003 20:02:34 ART Date: Sun, 27 Apr 2003 20:02:34 -0300 (ART) From: =?iso-8859-1?q?Rafael?= Subject: [obm-l] complexos To: OBM MIME-Version: 1.0 Content-Type: text/plain; charset=iso-8859-1 Content-Transfer-Encoding: 8bit Sender: owner-obm-l@sucuri.mat.puc-rio.br Precedence: bulk Reply-To: obm-l@mat.puc-rio.br Olá Pessoal! Não sei como resolver essa questão de complexos: Representando as raízes da equação: x^5 = -1 + i.raiz(3) no plano complexo, temos dois afixos distintos no: a) eixo real b) eixo imaginário c) 2° quadrante d) 3° quadrante e) 4° quadrante Na verdade eu queria saber mais sobre representar as raízes reais de uma equação com números complexos como vértices de um polígono regular também, não sei se é esse o caso... Abraços, Rafael. _______________________________________________________________________ Yahoo! Mail O melhor e-mail gratuito da internet: 6MB de espaço, antivírus, acesso POP3, filtro contra spam. http://br.mail.yahoo.com/ ========================================================================= Instruções para entrar na lista, sair da lista e usar a lista em http://www.mat.puc-rio.br/~nicolau/olimp/obm-l.html ========================================================================= From owner-obm-l@sucuri.mat.puc-rio.br Sun Apr 27 20:07:16 2003 Return-Path: Received: (from majordom@localhost) by sucuri.mat.puc-rio.br (8.9.3/8.9.3) id UAA08520 for obm-l-MTTP; Sun, 27 Apr 2003 20:04:44 -0300 Received: from itaqui.terra.com.br (itaqui.terra.com.br [200.176.3.19]) by sucuri.mat.puc-rio.br (8.9.3/8.9.3) with ESMTP id UAA08516 for ; Sun, 27 Apr 2003 20:04:40 -0300 Received: from botucatu.terra.com.br (botucatu.terra.com.br [200.176.3.78]) by itaqui.terra.com.br (Postfix) with ESMTP id 9DCF63BC64E for ; Sun, 27 Apr 2003 20:04:06 -0300 (BRT) Received: from [200.177.180.106] (dl-nas3-sao-C8B1B46A.p001.terra.com.br [200.177.180.106]) by botucatu.terra.com.br (Postfix) with ESMTP id 53F6529C083 for ; Sun, 27 Apr 2003 20:04:05 -0300 (BRT) User-Agent: Microsoft-Outlook-Express-Macintosh-Edition/5.02.2022 Date: Sun, 27 Apr 2003 20:06:12 -0300 Subject: FW: [obm-l] Indu=?ISO-8859-1?B?5+M=?=o Finita From: Claudio Buffara To: Lista OBM Message-ID: In-Reply-To: Mime-version: 1.0 Content-type: multipart/alternative; boundary="MS_Mac_OE_3134318773_57420_MIME_Part" Sender: owner-obm-l@sucuri.mat.puc-rio.br Precedence: bulk Reply-To: obm-l@mat.puc-rio.br > This message is in MIME format. Since your mail reader does not understand this format, some or all of this message may not be legible. --MS_Mac_OE_3134318773_57420_MIME_Part Content-type: text/plain; charset="ISO-8859-1" Content-transfer-encoding: quoted-printable Correcao: A fatoracao e 2^n + 1 abaixo deveria ser: =3D (a + 1)*(a^(p-1) - a^(p-2) + ... - a + 1). Claudio. ---------- From: Claudio Buffara Reply-To: obm-l@mat.puc-rio.br Date: Sun, 27 Apr 2003 19:16:37 -0300 To: Subject: Re: [obm-l] Indu=E7=E3o Finita on 27.04.03 10:53, Andr=E9 Lu=EDz at conectado2002@ieg.com.br wrote: Oi amigos, gostaria que voc=EAs me ajudasse nessas tr=EAs quest=F5es de indu=E7=E3o: =20 1) Para n >=3D 3 mostre que (n+1)^n < n^(n+1) =20 2) Para n >=3D 2 mostre que 1 * 3 * 5 * ..... * (2n - 1) < n^n =20 3) Mostre que =E9 poss=EDvel pagar, sem receber troco, qualquer quantia inteira de reais, maior do que 7, com notas de 3 reais e 5 reais. =20 4) Para n >=3D 3, mostre que 2^n + 1 =E9 um n=FAmero composto se n n=E3o =E9 uma pot=EAncia de 2. =20 Desde j=E1 muito obrigado. O no. 4 pode ser feito sem inducao. Suponha que n nao seja uma potencia de 2. Seja p o maior divisor impar de n. Como n >=3D 3, temos que p >=3D 3, e podemos escrever n =3D (2^k)*p, onde k eh um inteiro nao negativo. Seja a =3D 2^k 2^n + 1 =3D 2^((2^k)*p) + 1 =3D a^p + 1 =3D =3D (a + 1)*(a^(p-1) + a^(p-2) + ... + a + 1). Como a >=3D 1 e p >=3D 3, ambos os fatores sao >=3D 2. Logo, 2^n + 1 eh composto. Um abraco, Claudio.=20 --MS_Mac_OE_3134318773_57420_MIME_Part Content-type: text/html; charset="ISO-8859-1" Content-transfer-encoding: quoted-printable FW: [obm-l] Indu=E7=E3o Finita Correcao:

A fatoracao e 2^n + 1 abaixo deveria ser:

=3D (a + 1)*(a^(p-1) - a^(p-2) + ... - a + 1).

Claudio.
----------
From: Claudio Buffara <claudio.buffara@terra.com.br>
Reply-To: obm-l@mat.puc-rio.br
Date: Sun, 27 Apr 2003 19:16:37 -0300
To: <obm-l@mat.puc-rio.br>
Subject: Re: [obm-l] Indu=E7=E3o Finita

on 27.04.03 10:53, Andr=E9 Lu=EDz at conectado2002@ieg.com.br wrote:

Oi amigos, gostaria que voc=EAs= me ajudasse nessas tr=EAs quest=F5es de indu=E7=E3o:

1) Para n >=3D 3 mostre que (n+1)^n <= n^(n+1)

2) Para n >=3D 2 mostre que 1 * 3 * 5 * = ..... * (2n - 1) < n^n

3) Mostre que =E9 poss=EDvel pagar, sem receb= er troco, qualquer quantia inteira de reais, maior do que 7, com notas de 3 = reais e 5 reais.

4) Para n >=3D 3, mostre que 2^n + 1 =E9 u= m n=FAmero composto se n n=E3o =E9 uma pot=EAncia de 2.

Desde j=E1 muito obrigado.


O no. 4 pode ser feito sem inducao.

Suponha que n nao seja uma potencia de 2.
Seja p o maior divisor impar de n.
Como n >=3D 3, temos que p >=3D 3, e podemos escrever n =3D (2^k)*p, onde k= eh um inteiro nao negativo.

Seja a =3D 2^k
2^n + 1 =3D 2^((2^k)*p) + 1 =3D a^p + 1 =3D
=3D (a + 1)*(a^(p-1) + a^(p-2) + ... + a + 1).

Como a >=3D 1 e p >=3D 3, ambos os fatores sao >=3D 2. Logo, 2^n + 1 eh = composto.

Um abraco,
Claudio.
--MS_Mac_OE_3134318773_57420_MIME_Part-- ========================================================================= Instruções para entrar na lista, sair da lista e usar a lista em http://www.mat.puc-rio.br/~nicolau/olimp/obm-l.html ========================================================================= From owner-obm-l@sucuri.mat.puc-rio.br Sun Apr 27 20:55:32 2003 Return-Path: Received: (from majordom@localhost) by sucuri.mat.puc-rio.br (8.9.3/8.9.3) id UAA10164 for obm-l-MTTP; Sun, 27 Apr 2003 20:52:55 -0300 Received: from artemis.opendf.com.br (artemis.opengate.com.br [200.181.71.14]) by sucuri.mat.puc-rio.br (8.9.3/8.9.3) with ESMTP id UAA10160 for ; Sun, 27 Apr 2003 20:52:51 -0300 Received: from localhost (localhost [127.0.0.1]) by artemis.opendf.com.br (Postfix) with ESMTP id 1F3A42BED7 for ; Sun, 27 Apr 2003 20:53:46 -0300 (BRT) Received: from artemis.opendf.com.br ([127.0.0.1]) by localhost (artemis.opengate.com.br [127.0.0.1:10024]) (amavisd-new) with ESMTP id 15838-02 for ; Sun, 27 Apr 2003 20:53:45 -0300 (BRT) Received: from computer (200-140-002-143.bsace7001.dsl.brasiltelecom.net.br [200.140.2.143]) by artemis.opendf.com.br (Postfix) with ESMTP id D825A2BED3 for ; Sun, 27 Apr 2003 20:53:44 -0300 (BRT) From: "Artur Costa Steiner" To: Subject: [obm-l] =?iso-8859-1?Q?RE:_=5Bobm-l=5D_Indu=E7=E3o_Finita?= Date: Sun, 27 Apr 2003 20:52:19 -0300 Organization: Steiner Consultoria LTDA Message-ID: <001501c30d18$0c4a2db0$9865fea9@computer> MIME-Version: 1.0 Content-Type: text/plain; charset="iso-8859-1" Content-Transfer-Encoding: 7bit X-Priority: 3 (Normal) X-MSMail-Priority: Normal X-Mailer: Microsoft Outlook, Build 10.0.2627 X-MimeOLE: Produced By Microsoft MimeOLE V6.00.2800.1165 In-Reply-To: Importance: Normal X-Virus-Scanned: by amavisd-new Sender: owner-obm-l@sucuri.mat.puc-rio.br Precedence: bulk Reply-To: obm-l@mat.puc-rio.br O no. 4 pode ser feito sem inducao. Suponha que n nao seja uma potencia de 2. Seja p o maior divisor impar de n. Como n >= 3, temos que p >= 3, e podemos escrever n = (2^k)*p, onde k eh um inteiro nao negativo. [Artur Costa Steiner] Nao hah um engano aqui? Suponhamos que n =21. 21 nao eh potencia de 2 e seu maior divisor impar eh 7. Entretanto, nao existe qualquer inteiro nao negativo k para o qual 21 = (2^k)*7. Um abraco Artur Seja a = 2^k 2^n + 1 = 2^((2^k)*p) + 1 = a^p + 1 = = (a + 1)*(a^(p-1) + a^(p-2) + ... + a + 1). Como a >= 1 e p >= 3, ambos os fatores sao >= 2. Logo, 2^n + 1 eh composto. Um abraco, Claudio. ========================================================================= Instruções para entrar na lista, sair da lista e usar a lista em http://www.mat.puc-rio.br/~nicolau/olimp/obm-l.html ========================================================================= From owner-obm-l@sucuri.mat.puc-rio.br Sun Apr 27 21:31:11 2003 Return-Path: Received: (from majordom@localhost) by sucuri.mat.puc-rio.br (8.9.3/8.9.3) id VAA10980 for obm-l-MTTP; Sun, 27 Apr 2003 21:28:27 -0300 Received: from web80505.mail.yahoo.com (web80505.mail.yahoo.com [66.218.79.75]) by sucuri.mat.puc-rio.br (8.9.3/8.9.3) with SMTP id VAA10976 for ; Sun, 27 Apr 2003 21:28:23 -0300 Message-ID: <20030428002751.91290.qmail@web80505.mail.yahoo.com> Received: from [200.146.0.250] by web80505.mail.yahoo.com via HTTP; Sun, 27 Apr 2003 21:27:51 ART Date: Sun, 27 Apr 2003 21:27:51 -0300 (ART) From: "=?iso-8859-1?q?guilherme=20S.?=" Subject: Re: [obm-l] complexos To: obm-l@mat.puc-rio.br In-Reply-To: <20030427230234.66911.qmail@web14311.mail.yahoo.com> MIME-Version: 1.0 Content-Type: text/plain; charset=iso-8859-1 Content-Transfer-Encoding: 8bit Sender: owner-obm-l@sucuri.mat.puc-rio.br Precedence: bulk Reply-To: obm-l@mat.puc-rio.br --- Rafael escreveu: > Olá Pessoal! > > Não sei como resolver essa questão de complexos: > Representando as raízes da equação: > x^5 = -1 + i.raiz(3) > > no plano complexo, temos dois afixos distintos no: > a) eixo real > b) eixo imaginário > c) 2° quadrante > d) 3° quadrante > e) 4° quadrante > > Na verdade eu queria saber mais sobre representar as > raízes reais de uma equação com números complexos > como > vértices de um polígono regular também, não sei se é > esse o caso... > > > x^5=-1+i.raiz(3)=2[cos(a)+i sen(a)] , onde a=2pi/3+2kpi logo: x=2^(1/5)*[cos(a/5)+i sen(a/5)] quando : k=0=>a/5=2pi/15 k=1=>a/5=8pi/15 (seg. quad) k=2=>a/5=14pi/15 (seg. quad) k=3=>a/5=4pi/3 k=4=>a/5=26pi/15 ..... alt .c ou seja as 5 raizes desta equação têm argumentos que estão esm uma P.a. de razão 2pi/5, e portanto formam um pentagono regular cujo raio tem dimensão de 2^1/5. ===== _______________________________________________________________________ Yahoo! Mail O melhor e-mail gratuito da internet: 6MB de espaço, antivírus, acesso POP3, filtro contra spam. http://br.mail.yahoo.com/ ========================================================================= Instruções para entrar na lista, sair da lista e usar a lista em http://www.mat.puc-rio.br/~nicolau/olimp/obm-l.html ========================================================================= From owner-obm-l@sucuri.mat.puc-rio.br Sun Apr 27 22:34:41 2003 Return-Path: Received: (from majordom@localhost) by sucuri.mat.puc-rio.br (8.9.3/8.9.3) id WAA12245 for obm-l-MTTP; Sun, 27 Apr 2003 22:31:48 -0300 Received: from smtp.ieg.com.br (sharon.protocoloweb.com.br [200.226.139.12]) by sucuri.mat.puc-rio.br (8.9.3/8.9.3) with ESMTP id WAA12241 for ; Sun, 27 Apr 2003 22:31:44 -0300 From: cnaval@ieg.com.br Received: from ieg.com.br (jimi.protocoloweb.com.br [200.226.139.29]) by smtp.ieg.com.br (8.12.8/8.9.3) with SMTP id h3S1NiWj072949; Sun, 27 Apr 2003 22:23:45 -0300 (BRT) To: "obm-l@mat.puc-rio.br" Date: Mon, 28 Apr 2003 01:30:51 GMT Subject: Re: [obm-l] ajuda com desigualdade X-Mailer: DMailWeb Web to Mail Gateway 2.7v, http://netwinsite.com/top_mail.htm Message-id: <3eac844b.2eb1.0@ieg.com.br> X-User-Info: 200.151.59.104 MIME-Version: 1.0 Content-Type: text/plain; charset="iso-8859-1" Content-Transfer-Encoding: 8bit X-MIME-Autoconverted: from quoted-printable to 8bit by sucuri.mat.puc-rio.br id WAA12242 Sender: owner-obm-l@sucuri.mat.puc-rio.br Precedence: bulk Reply-To: obm-l@mat.puc-rio.br Oi guilherme, como a expressão eh simétrica em relação à a, b e c, podemos supor sem perda generalidade que a =>b=>c e pela desigualdade do rearranjo abc + abc + abc <= a^3 + b^3 + abc (na verdade neste ponto n tenho mta certeza, abc + abc + abc <= a^3 +c^3 + abc peço a ajuda aos mestres da lista :P) abc + abc + abc <= b^3 +c^3 + abc invertando cada desigualdade: 1/(3abc) => 1/(a^3 + b^3 + abc) 1/(3abc) => 1/(a^3 + c^3 + abc) 1/(3abc) => 1/(b^3 + c^3 + abc) somando as três: 1/abc => 1/(a^3 + b^3 + abc) + 1/(a^3 + b^3 + abc) + 1/(a^3 + b^3 + abc) [cqd] Bem, acho que se aquele rearranjo usado estiver certo, esta seria uma solução. Seria legal se mais pessoas enviassem outras.. Abraços.. Igor Castro Icq: 37878785 www.cnaval.hpg.com.br ----- Original Message ----- From: "guilherme S." To: Sent: Sunday, April 27, 2003 10:29 AM Subject: [obm-l] ajuda com desigualdade > prove que: > 1/(a^3+b^3+abc)+1/(a^3+c^3+abc)+1/(b^3+c^3+abc)<=1/abc > sendo a,b e c reais positivos > > _______________________________________________________________________ > Yahoo! Mail > O melhor e-mail gratuito da internet: 6MB de espaço, antivírus, acesso POP3, filtro contra spam. > http://br.mail.yahoo.com/ > ========================================================================= > Instruções para entrar na lista, sair da lista e usar a lista em > http://www.mat.puc-rio.br/~nicolau/olimp/obm-l.html > ========================================================================= http://www.ieg.com.br ========================================================================= Instruções para entrar na lista, sair da lista e usar a lista em http://www.mat.puc-rio.br/~nicolau/olimp/obm-l.html ========================================================================= From owner-obm-l@sucuri.mat.puc-rio.br Sun Apr 27 22:44:48 2003 Return-Path: Received: (from majordom@localhost) by sucuri.mat.puc-rio.br (8.9.3/8.9.3) id WAA12484 for obm-l-MTTP; Sun, 27 Apr 2003 22:42:11 -0300 Received: from itaqui.terra.com.br (itaqui.terra.com.br [200.176.3.19]) by sucuri.mat.puc-rio.br (8.9.3/8.9.3) with ESMTP id WAA12480 for ; Sun, 27 Apr 2003 22:42:07 -0300 Received: from botucatu.terra.com.br (botucatu.terra.com.br [200.176.3.78]) by itaqui.terra.com.br (Postfix) with ESMTP id EAA6F3BC18B for ; Sun, 27 Apr 2003 22:41:36 -0300 (BRT) Received: from riemann.localdomain (RJ173091.user.veloxzone.com.br [200.149.173.91]) (authenticated user fabio.dias.moreira) by botucatu.terra.com.br (Postfix) with ESMTP id 7E61929C0BE for ; Sun, 27 Apr 2003 22:41:36 -0300 (BRT) Content-Type: text/plain; charset="iso-8859-1" From: =?iso-8859-1?q?F=E1bio=20Dias=20Moreira?= To: obm-l@mat.puc-rio.br Subject: Re: [obm-l] ajuda com desigualdade Date: Sun, 27 Apr 2003 22:41:25 -0300 User-Agent: KMail/1.4.3 References: <3eac844b.2eb1.0@ieg.com.br> In-Reply-To: <3eac844b.2eb1.0@ieg.com.br> MIME-Version: 1.0 Content-Transfer-Encoding: 8bit Message-Id: <200304272241.32514.fabio.dias.moreira@terra.com.br> Sender: owner-obm-l@sucuri.mat.puc-rio.br Precedence: bulk Reply-To: obm-l@mat.puc-rio.br -----BEGIN PGP SIGNED MESSAGE----- Hash: SHA1 On Sunday 27 April 2003 22:30, cnaval@ieg.com.br wrote: > [...] abc + abc + abc <= b^3 > +c^3 + abc > [...] Falso: para a=1000, b=1, c=1, 3000 <= 1002. []s, - -- Fábio "ctg \pi" Dias Moreira -----BEGIN PGP SIGNATURE----- Version: GnuPG v1.0.6 (GNU/Linux) Comment: For info see http://www.gnupg.org iD8DBQE+rIbMalOQFrvzGQoRAuqiAJ9s1x+JWBKPDMlvVax63nA16sb+RwCgiwrL p9FMM1cOna7c4ExC4nuPfVU= =sIny -----END PGP SIGNATURE----- ========================================================================= Instruções para entrar na lista, sair da lista e usar a lista em http://www.mat.puc-rio.br/~nicolau/olimp/obm-l.html ========================================================================= From owner-obm-l@sucuri.mat.puc-rio.br Sun Apr 27 22:59:07 2003 Return-Path: Received: (from majordom@localhost) by sucuri.mat.puc-rio.br (8.9.3/8.9.3) id WAA12955 for obm-l-MTTP; Sun, 27 Apr 2003 22:56:31 -0300 Received: from itaqui.terra.com.br (itaqui.terra.com.br [200.176.3.19]) by sucuri.mat.puc-rio.br (8.9.3/8.9.3) with ESMTP id WAA12951 for ; Sun, 27 Apr 2003 22:56:27 -0300 Received: from una.terra.com.br (una.terra.com.br [200.176.3.32]) by itaqui.terra.com.br (Postfix) with ESMTP id 42A673BC6F3 for ; Sun, 27 Apr 2003 22:55:57 -0300 (BRT) Received: from [200.177.193.64] (dl-nas2-sao-C8B1C140.p001.terra.com.br [200.177.193.64]) by una.terra.com.br (Postfix) with ESMTP id 749DE2F005C for ; Sun, 27 Apr 2003 22:55:56 -0300 (BRT) User-Agent: Microsoft-Outlook-Express-Macintosh-Edition/5.02.2022 Date: Sun, 27 Apr 2003 22:57:22 -0300 Subject: Re: [obm-l] C=?ISO-8859-1?B?4Q==?=lculo em porvas militares From: Claudio Buffara To: Message-ID: In-Reply-To: Mime-version: 1.0 Content-type: text/plain; charset="ISO-8859-1" Content-Transfer-Encoding: 8bit X-MIME-Autoconverted: from quoted-printable to 8bit by sucuri.mat.puc-rio.br id WAA12952 Sender: owner-obm-l@sucuri.mat.puc-rio.br Precedence: bulk Reply-To: obm-l@mat.puc-rio.br on 27.04.03 17:07, basketboy_igor at basketboy_igor@bol.com.br wrote: > Saudações a todos, > Estou com algumas dúvidas em quetões de calculo I em > provas militeres: > > i)(EN/95) O lim(x->0) [raiz(x+b)+raiz(x+a)-raiz(b)-raiz > (a)]/x é igual a: > resp: 1/[2*raiz(b)] + 1/[2*raiz(a)] > Use o fato de que: (raiz(x+b) - raiz(b))/x = 1/(raiz(x+b) + raiz(b)) (multiplique o numerador e o denominador por raiz(x+b) + raiz(b) > ii)(EN/95) Se f(x) = e^(2x) + (x+1)*cos(x), então f'(0) > é igual a: > resp: 3. > f'(x) = 2e^(2x) + cos(x) - (x+1)sen(x) ==> f'(0) = 2 + 1 - (0+1)*0 = 3 > iii)(EN/97) O valor da integral de raiz[1 + 9*x*(dx)] é: > a)(2/27)*(1+9*x)^(3/2) + c > O integrando deve ser raiz(1 + 9x)*dx, certo? Faca u = 1+9x ==> du = 9dx. Logo a integral passa a ser de: raiz(u)*(du/9) = (1/9)*u^(1/2)*du. Integrando, obtemos: (1/9)*u^(3/2)/(3/2) + C = (2/27)*u^(3/2) + c. Ou seja: (2/27)*(1+9x)^(3/2) + C. Um abraco, Claudio. ========================================================================= Instruções para entrar na lista, sair da lista e usar a lista em http://www.mat.puc-rio.br/~nicolau/olimp/obm-l.html ========================================================================= From owner-obm-l@sucuri.mat.puc-rio.br Sun Apr 27 23:03:26 2003 Return-Path: Received: (from majordom@localhost) by sucuri.mat.puc-rio.br (8.9.3/8.9.3) id XAA13206 for obm-l-MTTP; Sun, 27 Apr 2003 23:00:46 -0300 Received: from itaqui.terra.com.br (itaqui.terra.com.br [200.176.3.19]) by sucuri.mat.puc-rio.br (8.9.3/8.9.3) with ESMTP id XAA13187 for ; Sun, 27 Apr 2003 23:00:37 -0300 Received: from botucatu.terra.com.br (botucatu.terra.com.br [200.176.3.78]) by itaqui.terra.com.br (Postfix) with ESMTP id 1E7143BC08A for ; Sun, 27 Apr 2003 23:00:07 -0300 (BRT) Received: from [200.177.186.69] (dl-nas4-sao-C8B1BA45.p001.terra.com.br [200.177.186.69]) by botucatu.terra.com.br (Postfix) with ESMTP id 424DD29C0B2 for ; Sun, 27 Apr 2003 23:00:06 -0300 (BRT) User-Agent: Microsoft-Outlook-Express-Macintosh-Edition/5.02.2022 Date: Sun, 27 Apr 2003 23:01:34 -0300 Subject: Re: [obm-l] RE: [obm-l] Indu=?ISO-8859-1?B?5+M=?=o Finita From: Claudio Buffara To: Message-ID: In-Reply-To: <001501c30d18$0c4a2db0$9865fea9@computer> Mime-version: 1.0 Content-type: text/plain; charset="ISO-8859-1" Content-Transfer-Encoding: 8bit X-MIME-Autoconverted: from quoted-printable to 8bit by sucuri.mat.puc-rio.br id XAA13193 Sender: owner-obm-l@sucuri.mat.puc-rio.br Precedence: bulk Reply-To: obm-l@mat.puc-rio.br Oi, Artur: O maior divisor impar de 21*21 = 441 eh 441. Assim, teremos: 441 = 2^0 * 441. 7 eh o maior divisor PRIMO de 441. Um abraco, Claudio. on 27.04.03 20:52, Artur Costa Steiner at artur@opendf.com.br wrote: > > O no. 4 pode ser feito sem inducao. > > Suponha que n nao seja uma potencia de 2. > Seja p o maior divisor impar de n. > Como n >= 3, temos que p >= 3, e podemos escrever n = (2^k)*p, onde k eh > um inteiro nao negativo. > [Artur Costa Steiner] > Nao hah um engano aqui? Suponhamos que n =21. 21 nao eh potencia de 2 e > seu maior divisor impar eh 7. Entretanto, nao existe qualquer inteiro > nao negativo k para o qual 21 = (2^k)*7. > Um abraco > Artur > > > Seja a = 2^k > 2^n + 1 = 2^((2^k)*p) + 1 = a^p + 1 = > = (a + 1)*(a^(p-1) + a^(p-2) + ... + a + 1). > > Como a >= 1 e p >= 3, ambos os fatores sao >= 2. Logo, 2^n + 1 eh > composto. > > Um abraco, > Claudio. > > ========================================================================= > Instruções para entrar na lista, sair da lista e usar a lista em > http://www.mat.puc-rio.br/~nicolau/olimp/obm-l.html > ========================================================================= > ========================================================================= Instruções para entrar na lista, sair da lista e usar a lista em http://www.mat.puc-rio.br/~nicolau/olimp/obm-l.html ========================================================================= From owner-obm-l@sucuri.mat.puc-rio.br Sun Apr 27 23:04:22 2003 Return-Path: Received: (from majordom@localhost) by sucuri.mat.puc-rio.br (8.9.3/8.9.3) id XAA13234 for obm-l-MTTP; Sun, 27 Apr 2003 23:01:47 -0300 Received: from smtp-27.ig.com.br (smtp-27.ig.com.br [200.226.132.159]) by sucuri.mat.puc-rio.br (8.9.3/8.9.3) with SMTP id XAA13229 for ; Sun, 27 Apr 2003 23:01:43 -0300 Received: (qmail 2420 invoked from network); 28 Apr 2003 02:01:26 -0000 Received: from unknown (HELO henrique) (200.140.6.162) by smtp-27.ig.com.br with SMTP; 28 Apr 2003 02:01:26 -0000 Message-ID: <014201c30d2a$0d95bb50$019da8c0@henrique> From: "=?iso-8859-1?Q?Henrique_Patr=EDcio_Sant'Anna_Branco?=" To: "OBM" Subject: [obm-l] =?iso-8859-1?Q?Irracionaliza=E7=E3o?= Date: Sun, 27 Apr 2003 23:01:16 -0300 MIME-Version: 1.0 Content-Type: text/plain; charset="iso-8859-1" Content-Transfer-Encoding: 8bit X-Priority: 3 X-MSMail-Priority: Normal X-Mailer: Microsoft Outlook Express 6.00.2800.1158 X-MimeOLE: Produced By Microsoft MimeOLE V6.00.2800.1165 Sender: owner-obm-l@sucuri.mat.puc-rio.br Precedence: bulk Reply-To: obm-l@mat.puc-rio.br Pessoal, Como eu procedo pra irracionalização de funções? Por exemplo, pra calcular o limite de sqrt(x^2 + x) -x com x tendendo a infinito, eu teria que passar a raiz pra baixo, certo? Como? Grato, Henrique. ========================================================================= Instruções para entrar na lista, sair da lista e usar a lista em http://www.mat.puc-rio.br/~nicolau/olimp/obm-l.html ========================================================================= From owner-obm-l@sucuri.mat.puc-rio.br Sun Apr 27 23:34:01 2003 Return-Path: Received: (from majordom@localhost) by sucuri.mat.puc-rio.br (8.9.3/8.9.3) id XAA14814 for obm-l-MTTP; Sun, 27 Apr 2003 23:31:23 -0300 Received: from trex-b.centroin.com.br (trex-b.centroin.com.br [200.225.63.136]) by sucuri.mat.puc-rio.br (8.9.3/8.9.3) with ESMTP id XAA14810 for ; Sun, 27 Apr 2003 23:31:19 -0300 Received: from centroin.com.br (RJ119162.user.veloxzone.com.br [200.141.119.162] (may be forged)) (authenticated bits=0) by trex-b.centroin.com.br (8.12.9/8.12.9) with ESMTP id h3S2UnVi028377 for ; Sun, 27 Apr 2003 23:30:49 -0300 (EST) Message-ID: <3EAC9299.2040605@centroin.com.br> Date: Sun, 27 Apr 2003 23:31:53 -0300 From: "A. C. Morgado" User-Agent: Mozilla/5.0 (Windows; U; Windows NT 5.0; en-US; rv:1.0.2) Gecko/20030208 Netscape/7.02 X-Accept-Language: en-us, en MIME-Version: 1.0 To: obm-l@mat.puc-rio.br Subject: Re: [obm-l] =?ISO-8859-1?Q?Irracionaliza=E7=E3o?= References: <014201c30d2a$0d95bb50$019da8c0@henrique> Content-Type: text/plain; charset=ISO-8859-1; format=flowed Content-Transfer-Encoding: 8bit Sender: owner-obm-l@sucuri.mat.puc-rio.br Precedence: bulk Reply-To: obm-l@mat.puc-rio.br Multiplicando e dividindo por sqrt (x^2+x) + x. Voce obtera [(x^2+x)-(x^2)]/[sqrt (x^2+x) + x] = x/[sqrt (x^2+x) + x] Henrique Patrício Sant'Anna Branco wrote: >Pessoal, > >Como eu procedo pra irracionalização de funções? >Por exemplo, pra calcular o limite de sqrt(x^2 + x) -x com x tendendo a >infinito, eu teria que passar a raiz pra baixo, certo? Como? > >Grato, >Henrique. > >========================================================================= >Instruções para entrar na lista, sair da lista e usar a lista em >http://www.mat.puc-rio.br/~nicolau/olimp/obm-l.html >========================================================================= > > > > ========================================================================= Instruções para entrar na lista, sair da lista e usar a lista em http://www.mat.puc-rio.br/~nicolau/olimp/obm-l.html ========================================================================= From owner-obm-l@sucuri.mat.puc-rio.br Mon Apr 28 00:31:36 2003 Return-Path: Received: (from majordom@localhost) by sucuri.mat.puc-rio.br (8.9.3/8.9.3) id AAA16516 for obm-l-MTTP; Mon, 28 Apr 2003 00:28:48 -0300 Received: from paiol.terra.com.br (paiol.terra.com.br [200.176.3.18]) by sucuri.mat.puc-rio.br (8.9.3/8.9.3) with ESMTP id AAA16512 for ; Mon, 28 Apr 2003 00:28:45 -0300 Received: from una.terra.com.br (una.terra.com.br [200.176.3.32]) by paiol.terra.com.br (Postfix) with ESMTP id 5002C87DAD for ; Mon, 28 Apr 2003 00:28:15 -0300 (BRT) Received: from [200.177.188.52] (dl-nas7-sao-C8B1BC34.p001.terra.com.br [200.177.188.52]) by una.terra.com.br (Postfix) with ESMTP id 756C82F004D for ; Mon, 28 Apr 2003 00:28:14 -0300 (BRT) User-Agent: Microsoft-Outlook-Express-Macintosh-Edition/5.02.2022 Date: Mon, 28 Apr 2003 00:29:42 -0300 Subject: Re: [obm-l] Pontos pintados From: Claudio Buffara To: Message-ID: In-Reply-To: <200304272112.h3RLCL507340@Gauss.impa.br> Mime-version: 1.0 Content-type: text/plain; charset="US-ASCII" Content-transfer-encoding: 7bit Sender: owner-obm-l@sucuri.mat.puc-rio.br Precedence: bulk Reply-To: obm-l@mat.puc-rio.br Oi, Gugu: Peguei a sutileza. Tudo compreendido! Gostei muito do argumento. Serah que 4 cores sao suficientes pra garantir que cada segmento unitario tenha extremidades de cores distintas? Um abraco, Claudio. >> on 27.04.03 02:06, Carlos Gustavo Tamm de Araujo Moreira at gugu@impa.br >> wrote: >> >>> Caro Claudio, >>> Suponha que em todo segmento unitario as extremidades tem cores >>> distintas.Se X e Y estao a distancia raiz(3), e' possivel achar A e B no >>> plano tais que os triangulos XAB e ABY sejam equilateros de lado 1. Assim, >>> A e B tem cores distintas e diferentes da cor de X, donde a cor de Y deve >>> ser igual a cor de X. >>> Considere agora um triangulo XYZ com lados >>> XY=XZ=raiz(3) e YZ=1. Temos que as cores de Y e de Z devem ser iguais a cor >>> de X mas a cor de Y deve ser distinta da cor de Z, absurdo. > > Temos ZX=raiz(3) e eu tinha mostrado que quaisquer dois pontos a > distancia raiz(3) devem ter a mesma cor. > ========================================================================= Instruções para entrar na lista, sair da lista e usar a lista em http://www.mat.puc-rio.br/~nicolau/olimp/obm-l.html ========================================================================= From owner-obm-l@sucuri.mat.puc-rio.br Mon Apr 28 00:46:20 2003 Return-Path: Received: (from majordom@localhost) by sucuri.mat.puc-rio.br (8.9.3/8.9.3) id AAA16886 for obm-l-MTTP; Mon, 28 Apr 2003 00:43:40 -0300 Received: from ivoti.terra.com.br (ivoti.terra.com.br [200.176.3.20]) by sucuri.mat.puc-rio.br (8.9.3/8.9.3) with ESMTP id AAA16882 for ; Mon, 28 Apr 2003 00:43:36 -0300 Received: from botucatu.terra.com.br (botucatu.terra.com.br [200.176.3.78]) by ivoti.terra.com.br (Postfix) with ESMTP id 6D8A7408314 for ; Mon, 28 Apr 2003 00:43:06 -0300 (BRT) Received: from [200.177.179.59] (dl-nas3-sao-C8B1B33B.p001.terra.com.br [200.177.179.59]) by botucatu.terra.com.br (Postfix) with ESMTP id 7CF1029C084 for ; Mon, 28 Apr 2003 00:43:05 -0300 (BRT) User-Agent: Microsoft-Outlook-Express-Macintosh-Edition/5.02.2022 Date: Mon, 28 Apr 2003 00:45:12 -0300 Subject: Re: [obm-l] ajuda com desigualdade From: Claudio Buffara To: Message-ID: In-Reply-To: <3eac844b.2eb1.0@ieg.com.br> Mime-version: 1.0 Content-type: text/plain; charset="ISO-8859-1" Content-Transfer-Encoding: 8bit X-MIME-Autoconverted: from quoted-printable to 8bit by sucuri.mat.puc-rio.br id AAA16883 Sender: owner-obm-l@sucuri.mat.puc-rio.br Precedence: bulk Reply-To: obm-l@mat.puc-rio.br Oi, Igor: Infelizmente, nem todas as suas desigualdades intermediarias sao verdadeiras. Se a >= b >= c > 0, entao a^3 + b^3 >= 2abc mas nao eh verdade que: a^3 + c^3 >= 2abc nem que b^3 + c^3 >= 2abc (tome a = b = 3, c = 2) Para ver que a^3 + b^3 >= 2abc basta demonstrar que a^3 + b^3 >= 2ab^2 Teremos: a^3 - 2ab^2 + b^3 = (a - b)(a^2 + ab - b^2). Como ambos os fatores sao nao-negativos, teremos que: a^3 - 2ab^2 + b^3 >= 0 ==> a^3 + b^3 >= 2ab^2 >= 2abc, pois b >= c > 0. **** A desiguladade do rearranjo, aplicada aos reais positivos a, b, c, implica que: abc + abc + abc <= a^3 + b^3 + c^3. (isso, alias, eh a desigualdade MG <= MA aplicada a a^3, b^3, c^3). **** Esta desigualdade foi uma questao da USAMO de 1997. A solucao estah aqui: http://www.kalva.demon.co.uk/usa/usa97.html Um abraco, Claudio. on 27.04.03 22:30, cnaval@ieg.com.br at cnaval@ieg.com.br wrote: > Oi guilherme, > como a expressão eh simétrica em relação à a, b e c, podemos supor sem perda > generalidade que a =>b=>c > e pela desigualdade do rearranjo abc + abc + abc <= a^3 + b^3 + abc (na > verdade neste ponto n tenho mta certeza, > abc + abc + abc <= a^3 +c^3 > + abc peço a ajuda aos mestres da lista :P) > abc + abc + abc <= b^3 +c^3 > + abc > invertando cada desigualdade: > 1/(3abc) => 1/(a^3 + b^3 + abc) > 1/(3abc) => 1/(a^3 + c^3 + abc) > 1/(3abc) => 1/(b^3 + c^3 + abc) > somando as três: > 1/abc => 1/(a^3 + b^3 + abc) + 1/(a^3 + b^3 + abc) + 1/(a^3 + b^3 + abc) > [cqd] > > Bem, acho que se aquele rearranjo usado estiver certo, esta seria uma > solução. Seria legal se mais pessoas enviassem outras.. > Abraços.. > > Igor Castro > Icq: 37878785 > www.cnaval.hpg.com.br > > ----- Original Message ----- > From: "guilherme S." > To: > Sent: Sunday, April 27, 2003 10:29 AM > Subject: [obm-l] ajuda com desigualdade > > >> prove que: >> 1/(a^3+b^3+abc)+1/(a^3+c^3+abc)+1/(b^3+c^3+abc)<=1/abc >> sendo a,b e c reais positivos >> ========================================================================= Instruções para entrar na lista, sair da lista e usar a lista em http://www.mat.puc-rio.br/~nicolau/olimp/obm-l.html ========================================================================= From owner-obm-l@sucuri.mat.puc-rio.br Mon Apr 28 01:05:22 2003 Return-Path: Received: (from majordom@localhost) by sucuri.mat.puc-rio.br (8.9.3/8.9.3) id BAA17950 for obm-l-MTTP; Mon, 28 Apr 2003 01:02:24 -0300 Received: from artemis.opendf.com.br (artemis.opengate.com.br [200.181.71.14]) by sucuri.mat.puc-rio.br (8.9.3/8.9.3) with ESMTP id BAA17943 for ; Mon, 28 Apr 2003 01:02:20 -0300 Received: from localhost (localhost [127.0.0.1]) by artemis.opendf.com.br (Postfix) with ESMTP id AE90C2BF0A for ; Mon, 28 Apr 2003 01:01:49 -0300 (BRT) Received: from artemis.opendf.com.br ([127.0.0.1]) by localhost (artemis.opengate.com.br [127.0.0.1:10024]) (amavisd-new) with ESMTP id 20017-04 for ; Mon, 28 Apr 2003 01:01:48 -0300 (BRT) Received: from computer (200-140-002-143.bsace7001.dsl.brasiltelecom.net.br [200.140.2.143]) by artemis.opendf.com.br (Postfix) with ESMTP id 941252BED7 for ; Mon, 28 Apr 2003 01:01:48 -0300 (BRT) From: "Artur Costa Steiner" To: Subject: [obm-l] =?iso-8859-1?Q?RE:_=5Bobm-l=5D_RE:_=5Bobm-l=5D_Indu=E7=E3o_Finita?= Date: Mon, 28 Apr 2003 01:01:50 -0300 Organization: Steiner Consultoria LTDA Message-ID: <001c01c30d3a$e60aaf30$9865fea9@computer> MIME-Version: 1.0 Content-Type: text/plain; charset="iso-8859-1" Content-Transfer-Encoding: 7bit X-Priority: 3 (Normal) X-MSMail-Priority: Normal X-Mailer: Microsoft Outlook, Build 10.0.2627 X-MimeOLE: Produced By Microsoft MimeOLE V6.00.2800.1165 In-Reply-To: Importance: Normal X-Virus-Scanned: by amavisd-new Sender: owner-obm-l@sucuri.mat.puc-rio.br Precedence: bulk Reply-To: obm-l@mat.puc-rio.br Oi Claudio, Houve um problema de grafia no email. Na realidade o ponto que eu coloquei, dando a ideia de 21 .21 = 21 X 21 =441 nao era um sinal matematico, mas sim o sinal de pontuacao indicando final de sentenca.... Eu quis dizer 21, enao 441 (risos...) Um abraco Artur >Oi, Artur: > >O maior divisor impar de 21*21 = 441 eh 441. > >Assim, teremos: 441 = 2^0 * 441. > >7 eh o maior divisor PRIMO de 441. > >Um abraco, >Claudio. ========================================================================= Instruções para entrar na lista, sair da lista e usar a lista em http://www.mat.puc-rio.br/~nicolau/olimp/obm-l.html ========================================================================= From owner-obm-l@sucuri.mat.puc-rio.br Mon Apr 28 01:49:05 2003 Return-Path: Received: (from majordom@localhost) by sucuri.mat.puc-rio.br (8.9.3/8.9.3) id BAA19575 for obm-l-MTTP; Mon, 28 Apr 2003 01:45:03 -0300 Received: from hotmail.com (oe34.law10.hotmail.com [64.4.14.91]) by sucuri.mat.puc-rio.br (8.9.3/8.9.3) with ESMTP id BAA19571 for ; Mon, 28 Apr 2003 01:44:59 -0300 Received: from mail pickup service by hotmail.com with Microsoft SMTPSVC; Sun, 27 Apr 2003 21:44:27 -0700 Received: from 67.24.122.29 by oe34.law10.hotmail.com with DAV; Mon, 28 Apr 2003 04:44:26 +0000 X-Originating-IP: [67.24.122.29] X-Originating-Email: [lrecova@hotmail.com] From: =?iso-8859-1?Q?Leandro_Lacorte_Rec=F4va?= To: Subject: RE: [obm-l] Limites Date: Sun, 27 Apr 2003 21:44:26 -0700 Message-ID: <000501c30d40$d8e28de0$1d7a1843@LeandroRecova> MIME-Version: 1.0 Content-Type: text/plain; charset="iso-8859-1" X-Priority: 3 (Normal) X-MSMail-Priority: Normal X-Mailer: Microsoft Outlook, Build 10.0.3416 Importance: Normal In-Reply-To: X-MimeOLE: Produced By Microsoft MimeOLE V6.00.2800.1165 X-OriginalArrivalTime: 28 Apr 2003 04:44:27.0247 (UTC) FILETIME=[D95F37F0:01C30D40] Content-Transfer-Encoding: 8bit X-MIME-Autoconverted: from quoted-printable to 8bit by sucuri.mat.puc-rio.br id BAA19572 Sender: owner-obm-l@sucuri.mat.puc-rio.br Precedence: bulk Reply-To: obm-l@mat.puc-rio.br Quero me desculpar pelo erro cometido ! Acho que olhei o limite muito rapido e nao prestei atencao... Obrigado Prof. Morgado e Claudio por notarem o erro ! Leandro -----Original Message----- From: owner-obm-l@sucuri.mat.puc-rio.br [mailto:owner-obm-l@sucuri.mat.puc-rio.br] On Behalf Of Claudio Buffara Sent: Sunday, April 27, 2003 6:12 AM To: obm-l@mat.puc-rio.br Subject: Re: [obm-l] Limites on 27.04.03 02:08, niski at fabio@niski.com wrote: > Pretendo nesta mensagem responder ao Leandro Recova, ao prof. Morgado, e > ao Claudio Buffara. > > prof. Morgado, desculpe.Eu estava acostumado com o acrônimo cbrt (raiz > cubica) e não fui cuidadoso o suficiente para fazer tal referencia. > > Claudio e Leandro : > Muito estranho... > Para o Leandro o limite deu 0, para o Buffara não existe, no Mathematica > deu -infinito e a resposta do livro é infinito !!!! e agora? > > Quanto a utilizar sin no lugar de sen , não acho que seja um grande > problema, alias sin remete ao termo original em latim "sinus". > Na minha opinião nao acho uma boa a lingua portuguesa mudar alguns > termos originais...matriz por exemplo..pq este Z? pq nao deixar do > originial cunhado por Sylvester!?!? Já vi coisas tb do tipo "Onduletas" > para designar wavelets...eu acho que seria a mesma coisa que se referir > ao Newton como "Isaque Newton". Oi, Niski: Nao ligue muito pros meus comentarios sobre notacao - o importante eh se fazer entender e isso voce conseguiu (mesmo com cbrt e sin). No mais, toda vez que eu fizer um comentario mais acido, pode ter certeza que eh soh brincadeira - eu sou o tipo de pessoa que "perde o amigo mas nao perde a piada" - portanto, nao me leve muito a serio nestas horas. Eu ja me ferrei varias vezes porque nem too mundo tem o mesmo senso de humor, mas nao consigo evitar. Assim, ja fica o aviso... Sobre o limite, eu cheguei a conclusao da que os limites laterais eram + e - infinito mas fiquei com um pouco de medo de ter errado as contas. Sabe o que eu fiz? Calculei no computador o valor da expressao original em x = 1,5707 e 1,5709 e achei valores enormes com sinais opostos - essa evidencia empirica me convenceu. Nao eh aceitavel como prova, mas muita matematica de 1a. qualidade ja foi descoberta atraves de observacoes empiricas (por exemplo, Gauss conjecturou o teorema dos nos. primos com base em analise numerica de uma tabela de primos - haja falto do que fazer, hem???) Um abraco, Claudio. ======================================================================== = Instruções para entrar na lista, sair da lista e usar a lista em http://www.mat.puc-rio.br/~nicolau/olimp/obm-l.html ======================================================================== = ========================================================================= Instruções para entrar na lista, sair da lista e usar a lista em http://www.mat.puc-rio.br/~nicolau/olimp/obm-l.html ========================================================================= From owner-obm-l@sucuri.mat.puc-rio.br Mon Apr 28 01:56:15 2003 Return-Path: Received: (from majordom@localhost) by sucuri.mat.puc-rio.br (8.9.3/8.9.3) id BAA19723 for obm-l-MTTP; Mon, 28 Apr 2003 01:53:41 -0300 Received: from Euler.impa.br (euler.impa.br [147.65.1.3]) by sucuri.mat.puc-rio.br (8.9.3/8.9.3) with ESMTP id BAA19718 for ; Mon, 28 Apr 2003 01:53:38 -0300 Received: from Gauss.impa.br (Gauss [147.65.4.1]) by Euler.impa.br (8.11.6p2/8.11.6) with ESMTP id h3S4r8027912 for ; Mon, 28 Apr 2003 01:53:08 -0300 (EST) From: Carlos Gustavo Tamm de Araujo Moreira Received: by Gauss.impa.br (8.11.6p2) id h3S4qp600778; Mon, 28 Apr 2003 01:52:51 -0300 (EST) Message-Id: <200304280452.h3S4qp600778@Gauss.impa.br> Subject: Re: [obm-l] Pontos pintados To: obm-l@mat.puc-rio.br Date: Mon, 28 Apr 2003 01:52:51 -0300 (EST) In-Reply-To: from "Claudio Buffara" at Apr 28, 3 00:29:42 am X-Mailer: ELM [version 2.4 PL25] MIME-Version: 1.0 Content-Type: text/plain; charset=US-ASCII Content-Transfer-Encoding: 7bit Sender: owner-obm-l@sucuri.mat.puc-rio.br Precedence: bulk Reply-To: obm-l@mat.puc-rio.br Caro Claudio, Nao se sabe. Por outro lado e' possivel construir (tentem, e' interessante) uma tal coloracao com 7 cores. Os casos de 4, 5 e 6 cores estao em aberto. Vejam a secao sobre o numero cromatico de R^n no meu livro com o Yoshi "Topicos em Combinatoria Contemporanea" (esta' na minha pagina www.impa.br/~gugu , mais precisamente em www.impa.br/~gugu/coloyoshi.ps). Abracos, Gugu > >Oi, Gugu: > >Peguei a sutileza. Tudo compreendido! Gostei muito do argumento. > >Serah que 4 cores sao suficientes pra garantir que cada segmento unitario >tenha extremidades de cores distintas? > >Um abraco, >Claudio. > >>> on 27.04.03 02:06, Carlos Gustavo Tamm de Araujo Moreira at gugu@impa.br >>> wrote: >>> >>>> Caro Claudio, >>>> Suponha que em todo segmento unitario as extremidades tem cores >>>> distintas.Se X e Y estao a distancia raiz(3), e' possivel achar A e B no >>>> plano tais que os triangulos XAB e ABY sejam equilateros de lado 1. Assim, >>>> A e B tem cores distintas e diferentes da cor de X, donde a cor de Y deve >>>> ser igual a cor de X. >>>> Considere agora um triangulo XYZ com lados >>>> XY=XZ=raiz(3) e YZ=1. Temos que as cores de Y e de Z devem ser iguais a cor >>>> de X mas a cor de Y deve ser distinta da cor de Z, absurdo. >> >> Temos ZX=raiz(3) e eu tinha mostrado que quaisquer dois pontos a >> distancia raiz(3) devem ter a mesma cor. >> > >========================================================================= >Instruções para entrar na lista, sair da lista e usar a lista em >http://www.mat.puc-rio.br/~nicolau/olimp/obm-l.html >========================================================================= ========================================================================= Instruções para entrar na lista, sair da lista e usar a lista em http://www.mat.puc-rio.br/~nicolau/olimp/obm-l.html ========================================================================= From owner-obm-l@sucuri.mat.puc-rio.br Mon Apr 28 03:01:24 2003 Return-Path: Received: (from majordom@localhost) by sucuri.mat.puc-rio.br (8.9.3/8.9.3) id CAA21134 for obm-l-MTTP; Mon, 28 Apr 2003 02:58:41 -0300 Received: from hotmail.com (oe66.law10.hotmail.com [64.4.14.201]) by sucuri.mat.puc-rio.br (8.9.3/8.9.3) with ESMTP id CAA21130 for ; Mon, 28 Apr 2003 02:58:37 -0300 Received: from mail pickup service by hotmail.com with Microsoft SMTPSVC; Sun, 27 Apr 2003 22:58:06 -0700 Received: from 67.24.122.29 by oe66.law10.hotmail.com with DAV; Mon, 28 Apr 2003 05:58:05 +0000 X-Originating-IP: [67.24.122.29] X-Originating-Email: [lrecova@hotmail.com] From: =?iso-8859-1?Q?Leandro_Lacorte_Rec=F4va?= To: Subject: [obm-l] =?iso-8859-1?Q?RE:_=5Bobm-l=5D_C=E1lculo_em_porvas_militares?= Date: Sun, 27 Apr 2003 22:58:05 -0700 Message-ID: <000c01c30d4b$22ba2a40$1d7a1843@LeandroRecova> MIME-Version: 1.0 Content-Type: text/plain; charset="iso-8859-1" X-Priority: 3 (Normal) X-MSMail-Priority: Normal X-Mailer: Microsoft Outlook, Build 10.0.3416 Importance: Normal In-Reply-To: X-MimeOLE: Produced By Microsoft MimeOLE V6.00.2800.1165 X-OriginalArrivalTime: 28 Apr 2003 05:58:06.0277 (UTC) FILETIME=[2351AF50:01C30D4B] Content-Transfer-Encoding: 8bit X-MIME-Autoconverted: from quoted-printable to 8bit by sucuri.mat.puc-rio.br id CAA21131 Sender: owner-obm-l@sucuri.mat.puc-rio.br Precedence: bulk Reply-To: obm-l@mat.puc-rio.br i)(EN/95) O lim(x->0) [raiz(x+b)+raiz(x+a)-raiz(b)-raiz (a)]/x é igual a: resp: 1/[2*raiz(b)] + 1/[2*raiz(a)] Separe os termos e "desracionalize" lim(x->0) [raiz(x+b)-raiz(b)]/x + {raiz(x+a)-a]/x= lim(x->0) 1/[raiz(x+b)+raiz(b)] + 1/[raiz(x+a)+raiz(a)]= 1/2(raiz(b) + 1/2(raiz(a)) ii)(EN/95) Se f(x) = e^(2x) + (x+1)*cos(x), então f'(0) é igual a: resp: 3. f'(x)=2e^(2x)+cos^2(x)-(x+1)sin(x) f'(0)=2+1-0 = 3. iii)(EN/97) O valor da integral de raiz[1 + 9*x*(dx)] é: a)(2/27)*(1+9*x)^(3/2) + c Seria integral (raiz(1+9x))dx ? Se for isso, entao temos Chame u = 1 + 9x entao du = 9dx, logo, a integral fica, Int(raiz(1+9x))dx = Int[(raiz(u))/9 ]du = 2/3 (raiz(u))^3/3 + c, Substituindo u=1+9x, chegamos na resposta desejada. iv)(EN/98)A equação do movimento de um progétil que se desloca ao longo do eixo x é x(t)={e^[-(t – Pi/4)]}*sen (t) + cotg²(t), t >=0 . A aceleração do projétil no instante t=0 é: Resp: d) 16 –2*raiz(2) Basta derivar duas vezes a expressao e lembrar da regra da cadeia. v) No ultimo limite, use o limite fundamental da trigonometria em A e no segundo problema mude a base do logaritmo pra base 2 que o resultado sai. Leandro -----Original Message----- From: owner-obm-l@sucuri.mat.puc-rio.br [mailto:owner-obm-l@sucuri.mat.puc-rio.br] On Behalf Of basketboy_igor Sent: Sunday, April 27, 2003 1:08 PM To: obm-l@mat.puc-rio.br Subject: [obm-l] Cálculo em porvas militares Saldações à todos, Estou com algumas dúvidas em quetões de calculo I em provas militeres: i)(EN/95) O lim(x->0) [raiz(x+b)+raiz(x+a)-raiz(b)-raiz (a)]/x é igual a: resp: 1/[2*raiz(b)] + 1/[2*raiz(a)] ii)(EN/95) Se f(x) = e^(2x) + (x+1)*cos(x), então f'(0) é igual a: resp: 3. iii)(EN/97) O valor da integral de raiz[1 + 9*x*(dx)] é: a)(2/27)*(1+9*x)^(3/2) + c iv)(EN/98)A equação do movimento de um progétil que se desloca ao longo do eixo x é x(t)={e^[-(t – Pi/4)]}*sen (t) + cotg²(t), t >=0 . A aceleração do projétil no instante t=0 é: Resp: d) 16 –2*raiz(2) v)(MM/98) Sendo A = Lim(x->0) {2*raiz[x*sen(6x)]}/ {[cossec(6x)]*[1 - cos²(6x)]} e B = Lim (x->log 2 na base 3)[2^(2x+1], [(A²)*B]/2 vale: Resp: b) 6 Igor Correia, #Mathematics ________________________________________________________________________ __ Seleção de Softwares UOL. 10 softwares escolhidos pelo UOL para você e sua família. http://www.uol.com.br/selecao ======================================================================== = Instruções para entrar na lista, sair da lista e usar a lista em http://www.mat.puc-rio.br/~nicolau/olimp/obm-l.html ======================================================================== = ========================================================================= Instruções para entrar na lista, sair da lista e usar a lista em http://www.mat.puc-rio.br/~nicolau/olimp/obm-l.html ========================================================================= From owner-obm-l@sucuri.mat.puc-rio.br Mon Apr 28 08:52:42 2003 Return-Path: Received: (from majordom@localhost) by sucuri.mat.puc-rio.br (8.9.3/8.9.3) id IAA25752 for obm-l-MTTP; Mon, 28 Apr 2003 08:49:29 -0300 Received: (from nicolau@localhost) by sucuri.mat.puc-rio.br (8.9.3/8.9.3) id IAA25747 for obm-l@mat.puc-rio.br; Mon, 28 Apr 2003 08:49:28 -0300 Date: Mon, 28 Apr 2003 08:49:28 -0300 From: "Nicolau C. Saldanha" To: obm-l@mat.puc-rio.br Subject: Re: [obm-l] ???? Message-ID: <20030428084928.A25567@sucuri.mat.puc-rio.br> References: Mime-Version: 1.0 Content-Type: text/plain; charset=iso-8859-1 Content-Disposition: inline Content-Transfer-Encoding: 8bit User-Agent: Mutt/1.2.5i In-Reply-To: ; from thiagobarrosilva@bol.com.br on Sun, Apr 27, 2003 at 12:15:05PM -0300 Sender: owner-obm-l@sucuri.mat.puc-rio.br Precedence: bulk Reply-To: obm-l@mat.puc-rio.br On Sun, Apr 27, 2003 at 12:15:05PM -0300, THIAGO wrote: > Bom dia! Meu nome é Thiago e acabo de entrar na > lista....Mas tenho uma dúvida cruel: acabei o ensino > médio; faço cursinho pré-vestibular e gostaria de saber > se eu posso participar da Olímpíada Brasileira de > Matemática, mesmo não estando no 2º grau. Um dos meus > professores de matematica do curso ministrou a olimpiada > ano passado no meu colégio..... O que posso fazer? A sua situação não é rara e foi discutida e explicitamente incorporada ao regulamento. Você pode fazer OBM nível 3 sim, desde que tenha concluido o 2o grau há menos de um ano e não tenha entrado para a universidade. O regulamento está em www.obm.org.br []s, N. ========================================================================= Instruções para entrar na lista, sair da lista e usar a lista em http://www.mat.puc-rio.br/~nicolau/olimp/obm-l.html ========================================================================= From owner-obm-l@sucuri.mat.puc-rio.br Mon Apr 28 08:53:17 2003 Return-Path: Received: (from majordom@localhost) by sucuri.mat.puc-rio.br (8.9.3/8.9.3) id IAA25792 for obm-l-MTTP; Mon, 28 Apr 2003 08:50:45 -0300 Received: (from nicolau@localhost) by sucuri.mat.puc-rio.br (8.9.3/8.9.3) id IAA25783 for obm-l@mat.puc-rio.br; Mon, 28 Apr 2003 08:50:45 -0300 Date: Mon, 28 Apr 2003 08:50:44 -0300 From: "Nicolau C. Saldanha" To: obm-l@mat.puc-rio.br Subject: Re: [obm-l] Limites Message-ID: <20030428085044.B25567@sucuri.mat.puc-rio.br> References: <200304270935.h3R9ZTxs012032@trex.centroin.com.br> <3EABE658.9080500@niski.com> Mime-Version: 1.0 Content-Type: text/plain; charset=iso-8859-1 Content-Disposition: inline Content-Transfer-Encoding: 8bit User-Agent: Mutt/1.2.5i In-Reply-To: <3EABE658.9080500@niski.com>; from fabio@niski.com on Sun, Apr 27, 2003 at 07:16:56AM -0700 Sender: owner-obm-l@sucuri.mat.puc-rio.br Precedence: bulk Reply-To: obm-l@mat.puc-rio.br On Sun, Apr 27, 2003 at 07:16:56AM -0700, niski wrote: > > 2) Onduletas eh dose! Mas ondaletas eh comum e aceitavel. > > Desculpe, me equivoquei, onduletas é em espanhol. De qualquer forma eu > prefiro muito mais Wavelets. Acho que deve ser traduzidos os > substantivos por exemplo multiplicadores ao inves de multipliers, > wavelets significa pequena ondas...ondaletas eu nao encontrei em nenhum > dicionario. Há quem defenda o uso da palavra 'marola'. []s, N. ========================================================================= Instruções para entrar na lista, sair da lista e usar a lista em http://www.mat.puc-rio.br/~nicolau/olimp/obm-l.html ========================================================================= From owner-obm-l@sucuri.mat.puc-rio.br Mon Apr 28 09:00:43 2003 Return-Path: Received: (from majordom@localhost) by sucuri.mat.puc-rio.br (8.9.3/8.9.3) id IAA26010 for obm-l-MTTP; Mon, 28 Apr 2003 08:58:11 -0300 Received: (from nicolau@localhost) by sucuri.mat.puc-rio.br (8.9.3/8.9.3) id IAA26005 for obm-l@mat.puc-rio.br; Mon, 28 Apr 2003 08:58:10 -0300 Date: Mon, 28 Apr 2003 08:58:10 -0300 From: "Nicolau C. Saldanha" To: obm-l@mat.puc-rio.br Subject: Re: [obm-l] Falha nossa Message-ID: <20030428085810.C25567@sucuri.mat.puc-rio.br> References: <20030426104039.A8053@sucuri.mat.puc-rio.br> Mime-Version: 1.0 Content-Type: text/plain; charset=iso-8859-1 Content-Disposition: inline Content-Transfer-Encoding: 8bit User-Agent: Mutt/1.2.5i In-Reply-To: <20030426104039.A8053@sucuri.mat.puc-rio.br>; from nicolau@sucuri.mat.puc-rio.br on Sat, Apr 26, 2003 at 10:40:39AM -0300 Sender: owner-obm-l@sucuri.mat.puc-rio.br Precedence: bulk Reply-To: obm-l@mat.puc-rio.br On Sat, Apr 26, 2003 at 10:40:39AM -0300, Nicolau C. Saldanha wrote: > On Fri, Apr 25, 2003 at 01:29:25PM +0000, Antonio Neto wrote: > > Falei besteira, Igor. Nao reparei que era a soma, e troquei pela > > determinacao do termo geral. O que eu disse aplica-se ao termo geral, mas > > nao aa soma. Para tirar a dúvida, fui aos arquivos da lista para pegar os > > valores de a, b, c, d e e. Escrevi o polinomio e calculei S(-1)= 1 e S(-2)= > > 0, o que nao faz sentido, pelo menos para mim. Desculpem a falha, abracos, > > olavo. > > Para mim faz todo o sentido falar em S(n) para qualquer inteiro. > Temos S(1) = 1^3 = 1 > S(2) = 1^3 + 2^3 = 9 > S(3) = 1^3 + 2^3 + 3^3 = 36 > > A propriedade importante é S(n+1) = S(n) + (n+1)^3 o que, > junto com S(0) = 0, define S. > > Mas se você desejar uma interpretação mais explícita, tome > > S(n) = 1^3 + 2^3 + ... + n^3, n >= 1 > S(n) = - ( (-1)^3 + (-2)^3 + ... + (n+1)^3 ), n < -1 Pensando mais no assunto achei que isso pode não ter ficado muito claro, vou dar outra explicação. A notação de Iverson é [frase] = 1, se 'frase' é verdadeira 0, se 'frase' é falsa. Usando esta notação temos S(n) = soma_k ([k > 0] - [k > n]) (k^3) onde o somatório é tomado sobre todos os inteiros k. É claro que só um número finito de termos é não nula. Esta definição vale para qualquer valor inteiro n e com ela é bem claro que S(n+1) = S(n) + (n+1)^3. []s, N. ========================================================================= Instruções para entrar na lista, sair da lista e usar a lista em http://www.mat.puc-rio.br/~nicolau/olimp/obm-l.html ========================================================================= From owner-obm-l@sucuri.mat.puc-rio.br Mon Apr 28 09:45:23 2003 Return-Path: Received: (from majordom@localhost) by sucuri.mat.puc-rio.br (8.9.3/8.9.3) id JAA27875 for obm-l-MTTP; Mon, 28 Apr 2003 09:42:07 -0300 Received: from hotmail.com (bay1-f21.bay1.hotmail.com [65.54.245.21]) by sucuri.mat.puc-rio.br (8.9.3/8.9.3) with ESMTP id JAA27870 for ; Mon, 28 Apr 2003 09:42:03 -0300 Received: from mail pickup service by hotmail.com with Microsoft SMTPSVC; Mon, 28 Apr 2003 05:41:31 -0700 Received: from 146.164.44.51 by by1fd.bay1.hotmail.msn.com with HTTP; Mon, 28 Apr 2003 12:41:31 GMT X-Originating-IP: [146.164.44.51] X-Originating-Email: [marcelo_souza7@hotmail.com] From: "Marcelo Souza" To: obm-l@mat.puc-rio.br Subject: Re: [obm-l] calculo c Date: Mon, 28 Apr 2003 12:41:31 +0000 Mime-Version: 1.0 Content-Type: text/plain; format=flowed Message-ID: X-OriginalArrivalTime: 28 Apr 2003 12:41:31.0651 (UTC) FILETIME=[7ED8A530:01C30D83] Sender: owner-obm-l@sucuri.mat.puc-rio.br Precedence: bulk Reply-To: obm-l@mat.puc-rio.br >From: "Marcelo" >Reply-To: obm-l@mat.puc-rio.br >To: >Subject: [obm-l] calculo c >Date: Sun, 27 Apr 2003 19:07:17 -0300 > >olá pessoal... > >não estou conseguindo provar essas questões: > >a) x³-y³ = (x-y)(x² + xy + y²) Escreva x^3-y^3 = (x-y)^3+3x^2y-3xy^2 = (x-y)^3+3xy(x-y)= =(x-y)(x^2-2xy+y^2+3xy)=(x-y)(x^2+xy+y^2) ok? >b) x^n - y^n = (x-y)(x^n-1 + x^n-2*y + ... + xy^n-2 + y^n-1) Aqui faz por induçâo em n >c) x³+y³ = (x+y)(x² - xy + y²) Aqui vou deixar como exercício já que é análogo ao primeiro []'s, Marcelo. >--- >Outgoing mail is certified Virus Free. >Checked by AVG anti-virus system (http://www.grisoft.com). >Version: 6.0.474 / Virus Database: 272 - Release Date: 18/4/2003 _________________________________________________________________ Add photos to your e-mail with MSN 8. Get 2 months FREE*. http://join.msn.com/?page=features/featuredemail ========================================================================= Instruções para entrar na lista, sair da lista e usar a lista em http://www.mat.puc-rio.br/~nicolau/olimp/obm-l.html ========================================================================= From owner-obm-l@sucuri.mat.puc-rio.br Mon Apr 28 10:02:40 2003 Return-Path: Received: (from majordom@localhost) by sucuri.mat.puc-rio.br (8.9.3/8.9.3) id JAA28347 for obm-l-MTTP; Mon, 28 Apr 2003 09:59:11 -0300 Received: from ivoti.terra.com.br (ivoti.terra.com.br [200.176.3.20]) by sucuri.mat.puc-rio.br (8.9.3/8.9.3) with ESMTP id JAA28343 for ; Mon, 28 Apr 2003 09:59:06 -0300 Received: from canela.terra.com.br (canela.terra.com.br [200.176.3.79]) by ivoti.terra.com.br (Postfix) with ESMTP id B42124082C9 for ; Mon, 28 Apr 2003 09:58:27 -0300 (BRT) Received: from [200.177.179.14] (dl-nas3-sao-C8B1B30E.p001.terra.com.br [200.177.179.14]) by canela.terra.com.br (Postfix) with ESMTP id 7E57B2240BA for ; Mon, 28 Apr 2003 09:58:26 -0300 (BRT) User-Agent: Microsoft-Outlook-Express-Macintosh-Edition/5.02.2022 Date: Mon, 28 Apr 2003 09:59:54 -0300 Subject: Re: [obm-l] RE: [obm-l] RE: [obm-l] Indu=?ISO-8859-1?B?5+M=?=o Finita From: Claudio Buffara To: Message-ID: In-Reply-To: <001c01c30d3a$e60aaf30$9865fea9@computer> Mime-version: 1.0 Content-type: text/plain; charset="ISO-8859-1" Content-Transfer-Encoding: 8bit X-MIME-Autoconverted: from quoted-printable to 8bit by sucuri.mat.puc-rio.br id JAA28344 Sender: owner-obm-l@sucuri.mat.puc-rio.br Precedence: bulk Reply-To: obm-l@mat.puc-rio.br Oi, Artur: De qualquer forma, vale o mesmo raciocinio: o maior divisor impar de 21 eh 21 e nao 7. Um abraco, Claudio. on 28.04.03 01:01, Artur Costa Steiner at artur_steiner@usa.net wrote: > Oi Claudio, > Houve um problema de grafia no email. Na realidade o ponto que eu > coloquei, dando a ideia de 21 .21 = 21 X 21 =441 nao era um sinal > matematico, mas sim o sinal de pontuacao indicando final de sentenca.... > Eu quis dizer 21, enao 441 (risos...) > Um abraco > Artur > >> Oi, Artur: >> >> O maior divisor impar de 21*21 = 441 eh 441. >> >> Assim, teremos: 441 = 2^0 * 441. >> >> 7 eh o maior divisor PRIMO de 441. >> >> Um abraco, >> Claudio. > > ========================================================================= > Instruções para entrar na lista, sair da lista e usar a lista em > http://www.mat.puc-rio.br/~nicolau/olimp/obm-l.html > ========================================================================= > ========================================================================= Instruções para entrar na lista, sair da lista e usar a lista em http://www.mat.puc-rio.br/~nicolau/olimp/obm-l.html ========================================================================= From owner-obm-l@sucuri.mat.puc-rio.br Mon Apr 28 10:29:23 2003 Return-Path: Received: (from majordom@localhost) by sucuri.mat.puc-rio.br (8.9.3/8.9.3) id KAA29420 for obm-l-MTTP; Mon, 28 Apr 2003 10:26:25 -0300 Received: from Euler.impa.br (euler.impa.br [147.65.1.3]) by sucuri.mat.puc-rio.br (8.9.3/8.9.3) with ESMTP id KAA29395 for ; Mon, 28 Apr 2003 10:26:16 -0300 Received: from sbm.impa.br (obm-01.impa.br [147.65.2.170]) by Euler.impa.br (8.11.6p2/8.11.6) with ESMTP id h3SDPj012145 for ; Mon, 28 Apr 2003 10:25:45 -0300 (EST) Message-Id: <5.0.0.25.0.20030428222543.00a0a4b0@pop.impa.br> X-Sender: obm@pop.impa.br X-Mailer: QUALCOMM Windows Eudora Version 5.0 Date: Mon, 28 Apr 2003 22:26:32 -0700 To: obm-l@mat.puc-rio.br From: Olimpiada Brasileira de Matematica Subject: [obm-l] Equipe Brasil - XIV Cone Sul. Mime-Version: 1.0 Content-Type: text/plain; charset="us-ascii"; format=flowed Sender: owner-obm-l@sucuri.mat.puc-rio.br Precedence: bulk Reply-To: obm-l@mat.puc-rio.br Caros(as) amigos(as) da lista: Equipe de alunos que representara o Brasil na XIV Olimpiada de Matematica do Cone Sul: BRA 1: Fabio Dias Moreira (Rio de Janeiro - RJ) BRA 2: Henry Wei Cheng Hsu (Sao Paulo - SP) BRA 3: Rodrigo Aguiar Pinheiro (Fortaleza - CE) BRA 4: Thiago Costa Leite Santos (Sao Paulo - SP) Abracos, Nelly. ========================================================================= Instruções para entrar na lista, sair da lista e usar a lista em http://www.mat.puc-rio.br/~nicolau/olimp/obm-l.html ========================================================================= From owner-obm-l@sucuri.mat.puc-rio.br Mon Apr 28 10:48:48 2003 Return-Path: Received: (from majordom@localhost) by sucuri.mat.puc-rio.br (8.9.3/8.9.3) id KAA30269 for obm-l-MTTP; Mon, 28 Apr 2003 10:45:10 -0300 Received: from cmsrelay01.mx.net (cmsrelay01.mx.net [165.212.11.110]) by sucuri.mat.puc-rio.br (8.9.3/8.9.3) with SMTP id KAA30265 for ; Mon, 28 Apr 2003 10:45:05 -0300 Received: from uadvg128.cms.usa.net (165.212.11.128) by cmsoutbound.mx.net with SMTP; 28 Apr 2003 13:44:33 -0000 Received: from uwdvg016.cms.usa.net [165.212.8.16] by uadvg128.cms.usa.net (ASMTP/) via mtad (C8.MAIN.3.05) with ESMTP id 400HDbNSE0459M28; Mon, 28 Apr 2003 13:44:31 GMT Received: from 200.181.4.100 [200.181.4.100] by uwdvg016.cms.usa.net (USANET web-mailer CM.0402.5.2B); Mon, 28 Apr 2003 13:44:29 -0000 Date: Mon, 28 Apr 2003 10:44:29 -0300 From: Artur Costa Steiner To: Subject: [obm-l] =?ISO-8859-1?Q?Re=3A=20=5BRe=3A=20=5Bobm=2Dl=5D=20RE=3A=20=5Bobm?= =?ISO-8859-1?Q?=2Dl=5D=20RE=3A=20=5Bobm=2Dl=5D=20Indu=E7=E3o?= =?ISO-8859-1?Q?=09Finita=5D?= X-Mailer: USANET web-mailer (CM.0402.5.2B) Mime-Version: 1.0 Message-ID: <154HDbNSd7488S16.1051537469@uwdvg016.cms.usa.net> Content-Type: text/plain; charset=ISO-8859-1 Content-Transfer-Encoding: 8bit X-MIME-Autoconverted: from quoted-printable to 8bit by sucuri.mat.puc-rio.br id KAA30266 Sender: owner-obm-l@sucuri.mat.puc-rio.br Precedence: bulk Reply-To: obm-l@mat.puc-rio.br Claudio Buffara wrote: > Oi, Artur: > > De qualquer forma, vale o mesmo raciocinio: o maior divisor impar de 21 eh > 21 e nao 7. > > Um abraco, > Claudio. > > on 28.04.03 01:01, Artur Costa Steiner at artur_steiner@usa.net wrote: > > > Oi Claudio, > > Houve um problema de grafia no email. Na realidade o ponto que eu > > coloquei, dando a ideia de 21 .21 = 21 X 21 =441 nao era um sinal > > matematico, mas sim o sinal de pontuacao indicando final de sentenca.... > > Eu quis dizer 21, enao 441 (risos...) > > Um abraco > > Artur > > > >> Oi, Artur: > >> > >> O maior divisor impar de 21*21 = 441 eh 441. > >> > >> Assim, teremos: 441 = 2^0 * 441. > >> > >> 7 eh o maior divisor PRIMO de 441. > >> > >> Um abraco, > >> Claudio. Ah, ta certo, eu estava, na minha cabeca, com a ideia de que era o maior fator primo. Excesso de trabalho Um abraco Artur ========================================================================= Instruções para entrar na lista, sair da lista e usar a lista em http://www.mat.puc-rio.br/~nicolau/olimp/obm-l.html ========================================================================= From owner-obm-l@sucuri.mat.puc-rio.br Mon Apr 28 12:58:50 2003 Return-Path: Received: (from majordom@localhost) by sucuri.mat.puc-rio.br (8.9.3/8.9.3) id MAA01265 for obm-l-MTTP; Mon, 28 Apr 2003 12:54:53 -0300 Received: from sporus.bol.com.br (sporus.bol.com.br [200.221.24.23]) by sucuri.mat.puc-rio.br (8.9.3/8.9.3) with ESMTP id MAA01260 for ; Mon, 28 Apr 2003 12:54:36 -0300 Received: from bol.com.br (200.221.24.131) by sporus.bol.com.br (5.1.071) id 3EA97E7200099EA8 for obm-l@mat.puc-rio.br; Mon, 28 Apr 2003 12:54:01 -0300 Date: Mon, 28 Apr 2003 12:54:01 -0300 Message-Id: Subject: [obm-l] Provas Militares MIME-Version: 1.0 Content-Type: text/plain;charset="iso-8859-1" From: "basketboy_igor" To: obm-l@mat.puc-rio.br X-XaM3-API-Version: 2.4 R3 ( B4 ) X-SenderIP: 200.217.136.220 Content-Transfer-Encoding: 8bit X-MIME-Autoconverted: from quoted-printable to 8bit by sucuri.mat.puc-rio.br id MAA01262 Sender: owner-obm-l@sucuri.mat.puc-rio.br Precedence: bulk Reply-To: obm-l@mat.puc-rio.br Saldações à todos, Estou com algumas dúvidas em: 1°) Calculo I i) (EN/95) O lim [raiz(x+b)+raiz(x+a)-raiz(b)-raiz(a)]/x x -> 0 é? resp: 1/[2*raiz(b)] + 1/[2*raiz(a)] ii)(EN/95) Se f(x) = e^(2x) + (x+1)*cos(x), então f’(0) é igual a: resp: 3. iii)(EN/97) O valor da integral de raiz[1 + 9*x*(dx)] é: a)(2/27)*(1+9*x)^(3/2) + c iv) (EN/98) A equação do movimento de um progétil que se desloca ao longo do eixo x é x(t) = {e^[-(t – Pi/4)]}*sen (t) + cotg²(t), t >= 0. A aceleração do projétil no instante t=0 é: Resp: d) 16 –2*raiz(2) v) (MM/98) Sendo A = Lim (x -> 0) {2*raiz[x*sen(6x)]}/ {[cossec(6x)]*[1 - cos²(6x)]} e B = Lim (x-> log 2 na base 3) [2^(2x+1], [(A²)*B]/2 vale: Resp: b) 6 __________________________________________________________________________ Seleção de Softwares UOL. 10 softwares escolhidos pelo UOL para você e sua família. http://www.uol.com.br/selecao ========================================================================= Instruções para entrar na lista, sair da lista e usar a lista em http://www.mat.puc-rio.br/~nicolau/olimp/obm-l.html ========================================================================= From owner-obm-l@sucuri.mat.puc-rio.br Mon Apr 28 13:06:21 2003 Return-Path: Received: (from majordom@localhost) by sucuri.mat.puc-rio.br (8.9.3/8.9.3) id NAA01413 for obm-l-MTTP; Mon, 28 Apr 2003 13:03:44 -0300 Received: from sporus.bol.com.br (sporus.bol.com.br [200.221.24.23]) by sucuri.mat.puc-rio.br (8.9.3/8.9.3) with ESMTP id NAA01404 for ; Mon, 28 Apr 2003 13:03:28 -0300 Received: from bol.com.br (200.221.24.131) by sporus.bol.com.br (5.1.071) id 3EA97E720009A4E2 for obm-l@mat.puc-rio.br; Mon, 28 Apr 2003 13:02:54 -0300 Date: Mon, 28 Apr 2003 13:02:54 -0300 Message-Id: Subject: [obm-l] =?iso-8859-1?q?D=FAvidas?= MIME-Version: 1.0 Content-Type: text/plain;charset="iso-8859-1" From: "basketboy_igor" To: obm-l@mat.puc-rio.br X-XaM3-API-Version: 2.4 R3 ( B4 ) X-SenderIP: 200.217.136.220 Content-Transfer-Encoding: 8bit X-MIME-Autoconverted: from quoted-printable to 8bit by sucuri.mat.puc-rio.br id NAA01410 Sender: owner-obm-l@sucuri.mat.puc-rio.br Precedence: bulk Reply-To: obm-l@mat.puc-rio.br (IMO/1996) Let ABCDEF be a convex hexagon such that AB||DB, BC||///ef and CD||AF. Let Ra, Rc, Re denote the circumradii or triangles FAB, BCD, DEF, respectively, and let P denote the perimeter or hexagon. Prova that Ra + Rc + Re >= P/2. (Vietnam) Solve the system of equations raiz(3x)*[1+1/(x+y)]=2 riaz(7y)*[1-1/(x+y)]=4*raiz(2) (Vietnam) Let a,b,c,d be four nonnegative real numbers satisfying the condition 2(ab + ac + ad + bc+ bd + cd) + abc + abd + acd + bcd = 16. Prove that a + b + c + d >= (2/3)*(ab + ac + ad + bc + bd + cd) and determine when equality occurs. __________________________________________________________________________ Seleção de Softwares UOL. 10 softwares escolhidos pelo UOL para você e sua família. http://www.uol.com.br/selecao ========================================================================= Instruções para entrar na lista, sair da lista e usar a lista em http://www.mat.puc-rio.br/~nicolau/olimp/obm-l.html ========================================================================= From owner-obm-l@sucuri.mat.puc-rio.br Mon Apr 28 13:13:38 2003 Return-Path: Received: (from majordom@localhost) by sucuri.mat.puc-rio.br (8.9.3/8.9.3) id NAA01761 for obm-l-MTTP; Mon, 28 Apr 2003 13:10:51 -0300 Received: from web12906.mail.yahoo.com (web12906.mail.yahoo.com [216.136.174.73]) by sucuri.mat.puc-rio.br (8.9.3/8.9.3) with SMTP id NAA01757 for ; Mon, 28 Apr 2003 13:10:47 -0300 Message-ID: <20030428161015.814.qmail@web12906.mail.yahoo.com> Received: from [200.206.103.3] by web12906.mail.yahoo.com via HTTP; Mon, 28 Apr 2003 13:10:15 ART Date: Mon, 28 Apr 2003 13:10:15 -0300 (ART) From: =?iso-8859-1?q?Johann=20Peter=20Gustav=20Lejeune=20Dirichlet?= Subject: Re: [obm-l] Problema do Dirichlet To: obm-l@mat.puc-rio.br In-Reply-To: <010401c30b58$22b942a0$3300c57d@bovespa.com> MIME-Version: 1.0 Content-Type: multipart/alternative; boundary="0-888249960-1051546215=:97373" Content-Transfer-Encoding: 8bit Sender: owner-obm-l@sucuri.mat.puc-rio.br Precedence: bulk Reply-To: obm-l@mat.puc-rio.br --0-888249960-1051546215=:97373 Content-Type: text/plain; charset=iso-8859-1 Content-Transfer-Encoding: 8bit Valeu cara!!!!!!!!Bem,se e assim...Alias,eu curto um pouco de braço as vezes.Devia ter pensado em vetores mas tudo bem,ce fez o trabalho sujo pra mim.MUITO OBRIGADO!!!!!!Agora brinque um pouco com essas duas aplicaçoes. "Uma reta contendo o incentro de um triangulo bissecta a area se e somente se bissecta o perimetro do triangulo." a segunda eu mando depois... Ass.:Johann Cláudio_(Prática) wrote:Ok, seu Dirichlet: Como você está se compromentando publicamente a ter nexo, relevância e educação (pelo menos nas suas mensagens pra lista), eu vou cumprir minha parte do trato. Aqui vai: O problema: Considere o triangulo ABC e um ponto T. Duas cevianas CT_c e BT_b se cortam em T e os pontos R_c,R_b e T sao alinhados,com R_c em AB e R_b em AC. Mostre que: (AT_c * BR_c) / (BT_c * AR_c) + ( AT_b * CR_b ) / (CT_b * AR_b ) = 1 ***** Solução usando vetores: Tomando A como origem, sejam U e V vetores unitários nas direções de AB e AC, respectivamente. Como ABC é não-degenerado, U e V são L.I. Assim, existem números reais h, k, a, b, c, d tais que: AB = hU, AC = kV, AT_c = aU, AR_c = bU, AT_b = cV, AR_b = dV Como | U | = | V | = 1, podemos re-escrever escrever a expressão do enunciado em função apenas de h, k, a, b, c, d. Assim: (AT_c * BR_c) / (BT_c * AR_c) + ( AT_b * CR_b ) / (CT_b * AR_b ) = = (a*(h - b))/((h - a)*b) + (c*(k - d))/((k - c)*d) = = (ha - ab)/(hb - ab) + (kc - cd)/(kd - cd) = = NUM / DEN onde: NUM = hkad - hacd - kabd + abcd + hkbc - hbcd - kabc + abcd e DEN = hkbd - hbcd - kabd + abcd Temos que provar que NUM / DEN = 1, ou seja, que NUM = DEN. Em outras palavras, basta provar que: NUM - DEN = abcd - hacd - kabc + hkad + hkbc - hkbd = 0 ***** Vamos agora à geometria: R_c, T, R_b são colineares ==> existe um no. real x tal que: AT = x*AR_c + (1-x)*AR_b ==> AT = xbU + (1-x)dV (1) B, T, T_b são colineares ==> existe um no. real y tal que: AT = y*AB + (1-y)*AT_b ==> AT = yhU + (1-y)cV (2) C, T, T_c são colineares ==> existe um no. real z tal que: AT = z*AC + (1-z)*AT_c ==> AT = zkV + (1-z)aU (3) (1) e (2) ==> xbU + (1-x)dV = yhU + (1-y)cV ==> (xb - yh)U + ((1-x)d - (1-y)c)V = 0 (1) e (3) ==> xbU + (1-x)dV = zkV + (1-z)aU ==> (xb - (1-z)a)U + ((1-x)d - zk)V = 0 Como U e V são LI, podemos concluir que: xb - yh = 0 (1-x)d - (1-y)c = 0 xb - (1-z)a = 0 (1-x)d - zk = 0 Ou seja, rearranjando: bx - hy = 0 -dx + cy = c - d bx + az = a dx + kz = d Resolvendo para x, achamos que: x = (hc - hd)/(bc - hd) e x = (ka - ad)/(kb - ad) Igualando estas expressões, multiplicando e simplificando, caímos em: abcd - hacd - kabc + hkad + hkbc - hkbd = 0. No entanto, esta expressão é precisamente igual a NUM - DEN (Lembre-se: NUM - DEN = abcd - hacd - kabc + hkad + hkbc - hkbd). Logo, NUM - DEN = 0 ==> NUM = DEN ==> NUM / DEN = 1 ==> acabou!!! ***** Repare que, apesar da álgebra meio braçal no final, a idéia central da solução é bastante simples, o que ilustra (espero) o grande poder do método vetorial em geometria. Pense na sua dificuldade em resolver este problema via Menelaus ou algum outro teorema mais elegante... Um abraço, Claudio. TRANSIRE SVVM PECTVS MVNDOQVE POTIRI CONGREGATI EX TOTO ORBE MATHEMATICI OB SCRIPTA INSIGNIA TRIBVERE Fields Medal(John Charles Fields) --------------------------------- Yahoo! Mail O melhor e-mail gratuito da internet: 6MB de espaço, antivírus, acesso POP3, filtro contra spam. --0-888249960-1051546215=:97373 Content-Type: text/html; charset=iso-8859-1 Content-Transfer-Encoding: 8bit

Valeu cara!!!!!!!!Bem,se e assim...Alias,eu curto um pouco de braço as vezes.Devia ter pensado em vetores mas tudo bem,ce fez o trabalho sujo pra mim.MUITO OBRIGADO!!!!!!Agora brinque um pouco com essas duas aplicaçoes.

"Uma reta contendo o incentro de um triangulo bissecta a area se e somente se bissecta o perimetro do triangulo."

a segunda eu mando depois...

Ass.:Johann

 Cláudio_(Prática) <claudio@praticacorretora.com.br> wrote:

Ok, seu Dirichlet:
 
Como você está se compromentando publicamente a ter nexo, relevância e educação (pelo menos nas suas mensagens pra lista), eu vou cumprir minha parte do trato. Aqui vai:
 
O problema:
 
Considere o triangulo ABC e um ponto T. Duas cevianas CT_c e BT_b se cortam
em T e os pontos R_c,R_b e T sao alinhados,com R_c em AB e R_b em AC.
Mostre que:

(AT_c * BR_c) / (BT_c * AR_c)  +  ( AT_b * CR_b ) / (CT_b * AR_b ) = 1

*****

Solução usando vetores:

Tomando A como origem, sejam U e V vetores unitários nas direções de AB e AC, respectivamente.
Como ABC é não-degenerado, U e V são L.I.

Assim, existem números reais h, k, a, b, c, d tais que:
AB = hU, AC = kV, AT_c = aU, AR_c = bU, AT_b = cV, AR_b = dV

Como | U | = | V | = 1, podemos re-escrever escrever a expressão do enunciado em função
apenas de h, k, a, b, c, d.
Assim:
(AT_c * BR_c) / (BT_c * AR_c)  +  ( AT_b * CR_b ) / (CT_b * AR_b ) =
= (a*(h - b))/((h - a)*b) + (c*(k - d))/((k - c)*d) =
= (ha - ab)/(hb - ab) + (kc - cd)/(kd - cd) =
 
= NUM / DEN
onde:
NUM = hkad - hacd - kabd + abcd + hkbc - hbcd - kabc + abcd
e
DEN = hkbd - hbcd - kabd + abcd

Temos que provar que NUM / DEN = 1, ou seja, que NUM = DEN.
Em outras palavras, basta provar que:
NUM - DEN = abcd - hacd - kabc + hkad + hkbc - hkbd = 0

*****

Vamos agora à geometria:

R_c, T, R_b são colineares ==>
existe um no. real x tal que: AT = x*AR_c + (1-x)*AR_b ==>
AT = xbU + (1-x)dV  (1)

B, T, T_b são colineares ==>
existe um no. real y tal que: AT = y*AB + (1-y)*AT_b ==>
AT = yhU + (1-y)cV  (2)

C, T, T_c são colineares ==>
existe um no. real z tal que: AT = z*AC + (1-z)*AT_c ==>
AT = zkV + (1-z)aU  (3)

(1) e (2) ==> xbU + (1-x)dV = yhU + (1-y)cV ==>
(xb - yh)U + ((1-x)d - (1-y)c)V = 0

(1) e (3) ==> xbU + (1-x)dV = zkV + (1-z)aU ==>
(xb - (1-z)a)U + ((1-x)d - zk)V = 0

Como U e V são LI, podemos concluir que:
xb - yh = 0
(1-x)d - (1-y)c = 0
xb - (1-z)a = 0
(1-x)d - zk = 0

Ou seja, rearranjando:
bx - hy = 0
-dx + cy = c - d
bx + az = a
dx + kz = d

Resolvendo para x, achamos que:
x = (hc - hd)/(bc - hd)
e
x = (ka - ad)/(kb - ad)

Igualando estas expressões, multiplicando e simplificando, caímos em:
abcd - hacd - kabc + hkad + hkbc - hkbd = 0.

No entanto, esta expressão é precisamente igual a NUM - DEN
(Lembre-se: NUM - DEN = abcd - hacd - kabc + hkad + hkbc - hkbd).

Logo, NUM - DEN = 0  ==>
NUM = DEN  ==> 
NUM / DEN = 1  ==> 
acabou!!!
*****
Repare que, apesar da álgebra meio braçal no final, a idéia central da solução é bastante simples, o que ilustra (espero) o grande poder do método vetorial em geometria. Pense na sua dificuldade em resolver este problema via Menelaus ou algum outro teorema mais elegante...
 
 
Um abraço,
Claudio.


TRANSIRE SVVM PECTVS MVNDOQVE POTIRI

CONGREGATI EX TOTO ORBE MATHEMATICI OB SCRIPTA INSIGNIA TRIBVERE

Fields Medal(John Charles Fields)



Yahoo! Mail
O melhor e-mail gratuito da internet: 6MB de espaço, antivírus, acesso POP3, filtro contra spam. --0-888249960-1051546215=:97373-- ========================================================================= Instruções para entrar na lista, sair da lista e usar a lista em http://www.mat.puc-rio.br/~nicolau/olimp/obm-l.html ========================================================================= From owner-obm-l@sucuri.mat.puc-rio.br Mon Apr 28 13:27:31 2003 Return-Path: Received: (from majordom@localhost) by sucuri.mat.puc-rio.br (8.9.3/8.9.3) id NAA02348 for obm-l-MTTP; Mon, 28 Apr 2003 13:24:33 -0300 Received: from ns3bind.localdomain ([200.230.34.5]) by sucuri.mat.puc-rio.br (8.9.3/8.9.3) with ESMTP id NAA02344 for ; Mon, 28 Apr 2003 13:24:29 -0300 Received: from servico2 ([200.230.34.227]) by ns3bind.localdomain (8.11.6/X.XX.X) with SMTP id h3SGJeO05055 for ; Mon, 28 Apr 2003 13:19:40 -0300 Message-ID: <01b101c30da2$c39b3e20$3300c57d@bovespa.com> From: "=?iso-8859-1?Q?Cl=E1udio_\=28Pr=E1tica\=29?=" To: References: <003201c30cc4$56050a00$e8c797c8@computador> <00d301c30cef$05231c50$176df1c8@itchy> Subject: [obm-l] Basquete Bizantino Date: Mon, 28 Apr 2003 13:25:14 -0300 MIME-Version: 1.0 Content-Type: text/plain; charset="iso-8859-1" Content-Transfer-Encoding: 8bit X-Priority: 3 X-MSMail-Priority: Normal X-Mailer: Microsoft Outlook Express 5.50.4920.2300 X-MimeOLE: Produced By Microsoft MimeOLE V5.50.4920.2300 Sender: owner-obm-l@sucuri.mat.puc-rio.br Precedence: bulk Reply-To: obm-l@mat.puc-rio.br Caros colegas da lista: Esse problema das notas e do troco tem uma variante interessante e um pouco mais difícil: O jogo de basquete bizantino é igual ao basquete normal, exceto que cestas curtas valem "a" pontos ao invés de 2 pontos e cestas de longa distância valem "b" pontos ao invés de 3 pontos. No basquete bizantino existem exatamente 35 pontuações que nunca ocorrem numa partida. Uma delas é 58. Quanto valem "a" e "b"? Um abraço, Claudio. ----- Original Message ----- From: "David Ricardo" To: Sent: Sunday, April 27, 2003 3:58 PM Subject: [obm-l] Re: [obm-l] Indução Finita > 3) Mostre que é possível pagar, sem receber troco, qualquer quantia inteira > de reais, maior do que 7, com notas de 3 reais e 5 reais. ========================================================================= Instruções para entrar na lista, sair da lista e usar a lista em http://www.mat.puc-rio.br/~nicolau/olimp/obm-l.html ========================================================================= From owner-obm-l@sucuri.mat.puc-rio.br Mon Apr 28 14:15:53 2003 Return-Path: Received: (from majordom@localhost) by sucuri.mat.puc-rio.br (8.9.3/8.9.3) id OAA04194 for obm-l-MTTP; Mon, 28 Apr 2003 14:12:30 -0300 Received: from ns3bind.localdomain ([200.230.34.5]) by sucuri.mat.puc-rio.br (8.9.3/8.9.3) with ESMTP id OAA04190 for ; Mon, 28 Apr 2003 14:12:26 -0300 Received: from servico2 ([200.230.34.227]) by ns3bind.localdomain (8.11.6/X.XX.X) with SMTP id h3SH7cO08461 for ; Mon, 28 Apr 2003 14:07:38 -0300 Message-ID: <01c301c30da9$76e418c0$3300c57d@bovespa.com> From: "=?iso-8859-1?Q?Cl=E1udio_\=28Pr=E1tica\=29?=" To: References: Subject: [obm-l] =?iso-8859-1?Q?Re:_=5Bobm-l=5D_D=FAvidas?= Date: Mon, 28 Apr 2003 14:13:17 -0300 MIME-Version: 1.0 Content-Type: text/plain; charset="iso-8859-1" Content-Transfer-Encoding: 8bit X-Priority: 3 X-MSMail-Priority: Normal X-Mailer: Microsoft Outlook Express 5.50.4920.2300 X-MimeOLE: Produced By Microsoft MimeOLE V5.50.4920.2300 Sender: owner-obm-l@sucuri.mat.puc-rio.br Precedence: bulk Reply-To: obm-l@mat.puc-rio.br Oi, Igor: Uma solução pro primeiro você encontra em: http://www.kalva.demon.co.uk/imo/isoln/isoln965.html Aliás, esse site é um dos melhores sobre olimpíadas de matemática em geral. Os outros dois já foram resolvidos recentemente pelo Gugu aqui na lista. O arquivo de mensagens está em: http://www.mat.puc-rio.br/~nicolau/olimp/obm-l.html Um abraço, Claudio. ----- Original Message ----- From: "basketboy_igor" To: Sent: Monday, April 28, 2003 1:02 PM Subject: [obm-l] Dúvidas > (IMO/1996) Let ABCDEF be a convex hexagon such that > AB||DB, BC||///ef and CD||AF. Let Ra, Rc, Re denote the > circumradii or triangles FAB, BCD, DEF, respectively, > and let P denote the perimeter or hexagon. Prova that > Ra + Rc + Re >= P/2. > > (Vietnam) Solve the system of equations > raiz(3x)*[1+1/(x+y)]=2 > riaz(7y)*[1-1/(x+y)]=4*raiz(2) > > (Vietnam) Let a,b,c,d be four nonnegative real numbers > satisfying the condition 2(ab + ac + ad + bc+ bd + cd) > + abc + abd + acd + bcd = 16. > Prove that a + b + c + d >= (2/3)*(ab + ac + ad + bc + > bd + cd) and determine when equality occurs. > ========================================================================= Instruções para entrar na lista, sair da lista e usar a lista em http://www.mat.puc-rio.br/~nicolau/olimp/obm-l.html ========================================================================= From owner-obm-l@sucuri.mat.puc-rio.br Mon Apr 28 14:29:00 2003 Return-Path: Received: (from majordom@localhost) by sucuri.mat.puc-rio.br (8.9.3/8.9.3) id OAA04538 for obm-l-MTTP; Mon, 28 Apr 2003 14:26:15 -0300 Received: from ns3bind.localdomain ([200.230.34.5]) by sucuri.mat.puc-rio.br (8.9.3/8.9.3) with ESMTP id OAA04524 for ; Mon, 28 Apr 2003 14:26:09 -0300 Received: from servico2 ([200.230.34.229]) by ns3bind.localdomain (8.11.6/X.XX.X) with SMTP id h3SHLIO09443 for ; Mon, 28 Apr 2003 14:21:18 -0300 Message-ID: <01e601c30dab$5fa31d80$3300c57d@bovespa.com> From: "=?iso-8859-1?Q?Cl=E1udio_\=28Pr=E1tica\=29?=" To: References: <20030428161015.814.qmail@web12906.mail.yahoo.com> Subject: Re: [obm-l] Problema do Dirichlet Date: Mon, 28 Apr 2003 14:26:57 -0300 MIME-Version: 1.0 Content-Type: multipart/alternative; boundary="----=_NextPart_000_01E3_01C30D92.3984D9E0" X-Priority: 3 X-MSMail-Priority: Normal X-Mailer: Microsoft Outlook Express 5.50.4920.2300 X-MimeOLE: Produced By Microsoft MimeOLE V5.50.4920.2300 Sender: owner-obm-l@sucuri.mat.puc-rio.br Precedence: bulk Reply-To: obm-l@mat.puc-rio.br This is a multi-part message in MIME format. ------=_NextPart_000_01E3_01C30D92.3984D9E0 Content-Type: text/plain; charset="iso-8859-1" Content-Transfer-Encoding: quoted-printable Oi, JP: Foi um prazer ajudar! Uma solu=E7=E3o do da reta contendo o incentro eu mandei no dia 25/04, = apesar de n=E3o usar o resultado do outro problema. Checa no site do Nicolau. Um abra=E7o, Claudio. ----- Original Message -----=20 From: Johann Peter Gustav Lejeune Dirichlet=20 To: obm-l@mat.puc-rio.br=20 Sent: Monday, April 28, 2003 1:10 PM Subject: Re: [obm-l] Problema do Dirichlet Valeu cara!!!!!!!!Bem,se e assim...Alias,eu curto um pouco de bra=E7o = as vezes.Devia ter pensado em vetores mas tudo bem,ce fez o trabalho = sujo pra mim.MUITO OBRIGADO!!!!!!Agora brinque um pouco com essas duas = aplica=E7oes.=20 "Uma reta contendo o incentro de um triangulo bissecta a area se e = somente se bissecta o perimetro do triangulo."=20 a segunda eu mando depois...=20 Ass.:Johann=20 ------=_NextPart_000_01E3_01C30D92.3984D9E0 Content-Type: text/html; charset="iso-8859-1" Content-Transfer-Encoding: quoted-printable
Oi, JP:
 
Foi um prazer ajudar!
 
Uma solu=E7=E3o do da reta contendo o = incentro eu=20 mandei no dia 25/04, apesar de n=E3o usar o resultado do outro=20 problema.
Checa no site do Nicolau.
 
Um abra=E7o,
Claudio.
 
----- Original Message -----
From:=20 Johann Peter Gustav = Lejeune=20 Dirichlet
Sent: Monday, April 28, 2003 = 1:10=20 PM
Subject: Re: [obm-l] Problema = do=20 Dirichlet

Valeu cara!!!!!!!!Bem,se e assim...Alias,eu curto um pouco de = bra=E7o as=20 vezes.Devia ter pensado em vetores mas tudo bem,ce fez o trabalho sujo = pra=20 mim.MUITO OBRIGADO!!!!!!Agora brinque um pouco com essas duas = aplica=E7oes.=20

"Uma reta contendo o incentro de um triangulo bissecta a area se e = somente=20 se bissecta o perimetro do triangulo."=20

a segunda eu mando depois...=20

Ass.:Johann

------=_NextPart_000_01E3_01C30D92.3984D9E0-- ========================================================================= Instruções para entrar na lista, sair da lista e usar a lista em http://www.mat.puc-rio.br/~nicolau/olimp/obm-l.html ========================================================================= From owner-obm-l@sucuri.mat.puc-rio.br Mon Apr 28 14:56:05 2003 Return-Path: Received: (from majordom@localhost) by sucuri.mat.puc-rio.br (8.9.3/8.9.3) id OAA05517 for obm-l-MTTP; Mon, 28 Apr 2003 14:52:45 -0300 Received: from hotmail.com (bay2-dav34.bay2.hotmail.com [65.54.246.91]) by sucuri.mat.puc-rio.br (8.9.3/8.9.3) with ESMTP id OAA05512 for ; Mon, 28 Apr 2003 14:52:41 -0300 Received: from mail pickup service by hotmail.com with Microsoft SMTPSVC; Mon, 28 Apr 2003 10:52:09 -0700 Received: from 200.151.183.227 by bay2-dav34.bay2.hotmail.com with DAV; Mon, 28 Apr 2003 17:52:09 +0000 X-Originating-IP: [200.151.183.227] X-Originating-Email: [marcelo_rufino@hotmail.com] From: "Marcelo Rufino de Oliveira" To: References: <20030427132914.40459.qmail@web80513.mail.yahoo.com> Subject: Re: [obm-l] ajuda com desigualdade Date: Mon, 14 Aug 2000 07:55:39 -0300 MIME-Version: 1.0 Content-Type: text/plain; charset="iso-8859-1" Content-Transfer-Encoding: 8bit X-Priority: 3 X-MSMail-Priority: Normal X-Mailer: Microsoft Outlook Express 5.00.2919.6700 X-MimeOLE: Produced By Microsoft MimeOLE V5.00.2919.6700 Message-ID: X-OriginalArrivalTime: 28 Apr 2003 17:52:09.0449 (UTC) FILETIME=[E3D6ED90:01C30DAE] Sender: owner-obm-l@sucuri.mat.puc-rio.br Precedence: bulk Reply-To: obm-l@mat.puc-rio.br Esta é meio braçal. Tira o mínimo e depois multiplique cruzado. Depois de fazer umas mil contas e cortes possíveis, a desigualdade se transforma em: a^6.b^3 + a^6.c^3 + b^6.a^3 + b^6.c^3 + c^6.b^3 + c^6.a^3 >= 2(a^3 + b^3 + c^3)(a^2.b^2.c^2) Vamos provar agora que ----- Original Message ----- From: "guilherme S." To: Sent: Sunday, April 27, 2003 10:29 AM Subject: [obm-l] ajuda com desigualdade > prove que: > 1/(a^3+b^3+abc)+1/(a^3+c^3+abc)+1/(b^3+c^3+abc)<=1/abc > sendo a,b e c reais positivos > > _______________________________________________________________________ > Yahoo! Mail > O melhor e-mail gratuito da internet: 6MB de espaço, antivírus, acesso POP3, filtro contra spam. > http://br.mail.yahoo.com/ > ========================================================================= > Instruções para entrar na lista, sair da lista e usar a lista em > http://www.mat.puc-rio.br/~nicolau/olimp/obm-l.html > ========================================================================= > ========================================================================= Instruções para entrar na lista, sair da lista e usar a lista em http://www.mat.puc-rio.br/~nicolau/olimp/obm-l.html ========================================================================= From owner-obm-l@sucuri.mat.puc-rio.br Mon Apr 28 15:36:30 2003 Return-Path: Received: (from majordom@localhost) by sucuri.mat.puc-rio.br (8.9.3/8.9.3) id PAA06698 for obm-l-MTTP; Mon, 28 Apr 2003 15:32:23 -0300 Received: from web14304.mail.yahoo.com (web14304.mail.yahoo.com [216.136.173.80]) by sucuri.mat.puc-rio.br (8.9.3/8.9.3) with SMTP id PAA06692 for ; Mon, 28 Apr 2003 15:32:19 -0300 Message-ID: <20030428183144.20738.qmail@web14304.mail.yahoo.com> Received: from [200.144.49.39] by web14304.mail.yahoo.com via HTTP; Mon, 28 Apr 2003 15:31:44 ART Date: Mon, 28 Apr 2003 15:31:44 -0300 (ART) From: =?iso-8859-1?q?Rafael?= Subject: [obm-l] 3 circunferências To: OBM MIME-Version: 1.0 Content-Type: text/plain; charset=iso-8859-1 Content-Transfer-Encoding: 8bit Sender: owner-obm-l@sucuri.mat.puc-rio.br Precedence: bulk Reply-To: obm-l@mat.puc-rio.br Olá pessoal! Será que alguém consegue me dar uma ajuda nessa questão: Três circunferência de raios r , r' e R são tangentes, duas a duas, externamente. A tangente comun interna às duas primeiras circunferências intercepta a circunferência de raio R nos pontos A e B. Calcular a corda AB. resp: (4R raiz(rr')/(r + r') Abraços, Rafael. _______________________________________________________________________ Yahoo! Mail O melhor e-mail gratuito da internet: 6MB de espaço, antivírus, acesso POP3, filtro contra spam. http://br.mail.yahoo.com/ ========================================================================= Instruções para entrar na lista, sair da lista e usar a lista em http://www.mat.puc-rio.br/~nicolau/olimp/obm-l.html ========================================================================= From owner-obm-l@sucuri.mat.puc-rio.br Mon Apr 28 16:27:04 2003 Return-Path: Received: (from majordom@localhost) by sucuri.mat.puc-rio.br (8.9.3/8.9.3) id QAA08044 for obm-l-MTTP; Mon, 28 Apr 2003 16:23:51 -0300 Received: from salem.bol.com.br (salem.bol.com.br [200.221.24.25]) by sucuri.mat.puc-rio.br (8.9.3/8.9.3) with ESMTP id QAA08038 for ; Mon, 28 Apr 2003 16:23:42 -0300 Received: from bol.com.br (200.221.24.128) by salem.bol.com.br (5.1.071) id 3EA6DA33001767CE for obm-l@mat.puc-rio.br; Mon, 28 Apr 2003 16:23:07 -0300 Date: Mon, 28 Apr 2003 16:23:07 -0300 Message-Id: Subject: Re:[obm-l] complexos MIME-Version: 1.0 Content-Type: text/plain;charset="iso-8859-1" From: "rmr-olimp" To: obm-l@mat.puc-rio.br X-XaM3-API-Version: 2.4 R3 ( B4 ) X-SenderIP: 200.206.193.76 Content-Transfer-Encoding: 8bit X-MIME-Autoconverted: from quoted-printable to 8bit by sucuri.mat.puc-rio.br id QAA08041 Sender: owner-obm-l@sucuri.mat.puc-rio.br Precedence: bulk Reply-To: obm-l@mat.puc-rio.br E aí Rafael... De uma olhada no dite abaixo: www.educ.fc.ul.pt/icm/icm2000/icm26/complexos.htm lá explica tudo de forma clara e com desenhos. Para resover a sua questão basta calcular a raiz quintupla de z, ou seja, de ( -1 + i.raiz(3) ). vc não precisará encontar as raízes, apenas saber onde os afixos se localizam, fique atento a isso. Espero ter ajudado.. Rodrigo > Olá Pessoal! > > Não sei como resolver essa questão de complexos: > Representando as raízes da equação: > x^5 = -1 + i.raiz(3) > > no plano complexo, temos dois afixos distintos no: > a) eixo real > b) eixo imaginário > c) 2° quadrante > d) 3° quadrante > e) 4° quadrante > > Na verdade eu queria saber mais sobre representar as > raízes reais de uma equação com números complexos como > vértices de um polígono regular também, não sei se é > esse o caso... > > Abraços, > > Rafael. > __________________________________________________________________________ Seleção de Softwares UOL. 10 softwares escolhidos pelo UOL para você e sua família. http://www.uol.com.br/selecao ========================================================================= Instruções para entrar na lista, sair da lista e usar a lista em http://www.mat.puc-rio.br/~nicolau/olimp/obm-l.html ========================================================================= From owner-obm-l@sucuri.mat.puc-rio.br Mon Apr 28 17:15:50 2003 Return-Path: Received: (from majordom@localhost) by sucuri.mat.puc-rio.br (8.9.3/8.9.3) id QAA08667 for obm-l-MTTP; Mon, 28 Apr 2003 16:56:43 -0300 Received: from cmsrelay05.mx.net (cmsrelay05.mx.net [165.212.11.2]) by sucuri.mat.puc-rio.br (8.9.3/8.9.3) with SMTP id QAA08658 for ; Mon, 28 Apr 2003 16:56:37 -0300 Received: from uadvg130.cms.usa.net (165.212.11.130) by cmsoutbound.mx.net with SMTP; 28 Apr 2003 19:56:03 -0000 Received: from uwdvg001.cms.usa.net [165.212.8.11] by uadvg130.cms.usa.net (ASMTP/) via mtad (C8.MAIN.3.05) with ESMTP id 477HDbT5B0316M30; Mon, 28 Apr 2003 19:56:01 GMT Received: from 200.181.4.100 [200.181.4.100] by uwdvg001.cms.usa.net (USANET web-mailer CM.0402.5.2B); Mon, 28 Apr 2003 19:55:59 -0000 Date: Mon, 28 Apr 2003 16:55:59 -0300 From: Artur Costa Steiner To: Subject: Re: [[obm-l] 3 circunfer-ncias] X-Mailer: USANET web-mailer (CM.0402.5.2B) Mime-Version: 1.0 Message-ID: <785HDbT485744S01.1051559759@uwdvg001.cms.usa.net> Content-Type: text/plain; charset=ISO-8859-1 Content-Transfer-Encoding: 8bit X-MIME-Autoconverted: from quoted-printable to 8bit by sucuri.mat.puc-rio.br id QAA08663 Sender: owner-obm-l@sucuri.mat.puc-rio.br Precedence: bulk Reply-To: obm-l@mat.puc-rio.br Rafael wrote: Eu estou consideravelmente enferrujado em Geometria, mas uma sugestao que talvez sirva para alguma coisa eh observar que o centro radical dos 3 circulos eh colinear com a corda AB. Artur > Olá pessoal! > > Será que alguém consegue me dar uma ajuda nessa > questão: > > Três circunferência de raios r , r' e R são tangentes, > duas a duas, externamente. A tangente comun interna às > duas primeiras circunferências intercepta a > circunferência de raio R nos pontos A e B. Calcular a > corda AB. > > resp: (4R raiz(rr')/(r + r') > > Abraços, > > Rafael. ========================================================================= Instruções para entrar na lista, sair da lista e usar a lista em http://www.mat.puc-rio.br/~nicolau/olimp/obm-l.html ========================================================================= From owner-obm-l@sucuri.mat.puc-rio.br Mon Apr 28 17:30:08 2003 Return-Path: Received: (from majordom@localhost) by sucuri.mat.puc-rio.br (8.9.3/8.9.3) id RAA09475 for obm-l-MTTP; Mon, 28 Apr 2003 17:27:12 -0300 Received: from paiol.terra.com.br (paiol.terra.com.br [200.176.3.18]) by sucuri.mat.puc-rio.br (8.9.3/8.9.3) with ESMTP id RAA09470 for ; Mon, 28 Apr 2003 17:27:08 -0300 Received: from botucatu.terra.com.br (botucatu.terra.com.br [200.176.3.78]) by paiol.terra.com.br (Postfix) with ESMTP id C46DF87FBF for ; Mon, 28 Apr 2003 17:26:37 -0300 (BRT) Received: from nt (RJ231083.user.veloxzone.com.br [200.165.231.83]) (authenticated user ensr) by botucatu.terra.com.br (Postfix) with ESMTP id 8D6E729C072 for ; Mon, 28 Apr 2003 17:26:33 -0300 (BRT) Message-ID: <01cc01c30dc4$506d5740$5400a8c0@ensrbr> From: "Luis Lopes" To: References: <200304270935.h3R9ZTxs012032@trex.centroin.com.br> <3EABE658.9080500@niski.com> <20030428085044.B25567@sucuri.mat.puc-rio.br> Subject: [obm-l] termos matematicos [era: Limites] Date: Mon, 28 Apr 2003 17:25:21 -0300 MIME-Version: 1.0 Content-Type: text/plain; charset="iso-8859-1" Content-Transfer-Encoding: 8bit X-Priority: 3 X-MSMail-Priority: Normal X-Mailer: Microsoft Outlook Express 5.50.4807.1700 X-MimeOLE: Produced By Microsoft MimeOLE V5.50.4807.1700 Sender: owner-obm-l@sucuri.mat.puc-rio.br Precedence: bulk Reply-To: obm-l@mat.puc-rio.br Sauda,c~oes, Bissectar, intersectar, 'marola' (!!?? não teria coragem de usar este termo se não o tivesse visto num texto antes)... Sou então levado a perguntar como devemos dizer "generating function" em particular e generating em outros contextos como probabilidade. Não tem algo como função geratriz de momento? Tenho usado função geratriz no contexto de função que gera uma seqüência mas acho que função geradora é possível e até preferível?!. Num contexto próximo, vimos recentemente uma discussão sobre resolver uma desigualdade. Já há algum tempo fico implicando mentalmente e me perguntando qual a diferença ou também como usar termos como equação, igualdade, identidade inequação e desigualdade Dizemos resolva a equação e não, resolva a igualdade. Deveríamos então dizer resolva a inequação. No momento não me ocorrem outros exemplos mas que há outros, ah isto há. Embora on topic (oups, pertinente), desculpem pela mensagem um pouco cri-cri []'s Luís -----Mensagem Original----- De: "Nicolau C. Saldanha" Para: Enviada em: segunda-feira, 28 de abril de 2003 08:50 Assunto: Re: [obm-l] Limites > On Sun, Apr 27, 2003 at 07:16:56AM -0700, niski wrote: > > > 2) Onduletas eh dose! Mas ondaletas eh comum e aceitavel. > > > > Desculpe, me equivoquei, onduletas é em espanhol. De qualquer forma eu > > prefiro muito mais Wavelets. Acho que deve ser traduzidos os > > substantivos por exemplo multiplicadores ao inves de multipliers, > > wavelets significa pequena ondas...ondaletas eu nao encontrei em nenhum > > dicionario. > > Há quem defenda o uso da palavra 'marola'. > > []s, N. > ========================================================================= Instruções para entrar na lista, sair da lista e usar a lista em http://www.mat.puc-rio.br/~nicolau/olimp/obm-l.html ========================================================================= From owner-obm-l@sucuri.mat.puc-rio.br Mon Apr 28 17:47:09 2003 Return-Path: Received: (from majordom@localhost) by sucuri.mat.puc-rio.br (8.9.3/8.9.3) id RAA10089 for obm-l-MTTP; Mon, 28 Apr 2003 17:44:21 -0300 Received: from videira.terra.com.br (videira.terra.com.br [200.176.3.30]) by sucuri.mat.puc-rio.br (8.9.3/8.9.3) with ESMTP id RAA10074 for ; Mon, 28 Apr 2003 17:44:02 -0300 Received: from ivoti.terra.com.br (ivoti.terra.com.br [200.176.3.20]) by videira.terra.com.br (Postfix) with ESMTP id 8745A33B7B6 for ; Mon, 28 Apr 2003 16:56:56 -0300 (BRT) Received: from canela.terra.com.br (canela.terra.com.br [200.176.3.79]) by ivoti.terra.com.br (Postfix) with ESMTP id 6B8194080B3 for ; Mon, 28 Apr 2003 16:55:56 -0300 (BRT) Received: from nt (RJ231083.user.veloxzone.com.br [200.165.231.83]) (authenticated user ensr) by canela.terra.com.br (Postfix) with ESMTP id 416052240F4 for ; Mon, 28 Apr 2003 16:55:55 -0300 (BRT) Message-ID: <01b801c30dc0$0788e5c0$5400a8c0@ensrbr> From: "Luis Lopes" To: References: <003201c30cc4$56050a00$e8c797c8@computador> <00d301c30cef$05231c50$176df1c8@itchy> Subject: [obm-l] =?iso-8859-1?Q?Re:_=5Bobm-l=5D_Re:_=5Bobm-l=5D_Indu=E7=E3o_Finita?= Date: Mon, 28 Apr 2003 16:54:49 -0300 MIME-Version: 1.0 Content-Type: text/plain; charset="iso-8859-1" Content-Transfer-Encoding: 8bit X-Priority: 3 X-MSMail-Priority: Normal X-Mailer: Microsoft Outlook Express 5.50.4807.1700 X-MimeOLE: Produced By Microsoft MimeOLE V5.50.4807.1700 Sender: owner-obm-l@sucuri.mat.puc-rio.br Precedence: bulk Reply-To: obm-l@mat.puc-rio.br Sauda,c~oes, Os 4 problemas desta discussão estão resolvidos no livro "Manual de Indução Matemática". Ver www.escolademestres.com/qedtexte A solução abaixo é muito parecida com a do livro. []'s Luís -----Mensagem Original----- De: "David Ricardo" Para: Enviada em: domingo, 27 de abril de 2003 15:58 Assunto: [obm-l] Re: [obm-l] Indução Finita > 3) Mostre que é possível pagar, sem receber troco, qualquer quantia inteira > de reais, maior do que 7, com notas de 3 reais e 5 reais. > > Defina o predicado P(k) como sendo 'é possível pagar uma quantia k com notas > de 3 e 5 reais'. > > Mostre que P(8) é verdadeiro (3 + 5 = 8). Assuma que P(k) é verdadeiro e > tente provar que se P(k) é verdadeiro, P(k+1) também é para os inteiros > maiores que 7. > > Para provar isso, veja os seguintes casos: > > 1) Se existe uma nota de 5 reais fazendo parte da quantia k, troque os 5 > reais por duas notas de três reais para obter a quantia k+1. > 2) Se não tiver nenhuma nota de 5 reais, deve haver pelo menos 3 notas de > três reais na quantia k (para satisfazer a condição k > 7). Troque essas > três notas de três reais por duas notas de 5 reais para obter a quantia k+1. > > Então é possível a partir de uma quantidade k se chegar a uma quantidade k+1 > com notas de 3 e 5 reais para k > 7, portanto o predicado é verdadeiro pra > todo k inteiro maior que 7. > > Não sei se é a melhor solução para o problema, mas eu acho que tá certo. > > []s > David ========================================================================= Instruções para entrar na lista, sair da lista e usar a lista em http://www.mat.puc-rio.br/~nicolau/olimp/obm-l.html ========================================================================= From owner-obm-l@sucuri.mat.puc-rio.br Mon Apr 28 18:17:00 2003 Return-Path: Received: (from majordom@localhost) by sucuri.mat.puc-rio.br (8.9.3/8.9.3) id SAA11518 for obm-l-MTTP; Mon, 28 Apr 2003 18:13:56 -0300 Received: from ns3bind.localdomain ([200.230.34.5]) by sucuri.mat.puc-rio.br (8.9.3/8.9.3) with ESMTP id SAA11514 for ; Mon, 28 Apr 2003 18:13:51 -0300 Received: from servico2 ([200.230.34.229]) by ns3bind.localdomain (8.11.6/X.XX.X) with SMTP id h3SL92O25173 for ; Mon, 28 Apr 2003 18:09:02 -0300 Message-ID: <02e401c30dcb$305c8dc0$3300c57d@bovespa.com> From: "=?iso-8859-1?Q?Cl=E1udio_\=28Pr=E1tica\=29?=" To: References: <20030428183144.20738.qmail@web14304.mail.yahoo.com> Subject: [obm-l] =?iso-8859-1?Q?Re:_=5Bobm-l=5D_3_circunfer=EAncias?= Date: Mon, 28 Apr 2003 18:14:42 -0300 MIME-Version: 1.0 Content-Type: text/plain; charset="iso-8859-1" Content-Transfer-Encoding: 8bit X-Priority: 3 X-MSMail-Priority: Normal X-Mailer: Microsoft Outlook Express 5.50.4920.2300 X-MimeOLE: Produced By Microsoft MimeOLE V5.50.4920.2300 Sender: owner-obm-l@sucuri.mat.puc-rio.br Precedence: bulk Reply-To: obm-l@mat.puc-rio.br ----- Original Message ----- From: "Rafael" To: "OBM" Sent: Monday, April 28, 2003 3:31 PM Subject: [obm-l] 3 circunferências > Olá pessoal! > > Será que alguém consegue me dar uma ajuda nessa > questão: > > Três circunferência de raios r , r' e R são tangentes, > duas a duas, externamente. A tangente comun interna às > duas primeiras circunferências intercepta a > circunferência de raio R nos pontos A e B. Calcular a > corda AB. > > resp: (4R raiz(rr')/(r + r') > Oi, Rafael: Como geometria não é o meu forte, aqui vai uma solução horrorosa, usando geometria analítica: Circunferencia de raio r: Centro em (-r,0): (x+r)^2 + y^2 = r^2 Circunferencia de raio r': Centro em (r',0): (x-r')^2 + y^2 = r'^2 Equação da reta tangente comum interna: x = 0 (a tangente é o eixo y). Equação da circunferência de raio R: (x-a)^2 + (y-b)^2 = R^2 (a,b): coordenadas do centro (a determinar) Como as circunferências são tangentes, temos que a distância entre os centros é igual à soma dos raios: (a+r)^2 + b^2 = (R+r)^2 (a-r')^2 + b^2 = (R+r')^2 Subtraindo e simplificando, obtemos: a = R(r - r')/(r + r') Para achra b, vou calcular de 2 formas a área do triangulo formado pelos 3 centros: (r+r')*|b|/2 = raiz((r+r'+R)*R*r*r') (formula de Heron) ==> b = + ou - 2*raiz((r+r'+R)*R*r*r')/(r+r') Ou seja, a equação da 3a. circunferência é (supondo s.p.d.g. que b > 0): (x - R(r-r')/(r+r'))^2 + (y - 2*raiz((r+r'+R)*R*r*r')/(r+r'))^2 = R^2 Fazendo x = 0, teremos as ordenadas dos dois pontos de interseção: (y - 2*raiz((r+r'+R)*R*r*r')/(r+r'))^2 = R^2 - R^2(r-r')^2/(r+r')^2 ==> (y - 2*raiz((r+r'+R)*R*r*r')/(r+r'))^2 = 4*R^2*r*r'/(r+r')^2 ==> y = 2*raiz((r+r'+R)*R*r*r')/(r+r') +ou- 2*R*raiz(r*r')/(r+r') Subtraindo os dois valores de y, obtemos o comprimento de AB: m(AB) = 4*R*raiz(r*r')/(r+r') Um abraço, Claudio. ========================================================================= Instruções para entrar na lista, sair da lista e usar a lista em http://www.mat.puc-rio.br/~nicolau/olimp/obm-l.html ========================================================================= From owner-obm-l@sucuri.mat.puc-rio.br Mon Apr 28 18:41:01 2003 Return-Path: Received: (from majordom@localhost) by sucuri.mat.puc-rio.br (8.9.3/8.9.3) id SAA12496 for obm-l-MTTP; Mon, 28 Apr 2003 18:38:17 -0300 Received: from itaqui.terra.com.br (itaqui.terra.com.br [200.176.3.19]) by sucuri.mat.puc-rio.br (8.9.3/8.9.3) with ESMTP id SAA12492 for ; Mon, 28 Apr 2003 18:38:13 -0300 Received: from barra.terra.com.br (barra.terra.com.br [200.176.3.52]) by itaqui.terra.com.br (Postfix) with ESMTP id 03F1D3BC1FF for ; Mon, 28 Apr 2003 18:37:42 -0300 (BRT) Received: from nt (RJ231083.user.veloxzone.com.br [200.165.231.83]) (authenticated user ensr) by barra.terra.com.br (Postfix) with ESMTP id 7A95723407C for ; Mon, 28 Apr 2003 18:37:40 -0300 (BRT) Message-ID: <020601c30dce$3fa7ee20$5400a8c0@ensrbr> From: "Luis Lopes" To: References: <200304270416.h3R4GKp10274@Gauss.impa.br> Subject: Re: [obm-l] serie do Marcio Date: Mon, 28 Apr 2003 18:36:34 -0300 MIME-Version: 1.0 Content-Type: text/plain; charset="iso-8859-1" Content-Transfer-Encoding: 8bit X-Priority: 3 X-MSMail-Priority: Normal X-Mailer: Microsoft Outlook Express 5.50.4807.1700 X-MimeOLE: Produced By Microsoft MimeOLE V5.50.4807.1700 Sender: owner-obm-l@sucuri.mat.puc-rio.br Precedence: bulk Reply-To: obm-l@mat.puc-rio.br Sauda,c~oes, Apesar de certa (vou acreditar no teorema do Gugu que não conhecia e não entendi a exposição, mas não importa), acredito que não seja a maneira correta de calcular a soma. Devemos usar as somas parciais em termos dos números harmônicos e daí calcular o limite. Senão vejamos (mas vou apelar pro LaTeX pois é copiar/colar de outra msg): Seja H_n = \sum_{k=1}^n \frac{1}{k} . S_n = \sum_{k=0}^n ( \frac{1}{4k+1} + \frac{1}{4k+3} - \frac{1}{2(k+1)} ) = 1 + \frac{1}{3} + \cdots + \frac{1}{4n+1} + \frac{1}{4n+3} - \frac{H_{n+1}}{2} = H_{4n+3} - \frac{H_{2n+1}}{2} - \frac{H_{n+1}}{2} . Sabe-se que (pelo menos para quem quer calcular tal soma) H_n = \log n + \gamma + o(1); (leia isto como H_n \approx \log n + \gamma para n muito grande e H_n = \log n + \gamma quando n\to\infty). onde \log é o logaritmo natural e \gamma=0.57721566490153286 ... é a constante de Euler. Então usando \log(4n+3) = \log( 4n(1 + \frac{3}{4n}) ) = \log 4 + \log n + \log(1 + 3/(4n)) = \log 4 + \log n + o(1) ...., obtemos S_n = \log(4n+3) + \gamma - \frac{\log(2n+1) + \log(n+1) + 2\gamma}{2} + o(1) = \log 4 - \frac{\log 2}{2} + o(1) = \frac{3 \log 2}{2} + o(1). Logo, \lim_{n \to\infty} S_n = \frac{3 \log 2}{2}. cqd É complicado, mas é assim que tem que ser feito! []'s Luís -----Mensagem Original----- De: "Carlos Gustavo Tamm de Araujo Moreira" Para: Enviada em: domingo, 27 de abril de 2003 01:16 Assunto: Re: [obm-l] serie do Marcio > Oi Claudio, > Sua prova esta' certa, e segue da seguinte proposicao facil de provar: > se soma(n=1 a infinito)(a_n) converge a A e soma(n=1 a infinito)(b_n) > converge a B entao > a_1+a_2+...+a_k(1)+b_1+a_(k(1)+1)+a_(k(1)+2)+...+a_k(2)+b_2+... > converge a A+B, para qualquer sequencia crescente de inteiros positivos > k(n) (note que os termos de a(n) e de b(n) estao entrando na ordem original > e se somamos muitos termos da terceira serie estamos somando muitos termos > da primeira serie e muitos termos da segunda serie). > Abracos, > Gugu > > > >Oi, Marcio e Luis: > > > >Realmente eh um problema intrigante. Empiricamente (com uma planilha) eu me > >convenci que a soma eh 1,5*Ln(2). > > > >Alem disso, vejam esse algebrismo: > > > > Ln(2) = 1-1/2+1/3-1/4+1/5-1/6+1/7-1/8+1/9-1/10+1/11-1/12+1/13-1/14+... > > > >0,5*Ln(2) = 1/2 -1/4 +1/6 -1/8 +1/10 -1/12 +1/14 - ... > > > >Somando: > >1,5*Ln(2) = 1 +1/3-1/2+1/5 +1/7-1/4+1/9 +1/11-1/6+1/3 + ... > > > >Rearranjando os termos: > >1,5*Ln(2) = (1+1/3-1/2) + (1/5+1/7-1/4) + (1/9+1/11-1/6) + ... > > > >Ou seja, > >1,5*Ln(2) = SOMA(n >= 0) [1/(4n+1) + 1/(4n+3) - 1/(2n+2)] > > > >Assim, o algebrismo funciona apesar de eu nao ter certeza do rigor, dado que > >estamos lidando com uma serie condicionalmente convergente. > > > >Pergunta: o que eu fiz pode ser justificado com rigor ou eu achei o > >resultado correto por pura sorte? > > > >Um abraco, > >Claudio. > > ========================================================================= Instruções para entrar na lista, sair da lista e usar a lista em http://www.mat.puc-rio.br/~nicolau/olimp/obm-l.html ========================================================================= From owner-obm-l@sucuri.mat.puc-rio.br Mon Apr 28 19:38:19 2003 Return-Path: Received: (from majordom@localhost) by sucuri.mat.puc-rio.br (8.9.3/8.9.3) id TAA13854 for obm-l-MTTP; Mon, 28 Apr 2003 19:35:17 -0300 Received: from fnn.net ([200.175.38.9]) by sucuri.mat.puc-rio.br (8.9.3/8.9.3) with SMTP id TAA13848 for ; Mon, 28 Apr 2003 19:35:13 -0300 Received: (qmail 3295 invoked from network); 28 Apr 2003 22:19:55 -0000 Received: from unknown (HELO windows98) (200.175.39.117) by mail.fnn.net with SMTP; 28 Apr 2003 22:19:55 -0000 Message-ID: <002601c30dd7$aaeb8f80$9a75fea9@windows98> From: "Daniel Pini" To: Subject: [obm-l] geometria Date: Mon, 28 Apr 2003 19:44:01 -0300 MIME-Version: 1.0 Content-Type: multipart/alternative; boundary="----=_NextPart_000_0023_01C30DBE.84B35B40" X-Priority: 3 X-MSMail-Priority: Normal X-Mailer: Microsoft Outlook Express 5.00.2615.200 X-MimeOLE: Produced By Microsoft MimeOLE V5.00.2615.200 Sender: owner-obm-l@sucuri.mat.puc-rio.br Precedence: bulk Reply-To: obm-l@mat.puc-rio.br This is a multi-part message in MIME format. ------=_NextPart_000_0023_01C30DBE.84B35B40 Content-Type: text/plain; charset="iso-8859-1" Content-Transfer-Encoding: quoted-printable Num c=EDrculo de raio igual a 12 est=E1 inscrito um triangulo ABC cujos = lados AB e AC medem 8 e 9, respectivamente. A altura relativa ao lado = BC =E9 igual a? Se um trapezio retangulo tem diagonais perpendiculars e bases = iguais(??), a sua altura =E9 igual a? R:18 Estes problemas foram retirados do Geometria2 do Morgado. ------=_NextPart_000_0023_01C30DBE.84B35B40 Content-Type: text/html; charset="iso-8859-1" Content-Transfer-Encoding: quoted-printable
Num c=EDrculo de raio igual a 12  = est=E1 inscrito=20 um triangulo ABC cujos lados AB e AC medem  8 e 9, respectivamente. = A=20 altura  relativa ao lado BC =E9 igual a?
Se um trapezio retangulo tem diagonais = perpendiculars e=20 bases iguais(??), a sua altura =E9 igual a? R:18
 
Estes problemas foram retirados do Geometria2 do = Morgado.
------=_NextPart_000_0023_01C30DBE.84B35B40-- ========================================================================= Instruções para entrar na lista, sair da lista e usar a lista em http://www.mat.puc-rio.br/~nicolau/olimp/obm-l.html ========================================================================= From owner-obm-l@sucuri.mat.puc-rio.br Mon Apr 28 19:44:05 2003 Return-Path: Received: (from majordom@localhost) by sucuri.mat.puc-rio.br (8.9.3/8.9.3) id TAA14091 for obm-l-MTTP; Mon, 28 Apr 2003 19:41:34 -0300 Received: from traven9.uol.com.br (traven9.uol.com.br [200.221.29.35]) by sucuri.mat.puc-rio.br (8.9.3/8.9.3) with ESMTP id TAA14074 for ; Mon, 28 Apr 2003 19:41:21 -0300 Received: from gauss ([200.158.97.150]) by traven9.uol.com.br (8.9.1/8.9.1) with SMTP id TAA25376 for ; Mon, 28 Apr 2003 19:40:50 -0300 (BRT) Message-ID: <006601c30dd7$fec4dcb0$96619ec8@gauss> From: "Domingos Jr." To: References: <003201c30cc4$56050a00$e8c797c8@computador> <00d301c30cef$05231c50$176df1c8@itchy> <01b101c30da2$c39b3e20$3300c57d@bovespa.com> Subject: Re: [obm-l] Basquete Bizantino Date: Mon, 28 Apr 2003 19:46:22 -0300 MIME-Version: 1.0 Content-Type: text/plain; charset="iso-8859-1" Content-Transfer-Encoding: 8bit X-Priority: 3 X-MSMail-Priority: Normal X-Mailer: Microsoft Outlook Express 6.00.2800.1158 X-MimeOLE: Produced By Microsoft MimeOLE V6.00.2800.1165 Sender: owner-obm-l@sucuri.mat.puc-rio.br Precedence: bulk Reply-To: obm-l@mat.puc-rio.br > O jogo de basquete bizantino é igual ao basquete normal, exceto que cestas > curtas valem "a" pontos ao invés de 2 pontos e cestas de longa distância > valem "b" pontos ao invés de 3 pontos. No basquete bizantino existem > exatamente 35 pontuações que nunca ocorrem numa partida. Uma delas é 58. > Quanto valem "a" e "b"? os pontos possíveis são { aX + bY, X, Y >= 0 inteiros } se 58 não é uma pontuação possível, temos necessariamente que nem a nem b dividem 58, ou seja: a, b != { 1, 2, 29, 58 } se existe um número finito de pontuações que nunca são obtidas temos que existe um inteiro N tal que, para todo n > N, existem X, Y tal que n = aX + bY seja n + 1 = aX' + bY' 1 = a(X'-X) + b(Y'-Y) isso nos diz que mdc(a, b) = 1 assuma a < b (sem perda de generalidade) aX + bY = aX' + bY' <=> a(X - X') = b(Y'-Y) mas então b|(X - X') e a|(Y - Y') a.X + b.Y tem então valores distintos para 0 <= X <= b-1, 0 <= Y <= a-1 ou seja há exatamente a.b pontuações distintas entre { 0, 1, ..., 2ab - a - b } 2ab - a - b + 1 - ab <= 35 ab - a - b <= 34 a >= 3 pois a != { 1, 2 } ab - a - b <= 34 => b(a-1) <= 34 + a b <= (34 + a)/(a-1) dá pra limitar o problema a: podemos ver que a = { 3, 4, 5, 6 } pois se a = 7, b <= (34 + 7)/6 = 41/6 < 7 existem 20 possíveis valores de (a, b) que satisfazem a desigualdade acima e mdc(a, b)= 1, talvez dê pra rodar um programinha e verificar qual dessas 20 comb. tem a propriedade desejada. [ ]'s ========================================================================= Instruções para entrar na lista, sair da lista e usar a lista em http://www.mat.puc-rio.br/~nicolau/olimp/obm-l.html ========================================================================= From owner-obm-l@sucuri.mat.puc-rio.br Mon Apr 28 20:45:52 2003 Return-Path: Received: (from majordom@localhost) by sucuri.mat.puc-rio.br (8.9.3/8.9.3) id UAA16283 for obm-l-MTTP; Mon, 28 Apr 2003 20:42:16 -0300 Received: from ivoti.terra.com.br (ivoti.terra.com.br [200.176.3.20]) by sucuri.mat.puc-rio.br (8.9.3/8.9.3) with ESMTP id UAA16278 for ; Mon, 28 Apr 2003 20:42:12 -0300 Received: from araci.terra.com.br (araci.terra.com.br [200.176.3.44]) by ivoti.terra.com.br (Postfix) with ESMTP id 9D9CC4080F2 for ; Mon, 28 Apr 2003 20:41:42 -0300 (BRT) Received: from nt (RJ231083.user.veloxzone.com.br [200.165.231.83]) (authenticated user ensr) by araci.terra.com.br (Postfix) with ESMTP id 3225421EF8E for ; Mon, 28 Apr 2003 20:41:42 -0300 (BRT) Message-ID: <025c01c30ddf$933fb5c0$5400a8c0@ensrbr> From: "Luis Lopes" To: References: <20030426104039.A8053@sucuri.mat.puc-rio.br> <20030428085810.C25567@sucuri.mat.puc-rio.br> Subject: Re: [obm-l] Falha nossa Date: Mon, 28 Apr 2003 20:40:36 -0300 MIME-Version: 1.0 Content-Type: text/plain; charset="iso-8859-1" Content-Transfer-Encoding: 8bit X-Priority: 3 X-MSMail-Priority: Normal X-Mailer: Microsoft Outlook Express 5.50.4807.1700 X-MimeOLE: Produced By Microsoft MimeOLE V5.50.4807.1700 Sender: owner-obm-l@sucuri.mat.puc-rio.br Precedence: bulk Reply-To: obm-l@mat.puc-rio.br Sauda,c~oes, Oi Nicolau, Sabemos que S(0)=0 e subindo uma linha na tabela de diferenças encontramos S(-1), S(-2), etc. Estamos falando de PA de ordem k e em particular k=3. \Delta^{-1} a_i = S(i) \Delta^{0} a_i = a_i Assim faz sentido S(n) para TODO n. Mas não sabia dar uma interpretação para a soma quando n<= 0. Na verdade é convencionado que S(n)=0 para S(n)= \sum_{i=1}^n a_i se n<=0. Fiquei intrigado quando vc escreveu > > S(n) = - ( (-1)^3 + (-2)^3 + ... + (n+1)^3 ), n < -1 e pensava em como seria uma expressão equivalente para uma outra PA de ordem 3. Estava matutando com isso e agora chegou esta sua msg. Pelo que entendi da notação de Iverson, S(n)=0 para n<=0. []'s Luís -----Mensagem Original----- De: "Nicolau C. Saldanha" Para: Enviada em: segunda-feira, 28 de abril de 2003 08:58 Assunto: Re: [obm-l] Falha nossa > On Sat, Apr 26, 2003 at 10:40:39AM -0300, Nicolau C. Saldanha wrote: > > On Fri, Apr 25, 2003 at 01:29:25PM +0000, Antonio Neto wrote: > > > Falei besteira, Igor. Nao reparei que era a soma, e troquei pela > > > determinacao do termo geral. O que eu disse aplica-se ao termo geral, mas > > > nao aa soma. Para tirar a dúvida, fui aos arquivos da lista para pegar os > > > valores de a, b, c, d e e. Escrevi o polinomio e calculei S(-1)= 1 e S(-2)= > > > 0, o que nao faz sentido, pelo menos para mim. Desculpem a falha, abracos, > > > olavo. > > > > Para mim faz todo o sentido falar em S(n) para qualquer inteiro. > > Temos S(1) = 1^3 = 1 > > S(2) = 1^3 + 2^3 = 9 > > S(3) = 1^3 + 2^3 + 3^3 = 36 > > > > A propriedade importante é S(n+1) = S(n) + (n+1)^3 o que, > > junto com S(0) = 0, define S. > > > > Mas se você desejar uma interpretação mais explícita, tome > > > > S(n) = 1^3 + 2^3 + ... + n^3, n >= 1 > > S(n) = - ( (-1)^3 + (-2)^3 + ... + (n+1)^3 ), n < -1 > > Pensando mais no assunto achei que isso pode não ter ficado muito > claro, vou dar outra explicação. A notação de Iverson é > > [frase] = 1, se 'frase' é verdadeira > 0, se 'frase' é falsa. > > Usando esta notação temos > > S(n) = soma_k ([k > 0] - [k > n]) (k^3) > > onde o somatório é tomado sobre todos os inteiros k. > É claro que só um número finito de termos é não nula. > Esta definição vale para qualquer valor inteiro n e com > ela é bem claro que S(n+1) = S(n) + (n+1)^3. > > []s, N. > ========================================================================= Instruções para entrar na lista, sair da lista e usar a lista em http://www.mat.puc-rio.br/~nicolau/olimp/obm-l.html ========================================================================= From owner-obm-l@sucuri.mat.puc-rio.br Mon Apr 28 22:35:41 2003 Return-Path: Received: (from majordom@localhost) by sucuri.mat.puc-rio.br (8.9.3/8.9.3) id WAA17348 for obm-l-MTTP; Mon, 28 Apr 2003 22:32:45 -0300 Received: from hotmail.com (bay2-dav61.bay2.hotmail.com [65.54.246.196]) by sucuri.mat.puc-rio.br (8.9.3/8.9.3) with ESMTP id WAA17344 for ; Mon, 28 Apr 2003 22:32:42 -0300 Received: from mail pickup service by hotmail.com with Microsoft SMTPSVC; Mon, 28 Apr 2003 18:32:10 -0700 Received: from 200.161.14.239 by bay2-dav61.bay2.hotmail.com with DAV; Tue, 29 Apr 2003 01:32:10 +0000 X-Originating-IP: [200.161.14.239] X-Originating-Email: [obm_2003@hotmail.com] From: "Raphael Marx" To: Subject: [obm-l] Desigualdades Date: Mon, 28 Apr 2003 22:32:22 -0300 MIME-Version: 1.0 Content-Type: text/plain; charset="Windows-1252" Content-Transfer-Encoding: 8bit X-Priority: 3 X-MSMail-Priority: Normal X-Mailer: Microsoft Outlook Express 6.00.2600.0000 X-MimeOLE: Produced By Microsoft MimeOLE V6.00.2600.0000 Message-ID: X-OriginalArrivalTime: 29 Apr 2003 01:32:10.0301 (UTC) FILETIME=[273AB6D0:01C30DEF] Sender: owner-obm-l@sucuri.mat.puc-rio.br Precedence: bulk Reply-To: obm-l@mat.puc-rio.br Olá a todos. Gostaria de pedir uma ajudinha em desigualdades: 1-Prove que se v8(vy+xz) 2-Existiriam algumas recomendações de sites sobre o assunto. 3-Teria algumas recomendações de livros sobre o assunto. 4-Um local onde possa pegar exercícios de teoria dos números,mais especificamente, relações binárias entre números inteiros(menor ou igual, divide e congruente módulo m). Estou precisando ver um pouquinho de teoria e estratégias para ter um pouco mais de sucesso em desigualdades. Um abraço a todos. ========================================================================= Instruções para entrar na lista, sair da lista e usar a lista em http://www.mat.puc-rio.br/~nicolau/olimp/obm-l.html ========================================================================= From owner-obm-l@sucuri.mat.puc-rio.br Mon Apr 28 22:35:48 2003 Return-Path: Received: (from majordom@localhost) by sucuri.mat.puc-rio.br (8.9.3/8.9.3) id WAA17309 for obm-l-MTTP; Mon, 28 Apr 2003 22:32:23 -0300 Received: from mail.ajato.com.br (200-162-192-51.mail.ajato.com.br [200.162.192.51]) by sucuri.mat.puc-rio.br (8.9.3/8.9.3) with SMTP id WAA17300 for ; Mon, 28 Apr 2003 22:32:11 -0300 Received: from mparaujo (200.162.245.161) by mail.ajato.com.br (5.1.061) id 3E9B665E0020CF81 for obm-l@mat.puc-rio.br; Mon, 28 Apr 2003 22:31:40 -0300 Message-ID: <003301c30c54$b7cda640$a1f5a2c8@ajato.com.br> From: "Marcos Paulo" To: References: <002601c30dd7$aaeb8f80$9a75fea9@windows98> Subject: Re: [obm-l] geometria Date: Sat, 26 Apr 2003 21:33:59 -0300 MIME-Version: 1.0 Content-Type: multipart/alternative; boundary="----=_NextPart_000_0030_01C30C3B.8CB546C0" X-Priority: 3 X-MSMail-Priority: Normal X-Mailer: Microsoft Outlook Express 6.00.2800.1158 X-MimeOLE: Produced By Microsoft MimeOLE V6.00.2800.1165 Sender: owner-obm-l@sucuri.mat.puc-rio.br Precedence: bulk Reply-To: obm-l@mat.puc-rio.br This is a multi-part message in MIME format. ------=_NextPart_000_0030_01C30C3B.8CB546C0 Content-Type: text/plain; charset="iso-8859-1" Content-Transfer-Encoding: quoted-printable Num c=EDrculo de raio igual a 12 est=E1 inscrito um triangulo ABC cujos = lados AB e AC medem 8 e 9, respectivamente. A altura relativa ao lado = BC =E9 igual a? Seja H o p=E9 da altura desejada. med(AC)/sen B =3D 2R, ou seja, sen B = =3D 9/24 e sen B =3D med(AH)/ AB, pois o tri=E2ngulo AHB =E9 = ret=E2ngulo. dessa maneira acho q, salvo erros de contas, AH mede 3. O segundo problema est=E1 incompleto e talvez seu enunciado fosse: Se um trapezio retangulo tem diagonais perpendiculars e bases iguais a 9 = e 36, a sua altura =E9 igual a? Nesse caso, h=E1 uma rela=E7=E3o demonstrada no livro, que diz que a = altura =E9 a m=E9dia geom=E9trica das bases. Como vc deu a resposta eu = escolhi as bases de forma que sua m=E9dia geome'trica fosse 18. []'s MP ----- Original Message -----=20 From: Daniel Pini=20 To: obm-l@mat.puc-rio.br=20 Sent: Monday, April 28, 2003 7:44 PM Subject: [obm-l] geometria R:18 Estes problemas foram retirados do Geometria2 do Morgado. ------=_NextPart_000_0030_01C30C3B.8CB546C0 Content-Type: text/html; charset="iso-8859-1" Content-Transfer-Encoding: quoted-printable
Num c=EDrculo de raio igual a 12  = est=E1 inscrito=20 um triangulo ABC cujos lados AB e AC medem  8 e 9, respectivamente. = A=20 altura  relativa ao lado BC =E9 igual a?
Seja H o p=E9 da altura desejada. med(AC)/sen B =3D 2R, ou seja, = sen B =3D 9/24 e=20 sen B =3D med(AH)/ AB, pois o tri=E2ngulo AHB =E9 ret=E2ngulo. dessa = maneira acho q,=20 salvo erros de contas, AH mede 3.
 
 
O segundo problema est=E1 incompleto e talvez seu enunciado = fosse:
Se um trapezio retangulo tem diagonais perpendiculars e bases = iguais a 9 e=20 36, a sua altura =E9 igual a?
 
Nesse caso, h=E1 uma rela=E7=E3o demonstrada no livro, que diz que = a altura =E9 a=20 m=E9dia geom=E9trica das bases. Como vc deu a resposta eu escolhi as = bases de forma=20 que sua m=E9dia geome'trica fosse 18.
 
[]'s MP
----- Original Message -----
From:=20 Daniel = Pini
Sent: Monday, April 28, 2003 = 7:44=20 PM
Subject: [obm-l] = geometria

 
 
 R:18
 
Estes problemas foram retirados do Geometria2 = do=20 Morgado.
------=_NextPart_000_0030_01C30C3B.8CB546C0-- ========================================================================= Instruções para entrar na lista, sair da lista e usar a lista em http://www.mat.puc-rio.br/~nicolau/olimp/obm-l.html ========================================================================= From owner-obm-l@sucuri.mat.puc-rio.br Mon Apr 28 22:58:06 2003 Return-Path: Received: (from majordom@localhost) by sucuri.mat.puc-rio.br (8.9.3/8.9.3) id WAA18186 for obm-l-MTTP; Mon, 28 Apr 2003 22:55:31 -0300 Received: from paiol.terra.com.br (paiol.terra.com.br [200.176.3.18]) by sucuri.mat.puc-rio.br (8.9.3/8.9.3) with ESMTP id WAA18182 for ; Mon, 28 Apr 2003 22:55:27 -0300 Received: from jurere.terra.com.br (jurere.terra.com.br [200.176.3.49]) by paiol.terra.com.br (Postfix) with ESMTP id AC321882E6 for ; Mon, 28 Apr 2003 22:54:56 -0300 (BRT) Received: from [200.177.182.136] (dl-nas6-sao-C8B1B688.p001.terra.com.br [200.177.182.136]) by jurere.terra.com.br (Postfix) with ESMTP id 8DE761380CB for ; Mon, 28 Apr 2003 22:54:55 -0300 (BRT) User-Agent: Microsoft-Outlook-Express-Macintosh-Edition/5.02.2022 Date: Mon, 28 Apr 2003 22:57:07 -0300 Subject: Re: [obm-l] Basquete Bizantino From: Claudio Buffara To: Message-ID: In-Reply-To: <006601c30dd7$fec4dcb0$96619ec8@gauss> Mime-version: 1.0 Content-type: text/plain; charset="ISO-8859-1" Content-Transfer-Encoding: 8bit X-MIME-Autoconverted: from quoted-printable to 8bit by sucuri.mat.puc-rio.br id WAA18183 Sender: owner-obm-l@sucuri.mat.puc-rio.br Precedence: bulk Reply-To: obm-l@mat.puc-rio.br Oi, Domingos: Rodar um programinha??? Essa ideia so podia vir um cientista da computacao!!! Mas tambem da pra fazer analiticamente e sem muito braco. mdc(a,b) = 1 decorre do fato de haver apenas um numero finito de pontuacoes impossiveis, ja que, por Bezout, todas as pontuacoes possiveis sao multiplas de mdc(a,b). Assim, se este fosse > 1, haveria uma infinidade de pontuacoes impossiveis. Sugestao: voce ja quase provou que existe N, tal que para todo n > N, a equacao aX + bY = n tem solucao inteira nao negativa mas aX + by = N nao tem uma tal solucao. Este N pode ser expresso em funcao de a e b. Depois, um pouco de exploracao numerica pode sugerir uma conjectura para o numero de pontuacoes impossiveis em funcao de a e b. Esta conjectura pode ser provada de varias maneiras, mas a demonstracao mais elegante que eu conheco eh bijetiva. Um abraco, Claudio. on 28.04.03 19:46, Domingos Jr. at dopikas@uol.com.br wrote: >> O jogo de basquete bizantino é igual ao basquete normal, exceto que cestas >> curtas valem "a" pontos ao invés de 2 pontos e cestas de longa distância >> valem "b" pontos ao invés de 3 pontos. No basquete bizantino existem >> exatamente 35 pontuações que nunca ocorrem numa partida. Uma delas é 58. >> Quanto valem "a" e "b"? > > > os pontos possíveis são { aX + bY, X, Y >= 0 inteiros } > se 58 não é uma pontuação possível, temos necessariamente que nem a nem b > dividem 58, ou seja: > a, b != { 1, 2, 29, 58 } > > se existe um número finito de pontuações que nunca são obtidas temos que > existe um inteiro N tal que, para todo n > N, existem X, Y tal que n = aX + > bY > > seja n + 1 = aX' + bY' > 1 = a(X'-X) + b(Y'-Y) > > isso nos diz que mdc(a, b) = 1 > assuma a < b (sem perda de generalidade) > > aX + bY = aX' + bY' <=> > a(X - X') = b(Y'-Y) > mas então b|(X - X') e a|(Y - Y') > > a.X + b.Y tem então valores distintos para 0 <= X <= b-1, 0 <= Y <= a-1 > ou seja há exatamente a.b pontuações distintas entre { 0, 1, ..., 2ab - a - > b } > 2ab - a - b + 1 - ab <= 35 > ab - a - b <= 34 > > a >= 3 pois a != { 1, 2 } > ab - a - b <= 34 => > b(a-1) <= 34 + a > b <= (34 + a)/(a-1) > > dá pra limitar o problema a: > podemos ver que a = { 3, 4, 5, 6 } > pois se a = 7, b <= (34 + 7)/6 = 41/6 < 7 > > existem 20 possíveis valores de (a, b) que satisfazem a desigualdade acima e > mdc(a, b)= 1, talvez dê pra rodar um programinha e verificar qual dessas 20 > comb. tem a propriedade desejada. > > [ ]'s > > ========================================================================= > Instruções para entrar na lista, sair da lista e usar a lista em > http://www.mat.puc-rio.br/~nicolau/olimp/obm-l.html > ========================================================================= > ========================================================================= Instruções para entrar na lista, sair da lista e usar a lista em http://www.mat.puc-rio.br/~nicolau/olimp/obm-l.html ========================================================================= From owner-obm-l@sucuri.mat.puc-rio.br Mon Apr 28 22:58:07 2003 Return-Path: Received: (from majordom@localhost) by sucuri.mat.puc-rio.br (8.9.3/8.9.3) id WAA18177 for obm-l-MTTP; Mon, 28 Apr 2003 22:55:25 -0300 Received: from ivoti.terra.com.br (ivoti.terra.com.br [200.176.3.20]) by sucuri.mat.puc-rio.br (8.9.3/8.9.3) with ESMTP id WAA18173 for ; Mon, 28 Apr 2003 22:55:20 -0300 Received: from itaim.terra.com.br (itaim.terra.com.br [200.176.3.76]) by ivoti.terra.com.br (Postfix) with ESMTP id 49AA940886E for ; Mon, 28 Apr 2003 22:54:50 -0300 (BRT) Received: from [200.177.182.136] (dl-nas6-sao-C8B1B688.p001.terra.com.br [200.177.182.136]) by itaim.terra.com.br (Postfix) with ESMTP id 2D9882E0047 for ; Mon, 28 Apr 2003 22:54:49 -0300 (BRT) User-Agent: Microsoft-Outlook-Express-Macintosh-Edition/5.02.2022 Date: Mon, 28 Apr 2003 22:56:20 -0300 Subject: Re: [obm-l] geometria From: Claudio Buffara To: Message-ID: In-Reply-To: <002601c30dd7$aaeb8f80$9a75fea9@windows98> Mime-version: 1.0 Content-type: multipart/alternative; boundary="MS_Mac_OE_3134415380_522706_MIME_Part" Sender: owner-obm-l@sucuri.mat.puc-rio.br Precedence: bulk Reply-To: obm-l@mat.puc-rio.br > This message is in MIME format. Since your mail reader does not understand this format, some or all of this message may not be legible. --MS_Mac_OE_3134415380_522706_MIME_Part Content-type: text/plain; charset="ISO-8859-1" Content-transfer-encoding: quoted-printable Oi, Daniel: on 28.04.03 19:44, Daniel Pini at daniel@fnn.net wrote: Num c=EDrculo de raio igual a 12 est=E1 inscrito um triangulo ABC cujos lados AB e AC medem 8 e 9, respectivamente. A altura relativa ao lado BC =E9 igua= l a? [ABC] =3D (1/2)*AB*AC*sen(A) =3D (1/2)*BC*h =3D=3D> h =3D AB*AC*sen(A)/BC Lei dos senos: AB =3D 2*R*sen(A) =3D=3D> sen(A)/AB =3D 1/(2*R) Logo, h =3D AB*AC/(2*R) =3D 8*9/(2*12) =3D 3. ***** Se um trapezio retangulo tem diagonais perpendiculars e bases iguais(??), a sua altura =E9 igual a? Dados insuficientes. De qualquer forma, um trapezio retangulo tem bases iguasi, entao ele eh um retangulo. Alem disso, se as diagonais sao perpendiculares, ele eh um losango. Um retangulo que eh losango eh um quadrado =3D=3D> Altura =3D Base, mas falta a mediad de algum elemento linear. R:18 =20 Estes problemas foram retirados do Geometria2 do Morgado. *** Xiii! Se voce copiou errado o 2o. problema, o Morgado vai te dar uma bronca... Um abraco, Claudio. --MS_Mac_OE_3134415380_522706_MIME_Part Content-type: text/html; charset="ISO-8859-1" Content-transfer-encoding: quoted-printable Re: [obm-l] geometria Oi, Daniel:

on 28.04.03 19:44, Daniel Pini at daniel@fnn.net wrote:

Num c=EDrculo de raio igual a 12  est=E1 in= scrito um triangulo ABC cujos lados AB e AC medem  8 e 9, respectivamen= te. A altura  relativa ao lado BC =E9 igual a?

[ABC] =3D (1/2)*AB*AC*sen(A) =3D (1/2)*BC*h =3D=3D> h =3D AB*AC*sen(A)/BC

Lei dos senos: AB =3D 2*R*sen(A) =3D=3D> sen(A)/AB =3D 1/(2*R)

Logo, h =3D AB*AC/(2*R) =3D 8*9/(2*12) =3D 3.

*****

Se um trapezio retangulo tem diagonais perpendiculars e = bases iguais(??), a sua altura =E9 igual a?

Dados insuficientes.

De qualquer forma, um trapezio retangulo tem bases iguasi, entao ele eh um = retangulo.
Alem disso, se as diagonais sao perpendiculares, ele eh um losango.
Um retangulo que eh losango eh um quadrado =3D=3D> Altura =3D Base, mas falta = a mediad de algum elemento linear.

R:18

Estes problemas foram retirados do Geometria2 do Morgado= .

*** Xiii! Se voce copiou errado o 2o. problema, o Morgado vai te dar uma br= onca...


       Um abraco,
       Claudio.
--MS_Mac_OE_3134415380_522706_MIME_Part-- ========================================================================= Instruções para entrar na lista, sair da lista e usar a lista em http://www.mat.puc-rio.br/~nicolau/olimp/obm-l.html ========================================================================= From owner-obm-l@sucuri.mat.puc-rio.br Mon Apr 28 23:02:17 2003 Return-Path: Received: (from majordom@localhost) by sucuri.mat.puc-rio.br (8.9.3/8.9.3) id WAA18346 for obm-l-MTTP; Mon, 28 Apr 2003 22:59:43 -0300 Received: from itaqui.terra.com.br (itaqui.terra.com.br [200.176.3.19]) by sucuri.mat.puc-rio.br (8.9.3/8.9.3) with ESMTP id WAA18340 for ; Mon, 28 Apr 2003 22:59:37 -0300 Received: from altamira.terra.com.br (altamira.terra.com.br [200.176.3.40]) by itaqui.terra.com.br (Postfix) with ESMTP id AE4F73BC8F2 for ; Mon, 28 Apr 2003 22:59:06 -0300 (BRT) Received: from [200.177.182.121] (dl-nas6-sao-C8B1B679.p001.terra.com.br [200.177.182.121]) by altamira.terra.com.br (Postfix) with ESMTP id 68B8E3DC113 for ; Mon, 28 Apr 2003 22:59:05 -0300 (BRT) User-Agent: Microsoft-Outlook-Express-Macintosh-Edition/5.02.2022 Date: Mon, 28 Apr 2003 23:00:37 -0300 Subject: FW: [obm-l] geometria From: Claudio Buffara To: Lista OBM Message-ID: In-Reply-To: Mime-version: 1.0 Content-type: multipart/alternative; boundary="MS_Mac_OE_3134415637_538142_MIME_Part" Sender: owner-obm-l@sucuri.mat.puc-rio.br Precedence: bulk Reply-To: obm-l@mat.puc-rio.br > This message is in MIME format. Since your mail reader does not understand this format, some or all of this message may not be legible. --MS_Mac_OE_3134415637_538142_MIME_Part Content-type: text/plain; charset="ISO-8859-1" Content-transfer-encoding: quoted-printable Ops! Uma correcao: No primeiro problema, na linha da lei dos senos, onde esta AB, leia-se BC (ja que este e nao AB eh o lado oposto ao angulo A). Pelo visto, eu tambem vou levar uma bronca do Morgado... Um abraco, Claudio. ---------- From: Claudio Buffara Date: Mon, 28 Apr 2003 22:56:20 -0300 To: Subject: Re: [obm-l] geometria Oi, Daniel: on 28.04.03 19:44, Daniel Pini at daniel@fnn.net wrote: Num c=EDrculo de raio igual a 12 est=E1 inscrito um triangulo ABC cujos lados AB e AC medem 8 e 9, respectivamente. A altura relativa ao lado BC =E9 igua= l a? [ABC] =3D (1/2)*AB*AC*sen(A) =3D (1/2)*BC*h =3D=3D> h =3D AB*AC*sen(A)/BC Lei dos senos: AB =3D 2*R*sen(A) =3D=3D> sen(A)/AB =3D 1/(2*R) Logo, h =3D AB*AC/(2*R) =3D 8*9/(2*12) =3D 3. ***** Se um trapezio retangulo tem diagonais perpendiculars e bases iguais(??), a sua altura =E9 igual a? Dados insuficientes. De qualquer forma, um trapezio retangulo tem bases iguasi, entao ele eh um retangulo. Alem disso, se as diagonais sao perpendiculares, ele eh um losango. Um retangulo que eh losango eh um quadrado =3D=3D> Altura =3D Base, mas falta a mediad de algum elemento linear. R:18 =20 Estes problemas foram retirados do Geometria2 do Morgado. *** Xiii! Se voce copiou errado o 2o. problema, o Morgado vai te dar uma bronca... Um abraco, Claudio. --MS_Mac_OE_3134415637_538142_MIME_Part Content-type: text/html; charset="ISO-8859-1" Content-transfer-encoding: quoted-printable FW: [obm-l] geometria Ops! Uma correcao: No primeiro problema, na linha da lei dos senos, onde es= ta AB, leia-se BC (ja que este e nao AB eh o lado oposto ao angulo A).

Pelo visto, eu tambem vou levar uma bronca do Morgado...

Um abraco,
Claudio.

----------
From: Claudio Buffara <claudio.buffara@terra.com.br>
Date: Mon, 28 Apr 2003 22:56:20 -0300
To: <obm-l@mat.puc-rio.br>
Subject: Re: [obm-l] geometria

Oi, Daniel:

on 28.04.03 19:44, Daniel Pini at daniel@fnn.net wrote:

Num c=EDrculo de raio igual a 12  est=E1 in= scrito um triangulo ABC cujos lados AB e AC medem  8 e 9, respectivamen= te. A altura  relativa ao lado BC =E9 igual a?

[ABC] =3D (1/2)*AB*AC*sen(A) =3D (1/2)*BC*h =3D=3D> h =3D AB*AC*sen(A)/BC

Lei dos senos: AB =3D 2*R*sen(A) =3D=3D> sen(A)/AB =3D 1/(2*R)

Logo, h =3D AB*AC/(2*R) =3D 8*9/(2*12) =3D 3.

*****

Se um trapezio retangulo tem diagonais perpendiculars e = bases iguais(??), a sua altura =E9 igual a?

Dados insuficientes.

De qualquer forma, um trapezio retangulo tem bases iguasi, entao ele eh um = retangulo.
Alem disso, se as diagonais sao perpendiculares, ele eh um losango.
Um retangulo que eh losango eh um quadrado =3D=3D> Altura =3D Base, mas falta = a mediad de algum elemento linear.

R:18

Estes problemas foram retirados do Geometria2 do Morgado= .

*** Xiii! Se voce copiou errado o 2o. problema, o Morgado vai te dar uma br= onca...


       Um abraco,
      Claudio.
--MS_Mac_OE_3134415637_538142_MIME_Part-- ========================================================================= Instruções para entrar na lista, sair da lista e usar a lista em http://www.mat.puc-rio.br/~nicolau/olimp/obm-l.html ========================================================================= From owner-obm-l@sucuri.mat.puc-rio.br Mon Apr 28 23:02:20 2003 Return-Path: Received: (from majordom@localhost) by sucuri.mat.puc-rio.br (8.9.3/8.9.3) id WAA18357 for obm-l-MTTP; Mon, 28 Apr 2003 22:59:47 -0300 Received: from mx.pop.com.br (relay2.pop.com.br [200.175.8.31]) by sucuri.mat.puc-rio.br (8.9.3/8.9.3) with ESMTP id WAA18342 for ; Mon, 28 Apr 2003 22:59:42 -0300 Received: from smtp.pop.com.br (smtp2.pop.com.br [200.175.8.19]) by mx.pop.com.br (Postfix) with SMTP id 19288BF584 for ; Mon, 28 Apr 2003 22:59:12 -0300 (BRT) Received: (qmail 1825 invoked by uid 0); 29 Apr 2003 01:59:08 -0000 Received: from 200.146.7.217.dialup.gvt.net.br (HELO MJEPYON) (200.146.7.217) by smtp2.pop.com.br with SMTP; 29 Apr 2003 01:59:08 -0000 Message-ID: <00be01c30df2$e27bb270$0100007f@MJEPYON> From: "Marcelo" To: References: Subject: Re: [obm-l] calculo c Date: Mon, 28 Apr 2003 22:45:45 -0300 MIME-Version: 1.0 Content-Type: text/plain; charset="iso-8859-1" Content-Transfer-Encoding: 8bit X-Priority: 3 X-MSMail-Priority: Normal X-Mailer: Microsoft Outlook Express 6.00.2720.3000 X-MimeOLE: Produced By Microsoft MimeOLE V6.00.2600.0000 Sender: owner-obm-l@sucuri.mat.puc-rio.br Precedence: bulk Reply-To: obm-l@mat.puc-rio.br olá marcelo... obrigado por me ajudar, mas existe um pequeno porém: eu já havia tentado por indução, mas eu não posso usar indução (um amigo meu ke faz ciencia da computação me explicou um pouco de indução), mas em calculo c, nós temos ke provar as formulas através das propriedades dos números, por isso não posso usar indução. > >olá pessoal... > > > >não estou conseguindo provar essas questões: > > > >a) x³-y³ = (x-y)(x² + xy + y²) > > Escreva x^3-y^3 = (x-y)^3+3x^2y-3xy^2 = (x-y)^3+3xy(x-y)= > > =(x-y)(x^2-2xy+y^2+3xy)=(x-y)(x^2+xy+y^2) > ok? > > >b) x^n - y^n = (x-y)(x^n-1 + x^n-2*y + ... + xy^n-2 + y^n-1) > > Aqui faz por induçâo em n > > >c) x³+y³ = (x+y)(x² - xy + y²) > > Aqui vou deixar como exercício já que é análogo ao primeiro > > []'s, Marcelo. > >--- > >Outgoing mail is certified Virus Free. > >Checked by AVG anti-virus system (http://www.grisoft.com). > >Version: 6.0.474 / Virus Database: 272 - Release Date: 18/4/2003 > > > _________________________________________________________________ > Add photos to your e-mail with MSN 8. Get 2 months FREE*. > http://join.msn.com/?page=features/featuredemail > > ========================================================================= > Instruções para entrar na lista, sair da lista e usar a lista em > http://www.mat.puc-rio.br/~nicolau/olimp/obm-l.html > ========================================================================= > --- Outgoing mail is certified Virus Free. Checked by AVG anti-virus system (http://www.grisoft.com). Version: 6.0.474 / Virus Database: 272 - Release Date: 18/4/2003 ========================================================================= Instruções para entrar na lista, sair da lista e usar a lista em http://www.mat.puc-rio.br/~nicolau/olimp/obm-l.html ========================================================================= From owner-obm-l@sucuri.mat.puc-rio.br Mon Apr 28 23:05:47 2003 Return-Path: Received: (from majordom@localhost) by sucuri.mat.puc-rio.br (8.9.3/8.9.3) id XAA18515 for obm-l-MTTP; Mon, 28 Apr 2003 23:02:53 -0300 Received: from paiol.terra.com.br (paiol.terra.com.br [200.176.3.18]) by sucuri.mat.puc-rio.br (8.9.3/8.9.3) with ESMTP id XAA18510 for ; Mon, 28 Apr 2003 23:02:49 -0300 Received: from marova.terra.com.br (marova.terra.com.br [200.176.3.39]) by paiol.terra.com.br (Postfix) with ESMTP id E7FAF88123 for ; Mon, 28 Apr 2003 23:02:18 -0300 (BRT) Received: from riemann.localdomain (RJ174112.user.veloxzone.com.br [200.149.174.112]) (authenticated user fabio.dias.moreira) by marova.terra.com.br (Postfix) with ESMTP id 8566A3DC07A for ; Mon, 28 Apr 2003 23:02:18 -0300 (BRT) Content-Type: text/plain; charset="windows-1252" From: =?windows-1252?q?F=E1bio=20Dias=20Moreira?= To: obm-l@mat.puc-rio.br Subject: Re: [obm-l] Desigualdades Date: Mon, 28 Apr 2003 23:01:56 -0300 User-Agent: KMail/1.4.3 References: In-Reply-To: MIME-Version: 1.0 Content-Transfer-Encoding: 8bit Message-Id: <200304282302.21328.fabio.dias.moreira@terra.com.br> Sender: owner-obm-l@sucuri.mat.puc-rio.br Precedence: bulk Reply-To: obm-l@mat.puc-rio.br -----BEGIN PGP SIGNED MESSAGE----- Hash: SHA1 On Monday 28 April 2003 22:32, Raphael Marx wrote: > Olá a todos. Gostaria de pedir uma ajudinha em desigualdades: > 1-Prove que se v (v+x+y+z)^2>8(vy+xz) > [...] Divida por 16: ((v+x+y+z)/4)^2 > (vy+xz)/2 > sqrt(xyzv) (v+x+y+z)/4 > (xyzv)^(1/4) verdadeiro por MA-MG. []s, - -- Fábio "ctg \pi" Dias Moreira -----BEGIN PGP SIGNATURE----- Version: GnuPG v1.0.6 (GNU/Linux) Comment: For info see http://www.gnupg.org iD8DBQE+rd0salOQFrvzGQoRAh43AJ4zphB28Q0cPIs2Q4qWrQpzBI7OeACfVHO4 O/sD7hsGmU7IR6o4g4XnDa8= =ob8a -----END PGP SIGNATURE----- ========================================================================= Instruções para entrar na lista, sair da lista e usar a lista em http://www.mat.puc-rio.br/~nicolau/olimp/obm-l.html ========================================================================= From owner-obm-l@sucuri.mat.puc-rio.br Mon Apr 28 23:37:25 2003 Return-Path: Received: (from majordom@localhost) by sucuri.mat.puc-rio.br (8.9.3/8.9.3) id XAA20965 for obm-l-MTTP; Mon, 28 Apr 2003 23:34:07 -0300 Received: from www.zipmail.com.br (smtp.zipmail.com.br [200.221.11.147]) by sucuri.mat.puc-rio.br (8.9.3/8.9.3) with ESMTP id XAA20961 for ; Mon, 28 Apr 2003 23:34:03 -0300 From: camilojr@zipmail.com.br Received: from [200.214.79.186] by www.zipmail.com.br with HTTP; Mon, 28 Apr 2003 23:31:23 -0300 Message-ID: <3EA9925000004F60@www.zipmail.com.br> Date: Mon, 28 Apr 2003 23:31:23 -0300 In-Reply-To: <00be01c30df2$e27bb270$0100007f@MJEPYON> Subject: [obm-l] =?iso-8859-1?Q?Re=3A=20=5Bobm=2Dl=5D=20calculo=20c?= To: obm-l@mat.puc-rio.br MIME-Version: 1.0 Content-Type: text/plain; charset="iso-8859-1" Content-Transfer-Encoding: 8bit X-MIME-Autoconverted: from quoted-printable to 8bit by sucuri.mat.puc-rio.br id XAA20962 Sender: owner-obm-l@sucuri.mat.puc-rio.br Precedence: bulk Reply-To: obm-l@mat.puc-rio.br Oi Marcelo, Vê se isso resolve o teu problema: b)(x-y)(x^n-1 + x^n-2*y + ... + xy^n-2 + y^n-1) = (x-y)*sum (k=0,n-1)(x^(n-1-k)*y^k)= sum (k=0,n-1)(x^(n-k)*y^k) - sum (k=0,n-1)(x^(n-1-k)*y^(k+1))= sum (k=0,n-1)(x^(n-k)*y^k) - sum (k=1,n)(x^(n-k)*y^k) Olhando os somatórios acima, é fácil ver que todos os termos se cancelam, menos k=0 e k=n, logo: (x-y)(x^n-1 + x^n-2*y + ... + xy^n-2 + y^n-1) = x^n - y^n um abraço, Camilo PS: sum (k=0,n): somatório com k variando de 0 a n. -- Mensagem original -- >olá marcelo... obrigado por me ajudar, mas existe um pequeno porém: eu já >havia tentado por indução, mas eu não posso usar indução (um amigo meu ke >faz ciencia da computação me explicou um pouco de indução), mas em calculo >c, nós temos ke provar as formulas através das propriedades dos números, >por >isso não posso usar indução. > >> >olá pessoal... >> > >> >não estou conseguindo provar essas questões: >> > >> >a) x³-y³ = (x-y)(x² + xy + y²) >> >> Escreva x^3-y^3 = (x-y)^3+3x^2y-3xy^2 = (x-y)^3+3xy(x-y)= >> >> =(x-y)(x^2-2xy+y^2+3xy)=(x-y)(x^2+xy+y^2) >> ok? >> >> >b) x^n - y^n = (x-y)(x^n-1 + x^n-2*y + ... + xy^n-2 + y^n-1) >> >> Aqui faz por induçâo em n >> >> >c) x³+y³ = (x+y)(x² - xy + y²) >> >> Aqui vou deixar como exercício já que é análogo ao primeiro >> >> []'s, Marcelo. >> >--- >> >Outgoing mail is certified Virus Free. >> >Checked by AVG anti-virus system (http://www.grisoft.com). >> >Version: 6.0.474 / Virus Database: 272 - Release Date: 18/4/2003 >> >> >> _________________________________________________________________ >> Add photos to your e-mail with MSN 8. Get 2 months FREE*. >> http://join.msn.com/?page=features/featuredemail >> >> ========================================================================= >> Instruções para entrar na lista, sair da lista e usar a lista em >> http://www.mat.puc-rio.br/~nicolau/olimp/obm-l.html >> ========================================================================= >> > > >--- >Outgoing mail is certified Virus Free. >Checked by AVG anti-virus system (http://www.grisoft.com). >Version: 6.0.474 / Virus Database: 272 - Release Date: 18/4/2003 > >========================================================================= >Instruções para entrar na lista, sair da lista e usar a lista em >http://www.mat.puc-rio.br/~nicolau/olimp/obm-l.html >========================================================================= > ------------------------------------------ Use o melhor sistema de busca da Internet Radar UOL - http://www.radaruol.com.br ========================================================================= Instruções para entrar na lista, sair da lista e usar a lista em http://www.mat.puc-rio.br/~nicolau/olimp/obm-l.html ========================================================================= From owner-obm-l@sucuri.mat.puc-rio.br Mon Apr 28 23:53:04 2003 Return-Path: Received: (from majordom@localhost) by sucuri.mat.puc-rio.br (8.9.3/8.9.3) id XAA21712 for obm-l-MTTP; Mon, 28 Apr 2003 23:50:14 -0300 Received: from krypton.hosting4u.net (krypton.hosting4u.net [209.15.2.78]) by sucuri.mat.puc-rio.br (8.9.3/8.9.3) with ESMTP id XAA21706 for ; Mon, 28 Apr 2003 23:50:10 -0300 Received: from gargamel (200-158-200-15.dsl.telesp.net.br [200.158.200.15]) by krypton.hosting4u.net (Postfix) with ESMTP id B63B3A078A for ; Mon, 28 Apr 2003 21:49:37 -0500 (CDT) Message-ID: <200304282353520870.059D4A06@smtp.watersportsbrazil.com> In-Reply-To: References: X-Mailer: Calypso Version 3.30.00.00 (3) Date: Mon, 28 Apr 2003 23:53:52 -0300 From: "Ariel de Silvio" To: obm-l@mat.puc-rio.br Subject: Re: [obm-l] geometria Mime-Version: 1.0 Content-Type: multipart/alternative; boundary="=====_105158483229385=_" Sender: owner-obm-l@sucuri.mat.puc-rio.br Precedence: bulk Reply-To: obm-l@mat.puc-rio.br --=====_105158483229385=_ Content-Type: text/plain; charset="us-ascii" Content-Transfer-Encoding: quoted-printable Claudio, n=E3o entendi a ultima passagem, vc substituiu sen(A)/BC por= sen(A)/AB?? []s =C1riel *********** MENSAGEM ORIGINAL *********** As 22:56 de 28/4/2003 Claudio Buffara escreveu: Oi, Daniel: on 28.04.03 19:44, Daniel Pini at daniel@fnn.net wrote: Num c=EDrculo de raio igual a 12 est=E1 inscrito um triangulo ABC cujos= lados AB e AC medem 8 e 9, respectivamente. A altura relativa ao lado BC= =E9 igual a? [ABC] =3D (1/2)*AB*AC*sen(A) =3D (1/2)*BC*h =3D=3D> h =3D AB*AC*sen(A)/BC Lei dos senos: AB =3D 2*R*sen(A) =3D=3D> sen(A)/AB =3D 1/(2*R) Logo, h =3D AB*AC/(2*R) =3D 8*9/(2*12) =3D 3. --=====_105158483229385=_ Content-Type: text/html; charset="us-ascii" Re: [obm-l] geometria
Claudio, não entendi a ultima passagem, vc substituiu  sen(A)/BC por sen(A)/AB??
 
[]s
Áriel

*********** MENSAGEM ORIGINAL ***********

As 22:56 de 28/4/2003 Claudio Buffara escreveu:
Oi, Daniel:

on 28.04.03 19:44, Daniel Pini at daniel@fnn.net wrote:

Num círculo de raio igual a 12  está inscrito um triangulo ABC cujos lados AB e AC medem  8 e 9, respectivamente. A altura  relativa ao lado BC é igual a?

[ABC] = (1/2)*AB*AC*sen(A) = (1/2)*BC*h ==> h = AB*AC*sen(A)/BC

Lei dos senos: AB = 2*R*sen(A) ==> sen(A)/AB = 1/(2*R)

Logo, h = AB*AC/(2*R) = 8*9/(2*12) = 3.
--=====_105158483229385=_-- ========================================================================= Instruções para entrar na lista, sair da lista e usar a lista em http://www.mat.puc-rio.br/~nicolau/olimp/obm-l.html ========================================================================= From owner-obm-l@sucuri.mat.puc-rio.br Tue Apr 29 00:33:16 2003 Return-Path: Received: (from majordom@localhost) by sucuri.mat.puc-rio.br (8.9.3/8.9.3) id AAA23086 for obm-l-MTTP; Tue, 29 Apr 2003 00:29:57 -0300 Received: from ivoti.terra.com.br (ivoti.terra.com.br [200.176.3.20]) by sucuri.mat.puc-rio.br (8.9.3/8.9.3) with ESMTP id AAA23082 for ; Tue, 29 Apr 2003 00:29:54 -0300 Received: from gunga.terra.com.br (gunga.terra.com.br [200.176.3.45]) by ivoti.terra.com.br (Postfix) with ESMTP id D28A040843D for ; Tue, 29 Apr 2003 00:29:23 -0300 (BRT) Received: from [200.177.182.163] (dl-nas6-sao-C8B1B6A3.p001.terra.com.br [200.177.182.163]) by gunga.terra.com.br (Postfix) with ESMTP id 10CB2128073 for ; Tue, 29 Apr 2003 00:29:23 -0300 (BRT) User-Agent: Microsoft-Outlook-Express-Macintosh-Edition/5.02.2022 Date: Tue, 29 Apr 2003 00:30:54 -0300 Subject: Re: [obm-l] Desigualdades From: Claudio Buffara To: Message-ID: In-Reply-To: <200304282302.21328.fabio.dias.moreira@terra.com.br> Mime-version: 1.0 Content-type: text/plain; charset="ISO-8859-1" Content-Transfer-Encoding: 8bit X-MIME-Autoconverted: from quoted-printable to 8bit by sucuri.mat.puc-rio.br id AAA23083 Sender: owner-obm-l@sucuri.mat.puc-rio.br Precedence: bulk Reply-To: obm-l@mat.puc-rio.br on 28.04.03 23:01, Fábio Dias Moreira at fabio.dias.moreira@terra.com.br wrote: > -----BEGIN PGP SIGNED MESSAGE----- > Hash: SHA1 > > On Monday 28 April 2003 22:32, Raphael Marx wrote: >> Olá a todos. Gostaria de pedir uma ajudinha em desigualdades: >> 1-Prove que se v> (v+x+y+z)^2>8(vy+xz) >> [...] > > Divida por 16: > > ((v+x+y+z)/4)^2 > (vy+xz)/2 > sqrt(xyzv) > (v+x+y+z)/4 > (xyzv)^(1/4) > > verdadeiro por MA-MG. > > []s, > Oi, Fabio (e Raphael): MA-MG soh pode ser usada se v > 0, o que nao estah dito no enunciado. Alem disso, acho que tem um probleminha na logica: Supondo v > 0, ((v+x+y+z)/4)^2 > raiz(xyzv) e (vy+xz)/2 > raiz(xyzv) sao ambas verdadeiras, mas isso nao implica necessariamente que: ((v+x+y+z)/4)^2 > raiz(xyzv). Na sua primeira desigualdade, voce parece estar assumindo justamente o que se quer provar. Eu diria que a condicao v < x < y < z e a ausencia da condicao v > 0 indicam que este pode ser um trabalha pra desigualdade do rearranjo. Um abraco, Claudio. ========================================================================= Instruções para entrar na lista, sair da lista e usar a lista em http://www.mat.puc-rio.br/~nicolau/olimp/obm-l.html ========================================================================= From owner-obm-l@sucuri.mat.puc-rio.br Tue Apr 29 00:39:49 2003 Return-Path: Received: (from majordom@localhost) by sucuri.mat.puc-rio.br (8.9.3/8.9.3) id AAA23298 for obm-l-MTTP; Tue, 29 Apr 2003 00:37:05 -0300 Received: from itaqui.terra.com.br (itaqui.terra.com.br [200.176.3.19]) by sucuri.mat.puc-rio.br (8.9.3/8.9.3) with ESMTP id AAA23293 for ; Tue, 29 Apr 2003 00:37:00 -0300 Received: from canela.terra.com.br (canela.terra.com.br [200.176.3.79]) by itaqui.terra.com.br (Postfix) with ESMTP id 9B1FE3BC164 for ; Tue, 29 Apr 2003 00:36:30 -0300 (BRT) Received: from [200.177.186.188] (dl-nas4-sao-C8B1BABC.p001.terra.com.br [200.177.186.188]) by canela.terra.com.br (Postfix) with ESMTP id 4E76022407F for ; Tue, 29 Apr 2003 00:36:29 -0300 (BRT) User-Agent: Microsoft-Outlook-Express-Macintosh-Edition/5.02.2022 Date: Tue, 29 Apr 2003 00:38:00 -0300 Subject: Re: [obm-l] Re: [obm-l] calculo c From: Claudio Buffara To: Message-ID: In-Reply-To: <3EA9925000004F60@www.zipmail.com.br> Mime-version: 1.0 Content-type: text/plain; charset="ISO-8859-1" Content-Transfer-Encoding: 8bit X-MIME-Autoconverted: from quoted-printable to 8bit by sucuri.mat.puc-rio.br id AAA23294 Sender: owner-obm-l@sucuri.mat.puc-rio.br Precedence: bulk Reply-To: obm-l@mat.puc-rio.br Oi, Marcelo: Se em calculo c voce puder usar a formula da soma dos termos de uma PG, entao voce pode fazer o seguinte: x^(n-1) + x^(n-2)*y + ... + x*y^(n-2) + y^(n-1) = = y^(n-1)*[(x/y)^(n-1) + (x/y)^(n-2) + ... + (x/y) + 1] = = y^(n-1)*[(x/y)^n - 1]/[(x/y) - 1] = = [x^n/y - y^(n-1)]*y/[x - y] = = [x^n - y^n]/[x - y] Ai, multiplicando ambos os membros por x - y, voce chega na identidade desejada. Um abraco, Claudio. on 28.04.03 23:31, camilojr@zipmail.com.br at camilojr@zipmail.com.br wrote: > Oi Marcelo, > > Vê se isso resolve o teu problema: > > b)(x-y)(x^n-1 + x^n-2*y + ... + xy^n-2 + y^n-1) = > (x-y)*sum (k=0,n-1)(x^(n-1-k)*y^k)= > sum (k=0,n-1)(x^(n-k)*y^k) - sum (k=0,n-1)(x^(n-1-k)*y^(k+1))= > sum (k=0,n-1)(x^(n-k)*y^k) - sum (k=1,n)(x^(n-k)*y^k) > > Olhando os somatórios acima, é fácil ver que todos os termos se cancelam, > menos k=0 e k=n, logo: > > (x-y)(x^n-1 + x^n-2*y + ... + xy^n-2 + y^n-1) = x^n - y^n > > um abraço, > Camilo > > PS: sum (k=0,n): somatório com k variando de 0 a n. > > -- Mensagem original -- > >> olá marcelo... obrigado por me ajudar, mas existe um pequeno porém: eu > já >> havia tentado por indução, mas eu não posso usar indução (um amigo meu > ke >> faz ciencia da computação me explicou um pouco de indução), mas em calculo >> c, nós temos ke provar as formulas através das propriedades dos números, >> por >> isso não posso usar indução. >> >>>> olá pessoal... >>>> >>>> não estou conseguindo provar essas questões: >>>> >>>> a) x³-y³ = (x-y)(x² + xy + y²) >>> >>> Escreva x^3-y^3 = (x-y)^3+3x^2y-3xy^2 = (x-y)^3+3xy(x-y)= >>> >>> =(x-y)(x^2-2xy+y^2+3xy)=(x-y)(x^2+xy+y^2) >>> ok? >>> >>>> b) x^n - y^n = (x-y)(x^n-1 + x^n-2*y + ... + xy^n-2 + y^n-1) >>> >>> Aqui faz por induçâo em n >>> >>>> c) x³+y³ = (x+y)(x² - xy + y²) >>> >>> Aqui vou deixar como exercício já que é análogo ao primeiro >>> >>> []'s, Marcelo. >>>> --- >>>> Outgoing mail is certified Virus Free. >>>> Checked by AVG anti-virus system (http://www.grisoft.com). >>>> Version: 6.0.474 / Virus Database: 272 - Release Date: 18/4/2003 >>> >>> >>> _________________________________________________________________ >>> Add photos to your e-mail with MSN 8. Get 2 months FREE*. >>> http://join.msn.com/?page=features/featuredemail >>> >>> ========================================================================= >>> Instruções para entrar na lista, sair da lista e usar a lista em >>> http://www.mat.puc-rio.br/~nicolau/olimp/obm-l.html >>> ========================================================================= >>> >> >> >> --- >> Outgoing mail is certified Virus Free. >> Checked by AVG anti-virus system (http://www.grisoft.com). >> Version: 6.0.474 / Virus Database: 272 - Release Date: 18/4/2003 >> >> ========================================================================= >> Instruções para entrar na lista, sair da lista e usar a lista em >> http://www.mat.puc-rio.br/~nicolau/olimp/obm-l.html >> ========================================================================= >> > > > > ------------------------------------------ > Use o melhor sistema de busca da Internet > Radar UOL - http://www.radaruol.com.br > > > > ========================================================================= > Instruções para entrar na lista, sair da lista e usar a lista em > http://www.mat.puc-rio.br/~nicolau/olimp/obm-l.html > ========================================================================= > ========================================================================= Instruções para entrar na lista, sair da lista e usar a lista em http://www.mat.puc-rio.br/~nicolau/olimp/obm-l.html ========================================================================= From owner-obm-l@sucuri.mat.puc-rio.br Tue Apr 29 09:38:52 2003 Return-Path: Received: (from majordom@localhost) by sucuri.mat.puc-rio.br (8.9.3/8.9.3) id JAA01106 for obm-l-MTTP; Tue, 29 Apr 2003 09:36:02 -0300 Received: from itaqui.terra.com.br (itaqui.terra.com.br [200.176.3.19]) by sucuri.mat.puc-rio.br (8.9.3/8.9.3) with ESMTP id JAA01101 for ; Tue, 29 Apr 2003 09:35:57 -0300 Received: from gunga.terra.com.br (gunga.terra.com.br [200.176.3.45]) by itaqui.terra.com.br (Postfix) with ESMTP id 90A0D3BC327 for ; Tue, 29 Apr 2003 09:35:26 -0300 (BRT) Received: from [200.177.192.207] (dl-nas2-sao-C8B1C0CF.p001.terra.com.br [200.177.192.207]) by gunga.terra.com.br (Postfix) with ESMTP id 1AA481280D4 for ; Tue, 29 Apr 2003 09:35:25 -0300 (BRT) User-Agent: Microsoft-Outlook-Express-Macintosh-Edition/5.02.2022 Date: Tue, 29 Apr 2003 09:36:28 -0300 Subject: Re: [obm-l] geometria From: Claudio Buffara To: Message-ID: In-Reply-To: <003301c30c54$b7cda640$a1f5a2c8@ajato.com.br> Mime-version: 1.0 Content-type: multipart/alternative; boundary="MS_Mac_OE_3134453790_74562_MIME_Part" Sender: owner-obm-l@sucuri.mat.puc-rio.br Precedence: bulk Reply-To: obm-l@mat.puc-rio.br > This message is in MIME format. Since your mail reader does not understand this format, some or all of this message may not be legible. --MS_Mac_OE_3134453790_74562_MIME_Part Content-type: text/plain; charset="ISO-8859-1" Content-transfer-encoding: quoted-printable Oi, Marcos Paulo (e demais colegas da lista): Acho que o mais interessante eh esse resultado sobre o trapezio retangulo, = o qual eu nao conhecia: as diagonais sao perpendiculares se e somente se a altura eh a media geometrica das bases. Isso da pra provar facilmente com o uso de vetores: Sejam i e j vetores unitarios ortogonais nas direcoes da base maior e da altura, respectivamente. Sejam B e b as medidas das bases (B =3D base maior) e h a altura do trapezio. Entao, as diagonais serao: d1 =3D -Bi + hj e d2 =3D bi + hj d1 e d2 sao perpendiculares <=3D=3D> d1 . d2 =3D 0 (produto escalar) <=3D=3D> d1 . d2 =3D (-Bi + hj).(bi + hj) =3D -Bb + h^2 =3D 0 <=3D=3D> h^2 =3D Bb e acabou! Um abraco, Claudio. on 26.04.03 21:33, Marcos Paulo at mparaujo@ajato.com.br wrote: Num c=EDrculo de raio igual a 12 est=E1 inscrito um triangulo ABC cujos lados AB e AC medem 8 e 9, respectivamente. A altura relativa ao lado BC =E9 igua= l a? Seja H o p=E9 da altura desejada. med(AC)/sen B =3D 2R, ou seja, sen B =3D 9/24 e sen B =3D med(AH)/ AB, pois o tri=E2ngulo AHB =E9 ret=E2ngulo. dessa maneira acho q= , salvo erros de contas, AH mede 3. O segundo problema est=E1 incompleto e talvez seu enunciado fosse: Se um trapezio retangulo tem diagonais perpendiculars e bases iguais a 9 e 36, a sua altura =E9 igual a? Nesse caso, h=E1 uma rela=E7=E3o demonstrada no livro, que diz que a altura =E9 a m=E9dia geom=E9trica das bases. Como vc deu a resposta eu escolhi as bases de forma que sua m=E9dia geome'trica fosse 18. []'s MP ----- Original Message ----- From: Daniel Pini To: obm-l@mat.puc-rio.br Sent: Monday, April 28, 2003 7:44 PM Subject: [obm-l] geometria R:18 =20 Estes problemas foram retirados do Geometria2 do Morgado. --MS_Mac_OE_3134453790_74562_MIME_Part Content-type: text/html; charset="ISO-8859-1" Content-transfer-encoding: quoted-printable Re: [obm-l] geometria Oi, Marcos Paulo (e demais colegas da lista):

Acho que o mais interessante eh esse resultado sobre o trapezio retangulo, = o qual eu nao conhecia:
as diagonais sao perpendiculares se e somente se a altura eh a media geomet= rica das bases.

Isso da pra provar facilmente com o uso de vetores:

Sejam i e j vetores unitarios ortogonais nas direcoes da base maior e da al= tura, respectivamente.
Sejam B e b as medidas das bases (B =3D base maior) e h a altura do trapezio.=

Entao, as diagonais serao:
d1 =3D -Bi + hj     e     d2 =3D bi + h= j

d1 e d2 sao perpendiculares <=3D=3D>
d1 . d2 =3D 0 (produto escalar) <=3D=3D>
d1 . d2 =3D (-Bi + hj).(bi + hj) =3D -Bb + h^2 =3D 0 <=3D=3D>
h^2 =3D Bb
e acabou!

Um abraco,
Claudio.

on 26.04.03 21:33, Marcos Paulo at mparaujo@ajato.com.br wrote:

Num c=EDrculo de raio igual a 1= 2  est=E1 inscrito um triangulo ABC cujos lados AB e AC medem  8 e 9= , respectivamente. A altura  relativa ao lado BC =E9 igual a?
Seja H o p=E9 da altura desejada. med(AC)/sen B =3D 2R, ou seja, sen B =3D 9/24 e= sen B =3D med(AH)/ AB, pois o tri=E2ngulo AHB =E9 ret=E2ngulo. dessa maneira acho q= , salvo erros de contas, AH mede 3.


O segundo problema est=E1 incompleto e talvez seu enunciado fosse:
Se um trapezio retangulo tem diagonais perpendiculars e bases iguais a 9 e = 36, a sua altura =E9 igual a?

Nesse caso, h=E1 uma rela=E7=E3o demonstrada no livro, que diz que a altura =E9 a m= =E9dia geom=E9trica das bases. Como vc deu a resposta eu escolhi as bases de for= ma que sua m=E9dia geome'trica fosse 18.

[]'s MP
----- Original Message -----
From: Daniel Pini <mailto:daniel@fnn.net>  
To: obm-l@mat.puc-rio.br
Sent: Monday, April 28, 2003 7:44 PM
Subject: [obm-l] geometria



R:18

Estes problemas foram retirados do Geometria2 do Morgado= .


--MS_Mac_OE_3134453790_74562_MIME_Part-- ========================================================================= Instruções para entrar na lista, sair da lista e usar a lista em http://www.mat.puc-rio.br/~nicolau/olimp/obm-l.html ========================================================================= From owner-obm-l@sucuri.mat.puc-rio.br Tue Apr 29 09:39:04 2003 Return-Path: Received: (from majordom@localhost) by sucuri.mat.puc-rio.br (8.9.3/8.9.3) id JAA01084 for obm-l-MTTP; Tue, 29 Apr 2003 09:34:38 -0300 Received: from hotmail.com (bay1-f146.bay1.hotmail.com [65.54.245.146]) by sucuri.mat.puc-rio.br (8.9.3/8.9.3) with ESMTP id JAA01080 for ; Tue, 29 Apr 2003 09:34:33 -0300 Received: from mail pickup service by hotmail.com with Microsoft SMTPSVC; Tue, 29 Apr 2003 05:34:01 -0700 Received: from 146.164.44.49 by by1fd.bay1.hotmail.msn.com with HTTP; Tue, 29 Apr 2003 12:34:00 GMT X-Originating-IP: [146.164.44.49] X-Originating-Email: [marcelo_souza7@hotmail.com] From: "Marcelo Souza" To: obm-l@mat.puc-rio.br Subject: [obm-l] Ajuda em teorema Date: Tue, 29 Apr 2003 12:34:00 +0000 Mime-Version: 1.0 Content-Type: text/plain; format=flowed Message-ID: X-OriginalArrivalTime: 29 Apr 2003 12:34:01.0276 (UTC) FILETIME=[9CD09FC0:01C30E4B] Sender: owner-obm-l@sucuri.mat.puc-rio.br Precedence: bulk Reply-To: obm-l@mat.puc-rio.br Alguém poderia exibir a prova do seguinte teorema para mim -Dada f(x) contínua e f'(t)<>0, então a curva pode ser expressa na forma y=f(x). obs.: <>=diferente. Agradeço []',M. _________________________________________________________________ Help STOP SPAM with the new MSN 8 and get 2 months FREE* http://join.msn.com/?page=features/junkmail ========================================================================= Instruções para entrar na lista, sair da lista e usar a lista em http://www.mat.puc-rio.br/~nicolau/olimp/obm-l.html ========================================================================= From owner-obm-l@sucuri.mat.puc-rio.br Tue Apr 29 10:41:02 2003 Return-Path: Received: (from majordom@localhost) by sucuri.mat.puc-rio.br (8.9.3/8.9.3) id KAA02649 for obm-l-MTTP; Tue, 29 Apr 2003 10:38:50 -0300 Received: from itaqui.terra.com.br (itaqui.terra.com.br [200.176.3.19]) by sucuri.mat.puc-rio.br (8.9.3/8.9.3) with ESMTP id KAA02643 for ; Tue, 29 Apr 2003 10:38:47 -0300 Received: from canela.terra.com.br (canela.terra.com.br [200.176.3.79]) by itaqui.terra.com.br (Postfix) with ESMTP id AF3D63BC695 for ; Tue, 29 Apr 2003 10:38:12 -0300 (BRT) Received: from riemann.localdomain (RJ174091.user.veloxzone.com.br [200.149.174.91]) (authenticated user fabio.dias.moreira) by canela.terra.com.br (Postfix) with ESMTP id 2CB942240D4 for ; Tue, 29 Apr 2003 10:38:12 -0300 (BRT) Content-Type: text/plain; charset="iso-8859-1" From: =?iso-8859-1?q?F=E1bio=20Dias=20Moreira?= To: obm-l@mat.puc-rio.br Subject: Re: [obm-l] Desigualdades Date: Tue, 29 Apr 2003 10:38:10 -0300 User-Agent: KMail/1.4.3 References: In-Reply-To: MIME-Version: 1.0 Content-Transfer-Encoding: 8bit Message-Id: <200304291038.17188.fabio.dias.moreira@terra.com.br> Sender: owner-obm-l@sucuri.mat.puc-rio.br Precedence: bulk Reply-To: obm-l@mat.puc-rio.br -----BEGIN PGP SIGNED MESSAGE----- Hash: SHA1 On Tuesday 29 April 2003 00:30, Claudio Buffara wrote: > on 28.04.03 23:01, Fábio Dias Moreira at fabio.dias.moreira@terra.com.br > > wrote: > > -----BEGIN PGP SIGNED MESSAGE----- > > Hash: SHA1 > > > > On Monday 28 April 2003 22:32, Raphael Marx wrote: > >> Olá a todos. Gostaria de pedir uma ajudinha em desigualdades: > >> 1-Prove que se v >> (v+x+y+z)^2>8(vy+xz) > >> [...] > > > > Divida por 16: > > > > ((v+x+y+z)/4)^2 > (vy+xz)/2 > sqrt(xyzv) > > (v+x+y+z)/4 > (xyzv)^(1/4) > > > > verdadeiro por MA-MG. > > > > []s, > > Oi, Fabio (e Raphael): > > MA-MG soh pode ser usada se v > 0, o que nao estah dito no enunciado. > [...] Mas acho que isso é necessário. Se v=-1, x=0, y=1/4 e z=3/4, temos que 0^2 > 8*(-1*0 + 1/4*3/4) <==> 3/2 < 0. > Alem disso, acho que tem um probleminha na logica: > > Supondo v > 0, ((v+x+y+z)/4)^2 > raiz(xyzv) e (vy+xz)/2 > raiz(xyzv) sao > ambas verdadeiras, mas isso nao implica necessariamente que: > ((v+x+y+z)/4)^2 > raiz(xyzv). > [...] A afirmação que usei foi: ((v+x+y+z)/4)^2 > (vy+xz)/2 e (vy+xz)/2 > raiz(xyzv), logo ((v+x+y+z)/4)^2 > raiz(xyzv), verdadeiro por transitividade. []s, - -- Fábio "ctg \pi" Dias Moreira -----BEGIN PGP SIGNATURE----- Version: GnuPG v1.0.6 (GNU/Linux) Comment: For info see http://www.gnupg.org iD8DBQE+roBIalOQFrvzGQoRAnSqAKCq7bW3Fq40d9750UfL/xEweMHSYACdEkUF PJmCZ9EuB91OjaNqPTnZhJs= =dNgk -----END PGP SIGNATURE----- ========================================================================= Instruções para entrar na lista, sair da lista e usar a lista em http://www.mat.puc-rio.br/~nicolau/olimp/obm-l.html ========================================================================= From owner-obm-l@sucuri.mat.puc-rio.br Tue Apr 29 13:32:19 2003 Return-Path: Received: (from majordom@localhost) by sucuri.mat.puc-rio.br (8.9.3/8.9.3) id NAA05933 for obm-l-MTTP; Tue, 29 Apr 2003 13:29:33 -0300 Received: from web12905.mail.yahoo.com (web12905.mail.yahoo.com [216.136.174.72]) by sucuri.mat.puc-rio.br (8.9.3/8.9.3) with SMTP id NAA05929 for ; Tue, 29 Apr 2003 13:29:29 -0300 Message-ID: <20030429162857.10358.qmail@web12905.mail.yahoo.com> Received: from [200.206.103.3] by web12905.mail.yahoo.com via HTTP; Tue, 29 Apr 2003 13:28:57 ART Date: Tue, 29 Apr 2003 13:28:57 -0300 (ART) From: =?iso-8859-1?q?Johann=20Peter=20Gustav=20Lejeune=20Dirichlet?= Subject: Re: [obm-l] ajuda com desigualdade(outra soluçao) To: obm-l@mat.puc-rio.br In-Reply-To: MIME-Version: 1.0 Content-Type: multipart/alternative; boundary="0-547771686-1051633737=:9490" Content-Transfer-Encoding: 8bit Sender: owner-obm-l@sucuri.mat.puc-rio.br Precedence: bulk Reply-To: obm-l@mat.puc-rio.br --0-547771686-1051633737=:9490 Content-Type: text/plain; charset=iso-8859-1 Content-Transfer-Encoding: 8bit Nao sei direito mas pode-se usar um teorema esperto,a Desigualdade da Abertura.Primeiro,abra tudo e cancele o que for preciso.Depois veja que tudo e Medias.Ou Abertura.Veja esta no site da OPM,seçao Links(mais precisamente em http://www.unl.edu/amc/a-activities/a4-for-students/ um otimo site com o artigo A=0(fatore!),a^3+b^3>=ab(a+b),e ai sai que 1/(a^3+b^3+abc)<=c/abc(a+b+c).Faz tres vezes e fim!!! Te Marcelo Rufino de Oliveira wrote:Esta é meio braçal. Tira o mínimo e depois multiplique cruzado. Depois de fazer umas mil contas e cortes possíveis, a desigualdade se transforma em: a^6.b^3 + a^6.c^3 + b^6.a^3 + b^6.c^3 + c^6.b^3 + c^6.a^3 >= 2(a^3 + b^3 + c^3)(a^2.b^2.c^2) Vamos provar agora que ----- Original Message ----- From: "guilherme S." To: Sent: Sunday, April 27, 2003 10:29 AM Subject: [obm-l] ajuda com desigualdade > prove que: > 1/(a^3+b^3+abc)+1/(a^3+c^3+abc)+1/(b^3+c^3+abc)<=1/abc > sendo a,b e c reais positivos > > _______________________________________________________________________ > Yahoo! Mail > O melhor e-mail gratuito da internet: 6MB de espaço, antivírus, acesso POP3, filtro contra spam. > http://br.mail.yahoo.com/ > ========================================================================= > Instruções para entrar na lista, sair da lista e usar a lista em > http://www.mat.puc-rio.br/~nicolau/olimp/obm-l.html > ========================================================================= > ========================================================================= Instruções para entrar na lista, sair da lista e usar a lista em http://www.mat.puc-rio.br/~nicolau/olimp/obm-l.html ========================================================================= --------------------------------- Yahoo! Mail O melhor e-mail gratuito da internet: 6MB de espaço, antivírus, acesso POP3, filtro contra spam. --0-547771686-1051633737=:9490 Content-Type: text/html; charset=iso-8859-1 Content-Transfer-Encoding: 8bit

Nao sei direito mas pode-se usar um teorema esperto,a Desigualdade da Abertura.Primeiro,abra tudo e cancele o que for preciso.Depois veja que tudo e Medias.Ou Abertura.Veja esta no site da OPM,seçao Links(mais precisamente em

http://www.unl.edu/amc/a-activities/a4-for-students/  um otimo site com o artigo A<B.)

Tem outra soluçao bem magica:

(a-b)(a²-b²)>=0(fatore!),a^3+b^3>=ab(a+b),e ai sai que 

1/(a^3+b^3+abc)<=c/abc(a+b+c).Faz tres vezes e fim!!!

Te

 Marcelo Rufino de Oliveira <marcelo_rufino@hotmail.com> wrote:

Esta é meio braçal.
Tira o mínimo e depois multiplique cruzado. Depois de fazer umas mil contas
e cortes possíveis, a desigualdade se transforma em:
a^6.b^3 + a^6.c^3 + b^6.a^3 + b^6.c^3 + c^6.b^3 + c^6.a^3 >= 2(a^3 + b^3 +
c^3)(a^2.b^2.c^2)

Vamos provar agora que




----- Original Message -----
From: "guilherme S."
To:
Sent: Sunday, April 27, 2003 10:29 AM
Subject: [obm-l] ajuda com desigualdade


> prove que:
> 1/(a^3+b^3+abc)+1/(a^3+c^3+abc)+1/(b^3+c^3+abc)<=1/abc
> sendo a,b e c reais positivos
>
> _______________________________________________________________________
> Yahoo! Mail
> O melhor e-mail gratuito da internet: 6MB de espaço, antivírus, acesso
POP3, filtro contra spam.
> http://br.mail.yahoo.com/
> =========================================================================
> Instruções para entrar na lista, sair da lista e usar a lista em
> http://www.mat.puc-rio.br/~nicolau/olimp/obm-l.html
> =========================================================================
>
=========================================================================
Instruções para entrar na lista, sair da lista e usar a lista em
http://www.mat.puc-rio.br/~nicolau/olimp/obm-l.html
=========================================================================



Yahoo! Mail
O melhor e-mail gratuito da internet: 6MB de espaço, antivírus, acesso POP3, filtro contra spam. --0-547771686-1051633737=:9490-- ========================================================================= Instruções para entrar na lista, sair da lista e usar a lista em http://www.mat.puc-rio.br/~nicolau/olimp/obm-l.html ========================================================================= From owner-obm-l@sucuri.mat.puc-rio.br Tue Apr 29 13:32:20 2003 Return-Path: Received: (from majordom@localhost) by sucuri.mat.puc-rio.br (8.9.3/8.9.3) id NAA05941 for obm-l-MTTP; Tue, 29 Apr 2003 13:29:40 -0300 Received: from web12905.mail.yahoo.com (web12905.mail.yahoo.com [216.136.174.72]) by sucuri.mat.puc-rio.br (8.9.3/8.9.3) with SMTP id NAA05937 for ; Tue, 29 Apr 2003 13:29:35 -0300 Message-ID: <20030429162904.10376.qmail@web12905.mail.yahoo.com> Received: from [200.206.103.3] by web12905.mail.yahoo.com via HTTP; Tue, 29 Apr 2003 13:29:04 ART Date: Tue, 29 Apr 2003 13:29:04 -0300 (ART) From: =?iso-8859-1?q?Johann=20Peter=20Gustav=20Lejeune=20Dirichlet?= Subject: Re: [obm-l] ajuda com desigualdade(outra soluçao) To: obm-l@mat.puc-rio.br In-Reply-To: MIME-Version: 1.0 Content-Type: multipart/alternative; boundary="0-1002073621-1051633744=:9485" Content-Transfer-Encoding: 8bit Sender: owner-obm-l@sucuri.mat.puc-rio.br Precedence: bulk Reply-To: obm-l@mat.puc-rio.br --0-1002073621-1051633744=:9485 Content-Type: text/plain; charset=iso-8859-1 Content-Transfer-Encoding: 8bit Nao sei direito mas pode-se usar um teorema esperto,a Desigualdade da Abertura.Primeiro,abra tudo e cancele o que for preciso.Depois veja que tudo e Medias.Ou Abertura.Veja esta no site da OPM,seçao Links(mais precisamente em http://www.unl.edu/amc/a-activities/a4-for-students/ um otimo site com o artigo A=0(fatore!),a^3+b^3>=ab(a+b),e ai sai que 1/(a^3+b^3+abc)<=c/abc(a+b+c).Faz tres vezes e fim!!! Te mais!!!! Marcelo Rufino de Oliveira wrote:Esta é meio braçal. Tira o mínimo e depois multiplique cruzado. Depois de fazer umas mil contas e cortes possíveis, a desigualdade se transforma em: a^6.b^3 + a^6.c^3 + b^6.a^3 + b^6.c^3 + c^6.b^3 + c^6.a^3 >= 2(a^3 + b^3 + c^3)(a^2.b^2.c^2) Vamos provar agora que ----- Original Message ----- From: "guilherme S." To: Sent: Sunday, April 27, 2003 10:29 AM Subject: [obm-l] ajuda com desigualdade > prove que: > 1/(a^3+b^3+abc)+1/(a^3+c^3+abc)+1/(b^3+c^3+abc)<=1/abc > sendo a,b e c reais positivos > > _______________________________________________________________________ > Yahoo! Mail > O melhor e-mail gratuito da internet: 6MB de espaço, antivírus, acesso POP3, filtro contra spam. > http://br.mail.yahoo.com/ > ========================================================================= > Instruções para entrar na lista, sair da lista e usar a lista em > http://www.mat.puc-rio.br/~nicolau/olimp/obm-l.html > ========================================================================= > ========================================================================= Instruções para entrar na lista, sair da lista e usar a lista em http://www.mat.puc-rio.br/~nicolau/olimp/obm-l.html ========================================================================= --------------------------------- Yahoo! Mail O melhor e-mail gratuito da internet: 6MB de espaço, antivírus, acesso POP3, filtro contra spam. --0-1002073621-1051633744=:9485 Content-Type: text/html; charset=iso-8859-1 Content-Transfer-Encoding: 8bit

Nao sei direito mas pode-se usar um teorema esperto,a Desigualdade da Abertura.Primeiro,abra tudo e cancele o que for preciso.Depois veja que tudo e Medias.Ou Abertura.Veja esta no site da OPM,seçao Links(mais precisamente em

http://www.unl.edu/amc/a-activities/a4-for-students/  um otimo site com o artigo A<B.)

Tem outra soluçao bem magica:

(a-b)(a²-b²)>=0(fatore!),a^3+b^3>=ab(a+b),e ai sai que 

1/(a^3+b^3+abc)<=c/abc(a+b+c).Faz tres vezes e fim!!!

Te mais!!!!

 Marcelo Rufino de Oliveira <marcelo_rufino@hotmail.com> wrote:

Esta é meio braçal.
Tira o mínimo e depois multiplique cruzado. Depois de fazer umas mil contas
e cortes possíveis, a desigualdade se transforma em:
a^6.b^3 + a^6.c^3 + b^6.a^3 + b^6.c^3 + c^6.b^3 + c^6.a^3 >= 2(a^3 + b^3 +
c^3)(a^2.b^2.c^2)

Vamos provar agora que




----- Original Message -----
From: "guilherme S."
To:
Sent: Sunday, April 27, 2003 10:29 AM
Subject: [obm-l] ajuda com desigualdade


> prove que:
> 1/(a^3+b^3+abc)+1/(a^3+c^3+abc)+1/(b^3+c^3+abc)<=1/abc
> sendo a,b e c reais positivos
>
> _______________________________________________________________________
> Yahoo! Mail
> O melhor e-mail gratuito da internet: 6MB de espaço, antivírus, acesso
POP3, filtro contra spam.
> http://br.mail.yahoo.com/
> =========================================================================
> Instruções para entrar na lista, sair da lista e usar a lista em
> http://www.mat.puc-rio.br/~nicolau/olimp/obm-l.html
> =========================================================================
>
=========================================================================
Instruções para entrar na lista, sair da lista e usar a lista em
http://www.mat.puc-rio.br/~nicolau/olimp/obm-l.html
=========================================================================



Yahoo! Mail
O melhor e-mail gratuito da internet: 6MB de espaço, antivírus, acesso POP3, filtro contra spam. --0-1002073621-1051633744=:9485-- ========================================================================= Instruções para entrar na lista, sair da lista e usar a lista em http://www.mat.puc-rio.br/~nicolau/olimp/obm-l.html ========================================================================= From owner-obm-l@sucuri.mat.puc-rio.br Tue Apr 29 13:47:38 2003 Return-Path: Received: (from majordom@localhost) by sucuri.mat.puc-rio.br (8.9.3/8.9.3) id NAA06505 for obm-l-MTTP; Tue, 29 Apr 2003 13:45:52 -0300 Received: from hotmail.com (f169.sea1.hotmail.com [207.68.163.169]) by sucuri.mat.puc-rio.br (8.9.3/8.9.3) with ESMTP id NAA06501 for ; Tue, 29 Apr 2003 13:45:47 -0300 Received: from mail pickup service by hotmail.com with Microsoft SMTPSVC; Tue, 29 Apr 2003 09:45:14 -0700 Received: from 198.81.9.1 by sea1fd.sea1.hotmail.msn.com with HTTP; Tue, 29 Apr 2003 16:45:14 GMT X-Originating-IP: [198.81.9.1] X-Originating-Email: [fredericor@hotmail.com] From: "Frederico Reis Marques de Brito" To: obm-l@mat.puc-rio.br Subject: Re: [obm-l] Ajuda em teorema Date: Tue, 29 Apr 2003 13:45:14 -0300 Mime-Version: 1.0 Content-Type: text/plain; charset=iso-8859-1; format=flowed Message-ID: X-OriginalArrivalTime: 29 Apr 2003 16:45:14.0720 (UTC) FILETIME=[B549A600:01C30E6E] Sender: owner-obm-l@sucuri.mat.puc-rio.br Precedence: bulk Reply-To: obm-l@mat.puc-rio.br Que curva??? >From: "Marcelo Souza" >Reply-To: obm-l@mat.puc-rio.br >To: obm-l@mat.puc-rio.br >Subject: [obm-l] Ajuda em teorema >Date: Tue, 29 Apr 2003 12:34:00 +0000 > >Alguém poderia exibir a prova do seguinte teorema para mim > >-Dada f(x) contínua e f'(t)<>0, então a curva pode ser expressa na forma >y=f(x). > >obs.: <>=diferente. >Agradeço >[]',M. > >_________________________________________________________________ >Help STOP SPAM with the new MSN 8 and get 2 months FREE* >http://join.msn.com/?page=features/junkmail > >========================================================================= >Instruções para entrar na lista, sair da lista e usar a lista em >http://www.mat.puc-rio.br/~nicolau/olimp/obm-l.html >========================================================================= _________________________________________________________________ MSN Hotmail, o maior webmail do Brasil. http://www.hotmail.com ========================================================================= Instruções para entrar na lista, sair da lista e usar a lista em http://www.mat.puc-rio.br/~nicolau/olimp/obm-l.html ========================================================================= From owner-obm-l@sucuri.mat.puc-rio.br Tue Apr 29 14:06:15 2003 Return-Path: Received: (from majordom@localhost) by sucuri.mat.puc-rio.br (8.9.3/8.9.3) id OAA07362 for obm-l-MTTP; Tue, 29 Apr 2003 14:04:22 -0300 Received: from web12906.mail.yahoo.com (web12906.mail.yahoo.com [216.136.174.73]) by sucuri.mat.puc-rio.br (8.9.3/8.9.3) with SMTP id OAA07347 for ; Tue, 29 Apr 2003 14:04:14 -0300 Message-ID: <20030429170341.7229.qmail@web12906.mail.yahoo.com> Received: from [200.206.103.3] by web12906.mail.yahoo.com via HTTP; Tue, 29 Apr 2003 14:03:41 ART Date: Tue, 29 Apr 2003 14:03:41 -0300 (ART) From: =?iso-8859-1?q?Johann=20Peter=20Gustav=20Lejeune=20Dirichlet?= Subject: Re: [obm-l] 3 circunferências To: obm-l@mat.puc-rio.br In-Reply-To: <20030428183144.20738.qmail@web14304.mail.yahoo.com> MIME-Version: 1.0 Content-Type: multipart/alternative; boundary="0-1529753429-1051635821=:6671" Content-Transfer-Encoding: 8bit Sender: owner-obm-l@sucuri.mat.puc-rio.br Precedence: bulk Reply-To: obm-l@mat.puc-rio.br --0-1529753429-1051635821=:6671 Content-Type: text/plain; charset=iso-8859-1 Content-Transfer-Encoding: 8bit Use Inversao ou trigonometria de gente grande Rafael wrote:Olá pessoal! Será que alguém consegue me dar uma ajuda nessa questão: Três circunferência de raios r , r' e R são tangentes, duas a duas, externamente. A tangente comun interna às duas primeiras circunferências intercepta a circunferência de raio R nos pontos A e B. Calcular a corda AB. resp: (4R raiz(rr')/(r + r') Abraços, Rafael. _______________________________________________________________________ Yahoo! Mail O melhor e-mail gratuito da internet: 6MB de espaço, antivírus, acesso POP3, filtro contra spam. http://br.mail.yahoo.com/ ========================================================================= Instruções para entrar na lista, sair da lista e usar a lista em http://www.mat.puc-rio.br/~nicolau/olimp/obm-l.html ========================================================================= --------------------------------- Yahoo! Mail O melhor e-mail gratuito da internet: 6MB de espaço, antivírus, acesso POP3, filtro contra spam. --0-1529753429-1051635821=:6671 Content-Type: text/html; charset=iso-8859-1 Content-Transfer-Encoding: 8bit

Use Inversao ou trigonometria de gente grande

 Rafael <matduvidas@yahoo.com.br> wrote:

Olá pessoal!

Será que alguém consegue me dar uma ajuda nessa
questão:

Três circunferência de raios r , r' e R são tangentes,
duas a duas, externamente. A tangente comun interna às
duas primeiras circunferências intercepta a
circunferência de raio R nos pontos A e B. Calcular a
corda AB.

resp: (4R raiz(rr')/(r + r')

Abraços,

Rafael.

_______________________________________________________________________
Yahoo! Mail
O melhor e-mail gratuito da internet: 6MB de espaço, antivírus, acesso POP3, filtro contra spam.
http://br.mail.yahoo.com/
=========================================================================
Instruções para entrar na lista, sair da lista e usar a lista em
http://www.mat.puc-rio.br/~nicolau/olimp/obm-l.html
=========================================================================



Yahoo! Mail
O melhor e-mail gratuito da internet: 6MB de espaço, antivírus, acesso POP3, filtro contra spam. --0-1529753429-1051635821=:6671-- ========================================================================= Instruções para entrar na lista, sair da lista e usar a lista em http://www.mat.puc-rio.br/~nicolau/olimp/obm-l.html ========================================================================= From owner-obm-l@sucuri.mat.puc-rio.br Tue Apr 29 14:35:21 2003 Return-Path: Received: (from majordom@localhost) by sucuri.mat.puc-rio.br (8.9.3/8.9.3) id OAA08597 for obm-l-MTTP; Tue, 29 Apr 2003 14:33:09 -0300 Received: from web12906.mail.yahoo.com (web12906.mail.yahoo.com [216.136.174.73]) by sucuri.mat.puc-rio.br (8.9.3/8.9.3) with SMTP id OAA08592 for ; Tue, 29 Apr 2003 14:33:04 -0300 Message-ID: <20030429173232.15090.qmail@web12906.mail.yahoo.com> Received: from [200.206.103.3] by web12906.mail.yahoo.com via HTTP; Tue, 29 Apr 2003 14:32:32 ART Date: Tue, 29 Apr 2003 14:32:32 -0300 (ART) From: =?iso-8859-1?q?Johann=20Peter=20Gustav=20Lejeune=20Dirichlet?= Subject: [obm-l] Problema da segunda fase da OBM-Outra Soluçao To: obm-l@mat.puc-rio.br MIME-Version: 1.0 Content-Type: multipart/alternative; boundary="0-895718206-1051637552=:14097" Content-Transfer-Encoding: 8bit Sender: owner-obm-l@sucuri.mat.puc-rio.br Precedence: bulk Reply-To: obm-l@mat.puc-rio.br --0-895718206-1051637552=:14097 Content-Type: text/plain; charset=iso-8859-1 Content-Transfer-Encoding: 8bit Ola pessoal!!!!!Ano passado consegui essa soluçao.Sera que alguem pode corrigi-la?(2ª fase OBM 2003)Considere o quadrilatero ABCD com AB=CD.Mostre que a reta que liga os pontos medios de AC e BD determina angulos iguais com os lados AB e CD. SOLUÇAO:considere o ponto E de intersecçao de AB e CD,M medio de BD,N medio de AC,X e Y onde MN corta AB e CD,na ordem,K monde AD corta XY.Suponha que MN e AD sejam concorrentes(se nao forem o teorema e obvio).Vamos provar que EX=EYMenelaos na reta XMKtriangulo ABD:AK/KD*DM/MB*BX/XA=1triangulo ACD:AK/KD*DY/YC*CN/NA=1triangulo EBD:EX/XB*BM/MD*DY/YE=1 Logo BX/XA=DY/YC(as duas primeiras),e como AB=CD,temos BX=DY.Pondo na ultima vemos que sai o pedido:EX=EY,e ai o triangulo XEY e isosceles.Pronto,fim!Essa dava pra ir no site da OBM ou Eureka!Te mais!!!!!!Ass.:Johann --------------------------------- Yahoo! Mail O melhor e-mail gratuito da internet: 6MB de espaço, antivírus, acesso POP3, filtro contra spam. --0-895718206-1051637552=:14097 Content-Type: text/html; charset=iso-8859-1 Content-Transfer-Encoding: 8bit
Ola pessoal!!!!!Ano passado consegui essa soluçao.Sera que alguem pode corrigi-la?
(2ª fase OBM 2003)
Considere o quadrilatero ABCD com AB=CD.Mostre que a reta que liga os pontos medios de AC e BD determina angulos iguais com os lados AB e CD.
 
SOLUÇAO:considere o ponto E de intersecçao de AB e CD,M medio de BD,N medio de AC,X e Y onde MN corta AB e CD,na ordem,K monde AD corta XY.Suponha que MN e AD sejam concorrentes(se nao forem o teorema e obvio).Vamos provar que EX=EY
Menelaos na reta XMK
triangulo ABD:
AK/KD*DM/MB*BX/XA=1
triangulo ACD:
AK/KD*DY/YC*CN/NA=1
triangulo EBD:
EX/XB*BM/MD*DY/YE=1
 
Logo BX/XA=DY/YC(as duas primeiras),e como AB=CD,temos BX=DY.Pondo na ultima vemos que sai o pedido:EX=EY,e ai o triangulo XEY e isosceles.Pronto,fim!
Essa dava pra ir no site da OBM ou Eureka!
Te mais!!!!!!Ass.:Johann



Yahoo! Mail
O melhor e-mail gratuito da internet: 6MB de espaço, antivírus, acesso POP3, filtro contra spam. --0-895718206-1051637552=:14097-- ========================================================================= Instruções para entrar na lista, sair da lista e usar a lista em http://www.mat.puc-rio.br/~nicolau/olimp/obm-l.html ========================================================================= From owner-obm-l@sucuri.mat.puc-rio.br Tue Apr 29 15:03:10 2003 Return-Path: Received: (from majordom@localhost) by sucuri.mat.puc-rio.br (8.9.3/8.9.3) id PAA09401 for obm-l-MTTP; Tue, 29 Apr 2003 15:01:14 -0300 Received: from web12908.mail.yahoo.com (web12908.mail.yahoo.com [216.136.174.75]) by sucuri.mat.puc-rio.br (8.9.3/8.9.3) with SMTP id PAA09396 for ; Tue, 29 Apr 2003 15:01:10 -0300 Message-ID: <20030429180038.19362.qmail@web12908.mail.yahoo.com> Received: from [200.206.103.3] by web12908.mail.yahoo.com via HTTP; Tue, 29 Apr 2003 15:00:38 ART Date: Tue, 29 Apr 2003 15:00:38 -0300 (ART) From: =?iso-8859-1?q?Johann=20Peter=20Gustav=20Lejeune=20Dirichlet?= Subject: Re: [obm-l] Problema do Dirichlet To: obm-l@mat.puc-rio.br In-Reply-To: <20030428161015.814.qmail@web12906.mail.yahoo.com> MIME-Version: 1.0 Content-Type: multipart/alternative; boundary="0-192615005-1051639238=:19346" Content-Transfer-Encoding: 8bit Sender: owner-obm-l@sucuri.mat.puc-rio.br Precedence: bulk Reply-To: obm-l@mat.puc-rio.br --0-192615005-1051639238=:19346 Content-Type: text/plain; charset=iso-8859-1 Content-Transfer-Encoding: 8bit Na verdade se vermos bem Menelaos e Ceva sao teoremas puramente vetoriais.Alias na minha opiniao:pontos perdidos no meio do nada sao a primeira pista de que Vetores ajuda. Vou passar um bem interessante da Balcanica(acho).Quem resolver com trigonometria meus parabens!!!!! No triangulo ABC de circuncentro O,seja D medio de AB,G centroide(ou baricentro) de ACDMostre que CD faz () graus com OG se e so se BA=AC. --------------------------------- Yahoo! Mail O melhor e-mail gratuito da internet: 6MB de espaço, antivírus, acesso POP3, filtro contra spam. --0-192615005-1051639238=:19346 Content-Type: text/html; charset=iso-8859-1 Content-Transfer-Encoding: 8bit

Na verdade se vermos bem Menelaos e Ceva sao teoremas puramente vetoriais.Alias na minha opiniao:pontos perdidos no meio do nada sao a primeira pista de que Vetores ajuda.

Vou passar um bem interessante da Balcanica(acho).Quem resolver com trigonometria meus parabens!!!!!

No triangulo ABC de circuncentro O,seja D medio de AB,G centroide(ou baricentro) de ACDMostre que CD faz () graus com OG se e so se BA=AC.



Yahoo! Mail
O melhor e-mail gratuito da internet: 6MB de espaço, antivírus, acesso POP3, filtro contra spam. --0-192615005-1051639238=:19346-- ========================================================================= Instruções para entrar na lista, sair da lista e usar a lista em http://www.mat.puc-rio.br/~nicolau/olimp/obm-l.html ========================================================================= From owner-obm-l@sucuri.mat.puc-rio.br Tue Apr 29 15:12:49 2003 Return-Path: Received: (from majordom@localhost) by sucuri.mat.puc-rio.br (8.9.3/8.9.3) id PAA09842 for obm-l-MTTP; Tue, 29 Apr 2003 15:11:11 -0300 Received: from hotmail.com (f22.law15.hotmail.com [64.4.23.22]) by sucuri.mat.puc-rio.br (8.9.3/8.9.3) with ESMTP id PAA09831 for ; Tue, 29 Apr 2003 15:11:05 -0300 Received: from mail pickup service by hotmail.com with Microsoft SMTPSVC; Tue, 29 Apr 2003 11:10:33 -0700 Received: from 200.174.69.242 by lw15fd.law15.hotmail.msn.com with HTTP; Tue, 29 Apr 2003 18:10:32 GMT X-Originating-IP: [200.174.69.242] X-Originating-Email: [piuwee@hotmail.com] From: "Felipe Marinho" To: obm-l@mat.puc-rio.br Subject: [obm-l] =?iso-8859-1?B?Tm92YW1lbnRlOiBFc3Bh528gVmV0b3JpYWwg?= Date: Tue, 29 Apr 2003 14:10:32 -0400 Mime-Version: 1.0 Content-Type: text/plain; charset=iso-8859-1; format=flowed Message-ID: X-OriginalArrivalTime: 29 Apr 2003 18:10:33.0147 (UTC) FILETIME=[A01BA0B0:01C30E7A] Sender: owner-obm-l@sucuri.mat.puc-rio.br Precedence: bulk Reply-To: obm-l@mat.puc-rio.br Olá pessoal, Venho aqui agradecer a todos pelas respostas enviadas a respeito do problema que eu havia apresentado. Mas olha, na verdade, acho que as respostas de vocês ficaram um pouco difíceis de serem entendidas por mim... Os conhecimentos de vocês vão muito além dos meus.. ;) Então olha, soh... para o problema: "Um Espaço Vetorial pode ser formado por EXATAMENTE dois vetores distintos?" Bem, eu andei pensando bastante... e conclui o seguinte: Todo Espaço Vetorial V qualquer sobre um corpo K, admite como subespaços vetoriais de imediato ele mesmo e o {0}. Ou seja, o próprio V e {0} são subespaços vetoriais de V. Para provar entao que um espaço vetorial pode ser formado por exatamente 2 vetores distintos, o problema limita-se a provar a existencia de subespaços vetoriais formados por exatamente 2 vetores distintos. Conclui entao que estes vetores existem e devem ter direções paralelas(pois aí sao fechados na adição e multiplicacao por escalar). Entao existe um Espaco Vetorial formado por exatamente 2 vetores distintos SE E SOMENTE SE suas direções forem paralelas, pois caso contrário, os 2 vetores não serão subespaços vetorias de V. Agora peço a ajuda de vocês para analisarem minha resposta. Tem nexo oque eu disse ? ;)) Aguardo mesmo a ajuda de vocês! E espero ter conseguido passar as minhas idéias. Agradeço desde já... E um grande abraço a todos... Felipe Marinho _________________________________________________________________ MSN Messenger: converse com os seus amigos online. http://messenger.msn.com.br ========================================================================= Instruções para entrar na lista, sair da lista e usar a lista em http://www.mat.puc-rio.br/~nicolau/olimp/obm-l.html ========================================================================= From owner-obm-l@sucuri.mat.puc-rio.br Tue Apr 29 15:19:01 2003 Return-Path: Received: (from majordom@localhost) by sucuri.mat.puc-rio.br (8.9.3/8.9.3) id PAA10086 for obm-l-MTTP; Tue, 29 Apr 2003 15:17:18 -0300 Received: from hotmail.com (bay1-f41.bay1.hotmail.com [65.54.245.41]) by sucuri.mat.puc-rio.br (8.9.3/8.9.3) with ESMTP id PAA10082 for ; Tue, 29 Apr 2003 15:17:10 -0300 Received: from mail pickup service by hotmail.com with Microsoft SMTPSVC; Tue, 29 Apr 2003 11:16:30 -0700 Received: from 146.164.44.58 by by1fd.bay1.hotmail.msn.com with HTTP; Tue, 29 Apr 2003 18:16:30 GMT X-Originating-IP: [146.164.44.58] X-Originating-Email: [marcelo_souza7@hotmail.com] From: "Marcelo Souza" To: obm-l@mat.puc-rio.br Subject: Re: [obm-l] Ajuda em teorema Date: Tue, 29 Apr 2003 18:16:30 +0000 Mime-Version: 1.0 Content-Type: text/plain; format=flowed Message-ID: X-OriginalArrivalTime: 29 Apr 2003 18:16:30.0479 (UTC) FILETIME=[751829F0:01C30E7B] Sender: owner-obm-l@sucuri.mat.puc-rio.br Precedence: bulk Reply-To: obm-l@mat.puc-rio.br Bom, o problema foi me passado assim, mas creio que seja a curva descrita por f(x)...corrijam-me se estiver errado... []'s, M. >From: "Frederico Reis Marques de Brito" >Reply-To: obm-l@mat.puc-rio.br >To: obm-l@mat.puc-rio.br >Subject: Re: [obm-l] Ajuda em teorema >Date: Tue, 29 Apr 2003 13:45:14 -0300 > >Que curva??? > > > > > > >>From: "Marcelo Souza" >>Reply-To: obm-l@mat.puc-rio.br >>To: obm-l@mat.puc-rio.br >>Subject: [obm-l] Ajuda em teorema >>Date: Tue, 29 Apr 2003 12:34:00 +0000 >> >>Alguém poderia exibir a prova do seguinte teorema para mim >> >>-Dada f(x) contínua e f'(t)<>0, então a curva pode ser expressa na forma >>y=f(x). >> >>obs.: <>=diferente. >>Agradeço >>[]',M. >> >>_________________________________________________________________ >>Help STOP SPAM with the new MSN 8 and get 2 months FREE* >>http://join.msn.com/?page=features/junkmail >> >>========================================================================= >>Instruções para entrar na lista, sair da lista e usar a lista em >>http://www.mat.puc-rio.br/~nicolau/olimp/obm-l.html >>========================================================================= > > >_________________________________________________________________ >MSN Hotmail, o maior webmail do Brasil. http://www.hotmail.com > >========================================================================= >Instruções para entrar na lista, sair da lista e usar a lista em >http://www.mat.puc-rio.br/~nicolau/olimp/obm-l.html >========================================================================= _________________________________________________________________ Protect your PC - get McAfee.com VirusScan Online http://clinic.mcafee.com/clinic/ibuy/campaign.asp?cid=3963 ========================================================================= Instruções para entrar na lista, sair da lista e usar a lista em http://www.mat.puc-rio.br/~nicolau/olimp/obm-l.html ========================================================================= From owner-obm-l@sucuri.mat.puc-rio.br Tue Apr 29 15:43:39 2003 Return-Path: Received: (from majordom@localhost) by sucuri.mat.puc-rio.br (8.9.3/8.9.3) id PAA11345 for obm-l-MTTP; Tue, 29 Apr 2003 15:41:31 -0300 Received: (from nicolau@localhost) by sucuri.mat.puc-rio.br (8.9.3/8.9.3) id PAA11340 for obm-l@mat.puc-rio.br; Tue, 29 Apr 2003 15:41:30 -0300 Date: Tue, 29 Apr 2003 15:41:30 -0300 From: "Nicolau C. Saldanha" To: obm-l@mat.puc-rio.br Subject: Re: [obm-l] Falha nossa Message-ID: <20030429154130.D10163@sucuri.mat.puc-rio.br> References: <20030426104039.A8053@sucuri.mat.puc-rio.br> <20030428085810.C25567@sucuri.mat.puc-rio.br> <025c01c30ddf$933fb5c0$5400a8c0@ensrbr> Mime-Version: 1.0 Content-Type: text/plain; charset=iso-8859-1 Content-Disposition: inline Content-Transfer-Encoding: 8bit User-Agent: Mutt/1.2.5i In-Reply-To: <025c01c30ddf$933fb5c0$5400a8c0@ensrbr>; from llopes@ensrbr.com.br on Mon, Apr 28, 2003 at 08:40:36PM -0300 Sender: owner-obm-l@sucuri.mat.puc-rio.br Precedence: bulk Reply-To: obm-l@mat.puc-rio.br On Mon, Apr 28, 2003 at 08:40:36PM -0300, Luis Lopes wrote: > Sauda,c~oes, > > Oi Nicolau, > > Sabemos que S(0)=0 e subindo uma linha > na tabela de diferenças encontramos > S(-1), S(-2), etc. Estamos falando de PA > de ordem k e em particular k=3. > > \Delta^{-1} a_i = S(i) > \Delta^{0} a_i = a_i > > Assim faz sentido S(n) para TODO n. Mas > não sabia dar uma interpretação para a > soma quando n<= 0. > > Na verdade é convencionado que S(n)=0 > para S(n)= \sum_{i=1}^n a_i se n<=0. > > Fiquei intrigado quando vc escreveu > > > > S(n) = - ( (-1)^3 + (-2)^3 + ... + (n+1)^3 ), n < -1 > > e pensava em como seria uma expressão > equivalente para uma outra PA de ordem 3. > > Estava matutando com isso e agora chegou > esta sua msg. > > Pelo que entendi da notação de Iverson, > S(n)=0 para n<=0. Não. S(3) = a_1 + a_2 + a_3 S(2) = a_1 + a_2 S(1) = a_1 S(0) = 0 S(-1) = - a_0 S(-2) = - a_0 - a_{-1} S(-3) = - a_0 - a_{-1} - a_{-2} Ou mais geralmente para qq n > 1 S(-n) = - a_0 - a_{-1} - ... - a_{-n+1} ou equivalentemente, para n < -1, S(n) = - a_0 - a_{-1} - ... - a_{n+1} []s, N. ========================================================================= Instruções para entrar na lista, sair da lista e usar a lista em http://www.mat.puc-rio.br/~nicolau/olimp/obm-l.html ========================================================================= From owner-obm-l@sucuri.mat.puc-rio.br Tue Apr 29 15:46:29 2003 Return-Path: Received: (from majordom@localhost) by sucuri.mat.puc-rio.br (8.9.3/8.9.3) id PAA11497 for obm-l-MTTP; Tue, 29 Apr 2003 15:45:08 -0300 Received: (from nicolau@localhost) by sucuri.mat.puc-rio.br (8.9.3/8.9.3) id PAA11492 for obm-l@mat.puc-rio.br; Tue, 29 Apr 2003 15:45:07 -0300 Date: Tue, 29 Apr 2003 15:45:07 -0300 From: "Nicolau C. Saldanha" To: obm-l@mat.puc-rio.br Subject: Re: [obm-l] Falha nossa Message-ID: <20030429154507.E10163@sucuri.mat.puc-rio.br> References: <20030426104039.A8053@sucuri.mat.puc-rio.br> <20030428085810.C25567@sucuri.mat.puc-rio.br> <025c01c30ddf$933fb5c0$5400a8c0@ensrbr> Mime-Version: 1.0 Content-Type: text/plain; charset=iso-8859-1 Content-Disposition: inline Content-Transfer-Encoding: 8bit User-Agent: Mutt/1.2.5i In-Reply-To: <025c01c30ddf$933fb5c0$5400a8c0@ensrbr>; from llopes@ensrbr.com.br on Mon, Apr 28, 2003 at 08:40:36PM -0300 Sender: owner-obm-l@sucuri.mat.puc-rio.br Precedence: bulk Reply-To: obm-l@mat.puc-rio.br On Mon, Apr 28, 2003 at 08:40:36PM -0300, Luis Lopes wrote: > Pelo que entendi da notação de Iverson, > S(n)=0 para n<=0. S(n) = soma_k ([k > 0] - [k > n]) (k^3) Se n < 0 então ([k > 0] - [k > n]) vale -1 para os inteiros k que satisfazem k>n mas não satisfazem k>0, ou seja, n+1, n+2, ...., -1, 0. []s, N. ========================================================================= Instruções para entrar na lista, sair da lista e usar a lista em http://www.mat.puc-rio.br/~nicolau/olimp/obm-l.html ========================================================================= From owner-obm-l@sucuri.mat.puc-rio.br Tue Apr 29 16:12:09 2003 Return-Path: Received: (from majordom@localhost) by sucuri.mat.puc-rio.br (8.9.3/8.9.3) id QAA12999 for obm-l-MTTP; Tue, 29 Apr 2003 16:10:28 -0300 Received: from web14311.mail.yahoo.com (web14311.mail.yahoo.com [216.136.224.61]) by sucuri.mat.puc-rio.br (8.9.3/8.9.3) with SMTP id QAA12984 for ; Tue, 29 Apr 2003 16:10:21 -0300 Message-ID: <20030429190946.52281.qmail@web14311.mail.yahoo.com> Received: from [200.17.25.3] by web14311.mail.yahoo.com via HTTP; Tue, 29 Apr 2003 16:09:46 ART Date: Tue, 29 Apr 2003 16:09:46 -0300 (ART) From: =?iso-8859-1?q?Rafael?= Subject: Re: [obm-l] 3 circunferências To: obm-l@mat.puc-rio.br In-Reply-To: <20030429170341.7229.qmail@web12906.mail.yahoo.com> MIME-Version: 1.0 Content-Type: text/plain; charset=iso-8859-1 Content-Transfer-Encoding: 8bit Sender: owner-obm-l@sucuri.mat.puc-rio.br Precedence: bulk Reply-To: obm-l@mat.puc-rio.br Por exemplo? Rafael. --- Johann Peter Gustav Lejeune Dirichlet escreveu: > > Use Inversao ou trigonometria de gente grande > Rafael wrote:Olá pessoal! > > Será que alguém consegue me dar uma ajuda nessa > questão: > > Três circunferência de raios r , r' e R são > tangentes, > duas a duas, externamente. A tangente comun interna > às > duas primeiras circunferências intercepta a > circunferência de raio R nos pontos A e B. Calcular > a > corda AB. > > resp: (4R raiz(rr')/(r + r') > > Abraços, > > Rafael. > > _______________________________________________________________________ > Yahoo! Mail > O melhor e-mail gratuito da internet: 6MB de espaço, > antivírus, acesso POP3, filtro contra spam. > http://br.mail.yahoo.com/ > ========================================================================= > Instruções para entrar na lista, sair da lista e > usar a lista em > http://www.mat.puc-rio.br/~nicolau/olimp/obm-l.html > ========================================================================= > > > --------------------------------- > Yahoo! Mail > O melhor e-mail gratuito da internet: 6MB de espaço, > antivírus, acesso POP3, filtro contra spam. _______________________________________________________________________ Yahoo! Mail O melhor e-mail gratuito da internet: 6MB de espaço, antivírus, acesso POP3, filtro contra spam. http://br.mail.yahoo.com/ ========================================================================= Instruções para entrar na lista, sair da lista e usar a lista em http://www.mat.puc-rio.br/~nicolau/olimp/obm-l.html ========================================================================= From owner-obm-l@sucuri.mat.puc-rio.br Tue Apr 29 16:31:41 2003 Return-Path: Received: (from majordom@localhost) by sucuri.mat.puc-rio.br (8.9.3/8.9.3) id QAA14181 for obm-l-MTTP; Tue, 29 Apr 2003 16:29:21 -0300 Received: from salem.bol.com.br (salem.bol.com.br [200.221.24.25]) by sucuri.mat.puc-rio.br (8.9.3/8.9.3) with ESMTP id QAA14177 for ; Tue, 29 Apr 2003 16:29:17 -0300 Received: from bol.com.br (200.221.24.136) by salem.bol.com.br (5.1.071) id 3EA6DA33001CE4C0 for obm-l@mat.puc-rio.br; Tue, 29 Apr 2003 16:28:39 -0300 Date: Tue, 29 Apr 2003 16:28:39 -0300 Message-Id: Subject: [obm-l] =?iso-8859-1?q?Prazo_de_inscri=E7=E3o_na_OBM?= MIME-Version: 1.0 Content-Type: text/plain;charset="iso-8859-1" From: "rmr-olimp" To: obm-l@mat.puc-rio.br X-XaM3-API-Version: 2.4 R3 ( B4 ) X-SenderIP: 200.206.193.76 Content-Transfer-Encoding: 8bit X-MIME-Autoconverted: from quoted-printable to 8bit by sucuri.mat.puc-rio.br id QAA14178 Sender: owner-obm-l@sucuri.mat.puc-rio.br Precedence: bulk Reply-To: obm-l@mat.puc-rio.br E aí galera... alguém sabe até que dia posso me inscrever na OBM? __________________________________________________________________________ Seleção de Softwares UOL. 10 softwares escolhidos pelo UOL para você e sua família. http://www.uol.com.br/selecao ========================================================================= Instruções para entrar na lista, sair da lista e usar a lista em http://www.mat.puc-rio.br/~nicolau/olimp/obm-l.html ========================================================================= From owner-obm-l@sucuri.mat.puc-rio.br Tue Apr 29 16:41:30 2003 Return-Path: Received: (from majordom@localhost) by sucuri.mat.puc-rio.br (8.9.3/8.9.3) id QAA14712 for obm-l-MTTP; Tue, 29 Apr 2003 16:40:08 -0300 Received: from paiol.terra.com.br (paiol.terra.com.br [200.176.3.18]) by sucuri.mat.puc-rio.br (8.9.3/8.9.3) with ESMTP id QAA14708 for ; Tue, 29 Apr 2003 16:40:03 -0300 Received: from barra.terra.com.br (barra.terra.com.br [200.176.3.52]) by paiol.terra.com.br (Postfix) with ESMTP id D9BF287F03 for ; Tue, 29 Apr 2003 16:39:27 -0300 (BRT) Received: from niski.com (unknown [200.148.197.82]) (authenticated user fniski) by barra.terra.com.br (Postfix) with ESMTP id 217972340C2 for ; Tue, 29 Apr 2003 16:39:20 -0300 (BRT) Message-ID: <3EAED4F0.9040000@niski.com> Date: Tue, 29 Apr 2003 12:39:28 -0700 From: niski User-Agent: Mozilla/5.0 (Windows; U; Windows NT 5.1; en-US; rv:1.0.2) Gecko/20030208 Netscape/7.02 X-Accept-Language: en-us, en MIME-Version: 1.0 To: obm-l@mat.puc-rio.br Subject: [obm-l] Limites no infinito Content-Type: text/plain; charset=ISO-8859-1; format=flowed Content-Transfer-Encoding: 8bit Sender: owner-obm-l@sucuri.mat.puc-rio.br Precedence: bulk Reply-To: obm-l@mat.puc-rio.br Por favor pessoal, me ajudem neste exercicio que empaquei. Obrigado Sejam f e g definidas em [a,+inf[ tais que f(x) >= e g(x) > 0 para todo x >= a. Suponha que lim[x->+inf] f(x)/g(x) = L, L >0. Prove que existe r > 0 , r > a, tal que para todo x>r (L/2)g(x) < f(x) < (3L/2)g(x). Conclua daí que se lim[x->+inf] g(x) = 0, entao lim[x->+inf] f(x) = 0. mais uma vez, obrigado. -- [about him:] It is rare to find learned men who are clean, do not stink and have a sense of humour. -Gottfried Whilhem Leibniz ========================================================================= Instruções para entrar na lista, sair da lista e usar a lista em http://www.mat.puc-rio.br/~nicolau/olimp/obm-l.html ========================================================================= From owner-obm-l@sucuri.mat.puc-rio.br Tue Apr 29 16:44:51 2003 Return-Path: Received: (from majordom@localhost) by sucuri.mat.puc-rio.br (8.9.3/8.9.3) id QAA14985 for obm-l-MTTP; Tue, 29 Apr 2003 16:43:12 -0300 Received: from web14304.mail.yahoo.com (web14304.mail.yahoo.com [216.136.173.80]) by sucuri.mat.puc-rio.br (8.9.3/8.9.3) with SMTP id QAA14977 for ; Tue, 29 Apr 2003 16:43:07 -0300 Message-ID: <20030429194234.19832.qmail@web14304.mail.yahoo.com> Received: from [200.17.25.3] by web14304.mail.yahoo.com via HTTP; Tue, 29 Apr 2003 16:42:34 ART Date: Tue, 29 Apr 2003 16:42:34 -0300 (ART) From: =?iso-8859-1?q?Rafael?= Subject: [obm-l] fatoração To: OBM MIME-Version: 1.0 Content-Type: text/plain; charset=iso-8859-1 Content-Transfer-Encoding: 8bit Sender: owner-obm-l@sucuri.mat.puc-rio.br Precedence: bulk Reply-To: obm-l@mat.puc-rio.br Pessoal, Achei essa pergunta num livro: Onúmero de fatores primos do número 100895598169 é: Tudo bem, achei que a resposta é 2 porque fui no computador e procurei que: 100895598169 = 112303 * 898423 Mas estando esse exercício num livro de segundo grau, deve haver uma maneira de se resolver isso sem uma calculadora e sem ir tentando primo por primo até o 112303... Alguém saberia? Abraços, Rafael. _______________________________________________________________________ Yahoo! Mail O melhor e-mail gratuito da internet: 6MB de espaço, antivírus, acesso POP3, filtro contra spam. http://br.mail.yahoo.com/ ========================================================================= Instruções para entrar na lista, sair da lista e usar a lista em http://www.mat.puc-rio.br/~nicolau/olimp/obm-l.html ========================================================================= From owner-obm-l@sucuri.mat.puc-rio.br Tue Apr 29 17:07:51 2003 Return-Path: Received: (from majordom@localhost) by sucuri.mat.puc-rio.br (8.9.3/8.9.3) id RAA16251 for obm-l-MTTP; Tue, 29 Apr 2003 17:06:27 -0300 Received: from trex-b.centroin.com.br (trex-b.centroin.com.br [200.225.63.136]) by sucuri.mat.puc-rio.br (8.9.3/8.9.3) with ESMTP id RAA16247 for ; Tue, 29 Apr 2003 17:06:23 -0300 Received: from centroin.com.br (RJ091019.user.veloxzone.com.br [200.141.91.19] (may be forged)) (authenticated bits=0) by trex-b.centroin.com.br (8.12.9/8.12.9) with ESMTP id h3TK5qVi021645 for ; Tue, 29 Apr 2003 17:05:53 -0300 (EST) Message-ID: <3EAEDB61.1010108@centroin.com.br> Date: Tue, 29 Apr 2003 17:06:57 -0300 From: "A. C. Morgado" User-Agent: Mozilla/5.0 (Windows; U; Windows NT 5.0; en-US; rv:1.0.2) Gecko/20030208 Netscape/7.02 X-Accept-Language: en-us, en MIME-Version: 1.0 To: obm-l@mat.puc-rio.br Subject: Re: [obm-l] Limites no infinito References: <3EAED4F0.9040000@niski.com> Content-Type: text/plain; charset=ISO-8859-1; format=flowed Content-Transfer-Encoding: 8bit Sender: owner-obm-l@sucuri.mat.puc-rio.br Precedence: bulk Reply-To: obm-l@mat.puc-rio.br Uai, a desigualdade eh a definiçao de lim f(x)/g(x) = L, com epsilon igual a L/2 e com o denominador eliminado por uma multiplicaçao por g(x). A conclusao eh Sanduiche! niski wrote: > Por favor pessoal, me ajudem neste exercicio que empaquei. Obrigado > > Sejam f e g definidas em [a,+inf[ tais que f(x) >= e g(x) > 0 para > todo x >= a. Suponha que lim[x->+inf] f(x)/g(x) = L, L >0. > Prove que existe r > 0 , r > a, tal que para todo x>r > > (L/2)g(x) < f(x) < (3L/2)g(x). > > Conclua daí que se lim[x->+inf] g(x) = 0, entao lim[x->+inf] f(x) = 0. > > mais uma vez, obrigado. > ========================================================================= Instruções para entrar na lista, sair da lista e usar a lista em http://www.mat.puc-rio.br/~nicolau/olimp/obm-l.html ========================================================================= From owner-obm-l@sucuri.mat.puc-rio.br Tue Apr 29 17:15:20 2003 Return-Path: Received: (from majordom@localhost) by sucuri.mat.puc-rio.br (8.9.3/8.9.3) id RAA16538 for obm-l-MTTP; Tue, 29 Apr 2003 17:13:55 -0300 Received: from hotmail.com (oe47.law10.hotmail.com [64.4.14.19]) by sucuri.mat.puc-rio.br (8.9.3/8.9.3) with ESMTP id RAA16530 for ; Tue, 29 Apr 2003 17:13:50 -0300 Received: from mail pickup service by hotmail.com with Microsoft SMTPSVC; Tue, 29 Apr 2003 13:11:54 -0700 Received: from 64.60.139.18 by oe47.law10.hotmail.com with DAV; Tue, 29 Apr 2003 20:11:54 +0000 X-Originating-IP: [64.60.139.18] X-Originating-Email: [lrecova@hotmail.com] From: =?iso-8859-1?Q?Leandro_Lacorte_Rec=F4va?= To: Subject: RE: [obm-l] Provas Militares Date: Tue, 29 Apr 2003 13:11:54 -0700 Message-ID: <000401c30e8b$94315590$28029b9b@LeandroRecova> MIME-Version: 1.0 Content-Type: text/plain; charset="iso-8859-1" X-Priority: 3 (Normal) X-MSMail-Priority: Normal X-Mailer: Microsoft Outlook, Build 10.0.3416 In-Reply-To: X-MimeOLE: Produced By Microsoft MimeOLE V6.00.2800.1165 Importance: Normal X-OriginalArrivalTime: 29 Apr 2003 20:11:54.0453 (UTC) FILETIME=[941AC050:01C30E8B] Content-Transfer-Encoding: 8bit X-MIME-Autoconverted: from quoted-printable to 8bit by sucuri.mat.puc-rio.br id RAA16532 Sender: owner-obm-l@sucuri.mat.puc-rio.br Precedence: bulk Reply-To: obm-l@mat.puc-rio.br Eu ja forneci a resposta em um email da lista. Verifique ok ! Anyway, aqui esta a resposta novamente ! i)(EN/95) O lim(x->0) [raiz(x+b)+raiz(x+a)-raiz(b)-raiz (a)]/x é igual a: resp: 1/[2*raiz(b)] + 1/[2*raiz(a)] Separe os termos e "desracionalize" lim(x->0) [raiz(x+b)-raiz(b)]/x + {raiz(x+a)-a]/x= lim(x->0) 1/[raiz(x+b)+raiz(b)] + 1/[raiz(x+a)+raiz(a)]= 1/2(raiz(b) + 1/2(raiz(a)) ii)(EN/95) Se f(x) = e^(2x) + (x+1)*cos(x), então f'(0) é igual a: resp: 3. f'(x)=2e^(2x)+cos^2(x)-(x+1)sin(x) f'(0)=2+1-0 = 3. iii)(EN/97) O valor da integral de raiz[1 + 9*x*(dx)] é: a)(2/27)*(1+9*x)^(3/2) + c Seria integral (raiz(1+9x))dx ? Se for isso, entao temos Chame u = 1 + 9x entao du = 9dx, logo, a integral fica, Int(raiz(1+9x))dx = Int[(raiz(u))/9 ]du = 2/3 (raiz(u))^3/9 + c, Substituindo u=1+9x, chegamos na resposta desejada. iv)(EN/98)A equação do movimento de um progétil que se desloca ao longo do eixo x é x(t)={e^[-(t – Pi/4)]}*sen (t) + cotg²(t), t >=0 . A aceleração do projétil no instante t=0 é: Resp: d) 16 –2*raiz(2) Basta derivar duas vezes a expressao e lembrar da regra da cadeia. v) No ultimo limite, use o limite fundamental da trigonometria em A e no segundo problema mude a base do logaritmo pra base 2 que o resultado sai. Leandro -----Original Message----- From: owner-obm-l@sucuri.mat.puc-rio.br [mailto:owner-obm-l@sucuri.mat.puc-rio.br] On Behalf Of basketboy_igor Sent: Monday, April 28, 2003 8:54 AM To: obm-l@mat.puc-rio.br Subject: [obm-l] Provas Militares Saldações à todos, Estou com algumas dúvidas em: 1°) Calculo I i) (EN/95) O lim [raiz(x+b)+raiz(x+a)-raiz(b)-raiz(a)]/x x -> 0 é? resp: 1/[2*raiz(b)] + 1/[2*raiz(a)] ii)(EN/95) Se f(x) = e^(2x) + (x+1)*cos(x), então f’(0) é igual a: resp: 3. iii)(EN/97) O valor da integral de raiz[1 + 9*x*(dx)] é: a)(2/27)*(1+9*x)^(3/2) + c iv) (EN/98) A equação do movimento de um progétil que se desloca ao longo do eixo x é x(t) = {e^[-(t – Pi/4)]}*sen (t) + cotg²(t), t >= 0. A aceleração do projétil no instante t=0 é: Resp: d) 16 –2*raiz(2) v) (MM/98) Sendo A = Lim (x -> 0) {2*raiz[x*sen(6x)]}/ {[cossec(6x)]*[1 - cos²(6x)]} e B = Lim (x-> log 2 na base 3) [2^(2x+1], [(A²)*B]/2 vale: Resp: b) 6 ________________________________________________________________________ __ Seleção de Softwares UOL. 10 softwares escolhidos pelo UOL para você e sua família. http://www.uol.com.br/selecao ======================================================================== = Instruções para entrar na lista, sair da lista e usar a lista em http://www.mat.puc-rio.br/~nicolau/olimp/obm-l.html ======================================================================== = ========================================================================= Instruções para entrar na lista, sair da lista e usar a lista em http://www.mat.puc-rio.br/~nicolau/olimp/obm-l.html ========================================================================= From owner-obm-l@sucuri.mat.puc-rio.br Tue Apr 29 18:07:31 2003 Return-Path: Received: (from majordom@localhost) by sucuri.mat.puc-rio.br (8.9.3/8.9.3) id SAA18676 for obm-l-MTTP; Tue, 29 Apr 2003 18:05:38 -0300 Received: from web13008.mail.yahoo.com (web13008.mail.yahoo.com [216.136.174.18]) by sucuri.mat.puc-rio.br (8.9.3/8.9.3) with SMTP id SAA18671 for ; Tue, 29 Apr 2003 18:05:34 -0300 Message-ID: <20030429210501.73734.qmail@web13008.mail.yahoo.com> Received: from [200.223.186.2] by web13008.mail.yahoo.com via HTTP; Tue, 29 Apr 2003 18:05:01 ART Date: Tue, 29 Apr 2003 18:05:01 -0300 (ART) From: =?iso-8859-1?q?carlos=20augusto?= Subject: [obm-l] derivadas (interessante) To: obm-l@mat.puc-rio.br Cc: augusto_math@yahoo.com.br MIME-Version: 1.0 Content-Type: text/plain; charset=iso-8859-1 Content-Transfer-Encoding: 8bit Sender: owner-obm-l@sucuri.mat.puc-rio.br Precedence: bulk Reply-To: obm-l@mat.puc-rio.br Demonstrar que a derivada de (x - 1)^(2/5) y = ------------------------------- é igual a ((x - 2)^(3/4))*((x - 3)^(7/3)) -161x^2 + 480x - 271 y´= -------------------------------------------------- 60*((x - 1)^(3/5))*((x - 2)^(7/4))*((x - 3)^(10/3)) Questão retirada do livro "Differential and integral calculus", autor: N. Piskunov _______________________________________________________________________ Yahoo! Mail O melhor e-mail gratuito da internet: 6MB de espaço, antivírus, acesso POP3, filtro contra spam. http://br.mail.yahoo.com/ ========================================================================= Instruções para entrar na lista, sair da lista e usar a lista em http://www.mat.puc-rio.br/~nicolau/olimp/obm-l.html ========================================================================= From owner-obm-l@sucuri.mat.puc-rio.br Tue Apr 29 18:07:49 2003 Return-Path: Received: (from majordom@localhost) by sucuri.mat.puc-rio.br (8.9.3/8.9.3) id SAA18710 for obm-l-MTTP; Tue, 29 Apr 2003 18:06:30 -0300 Received: from hotmail.com (oe56.law10.hotmail.com [64.4.14.191]) by sucuri.mat.puc-rio.br (8.9.3/8.9.3) with ESMTP id SAA18706 for ; Tue, 29 Apr 2003 18:06:25 -0300 Received: from mail pickup service by hotmail.com with Microsoft SMTPSVC; Tue, 29 Apr 2003 14:05:54 -0700 Received: from 64.60.139.18 by oe56.law10.hotmail.com with DAV; Tue, 29 Apr 2003 21:05:53 +0000 X-Originating-IP: [64.60.139.18] X-Originating-Email: [lrecova@hotmail.com] From: =?iso-8859-1?Q?Leandro_Lacorte_Rec=F4va?= To: Subject: RE: [obm-l] Limites no infinito Date: Tue, 29 Apr 2003 14:05:54 -0700 Message-ID: <001a01c30e93$1f5ee540$28029b9b@LeandroRecova> MIME-Version: 1.0 Content-Type: text/plain; charset="iso-8859-1" X-Priority: 3 (Normal) X-MSMail-Priority: Normal X-Mailer: Microsoft Outlook, Build 10.0.3416 In-Reply-To: <3EAED4F0.9040000@niski.com> X-MimeOLE: Produced By Microsoft MimeOLE V6.00.2800.1165 Importance: Normal X-OriginalArrivalTime: 29 Apr 2003 21:05:54.0081 (UTC) FILETIME=[1F12C110:01C30E93] Content-Transfer-Encoding: 8bit X-MIME-Autoconverted: from quoted-printable to 8bit by sucuri.mat.puc-rio.br id SAA18707 Sender: owner-obm-l@sucuri.mat.puc-rio.br Precedence: bulk Reply-To: obm-l@mat.puc-rio.br Niski, Voce colocou f(x) >=e ou f(x)>=0 ? Acho que voce pode usar a definicao de limite, veja so, Dado eps > 0 existe r > a > 0 tal que x > r => |f(x)/g(x) - L| < eps. Da ultima desigualdade voce tem L-eps < f(x)/g(x) < L + eps Tome eps=L/2 entao voce tera L/2 < f(x)/g(x) < 3L/2 Como g(x) > 0 , entao segue a desigualdade. O r que voce procura vem justamente da definicao de limite quando x->inf. -----Original Message----- From: owner-obm-l@sucuri.mat.puc-rio.br [mailto:owner-obm-l@sucuri.mat.puc-rio.br] On Behalf Of niski Sent: Tuesday, April 29, 2003 12:39 PM To: obm-l@mat.puc-rio.br Subject: [obm-l] Limites no infinito Por favor pessoal, me ajudem neste exercicio que empaquei. Obrigado Sejam f e g definidas em [a,+inf[ tais que f(x) >= e g(x) > 0 para todo x >= a. Suponha que lim[x->+inf] f(x)/g(x) = L, L >0. Prove que existe r > 0 , r > a, tal que para todo x>r (L/2)g(x) < f(x) < (3L/2)g(x). Conclua daí que se lim[x->+inf] g(x) = 0, entao lim[x->+inf] f(x) = 0. mais uma vez, obrigado. -- [about him:] It is rare to find learned men who are clean, do not stink and have a sense of humour. -Gottfried Whilhem Leibniz ======================================================================== = Instruções para entrar na lista, sair da lista e usar a lista em http://www.mat.puc-rio.br/~nicolau/olimp/obm-l.html ======================================================================== = ========================================================================= Instruções para entrar na lista, sair da lista e usar a lista em http://www.mat.puc-rio.br/~nicolau/olimp/obm-l.html ========================================================================= From owner-obm-l@sucuri.mat.puc-rio.br Tue Apr 29 19:17:57 2003 Return-Path: Received: (from majordom@localhost) by sucuri.mat.puc-rio.br (8.9.3/8.9.3) id TAA21122 for obm-l-MTTP; Tue, 29 Apr 2003 19:16:29 -0300 Received: from fnn.net ([200.175.38.9]) by sucuri.mat.puc-rio.br (8.9.3/8.9.3) with SMTP id TAA21117 for ; Tue, 29 Apr 2003 19:16:24 -0300 Received: (qmail 16922 invoked from network); 29 Apr 2003 22:01:02 -0000 Received: from unknown (HELO windows98) (200.175.39.84) by mail.fnn.net with SMTP; 29 Apr 2003 22:01:02 -0000 Message-ID: <003501c30e9e$38118600$9a75fea9@windows98> From: "Daniel Pini" To: Subject: [obm-l] corridas Date: Tue, 29 Apr 2003 19:25:19 -0300 MIME-Version: 1.0 Content-Type: multipart/alternative; boundary="----=_NextPart_000_0032_01C30E85.120284A0" X-Priority: 3 X-MSMail-Priority: Normal X-Mailer: Microsoft Outlook Express 5.00.2615.200 X-MimeOLE: Produced By Microsoft MimeOLE V5.00.2615.200 Sender: owner-obm-l@sucuri.mat.puc-rio.br Precedence: bulk Reply-To: obm-l@mat.puc-rio.br This is a multi-part message in MIME format. ------=_NextPart_000_0032_01C30E85.120284A0 Content-Type: text/plain; charset="iso-8859-1" Content-Transfer-Encoding: quoted-printable Numa corrida de 1760 metros, A vence B por 330 metros e A vence C por = 460 metros. Por quantos metros B vence C? R:160 Numa corrida de 5000m A vence B por 500m e B vence C por 1000m.Por = quantos metros B vence C?=20 ------=_NextPart_000_0032_01C30E85.120284A0 Content-Type: text/html; charset="iso-8859-1" Content-Transfer-Encoding: quoted-printable
Numa corrida de 1760 metros, A vence B = por 330=20 metros e A vence C por 460 metros. Por quantos metros B vence C?=20 R:160
 
Numa corrida de 5000m A vence B por 500m e B = vence C por=20 1000m.Por quantos metros B vence C?
------=_NextPart_000_0032_01C30E85.120284A0-- ========================================================================= Instruções para entrar na lista, sair da lista e usar a lista em http://www.mat.puc-rio.br/~nicolau/olimp/obm-l.html ========================================================================= From owner-obm-l@sucuri.mat.puc-rio.br Tue Apr 29 19:17:58 2003 Return-Path: Received: (from majordom@localhost) by sucuri.mat.puc-rio.br (8.9.3/8.9.3) id TAA21129 for obm-l-MTTP; Tue, 29 Apr 2003 19:16:32 -0300 Received: from fnn.net ([200.175.38.9]) by sucuri.mat.puc-rio.br (8.9.3/8.9.3) with SMTP id TAA21118 for ; Tue, 29 Apr 2003 19:16:26 -0300 Received: (qmail 9912 invoked from network); 29 Apr 2003 22:01:04 -0000 Received: from unknown (HELO windows98) (200.175.39.84) by mail.fnn.net with SMTP; 29 Apr 2003 22:01:04 -0000 Message-ID: <003e01c30e9e$39367e00$9a75fea9@windows98> From: "Daniel Pini" To: Subject: [obm-l] duvida Date: Tue, 29 Apr 2003 19:25:21 -0300 MIME-Version: 1.0 Content-Type: multipart/alternative; boundary="----=_NextPart_000_0036_01C30E85.13183A60" X-Priority: 3 X-MSMail-Priority: Normal X-Mailer: Microsoft Outlook Express 5.00.2615.200 X-MimeOLE: Produced By Microsoft MimeOLE V5.00.2615.200 Sender: owner-obm-l@sucuri.mat.puc-rio.br Precedence: bulk Reply-To: obm-l@mat.puc-rio.br This is a multi-part message in MIME format. ------=_NextPart_000_0036_01C30E85.13183A60 Content-Type: text/plain; charset="iso-8859-1" Content-Transfer-Encoding: quoted-printable Na finalissima do campeonato carioca de futebol de 1991 o quadro das = apostas era o seguinte: Para o Flamengo: cada R$175,00 apostado dava ao apostador R$100,00. Para o Fluminense: cada R$100,00 apstado dava ao apostador R$155,00. Assim, por exemplo se o Flamrngo fosse o vencedor do jogo um pessoa que = tivesse apostado R$175,00 no Flamengo teria de volta seus R$175,00 e = ainda ganharia R$100,00 , enquantoque uma pessoa que tivesse apostado = R$100,00 no Fluminense, perderia seus R$100,00. Supondo que uma casa de apostas tenha aceito 51 aposta a R$175,00 no = Flamengo, o n=EDmero de apostas a R$100,00 que ela deve aceitar para que = seu lucro seja o mesmo independente de quemganhe o jogo =E9? R:55 Dispondo-se de duas ligas de ouro e prata cujos metais est=E3o nas = raz=F5es 2:3 e 3:7 respectivamente, desejamos obter 8 quilos de uma nova = liga na qual estes metais estejam na raz=E3o 5:11. A difere=E7a entre as = quantidades, em kg, que devemos tomar de cada uma das duas ligas =E9? = R:6 ------=_NextPart_000_0036_01C30E85.13183A60 Content-Type: text/html; charset="iso-8859-1" Content-Transfer-Encoding: quoted-printable
Na finalissima do campeonato carioca de = futebol de=20 1991 o quadro das apostas era o seguinte:
Para o Flamengo: cada R$175,00 apostado dava ao = apostador=20 R$100,00.
Para o Fluminense: cada R$100,00 apstado dava ao = apostador=20 R$155,00.
Assim, por exemplo se o Flamrngo fosse o = vencedor do jogo=20 um pessoa que tivesse apostado R$175,00 no Flamengo teria de volta=20 seus  R$175,00 e ainda ganharia R$100,00 , enquantoque uma pessoa = que=20 tivesse apostado R$100,00 no Fluminense, perderia seus = R$100,00.
Supondo que uma casa de apostas tenha aceito 51 = aposta a=20 R$175,00 no Flamengo, o n=EDmero de apostas a R$100,00 que ela deve = aceitar=20 para que seu lucro seja o mesmo independente de quemganhe o jogo = =E9?
R:55
Dispondo-se de duas ligas de ouro e prata cujos = metais=20 est=E3o nas raz=F5es 2:3 e 3:7 respectivamente, desejamos obter 8 quilos = de uma nova=20 liga na qual estes metais estejam na raz=E3o 5:11. A difere=E7a entre = as =20 quantidades, em kg, que devemos tomar de cada uma das duas ligas =E9?=20 R:6
------=_NextPart_000_0036_01C30E85.13183A60-- ========================================================================= Instruções para entrar na lista, sair da lista e usar a lista em http://www.mat.puc-rio.br/~nicolau/olimp/obm-l.html ========================================================================= From owner-obm-l@sucuri.mat.puc-rio.br Tue Apr 29 19:21:09 2003 Return-Path: Received: (from majordom@localhost) by sucuri.mat.puc-rio.br (8.9.3/8.9.3) id TAA21222 for obm-l-MTTP; Tue, 29 Apr 2003 19:19:44 -0300 Received: from smtp017.mail.yahoo.com (smtp017.mail.yahoo.com [216.136.174.114]) by sucuri.mat.puc-rio.br (8.9.3/8.9.3) with SMTP id TAA21207 for ; Tue, 29 Apr 2003 19:19:34 -0300 Received: from 241.2-254.206.217.200.telemar.net.br (HELO itchy) (davidrvp@200.217.206.241 with login) by smtp.mail.vip.sc5.yahoo.com with SMTP; 29 Apr 2003 22:19:02 -0000 Message-ID: <000801c30e9d$601921e0$df8bfea9@itchy> From: "David Ricardo" To: Subject: [obm-l] Exponencial de uma matriz Date: Tue, 29 Apr 2003 19:16:19 -0300 MIME-Version: 1.0 Content-Type: text/plain; charset="iso-8859-1" Content-Transfer-Encoding: 8bit X-Priority: 3 X-MSMail-Priority: Normal X-Mailer: Microsoft Outlook Express 6.00.2600.0000 X-MimeOLE: Produced By Microsoft MimeOLE V6.00.2600.0000 Sender: owner-obm-l@sucuri.mat.puc-rio.br Precedence: bulk Reply-To: obm-l@mat.puc-rio.br Pessoal, eu tenho uma dúvida que já me perturba há uns dois anos e nunca perguntei a ninguém... Em Teoria de Controle utilizamos muito a exponencial de uma matriz (exp(At), onde A é uma matriz e t é o tempo) para fazer a representação de um sistema discreto em variáveis de estado. Eu sei calcular isso utilizando transformada de Laplace inversa de (sI-A)^-1 ou expansão em série de Taylor, mas o que significa um número elevado a uma matriz? Eu não estou conseguindo exergar bem isso... Não sei se a minha pergunta foi bem formulada, mas espero que vocês consigam me ajudar. []s David ========================================================================= Instruções para entrar na lista, sair da lista e usar a lista em http://www.mat.puc-rio.br/~nicolau/olimp/obm-l.html ========================================================================= From owner-obm-l@sucuri.mat.puc-rio.br Tue Apr 29 19:23:40 2003 Return-Path: Received: (from majordom@localhost) by sucuri.mat.puc-rio.br (8.9.3/8.9.3) id TAA21404 for obm-l-MTTP; Tue, 29 Apr 2003 19:22:23 -0300 Received: from fnn.net ([200.175.38.9]) by sucuri.mat.puc-rio.br (8.9.3/8.9.3) with SMTP id TAA21391 for ; Tue, 29 Apr 2003 19:22:18 -0300 Received: (qmail 8224 invoked from network); 29 Apr 2003 22:06:51 -0000 Received: from unknown (HELO windows98) (200.175.39.84) by mail.fnn.net with SMTP; 29 Apr 2003 22:06:51 -0000 Message-ID: <004201c30e9f$08478680$9a75fea9@windows98> From: "Daniel Pini" To: Subject: [obm-l] idade Date: Tue, 29 Apr 2003 19:31:08 -0300 MIME-Version: 1.0 Content-Type: multipart/alternative; boundary="----=_NextPart_000_003F_01C30E85.E2326AA0" X-Priority: 3 X-MSMail-Priority: Normal X-Mailer: Microsoft Outlook Express 5.00.2615.200 X-MimeOLE: Produced By Microsoft MimeOLE V5.00.2615.200 Sender: owner-obm-l@sucuri.mat.puc-rio.br Precedence: bulk Reply-To: obm-l@mat.puc-rio.br This is a multi-part message in MIME format. ------=_NextPart_000_003F_01C30E85.E2326AA0 Content-Type: text/plain; charset="iso-8859-1" Content-Transfer-Encoding: quoted-printable A soma das idades atuais de Antonio e Raul =E9 44 anos. Antonio tem o = dobro da idade que Raul tinha quando Antonio tinha a metade da idade que = Raul ter=E1 quando Raul tiver tr=EAs vezes a idade que Antonio tinha = quando era tr=EAs vezes mais velho que Raul. A diferen=E7a entre as = idades em anos =E9? A resposta em que cheguei foi 12 anos, ser=E1 que algu=E9m pode = confirmar a minha resposta?=20 ------=_NextPart_000_003F_01C30E85.E2326AA0 Content-Type: text/html; charset="iso-8859-1" Content-Transfer-Encoding: quoted-printable
A soma das idades atuais de Antonio e = Raul =E9 44=20 anos. Antonio tem o dobro da idade que Raul tinha quando Antonio tinha a = metade=20 da idade que Raul ter=E1 quando Raul tiver tr=EAs vezes a idade que = Antonio tinha=20 quando era tr=EAs vezes mais velho que Raul. A diferen=E7a entre as = idades em anos=20 =E9?
A resposta em que cheguei foi 12 anos, ser=E1 = que algu=E9m=20 pode confirmar a minha resposta?
------=_NextPart_000_003F_01C30E85.E2326AA0-- ========================================================================= Instruções para entrar na lista, sair da lista e usar a lista em http://www.mat.puc-rio.br/~nicolau/olimp/obm-l.html ========================================================================= From owner-obm-l@sucuri.mat.puc-rio.br Tue Apr 29 19:25:01 2003 Return-Path: Received: (from majordom@localhost) by sucuri.mat.puc-rio.br (8.9.3/8.9.3) id TAA21457 for obm-l-MTTP; Tue, 29 Apr 2003 19:23:44 -0300 Received: from itaqui.terra.com.br (itaqui.terra.com.br [200.176.3.19]) by sucuri.mat.puc-rio.br (8.9.3/8.9.3) with ESMTP id TAA21453 for ; Tue, 29 Apr 2003 19:23:41 -0300 Received: from gunga.terra.com.br (gunga.terra.com.br [200.176.3.45]) by itaqui.terra.com.br (Postfix) with ESMTP id E4F683BCACD for ; Tue, 29 Apr 2003 19:23:09 -0300 (BRT) Received: from niski.com (unknown [200.148.197.82]) (authenticated user fniski) by gunga.terra.com.br (Postfix) with ESMTP id BDA5F128072 for ; Tue, 29 Apr 2003 19:23:07 -0300 (BRT) Message-ID: <3EAEFB53.1090301@niski.com> Date: Tue, 29 Apr 2003 15:23:15 -0700 From: niski User-Agent: Mozilla/5.0 (Windows; U; Windows NT 5.1; en-US; rv:1.0.2) Gecko/20030208 Netscape/7.02 X-Accept-Language: en-us, en MIME-Version: 1.0 To: obm-l@mat.puc-rio.br Subject: Re: [obm-l] Limites no infinito References: <001a01c30e93$1f5ee540$28029b9b@LeandroRecova> Content-Type: text/plain; charset=ISO-8859-1; format=flowed Content-Transfer-Encoding: 8bit Sender: owner-obm-l@sucuri.mat.puc-rio.br Precedence: bulk Reply-To: obm-l@mat.puc-rio.br obrigado Leandro e Morgado. O fato é que fui tentar fazer esse problema sem antes ler direito a definicao de limites no infinito. obrigado e serei mais cuidadoso. Leandro Lacorte Recôva wrote: > Niski, > > Voce colocou f(x) >=e ou f(x)>=0 ? > > Acho que voce pode usar a definicao de limite, veja so, > > Dado eps > 0 existe r > a > 0 tal que > > x > r => |f(x)/g(x) - L| < eps. > > Da ultima desigualdade voce tem > > L-eps < f(x)/g(x) < L + eps > > Tome eps=L/2 entao voce tera > > L/2 < f(x)/g(x) < 3L/2 > > Como g(x) > 0 , entao segue a desigualdade. O r que voce procura vem > justamente da definicao de limite quando x->inf. > > -----Original Message----- > From: owner-obm-l@sucuri.mat.puc-rio.br > [mailto:owner-obm-l@sucuri.mat.puc-rio.br] On Behalf Of niski > Sent: Tuesday, April 29, 2003 12:39 PM > To: obm-l@mat.puc-rio.br > Subject: [obm-l] Limites no infinito > > Por favor pessoal, me ajudem neste exercicio que empaquei. Obrigado > > Sejam f e g definidas em [a,+inf[ tais que f(x) >= e g(x) > 0 para todo > x >= a. Suponha que lim[x->+inf] f(x)/g(x) = L, L >0. > Prove que existe r > 0 , r > a, tal que para todo x>r > > (L/2)g(x) < f(x) < (3L/2)g(x). > > Conclua daí que se lim[x->+inf] g(x) = 0, entao lim[x->+inf] f(x) = 0. > > mais uma vez, obrigado. > -- [about him:] It is rare to find learned men who are clean, do not stink and have a sense of humour. -Gottfried Whilhem Leibniz ========================================================================= Instruções para entrar na lista, sair da lista e usar a lista em http://www.mat.puc-rio.br/~nicolau/olimp/obm-l.html ========================================================================= From owner-obm-l@sucuri.mat.puc-rio.br Tue Apr 29 19:25:11 2003 Return-Path: Received: (from majordom@localhost) by sucuri.mat.puc-rio.br (8.9.3/8.9.3) id TAA21466 for obm-l-MTTP; Tue, 29 Apr 2003 19:23:54 -0300 Received: from hotmail.com (oe18.law10.hotmail.com [64.4.14.122]) by sucuri.mat.puc-rio.br (8.9.3/8.9.3) with ESMTP id TAA21461 for ; Tue, 29 Apr 2003 19:23:49 -0300 Received: from mail pickup service by hotmail.com with Microsoft SMTPSVC; Tue, 29 Apr 2003 15:23:18 -0700 Received: from 64.60.139.18 by oe18.law10.hotmail.com with DAV; Tue, 29 Apr 2003 22:23:18 +0000 X-Originating-IP: [64.60.139.18] X-Originating-Email: [lrecova@hotmail.com] From: =?iso-8859-1?Q?Leandro_Lacorte_Rec=F4va?= To: Subject: RE: [obm-l] Limites no infinito Date: Tue, 29 Apr 2003 15:23:18 -0700 Message-ID: <002401c30e9d$ef85bf00$28029b9b@LeandroRecova> MIME-Version: 1.0 Content-Type: text/plain; charset="iso-8859-1" X-Priority: 3 (Normal) X-MSMail-Priority: Normal X-Mailer: Microsoft Outlook, Build 10.0.3416 In-Reply-To: <001a01c30e93$1f5ee540$28029b9b@LeandroRecova> X-MimeOLE: Produced By Microsoft MimeOLE V6.00.2800.1165 Importance: Normal X-OriginalArrivalTime: 29 Apr 2003 22:23:18.0643 (UTC) FILETIME=[EF72AC30:01C30E9D] Content-Transfer-Encoding: 8bit X-MIME-Autoconverted: from quoted-printable to 8bit by sucuri.mat.puc-rio.br id TAA21462 Sender: owner-obm-l@sucuri.mat.puc-rio.br Precedence: bulk Reply-To: obm-l@mat.puc-rio.br Esqueci de falar da conclusao: Pelo teorema do sanduiche voce conclui facilmente que Conclua daí que se lim[x->+inf] g(x) = 0, entao lim[x->+inf] f(x) = 0. Regards. -----Original Message----- From: owner-obm-l@sucuri.mat.puc-rio.br [mailto:owner-obm-l@sucuri.mat.puc-rio.br] On Behalf Of Leandro Lacorte Recôva Sent: Tuesday, April 29, 2003 2:06 PM To: obm-l@mat.puc-rio.br Subject: RE: [obm-l] Limites no infinito Niski, Voce colocou f(x) >=e ou f(x)>=0 ? Acho que voce pode usar a definicao de limite, veja so, Dado eps > 0 existe r > a > 0 tal que x > r => |f(x)/g(x) - L| < eps. Da ultima desigualdade voce tem L-eps < f(x)/g(x) < L + eps Tome eps=L/2 entao voce tera L/2 < f(x)/g(x) < 3L/2 Como g(x) > 0 , entao segue a desigualdade. O r que voce procura vem justamente da definicao de limite quando x->inf. -----Original Message----- From: owner-obm-l@sucuri.mat.puc-rio.br [mailto:owner-obm-l@sucuri.mat.puc-rio.br] On Behalf Of niski Sent: Tuesday, April 29, 2003 12:39 PM To: obm-l@mat.puc-rio.br Subject: [obm-l] Limites no infinito Por favor pessoal, me ajudem neste exercicio que empaquei. Obrigado Sejam f e g definidas em [a,+inf[ tais que f(x) >= e g(x) > 0 para todo x >= a. Suponha que lim[x->+inf] f(x)/g(x) = L, L >0. Prove que existe r > 0 , r > a, tal que para todo x>r (L/2)g(x) < f(x) < (3L/2)g(x). Conclua daí que se lim[x->+inf] g(x) = 0, entao lim[x->+inf] f(x) = 0. mais uma vez, obrigado. -- [about him:] It is rare to find learned men who are clean, do not stink and have a sense of humour. -Gottfried Whilhem Leibniz ======================================================================== = Instruções para entrar na lista, sair da lista e usar a lista em http://www.mat.puc-rio.br/~nicolau/olimp/obm-l.html ======================================================================== = ======================================================================== = Instruções para entrar na lista, sair da lista e usar a lista em http://www.mat.puc-rio.br/~nicolau/olimp/obm-l.html ======================================================================== = ========================================================================= Instruções para entrar na lista, sair da lista e usar a lista em http://www.mat.puc-rio.br/~nicolau/olimp/obm-l.html ========================================================================= From owner-obm-l@sucuri.mat.puc-rio.br Tue Apr 29 19:36:49 2003 Return-Path: Received: (from majordom@localhost) by sucuri.mat.puc-rio.br (8.9.3/8.9.3) id TAA22444 for obm-l-MTTP; Tue, 29 Apr 2003 19:35:18 -0300 Received: from paiol.terra.com.br (paiol.terra.com.br [200.176.3.18]) by sucuri.mat.puc-rio.br (8.9.3/8.9.3) with ESMTP id TAA22440 for ; Tue, 29 Apr 2003 19:35:15 -0300 Received: from jurere.terra.com.br (jurere.terra.com.br [200.176.3.49]) by paiol.terra.com.br (Postfix) with ESMTP id 3884B88363 for ; Tue, 29 Apr 2003 19:34:45 -0300 (BRT) Received: from [200.177.179.184] (dl-nas3-sao-C8B1B3B8.p001.terra.com.br [200.177.179.184]) by jurere.terra.com.br (Postfix) with ESMTP id 2611F1380D2 for ; Tue, 29 Apr 2003 19:34:44 -0300 (BRT) User-Agent: Microsoft-Outlook-Express-Macintosh-Edition/5.02.2022 Date: Tue, 29 Apr 2003 19:36:11 -0300 Subject: Re: [obm-l] Desigualdades From: Claudio Buffara To: Message-ID: In-Reply-To: <200304291038.17188.fabio.dias.moreira@terra.com.br> Mime-version: 1.0 Content-type: text/plain; charset="ISO-8859-1" Content-Transfer-Encoding: 8bit X-MIME-Autoconverted: from quoted-printable to 8bit by sucuri.mat.puc-rio.br id TAA22441 Sender: owner-obm-l@sucuri.mat.puc-rio.br Precedence: bulk Reply-To: obm-l@mat.puc-rio.br Oi, Fabio: Desculpe a amolacao, mas eh que este problema estah me parecendo bem mais dificil do que eu imaginava inicialmente. Veja meus cometarios abaixo. Um abraco, Claudio. on 29.04.03 10:38, Fábio Dias Moreira at fabio.dias.moreira@terra.com.br wrote: > -----BEGIN PGP SIGNED MESSAGE----- > Hash: SHA1 > > On Tuesday 29 April 2003 00:30, Claudio Buffara wrote: >> on 28.04.03 23:01, Fábio Dias Moreira at fabio.dias.moreira@terra.com.br >> >> wrote: >>> -----BEGIN PGP SIGNED MESSAGE----- >>> Hash: SHA1 >>> >>> On Monday 28 April 2003 22:32, Raphael Marx wrote: >>>> Olá a todos. Gostaria de pedir uma ajudinha em desigualdades: >>>> 1-Prove que se v>>> (v+x+y+z)^2>8(vy+xz) >>>> [...] >>> >>> Divida por 16: >>> >>> ((v+x+y+z)/4)^2 > (vy+xz)/2 > sqrt(xyzv) >>> (v+x+y+z)/4 > (xyzv)^(1/4) >>> >>> verdadeiro por MA-MG. >>> >>> []s, >> >> Oi, Fabio (e Raphael): >> >> MA-MG soh pode ser usada se v > 0, o que nao estah dito no enunciado. >> [...] > > Mas acho que isso é necessário. Se v=-1, x=0, y=1/4 e z=3/4, temos que > 0^2 > 8*(-1*0 + 1/4*3/4) <==> 3/2 < 0. > Na verdade, com a sua escolha de v, x, y, z, o lado direito fica: 8*(vy+xz) = 8*((-1)*1/4 + 0*3/4) = -2, que eh de fato < 0, de forma que este contra-exemplo nao vale. >> Alem disso, acho que tem um probleminha na logica: >> >> Supondo v > 0, ((v+x+y+z)/4)^2 > raiz(xyzv) e (vy+xz)/2 > raiz(xyzv) sao >> ambas verdadeiras, mas isso nao implica necessariamente que: >> ((v+x+y+z)/4)^2 > raiz(xyzv). >> [...] Eu escrevi besteira na ultima linha. O que eu queria ter dito eh: Supondo v > 0, ((v+x+y+z)/4)^2 > raiz(xyzv) e (vy+xz)/2 > raiz(xyzv) sao ambas verdadeiras, mas isso nao implica necessariamente que: ((v+x+y+z)/4)^2 > (vy + xz)/2. Ou seja, eu posso estar enganado, mas me pareceu que sua afirmativa foi: Se A > C e B > C entao A > B, o que nao eh uma inferencia valida. > A afirmação que usei foi: > > ((v+x+y+z)/4)^2 > (vy+xz)/2 e (vy+xz)/2 > raiz(xyzv), logo > ((v+x+y+z)/4)^2 > raiz(xyzv), verdadeiro por transitividade. > Como voce provou que ((v+x+y+z)/4)^2 > (vy+xz)/2 ? Usando MA-MG (o que so vale se v > 0), eu chego em: ((v+x+y+z)/4)^2 = (((v+y)/2 + (x+z)/2)/2)^2 > ((raiz(vy) + raiz(xz))/2)^2 = = (vy + xz + 2*raiz(vyxz))/4 = (vy + xz)/4 + raiz(vyxz)/2 So que como (vy+xz)/2 > raiz(xyzv), nao da pra concluir o que queremos. Estou convencido de que a condicao v < x < y < z entra na demonstracao em algum ponto e minha aposta ainda eh na desigualdade do rearranjo. ========================================================================= Instruções para entrar na lista, sair da lista e usar a lista em http://www.mat.puc-rio.br/~nicolau/olimp/obm-l.html ========================================================================= From owner-obm-l@sucuri.mat.puc-rio.br Tue Apr 29 21:11:50 2003 Return-Path: Received: (from majordom@localhost) by sucuri.mat.puc-rio.br (8.9.3/8.9.3) id VAA25529 for obm-l-MTTP; Tue, 29 Apr 2003 21:10:24 -0300 Received: from trex.centroin.com.br (trex.centroin.com.br [200.225.63.134]) by sucuri.mat.puc-rio.br (8.9.3/8.9.3) with ESMTP id VAA25525 for ; Tue, 29 Apr 2003 21:10:20 -0300 Received: from trex.centroin.com.br (localhost [127.0.0.1]) by trex.centroin.com.br (8.12.9/8.12.9) with ESMTP id h3U09nkZ022692; Tue, 29 Apr 2003 21:09:49 -0300 (EST) Received: by trex.centroin.com.br (8.12.9/8.12.5/Submit) id h3U09nX3022690; Tue, 29 Apr 2003 21:09:49 -0300 (EST) Message-Id: <200304300009.h3U09nX3022690@trex.centroin.com.br> Received: from 200.141.119.135 by trex.centroin.com.br (CIPWM versao 1.4C1) with HTTPS for ; Tue, 29 Apr 2003 21:09:49 -0300 (EST) Date: Tue, 29 Apr 2003 21:09:49 -0300 (EST) From: Augusto Cesar de Oliveira Morgado To: obm-l@mat.puc-rio.br Cc: augusto_math@yahoo.com.br Subject: Re: [obm-l] derivadas (interessante) MIME-Version: 1.0 X-Mailer: CentroIn Internet Provider WebMail v. 1.4C1 (http://www.centroin.com.br/) Content-Type: text/plain; charset="iso-8859-1" Content-Transfer-Encoding: 8bit X-MIME-Autoconverted: from quoted-printable to 8bit by sucuri.mat.puc-rio.br id VAA25526 Sender: owner-obm-l@sucuri.mat.puc-rio.br Precedence: bulk Reply-To: obm-l@mat.puc-rio.br Se isso eh interessante, o que seria desinteressante? Em Tue, 29 Apr 2003 18:05:01 -0300 (ART), carlos augusto disse: > Demonstrar que a derivada de > > (x - 1)^(2/5) > y = ------------------------------- é igual a > ((x - 2)^(3/4))*((x - 3)^(7/3)) > > -161x^2 + 480x - 271 > y´= -------------------------------------------------- > 60*((x - 1)^(3/5))*((x - 2)^(7/4))*((x - > 3)^(10/3)) > > > > Questão retirada do livro "Differential and integral > calculus", autor: N. Piskunov > > _______________________________________________________________________ > Yahoo! Mail > O melhor e-mail gratuito da internet: 6MB de espaço, antivírus, acesso POP3, filtro contra spam. > http://br.mail.yahoo.com/ > ========================================================================= > Instruções para entrar na lista, sair da lista e usar a lista em > http://www.mat.puc-rio.br/~nicolau/olimp/obm-l.html > ========================================================================= > > ========================================================================= Instruções para entrar na lista, sair da lista e usar a lista em http://www.mat.puc-rio.br/~nicolau/olimp/obm-l.html ========================================================================= From owner-obm-l@sucuri.mat.puc-rio.br Tue Apr 29 21:11:50 2003 Return-Path: Received: (from majordom@localhost) by sucuri.mat.puc-rio.br (8.9.3/8.9.3) id VAA25521 for obm-l-MTTP; Tue, 29 Apr 2003 21:10:15 -0300 Received: from trex.centroin.com.br (trex.centroin.com.br [200.225.63.134]) by sucuri.mat.puc-rio.br (8.9.3/8.9.3) with ESMTP id VAA25517 for ; Tue, 29 Apr 2003 21:10:11 -0300 Received: from trex.centroin.com.br (localhost [127.0.0.1]) by trex.centroin.com.br (8.12.9/8.12.9) with ESMTP id h3U09fkZ022625; Tue, 29 Apr 2003 21:09:41 -0300 (EST) Received: by trex.centroin.com.br (8.12.9/8.12.5/Submit) id h3U09fVN022621; Tue, 29 Apr 2003 21:09:41 -0300 (EST) Message-Id: <200304300009.h3U09fVN022621@trex.centroin.com.br> Received: from 200.141.119.135 by trex.centroin.com.br (CIPWM versao 1.4C1) with HTTPS for ; Tue, 29 Apr 2003 21:09:41 -0300 (EST) Date: Tue, 29 Apr 2003 21:09:41 -0300 (EST) From: Augusto Cesar de Oliveira Morgado To: obm-l@mat.puc-rio.br Cc: augusto_math@yahoo.com.br Subject: Re: [obm-l] derivadas (interessante) MIME-Version: 1.0 X-Mailer: CentroIn Internet Provider WebMail v. 1.4C1 (http://www.centroin.com.br/) Content-Type: text/plain; charset="iso-8859-1" Content-Transfer-Encoding: 8bit X-MIME-Autoconverted: from quoted-printable to 8bit by sucuri.mat.puc-rio.br id VAA25518 Sender: owner-obm-l@sucuri.mat.puc-rio.br Precedence: bulk Reply-To: obm-l@mat.puc-rio.br Se isso eh interessante, o que seria desinteressante? Em Tue, 29 Apr 2003 18:05:01 -0300 (ART), carlos augusto disse: > Demonstrar que a derivada de > > (x - 1)^(2/5) > y = ------------------------------- é igual a > ((x - 2)^(3/4))*((x - 3)^(7/3)) > > -161x^2 + 480x - 271 > y´= -------------------------------------------------- > 60*((x - 1)^(3/5))*((x - 2)^(7/4))*((x - > 3)^(10/3)) > > > > Questão retirada do livro "Differential and integral > calculus", autor: N. Piskunov > > _______________________________________________________________________ > Yahoo! Mail > O melhor e-mail gratuito da internet: 6MB de espaço, antivírus, acesso POP3, filtro contra spam. > http://br.mail.yahoo.com/ > ========================================================================= > Instruções para entrar na lista, sair da lista e usar a lista em > http://www.mat.puc-rio.br/~nicolau/olimp/obm-l.html > ========================================================================= > > ========================================================================= Instruções para entrar na lista, sair da lista e usar a lista em http://www.mat.puc-rio.br/~nicolau/olimp/obm-l.html ========================================================================= From owner-obm-l@sucuri.mat.puc-rio.br Tue Apr 29 21:11:53 2003 Return-Path: Received: (from majordom@localhost) by sucuri.mat.puc-rio.br (8.9.3/8.9.3) id VAA25537 for obm-l-MTTP; Tue, 29 Apr 2003 21:10:32 -0300 Received: from trex.centroin.com.br (trex.centroin.com.br [200.225.63.134]) by sucuri.mat.puc-rio.br (8.9.3/8.9.3) with ESMTP id VAA25533 for ; Tue, 29 Apr 2003 21:10:28 -0300 Received: from trex.centroin.com.br (localhost [127.0.0.1]) by trex.centroin.com.br (8.12.9/8.12.9) with ESMTP id h3U09wkZ022746; Tue, 29 Apr 2003 21:09:58 -0300 (EST) Received: by trex.centroin.com.br (8.12.9/8.12.5/Submit) id h3U09wPG022745; Tue, 29 Apr 2003 21:09:58 -0300 (EST) Message-Id: <200304300009.h3U09wPG022745@trex.centroin.com.br> Received: from 200.141.119.135 by trex.centroin.com.br (CIPWM versao 1.4C1) with HTTPS for ; Tue, 29 Apr 2003 21:09:58 -0300 (EST) Date: Tue, 29 Apr 2003 21:09:58 -0300 (EST) From: Augusto Cesar de Oliveira Morgado To: obm-l@mat.puc-rio.br Cc: augusto_math@yahoo.com.br Subject: Re: [obm-l] derivadas (interessante) MIME-Version: 1.0 X-Mailer: CentroIn Internet Provider WebMail v. 1.4C1 (http://www.centroin.com.br/) Content-Type: text/plain; charset="iso-8859-1" Content-Transfer-Encoding: 8bit X-MIME-Autoconverted: from quoted-printable to 8bit by sucuri.mat.puc-rio.br id VAA25534 Sender: owner-obm-l@sucuri.mat.puc-rio.br Precedence: bulk Reply-To: obm-l@mat.puc-rio.br Se isso eh interessante, o que seria desinteressante? Em Tue, 29 Apr 2003 18:05:01 -0300 (ART), carlos augusto disse: > Demonstrar que a derivada de > > (x - 1)^(2/5) > y = ------------------------------- é igual a > ((x - 2)^(3/4))*((x - 3)^(7/3)) > > -161x^2 + 480x - 271 > y´= -------------------------------------------------- > 60*((x - 1)^(3/5))*((x - 2)^(7/4))*((x - > 3)^(10/3)) > > > > Questão retirada do livro "Differential and integral > calculus", autor: N. Piskunov > > _______________________________________________________________________ > Yahoo! Mail > O melhor e-mail gratuito da internet: 6MB de espaço, antivírus, acesso POP3, filtro contra spam. > http://br.mail.yahoo.com/ > ========================================================================= > Instruções para entrar na lista, sair da lista e usar a lista em > http://www.mat.puc-rio.br/~nicolau/olimp/obm-l.html > ========================================================================= > > ========================================================================= Instruções para entrar na lista, sair da lista e usar a lista em http://www.mat.puc-rio.br/~nicolau/olimp/obm-l.html ========================================================================= From owner-obm-l@sucuri.mat.puc-rio.br Tue Apr 29 22:01:08 2003 Return-Path: Received: (from majordom@localhost) by sucuri.mat.puc-rio.br (8.9.3/8.9.3) id VAA27477 for obm-l-MTTP; Tue, 29 Apr 2003 21:59:23 -0300 Received: from mail.ajato.com.br (200-162-192-51.mail.ajato.com.br [200.162.192.51]) by sucuri.mat.puc-rio.br (8.9.3/8.9.3) with SMTP id VAA27472 for ; Tue, 29 Apr 2003 21:59:16 -0300 Received: from mparaujo (200.162.245.188) by mail.ajato.com.br (5.1.061) id 3EAE55B200027A60 for obm-l@mat.puc-rio.br; Tue, 29 Apr 2003 21:58:45 -0300 Message-ID: <004201c30ca0$0c68c7c0$bcf5a2c8@ajato.com.br> From: "Marcos Paulo" To: References: <003501c30e9e$38118600$9a75fea9@windows98> Subject: Re: [obm-l] corridas Date: Sun, 27 Apr 2003 06:33:23 -0300 MIME-Version: 1.0 Content-Type: multipart/alternative; boundary="----=_NextPart_000_003F_01C30C86.E6B6DA80" X-Priority: 3 X-MSMail-Priority: Normal X-Mailer: Microsoft Outlook Express 6.00.2800.1158 X-MimeOLE: Produced By Microsoft MimeOLE V6.00.2800.1165 Sender: owner-obm-l@sucuri.mat.puc-rio.br Precedence: bulk Reply-To: obm-l@mat.puc-rio.br This is a multi-part message in MIME format. ------=_NextPart_000_003F_01C30C86.E6B6DA80 Content-Type: text/plain; charset="iso-8859-1" Content-Transfer-Encoding: quoted-printable A percorre 1760 enquanto B percorre 1430 e C percorre 1300. Se o tempo = que A demora para chegar ao fim da prova =E9 tomado como unidade, o = tempo que B demora pra chegar ao fim da prova (depois de A ter concluido = a mesma) =E9 3/13 e nesse tempo C ter=E1 andado 3*1300/13 =3D 300, = Ent=E3o Quando B chega C est=E1 na posi=E7=E3o 1600, ou seja, 160 metros = atras de B. A segunda =E9 an=E1loga. []'s MP ----- Original Message -----=20 From: Daniel Pini=20 To: obm-l@mat.puc-rio.br=20 Sent: Tuesday, April 29, 2003 7:25 PM Subject: [obm-l] corridas Numa corrida de 1760 metros, A vence B por 330 metros e A vence C por = 460 metros. Por quantos metros B vence C? R:160 Numa corrida de 5000m A vence B por 500m e B vence C por 1000m.Por = quantos metros B vence C? ------=_NextPart_000_003F_01C30C86.E6B6DA80 Content-Type: text/html; charset="iso-8859-1" Content-Transfer-Encoding: quoted-printable
A percorre 1760 enquanto B percorre = 1430 e C=20 percorre 1300. Se o tempo que A demora para chegar ao fim da prova =E9 = tomado como=20 unidade, o tempo que B demora pra chegar ao fim da prova (depois de = A ter=20 concluido a mesma) =E9 3/13 e nesse tempo C ter=E1 andado  = 3*1300/13 =3D=20 300, Ent=E3o Quando B chega C est=E1 na posi=E7=E3o 1600, ou seja, 160 = metros atras de=20 B.
 
A segunda =E9 an=E1loga.
 
[]'s MP
----- Original Message -----
From:=20 Daniel = Pini
Sent: Tuesday, April 29, 2003 = 7:25=20 PM
Subject: [obm-l] corridas

Numa corrida de 1760 metros, A vence = B por 330=20 metros e A vence C por 460 metros. Por quantos metros B vence C?=20 R:160
 
Numa corrida de 5000m A vence B por 500m e B = vence C por=20 1000m.Por quantos metros B vence C? =
------=_NextPart_000_003F_01C30C86.E6B6DA80-- ========================================================================= Instruções para entrar na lista, sair da lista e usar a lista em http://www.mat.puc-rio.br/~nicolau/olimp/obm-l.html ========================================================================= From owner-obm-l@sucuri.mat.puc-rio.br Tue Apr 29 22:11:10 2003 Return-Path: Received: (from majordom@localhost) by sucuri.mat.puc-rio.br (8.9.3/8.9.3) id WAA27799 for obm-l-MTTP; Tue, 29 Apr 2003 22:09:51 -0300 Received: from itaqui.terra.com.br (itaqui.terra.com.br [200.176.3.19]) by sucuri.mat.puc-rio.br (8.9.3/8.9.3) with ESMTP id WAA27795 for ; Tue, 29 Apr 2003 22:09:47 -0300 Received: from barra.terra.com.br (barra.terra.com.br [200.176.3.52]) by itaqui.terra.com.br (Postfix) with ESMTP id 2AAC43BCB79 for ; Tue, 29 Apr 2003 22:09:17 -0300 (BRT) Received: from [200.177.192.98] (dl-nas2-sao-C8B1C062.p001.terra.com.br [200.177.192.98]) by barra.terra.com.br (Postfix) with ESMTP id 2D6BF23404D for ; Tue, 29 Apr 2003 22:09:16 -0300 (BRT) User-Agent: Microsoft-Outlook-Express-Macintosh-Edition/5.02.2022 Date: Tue, 29 Apr 2003 22:10:18 -0300 Subject: Re: [obm-l] derivadas (interessante) From: Claudio Buffara To: Message-ID: In-Reply-To: <200304300009.h3U09fVN022621@trex.centroin.com.br> Mime-version: 1.0 Content-type: text/plain; charset="ISO-8859-1" Content-Transfer-Encoding: 8bit X-MIME-Autoconverted: from quoted-printable to 8bit by sucuri.mat.puc-rio.br id WAA27796 Sender: owner-obm-l@sucuri.mat.puc-rio.br Precedence: bulk Reply-To: obm-l@mat.puc-rio.br Oi, Carlos Augusto (e Morgado): Vou tentar salvar a mensagem: Se: (x - 1)^(2/5) y = ------------------------------- ((x - 2)^(3/4))*((x - 3)^(7/3)) entao, tomando logaritmos naturais: Ln(y) = (2/5)*Ln(x-1) - (3/4)*Ln(x-2) - (7/3)*Ln(x-3) Diferenciando, teremos: dy/y = [(2/5)/(x-1) - (3/4)/(x-2) - (7/3)/(x-3)]dx ==> dy/dx = y*[(2/5)/(x-1) - (3/4)/(x-2) - (7/3)/(x-3)] onde y eh dado pela expressao acima. Se, numa prova, voce nao receber credito total por essa resposta, entao abandone a cadeira pois seu professor eh um idiota. Um abraco, Claudio. on 29.04.03 21:09, Augusto Cesar de Oliveira Morgado at morgado@centroin.com.br wrote: > Se isso eh interessante, o que seria desinteressante? > > > Em Tue, 29 Apr 2003 18:05:01 -0300 (ART), carlos augusto > disse: > >> Demonstrar que a derivada de >> >> (x - 1)^(2/5) >> y = ------------------------------- é igual a >> ((x - 2)^(3/4))*((x - 3)^(7/3)) >> >> -161x^2 + 480x - 271 >> y´= -------------------------------------------------- >> 60*((x - 1)^(3/5))*((x - 2)^(7/4))*((x - >> 3)^(10/3)) >> >> >> >> Questão retirada do livro "Differential and integral >> calculus", autor: N. Piskunov >> >> _______________________________________________________________________ >> Yahoo! Mail >> O melhor e-mail gratuito da internet: 6MB de espaço, antivírus, acesso POP3, >> filtro contra spam. >> http://br.mail.yahoo.com/ >> ========================================================================= >> Instruções para entrar na lista, sair da lista e usar a lista em >> http://www.mat.puc-rio.br/~nicolau/olimp/obm-l.html >> ========================================================================= >> >> > > ========================================================================= > Instruções para entrar na lista, sair da lista e usar a lista em > http://www.mat.puc-rio.br/~nicolau/olimp/obm-l.html > ========================================================================= > ========================================================================= Instruções para entrar na lista, sair da lista e usar a lista em http://www.mat.puc-rio.br/~nicolau/olimp/obm-l.html ========================================================================= From owner-obm-l@sucuri.mat.puc-rio.br Tue Apr 29 22:21:34 2003 Return-Path: Received: (from majordom@localhost) by sucuri.mat.puc-rio.br (8.9.3/8.9.3) id WAA28164 for obm-l-MTTP; Tue, 29 Apr 2003 22:20:00 -0300 Received: from mail.ajato.com.br (200-162-192-51.mail.ajato.com.br [200.162.192.51]) by sucuri.mat.puc-rio.br (8.9.3/8.9.3) with SMTP id WAA28151 for ; Tue, 29 Apr 2003 22:19:51 -0300 Received: from mparaujo (200.162.245.188) by mail.ajato.com.br (5.1.061) id 3EAE55B2000284EB for obm-l@mat.puc-rio.br; Tue, 29 Apr 2003 22:19:20 -0300 Message-ID: <006201c30ca2$ec86d340$bcf5a2c8@ajato.com.br> From: "Marcos Paulo" To: References: <003e01c30e9e$39367e00$9a75fea9@windows98> Subject: Re: [obm-l] duvida Date: Sun, 27 Apr 2003 06:53:31 -0300 MIME-Version: 1.0 Content-Type: multipart/alternative; boundary="----=_NextPart_000_005F_01C30C89.B71586C0" X-Priority: 3 X-MSMail-Priority: Normal X-Mailer: Microsoft Outlook Express 6.00.2800.1158 X-MimeOLE: Produced By Microsoft MimeOLE V6.00.2800.1165 Sender: owner-obm-l@sucuri.mat.puc-rio.br Precedence: bulk Reply-To: obm-l@mat.puc-rio.br This is a multi-part message in MIME format. ------=_NextPart_000_005F_01C30C89.B71586C0 Content-Type: text/plain; charset="iso-8859-1" Content-Transfer-Encoding: quoted-printable Se h=E1 X apostas no flamengo e Y apostas no Fluminense, o dono da casa = de jogos arrecadar=E1 175X + 100Y. Se o Flamengo ganhar, ele ter=E1 que pagar: 175X + 100X =3D 275X,=20 Se o Fluminense ganhar, el;e ter=E1 que pagar 100Y + 155Y =3D 255Y=20 Se =E9 desejado que o lucro seja o mesmo independente do resultado, = teremos que ter 275X =3D 255Y (mesmo lucro =3D memso prejuizo). Como X = =3D 51, temos que 255Y =3D 51*275 e portanto Y =3D 55. Vamos utilizar X quilogramas da primeira liga (2:3) e 8 - X quilogramas = da segunda (3:7) A quantidade de ouro da primeira liga ser=E1 2/5 de X e a quantidade de = ouro da segunda ser=E1 3/10 de (8 - X) e a soma das quantidades de ouro = ser=E1 5/16 de 8 kg. Assim sendo, 2x/5 + 24/10 - 3x/10 =3D 40/16 x/10 =3D 40/16 - 24/10 x =3D 400/16 - 24 =3D 1 Devemos ent=E3o tomar 1 kg da primeira liga e 7 kg da segunda. A = diferen=E7a entre as quantidades =E9 6. []'s MP=20 ----- Original Message -----=20 From: Daniel Pini=20 To: obm-l@mat.puc-rio.br=20 Sent: Tuesday, April 29, 2003 7:25 PM Subject: [obm-l] duvida Na finalissima do campeonato carioca de futebol de 1991 o quadro das = apostas era o seguinte: Para o Flamengo: cada R$175,00 apostado dava ao apostador R$100,00. Para o Fluminense: cada R$100,00 apstado dava ao apostador R$155,00. Assim, por exemplo se o Flamrngo fosse o vencedor do jogo um pessoa = que tivesse apostado R$175,00 no Flamengo teria de volta seus R$175,00 = e ainda ganharia R$100,00 , enquantoque uma pessoa que tivesse apostado = R$100,00 no Fluminense, perderia seus R$100,00. Supondo que uma casa de apostas tenha aceito 51 aposta a R$175,00 no = Flamengo, o n=EDmero de apostas a R$100,00 que ela deve aceitar para que = seu lucro seja o mesmo independente de quemganhe o jogo =E9? R:55 Dispondo-se de duas ligas de ouro e prata cujos metais est=E3o nas = raz=F5es 2:3 e 3:7 respectivamente, desejamos obter 8 quilos de uma nova = liga na qual estes metais estejam na raz=E3o 5:11. A difere=E7a entre as = quantidades, em kg, que devemos tomar de cada uma das duas ligas =E9? = R:6 ------=_NextPart_000_005F_01C30C89.B71586C0 Content-Type: text/html; charset="iso-8859-1" Content-Transfer-Encoding: quoted-printable
Se h=E1 X apostas no flamengo e Y = apostas no=20 Fluminense, o dono da casa de jogos arrecadar=E1 175X + = 100Y.
 
Se o Flamengo ganhar, ele ter=E1 que = pagar: 175X +=20 100X =3D 275X,
Se o Fluminense ganhar, el;e = ter=E1 que pagar=20 100Y + 155Y =3D 255Y
Se =E9 desejado que o lucro seja o = mesmo independente=20 do resultado, teremos que ter 275X =3D 255Y (mesmo lucro =3D memso = prejuizo).=20 Como X =3D 51, temos que 255Y =3D 51*275 e portanto Y =3D = 55.
 
Vamos utilizar X quilogramas da = primeira liga=20 (2:3) e 8 - X quilogramas da segunda (3:7)
A quantidade de ouro da primeira liga = ser=E1 2/5 de X=20 e a quantidade de ouro da segunda ser=E1 3/10 de (8 - X) e a soma das = quantidades=20 de ouro ser=E1 5/16 de 8 kg.
Assim sendo, 2x/5 + 24/10 - 3x/10 =3D=20 40/16
x/10 =3D 40/16 - = 24/10
x =3D 400/16 - 24 =3D = 1
Devemos ent=E3o tomar 1 kg da primeira = liga e 7 kg da=20 segunda. A diferen=E7a entre as quantidades =E9 6.
 
[]'s MP 
 
----- Original Message -----
From:=20 Daniel = Pini
Sent: Tuesday, April 29, 2003 = 7:25=20 PM
Subject: [obm-l] duvida

Na finalissima do campeonato carioca = de futebol=20 de 1991 o quadro das apostas era o seguinte:
Para o Flamengo: cada R$175,00 apostado dava = ao=20 apostador R$100,00.
Para o Fluminense: cada R$100,00 apstado dava = ao=20 apostador R$155,00.
Assim, por exemplo se o Flamrngo fosse o = vencedor do=20 jogo um pessoa que tivesse apostado R$175,00 no Flamengo teria de = volta=20 seus  R$175,00 e ainda ganharia R$100,00 , enquantoque uma pessoa = que=20 tivesse apostado R$100,00 no Fluminense, perderia seus = R$100,00.
Supondo que uma casa de apostas tenha aceito = 51 aposta a=20 R$175,00 no Flamengo, o n=EDmero de apostas a R$100,00 que ela = deve aceitar=20 para que seu lucro seja o mesmo independente de quemganhe o jogo=20 =E9?
R:55
Dispondo-se de duas ligas de ouro e prata = cujos metais=20 est=E3o nas raz=F5es 2:3 e 3:7 respectivamente, desejamos obter 8 = quilos de uma=20 nova liga na qual estes metais estejam na raz=E3o 5:11. A difere=E7a = entre=20 as  quantidades, em kg, que devemos tomar de cada uma das duas = ligas =E9?=20 R:6
------=_NextPart_000_005F_01C30C89.B71586C0-- ========================================================================= Instruções para entrar na lista, sair da lista e usar a lista em http://www.mat.puc-rio.br/~nicolau/olimp/obm-l.html ========================================================================= From owner-obm-l@sucuri.mat.puc-rio.br Tue Apr 29 22:29:12 2003 Return-Path: Received: (from majordom@localhost) by sucuri.mat.puc-rio.br (8.9.3/8.9.3) id WAA28491 for obm-l-MTTP; Tue, 29 Apr 2003 22:27:52 -0300 Received: from smtp-26.ig.com.br (smtp-26.ig.com.br [200.226.132.160]) by sucuri.mat.puc-rio.br (8.9.3/8.9.3) with SMTP id WAA28487 for ; Tue, 29 Apr 2003 22:27:49 -0300 Received: (qmail 19087 invoked from network); 30 Apr 2003 01:27:31 -0000 Received: from unknown (HELO xxxx) (200.165.168.230) by smtp-26.ig.com.br with SMTP; 30 Apr 2003 01:27:31 -0000 Message-ID: <004601c30eb8$75924f40$e6a8a5c8@epq.ime.eb.br> From: "Marcio" To: References: Subject: Re: [obm-l] Desigualdades Date: Tue, 29 Apr 2003 22:33:10 -0300 MIME-Version: 1.0 Content-Type: text/plain; charset="iso-8859-1" Content-Transfer-Encoding: 8bit X-Priority: 3 X-MSMail-Priority: Normal X-Mailer: Microsoft Outlook Express 5.50.4133.2400 X-MimeOLE: Produced By Microsoft MimeOLE V5.50.4133.2400 Sender: owner-obm-l@sucuri.mat.puc-rio.br Precedence: bulk Reply-To: obm-l@mat.puc-rio.br A desigualdade vale mesmo para v negativo, e eu tmb ainda nao entendi a solucao do Fabio... Escreva x=v+a, y=v+a+b, z=v+a+b+c, com a,b,c positivos. Entao, a desigualdade eh equivalente : (4v + 3a + 2b + c)^2 > 8( v(v+a+b) + (a+v)(v+a+b+c) ) Agora note que o termo em v^2 eh cancelado nessa desigualdade. Repare que o termo em v também é cancelado, pois em ambos os lados ele vale 8v(3a+2b+c). Portanto, a desigualdade inicial eh verdadeira se e somente se, para todos a,b,c positivos tivermos o termos independente de v do LE maior que o correspondente no LD, i.e, se dados a,b,c positivos, for sempre verdadeiro que: (3a+2b+c)^2 > 8a(a+b+c) sse a^2 + 2(2b-c)a + (2b+c)^2 > 0 Mas isso eh verdadeiro, pois: a^2 + 2(2b-c)a + (2b+c)^2 = a^2 + 2(2b-c)a + (2b-c)^2 + 8bc > [a - (2b-c)]^2 > 0. Em particular, a desigualdade soh eh verdadeira no caso em que o produto bc eh positivo (que nao precisa ser verdadeiro se nao tivermos a condicao inicial v To: Sent: Tuesday, April 29, 2003 7:36 PM Subject: Re: [obm-l] Desigualdades > Oi, Fabio: > > Desculpe a amolacao, mas eh que este problema estah me parecendo bem mais > dificil do que eu imaginava inicialmente. > > Veja meus cometarios abaixo. > > Um abraco, > Claudio. > > on 29.04.03 10:38, Fábio Dias Moreira at fabio.dias.moreira@terra.com.br > wrote: > > > -----BEGIN PGP SIGNED MESSAGE----- > > Hash: SHA1 > > > > On Tuesday 29 April 2003 00:30, Claudio Buffara wrote: > >> on 28.04.03 23:01, Fábio Dias Moreira at fabio.dias.moreira@terra.com.br > >> > >> wrote: > >>> On Monday 28 April 2003 22:32, Raphael Marx wrote: > >>>> Olá a todos. Gostaria de pedir uma ajudinha em desigualdades: > >>>> 1-Prove que se v >>>> (v+x+y+z)^2>8(vy+xz) > >>>> [...] > >>> ========================================================================= Instruções para entrar na lista, sair da lista e usar a lista em http://www.mat.puc-rio.br/~nicolau/olimp/obm-l.html ========================================================================= From owner-obm-l@sucuri.mat.puc-rio.br Tue Apr 29 23:53:32 2003 Return-Path: Received: (from majordom@localhost) by sucuri.mat.puc-rio.br (8.9.3/8.9.3) id XAA30465 for obm-l-MTTP; Tue, 29 Apr 2003 23:52:01 -0300 Received: from salvatore4.bol.com.br (salvatore4.bol.com.br [200.221.24.52]) by sucuri.mat.puc-rio.br (8.9.3/8.9.3) with ESMTP id XAA30461 for ; Tue, 29 Apr 2003 23:51:57 -0300 Received: from bol.com.br (200.221.24.139) by salvatore4.bol.com.br (5.1.071) id 3E7669FA0082F5BD for obm-l@mat.puc-rio.br; Tue, 29 Apr 2003 23:51:27 -0300 Date: Tue, 29 Apr 2003 23:51:27 -0300 Message-Id: Subject: [obm-l] =?iso-8859-1?q?Prepara=E7=E3o?= MIME-Version: 1.0 Content-Type: text/plain;charset="iso-8859-1" From: "tarsis19" To: obm-l@mat.puc-rio.br X-XaM3-API-Version: 2.4 R3 ( B4 ) X-SenderIP: 198.81.8.1 Content-Transfer-Encoding: 8bit X-MIME-Autoconverted: from quoted-printable to 8bit by sucuri.mat.puc-rio.br id XAA30462 Sender: owner-obm-l@sucuri.mat.puc-rio.br Precedence: bulk Reply-To: obm-l@mat.puc-rio.br Se alguém souber de algum site que tenha artigos de preparação para a olimpíada universitária por favor responda-me. Eu já baixei todas as provas brasileiras, colombianas, iberos e mundiais. Se alguém souber de outras importante, mande-me o site por favor. Obrigado. __________________________________________________________________________ Seleção de Softwares UOL. 10 softwares escolhidos pelo UOL para você e sua família. http://www.uol.com.br/selecao ========================================================================= Instruções para entrar na lista, sair da lista e usar a lista em http://www.mat.puc-rio.br/~nicolau/olimp/obm-l.html ========================================================================= From owner-obm-l@sucuri.mat.puc-rio.br Wed Apr 30 08:57:22 2003 Return-Path: Received: (from majordom@localhost) by sucuri.mat.puc-rio.br (8.9.3/8.9.3) id IAA03203 for obm-l-MTTP; Wed, 30 Apr 2003 08:56:02 -0300 Received: from hotmail.com (bay1-f77.bay1.hotmail.com [65.54.245.77]) by sucuri.mat.puc-rio.br (8.9.3/8.9.3) with ESMTP id IAA03197 for ; Wed, 30 Apr 2003 08:55:58 -0300 Received: from mail pickup service by hotmail.com with Microsoft SMTPSVC; Wed, 30 Apr 2003 04:55:27 -0700 Received: from 200.194.208.219 by by1fd.bay1.hotmail.msn.com with HTTP; Wed, 30 Apr 2003 11:55:27 GMT X-Originating-IP: [200.194.208.219] X-Originating-Email: [lauritoalves@hotmail.com] From: "Laurito Alves" To: obm-l@mat.puc-rio.br Subject: [obm-l] =?iso-8859-1?B?UmU6IFtvYm0tbF0gTm92YW1lbnRlOiBFc3Bh528gVmV0b3JpYWw=?= Date: Wed, 30 Apr 2003 11:55:27 +0000 Mime-Version: 1.0 Content-Type: text/plain; charset=iso-8859-1; format=flowed Message-ID: X-OriginalArrivalTime: 30 Apr 2003 11:55:27.0467 (UTC) FILETIME=[641723B0:01C30F0F] Sender: owner-obm-l@sucuri.mat.puc-rio.br Precedence: bulk Reply-To: obm-l@mat.puc-rio.br Felipe, Z2 (conjunto das classes residuais módulo 2) munido das operações de adiçao e multiplicação é um corpo e possui apenas 2 elementos. Todo corpo é um espaço vetorial sobre si mesmo. Logo, Z2 é um exemplo de espaço vetorial de dois elementos. Laurito >From: "Felipe Marinho" >Reply-To: obm-l@mat.puc-rio.br >To: obm-l@mat.puc-rio.br >Subject: [obm-l] Novamente: Espaço Vetorial Date: Tue, 29 Apr 2003 14:10:32 >-0400 > >Olá pessoal, > >Venho aqui agradecer a todos pelas respostas enviadas a respeito do >problema que eu havia apresentado. > >Mas olha, na verdade, acho que as respostas de vocês ficaram um pouco >difíceis de serem entendidas por mim... Os conhecimentos de vocês vão muito >além dos meus.. ;) > >Então olha, soh... para o problema: > >"Um Espaço Vetorial pode ser formado por EXATAMENTE dois vetores >distintos?" > >Bem, eu andei pensando bastante... e conclui o seguinte: > >Todo Espaço Vetorial V qualquer sobre um corpo K, admite como subespaços >vetoriais de imediato ele mesmo e o {0}. > >Ou seja, o próprio V e {0} são subespaços vetoriais de V. > >Para provar entao que um espaço vetorial pode ser formado por exatamente 2 >vetores distintos, o problema limita-se a provar a existencia de subespaços >vetoriais formados por exatamente 2 vetores distintos. > >Conclui entao que estes vetores existem e devem ter direções paralelas(pois >aí sao fechados na adição e multiplicacao por escalar). > >Entao existe um Espaco Vetorial formado por exatamente 2 vetores distintos >SE E SOMENTE SE suas direções forem paralelas, pois caso contrário, os 2 >vetores não serão subespaços vetorias de V. > >Agora peço a ajuda de vocês para analisarem minha resposta. Tem nexo oque >eu disse ? ;)) >Aguardo mesmo a ajuda de vocês! >E espero ter conseguido passar as minhas idéias. >Agradeço desde já... >E um grande abraço a todos... > >Felipe Marinho > >_________________________________________________________________ >MSN Messenger: converse com os seus amigos online. >http://messenger.msn.com.br > >========================================================================= >Instruções para entrar na lista, sair da lista e usar a lista em >http://www.mat.puc-rio.br/~nicolau/olimp/obm-l.html >========================================================================= _________________________________________________________________ MSN Hotmail, o maior webmail do Brasil. http://www.hotmail.com ========================================================================= Instruções para entrar na lista, sair da lista e usar a lista em http://www.mat.puc-rio.br/~nicolau/olimp/obm-l.html ========================================================================= From owner-obm-l@sucuri.mat.puc-rio.br Wed Apr 30 09:08:33 2003 Return-Path: Received: (from majordom@localhost) by sucuri.mat.puc-rio.br (8.9.3/8.9.3) id JAA03557 for obm-l-MTTP; Wed, 30 Apr 2003 09:07:12 -0300 Received: from aacpdlotus.net.ms.gov.br (ns1.ms.gov.br [200.181.116.3]) by sucuri.mat.puc-rio.br (8.9.3/8.9.3) with ESMTP id JAA03553 for ; Wed, 30 Apr 2003 09:07:08 -0300 From: JoaoCarlos_Junior@net.ms.gov.br Importance: High X-Priority: 1 (High) Subject: [obm-l] Geometria Plana To: obm-l@mat.puc-rio.br X-Mailer: Lotus Notes Release 5.0.9a January 7, 2002 Message-ID: Date: Wed, 30 Apr 2003 08:11:31 -0400 X-MIMETrack: Serialize by Router on aacpdlotus/NETMS(Release 5.0.9a |January 7, 2002) at 04/30/2003 08:12:02 AM MIME-Version: 1.0 Content-type: text/plain; charset=iso-8859-1 Content-Transfer-Encoding: 8bit X-MIME-Autoconverted: from quoted-printable to 8bit by sucuri.mat.puc-rio.br id JAA03554 Sender: owner-obm-l@sucuri.mat.puc-rio.br Precedence: bulk Reply-To: obm-l@mat.puc-rio.br Que saudade das aulas de Geometria Plana de meu professor Ubirajara Pinheiro Borges. OBS: Nao estou conseguindo resolver o item 3 da questão abaixo enunciada. Os demais, consegui. Porem, repito a questao na integra, visto que alguém pode interessar-se por ela. Da apostila deste professor, questao 148: Sao dados um segmento AB e dois angulos XAY e ZBT iguais a alfa, nao fixos, tais que AY e AX se mantem respectivamente perpendiculares a BZ e BT. Sabendo-se que: AX intersecçao BZ = P, AX intersecçao BT = M, AY intersecçao BZ = N e AY intersecçao BT = Q, pede-se: 1) determinar o lugar geometrico dos pontos M e N; 2) provar que PQ se mantem constante; 3) determinar o angulo que MN forma com AB. Um forte abraço a todos, Joao Carlos. ========================================================================= Instruções para entrar na lista, sair da lista e usar a lista em http://www.mat.puc-rio.br/~nicolau/olimp/obm-l.html ========================================================================= From owner-obm-l@sucuri.mat.puc-rio.br Wed Apr 30 09:29:04 2003 Return-Path: Received: (from majordom@localhost) by sucuri.mat.puc-rio.br (8.9.3/8.9.3) id JAA04316 for obm-l-MTTP; Wed, 30 Apr 2003 09:27:43 -0300 Received: from aacpdlotus.net.ms.gov.br (ns1.ms.gov.br [200.181.116.3]) by sucuri.mat.puc-rio.br (8.9.3/8.9.3) with ESMTP id JAA04303 for ; Wed, 30 Apr 2003 09:27:36 -0300 From: JoaoCarlos_Junior@net.ms.gov.br Importance: High X-Priority: 1 (High) Subject: [obm-l] Tentativa de =?iso-8859-1?Q?Solu=E7=E3o_para_Representa=E7=E3o_de_S=EDmbolos?= To: obm-l@mat.puc-rio.br X-Mailer: Lotus Notes Release 5.0.9a January 7, 2002 Message-ID: Date: Wed, 30 Apr 2003 08:32:01 -0400 X-MIMETrack: Serialize by Router on aacpdlotus/NETMS(Release 5.0.9a |January 7, 2002) at 04/30/2003 08:32:31 AM MIME-Version: 1.0 Content-type: text/plain; charset=iso-8859-1 Content-Transfer-Encoding: 8bit X-MIME-Autoconverted: from quoted-printable to 8bit by sucuri.mat.puc-rio.br id JAA04307 Sender: owner-obm-l@sucuri.mat.puc-rio.br Precedence: bulk Reply-To: obm-l@mat.puc-rio.br Tentativa de Solução para Representação do Til, acento agudo e outros. No intuito de retribuir os ensinamentos recebidos de todos, passei a questão da representação simbólica do til, acento agudo e outros ao departamento de informática de meu trabalho. Após duas semanas, hoje, sugeriram-me o que segue: Devemos atualizar a versão do Windows, do Internet Explorer ou do Netscape em nossos computadores. Acredito que se trata de solução por tentativa e erro, se não resolver, afirmaram-me que continuarão tentando. Muita mensagens chegam-me com representações corretas de tais símbolos, poucos é que estes vêm indevidamente representados. Professor Carlos Gustavo Tamm de Araújo Moreira, sei que o senhor está dentre aqueles que deve realizar a atualização acima. Como recebem vocês esta mensagem? Um forte abraço, João Carlos. ========================================================================= Instruções para entrar na lista, sair da lista e usar a lista em http://www.mat.puc-rio.br/~nicolau/olimp/obm-l.html ========================================================================= From owner-obm-l@sucuri.mat.puc-rio.br Wed Apr 30 09:29:06 2003 Return-Path: Received: (from majordom@localhost) by sucuri.mat.puc-rio.br (8.9.3/8.9.3) id JAA04292 for obm-l-MTTP; Wed, 30 Apr 2003 09:27:24 -0300 Received: (from nicolau@localhost) by sucuri.mat.puc-rio.br (8.9.3/8.9.3) id JAA04287 for obm-l@mat.puc-rio.br; Wed, 30 Apr 2003 09:27:23 -0300 Date: Wed, 30 Apr 2003 09:27:23 -0300 From: "Nicolau C. Saldanha" To: obm-l@mat.puc-rio.br Subject: Re: [obm-l] Exponencial de uma matriz Message-ID: <20030430092723.A3632@sucuri.mat.puc-rio.br> References: <000801c30e9d$601921e0$df8bfea9@itchy> Mime-Version: 1.0 Content-Type: text/plain; charset=iso-8859-1 Content-Disposition: inline Content-Transfer-Encoding: 8bit User-Agent: Mutt/1.2.5i In-Reply-To: <000801c30e9d$601921e0$df8bfea9@itchy>; from davidrvp@yahoo.com.br on Tue, Apr 29, 2003 at 07:16:19PM -0300 Sender: owner-obm-l@sucuri.mat.puc-rio.br Precedence: bulk Reply-To: obm-l@mat.puc-rio.br On Tue, Apr 29, 2003 at 07:16:19PM -0300, David Ricardo wrote: > Pessoal, eu tenho uma dúvida que já me perturba há uns dois anos e nunca > perguntei a ninguém... > > Em Teoria de Controle utilizamos muito a exponencial de uma matriz (exp(At), > onde A é uma matriz e t é o tempo) para fazer a representação de um sistema > discreto em variáveis de estado. > > Eu sei calcular isso utilizando transformada de Laplace inversa de (sI-A)^-1 > ou expansão em série de Taylor, mas o que significa um número elevado a uma > matriz? Eu não estou conseguindo exergar bem isso... Acho que a interpretação que você quer é a seguinte: A equação diferencial v' = Av, v(0) = v0 (onde v: R -> R^n é uma função cujos valores são vetores e A é uma matriz real nxn fixa) tem por solução v(t) = exp(tA) v0 Você pode, se desejar, escrever exp(tA) = (e^t)^A. Ou talvez você queira discutir o que significa f(A) onde A é uma matriz quadrada nxn (real ou complexa) e f: D -> C é uma função analítica, onde C é o conjunto dos números complexos e D é um subconjunto aberto de C contendo todos os elementos do espectro de A (os autovalores de A). Uma definição é a seguinte. Se A é diagonalizável escreva A = X^(-1)BX onde B é diagonal. A matriz f(B) também é (por definição) diagonal e tem entrada ii igual a f(b_ii); defina f(A) = X^(-1) f(B) X. A função f assim definida pode ser estendida continuamente (aliás analiticamente) a DD, o conjunto de todas as matrizes nxn complexas com espectro contido em D, definindo assim f: DD -> C^(nxn). A exponencial da matriz é um caso particular disso. []s, N. ========================================================================= Instruções para entrar na lista, sair da lista e usar a lista em http://www.mat.puc-rio.br/~nicolau/olimp/obm-l.html ========================================================================= From owner-obm-l@sucuri.mat.puc-rio.br Wed Apr 30 09:33:50 2003 Return-Path: Received: (from majordom@localhost) by sucuri.mat.puc-rio.br (8.9.3/8.9.3) id JAA04464 for obm-l-MTTP; Wed, 30 Apr 2003 09:32:33 -0300 Received: (from nicolau@localhost) by sucuri.mat.puc-rio.br (8.9.3/8.9.3) id JAA04459 for obm-l@mat.puc-rio.br; Wed, 30 Apr 2003 09:32:32 -0300 Date: Wed, 30 Apr 2003 09:32:32 -0300 From: "Nicolau C. Saldanha" To: obm-l@mat.puc-rio.br Subject: [obm-l] Re: =?iso-8859-1?Q?=5Bobm-l=5D_Prepara=E7=E3o?= Message-ID: <20030430093232.B3632@sucuri.mat.puc-rio.br> References: Mime-Version: 1.0 Content-Type: text/plain; charset=iso-8859-1 Content-Disposition: inline Content-Transfer-Encoding: 8bit User-Agent: Mutt/1.2.5i In-Reply-To: ; from tarsis19@bol.com.br on Tue, Apr 29, 2003 at 11:51:27PM -0300 Sender: owner-obm-l@sucuri.mat.puc-rio.br Precedence: bulk Reply-To: obm-l@mat.puc-rio.br On Tue, Apr 29, 2003 at 11:51:27PM -0300, tarsis19 wrote: > Se alguém souber de algum site que tenha artigos de > preparação para a olimpíada universitária por favor > responda-me. > Eu já baixei todas as provas brasileiras, colombianas, > iberos e mundiais. Eu acho que você já tem material até demais. O meu conselho para quem quer se preparar para olimpíadas é sempre o de não tentar vencer pela quantidade. Um problema no qual você pensou e resolveu sozinho, ou no qual você pensou seriamente antes de ler a solução, vale mais do que dez problemas para os quais você leu a solução sem ter nem tentado resolver sozinho. Na IMO espera-se que você resolva 3 problemas em um dia: tentar ler ou resolver mais do que isso é um erro. []s, N. ========================================================================= Instruções para entrar na lista, sair da lista e usar a lista em http://www.mat.puc-rio.br/~nicolau/olimp/obm-l.html ========================================================================= From owner-obm-l@sucuri.mat.puc-rio.br Wed Apr 30 09:45:24 2003 Return-Path: Received: (from majordom@localhost) by sucuri.mat.puc-rio.br (8.9.3/8.9.3) id JAA05031 for obm-l-MTTP; Wed, 30 Apr 2003 09:43:34 -0300 Received: (from nicolau@localhost) by sucuri.mat.puc-rio.br (8.9.3/8.9.3) id JAA05026 for obm-l@mat.puc-rio.br; Wed, 30 Apr 2003 09:43:33 -0300 Date: Wed, 30 Apr 2003 09:43:33 -0300 From: "Nicolau C. Saldanha" To: obm-l@mat.puc-rio.br Subject: [obm-l] Re: =?iso-8859-1?Q?=5Bobm-l=5D_Tentativa_de_Solu=E7=E3o_para_Representa=E7?= =?iso-8859-1?Q?=E3o_de_S=EDmbolos?= Message-ID: <20030430094333.C3632@sucuri.mat.puc-rio.br> References: Mime-Version: 1.0 Content-Type: text/plain; charset=iso-8859-1 Content-Disposition: inline Content-Transfer-Encoding: 8bit User-Agent: Mutt/1.2.5i In-Reply-To: ; from JoaoCarlos_Junior@net.ms.gov.br on Wed, Apr 30, 2003 at 08:32:01AM -0400 Sender: owner-obm-l@sucuri.mat.puc-rio.br Precedence: bulk Reply-To: obm-l@mat.puc-rio.br On Wed, Apr 30, 2003 at 08:32:01AM -0400, JoaoCarlos_Junior@net.ms.gov.br wrote: > Devemos atualizar a versão do Windows, do Internet Explorer ou do > Netscape em nossos computadores. ... > Professor Carlos Gustavo Tamm de Araújo Moreira, sei que o senhor > está dentre aqueles que deve realizar a atualização acima. Isso é off-topic, este tipo de consideração administrativa-informática não deve ir para a lista e sim para o administrador (eu) ou para o interessado (talvez o Gugu); já que ela indevidamente foi enviada para a lista não resisto a dar uma resposta parcial. Eu não uso nem Windows, nem IE nem Netscape: uso Linux (Red Hat) e mutt para ler e-mail (e Mozilla como browser). Tanto quanto eu sei o Gugu usa Solaris e elm para ler o e-mail dele. Campanhas no sentido de que os outros devem mudar de plataforma são totalmente off-topic. Esta lista deve ser e permanecer independente de plataforma e seu conselho portanto não faz muito sentido. > Como recebem vocês esta mensagem? Os acentos funcionam bem mas o conteúdo é infelizmente muito equivocado. []s, N. ========================================================================= Instruções para entrar na lista, sair da lista e usar a lista em http://www.mat.puc-rio.br/~nicolau/olimp/obm-l.html ========================================================================= From owner-obm-l@sucuri.mat.puc-rio.br Wed Apr 30 10:04:01 2003 Return-Path: Received: (from majordom@localhost) by sucuri.mat.puc-rio.br (8.9.3/8.9.3) id KAA05984 for obm-l-MTTP; Wed, 30 Apr 2003 10:02:01 -0300 Received: from hotmail.com (f131.sea1.hotmail.com [207.68.163.131]) by sucuri.mat.puc-rio.br (8.9.3/8.9.3) with ESMTP id KAA05979 for ; Wed, 30 Apr 2003 10:01:56 -0300 Received: from mail pickup service by hotmail.com with Microsoft SMTPSVC; Wed, 30 Apr 2003 06:01:25 -0700 Received: from 198.81.9.3 by sea1fd.sea1.hotmail.msn.com with HTTP; Wed, 30 Apr 2003 13:01:25 GMT X-Originating-IP: [198.81.9.3] X-Originating-Email: [fredericor@hotmail.com] From: "Frederico Reis Marques de Brito" To: obm-l@mat.puc-rio.br Subject: Re: [obm-l] Ajuda em teorema Date: Wed, 30 Apr 2003 10:01:25 -0300 Mime-Version: 1.0 Content-Type: text/plain; charset=iso-8859-1; format=flowed Message-ID: X-OriginalArrivalTime: 30 Apr 2003 13:01:25.0371 (UTC) FILETIME=[9B2F5CB0:01C30F18] Sender: owner-obm-l@sucuri.mat.puc-rio.br Precedence: bulk Reply-To: obm-l@mat.puc-rio.br Acho que te passaram o problema errado. Tal como vc o enunciou, nada há a ser demonstrado. Se a curva for a "descrita" opor f, isto é, o gráfico de f , então, y=f(x), obviamente. Imagino, que o problema correto seja um caso especial do Teorema da Função Implícita, que pode ser enunciado, informalmente, assim: " Dada uma curva (x(t),y(t)) se x´(t) <> 0 então podemos, localmente, descrever essa curva como gráfico de y=f(x). " Para a demonstração, veja, por exemplo, ELON LAGES LIMA - Curso de Análise - vol I. Ed. SBM. Frederico. >From: "Marcelo Souza" >Reply-To: obm-l@mat.puc-rio.br >To: obm-l@mat.puc-rio.br >Subject: Re: [obm-l] Ajuda em teorema >Date: Tue, 29 Apr 2003 18:16:30 +0000 > >Bom, o problema foi me passado assim, mas creio que seja a curva descrita >por f(x)...corrijam-me se estiver errado... > >[]'s, M. > > >>From: "Frederico Reis Marques de Brito" >>Reply-To: obm-l@mat.puc-rio.br >>To: obm-l@mat.puc-rio.br >>Subject: Re: [obm-l] Ajuda em teorema >>Date: Tue, 29 Apr 2003 13:45:14 -0300 >> >>Que curva??? >> >> >> >> >> >> >>>From: "Marcelo Souza" >>>Reply-To: obm-l@mat.puc-rio.br >>>To: obm-l@mat.puc-rio.br >>>Subject: [obm-l] Ajuda em teorema >>>Date: Tue, 29 Apr 2003 12:34:00 +0000 >>> >>>Alguém poderia exibir a prova do seguinte teorema para mim >>> >>>-Dada f(x) contínua e f'(t)<>0, então a curva pode ser expressa na forma >>>y=f(x). >>> >>>obs.: <>=diferente. >>>Agradeço >>>[]',M. >>> >>>_________________________________________________________________ >>>Help STOP SPAM with the new MSN 8 and get 2 months FREE* >>>http://join.msn.com/?page=features/junkmail >>> >>>========================================================================= >>>Instruções para entrar na lista, sair da lista e usar a lista em >>>http://www.mat.puc-rio.br/~nicolau/olimp/obm-l.html >>>========================================================================= >> >> >>_________________________________________________________________ >>MSN Hotmail, o maior webmail do Brasil. http://www.hotmail.com >> >>========================================================================= >>Instruções para entrar na lista, sair da lista e usar a lista em >>http://www.mat.puc-rio.br/~nicolau/olimp/obm-l.html >>========================================================================= > > >_________________________________________________________________ >Protect your PC - get McAfee.com VirusScan Online >http://clinic.mcafee.com/clinic/ibuy/campaign.asp?cid=3963 > >========================================================================= >Instruções para entrar na lista, sair da lista e usar a lista em >http://www.mat.puc-rio.br/~nicolau/olimp/obm-l.html >========================================================================= _________________________________________________________________ MSN Messenger: converse com os seus amigos online. http://messenger.msn.com.br ========================================================================= Instruções para entrar na lista, sair da lista e usar a lista em http://www.mat.puc-rio.br/~nicolau/olimp/obm-l.html ========================================================================= From owner-obm-l@sucuri.mat.puc-rio.br Wed Apr 30 14:56:54 2003 Return-Path: Received: (from majordom@localhost) by sucuri.mat.puc-rio.br (8.9.3/8.9.3) id OAA11882 for obm-l-MTTP; Wed, 30 Apr 2003 14:53:41 -0300 Received: from web14308.mail.yahoo.com (web14308.mail.yahoo.com [216.136.173.156]) by sucuri.mat.puc-rio.br (8.9.3/8.9.3) with SMTP id OAA11878 for ; Wed, 30 Apr 2003 14:53:36 -0300 Message-ID: <20030430175256.92829.qmail@web14308.mail.yahoo.com> Received: from [200.17.25.3] by web14308.mail.yahoo.com via HTTP; Wed, 30 Apr 2003 14:52:56 ART Date: Wed, 30 Apr 2003 14:52:56 -0300 (ART) From: =?iso-8859-1?q?Rafael?= Subject: Re: [obm-l] idade To: obm-l@mat.puc-rio.br In-Reply-To: <004201c30e9f$08478680$9a75fea9@windows98> MIME-Version: 1.0 Content-Type: text/plain; charset=iso-8859-1 Content-Transfer-Encoding: 8bit Sender: owner-obm-l@sucuri.mat.puc-rio.br Precedence: bulk Reply-To: obm-l@mat.puc-rio.br Olá Daniel! Antonio = A Raul = R "A soma das idades de A e R é 44 anos". Se A tem "A" anos e R tem "R" anos, então: A + R = 44 (i) Vamos dividir essa frase em algumas partes, pegando do final pro começo: "A tem o dobro da idade que R tinha, quando A tinha metade da idade que R terá quando R tiver 3 vezes a idade que A tinha quando A era 3 vezes mais velho que R". Antes de mais nada, sabemos que a diferença em anos entre a idade de duas pessoas é sempre constante. Vamos chamar essa diferença de "y". Então: A - R = y (ii) "a idade que A tinha quando A era 3 vezes mais velho que R" Há alguns anos atrás, a idade de A era o triplo da idade de R. Digamos que isso foi a "x" anos atrás. Nessa época, A tinha "A - x" anos e R tinha "R - x" anos. Como A tinha o triplo: A - x = 3.(R - x) (iii) "R terá quando R tiver 3 vezes a idade que A tinha" Como A tinha "A - x", quando R tiver 3 vezes a idade de A será: 3.(A - x) "quando A tinha metade da idade que R" Se R tinha 3.(A - x), e A tinha a metade disso, A tinha: 3.(A - x)/2 "idade que R tinha, quando A tinha metade da idade que R terá " Se A tinha 3.(A - x)/2 e a diferença entre as idades de A e de R é "y", se tirarmos "y" da idade de A, teremos a idade de R: 3.(A - x)/2 - y "A tem o dobro da idade que R tinha" Hoje A tem "A" anos. Se R tinha 3.(A - x)/2 - y, e A tem o dobro disso, então A tem: A = 2.[3.(A - x)/2 - y] A = 3.(A - x) - 2y (iv) Agora se pegarmos as equações (i), (ii), (iii) e (iv) podemos resolver um sistema: A + R = 44 A - R = y A - x = 3.(R - x) A = 3.(A - x) - 2y A + R = 44 A - R = y A - x = 3R - 3x A = 3A - 3x - 2y A + R = 44 A - R = y A - x + 3x = 3R 0 = 3A - A - 3x - 2y A + R = 44 (i) A - R = y (ii) A + 2x = 3R (iii) 2A - 3x - 2y = 0 (iv) Colocando o valor de y da equção ii na equação iv: 2A - 3x - 2y = 0 2A - 3x - 2(A - R) = 0 2A - 3x - 2A + 2R = 0 - 3x + 2R = 0 2R = 3x (v) Agora fazendo iii - i: A + 2x - A - R = 3R - 44 2x - R - 3R = - 44 2x - 4R = - 44, dividindo por 2, x - 2R = - 22 Colocando o valor de 2R da equação (v) nesse resultado, temos: x - 2R = - 22 x - 3x = - 22 - 2x = - 22 2x = 22 x = 22/2 x = 11 Da equação (v) achamos R: 2R = 3x 2R = 3.11 2R = 33 R = 33/2 Como A + R = 44 da equação (i): A + R = 44 A + 33/2 = 44 A = 44 - 33/2 A = (88 - 33)/2 A = 55/2 Já temos as idades de A e R, mas o problema pediu a diferença entre elas, que é o valor de y: y = A - R y = 55/2 - 33/2 y = (55 - 33)/2 y = 22/2 y = 11 Abraços, Rafael. --- Daniel Pini escreveu: > A soma das idades atuais de Antonio e Raul é 44 > anos. Antonio tem o dobro da idade que Raul tinha > quando Antonio tinha a metade da idade que Raul terá > quando Raul tiver três vezes a idade que Antonio > tinha quando era três vezes mais velho que Raul. A > diferença entre as idades em anos é? > A resposta em que cheguei foi 12 anos, será que > alguém pode confirmar a minha resposta? _______________________________________________________________________ Yahoo! Mail O melhor e-mail gratuito da internet: 6MB de espaço, antivírus, acesso POP3, filtro contra spam. http://br.mail.yahoo.com/ ========================================================================= Instruções para entrar na lista, sair da lista e usar a lista em http://www.mat.puc-rio.br/~nicolau/olimp/obm-l.html ========================================================================= From owner-obm-l@sucuri.mat.puc-rio.br Wed Apr 30 18:45:33 2003 Return-Path: Received: (from majordom@localhost) by sucuri.mat.puc-rio.br (8.9.3/8.9.3) id SAA16343 for obm-l-MTTP; Wed, 30 Apr 2003 18:43:53 -0300 Received: from fnn.net ([200.175.38.9]) by sucuri.mat.puc-rio.br (8.9.3/8.9.3) with SMTP id SAA16339 for ; Wed, 30 Apr 2003 18:43:50 -0300 Received: (qmail 24253 invoked from network); 30 Apr 2003 21:28:24 -0000 Received: from unknown (HELO windows98) (200.175.39.124) by mail.fnn.net with SMTP; 30 Apr 2003 21:28:24 -0000 Message-ID: <003401c30f62$d6de2440$9a75fea9@windows98> From: "Daniel Pini" To: References: <20030430175256.92829.qmail@web14308.mail.yahoo.com> Subject: Re: [obm-l] idade Date: Wed, 30 Apr 2003 18:52:46 -0300 MIME-Version: 1.0 Content-Type: text/plain; charset="iso-8859-1" Content-Transfer-Encoding: 8bit X-Priority: 3 X-MSMail-Priority: Normal X-Mailer: Microsoft Outlook Express 5.00.2615.200 X-MimeOLE: Produced By Microsoft MimeOLE V5.00.2615.200 Sender: owner-obm-l@sucuri.mat.puc-rio.br Precedence: bulk Reply-To: obm-l@mat.puc-rio.br Realmente valeu pela resposta. Eu tinha deduzido todas as sentenças corretamente, mas na hora de efetuar as expressões, eu cometi um erro e cheguei a resposta 12. ----- Original Message ----- From: Rafael To: Sent: Wednesday, April 30, 2003 2:52 PM Subject: Re: [obm-l] idade > Olá Daniel! > > Antonio = A > Raul = R > > "A soma das idades de A e R é 44 anos". > Se A tem "A" anos e R tem "R" anos, então: > A + R = 44 (i) > > Vamos dividir essa frase em algumas partes, pegando do > final pro começo: > "A tem o dobro da idade que R tinha, quando A tinha > metade da idade que R terá quando R tiver 3 vezes a > idade que A tinha quando A era 3 vezes mais velho que > R". > > Antes de mais nada, sabemos que a diferença em anos > entre a idade de duas pessoas é sempre constante. > Vamos chamar essa diferença de "y". Então: > A - R = y (ii) > > "a idade que A tinha quando A era 3 vezes mais velho > que R" > Há alguns anos atrás, a idade de A era o triplo da > idade de R. Digamos que isso foi a "x" anos atrás. > Nessa época, A tinha "A - x" anos e R tinha "R - x" > anos. Como A tinha o triplo: > A - x = 3.(R - x) (iii) > > "R terá quando R tiver 3 vezes a idade que A tinha" > Como A tinha "A - x", quando R tiver 3 vezes a idade > de A será: > 3.(A - x) > > "quando A tinha metade da idade que R" > Se R tinha 3.(A - x), e A tinha a metade disso, A > tinha: > 3.(A - x)/2 > > "idade que R tinha, quando A tinha metade da idade que > R terá " > Se A tinha 3.(A - x)/2 e a diferença entre as idades > de A e de R é "y", se tirarmos "y" da idade de A, > teremos a idade de R: > 3.(A - x)/2 - y > > "A tem o dobro da idade que R tinha" > Hoje A tem "A" anos. Se R tinha 3.(A - x)/2 - y, e A > tem o dobro disso, então A tem: > A = 2.[3.(A - x)/2 - y] > A = 3.(A - x) - 2y (iv) > > Agora se pegarmos as equações (i), (ii), (iii) e (iv) > podemos resolver um sistema: > A + R = 44 > A - R = y > A - x = 3.(R - x) > A = 3.(A - x) - 2y > > A + R = 44 > A - R = y > A - x = 3R - 3x > A = 3A - 3x - 2y > > A + R = 44 > A - R = y > A - x + 3x = 3R > 0 = 3A - A - 3x - 2y > > A + R = 44 (i) > A - R = y (ii) > A + 2x = 3R (iii) > 2A - 3x - 2y = 0 (iv) > > Colocando o valor de y da equção ii na equação iv: > 2A - 3x - 2y = 0 > 2A - 3x - 2(A - R) = 0 > 2A - 3x - 2A + 2R = 0 > - 3x + 2R = 0 > 2R = 3x (v) > > Agora fazendo iii - i: > A + 2x - A - R = 3R - 44 > 2x - R - 3R = - 44 > 2x - 4R = - 44, dividindo por 2, > x - 2R = - 22 > > Colocando o valor de 2R da equação (v) nesse > resultado, temos: > x - 2R = - 22 > x - 3x = - 22 > - 2x = - 22 > 2x = 22 > x = 22/2 > x = 11 > > Da equação (v) achamos R: > 2R = 3x > 2R = 3.11 > 2R = 33 > R = 33/2 > > Como A + R = 44 da equação (i): > A + R = 44 > A + 33/2 = 44 > A = 44 - 33/2 > A = (88 - 33)/2 > A = 55/2 > > Já temos as idades de A e R, mas o problema pediu a > diferença entre elas, que é o valor de y: > y = A - R > y = 55/2 - 33/2 > y = (55 - 33)/2 > y = 22/2 > y = 11 > > Abraços, > > Rafael. > > --- Daniel Pini escreveu: > A soma > das idades atuais de Antonio e Raul é 44 > > anos. Antonio tem o dobro da idade que Raul tinha > > quando Antonio tinha a metade da idade que Raul terá > > quando Raul tiver três vezes a idade que Antonio > > tinha quando era três vezes mais velho que Raul. A > > diferença entre as idades em anos é? > > A resposta em que cheguei foi 12 anos, será que > > alguém pode confirmar a minha resposta? > > _______________________________________________________________________ > Yahoo! Mail > O melhor e-mail gratuito da internet: 6MB de espaço, antivírus, acesso POP3, filtro contra spam. > http://br.mail.yahoo.com/ > ========================================================================= > Instruções para entrar na lista, sair da lista e usar a lista em > http://www.mat.puc-rio.br/~nicolau/olimp/obm-l.html > ========================================================================= > ========================================================================= Instruções para entrar na lista, sair da lista e usar a lista em http://www.mat.puc-rio.br/~nicolau/olimp/obm-l.html ========================================================================= From owner-obm-l@sucuri.mat.puc-rio.br Wed Apr 30 19:22:49 2003 Return-Path: Received: (from majordom@localhost) by sucuri.mat.puc-rio.br (8.9.3/8.9.3) id TAA17029 for obm-l-MTTP; Wed, 30 Apr 2003 19:21:29 -0300 Received: from fnn.net ([200.175.38.9]) by sucuri.mat.puc-rio.br (8.9.3/8.9.3) with SMTP id TAA17019 for ; Wed, 30 Apr 2003 19:21:24 -0300 Received: (qmail 24124 invoked from network); 30 Apr 2003 22:05:59 -0000 Received: from unknown (HELO windows98) (200.175.39.124) by mail.fnn.net with SMTP; 30 Apr 2003 22:05:59 -0000 Message-ID: <007201c30f68$184e35a0$9a75fea9@windows98> From: "Daniel Pini" To: Subject: [obm-l] geometria Date: Wed, 30 Apr 2003 19:30:15 -0300 MIME-Version: 1.0 Content-Type: multipart/alternative; boundary="----=_NextPart_000_006F_01C30F4E.ED250D40" X-Priority: 3 X-MSMail-Priority: Normal X-Mailer: Microsoft Outlook Express 5.00.2615.200 X-MimeOLE: Produced By Microsoft MimeOLE V5.00.2615.200 Sender: owner-obm-l@sucuri.mat.puc-rio.br Precedence: bulk Reply-To: obm-l@mat.puc-rio.br This is a multi-part message in MIME format. ------=_NextPart_000_006F_01C30F4E.ED250D40 Content-Type: text/plain; charset="iso-8859-1" Content-Transfer-Encoding: quoted-printable A distancia entre os centros de dois c=EDrculos =E9 37.Se os raios = desses circulos medem20 e 8, o segmento da tangente comum externa = mede:35 A distancia entre os centros de dois c=EDrculos =E9 53.Se os raios = desses circulos medem20 e 8, o segmento da tangente comum interna = vale:45 O raio do circulo inscrito em um setor circular de raio r e angulo de = 60=BA =E9:r/3 ------=_NextPart_000_006F_01C30F4E.ED250D40 Content-Type: text/html; charset="iso-8859-1" Content-Transfer-Encoding: quoted-printable
A distancia entre os centros de dois = c=EDrculos =E9=20 37.Se os raios desses circulos medem20 e 8, o segmento da tangente comum = externa=20 mede:35
 
A distancia entre os centros de dois = c=EDrculos =E9=20 53.Se os raios desses circulos medem20 e 8, o segmento da tangente comum = interna=20 vale:45
 
O raio do circulo inscrito  em um setor circular de raio r e = angulo de=20 60=BA =E9:r/3
------=_NextPart_000_006F_01C30F4E.ED250D40-- ========================================================================= Instruções para entrar na lista, sair da lista e usar a lista em http://www.mat.puc-rio.br/~nicolau/olimp/obm-l.html ========================================================================= From owner-obm-l@sucuri.mat.puc-rio.br Wed Apr 30 20:04:06 2003 Return-Path: Received: (from majordom@localhost) by sucuri.mat.puc-rio.br (8.9.3/8.9.3) id UAA18018 for obm-l-MTTP; Wed, 30 Apr 2003 20:02:37 -0300 Received: from smtp-27.ig.com.br (smtp-27.ig.com.br [200.226.132.159]) by sucuri.mat.puc-rio.br (8.9.3/8.9.3) with SMTP id UAA18014 for ; Wed, 30 Apr 2003 20:02:34 -0300 Received: (qmail 26344 invoked from network); 30 Apr 2003 23:02:14 -0000 Received: from unknown (HELO henrique) (200.140.81.207) by smtp-27.ig.com.br with SMTP; 30 Apr 2003 23:02:14 -0000 Message-ID: <000701c30f6c$87399b90$019da8c0@henrique> From: "=?iso-8859-1?Q?Henrique_Patr=EDcio_Sant'Anna_Branco?=" To: "OBM" Subject: [obm-l] =?iso-8859-1?Q?Somat=F3rio?= Date: Wed, 30 Apr 2003 20:01:59 -0300 MIME-Version: 1.0 Content-Type: text/plain; charset="iso-8859-1" Content-Transfer-Encoding: 8bit X-Priority: 3 X-MSMail-Priority: Normal X-Mailer: Microsoft Outlook Express 6.00.2800.1158 X-MimeOLE: Produced By Microsoft MimeOLE V6.00.2800.1165 Sender: owner-obm-l@sucuri.mat.puc-rio.br Precedence: bulk Reply-To: obm-l@mat.puc-rio.br Pessoal, me ajudem nessa, por favor. Calcular somatório de k/k! com k variando de 2 a infinito. Se escrevermos k * 1/k!, é facil ver que a segunda parcela converge para e - 2, mas o k da primeira parcela embaça tudo. Alguma idéia? Grato, Henrique. ========================================================================= Instruções para entrar na lista, sair da lista e usar a lista em http://www.mat.puc-rio.br/~nicolau/olimp/obm-l.html ========================================================================= From owner-obm-l@sucuri.mat.puc-rio.br Wed Apr 30 20:34:41 2003 Return-Path: Received: (from majordom@localhost) by sucuri.mat.puc-rio.br (8.9.3/8.9.3) id UAA18694 for obm-l-MTTP; Wed, 30 Apr 2003 20:33:17 -0300 Received: from trex.centroin.com.br (trex.centroin.com.br [200.225.63.134]) by sucuri.mat.puc-rio.br (8.9.3/8.9.3) with ESMTP id UAA18690 for ; Wed, 30 Apr 2003 20:33:14 -0300 Received: from trex.centroin.com.br (localhost [127.0.0.1]) by trex.centroin.com.br (8.12.9/8.12.9) with ESMTP id h3UNWikZ026496 for ; Wed, 30 Apr 2003 20:32:44 -0300 (EST) Received: by trex.centroin.com.br (8.12.9/8.12.5/Submit) id h3UNWi4J026495; Wed, 30 Apr 2003 20:32:44 -0300 (EST) Message-Id: <200304302332.h3UNWi4J026495@trex.centroin.com.br> Received: from 200.141.90.197 by trex.centroin.com.br (CIPWM versao 1.4C1) with HTTPS for ; Wed, 30 Apr 2003 20:32:44 -0300 (EST) Date: Wed, 30 Apr 2003 20:32:44 -0300 (EST) From: Augusto Cesar de Oliveira Morgado To: obm-l@mat.puc-rio.br Subject: =?iso-8859-1?q?Re: [obm-l] Somat=F3rio?= MIME-Version: 1.0 X-Mailer: CentroIn Internet Provider WebMail v. 1.4C1 (http://www.centroin.com.br/) Content-Type: text/plain; charset="iso-8859-1" Content-Transfer-Encoding: 8bit X-MIME-Autoconverted: from quoted-printable to 8bit by sucuri.mat.puc-rio.br id UAA18691 Sender: owner-obm-l@sucuri.mat.puc-rio.br Precedence: bulk Reply-To: obm-l@mat.puc-rio.br k/k! = 1/(k-1)! soma (k de 2 a infinito) k/k! = soma (k de 2 a infinito) 1/(k-1)! = soma (k de 1 a infinito) 1/k! = e - 1 Em Wed, 30 Apr 2003 20:01:59 -0300, Henrique_Patrício_Sant'Anna_Branco disse: > Pessoal, me ajudem nessa, por favor. > > Calcular somatório de k/k! com k variando de 2 a infinito. > > Se escrevermos k * 1/k!, é facil ver que a segunda parcela converge para e - > 2, mas o k da primeira parcela embaça tudo. Alguma idéia? > > Grato, > Henrique. > > ========================================================================= > Instruções para entrar na lista, sair da lista e usar a lista em > http://www.mat.puc-rio.br/~nicolau/olimp/obm-l.html > ========================================================================= > > ========================================================================= Instruções para entrar na lista, sair da lista e usar a lista em http://www.mat.puc-rio.br/~nicolau/olimp/obm-l.html ========================================================================= From owner-obm-l@sucuri.mat.puc-rio.br Wed Apr 30 23:01:09 2003 Return-Path: Received: (from majordom@localhost) by sucuri.mat.puc-rio.br (8.9.3/8.9.3) id WAA20760 for obm-l-MTTP; Wed, 30 Apr 2003 22:59:11 -0300 Received: from paiol.terra.com.br (paiol.terra.com.br [200.176.3.18]) by sucuri.mat.puc-rio.br (8.9.3/8.9.3) with ESMTP id WAA20756 for ; Wed, 30 Apr 2003 22:59:08 -0300 Received: from itaim.terra.com.br (itaim.terra.com.br [200.176.3.76]) by paiol.terra.com.br (Postfix) with ESMTP id EF1C987C33 for ; Wed, 30 Apr 2003 22:58:37 -0300 (BRT) Received: from riemann.localdomain (RJ175076.user.veloxzone.com.br [200.149.175.76]) (authenticated user fabio.dias.moreira) by itaim.terra.com.br (Postfix) with ESMTP id 876992E0042 for ; Wed, 30 Apr 2003 22:58:37 -0300 (BRT) Content-Type: text/plain; charset="iso-8859-1" From: =?iso-8859-1?q?F=E1bio=20Dias=20Moreira?= To: obm-l@mat.puc-rio.br Subject: Re: [obm-l] geometria Date: Wed, 30 Apr 2003 22:58:30 -0300 User-Agent: KMail/1.4.3 References: <007201c30f68$184e35a0$9a75fea9@windows98> In-Reply-To: <007201c30f68$184e35a0$9a75fea9@windows98> MIME-Version: 1.0 Content-Transfer-Encoding: 8bit Message-Id: <200304302258.44288.fabio.dias.moreira@terra.com.br> Sender: owner-obm-l@sucuri.mat.puc-rio.br Precedence: bulk Reply-To: obm-l@mat.puc-rio.br -----BEGIN PGP SIGNED MESSAGE----- Hash: SHA1 On Wednesday 30 April 2003 19:30, Daniel Pini wrote: > A distancia entre os centros de dois círculos é 37.Se os raios desses > circulos medem20 e 8, o segmento da tangente comum externa mede:35 > > A distancia entre os centros de dois círculos é 53.Se os raios desses > circulos medem20 e 8, o segmento da tangente comum interna vale:45 > [...] Nos dois problemas, trace os raios que incidem nos pontos de contato das tangentes. No primeiro, forma-se um trapézio retângulo (trace a altura dele) e no segundo formam-se dois triângulos retângulos (use semelhanças). []s, - -- Fábio "ctg \pi" Dias Moreira -----BEGIN PGP SIGNATURE----- Version: GnuPG v1.0.6 (GNU/Linux) Comment: For info see http://www.gnupg.org iD8DBQE+sH9TalOQFrvzGQoRApSVAJ48iG7PE1U3T9N54gZqWqm7Y3NJqgCeNQtg jJ6lyVNSNxZa5uK7Gb49q9Q= =rm35 -----END PGP SIGNATURE----- ========================================================================= Instruções para entrar na lista, sair da lista e usar a lista em http://www.mat.puc-rio.br/~nicolau/olimp/obm-l.html ========================================================================= From owner-obm-l@sucuri.mat.puc-rio.br Wed Apr 30 23:21:49 2003 Return-Path: Received: (from majordom@localhost) by sucuri.mat.puc-rio.br (8.9.3/8.9.3) id XAA21214 for obm-l-MTTP; Wed, 30 Apr 2003 23:20:21 -0300 Received: from mail.ajato.com.br (200-162-192-51.mail.ajato.com.br [200.162.192.51]) by sucuri.mat.puc-rio.br (8.9.3/8.9.3) with SMTP id XAA21210 for ; Wed, 30 Apr 2003 23:20:18 -0300 Received: from mparaujo (200.162.245.68) by mail.ajato.com.br (5.1.061) id 3EAE55B20005970F for obm-l@mat.puc-rio.br; Wed, 30 Apr 2003 23:19:47 -0300 Message-ID: <003801c30f23$550a0d60$44f5a2c8@ajato.com.br> From: "Marcos Paulo" To: References: Subject: Re: [obm-l] Geometria Plana Date: Wed, 30 Apr 2003 11:18:11 -0300 MIME-Version: 1.0 Content-Type: text/plain; charset="iso-8859-1" X-Priority: 3 X-MSMail-Priority: Normal X-Mailer: Microsoft Outlook Express 6.00.2800.1158 X-MimeOLE: Produced By Microsoft MimeOLE V6.00.2800.1165 Content-Transfer-Encoding: 8bit X-MIME-Autoconverted: from Quoted-Printable to 8bit by sucuri.mat.puc-rio.br id XAA21211 Sender: owner-obm-l@sucuri.mat.puc-rio.br Precedence: bulk Reply-To: obm-l@mat.puc-rio.br Se eu entendi corretamente a figura N é o ortocentro do triângulo AMB e portanto MN é perpendicular a AB. []'s MP ----- Original Message ----- From: To: Sent: Wednesday, April 30, 2003 9:11 AM Subject: [obm-l] Geometria Plana > Que saudade das aulas de Geometria Plana de meu professor Ubirajara > Pinheiro Borges. > OBS: Nao estou conseguindo resolver o item 3 da questão abaixo > > enunciada. Os demais, consegui. Porem, repito a questao na integra, visto > > que alguém pode interessar-se por ela. > > Da apostila deste professor, questao 148: > > Sao dados um segmento AB e dois angulos XAY e ZBT iguais a alfa, > > nao fixos, tais que AY e AX se mantem respectivamente perpendiculares a > > BZ e BT. Sabendo-se que: > > AX intersecçao BZ = P, AX intersecçao BT = M, AY intersecçao BZ = N > > e AY intersecçao BT = Q, pede-se: > > 1) determinar o lugar geometrico dos pontos M e N; > 2) provar que PQ se mantem constante; > 3) determinar o angulo que MN forma com AB. > > > Um forte abraço a todos, Joao Carlos. > > ========================================================================= > Instruções para entrar na lista, sair da lista e usar a lista em > http://www.mat.puc-rio.br/~nicolau/olimp/obm-l.html > ========================================================================= ========================================================================= Instruções para entrar na lista, sair da lista e usar a lista em http://www.mat.puc-rio.br/~nicolau/olimp/obm-l.html =========================================================================